SSC 2024 General English Chapter-wise Solved Papers 9415650134


118 58 12MB

English Pages 912 Year 2023

Report DMCA / Copyright

DOWNLOAD PDF FILE

Recommend Papers

SSC 2024 General English Chapter-wise Solved Papers
 9415650134

  • 0 0 0
  • Like this paper and download? You can publish your own PDF file online for free in a few minutes! Sign Up
File loading please wait...
Citation preview

SSC ENGLISH (Based on TCS PYQ)

Chapter, Topic & Sub Topic wise

Solved Papers Chief Editor A.K. Mahajan Compiled & Edited By Anand Soni, Ras Bihari Sachin yadav, Ashish Computer Graphics By Balkrishna Tripathi & Vinay Sahu Editorial Office 12, Church Lane Prayagraj-211002 Mob. : 9415650134 Email : [email protected] website : www.yctbooks.com/ www.yctfastbook.com  All Rights Reserved with Publisher Publisher Declaration Edited and Published by A.K. Mahajan for YCT Publications Pvt. Ltd. and printed by Om Sai Offset. In order to Publish the book, full care has been taken by the Editor and the Publisher, Rs. : 1195/still your suggestions and queries are welcomed. In the event of any dispute, the judicial area will be Prayagraj.

INDEX Section-I : Grammar 1.

Spotting the Error........................................................................................................... 7-96 A. Question tag ....................................................................................................................10 B. Subject and Verb Agreement ..........................................................................................10 C. Form of Verb (Tense/number) ........................................................................................22 D. Preposition ......................................................................................................................39 E. Conjunction .....................................................................................................................53 F. Article .............................................................................................................................56 G. Correct use of Pronoun, Adjective and Adverb ..............................................................63 H. Conditional Sentence ......................................................................................................77 I. Noun (Case, Gender, Number) ......................................................................................81 J. Voice & Narration ..........................................................................................................85 K. Miscellaneous ................................................................................................................90

2.

Sentence Improvement .............................................................................................. 97-194 A. Form of verb (Tense, Number, Modal) ........................................................................100 B. Subject & Verb agreement ...........................................................................................124 C. Conditional Sentence ...................................................................................................131 D. Question tag .................................................................................................................135 E. Article ..........................................................................................................................138 F. Correct use of Pronoun, Adjective & Adverb ..............................................................140 G. Conjunction ..................................................................................................................150 H. Preposition ...................................................................................................................156 I. Voice and Narration ......................................................................................................168 J. Noun (Case, Gender, Number) ....................................................................................172 K. Miscellaneous ..............................................................................................................175 2

3.

Fill in the blanks........................................................................................................ 195-289 A. Subject and Verb Agreement ........................................................................................198 B. Form of Verb (Tense/number) ......................................................................................199 C. Preposition ....................................................................................................................212 D. Conjunction ...................................................................................................................217 E. Article ...........................................................................................................................220 F. Correct use of Pronoun, Adjective & Adverb ...............................................................221 G. Question Tag .................................................................................................................230 H. Interjection ...................................................................................................................230 I. Conditional Sentence ...................................................................................................230 J. Miscellaneous ...............................................................................................................232

4.

Active/Passive Voice ................................................................................................ 290-342

5.

Direct and Indirect Speech ...................................................................................... 343-370 A. Assertive Sentence .......................................................................................................345 B. Interrogative Sentence .................................................................................................358 C. Imperative Sentence .....................................................................................................365 D. Exclamatory Sentence ..................................................................................................369

Section-II : Vocabulary 6.

Synonyms .................................................................................................................. 371-437

7.

Antonyms .................................................................................................................. 438-512

8.

Misspelt word .......................................................................................................... 513-572

9.

One word substitution ............................................................................................. 573-646

10. Idiom & Phrases ....................................................................................................... 647-713

Section-III : Comprehension 11. Reading comprehension .......................................................................................... 714-754 12. Cloze test ................................................................................................................... 755-858 13. Parajumble/P.Q.R.S. ................................................................................................ 859-912 3

SSC Previous Exam Papers Analysis Chart S.N.

Exam.

Year

1. 2. 3. 4. 5. 6. 7. 8. 9. 10. 11. 12. 13. 14. 15. 16. 17. 18. 19. 20.

SSC CGL (Tier-II) (October) SSC CGL (Tier-I) SSC CGL (Tier-II) (March) SSC CHSL (Tier-I) (March) SSC CHSL (Tier-II) SSC CHSL (Tier-I) (August) SSC MTS (September) SSC MTS SSC Selection Post Phase-XI SSC GD (Constable) SSC CGL (Tier-I) SSC CPO (Tier-I) SSC CGL (Tier-II) SSC CGL (Tier-I) SSC CHSL SSC MTS SSC CGL (Tier-I) SSC CHSL SSC CPO-SI SSC Selection Post Phase VIII (Graduate Level) SSC Selection Post Phase VIII (H.S. Level) SSC Selection Post Phase VIII (Matriculation Level) SSC CGL (Tier-II) SSC CHSL SSC CGL (Tier-I) SSC CPO-SI (Tier-II) SSC CPO-SI SSC Stenographer Grade C&D SSC Selection Post Phase VII (Graduate Level) SSC Selection Post Phase VII (H.S. Level) SSC Selection Post Phase VII (Matriculation Level) SSC CGL (Tier-II) SSC CGL (Tier-I) SSC MTS SSC CHSL SSC GD SSC CGL (Tier-II) SSC MTS Total

2023 2023 2023 2023 2023 2023 2023 2023 2023 2023 2022 2022 2022 2022 2022 2022 2021 2021 2020 2020

Total Question Papers 1 39 4 36 1 40 27 57 12 76 40 9 3 21 42 48 21 36 6 4

2020

3

3 × 25 = 75

2020

5

5 × 25=125

2020 2020 2020 2019 2019 2019 2019

3 36 18 1 8 12 4

3 × 200 = 600 36 × 25 = 900 18 × 25 = 450 1 × 200 = 200 8 × 50 = 400 12 × 100 = 1200 4 × 25 = 100

2019

4

4 × 25 = 100

2019

4

4 × 25 = 100

2019 2019 2019 2019 2019 2017 2017

3 22 39 25 40 7 17 774

3 × 200 = 600 22 × 25 = 550 39 × 25 = 975 25 × 25 = 625 40 × 25 = 1000 7 × 200 = 1400 17 × 25 = 425 23,535

21. 22. 23. 24. 25. 26. 27. 28. 29. 30. 31. 32. 33. 34. 35. 36. 37. 38.

Note–

Total Question 1 × 45 = 45 39 × 25 = 975 4 × 45 = 180 36 × 25 = 900 1 × 40 = 40 40 × 25 = 1000 27 × 25 = 675 57 × 25 = 1425 12 × 25 = 300 76 × 20 = 1520 40 × 25 = 1000 9 × 50 = 450 3 × 200 = 600 21 × 25 = 525 42 × 25 = 1050 48 × 25 = 1200 21 × 25 = 525 36 × 25 = 900 6 × 50 = 300 4 × 25 = 100

After Detailed Analysis of the above 774 Question papers of SSC Exams related to English 23,535 questions have been presented chapterwise. Questions of repetitive and similar nature have been included so that the technique of asking question can benefit the competitors. 4

Trend Analysis of Previous Year SSC English Papers Through Pie Chart and Bar Graph

Parajumble/ P.Q.R.S., 368 Cloze test , 945

Spotting the Error, 669

Reading comprehension , 260

Sentence Improvement, 747

Fill in the blanks, 750 Idiom & Phrases , 632 Active/Passive Voice, 325 One word substitution , 422

Direct and Indirect Speech, 152

Misspelt word , 604

Synonyms , 679

Antonyms , 680

945

747

750 679

669

680 632

604

422 368 325 260 152

5

Trend Analysis of Questions topicwise from CGL (Pre & Mains) CHSL (Pre & Mains) Selection Post XI, SSC MTS, SSC GD & Other Exams (2017-2023)

6

01. SPOTTING ERROR TCS hewšve& hej DeeOeeefjle (Based On TCS Pattern) Chapterwise 1

Question tag

2

Subject and Verb Agreement

3

Form of Verb (Tense/number)

4

Preposition

5

Conjunction

Spotting the Error

Exam

Question No.

CGL (Tier-1) CGL (Tier-2) CHSL (Tier-1) CHSL (Tier-2) Selection Post VII, VIII, XI SSC MTS SSC GD SSC CPO SI CGL (Tier-1) CGL (Tier-2) CHSL (Tier-1) CHSL (Tier-2) Selection Post VII, VIII, XI SSC MTS SSC GD SSC CPO SI CGL (Tier-1) CGL (Tier-2) CHSL (Tier-1) CHSL (Tier-2) Selection Post VII, VIII, XI SSC MTS SSC GD SSC CPO SI CGL (Tier-1) CGL (Tier-2) CHSL (Tier-1) CHSL (Tier-2) Selection Post VII, VIII, XI SSC MTS SSC GD SSC CPO SI CGL (Tier-1)

1 1 1 1 1 1 20 7 15 10 5 10 5 15 30 10 35 15 6 7 3 30 25 5 80 10 3 5 8 20 5

CGL (Tier-2)

3

CHSL (Tier-1)

10

CHSL (Tier-2)

2

Selection Post VII, VIII, XI

1

SSC MTS

2

SSC GD

-

SSC CPO SI

6

7

Years

(2017–2023)

(2017–2023)

(2017–2023)

(2017–2023)

(2017–2023)

YCT

6

Article

7

Correct use of Pronoun, Adjective and Adverb 8

Conditional Sentence

9

Noun (Case, Gender, Number) 10

Voice & Narration

11

Miscellaneous

Spotting Error

CGL (Tier-1) CGL (Tier-2) CHSL (Tier-1) CHSL (Tier-2) Selection Post VII, VIII, XI SSC MTS SSC GD SSC CPO SI CGL (Tier-1) CGL (Tier-2) CHSL (Tier-1) CHSL (Tier-2) Selection Post VII, VIII, XI SSC MTS SSC GD SSC CPO SI CGL (Tier-1) CGL (Tier-2) CHSL (Tier-1) CHSL (Tier-2) Selection Post VII, VIII, XI SSC MTS SSC GD SSC CPO SI CGL (Tier-1) CGL (Tier-2) CHSL (Tier-1) CHSL (Tier-2) Selection Post VII, VIII, XI SSC MTS SSC GD SSC CPO SI CGL (Tier-1) CGL (Tier-2) CHSL (Tier-1) CHSL (Tier-2) Selection Post VII, VIII, XI SSC MTS SSC GD SSC CPO SI CGL (Tier-1) CGL (Tier-2) CHSL (Tier-1) CHSL (Tier-2) Selection Post VII, VIII, XI SSC MTS SSC GD SSC CPO SI 8

10 2 12 8 10 10 20 7 25 6 5 25 2 20 7 3 8 2 1 2 2 9 2 7 3 1 4 1 3 11 3 6 3 2 10 1 1 11 4 12 8 5 3 2 6

(2017–2023)

(2017–2023)

(2017–2023)

(2017–2023)

(2017–2023)

(2017–2023)

YCT

Trend Analysis of Questions topicwise from CGL (Pre & Mains) CHSL (Pre & Mains) Selection Post VII, VIII, XI, SSC MTS, SSC GD & Other Exams (2017-2023)

Spotting Error

9

YCT

01. SPOTTING ERROR A.

Question tag

1.

Note – Question tag of positive sentence is in Negative and Negative Sentence is in positive. Correct Sentence The education of girls is a primary concern for the country so it should be started early shouldn't it. 4. The following sentence has been split into four segments. Identify the segment that contains a grammatical error. My friend / never tells / a lie, / doesn’t he? (a) My friend (b) never tells (c) dosen't he (d) a lie SSC Constable GD-14/12/2021 Shift-II Ans. (c) : In the above sentence 'does he' will be used in place of 'doesn't he' because 'Never' gives Negative sense. Hence question tag will be positive. Correct sentence - My friend never tells a lie does he ? 5. Identify the segment in the sentence which contains a grammatical error. I am suitably dressed for the occasion, am I? (a) for the occasion (b) am I (c) I am (d) suitably dressed SSC CPO-SI – 25/11/2020 (Shift-II) Ans. (b) : In the above sentence 'aren't I' will be used in place of 'am I' because if 'am' is used in a positive sentence, its Negative question tag is 'aren't'. The question tag of positive sentence is Negative. Correct Sentence : I am suitably dressed for the occasion, aren't I? 6. Ram never came again, did he? (a) Ram never (b) Came (c) did he (d) No error SSC GD-14/02/2019 (Shift – I) Ans. (d) : No error. Note:- The Question tag being positive or Negative depend on whether the sense of the sentence is positive or Negative. Since it is clear from the use of 'Never' in the sentence that the sentence is giving negative sence and it is clear from the use of came (V2) that the sentence is in past indefinite. Hence its question tag will be positive & Past Indefinite helping verb 'did' will be used. And the appropriate pronoun he will be used for Ram. Hence 'did he' will be used.

identify the segment in the sentence which contains a grammatical error. If there is no error, select 'No error'. Every Saturday, your mother prepares a pudding, isn't she ? (a) isn't she (b) Every Saturday, your mother (c) Prepares a pudding (d) No error. SSC CGL (Tier-II) 08.08.2022 Shift-II Ans. (a) : In the above sentence 'doesn't she' will be used in place of isn't she because sentence is affirmative sentence of present indefinite tense. Hence question tag of present indefinite tense, singular helping verb, 'does' will be used. Since sentence is in positive. Hence question tag is in Negative. Note:– Question tag of positive sentence is in Negative and question tag of negative sentence is in positive. Correct sentence – Every Saturday, your mother prepares a pudding doesn't she? 2. The following sentence has been split into four segments. Identify the segment that contains a grammatical error. Your eldest sister/ lives in / a big city,/ does she? (a) a big city (b) lives in (c) does she (d) Your eldest sister SSC CGL (Tier-I) 13/04/2022 Shift-I Ans. (c) : In the above sentence 'doesn't she' will be used in place of 'does she' because question tag of positive sentence, is in Negative and question tag of Negative Sentence is in positive. Correct SentenceYour eldest sister lives in a big city doesn't she? 3. The following sentence has been split into four segments. Identify the segment that contains a grammatical error. The education of girls/ is a primary concern for B. the country,/ so it should be started early,/isn't 7. it? (a) The education of girls (b) isn't it (c) so it should be started early (d) is a primary concern for the country SSC Constable GD-17/11/2021 Shift-II Ans. (b) : In the above sentence 'Shouldn't it' will be used in place of 'isn't it' because Question tag of should is Shouldn't . Spotting Error

10

( Subject and Verb Agreement Find the part of the given sentence that has an error in it. If there is no error, choose 'No error'. To treat nightmare disorder, there is a number of medications and therapies. (a) of medications and therapies (b) To treat nightmare disorder (c) there is a number (d) No error SSC MTS 04/05/2023 (Shift-II) YCT

Ans. (c) : There is error in part (c) because 'there are a number' will be used in the place of 'there is a number'. There is not the subject, the verb agrees with what follows the verb. A number→ Plural (verb) Thus, the correct answer is option (c). 8. Find the part of the given sentence that has an error in it. If there is no error, choose ‘No error’. I am thirty-two years old and my sister are fifteen. (a) I am thirty-two (b) years old and (c) my sister are fifteen. (d) No error SSC MTS 19/05/2023 (Shift-III) Ans. (c) : The segment of the above given sentence – ‘my sister are fifteen’ is grammatically error because the verb ‘is’ is used in the place of ‘are’ as the subject is singular. Singular subject + sing. Verb. Thus, the correct answer is option (c). 9. Identify the segment that contains a grammatical error. If there is no error, select 'No error'. Rapid climate warming in/the tundra biome have been linked/to increasing shrub dominance. (a) Rapid climate warming in (b) No error (c) the tundra biome have been linked (d) to increasing shrub dominance. SSC MTS 18/05/2023 (Shift-I) Ans. (c) : The segment of the above given sentence – ‘the tundra biome have been linked’ is grammatically error, because the subject of the sentence is singular, singular verb (has been) is used in the place of plural verb (have been). The correct segment is- ‘the tundra biome has been linked’. 10. Find the part of the given sentence that has an error in it. If there is no error, choose ‘No error’. India’s claim to be a world leader in pharmaceuticals are heavily compromised. (a) world leader in pharmaceuticals (b) No error (c) are heavily compromised. (d) India’s claim to be a SSC MTS 16/05/2023 (Shift-II) Ans. (c) : The above given sentence the part (c) 'are heavily compromised' has grammatically error, because in this sentence, the subject is singular (India's) therefore the verb (is) is used in the place of 'are'. Thus the correct answer is (c). 11. Parts of the following sentence have been underlined and given as options. Select the option that contains an error. Jiten and Hiru loves to eat french fries for lunch. (a) French fries (b) Loves (c) Lunch (d) Eat SSC GD 16/01/2023 (Shift-III) Spotting Error

Ans. (b) : In the underlined part of the above sentence 'love' will be used in place of 'loves' because when two subjects (Jiten and Hiru) are joined by and then plural verb (love) is used. Correct Sentence Jiten and Hiru love to eat french fries for lunch. 12. The following sentence has been split into four segments. Identify the segment that contains a grammatical error. I were/ writing/ this poem/ last evening. (a) I were (b) writing (c) This poem (d) Last evening SSC GD 08/02/2023 (Shift-IV) Ans. (a) : In the given sentence of the above segment, 'I was' will be used in place of 'I were'. Because 'I' is used as singular number in past continuous tense. It is the first person pronoun. Hence the correct sentence is– 'I was writing this poem last evening'. 13. The following sentence has been split into four segments. Identify the segment that contains a grammatical error. One of these / boxes have / the portrait / of the heiress. (a) of the heiress (b) One of these (c) the portrait (d) boxes have SSC CGL (Tier-I) 20/07/2023 (Shift-I) Ans. (d) : The most appropriate option is (d) because ‘boxes have’ is replaced by ‘boxes has’ because ‘one of’ is used as a singular Subject. Note- One of + plural subject + singular verb Thus, the correct answer is option (d). 14. The following sentence has been split into four segments. Identify the segment that contains an error. Neetu have been/ waiting for me/ since 10 o’clock / in the morning. (a) Neetu have been (b) in the morning (c) since 10 o’clock (d) waiting for me SSC CGL (Tier-I) 14/07/2023 (Shift-I) Ans. (a) : In the above sentence singular helping verb ‘has been’ will be used in the place of plural helping verb ‘have been’ because the subject of the sentence (Neetu) is singular. Correct sentence – Neetu has been waiting for since 100° clock in the morning. 15. The following sentence has been split into four segments. Identify the segment that contains a grammatical error. She always / invite me / to / dinner. (a) to (b) invite me (c) She always (d) dinner SSC CGL (Tier-I) 17/07/2023 (Shift-II) Ans. (b) : In the above sentence 'Invites' (singular verb) will be used in place of Invite (Plural verb) because subject (she) is singular . Hence singular verb will be used. Correct sentence :– She always invites me to dinner.

11

YCT

16.

Find the part of the given sentence that has an error in it. If there is no error, choose 'No error'. Bushfire have created havoc and exacerbated the conditions of animals in the forest. (a) Bushfire have created havoc (b) and exacerbated the conditions (c) No error (d) of animals in the forest. SSC CHSL (Tier-I) 17/03/2023 (Shift-IV) Ans. (a) : In the above question 'has' will be used in place of 'have' because 'Bushfire' is Singular Subject. Hence Singular Verb (has) will be used. 17. Identify the segment in the sentence which contains a grammatical error. If there is no error, then select the option "No error". Although people had been on watch and/ the watchkeepers were very tired/ everyone were up and buzzing. (a) No error (b) the watchkeepers were very tired (c) everyone were up and buzzing. (d) Although people had been on watch and SSC CHSL (Tier-I) 15/03/2023 (Shift-III) Ans. (c) : In the above sentence ''was'' will be used in place of ''were'' because Singular verb is used after ''everyone''. Correct sentenceAlthough People had been on watch and the watch keepers were very tired, everyone was up and buzzing. 18. Find the part of the given sentence that has an error in it. If there is no error, choose 'No error'. Plastic debris in the ocean can become coated with algae and other marine microbes that produces a chemical scent. (a) coated with algae and other (b) marine microbes that produces a chemical scent. (c) Plastic debris in the ocean can become (d) No error SSC CHSL (Tier-I) 09/03/2023 (Shift-IV) Ans. (b) : In the above sentence "Produce" will be used in place of "Produces" because Relative pronoun 'that' is used for plural noun (marine microbes). Hence plural "verb" will be used. Note : By adding s/es to the main verb, it becomes a singular verb. Correct Sentence: Plastic debris in the ocean can become coated with algae and other marine microbes that produce a chemical scent. 19. In the sentence identify the segment that contains a grammatical error. From my experience in mountain climbing I have learn that physical fitness is very important. (a) I have learn (b) from my experience (c) is very important (d) that physical fitness SSC Stenographer (Grade C & D) 12.11.2021 Shift-I Spotting Error

Ans. (a) : In the above sentence 'I have Learned' will be used in place of 'I have Learn' because after has/have/had the main verb is always used in V3 form.

Infinitive

Present Participle

To + Learn

learning

Past Past tense Participle learned learned or learnt or learnt

Correct Sentence From my experience in mountain climbing. I have learned learnt that physical fitness is very important. 20. Identify the segment in the sentence which contains a grammatical error. Apple growers in Himachal Pradesh says they are harvesting a bumper crop this year due to good weather conditions. (a) they are harvesting (b) Apple growers in Himachal Pradesh says (c) a bumper crop this year (d) due to good weather conditions SSC Stenographer (Grade C & D) 12.11.2021 Shift-II Ans. (b) : In the above sentence 'Apple growers in Himachal Pradesh Say' will be used in place of 'Apple growers in Himachal Pradesh Says' because the Subject (apple growers) of sentence is plural. Hence plural verb (say) will be used. Correct sentence– Apple growers in Himachal Pradesh say they are harvesting. 21. The following sentence has been split into four segments. Identify the segment that contains a grammatical error. A pack of wolves/were chasing/the deer/in the forest. (a) A pack of wolves (b) were chasing (c) in the forest (d) the deer SSC Stenographer (Grade C & D) 15.11.2021 Shift-II Ans. (b) : In the above sentence, 'A pack of wolves' is a singular noun, singular verb is used with any singular Noun, So 'was' will be used in place of 'were'. Correct SentenceA pack of wolves was chasing the deer in the forest. 22. The following sentence has been split into segments. One of them may contain an error. Identify the segment that contains a grammatical error. If you don't find any error, mark 'No error' as your answer. A shop nearby/sell all the goods/of daily use. (a) A shop nearby (b) No error (c) Of daily use (d) Sell all the goods SSC CGL (Tier-I) 19/04/2022 Shift-III Ans. (d) : In the above sentence 'Sells all the goods' will be used in place of 'Sell all the goods' because 'A shop' is singular subject so singular verb will be used. Correct SentenceA Shop nearby (Sells) all the goods of daily use. 23. The following sentence has been split into four segments. Identify the segment that contains a grammatical error. The contrast between / Britain and other countries / of Europe / are striking.

12

YCT

(a) (b) (c) (d)

The contrast between Britain and other countries of Europe are striking SSC CGL (Tier-I) 18/04/2022 Shift-I Ans. (d) : In the above sentence 'is striking' will be used in place of 'are striking' because 'contrast' is a singular subject so singular verb 'is' will be used. Correct SentenceThe contrast between Britain and other countries of Europe is striking. 24. The following sentence has been divided into parts. One of them may contain an error. Select the part that contains the error from the given options. If you don't find any error, mark ‘No error’ as your answer. Every Saturday, / the workers gets / their weekly wages. (a) the workers gets (b) Every Saturday (c) No error (d) their weekly wages SSC CGL (Tier-I) 18/04/2022 Shift-I Ans. (a) : In the above sentence 'workers get' will be used in place of 'workers gets' because plural verb (get) is used with plural subject. (the workers) Correct sentence– Every saturday. the workers get their weekly wages. 25. The following sentence has been divided into parts. One of them may contain an error. Select the part that contains the error from the given options. If you don’t find any error, mark ‘No error’ as your answer. We ordered a Pizza/ which are one of/ my favourite foods. (a) No error (b) We ordered a Pizza (c) my favourite foods (d) which are one of SSC CGL (Tier-I) 13/04/2022 Shift-III Ans. (d) : In the above sentence 'is' will be used in place of 'are' because 'which' relative Pronoun is used for 'Pizza' which is singular, after that singular verb will be used so 'is' will be used in place of 'are'. Correct Sentence We ordered a pizza which is one of my favourite foods. 26. Parts of the following sentence have been given as options. One of them may contain an error. Select the part that contains the error from the given options. If you don't find any error, mark 'No error' as your answer. A significant number of students volunteers each year for environmental projects. (a) volunteers each year for (b) No error (c) A significant number of students (d) environmental projects SSC MTS-07/07/2022 Shift-I Ans. (a) : In the above sentence ''volunteer each year for" will be used in place of ''volunteers each year'' because plural verb (Volunteer) will be used for plural subject (A significant number of Students). Correct Sentence– A significant number of students volunteer each year for environmental projects. Spotting Error

27.

The following sentence has been divided into parts. One of them may contain an error. Select the part that contains the error from the given options. If you don't find any error, mark 'No error' as your answer. People sometimes asks me / why carbon dioxide / is important for the climate. (a) is important for the climate (b) why carbon dioxide (c) No error (d) People sometimes asks me SSC MTS-06/07/2022 Shift-II Ans. (d) : In the above sentence ''people sometimes ask me'' will be used in place of "people sometimes asks me" because the subject of the sentence 'people' is plural. Hence the verb 'ask' (plural) will be used. Correct Sentence – People sometimes ask me why carbon dioxide is important for the climate. 28. The following sentence has been divided into parts. One of them may contain an error. Select the part that contains the error from the given options. If you don't find any error, mark 'No error' as your answer. Safety measures makes risky activities less risky. (a) risky activities (b) No error (c) Safety measures makes (d) less risky SSC MTS-06/07/2022 Shift-III Ans. (c) : In the above sentence "Safety measures make" will be used in place of "Safety measures makes" because the subject of the sentence is Safety measures 'plural'. Hence the plural verb 'make' will be used Note – By adding's' to a verb, the verb become singular and to make it plural, 's' removed from it. Correct Sentence– Safety measures make risky activities less risky. 29. The following sentence has been divided into parts. One of them may contain an error. Select the part that contains the error from the given options. If you don’t find any error, mark ‘No error’ as your answer. The App has been so well-developed / that it anticipate / all the customers’ needs. (a) that it anticipate (b) No error (c) The App has been so well - developed (d) all the customer's needs SSC Constable GD-29/11/2021 Shift-II Ans. (a) : In the above sentence "that it anticipates" will be used in place of "that it anticipate" because singular verb (Anticipates) is used after 'it'. Correct Sentence - The App has been so well developed that it anticipates all the customers' needs. 30. The following sentence has been split into four segments. Identify the segment that contains a grammatical error. The poet / and the author / is awarded / by the President. (a) The poet (b) by the President (c) is awarded (d) and the author SSC Constable GD-24/11/2021 Shift-II

13

YCT

Ans. (c) : In the above sentence 'are' will be used in place of 'is' because when two subjects are joined by and, And 'the' (article) is used before both subjects then both subject considers they are different person/things and it will be a plural subject so plural verb will be used. Correct Sentence The poet and the author are awarded by the president. 31. The following sentence has been split into four segments. Identify the segment that contains a grammatical error. Even though Mahindra & Mahindra warn / consumers not to use low-standard engine oil, / people refuse to read the instructions / and damage their vehicles. (a) consumers not to use low standard engine oil (b) people refuse to read the instructions (c) and damage their vehicles (d) Even though Mahindra & Mahindra warn SSC Constable GD-23/11/2021 Shift-III Ans. (d) : In the above sentence "Even though Mahindra & Mahindra warns" will be used in place of 'Even though Mahindra & Mahindra warn' because Mahindra and Mahindra is a company and it is considered a singular Noun. Hence singular verb (V+s/es) will be used. Correct sentence - Even though Mahindra & Mahindra warns consumers Not to use low standard engine oil ………….. 32. In the given sentence, identify the segment which contains a grammatical error. People has been warning in advance about the coming economic crisis. (a) People (b) economic crisis (c) in advance (d) has been warning SSC CHSL (Tier-I) –15/04/2021 (Shift-I) Ans. (d) In the above sentence 'people have been' will be used in place of 'people has been' because plural verb (have) is used with people (plural subject). Correct sentence– People have been warning in advance about the coming economic crisis. 33. Identify the segment in the sentence which contains a grammatical error. Neither Sam nor I are interested in attending the meeting. (a) are interested (b) in attending (c) Neither Sam nor I (d) the meeting SSC CGL (Tier-II) 16/11/2020 (3 pm - 5 pm) Ans. (a) : In the above sentence 'am' will be used in place of 'are' because when two subjects are connected with neither ----- nor either ----- or then the verb is used according to the latter subject. (I) Correct sentence- Neither Sam nor I am interested in attending the meeting. 34. Identify the segment in the sentence which contains a grammatical error. The venue for the wedding have not yet been finalised (a) been finalised (b) have not yet (c) The venue (d) for the wedding SSC CGL (Tier-II) 16/11/2020 (3 pm - 5 pm) Spotting Error

Ans. (b) : In the above sentence Subject 'the venue' is in singular form. Hence verb also will be singular, so 'has' will be used in place of 'have'. Correct sentence- The venue for the wedding has not yet been finalised. 35. Identify the segment in the sentence, which contains the grammatical error. Neither Amit nor Raju are staying with his parents in Mumbai. (a) are staying (b) in Mumbai (c) with his parents (d) Neither Amit nor Raju SSC CPO-SI (Tier-II) 27/09/2019 (3 pm - 5 pm) Ans. (a) : In the above sentence 'is staying' will be used in place of 'are staying' because when two Subjects are connected with either ------ or, Neither ---nor then the verb is used according to the latter subject. (Raju) Correct sentence- Neither Amit nor Raju is staying with his parents in Mumbai. 36. Identify the segment in the sentence, which contains the grammatical error. The list of candidates to be called for the interview were put up on the board. (a) for the interview (b) The list of candidates (c) were put up on the board (d) to be called SSC CPO-SI (Tier-II) 27/09/2019 (3 pm - 5 pm) Ans. (c) : In the above sentence 'was' will be used in place of 'were' because the main Subject (The list) of the sentence is singular. Hence singular verb (was) will be used. Correct sentence- The list of candidates to be called for the interview was put up on the board. 37. Identify the segment in the sentence, which contains the grammatical error. One should be careful to re-read what they has written. (a) to re-read what (b) be careful (c) One should (d) they has written SSC CPO-SI (Tier-II) 27/09/2019 (3 pm - 5 pm) Ans. (d) : In the above sentence 'they have written' will be used in place of 'they has written' because plural verb (have) will be used with plural subject (they). Correct sentence- One should be careful to re-read what they have written. 38. Identify the segment in the sentence, which contains the grammatical error. The box of paper clips are kept in the drawer. (a) are kept (b) The box (c) of paper clips (d) in the drawer SSC CPO-SI (Tier-II) 27/09/2019 (3 pm - 5 pm) Ans. (a) : In the above sentence The main Subject 'The box' is singular. Hence singular verb 'is' will be used in the place of 'are'. Correct sentence- The box of paper clips is kept in the drawer.

14

YCT

39.

Identify the segment in the sentence, which contains the grammatical error. Prema is the girl in my class who write beautiful poems. (a) Prema is the girl (b) who write (c) beautiful poems (d) in my class SSC CPO-SI (Tier-II) 27/09/2019 (3 pm - 5 pm) Ans. (b) : In the above sentence 'writes' will be used in place of 'write' because the subject (Prema) of the sentence is singular and relative pronoun 'who' is used for singular subject 'prema' hence 'who' will be treated as singular. Hence singular form of verb (V1 + s/es) will be used. Correct sentence- Prema is the girl in my class who writes beautiful poems. 40. Identify the segment in the sentence which contains the grammatical error. Very few countries in the world is as big as India. (a) is (b) Very few countries (c) as big as (d) in the world SSC Stenographer Grade C&D 05/02/2019 (3:5pm) Ans. (a) : In the above sentence 'are' will be used in place of 'is' because the subject (very few countries) of the sentence is plural. Hence plural verb (are) will be used in the place of 'is'. Correct sentence- Very few countries in the world are as big as India. 41. Identify the segment in the sentence which contains the grammatical error. Neither Nita nor her sisters has applied for this job (a) nor her (b) has applied (c) Neither Nita (d) for this job SSC Stenographer Grade C&D 05/02/2019 (3:5pm) Ans. (b) : In the above sentence 'have' will be used in place of 'has' because when two subjects are connected with neither ---- nor, either ---- or then the verb is used according to the latter subject. (her sisters) Correct sentence- Neither Nita nor her sisters have applied for this job. 42. In the following, some part of the sentence may have errors. Find out which part of the sentence has an error and select the appropriate option. If a sentence is free from error, select No Error. Modern medicines has scored significant victories (1)/ against both infection and trauma- (2)/ the major causes of ill health and death. (3)/ No error (4) (a) 1 (b) 2 (c) 3 (d) 4 SSC CGL (Phase-II) 17/02/2018 Ans. (a) In the above sentence 'have' will be used in place of 'has' because the subject (Modern medicines) is plural. Hence plural verb (have) will be used. Correct sentenceModern medicines have scored significant victories. 43. Identify the segment in the sentence which contains a grammatical error. If there is no error, then select the option 'No error'. All of us were a little nervous when we came into this room today. Spotting Error

a little nervous when when we came into this room today. all of us were No error SSC CPO-SI – 23/11/2020 (Shift-I) Ans. (d) : No error Note— The subject of the sentence 'All of us' is plural , So the use of 'were' is correct according to the Past sense. (ii) little/a little is used with uncountable nouns and few/a few are used with countable nouns. 44. Identify the segment in the sentence which contains a grammatical error. Either Ramesh or Reema have the keys to the cupboard. (a) to the cupboard (b) or Reema have (c) the keys (d) Either Ramesh SSC CPO-SI – 25/11/2020 (Shift-II) Ans. (b) : In the above sentence 'has' will be used in place of 'have' because if two subjects are joined with either ---- or then the verb is determined according to the subject that comes after or. Hence, according to 'Reema' the use of 'has' would be appropriate. Correct sentenceEither Ramesh or Reema has the keys to the cupboard. 45. Identify the segment in the sentence which contains a grammatical error. He is over eighty and dare not read without glasses. (a) he is (b) over eighty and (c) without glasses (d) dare not read SSC CPO-SI – 25/11/2020 (Shift-II) Ans. (d) : In the above sentence 'can not' will be used in place of 'dare not' because the use of 'can not' (capacity) gives the appropriate meaning according to the sense of sentence. Correct sentenceHe is over eighty and can not read without glasses. 46. Identify the segment in the sentence which contains a grammatical error. Our new secretary’s performance have been quite impressive. (a) secretary’s performance (b) quite impressive (c) have been (d) our new SSC CPO-SI – 23/11/2020 (Shift-II) Ans. (c) : In the above sentence 'has been' will be used in place of 'have been' because 'Our New Secretary's performance' is a singular Subject. Hence singular verb 'has been' will be used after it. Correct sentenceOur new secretary’s performance has been quite impressive. 47. In the sentence identify the segment which contains the grammatical error. Every employee of the company were given a two bedroom flat as Diwali bonus. (a) a two bedroom flat (b) as Diwali bonus (c) Every employee (d) were given SSC CGL (Tier-I) – 06/06/2019 (Shift-I)

15

(a) (b) (c) (d)

YCT

(a) (b) (c) (d)

Ans. (d) : In the above sentence, Every employee of the company is used as singular Noun, so 'was' will be used in place of 'were' Every + Singular noun + Singular Verb Correct Sentence : Every employee of the company was given a two bedroom flat as Diwali bonus. 48. In the sentence identify segment which contains the grammatical error. Each of the girls have given an impressive dance performance. (a) have given (b) Each of the girls (c) an impressive (d) dance performance SSC CGL (Tier-I) – 07/06/2019 (Shift-III) Ans : (a) In the above sentence singular verb 'has given' will be used in place of plural verb 'have given' because after the Distributive pronoun each of / either of /Neither of Plural Noun/ Pronoun is used but the auxiliary verb is singular. Each of / Neither of + Plural subject + Singular verb Correct Sentence Each of the girls has given an impressive dance performance. 49. In the sentence identify the segment which contains the grammatical error. The Doon Valley with all its lights look beautiful at night from the top of the mountain. (a) look beautiful (b) from the top (c) at night (d) with all its lights SSC CGL (Tier-I) – 07/06/2019 (Shift-II) Ans : (a) In the above sentence The main subject 'The Doon Valley is singular, hence the sentence being in present Indefinite tense, the singular verb 'Looks' will be used by adding s/es at the end of the main verb. Correct Sentence The Doon Valley with all its lights looks beautiful at night from the top of the mountain. 50. In the sentence identify the segment which contains the grammatical error. The match is about to begin since the captain as well as the team are on the field. (a) are on the field (b) The matchis about to begin (c) since the captain (d) as well as the team SSC CGL (Tier-I) – 10/06/2019 (Shift-II) Ans : (a) In the above sentence 'is' will be used in place of 'are' because when two subjects are connected with' as well as, like, with, besides, together with, no less than, etc then the verb is used according to the first Subject. (The captain) Correct Sentence The match is about to begin since the captain as well as the team is on the field. 51. In the sentence identify the segment which contains the grammatical error. Cows are amongst the gentlest of animals; none shows more passionate tenderness towards their young. Spotting Error

Cows are amongst towards their young none shows more the gentlest of animals SSC CGL (Tier-I) – 13/06/2019 (Shift-I) Ans : (c) In the above sentence plural verb 'Show' will be used in place of singular verb 'shows' because the verb coming after 'None' will be Singular or Plural depending on the type of subject. In case of Singular Subject, singular verb will be used and in case of plural Subject, Plural verb will be used. and here 'no he' is used for cows (plural). Correct Sentence Cows are amongst the gentlest of animals; none show more passionate tenderness towards their young. 52. In the sentence identify the segment which contains the grammatical error. The cost of fruits and vegetables have risen abnormally this month. (a) have risen (b) The cost of (c) this month (d) abnormally SSC CGL (Tier-I) – 13/06/2019 (Shift-I) Ans : (a) In the above sentence, 'has' will be used instead of 'have' because the main subject is 'the cost'. Which is singular. hence the following verb will be singular accordingly. Hence has will be used. Correct sentence The cost of fruits and vegetables has risen abnormally this month. 53. In the sentence identify the segment which contains the grammatical error. Friction reduce the efficiency of machines so their value also gets reduced. (a) Friction reduce (b) also gets reduced. (c) the efficiency of machines (d) so their value SSC CGL (Tier-I) – 19/06/2019 (Shift-III) Ans. (a) : In the above sentence 'Friction reduces' will be used in place of 'Friction reduce' because friction is singular uncountable Noun, so singular verb (reduces) will be used. Correct sentenceFriction reduces the efficiency of machines so their value also gets reduced. 54. In the sentence identify the segment which contains the grammatical error. I am sure that neither Juhi nor Jaya are responsible for breaking the vase. (a) neither Juhi nor Jaya (b) are responsible (c) for breaking the vase (d) I am sure that SSC CGL (Tier-I) – 19/06/2019 (Shift-III) Ans. (b) : In the above sentence 'is responsible' will be used in place of 'are responsible' because when two subjects are connected with neither ----- nor either ---or then the verb 'is' used according to the Letter Subject. (Jaya) which is singular. Correct sentence I am sure that neither Juhi nor Jaya is responsible for breaking the vase.

16

YCT

55.

In the sentence identify the segment which contains the grammatical error. There is many modes of travel to go to Agra but I prefer road travel. (a) to go to (b) prefer road travel (c) Agra but I (d) There is many modes SSC CGL (Tier-I) – 12/06/2019 (Shift-III) Ans. (d) : The main subject of the sentence is Many modes of travel which is plural, so helping verb will be also plural. Note- If any sentence starts with Introductory subject 'there' then the main subject is used after 'verb'. Correct SentenceThere are many modes of travel to go to Agra but I prefer road travel. 56. In the Sentence identify the segment which contains the grammatical error. Ten kilometers are a long distance to cover on foot for a child. (a) Ten kilometers are (b) for a child (c) to cover on foot (d) a long distance SSC CGL (Tier-I) – 11/06/2019 (Shift-I) Ans. (a) : In the above sentence 'Ten kilometers is' will be used in place of 'Ten kilometers are' because 'Ten kilometers is used as a unit so it is considered as a singular subject. so singular verb (is) is used after it'. Correct SentenceTen kilometers is a long distance to cover on foot for a child. 57. In the sentence identify the segment which contains the grammatical error. The child along with his parents were waiting for the programme to begin. (a) The child (b) along with his parents (c) programme to begin (d) were waiting for the SSC CGL (Tier-I) – 11/06/2019 (Shift-III) Ans. (d) : In the above sentence 'was waiting for the' will be used in place of 'were waiting for the' because when two subject are connected with as well as', with, together with, along with etc the verb is used to according to the first subject. (The child) Correct SentenceThe child along with his parents was waiting for the programme to begin. 58. Identify the segment in the sentence which contains the grammatical error. Note down my phone number in case you needs to ring me up. (a) Note down (b) to ring me up (c) my phone number (d) in case you needs SSC CPO-SI – 09/12/2019 (Shift-II) Ans : (d) In the above sentence 'need' (Plural verb) will be used in place of needs (Singular verb) because the subject (you) of the sentence is in plural Number. Correct SentenceNote down my phone number in case you need to ring me up. Spotting Error

59.

In the sentence identify the segment which contains the grammatical error. Saurav Ganguly have emerged as the new President of the BCCI after a night long discussion. (a) Saurav Ganguly have emerged (b) as the new President (c) of the BCCI (d) after a night long discussion SSC CPO-SI – 11/12/2019 (Shift-I) Ans : (a) In the above sentence Singular verb 'has' will be used in place of plural verb 'have' because the subject (Saurav Ganguly) of the sentence is in Singular Number. Correct sentenceSaurav Ganguly has emerged as the new President of the BCCI after a night long discussion. 60. Identify the segment in the sentence which contains the grammtical error. If there is no error, select 'No error'. The monsoon have, in the past five days, retreated from most parts of the country. (a) No error (b) The monsoon have (c) retreated from most parts (d) in the past five days SSC CPO-SI – 12/12/2019 (Shift-I) Ans : (b) In the above Sentence 'The monsoon has' will be used in place of 'The monsoon have' because 'Monsoon' is an uncountable Noun, So Singular verb (has) will be used for it. Correct sentenceThe monsoon has, in the past five days, retreated from most parts of the country. 61. In the sentence identify the segment which contains the grammatical error. Only one copy of the book, running into 1200 pages, on Gandhiji's visit to the State of Andhra Pradesh, remain now. (a) running into 1200 pages (b) remain now (c) only one copy of the book (d) on Gandhiji's visit to the State SSC CPO-SI – 13/12/2019 (Shift-II) Ans. (b) : In the above sentence singular verb 'Remains' will be used in place of Plural verb 'Remain' because the Subject (only one copy) of the sentence is singular. Correct sentenceOnly one copy of the book, running into 1200 pages, on Gandhiji's visit to the State of Andhra Pradesh, remains now. 62. Identify the segment in the sentence which contains the grammatical error. The two states frequently has differences over the use of the river water especially during the summer months. (a) has differences (b) over the use of the river water (c) especially during the summer months (d) The two states frequently SSC CHSL (Tier-I) –09/07/2019 (Shift-III)

17

YCT

Ans : (a) In the above sentence 'have differences' will be used in place of 'has differences' because the subject (Two states) of the Sentence is plural. Hence plural verb 'have' will be used. Correct sentenceThe two states frequently have differences over the use of the river water especially during the summer months. 63. Identify the segment which contains the grammatical error. The members of the victorious football team as well as the captain was congratulated. (a) was congratulated (b) as well as the captain (c) The members of the (d) victorious football team SSC CHSL (Tier-I) –05/07/2019 (Shift-II) Ans : (a) In the above sentence Plural helping verb 'were' will be used in place of singular helping verb 'was' because when two subjects are connected with as well as, with, like, besides etc the verb is used according to the first subject. (football team) Correct sentenceThe members of the victorious football team as well as the captain were congratulated. 64. Identify the segment which contains the grammatical error. Each of the pictures have been signed by the football star. (a) the pictures (b) Each of (c) have been (d) signed by SSC CHSL (Tier-I) –05/07/2019 (Shift-III) Ans : (c) In the above Sentence 'has been' will be used in place of ' have been' because each of, either of, always followed by plural Noun and Singular verb (has). Each of / Either Of / Neither of + Plural Noun + Singular verb

Correct SentenceEach of the pictures has been signed by the football star. 65. Identify the segment which contains the grammatical error. The number of visitors at the fair were much larger than expected. (a) than expected (b) were much larger (c) the number of (d) visitors at the fair SSC CHSL (Tier-I) –04/07/2019 (Shift-II) Ans : (b) In the above Sentence 'was much larger' will be used in place of 'were much larger' because the main subject (the number) of the sentence is singular. Hence singular verb (was) will be used. Correct SentenceThe number of visitors at the fair was much larger than expected. 66. In the sentence identify the segment which contains the grammatical error. The promoters of Med Hospitals has agreed to sell their business to Pal Hospitals. (a) The promoters of (b) to sell their business (c) Med Hospitals has agreed (d) to Pal Hospitals SSC CHSL (Tier-I) –03/07/2019 (Shift-I) Spotting Error

Ans : (c) In the above Sentence 'have agreed' will be used in place of 'has agreed' because the main subject (promoters) of the sentence is plural. Hence plural verb (have) will be used. Correct SentenceThe promoters of Med. Hospitals have agreed to sell their business to Pal Hospitals. 67. In the sentence identify the segment which contains the grammatical error. Many of China's exports has been redirected to the domestic market in view of the more consumption-driven economy. (a) has been redirected (b) Many of China's exports (c) in view of the more consumption-driven economy (d) to the domestic market SSC CHSL (Tier-I) –03/07/2019 (Shift-II) Ans. (a) : In the above sentence 'have been redirected' will be used in place of 'has been redirected' because the subject 'exports' is plural. Hence plural verb 'have' will be used. Note- Many a + singular noun + singular Verb Correct SentenceMany of China's exports have been redirected to the domestic market in view of the more consumptiondriven economy. 68. In the sentence identify the segment which contains the grammatical error. The Cannes Film Festival attract some of the world's most famous people. (a) The Cannes Film Festival (b) most famous people (c) attract some of (d) the world's SSC CHSL (Tier-I) –02/07/2019 (Shift-III) Ans : (c) In the above sentence 'attracts' will be used in place of 'attract' because the main subject (The Cannes film festival) is singular. Hence singular verb (attracts) will be used. Correct SentenceThe Cannes film festival attracts some of the world's most famous people. 69. In the sentence identify the segment which contains a grammatical error. The environment protection policy need to become stricter if we wish to combat climate change. (a) The environment protection policy (b) need to become stricter (c) combat climate change (d) if we wish to SSC CHSL (Tier-I) 16/10/2020 (Shift-II) Ans. (b) In the above sentence 'needs to become stricter' will be used in place of 'need to become stricter' because the main subject 'The Environment protection policy' is singular. Hence singular verb ' needs' will be used. Correct SentenceThe environment protection policy needs to become stricter if we wish to combat climate change.

18

YCT

70.

Select the segment in the sentence, which 74. In the sentence identify the segment which contains a grammatical error. contains the grammatical error. Plenty of people likes to celebrate New Year's Thomas is a man of word who have been paying eve with their family and friends. back the borrowed money in instalments. (a) likes to celebrate (a) who have been paying back (b) Plenty of people (b) in instalments (c) New year's eve with (c) Thomas is a man of word (d) their family and friends (d) the borrowed money SSC CHSL (Tier-I) 17/03/2020 (Shift-II) SSC CHSL (Tier-I) 18/03/2020 (Shift-I) Ans. (a) : In the above sentence 'like' will be used in Ans. (a) : In the above sentence 'Who has been playing place of 'likes' because the noun used after 'plenty of' is back' will be used in place of 'Who have been playing plural so verb will also be used in plural form. back' because pronoun 'who' is used for the subject 'Thomas' which is singular. Hence singular verb (has) Plenty of + Countable plural noun + Plural verb will be used. Plenty of + Uncountable noun + Singular verb Correct sentenceNote-Hate, stop, finish, enjoy, prefer, keep, like + Thomas is a man of word who has been paying back present participle (–ing form of verb) the borrowed money in instalments. Correct Sentence75. In the given sentence identify the segment Plenty of people like celebrating New Year's eve with which contains the grammatical error. their family and friends. The neighbourhood were getting disturbed 71. In the given sentence identify the segment with the loud music that was playing at the which contains the grammatical error. wedding party in the big bungalow. Scientist have recently announced that a more (a) at the wedding party effective antidote for snake bite venom has (b) in the big bungalow been made. (c) that was playing (a) Scientist have (d) The neighbourhood were getting disturbed (b) for snake bite venom has been made SSC CHSL (Tier-I) 15/10/2020 (Shift-III) (c) recently announced Ans. (d) : In the above sentence 'The neighbourhood (d) that a more effective antidote SSC CHSL (Tier-I) 13/10/2020 (Shift-I) was getting disturbed' will be used in place of 'The neighbourhood were getting disturbed' because the Ans. (a) : In the above sentence 'has' will be used in subject of the sentence 'neighbourhood' is uncountable place of 'have' because scientist is a singular subject, Noun. Hence singular verb (was) will be used. Hence the verb will also be in singular. Correct sentenceCorrect SentenceThe neighbourhood was getting disturbed with the "Scientist has recently announced that a more effective loud music that was playing at the wedding party in the antidote for snakebite venom has been made." 72. In the sentence identify the segment which big bungalow. contains the grammatical error. 76. In the given sentence identify the segment Everyone are very well equipped with his own which contains a grammatical error. weapons. Someone have walked away with my umbrella. (a) Everyone are (b) very well equipped (a) away with (b) Someone (c) own weapons (d) with his (c) have walked (d) my umbrella SSC CHSL (Tier-I) 19/03/2020 (Shift-I) SSC CHSL (Tier-I) 19/10/2020 (Shift-III) Ans. (a) : In the above sentence 'Everyone is' will be Ans. (c) : In the above sentence 'has walked' will be used in place of 'Everyone are' because each one, used in place of 'have walked' because someone, each, everyone etc. is used as singular subject. Hence one etc if used as a subject then singular verb is used helping verb will be in singular (is). with them. Correct SentenceCorrect sentenceEveryone is very well equipped with his own weapons. Someone has walked away with my umbrella. 73. In the sentence identify the segment which 77. Identify the segment in the given sentence contains the grammatical error. which contains the grammatical error. The simplest way to get proper recognition are Soni take her daughter to the park in the through doing things with dedication and evening in a stroller. sincerity. (a) Soni take her daughter (b) to the park (a) are through doing things (c) in a stroller (d) in the evening (b) The simplest way SSC CHSL (Tier-I) 20/10/2020 (Shift-II) (c) to get proper recognition (d) with dictation and sincerity Ans. (a) : In the above sentence 'Soni takes her SSC CHSL (Tier-I) 18/03/2020 (Shift-III) daughter' will used in place of 'Soni take her daughter' because Soni is in singular Number. Hence singular Ans. (a) : In the above sentence subject 'The simplest way is in singular form, so the verb will also be in verb (takes) will be used. Since the sentence is in singular. Hence 'is through doing things' will be used present indefinite tense. Verb is made singular by in place of 'are through doing things'. adding 's/es' in the end of the verb. Correct SentenceCorrect sentenceThe simplest way to get proper recognition is through Soni takes her daughter to the park in the evening in a doing things with dedication and sincerity. stroller. Spotting Error

19

YCT

78.

In the given sentences, identify the segment which contains a grammatical error. Everyone are asking for passes to the music festival at Nehru Park. (a) at Nehru Park (b) for passes (c) to the music festival (d) Everyone are asking SSC CHSL (Tier-I) 19/10/2020 (Shift-II) Ans. (d) : In the above sentence "Everyone is asking" will be used in place of "Everyone are asking" because if each, Everyone, one etc used as a singular subject then the verb is used in singular form. Correct sentenceEveryone is asking for passes to the music festival at Nehru Park. 79. In the sentence identify the segment which contains the grammatical error. Some of the issues discussed on the media nowadays has no relevance to the nationbuilding process. (a) on the media nowadays (b) to the nation-building process (c) some of the issues discussed (d) has no relevance SSC CHSL (Tier-I) 19/03/2020 (Shift-II) Ans. (d) : In the above sentence 'have no relevance' will be used in place of 'has no relevance' because the subject (Some of the issues) of the sentence is plural. hence plural verb (have) will be used. Correct sentenceSome of the issues discussed on the media nowadays have no relevance to the nation-building process. 80. In the given sentence identify the segment which contains the grammatical error. Your skill lie in cheering up people who are stressed out, wound up or generally just annoyed. (a) Your skill lie (b) who are stressed out wound up (c) or generally just annoyed (d) in cheering up people SSC CHSL (Tier-I) 12/10/2020 (Shift-I) Ans. (a) : In the above sentence 'your skill lies' will be used in place of 'your skill lie' because the subject ' your skill' is singular Noun. Hence singular verb (Lies) will be used. Verb is made singular by adding 's/es'. Correct sentenceYour skill lies in cheering up people who are stressed out, wound up or generally just annoyed. 81. Find the part of the given sentence that has an error in it. If there is no error, choose 'No error'. Sterling is only twenty four but have achieved so much in his professional football career. (a) but have achieved so much (b) in his professional football career (c) No error (d) Sterling is only twenty four SSC MTS – 05/08/2019 (Shift-III) Ans. (a) : In the above sentence 'but has achieved so much' will be used in place of 'but have achieved so much' because the subject (Sterling) is singular. Hence singular verb 'has' will be used. Correct sentenceSterling is only twenty four but has achieved so much in his professional football career. Spotting Error

82.

Find the part of the given sentence that has an error in it. If there is no error, choose 'No error'. The doctor with his interns are due to arrive here by noon./No error (a) interns are due (b) No error (c) The doctor with his (d) to arrive here by noon SSC MTS – 05/08/2019 (Shift-II) Ans. (a) : In the above sentence 'is' will be used in place of 'are' because the main subject 'the doctor' is singular. Hence singular verb will be used. Correct sentenceThe doctor with his interns is due to arrive here by noon. 83. Identify the segment in the sentence which contains the grammatical error. Virat Kohli is one of the best batsmen that world have seen. (a) Virat Kohli is (b) one of the best (c) world have seen (d) batsmen that the SSC MTS – 22/08/2019 (Shift-I) Ans. (c) : In the sentence singular verb 'has' will be used in place of plural verb "have" because 'that' is used for 'Virat Kohli' and it is singular Noun and singular verb is used after singular Noun. Correct sentence[Virat Kohli is one of the best batsmen that world has seen] 84. Identify the segment in the sentence which contains the grammatical error. One of the friends were doubtful that the plan would work. (a) that the plan (b) were doubtful (c) would work (d) one of the friends SSC MTS – 20/08/2019 (Shift-III) Ans. (b) : In the above sentence 'was doubtful' will be used in place of 'were doubtful' because plural Noun and singular verb is used after 'one of the'. One of the + Plural noun + Singular verb Correct sentence''One of the friends was doubtful that the plan would work.'' 85. Identify the segment in the sentence which contains the grammatical error. Each of these players have been warned not to repeat the silly mistakes. (a) Each of these players (b) the silly mistakes (c) warned not to repeat (d) have been SSC MTS – 16/08/2019 (Shift-III) Ans. (d) : In the above sentence singular verb 'has been' used in place of plural verb 'have been' because each of either of, Neither of always followed by plural Noun and singular verb.

20

Each of / Either of / Neither of + Plural noun + Singular verb

Correct sentenceEach of these players has been warned not to repeat the silly mistakes. YCT

86.

Identify the segment in the sentence which Ans. (d) : In the above sentence 'has been a fan' will be contains the grammatical error. used in place of 'have been a fan' because the subject' A Superintendent of Police have the authority The 91 year old lady' of The sentence is singular. to suspend an inspector. Hence singular verb 'has' will be used for it. (a) authority to suspend Correct Sentence(b) have the The 91 year old lady has been a fan of cricket for decades. (c) A Superintendent of Police (d) an inspector 90. From the given options, identify the segment in SSC MTS – 16/08/2019 (Shift-I) the sentence which contains the grammatical error. Ans. (b) : In the above sentence 'A superintendent of The constitutional order on which the Indian police is a singular subject and singular verb always State run is that there shall be no used with singular subject. Hence singular verb 'has' discrimination on the basis of religion. will be used in place of plural verb 'have'. (a) There shall be (b) order on which Correct sentence(c) on the basis of (d) Indian State run A Superintendent of Police has the authority to SSC GD – 02/03/2019 (Shift-II) suspend an inspector. 87. Identify the segment in the sentence which Ans. (d) : In the above sentence 'Indian state runs' will be used in place of 'Indian state run' because the contains the grammatical error. subject "The Indian state" is singular. Hence singular The mother as well as her children were verb (runs) will be used. brought to the police station for interrogation. Correct sentence(a) her children were brought The constitutional order on which the Indian State runs (b) to the police station is that there shall be no discrimination on the basis of (c) for interrogation religion. (d) The mother as well as 91. From the given options, identify the segment in SSC MTS – 14/08/2019 (Shift-II) the sentence which contains the grammatical Ans. (a) : In the above sentence 'her children was error. brought' will be used in place of her children were We must visit this ancient temple which are a brought' because when Two nouns are connected with hundred years old. "as well as" then the verb is used according to the first (a) this ancient temple (b) a hundred years old Noun (Mother) which is singular. Hence singular verb (c) We must visit (d) which are (was) will be used. SSC GD – 18/02/2019 (Shift-III) Correct sentenceAns. (d) : 'This ancient temple' used in above sentence The mother as well as her children was brought to the is in singular number, so after which it would be police station for interrogation. appropriate to use singular verb 'is' in place of plural 88. Find the part of the given sentence that has an verb 'are' because 'which' is relative pronoun used for error in it. If there is no error, choose 'No 'this ancient temple'. error'. correct sentence It take her a long time to get over her failure in We must visit this ancient temple which is a hundred the examination./ No error years old. (a) long time to get over 92. From the given options, identify the segment in (b) her failure in the examination the sentence which contains the grammatical (c) No error error. (d) It take her a The researchers have found that all species SSC MTS – 07/08/2019 (Shift-III) makes a constructive contribution to the ecology of the earth. Ans. (d) : : In the above sentence singular verb 'takes' (a) a construction contribution to will be used in place of plural verb 'take' because (b) that all species makes Pronoun 'it' is singular and singular verb used for it. (c) the ecology of the earth Correct sentence(d) The researchers have found [It takes her a long time to get over her failure in the SSC GD – 18/02/2019 (Shift-II) examination]. 89. Find the part of the given sentence that has an Ans. (b) : In the above sentence 'That all species make' error in it. If there is no error, choose 'No will be used in place of "that all species makes" because plural verb (make) will be used according to error'. The 91 year old lady have been a fan of cricket plural subject 'all species'. Note— Assertive sentence of Present Indefinite tense for decades./No error is formed by adding s/es at the end of the verb to make (a) of cricket for decades singular verb. (b) No error Correct sentence (c) The 91 year old lady The researchers have found that all species make a (d) have been a fan SSC MTS – 06/08/2019 (Shift-II) constructive contribution to the ecology of the earth. Spotting Error

21

YCT

93.

From the given options, identify the segment in the sentence which contains the grammatical error. The population of Sri Lanka are less than that of India. (a) than that (b) Sri Lanka are (c) of India (d) The population SSC GD – 11/02/2019 (Shift-II) Ans. (b) : In the above sentence subject 'The population' is in singular, hence the use of helping verb will be in singular. So in the place of plural verb 'are' use of singular verb 'is' would be appropriate. Correct sentence The population of Sri Lanka is less than that of India.

C. Form of Verb (Tense/number) 94.

Find the part of the given sentence that has an error in it. If there is no error, choose 'No error'. There is no big deal if she didn't came to the party. (a) came to the party (b) No error (c) There is no big (d) deal if she didn't SSC MTS 04/05/2023 (Shift-II) Ans. (a) : For the above given parts of sentence part (a) will be incorrect because did not come did + not + VI is used in the place of didn't came (didn't + v2). Thus, the correct answer is option (a). 95. The following sentence has been split into four segments. Identify the segment that contains a grammatical error. Her bookstore flourished/because/she picks/ a good location. (a) Her bookstore flourished (b) Because (c) A good location (d) She picks SSC GD 31/01/2023 (Shift-II) Ans. (d) : There is an error in option (d) and 'picked' will be used instead of 'picks' because from the use of 'flourished' (V2) in the sentence it is clear that the given sentence is in past tense. Therefore, use of 'picked' (V2) would be appropriate. Correct SentenceHer bookstore flourished because she picked a good location. 96. The given sentence is decided into three segments. Select the option that has the segment with a grammatical error. If there is no error, select 'no error' as your answer. It will take/5 minutes to/preparing a cup of tea. (a) No error (b) Preparing a cup of tea (c) It will take (d) 5 minutes to SSC GD 27/01/2023 (Shift-I) Ans. (b) : There is an error is option (b) of the given sentence. 'Prepare' will be used instead of 'prepring' because the first form of the verb (V1) is used after the infinitive 'to' Therefore, use of 'prepare' would be appropriate. Correct SentenceIn will take 5 minutes to prepare a cup of tea. Spotting Error

97.

The following sentence has been split into four segments. Identify the segment that contains a grammatical error. My sister / is come for a short holiday / from the US. (a) My sister (b) from the US (c) a short holiday (d) is come for SSC GD 13/02/2023 (Shift-I) Ans. (d) : In the above sentence 'has' will be used in place of 'is' because according to sense of the sentence it is in present perfect tense and in it, 'has/have' is used as a helping verb. Hence option (d) is correct answer. 98. Parts of the following sentence have been underlined and given as options. Select the option that contains an error. The Dutta company has not being making new school bags since the lockdown commenced. (a) commenced (b) has not (c) being (d) since SSC CGL (Tier-I) 19/07/2023 (Shift-IV) Ans. (c) : In the above given underlined words are correct except 'being' so been (V3) will be used in place of 'being' because sentence is in present perfect tense then his structure will be Sub + has/have + V3 + Obj. Thus, the correct answer is option (c) 99. The following sentence has been split into four segments. Identify the segment that contains a grammatical error. Binay said that / he is coming / from / his village, Jaygarh. (a) from (b) his village, Jaygarh (c) he is coming (d) Binary said that SSC CGL (Tier-I) 21/07/2023 (Shift-II) Ans. (c) : In the above given sentence, the segment 'he is coming' is grammatically error because this sentence will be indirect form of past continuous tense and when the reporting verb (in indirect narration) is in past tense, verb of the reported speech is in past tense. Hence 'he was coming' will be used in place of 'he is coming'. The correct sentence- 'Binary said that he was coming from his village, Jaygarh.' 100. The following sentence has been split into four segments. Identify the segment that contains a grammatical error. The doctor came / after the patient / had / pass away. (a) after the patient (b) pass away (c) The doctor came (d) had SSC CGL (Tier-I) 25/07/2023 (Shift-IV) Ans. (b) : The segment of the above given sentence 'pass away is grammatically error, because Passed away (V3) is used in the place of 'pass away' (V1), Note- The given sentence is in Past Perfect Tense in which 'V3' is used with 'had'. The correct sentence is- 'The docter come after the patient had passed away'.

22

YCT

101. The following sentence has been divided into parts. One of them may contain an error. Select the part that contains the error from the given options. If you don’t find any error, mark ‘No error’ as your answer. My students / had expect / guidance from my end. (a) had expect (b) My students (c) guidance from my end (d) No error SSC CGL (Tier-I) 20/07/2023 (Shift-I) Ans. (a) : In the above given sentence there is containing an error in option (a) ‘had expect’ because sentence is in ‘past perfect tense’ (S + had + v3 +other) ‘The word ‘had expect’ is replaced by ‘had expected’. Correct SenteceThe my students had expected guidance from my end. 102. The following sentence has been split into four segments. Identify the segment that contains a grammatical error. I play / cricket in / my childhood / with my sister. (a) Play (b) With my sister (c) Cricket in (d) My childhood SSC CGL (Tier-I) 24/07/2023 (Shift-III) Ans. (a) : In the given sentence, 'Played' will be used in the place of 'Play' because the sense of the sentence is Past Indifinite tense and (verb2) will be used. Correct SenteceI played cricket in my childhood with my sister. 103. The following sentence has been split into four segments. Identify the segment that contains a grammatical error. When I was in 5th grade,/I use to / ride on my / red bicycle. (a) When I was in 5th grade, (b) ride on my (c) I use to (d) red bicycle SSC CHSL (Tier-I) 17/08/2023 (Shift-II) Ans. (c) : Part (c) of the sentence 'I use to' is erroneous. It has to be replaced with modal verb 'I used to' . used to refers to talking about something that happened in the past or existed in the past. used to + infinitive is used to talk about past situation that in no longer true. Correct Sentence when I was in 5th grade I used to ride on my red bicycle. 104. Parts of the following sentence have been given as options. One of them may contain an error. Select the option that contains the error. If you don’t find any error, mark ‘No error’ as your answer. Nobody from the gathering of odd people could guess the accident occurring blast. (a) Nobody from the gathering (b) the accident occurring blast (c) No error (d) of odd people could guess SSC CHSL (Tier-I) 08/08/2023 (Shift-II) Spotting Error

Ans. (b) : Parts of the above sentence in option (b) 'the accident occuring blast' is grammatically error because 'Occurred' (past participle) is used instead of 'occurring'. The correct segment – 'the accident occurred blast.' Thus, the correct answer is option (b). 105. Find the part of the given sentence that has an error in it. If there is on error, choose 'No error' . Workers and companies in all sectors can contribute their skills to meet society new needs. (a) sectors can contribe their skills (b) No error (c) to meet society new needs. (d) Workers and companies in all SSC CHSL (Tier-I) 21/03/2023 (Shift-II) Ans. (c) : In the above sentence, 'meeting' will be used in place of 'meet'. because 'gerund' (v1 + ing) is used after contribute Correct Sentence Workers and companies in all sectors can contribute their skills to meeting society new needs. 106. The following sentence has been split into four segments. Identify the segment that contains a grammatical error. We / has seen / that movie / before. (a) before (b) We (c) that movie (d) has seen SSC CGL Mains 26/10/2023 (Shift-I) Ans. (d) : In the above sentence 'have seen' will be used in place of 'has seen' because the subject (we) of the sentence is plural noun. Hence plural verb 'have' will be used. Correct Sentence We have seen that movie before. 107. Parts of the following sentence are given as option. Ldentify the part contains a grammatical error. If there is on error, select ‘No error’. Shyam, so good with weapons, fail to find the words / to tell him everything he’d dome / so far, all the compromises he’d made, had been for love. (a) to tell him that everything he’d done (b) Shyam, so good with weapons, fail to find the words (c) so far, all the compromises he’d made, had been for love. (d) No error SSC CHSL (Tier-I) 20/03/2023 (Shift-I) Ans. (b) : In the above sentence, "Failed" will be used in place of "fail" because given sentence is in 'Past Indefinite' tense. Hence V2 will be used. Correct Sentence: Shyam, so good with weapons failed to find the words to tell him that everything he'd done so far, all the compromises held made, had been for love.

23

YCT

108. Parts of the following sentence are given as option. Identify the part that contains a grammatical error. If there is no error, select ‘No error’. That will truly / been a milestone to / celebrate. (a) No error (b) celebrate. (c) been a milestone to (d) That will truly SSC CHSL (Tier-I) 20/03/2023 (Shift-I) Ans. (c) : In the above sentence, "be" will be used in place of 'been' because will be/shall be is used in Simple Future Tense. Correct Sentence: This will truly be a milestone to celebrate. 109. Identify the segment in the sentence which contains the grammatical error. If there is no error, select 'No error' We watched the workmen to repair the tiles of the kitchen. (a) of the kitchen (b) We watched the workmen (c) No error (d) to repair the tiles SSC CPO-SI – 09/12/2019 (Shift-I) Ans : (d) In the above sentence ‘Repairing’ will be used in place of ‘repair’ because gerund (verb1 + ing) is used after 'watch'. (verb) Watch + somebody + doing + something Correct sentence :We watched the workmen repairing the tiles of the kitchen. 110. Identify the segment in the sentence which contains a grammatical error. If there is no error, then select the option 'No error'. Sh! Someone listens to our conversation. (a) No error (b) listens to (c) Sh! Someone (d) our conversation SSC CPO-SI – 23/11/2020 (Shift-I) Ans. (b) : In the above sentence, ‘is listening to’ would be appropriate instead of ‘listens to’. Because the sentence is giving continuous sense. Hence the structure of present continuous tense will be used. subject + is / am / are + verb1 + ing + object + other words Correct sentence :Sh! someone is listening to our conversation 111. The following sentence has been split into four segments. Identify the segment that contains a grammatical error. My cousin, Jawahar,/visited me at home/and asked me/what I am reading. (a) what I am reading (b) visited me at home (c) and asked me (d) My cousin, Jawahar SSC Stenographer (Grade C & D) 12.11.2021 Shift-I Ans. (a) : In the above sentence 'What I was reading' will be used in place of 'What I am reading' because if 'first clause is in past, then second clause will also be in past tense. Correct SentenceMy cousin, Jawahar, visited me at home and asked me what I was reading. Spotting Error

112. The following sentence has been split into four segments. Identify the segment that contains a grammatical error. She has given/me a book/last week/which I returned. (a) last week (b) She has given (c) me a book (d) which I returned SSC Stenographer (Grade C & D) 12.11.2021 Shift-I Ans. (b) : In the above sentence 'She had given' will be used in place of 'She has given' because last week makes it clear that the sentence in past tense. So the helping verb of past perfect tense (had) will be used. Correct SentenceShe had given me a book last week which I returned. 113. In the given sentence, identify the segment that contains a grammatical error. 'Occupation' means the work or job people do not to earning money. (a) 'Occupation' means (b) the work or job (c) to earning money (d) people do SSC Stenographer (Grade C & D) 12.11.2021 Shift-II Ans. (c) : In the above sentence 'to earn' will be used in place of 'to earning' because 'VerbI' is used after infinitive 'to'. Correct sentence– 'Occupation' means the work or job people do not to earn money. 114. In the sentence identify the segment that contains a grammatical error. Suman told me that she didn't know how to operates her net banking account. (a) operates (b) how to (c) told me (d) she didn't know SSC Stenographer (Grade C & D) 15.11.2021 Shift-II Ans. (a) : In the above sentence 'operate' will be used in place of 'operates' because 'to + VI' is used (to + VI without s/es) Correct sentence:Suman told me that she didn't know how to operate her net banking account. 115. Identify the segment in the sentence which contains a grammatical error. I remember to meet her at a conference in Delhi last year. (a) to meet her (b) I remember (c) at a conference (d) in Delhi last year SSC Stenographer (Grade C & D) 15.11.2021 Shift-I Ans. (a) : Use of both indefinite and gerund is possible with Remember. But if we talk about the past in the sentence. We use the gerund and in the future, we will use the infinitive. In the given sentence, Last year means past has been talked about. So 'meet her' will be replaced by "meeting her". Hence option (a) will be correct. 116. The following sentence has been split into four segments. Identify the segment that contains a grammatical error. She had resign/from the/post before/he apologised. (a) he apologised (b) post before (c) she had resign (d) from the SSC CHSL 24.05.2022 Shift-III

24

YCT

Ans. (c) : In the above sentence 'had resigned' will be used in place of 'had resign' because verb3 is used after 'had' Note:– If the two clauses connected with After/Before, the work that was completed earlier is used in past perfect and the work that was completed later is used in past indefinite tense. Correct SentenceShe had resigned from the post before he apologised. 117. The following sentence has been split into four segments. Identify the segment that contains a grammatical error. My boss keeps telling me/all the time/ that money/doesn't grows on trees. (a) My boss keeps telling me (b) all the time (c) doesn't grows on trees (d) that money SSC CHSL 27.05.2022 Shift-I Ans. (c) : In the above sentence 'doesn't grow on tree' will be used in place of doesn't grows on tree because the verb of present Indefinite negative sentence, is used without s/es. Whereas in affirmative sentence VI + s/es is used. Correct Sentence My boss keeps telling me all the time that money doesn't grow on tree. 118. The following sentence has been split into four segments. Identify the segment that contains a grammatical error. I goes/to the school/every day/irrespective of the weather. (a) to the school (b) every day (c) irrespective of the weather (d) I goes SSC CHSL 27.05.2022 Shift-I Ans. (d) : In the above sentence 'go' will be used in place of 'goes' because plural verb is always used with 'I'. Note– The natural form of all verb (form) is plural and it is made singular by adding s/es. Correct SentenceI go to the school every day irrespective of the weather. 119. The following sentence has been split into four segments. Identify the segment that contains a grammatical error. Elina is/travelling/since/yesterday. (a) yesterday (b) Elina is (c) since (d) travelling SSC CHSL 27.05.2022 Shift-III Ans. (b) : In the above sentence 'has been' will be used in place of 'is' because it is clear from the use of since + time (yesterday) that the sentence is in present perfect continuous tense. Therefore, 'has been' will be used as a helping verb for the singular subject (Elina) in the sentence. Correct sentence Elian has been travelling since yesterday. Spotting Error

120. The following sentence has been split into four segments. Identify the segment that contains a grammatical error. There is many ways/to make/a stranger feel/welcome in the community. (a) welcome in the community (b) a stranger feel (c) to make (d) there is many ways SSC CHSL 26.05.2022 Shift-II Ans. (d) : In the above sentence 'are' will be used in the place of 'is' because the subject 'many ways' is plural. Hence plural verb (are) will be used. Note– When 'there' is used at the beginning of the sentence, the verb is used according to the subject used after the verb. Correct sentence– There are many ways to make a stranger feel welcome in the community. 121. Parts of the following sentence have been given as options. One of them may contain an error. Select the part that contains the error from the given options. If you don't find any error, mark 'No error' as your answer. The goons fled after kill a boy in the street. (a) No error (b) boy in the street (c) after kill a (d) The goons fled SSC MTS-18/07/2022 Shift-I Ans. (c) : In the above sentence 'after killing a' will be used in place of 'after kill a' because (V + ing) form of verb will be used after preposition (after). Correct Sentence – The goons fled after killing a boy in the street. 122. The following sentence has been split into segments. Identify the segment that contains a grammatical error. My guide / was blind / but he bring / me home safely. (a) but he bring (b) My guide (c) was blind (d) me home safely SSC CGL (Tier-I) 21/04/2022 Shift-III Ans. (a) : In the above sentence 'but he brought' will be used in place of 'but he bring' because the first clause of the sentence is in past. Then sub-ordinate Clause (if clause) will also be in the 'past'. So 'verb2' will be used. Correct sentenceMy guide was blind but he brought me home safely 123. The following sentence has been split into four segments. Identify the segment that contains a grammatical error. Will you / care for / a cup of / hot coffee? (a) a cup of (b) care for (c) Will you (d) hot coffee SSC CGL (Tier-I) 20/04/2022 Shift-III Ans. (c) : In the above sentence 'would you' will be used in place of 'will you' because, 'Would' is used for formal "Invitation/ Proposal". Correct sentence– Would you care for a cup of hot coffee?

25

YCT

124. The following sentence has been split into four segments. Identify the segment that contains a grammatical error. He allowed / his son to drive / but he warn him/ of the danger. (a) his son to drive (b) of the danger (c) but he warn him (d) He allowed SSC CGL (Tier-I) 11/04/2022 Shift-II Ans. (c) : In the above sentence 'but he warned him' will be used in place of 'but he warn him' because it is in Past Tense, then the second clause will also be in Past tense in which 'verb2' is used. Correct Sentence– He allowed his son to drive but he warned him of the danger. 125. The following sentence has been split into four segments. Identify the segment that contains a grammatical error. Did you / recall to meet / her at the party / last night? (a) last night (b) Do you (c) recall to meet (d) her at the party SSC CGL (Tier-I) 11/04/2022 Shift-II Ans. (c) : In the above sentence 'recall meeting' will be used in place of 'recall to meet' because recall is followed by gerund (V + ing) not infinitive (to +V1) correct sentence – Did you recall meeting her at the party last night 126. The following sentence has been split into four segments. Identify the segment that contains a grammatical error. you should / have respond / to my query / sooner. (a) to my query (b) sooner (c) have respond (d) you should SSC CGL (Tier-I) 13/04/2022 Shift-II Ans. (c) : In the above sentence 'have responded' will be used in the place of 'have respond' because past participle (V3) is used after 'should have'. Correct sentence you should have responded to my query sooner. 127. The following sentence has been divided into parts. One of them may contain a grammatical error. Select the part that contains the error from the given options. If you don’t find any error, mark ‘No error’ as your answer. If the economy fails / this year it reflect badly / on the government. (a) on the government (b) this year it reflect badly (c) No error (d) If the economy fails SSC CGL (Tier-I) 11/04/2022 Shift-I Ans. (b) : In the above sentence 'this year it reflects badly' will be used in place of 'this year it reflect badly' because subject 'it' is singular. Hence singular verb reflects) will be used. Note :– Verb is made singular by adding s/es while Noun is made plural by adding s/es/ Correct sentence– If the economy fails this year it reflects badly on the government. Spotting Error

128. The following sentence has been divided into parts. One of them may contain an error. Select the part that contains the error from the given options. If you don't find any error, mark 'No error' as your answer. Sometimes I act / like a fool, and she has / to step into save me. (a) Sometimes I act (b) like a fool and she has (c) No error (d) to step into save me SSC MTS-05/07/2022 Shift-III Ans. (d) : In the above sentence 'to step to save me' will be used in place of 'to step into save me' because Noun form is needed in the sentence, so infinitive (to + VI) will be used here as a noun. Correct Sentence– Sometime I act like a fool, and she has to step to save me. 129. The following sentence has been divided into parts. One of them may contain an error. Select the part that contains the error from the given options. If you don't find any error, mark 'No error' as your answer. Frank has being making / the rounds, advancing a theory ' that I believe is not without merit. (a) the rounds, advancing a theory (b) No error (c) Frank has being making (d) that I believe is not without merit SSC MTS-07/07/2022 Shift-II Ans. (c) : In the above sentence frank has been making' will be used in place of 'Frank has being making' because the sentence is in present perfect continuous tense. Correct Sentence – Frank has been making the rounds, advancing a theory that I believe is not without merit. 130. Parts of the following sentence have been given as options. One of them may contain an error. Select the part that contains the error from the given options. If you don's find any error, mark 'No error' as your answer. At Seven began the education that was to making the young boy a citizen and a soldier. (a) No error (b) At seven began he education (c) boy a citizen and a soldier (d) that was to making the young SSC MTS-07/07/2022 Shift-II Ans. (d) : In the above sentence 'that was to make the young' will be used in place of 'that was to making the young' because 'to + VI' is use as an infinitive which works as a noun. Correct sentence – At seven began the education that was to make the young boy a citizen and a soldier.

26

YCT

131. The following sentence has been divided into parts. One of them may contain an error. Select the part that contains the error from the given options. If you don’t find any error, mark ‘No error’ as your answer. She didn’t liked / any disturbance / during her studies. (a) She didn’t liked (b) during her studies (c) any disturbance (d) No error SSC Constable GD-23/11/2021 Shift-III Ans. (a) : In the above sentence 'didn't like' will be used in the place of 'didn't liked' because first form of verb (VI) is used after helping verb do/did/does. Correct Sentence- She didn't like any disturbance during her studies. 132. The following sentence has been split into four segments. Identify the segment that contains a grammatical error. Please ensure / that you covering your face / with a mask / when you go out. (a) that you covering your face (b) with a mask (c) Please ensure (d) when you go out SSC Constable GD-22/11/2021 Shift-II Ans. (a) : In the above sentence 'should cover' will be used in place of 'covering' because sentence is in imperative and according to the sense of sentence modal verb 'should' after that 'verb1' is used. Correct sentence– Please ensure that you should cover your face with a mask when you go out. 133. The following sentence has been split into four segments. Identify the segment that contains a grammatical error. I entered my room / and switch on the lights / but it was / still dark. (a) and switch on the lights (b) I entered my room (c) but it was (d) still dark SSC Constable GD-22/11/2021 Shift-II Ans. (a) : In the above sentence 'Switched on' will be used in place of 'Switch on' because it is clear from the use of 'entered was' in the sentence that the sentence in the past tesne. Hence Simple past verb (2nd form) will be used. Correct Sentence– I entered my room and switched on the lights but it was still dark. 134. The following sentence has been split into four segments. Identify the segment that contains a grammatical error. Who do you think / who stole / the painting / from the museum? (a) who stole (b) the painting (c) from the museum (d) Who do you think SSC Constable GD-18/11/2021 Shift-I Spotting Error

Ans. (a) : In the above sentence 'Who has stolen' will be used in place of 'who stole' because it is clear from the use of 'do' that if first clause is in present tense then second clause also be in present tense. Correct Sentence– Who do you think who has stolen the painting from the museum. 135. The following sentence has been split into four segments. Identify the segment that contains a grammatical error. Mona is from Paris. / She has live / there all / her life. (a) there all (b) her life (c) Mona is from Paris (d) she has live SSC Constable GD-14/12/2021 Shift-III Ans. (d) : In the above sentence 'She has lived' will be used in place of 'She has live' because verb3 will be used after 'has'. Correct sentence– Mona is from Paris she has lived there all her life 136. The following sentence has been split into four segments. Identify the segment that contains a grammatical error. Fortunately / the fire did not / caused much damage / to the building. (a) caused much damage (b) Fortunately (c) to the building (d) the fire did not SSC Constable GD-14/12/2021 Shift-II Ans. (a) : In the above sentence 'Cause' will be used in place of Caused' because base form of verb (VI) is used after helping verb do/does/did. Correct SentenceFortunately the fire did not cause much damage to the building. 137. Select the most appropriate option that can substitute the underlined segment in the given sentence. If there is an need to substitute it, 'No substitution required'. I have completed the work two hours ago. (a) has completed (b) completed (c) will have completed (d) No substitution required SSC Constable GD-10/12/2021 Shift-III Ans. (b) : In the above sentence 'Completed' will be used in the place of 'have completed' because The use of "two hours ago" makes it clear that the sense of the sentence is in past tense. So the verb of past Indefinite tense 'Verb2' will be used. Correct sentence– I completed the work two hours ago. 138. The following sentence has been split into four segments. Identify the segment that contains a grammatical error. I am thinking / your idea will be / really appreciated / by everyone. (a) I am thinking (b) by everyone (c) your idea will be (d) really appreciated SSC Constable GD-07/12/2021 Shift-II

27

YCT

Ans. (a) : In the above sentence 'I think' will be used in place of 'I am thinking' because as a verb 'think' is not used in continuous tense. Note :– Some verb like think, feel, believe not used in progressive sense or continuous tense. Correct sentence– I think your idea will be really appreciated by everyone. 139. Parts of the given sentence have been given as options. One of them contains a grammatical error. Select the option that has the error. The news reporter highlighting the mysterious death of the film actor repeatedly. (a) highlighting (b) repeatedly (c) the mysterious death (d) of the film actor SSC Constable GD-06/12/2021 Shift-III Ans. (a) : In the above sentence 'highlighted' will be used in place of 'highlighting' because News reporter is telling the incident of 'Past', So V2 (Highlighted) will be used. Correct sentence– The news reporter highlighted the mysterious death of the film actor repeatedly. 140. The following sentence has been split into four segments, Identify the segment that contains a grammatical error. How many / languages do / most children / in India known? (a) in India known (b) most children (c) languages do (d) How many SSC Constable GD-03/12/2021 Shift-II Ans. (a) : In the above sentence 'Know' will be used in place of 'known' because the use of 'do' makes it clear that the sentence is in Present Indefinite. Hence 'Verb Ist' (VI) will be used. Correct Sentence How many languages do most children in India Know? 141. The following sentence has been split into four segments. Identify the segment that contains a grammatical error. God is great/and we could/be grateful to him for all the gifts/he has bestowed on us. (a) God is great (b) and we could (c) he hs bestowed on us (d) be grateful to him for all the gifts SSC Constable GD-01/12/2021 Shift-I Ans. (b) : In the above sentence 'and we must' will be used in place of 'and we could' because can/could (modal) is used to express a possibility, while 'must' (modal) is used to express an obligation to do something. Correct sentence– God is great and we must be grateful to him for all the gifts he has bestowed on us. 142. In the sentence identify the segment which contains the grammatical error. Torrential rains and winds of upto 170 km per hour swept away roads, homes and bridges and knocking down power and communication lines. Spotting Error

(a) homes and bridges (b) knocking down (c) winds of upto (d) swept away roads SSC CGL (Tier-I) – 04/06/2019 (Shift-III) Ans. (b) : In the above sentence 'Knocked down' will be used in place of 'knocking down' because simple past tense is used in the sentence according to the sense of the sentence. Correct SentenceTorrential rains and wind of upto 170 km per hour swept away roads, homes and bridges and knocked down power and communication lines. 143. The following sentence has been divided into parts. One of them contains an error. Select the part that contains the error from the given options. Ranbir could not went/ to the award ceremony/ as he was busy/shooting/for a film/in Maldives. (a) Ranbir could not went (b) for a film (c) to the award ceremony (d) as he was busy SSC CGL (Tier-I) –20/08/2021 (Shift-I) Ans. (a) : In the above sentence 'Ranbir Could not go' will be used in place of 'Ranbir could not went' because original form (Verb1) comes after model (could). Correct SentenceRanbir could not go to the award ceremony as he was busy shooting for a film in Maldives. 144. Identify the segment in the sentence which contains a grammatical error. The tired and vexed travellers waiting at the airport for a long time. (a) vexed travellers (b) The tired and (c) waiting at the airport (d) for a long time SSC CGL (Tier-II) 16/11/2020 (3 pm - 5 pm) Ans. (c) : In the above sentence 'have been waiting' will be used in place of 'waiting' because the use of 'for a long time' makes. It clear that the sentence is in present perfect continuous tense. Hence verb (has/have + verb + ing) will be used. Correct sentence- The tired and vexed travellers have been waiting at the airport for a long time. 145. Identify the segment in the sentence which contains a grammatical error. Laws and rules are made to safeguarding our rights and protect us. (a) are made to (b) safeguarding our rights (c) Laws and rules (d) and protect us SSC CGL (Tier-II) 16/11/2020 (3 pm - 5 pm) Ans. (b) : In the above sentence 'to safeguard' will be used in place of 'to safeguarding' because as a infinitive (to+V1) will be used as a noun. Correct sentence- Laws and rules are made to safeguard our rights and protect us. 146. In the sentence identify the segment which contains the grammatical error. She forgot lock the door when she went out in a hurry. (a) in a hurry (b) She forgot lock (c) the door when (d) she went out SSC CGL (Tier-I) – 07/06/2019 (Shift-I)

28

YCT

Ans : (b) The use of 'forget to' would be appropriate Because when two verbs (forgot, Lock) are used side by side in a sentence, then the noun from the later verb (infinitive) (to + VI) to will be used. Correct sentence She forgot to lock the door when she went out in a hurry. 147. Identify the segment in the sentence which contains a grammatical error. I didn't knew you had gone to Goa for a vacation. (a) you had gone (b) to Goa (c) I didn't knew (d) for a vacation SSC CGL (Tier-II) 16/11/2020 (3 pm - 5 pm) Ans. (c) : In the above sentence 'know' will be used in place of 'knew' (V2) because (V1) will be used after helping verb do/did/does. Correct sentence- I didn't know you had gone to Goa for a vacation. 148. Identify the segment in the sentence, which contains the grammatical error. The files you were look for are placed on the table. (a) are placed (b) you were look for (c) The files (d) on the table SSC CPO-SI (Tier-II) 27/09/2019 (3 pm - 5 pm) Ans. (b) : In the above sentence 'Looking for' will be used in place of 'Look for' because sentence is in past continuous tense. was/were + verb1+ing Correct sentence- The files you were looking for are placed on the table. 149. Identify the segment in the given sentence which contains the grammatical error. You had better not to try and feed the animals in the zoo, it is forbidden. (a) and feed the animals (b) it is forbidden (c) in the zoo (d) You had better not to try SSC CHSL (Tier-I) 14/10/2020 (Shift-III) Ans. (d) : In the above sentence 'try' will be used in place of 'to try' because infinitive 'to' is not used after 'had better'. Note - Needn't, dare not, would rather, rather than, had better etc. bare infinitive (verb without to). Correct sentence : You had better not try and feed the animals in the zoo, it is forbidden. 150. Identify the segment in the given sentence which contains the grammatical error. We noticed that most of the shoppers in the mall looking for packets of ready to cook food. (a) We noticed that (b) most of the shoppers (c) packets of ready to cook food (d) in the mall looking for SSC CHSL (Tier-I) 13/10/2020 (Shift-III) Ans. (d) In the above sentence 'in the mall were Looking for' will be used in place of 'in the mall looking for' because sub-ordinate clause is in past continuous tense. Which subject 'shoppers' is plural. Hence plural helping verb 'were' will be used. Correct Sentence :We noticed that most of the shoppers in the mall were looking for packets of ready to cook food. Spotting Error

151. Identify the segment in the sentence which contains a grammatical error. If there is no error, then select the option 'No error'. The sun rises at 6:35 yesterday morning. (a) No error (b) The sun (c) yesterday morning (d) rises at 6:35 SSC CPO-SI – 23/11/2020 (Shift-I) Ans. (d) : In the above sentence "rose at 6:35" will be used in place of "rises at 6:35" because it is clear from the use of yesterday, the sentence is in past Indefinite tense. Hence second form (V2) of verb will be used. Correct sentence :The sun rose at 6:35 a.m. yesterday morning. 152. In the following, some part of the sentence may have errors. Find out which part of the sentence has an error and select the appropriate option. If a sentence is free from error, select No Error. This will explains the decision taken (1)/to impose a blockade on the country in (2)/ the midst of a crisis. (3)/ No error. (4) (a) 1 (b) 2 (c) 3 (d) 4 SSC CGL (Phase-II) 17/02/2018 Ans. (a) In the above sentence 'explain' will be used in place of 'Explains'. because "Verb1" is used after will in future Indefinite tense. Correct Sentence'This will explain the decision taken to impose ...... 153. In the following, some part of the sentence may have errors. Find out which part of the sentence has an error and select the appropriate option. If a sentence is free from error, select No Error. Many firms fail because when they begin (1)/exporting, they have not research the (2)/ target Markets or have not planned enough.(3)/No error(4) (a) 1 (b) 2 (c) 3 (d) 4 SSC CGL (Phase-II) 17/02/2018 Ans. (b) In the above sentence 'have' will be used as an auxiliary verb, So the verb coming after it will be in third form of verb (V3). Hence "Researched" will be used in place of "research." Correct Sentence Many firms fail because when they begin exporting, they have not researched the.... 154. Identify the segment in the sentence which contains the grammatical error Nobody is allowed to entered the colony during the lockdown. (a) Is allowed (b) Nobody (c) During the lockdown (d) To entered

29

SSC Sel. Post Phase-VIII (H.L.) 09.11.2020 (Shift-I)

Ans. (d) : In the above sentence 'to enter' will be used in place of 'to entered' because verb (V1) is used after infinitive 'to'. Correct Sentence:Nobody is allowed to enter the colony during the lockdown. YCT

155.

In the following, some part of the sentence may have errors. Find out which part of the sentence has an error and select the appropriate option. If a sentence is free from error, select No Error. As with people, some monkeys are lazy, (1)/like those who sleep all day in the zoo, (2)/and some were industrious. (3)/ No error (4) (a) 1 (b) 2 (c) 3 (d) 4 SSC CGL (Phase-II) 17/02/2018 Ans. (c) The main clause of the above sentence is in present tense, therefore, the sub-ordinate clause will also be in present tense. Hence 'are' will be used in place of 'were'. Correct sentenceAs with people, some monkeys are lazy, like those who sleep all day in the zoo, and some are industrious. 156. In the following, some part of the sentence may have errors. Find out which part of the sentence has an error and select the appropriate option. If a sentence is free from error, select No Error. No matter what Ashoka did in his earlier (1)/ years, in the ending he proved to be uncommonly (2)/ virtuous and wise. (3)/ No error (4) (a) 1 (b) 2 (c) 4 (d) 4 SSC CGL (Phase-II) 17/02/2018 Ans. (b) In the given sentence 'end' will be used in place of 'Ending', because according to the structure of the sentence, article 'the' is not used before the gerund. (V+ing) (in the End). Correct sentenceNo matter......years, in the end he proued to be uncommonly… 157. Identify the segment in the sentence which contains a grammatical error. She picked up the books lie on the table and put them on the shelf. (a) on the shelf (b) she picked up (c) put them (d) lie on the table SSC CPO-SI – 25/11/2020 (Shift-I) Ans. (d) : In the above sentence 'lay on' will be used in place of 'Lie on' because the sentence is in Past Indefinite. and verb2 "Lay" is used in this tense. lie ⇒ lay ⇒ lain

Ans. (b) : In the above sentence 'seemed' will be used in place of 'seem' as a past participle because the sentence in in past tense. Correct sentence One of the proposals received by us seemed very interesting. 159. Identify the segment in the sentence which contains a grammatical error. I was sorry to be hearing of his misfortune. (a) I was (b) sorry to (c) of his misfortune (d) be hearing SSC CPO-SI – 24/11/2020 (Shift-I) Ans. (d) : In the above sentence 'hear' will be used in place of 'be hearing' because according to sense of the sentence, Noun is needed here and (to + vI) infinitive works as noun. Note— Usually 'Sorry' is followed by infinitive (to+vI) Correct sentence : I was sorry to hear of his misfortune. 160. Identify the segment in the sentence which contains a grammatical error. If there is no error, then select the option 'No error'. I am hearing a strange noise now. (a) a strange (b) noise now (c) No error (d) I am hearing SSC CPO-SI – 23/11/2020 (Shift-I) Ans. (d) : In the above sentence 'hear' will be used in place of 'am hearing' because it is clear from the used of 'now' that the sentence is in Present Indefinite tense. Hence the verb 'hear' (verb1) will be used, because the subject of the sentence is I. Correct sentenceI hear a strange noise now. 161. Identify the segment in the sentence which contains a grammatical error. Sam took a little time to thought before he replied. (a) before he replied (b) a little time (c) Sam took (d) to thought SSC CPO-SI – 24/11/2020 (Shift-II) Ans. (d) : In the above sentence 'to think' will be used in place of 'to thought' because to + V1 (infinitive) is used. Note :– 1st form of verb is used to make infinitive which works as a 'noun'. Correct sentenceSam took a little time to think before he replied. 162. Identify the segment in the sentence which contains a grammatical error. The two trains gave the illusion of been stationary. ↓ ↓ ↓ (a) The two (b) trains gave Ist IInd IIIrd v form v form v form (c) of been stationary (d) the illusion Correct sentence SSC CPO-SI – 24/11/2020 (Shift-II) She picked up the books lay on the table and put them Ans. (c) : In the above sentence 'of being stationary' on the shelf. will be used in place of 'of been stationary' because 158. Identify the segment in the sentence which Gerund (V1 + ing) is used after the preposition. contains a grammatical error. Correct SentenceOne of the proposals received by us seem very The two trains gave the illusion of being stationary. interesting. 163. In the sentence identify the segment which (a) the proposals contains the grammatical error. (b) seem very interesting Due to the cyclone Idai vast areas of land have (c) received by us been flooded, roads destroyed and (d) one of communications disrupting in Zimbabwe and SSC CPO-SI – 24/11/2020 (Shift-I) Mosambique. Spotting Error

30

YCT

(a) (b) (c) (d)

vast areas of land have been flooded roads destroyed Due to the cyclone Idai and communications disrupting SSC CGL (Tier-I) – 04/06/2019 (Shift-II) Ans : (d) In the above sentence 'Disrupted' will be used in place of 'Disrupting' because when two clause connected with 'and' then both clause will be in same tense. Since 'roads destroyed' is in past form so communication disrupting also in past form. Correct SentenceDue to the cyclone Idai vast areas of land have been flooded, roads destroyed and communications disrupted in Zimbabwe and Mosambique. 164. In the sentences identity the segment which contains the grammatical error. You may left the class when you have completed the test. (a) You may left (b) completed the best (c) the class (d) when you have SSC CGL (Tier-I) – 13/06/2019 (Shift-II) Ans. (a) : In the above sentence 'leave' will be used in place of 'left' because after modal verb (may) first from of the verb is used. Correct sentenceYou may leave the class when you have completed the test. 165. In the sentence identify the segment which contains the grammatical error. 'Plogging' is an international trend, in which someone picking up trash while jogging or brisk walking. (a) while jogging or brisk walking (b) picking up trash (c) 'Plogging' is an international trend (d) in which someone SSC CPO-SI – 09/12/2019 (Shift-I) Ans : (b) In the above sentence 'Picks up trash' will be used in place of 'picking up trash' because according to the meaning of the sentence regular activity is being understood, thereafter the sentence is in simple present will be used. Correct sentence :'Plogging' is an international trend, in which someone picks up trash while jogging or brisk walking. 166. Identify the segment that contains a grammatical error. If there is no error, select 'No error' Need you to buy so many books? (a) to buy (b) Need you (c) so many books (d) No error SSC CPO-SI – 11/12/2019 (Shift-II) Ans : (a) In the given sentence 'buy' will be used in place of 'to buy' because in the given sentence semimodal 'Need' is used as a helping verb at the beginning of sentence. The sentence is yes/No answer type question Its structure be as follows. Helping verb + subject + verb + other words

167. Identify the segment in the sentence which contains the grammatical error. If there is no error, select 'No error' Anita enjoyed to swim in the sea. (a) to swim (b) Anita enjoyed (c) in the sea (d) No error SSC CPO-SI – 11/12/2019 (Shift-I) Ans : (a) In the above sentence 'Swimming' will be used in place of ' to swim' because gerund (verb1 + ing) is used after verb enjoy. Note- Admit, avoid, delay, deny, detest, enjoy, excuse, finish, forgive, mind, prevent, keep, resent, it is no use, it is no good, can't help, can't stand etc. always followed by v+ing form. Correct sentence :Anita enjoyed swimming in the sea. 168. Identify the segment in the sentence which contains the grammatical error. If there is no error, select 'No error'. He wouldn't let anyone to drive his new car. (a) No error (b) his new car (c) anyone to drive (d) He wouldn't let SSC CPO-SI – 12/12/2019 (Shift-I) Ans : (c) In the above sentence 'anyone drive' will be used in place of 'anyone to drive' because after act, bid, watch, behold, see, make, feel bare infinitive (VI) is used, in active voice but if it is in passive, then –infinitive (to + VI) is used and the sentence is in active voice. Correct sentence :He wouldn't let anyone drive his new car. 169. In the sentence identify the segment which contains the grammatical error. Are you having your own transport to go home from work? (a) from work (b) your own transport (c) Are you having (d) to go home SSC CPO-SI – 13/12/2019 (Shift-II) Ans. (c) : In the above sentence 'do you have' will be used in place of 'Are you having' because in interrogative sentence, when the subject gives sense of possession then According to grammar rule 'Do + Plural Subject + have + object + etc' is used. Correct sentence :Do you have your own transport to go home from work? 170. Identify the segment in the sentence, which contains the grammatical error. My grandmother has been lives in Shimla since her childhood days. (a) has been lives (b) her childhood days (c) My grandmother (d) in Shimla since SSC CHSL (Tier-I) –10/07/2019 (Shift-III) Ans : (a) In the above sentence 'Living' will be used in place of 'Lives' because it is clear from the use of helping verb 'has been' and since/for that the sentence is in Present Perfect continuous. The structure of this tense is as follows. Subject + has/have + been + verb + ing + since/for + Since 'to + buy' is used as a noun in the form of time infinitive whereas here verb is required. Correct sentenceCorrect sentence My grandmother has been living in Shimla since her Need you buy so many books? childhood.

Spotting Error

31

YCT

171. Identify the segment in the sentence which contains the grammatical error. When we went to Ramgarh we see the snowcapped mountains in the distance. (a) in the distance (b) to Ramgarh we see (c) the snow-capped mountains (d) when we went SSC CHSL (Tier-I) –09/07/2019 (Shift-II) Ans : (b) In the above sentence 'Saw' will be used in place of 'See' because it is clear from the use of went (V2) that the sentence is in Past Indefinite. So the second verb also be in 2nd form. Correct sentenceWhen we went to Ramgarh we saw the snow-capped mountains in the distance. 172. Identify the segment which contains the grammatical error. It was windy autumn day and leaves were fallen rapidly from the trees. (a) It was a (b) rapidly from trees (c) leaves were fallen (d) windy autumn day SSC CHSL (Tier-I) –05/07/2019 (Shift-III) Ans : (c) In the above sentence 'Leaves were falling' will be used in place of 'Leaves were fallen' since was/were' is the helping verb of Past continuous. After which 'ing' is added to the main verb. Although verb IIIrd form can be used after was/were but the sentence should in Passive form. Correct sentenceIt was a windy autumn day and leaves were falling rapidly from the trees. 173. Identify the segment which contains the grammatical error. The ambulance have arrived on time, the accident victim was taken to the hospital. (a) the accident victim (b) have arrived on time (c) The ambulance (d) was taken to the hospital SSC CHSL (Tier-I) –04/07/2019 (Shift-III) Ans : (b) In the above sentence 'had arrived on time' will be used in place of 'have arrived on time' because it is clear that the second clause is in past tense. So, the other clause will also be in past tense. and helping verb of past perfect tense 'had' will be used. Correct sentence :The ambulance had arrived on time, the accident victim was taken to the hospital. 174. In the sentence identify the segment which contains the grammatical error. The Public Works Department has propose to construct an elevated corridor which will run parallel to the National highway. (a) to the National highway (b) The Public Works Department has propose (c) which will run parallel (d) to construct an elevated corridor SSC CHSL (Tier-I) –02/07/2019 (Shift-I) Ans. (b) : It is clear from the use of Helping verb 'has' in the above sentence that the sentence is in Present Perfect tense and its structure is as follows Spotting Error

Subject + has / have + V III form + other words Thereafter in place of 'propose', 'proposed' verb3 will be used in the sentence. Correct SentenceThe Public Works Department has proposed to construct an elevated corridor which will run parallel to the National highway 175. In the sentence identify the segment which contains the grammatical error. I declined the lunch offer by Suresh, not because I do not want to go, but because I had no time. (a) by Suresh not because (b) but because I had no time (c) I do not want to go (d) I declined the lunch offer SSC CHSL (Tier-I) 26/10/2020 (Shift-II) Ans. (c) : In the above sentence ' I did not' will be used in place of 'I do not' because It is clear from V2 (Declaimed) that the sentence is in past tense. Hence the next clause is also in Past tense so 'did' will be used in place of 'do'. Correct sentenceI declined the lunch offer by Suresh, not because I did not want to go, but because I had no time. 176. Select the segment in the sentence, which contains the grammatical error. Did you know whether we can exchange the dress if my sister doesn't like it? (a) doesn't like it (b) we can exchange the dress (c) Did you know whether (d) if my sister SSC CHSL (Tier-I) 17/03/2020 (Shift-I) Ans. (c) In the above sentence 'Do you know whether will be used in place of 'Did you know whether' because according to the sense of the sentence it is in present tense. As it is clear from sub-ordinate clause (doesn't like) Correct SentenceDo you know whether we can exchange the dress if my sister doesn't like it? 177. In the given sentence, identify the segment which contains the grammatical error. A Delhi bound aircraft has to make an emergency landing at Mumbai airport as a woman passenger suddenly developed a serious breathing problem. (a) suddenly developed (b) as a woman passenger (c) has to make (d) a serious breathing problem SSC CHSL (Tier-I) 16/10/2020 (Shift-I) Ans. (c) : In the above sentence 'had to make' will be used in place of 'has to make' because it is clear from the second clause that the sentence is in past tense. Correct SentenceA Delhi bound aircraft had to make an emergency landing at Mumbai airport as a woman passenger suddenly developed a serious breathing problem.

32

YCT

(a) may have sleep problems 178. In the sentence identify the segment which (b) people with full time sedentary jobs contains a grammatical error. (c) due to lack of exercise Samurai jack drawn his sword to exact revenge (d) who commuting long distances on his superiors. SSC CHSL (Tier-I) 21/10/2020 (Shift-I) (a) to exact revenge (b) on his superiors Ans. (d) : In the above sentence 'who commute long (c) Samurai jack drawn (d) his sword to SSC CHSL (Tier-I) 26/10/2020 (Shift-I) distance' will be used in place of 'who commuting long distances' because here V1 (commute) is needed in Ans. (c) : In the above sentence V2 (drew) will be used present Indefinite tense. Since relative pronoun 'who' 3 in place of V (drawn) because above sentence is is used for subject people (plural noun) hence plural Affirmative sentence of past Indefinite tense and verb (verb1) without 's/es' is used after 'who'. second form of verb is used in it. Correct Sentence : Note- Past Indefinite Negative Interrogative Sentence, People with full time sedentary jobs who commute helping verb, 'did' and verb first form is used. long distances may have sleep problem due to lack of Correct Sentence exercise. Samurai jack drew his sword to exact revenge on his 182. In the given sentences identify the segment superiors, which contains a grammatical error. 179. In the sentence identify the segment which The Indian cricket team was done very well this year and is ranked number one now. contains the grammatical error. (a) was done very well Often hailed as one of the greatest industry (b) this year leaders of india, Ratan Tata take across the (c) and is ranked number one now reins of the company, leading from the front. (d) The Indian cricket team (a) leading from the front SSC CHSL (Tier-I) 19/10/2020 (Shift-I) (b) Often hailed as one of Ans. (a) : In the above sentence 'has' will be used in (c) the greatest industry leaders from india, (d) Ratan Tata take across the reins of the place of 'was' because the use of Adverb 'now' makes it clear Present time moment. Hence, present perfect company SSC CHSL (Tier-I) 26/10/2020 (Shift-II) tense is used. Note- was + done (V3) is used in passive voice. Ans. (d) : In the above sentence 'took over' will be Correct Sentence : used in place of 'take across' because according to the The Indian cricket team has done very well this year sense of the sentence it is in Past Indefinite tense. and is ranked number one now. Hence second form of verb (took) will be used. 183. In the given sentence, identify the segment Correct Sentence which contains a grammatical error. Often hailed as one of the greatest industry leaders of The country has to struggled with the problems india, Ratan Tata took over the reins of the company, of child labour. leading from the front. (a) child labour (b) the country has (c) to struggled with (d) the problems of 180. In the sentence identify the segment which SSC CHSL (Tier-I) 19/10/2020 (Shift-I) contains the grammatical error. Mani Kaul begun his career as an actor and a Ans. (c) : In the above sentence 'to struggle' will be director of short films before doing his debut used in place of 'to struggled' because first from of verb (VI) is used after 'to' feature film, 'Uski Roti,' (a) debut feature film, 'Uski Roti,' Infinitive − ( To + V1 ) (b) begun his career as an actor Correct Sentence : (c) a director of short films The country has to struggle with the problems of child (d) before doing his SSC CHSL (Tier-I) 26/10/2020 (Shift-II) labour. 184. In the given sentence identify the segment Ans. (b) : In the above sentence second form of verb which contains the grammatical error. 'began' will be used in place of third form of verb Freedom of expression meaning the right to 'begun' because the sentence is in past Indefinite tense. express one's own convictions and beliefs. (a) meaning the right begin → began → begun (b) to express one's own Ist IInd IIIrd (c) Freedom of expression (d) convictions and beliefs Correct sentence SSC CHSL (Tier-I) 12/10/2020 (Shift-II) Mani Kaul began his career as an actor and a director of short films before doing his debut feature film, 'Uski Ans. (a) : In the above sentence 'means the right' will be used in place of 'meaning the right' because the Roti, 181. In the sentence identify the segment which sentence is in Present Indefinite tense. Hence verb1 will be used. contains the grammatical error. People with full time sedentary job who Correct Sentence : commuting long distances, may have sleep Freedom of expression means the right to express one's own convictions and beliefs. problems due to lack of exercise.

Spotting Error

33

YCT

185. Identify the segment in the given sentence which contains the grammatical error. Have I ever told you what lovely pickles my grandmother used to made when she was in Visakhapatnam? (a) When she was in (b) have I ever told you (c) used to made (d) what lovely pickles SSC CHSL (Tier-I) 15/10/2020 (Shift-I) Ans. (c) : In the above sentence 'used to make' will be used in place of 'used to made' because the structure 'to + VI' is used as an infinitive. Correct Sentence : Have I ever told you what lovely pickles my grandmother used to make when she was in Visakhapatnam? 186. Identify the segment in the given sentence which contains the grammatical error. Ayan told Sriram, "Don't worry I can be able to climb that tree to reach the fruit." (a) Don't worry (b) I can be able to (c) climb that tree (d) to reach the fruit SSC CHSL (Tier-I) 15/10/2020 (Shift-I) Ans. (b) : In the above sentence 'I am able to' will be used in place of 'I can be able to' because the use of able with can is superfluous. Hence 'am able' gives appropriate meaning according to the sense of the sentence. Correct Sentence : Ayan told Sriram, "Don't worry I am able to climb that tree to reach the fruit." 187. In the sentence identify the segment which contains the grammatical error. Firecrackers have seized by the government went off while being defused on the banks of the river Ganga, in a town in West Bengal. (a) while being defused (b) on the banks of the river Ganga (c) Firecrackers have seized by the government (d) in a town in West Bengal SSC CHSL (Tier-I) 21/10/2020 (Shift-III) Ans. (c) : The above sentence is in passive voice So Firecrackers have been seized by the government is used in place of 'Firecrackers have seized by the government' because the passive voice structure of present perfect tense is as follows :– Object + has/have + been + VIII + by + subject + other. Correct Sentence : Firecrackers have been seized by the government went off while being defused on the banks of the river Ganga, in a town in West Bengal. 188. In the sentence identify the segment which contains the grammatical error. A 12 year old archer was airlifted to Delhi when he met with an accident while practice for the Khelo India Games in Guwahati. (a) was airlifted to Delhi (b) when he met with an accident (c) A 12 year old archer (d) while practice for the Khelo India Games SSC CHSL (Tier-I) 21/10/2020 (Shift-III) Spotting Error

Ans. (d) : In the above sentence 'While practicing for the Khelo India Games' will be used in place of 'While practice for the Khelo India Games' because 'ing' form of the verb is generally used with 'while'. Correct Sentence : A 12 year old archer was airlifted to Delhi when he met with an accident while practicing for the Khelo India Games in Guwahati. 189. In the sentence identify the segment which contains the grammatical error. When Harry come back home, he saw blood everywhere. (a) When Harry (b) blood everywhere (c) come back home (d) he saw SSC CHSL (Tier-I) 20/10/2020 (Shift-III) Ans. (c) : In the above sentence 'came back home' will be used in place of 'come back home' because the main clause of sentence, simple past, hence the subordinate clause will also be in simple past. Hence came (V2) will be used instead of come (VI) Correct Sentence : When Harry came back home, he saw blood everywhere. 190. In the following question, some part of the sentence may have errors. Find out which part of the sentence has an error and select the appropriate option. If a sentence is free from error, select 'No Error'. If revenues remain healthy, the (1)/ government would, over time, get the necessary (2)/fiscal room to rationalizes multiple GST rate. (3)/No error (4) (a) 1 (b) 2 (c) 3 (d) 4 SSC MTS 7-10-2017 (Shift-I) Ans. (c) : In the above sentence 'to rationalize' will be used in place of 'to rationalizes' because 'to + VI' is used. Hence Rationalize will be used without s/es. Correct Sentence : If revenues remain healthy, the government would, over time, get the necessary fiscal room to rationalize multiple GST rate. 191. In the following question, some part of the sentence may have errors. Find out which part of the sentence has an error and select the appropriate option. If a sentence is free from error, select 'No Error'. In July, the government promulgate(1)/a law that enables the banning (2)/of political parties.(3)/ No error (4) (a) 1 (b) 2 (c) 3 (d) 4 SSC MTS 7-10-2017 (Shift-I) Ans. (a) : In the above sentence 'promulgated' will be used in place of 'promulgate' because according to the sense of sentence Past Indefinite tense will be appropriate. Hence verb2 (V2) will be used. Correct Sentence : In July, the government promulgated a law that enables the banning of political parties.

34

YCT

192.

In the following question, some of the sentences have been errors. Find out which part of the sentence is an error and sefect the appropriate option, If a sentence is free from error, select 'No Error'. Mumbai was crawling at a (1) / snail's pace yesterday, is slowly (2) / returning to normalcy. (3) / No Error (4) (a) 1 (b) 2 (c) 3 (d) 4 SSC MTS 9-10-2017 (Shift-III) Ans : (a) In the above sentence 'which was crawling' will be used in place of 'was crawling' because Pronoun 'which' will be used for subject (Mumbai). because two verb (was, is) are given in the sentence while only one subject (mumbai) is used. So another suitable subject will be used according to the sense of sentence. Correct Sentence Mumbai, which was crawling at a snail's pace yesterday, is slowly returning to normalcy. 193. In the following question, some part of the sentence may have errors. Find out which part of the sentence has an error and select the appropriate option. If a sentence is free from error, select 'No Error'. But migrant laborers who (a)/ are being working inside the (b)/ premises for years. (c)/ No Error (d). (a) a (b) b (c) c (d) d SSC MTS 10-10-2017 (Shift-III) Ans. (b) : In the above sentence 'have been working' will be used in place of 'are being working' because 'for + period of time' is given in the following sentence. So Present Perfect continuous tense will be used. has/have + been + verb1 + ing Correct Sentence : But migrant laborers who have been working inside the premises for years. 194. In the following question, Some part of sentence have errors. Find out which part of the sentence has an error and select the appropriate option. If a sentence is free from error, select "No Error". Since they got something (1)/ different each month, (2)/they keep coming back to it. (3)/ No Error (4) (a) 1 (b) 2 (c) 3 (d) 4 SSC MTS 10-10-2017 (Shift-I) Ans : (a) In the above sentence get (V1) will be used in place of got (V2) because second clause is in present indefinite tense (keep). Hence v1 (get) will be used in the other clause. Correct Sentence : Since they get something different each month, they keep coming back to it. 195. In the following question, some part of the sentence may have errors. Find out which part of the sentence has an error and select the appropriate option. If a sentence is free from error, select 'No Error'. Spotting Error

Even if we missed a day's work, (1)/ out salaries are not cut and we (2) / were paid in full. (3)/ No Error (4) (a) 1 (b) 2 (c) 3 (d) 4 SSC MTS 11-10-2017 (Shift-III) Ans. (b) : In the above sentence 'were' will be used in place of 'are' because it is clear from the use of 'we missed and were paid' in the sentence that the sentence is in past tense. Correct Sentence : Even if we missed a day's work, out salaries were not cut and we were paid in full. 196. In the following question, some part of the sentence may have errors. Find out which part of the sentence has an error and select the appropriate option. If a sentence is free from error, select 'No Error'. Tension prevail late on Friday after (a)/ angry protesters pelted stones at the police, (b)/torched vehicles and vandalized publics property (c)/ No Error (d) (a) a (b) b (c) c (d) d SSC MTS 11-10-2017 (Shift-II) Ans. (a) : It is clear from the second clause of the above sentence that the sentence is in Past Indefinite Tense. So in the first part of sentence prevailed (V2) will be used in place of prevail. Correct Sentence : Tension prevailed late on Friday after angry protesters pelted stones at the police, torched vehicles and vandalized publics property. 197. In the following question, some part of the sentence may have errors. Find out which part of the sentence has an error and select the appropriate option. Over the years our trainers (1)/did helped students to clear the (2)/language tests for various countries. (3)/ No Error (4). (a) 1 (b) 2 (c) 3 (d) 4 SSC MTS 11-10-2017 (Shift-I) Ans : (b) In the above sentence 'helped' will be used in place of 'did helped' because in affirmative sentences of past Indefinite tense, only second form of verb is used without 'did'. Note–S + VIInd form + Obj. (Assertive) Sub + did not + V1 + Obj. (Negative) Did + Sub + V1 + Obj. (Interrogative) Correct Sentence : Over the years our trainers helped students to clear the language tests for various countries. 198. Find the part of the given sentence that has an error in it. If there is no error, choose 'No error'. He went up to her and asks her why she had insulted him./No error (a) and asks her why (b) she had insulted him (c) No error (d) He went up to her SSC MTS – 06/08/2019 (Shift-III)

35

YCT

Ans. (a) : In the above sentence 'asked' will be used in place of 'ask' because first clause (went) in past tense. Hence second clause will be in past Indefinite tense. Correct SentenceHe went up to her and asked her why she had insulted him. 199. Find the part of the given sentence that has an error in it. If there is no error, choose 'No error'. He has deposits all his money in banks./No error (a) He has deposits (b) money in banks (c) up all his (d) No error SSC MTS – 06/08/2019 (Shift-III) Ans. (a) : In the above sentence Deposited (V3) will be used in place of 'Deposits' because it is clear from the use of 'has' that the sentence is in present perfect tense. has/ have + V3 hence will be used. Correct sentence : He has deposited all his money in banks. 200. Identify the segment which contains the grammatical error. Kaira has upset with me since the accident. (a) the accident (b) me since (c) upset with (d) Kaira has SSC MTS – 22/08/2019 (Shift-III) Ans. (d) : In the above sentence 'has been' will be used in place of 'has' because it is clear from the use of 'Since + point of time' that the sentence is in Present Perfect continuous hence 'has been' + v3' is used. Correct Sentence :Kaira has been upset with me since the accident. 201. Identify the segment in the sentence which contains the grammatical error. Radha woke up early that morning so that she should watch the beautiful sunrise. (a) so that she should watch (b) Radha woke up (c) early that morning (d) the beautiful sunrise SSC MTS – 22/08/2019 (Shift-II) Ans. (a) : In the above sentence 'could' will be used in place of 'should' because could will be used for possibility. Correct sentence : Radha woke up that morning so that she could watch the beautiful sunrise. 202. Identify the segment in the sentence which contains the grammatical error. Harish is spending two hours every morning reading newspapers in different languages. (a) two hours every morning (b) in different languages (c) reading newspapers (d) Harish is spending SSC MTS – 22/08/2019 (Shift-II) Ans. (d) : In the above sentence 'spends' will be used in place of 'is spending' because Present Indefinite tense is used for habitual action. Correct Sentence : Harish spends two hours every morning reading newspapers in different languages. Spotting Error

203. Identify the segment in the sentence which contains the grammatical error from the given options. He started life as a teacher before turn to journalism. (a) He started life (b) as a teacher (c) to journalism (d) before turn SSC MTS – 22/08/2019 (Shift-II) SSC MTS – 08/08/2019 (Shift-III) Ans. (d) : In the above sentence 'before turning' will be used in place of 'before turn' Rule :– Preposition at, in, on, to , before, after, for, from, about etc are followed by verb's ing form. Correct sentence :He started life as a teacher before turning to journalism. 204. Find the part of the given sentence that has an error in it. If there is no error, choose 'No error'. The more he earned the more he want to earn./No error. (a) the more he (b) No error (c) want to earn (d) The more he earned SSC MTS – 08/08/2019 (Shift-I) Ans. (c) : In the above sentence 'wanted' will be used in place of 'want' because if the main clause of the sentence is in the past tense then other clause also will be used in the past. Correct sentence :The more he earned the more he wanted to earn. 205. Find the part of the given sentence that has an error in it. If there is no error, choose 'No error'. Either of the two books will met my requirement./No error (a) Either of the (b) No error (c) will met my requirement (d) two books SSC MTS – 06/08/2019 (Shift-I) Ans. (c) : In the above sentence 'will meet' will be used in place of 'will met' because Ist form of verb is used with modal verb 'will'. Correct sentence :Either of the two books will meet my requirement. 206. Find the part of the given sentence that has an error in it. If there is no error, choose No error'. She wanted to telling you who you could approach for your problem./No error (a) No error (b) who you could approach (c) for your problem (d) She wanted telling you SSC MTS – 02/08/2019 (Shift-III) Ans. (d) : In the above sentence 'she wanted to tell you' will be used in place of 'she wanted to telling you' because verb1 with infinitive 'to' Note– Want, promise, learn, manage etc. (to +V1) Correct sentence :She wanted to tell you who you could approach for your problem.

36

YCT

207. Find the part of the given sentence that has an error in it. If there is no error, choose 'No error'. All the students/is done/better this year. (a) All the students (b) is done (c) No error (d) better this year SSC MTS – 02/08/2019 (Shift-II) Ans. (b) : In the above sentence 'have done' will be used in place of 'is done' because according to the sense of the sentence it is in present perfect tense. Hence has/have + V3 will be used. Correct sentence :All the students have done better this year. 208. Find the part of the given sentence that has an error in it. If there is no error, choose 'No error'. Rakesh is foolish/to said/such things. (a) to said (b) No error (c) such things (d) Rakesh is foolish SSC MTS – 02/08/2019 (Shift-II) Ans. (a) : In the above sentence 'to say' will be used in place of 'to said' because the verb coming after 'to' is in VIst form. Note - to + verb1 Correct sentence :Rakesh is foolish to say such things. 209. Find the part of the given sentence that has an error in it. If there is no error, choose 'No error'. I told the tailor/to made a new/dress for me./No error. (a) dress for me (b) I told the tailor (c) No error (d) to made a new SSC MTS – 02/08/2019 (Shift-I) Ans. (d) : In the above sentence 'To make a new' will be used in place of 'to made a new' because to + VI is use as infinitive. Correct sentence :I told the tailor to make a new dress for me. 210. Identify the segment in the sentence which contains the grammatical error. Jaya asked Sushmita to reminds her about the meeting. (a) Sushmita to reminds (b) Jaya asked (c) the meeting (d) her about SSC MTS – 20/08/2019 (Shift-II) Ans. (a) : In the above sentence 'to remind' will be used in place of 'to reminds' because here infinitive has been used in which is appropriate Correct sentence :Jaya asked Sushmita to remind her about the meeting. 211. Identify the segment in the sentence which contains the grammatical error. Zainab was played with/her friends in the park/when she noticed Disha/walking away with a stranger. (a) Zainab was played with (b) walking away with a stranger (c) when she noticed Disha (d) her friends in the park SSC MTS – 20/08/2019 (Shift-I) Spotting Error

Ans. (a) : In the above sentence 'Playing' will be used in place of 'Played' because the given sentence is in active voice of Past continuous. Hence (was/were + V1 + ing) will be used. Correct sentence : ''Zainab was playing with her friends in the park when She noticed Disha walking away with a stranger.'' 212. Identify the segment in the sentence which contains the grammatical error. The priest request all the wealthy men to donate money for building a new temple. (a) for building a new temple (b) The priest request (c) to donate money (d) all the wealthy men SSC MTS – 16/08/2019 (Shift-II) Ans. (b) : In the above Sentence 'requested' will be used in place of 'request' because according to sense of the given sentence it will be used in past Indefinite tense. Correct sentence : The priest requested all the wealthy men to donate money for building a new temple. 213. Identify the segment in the sentence which contains the grammatical error. She misplaced her spectacles and now finding it difficult to read. (a) her spectacles (b) difficult to read (c) and now finding it (d) She misplaced SSC MTS – 14/08/2019 (Shift-I) Ans. (c) : In the above sentence 'found' will be used in place of finding' because the first clause of the sentence she (misplaced) is in past tense. Hence the later clause will also be used in past. Correct sentence : She misplaced her spectacles and found it difficult to read. 214. Indentify the segment in the sentence which contains the grammatical error. If there is no error, select 'no error' I don't think I meet him before (a) No error (b) him before (c) I meet (d) I don't think SSC MTS – 14/08/2019 (Shift-I) Ans. (c) : In the above sentence 'I met' will be used in place of 'I meet' because according to sense of the sentence. The verb of Past Indefinite tense will be used according to the sense of the sentence. Correct sentence : 'I don't think I met him before'. 215. Identify the segment in the sentence which contains the grammatical error from the given options. The company has invested a great deal of time and effort to setting up new training schemes. (a) The company has (b) time and effort to setting up (c) invested a great deal of (d) new training schemes SSC MTS – 09/08/2019 (Shift-III)

37

YCT

Ans. (b) : In the above sentence 'to set up' will be used in place of 'to setting up' because the verb after the infinitive 'to' is used in the first form. Correct sentenceThe company has invested a great deal of time and effort to set up new training schemes. 216. Identify the segment in the sentence which contains the grammatical error from the given options . For most citizens today. liberty meant the freedon to practise their religious or political beliefs. (a) For most citizens today (b) practice their religious (c) liberty meant the freedom to (d) or political beliefs. SSC MTS – 09/08/2019 (Shift-II) Ans. (c) : In the above sentence 'means' will be used in place of 'meant' because the word 'today' makes it clear that the sentence is in Present Indefinite Tense then verb1 + s/es (singular verb) is used. Correct sentenceFor most citizens today, liberty means the freedom to practice their religious or political beliefs. 217. Identify the segment in the sentence which contains the grammatical error from the given options . When they started, they had great ideas but lack the business know-how. (a) When they started (b) they had great ideas (c) but lack (d) the business know-how SSC MTS – 09/08/2019 (Shift-II) Ans. (c) : It is clear from the verb 'started' in the above sentence that the sentence is in past tense because verb2 is used in the sentence which is clear that the next part will also in the past tense. Hence 'lacked' will be used in place of 'lack'. Correct sentenceWhen they started, they had great ideas but lacked the business know-how. 218. Identify the segment in the sentence which contains the grammatical error from the given options. The government's proposal has been set alarm bells ringing for people with low incomes. (a) with low incomes (b) ringing for people (c) The government's proposal (d) has been set alarm bells SSC MTS – 08/08/2019 (Shift-II) Ans. (d) : In the above sentence 'has' will be used in place of 'has been' because the sentence is in Active voice. in which has/have + verb3 is used. Correct sentenceThe government's proposal has set alarm bells ringing for people with low incomes. 219. Find the part of the given sentence that has an error in it. If there is no error, choose 'No error'. Some people is always talking about themselves./No error (a) No error (b) about themselves (c) some people (d) is always talking SSC MTS – 07/08/2019 (Shift-II) Spotting Error

Ans. (d) : In the above sentence 'always talk' will be used in place of 'is always talking' because always shows a habitual action. Hence simple present tense will be used. and with plural noun (people) is used with plural verb (verb1) without s/es. Correct Sentence Some people always talk about themselves. 220. Identify the segment in the sentence which contains the grammatical error. If there is no error then select 'No error'. I have got my Master's degree in 2005. (a) I have got (b) my Master's degree (c) in 2005 (d) No error SSC MTS – 21/08/2019 (Shift-III) Ans. (a) : In the given sentence 'I got' (simple past tense) will be used in place of I have got Present Perfect tense because the time of past (2005) is given in the sentence. Correct Sentence I got my Master's degree in 2005. 221. Identify the segment in the sentence which contains the grammatical error from the given options. The use of new technology has bring down the price of books. (a) of books (b) bring down (c) The use of (d) technology SSC GD – 08/03/2019 (Shift-III) Ans. (b) : In the above sentence 'brought down' will be used in place of 'bring' down' because it is clear from the use of 'has' that the sentence is in Present Perfect tense. In which has/have + V3 is used. Correct sentenceThe use of new technology has brought down the price of books. 222. Identify the segment in the sentence which contains the grammatical error from the given option. For a long time the publishing market excluding the poor. (a) For a long (b) excluding (c) publishing (d) the poor SSC GD – 08/03/2019 (Shift-III) Ans. (b) : In the above sentence 'has been excluding' will be used in place of 'excluding' because the structure of perfect continuous tense because for + time (period of time) is given. Correct sentenceFor a long time the publishing market has been excluding the poor. 223. Identify the segment in the sentence which contains the grammatical error from the given options. The Supreme Court has the power to modification or cancel laws in the country. (a) the power (b) cancel laws (c) to modification (d) The Supreme Court SSC GD – 08/03/2019 (Shift-III)

38

YCT

Ans. (c) : In the above sentence 'to modify' will be used in place of 'to modification' because to + VI is used as an infinitive. Which works as a noun. Correct sentenceThe Supreme Court has the power to modify or cancel laws in the country. 224. From the given options, identify the segment in the sentence which contains the grammatical error. Hackers can either tampers with a random number generator of figure out the resultant number easily in case of online gaming. (a) figure out (b) with a random number (c) in case of (d) Hackers can either tampers SSC GD – 02/03/2019 (Shift-II) Ans. (d) : In the above sentence 'Hackers can either tamper' will be used in place of 'Hackers can either tampers' because the verb (VI) is used after the modal 'can'. Therefore temper without s/es will be used instead of tempers. Correct sentenceHackers can either tamper with a random number generator of figure out the resultant number easily in case of online gaming. 225. From the given options, identify the segment in the sentence which contains the grammatical error. Computers everywhere use random numbers as keys to locked or unlock encrypted information. (a) use random numbers as (b) keys to locked or unlock (c) encrypted information (d) computers everywhere SSC GD – 02/03/2019 (Shift-I) Ans. (b) : In the above sentence 'Lock' will be used in place of 'Locked' because infinitive 'to' is generally followed by verb Ist form. Correct Sentence : Computers everywhere use random numbers as keys to lock or unlock encrypted information. 226. From the given options identify the segment in the sentence which contains the grammatical error. With the help of skill-based education offer in colleges, many students will become eligible for various jobs. (a) With the help of (b) many students will become (c) skill based education offer in (d) eligible for various jobs SSC GD – 02/03/2019 (Shift-I) Ans. (b) : In the above sentence 'will be' is used in place of 'will become' because the sentence is in simple future. In which subject + will/shall + be + other word is used. Correct Sentence : With the help of skill-based education offer in colleges, many students will be eligible for various jobs. Spotting Error

227. From the given options, identify the segment in the sentence which contains the grammatical error. Dr. Rana is our teacher of English since this semester. (a) our teacher (b) this semester (c) Dr. Rana is (d) of English since SSC GD – 18/02/2019 (Shift-III) Ans. (c) : In the above sentence 'has been' will be used in place of 'is' because it is clear from the use of 'Since' that the sentence is in present perfect continuous tense. Sub + has/have + been + verb1 + ing + object +

since/for + time Correct sentence Dr. Rana has been our teacher of English since this semester. 228. From the given options identify the segment in the sentence which contains the grammatical error. Ram always tries to do things very carefully and made sure he does them correctly. (a) to do things (b) Ram always tries (c) does them (d) and made sure SSC GD – 14/02/2019 (Shift-II) Ans. (d) : In the above sentence 'and make sure' will be used in place of 'and made sure' because the sentence is in present Indefinite Tense. and 'Ram' is a singular subject so singular verb verb1 + s/es will be used for it. Correct sentence Ram always tries to do things very carefully and makes sure he does them correctly. 229. From the given options, identify the segment in the sentence which contains the grammatical error. The man greeted his secretary and told her that he need her help to complete the work immediately. (a) to complete the work (b) told her that (c) he need her help (d) greeted his secretary SSC GD–14/02/2019 (Shift-I) Ans. (c) : In the above sentence the main clause is in past tense. Therefore the subordinate clause will be also in past tense Hence 'needed' will be used in place of 'need'. Correct sentence The man greeted his secretary and told her that he needed her help to complete the work immediately.

D.

Preposition

230. Find the part of the given sentence that has an error in it. If there is no error, choose 'No error'. Neither I nor my friends are going too walk down that street. (a) down that street (b) Neither I nor my (c) friends are going too walk (d) No error SSC MTS 10/05/2023 (Shift-I)

39

YCT

Ans. (c) : There is an error of option (c) because adverb 'too' will be replaced by preposition 'to'. to + verb1 → work as a noon The correct sentence is'Neither I nor my friends are going to walk down that street. 231. Identify the segment that contains a grammatical error. If there is no error, select 'No error'. Never put of/until tomorrow what /you can do today. (a) Never put of (b) No error (c) you can do today (d) until tomorrow what SSC MTS 08/05/2023 (Shift-III) Ans. (a) : For the above given segment option (a) will be used because 'put of' is replace by ' put off' which means 'To postpone doing something' The correct sentence is– 'Never put off until tomorrow what you can do today' 232. Find the part of the given sentence that has an error in it. If there is no error, choose 'No error'. Can you please help me to this project? (a) Can you please (b) help me to (c) No error (d) this project? SSC MTS 12/05/2023 (Shift-III) Ans. (b) : In the above given sentence in part (b) 'help me to' has an error, because the preposition 'with' is used in place of 'to'. Help → with → something The correct sentence – Can you please help me with this project ?' 233. Parts of the following sentence have been given as options. One of them may contain an error. Select the option that contains the error. If you don’t find any error, mark ‘No error’ as your answer. With reference of your letter, the organisation wishes to hire you for the security services. (a) No error (b) With reference of your letter (c) for the security services (d) the organisation wishes to hire you SSC CHSL (Tier-I) 10/08/2023 (Shift-I) Ans. (b) : The part of the sentence mentioned in option (b) with reference of your letter. contains error. It need to be replaced with 'with reference to it is an idiom which means about or concerning something/someone or in relation to. Correct Sentence with reference to your letter, the organization wishes to hire for the security services. 234. Parts of the following sentence have been given as options. Select the option that contains an error. This region was struck by unusual wave of violence last year. Spotting Error

was struck by unusual This region last year. wave of violence SSC CHSL (Tier-I) 10/08/2023 (Shift-I) Ans. (a) : Part (a) "was struck by unusual" being erroneous, will be replaced by 'was struck with unusual'. Because 'struck with' means to have something you do not want because you can not get rid of it. Hence it is appropriate to be used in place of erroneous part. Correct sentence The region was struck with unusual wave of violence last year. 235. Parts of the following sentence have been given as options. Select the option that contains an error. The government has been accused in not doing enough to combat climate change. (a) accused in (b) The government (c) enough to combat (d) climate change. SSC CHSL (Tier-I) 09/08/2023 (Shift-III) Ans. (a) : Part (a) 'accused in' of the sentence is erroneous. it has to be substituted with 'Accused of' to make the sentence grammatically correct. Because Accused is always followed by preposition (of). Correct Sentence The government has been accused of not doing enough to combat climate change. 236. The following sentence has been divided into parts. One of them may contain an error. Select the part that contains the error from the given options. The current state of the literature / on the efficacy under cognitive-behavioural therapy / for individuals with generalised anxiety disorder / suggests that it is a promising treatment option. (a) on the efficacy under cognitive-behavioural therapy (b) for individuals with generalised anxiety disorder (c) The current state of the literature (d) suggests that it is a promising treatment option. SSC CHSL (Tier-I) 04/08/2023 (Shift-III) Ans. (a) : The error part of the given sentence is option (a) 'on the efficacy under cognitive–behavioural therapy.' Because 'of ' preposition is used in the place of 'under' according to the context of this sentence as it shows possession. The correct segment is– 'on the efficacy of cognitive – behavioural therapy'. 237. Find the part of the given sentence that has an error in it. If there is no error, choose 'No error'. You will never regret this wonderful experience with your life. (a) regret this wonderful (b) experience with your life. (c) You will never (d) No error SSC CHSL (Tier-I) 17/03/2023 (Shift-IV)

40

(a) (b) (c) (d)

YCT

Ans. (b) : In the above sentence 'in' will be used in place of 'with' because Preposition 'in' is used with Experience. Correct SentenceYou will never regret this wonderful experience in your life. 238. Find the part of the given sentence that has an error in it. If there is no error, choose 'No error'. No doubt, human minds are good on predicting the worst. (a) No error (b) No doubt, human minds (c) predicting the worst (d) are good on SSC CHSL (Tier-I) 09/03/2023 (Shift-IV) Ans. (d) : In the above sentence "good at" will be used in place of "good on" because 'at' Preposition is used when 'good' use as an Adjective. Correct SentenceNo doubt, human minds are good at predicting the worst. 239. The following sentence has been split into four segments. Identify the segment that contains a grammatical error. She persisted / to doing / what she wanted / despite opposition. (a) what she wanted (b) despite opposition (c) to doing (d) She persisted SSC CGL (Tier-I) 12/04/2022 Shift-II Ans. (c) : In the above sentence 'in doing' will be used in place of 'to doing' because 'in' Preposition is used with persist. Persist + in + (Verb1 + ing) Correct SentenceShe persisted in doing what she wanted despite opposition. 240. In the sentence identify the segment which contains the grammatical error. The invited guests preferred orange juice than hot coffee or chilled grapes. (a) preferred orange juice (b) or chilled grapes (c) than hot coffee (d) the invited guests SSC CHSL (Tier-I) 19/03/2020 (Shift-I) Ans. (c) : In the above sentence, ‘to not coffee’ would be appropriate instead of ‘than not coffee’, because preposition ‘to’ is used with prefer. Correct Sentence :The invited guests preferred orange juice to hot coffee or chilled grapes. 241. Identify the segment in the sentence which contains the grammatical error. She was unable to produce sufficient evidences for support her accusations. (a) to produce (b) She was unable (c) sufficient evidences for (d) support her accusations SSC CHSL (Tier-I) –11/07/2019 (Shift-I) Spotting Error

Ans : (c) In the given sentence 'sufficient evidence to' will be used in place of 'sufficient evidences for' because here (to + VI) infinitive 'to support' will be used Correct SentenceShe was unable to produce sufficient evidence to support her accusations. 242. Identify the segment in the sentence, which contains the grammatical error. Rahul was ready for accept any job, even a part-time one. (a) Rahul was ready (b) even a part-time one (c) any job (d) for accept SSC CPO-SI (Tier-II) 27/09/2019 (3 pm - 5 pm) Ans. (d) : In the above sentence 'to' will be used in place of 'for' because in the sentence requires an infinitive (to + VI) as a Noun. Hence the use of 'to accept' would be appropriate. Correct sentence- Rahul was ready to accept any job, even a part-time one. 243. In the sentence identify the segment which contains the grammatical error. The street artist Satish Munjal has been painting this wall since the past one week. (a) has been painting (b) the street artist (c) this wall (d) since the past one week SSC CGL (Tier-I) – 06/06/2019 (Shift-II) Ans : (d) In the above sentence, ‘for’ will be used in place of ‘since’. Because 'for' is used for period of time in present continuous tense while ‘since’ is used for point of time. Rule:- Sub + has/have + been + V1+ing + for + Period of time. Correct sentence :The street artist Satish Munjal has been painting this wall for the past one week. 244. In the sentence identify the segment that contains a grammatical error. Ritu was really annoyed as she had been waiting for me at the metro station for five O'clock. (a) at the metro station (b) was really annoyed (c) for five o'clock (d) has been waiting SSC Stenographer (Grade C & D) 12.11.2021 Shift-I Ans. (c) : In the above sentence 'Since five O'clock' will be used in place of 'for five O'clock' use of for - The word 'for' is used to show a period of time. Two hours, two years etc. use of since - The word 'since' is used to refer to a point of time 15 August, March, 1988 etc. Correct Sentence Ritu was really annoyed as she had been waiting for me at the metro station since five O'clock. 245. The following sentence has been split into four segments. Identify the segment that contains a grammatical error. I have the courage/to say what's right;/I'm not afraid/from the consequences.

41

YCT

(a) I'm not afraid (b) I have the courage (c) to say what's right (d) from the consequences SSC Stenographer (Grade C & D) 12.11.2021 Shift-I Ans. (d) : In the above sentence 'of the consequences' will be used in place of 'from the consequences' because afraid generally followed by 'of' preposition. Correct SentenceI have the courage to say what's right, I'm not afraid of the consequences. 246. Identify the segment in the sentence which contains a grammatical error. If there is no error, select 'No error'. The gentleman had a suitcase full with wigs, ornaments and dresses. (a) a suitcase full with (b) The gentleman had (c) wigs, ornaments and dresses (d) No error SSC CGL (Tier-II) 08.08.2022 Shift-II Ans. (a) : In the above sentence 'full of' will be used in place of 'full with' because preposition 'of' is used with 'full'. Correct sentence– The gentleman had a suitcase full of wigs, ornaments and dresses. 247. Identify the segment in the sentence which contains a grammatical error. The officer made me to type the letter again. (a) The officer (b) made me (c) to type (d) the letter again SSC Stenographer (Grade C & D) 11.11.2021 Shift-II Ans. (c) : If the sentence is in Active voice of caustive verb then after make, bare infinitive (infinitive without to) is used but if the sentence is in passive voice then after 'make' infinitive with 'to' is used. Hence Bare infinitive (without 'to') will be used in the above sentence. 248. The following sentence has been split into four segments. Identify the segment that contains a grammatical error. After a long trek/through the forest,/we arrived to the camp/around 8 p.m. (a) through the forest (b) After a long trek (c) we arrived to the camp (d) around 8 p.m. SSC Stenographer (Grade C & D) 11.11.2021 Shift-II Ans. (c) : In the above sentence 'at is used in place of to' because 'at' preposition is used for a definite place. Hence option (c) would be correct. 249. In the given sentence, identify the segment which contains a grammatical error. The migrating labourers were served hot meals for every town or village on their way. (a) on their way (b) were served (c) migrating labourers (d) for every town SSC Stenographer (Grade C & D) 11.11.2021 Shift-I

Spotting Error

Ans. (d) : In the above sentence 'in every town' will be used in place of 'for every town' because Preposition 'in' is used with big place. Correct sentence The migrating labourers were served hot meals in every town or village on their way. 250. The following sentence has been split into four segments. Identify the segment that contains a grammatical error. In about two months,/our family will be/leaving to Australia/to settle there. (a) leaving to Australia (b) In about two months (c) to settle there (d) our family will be SSC Stenographer (Grade C & D) 11.11.2021 Shift-I Ans. (a) : In the above sentence 'Leaving for Australia' will be used in place of 'Leaving to Australia' because here 'for' preposition is used in place of 'to' preposition. (When we go away from one place and to another place, we use 'Leave for) Correct sentenceIn about two month, our family will be leaving for Australia to settle there. 251. Identify the segment in the sentence which contains a grammatical error. As I was sitting in the back of the theatre, I couldn't see the performance very well. (a) As I was sitting (b) the performance very well (c) I couldn't see (d) in the back of the theatre SSC Stenographer (Grade C & D) 12.11.2021 Shift-II Ans. (d) : There is a mistake in the above sentence in option (d). 'at' will be used in place of 'in' because 'at' preposition used for small definite place. Correct Sentence– 'As I was sitting at the back of the theatre, I couldn't see the performance very well. 252. The following sentence has been split into four segments. Identify the segment that contains a grammatical error. The Konark temple/is dedicated/from the/Sun god. (a) the Konark temple (b) from the (c) is dedicated (d) Sun god SSC Stenographer (Grade C & D) 15.11.2021 Shift-II Ans. (b) : In the above sentence 'to' will be used in place of 'from' because dedicated is followed by 'to' preposition. Correct SentenceThe Konark temple is dedicated to the sun god. 253. The following sentence has been split into four segments. Identify the segment that contains a grammatical error. Whenever he is/on a holiday,/he travels by foot/to the temple nearby. (a) he travels by foot (b) On a holiday (c) to the temple nearby (d) whenever he is SSC CHSL 30.05.2022 Shift-I

42

YCT

Ans. (a) : In the above sentence 'on foot' will be used in place of 'by foot' because 'on foot' is used in the sense of walking. Correct SentenceWhenever he is on a holiday, he travels on foot to the temple nearby. 254. The following sentence has been split into four segments. Identify the segment that contains a grammatical error. The river / flowed besides / the hillock / in our village. (a) The river (b) In our village (c) flowed besides (d) the hillock SSC CHSL 25.05.2022 Shift-I Ans. (c) : In the above sentence 'beside' will be used in place of 'besides' is appropriate according to the sense of 'to the side of'. Besides → In addition to Correct sentence– The river flowed beside the hillock in our villiage. 255. The following sentence has been split into segments. One of them may contain an error. Identify the segment that contains a grammatical error. If you don't find any error, mark 'No error' as your answer. He was formerly/a doctor of/the corporate hospital. (a) a doctor of (b) No error (c) He was formerly (d) the corporate hospital SSC CGL (Tier-I) 19/04/2022 Shift-III Ans. (a) : There is an error of preposition 'of in' the given sentence 'At' is used to show the definite place/ Location, hence 'of' will replace by 'At'. Correct Sentence He was formerly a doctor at the corporate hospital. 256. The following sentence has been divided into parts. One of them contains an error, select the part that contains the error from the given options. He conceded/ a very crucial / point at / his opponent. (a) a very crucial (b) point at (c) his opponent (d) He conceded SSC CGL (Tier-I) 21/04/2022 Shift-III Ans. (b) : In the above sentence 'Point to' will be used in place of 'Point at' because preposition 'to' is used in the case of acceptance. Correct sentence – He conceded a very crucial point to his opponent 257. The following sentence has been split into four segments. Identify the segment that contains a grammatical error. As it has been raining heavily / since two hours, / the children are / at home. (a) the children are (b) since two hours (c) at home (d) As it has been raining heavily SSC CGL (Tier-I) 12/04/2022 Shift-III Spotting Error

Ans. (b) : In the above sentence 'for' will be used in place of 'since' because 'for' is used before period of time and 'since' is used before point of time. Correct SentenceAs it has been raining heavily for two hours the children are at home. 258. The following sentence has been split into four segments. Identity the segment that contains a grammatical error. We have/not met/some of our friends/since six months. (a) not met (b) We have (c) since six months (d) some of our friends SSC CGL (Tier-I) 12/04/2022 Shift-III Ans. (c) : In the above sentence 'for' will be used in place of 'Since' because since - point of time, (9 'o' clock, 1 January) for - period of time (six years, five minutes) Correct SentenceWe have not met some of our friends for six months. 259. The following sentence has been split into four segments. Identify the segment that contains a grammatical error. Your name/precedes before mine/in the/admission list. (a) admission list (b) Your name (c) in the (d) precedes before mine SSC CGL (Tier-I) 11/04/2022 Shift-III Ans. (d) : In the above sentence 'Precedes mine' will be used in place of 'precedes before mine' because preposition 'before' is superfluous with precede. correct sentence – Your name precedes mine in the admission list. 260. Parts of the following sentence have been given as options. One of them may contain an error. Select the part that contains the error from the given options. If you don't find any error, mark 'No error' as your answer. Aid workers have delivered vast quantities of food on the refugee camps. (a) No error (b) vast quantities of food (c) on the refugee camps (d) Aid workers have delivered SSC MTS-05/07/2022 Shift-III Ans. (c) : In the above sentence 'to the refugee Camps' will be used in place of 'on the 'refugee Camps' because 'to' preposition is used with delivered. Note- Deliver something → to → something/somebody Correct Sentence– Aid workers have delivered vast quantities of food to the refugee camps. 261. The following sentence has been split into four segments. Identify the segment that contains a grammatical error. Concern above rising pollution / led to the / formulation of / various action plans.

43

YCT

(a) (b) (c) (d)

Concern above rising pollution formulation of led to the various action plans SSC MTS-06/07/2022 Shift-II Ans. (a) : In the above sentence 'concern about rising pollution' will be used in place of 'Concern above rising pollution' because 'about' preposition is used with concern. Note – Concern about something Concern over something /some body Correct Sentence– Concern about rising pollution led to the formulation of various action plans. 262. Parts of the following sentence have been given as options. One of them may contain an error. Select the part that contains the error from the given options. If you don't find any error, mark 'No error' as your answer. The reduced air pressure on airline flights can lessen the amount of oxygen in passengers' blood for twenty-five percent. (a) The reduced air pressure on airline flights (b) No error (c) in passengers' blood for twenty-five percent (d) can lessen the amount of oxygen SSC MTS-05/07/2022 Shift-I Ans. (c) : In the above sentence 'to' will be used in place of 'for' because the preposition 'to' is generally used for 'Lessen'. Correct sentence– The reduced air pressure on airline fights can lessen the amount of oxygen in passengers blood to twenty-five percent. 263. The following sentence has been divided into parts. One of them may contain an error. Select the part that contains the error from the given options. If you don't find any error, mark 'No error' as your answer. I was at a party / with some off my / friends in Mumbai. (a) friends in Mumbai (b) with some off my (c) No error (d) I was at a party SSC MTS-12/07/2022 Shift-I Ans. (b) : In the above sentence 'with some of my' will be used in place of 'with some off my' because after some preposition 'of' is used. Note – Some → of → somebody / something. Correct Sentence – I was at a party with some of my friends in Mumbai. 264. The following sentence has been split into four segments. Identify the segment that contains a grammatical error. Mohit is expecting / a huge profit / at his / recent investment. (a) a huge profit (b) at his (c) Mohit is expecting (d) recent investment SSC Constable GD-30/11/2021 Shift-III

Spotting Error

Ans. (b) : In the above sentence 'from his' will be used in place of 'at his' because 'from' is generally used as preposition in the sense of something with 'profit'. Correct Sentence Mohit is expecting a huge profit from his recent investment. 265. The following sentence has been split into four segments. Identify the segment that contains a grammatical error. He moved on the undergrowth / quickly and silently, / hiding himself / from the enemy. (a) quickly and silently (b) hiding himself (c) He moved on the undergrowth (d) from the enemy SSC Constable GD-22/11/2021 Shift-II Ans. (c) : In the above sentence 'behind the undergrowth' will be used in place of 'on the undergrowth' because it gives appropriate meaning of the sentence. Correct Sentence– He moved behind the undergrowth quickly and silenty, hiding himself from the enemy. 266. The following sentence has been split into four segments. Identify the segment that contains a grammatical error. She arrived / at / Monday morning / to attend the marriage. (a) to attend the marriage(b) She arrived (c) at (d) Monday morning SSC Constable GD-13/12/2021 Shift-III Ans. (c) : In the above sentence 'on' will be used in place of 'at' because the preposition 'on' is used before any day or date. On monday, On 6th June Correct sentence– She arrived on monday morning to attend the marriage. 267. The following sentence has been split into four segments. Identify the segment that contains a grammatical error. I saw some / children playing happily / below the shade / of a tree. (a) children playing happily (b) of a tree (c) I saw some (d) below the shade SSC Constable GD-16/11/2021 Shift-III Ans. (d) : In the above sentence 'under the shade' will be used in place of 'below the shade' because' under the shade' is used in the sense of sit under the tree. Correct Sentence I saw some children playing happily under the shade of a tree. 268. The following sentence has been into four segments. Identify the segment that contains a grammatical error. Water / freeze at / 0 degree / Celsius. (a) freeze at (b) water (c) celsius (d) 0 degree SSC Constable GD-07/12/2021 Shift-II

44

YCT

Ans. (a) : In the above sentence 'Freeze' will be used in place of 'Freeze at' because in the sense of freezing 'at' preposition is not used. Correct sentence– Water freeze 0º degree celsius. 269. The following sentence has been split into four segments. Identity the segment that contains a grammatical error. Why do they / keep talking on / about money / all the time? (a) keep talking on (b) Why do they (c) all the time (d) about money SSC Constable GD-03/12/2021 Shift-II Ans. (a) : In the above sentence 'Keep talking' will be used in the place of 'keep talking on' because used of 'on' preposition is superfluous. Correct sentence Why do they keep talking about money all the time. 270. The following sentence has been divided into parts. One of them contains an error. Select the part that contains the error from the given option. We forgot / to wish Susan / in her birthday / this year. (a) in her birthday (b) this year (c) We forgot (d) to wish Susan SSC Constable GD-01/12/2021 Shift-I Ans. (a) : In the above sentence 'her birthday' will be used in place of 'in her birthday' because wish is followed by direct object with preposition. Note – wish for sth wish sb / sth on sb Correct sentence– ∗ We forgot to wish Susan her birthday this year. 271. The following sentence has been divided into parts. One of them may contain an error. Select the part that contains the error from the given options. If you don't find any error, mark 'No error' as your answer. A probe has been ordered/ by the incident/ that occurred at the celebrations. (a) A probe has been ordered (b) that occurred at the celebrations (c) by the incident (d) No error SSC CHSL (Tier-I) –05/08/2021 (Shift-I) Ans. (c) : In the above sentence 'for' preposition will be used in place of 'by' because 'for' is used to denote something contains with order. * Order for something. * Order to somebody. Correct Sentence A probe has been ordered for the incident that occurred at the celebrations 272. The following sentence has been split into four segments. Identify the segment that contains a grammatical error. The sports day events/ will be conducted/ from 3:30 pm and 5:30 pm/ on Saturday. Spotting Error

The sports day events from 3:30 pm and 5:30 pm on Saturday will be conducted SSC CHSL (Tier-I) –05/08/2021 (Shift-I) Ans. (b) : In the above sentence 'to' will be used in place of 'and' because 'From......to' is used to show period of time. Correct Sentence The sports day events will be conducted from 3:30 pm to 5:30 pm on Saturday. 273. Identify the segment in the given sentence which contains the grammatical error. Employees were given incentives based at their performance. (a) based at (b) given incentives (c) Employees were (d) their performance SSC CHSL (Tier-I) –13/04/2021 (Shift-I) Ans. (a) : In the above sentence 'based on' will be use in place of 'based at' because based is always followed by 'on' preposition. Correct Sentence Employees were given incentives based on their performance. 274. The following sentence has been divided into parts. One of them contains an error. Select the part that contains the error from the given options. Several prominent film stars/have appeared on television/on behalf of awareness/about hand hygiene. (a) on behalf of awareness (b) about hand hygiene (c) Several prominent film stars (d) have appeared on television SSC CGL (Tier-I) –23/08/2021 (Shift-I) Ans. (a) : In the above sentence 'for' will be used in place of 'on behalf of' because according to the sentence it given appropriate meaning. Correct Sentence Several prominent film stars have appeared on television for awareness about hand hygiene. 275. The following sentence has been split into four segments. Identify the segment that contains a grammatical error. Won't you/please come/to help me/along this heavy box ? (a) to help me (b) won't you (c) please come (d) along this heavy box SSC CGL (Tier-I) –17/08/2021 (Shift-I) Ans. (d) : In the above sentence 'for this heavy box' will be used in place of 'along the heavy box' because 'for' preposition gives suitable meaning of the sentence. Correct Sentence Won't you please come to help me for this heavy box. 276. Identify the segment in the sentence which contains a grammatical error. This jewellery box is made from silver and is an antique piece. (a) an antique piece (b) This jewellery box (c) and is (d) is made from silver SSC CGL (Tier-II) 16/11/2020 (3 pm - 5 pm)

45

(a) (b) (c) (d)

YCT

Ans. (d) : In the above sentence 'of' will be used in place of 'from' because 'from' is used for chemical change and 'of' is used for physical change' The sentence is talking about making a silver box. Hence preposition 'of' would be appropriate. Correct sentence- This jewellery box is made of silver and is an antique piece. 277. Identify the segment in the sentence which contains a grammatical error: Poor people have run down of food supplies during the lockdown. (a) Poor people have (b) during the lockdown (c) food supplies (d) run down of SSC CGL (Tier-II) 16/11/2020 (3 pm - 5 pm) Ans. (d) : In the above sentence 'run out of' will be used in place of 'run down of' because it gives appropriate meaning. run down of - Slow down run out of - To End Correct sentence - Poor people have run out of food supplies during the lockdown. 278. Identify the segment in the sentence, which contains the grammatical error. We reserved tickets for a journey on train for the next morning for my sisters and me. (a) for a journey on train (b) for my sisters and me (c) for the next morning (d) We reserved tickets SSC CPO-SI (Tier-II) 27/09/2019 (3 pm - 5 pm) Ans. (a) : In the above sentence 'by' preposition will be used in place of 'on' because it gives appropriate meaning. Correct sentenceWe reserved by tickets for a journey by train for the next morning for my sisters and me. 279. Identify the segment in the sentence, which contains the grammatical error. The Principal was extremely angry on the boys who threw the pieces of chalk at the teacher. (a) at the teacher (b) extremely angry on the boys (c) who threw the pieces of chalk (d) The Principal was SSC CPO-SI (Tier-II) 27/09/2019 (3 pm - 5 pm) Ans. (b) : In the above sentence 'angry at' will be used in place of 'angry on' because angry is followed by following preposition. angry on something/material. angry at somebody/someone. Correct sentence- The Principal was extremely angry at the boys who threw the pieces of chalk at the teacher. 280. Identify the segment in the sentence, which contains the grammatical error. The actor smiled to me when I entered the room as if she knew me. (a) The actor smiled to me (b) knew me (c) as if she (d) when I entered the room SSC CPO-SI (Tier-II) 27/09/2019 (3 pm - 5 pm) Spotting Error

Ans. (a) : In the above sentence 'at' will be used in place of 'to' because smile at someone/ somebody is used. Correct sentenceThe actor smiled at me when I entered the room as if she knew me. 281. Identify the segment in the sentence, which contains the grammatical error. It is difficult to make out that they are twins because they do not resemble to each other. (a) resemble to each other (b) it is difficult to make out (c) because they do not (d) that they are twins SSC CHSL (Tier-I) 18/03/2020 (Shift-II) Ans. (a) : In the above sentence 'resemble' will be used in the place of 'resemble to' because resemble is not followed by any preposition. Correct SentenceIt is difficult to make out that they are twins because they do not resemble each other. 282. Identify the segment in the sentence, which contains the grammatical error. Each student will have to carry his own lunch on the picnic. (a) Each student (b) his own lunch (c) on the picnic (d) will have to carry SSC CPO-SI (Tier-II) 27/09/2019 (3 pm - 5 pm) Ans. (c) : According to the sense of above sentence 'to' preposition will be used in place of 'on' because in the sense of destination preposition 'to' is used. Correct sentence- Each student will have to carry his own lunch to the picnic. 283. Identify the segment in the sentence which contains the grammatical error. I have been living in Delhi since many years (a) since (b) have been living (c) in Delhi (d) many years SSC Stenographer Grade C&D –05/02/2019 (3:5pm)

Ans. (a) : In the above sentence 'for' will be used in place of 'since' because many years is a period of time. since- used to show a point of time. for- used to show a period of time. Correct sentence- I have been living in Delhi for many years. 284. Identify the segment in the sentence which contains the grammatical error. The children play on the park every evening. (a) on the park (b) The children (c) every evening (d) play SSC Stenographer Grade C&D –05/02/2019 (3:5pm)

Ans. (a) : In the above sentence 'in' will be used in place of 'on' because– In - used to show a situation when something is enclosed (a park or ground) On - used for a situation when something is positioned above something else. Correct sentence- The children play in the park every evening. 285. Identify the segment in the sentence which contains the grammatical error. This beautiful ring is made from gold. (a) from gold (b) This (c) beautiful ring (d) is made

46

SSC Stenographer Grade C&D –05/02/2019 (3:5pm)

YCT

Ans. (a) : In the above sentence 'of' will be used in place of 'from' because from is used for chemical change and 'of' is for physical change. Correct sentence- This beautiful ring is made of gold. 286. Identify the segment in the sentence which contains the grammatical error. Despite of working hard he failed the test. (a) the test (b) he failed (c) Despite of (d) working hard SSC Stenographer Grade C&D –05/02/2019 (3:5pm)

Ans. (c) : In the above sentence 'Despite' will be used in place of 'Despite of' because despite is followed by no preposition and inspite is followed by 'of' preposition in the same sense. Correct sentenceDespite working hard he failed the test. 287. In the sentence identify the segment which contains the grammatical error. Behavior modification techniques are often used to the training of police dogs. (a) Behavior modification techniques (b) of police dogs (c) to the training (d) are often used SSC CHSL (Tier-I) 12/10/2020 (Shift-III) Ans. (c) : In the above sentence 'for the training' will be used in the place of 'to the training' because after 'used' (verb) 'for' preposition is used. Use something for something. Correct Sentence :Behavior modification techniques are often used for training of police dogs. 288. In the following, some part of the sentence may have errors. Find out which part of the sentence has an error and select the appropriate option. If a sentence is free from error, select No Error. He said that he thought off politics right when (1)/he was studying Intermediate and that (2)/ he had no fear of politics. (3)/ No error. (4) (a) 1 (b) 2 (c) 3 (d) 4 SSC CGL (Phase-II) 17/02/2018 Ans. (a) In the above sentence 'about' will be used in place of 'off' because Thought is followed by 'about' preposition. Correct sentence He said that he thought about politics right when..... 289. In the following question, some part of the sentence may have errors. Find out which part of the sentence has an error and select the appropriate option. The information contained in this electronic (1)/ message and any attachments to this message (2)/ is intended for the exclusive use of this recipient. (3)/ No error (4). (a) 1 (b) 2 (c) 3 (d) 4 SSC MTS 11-10-2017 (Shift-I) Spotting Error

Ans : (c) In the above sentence 'is intended to' will be used in place of 'is intended for' because 'intended to something' or 'intended to doing something' is used. Correct Sentence : The information contained in this electronic message and any attachments to this message is intended to the exclusive use of this recipient. 290. In the following, some part of the sentence may have errors. Find out which part of the sentence has an error and select the appropriate option. If a sentence is free from error, select No Error. While provisions on equality and nondiscrimination (1)/ would promote equal opportunity, (2)/ in the process, reservation at jobs should not be denied. (3)/ No error. (4) (a) 1 (b) 2 (c) 3 (d) 4 SSC CGL (Phase-II) 17/02/2018 Ans. (c) In the above sentence 'in' will be used in place of 'at' because Reservation is followed by preposition 'in' Correct sentenceIn the process in reservation in jobs should not be denied. 291. In the following, some part of the sentence may have errors. Find out which part of the sentence has an error and select the appropriate option. If a sentence is free from error, select No Error. There is a barrier among the egghead and (1)/ the hoipolloi and it would be lazy (2)/ idealism to ignore it. (3)/ No error (4) (a) 1 (b) 2 (c) 3 (d) 4 SSC CGL (Phase-II) 17/02/2018 Ans. (a) In the above sentence 'between' will be used in place of 'among' because for comparing two objects use of 'between' and among for more than two things. Correct sentenceThere is a barrier between the egghead and…. 292. Identify the segment in the sentence which contains a grammatical error. The bottle slipped with my hand and broke into a thousand pieces. (a) into a thousand pieces (b) with my hand (c) the bottle slipped (d) broke into SSC CPO-SI – 25/11/2020 (Shift-I) Ans. (b) : In the above sentence 'from my hand' will be used in place of 'with my hand' because preposition 'from' is used for separation of one thing from another. Correct sentence The bottle slipped from my hand and broke into a thousand pieces. 293. Identify the segment in the sentence which contains a grammatical error. She has been working tirelessly since hours. (a) She has (b) since hours (c) tirelessly (d) been working SSC CPO-SI – 24/11/2020 (Shift-I)

47

YCT

Ans. (b) : In the above sentence 'for hours' will be used in place of 'Since hour' because since is used for definite time and for is used for indefinite time. Here hours is showing indefinite time. So it is appropriate to use 'for' before it.

since + point of time for + period of time Correct sentence She has been working tirelessly for hours. 294. Identify the segment in the sentence which contains a grammatical error. I remember a friend to me buying several pairs of shoes at a sale. (a) of shoes at a sale (b) a friend to me (c) I remember (d) buying several pairs SSC CPO-SI – 23/11/2020 (Shift-II) Ans. (b) : In the above sentence 'a friend of mine' will be used in place of 'a friend to me' because To show possession in a sentence. it would be appropriate to use possessive pronoun 'mine' and preposition 'of' in place of to. Correct sentence I remember a friend of mine buying several pairs of shoes at a sale. 295. In the sentence identify the segment which contains the grammatical error. The Japanese artist Yoh Nagao was busy splashing the well from colours. (a) splashing the well (b) was busy (c) from colours (d) The Japanese artist SSC CGL (Tier-I) – 07/06/2019 (Shift-II) Ans : (c) In the above sentence 'with colours' will be used in place of 'From colours' because splash is followed by 'with' preposition. Correct sentence The Japanese artist Yoh Nagao was busy splashing the well with colours. 296. In the sentence identify the segment which contains the grammatical error. Although there are more than a hundred known elements, they rarely occur at a pure state. (a) more than a hundred (b) they rarely occur (c) at a pure state (d) Although there are SSC CGL (Tier-I) – 13/06/2019 (Shift-II) Ans. (c) : In the above sentence preposition 'in' will be used in the place of 'at' because 'occur' is followed by preposition 'in', it means 'happen'. Correct sentence Although there are more than a hundred known elements, they rarely occur in a pure state. 297. In the sentence identify the segment which contains the grammatical error. Not complying by any of the laws can land you into serious trouble. (a) Not complying by (b) any of the laws (c) into senous trouble (d) can land you SSC CGL (Tier-I) – 12/06/2019 (Shift-I) Spotting Error

Ans : (a) In the above sentence ' Complying with' will be used in place of 'Complying by' because as a verb comply is always followed by preposition 'with'. Correct SentenceNot complying with any of the laws can land you into serious trouble. 298. In the sentence identify the segment which contains the grammatical error. My father dissuaded me to try for a job as he wanted me to pursue higher studies. (a) to try for a job (b) to pursue higher studies (c) as he wanted me (d) My father dissuaded me SSC CGL (Tier-I) – 11/06/2019 (Shift-II) Ans. (a) : In the above sentence 'from trying for a job' will be used in place of 'to try for a job' because dissuaded is followed by 'from' preposition and gerund (V1 + ing). Correct SentenceMy father dissuaded me from trying for a job as he wanted me to persue higher studies. 299. Identify the segment that contains a grammatical error. If there is no error, select 'No error' If he wants to recover, he must abstain for alcohol. (a) No error (b) wants to recover (c) he must (d) abstain for alcohol SSC CPO-SI – 11/12/2019 (Shift-II) Ans : (d) In the above sentence 'from' will be used in place of 'for' because Abstain is followed by 'from' preposition. Abstain from - to give something up. Correct sentenceIf he wants to recover, he must abstain from alcohol. 300. Identify the segment that contains a grammatical error. If there is no error, select 'No error' These facts should not have been disclosed for the public. (a) These facts should (b) not have been disclosed (c) No error (d) for the public SSC CPO-SI – 11/12/2019 (Shift-II) Ans : (d) In the above sentence 'to' will be used in place of 'for' because disclose is generally followed by preposition 'to'. 'Disclose to somebody' …. (to give information about something.) Correct Sentence These facts should not have been disclosed to the public. 301. In the sentence identify the segment which contains the grammatical error. After two months the phone signals were back at the valley. (a) were back (b) After two months (c) at the valley (d) the phone signals SSC CPO-SI – 11/12/2019 (Shift-I) Ans : (c) In the above sentence 'in' will be used in place of 'at' because preposition 'at' is used for definite area/field and preposition 'in' is used for big area. Correct SentenceAfter two months the phone signals were back in the valley.

48

YCT

302. Identify the segment that contains a grammatical error. If there is no error, select 'No error' Fate smiled above him in all his business ventures. (a) business ventures (b) in all his (c) No error (d) Fate smiled above him SSC CPO-SI – 13/12/2019 (Shift-I) Ans. (d) : In the above sentence 'on' preposition will be used with smile because smile on/upon is phrasal verb which means to make someone have good Luck/Success. Correct SentenceFate smiled on him in all his business ventures. 303. Identify the segment which contains the grammatical error. Lack of formal education did not prevent Lincoln to become a great leader. (a) did not prevent (b) a great leader (c) to become (d) Lack of SSC CHSL (Tier-I) –08/07/2019 (Shift-I) Ans : (c) In the above sentence 'form becoming' will be used in place of 'to become' because prevent always followed by 'from' preposition and v + ing form. Note :– Prevent sth/sb from doing sth Correct sentenceLack of formal education did not prevent Lincoln from becoming a great leader. 304. In the sentence identify the segment which contains the grammatical error. Darkness spilled like ink and began to spread at everything. (a) Darkness spilled (b) spread at everything (c) like ink (d) and began to SSC CHSL (Tier-I) –05/07/2019 (Shift-I) Ans : (b) In the above sentence 'spread over' will be used in place of 'spread at' because spread is followed by 'over' preposition. Correct sentenceDarkness spilled like ink and began to spread over everything. 305. In the sentence identify the segment which contains the grammatical error. Despite of his efforts he couldn't get a decent job. (a) his efforts (b) Despite of (c) decent job (d) he couldn't get SSC CHSL (Tier-I) –05/07/2019 (Shift-I) Ans : (b) In the above sentence 'Despite' will be used in place of 'Despite of' because preposition 'of' is not used with despite. Note- Inspite of/Despite Correct sentenceDespite his efforts he couldn't get a decent job. 306. Identify the segment which contains the grammatical error. The Principal requested to the Chief Guest to inaugurate the sports meet. Spotting Error

The principal requested the sports meet to the Chief Guest to inaugurate SSC CHSL (Tier-I) –04/07/2019 (Shift-III) Ans : (c) In the above sentence 'the chief Guest to' will be used in place of 'to the chief Guest' because there is no need of 'to' preposition here with request, as it is in indirect speech of imperative sentence. Correct SentenceThe Principal requested the Chief Guest to inaugurate the sports meet. 307. In the sentence identify the segment which contains the grammatical error. The launch of the small electric car will depend to the willingness of buyers to pay more for it. (a) depend to the willingness (b) of buyers to pay more for it (c) The launch of the (d) small electric car will SSC CHSL (Tier-I) –03/07/2019 (Shift-I) Ans : (a) In the above sentence 'depend on the willingness' will be used in place of 'depend to the willingness' because 'on' preposition is used with depend. Correct SentenceThe launch of the small electric car will depend on willingness of buyers to pay more for. 308. In the sentence identify the segment which contains the grammatical error. India's largest car maker is evaluating the launch to a small electric car next year. (a) next year (b) is evaluating the launch (c) India's largest car maker (d) to a small electic car SSC CHSL (Tier-I) –03/07/2019 (Shift-II) Ans. (d) : In the above sentence 'of' will be used in place of 'to' because according to the sense of the sentence it gives appropriate meaning with Launch. Correct SentenceIndia's largest car maker is evaluating the launch of a small electric car next year. 309. In the sentence identify the segment which contains the grammatical error. I expect to return from Singapore during about a week's time. (a) I expect to return (b) during about (c) a week's time (d) from Singapore SSC CHSL (Tier-I) –03/07/2019 (Shift-III) Ans. (b) : In the above sentence the use of During is superfluous and 'about' gives suitable meaning for the sentence. Correct sentenceI expect to return from Singapore about a week's time. 310. In the sentence identify the segment which contains the grammatical error. I visited my friend to whom I had made an appointment. (a) I visited my friend (b) I had made (c) to whom (d) an appointment SSC CHSL (Tier-I) –02/07/2019 (Shift-I)

49

(a) (b) (c) (d)

YCT

Ans. (c) : In the above sentence 'with Whom' preposition will be used in place of 'to whom' because it gives suitable meaning to the sentence. Correct sentenceI visited my friend with whom I had made an appointment. 311. In the sentence identify the segment which contains the grammatical error. On September 11, 2001, two hijacked aircrafts crashed to the World Trade Centre in New York. (a) two hijacked aircrafts (b) On September 11, 2001 (c) crashed to the (d) World Trade Centre in New York SSC CHSL (Tier-I) –02/07/2019 (Shift-II) Ans : (c) In the above sentence preposition 'into' will be used with crash. Crash into something (of vehicle) Correct sentenceOn September 11, 2001, two hijacked aircrafts crashed into the World Trade Centre in New York. 312. In the sentence identify the segment which contains the grammatical error. Tilak rowed through the river and tied his boat on the other side. (a) and tied his boat (b) on the other side (c) Tilak rowed (d) through the river SSC CHSL (Tier-I) –01/07/2019 (Shift-III) Ans : (d) In the above sentence preposition 'across' will be used in place of 'through' because it gives appropriate meaning according to the sense of sentence. Correct sentenceTilak rowed across the river and tied his boat on the other side. 313. In the sentence identify the segment which contains the grammatical error. The country has the lowest per capita income and needs to create jobs to the youth. (a) needs to create (b) the lowest per capita income (c) the country has (d) jobs to the youth SSC CHSL (Tier-I) 26/10/2020 (Shift-II) Ans. (d) : In the above sentence 'Jobs for the youth' will be used in place of 'Jobs to the youth' because 'for' preposition give appropriate meaning to the sentence. Correct sentence : The country has the lowest per capita income and needs to create jobs for the youth. 314. In the sentence identify the segment which contains the grammatical error. The manager was referring about the previous instances of loss to the employees during his address. (a) the previous instances (b) of loss to the employees (c) during his address (d) The manager was referring about SSC CHSL (Tier-I) 19/03/2020 (Shift-III) Spotting Error

Ans. (d) : In the above sentence 'to' will be used in place of 'about' because verb 'refer' followed by 'to' preposition. Correct sentence : The manager was referring to the previous instances of loss to the employees during his address. 315. In the sentence identify the segment which contains the grammatical error. Though she was able to finish the work on time she couldn't do that out in fear. (a) she couldn't do that (b) Though she was able to (c) finish the work on time (d) out in fear SSC CHSL (Tier-I) 18/03/2020 (Shift-I) Ans. (d) : In the above sentence 'out of fear' will be used in place of 'out in fear' because it gives appropriate meaning of the sentence. Correct sentence : Though she was able to finish the work on time she couldn't do that out of fear. 316. In the given sentence identify the segment which contains the grammatical error. My hotel in Goa had a wonderful view of the sea in the window. (a) of the sea (b) My hotel in Goa (c) in the window (d) had a wonderful view SSC CHSL (Tier-I) 12/10/2020 (Shift-II) Ans. (c) : In the above sentence 'From the window' will be used in place of in 'in the window'. Because it gives suitable meaning according to the sense of the sentence. Correct sentence : My hotel in Goa had a wonderful view of the sea from the window. 317. Identify the segment in the given sentence, which contains the grammatical error. Did you tell to your mother that you saw her friend at the theater? (a) Did you tell to (b) at the theater (c) your mother that (d) you saw her friend SSC CHSL (Tier-I) 14/10/2020 (Shift-III) Ans. (a) : In the above sentence 'Did you tell' will be used instead of 'Did you tell to', because preposition 'to' is not used after verb 'tell'. Correct sentence : Did you tell your mother that you saw her friend at the theater? 318. In the given sentence, identify the segment which contains the grammatical error. A panel has been formed to look within the security needs of college students. (a) of college students (b) to look within (c) has been formed (d) the security needs SSC CHSL (Tier-I) 14/10/2020 (Shift-II) Ans. (b) : In the above sentence 'To look into' will be used in the place of 'to look within' because it give suitable meaning. Phrase :– Look into– to try discover the fact about something. Correct sentence : A panel has been formed to look into the security needs of college students.

50

YCT

319. Identify the segment in the sentence which 323. Identify the segment in the sentence which contains the grammatical error from the given contains the grammatical error. options. Pallavi was relaxing in the beach at this time In the evening he likes to read books and yesterday. articles that have nothing to do of his work. (a) beach at this (b) Pallavi was (a) do of his work (c) time yesterday (d) relaxing in the (b) In the evening he likes SSC MTS – 21/08/2019 (Shift-II) (c) that have nothing to Ans. (d) : In the above sentence 'Relaxing on the' will (d) to read books and articles SSC MTS – 09/08/2019 (Shift-I) be used in place of 'relaxing in the'. because On the beach-it means you are literally on the surface Ans. (a) : In the above sentence 'Do with his work' of a beach. will be used in place of 'do of his work', because the Correct sentence : use of 'do with' is appropriate in the sense of passing Pallavi was relaxing on the beach at this time time. Which means having or possessing. yesterday. Correct sentence : In the evening he likes to read books and articles that 324. Identify the segment in the sentence which have nothing to do with his work. contains the grammatical error. 320. Identify the segment in the sentence which The travel agent is making the arrangements contains the grammatical error from the given from my trips. options. (a) the arrangements (b) is making The city of Banaras is widely known from its (c) from my trips (d) The travel agent ghats and temples. SSC MTS – 21/08/2019 (Shift-II) (a) ghats and temples (b) The city of Banaras Ans. (c) : In the above sentence 'For my trips' will be (c) from its (d) is widely known SSC MTS – 08/08/2019 (Shift-III) used in place of 'from my trips'. Because for is used when something is related or concerned with someone. Ans. (c) : In the above sentence 'for' will be used in It also refers to when use talk about a purpose or a place of 'from'. because it gives appropriate meaning to reason for something. the sentence. because from is used to indicate. The point Correct sentence : in space at which a journey, motion or action parts. The travel agent is making the arrangements for my known for something trips. known to somebody 325. Identify the segment in the sentence which Correct sentence : contains the grammatical error. The city of Banaras is widely known for its Ghats and Diksha has been in a grumpy mood from she temples. got up. 321. Find the part of the given sentence that has an (a) from she got up (b) Diksha has error in it. If there is no error, choose 'No (c) grumpy mood (d) been in a error'. SSC MTS – 21/08/2019 (Shift-I) The child always goes for sleep with his teddy Ans : (a) : In the above sentence 'Since' will be used in bear./No error place of 'from'. Because it is clear from the use of 'has (a) with his teddy bear (b) goes for sleep been' that 'the sentence' is in present perfect continuous (c) No error (d) The child always SSC MTS – 05/08/2019 (Shift-I) tense. Hence 'since' will be used before point of time Ans. (b) : In the above sentence 'Goes to sleep' will be (She got up). used instead of 'goes for sleep', because 'Go to sleep' Correct sentence : usually implies that the person is in a place designated Diksha has been in a grumpy mood since she got up. for sleeping. 326. Identify the segment in the sentence which Correct sentence : contains the grammatical error. The child always goes to sleep with his teddy bear. To escape floods, a tigress took refuge at a 322. Identify the segment in the sentence which vegetable garden contains the grammatical error. (a) took refuge Sakshi discussed about the problem with her (b) at a vegetable garden manager very briefly. (c) a tigress (a) with her manager (b) about the problem (d) To escape floods (c) Sakshi discussed (d) very briefly SSC MTS – 20/08/2019 (Shift-III) SSC MTS – 22/08/2019 (Shift-I) Ans. (b) : 'In' preposition will be used instead of 'at' in Ans. (b) : The use of 'about' will not be appropriate in the above sentence. the above sentence. 'Take' refuse in :– (protection from danger) is used as Note- The verbs discuss, order and stress are transitive idiomatic. and therefore they are not followed by any preposition. Correct sentence : Correct sentence : Sakshi discussed the problem with her manager very To escape floods, a tigress took refuge in a vegetable briefly. garden Spotting Error

51

YCT

327. Identify the segment in the sentence which contains the grammatical error. The robbers broke in the house at night. (a) at night (b) the house (c) the robbers (d) broke in SSC MTS – 20/08/2019 (Shift-II) Ans. (d) : In the above sentence 'Broke into' will be used in place of 'broke in' because 'broke into (to enter a building by force)' gives appropriate meaning for the sentence. Correct sentence : The robbers broke into the house at night. 328. Identify the segment in the sentence which contains the grammatical error. He sat in his stool and shut his eyes to recollect what work he had for the day. (a) he had for the day (b) to recollect what work (c) He sat in his stool (d) and shut his eyes SSC MTS – 19/08/2019 (Shift-II) Ans. (c) : In the above sentence, 'On' is used instead of 'in'. Because the use of 'on' is appropriate in the sense of sitting on a stool. Because there is no hand in it. While 'in the chair, in the sofa, in the bed' is used. Correct sentence : ''He sat on his stool and shut his eyes to recollect what work he had for the day.'' 329. Identify the segment in the sentence which contains the grammatical error. We have been staying at the hotel since the last ten days. (a) since the (b) we have been (c) last ten days (d) staying at the hotel SSC MTS – 16/08/2019 (Shift-III) Ans. (a) : In the above sentence , 'for' will be used in place of 'since'. Because the use of 'for' shows an indefinite time. (the last ten days) Correct sentence : We have been staying at the hotel for the last ten days. 330. Identify the segment in the sentence which contains the grammatical error from the given options. There is no point of arguing over something so unimportant. (a) over something (b) there is no (c) point of arguing (d) so unimportant SSC MTS – 13/08/2019 (Shift-III) Ans. (c) : In the above sentence 'No point in' will be used instead of 'no point of'. Because 'no point in' is a phrase which means- 'will not yield any worthwhile results'. Correct Sentence : There is no point in arguing over something so unimportant. 331. Identify the segment in the sentence which contains the grammatical error. They have been living in Boston from ten years. (a) in Boston (b) been living (c) They have (d) from ten years SSC MTS – 13/08/2019 (Shift-I) Spotting Error

Ans. (d) : In the above sentence, the use of 'for' will be appropriate in place of 'from'. because it is clear from the use of have been + verb + ing that the sentence is in present perfect continuous tense. And since/for + time is used in this tense. Since– point of time (2 o' clock , January 1988)/for period of time (2 hours, 2 months, 10 years). Correct sentence : They have been living in Boston for ten years. 332. From the given options, identify the segment in the sentence which contains the grammatical error. One could argue that these are the compulsions of parliamentary democracy for smooth functioning for the State. (a) of parliamentary democracy (b) these are the compulsions (c) one could argue that (d) functioning for the State SSC GD – 18/02/2019 (Shift-II) Ans. (d) : In the above sentence 'Functioning of the state' will be used in place of Functioning for the state. Because preposition 'of' will be used to denote the possession. Correct sentence : One could argue that these are the compulsions of parliamentary democracy for smooth functioning of the State. 333. From the given option, identify the segment in the sentence which contains the grammatical error. After finishing his tiring game, Rohan leaned by the wall. (a) his game (b) Rahan leaned (c) After finishing (d) by the wall SSC GD – 14/02/2019 (Shift-I) Ans. (d) : In the above sentence, 'against the wall' will be used in place of 'by the wall'. Lean against something - to sit or stand with part of your body touching something as a support. Correct Sentence : After finishing his tiring game, Rohan leaned against the wall. 334. Identify the segment in the sentence which contains the grammatical error from the given options. She complained about her weak eyes and lamented about the necessity from spectacles. (a) lamented about (b) from spectacles (c) she complained (d) her weak eyes SSC GD – 11/02/2019 (Shift-III) Ans. (b) : The above sentence 'Of' (preposition) will be used in place of 'from', because the use of 'necessity of + something' is appropriate. Correct Sentence : She complained about her weak eyes and lamented about the necessity of spectacles. 335. From the given options, identify the segment in the sentence which contains the grammatical error. Does he like to go for sightseeing when he is in holiday? (a) when he is (b) Does he like (c) in holiday (d) to go for sightseeing SSC GD – 11/02/2019 (Shift-II)

52

YCT

Ans. (c) : According to the sense of the above sentence 'On' preposition will be used in place of 'in' before the word holiday. Because 'on' gives proper meaning in the sense. 'On' is used in the following way. On Diwali, On Holi, On Monday, On Holiday, On vacation etc. Correct sentence : Does he like to go for sightseeing when he is on holiday?

E.

Conjunction

336. Identify the segment which contains the grammatical error. The soldiers had no choice when to obey the commander. (a) The soldiers (b) had no choice (c) when to obey (d) the commander SSC CHSL (Tier-I) –04/07/2019 (Shift-II) Ans : (c) In the above sentence ‘But to obey’ would be appropriate in place ‘when to obey’. Because ‘but’ is worked as a conjunction to join two contrast sentences. Correct Sentence :The soldiers had no choice but to obey the commander. 337. In the given sentence, identify the segment which contains a grammatical error. While the night we felt afraid as the resort was very dark and secluded. (a) While the night (b) was very dark (c) and secluded (d) we felt afraid SSC Stenographer (Grade C & D) 15.11.2021 Shift-II Ans. (a) : In the above sentence, ' as the night' will be used in place of 'while the night', because some conjunctions are not used alone, they are always used in pairs like So____as,/Too_____to,/Either_______or,/as_____as,/ Neither_____nor etc. Correct Sentence– As the night we felt afraid as the resort was very dark and secluded. 338. The following sentence has been split into four segments. Identify the segment that contains a grammatical error. I can clearly/recollect the day/as I first/met you. (a) as I first (b) recollect the day (c) I can clearly (d) met you SSC Stenographer (Grade C & D) 15.11.2021 Shift-I Ans. (a) : In the above sentence 'When' (conjunction) will be used in place of conjunction 'as'. because the correct from will be "When I first meet you" Correct SentenceI can clearly recollect the day when I first met you. 339. The following sentences has been splite into four segments. Identify the segment that contains a grammatical error. No sooner did/the child start crying/then the mother/hugged him. (a) hugged him (b) No sooner did (c) the child start crying (d) then the mother SSC CGL (Tier-I) 11/04/2022 Shift-III Spotting Error

Ans. (d) : In the given sentence, 'than the mother' will be used instead of 'then the mother, because 'than' is always used with no-sooner. No sooner................than Scarcely/hardly..............when Correct Sentence– No sooner did the child start crying than the mother hugged him. 340. The following sentence has been split into four segments. Identify the segment which contains a grammatical error. Until I was/still a little weak, /I decided to walk home/from the metro station. (a) still a little weak (b) I decided to walk home (c) from the metro station (d) Until I was SSC CHSL (Tier-I) –16/04/2021 (Shift-I) Ans. (d) : In the above sentence 'Although I was' will be used in place of 'Until I was' because 'until' is a time denoting conjunction while 'although' is a contrast denoting conjunction which gives the appropriate meaning to the sentence. Correct Sentence Although I was still a little weak I decided to walk home from the metro station. 341. In the given sentence, identify the segment which contains a grammatical error. If you rent a house near your office, it will save you time as well as also money. (a) If you rent a house (b) as well as also money (c) near your office (d) it will save you time SSC CHSL (Tier-I)–06/08/2021 (Shift-I) Ans. (b) : In the above sentence 'Also' will not be used after 'as well as' because the use of also is superfluous in the sentence. Correct Sentence If you rent a house near your office, it will save you time as well as money. 342. The following sentence has been split into four segments. Identify the segment which contains a grammatical error. Notwithstanding we were / all busy that weekend / we had to / cancel the outing. (a) cancel the outing (b) we had to (c) Notwithstanding we were (d) all busy that weekend SSC CHSL (Tier-I)–06/08/2021 (Shift-I) Ans. (c) : In the above sentence, 'because of' will be used in place of 'Notwithstanding' because it gives suitable meaning, according to the sense of sentence. Correct Sentence Because of we were all busy that weekend we had to cancel the outing. 343. Identify the segment in the sentence which contains a grammatical error. We must plan how can we cope with the present situation. (a) We must plan (b) present situation (c) how can we (d) cope with the SSC CGL (Tier-II) 16/11/2020 (3 pm - 5 pm)

53

YCT

Ans. (c) : In the above sentence 'How we can' will be used in place of 'how can we' because 'how' is used as a conjunction not as a question word. because the sentence is in affermative. Correct sentence- We must plan how we can cope with the present situation. 344. In the following, some part of the sentence may have errors. Find out which part of the sentence has an error and select the appropriate option. If a sentence is free from error, select No Error. The postman travelled by the cycle, often wading through (1)/swamps of passed through jungles (2)/ in order to reach the villages (3)/ No error (4) (a) 1 (b) 2 (c) 3 (d) 4 SSC CGL (Phase-II) 17/02/2018 Ans. (b) In the above sentence 'And' will be used in place of 'of' because 'and' is used to join two noun or clause. Correct sentenceThe postman travelled by the cycle, often wading through swamps and passed through jungles in order to reach the villages. 345. Identify the segment in the sentence which contains the grammatical error. Preeti is absent from the meeting due to she is out of station. (a) Preeti is absent (b) due to she is (c) from the meeting (d) out of station SSC MTS – 21/08/2019 (Shift-I) Ans : (b) In the above sentence, the use of because in place of 'due to' would be appropriate. Because in the sense of reason 'because' is used as a conjunction. Correct sentence : Preeti is absent from the meeting because she is out of station. 346. Identify the segment in the sentence which contains a grammatical error. Zoya won the first prize in the race unless she stumbled and fell. (a) Zoya won the first (b) prize in the race (c) unless she (d) stumbled and fell SSC CGL (Tier-II) 16/11/2020 (3 pm - 5 pm) Ans. (c) : 'Inspite of /despite' would be appropriate in place of 'unless' in the above sentence. It means –: in spite of (used to show that somebody did not intend to do the thing mentioned). Hence it gives suitable meaning, according to the sense of sentence. Correct sentence- Zoya won the first prize in the race inspite of she stumbled and fell. 347. Identify the segment in the sentence which contains a grammatical error. Miss Marple is neither a good singer or the good stage artist. (a) stage artist (b) Miss Marple is (c) neither a good singer (d) or the good SSC CPO-SI – 24/11/2020 (Shift-II) Spotting Error

Ans. (d) : 'Nor the good' will be used in place of 'or the good' in the above sentence. Because co-relative conjunction of neither is used with 'nor'.

Neither..............Nor Either................Or Correct sentence :Miss Marple is neither a good singer nor the good stage artist. 348. In the sentence identify the segment which contains the grammatical error. We had to decline several orders in case that the production was held up due to labour strike. (a) in case that (b) due to labour strike (c) the production was held up (d) we had to decline SSC CGL (Tier-I) – 04/06/2019 (Shift-I) Ans : (a) : In the above sentence 'Because' conjunction would be appropriate in place of 'in case that'. Because the use of 'because' conjunction shows the reason which gives suitable meaning for the sentence. Correct sentenceWe had to decline several orders because the production was held up due to labour strike. 349. In the sentence identify the segment which contains the grammatical error. She lost a big order from a known showroom in case of her own carelessness. (a) She lost a big order (b) her own carelessness (c) from a known showroom (d) in case of SSC CGL (Tier-I) – 04/06/2019 (Shift-III) Ans. (d) : In the above sentence, 'because' is used in place of 'in case of '. The use of 'because' (conjunction) is showing the reason. Which gives the complete meaning of sentence. Correct sentence :She lost a big order from a known showroom because her own carelessness. 350. In the sentence identify the segment which contains the grammatical error. Cyclone ldai killed at least 157 people in Zimbabwe and Mozambique although it tore across Southern Africa. (a) Although (b) It tore across (c) At least 157 people (d) Cyclone ldai killed SSC CGL (Tier-I) – 04/06/2019 (Shift-I) Ans : (a) In the above sentence Conjunction 'because/as' would be appropriate in place of 'Although'. Rule- Because is used before another part of speech to give the reason for something. Correct sentence :Cyclone ldai killed at least 157 people in Zimbabwe and Mozambique because/as it tore across Southern Africa. 351. In the sentence identify the segment which contains the grammatical error. She got two quick promotions in order that she has good communication skills.

54

YCT

(a) (b) (c) (d)

two quick promotions in order that she has good communication skills She got SSC CGL (Tier-I) – 04/06/2019 (Shift-II) Ans : (b) In the above sentence 'Because' will be used in place of 'In order that'. Because is used to talk about in the sense of reason. Correct SentenceShe got two quick promotions because she has good communication skills. 352. In the sentence identify the segment which contains the grammatical error. No sooner did the dark clouds gather in the sky when the electricity was switched off. (a) No sooner did (b) the dark clouds gather. (c) in the sky (d) when the electricity was switched off SSC CPO-SI – 13/12/2019 (Shift-II) Ans. (d) : In the above sentence, 'than' will be used in place of 'when'. Because the following structure is used.

No sooner + did + sub + verb1 .........than No sooner + had + sub+verb3 .........than Hardly / Scarcely + had + sub + verb3 .........when Correct Sentence :No sooner did the dark clouds gather in the sky than the electricity was switched off. 353. In the sentence identify the segment which contains the grammatical error. Our nation can make significant growth but if we all work collectively. (a) work collectively (b) but if we all (c) our nation can (d) make significant growth SSC CHSL (Tier-I) –11/07/2019 (Shift-III) Ans : (b) In the above sentence the use of conjunction 'but' is superfluous. because conjunction 'if' will be used for condition. Which gives the complete meaning of the sentence. Correct Sentence :Our nation can make significant growth if we all work collectively. 354. Identify the segment in the sentence which contains the grammatical error. Inspite Smita had taken part in many television serials earlier, this was the first time she was going to host a live show. (a) in many television serials earlier (b) she was going to host (c) this was the first time (d) Inspite Smita has taken part SSC CHSL (Tier-I) –10/07/2019 (Shift-I) Ans : (d) In the above sentence, 'Inspite of' will be used in place of 'Inspite'. Because preposition 'of' is used with 'Inspite'. While 'despite' is used without 'of' in the same sense. Correct sentence : Inspite of Smita had taken part in many television serials earlier, this was the first time she was going to host a live show. Spotting Error

355.

Identify the segment in the sentence which contains the grammatical error. No sooner did I reach the station when the train left. (a) I reach the (b) the train left (c) No sooner did (d) station when SSC MTS – 21/08/2019 (Shift-I) Ans : (d) In the above sentence , 'than' will be used in place of 'when'. Note- Here 'No sooner––––than' will be used as a Conjunction. Correct sentence : No sooner did I reach the station than the train left. 356. Identify the segment in the sentence which contains the grammatical error. Even Sharat tried his best, he could not clear all the examination papers in one attempt. (a) he could not clear (b) Even Sharat tried his best (c) all the examination papers (d) in one attempt SSC CHSL (Tier-I) –10/07/2019 (Shift-II) Ans : (b) In the above sentence Conjunction 'Even though' will be used in the place of adverb 'even'. Because 'even though' is used when Result have not been received according to the work. Correct sentence :Even though sharat tried his best, he could not clear all the examination papers in one attempt. 357. Identify the segment in the sentence which contains the grammatical error. As you know that I always like to have icecream after my dinner. (a) As you know that (b) to have ice-cream (c) after my dinner (d) I always like SSC CHSL (Tier-I) –09/07/2019 (Shift-I) Ans : (a) In the above sentence' 'As you know' will be used in place of 'As you know that'. Because when 'as' is used at the beginning of the sentence, then 'that' is not used as a conjunction while (,) comma is used in place of 'that'. Correct sentence :As you know, I always like to have ice cream after my dinner. 358. Select the sentence identify the segment which contains the grammatical error. Different-coloured natural foods not only put a smile on children's faces and also benefit their overall growth. (a) Different-coloured natural foods (b) put a smile on children's faces (c) and also benefit (d) benefit their overall growth SSC CHSL (Tier-I) –02/07/2019 (Shift-III) Ans : (c) In the above sentence 'But also benefit' would be appropriate in place of 'And also benefit'. Because co-relative conjunction 'but also' is used with not only. Correct sentence :Different-coloured natural foods not only put a smile on children's faces but also benefit their overall growth.

55

YCT

359. Identify the segment in the sentence, which contains the grammatical error. Despite of her tiredness Meera went into the kitchen to make some dinner for the children. (a) for the children (b) Meera went into the kitchen (c) to make some dinner (d) Despite of her tiredness SSC CHSL (Tier-I) 14/10/2020 (Shift-I) Ans. (d) : In the above sentence, 'Despite' will be used in place of 'Despite of'. Because 'of' is not used with Despite. Correct sentence :Despite her tiredness Meera went into the kitchen to make some dinner for the children. 360. Select the segment in the sentence, which contains the grammatical error. No matter what that the mother did the baby didn't stop crying. (a) what that (b) the mother did (c) the baby didn't stop crying (d) no matter SSC CHSL (Tier-I) 17/03/2020 (Shift-III) Ans. (a) : In the above sentence, 'what' will be used as a conjunction in place of 'what that', because 'no matter what' is a phrase followed by a noun. Hence the use of 'that' is unnecessary. Correct sentence :No matter what the mother did the baby didn't stop crying. 361. Identify the segment in the sentence which contains the grammatical error from the given options. If they said nothing, she could sense their disapproval of her suggestion. (a) If they said nothing (b) her suggestion (c) their disapproval of (d) she could sense SSC MTS – 09/08/2019 (Shift-I) Ans. (a) : In the above sentence, 'although they said nothing' will be used in place of 'if they said nothing'. Because 'although' is used connector between the two contrast sentences. While 'if' is used in the sense of condition. Correct sentence :Although they said nothing, she could sense their disapproval of her suggestion. 362. Find the part of the given sentence that has an error in it. If there is no error, choose 'No error'. Do not let the opportunity slip away and you will repent./No error (a) No error (b) and you will repent (c) Do not let the (d) opportunity slip away SSC MTS – 07/08/2019 (Shift-I) Ans. (b) : In the above sentence 'Or' will be used in place of 'And'. because 'Or' is used to say otherwise. It is used to introduce the consequences of something not being done. Correct sentence :Do not let the opportunity slip away or you will repent. Spotting Error

363. Identify the segment in the sentence which contains the grammatical error. Ram cannot withdraw all his money unless he does not given a prior notice. (a) all his money (b) Ram cannot withdraw (c) a prior notice (d) unless he does not give SSC MTS – 19/08/2019 (Shift-I) Ans. (d) : In the above sentence, unless he gives' will be used in place of 'unless he does not give'. Because 'not' is not used with 'unless'. 'Unless' is itself a negative word (conjunction) which means – if ---- not, except if (if not, not until) here not is superflous. Correct Sentence :Ram cannot withdraw all his money unless he gives a prior notice. 364. Identify the segment in the sentence whih contains the grammatical error. If there is no error, select 'No Error'. Sushmita's youngest sister is not only beautiful but even intelligent also. (a) but even intelligent also (b) is not only beautiful (c) No Error (d) sushmita's youngest sister SSC MTS – 19/08/2019 (Shift-I) Ans. (a) : In the above sentence, 'but also intelligent' will be use in place of 'but even intelligent also in the sentence. because not only------------but also'' is used as correlative Conjunction, therefore words of the same parts of speech are added to it. Correct Sentence :Sushmita's youngest sister is not only beautiful but also intelligent.

F.

Article

365. Identify the segment that contains a grammatical error. If there is no error, select 'No error' In 79 A.D., the Mount Vesuvius blew/its top, burying the ancient/ cities of Pompeii and Herculaneum. (a) cities of Pompeii and Herculaneum (b) its top, burying the ancient (c) In 79 A.D., the Mount Vesuvius blew (d) No Error SSC MTS 10/05/2023 (Shift-I) Ans. (c) : In the above sentence article 'the'will not be used before mountain name. The correct sentence is'In 79, A.D. Mount Versuvius blew ..... .' Thus, the correct answer is option (c). 366. Identify the segment that contains a grammatical error. Ir there is no error, select 'No error' / An appeasements policies/were all but a sham to misdirect/the local groups. (a) An appeasements policies (b) the local groups (c) were all but a sham to misdirect (d) No error SSC MTS 19/05/2023 (Shift-III)

56

YCT

Ans. (a) : The segment of the above sentence – ‘An appeasements policies’ is grammatically incorrect because the article ‘An’ is used before indefinite singular countable noun. Article ‘the’ is used before the singular/plural countable noun in the place of article ‘An’. The correct segment is – ‘The appeasements policies’. 367. Identify the segment that contains a grammatical error. If there is no error, select 'No error'. Her evaluation will give/me a best indication of/where I stand in the class. (a) Her evaluation will give (b) me a best indication of (c) where I stand in the class (d) No error SSC MTS 15/05/2023 (Shift-I) Ans. (b) : In the above given sentence, the segment 'me a best indication of' is grammatically incorrect, because before the superlative adjective (best) article 'the' is used in the place of 'a'. The correct segment is – 'me the best indication of'. Thus, the correct answer is option (b). 368. Parts of the following sentence have been given as options. Select the option that contains an error. She was waiting for a opportunity to start her own business. (a) to start her (b) own business (c) for a opportunity (d) She was waiting SSC CGL (Tier-I) 21/07/2023 (Shift-II) Ans. (c) : In the above sentence option (c) 'for a opportunity' are grammatically incorrect because indefinite article 'an' is used in the place of article 'a', because Article 'An' is used before a singular Noun with a vowel sound. ⇒ 'A' is used before a singular noun with a consonant sound. Correct SentenceShe was waiting for an opportunity to start own business. 369. The following sentence has been split into four segments. Identify the segment that contains a grammatical error. You should / see a oculist; / your eyes look / worse today. (a) worse today (b) see a oculist; (c) your eyes look (d) You should SSC CGL (Tier-I) 26/07/2023 (Shift-II) Ans. (b) : For the above given sentence, the option (b) is incorrect because 'see a oculist' is replaced by 'see an oculist'. Note- Indefinite article 'an' will be used before vowel sound. (oculist) Thus, the correct answer is option (b). Correct SentenceYou should see an oculist your eyes look warse today. Spotting Error

370. Select the grammatically correct sentence. (a) It was the hectic day for Susan with a lot of purchase and the couple of meetings. (b) It was a hectic day for Susan with the lot of purchase and a couple of meetings. (c) It was a hectic day for Susan with the lot of purchase and an couple of meetings. (d) It was a hectic day for Susan with the lot of purchase and an couple of meetings. SSC CGL (Tier-I) 25/07/2023 (Shift-IV) Ans. (d) : The grammatically correct sentence of the above sentence is option (d) 'It was a hectic day for Susan with a lot of purchases and a couple of meetings.' Note:- 'A' is the 'indefinite Article', use before the singular countable noun with consonant sound. ⇒ 'A lot of'/lots of' can both be used with plural countable nouns and singular uncountable nouns. Hence, the correct answer is option (d). 371. The following sentence has been divided into three segments, A, B, C. One of them may contain a grammatical error. Select the segment that contains the error, from the given options. If you don’t find any error, mark ‘No error’ as your answer. Aniket lives (A) / in the small apartment (B) / in the suburbs (C). (a) No error (b) B (c) C (d) A SSC CGL (Tier-I) 18/07/2023 (Shift-III) Ans. (b) : The above given sentence, the segment 'in the small apartment' is grammatically incorrect, because article 'the' will be changed into 'a' before 'small apartment'. It is not specifying a particular or specific apartment. Hence, article 'the' is inappropriate. Thus, the correct answer is option (b). Correct SentenceAniket lives in a small apartment in the suburbs. 372. Identify the sentence that correctly uses the indefinite article. (a) She is a Indian artist married to a European engineer. (b) She is an Indian artist married to a European engineer. (c) She is a Indian artist married to an European engineer. (d) She is an Indian artist married to an European engineer. SSC CGL (Tier-I) 17/07/2023 (Shift-II) Ans. (b) : In the above sentence correctly use of the indefinite article is in option (b). Correct SentenceShe is an Indian artist married to a European engineer. 373. The following sentence has been split into four segments. Identify the segment that contains a grammatical error. The historians / are a great / asset to the / field of academia. (a) The historians (b) Are a great (c) Field of academia (d) Asset to the SSC CGL (Tier-I) 24/07/2023 (Shift-III)

57

YCT

Ans. (a) : In the above sentence 'Historians' will be used in place of 'the historians' because article 'the' is used in definite sense and here historians are not definite. Correct SenteceHistorians are a great asset to the field of academia. 374. The following sentence has been split into four segments. Identify the segment that contains a grammatical error. Babbal is / a oldest member / in our / Black Ice Club. (a) Black Ice Club (b) a oldest member (c) in our (d) Babbal is SSC CHSL (Tier-I) 02/08/2023 (Shift-I) Ans. (b) : In the above sentence, the segment ' a oldest member' is grammatically incorrect because article 'the' is used before the superlative adjective (oldest member). Correct sentence – Babbal is the oldest member in our black Ice club. 375. The following sentence has been divided into four parts. One of them contains an error. Select the part that contains the error from the given options. Honesty / is / a best / policy. (a) a best (b) is (c) Honesty (d) policy SSC CHSL (Tier-I) 04/08/2023 (Shift-III) Ans. (a) : In the above sentence option (a) 'a best is grammatically incorrect part. Because article 'the' is used before the superlative adjective (best). Correct sentence – Honesty is the best policy. 376. Identify the segment in the sentence which contains a grammatical error. A writer who wrote this book has been selected for this year's literature award. (a) wrote this book (b) A writer who (c) has been selected (d) for this year's literature award SSC Stenographer (Grade C & D) 11.11.2021 Shift-I Ans. (b) : In the above sentence, 'the writer who' will be used in place of 'A writer who', because the writer is adefinite noun. Therefore definite article 'the' will be used before writer. Correct sentence The writer who wrote this book has been selected for this year's literature award. 377. Identify the segment in the sentence which contains a grammatical error. We require the efficient electrician for changing the wiring of our house. (a) the efficient electrician (b) for changing the wiring (c) We require (d) of our house SSC Stenographer (Grade C & D) 12.11.2021 Shift-II Ans. (a) : In the above sentence 'An' will be used in place of 'the' in the above sentence because in indefinite sense, article 'an' is used before vowel sound word. Correct sentenceWe require an efficient electrician for changing the wiring of our house. Spotting Error

378. The following sentence has been split into four segments. Identify the segment that contains a grammatical error. Ronaldo is/a famous/player of/the football. (a) a famous (b) Ronaldo is (c) player of (d) the football SSC CHSL 27.05.2022 Shift-III Ans. (d) : In the above sentence, 'Football' will be used in place of 'the football'. Because article 'the' is used in definite sense. In general sense a noun is used without an article. Correct SentenceRonaldo is a famous player of football. 379. The following sentence has been split into four segments. Identify the segment that contains a grammatical error. This is /too grave sin/to be/pardoned. (a) This is (b) too grave sin (c) pardoned (d) to be SSC CGL (Tier-I) 12/04/2022 Shift-II Ans. (b) : In the above sentence, 'too grave a sin' will be used in place of 'too grave sin' because here the use of sin (noun) is used in indefinite sense. Hence Indefinite article 'a' will be used. Correct Sentence This is too grave a sin to be pardoned. 380. The following sentence has been divided into parts. One of them may contain a grammatical error. Select the part that contains the error from the given options. If you don’t find any error, mark ‘No error’ as your answer. Antique diamond necklace / was stolen / from the museum. (a) was stolen (b) No error (c) from the museum (d) Antique diamond necklace SSC CGL (Tier-I) 12/04/2022 Shift-II Ans. (d) : In the above sentence 'An antique' will be used in place of 'Antique' because in indefinite sense, article 'an' is used before vowel sound word. Correct SentenceAn Antique diamond necklace was stolen from the museum. 381. The following sentence has been into four segments. Identify the segment that contains grammatical error. Athens was/most luminous/ of all city-states/ for ancient Greece. (a) of all city-states (b) of ancient Greece (c) a most luminous (d) Athens was SSC CGL (Tier-I) 13/04/2022 Shift-III Ans. (c) : Definite article 'the' is used with superlative degree (most). Hence 'The' most luminous will be used in place of 'most luminous'. In the above sentence Correct sentence – Athens was the most luminous of all city-states of ancient Greece.

58

YCT

382. The following sentence has been divided into parts. One of them may contain an error. Select the part that contains the error from the given options. If you don’t find any error, mark ‘No error’ as your answer. It was / the very well-directed film / and we enjoyed it. (a) and we enjoyed it (b) the very well-directed film (c) No error (d) It was SSC CGL (Tier-I) 11/04/2022 Shift-I Ans. (b) : In the above sentence, ' a very well directed film, will be used instead of 'the very well directed film', because 'the' is used in definite sense while 'a' is used in indefinite sense. Hence Article 'a' will be used. Correct sentence – It was a very well directed film and we enjoyed it. 383. The following sentence has been split into four segments. Identify the segment that contains a grammatical error. The earthy / fragrance of soil / after a first rain / is always refreshing. (a) is always refreshing (b) fragrance of soil (c) The earthy (d) after a first rain SSC MTS-15/07/2022 Shift-I Ans. (d) : In the above sentence, 'After the first rain' will be used instead of 'after a first rain' because definite article 'the' is used before ordinal number (first, second, third etc). Correct Sentence – The earthy fragrance of soil after the first rain is always refreshing. 384. Parts of the following sentence have been given as options. One of them may contain an error. Select the part that contains the error from the given options. If you don't find any error, mark 'No error' as your answer. The telecom growth story is a important component of the broader story of India. (a) No error (b) The telecom growth story is (c) the broader story of India. (d) a important component of SSC MTS-06/07/2022 Shift-I Ans. (d) : In the above sentence 'An important component' will be used in place of 'a important component', because article 'an' is used before the word starting with vowel sound. (important) Correct Sentence – The telecom growth story is an important component of the broader story of India. 385. The following sentence has been divided into parts. One of them may contain an error. Select the part that contains the error from the given options. If you don't find any error, mark 'No error' as your answer. he is / struggling to make / a honest living . (a) Struggling to make (b) No error (c) a honest living (d) He is SSC MTS-08/07/2022 Shift-I Spotting Error

Ans. (c) : In the above sentence 'An honest living' will be used in place of a honest living. Because indefinite article 'an' is used before the noun of the vowel sound. Correct Sentence – He is struggling to make an honest living. 386. The following sentence has been split into four segments. Identify the segment that contains a grammatical error. Our team leader / told us to be at the work / by 9 a.m. / the next morning. (a) by 9 a.m. (b) the next morning (c) Our team leader (d) told us to be at the work SSC Constable GD-29/11/2021 Shift-II Ans. (d) : In the above sentence, 'at work' will be used instead of 'at the work', because article 'the' is used in specific sense. Hence, definite article 'the' will not be used. Correct Sentence Our team leader told us to be at work by 9 a.m. the next morning. 387. Parts of the given sentence have been given as options. One of them contains a grammatical error. Select the option that has the error. Go to the ninth block on the left-hand side and take a sharpest turn to reach Bunny’s house on the right. (a) Bunny's house (b) on the right1 (c) Go to the ninth block (d) take a sharpest turn SSC Constable GD-25/11/2021 Shift-III Ans. (d) : In the above sentence, article 'the' will be used in place of 'a', because definite article 'the' is used before superlative degree. (sharpest) Correct Sentence – Go to the ninth block on the left hand side and take the sharpest turn to reach Bunny's house on the right. 388. The following sentence has been split into four segments. Identify the segment that contains a grammatical error. I think/he has/settled/in United States. (a) settled (b) in united states (c) I think (d) he has SSC Constable GD-02/12/2021 Shift-I Ans. (b) : In the above sentence, 'In the united states' will be used instead of in united states. Because article 'the' is used before the name of some country. Note → Some countries as - The United States of America (USA), The Athens, The Philippines, The West Indies Correct sentence - I think he has settled in the united states. 389. The following sentence has been divided into parts. One of them contains an error. Select the part that contains the error the given options. The sea has become/an cheap dumping ground/ for all kinds/of waste products.

59

YCT

(a) (b) (c) (d)

an cheap dumping ground for all kinds of waste products the sea has become SSC CGL (Tier-I) –17/08/2021 (Shift-I) Ans. (a) : In the above sentence, 'a' will be used in place of 'an', because the pronunciation of 'cheap' is pronounced with (consonant sound). So Indefinite article 'a' will be used before it. While 'an' is used before vowel sound. Correct Sentence The sea has become a cheap dumping ground for all kind of waste products. 390. Identify the segment in the sentence which contains a grammatical error. There was enough evidence to convict him on selling of fake medicines. (a) There was enough evidence (b) Fake medicines (c) to convict him (d) on selling of SSC CGL (Tier-II) 16/11/2020 (3 pm - 5 pm) Ans. (d) : In the above sentence, 'on the selling of' will be used in place of 'on selling of', because when two nouns are joined by 'of' then 'the' is used before the first noun. Correct sentence- There was enough evidence to convict him on the selling of fake medicines. 391. Identify the segment in the sentence which contains a grammatical error. Although this is a narrowest street, many large trucks can enter it. (a) many large trucks (b) can enter it (c) Although this is (d) a narrowest street SSC CGL (Tier-II) 16/11/2020 (3 pm - 5 pm) Ans. (d) : In the above sentence, 'The narrowest' will be used instead of 'a narrowest', because definite article 'the' is used before superlative form (narrowest). Correct sentence- Although this is the narrowest street, many large trucks can enter it. 392. Identify the segment in the sentence which contains a grammatical error : Do you mind lending me your book for a hour? (a) lending me (b) Do you mind (c) for a hour (d) your book SSC CGL (Tier-II) 16/11/2020 (3 pm - 5 pm) Ans. (c) : 'An hour' will be used in place of 'a hour' in the above sentence. Because 'hour' is a vowel sounding word so article 'an' will be used before it. Correct sentence- Do you mind lending me your book for an hour? 393. Identify the segment in the sentence which contains the grammatical error. A Earth moves round the Sun. (a) moves (b) the Sun (c) A Earth (d) round SSC Stenographer Grade C&D –05/02/2019 (3:5pm) Ans. (c) : In the above sentence, 'The' Earth' will be used in place of 'A Earth', because definite article 'the' is used with celestial bodies (Earth, Sun, Moon etc.) Correct sentence- The Earth moves round the Sun.

Spotting Error

394. Select the segment in the sentence, which contains the grammatical error. We were busy at the work and went for lunch only at 2.30 pm. (a) We were busy (b) at the work (c) only at 2.30 pm (d) and went for lunch SSC CHSL (Tier-I) 17/03/2020 (Shift-III) Ans. (b) : In the above sentence, 'at work' will be used in place of 'at the work'. The definite article 'the' is usually not used before 'work'. Because in the sentence 'work' is used in indefinite sense and uncountable noon. Correct sentence : We were busy at work and went for lunch only at 2.30 pm. 395. Identify the segment in the sentence which contains a grammatical error. Rice of a best quality is available in the nearby supermarket at eighty rupees per kilo. (a) a best quality (b) at eighty rupees (c) in the nearby (d) is available SSC CPO-SI – 25/11/2020 (Shift-I) Ans. (a) : In the above sentence, 'The best' will be used instead of 'a best', because article 'the' is used before superlative degree. Correct sentence :Rice of the best quality is available in the nearby supermarket at eighty rupees per kilo. 396. Identify the segment in the sentence which contains a grammatical error. Two elderly ladies narrated an rather improbable story to the police. (a) to the police (b) improbable story (c) narrated an rather (d) Two elderly ladies SSC CPO-SI – 24/11/2020 (Shift-II) Ans. (c) : In the above sentence, the use of 'An rather' is incorrect. In its place ' rather an' will be used because the use of 'an' is used for improbable story in the sentence. Note –: Improbable is pronounced with 'vowel sound'. Therefore before this, indefinite article 'An' will be used. Correct Sentence :Two elderly ladies narrated rather an improbable story to the police. 397. In the sentence identify the segment which contains the grammatical error. In the northern suburbs of Bengaluru, home to the bulk of the information technology industry, the water crisis is even worst. (a) home to the bulk of (b) the water crisis (c) is even worst (d) In the northern suburbs SSC CGL (Tier-I) – 10/06/2019 (Shift-I) Ans : (c) In the above sentence, ' is even the worst' will be used instead of 'is even worst'. Because in the given sentence 'worst' which is superlative form of bad and 'the' is used before the superlative degree. Correct sentence :In the northern suburbs of Bengaluru, home to the bulk of the information technology industry, the water crisis is even the worst.

60

YCT

398. Identify the segment that contains a grammatical error. If there is no error, select 'No error' The day before yesterday he left for Europe by the air. (a) The day before yesterday (b) he left for Europe (c) No error (d) by the air SSC CPO-SI – 13/12/2019 (Shift-I) Ans. (d) : In the above sentence, 'by air' would be appropriate in place of 'by the air'. Because 'by' is used before the medium of transport but article 'the' is not used like – By air, by railway, by road, by water. Correct sentence :The day before yesterday he left for Europe by air. 399. Identify the segment in the sentence which contains the grammatical error. We will use enamel paint on this wall because it gives a best finish. (a) enamel paint on this wall (b) We will use (c) a best finish (d) because it gives SSC CHSL (Tier-I) –11/07/2019 (Shift-II) Ans : (c) In the above sentence, 'the best finish' will be used instead of 'a best finish', because definite article 'the' is used before superlative degree. Correct sentence :We will use enamel paint on this wall because it gives the best finish. 400. Identify the segment in the sentence which contains the grammatical error. With this heat wave on, we are having a terrible weather. (a) heat wave on (b) a terrible weather (c) with this (d) we are having SSC CHSL (Tier-I) –09/07/2019 (Shift-I) Ans : (b) In the above sentence, 'terrible weather' will be used instead of 'a terrible weather'. Because weather' is uncountable noun. Therefore before it, article (a/an) is not used. Correct sentence :With this heat wave on we are having terrible weather. 401. Identify the segment in the sentence which contains the grammatical error. The young girl was helping her father to look after shop. (a) her father (b) The young girl (c) was helping (d) to look after shop SSC CHSL (Tier-I) –09/07/2019 (Shift-II) Ans : (d) In the above sentence, 'to look after the shop' will be used in place of 'to look after shop'. Because in the above sentence, 'shop' is used as definite noun. Hence, before it, 'the' should be used. Note- 'A/An' is used before Indefinite noun. Correct sentence :The young girl was helping her father to look after the shop. 402. Identify the segment which contains the grammatical error. Climate change are a greatest threat facing our planet which needs our immediate attention. Spotting Error

our immediate attention are a greatest which needs facing planet SSC CHSL (Tier-I) –08/07/2019 (Shift-III) Ans : (b) In the above sentence, 'the greatest' will be used instead of 'greatest'. Because definite article 'the' is used before superlative degree. Correct sentence :Climate change are the greatest threat facing our planet which needs our immediate attention. 403. In the given sentence, identify the segment which contains a grammatical error. I think Virat Kohli will remain the captain of the Indian team in a year 2020. (a) of the Indian team (b) in a year 2020 (c) will remain (d) the captain SSC CHSL (Tier-I) 16/10/2020 (Shift-III) Ans. (b) : 'In the year 2020' would be appropriate instead of 'in a year 2020', in the above sentence, because article 'the' will be used as definite sense and in the sentence, 'year 2020' is giving the definite sense. Correct sentence :I think Virat Kohli will remain the captain of the Indian team in the year 2020. 404. In the following question, some part of the sentence may have errors. Find out which part of the sentence has as error and select the appropriate option. If a sentence is free from error, select 'No Error'. Teachers Day shall be observed (1)/ as non– teaching day (2)/ in our campus (3)/ No Error (4) (a) 1 (b) 2 (c) 3 (d) 4 SSC MTS 10-10-2017 (Shift-II) Ans. (b) : In the above sentence, 'as a non-teaching day' will be used in place of 'as non-teaching day. Because 'day' is countable noun and it is used as indefinite sense therefore before it, indefinite article 'a' will be used. Correct sentence : Teachers Day shall be observed as a non–teaching day in our campus. 405. In the sentence identify the segment which contains the grammatical error. According to a recent study, drinking green tea thrice a week is associated with a longest and healthy life. (a) a longest and healthy life (b) According to a recent study (c) is associated with (d) drinking green tea thrice a week SSC CHSL (Tier-I) 21/10/2020 (Shift-I) Ans. (a) : 'The longest and healthy life' would be appropriate instead of 'a longest and healthy life' in the above sentence. Because 'the' is always used before superlative degree and the superlative form of long is longest. Correct Sentence :According to a recent study, drinking green tea thrice a week is associated with the longest and healthy life.

61

(a) (b) (c) (d)

YCT

406. Identify the segment in the given sentence which contains the grammatical error. Most of voters in Taiwan seem to have once again voted for the democratic party and President Tsai Ing-wen. (a) once again voted for (b) the democratic pary (c) in Taiwan seem to have (d) Most of voters SSC CHSL (Tier-I) 13/10/2020 (Shift-III) Ans. (d) In the above sentence, 'Most of the Voters' will be used in place of 'most of voters'. Because with most of the following structure is used. most of + the + plural noun Correct Sentence :Most of the voters in Taiwan seem to have once again voted for the democratic party and President Tsai Ingwen. 407. In the given sentence identify the segment which contains the grammatical error. When I met Akbar Padamsee, one of a pioneers of Indian art, he was 87 but still alert and active. (a) he was 87 (b) when I met (c) one of a pioneers of Indian art (d) but still alert and active SSC CHSL (Tier-I) 12/10/2020 (Shift-I) Ans. (c) : 'One of the pioneers of Indian art' will be used instead of 'one of a pioneers of Indian art' in the above sentence. Because after one of' 'article the' and plural countable noun is used. Therefore here 'the' will be used in place of 'a'. Correct Sentence :When I met Akbar Padamsee, one of the pioneers of Indian art, he was 87 but still alert and active. 408. In the following question, some part of the sentence may have errors. Find out which part of the sentence has an error and select the appropriate option. If a sentence is free from error, select 'No Error'. An Country's Central News Agency (1)/said the launch was a (2)/''muscle-flexing''. (3)/ No Error (4) (a) 1 (b) 2 (c) 3 (d) 4 SSC MTS 9-10-2017 (Shift-I) Ans : (a) In the above sentence, 'A' will be used in place of 'An', because 'A' is used before consonant while article 'An' is used before vowel sound. Note :– If the pronunciation of consonant is vowel, then Article 'An' is used before it. Correct Sentence :A Country's Central News Agency said the launch was a ''muscle-flexing''. 409. In the following question, the sentence may have errors Find out which part of the sentence is an error and select the appropriate option. If a sentence is free from error, select 'No Error'. Spotting Error

However, the essential services (1) / and critical staff in a government (2) / will be on duty today (3) / No Error (4) (a) 1 (b) 2 (c) 3 (d) 4 SSC MTS 9-10-2017 (Shift-III) Ans : (b) In the above sentence, 'the government' would be appropriate in place of 'a government', because here the use of Government is used as specific sense. Therefore before it, definite article 'the' will be used. Correct Sentence :However, the essential services and critical staff in the government will be on duty today. 410. In the following question, some part of the sentence may have errors. Find out which part of the sentence has as error and select the appropriate option. If a sentence is free from error, select 'No Error'. We look forward to an (1)/ successful working relationship (2)/ in the future (3)/ No Error (4) (a) 1 (b) 2 (c) 3 (d) 4 SSC MTS 10-10-2017 (Shift-II) Ans. (a) : In the above sentence 'a' will be used instead of 'an', because article 'a' is used before the word consonant sound (successful), while article 'an' is used before vowel sound. Correct Sentence :We look forward to a successful working relationship in the future. 411. In the following question, some part of the sentence may have errors. Find out which part of the sentence has an error and select the appropriate option. If a sentence is free from error, select 'No Error'. A ethos of equality was one (1)/ of the major reasons for their (2) / belief in 'satguru'. (3)/ No Error (4) (a) 1 (b) 2 (c) 3 (d) 4 SSC MTS 11-10-2017 (Shift-III) Ans. (a) : In the above sentence, 'ethos of equality (collective noun) is being used as a collective noun and due to the pronunciation of ethos with vowel sound, Hence, Indefinite article 'an' will be used before it. So instead of 'A', 'An' will be used. Correct Sentence :An ethos of equality was one of the major reasons for their belief in 'satguru'. 412. Identify the segment in the sentence which contains the grammatical error. Patralekha finished the cake in the blink of a eye. (a) of a eye (b) in the blink (c) Patralekha finished (d) the cake SSC MTS – 22/08/2019 (Shift-III) Ans. (a) : 'Of an eye' would be appropriate in place of 'of a eye', in the above sentence. Because the pronunciation of eye is being used as vowel sound. Therefore article 'an' will be used before 'eye'. Correct Sentece :Patralekha finished the cake in the blink of an eye.

62

YCT

413. Identify the segment in the sentence which contains the grammatical error. Nisha rejected the job offer as a salary offered was too low (a) as a salary offered (b) Nisha rejected (c) was too low (d) the job offer SSC MTS – 20/08/2019 (Shift-III) Ans. (a) : In the above sentence, as the salary offered' will be used instead of 'as a salary offered'. Because in the sentence, the use of salary would be appropriate in definite sense. Correct Sentence :''Nisha rejected the job offer as the salary offered was too low''. 414. Identify the segment in the sentence which contains the grammatical error. Due to a last minute snag, the mission on a moon had to be cancelled. (a) on a moon (b) Due to a last minute snag (c) the mission (d) had to be cancelled SSC MTS – 16/08/2019 (Shift-II) Ans. (a) : In the above sentence, instead of 'on a moon', 'On the Moon' would be appropriate. Because definite article 'the' is used before celestial object of the world like – Sun, Moon, Earth, Planet, North pole, South Pole equator etc. Correct sentence :Due to a last minute snag, the mission on the moon had to be cancelled. 415. Identify the segment in the sentence which contains the grammatical error from the given options. Outside the cinema hall, there was queue of people waiting to see the movie. (a) there was queue of (b) people waiting (c) to see the movie (d) Outside the cinema hall SSC MTS – 13/08/2019 (Shift-II) Ans. (a) : In the above sentence, Article 'a' will be used before queue. Because article 'a/an' is used before singular countable noun. Correct sentence :Outside the Cinema hall, there was a queue of people waiting to see the movie. 416. Identify the segment in the sentence which contains the grammatical error from the given options. They visited the Qutub Minar, which is most attractive spot in Delhi. (a) Attractive spot (b) which is most (c) they visited (d) in Delhi SSC GD – 11/02/2019 (Shift-III) Ans. (b) : 'The most' would be appropriate in place of 'most' in the above sentence, because article 'the' is used before superlative degree. much − more − most Correct Sentence : They visited the Qutub Minar, which is the most attractive spot in Delhi. Spotting Error

G.

Correct use of Pronoun, Adjective and Adverb

417. Find the part of the given sentence that has an error in it. If there is no error, choose ‘No error’. Can you please pass me that glittery pen black? (a) pass me that (b) glittery pen black? (c) No error (d) Can you please SSC MTS 15/05/2023 (Shift-I) Ans. (b) : In the above sentence part (b) 'glittery pen black ?' is grammatically incorrect Because the part show irregular sequence of adjective and noun, the correct sequence is 'adjective + noun' (pen). The correct sentence– can you please pass me glittery black pen. 418. The following sentences has been split into four segments. Identify the segment that contains a grammatical error. It can / get extreme cold / during / the wniters. (a) get extreme cold (b) it can (c) the winters (d) during SSC CGL (Tier-I) 27/07/2023 (Shift-III) Ans. (a) : The segment of the above given sentence 'get extream cold'' is grammatically incorrect because the adverb (extremely) is used in the place of 'extreme'. It modifies the adjective (cold) / the correct. Sentence is- 'It can get extremely cold during the winters.' 419. The following sentence has been divided into parts. One of them may contain an error. Select the part that contains the error from the given options. If you don’t find any error, mark ‘No error’ as your answer. The food container / is not enough big to meet / the demands of the affected people. (a) The food container (b) the demands of the affected people. (c) No error (d) is not enough big to meet SSC CHSL (Tier-I) 09/08/2023 (Shift-III) Ans. (d) : In the above sentence "is not enough big to meet" need to be replaced with "is not big enough to meet". Because Adverb "enough" always comes after the adjective, while it can come before a noun. 'Big enough' means sufficient to Fulfill a need' demand or requirement". Correct Sentence The food container is not big enough to meet the demands of the affected people. 420. Select the sentence that has a grammatical error. (a) Don't talk so loudly. (b) Rama can bowl fastly. (c) Go back to your room. (d) Run quickly. SSC CHSL (Tier-I) 14/08/2023 (Shift-IV)

63

YCT

Ans. (b) : In the above sentence fastly will be replaced with fast (Adverb) and fast words as an adjective also. Fast does not require – 'ly' in the end to make an adverb. as it is an adverb itself. Correct Sentence– Rama can bowl fast. 421. Find the part of the given sentence that has an error in it. If there is no error, choose ‘No error’. What is the price of this fancy dress pink? (a) fancy dress pink? (b) No error (c) price of this (d) what is the SSC CHSL (Tier-I) 21/03/2023 (Shift-IV) Ans. (a) : In the above sentence 'Pink fancy dress' will be used in place of 'fancy dress Pink' if more than one Adjective is used before noun then its proper sequence will be size/shape/age/color/origin/material/ Purpose. Correct Sentence : What is the price of this Pink fancy dress. 422. Find the part of the given sentence that has an error in it. If there is no error, choose ‘No error’. Its impossible to conduct truly causal research on media consumption and suicide. (a) Its impossible to conduct (b) media consumption and suicide. (c) No error (d) truly causal research on SSC CHSL (Tier-I) 21/03/2023 (Shift-IV) Ans. (a) : In the above sentence It's (It is) will be used in place of Its (Possessive adjective) because 'sub + verb' is needed in the sentence and 'Its' is not used as Possessive Pronoun. Correct Sentence : It is impossible to conduct truly casual research on media consumption and Suicide. 423. Identify the segment in the sentence which contains a grammatical error. If there is no error, then select the option "No error". The credit for organizing/the successful event was/ given to both Neha and I. (a) the successful event was (b) given to both Neha and I. (c) The credit for organizing (d) No error SSC CHSL (Tier-I) 15/03/2023 (Shift-III) Ans. (b) : In the above sentence 'me' will be used in place of 'I' because objective case is used after verb/Preposition (given to). Hence objective case 'after' of Pronoun 'I' will be used. Correct sentenceThe credit for organising the successful event was given to both Neha and me. 424. Find the part of the given sentence that has an error in it. If there is no error, choose 'No error'. A key institution is the judiciary, especially the Supreme Court, with a widely remit and a wise legacy. Spotting Error

(a) (b) (c) (d)

a widely remit and a wise legacy especially the Supreme Court, with A key institution is the judiciary No error SSC CHSL (Tier-I) 15/03/2023 (Shift-I) Ans. (a) : In the above sentence 'a wide remission' will be used in place of 'a widely remit' because 'adjective + noun' will be used after Article 'A' while widely and remit is used as adverb and verb respectively. Correct SentenceA key institution is the judiciary especially the Supreme Court with a wide remission and a wise legacy.

425. The following sentence has been divided into four parts. One of them contains an error. Select the part that contains theerror from the given options. They were / married happily / before she / quit her job. ABCD (a) A (b) D (c) C (d) B SSC CGL Mains 26/10/2023 (Shift-I) Ans. (d) : In the above sentence 'happily married' will be used in place of married happily because adverb 'happily' qualify the verb married'. Correct Sentence The were happily married before she quit her job. 426. Find the part of the given sentence that has an error in it. If there is no error, choose 'No error'. India has turned 70 years older as a Republic. (a) 70 years older (b) as a Republic (c) No error (d) India has turned SSC CHSL (Tier-I) 09/02/2023 (Shift-II) Ans. (a) : In the above sentence 'old' will be used in place of "older" because according to the sense of the sentence here the need of Adjective (old) of Positive degree whereas "older" (comperative degree) is used with "than" to show comparision between Two objects. Correct SentenceIndia has turned 70 years old as a Republic. 427. Parts of the following sentence have been given as options. Select the option that contains an error. Everyone should respect those parents, teachers, and elders. (a) and elders. (b) respect those (c) parents, teachers (d) Everyone should SSC CGL Mains 26/10/2023 (Shift-I) Ans. (b) : In the above sentence 'respect their/his' will be used in place of 'respect those' because when everyone, everybody is subject of the sentence then possessive pronoun their/his is used. Correct Sentence Everyone should respect their/his parents, teachers and elders.

64

YCT

428. Parts of the following sentence have been given as options. Select the option that contains an error. This is the book whom I wanted to buy. (a) the book (b) whom I (c) wanted to buy. (d) This is SSC CGL Mains 26/10/2023 (Shift-I) Ans. (b) : In the above sentence 'which I' will be used in place of 'whom I' because Relative pronoun 'which' is used for non-living things. Note- Relative pronoun 'whom' is used for living thing in objective case. Correct Sentence This is the book which I wanted to buy. 429. Parts of the given sentence have been given as options. One of them may contain an error. Select the part that contains the error from the given options. If you don't find any error, mark 'No error' as your answer. In many places, restoring mangrove forests can be an extreme effective strategy (a) forests can be an (b) No error (c) In many places, restoring mangrove (d) extreme effective strategy SSC MTS-05/07/2022 Shift-II Ans. (d) : In the above sentence 'extremely effective' will be used in place of 'extreme effective' because 'effective' is an adjective, So Adverb form (extreamely) will be used to modify 'effective'. Correct Sentence – In many places, restoring mangrove forests can be an extremely effective strategy. 430. Parts of the following sentence have been given as options. One of them may contain an error. Select the part that contains the error from the given options. If you don't find any error, mark 'No error' as your answer. Educators must roll up their sleeves and do a more best job of teaching critical thinking. (a) Educators must roll up their sleeves (b) No error (c) and do a more best job (d) of teaching critical thinking SSC MTS-14/07/2022 Shift-III Ans. (c) : In the above sentence 'and do the best Job' will be used in place of 'and do a more best Job' because comparative degree (more) and superlative degree (best) are not used together. Note- Article the is used before the superlative degree. Correct Sentence – Educators must roll up their sleeves and do the best job of teaching critical thinking. 431. The following sentence has been split into four segments. Identify the segment that contains a grammatical error. The mountain / seems to steep / for us / to climb. Spotting Error

(a) seems to steep (b) for us (c) The mountain (d) to climb SSC Constable GD-18/11/2021 Shift-I Ans. (a) : In the above sentence 'Seems too steep' will be used in place of 'seems to steep' because (too + Adjective -----to) is used to give Negative sense. Which gives suitable sense for the sentence. Correct Sentence– The mountain seems too steep for us to climb. 432. Identify the segment in the sentence which contains a grammatical error. One must not boast about their success. (a) not boast (b) their success (c) about (d) One must SSC Stenographer (Grade C & D) 12.11.2021 Shift-I Ans. (b) : In the above sentence, 'One's success' will be used in place of 'their success' because the possessive pronoun of 'one' is 'one's'. Correct Sentence One must not boast about one's success. 433. Identify the segment in the sentence which contains a grammatical error. If there is no eror, select 'No error'. At one time birds and animals lived peaceful together. (a) peaceful together (b) birds and animals lived (c) No error (d) At one time SSC CGL (Tier-II) 08.08.2022 Shift-II Ans. (a) : 'Peacefully (adverb) will be used instead of peaceful (noun) in the above sentence. Because 'adverb' is used to modify 'lived (verb). Correct sentence– At one time birds and animals lived peacefully together. 434. Identify the segment that contains a grammatical error. A misunderstanding has crept between he and his sister. (a) and his sister (b) between he (c) has crept (d) A misunderstanding SSC CGL (Tier-II) 08.08.2022 Shift-II Ans. (b) : In the above sentence, 'him' will be used in place of 'he'. Because objective case (him) is used after preposition (between). While 'he' is used as subjective case. Correct sentence– A misunderstanding has crept between him and his sister. 435. Identify the segment in the sentence which contains a grammatical error. If there is no error, select 'No error'. If you are going downhill you can go much fast. (a) No error (b) If you are going downhill (c) much fast (d) you can go SSC CGL (Tier-II) 08.08.2022 Shift-II

65

YCT

Ans. (c) : 'Very' will be used instead of 'much' in the above sentence. Because adverb 'very' is used to modify the adverb 'fast'. Note : - Generally 'much' is used before adjective. Correct sentence– If you are going downhill you can go very fast. 436. Identify the segment in the sentence which contains a grammatical error. If there is no error, select 'No error'. You should tell these children to complete their projects himself. (a) You should tell (b) No error (c) these children to complete (d) their projects himself. SSC CGL (Tier-II) 08.08.2022 Shift-II Ans. (d) : 'Themselves' will be used in place of himself in the above sentence. Because the subject (children) of the sentence is plural and plural possessive adjective 'their' is used for it. Therefore for it, plural reflexive pronoun, themselves will be used. Correct sentence– You should tell these children to complete their projects themselves. 437. Select the most appropriate option to substitute the underlined segment in the given sentence. If there is no need to substitute it, select 'No substitution'. Her coach allowed she swimming in deep water. (a) her swimming (b) No substitution (c) her to swim (d) she to swim SSC Stenographer (Grade C & D) 11.11.2021 Shift-II Ans. (c) : In the above sentence 'allowed' is a verb, after it. pronoun will be used in objective case. So to make correct sentence, 'her to swim' will be used in place of 'she swimming'. Hence, Option (c) is correct 438. The following sentence has been split into four segments. Identify the segment that contains a grammatical error. It is difficult to say/which of the/two girls in that house/is the eldest? (a) which of the (b) It is difficult to say (c) two girls in that house (d) is the eldest SSC Stenographer (Grade C & D) 11.11.2021 Shift-I Ans. (d) : In the above sentence, 'elder' will be used in place of 'eldest', because comparative degree is used for two person/things. Correct sentence It is difficult to say which of the two girls in that house is elder? 439. In the sentence identify the segment that contains a grammatical error. There are much advantages of working together in a group. Spotting Error

(a) much advantages (b) in a group (c) of working together (d) There are SSC Stenographer (Grade C & D) 12.11.2021 Shift-II Ans. (a) : In the above sentence 'Many advantages' will be used in place of 'much advantages'. (We use many to refer to a large number of something in countable sense.) Correct Sentence– There are many advantages of working together in a group. 440. In the given sentence, identify the segment that contains a grammatical error. Draw a lotus plant and label their parts. (a) and label (b) Draw a (c) lotus plant (d) their parts SSC Stenographer (Grade C & D) 12.11.2021 Shift-II Ans. (d) : In the above sentence, 'its' will be used instead of 'their', because possessive adjective 'its' is used for non-living things. 'It' is a determiner which shows belonging to a thing. Correct sentence–Draw a lotus plant and label its parts'. 441. In the given sentence, identify the segment that contains a grammatical error. A community is a group of people who shares a common culture. (a) A community is (b) a group of people (c) common culture (d) who shares a SSC Stenographer (Grade C & D) 15.11.2021 Shift-II Ans. (d) : In the above sentence, 'that' would be appropriate instead of 'who' because 'group' is a nonliving and pronoun 'that' is used for nonliving thing. Correct Sentence:A community is a group of people that shares a common culture. 442. Identify the segment in the sentence which contains a grammatical error. I will come to yours house in the evening to play chess. (a) to yours house (b) in the evening (c) to play chess (d) I will come SSC Stenographer (Grade C & D) 15.11.2021 Shift-II Ans. (a) : In the above sentence, 'to your house' will be used in place of 'to yours house' because 'house' is a noun and adjective is always used to qualify noun. Your → Possessive adjective Yours → Possessive Pronoun

443. In the given sentence, identify the segment that contains a grammatical order. There is a few water in the bowl on the table. (a) There is (b) a few water (c) in the bowl (d) on the table SSC Stenographer (Grade C & D) 15.11.2021 Shift-I Ans. (b) : 'Few/a few/the few' is used before countable noun, while 'little/a little/the little' is used before uncountable noun and 'water' is an uncountable noun so before it, 'a little' will be used. Hence option (b) is correct answer.

66

YCT

444. The following sentence has been split into four segments. Identify the segment that contains a grammatical error. The birds flew/fastly above/as we stood/on outterrace. (a) The birds flew (b) On our terrace (c) Fastly above (d) As we stood SSC CHSL 27.05.2022 Shift-II Ans. (c) : In the above sentence, 'Fast' will be used in place of 'fastly', because adverb 'fast' is used to modify the verb 'flew' while 'fastly' is a incorrect word. Correct SentenceThe birds flew fast above as we stood on our terrace. 445. The following sentence has been split into four segments. Identify the segment that contains a grammatical error. I am not sure/if the red car/ in the parking slot/is your. (a) if the red car (b) in the parking slot (c) is your (d) I am not sure SSC CHSL 27.05.2022 Shift-I Ans. (c) : In the above sentence, 'yours' will be used instead of 'your'. According to the sense of the sentence, possessive pronoun (yours) is required at the end of the sentence, not possessive adjective (your). Correct Sentence I am not sure if the red car in the parking slot is yours. 446. Parts of the following sentence have been given as options. One of them may contain an error. Select the part that contains the error from the given options. if you don't find any error, mark 'No error' as your answer. He sat down in a darkly corner and covered his face, and would not say a single word to anyone. (a) No error (b) and covered his face, and would (c) He sat down in a darkly corner (d) not say a single word to anyone SSC MTS-18/07/2022 Shift-I Ans. (c) : In the above sentence, 'he sat' down in a dark corner' would be appropriate instead of 'He sat down in a darkly corner,' because 'corner' is a noun, hence before it, adjective 'Dark' will be used, not adverb (darkly). Correct Sentence – He sat down in a dark corner and covered his face, and would not say a single word to anyone 447. The following sentence has been split into four segments. identify the segment that contains a grammatical error. I find that / I get myself tired / very easily / these days. (a) these days (b) I get myself tired (c) I find that (d) very easily SSC CGL (Tier-I) 21/04/2022 Shift-III Spotting Error

Ans. (b) : 'I get tired' will be used instead of 'I get myself tired, in the above sentence, because in the sentence, the use of Reflexive pronoun (myself) is superfluous. Correct sentence → I find that I get tired very easily these days. 448. The following sentence has been divided into parts. One of them may contain a grammatical error. Select the part that contains the error from the given options. if you don't find any error, mark 'No error' as your answer. They had hardly completed / half of the work / than the boss called them. (a) than the boss called them (b) They had hardly completed (c) No error (d) half of the work SSC CGL (Tier-I) 20/04/2022 Shift-III Ans. (a) : In the given sentence, the use of 'when' would be appropriate in the place of 'than' because 'When' is used with hardly/ scarcely.' Hence 'when' will be used in place of 'than'. Scarcely/Hardly............when Correct SentenceThey had hardly completed half of the work when the boss called them. 449. The following sentence has been split into four segments. Identify the segment that contains a grammatical error. My friend / who's leg / was fractured, / has recovered. (a) who's leg (b) was fractured (c) My friend (d) has recovered SSC CGL (Tier-I) 20/04/2022 Shift-III Ans. (a) : In the given sentence, 'Whose leg' will be used instead of 'who's leg' because 'who' is used as subjective case while 'whose' is used as possessive case 'as an adjective'. Correct sentence– My friend whose leg was fractured has recovered. 450. The following sentence has been split into four segments. Identify the segment that contains a grammatical error. If he wants/ farther information,/ send him/ to me. (a) to me (b) send him (c) If he wants (d) farther information SSC CGL (Tier-I) 13/04/2022 Shift-I Ans. (d) : In the given sentence, the segment 'farther information' is grammatically incorrect. Here 'farther' means 'at a distance' 'further' will be used which means 'next'. Correct sentenceIf he wants further information, send him to me.

67

YCT

451. The following sentence has been divided into parts. One of them contains an error. Select the part the contains the error from the given option. This question / is quite too / simple for me / to answer. (a) simple for me (b) is quite too (c) This question (d) to answer SSC CGL (Tier-I) 13/04/2022 Shift-I Ans. (b) : In the given sentence, the part 'is quite too' contains an error, because 'quite' and 'too' means the same. so 'too' is superfluous in the sentence. Correct sentence– 'This question is quite simple for me to answer.' 452. The following sentence has been split into four segments. Identify the segment that contains a grammatical error. The recently hike/in the price of petroleum products/will hit/household budgets severely. (a) The recently hike (b) household budgets severely (c) will hit (d) in the price of petroleum products SSC CGL (Tier-I) 11/04/2022 Shift-III Ans. (a) : In the above sentence, 'The recent hike' will be used in place of 'The recently hike' because 'hike' is a noun and adjective 'recent' will be used before noun, not adverb (Recently). 453. The following sentence has been split into four segments. Identify the segment that contains a grammatical error. My older brother,/ which you’ll / meet later,/ is a dentist. (a) which you’ll (b) My older brother (c) is a dentist (d) meet later SSC CGL (Tier-I) 12/04/2022 Shift-I Ans. (a) : In the above sentence, 'Whom' would be appropriate instead of 'which' because the relative pronoun 'whom' (objective case) is used for person. While 'which' is used for animal/thing. Correct Sentence My older brother whom you'll meet later, is a dentist. 454. The following sentence has been split into four segments. Identify the segment that contains a grammatical error. My house / is more / spacious than / my sister. (a) spacious than (b) My house (c) is more (d) my sister SSC CGL (Tier-I) 12/04/2022 Shift-I Ans. (d) : Instead of 'my sister' 'that of my sister will be used in the above sentence, because 'house' is being compared in the sentence and comparison is made between the same noun. Note- "Than my sister's house" can also be used. Correct Sentence– My house is more spacious than that of my sister. Spotting Error

455.

Parts of the given sentence have been given as options. One of them may contain an error. Select the part that contains the error from the given options. If you don't find any error, mark 'No error' as your answer. It feels greatly to be with you at this ceremony. (a) at this ceremony (b) to be with you (c) It feels greatly (d) No error SSC MTS-15/07/2022 Shift-I Ans. (c) : 'It greatly feels' will be used in place of 'It feels greatly' in the above sentence, because 'greatly' (adverb) is qualifying 'feels (verb) will be used before it. Correct Sentence – It greatly feels to be with you at this ceremony. 456. The following sentence has been divided into parts. One of them may contain an error. Select the part that contains the error from the given options. If you don't find any error, mark 'No error' as your answer. Solar storms can / impact Earth-dwellers ' in significantly ways. (a) Solar storms can (b) impact Earth-dwellers (c) No error (d) in significantly ways SSC MTS-06/07/2022 Shift-I Ans. (d) : In the above sentence, 'significant' will be used in place of 'significantly (adv)'. Because significant (adjective) is used to qualify 'the ways' (noun). Correct Sentence – Solar storms can impact Earth-dwellers in significant ways. 457. The following sentence has been split into four segments. Identify the segment that contains a grammatical error. He had begun to dig / diligent along the / low tideline and had / captured two clams. (a) captured two clams (b) low tideline and had (c) diligent along the (d) He had begun to dig SSC MTS-07/07/2022 Shift-I Ans. (c) : In the above sentence, 'diligently along the' will be used in place of 'diligent along the' because adverb 'diligently' will be used to modify the verb 'begun'. while 'diligent' is an adjective. Correct Sentence – He had begun to dig diligently along the low tideline and had captured two clams. 458. Select the most appropriate option to substitute the underlined segment in the given sentence. If there is no need to substitute it, select 'No substitution required'. The playground was way large then I had imagined, and in cleaning it, my back began to ache.

68

YCT

(a) (b) (c) (d)

larger than I had No substitution required largest to what I have large then I have SSC MTS-07/07/2022 Shift-I Ans. (a) : The underlined part of the above sentence, 'larger than I had' would be appropriate because the sense of comparison is being displayed and in the comparative sense 'than' is used for comparison. Correct Sentence – The Playground was way larger than I had imagined, and in cleaning it, my back began to ache. 459. The following sentence has been divided into parts. One of them may contain an error. Selec the part that contains the error from the given options. If your don't find any error, mark 'No error' as your answer. A man tends to one / of the horses after it's / training at a stud farm. (a) of the horses after it's (b) A man tends to one (c) No error (d) training at a stud farm SSC MTS-06/07/2022 Shift-II Ans. (a) : In the above sentence, 'of the horses after its, will be used instead of 'of the horses after it's', because in sentence, according to horses, possessive adjective its' will be used while 'it's' is the short form of 'it is '. because possessive adjective is used before noun (training). Correct Sentence – A man tends to one of the horses after its training at a stud farm. 460. Parts of the following sentence have been given as options. One of them may contain an error. Select the part that contains the error from the given options. If you don't find any error, mark 'No error' as your answer. When you are under significant stress, it is hardly to process details and nuances. (a) significant stress, it is hardly (b) to process details and nuances (c) When you are under (d) No error SSC MTS-05/07/2022 Shift-I Ans. (a) : In the above sentence, 'Hard' is used in place of Hardly because according to the sense of the sentence, it gives suitable meaning. Hence adjective (Hard) will be used after 'it is'. Correct Sentence – When you are under significant stress, it is hard to process details and nuances. 461. The following sentence has been divided into parts. One of them may contain an error. Select the part that contains the error from the given options. If you don't find any error, mark 'No error' as your answer. The mountain lion is / one of the slyest and more / elusive animals in the woods.

Spotting Error

one of the slyest and more No error The mountain lion is elusive animals in the woods. SSC MTS-13/07/2022 Shift-I Ans. (a) : In the above sentence, 'One of the slyest and most' will be used in place of one of the slyest and more, because same part of speech, is added with 'And' 'the slyest is superlative degree so superlation degree (most) will be used after 'and'. Correct Sentence– The mountain lion is one of the slyest and most elusive animal in the woods. 462. The following sentence has been split into four segments. Identify the segment that contains a grammatical error. The brightest stars/can be 30,000 times/most luminous / than the Sun (a) The brightest stars (b) most luminous (c) can be 30,000 times (d) than the Sun SSC MTS-08/07/2022 Shift-I Ans. (b) : In the above sentence, 'more luminous' will be used in place of 'most luminous' because the use of 'than' is showing the sense of comparison so comparative degree 'more' will be used. Correct Sentence – The brightest stars can be 30,000 times more Luminous than the sun. 463. The following sentence has been divided into parts. One of them may contain an error. Select the part that contains the error from the given options. If you don’t find any error, mark ‘No error’ as your answer. May I / please borrow / yours book? (a) May I (b) yours book (c) No error (d) please borrow SSC Constable GD-30/11/2021 Shift-III Ans. (b) : In the above sentence, your book' will be used in place of 'yours book'. Because possessive adjective (your) is used before noun (book). While 'Yours' is possessive pronoun. Correct SentenceMay I please borrow your book ? 464. The following sentence has been split into four segments. Identify the segment that contains a grammatical error. Laughing for about ten minutes / continuous each day / is supposed to be / good for your health. (a) good for your health (b) is supposed to be (c) continuous each day (d) Laughing for about ten minutes SSC Constable GD-26/11/2021 Shift-I Ans. (c) : 'Continuously' (adverb) will be used in place of continuous (adjective) in the above sentence. Because an adverb is needed to qualify the whole clause.

69

(a) (b) (c) (d)

YCT

Note– Adverb is used to qualify-verb, adverb, adjective preposition and sometimes the whole clauses. Correct sentence– Laughing for about ten minutes continuously each day is supposed to be good for your health. 465. The following sentence has been split into four segments. Identify the segment that contains a grammatical error. Philip was / too proud / for extending / a hand of friendship. (a) too proud (b) a hand of friendship (c) Philip was (d) for extending SSC Constable GD-25/11/2021 Shift-III Ans. (d) : In the above sentence, 'to extend' will be used instead of 'for extending' because 'too ---- to' is used in the sense of 'so much that' and after 'to' first form of verb is used. Correct Sentence – Philip was too proud to extend a hand of friendship. 466. Parts of the given sentence have been given as options. One of them contains a grammatical error. Select the option that has the error. Pray to the Almighty God with utmost devotion and help the needy and poored people. (a) with utmost devotion (b) help the needy (c) poored people (d) to the Almighty God SSC Constable GD-16/11/2021 Shift-III Ans. (c) : 'Poor people' will be used in place of 'Poored people' in the above sentence. Because in the given sentence, 'people' is a noun. Hence, adjective (Poor) will be used before it. Correct Sentence Pray to the Almighty God with utmost devotion and help the needy and poor people. 467. The following sentence has been split into four segments. Identify the segment that contains a grammatical error. This apartment / is / nice than / that one. (a) that one (b) is (c) nice than (d) This apartment SSC Constable GD-14/12/2021 Shift-II Ans. (c) : In the above sentence, 'Nicer than' will be used instead of 'Nice than', because 'than' is used in the sense of comparison. So comparative degree (Nicer) would be appropriate. Correct Sentence - This apartment is nicer than that one. 468. The following sentence has been split into four segments. identify the segment that contains a grammatical error. The children played / merry in the park / while their parents / kept a watch. (a) while their parents (b) merry in the park (c) The children played (d) kept a watch SSC Constable GD-10/12/2021 Shift-III Spotting Error

Ans. (b) : 'Merrily' will be used in place of 'merry' in the above sentence, because according to the sense of sentence, 'adverb' (merrily) is needed here to modify verb (played). Hence adverb (merrily) will be used. Correct sentence:– The children played merrily in the park while their parents kept a watch. 469. The following sentence has been divided into parts. One of them may contain an error. Select the part that contains the error from the given options. If you don't find any error, select 'No error' as your answer. However hardly you / may work, / nobody will praise you. (a) nobody will praise you (b) No error (c) may work (d) However hardly you SSC Constable GD-10/12/2021 Shift-III Ans. (d) : 'Hard' will be used instead of 'Hardly' in the above sentence, because 'hardly' is a negative word which is not suitable according to the sense of sentence. Hence, adverb 'hard' will be used. Correct sentence:– However hard you may work, nobody will praise you. 470. The following sentence has been divided into parts. One of them may contain an error. Select the part that contains the error from the given options. If you don't find any error, select 'No error' as your answer. This newspaper has/a bigger circulation/from any other newspaper. (a) a bigger circulation (b) from any other newspaper (c) this newspaper has (d) No error SSC Constable GD-06/12/2021 Shift-III Ans. (b) : In the above sentence, 'than' would be appropriate in place of 'from'. Because 'than' is used with comparative degree. Correct sentence – This newspaper has a bigger circulation than any other newspaper. 471. The following sentence has been split into four segments. Identify the segment which contains a grammatical error. Deepu's mother / has repeated asked him/ to lower the volume / of his mobile. (a) has repeated asked him (b) to lower the volume (c) of his mobile (d) Deepu's mother SSC CHSL (Tier-I) –04/08/2021 (Shift-I) Ans. (a) : 'Repeatedly' will be used in place of has 'repeated' in the above sentence, because according to the use of asked (verb). an adverb would be appropriate to modify it. Correct Sentence Deepu's mother repeatedly asked him to lower the volume of his mobile.

70

YCT

472. In the given sentence, identify the segment which contains the grammatical error. The government has announced a gradual reboot of air travel who was halted two months ago due to lockdown. (a) who was halted (b) due to lockdown (c) The government has announced (d) reboot of air travel SSC CHSL (Tier-I) –16/04/2021 (Shift-I) Ans. (a) : In the above sentence, 'that was halted' would be appropriate in place of 'Who was halted'. Because 'who' is used for living thing while 'that' is used for non livings as a relative pronoun. Correct SentenceThe government has announced a gradual reboot of air travel that was halted two months ago due to lockdown. 473. The following sentence has been split into four segments. Identify the segment that contains a grammatical error. After a long gap,/we went shopping/last sunday and bought/ that we wanted. (a) that we wanted (b) last sunday and bought (c) we went shopping (d) after a long gap SSC CHSL (Tier-I) –19/04/2021 (Shift-I) Ans. (a) : In the above sentence, 'which we wanted' will be used instead of 'that we wanted', because 'which' is used for nonliving things as a relative pronoun. Correct Sentence After a long gap we went shopping last sunday and bought which we wanted. 474. The following sentence has been divided into parts. One of them contains an error. Select the part that contains the error the given options. The food/in the new eatery/is much best than/the one in RK Puram. (a) is much best than (b) in the new eatery (c) the one in RK Puram (d) The food SSC CGL (Tier-I) –24/08/2021 (Shift-I) Ans. (a) : 'Is much better than' will be used instead of 'is much best than' in the above sentence, because comparative degree (better) is used before than. Correct Sentence The food in the new eatery is much better than the one in RK Puram. 475. The following sentence has been divided into parts. One of them contains an error. Select the part that contains the error from the given options. The army/marched fearless/towards/the enemy cantonment. (a) marched fearless (b) towards (c) the enemy cantonment (d) the army SSC CGL (Tier-I) –23/08/2021 (Shift-I) Spotting Error

Ans. (a) : In the above sentence, instead of fearless (adjective), fearlessly (adverb) will be used because adverb(fearlessly) is used to modify the verb (marched). Correct SentenceThe army marched fearlessly towards the enemy cantonment. 476. Identify the segment in the sentence which contains a grammatical error. The small cafe at the end of the road is her. (a) of the road (b) is her (c) at the end (d) The small cafe SSC CGL (Tier-II) 16/11/2020 (3 pm - 5 pm) Ans. (b) : 'Hers' will be used in place of 'her' in the above sentence, because possessive pronoun would be appropriate to show the possession. While 'her' is a possessive adjectives. Correct sentenceThe small cafe at the end of the road is hers. 477. Identify the segment in the sentence which contains a grammatical error. She has been a member of this club since it's formation. (a) this club (b) a member of (c) since it's formation (d) She has been SSC CGL (Tier-II) 16/11/2020 (3 pm - 5 pm) Ans. (c) : 'Its' will be used in place of 'it's' in the above sentence, because 'its' is used as possessive adjective and after it noun (formation) is used. Correct sentence- She has been a member of this club since its formation. 478. Identify the segment in the sentence which contains a grammatical error. The three of them shouted out to each another when the train started moving. (a) The three of them (b) shouted out to each another (c) when the train (d) started moving SSC CGL (Tier-II) 16/11/2020 (3 pm - 5 pm) Ans. (b) : 'One another' will be used in place of' each another' in the above sentence. Because 'each other' is used for two people while 'one another' is used for more than two. Correct sentence- The three of them shouted out to one another when the train started moving. 479. Identify the segment in the sentence, which contains the grammatical error. Nikhil is more braver than Sushant who is afraid of the dark. (a) more braver than (b) Nikhil is (c) who is afraid (d) of the dark SSC CPO-SI (Tier-II) 27/09/2019 (3 pm - 5 pm) Ans. (a) : In the above sentence, 'braver' will be used in place of 'more braver' because more and braver both are comparative degree and double comparative is not used together. Correct sentence- Nikhil is braver than Sushant who is afraid of the dark.

71

YCT

480. Identify the segment in the sentence, which 485. Identify the segment in the sentence which contains the grammatical error. contains a grammatical error. No matter he tries hard he cannot play the Nobody has ever listened to me so attentive. guitar. (a) ever listened (b) so attentive (a) No matter (b) he tries hard (c) to me (d) nobody me (c) he cannot (d) play the guitar SSC CPO-SI – 23/11/2020 (Shift-II) SSC CPO-SI (Tier-II) 27/09/2019 (3 pm - 5 pm) Ans. (b) : In the above sentence, 'so attentively' would Ans. (b) : In the above sentence, 'how' will be used be appropriate instead of 'so attentive'. because adverb after ' No matter' because no matter is followed by 'attentively' would be appropriate for verb 'listened' not verb/adverb. adjective 'attentive'. Correct sentence- No matter how hard he tries, he cannot play the guitar. Correct Sentence : 481. Identify the segment in the sentence which Nobody has ever listened to me so attentively. contains the grammatical error. 486. In the sentence identify the segment which My sister and myself are pleased to accept your contains the grammatical error. invitation to dinner. There isn't many rice left in the house so we (a) My sister and myself must replenish our stock soon. (b) are pleased (a) left in the house (c) to accept (b) our stock soon (d) your invitation to dinner (c) so we must replenish SSC Stenographer Grade C&D –05/02/2019 (3:5pm) (d) There isn't many rice Ans. (a) : In the above sentence, 'I' will be used in place of 'myself', because Reflexive pronoun (myself) SSC CGL (Tier-I) – 11/06/2019 (Shift-I) is not used as a subject. Ans. (d) : 'There isn't much rice' would be appropriate Correct sentence- My sister and I are plaeased to in place of 'There isn't many rice' in the above accept your invitation to dinner. sentence, because 'rice' is a singular uncountable noun 482. Identify the segment in the sentence which for this 'much' is used while 'many' is used with contains the grammatical error. The cheetah can run more faster than any uncountable noun. example- much rice, much milk, much bread other animal. (uncountable noun) (a) any other animal (b) more faster many students, many pens, many cats (countable noun) (c) than (d) can run SSC Stenographer Grade C&D –05/02/2019 (3:5pm) Correct Sentence : Ans. (b) : 'More and faster' will not be used together, There isn't much rice left in the house so we must because both are comparative and double comparative replenish our stock soon. is not used together. 487. In the sentence identify the segment which Correct sentence- The cheetah can run faster than any contains the grammatical error. other animal. He is more smarter than his brother though he 483. Identify the segment in the sentence which does not earn much money. contains a grammatical error. (a) though he does not earn I had to hold on to the wall to steady itself or I (b) much money could have fallen. (c) He is more smarter (a) to the wall (b) I had to hold on (d) than his brother (c) to steady itself (d) could have fallen SSC CPO-SI – 24/11/2020 (Shift-I) SSC CGL (Tier-I) – 11/06/2019 (Shift-III) Ans. (c) : In the above sentence, 'to steady myself' will Ans. (c) : In the above sentence, 'He is smarter will be be used in place of 'to steady itself'. Because the used instead of 'he is more smarter', because double reflexive pronoun of the subject 'I' is 'myself'. comparative degree can not be used together. Correct Sentence : Correct Sentence : I had to hold on to the wall to steady myself or I could He is smarter than his brother though he does not earn have fallen. much money. 484. Identify the segment in the sentence which 488. In the sentence identify the segment which contains a grammatical error. The information you are asking for is quite contains the grammatical error. irrelevance. I know the cobbler which mends the shoes (a) is quite irrelevance (b) you are outside the gate of our colony. (c) asking for (d) The information (a) I know the cobbler (b) the shoes SSC CPO-SI – 25/11/2020 (Shift-II) (c) which mends (d) outside the gate Ans. (a) : 'Is quite irrelevant' will be used in place of SSC CGL (Tier-I) – 06/06/2019 (Shift-II) 'is quite irrelevance' in the above sentence, because an adjective is required to modify the information (noun) Ans : (c) 'Who mends' would be appropriate in place of in which 'irrelevant' will be used. While 'Irrelevance' is 'which mends' in the above sentence. Because in the given sentence, the use of 'which' is wrong for cobbler. used as a noun. because Cobbler is a living thing for which relative Correct sentence :pronoun 'who' is used. The information you are asking for is quite irrelevant. Spotting Error

72

YCT

'Who' is used for living thing while 'which' is used for non-living thing and 'that' is used for both (living and non-living) Correct Sentence : I know the cobbler who mends the shoes outside the gate of our colony. 489. In the sentence identify the segment which contains the grammatical error. With food granaries overflowing, the Government is looking to liquidate his grain stock to prevent damage. (a) the Government is looking (b) to liquidate his grain stock (c) With food granaries overflowing (d) to prevent damage SSC CPO-SI – 11/12/2019 (Shift-I) Ans : (b) In the above sentence, 'its' will be used in place of 'His'. Because possessive adjective 'its' is used for the government' (non-living thing). Correct SentenceWith food granaries overflowing, the Government is looking to liquidate its grain stock to prevent damage. 490. In the sentence identify the segment which contains the grammatical error. If the sentence has no error, then select 'No error'. How many money did Sudhir win in the lottery? (a) How many money (b) No error (c) in the lottery (d) did Sudhir win SSC CPO-SI – 12/12/2019 (Shift-II) Ans : (a) In the above sentence, 'how much money' would be appropriate in place of 'How many money'. Because 'money' is an uncountable noun. 'Much' is used before uncountable noun. Correct Sentence : How much money did Sudhir win in the lottery? 491. In the sentence identify the segment which contains the grammatical error. Vini bought two new dresses but either is suitable for the festival. (a) but either is (b) Vini bought (c) two new dresses (d) suitable for the festival SSC CPO-SI – 12/12/2019 (Shift-II) Ans : (a) 'But neither is' will be used instead of 'but either is' in the above sentence. Because the use of 'neither' would be appropriate for negative sense. Neither- not one nor the other of two things or people. Correct Sentence : Vini bought two new dresses but neither is suitable for the festival. 492. In the sentence identify the segment which contains the grammatical error. They informed the teacher that all of they family members would be going to Visakhapatnam. (a) would be going (b) all of they family members (c) to Visakhapatnam (d) They informed the teacher that SSC CPO-SI – 12/12/2019 (Shift-II)

Spotting Error

Ans : (b) 'All of their family members; would be appropriate instead of 'all of they family members' in the above sentence. Because the possessive adjective of 'they' is 'their' which qualify the family members. Correct Sentence : They informed the teacher that all of their family members would be going to Visakhapatnam. 493. Identify the segment in the sentence which contains the grammatical error. The team began searching for reasons for their poor performance in the tournament. (a) The team began (b) in the tournament (c) searching for reasons (d) for their poor performance SSC CHSL (Tier-I) –11/07/2019 (Shift-I) Ans : (d) In the above sentence, 'for its poor performance' will be used instead of 'for their poor performance, because 'team' is a collective noun. Which is used in singular. So with 'team' third person singular pronoun (its) will be used. Whereas their is used for plural noun (they). Correct Sentence : The team began searching for reasons for its poor performance in the tournament. 494. Identify the segment in the sentence which contains the grammatical error. Deepak said he would pack few things he had and vacate the hostel room the next day. (a) the next day (b) Deepak said he would pack (c) pack few things he had (d) and vacate the hostel room SSC CHSL (Tier-I) –10/07/2019 (Shift-II) Ans : (c) 'The few' will be used in place of 'few' in the above sentence. Few – not many (in the sense of negative) A few – some (in the sense of positive) The few – some (whatever is but all) Correct Sentence :Deepak said he would pack the few things he had and vacate the hostel from the next day. 495. Identify the segment in the sentence, which contains the grammatical error. Very little metals are as precious as gold. (a) as precious (b) metals are (c) as gold (d) very little SSC CHSL (Tier-I) –10/07/2019 (Shift-III) Ans : (d) In the above sentence, 'few will be used in place of 'little' because 'few' is used for countable things. While 'little' is used for countable things. Correct sentence :Very few metals are as precious as gold. 496. In the sentence identify the segment which contains the grammatical error. Billionaire Philip is the owner of the luxury yacht Lionheart, which worth an estimated $150 million. (a) which worth (b) the owner of (c) Billionaire Philip is (d) an estimated $150 million SSC CHSL (Tier-I) –02/07/2019 (Shift-II)

73

YCT

Ans : (a) In the above sentence, 'Whose worth' will be used in place of 'Which worth'. Because the subject of the sentence is alive and relative pronoun who/whose is used for living things. Correct sentence :Billionaire Philip is the owner of the luxury yacht Lionheart, whose worth an estimated $150 million. 497. In the given sentence, identify the segment which contains a grammatical error. We can see much changes in Government schools these days. (a) in Government schools (b) We can see (c) much changes (d) these days SSC CHSL (Tier-I) 16/10/2020 (Shift-III) Ans. (c) : In the above sentence 'Many changes' will be used in place of 'much changes' beacuse 'Many' will be used with countable noun. (change) Correct sentence :We can see many changes in Government schools these days. 498. In the sentence identify the segment which contains the grammatical error. You must make a choice immediately because there is few time left. (a) because there is (b) a choice immediately (c) You must make (d) few time left SSC CHSL (Tier-I) 21/10/2020 (Shift-II) Ans. (d) : In the above sentence, 'little time' will be used instead of 'few time', because 'few' is used with countable noun, while 'little' is used with uncountable. (time) Correct Sentence You must make a choice immediately because there is little time left. 499. In the sentence identify the segment which contains a grammatical error. Queen elizabeth got marrying to Philip, former Prince of Greece and Denmark. (a) marrying to Philip (b) Greece and Denmark (c) Queen Elizabeth got (d) former Prince of SSC CHSL (Tier-I) 26/10/2020 (Shift-I) Ans. (a) : In the place of 'marrying', 'married' will be used in the above sentence, because in any sentence, either noun or adjective is used after verb. Hence, Adjective' married' will be used after got. Correct Sentence :Queen elizabeth got married to Philip, former Prince of Greece and Denmark. 500. In the given sentence identify the segment which contains the grammatical error. The park was well lighted at night but it was deserted, so it was a bit scared. (a) but it was deserted (b) The park was well lighted (c) so it was a bit scared (d) at night SSC CHSL (Tier-I) 15/10/2020 (Shift-III) Spotting Error

Ans. (c) : In the above sentence, 'so it was a bit scary' will be used in place of 'so it was a bit scared', because adjective scary (something or someone that causes fear of terror. Which gives proper meaning for the sentence. Correct Sentence :The park was well lighted at night but it was deserted, so it was a bit scary. 501. In the given sentence identify the segment which contains a grammatical error. There are some members who haven't paid his subscription yet. (a) who haven't paid (b) his subscription yet (c) some members (d) There are SSC CHSL (Tier-I) 19/10/2020 (Shift-III) Ans. (b) : In the above sentence, their subscription yet' will be used in place of 'his subscription yet' because the subject of this sentence (some members) is plural. Hence, plural possessive adjective (their) will be used for it. Correct Sentence :There are some members who haven't paid their subscription yet. 502. In the given sentence, identify the segment which contains a grammatical error. All these boys have come to college without few knowledge of English. (a) All these boys (b) without few knowledge (c) have come to college (d) of English SSC CHSL (Tier-I) 19/10/2020 (Shift-II) Ans. (b) : In the above sentence 'without little knowledge' will be used in place of without few knowledge. Rule– 'few' is used as a countable noun and 'little' is used uncountable noun while knowledge is uncountable noun. Hence, before 'knowledge' 'little' would be appropriate. Correct Sentence :All these boys have come to college without little knowledge of English. 503. In the sentence identify the segment which contains the grammatical error. Cats cannot be blamed for its behaviour to their masters as they are tamed so. (a) to their masters (b) Cats cannot be blamed (c) for its behaviour (d) as they are tamed so SSC CHSL (Tier-I) 19/03/2020 (Shift-II) Ans. (c) : In the above sentence 'For their behaviour' would be appropriate instead of 'for its behaviour'. Because 'its' is used for non-living things. While the subject of sentence is 'cats' (living thing). Hence possessive adjective (their) would be appropriate for it. Correct Sentence :Cats cannot be blamed for their behaviour to their master as they are tamed so. 504. In the given sentence, identify the segment which contains the grammatical error. The performance of our players is worst than I had expected.

74

YCT

(a) The performance (b) our players Ans. (a) : In the above sentence 'Fast' will be used (c) I had (d) worst than instead of 'fastly' because 'fast' is an adverb itself, SSC CHSL (Tier-I) 14/10/2020 (Shift-II) which modify the verb (ran). Ans. (d) : In the above sentence 'worse than' will be Correct sentence : used instead of 'worst than', because comparative She ran so fast that I could not overtake her. degree is used before 'than'. 509. Find the part of the given sentence that has an Bad → Worse → Worst error in it. If there is no error, choose 'No error'. ( positive ) ( comparative ) ( superlative ) My brother brought a few rice while coming Correct Sentence :from the market./No error The performance of our players is worse than I had (a) No error expected. (b) My brother brought 505. Identify the segment in the sentence, which (c) a few rice while contains the grammatical error. (d) coming from the market Yesterday I searched in every my cupboard for SSC MTS – 06/08/2019 (Shift-I) the shirt I couldn't find. Ans. (c) : In the above sentence, 'a little rice' will be (a) I couldn't find (b) in every my cupboard used instead of 'a few rice', because 'A few' is used as a (c) for the shirt (d) Yesterday I searched SSC CHSL (Tier-I) 14/10/2020 (Shift-I) countable sense and 'a little' is used uncountable sense. While 'Rice' is an uncountable noun. Hence the use of Ans. (b) : In the above sentence, 'instead of' in every little would be appropriate. my cupboard', 'in every cupboard' would be Correct sentence : appropriate. Because possessive adjective is not used My brother brought a little rice while coming from the after every. market. Correct Sentence :Yesterday I searched in every cupboards for the shirt I 510. Find the part of the given sentence that has an error in it. If there is no error, choose 'No couldn't find. error'. 506. Identify the segment in the given sentence Pallavi is the cleverest of the two sisters./No which contains the grammatical error. error I don't recognise a none of the politicians who (a) of the two sisters (b) Pallavi is have been invited to be the speakers today. (c) the cleverest (d) No error (a) invited to be (b) the speakers today SSC MTS – 06/08/2019 (Shift-I) (c) I don't recognise none of Ans. (c) : In the given sentence, 'the cleverer' will be (d) the politicians who nave been used instead of 'the cleverest' because the comparative SSC CHSL (Tier-I) 13/10/2020 (Shift-II) degree is used for two men or two things and Ans. (c) : In the above sentence, 'I don't recognise any of superlative Degree is used more than two men or two ' will be used in place of 'I don't recognise none of'. things. Because the use of 'don't. makes it clear that Sentence is Correct sentence : negative. Hence 'none' will not be used but 'any' will be Pallavi is the cleverer of the two sisters. used. which gives negative sense. 511. Find the part of the given sentence that has an Correct Sentence :error in it. If there is no error, choose 'No I don't recognise any of the politicians who have been error'. invited to be the speakers today. He did not have few hopes when he contested 507. Find the part of the given sentence that has an the election./No error error in it. If there is no error, choose 'No (a) He did not have error'. (b) few hopes when he I know that he wanted to go home earliest./ No (c) contested the election error. (d) No error (a) go home earliest (b) I know that SSC MTS – 05/08/2019 (Shift-III) (c) No error (d) he wanted to Ans. (b) : In the above sentence 'Any' would be SSC MTS – 08/08/2019 (Shift-I) appropriate instead of 'few' because 'any' is used for Ans. (a) : In the above sentence, 'early' (adverb) will negative sense and 'hope' would be appropriate instead be used instead of earliest (adjective). Because an of 'hopes' because 'hope' is an uncountable noun. adverb will be used to modify the verb, (go). Note- Any is used in the sense of negative and Correct sentence :interrogative sentence. I know that he wanted to go home early. 508. Find the part of the given sentence that has an Correct sentence : error in it. If there is no error, choose 'No He did not have any hope when he contested the election. error'. 512. Find the part of the given sentence that has an She ran so fastly that I could not overtake error in it. If there is no error, choose 'No her./No error error'. (a) She ran so fastly (b) not overtake her It was not possible/for we/to understand her/No (c) No error (d) that I could error. SSC MTS – 08/08/2019 (Shift-I)

Spotting Error

75

YCT

(a) for we (b) to understand her (c) It was not possible (d) No error SSC MTS – 02/08/2019 (Shift-I) Ans. (a) : In the above sentence, 'for us' will be used instead of 'for we', because the pronoun comes in objective case after the preposition. Hence, the objective case of 'we' will be 'us'. Correct sentence : It was not possible for us to understand her. 513. Identify the segment in the sentence which contains the grammatical error. The recently announcement about various government schemes impacted the society positively. (a) about various government (b) the society positively (c) schemes impacted (d) The recently announcement SSC MTS – 21/08/2019 (Shift-III) Ans. (d) :In the above sentence, 'recent announcement' will be used instead of recently announcement. Because 'announcement' is a noun in sentence. Hence Adjective 'recent' will be used to qualify before it, not recently (adverb). Correct sentence : The recent announcement about various government schemes impacted the society positively. 514. Identify the segment in the sentence which contains the grammatical error. After the match, every player of the team went back to its house. (a) team went back (b) After the match (c) to its house (d) every player of the SSC MTS – 21/08/2019 (Shift-III) Ans. (c) : In the above sentence 'To his house' will be used instead of 'to its house' because in sentence, subject is 'every player of the team', which denotes singular person. So 'his' would be appropriate as a possessive adjective for singular person. Correct sentence : After the match, every player of the team went back to his house. 515. Identify the segment in the sentence which contains the grammatical error. Rita was a model so she would starve herself to maintain their body weight. (a) so she would starve herself (b) their body weight (c) Rita was a model (d) to maintain SSC MTS – 16/08/2019 (Shift-II) Ans. (b) : In the above sentence, 'her' will be used instead of 'their'. Because possessive adjective 'her' would be appropriate for Rita (girl). Note– 'Their' is a possessive adjective of 'they'. Correct sentence : Rita was a model so she would starve herself to maintain her body weight. 516. Identify the segment in the sentence which contains the grammatical error. David and me are brothers but from different mothers.

Spotting Error

(a) but from (b) different mothers (c) David and me (d) are brothers SSC MTS – 14/08/2019 (Shift-III) Ans. (c) : In the above sentence 'I' (subjective case) would be appropriate instead of 'me' (objective case) because 'same part of speech' join with and. Note:– All pronouns are used as subjective case before main verb. Correct sentence : 'David and I are brothers but from different mothers.' 517. Identify the segment in the sentence which contains the grammatical error. The master did not know who of the servants had broken the glass. (a) the glass (b) who of the servants (c) had broken (d) The master did not know SSC MTS – 14/08/2019 (Shift-II) Ans. (b) : In the above sentence, 'which' will be used in place of 'who'. Because which of the + plural noun/pronoun is used. Here 'which' would be appropriate and gives suitable meaning for the sentence. Correct sentence :The master did not know which of the servants had broken the glass. 518. Identify the segment in the sentence which contains the grammatical error from the given options. The essay is divided into five paragraphs and every of these has seven sentences. (a) has seven sentences (b) into five paragraphs (c) The essay is divided (d) and every of these SSC MTS – 13/08/2019 (Shift-II) Ans. (d) : In the above sentence, 'Each' will be used instead of 'every', because 'each' would be appropriate as a pronoun, while 'every' is only used as an adjective. Notes- Every is only used as an adjective, while 'each' is used both pronoun and adjective. Correct sentence : The essay is divided into five paragraphs and each of these has seven sentences. 519. Identify the segment in the sentence which contains the grammatical error : Grandmother was knitting a sweater for his grand-daughter. (a) sweater for his (b) Grandmother was (c) grand-daughter (d) knitting a SSC MTS – 13/08/2019 (Shift-I) Ans. (a) : In the above sentence, 'her' will be used instead of 'his'. Because possessive adjective 'her' is used for female (Grand-mother). While 'his' is used for masculine gender. Correct sentence :‘‘Grandmother was knitting a sweater for her granddaugher.'' 520. Identify the segment in the sentence which contains the grammatical error from the given options. It's important for media to be more responsibly and careful.

76

YCT

(a) It's important (b) for media to be (c) more responsibly (d) and careful SSC MTS – 09/08/2019 (Shift-III) Ans. (c) : In the above sentence 'More responsible' will be used in place of 'more responsibly' because when two words joined by 'and', both the words are used in the same parts of speech. Hence 'careful' is used as an adjective then, Responsible (adjective) will be used instead of responsibly (adverb). Note :– In sentence An adjective is used to modify media (noun), not an adverb. Correct sentence :It's important for media to be more responsible and careful. 521. Find the part of the given sentence that has an error in it. If there is no error choose 'No error. He lost all his hardly earned money./ No error (a) all his (b) hardly earned money (c) No error (d) He lost SSC MTS – 07/08/2019 (Shift-III) Ans. (b) : In the above sentence 'Hard' will be used instead of 'Hardly', because 'Hardly' is used as a negative sense, while 'hard' is used as a positive sense which gives suitable meaning. Correct Sentence :He lost all his hard earned money. 522. Find the part of the given sentence that has an error in it. If there in no error, choose 'No error'. Don't worry, there is less time for the train to arrive./No error (a) Don't worry (b) there is less time (c) No error (d) for the train to arrive SSC MTS – 07/08/2019 (Shift-III) Ans. (b) : In the given sentence, according to the sense, 'much' is used in place of 'less'. Because it is clear from the use of 'don't worry' make it clear' that there is still more time for the train to arrive. Hence the use of 'much' would be appropriate. Correct sentence :Don't worry, there is much time for the train to arrive. 523. Find the part of the given sentence that has an error in it. If there is no error, choose 'No error'. She loved herself so more that she thought of no one else./No error (a) thought of no one else (b) No error (c) so more that she (d) She loved herself SSC MTS – 07/08/2019 (Shift-II) Ans. (c) : In the above sentence, 'So much' would be appropriate instead of 'so more', because here 'more' is used as countable sense while 'much' is used as uncountable sense. In this sentence, 'love' is used as uncountable sense. Correct sentence :She loved herself so much that she thought of no one else.

Spotting Error

524. Find the part of the given sentence that has an error in it. If there is no error, choose 'No error'. It would take up few of my time to explain to you the whole case./No error (a) few of my time to (b) No error (c) explain to you the whole case (d) It would take up SSC MTS – 06/08/2019 (Shift-II) Ans. (a) : In the above sentence 'A little of my time to' would be appropriate in place of 'few of my time of' because adjective 'a little' is used for uncountable noun. While 'a few' is used for countable sense. Correct sentence : It would take up a little of my time to explain to you the whole case. 525. From the given options, identify the segment in the sentence which contains the grammatical error. Nowadays spectacles are costly than they used to be. (a) Nowadays (b) they used to be (c) spectacles are (d) costly than SSC GD – 18/02/2019 (Shift-III) Ans. (d) : In the above sentence, in the sense of showing comparison, 'costlier' would be appropriate in place of 'costly'. Because comparative degree is always used to show comparison and it is clear from the use of 'than' in the sentence. Correct sentence :Nowadays spectacles are costlier than they used to be. 526. From the given options, identify the segment in the sentence which contains the grammatical error. She was known for her skills in dance and music; she is equal proficient in the allied art of painting. (a) skills in dance (b) allied art of painting (c) was known (d) is equal proficient SSC GD – 11/02/2019 (Shift-II) Ans. (d) : In the above sentence, 'Adverb, 'equally' would be appropriate instead of 'equal'. Because adverb is used to modify an adjective (proficient). Correct sentence :She was known for her skills in dance and music, she is equally proficient in the allied art of painting.

H.

Conditional Sentence

527. The following sentence has been split into four segments. Identify the segment that contains a grammatical error. When Mayank came, / I shall go / with him to / the circus. (a) When Mayank came (b) with him to (c) the circus (d) I shall go SSC CHSL (Tier-I) 03/08/2023 (Shift-II)

77

YCT

Ans. (a) : In the above given sentence, the segment – 'when Mayank came ' is grammatically incorrect, because the sentence is showing the condition, when the first clause is in simple present, then the second clause will be in simple future tense. (When + first clause (simple present), second clause (simple future tense)] Correct sentence – 'When Mayank Comes, I shall go with him to the circus.' 528. The following sentence has been split into four segments. Identify the segment that contains a grammatical error. If you plant a seed, / you will be noticed / that it takes some / time to grow. (a) that it takes some (b) If you plant a seed (c) you will be noticed (d) time to grow SSC MTS-13/07/2022 Shift-I Ans. (c) : In the above sentence, 'you will notice' will be used instead of 'you will be noticed', because in this type of conditional sentence if one clause is in present tense, the main clause is future indefinite tense. Correct Sentence – If you plant a seed you will notice that it makes some time to grow. 529. The following sentence has been divided into parts. One of them may contain an error. Select the part that contains the error from the given options. If you don't find any error, mark 'No error' as your answer. If he had not dropped out, / he will most certainly have completed / his education by 2023. (a) his education by 2023 (b) No error (c) If he had not dropped out (d) he will most certainly have completed SSC MTS-14/07/2022 Shift-III Ans. (d) : In the above sentence, 'he would most certainly have completed' will be used in place of 'he will most certainly have completed', because 'if in first clause' had + v3 is used then in second clause 'would have + v3' will be is used. Correct Sentence – If he had not dropped out, he would most certainly have completed his education by 2023. 530. The following sentence has been split into four segments. Identify the segment that contains a grammatical error. Has the situation / been favourable, / I would have / attended the marriage. (a) attended the marriage (b) Has the situation (c) I would have (d) been favourable SSC Constable GD-26/11/2021 Shift-I Ans. (b) : In the above sentence, 'had' is used instead of 'Has' because the sentence is conditional and if 'would have' is used in second clause then had + V3 form is used in the first clause. Spotting Error

Note – 'Had' is used at the beginning of the sentence to show the condition. Correct Sentence– Had the situation been favourable, I would have attended the marriage. 531. In the given sentence, identify the segment which contains the grammatical error. He wonder for a moment if he had mistaken the direction. (a) if he had mistaken (b) He wonder (c) for a moment (d) the direction SSC CHSL (Tier-I) –15/04/2021 (Shift-I) Ans. (b) : In the above sentence 'He would have wondered' will be used instead of 'he wonder' If + sub + had + v3 ......would have + v3 Correct SentenceHe would have wondered for a moment if he had mistaken the direction. 532. In the sentence identify the segment which contains the grammatical error. That man would have attacked you, if you went that way. (a) That man would have (b) that way (c) if you went (d) attacked you SSC CHSL (Tier-I) –13/04/2021 (Shift-I) Ans. (c) : In the above sentence, 'If you had gone' will be used instead of 'if you went'. Because when 'would have' is used in principal clause, then 'if + had + V3' is used in sub-ordinate clause. Correct Sentence That man would have attacked you, if you had gone that way. 533. Identify the segment in the sentence which contains a grammatical error. If there is no error, then select the option 'No error'. When he goes on a business trip, he is usually taking his wife. (a) on a business trip (b) No error (c) he is usually taking his wife (d) when he goes SSC CPO-SI – 23/11/2020 (Shift-I) Ans. (c) : In the above sentence, 'takes' will be used instead of 'taking'. Because the above sentence is showing condition in such sentences, if first clause is in present indefinite. then the second one will also be in present indefinite. Note :– 'verb1 + s/es' is used with singular subject in present indefinite tense and simple sentence. Correct sentence :When he goes on a business trip, he usually takes his wife. 534. Identify the segment in the sentence which contains a grammatical error. She will come to work though it doesn't rain tomorrow. (a) She will come (b) though it doesn’t (c) to work (d) rain tomorrow SSC CPO-SI – 25/11/2020 (Shift-II)

78

YCT

Ans. (b) : In the above sentence The use of (if) would 538. Identify the segment in the sentence which contains the grammatical error. be appropriate instead of 'though', because if would be If I knew about him before I will have used as a conjunction in conditional sentence. Which recommended him. gives appropriate meaning to the sentence. (a) If I knew (b) recommended him Correct sentence :(c) about him before (d) I will have She will come to work if it doesn't rain tomorrow. SSC MTS – 20/08/2019 (Shift-II) 535. In the sentence identify the segment which In the above sentence, 'would have' will be Ans. (d) : contains the grammatical error. used instead of 'I will have' because given sentence is If so many catches had not being dropped, we mix conditional whose structure is as follows. would have won the match. (a) If so many catches If + simple past, + would have + V III form (b) had not being dropped Correct sentence :(c) won the match If I knew about him before, I would have (d) we would have recommended him. SSC CGL (Tier-I) – 07/06/2019 (Shift-III) 539. In the sentence identify the segment which Ans : (b) In the above sentence, 'been' would be contains the grammatical error. appropriate instead of 'being'. Because the given We will rest for sometime when we will reach sentence is conditional sentence, type III and passive the top. voice is used in 'if clause'. (a) We will rest (b) when we will (c) reach the top (d) for sometime A.V.– If + had + V 3 ...... would have + V 3 SSC CGL (Tier-I) – 06/06/2019 (Shift-III) P.V.— If + had + been + V 3 ...... would have + V3 Ans. (b) : In the above sentence 'When we reach' would be appropriate instead of 'when we will reach' Correct sentence :If so many catches had not been dropped, we would because when two correlated sentence is in future tense, then subordinate clause is always in present have won the match. indefinite tense and principal clause is always in future 536. In the sentence identify the segment which indefinite tense. contains the grammatical error. Correct Sentence :No sooner did he see the tiger when he ran as We will rest for sometime when we reach the top. fast as he could. 540. In the sentence identify the segment which (a) no sooner did (b) he see the tiger contains the grammatical error. (c) when he ran (d) as fast as he could If the growth of population had been controlled SSC CGL (Tier-I) – 10/06/2019 (Shift-II) earlier, mankind wouldn't lived in a polluted Ans : (c) In the above sentence, 'than' will be used world now. instead of 'when'. Because co-relative conjunction 'than' (a) mankind wouldn't lived is used with No-sooner' (b) in a polluted world now Rule- No sooner did → V1st........... than (c) if the growth of population No sooner had → V3rd........... than (d) had been controlled earlier SSC CPO-SI – 13/12/2019 (Shift-II) butAns. (a) : In the above sentence, 'Mankind wouldn't Hardly had → V3rd........... when have lived' would be appropriate instead of 'mankind Scarcely had → V3rd........... when wouldn't lived'. Because if clause is a conditional As soon as → V3rd..........., (comma) sentence whose correct syntax is as followsAll these rules are used in the same sense. If + sub + had + ...., sub + would + have + v3 + ... Correct sentence :No sooner did he see the tiger than he ran as fast as he Correct Sentence :If the growth of population had been controlled earlier, could. mankind wouldn't have lived in a polluted world now. 537. In the sentence identify the segment which 541. The following sentence has been split into four contains the grammatical error. segments. Identify the segment that contains a Entering the hall, the show had started. grammatical error. (a) had started (b) the show Had you/not reached in time,/we will have/lost (c) Entering (d) the hall our lives. SSC CGL (Tier-I) – 13/06/2019 (Shift-III) (a) Had you (b) lost our lives (c) not reached in time (d) we will have Ans : (c) In the above sentence, 'entered' will be used SSC CGL (Tier-I) –13/08/2021 (Shift-I) instead of 'entering'. Because as a rule, there is a sense of having two actions in the past, then for the action that Ans. (d) : In the above sentence, 'we would have' will happened first, the verb is in pasts perfect and for the be used instead of 'we will have' because the sentence is action that happened later, the verb is in past indefinite. in conditional sentence and in the first clause of conditional sentence- Had + V3 and in second clause Correct sentence :would have + V3 is used. When he entered into the hall, the show had started. Spotting Error

79

YCT

Note- Had + Subject + V3 + Other word + Subject + Would/Could/Might + have + V3 + Other word. Correct Sentence Had you not reached in time we would have lost our lives. 542. Identify the segment which contains the grammatical error. No sooner did the comedian begin his act when the audience roared with laughter. (a) when the audience (b) roared with laughter (c) No sooner did (d) the comedian begin SSC CHSL (Tier-I) –05/07/2019 (Shift-II) Ans : (a) In the above sentence 'Than' will be used instead of 'when' because the structure rule of no sooner in the sentence is as followNo sooner did + VIst .......... than No sooner had + VIIIrd .........than Correct sentence :No sooner did the comedian begin his act than the audience roared with laughter. 543. In the given sentence identify the segment which contains a grammatical error. When the boys were playing the match, it suddenly starts raining. (a) When the boys (b) starts raining (c) were playing the match (d) it suddenly SSC CHSL (Tier-I) 20/10/2020 (Shift-I) Ans. (b) : In the above sentence, 'started raining' would be appropriate in place of 'starts raining'. Because 'when' is used as a conjunction and introduces sub ordinate clause. In this situation, main clause is used in simple past. Hence the use of V2 form would be appropriate. Correct sentence :When the boys were playing the match, it suddenly started raining. 544. Identify the segment in the given sentence which contains the grammatical error. If India will win this match they will win the series. (a) the series (b) they will win (c) if India will win (d) this match SSC CHSL (Tier-I) 13/10/2020 (Shift-II) Ans. (c) : In the above sentence 'If India wins this match' would be appropriate instead of 'if India will win this match'. Because 'will/shall (modals) is not used in if clause. If + simple present,.... + ..... simple future Correct sentence :If India win this match, they will win the series. 545. Identify the segment in the sentence, which contains the grammatical error. If I join a software company I am getting a good deal of experience. (a) If I join (b) a software company (c) I am getting (d) a good deal of experience SSC CPO-SI (Tier-II) 27/09/2019 (3 pm - 5 pm) Spotting Error

Ans. (c) : In the above sentence, 'I will get' would be appropriate instead of 'I am getting', because in conditional sentence 'if' clause is in the present indefinite, then the letter second clause will be used in future indefinite. Correct sentence- If I join a software company ......... I will get a good deal of experience. 546. Identify the segment in the sentence which contains the grammatical error. Rakesh was studied when his friends called him. (a) studied when (b) his friends (c) called him (d) Rakesh was SSC MTS – 22/08/2019 (Shift-I) Ans. (a) : In the above sentence, 'studying' will be used instead of 'studies'. Rule - If some work is going on in the past and in the meantime. Any event happened (for a short time) then ongoing work is kept in past continuous and the event that happened in the middle is kept in 'past' indefinite. Correct sentence : [Rakesh was studying when his friends called him.] 547. Identify the segment in the sentence which contains the grammatical error. Had Rajani not taken/a cab to the station/she would have definitely/miss the train. (a) she would have definitely (b) Had Rajani not taken (c) miss the train (d) a cab to the station SSC MTS – 19/08/2019 (Shift-III) Ans. (c) : In the above sentence, 'missed the train' would be appropriate instead of 'miss the train'. Rule- IIIrd Conditional sentence structure. [If + subject + Had + v3 ---sub +would have + v3] or [Had + subject + v3, --- subject + would have + v3] Correct sentence : ''Had Rajani not taken a cab to the station she would have definitely missed the train.'' 548. From the given options, identify the segment in the sentence which contains the grammatical error. Having worked for the whole day, you shall have taken some rest and begun again. (a) some rest and begun again (b) the whole day (c) Having worked for (d) you should have taken SSC GD – 18/02/2019 (Shift-II) Ans. (d) : In the above sentence, 'You should have taken, will be used instead of 'you shall have taken', because it is the conditional sentence of past in which 'should have + VIIIrd form' would be appropriate. Correct sentence :Having worked for the whole day, you should have taken some rest and begun again. 549. Identify the segment in the sentence which contains the grammatical error from the given options. If children are treating with respect, they will behave accordingly.

80

YCT

(a) (b) (c) (d)

behave accordingly treating with respect If children are they will SSC MTS – 08/08/2019 (Shift-II) Ans. (b) : In the above sentence, 'are treated' would be appropriate in place of 'are treating'. Given sentence is conditional sentence type-1, in which if clause is used in simple present tense and main clause is used in future indefinite tense.

If .simple present, + will / shall + V I form because 'if' clause is in passive voice. Hence, use of the are + V3 would be appropriate. Correct sentenceIf children are treated with respect, they will behave accordingly. 550. Identify the segment in the sentence which contains the grammatical error. If I had gone to the library I can have borrowed a book. (a) I can have (b) gone to the library (c) borrowed a book (d) If I had SSC MTS – 16/08/2019 (Shift-I) Ans. (a) : In the above sentence, 'could have' would be appropriate in place of 'can have'. The Given sentence is conditional type-III. Whose structure is as follows– If + past perfect, (would have / could have) + V III form Correct sentence : If I had gone to the library I could have borrowed a book. 551. From the given options identify the segment in the sentence which contains the grammatical error. If they had a playground here, they can play after their school is over. (a) after their (b) If they had (c) school is over (d) they can play SSC GD – 14/02/2019 (Shift-II) Ans. (d) : In the above sentence, 'they could play' would be appropriate instead of 'they can play' Because this is conditional sentence of simple sentence whose structure is– If + subject + simple past tense + subject + would/could/might + verb in base form (V3).

If + simple past, would / could + V1 Correct sentence :If they had a playground here, they could play after their school is over. J.

I.

Noun Case, Gender, Number

552. Find the part of the given sentence that has an error in it. If there is no error, choose 'No error'. I just don't like the themes of todays music. (a) No error (b) I just don't (c) like the themes of (d) todays music. SSC CHSL (Tier-I) 10/03/2023 (Shift-III) Spotting Error

Ans. (d) : In the above sentence "today's" will be used in place of "todays" because to show Possession Apostrophe ('s) is used with related noun. Today's music – Music of Today Correct Sentence– I just don't like the themes of today's music 553. The following sentence has been split into four segments. Identify the segment that contains a grammatical error. We bought / two dozen's / mangoes from / the market. (a) mangoes from (b) We bought (c) the market (d) two dozen's SSC CGL (Tier-I) 20/04/2022 Shift-III Ans. (d) : Some numbers (hundred, thousand, lakh, Million, billion dozens) are used both singular and plural. If definite number (one, two, three.....) has been used before them, then their plural is not made. So we will replace two dozens with two dozen. Correct Sentence We bought two dozen mangoes from the market. 554. The following sentence has been divided into parts. One of them contain an error. Select the part that contains the error from the given options. If you don't find any error, mark 'No error' as your answer. My neighbour's grandson / is only / five month old. (a) No error (b) five month old (c) My neighbour's grandson (d) is only SSC CGL (Tier-I) 13/04/2022 Shift-II Ans. (b) : In the above sentence, 'Months' will be used in place of 'Month'. Because plural noun is used after five (plural number). Correct SentenceMy neighbour's grandson is only Five months old. 555. The following sentence has been divided into parts. One of them may contain an error. Select the part that contains the error from the given options. If you don’t find any error, mark ‘No error’ as your answer. It was a surprising / to receive the gift/ from my brother. (a) from my brother (b) It was a surprising (c) No error (d) to receive the gift SSC CGL (Tier-I) 12/04/2022 Shift-I Ans. (b) : In the above sentence 'A surprise' will be used instead of 'a surprising' because according to the sense of sentence, Noun form is required here. Correct Sentence – It was a surprise to receive the gift from my brother. 556. The following sentence has been divided into parts. One of them may contain an error. Select the part that contains the error from the given options. If you don't find any error, mark 'No error' as your answer.

81

YCT

They finally did what / we advice them to do / before the final match. (a) we advice them to do (b) before the final match (c) They finally did what (d) No error SSC MTS-13/07/2022 Shift-I Ans. (a) : In the above sentence, 'we advise (verb) them to do' will be used instead of 'we advice (noun) them to do', because verb (advise) is used after subject 'we', not noun (advice). Correct Sentence – They finally did what we advise them to do before the final match 557. The following sentence has been split into four segments. Identify the segment that contains a grammatical error. Human being are / the producers and the / consumers of Earth's / resources. (a) Human being are (b) resources (c) the producers and the (d) consumers of Earth's SSC MTS-06/07/2022 Shift-III Ans. (a) : In the above sentence Instead of 'Human being are', 'human beings are' will be used because 'Human being' is always used in plural sense. Correct Sentence – Human beings are the producers and the consumers of Earth's resources. 558. The following sentence has been divided into parts. One of them may contain an error. Select the part that contains the error from the given options. If you don't find any error, mark 'No error' as your answer. Land revenue was one of the / major / source of income / for Britishers in India. (a) No error (b) land revenue was one of the (c) major source of income (d) for Britishers in India SSC MTS-08/07/2022 Shift-I Ans. (c) : In the above sentence, 'major sources of income' would be appropriate instead of 'major source of income', because plural noun (Major Sources) is used after 'one of the'. Correct Sentence – Land revenue was one of the major sources of income for Britishers in India. 559. The following sentence has been split into four segments. Identify the segment that contains a grammatical error. The man turned to him with a look of disapprove. (a) with a look (b) turned to him (c) The man (d) of disapprove SSC Constable GD-16/11/2021 Shift-III Spotting Error

Ans. (d) : In the above sentence 'Disapproval' (noun) would be appropriate instead of 'disapprove' (verb) because according to the sense of sentence, Noun form (disapproval) is required here. Correct sentence The man turned to him with a look of disapproval. 560. The following sentence has been split into four segments. Identify the segment that contains a grammatical error. The manager told/the staff to complete/all the works/before going home. (a) before going home (b) the staff to complete (c) the manager told (d) all the works SSC Constable GD-08/12/2021 Shift-I Ans. (d) : In the above sentence, 'all the work' will be used in place of 'all the works', because 'work' is an uncountable noun and uncountable noun is not made plural by adding 's/es'. Correct sentence - The manger told the staff to complete all the work before going home. 561. Identify the segment in the sentence which contains a grammatical error. He loses his tempers on the slightest provocation. (a) provocation (b) He loses (c) the slightest (d) his tempers on SSC CGL (Tier-II) 16/11/2020 (3 pm - 5 pm) Ans. (d) : In the above sentence, 'his temper on' will be used in place of 'his tempers on', because 'temper' is an uncountable. Hence its plural will not be formed. Correct sentence- He loses his temper on the slightest provocation. 562. Identify the segment in the sentence which contains a grammatical error. The man played the flute and led all the mouse out of the town. (a) The man played (b) The flute and led (c) all the mouse (d) out of the town SSC CGL (Tier-II) 16/11/2020 (3 pm - 5 pm) Ans. (c) : In the above sentence, 'Mice' will be used in place of 'Mouse', because 'all' is used with plural noun and 'mice' is the plural number of 'mouse'. Correct sentence- The man played the flute and led all the mice out of the town. 563. Identify the segment in the sentence, which contains the grammatical error. The chart papers were distributed among all the childrens. (a) The chart papers (b) among all (c) were distributed (d) the childrens

82

SSC CPO-SI (Tier-II) 27/09/2019 (3 pm - 5 pm)

Ans. (d) : In the above sentence Instead of 'childrens', 'Children 'will be used because'. Children is a plural number of 'child'. Correct sentenceThe chart papers were distributed among all the children. YCT

564. The following sentence has been split into four (a) Subhash Gidwani is selling dolls segments. Identify the segment that contains a (b) held in january every year in Delhi grammatical error. (c) which are type of Madhya Pradesh's village Public speaking/is one of/ the most feared culture form/of communication. (d) in the annual Dastakar Bazaar (a) the most feared form SSC CHSL (Tier-I) 21/10/2020 (Shift-II) (b) is one of In the above sentence, 'the village culture of Ans. (c) : (c) of communication Madhya Pradesh' will be used instead of 'Madhya (d) Public speaking SSC CHSL (Tier-I) –19/04/2021 (Shift-I) Pradesh's village culture. Because apostrophe 's' is Ans. (a) : In the above sentence, 'forms' will be used in used for living thing noun to form possessive case and place of 'form' because plural noun is used after 'one of 'of' is used for non living thing to form possession. Correct Sentence :the'. Subhash Gidwani is selling dolls, which are type of the Correct Sentence Public speaking is one of the most feared forms of village culture of Madhya Pradesh at the Annual Dastakar Bazaar, held in January every year in Delhi. communication. 565. Identify the segment in the sentence which 569. Select the segment in the sentence, which contains the grammatical error. contains a grammatical error. The furnitures in this shop is very costly. This furnitures has been taken on hire for the (a) in this shop (b) The furnitures function in the college. (c) is (d) very costly (a) This furnitures has (b) in the college SSC Stenographer Grade C&D –05/02/2019 (3:5pm) (c) been taken (d) on hire for the Ans. (b) : In the above sentence 'Furniture' will be function used instead of 'furnitures' because 'furniture' can not SSC CHSL (Tier-I) 17/03/2020 (Shift-II) be formed plural by adding 's/es'. Ans. (a) : In the above sentence 'This furniture has' Correct sentencewould be appropriate instead of 'this furnitures has' The furniture in this shop is very costly. 566. Identify the segment in the sentence which because the plural of furniture is not formed by adding s/es. contains the grammatical error. The childrens are fond of climbing the mango One piece of furniture Two pieces of furniture tree in the garden. (a) are fond of Correct Sentence :(b) in the garden This furniture has been taken on hire for the function (c) The childrens in the college. (d) climbing the mango tree 570. In the given sentence identify the segment SSC CHSL (Tier-I) –09/07/2019 (Shift-III) which contains the grammatical error. Ans : (c) In the above sentence, 'the children' will be The Commonwealth Games had a grand used instead of 'the childrens', because children itself is opening ceremony with spectacular cultural plural, its singular is child. Hence there is no need to performings. make plural by putting 's' in it. (a) a grand opening Correct Sentence :(b) the commonwealth Games had The children are fond of climbing the mango tree in the (c) cultural performings garden. (d) with spectacular 567. Select the segment in the sentence, which SSC CHSL (Tier-I) 15/10/2020 (Shift-II) contains the grammatical error. They had to wait for the luggages to be put into Ans. (c) : In the above sentence, 'cultural performances' would be appropriate in place of cultural the cab before they themselves got into it. (a) before they themselves got into it performings. Because 'noun' will be used after cultural (b) for the luggages (adjective). Hence the use of performance (noun) is (c) They had to wait appropriate. (d) to be put into the cab Correct Sentence :SSC CHSL (Tier-I) 17/03/2020 (Shift-I) The Commonwealth Games had a grand opening Ans. (b) In the above sentence, 'for the luggage, will be ceremony with spectacular cultural performances. used in place of for the Luggages, because plural of 571. Find the part of the given sentence that has an luggage can not be formed by adding 's/es'. error in it. If there is no error, choose 'No Correct Sentence : error'. The had to wait for the lugguage to be put into the cab Her relatives were presented at the station to before they themselve got into it. see her off./No error 568. In the sentence identify the segment which (a) to see her off contains the grammatical error. (b) presented at the station Subhash Gidwani is selling dolls, which are (c) Her relatives were type of Madhya Pradesh's village culture at the (d) No error Annual Dastakar Bazaar, held in January SSC MTS – 06/08/2019 (Shift-III) every year in Delhi. Spotting Error

83

YCT

Ans. (b) : In the above sentence Present (Noun) will be used in place of 'presented' (verb) because if were presented (III form of verb) is used in the sentence, it will be in passive voice. Which will not give proper sense. Correct Sentence :Her relatives were present at the station to see her off. 572. Identify the segment in the sentence which contains the grammatical error. It is not possible to discuss anything with her because of her angry. (a) possible to discuss (b) anything with her (c) because of her angry (d) It is not SSC MTS – 22/08/2019 (Shift-III) Ans. (c) : In the above sentence 'Because of her anger' will be used instead of 'because of her angry' because 'angry' is an adjective' while noun is used after her (possessive adjective). Hence here 'anger' will be used because as a noun. Correct sentence :It is not possible to discuss anything with her because of her anger. 573. Find the part of the given sentence that has an error in it. If there is no error, choose 'No error'. My visiting to my family are few and far between. (a) No error (b) few and far between (c) My visiting to (d) my family are SSC MTS – 02/08/2019 (Shift-III) Ans. (c) : In the above sentence, 'my visits' will be used instead of 'my visiting to'. Because 'my' is a determiner which is always used before noun and for plural verb 'are' 'visits' (plural noun) will be used. Correct sentence :My visits to my family are few and far between. 574. Identify the segment in the sentence which contains the grammatical error. Platinum is costlier than any other metals found on earth. (a) Costlier than (b) Platinum is (c) found on earth (d) any other metals SSC MTS – 19/08/2019 (Shift-II) Ans. (d) : In the above sentence 'Metal' would be appropriate in place of 'metals' because singular noun is used with any other. Correct sentence : ''Platinum is costlier than any other metal found on earth." 575. Identify the segment in the sentence which contains the grammatical error. After a heated argument they went to their respective place. (a) they went (b) after a heated argument (c) respective place (d) to their SSC MTS – 19/08/2019 (Shift-I) Spotting Error

Ans. (c) : In the above sentence, 'respective places' will be used instead of 'respective place', because it is clear from the use of plural possessive adjective (their) in the sentence that noun also will be plural after it. Hence, the use of 'places' would be appropriate. Correct Sentence :''After a heated argument they went to their respective Places.'' 576. Identify the segment in the sentence which contains the grammatical error. This coffee is not available in any of the shop in the neighborhood market. (a) not available in (b) in the neighborhood market (c) any of the shop (d) this coffee is SSC MTS – 14/08/2019 (Shift-III) Ans. (c) : In the above sentence, 'any of the shops, will be used in place of 'any of the shop'. Because plural form of noun/pronoun is used after 'any of the'. Hence the use of shops (plural) would be appropriate. Correct Sentence :This coffee is not available in any of the shops in the neighborhood market. 577. Identify the segment in the sentence which contains the grammatical error from the given options. Its a good idea to read up on a company before going for an interview. (a) before going for (b) its a good idea to (c) an interview (d) read up on a company SSC MTS – 09/08/2019 (Shift-II) Ans. (b) : In the above sentence 'It's' would be appropriate instead of 'its' because 'its' is possessive adhective which will not be used instead of subject. The sort form of it's is it is and it is used in place of subject + verb. Correct Sentence :It's a good idea to read up on a company before going for an interview. 578. Identify the segment in the sentence which contains the grammatical error from the given options. The sense of sight is one of the most important senses gift to us. (a) The sense (b) most important (c) gift to us (d) is one of the SSC GD – 08/03/2019 (Shift-II) Ans. (c) : In the above sentence, 'gifts' will be used in place of gift, because noun is always used in plural form after one of the. Correct sentence :The sense of sight is one of the most important senses gifts to us. 579. Identify the segment in the sentence which contains the grammatical error from the given options. Some people lose their eye sights because of a disease or injury. (a) because of (b) disease or injury (c) their eye sights (d) some people SSC GD – 08/03/2019 (Shift-II)

84

YCT

Ans. (c) : In the above sentence, 'their eye sight' will be used instead of 'their eye sights' because uncountable noun are usually not made plural by adding s/es. Since 'eye sight' is an uncountable noun, hence s/es will not be used. Correct sentence :Some people lose their eye sight because of a disease or injury. 580. From the given options, identify the segment in the sentence which contains the grammatical error. Animals experience feelings of love and they also protect their young ones from difficulties as human does. (a) they also protect (b) Animals experience (c) as human does (d) feelings of love SSC GD – 02/03/2019 (Shift-II) Ans. (d) : In the above sentence, 'feeling of love' will be used instead of 'feelings of love' because plural is not made by adding s/es in abstract noun. Hence 'feeling' is used without s/es. Correct sentence :Animals experience feeling of love and they also protect their young ones from difficulties as human does. 581. Find the part of the given sentence that has an error in it. If there is no error, choose 'No error'. There is few to be said on both side./No error (a) to be said (b) There is few (c) on both sides (d) No error SSC MTS – 07/08/2019 (Shift-II) Ans. (b) : In the above sentence, 'There is little' will be used instead of 'there is few', because 'few' is used for countable noun while little is used for uncountable noun. Hence 'to be said' is used here in uncountable sense. Hence the use of little would be appropriate. Correct SentenceThere is little to be said on both sides.

J.

Voice.and Narration

582. The following sentence has been split into four segments. Identify the segment that contains a grammatical error. Ajit asked me/how could I solve/the complex puzzle/so easily. (a) so easily (b) the complex puzzle (c) Ajit asked me (d) how could I solve SSC Constable GD-02/12/2021 Shift-I Ans. (d) : In the above sentence 'how I could solve' will be used in place of 'how could I solve' because the sentence is in interrogative sentence of Indirect. So it will be written in the form of assertive sentence. Correct sentenceAjit asked me how I could solved the complex puzzle so easily. Spotting Error

583. The following sentence has been split into four segments. Identify the segment that contains a grammatical error. When Nancy saw the mess / the burglar was made, / she broke down / in tears. (a) she broke down (b) the burglar was made (c) in tears (d) When Nancy saw the mess SSC Constable GD-14/12/2021 Shift-III Ans. (b) : In the above sentence 'the burglar made' will be used in place of 'the burglar was made' because the sense of the sentence is in past and active voice, So "Subject +_ verb2" will be used. And 'that' will be used as conjunction. Correct Sentence– When Nancy saw the mess that burglar made she broke down in tears. 584. Parts of the given sentence have been given as options. One of them contains a grammatical error. Select the option that has the error. The easiest way to make pasta is to be followed Chef Kapoor's instructions on YouTube. (a) instructions on YouTube (b) to be followed (c) the easiest way (d) make pasta SSC Constable GD-07/12/2021 Shift-II Ans. (b) : In the above sentence 'to follow' will be used in place of 'to be followed' because sentence is in Active voice of present tense. Correct sentence – The easiest way to make pasta is to follow chef Kapoor’s instructions on you tube. 585. Identify the segment in the sentence which contains a grammatical error. A heat wave is declare when the maximum temperature hits 45º Celsius or is above 40º Celsius in summer. (a) A heat wave (b) when the (c) is declare (d) or is above SSC Stenographer (Grade C & D) 11.11.2021 Shift-I Ans. (c) : In the given sentence 'is declared' will be used in the place of 'is declare' because the sentence is in passive voice of present Indefinite tense. Active voice - sub + V1 + object Passive voice - obj + is/am/are + v3 + by + sub Correct Sentence(A heat wave is declared when the maximum temperature hits 45º celsius or is above 40º celsius in summer.) 586. Identify the segment in the sentence which contains a grammatical error. He wondered what was the reason for her refusal to accompany him. (a) was the reason (b) He wondered what (c) for her refusal (d) to accompany him SSC CPO-SI – 23/11/2020 (Shift-II)

85

YCT

Ans. (a) : In the above sentence, ‘the reason was’ will be used in place of ‘was the reason’. Because the above sentence is in indirect speech. So the interrogative of reported speech will change into assertive. In sentence ‘what’ is used as a conjunction to connect both clauses. Correct sentence :He wondered what the reason was for her refusal to accompany him. 587. In the given sentence, identify the segment which contains a grammatical error. The staff is being advise to maintain distance while seated in office. (a) The staff (b) in office (c) to maintain (d) is being advise SSC Stenographer (Grade C & D) 11.11.2021 Shift-I Ans. (d) : In the above sentence, ‘is being advise’ would be appropriate instead of ‘is being advised’. Because ‘sentence’ is used in passive voice. In passiveis + being + V3 Correct Sentence The staff is being advised to maintain distance while seated in office. 588. In the sentence identify the segment that contains a grammatical error. In today's world new ways of teaching young children have to be adopting. (a) today's world (b) teaching your children (c) have to be adopting (d) new ways of SSC Stenographer (Grade C & D) 15.11.2021 Shift-II Ans. (c) : In the above sentence ‘Have to be adopted’ would be appropriate instead of ‘have to be adopting’ because ‘be + VIII’ is used in passive voice. Correct sentence:In today's world new ways of teaching young children have to be adopted. 589. In the sentences identify the segment that contains a grammatical error. When I realised I was following by some street boys I decided to change my route. (a) was following by (b) I decided (c) I realised (d) to change SSC Stenographer (Grade C & D) 15.11.2021 Shift-I Ans. (a) : In the given sentence, ‘was being followed by’ will be used instead of ‘was following by’ Because ‘sentence’ is used in passive voice. In passivewas + being + V3 is used. 590. The following sentence has been split into four segments. Identify the segment that contains a grammatical error. I asked my friend/where had he/learnt to dance/so well. (a) I asked my friend (b) so well (c) where had he (d) learnt to dance SSC CGL (Tier-I) 19/04/2022 Shift-II Spotting Error

Ans. (c) : In Indirect speech interrogative sentence of reported speech is always used in Assertive form. Hence ‘Where he had’ will be used in place of ‘Where had he’. Correct sentence – I asked my friend where he had learnt to dance. 591. The following sentence has been split into four segments. Identify the segment that contains a grammatical error. The policeman asked / many people but / no one was knowing / how the accident happened. (a) The policeman asked (b) many people but (c) how the accident happened (d) no one was knowing SSC CGL (Tier-I) 12/04/2022 Shift-III Ans. (d) : In the above sentence, ‘no one had known’ will be used instead of ‘no one was knowing’. the given sentence is used in direct speech if it changes into indirect, ‘Had + V3’ will be used. Correct SentenceThe Policeman asked many people but no one had known how the accident happened. 592. The following sentence has been split into four segments. Identify the segment that contains a grammatical error. The visitors / were being showed / a collection of / old manuscripts. (a) a collection of (b) were being showed (c) old manuscripts (d) The visitors SSC Constable GD-30/11/2021 Shift-III Ans. (b) : In the above sentence, ‘were being shown’ will be used instead of ‘were being showed’. Because the sentence is in Passive Voice and the following structure is used in this sentenceWas/were + being + verb3 Show (I) → Showed (II) → Shown (III) Correct sentence The visitors were being shown a collection of old manuscripts. 593. The following sentence has been split into four segments. Identify the segment that contains a grammatical error. The teacher punished him / and told him / that he does not / need to give a clarification. (a) need to give a clarification (b) The teacher punished him (c) that he does not (d) and told him SSC Constable GD-25/11/2021 Shift-III Ans. (c) : In the above sentence, ‘did not’ will be used instead of ‘does not’, because above sentence is in Indirect speech. and reporting is in past tense. Hence reported speech would be appropriate in past indefinite. Correct sentence – The teacher punished him and told him that he did not need to give a clarification.

86

YCT

594. The following sentence has been split into four segments. Identify the segment that contains a grammatical error. The president forced/ to leave the country/when the people rose up against / the government. (a) to leave the country (b) the government (c) The president forced (d) when the people rose up against SSC Constable GD-17/11/2021 Shift-II Ans. (c) : In the above sentence, ‘was forced’ will be used in place of ‘forced’, because the sense of the sentence is in passive. The passive structure is as followswas/were + viii Correct Sentence The president was forced to leave the country when the people rose up against the government. 595. The following sentence has been split into four segments. Identify the segment that contains a grammatical error. Peter said / then / he was / feeling unwell. (a) Feeling unwell (b) he was (c) Peter said (d) then SSC Constable GD-01/12/2021 Shift-I Ans. (d) : In the above sentence, ‘that’ will be used instead of ‘then’, because while making simple sentence of indirect narration, ‘that’ is used as a Conjunction. Correct sentence– ∗ Peter said that he was feeling unwell. 596. Identify the segment in the sentence, which contains the grammatical error. The author's new novel, which is about social change, will launched soon. (a) The author’s new novel (b) will launched soon (c) which is (d) about social change SSC CPO-SI (Tier-II) 27/09/2019 (3 pm - 5 pm) Ans. (b) : In the above sentence 'will be launched' is used is the place of 'will launched'. because the sense of the above sentence is in passive voice so the passive structure will/shall + be + verb3 will be used. Correct sentenceThe author's new novel, which is about social change, will launch soon. 597. In the sentence identify the segment which contains the grammatical error. On the way he was bited on his toe by a poisonous snake. (a) on the way (b) he was bited (c) on his toe (d) by a poisonous snake SSC CGL (Tier-I) – 10/06/2019 (Shift-III) Ans. (b) : In the above sentence The use of ‘he was bitten’ would be appropriate instead of ‘he was bited’. Because sentence is passive voice. In passive voice, the third form of the main verb is used. Spotting Error

Bite − Bit − Bitten P.V.- object − was / were + verb3 + by + subject

Correct sentence :On the way he was bitten on his toe by a poisonous snake. 598. In the sentence identify the segment which contains the grammatical error. Supriya asked Kiran that where had her mother gone when the results of the contest were being declared. (a) that where had her mother gone (b) when the results of the contest (c) Supriya asked Kiran (d) were being declared SSC CGL (Tier-I) – 10/06/2019 (Shift-III) Ans. (a) : In the above sentence ‘Where’ will be used instead of ‘that where’. Because any other conjunction is not used before wh-word. In Interrogative sentence of Indirect speech, ‘Wh-word’ is used as a conjunction. Hence the use of ‘that’ is superfluous and will convert the sentence into assertive. Correct sentence :Supriya asked Kiran where her mother had gone when the results of the contest were being declared. 599. Identify the segment in the sentence, which contains the grammatical error. My father did never have an opportunity to go to a University. (a) an opportunity (b) a University (c) to go to (d) did never have SSC CPO-SI (Tier-II) 27/09/2019 (3 pm - 5 pm) Ans. (d) : Here the correct answer is (d) did not have. In the above sentence the error is related to adverb placement and adverb must come before the auxiliary verb in a sentence. the structure of the sentence is incorrect. Thus 'did never' will be replaced by 'Never did'. 600. In the following, some part of the sentence may have errors. Find out which part of the sentence has an error and select the appropriate option. If a sentence is free from error, select No Error. Service providers cannot stand in the way of a (1)/ consumer's access to content, it should have being provided (2)/ without any hindrance. (3)/ No error. (4) (a) 1 (b) 2 (c) 3 (d) 4 SSC CGL (Phase-II) 17/02/2018 Ans. (b) In the given sentence, 'been' will be used in place of 'being' because the sentence is in passive voice. has/have + been + verb3 Correct sentence'Consumer's access to content. It should have been provided without any hindrance. 601. Identify the segment in the sentence which contains the grammatical error. If there is no error, select 'No error' The letter was posting yesterday by my brother.

87

YCT

(a) (b) (c) (d)

by my brother 605. Identify the segment in the sentence which No error contains the grammatical error. was posting yesterday A soyabean processing unit is going to set up The letter soon in our village. SSC CPO-SI – 11/12/2019 (Shift-I) (a) A soyabean processing unit (b) is going Ans : (c) In the above sentence, ‘Posted’ would be (c) to set up appropriate instead ‘Posting’ because given sentence is (d) soon in our village in passive voice. SSC MTS – 16/08/2019 (Shift-III) Hence the use of third form of verb would be appropriate. Ans. (c) : In the above sentence, ‘to be set up’ would be appropriate instead of ‘to set up’ because according P.V.- object + was / were + verb3 + by + subject to the sense, the sentence will be in passive voice. Correct sentence :Hence to be + verb3 will be used. The letter was posted yesterday by my brother. Correct sentence :602. Identify the segment which contains the A soyabean processing unit is going to be set up soon grammatical error in our village. Pleasure cannot derive from giving pain to 606. In the given sentence identify the segment innocent people. which contains the grammatical error. (a) pleasure cannot derive A Royal Bengal tigress and her three cubs was (b) pain to poison and their carcasses left to rot in the (c) innocent people Wildlife Sanctuary in Goa. (d) from giving (a) A Royal Bengal tigress SSC CHSL (Tier-I) –08/07/2019 (Shift-I) (b) in the Wildlife Sanctuary in Goa Ans : (a) According to the sense of above sentence, ‘be (c) and their carcasses left to rot derived’ will be used instead of ‘derive’. The passive (d) and her three cubs was poison voice structure of the modal ‘can’ is as followsSSC CHSL (Tier-I) 13/10/2020 (Shift-I) can + be + V 3 Ans. (d) : ‘Were poisoned’ will be used instead of Correct sentence‘was poison’ in the above sentence. Because according Pleasure cannot be derived from giving pain to innocent to sense, sentence is in passive voice. Hence the third people. form of main verb will be used and instead of poison, 603. Identify the segment which contains the ‘poisoned’ will be used and subject (A royal Bengal grammatical error. Tigress and her three cubs) is plural, plural verb ‘were’ He asked from me if I was interested in a would be appropriate in place of ‘singular verb ‘was’. career in modeling. Correct Sentence(a) from me if (b) in a career "A Royal Bengal Tigers and her three cubs were (c) I was interested (d) He asked poisoned and their carcasses left to rot in the wildlife SSC CHSL (Tier-I) –08/07/2019 (Shift-II) sanctuary in Goa." Ans : (a) In the above sentence, ‘asked me’ would be 607. Identify the segment in the sentence which appropriate instead of ‘asked from me’, because making contains the grammatical error. indirect. preposition is not used in indirect speech after You are not expect to do the work alone. reporting verb. (told, asked etc) (a) not expect (b) to do Hence ‘from’ will not be used. (c) work alone (d) You are Correct sentence :SSC MTS – 14/08/2019 (Shift-II) He asked me if I was interested in a career in modeling. 604. In the sentence identify the segment which Ans. (a) : In the above sentence, instead of ‘not expect’, ‘not expected’ will be used. Because contains the grammatical error. Three more elevated roads have been propose according to the sense of sentence. it is in passive voice. in the new plan to unclog the traffic in Delhi. Therefore, the following structure will be used for this. (a) in the new plan (b) Three more elevated roads object+is/am/are+not+verb3 +other words (by subj.) (c) to unclog the traffic in Delhi Correct sentence :(d) have been propose SSC CHSL (Tier-I) –01/07/2019 (Shift-III) You are not expected to do the work alone. Ans : (d) In the above sentence ‘Have been proposed’ 608. Identify the segment in the sentence which will be appropriate instead of ‘have been propose’. contains the grammatical error from the given Because ‘given sentence’ is in passive voice. ‘Main options verb’ is always used past participle or third form. Many Indian languages can be reading using the Braille system. object + has / have + been + verb3 + by + subject (a) using (b) many India Correct sentence :(c) can be reading (d) the Braille Three more elevated roads have been proposed in the new plan to unclog the traffic in Delhi. SSC GD – 08/03/2019 (Shift-II)

Spotting Error

88

YCT

Ans. (c) : In the above sentence, ‘The third form of verb ‘read’ will be used in place of ‘reading’, because sentence is in passive sense. Whose structure will be as followsObject + modal + be + verb3 + other words (by + subject)

read - read - read 1st 2nd 3rd Correct sentence :Many Indian languages can be read using the Braille system. 609. In the following question, some part of the sentence may have errors. Find out which part of the sentence has an error and select the appropritate option. If a sentence is free from error, select 'No Error'. It was also claim that the players (a)/ were confused with drastic changes (b)/ in the strategy of every match (c)/ No Error (d) (a) a (b) b (c) c (d) d SSC MTS 11-10-2017 (Shift-II) Ans. (a) : In the given sentence, ‘claimed’ will be used instead of ‘claim’ because the given sentence is in passive structure, in which the 3rd form of the main verb is used with helping verb. (was) Correct Sentence : It was also claimed that the players were confused with drastic changes in the strategy of every match. 610. Identify the segment in the sentence which contains the grammatical error from the given options. The player was declare run-out as he failed to complete the run. (a) run-out as (b) he failed to (c) complete the run (d) the player was declare SSC MTS – 09/08/2019 (Shift-I) Ans. (d) : In the above sentence, ‘the player was declared’ will be used instead of ‘the player was declare’ because sentence is in passive voice. was/were + verb3 Correct sentence : The player was declared run-out as he failed to complete the run. 611. Identify the segment in the sentence which contains a grammatical error. Anybody who does not stop at the red light will fined. (a) will fined (b) at the red light (c) does not stop (d) Anybody who SSC CPO-SI – 25/11/2020 (Shift-I) Ans. (a) : In the above sentence, ‘will be fined’ would be appropriate instead of ‘will fined’ because the sentence is giving the sense of passive voice. Hence Modal + be + verb3 will be used. Correct sentence Anybody who does not stop at the red light will be fined. Spotting Error

612. In the sentence identify the segment which contains a grammatical error. Our twenty-first century society expects all relevant information to simply be handing to us. (a) to simply be (b) expects all relevant information (c) handing to us (d) Out twenty-first century society SSC CHSL (Tier-I) 12/10/2020 (Shift-III) Ans. (c) : In the above sentence, instead of handing to us, ‘handed to us’ would be appropriate. Because the passive form of the infinitive should be used here. Note( to + V1 = inf initive in active voice )  ( to + be + V3 = inf initive inpassive voice ) Correct Sentence : Our twenty-first century society expects all relevant information to simply be handed to us. 613. Find the part of the given sentence that has an error in it. If there is no error, choose 'No error'. The servant was beaten through his master with a stick./No error (a) No error (b) with a stick (c) Through his master (d) The servant was beaten SSC MTS – 05/08/2019 (Shift-I) Ans. (c) : In the above sentence, ‘By his master’ will be used in place of ‘Through his master’ because the sentence is in passive voice and the structure of passive voice is as follow. Active structureSub + VIInd form + Object. Passive structureObject + was/were + VIIIrd from + by + Subject. Correct sentence : The servant was beaten by his master with a stick. 614. Find the part of the given sentence that has an error in it. If there is no error, choose 'No error'. Mukherjee is a famous politician who is love by most people./No error (a) Mukherjee is a (b) famous politician who (c) No error (d) is love by most people SSC MTS – 05/08/2019 (Shift-II) Ans. (d) : In the above sentence ‘Is loved by most people’ will be used instead of ‘Is love by most people’ because the given sentence is in passive voice. Hence– Is/am/are + V3 + by + Object will be used. Correct sentence :'Mukherjee is a famous politician who is loved by most people.' 615. Identify the segment which contains a grammatical error. The old woman in our locality is knowing for her hot temper. (a) is knowing for (b) her hot temper (c) The old woman (d) in our locality SSC CHSL (Tier-I) 20/10/2020 (Shift-III)

89

YCT

Ans. (a) : In the given sentence, ‘is known for’ will be used instead of ‘is knowing for’ because sentence is in passive form. Hence the third form of main verb will be used. Correct Sentence : 'The old woman in our locality is known for her hot temper. 616. Identify the segment in the sentence which contains the grammatical error. I was reading the newspaper when I heard the sound of bullets been fired. (a) when I heard (b) the sound of bullets (c) been fired (d) I was reading the newspaper SSC MTS – 22/08/2019 (Shift-II) Ans. (c) : In the sentence, ‘being fired’ will be used instead of ‘been fired’. Because the second clause is in passive voice. Hence being + verb3 will be used. Correct sentence : I was reading the newspaper when I heard the sound of bullets being fired. 617. Identify the segment in the sentence which contains the grammatical error from the given options. Jaipur, which is renown for its architecture, was founded about 300 years ago. (a) about 300 years ago (b) was founded (c) for its architecture (d) Jaipur, which is renown SSC MTS – 08/08/2019 (Shift-III) SSC MTS – 22/08/2019 (Shift-II) Ans. (a) : In the above sentence, ‘Renowned’ will be used. instead of ‘Renown’, Because the sentence is in passive voice. Hence the use of V3 would be appropriate. Correct sentence : Jaipur, which is renowned for its architecture, was founded about 300 years ago. 618. From the given options identify the segment in the sentence which contains the grammatical error. I said you to prepare the article yesterday but you didn't. (a) the article (b) I said you (c) you didn't (d) to prepare SSC GD – 14/02/2019 (Shift-II) Ans. (b) : ‘I told you’ would be appropriate instead of ‘I said you’ in the above sentence. Because when the sentence of Reported speech is in assertive, then in indirect speech ‘sub + said to + object’ is converted into sub + told + object. Correct sentence :I told you to prepare the article yesterday but you didn't. Spotting Error

K.

Miscellaneous

619. Find the part of the given sentence that has an error in it. If there is no error, choose 'No error'. I particular like that kitten because it has white feet. (a) that kitten because it (b) has white feet (c) No error (d) I particular like SSC MTS 04/05/2023 (Shift-II) Ans. (d) : For the above given options, option (d) has an error because 'I like' will be used in the place of 'I particular like'. because the use of 'particular' is superfluous. The correct sentence- 'I like that kitten because it has white feet. Thus, the correct answer is option (d). 620. Select the option that expresses the given sentence in past tense form. Aksa Thomas comes to attend the session so early. (a) Aksa Thomas came to attend the session so early. (b) Aksa Thomas has come to attend the session so early. (c) Aksa Thomas may come to attend the session so early. (d) Aksa Thomas will come to attend the session so early. SSC GD 23/01/2023 (Shift-IV) Ans. (a) : Among the given options, the sentence in option (a) is in past tense. Formation of simple past tenseSubject + Verb2 + Object. Correct SentenceAksa Thomas came to attend the session so early. 621. Choose the sentence that is in simple past tense: A. I was giving some of the chocolates to my younger sister. B. I had been giving some of the chocolate to my younger sister. C. I had given some of the chocolates to my younger sister. D. I gave some of the chocolates to my younger sister. (a) A (b) D (c) B (d) C SSC GD 12/01/2023 (Shift-II) Ans. (b) : Among the given option, the sentence in option (b) 'I had been giving some of the chocolate to my younger sister' is in simple past tense. Structure of simple past tenseSubject + verb2 + object + other words 622. Select the option that expresses the given sentence in past tense form. The methods Arunima adopts for taming the dogs create awe and wonder among them.

90

YCT

(a) The methods Arunima adopted for taming the dogs create awe and wonder among them. (b) The methods Arunima adopted for taming the dogs created awe and wonder among them. (c) The methods Arunima adopts for taming the dogs created awe and wonder among them. (d) The methods Arunima adopts for taming the dogs will create awe and wonder among them. SSC CGL (Tier-I) 26/07/2023 (Shift-II) Ans. (b) : For the above given sentence, option (b) is in past form, because the word 'adopted' and 'created' are indicated second form of verb (v2). The correct sentence is– The Methods Arunima adopted for taming the dogs created awe and wonder among them. Thus, the correct answer is option (b). 623. Select the grammatically correct version of the following sentence. He was youngest than his brother. (a) He was young to his brother. (b) He was youngest to his brother. (c) He was too young than his brother. (d) He was younger than his brother. SSC CHSL (Tier-I) 17/08/2023 (Shift-II) Ans. (d) : The grammatically correct version of the given sentence, is option (d) ' He was younger than his brother'. Because before 'than' comparative degree will be used. (Younger) Correct sentence He was younger than his brother. 624. Select the grammatically correct sentence. A. I would like to spend an evening near pool with a cup of coffee. B. I would like to spend an evening near pool with cup of coffee. C. I would like to spend a evening near a pool with a cup of coffee. D. I would like to spend an evening near a pool with a cup of coffee. (a) C (b) A (c) B (d) D SSC Selection Posts XI-28/06/2023 (Shift-III) Ans. (d) : In the above sentence option (d) " I would like to spend an evening near a pool with a cup of coffee" is grammatically correct. Other options are not correct. 625. The given sentence has an error. Choose the option that corrects the error Throw a rounder stone to create ripples in the water. (a) Throw a more rounder stone to create ripples in the water. (b) Throw a round stone to create ripples in the water. (c) Throw the roundest stone to create ripples in the water.

Spotting Error

(d) Throw a more round stone to create ripples in the water. SSC CGL Mains 26/10/2023 (Shift-I) Ans. (b) : In the above sentence 'round' will be used in place of 'rounder' because rounder is not used or existence. Hence option (b) will be correct. Correct Sentence Throw a round stone to create ripple is the water. 626. Find the part of the given sentence that has an error in it. If there is no eroor, choose 'No error' . We have more then 6 years of experience in field hockey. (a) then 6 year of (b) We have more (c) experience in field hockey. (d) No error SSC CHSL (Tier-I) 21/03/2023 (Shift-II) Ans. (a) : In the above sentence, 'more than' will be used in place of 'more then'. because 'than' is used for comperision degree. Correct sentence We have more than 6 years of experience in field hockey. 627. Select the option that arranges the given words to form a grammatically correct and meaningful sentence. The students eagerly their awaited results. (a) Eagerly awaited the students their results. (b) The awaited eagerly results their students. (c) The students eagerly awaited their results. (d) The students their eagerly awaited results. SSC CGL Mains 26/10/2023 (Shift-I) Ans. (c) : The above option (c) is grammatically correct because adverb (eagerly) qualify the verb (awaited). Other options are not correct. Correct Sentence The student eayerly awaited their result. 628. Identify the sentence that contains no spelling errors. (a) Games instil in people the values of timeliness, honesty and consistency in their routines. (b) Games instil in people the values of timeliness, honesty and consistensy in their routeenes. (c) Games insteal in people the values of timliness, honesty and consistency in their routines. (d) Games insteal in people the values of timeliness, honesty and consistensy in their routines SSC Selection Posts XI-30/06/2023 (Shift-IV) Ans. (a) : In the above sentences, in option (a) no spelling error. Other options wrongly spelt. Correct SentenceGames instil in people the values of timeliness, honesty and consistency in their routines.

91

YCT

629. Find the part of the given sentence that has an error in it. If there is no error, choose 'No error'. Our hands have been crucial tools for finding and eat food, and helping us navigate the world. (a) tools for finding and eat food, (b) Our hands have been crucial (c) No error (d) and helping us navigate the world. SSC CHSL (Tier-I) 17/03/2023 (Shift-III) Ans. (a) : In the above sentence 'eating' will be used in place of 'eat' because same parts of speech is used with 'and'. Correct Sentence: Our hands have been crucial tools for finding and eating food and helping us navigate the world. 630. Parts of the following sentence have been given as options. Select the option that contains an error. The speaker’s emphasiss is on the present-day problems that are being faced by ordinary men of India. (a) by ordinary men of India (b) that are being faced (c) on the present-day problems (d) The speaker’s emphasiss is SSC CGL Mains 26/10/2023 (Shift-I) Ans. (d) : In the above sentence 'the speakar's emphasis is' will be used in place of 'The speakar's emphasiss is' because 'emphasiss' is wrongly splet its correct spelling is emphasis (force). Correct Sentence The speakar's emphasis is on the present-day problems that are being faced by ordinary men of India. 631. Parts of the following sentence are given as options. Identify the part that contains a grammatical error. If there is no error, select 'No error'. He has been/told many a times to/mind his manners. (a) No error (b) He has been (c) told many a times to (d) mind his manners. SSC CHSL (Tier-I) 10/03/2023 (Shift-III) Ans. (c) : In the above sentence "Many a time" will be used in place of "Many a times" because Singular Noun (time) is used after "many a/an". Correct Sentence– He has been told many times to mind his manners. 632. Identify the segment in the sentence which contains a grammatical error. If there is no error, then select the option "No error". Today, in fit of frenzied rage and / jealousy, you would have / killed me, your brother. (a) killed me, your brother (b) No error (c) jealousy, you would have (d) Today, in fit of frenzied rage and SSC CHSL (Tier-I) 09/02/2023 (Shift-II) Spotting Error

Ans. (d) : In the above sentence, 'frenzied' or 'rage' is used any of them because 'frenzied' and 'rage' have the same meaning so it is 'superflous'. Correct Sentence: Today, in fit of frenzied and jealousy, you would have killed me, your brother. 633. The following sentence has been split into four segments. Identify the segment that contains a grammatical error. The book that/I lost it/is too expensive/for me to buy now. (a) is too expensive (b) The book that (c) for me to buy now (d) I lost it SSC Stenographer (Grade C & D) 11.11.2021 Shift-I Ans. (d) : In the above sentence, ‘I lost’ will be used in place of ‘I lost it’ because the sense of the sentence is present and the use of 'it' is superflous and after last comma (,) will be used the seperate the clause. Correct sentence The book that I lost is too expensive for me to buy now. 634. In the given sentence, identify the segment that contains a grammatical error. The backbone in the human body is also called as the vertebral column. (a) The backbone (b) in the human body (c) as the vertebral column (d) is also called SSC Stenographer (Grade C & D) 11.11.2021 Shift-I Ans. (c) : In the above sentence, the use of ‘as’ is it is superfluous according to the sense of the sentence. Correct sentence The backbone in the human body is also called the vertebral column. 635. The following sentences has been split into four segments. Identify the segment that contains a grammatical error. After/a long and fun-filled day,/the children/slept themselves peacefully. (a) slept themselves peacefully (b) after (c) a long and fun-filled day (d) the children SSC CHSL 24.05.2022 Shift-III Ans. (a) : According to the meaning of the above sentence, the use of themselves is superfluous. Hence it will not be used. Correct SentenceAfter a long and fun-filled day the children slept peacefully. 636. Parts of the following sentence have been given as given as options. One of them may contain an error. Select the part that contains the error from the given options. If you don't find any error, mark 'No error' as you answer. Both of my parents are dependent on my salary of their living.

92

YCT

(a) (b) (c) (d)

are dependent on my Both of my parents salary of their living No error SSC MTS-18/07/2022 Shift-I Ans. (c) : In the above sentence, ‘salary of their livelihood’ will be used in place of ‘salary of their living’, because livelihood is used for earning livelihood as a 'noun'. Correct Sentence – Both of my parents are dependent on my salary of their livelihood. 637. The following sentence has been divided into parts. One of them may contain an error. Select the part that contains the error from the given options. If you don’t find any error, mark ‘No error’ as your answer. The team won/ very easily / at the recent sports event. (a) Very easily (b) at the recent sports event (c) The team won (d) No error SSC CGL (Tier-I) 21/04/2022 Shift-III Ans. (d) : No error

638. The following sentence has been divided into parts. One of them may contain an error. Select the part that contains the error from the given options. If you don’t find any error, mark ‘No error’ as your answer. The unseasonal rain/ caused a lot of damage/ to the crops. (a) No error (b) The unseasonal rain (c) to the crops (d) caused a lot of damage SSC CGL (Tier-I) 12/04/2022 Shift-II Ans. (a) : No error. 639. The following sentence has been split into four segments. Identify the segment that contains a grammatical error. The manager said, "Could you please / confirm me whether you / have received all the items / that you had ordered?" (a) have received all the items (b) that you had ordered (c) The manager said, "Could you please (d) Confirm me whether you SSC CGL (Tier-I) 13/04/2022 Shift-II Ans. (d) : In the given sentence, ‘confirm me whether you’ is grammatically incorrect. ‘confirm whether you’ is correct sentence. Here the use of pronoun ‘me’ is superfluous. Hence it will be removed. 640. The following sentence has been split into four segments. Identify the segment that contains a grammatical error. It is not worth/having the trouble/to write to him/as he never replies. Spotting Error

(a) To write to him (b) having the trouble (c) as he never replies (d) It is not worth SSC CGL (Tier-I) 11/04/2022 Shift-III Ans. (b) : Instead of ‘having the trouble,’ ‘taking the trouble’ would be appropriate in the above sentence, because ‘taking the trouble’ is used as an idiom. 641. The following sentence has been divided into parts. One of them may contain an error. Select the part that contains the error from the given options. If you don't find any error, mark 'No error' as your answer. He got afflicted with the disease / when he was / a teenager. (a) No error (b) He got afflicted with the disease (c) a teenager (d) when he was SSC MTS-15/07/2022 Shift-I Ans. (a) : No error. 642. Parts of the following sentence have been given as options. One of them contains an error. Select the part that contains the error from the given options. Time and technology are two important factor that change substances into resources. (a) Time and technology (b) resources (c) factor that (d) two important SSC MTS-07/07/2022 Shift-I Ans. (c) : In the above sentence, ‘factor in’ will be used instead of ‘factor that’ because ‘in’ is used as a phrasal verb with factor. Note - factor ↔ in (specialist) Correct Sentence – Time and technology are two important factor in change substance into resources. 643. The following sentence has been divided into parts. One of them may contain an error. Select the part that contains the error from the given options. If you don't find any error, mark 'No error' as your answer. Marine vertebrates are / valuable for / many reasons. (a) valuable for (b) No error (c) many reasons (d) Marine vertebrates are SSC MTS-05/07/2022 Shift-II Ans. (b) : No error 644. The following sentence has been divided into parts. One of them may contain an error. Select the part that contains the error from the given options. If you don't find any error, mark 'No error' as your answer. Whatever a writer puts / into a story probably comes with / his own life experience.

93

YCT

(a) (b) (c) (d)

with his own life experience No error into a story probably comes whatever a writer puts SSC MTS-07/07/2022 Shift-II Ans. (a) : In the above sentence ‘With own life experience’ will be used in place of ‘with his own life experience’. Because in the sentence, his is superfluous and the use of ‘with own life’ is appropriate to give a sense of emphasise. Correct Sentence – Whatever a writer put into a story probably comes with own life experience.

645. The following sentence has been divided into parts. One of them may contain an error. Selec the part that contains the error from the given options. If you don't find any error, mark 'No error' as your answer. Young scientists have fresh ideas; ideas; they are ambitious and highly productive. (a) and highly productive (b) Young scientists have fresh ideas (c) No error (d) they are ambitious SSC MTS-06/07/2022 Shift-III Ans. (c) : No error 646. The following sentence has been split into four segments. Identify the segment that contains a grammatical error. Do you play / any other sports / beside of / cricket and football? (a) any other sports (b) cricket and football (c) Do you play (d) beside of SSC Constable GD-29/11/2021 Shift-II Ans. (d) : In the above sentence, ‘besides’ will be used in place of ‘beside of’ because ‘besides’ is used in the sense of except. Correct Sentence Do you play any other sports besides cricket and football ? 647. The following sentence has been split into four segments. Identify the segment that contains a grammatical error. The road was blocked / so we had / to turn back and / find another one route. (a) so we had (b) to turn back and (c) the road was blocked (d) find another one route SSC Constable GD-26/11/2021 Shift-I Ans. (d) : In the given sentence, ‘another’ will be used instead of ‘another one’. Which gives complete sense. The use of ‘one’ is superfluous with it. Correct sentence– The road was blocked so we had to turn back and find another route. Spotting Error

648. The following sentence has been split into four segments. Identify the segment that contains a grammatical error. I will / be in school / for / 7 a.m. to 2 p.m. (a) 7 a.m. to 2 p.m. (b) for (c) be in school (d) I will SSC Constable GD-24/11/2021 Shift-II Ans. (b) : In the above sentence, ‘from’ will be used in place of ‘for’, because according to the sense of the sentence it would be appropriate. From + time + to + time Correct SentenceI will be in school from 7 a.m. to 2 p.m 649. Select the most appropriate option that can substitute the underlined segment in the given sentence. If there is no need to substitute it, select ‘No substitution required’. I recently visited the school when I studied in Class 10. (a) wherever (b) where (c) No substitution required (d) whenever SSC Constable GD-22/11/2021 Shift-II Ans. (b) : In the above sentence, instead of ‘when’ ‘where’ will be used. Because it gives a suitable meaning to the sentences. Other options have different meaning. Correct sentence– I recently visited the school where I studied in class 10. 650. The following sentence has been split into four segments. Identify the segment that contains a grammatical error. It was cold / and / we still / went swimming. (a) and (b) we still (c) went swimming (d) It was cold SSC Constable GD-14/12/2021 Shift-III Ans. (a) : ‘yet’ will be used instead of 'and' in the above sentence because the sense of the sentence is in constrast. So ‘yet’ is giving appropriate meaning. Correct sentence– It was cold yet we still went swimming. 651. In the given sentence, identify the segment which contains a grammatical error. Doctors say that home isolation is far more effective then hospitalise in cases of mild infection. (a) far more effective (b) than hospitalise (c) isolation is (d) Doctors say that SSC CHSL (Tier-I) –09/08/2021 (Shift-I) Ans. (b) : In the above sentence ‘hospitals’ (noun) will be used in place of ‘hospitalise’ (verb) because similar part of speech (Home isolation) is compared. Correct Sentence Doctors say that home isolation is far more effective than hospitals in cases of mild infection.

94

YCT

652. The following sentence has been divided into parts. One of them may contain an error. Select the part that contains the error from the given options. If you don't find any error, mark 'No error' as your answer. The need today is for both sides/to introspect and re-examine the issue carefully/ and resolve it amicably. (a) to introspect and re-examine the issue carefully (b) and resolve it amicably (c) No error (d) The need today is for both sides SSC CHSL (Tier-I) –09/08/2021 (Shift-I) Ans. (c) : No error. 653. The following sentence has been divided into parts. One of them contains an error. Select the part that contains the error from the given options. You must and avoid riding in crowded bus/or travelling in a metro/during rush hour/as both are quiet unpleasant experiences. (a) You must avoid riding in a crowded bus. (b) during rush hour (c) as both are quiet unpleasant experiences (d) or travelling in a metro SSC CGL (Tier-I) –13/08/2021 (Shift-I) Ans. (c) : In the above sentence 'quite' will be used instead of ‘quiet’ because it gives suitable meaning for the sentence. Correct Sentence You must and avoid riding in crowded bus or travelling in a metro during rush hour as both are quite unpleasant experiences. 654. In the following, some part of the sentence may have errors. Find out which part of the sentence has an error and select the appropriate option. If a sentence is free from error, select No Error. The priest class took upon itself the monopoly (1)/ of scriptural knowledge and interpretation (2)/ of the same to its own advantage. (3)/ No error (4) (a) 1 (b) 2 (c) 3 (d) 4 SSC CGL (Phase-II) 17/02/2018 Ans. (a) In the given sentence, ‘itself’ is not used because the structure of the sentence is grammatically correct without 'itself' and it is super flours. Correct sentenceThe priest class took upon the monopoly. 655. In the sentence identify the segment which contains the grammatical error. If the sentence has no error, then select 'No error'. Whom did you give the packet to ? (a) Whom (b) did you give (c) the packet to (d) No error SSC CPO-SI – 12/12/2019 (Shift-II) Ans : (d) No error Spotting Error

656. Identify the segment in the sentence which contains the grammatical error. If there is no error, select 'No error'. Among the reasons cited by the employee for quitting the job were the pressure of work, the outstation duties and desiring to pursue her hobbies. (a) Among the reasons cited by the employee (b) were the pressure of work the outstation duties (c) and desiring to pursue her hobbies (d) No error SSC CPO-SI – 12/12/2019 (Shift-I) Ans : (c) In the above sentence, ‘desire to pursue’ will be used in place of ‘desiring to pursue. Because same part of speech is used with ‘and’. Hence desire (Noun) will be used with duties (Noun). Correct sentence :Among the reasons cited by the employee for quitting the job were the pressure of work, the outstation duties and desire to pursue her hobbies. 657. Find the part of the given sentence that has an error in it. If there is no error, choose 'No error'. My aunt/sold her boutique/ to a billionaire. (a) to a billionaire (b) sold her boutique (c) My aunt (d) No error SSC MTS – 02/08/2019 (Shift-II) Ans. (d) : No error 658. Identify the segment which contains the grammatical error. Great many students who are good at other subjects perform poorly in English. (a) perform poorly (b) in English (c) who are good at (d) Great many students SSC CHSL (Tier-I) –08/07/2019 (Shift-III) Ans : (d) ‘A great many’ will be used instead of ‘Great many’ in the above sentence. Because its use is idiomatics. A great many – ‘large number of things or persons’. A great many/A good many + plural noun Correct Sentence :A Great many students who are good at other subjects perform poorly in English. 659. Identify the segment in the sentence which contains the grammtical error. If there is no error, select 'No error'. If she left now, she would reach the station on time. (a) the station on time (b) she would reach (c) if she left now (d) No error SSC CPO-SI – 12/12/2019 (Shift-I) Ans : (d) No error Note- If + subject + verb2... subject + would + verb1 is used. 660. In the sentence identify the segment which contains the grammatical error. On Saturday Rahul visited his professor to whom he had an appointment. (a) to whom he had (b) Rahul visited his professor (c) an appointment (d) On Saturday SSC CHSL (Tier-I) –04/07/2019 (Shift-I)

95

YCT

Ans : (a) In the above sentence, ‘with whom’ would be appropriate in place of ‘to whom’. Here ‘Whom’ is object of ‘with’ preposition. Which gives proper meaning for the sentence. Correct Sentence :On Saturday Rahul visited his professor with whom he had an appointment. 661. Find the part of the given sentence that has an error in if. If there is no error, choose 'No error'. Their friendship will not last long./No error (a) will not (b) last long (c) No error (d) Their friendship SSC MTS – 02/08/2019 (Shift-III) Ans. (c) : No error 662. In the following question, some part of the sentence may have errors. Find out which part of the sentence has as error and select the appropriate option. If a sentence is free from error, select 'No Error'. He added that his previous employee (1)/ in Lucknow never paid him on time (2)/ or gave him his full dues. (3)/ No Error (4) (a) 1 (b) 2 (c) 3 (d) 4 SSC MTS 9-10-2017 (Shift-II) Ans. (a) : In the above sentence, ‘former employee’ would be appropriate in place of ‘previous employee’. Because it gives appropriate according to the sense of the sentence. Correct Sentence :He added that his former employee in Lucknow never paid him on time or gave him his full dues. 663. Find the part of the given sentence that has an error in it. If there is no error, choose 'No error'. I did not see anybody in the auditorium./No error (a) No error (b) see anybody (c) I did not (d) in the auditorium SSC MTS – 07/08/2019 (Shift-I) Ans. (a) : No error. 664. Find the part of the given sentence that has an error in it. If there is no error, choose 'No error'. It is easier to preach then to follow./No error (a) to preach (b) it is easier (c) then to follow (d) No error SSC MTS – 05/08/2019 (Shift-III) Ans. (c) : In the above sentence, ‘than to follow’ will be used instead of ‘then to follow’, because ‘than’ is used for comparison. Correct Sentence :It is easier to preach than to follow. 665. Find the part of the given sentence that has an error in it. If there is no error, choose 'No error'. The people in my office are smarter than other offices./ No error Spotting Error

No error my office are smarter than other offices The people in SSC MTS – 05/08/2019 (Shift-II) Ans. (c) : In the given sentence, instead of ‘smarter than other offices’, ‘smarter than those of other offices’ will be used, because in the sentence ‘people are compared to people of other offices. Hence ‘those (people) of’ would be used before other offices. Correct Sentence :'The people in my office are smarter than those of other offices.' 666. Find the part of the given sentence that has an error in it. If there is no error, choose 'No error'. Unless you prepare well, you will not get a first class./No error (a) No error (b) a first class (c) Unless you prepare well (d) you will not get SSC MTS – 05/08/2019 (Shift-I) Ans. (a) : No error. 667. Indentify the segment in the sentence which contains the grammatical error. If there is no error, select 'no error' The climate of Dehradun is better than Meerut. (a) is better (b) The climate of Dehradun (c) no error (d) than Meerut SSC MTS – 14/08/2019 (Shift-I) Ans. (d) : In the above sentence, ‘than that of Meerut’ would be appropriate in place of ‘than Meerut’. Because in the sentence, the climate is being compared to not 'Meerut city'. So 'that of' (climate) will be used. Correct Sentence :'The climate of Dehradun is better than that of Meerut'. 668. Identify the segment in the sentence which contains the grammatical error. If no error then select 'No error'. She wanted to bake a cake for both her brothers. (a) No error (b) to bake a cake (c) for both her brothers (d) She wanted SSC MTS – 19/08/2019 (Shift-II) Ans. (a) : No error 669. Find the part of the given sentence that has an error in it. If there is no error, choose 'No error'. The lady/standing in the/corner is fat./No error. (a) corner is fat (b) No error (c) standing in the (d) The lady SSC MTS – 02/08/2019 (Shift-I) Ans. (b) : No error

96

(a) (b) (c) (d)

YCT

02. SENTENCE IMPROVEMENT TCS hewšve& hej DeeOeeefjle (Based On TCS Pattern) Chapterwise 1

Form of verb (Tense, Number, Modal)

2

Subject & Verb agreement

3

Conditional Sentence

4

Question tag

5

Article

Sentence Improvement

Exam

Question No.

CGL (Tier-1) CGL (Tier-2) CHSL (Tier-1) CHSL (Tier-2) Selection Post VII, VIII, XI SSC MTS SSC GD SSC CPO SI CGL (Tier-1) CGL (Tier-2) CHSL (Tier-1) CHSL (Tier-2) Selection Post VII, VIII, XI SSC MTS SSC GD SSC CPO SI CGL (Tier-1) CGL (Tier-2) CHSL (Tier-1) CHSL (Tier-2) Selection Post VII, VIII, XI SSC MTS SSC GD SSC CPO SI CGL (Tier-1) CGL (Tier-2) CHSL (Tier-1) CHSL (Tier-2) Selection Post VII, VIII, XI SSC MTS SSC GD SSC CPO SI CGL (Tier-1)

30 20 40 22 15 35 10 20 15 7 18 5 2 3 1 3 7 3 6 2 1 6 2 5 7 2 5 2 1 4 1 2 2

CGL (Tier-2)

1

CHSL (Tier-1)

2

CHSL (Tier-2)

1

Selection Post VII, VIII, XI

-

SSC MTS

1

SSC GD

-

SSC CPO SI

3

97

Years

(2017–2023)

(2017–2023)

(2017–2023)

(2017–2023)

(2017–2023)

YCT

6

Correct use of Pronoun, Adjective & Adverb

7

Conjunction

8

Preposition

9

Voice and Narration

10

Noun (Case, Gender, Number)

11

Miscellaneous

Sentence Improvement

CGL (Tier-1) CGL (Tier-2) CHSL (Tier-1) CHSL (Tier-2) Selection Post VII, VIII, XI SSC MTS SSC GD SSC CPO SI CGL (Tier-1) CGL (Tier-2) CHSL (Tier-1) CHSL (Tier-2) Selection Post VII, VIII, XI SSC MTS SSC GD SSC CPO SI CGL (Tier-1) CGL (Tier-2) CHSL (Tier-1) CHSL (Tier-2) Selection Post VII, VIII, XI SSC MTS SSC GD SSC CPO SI CGL (Tier-1) CGL (Tier-2) CHSL (Tier-1) CHSL (Tier-2) Selection Post VII, VIII, XI SSC MTS SSC GD SSC CPO SI CGL (Tier-1) CGL (Tier-2) CHSL (Tier-1) CHSL (Tier-2)2 Selection Post VII, VIII, XI SSC MTS SSC GD SSC CPO SI CGL (Tier-1) CGL (Tier-2) CHSL (Tier-1) CHSL (Tier-2) Selection Post VII, VIII, XI SSC MTS SSC GD SSC CPO SI 98

22 8 5 25 5 15 1 9 11 3 6 2 1 5 2 10 25 10 5 30 5 10 3 5 4 2 8 2 4 5 1 2 3 2 5 2 1 3 2 3 35 20 19 25 11 35 6 30

(2017–2023)

(2017–2023)

(2017–2023)

(2017–2023)

(2017–2023)

(2017–2023)

YCT

Trend Analysis of Questions topicwise from CGL (Pre & Mains) CHSL (Pre & Mains) Selection Post VII, VIII, XI, SSC MTS, SSC GD & Other Exams (2017-2023)

Sentence Improvement

99

YCT

02. SENTENCE IMPROVEMENT A. Form of Verb (Tense, Number, Modal) 1.

Improve the underlined part of the sentence. Choose 'No improvement' as an answer if the sentence is grammatically correct. Don't try to force the window open, or you might broken it. (a) you might breaks (b) you have break (c) No improvement (d) you might break SSC MTS 04/05/2023 (Shift-II) Ans. (d) : For the above given underlined part of the sentence will be removed by 'you might break' because modal verb always takes basic form of verb. (Modal + V1). Thus, the correct answer is option (d). 2. Improve the underlined part of the sentence. Choose 'No improvement' as an answer if the sentence is grammatically correct. The day are warm, so the student ate their lunch outside. (a) was warm (b) was warming (c) No improvement (d) is warm SSC MTS 04/05/2023 (Shift-II) Ans. (a) : The most appropriate option for the underlined part of the sentence (a) 'was warm' will be used because word 'ate' indicates that the sentence is in past tense. Thus, the correct answer is option (a). 3. Select the option that will improve the (bracketed) part of the sentence. In case of no improvement select the option 'No improvement'. When you ride a horse, (you sat on it and control it's) movements. (a) you sit on it and afterwards control it's (b) you seated on it and control it's (c) you sit on it and control its (d) No improvement SSC CHSL (Tier-I) 09/03/2023 (Shift-IV) Ans. (c) : For the above sentence, 'sit' (V1) will be used in place of 'sat' (V2) because the use of 'ride' (V1) makes it clear that the sentence is in Present Indefinite tense. Hence 'Verb1' is used after the subject you. Correct Sentence When you ride a horse, you sit on it and control its movements. 4. Improve the underlined part of the sentence. Choose 'No improvement' as an answer if the sentence is grammatically correct. That movie is so sad; the ending always makes me bawling. Sentence Improvement

(a) made me bawl (b) No improvement (c) makes me bawl (d) makes me bawled SSC MTS 02/05/2023 (Shift-I) Ans. (c) : The most appropriate option for the underlined part of the sentence is (c) 'makes me bawl' because verb1 + ing is not used after (verb)'make'. make + object + noun complement Thus, the correct sentence is'That movie is so sad; the ending always makes me bawl. 5. Improve the underlined part of the sentence. Choose ‘No improvement’ as an answer if the sentence is grammatically correct. She blew out the candle and gone to sleepy. (a) go to sleep (b) No improvement (c) went to sleeps (d) went to sleep SSC MTS 18/05/2023 (Shift-I) Ans. (d) : The underlined part of the sentence – ‘gone to sleepy’ is grammatically incorrect because the sentence is in simple past tense and (VII) ‘went’ is used in the place of ‘gone’. ⇒ (to + V1) is used as an infinitive in the place of (to sleepy). Hence, the correct option is (d) ‘went to sleep’. 6. Improve the underlined part of the sentence. Choose ‘No improvement’ as an answer if the sentence is grammatically correct. Though a foreigner, Mother Teresa settle in Calcutta and made it her home. (a) settled in Calcutta and (b) No improvement (c) settle over Calcutta as (d) settling in Calcutta or SSC MTS 16/05/2023 (Shift-II) Ans. (a) : The above underlined part of the sentence'settle in Calcutta and' is grammatically error, because the given sentence is in past tense, Verb III (settled) will be used in the place of (V1) settle. The correct part is- 'settled in Calcutta and' 7. Select the word segment that substitutes (replaces) the bracketed word segment correctly and completes the sentence meaningfully. Select the option 'no correction required' if the sentence is correct as given. The (roof collapsing) in a roar of rock and rubble. (a) roof collapsed (b) roof was collapse (c) No correction required (d) roof have collapsed SSC MTS 12/05/2023 (Shift-III)

100

YCT

Ans. (a) : The incorrect segment of the given sentence is (roof collapsing). The correct segment is option (a) 'roof collapsed' because the given sentence is in simple past tense, the verb (V2) 'collapsed' is used with the subject 'The roof '. 8. Select the most appropriate option to substitute the underlined segment in the given sentence. The manager have get hot under the collar when the workers demanded higher wages. (a) Had get under the collar (b) Got hot under the collar (c) Get hot under the collar (d) Will got under the collar SSC GD 16/01/2023 (Shift-III) Ans. (b) : Option (b) 'Got hot under the collar' will be used in the underlined part of the given sentence because the phrase 'have get hot under the collar' is grammatically incorrect. The correct phrase should be "got hot under the collar" which means the manager became angry. Correct SentenceThe manager got hot under the collar when the workers demanded higher wages. 9. Improve the underlined part of the sentence. Choose ‘No improvement’ as an answer if the sentence is grammatically correct. She works for eight hours every weekday. (a) No improvement (b) for eighth hours (c) of eighth hour (d) for eight hourly SSC MTS 16/05/2023 (Shift-II) Ans. (a) : No improvement. The underlined part of the above sentence- 'for eight hours' is grammatically correct because the sentence is in simple present and shows the habitual action. 10. Select the most appropriate option that can substitute the underlined segment I the given sentence. Shubhda has not finished to paint the window. (a) paining with (b) painting on (c) painting (d) painting of SSC GD 13/02/2023 (Shift-I) Ans. (c) : The underlined segment of the above sentence, 'painting' would be appropriate in place of 'to paint'. because Verb1 + ing is followed by verb 'finish'. The other options give different meaning. Hence options (c) 'painting' is correct answer. 11. Select the most appropriate options that can substitute the underlined segment in the given sentence. Don't sit on this bench as it is just painted. (a) It was painted just (b) It had been painted just (c) It had painted just (d) It has just been painted SSC GD 06/02/2023 (Shift-III) Ans. (d) : According to the sense of the above sentence, 'It has just been painted' will be used in place of 'it is just painted'. Hence options (d) is correct answer. Sentence Improvement

12.

Select the most appropriate option that can substitute the underlined segment in the given sentence. If there is no need to substitute it, select ‘No substitution’. My friend is owing an expensive cell phone. (a) own an expensive cell phone (b) No substitution (c) is own an expensive cell phone (d) owns an expensive cell phone. SSC CGL (Tier-I) 21/07/2023 (Shift-II) Ans. (d) : The most appropriate substitute for the underlined segment in the given sentence 'is owing an expensive cell phone' is option (d) 'owns an expensive cell phone,' because the sentence is in a simple present tense and the singular verb (owns) is used in the place of 'is owing.' Correct SentenceMy friend owns an expensive cell phone. 13. Select the most appropriate option to substitute the underlined segment in the given sentence. I don’t wear the sunglasses at the moment. (a) I didn’t wear (b) I was wearing (c) I wore (d) I’m not wearing SSC CGL (Tier-I) 18/07/2023 (Shift-III) Ans. (d) : The most appropriate substitute of the above underlined word 'don't wear' is 'I'm not wearing' because the phrase 'at the moment' is used in present continuous tense. Hence, the correct option is (d). Correct SentenceI am not wearing the sunglasses at the moment. 14. Select the most appropriate option that can substitute the underlined segment in the given sentence. If there is no need to substitute it, select 'No substitution'. Delegates of the conflicting countries formally meet at a conference to ironing out their differences. (a) To ironed (b) No substitution (c) To iron out (d) For iron in SSC CGL (Tier-I) 24/07/2023 (Shift-III) Ans. (c) : In the above sentence 'to iron out' is used as an infinitive (to + verb1) in the place of 'to ironing out'. Other options give different meanings. Correct SentenceDelegates of the conflicting countries formally meet at a conference to iron out their differences. 15. Select the most appropriate option that can substitute the underlined words in the given sentence. No sooner did we arrieved at the airport than we got onto our flight to Atlanta. (a) have we arrived at (b) can we arrive by (c) had we arrive at (d) did we arrive at SSC CHSL (Tier-I) 14/08/2023 (Shift-IV) Ans. (d) : The underlined words in the given sentence 'did we arrived at' will be substituted by option (d) 'did we arrive at' Because conjunction 'No sooner than' has been used here, in the sentence of past tense, where did is used followed by the first form of verb.

101

YCT

Structure NO sooner + did + subject + V1 + than...... It can used only in sentence in which two actions place. Correct Sentence No sooner did we arrive at the airport than we got onto our flight to Atlanta. 16. Select the most appropriate option that can substitute the underlined words in the following sentence. It is raining a lot in April and May, but the summer is very dry. (a) was raining a lot on (b) rains a lot in (c) rain a lot at (d) will rains a lot of SSC CHSL (Tier-I) 03/08/2023 (Shift-II) Ans. (b) : The most appropriate substitute for the above underlined words 'is raining a lot in' is option (b) 'rains a lot in '. Because simple present tense is used to describe an action that is regular, true or general. Correct sentence – It rains a lot in April and May, but the summer is very dry. 17. Choose the best substitute for the bracketed segment in the following sentence to complete the sentence meaningfully. (No sooner did she shouted), than the thieves ran away leaving all the jewellery on the table. (a) No sooner had she shout (b) No sooner did she shout (c) No sooner does she shout (d) No sooner do she shouts SSC CHSL (Tier-I) 03/08/2023 (Shift-II) Ans. (b) : The best substitute for the above bracketed Segment (No sooner did she shouted) is option (b) 'No sooner did she shout', because 'inversion' is used in the beginning of the sentence and (V1) is used after helping verb (did). No sooner + did + sub. + V1 + other words 18. Improve the underlined part of the sentence. Choose 'No improvement' as an answer if the sentence is grammatically correct. I won't bore you with the details, but I finally finish my taxes. (a) No improvement (b) finally finished (c) final finishing (d) finally finishing SSC CHSL (Tier-I) 17/03/2023 (Shift-III) Ans. (b) : The sense of given sentence is in Past Tense. Hence 'finally finished' will be used in place of 'Finally finish'. Correct Sentence - I won't bore you with the details, but I finally finished my taxes. 19. Improve the underlined part of the sentence. Choose 'No improvement' as an answer if the sentence is grammatically correct. I try to travelling lightly, packing only the bare essentials. (a) No improvement (b) travelled light (c) travels lightly (d) travel lightly SSC CHSL (Tier-I) 15/03/2023 (Shift-I) Ans. (d) : In the above underlined part of the sentence ''travel'' will be used in place of 'travelling' because V1 (First form of verb) is used after Infinitive 'to'. Note- Lightly (adverb) is used for to modify verb ''try''. Correct sentence- I try to travel lightly Packing only the bare essentials. Sentence Improvement

20.

Improve the underlined part of the sentence. Choose 'No improvement' as an answer if the sentence is grammatically correct. He sharpened his pencil, but the lead broken as soon as he started writing. (a) No improvement (b) lead broke (c) lead breaking (d) led break SSC CHSL (Tier-I) 09/03/2023 (Shift-IV) Ans. (b) : For the above underlined part of the sentence 'broken' (V2) will be used in place of 'broke' (V3) makes it clear that first clause of sentence is in Past Indefinite tense. Hence second Clause also will be in Past Indefinite tense so 'broke' (V2) will be used after Noun 'the lead'. Correct Sentence : he sharpened his pencil, but the lead broke as soon as he started writing. 21. Improve the underlined part of the sentence. Choose 'No improvement' as an answer if the sentence is grammatically correct. The Government has been building more schools to provided education at the secondary level. (a) provide education at (b) provided educationally on (c) providing education to (d) No improvement SSC CHSL (Tier-I) 09/02/2023 (Shift-II) Ans. (a) : For the above sentence 'provide' (V1) will be used in the place of underlined part "Provided" because 'V1' (first form of verb) is used after infinitive "to" which work as "noun". Correct Sentence : The Government has been building more schools to provide education at the secondary level. 22. Select the most appropriate option that can substitute the underlined segment in the given sentence. If there is no need to substitute it, select ‘No substitution required’. None of the boys was willing to admit that he was in the wrong. (a) admit (b) ready to admitting (c) No substitution required (d) willing to admitting SSC CGL (Tier-I) 13/04/2022 Shift-III Ans. (c) : No substitution required In past continuous tense was/were + V1 + ing form is required so there is no mistake in the sentence. 23. Select the most appropriate option to substitute the underlined segment in the given sentence. If no substitution is required, select 'No substitution'. The markets will be slow to recovers from the current recession. (a) to recover (b) to recovered (c) No substitution (d) to recovering SSC Stenographer (Grade C & D) 12.11.2021 Shift-I

102

YCT

Ans. (a) : In the given sentence underlined part, 'to recovers' will be replaced by 'to recover' because base form of verb (V1 ) is used after infinitive 'to' Correct Sentence : The markets will be slow to recover from the current recession. 24. Select the option that will improve the underlined part of the sentence. In case no improvement is needed, select 'No improvement'. The scientists are busy to explore new ideas. (a) exploring (b) No improvement (c) for exploring (d) to be exploring SSC CGL (Tier-II) 08.08.2022 Shift-II Ans. (a) : In the above sentence, 'Exploring' will be used in the place of underline part 'to explore' because gerund form (V1 + ing) is used after 'Busy'. Correct sentence– The scientists are busy exploring new ideas. 25. Select the most appropriate option to improve the underlined segment in the given sentence. If there is no need to improve it, 'No improvement required'. Whoever performs exceptionally well he will be promoted immediately. (a) No improvement required (b) he should be (c) he would be (d) will be SSC Stenographer (Grade C & D) 11.11.2021 Shift-II Ans. (d) : In the above sentence, 'he will be' will be replaced by 'will be' because 'whoever' is used as subject. Hence subject 'He' will not be used again as a subject. Correct SentenceWhoever perform exceptionally well will be promoted immediately. 26. Select the most appropriate option to substitute the underlined segment in the given sentence. If no substitution is required, select 'No substitution'. Does this plant be kept indoors? (a) Did (b) Can (c) Do (d) No substitution SSC Stenographer (Grade C & D) 11.11.2021 Shift-II Ans. (b) : In the above underline part 'Can' will be used in place of 'Does' because modal 'Can' is used for the sense of possibility/capability Correct SentenceCan this plant be kept indoors. 27. Select the most appropriate option to substitute the underlined segment in the given sentence. If no substitution is required, select 'No substitution'. By withhold the officer's promotion, the department caused her a great deal of frustration. (a) To withholding (b) In withhold (c) By withholding (d) No substitution SSC Stenographer (Grade C & D) 11.11.2021 Shift-II Sentence Improvement

Ans. (c) : In the given underline part, 'By withholding' will be used in place of 'By withhold' because 'V1 + ing' form is always used after Preposition. Hence option (c) is correct. Correct Sentence : By withholding the officer's promotion the department caused her a great deal of frustation. 28. Select the most appropriate option to substitute the underlined segment in the given sentence. If there is no need to substitute it, select 'No substitution'. What happened to your family if you continue to splurge money like this? (a) is happening (b) will happen (c) has happened (d) No substitution SSC Stenographer (Grade C & D) 12.11.2021 Shift-II Ans. (b) : : In the above sentence, 'will happen' will be used in place of 'happened' because 'what + helping verb + main verb' is used in interrogative sentence. Correct sentence–What will happen to your family if you continue to splurge money like this? 29. Select the most appropriate option to substitute the underlined segment in the given sentence. If no substitution is required, select 'No substitution'. Online crimes are on the increase so everyone needing to be careful and alert. (a) needs to be careful (b) No substitution (c) need to be careful (d) need to have careful SSC Stenographer (Grade C & D) 12.11.2021 Shift-II Ans. (a) : In the above sentence, 'needs to be careful' will be used in place of underlined part 'needing to be careful' because subject is singular. Hence singular verb will be used. Correct sentence– Online crimes are on the increase so everyone needs to be careful and alert. 30. Select the most appropriate option to substitute the underlined segment in the given sentence. If no substitution is required, select 'No substitution'. Two stadiums in the city are set to reopened after a gap of 65 days. (a) to be reopens (b) for reopen (c) No substitution (d) to reopen SSC Stenographer (Grade C & D) 15.11.2021 Shift-II

Ans. (d) : In the given sentence, 'to reopen' will be used in place of underlined part 'to reopened' because 'to + V1' (Base form of verb) is used in infinitive. The verb forms of Reopen infinitive Present Past tense Past participle Participle Reopen reopening reopened reopened Correct sentence Two stadiums in the city are set to reopen after a gap of 65 days. 31. Select the most appropriate option to substitute the underlined segment in the given sentence. If no substitution is required, select 'No substitution required'. The cyclone have worsen the situation in Mumbai.

103

YCT

(a) (b) (c) (d)

No substitution required has worsening have worsened has worsened

SSC Stenographer (Grade C & D) 15.11.2021 Shift-II

Ans. (d) : In the above sentence, 'has worsened' will be used in place of underlined part 'have worsen' because 'The cyclone' is singular subject. Hence singular verb 'has + V3' will be used. The verb for of worsen:Present Past infinitive Past tense participle Participle worsen worsening worsened worsened Correct SentenceThe Cyclone has worsened the situation in Mumbai. 32. Select the most appropriate option to substitute the underlined segment in the given sentence. If there is no need to substitute it, select 'No substitution'. My father promised to be bringing a new mobile phone for me. (a) for bringing (b) No substitution (c) to bring (d) to have brought SSC Stenographer (Grade C & D) 15.11.2021 Shift-II Ans. (c) : In the above sentence, 'to bring' will be used in place of underlined part 'to be bringing' because base form of verb (V1) is used after infinitive 'to' Correct Sentence– My father promised to bring a new mobile phone for me. 33. Select the most appropriate option to substitute the underlined segment in the given sentence. If there is no need to substitute it, select 'No substitution'. The driver loss control as soon as he was taking a right turn. (a) No substitution (b) lose control where (c) lost control when (d) was losing control after SSC Stenographer (Grade C & D) 15.11.2021 Shift-I Ans. (c) : In the above sentence, 'lost control when' will be used in place of underlined part 'loss control as soon as' because sentence is in past tense. Correct Sentence : The driver lost control when he was taking a right turn. 34. Select the most appropriate option to substitute the underlined segment in the given sentence. If no substitution is required, select 'No substitution required'. Ever since my daughter started math tuition, she is done quite better in the subject. (a) No substitution required (b) had done quite good (c) has been doing quite well (d) has done quite best SSC Stenographer (Grade C & D) 15.11.2021 Shift-I Sentence Improvement

Ans. (c) : In the given sentence, 'has been doing quite well' will be used in place of underlined part 'is done quite better' because second clause of the sentence will be in Present Perfect Continuous tense. Correct Sentence : Ever since my doughter started math tution she has been doing quite well in the subject. 35. Select the most appropriate option to substitute the underlined segment in the given sentence. If there is no need to substitute it, select 'No substitution'. I am sorry for all which happen yesterday. (a) all that happen (b) all what happens (c) No substitution (d) all that happened SSC Stenographer (Grade C & D) 15.11.2021 Shift-I Ans. (d) : In the given underlined part, 'all that happened' will be used in place of underlined part 'all which happen' because use of 'yesterday' makes it clear that the sentence is in past tense. Correct Sentence : I am sorry for all that happend yesterday. 36. Select the option that will improve the underlined part of the given sentence. The children opens the door silently, yesterday. (a) Will have opened the door (b) opened the door (c) open the door (d) will open the door SSC CHSL 31.05.2022 Shift-III Ans. (b) : In the above sentence 'opened' will be used in place of underlined part 'opens' because use of 'yesterday' makes it clear that the sentence is in past tense. Correct SentenceThe children opened the door silently, yesterday. 37. Select the most appropriate option that can substitute the underlined segment in the given sentence. If there is no need to substitute it, select ‘No substitution required’. The teacher said, “the sun raises in the east every day”. (a) No substitution required (b) the sun will rise in the east (c) the sun rises at the east (d) the sun rises in the east SSC CHSL 26.05.2022 Shift-II Ans. (d) : In the above sentence, 'rises' will be used in place of underlined part 'raises' because it gives suitable meaning according to the sense of the sentence. Correct sentence– The Teacher said, "The sun rises in the east every day. 38. Select the option that will improve the underlined part of the sentence. In case no improvement is needed, select 'No improvement required'. He blew out the candle and went to sleepy. (a) went to sleep (b) go on sleep (c) No improvement required (d) gone to sleep SSC MTS-18/07/2022 Shift-I

104

YCT

Ans. (a) : In the given sentence 'went to sleep' will be used in place of underlined part 'went to sleepy' because first form of verb (V1) is used after infinitive 'to'. Correct Sentence– He blew out the candle and went to sleep. 39. Select the option that will improve the underlined part of the given sentence. In case no improvement is needed, select 'No improvement required'. Next month, she was going to the institute to study for a master's degree. (a) went to (b) has gone to (c) is going to (d) No improvement required SSC MTS-18/07/2022 Shift-I Ans. (c) : In the above sentence, 'is going to' will be used in place of underlined part 'was going to' because 'is going to' is used for the sense of immediate future planning. Correct Sentence– Next month, she is going to the institute to study for a master's degree. 40. Select the option that will improve the underlined part of the given sentence. In case no improvement is needed, select ‘No improvement required’. I don’t fancy to go out this evening. (a) going out (b) to be going out (c) No improvement required (d) to going outside SSC CGL (Tier-I) 21/04/2022 Shift-III Ans. (a) : In the above sentence, 'going out' will be used in place of underlined part 'to go out' because gerund form (v1 + ing) is used after 'Fancy'. Correct Sentence– I don't fancy going out this evening 41. Select the most appropriate option that can substitute the underlined segment in the given sentence. If there is no need to substitute it, select ‘No substitution required’. As it was getting late Lata leave the party midway. (a) left the party (b) Leaving the party (c) is left the party (d) No substitution required SSC CGL (Tier-I) 21/04/2022 Shift-III Ans. (a) : In the given sentence, 'left the party' will be used in place of 'leave the party' because first clause of the sentence is in past tense. Hence, second clause will also be in past tense. 42. Select the most appropriate option that can substitute the underlined segment in the given sentence. If there is no need to substitute it, select ‘No substitution required’. Sentence Improvement

The first meeting of the Standing Committee will be hold next week. (a) No substitution required (b) Will be held next week (c) Will be holding next week (d) Will held next week SSC CGL (Tier-I) 11/04/2022 Shift-II Ans. (b) : In the above sentence, 'will be held next week' will be used in place of 'will be hold next week' because III rd form of verb (V3) is used after 'will be'. Correct Sentence– The first meeting of the standing committee will be held next week. 43. Select the most appropriate option that can substitute the underlined segment in the given sentence. If there is no need to substitute it, select ‘No substitution required’. When Sirisha knock at the door someone peeped through the window (a) knocked door (b) knocked on door (c) knocked at the door (d) No substitution requied SSC CGL (Tier-I) 13/04/2022 Shift-III Ans. (c) : In the given sentence, 'knocked at the door' will be used in place of 'knock at the door' because sentence is in past tense. Correct SentenceWhen Sirisha Knocked at the door someone peeped through the window. 44. Select the most appropriate option that can substitute the underlined segment in the given sentence. If there is no need to substitute it, select 'No substitution required'. Computer crime have been in the news since the nineteen eighties. (a) has been on the news since (b) has been in the news since (c) has been in the news from (d) No substitution required SSC CGL (Tier-I) 11/04/2022 Shift-III Ans. (b) : In the above sentence, 'has been in the news since' will be used in place of 'have been in the news since' because subject is singular. Hence singular verb will be used. Correct sentence Computer crime has been in the news since the nineteen eighties. 45. Select the option that will improve the underlined part of the sentence. In case no improvement is needed, select 'No improvement required'. Why doesn't you try calling your family again? (a) you don't tries (b) doesn't you tries (c) don't you try (d) No improvement required SSC MTS-06/07/2022 Shift-I

105

YCT

Ans. (c) : In the given sentence, 'don't you try' will be 49. used in place of 'doesn't you try' because subject (you) is in plural number' Hence, plural helping verb 'do' will be used. Correct Sentence – Why don't you try calling your family again? 46.

Select the option that will improve the underlined part of the given sentence. In case no improvement is needed, select 'No improvement required'. It has been my great misfortune to follow a speech more notable for its volume than for any sense. (a) following a speech most notable (b) No improvement required (c) to follow a speech much notable (d) to have being following a speech more notable SSC MTS-07/07/2022 Shift-I Ans. (a) : In the above sentence, 'following a speech most notable' will be used because sentence is in present perfect continuous and 'has/have + been + Ving' is used in it. Correct Sentence – It has been my great misfortune following a speech most notable for its volume than for any sense.

47.

Select the option that will improve the underlined part of the sentence, in case no improvement is needed, select 'No improvement required'. Shall you pass me that pasta bowl? (a) Shall pass you (b) Can you pass (c) Could passing you (d) No improvement required SSC MTS-07/07/2022 Shift-I Ans. (b) : In the above sentence, 'can you pass' will be used in place of 'Shall you pass' because modal 'Can' is used for polite request. Correct Sentence – Can you pass me that pasta bowl? 48. Select the option that will improve the underlined part of the given sentence. Incase no improvement is needed, select 'No improvement required'. I am trying to finding an investor from abroad. (a) trying too find (b) trying to find (c) No improvement required (d) try to find SSC MTS-05/07/2022 Shift-III Ans. (b) : In the given sentence, 'trying to find' will be used in place of 'trying to finding' because 'to +V1' is used as a infinitive Correct sentence – I am trying to find an investor from abroad. Sentence Improvement

Select the option that will improve the underlined part of the given sentence. In case no improvement is needed, select 'No improvement required'. The snowstorm in that region will effect thousands of travellers. (a) would affecting thousand (b) will effect thousands (c) No improvement required (d) will affect thousands SSC MTS-06/07/2022 Shift-II Ans. (d) : In the given underlined part, 'will affect thousands' will be used in place of 'will effect thousands' because main verb (affect) is used after auxiliary verb (will), as 'effect' is noun form so verb form 'affect' will be used . Correct Sentence– The snowstorm in that region will affect thousands of travellers. 50. Select the option that will improve the underlined part of the given sentence. In case no improvement is needed, select 'No improvement required'. In India, processed foods becomes popular in the 20th century. (a) became popular in (b) become popularity in (c) No improvement required (d) become popular at SSC MTS-13/07/2022 Shift-I Ans. (a) : In the given sentence, 'became popular in' will be used in place of 'becomes popular in' because '20th century' makes it clear that the sentence is in past tense. Hence 'became' will be used. Correct Sentence – In India, processed foods became popular in the 20th century. 51. Select the option that will improve the underlined part of the given sentence. In case no improvement is needed, select 'No improvement required'. I work hardly too earn bread and butter for my family. (a) work hardest too (b) work hard to (c) No improvement required (d) works hardly to SSC MTS-12/07/2022 Shift-I Ans. (b) : In the above underlined part, 'work hard to' will be used in place of 'work hardly too' because adverb 'hard' makes meaningful instead of 'hardly' and 'to + V1' is used after 'hard' Note :– Hard to do something Correct Sentence – I work hard to earn bread and butter for my family. 52. Select the option that will improve the underlined part of the sentence. In case no improvement is needed, select 'No improvement required'. He sounded like he were to going cry.

106

YCT

(a) (b) (c) (d)

No improvement required were going to going to was going to SSC MTS-14/07/2022 Shift-III Ans. (d) : In the given sentence, 'was going to cry' will be used in place of 'were to going cry' because singular verb (was) is used for singular subject (He). Correct Sentence – He sounded like he was going to cry 53. Select the option that will improve the underlined part of the given sentence. In case no improvement is needed, select 'No improvement required'. Aphrodite vowed that Paris would had the most beautiful woman as his wife. (a) would be having (b) No improvement required (c) will be having (d) would have SSC Constable GD-30/11/2021 Shift-III Ans. (d) : In the above underlined part, ‘would have’ will be used in place of ‘would had’ because ‘have’ is used after would, should, could, etc. Correct Sentence Aphrodite vowed that Paris would have the most beautiful woman as his wife. 54.

Select the most appropriate option that can substitute the underlined segment in the given sentence. If there is no need to substitute it, select ‘No substitution required’. “The players needing a break and we think it’s time for them to rest,” the coach told the reporters. (a) No substitution required (b) need a break (c) needs a break (d) are need a break SSC Constable GD-26/11/2021 Shift-I Ans. (b) : In the given sentence, ‘need’ will be used in place of ‘needing’ because sentence is in present indefinite. And subject (players) is plural. Hence, Plural verb (without s/es) will be used. Correct Sentence– The players need a break and we think it's time for them to rest. the coach told the reporters. 55. Select the option that will improve the underlined part of the given sentence. In case no improvement is needed, select 'No improvement required'. She just completed the work before her friend came. (a) just has completed the work (b) had just completed the work (c) has just completed the work (d) No improvement required SSC Constable GD-26/11/2021 Shift-I Sentence Improvement

Ans. (b) In the given sentence, ‘had just completed’ will be used in place of ‘just completed’ because in past perfect tense, the first completed and in past indefinite tense, the second completed work is used. Correct Sentence She had just completed the work before her friend came. 56. Select the most appropriate option that can substitute the underlined segment in the given sentence. If there is no need to substitute it, select ‘No substitution required’. We watched the carpenter to repair our broken table. (a) to repairing (b) No substitution required (c) repairing (d) repaired SSC Constable GD-23/11/2021 Shift-III Ans. (c) : In the given sentence, ‘repairing’ will be used in place of ‘to repair’ because V1 + ing form is used after ‘watch’. Correct Sentence - We watched the carpenter repairing our broken table. 57. Select the most appropriate option that can substitute the underlined segment in the given sentence. If there is no need to substitute it, select ‘No substitution required’. Don’t disturb me while I talk to someone on the phone. (a) was talking (b) am talking (c) No substitution required (d) talked SSC Constable GD-18/11/2021 Shift-I Ans. (b) : In the given sentence, ‘am taking’ will be used in place of ‘talk’ because continuous form is always used after conjunction ‘while’. Correct sentence– Don't disturb me while I am talking to someone on the phone. 58. Select the most appropriate option that can substitute the underlined segment in the given sentence. If there is no need to substitute it, select 'No substitution required'. Using a word processor for writing have a number of advantages. (a) to write have a number (b) for writing has a number (c) for writing having number (d) No substitution required SSC Constable GD-16/11/2021 Shift-III Ans. (b) : In the above sentence, ‘for writing has a number will be used in place of ‘for writing have a number’ because subject is singular. Hence singular verb ‘has’ will be used. Correct Sentence Using a word processor for writing has a number of advantages. 59. Select the most appropriate option that can substitute the underlined segment in the given sentence. If there is no need to substitute it, select ‘No substitution required’. People in North India doesn’t know much about South Indian culture.

107

YCT

(a) (b) (c) (d)

are not knowing doesn’t knows No substitution required don’t know SSC Constable GD-14/12/2021 Shift-III Ans. (d) : In the above sentence, ‘Don’t know’ will be used in place of ‘doesn’t know’ because ‘people’ is plural subject. Hence plural verb will be used. Correct sentence– People in north India don't know much about South Indian culture. 60. Select the option that will improve the underlined part of the sentence. In case no improvement is needed, select 'No improvement required'. My father is wishing me to give all my time to studies now. (a) No improvement required (b) has wished (c) wishes (d) was wishing SSC Constable GD-14/12/2021 Shift-II Ans. (c) : In the above sentence, ‘wishes’ will be used in place of ‘is wishing’ because sentence is in simple present. Correct sentence - My father wishes me to give all my time to studies now. 61. Select the most appropriate option that can substitute the underlined segment in the given sentence. If there is no need to substitute it, select 'No substitution required'. The first meeting of the Standing Committee were on growth financing because investment is crucial for us in revival after the pandemic. (a) No substitution required (b) was on growth financing (c) are on growth financing (d) have been in growth financing SSC Constable GD-06/12/2021 Shift-III Ans. (b) : For the above underlined part, option (b) ‘was on growth financing’ will be used in place of 'were on growth financing' because subject is singular. Hence verb will be in singular form. Correct sentence – The first meeting of the standing committee was on growth financing because.......... 62. Select the most appropriate option that can substitute the underlined segment in the given sentence. If there is no need to substitute it, select 'No substitution required'. I play football yesterday. (a) I been playing (b) I playing (c) No substitution required (d) I played SSC Constable GD-03/12/2021 Shift-II Ans. (d) : for the above underlined part, option(d) “I played’ will be used in place of 'I play' because use of ‘yesterday’ makes it clear that the sentence is in past tense. Correct SentenceI played football yesterday. Sentence Improvement

63.

Select the most appropriate option to substitue the underlined segment in the given sentence. If there is no need to substitute it, select 'No substitution'. If he doesn't reach the station, in time, he would sure miss the train. (a) would surely miss (b) No substitution (c) will sure missing (d) will surely miss SSC CHSL (Tier-I) –04/08/2021 (Shift-I) Ans. (d) : In the given sentence, ‘will surely miss’ will be used in place of ‘would sure miss’ because it is a conditional sentence. In these sentences, ‘if’ clause is used in present indefinite and second clause is used in future indefinite. Correct Sentence If he doesn't reach the station in time, he will surely miss the train. 64. Select the most appropriate option to substitute the underlined segment in the given sentence. If there is no need to substitute it, select 'No substitution required'. Lavanya was happy to see that the tree they had planted the previous year has grow quite taller. (a) No substitution required (b) had grown quite tall (c) had grow quite taller (d) had grown quite tallest SSC CHSL (Tier-I) –05/08/2021 (Shift-I) Ans. (b) : In the given sentence, ‘had grown quite tall’ will be used in place of ‘has grow quite taller’ because sentence is in past tense. Hence this clause will also be used in past tense. Correct Sentence Lavanya was happy to see that the tree they had planted the previous year has grown quite tall. 65. Select the most appropriate option to substitute the underlined segment in the given sentence. If there is no need to substitute it, select 'No substitution required'. Sometimes the family liking to play a game together after dinner. (a) Sometimes the family likes (b) Sometime the family likes (c) No substitution required (d) Sometimes the family are like SSC CHSL (Tier-I) –09/08/2021 (Shift-I) Ans. (a) : In the given sentence, ‘sometimes the family likes’ will be used in place of ‘sometimes the family liking’ because subject is singular. Hence singular verb (likes) will be used. Correct Sentence Sometimes the family likes to play a game together after dinner. 66. Select the most appropriate option to substitute the underlined segment in the given sentence. If there is no need to substitute it, select 'No substitution'. Had you not interfering in my affairs, I would have been in the US now. (a) No substitution (b) Had you not interfered (c) Was you not interfere (d) Have you not been interfering SSC CHSL (Tier-I) –09/08/2021 (Shift-I)

108

YCT

Ans. (b) : In the given sentence, ‘Had you not interfered’ will be used in place of ‘Had you not interfering' because 'V3' form is used after ‘Had’. Correct Sentence Had you not interfered in my affairs, I would have been in the US now. 67. Select the most appropriate option to substitute the underlined segment in the given sentence. If no substitution is required, select 'No substitution'. If you are not able to go out, did not worry. (a) don't worried (b) worrying not (c) do not worry (d) No substitution SSC CGL (Tier-I) –23/08/2021 (Shift-I) Ans. (c) : In the given sentence, ‘do not worry’ will be used in place of ‘did not worry’ because sentence is in present tense. Hence ‘do not’ will be used. Correct Sentence If you are not able to go out, do not worry. 68. Select the most appropriate option to substitute the underlined segment in the given sentence. If no substitution is required, select 'No substitution'. What were the examples that the writer gave to proved his point ? (a) No substitution (b) given to prove his point (c) gave to prove his point (d) give for proving his point SSC CGL (Tier-I)–23/08/2021 (Shift-I) Ans. (c) : For the above underlined part ‘ gave to prove his point’ will be used in place of 'gave to proved his point' because ‘to+V1’ is used as infinitive. Correct Sentence What were the examples that the writer gave to prove his point? 69. Select the option that will improve the underlined part of the sentence. In case no improvement is needed, select 'No improvement required'. I wish I have come an hour sooner. (a) has (b) will (c) had (d) No improvement required SSC CGL (Tier-I) –16/08/2021 (Shift-I) Ans. (c) : for the above underlined part, ‘Had’ will be used in place of ‘have’ because past indefinite (V2) is used after ‘I wish’. Correct SentenceI wish I had come an hour sooner. 70. Select the option that will improve the underlined part of the sentence. In case no improvement is needed, select 'No improvement'. The teacher advised the children to not went near the lake. (a) No improvement (b) not to go (c) don't go (d) not to be going SSC CGL (Tier-I) –18/08/2021 (Shift-I) Sentence Improvement

Ans. (b) : In the given sentence, ‘ not to go’ will be used in place of ‘to not went’ because ‘not’ is used before conjunction ‘to’ in indirect speech and V1 is used after ‘to’ Correct SentenceThe teacher advised the children not to go near the lake. 71. Select the most appropriate option to improve the underlined segment in the given sentence. If there is no need to improve it, select 'No improvement'. I have been working with children before, so I know that to expect. (a) am working (b) have worked (c) work (d) No improvement SSC CGL (Tier-II) 16/11/2020 (3 pm - 5 pm) Ans. (b) : For the above underlined part, ‘have worked’ will be used in place of have been working because use of 'before' makes it clear that the sentence will be in present perfect tense. Correct SentenceI have worked with children before, so I know that to expect. 72. Select the most appropriate option to improve the underlined segment in the given sentence. If there is no need to improve it, select 'No improvement'. He has being put behind prison for life. (a) No improvement (b) being putting in (c) been put behind (d) been put in SSC CGL (Tier-II) 16/11/2020 (3 pm - 5 pm) Ans. (d) : In the given sentence, ‘been put in’ will be used in place of ‘being put behind’ because IIIrd form of verb is used after has/have and ‘Put in’ is used to keep in prison. Correct SentenceHe has been put in prison for life 73. Select the most appropriate option to improve the underlined segment in the given sentence. If there is no need to improve it, select 'No improvement'. I am really looking forward to meet you. (a) to have met you (b) to be meeting you (c) to meeting you (d) No improvement SSC CGL (Tier-II) 16/11/2020 (3 pm - 5 pm) Ans. (c) : In the given sentence, ‘to meeting you’ will be used in place of ‘to meet you’ because gerund form (Ving) is used after ‘look forward to’. Correct SentenceI am really looking forward to meeting you. 74. Select the most appropriate option to improve the underlined segment in the given sentence. If there is no need to improve it, select 'No improvement'. If you had asked me, I had told you not to invest in that property. (a) No improvement (b) I would have told you (c) I will tell you (d) I would tell you SSC CGL (Tier-II) 16/11/2020 (3 pm-5 pm)

109

YCT

Ans. (b) : In the given sentence, ‘I would have told you’ will be used in place of ‘I had told, you’, because if ‘if’ clause is used in ‘had + V3’ then principal clause will be in would have + V3’. Note– If +present ............will + VI If+had+VIII ........... would have + VIII If+ (VII) ...........would +VI Correct Sentence If you had asked me, I would have told you not to invest in that property. 75. Select the most appropriate option to improve the underlined segment in the given sentence. If there is no need to improve it, select 'No improvement'. There was an argument about if we shall move to another city. (a) about if we should move (b) about whether we should move (c) on if we shall be moving (d) No improvement SSC CGL (Tier-II) 16/11/2020 (3 pm - 5 pm) Ans. (b) : For the above underlined part, ‘about whether we should move’ will be used because whether is used for the sense of option choice. And sentence is in past tense ‘should’ will be used. Correct SentenceThere was an argument about whether we should move to another city. 76. Select the most appropriate option to improve the underlined segment in the given sentence. If there is no need to improve it, select 'No improvement'. I am not use to drink coffee. (a) not use to drank (b) No improvement (c) not used to drink (d) not used to drinking SSC CGL (Tier-II) 16/11/2020 (3 pm - 5 pm) Ans. (d) : For the above underlined part, ‘not used to drinking’ will be used because gerund form (Ving) is used after ‘is/am/are + used to’. Note- Is/am/are+used to+Ving Correct SentenceI am not used to drinking coffee. 77. Select the most appropriate option to improve the underlined segment in the given sentence. If there is no need to improve it, select 'No improvement'. Why not you to be a good boy and sit down? (a) don't you be (b) you not be (c) you are not being (d) No improvement SSC CGL (Tier-II) 16/11/2020 (3 pm - 5 pm) Ans. (a) : for the above underlined part, ‘don’t you be’ will be used because ‘helping verb’ is used after Whword. Correct Sentence - Why don't you be a good boy and sit down? 78. Select the most appropriate option to improve the underlined segment in the given sentence. If there is no need to improve it, select 'No improvement'. She are having two brothers and three sisters. (a) has (b) No improvement (c) is having (d) have SSC CGL (Tier-II) 16/11/2020 (3 pm - 5 pm) Sentence Improvement

Ans. (a) : for the above underlined part, ‘has’ will be used because subject (She) is singular. Hence singular verb is used. And ‘has’ is used for possession. Correct Sentence - She has two brothers and three sisters. 79. Select the most appropriate option to improve the underlined segment in the given sentence. If there is no need to improve it, select 'No improvement'. You have not showed any improvement in your handwriting. (a) had not showed (b) has not shown (c) No improvement (d) have not shown SSC CGL (Tier-II) 16/11/2020 (3 pm - 5 pm) Ans. (d) : for the above underlined part, ‘have not shown’ will be used because IIIrd form is used after has/have. I II III Show Showed Shown Correct Sentence– You have not shown any improvement in your handwriting. 80. Select the alternative that will improve the underlined part of the sentence in case there is no improvement select “No improvement”. The watchman prevent him from parking his car near the gate. (a) prevents him for parking (b) prevented him to park (c) prevented him from parking (d) No improvement SSC CPO-SI (Tier-II) 27/09/2019 (3 pm - 5 pm) Ans. (c) : For the above underlined part, ‘prevented him’ will be used because sentence will be in past tense. Hence 'sub+V2' will be used. Correct Sentence– The watchman prevented him from parking his car near the gate. 81. Select the alternative that will improve the underlined part of the sentence in case there is no improvement select “No improvement”. 'Did you brought your drawing books?’the teacher asked the students. (a) Did you bring (b) Do you bringing (c) Do you brought (d) No improvement SSC CPO-SI (Tier-II) 27/09/2019 (3 pm - 5 pm) Ans. (a) : For the above underlined part., ‘did you bring’ will be used because first form of verb (V1) is used after 'do/does/did'. Correct Sentence - Did you Bring your drawing books? 82. Select the alternative that will improve the underlined part of the sentence in case there is no improvement select “No improvement". The news reported that the police have catched the guilty person and are interrogating him. (a) had catched the guilty (b) No improvement (c) have caught the guilty (d) are caught a guilty SSC CPO-SI (Tier-II) 27/09/2019 (3 pm - 5 pm)

110

YCT

Ans. (c) : for the above underlined part. ‘have caught the guilty’ will be used because IIIrd form of verb is used after has/have. Correct Sentence - The news reported that the police have caught the guilty person and are interrogating him. 83. Select the alternative that will improve the underlined part of the sentence in case there is no improvement select “No improvement”. Ajit has been played bridge since 2017. (a) is been playing bridge from 2017 (b) has been play bridge from 2017 (c) No improvement (d) has been playing bridge since 2017 SSC CPO-SI (Tier-II) 27/09/2019 (3 pm - 5 pm) Ans. (d) : for the above underlined part, ‘has been playing bridge since 2017’ will be used because ‘Ving’ form is used after 'has been' in present perfect continuous tense. Correct Sentence - Ajit has been playing bridge since 2017. 84. Select the alternative that will improve the underlined part of the sentence in case there is no improvement select “No improvement”. If I had been there I would surely solve the problem for you. (a) I would surely have solved (b) I will surely solves (c) I would surely solving (d) No improvement SSC CPO-SI (Tier-II) 27/09/2019 (3 pm - 5 pm) Ans. (a) : for the above underlined part ‘I would surely have solved’ will be used because if ‘if clause’ is used with had + V3 then result clause will be in would + have + V3. Correct Sentence - If I had been there I would surely have solved the problem for you. 85. Select the alternative that will improve the underlined part of the sentence in case there is no improvement select “No improvement”. The floods this year cause a lot of damage to the crops. (a) causing a lot of (b) cause lots of (c) have caused a lot of (d) No improvement SSC CPO-SI (Tier-II) 27/09/2019 (3 pm - 5 pm) Ans. (c) : For the above underlined part, ‘have caused a lot of’ will be used because it gives suitable meaning for the sentence. Correct Sentence - The floods this year have caused a lot of damage to the crops. 86. Select the alternative that will improve the underlined part of the sentence in case there is no improvement select “No improvement”. Some people like to watches television all day. (a) No improvement (b) likes to watch (c) like to watch (d) like to watching SSC CPO-SI (Tier-II) 27/09/2019 (3 pm - 5 pm) Ans. (c) : for the above underlined part, ‘like to watch will be used because base form of verb (V1) is used after infinite ‘to’. Correct Sentence - Some people like to watch television all day. Sentence Improvement

87.

Select the alternative that will improve the underlined part of the sentence in case there is no improvement select “No improvement”. The milk has boiled over and falling onto the stove. (a) falls into the stove (b) fallen onto the stove (c) No improvement (d) fall over in the stove SSC CPO-SI (Tier-II) 27/09/2019 (3 pm - 5 pm) Ans. (d) : for the above underlined part, ‘fall over in the stove will be used because it gives suitable meaning for the sentence. Correct sentence – The milk has boiled over and fall over in the stove. 88. Select the alternative that will improve the underlined part of the sentence in case there is no improvement select "No improvement". Riya went into the shop because it has sale. (a) No improvement (b) It is having sale (c) It having a sale (d) it had a sale SSC CPO-SI (Tier-II) 27/09/2019 (3 pm - 5 pm) Ans. (d) : for the above underlined part, ‘it had a sale’ will be used because sentence is in past tense. Correct Sentence - Riya went into the shop because it had a sale. 89. Select the alternative that will improve the underlined part of the sentence. In case there is no improvement select "No improvement". The manager could not able to understand the problem the factory workers faced. (a) No improvement (b) could not to (c) was not able to (d) never able to SSC Stenographer Grade C&D –05/02/2019 (3:5pm)

Ans. (c) : For the above underlined part ‘ was not able to’ will be used because use of could with ‘able’ is superfluous. Correct Sentence - The manager was not able to understand the problem the factory workers faced. 90. Select the alternative that will improve the underlined part of the sentence. In case there is no improvement select "No improvement". Gita suggested that her daughter to open a savings bank account. (a) No improvement (b) that her daughter open (c) her daughter opening (d) her daughter to start SSC Stenographer Grade C&D –05/02/2019 (3:5pm)

Ans. (b) : for the above underlined part, ‘open (verb)’ will be used because verb base form is used after noun and to + V1 (infinite) is used as noun. Correct Sentence - Gita suggested that her daughter open a savings bank account. 91. Select the alternative that will improve the underlined part of the sentence. In case there is no improvement select "No improvement". Ravi tried to explained him the problem but he refused to listen. (a) No improvement (b) Explain the problem to him (c) explaining him the problem (d) explain the problem

111

SSC Stenographer Grade C&D –05/02/2019 (3:5pm)

YCT

Ans. (b) : for the above underlined part, ‘explain the problem to him' will be used because base form of verb (V1) is used after infinite ‘to’. Correct Sentence - Ravi tried to explain the problem to him but he refused to listen. 92. Select the alternative that will improve the underlined part of the sentence. In case there is no improvement select "No improvement". No sooner did the bell range than the children ran out of the class. (a) did the bell ring than (b) the bell ring (c) No improvement (d) when the bell range SSC Stenographer Grade C&D –05/02/2019 (3:5pm)

Ans. (a) : for the above underlined part, ‘did the bell ring than’ will be used because first form of the verb (V1) is used after do/does/did. Correct Sentence - No sooner did the bell ring than the children ran out of the class. 93. Improve the bracketed part of the sentence. (They have hold), it is in the fitness of things that the legislature should be taken into confidence on Brexit. (a) They have holding (b) They have held (c) They has held (d) No improvement SSC CGL (Phase-II) 17/02/2018 Ans. (b) For the above bracketed part,’ they have held’ will be used because V3 form is used after has/have. sub + has/have + VIIIrd + object + other word Correct Sentence (They have held), it is in the fitness of things that the legislature should be taken into confidence on Brexit. 94. Improve the bracketed part of the sentence. It nuances the larger debate on whether such votes should override the will of the legislature, (or guided it further). (a) or guide it further (b) or guide it to farther (c) or guidance further (d) No improvement SSC CGL (Phase-II) 17/02/2018 Ans. (a) for the above bracketed part ‘or guide it further’ will be used because first form (V1) is used after model should. Correct Sentence In nuance the larger debate on whether such votes should override the will of the legislature or guide it further. 95. Improve the bracketed part of the sentence. A new chapter (may been put forth) that India gained Independence only in the last one year and not in 1947. (a) may have been putted forth (b) may be put forth (c) may be left forth (d) No improvement SSC CGL (Phase-II) 17/02/2018 Ans. (b) for the above bracketed part, ‘May be put forth' will be used because 'be' (Verb2) is used after model ‘May’. Correct Sentence - A new chapter (may be put forth) that India gained Independence only in the last one year and not in 1947 Sentence Improvement

96.

Improve the bracketed part of the sentence. The duo’s network was (made to learn identify) true signals using previously vetted signals; they then studied the weaker signals. (a) made to learn to identify (b) made to learning to identify (c) made learning to identify (d) No improvement SSC CGL (Phase-II) 17/02/2018 Ans. (a) For the above bracketed part, ‘made to learn to identify’ will be used because when two verb is used together then infinite (to + V1) form is used for later verb. Correct Sentence– The duo’s network was (made to learn to identify) true signals using previously vetted signals; they then studied the weaker signals. 97. Improve the bracketed part of the sentence. It also indicates the caveats and failure modes in the model need to be improved before (been used independently). (a) been using independently (b) being used independently (c) been independently (d) No improvement SSC CGL (Phase-II) 17/02/2018 Ans. (b) For the above bracketed. Part. ‘being used independently’ will be used because ‘need to be + V3' makes it clear that the sentence is in passive voice hence ‘being + V3’ will be used. Correct Sentence - It also indicates the caveats and failure modes in the model need to be improved before (being used independently). 98. Improve the bracketed part of the sentence. We have made progress in our development journey, and people (came) out in large numbers to vote for development. (a) had come (b) coming (c) will come (d) No improvement SSC CGL (Phase-II) 17/02/2018 Ans. (c) for the above bracketed part, ‘will come’ will be used because main clause is in present perfect tense Hence co-ordinate clause will be in simple future tense. Correct Sentence We have made progress in our development journey, and people (will come) out in large numbers to vote for development. 99. Select the most appropriate option to substitute the underlined segment in the given sentence. If there is no need to substitute it, select ‘No substitution required’. She is taking all kinds of medicine wherever she goes. (a) No substitution required (b) was taking (c) takes (d) took SSC CPO-SI – 23/11/2020 (Shift-I) Ans. (c) : In the above sentence, ‘takes’ will be used in place of ‘taking’ because ‘Present indefinite tense’ is used for habitual sense. Correct SentenceShe takes all kinds of medicine wherever she goes.

112

YCT

100. Select the most appropriate option to substitute the underlined segment in the given sentence. If there is no need to substitute it, select ‘No substitution required’. The secretary talks over the phone right now. (a) No substitution required (b) talked (c) was talking (d) is talking SSC CPO-SI – 23/11/2020 (Shift-I) Ans. (d) : for the above underlined part, ‘is talking’ will be used because ‘right now’ makes it clear that the sentence is in present continuous tense. Correct Sentence– The secretary is talking over the phone right now. 101. Select the most appropriate option to substitute the underlined segment in the given sentence. If there is no need to substitute it, select ‘No substitution required’. I am certain Ashok has finished his work by tomorrow. (a) had finished (b) No substitution required (c) will have finished (d) would have finish SSC CPO-SI – 25/11/2020 (Shift-II) Ans. (c) : for the above underlined part, ‘will have finished’ will be used because 'will have + V3' is used for the sense of completion of an action in the future. Correct sentence–I am certain Ashok will have finished his work by tomorrow. 102. Select the most appropriate option to substitute the underlined segment in the given sentence. If there is no need to substitute it, select ‘No substitution required’. Have taken permission, Shivam went horse riding. (a) Having taking permission (b) No substitution required (c) Have took permission (d) Having taken permission SSC CPO-SI – 25/11/2020 (Shift-II) Ans. (d) : For the above sentence, ‘ having taken permission’ will be used in place of ‘ have taken permission’ because present participle is needed for the sentence. Present participle–having + VIIIrd form Correct sentence– Having taken permission, Shivam went horse riding 103. Select the most appropriate option to substitute the underlined segment in the given sentence. If there is no need to substitute it, select ‘No substitution required’. The young man lifts weights at the gym when he pulled a muscle. (a) is lifting (b) has lifting (c) was lifting (d) No substitution required SSC CPO-SI – 23/11/2020 (Shift-II) Ans. (c) : for the above sentence, ‘was lifting’ will be used in place of 'lifts' because second clause is in Past tense. Hence first clause will also be in past tense. Correct SentenceThe young man was lifting weights at the gym when he pulled a muscle. Sentence Improvement

104. Select the most appropriate option to substitute the underlined segment in the given sentence. If there is no need to substitute it, select ‘No substitution required’. I want to learn yoga when I was in school. (a) ought to (b) No substitution required (c) need to (d) used to SSC CPO-SI – 23/11/2020 (Shift-II) Ans. (d) : In the above underlined part, ‘used to’ will be used because ‘used to’ is used to refer to things in past which no longer true. Correct Sentence - I used to learn yoga when I was in school. 105. Select the most appropriate option to substitute the underlined segment in the given sentence. If there is no need to substitute it, select ‘No substitution required’. She is being waited for you for two hours. (a) has been waiting (b) had wait (c) No substitution required (d) have been waited SSC CPO-SI – 24/11/2020 (Shift-II) Ans. (a) : In the above underlined part, ‘has been waiting’ will be used because sentence is in present perfect continuous tense. Hence 'has + been + Ving' will be used. 'Being' can be used as gerund or in passive voice of present or past continuous tenses. Correct sentence– She has been waiting for you for two hours. 106. Select the most appropriate option to substitute the underlined segment in the given sentence. If there is no need to substitute it, select 'No substitution required'. Everybody who know him agree that he rarely loses his temper. (a) No substitution required (b) who knows him agrees that (c) who knows him agrees if (d) whom know him agrees that SSC CPO-SI – 25/11/2020 (Shift-I) Ans. (b) : for the above underlined part, ‘who knows him agrees that’ will be used because 'singular verb' is used after everybody, everyone etc. Correct sentence–Everybody who knows him agrees that he rarely loses his temper. 107. Select the most appropriate option to substitute the underlined segment in the given sentence. If no substitution is required, select No improvement. The female ostrich guards the nest at night and the male guard it in the day. (a) guard it during the day (b) No improvement (c) guarding it during day (d) guards it during the day SSC CGL (Tier-I) – 13/06/2019 (Shift-II) Ans. (d) : For the above underlined part, ‘guards it during the day’ will be used because subject is singular Hence 'V1' (with s/es) will be used and during will be used for a period of time. Correct Sentence - The female ostrich guards the nest at night and the male guards it during the day.

113

YCT

108.

Select the most appropriate option to substitute the underlined segment in the given sentence. If no substitution is required, select no improvement. Though many European traders visit Puducherry in the sixteenth century onwards, what remains today is a relic of its French past. (a) traders visited Puducherry from the sixteenth century onwards (b) traders visited Puducherry in a sixteenth century onward (c) No improvement (d) trader visit Puducherry from the sixteenth century onwards SSC CGL (Tier-I) – 10/06/2019 (Shift-I) Ans : (a) For the above underlined part, ‘visited’ will be used in place of 'visit' because first clause of the sentence is in past tense and ‘from…….onwards’ will be used. Correct sentenseThough many European traders visited Puducherry from the sixteenth century onwards, what remains today is a relic of its French past. 109. Select the most appropriate option to substitute the underlined segment in the given sentence. If no substitution is required, select No improvement. The workers of this textile factory demand higher wages for a long time. (a) has demanded higher wages (b) No improvement (c) demanded higher wages (d) have been demanding higher wages SSC CGL (Tier-I) – 04/06/2019 (Shift-I) Ans : (d) For the above underlined part, ‘have been demanding higher wages’ will be used because ‘for + time’ makes it clear that the sentence is in present perfect continuous tense. Sub + has/have + been + v + ing + since/for + time Correct sentence– The workers of this textile factory have been demanding higher wages for a long time. 110. Select the most appropriate option to substitute the underline segment in the given sentence. If substitution is required, select no. improvement. I look for a better job for the last two months, but nothing is in sight. (a) have looked for a better job (b) looked for a better job (c) have been looking for a better job (d) No. improvement SSC CGL (Tier-I) – 04/06/2019 (Shift-II) Ans : (c) For the above underlined part, ‘have been looking for a better job’ will be used because 'for + time' makes it clear that the sentence is in present perfect continuous tense. Correct Sentence I have been looking for a better job for the last two months, but nothing is in sight. 111. Select the most appropriate option to substitute the underlined segment in the given sentence. If no substitution is required, select No improvement. She read that novel since she got up in the morning. Sentence Improvement

No improvement has been reading that novel has read that novel reads that novel SSC CGL (Tier-I) – 04/06/2019 (Shift-III) Ans. (b) : For the above underlined part, ‘has been reading that novel’ will be used because when since is used as conjunction then its principal clause is used in present perfect tense and second clause is used in Past indefinite tense. Present perfect/Present Perfect continuous tense + Since + Past Indefinite Tense Correct SentenceShe has been reading that novel since she got up in the morning. 112. Select the most appropriate option to substitute the underlined segment in the given sentence. If there is no need to substitute it, select 'No substitution required'. I made them cleaning the room thoroughly. (a) I make them cleaning (b) No substitution required (c) I making them clean (d) I made them clean SSC CPO-SI – 09/12/2019 (Shift-II) Ans : (d) In the above sentence, ‘I made them clean’ will be used in place of ‘I made them cleaning’ because causative verb is used here. Causative use of verb– (i) sub + Make + Object + VI (ii) Sub + Cause + Object + To + VI (iii) Sub + Have/get + Object + VIII Correct Sentence– I made them clean the room thoroughly. 113. Select the most appropriate option to substitute the underlined segment in the given sentence. If there is no need to substitute it, select 'No substitution'. I enjoy to watch a good detective movie. (a) watching (b) watched (c) No substitution (d) watch SSC CPO-SI – 09/12/2019 (Shift-I) Ans : (a) For the above underlined part, ‘watching’ is used instead of ‘to watch’ because gerund form (V1 + ing) is used after enjoy, hate, detest, mind, worth, etc Correct sentence– I enjoy watching a good detective movie. 114. Select the most appropriate option to substitute the underlined segment in the given sentence. If there is no need to substitute it, select 'No substitution'. When we arrived at Jhansi station, we are keeping our luggages in the cloak room. (a) are keeping our luggage (b) kept our luggages (c) kept our luggage (d) No substitution SSC CPO-SI – 11/12/2019 (Shift-I) Ans : (c) For the above underlined part, ‘kept our luggage’ will be used because first clause is in past tense. Hence second clause will also be in past tense and ‘luggage’ is used without ‘s/es.’ Correct sentence– When we arrived at Jhansi station, we kept our luggage in the cloak room

114

(a) (b) (c) (d)

YCT

115. Select the most appropriate option to substitute the underlined segment in the given sentence. If there is no need to substitute it, select 'No substitution' He waits to you for about two hours. (a) has waiting for you (b) No substitution (c) has been waiting you (d) has been waiting for you SSC CPO-SI – 11/12/2019 (Shift-I) Ans : (d) For the above underlined part, ‘has been waiting for you’ will be used because ‘for + time’ makes it clear that the sentence is in present perfect continuous tense. Correct sentence– He has been waiting for you for about two hours. 116. Select the most appropriate option to substitute the underlined segment. If no substitution is required select 'No substitution'. His employees were asking him for a raise for a long time now. (a) are asking him of a raise (b) have been asking him for a raise (c) No substitution (d) asked him for a rise SSC CPO-SI – 12/12/2019 (Shift-I) Ans : (b) For the above underlined part, ‘have been asking him for a raise’ will be used because ‘for + time’ makes it clear that the sentence is in present perfect continuous tense. Correct Sentence– His employees have been asking him for a raise for a long time now. 117. Select the most appropriate option to substitute the underlined segment in the given sentence. If there is no need to substitute it, select No improvement. The children trying hardly and learnt the poems. (a) hardly tried (b) hardly trying (c) tried hard (d) No improvement SSC CPO-SI – 13/12/2019 (Shift-II) Ans. (c) : For the above underlined part, ‘tried hard’ will be used because sentence is in past tense. Correct Sentence The children tried hard and learnt the poems. 118. Select the most appropriate option to substitute the underlined segment in the given sentence. If there is no need to substitute it, select No improvement. Pedestrians is to be careful when crossing busy roads. (a) need be caring when crossing (b) would care if they are crossing (c) No improvement (d) must be careful when crossing SSC CPO-SI – 13/12/2019 (Shift-II) Ans. (d) : For the above underlined part, ‘must be careful when crossing’ will be used because ‘must’ is always used to express obligation. Correct sentence– Pedestrians must be careful when crossing busy roads. Sentence Improvement

119. Select the most appropriate option to substitute the underlined segment in the given sentence. If there is no need to Substitute it, select No improvement. The minister promise to looked into the matter of fuel emissions and air pollution. (a) promise to looks into (b) No improvement (c) promised to look into (d) promises to looking at SSC CHSL (Tier-I) –11/07/2019 (Shift-I) Ans : (c) For the above underlined part, ‘promised to look into’ will be used because first form of verb (V1) is used after infintive ‘to’. Infinitive = (to + v1) look into. Correct sentence– The minister promised to look into the matter of fuel emissions and air pollution. 120. Select the most appropriate option to substitute the underlined segment in the given sentence. If there is no need to substitute it, select No improvment. After going through their brochure I has decided to make a donation to help their campaign. (a) I decided to make a donation to help (b) No improvement (c) I decided to make a donation for helping (d) I decided for making a donation to help SSC CHSL (Tier-I) –11/07/2019 (Shift-II) Ans : (a) For the above underlined part, ‘I decided’ will be used because given sentence will be in past indefinite tense according to the sense. Correct sentence After going through their brochure I decided to make a donation to help their campaign. 121. Select the most appropriate option to substitute the underlined segment in the given sentence. If there is no need to substitute it, select No improvement. With each passing day he was going in more trouble. (a) he was getting into (b) he were going to (c) he are getting in (d) No improvement SSC CHSL (Tier-I) –11/07/2019 (Shift-III) Ans : (a) For the above underlined part, ‘ he was getting into’ will be used because ‘get into something’ is used to reach a particular condition. And the sentence is in continuous form. Hence 'V1 + ing' will be used. Correct sentence–With each passing day he was getting into more trouble. 122. Select the most appropriate option to substitute the underlined segment in the given sentence. If there is no need to substitute it, select No improvement. Women in traditional costumes welcoming a foreign dignitaries. (a) welcome to foreign dignitanes (b) No improvement (c) welcomed the foreign dignitaries (d) welcomes the foreigner dignitaries SSC CHSL (Tier-I) –11/07/2019 (Shift-III)

115

YCT

Ans : (c) For the above underlined part, ‘ welcomed the foreign dignitaries’ will be used because sentence is in past tense according to the sense. Correct Sentence- Women in traditional costumes welcomed the foreign dignitaries. 123. Select the most appropriate option to substitute the underlined segment in the given sentence. If there is no need to substitute it, select No improvement "I'm afraid I must left now", said the guest after the inaugural function was over. (a) I'm afraid I has to be leaving now (b) I'm afraid I can left now (c) No improvement (d) I'm afraid I must leave now SSC CHSL (Tier-I) –10/07/2019 (Shift-II) Ans : (d) For the above underlined part, ‘leave’ will be used in place of ‘left’ because first form of verb (V1) is used after modal ‘must’. Correct Sentence– “I'am afraid I must leave now” said; the guest after the inaugural function was over. 124. Select the most appropriate option to substitute the underlined segment in the given sentence. If there is no need to substitute it, select No improvement. I'm afraid I am not knowing how to swim so I can't join you all that day. (a) I'm afraid I do not know (b) I are afraid I be not knowing (c) I'm afraid I is not knowing (d) No improvement SSC CHSL (Tier-I) –09/07/2019 (Shift-I) Ans : (a) For the above underlined part, ‘I’m afraid I do not know’ will be used because when ‘how + to + V1’ is used then present indefinite tense is used in first clause (before how to) I am afraid I do not know how to swim so I can't join ↓ present Indefinite sentence you all that day. 125. Identify the most appropriate option to substitute the underlined segment. If no substitution is required, select No substitution. All the accident victims have been admitted and received attention. (a) and have received (b) but are receiving (c) No substitution (d) so had received SSC CHSL (Tier-I) –08/07/2019 (Shift-II) Ans : (a) For the above underlined part, ‘and have received’ will be used because same part of speech is used with ‘and’. Correct sentence– All the accident victims have been admitted and have received attention. 126. Identify the most appropriate option to substitute the underlined segment. If no substitution is required, select No substitution. We are going to have pleasure in accepting your kind invitation. (a) We will have a pleasure (b) We have pleasure (c) We are having pleasure (d) No substitution SSC CHSL (Tier-I) –05/07/2019 (Shift-I)

Sentence Improvement

Ans : (b) For the above underlined part, ‘have pleasure will be used because ‘have’ is used for possession. Correct Sentence We have pleasure in accepting your kind invitation. 127. Select the most appropriate option to substitute the underlined segment in the given sentence. If there is no need to substitute it, select No improvement. The government has ordered that all the coaching centers without fire safety arrangements housing above the fourth floor be shut down immediately. (a) house up to the fourth floor (b) housed above the fourth floor (c) No improvement (d) housed over a fourth floor SSC CHSL (Tier-I) –04/07/2019 (Shift-I) Ans : (b) For the above underlined part, ‘housed above the fourth floor’ will be used because ‘verb’ form is required for the sentence. Correct Sentence– The government has ordered that all the coaching centers without fire safety arrangements housed above the fourth floor be shut down immediately. 128. Select the most appropriate option to substitute the underlined segment in the given sentence. If there is no need to substitute it, select No improvement. The ceremony will following by tea and refreshments for the guests. (a) No improvement (b) will followed by (c) will be followed by (d) will follow with SSC CHSL (Tier-I) –04/07/2019 (Shift-I) Ans : (c) For the above underlined part, ‘will be followed by’ will be used because the given sentence is in passive voice. Subject + will/shall + be + verbIIIrd + by + object + other words. Correct sentence–The ceremony will be followed by tea and refreshments for the guests. 129. Identify the most appropriate option to substitute the underlined segment. If no substitution is required, select No substitution. By next week I will have been made all the reservations for the trip to Europe. (a) will had make (b) will have been making (c) will have made (d) No substitution. SSC CHSL (Tier-I) –04/07/2019 (Shift-III) Ans : (c) For the above underlined part, ‘will have made will be used because the sentence is in future tense. Structure- Sub. + shall/will + have + VIIIrd form + obj. Correct sentence– By next week I will have made all the reservations for the trip to Europe. 130. Select the most appropriate option to substitute the underlined segment in the given sentence. If there is no need to substitute it, select No improvement. The Urban Art Commission has suggested that the landscape be developing by adding water fountains around the sculpture.

116

YCT

(a) be developed with (b) to be developed by (c) No improvement (d) be developed by SSC CHSL (Tier-I) –03/07/2019 (Shift-III) Ans. (d) : For the above underlined part, ‘be developed’ will be used because the clause is in passive voice. Hence 'Be + V3' will be used. Correct Sentence– The Urban Art Commission has suggested that the landscape to be developed by adding water fountains around the sculptare. 131. Select the most appropriate option to substitute the underlined segment in the given sentence. If there is no need to substitute it, select No improvement. Oh, what a lovely necklace! You need no buy such an expensive gift. (a) hadn't needed to buy (b) needn't have bought (c) No improvement (d) didn't need to buying SSC CHSL (Tier-I) –02/07/2019 (Shift-I) Ans. (b) : For the above underlined part, ‘needn’t have bought’ will be used because ‘needn’t have + V3’ is used when the action is done but it was not necessary and even a waste of time. Correct Sentence– Oh, what a lovely necklace! you needn't have bought such an expensive gift. 132. Select the most appropriate option to substitute the underlined segment in the given sentence. If there is no need to substitute it, select No improvement. I try to avoid travel in the rush hour. (a) to travel at (b) travelling in (c) travelling on (d) No improvement SSC CHSL (Tier-I) –02/07/2019 (Shift-III) Ans : (b) For the above underlined part, ‘travelling in’ will be used because gerund (V1+ ing) form is used after ‘avoid’ (v). Correct sentence– I try to avoid travelling in the rush hour. 133. Select the most appropriate option to substitute the underlined segment in the given sentence. If there is no need to substitute it, select 'No improvement'. When the actress comes to know that she was declared winner of the best actress award, her joy knew no bounds. (a) No improvement (b) came to know (c) was come to know (d) was knowing SSC CHSL (Tier-I) 26/10/2020 (Shift-II) Ans. (b) : For the above underlined part, ‘came to know’ will be used because second clause of the sentence is in past tense . Hence first clause will also be in past tense. correct sentence–When the actress came to know that she was declared winner of the best actress award, her joy knew no bounds. 134. Select the most appropriate option to substitute the underlined segment in the given sentence. If there is no need to substitute it, select ‘No improvement’. After walking a few steps in the forest, our guide sensing some sort of movement in the foliage behind us.

Sentence Improvement

(a) had been senses (b) sensed (c) No improvement (d) was sensed SSC CHSL (Tier-I) 16/10/2020 (Shift-III) Ans. (b) :For the above underline part, ‘sensed’ will be used because verb (V2) form is needed here. Hence 'V2' (Past indefinite) will be used. Correct sentence–After walking a few steps in the forest, our guide sensed some sort of movement in the foliage behind us. 135. Select the most appropriate option to substitute the underlined segment in the given sentence. If there is no need to substitute it, select ‘No improvement’. The new product will be arrive on supermarket shelves only next year. (a) No improvement (b) will arrived on (c) will arrive on (d) will arriving on SSC CHSL (Tier-I) 16/10/2020 (Shift-III) Ans. (c) :For the above underlined part, ‘will arrive on’ will be used because the sentence is in simple future. Hence 'will + V1' will be used. Correct sentence– The new product will arrive on supermarket shelves only next year. 136. Select the most appropriate option to substitute the underlined segment in the given sentence. If there is no need to substitute it, select 'No improvement'. For the last two years Shweta has being diligently working with Rohingya refugees to create colourul table mats using their local weaving techniques. (a) have (b) have being (c) has been (d) No improvement SSC CHSL (Tier-I) 21/10/2020 (Shift-II) Ans. (c) : For the above underlined part, ‘has been’ will be used because ‘for + time’ makes it clear that the sentence is in present perfect continuous tense. Correct Sentence For the last two years Shweta has been diligently working with Rohingya refugees to create colourul table mats using their local weaving techniques. 137 Select the most appropriate option to substitute the underlined segment in the given sentence. If there is no need to substitute it, select "No improvement'. Swami Vivekanand's vast knowledge of Eastern and Western culture appealed to all those who come in contacts with him. (a) No improvement (b) who came in contact (c) that comes to contact (d) which contact SSC CHSL (Tier-I) 19/10/2020 (Shift-I) Ans. (b) : For the above underlined part, ‘who came in contact’ will be used because the sentence is in past tense. Hence 'V2' will be used. Correct sentence– Swami Vivekanand's vast knowledge of Eastern and Western culture appealed to all those who came in contact with him. 138. Select the most appropriate option to substitute the underlined segment in the given sentence. If there is no need to substitute it, select No improvement.

117

YCT

''Did you tell your parents that you would going for movie with me this evening?'' My friend asked me. (a) then you were being gone (b) when you will going (c) this that you was going (d) that you are going SSC CHSL (Tier-I) 15/10/2020 (Shift-III) Ans. (d) : For the above underlined part, ‘That you are going’ will be used because the sentence is in direct form. Hence ‘Are’ will be used in place of ‘would’ Correct sentence–''Did you tell your parents that you are going for movie with me this evening?'' My friend asked me. 139. Select the most appropriate option to substitute the underlined segment in the given sentence. If there in no need to substitute it, No improvement There is a double-digit growth in salaries of vocational jobs now and the trend are likely to continue in the coming years. (a) No improvement (b) is likely for continuing (c) is likely to continue (d) is likely to be continuing SSC CHSL (Tier-I) 15/10/2020 (Shift-I) Ans. (c) : For the above underlined part, 'is likely to continue' will be used because subject of the sentence is in singular number. Correct sentence– There is a double-digit growth in salaries of vocational jobs now and the trend is likely to continue in the coming years. 140. Select the most appropriate option to substitute the underlined segment in the given sentence. If there is no need to substitute it, select No, improvement. Since it falls on the floor, her dog, a black Labrador, gulped the biscuit down. (a) As it have fallen (b) Because it is falling (c) No improvement (d) Since it fell SSC CHSL (Tier-I) 13/10/2020 (Shift-III) Ans. (d) For the above underlined part, ‘since it fell will be used because the given sentence is in past tense. Hence 'V2' will be used. Correct sentence– Since it fell on the floor, her dog, a black Labrador, gulped the biscuit down. 141. Select the most appropriate option to substitute the underlined segment in the given sentence. If the is no need to substitute it, select No improvement. There are too less space for Naveen to park his car in front of the shop (a) was so few space (b) were too little space (c) No improvement (d) was too little space SSC CHSL (Tier-I) 14/10/2020 (Shift-III) Ans. (d) : For the above underlined part, ‘was too little space’ will be used because sentence is in past tense and its subject (space) is singular. Hence ‘was’ will be used. Correct sentence– There was too little space for Naveen to park his car in front of the shop. Sentence Improvement

142. Select the most appropriate option to substitute the underlined segment in the given sentence. If there is no need to substitute it, select No improvement . It is evident from current research that young adults is more like to develop an interest in spoken English if there is a change in methodology. (a) are more likely (b) No Improvement (c) are more like (d) is more likely SSC CHSL (Tier-I) 21/10/2020 (Shift-III) Ans. (a) : For the above underline part, are more likely’ will be used because subject of the sentence is in plural number. And 'likely' (adverb form) is needed here. Correct sentence– It is evident from current research that young adults are more likely to develop an interest in spoken English if there is a change in methodology. 143. Select the most appropriate option to substitute the underlined segment in the given sentence. If there is no need to substitute it, select No improvement. I think you’d better paid online immediately as you may not get time to visit the bank later on during the day. (a) you’d better pay (b) No Improvement (c) you’d better paying (d) you’d have better pay SSC CHSL (Tier-I) 21/10/2020 (Shift-III) Ans. (a) : For the above underlined part.’ You’d better pay’ will be used because bare infinitive (without ‘to’) and first form of verb (V2) is used after ‘had better’. Correct sentence– I think you’d better pay online immediately as you may not get time to visit the bank later on during the day. 144. Select the most appropriate option to substitute the underlined segment in the given sentence. If there is no need to substitute it, select 'No improvement'. All the shops and restaurants have been closed during the bandh last Saturday. (a) No improvement (b) are being closed (c) were closed (d) has been closed SSC CHSL (Tier-I) 19/10/2020 (Shift-III) Ans. (c) : For the above underlined part, ‘were closed’ will be used because use of 'last Saturday' makes it clear that the sentence is in past tense. Correct sentence–All the shope and restaurants were closed during the bandh last Saturday. 145. Select the most appropriate option to substitute the underlined segment in the given sentence. If there is no need to substitute it, select No improvement. Gitanjali did not had school yesterday because it was a second Saturday. (a) does not have (b) No improvement (c) did not have (d) was not having SSC CHSL (Tier-I) 14/10/2020 (Shift-I) Ans. (c) : For the above underlined part, ‘did not have’ will be used because first form of verb (have) is used after 'did/do/does'. Correct sentence– Gitanjali did not have school yesterday because it was a second Saturday.

118

YCT

146. Select the most appropriate option to substitute the underlined segment in the given sentence. If there is no need to substitute it, select 'No improvement'. It is reported that everyday India witnessing the death of over 2000 babies aged less that one year. (a) India witness the death (b) No improvement (c) India will be witness the death (d) India witnesses the death SSC CHSL (Tier-I) 12/10/2020 (Shift-I) Ans. (d) : For the above underlined part, ‘India witnesses the death’ will be used because the use of 'every-day' makes it clear that the given sentence is in present tense. Correct sentence–It is reported that everyday India witnesses the death of over 2000 babies aged less that one year. 147. Select the most appropriate option to substitute the underlined segment in the given sentence. If there is no need to substitute it, select 'No improvement'. Many attempts has been made to climb the Everest before Tenzing and Hillary succeeded. (a) have been made (b) had been made (c) was made (d) No improvement SSC CHSL (Tier-I) 19/10/2020 (Shift-II) Ans. (b) : For the above underlined part, ‘had been made’ will be used because sentence in past perfect tense, the first complete work is used in past perfect tense, and the later complete work in used in past indefinite. Correct sentence– Many attempts had been made to climb the Everest before Tenzing and Hillary succeeded. 148. Improve the bracketed part of the sentence. Now that the child rights commission has (register) the case, it will go to the police for investigation, he said. (a) registered (b) registering (c) will register (d) No improvement SSC MTS 7-10-2017 (Shift-I) Ans. (a) : For the above bracketed part, ‘registered’ will be used because 'has/have + V3' is used in Present perfect tense. Correct sentence– Now that the child rights commission has registered the case, it will go to the police for investigation, he said. 149. Improve the bracketed part of the sentence. It was a formidable challenge that they (offer) going into the Glasgow championships. (a) offered (b) offering (c) will offering (d) No improvement SSC MTS 9-10-2017 (Shift-II) Ans. (a) : For the above bracketed part, ‘offered’ will be used because the given sentence is in past tense. Correct sentence– It was a formidable challenge that they offered going into the Glasgow championships. 150. Improve the bracketed part of the sentence. This rivalry is (prove to) be extremely beneficial for Indian badminton. (a) to prove to (b) proving to (c) proving by (d) No improvement SSC MTS 9-10-2017 (Shift-II) Sentence Improvement

Ans. (b) : For the above bracketed part, ‘Proving to’ will be used because 'V1 + ing' is used after 'is/am/are'. Correct sentence– This rivalry is proving to be extremely beneficial for Indian badminton. 151. Improve the bracketed part of the sentence. Yet, it would be a mistake to presume that ties can so easily return to their pre-2015 strength, as the ground (had) shifted in too many ways since then. (a) have (b) has (c) was (d) No improvement SSC MTS 9-10-2017 (Shift-I) Ans : (b) For the above bracketed part, ‘has’ will be used, because the given sentence is in present perfect tense. Correct sentence– Yet, it would be a mistake to presume that ties can so easily return to their pre-2015 strength, as the ground has shifted in too many ways since then. 152. Improve the bracketed part of the sentence. But she (win more) hearts for her gallant display of endurance. (a) wined more (b) won more (c) wan more (d) No improvement SSC MTS 10-10-2017 (Shift-III) Ans. (b) : For the above bracketed part, ‘won more’ will be used because sentence will be in past tense. Correct sentence– But she won more hearts for her gallant display of endurance. 153. Improve the bracketed part of the sentence. Last month (he makes news with his) "go whistle" comment to the leaders over outstanding dues. (a) he will make news with him (b) he made news with his (c) he makes news with him (d) No improvement SSC MTS 10-10-2017 (Shift-II) Ans. (b) : For the above bracketed part, ‘he made news with his’ will be used because use of 'last month’s' makes it clear that the sentence is in past tense. Correct sentence– Last month he made news with his "go whistle" comment to the leaders over outstanding dues. 154. Improve the bracketed part of the sentence. Sindhu's latest medal, to go with the Rio Olympics silver last year, has clearly (took her past) Nehwal. (a) taking her past (b) takes her past (c) taken her past (d) No improvement SSC MTS 11-10-2017 (Shift-III) Ans. (c) : For the above bracket part, ‘taken her past’ will be used because 'third form of verb' is used after has/have/had. Correct Sentence– Sindhu's latest medal, to go with the Rio Olympics silver last year, has clearly taken her past Nehwal. 155. Improve the bracketed part of the sentence. For a change, performance and not political expedience, (seem to have) dictated the nature and extent of the shuffle in the Council of Ministers. (a) seemingly has (b) seems to have (c) seemed to (d) No improvement SSC MTS 11-10-2017 (Shift-II)

119

YCT

Ans. (b) : For the above bracketed part, ‘seems to have’ will be used because the given sentence is in present indefinite tense. Hence singular verb (V1 + s/es) will be used. Correct sentence– For a change, performance and not political expedience, seems to have dictated the nature and extent of the shuffle in the Council of Ministers. 156. Improve the bracketed part of the sentence. Raman (has all along being) equated with best shooter among his peers. (a) had all along been (b) had all along being (c) has all along been (d) No improvement SSC MTS 11-10-2017 (Shift-I) Ans : (a) For the above bracketed part, ‘had all along been’ will be used because the given sentence will be in passive voice of past perfect tense. Correct sentence–Raman had all along been equated with best shooter among his peers. 157. Select the most appropriate option to substitute the underlined segment in the given sentence. If there is no need to substitute it, select 'No substitution required'. They couldn't find what they wanted and decide to look elsewhere. (a) No substitution required (b) have decided to look (c) had decided looking (d) decided to look SSC MTS – 09/08/2019 (Shift-I) Ans. (d) : For the above underlined part, ‘decided to look’ will be used because same part of speech (V) connects with ‘and’. Hence V2 will be used. Correct sentence– They couldn't find what they wanted and decided to look elsewhere. 158. Select the most appropriate option to substitute the underlined segment in the given sentence. If there is no need to substitute it, select 'No substitution required'. The government is set strict limits on public spending this year. (a) had been set (b) have set (c) No substitution required (d) has set SSC MTS – 08/08/2019 (Shift-III) Ans. (d) : For the above underlined part, ‘has set’ will be used because sentence is in present perfect tense. Correct sentence–The government has set strict limits on public spending this year. 159. Select the most appropriate option to substitute the underlined segment in the given sentence. If there is no need to substitute it, select No improvement. He lives in this remote colony since 2001. (a) has been living (b) continues to live (c) have been living (d) No improvement SSC MTS – 07/08/2019 (Shift-I) Ans. (a) : For the above underlined part, ‘has been living’ will be used because use of ‘since + time’ makes it clear that the sentence is in present perfect continuous tense. Correct Sentence–He has been living in this remote colony since 2001. Sentence Improvement

160. Select the most appropriate option to substitute the underlined segment in the given sentence. If there is no need to substitute it, select No Improvement. The housing problem in Mumbai becomes more serious day by day. (a) No Improvement (b) become (c) had become (d) is becoming SSC MTS – 05/08/2019 (Shift-III) Ans. (d) : For the above underlined part, ‘is becoming’ will be used because use of ‘day by day’ makes it clear that the sentence is in continuous tense. Correct Sentence– The housing problem in Mumbai is becoming more serious day by day. 161. Select the most appropriate option to substitute the underlined segment in the given sentence. If there is no need to substitute it, select No Improvement. The doctor has reassure me that the operation was a routine one. (a) No Improvement (b) had reassured me (c) is reassured me (d) reassure me SSC MTS – 02/08/2019 (Shift-II) Ans. (b) : For the above underlined part, ‘had reassured me’ will be used because subordinate clause is in past tense. Hence principal clause will also be in past tense. Correct sentence– The doctor had reassured me that the operation was a routine one. 162. Select the most appropriate option to substitute the underlined segment in the given sentence. If there is no need to substitute it, select No improvement. The teacher expected the students had finishing the project on time. (a) finishing (b) No improvement (c) to finish (d) finish SSC MTS – 22/08/2019 (Shift-I) Ans. (c) : for the above underlined part, ‘ to finish’ will be used because ‘object + infinitive (to + v1)’ is used after verbs 'ask, advise, allow, permit, teach, imagine, expect etc'. Correct sentence– The teacher expected the students to finish the project on time. 163. Select the most appropriate option to substitute the underlined segment in the given sentence. If there is no need to substitute it, select No improvement. The student asked the teacher if he can come in. (a) No improvement (b) could come (c) will come (d) can came SSC MTS – 21/08/2019 (Shift-III) Ans. (b) : For the above underlined part, ‘could come’ will be used because reporting verb is in past tense . Hence reported speech will also be in past tense. Correct sentence–The student asked the teacher if he could come in. 164. Select the most appropriate option to substitute the underlined segment in the given sentence. If there is no need to substitute it, select No improvement.

120

YCT

Bhawna will visited the Eiffel Tower during her trip. (a) will visit (b) will visiting (c) would visiting (d) No improvement SSC MTS – 21/08/2019 (Shift-I) Ans : (a) For the above underlined part, ‘will visit will be used because 'first form of verb' is used after modals will, shall can, could, may, might etc. Modals –Can, could, shall, should, will, would, may, might, must, used (to), ought (to), need, dare etc. Correct sentence–Bhawna will visit the Eiffel Tower during her trip. 165. Select the most appropriate option to substitute the underlined segment. If there is no need to substitute it, select No improvement. Emperor Ashoka have conquered Kalinga before he embraced Buddhism. (a) had conquered (b) was conquering (c) No improvement (d) has conquered SSC MTS – 19/08/2019 (Shift-II) Ans. (a) : For the above underlined part, ‘had conquered will be used because sentence is in past perfect tense, first completed work is used in past perfect and later completed work is used in past indefinite. Correct Sentence –''Emperor Ashoka had conquered Kalinga before he embraced Buddhism.'' 166. Select the most appropriate option to substitute the underlined segment in the given sentence. If there is no need to substitute it, select No improvement. I had to reconsider my decision as going to Syria might been dangerous. (a) had to be (b) might have been (c) may have (d) No improvement SSC MTS – 19/08/2019 (Shift-I) Ans. (b) : For the above underlined part, ‘might have been’ will be used because 'V3' form is used after ‘might have’. Correct sentence– '' I had to reconsider my decision as going to Syria might have been dangerous. 167. Select the most appropriate option to substitute the underlined segment in the given sentence. If there is no need to substitute it, select No improvement. A little carefulness can have averted the accident. (a) can averted (b) could have averted (c) could have avert (d) No improvement SSC MTS – 19/08/2019 (Shift-I) Ans. (b) : For the above underlined part, ‘could’ will be used because 'could have + V3' is used for something that was possible in the past but that you didn’t do it. Correct Sentence–''A little carefulness could have averted the accident''. 168. Select the most appropriate option to substitute the underlined segment. If there is no need to substitute it, select No improvement. He is living in this house for the past fifteen years before he shifted abroad. (a) has been living (b) No improvement (c) had been living (d) was lived SSC MTS – 16/08/2019 (Shift-III) Sentence Improvement

Ans. (c) : For the above underlined part, ‘had been living’ will be used because the given clause will be in past perfect continuous tense. Correct sentence– He had been living in this house for the past fifteen years before he shifted abroad. 169. Select the most appropriate option to substitute the underlined segment. If there is no need to substitute it, select No improvement Sonia is learnt to play the guitar these days. (a) was learning playing (b) was learning to play (c) is learning to play (d) No improvement SSC MTS – 16/08/2019 (Shift-I) Ans. (c) : For the above underlined part, ‘is learning to play’ will be used because use of ‘these days’ makes it clear that the sentence will be in present continuous tense. Correct sentence–Sonia is learning to play the guitar these days. 170. Select the most appropriate option to substitute the underlined segment. If there is no need to substitute it, select No improvement Have you ever being by New York? (a) No improvement (b) ever being at (c) ever been to (d) never been with SSC MTS – 14/08/2019 (Shift-II) Ans. (c) : For the above underlined part, ‘ever been to’ will be used because 'V3 form of verb' is used after has/have/had. Correct sentence–Have you ever been to New York? 171. Select the most appropriate option to substitute the underlined segment in the given sentence. If there is no need to substitute it, select No improvement. Do you mind to lending me your umbrella? (a) to lend me (b) lending me (c) No improvement (d) lend me SSC MTS – 14/08/2019 (Shift-II) Ans. (b) : For the above underlined part, ‘lending me’ will be used because gerund form (V1 + ing) is used after mind, enjoy, worth etc. Correct sentence–Do you mind lending me your umbrella? 172. Select the most appropriate option to substitute the underlined segment in the given sentence. If there is no need to substitute it, select 'No substitution required'. She has decided on to do a Chinese language course. (a) No substitution required (b) on doing (c) on to doing (d) to doing SSC MTS – 13/08/2019 (Shift-III) Ans. (b) : For the above underlined part, ‘on doing’ will be used because gerund form (V1 + ing) is used after preposition. Correct sentence–She has decided on doing a Chinese language course. 173. Select the most appropriate option to substitute the underlined segment in the given sentence. If there is no need to substitute it, select option 'No substitution required'.

121

YCT

The trainee hand a copy of his appointment letter to the HR head. (a) No substitution required (b) handed over a copy (c) hand over the copy (d) handed of a copy SSC MTS – 09/08/2019 (Shift-III) Ans. (b) : For the above underlined part, ‘handed over a copy’ will be used because past form of verb (V2) is needed according to the sense. Correct sentence–The trainee handed over a copy of his appointment letter to the HR head. 174. Select the most appropriate option to substitute the underlined segment in the given sentence. If there is no need to substitute it, select 'No substitution required'. For the past six months, she is searching for a job. (a) had been searching (b) No substitution required (c) has searching (d) has been searching SSC MTS – 09/08/2019 (Shift-II) Ans. (d) : For the above underlined part, ‘has been searching’ will be used because ‘for + time’ makes it clear that the sentence is in present perfect continuous tense. Correct sentence– For the past six months, she has been searching for a job. 175. Select the most appropriate option to substitute the underlined segment in the given sentence. If there is no need to substitute it, select 'No substitution required'. The detectives searched the house from top to bottom, but find any sign of the stolen goods. (a) No substitution required (b) while finding no sign (c) despite found any sign (d) but found no sign SSC MTS – 09/08/2019 (Shift-II) Ans. (d) : For the above underlined part, ‘but found no sign of’ will be used because first clause of the sentence is in past tense. Hence second clause will also be in past tense. Correct sentence–The detectives searched the house from top to bottom, but found no sign of the stolen goods. 176. Select the most appropriate option to substitute the underlined segment in the given sentence. If there is no need to substitute it, select 'No substitution required'. This training has intended to give people the skills they need for a particular job. (a) have intended to (b) is intend towards (c) No substitution required (d) is intended to SSC MTS – 08/08/2019 (Shift-II) Ans. (d) : For the above underlined part, ‘is intended to’ will be used because sentence will be in passive voice of present indefinite tense. Correct sentence This training is intended to give people the skills they need for a particular job. Sentence Improvement

177. Select the most appropriate option to substitute the underlined segment in the given sentence. If there is no need to substitute it, select No Improvement. The inflation rate raised up 8% last month. (a) rose up (b) No Improvement (c) rise to (d) raise up SSC MTS – 07/08/2019 (Shift-III) Ans. (a) : For the above underlined part, ‘rose to’ will be used because 'rose' (Past form of rise) is needed according to the sense. Correct sentence –The inflation rate rose up 8% last month. 178. Select the most appropriate option to substitute the underlined segment in the given sentence. If there is no need to substitute it, select No improvement. I will write to you when I will reach New Delhi. (a) I reach (b) I will be reaching (c) I am reached (d) No improvement SSC MTS – 07/08/2019 (Shift-II) Ans. (a) : For the above underlined part, ‘I reach’ will be used because of conditional sentence. Structure of type I conditional isMain clause (Future indefinite) , Subordinate clause (If/when + Present + indefinite) If/When + Present Indefinite, Future Indefinite (main clause)

Correct Sentence I will write to you when I reach new Delhi. 179. Select the most appropriate option to substitute the underlined segment in the given sentence. If there is no need to substitute it, select No Improvement. My car broke off on my way to home. (a) down (b) out (c) on (d) No improvement SSC MTS – 07/08/2019 (Shift-II) Ans. (a) : For the above underlined part, Phrasal verb ‘break down’ will be used. Hence ‘down’ will be used in place of ‘off’. Correct Sentence–My car broke down on my way to home. 180. Select the most appropriate option to substitute the underlined segment in the given sentence. If there is no need to substitute it, select No Improvement. You need have brought your umbrella as it is a cloudy day. (a) can have (b) could has (c) should have (d) No Improvement SSC MTS – 06/08/2019 (Shift-II) Ans. (c) : For the above underlined part, ‘should have’ will be used, because 'should' is used for advice or suggestion. Correct sentence– You Should have brought your umbrella as it is a cloudy day. 181. Select the most appropriate option to substitute the underlined segment in the given sentence. If there is no need to substitute it, select No improvement. Permit me to speak the truth and I have say it without any fear.

122

YCT

(a) (b) (c) (d)

No improvement I had say it without any fear I shall said it without any fear I will say it without any fear SSC GD – 02/03/2019 (Shift-II) Ans. (d) : For the above underlined part, ‘I will say’ will be used because second clause will be in 'future indefinite tense. ' Correct Sentence–Permit me to speak the truth and I will say it without any fear. 182. Select the most appropriate option to substitute the underlined segment in the given sentence. If there is no need to substitute it, select No improvement. The family had been without food for four days. (a) family are been (b) family is been (c) family is being (d) No improvement SSC GD – 02/03/2019 (Shift-II) Ans. (d) : No improvement 183. Select the most appropriate option to substitute the underlined segment in the given sentence. If there is no need to substitute it, select No improvement. He is felt extremely unhappy yesterday because of the inordinate delay of the messenger. (a) No improvement (b) He has feeling extremely unhappy yesterday (c) He have been feeling extremely unhappy yesterday (d) He was feeling extremely unhappy yesterday SSC GD – 02/03/2019 (Shift-I) Ans. (d) : For the above underlined part, ‘He was feeling extremely unhappy’ will be used because use of ‘yesterday’ makes it clear that the sentence is in past tense. Correct Sentence He was feeling extremely unhappy yesterday because of the inordinate delay of the messenger. 184. Select the most appropriate option to substitute the underlined segment in the given sentence. If there is no need to substitute it, select No improvement. The newly- launched mobile phone application has been a great support for people in remote areas of the country. (a) applications will been a great support (b) applications has been a great support (c) No improvement (d) applications have been a great support SSC GD – 02/03/2019 (Shift-I) Ans. (c) : No improvement. 185. Select the most appropriate option to substitute the underlined segment in the given sentence. If there is no need to substitute it, select No improvement. The convict was informed that the judge will heard the case tomorrow. (a) judge hears a case (b) judge will hear the case (c) judge will hearing a case (d) No improvement SSC GD – 02/03/2019 (Shift-I)

Sentence Improvement

Ans. (b) : For the above underlined part, ‘judge will hear the case’ will be used because first form of the verb(V1) is used after ‘will’. Correct Sentence The convict was informed that the judge will hear the case tomorrow. 186. Select the most appropriate option to substitute the underlined in the given sentence. If there is no need to substitute it, select 'No improvement. As you can see from my resume, I am working in Mumbai since 2016. (a) I have been working in Mumbai since 2016 (b) I am working in Mumbai since 2016 (c) No improvement (d) I am work in Mumbai since 2016 SSC GD – 18/02/2019 (Shift-II) Ans. (a) : For the above underlined part, ‘ I have been working in Mumbai since 2016’, will be used because use of ‘since + time’ makes it clear that the sentence is in perfect continuous tense. CORRECT SENTENCE–As you can see from my resume, I have been working in Mumbai since 2016. 187. Select the most appropriate option to substitute the underlined in the given sentence. If there is no need to substitute it, select 'No improvement. In the interview held recently, I answer the question as best as I could. (a) I answered the question as best as I can. (b) I answer the question as best as I could. (c) I answered the question as best as I could. (d) No improvement SSC GD – 18/02/2019 (Shift-II) Ans. (c) : For the above underlined part, ‘I answered the question as best as I could’ will be used because use of ‘ held recently’ makes it clear that the sentence is in past tense. Correct Sentence In the interview held recently, I answered the question as best as I could. 188. Select the most appropriate option to substitute the underlined segment in the given sentence. If there is no need to substitute it, select No improvement. Tourists visiting Leh take home photos and memories but leave back a mountains of waste. (a) leave away (b) left back (c) No improvement (d) leave behind SSC GD – 14/02/2019 (Shift-II) Ans. (d) : for the above underlined part, ‘leave behind’ will be used because it gives suitable meaning for the sentence. Correct Sentence–Tourists visiting Leh take home photos and memories but leave behind a mountains of waste. 189. Select the most appropriate option to substitute the underlined segment in the given sentence. If there is no need to substitute it, select No Improvement When you go by rail, make sure taking the express train that stops only at big stations. (a) make sure to taking (b) No improvement (c) make sure to take (d) making sure to take SSC GD – 14/02/2019 (Shift-I)

123

YCT

Ans. (c) : for the above underlined part ‘make sure to take’ will be used because infinitive ‘to + V1’ is used after ‘make sure’. correct sentence–When you go by rail, make sure to take the express train that stops only at big stations. 190. Select the most appropriate option to substitute the underlined segment in the given sentence. If there is no need to substitute it, select 'No substitution' She give me everything I asked for. (a) Is giving me everything (b) gave me everything (c) No improvement (d) gave myself everything SSC GD – 11/02/2019 (Shift-II) Ans. (b) : For the above underlined part, ‘gave me everything’ will be used because sentence is in past tense. Correct sentence–She gave me everything I asked for. 191. Select the most appropriate option to substitute the underlined segment in the given sentence. If there is no need to substitute it, select 'No substitution' A study has shown that the use of mobile data in India have increased 50 times over the last three years (a) No improvement (b) has increased 50 times (c) has been increased to 50 times (d) Is increasing 50 times SSC GD – 11/02/2019 (Shift-II) Ans. (b) : For the above underlined part, ‘has increased 50 times’ will be used because subject (The use of mobile data) is singular. Hence verb will be singular. Correct sentence–A study has shown that the use of mobile data in India has increased 50 times over the last three years 192. Select the most appropriate option to substitute the underlined segment in the given sentence. If there is no need to substitute it, select No substitution The football coach urged the team were give its best to the final match. (a) No Improvement (b) That it give its best (c) Try to give it best (d) To give its best SSC GD – 11/02/2019 (Shift-II) Ans. (d) : For the above underlined part,’ to give its best’ will be used because 'urge somebody to do something' will be used. urge somebody to dosomething

Correct sentence–The football coach urged the team to give its best to the final match.

B.

Subject and Verb Agreement

193. Improve the underlined part of the sentence. Choose 'No improvement' as an answer if the sentence is grammatically correct. They selling postage stamps at the corner store. (a) sold postal stamped (b) sell postage stamps (c) No improvement (d) sells postage stamps SSC MTS 08/05/2023 (Shift-III) Sentence Improvement

Ans. (b) : For the above given underlined part of the sentence 'selling postage stamps' will be replaced by 'sell postage stamps' because this sequence is present Indefinite tense. Subject 'They' is plural so verb is also used plural (sell). Thus, the corrct answer is option (b) 194. Select the most appropriate option to substitute the underlined segment in the given sentence. The syllabus designers has tried to address the problem of time available for teaching. (a) Have tried to address (b) Having tried to address (c) Has tried addressing (d) has been tried to address SSC GD 08/02/2023 (Shift-IV) Ans. (a) : 'Have tried to address' is the most appropriate option to substitute the underlined segment in the given sentence 'has tried to address', because the subject of the sentence designers is plural. Hence plural verb (have) is used for it. According to the sense of the above sentence, the other options do not give proper meaning. Hence option (a) is correct answer. 195. Select the most appropriate option to substitute the underlined segment in the given sentence. The monitor of the class don't know the answer. (a) The class doesn't knew (b) The class doesn't know (c) The class didn't knew (d) The class don't knew SSC GD 06/02/2023 (Shift-III) Ans. (b) : The underline segment in the given sentence, 'The class doesn't know' will be used because the subject of the sentence is 'The monitor' (singular) so according to the singular subject, 'does not' would be appropriate. Hence option (b) is correct answer. 196. Select the most appropriate option that can substitute the underlined segment in the given sentence. If there is no need to substitute it, select ‘No substitution’. Honesty as well as discipline are required to succeed in life. (a) is required to succeed in life (b) are requirement to succeed in life (c) has required to succeed in life (d) No substitution SSC CGL (Tier-I) 26/07/2023 (Shift-II) Ans. (a) : The most appropriate option for the underlined segment 'is required to succeed in life' will be used in place of 'are required to succeed in life, because when, subject is followed by words 'such as as well as', 'along with ', 'besides' are takes verb as per first subject. Thus, the correct answer is option (a).

124

YCT

197. Select the most appropriate option that can substitute the underlined segment in the following sentence. The number of seats in the movie theatre were reduced to comply with the new social distancing guidelines. (a) of the movie theatre were reduced (b) in the movie theatre are reduced (c) in the movie theatre was reduced (d) in the movie theatre were being reduced SSC CHSL (Tier-I) 09/08/2023 (Shift-III) Ans. (c) : The underlined segment of the sentence 'in the movie theatre were reduced' need to be substituted with option (c) 'in the movie theatre was reduced' . Because 'The number of' is singular and always takes a singular verb. Correct Sentence The number of seats in the movie theatre was reduced to comply with the new social distancing guidelines. 198. Improve the underlined part of the sentence. Choose 'No improvement' as an answer if the sentence is grammatically correct. We keeps a horse, two cows, and six chickens. (a) has keeps a (b) keep a (c) keeping a (d) No improvement SSC CHSL (Tier-I) 21/03/2023 (Shift-II) Ans. (b) : In the above sentence, 'keep a horse' will be used in place of 'keeps a horse' because Plural verb is used with plural subject (we). Correct Sentence: We keep a horse, two cows, and six chickens. 199. Select the option that will improve the underlined part of the sentence. In case no improvement is needed, select 'No Improvement'. Many battle was fought on the soil of India. (a) Many a battle was (b) No Improvement (c) Much battle was (d) Many battle were SSC CGL (Tier-II) 08.08.2022 Shift-II Ans. (a) : For the above underlined part, ‘Many a battle was’ will be used because ‘battle’ is not used in plural. Hence ‘Many a + singular noun + singular verb’ will be used. many a + sin gular noun + singular verb Correct sentence – Many a battle was fought on the soil of India. 200. Select the most appropriate option to substitute the underlined segment in the given sentence. If no substitution is required, select 'No substitution required'. None of the soldiers did not survive when trapped in the glacier. (a) could survive (b) had not survived (c) could not survives (d) No substitution required SSC Stenographer (Grade C & D) 12.11.2021 Shift-II Sentence Improvement

Ans. (a) : For the above underlined part, ‘could service’ will be used because none is used for negative sense Hence ‘not’ will not be used. Correct sentence–None of the soldiers could survive when trapped in the glacier. 201. Select the most appropriate option to improve the underlined segment in the given sentence. If there is no need to improve it, select 'No Improvement required'. Fifty dollars are a high price to pay for this shirt. (a) are the (b) No improvement required (c) is a (d) were a SSC Stenographer (Grade C & D) 12.11.2021 Shift-II Ans. (c) : For the above underlined part, ‘is a’ will be used because ‘fifty dollars’ is in singular form hence singular helping verb will be used. Correct sentence – Fifty dollars is a high price to pay for this shirt. 202. Select the most appropriate option that can substitute the underlined segment in the given sentence. People was empty out of the malls and roads. (a) was emptying out of (b) was emptying of (c) were emptying of (d) were emptying out of SSC CHSL 27.05.2022 Shift-I Ans. (d) : For the above underlined part, ‘were emptying out of’ will be used because plural helping verb will be used with plural subject (people). Correct Sentence People were emptying out of the malls and roads. 203. Select the option that will improve the underlined part of the given sentence. In case no improvement is needed, select 'No improvement required'. Many a man has succumbed to his temptations. (a) No improvement required (b) men has (c) a men has (d) man has SSC CGL (Tier-I) 13/04/2022 Shift-II Ans. (a) : No Improvement required Note:- "Many a + singular noun + singular verb structure follow’’ 204. Select the most appropriate option to substitute the underlined segment in the given sentence. If there is no need to substitute it, select ‘No substitution required’. I wish I were listening to my parents. (a) have listened (b) had listened (c) No substitution required (d) am listening SSC CGL (Tier-I) 12/04/2022 Shift-I

125

YCT

Ans. (b) : For the above underlined part, ‘had listened’ will be used because past tense is used after ‘I wish’ for the regret from the past. Correct Sentence – I wish I had listened to my parents. 205. Select the option that will improve the underlined part of the given sentence. In case no improvement is needed, select 'No improvement required'. Although the waters may have receded, the aftermath of one of the world's biggest natural disaster had only just begun. (a) the aftermath of one of the world's biggest natural disasters (b) No improvement required (c) the aftermath of one of the worlds' biggest natural disasters has (d) the aftermath of one the worlds' biggest natural disaster SSC MTS-06/07/2022 Shift-I Ans. (a) : For the above underlined part ‘the aftermath of one of the world’s biggest disasters’ will be used because plural noun is used of ‘one of’. Correct Sentence – Although the waters may have receded, the aftermath of the of the world's biggest natural disasters had only just begun. 206. Select the option that will improve the underlined part of the given sentence. In case no improvement is needed, select 'No improvement required'. Scientific discoveries is often result of hard word by many people. (a) No improvement required (b) is often resulting (c) are often resulted (d) are often the result SSC MTS-07/07/2022 Shift-II Ans. (d) : for the above underlined part ‘are often the result’ will be used because its subject is plural. Hence plural verb ‘are’ will be used. Correct Sentence – Scientific discoveries are often the result of hard work by many people. 207. Select the most appropriate option that can substitute the underlined segment in the given sentence. If there is no need to substitute it, select ‘No substitution required’. Wash and dry the raw mangoes, and use a fork to prick it all over. (a) for prick it all over (b) to prick them all over (c) No substitution required (d) for all over pricking them SSC Constable GD-29/11/2021 Shift-II Ans. (b) : For the above underlined part ‘to prick them all over’ will be used because subject is plural. Hence plural objective case ‘them’ will be used. Correct Sentence Wash and dry the raw mangoes, and use a fork to prick them all over. Sentence Improvement

208. Select the most appropriate option that can substitute the underlined segment in the given sentence. If there is no need to substitute it, select ‘No substitution required’. The book are not owned by him. (a) are not being owned (b) No substitution required (c) is not owned (d) is not own SSC Constable GD-23/11/2021 Shift-III Ans. (c) : For the above underlined part, ‘is not owned’ will be used because ‘the book’ is singular noun. Hence singular verb will be used. Correct Sentence - The book is not owned by him. 209. Select the option that will improve the underlined part of the given sentence. In case no improvement is needed, select 'No improvement required'. What fun is it to ride a rollercoaster with friends! (a) No improvement rquired (b) it is for riding (c) is it riding (d) it is to ride SSC Constable GD-13/12/2021 Shift-III Ans. (d) : For the above underlined part, ‘ it is to ride’ will be used because sentence is exclamatory. Hence simple sentence structure (Sub + V) will be used. Correct sentence– What fun it is to ride a rollercoaster with friends! 210. Select the most appropriate option that can substitute the underlined segment in the given sentence. If there is no need to substitute it, select ‘No substitution required’. Tourism is one in the biggest businesses in the world. (a) are one of the (b) is one of the (c) are one in the (d) No substitution required SSC Constable GD-14/12/2021 Shift-II Ans. (b) : For the above underlined part, ‘is one of the’ will be used because ‘one of the’ is a phrase Hence ‘of’ will be used instead of 'in'. Correct sentence - Tourism is one of the biggest businesses in the world. 211. Select the most appropriate option to substitute the underlined segment in the given sentence. If there is no need to substitute it, select 'No substitution'. Neither the parents nor the child were happy about the result. (a) were happy with (b) was happy with (c) No substitution (d) was happy for SSC CHSL (Tier-I) –13/04/2021 (Shift-I) Ans. (b) : For the above underline part, ‘was happy with’ will be used because when two subjects connect with neither--------- nor, then verb is used according to later subject. Hence singular verb will be used. Correct Sentence Neither the parents nor the child was happy with the result.

126

YCT

212. Select the most appropriate option to improve the underlined segment in the given sentence. If there is no need to improve it, select 'No improvement'. Many a person are unable of distinguish right from wrong. (a) are capable of (b) No improvement (c) are unable to (d) is unable to SSC CGL (Tier-II) 16/11/2020 (3 pm - 5 pm) Ans. (d) : for the above underlined part, ‘is unable to’ will be used because singular verb is used after ‘many a', and ‘to’ preposition is used with unable. Correct SentenceMany a person is unable to distinguish right from wrong. 213. Select the alternative that will improve the underlined part of the sentence in case there is no improvement select “No improvement”. You have to wear this uniform whether you likes it or not. (a) whether you like it or not (b) whether you are liking it or not (c) No improvement (d) Whether if you like it or not SSC CPO-SI (Tier-II) 27/09/2019 (3 pm - 5 pm) Ans. (a) : For the above underlined part ‘whether you like it or not’ will be used because plural verb is used with plural subject (you). Correct Sentence–You have to wear this uniform whether you like it or not. 214. Select the alternative that will improve the underlined part of the sentence. In case there is no improvement select "No improvement". All students has a dream to study in a good university. (a) have dream (b) No improvement (c) has dream (d) have a dream SSC Stenographer Grade C&D –05/02/2019 (3:5pm)

Ans. (d) : For the above underlined part, ‘have a dream’ will be used because ‘all students’ is plural subject. Hence plural verb ‘have’ will be used. Correct Sentence - All students have a dream to study in a good university. 215. Select the alternative that will improve the underlined part of the sentence. In case there is no improvement select "No improvement". Many of my friends come to school by bus. (a) Much of my friends (b) Many who are friends (c) No improvement (d) Many of all my friends SSC Stenographer Grade C&D –05/02/2019 (3:5pm)

Ans. (c) : No improvement 216. Select the alternative that will improve the underlined part of the sentence. In case there is no improvement select "No improvement". Success and failure in life depends on how well a person uses his time. (a) depending upon (b) dependent on (c) No improvement (d) depend on SSC Stenographer Grade C&D –05/02/2019 (3:5pm)

Sentence Improvement

Ans. (d) : For the above underlined part ‘depend on’ will be used because plural verb is used for plural subject (Success and failure). Correct Sentence - Success and failure in life depend on how well a person uses his time. 217. Improve the bracketed part of the sentence. The Centre’s bid to dispel the pall of gloom over the economy (have been helped) in recent weeks by a rating. (a) had been help (b) has been helped (c) have been helping (d) No improvement SSC CGL (Phase-II) 17/02/2018 Ans. (b) For the above bracketed part, ‘has been helped’ will be used because subject (The centre’s bid) is singular. Hence singular verb will be used. Correct Sentence– The centre's bid to dispel the pall of gloom over the economy has been helped in recent weeks by a rating. 218. Improve the bracketed part of the sentence. Without any exaggeration, the provision will ensure that legislators (keep a voice) in finalizing the terms and conditions of the agreement. (a) have a voice (b) head a voice (c) lost a voice (d) No improvement SSC CGL (Phase-II) 17/02/2018 Ans. (a) For the above bracketed part, ‘have a voice’ will be used because ‘have’ is used for possession and subject is plural hence helping verb will be in plural form. Correct Sentence Without any exaggeration the provision will ensure that legislators have a voice in finalizing the terms and conditions of the agreement. 219. Select the most appropriate option to substitute the underlined segment in the given sentence. If there is no need to substitute it, select ‘No substitution required’ Have either of you been to the planetarium before? (a) No substitution required (b) Have everyone (c) Has either (d) Has both SSC CPO-SI – 24/11/2020 (Shift-I) Ans. (c) : For the above underlined part, ‘Has either’ will be used because either means ‘one’ or 'the other of two things'. Hence singular verb is used with it . Correct Sentence– Has either of you been to the planetarium before? 220. Select the most appropriate option to substitute the underlined segment in the given sentence. If no substitution is required, select No improvement. Knowing the particular things that motivate each person help you add power to their motivation. (a) No improvement (b) helps you add power (c) help you to add power (d) help you adding power SSC CGL (Tier-I) – 11/06/2019 (Shift-I)

127

YCT

Ans. (b) : For the above underlined part, ‘helps you add power’ will be used because ‘each’ is used as singular subject. Hence singular verb will be used. Correct sentence– Knowing the particular things that motivate each person helps you add power to their motivation. 221. Select the most appropriate option to substitute the underlined segment in the given sentence. If no substitution is required, select No improvement. Each of the guests were given a return gift. (a) No improvement (b) Each of the guests was given (c) Each guests were given (d) Each and every guest were given SSC CGL (Tier-I) – 19/06/2019 (Shift-III) Ans. (b) : For the above underlined part, ‘Each of the guests was given’ will be used because plural noun and singular verb is used with ‘Each of the’. Correct sentence– Each of the guests was given a return gift. 222. Select the most appropriate option to substitute the underlined segment in the given sentence. If no substitution is required, select No improvement. How long you are living in this city? (a) have you lived (b) are you been living (c) are you live (d) No improvement SSC CGL (Tier-I) – 13/06/2019 (Shift-II) Ans. (a) : For the above underlined part, ‘have you lived’ will be used because the given sentence will be in present perfect tense. Correct sentence– How long have you lived in this city? 223. Select the most appropriate option to substitute the underlined segment in the given sentence. If no substitution is required, select No improvement. The India consensus study highlight a benefits by adding domestic violence prevention approaches to current government policy. (a) highlights the benefits of adding (b) No improvement (c) highlight the benefits from added (d) highlight a benefit in addition SSC CGL (Tier-I) – 10/06/2019 (Shift-I) Ans : (a) For the above underlined part ‘highlights the benefits of adding’ will be used because subject is singular. Hence singular verb will be used. Correct Sentence– The India consensus study highlights the benefits of adding domestic violence prevention approaches to current government policy. 224. Select the most appropriate option to substitute the underlined segment in the given sentence. If no substitution is required, select No improvement. The world's agricultural land are in pressure to raising more and more crops. (a) land is under pressure to raise (b) land is under pressure so raising (c) No improvement (d) land is at pressure to raise SSC CGL (Tier-I) – 10/06/2019 (Shift-III) Sentence Improvement

Ans. (a) : For the above underlined part ‘land is under pressure to raise’ will be used because subject is singular . Hence singular verb will be used. Correct Sentense– The world's agricultural land is under pressure to raise more and more crops. 225. Select the most appropriate option to substitute the underlined segment in the given sentence. If there is no need to substitute it, select No improvement. The children has had their dinner but the adults still have to eat. (a) No improvement (b) have had their dinner (c) are have their dinner (d) is having the dinner SSC CPO-SI – 12/12/2019 (Shift-II) Ans : (b) For the above underlined part, ‘have had their dinner’ will be used because subject (‘the children’ is plural . Hence Plural verb (have) will be used. Correct sentence– The children have had their dinner but the adults still have to eat. 226. Select the most appropriate option to substitution the underlined segment. If no substitution is required select 'No substitution'. This is one of the boldest economic reform that have ever been introduced. (a) economic reforms that has ever (b) economic reforms that have ever (c) No substitution (d) economic reform that has ever SSC CPO-SI – 12/12/2019 (Shift-I) Ans : (b) For the above underlined part, ‘reforms’ will be used. Because Plural noun/Pronoun is used after ‘one of the’. One of + plural noun + relative pronoun + plural verb

Correct sentence– This is one of the boldest economic reforms that have ever been introduced. 227. Select the most appropriate option to substitute the underlined segment in the given sentence. If there is no need to substitute it, select no improvement. In the park I happened to meet two of my father's friend who had known him for several years. (a) No improvement (b) two of my father's friends (c) two friends of my fathers (d) two of my fathers friend SSC CPO-SI – 13/12/2019 (Shift-II) Ans. (b) : For the above underlined part, ‘two of my father’s friends’ will be used because plural noun is used after ‘two’. Hence ‘friends’ will be used. Correct sentence–In the park I happened to meet two of my father's friends who had known him for several years. 228. Select the most appropriate option to substitute the underlined segment in the given sentence. If there is no need to substitute it, select No improvement. If I does not pay my post-paid mobile bill by tomorrow I will probably have to pay a fine. (a) If I had not paid (b) If I do not pay (c) No improvement (d) If I has not paid SSC CHSL (Tier-I) –09/07/2019 (Shift-III)

128

YCT

Ans : (b) For the above underlined part, ‘If I do not pay’ will be used because when second clause is in future indefinite, first clause will be in present indefinite. Hence plural helping verb ‘do’ will be used. Correct Sentence–If I do not pay my post-paid mobile bill by tomorrow, I will probably have to pay a fine 229. Identify the most appropriate option to substitute the underlined segment. If no substitution is required, select No substitution. The commander of the ship as well as the crew are caught by the storm. (a) are caught in (b) is caught in (c) No substitution (d) are being caught by SSC CHSL (Tier-I) –08/07/2019 (Shift-II) Ans : (b) For the above underlined part, ‘is caught in’ will be used because its subject is singular. Hence singular verb ‘is’ will be used. Note-When two subjects are connect with as well as, with, along with, then verb is used according to first subject. Correct sentence–The commander of the ship as well as the crew is caught by the storm. 230. Identify the most appropriate option to substitute the underlined segment. If no substitution is required, select No substitution. He persisted to do that he thought was right. (a) for doing this which he thought (b) in doing what he thought (c) No substitution (d) to do what he thinks SSC CHSL (Tier-I) –05/07/2019 (Shift-I) Ans : (b) For the above underlined part, ‘in doing what he thought’ will be used because ‘in’ preposition is used with persist. And gerund form (V1+ ing) is used after prepositions. Note- Persist in doing something Correct sentence– He persisted in doing what he thought was right. 231. Select the alternative that will improve the underlined part of the sentence. In case there is no improvement select 'No improvement'. Sidharth want to become a football player but needs to practice more rigorously. (a) No improvement (b) wants to become (c) wanted becoming (d) is wanting to become SSC CHSL (Tier-I) 17/03/2020 (Shift-II) Ans. (b) : For the above underlined part, ‘wants to become’ will be used because subjects (Sidharth) is singular Hence verb will be singular. Note- s/es is used to make singular form of verb Correct sentence– Sidharth wants to become a football player but needs to practice more rigorously. 232. Select the most appropriate option to substitute the underlined segment in the given sentence. If there is no need to substitute it, select No improvement. The book's page seem to have been torn by someone. (a) No improvement (b) The book's page seems (c) The page of the book was seems (d) The page of the book has seem SSC CHSL (Tier-I) 20/10/2020 (Shift-II) Sentence Improvement

Ans. (b) : For the above underlined part, ‘ the book’s page seems’ will be used because subject (The book’s page) is singular. Hence singular verb will be used. Correct Sentence- The book's page seems to have been torn by someone. 233. Select the most appropriate option to substitute the underlined segment in the given sentence. If there is no need to substitute it, select 'NO improvement'. She is tender-hearted by nature and I have found her eyes moist many a times over the sad face of any children. (a) many a time (b) No improvement (c) many time (d) so many a times SSC CHSL (Tier-I) 19/03/2020 (Shift-II) Ans. (a) : For the above underlined part, ‘many a time’ will be used because singular noun and singular verb is used after ‘Many a’. Correct sentence–She is tender-hearted by nature and I have found her eyes moist many a time over the sad face of any children. 234. Improve the bracketed part of the sentence. The Home Department (have issued) separate guidelines on the participation of children in processions and public functions. (a) will issued (b) has issued (c) were issued (d) No improvement SSC MTS 7-10-2017 (Shift-I) Ans : (b) For the above underlined part, 'has issued' will be used because subject is singular Hence singular verb will be used. Correct sentence– The Home Department has issued separate guidelines on the participation of children in processions and public functions. 235. Improve the bracketed part of the sentence. The Finance Minister has acknowledged that the challenge before the government now (are to workout) both policy and investment measures to boost momentum. (a) were to work out (b) will to work out (c) is to work out (d) No improvement SSC MTS 11-10-2017 (Shift-II) Ans. (c) : For the above underlined part, ‘is to workout’ will be used because subject is singular. Hence singular verb will be used. Correct sentence– The Finance Minister has acknowledged that the challenge before the government now is to workout both policy and investment measures to boost momentum. 236. Select the most appropriate option to substitute the underlined segment in the given sentence. If there is no need to substitute it, select No improvement. The city have not receive much rais for a week now. (a) have not received (b) has not received (c) No improvement (d) has not receive SSC MTS – 08/08/2019 (Shift-I) Ans. (b) : For the above underlined part, ‘has not received will be used because subject (The city) is singular Hence singular verb will be used and V3 form is used after has/have. Correct sentenceThe city has not received much rais for a week now.

129

YCT

237. Select the most appropriate option to substitute the underlined segment in the given sentence. If there is no need to substitute it, select No Improvement. The father with his children were expected to attend the function. (a) are (b) have (c) No Improvement (d) was SSC MTS – 02/08/2019 (Shift-I) Ans. (d) : For the above underlined part, ‘was’ will be used because when two subjects are connected 'with', 'together with', 'as well as', then verb is used, according to first subject. Correct Sentence The father with his children was expected to attend the function. 238. Select the most appropriate option to substitute the underlined segment in the given sentence. If there is no need to substitute it, select no substitution required More than 60% of India's population live under the poverty line. (a) lives beside the poverty line (b) no substitution required (c) live by the poverty line (d) lives below the poverty line SSC MTS–14/08/2019 (Shift-III) Ans. (d) : For the above underlined part, ‘lives below the poverty line’ will be used, because subject is singular. Hence singular verb will be used. Correct Sentence– More than 60% of India's population lives below the poverty line. 239. Select the most appropriate option to substitute the underlined segment in the given sentence. If there is no need to substitute it, select No improvement. The new traffic signals is not clear visible from a distance (a) No improvement (b) are not clearly (c) is not clearly (d) are not clear SSC MTS – 13/08/2019 (Shift-I) Ans. (b) : For the above underlined part ‘are not clearly’ will be used because subject is plural hence plural verb will be used and adverb form ‘clearly’ will be used to modify visible (adjective). Correct sentence ''The new traffic signals are not clearly visible from a distance.'' 240. Select the most appropriate option to substitute the underlined segment in the given sentence. If there is no need to substitute it, select No Improvement. A study suggests that woman is good at handling emotions than men. (a) women is better (b) No improvement (c) woman are best (d) women are better SSC MTS – 07/08/2019 (Shift-III) Ans. (d) : For the above underlined part, ‘women are better’ will be used because here, women are compared to men and comparison is in same word. Hence plural form (women) will be used. Correct sentence A study suggests that women are better at handling emotions than men. Sentence Improvement

241. Select the most appropriate to substitute the underline segment. If there is no need to substitute it, select No improvement. One of the factor for her failure is the lack of concentration in studies. (a) one of the factors (b) each of the factors (c) one of a factor (d) No improvement SSC MTS–14/08/2019 (Shift-I) Ans. (a) : For the above underlined part, ‘one of the factors' will be used because plural noun/pronoun and singular verb is used after ‘one of’. One of + plural noun + singular verb

Correct sentence– One of the factors for her failure is the lack of concentration in studies. 242. Select the most appropriate option to substitute the underlined in the given sentence. If there is no need to substitute it, select 'No substitution required'. The novels of Jane Austen give us a glimpse of the world of women. (a) has given the (b) No substitution required (c) gave them the (d) gives us a SSC GD – 08/03/2019 (Shift-III) Ans. (b) : No substitution required 243. Select the most appropriate option to substitute the underlined in the given sentence. If there in no need to substitute it, select 'No substitution required'. Scientific discoveries is often result of hard work by many people. (a) are resulted (b) is resulted (c) are often a resulted (d) No substitution required SSC GD – 08/03/2019 (Shift-II) Ans. (c) : For the above underlined part, ‘are often a resulted’ will be used because subject is plural. Hence plural verb (are) will be used. Correct Sentence– Scientific discoveries are often resulted of hard work by many people. 244. Select the most appropriate option to substitute the underlined segment in the given sentence. If there is no need to substitute it, select No substitution Regular exercise lead to fitness of the body. (a) leads to fitness of the body (b) will leads to fitness of the body (c) leading to fitness of the body (d) No improvement SSC GD – 18/02/2019 (Shift-III) Ans. (a) : For the above underlined part, ‘leads to fitness of the body’ will be used because subject (Regular exercise ) is singular. Hence singular verb will be used. Correct Sentence– Regular exercise leads to fitness of the body.

130

YCT

245. Select the most appropriate option to substitute the underlined segment in the given sentence. If there is no need to substitute it, select No improvement. Yesterday eleven coaches of a train headed for Delhi was derailed at Vaishali district of Bihar. (a) was derailed in (b) were derailed in (c) No improvement (d) were derail at SSC GD – 14/02/2019 (Shift-II) Ans. (b) : For the above underlined part, ‘were derailed in’ will be used because subject is plural Hence plural verb will be used. Correct Sentence Yesterday eleven coaches of a train headed for Delhi were derailed in Vaishali district of Bihar. 246. Select the most appropriate option to substitute the underlined segment in the given sentence. If there is no need to substitute it, select No improvement. The furniture in this room are old and need to be replaced. (a) No improvement (b) is old and needs (c) is old and need (d) are old and needing SSC GD – 11/02/2019 (Shift-III) Ans. (b) : For the above underlined part, ‘is old and needs’ will be used because subject (the furniture) is singular. Hence singular verb will be used. Correct Sentence The furniture in this room is old and need to be replaced.

C.

Conditional Sentence

247. Select the most appropriate option that can substitute the underlined segment in the given sentence. Hardly we had boarded the train when it started to move. (a) we are boarded (b) had we boarded (c) will we boarded (d) is we boarded SSC CGL (Tier-I) 21/07/2023 (Shift-II) Ans. (b) : The most appropriate substitute the underlined segment in the given sentence 'we had boarded' is option (b) 'had we boarded,' because these sentence is past perfect tense and inversion is used with pair confunction 'hardly/scarcely……when, Inversion rules[Hardly/scarcely + had + subject + VIII + object + when + other clause.] Correct SentenceHardly had we boarded the train when it started to move. 248. Select the most appropriate option to substitute the underlined segment in the given sentence. If there is no need to substitute it, select 'No substitution'. If you had learned French, you had gone to France for studies. (a) would have gone (b) No substitution (c) would go (d) will bo SSC Stenographer (Grade C & D) 12.11.2021 Shift-I Sentence Improvement

Ans. (a) : For the underlined part, ‘would have gone’ will be used because sentence is conditional. Structure of the conditional sentence is belowIf + Sub. + had + V3 + Obj., Sub. + would have + V3 + Obj. Correct Sentence If you had learned French, you would have gone to Frence for studies. 249. Select the most appropriate option to improve the underlined segment in the given sentence. If there is no need to improve it, 'No improvement required'. You might spot a tiger in the Sariska reserve if you would be lucky. (a) if you are (b) if it was (c) then you are (d) No improvement required SSC Stenographer (Grade C & D) 11.11.2021 Shift-II Ans. (a) : For the above underlined part, 'if you are' will be used because sentence is in real condition. Present Indefinite If + , Condition 250. Select the option that will improve the underlined part of the given sentence. In case no improvement is needed, select 'No improvement required'. In his memoir, Attenborough said that, "Lisa would have been 11 if she was with us today." (a) Attenborough said that, "Lisa will have been 11 if she were (b) Attenborough said, "Lisa would have been 11 if she were (c) Attenborough said, "Lisa would have been 11 if she was (d) No improvement required SSC MTS-06/07/2022 Shift-III Ans. (b) : for the above underlined part, ‘Attenborough said, "Lisa would have been 11 if she were" will be used because sentence is conditional. Structure of the conditional sentence is belowIf + sub + were, sub + would have + v3 + obj. Correct Sentence – In his memoir, Attenborough said that, "Lisa would have been 11 if she were with us 251. Select the option that will improve the underlined part of the given sentence. In case no improvement is needed, select 'No improvement required'. Since he is strong, he is quite timid. (a) Although (b) No improvement required (c) Because (d) Besides SSC Constable GD-25/11/2021 Shift-III Ans. (a) : For the above underlined part, ‘Although’ will be used because ‘Although’ is used to show contrast sense. Correct sentence – Although he is strong, he is quite timid.

131

YCT

252. Select the option that will improve the underlined part of the given sentence. In case no improvement is needed, select 'No improvement required'. Unless we sat in an incorrect posture, we may strain our back. (a) Although we are sitting (b) Until we sit (c) No improvement required (d) If we sit SSC Constable GD-17/11/2021 Shift-II Ans. (d) : For the above underlined part, ‘if we sit’ will be used because it gives suitable meaning for the sentence. Correct Sentence- if we sit in an incorrect posture, we may strain our back. 253. Select the option that will improve the underlined part of the given sentence. In case no improvement is needed, select 'No improvement required'. Father asked me that if I will accompany him to the park. (a) whether I will (b) No improvement required (c) if I would (d) that if I would SSC Constable GD-02/12/2021 Shift-I Ans. (c) : For the above underline part, ‘if I would’ will be used because the given sentence is in Conditional hence ‘that’ will not be used and past form of will (would) will be used. Correct sentence Father asked me if I would accompany him to the park. 254. Select the most appropriate option that can substitute the underlined segment in the given sentence. If there is no need to substitute it, select 'No substitution required'. If that mango falls it has landed on your head. (a) No substitution required (b) will land (c) was landing (d) is landing SSC Constable GD-10/12/2021 Shift-III Ans. (b) : For the above underlined part, ‘will land’ will be used because of conditional sentence. In these kinds of sentences, if ‘if clause’ is in present indefinite then second clause will be in future indefinite. Correct sentence:– If that mango falls it will land on your head. 255. Select the most appropriate option that can substitute the underlined segment in the given sentence. If there is no need to substitute it, select 'No substitution required'. If the air pollutes with small particles, natural or otherwise, the sunset will be more red. (a) will be polluted with (b) is polluted with (c) No substitution required (d) was polluted with SSC Constable GD-07/12/2021 Shift-II Sentence Improvement

Ans. (b) : for the above underlined part, ‘is polluted with’ will be used because when principal clause is in future indefinite, then subordinate clause will be in present indefinite tense. Correct sentence – If the air is polluted with small particles, natural or otherwise, the sunset will be more red. 256. Select the most appropriate option to substitute the underlined segment in the given sentence. If there is no need to substitute it, select ‘No substitution required’. Had I a rich person, I would share my wealth. (a) were I (b) No substitution required (c) I am (d) Had I been SSC CPO-SI – 23/11/2020 (Shift-I) Ans. (a) : For the above underlined part ‘were I’ will be used because of conditional sentence. Structure of the conditional (II) is belowIf + V2/were form, sub + would/could + V1 form. If + were/V II from, would/could + V I from were + subject, would/could+V I form

Correct Sentence– Were I a rich person, I would share my wealth. 257. Select the most appropriate option to substitute the underlined segment in the given sentence. If there is no need to substitute it, select No improvement. He has been solving this sum for a couple of hours but he did not solve it yet. (a) No improvement (b) has not solved it (c) has not solve it (d) had not solved it SSC CHSL (Tier-I) –03/07/2019 (Shift-II) Ans. (b) : For the above underlined part, ‘has not solved' will be used because sentence is in present tense. Hence present perfect tense (has) will be used. Correct sentence He has been solving this sum for a couple of hours but he has not solved it yet. 258. Select the most appropriate option to substitute the underlined segment in the given sentence. If there is no need to substitute it, select No improvement. There was a long queue at the post office and by the time I have reached the counter I was told the printer was not working. (a) I reached the counter (b) No improvement (c) I reach the counter (d) I am reaching the counter SSC CHSL (Tier-I) –10/07/2019 (Shift-I) Ans : (a) For the above underlined part, 'I reached the counter' will be used because principle clause is in past tense. Hence subordinate clause will also be in past tense. Correct Sentence There was a long queue at the post office and by the time I reached the counter I was told the printer was not working.

132

YCT

259. Select the most appropriate option to substitute the underlined segment in the given sentnce. If no substitution is required, select No improvement. Scarcely had he gone out than a client came to meet him. (a) he had gone out than (b) No improvement (c) he had go out when (d) had he gone out when SSC CGL (Tier-I) – 13/06/2019 (Shift-III) Ans : (d) For the above underlined part, ‘Had he gone out when’ will be used because when is used with scarcely. Correct Sentence– Scarcely had he gone out when a client came to meet him. 260. Selected the most appropriate option to substitute the underlined segment in the given sentence. If no substitution is required, select No improvement. When we went to the cinema yesterday, the film had already start. (a) the film had already started (b) the film have already start (c) No improvement (d) the film was already start SSC CGL (Tier-I) – 12/06/2019 (Shift-III) Ans. (a) : For the above underlined part, 'the film had already started' will be used because 'V3' form is used with had/has/ have. Correct Sentence When we went to the cinema yesterday, the film had already started. 261. Select the most appropriate option to substitute the underlined segment in the given sentence. If no substitution is required, select No improvement. I am think that tomorrow I will take leave and stay at home. (a) No improvement (b) I was thinking that the next day I will be taking (c) I think that tomorrow I will take (d) I have thought that tomorrow I am taking SSC CGL (Tier-I) – 12/06/2019 (Shift-III) Ans. (c) : For the above underlined part, ‘ I think that tomorrow’ will be used because when subordinate clause is in future indefinite then principle clause should be in present indefinite tense. Correct Sentence– I think that tomorrow I will take leave and stay at home. 262. Select the most appropriate option to substitute the underlined segment in the given sentence. If no substitution is required, select No improvement. If you listen to the English news, it improve your English. (a) it would improve (b) No improvement (c) it improved (d) it is improving SSC CGL (Tier-I) – 06/06/2019 (Shift-I) Sentence Improvement

Ans. (a) : For the above underlined part. It would improve will be used because given sentence is conditional sentence of past tense. Hence if + subject + V2, subject + would + V1 structure will follow. Correct Sentence If you listened to the English news, It would improve your English. 263. Select the most appropriate option to substitute the underlined segment in the given sentence. If no substitution is required, select No improvement. If you park your car here, the traffic police has fined you. (a) fine you (b) will fine you (c) fined you (d) No improvement SSC CGL (Tier-I) – 04/06/2019 (Shift-I) Ans : (b) For the underlined part ‘ will fine you’ will be used because the sentence is conditional sentence typeI and its structure is . If +Present Indefinite+ Future Indefinite

Correct Sentence If you park your car here, the traffic police will fine you. 264. Select the most appropriate option to substitute the underlined segment in the given sentence. If no substitution is required, select No improvement. If you join this job now, it proves to be good in the long run. (a) No improvement (b) it has proved to be good (c) it proves good (d) it will prove to be good SSC CGL (Tier-I) – 04/06/2019 (Shift-II) Ans : (d) For the underlined part “It will prove to be good” will be used because it is conditional type-I sentence and its structure isIf +Present Indefinite+ Future Indefinite

Correct Sentence– If you join this job now, it will prove to be good in the long run. 265. Select the most appropriate option to substitute the underlined segment in the given sentence. If no substitution is required, select No improvement. If I have money, I purchase this house. (a) I purchased (b) I will purchase (c) No improvement (d) I have purchased SSC CGL (Tier-I)–04/06/2019 (Shift-III) Ans. (b) : For the underlined part “I will purchase” will be used becauseNote-

133

(i) If + Present Tense - will + verb1 (ii) If + Past Tense - would + verb1 (iii) If + Past Perfect - would have + VIII Correct Sentence If I have money, I will purchase this house. YCT

266. Select the most appropriate option to substitute Ans. (a) : For the underlined part “ had been doing” the underlined segment. If no substitution is will be used because subordinate clause of the sentence required select 'No substitution' is a conditional clause. If I would have a degree from the university, I Correct Sentence would get a good job. My doctor knew that I would eventually recover and do (a) No substitution the kind of work I had been doing before. (b) If I had a degree 270. Select the most appropriate option to substitute (c) When I will have a degree the underlined segment in the given sentence. If (d) If I am having the degree there is no need to substitute it, select 'No SSC CPO-SI – 11/12/2019 (Shift-II) improvement'. Ans : (b) For the underlined part “If I had a degree” While my parents talked to the visitor, I made will be used because it is conditional Type – II sent and tea in the kitchen. its structure is(a) parents have talked (b) No improvement If + past (V2 ) (had), would + verb1 (c) parents were talking (d) parents are talking SSC CHSL (Tier-I) 20/10/2020 (Shift-I) Correct sentence Ans. (c) : For the underlined part parents were talking If I had a degree from the university, I would get a good will be used because when two past actions are denote job. 267. Select the most appropriate option to substitute in a sentence. If you have gone then verb past the underlined segment. If no substitution is continuous for the work that is already going on and verb past for the work that is going to happen later is in required select 'No substitution'. If you will bring a letter of introduction, I will the indefinite. Correct Sentence consider your request. While my parents were talking to the visitor, I made (a) If you bring tea in the kitchen. (b) If you brings (c) If you will be bringing 271. Select the most appropriate option to substitute (d) No substitution the underlined segment in the given sentence. If SSC CPO-SI – 12/12/2019 (Shift-I) there is no need to substitute it, select No Improvement. Ans : (a) For the underlined part “if you bring” will be When the little girl loses her doll, she began to used because it is conditional type-I sentence and its cry. structure is(a) loss her doll (b) No Improvement If +Present Indefinite+ Future Indefinite (c) losing her doll (d) lost her doll Correct Sentence– SSC MTS – 06/08/2019 (Shift-I) If you bring a letter of introduction, I will consider your Ans. (d) : For the underlined part "Lost her doll" request. phrase will be used because the action of the principal 268. Select the most appropriate option to substitute clause is in past tense 'began' (V2) Hence subordinate the underlined segment in the given sentence. If clause is also in past tense (last). there is no need to substitute it, select No. Correct Sentence– improvement. She has been writing this novel for a year but When the little girl lost her doll, she began to cry. 272. Select the most appropriate option to substitute she did not finish it yet. the underlined segment in the given sentence. If (a) had not finished it (b) has not finish it there is no need to substitute it, select No (c) No. improvement (d) has not finished it Improvement. SSC CHSL (Tier-I) –03/07/2019 (Shift-I) If he would tried he would have succeeded. Ans : (d) For the underlined part “has not finished it” (a) was tried (b) No Improvement will be used because negative sentence of present (c) had tried (d) is tried perfect is also used ‘yet’. SSC MTS – 05/08/2019 (Shift-II) Correct Sentence Ans. (c) : For the underlined part ‘had tried’ will be She has been writing this novel for a year but she has used because given sentence is conditional sentence not finished it yet. 269. Select the most appropriate option to substitute type – II and its structure is IIIrd the underlined segment in the given sentence. If If + sub + past perfect, sub + would have + v form Correct Sentence If he had tried he would have there is no need to substitute it, select ‘No succeeded. improvement’. My doctor knew that I would eventually 273. Select the most appropriate option to substitute recover and do the kind of work I would be the underlined segment in the given sentence. If doing before. there is no need to substitute it, select No (a) had been doing Improvement. (b) would have been doing He did not like the novel, nor I did. (c) would have done (a) nor did I (b) No Improvement (d) No improvement (c) nor I like it (d) nor did I likes it SSC MTS – 02/08/2019 (Shift-I) SSC CHSL (Tier-I) 16/10/2020 (Shift-I) Sentence Improvement

134

YCT

Ans. (a) : For the underlined ‘nor did I’ will be used because nor is generally followed by rule of inversion. Inversion is - putting the verb before the subject. Correct Sentence– He did not like the novel, nor did I. 274. Select the most appropriate option to substitute the underlined segment in the given sentence. If there is no need to substitute it, select No improvement. The meeting will start when the manager is arriving. (a) arrives (b) No improvement (c) arrived (d) was arrived SSC MTS – 21/08/2019 (Shift-III) Ans. (a) : For the underlined “arrives” will be used because given sentence is conditional sentence type-I and in which main clause is Simple future tense and subordinate clause is simple present tense. It structure isIf/when+ present Indefinite tense+Future Indefinite Correct Sentence– The meeting will start when the manager arrives. 275. Select the most appropriate option to substitute the underlined segment in the given sentence. If there is no need to substitute it, select No improvement. Her dance teacher have come twice a week but there is no improvement in her dance. (a) No improvement (b) will be coming (c) has been coming (d) were coming SSC MTS – 21/08/2019 (Shift-I) Ans : (c) For the underlined part teacher “has been coming” will be used because when any action start in the past and continuous in the present in regular time gap, the sentence used is Present Perfect continuous tense. Correct Sentence– Her dance teacher has been coming twice a week but there is no improvement in her dance. 276. Select the most appropriate option to substitute the underlined segment. If there is no need to substitute it select No improvement. I will not be able to leave for home until my boss arrived in the office. (a) untill my boss arrives at the office (b) No improvement (c) until my boss has arrive to the office (d) untill my boss is arrived in the office SSC MTS – 20/08/2019 (Shift-I) Ans. (a) : For the underlined part ‘arrives’ will be used because in the conditional sentence of future indefinite tense the clause stating the condition (which starts with if, as if , until, unless) used in present indefinite tense. Correct Sentence–I will not be able to leave for home until my boss arrives in the office. 277. Select the most appropriate option to substitute the underlined segment. If there is no need to substitute it, select No improvement. Hardly he had entered the station, when the train whistled. Sentence Improvement

(a) (b) (c) (d)

hardly he entered hardly had he entered No improvement he hardly had entered SSC MTS – 14/08/2019 (Shift-I) Ans. (b) : For the underlined part 'hardly had he entered' will be used because hardly, scarcely, no sooner, when used in beginning of the sentence then the method of inversion is applicable and the verb is used before subject. Correct Sentence–Hardly had he entered the station, when the train whistled. 278. Select the most appropriate option to substitute the underlined segment. If there is no need to substitute it, select No improvement. If I was you I would not sign the document. (a) No improvement (b) If I were you (c) If I have been you (d) If I had been you SSC MTS – 14/08/2019 (Shift-I) Ans. (b) : For the underlined part ‘If I were’ will be used because the subject of the sentence shows imaginary situation and in Hypothetical sentence if subject is singular or plural both situation were is used as a helping verb. If + were/II form of verb, would + V I form of verb

Correct Sentence– If I were you I would not sign the document.

D.

Question Tag

279. Select the most appropriate option to substitute the underlined segment in the given sentence. If there is no need to substitute it, select 'No substitution'? CV Raman became famous when he got the Nobel Prize, hadn't he? (a) didn't he (b) wasn't he (c) isn't it (d) No substitution SSC Stenographer (Grade C & D) 12.11.2021 Shift-II Ans. (a) : For the underlined part ‘didn’t he’ will be used because question tag of Negative sentence is positive and question tag of positive sentence is Negative. Correct sentence–CV Raman became famous when he got the Nobel Prize, didn't he? 280. Select the most appropriate option to substitute the underlined segment in the given sentence. If there is no need to substitute it, select 'No substitution'. You have booked your seats in the next flight, have you? (a) did you (b) haven't you (c) isn't it (d) No substitution SSC Stenographer (Grade C & D) 15.11.2021 Shift-II Ans. (b) : For the underlined part ‘haven't’ you’ will be used because question tag of positive sentence is negative and Negative sentence is positive. Correct Sentence You have booked your seats in the next flight, haven't you?

135

YCT

281. Select the most appropriate option to substitute the underlined segment in the given sentence. If there is no need to substitute it, select 'No substitution'. He can't climb this tree, can't he? (a) No substitution (b) can he (c) isn't it (d) couldn't he SSC Stenographer (Grade C & D) 15.11.2021 Shift-I Ans. (b) : question tag of Negative sentence is positive and positive sentence is negative. Hence for the underline part ‘Can he’ will be used. 282. Select the most appropriate option that can substitute the underlined segment in the given sentence. If there is no need to substitute it, select ' No substitution required.' I imagine you have learnt a valuble lesson from this experience, didn't you? (a) Have you ? (b) No substitution required (c) did you? (d) Haven't you? SSC CGL (Tier-I) 13/04/2022 Shift-I Ans. (d) : In the given sentence, 'didn't you' will be substituted by 'haven't you' because the question tag of 'you have' is 'haven't you' because sentence is in positive hence question tag will be negative. Hence option (d) is correct. 283. Select the most appropriate option that can substitute the underlined segment in the given sentence. If there is no need to substitute it, select ‘No substitution required’. During foggy weather, flights are often cancelled, isn’t it? (a) aren't they (b) No substitution required (c) haven't they (d) do they SSC Constable GD-24/11/2021 Shift-II Ans. (a) : For the above underlined part “aren’t they” will be used because question tag of 'are' is 'aren’t'. Correct Sentence During foggy weather, flights are often cancelled, aren't they ? 284. Select the option that will improve the underlined part of the given sentence. In case no improvement is needed, select 'No improvement required'. You are going abroad for studies, are you ? (a) No improvment required (b) wasn't you (c) isn't it (d) aren't you SSC Constable GD-02/12/2021 Shift-I Ans. (d) : For the above underlined part aren’t you will be used because question tag of 'are' is 'are n't'. Correct Sentence→ you are going abroad for studies, aren't you? Sentence Improvement

285. Select the option that will improve the underlined part of the given sentence. In case no improvement is needed, select 'No improvement required'. Your brother can't drive, is it ? (a) No improvement required (b) couldn't he (c) can he (d) will he SSC Constable GD-08/12/2021 Shift-I Ans. (c) : For the above underlined part 'can he' is used because question tag of 'can’t' is 'can'. Correct SentenceYour brother can't drive, can he? 286. Select the most appropriate option that can substitute the underlined sagment in the given sentence. If there is no need to substitute it, select 'No substitution required'. They have decorated their house nicely, didn't they? (a) No substitution required (b) have they (c) haven't they (d) did they SSC Constable GD-01/12/2021 Shift-I Ans. (c) : For the above underlined part 'haven’t they' will be used because question tag is used according to subject and helping verb. Correct Sentence– They have decorated their house nicely, haven't they? 287. Select the most appropriate option to substitute the underlined segment in the given sentence. If there is no need to substitute it, select 'No substitution'. You have locked the gate, isn't it? (a) haven't you (b) did you (c) No substitution (d) have you SSC CHSL (Tier-I) –15/04/2021 (Shift-I) Ans. (a) : For the underlined part ‘haven’t you’ will be used because question tag of positive sentence is negative. And pronoun of question tag used according to the subject of the sentence. Correct Sentence You have locked the gate, haven't you. 288. Select the most appropriate option to improve the underlined segment in the given sentence. If there is no need to improve it, select 'No improvement'. She is not ready for marriage, isn't it? (a) No improvement (b) is it (c) isn't she (d) is she SSC CGL (Tier-II) 16/11/2020 (3 pm - 5 pm) Ans. (d) : For the underlined part ‘is she’ will be used because question tag of negative sentence is positive. And pronoun of question tag used according to the subject of the sentence. Correct Sentence - She is not ready for marriage, is she.

136

YCT

289. Select the alternative that will improve the underlined part of the sentence in case there is no improvement select “No improvement". You have my mobile number, isn't it? (a) don’t you (b) No improvement (c) do you (d) has you SSC CPO-SI (Tier-II) 27/09/2019 (3 pm - 5 pm) Ans. (a) : For the underlined part ‘don’t you’ will be used because question tag is used according to the helping of verb and subject of the sentence and the sentence is present indefinite helping verb do/does. Correct Sentence - You have my mobile number, don't you? 290. Select the alternative that will improve the underlined part of the sentence. In case there is no improvement select "No improvement". "Let's go for a movie this weekend, shall we?" (a) can't we? (b) No improvement (c) shan't we? (d) would we? SSC Stenographer Grade C&D –05/02/2019 (3:5pm)

Ans. (b) : No improvement Note- 'Shall we' is the question tag of 'let'. 291. Select the most appropriate option to substitute the underlined segment in the given sentence. If there is no need to substitute it, select 'No substitution required'. She will have to go to another school when she moves from here, has she? (a) hasn’t she (b) No substitution required (c) will she (d) won't she SSC CPO-SI – 25/11/2020 (Shift-I) Ans. (d) : For the above underlined part ‘won’t she’ will be used because question tag of positive sentence is negative and negative sentence is positive. Correct sentecneShe will have to go to another school when she moves from here, won't she? 292. Select the most appropriate option to substitute the underlined segment in the given sentence. If no substitution is required, select No improvement. He came late, wasn't it? (a) didn't he (b) did he (c) No improvement (d) isn't it SSC CGL (Tier-I) – 07/06/2019 (Shift-I) Ans : (a) For the above underlined part ‘didn’t he’ will be used because given sentence is affirmative of past Indefinite tense. Hence its question tag is negative form of part indefinite tense. Correct Sentence He came late, didn’t he? 293 Select the most appropriate option to substitute the underlined segment in the given sentence. If no substitution is required, select No improvement. We can't live without water, will we? (a) won't we (b) can we (c) do we (d) No improvement SSC CGL (Tier-I) – 07/06/2019 (Shift-II) Sentence Improvement

Ans : (b) For the underlined part 'can we' will be used because the given question tag is based on modal Auxiliary verb Negative sentence in of can so its question tag will be positive sentence since the verb of the sentence is can then the question. (can + subject). Correct Sentence– We can't live without water, can we? 294. Select the most appropriate option to substitute the underlined segment in the given sentence. If no substitution is required, select No improvement You won't tell the secret, won't you? (a) No improvement (b) would you (c) will you (d) isn't it SSC CGL (Tier-I) – 06/06/2019 (Shift-II) Ans : (c) For the underlined part ‘will you’ will be used because won’t (Negative) is used in the sentence. Hence question tag positive (will you) is used. Correct Sentence You won't tell the secret, will you? 295. Select the most appropriate option to substitute the underlined segment in the given sentence. If no substitution is required, select No improvement. They didn't see you, is it? (a) no improvement (b) did they (c) wasn't it (d) have they SSC CGL (Tier-I) – 06/06/2019 (Shift-III) Ans. (b) : For the underlined part ‘did they’ will be used because the sentence and question tag is in same tense. Correct Sentence– They didn't see you, did they? 296. Select the most appropriate option to substitute the underlined segment in the given sentence. If there is no need to substitute it, select 'No substitution'. The tree was uprooted by the storm last evening, isn't it? (a) didn't it (b) wasn't it (c) was it (d) No substitution SSC CPO-SI – 09/12/2019 (Shift-I) Ans : (b) For the underlined part ‘ wasn’t it’ will be used because question tag of positive sentence is negative and verb use in same sentence. Correct Sentence– The tree was uprooted by the storm last evening, wasn't it? 297. Select the most appropriate option to substitute the underlined segment in the given sentence. If there is no need to substitute it, select 'No substitution' You have finished your work, have you? (a) haven't you (b) didn't you (c) No substitution (d) did you SSC CPO-SI – 11/12/2019 (Shift-I) Ans : (a) For the underlined part ‘ Haven’t you’ will be used because question tag of affirmative sentence is negative. Correct Sentence You have finished your work, haven't you?

137

YCT

298. Select the most appropriate option to substitute the underlined segment in the given sentence. If there is no need to substitute it, select ‘No improvement’. We've had a long day. We should rest now, shouldn't we? (a) hadn’t we (b) haven't we (c) No improvement (d) won't we SSC CHSL (Tier-I) 16/10/2020 (Shift-II) Ans. (c) : No improvement 299. Select the most appropriate option to substitute the underlined segment in the given sentence. If there is no need to substitute it, select 'No improvement'. Avika understands French, does she? (a) is she (b) No improvement (c) doesn't she (d) isn't SSC CHSL (Tier-I) 19/10/2020 (Shift-III) Ans. (c) : For the underlined part “doesn’t she” will be used because question tag of positive sentence is negative since the sentence is in present indefinite so indefinite verb “does” is used. Correct Sentence– Avika understands French, doesn't she? 300. Select the most appropriate option to substitute the underlined segment in the given sentence. If there is no need to substitute it, select 'No improvement'. The house has a garden in front, is it? (a) No improvement (b) does it (c) hasn't it (d) isn't it SSC CHSL (Tier-I) 19/10/2020 (Shift-II) Ans. (c) : for the underlined part “hasn’t It” will be used because question tag of positive sentence is Negative and Negative sentence is positive. Correct Sentence– The house has a garden in front, hasn't it. 301. Select the most appropriate option to substitute the underlined segment in the given sentence. If there is no need to substitute it, select 'No improvement'. She is quite capable of looking after herself, aren't she? (a) wasn't she (b) No improvement (c) isn't she (d) won't she SSC CHSL (Tier-I) 14/10/2020 (Shift-II) Ans. (c) : For the underlined part “isn’t she” will be used because question is used according to the helping verb and subject of the sentence and question tag of positive sentence is negative and negative sentence is positive. Correct Sentence - She is quite capable of looking after herself, isn't she? 302. Select the most appropriate option to substitute the underlined segment. If there is no need to substitute it, select No improvement He does not tell lies, doesn't he ? (a) does he (b) isn't he (c) No improvement (d) didn't he SSC MTS – 14/08/2019 (Shift-II) Ans. (a) : For the underlined part “does he” will be used because sentence is negative. Hence question tag will be positive. Correct Sentence– He does not tell lies, does he ? Sentence Improvement

E.

Article

303. Select the most appropriate option to substitute the segment in the given sentence. If there is no need to substitute it, select 'no substitution required'. Chia is currently pursuing a (master's in developmental studies in Netherlands). (a) master's in developmental studies in the Netherlands. (b) master's in developmental studies at Netherlands (c) No substitution required (d) master's in developmental studies in the Netherlands. SSC MTS 08/05/2023 (Shift-III) Ans. (d) : The most appropriate option to substitute the Segment is option (d) 'master's in developmental studies in the Nether lands' and article 'The' is used before country name ' Netherlands' because it is a group of many provinces. These, the correct answer is option (d). 304. Select the option that rectifies the underlined part of the given sentence. In case no correction is needed, select ‘Nocorrection required’. Cinema provides the most universal entertainment. (a) a universal (b) more universal (c) an universal (d) No correction required SSC CGL Mains 26/10/2023 (Shift-I) Ans. (a) : In the above sentence 'a universal' will be used in place of underlined part 'the most universal because article 'the' and superlative degree 'most' is not used with universal. Correct Sentence Cinema provides a universal entertainment. 305. Select the option that will improve the underlined part of the given sentence. In case no improvement is needed, select 'No improvement required'. The movie theatre is a excited place where people can relax and enjoy the experience. (a) No improvement required (b) exciting (c) an exciting (d) the excited SSC Constable GD-17/11/2021 Shift-II Ans. (c) : For the above underlined part 'an exciting' will be used because indefinite article 'an' is used before vowel sound. Correct SentenceThe movie theatre is an exciting place Where people can relax and enjoy the experience. 306. Select the most appropriate option that can substitute the underlined segment in the given sentence. If there is no need to substitute it, select 'No substitution required'.

138

YCT

Delhi is a most affected city by the virus nowadays. (a) Delhi was a most affected (b) Delhi is the most affected (c) No substitution required (d) Delhi has a most affected SSC Constable GD-02/12/2021 Shift-I Ans. (b) : For the above underlined part “Delhi is the most affected” will be used because definite article ‘the’ is used before superlative degree “most”. Correct sentence Delhi is the most affected city by the virus Nowadays. 307. Select the most appropriate option that can substitute the underlined segment in the given sentence. If there is no need to substitute it, select ‘No substitution required’. When news of the concert leaks out, there will be a rush for the ticket. (a) the rush for the tickets (b) a rush for tickets (c) No substitution required (d) a rush for a ticket SSC Constable GD-14/12/2021 Shift-III Ans. (b) : For the underlined part “a rush for ticket” will be used because use of ticket in the sentence is in general sense. Hence definite article ‘the’ will not be used. Correct sentence– When news of the concert leaks out, there will be a rush for tickets. 308. Select the option that will improve the underlined part of the given sentences. In case no improvement is needed select 'No improvement required'. A need of an hour is to provide employment to the youth of the country. (a) The need in an (b) The need of an (c) The need of the (d) No improvement required SSC CGL (Tier-I) –24/08/2021 (Shift-I) Ans. (c) : For the underlined part ‘the need of the’ will be used because the construction of the sentence is “Noun + Preposition + Noun” them article ‘the’ is used before first Noun and in the sentence discuss about specific time period. Hence definite article ‘the’ will be used. Note:- Definite article ‘the’ is used emphasize for the sentence. Correct Sentence The need of the hour is to provide employment to the youth of the country. 309. Select the most appropriate option to substitute the underlined segment in the given sentence. If no substitution is required, select 'No substitution'. The apple a day keeps the doctor away. (a) No substitution (b) The apples a day (c) A apple a day (d) An apple a day SSC CGL (Tier-I) –20/08/2021 (Shift-I) Sentence Improvement

Ans. (d) : For the underlined part ‘An apple a day’ will be used because before indefinite Noun, whose pronunciation is a vowel sound, indefinite article ‘an’ is used. Correct Sentence An apple a day keeps the doctor away. 310. Select the most appropriate option to substitute the underlined segment in the given sentence. If there is no need to substitute it, select 'No substitution'. Rahul thinks that this is quite the cheap restaurant. (a) so a cheap (b) so the cheap (c) No substitution (d) quite a cheap SSC CPO-SI – 09/12/2019 (Shift-I) Ans : (d) In the underlined Phrase for the indefinite noun ‘restaurant’, it is appropriate to used indefinite article ‘a’ in place of define article ‘the’. Correct Sentence– Rahul thinks that this is quite a cheap restaurant. 311. Select the most appropriate option to substitute the underlined segment in the given sentence. If there is no need to substitute it, select No improvement. Durga is now the Executive Vice-President of the company with a responsibility to marketing for the entire northern zone. (a) with the responsibility of marketing (b) No improvement (c) with a responsibility by marketing (d) with responsibility from marketing SSC CHSL (Tier-I) –10/07/2019 (Shift-II) Ans : (a) For the underlined part ‘the and of’ will be used respectively because when two nouns are connected with ‘of’ then article ‘the’ is used before first Noun and to showing possession ‘of’ will be used. Hence the responsibility of marketing is appropriate. Correct Sentence– Durga is now the Executive Vive-President of the company with the responsibility of marketing for the entire northern zone. 312. Select the most appropriate option to substitute the underlined segment. If there is no need to substitute it, select No improvement. Bulbul is honest girl but she is also very rude. (a) Bulbul is an honest girl (b) Bulbul is very honest girl (c) No improvement (d) Bulbul is a honest girl SSC MTS – 19/08/2019 (Shift-III) Ans. (a) : For the underlined part “ Bulbul is an honest girl” will be used because article ‘an’ is used before vowel sound word (honest) because after this indefinite noun (girl) is used. Correct sentence– "Bulbul is an honest girl but she is also very rude.''

139

YCT

F.

Correct use of Pronoun, Adjective & Adverb

313. Select the most appropriate option to substitute the bracketed segment in the given sentence. If there is no need to substitute it, select 'no substitution required'. (In fact, climbing fem is spreading so rapidly that its) now the state's worst invasive weed. (a) No substitution required (b) In fact, climbing fern is spreading so rapidly that it's (c) In fact, climbing fern is spread so rapidly that is (d) In fact, climbing fern is spreading so rapid that it's SSC MTS 10/05/2023 (Shift-I) Ans. (b) : The most appropriate option to substitute the bracketed segment in the given sentence is option (b) because 'that its' will be replaced by 'that it's'. Here it's gives sense of it is, not possessive adjective (its). Thus, the correct answer is option (b). 314. Select the most appropriate option to substitute the bracketed segment in the given sentence. If there is no need to substitute it, select 'no substitution required'. The unit twitched like a bird (preening it's plumage and becomes motionless). (a) No substitution required (b) preening its plumage and became motionless (c) to preen it's plumage and becomes motionless (d) preening it's plumage while becoming motionless SSC MTS 02/05/2023 (Shift-I) Ans. (b) : The most appropriate option to substitute the bracketed segment is option (b) 'preening its plumage and became motionless' will be used in the place of bracketed segment. The word 'twitched' indicated that past indefinite tense (S + V2 + O) so verb2 'became' will be used and pronoun it's changed into 'Possessive adjective' 'its'. Thus, the correct answer is option (b). 315. Select the word segment that substitutes (replaces) the bracketed word segment correctly and completes the sentence meaningfully. Select the option 'no correction required if the sentence is correct as given. The dessert was not just extremely hot but (also to sweet as well). (a) No correction required (b) also being too sweet (c) Also too sweet (d) besides too sweet as well SSC MTS 19/05/2023 (Shift-III) Ans. (c) : The bracketed word of the above sentence(also to sweet as well) has grammatically error, because adverb ‘too’ is used, in the place of infinitive ‘to’ and remove the ‘as well’. Thus, the correct segment is option (c) ‘Also too sweet’. Sentence Improvement

316. Improve the underlined part of the sentence. Choose ‘No improvement’ as an answer if the sentence is grammatically correct. The teacher let the students choose there essay topics. (a) has choose their (b) No improvement (c) choose their (d) chosen there SSC MTS 15/05/2023 (Shift-I) Ans. (c) : The underline part of the given sentence is – 'choose there' which is grammatically incorrect, because possessive pronoun (their) is used before the noun (essay) in the place of 'there'. Hence, the correct option is (c). 317. Select the option that expresses the given sentence in positive degree of comparison. She is the dullest child in the class. (a) She is very duller in the class. (b) No other child in the class is as dull as she. (c) In the class of all she is dull. (d) Dullest child in the class is she. SSC CHSL (Tier-I) 02/08/2023 (Shift-I) Ans. (b) : The above sentence is in 'superlative degree; The positive degree of this sentence is option (b) 'No other child in the class is as dull as she'. Structure of 'superlative and positive degree' – Superlative– Sub + is/am/are + the + superlative adjective + others. Possitive– No other + noun + is/am/are + as + positive adj: + as + obj. 318. Improve the underlined part of the sentence. Choose 'No improvement' as an answer if the sentence is grammatically correct. The dessert was many sweet for my tastes. (a) No improvement (b) too sweetly (c) too sweet (d) more sweeter SSC MTS 12/05/2023 (Shift-III) Ans. (c) : The underlined part of the above sentence (Many sweet) is grammatically incorrect. The Correct option (c) 'too sweet' is used instead of 'many sweet'. Note– Many' is used before the 'countable noun'. Sweet is 'Uncountable noun' there for 'too' is used before the uncountable adjective. 319. Select the most appropriate option that can substitute the underlined segment in the given sentence. If there is no need to substitute it, select 'No substitution'. The travelling gypsies wanted to display they're skilful antics to the crowd. (a) their skilful antics (b) there skilful antics (c) No substitution (d) dare skilful antics SSC GD 13/02/2023 (Shift-I) Ans. (a) : The underlined segment in the given sentence, 'their skilful antics' would be appropriate in place of 'they are skilful antics'. Because before noun (skilful antics), possessive adjective 'their' is used. The correct sentence is– 'The travelling gypsies wanted to display their skilful antics to the crowd'.

140

YCT

320. Select the most appropriate option that can substitute the underlined segment in the given sentence. I don’t think he can dance as wellest Elena. (a) weller than (b) well than (c) as well as (d) wellest SSC CHSL (Tier-I) 03/08/2023 (Shift-II) Ans. (c) : The underlined segment of the above sentence 'as wellest' is grammatically error. The correct segment is option (c) 'as well as', because positive degree of comparison is used in the place of 'as wellest'. as +adj/adv. + as → positive degree Other options are not correct. 321. Improve the underlined part of the sentence. Choose ‘No improvement’ as an answer if the sentence is grammatically correct. The number of women working outside homes has been steady increasing. (a) has been steadily (b) had been steadier (c) No improvement (d) have been steady SSC CHSL (Tier-I) 21/03/2023 (Shift-IV) Ans. (a) : For the above underlined part of the sentence "has been steadily" will be used in place of "has been steady" because adverb (steadily) is needed to modify verb (increasing). Hence steadily (adverb) will be used in place of steady (adjective). Correct Sentence : The number of woman working outside homes has been steadily increasing. 322. Select the options that will improve the nderlined part of the sentence. In case no improvement is needed, select 'No improvement'. But what of the young people whom are part of the cricket crowds, some of who are presumbly born in England? (a) who are part of the cricket crowds, some of who are presumably born in England (b) No improvement (c) whom are part of he cricket crowds, some of whom are presumably born in England (d) who are part of the cricket crowds, some of whom are presumably born in England SSC CHSL (Tier-I) 21/03/2023 (Shift-II) Ans. (d) : In the above sentence, 'who' will be used in place of 'whom' and 'whom' will be used in place of 'who' because 'who' is used in subjective case while 'whom' is used in objective case. Correct Sentnce: But what of the young who are part of the cricket crowds some of whom are presumably born in England. 323. Select the option that will improve the (bracketed) part of the sentence. In case of no improvement is needed, select the option 'No improvement'. Check all radiators for (meagre leaks, special) round pipework connections. (a) small leaks, especially (b) No improvement (c) modest leaks, specially (d) cramped leaks, specially SSC CHSL (Tier-I) 17/03/2023 (Shift-III) Sentence Improvement

Ans. (a) : In the above bracketed Part of the sentence ' 'Small leaks, especially' will be used in place of 'meagre leaks, special' because it gives appropriate meaning. 324. Select the most appropriate option in the superlative degree that can substitute the underlined segment in the givensentence. This is the challenge task I have ever encountered. (a) more challenging (b) much challenging (c) challenging (d) most challenging SSC CGL Mains 26/10/2023 (Shift-I) Ans. (d) : In the above sentence 'the most challenging' will be used in place of challenge because adjective 'Challenging' is used to qualify noun 'Task' and superlative degree 'most' is used after article 'the'. Correct Sentence This is the most challenging task I have ever encountered. 325. Select the option that will improve the underlined part of the following sentence. He is taller and most handsome than his friend. (a) very more (b) more (c) the more (d) the most SSC CGL Mains 26/10/2023 (Shift-I) Ans. (b) : In the above sentence 'more' will be used in place of underlined word 'most' because 'and' follow same part of speech and than is used with comparative degree 'more'. Correct Sentence He is taller and more handsome than his friend. 326. Improve the underlined part of the sentence. Choose 'No improvement' as an answer if the sentence is grammatically correct. Add more flour if the dough is too stickiest. (a) too stickier (b) too sticky (c) No improvement (d) much stick SSC CHSL (Tier-I) 15/03/2023 (Shift-III) Ans. (b) : In the above underlined part of the sentence ''too sticky'' will be used is place of ''too stickiest'' because Positive degree adjective is used after 'too' Adverb while 'stickiest' is used as superlative degree. Correct sentenceAdd more flour if the dough is too sticky. 327. Improve the underlined part of the sentence. Choose 'No improvement' as an answer if the sentence is grammatically correct. The Blue House is practising it's songs for the competition. (a) No improvement (b) practised it's (c) practising its (d) practise its SSC CHSL (Tier-I) 09/03/2023 (Shift-IV) Ans. (c) : For the above Underlined Part of the sentence its will be used in place of it's because Possessive Adjective 'its' will be used before noun "songs" whereas it's (subject + verb) short form of 'It + is' which is followed by verb or Adjective. Correct Sentence The Blue House is practising its songs for the competition.

141

YCT

328. Select the option that will improve the underlined part of the given sentence. In case no improvement is needed, select 'No improvement required'. I don't know why he is such secretly. (a) are so secretly (b) No improvement required (c) is many secretive (d) is so secretive SSC MTS-05/07/2022 Shift-II Ans. (d) : The underlined part of the above sentence, ‘is so secretive’ will be used in place of ‘is such secretly’. because Adverb ‘so’ is used in the sense of ‘showing excess/emphasis'. Correct Sentence – I don't know why he is so secretive. 329. Select the most appropriate option to substitute the underlined segment in the given sentence. If no substitution is required, select 'No substitution required'. The examination was more simpler than I expected it to be. (a) much simpler (b) most simple (c) very simple (d) No substitution required SSC Stenographer (Grade C & D) 12.11.2021 Shift-I Ans. (a) : Since simple comparative degree is “simpler” and double comparative degree will not be used together in a sentence, so replace more simpler with much simpler. Correct sentence– The examination was much simpler than I expected it to be. 330. Select the most appropriate option to improve the underlined segment in the given sentence. If there is no need to improve it, select 'no improvement required'. Nobody was interested in the fish who Gopal had cach from the river. (a) which Gopal had catch (b) no improvement required (c) who Gopal had caught (d) which Gopal had caught SSC CGL (Tier-II) 08.08.2022 Shift-II Ans. (d) : For the underlined part “which Gopal had caught” will be used because pronoun ‘which’ is used for inanimate and small living being while ‘who’ is used for the living and V3 (cought) form will be used after had. Correct sentence– Nobody was interested in the fish which Gopal had caught from the river. 331. The following sentence has been split into four segments. Identify the segment that contains a grammatical error. If you want to/win the race,/you need to/believe in you. (a) you need to (b) believe in you (c) If you want to (d) win the race SSC Constable GD-08/12/2021 Shift-I Sentence Improvement

Ans. (b) : In the above sentence, ‘believe in yourself’ will be used in place of ‘believe in you’ because the reflexive pronoun is needed for the sentence. Correct sentence– If you want to win the race you need to believe in yourself. 332. Select the most appropriate option to substitute the underlined segment in the given sentence. If no substitution is required, select 'No substitution'. You must be quite good acquainted with the terrain in this jungle. (a) well acquainted (b) No substitution (c) better acquaints (d) best acquaint SSC Stenographer (Grade C & D) 11.11.2021 Shift-II Ans. (a) : Since Acquainted is an 'Adjective', adverb is needed to qualify an Adjective, therefore Adjective ‘Good’ is replace by an adverb “will” 333. Select the most appropriate option to substitute the underlined segment in the given sentence. If no substitution is required, select 'No substitution'. Practising math sums in so many periods over time is better than in long periods at a time, for effective learning. (a) too much (b) much (c) No substitution (d) short SSC Stenographer (Grade C & D) 11.11.2021 Shift-II Ans. (d) : For the underline part “short” will be used because ‘many’ is used for countable Noun since A time duration is being talk about in the sentence which that would be uncountable . Hence short periods will be used. Option (d) is correct 334. There is an underlined segment in the given sentence. Select the most appropriate option to substitute the segment. If there is no need to substitute it, select 'No substitution required'. I like Rahane's style of batting more than Kohli. (a) No substitution required (b) more from Kohli (c) more from Kohli's (d) more than Kohli's SSC Stenographer (Grade C & D) 11.11.2021 Shift-II Ans. (d) : The comparison is always in like term i.e. Rahane’s batting style will be compared to Kohli’s batting style, so Apostrophe (‘s) will be used with Kohli, so replace “More than kohli” with 'more than Kohli’s'. Option (d) is correct 335. Select the most appropriate option to improve the underlined segment in the given sentence. If there is no need to improve it, select 'No improvement required'. He is no clever to make sound business decisions.

142

YCT

(a) No improvement required (b) not clever enough to make (c) not clever to be making (d) not enough clever to make SSC Stenographer (Grade C & D) 11.11.2021 Shift-I Ans. (b) : For the underlined part ‘not clever enough to make’ will be used because it gives appropriate meaning to the sentence. Note :- Enough (Adverb) is always used after Adjective to qualify them. Correct sentence– He is not clever enough to make sound business decisions. 336. Select the most appropriate option to improve the underlined segment in the given sentence. If there is no need to improve it, select 'No improvement required'. He waves to me whoever he sees me at the window. (a) No improvement required (b) whenever (c) whatever (d) wherever SSC Stenographer (Grade C & D) 11.11.2021 Shift-I Ans. (b) :For the underlined part “whenever” will be used because 'whoever' is a pronoun and “whenever” is a conjunction which gives appropriate meaning to the sentence. Because two clause connect with conjunction not pronoun. He waves to me whenever he sees me at the window.

337. Select the most appropriate option to substitute the underlined segment in the given sentence. If no substitution is required, select 'No substitution'. I've realised that working hard is the more important thing for me. (a) No substitution (b) the most important (c) a much important (d) of more importance SSC Stenographer (Grade C & D) 15.11.2021 Shift-I Ans. (b) : For the underlined part “the most important” will be used because comparative degree is used to compare two persons or thing whereas in the sentence one among many things is choosing is being revealed. Hence superlative degree will be used. 338. Select the most appropriate option that can substitute the underlined segment in the given sentence. In order to win the competition, one has to be creative. (a) more creative (b) most creative (c) creativer (d) creativest SSC CHSL 26.05.2022 Shift-III Ans. (a) : For the underlined part “more creative” will be used because in the sense of ‘more than one” gives appropriate meaning. Correct sentence– In order to win the competition, one has to be more creative. Sentence Improvement

339. Select the most appropriate option that can substitute the underlined segment in the given sentence. Show me the better dress in the shop. (a) a better (b) the best (c) the good (d) a good SSC CHSL 25.05.2022 Shift-I Ans. (b) : For the underlined part “ the best” will be used because superlative degree ‘best’ is used after ‘the’ whenever better is used in comparative degree. Correct sentence– Show me the best dress in the shop. 340. Select the most appropriate option that can substitute the undelined segment in the given sentence. One should work hard than others to succeed. (a) hardest that (b) harder than (c) hard to (d) harder to SSC CHSL 26.05.2022 Shift-II Ans. (b) : For the underlined part “harder than” will be used because comparative degree is used with than. Note:- To make some adjectives comparative ‘more’ is used before them Correct Sentence– One should work harder than others to succeed. 341. Select the option that will improve the underlined part of the given sentence. Robin has less marbles than George. (a) few marbles (b) lower marbles (c) fewer marbles (d) enough marbles SSC CHSL 26.05.2022 Shift-II Ans. (c) : For the underlined part “fewer” will be used because in countable sense “few” is used while “little” is used in an countable sense. Little → Less → Least Few → Fewer → Fewest Correct sentence– Robin has fewer marbles than George. 342. Select the most appropriate option to substitute the underlined segment in the given sentence. if there is no need to substitute it, select option 'No substitution required'. Often it makes one wonder at the sights you see. (a) one sees (b) No substitution required (c) one see (d) you saw SSC CGL (Tier-I) 20/04/2022 Shift-III Ans. (a) : For the underlined part “one sees” will be used because indefinite pronoun ‘one’ is used before sentence . In case of singular pronoun ‘one’ singular verb “sees” will be used. Correct sentence___ often it makes one wonder at the sights one sees.

143

YCT

343. Select the option that will improve the underlined part of the given sentence. In case no improvement is needed, select 'No improvement required'. He’s not so friendly like she is. (a) so friendly that (b) No improvement required (c) so friendly as (d) as friendly like SSC CGL (Tier-I) 12/04/2022 Shift-III Ans. (c) : For the underlined part “so friendly as” will be used because so….as is used. Correct sentenceHe's not so friendly as she is. 344. Select the most appropriate option to substitute the underlined segment in the given sentence. If there is no need to substitute it, select 'No substitution required'. The old lady needed love and care beyond money. (a) beside (b) besides (c) No substitution required (d) beneath SSC CGL (Tier-I) 13/04/2022 Shift-I Ans. (b) :For the underlined part ‘besides’ will be used because it gives appropriate meaning to the sentence. The correct sentence will be – The old lady needed love and care besides money. Hence the correct answer is (b) 345. Select the options that will improve the underlined part of the sentence. In case no improvement is needed, select 'No improvement required'. He stood up from their chair to signal the end of the meeting. (a) up from his (b) upon of his (c) No improvement required (d) below in their SSC MTS-06/07/2022 Shift-I Ans. (a) : For the underlined part “up from his” will be used because ‘He’ is the subject of sentence and possessive pronoun of ‘he’ is ‘his’ and they is their. Correct Sentence– He stood up from his chair to signal the end of the meeting. 346. Select the most appropriate option to substitute the underlined segment in the given sentence. If there is no need to substitute it, select 'No substitution required'. Gel ingredients work well on waved, curled, relax and texture hair in the long run. (a) relaxed and textured (b) No substitution textured (c) relaxed and texturing (d) relaxing and textured SSC MTS-05/07/2022 Shift-III Ans. (a) : For the above underlined part, option (a) relaxed and textured will be used because adjective form (past participle) is needed. Correct Sentence – Gel ingredients work well on waved, curled, relaxed and taxtured lair in the long run. Sentence Improvement

347. Select the most appropriate option to substitute the underlined segment in the given sentence. If there is no need to substitute it, select 'No substitution required.' Abraham also had one sister, of who not much has been recorded. (a) of what (b) of whose (c) of whom (d) No substitution required SSC MTS-06/07/2022 Shift-II Ans. (c) : For the above underlined part, option (c) 'of whom' will be used because objective form is always used after prepositions. Correct Sentence– Abraham also had one sister, of whom not much has been recorded. 348. Select the option that will improve the underlined part of the given sentence. In case no improvement is needed, select 'No improvement required'. These days, judging someone is easier then respecting them. (a) is easier than (b) is easiest than (c) is more easier then (d) No improvement required SSC MTS-06/07/2022 Shift-III Ans. (a) : For the above underlined part, option (a) ‘is easier than’ will be used because ‘than’ is used with comparative degree. Correct Sentence – These days, judging someone is easier than respecting them. 349. Select the option that will improve the underlined part of the given sentence. In case no improvement is needed, select 'No improvement required'. While travelling, one comes across the larger number of people. (a) the large number off (b) No improvement required (c) an larger number of (d) a large number of SSC MTS-08/07/2022 Shift-I Ans. (d) : For the above underlined part, option (d) ‘a large number of’ will be used because comparative form is not needed for the sentence. Correct Sentence – While travelling, one comes across a large number of people. 350. Select the most appropriate option that can substitute the underlined segment in the given sentence. If there is no need to substitute it, select ‘No substitution required’. I don’t have some money at all. (a) No substitution required (b) few (c) any (d) little SSC Constable GD-24/11/2021 Shift-II

144

YCT

Ans. (c) : For the above underlined part, option (c) ‘any’ will be used because ‘Any’ is used for negative sentence and ‘some’ is used for assertive sentence. Note - Positive Sense - some Negative sense - Any Correct Sentence I don't have any money at all. 351. Select the option that will improve the underlined part of the given sentence. In case no improvement is needed, select 'No improvement required'. He was too tired for walking anymore. (a) very tired for walking (b) too tired to walk (c) to tired for walk (d) No improvement required SSC Constable GD-23/11/2021 Shift-III Ans. (b) : For the above underlined part, option (b) too tired to walk will be used because 'too + adj./adverb + to + V1' is used in these kinds of sentences. Too + Adjective / Adverb + To Infinitive Correct Sentence - He was too tired to walk anymore. 352. Select the most appropriate option that can substitute the underlined segment in the given sentence. If there is no need to substitute it, select ‘No substitution required’. The laptop that I bought last week works very good. (a) No substitution required (b) too good (c) very well (d) very nice SSC Constable GD-18/11/2021 Shift-I Ans. (c) : For the above underlined part, option (c) ‘very well’ will be used because adverb form (well) is used after adjective (very). Correct sentence– The Laptop that I bought last week works very well. 353. Select the most appropriate option that can substitute the underlined segment in the given sentence. If there is no need to substitute it, select‘No substitution required'. Wool and silk are materials those are the easiest to dye. (a) that are the easiest (b) that is easy (c) that are the easier (d) No substitution required SSC Constable GD-16/11/2021 Shift-III Ans. (a) : For the above underlined part, option (a) that are the easiest’ will be used because Pronoun ‘that’ is used before superlative degree. Correct Sentence Wool and silk are materials that are the easiest to dye. 354. Select the most appropriate option that can substitute the underlined segment in the given sentence. If there is no need to substitute it, select 'No substitution required'. General speaking, the great discoverers were people with great courage. Sentence Improvement

No substitution required Generally speak Generally spoken Generally speaking SSC Constable GD-08/12/2021 Shift-I Ans. (d) : For the above underlined part, option (d) ‘Generally speaking” will be used because generally (adverb form) is needed for the sentence. Correct sentence Generally speaking, the great discoverers were people with great courage. 355. Select the option that will improve the underlined segment in the given sentence. In case no improvement is needed, select 'No improvement' What a palace home! (a) pushy (b) palatial (c) palatable (d) No improvement SSC CHSL (Tier-I) –04/08/2021 (Shift-I) Ans. (b) : For the above underlined part, option (b) 'palatial' will be used because adjective form is needed for the sentence. Correct Sentence–What a palatial home! 356. Select the most appropriate option to substitute the underlined segment in the given sentence. If there is no need to substitute it, select 'No substitution required.' India has identified a nose-based vaccine against covid-19 that could be a 'gamechanger.' (a) No substitution required (b) a nasal vaccine (c) an aural vaccine (d) a sense-based vaccine SSC CHSL (Tier-I) –05/08/2021 (Shift-I) Ans. (b) : For the above underlined part, option (b) ‘a nasal vaccine’ will be used because ‘vaccine’ is noun Hence adjective form (nasal) will be used before it. Correct Sentence - India has identified a nasal vaccine against covid-19 that could be a 'game-changer.' 357. Select the most appropriate option to substitute the underlined segment in the given sentence. If no substitution is required, select 'No substitution'. The place is too much noisy. (a) No substitution (b) too noisy (c) much noisy (d) much too much noisy SSC CGL (Tier-I) –16/08/2021 (Shift-I) Ans. (b) : For the above underlined part, option (b) ‘too noisy’ will be used because ‘much’ is superfluous. Note:- Normally, Negative words is used after ‘too’. Correct Sentence The place is too noisy. 358. Select the option that will substitute the underlined part of the given sentence. In case no substitution is needed, select 'No substitution rquired'. When I met the couple yesterday I noticed that the wife was more taller than her husband.

145

(a) (b) (c) (d)

YCT

(a) (b) (c) (d)

No substitution required much tallest much tall taller SSC CGL (Tier-I) –18/08/2021 (Shift-I) Ans. (d) : For the above underlined part, ‘taller’ will be used because ‘more’ is superfluous and double comparative is not used in a sentence. Correct Sentence When I met the couple yesterday I noticed that the wife was taller than her husband. 359. Select the most appropriate option to improve the underlined segment in the given sentence. If there is no need to improve it, select 'No improvement'. A hole is in my picket. (a) A hole are there (b) A hole there is (c) No improvement (d) There is a hole SSC CGL (Tier-II) 16/11/2020 (3 pm - 5 pm) Ans. (d) : For the above underlined part, option (d) ‘there is a hole’ will be used because ‘there’ is used as a introductory subject. Correct Sentence - There is a hole in my pocket. 360. Select the alternative that will improve the underlined part of the sentence in case there is no improvement select “No improvement”. Dinesh requested the lady at the counter to give him a seat besides the window. (a) No improvement (b) a seat beside the window (c) an seat beside a window (d) one seat besides the window SSC CPO-SI (Tier-II) 27/09/2019 (3 pm-5 pm) Ans. (b) : For the above underlined part, option (b) 'a seat beside the window' will be used because ‘beside’ gives suitable meaning for the sentence. Correct Sentence - Dinesh requested the lady at the counter to give him a seat beside the window. 361. Select the alternative that will improve the underlined part of the sentence in case there is no improvement select “No improvement”. It has been raining very hardly since morning. (a) is been raining very hardly (b) No improvement (c) have been raining very hardly (d) has been raining very hard SSC CPO-SI (Tier-II) 27/09/2019 (3 pm - 5 pm) Ans. (d) : For the above underlined part, ‘has been raining very hard’ will be used because adverb form (hard) is needed for the sentence. Correct Sentence - It has been raining very hard since morning. 362. Select the alternative that will improve the underlined part of the sentence in case there is no improvement select “No improvement”. Kalidasa was a greater classical Sanskrit poet of India. (a) No improvement (b) was a great classical (c) was the greater classic (d) was a greatest classical SSC CPO-SI (Tier-II) 27/09/2019 (3 pm - 5 pm)

Sentence Improvement

Ans. (b) : For the above under part, option (b) ‘was a great classical’. Will be used because comparative form is not needed for the sentence. Hence positive degree (great) will be used. Correct Sentence– Kalidasa was a great classical Sanskrit poet of India. 363. Select the alternative that will improve the underlined part of the sentence in case there is no improvement select "No improvement". Our cook puts too many salt in the food. (a) No improvement (b) Putting too much (c) puts too much (d) puts very much SSC CPO-SI (Tier-II) 27/09/2019 (3 pm - 5 pm) Ans. (c) : For the above underlined part, option (C) ‘puts too much’ will be used because ‘much’ is used with uncountable nouns and many is used with countable nouns. Correct Sentence - Our cook puts too much salt in the food. 364. Select the alternative that will improve the underlined part of the sentence. In case there is no improvement select "No improvement". Mohan is a good actor but his brother is better than him : (a) more better (b) quite better (c) No improvement (d) very better SSC Stenographer Grade C&D –05/02/2019 (3:5pm)

Ans. (c) : The underlined part of the sentence, there is no improvement. Correct sentence-Mohan is a good actor but his brother is better than him. 365. Improve the bracketed part of the sentence. The company hopes to eventually provide Singapore with the second-biggest electric carsharing service in the world, (second to Paris only). (a) second only to Paris (b) second to only Paris (c) only second to Paris (d) No improvement SSC CGL (Phase-II) 17/02/2018 Ans. (a) The bracket part of the given sentence, at the place of ‘second to Paris only’, ‘second only to Paris’ will be grammatically correct because use of adverb (only) apply before the verb and after the noun in a sentence to modify the verb. Note:- The use of ‘only’ apply before noun It modifies same noun. Correct Sentence– The company hopes to eventually provide Singapore with the second-biggest electric carsharing service in the world, (second only to Paris). 366. Improve the bracketed part of the sentence. But understanding what constitutes such (a miracle diet), and making sure that patients get it, isn’t as straightforward as it seems. (a) a miraculous diet (b) a miracling diet (c) an miracling diet (d) No improvement SSC CGL (Phase-II) 17/02/2018

146

YCT

Ans. (a) The bracketed part of the above sentence, At Ans. (b) : The underlined segment in the given the place of ‘miracle diet’, ‘A miraculous diet’ will be sentence, in the place of ‘over his reaching’, ‘beyond used because miraculous (Adjective) will be used his reach’ would be appropriate. Refers-: not close enough to be touched or picked up before diet (noun) by someone. Correct Sentence– But understanding what constitutes such a miraculous Hence, The use of 'beyond' would be appropriate diet, and making sure that patients get it, isn’t as Note:- We used ‘over’ to talk about movement or position at a higher level than something. straightforward as it seems. Correct Sentence– 367. Improve the bracketed part of the sentence. The large mansion was beyond his reaching. Expensive oil could hit consumption and public investment and dent private investment (what 371. Select the most appropriate option to substitute the underlined segment in the given sentence. is not a path) to a sustained revival. If no substitution is required, select No (a) who is not a path (b) whom is not a path improvement. (c) which is not a path (d) No improvement Meera has a friend who parents live in Dubai. SSC CGL (Phase-II) 17/02/2018 (a) have a friend whom parents live Ans. (c) The bracketed part of the given sentence , At (b) No improvement the place of ‘what’, ‘which’ will be used as a relative (c) has a friend whose parents live pronoun. (d) has a friend which parents live For Living things – will be used 'who' and for nonSSC CGL (Tier-I)–13/06/2019 (Shift-I) living things- ‘which’ as a relative pronoun. Ans (c) : For the underlined part, 'has a friend whose Correct Sentence parent live' will be used because here possessive Expensive oil could hit consumption and public adjective (whose) is needed. investment and dent private investment which is not a Correct Sentencepath to a sustained revival. Meera has a friend whose parents live in Dubai. 368. Improve the bracketed part of the sentence. 372. Select the most appropriate option to Whatever said and done, (not less vital) is the substitution the underlined segment in the future of the border separating Northern given sentence. If no substitution is required, Ireland from the Irish Republic. select No improvement. (a) any less vital (b) more less vita l Some agitating miners allegation that there is (c) no less vital (d) No improvement no emergency measures inside the mines. SSC CGL (Phase-II) 17/02/2018 (a) miners allege that there is (b) miner's allegation that there are Ans. (c) The bracketed part of above sentence, at the place (c) miners alleged that there were of ‘not less vital’, ‘no less vital’ will be used because for (d) No improvement modifying of an adjective (less) ‘no’ will be used. SSC CGL (Tier-I) – 11/06/2019 (Shift-II) Correct Sentence–Whatever said and done, no less vital is the future of the border separating Northern Ans. (c) : The underlined part of the above sentence, in Ireland from the Irish Republic. place of 'miners allegation that there is, 'miners alleged 369. Select the most appropriate option to substitute that there were' would be used because sentence, is in the underlined segment in the given sentence. If past tense. there is no need to substitute it, select 'No Hence ‘verb’ would be used in past tense and in place of ‘allegation (noun),’ alleged (adj.) is required. substitution required'. That tree fell down yesterday narrowly miss Correct Sentence– Some agitating miners alleged that there were no my car. emergency measures inside the mines. (a) narrowed missing 373. Select the most appropriate option to substitute (b) narrowly missed the underlined segment in the given sentence. If (c) No substitution required no substitution is required, select No (d) narrowly missing improvement. SSC CPO-SI – 25/11/2020 (Shift-I) He made a desperate but also partial successful Ans. (d) : The underlined segment in the given sentence, effort to change the ghastly topic. in the place of ‘narrowly miss’, 'narrowly missing' would (a) No improvement be appropriate because here the adjective form of miss, (b) but partially successful effort missing (V+ ing) will be used. (c) though part successful effort Correct Sentence– (d) yet also partial successful effort That tree fell down yesterday narrowly missing my car. SSC CGL (Tier-I) – 11/06/2019 (Shift-III) 370. Select the most appropriate option to substitute Ans. (b) : The underlined segment in the given the underlined segment in the given sentence. If there is no need to substitute it, select ‘No sentence, in place of ‘but also partial successful’, ‘but partially successful’ would be appropriate because substitution required’. partially (Adverb) modify successful (Adjective). The large mansion was over his reaching . Note:- ‘but also’ is used in the following structure. (a) away from reaching not only ......... but also (b) beyond his reach (c) No substitution required Correct Sentence– (d) above his reaching He made a desperate but partially successful effort to SSC CPO-SI – 24/11/2020 (Shift-II) change the ghastly topic. Sentence Improvement

147

YCT

374. Select the most appropriate option to substitute the underlined segment in the given sentences. If no substitution is required, select no improvement. A reason why there are so much misconception on dyslexia could be the sheer invisibility of the disorder. (a) there are so much misconceptions against dyslexia (b) there are so many misconceptions about dyslexia (c) there are so many misconception around dyslexia (d) No improvement SSC CGL (Tier-I) – 10/06/2019 (Shift-II) Ans : (b) In the given sentence of the underlined part, in the place of ‘there are so much misconception on dyslexia, ‘there are so many misconceptions about dyslexia’ would be appropriate because here misconception is used as a countable noun’ Hence countable adjective ‘many’ will be used. Correct Sentence– A reason why there are so many misconceptions about dyslexia could be the sheer invisibility of the disorder. 375. Select the most appropriate option to substitute the underlined segment. if no substitution is required select 'No substitution' The assignment was so difficult for Mohit to do on his own. (a) was too difficult (b) was so much difficult (c) were so difficult (d) No substitution SSC CPO-SI – 13/12/2019 (Shift-I) Ans. (a) : In the above sentence of the underlined part 'was too diffucult' will be used because 'too.....to' is used in negative sense. Correct Sentence– The assignment was too difficult for Mohit to do on his own. 376. Select the most appropriate option to substitute the underlined segment in the given sentence. If there is no need to substitute it, select No improvement. There is a lot of confusion among students due to recent changes in the examination pattern. (a) allot of (b) a lots of (c) No improvement (d) the lots of SSC CHSL (Tier-I) 26/10/2020 (Shift-II) Ans. (c) : The underlined segment in the given sentence, there is no need to substitute it. 377. Select the most appropriate option to substitute the underlined segment in the given sentence. If there is no need to substitute it, select ‘No improvement’. The teacher said that all that students who did not complete the task in the given time should attempt it again. (a) No improvement (b) those (c) this (d) them SSC CHSL (Tier-I) 16/10/2020 (Shift-I) Sentence Improvement

Ans. (b) : In the above sentence, in place of the underlined part ‘that’, the use of ‘those’ would be appropriate because in given sentence ‘students’ will be used in plural number. Hence the use of plural pronoun ‘those’ would be appropriate. Correct Sentence–The teacher said that all those students who did not complete the task in the given time should attempt it again. 378. Select the most appropriate option to substitute the underlined segment in the given sentence. If there is one need to substitute it, select 'No improvement'. The Arabian Sea region is very important for India as several major ports are locating there. (a) have been locating (b) No improvement (c) was locating (d) are located SSC CHSL (Tier-I) 13/10/2020 (Shift-I) Ans. (d) : The underlined segment in the given sentence, in place of ‘are locating’. ‘are located’ would be appropriate because here 'located' is used as 'past participle.' Correct Sentence– The Arabian Sea region is very important for India as several major ports are located there. 379. Select the most appropriate option to substitute the underlined segment in the given sentence. If there is no need to substitue it, select No improvement. The film by itself was good but the bit long. than I had expected. (a) a bit long (b) the bit longer (c) a bit longer (d) No improvement SSC CHSL (Tier-I) 21/10/2020 (Shift-I) Ans. (c) : The underlined part of the above sentence 'the bit long' will be replaced by ‘a bit longer’ because in the sentence, the use of ‘bit’ is used as indefinite sense. Hence It is clear before it, the use of indefinite article ‘A’ will be used. And the use of ‘than’ in the sentence is used for longer (Comparative degree). Correct Sentence– The film by itself was good but a bit longer than I had expected. 380. Select the most appropriate option to substiute the underlined segement in the given sentence. If there is no need to substitute it, select No improvement. This is a class who is expected to do exceptionally well in the quiz next week. (a) which is expected to do (b) No improvement. (c) that is expecting to done (d) which are expecting to be doing SSC CHSL (Tier-I) 13/10/2020 (Shift-III) Ans. (a) The underlined segment in the given sentence, in place of ‘who is expected to do’, ‘which is expected to do’ would be used because the use of relative pronoun ‘which’ is used as a non-living. Correct Sentence– This is a class which is expected to do exceptionally well in the quiz next week.

148

YCT

381. Select the most appropriate option to substitute the underlined segment in the given sentence. If there is no need to substitute it, select 'No improvement'. Doctors at the AIIMS trauma Centre managed to save the life of 12 year old archer Shivangini, whose were struck by an arrow in the neck during a practice session. (a) No improvement (b) who was striked (c) which was struck (d) who was struck SSC CHSL (Tier-I) 14/10/2020 (Shift-II) Ans. (d) : In place of the underlined segment of the above sentence. ‘Whose were struck’, ‘who was struck’ would be used, because for persons as a subject relative pronoun ‘who’ would be appropriate. Whereas whose is used in objective case and ‘Shivangini’ for singular noun, ‘was’ is appropriate. Correct SentenceDoctors at the AIIMS trauma Centre managed to save the life of 12 year old archer Shivangini who was struck by an arrow in the neck during a practice session. 382. Select the most appropriate option to substitute the underlined segment in the given sentence. If there is no need to substitute it, select No Improvement. Luckily we have got the few minutes to spare. (a) a little (b) No Improvement (c) a few (d) quite few SSC MTS – 06/08/2019 (Shift-III) Ans. (c) : The underlined part of the given sentence, in place of ‘the few’, the use of ‘A few’ will be used. The meaning of the other optionsFew -: Negative sense A Few -: Affirmative sense Note:- In countable sense, Few, a Few, the Few are used whereas In uncountable sense- little, a little and the little are used. Correct sentence Luckily we have got a few minutes to spare. 383. Select the most appropriate option to substitute the underlined segment in the given sentence. If there is no need to substitute it, select No improvement. In a recession-hit economy, automobiles is the mostly affected sector. (a) is the most (b) are the mostly (c) No improvement (d) are the most SSC MTS – 22/08/2019 (Shift-III) Ans. (a) : The underlined part in the given sentence, in place of ‘mostly’ (Adverb). The use of ‘most’ (Adjective) will be appropriate because here the use of adjective is appropriate, not adverb. Correct Sentence In a recession-hit economy, automobiles is the most affected sector. 384. Select the most appropriate option to substitute the underlined segment in the given sentence. If there is no need to substitute it, select No Improvement. Only me can solve the problem. (a) Only I can (b) Me can only (c) No Improvement (d) Only me could SSC MTS – 06/08/2019 (Shift-I) Sentence Improvement

Ans. (a) : In The underlined part of the given sentence, in place of ‘only me can’, The use of ‘only I can’ will be appropriate because the pronoun of the subjective case is used as the subject in the sentence. Hence in place of objective case ‘me’ the use of subjective case ‘I’ will be used. Correct Sentence– Only I can solve the problem. 385. Select the most appropriate option to substitute the underlined segment. If there is no need to substitute it, select No improvement. Whom you think, will be selected as the Head Boy of the school ? (a) which you thought (b) who do you think (c) whom you thinking (d) No improvement SSC MTS – 19/08/2019 (Shift-II) Ans. (b) : In the underlined part of the given sentence, in place of ‘whom you think’, ‘who do you think’ will be used because according to the sense of sentence, a subject is needed here. Correct sentence ‘Who do you think, will be selected as the Head Boy of the school ? 386. Select the most appropriate option to substitute the underlined segment. If there is no need to substitute it, select No improvement I had first gone to Srinagar with mine family in 1961. (a) along my family (b) with my family (c) along mine family (d) No improvement SSC MTS – 16/08/2019 (Shift-I) Ans. (b) : The underlined part of the given sentence, in place of ‘mine’ the use of possessive adjective ‘my’ will be used because my (adjective) is used to describe the specialty of family (noun). Correct Sentence– I had first gone to Srinagar with my family in 1961. 387. Select the most appropriate option to substitute the underlined segment in the given sentence. If there is no need to substitute it, select No improvement. The fire spread through the building very quick but fortunately nobody was injured. (a) spreads through the building quickly (b) spread through the building very quickly (c) spreading through building very quick (d) No improvement SSC MTS – 13/08/2019 (Shift-II) Ans. (b) : ‘Very quickly’ will be used instead of ‘very quick’, the underlined part of the given sentence because the use of an adverb (quickly) is appropriate here, not an Adjective (quick). Correct sentence The fire spread through the building very quickly but fortunately nobody was injured. 388. Select the most appropriate option to substitute the underlined segment in the given sentence. If there is no need to substitute it, select No Improvement. It is danger to cross the road unmindfully. (a) It is dangerous (b) It is full of dangerous (c) There is danger (d) No Improvement SSC MTS – 06/08/2019 (Shift-II)

149

YCT

Ans. (a) : The underlined part of the given sentence, in place of ‘It is danger,’ the use of ‘It is dangerous’ will be appropriate because according to sense here, the use of adjective (dangerous) is appropriate, not noun (danger). Correct Sentence - It is dangerous to cross the road unmindfully. 389. Select the most appropriate option to substitute the underlined in the given sentence. If there is no need to substitute it, select 'No substitution required'. The eighteenth century saw the middle classes becoming most prosperous. (a) No substitution required (b) being more than (c) become more (d) became most SSC GD – 08/03/2019 (Shift-III) Ans. (c) : In the underlined part of given sentence, in place of ‘becoming most’, ‘become more’ will be used because comparative degree is needed according to the sense of sentence. Correct Sentence– The eighteenth century saw the middle classes become more prosperous. 390. Select the most appropriate option to substitute the underlined in the given sentence. If there in no need to substitute it, select 'No substitution required'. When a plastic comb is rubbed with dry hair, they acquired a static charge. (a) No substitution required (b) it acquired (c) they acquire an (d) it acquires a SSC GD – 08/03/2019 (Shift-II) Ans. (d) : Here, it would be appropriate to use ‘it acquires a’, instead of ‘ they acquired a’ because the sentence should be in present tense and pronoun 'it' is used for non living thing. Given sentence is zero conditional sentence, whose structure is as followsIf / when + Present Indefinite+Present Indefinite

Correct Sentence– When a plastic comb is rubbed with dry hair, it acquires a static charge. 391. Select the most appropriate option to substitute the underlined segment in the given sentence. If there is no need to substitute it, select No Improvement Meeting my teacher after ten years was an unforgotten moment that I will always treasure. (a) a forgotten moment (b) an unforgettable moment (c) the unforgettable moment (d) No improvement SSC GD – 14/02/2019 (Shift-I) Ans. (b) : The underlined segment in the given sentence, in place of ‘unforgotten moment’, 'unforgettable moment' will be used because It is appropriate to use ‘unforgettable’ (adjective) to describe the moment (noun). Correct Sentence– Meeting my teacher after ten years was an unforgettable moment that I will always treasure. Sentence Improvement

392. Select the most appropriate option to substitute the underlined segment in the given sentence. If there is no need to substitute it, select No improvement. The questions are so simple that a child can understand it. (a) can understand them (b) No improvement (c) is understanding them (d) can understand this SSC GD – 11/02/2019 (Shift-III) Ans. (a) : The underlined part of the above sentence, in place of ‘can understand it,’ the use of ‘can understand them’ will be appropriate because the ‘questions’ is plural subject for which the use of plural object ‘them’ would be oppropriate. Correct SentenceThe questions are so simple that a child can understand them.

G.

Conjunction

393. Select the most appropriate option that can substitute the underlined segment in the given sentence. I asked Rita that how far her school was. (a) How far her school is (b) That how her school is (c) Then how far her school was (d) How far her school was SSC GD 12/01/2023 (Shift-II) Ans. (d) : In the underlined part of the above sentence 'how far her school was' will be used instead of 'that how far her school was'. Because the given sentence is in indirect speech of interrogative sentence and while changing interrogative sentence into direct speech. Wh word is used as a conjunction in place of 'that'. Correct sentenceI asked Rita how far her school was. 394. Select the most appropraite option to substitute the underlined segment in the given sentence. The secretary to my boss is very efficient as he not only gives him the required information and also handles correspondence independently. (a) but also hanldes (b) yet also handles (c) along with also handles (d) not also hanldes SSC CGL (Tier-I) 27/07/2023 (Shift-III) Ans. (a) : The most appropriate substitute for the underlined segment in the above given sentence 'and also handles' is gramatically error. The correct segment is 'but also handles.' Because 'not only but also' is used as a conjuction. Not only..........but also conjunction. Thus, the correct answer is option (a).

150

YCT

395. Select the option that will improve the underlined part of the sentence. In case no improvement is needed, select 'No improvement'. Keep him at arm's length lest you may not repent in the long run. (a) or you may no (b) No Improvement (c) unless you may (d) lest you should SSC CGL (Tier-II) 08.08.2022 Shift-II Ans. (d) : The underlined part of the above sentence, in place of ‘may’, ‘should’ will be used. Because ‘should’ always is used after ‘lest’. Correct sentence– Keep him at arm's length lest you should not repent in the long run. 396. Select the most appropriate option to substitute the underlined segment in the given sentence. If no substitution is required, select 'No substitution'. Are you trying to say that you are stupid so proud of it? (a) though (b) No substitution (c) so that (d) but SSC Stenographer (Grade C & D) 11.11.2021 Shift-I Ans. (d) : ‘But’ as a conjunction is used to introduce a word or phrase. ‘so’ as a conjunction to show the reason for something. Hence the underlined part of the given sentence, ‘but’ conjunction will be used. 397. Select the most appropriate option to substitute the underlined segment in the given sentence. If there is no need to substitute it, select 'No substitution'. They decided to move to Dehradun because they could live in a peaceful environment. (a) No substitution (b) as soon as (c) so that (d) in case SSC Stenographer (Grade C & D) 12.11.2021 Shift-II Ans. (c) : The underlined segment in the given sentence, in place of ‘because’ ‘so that’ will be used because it gives appropriate meaning according to sense of the sentence. Correct Sentence– They decided to move to Dehradun so that they could live in a peaceful environment. 398. Select the most appropriate option to substitute the underlined segment in the given sentence. If there is no need to substitute it, select 'No substitution required'. Hardly had we get in but the train started. (a) We get in than (b) We got in before (c) No substitution required (d) We got in when SSC CGL (Tier-I) 19/04/2022 Shift-III Ans. (d) : ‘when’ is paired with hardly / scarcely and the third form of verb (V3) is used after ‘had’. Hence in place of ‘we get in but,’ the use of ‘we got in when’ will be appropriate. Sentence Improvement

399. Select the option that will improve the underlined part of the given sentence. In case no improvement is needed, select 'No improvement required'. Banning food and beverages is not desirable nor feasible in a civilized society. (a) neither (b) No improvement required (c) none (d) nor SSC MTS-05/07/2022 Shift-I Ans. (a) : In the underlined part of the above sentence, ‘Neither’ will be used in place of ‘not’. Because the following structure is used in it. Neither ---------------nor Either ---------------or Correct Sentence– Banning food and beverages is neither desirable nor feasible in a civilized society. 400. Select the most appropriate option that can substitute the underlined segment in the given sentence. If there is no need to substitute it, select ‘No substitution required’. The interviews will be held between 11 a.m. to 5 p.m. (a) will hold during (b) will be held from (c) No substitution required (d) will be held at SSC Constable GD-25/11/2021 Shift-III Ans. (b) : The underlined segment in the given sentence, in place of ‘will be hold between’, the use of ‘will be held from’ will be appropriate. Because the use of ‘to’ structure will be appropriate with ‘from’. From ......... to Between--------and Correct sentence – The interviews will be held from 11 a.m. to 5 p.m. 401. Select the option that will improve the underlined part of the given sentence. In case no improvement is needed, select 'No improvement required'. Reading not only helps to pass the leisure hours as well as keeps you well informed. (a) No improvement required (b) along with (c) but also (d) and even SSC Constable GD-09/12/2021 Shift-II Ans. (c) : In the given sentence, ‘but also’ will be used in place of ‘as well as’ because ‘Not only-------but also’ is a correlative conjunction. It is used with each other. Correct sentence – Reading not only helps to pass the leisure hours But also keeps you well informed.

151

YCT

402. Select the most appropriate option to substitute the underlined segment in the given sentence. If there is no need to substitute it, select 'No substitution'. No other place in India gets much rainfall as Mawsynram. (a) more rainfall (b) No substitution (c) as much rainfall (d) the most rainfall SSC CHSL (Tier-I) –16/04/2021 (Shift-I) Ans. (c) : In the underlined part of the above sentence, in place of ‘much rainfall’ 'as much rainfall' will be used. Because ‘As much as’ is used as a co-relative conjunction which means:- used to say that two things are equal in amount or degree. Correct Sentence No other place in india gets as much rainfall as Mawsynram. 403. Select the most appropriate option to substitute the underlined segment in the given sentence. If there is no need to substitute, it select 'No substitution required'. I cannot help you if you tell me your problem. (a) unless you don't tell me (b) unless you tell me (c) until you don't tell me (d) No substitution required SSC CHSL (Tier-I) –19/04/2021 (Shift-I) Ans. (b) : The underlined part of the above sentence, in place of ‘if you tell me’, ‘Unless you tell me’ will be used. Because in the sense of contrast, the use of ‘unless’ will be appropriate. Note:- The use of ‘not’, will not be appropriate with ‘unless clause’. Correct Sentence I cannot help you unless you tell me your problem. 404. Select the most appropriate option to improve the underlined segment in the given sentence. If there is no need to improve it, select 'No improvement'. The place was not cold only and also damp. (a) only cold not also (b) No improvement (c) not only cold but also (d) not only cold and both SSC CGL (Tier-II) 16/11/2020 (3 pm - 5 pm) Ans. (c) : The underlined part of the given sentence, in place of ‘not cold only and also’, ‘not only cold but also’ will be used. Because the use of ‘correlative conjunction (but also) will be appropriate with ‘not only’. Structure – Not only------------------but also Correct Sentence - The place was not only cold but also damp. 405. Select the most appropriate option to improve the underlined segment in the given sentence. IF there is no need to improve it, select 'No improvement'. I will not go to the party lest you will promise to accompany me. (a) No improvement (b) unless you (c) if you will (d) until you should SSC CGL (Tier-II) 16/11/2020 (3 pm - 5 pm) Sentence Improvement

Ans. (b) : The underlined segment in the given sentence, in place of ‘lest you will’, ‘unless you’ will be used. Because the use of modal will not be appropriate with ‘if , until, unless’. Correct SentenceI will not go to the party unless you promise to accompany me. 406. Improve the bracketed part of the sentence. (As soon after the players arrived) in decorated cars, the ceremony began with traditional African drummers and dancers blowing conch shells. (a) Soon after the players arrive (b) Soon after the players arrived (c) As soon as the players arrived after (d) No improvement SSC CGL (Phase-II) 17/02/2018 Ans. (b) The underlined part of the above sentence, in place of ‘As soon after’, ‘soon after’ will be used according to the sense of sentence. Because the above conjunction is used to mean the followingAs soon as means -: immediately at or shortly after the time that Soon after means:- within a short period after this or that time, aunt etc. Correct Sentence Soon after the players arrived in decorated cars, the ceremony began with traditional African drummers and dancers blowing conch shells. 407. Select the most appropriate option to substitute the underlined segment in the given sentence. If there is no need to substitute it, select ‘No substitution required’. Because she didn’t love him, she had to marry him. (a) Despite (b) Although (c) Since (d) No substitution required SSC CPO-SI – 23/11/2020 (Shift-I) Ans. (b) : In the underlined part of the above sentence, instead of ‘because’, the use of 'although' would be appropriate Because ‘although’ is used as a conjunction. Note :- Because is usually not used at the beginning of the sentence , it is a subordinate conjunction which serves to connect two clauses. Which shows the reason. Correct Sentence– Although she didn't love him, she had to marry him. 408. Select the most appropriate option to substitute the underlined segment in the given sentence. If there is no need to substitute it, select ‘No substitution required’. The exam was not so easy that we had expected. (a) such easy as (b) as easy as (c) as easy so (d) No substitution required SSC CPO-SI – 25/11/2020 (Shift-II) Ans. (b) : The underlined part of the above sentence, in place of ‘so easy that’, ‘As easy as’ will be used because ‘as + adj + as’ will be appropriate. Correct Sentence– The exam was not as easy as we had expected.

152

YCT

409. Select the most appropriate option to improve the underlined statement in the given sentence. If there is no need to improve it, select 'No improvement'. I suggest you to see a solicitor. (a) No improvement (b) am suggesting you (c) suggest you that (d) suggest that you SSC CGL (Tier-II) 16/11/2020 (3 pm - 5 pm) Ans. (d) : for the above underlined part, ‘suggest that you’ will be used in place of suggest you because ‘that’ is used after suggest. Correct SentenceI suggest that you see a solicitor. 410. Select the most appropriate option to substitute the underlined segment in the given sentence. If there is no need to substitute it, select ‘No substitution required’. The train was delayed in order to an accident. (a) because (b) owing to (c) since (d) No substitution required SSC CPO-SI – 24/11/2020 (Shift-II) Ans. (b) : In the above sentence the underlined phrase ‘in order to’, ‘owing to’ will be used , because it gives a suitable meaning to the sentence. Correct Sentence– The train was delayed owing to an accident. 411. Select the most appropriate option to substitute the underlined segment in the given sentence. If no substitution is required, select No improvement. The Committee occupied itself with no other important question only this. (a) with no other important question but this (b) No improvement (c) with one other important question either this (d) with no other important question this SSC CGL (Tier-I) – 19/06/2019 (Shift-III) Ans. (a) : The underlined segment in the given sentence, in place of ‘ with no other important question only this’, ‘with no other important question but this’ will be appropriate, because here adversative conjunction ‘but’ will be used. Correct Sentence– The committee occupied itself with no other important question but this. 412. Select the most appropriate option to substitute the underlined segment in the given sentence. If no substitution is required, select No improvement. Let us not neglect important aspects of life although pursue momentary-pleasures. (a) by pursued moment pleasures. (b) through pursued in momentary pleasures (c) while pursuing momentary pleasures. (d) No improvement SSC CGL (Tier-I) – 07/06/2019 (Shift-III) Ans : (c) The underlined part of the above sentence, in place of ‘Although’ suitable conjunction. ‘while’ will be used. Because for this sentence, it gives suitable meaning. Correct Sentence– Let us not neglect important aspects of life while pursuing momentary pleasure. Sentence Improvement

413. Select the most appropriate option to substitute the underlined segment. If no substitution is required select 'No substitution' Even if he is an honest man, he has been accused of theft. (a) Although he is an honest man (b) No substitution (c) Since he are an honest man (d) Even though he is honest man SSC CPO-SI – 11/12/2019 (Shift-II) Ans : (a) The underlined segment of the given sentence, in place of ‘Even if he is an honest man’, the use of ‘Although he is an honest man’ will be used. Because although is used when two opposing clauses are used together. Correct Sentence– Although he is an honest man, he has been accused of theft. 414. Select the most appropriate option to substitute the underlined segment in the given sentence. If there is no need to substitute it, select 'No improvement'. Prithvi likes baseball as long as she likes cricket. (a) as many as (b) No improvement (c) so long as (d) as much as SSC CHSL (Tier-I) 19/03/2020 (Shift-III) Ans. (d) : In the underlined part of the above sentence, the use of ‘as much as’ would be appropriate in place of ‘as long as’. Because it is expressing the sense of ‘quantity’. As long as :- provided that. As much as :- used to say that an amount is as large as another amount. As many as :- used to Suggest that a number or amount is surprisingly large. So long as :- during and up to the end of the time that. Correct Sentence– Prithvi likes baseball as much as she likes cricket. 415. Select the most appropriate option to substitute the underlined segment in the given sentence. If there is no need to substitute it, select 'No improvement'. As long to funding is not made available, our construction dreams have been deferred. (a) As high as (b) In as much as (c) No improvement (d) So long as SSC CHSL (Tier-I) 19/03/2020 (Shift-III) Ans. (d) : The underlined segment of the above sentence, ‘so long as’ will be appropriate in place of ‘as long to’. Because the use of ‘so long as’ will be used as a conjunction. Which gives the appropriate meaning for this sentence. Correct Sentence– So long as funding is not made available, our construction dreams have been deferred. 416. Select the most appropriate option to substitute the underlined segment in the given sentence. If there is no need to substitute it, select 'No improvement'. So much as you are their class teacher, you are responsible for their performance in the examination.

153

YCT

(a) In as much as (b) No improvement (c) As long as (d) As much to SSC CHSL (Tier-I) 19/03/2020 (Shift-I) Ans. (c) : The underlined part of the above sentence, ‘as long as’ will be used in place of ‘so much as’. Because ‘As long as’ shows the time. Which gives suitable meaning for this sentence. Correct Sentence– As long as you are their class teacher, you are responsible for their performance in the examination. 417. Select the most appropriate option substitute the underlined segment in the given sentence. If there is no need to substitute it, select 'No improvement'. As long as they are here, Hari won't attend the training. (a) So long so (b) As long that (c) No improvement (d) So far as SSC CHSL (Tier-I) 18/03/2020 (Shift-III) Ans. (c) : The underlined part of the given sentence, there is no need to substitute it. 418. Select the most appropriate option to substitute the underlined segment in the given sentence. If there is no need to substitute it, select 'No improvement'. As of you are here with me, who cares about the outcome of the issue. (a) So long (b) As long as (c) As long to (d) No improvement SSC CHSL (Tier-I) 18/03/2020 (Shift-I) Ans. (b) : In the underlined part of the above sentences. ‘As long as’ will be used in place of ‘as of’ . Because It gives appropriate meaning for this sentence. Correct Sentence– As long as you are here with me, who cares about the outcome of the issue. 419. Select the most appropriate option to substitute the underlined segment in the given sentence. If there is no need to substitute it, select No improvement. The director wanted Aziz to play the role of the doctor as well also that of the apprentice. (a) and also of (b) No improvement. (c) as well as (d) as well SSC CHSL (Tier-I) 26/10/2020 (Shift-I) Ans. (c) : The underlined part of the above sentence ‘As well as’ will be used in place of ‘as well also’. Because ‘as well as’ is used as a co-ordinate conjunction. As well as :- in addition to somebody / something which gives suitable meaning for this sentence. Correct Sentence The director wanted Aziz to play the role of the doctor as well as that of the apprentice. 420. Select the most appropriate option to substitute the underlined segment in the given sentence. If there is no need to substitute it, select No improvement The Dean said, "you can have neither jam nor butter but not both" (a) No improvement (b) jam and butter (c) no jam or butter (d) either jam or butter SSC CHSL (Tier-I) 20/10/2020 (Shift-III)

Sentence Improvement

Ans. (d) : In the above sentence ‘Either jam or butter’ will be appropriate in place of the underlined part ‘neither jam nor butter’. Because the use of ‘either ---or’ gives the suitable meaning for this sentence. Correct Sentence– The Dean said, "you can have either jam or butter but not both" 421. Select the most appropriate option to substitute the underlined segment in the given sentence. If there is no need to substitute it, select No improvement. The firing of the missile not only killed innocent persons and also created political furore. (a) not only kill innocent persons and also (b) not only killed innocent persons but also (c) No improvement (d) but killed innocent person and also SSC CHSL (Tier-I) 14/10/2020 (Shift-I) Ans. (b) : ‘Not only killed innocent persons but also’ will be used in place of the underlined part of the above sentence ‘Not only killed innocent persons and also’. Because ‘Not only-------- but also’ is used together. Correct Sentence– The firing of the missile not only killed innocent persons but also created political furore. 422. Select the most appropriate option to substitute the underlined segment in the given sentence. If there is no need to substitute it select No improvement. So much as you behave like this, the decision of the management against you cannot as revoked. (a) As long to (b) As much to (c) As long as (d) No improvement SSC CHSL (Tier-I) 12/10/2020 (Shift-III) Ans. (c) : In the above sentence, there are two clause in question, Hence to join the clause, “As long as’ is used appropriate in place of conjunction ‘so much as’ Correct Sentence– As long as you behave like this, the decision of the management against you cannot as revoked. 423. Select the most appropriate option to substitute the underlined segment in the given sentence. If there is no need to substitute it, select 'No improvement'. Lenovo has launched ThinkPad XI Fold, the world's first foldable laptop which is claimed to be so durable as its other devices. (a) to be as durable as (b) to be most durable as (c) to be very durable as (d) No improvement SSC CHSL (Tier-I) 12/10/2020 (Shift-I) Ans. (a) : The underlined segment of the above sentence ‘as’ will be used in place of ‘so’. Because it is often used for comparison. Because the used of ‘as ----- as’ gives the appropriate meaning. Correct Sentence– Lenovo has launched ThinkPad XI Fold, the world's first foldable laptop which is claimed to be as durable as its other devices.

154

YCT

(a) unless (b) No improvement 424. Improve the bracketed part of the sentence. (c) even (d) because To conform to a set of rules (a) Regulations SSC MTS – 20/08/2019 (Shift-III) without a real voice in their formulation (a) and (b) to Ans. (a) : ‘Unless’ will be used in place of the (c) but (d) No improvement underlined part of the above sentence. According to SSC MTS 9-10-2017 (Shift-III) sense of sentence, ‘unless’ is appropriate for Ans : (a) In the given sentence, the use of ‘and’ will be conditional sentence. The use of ‘unless’ is used in appropriate in place of ‘a’. Which gives suitable negative sentence. meaning for the sentence. Correct Sentence ''Pratham will not go the party unless he is personally Correct Sentence– To conform to a set of rules and Regulations without a invited.'' real voice in their formulation. 429. Select the most appropriate option to substitute 425. Select the most appropriate option to substitute the underlined segment in the given sentence. If the underlined segment in the given sentence. If there is no need to substitute it, select No there is no need to substitute it, select No improvement. improvement. Rashmi is not only smart but very beautiful. (a) but beautiful Until you remain calm, you cannot concentrate. (a) Unless you remains (b) Unless you remain (b) No improvement (c) No improvement (d) If you remain (c) very beautiful (d) but also very beautiful SSC MTS – 08/08/2019 (Shift-I) SSC MTS – 20/08/2019 (Shift-II) Ans.(b): ‘unless you remain’ will be used in place of Ans. (d) :The underlined part of the above sentence, the underlined part of the above sentence ‘until you remain’. Because it gives suitable meaning. ‘But also very beautiful’ will be used in place of ‘But Until means :- up to the time that very beautiful.’ Unless means :- except on the condition that. Rule:- ‘But also’ is used with ‘not only’ as a corelative conjunction. Correct sentence will be Unless you remain calm, you can not concentrate. Correct Sentence 426. Select the most appropriate option to substitute Rashmi is not only smart but also very beautiful. the underlined segment in the given sentence. If 430. Select the most appropriate option to substitute there is no need to substitute it, select No the underlined segment. If there is no need to Improvement. substitute it, select No improvement. He not only speaks Hindi fluently but English The student stood up no sooner the Principal as well as. entered the class. (a) No improvement (a) no soon (b) as soon as (b) but also English (c) No improvement (d) soon (c) as well as English SSC MTS – 20/08/2019 (Shift-I) (d) but English too as well Ans. (b) : ‘As soon as’ Will be used in place of the SSC MTS – 07/08/2019 (Shift-I) underlined part of the above sentence ‘no sooner’. Ans. (b) : ‘But also English’ will be used in place of Because the use of ‘as soon as’ is used as a the underlined sentence ‘but English as well as’. conjunction. Because 'but also' is used with 'not only'. Correct Sentence– The student stood up as soon as the Principal entered Correct Sentence He not only speaks Hindi fluently but also English. the class. 427. Select the most appropriate option to substitute 431. Select the most appropriate option to substitute the underlined segment in the given sentence. If the underlined segment in the given sentence. If there is no need to substitute it, select No there is no need to substitute it, select No Improvement. improvement. Not only the parents but also the children were I asked two pedestrians the way to the station, sad at leaving the house. and neither of them couldn't help me. (a) as also (b) so even (a) and either them couldn't (c) and also (d) No Improvement (b) No improvement (c) but neither of them could SSC MTS – 05/08/2019 (Shift-III) (d) but none of they could Ans. (d) : The underlined segment in the given SSC MTS – 13/08/2019 (Shift-II) sentence, there is no need to substitute it. Ans. (c) : ‘could’ will be used in place of the Rule :- ‘But also’ is used with not only as a underlined part of the above sentence ‘couldn’t’. correlative conjunction. Because the use of ‘neither’ is used so ‘not’ will not be 428. Select the most appropriate option to substitute appropriate. the underlined segment in the given sentence. If Note:-The use of ‘not/negative’ is not used with until , there is no need to sibstitute it, select No unless, neither, nor, lest. improvement. Correct Sentence Pratham will not go the party if he is I asked two pedestrians the way to the station, but personally invited. neither of them could help me.]

Sentence Improvement

155

YCT

432. Select the most appropriate option to substitute the underlined segment in the given sentence. If there is no need to substitute it, select No improvement. We must start now unless it will be too late. (a) but (b) or (c) No improvement (d) until SSC MTS – 07/08/2019 (Shift-II) Ans. (b) : ‘or’ will be used in place of the underlined part of the above sentence ‘unless’. Because ‘or’ is used in negative sentence or introduce a different or opposite idea. Correct SentenceWe must start now or it will be too late.

H.

Preposition

433. Select the most appropriate option to substitute the bracketed segment in the given sentence. If there is no need to substitute it, select 'no substitution required'. (Because he has been in custody from last July.) Pickard will now be released in four months. (a) Because he had been under custody from last July, (b) No substitution required (c) Because he had been in custody from last July, (d) Because he has been in custody since last July, SSC MTS 19/05/2023 (Shift-III) Ans. (d) : The bracketed segment of the above given sentence (Because he has been in custody from last July) is grammatically error, because preposition ‘since’ is used in the place of ‘from’. because 'since' is used before definite point of time. 434. Select the most appropriate option to substitute the bracketed segment in the given sentence. If there is no need to substitute it, select 'no substitution required'. He has 33 years of professional experience (on the field of) Public Administration. (a) in the field off (b) over field of (c) No substitution required (d) in the field of SSC MTS 16/05/2023 (Shift-II) Ans. (d) : The bracketed segment of the given sentence (on the field of) has grammatically error, because 'in' preposition is used in the place of 'on'. The correct segment is- (in the field of). 435. Select the most appropriate option that can substitute the underlined segment in the given sentence. The pleasure created by the poet was primarily until his exaggerated descriptions. (a) Below his exaggerated descriptions (b) Beside his exaggerated descriptions (c) Off his exaggerated descriptions (d) Through his exaggerated descriptions SSC GD 31/01/2023 (Shift-II) Sentence Improvement

Ans. (d) : The most appropriate option that can substitute the underlined segment in the given sentence is option (b) 'through his exaggerated descriptions' because 'through' is used to show a result of something. Hence option (d) is correct. Correct SentenceThe pleasure created by the poet was primarily through his exaggerated descriptions. 436. Select the most appropriate option that can substitute the underlined segment in the given sentence. Their plan to action during that critical situation was well appreciated by all. (a) Of action during that (b) In action during that (c) Of action off that (d) Of action about that SSC GD 08/02/2023 (Shift-IV) Ans. (a) : 'of action during that' is the most appropriate option that can substitute the underlined segment in the given sentence– 'to action during that'. Hence option (a) 'of action during that' is correct answer. 437. Select the correct option substitute the underlined segment in the given sentence. If no substitution is required, select 'No substitution'. She is married with a doctor. (a) No substitution (b) to a doctor (c) a doctor (d) for a doctor SSC GD 01/02/2023 (Shift-II) Ans. (b) : In the underlined part of the above sentence, 'to a doctor' will be used in place of 'with a doctor'. Because we need a preposition when we use the verb 'to marry' in the passive voice. The correct sentence is– She is married to a doctor. Hence option (b) is correct answer. 438. Select the most appropriate option that can substitute the underlined segment in the given sentence. The players distributed the reward between themselves (a) among themselves (b) between them (c) amongst their self (d) between their selves SSC CGL (Tier-I) 21/07/2023 (Shift-II) Ans. (a) : The underlined segment in the above given sentence 'between themselves' is grammatically error. The most appropriate substitute is option (a) 'among themselves' because 'between' is used for two people or things and 'among' used more then two people or things. Correct SentenceThe players distributed the reward among themselves.

156

YCT

439. Select the most appropriate option that can substitute the underlined segment in the given sentence. I have not seen you for last year. (a) you for last years (b) me for last year (c) you since last year (d) you about last year SSC CGL (Tier-I) 18/07/2023 (Shift-III) Ans. (c) : The most appropriate substitute of the above underlined segment 'you for last year' is 'you since last year', because 'since' is used in the place of 'for' and show the relation to a specific date, time or period in the past while 'for' is used of a length of time. Note - Since + last year. for + the last year. Correct SentenceI have not seen you since last year. 440. Select the most appropriate option that can substitute the underlined segment in the following sentence. I need to put out my clothes before I leave for work this morning otherwise, I will be restless when I return. (a) need to put in (b) need to put away (c) need to put off (d) need for put out SSC CHSL (Tier-I) 10/08/2023 (Shift-I) Ans. (b) : The underlined part 'need to put out' is required to be substituted with option (b) need to put away. Because 'Put away' renderers the meaning, To put something into place where it is normally kept'. Using it will meet with the sense and context of the sentence. Correct sentence I need to put away my clothes before I leave for work this morning otherwise, I will be restless when I return. 441. Select the most appropriate option that can substitute the underlined segment in the given sentence. If there is no need to substitute it, select ‘No substitution required’. Ria sits right besides Shimona in the craft class. (a) besides to (b) beside (c) be side (d) No substitution required SSC Selection Posts XI-28/06/2023 (Shift-III) Ans. (b) : In the above sentence option (b) "besides" (Except) will be replaced by 'beside' (on one side) gives appropriate meaning according to sense of the sentence. Correct sentenceRita gifts right beside Shimona in the craft class. 442. Select the most appropriate option that can replace the underlined part of the following sentence. It has been raining from morning. (a) for (b) till (c) though (d) since SSC CHSL (Tier-I) 09/08/2023 (Shift-III) Sentence Improvement

Ans. (d) : The underlined part 'from' in the above mentioned sentence can be replaced by option (d) 'since' because 'since' is used to refer 'a point of time'. Ex – Since last week, since yesterday Since Wednesday, since 1990, Since evening, etc. Correct Sentence It has been raining since morning. 443. Select the most appropriate option that can substitute the underlined segment in the given sentence. Amar is very good in mathematics. (a) good at (b) good about (c) good on (d) good with SSC CHSL (Tier-I) 14/08/2023 (Shift-IV) Ans. (a) : The underlined segment in the sentence 'Good in' can be substituted with option (a) 'good at'. Because 'good + at' is used to talk about things that is done skillfully Correct Sentence – Amar is very good at mathematics. 444. Select the most appropriate option that can substitute the underlined segment in the following sentence. Can you please make sure that all the guest are gathered on the stage before the performance begins? (a) guests are gathered by (b) guest is gathered on (c) guests is gathered by (d) guests are gathered in SSC CHSL (Tier-I) 08/08/2023 (Shift-II) Ans. (a) : The most appropriate substitute of the above underlined segment is option (a) 'guests are gathered by' because 'all' is used with a plural noun, it means every and 'by' preposition is used in the place of 'on' in the sense of beside the stage. Hence, the correct options is (a). 445. Select the option that will improve the (bracketed) part of the sentence. In case of no improvement select the option 'No improvement'. There's even a (picture by which a witch is riding) a broomstick on the sides of the police cars. (a) No improvement (b) picture that of a witch riding (c) pictures of witch riding (d) picture of a witch riding SSC CHSL (Tier-I) 15/03/2023 (Shift-III) Ans. (d) : In the above bracketed part of the sentence ''picture by which a witch is riding'' in place of ''Picture of a witch riding'' will be used, because it gives according to the sense of the sentence. Correct sentenceThere's even a picture of a witch riding a broomstick on the sides of the police cars. 446. Select the option that will improve the underlined part of the sentence. In case no improvement is needed, select 'No improvement'. The apple tree was loaded of fruit.

157

YCT

(a) No improvement (b) loaded with (c) loaded from (d) laden with SSC CGL (Tier-II) 08.08.2022 Shift-II Ans. (b) : The underlined part of the sentence, ‘Loaded with’ will be used in place of ‘Loaded of’ because ‘with’ Preposition is used with ‘loaded’. Correct sentence– The apple tree was loaded with fruit. 447. Select the most appropriate option to improve the underlined segment in the given sentence. If there is no need to improve it, select 'No improvement required'. The little girl chatters away as she hurry up the road with her mother. (a) No improvement required (b) hurried up (c) hurries down (d) hurry on SSC Stenographer (Grade C & D) 11.11.2021 Shift-I Ans. (c) : The underlined segment in the given sentence ‘Hurries down’ will be used in place of ‘hurry up’ because pronoun ‘she’ is singular. She + V1 + s/ es / ies + object. Correct sentence The little girl chatters away as she hurries down the road with her mother. 448. Select the most appropriate option to substitute the underlined segment in the given sentence. If no substitution is required, select 'No substitution'. I had no choice but from agree to the terms of the contract. (a) but for (b) but was (c) but to (d) No substitution SSC Stenographer (Grade C & D) 11.11.2021 Shift-I Ans. (c) : The underlined part of the above sentence, ‘but to’ will be used in place of ‘but form’ because according to sense of sentence, it gives suitable meaning. Correct sentence I had no choice but to agree to the terms of the contract. 449. There is an underlined segment in the given sentence. Select the most appropriate option to substitute the segment. If there is no need to substitute it, select 'No substitution required'. Adding to the problem of heavy rain, people have to worry about potholes on roads. (a) No substitution required (b) By adding to the problem of (c) Added to the problem of (d) In addition to the problem of SSC Stenographer (Grade C & D) 12.11.2021 Shift-II Ans. (d) : The underlined part of above sentence , ‘In addition to the problem of’ will be used in place of ‘Adding to the problem’ because it gives proper meaning of this sentence. 450. Select the most appropriate option to improve the underlined segment in the given sentence. If there is no need to improve it, select 'No improvement required'.

Sentence Improvement

The football team has won the championship at the second time. (a) for the (b) in the (c) No improvement required (d) at a SSC Stenographer (Grade C & D) 15.11.2021 Shift-I Ans. (a) : The underlined part of above sentence, 'for the' will be used in place of 'at the' because it gives proper meaning of this sentence. 451. Select the most appropriate option that can substitute the underlined segment in the given sentence. We want to divide the expenses between the three of us. (a) from (b) for (c) among (d) at SSC CHSL 24.05.2022 Shift-III Ans. (c) : The underlined part of the above sentence, ‘among’ will be used in place of ‘between’ because ‘three of us’ is used in the sentence. ‘Among’ is used for more than two people / things. While ‘between’ is used for two. Correct Sentence We want to divide the expenses among the three of us. 452. Select the most approrpaite option that can substitute the underlined segment in the given sentence. I was waiting from two hours. (a) for (b) since (c) at (d) in SSC CHSL 26.05.2022 Shift-I Ans. (a) : The underlined part of the above sentence, preposition ‘For’ will be used in place of ‘from’ . Because ‘for’ is used to refer period of time. Correct sentence– I was waiting for two hours. 453. Select the most appropriate option that can substitute the underlined segment in the given sentence. If there is no need to substitute it, select 'No substitution required'. The investigation revealed that both of they had given false information to obtain the certificates. (a) No substitution required (b) reveals that how they both (c) revealed that both of them (d) reveal that them both SSC CGL (Tier-I) 12/04/2022 Shift-II Ans. (c) : ‘Revealed that both of them’ will be used in place of ‘raveled that both of they’ in the underlined part of the above sentence. Because objective form (them ) is always used after preposition. Correct Sentence The investigation revealed that both of them had given false information to obtain the certificates.

158

YCT

454. Select the option that will improve the underlined part of the given sentence. In case no improvement is needed, select ‘No improvement required’. You are welcome to partake this light refreshment. (a) for partaking (b) to partake of (c) in partaken (d) No improvement required SSC CGL (Tier-I) 13/04/2022 Shift-III Ans. (b) : In the given sentence, ‘to partake of’ will be used in place of ‘to partake’ because preposition ‘of’ is used after ‘partake’. 455. Select the most appropriate option that can substitute the underlined segment in the given sentence. If there is no need to substitute it, select ‘No substitution required’. The student apologised the teacher for the delay in submitting the assignment. (a) is apologising the teacher (b) No substitution required (c) apologised to the teacher (d) has apologised the teacher SSC CGL (Tier-I) 11/04/2022 Shift-III Ans. (c) : ‘Apologised to the teacher’ will be used in place of ‘apologised the teacher’ in the underlined part of the above sentence. Because – apologized to + somebody / someone. Correct sentence – The student apologised to the teacher for the delay in submitting the assignment. 456. Select the option that will improve the underlined part of the sentence. In case no improvement is needed, select 'No improvement required'. You are looking so pretty on these black outfit. (a) pretty in this (b) No improvement (c) prettier in these (d) prettiest with those SSC MTS-13/07/2022 Shift-I Ans. (a) : ‘Pretty in this’ will because in place of ‘pretty on these’ in the underlined part of the above sentence. Because ‘In’ Preposition will be used in the sense of ‘pretty in this cloth’. Correct Sentence – You are looking so pretty in this black outfit. 457. Select the most appropriate option that can substitute the underlined segment in the given sentence. If there is no need to substitute it, select ‘No substitution required’. The environment is on threat hedgerows and woodlands are vanishing. (a) into threat (b) under threat (c) No substitution required (d) in threat SSC Constable GD-22/11/2021 Shift-II Sentence Improvement

Ans. (b) : In the above sentence, the use of under threat (danger) will be appropriate in place of ‘on threat’. because it gives proper meaning for sentence. Another preposition is not suitable. Correct Sentence– The environment is under threat hedgerows and woodlands are vanishing. 458. Select the most appropriate option that can substitute the underlined segment in the given sentence. If there is no need to substitute it, select ‘No substitution required’. I thanked my teacher for explaining me the lesson. (a) explain me the lessons (b) No substitution required (c) her explaining me the lesson (d) explaining the lesson to me SSC Constable GD-18/11/2021 Shift-I Ans. (d) : The underlined part of the above sentence, ‘explaining the lesson to me’ will be used in place of ‘explaining me the lesson’. Because 'explain something to somebody' is used together. Note:- Explain to somebody. Correct Sentence– I thanked my teacher for explaining the lesson to me. 459. Select the option that will improve the underlined part of the given sentence. In case no improvement is needed, select 'No improvement required'. The farmer was deprived from his land by the money lender. (a) depriving in (b) No improvement required (c) deprived of (d) deprived on SSC Constable GD-06/12/2021 Shift-III Ans. (c) : ‘Deprived of’ will be used in place ‘deprived from’ in the underlined segment of the above sentence. Because ‘of’ preposition is used with ‘deprived’. Correct sentence – The farmer was deprived of his land by the money lender. 460. Select the option that will improve the underlined segment in the given sentence. In case no improvement is needed, select 'No improvement' It is wise to insure one's home on fire. (a) in (b) No improvement (c) by (d) against SSC CHSL (Tier-I) –16/04/2021 (Shift-I) Ans. (d) : According to sense of the above sentence , ‘against’ will be appropriate in place of the underlined word ‘on’ . Another options give different meaning. Correct Sentence It is wise to insure one's home against fire. 461. Select the most appropriate option to substitute the underlined segment in the given sentence. If there is no need to substitute it, select 'No substitution.' Small desert animals burrow underground across the days and come out only at night.

159

YCT

(a) within the days (b) during the day (c) No substitution (d) between days SSC CHSL (Tier-I) –06/08/2021 (Shift-I) Ans. (b) : In the underlined part of the above sentence, ‘during the day’ will be used in place of ‘across the day.’ Because ‘during’ is used for expressing the ‘period of time’. Correct Sentence Small desert animals burrow underground during the day and come out only at night. 462. Select the option that will substitute the underlined part of the given sentence. In case no substitution is needed, select 'No substitution required'. Japan has made important and original contributions inside some fields like photography and electronics. (a) for some field like (b) in some fields like (c) No substitution required (d) on some field like SSC CGL (Tier-I) –24/08/2021 (Shift-I) Ans. (b) : ‘In some field like’ will be used in place of ‘inside some fields’ in the underlined segment of the above sentence. Because the use of ‘in’ gives the suitable meaning. Correct SentenceJapan has made important and original contributions in some fields like photography and electronics. 463. Select the most appropriate option to substitute the underlined segment in the given sentence. If no substitution is required, select 'No substitution'. After the old building was pulled out the building contractor surveyed the progress of the work. (a) was pulled up (b) was pulled over (c) was pulled down (d) No substitution SSC CGL (Tier-I) –20/08/2021 (Shift-I) Ans. (c) : ‘was pulled down’ will be used in place of ‘was pulled out’ in the underlined segment of the above sentence. Because ‘pull down’ is used to 'demolish the building'. Another option is not suitable. Pull down :- to demolish a building. Pull over :- to remove someone or something form an activity. Pull up :- to bring to a stop. (Holt). Correct Sentence After the old building was pulled down the building contractor surveyed the progress of the work. 464. Select the most appropriate option to improve the underlined segment in the given sentence. If there is no need to improve it, select 'No improvement'. We are coping the problems with the best of our ability. (a) No improvement (b) coping in the problems by (c) coping with the problems to (d) coping the problems at SSC CGL (Tier-II) 16/11/2020 (3 pm - 5 pm)

Sentence Improvement

Ans. (c) : The underlined part of the above sentence ‘coping with the problems to’ will be used in place of ‘coping the problem with. Because ‘with’ is used with cop. Correct Sentence - We are coping with the problems to the best of our ability. 465. Select the most appropriate option to improve the underlined segment in the given sentence. If there is no need to improve it, select 'No improvement'. The child put a ladder on the wall and climbed up. (a) over a wall (b) above the wall (c) No improvement (d) against the wall SSC CGL (Tier-II) 16/11/2020 (3 pm - 5 pm) Ans. (d) : ‘Against the wall’ will be used in place of ‘on the wall’, in the underlined part of the above sentence. Because when something rest on some support then ‘against’ is used. Correct Sentence - The child put a ladder against the wall and climbed up. 466. Select the most appropriate option to improve the underlined segment in the given sentence. If there is no need to improve it, select 'No improvement'. This stain can be remove by lime juice. (a) remove from (b) No improvement (c) removed with (d) removed through SSC CGL (Tier-II) 16/11/2020 (3 pm - 5 pm) Ans. (c) : ‘Removed with’ will be used in place of ‘remove by’ in the underlined part of the above sentence. Because preposition ‘with’ is appropriate with ‘remove’. Correct Sentence - This stain can be remove with lime juice. 467. Select the most appropriate option to improve the underlined segment in the given sentence. If their is no need to improve it, select 'No improvement'. I bet you can't beat me by chess. (a) on chess (b) in the chess (c) No improvement (d) at chess SSC CGL (Tier-II) 16/11/2020 (3 pm - 5 pm) Ans. (d) : The underlined part of the above sentence, ‘at chess’ will be used in place of ‘by chess’. Because 'beat + somebody + at + something.' Correct Sentence - I bet you can't beat me at chess. 468. Select the alternative that will improve the underlined part of the sentence in case there is no improvement select “No improvement”. Using seeds for grow more plants is an exceptionally good way of gardening. (a) No improvement (b) to growing more plants (c) to grow more plants (d) for grows more plants SSC CPO-SI (Tier-II) 27/09/2019 (3 pm - 5 pm) Ans. (c) : In the above underlined part, 'to grow more plants’ will be used in place of ‘for grow more plants’. Because ‘ing’ form will be appropriate after ‘for’ preposition and the first form of verb (vI) is suitable after ‘to’. Correct sentence - Using seeds to grow more plants is an exceptionally good way of gardening.

160

YCT

469. Select the alternative that will improve the underlined part of the sentence. In case there is no improvement select "No improvement". My friend said, "Can you please suggest for me a good school in the neighbourhood?" (a) suggest to me (b) suggest (c) suggesting me (d) No improvement SSC Stenographer Grade C&D –05/02/2019 (3:5pm)

Ans. (a) : The underlined part of the above sentence, ‘suggest to me’ will be used in place of ‘suggest for me’. Because ‘to’ preposition is used after ‘suggest.’ (suggest to somebody) Correct Sentence - My friend said, "Can you please suggest to me a good school in the neighbourhood?" 470. Improve the bracketed part of the sentence. Election was an ‘opinion minus democracy’ in which the innocent and helpless (people were pitted among) government’s muscle power State machinery. (a) people were pitted against (b) people are pitted against (c) people would pitted again st (d) No improvement SSC CGL (Phase-II) 17/02/2018 Ans. (a) The bracket part, of the above sentence, correct preposition (against) will be used in place of people were pitted among. Note :- ‘Against’ is used for against (government's) while ‘Among’ is used for (between more than two people). Correct Sentence Election was an ‘opinion minus democracy’ in which the innocent and helpless people were pitted against government’s muscle power State machinery. 471. Select the most appropriate option to substitute the underlined segment in the given sentence. If there is no need to substitute it, select 'No substitution' The lockdown prevented people to go out from their homes. (a) to go outside from (b) No substitution (c) of going out from (d) from going out of SSC Sel. Post Phase-VIII (G.L.) 09.11.2020 (Shift-II)

Ans. (d) : In the above sentence, ‘From going out of’ will be used in place of ‘to go out from’. Because the following structure is used after prevent (verb). Prevent from doing something. Correct sentence The lockdown prevented people from going out of their homes. 472. Select the most appropriate option to substitute the underlined segment in the given sentence. If there is no need to substitute it, select ‘No substitution required’. The mare hurt its hooves when running to the forest. (a) with running across (b) No substitution required (c) while running through (d) when running into SSC CPO-SI – 24/11/2020 (Shift-I) Sentence Improvement

Ans. (c) : The underlined phrase of the given sentence. ‘while running through’ will be used in place of ‘when running to’. Because we use both ‘when’ and ‘while’ as a sub-ordinating [while + continuous form (V1+ing)] conjunctions to introduce adverbial clause of time. Correct Sentence– The mare hurt its hooves while running through the forest. 473. Select the most appropriate option to substitute the underlined segment in the given sentence. If there is no need to substitute it, select ‘No substitution required’. Every year the family goes for a pilgrim to the temple at Tirupathi. (a) at the pilgrim (b) on a pilgrimage (c) No substitution required (d) from the pilgrimage SSC CPO-SI – 24/11/2020 (Shift-I) Ans. (b) : The underlined phrase of the given sentence, ‘on a pilgrimage’ will be used in place of ‘for a pilgrim’. Because preposition ‘on’ is suitable in place of ‘preposition for’. Because ‘on’ is generally used when we want to indicate a position above or a top a space. ‘For’ is used to indicate reason and specify the use of something. Correct Sentence – Every year the family goes on a pilgrimage to the temple at Tirupathi. 474. Select the most appropriate option to substitute the underlined segment in the given sentence. If there is no need to substitute it, select ‘No substitution required’. The police was accused to tamper with the evidence. (a) accused of tampering (b) accusing to tamper (c) No substitution required (d) accused to tampered SSC CPO-SI – 24/11/2020 (Shift-II) Ans. (a) : The underlined part of the above sentence, ‘accused of tampering’ will be used in place of ‘accused to temper’. Because preposition ‘of’ is used after accused. Correct Sentence– The police was accused of tampering with the evidence. 475. Select the most appropriate option to substitute the underlined segment in the given sentence. If no substitution is required, select No improvement. We must endeavour to increase women's access for education also employment. (a) access in education also employment (b) excess to education or employment (c) No improvement (d) access to education and employment SSC CGL (Tier-I) – 07/06/2019 (Shift-III)

161

YCT

Ans : (d) The underlined part of the above sentence, ‘Access to’ will be used in place of ‘Access for’. Because suitable preposition ‘to’ is used after access. Access to - a way of entering or reaching a place. Correct Sentence We must endeavour to increase women's access to education also employment. 476. Select the most appropriate option to substitute the underlined segment in the given sentence. If no substitution is required, select No improvement. An old misunderstanding exists among the two families, so they are not friendly any more. (a) has exist within the two families (b) No improvement (c) exists between the two families (d) will exist among the two families SSC CGL (Tier-I) – 13/06/2019 (Shift-III) Ans : (c) In the above underlined part 'exists between' will be used because ‘Among’ is always used for more than two and ‘between’ is used for two. Hence for two families ‘between’ will be used. Correct Sentence– An old misunderstanding exists between the two families, so they are not friendly any more. 477. Select the most appropriate option to substitute the underlined segment in the given sentence. If there is no need to substitute it, select 'No substitution'. The authorities are looked on the matter. (a) No substitution (b) are looked into the matter (c) are looking into the matter (d) are looking on the matter SSC CPO-SI – 09/12/2019 (Shift-I) Ans : (c) In the above sentence, ‘Are looking into’ will be used in place of ‘are looked on’. Look into- to try to discover the facts about something such as a problem or a crime. Correct Sentence The authorities are looking into the matter. 478. Select the most appropriate option to substitute the underlined segment in the given sentence. If there is no need to substitute it, select No improvement. Raman's discovery, made during the voyaging over the Mediterranean sea, that water molecules could scatter light just like air molecules, was very radical in those days. (a) while an voyage across (b) during a voyage across (c) No improvement (d) during the voyage above SSC CPO-SI – 12/12/2019 (Shift-II) Ans : (b) In the underlined part of the above sentence, preposition ‘Across’ is suitable in place of ‘over’ because Across – from one side to other side of something. Correct Sentence Raman's discovery, made during a voyage across the Mediterranean sea, that water molecules could scatter light just like air molecules, was very radical in those days. Sentence Improvement

479. Select the most appropriate option to substitute the underlined segment. If no substitution is required select 'No substitution' Employees refrain to indulge in underable activities if they are monitored. (a) refrain against indulging (b) refrain from indulging (c) refrain for indulgence (d) No substitution SSC CPO-SI – 13/12/2019 (Shift-I) Ans. (b) : In place of the above underlined part ‘refrain from indulge’, ‘refrain from indulging’ will be used because preposition ‘from’ is used with refrain. Correct Sentence– Employees refrain from indulging in underable activities if they are monitored. 480. Select the most appropriate option to substitute the underlined segment. If no substitution is required select 'No substitution' The student, with his parents were asked for waiting outside the Principal's office. (a) were asked to wait (b) was asked to wait (c) were asked to be waiting (d) No substitution SSC CPO-SI – 13/12/2019 (Shift-I) Ans. (b) : The underlined part of the above sentence, ‘was asked to wait’ will be used in place of ‘were asked for waiting.’ Because the subject of this sentence is singular. Note:Ask for :- to ask someone to give you something. Correct Sentence– The student, with his parents was asked to wait outside the Principal's office. 481. Select the most appropriate option to substitute the underlined segment in the given sentence. If there is no need to substitute it, select No improvement. The government has adopted a tough stance with respect of terror and security. (a) in respect of (b) about respect to (c) No improvement (d) with respect to SSC CHSL (Tier-I) –11/07/2019 (Shift-II) Ans : (d) The underlined segment of the above sentence, ‘with respect to’ will be used in place of ‘with respect of’. Because preposition (to) is used with respect. Correct Sentence–The government has adopted a tough stance with respect to terror and security. 482. Select the most appropriate option to substitute the underlined segment in the given sentence. If there is no need to substitute it, select No improvement. What a relief to sit down after walking in the park from a hour! (a) by an hour (b) No improvement (c) for an hour (d) in a hour SSC CHSL (Tier-I) –09/07/2019 (Shift-II)

162

YCT

Ans : (c) ‘For an hour’ will be used in place of ‘from a hour’ in the underlined segment of the above sentence. Because ‘for’ is used for period of time and ‘hour’ is pronounced with a vowel sound. So ‘An’ will be used as its first article. Correct Sentence–What a relief to sitdown after walking in the park for an hour. 483. Identify the most appropriate option to substitute the underlined segment. If no substitution is required, select No substitution. I was astonished on the ignorance in the villagers. (a) over the ignorance between (b) No substitution (c) on the ignorance by (d) at the ignorance of SSC CHSL (Tier-I) –08/07/2019 (Shift-I) Ans : (d) The underlined part of the above sentence, ‘At’ and ‘of’ will be used in place of ‘on and in’ Respectively. Because Astonished is always followed by 'at' preposition. And According to the meaning of the sentence – ignorance of the villagers will be used. Correct Sentence I was astonished at the ignorance of the villagers. 484. Identify the most appropriate option to substitute the underlined segment. If no substitution is required, select No substitution. The manufacturer conformed with the specifications given from the client. (a) with the specifications given of (b) No substitution (c) to the specifications given by (d) by the specifications given for SSC CHSL (Tier-I) –05/07/2019 (Shift-II) Ans : (c) Preposition ‘to’ is used with conformed in the above sentence. Conform with - has the shape that is required by. Conform to – be/act in accordance with and 'from' will be replace ‘by’. ‘By’ will be used in place of ‘be/act in accordance with and from.' Correct Sentence The manufacturer conformed to the specifications given by the client 485. Identify the most appropriate option to substitute the underlined segment. If no substitution is required, select No substitution. You must not prevent me to do what I want. (a) by doing what I wanted (b) from doing what I want (c) to do what I am wanting (d) No substitution SSC CHSL (Tier-I) –05/07/2019 (Shift-II) Ans : (b) The underlined part of the above sentence, ‘for doing’ will be used in place of ‘to do’. Because prevent is always used with ‘from’ in the given sentence and verb, ing form’ is used after it. Correct Sentence– You must not prevent me from doing what I want. 486. Identify the most appropriate option to substitute the underlined segment. If no substitution is required, select No substitution. I have a terrible cold during several days now. Sentence Improvement

(a) (b) (c) (d)

had a terrible cold since have had a terrible cold for No substitution am having a terrible cold during SSC CHSL (Tier-I) –05/07/2019 (Shift-III) Ans : (b) The underlined part of the above sentence, ‘have had a terrible cold for’ will be used in place of ‘have a terrible cold during’ Because in sentence, ‘for’ is used for period of time so ‘sentence’ will be used in present perfect tense. "

v1 has / have

v2 →

had

v3 →

had

"

Correct Sentence– I have had a terrible cold for several days now. 487. Identify the most appropriate option to substitute the underlined segment. If no substitution is required, select No substitution. The shoebill stork can be recognise from its long legs and broad wings. (a) recognised by its (b) recognise through it's (c) recognising from it's (d) No substitution SSC CHSL (Tier-I) –05/07/2019 (Shift-III) Ans : (a) The underlined part of the above sentence, ‘recognised by its’ will be used in place of 'recognize form its.' When sentence is in passive form, then ‘Can be’ is followed by the third form of the verb. ‘By’ will be used instead of ‘from’ because ‘recognised by something’ is used. Correct Sentence The shoebill stork can be recognised by its long legs and broad wings. 488. Select the most appropriate option to substitute the underlined segment in the given sentence. If there is no need to substitute it, select No improvement. They dissuaded me of visiting the War Memorial but I really wanted to see it. (a) of visit (b) No improvement (c) from visiting (d) to visiting SSC CHSL (Tier-I) –03/07/2019 (Shift-I) Ans : (c) ‘From visiting’ will be used in place of the underlined part of the above sentence ‘of visiting’, because 'dissuade somebody + from + v1 + ing' form will be appropriate. Correct Sentence They dissuaded me from visiting the War Memorial but I really wanted to see it. 489. Select the most appropriate option to substitute the underlined segment in the given sentence. If there is no need to substitute it, select No improvement. Two senior officers have been entrusted with the responsibility by organize the reception of the guest. (a) No improvement (b) for organize (c) of organizing (d) to organizing SSC CHSL (Tier-I) –03/07/2019 (Shift-II) Ans. (b) : The underlined segment of the above sentence, ‘for organizing’ will be used in place of ‘by organize’. Because ‘for’ preposition is used with responsitbility. Correct Sentence Two senior officers have been entrusted with the responsibility for organize the reception of the guest.

163

YCT

490. Select the most appropriate option to substitute the underlined segment in the given sentence. If there is no need to substitute it, select No improvement. He tried to prevent his friend to smoke. (a) from smoking (b) No improvement (c) smoking (d) to smoking SSC CHSL (Tier-I) –02/07/2019 (Shift-II) Ans : (a) In the given sentence, ‘from’ will be used with prevent. Gerund form (V + ing) of verb will be used after ‘from’. Prevent from :- to stop somebody from doing something. Correct Sentence– He tried to prevent his friend from smoking. 491. Select the most appropriate option to substitute the underlined segment in the given sentence. If there is no need to substitute it, select No improvement. The boys were delighted on the idea of going to Goa. (a) at the idea of (b) on an idea of (c) at an idea of (d) No improvement SSC CHSL (Tier-I) –02/07/2019 (Shift-II) Ans : (a) The underlined part of the above sentence. ‘At the idea of’ Will be used in place of ‘on the idea of’. Because the proper preposition would be ‘at’ with delight. Correct Sentence The boys were delighted at the idea of going to Goa. 492. Select the most appropriate option to substitute the underlined segment in the given sentence. If there is no need to substitute it, select No improvement. The children are too young to sit up a long lecture. (a) sit across (b) sit through (c) No improvement (d) sit away SSC CHSL (Tier-I) –02/07/2019 (Shift-III) Ans : (b) In the underlined part of the above sentence, the used of ‘sit through’ (staying till end) would be appropriate. Sit through :- to stay until the end of a performance, speech etc. Note :- The use of too + adj + to’ expresses – negative sentence. Correct Sentence– The children are too young to sit through a long lecture. 493. Select the most appropriate option to substitute the underlined segment in the given sentence. If there is no need to substitute it, select 'NO improvement.' Sele, is a near extinct language, spoken for just five villages in Nepal, which lie close to Tibet. (a) spoken out (b) spoken in (c) No improvement (d) spoken of SSC CHSL (Tier-I) 13/10/2020 (Shift-I) Ans. (b) : ‘spoken in’ will be used in place of the underlined part of the above sentence. because ‘spoken in’ would be appropriate. Correct Sentence– Sele, is a near extinct language, spoken in just five villages in Nepal, which lie close to Tibet. Sentence Improvement

494. Select the alternative that will improve the underlined part of the sentence. In case there is no improvement select 'No improvement'. Priya has been working in this school from June 2017. (a) for June 2017 (b) No improvement (c) on June 2017 (d) since June 2017 SSC CHSL (Tier-I) 18/03/2020 (Shift-II) Ans. (d) : The sentence is present perfect continuous and in the underlined part of the sentence 'since June 2017' will be used. Hence ‘Since June 2017’ is appropriate in place of undelined part ‘from June 2017’. Correct Sentence– Priya has been working in this school since June 2017. 495. Select the most appropriate option to substitute the underlined segment in the given sentence. All the employees of the nationalized bank are in strike. (a) by (b) with (c) for (d) on SSC CHSL (Tier-I) 26/10/2020 (Shift-I) Ans. (d) : The underlined segment in the given sentence, ‘on’ will be used in place of in’. Because ‘on strike’ is used as Idiomatic. ‘On strike’ means :- to refuse to continue working because of an argument with an employer about working conditions. Correct Sentence All the employees of the nationalzed bank are on strike. 496. Select the most appropriate option to substitute the underlined segment in the given sentence. If there is no need to substitute it, select 'No improvement'. The week-long Theatre Festival has begun with a wonderful lines of plays. (a) No imrovement (b) lined up (c) lines up (d) line up SSC CHSL (Tier-I) 26/10/2020 (Shift-II) Ans. (d) : The underlined part of the above sentence, ‘line up’ will be used in place of ‘lines of’. Because the article (a) is used before the word 'line'. Correct Sentence The week-long Theatre Festival has begun with a wonderful line up plays. 497. Select the most appropriate option to substitute the underlined segment in the given sentence. If there is no need to substitute it, select No improvement. Lata told to her team leader that she would be leaving office early as she had some urgent personal work. (a) Lata tell to her team leader (b) Lata telling to her team leader (c) Lata told her team leader (d) No improvement SSC CHSL (Tier-I) 14/10/2020 (Shift-III) Ans. (c) : ‘Told her’ will be used in place of the underlined part of the above sentence ‘told to her’. Because no preposition is used after ‘told' (verb) and direct object comes after told. Correct Sentence– Lata told her team leader that she would be leaving office early as she had some urgent personal work.

164

YCT

498. Select the option that will improve the underlined part of the sentence. In case no improvement is needed, select 'No improvement'. Finally, Ananya found her pet kitten over the top branch of a tree in their garden. (a) across the top branch in the tree (b) No improvement (c) in a top branch of a tree (d) on the top branch of the tree SSC CHSL (Tier-I) 13/10/2020 (Shift-II) Ans. (d) : ‘On the top branch of the tree’ would be appropriate in place of the underlined part of the above sentence ‘over the top branch of a tree’. Here preposition ‘on’ will be used in place of preposition ‘over’, because here the meaning is being made on the top most branch of the tree. Correct Sentence– Finally, Ananya found her pet kitten on the top branch of a tree in their garden. 499. Select the option that will improve the underlined part of the sentence. In case no improvement is needed, select 'No improvement'. As we' ve been neighbours, I have known Mrs. Shashi Singh since more than 16 years. (a) No improvement (b) for more than 16 years (c) from more than 16 years (d) by more than 16 years SSC CHSL (Tier-I) 13/10/2020 (Shift-II) Ans. (b) : The underlined part of the above sentence, ‘for more than 16 years’ will be used in place of ‘since more than 16 years’. Because ‘for’ should be used for expressing the period of time. Correct Sentence As we have been neighbours, I have known Mrs. Shashi Singh for more than 16 years. 500. Improve the bracketed part of the sentence. No mention was made (to a) key amendment to the Constitution to accommodate demands that had been defeated just last Monday. (a) of a (b) in a (c) by a (d) No improvement SSC MTS 9-10-2017 (Shift-I) Ans : (a) ‘of a’ will be used in place of the above bracketed part ‘to a’, because ‘of’ preposition would be appropriate with make. Correct Sentence No mention was made of a key amendment to the Constitution to accommodate demands that had been defeated just last Monday. 501. It includes a (decision of the status) of British and EU migrants resident in their respective territories (a) decision to the status (b) decision on the status (c) decision of the status (d) No improvement SSC MTS 9-10-2017 (Shift-III) Ans : (b) ‘Decision on the status’ will be used in place of the underlined part of the above sentence ‘decision of the status’. Because with decision ‘on’ preposition is always used. Correct Sentence– It includes a decision on the status of British and EU migrants resident in their respective territories Sentence Improvement

502. Improve the bracketed part of the sentence. The terms of a future partnership would be vastly (inferior on) comparison with the benefit of full membership. (a) inferior by (b) inferior to (c) inferior in (d) No improvement SSC MTS 10-10-2017 (Shift-II) Ans. (b) : ‘Inferior to’ would be appropriate in place of in the bracket part of the above sentence ‘inferior on’, because preposition ‘to’ is always used with inferior. Correct Sentence– The terms of a future partnership would be vastly inferior to comparison with the benefit of full membership. 503. Improve the bracketed part of the sentence. Nothing (show on) the ineptness of the state more than unpreparedness in the face of predictable events. (a) Showa off (b) shows in (c) shows up (d) No improvement SSC MTS 10-10-2017 (Shift-I) Ans : (c) The use of ‘shows up’ is correct in bracketed part the above word ‘show on’, because show up Means-: to expose or discredit especially by revealing faults. Which is giving the correct meaning of the sentence. Other options-: Show off :- the act of showing off. Show up -: to lead (someone) to the entrance. Correct Sentence– Nothing shows up the ineptness of the state more than unpreparedness in the face of predictable events. 504. Select the mos appropriate option to substitute the underlined segment in the given in the given sentence. If there is no need to substitute it, select No Improvement. Friends stand by each other on time of need. (a) No Improvement (b) in times (c) for time (d) of times SSC MTS – 06/08/2019 (Shift-III) Ans. (b) : The underlined part of the above sentence. 'In time' will be appropriate according to sense of the sentence. Correct sentence Friends stand by each other in times of need. 505. Select the most appropriate option to substitute the underlined segment in the given sentence. If there is no need to substitute it, select 'No substitution required'. We found out that our competitors were selling product on a much higher price. (a) No substitution required. (b) at a more high price. (c) in highest price. (d) at a much higher price. SSC MTS – 09/08/2019 (Shift-I) Ans. (d) : ‘At a much higher price’ will be used in place of the underlined part of the above sentence ‘on a much higher price’. Because the use of ‘at’ is used beyond a specific condition or time. Correct Sentence– We found out that our competitors were selling product at a much higher price.

165

YCT

506. Select the most appropriate option to substitute the underlined segment in the given sentence. If there is no need to substitute it, select No Improvement. Eight people were injured after a tempo rammed into a car on the highway. (a) rammed with (b) rammed by (c) rams in (d) No improvement SSC MTS – 08/08/2019 (Shift-I) Ans. (c) : ‘Ram in’ will be used in place of the underlined part of the above sentence ‘Rammed into.? Because fix preposition ‘in’ will be used with Ram (bump). Correct sentence Eight people were injured after a tempo rams in a car on the highway. 507. Select the most appropriate option to substitute the underlined segment in the given sentence. If there is no need to substitute it, select No improvement. You must apologise with him for this. (a) No Improvement (b) from (c) of (d) to SSC MTS – 07/08/2019 (Shift-I) Ans. (d) : ‘To’ Would be appropriate in place of the underlined part of the above sentence ‘with’. Because the word ‘apologise’ takes the fix preposition ‘Apologise to a person’ and ‘ apologize for a problem; Correct sentence– You must apologise to him for this. 508. Select the most appropriate option to substitute the underlined segment in the given sentence. If there is no need to substitute it, select No Improvement. Mr. Kansal has been living in Delhi since 10 years. (a) until (b) for (c) No Improvement (d) from SSC MTS – 06/08/2019 (Shift-I) Ans. (b) : ‘For’ will be used in place of the underlined part of the above sentence ‘since’, because the use of ‘for’ is correct for showing the period of time, not since. Correct Sentence– Mr. Kansal has been living in Delhi for 10 years. 509. Select the most appropriate option to substitute the underlined segment in the given sentence. If there is no need to substitute it, select No Improvement. Applications are invited to eligible candidates for the post of Vice Principal in a reputed school. (a) No Improvement (b) invited by (c) inviting from (d) invited from SSC MTS – 05/08/2019 (Shift-II) Ans. (d) : The use of 'Invited from' Would be appropriate in place of the underlined part of the above sentence ‘invited to’ Because ‘invited something + form + somebody’ is used. Correct Sentence– Applications are invited from eligible candidates for the post of Vice Principal in a reputed school. Sentence Improvement

510. Select the most appropriate option to substitute the underlined segment in the given sentence. If there is no need to substitute it, select No Improvement. She insisted to driving her kids to school. (a) insist on (b) insisted on (c) insist to (d) No Improvement SSC MTS – 02/08/2019 (Shift-III) Ans. (b) : The use of ‘on’ preposition is used with the underlined part of the above sentence. ‘Insist’ which means Insist on :- To keep doing something that annoys people. Corrent sentenceShe insisted on driving her kids to school. 511. Select the most appropriate option to substitute the underlined segment in the given sentence. If there is no need to substitute it, select No Improvement. The government seems to have no interest for the welfare of the common man. (a) any interest in (b) no interest to (c) No improvement (d) no interest in SSC MTS – 02/08/2019 (Shift-III) Ans. (d) : ‘In’ preposition is used with the underlined part of the above sentence. Hence, 'no interest in’ will be used in place of ‘no interest for’. Correct Sentence– The government seems to have no interest in the welfare of the common man. 512. Select the most appropriate option to substitute the underlined segment in the given sentence. If there is no need to substitute it, select No Improvement. Who can prevent you from getting married now that you are within age ? (a) in (b) of (c) No Improvement (d) on SSC MTS – 02/08/2019 (Shift-II) Ans. (b) : The underlined part of the above sentence, ‘of’ preposition will be used in place of ‘within’ because ‘of’ is used in the sense of completion of age and ‘within’ is used in the sense of age to decrease. Correct Sentence– Who can prevent you from getting married now that you are of age ? 513. Select the most appropriate option to substitute the underlined segment in the given sentence. If there is no need to substitute it, select No improvement. Garima live at the 25th floor. (a) is living in (b) live in (c) lives on (d) No improvement SSC MTS – 22/08/2019 (Shift-I) Ans. (c) : ‘lives on’ is used in place of the underlined part of the above sentence ‘live at’ because it gives appropriate meaning according to the sentence. Correct sentence Garima lives on the 25th floor.

166

YCT

(a) No improvement 514. Select the most appropriate option to substitute the underlined segment in the given sentence. If (b) She is to fond of there is no need to substitute it, select No (c) She is extreme fond of improvement. (d) She is extremely fond of Manish has not been attending the class for last SSC MTS – 19/08/2019 (Shift-III) month. Ans. (d) : ‘Fond of’ will be used in place of the (a) since last (b) No improvement underlined part of the above sentence 'fond about' (c) from last (d) throughout later because fix preposition ‘of’ will be used with ‘fond’. SSC MTS – 21/08/2019 (Shift-III) Correct sentence– Ans. (a) : ‘Since last’ will be used in place of the She is extremely fond of chocolate and cookies. above underlined word ‘for last’ because ‘last month’ 519. Select the most appropriate option to substitute is point of time. the underlined segment. If there is no need to Since+last/month/year/week,for+the last+month/year/week substitute it, select No improvement. I would be eternally indebted from you if you Correct Sentence– could help me. Manish has not been attending the class since last (a) indebted in you (b) indebted to you month. (c) indebted for you (d) No improvement 515. Select the most appropriate option to substitute SSC MTS – 19/08/2019 (Shift-III) the underlined segment in the given sentence. If there is no need to substitute it, select No Ans. (b) : Preposition ‘to’ is used with the underlined part of the above sentence ‘Indebted’. because it gives improvement. Tommy's eyes grew wide on a sight of the new suitable meaning. Correct sentence bicycle. ''I would be eternally indebted to you if you could help (a) at the sight of (b) at a sight of me.'' (c) on the sight of (d) No improvement SSC MTS – 21/08/2019 (Shift-II) 520. Select the most appropriate option to substitute the underlined segment. If there is no need to Ans. (a) : The underlined segment of the above substitute it, select No Improvement. sentence, ‘at the sight of ‘ will be used in place of ‘on a The child smile on her mother. sight of’. Which is appropriate for the sentence. (a) No Improvement (b) smiled at Correct Sentence– (c) is smiling over (d) smiles on Tommy's eyes grew wide at the sight of the new SSC MTS – 16/08/2019 (Shift-III) bicycle. Ans. (b) : ‘At’ will because in place of the underlined 516. Select the most appropriate option to substitute the underlined segment in the given sentence. If part of the above sentence ‘on’, because ‘at’ there is no need to substitute it, select No preposition is used with smile. Smiled at (sb) means the smile was caused by improvement. someone. The little girl fall off from the terrace. Correct Sentence– (a) fell below from (b) fell under The child smile at her mother. (c) fell from (d) No improvement SSC MTS – 21/08/2019 (Shift-I) 521. Select the most appropriate option to substitute the underlined segment in the given sentence. If Ans : (c) ‘Fell from’ will be used in place of the above there is no need to substitute it, select No underlined part of the sentence ‘fall off from’. improvement. Fall from (Something):- to drop from something at The two thieves divided the loot among some height or to fall of someone or something. themselves. Correct Sentence– (a) between themselves (b) with them The little girl fell from the terrace. (c) No improvement (d) among them 517. Select the most appropriate option to substitute SSC MTS – 16/08/2019 (Shift-II) the underlined segment in the given sentence. If there is no need to substitute it, select No Ans. (a) : In the given sentence, ‘between themselves’ will be used in place of ‘among themselves’. improvement. Rule :- ‘Among’ is used for more than two persons. I'll be at the parlour near 5 pm. ‘Between’ is used for two persons. (a) along (b) No improvement Correct sentence (c) till (d) unless SSC MTS – 20/08/2019 (Shift-II) The two thieves divided the loot between themselves. Ans. (c) : The underlined part of the above sentence, 522. Select the most appropriate option to substitute the underlined segment in the given sentence. If ‘till’ will be used in place of ‘near’ because ‘till’ is there is no need to substitute it, select 'No used for a point of time. It is suitable for point of time substitution required'. because ‘5 P.M’ shows a point of time. You can buy a plastic cover to put into your Correct sentence computer if you're worried about dust. I'll be at the parlour till 5 P.M. (a) put above 518. Select the most appropriate option to substitute (b) No substitution required the underlined segment. If there is no need to (c) put onto subsstitute it, select No improvement. (d) put over She is extremely fond about chocolate and SSC MTS – 13/08/2019 (Shift-III) cookies.

Sentence Improvement

167

YCT

Ans. (d) : In the above question computer is covered with plastic cover. Hence here ‘put over’. Should be used in underline place of ‘put into’. Correct Sentence You can buy a plastic cover to put over your computer if you're worried about dust. 523. Select the most appropriate option to substitute the underlined segment in the given sentence. If there is no need to substitute it, select 'No substitution required'. Suresh thought of a splendid idea to celebrate his parent's anniversary. (a) think about a (b) No substitution required (c) thought of (d) thought at the SSC MTS – 09/08/2019 (Shift-II) Ans. (b) : There is no need to substitute. Note- ‘of’ preposition is used with ‘think’ when on idea has to be told about it. 524. Select the most appropriate option to substitute the underlined in the given sentence. If there is no need to substitute it, select 'No substitution required'. Charles Dickens writers for the effects in industrialization on people's lives. (a) is writing with (b) wrote about (c) No substitution required (d) was writing of SSC GD – 08/03/2019 (Shift-III) Ans. (b) : ‘Wrote about’ will be used in place of the underlined part of the given sentence ‘writes for’ because ‘sentence’ is simple past tense. Hence the use of wrote would be appropriate. Correct Sentence–Charles Dickens wrote about the effects in industrialization on people's lives. 525. Select the most appropriate option to substitute the underlined segment in the given sentence. If there is no need to substitute it, select. No improvement The Indian forces are known for their bravery. (a) force are known for (b) forces are know of (c) No improvement (d) forces in known for SSC GD – 18/02/2019 (Shift-III) Ans. (c) : No improvement Note- * be known for something * be known to somebody 526. Select the most appropriate option to substitute the underlined segment in the given sentence. If there is no need to substitute it, select No Improvement My favourite novelist in all was Charles Dickens, who really left his mark on me. (a) No improvement (b) My favourite novelist of all (c) My favourite novelist between all (d) My favourite novelist at all SSC GD – 14/02/2019 (Shift-I) Ans. (b) : The underlined part of the above sentence, My favourite novelist of all will be used in place of ‘Favourite novelist in all.’ Because it give appropriate meaning according to sense of the sentence. Correct Sentence– My favourite novelist of all was Charles Dickens, who really left his mark on me. Sentence Improvement

527. Select the most appropriate option to substitute the underlined segment in the given sentence. If there is no need to substitute it, select No improvement. The movie Uri tells the story of a surgical strike by the Indian army towards militants in POK. (a) for militants in POK (b) No improvement. (c) against militants in POK (d) over militants at POK SSC GD – 11/02/2019 (Shift-III) Ans. (c) : ‘Against militants in POK’ will be used in place of the underlined part of the above sentence 'towards militants in POK’. Because the use of ‘against; would be appropriate. Correct Sentence– The movie Uri tells the story of a surgical strike by the Indian army against militants in POK.

I.

Voice and Narration

528. Improve the underlined part of the sentence. Choose 'No improvement' as an answer if the sentence is grammatically correct. Each passenger is allows one piece of carry-on luggage. (a) No improvement (b) is allowed (c) is allow (d) is allowing SSC MTS 10/05/2023 (Shift-I) Ans. (b) : For the above given underlined part of the sentence 'is allows' replaced by 'is allowed' because sentence is in passive sense. The correct sentence will be 'Each passenger is allowed one piece of carry on luggage. Thus, the correct answer is option (b). 529. Select the most appropriate option to substitute the underlined segment in the given sentence. If there is no need to substitute it, select 'No substitution'. These buildings will demolish by the end of this year. (a) will be demolishing (b) No substitution (c) will be demolish (d) will be demolished SSC Stenographer (Grade C & D) 12.11.2021 Shift-I Ans. (d) : In the above sentence, 'will be demolished' will be used in place of underlined part 'will demolish' because sentence is in passive voice. P.V. → will + be + V3 Correct sentenceThese buildings will be demolished by the end of this year. 530. Improve the underlined part of the sentence. Choose 'No improvement' as an answer if the sentence is grammatically correct. The medieval knight was knew for his horsemanship. (a) were known (b) was knows (c) No improvement (d) was known SSC CHSL (Tier-I) 09/03/2023 (Shift-IV)

168

YCT

Ans. (d) : For the above underlined part of the sentence 'known' (V3) will be used in place of "knew" (V2) because 'verb3' will be used in Passive voice after 'was/were'. Note : 'was/were + V1+ing' is used in Active voice. Correct Sentence : The medieval knight was known for his horsemanship. 531. Select the most appropraite option that can substitute the underlined segment in the given sentence. Polythelene terephthalate, one of the most often recycled plastics and the material used in the majority of water and soda bottles, can be transformed into everything from polyester fabric to automotive parts. (a) may be converted (b) might be placed (c) may be devaluated (d) might be evolved SSC CGL (Tier-I) 27/07/2023 (Shift-III) Ans. (a) : The underlined segment of the above given sentence 'can be transformed' is grammatically error because 'may be transformed' is used in the place of 'can be'. Note- 'May' denotes of possibility of something. ⇒ (May be + VIII) is used in passive voice. Thus, the correct answer is option (a). 532. Select the option that will improve the underlined part of the given sentence. Adyasha would have been looked gorgeous in ethnic apparel. (a) would have looked (b) had looking (c) would be looked (d) was looked SSC CHSL (Tier-I) 02/08/2023 (Shift-I) Ans. (a) : The underlined part of the given sentence 'would have been looked ' is grammatically error, the correct segment is option (a) 'would have looked' because after 'would have '+ past participle (V3) is used in active voice. Correct sentence is – 'Adyasha would have looked gorgeous in ethnic apparel.' 533. Select the most appropriate option to substitute the underlined segment in the given sentence to make it a meaningful sentence. If no substitution is required, select 'No substitution'. The first step towards the prevention of cybercrimes will obviously to promote awareness about criminal activities and tighten up security. (a) Would obviously been to promote awareness about criminal activities (b) Would obviously be to promote awareness about criminal activities (c) No substitution (d) Would obviously be to promote awareness for criminal activates SSC GD 27/01/2023 (Shift-I) Sentence Improvement

Ans. (b) : Option (b) will be used in the underlined part of the above sentence because the given sentence is in passive voice and preposition 'about' is used with awareness. Hence, option (b) will be correct. Correct SentenceThe first step towards the prevention of cybercrimes would obviously be to promote awareness about criminal activities and tighten up security. 534. Improve the underlined part of the sentence. Choose 'No improvement' as an answer if the sentence is grammatically correct. MLA is a representative electing by the voters of a constituency to the legislature. (a) elected by a (b) elected by the (c) elects by a (d) No improvement SSC MTS 04/05/2023 (Shift-II) Ans. (b) : The most appropriate improve the underlined part of the sentence is option (b) because 'electing by the' will be replaced by 'elected by the'. The sentence is in passive voice so, verb (is/am/are + v3) will be used. Thus, the correct sentence is'MLA is a representative elected by the voters of a constituency to the legislature. 535. Select the most appropriate option that can substitute the underlined segment in the given sentence. If there is no need to substitute it, select ‘No substitution required’. The authorities are assured the people that they will look into the matter. (a) No substitution required (b) have been assured (c) have assured (d) has assured SSC CGL (Tier-I) 11/04/2022 Shift-I Ans. (c) : In the above sentence, 'have assured' will be used in place of 'are assured' because sentence is in active voice. Hence have + V3 will be used. Correct sentence– The authorities have assured the people that they will look into the matter. 536. Select the most appropriate option to substitute the underlined segment in the given sentence. If no substitution is required, select 'No substitution required'. The guard had gave a hard push by the crowd. (a) was given (b) gives (c) had given (d) No substitution required SSC Stenographer (Grade C & D) 12.11.2021 Shift-II Ans. (a) : In the given sentence underlined part, 'was given' will be used in place of 'had gave' because sentence is in passive voice. Correct sentence–The guard had given a hard push by the crowd. 537. Select the most appropriate option to substitute the underlined segment in the given sentence. If there is no need to substitute it, select 'No substitution'. I am decided to buy a globe for your birthday. (a) have deciding (b) No substitution (c) am decide (d) have decided SSC Stenographer (Grade C & D) 15.11.2021 Shift-II

169

YCT

Ans. (d) : In the given sentence, 'have decided' will be used in place of underlined part 'am decided' because sentence is in active voice. Hence V3 'will' not be used with 'am'. A. v. → am + Ving A.v. → have + V3 Correct sentence I have decided to buy a globe for your birthday. 538. Select the most appropriate option that can substitute the underlined segment in the given sentence. If there is no need to substitute it, select ‘No substitution required’. I have been worked hard to finish this project. (a) No substitution required (b) working (c) had been worked (d) have been working SSC Constable GD-24/11/2021 Shift-II Ans. (d) : In the above underlined part, ‘have been working’ will be used in place of ‘have been worked’ because 'has/have + been + Ving' is used in active voice. Correct Sentence I have been working hard to finish this project. 539. Select the most appropriate option that can substitute the underlined segment in the given sentence. If there is no need to substitute it, select 'No substitution required'. She made the shopkeeper to take back the damaged headset. (a) taking back (b) take back (c) No substitution required (d) taken back SSC Constable GD-13/12/2021 Shift-III Ans. (b) : In the above underlined part, ‘take back’ will be used in place of ‘ to take back’. Because ‘to’ is not used, after causative verb ‘make’ in active voice. Note–‘To; is used after causative verb ‘cause’ and ‘to’ is used after causative verb ‘make’ in passive voice. Correct sentence– She made the shopkeeper take back the damaged headset. 540. Select the option that will improve the underlined part of the given sentence. In case no improvement is needed, select 'No improvement required'. The children were been drove to the picnic spot in the school bus. (a) No improvement required (b) were being driven (c) were been driven (d) were been driving SSC Constable GD-30/11/2021 Shift-III Ans. (b) : ‘Were being driven’ will be used in place of the underlined part of the above sentence ‘were been driven’ because the ‘sentence’ is in passive and the following structure is used in it. was/were + being + verb3 Correct SentenceThe Children were being driven to the picnic spot in the school bus. Sentence Improvement

541. Select the option that will improve the underlined part of the given sentence. In case no improvement is needed, select 'No improvement required'. My teacher said me that I would be famous one day. (a) asked me (b) say to me (c) told me (d) No improvement required SSC Constable GD-16/11/2021 Shift-III Ans. (c) : ‘Told me’ will be used in place of the underlined part of the above sentence ‘ said me’. Because the above sentence is in Indirect speech, so ‘told’ will be used in place of reporting verb ‘said.? Correct Sentece My teacher told me that I would be famous one day. 542. Select the most appropriate option to substitute the underlined segment in the given sentence. If no substitution is required, select 'No substitution required.' The committee members was took a unanimous decision that they would invest in startups. (a) took (b) taken (c) No substitution required (d) takes SSC Stenographer (Grade C & D) 15.11.2021 Shift-I Ans. (a) : In the above sentence, 'took' will be used in place of underlined part 'was took' because sentence is in active voice. Hence 'V2' is used. Correct SentenceThe committee member took a unanimous decision that they would invest in startups. 543. Select the option that will improve the underlined part of the given sentence. In case no improvement is needed, select 'No improvement required'. Has her family being informed about the event? (a) been informed (b) be inform (c) being informing (d) No improvement required SSC Constable GD-10/12/2021 Shift-III Ans. (a) : ‘Been informed’ will be used in place of the underlined part of the above sentence ‘being informed’ because the above sentence is passive form of present perfect tense. Correct Sentence– Has her family been informed about the event? 544. Select the most appropriate option to improve the underlined segment in the given sentence. IF there is no need to improve it, select 'No improvement'. He was considered a genius by his school headmaster. (a) was considering (b) No improvement (c) was to be considered (d) was been considered SSC CGL (Tier-II) 16/11/2020 (3 pm - 5 pm) Ans. (b) :There is no need to improve sentence.

170

YCT

545.

Improve the bracketed part of the sentence. (It is been seen) as a "prestige battle" for the PM and a litmus test for the president of the opposition party. (a) It have been seen (b) It is being seen (c) It had being seen (d) No improvement SSC CGL (Phase-II) 17/02/2018 Ans. (b) ‘It is being seen’ will be used in place of the words given in brackets (It is been seen) because the above sentence is passive voice of present continuous tense. Whose structure is as follows. Subject +is/am/are + being + verb3 + other Correct Sentence It is being seen as a "Prestige battle" for the PM and a litmus test per the president of the opposition party. 546. Select the most appropriate option to substitute the underlined segment in the given sentence. If there is no need to substitute it, select ‘No improvement’. Rohit Sharma have declared the best batsman of 2019. (a) had declared (b) has been declared (c) No improvement (d) is declaring SSC CHSL (Tier-I) 16/10/2020 (Shift-II) Ans. (b) : ‘Has been declared’ will be used in place of the underlined part of the above sentence ‘have declared'. Because the above sentence is based on the structure of passive form. Hence passive form 'has been' will be used in place of 'have'. Correct Sentence– Rohit Sharma has been declared the best batsman of 2019. 547. Select the most appropriate option to substitute the underlined segment in the given sentence. If there is no need to substitute it, select No improvement. It has been announcing that the bus will be late by an hour. (a) No improvement (b) It is announcing (c) It has announced (d) It has been announced SSC MTS – 21/08/2019 (Shift-II) Ans. (d) : The underlined part of the above sentence, ‘it has been announced’ will be used in place of ‘it has been announcing’. Because the sentence is in formed passive voice, So ‘it has been announced’ would be appropriate. Correct Sentence– It has been announced that the bus will be late by an hour. 548. Improve the bracketed part of the sentence. Prohibitory orders (will not) enforced, with the government evidently viewing the looming protests as pressure relief valves instead of as trigger points for violence. (a) were not (b) willingly (c) were be (d) no improvement SSC MTS 10-10-2017 (Shift-I) Ans : (a) For the above bracketed part, ‘were not’ will be used because the given sentence is in passive voice. Hence 'was/were + V3' will be used. Sentence Improvement

Correct sentence– Prohibitory orders were not enforced, with the government evidently viewing the looming protests as pressure relief valves instead of as trigger points for violence. 549. Select the most appropriate option to substitute te underlined segment in the given sentence. If there is no need to substitute it, select No Improvement. In his coming film the Spider-Man will be seen in European cities. (a) No Improvement (b) had been seen (c) has seen (d) was seen SSC MTS – 06/08/2019 (Shift-III) Ans. (a) : No Improvement Note-The above sentence is in passive voice. Hence ‘ will + be + V3’ will be used. 550. Select the most appropriate option to substitute the underlined segment in the given sentence. If there is no need to substitute it, select No improvement. Every book is wrote by Ruskin Bond is a masterpiece. (a) written (b) has wrote (c) No improvement (d) have written SSC MTS – 22/08/2019 (Shift-III) Ans. (a) : For the above underlined part, ‘written’ will be used because the given sentence is in passive voice. Hence 'is + V3' will be used. Correct sentence will be [Every book written by Ruskin Bond is a masterpiece.] 551. Select the most appropriate option to substitute the underlined segment. If there is no need to substitute it, select No improvement. I would have definitely helped you if you had told me why you did need the money. (a) why did you need the money (b) why you needed the money (c) why you needs the money (d) No improvement. SSC MTS – 22/08/2019 (Shift-II) Ans. (b) : for the above underlined part, ‘needed’ will be used because assertive sentence is used after ‘WH’ word in indirect. Correct sentence– I would have definitely helped you if you had told me why you needed the money. 552. Select the most appropriate option to substitute the underlined segment. If there is no need to substitute it, select No improvement. I courteously asked her where was she going but she did not reply. (a) she was going (b) was she gone (c) No improvement (d) will she go SSC MTS – 19/08/2019 (Shift-II) Ans. (a) : ‘she was going’ will be used in place of the underlined segment of the above sentence ‘was the going. Note :- In Indirect speech after conjunction (wh-word) 'Assertive sentence' is used in place of Interrogative sentence. Correct Sentence– I courteously asked her where she was going but she did not reply.

171

YCT

553.

Improve the bracketed part of the sentence. Internet providers would not block content because it would (not to be make economic sense) and consumers would not stand for it. (a) not be making economical sense (b) not be make economical sense (c) not make economic sense (d) No improvement SSC CGL (Phase-II) 17/02/2018 Ans. (c) For the above bracketed part, ‘to be’ will not be used because sentence is in active voice. Hence would + not + V1 will be appropriate. Correct Sentence Internet providers would not block content because it would not make economic sense and consumers would not stand for it. 554. Select the most appropriate option to substitute the underlined segment in the given sentence. If there is no need to substitute it, select no substitution required The banquet hall is been renovate recently. (a) is being renovated (b) has been renovated (c) no substitution required (d) are being renovate SSC MTS – 14/08/2019 (Shift-III) Ans. (b) : ‘Has been renovated’ will be used in place of the underlined segment of the above sentence ‘is been renovate’, because sentence is based on the structure of passive voice. As a rule if an action has been completed recently, then the verb will be in present perfect. Correct Sentence– The banquet hall has been renovated recently. 555. Select the most appropriate option to substitute the underlined segment in the given sentence. If there is no need to substitute it, select No improvement. On inspection a hundred year old bridge was being find too weak to take the load of lakhs of vehicles every day. (a) was being found (b) No improvement (c) was found to be (d) has to be found SSC GD – 14/02/2019 (Shift-II) Ans. (a) : ‘Was being found’ will be used in place of the underlined part of the above sentence 'was being find' because this sentence is expressing the sense of passive voice and ‘the use of V3rd form ‘ is used after being. Correct Sentence– On inspection a hundred year old bridge was being found too weak to take the load of lakhs of vehicles every day.

J.

Noun (Case, Gender, Number)

556. Select the most appropriate option to substitute the bracketed segment in the given sentence. If there is no need to substitute it, select 'no substitution required'. The Yoruba was (one of the west African citystate culture). Sentence Improvement

No substitution required one of a west African city-state cultures one of the west's African city-state culture one of the west African city-state cultures SSC MTS 15/05/2023 (Shift-I) Ans. (d) : In the bracketed segment of the above given sentence cultures' (plural) is used in the place of 'culture' (Singular). Note– One of the + Plural Noun + Singular Verb. Hence, the correct option is (d). 557. Select the most appropriate option that can substitute the underlined segment in the given sentence. If there is no need to substitute it, select 'No substitution'. Some passengers misplaced their luggages before boarding the train. (a) No substitution (b) misplaced his luggages (c) misplaces their luggages (d) misplaced their luggage SSC GD 01/02/2023 (Shift-II) Ans. (d) : The underlined segment of the above sentence, 'misplaced their luggage' would be appropriate in place of 'misplace their luggages'. Hence option (d) is correct answer. 558. Select the most appropriate option that can substitute the underlined words in the given sentence. She has gone to the stationery store to buy a scissor. (a) buy a pair of scissor (b) buy a scissors (c) buying a pair of scissors (d) buy a pair of scissors SSC CHSL (Tier-I) 09/08/2023 (Shift-III) Ans. (d) : underlined words of the sentence by a scissor will be substituted by option (d) 'buy a pair of scissors.' In English 'A pair of scissors Can be simply called 'scissors' as it refers to one object that consists of two blades. Nouns like Scissors, Spectacles, trousers are plural in nature. To make such nouns singular, phrases like 'a pair of' is used – such singular nouns after adding 'a pair of' are – Ex – a pair of jeans, a pair of pants, a pair of eyeglasses Correct Sentence She has gone to the stationary store to buy a pair of scissors. 559. Improve the underlined part of the sentence. Choose ‘No improvement’ as an answer if the sentence is grammatically correct. I didn't have times to watch the whole movie. (a) time to watch (b) timer to watched (c) time to watching (d) No improvement SSC CHSL (Tier-I) 21/03/2023 (Shift-IV) Ans. (a) : For the above underlined part of the sentence 'time to watch' will be used in place of 'times to watch' because 'time' is uncountable noun. Hence s/es not used after it. Correct Sentence : I didn't have time to watch the whole movie.

172

(a) (b) (c) (d)

YCT

560. Improve the underlined part of the sentence. Choose 'No improvement' as an answer if the sentence is grammatically correct. None of the witnessed could identify the suspect. (a) No improvement (b) witness had identify (c) witnessed could identified (d) witnesses could identify SSC CHSL (Tier-I) 17/03/2023 (Shift-IV) Ans. (d) : In the above sentence, 'witnesses could identify' will be used in underlined part, because plural noun form (witnesses) is needed here after 'none of' the. 561. Select the word segment that substitutes (replaces) the bracketed word segment correctly and completes the sentence meaningfully. In case no improvement is needed, select 'No improvement'. To be ready for a conflict (is one of the most effectual mean of) preserving harmony. (a) No improvement (b) is one of most effectual means of (c) is one of the most effectual means of (d) is one of most effectual mean with SSC CHSL (Tier-I) 15/03/2023 (Shift-III) Ans. (c) : In the above bracketed Part of the sentence 'means' will be used in place of 'mean' because 'Plural noun' is used after 'one of the'. Note- One of the + Plural noun + singular verb Correct sentenceTo be ready for a conflict is one of the most effectual means of preserving harmony. 562. Select the word segment that substitutes (replaces) the bracketed word segment correctly and completes the sentence meaningfully. In case no improvement is needed, select 'No improvement'. (That's one of the strongest swear word) in Spanish. (a) That is of the strongest swear words (b) That's the strongest swear words (c) That's one of the strongest swear words (d) No improvement SSC CHSL (Tier-I) 15/03/2023 (Shift-I) Ans. (c) : Bracketed Part of the sentence 'words' will be used in place of 'word' because plural noun is used after 'one of the'. Note- One of the + Plural noun + singular verb Correct SentenceThat's one of the strongest swear words in Spanish. 563. Improve the underlined part of the sentence. Choose 'No improvement' as an answer if the sentence is grammatically correct. She cuts her haired short in the summer. (a) hair short (b) hairy short (c) No improvement (d) hair shortest SSC CHSL (Tier-I) 09/02/2023 (Shift-II) Ans. (a) : For the above sentence, 'Hair' will be used in place of underlined part "Haired" because Noun (Hair) will be used after Possessive Adjective (her) whereas 'Haired' is used as Adjective. Sentence Improvement

564. Select the most appropriate option to substitute the underlined segment in the given sentence. If there is no need to substitute it, select 'No substitution'. Many of the food required for the moving migrant workers is arranged by the state government. (a) Much of food (b) Any of the food (c) Most of the food (d) No substitution SSC Stenographer (Grade C & D) 11.11.2021 Shift-I Ans. (c) : ‘Most of the food’ will be used in place of the underlined part of the above sentence 'many of the food' because ‘food’ is an uncountable noun. Hence 'Most' will be used. Correct SentenceMost of the food required for the moving migrant workers is arranged by the state government. 565. Select the option that will improve the underlined part of the given sentence. In case no improvement is needed, select 'No inprovement required'. My company beliefs that no one should up late for work. (a) believing (b) believes (c) No improvement required (d) belief SSC MTS-06/07/2022 Shift-III Ans. (b) : ‘believes’ (Verb) will be used in place of the underlined part of the above sentence ‘beliefs’ (Noun) because ‘verb’ is used after subject ‘My company’, not noun. Correct Sentence – My company believes that no one should turn up late for work. 566. Select the alternative that will improve the underlined part of the sentence in case there is no improvement select “No improvement”. In the major cities cost of life is very high. (a) cost of life are (b) the cost of living are (c) No improvement (d) the cost of living is SSC CPO-SI (Tier-II) 27/09/2019 (3 pm - 5 pm) Ans. (d) : ‘The cost of living is’ will be used in place of the above sentence ‘the cost of life is’, because in the sense of livelihood, ‘living’ will be used in place of ‘life’. And ‘the’ will be used before cost. Because two nouns joined by ‘of’. Then ‘the’ is used before the first noun. Correct Sentence– In the major cities the cost of living is very high. 567. Improve the bracketed part of the sentence. The Centre would reconsider its draft and limit its scope to just providing relief to women, (instead of creating a new regulative). (a) instead of creating a new regulatory (b) instead of making a new regulator (c) instead of creating a new regulation (d) No improvement SSC CGL (Phase-II) 17/02/2018

173

YCT

Ans. (c) In the above sentence, Regulative (adj) and new (adj) both have come together which is wrong with the rules of sentence structure. Noun is always used after New (adj). Therefore, regulation (noun) will be used in place of ‘regulative’. Correct Sentence– The Centre would reconsider its draft and limit its scope to just providing relief to women, instead of creating a new regulation. 568. Improve the bracketed part of the sentence. "An interview after the campaign ends (is the normal) for every candidate and every campaigner in every election," he said. (a) is the norm (b) is the norm ally (c) is continuality (d) No improvement SSC CGL (Phase-II) 17/02/2018 Ans. (a) According to the sense of the above sentence, ‘Norm’ will be used in place of in bracket part 'normal'. Which give the suitable meaning for the sentence. Correct Sentence– “An interview after the campaign ends is the norm for every candidate and every campaigner in every election”, he said. 569. Improve the bracketed part of the sentence. The tribunal noted that the driver of the (offend) truck did not lead any evidence in rebuttal of the claims. (a) offencing (b) offending (c) offence (d) No improvement SSC CGL (Phase-II) 17/02/2018 Ans. (b) ‘offending’ will be used in place of in bracket part of the above sentence ‘offend’. Because ‘participle’ is used as an adjective. Because the used of noun ‘truck’ is correct after it. Correct Sentence The tribunal noted that the driver of the offending truck did not lead any evidence in rebuttal of the claims. 570. Select the alternative that will improve the underlined part of the sentence. In case there is no improvement select 'No improvement'. I had no informations about the bank strike so I went there yesterday. (a) not got informations about (b) no informations concerning (c) no information about (d) No improvement SSC CHSL (Tier-I) 17/03/2020 (Shift-I) Ans. (c) ‘No information about’ would be appropriate in place of the underlined part of the above sentence, because the plural form of ‘Information, scenery, Luggage is not used. Hence ‘s/es’ is not used at the end of it. Correct Sentence– I had no information about the bank strike so I went there yesterday. 571. Select the option that will improve the underlined part of the given sentence. In case no improvement is needed, select 'No improvement required'. He did much mistakes. Sentence Improvement

committed many mistakes make many mistakes No improvement required did many a mistakes SSC Constable GD-07/12/2021 Shift-II Ans. (a) : In the given sentence, ‘committed many mistakes’ will be used in place of did much mistakes because ‘commit’ is used with mistake and ‘many’ will be used in place of ‘much’ because ‘mistakes’ is used as a countable noun. Correct sentence – He committed many mistakes. 572. Select the most appropriate option to substitute the underlined segment in the given sentence. If there is no need to substitute it, select 'No improvement', The Delhi Birla International School alumnus occupy leading positions in different areas of profession and service. (a) alummus occupies (b) alumni occupy (c) No improvement (d) alumnus occupying SSC CHSL (Tier-I) 21/10/2020 (Shift-II) Ans. (b) : In the above sentence, instead of ‘Alumnus occupy’, ‘Alumni occupy’ will be used. Because ‘alumni’ is a plural noun and the sentence is talking about ‘plural noun’. Correct Sentence– The Delhi Birla International School alumni occupy leading positions in different areas of profession and service. 573. Select the most appropriate option to substitute the underlined segment in the given sentence. If there is no need to substitute it, select 'No improvement'. India has lost a veteran actor with the deceased of Shriram Lagoo. (a) No improvement (b) bereavement (c) expired (d) demise SSC CHSL (Tier-I) 12/10/2020 (Shift-II) Ans. (d) : The use of demise (the end or failure of something) would be appropriate in the underlined part of the above sentence. Here 'demise' will be used in place of ‘deceased’, because ‘noun’ will come after article ‘the’ and not adjective. The meaning of other optionsBereavement :- the state of having lost a relative or close friend because they have died. Expired:- No longer valid because the period of time for which it could be used. Has ended. Correct Sentence India has lost a veteran actor with the demise of Shriram Lagoo. 574. Select the most appropriate option to substitute the underlined segment in the given sentence. if there is no need to substitute is, select No improvement. Tons of sheep were grazing in the field. (a) Numerous sheeps (b) A flock sheep (c) No improvement (d) Many sheeps SSC CHSL (Tier-I) 20/10/2020 (Shift-III)

174

(a) (b) (c) (d)

YCT

Ans. (b) : In the given sentence the underlined part of ‘A flock of sheep’ will be used in place of ‘Tons of sheep’ because the word ‘Flock’ is used for the group of animals. Correct Sentence– A flock of sheep were grazing in the field. 575. Select the most appropriate option to substitute the underlined segment in the given sentence. If there is no need to substitute it, select No improvement. The student's knowing of the subject astonished the teacher. (a) students knowing (b) student's knowledge (c) students knowledge (d) No improvement SSC MTS – 20/08/2019 (Shift-III) Ans. (b) : ‘Knowledge’ is used in place of the underlined part of the above sentence ‘knowing’. Because ‘noun’ is used after Apostrophe(‘s). Hence the use of knowledge is appropriate. Correct Sentence– ''The student's knowledge of the subject astonished the teacher.'' 576. Select the most appropriate option to substitute the underlined segment in the given sentence. If there is no need to substitute it, select No improvement. In case of actual requirement, there is a dearth of woman army officer in our state. (a) women army officer (b) woman army officers (c) no improvement (d) women army officers SSC GD – 02/03/2019 (Shift-II) Ans. (d) : ‘Women army officers’ will be used in place of ‘woman army officer’ because here is a need for a plural noun. Note :- To make plural of women officer both the words are made plural. e.g. women officers. Correct SentenceIn case of actual requirement, there is a dearth of women army officers in our state.

K.

Miscellaneous

577. Select the word segment that substitutes (replaces) the bracketed word segment correctly and completes the sentence meaningfully. Select the option 'no correction required' if the sentence is correct as given. He (returned back) Cambridge shortly before three o'clock. (a) appeared (b) showed (c) No correction required (d) reached SSC MTS 10/05/2023 (Shift-I) Ans. (d) : The appropriate answer for the bracketed word is 'reached' (V2). Because the sentence is in past indefinite tense. The correct answer is– 'He reached combridge shortly before three o' clock'. Thus, the correct answer is option (d). Sentence Improvement

578. Improve the underlined part of the sentence. Choose 'No improvement' as an answer if the sentence is grammatically correct. The most worrying aspect of climate change is the effect it has on the world's food supply. (a) No improvement (b) A most worry aspect (c) A worried aspects (d) The most worrying aspect SSC MTS 08/05/2023 (Shift-III) Ans. (a) : For the above given, underlined part of sentence will not be Improve because it is correct sentence. Thus, the correct answer is option (a) 579. Improve the underlined part of the sentence. Choose 'No improvement' as an answer if the sentence is grammatically correct. It is the most effective means of mass communication. (a) the most effectively (b) a most effectives (c) a most effecting (d) No improvement SSC MTS 02/05/2023 (Shift-I) Ans. (d) : There is no improvement for the underlined part of the sentence because it is appropriate already. 'It is the most effective means of mass communication'. Thus, the correct answer is option (d). 580. Improve the underlined part of the sentence. Choose 'No improvement' as an answer if the sentence is grammatically correct. Drive two blocks and turn right at the traffic lights. (a) turn right in (b) No improvement (c) turns right at (d) turn rightly at SSC MTS 19/05/2023 (Shift-III) Ans. (b) : No improvement. The underlined part of the above sentence ‘turn right at’ is grammatically correct. The correct sentence is – ‘Drive two blocks and turn right at the traffic lights’. Hence, the correct answer is option (b). 581. Improve the underlined part of the sentence. Choose ‘No improvement’ as an answer if the sentence is grammatically correct. Cell phones keeps loved ones and those nearly them well connected. (a) keep loved ones and those near (b) keeping loved and nearly (c) kept love and near (d) No improvement SSC MTS 18/05/2023 (Shift-I) Ans. (a) : The underlined part of the above given sentence – ‘Keeps loved ones and those nearly’ is grammatically incorrect because the given sentence is in ‘Simple Present tense’ therefore, ‘Verb1’ is used with plural subject (Cell phones), and ‘near’ is used in the place of ‘nearly’. The correct sentence – Cell phones keep loved ones and those near them well connected.

175

YCT

582. Select the option that can be used as a oneword substitute for the underlined phrase in the following sentence. She was known for her ability to communicate effectively and express herself clearly in writing and speaking. (a) articulation (b) mumbling (c) incoherence (d) rambling SSC CGL Mains 26/10/2023 (Shift-I) Ans. (a) : For the above underlined group of word 'ability to communicate effectively and express herself clearly in writing and speaking is option (a) Articulation gives appropriate meaning. Other options give different meaning. Mumbling Grumble Incoherence Irrational Rambling Garrulous 583. Improve the underlined part of the sentence. Choose ‘No improvement’ as an answer if the sentence is grammatically correct. The Tagore House is rehearsing a skit for the school’s Annual Day Function. (a) was rehearse a skit (b) No improvement (c) is rehearsed an skit (d) has been rehearsing a skits SSC MTS 15/05/2023 (Shift-I) Ans. (b) : No improvement. The underline part of the above given sentence 'is rehearsing a skit' is grammatically correct because the sentence is in present continuous tense, the verb is used according to the (sub.). Thus, the correct answer is option (b). 584. Select the most appropriate option to substitute the bracketed segment in the given sentence. If there is no need to substitute it, select 'no substitution required'. The query isn't (whether he will admit it, but when). (a) whether he will admit it or not, but when? (b) whether he will be admitting it or not, but when? (c) No substitution required (d) whether or nor he will admit it, but when? SSC MTS 12/05/2023 (Shift-III) Ans. (c) : No substitution required. 585. Select the most appropriate option that can substitute the underlined segment in the given sentence. They took a while to establish the threat. (a) Recognise the threat (b) manifest the threat (c) Negotiate the threat (d) Move the threat SSC GD 31/01/2023 (Shift-II) Ans. (a) : "Recognize the threat" will be used in the underlined part of the given sentence because threat is recognized not established. Other options give different meaning. Correct SentenceThey took a while to recognize the threat. Sentence Improvement

586. Select the most appropriate option that can substitute the underlined segment in the given sentence. Dogs are the most adorable and lovable beings to tie. (a) Beings to see (b) animals to play (c) organisms to watch (d) pets to keep SSC GD 23/01/2023 (Shift-IV) Ans. (d) : 'Pets to keep' will be used in the underlined part of the given sentence because it is giving proper meaning according to the sense of the sentence. Correct SentenceDogs are the most adorable and lovable pets to keep. 587. Select the most appropiate option to substitute the underlined segment in the given sentence. The bufotoxin present on a toad's skin can irritate you but touching a toad shall not cause you warts. (a) Was not cause (b) Do not cause (c) Has not given (d) Won't give SSC GD 16/01/2023 (Shift-III) Ans. (d) : Option (d) 'won't give' will be used in the underlined part of the given sentence because the future tense 'shall' isn't typically used in this type of conditional statement about a general truth. Hence option (d) will be used. Correct sentenceThe bufotoxin present on a toad's skin can irritate you but touching a toad won't give you warts. 588. Select the most appropriate option that can substitute the underlined segment in the given sentence. We should also learn to trust what he says. after all, he is a reformed person now. (a) To give someone the benefit of the doubt (b) To make matters worse (c) No pain, no gain (d) To wrap our head around something SSC GD 16/01/2023 (Shift-III) Ans. (a) : Option (a) 'To give someone the benefit of the doubt' will be used in the underlined part of the above sentence because in the sentence, the speaker suggest they should trust a person who has reformed, implying giving them chance despite doubts that may exist due to past behaviour. 'To give someone the benefit of the doubt' is an idiom. Correct SentenceWe should also learn to give someone the benefit of the doubt after all, he is a reformed person now. 589. Select the most appropriate option that can substitute the underlined segment in the given sentence. The way someone talks and behaves introduces the characteristics of a person. (a) Symbolises the gestures (b) Effects the aura (c) Defines the personality (d) Postulates the personality SSC GD 12/01/2023 (Shift-II)

176

YCT

Ans. (c) : Option (c) 'defines the personality' will be used in the underlined part of the given sentence because it is giving proper meaning according to the sense of the sentence. Correct sentenceThe way someone talks and behaves defines the personality of a person. 590. Select the most appropriate option to substitute the underlined segment in the given sentence. Purchasing furniture at a low cost could not involve abandoning design or quality. (a) should not entail sacrificing (b) did not provoke compromising (c) have not done regretting (d) cannot force forgetting SSC GD 10/01/2023 (Shift-I) Ans. (a) : In the underlined part of the above sentence. instead of 'could not involve abandoning', 'should not entail sacrificing' will be used. Because there is a sense of advice in the sentence. Hence the use of should will be appropriate. Correct sentencePurchasing furniture at a low cost should not entail sacrificing design or quality. 591. Select the most appropriate option that can substitute the underlined segment in the given sentence. Athira would like to get him a special birthday present, but nothing springs to mind. (a) from the beginning of time (b) dawns on (c) almost always (d) forget about something SSC Selection Posts XI-30/06/2023 (Shift-IV) Ans. (b) : In the above sentence, the most appropriate substitute of underlined part 'Springs to mind' is option (b) "dawns on" which means "to come quickly into your mind, gives appropriate meaning." Other options give different meaning. 592. Select the most appropriate option that can substitute the underlined segment in the given sentence. I regarded my brother on passing his graduation examination. (a) congratulated (b) disparaged (c) flattered (d) kowtowed SSC GD 10/01/2023 (Shift-I) Ans. (a) : The most appropriate option that can substitute the underlined segment 'regarded' is 'Congratulated' (Applaided). Meaning of the other optionsDisparaged - Demean Flattered - Cojoled Kowtowed - Solution, hallo Correct SentenceI Congratulated my brother on passing his graduation examination. Sentence Improvement

593. Select the most appropriate option that can substitute the underlined segment in the given sentence. To enable women to become literate and provide for themselves, education for girls must be done voluntary. (a) Made mandatory (b) Enabled extensively (c) Kept secret (d) done compulsory SSC GD 08/02/2023 (Shift-IV) Ans. (a) : 'Made mandatory' is the most appropriate option that can substitute the underlined segment in the given sentence. Hence option (a) 'Made mandatory' is correct answer. 594. Select the most appropriate option that can substitute the underlined segment in the given sentence. Land with unoccupied possession is pretending to be more valuable. (a) are likely to be (b) was similarly to be (c) is likely to be (d) have likely to be SSC GD 01/02/2023 (Shift-II) Ans. (c) : According to the sense of the above underlined segment, 'is likely to be' will be used in place of 'pretending to be'. The other options do not give the suitable meaning. Hence option (c) is correct answer. 595. Select the most appropriate option that can substitute the underlined segment in the given sentence. From a recent research, evident is it that a lot more women have been opting for professional courses. (a) it is evident that more a lot women (b) it is evident that a lot more women (c) it is evidence that a lot more women (d) it is evidently that a lot more women SSC CHSL (Tier-I) 17/08/2023 (Shift-II) Ans. (b) : The underlined segment 'evident is it that a lot more women' can be replaced by option (b) 'it is evident that a lot more women' . Because inversion of order of sentences is unnecessary have as sentence does not have interrogative nature. Correct Sentence From a recent research, It is evident that a lot more women have been opting for professional course. 596. Select the most appropriate option that can substitute the underlined segment in the following sentence. As the storm raged outside, the children huddled together under blankets, listening to the sound of the rain hitting the windows hard and hoping that the power wouldn’t go out. (a) climbing up to the windows (b) beating off the windows (c) setting off the windows (d) pounding against the windows SSC CHSL (Tier-I) 10/08/2023 (Shift-I)

177

YCT

Ans. (d) : The correct substitution of the underlined segment 'hitting the windows hard' is option (d) 'pounding against the windows.' Because "Pounding " means to hit something hard option (d) correctly replaces the underlined segment. Correct sentence As the storm raged outside, the children huddled together under blankets, listening to the sounds of the rain pounding against the windows and hoping that the power wouldn't go out. 597. Select the most appropriate option to substitute the underlined segment in the following sentence. With the harsh words of the boss, Srujan felt sad, but his colleague’s words created insult to injury. (a) attached insult to injury (b) provoked insult to injury (c) added insult to injury (d) added injury to insult SSC CHSL (Tier-I) 02/08/2023 (Shift-I) Ans. (c) : The most appropriate substitute of the underlined segment 'created insult to injury' is 'added insult to injury'. It is an idiom, which means 'to make a bad situation even worse'. Hence, the correct option is (c). 598. Select the most appropriate option that can substitute the underlined words in the given sentence. Students were asked to write the historical events in the order of their occurrence. (a) chronological order (b) historic order (c) aesthetic order (d) diachronic order SSC Selection Posts XI-30/06/2023 (Shift-IV) Ans. (a) : For the above sentence underlined group of word, 'the order of their occurrence' option (a) 'Chronological order' is appropriate. Other options give different meaning. 599. Improve the underlined part of the sentence. Choose 'No improvement' as an answer if the sentence is grammatically correct. Remember to close the windows before you leave. (a) you leave (b) No improvement (c) you leaves (d) you have leaving SSC CHSL (Tier-I) 17/03/2023 (Shift-IV) Ans. (b) : No improvement 600. Select the word segment that substitutes (replaces) the bracketed word segment correctly and complete the sentence meaningfully. Select the option 'no correction required' if the sentence is correct as given. Titanic (was sink) after midnight. (a) was drown (b) sunked (c) No correction required (d) drowned SSC CHSL (Tier-I) 17/03/2023 (Shift-IV) Sentence Improvement

Ans. (d) : In the above sentence, 'drowen' will be used in bracketed part. because sentence is in past tense. Correct sentenceTitanic drowned after midnight. 601. Improve the underlined part of the sentence. Choose 'No improvement' as an answer if the sentence is grammatically correct. I can hear the band which is playing patriotic tunes, in the playground. (a) No improvement (b) patriot tunes (c) patriotic tuning (d) patriotically tuned SSC CHSL (Tier-I) 17/03/2023 (Shift-III) Ans. (a) : No improvement 602. Improve the underlined part of the sentence. Choose 'No improvement' as an answer if the sentence is grammatically correct. The dog chased the ball into the neighbour's yard. (a) chase the ball (b) No improvement (c) chasing the ball (d) has chasing the ball SSC CHSL (Tier-I) 15/03/2023 (Shift-III) Ans. (b) : No improvement 603. Select the option that will improve the underlined part of the sentence. In case no improvement is needed, select 'No improvement'. Herbalism had become an all but extinct skill in the Western world. (a) No improvement (b) Herbalism has become but all an extinct skill (c) Herbalism become an extinct skill (d) Herbalism has become but an extinct skill SSC CHSL (Tier-I) 09/02/2023 (Shift-II) Ans. (a) : No improvement 604. Select the most appropriate option to improve the underlined segment in the given sentence. If there is no need to improve it, select 'no improvement required'. Learning a poem by heart is meaningless if you do not understand it. (a) To learning a poem at heart (b) no improvement required (c) Learning a poem in heart (d) Learn a poem by heart SSC CGL (Tier-II) 08.08.2022 Shift-II Ans. (b) : No. improvement required. Note : In sentence, the use of learning is used as present participle. 605. Select the most appropriate option to substitute the underlined segment in the given sentence. If there is no need to substitute it, select 'No substitution'. The situation in the city is now fully under control. (a) beside (b) No substitution (c) below (d) beneath SSC Stenographer (Grade C & D) 11.11.2021 Shift-II Ans. (b) : No correction required in the sentence. Hence option (b) is correct. Option (b) is correct

178

YCT

606. Select the most appropriate option to improve the underlined segment in thie given sentence. If there is no need to improve it, select 'No improvement required'. Sonali as well as her brother is performing at the show tonight. (a) were performing (b) are performing (c) have performance (d) No improvement required SSC Stenographer (Grade C & D) 11.11.2021 Shift-II Ans. (d) : In the given sentence , ‘Main subject’ is – Sonali: which is in singular Number whereas ‘As well as her brother’ is Adjective phrase. Hence according to Sonali. Singular verb is used and there is no correction in the sentence. Hence option (d) is correct. 607. Select the most appropriate option to substitute the underlined segment in the given sentence. If no substitution is required, select 'No substitution required'. Hears to the news tonight at 9 pm. as there will be an important announcement about the budget. (a) Hear to (b) Listened to (c) No substitution required (d) Listen to SSC CGL (Tier-I) –17/08/2021 (Shift-I) Ans. (d) : In the given sentence, ‘Listen to’ will be used in place of ‘Hears to’. Because we use hear for sounds that come to our ears, and listen is used to describe paying attention to sounds that are going on. Correct SentenceListen to the news tonight at 9pm. as there will be an important announcement about the budget. 608. Select the most appropriate option to substitute the underlined segment in the given sentence. If no substitution is required, select 'No substitution'. Impending natural disasters produce us constant worry. (a) instill (b) cause (c) No substitution (d) provoke SSC Stenographer (Grade C & D) 11.11.2021 Shift-I Ans. (b) : The underlined part of the above sentence, ‘cause’ will be used in place of ‘produce’. Because it gives appropriate explanation to the sentence. Correct SentenceImpending natural disasters cause us constant worry. 609. Select the most appropriate option to substitute the underlined segment in the given sentence. If no substitution is required, select 'No substitution required'. The crisis was adverted due to timely intervention by the government. (a) was been averted (b) be averted (c) No substitution required (d) is avert SSC Stenographer (Grade C & D) 12.11.2021 Shift-II Sentence Improvement

Ans. (c) : There is no grammatical error in this sentence. Hence no substitution is required, option (c) will be correct answer. 610. Select the most appropriate option to substitute the underline segment in the given sentence. The affection of her husband carried her up in the midst of all her problems. (a) stood her up (b) tore her up (c) drove her up (d) bore her up SSC CHSL 26.05.2022 Shift-I Ans. (d) : ‘Bore her up’ is appropriate in place of the underlined part of the above sentence ‘carried her up’. Because other option gives the different meaning. Correct sentence– The affection of her husband bore her up in the midst of all her problems. 611. Select the most appropriate option to substitute the underline segment in the given sentence. The Ukranian soldiers passed in for several weeks. (a) held out (b) hanged to (c) hanged in (d) get off SSC CHSL 26.05.2022 Shift-III Ans. (a) : The use of ‘held out’ instead of ‘Passed in’ in the underlined portion of the above sentence gives a suitable meaning to the sentence. Other options have different meaning. Correct sentence – The Ukranian soldiers held out for several weeks. 612. Select the most appropriate option that can substitute the underlined segment in the given sentence. I told Pooja that she could have use my car to get around while she is in my hometown. (a) that she can use my car to get away with (b) that she could have use my car to get away (c) that she could have use my car to get around with (d) that she can use my car to get around SSC CHSL 26.05.2022 Shift-III Ans. (d) : The underlined part of the above sentence, ‘can use’ will be used in place of ‘could have use’. Because the use of can gives appropriate meaning according to the sentence. Correct sentence– I told pooja that she can use my car to get around while she is in my hometown. 613. Select the most appropriate option that can substitute the underlined word in the given sentence. Due to his hard work, he established a succesful career. (a) successful (b) sucessfful (c) sucessful (d) sucessful SSC CHSL 26.05.2022 Shift-III Ans. (a) : ‘Successful’ will be used in the underlined part of the above sentence because successful is spelled correctly. The spelling given in the other options is incorrect. Correct sentence– Due to his hard work, he established a successful career.

179

YCT

614. Select the most appropriate option that can substitute the underlined segment in the given sentence. If there is no need to substitute it, select ‘No substitution required’. At the end of our first two weeks on the island, we were informed that our lawyers, Bram Fischer and Joel Joffe, were going to be visiting the following day. (a) we are informed (b) we were inform (c) No substitution required (d) we get inform SSC CHSL 25.05.2022 Shift-I Ans. (a) : There is no need to substitute it. 615. Select the option that will improve the underlined part of the given sentence. Life is hard, unpredictable, but though manageable. (a) though achievable (b) although bearable (c) despite practicable (d) still controllable SSC CHSL 27.05.2022 Shift-II Ans. (d) : The use of ‘still controllable’ is used in place of the underlined part of the above sentence ‘though manageable’. It gives a suitable meaning to the sentence. Other options have different meanings. Correct Sentence Life is hard, unpredictable, but still controllable. 616. Select the option that will improve the underlined part of the given sentence. I would like to thank you for your coming out tonight. (a) for your will come out (b) for comes out (c) for coming out (d) for you came out SSC CHSL 27.05.2022 Shift-I Ans. (c) : The use of ‘your’ is superfluous in the underlined part of the above sentence. Because ‘you’ has used as an object and gives complete meaning for the sentence. Correct SentenceI would like to thank you for coming out tonight. 617. Select the option that will improve the underlined part of the given sentence. In this scene, a close-up shot was taken by the cameraman for display the emotions. (a) in order of emotions (b) in order to display the emotions (c) for show of emotions (d) to show emotions SSC CHSL 27.05.2022 Shift-I Ans. (b) : ‘In order to display the emotions’ will be used in place of the underlined segment of the above sentence ‘for display the emotions’. Because the use of ‘in order to’ gives the proper meaning for the sentence. Correct Sentence In this scene, a close-up shot was taken by the cameraman in order to display the emotions. Sentence Improvement

618. Select the most appropriate option to substitute the underline segment in the given sentence. I called of an explanation for his rude behaviour. (a) called in (b) called into (c) called out (d) called SSC CHSL 27.05.2022 Shift-I Ans. (c) : The underlined segment of the above sentence, called out (to say something in a loud voice) will be used in place of called of. Because it gives suitable meaning for this sentence. Other options have different meaning. Correct SentenceI called out an explanation for his rude behaviour. 619. Select the option that will improve the underlined part of the given sentence. Reema and Aasha were frolicking in the park (a) were performing in the garden (b) were participating in the garden (c) were enjoying in the garden (d) were playing in the garden SSC CHSL 27.05.2022 Shift-III Ans. (d) : ‘Were playing in the garden’ will be used in place of the underlined part of the above sentence ‘Were frolicking in the park. Garden :- An area next to one’s house where you can grow flower’s fruit, vegetables etc. usually with a later. Park :- An area where people go to walk and relax. Note:- The sense of the above sentence is already suitable. But the phrase given in option (d) can also be used in the sentence. 620. Select the most appropriate option to substitute the underlined segment in the given sentence. I was carried by her rapturous song. (a) carried of (b) carried away (c) carried off (d) carried to SSC CHSL 26.05.2022 Shift-II Ans. (b) : The use of ‘carried away’ is appropriate in place of the underlined part of the above sentence, carried’. It gives the proper meaning for the sentence. Other options have different meaning. Correct sentence– I was carried away by her raptorous song. 621. Select the option that will improve the underlined part of the given sentence. In case no improvement is needed, select 'No improvement required'. Her forty-fourth birthday had just passed, unnoticed by all but herself. (a) had just passed, unnoticed by but all herself (b) has just passed, unnoticed by all herself (c) has just passed, unnoticed by but all herself (d) No improvement required SSC MTS-18/07/2022 Shift-I Ans. (d) : There is no improvement required. 622. Select the most appropriate option that can substitute the underlined segment in the given sentence. If there is no need to substitute it, select 'No substitution required'.

180

YCT

I can't make out my mind about taking a vacation in winter. (a) made out (b) make up (c) No substitution required (d) make on SSC CGL (Tier-I) 19/04/2022 Shift-III Ans. (b) : In the underlined part of the given sentence, ‘make up’ will be used in place of ‘make at’. Make out means (to see, hear or understand something or someone with difficulty.) While Make up :- to invent something, such as an excuse or a story, often in order to deceive. I can’t make up my mind. 623. Select the most appropriate option to substitute the underlined segment in the given sentence. If there is no need to substitute it, select 'No substitution required'. It is quite difficult to make on what this doctor writes. (a) made on (b) make out (c) make in (d) No substitution required SSC CGL (Tier-I) 20/04/2022 Shift-III Ans. (b) : In the given sentence, ‘make out’ (understand). Will be used in place of ‘make on’ (continue). According to the sense of the sentence. Correct Sentence– It is difficult to make out what this doctor writes. 624. Select the most appropriate option that can substitute the underlined segment in the given sentence. If there is no need to substitute it, select 'No substitution required'. The dress may pleasantly be delivered to us at the earliest. (a) may kindly (b) must pleasantly (c) might kindly (d) No substitution required SSC CGL (Tier-I) 19/04/2022 Shift-II Ans. (a) : Kindly mean- In a kind way. Hence according to the sense of sentence, ‘may kindly’ will be used in place of ‘may pleasantly’. Pleasantly means- in a way that is fun attractive or gives pleasure. 625. Select the most appropriate option that can substitute the underlined segment in the given sentence. If there is no need to substitute it, select ‘No substitution required’. In spite of my niece's dog Casper is very mischievous, he is lovable. (a) Nevertheless my niece (b) However my nieces (c) No substitution required (d) Although my niece's SSC CGL (Tier-I) 18/04/2022 Shift-I Ans. (d) : In given underline part, sentence tries to convey the meaning that the narrator's niece's dog casper is very mischievous, yet he is lovable. In this sentence, ‘although’ will be used in place of ‘in spite of’ Hence option (d) is correct answer. Sentence Improvement

626. Select the most appropriate option to substitute the underlined segment in the given sentence. If there is no need to substitute it, select ‘No substitution required’. Of the two plans submitted by the architect, this is the one more likely to be accepted. (a) most likely (b) most likelihood (c) No substitution required (d) much likely SSC CGL (Tier-I) 11/04/2022 Shift-II Ans. (c) : There is no need to substitute it. 627. Select the option that will improve the underlined part of the given sentence. In case no improvement is needed, select 'No improvement required'. We might as well watch a film on TV as there’s nothing much to do. (a) No improvement required (b) may as well as (c) may as such (d) might well SSC CGL (Tier-I) 12/04/2022 Shift-III Ans. (a) : There is no need to substitute it. 628. Select the most appropriate option to substitute the underlined segment in the given sentence. If there is no need to substitute it, select ‘No substitution required.’ A tigress has given birth to a cub in the Ranthambore Tiger Reserve, taking the big cat population to 78. (a) was birthed (b) no substitution required (c) is given births (d) has give birth SSC CGL (Tier-I) 11/04/2022 Shift-I Ans. (b) : No substitution required. 629. Select the option that will improve the underlined part of the given sentence. In case no improvement is needed, select 'No improvement required'. Carrying an umbrella is not at all a good idea during thunderstorms. (a) Carried an umbrella (b) Carry an umbrella (c) No improvement required (d) Carrying a umbrella SSC MTS-15/07/2022 Shift-I Ans. (c) : No improvement required. 630. Select the option that will improve the underlined part of the given sentence. In case no improvement is needed, select 'No improvement required'. Shubham and I were watching football match in our bedroom. (a) I were watched (b) I was watching (c) me is watching (d) No improvement required SSC MTS-06/07/2022 Shift-I Ans. (d) : No improvement required.

181

YCT

631. Select the option that will improve the underlined part of the given sentence. In case no improvement is needed, select 'No improvement required'. I knew very little Spanish when I came here, but I have been learning and can now make myself understood. (a) when I come here, however (b) upon arriving her, since (c) No improvement required (d) as soon as I came here, but SSC MTS-05/07/2022 Shift-III Ans. (c) : No improvement required 632. Select the option that will improve the underlined part of the given sentence. In case no improvement is needed, select 'NO improvement required'. Please consult a doctor before you take any medicine. (a) consulting (b) consulted (c) No improvement required (d) consults SSC MTS-06/07/2022 Shift-II Ans. (c) : No improvement required. 633. Select the options that will improve the underlined part of the given sentence. In case no improvement is needed, select 'No improvement required'. The water at her place was cool and had a metallic taste. (a) was cooling and have (b) No improvement required (c) were cool and have (d) was coolest and has SSC MTS-05/07/2022 Shift-I Ans. (b) : No improvement required. 634. Select the options that will improve the underlined part of the given sentence. In case no improvement is needed, select 'No improvement required'. The eighteenth century saw the middle classes becoming most prosperous. (a) been more than (b) No improvement required (c) become more (d) became most SSC MTS-05/07/2022 Shift-II Ans. (c) : ‘Become more’ will be used in place of the above sentence ‘becoming most’ because generally, ‘more’ is used in the sense of showing excess. Correct Sentence– The eighteenth century saw the middle classes become more prosperous. 635. Select the option that will improve the underlined part f the given sentence. Incase no improvement is needed, select 'No improvement required'. I am sorry of which my mother said. Sentence Improvement

for what for which off that No improvement required SSC MTS-06/07/2022 Shift-III Ans. (a) : ‘For what (whatever) will be used in place of the underlined part of the above sentence ‘of which’. Because it gives appropriate meaning. According to the sense of the sentence. Other options are not suitable. Correct Sentence – I am sorry for what my mother said. 636. Select the option that will improve the underlined part of the given sentence. In case no improvement is needed, select 'No improvement required'. Why should we give up everything? (a) gives up (b) given off (c) gave at (d) No improvement required SSC MTS-06/07/2022 Shift-III Ans. (d) : No improvement required. 637. Select the option that will improve the underlined part of the given sentence. In case no improvement is needed, select 'No improvement required'. The courses provided by the training institute were more than enough. (a) more then enough (b) more enough (c) most enough (d) No improvement required SSC MTS-08/07/2022 Shift-I Ans. (d) : No improvement required. 638. Select the option that will improve the underlined part of the given sentence. In case no improvement is needed, select 'No improvement required'. Grammar is just a natural function of children's brains, and they apply it to whatever they find. (a) No improvement required (b) they applied them do (c) they apply them for (d) they apply that in SSC MTS-14/07/2022 Shift-III Ans. (a) : There is no need to substitute it. 639. Select the option that will improve the underlined part of the given sentence. In case no improvement is needed, select 'No improvement required'. He went to bed early to get up on time to attend his morning lecture. (a) get up over time (b) No improvement required (c) get up for time (d) get up at time SSC MTS-14/07/2022 Shift-III Ans. (b) : There is no need to substitute it.

182

(a) (b) (c) (d)

YCT

640. Select the most appropriate option that can substitute the underlined segment in the given sentence. If there is no need to substitute it, select ‘No substitution required’. Naveena won a scholarship to further studies in medicine and decided to go abroad. (a) for further study on medicine (b) in furthering studies with medicine (c) for further studies in medicine (d) No substitution required SSC Constable GD-29/11/2021 Shift-II Ans. (c) : ‘For further studies in medicine’ will be used in place of the underlined part of the above sentence ‘ to further studies in medicine’ because in the sentence ‘for’ will be used in the sense of ‘further styudeis? Correct Sentence Naveena won a scholarship for further studies in medicine and decided to go abroad. 641. Select the option that will improve the underlined part of the given sentence. In case no improvement is needed, select 'No improvement required'. The strong wind almost lifted me off my feet. (a) above my feet (b) No improvement required (c) over my feet (d) against my feet SSC Constable GD-26/11/2021 Shift-I Ans. (b) : No improvement required. 642. Select the most appropriate option that can substitute the underlined segment in the given sentence. If there is no need to substitute it, select ‘No substitution required’. Skydiving was a rare opportunity that I had got, so, even though I was exhausted, I decided to go for it. (a) No substitution required (b) so despite however (c) as yet though (d) still even if SSC Constable GD-25/11/2021 Shift-III Ans. (a) : No substitution required 643. Select the option that will improve the underlined part of the sentence. In case no improvement is needed, select 'No improvement required'. The decision to become a doctor is completely hers. (a) No improvement required (b) is completely her (c) is complete her's (d) are completely her SSC Constable GD-17/11/2021 Shift-II Ans. (a) : No improvement required. 644. Select the most appropriate option that can substitute the underlined word in thegiven sentence. In this modern world, everyone is associated in making easy money. Sentence Improvement

(a) bothered (b) interested (c) accounted (d) disassociated SSC Constable GD-14/12/2021 Shift-III Ans. (b) : ‘Interested’ will be used instead of the underlined part of the above sentence ‘associated’ because It gives suitable meaning, according to the sense of sentence. Correct sentence–In this modern world, everyone is interested in making easy money. 645. Select the most appropriate option that can substitute the underlined segment in the given sentence. If there is no need to substitute it, select 'No substitution required'. The ostrich is an interesting bird which has wings but cannot fly. (a) which has got wing but cannot fly (b) which have wings but cannot fly (c) which is having wings but cannot fly (d) No substitution required SSC Constable GD-14/12/2021 Shift-II Ans. (d) : No substitution required. 646. Select the most appropriate option that can substitute the underlined segment in the given sentence. If there is on need to substitute it, select 'No substitution required'. The actor along with his new wife is expected shortly. (a) No substitution required (b) are being (c) are expected (d) has expected SSC Constable GD-09/12/2021 Shift-II Ans. (a) : No substitution required 647. Select the most appropriate option that can substitute the underlined word in the given sentence. 'Angulimala' lived by training travellers and feared no one. (a) disillusioning (b) distilling (c) plundering (d) mischallenging SSC Constable GD-09/12/2021 Shift-II Ans. (c) : ‘Plundering’ (loot) will be used instead of the underlined part of the above sentence ‘training’. because it gives appropriate meaning according to sense of the sentence. Disillusioning :- to destroy somebody’s belief in or good opinion of somebody / something. Distilling :- to let fall, exude, to purify. Mischallenging :- bad challenge. Correct sentence – 'Angulimala' lived by plundering travellers and feared no one. 648. Select the option that will improve the underlined part of the given sentence. In case no improvement is needed, select 'No improvement required'. The sun had set by the time we reached the hilltop.

183

YCT

(a) (b) (c) (d)

we reach we were reaching we had reached No improvement required SSC Constable GD-08/12/2021 Shift-I Ans. (d) : No improvement required. 649. Select the most appropriate option that can substitute the underlined segment in the given sentence. If there is no need to substitute it, select 'No substitution required'. Many of the apples were rotten. (a) was rotten (b) No substitution required (c) is been rotten (d) is rotten SSC Constable GD-07/12/2021 Shift-II Ans. (b) : No substitution requred. 650. Select the most appropriate option to substitute the underlined segment in the given sentence. If there is no need to substitute it, select 'no substitution'. I need someone very strong to lift this heavy bag. (a) No substitution (b) Need (c) Am need (d) Am in need SSC Sel. Post Phase-VIII (H.L.) 09.11.2020 (Shift-I)

Ans. (a) : No substitution. 651. Select the most appropriate option that can substitute the underlined word in the given sentence. At the time I was driving near Delhi, I perfectly felt the need to stretch my legs. (a) effortlessly (b) suddenly (c) normally (d) partially SSC Constable GD-06/12/2021 Shift-III Ans. (b) : ‘Suddenly’ (quickly and unexpected) will be used instead of the underlined part of the above sentence ‘perfectly’, because it gives proper meaning, according to the sense of the sentence. Correct sentence– At the time I was driving near Delhi, I suddenly felt the need to stretch my legs. 652. Select the most appropriate option that can substitute the underlined word in the given sentence. As we all are aware, silk is dedicated from the cocoons of mulberry silkworms. (a) derived (b) decorated (c) decimated (d) deflected SSC Constable GD-03/12/2021 Shift-II Ans. (a) : In the above sentence, ‘ derived’ will be used instead of ‘dedicated’ (devote), Because it gives suitable meaning for this sentence. Other options have different meaning. Decorated – To furnish with something. Decimated – to cause great destruction or harm. Deflected – to turn coside. Correct sentence As we all are aware, silk is derived from the cocoons of mulberry silkworms. Sentence Improvement

653. Select the most appropriate option that can substitute the underlined segment in the given sentence. If there is no need to substitute it, select 'No substitution required'. The three friends spent over an hour discussing the problem with their teacher. (a) discussing about (b) discussing on (c) having discussion on (d) No substitution required SSC Constable GD-03/12/2021 Shift-II Ans. (d) : No substitution required. Discuss → something with somebody 654. Select the option that will improve the underlined part of the given sentence. In case no improvement is needed, select 'No improvement required'. Not only did he run away from home, but also stole the money. (a) No improvement required (b) He only did not run (c) Only he did not ran (d) Not only did he ran SSC Constable GD-01/12/2021 Shift-I Ans. (a) : No improvement required 655. Select the most appropriate option to substitute the underlined segment in the given sentence. If there is no need to substitute it, select 'No improvement'. It was cloudy in the morning but the sun is shining now. (a) the sun has shone (b) No improvement (c) the sun were shining (d) a sun is shining SSC CHSL (Tier-I) –13/04/2021 (Shift-I) Ans. (b) : No improvement. Note- The second clause of the sentence is giving the sense of present tense. Hence, 'is + verb + ing' will be used. 656. Select the option that will improve the underlined segment in the given sentence. In case no improvement is needed, select 'No improvement' I must have dropped one of my gloves in the car. (a) any (b) No improvement (c) an (d) the SSC CHSL (Tier-I) –06/08/2021 (Shift-I) Ans. (b) : There is no need to substitute it select ‘No substitution required’ 657. Select the most appropriate option to substitute the underlined segment in the given sentence. If there is no need to substitute it select 'No substitution required'. Fermented vegetables can be stored for about a year without going bad. (a) with going bad (b) to go bad (c) No substitution required (d) without going worst SSC CHSL (Tier-I) –19/04/2021 (Shift-I)

184

YCT

Ans. (c) : There is no need to substitute it select ‘No substitution required’. 658. Select the most appropriate option to substitute the underlined segment in the given sentence. If no substitution is required, select 'No substitution'. Either the bears or the lion has escaped from the zoo. (a) No substitution (b) have escaped from the zoo (c) are escaped from the zoo (d) is escaped from the zoo SSC CGL (Tier-I) –17/08/2021 (Shift-I) Ans. (a) : No substitution, required. 659. Select the most appropriate option to substitute the underlined segment in the given sentence. If no substitution is required, select 'No substitution'. While washing your hands, rub them together for 20 seconds to remove the microbes on them. (a) the microbes on their (b) the microbes on those (c) No substitution (d) the microbes on they SSC CGL (Tier-I) –13/08/2021 (Shift-I) Ans. (c) : No substitution. 660. Select the most appropriate option to substitute the underlined segment in the given sentence. If no substitution is required, select 'No substitution'. The toy drummer plays the drum if you press the button at the back. (a) played the drum if you are pressing (b) No substitution (c) will play the drum if you will press (d) playing the drum if you pressed SSC CGL (Tier-I) –13/08/2021 (Shift-I) Ans. (b) : No substitution. 661. Select the most appropriate option to improve the underlined segment in the given sentence. If there is no need to improve it, select 'No improvement'. I was being depressed when you called : (a) No improvement (b) being depressing (c) feeling depressed (d) going depressed SSC CGL (Tier-II) 16/11/2020 (3 pm - 5 pm) Ans. (c) : ‘Feeling depressed' will be used in place of the underlined part of the above sentence ‘being depressed' because it gives suitable meaning for the sentence. Correct Sentence - I was feeling depressed when you called 662. Select the alternative that will improve the underlined part of the sentence in case there is no improvement select “No improvement”. The children’s cricket ball hit their neighbour’s window and it broke. (a) and it is broke (b) and it broken (c) and it was breaking (d) No improvement SSC CPO-SI (Tier-II) 27/09/2019 (3 pm - 5 pm) Ans. (d) : No improvment. Sentence Improvement

663. Select the alternative that will improve the underlined part of the sentence in case there is no improvement select “No improvement”. Unless Arjun takes a taxi he will not reach the airport in time. (a) However Arjun takes a tax (b) If Arjun take a taxi (c) No improvement (d) Unless Arjun take taxi SSC CPO-SI (Tier-II) 27/09/2019 (3 pm - 5 pm) Ans. (c) : No improvement 664. Select the alternative that will improve the underlined part of the sentence in case there is no improvement select “No improvement”. The only vacant seat in the bus was right in the corner. (a) right on the corner (b) right into the corner (c) right up to the corner (d) No improvement SSC CPO-SI (Tier-II) 27/09/2019 (3 pm - 5 pm) Ans. (d) : No improvement 665. Select the alternative that will improve the underlined part of the sentence in case there is no improvement select “No improvement”. I had to go to Delhi, however, I changed my mind and decided not to go. (a) therefore I change my mind (b) nevertheless I change my mind (c) however I will change my mind (d) No improvement SSC CPO-SI (Tier-II) 27/09/2019 (3 pm - 5 pm) Ans. (d) : No improvement 666. Select the most appropriate option to substitute the underlined segment in the given sentence. If there is no need to substitute it, select 'No substitution required'. I cannot tell you now whether I will be able to come or not. (a) in case (b) unless (c) because (d) No substitution required SSC CPO-SI – 25/11/2020 (Shift-I) Ans. (d) : No substitution required 667. Select the most appropriate option to substitute the underlined segment in the given sentence. If there is no need to substitute it, select 'No substitution required'. One morning I went out with my cousin which is the lawyer. (a) who is the (b) which is a (c) No substitution required (d) who is a SSC CPO-SI – 25/11/2020 (Shift-I) Ans. (d) : ‘Who is a’ will be used instead of the underlined part of the above. Sentence ‘which is the’. Because ‘who’ is used as a relative pronoun living things (my causin). And generally article ‘the’ is not used before past name. Correct Sentence– One morning I went out with my cousin who is lawyer. 668. Select the most appropriate option to substitute the underlined segment in the given sentence. If there is no need to substitute it, select ‘No substitution required’. A large swarm of dogs was blocking the way. (a) No substitution required (b) herd of (c) school of (d) pack of SSC CPO-SI – 24/11/2020 (Shift-I)

185

YCT

Ans. (d) : ‘Pack of’ will be used in place of the underlined part of the above sentence ‘swarm of’. Swarm of (Swarm of bee) and pack of (Pack of dog) is called. Herd:- a group of animals of the same type that live and feed together. School :- a large number of fish or other sea animals, swimming together. Correct Sentence– A large pack of dogs was blocking the way. 669. Select the most appropriate option to substitute the underlined segment in the given sentence. If there is no need to substitute it, select ‘No substitution required’ The forecast predicts that there will be little rain in the month of July. (a) should be many rain (b) No substitution required (c) would be few rain (d) must be a little rain SSC CPO-SI – 24/11/2020 (Shift-I) Ans. (b) : No improvement 670. Select the most appropriate option to substitute the underlined segment in the given sentence. If here is no need to substitute it, select ‘No substitution required’. When I left my house this morning, it was raining. (a) rained (b) is raining (c) rains (d) No substitution required SSC CPO-SI – 23/11/2020 (Shift-I) Ans. (d) : No improvement 671. Select the most appropriate option to substitute the underlined segment in the given sentence. If there is no need to substitute it, select 'No substitution required'. I looked anywhere for my puppy but could not find it. (a) somewhere (b) No substitution required (c) everywhere (d) nowhere SSC CPO-SI – 25/11/2020 (Shift-II) Ans. (c) : ‘Everywhere (everyplace) will be used instead of the underlined part of the above sentence ‘any where’ (anyplace). Which is suitable, according to the sense of sentence . Correct Sentence– I looked everywhere for my puppy but could not find it. 672. Select the most appropriate option to substitute the underlined segment in the given sentence. If there is no need to substitute it, select 'No substitution required'. Suman performed at the festival well yesterday. (a) No substitution required (b) yesterday at the festival well (c) at the festival yesterday well (d) well at the festival yesterday SSC CPO-SI – 25/11/2020 (Shift-II) Sentence Improvement

Ans. (d) : ‘Well at the festival yesterday’ will be used in place of the underlined part of the above given sentence ‘at the festival well yesterday’. Because here the adverb ‘well’ is describing the quality of performed (Verb). Correct SentenceSuman performed well at the festival yesterday. 673. Select the most appropriate option to substitute the underlined segment in the given sentence. If there is no need to substitute it, select ’No substitution required’. When challenged, the man had no answer for his actions. (a) When in challenge (b) When to be challenged (c) No substitution required (d) While challenges SSC CPO-SI – 23/11/2020 (Shift-II) Ans. (c) : No substitution required. 674. Select the most appropriate option to substitute the underlined segment in the given sentence. If there is no need to substitute it, select ‘No substitution required’. She walked among me without greeting me. (a) No substitution required (b) about me (c) pass me (d) over me SSC CPO-SI – 23/11/2020 (Shift-II) Ans. (c) : ‘Pass me’ would be appropriate in place of the underlined segment of the above sentence ‘among me’ which gives appropriate meaning for the sense of sentence. Correct Sentence– She walked pass me without greeting me. 675. Select the most appropriate option to substitute the underlined segment in the given sentence. If there is no need to substitute it, select ‘No substitution required’. He decided not to take to heart the loss that he had incurred in business. (a) be taking heart (b) take heart of (c) No substitution required (d) takes to heart SSC CPO-SI – 24/11/2020 (Shift-II) Ans. (c) : No substitution required 676. Select the most appropriate option to substitute the underlined segment in the given sentence. If no substitution is required, select No improvement. We had never seen so strong blizzard. The wind speed was 130 km per hour. (a) so strong the blizzard (b) such strong a blizzard (c) such a strong blizzard (d) no improvement SSC CGL (Tier-I) – 06/06/2019 (Shift-III)

186

YCT

Ans. (c) : ‘Such a strong’ will be used in place of the above sentence ‘so strong’, Because blizzard (with very strong winds gives the appropriate meaning. Correct Sentence– We had never seen such a strong blizzard. The wind speed was 130 km per hour. 677. Select the most appropriate option to substitute the underlined segment in the given sentence. If no substitution is required, select No improvement. Biologists believe that increased human activity means an accelerated rate of change in habitat for all creatures of the world. (a) No improvement (b) human activity meant a acelerated rate in change (c) human activities mean an acelerated rate form change (d) human activity is meaning an acelerated rate of change SSC CGL (Tier-I) – 10/06/2019 (Shift-II) Ans : (a) No improvement 678. Select the most apropriate option to substitute the underlined segment in the given sentence. If no substitution is required, select No improvement. Application are to be sent to the Principal before 30th May. (a) were being send to (b) No improvement (c) will be send to (d) are sending to SSC CGL (Tier-I) – 13/06/2019 (Shift-I) Ans : (b) No improvement 679. Select the most appropriate option to substitute the underlined segment in the given sentence. If no substitution is required, select No improvement. Your coming home to dinner on time Should be a rule rather the exception. (a) should be a rule rather being the exception (b) should be the rule rather than the exception (c) No improvement (d) shall be a rule rather than a exception SSC CGL (Tier-I) – 11/06/2019 (Shift-I) Ans. (b) : 'should be the rule rather than' 'the exception' will be used instead of the underlined part of the above sentence ‘should be a rule rather the the exception. Because the use of ‘than’ is appropriate with rather. Rather than- instead of, in place of. Correct Sentence Your coming home to dinner on time should be the rule rather than the exception. 680. Select the most appropriate option to substitute the underlined segment in the given sentence. If no substitution is required, select No improvement. It is convey to all the residents by now that they are required to apply for parking stickers. (a) It will be conveyed to all the residents (b) It has been conveyed to all the residents (c) It is conveyed to all the residents (d) No improvement SSC CGL (Tier-I) – 10/06/2019 (Shift-III) Sentence Improvement

Ans. (b) : ‘It has been conveyed to all the residents’ will be used in place of the underlined part of the above sentence ‘it is convey to all the residents. Because in this sentence, the use of ‘by now’ is expressing the sense of present perfect tense. In the given sentence, In the sense of passive voice is appropriate explanation. Correct Sentence– It has been conveyed to all the residents by now that they are required to apply for parking stickers. 681. Select the most appropriate option to substitute the underlined segment in the given sentence. If no substitution is required, select No improvement. Each dancer performed very well that it was difficult to judge who the best was. (a) such well that (b) so well that (c) No improvement (d) such very well that SSC CGL (Tier-I) – 07/06/2019 (Shift-I) Ans : (b) ‘So’ will be used instead of the underlined segment of the above sentence ‘very’. Because ‘so’ is used with that. Correct Sentence Each dancer performed so well that it was difficult to judge who the best was. 682. Select the most appropriate option to substitute the underlined segment in the given sentence. If no substitution is required, select No improvement. His school was 7 km away his house. I wondered how he covered so long distance daily on foot. (a) so long the distance (b) No improvement (c) such long distance (d) such a long distance SSC CGL (Tier-I) – 07/06/2019 (Shift-II) Ans : (d) The use of ‘such a long distance’ is appropriate instead of the underlined part of the above sentence ‘so long distance’. Because ‘so’ is used before adjective or adverb. (without a noun) E.g. she was so beautiful. ‘such is used before a noun or an adjective. Correct Sentence– His school was 7km. away from his house. I wondered how he covered such a long distance daily on foot. 683. Select the most appropriate option to substitute the underlined segment in the given sentence. If no substitution is required, select No improvement. The Head has been so busy to go through these files and resolve the matter. (a) such busy to go through (b) too busy to go through (c) No improvement (d) too so busy to go through SSC CGL (Tier-I) – 06/06/2019 (Shift-II) Ans : (b) The use of ‘too busy to’ is appropriate instead of in the underlined part of the above sentence ‘so busy to’, because here according to the sentence negative sense is required, for which the used of ‘too’ is appropriate. Correct Sentence The Head has been too busy to go through these files and resolve the matter.

187

YCT

684. Select the most appropriate option to substitute the underlined segment in the given sentence. If there is no need to substitute it, select 'No substitution required'. Tomatoes sown around July in the southern states are harvested in October. (a) Tomatoes sow around July (b) Tomatoes sowing around July (c) No substitution required (d) Tomatoes sown under July SSC CPO-SI – 09/12/2019 (Shift-II) Ans : (c) No substitution required 685. Select the most appropriate option to substitute the underlined segment in the given sentence. If there is no need to substitute it, select 'No substitution'. I couldn't got some sleep because the people in the next room were talking very loudly. (a) couldn't get some sleep (b) couldn't get any sleep (c) couldn't got any sleep (d) No substitution SSC CPO-SI – 09/12/2019 (Shift-I) Ans : (b) ‘Couldn’t got any sleep’ will be used in place of in the underlined part of the above sentence couldn’t got some sleep’. Because the use of ‘any’ is used in negative or Interrogative and the use of ‘some’ is used – Positive sentence. Note :- In the given sentence, the use of could’ is used as a model verb and first form of the verb is used with modal verb. Correct Sentence I couldn't get any sleep because the people in the next room were talking very loudly. 686. Select the most appropriate option to substitute the underlined segment. If no substitution is required select 'No substitution' He had barely spoken the words when he realised his mistake. (a) barely speak the words that (b) barely spoke the words when (c) No substitution (d) barely spoken the words than SSC CPO-SI – 11/12/2019 (Shift-II) Ans : (c) No substitution. 687. Select the most appropriate option to substitute the underlined segment in the given sentence. If there is no need to substitute it, select No improvement. I tried to contact my dentist but her cell phone appeared to have been switched off. (a) appears to had been (b) appear to be (c) No improvement (d) appeared to been SSC CPO-SI – 12/12/2019 (Shift-II) Ans : (c) No improvement 688. Select the most appropriate option to substitute the underlined segment in the given sentence. If there is no need to substitute it, select No improvement. Be polite and courteous will greatly improve your chances of making friends at your new office. Sentence Improvement

For being polite and courteous Being polite and courteous No improvement By being polite and courteous SSC CPO-SI – 12/12/2019 (Shift-II) Ans : (b) 'Being polite and courteous' is used in the above underlined part because 'Being + adjective' structure tells about the activity and behavior. In the given sentence, polite and courteous is an adjective. Correct SentenceBeing polite and courteous will greatly improve your chances of making friends at your new office. 689. Select the most appropriate option to substitute the underlined segment. If no substitution is required select 'No substitution'. It would be wise to accept the company's offer lest you should regret later. (a) lest you should not regret (b) lest you would regret (c) lest you shall not regret (d) No substitution SSC CPO-SI – 12/12/2019 (Shift-I) Ans : (d) No substitution Note-: ‘Should’ is used with ‘lest’. 690. Select the most appropriate option to substitute the underlined segment. If no substitution is required select. 'No substitution'. She has no other hobby but that of collecting coins. (a) No substitution (b) no another hobby but (c) not other hobbies but (d) no other hobby than SSC CPO-SI – 12/12/2019 (Shift-I) Ans : (d) ‘No other hobby than’ will be used in place of the underlined part of the above sentence ‘no other hobby but? Because no other than is used when the emphasis is on only one of severed events / habits /decisions. Correct Sentence– She has no other hobby than that of collecting coins. 691. Select the most appropriate option to substitute the underlined segment in the given sentence. If there is no need to substitute it, select No improvement. Shailaja is prettier than either of her three cousins. (a) prettier than any of (b) most pretter than all of (c) pretty than either of (d) No improvement SSC CPO-SI – 13/12/2019 (Shift-II) Ans. (a) : ‘Prettier than any of’ is used instead of the underline part of the above sentence’ ‘Prettier than either of’. While comparison is happening in three cousins. Here the use of ‘any of’ will be appropriate, in which more than two things is comparing. Correct Sentence– Shailaja is prettier than any of her three cousins.

188

(a) (b) (c) (d)

YCT

692. Select the most appropriate option to substitute the underlined segment in the given sentence. If there is no need to substitute it, select No improvement. This college is not only equipped with the best laboratories but also has the most competent staff. (a) No improvement (b) and also had the most (c) but has the very (d) but also have the most SSC CPO-SI – 13/12/2019 (Shift-II) Ans. (a) : No improvement. 693. Select the most appropriate option to substitute the underlined segment. If no substitution is required select 'No substitution' Some people in the audience were asleep while the chief guest was speaking. (a) while a chief guest is speaking (b) when the chief guest has spoken (c) whenever the chief guest spoke (d) No substitution SSC CPO-SI – 13/12/2019 (Shift-I) Ans. (d) : No substitution. 694. Select the most appropriate option to substitute the underlined segment. If no substitution is required select 'No substitution' The colour of her cheeks is like a rose. (a) are just as a rose (b) are like a rose (c) No substitution (d) is like that of a rose SSC CPO-SI – 13/12/2019 (Shift-I) Ans. (d) : ‘Is like that of a rose’. Would be appropriate in place of the underlined part of the above sentence ‘is like a rose’. Because the underlined word ‘rose’ is compared to the colour of her cheeks. That’s why similar type of things are done with similar things. Because here the colour of the rose is not compared to the rose. Correct Sentence– The colour of her cheeks is like that of a rose. 695. Select the most appropriate option to substitute the underlined segment in the given sentence. If there is no need to substitute it, select No improvement. A spokesperson of the company explained that the media had misrepresented the facts about the accident in their building. (a) misrepresent the facts on the accident (b) misrepresenting the facts about the accident (c) No improvement (d) misrepresents the facts for an accident SSC CHSL (Tier-I) –10/07/2019 (Shift-I) Ans : (c) No improvement. 696. Select the alternative that will improve the underlined part of the sentence in case there is no improvement select No improvement The crowd wave feverishly when they saw their leader. (a) waving hard (b) wave well (c) No improvement (d) waved frantically SSC CHSL (Tier-I) –10/07/2019 (Shift-III) Sentence Improvement

Ans : (d) The use of ‘waved’ would be appropriate in place of the underlined part of given sentence ‘wave’ because the incident is of the past, so the waved will be correct. Feverishly :- in a way that shows strong feeling of excitement or worry, often with a lot of activity or quick movements.] Frantically :- quickly and with a lot of activity, but in away that is not very well organized. Correct Sentence– The crowd waved frantically when they saw their leader. 697. Select the alternative that will improve the underlined part of the sentence in case there is no improvement select No improvement. I experienced excruciating pain when the hammer fell on my foot. (a) exciting pain (b) affable paining (c) strong pains (d) No improvement SSC CHSL (Tier-I) –10/07/2019 (Shift-III) Ans : (d) No improvement. 698. Select the most appropriate option to substitute the underlined segment in the given sentence. If there is no need to substitute it, select No improvement. We have been waiting to watch this movie since we first read its review. (a) We are been waiting (b) We has been waiting (c) We have being waiting (d) No improvement SSC CHSL (Tier-I) –09/07/2019 (Shift-I) Ans : (d) No Improvement. 699. Select the most appropriate option to substitute the underlined segment in the given sentence. If there is no need to substitute it, select No improvement. The express bus generally stops at only the major towns on the route. (a) generally stopping on only the major towns (b) generally stop in only the major towns (c) generally stopped from only the major towns (d) No improvement SSC CHSL (Tier-I) –09/07/2019 (Shift-III) Ans : (d) No improvement 700. Identify the most appropriate option to substitute segment. If no substitution is required, select No substitution. Those who have not received the invitation he should leave. (a) he should be leaving (b) they should leave (c) should leave (d) No substitution SSC CHSL (Tier-I) –04/07/2019 (Shift-II) Ans : (c) The use of ‘should leave’ would be appropriate in place of the underlined part of the above sentence ‘he should leave’. Because in the given sentence pronoun ‘he’ will not be used because pronoun ‘Those’ has used before it. Hence the use of ‘he’ is superfluous. Correct Sentence– 'Those who have not received the invitation should leave'

189

YCT

701. Identify the most appropriate option to substitute the underlined segment. If no substitution is required, select No substitution. Of Rama and Usha the later is the most responsible. (a) the later is more (b) the latter is the more (c) the latter is most (d) No substitution. SSC CHSL (Tier-I) –04/07/2019 (Shift-III) Ans : (b) The underlined part of the above sentence, ‘the latter is the more’ will be used in place of ‘the later is the most’. Because this sentence is used as a comparative degree. Hence ‘the more’ is appropriate in place of the most. Later :- near the end of a period of time, life etc. Latter– used to refer to the second of two things. Correct sentence Of Rama and Usha the latter is the more responsible. 702. Select the most appropriate option to substitute the underlined in the given sentence. If there is no need to substitue it, select No improvement. I hope never to have another so experience as I had in Puri during Cyclone Fani. (a) No improvement (b) such experience as (c) as experience as (d) same experience as SSC CHSL (Tier-I) –02/07/2019 (Shift-I) Ans. (b) : ‘such’ is used in place of the underlined part of the above sentence ‘so’, because adjective (such) will be used to describe experience (noun) and not adverb 'so'. Correct Sentence I hope never to have another such experience as I had in Puri during Cyclone Fani. 703. Select the most appropriate option to substiute the underlined segment in the given sentence. If there is no need to substitute it, select No improvement. Harry and Meghan have decided to work and become financially independent. (a) No improvement (b) have decided to work and becam (c) have decide to work and become (d) has decided to work and become SSC CHSL (Tier-I) 17/03/2020 (Shift-I) Ans. (a) No improvement. 704. Select the alternative that will improve the underlined part of the sentence. In case there is no improvement select 'NO improvement'. I have prepared well for the exam and am ready to answer any question. (a) preparing (b) may prepare (c) will prepared (d) No improvement SSC CHSL (Tier-I) 17/03/2020 (Shift-II) Ans. (d) : No improvement 705. Select the alternative that will improve the underlined part of the sentence. In case there is no improvement select 'No improvement'. The struggle for climate change must be prioritised given that out population is increasing and not decreasing. (a) is prioritising (b) No improvement (c) may have prioritised (d) could prioritise SSC CHSL (Tier-I) 18/03/2020 (Shift-II) Ans. (b) : No improvement. Sentence Improvement

706. Select the most appropriate option to substitute the underlined segment in the given sentence. If there is no need to substitute it, select 'No improvement'. Before it was modified, the Law provided with the owner could take possession of the goods at any time. (a) provided that (b) provided on (c) No improvement (d) provided as SSC CHSL (Tier-I) 18/03/2020 (Shift-I) Ans. (a) : ‘Provided that’ will be used in place of the underlined part of the above sentence ‘provide with’. Because the use of ‘provided that’ is used as idiomatic which means – on the condition that. Correct Sentence– Before it was modified, the Law provided that the owner could take possession of the goods at any time. 707. Select the most appropriate option to substitute the underlined segment in the given sentence. If there is no need to substitute it, select 'No improvement'. Some people believe that 2020 is going to be a very significant year in our history. (a) No improvement (b) will going (c) were going (d) are going SSC CHSL (Tier-I) 26/10/2020 (Shift-II) Ans. (a) : No improvement 708. Select the most appropriate option to substitute the underlined segment in the given sentence. If there is no need to substitute it, select 'No improvement. Over the past 20 years, there has been a constant decline in the variety of vegetables that we eat because the native varieties are no longer available. (a) is being (b) No improvement (c) was been (d) had been SSC CHSL (Tier-I) 19/10/2020 (Shift-I) Ans. (b) : No improvement. 709. Select the most appropriate option to substitute the underlined segment in the given sentence. If there is no need to substitute it, select 'No improvement'. The rising prices of essential food items have placed an weighty burden on the poor. (a) lightest (b) trivial (c) enormous (d) No improvement SSC CHSL (Tier-I) 12/10/2020 (Shift-II) Ans. (c) : ‘Enormous (adj.) extremely large, huge’ will be used in place of the underlined segment of the above sentence ‘weighty’. Because enormous burden is used for ‘Rising prices’, not a weighty burden. The meaning of other optionslightest – very light, pole in colour. trivial – not important or series, not worth considering. Correct Sentence The rising prices of essential food items have placed an enormous burden on the poor.

190

YCT

710. Select the most appropriate option to substitute the underlined segment in the given sentence. If there is no need to substitute it, select No improvement. She was looking troubled when the teacher asked her to submit the homework. (a) is look trouble (b) No improvement (c) had been trouble (d) was troubling SSC CHSL (Tier-I) 15/10/2020 (Shift-III) Ans. (b) : No improvement. 711. Select the most appropriate option to substitute the underlined segment in the given sentence. If there is no need to substitute it, select 'No improvement'. Why did the teacher ask all four of you to stay back in class in the break ? (a) asking four of you (b) No improvement (c) asked of four of you (d) ask you all four SSC CHSL (Tier-I) 15/10/2020 (Shift-II) Ans. (b) : No improvement. 712. Select the most appropriate option to substitute the underlined segment in the given sentence. If there is not need to substitute it, select No improvement. His uncle advised Naveen to keep away from involving himself in the controversy. (a) No improvement (b) from keep away for (c) that he keeps away for (d) for keep away from SSC CHSL (Tier-I) 15/10/2020 (Shift-I) Ans. (a) : No improvement. 713. Select the most appropriate option to substitute the underlined segment in the given sentence. If there is no need to substitute the underlined segment in the given sentence. If there is no need to substiute it, select 'No improvement'. After switching off the light, he went off to sleep. (a) being switched off the light (b) he has switched off the light (c) No improvement (d) switching off a light SSC CHSL (Tier-I) 20/10/2020 (Shift-I) Ans. (c) : No improvement. 714. Select the most appropriate option to substitute the underlined segment in the given sentence. If there is no need to substitute it, select No improvement. Have you got confirmation of the receipt of the donation? (a) Have you get confirmation (b) Did you got confirmation (c) Has you got confirmations (d) No improvement SSC CHSL (Tier-I) 20/10/2020 (Shift-II) Ans. (d) : No improvement 715. Select the alternative that will improve the underlined part of the sentence. In case there is no improvement select 'No improvement'. There was no sign of recognition of his face when they met after ten years. Sentence Improvement

no sign for recognition no sign to recognise no improvement no any sign of recognition SSC CHSL (Tier-I) 17/03/2020 (Shift-III) Ans. (c) : No improvement. 716. Select the most appropriate option to substitute the underlined segment in the given sentence. If there is no need to substitute it, select 'No improvement'. Lot of times she advised him for not being lazy for completing such a tough task. (a) No improvement (b) Much of time (c) A lot of times (d) A lot of time SSC CHSL (Tier-I) 18/03/2020 (Shift-III) Ans. (c) : ‘A lot of times’ will be used in place of the underlined part of the above sentence ‘Lot of times’ . Because ‘A lot of times’ is appropriate. Because lot of time means:- single time or multiple time the task had been done while. A lot of time means:-the task has been done many/multiple accassions. Correct Sentence– A lot of times she advised him for not being lazy for completing such a tough task. 717. Select the most appropriate option to substitute the underlined segment in the given sentence. If there is no need to substitute it, select No improvement. As many as I care for her, I still have to disagree with her on this topic. (a) As much as (b) As long to (c) As high as (d) No improvement SSC CHSL (Tier-I) 19/03/2020 (Shift-I) Ans. (a) : ‘As much as’ will be used in place of the underlined part of the above sentence ‘as many as. Because the use of ‘as much as’ is used for uncountable things. As much as – almost As long as – provided that. as much as - almost as long as - provided that Correct Sentence– As much as I care for her, I still have to disagree with her on this topic. 718. Improve the bracketed part of the sentence. In doing so, she (farther) raised the profile of badminton and of women's sport itself in India. (a) far away (b) far off (c) further (d) No improvement SSC MTS 10-10-2017 (Shift-III) Ans. (c) : ‘Further’ will be used in place of in the given sentence ‘farther’. Because farther is used for distance and further is used for ‘forward’. Correct Sentence– In doing so, she ‘further’ raised the profile of badminton and of women's sport itself in India.

191

(a) (b) (c) (d)

YCT

719. Improve the bracketed part of the sentence. And makes it a point to remind (eachone) that her recovery is still incomplete, and the she would be back at her best in upcoming competitions. (a) everyone (b) each people (c) every people (d) No improvement SSC MTS 11-10-2017 (Shift-III) Ans. (a) : 'Each' has the same meaning as 'everyone'. ‘one’ is not used with each. Hence ‘everyone’ will be used in place of each one. 720. Select the most appropriate option to substitute the underlined segment in the given sentence. If there is no need to substitute it, select No improvement. It is not for nothing that Mumbai is talked about as the most cosmopolitan of Indian cities. (a) is talking (b) are talked (c) No improvement (d) was talking SSC MTS – 22/08/2019 (Shift-III) Ans. (c) : No improvement. 721. Select the most appropriate option to substitute the underlined segment. If there is no need to substitute it, select No improvement. If you do not vote, you do not have the right to protest. (a) right in protest (b) right on protest (c) right for protest (d) No improvement SSC MTS – 22/08/2019 (Shift-II) Ans. (d) : No improvement 722. Select the most appropriate option to substitute the underlined segment. If there is no need to substitute it, select No improvement. The lack of green spaces has led to an increase in pollution levels. (a) an increasing in (b) No improvement (c) a increase on (d) a increase of SSC MTS – 22/08/2019 (Shift-II) Ans. (b) : No improvement 723. Select the most appropriate option to substitute the underlined segment in the given sentence. If there is no need to substitute it, select 'No substitution required'. The study found that men who were married lived longer than those who were not. (a) that who were (b) the one who were (c) No substitution required (d) these who were SSC MTS – 09/08/2019 (Shift-I) Ans. (c) : No Improvement. 724. Select the most appropriate option to substitute the underlined segment in the given sentence. If there is no need to substitute it, select 'No substitution required'. They won't commit themselves until they see which way the wind is blowing. (a) their till (b) No substitution required (c) themself until (d) them until SSC MTS – 08/08/2019 (Shift-III) Ans. (b) : No substitution required. Sentence Improvement

725. Select the most appropriate option is substitute the underlined segment in the given sentence. If there is no need to substitute it, select No Improvement. The terrorist was arrested for the first time in December, 2001. (a) for first timing (b) No Improvement (c) in the first time (d) for one times SSC MTS – 05/08/2019 (Shift-II) Ans. (b) : No improvement Note- Article ‘the’ is used before ordinal adjective. 726. Select the most appropriate option to substitute the underlined segment in the given sentence. If there is no need to substitute it, select No Improvement. For her, money is only the means to an end. (a) the means for an end (b) means to end (c) No Improvement (d) a means to the end SSC MTS – 05/08/2019 (Shift-I) Ans. (c) : No Improvement. 727. Select the most appropriate option to substitute the underlined segment in the given sentence. If there is no need to substitute it, select No Improvement. Many accidents at the factory are caused of workers who don't read warning signs. (a) are caused because of (b) is the cause of (c) is caused by (d) No Improvement SSC MTS – 05/08/2019 (Shift-I) Ans. (a) : ‘Are caused because of’ is appropriate in place of the underlined part of the above sentence ‘are caused of’. Because it gives proper meaning for this sentence. Correct Sentence– Many accidents at the factory are caused because of workers who don't read warning signs. 728. Select the most appropriate option to substitute the underlined segment in the given sentence. If there is no need to substitute it, select No Improvement. I don't think many people will be able to attend to meeting tomorrow. (a) is able (b) No Improvement (c) have able (d) are able SSC MTS – 02/08/2019 (Shift-III) Ans. (b) : No improvement 729. Select the most appropriate option to substitute the underlined segment in the given sentence. If there is no need to substitute it, select No Improvement. Please tell the story in a nutshell. (a) at a nutshell (b) in the nutshell (c) in nutshells (d) No Improvement SSC MTS – 02/08/2019 (Shift-I) Ans. (d) : No Improvement. Note:- In a nutshell (in brief) we use in a nutshell to indicate that we are saying something in a brief way.

192

YCT

730. Select the most appropriate option to substitute the underlined segment in the given sentence. If there is no need to substitute it, select No improvement. Shashi couldn't walk no more. (a) more father (b) any farther (c) far more (d) No improvement SSC MTS – 22/08/2019 (Shift-I) Ans. (b) : ‘Any farther’ will be used instead of the underlined part of in the above sentence ‘No more’. Because here we are talking about moving forward. Hence it makes proper sense. Correct sentence– Shashi couldn't walk any farther. 731. Select the most appropriate option to substitute the underlined segment in the given sentence. If there is no need to substitute it, select No improvement. Greyhounds do not make good watchdogs but they are very gentle with children. (a) No improvement (b) very gentler with (c) more gentle to (d) much gentle with SSC MTS – 21/08/2019 (Shift-II) Ans. (a) : No improvement. Note- Generally ‘very’ is used as adjective of positive degree, since ‘gentle’ is Adjective of positive degree. That’s why, the use of ‘very’ is appropriate here. 732. Select the most appropriate option to substitute the underlined segment in the given sentence. If there is no need to substitute it, select No improvement. Simran is more intelligent than any other girl in the class. (a) No improvement (b) most intelligent (c) the more itelligent (d) the more intelligent SSC MTS – 20/08/2019 (Shift-III) Ans. (a) : No improvement 733. Select the most appropriate option to substitute the underlined segment in the given sentence. If there is no need to substitute it, select No improvement. Vivek was astonished at his failure in the exam. (a) at his failing (b) in his failing (c) No improvement (d) in his failure SSC MTS – 20/08/2019 (Shift-II) Ans. (c) : No improvement 734. Select the most approriate option to substitute the underlined segment. If there is no need to substitute it, select No improvement. Patel played every game that was played in the school. (a) that has been playing (b) that was playing (c) that is being played (d) No improvement SSC MTS – 20/08/2019 (Shift-I) Ans. (d) : No improvement Sentence Improvement

735. Select the most appropriate option to substitute the underlined segment. If there is no need to substitute it, select No improvement. She told the children not to stop the work. (a) don't stop (b) not stop (c) No improvement (d) not stopping SSC MTS – 16/08/2019 (Shift-III) Ans. (c) :No improvement. Note – The use of (to + stop) is used as a noun, infinitive. 736. Select the most appropriate option to substitute the underlined segment in the given sentence. If there is no need to substitute it, select No improvement. I don't remember exactly when did I go to Shimla last year. (a) when I was going (b) when I went (c) when I go (d) No improvement SSC MTS – 16/08/2019 (Shift-II) Ans. (b) : ‘When I went’ is used in place of the underlined segment of the above sentence ‘when did I go’. Because ‘sentence’ is not an interrogative because there is no question mark (?) at the end. Hence past indefinite tense (affirmative) will be used. Correct Sentence– I don't remember exactly when I went to Shimla last year. 736. Select the mot appropriate option to substitute the underlined segment. If there is no need to substitute it, select No improvement My parents both are doctors. (a) My parents are both (b) Both my parents (c) Both my parent (d) No improvement SSC MTS – 16/08/2019 (Shift-I) Ans. (b) : ‘Both my parents’ will be used instead of the underlined part of the above sentence ‘My parents both’. Because ‘both’ is always used before the subject in the sense of two. Correct Sentence– Both my parents are doctors. 738. Select the most appropriate option to substitute the underlined segment in the given sentence. If there is no need to substitute it, select 'No substitution required'. It's fascinating to watch how a baby changes and develops over time. (a) No substitution required (b) change and develop (c) changing and developing (d) changed and developed SSC MTS – 13/08/2019 (Shift-III) Ans. (a) : No substitution required 739. Select the most appropriate option to substitute the underlined segment in the given sentence. If there is no need to substitute it, select No improvement. Robert told me about his new job, which he's enjoying very much.

193

YCT

(a) (b) (c) (d)

No improvement Which he enjoying that which he's enjoyed which he had enjoy SSC MTS – 13/08/2019 (Shift-II) Ans. (a) : No improvement 740. Select the most appropriate option to substitute the underlined segment in the given sentence. If there is no need to substitute it, select No improvement. Rashi hasn't eaten everything from yesterday. (a) anything since (b) something for (c) nothing along (d) No improvement SSC MTS – 13/08/2019 (Shift-I) Ans. (a) : ‘Anything since’ will be used in place of the underlined part of the above sentence ‘everything from’. Because according to the sense, the use of ‘anything’ is appropriate because ‘the sentence’ is negative and point of time ‘yesterday’ is given, so ‘since’ will be use in place of ‘from’. Correct Sentence– Rashi hasn't eaten anything since yesterday. 741. Select the most appropriate option to substitute the underlined segment in the given sentence. If there is no need to substitute it, select option 'No substitution required'. The interview was broadcast on the same day he was sworn in as President. (a) had broadcast (b) broadcast (c) was broadcasted (d) No substitution required SSC MTS – 09/08/2019 (Shift-III) Ans. (d) : No substitution required Note – The above sentence is in passive form. Hence was + V3 (broadcast) will be used. Past form of Broadcast is used- broadcast/broadcasted. 742. Select the most appropriate option to substitute the underlined segment in the given sentence. If there is no need to substitute it, select 'No substitution required'. Passengers were asked to identify their suitcases before they were loaded on the plane. (a) they loaded on (b) it was loaded on (c) they were loaded over (d) No substitution required SSC MTS – 08/08/2019 (Shift-II) Ans. (d) : No substitution required. 743. Select the most appropriate option to substitute the underlined segment in the given sentence. If there is no need to substitute it, select 'No substitute required'. Vocational training should not be seen as less important than an academic education. (a) than a academic education (b) No substitution required (c) than an academic educate (d) then an academic education SSC MTS – 08/08/2019 (Shift-II) Ans. (b) : No substitution required

Sentence Improvement

744. Select the most appropriate option to substitute the underlined segment in the given sentence. If there is no need to substitute it, select No Improvement. MS Dhoni has no immediate plans to retire. (a) plan to retirement (b) plans retiring (c) planning for retire (d) No Improvement SSC MTS – 07/08/2019 (Shift-III) Ans. (d) : No improvement Note – ‘to + retire’ is used as infinitive (noun) 745. Select the most appropriate option to substitute the underlined in the given sentence. If there in no need to substitute it, select 'No substitution required'. Carrying an umbrella is not at all a good idea during thunderstorms. (a) To carry the umbrella (b) Carrying a umbrella (c) No substitution required (d) Carry umbralla SSC GD – 08/03/2019 (Shift-II) Ans. (c) : No substitution required 746. Select the most appropriate option to substitute the underlined segment in the given sentence. If there is no need to substitute it, select No Improvement We should offer due respects to all elderly persons. (a) offer due respect to (b) offers due respects to (c) offer due respects for (d) No improvement SSC GD–18/02/2019 (Shift-III) Ans. (a) : ‘offer due respect to’ will be used in place of in the underlined part of the above sentence ‘offer due respects to’. Because in according to sense of the sentence ‘due respect to (respectfully)’ is used. Correct Sentence We should offer due respect to all elderly persons. 747. Select the most appropriate option to substitute the underlined in the given sentence. If there is no need to substitute it, select 'No improvement. The river is flooded and it has over flown its banks. (a) it overflow its banks. (b) it has overflow its banks. (c) it overflowing its banks. (d) No improvement SSC GD – 18/02/2019 (Shift-II) Ans. (d) : No improvement

194

YCT

03. FILL IN THE BLANKS TCS hewšve& hej DeeOeeefjle (Based On TCS Pattern) Chapterwise 1

Subject and Verb Agreement

2

Form of Verb (Tense/number)

3

Preposition

4

Conjunction

5

Article

Fill in the Blanks

Exam CGL (Tier-1) CGL (Tier-2) CHSL (Tier-1) CHSL (Tier-2) Selection Post VII, VIII, XI SSC MTS SSC GD SSC CPO SI CGL (Tier-1) CGL (Tier-2) CHSL (Tier-1) CHSL (Tier-2) Selection Post VII, VIII, XI SSC MTS SSC GD SSC CPO SI CGL (Tier-1) CGL (Tier-2) CHSL (Tier-1) CHSL (Tier-2) Selection Post VII, VIII, XI SSC MTS SSC GD SSC CPO SI CGL (Tier-1) CGL (Tier-2) CHSL (Tier-1) CHSL (Tier-2) Selection Post VII, VIII, XI SSC MTS SSC GD SSC CPO SI CGL (Tier-1) CGL (Tier-2) CHSL (Tier-1) CHSL (Tier-2) Selection Post VII, VIII, XI SSC MTS SSC GD SSC CPO SI 195

Question No. 2 1 2 1 1 1 3 15 5 29 15 5 20 10 16 9 7 14 7 3 3 1 5 7 3 5 2 3 2 3 1 1 1 1 1 2 1 1

Years

(2017–2023)

(2017–2023)

(2017–2023)

(2017–2023)

(2017–2023)

YCT

6

Correct use of Pronoun, Adjective & Adverb

7

Question Tag

8

Interjection

9

Conditional Sentence

10

Miscellaneous

Fill in the Blanks

CGL (Tier-1)

20

CGL (Tier-2)

11

CHSL (Tier-1)

19

CHSL (Tier-2)

6

Selection Post VII, VIII, XI

4

SSC MTS

5

SSC GD

1

SSC CPO SI

4

CGL (Tier-1)

1

CGL (Tier-2)

-

CHSL (Tier-1)

1

CHSL (Tier-2)

-

Selection Post VII, VIII, XI

1

SSC MTS

-

SSC GD

-

SSC CPO SI

1

CGL (Tier-1)

1

CGL (Tier-2)

-

CHSL (Tier-1)

1

CHSL (Tier-2)

-

Selection Post VII, VIII, XI

-

SSC MTS

-

SSC GD

-

SSC CPO SI

-

CGL (Tier-1)

2

CGL (Tier-2)

1

CHSL (Tier-1)

2

CHSL (Tier-2)

1

Selection Post VII, VIII, XI

1

SSC MTS

1

SSC GD

-

SSC CPO SI

2

CGL (Tier-1)

80

CGL (Tier-2)

51

CHSL (Tier-1)

99

CHSL (Tier-2)

70

Selection Post VII, VIII, XI

19

SSC MTS

71

SSC GD

23

SSC CPO SI

80

196

(2017–2023)

(2017–2023)

(2017–2023)

(2017–2023)

(2017–2023)

YCT

Trend Analysis of Questions topicwise from CGL (Pre & Mains) CHSL (Pre & Mains) Selection Post VII, VIII, XI, SSC MTS, SSC GD & Other Exams (2017-2023)

Fill in the Blanks

197

YCT

03. FILL IN THE BLANKS A.

5.

Subject and Verb Agreement

1.

Select the most appropriate option to fill in the blank : The children's Series .......... recommended for young readers. (a) have (b) is (c) are (d) has SSC CGL (Tier-II) 16/11/2020 (3 pm-5 pm) Ans. (b) : In the above sentence, the subject 'The children's series' is singular noun, [S.N. + S.V.] so the helping verb/ main verb will also be singular. Correct sentence– The children's Series is recommended for young readers. 2. Fill in the blank with the most appropriate word. Second-hand furniture __________ here at reasonable prices. (a) were selling (b) are sold (c) is sold (d) has sold SSC CPO-SI – 24/11/2020 (Shift-I) Ans. (c) : 'Is sold' will be used in the blank space of the above sentence because 'furniture' is a singular noun and with it, singular verb is used. Correct sentence- Second-hand furniture is sold here at reasonable prices. 3. Fill in the blank with the most appropriate word. This is one of the matters that ______ me the most. (a) has disturb (b) are disturbing (c) is disturbed (d) disturb SSC CPO-SI – 23/11/2020 (Shift-II) Ans. (d) : The above given sentence in blank space, fill the most appropriate word is option (d) 'disturb'. If two clauses are connected with relative pronouns, verb is used according to the antecedent of 'that'. Here, the antecedent of the relative pronoun (that) 'matters' is plural, then the verb (disturb) is used plural. The correct sentence is'This is one of the matters that disturb me the most'. 4. Select the most appropriate word to fill in the blank. These apples _____ Rs. 100 per kg. Do you think they are expensive? (a) cost (b) is costing (c) has cost (d) costs SSC CPO-SI – 11/12/2019 (Shift-I) Ans : (a) In the above blank space, 'cost' will be used, because with these apples (plural noun), plural verb will be used. Since the sentence is in present indefinite tense in which the first form of verb (verb 1) is used. Correct sentence– These apples cost Rs. 100 per kg. Do you think they are expensive? Fill in the Blanks

Select the most appropriate option to fill in the blank. I who_____ your friend and wellwisher, should have been informed about your decision. (a) are (b) is (c) am (d) be SSC CPO-SI–11/12/2019 (Shift-II) Ans : (c) 'Am' will be used in the blank space of the above sentence because auxiliary verb 'am' is used with 'I', which is antecedent of the relative pronoun 'who'. Correct sentence- I who am your friend and wellwisher, should have been informed about your decision. 6. Select the most appropriate option to fill in the blank. Plenty of information about the flora and fauna of India _____ available in this book. (a) are (b) is (c) has (d) were SSC CPO-SI – 13/12/2019 (Shift-I) Ans. (b) : 'Is' will be used in the blank place of the above sentence because the subject of the sentence 'Information' is an 'uncountable noun' so singular verb (is) will be used. Correct sentence- Plenty of information about the flora and fauna of India is available in this book. 7. Select the most appropriate option to fill in the blank. Our salaries –––––– increased over the years. (a) have (b) are (c) has (d) is SSC MTS – 08/08/2019 (Shift-I) Ans. (a) : In the above blank space, plural verb 'have' will be used, because 'Our salaries' is plural subject and the given sentence is in 'present perfect tense'. Correct sentence- Our salaries have increased over the years. 8. Select the most appropriate option to fill in the blank. Man –––– a strong desire to accumulate wealth. (a) have (b) had (c) is having (d) has SSC MTS – 07/08/2019 (Shift-I) Ans. (d) : 'Has' will be used in blank space of the above sentence. Because when 'man' is used for the entire human race, it is considered singular. The sentence is a universal truth, which is always in the present tense. Correct sentence- Man has a strong desire to accumulate wealth. 9. Select the most appropriate option to fill in the blank and make a grammatically correct sentence. The recent lawsuit –––– put the firm in a tight spot. (a) has (b) is (c) have (d) are SSC MTS – 22/08/2019 (Shift-I)

198

YCT

Ans. (a) : In the blank space of the above sentence, the use of 'has' will be appropriate because this sentence is used in present perfect tense, in which 'has' is used with singular subject 'lawsuit'. Correct sentence- The recent lawsuit has put the firm in a tight spot. 10. Select the most appropriate option to fill in the blank and make a grammatically correct sentence. Timely alerts about floods in Bhutan –––––– a blessing go Indian villages. (a) are proved (b) have proved (c) is proved (d) has proved SSC MTS – 20/08/2019 (Shift-III) Ans. (b) : In the blank space of the above sentence, the use of 'have proved' would be appropriate because in the sentence 'alerts' is a plural subject so the verb will also be plural. Correct sentence- Timely alerts about floods in Bhutan have proved a blessing go Indian villages. 11. Select the most appropriate option to fill in the blank. In the United States of America two dozen people ______ due to the present cold wave. (a) have been killed (b) are being killed (c) has killed (d) has been killed SSC GD – 14/02/2019 (Shift-I) Ans. (a) : The use of verb 'have been killed' would be appropriate, in the blank space of the above sentence. Because 'two dozen people' is plural noun so the verb will also be used in plural. Correct SentenceIn the United States of America two dozen people have been killed due to the present cold wave.

B. Form of Verb (Tense/number) 12.

Thus, the correct answer is option (d). Correct SentenceSnow was falling lightly as everyone was leaving Church. 14. Select the most appropriate option to fill in the blank. If she hadn't _______ her cell phone on the bed, she could have called him. (a) through (b) thrown (c) throne (d) throng SSC MTS 16/05/2023 (Shift-II) Ans. (b) : In blank space of the above given sentence option (b) 'thrown' would be appropriate because the given sentence is past perfect tense and the verb (III) is used. Verb- 'Throw' formThrow (V1) threw (V2) thrown (V3) Thus, the correct answer is option (b). Correct SentenceIf she hadn't thrown her cell phone on the bed, she could have called him. 15. A sentence has been given with a blank to be filled with an appropriate option. Choose the correct alternative. Meteorologists are still ______ to understand sting jets. (a) works (b) on work (c) worked (d) working SSC MTS 15/05/2023 (Shift-I) Ans. (d) : In blank space of the above sentence 'working' will be used. because the sentence is present continuous tense– Rule– Subject + is/am/are + verb1 + ing + object. Correct SentenceMeteorologists are still working to understand sting jets. 16. A sentence has been given with a blank to be filled with an appropriate option. Choose the correct alternative. I bowed to him, but he ______ not to see me. (a) seems (b) seeming (c) seemed (d) seemingly SSC MTS 15/05/2023 (Shift-I) Ans. (c) : The blank space of the above given sentence is filled with options (c) 'seemed' because the given sentence is complex and the verb is used in past tense in the principle clause therefore 'VerbIII' is used in the subordinate clause. Thus, the correct answer is option (c). Correct SentenceI bowed to him, but he seemed not to see me.

A sentence has been given with a blank to be filled with an appropriate option. Choose the correct alternative. I always try to .......... chocholates for you. (a) brought (b) bought (c) bring (d) bringing SSC MTS 08/05/2023 (Shift-III) Ans. (c) : The appropriate option for the blank space is option (c) ' bring' because infinitive 'to' is used. Note :- to + Verb1 (first form of verb) Thus, the correct sentence is – I always try to bring chocholates for you. 13. A sentence has been given with a blank to be filled with an appropriate option. Choose the correct alternative. Snow .......... lightly as everyone was leaving church. 17. (a) fallen (b) falling (c) is falling (d) was falling SSC MTS 04/05/2023 (Shift-II) Ans. (d) : The most appropriate option for the blank space 'was falling' will be used because sentence is in past continuous. The word 'Everyone was leaving' indicates that sentence is in past tense. Fill in the Blanks

199

Select the most appropriate option to fill in the blank. Stories of dolphins’ saving human lives ________ throughout history. (a) have been said (b) have been told (c) told (d) had been said SSC CGL (Tier-I) 25/07/2023 (Shift-IV) YCT

Ans. (b) : The most appropriate option to fill in the above given sentence in blank space is option (b) 'have been told' because 'have been +VIII' is used in 'passive voice', Correct Sentence'Stories of dolphins saving human lives have been told throughout history. There, the correct option is (b). 18. Select the most appropriate phrasal verb to fill in the blank. The driver very subtly ________ the traffic violation he committed. (a) ironed aside (b) ironed out (c) ironed in (d) ironed through SSC CGL (Tier-I) 27/07/2023 (Shift-III) Ans. (b) : In the above sentence, the most appropriate phrasal verb to fill in the blank space is option (b) 'ironed out' which means 'issues or problems have been resolved.' Other options give different meanings. Thus, the correct answer is option (b). Correct SentenceThe driver very subtly ironed out the traffic violation he committed. 19. Select the most appropriate option to fill in the blank. Mosquitoes –––––– if there is no water logging. (a) Will breed (b) Will have been breeding (c) Breed (d) Will not breed SSC CGL (Tier-I) 24/07/2023 (Shift-III) Ans. (d) : In the above sentence 'will not breed' (spawn) will be used in the blank space because it gives appropriate meaning according to sense of the sentence. Other options give different meaning. Correct Sentence'Mosquitoes will not breed if there is no water logging. 20. Fill in the blank with the most suitable word. A haze of panic blurred my vision; something terrible was close to ............ (a) happened (b) happening (c) happens (d) has happened SSC CHSL (Tier-I) 15/03/2023 (Shift-I) Ans. (b) : In the above sentence 'happening' will be used in blank space because here 'Noun' form is needed and gerund is working as Verble Noun. Whereas other option is in verb form. Note- Infinitive (to + verb1) is working as noun. Correct sentenceA haze of panic blurred my vision ; something terrible was close to happening. 21. Fill in the blank with the most appropriate word. She is eagerly waiting for the day when she ............ to her fiance. (a) gets married (b) will marries (c) got marry (d) has married SSC Stenographer (Grade C & D) 11.11.2021 Shift-II Fill in the Blanks

Ans. (a) : In the blank space of the above sentence, the word 'gets married' will be used because (b) Base from of verb comes with will and not 's / es ' form. (c) 'Married' will be used after 'gets' , not Marry's. (d) 'Marry' does not take any preposition in active voice whereas in passive voice, the use of 'to' preposition is appropriate with it. Correct SentenceShe is eagerly waiting for the day when she gets married to her fiance. 22. Select the most appropriate option to fill in the blank. By this time tomorrow, I _______ the work assigned to me. (a) will do (b) will be done (c) will have did (d) will have done SSC CGL Mains 26/10/2023 (Shift-I) Ans. (d) : For the above blank space 'will have done' will be used because when sentence start with 'By this time' then Futer perpect tense will be used in the sentence. Correct Sentence By this time tomorrow, I will have done the work assigned to me. 23. Select the most appropriate option to fill in the blank. There ................. considerable room for improvement in your drawing. (a) were (b) be (c) is (d) are SSC Stenographer (Grade C & D) 11.11.2021 Shift-II Ans. (c) : In the blank space of above sentence, 'is' will be used because 'room' is singular countable noun so singular verb 'is' will be used. Hence option (c) is correct. Correct SentenceThere is considerable room for improvement your drawing. 24. Fill in the blank with the most appropriate word. She told me that she wants to ........... a teacher. (a) become (b) becoming (c) became (d) becomes SSC Stenographer (Grade C & D) 15.11.2021 Shift-I Ans. (a) : In the blank space of the above sentence. The use of 'become' will be appropriate because 'to' make infinitive, the base form of the verb (become) is used after the infinitive 'to' [to + v1]. Correct SentenceShe told me that she wants to become a teacher. 25.

200

Select the most appropriate option to fill in the blank. As soon as she opened the cages, the birds –––– ––––. (a) fly away (b) flies away (c) flown away (d) flew away SSC CHSL 26.05.2022 Shift-I YCT

Ans. (d) : In the blank space of the above sentence, 'flew away' (verb2) will be used because in the beginning of the sentence, 'opened' (v2) has been used, so the subsequent verb will also be used as v2. Correct Sentence– As soon as she opened the cages, the birds flew away. 26. Select the most appropriate option to fill in the blank. Yesterday, after a nice walk, we _____ to sit on the bench, watch the birds, and rest for a while. (a) decided (b) decide (c) decides (d) deciding SSC MTS-15/07/2022 Shift-I Ans. (a) : In the blank space of the above sentence. The use of 'decided' would be appropriate because the use of 'yesterday' makes it clear that the sentence is in past tense so the verb 'decided' (v2) is used. Correct Sentence– Yesterday, after a nice walk, we decided to sit on the bench, watch the birds, and rest for a while. 27. Select the most appropriate option to fill in the blank. He will _____ if they look after him carefully. (a) pull off (b) pull through (c) pull out (d) pull up SSC Constable GD-29/11/2021 Shift-II Ans. (b) : 'Pull through' (to survive) will be used in the blank space the above sentence. It gives appropriate meaning according to the sense of the sentence. Correct SentenceHe will pull through if they look after him carefully. 28. Select the most appropriate option to fill in the blank. The huge vase looks out of –––– with the surroundings. (a) size (b) shape (c) design (d) place SSC Constable GD-06/12/2021 Shift-III Ans. (d) : In the blank space of the above sentence, the word use of 'place' would be appropriate because 'out of place' is a phrase which means (Inappropriate). Correct Sentence– The huge vase looks out of place with the surroundings. 29. Select the most appropriate work to fill in the blank. His voice was ........... by the sound of the helicopter. (a) driven (b) drawn (c) dawned (d) drowned SSC CHSL (Tier-I) –05/08/2021 (Shift-I) Ans. (d) : The above sentence is the passive of past indefinite tense. So in the blank space the verb 'drowned' (v3) would be appropriate, according to the sense. was/were + verb3 Correct sentenceHis voice was drowned by the sound of the helicopter. 30. Select the most appropriate option to fill in the blank. ....... you mind lending me your book for a day? (a) Could (b) Would (c) Need (d) May SSC CGL (Tier-II) 16/11/2020 (3 pm - 5 pm) Fill in the Blanks

Ans. (b) : The option 'would' will be used in the blank space of the above sentence. Because 'would' is used for polite request. The remaining options have different meanings. Correct sentence- Would you mind lending me your book for a day? 31. Select the most appropriate option to fill in the blank. The bus ______ before I reached the bus stand (a) Is leaving (b) Left (c) Had left (d) Leaves SSC Sel. Post Phase-VIII (M.L.) 09.11.2020 (Shift-III)

Ans. (c) : 'Had left' will be used in the blank space of the above sentence because If any two actions happened in the past and which have a relation with each other then the action that happened earlier is called past. Let's translate into past perfect and the work that happened later, let's translate it into past indefinite tense. Correct sentenceThe bus had left before I reached the bus stand 32. Select the most appropriate word to fill in the blank. I ________ something burning now. (a) smelt (b) smell (c) have been smelling (d) have smelt SSC CPO-SI – 23/11/2020 (Shift-I) Ans. (b) : The above sentence is giving the meaning of present indefinite tense. So the use of verb 'smell' (vI) would be appropriate in the blank space. correct sentence - I smell something burning now. 33. Select the most appropriate word to fill in the blank. I’m sure I ______ them at the party last night. (a) am seeing (b) saw (c) have seen (d) was seeing SSC CPO-SI – 23/11/2020 (Shift-I) Ans. (b) : 'Last night' has been used at the end of the above sentence . So the sentence is expressing the meaning of past tense, so in the blank space of the sentence 'saw' will be used (past form of verb see') Correct sentence I’m sure I saw them at the party last night. 34. Fill in the blank with the most appropriate word. More than half the drain ............ been desilted. (a) was (b) have (c) has (d) is SSC CPO-SI – 24/11/2020 (Shift-II) Ans. (c) : In the blank space of above sentence, 'has' will be used because 'more than half the drain' is in singular number, so the verb' will also be singular. The structure of the sentence is based on passive voice. Correct Sentence - More than half the drain has been desilted. 35. Fill in the blank with the most appropriate word. The doctor often made his patients ______ for long. (a) waiting (b) to wait (c) waited (d) wait SSC CPO-SI – 23/11/2020 (Shift-II)

201

YCT

Ans. (d) : In the blank space of the above sentence, 'wait' will be used because in this sentence, the use of 'made' is used as a causative verb. 'Bare infinitive' is used with make. Correct sentence- The doctor often made his patients wait for long. 36. Select the most appropriate option to fill in the blank. We should never ––––– with the rules of driving. (a) temper (b) trifle (c) reckon (d) tamper SSC CGL (Tier-I) – 11/06/2019 (Shift-III) Ans. (d) : The use of word 'tamper' (to interfere) would be appropriate, in the blank space of above sentence. The meaning of other words isTemper :- attitude, trifle, destroy, Reckon:- to calculate an amount, a number etc. Tamper with (Phrasal Verb)- To change or touch (something) especially in a way that causes damage or harm. Correct sentence - We should never tamper with the rules of driving. 37. Select the most appropriate word to fill in the blank. The National war memorial is____ to our Armed Forces. (a) inscribed (b) dedicated (c) honoured (d) devoted SSC CGL (Tier-I) – 06/06/2019 (Shift-III) Ans. (b) : In the blank space of the above sentence, 'dedicated' will be used because it give appropriate meaning. The meaning of other words is not appropriateDevoted - Having great love for somebody/something and being loyal to them. Inscribed - To write or cut words, en+ something Honoured - Great respect and admiration for somebody. Correct sentence– The National war memorial is dedicated to our Armed Forces. 38. Select the most appropriate word to fill in the blank. A painting competition was held in the colony and Kavya was ____ winner in the age group 35 years. (a) publicized (b) supposed (c) described (d) declared SSC CGL (Tier-I) – 06/06/2019 (Shift-III) Ans. (d) : According to the sense of the above sentence, 'declared' will be used in the blank space. The meaning of other words isDescribed - To say what somebody / something is like Publicized - To make something known to the public to advertise something Supposed - To pretend that something is true. Correct sentence– A painting competition was held in the colony and Kavya was declared winner in the age group 3-5 years. Fill in the Blanks

39.

Select the most appropriate word to fill in the blank. In the wake of the recent cross-border tensions, forces have been _____ at strategic locations for immediate action, if required. (a) deported (b) deployed (c) deposited (d) deported SSC CGL (Tier-I) – 07/06/2019 (Shift-I) Ans : (b) The use of 'deployed' would be appropriate, in the blank space of the above sentence, because forces are deployed. The meaning of other words isDeport:- To force somebody to leave a country, because they have broken the law. Deposit :- A sum of money that is given as the first part of a larger payment, or a sum of money that is paid by somebody. Correct sentence– In the wake of the recent crossborder tensions, forces have been deployed at strategic locations for immediate action, if required. 40. Select the most appropriate option to fill in the blank. I sat ______ my life as nothing seemed to be working for me. (a) blessing (b) cursing (c) tormenting (d) invoking SSC CGL (Tier-I) – 12/06/2019 (Shift-I) Ans : (b) According to the sense of the above sentence, 'cursing' would be appropriate in the blank space. Other options do not give proper meaning according to the sense of sentence. The meaning of other options isBlessing - Something that is good or helpful. Tormenting - Intense feeling of suffering, acute mental or physical. Invoking - Request earnestly. (something from somebody) Correct Sentence I sat cursing my life as nothing seemed to be working for me. 41. Select the most appropriate option to fill in the blank. I am quite satisfied that I have not been ____in doing whatever was needful for building up their character. (a) caring (b) negligent (c) affectionate (d) devoted SSC CGL (Tier-I) – 13/06/2019 (Shift-II) Ans. (b) : According to the sense of the above sentence, the use of word 'negligent' would be appropriate in the blank space. The meaning of other words isCaring - Compassionate, lovingness. Affectionate - Having or displaying warmth or affection. Devoted - Dedicated. Correct sentence– I am quite satisfied that I have not been negligent in doing whatever was needful for building up their character.

202

YCT

42.

Select the most appropriate option to fill in the blank. Colours, they say have the power to calm, pacify and relax; they can energies, activate and ______ (a) invigorate (b) involve (c) interest (d) enrage SSC CGL (Tier-I) – 13/06/2019 (Shift-III) Ans : (a) In the blank space, according to the meaning of the above sentence, 'invigorate' would be appropriate. Other options do not make sense, according to the meaning of the sentence. The meaning of other words isInvolve :- Contain as a part. Interest :- A sense of concern with and curiosity about someone Enrage :- Put into a rage, make violently angry. Correct sentence– Colours, they say have the power to calm, pacify and relax; they can energies, activate and invigorate. 43. Select the most appropriate option to fill in the blank. In ancient Greece, women were not allowed to ______ in the Olympic Games. (a) compete (b) collide (c) comply (d) cope SSC CGL (Tier-I) – 13/06/2019 (Shift-III) Ans : (a) In the blank space of the given sentence, 'compete' (compete in a game or a contest) would be appropriate. The meaning of other options is Collide- Crash together with violent impact. Comply- Adopt, follow. Cope- Contend, manage. Correct sentence– In ancient Greece, women were not allowed to compete in the Olympic Games. 44. Select the most appropriate option to fill in the blank. In order to _____ to a new place you may need to adjust to the ways of that culture. (a) Avoid (b) Adopt (c) Adapt (d) Adhere SSC CGL (Tier-I) – 12/06/2019 (Shift-II) Ans : (c) In the blank space of the above sentence, 'adapt' will be used, which means - 'become suitable'. The meaning of other options isAvoid - Keep away from, refrain from doing something Adopt - Take into one's family (take in) Adhere - To stick firmly, (cling) Correct sentence - In order to adapt to a new place you may need to adjust to the ways of that culture. 45. Select the most appropriate option to fill in the blank. Following detailed deliberations, the meeting has been ........ till next week : (a) proposed (b) reviewed (c) cancelled (d) adjourned SSC CGL (Tier-I) – 12/06/2019 (Shift-III) Ans. (d) : In the blank space of the above sentence, 'adjourned (postponed)' would be appropriate, because 'adjourn' is used to postpone the meeting of a court. The meaning of other options is Fill in the Blanks

Proposed- Advise, suggest Reviewed- Examine again Cancelled- No longer planned. Correct sentence – Following detailed deliberations, the meeting has been adjourned till next week. 46. Select the most appropriate option to fill in the blanks. Hima Das, the reigning world junior sprinter who ........ the national record, won the gold in the Federation Cup. (a) holds (b) keeps (c) gets (d) plays SSC CGL (Tier-I) – 12/06/2019 (Shift-III) Ans. (a) : In the blank space of the above sentence, 'holds' (sustain, support) would be appropriate, The meaning of other options is Keep- Continue a certain state, condition or activity Get- Find, receive Play- Participate in games or sport. Correct sentence – Hima Das, the reigning world junior sprinter who holds the national record, won the gold in the Federation Cup. 47. Select the most appropriate option to fill in the blank. The accident victim –––––– to his injuries before he could be taken to the hospital. (a) succumbed (b) subscribed (c) surrendered (d) submitted SSC CGL (Tier-I) – 11/06/2019 (Shift-III) Ans. (a) : According to the sense of the above sentence, 'succumbed' (die) would be appropriate in the blank space. Succumbed- To die or suffer badly an illness / injury. The meaning of other options isSubscribe - Receive or obtain regularly. Surrender - Give up, yielding Submit - Hand over formally. (Present) Correct sentence – The accident victim succumbed to his injuries before he could be taken to the hospital. 48. Select the most appropriate option to fill in the blank. Technology can be ____ and people seem to be more attached to screens than ever before. (a) Addictive (b) infatuated (c) Incorrigible (d) contagious SSC CGL (Tier-I) – 10/06/2019 (Shift-II) Ans : (a) According to the sense of the above sentence 'addictive' (addiction) will be used in the blank space. The meaning of other options isInfatuated - Having a very strong feeling of love or attraction for somebody/ something so that you cannot think clearly. Incorrigible - Not able to be corrected or changed. Contagious - (Of disease) capable of being transmitted by infection. Correct sentence – Technology can be addictive and people seem to be more attached to screens than ever before. 49. Select the most appropriate option to fill in the blank. People of the older generation _____ over the good old school days. (a) remember (b) remind (c) retribute (d) reminisce SSC CGL (Tier-I) – 10/06/2019 (Shift-II)

203

YCT

Ans : (d) The use of word 'reminisce' (to write, think and talk about happy times) would be appropriate in the blank space of the above sentence according to the sense of the sentence. The meaning of other options isRemember - Recall knowledge form memory. (think) Remind - Help somebody remember something, especially something important. Attribute - An abstraction belong to or characters Correct Sentence – People of the older generation reminisce over the good old school days. 50. Select the most appropriate option to fill in the blank. I was frustated at not being able to____ of my old car. (a) devoid (b) depose (c) dispose (d) deal SSC CGL (Tier-I) – 10/06/2019 (Shift-III) Ans. (c) : In the blank space of the above sentence, 'dispose' (settle, resolve) would be appropriate. The meaning of other options isDevoid- Completely wanting or lacking (Barren) Depose- Force to leave (an office) Deal- Handle, manage. Correct sentence- I was frustated at not being able to dispose of my old car. 51. Fill in the blank with the most appropriate word. The committee members were in ____over the budget for the function. (a) difference (b) dissent (c) deference (d) descent SSC CGL (Tier-I) – 07/06/2019 (Shift-III) Ans : (b) According to the sense of the above sentence 'dissent' (disagreement) would be appropriate in the blank space. The meaning of other options isDeference - Respectfulness (courteous regard for people's filings) Difference - The quality of being unlike or dissimilar. Descent- Properties attributable to your ancestry. (origin)_ Correct sentence- The committee members were in dissent over the budget for the function. 52. The burning of the effigy of Ravana on Dussehra –––– the burning of all evils. (a) epitomizes (b) symbolizes (c) intensifies (d) personifies SSC CGL (Tier-I) – 06/06/2019 (Shift-I) Ans. (b) : The word 'Symbolizes' will be used in the blank space of the above sentence because according to the sense of sentence, 'symbolizes' would be appropriate. The meaning of other options isEpitomize - Symbolize Intensify - Escalate, step up. Personify - Represent , body. Correct sentence - The burning of the effigy of Ravana on Dussehra symbolizes the burning of all evils. Fill in the Blanks

53.

Select the most appropriate word to fill in the blank. The state Government argued that it could not _____ the increase in the teachers' salaries as awarded by the court. (a) spare (b) get (c) stand (d) afford SSC CGL (Tier-I) – 04/06/2019 (Shift-II) Ans : (d) According to the sense of the above sentence, the word 'afford' (to do something) would be appropriate in the blank space. The meaning of other options isSpare - More than is needed, desired, excess, Get - Find, acquire Stand - Be upright. Correct sentence- The state Government argued that it could not afford the increase in the teachers' salaries as awarded by the court. 54. Select the most appropriate word to fill in the blank. Scientists at Cambridge University are ___ how plants can give us sustainable energy. (a) scrutinizing (b) investigating (c) inspecting (d) looking SSC CGL (Tier-I) – 04/06/2019 (Shift-II) Ans : (b) 'Investigating' would be appropriate in the blank space of the above sentence because the sense of finding out by scientist is being revealed in the sentence. The meaning of other options isScrutinizing- Examining, seeing. Inspecting- Look over carefully. Looking- Appearing to be as specified, the act of searching visually. Correct sentence- Scientists at Cambridge University are investigating how plants can give us sustainable energy. 55. Select the most appropriate word to fill in the blank. Many items made of ivory were –––– from a dealer in antiques by the custom authorities at the Delhi airport. (a) annexed (b) confiscated (c) hijacked (d) appropriated SSC CGL (Tier-I) – 04/06/2019 (Shift-III) Ans. (b) : 'Confiscated' would be appropriate in the blank space of the above sentence because the sentence is talking about confiscating the item of ivory. Confiscate - Take temporary possession of as a security, by legal authority. (Seize) The meaning of other options isAnnexed - Attached or added, Appropriated - Suitable for a particular person, place, condition. Hijacked - Seize, control of . Correct sentence - Many items made of ivory were confiscated from a dealer in antiques by the custom authorities at the Delhi airport. 56. Select the most appropriate word to fill in the blank. You must apologize ––––– the punishment. (a) escaping (b) to escape (c) escapes (d) escape SSC CPO-SI – 09/12/2019 (Shift-II)

204

YCT

Ans : (b) The use of word 'to escape' is appropriate in the blank space of the above sentence because after apologize (verb), infinitive (to escape) would be appropriate in this sentence. Correct sentence- You must apologize to escape the punishment. 57. Select the most appropriate option to fill in the blank. Due to shortage of time, I was unable ____ the task satisfactorily. (a) in completing (b) for completing (c) from complete (d) to complete SSC CPO-SI – 11/12/2019 (Shift-II) Ans : (d) 'To complete' will be used in the blank space of the above sentence because 'to' preposition' is used with unable. unable to do something Correct Sentence Due to shortage of time, I was unable to complete the task satisfactorily. 58. Select the most appropriate option to fill in the blank. The first thing they did was to –––– the extent of damage done by the rains. (a) repair (b) search (c) ascertain (d) circulate SSC CPO-SI – 12/12/2019 (Shift-II) Ans : (c) The word 'ascertain' would be appropriate in the blank space of the above sentence according to the meaning of the sentence. The meaning of other options isRepair - Mend, restore. Search - Seek, look. Circulate - Move through a space. Correct Sentence The first thing they did was to ascertain the extent of damage done by the rains. 59. Select the most appropriate option to fill in the blank. The Chinese leader's formal Chinese suit ––––– Changpao. (a) will be called (b) is call (c) are called (d) is called SSC CPO-SI – 12/12/2019 (Shift-II) Ans : (d) The use of word 'is called' would be appropriate in the blank space of the above sentence because 'Chinese suit' is a singular noun so the verb will also be singular. Sentence structure is based on passive voice. Note- In Passive voice , the third form of the verb (v3) is used after helping verb. Correct sentence- The Chinese leader's formal Chinese suit is called Changpao. 60. Select the most appropriate option to fill in the blank. The majority –––––– that the country can progress under able leadership. (a) believes (b) is believing (c) believe (d) have believed SSC CPO-SI – 12/12/2019 (Shift-I) Fill in the Blanks

Ans : (a) The word 'believes' would be appropriate in the blank space of the above sentence because 'the majority' is a collective noun singular or plural verb is used according to the sense of sentence. Since here is the sense of unity so singular verb will be used. Correct sentence- The majority believes that the country can progress under able leadership. 61. Select the most appropriate word to fill in the blank. As he crossed the desert he nearly _____ to thirst when he was forced to go without water for four days and five nights. (a) succumbed (b) subsisted (c) survived (d) submerged SSC CHSL (Tier-I) –11/07/2019 (Shift-I) Ans : (a) In the blank space the word 'succumbed' would be appropriate according to the sense of the above sentence. Succumb-(past form-succumbed)- To stop fighting against something. The meaning of other options isSubsist- Support oneself (exist, survive) Survive- Live, remark Submerge- Cover completely or make imperceptible. (drown) Correct sentence- As he crossed the desert he nearly succumbed to thirst when he was forced to go without water for four days and five nights. 62. Select the most appropriate word to fill in the blank. A fresh interest rate cut by the Reserve Bank of India would largely ____ on how the monsoon turns out in the months ahead. (a) depend (b) determine (c) define (d) demonstrate SSC CHSL (Tier-I) –11/07/2019 (Shift-II) Ans : (a) In the blank space of the above sentence, 'depend' will be used because according to the meaning of the sentence, it is being talked about depending on monsoon. Depend on/ somebody/ something (for something) The meaning of other options isDetermine- To find out or come to a decision Define- Show, make or come to a decision Demonstrate- Establish, prove. Correct sentence - A fresh interest rate cut by the Reserve Bank of India would largely depend on how the monsoon turns out in the months ahead. 63. Select the most appropriate option to fill in the blank. The hotel room ____ of odour of leftover food. (a) smoked (b) flavoured (c) reeked (d) traced SSC CHSL (Tier-I) –11/07/2019 (Shift-III) Ans : (c) In the blank space of the above sentence, the use of word 'reeked' (smell) would be appropriate because in the sentence, is talking about the smell of leftover food so the use of 'reeked' would be used. The meaning of other options isSmoked – Fume. Flavoured – Favorite, best loved Traced- An indication that something has been present. Correct sentence- The hotel room reeked of odour of leftover food.

205

YCT

64.

Select the most appropriate word to fill in the blank. Necessary support to_____ the process of sale is being extended by the banks in the consortium. (a) felicitate (b) complicate (c) confuse (d) facilitate SSC CHSL (Tier-I) –10/07/2019 (Shift-I) Ans : (d) According to the sense of the above sentence, 'facilitate' (make easy) would be appropriate in the blank space. The meaning of other options isFelicitate- Congratulate. Complicate- Difficult, complex. Confuse- Distract. Correct sentence - Necessary support to facilitate the process of sale is being extended by the banks in the consortium. 65. Select the most appropriate option to fill in the blank. The Prime Minister was____ a ceremonial welcome. (a) initiated (b) extended (c) expected (d) invited SSC CHSL (Tier-I) –10/07/2019 (Shift-III) Ans : (b) According to the sense of the above sentence in blank space, 'extended' (spread) would be appropriate. The meaning of other options isInitiated- Begin, start an event or prepare the way for Expected- Hoped for. Invited- Have as a guest, remark Correct Sentence - The Prime Minister was extended a ceremonial welcome. 66. Select the most appropriate word to fill in the blank. The annual wholesale price index jumped to its highest level last month posing a fresh _____for policy makers to remain watchful on the price front. (a) chain (b) check (c) challenge (d) choice SSC CHSL (Tier-I) –09/07/2019 (Shift-I) Ans : (c) According to the sense of the above sentence, 'challenge' would be appropriate in the blank space. The meaning of other options isCheck - Make an examination or investigation. Choice - Option, alternation or investigation. Chain - A series of connected things or people. Correct sentence- The annual wholesale price index jumped to its highest level last month posing a fresh challenge for policy makers to remain watchful on the price front. 67. Select the most appropriate word to fill in the blank. The airline had earlier ____ payment only to those in the high-income bracket but now salaries have dried up for everyone. (a) demanded (b) decided (c) deferred (d) deployed SSC CHSL (Tier-I) –09/07/2019 (Shift-I) Fill in the Blanks

Ans : (c) In the blank space of the above sentence, the word 'deferred' (postpone) would be appropriate. The meaning of other options isDemanded - Claim, ask. Deployed - Place troops or weapons in battle formation. Decided - Determine, make up one's mind. Correct sentence- The airline had earlier deferred payment only to those in the high-income bracket but now salaries have dried up for everyone. 68. Select the most appropriate word to fill in the blank. The Chairman advised them to build a bridge between the two companies using _____ and compromise. (a) strength (b) control (c) competition (d) negotiation SSC CHSL (Tier-I) –09/07/2019 (Shift-III) Ans : (d) According to the sense of the above sentence, 'negotiation' (discussion) would be appropriate in the blank space. The meaning of other options isStrength- The property of being physically or mentally strong. Control- The activity of managing. Competition- Challeng, rivalry. Correct sentence - The Chairman advised them to build a bridge between the two companies using negotiation and compromise. 69. Fill in the blank with the most appropriate word. She was____ with joy at winning the talent show. (a) overturned (b) overruled (c) overpowered (d) overwhelmed SSC CHSL (Tier-I) –08/07/2019 (Shift-II) Ans : (d) In the blank space of the above sentence, 'overwhelmed' will be used because the sentence is expressing the feeling of being 'overwhelmed with happiness'. The meaning of other options isOverturned- Upturned (moved around an axis or center) Overruled- Rule against. Overpowered- Overcome by superior force. Correct sentence- She was overwhelmed with joy at winning the talent show. 70. Fill in the blank with the most appropriate word. The man _____ heavy losses in his investments. (a) reaped (b) incurred (c) earned (d) raised SSC CHSL (Tier-I) –05/07/2019 (Shift-I) Ans : (b) According to the sense of the above sentence, 'incurred' (acquire, get) would be appropriate in the blank space. The meaning of other options is Reaped- The act of harvesting crops. Earned- Gain, pull in. Raised- Lifted, upraised. Correct sentence - The man incurred heavy losses in his investments. 71. Fill in the blank with the most appropriate word. During the curfew the army was ____ at strategic locations. (a) employed (b) supplied (c) deployed (d) dispersed SSC CHSL (Tier-I) –05/07/2019 (Shift-I)

206

YCT

Ans : (c) In the blank space of the above sentence, 'deployed' would be appropriate because 'deployed' is used in relation to the deployment of soldiers. The meaning of other options isEmployed :- Busy, engaged. Supplied :- Circulate, issue, providing something. Dispersed :- Spread, distributed. Correct sentence- During the curfew the army was deployed at strategic locations. 72. Fill in the blank with the most appropriate word. The old man was _____ at the hospital after a long illness. (a) recuperating (b) relenting (c) rejoicing (d) relaxing SSC CHSL (Tier-I) –05/07/2019 (Shift-II) Ans : (a) In the blank space of the above sentence, 'recuperating' (recovering) would be appropriate because the sentence is talking about recovering of old man. The meaning of other options isRelenting :- Soften, yield. Rejoicing :- Joy, feel happiness. Relaxing :- Restful, reposeful Correct sentence- The old man was recuperating at the hospital after a long illness. 73. Fill in the blank with the most appropriate word. The soldiers planned to attack at a precise hour so they _____ their watches. (a) assessed (b) exchanged (c) restored (d) synchronized SSC CHSL (Tier-I) –05/07/2019 (Shift-III) Ans : (d) According to the sense of the above sentence 'synchronized' would be appropriate in the blank space. The meaning of other options isAssessed:- To estimate or judge the value, character, etc. Exchanged :- Transaction, interchange. Restored :- Rebuild, bring back into existence. Correct sentence - The soldiers planned to attack at a precise hour so they synchronized their watches. 74. Select the most appropriate words to fill in the blank. Leopards have learned to ____ to semi-urban living near Mumbai and Gurugram. (a) adapt (b) adopt (c) change (d) prepare SSC CHSL (Tier-I) –04/07/2019 (Shift-I) Ans : (a) According to the sense of the above sentence, 'adapt' would be appropriate in the blank space. The meaning of other options isAdopt- Take as one's own. (Accept) Change- To transform or convert, modify. Prepare- Arrange, produce, adapt Correct sentence- Leopards have learned to adapt to semi-urban living near Mumbai and Gurugram. 75. Select the most appropriate word to fill in the blank. Darkness ____ over the forest increasing the danger of wild animals. (a) collapsed (b) descended (c) grounded (d) declined SSC CHSL (Tier-I) –04/07/2019 (Shift-I) Fill in the Blanks

Ans : (b) According to the sense of the above sentence, 'descended' would be appropriate in the blank space. The meaning of the sentences isCollapsed- Damaged, demolished. Declined- Deterioration, recession. Grounded- The solid surface of the earth Correct sentence- Darkness descended over the forest increasing the danger of wild animals. 76. Select the most appropriate option to fill in the blank. Several environmentalists from city conducted a meet to discuss issues related to the _____ pollution levels created by the growing number of pharmaceutical companies. (a) rise in (b) raie in (c) raising of (d) rising from SSC CHSL (Tier-I) 17/03/2020 (Shift-I) Ans. (a) According to the sense of the above sentence, 'rise in' will be used in the blank space because the sentence is talking about increasing pollution levels. Correct sentence- Several environmentalists from city conducted a meet to discuss issues related to the rise in pollution levels created by the growing number of pharmaceutical companies. 77. Select the most appropriate word to fill in the blank. Prince Harry and his wife, Megan Markle announced their decision to ______ from their role as senior royals by leaving United Kingdom and limiting their royal commitments. (a) step up (b) step on (c) step in (d) step back SSC CHSL (Tier-I) 21/10/2020 (Shift-III) Ans. (d) : According to the sense of the above sentence, 'step back' would be appropriate in the blank space. The meaning of other option isStep up - Intensify, increase Step on - Exploit, to place or press the foot on. Step in - Enter, come, arrive Correct sentence- Prince Harry and his wife Megan Markle announced their decision to step back from their role as senior royals by leaving United Kingdom and limiting their royal commitments. 78. In the following question, the sentence given with blank to be filled in with an appropriate word. Select the correct alternative out of the four and indicate it by selecting the appropriate option He _______ office late that day. (a) reached (b) was reached (c) reaching (d) is reach SSC MTS 9-10-2017 (Shift-II) Ans. (a) : In the blank space of the above sentence, 'reached' will be used because it is clear from 'that day' that the sentence is in past tense. Hence 'reached' (V2) will be used. Correct sentence- He reached office late that day. 79. In the following question, the sentence given with blank to be filled in with an appropriate word. Select the correct alternative out of the four and indicate it by selecting the appropriate option.

207

YCT

I asked her if I ––––– borrow her necklace for a day. (b) would (a) could (c) should (d) can SSC MTS 9-10-2017 (Shift-I) Ans : (a) In the blank space of the above sentence, 'could' would be appropriate because 'could' is used in the sense of politely asking for something. Could- used as a more polite from of ''can'' when asking for permission. Note- If principal clause is in past tense, subordinate clause will also be in past. Correct sentence- I asked her if I could borrow her necklace for a day. 80. In the following question, the sentence given with blank to be filled in with an appropriate word. Select the correct alternative out of the four and indicate it by selecting the appropriate option. Sohail was busy when we _______ to see him. (a) went (b) go (c) goes (d) gone SSC MTS 10-10-2017 (Shift-III) Ans. (a) : In the blank space of the above sentence, 'went (second form of go)' would be appropriate. Because if principal clause is in past tense, subordinate clause will also be in past tense (V2) Correct sentence- Sohail was busy when we went to see him. 81. In the following question, the sentence given with blank to be filled in with an appropriate word. Select the correct alternative out of the four and indicate it by selecting the appropriate option When we got home last night, we found that the guest _______. (a) had arrived (b) has arrived (c) is arrived (d) have arrived SSC MTS 10-10-2017 (Shift-II) Ans. (a) : In the blank space of the above sentence, the verb 'had arrived' will be used. Usually two events of the past in a sentence denote then the form of the verb will be in past perfect for the action that has already happened, while for those that will happen later, the verb of action will be in past indefinite. Correct sentence- When we got home last night, we found that the guest had arrived. 82. Select the most appropriate option to fill in the blank. Raja ––––– the house before Hari got there. (a) left (b) will leave (c) leaves (d) had left SSC MTS – 06/08/2019 (Shift-III) Ans. (d) : In the blank space of the above sentence, 'had left' will be used. Because in the sentence two clauses are connected with 'before' then the first clause (the action which is completed earlier) will be used in past perfect, second clause (which action is happened later) will be past indefinite. Hence had + verb3 will be used in the blank space. Correct sentence- Raja had left the house before Hari got there. Fill in the Blanks

83.

Select the most appropriate option to fill in the blank and make a grammatically correct sentence. Nowadays, many home buyers –––––– interest in digital homes. (a) shows (b) showed (c) had shown (d) are showing SSC MTS – 22/08/2019 (Shift-II) Ans. (d) : In the blank space of the above sentence, 'are showing' will be suitable because 'home buyers' is a plural subject so verb will also be in plural. Correct Sentence- Now a days, many home buyers are showing interest in digital homes. 84. Select the most appropriate word to fill in the blank and make a meaningful sentence. The culprits should not be allowed to –––– easily. (a) get in (b) get on (c) get away (d) get up SSC MTS – 09/08/2019 (Shift-I) Ans. (c) : In the blank space of the above sentence, 'get away' will be suitable. The meaning of the other options isGet in - Come, enter Get on - Advance, ascend Get up - Awake, stand. Correct sentence- The culprits should not be allowed to get away easily. 85. Select the most appropriate to fill in the blank. The company ––––– its first all electric car in the Indian market soon. (a) is launching (b) launched (c) launch (d) has launched SSC MTS – 08/08/2019 (Shift-III) Ans. (a) : In the blank space of the above sentence, 'is launching' would be suitable because present continuous tense is used to tell a definite event in the near future. Correct sentence- The company is launching its first all electric car in the Indian market soon. 86. Select the most appropriate option to fill in the blank. When are you ––––– to write to your friend ? (a) gone (b) going (c) goes (d) go SSC MTS – 07/08/2019 (Shift-I) Ans. (b) : In the above sentence, 'present continuous' is used. Hence 'going' will be used in the blank space. (Is / am / are + V + ing) Correct sentence- When are you going to write to your friend? 87. Select the most appropriate option to fill in the blank. He ––––– done it. (a) should have been (b) shouldn't have (c) should had not (d) should not had SSC MTS – 06/08/2019 (Shift-I) Ans. (b) : In the blank space of the above sentence, 'shouldn't have' would be appropriate. 'Shouldn't have' is used in the sense of given sentence. Note- Model+hava+V3 Correct sentence- He shouldn't have done it.

208

YCT

88.

Select the most appropriate option to fill in the blank. I ––––– blood in the hospital tomorrow. (a) have donated (b) was donating (c) will donate (d) had donated SSC MTS – 05/08/2019 (Shift-III) Ans. (c) : In the blank space of the sentence, 'will donate' will be suitable because it is clear from the use of tomorrow in the sentence that the sentence will be in future indefinite only. Correct sentence- I will donate blood in the hospital tomorrow. 89. Select the most appropriate option to fill in the blank. I am always –––– for good books. (a) looking out (b) looking down (c) looking after (d) looking into SSC MTS – 05/08/2019 (Shift-III) Ans. (a) : In the blank space of the above sentence, 'looking out for' will be suitable, because looking out for (phrase) - to be aware. The meaning of other options isLook after - To take care of , be responsible for, Looking down - Dominate, look over, Look into - Examine, check out, Correct sentence- I am always looking out for good books. 90. Select the most appropriate option to fill in the blank. The little puppy was ––– by a speeding car. (a) run away (b) run into (c) run over (d) run at SSC MTS – 05/08/2019 (Shift-II) Ans. (c) : In the blank space of the above sentence, 'run over' will be used, because 'run over' means(injure) which would be appropriate according to the sentence. The meaning of other options isRun away - Escape Run into - Come together (encounter) Correct sentence- The little puppy was run over by a speeding car. 91. Select the most appropriate option to fill in the blank. Teachers would like –––– a long holiday. (a) having been (b) to have (c) have (d) having for SSC MTS – 05/08/2019 (Shift-II) Ans. (b) : In the blank space of the above sentence, 'to have' will be used, because the verb 'have' should be in to infinitive form, hence 'like' will be followed by 'to'. Correct sentence- Teachers would like to have a long holiday. 92. Select the most appropriate option to fill in the blank. I –––– very busy lately. (a) would be (b) have been (c) will be (d) should be SSC MTS – 05/08/2019 (Shift-I) Ans. (b) : In the blank space of the above sentence, 'have been' will be used because the sentence is expressing the sense of lately. Correct sentence– I have been very busy lately. Fill in the Blanks

93.

Select the most appropriate option to fill in the blank. I watched him ––––. (a) to falling (b) to fell (c) fell (d) fall SSC MTS – 02/08/2019 (Shift-III) Ans. (d) : 'Fall' will be used in the blank space of the above sentence because 'fall' is an intransitive verb, do not use infinitive and do not take with object. Hence- 'fall' will be used at the blank space. Correct sentence- I watched him fall. 94. Select the most appropriate option to fill in the blank. He ––––– his portrait in the main hall. (a) had hanged (b) hung (c) hang (d) hanged SSC MTS – 02/08/2019 (Shift-II) Ans. (b) : In the blank space of the above sentence, 'hung' would be appropriate, because the use of 'hung' (v2 form of hang) would be appropriate in the sense of 'hanging' of picture. Correct sentence- He hung his portrait in the main hall. 95. Select the most appropriate option to fill in the blank. ––––– you like some water? (a) Do (b) Would (c) Shall (d) can SSC MTS – 02/08/2019 (Shift-I) Ans. (b) : The above given sentence in blank space, the verb 'would' will be appropriate because 'would' is a past form of the verb 'will' and used to polite request. The correct sentence- 'would you like some water?' Thus, the correct answer is potion (b). 96. Select the most appropriate option to fill in the blank and make a grammatically correct sentence. Shailesh finally decided to take a back seat and let his son ––––– the family business. (a) running (b) to run (c) will run (d) run SSC MTS – 21/08/2019 (Shift-III) Ans. (d) : In the blank space of the above sentence, 'run' will be used. Note - 'Bare infinitive' is used with 'let' and make in causative verb. Correct sentence- Shailesh finally decided to take a back seat and let his son run the family business. 97. Select the most appropriate option to fill in the blank and make a grammatically correct sentence. Rachna was angry with her father because he did not take her to the zoo as he ––––––. (a) promise (b) promises (c) had promised (d) is promising SSC MTS – 20/08/2019 (Shift-I) Ans. (c) : 'Had promised' will be used in the blank space of the above sentence because the sentence is in past tense and 'past perfect tense' is used for past of past. Correct sentence- ''Rachna was angry with her father because he did not take her to the zoo as he had promised.''

209

YCT

98.

Select the most appropriate option to fill in the blank and make a grammatically correct sentence. The players returned to the dilapidated hotel that –––– them. (a) was been allocated to (b) were allocated to (c) had allocated to (d) was allocated to SSC MTS – 19/08/2019 (Shift-III) Ans. (d) : In the blank space of the above sentence, 'was allocated to' would be appropriate because according to singular noun 'hotel,' singular verb 'was' will be suitable. Correct sentence- The players returned to the dilapidated hotel that was allocated to them. 99. Select the most appropriate option to fill in the blank and make a grammatically correct sentence. Maryam –––––– to anyone as she was angry. (a) not spoke (b) did not spoken (c) did not spoke (d) did not speak SSC MTS – 19/08/2019 (Shift-III) Ans. (d) : In the blank space of the above sentence, 'did not speak' will be suitable because 'V1' (first form of verb) is used with 'did' in past indefinite. Rule- The first form of verb (v1) is used with 'do/ does/did.' Correct sentence- Maryam did not speak to anyone as she was angry.'' 100. Select the most appropriate word to fill in the blank and make a meaningful sentence. Suman found a hundred-rupee note ––––– on the road. (a) lied (b) laying (c) lying (d) lies SSC MTS – 19/08/2019 (Shift-III) Ans. (c) : According to the above sentence, the use of 'lying' is appropriate because in the sentence (note lying on the road), the sense of feeling is appearing . Correct sentence- Suman found a hundred-rupee note lying on the road. 101. Select the most appropriate option to fill in the blank and make a grammatically correct sentence. Our neighbours threatened to call the police if we ––––– stop the noise. (a) didn't (b) hasn't (c) doesn't (d) haven't SSC MTS – 19/08/2019 (Shift-II) Ans. (a) : 'Didn't' will be suitable in the blank space of the above sentence, because the first clause of the sentence is in past tense. Hence the second clause will also be in past tense. Note- Helping verb 'did' is used in past indefinite tense. Correct sentence- Our neighbours threatened to call the police if we did not stop the noise. 102. Select the most appropriate option to fill in the blank and make a grammatically correct sentence. Although I had never seen her before, I –––– her from a photograph. (a) am recognizing (b) recognize (c) recognized (d) have recognized SSC MTS – 16/08/2019 (Shift-II) Fill in the Blanks

Ans. (c) : The use of 'recognized' in the blank space of the above sentence would be appropriate because if the sentence denoted two actions of the past, then the action that is happened earlier will be used in past perfect, the action that is happened later will be used in past indefinite. Correct sentence- Although I had never seen her before, I recognized her from a photograph. 103. Select the most appropriate option to fill in the blank and make a grammatically correct sentence. Robert and Rooney aren't very good friends although they –––– each other for a long time. (a) have known (b) are knowing (c) has known (d) knew SSC MTS – 13/08/2019 (Shift-II) Ans. (a) : According to the sense of the above sentence, 'have known' will be suitable in the blank space because the first clause of the sentence is in present tense. Hence the second clause will also be in present tense. 'For a long time' shows the sentence is in present perfect tense. Correct sentence- Robert and Rooney aren't very good friends although they have known each other for a long time. 104. Select the most appropriate option to fill in the blank and make a grammatically correct sentence. The bodies –––– identified as those of two suspected drug dealers. (a) were (b) had (c) have (d) was SSC MTS – 09/08/2019 (Shift-III) Ans. (a) : According to the sense of the above sentence, the verb 'were' will be used in the blank space because the sentence is in passive voice. Since '3rd form of verb' is used. Correct sentence- The bodies were identified as those of two suspected drug dealers. 105. Select the most appropriate option to fill in the blank and make a grammatically correct sentence. You are sure to ––––– if you don't keep to the path. (a) get lose (b) got lost (c) get lost (d) got lose SSC MTS – 09/08/2019 (Shift-II) Ans. (c) : In the blank space of the above sentence, 'get lost' will be used because VIst form is used after infinitive 'to' and 'get lost' is used as a phrase. It is used to tell someone forcefully and quite rudely to go away. Correct sentence- You are sure to get lost if you don't keep to the path. 106. Select the most appropriate word to fill in the blank and make a meaningful sentence. His old car ––––– on the highway. (a) broke up (b) broke out (c) broke in (d) broke down SSC MTS – 09/08/2019 (Shift-II) Ans. (d) : In the blank space of the above sentence, 'broke down' will be suitable while the meaning of the other options is different. Break down- to stop working

210

YCT

Break up - to take apart Break out - begin suddenly and sometimes violently. Break in - Intrude. Correct sentence- His old car broke down on the highway. 107. Select the most appropriate option to fill in the blank and make a grammatically correct sentence. The driver of the car probably holds the key to –––– the crime. (a) solving (b) solves (c) solved (d) solution SSC MTS – 09/08/2019 (Shift-II) Ans. (a) : In the blank space of the above sentence, 'solving' will be suitable because 'key to solving' is used. Note- Key to + doing something Correct sentence- The driver of the car probably holds the key to solving the crime. 108. Select the most appropriate option to fill in the blank. He ––––– for a week when his father came. (a) has been ill (b) had been ill (c) is being ill (d) is ill SSC MTS – 07/08/2019 (Shift-III) Ans. (b) : 'Had been ill' will be used in the blank space of the above sentence, because it is clear from the use of for + a week (time), that given sentence is in past perfect continuous tense. So helping verb (had been) will be used. Rule - As a rule, if two actions of past tense are denoted in a sentence, then for the action which is finished first, verb is in past perfect and for the action that is happened later, verb is in past indefinite. Correct sentence- He had been ill for a weak when his father came. 109. Select the most appropriate option to fill in the blank. She returned the book after she ––––it. (a) reads (b) read (c) had read (d) was read SSC MTS – 07/08/2019 (Shift-II) Ans. (c) : In the blank space of the above sentence, 'had read' will be used. Because usually there are two action in past tense then for the action which is finished first, verb is in past perfect tense and for action that happened later, verb is in past indefinite tense. Rule - 'Past perfect tense' is used for past of past Note- Subject + VII + object + after + subject + had + main vIII + object. Correct sentence- She returned the book after she had read it 110. Select the most appropriate option to fill in the blank. A twelve year old boy drowned in a pit –––– up for the coastal road project. (a) dig (b) digs (c) digging (d) dug SSC MTS – 06/08/2019 (Shift-II) Ans. (d) : In the blank space of the above sentence, phrase verb 'dug up' will be suitable. Correct sentence- A twelve year old boy drowned in a pit dug up for the coastal road project. Fill in the Blanks

111. Select the most appropriate word to fill in the blank. Leading novelists of the nineteenth century _____ for a cause. (a) writes (b) has written (c) are writing (d) wrote SSC GD – 08/03/2019 (Shift-III) Ans. (d) : In the blank space of the above sentence, 'wrote' would be appropriate, because from the (nineteenth century) used in the sentence, it is clear that the sentence will be in past tense. Hence past indefinite verb 'wrote' (v2) will be used. Correct sentence- Leading novelists of the nineteenth century wrote for a cause. 112. Select the most appropriate word to fill in the blank. A major earthquake ______ in India on 8 October 2005 in Uri. (a) occurs (b) has been occurring (c) occurred (d) is occurring SSC GD – 08/03/2019 (Shift-II) Ans. (c) : 'Occurred' will be used in the blank space of the above sentence, because in the above sentence the events of the past have been described. Hence verb 'occurred' (v2) will be used in past indefinite. Correct sentence– A major earthquake occurred in India on 8 October 2005 in Uri. 113. Select the most appropriate option to fill in the blank. The earth _______ round the sun and not the vice-versa. (a) revolving (b) revolves (c) revolve (d) has revolve SSC GD – 02/03/2019 (Shift-II) Ans. (b) : In the blank space of the above sentence, 'revolves' will be used because the sentence with universal truth are kept in present indefinite tense and make singular by using s/es at the end of the verb. Correct sentence- The earth revolves round the sun and not the vice-versa. 114. Select the most appropriate option to fill in the blank. A toddler's brain ______ pretty much fully developed by the age of three. (a) was (b) are (c) were (d) is SSC GD – 02/03/2019 (Shift-I) Ans. (d) : The use of 'is' would be appropriate in the blank space of the above sentence. Because its subject (a toddler brain) is singular. Hence singular verb (is) will be used. Note - Present tense is used to show the general sense. Correct sentence- A toddler's brain is pretty much fully developed by the age of three. 115. Select the most appropriate option to fill in the blank. Researchers say that about 33% of people with diabetes may _______ a skin disorder in their lifetime. (a) have been (b) has been (c) have (d) has SSC GD – 02/03/2019 (Shift-I)

211

YCT

Ans. (c) : In the blank space of the above sentence, 'have' will be used. Because 'Have' (v1) is used after modal verb 'may'. Correct sentence- Researchers say that about 33% of people with diabetes may have a skin disorder in their lifetime. 116. Fill in the blanks with the most appropriate word. Shakespeare _______ this play. (a) write (b) is writing (c) has been writing (d) wrote SSC GD – 11/02/2019 (Shift-III) Ans. (d) : In the blank space of the above sentence, 'wrote' would be appropriate, because it is describes the past event, only simple past would be used here 'wrote' v2 form is used in past indefinite tense. Correct sentence– Shakespeare wrote this play. 117. Select the most appropriate word to fill in the blank. While in Rome we must do as Romans ______. (a) is doing (b) was doing (c) had done (d) do SSC GD – 11/02/2019 (Shift-II) Ans. (d) : According to the sense of the above sentence, 'do' would be appropriate in the blank space because in sentence, the use of the word 'Romans' is in plural number. Hence the use of the verb will also be in plural number. Correct sentenceWhile in Rome we must do as Romans do.

C.

Preposition

118. Fill in the blank with the most appropriate option. This change is responsible for a recent ........... the number of cases of cancer in this age group. (a) rise at (b) rise in (c) rise of (d) rise to SSC MTS 10/05/2023 (Shift-I) Ans. (b) : The appropriate option for the blank space is option (b) 'rise in' because 'rise' take preposition 'in'. Other option are incorrect answer. Thus, the correct answer is option (b). Correct SentenceThis charge is responsible for a recent rice in the number of cases of cancer in this age group. 119. Select the most appropriate option to fill in the blank. I will be joining the office ........... a week's time. (a) in (b) at (c) during (d) by SSC Stenographer (Grade C & D) 11.11.2021 Shift-I Ans. (a) : In the blank space of the above sentence, 'in' will be used. preposition 'in' is used with parts of the day (not specific times), months, year and seasons. It is used as phraseexp:- in a week's time. in two year's time. Correct SentenceI will be joining the office in a week's time. Fill in the Blanks

120. Select the most appropriate option to fill in the blank. He fell ............ the swing in the garden. (a) along (b) below (c) off (d) away SSC Stenographer (Grade C & D) 11.11.2021 Shift-II Ans. (c) : In the blank space of the above sentence preposition 'off' will be used because 'fall off' means – 'fall heavily or suddenly' which gives suitable meaning in the sentence. Hence, Option (c) is correct Correct SentenceHe fell off the swing in the garden. 121. Select the most appropriate option to fill in the blanks. ........ this time next week I will be ........ Mumbai. (a) By; in (b) In; in (c) On; at (d) At; from SSC Stenographer (Grade C & D) 12.11.2021 Shift-II Ans. (a) : In the blank space of the above sentence preposition, 'by' and 'in' will be used. because it gives appropriate meaning. The other options are not suitable. Correct sentence–By this time next week I will be in Mumbai. 122. Select the most appropriate option to fill in the blank. I organised my dressing table by putting all the rubber bands ______ the drawer. (a) for (b) in (c) on (d) at SSC MTS-18/07/2022 Shift-I Ans. (b) : The use of 'in' would be appropriate in the blank space of the above sentence while other prepositions give different meaning. For –: because of (something), used to indicate the place someone. On –: touching and being supported by the top surface of something. At –: used to indicate the place where someone or something is. Correct Sentence– I organised my dressing table by putting all the rubber bands in the drawer. 123. Select the most appropriate option to fill in the blank. The increasing concerns about climate change point to the need for enhanced efforts towards ______ sustained growth. (a) achieved (b) achieve (c) achieving (d) to achieve SSC CGL (Tier-I) 11/04/2022 Shift-I Ans. (c) : The word 'achieving' would be appropriate in the blank space of the above sentence because '(ing) form of verb is used after towards (preposition)'. Correct sentence– The increasing concerns about climate change point to the need for enhanced efforts towards achieving sustained growth.

212

YCT

124. Select the most appropriate option to fill in the blank. Tickets for the event are available ______ the door; there is no advance booking. (a) at (b) in (c) of (d) to SSC MTS-05/07/2022 Shift-III Ans. (a) : In the blank space of the above sentence, 'At' will be used before 'the door', because 'at' preposition is used to indicate a specific place or time. Correct sentence – Tickets for the event are available at the door; there is no advance booking. 125. Select the most appropriate option to fill in the blank. Many people like playing cricket, but _____ my opinion, it is too chaotic. (a) in (b) from (c) for (d) with SSC MTS-13/07/2022 Shift-I Ans. (a) : 'In' will be used in the blank space of the above sentence because 'in' preposition is used before 'My opinion' in the sentence of giving (personal opinion/advice). Correct Sentence – Many people like playing cricket, but in my opinion, it is too chaotic. 126. Select the most appropriate option to fill in the blank. He admonished his listeners ______ their wicked ways. (a) changes (b) changed (c) to change (d) change SSC MTS-07/07/2022 Shift-II Ans. (c) : In the blank space of the above sentence 'to change' will be used because 'to' preposition is used with admonish. Note– Admonish somebody to do something. Correct Sentence– He admonished his listeners to change their wicked ways. 127. Select the most appropriate option to fill in the blank. The artists will arrive ........ an hour. (a) between (b) along (c) within (d) before SSC CGL (Tier-II) 16/11/2020 (3 pm - 5 pm) Ans. (c) : According to the sense of above sentence, 'within' (before the end of) will be used in the blank space. The other options give different meaning. Correct sentence- The artists will arrive within an hour. 128. Select the most appropriate option to fill in the blank. You really need ______ some guidance in Chemistry. (a) take (b) taking (c) to take (d) to be taking SSC CPO-SI – 25/11/2020 (Shift-I) Fill in the Blanks

Ans. (c) : 'To take' will be used in blank space of the above sentence, the verb need + to (infinitive) is always used in the first form (v1st) of the verb. Correct sentence- You really need to take some guidance in Chemistry. 129. Select the most appropriate option to fill in the blank. Vivek has a strong resemblance _________ his grandfather. (a) about (b) to (c) with (d) from SSC CPO-SI – 25/11/2020 (Shift-I) Ans. (b) : In the blank space of the above sentence, 'to' (preposition)' would be appropriate, because 'to' is used with resemblance to denote similarity. Correct sentence- Vivek has a strong resemblance to his grandfather. 130. Fill in the blank with the most appropriate word. Eggs are sold ______ the dozen. (a) by (b) in (c) to (d) at SSC CPO-SI – 24/11/2020 (Shift-I) Ans. (a) : 'By' will be used in blank space of the above sentence because 'by' preposition is used to express the sense of dozen (measurement). Correct sentenceEggs are sold by the dozen. 131. Select the most appropriate words to fill in the blanks. Today computer viruses spread ______ a dizzying speed ______ way of file downloads. (a) by; by (b) at; from (c) at; by (d) in; from SSC CPO-SI – 23/11/2020 (Shift-I) Ans. (c) : In the above sentence, 'At and by' would be appropriate, the preposition 'by' is used for indicating the means of achieving something, and 'at' is used for expressing a particular state or condition. Hence option (c) is correct. Correct sentence- Today computer viruses spread at a dizzying speed by way of file downloads. 132. Select the most appropriate word to fill in the blank. We respect you but we don’t agree ______ your ideas. (a) to (b) for (c) by (d) with SSC CPO-SI – 23/11/2020 (Shift-I) Ans. (d) : In the above sentence, 'with' preposition would be appropriate with agree. The other options are not suitable. Note- (i) agree with someone or something. (ii) agree on a topic or subject. (iii) agree to do something. Correct sentence- We respect you but we don't agree with your ideas. 133. Fill in the blank with the most appropriate word. It is not easy to distinguish a loyal friend ______ a flatterer. (a) among (b) between (c) and (d) from SSC CPO-SI – 25/11/2020 (Shift-II)

213

YCT

Ans. (d) : In the blank space of the above sentence, preposition 'from' will be used. Because 'from' will be suitable with distinguish. The other options are not suitable. Correct sentence- It is not easy to distinguish a loyal friend from a flatterer. 134. Fill in the blank with the most appropriate word. The fan was directly ______ my head. (a) on (b) over (c) at (d) below SSC CPO-SI – 25/11/2020 (Shift-II) Ans. (b) : 'Over' would be appropriate in the blank space of the above sentence, because 'over' preposition is used in the sense of 'above'. Correct sentence- The fan was directly over my head. 135. Fill in the blank with the most appropriate word. ______ Sunny, two others have qualified for the final round of the quiz. (a) Along (b) Besides (c) Between (d) Apart SSC CPO-SI – 23/11/2020 (Shift-II) Ans. (b) : In the blank space of the above sentence, the use of 'besides' would be appropriate because in the sentence, the sense of addition is appearing. The other options give different meaning. Correct sentence- Besides Sunny, two others have qualified for the final round of the quiz. 136. Fill in the blank with the most appropriate word. She has inherited her fondness ______ tennis from her parents. (a) to (b) at (c) for (d) by SSC CPO-SI–23/11/2020 (Shift-II) Ans. (c) : 'For' preposition would be appropriate in the blank space of the above sentence. 'For' preposition is used with fondness. 'Fondness for somebody/something' is used. Correct sentence- She has inherited her fondness for tennis from her parents. 137. Fill in the blank with the most appropriate word. He was born in a business family and has a natural aptitude ______ business. (a) on (b) at (c) for (d) to SSC CPO-SI – 24/11/2020 (Shift-II) Ans. (c) : In the blank space of the above sentence, 'for' would be appropriate because 'for' preposition is used with aptitude. Correct sentence- He was born in a business family and has a natural aptitude for business. 138. Select the most appropriate word to fill in the blank. The boy was leaning against the pillar_____ his hands _____ his pockets. (a) with; from (b) with; in (c) for; into (d) for; in SSC CPO-SI – 11/12/2019 (Shift-I) Fill in the Blanks

Ans : (b) According to the sense of the above sentence, 'with and in' would be appropriate in the blank space. The other options do not give proper meaning . Correct sentence- The boy was leaning against the pillar with his hands in his pockets. 139. Select the most appropriate option to fill in the blanks. The vibrations from wind instruments ––––– always positive and good –––– your breathing and health. (a) is, in (b) are; for (c) were; to (d) was; in SSC CPO-SI – 12/12/2019 (Shift-II) Ans : (b) According to the sense of above sentence, 'are' and for' will be used in the blank space because 'vibrations' is plural noun so plural verb 'are' is correct. 'Good for something or somebody' is used. Correct sentence- The vibrations from wind instruments are always positive and good for your breathing and health. 140. Select the most appropriate option to fill in the blank. Please sign these documents ––––ink so that they can be preserved. (a) with (b) in (c) from (d) by SSC CPO-SI – 12/12/2019 (Shift-I) Ans : (b) In the blank space of the above sentence, 'In' will be used because 'in' preposition is to describe some characteristic intrinsic to the writing itself. Correct sentence- Please sign these documents in ink so that they can be preserved. 141. Select the most appropriate word to fill in the blank. It is said that children who learn to use their body, mind and emotions for their own wellbeing, work ______ the wellbeing of others. (a) within (b) into (c) forwards (d) towards SSC CHSL (Tier-I) 16/10/2020 (Shift-III) Ans. (d) : According to the sense of above sentence, 'towards' will be used in the blank space because it is a preposition which means – in the direction of something. The meaning of other options is – Within –: inside (a certain area or space) Into –: to or toward the inside of (something) Forward –: toward the future. Correct sentence- It is said that children who learn to use their body, mind and emotions for their own wellbeing, work towards the wellbeing of others. 142. Select the most appropriate option to fill in the blank in the given sentence. The area is _____ close circuit television surveillance. (a) about (b) over (c) into (d) under SSC CHSL (Tier-I) 14/10/2020 (Shift-III) Ans. (d) : In the blank space of the above sentence, 'under' would be appropriate which means –: within the group that has. The meaning of other options is – About –: very close to doing something

214

YCT

Over –: downward to a flat or horizontal position. Into –: to or toward the inside of (something) or in the direction of (something) Correct sentence- The area is under close circuit television surveillance. 143. Select the most appropriate option to fill in the blank. Madhuri is bringing ............... her sister's daughter after the death of the child's parents. (a) over (b) out (c) up (d) in SSC CHSL (Tier-I) 17/03/2020 (Shift-III) Ans. (c) : According to the sense of the above sentence, 'Bring up' would be appropriate in the blank space. Bring up means -: to take care of and teach. The meaning of other options are – Bring in –: to cause (someone) to become involved in a process, activity etc. Bring out -: reveal uncover. Correct sentence- Madhuri is bringing up her sister's daughter after the death of the child's parents. 144. Turn the AC _____before you leave the office. (a) out (b) off (c) down (d) in SSC MTS 9-10-2017 (Shift-III) Ans : (b) In the blank space of the above sentence, 'off' will be used because 'on/off' is used to start/off any device. Correct sentence- Turn the AC off before you leave the office. 145. In the following question, the sentence given with blank to be filled in with an appropriate word. Select the correct alternative out of the four and indicate it by selecting the appropriate option Throw a stone _______ the fierce cat. (a) above (b) on (c) at (d) forward SSC MTS 10-10-2017 (Shift-II) Ans. (c) : 'At' (preposition) will be used in the blank space of the above sentence, because 'at' is used to show point of place and 'on' is used to denote the time. Forward means – in a forward direction. Correct sentence- Throw a stone at the fierce cat. 146. In the following question, the sentence given with blank to be filled in with an appropriate word. Select the correct alternative out of the four and indicate it by selecting the appropriate option Mr. Jain has been in this college _______ 2000. (a) for (b) since (c) of (d) off SSC MTS 11-10-2017 (Shift-III) Ans. (b) : 'Since' will be used in the blank space of the above sentence, because the sentence is in present perfect continuous tense. Hence 'since is used for point of time. Note–: In perfect continuous tense, since is used for point of time and 'for' is used with period of time. Correct sentence- Mr. Jain has been in this college since 2000. Fill in the Blanks

147. In the following question, the sentence given with blank to be filled in with an appropriate word. Select the correct alternative out of the four and indicate it selecting the appropriate option. Anita was musing _______ memories of the past. (a) on (b) off (c) for (d) to SSC MTS 11-10-2017 (Shift-II) Ans. (a) : According to the sense of the above sentence, 'On' will be used in the blank space. Musing on memories –: to think carefully about something for a time. Correct sentence- Anita was musing on memories of the past. 148. Select the most appropriate option to fill in the blank. He is addicted to alcohol and exerts a bad influence ––––– his family. (a) in (b) about (c) for (d) on SSC MTS – 06/08/2019 (Shift-III) Ans. (d) : In the blank space of the above sentence, 'On' will be used because on/upon preposition is used with influence. Correct sentence- He is addicted to alcohol and exerts a bad influence on his family. 149. Select the most appropriate option to fill in the blank and make a grammatically correct sentence. Planets do not have light ––––– their own. (a) on (b) by (c) of (d) for SSC MTS – 22/08/2019 (Shift-III) Ans. (c) : In the blank space of the above sentence, 'of' preposition would be appropriate because the sense of possession is appearing. Correct Sentence- Planets do not have light of their own. 150. Select the most appropriate option to fill in the blank and make a grammatically correct sentence. There is no information ––––– the company's profit in the report. (a) about (b) along (c) among (d) with SSC MTS – 22/08/2019 (Shift-III) Ans. (a) : According to the sense of the above sentence, 'about' will be used in the blank space because preposition 'about' is used with 'information'. Correct sentence- There is no information about the company's profit in the report. 151. Select the most appropriate option to fill in the blank. He is a descendant –––– the Mughal royalty. (a) for (b) of (c) in (d) to SSC MTS – 06/08/2019 (Shift-I) Ans. (b) : In the blank space of the above sentence, preposition 'of' would be appropriate, because preposition 'of ' is used with descendant. Correct sentence- He is a descendant of the Mughal royalty.

215

YCT

152. Select the most appropriate option to fill in the blank. I congratulated him –––– his promotion. (a) because of (b) to (c) due of (d) on SSC MTS – 05/08/2019 (Shift-II) Ans. (d) : 'On' preposition would be appropriate in the blank space of the above sentence, because 'On' (preposition) is used with congratulated. Correct sentence- I congratulated him on his promotion. 153. Select the most appropriate option to fill in the blank. The origin of ice-cream dates back –––– nearly half-a-century. (a) to (b) at (c) from (d) into SSC MTS – 05/08/2019 (Shift-I) Ans. (a) : In the blank space of the above sentence, 'to' will be used because 'dates back to' is a phrase which means :- 'to have originated at an earlier time'. Correct sentence- The origin of ice-cream dates back to nearly half-a-century. 154. Select the most appropriate option to fill in the blank. Though I am tall, I feel inferior ––––– others. (a) to (b) than (c) upon (d) from SSC MTS – 02/08/2019 (Shift-III) Ans. (a) : 'To' will be used in the blank space of the above sentence because preposition 'to' is used with 'inferior'. Correct SentenceThough I am tall, I feel interior to others. 155. Select the most appropriate option to fill in the blank and make a grammatically correct sentence. There was a heated discussion ––––– two members of the club. (a) among (b) of (c) in (d) between SSC MTS – 21/08/2019 (Shift-II) Ans. (d) : In the blank space of the above sentence, ' Between' is used because there is talk about two person then 'Between' is used and when there is talk about more than two persons, then 'Among' is used. ex- 1. Father distributed the sweets between his two sons. 2. Father distributed the sweets among his three sons. Correct sentence- There was a heated discussion between two members of the club 156. Select the most appropriate option to fill in the blank and make a grammatically correct sentence. People tend to turn a blind eye ––––– crime against women. (a) to (b) at (c) on (d) in SSC MTS – 20/08/2019 (Shift-II) Ans. (a) : 'To' will be used in the blank space of the above sentence, because 'turn a blind eye to' will be suitable. It is a idiomatic phrase. Correct sentence- People tend to turn a blind eye to crime against women. Fill in the Blanks

157. Select the most appropriate word to fill in the blank and make a meaningful sentence. The children ––––– over the property of their father. (a) quarrelled (b) approved (c) confiscated (d) enjoyed SSC MTS – 16/08/2019 (Shift-III) Ans. (a) : In the blank space of the above sentence, 'quarrelled' will be the used, because preposition over is used with quarrel. Quarrel over (idiom) :- to have an argument or dispute about something. Correct sentence- The children quarrelled over the property of their father. 158. Select the most appropriate option to fill in the blank and make a grammatically correct sentence. A babysitter is somebody who looks ––––– other people's children. (a) for (b) on (c) over (d) after SSC MTS – 16/08/2019 (Shift-III) Ans. (d) : In the blank space of the above sentence, 'after' will be used. 'look after' is a phrasal verb which means :- 'take care of' The other options give different meaning – Look for :- try to find something that you have lost. Look on :- to consider to someone or something in a particular way. Look over :- to examine something or someone quickly. Correct sentence- A babysitter is somebody who looks after other people's children. 159. Select the most appropriate option to fill in the blank and make a grammatically correct sentence. It wasn't easy, but finally we succeeded ––––– finding a solution to the problem. (a) of (b) at (c) in (d) for SSC MTS – 16/08/2019 (Shift-I) Ans. (c) : In the blank space of the above sentence, 'In' preposition will be used. Succeed in (something) :- to complete or accomplish something as one desires hopes, or intends. Correct sentence- It wasn't easy, but finally we succeeded in finding a solution to the problem. 160. Select the most appropriate option to fill in the blank and make a grammatically correct sentence. I'm short –––––– funds at the moment so can I pay you back next week ? (a) of (b) in (c) off (d) at SSC MTS – 13/08/2019 (Shift-III) Ans. (a) : According to the sense of the above sentence, 'of' would be appropriate. 'Short of something' is used. Correct sentence- I'm short of funds at the moment so can I pay you back next week?

216

YCT

161. Select the most appropriate option to fill in the blank and make a grammatically correct sentence. The vase broke ––––– pieces when Simran dropped it. (a) from (b) in (c) among (d) into SSC MTS – 13/08/2019 (Shift-I) Ans. (d) : 'Into' would be appropriate in the blank space of the above sentence. Because 'into' is used with break, according to the sense of the sentence. 'Break into' means ;- 'to divide into pieces, as by bending or cutting.' Correct sentence- ''The vase broke into pieces when Simran dropped it.’’ 162. Select the most appropriate word to fill in the blank. She is highly skilled ––––– dealing with difficult customers. (a) of (b) into (c) at (d) on SSC MTS – 08/08/2019 (Shift-II) Ans. (c) : According to the sense of the above sentence, 'at' preposition would be appropriate in the blank space. 'At' is used with skill. The other options give different meaning. Correct sentence- She is highly skilled at dealing with difficult customers. 163. Select the most appropriate option to fill in the blank. They were astonished ––––– his failure in the competition. (a) in (b) at (c) on (d) from SSC MTS – 07/08/2019 (Shift-III) Ans. (b) : In the blank space of the above sentence, 'at' would be appropriate because fixed preposition 'at' is used with astonished. Correct sentence- They were astonished at his failure in the competition. 164. Select the most appropriate word to fill in the blank. The little girl protested _____ the hypocrisy of her elders. (a) against (b) apart (c) along (d) with SSC GD – 08/03/2019 (Shift-III) Ans. (a) : In the blank space of the above sentence, 'Against' will be used because preposition 'against' is used with protest'. Correct sentence- The little girl protested against the hypocrisy of her elders. 165. Select the most appropriate word to fill in the blank. Tin cans, used for storing food are made _____ electroplating tin onto iron. (a) of (b) with (c) by (d) to SSC GD – 08/03/2019 (Shift-II) Ans. (c) : In the blank space of the above sentence, 'by' will be suitable because preposition 'by' is used with made when mention the name of the company or person that has made something. Correct sentence- Tin cans, used for storing food are made by electroplating tin onto iron. Fill in the Blanks

166. Select the most appropriate option to fill in the blank. Disputes destroy your happiness and distract you ______ your everyday tasks at work and home. (a) off (b) from (c) away (d) at SSC GD – 02/03/2019 (Shift-II) Ans. (b) : In the blank space of the above sentence, 'from' is used (as a preposition) with distract . Distract+ somebody/something from something. Correct sentence- Disputes destroy your happiness and distract you from your everyday tasks at work and home.

D.

Conjunction

167. Fill in the blank with the most appropriate word. The mid-day meal ensures ............ each child studying in a government school has at least one nutritious meal a day. (a) that (b) these (c) this (d) those SSC Stenographer (Grade C & D) 11.11.2021 Shift-II Ans. (a) : In the blank space of the above sentence, 'that' will be used because 'that' is used as a conjunction after ensure. Correct SentenceThe mid-day meal ensures that each child studying in a government school has at least one nutritious meal a day. 168. Select the most appropriate option to fill in the blank. She is as fearless ........... a tigress. (a) also (b) as (c) so (d) like SSC Stenographer (Grade C & D) 11.11.2021 Shift-I Ans. (b) : In the blank space of the above sentence 'as' will be used in the blank space. Because when the similarity of a person with someone 'as something as' is used. 'as........as' is used in pair as a conjunction. Correct SentenceShe is as fearless as a tigress. 169. Select the most appropriate option to fill in the blank. Don't leave the room .......... I am back. (a) until (b) when (c) while (d) since SSC Stenographer (Grade C & D) 15.11.2021 Shift-II Ans. (a) : In the blank space of the above sentence, 'until' will be used. Because according to the meaning of the sentence, condition is being expressed time so conjunction 'until' will be suitable. The other option will not give proper meaning. Correct SentenceDon't leave the room until I am back.

217

YCT

170. Select the most appropriate option to fill in the blank. I won the online game of chess .......... I was out of practice. (a) even if (b) because (c) unless (d) even though SSC Stenographer (Grade C & D) 15.11.2021 Shift-I Ans. (d) : According to the sense of the above sentence, 'even though' would be appropriate, because 'even though' is used as subordinating conjunction in the same way as although/though. 'Even though' similar to although but it makes stronger contrast. The other options do not give proper meaning. Correct SentenceI won the online game of chess even though I was out of practice. 171. Select the most appropriate option to fill in the blank. She was –––––– right nor wrong. (a) whether (b) not (c) neither (d) either SSC CHSL 31.05.2022 Shift-III Ans. (c) : In the blank space of the above sentence, 'neither' is used. Because correlative conjunction 'neither'----- nor' is used together. The use of some correlative conjunctions – No sooner ---------- than Hardly/scarcely --------- when Correct Sentence She was neither right nor wrong. 172. Select the most appropriate option to fill in the blank. _____ did she open her lunch box than everyone gathered around her. (a) As soon as (b) No sooner (c) Just as (d) Hardly SSC CGL (Tier-I) 20/04/2022 Shift-III Ans. (b) : In the blank space of the above sentence, 'No sooner' will be used. Because it is a co-relative conjunction. 'No sooner ------- than' is used together. The use of some co-relative conjunctions – Hardly/scarcely --------- when Not only --------- but also 173. Select the most appropriate option to fill in the blank. _____or not you allow me. I am going to the fair. (a) If (b) Whether (c) Until (d) Unless SSC CGL (Tier-I) 19/04/2022 Shift-II Ans. (b) : In the given blank space 'whether' would be appropriate. Because 'unless/until' itself is a negative word, so after them, 'not' is used with 'whether'. It is used as subordinating conjunction to select one of the two item. Correct sentence– Whether or not you allow me, I am going to the fair. Fill in the Blanks

174. Select the most appropriate option to fill in the blank. The police will regard it as an accident........ any evidence to the contrary is found. (a) otherwise (b) unless (c) whenever (d) still SSC CGL (Tier-II) 16/11/2020 (3 pm - 5 pm) Ans. (b) : According to the sense of the above sentence subordinating conjunction, 'unless' will be used in the blank space. Unless + condition, + action/result. The other options give different meaning. Correct Sentence – The Police will regard it as an accident unless any evidence to the contrary is found. 175. Select the most appropriate word to fill in the blank. Let us know _________ you change your mind. (a) although (b) in case (c) suppose (d) however SSC CPO-SI – 23/11/2020 (Shift-I) Ans. (b) : According to the sense of the above sentence, 'In case' would be appropriate. Note- (i) 'In case' is used in the sense of 'we are prepared for future circumstances'. (ii) 'In case’ is used at the beginning of the sentence in the sense of 'if'. Main clause (Action) + In case + subordinating clause (Reason) Correct SentenceLet us know in case you change your mind. 176. Fill in the blank with the most appropriate word. We cannot delay the project ______ it is urgent. (a) than (b) although (c) as if (d) because SSC CPO-SI – 25/11/2020 (Shift-II) Ans. (d) : According to the sense of the above sentence, conjunction 'because' will be used in the blank space. As a subordinating conjunction 'because' is used to explain the reason. Correct sentence- We cannot delay the project because it is urgent. 177. Fill in the blank with the most appropriate word. The wise try to improve themselves ______ the fools keep blaming their luck. (a) whether (b) still (c) whereas (d) therefore SSC CPO-SI – 24/11/2020 (Shift-II) Ans. (c) : According to the sense of the above sentence, 'whereas' would be appropriate in the blank space. because it is used for showing a fact that is different. The meaning of other option is– Still :- but yet Whether :- used to express a doubt or choice between two possibilities. Therefore :- for that reason. Correct sentence- The wise try to improve themselves whereas the fools keep blaming their luck.

218

YCT

178. Select the most appropriate word to fill in the blank. ______ there were no buses, we had to take a taxi. (a) Even if (b) Hence (c) Since (d) Although SSC CPO-SI – 11/12/2019 (Shift-I) Ans : (c) According to the sense of the above sentence conjunction. 'since' would be appropriate in the blank space. The other options give different meaning. Even if :- Whether or not and has to do with conditions that may apply. Hence :- So, thus Although :- Used for introducing a statement that makes the main statement in a sentence seem surprising. Correct sentence- Since there were no buses, we had to take a taxi. 179. Select the most appropriate option to fill in the blank. This is the reason _______ I don't attend parties. (a) because (b) since (c) so (d) why SSC CPO-SI – 13/12/2019 (Shift-I) Ans. (d) : In the blank space of the above sentence, conjunction 'why' will be used. 'why' is used to express the reason in the subordinate clause. Correct sentence- This is the reason why I don't attend parties. 180. Select the most appropriate option to fill in the blank. He had hardly gone a few kilometers ______ his car broke down. (a) when (b) where (c) that (d) than SSC CPO-SI – 13/12/2019 (Shift-I) Ans. (a) : In the blank space of the above sentence, 'when' will be used because 'when' is used with Hardly, scarcely or barely while 'No sooner' is used with than. Correct sentence- He had hardly gone a few kilometers when his car broke down. 181. Select the most appropriate option to fill in the blank. This fabric is ________ though cheaper than other fabrics. (a) as good as (b) more good (c) as good (d) so good as SSC CPO-SI – 13/12/2019 (Shift-I) Ans. (a) : In the blank space of the above sentence, 'As good as' would be appropriate. Because the other options give different meaning. In option (b) – 'more good' (comparative and positive) con not use together. In option (c) 'As good' is not used in positive degree. In option (d) 'So good as' is used in negative sentence. Correct sentence- This fabric is as good as though cheaper than other fabrics. 182. In the following question, the sentence given with blank to be filled in with an appropriate word. Select the correct alternative out of the four and indicate it by selecting the appropriate option. Fill in the Blanks

She doesn't know –––––– to accept or refuse the proposal. (a) if (b) that (c) whether (d) which SSC MTS 9-10-2017 (Shift-I) Ans : (c) In the blank space of the above sentence, 'whether' would be appropriate. Because 'whether' is used to express doubt which means:- used to show that something is true in either of two cases. (Whether.........or) 'Or' is used with whether. It is a conjunction. Correct sentence- She doesn't know whether to accept or refuse the proposal. 183. Both Sushant ______Rohit are handsome. (a) a well as (b) and (c) or (d) neither SSC MTS 10-10-2017 (Shift-I) Ans : (b) In the blank space of the above sentence, 'and' will be used because 'and' is used with both. It is a co-relative conjunction. some co-relative conjunctions – both ----------- And Not only --------- but also Correct sentence- Both Sushant and Rohit are handsome. 184. Select the most appropriate word to fill in the blank. –––– they are twins, they are worlds apart in their attitude to life. (a) Although (b) Despite (c) Whereas (d) However SSC MTS – 08/08/2019 (Shift-III) Ans. (a) : According to the sense of the above sentence, 'Although' will be used in the blank space because. It is a conjunction that is after used to show a contrast. Other options give different meanings. Correct sentence- Although they are twins, they are worlds apart in their attitude to life. 185. Select the most appropriate option to fill in the blank. We must start now –––– it will be too late (a) until (b) unless (c) or (d) but SSC MTS – 02/08/2019 (Shift-I) Ans. (c) : In the blank space of the above sentence, 'or' will be used. Because the sense of 'or' is being revealed from the sentence. The other options are – Until :- up to the point in time or the event mentioned. Unless :- used to say that something can only happen or be true in a particular situation. But :- used to introduce the word or phrase that contrast with what was said before. Correct sentence- We must start now or it will be too late. 186. Select the most appropriate option to fill in the blank. The entry gates were closed ––––– we could enter in. (a) till (b) before (c) after (d) until SSC MTS – 02/08/2019 (Shift-I)

219

YCT

Ans. (b) : In the blank space of the above sentence, 'before' will be used. The word before/after are in time expression and these are used to indicate when something happens in the past, present or future. So 'Before' is the correct conjunction here. The other options are give different meaning. Till – until After :- At a time later than something. Correct sentence- The entry gates were closed before we could enter in. 187. Select the most appropriate option to fill in the blank and make a grammatically correct sentence. Simran was reading a book ––––– waiting for the bus. (a) while (b) from (c) during (d) at SSC MTS – 20/08/2019 (Shift-II) Ans. (a) : In the blank space of the above sentence, 'while' would be appropriate. 'While' is used, when two task are happening simultaneously. Correct sentence- ''Simran was reading a book while waiting for the bus. 188. Select the most appropriate option to fill in the blank and make a grammatically correct sentence. ––––––– he plays well he has not been selected in the team. (a) Though (b) Because (c) Since (d) Unless SSC MTS – 20/08/2019 (Shift-I) Ans. (a) : According to the sense of the above sentence 'Though' would be appropriate. 'Though' is used to add a fact that makes the previous statement less strong. Correct sentence- Though he plays well he has not been selected in the team. 189. Select the most appropriate option to fill in the blank and make a grammatically correct sentence. Children are not allowed to use the swimming pool ––––– they are accompanied by an adult. (a) unless (b) at least (c) if only (d) in case SSC MTS – 13/08/2019 (Shift-II) Ans. (a) : According to the sense of the above sentence, 'unless' will be used, because It is used in negative sense and told about condition. 'Until' is used in the sense of up to the time or when till. Correct sentence- Children are not allowed to use the swimming pool unless they are accompanied by an adult. 190. Select the most appropriate option to fill in the blank. We cannot quote a price ––––– we are allowed to examine the sample. (a) except (b) unless (c) because (d) whether SSC MTS – 06/08/2019 (Shift-II) Ans. (b) : In the blank space of the above sentence, 'unless' will be used, because the sentence is conditional. The other options are – Except :- only; with the exception. Fill in the Blanks

Because :- for the reason that, due to the fact that. Whether :- used to introduce the first of two. Correct sentence- We cannot quote a price unless we are allowed to examine the sample. 191. Select the most appropriate option to fill in the blank. Gita won't pass the examination and ______ will Rita. (a) nor (b) or (c) either (d) neither SSC GD – 14/02/2019 (Shift-II) Ans. (d) : In the blank space of the above sentence, 'neither' will be used because 'neither' is used in the sense of not one person. It is used in the sense of 'also'. Correct sentence- Gita won't pass the examination and neither will Rita.

E.

Article

192. Select the most appropriate option to fill in the blanks. I am ........ regular reader of ........ Times of India. (a) a; the (b) the; a (c) a; a (d) the; no word required SSC Stenographer (Grade C & D) 12.11.2021 Shift-I Ans. (a) : In the blank space of the above sentence. 'a' and 'the' will be used respectively because article 'a' is used before 'indefinite singular noun' and article 'the' is used before the name of any 'newspaper'. Correct Sentence I am a regular reader of the Times of India. 193. Select the most appropriate option to fill in the blanks. Andaman and Nicobar Islands is ........... union territory in ........... Bay of Bengal. (a) an, a (b) the, the (c) the, no word required (d) a, the SSC Stenographer (Grade C & D) 15.11.2021 Shift-I Ans. (d) : In the given sentence, Article 'a' will be used before 'union' because the pronunciation of union is y(u) consonant sound and article 'the' is used before the bays and Gulfs. Hence option (d) will be correct answer. 194. Select the most appropriate to fill in the blanks. You can go to ________ national Art Gallery but I want to visit _______ zoo first. (a) no word required; the (b) the; the (c) the; no word required (d) a; a SSC CPO-SI – 25/11/2020 (Shift-I) Ans. (b) : In the blank space of the above sentence, 'the' and 'the' will be used respectively because here both the nouns are used in specific form. Correct sentence- You can go to the national Art Gallery but I want to visit the zoo first.

220

YCT

195. Select the most appropriate option to fill in the blank. Educators have ––––– unique responsibility towards the empowerment of their students. (a) a (b) few (c) an (d) some SSC CPO-SI – 12/12/2019 (Shift-II) Ans : (a) In the blank space of the above sentence, 'a' will be used because 'a' is used before consonant sound. Even if they start with 'vowel' alphabet. 'Unique' starts with the consonant sound (u). Hence the use of 'a' is correct. Correct sentence- Educators have a unique responsibility towards the empowerment of their students. 196. Select the most appropriate option to fill in the blank and make a grammatically correct sentence. His clothes were way over –––– top for a sombre occasion. (a) the (b) no article (c) an (d) a SSC MTS – 21/08/2019 (Shift-III) Ans. (a) : In the blank space of the above sentence, definite article 'the' will be used. The other options give different meaning. Over the top - extremely or excessively flamboyant or outrageous. Correct Sentence – His clothes were way over the top for a sombre occasion. 197. Select the most appropriate option to fill in the blank and make a grammatically correct sentence. It was ––––– honour of seema to be selected as a teacher trainer. (a) the (b) No article (c) an (d) a SSC MTS – 21/08/2019 (Shift-II) Ans. (c) : In the blank space of the above sentence, 'An' will be used because when a word is pronounced with a vowel sound 'an' is used. Here the word 'honour' begins with Hindi vowel 'Aa'. Correct sentence- It was an honour of seema to be selected as a teacher trainer. 198. Select the most appropriate option to fill in the blank and make a grammatically correct sentence. Anand Kumar is______ unique teacher. (a) no article (b) an (c) the (d) a SSC MTS – 21/08/2019 (Shift-I) Ans : (d) In the blank space of the above sentence, 'a' will be used as an article. Use of 'a' :- 'A' is used before consonant sound word. eg :- a university. Use of 'an' :- 'An' is used before the vowel sound word. eg :- An hour Correct sentence- Anand Kumar is a unique teacher. 199. Select the most appropriate option to fill in the blank and make a grammatically correct sentence. London is ––––– European city. (a) a (b) No article (c) an (d) the SSC MTS – 13/08/2019 (Shift-I) Fill in the Blanks

Ans. (a) : Article 'A' will be used in the blank space of the above sentence. Use of 'a' before the word whose first letter of pronunciation is 'a' hindi consonant. 'An' is used before vowel sound. eg. An hour A university Correct sentence- London is a European city. 200. Select the most appropriate word to fill in the blank. You have to do ––––– intensive training to become a nurse. (a) the (b) a (c) an (d) one SSC MTS – 08/08/2019 (Shift-II) Ans. (c) : In the blank space of the above sentence, 'article' 'an' will be used because when the first word of pronunciation of Hindi vowel then 'an' is used when the pronunciation of consonant sound, then 'a' is used. Correct sentence- You have to do an intensive training to become a nurse.

F.

Correct use of Pronoun, Adjective & Adverb

201. A sentence has been given with a blank to be filled with an appropriate option. Choose the correct alternative. Farms are increasingly turning to automation for ______ reasons. (a) much (b) most (c) little (d) many SSC MTS 18/05/2023 (Shift-I) Ans. (d) : ‘many’ is used in the blank space of the above sentence, because ‘many’ is an adjective, used before the countable noun (reasons). Note- ‘Much’ is used before the uncountable noun. Thus, the correct answer is option (d). Correct SentenceFarms are increasingly turning to automation for many reasons. 202. A sentence has been given with a blank to be filled with an appropriate option. Choose the correct alternative. The school was _______ than a mile from their home. (a) more (b) most (c) many (d) much SSC MTS 16/05/2023 (Shift-II) Ans. (a) : The above given sentence, in blank space option (a) 'more' would be appropriate because the given sentence is in comparative degree, hence. 'more' is used before 'than'. Much/Many → more → most Thus, the correct answer is option (a). Correct SentenceThe school was more than a mile from their home. 203. Select the most appropritate option to fill in the blank. She is _______ a peacock in the blue satin saree.

221

YCT

(a) So beautiful as (b) as beautiful as (c) beautiful like (d) very beautiful as SSC CGL (Tier-I) 14/07/2023 (Shift-I) Ans. (b) : In the above sentence ‘as beautiful as’ will be used in the blank space. because it gives suitable meaning. Correct SentenceShe is as beautiful as a peacock in the blue stain saree. 204. Select the most appropriate option to fill in the blank. The police will let .......... go after seeing their passes. (a) he (b) they (c) we (d) them SSC Stenographer (Grade C & D) 12.11.2021 Shift-I Ans. (d) : The above given sentence, 'them' will be used in the blank space. Because the objective case pronoun is used after 'let' (verb). Correct SentenceThe police will let them go after seeing their passes. 205. Fill in the blanks with the most appropriate word. The speaker impressed the audience with her ........... speech. (a) fluent (b) fluently (c) effluent (d) fluency SSC Stenographer (Grade C & D) 12.11.2021 Shift-I Ans. (a) : In the blank space of the above sentence, Adjective 'fluent' would be appropriate because 'speech' is a noun. Hence, Adjective 'fluent' is used to qualify before the noun. Correct SentenceThe speaker impressed the audience with her fluent speech. 206. Fill in the blanks with the most appropriate word. The ............ bird collapsed due to the severe heat wave. (a) migratory (b) migrates (c) migration (d) migrate SSC Stenographer (Grade C & D) 12.11.2021 Shift-I Ans. (a) : In the blank space of the above sentence, 'migratory' would be appropriate because 'bird' is a noun. Adjective 'migratory' is used to qualify before the noun. Correct SentenceThe migratory bird collapsed due to the severe heat wave. 207. Fill in the blank with the most appropriate word. Your son hasn't caused any problems; in fact, he has been very ............... (a) cooperative (b) cooperatingly (c) cooperation (d) cooperated SSC Stenographer (Grade C & D) 11.11.2021 Shift-I Ans. (a) : In the given sentence, 'cooperative' is an adjective which is qulifies the adverb (very). Hence in the blank space of the given sentence, 'cooperative' will be used. Fill in the Blanks

• 'Cooperated' is a verb. • 'Cooperation' is a noun. • 'Cooperatingly' is not an adverb, its structure is wrong. Correct SentenceYour son hasn't caused any problems; in fact, he has been very cooperative. 208. Fill in the blank with the most appropriate word. Those who complete the test successfully will be moved into a/an ............ class. (a) advanced (b) rapid (c) crash (d) fast SSC Stenographer (Grade C & D) 12.11.2021 Shift-II Ans. (a) : According to the sense of the above sentence 'Advanced' will be used in the blank space, because 'advanced' is an adjective which is used before the noun. The other objective gives different meaning. Correct sentence–Those who complete the test successfully will be moved into an advanced class. 209. Select the most appropriate option to fill in the blank. I lost my purse ......... between my house and the metro station. (a) somewhere (b) there (c) anywhere (d) wherever SSC Stenographer (Grade C & D) 12.11.2021 Shift-II Ans. (a) : In the blank space of the above sentence, 'somewhere' will be used, because 'somewhere' is an adverb, refer to a place without saying exactly. Correct sentence–I lost my purse somewhere between my house and the metro station. 210. Select the most appropriate option to fill in the blank. It is quite hot today but it was .......... yesterday. (a) most hot (b) hotter (c) hottest (d) more hotter SSC Stenographer (Grade C & D) 15.11.2021 Shift-II Ans. (b) : Conjunction 'but' shows the constract against two sentence so 'adjective 'hotter' would be appropriate in the blank space. Correct SentenceIt is quite hot today but it was hotter yesterday. 211. Fill in the blank with the most appropriate word. As the sun was setting, the clouds were looking beautiful with myriad .......... colours. (a) differential (b) difference (c) different (d) differently SSC Stenographer (Grade C & D) 15.11.2021 Shift-II Ans. (c) : In the blank space of the given sentence, adjective 'different' will be used because 'colours' is a noun. The attribute of a noun is always described by an adjective. Here 'different' is an adjective. Correct SentenceAs the sun was setting the clouds were looking beautiful with myriad different colours.

222

YCT

212. Select the most appropriate option to fill in the blank. The writer declares that –––––– children of their childhood is a criminal act. (a) to rob (b) robbed (c) rob (d) robbing SSC CHSL 24.05.2022 Shift-III Ans. (d) : In the blank space of the above sentence, 'Robbing' would be appropriate. Because participle adjective (verb + ing) is used before children (noun). Correct Sentence The writer declares that robbing children of their childhood is a criminal act. 213. Select the most appropriate option to fill in the blank. He was a ––––– stranger to me. (a) full (b) settled (c) entire (d) complete SSC CHSL 27.05.2022 Shift-III Ans. (d) : In the blank space of the above sentence, 'complete' will be used because it gives the sense of an adjective. Which is qualifying the noun 'stranger' used in the sentence. Correct Sentence He was a complete stranger to me. 214. Select the most appropriate option to fill in the blank. I have visited ______ coaching institutes in your region. (a) many (b) mostly (c) much (d) any SSC MTS-18/07/2022 Shift-I Ans. (a) : In the blank space of the above sentence, 'Many' will be used because 'many' is used as an adjective with countable noun. ' Coaching'. While 'much' is used with uncountable noun. Correct SentenceI have visited many coaching institutes in your region. 215. Select the most appropriate options to fill in the blank. He claims to be the _______ kid in the school. (a) smartness (b) smartest (c) smarting (d) smarter SSC MTS-05/07/2022 Shift-I Ans. (b) : In the blank space of the above sentence (adj.) 'smartest' will be used. because 'The' is used before blank space Hence, 'superlative degree' is used 'the smartest'. The degree of the other options is not suitable. Correct Sentence – He claims to be the smartest kid in the school. 216. Select the most appropriate option to fill in the blank. He is the patient ____ called you in the morning. (a) which (b) whom (c) who (d) that SSC MTS-13/07/2022 Shift-I Fill in the Blanks

Ans. (c) : In the blank space of the above sentence, 'who' will be used because the subject of the sentence is 'He' and relative pronoun 'who' is used for subjective case. Note :– Who is always used before verb while 'verb' is not used after 'whom'. Correct sentence – He is the patient who called you in the morning. 217. Select the most appropriate option to fill in the blank. About three years ago, an old man, ______ name was Paul, used to live in this house. (a) whom (b) who (c) which (d) whose SSC MTS-05/07/2022 Shift-II Ans. (d) : In the blank space of the above sentence, 'whose' will be used because possessive adjective needs here. So 'whose' will be used. Correct Sentence – About three years ago, an old man, whose name was Paul, used to live in this house. 218. Select the most appropriate option to fill in the blank. I am not going to buy ______ toys for you. (a) them (b) these (c) they (d) this SSC MTS-07/07/2022 Shift-II Ans. (b) : According to the sense of the above sentence, 'these' would be appropriate in the blank space because, here, 'these' is an demonstrative adjective which is used before the noun 'toys'. The other options are not suitable. Correct Sentence – I am not going to buy these toys for you. 219. Select the most appropriate option to fill in the blank. He persuaded other towns near Amman to send _____ soldiers to help him. (a) its (b) there (c) their (d) they SSC MTS-08/07/2022 Shift-I Ans. (c) : In the blank space of the above sentence, 'their' will be used because possessive pronoun 'their' is used for soldiers. Correct SentenceHe persuaded other towns near Amman to send their soldiers to help him. 220. Select the most appropriate option to fill in the blank. The priority for the COVID-19 vaccine must be to cover the ____ number in the shortest time. (a) lengthiest (b) latest (c) longest (d) largest SSC Constable GD-10/12/2021 Shift-III Ans. (d) : In the blank space of the above sentence, superlative degree 'largest' will be used because according to the sense of above sentence, it gives proper or suitable meaning.

223

YCT

Note:– Definite article 'the' is used before superlative Ans. (a) : In the blank space of the above sentence, degree. 'myself' will be appropriate, because 'reflexive pronoun' Correct Sentence:– The priority for the COVID-19 of 'I' is 'myself'. vaccine must be to cover the largest number in the Correct sentence- When I solved the sum myself I shortest time. was very happy. 221. Select the most appropriate option to fill in the 226. Fill in the blank with the most appropriate blank. word. The lack of regular academic sessions has ______ of our success depends on our own meant that in 2020 the country has had the ––– efforts. – share of students who have attended (a) many (b) few educational institutions. (c) less (d) much (a) lower (b) low SSC CPO-SI – 24/11/2020 (Shift-I) (c) lowest (d) last SSC CGL (Tier-I) –24/08/2021 (Shift-I) Ans. (d) : In the blank space of the above sentence, Ans. (c) : In the blank space of the above sentence, 'much' will be used because 'much' is used for lowest' will be used because superlative degree (lowest) uncountable noun. is used after article 'the'. Correct sentence- Much of our success depends on Correct sentenceour own efforts. The lack of regular academic sessions has meant that in 227. Fill in the blank with the most appropriate 2020 the country has had the lowest share of students word. who have attended educational institutions. The man ______ book you are reading is my 222. Select the most appropriate option to fill in the father. blank. (a) that (b) whose The ......... chair was discarded by the ten Antonym. (c) who (d) whom (a) broken (b) broking SSC CPO-SI – 25/11/2020 (Shift-II) (c) break (d) broke Ans. (b) : In the blank space of the above sentence, SSC CGL (Tier-II) 16/11/2020 (3 pm - 5 pm) 'whose' will be used, because the possessive case Ans. (a) : In the above sentence, 'broken (verb)' will be (ownership denote) is needed here. used because 'broken' is used as an adjective and noun Correct sentence- The man whose book you are (chair) is used after 'it'. reading is my father. Correct sentence- The broken chair was discarded by 228. Select the most appropriate option to fill in the the ten Antonym. blank. 223. Select the most appropriate option to fill in the It is hard to conceive of a more _____ route blank. The Jupiter is the ______ planet in our solar than the one that this determined traveller system. chose to follow. (a) most big (b) biggest (a) stimulating (b) dashing (c) bigger (d) big (c) daunting (d) provoking SSC Sel. Post Phase-VIII (M.L.) 09.11.2020 (Shift-III) SSC CGL (Tier-I) – 13/06/2019 (Shift-I) Ans. (b) : In the blank space of the above sentence, Ans : (c) According to the sense of the above superlative degree 'Biggest' will be used because sentence, 'Daunting– (discouraging through fear) will superlative degree is used after 'the'. be used in the blank space. The meaning of other Correct sentenceoptions is – The Jupiter is the biggest planet in our solar system. 224. Select the most appropriate option to fill in the Stimulating– Rousing or quickening activity or the blank in the given sentence. senses. The girl _____ stood first in the class is a close Dashing– Lively and spirited. friend. Provoking– Causing or tending to cause. (a) Whose (b) Who Correct sentence- It is hard to conceive of a more (c) Which (d) Whom daunting route than the one that this determined SSC Sel. Post Phase-VIII (H.L.) 09.11.2020 (Shift-I) traveller chose to follow. Ans. (b) : In the blank space of the above sentence, 229. Select the most appropriate option to fill in the 'who' will be used because it is clear from the use of blank. the verb (stood), the pronoun of the subjective case Charlie Chaplin's rise to fame paralleled the (who) will be used. Correct sentence___growth of Hollywood movies in the first The girl who stood first in the class is a close friend. decades of the century. 225. Select the most appropriate option to fill in the (a) delayed (b) progressing blank. (c) sluggish (d) explosive When I solved the sum ______ I was very SSC CGL (Tier-I) – 13/06/2019 (Shift-I) happy. Ans : (d) According to the sense of the above (a) myself (b) itself sentence, 'Explosive' will be used in the blank space (c) oneself (d) herself SSC CPO-SI – 25/11/2020 (Shift-I) which gives complete sense. Fill in the Blanks

224

YCT

The meaning of other options is – Delayed– Not as for along as normal in development. Progressive – Industrial, promoting progress. Sluggish – Moving slowly. Correct sentence- Charlie Chaplin's rise to fame paralleled the explosive growth of Hollywood movies in the first decades of the century. 230. Select the most appropriate option to fill in the blank. Today ___ society is literally poisoning the earth with acid rain. (a) industrialised (b) developing (c) growing (d) cosmopolitan SSC CGL (Tier-I) – 13/06/2019 (Shift-II) Ans. (a) : According to the sense of the above sentence, 'Industrialised' would be appropriate in the blank space. The meaning of the other options is – Developing– Underdeveloped Growing– Development Cosmopolitan– Universal, comprehensive. CORRECT SENTENCEToday industrialised society is literally poisoning the earth with acid rain. 231. Select the most appropriate option to fill in the blank. The roads at 15000 feet are not easily navigable and the air is ____ and freezing. (a) rarefied (b) intensified (c) elevated (d) exalted SSC CGL (Tier-I) – 12/06/2019 (Shift-I) Ans : (a) According to the sense of the above sentence, 'Rarefied' will be used. The meaning of other options is – Intensified :- Made more intense (intense) Elevated :- Raised above the ground. (raises) Exalted :- Of high moral or intellectual value. Correct sentence- The roads at 15000 feet are not easily navigable and the air is rarefied and freezing. 232. Select the most appropriate option to fill in the blank. When you want to expand your vocabulary the best thing to do is to relate a known word with an _____ one and guess the meaning from the context. (a) unclear (b) essential (c) unnecessary (d) unfamiliar SSC CGL (Tier-I) – 12/06/2019 (Shift-II) Ans : (d) According to the sense of the above sentence, 'unfamiliar' would be appropriate in the blank space. The meaning of other options is – Unclear :- Poorly stated or described (ambiguous) Essential :- Basic and fundamental Unnecessary :- Not necessary. Correct sentence- When you want to expand your vocabulary the best thing to do is to relate a known word with an unfamiliar one and guess the meaning from the context. 233. Select the most appropriate option to fill in the blank. She got a lucrative job of a translator because she was –––––– in French. (a) deficient (b) proficient (c) sufficient (d) efficient SSC CGL (Tier-I) – 11/06/2019 (Shift-I) Fill in the Blanks

Ans. (b) : In the blank space of the above sentence, 'proficient' would be appropriate. because here we are talking about the formation in the French language. Note :- The word meaning of proficient and efficient is same but the use of proficient for skillful in a language and efficient for skillful in a any work. Correct sentence- She got a lucrative job of a translator because she was proficient in French. 234. Select the most appropriate option to fill in the blank. The committee reached ––––––– decision regarding the appointment of the chairman. (a) a compatible (b) an exemplary (c) a unanimous (d) an agreeable SSC CGL (Tier-I) – 11/06/2019 (Shift-I) Ans. (c) : In the blank space of the above sentence, 'a unanimous' will be used, because when a decision is taken 'unanimous' the use of 'unanimous decision' is appropriate for that. Correct sentence- The committee reached a unanimous decision regarding the appointment of the chairman. 235. Select the most appropriate option to fill in the blank. A –––––– speech is certainly more effective than one which is verbose. (a) surly (b) sullen (c) lengthy (d) laconic SSC CGL (Tier-I) – 11/06/2019 (Shift-II) Ans. (d) : In the blank space of the above sentence, 'laconic' will be used because laconic is being compared with verbose. The meaning of other options is – Surely :- certainly, for certain. Sullen :- showing a brooding ill humor. (morose) Lengthy :- extended, long. Correct sentence- A laconic speech is certainly more effective than one which is verbose. 236. Select the most appropriate option to fill in the blank. The ship sailed smoothly owing to ––––– winds. (a) boisterous (b) tempestulous (c) favourable (d) turbulent SSC CGL (Tier-I) – 11/06/2019 (Shift-II) Ans. (c) : In the blank space of the above sentence, 'favourable' would be appropriate because the ship is sailed to go easily in favor of the direction due to the wind. The meaning of other options is – Boisterous :- noisy and lacking in restraint or discipline. (unruly) Tempestulous :- stormy. Turbulent :- tumultuous, unquiet. Correct sentence- The ship sailed smoothly owing to favourable wind. 237. Select the most appropriate option to fill in the blank. She performs different roles very convincingly as she is a very_______ actor. (a) virtuous (b) versatile (c) verbose (d) voracious SSC CGL (Tier-I) – 10/06/2019 (Shift-I)

225

YCT

Ans : (b) According to the sense of the above sentence, 'versatile' would be appropriate in the blank space. The other options are – Virtuous :- morally excellent (Righteous) Verbose :- using or containing too many words. Voracious :- edacious, rapacious. Correct sentence- She performs different roles very convincingly as she is a very versatile actor. 238. Select the most appropriate option to fill in the blank. The owner of the house was____ at the watchman for letting in a stranger. (a) impatient (b) indifferent (c) indignant (d) indulgent SSC CGL (Tier-I) – 10/06/2019 (Shift-III) Ans. (c) : According to the sense of the above sentence, 'Indignant' would be appropriate, which means angry about an unfair situation. The other options give different meaning. Impatient :- agitated. Indifferent :- apathetic, uninterested. Indulgent :- tolerant or lenient. Correct sentenceThe owner of the house was indignant at the watchman for letting in a stranger. 239. Fill in the blank with the most appropriate word. The old man wished to donate his _____wealth for the upliftment of the downtrodden. (a) immense (b) elusive (c) intense (d) eminent SSC CGL (Tier-I) – 07/06/2019 (Shift-III) Ans : (a) In the blank space of the above sentence, 'immense' will be used. 'Immense' means 'very great in amount'. The meaning of other options gives different meaning– Intense :-extremely sharp (acute) Eminent :- important Immense :- huge, vast Elusive :- skillful, artful Correct sentence- The old man wished to donate his immense wealth for the upliftment of the downtrodden. 240. Select the most appropriate word to fill in the blank. A number of Indian goods face a –––––– competition from Chinese goods in terms of prices and looks. (a) fierce (b) bold (c) angry (d) powerful SSC CGL (Tier-I) – 04/06/2019 (Shift-III) Ans. (a) : In the blank space of the above sentence, 'fierce' (heavy, terrible) would be appropriate. Fierce :marked by extreme and violent energy. (Ferocious) The meaning of other options is – Bold :- fearless, adventuresome. Angry :- feeling or showing anger. Powerful :- having great power or force. Correct sentence- A number of Indian goods face a fierce competition from chinese goods in term of prices and looks. Fill in the Blanks

241. Select the most appropriate word to fill in the blank. Father didn't believe ______ I said. (a) that (b) which (c) who (d) what SSC CPO-SI – 11/12/2019 (Shift-I) Ans : (d) According to the sense of above sentence, relative pronoun, 'what' would be appropriate. Which gives proper meaning, according to the meaning of the sentence. Correct sentence- Father didn't believe what I said. 242. Select the most appropriate option to fill in the blank. –––––– of these boys is doing anything worthwhile. (a) Both (b) Each (c) None (d) Either SSC CPO-SI – 12/12/2019 (Shift-I) Ans : (c) In the blank space of the above sentence, 'none' will be used in the blank space. The pronoun 'None' is used more than two people. Rule :- 'Both' is used for two persons. 'Each' is used for more than two and in the sense of each. 'None' for more than two persons, in the sense of none. 'Either' for two persons, in the sense of one. Correct sentence- None of these boys is doing anything worthwhile. 243. Select the most appropriate option to fill in the blank. The company has appointed twice as –––– women as men. (a) many (b) more (c) much (d) most SSC CPO-SI – 12/12/2019 (Shift-I) Ans : (a) 'Many' would be appropriate in the blank space of the above sentence. Because it gives appropriate meaning. Rule :- 'Many' is used for plural countable noun in positive degree. 'More' is used for comparative degree as an adjective. 'Much'– Much is used for positive degree as an adjective. 'Most'– Most is used for superlative degree as an adjective. Correct sentence- The company has appointed twice as many women as men. 244. Select the most appropriate option to fill in the blank. Dozens of dogs in Norway have recently been hit by a mysterious and at times, _______ illness, raising concerns among dog owners. (a) dreary (b) desperate (c) suspicious (d) fatal SSC CPO-SI – 13/12/2019 (Shift-II) Ans. (d) : In the blank space of the above sentence, 'fatal' would be appropriate. The meaning of other options is – Dreary :- dull, gloomy. Desperate :- hopeless. Suspicious :- mistrustful, distrustful. Correct sentence- Dozens of dogs in Norway have recently been hit by a mysterious and at times, fatal illness, raising concerns among dog owners.

226

YCT

245. Select the most appropriate option to fill in the blank. Indian films have ____ added nature without its fury into long romantic songs. (a) fretfully (b) fussily (c) frequently (d) formerly SSC CPO-SI – 13/12/2019 (Shift-II) Ans. (c) : According to the sense of the above sentence, 'frequently' would be appropriate in the blank space of the sentence. Correct sentence- Indian films have frequently added nature without its fury into long romantic songs. 246. Select the most appropriate option to fill in the blank. The crisis would______ damage their reputation. (a) further (b) farther (c) either (d) neither SSC CHSL (Tier-I) –11/07/2019 (Shift-III) Ans : (a) According to the sense of the above sentence, 'further' would be appropriate in the blank space. Further means :- Distance. Correct sentence- The crisis would further damage their reputation. 247. Select the most appropriate word to fill in the blank. Any account of the reign of King Harsha would remain ____ without a reference to Hiuen Tsang. (a) incomplete (b) famous (c) eminent (d) unknown SSC CHSL (Tier-I) –10/07/2019 (Shift-I) Ans : (a) According to the sense of the above sentence, 'Incomplete' would be appropriate. The meaning of other options is – Famous :- notable, renowned Eminent :- noteworthy, prominent. Unknown :- anonymous, nameless. Correct sentence- Any account of the reign of King Harsha would remain incomplete without a reference to Hiuen Tsang. 248. Select the most appropriate option to fill in the blank. The employees of our company are not____of their freedom. (a) deployed (b) denied (c) dissipated (d) deprived SSC CHSL (Tier-I) –10/07/2019 (Shift-III) Ans :(d) According to the sense of the above sentence, 'deprived' would be appropriate in the blank space. The meaning of other options is – Deployed :- place troops or weapons in battle formation. Denied :- refuse to recognise. Dissipated :- spread out. Correct sentence- The employees of our company are not deprived of their freedom. 249. Select the most appropriate word to fill in the blank. Government personnel on official trips are ____ to claim reimbursement of their expenses. Fill in the Blanks

(a) entitled (b) incited (c) indicted (d) invited SSC CHSL (Tier-I) –09/07/2019 (Shift-II) Ans : (a) According to the sense of the above sentence, 'Entitled' would be appropriate. The meaning of other options is – Incited :- provoked or stir up. Indict :- accuse formally of a crime. Invite :- call for, quest, pay for. Correct sentence- Government personnel on official trips are entitled to claim reimbursement of their expenses. 250. Select the most appropriate word to fill in the blank. The critics were _____ about his remarks on the efficiency of the government and gave many examples to prove they didn't agree with him. (a) optimistic (b) morose (c) rational (d) cynical SSC CHSL (Tier-I) –09/07/2019 (Shift-II) Ans : (d) According to the sense of the above sentence, 'cynical' would be appropriate in the blank space. The meaning of other options is – Optimistic :- hopeful Morose :- sullen, moody, dark Rational :- logical Correct sentence- The critics were cynical about his remarks on the efficiency of the government and gave many examples to prove they didn't agree with him. 251. Fill in the blank with the most appropriate word. I had ____ pain in the stomach after taking lunch. (a) strict (b) heavy (c) huge (d) acute SSC CHSL (Tier-I) –08/07/2019 (Shift-I) Ans : (d) According to the sense of the above sentence, 'Acute' would be appropriate in the blank space, which means 'very much'. The meaning of other options is Strict :- rigid Heavy :- overweight Huge :- big, large. Correct sentence- I had acute pain in the stomach after taking lunch. 252. Fill in the blank with the most appropriate word. All organizations seek sincere and ____employees. (a) industrious (b) industrial (c) inactive (d) inert SSC CHSL (Tier-I) –08/07/2019 (Shift-I) Ans : (a) According to the sense of the above sentence, 'Industrious' would be appropriate because the sense of hardworking employees is being expressed in the sentence. The meaning of other options is – Industrial :- progressive, developed Inert :- unmoving Inactive :- sluggish Correct sentence- All organizations seek sincere and industrious employees.

227

YCT

253. Fill in the blank with the most appropriate word. .........people are remembered even after death for their good deeds. (a) Virtual (b) Violent (c) Vicious (d) Virtuous SSC CHSL (Tier-I) –08/07/2019 (Shift-II) Ans : (d) In the blank space of the above sentence, 'virtuous' would be appropriate. The meaning of other options is – Violent :- unpeaceful. Virtual :- realistic. Vicious :- evil Correct sentence- Virtuous people are remembered even after death for their good deed. 254. Fill in the blank with the most appropriate word. The landlord is very _____ about cleanliness and finds fault with all his tenants. (a) fabulous (b) fastidious (c) pompous (d) ominous SSC CHSL (Tier-I) –08/07/2019 (Shift-III) Ans : (b) According to the sense of the above sentence, 'Fastidious' would be appropriate in the blank space. The meaning of other options is – Fabulous :- Extremely pleasing, unreal. Pompous :- Grandiloquent, portentous. Ominous :- Baleful, menacious. Correct sentence- The landlord is very fastidious about cleanliness and finds fault with all his tenants. 255. Select the most appropriate word to fill in the blank. There should be _________ punishment for rule breakers. (a) too (b) many (c) severe (d) much SSC CHSL (Tier-I) 12/10/2020 (Shift-III) Ans. (c) : According to the sense of the above sentence, 'severe' will be used. The meaning of other options is – Many :- numerous. (for countable nouns) Much :- a lot of ( for uncountable nouns) Correct sentence- There should be severe punishment for rule breakers. 256. In the following question, the sentence given with blank to fill a proper word. Choose the correct option I dislike eating in a ______ restaurant. (a) Noiseful (b) Noising (c) Noised (d) noisy SSC MTS 9-10-2017 (Shift-III) Ans : (d) In the blank space of the above sentence, 'noisy' would be appropriate because noisy is used for noise. 'Noisy' is an adjective, it will be used before restaurant (Noun) in the sentence. Correct sentence- I dislike eating in a noisy restaurant. 257. In the following question, the sentence given with blank to be filled in with an appropriate word. Select the correct alternative out of the and indicate appropriate option. It was I –––––– saw Rajesh in the play-ground before school. (a) whose (b) whom (c) who (d) which SSC MTS 11-10-2017 (Shift-I) Fill in the Blanks

Ans : (c) In the blank space of the above sentence, 'who' would be appropriate because 'who' is used as relative pronoun subject and 'whom' is used with object. Sub + who Obj + whom Who is a subjective pronoun. Who is used as relative pronoun. 'Who' works as relative clause. Correct sentence- It was I who saw Rajesh in the playground before school. 258. Select the most appropriate option to fill in the blank and make a grammatically correct sentence. The passengers –––– left Paris airport at six arrived in Rome an hour later. (a) which (b) whom (c) who (d) whoever SSC MTS – 09/08/2019 (Shift-I) Ans. (c) : In the blank space of the above sentence, 'who' will be used, because 'who' is subjective pronoun and 'whom' is an objective pronoun, here passengers is a subject, for which subjective pronoun 'who' will be used. Correct sentence- The passengers who left Paris airport at six arrived in Rome an hour later. 259. Select the most appropriate option to fill in the blank. This is one of the ––––– remarkable cases I have handled. (a) more (b) most (c) much more (d) much SSC MTS – 08/08/2019 (Shift-I) Ans. (b) : In the blank space of the above sentence, 'Most' would be appropriate because superlative degree (most) is used after 'one of the' Correct sentence- This is one of the most remarkable cases I have handled. 260. Select the most appropriate option to fill in the blank. My mother is the ––––– inspiration in my life. (a) big (b) bigger (c) biggest (d) most biggest SSC MTS – 05/08/2019 (Shift-III) Ans. (c) : 'Biggest' would be appropriate in the blank space of the above sentence because article 'the' is used before the superlative degree. Note :- Option (d) is wrong because double superlative/ comparative is not used. Correct sentence- My mother is the biggest inspiration in my life. 261. Select the most appropriate option to fill in the blank. The businessman has ––––– connections in the industry. (a) respective (b) respectable (c) receptive (d) responsible SSC MTS – 02/08/2019 (Shift-III) Ans. (b) : According to the sense of the above sentence, 'respectable' would be appropriate. The meaning of other options is – Respective :- respectable own. Receptive :- approachable, amenable. Responsible :- answerable or accountable. Correct sentence- The businessman has respectable connections in the industry.

228

YCT

262. Select the most appropriate option to fill in the blank. My teacher is the ––––– of all. (a) kindest (b) kind (c) kinder (d) kindly SSC MTS – 02/08/2019 (Shift-II) Ans. (a) : In the blank space of the above sentence, 'Kindest' would be appropriate because article 'the' is used before superlative degree. Correct sentence- My teacher is the kindest of all. 263. Select the most appropriate option to fill in the blank and make a grammatically correct sentence. One must keep____ promise. (a) ones (b) one's (c) its (d) the SSC MTS – 21/08/2019 (Shift-I) Ans : (b) In the blank space of the above sentence, pronoun 'one's' would be appropriate because 'one' is used as a subject. 'One's' will be used for possessive Pronoun. Correct sentence- One must keep one's promise. 264. Select the most appropriate option to fill in the blank and make a grammatically correct sentence. Your need is certainly greater than ––––. (a) I (b) my (c) mine (d) me SSC MTS – 20/08/2019 (Shift-III) Ans. (c) : In the blank space of the above sentence, 'mine' will be used, because your need (Abstract noun) is compared to mine (possessive pronoun). 'My' is a possessive adjective while 'mine' is a possessive pronoun. Exp :- This is my school This school is mine. Correct sentence- Your need is certainly greater than mine. 265. Select the most appropriate option to fill in the blank and make a grammatically correct sentence. The more tired you are, the ---------- it is to concentrate. (a) harder (b) hardest (c) more harder (d) hard SSC MTS – 14/08/2019 (Shift-III) Ans. (a) : In the blank space of the above sentence, 'harder' would be appropriate because in this type of sentence both the clause begin with a comparative degree. Structure- The more -------- the more. Correct sentence- The more tired you are, the harder it is to concentrate. 266. Select the most appropriate word to fill in the blank and make a meaningful sentence. When I returned home from work I found my son weeping –––––. (a) sweetly (b) bitterly (c) calmly (d) gently SSC MTS – 13/08/2019 (Shift-II) Ans. (b) : According to the sense of the above sentence, 'bitterly' will be used in the blank space. The meaning of the other options is – Fill in the Blanks

Sweetly :- lovingly, charmingly Calmly :- peacefully, smoothly Gently :- a kind and amiable Correct sentence- When I returned home from work I found my son weeping bitterly. 267. Select the most appropriate option to fill in the blank. Akbar had a ––––– reign than Babar. (a) more long (b) longer (c) longest (d) long SSC MTS – 07/08/2019 (Shift-II) Ans. (b) : In the blank space of the above sentence, 'longer' will be used, because here the region of Akbar is being compared with the region of Babar. Hence comparative degree (longer) will be used before than. Correct sentence- Akbar had a longer reign than Babar. 268. Select the most appropriate option to fill in the blank and make a grammatically correct sentence. If children spend too much time watching television, they can create health problems for __________. (a) oneself (b) themselves (c) himself (d) itself SSC MTS – 14/08/2019 (Shift-I) Ans. (b) : In the blank space of the above sentence, 'themselves' would be appropriate, because the word 'themselves' is classified as a reflexive pronoun. It is the third person plural reflexive pronoun and here is the third person plural subject 'children' is given. Correct sentence- If children spend too much time watching television, they can create health problems for themselves. 269. Select the most appropriate to fill in the blank. I think that childhood is a time when there are ........ways to make life enjoyable. (a) many (b) less (c) little (d) more SSC GD – 14/02/2019 (Shift-I) Ans. (a) : According to the sense of the above sentence, 'many' would be appropriate in the blank space. The other options give different meaning. Correct sentence- I think that childhood is a time when there are many ways to make life enjoyable. 270. Fill in the blanks with the most appropriate words. _______ taught you such lessons as these? (a) What (b) Which (c) Who (d) Whom SSC GD – 11/02/2019 (Shift-III) Ans. (c) : In the blank space of the above sentence, 'who' will be used because subjective case relative pronoun, 'who' is used as a pronoun to describe people. Correct sentence- Who taught you such lessons as these.

229

YCT

G.

Question Tag

271. Select the most appropriate option to fill in the blank. She hardly works on weekends, ______? (a) isn’t she (b) is she (c) doesn’t she (d) does she SSC CGL (Tier-I) 12/04/2022 Shift-I Ans. (d) : In the blank space of the above sentence, 'does she' will be used because 'Hardly' gives negative sense. Hence 'positive question tag' will be used. Correct Sentence – She hardly works on weekends, does she? 272. Fill in the blank with the most appropriate word. You have memorised your speech, ______ you? (a) haven't (b) hasn't (c) didn't (d) couldn't SSC CPO-SI – 25/11/2020 (Shift-II) Ans. (a) : In the blank space of the above sentence, 'haven't' will be used because question tag negative is used for positive sentence. Correct Sentence – You have memorised your speech haven't you? 273. Fill in the blank with the most appropriate option. I am allowed to take photographs inside the church, ______? (a) can’t I (b) shall I (c) am I (d) aren’t I SSC CPO-SI – 24/11/2020 (Shift-II) Ans. (d) : In the blank space of the above sentence, 'aren't I' would be appropriate because 'aren't' is used for question tag. There is no contracted form of 'am not'. Note :- Negative question tag is used for positive sentence. Correct sentence- I am allowed to take photographs inside the church, aren’t I? 274. Select the most appropriate word to fill in the blank in the given sentence. He is your brother _______? (a) isn't it (b) isn't he (c) is it (d) is he SSC CHSL (Tier-I) 15/10/2020 (Shift-I) Ans. (b) : In the blank space of the above sentence, 'isn't he' will be used because negative question tag is used for positive sentence. Correct sentence- He is your brother isn't he?

Ans. (b) : In the blank space of the above sentence, 'Goodness gracious' will be used because goodness gracious is an exclamation of excitement surprise or annoyance. Correct sentence- Goodness gracious : What have you done to your hair? You are looking so odd. 276. Fill in the blank with the most appropriate word. ______! What a frightening experience it was! (a) Hurray (b) Alas (c) Bravo (d) Oh SSC CPO-SI – 24/11/2020 (Shift-II) Ans. (d) : In the blank space of the above sentence, proper word 'oh!' will be used. The other options are– Hurrah :- used for expressing great pleasure. Alas :- used for expressing sadness about something. Bravo :- a word that people shout to show that they have enjoyed something. Oh :- used to express a feeling such as surprise, pain or happiness etc. Correct sentence- Oh! What a frightening experience it was!

I.

Conditional Sentence

277. Select the most appropriate option to fill in the blank. If you had consulted me, I would have ––––––– you. (a) advised (b) advise (c) advises (d) advising SSC CHSL 30.05.2022 Shift-I Ans. (a) : In the blank space of the above sentence, 'advised' will be used. Because 'would have' is used before in the blank space and after it verb3 (advised) is used. If + sub + had + (VIII) + obj., Sub. + Would have + VIII + obj. Correct SentenceIf you had consulted me, I would have advised you. 278. Select the most appropriate option to fill in the blank. I _______ your remaining project if you get groceries for me from the market. (a) would do (b) will do (c) will have done (d) would have done SSC MTS-07/07/2022 Shift-I Ans. (b) : In the blank space of the above sentence, 'will do' will be used because in this type of conditional sentence ' if' clause (subordinate clause) is in present tense and principle clause is used in future tense. Correct SentenceI will do your remaining project if you get groceries for H. Interjection me from the market. 275. Fill in the blank with the most appropriate 279. Select the most appropriate option to fill in the blank. word. If I ______ a doctor, I would have treated you ______! What have you done to your hair? You all. are looking so odd. (a) were (b) would (a) Bravo (b) Goodness gracious (c) am (d) could (c) Bother (d) Hurrah SSC MTS-06/07/2022 Shift-III SSC CPO-SI – 23/11/2020 (Shift-II) Fill in the Blanks

230

YCT

Ans. (a) : 'Were'' will be used in the blank space of the above sentence because it is a conditional sentence and 'were' is used with 'if' clause in imaginary situation. Correct Sentence – If I were a doctor, I would have treated you all. 280. Select the most appropriate option to fill in the blank. If she likes something, she'II buy it ______ much it costs. (a) however (b) whatever (c) also (d) very SSC MTS-08/07/2022 Shift-I Ans. (a) : According to the sense of the above sentence, 'However' will be used . The meaning of other options is – Whatever :- whatsoever. Also :- in addition to. Very :- intensifier. Correct SentenceIt she like something, She'll buy it however much it costs. 281. Select the most appropriate word to fill in the blank. If you_____ your book, I'd like to borrow it. (a) finish (b) finished (c) will finish (d) have finished SSC CPO-SI – 11/12/2019 (Shift-I) Ans : (d) In the blank space of the above sentence, 'have finished' will be used. Here 'if' clause is giving the sense of completeness so 'have finished' will be used. Correct Sentence will be If you have finished your book, I'd like to borrow it. 282. Select the most appropriate option to fill in the blank. She had won the match when we ___ the venue. (a) had been reaching (b) are reaching (c) reached (d) reach SSC CPO-SI – 13/12/2019 (Shift-I) Ans. (c) : In the blank space of the above sentence, 'reached' will be used because if the sentence denoted two actions of the past, then the action is happened earlier will be used in past perfect, the action that is happened later will be used in past indefinite tense. Correct sentence- She had won the match when we reached the venue. 283. Select the most appropriate word to fill in the blank. The pilgrims grew increasinly aware of the ___ of sacred Buddhist texts in the Chinese language and decided to travel to India in search of them. (a) persistence (b) parity (c) preference (d) paucity SSC CHSL (Tier-I) –11/07/2019 (Shift-II) Ans : (d) In the blank space of the above sentence, 'paucity' would be appropriate. The meaning of the other options is – Parity :- functional equality. Persistence :- continuity. Preference :- chance to choose. Fill in the Blanks

Correct sentence- The pilgrims grew increasinly aware of the paucity of sacred Buddhist texts in the Chinese language and decided to travel to India in search of them. 284. In the following question, the sentence given with blank to be filled in with an appropriate word. Select the correct alternative out of the four and indicate it by selecting the appropriate option. Karishma has not been able to recall where _____. (a) did she lived (b) does she lived (c) lived her (d) she lived SSC MTS 11-10-2017 (Shift-II) Ans. (d) : In the blank space of the above sentence, 'she lived' will be used. because 'where' is used as a conjunction (adverb of place) Hence simple past tense will be used after it, because the first clause is talking about past action. Correct sentence- Karishma has not been able to recall where she lived. 285. Select the most appropriate option to fill in the blank and make a grammatically correct sentence. If I ––––– that you were coming over, I would have cancelled my programme. (a) know (b) knows (c) had known (d) am knowing SSC MTS – 19/08/2019 (Shift-I) Ans. (c) : In the blank space of the above sentence, 'had known' would be appropriate. The structure of IIIrd conditional sentence is – [If + subject + had +v3 -------- subject + would have + v3+-------.] Correct sentence- 'If I had known that you were coming over, I would have cancelled my programme.' 286. Select the most appropriate option to fill in the blank and make a grammatically correct sentence. The car-park was full, so we ––––– to look for another place to park. (a) having (b) had (c) has (d) have SSC MTS – 16/08/2019 (Shift-III) Ans. (b) : 'Had' would be appropriate in the blank space of the above sentence. Because the principle clause is in past tense hence subordinate clause will also be in past tense. Correct sentence- The car-park was full, so we had to look for another place to park. 287. Select the most appropriate word to fill in the blank and make a meaningful sentence. As a goodwill –––––, I took some colleagues out for a meal. (a) gesture (b) reminder (c) token (d) indication SSC MTS – 16/08/2019 (Shift-II) Ans. (a) : According to the sense of the above sentence, 'Gesture' would be appropriate in the blank space. The meaning of other option is– Reminder :- a message that helps you remember something.

231

YCT

Token :- something serving as a sign of something else. Indication :- something that serve to indicate or suggest. Correct sentence- As a goodwill gesture, I took some colleagues out for a meal.

J.

Miscellaneous

288. Fill in the blank with the most appropriate choice This .......... piece of mischief by his brother brought a quiet laugh from David. (a) vesical (b) classical (c) whimsical (d) nonsensical SSC MTS 10/05/2023 (Shift-I) Ans. (c) : The appropriate option for the blank space is (c) 'whimsical' (Ecentric) will be used. while other options areVesical means- Urinary classical means- Traditional Nonsensical means- Ridiculous Thus, the correct answer is option (c). 289. Fill in the blank with the most appropriate choice. During treatment, patients lay ......... on the treatment couch for around 3 hrs. (a) repine (b) alpine (c) supine (d) arsine SSC MTS 10/05/2023 (Shift-I) Ans. (c) : The most appropriate option for the blank space is 'supine' 'careless'. other words areRepine means- Lament Alpine means- Steep Thus, the correct answer is option (c). 290. Fill in the blank with the most appropriate option. For months, Drupad could not taste anything he ate as regret layered .......... mud. (a) his mouth like (b) his mouth comparable to (c) his mouth as though (d) his mouth allied to SSC MTS 10/05/2023 (Shift-I) Ans. (a) : The most appropriate option for the blank space 'his mouth like' will be used because. it gives correct meaning for the sentence. The correct sentence isFor Months, Drupad could not taste anything he ate as regret layered his mouth like mud. Thus, the correct answer is option (a). 291. Select the most appropriate option to fill in the blank. In enjoy finding ways to make the most of my budget by being ....... (a) frugal (b) miser (c) lethargic (d) gloomy SSC MTS 08/05/2023 (Shift-III) Fill in the Blanks

Ans. (a) : The most appropriate option for the blank Space 'frugal' will be used because it gives appropriate meaning for the sentence. Meaning for other words are– Miser means – Scrooge Lethargic means – Sluggish Gloomy means – Melancholy Thus, the correct answer is option (a) 292. Select the most appropriate option to fill in the blank. You must _____________ what you cannot cure. (a) embracing (b) endure (c) state (d) take SSC Selection Posts XI-27/06/2023 (Shift-I) Ans. (b) : In the blank space of the above sentence 'endure' would be appropriate , which means 'to suffer something unpleasant, or painful'. Other options give different meanings– Embracing – Take in' State – Situation, Take – Accept 293. Select the most appropriate option to fill in the blank. In 1971, he was awarded the silver ........ of the Society of Arts, London. (a) medieval (b) medal (c) meddle (d) metal SSC MTS 08/05/2023 (Shift-III) Ans. (b) : The most appropriate option for the blank Space (b) 'medal' will be use because it gives appropriate meaning of the sentence. Other words are– Medieval means – out dated Meddle means – Intervene metal means – Mineral These, the correct answer is option (b) 294. A sentence has been given with a blank to be filled with an appropriate option. Choose the correct alternative. When there is not ....... to eat, people starve to death. (a) some (b) a few (c) enough (d) any SSC MTS 08/05/2023 (Shift-III) Ans. (c) : The most appropriate option for the blank space (c) adjective enough' will be used because it modifies a noun 'to eat'. it gives correct meaning of the sentence. 'When these is not enough to eat. 'people Starve to death', These, The correct answer is option (c) 295. A sentence has been given with a blank to be filled with an appropriate option. Choose the correct alternative. Satya Nadella completed his primary education ....... Hyderabad Public School, Begumpet. (a) from (b) over (c) by (d) onto SSC MTS 04/05/2023 (Shift-II)

232

YCT

Ans. (a) : For the above given blank space option (a) 'from' will be used because it shows 'direction of the Noun. Thus, the correct sentence will be 'Satya Nadella completed his primary education from Hyderabad public school, Begumpet.' 296. Select the most appropriate option to fill in the blank. She was moving slowly and ......... yesterday when she fell. (a) firmly (b) vaguely (c) strongly (d) shakily SSC MTS 04/05/2023 (Shift-II) Ans. (d) : The most appropriate option for the blank space (d) 'shakily' which means 'guiveringly' Meaning of the other words are___ Firmly means- Forcibly Vaguely means- Roughly Strongly means- Vigorously Thus, the correct sentence will be 'She was moving slowly and shakily yesterday when she fell'. 297. Select the most appropriate option to fill in the blank. Robin watched as the smoke _________ steadily from the chimney. (a) left (b) rose (c) went (d) hung SSC CGL Mains 26/10/2023 (Shift-I) Ans. (b) : For the above blank space 'rose' will be used because it gives appropriate meaning according to sense of the sentence. Other options give different meaning. Correct Sentence Rabin watched as the smoke rose steadily from the chimney. 298. Select the most appropriate option to fill in the blank. The girl was ________ as she was drenched in the rain. (a) bolt (b) cold (c) gold (d) tight SSC CGL Mains 26/10/2023 (Shift-I) Ans. (b) : In the above sentence 'cold' will be used in the blank space because it gives appropriate meaning according to sense of the sentence. Other options give different meaning. Correct Sentence The girl was cold as she was drenched in the rain. 299. Fill in the blank with the most appropriate word that can be used here based on the given context. We as human beings usually tend to forget that life itself is an accumulated process of _________. (a) recalling (b) testing (c) learning (d) remembering SSC Selection Posts XI-27/06/2023 (Shift-I) Ans. (c) : According to the context of the above sentence in blank space 'learning' would be appropriate Other options give different meanings. Fill in the Blanks

300. Select the most appropriate phrasal verb to fill in the blank. . _________ that thought. It will be a great idea someday. (a) Hold on (b) Hold to (c) Hold onto (d) Hold against SSC CGL Mains 26/10/2023 (Shift-I) Ans. (c) : For the above blank space phrasal verb 'Hold on to' will be used because it gives appropriate meaning according to sense of the sentence. Other options give different meaning. Hold onto – To keep something for some body else or for longer than usual. Correct Sentence Hold onto that thought. It will be a great idea someday. 301. Select the most appropriate option to fill in the blank. You use an ________ to steer a boat. (a) oar (b) or (c) ore (d) are SSC CGL Mains 26/10/2023 (Shift-I) Ans. (a) : In the above sentence 'oar' will be used in the blank space because it give appropriate meaning according to sense of the sentence. Note- Oar - A long pole with a flat part at one end that is used for rowing a boat. ore - rock, earth etc from which metal can be obtained. Correct Sentence You use an oar to steer a boat. 302. Select the most appropriate option to fill in the blank. ________ being an officer, he lives in a very tiny apartment. (a) Except (b) Instead (c) Unlike (d) Despite SSC Selection Posts XI-28/06/2023 (Shift-III) Ans. (d) : In the above sentence option (d) "Despite" will be used in the blank space because it gives appropriate meaning according to sense of the sentence. Other options give different meaning. Correct sentenceDespite being an officer, he lives in a very tiny apartment. 303. Select the most appropriate option to fill in the blank. What to do in the event of a gas ............. (a) leak (b) leek (c) lead (d) leer SSC MTS 04/05/2023 (Shift-II) Ans. (a) : The most appropriate option for the blank space is 'leak' which means 'exposure'. other options are not correct. The correct sentence isWhat to do in the event of a gas leak. Thus, the correct answer is option (a). 304. Select the most appropriate option to fill in the blank. The propaganda was made to invade ________ neighbouring countries.

233

YCT

(a) smaller (b) stranger (c) strongest (d) largest SSC Selection Posts XI-30/06/2023 (Shift-IV) Ans. (a) : In the above sentence, 'smaller' will be used in the blank space because it gives appropriate meaning according to sense of the sentence. Other options give different meaning. Stranger - Unknown Strongest - Powerful Largest - Huge, Giant Correct SentenceThe propoganda was made to Invade smaller neighbouring countries. 305. A sentence has been given with a blank to be filled with an appropriate option. Choose the correct alternative. The person driving the car ......... control over the wheels and knocked her down. (a) loose (b) losing (c) lost (d) lose SSC MTS 02/05/2023 (Shift-I) Ans. (c) : The most appropriate option for the blank space 'lost' (c) because the sentence is in past Indifinite tense. lost lost lost The person driving the car lost control over the wheels and knocked her down. 306. Select the most appropriate option to fill in the blank. Police used ................ evidence to convict a victim who was innocent. (a) stunted (b) authentic (c) hateful (d) fabricated SSC MTS 02/05/2023 (Shift-I) Ans. (d) : The most appropriate option for the blank space is option (d) [Fabricated] because sentence is in past Indifinite tense which gives correct meaning. other words are____ Stunted means- Immature Authentic means- Reliable Hateful means- Disgusting Thus, the correct answer is option (d) 307. Fill in the blank with the most appropriate choice. His intent was the become sole lord and to ........ his tiny principality. (a) subsidize (b) jeopardize (c) methodize (d) aggrandize SSC MTS 02/05/2023 (Shift-I) Ans. (d) : For the above given blank space, the most appropriate answer is option (d) aggrandize which means 'embellish'. The correct sentence will be'His intent was the become sole lord and to aggrandize his tiny principality.' other words are____ Subsidize means- bonus, aid Jeopardize means- Threaten Methodize means- arrange, Marshal Fill in the Blanks

308. Select the most appropriate idiom to fill in the blank. Despite their differences, the two politicians decided to ________ and work together for the greater good. (a) burn the midnight oil (b) bark up the wrong tree (c) bury the hatchet (d) blow off steam SSC CGL Mains 26/10/2023 (Shift-I) Ans. (c) : For the above blank space idiom, bury the hatchet will be used which means 'to make peace' gives an appropriate meaning. Other options give different meaning. Burn the midnight oil - To study late might. Bark up the wrong tree - to make the wrong choice. Blow off steam - to release your stress or frustration. Correct Sentence Despite their differences, the two politicians decided to bury the hatchet and work together for the greater good. 309. Select the most appropriate option to fill in the blank. Wheat is widely ........ but the output is not large. (a) gross (b) groan (c) grown (d) grilled SSC MTS 02/05/2023 (Shift-I) Ans. (c) : For the above given blank space 'the most appropriate option is (c) 'grown' will be used because sentence is in passive voice. The correct sentence is'Wheat is widely grown but the output is not large'. Thus, the correct answer is option (c). 310. Select the most appropriate option to fill in the blank. Since these people used tobacco to _______ their deities, the herb itself was one of the instruments of godless, false religions. (a) trace (b) propitiate (c) expatiate (d) militate SSC MTS 19/05/2023 (Shift-III) Ans. (b) : According to the sense of the above sentence in blank space option (b) ‘propitiate’ would be appropriate, which mans – ‘Satisfy’. Meaning of other optionTrace – Notch, Expatiate – Sermonize, Militate – Fight, Interfere, 311. Select the most appropriate option to fill in the blank. I hope you ___________ what I taught you today. (a) divided (b) understood (c) cleared (d) summoned SSC Selection Posts XI-30/06/2023 (Shift-IV) Ans. (b) : In the above sentence, underlined will be used in the blank space because it gives appropriate meaning according to sense of the sentence.

234

YCT

Divided - Separate Cleared - Processed Summoned - Denominate Correct SentenceI hope you understood what I taught you today. 312. Select the most appropriate option to fill in the blank. There's plenty here to trap the unwary and baffle even the _______ gambler! (a) exenterate (b) reiterate (c) inveterate (d) alliterate SSC MTS 19/05/2023 (Shift-III) Ans. (c) : According to the sense of the above sentence in blank space ‘inveterate’ would be appropriate. Other options give different meaningsExenterate – clear out, Reiterate – Revision, Alliterate – (a phrase or line of verse) contain words which begin with the same round or letter. 313. Select the most appropriate option to fill in the blank. That night the dinner was special, and the _________ himself came out to read out the menu list. (a) guest (b) chef (c) public (d) servant SSC Selection Posts XI-28/06/2023 (Shift-III) Ans. (b) : In the above sentence option (b) "chef" will be used in the blank space because it gives appropriate meaning according to sense of the sentence. Other options give different meaning. Correct sentenceThat night the dinner was special, and the chef himself came out the menu list. 314. Select the most appropriate option to fill in the blank. Her notoriety increased after she single _________ shot down a Rhodesian army helicopter. (a) handedly (b) stranded (c) dose (d) plainly SSC MTS 19/05/2023 (Shift-III) Ans. (a) : According to the sense of the above sentence in blank space ‘handedly’ would be appropriate, Because it is an adverb, which means – ‘Without help from anyone else’. Other options give different meanings. 315. Select the most appropriate option to fill in the blank : Computers would replace the _______ technique of creating an image on paper, transferring it to transparent cells and again onto film. (a) audacious (b) austere (c) archaic (d) assuage SSC MTS 19/05/2023 (Shift-III) Ans. (c) : According to the sense of the above sentence in blank space ‘archaic’ (adj.) would be appropriate. Other options give different meaningsAudacious – Brave Austere – staunch, Tedious, Assuage – Soothe, Calm, Fill in the Blanks

316. Fill in the blank with the most appropriate option. Sam knew who those people were; the figure in front was ______ his cousin. (a) at mistake (b) unmistakably (c) mistaken of (d) by mistake SSC MTS 18/05/2023 (Shift-I) Ans. (b) : According to the sense of the above sentence in blank space ‘unmistakably’ would be appropriate, It is an adverb. Other options give different meanings. 317. Select the most appropriate option to fill in the blank. Participate in a mock interview to prepare for ______ on your feet and formulating answers under pressure. (a) cursing (b) building (c) thinking (d) blocking SSC MTS 18/05/2023 (Shift-I) Ans. (c) : According to the sense of the above sentence in blank space ‘thinking’ would be appropriate because ‘interview to prepare for thinking’ gives the proper meaning of this sentence. Other options give different meanings. 318. Select the most appropriate option to fill in the blank. The Thar desert is __________ in the state of Rajasthan. (a) bounded (b) seen (c) placed (d) situated SSC Selection Posts XI-30/06/2023 (Shift-IV) Ans. (b) : In the above sentences, 'situated' will be used in the blank space because 'in' preposition is used with situated. Other options give different meaning. Correct SentenceThe thar desert is situated in the state of Rajasthan. 319. Select the most appropriate option to fill in the blank. Sometimes we wish for things, but we do not think about their ______. (a) hazards (b) copiousness (c) fractions (d) consequences SSC MTS 16/05/2023 (Shift-II) Ans. (d) : According to the sense of the above given sentence in blank space 'consequences (noun) would be used. 320. A sentence has been given with a blank to be filled with an appropriate option. Choose the correct alternative. ________ from humans, lions are the key natural predators of elephants. (a) Along (b) Rather (c) Aside (d) Beside SSC MTS 16/05/2023 (Shift-II) Ans. (c) : According to the sense of the above passage, in the blank space the word 'Aside' would be used, which means 'besides' or the same as apart from.'

235

YCT

321. Select the most appropriate option to fill in the blank. The researcher team took air ______ from the room every five minutes to rule out the possibility of a virus. (a) samples (b) droops (c) slices (d) instances SSC MTS 15/05/2023 (Shift-I) Ans. (a) : According to the sense of the above sentence for blank space 'samples' would be appropriate. Meaning of other options– Droops :- Dwindle Slices :- Piece Instances :- Incidents, Example, 322. Select the most appropriate option to fill in the blank. Disturbing emotions begin to build into fear, and then ______. (a) boldness (b) server (c) assurance (d) terror SSC MTS 15/05/2023 (Shift-I) Ans. (d) : According to the sense of the above sentence, 'the most appropriate options is (d) 'terror' which means– 'Dread' Meaning of other options – Boldness :- courage, Server :- Liege, Assurance :- Conviction, 323. Select the most appropriate option to fill in the blank. The duke had capacity, but his life was so ______ that what influence he had upon the king was for evil. (a) lewd (b) scrumptious (c) seditious (d) ambitious SSC MTS 12/05/2023 (Shift-III) Ans. (a) : In the blank space of above given sentence option (a) 'lewd' (adjective) will be used, because 'so' is used an adverb, qualifying the adjective 'lewd'. Hence, the correct answer is option (a) 324. Select the most appropriate option to fill in the blank. They probably subsisted on insects and other small _______ invertebrates and perhaps even on aquatic vertebrates. (a) reverent (b) skittish (c) terrestrial (d) recluse SSC MTS 12/05/2023 (Shift-III) Ans. (c) : According to the sense of the above sentence in blank space the word 'terrestrial' would be used which means – 'relating to the planet earth , lining or existing on the land rather than in the sea or air.' meaning of other options – Reverent – Respectful Skittish – Unstable, Recluse – stoic, 325. Select the most appropriate option to fill in the blank. We cannot _______ a religion or a political party that ignores the rights of half the human race. (a) condemn (b) disseminate (c) forgive (d) desiccate SSC MTS 12/05/2023 (Shift-III) Fill in the Blanks

Ans. (c) : In the above sentence option (c) 'forgive' (V1) will be used in the blank space, which means- 'Pardom'. Meaning of other options– Condemn – Rebuke, Disseminate – Expand Desiccate – Dwindle, 326. Select the most appropriate option to fill in the blank. This initiative has been sharply criticized as grossly inefficient and _______. (a) Capricious (b) Counterproductive (c) Canonical (d) Calumny SSC MTS 12/05/2023 (Shift-III) Ans. (b) : In the above sentence option (b) 'Counterproductive' will be used in blank space which means– having on effect that is the opposite of what you desire. The correct sentence– 'This initiative has been sharply criticized as grossly inefficient and counterproductive.' 327. Fill in the blanks with the correct homophone. His eyes ______ up with fascination when he saw his idol standing in front of him. (a) Lye (b) Lit (c) Lite (d) Life SSC GD 31/01/2023 (Shift-II) Ans. (b) : The correct homophone 'lit' (illuminated) will be used in the blank space of the above sentence because it is giving proper meaning according to the sense of the sentence. Other option give different meaning. Lay - a strong solution of sodium or potassium hydroxide, Lite - Mild Life - Spirit, existence Correct SentenceHis eyes lit up with fascination when he saw his idel standing in front of him. 328. Select the most appropriate option to fill in the blank. The _____ of this beautiful garden is so exciting. (a) Site (b) Slight (c) Cite (d) Sight SSC GD 27/01/2023 (Shift-I) Ans. (d) : In the blank space of the above sentence 'Sight' (view) will be used because it is giving proper meaning according to the sense of the sentence. Other options give different meanings. Meaning of the other optionsSite Venue, Place Slight Little, Some Cite Mention Correct SentenceThe sight of this beautiful garden is so exciting. 329. Select the most appropriate option to fill in the blank. Janaki remains _______ while talking about her dark past.

236

YCT

(a) Gracious (c) Dignified

(b) Cautious (d) Turbulent SSC GD 27/01/2023 (Shift-I) Ans. (b) : In the blank space of the given sentence 'cautious' (Aware) is used because it is giving proper meaning according to the sense of the sentence. Other options give different meanings. Meaning of other optionsGracious Humble Dignified Arrogant Turbulent Unstable, Stormy Correct SentenceJanaki remains Cautious while talking about her dark part. 330. Select the most appropriate option to fill in the blank. APJ Abdul Kalam was born in a middle class family. He was a short boy _______ ordinary looks. (a) with either (b) with a (c) with rather (d) neither SSC GD 23/01/2023 (Shift-IV) Ans. (b) : According to the sense of the sentence option (b) 'with a' is used in the blank space of sentence While the other options give different meaning. 331. Select the most appropriate option to fill in the blank. He is usually calm but today he appears rather ______. (a) quiet (b) undisturbed (c) disturbed (d) relaxed SSC GD 23/01/2023 (Shift-IV) Ans. (c) : In the blank space of the given sentence option (c) disturbed (upset) will be used because it is giving proper meaning according to the sense of the sentence. Other options give different meanings. quit Leave, forsake Undisturbed Peaceful, Calm Relaxed Comfortable, ease Correct SentenceHe is usually calm but today he appears rather disturbed. 332. Select the most appropriate option to fill in the blank. The minister reached the venue of the programme before time which __________ wonder and awe among the organisers. (a) Strengthened (b) Draped (c) Created (d) Lagged SSC GD 16/01/2023 (Shift-III) Ans. (c) : Created (generated) will be used in the blank space of the above sentence because it is giving proper meaning according to the sense of the sentence. Other gives different meanings. Strenthened Fortify Draped Covered Lagged Straggle, Fall behind Fill in the Blanks

333. Select the most appropriate option to fill in the blank. His seat was the ______ in our row. (a) Farthest (b) Near to (c) Further (d) Far SSC GD 16/01/2023 (Shift-III) Ans. (a) : In the blank space of the above sentence option (a) Farthest (far, utmost) will be used because it is giving proper meaning as per the sense of the sentence. Other options give different meanings. Near to - Close to Further - Ahead Far - Away Currect SentenceHis seat was the farthest in our row. 334. Select the most appropriate option to fill in the blank. ______ is the scientific study of material remais like relics and monuments of past human life. (a) Geology (b) Theology (c) Biology (d) Archaeology SSC GD 13/02/2023 (Shift-I) Ans. (d) : In the blank space of the above sentence, 'Archeology' would be appropriate. The other options do not give the proper sense. Geology :- a science that deals with history of the earth as recorded in rocks. Theology :- divinity. Biology :- all the plant and animal life of a particular region. 335. Select the most appropriate option to fill in the blank. My mother was ______ at my negligence and scolded me. (a) Energetic (b) Furious (c) Thrilled (d) Pleased SSC GD 12/01/2023 (Shift-II) Ans. (b) : 'Furious' will be used is the blank space of the given sentence because it is giving proper meaning according to the sense of the sentence. Other option give different meaning. Energetic - Active, likely, Thrilled - excited Pleased - happy, delighted Correct sentenceMy mother was furious at my negligence and scolded me. 336. Select the most appropriate option to fill in the blank. Truthfully speaking, the victories of peace and harmony have a more _______ impact on human civilisation than the so-called victories of war. (a) Superior (b) Transient (c) Lasting (d) Greater SSC GD 10/01/2023 (Shift-I) Ans. (c) : 'Losting' (long-term) will be used in the blank space of the given sentence because it is giving proper meaning according to the sense of the sentenec. Other options give different meanings.

237

YCT

Superior - Better, Senior Transient - Momentary Greater - Major, higher Correct sentenceTruthfully speaking, the victories of peace and harmony have a more lasting impact on human civilization that the so-called victories of war. 337. Select the correct word for the blanks in the given sentence. Sivakashi is famous for its _________ box industry, and I am unable to find a _______ for my purple dress. (a) Sweat (b) match (c) Chatter (d) Bread SSC GD 10/01/2023 (Shift-I) Ans. (b) : 'Match' will be used in the blank space of the given sentence because it is giving proper meaning according to the sense of the sentence. Other options give different meanings. Sweat - Melodious, ducet Chatter - Rubbish Bread - Loof, Livelihood Correct SentenceSivkashi is famous for its match box industry, and I am unable to find a match for my purple dress. 338. Select the most appropriate option to fill in the blank. The teacher ______ with laughter, pleased with his joke, but then composed himself. (a) Brayed (b) Blade (c) Braid (d) Bread SSC GD 08/02/2023 (Shift-IV) Ans. (a) : In the blank space of the above sentence, 'brayed' would be appropriate answer. The meaning of other options is– Blade :-n cutting implement Braid :- decorate with braids or ribbons. Bread :- foods (like bread and cake and pastries) 339. Select the most appropriate options to fill in the blank. As l review the events of my past life, i realise how ______ are the influences that shape our destinies. (a) Subtle (b) Artless (c) Crube (d) Trumpet SSC GD 06/02/2023 (Shift-III) Ans. (a) : According to the sense of the above sentence, 'subtle' would be appropriate in the blank space. The other options give different meaning. Artless :- natural. Crube :- lacking in social refinement. Trumpet :- a brass musical instrument with brilliant tone. 340. Select the most appropriate options to fill in the blank. The operation is to be distinguished from of heat surgery and, in particular, from the repair of openings between the ______. (a) Oracle (b) Oculus (c) Oracles (d) Auricles SSC GD 06/02/2023 (Shift-III) Fill in the Blanks

Ans. (d) : In the blank space of the above sentence, the word 'Auricles' would be appropriate. The other options give different meaning– Oracle :- prophet, seer Oculus :- eye, optic Oracles :- a prophecy revealed by a priest or priestess. 341. Fill in the blanks with the correct homophone. _______ of anything is destructive in nature. (a) Axes (b) Excess (c) Axis (d) Access SSC GD 01/02/2023 (Shift-II) Ans. (b) : In the blank space of the above sentence, 'the correct homophone 'excess' (surfeit) would be appropriate. The other options give different meaning. Axes :- to get rid of a service, system etc. Axis :- the center around which something rotates. Access :- approach. 342. Select the most appropriate option to fill in the blanks and make the sentence meaningful. She hurt her ______ and _______. (a) heal; toe (b) heal; tow (c) heel; tow (d) heel; toe SSC GD 01/02/2023 (Shift-II) Ans. (d) : According to the sense of the above sentence, 'heel, toe' would be appropriate in the blank space. Because 'heel and toe' is an idiom. Which means– denoting a style of walking. Hence option (d) is correct answer. 343. Select the most appropriate option to fill in the blank. The doctor _________ two tablets per day. (a) ordered (b) prescribed (c) advised (d) proposed SSC CGL (Tier-I) 19/07/2023 (Shift-IV) Ans. (b) : The most appropriate option for the blank space is option (b) 'prescribed which means 'to something that has been officially recommended, ordered, such as doctor prescribing medication. Thus, the correct answer is option (b). Correct SentenceThe Doctor Prescribed two tablets per day. 344. Select the most appropriate homonym to fill in the blank. Please ___________more sugar to the juice so that it becomes sweeter (a) add (b) aid (c) ad (d) aide SSC CGL (Tier-I) 26/07/2023 (Shift-II) Ans. (a) : The most appropriate homonym for the blank space is option (a) 'add' (in clued). other option are– Aid means – Assist Ad means – short for advertisement Aide means – Associate Thus, the correct sentence is– please add more sugar to the juice so that it becomes sweets.

238

YCT

345. Select the most appropriate option to fill in the blank. The actress is very ______ and takes pride in her appearance all the time. (a) vein (b) vain (c) vane (d) wane SSC CGL (Tier-I) 26/07/2023 (Shift-II) Ans. (b) : The most appropriate option for the blank space 'vain' will be used because it qualify adjective 'very; other words give different meaning– vein means – Artery (Blood vensel) vane means – Blade, feathers Wane means – Diminish Thus, the correct answer is option (b) 346. Select the most appropriate option to fill in the blank. _________ are words which have the same spelling or pronunciation but not the same meaning. (a) Antonyms (b) Synonyms (c) Acronyms (d) Homonyms SSC CGL (Tier-I) 25/07/2023 (Shift-IV) Ans. (d) : In the above sentences in blank space 'Homonyms' would be appropriate. Meaning of other options Antonyms - Opposite, inversive, Synonyms - Alternative, Acronyms - Abbreviation, initialism, Correct SentenceHomonyms are words which have the same spelling or pronunciation but not the same meaning. 347. Select the most appropriate option to fill in the blank. We _________ off old clothes and bought new ones for Diwali. (a) cost (b) coast (c) cast (d) caste SSC CGL (Tier-I) 25/07/2023 (Shift-IV) Ans. (c) : The above given sentence in blank space option (c) 'cast' (v.). will be used because 'Cast-off' is a phrasal verb which means - to get rid of something because you no longer want or need it. Correct sentence - 'We cost off do clothes and bought new ones for Diwali.' 348. Select the most appropriate option to fill in the blank. There are rows of chairs on either side of the _______. (a) aisle (b) isle (c) I’ll (d) ail SSC CGL (Tier-I) 18/07/2023 (Shift-III) Ans. (a): The above given sentence in blank space 'aisle' (street) would be appropriate according to the sense of this sentence. Other options are not appropriate. Correct SentenceThere are rows of chairs on either side of the aisle. Fill in the Blanks

349. Select the most appropriate homonym to fill in the blank. The villagers kept the ________ out, to collect the rain water. (a) crus (b) cruse (c) crews (d) cruise SSC CGL (Tier-I) 20/07/2023 (Shift-I) Ans. (b) : The most appropriate homonym is ‘Cruse’ Which means a small container often used to hod liquids such as oil or water. Thus, the correct answer is option (b). 350. Select the most appropriate option to fill in teh blank. Keats and Shelly were poets of the same peirod; in other words, they were ______. (a) contemporaries (b) associates (c) co-writers (d) colleagues SSC CGL (Tier-I) 27/07/2023 (Shift-III) Ans. (a) : According to the sense of the above given sentence in blank space option (a) 'contemporaries' would be appropriate. Other options give different meaningAssociates - Peers, Colleague, Co-writers - Associates, Partners, Colleagues - Co-wrokers, Allies, Thus, the correct answer is option (a). 351. Select the most appropriate option to till in the blank. She could easily eat the ................ biryani by herself. (a) hole (b) haul (c) whole (d) hall SSC CGL (Tier-I) 27/07/2023 (Shift-III) Ans. (c) : According to the sense of the above given sentence in blank space 'Whole' would be appropriate. The Other options give different meaningHole- An empty space. Haul- To make a loud sound. Hall- A large room in a building. Thus, the correct answer is option (c). 352. Select the most appropriate homonym to fill in the blank. The hunter dogs followed the hyena’s ______. (a) cents (b) scent (c) cense (d) sense SSC CGL (Tier-I) 14/07/2023 (Shift-I) Ans. (b) : In the above sentence ‘scent’ (odour) will be used in the blank space because it gives appropriate meaning. Other options give different meaning. Correct sentence – The hunter dogs followed the hyena’s scent . 353. Select the most appropriate option to fill in the blank. Payments can be made by ____ or in cash. (a) cheeky (b) cheque (c) cheque (d) chick SSC CGL (Tier-I) 17/07/2023 (Shift-II)

239

YCT

Ans. (b) : In the above sentence 'Cheque' will be used in the blank space because it gives appropriate meaning according to sense of the sentence. other options give different meaning. Correct sentence– Payment can be made by cheque or in cash. 354. Select the most appropriate option to fill in the blank. The police were on high alert on account of the _____ convict who had killed many people. (a) run after (b) walk away (c) eloped (d) runaway SSC CGL (Tier-I) 17/07/2023 (Shift-II) Ans. (d) : In the above sentence "runaway' will be used in the blank space because it gives appropriate meaning according to sense of the sentence. other option give different meaning. walk away – move on. Eloped – Abscond Correct sentence – The police were on high alert on account of the runaway convict who had killed many people. 355. Select the most appropriate option to fill in the blank. The princess firmly held the ––––– in her hands. (a) Ran (b) Rein (c) Reign (d) Rain SSC CGL (Tier-I) 24/07/2023 (Shift-III) Ans. (b) : The most appropriate option for the blank space is option (b) (rein). (Bridle) because it gives appropriate meaning. Other options give different meaning. Reign - Dominate Rain - Precipitation Correct SentenceThe princess firmly held the Rein in her hands. 356. Select the most appropriate option to fill in the blank. Sarah just gave ________ to a baby boy. (a) breath (b) broth (c) berth (d) birth SSC CHSL (Tier-I) 17/08/2023 (Shift-II) Ans. (d) : option (d) Birth – ( Being born; coming out of a mother's body) is appropriate to be filled in the blank of the sentence. Because it gives the exact and intended meaning to the sentence. Other options means the following – Breath – The air that one takes Inter and blow out of one's lungs Broth – A thin soup, often with vegetables or rice in it Berth – An allotted place for sleeping on a train. Correct Sentence Sarah just gave birth to a baby boy. 357. Select the most appropriate option to fill in the blank. I can’t _______ this heat anymore. (a) bier (b) bear (c) bare (d) beer SSC CHSL (Tier-I) 10/08/2023 (Shift-I) Fill in the Blanks

Ans. (b) : Option (b) Bear – (To accept, tolerate or endure something unpleasant.) need to be filled in the provided blank to give the sentence appropriate of intended meaning. Meaning of other options Bier – A frame on which a dead body or coffen is carried before a funeral. Bare – without any covering or cloth Beer – A type of alcoholic drink that is made from grain. Correct Sentence – I can't bear this heat anymore. 358. Select the most appropriate homophone to fill in the blank. Even animals find it difficult to ______ the loss of their loved ones. (a) bare (b) where (c) bear (d) wear SSC CHSL (Tier-I) 02/08/2023 (Shift-I) Ans. (c) : The most appropriate homophone 'bear' (tolerate) would be appropriate in the blank space of the above sentence. Other options give different meaning – Bare – Empty, Nude, Wear – Put on, Robe, 359. Select the most appropriate option to fill in the blank. The CEO's _____ disregard for his employees' well-being led to high turnover and low morale in the company. (a) calus (b) calous (c) callus (d) callous SSC CHSL (Tier-I) 14/08/2023 (Shift-IV) Ans. (d) : Option (d) callous' has to be used in the blank space while the remaining options has incorrect spelling. callous – (showing or having an insensitive and cruet disregard for others) fulfills the demand and sense of the sentence. Correct Sentence The CEO's callous disregard for his employees will being led to high tween-over and low morale in the company. 360. Select the most appropriate homophones to fill in the blanks. I got ____ while replacing the ____. (a) tired, tyre (b) tired, tier (c) tyred, tire (d) tyre, tire SSC CHSL (Tier-I) 14/08/2023 (Shift-IV) Ans. (a) : Option (a) Tired, tyre is the most appropriate homophones which can be filled in the respective blanks as Both give suitable meaning. Tired means Drained of strength and energy and meaning of tyre will be. a thick rubber ring that fits around the outside of a while. Correct Sentence– I got tired while replacing the tyre.

240

YCT

361. Select the most appropriate homonym in the context of the following sentence to fill in the blank. The key was hanging on a ________by the door. (a) nail: a thin, hard area that covers the upper side of the end of each finger and each toe (b) nail: to catch someone, especially when they are doing something wrong, or guilty (c) nail: to do something successfully (d) nail: a small metal spike, driven into wood to join things together or to serve as a hook SSC CHSL (Tier-I) 08/08/2023 (Shift-II) Ans. (d) : The most appropriate homonym of the above sentence in blank space option (d) 'nail: a small metal spike, driven into wood to join things together or to serve as a hook' would be appropriate. Other options give different meanings. 362. Select the most appropriate option to fill in the blank. The ________ of the crime scene made it difficult for investigators to gather evidence. (a) cite (b) set (c) site (d) sleight SSC CHSL (Tier-I) 04/08/2023 (Shift-III) Ans. (c) : The most appropriate option (c) 'site' (spot) fill in the blank space of the above sentence. Other options give different meaning – Cite –Mention, Extract, Set – Group, team, Sleight – Craftiness, wile, 363. Fill in the blank with the most suitable word. The humour of the gods is cruel; or ______ they see more than we do. (a) unlikely (b) odds (c) perhaps (d) however SSC CHSL (Tier-I) 21/03/2023 (Shift-IV) Ans. (c) : For the above sentence, 'Perhaps' will be used in blank space because it gives appropriate meaning. Other options give different meaning. Correct SentenceThe humour of the gods is cruel; or perhaps they see more than we do. 364. Fill in the blank with the most suitable word. The ______ heard that the relationship was stormy and had broken up several times. (a) inquiry (b) impute (c) inordinate (d) impugn SSC CHSL (Tier-I) 21/03/2023 (Shift-IV) Ans. (a) : In the above sentence 'Inquiry' will be used in the blank space because it gives appropriate meaning. Other options give different meaning. 365. Fill in the blank with an appropriate option. Your ear has three main ______: outer, middle and inner. (a) parts (b) subjects (c) plugs (d) events SSC CHSL (Tier-I) 21/03/2023 (Shift-IV) Fill in the Blanks

Ans. (a) : For the above sentence 'parts' will be used in blank space because it gives appropriate meaning. Other options give different meaning. Plugs – Switch Subjects – Topic Events – Incident 366. Fill in the bland with an appropriate option. Taking a deep, ––––––– breath, I start again, successfully loading the filter and placing the cup underneath. (a) penetrating (b) surrogating (c) calming (d) jamming SSC CHSL (Tier-I) 21/03/2023 (Shift-II) Ans. (c) : According to the sense of the above sentence, option (c) 'calming' will be used in blank space. Other options are not suitable. 367. Fill in the blank with an appropriate option. _____ threats to elephants requires public and political with to take action. (a) Dignifying (b) Addressing (c) Perplexing (d) Stagnation SSC CHSL (Tier-I) 20/03/2023 (Shift-I) Ans. (b) : For the above sentence, option (b) 'Addressing' will be used in blank space. Other options are not correct. Correct Sentence: Addressing threats to elephants requires public and political will to take action. 368. Fill in the blank with an appropriate option Participants were _______ a meal of steak and French fries under dim light. (a) served (b) nerved (c) cured (d) calmed SSC CHSL (Tier-I) 20/03/2023 (Shift-I) Ans. (a) : For the above sentence, option (a) "served" will be used in blank space. because it gives suitable meaning. 369. Select the most appropriate option to fill in the blank. In the early autumn fierce fighting took place, the __________ Serbians bearing a prominent part. (a) cowardly (b) unabashed (c) gallant (d) bore SSC CHSL (Tier-I) 20/03/2023 (Shift-I) Ans. (c) : In the above sentence, 'gallant' will be used in blank space. 370. Fill in the blank with an appropriate option. We can usefully distinguish four types of claims that have been traditionally made using the ............. "human nature". (a) collateral (b) application (c) nuance (d) expression SSC CHSL (Tier-I) 17/03/2023 (Shift-IV) Ans. (d) : In the above sentence, 'expression' will be used in blank space. because it gives suitable meaning.

241

YCT

371. Fill in the blank with an appropriate option. The oil in the diya symbolises the dirt in the human mind such as greed, ............, hate, etc. (a) envy (b) slavery (c) joy (d) merry SSC CHSL (Tier-I) 17/03/2023 (Shift-IV) Ans. (a) : In the above sentence, 'envy' will be used in blank space. Other options are not correct. Correct SentenceThe oil in the diya symbolises the dirt in the human mind such as greed, envy hate etc. 372. Select the most appropriate option to fill in the blank. Howard tried to speak but all that come out was an ............., squeaky gargle. (a) implausible (b) sincere (c) easy (d) inarticulate SSC CHSL (Tier-I) 17/03/2023 (Shift-IV) Ans. (d) : In the above Sentence, 'inarticulate' will be used in blank space, because it gives suitable meaning for the sentence. Correct SentenceHoward tried to speak but all that come out was an inarticulate, sequeaky garagle. 373. Fill in the blank with an appropriate option. David always kept his ............. open for anything that might put them at risk. (a) black eye (b) starry eyes (c) eagle eyes (d) sheep eyes SSC CHSL (Tier-I) 17/03/2023 (Shift-III) Ans. (c) : In the above sentence 'eagle eyes' will be used in blank space because it gives suitable. Other option give different meaning. Starry eyes - Interested Sheep eyes - Shy Black eye - Mailignant 374. Fill in the blank with the most suitable word. He made me think of an old time magician more than ............. (a) a thing (b) everything (c) something (d) anything SSC CHSL (Tier-I) 15/03/2023 (Shift-III) Ans. (d) : In the above sentence 'anything' will be used because it gives appropriate meaning according to sense of the sentence. Other option give different meaning. Correct sentenceHe made me think of an old time magician more than anything. 375. Fill in the blank with an appropriate option. No matter how many times I try, I always end up in the same .............. (a) sauce (b) meal (c) pickle (d) account SSC CHSL (Tier-I) 15/03/2023 (Shift-I) Ans. (c) : In the above sentence 'Pickle' will be used in blank space because it gives appropriate meaning. Other options give different meaning. Fill in the Blanks

sauce → Ketchup account → Granary meal → Diet Correct SentenceNo matter how many times ? try, I always end up in the same Pickle. 376. Fill in the blank with an appropriate option. AWS offers various storage services that can ............ data for your website. (a) store (b) stack (c) pour (d) barge SSC CHSL (Tier-I) 15/03/2023 (Shift-I) Ans. (a) : In the above sentence 'Store' will be used in blank space because it gives appropriate meaning. Other options give different meaning. Pour→ Flow Comeout Stack → Stovepipe Barge → Hoy, Scow Correct sentenceAWS offers Various storage services that can store data for your website. 377. Fill in the blank with an appropriate option. In this engineering design project you will learn how to build an infinity mirror, with built-in lights that make the mirror look like a deep ............ with no end. (a) tunnel (b) lodge (c) funeral (d) signal SSC CHSL (Tier-I) 10/03/2023 (Shift-III) Ans. (a) : In the above sentence "Tunnel" will be used in the blank space because it gives appropriate meaning. Other options give different meaning. Signal – Sign, Indication Lodge – Stay, Keep Funeral – Exequies 378. Fill in the blank with an appropriate option. In humans, lack of sleep leads to ............. memory and reduced cognitive abilities. (a) attended (b) impaired (c) thankless (d) witnessed SSC CHSL (Tier-I) 10/03/2023 (Shift-III) Ans. (b) : In the above sentence "Impaired" will be used in blank space because it gives appropriate meaning. Other options give different meaning. Witnessed – Affirmed Attended – Visit, Report Thankless – Ungrateful Correct Sentence– In humans lack of sleep leads to impaired memory and reduced cognitive abilities. 379. Fill in the blank with an appropriate option. The social ............. is going to meet with all of us tomorrow with the plans so we can make a decision. (a) army (b) worker (c) toddler (d) labour SSC CHSL (Tier-I) 10/03/2023 (Shift-III) Ans. (b) : In the above sentence "worker" will be used in blank space because it gives appropriate meaning according to sense of the sentence. Other options give different meaning.

242

YCT

Toddler – Children Labour – Struggle Army – Military Correct Sentence – The social worker is going to meet with all of us tomorrow with the plans so we can make a decision. 380. Fill in the blank with an appropriate option. Wash, rinse and drain water from quinoa and ............ (a) set upon (b) set off (c) set aside (d) set in SSC CHSL (Tier-I) 09/03/2023 (Shift-IV) Ans. (c) : In the above sentence ‘‘set aside’’ will be used in blank space because it gives an appropriate meaning according to sense of the sentence. Other options give different meaning. Correct sentence wash, rinse and drain water from quinoa and set aside. 381. Fill in the blank with an appropriate option. The U.S. has military ............ in many Middle Eastern countries. (a) forces (b) denizens (c) personal (d) townies SSC CHSL (Tier-I) 09/03/2023 (Shift-IV) Ans. (a) : In the above sentence, 'Forces' will be used in blank space because it gives an appropriate meaning. Other options give different meaning. Denizens - Resident Personal - Particular Correct Sentence : The U.S. has military forces in many middle eastern countries. 382. Fill in the blank with an appropriate option. One of his rules was to always .............. and await instruction to enter. (a) knock (b) rub (c) blink (d) resist SSC CHSL (Tier-I) 09/02/2023 (Shift-II) Ans. (a) : Blank space of the above sentence option (a) 'knock' will be used because it gives appropriate meaning according to sense of the sentence. Other options give different meaning. Resist – Protest Blink – Nop, Doze Rub – Friction 383. Fill in the blank with the most suitable word. Remember that, little sister: wait for a man to ............ your honour, and you'll wait forever. (a) comfort (b) cheer (c) repeal (d) avenge SSC CHSL (Tier-I) 09/02/2023 (Shift-II) Ans. (d) : In the above sentence ‘‘Avenge’’ will be used in blank space which gives appropriate meaning according to sense of the sentence. Other options give different meaning. Comfort – Convenience Repeal – Breach Cheer – Encouragement Correct sentence : Remember that, little sister : wait for a man to avenge your honour and you'll wait forever. Fill in the Blanks

384. Fill in the blank with the most appropriate word. The boy ............ to take the money. (a) ordered (b) denied (c) reminded (d) refused SSC CGL (Tier-II) 08.08.2022 Shift-II Ans. (d) : In the blank space of the above sentence, 'refused' gives proper (suitable) meaning. The other options give different meaning. – Ordered :- arranged, controlled. Denied :- declare untrue (refuse) Reminded :- made aware, cautioned. Correct sentence– The boy refused to take the money. 385. Fill in the blank with the most appropriate word. The annual meeting of our club has been ........ till next month. (a) dragged (b) deferred (c) disrupted (d) deranged SSC CGL (Tier-II) 08.08.2022 Shift-II Ans. (b) : In the blank space of the above sentence, 'Deferred' would be appropriate. The other options give different meaning. Dragged :- crestfallen, cast down Disrupted :- confused, upset Deranged :- distract, unsettled. Correct sentence– The annual meeting of our club has been deferred till next month. 386. Fill in the blank with the most appropriate word. Gravity not only causes bodies to fall but also ........... their speed. (a) decreases (b) causes (c) increases (d) adds SSC Stenographer (Grade C & D) 11.11.2021 Shift-I Ans. (c) : According to the sense of the above sentence,' increase' gives suitable meaning. The other options give different meaning. Cause :- matter, purpose. Adds :- count up, calculate. Correct SentenceGravity not only causes bodies to fall but also increases their speed. 387. Select the most appropriate option to fill in the blanks. My talk will be on improving the .......... skills of students in English. (a) college (b) substitution (c) school (d) communication SSC Stenographer (Grade C & D) 15.11.2021 Shift-II Ans. (d) : In the blank space of the above sentence, 'communication' would be appropriate. The other options do not give suitable meaning. Substitution :- change, exchange Correct SentenceMy talk will be on improving the communication skills of students in English.

243

YCT

388. Select the most appropriate option to fill in the blank. A note of ––––––––during the leader's meeting at the UN. (a) discord surfaced (b) friction surfaced (c) strife surfaced (d) grievance surfaced SSC CHSL 30.05.2022 Shift-I Ans. (a) : In the blank space of the above sentence, 'discard surfaced' will be used, according to the sense of sentence it gives suitable meaning. The other options give different meaning. Correct Sentence A note of discord surfaced during the leader's meeting at the UN. 389. Select the most appropriate option fill in the blank. The death of her father had made her sullen and –––––––. (a) melancholic (b) joyful (c) smug (d) jovial SSC CHSL 30.05.2022 Shift-I Ans. (a) : In the blank space of the above sentence, 'melancholic' gives the suitable meaning. The other options give different meaning. Joyful :- very happy Smug :- looking or feeling too pleased about something. Jovial :- very cheerful and friendly. Correct Sentence The death of her father had made her sullen and melancholic. 390. Select the most appropriate option to fill in the blank. My grandfather was a farmer and he –––––– three acres of land. (a) advanced (b) cultivated (c) grown (d) cultured SSC CHSL 24.05.2022 Shift-III Ans. (b) : In the blank space of the above sentence, 'cultivated' gives suitable meaning for this sentence. The meaning of other options is – Advanced :- progressive. Grown :- developed. Culture :- civilized Correct Sentence My grandfather was a farmer and he cultivated three acres of land. 391. In the following passage, some word have been deleted. Read the passage carefully and select the most appropriate option to fill in each blank. Many times –––––– starts with the creative ––– –––– and the enduring passion of a single individual. (a) renovation, dullness (b) innovation, spark (c) stagnation, determination (d) tradition, lethargy SSC CHSL 31.05.2022 Shift-III Fill in the Blanks

Ans. (b) : In the blank space of the above sentence, 'innovation and spark' will be used respectively because it gives proper meaning for this sentence. The other options give different meaning. Correct SentenceMany times innovation starts with the creative spark and the enduring passion of a single individual. 392. Select the correctly spelt word to fill in the blank. The ––––– of the worksheets is not an easy job. (a) reconcilation (b) reconsilation (c) reconsiliation (d) reconciliation SSC CHSL 26.05.2022 Shift-I Ans. (d) : In the blank space of the above sentence, 'Reconciliation' will be used because it is spelled correctly. The other options are wrong. Correct sentence– The reconciliation of the worksheets is not an easy job. 393. Select the most appropriate option to fill in the blank. My success was complete and the delighted directors ––––––– me the privilege of developing automatic regulators which were much desired. (a) accorded (b) contrasted (c) disagreed (d) removed SSC CHSL 26.05.2022 Shift-I Ans. (a) : In the blank space of the above sentence, 'Accorded' give the proper meaning of this sentence. The other options give different meaning. Contrasted :- contradictory, converse. Disagreed :- dissent Removed :- detached. Correct sentence – My success was complete and the delighted directors accorded me the privilege of developing automatic regulators which were much desired. 394. Select the most appropriate option to fill in the blank. She was laughing from the –––––– exuberance of the performance. (a) total (b) sheer (c) perfect (d) utter SSC CHSL 26.05.2022 Shift-I Ans. (b) : In the blank space of the above sentence, 'sheer' (absolute) gives the suitable meaning. The other options give different meaning. Total :- sum (the whole amount) Perfect :- exact, complete. Utter :- express in speech. Correct sentence – She was laughing from the sheer exuberance of the performance. 395. Select the most appropriate option to fill in the blank. The beauty of our festivals lies in 'unity in –––– –––––'. (a) belongingness (b) richness (c) inclusion (d) diversity SSC CHSL 26.05.2022 Shift-I

244

YCT

Ans. (d) : In the blank space of the above sentence, 'Diversity' (variety) give the suitable meaning. The other options give different meaning. Richness :- Affluence Inclusion :- Comprehension. Correct Sentence – The beauty of our festivals lies in 'unity in diversity.' 396. Select the correct homonym from the given options to fill in the blank. The French perfume has a wonderful________ (a) scent (b) sent (c) send (d) cent SSC CHSL 26.05.2022 Shift-III Ans. (a) : In the blank space of the above sentence, 'scent' (aroma) gives suitable meaning. The other options give different meaning. Sent :- sent out. Send :- order to be taken. Cent :- penny. Correct Sentence– The French perfume has a wonderful scent. 397. Select the most appropriate option to fill in the blank. We should ______ his outfit as he has put on weight. (a) utter (b) alter (c) altar (d) ultra SSC CHSL 26.05.2022 Shift-III Ans. (b) : In the blank space of the above sentence, 'Alter' (change) gives suitable meaning in the sentence. The other options give different meaning. – Utter :- express in speech. Altar :- a holy table in a church or temple. Ultra :- a person who holds extreme views especially in politics. Correct Sentence– We should alter his outfit as he ha put on weight. 398. Select the most appropriate option to fill in the blank. Asha was ___________ made a fool by her friend. (a) unknowingly (b) insistently (c) deliberately (d) innocently SSC CHSL 25.05.2022 Shift-I Ans. (c) : In the blank space of the above sentence, 'Deliberately' (with intention) gives proper meaning. The other options are – Unknowingly:- without knowledge or intention. Insistently:- in a way that demands something firmly and refuse to accept any opposition or excuses. Innocently:- In a way that is not intended to cause harm or upset somebody. Correct Sentence– Asha was deliberately made a fool by her friend. 399. Select the most appropriate synonym of the word given in the bracket to fill in the blank. The western side of the country has some ––––– –––– (vacant) islands to explore. Fill in the Blanks

(b) uninterested (d) undesired SSC CHSL 27.05.2022 Shift-I Ans. (c) : In the blank space of the above sentence, 'uninhabited' (desolate) would be appropriate. The meaning of other options is– Unexplored :- undiscovered, unknown. Uninterested :- not having or showing interest. Undesired :- unwanted. Correct Sentence The western side of the country has some uninhabited (vacant) islands to explore. 400. Select the correct homonym from the given options to fill in the blank. The police returned to the –––––– of the crime. (a) cene (b) seen (c) scene (d) zen SSC CHSL 27.05.2022 Shift-I Ans. (c) : In the blank space of the above sentence, 'scene (vista)' gives the suitable meaning for the sentence. The meaning of other options is– Cane :- stick See :- to view Correct SentenceThe police returned to the scene of the crime. 401. Select the most appropriate option to fill in the blank. A student –––––– in ragging can be expelled from the institution. (a) curbing (b) avoiding (c) indulging (d) contrasting SSC CHSL 27.05.2022 Shift-III Ans. (c) : In the blank space of the above sentence, 'Indulging' (pander) gives the proper meaning. The other options give different meaning. Curbing :- to control or limit something. Avoiding :- refrain from doing something. Contrasting :- different. Correct Sentence A student indulging in ragging can be expelled from the institution. 402. Select the most appropriate option to fill in the blank. Ramu went with his friends on a trip to ––––– new paths. (a) explore (b) invent (c) determine (d) deploy SSC CHSL 26.05.2022 Shift-II Ans. (a) : In the blank space of the above sentence, 'Explore' would be appropriate. The meaning of other options is – Invent :- to produce or design something that has not existed before. Determine :- assure, insure Deploy :- to use something effectively. Correct sentence– Ramu went with his friends on a trip to explore new paths.

245

(a) unexplored (c) uninhabited

YCT

403. Select the most appropriate option to fill in the blank. _____ I learned that there was a gift for each child, I was delighted. (a) When (b) Though (c) While (d) Which SSC MTS-18/07/2022 Shift-I Ans. (a) : According to the sense of the above sentence, 'when' will be used in the blank space. Correct Sentence– When I learned that there was a gift for each child, I was delighted. 404. Select the most appropriate to fill in the blank. Most mainliners do not view modern culture as inherently _____ to religious belief. (a) inimical (b) inchoate (c) inconclusive (d) inadvertent SSC MTS-18/07/2022 Shift-I Ans. (a) : According to the sense of the above sentence, 'inimical' (harmful) would be appropriate in the blank space. The meaning of other options is – Inchoate :- only partly in existence. Inconclusive :- indecisive. Inadvertent :- unintended. 405. Select the most appropriate to fill in the blank. That fellow pushed me so hard that my vision got ______ for a second. (a) simplified (b) illuminated (c) exposed (d) blurred SSC MTS-18/07/2022 Shift-I Ans. (d) : In the blank space of the above sentence, 'blurred' (unclear) gives suitable meaning. The other options are – Simplified :- made easy. Illuminate :- illumine. Exposed :- open. Correct Sentence – That fellow pushed me so hard that my vision got blurred for a second. 406. Select the most appropriate option to fill in the blank. She smashed a ____ of glass with the metal vase. (a) pion (b) pawn (c) pane (d) pain SSC MTS-18/07/2022 Shift-I Ans. (c) : In the blank space of the above sentence, 'pane (window glass)' would be suitable because according to the sense of above sentence, it gives suitable meaning. Pawn :- a chess piece of the smallest size and least value. Pain :- a person or thing that is very annoying Correct Sentence – She smashed a pane of glass with the metal vase. Fill in the Blanks

407. Select the most appropriate option to fill in the blank. They called Mohan a ____ because he did not want to go into the dark lane. (a) brave (b) valiant (c) hero (d) coward SSC CGL (Tier-I) 19/04/2022 Shift-III Ans. (d) : According to the sense of sentence, 'Coward' would be appropriate in the blank space. The other word gives different meaning. Brave :- bold, heroic Valiant :- showing valor Hero :- champion, fighter. 408. Select the most appropriate option to fill in the blank. You will find it difficult ______ but later you will get used to wearing a mask. (a) at first (b) at the first (c) firstly (d) on first SSC CGL (Tier-I) 18/04/2022 Shift-I Ans. (a) : According to the sense of the above sentence, 'at first' will be used in the blank space. Thus, the correct option is (a). 409. Select the most appropriate option to fill in the blank. Global warming or climate change has today become a major ______ to mankind. (a) threat (b) penalty (c) punishment (d) endanger SSC CGL (Tier-I) 11/04/2022 Shift-II Ans. (a) : According to the sense of the above sentence, 'threat' would be appropriate. The other options give different meaning. Penalty :- painful consequences of an action or condition. Punishment :- act of punishing. Endanger :- Imperil. 410. Select the most appropriate option to fill in the blank. This app is a very ______ one for online shopping. (a) convenience (b) competency (c) capable (d) convenient SSC CGL (Tier-I) 12/04/2022 Shift-III Ans. (d) :In the blank space of the above sentence, 'Convenient' (accessible) would be appropriate. The meaning of other options is – Convenience :- the quality of being useful and convenient. Competence :- well prepared to do something. Capable :- having capacity or ability. 411. Select the most appropriate option to fill in the blank. Harish's voice ____ in the empty rooms of their new house. (a) reverted (b) resounded (c) relapsed (d) resorted SSC CGL (Tier-I) 13/04/2022 Shift-I

246

YCT

Ans. (b) : The most appropriate option in the given blank space is 'resounded' (noise). The meaning of other options is – Reverted :- returned, turned back Relapsed :-deteriorate in health. Resort :- to adopt a course of action to improve your situation. 412. Select the most appropriate option to fill in the blank. The reality of the effects of rising aviation industry emissions on climate change has been known for decades, yet the industry and its emissions has continued to grow without ______. (a) easiness (b) neglect (c) constraint (d) comfort SSC MTS-15/07/2022 Shift-I Ans. (c) : According to the sense of the above sentence, 'constraint' (restraint) would be appropriate. The other options give different meaning. Easiness :- simpleness, simplicity. Neglect :- Ignore, disregard. Comfort :- comfortableness. 413. Select the most appropriate to fill in the blank. Accuracy rates of facial recognition algorithms are _____ low in the case of minorities, women and children, as demonstrated in multiple studies across the world. (a) smoothly (b) Carelessly (c) Particularly (d) Beautifully SSC MTS-15/07/2022 Shift-I Ans. (c) : According to the sense of the above sentence, 'Particularly' (specially) gives the suitable meaning. The other options give different meaning. Smoothly :- swimmingly. Carelessly :- without care or concern. Beautifully :- attractively. 414. Select the most appropriate option to fill in the blank. I prefer to listen to relaxing and _____ music when meditating. (a) soothing (b) abrading (c) fretting (d) tormenting SSC MTS-15/07/2022 Shift-I Ans. (a) : In the blank space of the above sentence, 'soothing' would be appropriate. The other options give different meaning. Abrading :- rubbing down, abrasing Fretting :- worry unnecessarily Tormenting :- torture Correct Sentence – I prefer to listen to relaxing and soothing music when meditating. 415. Select the most appropriate option to fill in the blank. The ______ of directors will decide the penalty. (a) bowed (b) braid (c) bored (d) board SSC MTS-15/07/2022 Shift-I Fill in the Blanks

Ans. (d) : According to the sense of the above sentence, board (panel) would be appropriate in the blank space. The other options give different meaning. Bowed :- submissive. Braid :- thin coloured rope that is used to decorate furniture and military uniform. Bord :- tired. 416. Select the most appropriate option to fill in the blank. He has a high paying job, ______ he never seems to have any money. (a) yet (b) by (c) from (d) so SSC MTS-15/07/2022 Shift-I Ans. (a) : In the blank space of the above sentence, 'yet' will be used because according to the sense of the above sentence, it gives proper meaning. Correct Sentence – He has a high paying job, yet he never seems to have any money. 417. Select the most appropriate option to fill in the blank. The boys were tired, but not too tired to examine the little hut ______. (a) weirdly (b) weakly (c) thoroughly (d) sluggishly SSC MTS-06/07/2022 Shift-I Ans. (c) : According to the sense of the above sentence, 'thoroughly' would be appropriate. The other options give different meaning. Weirdly :- in a weird manner. Weakly :- feeble. Sluggishly :-more slowly than normal and in a way that seems lazy. Correct Sentence – The boys were tired, but not too tired to examine the little hut thoroughly. 418. Select the most appropriate option to fill in the blank. It is better to be prompt and organised than to be _____. (a) provident (b) meaningless (c) efficient (d) reckless SSC MTS-06/07/2022 Shift-I Ans. (d) : According to the sense of the above sentence, 'reckless' will be used in the blank space. The other options are– Provident :- careful, thrifty, foresighted. Meaningless :- unimportant. Efficient :- competent, effective. Correct Sentence – It is better to be prompt and organised than to be reckless 419. Select the most appropriate option to fill in the blank. The Lebanese army raided the boat and ____ dozens of water bottles, tuna cans, loaves of bread, boxes of Picon cheese and life jackets. (a) discovered (b) learnt (c) created (d) invented SSC MTS-07/07/2022 Shift-I

247

YCT

Ans. (a) : According to the sense of the sentence, 'discovered' would be appropriate in the blank space. The other options give different meaning. Learnt :- acquired, gained knowledge. Created :- make Invented :- to produce or design something that has not existed before. Correct Sentence – The Lebanese army raided the boat and discovered dozens of water bottles, tuna cans, loaves of bread, boxes of Picon cheese and life jackets. 420. Select the most appropriate option to fill in the blank. I can't control my_____ these days because I got selected for a job. (a) casualness (b) nepotism (c) pollution (d) excitement SSC MTS-07/07/2022 Shift-I Ans. (d) : According to the sense of the above sentence, 'excitement' will be used in the blank space. The meaning of other options is – Casualness :- familiarity. Nepotism :- favoritism shown to relatives or close friend. Pollution :- defilement. Correct Sentence – I can't control my excitement these days because I got selected for a job. 421. Select the most appropriate option to fill in the blank. Writing books about how to _____ on the internet seems like a great idea. (a) make a promise (b) make a fortune (c) make a habit (d) make a list SSC MTS-07/07/2022 Shift-I Ans. (b) : According to the sense of the above sentence, 'make a fortune' would be appropriate in the blank space. The other options give different meaning. Make a promise :- to tell someone that you will definitely to do something. Make a habit :- make manner. Make a list :- make a record. Correct Sentence – Writing books about how to make a fortune on the internet seems like a great idea. 422. Select the most appropriate option to fill in the blank. Which ______ should I take to reach the railway station? (a) root (b) route (c) rote (d) riot SSC MTS-07/07/2022 Shift-I Ans. (b) : According to the sense of the above sentence, 'route' would be appropriate in the blank space. The other sentence give different meaning. Fill in the Blanks

Root :- base, foundation. Rote :- routine. Riot :- a noisy, violent public disorder caused by a group. Correct Sentence – Which route should I take to reach the railway station. 423. Select the most appropriate option to fill in the blank. One of the children stood at the door to _____ if he saw the apparitor coming, who was an officer of the spiritual court. (a) give notice (b) give advice (c) give thought (d) give a hand SSC MTS-05/07/2022 Shift-III Ans. (a) : According to the sense of the above sentence, 'give notice' would be appropriate in the blank space. The other options give different meaning. Give Advice :- counsel Give thought :- thinking, idea or notion Give a hand :- Help Correct Sentence – One of the children stood at the door to give notice if he saw the apparitor coming, who was an officer of the spiritual court. 424. Select the most appropriate option to fill in the blank. In all cases, we have to ______ the facts from the various reports and documents. (a) Instil (b) Repel (c) Standstill (d) Distil SSC MTS-06/07/2022 Shift-II Ans. (d) : According to the sense of the above sentence, 'Distil' (to get the essential meaning or idea) will be used in the blank space. Other option give different meaning. Instil :- gradually make somebody feel, think. Repel :- force back. Standstill :- deadlock, impasse. Correct Sentence – In all case, we have to distil the facts from the various reports and documents. 425. Select the most appropriate option to fill in the blank. Thus, _____ of their purpose, the three knights tried another plan. (a) involved (b) foiled (c) toiled (d) failed SSC MTS-06/07/2022 Shift-II Ans. (b) : According to the sense of the above sentence, 'foiled' will be used, which means 'to present a plan from succeeding.' The other options are– Involved :- participating Toil :- work hard Failed :- neglected Correct Sentence– Thus, foiled of their purpose, the three knights tried another plan.

248

YCT

426. Select the most appropriate option to fill in the blank. If you want to save your money, start living an _____ life. (a) improper (b) economical (c) extravagant (d) indulgent SSC MTS-13/07/2022 Shift-I Ans. (b) : According to the sense of the above sentence, 'economical' would be appropriate in the blank space. The meaning of other options is – Improper :- not suitable or right (inappropriate) Extravagant :- prodigal, spendthrift. Indulgent :- lenient, soft. Correct Sentence – If you want to save your money, start living an economical life. 427. Select the most appropriate option to fill in the blank. This cruise ship _____ in November and December. (a) sails (b) sells (c) sales (d) seals SSC MTS-13/07/2022 Shift-I Ans. (a) : According to the sense of the above sentence, 'sails' will be used in the blank space. The other options give different meaning. Sells :- trade, betray Sales :- gross sales Seals :- a thin that makes something definite. 428. Select the most appropriate option to fill in the blank. If we don't reverse course now, will anyone take anything we say about justice and ________ with any seriousness for decades to come? (a) righteousness (b) mediation (c) wickedness (d) dishonesty SSC MTS-13/07/2022 Shift-I Ans. (a) : According to the sense of the above sentence, 'Righteousness' will be used in the blank space. The other options give different meaning. Mediation :- intermediation Wickedness :- evilness, sin. Dishonesty :- Knavery. Correct Sentence – If we don't reverse course now, will anyone take anything we say about justice and righteousness with any seriousness for decades to come. 429. Select the most appropriate option to fill in the blank. Many people may not find the rewards ______ with the time and effort required. (a) insignificant (b) gleeful (c) cautious (d) commensurate SSC MTS-13/07/2022 Shift-I Ans. (d) : According to the sense of the above sentence, 'Commensurate' would be appropriate in the blank space. The other options give different meaning. Fill in the Blanks

Insignificant :- meaningless Gleeful :- merry. Cautious :- careful. Correct Sentence – Many people may not find the rewards commensurate with the time and effort required. 430. Select the most appropriate option to fill in the blank. The ______ of the knife in his hand made her sick. (a) site (b) sight (c) slight (d) cite SSC MTS-05/07/2022 Shift-II Ans. (b) : In the blank space of the above sentence, 'sight' will be used because according to the sense of sentence, it gives proper meaning. Correct Sentence – The sight of the knife in his hand made her sick. 431. Select the most appropriate option to fill in the blank. The ______ needs of the inhabitants seem to be fruit, iced drinks, shiny boots and lottery tickets. (a) amiable (b) principle (c) invincible (d) principal SSC MTS-05/07/2022 Shift-II Ans. (d) : In the blank space of the above sentence, 'Principal' will be used because it gives suitable meaning according to the sense of this sentence. Correct Sentence – The principal needs of the inhabitants seem to be fruit, iced drinks, shiny boots and lottery tickets. 432. Select the most appropriate option to fill in the blank. There are good points to each side and we need more _____ people that are willing to look at them from all angles. (a) ubiquitous (b) unbiased (c) undermined (d) unadorned SSC MTS-05/07/2022 Shift-II Ans. (b) : In the blank space of the above sentence, 'unbiased' would be appropriate because it gives proper meaning according to the sense of this sentence. Correct Sentence – There are good points to each side and we need more unbiased people that are willing to look at them from all angles. 433. Select the most appropriate option to fill in the blank. My niece is very kind and ______ to everyone in the family. (a) callous (b) ruthless (c) generous (d) stern SSC MTS-05/07/2022 Shift-II Ans. (c) : In the blank space of the above sentence, 'generous' will be used because it gives suitable meaning, according to the sense of this sentence. Correct sentence – My niece is very kind and generous to everyone in the family.

249

YCT

434. Select the most appropriate options to fill in the blank. Thank you so _______ for helping me with this project. (a) too (b) many (c) most (d) much SSC MTS-05/07/2022 Shift-II Ans. (d) : In the blank space of the above sentence, 'much' would be appropriate because 'much' is used with so in the uncountable sense. Correct sentence– Thank you so much for helping me with this project. 435. Select the most appropriate option to fill in the blank. All of my efforts to save her life have gone in _____. (a) vain (b) vein (c) van (d) vane SSC MTS-07/07/2022 Shift-II Ans. (a) : In the blank space of the above sentence, 'vain' will be used because 'in vain' is an idiom which means – without success. Vein :- a blood vessels Van :- a truck with an enclosed cargo space. Vane :- blade. Correct Sentence – All of my efforts of saver her life have gone in vain. 436. Select the most appropriate option to fill in the blank. The US has become far more _____ to people with all kinds of food sensitivities and diet regimens. (a) deviated (b) lodging (c) selective (d) accommodating SSC MTS-07/07/2022 Shift-II Ans. (d) : According to the sense of the above sentence, 'Accommodating' would be appropriate in the blank space. The meaning of other options is – Deviated :- diverted Lodging :- lodgment Selective :- exclusive, tending to select. Correct Sentence – The US has become far more accommodating to people with all kinds of food sensitivities and diet regimens. 437. Select the most appropriate option to fill in the blank. My sister treats me well in the morning but at night she becomes______. (a) eloquent (b) awful (c) exemplary (d) acceptable SSC MTS-07/07/2022 Shift-II Ans. (b) : According to the sense of the above sentence, 'awful' will be used in the blank space. The other option give different meaning. Eloquent :- smooth - spoken. Exemplary :- model, worthy. Acceptable :- unexceptionable Correct Sentence – My sister treats me well in the morning but at night she becomes awful. Fill in the Blanks

438. Select the most appropriate option to fill in the blank. The ocean waves swelled and reached _____ her head, and she started screaming. (a) among (b) upon (c) above (d) at SSC MTS-06/07/2022 Shift-III Ans. (c) : According to the sense of the above sentence, 'above' would be appropriate in the blank space. The other options are not suitable. Correct Sentence – The ocean waves swelled and reached above her head, and she started screaming. 439. Select the most appropriate option to fill in the blank. He said the cooperative will continue to invest _____ per its usual yearly rate. (a) that (b) so (c) such (d) as SSC MTS-06/07/2022 Shift-III Ans. (d) : In the blank space of the above sentence, 'as' will be used because according to the sense of the above sentence 'as per' is used. The other options give different meaning. Correct Sentence – He said the cooperative will continue to invest as per its usual yearly rate. 440. Select the most appropriate option to fill in the blank. Ms. Gupta expressed her ______ for my political understanding. (a) admiration (b) location (c) strength (d) compassion SSC MTS-08/07/2022 Shift-I Ans. (a) : According to the sense of the above sentence, 'Admiration' would be appropriate. The other options give different meaning. Location :- a place or position. Strength :- the quality of being physically or mentally strong. Compassion :- a deep awareness of and sympathy for another's suffering. Correct Sentence – Ms. Gupta expressed her admiration for my political understanding. 441. Select the most appropriate option to fill in the blank. Pilots must be _____ at all times to keep their passengers safe during the flight. (a) thirsty (b) distant (c) vigilant (d) forceful SSC MTS-08/07/2022 Shift-I Ans. (c) : According to the sense of the sentence, 'Vigilant' (sharp-eyed) would be appropriate. The meaning of other options is – Thirsty :- feeling a need or desire to drink. Distant :- far, deep Forceful :- strong, firm, emphatic Correct Sentence – Pilots must be vigilant at all times to keep their passengers safe during the flight.

250

YCT

442. Select the most appropriate option to fill in the blank. Could you spell your name ______, please? (a) weakly (b) never (c) again (d) forever SSC MTS-08/07/2022 Shift-I Ans. (c) : According to the sense of the above sentence, 'again' will be used in the blank space. The meaning of other options is – Weakly :- every week. Never :- not at all, certainly not. Forever :- without interruption. (always) Correct Sentence – Could you spell your name again please? 443. Select the most appropriate option to fill in the blank. The sudden death of our uncle was _____ for all of us. (a) tragic (b) fragile (c) gentle (d) dissuasive SSC MTS-12/07/2022 Shift-I Ans. (a) : According to the sense of the above sentence, 'tragic' would be appropriate in the blank space. Fragile :- delicate, breakable. Gently :- mildly Dissuasive :- discouraging, deterring from action. Correct Sentence – The Sudden death of our uncle was tragic for all of us. 444. Select the most appropriate option to fill in the blank. It's ______ too late to start your own business. (a) never (b) no (c) neither (d) nor SSC MTS-12/07/2022 Shift-I Ans. (a) : According to the sense of the above sentence, 'never' will be used in the blank space. The other options do not give proper meaning. no :- none Correct Sentence – It's never too late to start your own business. 445. Select the most appropriate option to fill in the blank. India continues to ______ the highest amount of e-waste, more than China, the US, Japan and Germany. (a) generate (b) remake (c) stake (d) select SSC MTS-14/07/2022 Shift-III Ans. (a) : According to the sense of the above sentence, 'generate' would be appropriate. Remake :- creation that is created again. Stake :- bet, wager Select :- choice Correct Sentence– India continues to generate the highest amount of ewaste, more than china, the US. Japan and Germany. Fill in the Blanks

446. Select the most appropriate option to fill in the blank. A _____ body will fight off the illness naturally without any medicine. (a) happy (b) healthy (c) flabby (d) fresh SSC MTS-14/07/2022 Shift-III Ans. (b) : According to the sense of the above sentence, 'healthy' will be used in the blank space. Happy :- Cheerful Flabby :- flaccid, soft Fresh :- new-made Correct Sentence – A healthy body will fight off the illness naturally without any. 447. Select the most appropriate option to fill in the blank. Can you please ______ this sauce over the pasta? (a) pore (b) power (c) pour (d) purr SSC MTS-14/07/2022 Shift-III Ans. (c) : According to the sense of the above sentence, 'pour' would be appropriate in the blank space. pore :- concentrate power :- strength, force. purr :- make vibrant sounds. Correct Sentence – Can you please pour this sauce over the pasta? 448. Select the most appropriate option to fill in the blank. We deplore the use of ______ against innocent people. (a) reduction (b) violence (c) gentleness (d) humour SSC MTS-14/07/2022 Shift-III Ans. (b) : According to the sense of the above sentence, 'violence' would be appropriate in the blank space. The other options give different meaning. Reduction :- decrease Gentleness :- mildness Humour :- wit (the quality of being funny). Correct Sentence – We deplore the use of violence against innocent people. 449. Select the most appropriate option to fill in the blank. We _____ learn to meet adversity gracefully. (a) have (b) ought (c) had (d) must SSC MTS-14/07/2022 Shift-III Ans. (d) : In the blank space of the above sentence, 'must' would be appropriate because 'must' is used to express obligation, give orders and advice. While 'ought' is used to express correctness or duty, indicate that something is probable. Correct Sentence – We must learn to meet adversity gracefully.

251

YCT

450. Select the most appropriate option to fill in the blank. Mere classroom teaching-learning is boring and ______ for students. (a) monotonous (b) exciting (c) interesting (d) varied SSC Constable GD-30/11/2021 Shift-III Ans. (a) : In the blank space of the above sentence, 'Monotonous' would be appropriate. The other options five different meaning. Exciting :- creating Interesting :- amusing, entertaining. Varied :- different. Correct sentence Mere classroom teaching-learning is boring and monotonous for students. 451. Select the most appropriate option to fill in the blank. When his mother way away, he took the ____ of his little brother. (a) management (b) duty (c) power (d) charge SSC Constable GD-30/11/2021 Shift-III Ans. (d) : In the blank space of the above sentence, 'Charge' will be used. The other options give different meaning. Management :- direction Duty :- obligation, responsibility. Power :- ability Correct Sentence When his mother was away, he took the charge of his little brother. 452. Select the most appropriate option to fill in the blank. The word ‘ecology’ refers to the pattern and ______ of relationships among plants, animals and people and their environment. (a) balance (b) policy (c) plan (d) outline SSC Constable GD-29/11/2021 Shift-II Ans. (a) : In the blank space of the above sentence, 'balance' would be appropriate because it gives suitable meaning according to the sense of this sentence. Correct Sentence The word ‘ecology’ refers to the pattern and balance of relationships among plants, animals and people and their environment. 453. Select the most appropriate option to fill in the blank. No changes can be made to the bond without the ______ of the partners. (a) sympathy (b) harmony (c) consent (d) confession SSC Constable GD-29/11/2021 Shift-II Ans. (c) : In the blank space of the above sentence, 'consent' will be used because according to the sense of this sentence, it gives suitable meaning. Correct sentence No changes can be made to the bond without the consent of the partners. Fill in the Blanks

454. Select the most appropriate option to fill in the blank. Edit and proofread your essay ______ times. (a) much (b) few (c) less (d) several SSC Constable GD-26/11/2021 Shift-I Ans. (d) : In the blank space of the above sentence, 'several' gives suitable meaning. The other options give different meaning. Much :- great in quality or degree. (Uncountable) Few :- Hardly (Countable) Less :- comparative of little, lower in quality (Uncountable) Correct sentence – Edit and proofread your essay several times. 455. Select the most appropriate option to fill in the blank. He recovered after a ______ illness. (a) projected (b) stretched (c) prolonged (d) surprise SSC Constable GD-26/11/2021 Shift-I Ans. (c) : In the blank space of the above sentence, 'Prolonged' gives suitable meaning. The other options give different meaning. Projected :- predicted in advance. Stretched :- flexible, extended. Surprise :- the act of surprising someone. Correct sentence – He recovered after a prolonged illness. 456. Select the most appropriate option to fill in the blank. The book gives a ______ account of the horrors of war. (a) heroic (b) bulky (c) pleasant (d) graphic SSC Constable GD-26/11/2021 Shift-I Ans. (d) : In the blank space of the above sentence, 'graphic' gives suitable meaning. The other options give different meaning. Heroic :- bold, brave Bulky :- large Pleasant :- nice, good-natured. Correct sentenceThe book gives a graphic account of the horrors of war. 457. Select the most appropriate option to fill in the blank. He has been ______ in his performance throughout. (a) consistent (b) constructive (c) conducive (d) connotative SSC Constable GD-25/11/2021 Shift-III Ans. (a) : In the blank space of the above sentence, 'consistent' will be used because according to the sense of the sentence, it gives suitable meaning. Correct Sentence – He has been consistent in his performance throughout.

252

YCT

458. Select the most appropriate option to fill in the blank. The ______ cannot be a panacea for all respiratory infections. (a) care (b) healing (c) treatment (d) diagnosis SSC Constable GD-25/11/2021 Shift-III Ans. (c) : In the blank space of the above sentence, 'treatment' will be used because according to the sense of the sentence, it gives suitable meaning. Correct sentence – The treatment cannot be a Panacea for all respiratory infections. 459. Select the most appropriate option to fill in the blank. If you do not pay your ______ you will be imprisoned. (a) debt (b) debit (c) deed (d) damage SSC Constable GD-25/11/2021 Shift-III Ans. (a) : In the blank space of the above sentence, 'debt' will be used because, according to the sense of the sentence it gives proper meaning. Correct Sentence – If you do not pay your debt you will be imprisoned. 460. Select the most appropriate option to fill in the blank. The police ______ the protestors from entering the parliament. (a) prevented (b) presumed (c) protected (d) persuaded SSC Constable GD-24/11/2021 Shift-II Ans. (a) : According to the sense of the above sentence, 'prevented' will be used in the blank space. The other options give different meaning. Presumed :- assume, accept without verification or proof. Protected :- kept safe, defended from danger. Persuaded :- carry (with approval) Correct Sentence The police prevented the protestors from entering the parliament. 461. Select the most appropriate option to fill in the blank. The company is going to ______ the North Sea for oil. (a) search (b) explore (c) tour (d) traverse SSC Constable GD-24/11/2021 Shift-II Ans. (b) : According to the sense of the sentence, 'explore' would be appropriate in the blank space. The other options give different meaning. Search :- try to locate or discover. Tour :- a journey or route. Traverse :- cross, cut Correct Sentence The company is going to explore the North sea for oil. Fill in the Blanks

462. Select the most appropriate option to fill in the blank. Please ______ my subscription of the children’s magazine. (a) settle (b) renew (c) regain (d) finish SSC Constable GD-24/11/2021 Shift-II Ans. (b) : According to the sense of the above sentence, 'renew' gives the suitable meaning in the blank space. The other options give different meaning. Settle :- fixed, establish Regain :- recover, find Finish :- complete Correct Sentence Please renew my subscription of the children's magazine. 463. Select the most appropriate option to fill in the blank. This is the ______ clock that I have ever seen. (a) finest (b) purest (c) tastiest (d) sharpest SSC Constable GD-23/11/2021 Shift-III Ans. (a) : In the blank space of the above sentence, 'finest' would be appropriate. The other options give different meaning. Purest :- free of extraneous element. Tastiest :- taste perception, have flavor. Sharpest :- acute, pointed Correct Sentence - This is the finest clock that I have ever seen. 464. Select the most appropriate option to fill in the blank. Friendship increases happiness and ______ misery. (a) diminishes (b) determines (c) distinguishes (d) delegates SSC Constable GD-23/11/2021 Shift-III Ans. (a) : According to the sense of the sentence, 'diminishes' would be appropriate in the blank space. The other options give different meaning. Determines :- find out, set Distinguishes :- mark as different Delegates :- depute, designate. Correct sentence - Friendship increases happiness and diminishes misery. 465. Select the most appropriate option to fill in the blank. We have decided to go on a long ______ with our friends. (a) travel (b) passage (c) visit (d) drive SSC Constable GD-23/11/2021 Shift-III Ans. (d) : According to the sense of the above sentence, 'drive' would be appropriate in the blank space. The other options give different meaning. Travel :- journey Passage :- passing Visit :- a temporary stay. Correct Sentence - We have decided to go on a long drive with our friends.

253

YCT

466. Select the most appropriate option to fill in the blank. A soft breeze blows as the sun ______ in the distant waters. (a) basks (b) glazes (c) shimmers (d) ramifies SSC Constable GD-22/11/2021 Shift-II Ans. (c) : In the blank space of the above sentence, 'shimmers' gives the suitable meaning. The other options give different meaning. Bask :- enjoy, relish. Glaze :- glass over, shince, Ramifies :- separate. Correct sentence– A soft breeze blows as the sun shimmers in the distant waters. 467. Select the most appropriate option to fill in the blank. Hitopadesha tales are written in a readerfriendly style, which perhaps ______ to their success. (a) supplied (b) provided (c) contributed (d) arranged SSC Constable GD-22/11/2021 Shift-II Ans. (c) : In the blank space of the above sentence, 'contributed' will be used. The other options give different meaningsupply :- give what is desired or needed Provide :- give something useful or necessary. Arrange :- set up Correct sentence– Hitopadesha tales are written in a reader - friendly style, which perhaps contributed to their success. 468. Select the most appropriate option to fill in the blank. Many students are ______ by the life of great leaders. (a) interested (b) involved (c) inspired (d) infected SSC Constable GD-22/11/2021 Shift-II Ans. (c) : In the blank space of the above sentence, 'inspired' will be used. The other options give different meaning. Interested :- curious. Involved :- attached. Infected :- unhealthful. Correct Sentence– Many students are inspired by the life of great leaders. 469. Select the most appropriate option to fill in the blank. Visitors come from all over the world to ______ the Shalimar Gardens. (a) perceive (b) see (c) observe (d) look SSC Constable GD-18/11/2021 Shift-I Ans. (b) : According to the sense of the above sentence, 'see' will be used in the blank space. The other options give different meaning. Fill in the Blanks

Perceive :- comprehend Observe :- watch attentively Look :- seem. appear Correct sentence– Visitors come from all over the world to see the Shalimar Gardens. 470. Select the most appropriate option to fill in the blank. He ran towards his mother ______ and hugged her. (a) partially (b) spitefully (c) eventually (d) gaily SSC Constable GD-18/11/2021 Shift-I Ans. (d) : According to the sense of the above sentence, 'gaily' will be used in the blank space. The other options give different meaning. Partially :- imcompletely, in part, Spitefully :- despitefully Eventually :- finally. Correct sentence– He ran towards his mother gaily and hugged her. 471. Select the most appropriate option to fill in the blank. This antique sculpture is indeed ______. (a) harmless (b) helpless (c) priceless (d) endless SSC Constable GD-18/11/2021 Shift-I Ans. (c) : According to the sense of the above sentence, 'priceless' will be used in the blank space. The other options give different meaning. Harmless :- benign, safe. Helpless :- dependent. Endless :- continuous. Correct Sentence– This antique sculpture is indeed priceless. 472. Select the most appropriate option to fill in the blank. The socio-economic policy has been the ______ of much debate. (a) subject (b) talk (c) object (d) concept SSC Constable GD-17/11/2021 Shift-II Ans. (a) : According to the sense of the above sentence, 'subject' would be appropriate in the blank space. The other options give different meaning. Talk :- speak. Object :- thing Concept :- An abstract or general idea inferred or derived from specific instances. Correct Sentence The socio-economic policy has been the subject of much debate. 473. Select the most appropriate option to fill in the blank. For a member to borrow money from the association, three ______ need to be met. (a) accounts (b) conditions (c) points (d) restrictions SSC Constable GD-17/11/2021 Shift-II

254

YCT

Ans. (b) : According to the sense of the above sentence, 'Conditions' will be used in blank space. The meaning of other options is – Accounts :- history Points :- marks Restrictions :- limitation. Correct Sentence For a member to borrow money from the association, three conditions need to be met. 474. Select the most appropriate option to fill in the blank. Teams from all over India _____ in the talent show. (a) provided (b) participated (c) produced (d) prohibited SSC Constable GD-17/11/2021 Shift-II Ans. (b) : According to the sense of the above sentence, 'participated' would be appropriate in the blank space. Provided :- supply Produce :- grow, develop Prohibit :- forbid, disallow Correct Sentence Team from all over India participated in the talent show. 475. Select the most appropriate option to fill in the blank. When she ____ the hall, she was pleasantly surprised. (a) arrived (b) intruded (c) appeared (d) entered SSC Constable GD-13/12/2021 Shift-III Ans. (d) : In the blank space of the above sentence, 'entered' would be appropriate. The other options give different meaning. Arrived :- reach a destination Intruded :- enter without invitation, Appeared :- come out Correct sentence– when she entered the hall she was pleasantly surprised 476. Select the most appropriate option to fill in the blank. My grandmother is _____ to the bed. (a) concealed (b) confined (c) concerned (d) confirmed SSC Constable GD-13/12/2021 Shift-III Ans. (b) : In the blank space of the above sentence, 'Confined' gives suitable meaning. The other options give different meaning. Concealed :- hidden Concerned :- afraid Confirmed :- unchangeable. Correct sentence – My grandmother is confined to the bed. 477. Select the most appropriate option to fill in the blank. Traffic ____ is a very serious problem in metro cities. (a) congestion (b) collection (c) gathering (d) chain SSC Constable GD-13/12/2021 Shift-III Fill in the Blanks

Ans. (a) : In the blank space of the above sentence, 'Congestion' gives the proper meaning. The other options give different meaning. Collection :- Accumulation, assemblage. Gathering :- assembly. Chain :- connect or arrange into a chain Correct Sentence– Traffic congestion is a very serious problem in metro cities. 478. Select the most appropriate option to fill in the blank. There are five ––––– members of the United Nations. (a) forever (b) immortal (c) durable (d) permanent SSC Constable GD-02/12/2021 Shift-I Ans. (d) : According to the sense of the above sentence, 'Permanent' will be used in the blank space. The meaning of other options is – Forever :- always, constantly. Immortal :- Infinite. Durable :- long-lived. 479. Select the most appropriate option to fill in the blank. Two young men ––––– the chain from the woman's neck and ran away. (a) pulled (b) squeezed (c) pushed (d) turned SSC Constable GD-02/12/2021 Shift-I Ans. (a) : According to the sense of the above sentence, 'pulled' will be used in the blank space. The meaning of other options is – Squeezed :- press or force. Push :- move strenuously and with effort. Turned :- inverted, overturned. 480. Select the most appropriate option to fill in the blank. The –––––– was given a stipend during his period of training. (a) owner (b) intern (c) employer (d) entrant SSC Constable GD-02/12/2021 Shift-I Ans. (b) : According to the sense of the above sentence, 'intern' will be used in the blank space. The meaning of the other options is – Owner :- someone who owns a business. Employer :- a person or firm that employs workers. Entrant :- someone who enters. 481. Select the most appropriate option to fill in the blank. The man felt exhausted after climbing a ______ of stairs. (a) flight (b) heap (c) pack (d) fleet SSC Constable GD-16/11/2021 Shift-III

255

YCT

Ans. (a) : In the blank space of the above sentence, 'Flight' will be used because 'flight of stairs' is a phrase which means– two or more steps linked together by common riser and treads. Correct SentenceThe man felt exhausted after climbing a flight of stairs. 482. Select the most appropriate option to fill in the blank. The Egyptians developed ways to prevent the dead body from ______. (a) destroying (b) drooping (c) dying (d) decaying SSC Constable GD-16/11/2021 Shift-III Ans. (d) : In the blank space of the above sentence, 'decaying' will be used. Because according to the sense of the sentence, it gives proper meaning. 483. Select the most appropriate option to fill in the blank. She is on six months’ ______ leave. (a) paternal (b) maternity (c) paternity (d) material SSC Constable GD-16/11/2021 Shift-III Ans. (b) : In the blank space of the above sentence, 'Maternity' will be used. Because according to the sense of the above sentence, it gives proper meaning. Correct Sentence She is on six months' maternity leave. 484. Select the most appropriate option to fill in the blank. Suddenly, she jumped up and ______ his arm. (a) occupied (b) snatched (c) grabbed (d) captured SSC Constable GD-14/12/2021 Shift-III Ans. (c) : According to the sense of the above sentence, 'Grabbed' (seize, catch) will be used in the blank space. The other options gives different meaning. Occupied :- busy, engaged Snatched :- kidnapped. Captured :- catch. 485. Select the most appropriate option to fill in the blank. The little girl walked slowly ______ her burden. (a) holding (b) loading (c) bearing (d) storing SSC Constable GD-14/12/2021 Shift-III Ans. (c) : According to the sense of the above sentence, 'bearing' will be used. The other options give different meaning. Holding :- sustaining Loading :- burdening Storing :- stocking correct sentence– The little girl walked slowly bearing her burden. 486. Select the most appropriate option to fill in the blank. I was ______ by the sound of the doorbell. (a) enabled (b) awakened (c) enhanced (d) activated SSC Constable GD-14/12/2021 Shift-III Fill in the Blanks

Ans. (b) : According to the sense of the above sentence, 'awakened' will be used. The meaning of other options is – Enabled :- able for some task. Enhanced :- ameliorate, increased. Activated :- excited. Correct Sentence– I was awakened by the sound of the doorbell. 487. Select the most appropriate option to fill in the blank. Pahari is the most ______ spoken language in Himachal Pradesh. (a) quickly (b) carefully (c) strictly (d) commonly SSC Constable GD-14/12/2021 Shift-II Ans. (d) : According to the sense of the above sentence, 'Commonly' will be used in the blank space. The other options give different meaning. Quickly :- no delay, with rapid movements. Carefully :- cautiously. Strictly :- rigorously. 488. Select the most appropriate option to fill in the blank. The little girl tried to ______ the singer. (a) imply (b) imitate (c) languish (d) emit SSC Constable GD-14/12/2021 Shift-II Ans. (b) : According to the sense of the above sentence, 'Imitate' would be appropriate. The meaning of other options is – Imply :- have as a logical consequence Languish :- become feeble. Emit :- give off 489. Select the most appropriate option to fill in the blank. Making notes will ______ you to organise your thoughts in a better way. (a) disable (b) capable (c) adjustable (d) enable SSC Constable GD-14/12/2021 Shift-II Ans. (d) : According to the sense of the above sentence, 'enable' would be appropriate in the blank space. The meaning of other options is – Disable :- invalid, disenable. Capable :- able, competent. Adjustable :- changeable, adoptable. Correct sentence - Making notes will enable you to organise your thoughts in a better way. 490. Select the most appropriate option to fill in the blank. Films based on books are more ____ than original screenplays. (a) better (b) favourite (c) popular (d) different SSC Constable GD-10/12/2021 Shift-III Ans. (c) : In the blank spaces of the above sentence, 'popular' will be used because according to the sense of this sentence, it gives suitable meaning. Correct Sentence– Films based on books are more popular than original screenplays.

256

YCT

491. Select the most appropriate option to fill in the blank. The leaves of plants _____ soon if they do not get enough water. (a) wilt (b) ebb (c) melt (d) erode SSC Constable GD-10/12/2021 Shift-III Ans. (a) : In the blank space of the above sentence, 'wilt' will be used because according to the sense of this sentence , it gives proper meaning. Correct Sentence– The leaves of plants wilt soon if they do not get enough water. 492. Select the most appropriate option to fill in the blank. You should pay a little more attention to your ____. (a) apportion (b) apparition (c) appearance (d) apprehension SSC Constable GD-09/12/2021 Shift-II Ans. (c) : In the blank space of the above sentence, 'Appearance' will be used because according to the sense of sentence, it gives proper meaning. The meaning of the other options is– Apportion :- allocate, deal. Apparition :- fantasm, shadow. Apprehension :- misgiving, fearful expectation. 493. Select the most appropriate option to fill in the blank. The thief was taken to the police station but the ____ revealed nothing. (a) provocation (b) interrogation (c) objection (d) discussion SSC Constable GD-09/12/2021 Shift-II Ans. (b) : According to the sense of the above sentence, 'Interrogation' would be appropriate in the blank space. The meaning of other options is – Provocation :- incitation, incitement Objection :- protest, dissent. Discussion :- discourse, treatment. 494. Select the most appropriate option to fill in the blank. Candidates who successfully complete the entrance test will be ____ for an interview next week. (a) encouraged (b) asked (c) invited (d) required SSC Constable GD-09/12/2021 Shift-II Ans. (c) : According to the sense of the above sentence, 'Invited' would be appropriate in the blank space. The other options give different meaning. Correct sentence– Candidates who successfully complete the entrance test will be invited for an interview next week. 495. Select the most appropriate option to fill in the blank. He was ––––– from Turkey for his public condemnation of the Shah. (a) exiled (b) excluded (c) evacuated (d) escaped SSC Constable GD-08/12/2021 Shift-I Fill in the Blanks

Ans. (a) : According to the sense of the above sentence, 'exiled' will be used in the blank space. The other options give different meaning. Exclude :- debar, keep out. Evacuated :- empty. Escape :- get away. 496. Select the most appropriate option to fill in the blank. He asked me –––––– I knew you. (a) although (b) since (c) whether (d) unless SSC Constable GD-08/12/2021 Shift-I Ans. (c) : In the given sentence, it is clear that the sentence is in 'Indirect narration'. In Indirect, conjunction 'if or whether' is used with Interrogative sentence in the blank space. Correct sentence– He asked me whether I knew you. 497. Select the most appropriate option to fill in the blank. This case is limited to state –––––. (a) radicalisation (b) jurisdiction (c) preparation (d) ramification SSC Constable GD-08/12/2021 Shift-I Ans. (b) : According to the sense of the above sentence, 'jurisdiction' would be appropriate in the blank space. The other options give different meaning. Radicalisation :- The action or process of making somebody more extreme or radical in their opinions an political. Preparation :- planning, readying. Ramification :- complication 498. Select the most appropriate option to fill in the blank. The little girl –––– a puppy from the street. (a) preserved (b) transferred (c) passed (d) rescued SSC Constable GD-07/12/2021 Shift-II Ans. (d) : According ot the sense of the above sentence, 'rescued' will be used in the blank space. The other options give different meaning. Preserved :- conserved, protected. Transfer :- move from one place to another. Passed :- go across Correct sentence – The little girl rescued a puppy from the street. 499. Select the most appropriate option to fill in the blank. The systems –––––– on back-up battery when the electricity goes off. (a) operate (b) activate (c) drive (d) control SSC Constable GD-07/12/2021 Shift-II Ans. (a) : According to the sense of the above sentence, 'operate' will be used in the blank space. The other options give different meaning. Activate :- make active or more active Drive :- steer, ride. Control :- command. Correct sentence – The systems operate on back-up battery when the electricity goes off.

257

YCT

500. Select the most appropriate option to fill in the blank. It is a well-known fact that air pollution has a terrible –––––– on the health of people. (a) effect (b) shock (c) problem (d) force SSC Constable GD-07/12/2021 Shift-II Ans. (a) : According to the sense of the above sentence, 'effect' will be used in the blank space. The other options give different meaning. Shock :- an unpleasant or disappointing. Problem :- trouble, difficulty. Force :- strength. Correct sentence – It is a well-known fact that air pollution has a terrible effect on the health of people. 501. Select the most appropriate option to fill in the blank. Several companies have come up with technology at –––––– prices to suit consumers of all segments. (a) acceptable (b) agreeable (c) amenable (d) affordable SSC Constable GD-06/12/2021 Shift-III Ans. (d) : In the blank space of the above sentence, 'affordable' will be used because according to the sense of the above sentence, it gives proper meaning. Correct sentence – Several companies have come up with technology at affordable prices to suit consumers of all segments. 502. Select the most appropriate option to fill in the blank. The old man has earned the ––––– of settling disputes peacefully. (a) renown (b) reputation (c) resistance (d) restoration SSC Constable GD-06/12/2021 Shift-III Ans. (b) : According to the sense of the above sentence, 'reputation' will be used in the blank space. The other options give different meanings. Correct sentence – The old man has earned the reputation of settling disputes peacefully. 503. Select the most appropriate option to fill in the blank. Health cannot be ____at any cost. (a) rejected (b) neglected (c) despised (d) deserted SSC Constable GD-03/12/2021 Shift-II Ans. (b) : In the blank space of the above sentence, 'neglected' will be used in the blank space. Because it gives the suitable meaning for this sentence. Rejected :- disapproved. refused. Despised :- contemned, scorned. Deserted :- desolated. Correct sentence Health cannot be neglected at any cost. Fill in the Blanks

504. Select the most appropriate option to fill in the blank. He was fined for _____traffic rules. (a) refusing (b) obeying (c) violating (d) omitting SSC Constable GD-03/12/2021 Shift-II Ans. (c) : In the blank space of the above sentence, 'violating' will be used. Because it gives suitable meaning for this sentence. the other options give different meaning. Refusing :- declining. Obeying :- to be obedient Omitting :- exclude, except. Correct sentence He was fined for violating traffic rules. 505. Select the most appropriate option to fill in the blank. Giant pandas are black-and-white Chinese bears that are on the _____of extinction. (a) line (b) verge (c) margin (d) peak SSC Constable GD-03/12/2021 Shift-II Ans. (b) : In the blank space of the above sentence, 'verge' is used in the blank space. The other options give different meaning. Line :- a length without breadth or thickness. Margin :- border, perimeter Peak :- point, acme, apex, zenith. Correct sentence Giant pandas are blank-and-white Chinese bears that are on the verge of extinction. 506. Select the most appropriate option to fill in the blank. The child _______that he had broken the vase. (a) collected (b) requested (c) demanded (d) admitted SSC Constable GD-01/12/2021 Shift-I Ans. (d) : According to the sense of the above sentence, 'admitted' would be appropriate in the blank space. The meaning of other options is – Collected :- gather up, compile. Requested :- call for. Demanded :- require, need. Correct Sentence– The Child admitted that he had broken the vase. 507. Select the most appropriate option to fill in the blank. In the year 1973, there were only 9 tiger reserves, but ______the number has risen to fifty. (a) gradually (b) technically (c) adamantly (d) consequently SSC Constable GD-01/12/2021 Shift-I Ans. (a) : According to the sense of the above sentence, 'gradually' will be used in the blank space. The other options give different meaning. Technically :- with regard to technique. Adamantly :- Inflexibly. Consequently :- as a result. Correct sentence– In the year 1973, there were only 9 tiger reserves, but gradually the number has risen to fifty.

258

YCT

508. Select the most appropriate option to fill in the blank. Look at the hippo ______about happily in the river. (a) stammering (b) splashing (c) crackling (d) sputtering SSC Constable GD-01/12/2021 Shift-I Ans. (b) : According to the sense of the above sentence, 'splashing' would be appropriate in the blank space. The other options give different meaning. Stammering :- bumble, falter Crackling :- crackle (the sharp sound) Sputtering :- the noise of something spattering. Correct sentence– Look at the hippo splashing about happily in the river. 509. Select the most appropriate option to fill in the blank. It is claimed that if you take a certain homeopathic medicine, it will .......... you against the coronovirus. (a) shelter (b) immune (c) reinforce (d) fortify SSC CHSL (Tier-I) –04/08/2021 (Shift-I) Ans. (d) : In the blank space of the above sentence, 'fortify' will be used because according to the sense of the sentence, it gives the suitable meaning. Correct sentenceIt is claimed that if you take a certain homeopathic medicine, it will fortify you against the coronavirus. 510. Select the most appropriate word to fill in the blank. Debjyoti's short animation film .......... highlights the stark contrast between the rich and the poor in India. (a) poignantly (b) strictly (c) carelessly (d) sweetly SSC CHSL (Tier-I) –04/08/2021 (Shift-I) Ans. (a) : In the blank space of the above sentence, 'poignantly' would be appropriate because according to the sense of the sentence, it gives the suitable meaning. Correct sentenceDebjyoti's short animation film poignantly highlights the stark contrast between the rich and the poor in India 511. Today you may not .......... what you hear, but that is the truth and you have to accept it. (a) answer (b) reply (c) enjoy (d) like SSC CHSL (Tier-I) –05/08/2021 (Shift-I) Ans. (d) : In the blank space of the above sentence, 'like' will be used because according to the sense of the sentence, it gives the proper meaning. Correct sentenceToday you may not like what you hear, but that is the truth and you have to accept it. 512. Fill in the blank with most appropriate option. The craze for taking selfies has only increased our ____ with our own image. (a) expression (b) possession (c) obsession (d) impression SSC CHSL (Tier-I) –16/04/2021 (Shift-I) Fill in the Blanks

Ans. (c) : According to the sense of the above sentence, 'obsession' will be used in the blank space. The other options give different meaning Possession :- ownership Expression :- saying, reflexion. Impression :- effect. Correct sentenceThe craze for taking selfies has only increased our obsession with our own image. 513. Select the most appropriate option to fill in the blank. People have overcome poverty, drug addiction, abuse, divorce, mental illness and virtually every ____ known to man. (a) Defiance (b) Dispute (c) Objection (d) Challenge SSC CHSL (Tier-I) –16/04/2021 (Shift-I) Ans. (d) : According to the sense of the above sentence, 'Challenge' would be appropriate in the blank space. The other options are not giving the suitable meaning. Dispute :- contravention, conflict. Defiance :- rebelliousness Objection :- protest. 514. Select the most appropriate option to fill in the blank. The experience of having climbed to the Mt. Everest changes you ____. (a) rarely (b) regularly (c) completely (d) frequently SSC CHSL (Tier-I) –15/04/2021 (Shift-I) Ans. (c) : In the blank space of the above sentence, 'completely' would be appropriate. The other options give different meaning. Rarely :- seldom Regularly :- in a regular manner. Frequently :- often. 515. Select the most appropriate word to fill in the blank. These days you can transfer money ____ from one account to another. (a) clearly (b) instantly (c) publicly (d) heavily SSC CHSL (Tier-I) –15/04/2021 (Shift-I) Ans. (b) : According to the sense of the above sentence, 'instantly' would be appropriate in the blank space. The other options give different meaning. Heavily :- with great force. Publicly :- In a manner accessible to or observable by the public. Clearly :- with doubt or question. 516. Fill in the blank with the most appropriate option. She made an ____ to break the record for long jump. (a) attack (b) intention (c) exercise (d) attempt SSC CHSL (Tier-I) –13/04/2021 (Shift-I) Ans. (d) : In the blank space of the above sentence, 'attempt' would be appropriate because according to the sense of the sentence, it gives suitable meaning. The meaning of other options is– Exercise :- practice, example. Intention :- aim, mission. Attack :- an offensive against an enemy.

259

YCT

517. Select the most appropriate word to fill in the blank. As far as he is _____, he will love to attend the party. (a) interested (b) concerned (c) confined (d) anxious SSC CHSL (Tier-I) –13/04/2021 (Shift-I) Ans. (b) : According to the sense of the above sentence, 'concerned' will be used in the blank space. Because in the sentence, 'as far as he is concerned' is suitable for this sentence. The other options give different meaning. Interested :- Curious Confined :- bound, limit Anxious :- uneasy. 518. Fill in the blank with appropriate word. This part of the sea ........ with sharks. (a) expands (b) crowds (c) suffices (d) teems SSC CHSL (Tier-I) –06/08/2021 (Shift-I) Ans. (d) : In the blank space of the above sentence, 'teems' will be used because it gives the sense of the majority. Correct sentenceThis part of the sea teems with sharks. 519. Select the most appropriate word to fill in the blank. It is ......... to see so many of our fellow Indian helps the migrants in their struggle to go home. (a) savouring (b) heartening (c) reuniting (d) relishing SSC CHSL (Tier-I) –06/08/2021 (Shift-I) Ans. (b) : In the blank space of the above sentence, 'heartening' will be used because it gives the sense of the happy. Correct sentenceIt is heartening to see so many of our fellow Indian helps the migrants in their struggle to go home. 520. Select the most appropriate word to fill in the blank. Have you thought about the ............ of the recent decision you took? (a) conclusions (b) complexes (c) consequences (d) concerns SSC CHSL (Tier-I) –09/08/2021 (Shift-I) Ans. (c) : According to the sense of the above sentence, 'consequences' would be appropriate in the blank space. The other options give difference meaning. Concern :- fear, worry. Complexes :- compound Conclusion :- close, finale. Correct sentenceHave you thought about the consequences of the recent decision you took? 521. Select the most appropriate word to fill in the blank. The headmaster .......... on the stage and gave a short speech. (a) prescribed (b) appeared (c) delighted (d) remembered SSC CHSL (Tier-I) –09/08/2021 (Shift-I) Ans. (b) : In the blank space of the above sentence, 'appeared' will be used. Because according to the sense of the sentence, it gives suitable meaning. Fill in the Blanks

The other options give different meaning. Prescribed :- arbitrary. Remembered :- call back, recall. Delighted :- a feeling of extreme pleasure. 522. Select the most appropriate word to fill in the blank. We admit students with ––––––academic backgrounds like life sciences, physical sciences and business and commerce. (a) simple (b) similar (c) same (d) diverse SSC CHSL (Tier-I) –19/04/2021 (Shift-I) Ans. (d) : In the blank space of the above sentence, 'Diverse' will be used because according to the sense of the sentence, it gives suitable meaning. The other options give different meaning. Simple :- easy Similar :- alike Same :- equal Correct sentenceWe admit students with diverse academic backgrounds like life sciences, physical sciences and business and commerce. 523. Select the most appropriate word to fill in the blank. Please complete the short questionnaire ––––– and return it to us. (a) excluded (b) attached (c) concluded (d) separated SSC CHSL (Tier-I) –19/04/2021 (Shift-I) Ans. (b) : In the blank space of the above sentence, 'attached' will be used because according to the sense of the sentence, it gives proper meaning. The other options give different meaning. Exclude :- debar, omit Conclude :- close Separate :- divide, part Correct sentencePlease complete the short questionnaire attached and return it to us. 524. Select the most appropriate option to fill in the blank. His –––––– of the topic was so good that students had few doubts at the end of the class. (a) exposition (b) picturisation (c) exhibition (d) qualification SSC CGL (Tier-I) –24/08/2021 (Shift-I) Ans. (a) : According to the sense of the sentence, 'exposition' would be appropriate in the blank space. The other options give different meaning. Picturisation :- the process of adapting a story or film as a film. Qualification :- ability. Exhibition :- exposition Correct sentenceHis exposition of the topic was so good that students had few doubts at the end of the class. 525. Select the most appropriate option to fill in the blank. The government has hardened its ––––––– against the agitators and imposed a curfew in the area. (a) bearing (b) manner (c) orientation (d) stance SSC CGL (Tier-I) –23/08/2021 (Shift-I)

260

YCT

Ans. (d) : According to the sense of the sentence, 'stance' (position) would be appropriate in the blank space. The other options give different meaning. Bearing :- a person's manner or conduct. Manner :- Style Orientation :- preference. Correct sentenceThe government has hardened its stance against the agitators and imposed a curfew in the area. 526. Select the most appropriate option to fill in the blank. We are bewildered by the power which science has suddenly placed in our laps, and we are ––– by the realisation of our unpreparedness to deal with a crisis. (a) condemned (b) demeaned (c) humbled (d) proud SSC CGL (Tier-I) –23/08/2021 (Shift-I) Ans. (c) : According to the sense of the sentence, 'humbled' (gentle) will be used in the blank space. The other options give different meaning. Condemned :- objurgate, decry. Demeaned :- disgrace. Proud :- arrogant Correct sentenceWe are bewildered by the power which science has suddenly placed in our laps, and we are humbled by the realisation of our unpreparedness to deal with a crisis. 527. Select the most appropriate option to fill in the blank. A ––––––– souvenir will be released on the occasion of the World Meet. (a) voluminous (b) considerable (c) generous (d) bountiful SSC CGL (Tier-I) –16/08/2021 (Shift-I) Ans. (a) : According to the sense of the above sentence, 'voluminous' (huge) would be appropriate. The meaning of other options is– Considerable :- appreciable. Generous :- Charitable. Bountiful :- plentiful 528. Select the most appropriate option to fill in the blank. You need not –––––– so much fuss about wearing a mask when you go out. (a) generate (b) make (c) do (d) cause SSC CGL (Tier-I) –16/08/2021 (Shift-I) Ans. (b) : In the blank space of the above sentence, 'make' will be used because 'need not' is used as a modal and the first form of the verb (V1) is used after modal verb and 'make a fuss' is used as a phrase. Note :- (Need not do something) Correct sentenceYou need not make so much fuss about wearing a mask when you go out. 529. Select the most appropriate option to fill in the blank. Some people were tired of the long –––––– in front of the mall and left without entering. (a) waste (b) wait (c) waist (d) weight SSC CGL (Tier-I) –18/08/2021 (Shift-I) Fill in the Blanks

Ans. (b) : According to the sense of the above sentence, 'wait' will be used in the blank space. The other options give different meaning. Waste :- consume, squander. Waist :- shank. Weight :- burden, burthen. 530. Select the most appropriate option to fill in the blank. Keats and Shelley were poets of the same period; they were ______. (a) contemporaries (b) acquaintances (c) colleagues (d) associates SSC CGL (Tier-I) –18/08/2021 (Shift-I) Ans. (a) : According to the sense of the above sentence, 'contemporaries' will be used in the blank space. The other options give different meaning. Acquaintances :- familiarity. Colleagues :- co-workers, fellow. Associate :- comrade. Correct sentenceKeats and Shelley were poets of the same period; they were contemporaries. 531. Select the most appropriate option to fill in the blank. Credit card –––––– has increased in recent years. (a) fluke (b) freed (c) fraud (d) flake SSC CGL (Tier-I) –20/08/2021 (Shift-I) Ans. (c) : According to the sense of the above sentence, 'fraud' would be appropriate. The other options give different meaning. Fluke :- good luck. Freed :- unbound, independent. Flake :- peel, fleck 532. Select the most appropriate option to fill in the blank. My neighbour was admitted to the hospital when he developed an allergic ______ to some medicine he took. (a) result (b) rejoinder (c) reaction (d) retort SSC CGL (Tier-I) –20/08/2021 (Shift-I) Ans. (c) : According to the sense of the above sentence, 'reaction' will be used in the blank space. The other options give different meaning. Result :- consequence, event. Rejoinder :- counter Retort :- comeback, a quick reply to a question or remark. Correct sentenceMy neighbour was admitted to the hospital when he developed an allergic reaction to some medicine he took. 533. Select the most appropriate option to fill in the blank. Social isolation, overcrowding, the –––––– of our society and several factors are responsible for stress. (a) attractiveness (b) cheapness (c) affordability (d) competitiveness SSC CGL (Tier-I) –17/08/2021 (Shift-I)

261

YCT

Ans. (d) : According to the sense of the sentence, 'Competitiveness' will be used in the blank space. The other options give different meaning. Attractiveness :- attraction. Cheapness :- cut rate Affordability :- the cost or price of something. Correct sentenceSocial isolation, overcrowding, the competitiveness of our society and several factors are responsible for stress. 534. Select the most appropriate option to till in the blank. Advertisers often use rhyme and ––––––––. (a) alteration (b) alliteration (c) denunciation (d) designation SSC CGL (Tier-I) –17/08/2021 (Shift-I) Ans. (b) : According to the sense of the above sentence, 'Alliteration' will be used in the blank space. The other options give different meaning. Alteration :- change, modification. Denunciation :- denouncement. Designation :- identification. Correct sentenceAdvertisers often use rhyme and alliteration. 535. Select the most appropriate option to fill in the blank. Work and domestic –––––– made Kajal short tempered. (a) pressures (b) weights (c) gravities (d) forces SSC CGL (Tier-I) –13/08/2021 (Shift-I) Ans. (a) : According to the sense of the above sentence, 'Pressures' will be used in the blank space. The other options give different meaning. Weight :- burden Gravities :- soberness Force :- strength. Correct sentenceWork and domestic pressures made Kajal short tempered. 536. Select the most appropriate option to fill in the blank. India is formally moving ahead to –––––– 21 MIG-29 and 12 Sukhoi-30MKI fighters from Russia along with upgrades of their existing fleets. (a) accomplish (b) achieve (c) advance (d) procure SSC CGL (Tier-I) –13/08/2021 (Shift-I) Ans. (d) : According to the sense of the above sentence, 'procure' will be used in the blank space. The other options give different meaning. Accomplish :- attain, carry out. Achieve :- gain with effort. Advance :- bring forward. Correct Sentence India is formally moving ahead to procure 21 MIG-29 and 12 Sukhoi-30MKI fighters from Russia along with upgrades of their existing fleets. 537. Select the most appropriate option to fill in the blank. Bonsai gardening was first practiced in China but became _____ because of the Japanese. Fill in the Blanks

(a) frequent (b) prevailing (c) popular (d) customary SSC CPO-SI (Tier-II) 27/09/2019 (3 pm - 5 pm) Ans. (c) : In the blank space of the above sentence, 'popular' (famous) will be used in the blank space. The other options give different meaning. Correct sentence- Bonsai gardening was first practiced in China but became popular because of the Japanese. 538. Select the most appropriate option to fill in the blank. ______ was present in the hall took part in the voting process. (a) Whoever (b) Whichever (c) Wherever (d) Whatever SSC CPO-SI (Tier-II) 27/09/2019 (3 pm-5 pm) Ans. (a) : In the blank space of the above sentence, 'whoever' will be used in the blank space. The other options give different meaning. Correct sentence- Whoever was present in the hall took part in the voting process. 539. Select the most appropriate option to fill in the blank. During the night the express train picked up ______. (a) speed (b) distance (c) rate (d) quick SSC CPO-SI (Tier-II) 27/09/2019 (3 pm - 5 pm) Ans. (a) : According to the sense of the above sentence, 'speed' will be used in the blank space. The other options give different meaning. Correct sentence- During the night the express picked up speed. 540. Select the most appropriate option to fill in the blank. The committee’s suggestion was not acceptable to everyone as it was ______. (a) considerate (b) controversial (c) concrete (d) convenient SSC CPO-SI (Tier-II) 27/09/2019 (3 pm-5 pm) Ans. (b) : In the blank space of the above sentence, 'controversial' will be used in the blank space. The other options give different meaning. Correct sentence- The committee's suggestion was not acceptable to everyone as it was controversial. 541. Select the most appropriate option to fill in the blank. Generally people use ______ oils for their cooking. (a) cleared (b) refined (c) improved (d) washed SSC CPO-SI (Tier-II) 27/09/2019 (3 pm - 5 pm) Ans. (b) : According to the sense of the above sentence, 'Refined' would be appropriate. The other options give different meaning. Correct sentence- Generally people use refined oils for their cooking. 542. Select the most appropriate option to fill in the blank. The nationalists followed a policy of nonviolence to make their ______ successful. (a) encounter (b) struggle (c) politics (d) speeches SSC CPO-SI (Tier-II) 27/09/2019 (3 pm - 5 pm)

262

YCT

Ans. (b) : In the blank space of the above sentence, 'struggle' will be used. The other options give different meaning. Encounter :- come together. Politics :- social relations involving intrigue to gain authority or power. Speeches :- oral communication. Correct sentence– The nationalists followed a policy of non-violence to make their struggle success. 543. Select the most appropriate option to fill in the blank. Bengaluru is a beautiful city which ______ the modern with the traditional. (a) breaks (b) blends (c) grows (d) shares SSC CPO-SI (Tier-II) 27/09/2019 (3 pm - 5 pm) Ans. (b) : According to the sense of the above sentence, 'blends' will be used in the blank space. The other options give different meaning. Breaks :- Separate, split. Grows :- develop Shares :- percentage 544. Select the most appropriate option to fill in the blank. The pleasant ______ of the rain as it fell on the dust made me feel nostalgic. (a) incense (b) balm (c) aroma (d) perfume SSC CPO-SI (Tier-II) 27/09/2019 (3 pm - 5 pm) Ans. (c) : According to the sense of the above sentence, 'Aroma' will be used in the blank space. The other options give different meaning. Correct Sentence – The pleasant aroma of the rain as it fell on the dust made me feel nostalgic. 545. Select the most appropriate option to fill in the blank. Movie makers generally ______ historical facts to write their stories. (a) explain (b) correct (c) account (d) adapt

547. Select the most appropriate option to fill in the blank. We can meet tomorrow to ........ the chief guest's programme. (a) reveal (b) call (c) confirm (d) declare SSC Stenographer Grade C&D –05/02/2019 (3:5pm)

Ans. (c) : According to the sense of the above sentence, 'Confirm' will be used in the blank space. The other options give different meaning. Correct sentence- We can meet tomorrow to confirm the chief guest's programme. 548. Select the most appropriate option to fill in the blank. The library expects you to return each and every book that you have ........ (a) borrowed (b) kept (c) selected (d) lent SSC Stenographer Grade C&D –05/02/2019 (3:5pm)

Ans. (a) : According to the sense of the above sentence, 'borrowed' will be used in the blank space. The other options give different meaning. Correct sentence- The library expects you to return each and every book that you have borrowed. 549. Select the most appropriate option to fill in the blank. You must .......... your goals with all seriousness if you want to succeed in your life. (a) realize (b) pursue (c) chase (d) direct SSC Stenographer Grade C&D –05/02/2019 (3:5pm)

Ans. (b) : In the blank space of the above sentence, 'pursue' would be appropriate in the blank space. The other options give different meaning. Correct sentence- You must pursue your goals with all seriousness if you want to succeed in your life. 550. Select the most appropriate option to fill in the blank. These medicines will be ....... only if they are taken regularly. (a) useful (b) efficient SSC CPO-SI (Tier-II) 27/09/2019 (3 pm - 5 pm) (c) affective (d) effective SSC Stenographer Grade C&D –05/02/2019 (3:5pm) Ans. (d) : According to the sense of the above sentence, 'Adapt' will be used in the blank space. The Ans. (d) : In the blank space of the above sentence, 'effective' will be used in the blank space. other options give different meaning. Correct sentence- These medicines will be effective Correct sentence- Movie makers generally adapt only if they are taken regularly. historical facts to write their stories. 546. Select the most appropriate option to fill in the 551. In the following question, the sentence given blank. with blank to be filled in with an appropriate The truck was .......... the traffic and the word. Select the correct alternative out of the policeman asked the driver to move off. four and indicate it by selecting the (a) obstructing (b) hiding appropriate option. (c) closing (d) holding Time perception raises a number of ______ SSC Stenographer Grade C&D–05/02/2019 (3:5pm) puzzles, including what it means to say we Ans. (a) : In the blank space of the above sentence, perceive time. 'Obstructing' will be used in the blank space. Because (a) discriminating (b) intriguing (c) boring (d) befooling according to the sense of this sentence, it give suitable meaning. SSC CGL (Phase-II) 17/02/2018 The other options are– Ans. (b) According to the sense of the above sentence, Closing :- adjacent, oppressed. 'intriguing' will be used in the blank space. which is the Hiding :- concealing. gerund form of adjective. Holding :- belongings. Correct sentenceCorrect sentence- The truck was obstructing the Time perception raises a number of intriguing puzzles, including what it means to say we perceive time. traffic and the policeman asked the driver to move off. Fill in the Blanks

263

YCT

552.

In the following question, the sentence given with blank to be filled in with an appropriate word. Select the correct alternative out of the four and indicate it by selecting the appropriate option. The theoretical framework is finished by identifying indicators to be used in the ____ of the success of such policies. (a) evaluation (b) completion (c) formation (d) rotation SSC CGL (Phase-II) 17/02/2018 Ans. (a) According to the sense of the above sentence, 'evaluation' will be used as a noun in the blank space. Correct sentenceThe theoretical framework is finished by identifying indicators to be used in the evaluation of the success of such policies. 553. In the following question, the sentence given with blank to be filled in with an appropriate word. Select the correct alternative out of the four and indicate it by selecting the appropriate option. Many national surveys _____ that malnutrition is common in developed countries. (a) wheal (b) reveal (c) sheal (d) vineal SSC CGL (Phase-II) 17/02/2018 Ans. (b) According to the sense of the above sentence, 'reveal' would be appropriate in the blank space. The other options give different meaning. Correct sentenceMany national surveys reveal that malnutrition is common in developed countries. 554. In the following question, the sentence given with blank to be filled in with an appropriate word. Select the correct alternative out of the four and indicate it by selecting the appropriate option. Operant conditioning can be described as a learning _____ that is used to modify or change a person's behavior through experiences and consequences. (a) method (b) object (c) goal (d) suspect SSC CGL (Phase-II) 17/02/2018 Ans. (a) According to the sense of the above sentence, 'method' would be appropriate. The other options give different meaning. Correct sentenceOperant conditioning can be described as a learning method that is used to modify or change a person's behavior through experiences and consequences. 555. Select the alternative that is appropriate to fill the blank. The baby bird was _____ by its mother. (a) dissipated (b) discarded (c) abandoned (d) sidelined

556. Select the alternative that is appropriate to fill the blank. In case it rains, what______ arrangements have been made? (a) alternate (b) substitute (c) alternative (d) optional SSC Sel. Post Phase-VII (M.L.) 15.10.2019 (Shift-I)

Ans. (c) : According to the sense of the above sentence, 'Alternative' would be appropriate in the blank space. The other options give different meaning. Correct sentenceIn case it rains, what alternative arrangements have been made. 557. Select the most appropriate option to fill in the blank The doctors hope that the soldier’s wounds will soon____. (a) recover (b) mend (c) restore (d) heal SSC Sel. Post Phase-VIII (G.L.) 09.11.2020 (Shift-II)

Ans. (d) : According to the sense of the above sentence, 'Heal' would be appropriate in the blank space. The other options give different meaning. Correct sentence– The doctors hope that the soldier's wounds will soon heal. 558. Select the most appropriate option to fill in the blank. The family made _____ enquiries about his background. (a) disputed (b) disastrous (c) discreet (d) discarded SSC Sel. Post Phase-VIII (G.L.) 09.11.2020 (Shift-II)

Ans. (c) : According to the sense of the above sentence, 'Discreet' would be appropriate in the blank space. The other options give different meaning. Correct sentence:The family made discreet enquiries about his background. 559. Fill in the blank with the most appropriate word. ______ you deserve credit for the success of the show. (a) no less (b) such that (c) only if (d) none but SSC CPO-SI – 24/11/2020 (Shift-I) Ans. (d) : According to the sense of the above sentence, 'none but' would be appropriate in the blank space. SSC Sel. Post Phase-VII (M.L.) 15.10.2019 (Shift-I) The meaning of other options is– No less :- used to suggest often ironically that Ans. (c) : According to the sense of the above something is surprising or impressive. sentence, 'abandoned' would be appropriate in the Such that :- to the extend that. blank space. The other options give different meaning. Only if :- never except when. Correct sentenceCorrect sentence- none but you deserve credit for the The baby bird was abandoned by its mother. success of the show.

Fill in the Blanks

264

YCT

560. Fill in the blank with the most appropriate word. ______ of living life on his own terms, he made few compromises. (a) Desire (b) Desirous (c) Desired (d) Desiring SSC CPO-SI – 24/11/2020 (Shift-I) Ans. (a) : According to the sense of the above sentence, 'Desire' would be appropriate in the blank space. The other options give different meaning. Desirous :- having or expressing desire for something. (wishful) Desired :- wanted Desiring :- feel or have a desire. Correct sentence- Desire of living life on his own terms, he made few compromises. 561. Select the most appropriate option to fill in the blank. They postponed ––––––– on vacation due to adverse weather conditions. (a) going (b) for going (c) to go (d) to be gone SSC CPO-SI – 12/12/2019 (Shift-I) Ans : (a) In the blank space of the above sentence, 'going' will be used. Because 'postpone' is a transitive verb so here 'object' will be used, the gerund form of verb is worked as a noun. It is used as subject and object. Correct sentence- They postponed going on vacation due to adverse weather conditions. 562. Fill in the blank with the most appropriate word. The child made ____as he was getting late for school. (a) hurry (b) rapidity (c) haste (d) swiftness SSC CHSL (Tier-I) –05/07/2019 (Shift-II) Ans. : (c) In the blank space of the above sentence, 'haste' would be appropriate because 'made haste' is used as a idiom/phrase. Which means – hurry. Correct sentence- The child made haste as he was getting late for school. 563. Fill in the blank with the most appropriate word. She kept her clothes neatly arranged in the____ (a) godown (b) larder (c) wardrobe (d) warehouse SSC CHSL (Tier-I) –04/07/2019 (Shift-II) Ans : (c) In the blank space of the above sentence, 'wardrobe' would be appropriate. The meaning of other options is– Go-down :- a warehouse. Larder :- still room Warehouse :- a storehouse for goods and merchandise. Correct sentence- She kept her clothes neatly arranged in the wardrobe. 564. Fill in the blank with the most appropriate word. The old man has lost his memory. He is suffering from_____ Fill in the Blanks

(a) anaesthesia (b) acacia (c) ambrosia (d) amnesia SSC CHSL (Tier-I) –04/07/2019 (Shift-II) Ans : (d) In the blank space of the above sentence, 'amnesia' would be appropriate, which means 'memory less'. The meaning of other options is – Anaesthesia :-loss of bodily sensation with. Ambrosia :- nectar Acacia :- any of various spiny tress. Correct sentence- The old man has lost his memory. He is suffering from amnesia. 565. Fill in the blank with the most appropriate word. The_____ appears to be rather rough today for the ships to sail. (a) environment (b) climate (c) atmosphere (d) weather SSC CHSL (Tier-I) –04/07/2019 (Shift-III) Ans : (d) In the blank space of the above sentence, 'weather' would be appropriate. The meaning of other options is– Environment :- surround. Climate :- mood Atmosphere :- atmospheric state. Correct sentence- The weather appears to be rather rough today for the ships to sail. 566. Fill in the blank with the most appropriate word. The archaeologist made a _____discovery. (a) senile (b) sensational (c) sensible (d) sensitive SSC CHSL (Tier-I) –04/07/2019 (Shift-III) Ans : (b) In the blank space of the above sentence, 'sensational' would be appropriate. The meaning of other options – Senile :- mentally or physical infirm with age. Sensible :- conscious Sensitive :- delicate, responsive. Correct sentence- The archaeologist made a sensational discovery. 567. Select the most appropriate word to fill in the blank. Adali Khal is such a ____ village in Garhwal that hardly any tourist can reach there. (a) lonesome (b) dreaming (c) remote (d) lost SSC CHSL (Tier-I) –03/07/2019 (Shift-I) Ans : (c) In the blank space of the above sentence, 'remote' will be used as an adjective. The meaning of other options is – Lonesome :- lonely. Lost :- deep in thought. Dream :- daydream. Correct sentence- Adali Khal is such a remote village in Garhwal that hardly any tourist can reach there. 568. Select the most appropriate word to fill in the blank. She was ____ to have worked with some renowned scientists of the world. (a) favourable (b) affluent (c) charmed (d) fortunate SSC CHSL (Tier-I) –03/07/2019 (Shift-I)

265

YCT

Ans : (d) In the blank space of the above sentence, 'fortunate' will be used. The meaning of other options is – Favourable :- plausive, prosperous. Affluent :- wealthy. Charm :- enchant, fascinate. Correct sentence- She was fortunate to have worked with some renowned scientists of the world. 569. Select the most appropriate word to fill in the blank. He once had a very frightening ––––– with a wild bull. (a) appointment (b) encounter (c) enterprise (d) commitment SSC CHSL (Tier-I) –03/07/2019 (Shift-II) Ans. (b) : In the blank space of the above sentence, 'Encounter' will be used. The meaning of other options is– Appointment :- designation. Enterprise :- endeavour. Commitment :- loyalty, dedication. Correct sentence- He once had a very frightening encounter with a wild bull. 570. Select the most appropriate word to fill in the blank. During the elections political parties try to –––– ––––– their rivals in order to gain advantage. (a) laud (b) covet (c) commend (d) vilify SSC CHSL (Tier-I) –03/07/2019 (Shift-II) Ans. (d) : Here, in the blank space 'Vilify' will be used in the above sentence. The meaning of other options is – Laud :- praise, glorify or honor. Covet :- wish, long or crave for. Commend :- express a good opinion of. Correct sentence- During the elections political parties try to vilify their rivals in order to gain advantage. 571. Select the most appropriate word to fill in the blank. In view of his transfer to a far off place, he had to ––––––– his promotion. (a) resign (b) claim (c) assert (d) forgo SSC CHSL (Tier-I) –03/07/2019 (Shift-III) Ans. (d) : According to the sense of the above sentence, 'forgo' give the suitable meaning in the blank space. The other options give different meaning. Resign :- give up, vacate Claim :- demand, involve Assert :- affirm strongly. Correct sentence- In view of his transfer to a far off place, he had to forgo his promotion. 572. Select the most appropriate word to fill in the blank. The prisoner held up his fist in a defiant ––––– as he was led out of the courtroom. (a) wave (b) gesture (c) salute (d) token SSC CHSL (Tier-I) –03/07/2019 (Shift-III) Fill in the Blanks

Ans. (b) : According to the sense of the above sentence, 'Gesture' would be appropriate in the blank space. The other options give different meaning. Wave :- a movement up and down. Salute :- honor with a military ceremony. Token :- symbol. Correct sentence- The prisoner held up his fist in a defiant gesture as he was led out of the courtroom. 573. Select the most appropriate word to fill in the blank. People's will and determination can help them –––– most of the challenges of life. (a) overvalue (b) overdo (c) overview (d) overcome SSC CHSL (Tier-I) –02/07/2019 (Shift-I) Ans. (d) : According to the sense of the above sentence, 'overcome' (win) would be appropriate. The meaning of other options is – Overvalue :- assign too high a value to. Overdo :- exaggerate. Overview :- a general summary of subject. Correct sentence- People's will and determination can help them overcome most of the challenges of life. 574. Select the most appropriate word to fill in the blank. In spite of being born in an –––––– family, he chose to fight all odds and emerged as a notable statesman. (a) impeccable (b) immaculate (c) intelligent (d) impoverished SSC CHSL (Tier-I) –02/07/2019 (Shift-I) Ans. (d) : According to the sense of the above sentence, 'impoverished' (poor) gives the suitable meaning. The other options give different meaning. Impeccable :- faultless Immaculate :- spotless Intelligent :- smart, agile. Correct sentence- In spite of being born in an impoverished family, he chose to fight all odds and emerged as a notable statesman. 575. Select the most appropriate word to fill in the blank. There were not enough tinsmiths in Vienna to _______the lanterns for street lighting. (a) accomplish (b) compose (c) execute (d) manufacture SSC CHSL (Tier-I) –02/07/2019 (Shift-II) Ans : (d) In the blank space of the given sentence, manufacture' will be used because it gives appropriate meaning. The other options do not give the suitable meaning. The meaning of other options is– Accomplish :- carry out Compose :- compile, frame Execute :- put in effect. Correct sentence- There were not enough tinsmiths in Vienna to manufacture the lanterns for street lighting. 576. Select the most appropriate word to fill in the blank. Municipal corporations have started to take _____ of the problems of waste management.

266

YCT

(a) perception (b) cognizance (c) knowledge (d) attention SSC CHSL (Tier-I) –02/07/2019 (Shift-II) Ans : (b) According to the sense of the above sentence, 'Cognizance' would be appropriate in the blank space. The meaning of other options is – Perception :- insight, Knowledge :- deep understanding. Attention :- care Correct sentence- Municipal corporations have started to take cognizance of the problems of waste management. 577. Select the most appropriate word to fill in the blank. Financial institutions that had their premises in the World Trade Centre were crippled by the loss of ____ and hardware. (a) person (b) personnel (c) individual (d) group SSC CHSL (Tier-I) –02/07/2019 (Shift-III) Ans : (b) In the blank space of the above sentence; 'personnel' (working) would be appropriate. Meaning of other options isPerson- human, Individual, discrete Group- categpry, class, Correct sentence- Financial institutions that had their premises in the World Trade Centre were crippled by the loss of personnel and hardware. 578. Select the most appropriate word to fill in the blank. The thick smoke ____out of the landfill site is making the city's polluted air even more toxic. (a) tossing (b) rocking (c) billowing (d) burning SSC CHSL (Tier-I) –02/07/2019 (Shift-III) Ans : (c) In the blank space of the above sentence, 'billowing' will be used. The other options give different meaning. Tossing :- throw carelessly. Rocking :- shake, sway. Burning :- execution by fire. Correct sentence- The thick smoke billowing out of the landfill site is making the city's polluted air even more toxic. 579. Select the most appropriate word to fill in the blank. I had a stroke at an early age, but what I thought was a great ____at that time changed my life for the better. (a) injustice (b) indent (c) indication (d) incentive SSC CHSL (Tier-I) –01/07/2019 (Shift-III) Ans : (a) According to the sense of the above sentence, 'Injustice' would be appropriate in the blank space. The meaning of other options is – Indent :- to notch or give serrated edge to. Indication :- the act of indicating. Incentive :- bonus. Correct sentence- I had a stroke at an early age, but what I thought was a great injustice at that time changed my life for the better. Fill in the Blanks

580. Select the most appropriate word to fill in the blank. We have little choice early in life, but as we grow older choices______ (a) aggravate (b) adrift (c) abound (d) afloat SSC CHSL (Tier-I) –01/07/2019 (Shift-III) Ans : (c) In the blank space of the above sentence, 'abound' will be used. The meaning of other options is – Aggravate :- make worse, exasperate. Adrift :- purposeless, aimless Afloat :- flooded. Correct sentence- We have little choice early in life, but as we grow older choices abound. 581 Select the most appropriate word to fill in the blank. Even though we found ourselves in a ________ situation during our climb, we decided not to give up the expedition. (a) precarious (b) pleasant (c) permissive (d) plausible SSC CHSL (Tier-I) 26/10/2020 (Shift-II) Ans. (a) : According to the sense of the above sentence, 'precarious' will be used in the blank space. The meaning of other options is – Pleasant :- good natured, nice Permissive :- lenient Plausible :- credible Correct sentence- Even though we found ourselves in a precarious situation during our climb, we decided not to give up the expedition. 582. Select the most appropriate option for blank. When I first met my neighbours I took an _______ liking to them. (a) instant (b) instantly (c) instance (d) instaneously SSC CHSL (Tier-I) 26/10/2020 (Shift-II) Ans. (a) : According to the sense of the above sentence, 'instant' will be used in the blank space. The meaning of other options is– Instantly :- without delay. Instance :- example Correct sentence- When I first met my neighbours I took an instant liking to them. 583. Select the most appropriate option to fill in the blank. Astronomy enthusiasts and sky gazers will be able to watch six eclipses in 2020, but only three of them are expected to be _____ in India. (a) revealed (b) evident (c) exposed (d) visible SSC CHSL (Tier-I) 17/03/2020 (Shift-I) Ans. (d) In the blank space of the above sentence, 'visible' will be used in the blank space. The meaning of other options is– Revealed :- uncovered, unveil. Exposed :- exhibit Evident :- manifest, plain. Correct sentence- Astronomy enthusiasts and sky gazers will be able to watch six eclipses in 2020, but only three of them are expected to be visible in India.

267

YCT

584. Select the most appropriate word to fill in the blank. Parents who are very ______ later repent giving their children too much freedom. (a) strict (b) different (c) overbearing (d) permissive SSC CHSL (Tier-I) 16/10/2020 (Shift-III) Ans. (d) : In the blank space of the above sentence, 'permissive' would be appropriate. The other options give different meaning. Strict :- rigorous, rigid Different :- dissimilar, unlike. Overbearing :- naughty, disdainful. Correct sentence- Parents who are very permissive later repent giving their children too much freedom. 585. Select the most appropriate word to fill in the blank. I had a stroke at an early age, but what I thought was a great ______ at that time changed my life for the better. (a) lowliness (b) significance (c) setback (d) moodiness SSC CHSL (Tier-I) 16/10/2020 (Shift-II) Ans. (c) : According to the sense of the above sentence, 'setback' will be used in the blank space. The meaning of other options is– Lowliness :- humbleness Significance :- import. Moodiness :- a sullen gloomy feeling. Correct sentence- I had a stroke at an early age, but what I thought was a great setback at that time changed my life for the better. 586. Select the most appropriate word to fill in the blank. There was a range of ______ that kept tourists engaged during the International Film Festival in Goa. (a) moments (b) conversions (c) employments (d) activities SSC CHSL (Tier-I) 16/10/2020 (Shift-II) Ans. (d) : In the given sentence, 'activities' would be appropriate in the blank space. The meaning of other options is– Moments :- an indefinitely short time. Conversions :- change over transition. Employments :- employ, work. Correct sentence- There was a range of activities that kept tourists engaged during the International Film Festival in Goa. 587. Select the most appropriate word to fill in the blank. You've cleared the ______ examination, haven't you? (a) frequent (b) ability (c) conclusion (d) entrance SSC CHSL (Tier-I) 16/10/2020 (Shift-I) Ans. (d) : According to the sense of the above sentence, 'entrance' would be appropriate in the blank space. The meaning of other options is – Frequent :- regular Ability :- power Conclusion :- decision, verdict. Correct sentence- You've cleared the entrance examination, haven't you? Fill in the Blanks

588. Select the most appropriate word to fill in the blank. Ritu is an ____reader, I always find her with a book. (a) irritated (b) avid (c) erratic (d) inattentive SSC CHSL (Tier-I) 21/10/2020 (Shift-II) Ans. (b) : In the blank space of the above sentence, 'avid' will be used because according to the sense of the sentence, it gives the suitable meaning. The meaning of the other options is – Irritated :- annoyed Erratic :- fickle, changeable Inattentive :- absentminded, oblivious Correct sentence- Ritu is an avid reader, I always find her with a book. 589. Select the most appropriate word to fill in the blank. The police used tear gas to____ the crowd. (a) attack (b) disperse (c) disburse (d) collect SSC CHSL (Tier-I) 21/10/2020 (Shift-II) Ans. (b) : According to the sense of the above sentence, 'disperse' would be appropriate in the blank space. The meaning of other options is– Attack :- assail Disburse :- pay out Collect :- gather Correct sentence- The police used tear gas to disperse the crowd. 590. Select the most appropriate word to fill in the blank. The wall fell down with a big____ (a) bang (b) murmur (c) hiss (d) screech SSC CHSL (Tier-I) 19/03/2020 (Shift-III) Ans. (a) : In the blank space of the above sentence, 'big bang' would be appropriate. The meaning of other options is – Murmur :- grumble, mutter. Hiss :- siss Screech :- scream. Correct sentence- The wall fell down with a big bang. 591. Select the most appropriate word to fill in the blank. Although they searched for an hour, they were not able to find the____easliy. (a) location (b) whereabouts (c) space (d) meeting SSC CHSL (Tier-I) 19/03/2020 (Shift-III) Ans. (a) : In the blank space of the above sentence, 'location' will be used. The meaning of other options is – Whereabouts :- the general location where something is. Space :- outer space. Meeting :- get together. Correct sentence- Although they searched for an hour, they were not able to find the location easliy.

268

YCT

592. Select the most appropriate option to fill in the blank. The minister said, "Anyone using drones will have to register them on an____portal." (a) airport (b) airline (c) aeroplane (d) aviation SSC CHSL (Tier-I) 17/03/2020 (Shift-II) Ans. (d) : In the blank space of the above sentence, 'Aviation' would be appropriate. The other options give different meaning. Correct sentence- The minister said, "Anyone using drones will have to register them on an aviation portal." 593. Select the most appropriate option to fill in the blank. The strong Indian batting line-up will _____a big worry for the visiting cricket team. (a) put (b) place (c) pick (d) pose SSC CHSL (Tier-I) 17/03/2020 (Shift-II) Ans. (d) : In the blank space of the above sentence, 'Pose (lay)' would be appropriate. The meaning of other options is – Put :- set up Place :- blank space Pick :- choice, selection Correct sentence – The strong Indian batting line-up will pose a big worry for the visiting cricket team. 594. Select the most appropriate word to fill in the blank in the given sentence. This winter ______ weather conditions have led people to remain indoors. (a) averse (b) inverse (c) traverse (d) adverse SSC CHSL (Tier-I) 13/10/2020 (Shift-I) Ans. (d) : In the blank space of the above sentence, 'Adverse' (contrary) would be appropriate because in the blank space, the meaning of 'adverse weather' is appearing. The other options give different meaning. Averse :- antipathetic Inverse :- backward Traverse :- span. Correct sentence- This winter adverse weather conditions have led people to remain indoors. 595. Select the most appropriate word to fill in the blank in the given sentence. Some people think that the credit card culture encourages people to shop compulsively and merely ______ debts. (a) simulate (b) formulate (c) stimulate (d) accumulate SSC CHSL (Tier-I) 13/10/2020 (Shift-I) Ans. (d) : In the blank space of the above sentence, 'Accumulate' (collect) would be appropriate. The meaning of other options is – Simulate :- assume. Formulate :- develop Stimulate :- excited. Correct sentence- Some people think that the credit card culture encourages people to shop compulsively and merely accumulate debts. Fill in the Blanks

596. Select the most appropriate work to fill in the blank. There was a huge _________ in the stomach. (a) tumour (b) tuber (c) tremor (d) treasure SSC CHSL (Tier-I) 19/03/2020 (Shift-I) Ans. (a) : In the blank space of the above sentence, 'tumour' will be used because 'tumour' is used for stomach. The meaning of other options is – Tremor :- quake Treasure :- value, prize. Correct sentence- There was a huge tumour in the stomach. 597. Select the most appropriate word to fill in the blank. The _______ part of the issue is that the investigation officer himself turned out to be the culprit. (a) major (b) real (c) funniest (d) primary SSC CHSL (Tier-I) 19/03/2020 (Shift-I) Ans. (c) : In the blank space of the above sentence, 'funniest' will be used because article 'the' is used before superlative degree. The meaning of other options is – Major :- greater in scope or effect. Real :- generous Primary :- first, original. Correct sentence- The funniest part of the issue is that the investigation officer himself turned out to be the culprit. 598. Select the most appropriate word to fill in the blank. Success is the ________ of taking risks, everybody knows. (a) by-product (b) burden (c) tension (d) pressure SSC CHSL (Tier-I) 18/03/2020 (Shift-III) Ans. (a) : According to the sense of the above sentence, 'by-product' will be used in the blank space. The meaning of other options is– Burden :- load, weight. Tension :- stress Pressure :- force Correct sentence- Success is the by-product of taking risks, everybody knows. 599. Select the most appropriate word to fill in the blank. People readily accept any _______ of their leaders if supported by the media. (a) dispute (b) compromise (c) argument (d) concession SSC CHSL (Tier-I) 18/03/2020 (Shift-III) Ans. (c) : In the blank space of the above sentence, 'Argument' would be appropriate. The meaning of other options is – Dispute :- conflict Compromise :- accommodation, Concession :- allowance Correct sentence- People readily accepts any argument of their leaders if supported by the media.

269

YCT

600. Select the most appropriate option to fill in the Ans. (d) : According to the sense of the above blank. sentence, 'fracture' will be used in the blank space. Earthquake inspection _______ that the two The meaning of other options is – ancient buildings that remained intact after the Contamination :- pollute. Hiroshima bombing in Tokyo would not Infection :- contagion, transmission. withstand a strong tremor. Wound :- injury. (a) resumed (b) retracted Correct sentence- I had a broken bone in the hand (c) redirected (d) revealed SSC CHSL (Tier-I) 18/03/2020 (Shift-II) which the doctor called a fracture and suggested immediate surgery. Ans. (d) : According to the sense of the above sentence, 'revealed' will be used in the blank space. 604. Select the most appropriate word to fill in the The meaning of other options is – blank. Resumed :- restart Janki burnt her boats with her previous Retracted :- backward supervisor by ___him in public. Redirected :- to change the course or direction of. (a) flaunting (b) criticizing Correct Sentence(c) applauding (d) appreciating Earthquake inspection revealed that the two ancient SSC CHSL (Tier-I) 26/10/2020 (Shift-I) buildings that remained intact after the Hiroshima Ans. (b) : According to the sense of the above bombing in Tokyo would not withstand a strong tremor. sentence, 'criticizing' would be appropriate in the blank 601. Select the most appropriate option to fill in the space. blank. The meaning of other options is – The reporter asked the politician, "As a woman Flaunting :- display proudly. leader, what are some of the major _________ Applaud :- express approval of. that you face?" Appreciate :- apprise, revalue. (a) crunches (b) contests Correct sentence- Janki burnt her boats with her (c) crashes (d) challenges previous supervisor by criticizing him in public. SSC CHSL (Tier-I) 18/03/2020 (Shift-II) 605. Select the most appropriate word to fill in the Ans. (d) : According to the sense of the above blank. sentence, 'challenges' will be used in the blank space. The sales team ___the engineers for the While the meaning of other options is– Crunches :- reduce to small pieces, chew noisily. organization's failure to bag the mega deal. Contests :- a struggle between rivals. (a) liked (b) blamed Crashes :- break up, undergo damage. (c) appreciated (d) praised Correct SentenceSSC CHSL (Tier-I) 26/10/2020 (Shift-I) The reporter asked the politician, "As a woman leader, Ans. (b) : According to the sense of the above what are some of the major challenges that you face?" sentence, 'blamed' would be appropriate in the blank 602. Select the most appropriate word to fill in the space. blank. She has shown a great interest towards space The meaning of other options is – science, since her early children and a passion Liked :- likable (found pleasant or attractive) Appreciated :- prized, be grateful for. to ______ the outer space. (a) elicit (b) entertain Praised :- evaluate, extolment loud. (c) explore (d) enlighten Correct sentence- The sales team blamed the SSC CHSL (Tier-I) 18/03/2020 (Shift-I) engineers for the organization's failure to bag the mega Ans. (c) : According to the sense of the above deal. sentence, 'explore' (search) will be used in the blank 606. Select the most appropriate word to fill in the space. blank. The meaning of other options is – The politician was responsible for ____violence Elicit :- extract, draw at. in the university. Entertain :- toy with, amuse. (a) inciting (b) winning Enlighten :- give spiritual insight to. (c) losing (d) preparing Correct sentence- She has shown a great interest towards space science, since her early children and a SSC CHSL (Tier-I) 26/10/2020 (Shift-II) passion to explore the outer space. Ans. (a) : According to the sense of the above 603. Select the most appropriate word to fill in the sentence, 'inciting' would be appropriate in the blank space. blank. I had a broken bone in the hand which the The other options give different meaning. doctor called a _______ and suggested Winning :- victorious, successful. Losing :- fail to keep or to maintain. immediate surgery. Preparing :- ready. (a) contamination (b) infection Correct sentence- The politician was responsible for (c) wound (d) fracture SSC CHSL (Tier-I) 18/03/2020 (Shift-I) inciting violence in the university. Fill in the Blanks

270

YCT

607. Select the most appropriate word to fill in the blank. In our metro cities the contrast between the high rise buildings with the slums behind is_____. (a) comforting (b) interesting (c) laudable (d) appalling SSC CHSL (Tier-I) 26/10/2020 (Shift-II) Ans. (d) : According to the sense of the above sentence, 'appalling' (dismaying) would be appropriate. The meaning of other options is– Comforting :- satisfying. Interesting :- exciting, entertaining. Laudable :- commendable, praiseworthy. Correct sentence- In our metro cities the contrast between the high rise buildings with the slums behind is appalling. 608. Select the most appropriate word to fill in the blank. Shivangini, the 12 year old archer who ___an injury has been successfully operated by a team of doctors at A.I.I.M.S. (a) pertained (b) maintained (c) ordained (d) sustained SSC CHSL (Tier-I) 21/10/2020 (Shift-I) Ans. (d) : In the blank space of the above sentence, 'Sustained' (keep up) would be appropriate. Because according to the sense of this sentence, it gives the suitable meaning. Pertained :- be relevant to Maintain :- observe Ordained :- consecrate, dedicated. Correct sentence- Shivangini, the 12 year old archer who sustained an injury has been successfully operated by a team of doctors at A.I.I.M.S. 609. Select the most appropriate word (s) to fill in the blank: My family is my priority but there were times when I had to___ because of work pressures. (a) pull away (b) look into (c) put off (d) sit in SSC CHSL (Tier-I) 21/10/2020 (Shift-I) Ans. (a) : In the blank space of the above sentence, 'Pull away' (back away) will be used. According to the sense of this sentence, it gives the proper meaning. Look into :- check, investigate Put off :- postpone. Sit in :- attend as a visitor Correct sentence- My family is my priority but there were times when I had to pull away because of work pressures. 610. Select the most appropriate word to fill in the blanks In ______ of the great work he is doing for economically weaker children, he has been given a special award. (a) resolution (b) award (c) reward (d) recognition SSC CHSL (Tier-I) 19/10/2020 (Shift-I) Ans. (d) : According to the sense of the above sentence, 'recognition' (identification) will be used in the blank space. Fill in the Blanks

The meaning of other options is– Resolution :- a decision to do something. Award :- prize. Reward :- honour, laurels. Correct sentence- In recognition of the great work he is doing for economically weaker children, he has been given a special award. 611. Select the most appropriate word to fill in the blank. We found it _____ to climb further due to bad weather and decided to give up the expedition. (a) boring (b) pleasant (c) impossible (d) exciting SSC CHSL (Tier-I) 19/10/2020 (Shift-I) Ans. (c) : In the blank space of the above sentence, 'impossible' would be appropriate. Because according to the sense of this sentence, it gives the suitable meaning. The meaning of other options is– Boring :- tedious. Pleasant :- good-natured. Exciting :- provocative. Correct sentence- We found it impossible to climb further due to bad weather and decided to give up the expedition. 612. Select the most appropriate word to fill in the blank in the given sentence. The teacher ______ the student for cheating. (a) rebuked (b) refused (c) praised (d) invoked SSC CHSL (Tier-I) 12/10/2020 (Shift-II) Ans. (a) : According to the sense of the above sentence, ' rebuked' will be used in the blank space. The meaning of other options is– Refused :- decline, reject. Praised :- admiration, compliment. Invoke :- call, Invocation. Correct sentence- The teacher rebuked the student for cheating. 613. Select the most appropriate word to fill in the blank in the given sentence. Those earning less than Rs. 10000 are ______ from paying income tax. (a) collected (b) exempted (c) accepted (d) adjusted SSC CHSL (Tier-I) 12/10/2020 (Shift-II) Ans. (b) : In the blank space of the above sentence, 'exempted' (immune) will be used. The meaning of other options is– Collected :- accumulate, compile. Accepted :- admitted. Adjust :- correct, set. Correct sentence- Those earning less than Rs. 10,000 are exempted from paying income tax. 614. Select the most appropriate word to fill in the blank in the given sentence. The flyover was closed in October last year for repairs but its reopening has been –––– as it will be ready only in March this year. (a) delayed (b) built (c) broken (d) repaired SSC CHSL (Tier-I) 15/10/2020 (Shift-III)

271

YCT

Ans. (a) : In the blank space of the given sentence, 'Delayed' would be appropriate. The other options give different meaning. Built :- improved. Broken :- damaged, injured. Repaired :- mend, rectify. Correct Sentence – The flyover was closed in October last year for repairs but its reopening has been delayed as it will be ready only in March this year. 615. Select the most appropriate word to fill in the blank in the given sentence. Today's –––– are only interested in watching movies with the stories they can relate to the cinema. (a) audiences (b) accountants (c) politics (d) personalities SSC CHSL (Tier-I) 15/10/2020 (Shift-III) Ans. (a) : According to the sense of the above sentence, 'Audience' will be used in the blank space. The meaning of other options is– Accountants :- someone who maintains and audits business accounts. Politics :- refers to the government of a city state, or country. Personalities :- celebrity, famous person. Correct sentence- Today's audiences are only interested in watching movies with the stories they can relate to the cinema. 616. Select the most appropriate words to fill in the blank in the given sentence. I dislike people with –––– who say something and do something else. (a) double standards (b) many roles (c) multiple relations (d) strange achievements SSC CHSL (Tier-I) 15/10/2020 (Shift-II) Ans. (a) : In the blank space of the above sentence, 'double standards' would be appropriate. The other options give different meaning. Many roles :- many functions Multiple relations :- many relation Strange achievement :- unique benefit. Correct sentence- I dislike people with double standards who say something and do something else. 617. Select the most appropriate option to fill in the blank in the given sentence. Over 7,50,000 people from 192 countries have pledged to go vegan for a month which they say would reduce carbon ________ by 450000 tons. (a) exclusion (b) inclusion (c) omission (d) emission SSC CHSL (Tier-I) 15/10/2020 (Shift-I) Ans. (d) : According to the sense of the above sentence, 'emission' would be appropriate in the blank space. The other options give different meaning. Exclusion :- banishment, debarment. Inclusion :- the stare of being included. Omission :- neglect. Correct sentence- Over 7,50,000 people from 192 countries have pledged to go vegan for a month which they say would reduce carbon emission by 450000 tons. Fill in the Blanks

618. Select the most appropriate option to fill in the blank in the given sentence. The performances of the notable dancers and guest artists _____the audience. (a) absorbed (b) enthralled (c) immersed (d) occupied SSC CHSL (Tier-I) 13/10/2020 (Shift-III) Ans. (b) In the blank space of the above sentence, 'enthralled' will be used in the blank space. The other options– Absorbed :- engrossed. Immersed :- plunge Occupied :- busy, engaged. Correct Sentence- The performances of the notable dancers and guest artists enthralled the audience. 619. Select the most appropriate option to fill in the blank in the given sentence. A____ of small stones is called a mosaic. (a) structure (b) pattern (c) design (d) mound SSC CHSL (Tier-I) 13/10/2020 (Shift-III) Ans. (b) In the blank space of the above sentence, 'pattern' would be appropriate. The other options give different meaning. Structure :- construction. Design :- plan, pattern. Mound :- hill, neap. Correct sentence- 'A pattern of small stones is called a mosaic.' 620. Select the most appropriate option to fill in the blank in the given sentence. Students who are capable of understanding and managing their 'emotional intelligence' do better at school in ____ to their peers who lack the ability. (a) difference (b) similarity (c) resemblance (d) comparison SSC CHSL (Tier-I) 14/10/2020 (Shift-III) Ans. (d) : In the blank space of the above sentence, 'comparison' would be appropriate. The meaning of other options is– Difference :- conflict. Similarity :- alikeness. Resemblance :- parallel. Correct sentence- Students who are capable of understanding and managing their 'emotional intelligence' do better at school in comparison to their peers who lack the ability. 621. Select the most appropriate word to fill in the blank: As part of the ________ Classical Indian Music Festival leading vocalists and instrumentalist are taking the stage this evening. (a) postponed (b) foregone (c) on set (d) on going SSC CHSL (Tier-I) 21/10/2020 (Shift-III)

272

YCT

Ans. (d) : According to the sense of the above Ans. (d) : In the blank space of the above sentence, sentence, 'on going' would be appropriate in the blank 'restored' will be used because it gives appropriate space. meaning. The other options give different meaning. The other options give different meaning. Postpone :- stop, defer. Rebuilt :- reconstruct. Foregone :- forsake, departed. Repaired :- amend, rectify. On set :- begin. Recovered :- convalesce, get back. Correct sentence- As part of the on going Classical Correct sentence- Order was quickly restored in the Indian Music Festival leading vocalists and area after the scuffle between the two groups. instrumentalist are taking the stage this evening. 626. Select the most appropriate word to fill in the 622. Select the most appropriate word to fill in the blank in the given sentence. blank. When he came back home safe that night, The Yoga instructor was _______at the everybody ____ a sigh of relief. student's flexibility. (a) Inhaled (b) exhaled (a) ridiculed (b) amazed (c) heaved (d) raised (c) disturbed (d) angry SSC CHSL (Tier-I) 19/10/2020 (Shift-III) SSC CHSL (Tier-I) 20/10/2020 (Shift-III) Ans. (c) : In the blank space of the above sentence, Ans. (b) : In the blank space of the above sentence, 'heaved' will be used because 'heaved a sigh of relief' is 'Amazed' will be used. which give complete sense. a phrase which means– to feel very suddenly because Ridicule :- jest, laugh at. something has not happened or has ended. Disturbed :- disarranged, upset. The meaning of other options is – Angry :- wrathful, furious. Inhaled :- breathe in. Correct sentence- The Yoga instructor was amazed at Exhaled :- breathe out. the student's flexibility. 623. Select the most appropriate word to fill in the Raised :- appraised, lifted. Correct sentence- When he came back home safe that blank. The government enterprises were privatized as night, everybody heaved a sign of relief. they were ______ heavy losses on a daily basis. 627. Select the most appropriate word to fill in the (a) containing (b) earning blank in the given sentence. (c) incurring (d) demanding Our professor speaks English with a _____ SSC CHSL (Tier-I) 20/10/2020 (Shift-III) Oriya accent. (a) widespread (b) vague Ans. (c) : In the blank space of the above sentence, ' incurring will be used. because it gives appropriate (c) pronounced (d) remarkable meaning according to sense of the sentence. SSC CHSL (Tier-I) 19/10/2020 (Shift-III) The meaning of other options is– Ans. (c) : In the blank space of the above sentence, Contain :- bear, hold. 'pronounced' would be appropriate, according to the Earning :- pay. sense of sentence. Demanding :- needy, stern. The meaning of other options is– Correct sentence- The government enterprises were Widespread :- diffused, prevalent. privatized as they were incurring heavy losses on a Vague :- obscure. daily basis. Remarkable :- noteworthy, important. 624. Select the most appropriate word to fill in the Correct sentence- Our professor speaks English with blank in the given sentence. a pronounced Oriya accent. He –––– his studies even after leaving college 628. Select the most appropriate option to fill in the and finally got a Ph.D. degree. blank in the given sentence. (a) sought (b) persisted A literature festival on the beach: With waves (c) pursued (d) carried crashing on the beach, the Kerala Literature SSC CHSL (Tier-I) 20/10/2020 (Shift-I) Festival got _____ in Kozhikode with a range of Ans. (c) : In the blank space of the above sentence, panel discussions in English and Malayalam. 'pursued' would be appropriate. (a) apart (b) wayward The meaning of other options is– (c) underway (d) away Sought :- being searched for. SSC CHSL (Tier-I) 20/10/2020 (Shift-II) Persisted :- continue to exist. Ans. (c) : According to the sense of the above Carrier :- transport, conveyer. sentence, 'underway' will be used in the blank space. Correct sentence- He pursued his studies even after The other options– leaving college and finally got a Ph.D. degree. 625. Select the most appropriate word to fill in the Apart :- separated, aloof. Wayward :- erratic, temperamental. blank in the given sentence. Order was quickly ––––– in the area after the Away :- outside. Correct sentence- A literature festival on the beach: scuffle between the two groups. (a) recovered (b) rebuilt With waves crashing on the beach, the Kerala Literature Festival got underway in Kozhikode with a (c) repaired (d) restored SSC CHSL (Tier-I) 20/10/2020 (Shift-I) range of panel discussions in English and Malayalam. Fill in the Blanks

273

YCT

629. Select the most appropriate option to fill in the blank in the given sentence. We must not take the future for granted; rather, we should enjoy the _____ joys of our humdrum lives. (a) future (b) average (c) everyday (d) habitual SSC CHSL (Tier-I) 20/10/2020 (Shift-II) Ans. (c) : According to the sense of the above sentence, 'everyday' would be appropriate, in the blank space. 'Everyday' is an adjective which is used to describe something. The meaning of other options is– Future :- the time yet to come. Average :- medium Habitual :- normal, natural Correct sentence- We must not take the future for granted; rather, we should enjoy the everyday joys of our humdrum lives. 630. Select the most appropriate word to fill in the blank. The important ____ is where to arrange the funds from. (a) solution (b) issue (c) view (d) result SSC CHSL (Tier-I) 19/10/2020 (Shift-II) Ans. (b) : In the blank space of the above sentence, 'Issue' will be used. because it gives appropriate meaning. The meaning of other options is– Solution :- a method for solving a problem View :- aspect, prospect. Result :- consequence. Correct sentence– The important issue is where to arrange the funds from. 631. Select the most appropriate word to fill in the blank. Several villages in Kashmir have been _____ due to heavy snowfall. (a) confined (b) isolated (c) engaged (d) divided SSC CHSL (Tier-I) 19/10/2020 (Shift-II) Ans. (b) : In the blank space of the above sentence, 'Isolated' will be used. because it gives appropriate meaning. The meaning of other options is– Confined :- close, imprisoned. Engaged :- occupied. Divided :- separate. Correct sentence- Several villages in Kashmir have been isolated due to heavy snowfall. 632. Select the most appropriate word to fill in the blank. Children need special care and ____ from the side of their parents for making them emotionally strong. (a) compromise (b) suspicion (c) attention (d) reflection SSC CHSL (Tier-I) 19/03/2020 (Shift-II) Fill in the Blanks

Ans. (c) : In the blank space of the above sentence, 'Attention' will be used. because it gives appropriate meaning. The meaning of other options is– Compromise :- settle by concession. Suspicion :- distrust. Reflection :- transformation. Correct sentence- Children need special care and attention from the side of their parents for making them emotionally strong. 633. Select the most appropriate word to fill in the blank. Unnecessary suspicion and lack of trust against the others are caused by ____. (a) impartiality (b) contentment (c) prejudice (d) empathy SSC CHSL (Tier-I) 19/03/2020 (Shift-II) Ans. (c) : In the blank space of the above sentence, 'Prejudice' will be used. because it gives appropriate meaning. The meaning of other options is– Contentment :- happiness with one's situation in life. Impartiality :- an inclination to weigh both views. Empathy :- understanding and entering into another's feeling. Correct sentence- Unnecessary suspicion and lack of trust against the others are caused by prejudice. 634. Select the most appropriate word to fill in the blank. I found the scene of some people making fun of a beggar quite ____. (a) repulsive (b) destructive (c) sociable (d) admirable SSC CHSL (Tier-I) 14/10/2020 (Shift-II) Ans. (a) : In the blank space of the above sentence, 'Repulsive' gives the suitable meaning. The meaning of other options is– Destructive :- ruinous, blasting. Sociable :- friendly. Admirable :- pleasing. Correct sentence- I found the scene of some people making fun of a beggar quite repulsive. 635. Select the most appropriate word to fill in the blank. The scientist, who is known for his ____ ideas on space travel, is much admired. (a) anarchist (b) revolutionary (c) ridiculous (d) malicious SSC CHSL (Tier-I) 14/10/2020 (Shift-II) Ans. (b) : In the blank space of the above sentence, 'Revolutionary' would be appropriate. Because according to the sense of this sentence, it gives suitable meaning. Anarchist :- A person who rebels against any authority. Ridiculous :- absurd, ludicrous. Malicious :- despiteful, spiteful. Correct sentence- The scientist, who is known for his revolutionary ideas on space travel, is much admired. 636. Select the most appropriate option to fill in the blank. According to experts, some sectors remain _____ during these difficult times, including banking and financial services, FMCG, food and technology.

274

YCT

(a) repugnant (b) resilient (c) redundant (d) repentant SSC CHSL (Tier-I) 14/10/2020 (Shift-I) Ans. (b) : According to the sense of the above sentence, 'Resilient' will be used in the blank space. The meaning of other options is– Repugnant :- detestable. Redundant :- excess, surplus. Repentant :- penitent, regretful. Correct sentence- According to experts, some sectors remain resilient during these difficult times, including banking and financial services, FMCG, food and technology. 637. Select the most appropriate word to fill in the blank. Around one hundred _____ across the world participated in the conference. (a) delegates (b) talents (c) clever (d) intelligent SSC CHSL (Tier-I) 14/10/2020 (Shift-I) Ans. (a) : According to the sense of the above sentence, 'delegates' will be used in the blank space because it gives the suitable meaning. The meaning of other options isTalents :- natural abilities or qualities Clever :- cunning. Intelligent :- agile, nimble. Correct sentence- Around one hundred delegates across the world participated in the conference. 638. Select the most appropriate option to fill in the blank in the given sentence. Timers at traffic signals, that help motorists save fuel by switching off their engines while waiting, will soon be phased out due to technical ________. (a) glitches (b) illuminations (c) streamers (d) symbols SSC CHSL (Tier-I) 13/10/2020 (Shift-II) Ans. (a) : According to the sense of the above sentence, 'glitches' would be appropriate. in the blank space. The meaning of other options is– Illuminations :- light. Streamer :- a long flag, ray of light, mark, a sign. Symbol :- mark, a sign. Correct sentence- Timers at traffic signals, that help motorists save fuel by switching off their engines while waiting, will soon be phased out due to technical glitches. 639. Select the most appropriate option to fill in the blank in the given sentence. A ________ of small stones is called a mosaic. (a) Mound (b) Structure (c) Design (d) Pattern SSC CHSL (Tier-I) 13/10/2020 (Shift-II) Ans. (d) : In the blank space of the above sentence, 'Pattern' would be appropriate, according to the sense of the above sentence. The meaning of other options is– Mount :- climb, rise. Structure :- constructure. Design :- plan. Correct sentence- A pattern of small stones is called a mosaic. Fill in the Blanks

640. Select the most appropriate word to fill in the blank. For thousands of years, humans have been ________ the earth, making scars upon the land. (a) redecorating (b) remaking (c) damaging (d) distracting SSC CHSL (Tier-I) 12/10/2020 (Shift-III) Ans. (c) : In the blank space of the above sentence, 'damaging' will be used. The meaning of other options is– Redecorating :- redo the decoration of an apartment or house. Remaking :- creation that is created again or a new. Distracting :- disorder, disquiet. Correct sentence- For thousands of years, humans have been damaging the earth, making scars upon the land. 641. Select the most appropriate word to fill in the blank in the given sentence. Padamse worked like a mathematician and was a true _______ who experimented with numerous mediums. (a) literary (b) visionary (c) adversary (d) voluntary SSC CHSL (Tier-I) 12/10/2020 (Shift-I) Ans. (b) : According to the sense of the above sentence, 'Visionary' will be used in the blank space. The meaning of other options is– Literary :- knowledge about literature. Adversary :- opponent, apposer. Voluntary :- unpaid, volunteer. Correct sentence- Padamse worked like a mathematician and was a true visionary who experimented with numerous mediums. 642. Select the most appropriate word to fill in the blank in the given sentence. If the video clip of the murder had not been shared on social media, the world would likely have remained _______ of the tragedy. (a) unconfirmed (b) renowned (c) unaware (d) recognised SSC CHSL (Tier-I) 12/10/2020 (Shift-I) Ans. (c) According to the sense of the above sentence, 'unaware' will be used in the blank space. The other options give different meaning– Unconfirmed :- temporary, transitory. Renowned :- famous, notable Recognised :- established, acknowledge. Correct sentence- If the video clip of the murder had not been shared on social media, the world would likely have remained unaware of the tragedy. 643. Select the most appropriate option to fill in the blank. By accelerating digitisation and leveraging next generation technologies, the life insurance sector can double its ______ to Rupees 70 lakh crores, a study said. (a) assets (b) reports (c) policies (d) finding SSC CHSL (Tier-I) 17/03/2020 (Shift-III)

275

YCT

Ans. (a) : According to the sense of the above sentence, 'assets' would be appropriate in the blank space. The meaning of the other options is–– Reports :- complain about; make a charge Policies :- insurance. Finding :- the decision of a court on issues of fact of law. Correct sentence- By accelerating digitisation and leveraging next generation technologies, the life insurance sector can double its assets to Rupees 70 lakh crores, a study said. 644. In the following question, the sentence given with blank to be filled in with an appropriate word. Select the correct alternative out of the four and indicate it by selecting the appropriate option : I had ........... my right shoulder during the play. (a) browsed (b) broke (c) grounded (d) injured SSC MTS 7-10-2017 (Shift-I) Ans. (d) : In the blank space of the above sentence, 'injured' would be appropriate, Generally 'had + V3rd form' is used in past perfect tense. The meaning of other options is – Browsed :- pasture, graze. Broke :- come apart, separate. Ground :- earth, land. Correct sentence- I had injured my right shoulder during the play. 645. In the following question, the sentence given with blank to be filled in with an appropriate word. Select the correct alternative out of the four and indicate it by selecting the appropriate option : Police had .......... the driver. (a) obtained (b) detained (c) sustained (d) objected SSC MTS 7-10-2017 (Shift-I) Ans. (b) : In the blank space of the above sentence, ' detained' (confine) will be used. The other options give different meaning. Obtain :- find, get. Sustain :- keep up, maintain. Object :- to be against somebody. Correct sentence- Police had detained the driver. 646. In the following question, the sentence given with blank to be filled in with an appropriate word. Select the correct alternative out of the four and indicate it by selecting the appropriate option On the ________ occasion of Ganesh Chaturthi the Guptas bought a new house. (a) prosperous (b) auspicious (c) official (d) fortuitous SSC MTS 9-10-2017 (Shift-II) Ans. (b) : In the blank space of the above sentence, 'Auspicious' (hopeful) will be used. The meaning of other options is– Prosperous :- comfortable, well situated. Official :- authorised, confirmed. Fortuitous :- causeless, unintended. Correct sentence- On the auspicious occasion of Ganesh Chaturthi the Guptas bought a new house. Fill in the Blanks

647. In the following question, the sentence given with blank to be filled in with an appropriate word. Select the correct alternative out of the four and indicate it by selecting the appropriate option. My parents have given their ______ to the match. (a) refuse (b) accept (c) assent (d) expect SSC MTS 10-10-2017 (Shift-III) Ans. (c) : In the blank space of the above sentence, 'Assent' (allow) will be used. because it gives appropriate meaning. The other options give different meaning. Accept :- receive, admit Refuse :- decline, turn down. Expect :- require Correct sentence- My parents have given their assent to the match. 648. In the following question, the sentence given with blank to be filled in with an appropriate word. .........................indicate it by selecting the appropriate option. He killed three __________ for dinner. (a) forl (b) fowls (c) fool (d) flour SSC MTS 11-10-2017 (Shift-III) Ans. (b) : In the blank space of the above sentence, 'fowls' (domestic fowl, poultry) would be appropriate. The meaning of other options is– Fool :- jester. mug. Flour :- food product. Correct sentence- He killed three fowls for dinner. 649. In the following question, the sentence given with blank to be filled in with an appropriate word. Select the correct alternative out of the and indicate appropriate option. He is younger ––––––– you are, isn't he? (a) than (b) incase (c) if (d) on condition SSC MTS 11-10-2017 (Shift-I) Ans : (a) In the blank space of the above sentence, 'than' will be used because Adjective 'than' is used in comparative degree. Correct sentence- He is younger than you are, isn't he? 650. Select the most appropriate option to fill in the blank. Your answer book will be –––– by a computer. (a) replied (b) submitted (c) evaluated (d) tested SSC MTS – 06/08/2019 (Shift-III) Ans. (c) : In the blank of the above sentence, 'evaluated' would be appropriate. The meaning of other options is– Replied :- answer. respond. Submitted :- present. Tested :- well tried, proved. Correct sentence- Your answer book will be evaluated by a computer.

276

YCT

651. Select the most appropriate option to fill in the (a) occupancy (b) carrier blank. (c) postage (d) career The teacher was very ––––– in his subject. SSC MTS – 22/08/2019 (Shift-II) (a) best (b) master Ans. (d) : In the blank space of the above sentence, (c) top (d) proficient 'career' will be used. because it gives appropriate SSC MTS – 06/08/2019 (Shift-III) meaning. Ans. (d) : In the blank space of the above sentence, The meaning of other options is– proficient (skill) would be appropriate. Occupancy :- tenancy. The other options give different meaning. Postage :- stamp. Best :- champion, high grade. Carrier :- letter carrier, mail carrier, postman. Top :- upper side, peak Correct sentence- Raman exercises daily because he Master :- lord. is preparing for a career in the armed forces. Correct sentence- The teacher was very proficient in 656. Select the most appropriate word to fill in the his subject. blank and make a meaningful sentence. 652. Select the most appropriate word to fill in the The customers ––––– the company with blank and make a meaningful sentence. complaints against the price increase. The method ––––– by the teacher is very (a) failed (b) criticized tedious. (c) flooded (d) hoarded (a) regarded (b) reminded SSC MTS – 09/08/2019 (Shift-I) (c) allowed (d) suggested SSC MTS – 22/08/2019 (Shift-III) Ans. (c) : In the blank space of the above sentence, 'flooded' would be appropriate. Ans. (d) : According to the sense of the above The meaning of other options is– sentence, 'suggested' would be appropriate in the blank Failed :- unsuccessful. space. Criticized :- find fault with The meaning of other options is– Hoarded :- accumulate, amass. Regarded :- esteem, respect. Correct sentence- The customers flooded the Remind :- cue, prompt company with complaints against the price increase. Allow :- permit. Correct sentence- The method suggested by the 657. Select the most appropriate word to fill in the teacher is very tedious. blank and make a meaningful sentence. Conservative parents usually do not give their 653. Select the most appropriate option to fill in the –––––– for uncommon career choices of their blank and make a grammatically correct sentence. children. Jamuna found an old necklace and ––––– gold (a) consent (b) agreement bangles in the cupboard. (c) allowance (d) dissent (a) an pair of (b) pair of SSC MTS – 08/08/2019 (Shift-III) (c) a pair of (d) a pairs of Ans. (a) : According to the sense of the above SSC MTS – 22/08/2019 (Shift-II) sentence, 'consent' would be appropriate in the blank Ans. (c) : In the blank space of the above sentence, 'a space. pair of' will be used. The meaning of other options is– The other options give different meaning. Agreement :- accord Correct sentence- Jamuna found an old necklace and Allowance :- reimbursement, mileage. a pair of bangles in the cupboard. Dissent :- objection, protest. 654. Select the most appropriate word to fill in the Correct sentence- Conservative parents usually do not blank and make a meaningful sentence. give their consent for uncommon career choices of The student –––––– for a refund of fees. their children. (a) said (b) wanted 658. Select the most appropriate word to fill in the (c) requested (d) ordered blank and make a meaningful sentence. SSC MTS – 22/08/2019 (Shift-II) He ––––– me up on his way to work. Ans. (c) : In the blank space of the above sentence, (a) picked (b) raised 'requested' would be appropriate. (c) lifted (d) dropped The meaning of other options is– SSC MTS – 08/08/2019 (Shift-III) Said :- being the one previously mentioned or spoken Ans. (a) : According to the sense of the above of. sentence, 'picked' would be appropriate in the blank Wanted :- desired, craved. space. Ordered :- set up, an organized state. The meaning of other options is– Correct sentence- The student requested for a refund Raised :- upraised of fees. 655. Select the most appropriate word to fill in the Lifted :- held up in the air Dropped :- leave out, miss. blank and make a meaningful sentence. Raman exercises daily because he is preparing Correct sentence- He picked me up on his way to work. for a –––––– in the armed forces. Fill in the Blanks

277

YCT

659. Select the most appropriate option to fill in the blank. ––––– rivers have water throughout the year. (a) Ephemeral (b) Perennial (c) Seasonal (d) Chronic SSC MTS – 08/08/2019 (Shift-I) Ans. (b) : According to the sense of the above sentence, 'perennial' (continual) would be appropriate in the blank space. The meaning of other options is– Ephemeral :- anything short-lived. Seasonal :- dependent on a particular season. Chronic :- continuing, long. Correct sentence- Perennial rivers have water throughout the year. 660. Select the most appropriate option to fill in the blank. The accident occurred in the ––––– of the road. (a) way (b) middle (c) side (d) path SSC MTS – 08/08/2019 (Shift-I) Ans. (b) : According to the sense of the above sentence, 'middle' would be appropriate in the blank space. The meaning of other options is– Way :- mode, style. Side :- slope, incline. Path :- route. Correct sentence- The accident occurred in the middle of the road. 661. Select the most appropriate option to fill in the blank. I ––––him to use my bike to go to the office. (a) granted (b) allowed (c) admonished (d) sanctioned SSC MTS – 07/08/2019 (Shift-I) Ans. (b) : According to the sense of the above sentence, 'allowed' would be appropriate. The meaning of other options is– Grant :- allow. Admonish :- warn, discourage. Sanction :- official permission. Correct sentence- I allowed him to use my bike to go to the office. 662. Select the most appropriate option to fill in the blank. Marconi ––––– the radio. (a) discovered (b) invented (c) built (d) assembled SSC MTS – 07/08/2019 (Shift-I) Ans. (b) : In the blank space of the above sentence, 'invented' will be used because when we discover new things, then 'invent' is used and 'discover' is used for old things. The meaning of other options is– Built :- construct, make. Discovered :- detect, observe. Assembled :- put together. Correct sentence- Marconi invented the radio. Fill in the Blanks

663. Select the most appropriate option to fill in the blank. There is no –––– in the container for the water to flow. (a) link (b) outlet (c) room (d) escape SSC MTS – 06/08/2019 (Shift-I) Ans. (b) : According to the sense of the above sentence, 'outlet' would be appropriate in the blank space. Link :- connect, tie. Room :- space of movement. Escape :- get away. Correct sentence- There is no outlet in the container for the water to flow. 664. Select the most appropriate option to fill in the blank. The incident made a –––––– impression on Karan. (a) factual (b) rounded (c) perfect (d) deep SSC MTS – 06/08/2019 (Shift-I) Ans. (d) : In the blank space of the above sentence, 'deep' would be appropriate. The meaning of other options is– Factual :- actual, real. Rounded :- circular. Perfect :- exact, clear. Correct sentence- The incident made a deep impression on Karan. 665. Select the most appropriate option to fill in the blank. Young people from disadvantaged homes look for ––––– in government jobs. (a) utility (b) capability (c) ability (d) stability SSC MTS – 05/08/2019 (Shift-III) Ans. (d) : In the blank space of the above sentence, 'stability' would be appropriate. The meaning of other options is– Utility :- usefulness. Capability :- potentiality. Ability :- power. Correct sentence- Young people from disadvantaged homes look for stability in government jobs. 666. Select the most appropriate option to fill in the blank. The wonderful performance of the maestro left the audience –––––– . (a) puzzled (b) sleepy (c) hungry (d) entranced SSC MTS – 05/08/2019 (Shift-II) Ans. (d) : In the blank space of the above sentence, 'entranced' would be appropriate. The other options give different meaning. Puzzle :- perplex, baffle. Sleepy :- Drowsy, Lethargic, Hungry :- thirsty, wishful. Correct sentence- The wonderful performance of the maestro left the audience entranced.

278

YCT

667. Select the most appropriate option to fill in the blank. Sarthak –––– everything that a leader should be. (a) epitomizes (b) adores (c) worships (d) clones SSC MTS – 05/08/2019 (Shift-I) Ans. (a) : In the blank space of the above sentence, 'epitomizes' would be appropriate. The other options give different meaning. Adores :- love intensely. Worship :- show devotion to. Clones :- look like. Correct sentence- Sarthak epitomizes everything that a leader should be. 668. Select the most appropriate option to fill in the blank. The students are in great –––– of their teachers. (a) praise (b) awe (c) respect (d) gusto SSC MTS – 05/08/2019 (Shift-I) Ans. (b) : In the blank space of the above sentence, 'Awe' (fear) will be used. because it gives appropriate meaning. The meaning of other options is– Praise :- appraise. Respect :- honour. Gusto :- zest, relish. Correct sentence- The students are in great awe of their teachers. 669. Select the most appropriate option to fill in the blank. It is quite tough to ––––– which candidate will win the presidential elections. (a) pretend (b) elect (c) predict (d) argue SSC MTS – 02/08/2019 (Shift-III) Ans. (c) : According to the sense of the above sentence, 'predict' would be appropriate in the blank space. The meaning of other options is– Pretend :- dissemble. Elect :- select, chosen. Argue :- contend, reason. Correct sentence- It is quite tough to predict which candidate will win the presidential elections. 670. A sentence has been given with a blank to be filled with an appropriate word. Choose the correct alternative. I am ––––– of winning the competition. (a) resistent (b) confident (c) pertinent (d) reliant SSC MTS – 02/08/2019 (Shift-II) Ans. (b) : In the blank space of the above sentence, 'confident' would be appropriate. The meaning of the other options is– Resistent :- not wanting something and trying to prevent something happening. Pertinent :- relevant. Reliant :- dependent. Correct sentence- I am confident of winning the competition. Fill in the Blanks

671. A sentence has been given with a blank to be filled with an appropriate word. Choose the correct alternative. The bullet struck a wall and was ––––– from its course. (a) reflected (b) twisted (c) deflected (d) crashed SSC MTS – 02/08/2019 (Shift-II) Ans. (c) : In the blank space of the above sentence, 'deflected' would be appropriate. The other options give different meaning. Reflected :- mirrored. Twisted :- misrepresented. Deflect :- distract. Correct sentence- The bullet struck a wall and was deflected from its course. 672. Select the most appropriate option to fill in the blank. Her parents always –––––– her in her studies. (a) encouraged (b) interested (c) impressed (d) expected SSC MTS – 02/08/2019 (Shift-I) Ans. (a) : In the blank space of the above sentence, 'encouraged' will be used. The meaning of other options is – Interest :- excite the curiosity of. Impressed :- affect. Expected :- hoped. Correct sentence- Her parents always encouraged her in her studies. 673. Select the most appropriate option to fill in the blank and make a grammatically correct sentence. Meena had a whale –––– a time in Mussorie with her gang. (a) in (b) of (c) on (d) at SSC MTS – 22/08/2019 (Shift-I) Ans. (b) : In the blank space of the above sentence, 'of' will be used because 'whale of a time' (pleasant experience) would be appropriate. It is used as an idiomatic phrase. The other options do not give the suitable meaning. Correct sentence- Meena had a whale of a time in Mussorie with her gang. 674. Select the most appropriate option to fill in the blank and make a grammatically correct sentence. In the age of technology sending letters by postal mail seems ––––– to me. (a) superstition (b) surplus (c) supercilious (d) superfluous SSC MTS – 22/08/2019 (Shift-I) Ans. (d) : According to the sense of the above sentence, 'superfluous' would be appropriate in the blank space. The meaning of other options is– Superstition :- an irrational belief arising from ignorance or fear. Surplus :- excess, extra. Supercilious :- naughty, prideful. Correct sentence- In the age of technology, sending letters by postal mail seems superfluous to me

279

YCT

675. Select the most appropriate option to fill in the blank and make a grammatically correct sentence. Tourism has been ––––– in India for a long time. (a) postponed (b) declined (c) decayed (d) neglected SSC MTS – 22/08/2019 (Shift-I) Ans. (d) : According to the sense of the above sentence, 'neglected' would be appropriate. The meaning of other options is– Postpone :- put off Decline :- grow worse. Decay :- a gradual decrease. Correct sentence- Tourism has been neglected in India for a long time. 676. Select the most appropriate option to fill in the blank and make a grammatically correct sentence. Surendra tries to –––––– his opponents in all the matches. (a) intimidate (b) impersonate (c) intimate (d) initiate SSC MTS – 21/08/2019 (Shift-III) Ans. (a) : According to the sense of the above sentence, 'intimidate' (fearful) would be appropriate. The meaning of other options is– Impersonate :- pretend to be someone you are not. Intimate :- suggest. Initiate :- start, originate. Correct sentence- Surendra tries to intimidate his opponents in all the matches. 677. Select the most appropriate option to fill in the blank and make a grammatically correct sentence. Megha is attending a teachers' ––––– in London. (a) connection (b) combination (c) connotation (d) convention SSC MTS – 21/08/2019 (Shift-III) Ans. (d) : In the blank space of the above sentence, 'convention' will be used. because it gives appropriate meaning. The other options give different meaning. Connection :- link, joining. Combination :- combining. Connotation :- meaning, substance. Correct sentence- Megha is attending a teachers' convention in London. 678. Select the most appropriate word to fill in the blank and make a meaningful sentence. The ISRO team is working round the clock to – ––––– Chandrayaan-2. (a) navigate (b) introduce (c) launch (d) steer SSC MTS – 21/08/2019 (Shift-II) Ans. (c) : In the blank space of the above sentence, 'Launch' (set in motion) would be appropriate. The other options give different meaning. Navigate :- guide, maneuver, voyage. Introduce :- Innovate, inclose. Steer :- hint, lead. Correct sentence- The ISRO team is working round the clock to launch Chandrayaan-2. Fill in the Blanks

679. Select the most appropriate word to fill in the blank and make a meaningful sentence. The diamond necklace is too –––– for me. (a) luscious (b) insufficient (c) extravagant (d) austere SSC MTS – 21/08/2019 (Shift-II) Ans. (c) : In the blank space of the above sentence, 'extravagant' would be appropriate. The meaning of other options is– Insufficient :- deficient, meager. Luscious :- juicy, pleasant. Austere :- strict, severe. Correct sentence- The diamond necklace is too extravagant for me. 680. Select the most appropriate option to fill in the blank and make a grammatically correct sentence. The cost turned out to be higher than_____ (a) excepted (b) anticipated (c) accepted (d) emancipated SSC MTS – 21/08/2019 (Shift-I) Ans : (b) According to the sense of the above sentence, 'anticipated' (foresee) would be appropriate in the blank space. The other options give different meaning. Excepted :- leave off, exclude. Accepted :- to agree Emancipated :- liberate. Correct sentence- The cost turned out to be higher than anticipated. 681. Select the most appropriate option to fill in the blank and make a grammatically correct sentence. Working together gave us a chance to ______ about old times. (a) reminisce (b) reproduce (c) recreate (d) reminding SSC MTS – 21/08/2019 (Shift-I) Ans : (a) In the blank space of the above sentence, 'reminisce' (recall the past) would be appropriate. The other options give different meaning. Reproduce :- procreate, recreate a sound, idea, mood. Recreate :- renovate, revive. Reminding :- remember. Correct sentence- Working together gave us a chance to reminisce about old times. 682. Select the most appropriate option to fill in the blank and make a grammatically correct sentence. The blind man needed ––––– to cross the street. (a) guards (b) assistance (c) cooperation (d) porters SSC MTS – 20/08/2019 (Shift-III) Ans. (b) : According to the sense of the above sentence, 'assistance (helper) would be appropriate. The meaning of other options is– Guard :- defend, ward. Cooperation :- joint operation or action. Porters :- doorkeeper, ostiary. Correct sentence- 'The blind man needed assistance to cross the street.'

280

YCT

683. A word can have a different –––––– in different contexts. (a) connotation (b) constellation (c) contention (d) confrontation SSC MTS – 20/08/2019 (Shift-III) Ans. (a) : According to the sense of the above sentence, 'connotation' (substance) would be appropriate in the blank space. Constellation :- configuration. Contention :- rivalry, arguing. Confrontation :- opposition. Correct sentence- A word can have a different connotation in different contexts. 684. Select the most appropriate option to fill in the blank and make a grammatically correct sentence. Seema gave her teacher a beautiful ––––– of pink roses on Teachers' Day. (a) grove (b) reed (c) bouquet (d) wreath SSC MTS – 20/08/2019 (Shift-II) Ans. (c) : In the blank space of the above sentence, 'bouquet' would be appropriate, because in the sentence expresses the point of giving 'pink rose' which kept together make a bouquet. The meaning of the other options is– Grove :- orchard, plantation. Reed :- chionochloa conspicua, gramineous plant. Wreath :- garland. Correct sentence- Seema gave her teacher a beautiful bouquet of pink roses on Teacher's Day. 685. Select the most appropriate option to fill in the blank and make it grammatically correct sentence. For many, The Lion King is ––––– as a childhood memory. (a) invested (b) deposited (c) treasured (d) inherited SSC MTS – 20/08/2019 (Shift-II) Ans. (c) : In the blank space of the above sentence, 'treasured' (gem) would be appropriate because memories are protected. The lion king is a story that is a part of a childhood memory. Which is expressed in a sentence to be preserved. The meaning of other options is– Invested :- expend. Deposit :- down payment. Inherited :- genetic, inheritable. Correct sentence- For many, The Lion King is treasured as a childhood memory. 686. Select the most appropriate word to fill in the blank and make a meaningful sentence. Mahesh is very happy because the value of his shares has –––. (a) diminished (b) complimented (c) applauded (d) appreciated SSC MTS – 20/08/2019 (Shift-I) Ans. (d) : According to the sense of the above sentence, 'appreciated' would be appropriate in the blank space. Fill in the Blanks

The meaning of other options is– Diminished :- decrease, lessen. Compliment :- congratulate. Applauded :- praise. Correct sentence- Mahesh is very happy because the value of his shares has appreciated. 687. Select the most appropriate word to fill in the blank and make a meaningful sentence. The CEO went against the advisory committe's –––––. (a) Introduction (b) Information (c) Recommendation (d) Response SSC MTS – 20/08/2019 (Shift-I) Ans. (c) : According to the sense of the above sentence, 'recommendation' would be appropriate. The meaning of other options is– Introduction :- launching, first appearance. Information :- data, info. Response :- answer, reply. Correct sentence- The CEO went against the advisory committe's recommendation. 688. Select the mot appropriate word to fill in the blank and make a meaningful sentence. The doctor advised him to consume ––––– water without any additive. (a) pleasant (b) plain (c) potent (d) precious SSC MTS – 19/08/2019 (Shift-III) Ans. (b) : According to the sense of the above sentence, 'plain' will be used in the blank space. The meaning of other options is– Pleasant :- good-natured. Potent :- strong, powerful. Precious :- valuable. Correct sentence- The doctor advised him to consume plain water without any additive. 689. Select the most appropriate word to fill in the blank and make a meaningful sentence. He was physically strong and became a ––––– climber of high rocky cliffs. (a) timid (b) fearful (c) unwilling (d) daring SSC MTS – 19/08/2019 (Shift-II) Ans. (d) : According to the sense of the above sentence, 'daring' would be appropriate in the blank space. The meaning of other options is– Timid :- cowardly. Fearful :- awful, dreadful. Unwilling :- unintentional, loath. Correct sentence- He was physically strong and became a daring climber of high rocky cliffs. 690. Select the most appropriate word to fill in the blank and make a meaningful sentence. He had his own line of thinking and would defend his views with ––––. (a) conversion (b) conviction (c) conversation (d) convention SSC MTS – 19/08/2019 (Shift-II) Ans. (b) : According to the sense of the above sentence, 'conviction' (strong belief) will be used in the blank space.

281

YCT

The other options give different meaning. Conversion :- changeover, transition. Conversation :- a talk between two or more people. Convention :- rule a large formal assembly. Correct sentence- He had his own line of thinking and would defend his views with conviction. 691. Select the most appropriate word to fill in the blank and make a meaningful sentence. It's a venue of every occasion and one that offers ––––– meals on special days. (a) offensive (b) amusing (c) thrilling (d) delightful SSC MTS – 19/08/2019 (Shift-I) Ans. (d) : According to the sense of the above sentence, 'delightful' would appropriate in the blank space. The meaning of the other options is– Offensive :- hateful, disgustful. Amusing :- comical, funny. Thrilling :- exciting Correct sentence- It's a venue of every occasion and one that offers delightful meals on special days. 692. Select the most appropriate word to fill in the blank and make a meaningful sentence. Winning both the races has been a ––––– surprise, but the success is well deserved. (a) satisfying (b) pleasant (c) civilized (d) Charming SSC MTS – 19/08/2019 (Shift-I) Ans. (b) : According to the sense of the above sentence, 'pleasant' would be appropriate in the blank space. The meaning of other options is– Satisfying :- comforting. Civilized :- humane. Charming :- pleasing. Correct sentence- Winning both the races has been a pleasant surprise, but the success is well deserved. 693. Select the most appropriate option to fill in the blank and make a grammatically correct sentence. I wasn't feeling well yesterday, but I feel a little ––––– today. (a) more better (b) best (c) better (d) good SSC MTS – 19/08/2019 (Shift-I) Ans. (c) : In the blank space of the above sentence, 'better' will be used because in this sentence 'yesterday (noun) is compared from 'today'. Hence, comparative degree 'better' is used of positive degree 'well.' Ex. Lata is a better singer than Asha. Correct sentence I wasn't feeling well yesterday, but I feel a little better today. 694. Select the most appropriate word to fill in the blank and make a meaningful sentence. She gave important evidence and I found her to be a sensible and ––––– witness. (a) credible (b) imposing (c) conclusive (d) unlikely SSC MTS – 16/08/2019 (Shift-III) Fill in the Blanks

Ans. (a) : In the blank space of the above sentence, 'credible' would be appropriate. Credible– able to be believed; convincing. The meaning of other options is– Imposing :- impressive. Conclusive :- decisive. Unlikely :- unbelievable. Correct sentence- She gave important evidence and I found her to be a sensible and credible witness. 695. Select the most appropriate word to fill in the blank and make a meaningful sentence. The two friends were too ––––– to admit that they were wrong. (a) satisfied (b) stubborn (c) voluntary (d) reasonable SSC MTS – 16/08/2019 (Shift-II) Ans.(b) : According to the sense of the above sentence, 'stubborn' would be appropriate. The meaning of the other options is– Satisfied :- contented. Voluntary :-willful Reasonable :- logical. Correct sentence- The two friends were too stubborn to admit that they were wrong. 696. Select the most appropriate option to fill in the blank and make a grammatically correct sentence. I would have forgotten my appointment if Susan ––––– reminded me about it. (a) hadn't (b) wouldn't (c) haven't (d) had SSC MTS – 16/08/2019 (Shift-I) Ans. (a) : In the blank space of the above sentence, 'hadn't' will be used because if + past perfect, sub + would have + V3. Hence 'Hadn't' would be appropriate in the blank space. Correct sentence- I would have forgotten my appointment if Susan hadn't reminded me about it. 697. Select the most appropriate word to fill in the blank and make a meaningful sentence. Which –––– should we adopt to increase our sales ? (a) promotions (b) strategies (c) functions (d) intentions SSC MTS – 16/08/2019 (Shift-I) Ans. (b) : In the blank space of the above sentence, 'strategies' will be used. because it gives appropriate meaning. The meaning of other options is– Promotions :- advancement. Function :- operate, work Intention :- mission, aim. Correct sentence- Which strategies should we adopt to increase our sales ? 698. Select the most appropriate word to fill in the blank and make a meaningful sentence. We try to keep abreast of the new treatments, but have gained little ––––– experience of using them. (a) systematic (b) practical (c) orderly (d) doable SSC MTS – 16/08/2019 (Shift-I)

282

YCT

Ans. (b) : In the blank space of the above sentence, 'practical' will be used. because it gives appropriate meaning. The meaning of other options is– Systematic :- organized. Orderly :- neat, tidy. Doable :- possible. Correct sentence- We try to keep abreast of the new treatments, but have gained little practical experience of using them. 699. Select the most appropriate option to fill in the blank and make a grammatically correct sentence: I was walking along the street when suddenly I ----------- footsteps behind me. (a) hear (b) hearing (c) heard (d) was hearing SSC MTS – 14/08/2019 (Shift-III) Ans. (c) : In the blank space of the above sentence, 'Heard' would be appropriate because there are two past action in a sentence. Then the verb past continuous is used for the work that is already going on and the verb past indefinite is used for the work that is going to happen later. Correct sentence- I was walking along the street when suddenly I heard footsteps behind me. 700. Select the most appropriate word to fill in the blank and make a meaningful sentence. A concerted effort has to be make in order to use our mineral resources in a planned and -------------- manner. (a) reserved (b) collected (c) preserved (d) sustainable SSC MTS – 14/08/2019 (Shift-III) Ans. (d) : In the blank space of the above sentence, sustainable' would be appropriate. The meaning of other options is– Reserved :- booked, engaged. Collected :- gathered, composed. Preserved :- conserve, maintain. Correct sentence- A concerted effort has to be make in order to use our mineral resources in a planned and sustainable manner. 701. Select the most appropriate word to fill in the blank and make a meaningful sentence. The common occurrences of fatigue and ------decrease when we listen to music. (a) Depletion (b) distaste (c) boredom (d) liveliness SSC MTS – 14/08/2019 (Shift-III) Ans. (c) : In the blank space of above sentence, 'Boredom' would be appropriate, because the word fatigue and boredom is appropriate with each-other. The meaning of other options is– Deplete :- consume, wipe out. Distaste :- antipathy, aversion. Liveliness :- animation, life. Correct Sentence- The common occurrences of fatigue and boredom decrease when we listen to music. Fill in the Blanks

702. Select the most appropriate option to fill in the blank and make a grammatically correct sentence. Chris and I play tennis together regularly, but ––––––––– of us can play very well. (a) both (b) either (c) none (d) neither SSC MTS – 14/08/2019 (Shift-II) Ans. (d) : In the blank space of the above sentence, 'Neither' is used because 'neither' is a distributive pronoun, used to negate each of two persons. 'But' is a coordinating conjunction used to cnnect opposite ideas. The other options give different meaning. Either :- one or the other of two. None :- not any, not one. Both :- the one as well as the other. Correct sentence- Chris and I play tennis together regularly, but neither of us can play very well. 703. Select the most appropriate word to fill in the blank and make a meaningful sentence. When we talk of 'globalization' we often refer to an economic system that has ––––––– in the last 50 years or so. (a) proceeded (b) emerged (c) derived (d) flowed SSC MTS – 14/08/2019 (Shift-II) Ans. (b) : In the blank space of the above sentence, 'emerged' will be used. because it gives appropriate meaning. The meaning of other options is– Proceed :- continue, go forward. Derived :- plagiarised. Correct sentence- When we talk of 'globalization' we often refer to an economic system that has emerged in the last 50 years or so. 704. Select the most appropriate word to fill in the blank and make a meaningful sentence. The idea of Satyagraha –––––– the power of truth. (a) impressed (b) emphasized (c) created (d) insisted SSC MTS – 14/08/2019 (Shift-II) Ans. (b) : In the blank space of the above sentence, 'emphasized' will be used. because it gives appropriate meaning. The meaning of other options is– Impressed :- affected Created :- to cause something new to happen or exist. Insist :- take a firm stand. Correct sentence- The idea of Satyagraha emphasized the power of truth. 705. Select the most appropriate option to fill in the blank and make a grammatically correct sentence. I tried to contact her two or three times but ................. time there was no reply. (a) each (b) both (c) all (d) any SSC MTS – 14/08/2019 (Shift-II)

283

YCT

(a) pry (b) ransack Ans. (a) : In the blank space of the above sentence, (c) seek (d) poach 'each' will be used because we use 'each' to refer to SSC MTS – 13/08/2019 (Shift-III) individual things in a group or a list of two or more things. It is normally followed by a singular verb. Ans. (b) : According to the sense of the above The other meaning– sentence, 'ransack' (loot) would be appropriate in the Both :- the two blank space. All :- the whole of a thing. The meaning of other options is– Any :- used instead of 'some' in negative sentence and Pry :- intrude, poke. questions. Seek :- look for, search. Correct Sentence- I tried to contact her two or three Poach :- hint illegally. times but each time there was no reply. Correct sentence- Searching frantically to find all the 706. Select the most appropriate option to fill in the hidden jewels and cash, the burglars proceeded to blank and make a grammatically correct sentence ransack the entire house. Susan finally arrived at the party at midnight, 710. Select the most appropriate word to fill in the but by then, most of the guests _______. blank and make a meaningful sentence. (a) having left (b) have left Nowadays, digital services have made our cash (c) had left (d) left transactions easier and –––––. SSC MTS – 14/08/2019 (Shift-I) (a) softer (b) smoother Ans. (c) : In the blank space of the above sentence, (c) heavier (d) dearer 'had left' will be used because one of the two works is SSC MTS – 13/08/2019 (Shift-II) finished earlier and the other work is finished later Ans. (b) : According to the sense of the above then the first work is written in past perfect tense and sentence, 'smoother' (more easy) would be appropriate the later work is written in past indefinite. in the blank space. Correct sentence- Susan finally arrived at the party at The meaning of other options is– midnight, but by then, most of the guests had left. Softer :- flaccid, mushy. 707. Select the most appropriate word to fill in the Heavier :- overweight. blank and make a meaningful sentence. There are some dates or periods of time in the Dearer :- beloved. history of the world that are so ______that Correct sentence- Nowadays, digital services have made our cash transactions easier and smoother everyone knows and remembers them. 711. Select the most appropriate word to fill in the (a) trivial (b) representative blank and make a meaningful sentence. (c) significant (d) momentary The development of real estate has –––––– SSC MTS – 14/08/2019 (Shift-I) impacted the environment. Ans. (c) : In the blank space of the above sentence,. (a) narrowly (b) positively 'significant' will be used. because it gives appropriate (c) conveniently (d) adversely meaning. SSC MTS – 13/08/2019 (Shift-I) The meaning of the other options is– Trivial :- superficial. Ans. (d) : According to the sense of the above Representative :- symbolic, spokesperson. sentence, 'Adversely' (against) would be appropriate. Momentary :- fugitive. The other options give different meaning. Correct sentence- 'There are some dates or periods of Narrowly :- only by a small amount. time in the history of the world that are so significant Positively :- so as to be positive. that everyone knows and remembers them.' Conveniently :- handily. 708. Select the most appropriate word to fill in the Correct sentence- The development of real estate has blank and make a meaningful sentence. adversely impacted the environment. Mother__ some food for the old beggar woman. 712. Select the most appropriate word to fill in the (a) set up (b) set out blank and make a meaningful sentence. (c) set aside (d) set in Be careful not to drop this glass case; it's very – SSC MTS – 14/08/2019 (Shift-I) –––. Ans. (c) : In the blank space of the above sentence, 'set (a) fragile (b) rigid aside' would be appropriate, which means 'to save'. (c) smooth (d) hard The meaning of other options is– SSC MTS – 13/08/2019 (Shift-I) Set up :- establish Ans. (a) : According to the sense of the above set out :- depart, set off. sentence, 'fragile' (weak) would be appropriate in the set in :- become established, begin. blank space. Correct sentence- Mother set aside some food for the The meaning of other options is– old beggar woman. Rigid :- strict 709. Select the most appropriate word to fill in the Smooth :- slippy. blank and make a meaningful sentence. Searching frantically to find all the hidden Hard :- solid. jewels and cash, the burglars proceeded to –––– Correct sentence- Be careful not to drop this glass case; it's very fragile. the entire house. Fill in the Blanks

284

YCT

713. Select the most appropriate word to fill in the blank and make a meaningful sentence. The author touched the ––––– of her career with her second novel which won the Pulitzer Prize. (a) peek (b) nadir (c) base (d) peak SSC MTS – 09/08/2019 (Shift-III) Ans. (d) : According to the sense of the above sentence, 'Peak' gives appropriate meaning. Peek :- peep. Nadir :- Downfal, Base :- floor, basic Correct sentence- The author touched the peak of her career with her second novel which won the Pulitzer Prize. 714. Select the most appropriate word to fill in the blank and make a meaningful sentence. She seemed ––––about the plan as she did not react much. (a) uninterested (b) enthusiastic (c) excited (d) interested SSC MTS – 09/08/2019 (Shift-III) Ans. (a) : According to the sense of the above sentence, 'uninterested' (indifferent) give the suitable meaning. in the blank space. Other options give different meaning. Enthusiastic :- keen, warm. Excited :- agitated, thrilled. Interested :- having or showing interest, curious. Correct sentence- She seemed uninterested about the plan as she did not react much. 715. Select the most appropriate option to fill in the blank and make a grammatically correct sentence. I was determined not to stay in hospital for any longer –––– was strictly necessary. (a) when (b) than (c) then (d) that SSC MTS – 09/08/2019 (Shift-III) Ans. (b) : In the blank space of the above sentence, 'than' will be used because after comparative degree (longer), 'than' would be appropriate. In given sentence, 'longer' is used as a comparative degree. Correct sentence- I was determined not to stay in hospital for any longer than was strictly necessary. 716. Select the most appropriate word to fill in the blank and make a meaningful sentence. Owing to all the recent negative turn of events in her life, strongly felt –––– forces must be at work. (a) helpful (b) evil (c) angelic (d) haunting SSC MTS – 09/08/2019 (Shift-II) Ans. (b) In the blank space of the above sentence, 'evil' would be appropriate. The sentence, is showing negative meaning and 'haunting' is not used because 'evil force' is used as a phrase. Correct sentence- Owing to all the recent negative turn of events in her life, strongly felt evil forces must be at work. Fill in the Blanks

717. Select the most appropriate word to fill in the blank and make a meaningful sentence. Twenty families were ––––– from an old building before the monsoons. (a) erased (b) evacuated (c) emptied (d) expelled SSC MTS – 08/08/2019 (Shift-II) Ans. (b) : According to the sense of the above sentence, 'evacuated' would be appropriate. The meaning of other options is – Erase :- efface. Empty :- blank, vacant. Expel :- kick out, eject. Correct sentence- Twenty families were evacuated from an old building before the monsoons. 718. Select the most appropriate word to fill in the blank and make a meaningful sentence. In some disputes face-to-face talk can ––––– the issue. (a) restrict (b) refuse (c) resolve (d) reduce SSC MTS – 08/08/2019 (Shift-II) Ans. (c) : In the blank space of the above sentence, 'resolve' would be appropriate. The meaning of other options is– Restrict :- curtail. Refuse :- decline, reject. Reduce :- shrink. Correct sentence- In some disputes face-to-face talk can resolve the issue. 719. Select the most appropriate option to fill in the blank. The air-conditioner has made ceiling fans a little ––––– in today's world. (a) extinct (b) absurd (c) additional (d) redundant SSC MTS – 07/08/2019 (Shift-III) Ans. (d) : According to the sense of the above sentence, 'Redundant' (unnecessary) would be appropriate in the blank space. Extinct :- dead. Absurd :- illogical Additional :- Extra. Correct sentence- The air-conditioner has made ceiling fans a little redundant in today's world. 720. Select the most appropriate option to fill in the blank. My little cousin is ––––– a movie. (a) watching (b) looking (c) seeing (d) observing SSC MTS – 07/08/2019 (Shift-III) Ans. (a) : In the blank space of the above sentence, 'watching' will be used. Because 'watch' is used to see the movie, T.V. serial etc. Correct sentence- My little cousin is watching a movie. 721. Select the most appropriate option to fill in the blank. The frightened convict –––––– for mercy. (a) shouted (b) whispered (c) mumbled (d) pleaded SSC MTS – 07/08/2019 (Shift-II)

285

YCT

Ans. (c) : In the blank space of the above sentence, Ans. (d) : In the blank space of the above sentence, 'emphasising' will be used. 'pleated' would be appropriate. The meaning of other options is– The meaning of other options is– Shout :- cry, yell. Insisting :- continual and persistent Whisper :- rusted. Decrying :- condemn Mumbled :- mutter. Avoiding :- debar, prevent from happining. Correct sentence- The frightened convict pleaded for Correct sentence- The speaker had not written his mercy. speech properly and went on emphasising one point 722. Select the most appropriate option to fill in the only. blank. 726. Select the most appropriate word to fill in the The passengers were afraid, but the captain –– blank. ––– them that there was no danger. If you drink too much, it will ______ your (a) cautioned (b) assured judgment. (c) claimed (d) confided (a) Impel (b) Impede SSC MTS – 07/08/2019 (Shift-II) (c) Impose (d) Improve Ans. (b) : According to the sense of the above SSC GD – 08/03/2019 (Shift-III) sentence, 'Assured' would be appropriate in the blank Ans. (b) : In the blank space of the above sentence, space. 'impede' will be used. because it gives appropriate The meaning of other options is– meaning. Caution :- care, precaution. The meaning of other options is– Claim :- arrogate, lay claim. Impel :- force. Confide :- reveal in private. Impose :- enforce, bring down. Correct sentence- The passengers were afraid, but the Improve :- ameliorate, amend. captain assured them that there was no danger Correct sentence- If you drink too much, it will 723. Select the most appropriate option to fill in the impede your judgment. blank. 727. Select the most appropriate word to fill in the She had a bandage –––– round her finger. blank. (a) covered (b) twisted The great advantage of early rising is the good (c) wrapped (d) bound ______ is given us to our day. SSC MTS – 06/08/2019 (Shift-II) (a) sense (b) kick Ans. (c) : In the blank space of the above sentence, (c) start (d) humour 'wrapped' would be appropriate. SSC GD – 08/03/2019 (Shift-II) The other options give different meaning. Ans. (c) : In the blank space of the above sentence, Covered :- wrapped, crusted. 'start' (begin) will be used. because it gives appropriate Twisted :- bend, turn. meaning. Bound :- rolled, wired. The meaning of other options is– Correct sentence- She had a bandage wrapped round Sense :- sentience, sentiency. her finger. 724. Select the most appropriate option to fill in the Kick :- hit, strike with the foot. Humour :- wittiness. blank. They sat by the river bank with their legs –––– correct sentence– The great advantage of early rising is the good start is in the water. given us to our day. (a) remaining (b) swimming (c) washing (d) dangling 728. Select the most appropriate word to fill in the SSC MTS – 06/08/2019 (Shift-II) blank. Ans. (d) : In the blank space of the above sentence, The new textile policy lays great emphasis on 'dangling' (hanging) will be used. stabilizing cotton prices to ______ the interest of cotton growers. The meaning of other options is– Remaining :- left (a) improve (b) diversify Swimming :- to move your body through water. (c) protect (d) raise Washing :- laundry. SSC GD – 08/03/2019 (Shift-II) Correct sentence- They sat by the river bank with Ans. (d) : In the blank space of the above sentence, their legs dangling in the water. 'raise' would be appropriate. 725. Select the most appropriate word to fill in the The other options give different meaning. blank. Improve :- amend. The speaker had not written his speech Diversity :- vary, change. properly and went on _____ one point only. Protect :- secure. (a) Insisting (b) Decrying Correct sentence- The new textile policy lays great (c) Emphasising (d) Avoiding emphasis on stabilizing cotton prices to raise the SSC GD – 08/03/2019 (Shift-III) interest of cotton growers. Fill in the Blanks

286

YCT

729. Select the most appropriate word to fill in the Ans. (b) : According to the sense of the above blank. sentence, 'passion' would be appropriate in the blank In order to cut down on errors, a keyboard that space. can be ______ to suit individual typing style is The meaning of other options is– needed. Claim :-affirm strongly. (a) predicated (b) shared Response :- reply, answer. (c) projected (d) adjusted Demand :- necessitate, require. SSC GD – 02/03/2019 (Shift-II) Correct sentence- If you have a passion for Ans. (d) : In the blank space of the above sentence, photography, you do not have to wait for your big 'adjusted' would be appropriate. break. The meaning of other options is– 733. Select the most appropriate option to fill in the Predicate :- connote, proclaim. blank. Share :- part, portion. The baby bird was _______ by its mother. Projected :- plan, visualise. (a) thrown (b) renounced Correct sentence- In order to cut down on errors, a (c) abandoned (d) sacrificed keyboard that can be adjusted to suit individual typing SSC GD – 18/02/2019 (Shift-III) style is needed. 730. Select the most appropriate word to fill in the Ans. (c) : In the blank space of the above sentence, 'abandoned' would be appropriate. blank. When the bat pays attention to an object, the The meaning of other options is– Thrown :- to send something from your hand through ______ of that object sharpens in its brain. the air by moving your hand. (a) representation (b) depiction Renounced :-forsake, quit. (c) appearance (d) portrait SSC GD – 02/03/2019 (Shift-II) Sacrificed :- ritual killing, loss. Correct sentence- The baby bird was abandoned by Ans. (b) : According to the sense of the above its mother. sentence, 'depiction' would be appropriate. 734. Select the most appropriate word to fill in the The meaning of other options is– blank. Representation :- the way that somebody/something is shown or described. Dinosaurs are an _______ species now. Appearance :- visual aspect, show. (a) extinct (b) antique Portrait :- depiction, picture. (c) ancient (d) ageold Correct sentence- When the bat pays attention to an SSC GD – 18/02/2019 (Shift-III) object, the depiction of that object sharpens in its Ans. (a) : In the blank space of the above sentence, brain. 'extinct' (dead) would be appropriate. 731. Select the most appropriate word to fill in the The meaning of other options is– blank. Antique :- old, unstylish. The complexity of the current environment Ancient :- past (very old) includes many challenges in general, but air Age old :- aged. pollution in _______. Correct sentence- Dinosaurs are an extinct species (a) public (b) common now. (c) overall (d) particular 735. Select the most appropriate word to fill in the SSC GD – 02/03/2019 (Shift-I) blank. Ans. (d) : According to the sense of the above Apart from cows, buffaloes are also ______ for sentence, 'particular' will be used in the blank space. mass milk production for humans. The meaning of other options is– (a) developed (b) grown Public :- open, common (c) produced (d) reared Overall :- including, entire SSC GD – 18/02/2019 (Shift-III) Common :- average, ordinary. Ans. (d) : According to the sense of the above Correct sentence- The complexity of the current sentence, 'reared' (bring up) would be appropriate in environment includes many challenges in general, but the blank space. air pollution in particular. 732. Select the most appropriate word to fill in the The meaning of other options is– Developed :- grow, produce. blank. If you have a _______ for photography, you do Grown :- mature. Produce :- acquire, create. not have to wait for your big break. Correct sentence(a) claim (b) passion Apart from cows, buffaloes are also reared for mass (c) response (d) demand SSC GD – 02/03/2019 (Shift-I) milk production for humans. Fill in the Blanks

287

YCT

736. Select the most appropriate option to fill in the Ans. (d) : According to the sense of the above blank. sentence, 'Attraction' would be appropriate in the blank Scientists tried to find out the _____ behind the space. occurrence of the phenomenon. The meaning of other options is– (a) cause (b) clarification Displays :- show, presentation. (c) illustration (d) interpretation SSC GD – 18/02/2019 (Shift-III) Spectacle :- something or someone seen. Demonstrations :- protest, display. Ans. (a) : According to the sense of the above Correct sentence- India's Republic Day Parade had, sentence, 'cause' (reason) would be appropriate in the among the main attractions an all-women's marching blank space. contingent. The meaning of other options is– Clarification :- clearing 740. Select the most appropriate option to fill in the Illustration :- representative. blank. Interpretation :- explanation. Sometimes it is necessary for an author to Correct sentenceknow what is going on in the minds of his Scientists tried to find out the cause behind the ______. occurrence of the phenomenon. (a) audience (b) listeners 737. Select the most appropriate option to fill in the (c) viewers (d) characters blank. SSC GD – 18/02/2019 (Shift-II) If an individual keeps his fingers crossed he Ans. (d) : According to the sense of the above ______ for the best. sentence, 'characters' (plural noun) will be used in the (a) hopes (b) responds blank space. Because here to denote the author's own (c) reacts (d) suspects character/nature, the use of characters would be SSC GD – 18/02/2019 (Shift-II) appropriate. Ans. (a) : In the blank space of the above sentence, Audience :- hearing, viewers 'hopes' will be used because 'keeps his fingers crossed' Viewers :- a person who watches television. is a phrase which means– 'keep faith'. Listener :- auditor, attender. Given sentence is a zero conditional sentence. Correct sentence- Sometimes it is necessary for an If, Present Indefinite, Present Indefinite author to know what is going on in the minds of his The meaning of other options is– characters. Responds :- reply, answer. 741. Select the most appropriate word to fill in the Reacts :- respond, oppose, blank. Suspect :- distrust, disbelieve. _______ are to be avoided; they lead to conflict. Correct sentence- If an individual keeps his fingers (a) Question (b) Statements crossed he hopes for the best. (c) Arguments (d) Decisions 738. Select the most appropriate option to fill in the SSC GD – 14/02/2019 (Shift-II) blank. We need not worry about anything ______ his Ans. (c) : In the blank space of the above sentence, commitment to the work he is assigned. 'argument' would be appropriate. (a) except (b) access The meaning of other options is– (c) expect (d) accept Question :- Interrogate. SSC GD – 18/02/2019 (Shift-II) Statement :- assertion. Ans. (a) : According to the sense of the above Decision :- verdict. sentence, 'except' would be appropriate in the blank Correct sentence- Arguments are to be avoided; they space. lead to conflict. The other options give different meaning. 742. Select the most appropriate option to fill in the Access :- approach. blank. Accept :- receive. You can visit me on ............. at 4 o'clock. Except :- besides, (a) tomorrow (b) next month Correct sentence- We need not worry about anything except his commitment to the work he is assigned. (c) today (d) Sunday 739. Select the most appropriate word to fill in the SSC GD – 14/02/2019 (Shift-II) blank. Ans. (d) : In the above sentence, on (preposition) is India's Republic Day Parade had, among the used so in the blank space, 'Sunday' would be main______, an all-women's marching appropriate. Because preposition 'on' is used before the contingent. name of the days. (a) displays (b) spectacle Correct sentence- You can visit me on Sunday at 4 (c) demonstrations (d) attractions SSC GD – 18/02/2019 (Shift-II) o'clock. Fill in the Blanks

288

YCT

743. Select the most appropriate word to fill in the blank. Just are the _______ of God. (a) ways (b) deeds (c) plans (d) tricks SSC GD – 14/02/2019 (Shift-II) Ans. (a) : In the blank space of the above sentence, 'ways' will be used. because it gives appropriate meaning. The meaning of other options is– Deed :- human activity. Plan :- program. Trick :- deceitful action or device. Correct sentence- Just are the ways of God. 744. Select the most appropriate word to fill in the blank. Smoking is ______ to health (a) tedious (b) serious (c) anxious (d) injurious SSC GD – 14/02/2019 (Shift-I) Ans. (d) : In the blank space of the above sentence, 'injurious' would be appropriate. The meaning of other options is– Tedious :- dull. Serious :- sober, sincere. Anxious :- nervous, uneasy. Correct sentenceSmoking is injurious to health. 745. Select the most appropriate word to fill in the blank. Let us_______ our heads together to solve the problem. (a) join (b) put (c) bring (d) combine SSC GD – 14/02/2019 (Shift-I) Ans. (b) : According to the sense of the above sentence, 'put' will be used in the blank space because 'put their heads together' is an idiom which means– 'consult and work together'. Correct sentence- Let us put our heads together to solve the problem. 746. Select the most appropriate word to fill in the blank. The ________ of the museum showed us some ancient coins. (a) controller (b) conservator (c) collector (d) curator SSC GD – 11/02/2019 (Shift-III) Ans. (d) : In the blank space of the above sentence, 'curator' (conservator) would be appropriate. Because here talked about museum. The meaning of other options is– Controller :- restrainer. Conservator :- the protection of the natural world. Collector :- accumulator, gatherer. Correct sentence- The curator of the museum showed us some ancient coins. Fill in the Blanks

747. Select the most appropriate word to fill in the blank. The Prime Minister was as good as his ______. (a) word (b) office (c) sword (d) dress SSC GD – 11/02/2019 (Shift-II) Ans. (a) : According to the sense of the above sentence, 'word' (promise) would be appropriate in the blank space. The meaning of the other options is– Office :- federal, agency. Sword :- blade, weapon. Dress :- apparel. Correct sentence- The Prime Minister was as good as his word. 748. Select the most appropriate word to fill in the blank. They never ______ those who have done wrong. (a) condemn (b) outcast (c) blame (d) forgive SSC GD – 11/02/2019 (Shift-II) Ans. (d) : According to the sense of the above sentence, 'forgive' would be appropriate in the blank space. The other options give different meaning. Condemn :- decry, objurgate. Outcast :- religious outcast. Blame :- fault, charge. Correct sentence- They never forgive those who have done wrong. 749. Select the most appropriate word to fill in the blank. I make friends ______ I go. (a) moreover (b) wherever (c) however (d) whenever SSC GD – 11/02/2019 (Shift-II) Ans. (b) : According to the sense of the above sentence, 'wherever' would be appropriate. The meaning of other options is– Moreover :- furthermore. However :- nevertheless. Whenever :- at any time. Correct sentence- I make friends wherever I go 750. Select the most appropriate word to fill in the blank. They showed her the greatest (............). (a) praise (b) tribute (c) honour (d) respect SSC GD–11/02/2019 (Shift-III) Ans. (d) : In the blank space of the above sentence, 'respect' will be used. because it gives appropriate meaning. The meaning of other options is– Praise :- acclaim, homage, laurels. Tribute :- testimonial. Honour :-respect. Correct sentence- They showed her the greatest respect.

289

YCT

04.

ACTIVE/PASSIVE VOICE TCS hewšve& hej DeeOeeefjle (Based On TCS Pattern) Chapterwise

Exam

Question No.

Years

1

CGL (Tier-1) CGL (Tier-2) CHSL (Tier-1) CHSL (Tier-2) Selection Post VII, VIII, XI SSC MTS SSC GD SSC CPO SI

70 45 90 35 5 35 5 40

(2017–2023)

Active/Passive Voice

Trend Analysis of Questions topicwise from CGL (Pre & Mains) CHSL (Pre & Mains) Selection Post VII, VIII, XI, SSC MTS, SSC GD & Other Exams (2017-2023)

Active / Passive Voice

290

YCT

04. ACTIVE/PASSIVE VOICE 1.

Select the option that express the given sentence in active voice. The article had not been posted by Mr. Gupta. (a) Mr. Gupta still had not posted the article. (b) Mr. Gupta had not posted the article. (c) Mr. Gupta have not been posted the article. (d) Mr. Gupta has not posted the article. SSC CGL (Tier-I) 19/07/2023 (Shift-IV) Ans. (b) : The above sentence is in Passive Voice of Past Perfect tense. Its Active Voice structure will beA.V.- Sub. + had + V3 + Object P.V.- Object + had + been + V3 + by + Sub. Thus, the correct answer is option (b). Correct SentenceMr. Gupta had not posted the article. 2. Select the option that expresses the given sentence in passive voice. The ruler of the community had already passed a decree against that decision. (a) A decree had already passed by the ruler of the community against that decision. (b) A decree had already been passed by the ruler of the community against that decision. (c) A decree was already passed by the ruler of the community against that decision. (d) A decree was already being passed by the ruler of the community against that decision. SSC CGL (Tier-I) 19/07/2023 (Shift-IV) Ans. (b) : For the above given sentence is in active voice, in past perfect tense. Its Passive Voice structure will beA.V.- Subject + had + V3 + Object. P.V.- Obj + had + been + V3 + by Subject. Thus, the correct answer is option (b). Correct SentenceA decree had already been passed by the ruler of the community against that decision. 3. Select the option that expresses the given sentence in active voice. Wedding invitations will be sent by them. (a) They will send wedding invitations. (b) They will be sent wedding invitations. (c) He will sent wedding invitations. (d) He will be send wedding invitations. SSC CGL (Tier-I) 21/07/2023 (Shift-II) Ans. (a) : The above given sentence is in 'Passive voice.' of future Indefinite Tense and its 'Active voice' structure will beP.V.- Object + will/shall + be + VIII + by + Subject. A.V.- Subject + will/shall + VI + Object. Correct SentenceThey will send wedding invitations. Active / Passive Voice

4.

Select the option that expresses the given sentence in passive voice. He has not committed the crime. (a) The crime has not been committed by him. (b) The crime have not been committed by him. (c) The crime was not committed by him. (d) The crime have not being committed by him. SSC CGL (Tier-I) 21/07/2023 (Shift-II) Ans. (a) : The above given sentence is in Active Voice of present perfect tense. Its Passive structure will be A.V. ⇒ Subject + has/have + not + VIII + object. P.V. ⇒ Object + has/have + not + been + VIII + by + Subject. Correct SentenceThe crime has not been committed by him. 5. Select the option that expresses the given sentence in active voice. Were the cakes being baked by them? (a) Was cakes being baked by them? (b) Were they baking the cakes? (c) Were they baking the cake? (d) Did the cakes baked by them? SSC CGL (Tier-I) 26/07/2023 (Shift-II) Ans. (b) : The correct active voice is in the given sentence 'were the cakes being backed by them?' is 'were they baking the cakes?' because the given sentence is in passive voice of interogative of past continuous tense. The Structure of Action sentence is– A.V.- was/were + sub + V1 + ing + Obj? P.V.- was/were + object + being + v3 + by + sub? Thus, the correct answer is option (b) 6. Select the option that expresses the given sentence in active voice. The manager was being scolded by Rushikesh. (a) Rushikesh has been scolding the manager. (b) Rushikesh will be scolding the manager. (c) Rushikesh was scolding the manager. (d) Rushikesh scolded the manager. SSC Selection Posts XI-27/06/2023 (Shift-I) Ans. (c) : The above sentence is in passive voice of past continuous tense. Its active voice structure will be P.V. – Obj. + was/were + being + VIII + by + sub. A. V. – Sub. + was/ were + VI + ing + obj. The correct active voice – Rushikesh was scolding the manager. 7. Select the option that expresses the given sentence in passive voice. The tour guide always answers the visitors’ questions.

291

YCT

(a) The visitors’ questions are answered by the tour guide always. (b) The visitors’ questions are always answered by the tour guide. (c) The visitors’ question was always answered by the tour guide. (d) The visitors’ questions are answered always by the tour guide. SSC CGL (Tier-I) 26/07/2023 (Shift-II) Ans. (b) : For the above given sentence is Active Voice of the present indefinite tense. Its Passive voice structure will be A.V.- subject + V1 + s/es + object P. V.- object + is/am/are + V3 + by + subject Thus, the correct answer is option (b) 8. Select the option that expresses the given sentence in passive voice. You have adopted the plan. (a) The plan has being adopted by you. (b) The plan should be adopted by you. (c) The plan has been adopted by you. (d) The plan is adopted by you. SSC CGL (Tier-I) 25/07/2023 (Shift-IV) Ans. (c) : The above given sentence is in active voice of present perfect tense, its passive structure will be⇒ The structure of active voice Subject + has/have + VIII + Object ⇒ The structure of Passive voiceSubject + has/have + been + VIII + by + Subject Hence, the correct option is (c). The correct passive voice of this sentence is 'The plan has been adopted by you.' 9. Select the option that expresses the given sentence in passive voice. Each member of the literary club submitted various literary works for the magazine. (a) Various literary works were being submitted by each member of the literary club for the magazine. (b) Various literary works had been submitted by each member of the literary club for the magazine. (c) Various literary works were submitted by each member of the literary club for the magazine. (d) Various literary works are submitted by each member of the literary club for the magazine. SSC CGL (Tier-I) 25/07/2023 (Shift-IV) Ans. (c) : The above sentence is in active voice of past indefinite tense. Hence its passive voice structure will be (c) correct sentence 'Various literary works were submitted by each member of the literary club for the magazine.' A.V.- Subject + VII + object. P.V.- Object + were + VIII + by + Subject. Active / Passive Voice

10.

Describe how you will tell everyone that Mr. Akhil opened the gate in passive voice. (a) Mr.Akhil open the gate. (b) The gate was opened by Mr.Akhil. (c) The gate opened by Mr.Akhil. (d) The gate been open by Mr.Akhil. SSC CGL (Tier-I) 18/07/2023 (Shift-III) Ans. (b) : The above sentence is in active voice of simple past tense. Its passive voice structure will be A.V. ⇒ Sub + VII + obj P.V. ⇒ Obj. + was/were + VIII + by + sub Correct Passive VoiceThe gate was opened by Mr. Akhil. 11. Describe how you will tell your friends that Mr. Ram uses multimedia for teaching the children in passive voice. (a) Multimedia is used by Mr. Ram for teaching the children. (b) Multimedia is used for teach the children by Mr. Ram. (c) Multimedia is using for teaching the children. (d) Mr. Ram use multimedia for teaching the children. SSC CGL (Tier-I) 18/07/2023 (Shift-III) Ans. (a) : The above sentence is in active voice of simple present tense. Its passive voice structure will be A.V. ⇒ Sub. + V1 + s/es + obj. + other words. P.V. ⇒ Obj. + is/am/are + VIII + by + sub + other words Correct passive voice - Multimedia is used by Mr. Ram for teaching the children. 12. Select the option that expresses the given sentence in passive voice. He didn’t eat a single morsel of food at his daughter’s wedding. (a) A single morsel of food was not eaten by him at his daughter’s wedding. (b) A single morsel of food can not be eaten by him at his daughter’s wedding. (c) A single morsel of food was not being eaten by him at his daughter’s wedding. (d) A single morsel of food is not eaten by him at his daughter’s wedding. SSC CGL (Tier-I) 20/07/2023 (Shift-I) Ans. (a) : For the above given sentence is in ‘Active form’ of past Indefinite tense’ A.V.- Sub. + V2 + Obj. So, Structure of the passive of part Indefinite tense is– P.V.- Obj. + was/were + V3 + by +Sub. Thus, the correct answer is option (a). 13. Select the option that expresses the given sentence in passive voice. The workers had eaten all the pastries before the day broke. (a) All the pastries are being eaten by the workers before the day broke. (b) All the pastries were being eaten by the workers before the day broke. (c) All the pastries had been eaten by the workers before the day broke. (d) All the pastries have been eaten by the workers before the day broke. SSC CGL (Tier-I) 20/07/2023 (Shift-I)

292

YCT

Ans. (c) : For the above given sentence is in ‘Active form’ of past perfect tense. So, there is passive structure of past perfect tense is – A.V.- Sub. + had + v3 + obj. P.V.- Obj. + had + been + V3 + by + Sub Correct Passive Voice Thus, the correct answer is option (c). 14. Select the option that expresses the given sentences in passive voice. We are organising the charity function tomorrow. (a) The charity function is being organised tomorrow. (b) The charity function is being organise tomorrow. (c) The charity function is been organised tomorrow. (d) The charity function is organised tomorrow. SSC CGL (Tier-I) 27/07/2023 (Shift-III) Ans. (a) : The above given sentence is in 'Active Voice' of present continuous tense and the 'Passive Voice' of this sentence is option (a) 'The charity function is being organised tomorrow.' Rules of 'Active and Passive Voice'A.V.- Subject + is/am/are + v1 + ing + object. P.V.- Object + is/am/are + being + VIII + by + Subject. Thus, the correct answer is option (a). 15. Select the option that expresses the given sentence in active voice. The deer was killed by the boar. (a) The boar was killing the deer. (b) The boar was killed by the deer. (c) The boar killed the deer. (d) The deer killed the boar. SSC CGL (Tier-I) 27/07/2023 (Shift-III) Ans. (c) : The above given sentence is in 'Passive Voice', of past indefinite tense and the 'Active Voice' of this sentence is- 'The boar killed the deer'. Rules of 'Active and Passive Voice'A.V.- Subj. + V2 + object P.V.- object + was/were + VIII + by + subject Thus, the correct answer is option (c). 16. Select the option that expresses the given sentence in passive voice. The company’s board of directors will announce the financial results at the annual meeting tomorrow. (a) The financial results will have been announced by the company’s board of directors at the annual meeting tomorrow. (b) The financial results are being announced by the company’s board of directors at the annual meeting tomorrow. (c) The company’s board of directors announced the financial results at the annual meeting tomorrow. (d) The financial result will be announced by the company’s board of directors at the annual meeting tomorrow. SSC CGL (Tier-I) 14/07/2023 (Shift-I) Active / Passive Voice

Ans. (d) : The above sentence is in active voice of future indefinite tense and its passive structure is P.v – Object + will/shall + be + verb3 + by subject + other word. Correct sentence – The financial result will be announced by the company’s board of directors at the annual meeting tomorrow. 17.

Select the option that expresses the following sentence in active voice. By whom were you taught English grammar and composition? (a) Who has taught you English grammar and composition? (b) Who taught you English grammar and composition? (c) Who had taught you English grammar and composition? (d) Whom taught you English grammar and composition? SSC CGL Mains 26/10/2023 (Shift-I) Ans. (b) : The above sentence is passive voice of post indefinite. Its active voice structure will be P.V – By whom + was/were + obje. + VIII + Other words. A.V – Who + VII + Object + Other words. Correct Sentence Who taught you English grammar and Composition?

18.

Select the option that expresses the given sentence in active voice. Why was such a letter written by your brother? (a) Why is your brother writing such a letter? (b) Why did your brother write such a letter? (c) Why has your brother written such a letter? (d) Why your brother wrote such a letter? SSC Selection Posts XI-28/06/2023 (Shift-III) Ans. (b) : The above sentence is passive voice of interrogative of past indefinite tense. Its active structure will beP.V.- Why + was/were + object + VIII + by + Subject. A.V.- Why + did + subject + VI + object. Correct SentenceWhy did your brother write such a letter? 19. Select the option that expresses the given sentence in passive voice. He won’t receive any better choice than this from anywhere. (a) Any better choice won’t be received by him than this from anywhere. (b) Any better choice won’t have been be received by him than this from any where. (c) Any better choice wouldn’t have received by him than this from anywhere. (d) Any better choice shouldn’t be received by him than this from anywhere. SSC CGL (Tier-I) 14/07/2023 (Shift-I)

293

YCT

Ans. (a) : The above sentence is in active voice of future indefinite tense and its passive structure is – P.V.– object + will/shall + not + be + V3 + by + subject + other words. Correct Sentence– Any better choice won’t be received by him than this from anywhere. 20. Select the option that expresses the given sentence in passive voice She might have completed her research by that time. (a) Her research might have completed by her by that time. (b) Her research might had been completed by her by that time. (c) Her research might have been completed by her by that time. (d) Her research might be completed by her by that time. SSC CGL (Tier-I) 17/07/2023 (Shift-II) Ans. (c) : The above sentence is Active voice of present perfect tense. Its passive voice structure will be – A.V. – subject + might have + V3 + object P.V. – object + might have + been + V3 + by + subject. Correct sentence – Her research might have been completed by her by that time. 21. Select the option that expresses the given sentence in active voice. By whom was the coffee made? (a) Who had made the coffee? (b) Who makes the coffee? (c) Who made the coffee? (d) Who has made the coffee? SSC CGL (Tier-I) 17/07/2023 (Shift-II) Ans. (c) : The above sentence is passive voice of interrogative of past Indefinite tense, Its Active voice structure will beP.V.– By whom + was + v3 + object A.V.– who + V2 + object Correct sentence – who made the coffee ? 22. Select the option that expresses the given sentence in passive voice. One group of the club has raised an undigestible criticism against the manager. (a) An undigestible criticism had been raised by one group of the club against the manager. (b) An undigestible criticism would have been raised by one group of the club against the manager. (c) An undigestible criticism was being raised by one group of the club against the manager. (d) An undigestible criticism has been raised by one group of the club against the manager. SSC CGL (Tier-I) 24/07/2023 (Shift-III) Active / Passive Voice

Ans. (d) : The above sentence is Active Voice of Present Perfect tense. Its Passive Structure will beA.V.- Sub. + has/have + v3 + obj. P.V.- Obj. + has/have + been + v3 +by + sub. Correct SentenceAn undigestible criticism has been raised by one group of the club against the manager. 23. Select the option that expresses the given sentence in active voice. The award was presented to her by the director. (a) She presented the award to the director. (b) The director presented the award to her. (c) The award presented to her by the director. (d) She was presenting the award to the director. SSC CGL Mains 26/10/2023 (Shift-I) Ans. (b) : The above sentence is passive voice of past indefinite fense. He fetive voice structure will be P.V. - Object + was/were + VIII + by + subject A.V. - Subject + VII + Object. Correct Sentence The director presented the award to her. 24. Select the option that expresses the given sentence in passive voice. Access denied. (a) Let it be known that access will be denied. (b) Access is being denied. (c) Access has been denied. (d) Let the access be denied. SSC CGL Mains 26/10/2023 (Shift-I) Ans. (d) : The above sentence is Active Voice of Imperative sentence. Its passive structure will be A.V. - Object + V1 P.V. - Let + Object + be + VIII Correct Sentence Let the access be denied. 25. Select the option that expresses the given sentence in active voice. The first prize will be won by Williams in the singing competition. (a) Williams had won the first prize in the singing competition. (b) Williams wins the first prize in the singing competition. (c) Williams will be winning the first prize in the singing competition. (d) Williams will win the first prize in the singing competition. SSC Selection Posts XI-30/06/2023 (Shift-IV) Ans. (d) : The above sentence is Passive voice of future Indefinite tense. Its active voice structure will beP.V.- Object + will + be + VIII + by + Sub. + Others. A.V.- Subject + Will + V1 + Object. Correct SentenceWilliam will win the first prize in the singing competition.

294

YCT

26.

Select the option that expresses the given sentence in active voice. The newspaper was read by me. (a) I read the newspaper (b) I will read the newspaper (c) I had read the newspaper (d) I was reading the newspaper SSC CGL (Tier-I) 24/07/2023 (Shift-III) Ans. (a) : The above given sentence is in passive voice of 'Past Indefinite tense'. Its Active Voice structure will beP.V.- Obj. + was/were + v3 + by + sub. A.V.- Sub. + V2 + Obj. Correct SentenceI read the newspaper. 27. Select the option that expresses the given sentence in passive voice. The students must complete the assignment by Friday. (a) The students will have completed the assignment by Friday. (b) The assignment will be completed by the students by Friday. (c) The assignment must be complete by Friday by the students. (d) The assignment must be completed by the students by Friday. SSC CHSL (Tier-I) 17/08/2023 (Shift-II) Ans. (d) : Rules to change modal (must) into passive voice is – Active voice – Subject + Modal (must) + v1 + object + other words Passive voice – Object + Modal (must) + be + v3 + by + subject + other words Correct Sentence The assignment must be completed by the students by friday. 28. Select the option that correctly expresses the following sentence in active voice. This was talked about in the meeting by the officers. (a) The officers were talking about in the meeting. (b) The officers talked about this in the meeting. (c) The officers in the meeting had talked about it. (d) The officers talk about this in the meeting. SSC CHSL (Tier-I) 17/08/2023 (Shift-II) Ans. (b) : The sentence is in passive form of past indefinite tense, for changing the given sentence in active voice, below mentioned rules are to be used– Passive voice – object + was/were + v3 + by + subject Active voice – Subject + v2 + objects Correct Active form – The officers talked about this in the meeting 29. Select the option that expresses the given sentence in passive voice. The boy will bring the coffee. Active / Passive Voice

(a) The coffee must have been brought by the boy. (b) The coffee will have been brought by the boy. (c) The coffee would have been brought by the boy. (d) The coffee will be brought by the boy. SSC CHSL (Tier-I) 10/08/2023 (Shift-I) Ans. (d) : To convert the given active voice of future simple Tense into passive voice, The rules to be followed are– Active voice – Subject + will/shall + V1 + object. Passive voice – object + will/shall + be + V3 + by + subject. Correct passive form – The coffee will be brought by the boy. 30. Select the option that expresses the given sentence in active voice. The dog was run over by a truck. (a) A truck had run over the dog. (b) A truck runs over the dog. (c) A truck ran over the dog. (d) A truck has run over the dog. SSC CHSL (Tier-I) 10/08/2023 (Shift-I) Ans. (c) : The given sentence is in passive voice of past indefinite tense. It can be expressed in Active voice with the help of following rules/structures – Passive voice – object + was/were +V3 + by + Subject Active voice – subject + V2 +object Correct Active form A truck ran over the dog. 31. Select the option that expresses the following sentence in active voice. The tickets had already been booked by my friend. (a) The tickets had already booked my friend. (b) My friend has already booked the tickets. (c) My friend already booked the tickets. (d) My friend had already booked the tickets. SSC CHSL (Tier-I) 09/08/2023 (Shift-III) Ans. (d) : Rules to be used to change the given sentence from passive voice to active voice is– Passive voice – Object + had been + V3 + by + Subject Active voice – Subject + had + V3 + Object Correct Active form My friend had already booked the tickets 32. Select the option that expresses the given sentence in active voice. Let the book be read by me. (a) Let me read the book. (b) Let me had read the book. (c) Let me please read the book. (d) Let me have read the book. SSC CHSL (Tier-I) 09/08/2023 (Shift-III) Ans. (a) : following rules are to be followed to Convert the given sentence into active voicePassive voice – Let + object + be + V3 + by + Subject Active voice – Let + Subject + V1 + object Correct Active form– Let me read the book.

295

YCT

33.

Select the option that correctly expresses the following sentence in passive voice. The child will recite the poem. (a) The poem will be recited by the child. (b) The poem will recite by the child. (c) By child poem will be recited. (d) The poem is recited by the child. SSC CHSL (Tier-I) 02/08/2023 (Shift-I) Ans. (a) : The above given sentence is in active voice of simple future tense. Its passive voice will be made from the following structure – A.V. – sub. + will/shall + VI + other words.' P.V. – obj: + will/shall + be + VIII + by + sub.' Correct Passive voice – The poem will be recited by the child .' 34. Select the option that expresses the given sentence in active voice. French is taught to us by Miss Glenn. (a) Miss Glenn is teaching us French. (b) Miss Glenn teaches us French. (c) Miss Glenn teach us French. (d) Miss Glenn taught us French. SSC CHSL (Tier-I) 02/08/2023 (Shift-I) Ans. (b) : The above given sentence is in passive voice of simple present tense. Its active voice will be made from the following rules – A.V. – sub. + v1 + s/es + obj. P.V. – obj. + is/am/are +v3 + others + by + sub. Correct active voice – 'Miss Glenn teaches us French'. 35. Select the option that expresses the given sentence in active voice. The snake was killed by Shyam. (a) Shyam kills the snake. (b) Shyam had killed the snake. (c) Shyam killed the snake. (d) Shyam has killed the snake. SSC CHSL (Tier-I) 14/08/2023 (Shift-IV) Ans. (c) : The given sentence is passive voice of Past Indefinite tense. It will be changed into active voice, using following rules– Passive voice – object + was/were + V3 + by + Subject Active voice – Subject + V2 + object Correct Active VoiceShyam killed the snake. 36. Select the option that expresses the given sentence in active voice. Let the door be shut. (a) The door should be shut. (b) You are requested to shut the door. (c) Let you shout the door. (d) Shut the door. SSC CHSL (Tier-I) 14/08/2023 (Shift-IV) Ans. (d) : The given sentence after being converted into Active voice will get the form mentioned in option (b). The rules of conversion of imperative sentence– Passive voice – Let + object + be + V3 Active voice – verb1 + object. Correct active form – Shut the door. Active / Passive Voice

37.

Select the most appropriate option that expresses the given sentence in passive voice. The director hired the new employee after the interview. (a) The new employee hired by the director after the interview. (b) The new employee was hired by the director after the interview. (c) The new employee will be hired by the director after the interview. (d) The new employee is being hired by the director after the interview. SSC CHSL (Tier-I) 08/08/2023 (Shift-II) Ans. (b) : The above sentence is in active voice of past indefinite tense. Its passive voice structure will be – A.V. – Sub.+ VII + obj. + other words. P.V. – obj. + was/were + VIII + by + sub. + other words. Correct passive voice – The new employee was hired by the director after the interview. 38. Select the option that expresses the given sentence in passive voice. The speech of Dr. APJ Abdul Kalam influences young minds. (a) Young minds are influenced by the speech of Dr. APJ Abdul Kalam. (b) Young minds get influenced by Dr. APJ Abdul Kalam (c) Young minds were influenced by the speech of Dr. APJ Abdul Kalam. (d) The speech of Dr. APJ Adul Kalam is very influencing. SSC CHSL (Tier-I) 08/08/2023 (Shift-II) Ans. (a) : The above sentence is in active voice of simple present tense. Its passive voice structure will be– A.V. – Sub. + VI + s/es +obj. + other words. P.V. – obj. + is/are + VIII + by + sub. + other words. Correct passive voice – Young minds are influenced by the speech of Dr. APJ Abdul kalam. 39. Select the option that expresses the given sentence in passive voice. You should apply for the job before the deadline. (a) The deadline should not be crossed for the job application. (b) The job should be applied for before the deadline. (c) The job could be applied for after the deadline. (d) You should be applying for the job before the deadline is. SSC CHSL (Tier-I) 04/08/2023 (Shift-III) Ans. (b) : The above sentence is in active voice of modal form. its passive voice structure will be – A.V. – Sub. + would/should + VI + obj. + other words. P.V. – obj. + would/should + be + VIII + sub. + other words Correct Passive voice – 'The job should be applied for before the deadline.'

296

YCT

40.

Select the option that expresses the given sentence in active voice. Was the painting stolen by the thief? (a) Was the thief not stealing the painting? (b) Did the thief steal the painting? (c) Was the painting being stolen by the thief? (d) Did not the thief steal the painting? SSC CHSL (Tier-I) 04/08/2023 (Shift-III) Ans. (b) : The above sentence is in passive voice of interrogative sentence of past indefinite tence. Its active voice structure will be – A.V. – Did + Sub. + V1 + obj. + ? P.V. – Was/were + obj. + VIII + by + Sub. + ? Correct Active voice – Did the thief steal the painting? 41. Select the option that expresses the following sentence in active voice. A play is being staged by our group this evening. (a) Our group stages a play this evening. (b) Our group staged a play this evening. (c) Our group is staging a play this evening. (d) A play staged our group this evening. SSC CHSL (Tier-I) 03/08/2023 (Shift-II) Ans. (c) : The above sentence is in passive voice of present continuous tense. Its active voice structure will be – A.V. Sub. + is/am/ are + v+ ing + obj. + others. P.V. – Obj. + is/am/are + being + VIII + by + sub. + others. Correct Active voice – 'Our group is staging a play this evening.' 42. Select the option that expresses the given sentence in active voice. Mathematical problems are never solved daily by him. (a) He solves never mathematical problems daily. (b) He has solved never mathematical problems daily. (c) He never solves mathematical problems daily. (d) He solve never mathematical problems daily. SSC CHSL (Tier-I) 03/08/2023 (Shift-II) Ans. (c) : The above sentence is in passive voice of present tense. Its active voice will be made from Indefinite the following structure – A.V. – 'Sub.+ never + VI + s/es + object + others.' P.V. – obj. + is/am/are + never + VIII + others + by + sub.' Correct Active voice – 'He never solves mathematical problems daily.' 43. Select the correct passive form of the given sentence. The child locked the shutters. (a) The shutters were locked by the child. (b) The shutters are locked by the child. (c) The shutters would be locked by the child. (d) The shutters had been locked by the child. SSC CHSL (Tier-I) 21/03/2023 (Shift-IV) Active / Passive Voice

Ans. (a) : In the above sentence the use of 'locked' makes it clear that the sentence is Active voice of Past Indefinit tense. Its Passive structure will beA.V – Sub + v2 + obj P.V – obj + was/were + v3 +by + Sub. Correct Passive Voice: The shutters were locked by the child. 44. Select the correct passive form of fthe given sentence. Susan writes recipes for the cookbook. (a) Recipes are write by Susan for the cookbook. (b) Recipes were written by Susan for the cookbook. (c) Recipes are written by Susan for the cookbook. (d) Recipes are being written by Susan for the cookbook. SSC CHSL (Tier-I) 21/03/2023 (Shift-II) Ans. (c) : The above sentence is Active Voice of Simple Present tense. Its Passive structure will beA.V – Sub. + V1 + s/es + obj. P.V – Obj. + is/am/are + v3 + by + sub. Correct Passive Voice Recipes are written by Susan for the cookbook. 45. The given sentence is in active voice. Change the voice of the sentence. Select the Correct Option from the sentence given in options Vasant has has delered the speech. (a) Vasant delivers the speech on time. (b) The speech was timely delivered by Vasant. (c) The speech was timely delivered by Vasant. (d) The speech has been delivered by Vasant. SSC CHSL (Tier-I) 20/03/2023 (Shift-I) Ans. (d) : The above sentence is Active Voice of Present Perfect Tense. Its Passive voice will beA.V.- Subject + has / have + v3 + object P.V.- Object + has / have + been + v3 + by + subject. 46. Select the correct passive form of the given sentence. Picasso painted Guernica in 1937. (a) Guernica was painting by Picasso in 1937. (b) Guernica was been painted by Picasso in 1937. (c) Guernica was paint by Picasso in 1937. (d) Guernica was painted by Picasso in 1937. SSC CHSL (Tier-I) 17/03/2023 (Shift-IV) Ans. (a) : In the above sentence, Sub+V2 makes it clear that the sentence is in Simple Past Tense. Its Passive Structure isA.V.- Sub+V2 + obj P.V.- Obj + was/were + V3 + by + sub 47. Select the correct passive form of the given sentence. The manager lost the letter. (a) The letter was loss by the manager. (b) The letter was loose by the manager. (c) The letter was lost by the manager. (d) The letter was lose by the manager. SSC CHSL (Tier-I) 17/03/2023 (Shift-III)

297

YCT

Ans. (c) : In the given sentence the use of 'Lost makes it clear that the sentence is in simple Past. Hence its Passive Voice structure will beA.V. - Sub +V2+obj P.V.- obj + was/were+V3+by+Sub. Correct Passive voice- The letter was lost by the manager. 48. Choose the option that is the correct passive form of the sentence. Shivam does not play football. (a) Football will not played by Shivam. (b) Football is not played by Shivam. (c) Football was not played by Shivam. (d) Football has not played by Shivam. SSC CHSL (Tier-I) 15/03/2023 (Shift-III) Ans. (b) : In the above sentence the use of 'does not play' (v1) makes it clear that the sentence is Active voice of Present Indefinite tense. Hence its Passive structure will beA.v- Subject + does/do + (not) + verb1 + object. P.v - object + is/am/are + (not) + verb3 + by + subject. Correct SentenceFootball is not played by shivam. 49. Select the correct passive form of the given sentence. The tigress was trailing the stag. (a) The stag had been trailed by the tigress. (b) The stag was trailed by the tigress. (c) The stag was trailing by the tigress. (d) The stag was being trailed by the tigress. SSC CHSL (Tier-I) 15/03/2023 (Shift-I) Ans. (d) : In the above sentence the use of 'was + trailing' makes it clear that the sentence is Active voice of Past Continuous Tense. Hence its Passive voice structure will beA.v- Subject + was/were +V1 + ing + object. P.v- object + was/were + being +verb3 + by + Subject. Correct sentenceThe Stag was being trailed by the tigress. 50. The given sentence is in active voice. Change the voice of the sentence. Select the correct option from the sentences given in options. The story explores the conflicting range of human emotions. (a) The conflicting range of human emotions had explored by the story. (b) The conflicting range of human emotions did explored by the story. (c) The conflicting range of human emotions is explored by the story. (d) The conflicting range of human emotions has explored by the story. SSC CHSL (Tier-I) 10/03/2023 (Shift-III) Ans. (c) : In the above sentence the use of "Explores" (verb1 + s/es) makes it clear that the sentence is in Present Indefinete tense. Hence its Passive Voice Structure will be. AV – Subject + Verb1 (s/es) + object PV – object + is/am/are + Verb3 + by + Subject. Correct Sentence – The conflicting range of human emotions is explored by the story. Active / Passive Voice

51.

Select the correct passive form of the given sentence. The maid washes the windows every week. (a) The windows are wash every week by the maid. (b) The windows were washed every week by the maid. (c) The windows are washed every week by the maid. (d) The windows was washed every week by the maid. SSC CHSL (Tier-I) 09/03/2023 (Shift-IV) Ans. (c) : In the above sentence the use of "washes" (verb1 + s/es) makes it clear that the sentence is in Present Indefinite tense. Hence its Passive structure will beA.V – Subject + verb1 + (s/se) + object + other words. P.V– Object + is/am/are + verb3 + other words + by + Subject Correct Sentence The windows are washed every week by the maid. 52. Choose the option that is the correct passive form of the sentence. The culture was losing its meaning. (a) The culture lost it's meaning (b) The meaning of the culture was lost (c) The meaning of the culture was being lost (d) The meaning of the culture will lost SSC CHSL (Tier-I) 09/02/2023 (Shift-II) Ans. (c) : In the above sentence the use of was + losing (verb+ing) makes it clear that the sentence is in Past continuous. Hence its Passive voice structure will beA.V– Subject + was/were + verb1 + ing + object P.V – Object + was/were + being + verb3 + (by + Subject) Correct Sentence : The meaning of the culture was being lost. 53. Select the correct active form of the given sentence. A car full of explosives was apprehended today by the forces. (a) The forces are apprehending a car full of explosives today. (b) A car full of explosive has apprehended the forces today. (c) The forces apprehended a car full of explosives today. (d) The forces have apprehended a car full of explosive today. SSC Stenographer (Grade C & D) 12.11.2021 Shift-I Ans. (c) : The above sentence is in Passive Voice of the past Indefinite tense. Its Active Voice structure will be– A.V. → Sub. + v2 + obj. + others. P.V. → Obj. + was/were + v3 + by + sub. + others. Cor. Active Voice The forces apprehended a car full of explosives today. 54. Select the correct passive voice of the given sentence. A good conclusion summarises the highlights of a speech.

298

YCT

(a) The good conclusion is summarized by the highlights of the speech (b) The highlights of a speech are summarized in a good conclusion. (c) A good conclusion of a speech is summarized by the highlights. (d) A speech highlight is summarized by a good conclusion. SSC Stenographer (Grade C & D) 12.11.2021 Shift-I Ans. (b) : The above sentence is in Active voice of the present Indefinite tense. Its passive structure will be– A.V. → Sub. + V1 + obj. + others. P.V. → Obj. + is/am/are + v3 + by + sub. + others. Cor. Passive voice The highlights of a speech are summarised in a good conclusion. 55. Select the correct active voice of the given sentence. A standing ovation was received by the storytellers who participated in the International Festival. (a) The storytellers who participated in the International Festival will be receiving a standing ovation. (b) The storytellers who participated in the International Festival had been receiving a standing ovation. (c) In the International Festival the participating storytellers were receiving a standing ovation. (d) The storytellers who participated in the International Festival received a standing ovation. SSC Stenographer (Grade C & D) 12.11.2021 Shift-I Ans. (d) : The above sentence is in passive voice of past Indefinite tense. Its Active voice structure will be– A.V. → Sub. + V2 + obj. + others. P.V. → Obj. + was/were + v3 + by + sub. + others. Cor. Active voice The storytellers who participated in the International Festival received a standing ovation. 56. Select the correct active form of the given sentence. The environment has been ruthlessly destroyed in this area by the growth of industry. (a) The growth of industry is ruthlessly destroying the environment in this area. (b) The growth of industry will ruthlessly destroy the environment in this area. (c) The growth of industry has ruthlessly destroyed the environment in this area. (d) The environment in this area has ruthlessly destroyed the growth of industry. SSC Stenographer (Grade C & D) 12.11.2021 Shift-I Ans. (c) : The above sentence is in Passive voice of the Assertive sentence of the present perfect tense. Its Active voice structure will be – A.V. → Sub. + has/have + V3 + obj. + others. P.V. → Obj. + has/have + been + v3 + by + sub. + others. Cor. Active voice The growth of industry has ruthlessly destroyed the environment in this area. Active / Passive Voice

57.

Select the correct passive voice of the given sentence. I am reviewing a book by Ruskin Bond. (a) A book by Ruskin Bond is being reviewed by me. (b) A book by Ruskin Bond would be reviewed by me. (c) A book by Ruskin Bond was to be reviewed by me. (d) A book by Ruskin Bond has been reviewed by me. SSC Stenographer (Grade C & D) 12.11.2021 Shift-I Ans. (a) : The above given sentence is in Active voice of the present continuous tense. Its passive voice structure will be– A.V. → Sub. + is/am/are + vIng + obj. + others. P.V. → Obj. + is/am/are + being + v3 + by + sub. + others. Cor. Passive voice A book by Ruskin Bond is being reviewed by me. 58. Select the correct active voice of the given sentence. A brave fight was put by the cancer patient and she finally won the battle. (a) The cancer patient put up a brave fight and she finally won the battle. (b) The cancer patient had put up a brave fight and she finally won the battle. (c) A brave fight was being put up by the cancer patient and she finally won the battle. (d) A brave fight had been put up by the cancer patient and she finally won the battle. SSC Stenographer (Grade C & D) 12.11.2021 Shift-I Ans. (a) : The above sentence is in Passive voice of past Indefinite tense. Its Active voice structure will be– A.V. → Sub. + v2 + obj. + others. P.V. → Obj. + was/were + v3 + by + sub. + others. Cor. Active voice The cancer patient put up a brave fight and she finally won the battle. 59. Select the option that expresses the given sentence in passive voice. Robots are performing delicate surgeries. (a) Delicate surgeries are performed by robots. (b) Delicate surgeries were being performed by robots. (c) Delicate surgeries have been performed by robots. (d) Delicate surgeries are being performed by robots. SSC Stenographer (Grade C & D) 12.11.2021 Shift-I Ans. (d) : The above sentence is in Active voice of present continuous tense. Its Passive voice structure will be – A.V.→ Sub. + is/am/are + vIng + obj. + others. P.V.→ Obj. + is/am/are + being +v3+ by + sub. + others. Cor. Passive VoiceDelicate surgeries are being performed by robots.

299

YCT

60.

Select the option that expresses the given sentence in active voice. When will the new laws be implemented by the government? (a) When would the government implement the new laws? (b) When will the government implement the new laws? (c) When has the government implemented the new laws? (d) When did the government implement the new laws? SSC Stenographer (Grade C & D) 12.11.2021 Shift-I Ans. (b) : The given sentence is in Passive voice of future Indefinite tense. Its Active voice structure will be – A.V.→ Sub. + will/shall + v1 + obj. + others. P.V.→ Obj. + will/shall + be + v3 + by + sub. + others. Cor. Active voice When will the government implement the new laws. 61. Select the most appropriate active form of the given sentence. Those who help themselves are helped by God. (a) God helps those who help themselves. (b) Help yourself and God will help you. (c) God is helping those who are helping others (d) If you help yourself, God will help you. SSC CGL (Tier-II) 08.08.2022 Shift-II Ans. (a) : The above sentence is in Passive Voice of the simple present tense. Its Active Voice structure will beA.V.– Subject + verb1 (s/es) + object + other words. P.V.– Object + is/am/are + verb3 + by + subject. Correct Active VoiceGod helps those who help themselves. 62. Select the most appropriate passive form of the given sentence. Please do not pluck any flowers from the temple compound. (a) You are requested not to pluck any flowers from the temple compound. (b) No flowers can be plucked from the temple compound. (c) How can any flowers to plucked from the temple compound ? (d) Let any flowers not to be plucked from the temple compound. SSC CGL (Tier-II) 08.08.2022 Shift-II Ans. (a) : In the above sentence the use of requested word 'Please' makes it clear that the given sentence is in Active Voice of Imprative sentence. Its Passive voice structure will be– A.V.– (Please) + Do not + Verb1 + object + Other words. P.V. – You are requested/adviced/ordered + (not) + to verb1 + object + other words. Correct Passive Voice– You are requested not to pluck any flowers from the temple compound. Active / Passive Voice

63.

Select the option that expresses the given sentence in active voice. The money shall have been withdrawn from the bank by tomorrow. (a) We are going to withdraw the money from the bank by tomorrow. (b) We shall withdraw the money from the bank by tomorrow. (c) We shall have withdrawn the money from the bank by tomorrow. (d) We shall be withdrawing the money from the bank by tomorrow. SSC CGL (Tier-II) 08.08.2022 Shift-II Ans. (c) : In the above sentence the use of (Shall have been + withdrawn (V3) makes it clear that the given sentence is in Passive voice of Future perfect tense. It Active voice structure will be – A.V.– Subject + will/shall + have + verb3 + object + other words. P.V.– Subject + will/shall + have + been + verb3 + (by + subject) + other words Correct Active VoiceWe shall have withdrawn the money from the bank by tomorrow. 64. Select the option that expresses the given sentence in active voice. Has Rahul been declared fit to play the next match ? (a) Did they declare Rahul fit to play the next match ? (b) Has Rahul declared the next match fit to play? (c) Have they declared Rahul fit to play the next match ? (d) Are they declaring Rahul fit to play the next match ? SSC CGL (Tier-II) 08.08.2022 Shift-II Ans. (c) : In the above sentence the use of (has + Object + been + V3) makes it clear that the sentence is Passive voice of Interrogative sentence (Yes/No answer type) of present perfect tense. Its Active structure will be– A.V.– Has/Have + subject + verb3 + object + other words + ? P.V.– Has/Have + object + been + verb3 + other words. (by + subject) + ? Correct Active VoiceHave they declared Rahul fit to play the next match? 65. Select the most appropriate passive form of the given sentence. Granny had given Uncle Ken a good lecture on how to be a responsible adult. (a) Uncle Ken was giving a good lecture to Granny on how to be a responsible adult. (b) Granny was given a good lecture by Uncle Ken on how to be responsible adult. (c) Uncle Ken was being given a good lecture by Grany on how to be a responsible (d) Uncle Ken had been given a good lecture by Granny on how to be a responsible adult. SSC CGL (Tier-II) 08.08.2022 Shift-II

300

YCT

Ans. (d) : In the above sentence the use of (had + V3) makes it clear that the given sentence is in Active voice of past perfect tense. Its Passive structure will be– A.V.– Subject + had + verb3 + object + other words. P.V.– Object + had been + verb3 + by + subject + other words. Correct Passive VoiceUncle Ken had been given a good lecture by Granny on how to be a responsible adult. 66. Select the most appropriate active form of the given sentence. Rani's car is twelve years old but it has not been used much. (a) Rani's car is twelve years old but she isn't using it much. (b) Rani's car is twelve years old but she hasn't used it much (c) Rani's car is twelve years old but she didn't use it much (d) Rani's car is twelve years old but it hasn't used her much. SSC CGL (Tier-II) 08.08.2022 Shift-II Ans. (b) : The above sentence in which two clauses are given the voice of the clause changes in which the object is given. Hence there will be change in the second clause of the above sentence. In which "has been used (V3)" given it is clear that the sentence is in passive voice of Present Perfect tense. So its Active voice structure will be– AV – Subject + has/have + (not) + verb3 + object + other words. P.V. – Object + has/have + (not) + been + verb3 + (by + subject) Correct Active VoiceRani's car is twelve years old but she hasn't used it much. 67. Select the correct passive form of the given sentence. The family was watching Ramayana on the TV. (a) Ramayan was watching the family on the TV. (b) Ramayan is being watched by the family on the TV. (c) Ramayan was being watched by the family on the TV. (d) Ramayan has been watched by the family on the TV. SSC Stenographer (Grade C & D) 11.11.2021 Shift-II Ans. (c) : The above sentence is in Active voice of Present continuous tense. Its Passive voice structure will be– A.V.- Sub. + Was/Were + Ving + Obj. + other P.V.- Obj. + was/were + being + verb3 + by + Sub. other words. Correct Passive Voice– Ramayan was being watched by the family on the TV. 68. Select the correct passive form of the given sentence. The country is witnessing a severe locust outbreak in Rajasthan and Gujarat. Active / Passive Voice

(a) A severe locust outbreak in Rajasthan and Gujarat has been witnessed by the country. (b) A severe locust outbreak in the country is being witnessed by Rajasthan and Gujarat. (c) A severe locust outbreak in Rajasthan and Gujarat is witnessing the country. (d) A severe locust outbreak in Rajasthan and Gujarat is being witnessed by the country. SSC Stenographer (Grade C & D) 11.11.2021 Shift-II Ans. (d) : The above sentence is in Active voice of Present continuous tense. Its Passive voice structure will be– A.V. - Sub + is/am/are + Ving + Obj + Other words. P.V. - Objec + is/am/are + being + VIII + Subjct + by + Other words. Correct Passive Voice– A severe locust outbreak in Rajasthan and Gujarat is being witnessed by the country. 69. Select the option that expresses the given sentence in passive voice. His employer trusts him completely. (a) He was completely trusted by his employer (b) His employer was completely trusted by him (c) He has been completely trusted by his employer (d) He is completely trusted by his employer SSC Stenographer (Grade C & D) 11.11.2021 Shift-I Ans. (d) : The above sentence is in Active voice of Assertive sentence of Present Indefinite tense. Its Passive structure will be– Active voice - Sub + V1 + object + Other words Passive voice - Obj. + is/am/are + V3 + by + subject + Other words Currect Passive VoiceHe is completely trusted by his employer. 70. Select the option that expresses the given sentenc in active voice. Let a strict eye be kept on romour mongers. (a) Let rumour mongers keep a strict eye. (b) Rumour mongers should keep a strict eye. (c) Keep a strict eye on rumour mongers. (d) You should have kept a strict eye on rumour mongers. SSC Stenographer (Grade C & D) 11.11.2021 Shift-I Ans. (c) : The above sentence is in Passive voice of imperative sentence. Its active structure will be– • Active voice - Verb (V1 Form) + Object Passive voice - Let + Object + be + V3 + Other words Currect Active VoiceKeep a strict eye on rumour mongers. 71. Select the correct passive form of the given sentence. Anurag has given me this book to read. (a) I have given this book to Anurag to read. (b) This book has given me Anurag to read. (c) This book has been given by me to Anurag to read. (d) I have been given this book by Anurag to read. SSC Stenographer (Grade C & D) 12.11.2021 Shift-II

301

YCT

Ans. (d) : The given active voice of the above sentence is in present perfect tense. Its Passive Voice structure will beActive voice– Subject + has/have + V3 + object. Passive voice– Object + have/has + been + V3 + by + Subject. The correct passive voice will be– 'I have been given this book by Anurag to read.' 72. Select the correct passive form of the given sentence. Authorities have stepped up efforts to control the invasion of locusts in Rajasthan and Gujarat. (a) Efforts to control the invasion of locusts in Rajasthan and Gujarat are being stepped up by the authorities. (b) Efforts to control the invasion of locusts in Rajasthan and Gujarat were stepped up by the authorities. (c) Efforts to control the invasion of locusts in Rajasthan and Gujarat were being stepped up by the authorities. (d) Efforts to control the invasion of locusts in Rajasthan and Gujarat have been stepped up by the authorities. SSC Stenographer (Grade C & D) 15.11.2021 Shift-II Ans. (d) : In the above sentence the use of "have + V3" makes it clear that the sentence is in Active voice of Present Perfect tense. Its Passive voice structure will be– A. v. → Sub. + has/have + V3 + obj. + others. P. v. → Obj. + has/have + been + V3 + by + sub. + others. Cor. Passive voice – Efforts to control the invasion of locusts in Rajasthan and Gujarat have been stepped up by the authorities. 73. Select the option that expresses the given sentence in passive voice. Dalal Street houses the headquarters of Bombay Stock Exchange. (a) The headquarters of Bombay Stock Exchange are housed on Dalal Street. (b) Dalal Street is housed the headquarters of Bombay Stock Exchange. (c) The headquarters of Bombay Stock Exchange is housed by Dalal Street. (d) The headquarters of Bombay Stock Exchange is being housed on Dalal Street. SSC Stenographer (Grade C & D) 15.11.2021 Shift-II Ans. (a) : The above sentence is in Active voice of Assertive sentence of simple present tense. Its Passive voice structure will be– A. v.→ Sub. + VI + obj. + others P. v. → Obj. + is/am/are + V3 + by + sub. + others. Cor. Passive voice:The headquarters of Bombay Stock Exchange are housed on Dalal Street. 74. Select the option that expresses the given sentence in active voice. The fifty thousand mark has been crossed by Sensex in the last few days. Active / Passive Voice

(a) The last few days have crossed the fifty thousand marks in Sensex (b) The fifty thousand marks has crossed Sensex in the last few days. (c) Sensex have crossed the fifty thousand mark in the last few days. (d) Sensex has been crossed in the last few days by the fifty thousand mark. SSC Stenographer (Grade C & D) 15.11.2021 Shift-II Ans. (c) : The above sentence is in Passive voice of Present Perfect tense. Its Active voice structure will be– A. v. → Sub. + has/have + V3 + obj. + others. P. v. → Obj. + has/have + been + V3 + by + sub. + others. Correct Active voice– Sensex have crossed the fifty thousand mark in the last few days. 75. Select the correct active form of the given sentence. How much was paid by you for this watch? (a) How much were you paid for this watch? (b) How much have you paid for this watch? (c) How much did you pay for this watch? (d) How much are you paying for this watch? SSC Stenographer (Grade C & D) 15.11.2021 Shift-II Ans. (c) : The above sentence is in Passive voice of Interrogative sentence of past indefinite tense. Its Active voice structure will be– A.v. → How much + did + Sub. + VI + obj + others? P.v. → How much + was/were + Obj. + V3. + by + sub. + others? Correct Active VoiceHow much did you pay for this watch? 76. Select the correct active voice of the given sentence. What should be done with my hands when I speak in public? (a) What should be done when I speak in public with my hands? (b) What shall I do when I speak with my hands in public? (c) What should I do with my hands when I speak in public? (d) What should I do when I speak with my hands in public? SSC Stenographer (Grade C & D) 15.11.2021 Shift-I Ans. (c) : The above sentence is in Passive voice of modal verb of Interrogative sentence. Its Active voice structure will be– A.V. – Wh-word + modal verb + sub + V1 + object + Other words + ? P.V. – Wh-word + modal verb + sub + be + V3 + by + object + ? Thus, the correct answer is option (c). 77. Select the option that expresses the given sentence in passive voice. Enjoy instant benefits by becoming a member. (a) Let instant benefits be enjoyed by becoming a member. (b) Instant benefits for members can be enjoyed.

302

YCT

(c) Instant benefits will be enjoyed if you become a member. (d) Instant members can enjoy benefits. SSC Stenographer (Grade C & D) 15.11.2021 Shift-I Ans. (a) : The above sentence is in Active voice of Imperative sentence. Its Passive voice structure will be– A.V.–V1 + object + other words. P.V. – Let + object + be + V3 + other words. Thus, the correct answer is option (a). 78. Select the correct passive voice of the given sentence. The doctor told me to start taking medicines prescribed by him. (a) The doctor had told me to start taking medicines prescribed by him. (b) I was told by the doctor to start taking medicines prescribed by him. (c) The doctor was told to start taking medicines prescribed by me daily. (d) I had been told by the doctor to start taking medicines prescribed by him. SSC Stenographer (Grade C & D) 15.11.2021 Shift-I Ans. (b) : The above sentence is in Active voice of Assertive sentence of Past Indefinite tense. Its Passive voice structure will be– A.V. – Sub. + V2 + object + Other words P.V. – Object + was/were + V3 + by + Sub. + Other words Thus, the correct answer is option (b). 79. Select the correct active form of the given sentence. The food can now be removed from the dining table. (a) We will now remove the food from the dining table. (b) We should now remove the food from the dining table. (c) We might now remove the food from the dining table. (d) We can now remove the food from the dining table. SSC Stenographer (Grade C & D) 15.11.2021 Shift-I Ans. (d) : The given sentence is in Passive voice of model sentence. Its Active voice structure will be– A.V.–Sub + modal verb + VI + object P.V.–Object + modal verb + be + V3 + (by + sub) Thus, the correct answer is option (d). 80. Select the option that expresses the given sentence in active voice. The utensils have been cleaned. (a) Someone was cleaning the utensils. (b) Someone has cleaned the utensils. (c) Someone cleaned the utensils. (d) Someone is cleaning the utensils. SSC CHSL 30.05.2022 Shift-I Ans. (b) : In the above sentence the use of (have been + V3) makes it clear that the given sentence is in Passive voice of Present Perfect tense. Its Active structure will be– Active / Passive Voice

A.V.– Subject + has/have + verb3 + object P.V.– Object + has/have + been + verb3 + (by + subject) Correct Active Voice– Someone has cleaned the utensils. 81. Select the option that expresses the given sentence in active voice. The car was being washed by me. (a) I was washing the car (b) I washed the car (c) I had been washing the car (d) I did the washing car SSC CHSL 30.05.2022 Shift-I Ans. (a) : In the above sentence the use of "was being + V3" makes it clear that the given sentence is in Passive voice of Past continuous tense. Its Active voice structure will be– A.V. – Subject + was/were + verb + ing + object + other words. P.V. – Object + was/were + being + verb3 + by + subject (objective case) Correct Active VoiceI was washing the car. 82. Select the option that expresses the given sentence in active voice. All the electric poles in the suburbs were destroyed by the cyclone last year. (a) The cyclone destroyed all the electric poles in the suburbs last year. (b) The cyclone destroyed all the suburbs by the electric poles last year. (c) The cyclone was destroyed by the all the electric poles in the suburbs last year. (d) The cyclone was being destroyed with all the electric poles in the suburbs last year. SSC CHSL 30.05.2022 Shift-I Ans. (a) : In the above sentence the use of "were + V3" makes it clear that the given sentence is in Passive voice of Past Indefinite tense. Its Active voice structure will be – A.V. – Subject + verb2 + object + other words. P.V.– Object + was/were + verb3 + by + subject + other words. Correct Active VoiceThe cyclone distroyed all the electric poles in the suburbs last year. 83. Select the option that expresses the given sentence in active voice. The flyer for the international symposium is being sent by students to institutes all over the country. (a) The flyer for the international symposium is sent by students to institutes all over the country. (b) Students are sending the flyer for the international symposium to institutes all over the country. (c) The flyer is sending the students to institutes all over the country for the international symposium. (d) Students are being sent the flyer for the international symposium to institutes all over the country. SSC CHSL 31.05.2022 Shift-III

303

YCT

Ans. (b) : In the given sentence the use of "is being + V3" makes it clear that the given sentence is in Passive voice of Present continuous tense. Its Active voice structure will be– A.V. – Subject + is/am/are + verb + ing + object + (other words) P.V. – Object + is/am/are + being + verb3 + by + subject + (other word), 84. Select the option that expresses the given sentence in passive voice. Alfred's doctor treated Rohan for his fever. (a) Rohan was treats by Alfred's doctor for his fever. (b) Rohan will be treated by Alfred's doctor for his fever. (c) Rohan has been treated by Alfred's doctor for his fever. (d) Rohan was treated by Alfred's doctor for his fever. SSC CHSL 31.05.2022 Shift-III Ans. (d) : In the above sentence the use of treated (V2) makes it clear that the given sentence is in Active voice of Past Indefinite tense. Its Passive voice structure will be– A.V. – Subject + verb2 + object + other words P.V.– Object + was/were + verb3 + by + subject + other words. Correct Passive VoiceRohan was treated by Alfred's doctor for his fever. 85. Select the option that expresses the given sentence in passive voice. You have lost your note book. (a) Your note book got lost. (b) You note book is lost. (c) Your note book has been lost. (d) Your not book have been lost. SSC CHSL 26.05.2022 Shift-I Ans. (c) : In the above sentence the use of "have V3" makes it clear that the given sentence is in Active voice of Present Perfect tense. Its Passive voice structure will be– A.V.– Subject + has/have + verb3 + object P.V.– Object + has/have+been + verb3 + (by + subject) Correct Passive Voice– Your note book has been lost. 86. Select the option that expresses the given sentence in passive voice. Siddharth is floating a grand plan by next week. (a) Next week is keeping afloat by Siddharth’s grand plan. (b) Siddharth is being floated by a grand plan next week. (c) A grand plan is being floating by Siddharth next week. (d) A grand plan is being floated by Siddharth next week. SSC CHSL 26.05.2022 Shift-I Active / Passive Voice

Ans. (d) : In the above sentence the use of "Is floating (V1 + ing)" makes it clear that the given sentence is in Active voice of Present continuous tense. Its Passive structure will be– A.V. – Subject + is/am/are + verb1 + ing + object P.V.– Object + is/am/are + being + verb3 + by + subject Correct Passive Voice– A grand plan is being floated by Siddharth next week. 87. Select the option that expresses the given sentence in active voice. The incident was narrated by somebody to Naresh. (a) Somebody narrated the incident to Naresh. (b) Somebody was about to narrate the incident to Naresh. (c) Somebody will narrate the incident to Naresh. (d) Somebody was narrating the incident to Naresh. SSC CHSL 26.05.2022 Shift-I Ans. (a) : In the above sentence the use of 'was + V3" makes it clear that the given sentence is in Passive voice of Past Indefinite tense. Its Active voice structure will be– P.V.– Object + was/were + verb3 + by + subject + otherword. A.V.– Subjet + verb2 + object + otherwords. Correct Active Voice– Somebody narrated the incident to Naresh. 88. Select the option that expresses the given sentence in passive voice. The government presented awards to eminent people. (a) Eminent people were being presented awards by the government. (b) Eminent people were presented awards by the government. (c) Eminent people are being presented awards by the government. (d) Eminent people are presented awards by the government. SSC CHSL 26.05.2022 Shift-III Ans. (b) : The above sentence is in Active voice of past Indefinite tense. Its Passive voice structure will be– A.V.– Subject + verb2 + object P.V.– Object + was/were + verb3 + by + subject. Correct Passive Voice– Eminent people were presented awards by the government. 89. Select the option that expresses the given sentence in active voice. The new bride is going to be admired by everyone in the family. (a) Everybody in the family was going to admire the new bride. (b) Everybody in the family is going to admire the new bride. (c) Everybody in the family has been going to admire the new bride. (d) Everybody in the family was admiring the new bride. SSC CHSL 26.05.2022 Shift-III

304

YCT

Ans. (b) : In the above sentence the use of "To be admired" makes it clear that the given sentence is in Passive voice of Infinitive. Its Active voice structure will be– P.V.– Object (Phrase) + to + be + Verb3 + subject (Phrase) A.V. – Subject (Phrase) + to + verb1 + object (Phrase) Correct Active Voice– Everybody in the family is going to admire the new bride. 90. Select the option that expresses the given sentence in active voice. Admittance was refused to him by the airport security. (a) The airport security refused him admittance. (b) The airport security had refused him admittance . (c) The airport security has refused him admittance. (d) The airport security is refusing him admittance. SSC CHSL 26.05.2022 Shift-III Ans. (a) : In the above sentence the use of "was + V3" makes it clear that the given sentence is in Passive voice of Past Indefinite tense. Its Active voice structure will be– P.V.– Object + was + verb3 + by + subject A.V.– Subject + verb2 + object Correct Active Voice– The airport security refused him admittance. 91. Select the option that expresses the given sentence in passive voice. You never hear of a happy billionaire. (a) A happy billionaire never heard of. (b) A happy billionaire is never heard by you. (c) No one hears a happy billionaire. (d) A happy billionaire is never heard of SSC CHSL 25.05.2022 Shift-I Ans. (d) : In the above sentence the use of 'Hear' (V1) makes it clear that the give sentence is in Active voice of Present Indefinite tense. Its Passive voice structure will be– A.V.– Subject + verb1 + object P.V. – Object + is/am/are+verb3 + by + subject Correct Passive Voice– A happy billionaire is never heard of. 92. Select the option that expresses the given sentence in passive voice. The most recent studies on the human body are likely to alter the face of health sciences. (a) The face of health sciences is likely to be altered by the most recent studies on the human body. (b) The face of health sciences will be altering the most recent studies on the human body. (c) The most face of health sciences is likely to alter the recent studies on the human body. (d) The most recent studies on the human body are likely to be altered by the face of health sciences. SSC CHSL 25.05.2022 Shift-I Active / Passive Voice

Ans. (a) : In the above sentence the use of 'To alter" makes it clear that the given sentence is in Active voice of Infinitive. Its Passive voice structure will be– P.V.- Object (Phrase) + to + be + verb3 + by + subject (phrase) A.V.- Sub.( Phrase) + to + V1 + Obj. (Phrase) Correct Passive Voice– The face of health sciences is likely to be altered by the most recent studies on the human body. 93. Select the option that expresses the given sentence in Passive voice. Tell her to finish the work. (a) The work is to be finished by her. (b) To finish the work be told to her. (c) Let she finish the work. (d) Let her be told to finish the work. SSC CHSL 27.05.2022 Shift-II Ans. (d) : The above sentence is started form verb 'Tell (VI)' and it makes clear that the sentence is in Active voice of Imperative sentence. Its Passive structure will be– A.V – Verb1 + Object + other words P.V – Let + Object + be + Verb3 + other words Correct Passive VoiceLet her be told to finish the work. 94. Select the option that expresses the given sentence in passive voice. They will meet Freddy outside the airport. (a) Freddy will meet them outside the airport. (b) They would be met by Freddy outside the airport. (c) Freddy would have been met outside the airport. (d) Freddy will be met by them outside the airport. SSC CHSL 27.05.2022 Shift-II Ans. (d) : In the above sentence the use of verb 'will meet' (will + VI) makes it clear that the given sentence is in Active voice of Future Indefinite Tense. Its Passive voice structure will be– A.V – Subject + will/shall + Verb1 + Object + other words. P.V – Object + will/shall + be + verb3 + by + subject (objective form) + other words. Correct passive Voice– Freddy will be met by them outside the airport. 95. Select the option that expresses the given sentence in active voice. Four grams of sodium chloride was added to the mixture by the research team. (a) The mixture added four grams of sodium chloride to the research team. (b) The research team added four grams of sodium chloride to the mixture. (c) The research team added the mixture to four grams of sodium chloride. (d) Four grams of sodium chloride added the mixture to the research team. SSC CHSL 27.05.2022 Shift-III

305

YCT

Ans. (b) : In the above sentence the use of 'was added' (was + V3) makes it clear that the given sentence is Passive voice of Past Indefinite tense. Its Active voice structure will be– P.V. – Object + was/were + verb3 + by + subject + (other words) A.V. – Subject + verb2 + object + (other words) Note- The use of 'other words' used in just after verb or in the end of the sentence. Correct Active VoiceThe research team added four grams of sodium chloride to the mixture. 96. Select the option that expresses the given sentence in passive voice. My father teaches students. (a) Students are being taught by my father. (b) Students are taught by my father. (c) A student is taught by my father. (d) My father is teaching students. SSC CHSL 27.05.2022 Shift-III Ans. (b) : In the above sentence the use of "teaches' (VI + s/es) makes it clear that the given sentence is in Active voice of Present Indefinite tense. Its Passive voice structure will be– A.V.– Subject + Verb1 + Object P.V.– Object + is/am/are + verb3 + by + Subject Correct Passive Voice Students are taught by my father. 97. Select the option that expresses the given sentence in active voice. Pygmalion was written by GB Shaw. (a) GB Shaw was writing Pygmalion. (b) Pygmalion wrote GB Shaw. (c) GB Shaw wrote Pygmalion. (d) GB Shaw was written by Pygmalion. SSC CHSL 26.05.2022 Shift-II Ans. (c) : In the above sentence the use of 'was + written (V3)' makes it clear that the given sentence is Passive voice of Past Indefinite tense. Its Active voice structure will be– P.V.– Object + was/were + verb3 + by + subject A.V.– Subject + Verb2 + object Correct Active Voice– GB Shaw wrote Pygmalion. 98. Select the correct active voice of the given sentence. The watchman has been bitten by a stray dog. (a) A stray dog bit the watchman. (b) A stray dog is biting the watchman. (c) A watchman has bitten the stray dog. (d) A stray dog has bitten the watchman. SSC CGL (Tier-I) 19/04/2022 Shift-III Ans. (d) : The above sentence is in Passive voice of Assertive Form of Present Perfect Tense. Its Active voice structure will be– A.V. - Subject + has/have + V3 + object + others P.V. - Obj. + has/have + been + V3 + by + Sub + others Correct Active voiceA Stray dog has bitten the watchman. Active / Passive Voice

99.

Select the option that expresses the given sentence in active voice. I have been informed of their decision. (a) They are informing me of their decision. (b) They had informed me of their decision. (c) They informed me of their decision. (d) They have informed me of their decision. SSC CGL (Tier-I) 19/04/2022 Shift-III Ans. (d) : The above sentence is in Passive voice of Assertive form of Present Perfect tense. Its Active voice structure will be– A.V.- Sub + has/have + V3 + object + others. P.V.- Obj. + has/have + been + V3 + by + Sub + other. Hence, Correct Active voiceThey have informed me of their decision. 100. Select the option that expresses the given sentence in active voice. I was taught by shyam how to drive. (a) Shyam has taught me how to drive. (b) I taught Shyam how to drive. (c) Shyam is teaching me how to drive. (d) Shyam taught me how to drive. SSC CGL (Tier-I) 21/04/2022 Shift-III Ans. (d) : In the above sentence the use of verb (was + taught) makes it clear that the given sentence is in Passive voice of Simple Past Tense. Its Active voice structure will be– A.V = Subject +v2 + object + other words P. V = Object + was/were + v3 + by + subject + other words. Correct Active Voice– Shyam taught me how to drive. 101. Select the correct passive voice of the given sentence. Hold the rope firmly. (a) Let the rope hold you firmly. (b) Let the rope firmly hold you. (c) Let the rope be held firmly. (d) Let the rope be hold firmly. SSC CGL (Tier-I) 20/04/2022 Shift-III Ans. (c) : The above sentence is in Active voice of Imperative sentence. Its Passive voice structure will be– A.V. → VI + object + Others words. P.V. → Let + object + be + v3 + other words. Correct Passive VoiceLet the rope be held firmly. 102. Select the option that expresses the given sentence in active voice. He is given proper guidance by his boss. (a) His boss is giving him proper guidance. (b) His boss gives him proper guidance. (c) He gives proper guidance to his boss. (d) His boss has been giving him proper guidance. SSC CGL (Tier-I) 20/04/2022 Shift-III

306

YCT

Ans. (b) : The above sentence is in Passive voice of Assertive form of Present Indefinite tense. Its Active voice structure will be– A.V.– Subject + V1(s/es) + object + others words. P.V.– Obj. of A.V. + is/am/are + V3 + by + Sub. of A.V. + others. The Correct Active Voice is His boss gives him proper guidance. 103. Select the option that expresses the given sentence in passive voice. Can you carry my bag upstairs? (a) Can my bag be carried upstairs by you? (b) Can your bag be carried upstairs by me? (c) Was my bag carried upstairs by you? (d) Could your bag be carried upstairs by me? SSC CGL (Tier-I) 19/04/2022 Shift-II Ans. (a) : The above sentence is Active voice of Interrogative form of modal verb. Its Passive voice structure will be– A.V – Modal verb + sub + VI + object+other words + ?] P.V- Modal verb + obj of A.V + be + V3 +by+ sub of A.V + ? 104. Select the option that expresses the given sentence in passive voice. The judge found the prisoner guilty. (a) The judge was found guilty by the prisoner. (b) The prisoner had been found guilty by the judge. (c) The prisoner has been found guilty by the judge. (d) The prisoner was found guilty by the judge. SSC CGL (Tier-I) 18/04/2022 Shift-I Ans. (d) : The above sentence is in Active voice of Past Indefinite tense. Its Passive voice structure will be– P.V.- [Obj (Sub.case) + was/were + V3 + by + sub. (obj. case)] A.V.- Sub. + V2 + Object. Thus, the correct answer is option (d). 105. Select the option that expresses the given sentence in passive voice. Who is creating this mess? (a) By whom is this mess being created? (b) Who has created this mess? (c) By whom this mess being created? (d) By whom has this mess been created? SSC CGL (Tier-I) 18/04/2022 Shift-I Ans. (a) : The above sentence is Active voice of Interrogative form of Present continuous tense. Its Passive voice structure will be– A.V.- Who + is/am/are + v1+ing + obj + ? P.V.- By whom + is/am/are + object + being + V3 + ? The correct answer is potion (a). 106. Select the correct active voice of the given sentence. The poems of great English poets are being translated into other languages. (a) They were translating the poems of great English poets into other languages. (b) They translated the poems of great English poets into other languages. Active / Passive Voice

(c) They are translating the poems of great English poets into other languages. (d) They have translated the poems of great English poets into other languages. SSC CGL (Tier-I) 12/04/2022 Shift-II Ans. (c) : In the above sentence the use of "are + being + V3" makes it clear that the given sentence is in Passive voice of Present Continuous tense. Its Active voice structure will be– A.V – Sub + is/am/are + V1 + ing + obj + others P.V. – Obj.+is/am/are + being + V3 + by + Sub + others. Correct Active VoiceThey are translating the poems of great English poets into other languages. 107. Select the option that expresses the given sentence in passive voice. He opened the door for his mother. (a) The door was open by him for his mother. (b) The door had been opened by him for his mother. (c) The door is opened by him for his mother. (d) The door was opened by him for his mother. SSC CGL (Tier-I) 12/04/2022 Shift-II Ans. (d) : In the above sentence the use of "Subject + V2" makes it clear that the given sentence is in Active voice of Past Indefinite tense. Its Passive voice structure will be– A.V. – Sub + V2 + obj. + others. P.V. – Obj. + was/were + V3 + by + sub. + others. Correct Passive VoiceThe door was opened by him for his mother. 108. Select the option that expresses the given sentence in passive voice. She gifted me a beautiful handbag. (a) I am being gifted a beautiful handbag by her. (b) I have been gifted a beautiful handbag by her. (c) I was gifted a beautiful handbag by her. (d) I am gifted a beautiful handbag by her. SSC CGL (Tier-I) 11/04/2022 Shift-II Ans. (c) : In the above sentence the use of 'gifted' (V2) makes it clear that the given sentence is in Active voice of simple Past tense. Its Passive voice structure will be– A.V → Subject + V2 + Object + Other words P.V → Object + was/were + V3 + by + subject + other words Correct Passive Voice– I was gifted a beautiful handbag by her. 109. Select the option that expresses the given sentence in passive voice. Give the child a nourishing diet. (a) The child is given a nourishing diet. (b) The child must have given a nourishing diet. (c) The child should be given a nourishing diet. (d) The child was given a nourishing diet. SSC CGL (Tier-I) 11/04/2022 Shift-II Ans. (c) : The above sentence is in Active voice of imperative sentence. Its Passive voice structure will be– A.V → V1+ object + other words P.V → Object + Should +be + V3 + other words Correct Passive Voice– The child should be given a nourishing diet.

307

YCT

110. Select the correct active voice from of the given sentence A cartoon film is being watched by Praveen. (a) Praveen had been watching cartoon film. (b) Praveen is watching a cartoon film. (c) Praveen would like to watch a cartoon film. (d) Praveen is watched a cartoon film. SSC CGL (Tier-I) 13/04/2022 Shift-III Ans. (b) : The given sentence is in Passive voice of Assertive form of Present Continuous tense. Its Active voice structure will be– A.V. Subject + is/am/are + v1+ing + object + others. P.V. Obj. + is/am/are + being +V3 + by + Sub. + others. Correct Passive Voice– Thus, the correct answer is option (b). 111. Select the option that expresses the given sentence in passive voice. They catch whale sharks commercially in some places around the world. (a) Whale sharks are caught commercially in some places around the world. (b) Whale sharks have been caught commercially in some places around the World. (c) Whale sharks were caught commercially in some places around the world. (d) Whale sharks are being caught commercially in some places around the world. SSC CGL (Tier-I) 13/04/2022 Shift-III Ans. (a) : The above given sentence is in Active voice of Assertive form of Present Indefinite tense. Its Passive voice structure will be– A.V.- Subject +VI (s/es) + object P.V.- Obj. + is/am/are + V3 + by + Sub. 112. Select the correct active voice form of the given sentence. All the prize winning books have been displayed on the tables. (a) We have displayed all the prize winning books on the tables. (b) We have to display all the prize winning books on the tables. (c) We will be displaying all the prize winning books on the tables. (d) We are displaying all the prize winning books on the tables. SSC CGL (Tier-I) 12/04/2022 Shift-III Ans. (a) : The above sentence is in Passive voice of Present Perfect tense. Its Active voice structure will beA.V.- Sub. + has/have + V3 + obj. P.V.- Obj. + has/have + been + V3 + subject Correct Passive Voice– We have displayed all the prize winning books on the table. 113. Select the correct active voice form of the given sentence. All the inmates were rescued from the building by the firemen. (a) The firemen have been rescuing all the inmates from the building. (b) The firemen are rescuing all the inmates from the building. Active / Passive Voice

(c) The firemen rescued all the inmates from the building. (d) The firemen have rescued all the inmates from the building. SSC CGL (Tier-I) 12/04/2022 Shift-III Ans. (c) : In the above sentence the use of verb "Were + V3" makes it clear that the given sentence is in Passive voice of Past Indefinite tense. Its Active Voice structure will be– A.V.- Subject + V2 + object P.A.- Subject + was/were + V3 + by + object Correct Active Voice– The firemen rescued all the inmates from the building. 114. Select the correct passive voice of the given sentence. She is making a beautiful beaded curtain. (a) A beautiful beaded curtain is made by her. (b) A beautiful beaded curtain was made by her. (c) A beautiful beaded curtain is being made by her. (d) A beautiful beaded curtain was being made by her. SSC CGL (Tier-I) 13/04/2022 Shift-II Ans. (c) : In the above sentence the use of verb 'is + making' (V1 + ing) makes it clear that the given sentence is in Active voice of Present Continuous tense. Its Passive voice structure will be– A.V. → Subject + is/am/are + V1 + ing + object P.V. → object (subjective case) + is/am/are + being + V3 + by + subject (objective case) Correct Passive Voice– A beautiful beaded curtain is being made by her. 115. Select the correct active voice of the given sentence. Rohan was pushed into the pool by karan. (a) Rohan pushed karan into the pool. (b) Karan was pushing Rohan into the pool. (c) Karan pushed Rohan into the pool. (d) Rohan had pushed Karan into the pool. SSC CGL (Tier-I) 13/04/2022 Shift-II Ans. (c) : In the above sentence the use of "was + pushed (V3)" makes it clear that the given sentence is in Passive voice of Past Indefinite tense. Its Active voice structure will be– A.V.- Subject + V2 + object P.V. - Object + was/were + V3 + by + subject Thus, the correct answer is option (c). 116. Select the correct passive voice of the given sentence. Preeti invited Ritu for a party. (a) Preeti has been invited for a party by Ritu. (b) Preeti was invited for a party by Ritu. (c) Ritu was invited for a party by Preeti. (d) Ritu is invited for a party by Preeti. SSC CGL (Tier-I) 13/04/2022 Shift-I

308

YCT

Ans. (c) : The above sentence is Active voice of Past Indefinite tense. Its Passive voice structure will be– 'Ritu was invited for a party by preeti'. Structure – A.V.– Sub. + V2 + object P.V. – Obj. + was/ were + v3 + by + sub. Hence, the correct option is (c) 117. Select the option that expresses the given sentence in passive voice. The Smiths have put a huge Christmas tree. (a) A huge Christmas tree is being but up by the Smiths. (b) A huge Christmas tree was put up by the Smiths. (c) A huge Christmas tree is put up by the Smiths. (d) A huge Christmas tree has been put up by the Smiths. SSC CGL (Tier-I) 13/04/2022 Shift-I Ans. (d) : The above sentence is in Active voice of Present Perfect tense. Its Passive voice structure will be– 'A huge Christmas tree has been put up by the Smiths.' The structure of the sentence is – A.V.- Sub. + has/ have + v3 + object P.V.- Obj. + has/have + been + v3 + by + sub. Hence the correct answer is (d). 118. Select the option that expresses the given sentence in passive voice. She handles all tasks efficiently. (a) All tasks were handled efficiently by her. (b) All tasks are being handled efficiently by her. (c) All tasks have been handled efficiently by her. (d) All tasks are handled efficiently by her. SSC CGL (Tier-I) 11/04/2022 Shift-I Ans. (d) : In the above sentence the use of 'handles' (v1 + s/es) makes it clear that the given sentence is in Active voice of Present Indefinite tense. Its Passive voice structure will be– A.V. → Subject + (v1 + s/es) + object + other words P.V. → Object + is/am/are + v3 + by + subject + other words. Correct Active Voice→ All tasks are handled efficiently by her. 119. Select the option that expresses the given sentence in active voice. All weapons were surrendered by them. (a) They had surrendered all weapons. (b) They have surrendered all weapons. (c) They are surrendering all weapons. (d) They surrendered all weapons. SSC CGL (Tier-I) 11/04/2022 Shift-I Ans. (d) : In the above sentence the use of 'were surrendered' (were + V3) makes it clear that the given sentence is in Passive voice of Past Indefinite tense. Its Active voice structure will be– Active / Passive Voice

P.V.→ Object + was/were + v3 + by + subject + other words. A.V.→ Subject + v2 + object + other words. Correct Passive Voice– They surrendered all weapons. 120. Select the option that expresses the given sentence in active voice. Trumpets are being blown by the soldiers to announce the victory. (a) The soldiers were blowing the trumpets to announce the victory. (b) The soldiers blew the trumpets to announce the victory. (c) The soldiers below the trumpets to announce the victory. (d) The soldiers are blowing the trumpets to announce the victory. SSC CGL (Tier-I) 11/04/2022 Shift-III Ans. (d) : The above sentence is in Passive from of Present continuous tense. Its Active voice structure will be– P.V.→ Obj. + is/am/ are + being + v3. + by + sub. + other. words. A.V.→ Sub. + is/am/ are + V1+Ing + obj. + other words. Correct Active voice – The soldiers are blowing the trumpets to announce the victory. 121. Select the option that expresses the given sentence in active voice. How many candidates had been interviewed by him before lunch? (a) How many candidates had he interviewed before lunch? (b) How many candidates was he interviewing before lunch? (c) How many candidates did he interview before lunch? (d) Hown many candidates has he interviewed before lunch? SSC CGL (Tier-I) 11/04/2022 Shift-III Ans. (a) : The above sentence is in Passive voice of Past Perfect tense. Its Active voice structure will be– P.V.→ Wh. word (Phrase) + had + been + v3 + by + sub + other words? A.V.→ Wh. word (Phrase) + had + sub + v3 + other words? Correct Active VoiceHow many candidates had he interviewed before lunch? 122. Select the option that expresses the given sentence in passive voice. Do you trust me? (a) Do I was trusted by you? (b) Am I trusted by you? (c) Do I am trusted by you? (d) I am trusted by you. SSC CGL (Tier-I) 12/04/2022 Shift-I Ans. (b) : The above sentence starting with "Do+V1" makes it clear that the given sentence is in Active voice of interrogative form of Present Indefinite tense. Its Passive voice structure will be–

309

YCT

A.V. - Do/Does + sub+V1 + obj. + others.? P.V. - Am/is/are+obj.+V3+by + Sub. + others? Correct Passive Voice– Am I trusted by you? 123. Select the option that expresses the given sentence in active voice. Hatred can be overcome by love. (a) Love has overcome hatred. (b) Love can overcame hatred. (c) Love is overcoming hatred. (d) Love can overcome hatred. SSC CGL (Tier-I) 12/04/2022 Shift-I Ans. (d) : In the above sentence the use of 'Can + be + V3' makes it clear that the given sentence is in Passive voice of modal sentence. Its Active voice structure will be– A.V. – Sub + Modals + V1 + obj. + others. P.V. – Obj. + Modals + be + V3 + by + sub + others. Correct Active Voice– Love can overcome hatred. 124. Select the correct passive form of the given sentence. The Central Board of Secondary Education has launched a new helpline for students. (a) A new helpline for students is launched by the Central Board of Secondary Education. (b) A new helpline for students will be launched by the Central Board of Secondary Education. (c) A new helpline for students had been launched by the Central Board of Secondary Education. (d) A new helpline for students has been launched by the Central Board of Secondary Education. SSC CHSL (Tier-I) –04/08/2021 (Shift-I) Ans. (d) : The above sentence is in Active voice of Present Perfect tense. Its passive structure will be – A.V. – Sub + has/have + V3 + Object + Others P.V. – Object + has/have + been + V3 + by + Subject + Others. Correct Passive form A new helpline for students has been launched by the Central Board of Secondary Education. 125. Select the option that expresses the given sentence in passive voice. Sai Kumari gave Akhila a bouquet of lillies on her birthday. (a) Akhila was given a bouquet of lillies on her birthday by Sai Kumari (b) Akhila was gave a bouquet of lillies on her birthday by Sai Kumari (c) Akhila is given a bouquet of lillies on her birthday by Sai Kumari (d) Akhila gave a bouquet of lillies on her birthday to Sai Kumari SSC CHSL (Tier-I) –05/08/2021 (Shift-I) Ans. (a) : The above sentence is in Active voice of Past Indefinite tense. Its Passive voice structure will be– A.V. – Sub + V2 + Object + Others P.V. – Object + was/were + V3 + by + Sub + Others. Correct Passive Voice– Akhila was given a bouquet of lillies on her birthday by Sai Kumari. Active / Passive Voice

126. Select the correct passive form of the given sentence. You can see vast valleys sloping into the distance. (a) Vast valleys will be seen sloping into the distance. (b) Vast valleys had been seen sloping into the distance. (c) Vast Valleys can be seen sloping into the distance. (d) Vast Valleys are seen sloping into the distance. SSC CHSL (Tier-I) –16/04/2021 (Shift-I) Ans. (c) : In the above sentence the use of modal 'Can' makes it clear that the given sentence is in Active voice of modal sentence. Its Passive voice structure will be– A.V.– Sub. + modal (can) + V1 + Object + Other words P.V.– Obj. + modal (can) + be + V3 + by + Sub. + Other words. Correct Passive Voice– Vast Valleys can be seen sloping into the distance. 127. Select the correct passive form of the given sentence. The government is helping the states affected by the cyclone. (a) The states affected by the cyclone have been helped by the government (b) The states affected by the cyclone will be helped by the government. (c) The states affected by the cyclone are being helped by the govenment. (d) The government is going to help the states affected by the cyclone. SSC CHSL (Tier-I) –15/04/2021 (Shift-I) Ans. (c) : In the above sentence the use of 'is + helping' (V1+ing) makes it clear that the given sentence is in Active voice of Present continuous tense. Its Passive structure will be– A.V.– Sub + is/am/are + v1 + ing + object + other words. P. V.– Object + is/are/are + being + v3 + by + subject + other words. Correct Passive voiceThe states affected by the cyclone are being helped by the government. 128. Select the correct passive form of the given sentence. A traffic policeman fined Taru for jumping the red light. (a) A traffic policeman was fined by Taru for jumping the red light. (b) Taru was fined by a traffic policeman for jumping the red light. (c) Taru is fined by a traffic policeman for jumping the red light. (d) Taru has been fined by a traffic policeman for jumping the red light. SSC CHSL (Tier-I) –13/04/2021 (Shift-I) Ans. (b) : In the above sentence the use of 'fined (VII)' makes it clear that the given sentence is in Active voice of Past Indefinite tense. Its Passive voice structure will be–

310

YCT

A.V.– Subject + V2 + Object + Other words P.V.– Object + was/were + VIII + by + Subject + Other words Correct Passive Voice– Taru was fined by a traffic policeman for jumping the red light. 129. Select the correct passive form of the given sentence. He drove the car very carefully on the hilly terrain. (a) The car was driven very carefully on the hilly terrain. (b) The car is being driven very carefully on the hilly terrain. (c) The car has been driven very carefully on the hilly terrain. (d) The car is driven very carefully on the hilly terrain. SSC CHSL (Tier-I) –06/08/2021 (Shift-I) Ans. (a) : In the above sentence the use of 'drove' (V2) makes it clear that the given sentence is in Active voice of Past Indefinite tense. Its Passive voice structure will be– A.V. – Sub + V2 + Object + Other words P.V. – Object + was/were + V3 + by + Subject + Other words Correct Passive Voice– The car was driven very carefully on the hilly terrain. 130. Select the option that expresses the given sentence in passive voice. Aditya passed the driving test quite easily. (a) The driving test had passed quite easily by Aditya. (b) The driving test have been passed quite easily by Aditya. (c) The driving test could passed quite easily by Aditya. (d) The driving test was passed quite easily by Aditya. SSC CHSL (Tier-I) –09/08/2021 (Shift-I) Ans. (d) : The above sentence is in Active voice of Past Indefinite tense. Its Passive voice structure will be– A.V.- Sub. + Verb2 + Obj. + Other words. P.V.- Obj. + was/were + VIII + by + Sub. + Other words. Correct Passive Voice– The driving test was passed quite easily by Aditya. 131. Select the option that expresses the given sentences in the active or passive voice. Celebrate the New Year with amazing offers. (a) The New Year will be celebrated with amazing offers. (b) Amazing offers will be celebrate in the New Year. (c) Let the New Year be celebrated with amazing offers. (d) The New Year should be celebrated with amazing offers. SSC CHSL (Tier-I) –19/04/2021 (Shift-I) Ans. (c) : In the above sentence the use of "V1 + object" makes it clear that the given sentence is in Active voice of Imperative sentence. Its Passive voice structure will be– Active / Passive Voice

Active - VI + Object + other words. Passive - Let + Object + be + vIIIrd + other words. or You are advised to + VI + Object. Correct Passive VoiceLet the New Year be celebrated with amazing offers. 132. Select the option that expresses the given sentence in passive voice. Richard wore a broad belt with shiny buckles. (a) A broad belt with shiny buckles had been worn by Richard. (b) A broad belt with shiny buckles was wear by Richard. (c) A broad belt with shiny buckles wore Richard. (d) A broad belt with shiny buckles was worn by Richard. SSC CGL (Tier-I) –24/08/2021 (Shift-I) Ans. (d) : In the above sentence the use of 'wore (V2)" makes it clear that the given sentence is in Active voice of Past Indefinite tense. Its Passive voice structure will be– A.V. – Subject + V2 + Object P.V. – Object + was/were + V3 + by + Subject Correct Passive Voice– A broad belt with shiny buckles was worn by Richard. 133. Select the option that expresses the given sentence in passive voice. He gave Kaveri a beautiful yellow dress. (a) Kaveri has been given a beautiful yellow dress by him. (b) Kaveri was given a beautiful yellow dress by him. (c) Kaveri is given a beautiful yellow dress by him. (d) Kaveri has given a beautiful yellow dress by him. SSC CGL (Tier-I) –23/08/2021 (Shift-I) Ans. (b) : In the above sentence the use of 'gave (v2)' makes it clear that the given sentence is in Active voice of Past Indefinite tense. Its Passive voice structure will be– A.V. – Subject + V2 + Object P.V. – Object + was/were + V3 + by + Subject Correct Passive VoiceKaveri was given a beautiful yellow dress by him. 134. Select the option that expresses the given sentence in passive voice. Someone stole his traveller's cheques when he was travelling in Europe. (a) Someone had been stole his traveller's cheques when he was travelling in Europe. (b) His traveller's cheques are stole when he was travelling in Europe. (c) His traveller's cheques was stole when he was travelling in Europe. (d) His traveller's cheques were stolen when he was travelling in Europe. SSC CGL (Tier-I) –16/08/2021 (Shift-I) Ans. (d) : In the above sentence in which two clauses are given, make the voice of only Principle clause. In this sentence the use of verb 'stole (V2)' makes it clear that the given sentence is in Active voice of Past Indefinite Tense. Its Passive voice structure will be–

311

YCT

A.V.– Subject + V2 + Object + Subordinate Clause. P.V.– Object + was/were + V3 + (by agent) + Subordinate Clause. Note- Sometimes 'by + Subject' is understood in case of when someone, somebody is given. Correct Passive Voice– His traveller's cheques were stolen when he was travelling in Europe. 135. Select the option that expresses the given sentence in passive voice. Imran has found two good jobs in the advertisements of the newspaper. (a) Two good jobs had found by Imran in the advertisements of the newspaper. (b) Two good jobs has found by Imran in the advertisements of the newspaper. (c) Two good jobs are being found by Imran in the advertisements of the newspaper. (d) Two good jobs have been found by Imran in the advertisements of the newspaper. SSC CGL (Tier-I) –18/08/2021 (Shift-I) Ans. (d) : In the above sentence the use of verb 'has + found (V3)' makes it clear that the given sentence is in Active voice of Present Perfect tense. Its Passive voice structure will be– A.V. – Subject + has/have + V3 + Object P.V. – Object + has/have + been + V3 + by + Subject Correct Passive Voice– Two good jobs have been found by Imran in the advertisements of the newspaper. 136. Select the option that expresses the given sentence in passive voice. You can solve this problem. (a) This problem ought to be solved by you. (b) This problem should be solved by you. (c) This problem can be solved by you. (d) This problem might be solved by you. SSC CGL (Tier-I) –20/08/2021 (Shift-I) Ans. (c) : The above sentence is in Active voice of Modal verb (Can). Its Passive voice structure will be– A.V. – Subject + Can + VI + Object. P.V. – Object + Can + be + VIII + by + Subject. Correct Passive VoiceThis problem can be solved by you. 137. Select the option that expresses the given sentence in passive voice. The man bought a parrot that could speak five languages. (a) A parrot that could spoke five languages bought the man. (b) A parrot that spoken five languages was bought by the man. (c) A parrot that spoke five languages had been bought by the man. (d) A parrot that spoke five languages was bought by the man. SSC CGL (Tier-I) –17/08/2021 (Shift-I) Ans. (d) : In the above sentence the use of verb 'bought (V2)' makes it clear that the given sentence is in Active voice of Past Indefinite tense. Its Passive voice structure will be– Active / Passive Voice

A.V. – Subject + V2 + Object 3 P.V. – Object + Was/Were + V + by + Subject Correct Passive VoiceA parrot that spoke five languages was bought by the man. 138. Select the option that expresses the given sentence in passive voice. The invigilator is advising the students not to carry calculators into the examination hall. (a) The students have been advised not to carry calculators into the examination hall by the invigilator. (b) The students were advised not to carry calculators into the examination hall. (c) The students were advised the invigilator not to carry calculators into the examination hall. (d) The students are being advised by the invigilator not to carry calculators into the examination hall. SSC CGL (Tier-I) –13/08/2021 (Shift-I) Ans. (d) : In the above sentence the use of (is + advising) makes it clear that the given sentence is in Active voice of Present Continuous tense. Its Passive voice structure will be– A.V. – Subject + is/am/are + V1+ing + Object. P.V. – Object + is/am/are + being + V3 + by + Subject Correct Passive VoiceThe students are being advised by the invigilator not to carry calculators into the examination hall. 139. Select the correct active form of the given sentence. Let the shops be opened. (a) The shops should be opened (b) Have the shops opened? (c) Open the shops (d) The shops are open SSC CGL (Tier-II) 16/11/2020 (3 pm - 5 pm) Ans. (c) : In the above sentence the use of "Let + Subject + be + V3" makes it clear that the given sentence is in Passive voice of Imperative sentence. Its Active voice structure will be – Active voice- VI + object + other words Passive voice- Let + object + be + V3 + other words Correct Active Voice- Open the shops. 140. Select the correct active form of the given sentence. Was your bag left in the bus? (a) Did you left your bag in the bus? (b) Have you left your bag in the bus? (c) Was you leaving your bag in the bus? (d) Did you leave your bag in the bus? SSC CGL (Tier-II) 16/11/2020 (3 pm - 5 pm) Ans. (d) : In the above sentence the use of verb "was + V3" makes it clear that the given sentence is in Passive voice of interrogative form of Past Indefinite tense. Its Active voice structure will be – P.V.- was+object+V3+other words + ? A.V.- Did+sub+V1+obj+other words + ? Correct Active VoiceDid you leave your bag in the bus?

312

YCT

141. Select the correct active form of the given Ans. (c) : In the above sentence the use of verb "has sentence. been + V3" makes it clear that the given sentence is in The catch should not be dropped. Passive Voice of Present Perfect tense. Its Active voice (a) Let the catch not be dropped structure will be – (b) You would not drop the catch P.V.- Obj. + has/have + been + V3 + by + sub. (c) Have you dropped the catch A.V.- Sub. + has/have + V3 + obj. (d) Don't drop the catch Correct Active Voice– SSC CGL (Tier-II) 16/11/2020 (3 pm - 5 pm) Mother has packed your lunch box. Ans. (d) : In the above sentence the use of verb 145. Select the correct passive form of the given sentence. "Should + be + V3" makes it clear that the given We all regard Liza as an expert. sentence is in Passive voice of Imperative sentence. Its (a) Liza should be regarded as an expert by all of us. Active voice structure will be – (b) Liza has been regarded as an expert by all of us A.V.- Don't + V1 + Object (c) Liza is regarded as an expert by all of us P.V.- Object + should not + be + V3. (d) Liza was regarded as an expert by all of us Correct Active Voice- Don't drop the catch. SSC CGL (Tier-II) 16/11/2020 (3 pm - 5 pm) 142. Select the correct passive form of the given Ans. (c) : In the above sentence the use of verb Regard sentence. They are making elaborate arrangements for V1 makes it clear that the given sentence is in Active voice of Present Indefinite tense. Its Passive voice the party. (a) Elaborate arrangements have been made for structure will be – A.V.- Sub. + V1 + obj. + others. the party. P.V.- Obj. + is/am/are + V3 + by + sub. + others. (b) Elaborate arrangements are made for the Correct Passive Voice– party. Liza is regarded as an expert by all of us. (c) Elaborate arrangements were being made for 146. Select the correct passive form of the given the party. sentence. (d) Elaborate arrangements are being made for They will lay the foundation stone next week. the party. (a) The foundation stone will be laid by them SSC CGL (Tier-II) 16/11/2020 (3 pm - 5 pm) next week Ans. (d) : In the above sentence the use of verb 'are + (b) The foundation stone is being laid by them v + ing" makes it clear that the given sentence is in next week Active voice of Present Continuous tense. Its Passive (c) The foundation stone will be laying by next structure will be– week A.V.- Sub + is/am/are + V1+ing + object + others (d) The foundation stone will have been laid by P.V.- Object + is/am/are + being + V3 + subject + them next week others. SSC CGL (Tier-II) 16/11/2020 (3 pm-5 pm) Correct Passive Voice- Elaborate arrangements are Ans. (a) : In the above sentence the use of verb "Sub + being made for the party will + V1" makes it clear that the given sentence is in 143. Select the correct passive form of the given Active voice of Future Indefinite tense. Its Passive sentence. voice structure will be– He asked me to sing a song. A.V.- Sub + will/shall + V1 + obj. + other words. (a) I was being asked to sing a song. P.V.- Obj + will/shall + be + V3 + by + sub + other (b) I was asked to sing a song. words. (c) I am asked to sing a song. Correct Passive Voice– The foundation stone will be laid by them next week. (d) I had been asked to sing a song. SSC CGL (Tier-II) 16/11/2020 (3 pm - 5 pm) 147. Select the correct active form of the given sentence. Ans. (b) : In the above sentence the use of verb "Sub + Only home-cooked food is eaten by us. V2" makes it clear that the given sentence is in Active (a) We are eating only home-cooked food voice of Past Indefinite tense. Its Passive voice (b) We ate only home-cooked food structure will be – (c) We eat only home-cooked food A.V.- Sub. + V2 + Object + others (d) We have eaten only home-cooked food P.V.- Object + was/were + V3 + sub + others SSC CGL (Tier-II) 16/11/2020 (3 pm - 5 pm) Correct Passive Voice– I was asked to sing a song. Ans. (c) : In the above sentence the use of verb "is + 144. Select the correct active form of the given V3" makes it clear that the given sentence is in Passive sentence. voice of Present Indefinite tense. Its Active voice Your lunch box has been packed by mother. structure will be – (a) Mother had packed your lunch box P.V.- Obj+is/am/are+V3+by+sub. (b) Mother is packing your lunch box A.V.- Sub+V1+Object (c) Mother has packed your lunch box Correct Active Voice– (d) Mother will pack your lunch box SSC CGL (Tier-II) 16/11/2020 (3 pm - 5 pm) We eat only home-cooked food. Active / Passive Voice

313

YCT

148. Select the correct passive form of the given Ans. (a) : In the above sentence the use of verb "are + sentence. V1 + ing" makes it clear that the given sentence is in We have looked at the plan carefully. Active voice of Present continuous tense. Its Passive (a) The plan is being looked at carefully voice structure will be– (b) The plan have been looked at carefully A.V.- Subject + is/am/are + Ving+ obj. + others. (c) The plan has been looked at carefully P.V.- Obj. + is/am/are + being + V3 + by + sub. + (d) The plan was looked at carefully others. SSC CGL (Tier-II) 16/11/2020 (3 pm - 5 pm) Correct Passive Voice- The car is being lifted with a crane. Ans. (c) : In the above sentence the use of verb 'have + V3" makes it clear that the given sentence is in Active 152. Select the correct passive form of the given voice of Present Perfect tense. Its Passive voice sentence. structure will be– My mother gave me an interesting book. A.V.- Sub + has/have + V3 + obj. + others. (a) I was given an interesting book by my mother P.V.- Obj. + has/have + been + V3 + by + sub + others. (b) An interesting book is given to me by my Correct Passive Voice– mother The plan has been looked at carefully. (c) I am given an interesting book by my mother (d) An interesting book has been given to my 149. Select the correct passive form of the given mother sentence. SSC CGL (Tier-II) 16/11/2020 (3 pm - 5 pm) Razia Sultana ruled over the northern part of India. Ans. (a) : In the given sentence the use of "Subject + (a) The northen part of India was ruled over by V2" makes it clear that the given sentence is in Active Razia Sultana. voice of Past Indefinite tense. Its Passive voice (b) The northen part of India had been ruled over structure will be– by Razia Sultana. A.V.- Sub. + V2 + obj. + others. (c) The northen part of India was being ruled P.V.- Object + was/were + V3 + by + sub + others. over by Razia Sultana. Correct Passive Voice– I was given an interesting (d) The northen part of India is ruled over by book by my mother. Razia Sultana. 153. Select the correct active form of the given SSC CGL (Tier-II) 16/11/2020 (3 pm - 5 pm) sentence. Ans. (a) : In the above sentence the use of verb "Sub + By whom were you pushed into the mud? (a) Who was pushing you into the mud? V2" makes it clear that the given is in Active voice of (b) Whom did you push into the mud? Past Indefinite tense. Its Passive voice structure will (c) Who has pushed you into the mud? be– (d) Who pushed you into the mud? A.V.- Subject + V2 + obj. + other words. SSC CGL (Tier-II) 16/11/2020 (3 pm - 5 pm) P.V.- Object + was/were + V3 + by + sub. + other words. Ans. (d) : In the above sentence the use of verb "were Correct Passive Voice– The northern part of India + V3" makes it clear that the given sentence is in was ruled over by Razia Sultana. Passive voice of Past Indefinite tense. Its Active voice 150. Select the correct active form of the given structure will be – P.V.- By whom + was/were + subject + V3 + others + ? sentence. A.V.- Who + V2 + object + others + ? You might be promoted this year. Note- When an interrogative sentence starts with 'who' (a) They might promote you this year then the Passive Voice the beginning of the sentence is (b) They will promote you this year 'by whom' because "Who" is replaced by object 'by (c) They should have promoted you this year whom and object' to the subject of Passive voice. (d) You may promote them this year SSC CGL (Tier-II) 16/11/2020 (3 pm - 5 pm) Correct Active Voice- Who pushed you into the mud? Ans. (a) : In the above sentence the use of verb "might 154. Select the correct active form of the given sentence : + be + V3" makes it clear that the given sentence is in Kites have been bought by many children. Passive voice of modal sentence. Its Active voice (a) Many children had bought kites structure will be– (b) Many children have bought kites P.V.- Obj. + might + be + V3 + by + sub. + others. (c) Many children bought kites A.V.- Sub. + might + V1 + object + others. (d) Many children are buying kites Correct Active VoiceSSC CGL (Tier-II) 16/11/2020 (3 pm - 5 pm) They might promote you this year. 151. Select the correct passive form of the given Ans. (b) : In the above sentence the use of verb "have been + V3" makes it clear that the given sentence is in sentence. Passive voice of Present Perfect tense. Its Active voice They are lifting the car with a crane. structure will be– (a) The car is being lifted with a crane. P.V.- Object + have/has + been + V3 + by + sub. (b) The car is lifted with a crane A.V.- Sub+has/have+V3+object. (c) The car was being lifted with a crane Correct Active Voice- Many children have bought (d) The car had lifted with a crane SSC CGL (Tier-II) 16/11/2020 (3 pm - 5 pm) kites. Active / Passive Voice

314

YCT

155. Select the correct passive form of the given 159. Choose the option that is the passive form of sentence : the sentence. Hang the washing on the clothesline. You have not yet responded to several of my (a) Let the washing be hung on the clothesline complaints. (b) Let the washing to be hung on the clothesline (a) Several of my complaints you have not been (c) Let the washing be hanged on the clothesline responded to yet. (d) Let the washing hanging on the clothesline. (b) My complaints are not been responding yet SSC CGL (Tier-II) 16/11/2020 (3 pm - 5 pm) by you. Ans. (a) : In the above sentence the use of "V1 in the (c) Several of my complaints are not being beginning of the sentence, makes it clear that the given responded yet. sentence is in Active voice of imperative sentence. Its (d) Several of my complaints have not been Passive voice structure will be – responded to yet. A.V.- V1 + Obj. + other words. SSC CPO-SI (Tier-II) 27/09/2019 (3 pm - 5 pm) P.V.- Let + Obj. + be + V3 + other words. Correct Passive Voice– Let the washing be hung on Ans. (d) : In the above sentence the use of verb "have the clothesline. + not + V3" makes it clear that the given sentence is in 156. Select the correct passive form of the given Active voice of Present Perfect tense. Its Passive voice sentence : structure will be– Harsh has applied for leave A.V.- Sub + has/have + not + V3 + object. (a) Leave is applied for by Harsh. P.V.- Object + has/have + not + been + V3 + by + sub. (b) Leave had been applied by Harsh. Correct Passive Voice– Several of my complaints (c) Leave has been applied for by Harsh. have not been responded to yet. (d) Leave was applied by Harsh. 160. Choose the option that is the active form of the SSC CGL (Tier-II) 16/11/2020 (3 pm - 5 pm) sentence. Ans. (c) : In the above sentence the use of verb 'has + I am sometimes puzzled by her actions. V3" makes it clear that the given sentence is in Active (a) Her actions are sometimes puzzled by me. voice of Present Perfect tense. Its Passive voice (b) Her actions sometimes puzzle me. structure will be– (c) She can sometimes puzzle me. A.V.- Sub. + has/have + V3 + obj. + others. (d) Her actions were sometimes puzzling me. P.V.- Obj. + has/have + been + V3 + by + sub + others. SSC CPO-SI (Tier-II) 27/09/2019 (3 pm - 5 pm) Correct Passive Voice– Leave has been applied for by Harsh. Ans. (b) : In the above sentence the use of verb "Am + 157. Select the correct active form of the given V3" makes it clear that the given sentence is in Passive sentence : voice of Present Indefinite tense. Its Active structure He was hit in the eye by an arrow : will be– (a) An arrow hit him in the eye. P.V.- Object + is/am/are + V3 + by + sub. (b) An arrow will hit him in the eye. A.V.- Subject + V1 + (s/es) + object. (c) An arrow was hitting him in the eye. Correct Active Voice(d) An arrow has hit him in the eye. Her actions sometimes puzzle me. SSC CGL (Tier-II) 16/11/2020 (3 pm - 5 pm) 161. Choose the option that is the passive form of Ans. (a) : In the above sentence the use of verb "was + 3 the sentence. V " makes it clear that the given sentence is in Passive The first settlers displaced the original voice of Past Indefinite tense. Its Active voice inhabitants. structure will be – (a) The original inhabitants are displacing the P.V.- Object + was/were + V3 + by + sub. first settlers. A.V.- Subject + V2 + Object + others. Correct Active Voice- An arrow hit him in the eye. (b) The original inhabitants were displaced by the first settlers. 158. Select the correct active form of the given sentence : (c) The original inhabitants will displace the first A new song is being composed by her. settlers. (a) She is composing a new song. (d) The original inhabitants are being displace by (b) She has composed a new song. the first settlers. (c) She has been composing a new song. SSC CPO-SI (Tier-II) 27/09/2019 (3 pm - 5 pm) (d) She composed a new song. Ans. (b) : In the above sentence the use of 'verb2' SSC CGL (Tier-II) 16/11/2020 (3 pm - 5 pm) (displaced) makes it clear that the given sentence is in Ans. (a) : In the above sentence the use of verb "is + Active voice of Past Indefinite tense. Its Passive voice being + V3" makes it clear that the given sentence is structure will be– Passive voice of Present Continuous Tense. Its Active A.V.- Subject + V2 + object. voice structure will be– P.V.- Object + was/were + V3 + by + subject. P.V.- Object + is + being + V3 + by + sub. Correct Passive Voice– The original inhabitants were A.V.- Sub + is + Ving + object. Correct Active Voice- She is composing a new song. displaced by the first settlers. Active / Passive Voice

315

YCT

162. Choose the option that is the passive form of the sentence. Avoid taking the flyover. (a) The flyover can be avoid take. (b) The flyover should be avoided. (c) The flyover should avoid taking. (d) The flyover should be avoid. SSC CPO-SI (Tier-II) 27/09/2019 (3 pm - 5 pm) Ans. (b) : In the above sentence the use of verb "V1 + Subject" makes it clear that the given sentence is in Active voice of Imperative sentence. Its Passive voice structure will be– A.V.- V1 + Obj. + others. P.V.- Obj. + should + be + V3 Correct Passive Voice– The flyover should be avoided. 163. Choose the option that is the active form of the sentence. Has your message been despatched? (a) Has your message despatched? (b) Have you despatched your message? (c) Had you despatch your message? (d) Is your message despatching? SSC CPO-SI (Tier-II) 27/09/2019 (3 pm - 5 pm) Ans. (b) : In the above sentence the use of "has + Subject + been + V3" makes clear that the given sentence is in Passive voice of Interrogative form of Past Perfect tense. Its Active Voice structure will be – P.V.- has/have + object + been + V3 + by + subject? A.C.- Has/have + subject + V3 + object? Correct Active Voice- Have you despatched your message? 164. Choose the option that is the passive form of the sentence. They need 104 more runs to win the match. (a) 104 more runs are needed for them to win the match. (b) 104 more runs will need for them to win the match. (c) They are needed 104 more runs to win the match. (d) 104 more runs can be needed by them to win the match. SSC CPO-SI (Tier-II) 27/09/2019 (3 pm - 5 pm) Ans. (a) : In the above sentence the use of 'verb1' (need) makes it clear that the given sentence is in Active voice of Present Indefinite tense. Its Passive voice structure will be– A.V.- Sub. + V1 + object + other words P.V.- Obj. + is/am/are + V3 + by + sub. + other words. Correct Passive Voice– 104 more runs are needed for them to win the match. 165. Choose the option that is the passive form of the sentence. The security guard opened the gate using his pass. (a) The gate is open by the security guard by using his pass. (b) The gate was opened by the security guard using his pass. (c) The gate opened the security guard using his pass. (d) The gate was open by the security guard use his pass. SSC CPO-SI (Tier-II) 27/09/2019 (3 pm - 5 pm) Active / Passive Voice

Ans. (b) : In the above sentence the use of verb2 (opened) makes it clear that the given sentence is in Active voice of Past Indefinite tense. Its Passive voice structure will be– A.V.- Sub. + V2 + object + other words. P.V.- Obj. + was/were + V3 + by + sub + other words. Correct Passive Voice– The gate was opened by the security guard using his pass. 166. Choose the option that is the passive form of the sentence. The detective found the lost necklace. (a) The lost necklace should find the detective. (b) The lost necklace was found by the detective. (c) The lost necklace founded by the detective. (d) The lost necklace had been find by the detective. SSC CPO-SI (Tier-II) 27/09/2019 (3 pm - 5 pm) Ans. (b) : In the above sentence the use of verb2 (found) makes it clear that the given sentence is Active voice of Past Indefinite tense. Its Passive voice structure will be – A.V.- Sub. + V2 + object + others. P.V.- Obj. + was/were + V3 + by + sub + others. Correct Passive Voice– The lost necklace was found by the detective. 167. Choose the option that is the passive form of the sentence. The fisherman caught a large fish. (a) A large fish should be caught by the fisherman. (b) A large fish was catching the fisherman. (c) A large fish was caught by the fisherman. (d) A large fish had been catched by the fisherman. SSC CPO-SI (Tier-II) 27/09/2019 (3 pm - 5 pm) Ans. (c) : In the above sentence the use of "Subject + V2" makes it clear that the given sentence is in Active voice of Past Indefinite tense. Its Passive voice structure will be– A.V.- Sub. + V2 + object. P.V.- Obj. + was/were + V3 + by + sub. Correct Passive Voice– A large fish was caught by the fisherman. 168. Choose the option that is the passive form of the sentence. They found her guilty of theft. (a) She found them guilty of theft. (b) She was found guilty of theft. (c) She had been find guilty of theft. (d) She is find guilty of theft by them. SSC CPO-SI (Tier-II) 27/09/2019 (3 pm - 5 pm) Ans. (b) : In the above sentence the use "Sub + V2" makes it clear that the given sentence is in Active voice of Past Indefinite tense. Its Passive voice structure will be– A.V.- Sub. + V2 + object. P.V.- Object + was/were + V3 + by + sub. Correct Passive Voice– She was found guilty of theft. 169. Choose the option that is the active form of the sentence. By whom was this poem written? (a) Who wrote this poem? (b) Who is wrote this poem? (c) Who write this poem? (d) This poem is wrote by whom? SSC CPO-SI (Tier-II) 27/09/2019 (3 pm - 5 pm)

316

YCT

Ans. (c) : In the above sentence the use of "Sub + V2" Ans. (a) : In the above sentence the use of "By whom 3 makes it clear that the given sentence is in Active + was + V " makes it clear that the given sentence is in voice of Past Indefinite tense. Its Passive voice Passive voice of Interrogative form of Past Indefinite structure will be– tense. Its Active voice structure will be– A.V.- Sub.+V2+obj.+other words. P.V.- By whom + was/were + sub. + V3 + others + ? P.V.- Obj.+was/were+V3+by+sub.+other words. A.V.- Who + V2 + obj. + others + ? Correct Passive Voice– The manhole covers were Correct Active Voice- Who wrote this poem? changed before the rainy season. 170. Choose the option that is the passive form of 173. Choose the option that is the passive form of the sentence. the sentence. The tennis ball hit Dhiraj on the head. Switch off the television. (a) Dhiraj had been hit on the head by the tennis (a) Can you switch off the television? (b) May I switch off the television? ball. (c) Let the television be switched off. (b) Dhiraj was hit on the head by the tennis ball. (d) Let the television being switch off. (c) The tennis ball was being hit by Dhiraj. SSC CPO-SI (Tier-II) 27/09/2019 (3 pm-5 pm) (d) Dhiraj was being hit on the head by the tennis Ans. (c) : In the above sentence starting with verb ball. SSC CPO-SI (Tier-II) 27/09/2019 (3 pm-5 pm) 'switch off', it is clear that the sentence is in Imperative sentence. Its Passive voice structure will be – Ans. (b) : In the above sentence the use of "Sub + V2" A.V.- V1 + obj. + others. makes it clear that the given sentence is in Active P.V.- Let + obj. + be + V3 + others. voice of Past Indefinite tense. Its Passive voice Correct Passive Voice– Let the television be switched structure will be – off. A.V.- Sub.+V2+obj.+other words. 174. Choose the option that is the passive form of P.V.- Obj.+was/were+V3+by+sub.+other words. the sentence. I burnt my hand yesterday while cooking. Correct Passive Voice– Dhiraj was hit on the head by (a) My hand was burning yesterday while the tennis ball. cooking. 171. Choose the option that is the passive form of (b) My hand could be burnt yesterday while the sentence. cooking. They considered it an impressive building. (c) My hand was burnt yesterday while cooking. (a) It was considered to be an impressive (d) My hand will be burnt yesterday while building. cooking. (b) It can be consider to be an impressive SSC CPO-SI (Tier-II) 27/09/2019 (3 pm - 5 pm) building. Ans. (c) : In the above sentence the use of "Sub + V2 (c) It was considering to being an impressive (brunt)" makes it clear that the given sentence is in building. Active voice of Past Indefinite tense. Its Passive voice structure will be– (d) It is considered to be an impressive building. SSC CPO-SI (Tier-II) 27/09/2019 (3 pm - 5 pm) A.V.- Sub.+V2+object+other words. P.V.- Obj.+was/were+V3+by+sub.+other words. Ans. (a) : In the above sentence the use of "Subject + Correct Passive Voice– V2" makes it clear that the given sentence is in Active My hand was burnt yesterday while cooking. voice of Past Indefinite tense. Its Passive voice 175. Choose the option that is the active form of the structure will be– sentence. A.V.- Sub. + V2 + obj. + other words. An award was given to the film 'Andhaa P.V.- Obj. + was/were + V3+ (to be) + sub. + other Dhund.' words. (a) The jury had given the film ‘Andhaa Dhund’ Correct Passive Voice– It was considered to be an an award. impressive building. (b) The jury gave the film ‘Andhaa Dhund’ an award. 172. Choose the option that is the passive form of (c) The jury was gave the film ‘Andhaa Dhund’ the sentence. an award. The Municipal Corporation changed the (d) The jury will give the award to the film manhole covers before the rainy season. ‘Andhaa Dhund’. (a) The manhole covers is being changed before SSC CPO-SI (Tier-II) 27/09/2019 (3 pm - 5 pm) the rainy season. Ans. (b) : In the above sentence the use of "was + V3" (b) The manhole covers are change before the makes it clear that the given sentence is in Passive rainy season. voice of Past Indefinite tense. Its Active voice (c) The manhole covers were changed before the structure will be– rainy season. P.V.- Obj.+was/were+V3+by+sub+other words. (d) The manhole covers can be changed before A.V.- Sub.+V2+object+other words. Correct Active Voice- The jury gave the film ‘Andhaa the rainy season. SSC CPO-SI (Tier-II) 27/09/2019 (3 pm - 5 pm) Dhund’ an award. Active / Passive Voice

317

YCT

(a) Abhishek has seen Shyam starting the car. 176. Choose the option that is the passive form of (b) Abhishek can be seen starting the car by the sentence. Shyam. The class teacher was taking the children to the (c) Abhishek was seen starting the car by Shyam. zoo. (d) Abhishek was saw by Shyam starting the car. (a) The children will go to the zoo by their class SSC CPO-SI (Tier-II) 27/09/2019 (3 pm - 5 pm) teacher. (b) The children can be taking to the zoo by their Ans. (c) : In the above sentence the use of "Sub + V2" (saw) makes it clear that the given sentence is in class teacher. (c) The children being taken to the zoo by their Active Voice of Past Indefinite tense. Its Passive voice structure will be– class teacher. (d) The children were being taken to the zoo by A.V.- Sub.+V2+obj.+other words. P.V.- Obj.+was/were+V3+by+sub.+other words. their class teacher SSC CPO-SI (Tier-II) 27/09/2019 (3 pm-5 pm) Correct Passive Voice– Abhishek was seen starting the car by Shyam. Ans. (d) : In the above sentence the use of "was + verb + ing" makes it clear that the given sentence is in 180. Choose the option that is the passive form of the sentence. Active voice of Past Continuous tense. Its Passive The little puppy chewed my new slippers. voice structure will be– (a) My new slippers were chewing the little A.V.- Sub. + was/were + Ving + object + other words. puppy. P.V. - Object + was/were + being + V3 + by + sub. + (b) My new slippers can be chewed by the little other words. puppy. Correct Passive Voice– The children were being (c) My new slippers were chewed by the little taken to the zoo by their class teacher. puppy. 177. Choose the option that is the passive form of (d) My new slippers are being chew by the little the sentence. puppy. The manager permitted women employees to SSC CPO-SI (Tier-II) 27/09/2019 (3 pm - 5 pm) leave office early on that day. Ans. (c) : In the above sentence the use of verb (a) Women employees permitted the manager to "Chewed" (V2) makes it clear that the given sentence is leave office early on that day. in Active voice of Past Indefinite tense. Its Passive (b) Women employees had to leave office early voice structure will be– on that day. A.V.- Sub.+V2+obj.+other words. (c) Women employees should leave office early P.V.- Obj.+was/were+V 3+by+sub.+other words. on that day. Correct Passive Voice– My new slippers were (d) Women employees were permitted to leave chewed by the little puppy. office early on that day. 181. Choose the option that is the passive form of SSC CPO-SI (Tier-II) 27/09/2019 (3 pm - 5 pm) the sentence. Ans. (d) : In the above sentence the use of "Sub + V2" The child tore the page of the book. makes it clear that the given sentence is in Active (a) The page of the book was torn by the child. voice of Past Indefinite tense. Its Passive voice (b) The page of the book tore by the child. structure will be– (c) The book’s page is torn by the child. A.V.- Sub.+V2+object+other words. (d) The page of the book is tearing by the child. P.V.- Object+was/were+V3+by+sub+other words. SSC CPO-SI (Tier-II) 27/09/2019 (3 pm - 5 pm) Correct Passive Voice– Ans. (a) : In the above sentence the use of verb "tore" Women employees were permitted to leave office (V2) makes it clear that the given sentence is in Active early on that day. voice of Past Indefinite tense. Its Passive voice 178. Choose the option that is the passive form of structure will be– the sentence. A.V.- Sub.+V2+obj.+other words. Give the command. P.V.- Obj.+was/were+V3+by+sub.+other words. (a) Let the command be given. Correct Passive Voice– The page of the book was (b) You can give the command. torn by the child. (c) The command can be given. (d) The command should be give by you. 182. Choose the option that is the active form of the sentence. SSC CPO-SI (Tier-II) 27/09/2019 (3 pm - 5 pm) The politician's speech was loudly cheered. Ans. (a) : The above sentence starting with "verbI + (a) The audience cheer the politician’s loud Obj". It is clear that the given sentence is in Active Voice of Imperative sentence. Its Passive voice speech structure will be– (b) The audience was loudly cheered by the A.V.- V1+obj.+others. politician’s speech P.V.- Let+obj.+be+V3+others. (c) The audience loudly cheered the politician’s Correct Passive Voice– Let the command be given. speech. 179. Choose the option that is the passive form of (d) The audience had been loudly cheered by the the sentence. politician. Shyam saw Abhishek starting the car. SSC CPO-SI (Tier-II) 27/09/2019 (3 pm - 5 pm) Active / Passive Voice

318

YCT

A.V.- Who+V2+obj.+others. Ans. (c) : In the above sentence the use of "was + V3" P.V.- By whom+was/were+obj.+V3+other. makes it clear that the given sentence is in Passive Correct Passive Voice– By whom was the cup voice of Past Indefinite tense. Its Active voice broken? structure will be– 186. Select the correct passive form of the given P.V.- Object+was/were+V3+by+sub. + other words. sentence. A.V.- Sub.+V2+object + other words. Someone has stolen my watch. Correct Active Voice- The audience loudly cheered (a) My watch was stolen by someone. the politician’s speech. (b) My watch has been stolen by someone. 183. Choose the option that is the active form of the (c) Someone steals my watch. sentence : (d) Someone stole my watch. By whom was this window broken? Five SSC Stenographer Grade C&D –05/02/2019 (3:5pm) packets of milk were delivered by the milkman. 3 (a) Who broke this window? The milkman Ans. (b) : In the above sentence the use of 'has + V ' makes it clear that the given sentence is in Active delivered five packets of milk. (b) Who had broke this window? The milkman voice of Present Perfect tense. Its Passive voice structure will be– had delivered five packets of milk. (c) Whom was break this window? The milkman A.V.- Sub.+has/have+V3+obj. P.V.- Object+has/have + been +V3+by+sub. will deliver five packets of milk. (d) Who is breaking this window? The milkman Correct Passive Voice– My watch has been stolen by someone. is delivering five packets of milk. SSC CPO-SI (Tier-II) 27/09/2019 (3 pm-5 pm) 187. Select the correct passive form of the given sentence. Ans. (a) : In the above sentence the use of was + Open the door. broken (V3) and "Were + delivered" makes it clear that (a) The door is opened by me. the given sentence is in Passive voice of Interrogative (b) Let the door be opened. sentence. Its Active voice structure will be– (c) You should open the door. P.V.- By whom+was/were+obj.+V3+others. (d) The door was opened by me. A.V.- Who+V2+obj.+others. SSC Stenographer Grade C&D –05/02/2019 (3:5pm) Note- Active voice changes the Passive voice-who into Ans. (b) : At the beginning of the above sentence by whom. "open" (V1) makes it clear that the given sentence is in Correct Active Voice- Who broke this window? The Active voice of Imperative sentence. Its Passive milkman delivered five packets of milk. 184. Select the correct passive form of the given structure will be– A.V.- V1+Obj+others. sentence. P.V.- Let+Obj+be+V3+others. The students will never forget a good teacher. (a) A good teacher has never been forgotten by Correct Passive Voice– Let the door be opened. 188. Select the correct passive form of the given the students. sentence. (b) A good teacher will never be forgotten by the His teachers have given him some books. students (a) Some books are given him some books (c) Good teachers are never forgotten by the (b) His teachers are giving him some books students (c) He was given some books by his teachers (d) A good teacher is never forgotten by the (d) Some books have been given to him by his students teachers SSC Stenographer Grade C&D –05/02/2019 (3:5pm) 1 SSC Stenographer Grade C&D –05/02/2019 (3:5pm) Ans. (b) : In the above sentence the use of "will + V " Ans. (d) : In the above sentence the use of have + makes it clear that the given sentence is in Active given (V3) makes it clear that the given sentence is in voice of Future Indefinite tense. Its Passive voice Active voice of Present Perfect tense. Its Passive voice structure will be– structure will be– A.V.- Subject+will+V1+obj+others. A.V.- Sub.+has/have+V3+obj.+others. P.V.- Object+will+be+V3+by+sub+others. P.V.- Obj.+has/have+been+V3+by+sub.+others. Correct Passive Voice– A good teacher will never be Note:- Passive voice with other words According to forgotten by the students. sense of the sentence placed "By + Subject" before or 185. Select the correct passive form of the given after. sentence. Correct Passive Voice– Some books have been given Who broke the cup? to him by his teachers. (a) By whom was the cup broken? 189. Select the correct passive form of the given (b) Was the cup broken by someone? sentence. (c) Who has broken the cup? My friend writes a letter to his father. (d) Was the cup broken by someone? (a) A letter is written by my friend to his father SSC Stenographer Grade C&D –05/02/2019 (3:5pm) (b) My friend wrote a letter to his father (c) A letter was written by my friend to his father Ans. (a) : In the above sentence the use of "broke" (d) A letter is being written by my friend to his (V2) makes it clear that the given sentence is Active father voice of Past Indefinite tense. Its Passive voice SSC Stenographer Grade C&D–05/02/2019 (3:5pm) structure will be– Active / Passive Voice

319

YCT

Ans. (a) : In the above sentence the use of "Writes" 193. Select the correct passive form of the given sentence : makes it clear that the given sentence is in Active The children had plucked all the flowers. voice of Present Indefinite tense. Its Passive voice (a) All the flowers has been plucked by the structure will be – children. A.V.- Sub.+V1+object+others. (b) All the flowers were plucked by the children. P.V.- Object+is/am/are+V3+by+sub.+others. (c) All the flowers are plucked by the children. Correct Passive Voice– A letter is written by my (d) All the flowers had been plucked by the friend to his father. children. 190. Select the correct passive form of the given SSC Stenographer Grade C&D–05/02/2019 (3:5pm) sentence. Ans. (d) : In the above sentence the use of "had + Can you read this book? plucked (V3)" makes it clear that the give sentence is in (a) Can you have read this book? Active Voice of Past Perfect tense. Its Passive voice (b) Could this book have been read by you? structure will be– (c) Can this book be read by you? A.V.- Sub.+had+verb3+object. (d) This book can be read by you. SSC Stenographer Grade C&D–05/02/2019 (3:5pm) P.V.- Object+had+been+verb3+by+subject. Correct Passive Voice– All the flowers had been Ans. (c) : In the above sentence modal verb "Can" is plucked by the children. used at the beginning of the sentence, hence it is an Interrogative sentence. Its Passive voice structure will 194. In the following question, a sentence has been be– given in Active/Passive voice. Out of the four A.V.- Can+sub.+V1+obj.+others? alternatives suggested, select the one which best P.V.- Can+obj.+be+V3+by+sub.? expresses the same sentence in Passive/Active Correct Passive Voice– Can this book be read by voice. you? You have to finish painting this fence by Friday. 191. Select the correct passive form of the given (a) This fence has to be finished painting by sentence. Friday. My brother won the Best Actor award in the (b) This fence is to be finishing painted by you inter-college drama competition. by Friday. (a) In the inter-college drama competition my (c) Painting of this fence has to be painted by brother won the Best Actor award. (b) The Best Actor award is being won by my you by Friday. brother in the inter-college drama competition (d) By Friday you will have finished painting the (c) The Best Actor award has been won by my fence. brother in the inter-college drama SSC CGL (Phase-II) 17/02/2018 competition. Ans. (a) The given Active voice sentence based on (d) The Best Actor award was won by my brother condition. Hence Passive voice change into– in the inter-college drama competition A.V.→ Sub. + has to / have to + V1st + object + other SSC Stenographer Grade C&D–05/02/2019 (3:5pm) words. 2 Ans. (d) : In the above sentence the use of won (V ) P.V.→ Obj. + have to / has to + be + V3rd + (by Sub) + makes it clear that the given sentence is in Active other words. voice of Past Indefinite tense. Its Passive voice Correct Passive Voice– structure will be– This fence has to be finished painting by Friday. A.V.- Sub.+V2+obj.+other words. 195 . The dealer will service the scooter for free the P.V.- Obj.+was/were+V3+by+Sub.+other words. first three times. Correct Passive Voice– The Best Actor award was (a) The scooter will be serviced for free by the won by my brother in the inter-college drama dealer for the first three times. competition. (b) The scooter was serviced for free by the 192. Select the correct passive form of the given dealer for the first three times. sentence. (c) For the first three times, the dealer will The doctor is treating the patient. service the scooter for free. (a) The patient is treated by the doctor (d) Servicing of the scooter for free will be done (b) The patient is being treated by the doctor by the dealer for the first three times. (c) The patient is being treating by the doctor SSC CGL (Phase-II) 17/02/2018 (d) The patient has been treated by the doctor (a) In the above sentence the use of "will + service SSC Stenographer Grade C&D –05/02/2019 (3:5pm) Ans. 1 Ans. (b) : In the above sentence the use of "is treating (V )" makes it clear that the given sentence is in Active (V + ing)" makes it clear that the given sentence is in voice of Simple Future tense. Its Passive voice structure Active voice of Present Continuous tense. Its Passive will be– A.V.→ Sub. + will/shall + V1st + object + other word. voice structure will be– P.V.→ Obj. + will be/shall be +V3rd + by + Sub. + other A.V.- Sub.+ is/am/are + verb1 + ing + object. P.V.- Object + is/am/are + being + verb3 + by + words. Correct Passive Voice– subject. Correct Passive Voice– The patient is being treated The scooter will be serviced for free by the dealer for the first three times. by the doctor. Active / Passive Voice

320

YCT

196.

The manager gave her a negative feedback during her performance appraisal. (a) A negative feedback was given to her by the manager during her performance appraisal. (b) The manager will give her negative feedback during her performance appraisal. (c) A negative feedback is being given to her by the manager during appraising her performance. (d) Giving of a negative feedback to her during her performance appraisal has been done by the manager. SSC CGL (Phase-II) 17/02/2018 Ans. (a) In the above sentence the use of "gave" (V2) makes it clear that the given sentence is in Active voice of Past Indefinite tense. Its Passive voice structure will be– A.V.→ Sub + V2 + object + other words. P.V.→ Obj + was/were + V3rd + by + Sub + other words. Correct Passive Voice– A negative feedback was given to her by the manager during her performance appraisal. 197. The General presented us a detailed battle plan. (a) A detailed battle plan will be presented to us by the General. (b) The General presents us a detailed battle plan. (c) A detailed battle plan was presented to us by the General. (d) Presenting of a detailed battle plan was being done by the General to us. SSC CGL (Phase-II) 17/02/2018 Ans. (c) In the above sentence the use of "presented" (V2) makes it clear that the given sentence is in Active voice of Past Indefinite tense. Its Passive voice structure will be– A.V.→ Sub + V2 + object + other words. P.V.→ Obj + was/were +V3rd + by sub + other words. Correct Passive Voice– A detailed battle plan was presented to us by the general. 198. For how long will you keep neglecting your duties? (a) For how long have you kept neglecting your duties? (b) For how long do your duties need to be neglected b y yourself? (c) For how long will your duties be kept neglected by you? (d) Neglecting of the duties by you have been going on since how long? SSC CGL (Phase-II) 17/02/2018 Ans. (c) In the above sentence the use of "Question word + will + you + keep (V1)" makes it clear that the given sentence is in Active voice of Simple Future tense. Its Passive voice structure will be – A.V.→ Question word+will/shall+sub.+verb1+object. P.V.→ Question word+will/shall+object+be verb3+by+ subject. Correct Passive Voice– For how long will your duties be kept neglected by you? Active / Passive Voice

199.

The doctor has given me a very painful injection. (a) A very painful injection has been given to me by the doctor. (b) The doctor gives myself a very painful injection. (c) A very painful injection is being given to me by the doctor. (d) Giving of a painful injection to me has been done by the doctor. SSC CGL (Phase-II) 17/02/2018 Ans. (a) In the above sentence the use of "has + given (V3)" makes it clear that the given sentence is in Active voice of Present perfect tense. Its Passive voice structure will be– A.V.→ Sub. + has/have + V3rd + object + other words. P.V.→ Obj. + has been/have been + V3rd + by + Sub. + other words. Correct Passive Voice– A very painful injection has been given to me by the doctor. 200. Uncle always welcomes us to his home with a big smile. (a) We are always welcomed by uncle to his home with a big smile. (b) Uncle is always welcoming us to his home with a big smile. (c) A big smile is used by uncle always when he is welcoming us. (d) Welcoming by uncle with a big smile to us is always done. SSC CGL (Phase-II) 17/02/2018 Ans. (a) In the above sentence the use of "welcomes" (V1 + s/es) makes it clear that the given sentence is in Active voice of Present Indefinite tense. Its Passive voice structure will be– A.V.→ Sub. + Verb(s/es) + object + other words. P.V.→ Obj. + is/am/are +Verb3rd + by + sub. + other words. Correct Passive Voice– We are always welcomed by uncle to his home with a big smile. 201. The speeding car knocked down the pedestrian. (a) The car which was speeding has knocked down the pedestrian. (b) The pedestrian was knocked down by the speeding car. (c) The pedestrian was being knocked down by the car which was speeding. (d) Knocking down of the pedestrian has been done by the speeding car. SSC CGL (Phase-II) 17/02/2018 Ans. (b). In the above sentence the use of "Knocked (V2)" makes it clear that the given sentence is in Active voice of Past Indefinite tense. Its Passive voice structure will be– A.V.→ Sub. + VerbIInd + object + other words. P.V.→ Obj. + was/were +VIIIrd + by Sub. + other words. Correct Passive Voice– The pedestrian was knocked down by the speeding car.

321

YCT

202.

I am sure she will like you. (a) I am sure you were liked by her. (b) She, I am sure shall like you. (c) Liking of you by her is a surety by me. (d) I am sure you will be liked by her. SSC CGL (Phase-II) 17/02/2018 Ans. (d). In the main clause of the above sentence the use of "Will + like (V1)" makes it clear that the given sentence is in Active voice of Future Indefinite tense. Its Passive voice structure will be– A.V.→ I am sure + Sub.+ will/shall + verbIst + Object + other words. P.V.→ I am sure + object + will be / shall be + VIIIrd + by + Sub + other words. Correct Passive Voice– I am sure you will be liked by her. 203. I rarely watch action movies. (a) Action movies are rarely watched by myself. (b) Action movies are rarely watched by me. (c) I had rarely watched action movies. (d) Watching of action movies is rarely d one by me. SSC CGL (Phase-II) 17/02/2018 Ans. (b) In the above sentence the use of "watch (V1)" makes it clear that the given sentence is in Active voice of Present Indefinite tense. Its Passive voice structure will be– A.V → Sub. + Verb (s + es) + Object + other words. P.V → Obj. + is/am/are + VIIIrd + by + Sub. + other words. Correct Passive Voice– Action movies are rarely watched by me. 204. His behaviour embarrasses me. (a) His behaviour is embarrassing for me. (b) I am embarrassed by his behaviour. (c) I am myself embarrassed by his behaviour. (d) Embarrassing of myself is done by his behaviour. SSC CGL (Phase-II) 17/02/2018 Ans. (b) In the above sentence the use of "embarrasses (V1 + s/es)" makes it clear that the given sentence is Active voice of Present Indefinite tense. Its Passive voice structure will be– A.V.→ Sub. + Verb (s + es) + Object + other words. P.V.→ Obj. + is/am/are + VIIIrd + by + Sub. + other words. Correct Passive Voice– I am embarrassed by his behaviour. 205. She ate the sweets greedily. (a) Greedily sweets are eaten by her. (b) She herself ate the sweets greedily. (c) Eating of the sweets was done by her greedily. (d) The sweets were eaten by her greedily. SSC CGL (Phase-II) 17/02/2018 Ans. (d) In the above sentence the use of "ate (V2)" makes it clear that the given sentence is in Active voice of Past Indefinite tense. Its Passive voice structure will be– A.V → Sub + VerbIInd + Object + other words. P.V→ Obj + was/were + VIIIrd + by + Sub + other words. Correct Passive Voice– The sweets were eaten by her greedily. Active / Passive Voice

206.

She will never tell you the truth. (a) You will never be told the truth by her. (b) She shall never be telling you the truth. (c) You were never be told the truth by her. (d) Telling of the truth to you be her will never be done. SSC CGL (Phase-II) 17/02/2018 Ans. (a) In the above sentence the use of "will + tell (V1)" makes it clear that the given sentence is in Active voice of Future Indefinite tense. Its Passive voice structure will be– Correct Passive Voice– A.V.→ Sub. + will/shall + verb1st + Object + other words. P.V.→ Obj. + will be/shall be + VIIIrd + by + Sub. + other words. Correct Passive Voice– You will never be told the truth by her. 207. You need to offer them a better deal. (a) They are in need of being offered a better deal by yourself. (b) They need to be offered a better deal by you. (c) A better deal is needed by them and you have to offer it. (d) The offering of a better deal has to be done by you for their need. SSC CGL (Phase-II) 17/02/2018 Ans. (b) In the above sentence "need to + VI" is used. It is clear that the given sentence is in Active Voice. Hence its Passive voice structure will be – A.V.→ Sub. + need to + verb1 + Object + other words. P.V.→ Obj. + need to + be + VIIIrd + by + Sub. + other words. Correct Passive Voice– They need to be offered a better deal by you. 208. You are making your life more difficult. (a) Your life is being made more difficult by you. (b) You yourself make your life more difficult. (c) Your life had been made more difficult by you yourself. (d) Making of your life more difficult has been done by yourself. SSC CGL (Phase-II) 17/02/2018 Ans. (a) In the above sentence the use of "are + making" makes it clear that the given sentence is in Active voice of Present Continuous tense. Its Passive voice structure will be – A.V.→ Sub. + is/am/are + verb (ing) + obj. + other words. P.V.→ Obj. + is/am/are + being + v3rd + by + sub. + other words. Correct Passive Voice– Your life is being made more difficult by you. 209. Mother needs your help in the kitchen. (a) In the kitchen mother needs help by yourself. (b) Your helping is needed by mother in the kitchen. (c) Your help is needed by mother in the kitchen. (d) Needing of your help is done by mother in the kitchen. SSC CGL (Phase-II) 17/02/2018

322

YCT

Ans. (c) In the above sentence the use of "needs (verb1 + s/es)" makes it clear that the given sentence is in Active voice of Present Indefinite tense. Its Passive voice structure will be– Rules to convert into P.V. A.V.→ Sub. + verb1 (s/es) + obj. + other words. P.V.→ Obj. + is/am/are + verb3rd + by + sub. + other words. Correct Passive Voice– Your help is needed by mother in the kitchen. 210. She always took her medicines on time. (a) The medicines was always took on time by her. (b) The medicines were always taken on time by her. (c) On time medicines she always took. (d) Taking of the medicines on time was being done by her. SSC CGL (Phase-II) 17/02/2018 Ans. (b). In the above sentence the use of "took (V2)" makes it clear that the given sentence is in Active voice of Past Indefinite tense. Its Passive voice structure will be– A.V.→ Sub. + verb (II form) + object + other words. P.V.→ Object + was/were + verb III form + by +sub. + other words. Correct Passive Voice– The medicines were always taken on time by her. 211. I have flown this plane for seven years. (a) This plane is flying me for seven years. (b) I am flying this plane for seven years. (c) Seven years have happened since I have been flying this plane. (d) This plane has been flown by me for seven years. SSC CGL (Phase-II) 17/02/2018 Ans. (d) In the above sentence the use of "have + flown (V3)" makes it clear that the given sentence is in Active voice of Present Perfect tense. Its Passive voice structure will be– A.V.→ Sub. + has/have + verb (III form) + object + other words. P.V.→ Obj + has/have + been + verb (III form) + by + sub. + other words. Correct Passive Voice– This plane has been flown by me for seven years. 212. Who has composed this song? (a) This song will be composed by whom? (b) Who composes this song? (c) Composing of this song ha s been done by who? (d) This song has been composed by whom? SSC CGL (Phase-II) 17/02/2018 Ans. (d) In the above sentence the use of "has + composed (V3)" makes it clear that the given sentence is in Active voice of Interrogative Sentence of Present Perfect tense. Its Passive voice structure will be– A.V.→ Who + has/have + verb (III form) + object + other words + ? P.V.→ Obj. + has/have + been + verb (III form) + by whom? Correct Passive Voice– This song has been composed by whom? Active / Passive Voice

213.

The organisers will give you a certificate. (a) A certificate will be given to you by the organisers. (b) The organisers had given you a certificate. (c) You were given a certificate by the organisers. (d) Giving of certificate to you was done by organisers. SSC CGL (Phase-II) 17/02/2018 Ans (a) In the above sentence the use of "will + give (V1)" makes it clear that the given sentence is in Active voice of Future Indefinite tense. Its Passive voice structure will be– A.V.→ Sub. + will/shall + verbI + obj. + other words. P.V.→ Obj. + will / shall + be + verb III + by + sub. + other words. Correct Passive Voice– A certificate will be given to you by the organizers. 214. Select the option that is the correct passive from of the given sentence Our boss appreciates all our efforts. (a) All our efforts were appreciated by our boss. (b) All our efforts will be appreciated by our boss (c) All our efforts are appreciated by our boss (d) All our efforts are appreciated by us. SSC Sel. Post Phase-VII (M.L.) 15.10.2019 (Shift-I)

Ans. (c) : In the above sentence the use of "appreciates (V1 + s/es)" makes it clear that the given sentence is in Active voice of Present Indefinite tense. Its Passive voice structure will be– A.V.- Subject + verb1 + (s/es) + object + other words P.V.- Object + is/am/are + verb3 + by + subject + other words. Correct Passive Voice– All our efforts are appreciated by our boss. 215. Select the correct passive form of the given sentence. Follow all the steps accurately. (a) All the steps should be following accurately (b) Let all the steps be follow accurately (c) Let all the steps be followed accurately (d) All the steps should be follow accurately SSC Sel. Post Phase-VIII (G.L.) 09.11.2020 (Shift-II)

Ans. (c) : In the beginning of the above sentence "verb1" makes it clear that the given sentence is in Active voice of Imperative sentence. Its Passive voice structure will be– A.V.- Verb + object + other words. P.V.- Let + object + be + verb3 + other words. Correct Passive Voice– Let all the steps be followed accurately. 216. Select the correct active form of the given sentence. Perfection is sought by him in everything he does. (a) He sought perfection in everything he did (b) He has sought perfection in everything he does (c) He seeks perfection in everything he does (d) He is seeking perfection in everything he does

323

SSC Sel. Post Phase-VIII (G.L.) 09.11.2020 (Shift-II)

YCT

Ans. (c) : In the above sentence the use of "is sought (V3)" makes it clear that the given sentence is in Passive voice of Present Indefinite tense. Its Active voice structure will be – P.V.- Object + is/am/are + verb3 + by + subject + other words. A.V.- Subject + verb1 + (s/es) + object + other words. Correct Active Voicehe seeks perfection in everything he does. 217. Select the correct passive form of the given sentence. The volunteers ensured that no one was hurt. (a) It was ensured by the volunteers that no one was hurt (b) It was to be ensured by the volunteers that no one was hurt (c) It was being ensured by the volunteers that no one was hurt (d) It had been ensured by the volunteers that no one was hurt SSC Sel. Post Phase-VIII (H.L.) 09.11.2020 (Shift-I)

Ans. (a) : In the above sentence the use of "ensured (V2)" makes it clear that the given sentence is in Active voice of Past Indefinite tense. Its Passive voice structure will be– A.V.- Subject + verb2 + object + other words. P.V.- Object (it) + was/were + verb3 + by + subject + other words Correct Passive Voice– It was ensured by the volunteers that no one was hurt. 218. Select the correct passive form of the given sentence. I have completed that project. (a) The project is completed by me (b) The project was completed by me (c) The project has been completed by me (d) The project is being completed by me SSC Sel. Post Phase-VIII (H.L.) 09.11.2020 (Shift-I)

Ans. (c) : In the above sentence the use of "have + V3" makes it clear that the given sentence is in Active voice of Present Perfect tense. Its Passive voice structure will be– A.V.- Sub. +has/have + v3 + obj P.V.- Obj. +has/have + been + v3 + by + sub Correct Passive Voice– The project has been completed by me. 219. Select the correct passive form of the given sentence. She did not accept the job of a clerk. (a) The job of a clerk has not accepted her (b) The job of a clerk did not accept her (c) The job of a clerk was not accepted by her (d) The job of a clerk is not accepted by her SSC Sel. Post Phase-VIII (M.L.) 09.11.2020 (Shift-III)

Ans. (c) : In the above sentence the use of "did not + verbI" makes it clear that the given sentence is in Active Voice of Past Indefinite tense. Its Passive voice structure will be– A.V.- Subject + did + not + verb1 + object + other words. P.V.- Object + was/were + not + verb3 + by + subject + other words. Correct Passive Voice– The job of a clerk was not accepted by her. Active / Passive Voice

220. Select the correct active form of the given sentence. The main gate of the building was being guarded by gun-totting guards. (a) The main gate of the building were guarding gun-totting guards. (b) Gun-totting guards were guarding the main gate of the building. (c) Gun-totting guards have been guarding the main gate of the building. (d) Gun-totting guards guarded the main gate of the building. SSC CGL (Tier-I) – 04/06/2019 (Shift-II) Ans : (b) In the above sentence the use of "was + being + V3" makes it clear that the given sentence is in Passive voice of Past Continuous tense. Its Active voice structure will beA.V- Sub. + was/were + VI + ing + obj. P.V- Obj. + was/were + being + VIII + by + sub. Correct Active VoiceGun-totting guards were guarding the main gate of the building. 221. Select the correct passive form of the given sentence. Show these visitors the butterfly farm. (a) These visitors should be showed the butterfly farm. (b) Let these visitors be showing the butterfly farm. (c) You must show these visitors the butterfly farm. (d) Let these visitors be shown the butterfly farm. SSC CGL (Tier-I) – 06/06/2019 (Shift-III) Ans. (d) : In the beginning of the above sentence "v1 (show)" makes it clear that the given sentence is in Active voice of Imperative sentence. Its Passive voice structure will be– A.V.- Verb1 + obj. + other words P.V.- Let + obj. + be + v3 + other words Correct Active Voice– Let these visitors be shown the butterfly farm. 222. Select the correct active form of the given sentence. The wall of this park is being painted by children. (a) Children are painting the wall of the park. (b) The wall is painting children of this park. (c) Children have been painting the wall of this park. (d) Children painted the wall of this park. SSC CGL (Tier-I) – 06/06/2019 (Shift-II) Ans : (a) In the above sentence the use of "is + being + painted" makes it clear that the given sentence is in Passive voice of Present Continuous tense. Its Active voice structure will be– P.V.- Obj. + is/am/are + being + VIIIrd form + by + sub. A.V.- Sub. + is/am/are + V1 + ing + obj. Correct Active Voice– Children are painting the wall of the park. 223. Select the correct passive form of the given sentence. Please take the guest to his room on the 6th floor.

324

YCT

(c) The glass table should be rubbed with a soft (a) The guest should be took to his room on the cloth to make it shine. 6th floor. (d) The glass table have to be rubbed with a soft (b) You are requested to take the guest to his cloth to make it shine. room on the 6th floor. (c) You must take the guest to his room on the SSC CGL (Tier-I) – 19/06/2019 (Shift-III) 6th floor Ans. (c) : In the beginning of the sentence the use of (d) Let the guest be taking to his room on the 6th "Rub (VI)" makes it clear that the given sentence is in floor Active voice of Imperative sentence. Its Passive SSC CGL (Tier-I) – 07/06/2019 (Shift-II) structure will be– IIIrd Ans : (b) In the above sentence the use of "Please take P.V.- Obj. + should + be + V + other word. (verb1)" makes it is clear that the given sentence is in Correct Passive Voice– Imperative sentence which shows 'request' Hence, its The glass table should be rubbed with a soft cloth to make it shine. Passive voice structure will be– P.V.- You are requested + to + V1st form + obj. + other 227. Select the correct active form of the given words. sentence. Correct Passive Voice– The results will be known to us by the end of You are requested to take the guest to his room on the the next month. 6th floor. (a) We have known the results by the end of the next month. 224. Select the correct active form of the given (b) We will know the results by the end of the sentence. next month. One of the passengers was being thoroughly (c) We are knowing the results by the end of the checked by the custom officers. next month. (a) The custom officers have been thoroughly (d) We know the results by the end of the next checking one of the passengers. month. (b) The custom officers thoroughly checked one SSC CGL (Tier-I)–19/06/2019 (Shift-III) of the passengers. (b) : In the above sentence the use of "will be + (c) One of the passengers was thoroughly Ans. V3" makes it clear that the given sentence is in Passive checking the custom officers. (d) The custom officers were thoroughly voice of Future Indefinite tense. Its Active voice structure will be– checking one of the passengers. SSC CGL (Tier-I)–07/06/2019 (Shift-I) Passive Structure IIIst Ans : (d) In the above sentence the use of "was + being Obj. + shall/will + be + V form + by + sub. Active Structure + checked (V3)" makes it clear that the given sentence is Ist in Passive voice of Past continuous tense. Its Active Sub. + shall/will + V form + obj. Correct Passive Voicevoice structure will be– A.V.- Subject + was/were + (VI + ing) + object. We will know the results by the end of the next month. Correct Active Voice– 228. Select the correct active form of the given The custom officers were thoroughly checking one of sentence. the passengers. He was given a book for his birthday. (a) She gave him a book for his birthday. 225. Select the correct passive form of the given (b) She will be giving him a book for is birthday. sentences. (c) She has gave him a book for his birthday. They opened a new mall nearby last month. (d) She give a book to him for his birthday (a) A new mall can be open nearby last month SSC CGL (Tier-I) – 13/06/2019 (Shift-I) (b) A new mall was opened nearby last month (c) A new mall will be opened nearby last month Ans : (a) In the above sentence the use of "was + V3" (d) Nearby a new mall is opened last month. makes it clear that that the given sentence is in Passive SSC CGL (Tier-I) – 12/06/2019 (Shift-III) voice of Past Indefinite tense. Its Active voice structure will be– Ans. (b) : In the above sentence the use of opened (V2) P.V.- Object + was/were + v3rd + subject + others. makes it clear that the given sentence is in Active A.V.- Understood subject + v2 + obj. + others. voice of Past Indefinite tense. Its Passive voice Correct Active Voice– structure will be– She gave him a book for his birthday. A.V.- Sub. + V2 + object + other words. P.V.- Object + was/were + V3 + by + sub. + other 229. Select the correct passive form of the given sentence. words. The artist played the violin. Correct Passive Voice– (a) The violin is being played by the artist. A new mall was opened nearby last month. (b) The violin will be played be the artist. 226. Select the correct passive form of the given (c) The violin had been played by the artist. sentence. (d) The violin was played by the artist. Rub the glass table with a soft cloth to make it SSC CGL (Tier-I) – 13/06/2019 (Shift-I) shine. 2 (a) The soft cloth should be rubbed with a glass Ans : (d) In the above sentence the use of "Sub + V " makes it clear that the given sentence is in Active table to make it shine. (b) The glass table must rub with a soft cloth to voice of Past Indefinite tense. Its Passive voice structure will be– make it shine. Active / Passive Voice

325

YCT

Active - Subject + verbIInd form + object. Passive - Object + was/were + vIIIrd form + by + subject. Correct Passive Voice– The violin was played by the artist. 230. Select the correct active form of the given sentence. Rudra was laughed at by all his friends when he wore his socks inside-out. (a) All his friends laughed at Rudra when he wore his socks inside-out. (b) All his friends will be laughing at Rudra for wearing his socks inside-out. (c) If Rudra wears his socks inside-out his friends will laugh at him. (d) When Rudra wore his socks inside-out all his were laughing at him. SSC CGL (Tier-I) – 13/06/2019 (Shift-II) Ans. (a) : In the above sentence the use of "was + VIIIrd" makes it clear that the given sentence is in Passive voice of Past Indefinite tense. Its Active voice structure will be– P.V.- Object + was/were + VIIIrd + by + subject. A.V.- Sub. + VIInd + object. Correct Active VoiceAll his friends laughed at Rudra when he wore his socks inside-out. 231. Select the correct passive form of the given sentence. They will put away their woollens after the festival of Holi. (a) Their woollens have been put away after the festival of Holi. (b) Their woollens are being put away after the festival of Holi. (c) Their woollens will be put away after the festival of Holi. (d) Their woollens are put away after the festival of Holi. SSC CGL (Tier-I) – 13/06/2019 (Shift-II) Ans. (c) : In the above sentence the use of "Subject + will + V1" makes it clear that the given sentence is Active voice of Future Indefinite tense. Its Passive voice structure will be– A.V.- Sub. + will/shall + VI + object. P.V.- Obj. + will be/shall be + V3 + by + subject. Correct Passive Voice– Their woollens will be put away after the festival of Holi. 232. Select the correct active form of the given sentence. Dinesh was looked after by his grandmother when his parents went abroad. (a) Dinesh's grandmother looks after him when his parents went abroad. (b) Dinesh's grandmother looked after him when his parents went abroad. (c) Dinesh looked after his grandmother when his parents went abroad. (d) Dinesh's grandmother was looking after him when his parents went abroad. SSC CGL (Tier-I) – 13/06/2019 (Shift-III) Active / Passive Voice

Ans : (b) In the above sentence the use of "was + VIIIrd" makes it clear that the given sentence is in Passive Voice of Past Indefinite tense. Its Active voice structure will be– P.V.- Obj. + was/were + v3 + by + sub.+other words. A.V.- Sub. + v2 + obj. + other words. Correct Active VoiceDinesh's grandmother looked after him when his parents went abroad. 233. Select the correct passive form of the given sentence. Ananya plucks fresh flowers from the garden every day. (a) Fresh flowers have been plucked by Ananya from the garden every day. (b) Fresh flowers were plucked by Ananya from the garden every day. (c) Fresh garden is plucked by Ananya from the flowers every day. (d) Fresh flowers are plucked by Ananya from the garden every day. SSC CGL (Tier-I) – 13/06/2019 (Shift-III) Ans : (d) In the above sentence the use of (Subject + V1 + s/es) makes it clear that the given sentence is in Active voice of Present Indefinite tense. Its Passive voice structure will be – A.V.- Sub. + v1 + (e/es) + obj. P.V.- Obj. + is/am/are + vIIIrd + by + sub. Correct Passive Voice– Fresh flowers are plucked by Ananya from the garden every day. 234. Select the correct passive form of the given sentence. Credit cards are replacing cash transactions. (a) Cash trensactions had been replaced by credit cards. (b) Cash transactions have been replaced by credit cards. (c) Cash transactions are being replaced by credit cards. (d) Cash transactions are replaced by credit cards. SSC CGL (Tier-I) – 12/06/2019 (Shift-I) Ans : (c) In the above sentence the use of "are + v1 + ing" makes it clear that the given sentence is in Active voice of Present Continuous tense. Its Passive voice structure will be– A.V.- sub. + is/am/are + V1 + ing + obj. P.V.- obj. + is/am/are + being + V3 + by + sub. Correct Passive Voice– Cash transactions are being replaced by credit cards. 235. Select the correct active form of the given sentence. We were given very little time to prepare the presentation. (a) They have given us very little time to prepare the presentation. (b) They gave us very little time to prepare the presentation. (c) They will give us very little time to prepare the presentation. (d) They give us very little time to prepare the presentation. SSC CGL (Tier-I) – 12/06/2019 (Shift-I)

326

YCT

Ans : (b) In the above sentence the use of "Were + V3" Ans. (b) : In the above sentence the use of "by whom makes it clear that the given sentence is in Passive voice + were + V3" makes it clear that the given sentence is of Past Indefinite tense. Its Active voice structure will in Passive voice of interrogative from of Past be – Indefinite tense. Its Active voice structure will be – P.V.- Sub. + was/were + VIIIrd + object + others. P.V.- By whom+was/were+ sub.+ v3+ other words. A.V.– Object +V2 + Subject + other words. A.V.- Who + v2+ obj + other words. Correct Active Voice239. Select the correct passive form of the given They gave us very little time to prepare the presentation. sentence. 236. Choose the option that is the passive form of The sailors had never encountered such a the sentence. rough sea. A campus fire in California caused the death of (a) Such a rough sea is never encountered by the at least twenty-three persons. sailors. (a) The death of at least twenty three persons (b) Such a rough sea had never been encountered caused a campuse fire in California by the sailors. (b) The death of at least twenty three persons (c) Such a rough sea has never been encountered will be caused in a campus fire in California. by the sailors. (c) At least twenty-three persons' death was (d) Such a rough sea was never encountered by caused in a campus fire in California. (d) The death of at least twenty three persons the sailors. was caused by a campus fire in California. SSC CGL (Tier-I) – 11/06/2019 (Shift-I) SSC CGL (Tier-I) – 12/06/2019 (Shift-II) Ans. (b) : In the above sentence the use of "had + v3" Ans : (d) In the above sentence the use of "Subject + makes it clear that the given sentence is in Active V2" makes it clear that the given sentence is Active voice of Past Perfect tense. Its Passive voice structure voice of Past Indefinite tense. Its Passive voice structure will be– will be– Active structure Active Structure: Sub. + had + vIIIrd form + obj. IInd Subject + V form + Object. + other words. Passive structure Passive Structure: Obj. + had been + vIIIrd form + by + sub. IIIrd Obj. + was/were + V form + by + sub. + other words. Correct Passive Voice– The death of at least twenty Correct Passive Voice– three persons was caused by a campus fire in California. Such a rough sea had never been encountered by the sailors. 237. Choose the option that is the active form of the 240. Select the correct active form of the given sentence. sentence. It was decided by the members that the report We will all be greatly benefitted by this scheme. would be placed before the Chairman for his (a) This scheme will greatly benefit us all. comments. (b) This scheme has greatly benefitted us all. (a) The Chairman's comments were to be placed on the report of the members. (c) This scheme is going to greatly benefit us all. (b) Did the members decide to place the report (d) This scheme would greatly benefit we all. before the Chairman? SSC CGL (Tier-I) – 11/06/2019 (Shift-I) (c) The Chairman decided to place the report Ans. (a) : In the above sentence the use of "will be + before the members. v3" makes it clear that the given sentence is in Passive (d) The members decided to place the report voice of Future Indefinite tense. Its Active voice before the Chairman for his comments. structure will be– SSC CGL (Tier-I) – 12/06/2019 (Shift-II) Passive Structure– Ans : (d) In the above sentence the use of "was + V3" Obj. + shall/will + be + vIIIrd form + by + sub. makes it clear that the given sentence is in Passive voice of Past Indefinite tense. Its Active voice structure will Active structure Ist Sub. + shall/will + v form + obj. be– Correct SentencePassive StructureIIIrd This scheme will greatly benefit us all. Obj. + was/were + V form + by + sub. Active Structure 241. Select the correct passive form of the given Subject + VIInd form + Object. sentence. Correct Active Voice– She never trusted anyone. The members decided to place the report before the (a) Anyone is not trusted by her. Chairman for his comments. (b) She was never trusted by anyone. 238. Select the correct active form of the given (c) No one was ever trusted by her. sentences. (d) Her trust was never for anyone. By whom were you taught Mathematics ? SSC CGL (Tier-I) – 11/06/2019 (Shift-II) (a) Who will teach you Mathematics? Ans. (c) : In the above sentence the use of "Subject + (b) Who taught you Mathematics? V2" makes it clear that the given sentence is in Active (c) Who teaching you Mathematics? voice of Past Indefinite tense. Its Passive voice (d) Whom are you teaching Mathematics? SSC CGL (Tier-I) – 12/06/2019 (Shift-III) structure will be– Active / Passive Voice

327

YCT

P.V.- Sub.+was/were+V3+ by+obj. Note– While making 'anyone' Passive, 'No one' will be written and 'never' will be changed to 'ever'. Correct Passive VoiceNo one was ever trusted by her. 242. Select the correct active form of the given sentence. Heavy taxes have been imposed on luxury items by the government. (a) The government had imposed heavy taxes on luxury items. (b) The government imposed heavy taxes on luxury items. (c) The government is imposing heavy taxes on luxury items. (d) The government has imposed heavy taxes on luxury items. SSC CGL (Tier-I) – 11/06/2019 (Shift-II) Ans. (d) : In the above sentence the use of "have been + V3" makes it clear that the given sentence is in Passive voice of Present Perfect tense. Its Active voice structure will be– Passive Structure Obj. + has/have + been + vIIIrd form + by + sub. Active Structure Sub. + has/have + vIIIrd form + obj. Correct Active Voice– The government has imposed heavy taxes on luxury items. 243. Select the correct passive form of the given sentence It is time to serve dinner. (a) It is time for dinner to be served. (b) It is time that dinner is serve. (c) It is time so dinner should be served. (d) It is dinner serving time. SSC CGL (Tier-I) – 11/06/2019 (Shift-III) Ans. (a) : The given sentence is in Active voice of Imperative sentence. Its Passive voice structure is – It is time + object (for dinner) + to + be + VIII form (served). Note:- "By + subject" is not used in Imperative sentence. Correct Passive VoiceIt is time for dinner to be served. 244. Select the correct active form of the given sentence. Superstitions are still believed in by people. (a) People still believed in superstitions. (b) People still believe in superstitions. (c) People are still believing in superstitions. (d) People have still believed in superstitions. SSC CGL (Tier-I) – 11/06/2019 (Shift-III) Ans. (b) : In the above sentence the use of "are + V3" makes it clear that the given sentence is in Passive voice of Present Indefinite tense. Its Active voice structure will be– Active Structure Sub. + VIst form + obj. Passive Structure Obj. + is/am/are + VIIIrd form + by + sub. Correct Active Voice– People still believe in superstitions. Active / Passive Voice

245.

Select the correct active form of the given sentence. She was seen sitting in the last row. (a) We see her sitting in the last row (b) We saw her sitting in the last row (c) We had seen her sitting in the last row (d) We have seen her sitting in the last row SSC CGL (Tier-I) – 10/06/2019 (Shift-I) Ans : (b) In the above sentence the use of "was + V3" makes it clear that the given sentence is Passive voice of Past Indefinite tense. Its Active voice structure will be– P.V.→ Obj. + was/were + v3 + by + subject. A.V.→ Sub. + v2 + object. Correct Active VoiceWe saw her sitting in the last row. 246. Select the correct passive form of the given sentence. The enemy will have seized the fort before nightfall. (a) The enemy will be seized by the fort before nightfall. (b) The fort would have been seized by the enemy before nightfall. (c) The fort will have been seized by the enemy before nightfall. (d) The enemy will seize the fort before nightfall. SSC CGL (Tier-I) – 10/06/2019 (Shift-I) Ans : (c) In the above sentence the use of "will have + V3" makes it clear that the given sentence is in Active voice of Future Perfect tense. Its Passive voice structure will be– Active– Sub. + will/shall + have + verbIIIrd + object. Passive– Obj. + will/shall + have been + verbIIIrd + by + subject. Correct Passive Voice– The fort will have been seized by the enemy before nightfall. 247. Select the correct active form of the given sentence. Let the bell be rung every forty minutes. (a) The bell should be rung every forty minutes (b) Ring the bell every forty minutes (c) The bell ought to be rung every forty minutes (d) Let the bell keep ringing every forty minutes SSC CGL (Tier-I) – 10/06/2019 (Shift-II) Ans : (b) In the above sentence the use of "Let + obj + be + V3" makes it clear that the given sentence is in Passive voice of imperative sentence. Passive voice structure will be– A.V.- VIst + object + others. P.V.- Let + obj + be + v3 + others. Correct active VoiceRing the bell every forty minutes. 248. Select the correct passive form of the given sentence. He is clicking good pictures with his new camera. (a) Good pictures were clicked with his new camera. (b) Good pictures have been clicked with his new camera.

328

YCT

(c) Good pictures are being clicked with his new camera. (d) Good pictures are clicked with his new camera. SSC CGL (Tier-I) – 10/06/2019 (Shift-II) Ans : (c) In the above sentence the use of "is + verb + ing" makes it clear that the given sentence is in Active voice of Present Continuous tense. Its Passive voice structure will be– A.V.- Sub. + is/am/are + V1 + ing + obj. P.V.- Obj. + is/am/are + being + V3 + by + sub. Correct Passive VoiceGood pictures are being clicked with his new camera. 249. Select the correct passive form of the given sentence. Have they announced the world cup cricket team? (a) Have the world cup cricket team announced? (b) Have the world cup cricket team been announced? (c) Has the world cup cricket team being announced? (d) Has the world cup cricket team been announced? SSC CGL (Tier-I) – 10/06/2019 (Shift-III) Ans. (d) : In the above sentence the use of "Have + Subject + V3" makes it clear that the given sentence is in Active voice of Interrogative of Present Perfect tense. Its Passive voice structure will be– Passive- Has/have + obj. + been + V3 + by + sub. Correct Passive VoiceHas the world cup cricket team been announced? 250. Select the correct active form of the given sentence. You will either be taken prisoner or shot by the enemy. (a) The enemy will be either taking you prisoner or shooting you. (b) The enemy would either take you prisoner or shoot. (c) The enemy will either take you prisoner or shoot you. (d) The enemy either takes you prisoner or shoots you. SSC CGL (Tier-I) – 10/06/2019 (Shift-III) Ans. (c) : In the above sentence the use of "will + be + V3" makes it clear that the given sentence is in Passive voice of Future Indefinite tense. Its Active voice structure will be– A.V.- Sub. + will/shall + VIst + object. P.V.- Obj. + will/shall + be + VIIIrd + by + sub. Correct Active Voice– The enemy will either take you prisoner or shoot you. 251. Select the correct passive form of the given sentence. Please take these students round the biscuit factory. (a) These students should be taking round the biscuit factory. (b) You are requested to take these students round the biscuit factory. Active / Passive Voice

(c) Let these students to be taken round the biscuit factory. (d) You must take these students round the biscuit factory. SSC CGL (Tier-I) – 07/06/2019 (Shift-I) Ans : (b) In the above sentence the use of "Please + V1" makes it clear that the sentence is in Imperative sentence which shows request. Hence, Its Passive voice structure will be– Active Structure- Please + V1 + obj. + others. Passive Structure- You are requested + to + verb1 + object + other words. Correct Passive Voice– You are requested to take these students round the biscuit factory. 252. Select the correct active form of the given sentence. The chairs were being arranged in the examination hall by the staff. (a) The chairs were arranging the staff in the examination hall. (b) The staff was arranging the chairs in the examination hall. (c) The staff has been arranging the chairs in the examination hall. (d) The staff has arranged the chairs in the examination hall. SSC CGL (Tier-I)–07/06/2019 (Shift-II) Ans : (b) In the above sentence the use of "were +being + v3" makes it clear that the given sentence is in Passive voice of Past Continuous tense. Its Active voice structure will be– P.V.- Obj. + was/were + being + V3 + by + sub. A.V.- Sub. + was/were + V1 + ing + obj. Correct Active Voice– The staff was arranging the chairs in the examination hall. 253. Select the correct active form of the given sentence. The hunchback was being laughed at by everyone. (a) Everyone was laughing at the hunchback. (b) Everyone is laughing at the hunchback. (c) Everyone laughed at the hunchback. (d) Everyone laughs at the hunchback. SSC CGL (Tier-I) – 07/06/2019 (Shift-III) Ans : (a) In the above sentence the use of "was + being + V3" makes it clear that the given sentence is in Passive voice of Past Continuous tense. Its Active voice structure will be– P.V.- Object + was/were + being + V3 + by + sub. A.V.- Sub + was/were + V1 + ing + obj. Correct Active VoiceEveryone was laughing at the hunchback. 254. Select the correct passive form of the given sentence. Some people believe that discipline means blind submission to authority. (a) It is belief by some people that discipline means blind submission to authority (b) It was believed by some people that discipline meant blind submission to authority.

329

YCT

(c) It is believed by some people that discipline means blind submission to authority. (d) It has been believed by some people that discipline means blind submission to. SSC CGL (Tier-I) – 07/06/2019 (Shift-III) Ans : (c) In the above sentence the use of "Subject + V1" makes it clear that the given sentence is in Active voice of Present Indefinite tense. Its Passive voice structure will be– Active Voice- Sub. + V1 + Object. Passive Voice- Obj. + is/am/are + V3 + by + Sub. Correct Passive VoiceIt is believed by some people that discipline means blind submission to authority. 255. Select the correct passive form to the given sentence. At night, lock the outer gate. (a) The outer gate is requested to be locked at night. (b) The outer gate is locked at night. (c) The outer gate be locked at night. (d) Let the outer gate be locked at night. SSC CGL (Tier-I) – 06/06/2019 (Shift-I) Ans. (d) : The above sentence is in Active voice of Imperative sentence, Its Passive voice structure will be– A.V.- V1 + obj. + other words. P.V.- Let + obj. + be + V3rd + object + other words. Correct Passive Voice– Let the outer gate be locked at night. 256. Select the correct active form of the given sentence. Their children were brought up with great care. (a) They have been bringing up their children with great care. (b) They had brought up their children with great care. (c) They brought up their children with great care. (d) Their children brought them up with great care. SSC CGL (Tier-I) – 06/06/2019 (Shift-I) Ans. (c) : In the above sentence the use of "were + V3" makes it clear that the given sentence is in Passive voice of Past Indefinite tense. Its Active voice structure will be– P.V.- Object + was/were/+ VIIIrd + by + subject. A.V.- Sub + VIInd + object. Correct Active Voice– They brought up their children with great care. 257. Select the correct passive form of the given sentence. Take this suitcase upstairs. (a) You are requested to take this suitcase upstairs. (b) You must be taking this suitcase upstairs. (c) This suitcase should be taken upstairs. (d) Let this suitcase been taken upstairs. SSC CGL (Tier-I)–06/06/2019 (Shift-II) Active / Passive Voice

Ans : (c) In the beginning of the sentence "V1" makes it clear that the given sentence is in Imperative Sentence. Its Passive voice structure will be– A.V.- V1 + obj. + others. P.V.- Object + should be + verb (III) + others. Correct Passive Voice– This suitcase should be taken upstairs. 258. Select the correct active form of the given sentence. The injured were being removed to the hospital by the police. (a) The police removed the injured to the hospital. (b) The injured was removing the police to the hospital. (c) The police was removing the injured to the hospital. (d) The police has been removing the injured to the hospital. SSC CGL (Tier-I) – 06/06/2019 (Shift-III) Ans. (c) : In the above sentence the use of "was + being + V3" makes it clear that the given sentence is Passive voice of Past Continuous tense. Its Active voice structure will be– P.V.- Obj. + was/were + being + V3 + by + sub. A.V.- Sub. + was/were + V1 + ing + obj. Correct Active Voice– The police was removing the injured to the hospital. 259. Select the correct active form of the given sentence. Every passing vehicle was being thoroughly checked by the guards. (a) The guards have been thoroughly checking every passing vehicle. (b) Every passing vehicle were thoroughly checking the guards. (c) The guards have thoroughly checked every passing vehicle. (d) The guards were thoroughly checking every passing vehicle. SSC CGL (Tier-I) – 04/06/2019 (Shift-I) Ans : (d) In the above sentence the use of "was + being + V3" makes it clear that the given sentence is in Passive voice of Past Continuous tense. Its Active voice structure will be– P.V.- Obj. + was/were + being + VIIIrd + by + sub. A.V.- Sub. + was/were + VI + ing + object. Correct Active Voice– The guards were thoroughly checking every passing vehicle. 260. Select the correct passive form of the given sentence. Do not buy medicines without the doctor's prescription. (a) Medicines could not be bought without the doctor's prescription (b) Medicines need not be bought without the doctor's prescription (c) Medicines should not be bought without the doctor's prescription (d) Medicines might not be bought without the doctor's prescription SSC CGL (Tier-I) – 04/06/2019 (Shift-I)

330

YCT

Ans : (c) In the beginning of above sentence the use of "Do not + V1" makes it clear that the given sentence is in Imperative Sentence. Its Passive voice structure will be– A.V.- Do not + V1 + obj. + others. P.V.- Obj. + should + not + be + V3 + other words. Correct Passive VoiceMedicines should not be bought without the doctor's Prescription. 261. Select the correct passive form of the given sentence. Do not park your car in front of my house. (a) My house should not be parked in front of your car (b) Your car need not be parked in front of my house. (c) Your car could not be parked in front of my house. (d) Your car should not be parked in front of my house. SSC CGL (Tier-I) – 04/06/2019 (Shift-II) Ans : (d) In the above sentence the use of "Do not + V1" makes it clear that the sentence is in Imperative Sentence. Its Passive voice structure will be– A.V.- Do not + v1 + obj. + others. P.V.- Obj. + should + not + be + V3 + other words. Correct Passive VoiceYour car should not be parked in front of my house. 262. Select the correct passive form of the given sentence. Please show me my son's Mathematics notebook. (a) I may please be shown my son's Mathematics notebook. (b) My son may please be shown the Mathematics notebook. (c) I will please be shown my son's Mathematics notebook. (d) My son's Mathematics notebook was please shown to me. SSC CGL (Tier-I) – 04/06/2019 (Shift-III) Ans. (a) : The use of ''please show' (V1) in the starting of the sentence makes it clear that the sentence is Active Voice of imperative sentence and its passive voice structure will beA.V.- (Please) + (V1) + obj. + other words. P.V.- obj. + modal verb (may) + please + be + verb3 + other words. Correct Passive VoiceI may please be shown my son's Mathematics notebook. 263. Select the correct active form of the given sentence. The crop was adversely affected by the inadequate rainfall. (a) The inadequate rainfall adversely affected the crop. (b) The inadequate rainfall was adversely affecting the crop. (c) The inadequate crop adversely affected the rainfall. (d) The adversely rainfall has affected the inadequate crop. SSC CGL (Tier-I) – 04/06/2019 (Shift-III) Active / Passive Voice

Ans. (a) : In the above sentence the use of "was + V3" makes it clear that the given sentence is in Passive voice of Past Indefinite tense. Its Active voice structure will be– Active structure- Sub. + VIInd form + obj. Passive structure- Obj. + was/were + VIIIrd form + by + sub. Correct Active VoiceThe inadequate rainfall adversely affected the crop. 264. Select the correct passive form of the given sentence. Someone gave him a new case for his credit cards. (a) He is given a new case for his credit cards. (b) He had given a new case for his credit cards. (c) He was given a new case for his credit cards. (d) He has given a new case for his credit cards. SSC CHSL (Tier-I) –11/07/2019 (Shift-I) Ans : (c) In the above sentence the use of "Sub + V2" makes it clear that the given sentence is in Active voice of Past Indefinite tense. Its Passive voice structure will be– Active Structure- Sub + v2nd + object. Passive Structure- Obj. + was/were + v3rd + by + subject. Correct Passive VoiceHe was given a new case for his credit cards. 265. Select the correct passive form of the given sentence. The cyclone has damaged several crops. (a) Several crops are being damaged by the cyclone. (b) Several corps had been damaged by the cyclone. (c) Several crops have been damaged by the cyclone. (d) Several crops have damaged the cyclone. SSC CHSL (Tier-I) –11/07/2019 (Shift-II) Ans : (c) In the above sentence the use of "has + V3" makes it clear that the given sentence is in Active voice of Present Perfect tense. Its Passive voice structure will be– Active structure- Sub. + has/have + VIIIrd form + obj. Passive structure- Obj. + has/have + been + VIIIrd form + by + sub. Correct Passive VoiceSeveral crops have been damaged by the cyclone. 266. Select the correct passive form of the given sentence. Everyone should obey traffic regulations. (a) Traffic regulation will be obeyed by everyone. (b) Traffic regulations should be obey by everyone. (c) Traffic regulations are obeyed by everyone. (d) Traffic regulations should be obeyed by everyone. SSC CHSL (Tier-I) –11/07/2019 (Shift-III) Ans : (d) While making Modal verb 'should' a Passive form, 'be' is used after 'should' and the 'subject' is the 'object' and 'object' into the subject. Changed in Passive VIII form of verb is used and finally by is used before subject.

331

YCT

A.V.- Sub. + should + V1 + obj. P.V.- Obj. + should + be + VIIIrd + by + sub. Correct Passive VoiceTraffic regulations should be obeyed by everyone. 267. Select the correct passive form of the given sentence. You have made a mistake in writing your answer. (a) A mistake has been made by you in writing your answer. (b) A mistake have been made by you in writing your answer. (c) A mistake is being made by you in writing your answer. (d) A mistake had been made by you in writing your answer. SSC CHSL (Tier-I) –10/07/2019 (Shift-I) Ans : (a) In the above sentence the use of "have + V3" makes it clear that the given sentence is in Active voice of Present Perfect tense. Its Passive voice structure will be– A.V.- Subject + has/have + verb3rd form + object + others. P.V.- Object + has/have + been + verb3rd form + by + subject + others. Correct Passive VoiceA mistake has been made by you in writing your answer. 268. Select the correct passive form of the given sentence. His mother gave him a note for his teacher. (a) He had been given a note for his teacher by his mother. (b) He is given a note for his teacher by his mother. (c) He was given a note for his teacher by his mother. (d) His teacher was given a note for his mother. SSC CHSL (Tier-I) –10/07/2019 (Shift-II) Ans : (c) In the above sentence the use of "gave (V2)" makes it clear that the given sentence is in Active voice of Past Indefinite tense. Its Passive voice structure will be– A.V.- Subject + VIInd + object1 + object2 + others. P.V.- Object (indirect) + was/were+VIIIrd + object2 (direct) + others + by + subject. Note:– Sometimes in the case of two objects being given in the sentence Indirect object (animate) and Direct object (inanimate). Indirect object (animate) as subject of Passive is considered better. Correct Passive VoiceHe was given a note for his teacher by his mother. 269. Choose the option that is the passive form of the sentence. The chief guest gave prizes to the children. (a) The children were being given prizes by the chief guest. (b) The children are given prizes by the chief guest. (c) The children were given prizes by the chief guest. (d) The children was given prizes by the chief guest. SSC CHSL (Tier-I) –10/07/2019 (Shift-III) Active / Passive Voice

Ans : (c) In the above sentence the use of "gave (V2)" makes it clear that the given sentence is in Active voice of Past Indefinite tense. Its Passive voice structure will be– Active- Subject + verb (2nd form) + object + others. Passive- Object + was/were + verb(3rd form) + by + subject + others. Correct Passive VoiceThe children were given prizes by the chief guest. 270. Select the correct passive form of the given sentence. They will complete the painting in a week's time. (a) The painting will be completed in a week's time. (b) The painting was completed in a week's time. (c) The painting is completed in a week's time. (d) The painting had been completed in a week's time. SSC CHSL (Tier-I) –09/07/2019 (Shift-I) Ans : (a) In the above sentence the use of "will + V1" makes it clear that the given sentence is in Active voice of Future Indefinite tense. Its Passive voice structure will be– Active- Sub. + shall/will + v1 + obj. Passive- Obj. + shall/will + be + v3 + by + sub. Note- Sometimes 'by + subject' is understood. Correct Passive VoiceThe painting will be completed in a week's time 271. Select the correct passive form of the given sentence. They are going to open the shops at eleven o'clock. (a) The shops must be opened at eleven o'clock. (b) The shops were opened at eleven o'clock. (c) The shops are going to be opened at eleven o'clock. (d) The shops have been opened at eleven o'clock. SSC CHSL (Tier-I) –09/07/2019 (Shift-II) Ans : (c) The above sentence is in Active voice of Present continuous tense. Its Passive voice structure will be– Active- Subject + is/am/are + verb1 + ing + to + V1 + object + others. Passive- Object + is/am/are + V1 + ing + to + be + verbIIIrd form + by + Object + Others. Note :- In the above sentence Main Verb infinitive 'to open' is used while converting it into Passive to + be + V3 (opened ) is used. Correct Passive SentenceThe shops are going to be opend at eleven o'clock. 272. Select the correct passive form of the given sentence. They will not allow mobile phones in the examination hall. (a) Mobile phones are not allowed in the examination hall. (b) Mobile phones are not being allowed in the examination hall. (c) Mobile phones will not be allowed in the examination hall. (d) Mobile phones were not allowed in the examination hall. SSC CHSL (Tier-I) –09/07/2019 (Shift-III)

332

YCT

Ans : (c) The above sentence is in Active Voice of Negative sentence of Future Indefinite tense. Its Passive voice structure will be– Active structureSub. + shall/will + not + VIst form + obj. Passive structureObj. + will/shall + not + be + VIIIrd form + by + sub. Note-1: Sometimes by + object is understood. Correct Passive VoiceMobile phones will not be allowed in the examination hall. 273. Select the most appropriate passive form of the given sentence. The surgeon successfully operated upon the patient. (a) The patient is being successfully operated upon by the surgeon. (b) The patient is successfully operated upon by the surgeon. (c) The patient was successfully operated upon by the surgeon. (d) The surgeon has been successfully operated upon by the patient. SSC CHSL (Tier-I) –08/07/2019 (Shift-I) Ans : (c) In the above sentence the use of "V2 (operated)" makes it clear that the given sentence is in Active voice of Past Indefinite tense. Its Passive voice structure will be– Active- Subject + vIInd form + object + others words. Passive- Object + was/were + vIIIrd form + by + subject + other words. Correct Passive VoiceThe patient was successfully operated upon by the surgeon. 274. Select the most appropriate passive form of the given sentence. He threw the ball towards his little brother. (a) His little brother's ball was thrown at him. (b) The ball was thrown towards his little brother. (c) The ball is thrown towards his little brother. (d) The ball is being thrown towards his little brother. SSC CHSL (Tier-I) –08/07/2019 (Shift-II) Ans : (b) In the above sentence the use of "Sub + V2" makes it clear that the given sentence is in Active voice of Past Indefinite tense. Its Passive voice structure will be– Active Structure- Sub. + VIInd form + obj. Passive Structure- Obj.+was/were+VIIIrd form + by + sub. Correct Passive Sentence– The ball was thrown towards his little brother. 275. Select the most appropriate passive form of the given sentence. Please lock the door. (a) The door should kindly being locked. (b) You are requested to lock the door. (c) You are being ordered to lock the door. (d) Please let the door be lock. SSC CHSL (Tier-I) –08/07/2019 (Shift-III) Active / Passive Voice

Ans : (b) In the above sentence the use of "Please + V1" makes it clear that the given sentence is in Active voice of Imerative sentence. Its passive voice structure will be– A.V.- Please + V1 + obj. P.V.- You are requested to + v1 + obj. You are requested/adviced/ordered+Active Sentence. Note- If the sense of order appears from Active sentence and object is also given in it, then its Passive can also be made in the following way– Let + obj. + be + verb3rd form. Correct Passive VoiceYou are requested to lock the door. 276. Select the most appropriate passive form of the given sentence. She answered fifty questions in one hour. (a) Fifty questions were answered by her in one hour. (b) Fifty questions were being answered by her in one hour. (c) Fifty questions have been answered by her in one hour. (d) Fifty questions are answered by her in one hour. SSC CHSL (Tier-I) –05/07/2019 (Shift-I) Ans : (a) In the above sentence the use of "V2 (answered)" makes it clear that the given sentence is in Active voice of Past Indefinite tense. Its Passive voice structure will be– Active structure Sub. + VIInd form + obj. Passive structure Obj. + was/were + VIIIrd form + by + sub. Note- While making Active to Passive, subject to object and object to subject is converted. Correct Passive Voice– Fifty questions were answered by her in one hour. 277. Select the most appropriate passive form of the given sentence. They were distributing food grains among the poor. (a) The poor will be distributed among food grains. (b) Food grains were being distributed among the poor. (c) Food grains were distributed among the poor. (d) Food grains will be distributed among the poor. SSC CHSL (Tier-I) –05/07/2019 (Shift-II) Ans : (b) In the above sentence the use of "were + ing" makes it clear that the given sentence is in Active voice of Past Continuous tense. Its Passive voice structure will be– Active Structure Sub. + was / were + v + ing form + obj. Passive Structure

Obj. + was / were + being + v IIIrd form + by + sub. Correct Passive VoiceFood grains were being distributed among the poor.

333

YCT

278. Select the most appropriate passive form of the given sentence. The store manager caught the teenager shoplifting. (a) The teenager is caught shoplifting by the store manager. (b) The teenager was caught shoplifting by the store manager. (c) The store manager is caught shoplifting by the teenager (d) The teenager has been caught shoplifting by the store manager. SSC CHSL (Tier-I) –05/07/2019 (Shift-III) Ans : (b) In the above sentence the use of "V2 (Caught)" makes it clear that the given sentence is in Active voice of Past Indefinite tense. Its Passive voice structure will be– Active structure Sub. + vIInd form + obj. Passive structure Obj. + was/were + vIIIrd form + by + sub. Correct Passive SentenceThe teenager was caught shoplifting by the store manager. 279. Select the correct passive form of the given sentence. The mechanic repaired my car quite satisfactorily. (a) My car repaired quite satisfactorily by the mechanic. (b) The mechanic was repaired quite satisfactorily by my car. (c) My car was repaired quite satisfactorily by the mechanic. (d) My car is repaired quite satisfactorily by the mechanic. SSC CHSL (Tier-I) –04/07/2019 (Shift-I) Ans : (c) In the above sentence the use of "Sub + V2" makes it clear that the given sentence is in Active voice of Past Indefinite tense. Its Passive voice structure will be– Active- Subject + VIInd + object + others. Passive- Object + was/were + VIIInd + by + subject. Correct Passive VoiceMy car was repaired quite satisfactorily by the mechanic. 280. Select the most appropriate passive form of the given sentence. How much monthly rent did you pay for the house? (a) How much monthly rent is paid for the house? (b) How much monthly rent was being paid for the house? (c) How much monthly rent was paid for the house? (d) How much monthly rent is being paid for the house? SSC CHSL (Tier-I) –04/07/2019 (Shift-II) Ans : (c) In the above sentence the use of verb 'Wh+Did' makes it clear that the given sentence is in Interrogative of Past Indefinite tense. Its Passive structure will be– Active / Passive Voice

How much montly rent (question word)+was/were + verb3 + by + sub + other words + ? Correct Passive VoiceHow much monthly rent was paid for the house? 281. Select the most appropriate passive form of the given sentence. Have you paid your outstanding bills? (a) Have your outstanding bills been paid? (b) Are your outstanding bills being paid? (c) Have your outstanding bills being paid? (d) Will your outstanding bills be paid? SSC CHSL (Tier-I) –04/07/2019 (Shift-III) Ans : (a) In the above sentence the use of 'have + sub + V3" makes it clear that the given sentence is in Interrogative form of Present Perfect tense. Its Passive voice structure will be– Active structure- Has/have + sub. + VIIIrd form + obj. Passive structure- Has/have + obj. + been + VIIIrd form + by + sub.? Note :- While making Active to Passive, sub into object and obj. into subject is created. Sometimes 'by + subject' is understood. Correct Passive Voice Have your outstanding bills been paid ? 282. Select the correct passive form of the given sentence. The merchant took out the dead parrot from the cage. (a) The dead parrot took out from the cage by the merchant. (b) The dead parrot is taken out from the cage by the merchant. (c) The dead parrot was taking out from the cage by the merchant. (d) The dead parrot was taken out from the cage by the merchant. SSC CHSL (Tier-I) –03/07/2019 (Shift-I) Ans : (d) In the above sentence the use of "Sub + V2" makes it clear that the given sentence is in Active voice of Past Indefinite tense. Its Passive voice structure will be– Active structure- Sub. + VIInd form + obj. Passive structure- Obj.+was/were + VIIIrd form + by + sub. Correct Passive Voice– The dead parrot was taken out from the cage by the merchant. 283. Select the correct passive form of the given sentence. The merchant brought many gifts for his family from India. (a) Many gifts were brought by the merchant for his family from India. (b) His family were brought by the merchant for his family from India. (c) Many gifts brought by the merchant for his family from India. (d) Many gifts are brought by the merchant for his family from India. SSC CHSL (Tier-I) –03/07/2019 (Shift-II)

334

YCT

Ans. (a) : In the above sentence the use of "Sub + V2" makes it clear that the given sentence is in Active voice of Past Indefinite tense. Its Passive voice structure will be– Active Structure Sub. + VIIrd form + obj. Passive Structure Obj. + was/were + VIIIrd form + by + sub. Correct Passive VoiceMany gifts were brought by the merchant for his family from India. 284. Select the correct passive form of the given sentence. The new gardener looks after the plants well. (a) The plants looked after well by the new gardener. (b) The plants were looked after well by the new gardener. (c) The plants are looked after well by the new gardener. (d) The new gardener is looked after well by the plants. SSC CHSL (Tier-I) –03/07/2019 (Shift-III) Ans. (c) : In the above sentence the use of "Subject + V1(e/es)" makes it clear that the given sentence is in Active voice of Present Indefinite tense. Its Passive voice structure will be– Active- Subject + verb + s/es + object + others. Passive- Object + is/am/are + verbIIIrd + by + subject + others. Correct Passive VoiceThe plants are looked after well by the new gardener. 285. Select the correct passive form of the given sentence. The gardener has mowed the lawn. (a) The lawn was mowed by the gardener. (b) The lawn has been mowed by the gardener. (c) The lawn is mowed by the gardener. (d) The gardener has been mowed by the lawn. SSC CHSL (Tier-I) –02/07/2019 (Shift-I) Ans. (b) : In the above sentence the use of "has + V3" makes it clear that the given sentence is in Active Voice of Present Perfect tense. Its Passive voice structure will be– Active- Subject + has/have + VerbIIIrd form + Object + Others. Passive- Object + has/have + been + VerbIIIrd form + by + Subject + Others. Correct Passive Voice– The lawn has been mowed by the gardener. 286. Select the correct passive form of the given sentence. The Japanese Emperor Akihito abdicated the Chrysanthemum Throne in April, 2019. (a) The Japanese Emperor Akihito was abdicated by the Chrysanthemum Throne in April, 2019. (b) The Chrysanthemum Throne was abdicated by the Japanese Emperor Akihito in April, 2019. Active / Passive Voice

(c) The Chrysanthemum Throne has abdicated by the Japanese Emperor Akihito in April, 2019. (d) The Chrysanthemum Throne is abdicated by the Japanese Emperor Akihito in April, 2019. SSC CHSL (Tier-I) –02/07/2019 (Shift-II) Ans : (b) In the above sentence the use of "Sub + V2" makes it clear that the given sentence is in Active voice of Past Indefinite tense. Its Passive voice structure will be– A.V.- Sub. + v2 + object + other words. P.V.- Object + was/were + V3rd + by + subject + other words. Correct Passive Voice– The Chrysanthemum Throne was abdicated by the Japanese Emperor Akihito in April, 2019. 287. Select the correct passive form of the given sentence. Bill Gates has given away twenty seven percent of his wealth in charity. (a) Twenty seven percent of his wealth had been given away by Bill Gates in charity. (b) Twenty seven percent of his wealth has given away Bill Gates in charity. (c) Twenty seven percent of his wealth has been given away by Bill Gates in charity. (d) Bill Gates has been given away by twenty seven percent of his wealth in charity. SSC CHSL (Tier-I) –02/07/2019 (Shift-III) Ans : (c) In the above sentence the use of "has + V3" makes it clear that the given sentence is in Active voice of Present Perfect tense. Its Passive voice structure will be– A.V.- Sub. + has/have + VIIIrd form + obj. P.V.- Obj. + has/have + been + VIIIrd form + by + sub. Correct Passive VoiceTwenty seven percent of his wealth has been given away by Bill Gates in charity. 288. Select the correct passive form of the given sentence. The prisoners have made these paintings. (a) These prisoners have been made by the paintings. (b) These paintings were made by the prisoners. (c) These paintings are made by the prisoners. (d) These paintings have been made by the prisoners. SSC CHSL (Tier-I) –01/07/2019 (Shift-III) Ans : (d) In the above sentence the use of "have + V3" makes it clear that the given sentence is in Active voice of Present Perfect tense. Its Passive voice structure will be– Active voice- (Sub. + has/ have + v3 + obj.) Passive voice- (Obj. + has/have + been + v3 + by + sub.) Correct Passive VoiceThese paintings have been made by the prisoners. 289. Select the correct passive form of the given sentence. Savita will deliver a speech on the occasion of Ambedkar Jayanti.

335

YCT

(a) On the occasion of Ambedkar Jayanti a A.V.– Subject + is/am/are + verb1 + ing + object + speech is being delivered by Savita. other words. (b) On the occasion of Ambedkar Jayanti a P.V.– Object + is/am/are + being + v3 + by + subject + speech was delivered by Savita. other words. (c) On the occasion of Ambedkar Jayanti a Correct Passive Voicespeech will be delivered by Savita. (d) On the occasion of Ambedkar Jayanti a A new serial about the life of Dr. Ambedkar is being watched by my mother. speech is to be delivered by Savita. SSC CHSL (Tier-I) 26/10/2020 (Shift-II) 292. Select the correct passive form of the given sentence. Ans. (c) : In the above sentence the use of "Will + V1" We are arranging an exhibition of sarees from makes it clear that the given sentence is in Active different states of India at Dilli Haat. voice of Future Indefinite tense. Its Passive voice (a) An exhibition of sarees from different states structure will be– of India is being arranged by us at Dilli Haat. Active- Subject + will/shall + verb1 + object + other words. (b) An exhibition of sarees from different states Passive- Object + will/shall + be + verb3 + by + of India was arranged by us at Dilli Haat. subject + other words. (c) An exhibition of sarees from different states Correct Passive Voiceof India has been arranged by us at Dilli Haat. On the occasion of Ambedkar Jayanti a speech will be (d) An exhibition of sarees from different states delivered by Savita. of India will be arranged by us at Dilli Haat. 290. Select the option that is the passive form of the SSC CHSL (Tier-I) 16/10/2020 (Shift-II) sentence. Ans. (a) : In the above sentence the use of 'are + VIst + The sub-committee is presenting its report in a ing' makes it clear that the given sentence is in Active week's time. voice of Present Continuous tense. Its Passive voice (a) The report of the sub-committee had been structure will be– presented in a week's time. A.V.– Subject + is/am/are + V1 + ing + object + other (b) The report of the sub-committee is presenting words. in a week's time. (c) The report of the sub-committee is being P.V.– Object + is/am/are + being + V3 + by + subject + other words. presented in a week's time (d) The report of the sub-committee has been Correct Passive VoiceAn exhibition of sarees from different states of India is presented in a week's time. SSC CHSL (Tier-I) 17/03/2020 (Shift-I) being arranged by us at Dilli Haat. Ans. (c) In the above sentence the use of "is + V1 + ing" 293. Select the correct passive form of the given makes it clear that the given sentence is in Active voice sentence. of Present Continuous tense. Its Passive voice structure My friend narrated a very interesting story at will be– the International Storytelling Festival. Active structure (a) A very interesting story was being narrated by ing Sub. + is/am/are + V form + obj. my friend at the International Storytelling Passive Structure Festival. IIIrd Obj. + is/am/are + being + V + by + sub. (b) A very interesting story has been narrated by Correct Passive Voicemy friend at the International Storytelling The report of the sub-committee is being presented in a Festival. week's time. (c) A very interesting story was narrated by my 291. Select the correct passive form of given friend at the International Storytelling sentence. Festival. My mother is watching a new serial about the (d) A very interesting story is being narrated by life of Dr. Ambedkar. my friend at the International Storytelling (a) A new serial about the life of Dr. Ambedkar Festival. is being watched by my mother. SSC CHSL (Tier-I) 16/10/2020 (Shift-I) (b) A new serial about the life of Dr. Ambedkar Ans. (c) : In the above sentence the use of "V2 was being watched by my mother. (narrated)" makes it clear that the given sentence is in (c) A new serial about the life of Dr. Ambedkar Active voice of Past Indefinite tense. Its Passive voice is watched by my mother. structure will be– (d) A new serial about the life of Dr. Ambedkar A.V.- Subject + V + object+ other words. 2 will be watched by my mother. P.V.- Object + was/were + V3 + by + subject + other SSC CHSL (Tier-I) 16/10/2020 (Shift-III) words. Ans. (a) : In the above sentence the use of "is + VIst + Correct Passive Voiceing" makes it clear that the given sentence is in Active voice of Present Continuous tense. Its Passive voice A very interesting story was narrated by my friend at the International Storytelling Festival. structure will be– Active / Passive Voice

336

YCT

294. Select the correct passive form of the given Ans. (c) : In the above sentence the use of 'have + v3' sentence. makes it clear that the given sentence is in Active Chhattisgarh organised the first ever 'All India voice of Present Perfect tense. Its Passive voice Folk Dance Festival' in January 2020. structure will be– (a) The first ever 'All India Folk Dance Festival' Active Structure will be organised by the Chhaattisgarh State Sub. + has/have + VIIIrd form + Obj. in January 2020. Passive Structure (b) The first ever 'All India Folk Dance Festival' IIIrd was being organised by the Chhaattisgarh Obj. + has/have + been V form + by + sub. Correct Passive VoiceState in January 2020. (c) The first ever 'All India Folk Dance Festival' The colour of the new car we want to buy has been was organised by the Chhaattisgarh State in chosen by us. January 2020. 297. Select the correct passive form of the given (d) The first ever 'All India Folk Dance Festival' sentence. had been organised by the Chhaattisgarh State Rains lashed Delhi and brought the in January 2020. temperature down by a few degrees. SSC CHSL (Tier-I) 21/10/2020 (Shift-II) (a) Delhi had been lashed by rain and the Ans. (c) : In the above sentence the use of V2 temperature had been brought down by a few (organised) makes it clear that the given sentence is in (b) Delhi was being lashed by rain and the Active voice of Past Indefinite tense. Its passive voice temperature was being brought down by a structure will be– few degrees. A.V.– Subject + V2 + object + others. (c) Delhi was lashed by rains and the temperature P.V.– Object + was/were + V3 + by + subject + others. was brought down by a few degrees. Correct Passive Voice (d) Delhi is being lashed by rain and the The first ever 'All India Folk Dance Festival' was temperature was brought down by a few organised by the Chhaattisgarh State in January 2020. degrees. 295. Select the correct Passive form of the given SSC CHSL (Tier-I) 13/10/2020 (Shift-I) sentence. Ans. (c) : In the above sentence the use of "Subject + The Inspector had already proved the case V2" makes it clear that the given sentence is in Active before the investigation team took charge of it. Voice of Past Indefinite tense. Its Passive voice (a) The case had already been proved by the Inspector before the investigation team took structure will be – A.V.- Subject + V2 + object + others. charge of it. (b) The investigation team took chage of the case P.V.- Object + was/were + V3 + by + subject + others. out the inspector had already proved before it. Correct Passive Voice(c) The case was already proved by the Inspector Delhi was lashed by rains and the temperature was brought down by a few degrees. before the investigation team took charge of it. (d) The case had already proved by the Inspector 298. Select the correct passive form of the given before the investigation team took charge of it. sentence. SSC CHSL (Tier-I) 19/03/2020 (Shift-III) The coach received the members of the team Ans. (a) : In the above sentence the use of "had + V3" with great excitement. makes it clear that the given sentence is in Active (a) The members of the team were received by voice of Past Perfect tense. Its Passive voice structure the coach with great excitement. will be– (b) The member of the team had been received Active Structure by the coach with great excitement. Sub. + had + VIIIrd form + Obj. (c) The member of the team was received by the Passive Structure coach with great excitement. Obj. + had + been + VIIIrd form + by + sub. (d) The member of the team could be received by Correct Passive Voicethe coach with great excitement. SSC CHSL (Tier-I) 19/03/2020 (Shift-I) The case had already been proved by the Inspector before the investigation team took charge of it. Ans. (a) : In the above sentence the use of "V2 296. Select the option that is the passive form of the (received)" makes it clear that the given sentence is in sentence. Active voice of Past Indefinite tense. Its Passive voice We have chosen the colour of the new car we structure will be– want to buy. Active structure (a) The colour of the new car we want to buy is nd Sub + V II form + object being chosen by us. (b) The colour of the new car we want to buy had Passive structure been chosen. rd (c) The colour of the new car we want to buy has Obj + was / were + V III form + by + sub been chosen by us. (d) The colour of the new car we want to buy can Correct Passive VoiceThe members of the team were received by the coach be chosen by us. SSC CHSL (Tier-I) 17/03/2020 (Shift-II) with great excitement. Active / Passive Voice

337

YCT

299. Select the correct passive form of the given A.V.– Subject + has/have + V3 + object. P.V.– Object + has/have + been + V3 + by + subject. sentence. Sheetal might have misplaced Mohan's wallet Correct Passive Voicein their rush. A new version of Shakespeare's 'Macbeth' has been (a) Mohan's wallet may have been misplaced by performed by the students. Sheetal in their rush. 302. Select the correct passive form of the given (b) Mohan's wallet might have been misplaced by sentence? Sheetal in their rush. Who killed Ravana? (c) Mohan's wallet may be misplaced by Sheetal (a) By whom was Ravana killed? in their rush. (b) Ravana is killed by whom? (d) Mohan's wallet might be misplaced by (c) By whom Ravana was killed? Sheetal in their rush. (d) Ravana was killed by whom? SSC CHSL (Tier-I) 18/03/2020 (Shift-III) SSC CHSL (Tier-I) 26/10/2020 (Shift-I) Ans. (b) : In the above sentence the use of 'might + Ans. (a) : In the above sentence the use of "Who + V2 have' makes it clear that the given sentence is in (killed)" makes it clear that the given sentence is Active voice of Interrogative form of Past Indefinite Active voice of Past form of modal verb. Its Passive tense. Its Passive voice structure will be– structure will be– A.V.– Subject + modal + have + V3 + object + others. Active − who + V IInd form + object ? P.V.– Object + modal + have + been + V3 + by + subject + others. Passive − By whom + was / were + object + VIIIrd form + ? Correct Passive VoiceNote- When an interrogative sentence start with who, 'Mohan's wallet might have been misplaced by Sheetal while making it Passive, "By whom" at the beginning in their rush.' of the sentence. Because who (Sub) is used in object 300. Select the option that is the passive form of the "by whom" putting an object in passive voice. sentence. Correct Passive VoiceThe batsman had hit the ball towards the By whom was Ravana killed? boundary. (a) The ball is hit towards the boundary by the 303. Select the correct passive form of the given sentence. batsman. Did you inform him about the ticket (b) The ball is being towards the boundary by the cancellation? batsman. (a) Was he informed about the ticket cancellation (c) The ball was hit towards the boundary by the by you? batsman. (b) Will he be informed about the ticket (d) The ball had been hit towards the boundary cancellation by you? by the batsman. (c) Is he being informed about the ticket SSC CHSL (Tier-I) 18/03/2020 (Shift-II) cancellation by you? Ans. (d) : In the above sentence the use of 'had + V3" (d) Was he being informed about the ticket makes it clear that the given sentence is in Active voice cancellation by you? of Past Perfect tense. Its Passive voice structure will SSC CHSL (Tier-I) 26/10/2020 (Shift-II) be– Ans. (a) : In the above sentence the use of "Did + Sub A.V.– Subject + had + v3 + object + others. + V1" makes it clear that the given sentence is in P.V.– Object + had + been + v3 + by + subject + others. Active voice of interrogative form of Past Indefinite Correct Passive Voicetense. Its Passive voice structure will be– 'The ball had been hit towards the boundary by the Active − Did + subject + verb1 + object + other words + ? batsman.' 301. Select the correct passive form of the given Passive − Was / were + object + verb3 + other words + sentence. by + subject + ? The students have performed a new version of Correct Passive Voice– Shakespeare's 'Macbeth'. Was he informed about the ticket cancellation by you? (a) Shakespeare's 'Macbeth' has been performed 304. Select the correct passive form of the given by the new version of the students. sentence. (b) A new version of Shakespeare's 'Macbeth' Dr. Verma advised patients to try out a new have been performed by the students. medicine for migraine. (c) A new version of Shakespeare's 'Macbeth' has (a) Patients were advised to try out a new been performed by the students. medicine for migraine by Dr. Verma. (d) Shakespeare's 'Macbeth' have been performed (b) Patients have been advised to try out a new by the new version of the students. medicine for migraine by Dr. Verma. SSC CHSL (Tier-I) 18/03/2020 (Shift-I) (c) The old patients had been advised to try out a new medicine for migraine by Dr. Verma. Ans. (c) : In the above sentence the use of "have + V3" (d) The patients were being advised to try out a makes it clear that the given sentence is in Active new medicine for migraine by Dr. Verma. voice of Present Perfect tense. Its passive voice SSC CHSL (Tier-I) 21/10/2020 (Shift-I) structure will be– Active / Passive Voice

338

YCT

(a) Diamonds can be bought and sympathy Ans. (a) : In the above sentence the use of "Subject + cannot be bought. V2" makes it clear that the given sentence is in Active (b) Diamonds can be bought but sympathy voice of Past Indefinite tense. Its Passive voice cannot be bought. structure will be– (c) Diamonds can be bought but sympathy can be A.V.– Subject + V2 + object + others. bought. P.V.– Object + was/were + V3 + by + subject + others. (d) Diamonds cannot be bought but sympathy Correct Passive Voicecannot be bought. Patients were advised to try out a new medicine for SSC CHSL (Tier-I) 15/10/2020 (Shift-III) migraine by Dr. Verma. Ans. (b) : In the above sentence the use of modal verb 305. Select the correct passive form of the given "can" makes it clear that the given sentence is Active sentence. voice of Modal Verb. Its passive voice structure will They are laying new telephone cables our be – colony. A.V.- Sub + modal (can) + V1 + others. (a) New telephone cables are being laid in our P.V.- Subject + modal(can) + be + V3 + others. colony. Correct Passive Voice(b) New telephone cables were being laid in our Diamonds can be bought but sympathy cannot be colony. bought. (c) New telephone cables will be laid in our 308. Select the correct passive form of the given colony. sentence. (d) New telephone cables were laid in our I use jaggery to sweeten my tea in winters. colony. (a) In winters jaggery was used by me to sweeten SSC CHSL (Tier-I) 19/10/2020 (Shift-I) my tea. (b) Jaggery is being used by me to sweeten my Ans. (a): In the above sentence the use of "are + V1st + tea in winters. ing" makes it clear that the given sentence is in Active (c) I have been using jaggery to sweeten my tea voice of Present Continuous tense. Its Passive voice in winters. structure will be– (d) Jaggery is used by me to sweeten my tea in A.V.- Subject + is/am/are + V1 + ing + object + others. winters. P.V.- Object + is/am/are + being + Past participle (V3) SSC CHSL (Tier-I) 15/10/2020 (Shift-II) + by + subject + others. Ans. (d) : In the above sentence the use of "Sub + V1" Correct Passive Voicemakes it clear that the given sentence is in Active New telephone cables are being laid in our colony. voice of Present Indefinite tense. Its Passive voice 306. Select the correct passive from of the sentence. structure will be– The Delhi Police has shot an educational video Active structure on Traffic Rules for school students. Sub. + v Ist form + obj. (a) An educational video on Traffic Rules for school students has been shot by the Delhi Passive structure Police. Obj. + is / am / are + V IIIrd form + by + sub. (b) An educational video on Traffic Rules for school students is being shot by the Delhi Correct Passive VoicePolice Jaggery is used by me to sweeten my tea in winters. (c) An educational video on Traffic Rules for 309. Select the option that is the passive form of the school students is being shot by the Delhi given sentence. Police. The Principal gave her a warning about (d) An educational video on Traffic Rules for coming late to school everyday. (a) She was giving a warning be the Principal school students had been shot by the Delhi about coming late to school everyday. Police. (b) She gives a warning to the Principal about SSC CHSL (Tier-I) 12/10/2020 (Shift-II) coming late to school everyday. Ans. (a) : In the above sentence the use of 'has + V3" (c) She was given a warning about coming late to makes it clear that the given sentence is in Active school everyday by the Principal. voice of Present Perfect tense. Its Passive voice (d) She has giving a warning by the Principal structure will be– about coming late to school everyday. A.V.– Subject + has/have + V3 + object + others. SSC CHSL (Tier-I) 15/10/2020 (Shift-I) P.V.– Object + has/have + been + V3 + by + object + Ans. (c) : In the above sentence the use of "V2 (gave)" others. makes it clear that the given sentence is in Active voice Correct Passive Voiceof Past Indefinite tense. Its Passive voice structure will An educational video on Traffic Rules for school be – students has been shot by the Delhi Police. A.V.– Subject + V2 + object + others. 307. Select the correct passive form of given P.V.– Object + was/were + V3 + by + subject + others. Correct Passive Voicesentence. We can buy diamonds, but we cannot buy She was given a warning about coming late to school everyday by the Principal. sympathy. Active / Passive Voice

339

YCT

310. Select the option that is the passive form of the 313. Select the correct passive form of the given given sentence. sentence. The police brought a search warrant when they The police arrested the thieves yesterday. raided his house. (a) The thieves have been arrested yesterday by (a) A search warrant was brought by the police the police. when they raided his house. (b) The thieves were arrested yesterday by the (b) The police had raided his house and brought police. a search warrant. (c) The thieves were being arrested yesterday by (c) A search warrant had been brought when the the police. police raided his house. (d) The thieves had been arrested yesterday by (d) A search warrant was brought when they the police. raided his house by police. SSC CHSL (Tier-I) 20/10/2020 (Shift-III) SSC CHSL (Tier-I) 13/10/2020 (Shift-III) Ans. (b) : In the above sentence the use of "V2 2 Ans. (a) In the above sentence the use of "V " (brought) (arrested)" makes it clear that the given sentence is in makes it clear that the given sentence is in Active voice Active voice of Past Indefinite tense. Its Passive voice of Past Indefinite tense. Its Passive voice structure will structure will be– be– A.V.– Subject + V2 + object + others. A.V.– Subject + V2 + Object + others. P.V.– Object + was/were + V3 + by + subject + others. P.V.– Object + was/were + V3 + by + subject + others. Correct Passive VoiceCorrect Passive VoiceA search warrant was brought by the police when they The thieves were arrested yesterday by the police. 314. Select the correct passive form of the given raided his house. sentence. 311. Select the correct passive form of the given Gulliver easily pulled all the warships of the sentence. enemy after him. Please make a cup of coffee for me. (a) All the warships of the enemy had been easily (a) Can you please make a cup of coffee for me? pulled by Gulliver after him. (b) A cup of coffee can be made for me by you. (b) All the warships of Gulliver were easily (c) I request you to make a cup of coffee for me. pulled by the enemy after him. (d) Let a cup of coffee be made for me. (c) All the warships of the enemy were easily SSC CHSL (Tier-I) 14/10/2020 (Shift-III) pulled by Gulliver after him. Ans. (d) : In the above sentence the use of "Please + (d) All the warships of the enemy were being V1" makes it clear that the given sentence is in easily pulled by Gulliver after him. Imperative sentence. Its Passive voice structure will SSC CHSL (Tier-I) 20/10/2020 (Shift-I) be– A.V.- Please + V1 + object + others. Ans. (c) : In the above sentence the use of "V2 P.V.- Let + object + be + V3 + others. (pulled)" makes it clear that the given sentence is Correct Passive VoiceActive voice of Past Indefinite tense. Its Passive voice Let a cup of coffee be made for me. structure will be – 312. Select the correct passive form of the given Active structure sentence. Sub. + VIInd form + obj. The vet told me to put eye drops and ointment Passive structure in my pet’s eyes. (a) I had been told by the vet to put eye drops and Obj. + was / were + VIIIrd form + by + sub. ointment in my pet’s eyes. (b) I am being told by the vet to put eye drops Correct Passive VoiceAll the warships of the enemy were easily pulled by and ointment in my pet’s eyes. (c) I was told by the vet to put eye drops and Gulliver after him ointment in my pet’s eyes. 315. Select the correct passive form of the given (d) I was being told by the vet to put eye drops sentence. and ointment in my pet’s eyes. For reaching the station, they hired an auto. SSC CHSL (Tier-I) 21/10/2020 (Shift-III) (a) For reaching the station, an auto had been Ans. (c) : In the above sentence the use of "V2 (told)" hired by them. makes it clear that the given sentence is in Active voice (b) For reaching the station, an auto was hired by of Past Indefinite tense. Its Passive voice structure will them. be– (c) For reaching the station, an auto is hired by Active voicethem. Subject + verb2 + object + other words. (d) For reaching the station, an auto was being Passive voicehired by them. Object + was/were + VIII form + by + subject + other SSC CHSL (Tier-I) 19/10/2020 (Shift-III) words. Ans. (b) : In the above sentence the use of "V2 (hired)" Correct Passive Voicemakes it clear that the given sentence is in Active I was told by the vet to put eye drops and ointment in voice of Past Indefinite tense. Its Passive voice my pet’s eyes. structure will be – Active / Passive Voice

340

YCT

Active structure Ans. (c) : In the above sentence Modal 'can' is used. Sub. + VIInd form + obj. Hence its Passive voice structure will be– Passive Structure Active structure Obj. + was/were + VIIIrd form + by + sub. Sub. + modal + V1st form + Obj. Correct Passive VoicePassive Structure For reaching the station, an auto was hired by them. Obj. + modals + be + V3rd from + by + sub. 316. Select the option that is the passive form of the Note– While making Active to Passive, subject into given sentence. object and object to subject is replaced. This book explores the secret of leading a long Correct Passive Voiceand meaningful life. All the programmes can be performed perfectly well (a) The secret of leading a long and meaningful by this android. life was being explored by this book. (b) The secret of leading a long and meaningful 319. Select the correct passive form of the given sentence. life is explored in this book. I use baking soda to remove stains from my (c) The secret of leading a long and meaningful clothes. have been explored by this book. (a) I have been using baking soda to remove (d) The secret of leading a long and meaningful stains from my clothes. life had been explored by this book. (b) Baking soda was used by me to remove stains SSC CHSL (Tier-I) 20/10/2020 (Shift-II) from my clothes. Ans. (b) : In the above sentence the use of "Sub + V1" (c) Baking soda is used by me to be remove makes it clear that the given sentence is in Active voice of Present Indefinite tense. its Passive voice stains from my clothes. structure will be– (d) Baking soda is used by me to remove stains from my clothes. A.V. − Sub. + verb1 + ( s / es) + obj. + other words. SSC CHSL (Tier-I) 14/10/2020 (Shift-II) P.V. − Obj. + ( is / am/ are ) + verb3 + by + sub. + other words. Ans. (d) : In the above sentence the use of "Subject + Correct Passive VoiceV1" makes it clear that the given sentence is in Active The secret of leading a long and meaningful life is voice of Present Indefinite tense. Its Passive voice explored in this book. structure will be – 317. Select the correct passive form of the given Active Structure sentence. Sub. + V1st form + obj. He spent a lot of money on movies. Passive structure (a) A lot of money is spent by him on movies. (b) A lot of money had been spent by him on Obj. + is/am/are + VIIIrd form + by + sub. movies. Correct Passive Voice(c) a lot of money was being spent by him on Baking soda is used by me to remove stains from my movies. clothes. (d) A lot of money was spent by him on movies SSC CHSL (Tier-I) 19/10/2020 (Shift-II) 320. Select the option that expresses the given Ans. (d) : In the above sentence the use of "V2 (spent)" sentence in passive voice. makes it clear that the given sentence is in Active Sujata shared the results of her experiment voice of Past Indefinite tense. Its Passive voice with her colleagues. structure will be – (a) The results of her experiment can be shared Active Structure with her colleagues by Sujata. IInd Sub. + V form + Obj. (b) The results of her experiment were shared Passive Structure with her colleagues by Sujata. Obj. + was/were + VIIIrd form + by + sub. (c) The results of her experiment are being Correct Passive Voiceshared with her colleagues by Sujata. A lot of money was spent by him on movies. (d) He colleagues shared the results of her 318. Select the correct passive form of the given experiment with Sujata. sentence. SSC CHSL (Tier-I) 14/10/2020 (Shift-I) This android can perform all the programmes Ans. (b) : In the above sentence the use of "Subject + perfectly well. 2 (a) All the programmes can have been performed V " makes it clear that the given sentence is in Active perfectly well by this android. voice of Past Indefinite tense. Its Passive voice (b) All the programmes could have been structure will be– performed perfectly well by this android A.V.– Subject + V2 + object + others. (c) All the programmes can be performed P.V.– Object + was/were + V + by + subject + others. 3 perfectly well by this android. Correct Passive Voice (d) All the programmes could be performed The results of her experiment were shared with her perfectly well by this android. SSC CHSL (Tier-I) 19/03/2020 (Shift-II) colleagues by Sujata. Active / Passive Voice

341

YCT

321. Select the correct passive form of the given sentence. The mechanic repaired my bike. (a) My bike has been repaired by the mechanic (b) My bike was repaired by the mechanic (c) My bike was being repaired by my mechanic (d) My bike is repaired by my mechanic SSC CHSL (Tier-I) 12/10/2020 (Shift-III) Ans. (b) : In the above sentence the use of "repaired (V2)" makes it clear that the given sentence is in Active voice of Past Indefinite tense. Its Passive voice structure will be– A.V.– Subject + V2 + object + others. P.V.– Object + was/were + V3 + by + subject + others. Correct passive sentence– My bike was repaired by the mechanic 322. Select the correct passive form of the given sentence. The police beat a number of protestors last night. (a) A number of protestors has been beaten by the police the night before. (b) A number of protestors were being beaten by the police last night. (c) A number of protestors were beaten by the police the night before. (d) A number of protestors had been beaten by the police last night. SSC CHSL (Tier-I) 12/10/2020 (Shift-I) Ans. (c) : In the above sentence the use "V2 (beat)" makes it clear that the given sentence is Active voice of Past Indefinite tense. Its Passive voice structure will be– A.V.– Subject + V2 + object + other words. P.V.– Object + was/were + V3 + by + subject + other words. Correct Passive Voice-

Ans. (d) : In the above sentence the use of "Subject + V2" makes it clear that the given sentence is in Active voice of Past Indefinite tense. Its Passive voice structure will be– Active structure Sub. + VIInd form + obj. Passive structure Obj. + was/were + VIIIrd form + by + sub. + other words. Note:– While making Active to passive, subject into object and object into subject is converted. Correct Passive Voice– The research paper was written by Jiya with the help of her tutor. 324. Select the option that expresses the given sentence in active voice. Nothing can be achieved without hard work. (a) One has achieved nothing without hard work. (b) One will achieve nothing without hard work. (c) One could achieve nothing without hard work. (d) One can achieve nothing without hard work. SSC CGL (Tier-II) 08.08.2022 Shift-II Ans. (d) : The given sentence is in Passive voice 'Nothing can be achieved without hard work' will be converted into Active Voice 'One can achieve nothing without hard work' because can + be + achieved ( v3 ) indicated passive of Modal 'can'. Thus, the correct answer is option (d). 325. Select the option that expresses the given sentence in passive voice. Do we need high taxed on senior citizens' savings? (a) Are senior citizens' high taxes needed for savings? (b) Do senior citizens need high taxes on their savings? (c) Are high taxes needed on senior citizens savings? (d) Do we need senior citizens savings on high taxes? SSC Stenographer (Grade C & D) 15.11.2021 Shift-II Ans. (c) : In the above sentence the use of 'Do' is the starting of the sentence makes it clear that the sentence is Interrogative form of Present indefinite tense and it passive voice structure will beA.v. → Do/Does + sub. + V1 + obj. + others words? P.v. → is/am/are + obj. + V3 + others words (by + sub)? Note- According to the sense of the setence by + subject is understood and it will not be used.

A number of protestors were beaten by the police the night before. 323. Select the option is the passive form of the sentence. Jiya wrote the research paper with the help of her tutor. (a) The research paper is being written by Jiya with the help of her tutor. (b) The research paper was wrote by Jiya with the help of the tutor. (c) The research paper can be written by the tutor with the help of Jiya. (d) The research paper was written by Jiya with Correct Passive Sentence→ the help of her tutor. SSC CHSL (Tier-I) 17/03/2020 (Shift-III) Are high taxes needed on senior citizens saving. Active / Passive Voice

342

YCT

05. DIRECT AND INDIRECT SPEECH TCS hewšve& hej DeeOeeefjle (Based On TCS Pattern) Chapterwise

Exam

Question No.

1

CGL (Tier-1)

21

CGL (Tier-2)

9

CHSL (Tier-1)

23

CHSL (Tier-2)

7

Selection Post VII, VIII, XI

6

SSC MTS

5

SSC GD

4

Assertive Sentence

SSC CPO SI 2

Interrogative Sentence

3

Imperative Sentence

4

Exclamatory Sentence

Direct and Indirect Speech

9

CGL (Tier-2)

5

CHSL (Tier-1)

6

CHSL (Tier-2)

9

Selection Post VII, VIII, XI

1

SSC MTS

2

SSC GD

1

SSC CPO SI

3

CGL (Tier-1)

4

CGL (Tier-2)

2

CHSL (Tier-1)

8

CHSL (Tier-2)

2

Selection Post VII, VIII, XI

1

SSC MTS

2

SSC GD

1

SSC CPO SI

2

CGL (Tier-1)

2

CGL (Tier-2)

1

CHSL (Tier-1)

1

CHSL (Tier-2)

1

Selection Post VII, VIII, XI

1

(2017–2023)

(2017–2023)

(2017–2023)

1

SSC GD

-

SSC CPO SI

2

343

(2017–2023)

10

CGL (Tier-1)

SSC MTS

Years

YCT

Trend Analysis of Questions topicwise from CGL (Pre & Mains) CHSL (Pre & Mains) Selection Post VII, VIII, XI, SSC MTS, SSC GD & Other Exams (2017-2023)

Direct and Indirect Speech

344

YCT

05. DIRECT AND INDIRECT SPEECH A.

Assertive Sentence

1.

Select the correct indirect form of the given sentence. Lee said to his wife, “I am very happy today.” (a) Lee told his wife that he was very happy today. (b) Lee told his wife that he has been very happy that day. (c) Lee told his wife he was very happy today. (d) Lee told his wife that he was very happy that day SSC CHSL (Tier-I) 21/03/2023 (Shift-IV) Ans. (d) : Reported speech of the above direct speech sentence is in 'Present countinous tense'. Hence its indirect speech will be changed by these rules1. 'told' will be used in place of 'said to'. 2. Conjunction 'that' will be used in place of comma and inverted comma. 3. According to narration rule tense and Pronoun will be change Correct Sentence : Lee told his wife that he was very happy that day. 2. Select the correct indirect form of the given sentence. Miranda said to her teacher, "I was sick for a day." (a) Miranda told her teacher that she had been sick for a day. (b) Miranda told her teacher that she is sick fior a day. (c) Miranda told her teacher that she was sick for a day. (d) Miranda told her teacher that she has sick for a day. SSC CHSL (Tier-I) 21/03/2023 (Shift-II) Ans. (a) : The above sentence is direct form of Past countinuous tense. Its indirect form will be(i) 'told' will be used in place of Said to'. (ii) Conjunction 'that' will be used in place of Comma, inverted comma. (iii) Tense & Pronoun will be changed according to narration rule. Correct indirect form– Miranda told her teacher that she had been sick for a day. 3. Choose the option that is the correct direct form of the sentence. She said, since evening, she had go for a run. Direct and Indirect Speech

She said, “It is evening I must ran” She said, “It is evening I might go for a run.” She said, “It is evening I should go for a run.” She said, “It is evening I must go for a run.” SSC CHSL (Tier-I) 20/03/2023 (Shift-I) Ans. (d) : Above sentence is in indirect speech following rule will be used to change in direct speech. 1. comma and Inverted Comma will be used in place of 'since'. 2. Reporting verb 'said' will not be Changed. 3. Tense & Pronoun will be changed according to Narration rule. 4. Select the correct indirect form of the given sentence. Pritam said, "The prisoner had slept throughout the journey." (a) Pritam said that the prisoner had been sleeping throughout the journey. (b) Pritam said that the prisoner had sleep throughout the journey. (c) Pritam said that the prisoner had slept throughout the journey. (d) Pritam said that the prisoner has slept throughout the journey. SSC CHSL (Tier-I) 17/03/2023 (Shift-IV) Ans. (c) : In the above question conjunction 'that' will be used in place of comma inverted comma and tense will be changed according to the narration rule. Correct SentencePritam said that the prisoner had slept throughout the journey. 5. Choose the option that is the correct direct form of the sentence. He said that he would be taking a leave of absence for two weeks as there had been a death in his family. (a) He said "I am going to take a leave for two weeks as there had been a death in my family". (b) He said "I would be taking a leave of absence for 2 weeks as there been a death in my family". (c) He said "I will be taking a leave of absence for two weeks as there is a death in my family". (d) He said "I will be taking a leave of absence for two weeks as there has been a death in my family". SSC CHSL (Tier-I) 15/03/2023 (Shift-III)

345

(a) (b) (c) (d)

YCT

Ans. (d) : In the above sentence the use of ''said that'' makes it clear that the sentence is in Indirect speech and its Reported speech is a Assertive sentence. Hence its direct speech will be1- Comma and Inverted comma will be used in place of Conjunction. (that) 2- No change in Reporting verb because Reporting object is not given. 3- According to Narration Rules Tense and Pronoun will be change. Correct sentenceHe said "I will be taking a leave of absence for two weeks as there has been a death in my family." 6. Choose the option that is the correct direct form of the sentence. Shubham said that his father was playing cricket with him. (a) Shubham said, "My father played cricket with me." (b) Shubham said, "My father is playing cricket with me." (c) Shubham said, "My father was playing cricket with me." (d) Shubham said, "My father had played cricket with me." SSC CHSL (Tier-I) 15/03/2023 (Shift-I) Ans. (b) : In the above sentence the use of 'said that' makes it clear that the sentence is in Indirect speech of Assertive. Its direct speech will be1- Comma and Inverted comma will be used in place of Conjunction 'that'. 2- No change in Reporting verb 'said'. 3- According to Narration Rule Tense and Pronoun will be changed. Correct sentenceShubham said, ''My father is playing cricket with me.'' 7. Select the correct indirect form of the given sentence. Gloria said, "His relatives are landing tomorrow." (a) Gloria said that his relatives are landing the next day. (b) Gloria said that his relatives were landing the previous day. (c) Gloria said that his relatives were landing the next day. (d) Gloria said that his relatives have been landing the next day. SSC CHSL (Tier-I) 10/03/2023 (Shift-III) Ans. (c) : Reported speech of the above direct speech sentence is in assertive. Hence its Indirect speech will be(1) Conjunction 'that' will be used in place of Comma and Inverted comma. (2) No change in Reporing Verb "said'. (3) According to Narration Rules Tense and Pronoun will be changed and "The next day" is used in place of Tomorrow. Correct Sentence – Gloria said that his relatives were landing the next day. Direct and Indirect Speech

8.

Select the correct indirect form of the given sentence. He said, "I will never hurt you." (a) He said he should never hurt me. (b) He said that he would never hurt me. (c) He says that he will never hurt me. (d) He said that he will never hurt me. SSC CHSL (Tier-I) 09/03/2023 (Shift-IV) Ans. (b) : Reported speech of the above sentence is Assertive sentence. Hence its Indirect speech will be changed by following rules1. Conjunction "that" will be used in place of Comma and Inverted comma. 2. No change in Reorting verb "said". 3. According to narration rule tense and Pronoun will be changed. Correct Sentence: He said that he would never hurt me. 9. Select the correct indirect form of the given sentence. Aditi said, "I am not invested in the project anymore." (a) Aditi said that she had not invested in the project anymore. (b) Aditi said that she is not invested in the project anymore. (c) Aditi said that she was not invested in the project anymore. (d) Aditi said that she do not invested in the project anymore. SSC CHSL (Tier-I) 09/02/2023 (Shift-II) Ans. (c) : Reported speech of the above sentence is a Assertive sentnece. Therefore, the following Rule will be used to make Indirect speech. 1. Comma and Inverted comma will be used in place of conjunction 'that'. 2. No change in Reporting verb (said). 3. According to Narration Rule Tense and Pronoun will be changed. Correct Sentence : Aditi said that she was not invested in the project anymore. 10. Select the option that expresses the given sentence in indirect speech. Arya said, “I am very busy now.” (a) Arya said that she is being very busy now. (b) Arya said that she was being very busy then. (c) Arya said that she is be very busy now. (d) Arya said that she was very busy then. SSC Selection Posts XI-30/06/2023 (Shift-IV) Ans. (d) : Reported speech of the above direct sentence is in assertive. Its Indirect speech will be1. Conjunction 'that' will be used in place of comma and inverted comma. 2. According to narration rule tense and pronoun will be changed. 3. 'Now' will be changed into 'then'. Correct SentenceArya said that she was very busy then.

346

YCT

11.

Select the most appropriate option in direct speech. (a) The principal say me, “You are in-charge of the admission cell.” (b) The principal said me, You are in-charge of the admission cell. (c) The principal said to I, “You are in-charge of the admission cell.” (d) The principal said to me, “You are in-charge of the admission cell.” SSC CGL Mains 26/10/2023 Ans. (d) : In the above options, only option (d) 'The principal said to me, ''you are in-charge of the admission cell'' is correctly framed in direct speech. Other options are grammatically incorrect. 12. Select the option that expresses the given sentence in indirect speech. He said, “I bought this book for my brother.” (a) He said that he buy this book for his brother. (b) He said that he had bought that book for his brother. (c) He said that he bought this book for his brother. (d) He said that he has bought this book for his brother. SSC CGL Mains 26/10/2023 Ans. (b) : Reported speech of the above direct sentence is past indefinite tense. Its Indirect speech will be 1. Conjunction 'that' will be used in place of comma and inverted comma. 2. According to marration rule tense and pronoun will be changed. Correct Sentence He said that he had bought that book for his brother. 13. Select the correct indirect narration of the given sentence. He said to me, “My phone is switched off.” (a) He told me that his phone was switched off. (b) He told me that our phone had switch off. (c) He told me that his phone was switch off. (d) He told me that my phone was switched off. SSC Selection Posts XI-28/06/2023 (Shift-III) Ans. (a) : Reported speech of the above direct sentence is in assertive sentence. Its indirect speech will be1. 'Said to' change into 'told in' case of when reporting object (me) is given. 2. Conjunction "that" will be used in place of comma and inverted comma. 3. According to Narration rule, tense and pronoun will be changed. Correct SentenceHe told me that his phone was switched off. 14. Select the option that expresses the given sentence in indirect speech. He said, “Two and two make four.” (a) He says that two and two made four. (b) He said that two and two made four. (c) He said that two and two make four. (d) He says that two and two make four. SSC CGL Mains 26/10/2023 Direct and Indirect Speech

Ans. (c) : Reported speech of the above direct sentence is a fact. Its Indirect speech will be 1. Conjunction 'that' will be used in place of comma and inverted comma. 2. According to narration rule tense and pronoun will be changed. Correct Sentence He said that two and two make four. 15. Select the option that expresses the given sentence in indirect speech. Aravind said, “My teacher is correcting the answer sheets.” (a) Aravind said that his teacher was correcting the answer sheets. (b) Aravind said his teacher was be corrected the answer sheets. (c) Aravind said that his teacher is being correcting the answer sheets. (d) Aravind said that teacher is corrected the answer sheets. SSC Selection Posts XI-27/06/2023 (Shift-I) Ans. (a) : Reported speech of the above direct sentence is in assertive sentence. Its indirect speech will be made from the following rule – 1. Conjunction 'That' will be used in the place of comma and inverted comma. 2. According to the narration rule tense and pronoun will be changed. The correct indirect speech Aravind said that his teacher was correcting the answer sheets. 16. Select the correct indirect form of the given sentence. Sana said, "I am going to pick up the children now." (a) Sana said that she was going to pick up the children then. (b) Sana said that I am going to pick up the children now. (c) Sana said that I am going to pick up the children then. (d) Sana said that she was going to pick up the children now. SSC Stenographer (Grade C & D) 12.11.2021 Shift-I Ans. (a) : The above given sentence to change into Indirect Speech the following change will be– 1– Conjunction “that” will be used in place of comma and inverted comma. 2– According to the Narration Rules 'Tense and Pronoun' will be changed. 3– ‘Then’ will be used in the place of ‘Now’ 17. Select the correct direct form of the given sentence. I told Kavya that she was late for her class. (a) I said to Kavya, "She was late for her class." (b) I said to Kavya, "You were late for your class." (c) I said to Kavya, "why are you late for your class?" (d) I said to Kavya, "You are late for your class." SSC Stenographer (Grade C & D) 12.11.2021 Shift-I

347

YCT

Ans. (d) : The above given sentence is in indirect speech to change into direct speech the following change will be. 1. – ‘Said to’ will be used in the place of ‘told’ 2. – Comma, inverted comma will be used in the place of conjunction ‘that’. 3. – According to the Narration Rule, 'Tense and Pronoun' will be changed. 18. Select the option that expresses the given sentence in indirect speech. She said, "Ruhi slipped when she was trying to board the bus." (a) She said that Ruhi had slipped when she was trying to board the bus. (b) She says that Ruhi slipped when she was trying to board the bus. (c) She said that Ruhi has slipped when she was trying to board the bus. (d) She said that Ruhi slipped when she was trying to board the bus SSC CGL (Tier-II) 08.08.2022 Shift-II Ans. (a) : Reported speech of the above sentence is in Assertive Sentence. Hence for indirect speech, the following change will be – 1. – Conjunction ‘that’ will be used in place of comma and inverted comma. 2. – There will be no change in “said” in case of the reporting object is not given. 3 – According to the Narration Rule, 'Tense and pronoun' will be changed. 19. Select the most appropriate indirect form of the given sentence. Jayesh said to Diwakar, "You can top the class if you want to." (a) Jayesh told Diwakar that he would top the class if he wants to. (b) Jayesh told Diwakar that he could top the class if he wanted to. (c) Jayesh told Diwakar that you could top the lass if you wanted to. (d) Jayesh told Diwakar that you can top the class if you want to. SSC CGL (Tier-II) 08.08.2022 Shift-II Ans. (b) : Reported speech of the above sentence is in assertive sentence. In which modal verb 'can' is used. Hence for indirect speech, the following change will be– 1. Conjunction ‘that’ will be used in the place of comma and inverted comma. 2. “Said to” change into ‘told’ in case of the Reporting object (Diwakar) is given. 3. According to the Narration Rules 'Tense and Pronoun' will be changed. 20. Select the option that expresses the given sentence in direct speech. Your sister will say that she has lost her pen again. (a) Your sister said, "She has lost my pen again." (b) Your sister will say, "I have lost my pen again." (c) Your sister will say, "I lost my pen again." (d) Your sister says, "I lost her pen again." SSC CGL (Tier-II) 08.08.2022 Shift-II Direct and Indirect Speech

Ans. (b) : In the above sentence the use of conjunction “that” makes it clear that the given sentence is in Reported speech of Assertive sentence. Hence its change into direct speech the following Rules will be– 1.– Comma and inverted comma will be used in the place of conjunction “that”. 2. – There will be no change in tense in case of (will +say) (future tense) is used in Reporting Verb. 21.

Select the most appropriate direct form of the given sentence. The policeman told the boys that they could not park their car there. (a) The policeman said to the boys, "You could not park their car there." (b) The policeman said to the boys, "You cannot park your car here." (c) The policeman said to the boys, "How could you park your car here?" (d) The policeman told to the boys, "They could not park their car there." SSC CGL (Tier-II) 08.08.2022 Shift-II Ans. (b) : In the above sentence , it is clear from the use of ‘told’ and conjunction ‘that’, that the Reported speech will be in Assertive sentence of indirect speech. Hence, for direct speech following change will be 1.– 'Comma and inverted comma' will be used in place of conjunction ‘that’. 2.– ‘Said to’ will be used in place of ‘told’. 3.– According to the Narration Rules, 'Tense and pronoun' will be changed.

22.

Select the correct indirect narration of the given sentence. Grandmother said to me, "You have been careless again. Remember to be careful whenever you are assigned a task." (a) Grandmother told me that I was being careless again. She further said I had to be careful whenever I am assigned a task. (b) Grandmother told me that I had been careless again. She further remined me to be careful whenever I was assigned a task. (c) Grandmother said to me that I was being careless again. She further said I have to be careful whenever I was assigned a task. (d) Grandmother tells me that I have been careless again. She further reminds me to be careful whenever I am assigned a task. SSC Stenographer (Grade C & D) 15.11.2021 Shift-I Ans. (b) : The above given direct speech is assertive form of present perfect tense. Hence for indirect speech the following change will be 1- Conjunction 'that' will be used in the place of comma and inverted comma. 2- 'told' will be used in place of 'said to' in case of the reporting object (me) is given. 3- According to the Narration Rule, 'Tense and Pronoun' will be changed.

348

YCT

23.

Select the option that expresses the given sentence in reported speech. Harry said, "My son is in danger and I need help." (a) Harry said his son was in danger and I needed help. (b) Harry said that his son was in danger and he needed help. (c) His son said that Harry was in danger and that he needed help. (d) Harry said his son is in danger and I need help. SSC Stenographer (Grade C & D) 15.11.2021 Shift-I Ans. (b) : The given sentence is in direct speech of assertive form of Present Indefinite Tense . To change direct into indirect speech the following rules will be– 1- Conjunction "that" will be used in the place of comma and inverted comma. 2- According to Narration Rule, 'Tense and Pronoun' will be changed. 24. Select the option that expresses the given sentence in direct speech. Megha said that she had taken the shortcut to her college. (a) Megha said, "I took the shortcut to my college". (b) Megha says, "She takes the shortcut to my college". (c) Megha said, "I take the shortcut to my college". (d) Megha said, "She takes the shortcut to her college". SSC Stenographer (Grade C & D) 15.11.2021 Shift-I Ans. (a) : The above given sentence is in indirect speech of 'past perfect tense.' For direct speech the following changes will be1- Comma and inverted comma will be used in place of conjunction "that". 2- According to narration rules, Tense and Pronoun' will be changed. 25. Select the option that expresses the given sentence in indirect speech. She said, ''I can play the guitar pretty well.'' (a) She said that she would play the guitar pretty well. (b) She said that she could play the guitar pretty well. (c) She said that she plays the guitar pretty well. (d) She said that she can play the guitar pretty well. SSC CHSL 27.05.2022 Shift-II Ans. (b) : Reported speech of the above sentence is in assertive sentence. Hence its indirect speech will be changed according to narration rules 1- Conjunction 'that' will be used in the place of comma, inverted comma. 2- According to the Narration Rule, 'Tense and Pronoun' will be changed. Direct and Indirect Speech

26.

Select the option that expresses the given sentence in direct speech. Manan said that he had no work to do that day. (a) Manan says, "I had no work to do that day." (b) Manan said, "I have no work to do that day." (c) Manan said, " I have no work to do today." (d) Manan said, "Have I no work to do today?" SSC CGL (Tier-I) 13/04/2022 Shift-I Ans. (c) : The direct speech of the given sentence will be- Manan said, "I have no work to do today." In the sentence, 'said' will not change because reporting verb does not have an object, the conjunction 'that' will be removed and 'he' will be changed into 'I' because in direct to indirect transformation the first-person Pronoun is changed according to the subject of the reporting verb. Hence, the correct answer is (c). 27. Select the option that expresses the given sentence in indirect speech. She said, "The stars are shining brightly tonight." (a) She said the stars were shining brightly tonight. (b) She asked if the stars were shining brightly that night. (c) She said that the stars were shining brightly that night. (d) She said the stars are shining brightly tonight. SSC CGL (Tier-I) 11/04/2022 Shift-III Ans. (c) : In the above sentence is in direct speech hence its indirect speech will be made from the following change1- Conjunction 'that' will be used in the place of comma and inverted comma. 2- According to the narration rule, 'Tense and Pronoun' will be changed. 3- 'That night' will be used in place of 'tonight'. 28. Select the option that expresses the given sentence in reported speech. Mother said, “Abhinav slipped while trying to board a bus.” (a) Mother says that Abhinav slipped while trying to board a bus. (b) Mother said that Abhinav slipped while trying to board a bus. (c) Mother told that Abhinav slipped while trying to board a bus. (d) Mother said that Abhinav had slipped while trying to board a bus. SSC CGL (Tier-I) 12/04/2022 Shift-I Ans. (d) : The above sentence is in direct speech of past indefinite tense. Hence its indirect speech will be made from the following change1- Conjunction 'that' will be used in the place of comma and inverted comma. 2- According to the narration rule, 'Tense and Pronoun' will be changed. 29. Select the correct indirect form of the given sentence. Anju said to Vijaya, "You can stay with us whenever you are in Delhi."

349

YCT

(a) Anju told Vijaya that she could stay with them whenever she was in Delhi. (b) Anju told Vijaya that she could stay with them whenever you were in Delhi. (c) Anju told Vijaya that she can stay with us whenever she is in Delhi. (d) Anju told Vijaya that you can stay with us whenever you are in Delhi. SSC CHSL (Tier-I) –04/08/2021 (Shift-I) Ans. (a) : Reported speech in the above direct speech it is in an assertive sentence. The following rule will be used to convert it into indirect speech as1- Conjunction 'that' will be used in the place of comma and inverted comma. 2- Reporting verb 'said to' change into "told". 3- According to the narration rule, 'Tense and Pronoun' will be changed. 30. Select the correct indirect form of the given sentence. "I am going to walk to the market to check which shops are open," said Anant. (a) Anant said that he went for a walk to the market to check which shops were open. (b) Anant said that he will be going to walk to the market to check which shops were open. (c) Anant said that he is going to walk to the market to check which shops were open. (d) Anant said that he was going to walk to the market to check which shops were open. SSC CHSL (Tier-I) –16/04/2021 (Shift-I) Ans. (d) : Reported speech in the direct form of the above sentence is in assertive sentence, hence its indirect speech will be made from the following change1- Conjunction 'that' will be used in place of comma and inverted comma. 2- According to the narration rule, 'Tense and Pronoun' will be changed. 31. Select the correct indirect form of the given sentence. Sam said, "pa, I am a big boy now and don't need to drink milk." (a) Sam told his father that he was a big boy then and didn't need to drink milk. (b) Sam told his father that he is a big boy now and don't need to drink milk. (c) Sam told his father that I am a big boy now and don't need to drink milk. (d) Sam told his father that I would be a big boy now and won't need to drink milk. SSC CHSL (Tier-I) –15/04/2021 (Shift-I) Ans. (a) : Reported speech of the above direct speech is in assertive sentence. Hence its indirect speech will be made from the following change1- In case of reporting object being given, the reporting verb, 'said' will be change into 'told'. 2- Conjunction 'that' will be used in place of comma and inverted comma. 3- According to the narration rule, 'Tense and Pronoun' will be changed. 32. Select the correct indirect form of the given sentence. "I love Bollywood music," Deepu said to Reena. Direct and Indirect Speech

(a) Deepu told Reena that he loved Bollywood music. (b) Deepu will say to Reena that he loved Bollywood music. (c) Deepu says to Reena that he loves Bollywood music. (d) Deepu had told Reena that he loved Bollywood music. SSC CHSL (Tier-I) –06/08/2021 (Shift-I) Ans. (a) : It is clear from the reported speech of the above direct speech that the sentence is in assertive sentence. Hence its indirect speech will be made from the following change 1- Conjunction 'that' will be used in place of comma and inverted comma. 2- Repotting verb 'said to' will be changed into 'told'. 3- According to the narration rule, 'Tense and Pronoun' will be changed. 33. Select the correct indirect form of the given sentence. My niece said, "I have decided to take coaching in badminton." (a) My niece told me that she had decided to take coaching in badminton. (b) My niece told that I have decided to take coaching in badminton (c) My niece said she is deciding to take coaching in badminton (d) My niece said she will decide to take coaching in badminton SSC CHSL (Tier-I) –09/08/2021 (Shift-I) Ans. (a) : It is clear from the reported speech of the above direct speech that the sentence is in assertive sentence. Hence its indirect speech will be made from the following change1- Conjunction 'that' will be used in the place of comma and inverted comma. 2- According to the narration rule, 'Tense and Pronoun' will be changed. 34. Select the option that expresses the given sentence in reported speech. 'I live in Mumbai', she said. (a) She said that she had been living in Mumbai. (b) She said that she lived in Mumbai. (c) She says that she was living in Mumbai. (d) She said that she has been living in Mumbai. SSC CGL (Tier-I) –24/08/2021 (Shift-I) Ans. (b) : Reported speech of the above direct sentence is in assertive sentence. Hence its indirect speech will be made from the following change1- Conjunction 'that' will be used in place of comma and inverted comma. 2- According to the narration rule, 'Tense and Pronoun' will be changed. 35. Select the option that expresses the given sentence in indirect speech. Sneha said to me, 'I was watching TV'. (a) Sneha told me that I was watching TV. (b) Sneha said to me that I was watching TV. (c) Sneha told me that she had been watching TV. (d) Sneha told me that she was watching TV. SSC CGL (Tier-I) –18/08/2021 (Shift-I)

350

YCT

(c) The teacher says that magnets attract objects Ans. (c) : Reported speech of the above direct sentence made of iron. is in assertive sentence. Hence its indirect speech will (d) The teacher said that magnets attracted be made from the following changeobjects made of iron 1- In case of reporting object being given, the reporting SSC CGL (Tier-II) 16/11/2020 (3 pm - 5 pm) verb 'said to' will be changed into 'told'. 2- Conjunction 'that' will be used in place of comma and Ans. (c) : The above direct speech is in assertive inverted comma. sentence. Its indirect speech will be made from the 3- According to the narration rule, 'Tense and Pronoun' following change will be changed. 1- Conjunction 'that' will be used in the place of 36. Select the option that expresses the given comma and inverted comma. 2- Reporting verb (says) is in present tense. Hence no sentence in indirect speech. The professor said to the media persons, “From change in reported speech tense. our research we have come to the conclusion 39. Select the correct indirect form of the given sentence. that there is life on that planet.” (a) The professor informed the media persons He said to me, "Your father is waiting for you that from our research we have come to the at the reception." conclusion that there had been life on that (a) He told me that my father was waiting for me planet. at the reception. (b) The professor said to the media persons from (b) He told to me my father was waiting for you our research you had come to the conclusion at the reception. that there was life on that planet. (c) He told me that your father was waiting for (c) The professor said to the media persons that him at the reception. from their research they had come to the (d) He told me that his father was waiting for you conclusion that there was life on that planet. at the reception. (d) The professor told to the media persons that SSC CGL (Tier-II) 16/11/2020 (3 pm - 5 pm) from your research you have come to the Ans. (a) : Reported speech of the above sentence is in conclusion that there is life on that planet. assertive sentence . Hence its indirect speech will be SSC CGL (Tier-I) –20/08/2021 (Shift-I) made from the following change. Ans. (c) : Reported speech of the above direct speech is 1- Reporting verb 'said to' will be changed into 'told' in in assertive sentence. Hence its indirect speech will be case of object is given. made from the following change 2- Conjunction 'that' will be used in place of comma 1- Conjunction 'that' will be used in the place of comma and inverted comma. and inverted comma. 3- According to the narration rule, 'Tense and Pronoun' 2- According to the narration rule, 'Tense and Pronoun' will be changed. will be changed. 40. Select the correct indirect form of the given 37. Select the option that expresses the given sentence. sentence in indirect speech. He said to Manoj, "I celebrated my birthday 'Everything is going to be alright,' said the two days ago." (a) He told Manoj that he celebrated his birthday doctor. (a) The doctor said that everything will be two days ago. (b) He told Manoj that he had celebrated his alright. birthday two days before. (b) The doctor said that everything was going to be alright. (c) He told Manoj that he celebrated my birthday (c) The doctor said that everything is going to be two days before. alright. (d) He told Manoj that I celebrated by birthday (d) The doctor said that everything are going to two days ago. be alright. SSC CGL (Tier-II) 16/11/2020 (3 pm - 5 pm) SSC CGL (Tier-I) –13/08/2021 (Shift-I) Ans. (b) : Reported speech of the above sentence is in Ans. (b) : Reported speech of the above direct sentence assertive sentence. Its indirect speech will be made is in assertive sentence. Hence its indirect speech will from the following rulebe made from the following change1- Conjunction 'that' will be used in place of comma 1- Conjunction 'that' will be used in the place of comma and inverted comma. and inverted comma. 2- Reporting verb 'said to' change into told. 2- According to the narration rule, 'Tense and Pronoun' 3- According to the narration rule, 'Tense and Pronoun' will be changed. will be changed. 38. Select the correct indirect form of the given 41. Select the correct indirect form of the given sentence. sentence. The teacher says, "Magnets attract objects The Principal says, "Hard work is the key to made of iron." success." (a) The teacher says magnets attracts objects (a) The Principal say that hard work is the key to made of iron. success. (b) The teacher said that magnets were attracting (b) The Principal said that hard work was the key objects made of iron. to success. Direct and Indirect Speech

351

YCT

(c) The Principal says that hard work is the key Ans. (c) : To convert the above sentence in to direct to success. speech the following rules will be applied(d) The Principal says that hard work was the key 1- Comma (,) and inverted (" ") comma will be used to success. into the place of conjunction "that". SSC CGL (Tier-II) 16/11/2020 (3 pm - 5 pm) 2- Reporting verb 'told' change into "said to". Ans. (c) : Reporting verb (says) of the above sentence 3- According to the narration rule, 'Tense and Pronoun' is in present tense. Hence not any change in tense of will be changed. reported speech only conjunction 'that' will be used in 45. Select the correct direct form of the given place of comma and inverted comma. sentence. 42. Select the correct indirect form of the given She told her brother that she was going to meet sentence. her friend. The Prime Minister has said, "The government (a) She said to her brother, "I am going to meet will extend help to the unorganised sector." my friend". (a) The Prime Minister said that the government (b) She said to her brother, "I was going to meet would extend help to the unorganised sector my friend". (b) The Prime Minister has said that the (c) She said to her brother, "She will go to meet government will extend help to the her friend. unorganised sector (d) She said to her brother, "I am gone to meet (c) The Prime Minister said that the government her friend". has extended help to the unorganised sector SSC CGL (Tier-II) 16/11/2020 (3 pm - 5 pm) (d) The Prime Minister has said that the government extended help to the Ans. (a) : Above indirect speech is in assertive sentence. Its direct speech will be made from the unorguanised sector. SSC CGL (Tier-II) 16/11/2020 (3 pm - 5 pm) following rule1- Comma (,) and inverted comma (" ") will be used in Ans. (b) : Reporting verb of the above sentence is in the place of conjunction 'that'. present tense. Hence not any change in tense of 2- Reporting verb 'told' change into 'said to'. reported speech. Only conjunction 'that' will be used in 3- According to the narration rule, 'Tense and Pronoun' the place of comma and inverted comma. 43. Select the correct indirect form of the given will be changed. 46. Select the correct indirect form of the given sentence. sentence. The tailor said to him, "Your shirt will be The librarian said to her, "You can borrow ready by tomorrow." only two books at a time". (a) The tailor told him that your shirt would be (a) The librarian told her that she could borrow ready by tomorrow. only two books at a time (b) The tailor told him that his shirt will be ready (b) The librarian told her that you can borrow by tomorrow. (c) The tailor told to him that your shirt will be only two books at a time ready by the next day. (c) The librarian told her that they could borrow (d) The tailor told him that his shirt would be only two books at a time ready by the next day. (d) The librarian told her that she can borrow SSC CGL (Tier-II) 16/11/2020 (3 pm - 5 pm) only two books at a time Ans. (d) : Reported speech of the above direct SSC CGL (Tier-II) 16/11/2020 (3 pm - 5 pm) sentence is in assertive sentence . Its indirect speech Ans. (a) : Reported speech of the above sentence is in will be made from the following ruleassertive sentence. Its indirect speech will be made 1- Conjunction 'that' will be used in the place of from the following rulecomma and inverted comma. 1- Conjunction "that" will be used in the place of 2- Reporting verb 'said to' change into 'told' in case of comma (,) and inverted comma (" "). reporting object (him) is given. 2- Reporting verb 'said to' change into "told" in case of 3- According to the narration rule, 'Tense and Pronoun' reporting object (her) is given. will be changed. 3- According to the narration rule tense and pronoun 4- 'Tomorrow' change into "the next day". will be changed. 44. Select the correct direct form of the given 47. Choose the option that is the indirect form of sentence. the sentence. I told him that if he went around the park he "I'm hungry" said the child to his mother. would see some rare flowers. (a) The child said his mother that he is hungry. (a) I said to him, "If you go around the park you (b) The child said to his mother that I am hungry. would saw some rare flowers." (c) The child told his mother that she was (b) I said to him, "If he went around the park he hungry. will see some rare flowers." (d) The child told his mother that he was hungry. (c) I said to him, "If you go around the park you SSC CPO-SI (Tier-II) 27/09/2019 (3 pm - 5 pm) will see some rare flowers." (d) I said to him, "If you went around the park Ans. (d) : Reported speech of the above sentence is in assertive sentence. Its indirect speech will be made you would see some rare flowers." SSC CGL (Tier-II) 16/11/2020 (3 pm - 5 pm) from the following ruleDirect and Indirect Speech

352

YCT

1- Conjunction 'that' will be used in the place of Ans. (c) : Above indirect speech is in assertive comma and inverted comma. sentence its direct speech will be made from the 2- Reporting verb 'said to' change into ''told'' in the following change. case of reporting object (mother) is given. 1- Comma (,) and inverted comma (" ") will be used in 3- According to the narration rule, Tense and Pronoun the place of conjunction 'that'. will be changed. 48. Choose the option that is the indirect form of 2- According to the narration rule, Tense and Pronoun will be changed. the sentence. 52. Choose the option that is the indirect form of Kishore said“I am leaving now.” the sentence. (a) Kishore said that he was leaving then. She said to him, "I don't want to go there." (b) Kishore said that he left then. (a) She said he didn’t want to go there. (c) Kishore says that I am leaving now. (d) Kishore said that he had to leaving then. (b) She said to him that she didn’t want to go SSC CPO-SI (Tier-II) 27/09/2019 (3 pm - 5 pm) there. (c) He said to her that she didn’t want to go Ans. (a) : Reported speech of the above sentence is in assertive sentence. Its indirect speech will be made there. from the following rule (d) She said to him that you do not want to go 1- Conjunction 'that' will be used in the place of there. comma and inverted comma. SSC CPO-SI (Tier-II) 27/09/2019 (3 pm - 5 pm) 2- According to the narration rule, Tense and Pronoun Ans. (b) : Reported speech of the above sentence is in will be changed. assertive sentence. Its indirect speech will be made 49. Choose the option that is the direct form of the from the following rulesentence. 1- Conjunction 'that' will be used in the place of Abdul said that he had seen that film the day comma and inverted comma. before. 2- According to the narration rule, Tense and Pronoun (a) Abdul said, “I had seen this film the day will be changed. before.” 53. Choose the option that is the direct form of the (b) Abdul said, “I saw this film yesterday.” sentence. (c) Abdul said, “I saw that film yesterday.” Vikas said to Navin that he hadn't met him (d) Abdul said, “I see this film on the previous since February the previous year. day.” (a) Vikas told Navin, “You haven’t meeting me SSC CPO-SI (Tier-II) 27/09/2019 (3 pm - 5 pm) since February last year.” Ans. (b) : In the above sentence conjunction ''that'' is (b) Vikas said to Navin, “I haven’t met you since used . Hence makes it clear that indirect speech is in assertive sentence. Its direct speech will be made from February last year.” the following rule(c) Vikas told to Navin, “I haven’t met him since 1- Comma (,) and inverted (" ") comma will be used in February last year.” the place of conjunction "that". (d) Vikas said, “Navin, I haven’t meet you since 2- According to the narration rule, Tense and Pronoun February last year.” will be changed and 'day before' change into SSC CPO-SI (Tier-II) 27/09/2019 (3 pm - 5 pm) 'yesterday'. Ans. (b) : Above indirect speech makes it clear that 50. Choose the option that is the direct form of the the sentence is in assertive sentence. Its direct speech sentence. will be made from the following ruleDevi replied that she was sorry but she could 1- Comma (,) and inverted comma (" ") will be used in not go. the place of conjunction 'that'. (a) Devi replied, “ She was sorry I could not go.” 2- According to the narration rule, Tense and Pronoun (b) Devi replied, “Sorry she could not go.” will be changed. (c) Devi replied, “I’m sorry but I cannot go.” 3- 'The previous year' change into 'last year'. (d) Devi replies, “I could not went but sorry.” SSC CPO-SI (Tier-II) 27/09/2019 (3 pm - 5 pm) 54. Choose the option that is the indirect form of the sentence. Ans. (c) : In the above indirect speech the use of “I have joined computer classes” Rudra said. conjunction 'that' makes it clear that the sentence is in (a) Rudra said that he had joined computer assertive. Its direct speech will be made from the following ruleclasses. 1- Comma (,) and inverted comma (" ") will be used in (b) Rudra said that I am joining computer classes. the place of conjunction "that". (c) Rudra said that I joined computer classes. 2- According to the narration rule, Tense and Pronoun (d) Rudra said that I join computer classes. will be changed and 'could not' change into 'can not'. SSC CPO-SI (Tier-II) 27/09/2019 (3 pm - 5 pm) 51. Choose the option that is the direct form of the Ans. (a) : Reported speech of the above sentence is in sentence. assertive sentence. Its indirect speech will be made Jai said that he wanted to be a soldier. from the following rule(a) Jai said, “I wanted to be a soldier.” 1- Conjunction 'that' will be used in the place of (b) Jai said that, “I wanted to become a soldier.” comma and inverted comma. (c) Jai said, “I want to be a soldier.” (d) Jai said, “He wants to become a soldier.” 2- According to the narration rule, Tense and Pronoun SSC CPO-SI (Tier-II) 27/09/2019 (3 pm - 5 pm) will be changed. Direct and Indirect Speech

353

YCT

55.

Choose the option that is the direct form of the 1- Conjunction 'that' will be used in the place of sentence. comma and inverted comma. The Principal said to them that he did not want 2- According to the narration rule, Tense and Pronoun to see any one to return with a complaint will be changed. against them. 58. Select the correct indirect form of the given (a) The Principal said to them, “I did not want to sentence. saw any one to return with a complaint Raju said to his friend, "I lost my book against them.” yesterday". (b) The Principal said to them, “I do not want to (a) Raju told his friend that he had lost his book see any one to return with a complaint against the previous day you.” (b) Raju told his friend that his book may have (c) The Principal said to them, “I do not wanted been lost the previous day to see any of them to be returning with a (c) Raju said to his friend that he lost his book complaint.” the previous day (d) The Principal said to them, “I do not want to (d) Raju said to his friend that his book had been see any one to returns with a complaint lost the previous day against you.” SSC Stenographer Grade C&D –05/02/2019 (3:5pm) SSC CPO-SI (Tier-II) 27/09/2019 (3 pm - 5 pm) Ans. (a) : Reported speech of the above sentence is in Ans. (d) : Reported speech of the above sentence is in assertive sentence. Its indirect speech will be made assertive. Its direct speech will be made from the from the following rulefollowing rule1- 'Said to' change into told in the case of reporting 1- Comma (,) and inverted comma (" ") will be used in object (his friend) is given the place of conjunction 'that' 2- Conjunction 'that' will be used in the place of 2- According to the narration rule, Tense and Pronoun comma and inverted comma. will be changed. 56. Choose the option that is the indirect form of 3- According to the narration rule, Tense and Pronoun will be changed. the sentence. 59. Select the correct indirect form of the given “We are going to Tirupati next week,” Deepa sentence. told her friends. Raj said, "I may buy a new mobile phone (a) Deepa told her friends that they will go to tomorrow." Tirupati next week. (a) Raj said that he may be buying a new mobile (b) Deepa told to her friends that she is going to phone the next day Tirupati following week. (b) Raj said that he might buy a new mobile (c) Deepa told her friends that we were going to phone the next day Tirupati the following week. (c) Raj told me that he might buy a new mobile (d) Deepa told her friends that they were going to phone tomorrow Tirupati the following week. (d) Raj told me that he would buy a new mobile SSC CPO-SI (Tier-II) 27/09/2019 (3 pm - 5 pm) phone the next day Ans. (d) : Reported speech of the above sentence is in SSC Stenographer Grade C&D –05/02/2019 (3:5pm) assertive. Its indirect speech will be made from the Ans. (b) : Reported speech of the above sentence is in following ruleassertive sentence. Its indirect speech will be made 1- Conjunction 'that' will be used in the place of from the following rulecomma and inverted comma. 1- Conjunction 'that' will be used in the place of 2- According to the narration rule, Tense and Pronoun will be changed. comma and inverted comma. 3- 'Next week' change into 'The following week'. 2- 'May' change into 'might' and 'tomorrow' change 57. Select the correct indirect form of the given into 'The next day'. 3- According to the narration rule, Pronoun will be sentence. The boy said, "I am happy to receive the changed. prize." 60. Select the correct indirect form of the given (a) The boy said that he is happy to receive the sentence. prize The manager said to his assistant, "You will get (b) The boy said that he was happy receiving the a bonous next month." prize (a) The manager tells his assistant that he will get (c) The boy said that he was happy to receive the a bonus the following month prize (b) The manager told his assistant that he will be (d) The boy said that he was happy to have getting a bonus the following month received the prize (c) The manager told his assistant that he would SSC Stenographer Grade C&D –05/02/2019 (3:5pm) get a bonus the following month Ans. (c) : Reported speech of the above sentence is in (d) The manager told his assistant that he should assertive sentence. Its indirect speech will be made get a bonus the following month from the following changeSSC Stenographer Grade C&D –05/02/2019 (3:5pm) Direct and Indirect Speech

354

YCT

Ans. (c) : Reported speech of the above sentence is in assertive sentence. Its indirect speech will be made from the following rule1- Reporting verb 'said to' change into 'told'. 2- Conjunction 'that' will be used in the place of comma and inverted comma. 3- According to the narration rule, Tense and Pronoun will be changed. 4- 'Next month' converted to the 'following month'. 61. Select the correct indirect form of the given sentence. They teacher says, "The Sun rises in the East." (a) The teacher says that the Sun rises in the East (b) The teacher said that the Sun rises in the East (c) The teacher says that the Sun rose in the East (d) The teacher told that the Sun rose in the East SSC Stenographer Grade C&D –05/02/2019 (3:5pm)

Ans. (a) : Reported speech of the above sentence is a universal truth. Its indirect speech will be made from the following rule1- When we talk about universal truth in the sentence there is no change in the tense of reported speech. 2- Conjunction 'that' will be used in the place of comma and inverted comma. 62. The jeweler said to us, "These diamonds are polished here." (a) The jeweler informed us that these diamonds were polished there. (b) The jeweler informed us that those diamonds were polished there. (c) The jeweller informed us that those diamonds were polished here. (d) The jeweller informed us that these diamonds were polished here. SSC CGL (Phase-II) 17/02/2018 Ans. (b) Reported speech of the above sentence is in assertive sentence. Its indirect speech will be made from the following rule1- In reporting verb 'said to' change into 'informed'. 2- Conjunction 'that' is used in the place of comma and inverted comma. 3- According to the narration rule, Tense and Pronoun will be changed. 63. The old man said to me, "Life has taught me some hard lessons." (a) The old man tells me that life has taught him some hard lessons. (b) The old man told me this that life has had taught him some hard lessons. (c) The old man tells me that life is teaching him some hard lessons. (d) The old man told me that life had taught him some hard lessons. SSC CGL (Phase-II) 17/02/2018 Ans. (d) Reporting speech of the above sentence is in assertive sentence. Its indirect speech will be made from the following rule. 1- 'Said to' change into 'told'. 2- Conjunction 'that' will be used in the place of comma and inverted comma. 3. According to the narration rule, Tense and Pronoun will be changed. Direct and Indirect Speech

64.

The groom said to the bride, "I will take good care of you." (a) The groom promised the bride that he would take good care of her. (b) The groom has promised the bride that he will be taking good care of her. (c) The groom had promised the bride that he would have taken good care of her. (d) The groom promises the bride that he would take good care of her. SSC CGL (Phase-II) 17/02/2018 Ans. (a) Reporting speech of the above sentence is in 'assertive sentence'. Its indirect speech will be made from the following rule1- 'Said to' change into 'promised' because 'I, + will' → show 'compulsion', and 'promise' etc. 2- Conjunction 'that' will be used in the place of comma and inverted comma. 3- According to the narration rule, Tense and Pronoun will be changed. 65. He said to his partner, "I will not be responsible if the project fails." (a) He has told his partner that he will not be responsible if the project fails. (b) He told his partner that he would not be responsible if the project failed. (c) He tells his partner that he would not be responsible if the project fails. (d) He told his partner that I will not be responsible if the project failed. SSC CGL (Phase-II) 17/02/2018 Ans. (b) Reported speech of the above sentence is in 'assertive sentence'. Its indirect speech will be made from the following rule1- 'Said to' change into 'told'. 2- Conjunction 'that' will be used in place of 'comma and inverted comma'. 3- According to the narration rule, 'Tense and Pronoun' will be changed. 66. I said to my father, "I want to build a raft which can hold four people ." (a) I told my father that I am wanting to build a raft which could hold four people. (b) I told my father that that I wanted to build a raft which could hold four people. (c) I told my father that that I am wanting to build a raft which could hold four people. (d) I told my father that I wanted to build a raft which could hold four people. SSC CGL (Phase-II) 17/02/2018 Ans. (d) Reporting speech of the above sentence is in 'assertive sentence'. Its indirect speech will be made from the following rule. 1- 'Said to' change into 'told'. 2- Conjunction 'that' will be used in place of comma and inverted comma. 3- According to the narration rule, tense and pronoun will be changed.

355

YCT

67.

My brother said, "I can climb this hill in less than an hour." (a) My brother claimed that he can climb that hill in less than an hour. (b) My brother claimed that he could climb this hill in less than an hour. (c) My brother claimed that he can climb this hill in less than an hour. (d) My brother claimed that he could climb that hill in less than an hour. SSC CGL (Phase-II) 17/02/2018 Ans. (d) 'Reported speech' of the above sentence is in 'assertive sentence'. Its indirect speech will be made from the following rule1- 'Said to' change into 'claimed' according to sense of the sentence. 2- Conjunction 'that' will be used in the place of 'comma and inverted comma'. 3- According to the narration rule, 'Tense and Pronoun' will be changed. 68. The officer said, "I am very busy now". (a) The officer said that he was very busy now. (b) The officer said that he is very busy then. (c) The officer said that he is very busy now. (d) The officer said that he was very busy then. SSC CGL (Phase-II) 17/02/2018 Ans. (d) 'Reported speech' of the above sentence is in 'assertive sentence'. Its indirect speech will be made from the following rule1- Conjunction 'that' will be used in place of 'comma and inverted comma'. 2- According to the narration rule, 'Tense and Pronoun' will be changed. 69. "I am proud of you," father said to me. (a) Father told me that he is proud of me. (b) Father told me that he was proud of me. (c) Father told me this that he was proud of me. (d) Father told me that that he is proud of me. SSC CGL (Phase-II) 17/02/2018 Ans. (b) 'Reporting verb' of the above sentence is in 'past tense' and 'reported speech' in 'assertive sentence'. So its indirect speech will be made from the following rule1- 'Said to' change into 'told'. 2- Conjunction 'that' will be used in place of 'comma and inverted comma'. 3- According to the narration rule, 'Tense and Pronoun' will be changed. 70. "The taxi is here," said the watchman. (a) The watchman said that the taxi is there. (b) The watchman said that the taxi was here . (c) The watchman said that the taxi is here. (d) The watchman said that the taxi was there. SSC CGL (Phase-II) 17/02/2018 Ans. (d) Reported speech of the above sentence is in 'assertive sentence'. Its indirect speech will be made from the following rule1- 'That' is used in place of 'comma and inverted comma'. 2- 'Tense and pronoun' will be changed according to the narration rule. Direct and Indirect Speech

71.

The student said, "I must study hard." (a) The student said that he had to study hard. (b) The student says that he had to study hard. (c) The student says that he must study hard. (d) The student say that he had to study hard. SSC CGL (Phase-II) 17/02/2018 Ans. (a) Reported speech of the above sentence is in 'assertive sentence'. Its indirect speech will be made from the following rule1- 'That' is used in place of 'comma and inverted comma'. 2- According to the narration rule, 'Tense and Pronoun' will be changed. 72. My brother said to me, "I was upset, but now I am fine." (a) My brother told me that he was upset, but now he was fine. (b) My brother told me that he had been upset, but then he will be fine. (c) My brother told me that he had been upset, but then he was fine. (d) My brother told me that he has been upset, but then he is fine. SSC CGL (Phase-II) 17/02/2018 Ans. (c) Reported speech of the above sentence is in 'assertive sentence'. Its indirect speech will be made from the following rule1- 'Said to' change into 'told'. 2- 'That' is used in place of 'comma and inverted comma'. 3- According to the narration rule, 'Tense and Pronoun' will be changed. 4- 'Now', change into 'then'. 73. My friend said to me, "For me running is like therapy." (a) My friend told me that for her running is like therapy. (b) My friend told me that for her running was like therapy. (c) My friend told me that for her having ran was like therapy. (d) My friend tells me that for her running is like therapy. SSC CGL (Phase-II) 17/02/2018 Ans. (b) Reported speech of the above sentence is in 'assertive sentence'. Its indirect speech will be made from the following rule1- 'Said to' change into 'told'. 2- 'That' is used in the place of 'comma and inverted comma'. 3- According to the narration rule, 'Tense and Pronoun' will be changed. 74. I said to the taxi driver, "You must be crazy to drive so fast." (a) I said to the taxi driver that he has to be crazy to drive so fast. (b) I told the taxi driver that he had to be crazy to drive so fast. (c) I had told the taxi driver that he has to be crazy to drive so fast. (d) I had said to the taxi driver that he has to be crazy to be driving so fast. SSC CGL (Phase-II) 17/02/2018

356

YCT

Ans. (b) 'Reporting verb' of the above sentence is in 'past tense' and 'reported speech' is in 'assertive sentence'. Its indirect speech will be made from the following rule1- 'Said to' change into 'told'. 2- 'That' is used in place of comma and inverted comma. 3- According to the narration rule, 'Tense and Pronoun' will be changed. 75. "My train will reach by noon," he explained. (a) He explained that his train will reach by noon. (b) He explained that his train would reach by noon. (c) He explained that his train will have reached by noon. (d) He explained that his train would have had reached by noon. SSC CGL (Phase-II) 17/02/2018 Ans. (b) Reported speech of the above sentence is in 'assertive sentence'. Its indirect speech will be made from the following rule1- 'That' is used in place of comma and inverted comma. 2- 'Will' change into 'would' and 'my' change into 'his'. 3- According to the narration rules. 76. Select the correct indirect form of the given sentence. The children said, "We are bored of sitting at home." (a) The children said they were getting bored of sitting at home (b) The children said that they were bored of sitting at home (c) The children say that they are bored of sitting at home (d) The children are saying that they are bored of sitting at home SSC Sel. Post Phase-VIII (H.L.) 09.11.2020 (Shift-I)

Ans. (b) : Reported speech of the above sentence is in 'assertive sentence'. Its indirect speech will be made from the following rule1- There will be no change in the reporting verb 'said' in case of reported speech in assertive and reporting object is not given. 2- 'that' is used in place of 'comma and inverted comma'. 3- According to the narration rule, 'Tense and Pronoun' will be changed. 77. Select the correct indirect form of the given sentence. Mahesh said to the interviewer, "Sir, I was in charge of the production unit at my previous office." (a) Mahesh told the interviewer that Sir was in charge of the production unit at my previous office. (b) Mahesh told the interviewer I was in charge of the production unit at his previous office. (c) Mahesh told the interviewer you were in charge of the production unit at my previous office. (d) Mahesh told the interviewer that he had been in charge of the production unit at his previous office. SSC CHSL (Tier-I) –11/07/2019 (Shift-II) Direct and Indirect Speech

Ans : (d) Reported speech of the above sentence is in assertive sentence. Its indirect speech will be made from the following rule1- Conjunction 'that' is used in place of 'comma and inverted comma'. 2- 'Said to' change into 'told'. 3- According to the narration rule, 'Tense and Pronoun' will be changed. 78. Select the correct indirect form of the given sentence. The boss said to his employee, "I will be happy if you finish this project by evening." (a) The boss told his employee that he would be happy if he will finish this project by evening. (b) The boss told his employee he will be happy if he finished this project by evening. (c) The boss told his employee that he could be happy if he finishes that project by evening. (d) The boss told his employee that he would be happy if he finished that project by evening. SSC CHSL (Tier-I) –11/07/2019 (Shift-III) Ans : (d) Reported speech of the above direct sentence, is in assertive sentence. Its indirect speech will be made from the following rule1- 'That' is used in place of comma and inverted comma. 2- 'Said to' will be changed into 'told' in case of reporting object (employees) is given. 3- According to the narration rule, 'Tense and Pronoun' will be changed. 79. Select the most appropriate indirect form of the given sentence. She says, "I go for a walk every morning." (a) She says that she goes for a walk every morning. (b) She said that she goes for a walk every morning. (c) She says that I go for a walk every morning. (d) She said that she went for a walk every morning. SSC CHSL (Tier-I) –08/07/2019 (Shift-II) Ans : (a) The above given 'direct sentence' is in 'assertive sentence'. It can be changed in indirect speech from the following rule1. Conjunction 'that' will be used in place of 'comma and inverted comma.' 2. There will be no change in the reporting verb 'says' because It has no object. 3. Reporting verb (says) is in the 'Present Indefinite tense', therefore, No change of the tense of reported speech. 4. 'I' (the first person pronoun), will be changed according to the reporting verb 'subject' (she). 80. Select the most appropriate indirect form of the given sentence. She said, "Deepak slipped when he was trying to skate." (a) She said that Deepak slipped when he was trying to skate. (b) She said that Deepak has slipped when he was trying to skate. (c) She said that Deepak was slipping when he was trying to skate. (d) She said that Deepak had slipped when he was trying to skate. SSC CHSL (Tier-I) –04/07/2019 (Shift-II)

357

YCT

Ans : (d) Reported speech of the above sentence is in 'assertive sentence'. Its indirect speech will be changed by the following rule1- 'That' is used in place of 'comma and inverted comma'. 2- 'Said to' change into 'told'. 3- According to the narration rule, 'Tense and Pronoun' will be changed. 81. Select the correct indirect form of the given sentence. Rama said to me, "I need your help for my new assignment." (a) Rama says to me that she needs my helps for her new assignment. (b) Rama said to me that she is needing my helps for her new assignment. (c) Rama said to me she needs your help for the new assignment. (d) Rama told me that she needed my help for her new assignment. SSC CHSL (Tier-I) 26/10/2020 (Shift-II) Ans. (d) : Reported speech of the above sentence is in 'assertive sentence'. Its indirect speech will be made from the following rule . 1- Reporting verb 'said to' change into 'told'. 2- 'that' is used in place of 'comma and inverted comma'. 3- According to the narration rule, 'Tense and Pronoun' will be changed. 82. Select the correct Indirect form of the given sentence. The minister said, "It is tragic that the Australian bush fires have claimed the lives of so many helpless animals." (a) The minister said that it was tragic that the Australian bush fires had claimed the lives of so many helpless animals. (b) The minister said that it was tragic that the Australian bush fires claimed the lives of so many helpless animals. (c) The minister said that it was tragic that the Australian bush fires were claiming the lives of so many helpless animals. (d) The minister said that it was tragic that the Australian bush fires have been claiming the lives of so many helpless animals. SSC CHSL (Tier-I) 26/10/2020 (Shift-II) Ans. (a) : Reported speech of the above sentence is in 'assertive sentence'. Its indirect speech will be made from the following rule1- No change in reporting verb 'said'. 2- 'that' is used in place of 'comma and inverted comma.' 3- According to the narration rule, 'Tense and Pronoun' will be changed. 83. Select the correct indirect form of the given sentence. Roshan said to me, ''Sandeep made a new year greeting card for his teacher''. (a) Roshan said Sandeep had made a year greeting card for his teacher. (b) Roshan was saying that Sandeep made a new year greeting card for his teacher. Direct and Indirect Speech

(c) Roshan told me that Sandeep had made a new greeting card for his teacher. (d) Sandeep made a new year greeting card for his teacher said Roshan. SSC CHSL (Tier-I) 15/10/2020 (Shift-II) Ans. (c) : Reported speech of the above sentence is in 'assertive sentence'. Its indirect speech will be made from the following rule1- Conjunction 'that' is used in place of 'comma and inverted comma.' 2- Reporting verb 'said to' change into 'told' in case of reporting object (me) is given. 3- According to the narration rule, 'Tense and Pronoun' will be changed. 84. Select the correct indirect form of the given sentence. Peter said, "He broke his glasses." (a) Peter said that he break his glasses. (b) Peter said that he had broken his glasses. (c) Peter said that he is breaking his glasses. (d) Peter said that he is going to break his glasses. SSC CHSL (Tier-I) 20/10/2020 (Shift-III) Ans. (b) : Reported speech of the above sentence is in an 'assertive sentence'. Its indirect speech will be made from the following rule1- Conjunction 'that' is used in place of 'comma and inverted comma.' 2- No change in reporting verb 'said' in case of reporting verb object is not given. 3- According to the narration rule, 'Tense and Pronoun' will be changed. 85. Select the option that is the direct form of the given sentence. Explaining her work she said that she made films for posterity to remember. (a) Explaining her work, she said, "I will make the films that will remember posterity." (b) Explaining her work, she said, "She makes films for posterity to remember." (c) Explaining her work, she said, "She made films for posterity to remember." (d) Explaining her work, she said, "I make films for posterity to remember." SSC CHSL (Tier-I) 20/10/2020 (Shift-II) Ans. (d) : The above 'indirect sentence' in assertive sentence. Its direct speech will be made from the following rule1- 'Comma and inverted comma' is used in place of conjunction 'that'. 2- According to the narration rule, 'Tense and Pronoun' will be changed.

B. 86.

358

Interrogative Sentence Choose the option that is the correct indirect form of the sentence. Rahul asked me, "Are we going to New York next year?" (a) Rahul asked that we are going to New York next year. (b) Rahul asked me that were we going to New York next year. YCT

(c) Rahul asked me that would we go to New York next year? (d) Rahul asked me if we were going to New York next year. SSC CHSL (Tier-I) 17/03/2023 (Shift-III) Ans. (d) : Direct Speech of the above sentence is interrogative sentence. Hence its Indirect Speech will be(1) Question mark (?) change into full stop (.). (2) Interrogative sentence change into Assertive Sentence. (3) According to Narration Rule Tense and Pronoun will be changed. (4) If/whether will be used in place of comma and inverted comma. Correct indirect form Rahul asked me if we were going to New York next year. 87. Select the option that expresses the given sentence in direct speech. The ticket collector asked the passengers where their tickets were. (a) The ticket collector said to the passengers, “Where your tickets were?” (b) The ticket collector said to the passengers, “Where were your tickets?” (c) The ticket collector said to the passengers, “Where are your tickets?” (d) The ticket collector demanded the passengers, “Show me your tickets” SSC Selection Posts XI-28/06/2023 (Shift-III) Ans. (c) : Reported speech of the above Indirect sentence is in Interrogative sentence. Its direct speech will be1. 'Asked' change into 'said to' in case of when reporting object is given. 2. Comma and inverted comma will be used in place of wh-word (conjunction). 3. According to Narration Rule tense and pronoun will be changed. 4. Assertive sentence changed into Interrogative sentence. 5. Question mark (?) will be used in place of full stop (.). 88. Select the correct indirect form of the given sentence. "Would you like to join me in the Earth Day campaign I am planning?" I said to Reena. (a) I asked Reena if she would like to join me in the Earth day campaign I was planning. (b) I asked Reena If she will like to join me in the Earth day campaign I was planning. (c) I asked Reena would if she would like to join me in the Earth day campaign I was planning. (d) I am asking Reena if she is ready to join me in the Earth day campaign I am planning. SSC Stenographer (Grade C & D) 12.11.2021 Shift-I Ans. (a) : The above sentence is in the interrogative of past tense. Its indirect narration will be made from the following rule1- 'Said to' change into 'asked'. Direct and Indirect Speech

2- Conjunction 'if/whether' is used in place of 'comma and inverted comma.' 3- According to the narration rule, 'Tense and Pronoun' will be changed. 89. Select the most appropriate direct form of the given sentence. The driver asked a passerby if he could tell him the way to the market. (a) The driver said to a passerby, "Tell me the way to the market, will you" (b) The driver said to a passerby, "Please, will you tell me the way to the market?" (c) The driver said to a passerby, "Can you tell me the way to the market"? (d) The driver said to a passerby, "Could he tell him the way to the market?" SSC CGL (Tier-II) 08.08.2022 Shift-II Ans. (c) : In the above sentence the use of 'asked' and conjunction 'if' makes it clear that that reported speech is in interrogative sentence (Yes/No type), its direct speech will be made from the following rule1- 'Comma and inverted comma' will be used in place of conjunction 'if'. 2- According to the narration rule, 'tense and pronoun' will be changed. 3- 'Said to' is used in place of 'asked'. 4- Change the latter clause from 'assertive' to 'interrogative sentence' and in place of full stop (.), question mark (?) will be used. 90. Select the option that expresses the given sentence in direct speech. The officer asked the accountant why the accounts had not been reconciled. (a) The officer said to the accountant, "Why the accounts have not been reconciled?" (b) The officer said to the accountant, "Why the accounts had not been reconciled?" (c) The officer said to the accountant, "Why have the accounts not been reconciled?" (d) The officer said to the accountant, "Why have the accounts not being reconciled?" SSC Stenographer (Grade C & D) 11.11.2021 Shift-I Ans. (c) : The given sentence is in indirect speech which is an interrogative sentence, its direct speech will be made from the following rule1- 'Asked' change into 'said to'. 2- 'Comma and inverted comma' will be used in place of question word 'why'. 3- According to the narration rule, 'Tense and Pronoun' will be changed. 4- Change to the latter clause from assertive to interrogative sentence. 5- In place of full stop. (.), question mark (?) will be used. 91. Select the correct indirect form of the given sentence. "Aren't you afraid of travelling alone in a foreign country?" I asked Aditi.

359

YCT

(a) I asked Aditi whether she was afraid of traveling alone in a foreign country. (b) I asked Aditi if you aren't afraid of traveling alone in a foreign country. (c) I asked Aditi whether you are afraid of traveling alone in a foreign country. (d) I asked Aditi if she wasn't afraid of traveling alone in a foreign country. SSC Stenographer (Grade C & D) 15.11.2021 Shift-II Ans. (d) : Above direct sentence is in interrogative of present continuous tense. Its indirect speech will be made from the following rule1- 'If' is used in place of 'comma and inverted comma' as a conjunction. 2- According to the narration rule, 'tense and pronoun' will be changed. 92. Select the option that expresses the given sentence in reported speech. David said, "Can someone please explain what is going on?" (a) David asked that if someone could explain what is going on. (b) David asked whether someone could explain what was going on. (c) David asked if someone can please explain what was going on. (d) David said can someone please explain what is going on. SSC Stenographer (Grade C & D) 15.11.2021 Shift-II Ans. (b) : The above sentence is an interrogative sentence of present indefinite tense. Its indirect narration will be made from the following rule1- 'Said' change into 'asked'. 2- Conjunction 'whether' is used in place of 'comma and inverted comma'. 3- According to the narration rule, 'Tense and Pronoun' will be changed. 93. Select the option that expresses the given sentence in direct speech. She asked me what my favourite cuisine was. (a) She said to me, ''What your favourite cuisine is?" (b) She said to me. "What my favourite cuisine is?" (c) She said to me, "What is my favourite cuisine?" (d) She said to me, "What is your favourite cuisine?" SSC CGL (Tier-I) 19/04/2022 Shift-II Ans. (d) : The given sentence is in indirect narration of present indefinite tense. Its direct narration will be made from the following rule1- 'Asked' change into 'said to'. 2- 'Comma and inverted comma' will be used in place of wh-word 'what'. 3- According to the narration rule, 'tense and pronoun' will be changed. 4- Change the latter clause from assertive to interrogative and in place of full stop (.) , question mark (?) will be used. Direct and Indirect Speech

94.

Select the option that expresses the given sentence in direct speech. He asked me when I had booked the flight tickets. (a) He said to me, "When did you book the flight tickets ?" (b) He said to me, "When do you book the flight tickets?" (c) He said to me, "When are you booking the flight tickets?" (d) He said to me, "When you had book the flight tickets ?" SSC CGL (Tier-I) 11/04/2022 Shift-I Ans. (a) : In the above sentence the use of conjunction 'when' makes it clear that the given sentence is in indirect form of interrogative sentence. Its direct speech will be made from the following rule1- 'Said to' will be used in place of 'asked'. 2- 'Comma and inverted comma' will be used in place of conjunction 'when'. 3- According to the narration rule, 'Tense and Pronoun' will be changed. 4- In place of full stop (.), question mark (?) will be used. 95. Select the option that expresses the given sentence in direct speech. The students asked the teacher where they could find the information for their project. (a) The students said to the teacher, "Where can you find the information for our project?" (b) The students asked the teacher, "Where can we find the information for our project?" (c) The teacher said to the student, "Where can you find the information for your project" (d) The teacher said to the student, "I will tell you where you can find the information for your project" SSC CHSL (Tier-I) –05/08/2021 (Shift-I) Ans. (b) : In the above sentence the use of conjunction "when" makes it clear that the given sentence is in indirect speech of interrogative sentence. Its direct speech will be made from the following rule1- 'Comma and inverted comma' will be used in place of conjunction "where". 2- According to the narration rule, 'Tense and Pronoun' will be changed. 96. Select the option that expresses the given sentence in indirect speech. The counter clerk asked the visitor, 'What is your name' ? (a) The counter clerk asked the visitor his name. (b) The counter clerk asks the visitor your name. (c) The counter clerk is asking the visitor your name. (d) The counter clerk has asked the visitor his name. SSC CGL (Tier-I) –23/08/2021 (Shift-I) Ans. (a) : In the above sentence the use of asked makes it clear that the sentence is in interrogative sentence of direct form. Its indirect speech will be made from the following rule-

360

YCT

1- "What" is used in place of 'comma and inverted comma' as a conjunction. 2- Change the latter clause from interrogative to assertive. 3- In place of question mark (?), full stop (.) will be used. 97. Select the option that expresses the given sentence in reported speech. 'Whom did you see at the shopping mall today ?' I asked my daughter. (a) I asked my daughter that whom did she saw at the shopping mall thay day. (b) I asked my daughter who she has seen at the shopping mall today. (c) I asked my daughter that whom she saw at the shopping mall on that day. (d) I asked my daughter whom she had seen at the shopping mall that day. SSC CGL (Tier-I) –16/08/2021 (Shift-I) Ans. (d) : Reported speech of the above sentence is in interrogative. Its indirect speech will be made from the following rule1- Question word 'whom' is used as a conjunction in place of 'comma and inverted comma'. 2- Interrogative sentence change into assertive sentence. 3- Full stop (.) will be used in place of mark of interrogation (?). 4- According to the narration rule, 'Tense and Pronoun' will be changed. 98. Select the correct indirect form of the given sentence. The interviewer asked Ramesh, "Do you have any idea about our products?" (a) The interviewer asked Ramesh that if he had any idea about the products. (b) The interviewer asked Ramesh if he have any idea about their products. (c) The interviewer asked Ramesh if he had any idea about their products. (d) The interviewer asked Ramesh if they had any idea about his products. SSC CHSL (Tier-I) –11/07/2019 (Shift-I) Ans : (c) Reported speech of the above sentence is in 'interrogative sentence'. Its indirect speech will be made from the following rule1- Conjunction "if/whether" is used in place of 'comma and inverted comma'. 2- "Said to" change into 'asked'. 3- According to the narration rule, 'Tense and Pronoun' will be changed. 4- Full stop (.) will be used in place of mark of interrogation (?). 99. Choose the option that is the indirect form of the sentence. A new student said to me, "Where is the Principal's office?" (a) A new student asked me where the Principal's office is. (b) A new student asked me where is the Principal's office. (c) A new student said to me where the Principal's office was. (d) A new student asked me where the Principal's office was. SSC CHSL (Tier-I) –10/07/2019 (Shift-III) Direct and Indirect Speech

Ans : (d) Reported speech of the above sentence is in "wh-type" interrogative sentence. Its indirect speech will be made from the following rule1- Question word "where" is used in place of comma and inverted comma as a conjunction. 2- Reporting verb "said to" change into "asked". 3- According to the narration rule, 'Tense and Pronoun' will be changed. 4- Reported speech sentence change into assertive sentence. 5- Full stop (.) will be used in place of mark of interrogation (?). 100. Select the correct indirect form of the given sentence. The interviewer asked Ashok, "Why did you leave that job last month?" (a) The interviewer asked Ashok that why he leave that job the last month. (b) The interviewer asked Ashok that why did he leave that job the previous month. (c) The interviewer asked Ashok why he had left that job the previous month. (d) The interviewer asked Ashok why you have left that job the previous month. SSC CHSL (Tier-I)–09/07/2019 (Shift-I) Ans : (c) Above sentence is in "wh-type" interrogative sentence. Its indirect speech will be made from the following rule1- Question word 'why' is used in place of comma and inverted comma as a conjunction. 2- Reporting verb 'said to' change into 'asked'. 3- According to narration rule, 'Tense and Pronoun' will be changed. 4- Reported speech interrogative sentence change into assertive sentence. 101. Select the correct indirect form of the given sentence. Uma asked me, "How's your brother?" (a) Uma asked me how was my brother. (b) Uma asked me how your brother was. (c) Uma asked me how my brother was. (d) Uma asked me how is my brother. SSC CHSL (Tier-I) –09/07/2019 (Shift-II) Ans : (c) Above sentence is in "wh-type" interrogative sentence. Its indirect speech will be made from the following rule1- Question word 'how' is used in place of 'comma and inverted comma'. 2- Reporting verb 'said to' change into 'asked'. 3- According to the narration rule, 'Tense and Pronoun' will be changed. 4- Reported speech sentence change into assertive sentence and full stop (.) will be used in place of question mark (,). 102. Select the most appropriate indirect form of the given sentence. He said to the grocer, "What is the price of tomatoes?" (a) He asked the grocer what was the price of tomatoes. (b) He asked the grocer what the price of tomatoes was.

361

YCT

(c) He asked the grocer what is the price of tomatoes. (d) He asked the grocer that what was the price of tomatoes. SSC CHSL (Tier-I) –05/07/2019 (Shift-II) Ans : (b) Above reported speech sentence is in "whtype" interrogative sentence. Its indirect speech will be made from the following rule1- Question word 'what' will be used in place of 'comma and inverted comma'. 2- Reporting verb 'said to' change into 'asked'. 3- According to the narration rule, 'Tense and Pronoun' will be changed. 4- Reported speech sentence change into 'assertive sentence'. 103. Select the correct indirect from of the given sentence. The stranger said to me, "Can you tell me the way to the River Bank Colony?" (a) The stranger asked me can you tell me the way to the River Bank Colony. (b) The stranger asked me if you could tell him the way to the River Bank Colony. (c) The stranger asked me if I could tell him the way to the River Bank Colony. (d) The stranger told me that I could tell him the way to the River Bank Colony. SSC CHSL (Tier-I) –04/07/2019 (Shift-I) Ans : (c) The above reported speech is interrogative (yes/no answer type) sentence. Its indirect speech will be made from the following rule1- If the reported speech begins with yes/no type interrogative, then in place of 'comma and inverted comma', 'if /whether' is used as a conjunction. 2- In reporting verb 'said to' is changed into asked. 3- According to the narration rule, 'tense and pronoun' will be changed. 104. Select the most appropriate indirect form of the given sentence. The lawyer said to the accused, "Where were you at the time of the burglary?" (a) The lawyer asked the burglar where he was at the time of burglary. (b) The lawyer asked the burglar that where was he at the time of burglary. (c) The lawyer asked the burglar if he was there at the time of burglary. (d) The lawyer asked the burglar where were you at the time of burglary. SSC CHSL (Tier-I) –04/07/2019 (Shift-III) Ans : (a) Above reported sentence is in "wh-type" interrogative sentence. Its indirect speech will be made from the following rule1- Question word 'where' is used a conjunction in place of 'comma and inverted comma'. 2- Reporting verb 'said to' change into 'asked'. 3- According to the narration rule, 'Tense and Pronoun' will be changed. 4- Reported speech sentence change into assertive sentence and full stop (.) will be used in place of question mark (?). Direct and Indirect Speech

105. Select the correct indirect form of the given sentence. Bhola said to his mother, "Why is this charcoal so black?" (a) Bhola asked his mother why was that charcoal so black. (b) Bhola told his mother why that charcoal was so black. (c) Bhola asked his mother why that charcoal was so black. (d) Bhola asked his mother why this charcoal is so black. SSC CHSL (Tier-I) –03/07/2019 (Shift-I) Ans : (c) Reported speech of the above sentence is in (wh-type) interrogative sentence. Its indirect speech will be made from the following rule1- Question word 'where' is used as a conjunction in place of 'comma and inverted comma'. 2- Reporting verb 'said to' change into 'asked'. 3- According to the narration rule, 'Tense and Pronoun' will be changed . 4- Reported speech sentence change into assertive sentence. 5- Full stop (.) will be used in place of question mark (?). 106. Select the correct indirect form of the given sentence. Bhola said to his friend, ''How can I make this charcoal white''? (a) Bhola asked his friend how he could make that charcoal white. (b) Bhola told his friend how he could make that charcoal white. (c) Bhola asked his friend how can I make this charcoal white. (d) Bhola asked his friend how could he make that charcoal white. SSC CHSL (Tier-I) –03/07/2019 (Shift-II) Ans. (a) : The above sentence is in "wh-type" interrogative sentence. Its indirect speech will be made from the following rule1- Question word "how" is used in place of 'comma and inverted comma'. 2- Reporting verb 'said to' change into 'asked'. 3- According to the narration rule, 'Tense and Pronoun' will be changed. 4- Reported speech sentence change into assertive sentence. 5- Full stop (.) will be used in place of question mark (?). 107. Select the correct indirect form of the given sentence. Father said to me, ''Are you going to keep me waiting all night''? (a) Father asked me if you were going to keep him waiting all night. (b) Father told me that you are going to keep me waiting all night. (c) Father asked me are you going to keep me waiting all night. (d) Father asked me if I was going to keep him waiting all night. SSC CHSL (Tier-I) –03/07/2019 (Shift-III)

362

YCT

Ans. (d) : Reported speech of the above direct sentence is in (yes/no answer type) interrogative sentence. Its indirect speech will be made from the following rule1- If the beginning of the reported speech is "yes/no type" interrogative, then in place of comma and inverted comma, if /whether is used as a conjunction. 2- Reporting verb 'said to' change into 'asked'. 3- According to the narration rule, 'Tense and Pronoun' will be changed. 108. Select the correct indirect of the given sentence. Mrs. Gupta said to me, ''Why are these boys standing in the sun?'' (a) Mrs. Gupta told me why those boys are standing in the sun. (b) Mrs. Gupta asked me why those boys were standing in the sun. (c) Mrs. Gupta said to me that why are these boys standing in the sun. (d) Mrs. Gupta asked me why were these boys standing in the sun. SSC CHSL (Tier-I) –02/07/2019 (Shift-I) Ans. (b) : Reported speech of the above sentence is in "wh-type" interrogative sentence. Its indirect speech will be made from the following rule1- Question word 'why' is used as a conjunction in place of 'comma and inverted comma'. 2- Reporting verb 'said to' change into 'asked'. 3- According to the narration rule, 'tense and pronoun' will be changed. 4- Reported speech sentence change into assertive sentence full stop (.) will be used in place of question mark (?). 109. Select the correct indirect form of the given sentence. The shopkeeper asked me, "Where is your car?" (a) The shopkeeper asked me where my car was. (b) The shopkeeper asked me where my car is. (c) The shopkeeper asked me where was my car. (d) The shopkeeper asked me where is your car. SSC CHSL (Tier-I)–02/07/2019 (Shift-II) Ans : (a) Reported speech of the above sentence is in "(wh-type)" interrogative sentence. Its indirect speech will be made from the following rule1- Question word 'what' is used as a conjunction in the place of 'comma and inverted comma'. 2- Second clause of the sentence change from interrogation into assertive sentence. 3- Full stop (.) will be used in place of question mark (?). 110. Select the correct indirect form of the given sentence. I asked the shopkeeper, "What is the price of this bike?" (a) I asked the shopkeeper what the price of that bike was. (b) I asked the shopkeeper that what the price of this bike was. (c) I asked the shopkeeper what is the price of this bike. (d) I asked the shopkeeper what was the price of that bike. SSC CHSL (Tier-I) –02/07/2019 (Shift-III) Direct and Indirect Speech

Ans : (a) Reported speech of the above sentence is in "(wh-type)" interrogative sentence. Its indirect speech will be made from the following rule1- Question word 'where' is used as a conjunction in place of .comma and inverted comma'. 2- Second clause of the sentence change from interrogative to assertive sentence. 3- Full stop (.) will be used in place of question mark (?). 111. Select the correct indirect form of the given sentence. Mr. Puri said to me. "Why are you taking down this fence?" (a) Mr. Puri asked me why was I taking down this fence. (b) Mr. Puri said to me why you were taking down that fence. (c) Mr. Puri asked me why I was taking down that fence. (d) Mr. Puri said to me that why are you taking down this fence. SSC CHSL (Tier-I) –01/07/2019 (Shift-III) Ans : (c) Reported speech of the above sentence is in "(wh-type)" interrogative sentence. Its indirect speech will be made from the following rule. 1- Question word 'why' is used as a conjunction in place of 'comma and inverted comma.' 2- Reporting verb 'said to' change into 'asked'. 3- According to the narration rule, 'Tense and Pronoun' will be changed. 112. Select the correct indirect form of given sentence. The foreigner said to the farmer, "Can you tell me the way to the nearest hotel?" (a) The foreigner suggested the farmer if he could tell him the way to the nearest hotel. (b) The foreigner exclaimed with the farmer if he could tell him the way to the nearest hotel. (c) The foreigner told the farmer if he could tell him the way to the nearest hotel. (d) The foreigner enquired of the farmer if he could tell him the way to the nearest hotel. SSC CHSL (Tier-I) 21/10/2020 (Shift-II) Ans. (d) : Reported speech of the above direct sentence is in interrogative sentence. Its indirect speech will be made from the following rule1- Reporting verb 'said to' change into enquired of. 2- 'If/whether' is used as a conjunction in the place of 'comma and inverted comma'. 3- According to the narration rule, 'tense and pronoun' will be changed. 4- Full stop (.) is used in place of question mark (?). 113. Select the correct indirect form of the given sentence. The lion asked the lioness, "Where will you hide yourself, until I return?" (a) The lion asked the lioness where she would hide herself, until he returned. (b) The lion asked the lioness where she will hide herself, until he returns.

363

YCT

(c) The lion asked the lioness where would she hide herself, until he returned. (d) The lion asked the lioness whether she would hide herself, until he returned. SSC CHSL (Tier-I) 19/03/2020 (Shift-III) Ans. (a) : Reported speech of the above sentence is in "(wh-type)" interrogative sentence. Its indirect speech will be made from the following rule1- Question word "where" is used as a conjunction in place of 'comma and inverted comma'. 2- Reporting verb 'said to' change into 'asked'. 3- According to the narration rule, 'Tense and Pronoun' will be changed. 4- Reported speech sentence change into assertive sentence. 114. Select the correct indirect form of the given sentence. Ravi asked, "Ritu, how many sets of class 12 practice papers have you finished?" (a) Ravi asked Ritu how many sets of class 12 practice papers she had finished. (b) Ravi asked Ritu how many sets of class 12 practice papers had she been finishing. (c) Ravi asked Ritu how many sets of class 12 practice papers was she finishing. (d) Ravi asked Ritu how many sets of class 12 practice papers was she finishing. SSC CHSL (Tier-I) 13/10/2020 (Shift-I) Ans. (a) : The reported speech of the above sentence is an interrogative sentence of Present perfect tense, so its indirect speech will be made from the following rule1- Question word 'How' is used as a conjunction in place of 'comma and inverted comma'. 2- Reporting verb object (Ritu) is changed into 'she', according to the reported speech subject (you). 3- According to the narration rule, 'Tense and Pronoun' will be changed. 4- Reported speech will change from interrogative sentence to assertive sentence. 115. Select the correct indirect form of the given sentence. The storekeeper asked the security, "Did you see the torch I kept here?" (a) The storekeeper asked the security whether he had seen the torch he had kept there. (b) The storekeeper asked the security whether he saw the torch he kept there. (c) The storekeeper asked the security if he saw the torch he kept there. (d) The storekeeper asked the security if had he seen the torch he kept there. SSC CHSL (Tier-I) 18/03/2020 (Shift-III) Ans. (a) : Reported speech of the above direct sentence is in (yes/no type) interrogative sentence. Its indirect speech will be made from the following rule1- No change in reporting verb 'asked'. 2- If/ whether is used in place of 'comma and inverted comma'. 3- According to the narration rule, 'Tense and Pronoun' will be changed. 4- Full stop (.) will be used in place of question mark (?). Direct and Indirect Speech

116. Select the option that is the direct form of the sentence. Kirti asked me if I had watched the movie on television the previous night. (a) Kirti asked me, "Had you watched the movie on television last night?" (b) Kirti asked me, "Did I watched the movie on television last night?" (c) Kirti asked me, "Did you watch the movie on television last night?" (d) Kirti asked me, "You had watched the movie on television last night?" SSC CHSL (Tier-I) 18/03/2020 (Shift-II) Ans. (c) : In the above sentence, the use of conjunction 'if' makes it clear that the given sentence is in indirect speech of interrogative sentence. Its direct speech will be made from the following rule1- 'Comma and inverted comma' will be used in place of conjunction 'if'. 2- According to the narration rule, 'Tense and Pronoun' will be changed. 3- 'The last night' is used in place of "the previous night." 4- Reported speech will changed from assertive to interrogative sentence. 117. Select the correct indirect form of the given sentence. Shanti asked me, "Why did you keep this smart phone in the bin?" (a) Shanti asked me why I was keeping that smart phone in the bin. (b) Shanti asked me why I had been keeping that smart phone in the bin. (c) Shanti asked me why I kept that smart phone in the bin. (d) Shanti asked me why I had kept that smart phone in the bin. SSC CHSL (Tier-I) 18/03/2020 (Shift-I) Ans. (d) : The above reported speech is in "(wh-type)" interrogative sentence. Its indirect speech will be made from the following rule1- Question word "why" is used as conjunction in place of 'comma and inverted comma'. 2- Reporting verb 'said to' change into 'asked'. 3- According to the narration rule, 'Tense and Pronoun' will be changed. 4- Reported speech will convert into interrogative to assertive sentence. 118. Select the correct indirect form of the given sentence. Sitab asked his mother, "Has it been snowing all night?" (a) Sitab asked his mother if it is snowing all night. (b) Sitab asked his mother if it had been snowing all night. (c) Sitab asked his mother if it has snowed all night. (d) Sitab asked his mother if it was snowing all night. SSC CHSL (Tier-I) 12/10/2020 (Shift-II)

364

YCT

Ans. (b) : Reported speech of the above direct sentence is in "(yes/no type)" interrogative sentence. Its indirect speech will be made from the following rule1- No change is reporting verb 'asked'. 2- If/whether is used in place of 'comma' and inverted comma'. 3- According to the narration rule, 'Tense and Pronoun' will be changed. 4- Full stop (.) will be used in place of question mark (?). 119. Select the option that is the direct form of the given sentence. The child asked his mother if he could swim in the pool there. (a) The child said to his mother, "Will swim in the pool here"? (b) The child asked to his mother, "May I swim in the pool here"? (c) The child asked to his mother, "Can he swim in the pool there"? (d) The child asked his mother, "Could he swim in the pool there"? SSC CHSL (Tier-I) 15/10/2020 (Shift-I) Ans. (b) : In the above sentence the use of conjunction 'if' makes it clear that reported speech of the above sentence is in indirect speech. Its direct speech will be made from the following rule1- 'Comma and inverted comma' used in place of 'if'. 2- According to the narration rule, 'Tense and Pronoun' will be changed. 3- The sentence convert into interrogative to assertive sentence. 120. Select the correct indirect form of the given sentence. Nakul asked her, "How do you know that Rahul has told it to him?" (a) Nakul asked her how she know that Rahul had told that to him. (b) Nakul asked her how she knew that Rahul had told that to him. (c) Nakul asked her that how she knows that Rahul had told to him. (d) Nakul asked her that how she has known that Rahul has told that to him. SSC CHSL (Tier-I) 19/03/2020 (Shift-II) Ans. (b) : Reported speech of the above direct sentence is in "(wh-type)" interrogative sentence. Its indirect speech will be made from the following rule1- No change is reporting verb 'asked'. 2- Question word 'how' is used as a conjunction is place of 'comma and inverted comma'. 3- Reported speech of interrogative sentence is converted into assertive sentence in indirect speech. 4- Full stop (.) will be used in place of question mark (?). 121. Select the correct indirect form of the given sentence. "Do you attend the film festival at Goa every year?" he asked. Direct and Indirect Speech

(a) He asks me if I attend the film festival at Goa every year. (b) He asked me if I attended the film festival at Goa every year. (c) He asks me if I was attending the film festival at Goa every year. (d) He is asking me if I attend the film festival at Goa every year. SSC CHSL (Tier-I) 12/10/2020 (Shift-I) Ans. (b) : Reported speech of the above direct sentence is in "(yes/no answer type)" interrogative sentence. Its indirect speech will be made from the following rule1- No change in reporting verb "asked". 2- 'If/whether' is used in place of 'comma and inverted comma'. 3- According to the narration rule, 'Tense and Pronoun' will be changed. 4- Full stop (.) is used in place of question mark (?).

C.

Imperative Sentence

122. Select the correct direct form of the given sentence. Prachi proposed that they should meet their friends at a restaurant. (a) Prachi said, "Let us meet their friends at a restaurant." (b) Prachi said, "We could meet their friends at a restaurant". (c) Prachi said, "Let them meet their friends at a restaurant". (d) Prachi said, "They should meet their friends at a restaurant". SSC Stenographer (Grade C & D) 12.11.2021 Shift-I Ans. (a) : In the above sentence, the use of 'proposed' makes it clear that the given sentence is in indirect speech of imperative sentence. Its direct speech will be made from the following rule1- 'Proposed' change into 'said'. 2- 'Comma and inverted comma' will be used in place of conjunction 'that'. 3- 'They should' change into 'let us'. 4- According to the narration rule, 'Tense and Pronoun' will be changed. 123. Select the option that expresses the given sentence in indirect speech. She said to me, "Don't worry about me." (a) She told me don't worry about her. (b) She told me to not be worried about her. (c) She told me to not to worry about me. (d) She told me not to worry about her. SSC CGL (Tier-II) 08.08.2022 Shift-II Ans. (d) : Reported speech of the above sentence is in 'Imperative sentence'. Its indirect speech will be made from the following rule1- Conjunction 'to' is used in place of 'comma and inverted comma'. '(not)' is used before conjunction 'to' in negative imperative sentence. 2- Reporting verb 'said to' change into 'told' in case of reporting object (me) is given. 3- According to the narration rule, 'Tense and Pronoun' will be changed.

365

YCT

124. Select the correct indirect narration of the given sentence. Father said to Swami, "Take whatever you like from this drawer." (a) Father said Swami to take whatever you like from this drawer. (b) Father told Swami that he can take whatever he likes from the drawer. (c) Father told Swami to take whatever he liked from that drawer. (d) Father said to Swami to have taken whatever he liked from that drawer. SSC Stenographer (Grade C & D) 15.11.2021 Shift-I Ans. (c) : Above given direct speech is in 'imperative sentence'. Its indirect speech will be made from the following rule1- Reporting verb 'said to' change into 'told'. 2- Conjunction 'to' is used in place of 'comma and inverted comma'. 3- According to the narration rule, 'tense and pronoun' will be changed. 125. Select the option that expresses the given sentence in reported speech. The teacher said, “Asif, go and wash your hands.” (a) The teacher told to Asif to go and wash his hands. (b) The teacher told Asif go and wash your hands. (c) The teacher told Asif go and wash his hands. (d) The teacher told Asif to go and wash his hands. SSC CGL (Tier-I) 12/04/2022 Shift-III Ans. (d) : Reported speech of the above sentence is in 'imperative sentence'. Its indirect will be made from the following rule1- Conjunction 'to' is used in place of 'comma and inverted comma'. 2- Reporting verb 'said to' change into 'told'. 3- According to the narration rule, 'Tense and Pronoun' will be changed. 126. Select the correct indirect form of the given sentence. Mother said to Megha, "Leave for your office now." (a) Mother said to Megha leave for her office then. (b) Mother told to Megha to leave for her office now. (c) Mother told Megha to leave for her office then. (d) Mother told Megha to leave for your office now. SSC CHSL (Tier-I) –13/04/2021 (Shift-I) Ans. (c) : Reported speech of the above direct narration makes it clear that the sentence talks about order. Therefore, this is 'imperative sentence'. Its indirect speech will be made from the following rule1- Reporting verb 'said to' convert into 'told/order'. 2- 'To' will be used in place of 'comma and inverted comma'. 3- According to the narration rule, 'Tense and Pronoun' will be changed. Direct and Indirect Speech

127. Select the option that expresses the given sentence in reported speech. He said to his sister, ''Please help me with my homework'. (a) He told his sister that she should help him with his homework. (b) His sister was asked by her brother that she should help him with his homework. (c) His sister told him that he should help her with her homework. (d) He requested his sister to help him with his homework. SSC CHSL (Tier-I) –19/04/2021 (Shift-I) Ans. (d) : Reported speech of the above sentence, the use of 'please' makes it clear that the given sentence is in imperative sentence. Its indirect speech will be made from the following rule1- Reporting verb 'said to' convert into 'requested'. 2- 'to' is used in place of 'comma and inverted comma'. 3- Remove 'please/kindly' etc. 4- According to the narration rule, 'Tense and Pronoun' will be changed. 128. Select the correct direct form of the given sentence. The policeman warned us not to block the traffic. (a) The policeman said to us, "We should not block the traffic" (b) The policeman said to us, "Do not block the traffic." (c) The policeman said to us, "Let us not block the traffic." (d) The policeman said to us, "You did not block the traffic". SSC CGL (Tier-II) 16/11/2020 (3 pm - 5 pm) Ans. (b) : In the above indirect speech use of 'to' marks it clear that the direct speech is in imperative sentence. Its direct speech will be made from the following rule1- Comma and inverted comma will be use in the place of conjunction 'to'. 2- 'Said to' will be used in the place of reported verb warned." 3- Not change into 'do not' because imperative sentence start with v1 (verb1). 129. Select the correct direct form of the given sentence : The young man prayed to God to help him clear that interview : (a) The young man said, "O God! Help him to clear that interview". (b) The young man said, "O God! Help me clear this interview". (c) The young man said to God, "You have to help me clear that interview". (d) The young man said to God, "Will you help me clear that interview"? SSC CGL (Tier-II) 16/11/2020 (3 pm - 5 pm) Ans. (b) : In the above sentence the use of conjunction to makes it clear that the sentence is in imperative sentence of indrect form. Its direct speech will be made from the following rule-

366

YCT

1- Comma and inverted comma will be used in the place of conjunction 'to'. 2- Reporting verb 'prayed' change into 'said' and exclamation word 'O God!' will be used. 3- According to narration rule, Tense and Pronoun will be changed. 130. Select the correct indirect form of the given sentence. Hemant said to the hotel receptionist, "Please tell Bharat I will come to see him tomorrow at 10 am." (a) Hemant said to the hotel receptionist please to tell Bharat he will come to see me tomorrow at 10 am. (b) Hemant requested to the hotel receptionist please inform Bharat I will come to see him tomorrow at 10 am. (c) Hemant said to the hotel receptionist to please tell Bharat that I will come to see him tomorrow at 10 am. (d) Hemant requested the hotel receptionist to inform Bharat that he would come to see him the next day at 10 am. SSC CHSL (Tier-I) –10/07/2019 (Shift-I) Ans : (d) In the above sentence the use of please + v1 makes it clear that the reported speech is in imperative sentence. Its indirect speech will be made from the following rule1- Conjunction 'to' is used in place of 'comma and inverted comma'. 2- Reporting verb 'said to' convert into 'requested' according to sense of reported speech. 3- According to the narration rule, 'tense and pronoun' will be changed. 4- 'Tomorrow' change into 'the next day.' 131. Select the correct indirect form of the given sentence. "Don't park here," the policeman said to them. (a) The policeman ordered them not to park here. (b) The policeman said to them not to park here. (c) Don't park here he said to the policeman. (d) The policeman ordered them not to park there. SSC CHSL (Tier-I) –10/07/2019 (Shift-II) Ans : (d) Reported speech of the above sentence beginning with 'don't. Hence it is imperative sentence of negative form. Its indirect speech will be made from the following rule1- Reporting verb 'said to' convert into 'ordered'. 2- Conjunction 'to' will be used in the place of comma (,) and inverted comma (" "). 3- In negative (imperative) sentence 'not' is used before the conjunction 'to'. 4- 'Tense and Pronoun' will be changed according to the narration rule. 132. Select the most appropriate indirect form of the given sentence. Seema said to her daughter, "Don't go near the well." (a) Seema requested to her daughter to go near the well. (b) Seema told her daughter that don't go near the well. Direct and Indirect Speech

(c) Seema said her daughter you shouldn't go near the well. (d) Seema warned her daughter not to go near the well. SSC CHSL (Tier-I) –05/07/2019 (Shift-I) Ans : (d) Reported speech of the above sentence beginning with 'don't'. Hence it's a negative imperative sentence. Its indirect speech will be made from the following rule1- Conjunction 'to' is used in place of 'comma and inverted comma' and 'not' is used before 'to' in case of its sentence is negative. 2- Reporting verb said convert into 'warned' according to sense of the sentence. 133. Select the correct indirect form of the given sentence. Roopa said to me, "Please accompany me to the market." (a) Roopa said please accompany me to the market. (b) Roopa said to me that you must accompany me to the market. (c) Roopa requested me to accompany her to the market. (d) Roopa is requesting me to accompany her to the market. SSC CHSL (Tier-I) 16/10/2020 (Shift-II) Ans. (c) : Reported speech of the above sentence beginning with 'please + v1 hence it's make clear that the sentence is imperative. Its indirect speech will be made from the following rule1- Conjunction 'to' is used in place of 'comma and inverted comma'. 2- Reporting verb 'said to' convert into 'requested' according to sense of the sentence. 3- According to the narration rule, 'Tense and Pronoun' will be changed. 134. Select the correct indirect form of the given sentence. Mother said to Avika, "Finish these sums before you go to the park." (a) Mother told Avika to finish those sums before she went to the park. (b) Mother told Avika to finish these sums before you go to the park. (c) Mother told Avika to finish those sums before she goes to the park. (d) Mother told Avika to finish those sums before she was going to the park. SSC CHSL (Tier-I) 19/10/2020 (Shift-III) Ans. (a) : Reported speech of the above sentence beginning with finish (V1). Hence it's imperative sentence. Its indirect speech will be made from the following rule1- Conjunction 'to' will be used in place of 'comma and inverted comma'. 2- Reporting verb 'said to' convert into 'told' according to sense of the sentence. 3- According to the narration rule, 'Tense and Pronoun' will be changed.

367

YCT

135. Select the correct indirect form of the given 138. Choose the option that is the indirect form of sentence. the sentence. Mother said to Kavya, "Switch off the light The judge said to Jia, “Stand in the witness before you go to sleep." box.” (a) Mother told Kavya to switch off the light (a) The judge told Jia you are standing in the before she went to sleep. witness box. (b) Mother told Kavya to switch off the light (b) The judge told Jia to stand in the witness box. before she goes to sleep. (c) The judge told to Jia stand in the witness box. (c) Mother told Kavya to switch off the light before you go to sleep. (d) The judge told Jia you will stand in the (d) Mother told Kavya to switch off the light witness box. before she is going to sleep. SSC CPO-SI (Tier-II) 27/09/2019 (3 pm - 5 pm) SSC CHSL (Tier-I) 19/10/2020 (Shift-II) Ans. (b) : Above reported speech beginning with v1 Ans. (a) : Reported speech of the above sentence (verb) makes it clear that the sentence is in imperative beginning with "switch off (V1)". Hence it's imperative sentence. Its indirect speech will be made from the sentence. Its indirect speech will be made from the following rule. following rule1- Conjunction 'to' is used in place of 'comma and 1- Reporting verb 'said' change into 'told' in the case of reporting object (Jia) is given . inverted comma'. 2- Reporting verb 'said to' convert into 'told' according 2- Conjunction 'to' is used in the place of comma and inverted comma. to sense of the sentence. 136. Select the option that expresses the given 3- According to narration rule, Tense and Pronoun will be changed. sentence in direct speech. Bharati told Rakesh to call her when he 139. Choose the option that is the direct form of the reached there. sentence. (a) "Call me when you reach here", Bharati told I told them to be quiet. Rakesh. (a) I said to them “Be you quiet!” (b) "Call her when you have reached here", (b) I told to them, “You must be quiet.” Bharati told Rakesh. (c) I told them, “Be quiet!” (c) "Call me when you reached there", Bharati (d) I said them, “You be quiet.” says to Rakesh. SSC CPO-SI (Tier-II) 27/09/2019 (3 pm - 5 pm) (d) "Call him when you are reaching there", Bharati told Rakesh. Ans. (c) : In above indirect speech the use conjunction SSC CHSL (Tier-I) 14/10/2020 (Shift-I) 'to' makes it clear that the given sentence is in Ans. (a) : Conjunction 'to' is used in above indirect Imprative sentence. It will be made from the following sentence. Hence its reported speech will be imperative. ruleIts direct speech will be made from the following rule1- Comma (,) and inverted comma (" ") will be used in 1- Comma and inverted comma will be used in the place of conjunction 'to'. place of conjunction 'to'. 2- Other change will be according to narration rules. 2- According to the narration rule, 'Tense and Pronoun' 140. Choose the option that is the indirect form of will be changed. the sentence. 3- Remaining part will be same. Please bring me a cup of coffee,” Shakila said 137. Choose the option that is the indirect form of to the waiter. the sentence. (a) Shakila told the waiter to bring her a cup of "Get out of this room", the officer shouted at coffee. the cadet. (b) Shakila said to the waiter he should bring her (a) The officer shouted at the cadet and asked a cup of coffee. him to get out of that room. (b) The officer shouted at the cadet and asked (c) Shakila told to the waiter that he should him to get out of this room. brought her a cup of coffee. (c) The officer shouts at the cadet and asks him (d) Shakila said to the waiter you bring me a cup to go out of that room. of coffee. (d) The officer shouting at the cadet and asking SSC CPO-SI (Tier-II) 27/09/2019 (3 pm - 5 pm) him to go out of the room. SSC CPO-SI (Tier-II) 27/09/2019 (3 pm - 5 pm) Ans. (a) : In the above indirect speech, the use of 'please' makes it clear that the given sentence is in Ans. (a) : The above reported speech use of 'get out ' imperative sentence. Its indirect speech will be made (v1) beginning of the sentence makes it clear that the from the following rule. sentence is in imperative sentence. Its indirect speech 1- 'Said to' change into 'told'. will be made from the following rule2- Conjunction 'to' is used in the place of of comma 1- By removing comma and inverted comma before the main verb conjunction 'to' is used. and inverted comma. 2- Change the pronoun 'this' of the reported speech into 3- According to narration rule, Tense and Pronoun will 'that'. be changed. Direct and Indirect Speech

368

YCT

141.

(a) The woman said, "How the kids are making too much noise!" (b) The woman said, "The kids were making too much noise." (c) The woman said, "What noise are the kids making?" (d) The woman said, "How much noise are the kids making!" SSC Stenographer (Grade C & D) 15.11.2021 Shift-II Ans. (d) : The above sentence is an exclamatory so this to convert the sentence into direct speech, 'too much noise' will be changed to 'how much noise' and 'were' change into 'are' and 'exclaimed' convert into 'said'. 145. Select the correct indirect narration of the given sentence. The team leader said to us, "Congratulations! You have prepared a very good presentation." (a) The team leader told us congratulations and we had prepared a very good presentation. (b) We told our team leader that we had prepared a very good presentation and he said congratulations. (c) The team leader said congratulations and told us that we have prepared a very good presentation. (d) The team leader congratulated us and said that we had prepared a very good presentation. SSC Stenographer (Grade C & D) 15.11.2021 Shift-I Ans. (d) : Reported speech of the above direct narration is an exclamatory sentence. Its indirect speech will be made from the following rule1- Reporting verb 'said to' convert into 'congratulated' according to sense of the sentence. 2- Conjunction 'that' is used in place of 'comma and inverted comma'. 3- According to the narration rule, 'Tense and Pronoun' will be changed. 146. Select the option that expresses the given sentence in indirect speech. 'How beautiful she is!' said Betty. (a) Betty exclaimed with joy that she was very beautiful. (b) Betty exclaimed with joy she is very beautiful. (c) Betty exclaimed with joy how beautiful she was. (d) Betty exclaimed with joy how beautiful she is. SSC CGL (Tier-I) –17/08/2021 (Shift-I) Ans. (a) : Reported speech of the above direct narration is an exclamatory sentence. Its indirect speech will be made from the following rule1- Conjunction 'that' is used in place of 'comma and D. Exclamatory Sentence inverted comma'. 144. Select the option that expresses the given 2- Reporting verb change into exclaimed with joy/ sentence in direct speech. surprised etc. The women exclaimed that the kids were 3- Full stop (.) is used in the place of mark of exclamation (!). making too much noise. The teacher said to me, "Stand here next to the podium." (a) The teacher tells to me to stand there next to the podium. (b) The teacher told to me to have stood there next to the podium. (c) The teacher told me to stand there next to the podium. (d) The teacher had told to me to have stood there next to the podium. SSC CGL (Phase-II) 17/02/2018 Ans. (c) Reported speech of the above sentence is in 'imperative sentence'. Its indirect speech will be made from the following rule1- 'Said to' change into 'told'. 2- 'To' conjunction is used in place of 'comma and inverted comma' in imperative sentence. 3- Place indicator word 'here' change into 'there'. 142. “Hurry up, get in the bus” the conductor said to us. (a) The conductor told us to hurry up and get in the bus. (b) The conductor tells us to hurry up and get in the bus. (c) The conductor told us to hurry up and got in the bus. (d) The conductor tells us to hurry up and got in the bus. SSC CGL (Phase-II) 17/02/2018 Ans. (a) Reported speech of the above sentence is in 'imperative sentence'. Its indirect speech will be made from the following rule1- 'Said to' change into 'told'. 2- 'to' is used in the place of 'comma and inverted comma' as a conjunction. 143. Choose the option that is the direct form of the sentence. He asked me to wait there until I got my turn. (a) He said to me, “Wait here until you get your turn.” (b) He told to me “Wait here until you get your turn.” (c) He said to me, “You are wait here until you get your turn.” (d) He asked me to “wait there until you get my turn.” SSC CPO-SI (Tier-II) 27/09/2019 (3 pm - 5 pm) Ans. (a) : At the above Indirect speech the use of conjunction 'to' makes it clear that the sentence is in imperative. Its direct speech will be made from the following rule1- Comma (,) and inverted comma (" ") will be used in the place of conjunction 'to' 2- 'Asked' change into 'Said to ' 3- According to narration , Tense and Pronoun will be changed.

Direct and Indirect Speech

369

YCT

147. Select the correct direct form of the given sentence. Neha exclaimed that it was a very pleasant surprise. (a) Neha said, "Is it a very pleasant surprise?" (b) Neha said, "It is a very pleasant surprise?" (c) Neha said, "What a pleasant surprise!" (d) Neha said, "How a pleasant surprise!" SSC CGL (Tier-II) 16/11/2020 (3 pm - 5 pm) Ans. (c) : In the above sentence the use of exclaimed makes it clear that the sentence is in exclamatory sentence. Its direct speech will be made from the following rule1- Comma (,) and inverted comma (" ") will be used in the place of conjunction 'that'. 2- For exclamatory sentence what a " is also added before the clause containing that. 3- Exclamation mark (!) will be used in the place of full stop (.) 148. Select the correct indirect form of the given sentence. My neighbour said to me, "Hello! How are you?" (a) My neighbour greeted me and asked how was I. (b) My neighbour said hello and asked how are you (c) My neighbour said hello and asked how were I (d) My neighbour greeted me and asked how I was SSC CGL (Tier-II) 16/11/2020 (3 pm - 5 pm) Ans. (d) : In the above sentence the use of 'Hello!' makes it clear that the sentence is in exclamatory sentence. Its indirect speech will be made from the following rule1- The exclamation word 'Hello' will change the reporting verb 'said to' into 'greeted.' 2- ' How' is used as a conjunction in the sentence. 3- According to the narration rule, 'Tense and Pronoun' will be changed. 149. Select the correct indirect form of the given sentence. He exclaimed, "What a fine piece of architecture it is !" (a) He exclaimed what a fine piece of architecture is it. (b) He exclaimed was it a fine piece of architecture. (c) He exclaimed that what a fine piece of architecture it was. (d) He exclaimed that it was a fine piece of architecture. SSC CGL (Tier-II) 16/11/2020 (3 pm - 5 pm) Ans. (d) : In the above sentence the use of "exclaimation" marks, makes it is clear that the sentence is in exclamatory sentence. Its indirect speech will be made from the following rule1- Conjunction 'that' will be used in the place of comma and inverted comma. 2- For exclamatory sentence "what a" is also added before the clause containing it was. 3- Full stop (.) will be used in the place of exclamation mark. Direct and Indirect Speech

150. Select the option that is the direct form of the given sentence. Nidhi exclaimed that it was a beautiful painting. (a) Nidhi said to the artist in surprise, "How beautiful you made that painting!" (b) Nidhi told him in wouder. "How beautiful!" (c) Nidhi said to the artist, "That looks like a beautiful painting!" (d) Nidhi exclaimed, "What a beautiful painting!" SSC CHSL (Tier-I) 13/10/2020 (Shift-III) Ans. (d) In the above sentence the use of exclaimed makes it clear that reported speech is in exclamatory sentence. Its direct speech will be made from the following rule1- 'Comma and inverted comma' will be used in the place of conjunction 'that'. 2- For exclamatory sentence "what a --- " is added before that clause. 3- Exclamation mark (!) is used in place of full stop (.). 151. Select the correct indirect form of the given sentence. Meena said "What a beautiful flower!" (a) Meena exclaimed how beautiful the flower was (b) Meena said that she had a beautifyl flower (c) Meena exclaimed what a beautiful flower it was (d) Meena exclaimed that it was a very beautiful flower. SSC Stenographer Grade C&D –05/02/2019 (3:5pm)

Ans. (d) : Reported speech of the above sentence is in Exclamatory sentence. Its indirect speech will be made from the following rule1- Conjunction 'that' will be used in the place of comma and inverted comma. 2- Remove 'what-----a " in reported speech. 3- Full stop (.) will be used in the place of exclamatory mark (!). 152. Choose the option that is the direct form of the sentence. He exclaimed sadly that it was a pity that so many lives had been lost in the floods. (a) He said sadly, “It is a pity that so many lives had been lost in the floods.” (b) He said sadly, “What a pity that so many lives are being lost in the floods!” (c) He said sadly, “What a pity that so many lives have been lost in the floods.” (d) He said sadly, “It was a pity that so many lives were loss in the floods.” SSC CPO-SI (Tier-II) 27/09/2019 (3 pm - 5 pm) Ans. (c) : Above Indirect speech is use of 'exclaimed' makes it clear that the sentence is exclamatory sentence. Its direct speech will be made from the following rule1. Comma (,) and inverted comma (" ") will be used in place of conjunction "that". 2- For exclamatory sentence "what" is also added before the clause countering 'that'. 3- Exclamation mark (!) will be used in the place of full stop (.).

370

YCT

06. SYNONYMS TCS hewšve& hej DeeOeeefjle (Based On TCS Pattern) Chapterwise

Exam

Question No.

1

CGL (Tier-1) CGL (Tier-2) CHSL (Tier-1) CHSL (Tier-2) Selection Post VII, VIII, XI SSC MTS SSC GD SSC CPO SI CGL (Tier-1) CGL (Tier-2) CHSL (Tier-1) CHSL (Tier-2) Selection Post VII, VIII, XI SSC MTS SSC GD SSC CPO SI CGL (Tier-1) CGL (Tier-2) CHSL (Tier-1) CHSL (Tier-2) Selection Post VII, VIII, XI SSC MTS SSC GD SSC CPO SI CGL (Tier-1) CGL (Tier-2) CHSL (Tier-1) CHSL (Tier-2) Selection Post VII, VIII, XI SSC MTS SSC GD SSC CPO SI CGL (Tier-1) CGL (Tier-2) CHSL (Tier-1) CHSL (Tier-2) Selection Post VII, VIII, XI SSC MTS SSC GD SSC CPO SI

15 5 20 10 9 11 1 2 3 2 1 1 2 5 1 4 10 8 12 6 4 10 2 5 10 3 17 5 5 5 2 5 7 1 10 5 5 9 1 7

CGL (Tier-1) CGL (Tier-2) CHSL (Tier-1) CHSL (Tier-2) Selection Post VII, VIII, XI SSC MTS SSC GD SSC CPO SI

8 2 7 3 4 5 1 5

A

2 B

3 C

4 D

5 E

6

F

Synonyms

371

Years

(2017–2023)

(2017–2023)

(2017–2023)

(2017–2023)

(2017–2023)

(2017–2023)

YCT

7

G

8

H

9

I

10

J

11

K

12 L

13 M

Synonyms

CGL (Tier-1) CGL (Tier-2) CHSL (Tier-1) CHSL (Tier-2) Selection Post VII, VIII, XI SSC MTS SSC GD SSC CPO SI CGL (Tier-1) CGL (Tier-2) CHSL (Tier-1) CHSL (Tier-2) Selection Post VII, VIII, XI SSC MTS SSC GD SSC CPO SI CGL (Tier-1) CGL (Tier-2) CHSL (Tier-1) CHSL (Tier-2) Selection Post VII, VIII, XI SSC MTS SSC GD SSC CPO SI CGL (Tier-1) CGL (Tier-2) CHSL (Tier-1) CHSL (Tier-2) Selection Post VII, VIII, XI SSC MTS SSC GD SSC CPO SI CGL (Tier-1) CGL (Tier-2) CHSL (Tier-1) CHSL (Tier-2) Selection Post VII, VIII, XI SSC MTS SSC GD SSC CPO SI CGL (Tier-1) CGL (Tier-2) CHSL (Tier-1) CHSL (Tier-2) Selection Post VII, VIII, XI SSC MTS SSC GD SSC CPO SI CGL (Tier-1) CGL (Tier-2) CHSL (Tier-1) CHSL (Tier-2) Selection Post VII, VIII, XI SSC MTS SSC GD SSC CPO SI 372

2 1 3 1 1 1 2 2 1 2 1 1 4 1 4 9 1 10 2 8 5 3 5 1 1 1 1 1 1 1 1 2 1 2 1 1 5 1 3 7 3 5 2 3 5 1 5

(2017–2023)

(2017–2023)

(2017–2023)

(2017–2023)

(2017–2023)

(2017–2023)

(2017–2023)

YCT

14 N

15 O

16 P

17 Q

18

R

19

S

20

T

Synonyms

CGL (Tier-1) CGL (Tier-2) CHSL (Tier-1) CHSL (Tier-2) Selection Post VII, VIII, XI SSC MTS SSC GD SSC CPO SI CGL (Tier-1) CGL (Tier-2) CHSL (Tier-1) CHSL (Tier-2) Selection Post VII, VIII, XI SSC MTS SSC GD SSC CPO SI CGL (Tier-1) CGL (Tier-2) CHSL (Tier-1) CHSL (Tier-2) Selection Post VII, VIII, XI SSC MTS SSC GD SSC CPO SI

1 1 2 1 2 4 3 2 5 2 5 3 1 2 8 2 12 8 5 10 2 5

CGL (Tier-1) CGL (Tier-2) CHSL (Tier-1) CHSL (Tier-2) Selection Post VII, VIII, XI SSC MTS SSC GD SSC CPO SI CGL (Tier-1) CGL (Tier-2) CHSL (Tier-1) CHSL (Tier-2) Selection Post VII, VIII, XI SSC MTS SSC GD SSC CPO SI CGL (Tier-1) CGL (Tier-2) CHSL (Tier-1) CHSL (Tier-2) Selection Post VII, VIII, XI SSC MTS SSC GD SSC CPO SI CGL (Tier-1) CGL (Tier-2) CHSL (Tier-1) CHSL (Tier-2) Selection Post VII, VIII, XI SSC MTS SSC GD SSC CPO SI

1 1 1 1 1 1 8 2 12 5 10 2 2 5 10 8 22 5 1 4 2 4 5 3 6 6 2 4 1 1

373

(2017–2023)

(2017–2023)

(2017–2023)

(2017–2023)

(2017–2023)

(2017–2023)

(2017–2023)

YCT

21

U

22

V

23

W

24

X

25

Y

26

Z

Synonyms

CGL (Tier-1) CGL (Tier-2) CHSL (Tier-1) CHSL (Tier-2) Selection Post VII, VIII, XI SSC MTS SSC GD SSC CPO SI CGL (Tier-1) CGL (Tier-2) CHSL (Tier-1) CHSL (Tier-2) Selection Post VII, VIII, XI SSC MTS SSC GD SSC CPO SI CGL (Tier-1) CGL (Tier-2) CHSL (Tier-1) CHSL (Tier-2) Selection Post VII, VIII, XI SSC MTS SSC GD SSC CPO SI CGL (Tier-1) CGL (Tier-2) CHSL (Tier-1) CHSL (Tier-2) Selection Post VII, VIII, XI SSC MTS SSC GD SSC CPO SI CGL (Tier-1) CGL (Tier-2) CHSL (Tier-1) CHSL (Tier-2) Selection Post VII, VIII, XI SSC MTS SSC GD SSC CPO SI CGL (Tier-1) CGL (Tier-2) CHSL (Tier-1) CHSL (Tier-2) Selection Post VII, VIII, XI SSC MTS SSC GD SSC CPO SI 374

1 1 2 1 1 1 1 1 1 1 1 1 1 2 1 1 1 1 1 1 2 1 1 1 1 1 1 1 1 1 1 1 1

(2017–2023)

(2017–2023)

(2017–2023)

(2017–2023)

(2017–2023)

(2017–2023)

YCT

Trend Analysis of Questions topicwise from CGL (Pre & Mains) CHSL (Pre & Mains) Selection Post VII, VIII, XI, SSC MTS, SSC GD & Other Exams (2017-2023)

Synonyms

375

YCT

06. SYNONYMS A 1.

Select the most appropriate synonym of the given word. Ample (a) Entertain (b) Sample (c) Temple (d) Sufficient SSC GD 23/01/2023 (Shift-IV) Ans. (d) : Ample (Enough), Synonym - Sufficient (Adequate) Meaning of the other optionsEntertain Amuse, Recreate Sample Pattern, Model Temple Holy Place, House of God 2. Select the most appropriate synonym of the given word. Awkward (a) Dormant (b) Asleep (c) Asleep (d) Unskillful SSC GD 12/01/2023 (Shift-II) Ans. (d) : Awkward (Clumsy) - Synonym - Unskillful (Inept) Meaning of the other optionsDormant - Inactive, Idle Asleep - Dormant, Napping 3. Select the most appropriate synonym of the given word. Absorbed (a) Preoccupied (b) Soaked (c) Replaced (d) Restored SSC CGL (Tier-I) 21/07/2023 (Shift-II) Ans. (b) : The most appropriate synonym of the above given word 'Absorbed' is 'Soaked.' Meaning of other optionsPreoccupied - Restless, Nervous, Replaced - Fungible, Restored - Render, Improve, Thus, the correct answer is option (b). 4. Select the most appropriate synonym of the underlined word. The boxer showed audacity by agreeing to fight the champion. (a) Desperation (b) Honesty (c) Courage (d) Reparation SSC CGL (Tier-I) 25/07/2023 (Shift-IV) Ans. (c) : The most appropriate synonym of the underlined word 'Audacity' is 'courage'. Meaning of other options Desperation - Desalation, Honesty - Integrity, Reparation - Indemnity, Thus, the correct option is (c). 5. Select the most appropriate synonym of the given word. Announced (a) Secret (b) Declared (c) Hid (d) Estrange SSC CGL (Tier-I) 24/07/2023 (Shift-III) Synonyms

Ans. (b) : The most appropriate Synonym of the given word 'Announced' is 'Declared'. Meaning of other words Secret - Confidential Hid - Conceal Estrong - Alienate 6. Select the most appropriate synonym of the given word. Awful (a) Inoffensive (b) Delightful (c) Terrible (d) Acceptable SSC Selection Posts XI-30/06/2023 (Shift-IV) Ans. (c) : Synonym of 'Awful' is 'Terrible'. Other options give different meaning. Inoffensive - Secure Delightful - Enjoyable Acceptable - Plausible 7. Select the word that is the closest meaning (synonym) to the word given below. Attune (a) Assimilate (b) Skew (c) Estrange (d) Disrupt SSC CHSL (Tier-I) 17/03/2023 (Shift-III) Ans. (a) : Attune - Synonym - Assimilate Meaning of other wordDisrupt - Break Estrange - Delete Skew - Inclined 8. Select the most appropriate synonym of the given word. August (a) Imminent (b) Eminent (c) Permanent (d) Illicit SSC MTS-14/07/2022 Shift-III Ans. (b): August (Esteemed), Synonym- Eminent (Famous) Meaning of the other optionsImminent – Adjacent Permanent – Stable, Lasting Illicit – Illegal, Unlawful 9. Select the most appropriate synonym of the given word. Accurate (a) General (b) False (c) Exact (d) Active SSC Constable GD-29/11/2021 Shift-II Ans. (c): Accurate (Actual), Synonym - Exact (Correct) Meaning of the other optionsGeneral - Usual, Common False - Wrong Active - Energetic, Lively

376

YCT

10.

Select the most appropriate synonym of the given word. Avert (a) Prevent (b) Confront (c) Face (d) Permit SSC CGL (Tier-I) 11/04/2022 Shift-I Ans. (a): Avert (Avoid), Synonym – Prevent (Interrupt, Stop) Meaning of the other optionsConfront –Face Face – Encounter Permit – Allow 11. Select the most appropriate synonym of the given word. Arid (a) Fertile (b) Lively (c) Dry (d) Damp SSC CGL (Tier-I) 19/04/2022 Shift-III Ans. (c): Arid (Dry), Synonym- Dry (Drought) Meaning of the other optionsFertile - Fecund Lively - Active, Energetic Damp - Wet, Moist 12. Select the most appropriate synonym of the given word. Abandon (a) Retain (b) Possess (c) Quit (d) Rescue SSC CGL (Tier-I) 19/04/2022 Shift-II Ans. (c): Abandon (Give up), Synonym- Quit (Leave) Meaning of the other optionsRetain – Sustain Possess – Hold, Own Rescue – Save 13. Select the most appropriate synonym of the given word. Audacity (a) Kindness (b) Daring (c) Caution (d) Meekness SSC CGL (Tier-I) 21/04/2022 Shift-III Ans. (b): Audacity (Courage), Synonym – Daring (Bold) Meaning of the other optionsKindness - Compassion, Generosity Caution - Warning Meekness - Modesty 14. Select the most appropriate synonym of the given word. abandon (a) leave (b) start (c) adopt (d) allow SSC CGL (Tier-II) 08.08.2022 Shift-II Ans. (a): Abandon (Give up), Synonym- Leave (Renounce) Meaning of the other optionsStart – Begin Adopt – Siphon Allow – Permit Synonyms

15.

Select the most appropriate synonym of the given word. Arraign (a) Prosecute (b) Arrive (c) Persecute (d) Arrange SSC CHSL 31.05.2022 Shift-III Ans. (a): Arraign (Blame), Synonym- Prosecute (Impaled, Indict) Meaning of the other optionsArrive - Reach Persecute - Torture, Afflict Arrange - Organize 16. Select the most appropriate synonym of the given word. Admire (a) Neglect (b) Appreciate (c) Forget (d) Admonish SSC CHSL 31.05.2022 Shift-III Ans. (b): Admire (Praise), Synonym - Appreciate (Applaud) Meaning of the other optionsNeglect - Disregard Forget - Bewilder Admonish - Reprimand, Rebuke, Scold 17. Select the most appropriate meaning of the underlined word in the given sentence. Police fear that terrorists may try to assassinate the president during his appearance at the summit. (a) Rebuke (b) Deceive (c) Murder (d) Distress SSC CHSL 27.05.2022 Shift-II Ans. (c): The appropriate meaning of the underlined word 'Assassinate' (Kill) in the above sentence is Murder (Slay). Other options have different meaningsRebuke - Reprimand Deceive - Hoodwink Distress - Danger, Risk 18. Select the most appropriate synonym of the underlined word in the given sentence. Simon had always been an ardent follower. (a) averse (b) objective (c) casual (d) passionate SSC MTS-18/07/2022 Shift-I Ans. (d): Ardent (Enthusiastic), Synonym – Passionate (Zealous) Meaning of the other optionsAverse – Unfavourable Objective – Aim Casual – Accidental 19.

377

Select the most appropriate synonym of the given word. Acute (a) Obtuse (b) Faithful (c) Chronic (d) Dire SSC MTS-18/07/2022 Shift-I YCT

Ans. (d): Acute (Intense), Synonym – Dire (Terrible) Meaning of the other optionsObtuse – Dull, Slow Faithful – Credible, Authentic Chronic – Old 20. Select the most appropriate synonym of the given word. ASSIST (a) injure (b) restrict (c) acquire (d) facilitate SSC Stenographer (Grade C & D) 12.11.2021 Shift-II Ans. (d): Assist (Aid), Synonym- Facilitate (Ease, Help) Meaning of the other optionsInjure - Harm, Hurt Restrict - Banned, Inhibit Acquire - Obtain, Gain 21. Select the most appropriate synonym of the given word. Achieve (a) Accuse (b) Adjoin (c) Abduct (d) Accomplish SSC Constable GD-09/12/2021 Shift-II Ans. (d): Achieve (Obtain), Synonym – Accomplish (Fulfill, Attain) Meaning of the other optionsAccuse – Allege Adjoin – Link, Joint Abduct – Kidnap 22. Select the most appropriate synonym of the given word. AUSTERE (a) unadorned (b) luxurious (c) elaborate (d) decorative SSC Stenographer (Grade C & D) 12.11.2021 Shift-II Ans. (a): Austere (Gullible, Rigorous), SynonymUnadorned (Simple) Meaning of the other optionsLuxurious - Opulent, Sumptuous Elaborate - Detailed, Comprehensive Decorative - Ornamental 23. Select the most appropriate synonym of the given word. Adversity (a) Misfortune (b) Emergency (c) Privilege (d) Fortune SSC Constable GD-14/12/2021 Shift-III Ans. (a): Adversity (Calamity, Disaster), Synonym – Misfortune (Bad luck, Tragedy) Meaning of the other optionsEmergency – Necessity, Crisis Privilege– Freedom, Exclusive Right Fortune– Fate, Luck 24. Select the most appropriate synonym of the given word. Aggregate (a) Disperse (b) Total (c) Separate (d) Divide SSC Constable GD-02/12/2021 Shift-I Synonyms

Ans. (b): Aggregate (Whole), Synonym - Total (Entire) Meaning of the other optionsDisperse - Spread Separate - Segregate, Distinct Divide - Distribute, Split 25. Select the most appropriate synonym of the given word. Alter (a) Beautify (b) Transform (c) Preserve (d) Retain SSC Constable GD-14/12/2021 Shift-III Ans. (b): Alter (Change), Synonym – Transform (Convert) Meaning of the other optionsBeautify – Adorn Preserve – Protect, Sustain Retain – Preserve 26. Select the most appropriate synonym of the given word. Attract (a) Repel (b) Disgust (c) Prevent (d) Entice SSC Constable GD-23/11/2021 Shift-III Ans. (d): Attract (Charm), Synonym - Entice (Tempt) Meaning of the other optionsRepel - Setback, Recapture Disgust - Aversion, Hate Prevent - Stop, Interrupt 27. Select the most appropriate SYNONYM of the given word. Appeal (a) Growth (b) Charm (c) Challenge (d) Bloom SSC Constable GD-22/11/2021 Shift-II Ans. (b): Appeal (Attraction/Request), SynonymCharm (Allure) Meaning of the other optionsGrowth – Development Challenge – Objection Bloom– Blossom, Flourish 28. Select the most appropriate synonym of the given word. Amiable (a) Dishonest (b) Disagreeable (c) Honest (d) Friendly SSC Constable GD-25/11/2021 Shift-III Ans. (d): Amiable (Cordial), Synonym – Friendly (Affable) Meaning of the other optionsDishonest – Untruthful, Corrupt Disagreeable – Obnoxious Honest – Faithful 29.

Select the most appropriate synonym of the given word. ASTONISH (a) simplify (b) create (c) surprise (d) cheat SSC Stenographer (Grade C & D) 15.11.2021 Shift-II

378

YCT

Ans. (c): Astonish (Amaze), synonym- Surprise (Wonder) Meaning of the other optionsSimplify - Facilitate Create - Build Cheat - Deceit, Deception 30. Select the most appropriate synonym of the given word. ALLEVIATE (a) aggravate (b) mitigate (c) irritate (d) agitate SSC Stenographer (Grade C & D) 15.11.2021 Shift-II Ans. (b): Alleviate (Reduce), Synonym- Mitigate (Curtail) Meaning of the other optionsAggravate - Provoke Agitate - Perturb, Incite Irritate - Infuriate, Arouse 31. Select the most appropriate synonym of the given word. Abode (a) Annex (b) Habituation (c) Dwelling (d) Aboard SSC CHSL (Tier-I) –19/04/2021 (Shift-I) Ans. (c): Abode - Synonym, Dwelling Meaning of other words– Annex - Affix, Habituation - Inurement, Aboard foreign - On board 32. Select the most appropriate synonym of the given word. Absolve (a) Condemn (b) Attack (c) Pardon (d) Accuse SSC CGL (Tier-I) –23/08/2021 (Shift-I) Ans. (c): Absolve - Synonym- Pardon Meaning of other words– Condemn – Censure, Attack – Assault, Accuse – Denounce 33. Select the most appropriate synonym of the given word. Abolish (a) Introduce (b) Eliminate (c) Continue (d) Establish SSC CGL (Tier-I) –18/08/2021 (Shift-I) Ans. (b): Abolish – Synonym – Eliminate Meaning of other words– Introduce – Acquaint, Continue – Keep on, Establish Install 34. Select the most appropriate synonym of the given word. APPLAUSE (a) approval (b) condemnation (c) attack (d) blame SSC CPO-SI (Tier-II) 27/09/2019 (3 pm - 5 pm) Ans. (a): Applause - Synonym – Approval Meaning of other words– Attack – Assault, Condemnation – Censure, Blame Blemish Synonyms

35.

Select the most appropriate synonym of the given word : ABUNDANT (a) plentiful (b) available (c) necessary (d) latest SSC Stenographer Grade C&D –05/02/2019 (3:5pm)

Ans. (a): 'Abundant' - Synonym- Plentiful Meaning of other words– Necessary – Essential Latest - Backmost Available - Attainable 36. Select the most appropriate synonym of the given word. AUSPICIOUS (a) suspicious (b) playful (c) favourable (d) hapless SSC CPO-SI – 25/11/2020 (Shift-II) Ans. (c): Auspicious – Synonym - Favourable Meaning of other words– Suspicious – Dubious, Playful - Fickle, Hapless Unlucky 37. Select the most appropriate synonym of the given word. ABSURD (a) meaningful (b) ridiculous (c) important (d) rational SSC CPO-SI – 23/11/2020 (Shift-II) Ans. (b): Absurd - Synonym – Ridiculous Meaning of other words– Meaningful - Useful, Important - Vital, Rational Logical 38. Select the most appropriate synonym of the given word. AUTHENTIC (a) incredible (b) counterfeit (c) dubious (d) genuine SSC CPO-SI – 25/11/2020 (Shift-II) Ans. (d): Authentic - Synonym - 'Genuine' Meaning of other words– Incredible – Dubious Counterfeit – Forge Dubious – Suspicious 39. Select the most appropriate synonym of the given word. Accurate (a) sincere (b) complete (c) correct (d) appropriate SSC CPO-SI – 23/11/2020 (Shift-I) Ans. (c): 'Accurate' – Synonym - 'Correct' Meaning of other words– Sincere - Honest, Complete - Overall, Appropriate Suitable 40. Select the most appropriate synonym of the given word. AUSTERE (a) careless (b) strict (c) generous (d) unwilling SSC CGL (Tier-I) – 19/06/2019 (Shift-III) Ans. (b): AUSTERE- Synonym -‘Strict’ Meaning of other words– Careless - Reckless, Generous - Liberal, Unwilling Reluctant

379

YCT

41.

Select the most appropriate synonym of the given word. ATTRIBUTE (a) respect (b) speech (c) praise (d) quality SSC CGL (Tier-I) – 13/06/2019 (Shift-III) Ans. (d) Attribute - Synonym – Quality Meaning of other words– Respect - Regard Praise - Admire Speech - Oration 42. Select the most appropriate synonym of the given word. ASSIST (a) Help (b) Mend (c) Change (d) Create SSC CGL (Tier-I) – 12/06/2019 (Shift-II) Ans. (a): Assist- Synonym- Help Meaning of other words– Mend- Reform, Change- Variation, Create - Build 43. Select the most appropriate synonym of the given word. ADEPT (a) alone (b) skilled (c) unknown (d) kind-hearted SSC CGL (Tier-I) – 12/06/2019 (Shift-III) Ans. (b): Adept– Synonym – Skilled Meaning of other words– Alone- Sole Unknown- Unaware Kind-hearted- Gracious 44. Select the most appropriate synonym of the given word. APPARENT (a) dubious (b) obvious (c) obscure (d) doubtful SSC CPO-SI – 09/12/2019 (Shift-II) Ans. (b): Apparent - Synonym – Obvious Meaning of other words Dubious - Suspicious, Obscure- Vague, Doubtful Suspicious 45. Select the most appropriate synonym of the given word. ADEQUATE (a) meagre (b) sufficient (c) inept (d) unfit SSC CPO-SI – 09/12/2019 (Shift-I) Ans. (b): Adequate - Synonym - 'Sufficient' Meaning of other words– Meagre - Scanty, Inept - Incapable, Unfit - Unworthy 46. Select the most appropriate synonym of the given word. ALOOF (a) protected (b) steady (c) detached (d) rocky SSC CPO-SI – 12/12/2019 (Shift-II) Ans. (c): Aloof- Synonym – Detached Meaning of other words– Protected - Safe, Steady - Firm, Rocky – Petrous Synonyms

47.

Select the most appropriate synonym of the given word. Accurate (a) Real (b) Sincere (c) Precise (d) Genuine SSC CHSL (Tier-I) –11/07/2019 (Shift-I) Ans. (c): Accurate- Synonym – Precise Meaning of other words– Real- Genuine, Sincere- Honest, Genuine-Actual 48. Select the most appropriate synonym of the given word. ANOMALOUS (a) unhealthy (b) indirect (c) peculiar (d) common SSC CHSL (Tier-I) –09/07/2019 (Shift-III) Ans. (c): Anomalous- Synonym- Peculiar Meaning of other words– Unhealthy Morbid Indirect Unobserved Common General 49. Select the most appropriate synonym of the given word. Avenge (a) Punish (b) Cheer (c) Encourage (d) Comfort SSC CHSL (Tier-I) –04/07/2019 (Shift-I) Ans. (a): Avenge- Synonym -'Punish' Meaning of other words– Cheer- Incentive Comfort- Rest Encourage- Stimulate 50. Select the most appropriate synonym of the given word. ADHERE (a) comply (b) ignore (c) release (d) detach SSC CHSL (Tier-I) –02/07/2019 (Shift-II) Ans. (a): Adhere- Synonym- Comply Meaning of other words– Ignore- Negligence Release- Discharge Detach- Separate 51. Select the most appropriate synonym of the given word. ADVERSARY (a) helper (b) assistant (c) rival (d) supporter SSC CHSL (Tier-I) –01/07/2019 (Shift-III) Ans. (c): ADVERSARY- Synonym - 'Rival Meaning of other words– Helper- Auxiliary Assistant- subsidiary Supporter- Advocate 52. Select the most appropriate synonym of the given word. ANODYNE (a) harmful (b) Benign (c) Gigantic (d) Hostile SSC CHSL (Tier-I) 18/03/2020 (Shift-II) Ans. (b): 'Anodyne'- Synonym - 'benign' Meaning of other words– Harmful - Detrimental Gigantic - Huge Hostile - Enemy

380

YCT

53.

Select the most appropriate synonym of the given word. Assemble (a) Disband (b) Detach (c) Address (d) Amass SSC CHSL (Tier-I) 26/10/2020 (Shift-I) Ans. (d): Assemble - Synonym - "Amass" Meaning of other words– Disband – Pluck Detach - Separate Address - Lecture 54. Select the most appropriate synonym of the given word. AGILE (a) Nimble (b) Visible (c) Docile (d) Guile SSC CHSL (Tier-I) 12/10/2020 (Shift-II) Ans. (a): Agile– Synonym – Nimble Meaning of other words– Visible – Manifest Docile – Courteous Guile – Deceit 55. Select the most appropriate synonym of the given word: ALTRUIST (a) Individualist (b) Philanthropist (c) Nutritionist (d) Impressionist SSC CHSL (Tier-I) 21/10/2020 (Shift-III) Ans. (b): 'Altruist'- Synonym- Philanthropist Meaning of other words– Individualist - Selfish, Nutritionist - Dietician, Impressionist - Expressionistic 56. Select the most appropriate synonym of the given word. AFFLUENT (a) Destitute (b) Proficient (c) Wealthy (d) Impoverished SSC CHSL (Tier-I) 19/10/2020 (Shift-II) Ans. (c): Affluent– Synonym – Wealthy Meaning of other words– Destitute – Penniless, Proficient – Skilled Impoverished – Fortuneless 57. Select the most appropriate synonym of the given word. AURA (a) Colour (b) Shriek (c) Halo (d) Noise SSC CHSL (Tier-I) 14/10/2020 (Shift-I) Ans. (c): 'Aura'- Synonym - 'Halo' Meaning of other words– Colour – Paint, Shriek – Scream, Noise – Clamor 58. Select the most appropriate synonym of the given word. Assertion (a) Continuation (b) Rejection (c) Discussion (d) Declaration SSC CHSL (Tier-I) 12/10/2020 (Shift-I) Ans. (d): Assertion - Synonym - 'Declaration' Meaning of other words– Continuation - Frequent Rejection - Abnegation Discussion - Debate Synonyms

59.

Select the most appropriate synonym of the given word. ACCRUE (a) Spread (b) Variety (c) Collect (d) Range SSC CHSL (Tier-I) 20/10/2020 (Shift-II) Ans. (c): 'Accrue' - Synonym - 'Collect' Meaning of other words– Spread – Dispersion, Variety – Variation, Range – Category 60. Select the most appropriate synonym of the given word. ADJUSTMENT (a) Cooperating (b) Balancing (c) Compromising (d) Coordinating SSC CHSL (Tier-I) 19/03/2020 (Shift-I) Ans. (b): Adjustment - Synonym - Balancing Meaning of other words– Cooperating - Collaborate, Compromising - Resolve, Coordinating - Adjust 61. Select the most appropriate synonym of the given word. ALLURING (a) Deceitful (b) Distracting (c) Attractive (d) Cheating SSC CHSL (Tier-I) 17/03/2020 (Shift-I) Ans. (c) Alluring – Synonym – Attractive Meaning of other words– Distracting – Ruffle, Cheating– Thuggery, Deceitful Beguiler 62. In the following question, out of the given four alternatives, select the one which best expresses the meaning of the given word. Abrupt (a) Deliberate (b) Gradual (c) Sudden (d) Regular SSC MTS 9-10-2017 (Shift-II) Ans. (c): Abrupt - Synonym – Sudden Meaning of other words– Deliberate – Intentional, Gradual – Slowly, Regular – General 63. Select the most appropriate synonym of the given word. ANXIOUS (a) Elegant (b) Determined (c) Nervous (d) Bored SSC MTS – 22/08/2019 (Shift-II) Ans. (c): Anxious - Synonym - 'Nervous' Meaning of other words– Elegant - Comely Determined - Firm Bored - Sullen 64. Select the synonym of the given word. Aroma (a) Sneeze (b) Stench (c) Stink (d) Scent SSC MTS – 09/08/2019 (Shift-I) Ans. (d): Aroma - Synonym - 'Scent' Meaning of other words– Sneeze - Sternutation Stench - Smell Stink - Fetor

381

YCT

65.

Select the synonym of the given word. Awkward (a) Likable (b) Clumsy (c) Worthy (d) Scared SSC MTS–08/08/2019 (Shift-III) Ans. (b): Awkward - Synonym - 'Clumsy' Meaning of other words– Likable – Catchy Worthy - Eligible Scared - Fearful 66. Select the most appropriate synonym of the given word. Agile (a) Dull (b) Stiff (c) Quick (d) Rigid SSC MTS – 07/08/2019 (Shift-I) Ans. (c): Agile - Synonym – Quick Meaning of other words– Dull - Floppy Stiff - Harsh Rigid - Tough 67. Select the most appropriate synonym of the given word. Accountable (a) Blameless (b) Slack (c) Unreliable (d) Answerable SSC MTS – 05/08/2019 (Shift-II) Ans. (d): Accountable - Synonym – Answerable Meaning of other words– Blameless - Innocent, Slack - Sluggish, UnreliableIncredible 68. Select the most appropriate synonym of the given word. Artful (a) Artistic (b) Naive (c) Crafty (d) Creative SSC MTS – 20/08/2019 (Shift-III) Ans. (c): Artful - Synonym - 'Crafty' Meaning of other words– Artistic - Beautiful Naive - Simple Creative - Innovative 69. Select the most appropriate synonym of the given word. ABSOLVED (a) scolded (b) Pardoned (c) Punished (d) Boycotted SSC MTS – 19/08/2019 (Shift-II) Ans. (b): Absolved - Synonym - 'Pardoned' Meaning of other words– Scold - Admonish Punish - Chastise Boycott - Estrange 70. Select the most appropriate synonym of the given word. ANTHOLOGY (a) Inheritance (b) Collection (c) Legacy (d) Currency SSC MTS – 19/08/2019 (Shift-II) Ans. (b): Anthology - Synonym – Collection Meaning of other words– Inheritance - Heritage Legacy - Bequest Currency - Money Synonyms

71.

Select the most appropriate synonym of the given word. ADVANTAGEOUS (a) Unlucky (b) Adverse (c) Favorable (d) Crucial SSC MTS – 19/08/2019 (Shift-I) Ans. (c): Advantageous - Synonym – Favorable Meaning of other words– Unlucky - Ominous Adverse - Hostile Crucial - Important 72. Select the most appropriate synonym of the given word. ABANDONED (a) Continued (b) Adopted (c) Initiated (d) Deserted SSC MTS – 19/08/2019 (Shift-I) Ans. (d): Abandoned - Synonym - 'Deserted' Meaning of other words– Continued - Sustained Adopted - Adoptive Initiate - Begin 73. Select the most appropriate synonym of the given word. AFFLUENCE (a) Hardship (b) Poverty (c) Wealth (d) Bankruptcy SSC MTS – 16/08/2019 (Shift-I) Ans. (c): Affluence - Synonym - ‘Wealth’ Meaning of other words– Hardship - Misery Poverty - Penury Bankruptcy - Insolvency 74. Select the most appropriate synonym of the given word. ACCLAIM (a) Censure (b) Praise (c) Blame (d) Criticism SSC MTS – 14/08/2019 (Shift-II) Ans. (b): 'Acclaim'- Synonym – 'Praise' Meaning of other words– Censure – Condemn, Blame – Flaw, Criticism– Review 75. Select the synonym of the given word. Apparent (a) Obsolete (b) Objective (c) Obvious (d) Obscure SSC MTS – 09/08/2019 (Shift-III) Ans. (c): Apparent - Synonym – Obvious Meaning of other words– Obsolete-Defunct, Objective-Purpose, Obscure-Vague 76. Select the synonym of the given word. AGILE (a) Aggressive (b) Active (c) Dull (d) Lazy SSC GD – 14/02/2019 (Shift-II) Ans. (b): Agile - Synonym – Active Meaning of other words– Aggressive - Rampant, Dull - Boring, Lazy Sluggish

382

YCT

77.

Select the synonym of the given word. APPARENT (a) Considerable (b) Doubtful (c) Obvious (d) Questionable SSC GD – 11/02/2019 (Shift-II) Ans. (c): Apparent - Synonym – Obvious Meaning of other words– Considerable - Significant Doubtful - Suspicious Questionable - Suspect 78. Select the synonym of the given word. ADORN (a) create (b) produce (c) generate (d) decorate SSC GD – 02/03/2019 (Shift-II) Ans. (d): Adorn - Synonym – Decorate Meaning of other words– Create - Build, Produce – Yield, Generate Proliferate 79. Select the most appropriate synonym of the given word. ABSOLUTE (a) limited (b) complete (c) partial (d) conditional SSC CHSL (Tier-I) –09/08/2021 (Shift-I) Ans. (b): Absolute - Synonym - Complete Meaning of other words– Limited - Confined, Conditional - Organized, Partial Fractional

B 80.

Select the most appropriate synonym of the given word. Blunt (a) Cheerful (b) Dull (c) Valiant (d) Sharp SSC GD 31/01/2023 (Shift-II) Ans. (b) : Blunt (edgless), Synonym - Dull (Boring) Meaning of the other optionsCheerful - Hilarious, Glad Valiant - Brave, Gallant Sharp - Intense, fast 81. Select the most appropriate synonym of the given word. Baffle (a) Amaze (b) Enlighten (c) Clear (d) Comprehend SSC GD 13/02/2023 (Shift-I) Ans. (a) : 'Amaze' (astonished) is the most appropriate synonym of the given word 'baffle'. The meaning of other options is – Enlighten :- make understand. Clear :- distinct. Comprehend :- get the meaning of something. 82. Select the most appropriate synonym of the given word. Blast (a) Blow up (b) Recall (c) Plead (d) Share SSC GD 10/01/2023 (Shift-I) Synonyms

Ans. (a) : Blast (Explosion, Eruption), Synonym - Blow up (Acurate, explosion) Meaning of the other optionsRecall - Memorize Plead - Appeal, Peition Share - Part, Portion 83. Select the most appropriate synonym for the underlined word in the following sentence. She’s so benevolent that she can’t even harm a fly, let alone hurting someone. (a) Gloomy (b) Unfortunate (c) Compassionate (d) Grudging SSC CHSL (Tier-I) 10/08/2023 (Shift-I) Ans. (c) : The synonym of the underlined word of the sentence, benevolent – (kind, friendly and helpful to others) will be option (c) compassionate - (To recognize the suffering of others and take action to help). Meaning possessed by other options – Gloomy – filled with melancholy. Unfortunate – Not favored by luck Grudging – Displaying reluctance while doing or giving something. 84. Select the most appropriate synonym of the underlined word. She was completely baffled by his strange behaviour. (a) unfazed (b) confused (c) angered (d) relieved SSC CHSL (Tier-I) 08/08/2023 (Shift-II) Ans. (b) : The most appropriate synonym of the above underlined word 'baffled' is 'Confused'. Meaning of other options – Unfazed – Undaunted, Composed, Angered – Annoy, Irritate, Relieved – Glad, Grate full, 85. Select the most appropriate synonym of the given word. Bargain (a) Disagreement (b) Gain (c) Dispute (d) Deal SSC Constable GD-13/12/2021 Shift-III Ans. (d): Bargain (Negotiate), Synonym- Deal (Business) Meaning of the other optionsDisagreement - Dissent, Discord Gain - Benefit Dispute - Controversy, Fight 86. Select the most appropriate synonym of the given word. Batter (a) Clash (b) Struggle (c) Beat (d) Fight SSC Stenographer (Grade C & D) 11.11.2021 Shift-I Ans. (c): Batter (Strike, Hit), Synonym - Beat (Hit) Meaning of the other wordsClash - Struggle Struggle - Contend Fight - Battle, Quarrel

383

YCT

87.

Select the most appropriate synonym of the given word. Bafflement (a) Clarity (b) Confusion (c) Pleasure (d) Cleanliness SSC CHSL 24.05.2022 Shift-III Ans. (b): Bafflement (Confusion), Synonym-Confusion (Perplexity) Meaning of the other optionsClarity - Lucidity Pleasure - Amusement, Happiness Cleanliness - Neatness 88. Select the most appropriate synonym of the given word. Bull-Headed (a) Headstrong (b) Clear-headed (c) Muddle-headed (d) Weak-minded SSC CHSL (Tier-I) –16/04/2021 (Shift-I) Ans. (a): Bull-Headed - Synonym, Headstrong Meaning of other words– Clear - Headed-Lucid, Muddle-Headed-Confused, Weak-Minded-Irresolute 89. Select the most appropriate synonym of the given word. Banner (a) March (b) Protest (c) Poster (d) Rally SSC CGL (Tier-I) –23/08/2021 (Shift-I) Ans. (c): Banner - Synonym - Poster Meaning of other words– March - Parade, Protest - Resist, Rally – Forgather 90. Select the most appropriate synonym of the given word. BENIGN (a) favourable (b) malignant (c) harsh (d) severe SSC CPO-SI – 25/11/2020 (Shift-I) Ans. (a): Benign - Synonym - Favourable Meaning of other words Malignant – Deadly, Harsh - Rigid, Severe - Drastic 91. Select the most appropriate synonym of the given word. BEGUILE (a) Disenchant (b) Hasten (c) Repulse (d) Deceive SSC CPO-SI – 13/12/2019 (Shift-I) Ans. (d): Beguile - Synonym – Deceive Meaning of other words– Disenchant - Fascination, Hasten – Hurry, Repulse Expel 92. Select the most appropriate synonym of the given word. BARREN (a) Fertile (b) Bountiful (c) Desolate (d) Fruitful SSC CHSL (Tier-I) 16/10/2020 (Shift-II) Ans. (c): 'Barren'- Synonym - 'Desolate' Meaning of other words– Fertile - Productive, Fruitful, Bountiful - Abundant Synonyms

93.

In the following question, out of the given four alternatives, select the one which best expresses the meaning of the given word: Bewitched (a) Disgusting (b) Captivated (c) Turn of (d) Prevent SSC MTS 11-10-2017 (Shift-II) Ans. (b): Bewitched – Synonym – Captivated Meaning of other words– Disgusting - Odious Prevent - Stop 94. Select the most appropriate synonym of the given word. Brisk (a) Frugal (b) Languid (c) Quick (d) Sluggish SSC MTS – 08/08/2019 (Shift-I) Ans. (c): Brisk- Synonym – Quick Meaning of other words– Frugal - Thrifty Languid - Drowsy Sluggish - Lazy 95. Select the most appropriate synonym of the given word. Benevolent (a) Miser (b) Cruel (c) Malevolent (d) Generous SSC MTS – 06/08/2019 (Shift-II) Ans. (d): Benevolent- Synonym – Generous Meaning of other words– Miser- Covetous, Cruel- Brutal Malevolent - Deceitful 96. Select the most appropriate synonym of the given word. BENEFICIAL (a) Worthless (b) Harmful (c) Advantageous (d) Adverse SSC MTS – 16/08/2019 (Shift-III) Ans. (c): Beneficial - Synonym – Advantageous Meaning of other words– Worthless - Waste, Harmful - Detrimental, Adverse Hostile 97. Select the synonym of the given word. BOUNTIFUL (a) Pretty (b) Religious (c) Spiritual (d) Generous SSC GD – 02/03/2019 (Shift-I) Ans. (d): Bountiful - Synonym – Generous Meaning of other words– Pretty - Lovely, Religious - Virtuous, Spiritual Mental 98. Select the synonym of the given word. BRITTLE (a) Coarse (b) Enduring (c) Delicate (d) Durable SSC GD – 02/03/2019 (Shift-II) Ans. (c): Brittle - Synonym – Delicate Meaning of other words– Coarse - Knobby, Enduring - Lasting, Durable Permanent

384

YCT

104. Select the most appropriate synonym of the given word. Colloquial 99. Select the most appropriate synonym of the (a) Lexical (b) Conversational given word. (c) Prosodic (d) Linguistic Console SSC GD 10/01/2023 (Shift-I) (a) Solace (b) Arteriole Ans. (b) : Colloquial (Informal), Synonym (c) Fret (d) Torment SSC MTS 10/05/2023 (Shift-I) Conversational (talktive) Ans. (a) : The most appropriate synonym of the given Meaning of the other optionword 'console' is 'solace'. The meaning of other words is Lexical - Dicrionary, etymological Prasodic - Poetic rhythm, metrics ____ Linguistic - Polyglot Arteriole means- capillary Fret means- worry 105. Select the most appropriate synonym of the Torment means- Agony given word. Confuse 100. Choose the word that means the same as the given word. (a) Puzzle (b) Disunion Confirm (c) Compose (d) Admit (a) Corrosion (b) Corroborate SSC GD 06/02/2023 (Shift-III) (c) Collision (d) Collaborate Ans. (a) : From the above word 'puzzle' is the same SSC MTS 04/05/2023 (Shift-II) meaning (synonym) as the word 'confuse'. Ans. (b) : The most appropriate similar meaning for the The other options give different meaning. given word 'confirm' is 'corroborate' Disunion :- the termination or destruction of union. Meaning of other words Compose :- compile, write. Corrosion means- erosion Admit :- allow in, accept. Collision means- concussion, crash 106. Select the most appropriate synonym of th Collaborate means- Cooperate given word. 101. Select the appropriate synonym for the Competent underlined word. (a) Unfit (b) Capable Make sure that you always challenge yourself. (c) Overqualified (d) Unskilled (a) Approve (b) Carry SSC GD 01/02/2023 (Shift-II) (c) Place (d) Question Ans. (b) : 'Capable' is the most appropriate synonym of SSC MTS 02/05/2023 (Shift-I) the given word– 'Competent'. Ans. (d) : The most appropriate synonym for the The meaning of other options is– underline word 'challenge' is 'question' which means Unfit :- unsuitable. defiance'. Meaning of other words ___ Overqualified :- overexperienced. Approve means- accept, applaud carry means bear. Unskilled :- incompetent. 102. Choose the word that means the same as the 107. Select the most appropriate synonym of the given word. given word. Consecutive Consequence (a) Executive (b) Important (a) Outcome (b) Begin (c) Organized (d) Successive (c) Origin (d) Start SSC MTS 15/05/2023 (Shift-I) SSC CGL (Tier-I) 18/07/2023 (Shift-III) Ans. (d) : The synonym of the above given word Ans. (a) : The most appropriate synonym of the above 'Consecutive' is Successive'. word 'consequence' (result) is 'Outcome'. Meaning of other options – Meaning of other options Executive :- Managerial, Begin - Induct, start up, Important :- Vital, Origin - Basic, Fount, Organized :- Unified, 103. Select the most appropriate synonym of the Start - Seed, Outset, 108. Select the most appropriate synonym to given word. substitute the underlined word. Commerce The weather forecast mentioned that there (a) Stream (b) Trade would be a cloud burst this afternoon. (c) Class (d) Bank (a) sandstorm (b) snowfall SSC GD 27/01/2023 (Shift-I) (c) rainstorm (d) famine Ans. (b) : Commerce (Business), Synonym- Trade SSC CGL (Tier-I) 27/07/2023 (Shift-III) (Market). Meaning of the other optionsAns. (c) : The most appropriate synonym of the Stream Current, flow underlined word 'Cloud burst' is 'rainstorm'. Class Category, Group Other options give different meaning. Bank Edge, Coast Thus, the correct answer is option (c).

C

Synonyms

385

YCT

109. Select the most appropriate synonym of the given word. Crooked (a) Twisted (b) Taken (c) Admirable (d) Toasted SSC CGL (Tier-I) 24/07/2023 (Shift-III) Ans. (a) : The most appropriate synonym of given word 'Crooked' is 'twisted'. Meaning of other wordTaken - Contracted Admirable - Plausible Toasted - Baked 110. Select the most appropriate synonym of the underlined word. In order to support countries with a concerted approach for an inclusive, and government-led assessment of post-disaster damaes, losses, and recovery needs, as well as the development of a comprehensive recovery plan, the European Commission, the World Bank, and the United Nation adopted the Post Disaster Needs Assessment (PDNA) process in 2008. (a) Clear (b) Sustainable (c) Restricted (d) Complete SSC CGL (Tier-I) 24/07/2023 (Shift-III) Ans. (d) : The most appropriate synonym of the underlined word 'comprehensive' is 'complete'. Meaning of other wordsSustainable - Renewable Restricted - Limited Clear - Apparent 111. Select the most appropriate synonym of the underlined word in the following sentence. UNESCO is committed to protecting individual privacy and securing the personal information made available to us when you visit unesco.org, as well as UNESCO pages on other sites. (a) devoted (b) inconstant (c) disloyal (d) unfaithful SSC Selection Posts XI-30/06/2023 (Shift-IV) Ans. (a) : In the above sentences, synonym of underline word, 'committed' is 'Devoted'. Other options give different meanings. Inconstant - unstable Disloyed - faithless Unfaithful - Betrayer 112. Select the most appropriate synonym of the underlined word in the given sentence. His conceit prevented him from accepting any criticism, even if it was meant to be constructive. (a) modesty (b) cowardice (c) egotism (d) prejudice SSC CHSL (Tier-I) 09/08/2023 (Shift-III) Ans. (c) : The appropriate synonym of the underlined word 'conceit' is 'egotism'. other options contain following meaning– Modesty – Socially acceptable behavior/ quality of not being too proud. Cowardice – lack of courage/ A failure of character in the face of a challenge. Prejudice – A partiality that prevents objective consideration Synonyms

113. Select the most appropriate synonym of the given word. Colleague (a) Distend (b) Co-worker (c) Consult (d) Exempt SSC CHSL (Tier-I) 08/08/2023 (Shift-II) Ans. (b) : The most appropriate synonym of the above given word 'Colleague' (comrade) is 'Co-worker' Meaning of other options – Distend – Expand, puff, Consult – Discuss, Examine, Exempt – Loose, Immune, 114. Choose the word that means the same as the given word. Calibrate (a) Gauze (b) Assail (c) Gauge (d) Affront SSC CHSL (Tier-I) 10/03/2023 (Shift-III) Ans. (c) : Calibrate - Synonym - Gauge Meaning of other wordAssail – Attack Affront – Insult Gauze – Smoke 115. Select the most appropriate SYNONYM of the given word. Compose (a) Collect (b) Cuddle (c) Confuse (d) Clutch SSC Constable GD-30/11/2021 Shift-III Ans. (a): Compose (Comprise), Synonym - Collect (Accumulate) Meaning of the other optionsCuddle - Embrace Confuse - Baffle, Perplex Clutch - Grip, Hold 116. Select the most appropriate synonym of the given word. Clandestine (a) Blatant (b) Omate (c) Open (d) Undercover SSC MTS-13/07/2022 Shift-I Ans. (d): Clandestine (Secret), Synonym- Undercover (Covert, Hidden) Meaning of the other optionsBlatant – Dominant, Potent Ornate – Adorn Open – Unclock, Unclose 117. Select the most appropriate synonym of the underlined word in the given sentence. He was critically injured in the accident. (a) Favourably (b) severely (c) trivially (d) Confidently SSC MTS-07/07/2022 Shift-II Ans. (b): Critically (Seriously), Synonym - Severely (Gravely) Meaning of the other optionsFavourably – Graciously Trivially – Frivolous Confidently – Expectantly, Positivity

386

YCT

118. Select the most appropriate synonym of the given word. Caustic (a) Acidic (b) Photogenic (c) Basic (d) Soothing SSC MTS-08/07/2022 Shift-I Ans. (a): Caustic (Tart), Synonym - Acidic (Bitter, Sour) Meaning of the other optionsPhotogenic – Attractive Basic – Fundamental Soothing – Calming 119. Select the most appropriate synonym of the given word. Callous (a) Brutal (b) Problematic (c) large (d) Humane SSC MTS-08/07/2022 Shift-I Ans. (a): Callous (Unkind), Synonym - Brutal (Ruthless) Meaning of the other optionsProblematic – Troublesome Large – Huge Humane – Kind 120. Select the most appropriate synonym of the given word. Confront (a) Mingle (b) Challenge (c) Scheme (d) Conceal SSC CHSL 24.05.2022 Shift-III Ans. (b): Confront (Face), Synonym- Challenge (Defiance, Dare) Meaning of the other optionsMingle - Conspire, Amalgamate Scheme - Plan, Project Conceal - Hide 121. Select the most appropriate synonym of the given word. Carnage (a) Slaughter (b) Lust (c) Plague (d) Revelry SSC Stenographer (Grade C & D) 11.11.2021 Shift-I Ans. (a): Carnage (Massacre), Synonym- Slaughter (Killing, Murder) Meaning of the other optionsPlague - Calamity, Pestilence Lust - Desire, Erotic Revelry - Celebration, Festivity 122. Select the most appropriate synonym of the given word. Common (a) Individual (b) Usual (c) Scarce (d) Particular SSC Constable GD-13/12/2021 Shift-III Ans. (b): Common (General), Synonym - Usual (Normal) Meaning of the other optionsIndividual - Personal Scarce - Scant Particular - Special Synonyms

123. Select the most appropriate synonym of the given word. Challenge (a) Answer (b) Acceptance (c) Objection (d) Peace SSC CHSL (Tier-I) –15/04/2021 (Shift-I) Ans. (c): Challenge - Synonym - Objection Meaning of other words– Peace - Silence, Answer – Reply, Acceptance - Assent 124. Select the most appropriate synonym of the given word. Crust (a) Root (b) Core (c) Kernel (d) Shell SSC Constable GD-03/12/2021 Shift-II Ans. (d): Crust (Coat, Scale), Synonym– Shell (Shield, Coat) Meaning of the other optionsRoot - Source, Origin Core - Basic Kernel - Seed, Core 125. Select the most appropriate synonym of the given word. CONSPICUOUS (a) evident (b) concealed (c) obscure (d) vague SSC Stenographer (Grade C & D) 15.11.2021 Shift-I Ans. (a): Conspicuous (Obvious, Visible), SynonymEvident (Apparent, Clear) Meaning of the other optionsConcealed - Secrete Obscure - Unclear Vague - Ambiguous 126. Select the most appropriate synonym of the given word. CONSEQUENCE (a) source (b) attraction (c) inception (d) outcome SSC Stenographer (Grade C & D) 12.11.2021 Shift-I Ans. (d): Consequence (Result), Synonym- Outcome (Effect) Meaning of the other optionsAttraction - Charm Inception - Beginning Source - Origin 127. Select the most appropriate synonym of the given word. COMMENCE (a) initiate (b) conclude (c) finish (d) cease SSC Stenographer (Grade C & D) 12.11.2021 Shift-I Ans. (a): Commence (Begin), Synonym-Initiate (Start) Meaning of the other optionsFinish - End Cease - Close, Stop Conclude - Deduce 128. Select the most appropriate synonym of the given word. CALLOUS (a) inaccurate (b) insensitive (c) incredible (d) inexperienced SSC CHSL (Tier-I) –06/08/2021 (Shift-I)

387

YCT

Ans. (b): Callous - Synonym - Insensitive Meaning of other words– Incredible - Unreliable, Inexperienced - Inexpert, Inaccurate - Wrong 129. Select the most appropriate synonym of the given word. CONCEITED (a) arrogant (b) diffident (c) inferior (d) tentative SSC CPO-SI – 23/11/2020 (Shift-II) Ans. (a): Conceited - Synonym - Arrogant Meaning of other words– Diffident - Bashful Inferior - Low Tentative - Inquiring 130. Select the most appropriate synonym of the given word. CORDIAL (a) cold (b) rude (c) friendly (d) aloof SSC CPO-SI – 23/11/2020 (Shift-II) Ans. (c): Cordial- Synonym – Friendly Meaning of other words– Cold - Chill Rude - Vulgar Aloof - Separate 131. Select the most appropriate synonym of the given word. CONDONE (a) impose (b) prevent (c) forgive (d) forbid SSC CPO-SI – 23/11/2020 (Shift-II) Ans. (c): Condone - synonym - Forgive Meaning of other words– Impose - Inflict Prevent - Stop Forbid - Stop 132. Select the most appropriate synonym of the given word. COVETED (a) despised (b) desired (c) covered (d) renounced SSC CPO-SI – 24/11/2020 (Shift-II) Ans. (b): 'Coveted'- Synonym 'Desired' Meaning of other words– Despised - Disdain Covered - Impotent Renounced - Leave 133. Select the most appropriate synonym of the given word. CATASTROPHE (a) expansion (b) calamity (c) restraint (d) prosperity SSC CPO-SI – 23/11/2020 (Shift-II) Ans. (b): Catastrophe - Synonym - Calamity Meaning of other words– Expansion - Detail Restraint - Control Prosperity - Affluence 134. Select the most appropriate synonym of the given word. CHASTE (a) divine (b) defiled (c) pure (d) liberated SSC CPO-SI – 25/11/2020 (Shift-I) Synonyms

Ans. (c): Chaste- Synonym – Pure Meaning of other words– Divine - Classic, Defiled - Profane, Liberated - Free 135. Select the most appropriate synonym of the given word. CONVERSE (Verb) (a) talk (b) display (c) oppose (d) agree SSC CGL (Tier-I) – 19/06/2019 (Shift-III) Ans. (a): Converse - Synonym- Talk Meaning of other words– Display- Show, Oppose- Stand, Agree- Accord 136. Select the most appropriate synonym of the given word. CONCISE (a) lengthy (b) detailed (c) brief (d) complex SSC CGL (Tier-I) – 11/06/2019 (Shift-III) Ans. (c): Concise - Synonym - ‘Brief’ Meaning of other words– Lengthy- Long, Detailed- Extensive, Complex- Complicated 137. Select the synonym of the given word. CHRONIC (a) persistent (b) common (c) ordinary (d) temporary SSC CGL (Tier-I) – 04/06/2019 (Shift-I) Ans. (a): Chronic - Synonym - 'Persistent' Meaning of other words– Common- General Ordinary- Normal Temporary- Temporal 138. Select the synonym of the given word. COERCE (a) pressurize (b) cajole (c) leave (d) enchant SSC CGL (Tier-I) – 04/06/2019 (Shift-I) Ans. (a): Coerce - Synonym - 'Pressurize' Meaning of other words– Cajole- Enticement Leave- Renounce Enchant- Spellbind 139. Select the most appropriate synonym of the given word. CONDEMN (a) denounce (b) commend (c) compliment (d) approve SSC CPO-SI – 09/12/2019 (Shift-II) Ans. (a): Condemn- Synonym - 'Denounce' Meaning of other words– Commend- Admire, Compliment - Praise, Approve Favor 140. Select the most appropriate synonym of the given word. CONTAMINATE (a) sanctify (b) cleanse (c) purify (d) pollute SSC CPO-SI – 09/12/2019 (Shift-I) Ans. (d) Contaminate- Synonym - 'Pollute' Meaning of other words– Sanctify - Redeem, Cleanse - Wash, Purify - Hallow

388

YCT

141. Select the most appropriate synonym of the given word. COMPLACENT (a) discontent (b) aggressive (c) amicable (d) smug SSC CPO-SI – 11/12/2019 (Shift-I) Ans. (d) Complacent - Synonym – Smug Meaning of other word Discontent - Chagrin, Aggressive - Rampant, Amicable - Friendly 142. Select the most appropriate synonym of the given word. Camouflage (a) Exhibit (b) Disguise (c) Divulge (d) Expose SSC CHSL (Tier-I) –11/07/2019 (Shift-II) Ans. (b) Camouflage- Synonym – Disguise Meaning of other words– Exhibit- Present, Divulge - Evolve Expose - Unbare 143. Select the most appropriate synonym of the given word. Climax (a) Preface (b) Epilogue (c) Culmination (d) Prologue SSC CHSL (Tier-I) –11/07/2019 (Shift-III) Ans. (c) Climax- Synonyms - Culmination Meaning of other words– Preface - Prelude, Epilouge - Postscript, Prologue Preface 144. Select the most appropriate synonym of the given word. CANDID (a) cruel (b) frank (c) arrogant (d) sweet SSC CHSL (Tier-I) –04/07/2019 (Shift-II) Ans. (b) Candid - Synonym- 'Frank' Meaning of other words– Cruel - Brutal, Arrogant – Conceited, Sweet - Dulcet 145. Select the most appropriate synonym of the given word. CURRENT (a) antiquated (b) uncommon (c) old (d) present SSC CHSL (Tier-I) –02/07/2019 (Shift-II) Ans. (d) Current- Synonym- Present Meaning of other words– Antiquated - Ancient Uncommon - Unusual Old - Decrepit 146. Select the most appropriate synonym of the given word. COMBUSTIBLE (a) unbreakable (b) nonexplosive (c) fragile (d) inflammable SSC CHSL (Tier-I) –02/07/2019 (Shift-III) Ans. (d): 'Combustible' - Synonyms- 'Inflammable' Meaning of other words– Unbreakable - Non breakable Nonexplosive - Fireproof Fragile - Delicate Synonyms

147. Select the most appropriate synonym of the given word. CONGENIAL (a) Aloof (b) Bombastic (c) Compatible (d) Servile SSC CHSL (Tier-I) 21/10/2020 (Shift-II) Ans. (c): Congenial - Synonym – Compatible Meaning of other words– Aloof – Separate Bombastic – Grandiloquent Servile – Sneaky 148. Select the most appropriate synonym of the given word. CONSTRAINT (a) Concern (b) Control (c) Hazard (d) Burden SSC CHSL (Tier-I) 13/10/2020 (Shift-II) Ans. (b): Constraint- Synonym- Control Meaning of other words– Concern - Anxiety Hazard - Danger Burden - Load 149. Select the most appropriate synonym of the given word. COVE (a) Crater (b) Pit (c) Hollow (d) Bay SSC CHSL (Tier-I) 17/03/2020 (Shift-II) Ans. (d): Cove – Synonym - Bay Meaning of other words– Crater – Delve Pit – Mine Hollow – Void 150. In the following question, out of the given four alternatives, select the one which best expresses the meaning of the given word. Concession. (a) Denial (b) Protest (c) Retrograde (d) Allowance SSC MTS 9-10-2017 (Shift-I) Ans. (d): Concession - Synonym- Allowance Meaning of other words– Denial - Negation, Protest - Resist, Retrograde - Fallen 151. In the following question, out of the given four alternatives, select the one which best expresses the meaning of the given word. Cavalier (a) Nobleman (b) Gentleman (c) Rude boy (d) Patrician SSC MTS 11-10-2017 (Shift-III) Ans. (c): Cavalier – Synonym- Rude boy Meaning of other words– Nobleman – Aristocrat Gentleman – Mynheer Patrician – Oligarch 152. In the followingquestion, out of the given four alternatives, select the one which best expresses the meaning of the given word. Commotion (a) Fuss (b) Calm (c) Silence (d) Quiet SSC MTS 11-10-2017 (Shift-II)

389

YCT

Ans. (a): Commotion - Synonym- Fuss Meaning of other words– Calm - Quiet, Silence - Peace 153. In the following question, out of the given four alternative, select the one which best expresses the meaning of the given word. Chagrin (a) Embarrassment (b) Delight (c) Pleasure (d) Happiness SSC MTS 11-10-2017 (Shift-II) Ans. (a): Chagrin – Synonym- Embarrassment Meaning of other words– Delight - Happiness, Pleasure 154. Select the most appropriate synonym of the given word. CONTROL (a) Reduce (b) Relieve (c) Resign (d) Regulate SSC MTS – 22/08/2019 (Shift-II) Ans. (d): 'Control'- Synonym - 'Regulate' Meaning of other words– Reduce - Detract Relieve- Assist Resign- Relinquish 155. Select the synonym of the given word. Confusion (a) Assurance (b) Muddle (c) Devotion (d) Clarity SSC MTS – 09/08/2019 (Shift-I) Ans. (b): Confusion - Synonym - 'Muddle' Meaning of other words– Assurance - Warranties Devotion - Loyalty Clarity - Lucidity 156. Select the most appropriate synonym of the given word. Courteous (a) Optional (b) Arrogant (c) Polite (d) Meritorious SSC MTS – 05/08/2019 (Shift-III) Ans. (c): Courteous- Synonym – Polite Meaning of other words– Optional - Alternative, Arrogant – Conceited, Meritorious - Godly 157. Select the most appropriate synonym of the given word. Chide (a) Flatter (b) Commend (c) Approve (d) Rebuke SSC MTS – 02/08/2019 (Shift-III) Ans. (d): Chide- Synonym – Rebuke Meaning of other words– Flatter - Cringe Commend - Applaud Approve - Favor 158. Select the most appropriate synonym of the given word. Capable (a) Competent (b) Conceited (c) Complacent (d) Consistent SSC MTS – 02/08/2019 (Shift-II) Synonyms

Ans. (a): Capable - Synonym – Competent Meaning of other words– Conceited - Arrogant Complacent - Glad Consistent - Relevant 159. Select the most appropriate synonym of the given word. Courage (a) Meekness (b) Valour (c) Humility (d) Timidity SSC MTS – 02/08/2019 (Shift-I) Ans. (b): Courage - Synonym - Valour Meaning of other words– Meekness - Modesty Humility - Delicacy Timidity - Cowardice 160. Select the most appropriate synonym of the given word. CALM (a) violent (b) peaceful (c) windy (d) stormy SSC MTS – 13/08/2019 (Shift-II) Ans. (b): Calm - Synonym – Peaceful Meaning of other words– Violent - Furious Windy - Gusty Stormy - Fierce 161. Select the most appropriate synonym of the given word. Chaste (a) Pure (b) Lustful (c) Fascinating (d) Loyal SSC MTS – 21/08/2019 (Shift-III) Ans. (a): Chaste - Synonym – Pure Meaning of other words– Lustful - Salacious, Fascinating - Delightful, Loyal Faithful 162. Select the most appropriate synonym of the given word : Contempt (a) Respect (b) Approve (c) Permit (d) Hatred SSC MTS – 22/08/2019 (Shift-I) Ans. (d): 'Contempt'- Synonym - 'Hatred' Meaning of other words– Respect - Regard Approve - Favor Permit - Allow 163. Select the synonym of the given word. Confine (a) Liberate (b) Restrict (c) Rescue (d) Release SSC GD – 08/03/2019 (Shift-III) Ans. (b): 'Confine'- Synonym - 'Restrict' Meaning of other words– Liberate - Release Rescue - Redeem Release - Discharge 164. Select the synonym of the given word. CONTRARY (a) Affiliated (b) Opposite (c) Similar (d) Linked SSC GD – 08/03/2019 (Shift-III)

390

YCT

Ans. (b): 'Contrary'- Synonym - 'Opposite' Meaning of other words– Affiliated - Allied Similar - Identical Linked - Mixed 165. Select the synonym of the given word. CURATIVE (a) Trembling (b) Damaging (c) Feeding (d) Healing SSC GD – 11/02/2019 (Shift-II) Ans. (d): Curative - Synonym – Healing Meaning of other words– Trembling - Thrill Damaging - Hurtful Feeding - Nourishing

D 166. Select the appropriate synonym for the underlined word. It was a distraction. (a) faith (b) conviction (c) confusion (d) assurance SSC MTS 08/05/2023 (Shift-III) Ans. (c) : The most appropriate synonym for the under lined word ' distraction' is 'Confusion'. Meaning of other words – Faith means – confidence Conviction means – Reliance Assurance means – Guarantee Thus, the correct answer is option (c) 167. Select the most appropriate synonym of the given word. Debased (a) Corrupted (b) Unamenable (c) Aggrandize (d) Amenable SSC MTS 12/05/2023 (Shift-III) Ans. (a) : The most appropriate synonym of the given word 'Debased' (Dissolute) is 'corrupted'. Meaning of other options– Unamenable – Incurable, Aggrandize – Increase, Amenable – Responsible, Hence, the correct answer is option (a) 168. Select the most appropriate synonym of the given word. Deficient (a) Insufficient (b) Distinct (c) Distinct (d) Abundant SSC GD 06/02/2023 (Shift-III) Ans. (a) : 'Insufficient' (meager) is the most appropriate synonym of the given word 'deficient' (lacking). The other meanings– Distinct :- separate. Abundant :- copious. 169. Select the most appropriate synonym of the underlined word. During the summer of 1893, Miss Sullivan and I visited the World's Fair with Dr. Alexander Graham Bell. I recall with unmixed delight those days when a thousand childish fancies became beautiful realities. Synonyms

(a) dismay (b) memory (c) enchant (d) emotion SSC CGL (Tier-I) 21/07/2023 (Shift-II) Ans. (c) : The most appropriate synonym of the above given underlined word 'delight' is 'enchant.' Meaning of other optionsDismay - Discomfort. Memory - Souvenir, Emotion - Sensation, Thus, the correct answer is option (c). 170. Select the most appropriate synonym of the given word from the sentence. Dangerous The traffic police have released a safe advisory advisory that driving without seat belts is risky, beneficial, and fearful for human life. (a) Risky (b) Beneficial (c) Safe (d) Released SSC CGL (Tier-I) 24/07/2023 (Shift-III) Ans. (a) : The most appropriate synonym of the given word 'Dangerous' is 'Risky'. Meaning of other wordsBeneficial - Remunerative Safe - Secure Released - Liberated 171. Select the word that is closest in meaning (SYNONYM) to the word given below. 'Decimate' (a) mend (b) wreck (c) harbour (d) establish SSC CHSL (Tier-I) 21/03/2023 (Shift-IV) Ans. (b) : Decimate - Synonym - 'wreck' Meaning of other wordestablish – Founded harbour – Refuge Mend – Improvement 172. Select the word that is closest in meaning (SYNONYM) to the word given below. DEPRESSED (a) DOLEFUL (b) DYNAMIC (c) STURDY (d) NOISY SSC CHSL (Tier-I) 20/03/2023 (Shift-I) Ans. (a) : Depressed - Synonym - "Doleful" Meaning of other wordsNoisy- Cheeky Sturdy- Strong Dynamic- Mobile 173. Select the most appropriate synonym of the underlined word in the given sentence. The plan is to demolish the kitchen area and build a new room. (a) curtail (b) forge (c) raze (d) refurbish SSC MTS-05/07/2022 Shift-I Ans. (c): The most appropriate synonym of the underlined word 'Demolish' (Destroy) is option (c) 'Raze' (Dissipate). Other options have different meaningsCurtail – Mitigate Forge – Counterfeit Refurbish – Renovate

391

YCT

174. Select the most appropriate synonym of the given word. Despicable (a) Commendable (b) Laudable (c) Contemptible (d) Desirable SSC CHSL (Tier-I) –05/08/2021 (Shift-I) Ans. (c): Despicable - Synonym - Contemptible Meaning of other words– Desirable - Pleasing, Commendable - Meritorious, Laudable - Admirable 175. Select the most appropriate synonym of the given word. DUO (a) Loan (b) Bond (c) Pair (d) Debt SSC CGL (Tier-I) 12/04/2022 Shift-I Ans. (c): Duo (Couple), Synonym - Pair (Match) Meaning of the other optionsLoan - Debt Bond - Restriction Debt - Due 176. Select the most appropriate synonym of the given word. Decipher (a) Calculate (b) Create (c) Interpret (d) Interrogate SSC CHSL 30.05.2022 Shift-I Ans. (c): Decipher (Decode), Synonym- Interpret (Explain) Meaning of the other optionsCalculate - Estimate, Evaluate Create - Build Interpret - Inquire, Investigate 177. Select the most appropriate synonym of the given word. Disrupt (a) Arrange (b) Injure (c) Organise (d) Breach SSC CGL (Tier-I) 13/04/2022 Shift-II Ans. (d): Disrupt (Interrupt), Synonym- Breach (Violation) Meaning of the other optionsArrange - Organize Injure - Hurt Organise - Manage 178. Select the most appropriate synonym of the given word. Disseminate (a) Facilitate (b) Merge (c) Circulate (d) Implement SSC CHSL 27.05.2022 Shift-I Ans. (c): Disseminate (Expand), Synonym- Circulate (Spread) Meaning of the other optionsFacilitate - Ease, Smooth Merge - Meet, Sink Implement - Execute, Apply Synonyms

179. Select the most appropriate synonym of the given word. Disgust (a) Dislike (b) Desire (c) Danger (d) Doubt SSC Constable GD-03/12/2021 Shift-II Ans. (a): Disgust (Hate), Synonym - Dislike (Aversion) Meaning of the other optionsDesire – Aspiration, Ambition Danger – Peril Doubt – Suspicion 180. Select the most appropriate synonym of the given word. Defy (a) Destroy (b) Abide (c) React (d) Disobey SSC Constable GD-17/11/2021 Shift-II Ans. (d): Defy (Neglect, Ignore), Synonym - Disobey (Contravene) Meaning of the other optionsDestroy - Dissipate Abide - Adhere React - Respond 181. Select the most appropriate synonym of the given word. Deficient (a) Challenging (b) Bold (c) Difficult (d) Lacking SSC Stenographer (Grade C & D) 11.11.2021 Shift-II Ans. (d): Deficient (Lacking, Meager), SynonymLacking (Inadequate) Meaning of the other optionsChallenging - Difficult Bold - Daring, Brave Difficult - Arduous 182. Select the most appropriate synonym of the given word. Deliberate (a) Disastrous (b) Unplanned (c) Planned (d) Accidental SSC Constable GD-23/11/2021 Shift-III Ans. (c): Deliberate (Intentional), Synonym - Planned (Willful) Meaning of the other optionsDisastrous - Catastrophic Unplanned - Unexpected, Accidental Accidental - Circumstantial, By chance 183. Select the most appropriate synonym of the given word. Destroy (a) Demolish (b) Construct (c) Rehabilitate (d) Detach SSC Constable GD-22/11/2021 Shift-II Ans. (a): Destroy (Decimate), Synonym - Demolish (Devastate) Meaning of the other optionsConstruct – Build Rehabilitate – Regenerate, Reconstruct Detach – Segregate

392

YCT

184. Select the most appropriate synonym of the given word. DEFER (a) Postpone (b) Match (c) Balance (d) Conflict SSC Constable GD-18/11/2021 Shift-I Ans. (a): Defer (Avoid/ Postpone), Synonym- Postpone (Delay) Meaning of the other optionsMatch – Competition, Contest Balance – Equilibrium, Harmony Conflict – Strife, Collision 185. Select the most appropriate synonym of the given word. Diffidence (a) Confidence (b) Boldness (c) Meekness (d) Courage SSC CHSL (Tier-I) –15/04/2021 (Shift-I) Ans. (c): Diffidence - Synonym, Meekness Meaning of other words– Courage - Bold, Confidence - Assuredness, Boldness - Spunk 186. Select the most appropriate synonym of the given word. DELEGATE (a) possess (b) nominate (c) create (d) detain SSC Stenographer (Grade C & D) 15.11.2021 Shift-II Ans. (b): Delegate (Representative), SynonymNominate (Designate) Meaning of the other optionsPossess - Own, Hold Create - Build Detain - Stop, Inhibit 187. Select the most appropriate synonym of the given word. Deficiency (a) Surplus (b) Excess (c) Shortage (d) Success SSC Stenographer (Grade C & D) 15.11.2021 Shift-II Ans. (c): Deficiency (Lack), Synonym - Shortage (Scarcity) Meaning of the other optionsSurplus - Extra, Additional Excess - Superfluous Success - Achievement 188. Select the most appropriate synonym of the given word. Detrimental (a) Harmful (b) Friendly (c) Smart (d) Intellectual SSC CGL (Tier-I) –20/08/2021 (Shift-I) Ans. (a): Detrimental - Synonym, Harmful Meaning of other words– Friendly - Amicable, Smart - Clever, Intellectual Mental Synonyms

189. Select the most appropriate synonym of the given word. DECEIVE (a) cheat (b) withdraw (c) provide (d) guide SSC Stenographer Grade C&D –05/02/2019 (3:5pm)

Ans. (a): 'Deceive' - Synonym – Cheat Meaning of other words– Provide - Endow Withdraw - Recant Guide - Leader 190. Select the most appropriate synonym of the given word. DISCONTENT (a) disobedience (b) dissatisfaction (c) dismissal (d) disconnect SSC Sel. Post Phase-VIII (G.L.) 09.11.2020 (Shift-II)

Ans. (b): Discontent - Synonym - Dissatisfaction Meaning of other words– Disobedience - Defiance Dismissal – Removal Disconnect - Separate 191. Select the most appropriate synonym of the given word. DISPUTE (a) harmony (b) peace (c) quarrel (d) agreement SSC Sel. Post Phase-VIII (M.L.) 09.11.2020 (Shift-III)

Ans. (c): Dispute - Synonym – Quarrel Meaning of other words– Harmony - Unison Peace - Serenity Agreement- Contract 192. Select the most appropriate synonym of the given word. DESOLATE (a) protected (b) bleak (c) inhabited (d) cultivated SSC CPO-SI – 25/11/2020 (Shift-I) Ans. (b): Desolate - Synonym – Bleak Meaning of other words– Protected - Conserved, Inhabited - Habitable, Cultivated – Ploughed 193. Select the most appropriate synonym of the given word. DESPISE (a) adore (b) admire (c) abhor (d) approve SSC CPO-SI – 24/11/2020 (Shift-I) Ans. (c): 'Despise'- Synonym - 'Abhor' Meaning of other words– Adore - Greet, Admire - Praise, Approve - Favor 194. Select the most appropriate synonym of the given word. DEVOUT (a) revered (b) loyal (c) pious (d) respectable SSC CGL (Tier-I) – 12/06/2019 (Shift-I) Ans. (c): Devout- Synonym - ‘Pious’ Meaning of other words– Revered Respectable Loyal Faithful Respectable Revered

393

YCT

195. Select the most appropriate synonym of the given word. DEFER (a) delay (b) despair (c) dictate (d) dread SSC CGL (Tier-I) – 11/06/2019 (Shift-I) Ans. (a): Defer - Synonym - ‘Delay’ Meaning of other words– Despair- Despond, Dictate- Fescue, Dread- Fear 196. Select the synonym of the given word. DUBIOUS (a) certain (b) fishy (c) loyal (d) steady SSC CGL (Tier-I) – 06/06/2019 (Shift-II) Ans. (b): Dubious - Synonym - 'Fishy' Meaning of other words– Loyal Faithful Steady Freeze Certain Sure 197. Select the synonym of the given word. DRAG (a) rush (b) pull (c) push (d) rest SSC CGL (Tier-I) – 06/06/2019 (Shift-III) Ans. (b): Drag - Synonym - 'Pull' Meaning of other words– Rest Comfort Rush Haste Push Shake 198. Select the most appropriate synonym of the given word. DISCERN (a) Daunt (b) Perceive (c) Neglect (d) Disregard SSC CPO-SI – 11/12/2019 (Shift-II) Ans. (b): Discern - Synonym – Perceive Meaning of other words– Daunt - Threaten, Neglect- Disregard, Disregard Dishonor 199. Select the most appropriate synonym of the given word. DISASTROUS (a) noisy (b) wicked (c) calamitous (d) impressive SSC CPO-SI – 13/12/2019 (Shift-II) Ans.(c): Disastrous - Synonym – Calamitous Meaning of other words– Noisy - Clamor, Wicked - Vicious, Impressive - Influential 200. Select the most appropriate synonym of the given word. DEFERENCE (a) Obeisance (b) Ignorance (c) Contrast (d) Delay SSC CPO-SI–13/12/2019 (Shift-I) Ans. (a): Deference- Synonym - 'Obeisance' Meaning of other words– Ignorance - Bullhead Contrast - Resist Delay - Lateness Synonyms

201. Select the most appropriate synonym of the given word. Dedication (a) Contentment (b) Trepidation (c) Determination (d) Commitment SSC CHSL (Tier-I) –11/07/2019 (Shift-I) Ans. (d): Dedication- Synonym – Commitment Meaning of other words– Contentment- Satisfaction, Trepidation- Anxiety, Determination- Fixation 203. Select the most appropriate synonym of the given word. DISCOVER (a) neglect (b) find (c) hide (d) ignore SSC CHSL (Tier-I) –08/07/2019 (Shift-III) Ans. (b): Discover- Synonym-'Find' Meaning of other words– Neglect Dishonor Hide Conceal Ignore Abandon 204. Select the most appropriate synonym of the given word. DEFIANCE (a) deference (b) assistance (c) resistance (d) pretence SSC CHSL (Tier-I) –05/07/2019 (Shift-II) Ans. (c) Defiance- Synonym -'Resistance' Meaning of other words– Deference - Respect Assistance - Help Pretence - Splash 205. Select the most appropriate synonym of the given word. DEROGATORY (a) Favourable (b) Appreciative (c) Sarcastic (d) Lethargic SSC CHSL (Tier-I) 16/10/2020 (Shift-I) Ans. (c): 'Derogatory' - Synonym - 'Sarcastic' Meaning of other words– Favourable – Suited, Appreciative - Applaud, Lethargic - Drowsy 206. Select the most appropriate synonym of the given word. DETERIORATE (a) Enliven (b) Worsen (c) Fasten (d) Improve SSC CHSL (Tier-I) 26/10/2020 (Shift-II) Ans. (b): Deteriorate – Synonym – Worsen Meaning of other words– Enliven – Provoke Fasten – Nail Improve – Repair 207. Select the most appropriate synonym of the given word. DESPONDENT (a) Energetic (b) Neutral (c) Cheerful (d) Depressed SSC CHSL (Tier-I) 26/10/2020 (Shift-II) Ans. (d): Despondent- Synonym -‘Depressed Meaning of other words– Energetic – Powerful Neutral – Indifferent Cheerful – Glad

394

YCT

208. Select the most appropriate synonym of the given word. Diligent (a) Inattentive (b) Lethargic (c) Weary (d) Untiring SSC CHSL (Tier-I) 12/10/2020 (Shift-III) Ans. (d): 'Diligent'- Synonym - 'Untiring' Meaning of other words– Inattentive - Heedless Lethargic - Sluggish Weary - Languid 209. Select the most appropriate of the given word. Delinquent (a) Rewarding (b) Despairing (c) Charming (d) Offending SSC CHSL (Tier-I) 20/10/2020 (Shift-III) Ans. (d): 'Delinquent'- Synonym - 'Offending' Meaning of other words– Rewarding - Profitable, Despairing - Despond, Charming - Glamorous 210. Select the most appropriate synonym of the given word. DEPLETE (a) Reduce (b) Complete (c) Raise (d) Increase SSC CHSL (Tier-I) 16/10/2020 (Shift-I) Ans. (a): 'Deplete' - Synonym - 'Reduce' Meaning of other words– Complete - Total, Raise - Uplift, Increase - Rise 211. Select the most appropriate synonym of the given word : Drowsy (a) Lively (b) Alert (c) Sleepy (d) Active SSC MTS – 22/08/2019 (Shift-III) Ans. (c): 'Drowsy' Synonym -'Sleepy' Meaning of other words– Lively - Fancy Active - Alive Alert - Alarm 212. Select the synonym of the given word. Dedicate (a) Decide (b) Design (c) Denounce (d) Devote SSC MTS – 08/08/2019 (Shift-III) Ans. (d): Dedicate- Synonym – Devote Meaning of other words– Decide - Judge Design - Creation Denounce-Impute 213. Select the most appropriate synonym of the given word. Demolish (a) Repair (b) Taunt (c) Reveal (d) Dismantle SSC MTS – 02/08/2019 (Shift-III) Ans. (d): 'Demolish'- Synonym –Dismantle Meaning of other words– Repair - Mending, Taunt - Ridicule, Reveal - Evince 214. Select the most appropriate synonym of the given word. Deepen (a) Soothe (b) Neutralize (c) Relieve (d) Intensify SSC MTS – 02/08/2019 (Shift-I) Synonyms

Ans. (d) Deepen - Synonym – Intensify Meaning of other words– Soothe - Calm Neutralize - Vitiate Relieve - Help 215. Select the most appropriate synonym of the given word. DESTROY (a) Assist (b) Preserve (c) Restore (d) Ruin SSC MTS – 19/08/2019 (Shift-III) Ans. (d): Destroy - Synonym – Ruin Meaning of other words– Assist - Help Preserve - Protect Restore - Return 216. Select the most appropriate synonym of the given word. Detain (a) Delay (b) Advance (c) Permit (d) Allow SSC MTS – 13/08/2019 (Shift-I) Ans. (a): Detain- Synonym- Delay Meaning of other words– Advance - Promoted Permit - Assent Allow - Permit 217. Select the most appropriate synonym of the given word. DUBIOUS (a) Certain (b) Doubtful (d) Sure (c) Definite SSC MTS – 16/08/2019 (Shift-III) Ans. (b): Dubious - Synonym – Doubtful Meaning of other words– Certain - Sure, Definite - Obvious, Sure - fixed

E 218. Select the most appropriate synonym of the given word. Enervate (a) Devitalize (b) Arouse (c) Boost (d) Invigorate SSC MTS 19/05/2023 (Shift-III) Ans. (a) : The most appropriate synonym of the above word ‘Enervate’ is ‘Devitalize’. Meaning of other options – Arouse – Wake up, Boost – Stimulus, Invigorate – Fortify, Thus, the correct option is (a). 219. Select the most appropriate synonym of the given word. Esteem (a) Excuse (b) Abuse (c) Ridicule (d) Respect SSC GD 31/01/2023 (Shift-II)

395

YCT

Ans. (d) : Esteem (Honour), Synonym- Respect (Deference). Meaning of the other optionsExcuse - Pardon, Pretex Abuse - Rebuke, Misconduct Ridicule - Derision 220. Select the most appropriate synonym of the given word. Eligible (a) Efficient (b) Qualified (c) Sufficient (d) Disinterested SSC GD 23/01/2023 (Shift-IV) Ans. (b) : Eligible (Capable), Synonym - Qualified (Deserving) Meaning of the other optionsEfficient - Skillful, Adept Sufficient - Adequate, ample Disinterested - Indifferent 221. Select the most appropriate synonym of the underlined word in the given sentence. No one can exterminate poverty from this world. (a) Plant (b) endure (c) Secure (d) Eradicate SSC GD 16/01/2023 (Shift-III) Ans. (d) : Exterminate (Osstruct), Synonym- Eradicate (Abolist, root out) Meaning of the other optionsPlant Tree, factory Endure Suffer, bear with Secure Safe 222. Select the most appropriate synonym of the underlined word in the following sentence. They put in their sincere efforts and were lauded for their endeavours. (a) drop (b) efforts (c) passivity (d) fun SSC GD 13/02/2023 (Shift-I) Ans. (b) : 'Effort' (attempt) is the most appropriate synonym of the underlined word 'endeavors'. The meaning of other words is– Drop :- fall off. Passivity :- the trait of remaining inactive. Fun :- merriment, playfulness. 223. Select the most appropriate synonym of the given word. Extinct (a) Vanished (b) Living (c) Active (d) Vibrant SSC CGL (Tier-I) 25/07/2023 (Shift-IV) Ans. (a) : The appropriate synonym of the above given word 'Extinct' is 'Vanished'. Meaning of other options Living - Existing, Alive, Active - Deedful, Effectual, Vibrant - Doddering, Quaky Synonyms

224. Select the most appropriate synonym of the underlined word. The committee wanted us to explain this in detail. (a) Collect (b) Elaborate (c) Keen (d) Keen SSC CGL (Tier-I) 18/07/2023 (Shift-III) Ans. (b) : The most appropriate synonym of the above underlined word 'explain' is 'elaborate'. Meaning of other options Collect - Gather, Hoard, Keen - Eager, Ardent, 225. Select the most appropriate synonym of the given word. Earnestly (a) Sincerely (b) Wordily (c) Verbosely (d) Ambiguously SSC CGL (Tier-I) 24/07/2023 (Shift-III) Ans. (a) : Synonym of 'Earnestly' is 'Sincerely'. Meaning of other wordsWordily - expansively Verbosely - Prolixly Ambiguously - Unclearly 226. Choose the word that means the same as the given word. 'Enigmatic' (a) Literate (b) Grumpy (c) Illiterate (d) Mysterious SSC CHSL (Tier-I) 21/03/2023 (Shift-IV) Ans. (d) : Enigmatic - Synonym - mysterious Meaning of other wordsGrumpy – Crabby Literate – Educated Illiterate – Uneducated 227. Select the most appropriate synonym of the underlined word. All branded shoes are priced extravagantly. (a) Modestly (b) Economically (c) Expensively (d) Meagerly SSC MTS-12/07/2022 Shift-I Ans. (c): Extravagantly (Luxuriously, Sumptuously), Synonym - Expensively (Costly) Meaning of the other optionsModestly - Politely Economically - Meagerly, Frugally Meagerly - Scantily 228. Select the most appropriate synonym of the underlined word. I don't know why she wears these eccentric outfits. (a) Expensive (b) Elegant (c) Bizarre (d) Lavish SSC MTS-06/07/2022 Shift-III Ans. (c): Eccentric (Whimsical, Abnormal), Synonym Bizzare (Weird, Peculiar) Meaning of the other optionsExpensive – Costly, Precious Elegant – Delightful Lavish – Extravagant, Generous

396

YCT

229. Select the most appropriate synonym of the given word. Erring (a) Blundering (b) Tidy (c) Sloppy (d) Flawless SSC MTS-15/07/2022 Shift-I Ans. (a): Erring (Gracious), Synonym – Blundering (Dropped) Meaning of the other optionsTidy – Clean Sloppy – Muddy Flawless – Faultless 230. Select the most appropriate synonym of the given word. Eventually (a) Ultimately (b) Primarily (c) Originally (d) Initially SSC CGL (Tier-I) 13/04/2022 Shift-III Ans. (a): Eventually (Finally), Synonym-Ultimately (At least) Meaning of the other optionsPrimarily - Mainly Originally - Radically Initially - In the beginning 231. Select the most appropriate synonym of the given word. Ecstasy (a) Mourning (b) Appreciation (c) Bliss (d) Fantacy SSC CHSL 27.05.2022 Shift-I Ans. (c): Ecstasy (Bliss, Zeal), Synonym- Bliss (Joy, Pleasure) Meaning of the other optionsMourning - Grief, Sorrow Appreciation - Admiration Fantasy - Imagination 232. Select the most appropriate synonym of the given word. Emblem (a) Symbol (b) Expression (c) Problem (d) Refinement SSC Stenographer (Grade C & D) 11.11.2021 Shift-I Ans. (a): Emblem (Symbol), Synonym- Symbol (Sign, Mark) Meaning of the other optionsExpression - Manifestation Problem - Trouble, Difficulty Refinement - Rectification, Purification 233. Select the most appropriate synonym of the given word. Erect (a) Topple (b) Lower (c) Raze (d) Raise SSC Constable GD-02/12/2021 Shift-I Ans. (d): Erect (Straight, Upright), Synonym - Raise (Lift, Elevate) Meaning of the other optionsTopple - Fall Lower - Lessen, Reduce Raze - Tear Synonyms

234. Select the most appropriate synonym of the given word. Enormous (a) Ordinary (b) Limited (c) Massive (d) Expensive SSC Constable GD-01/12/2021 Shift-I Ans. (c): Enormous (Huge), Synonym- Massive (Gigantic) Meaning of the other optionsOrdinary - Hackneyed, Mediocre Limited - Bounded Expensive - Precious, Valuable 235. Select the most appropriate synonym of the given word. Enjoyable (a) Delightful (b) Cloudy (c) Bright (d) Beautiful SSC Constable GD-18/11/2021 Shift-I Ans. (a): Enjoyable (Amusing, Pleasurable), Synonym Delightful (Entertaining, Interesting) Meaning of the other optionsCloudy – Dim, Hazy Bright – Radiant, Shiny Beautiful – Lovely 236. Select the most appropriate synonym of the underlined word. We expended most of our savings on this tour. (a) encashed (b) enjoyed (c) exhausted (d) earned SSC CHSL 26.05.2022 Shift-III Ans. (c): The appropriate synonym for the underlined word 'Expended' (Spend, Pay) in the above sentence will be 'Exhausted' (Weak, Feeble). Other options have different meaningsEncased - Inset Enjoyed - Delighted Earned - Acquired 237. Select the most appropriate synonym of the given word. Eternal (a) Balanced (b) Everlasting (c) Limited (d) Faithful SSC Constable GD-10/12/2021 Shift-III Ans. (b): Eternal (Endless), Synonym - Everlasting (Perpetual) Meaning of the other optionsBalanced - Fair, Unbiased Limited- Restricted Faithful- Reliable, Credible 238. Select the most appropriate synonym of the given word. EMERGE (a) Aspire (b) Appear (c) Admire (d) Acquire SSC CHSL (Tier-I) –13/04/2021 (Shift-I) Ans. (b): Emerge - Synonym - Appear Meaning of the wordsAspire- Desire Acquire - Enlist Admire - Praise

397

YCT

239. Select the most appropriate synonym of the given word : EFFICIENCY (a) Fluency (b) Capability (c) Tendency (d) Disability SSC CGL (Tier-II) 16/11/2020 (3 pm - 5 pm) Ans. (b): 'Efficiency' – Synonym – Capability Meaning of the wordsFluency- Flow Tendency- Leaning Disability- Inability 240. Select the most appropriate synonym of the given word. EMPHASIZE (a) repeat (b) draw (c) stress (d) show SSC Stenographer Grade C&D –05/02/2019 (3:5pm)

Ans. (c): 'Emphasize' – Synonym – Stress Meaning of other words– Repeat - Encore Draw - Exert Show – Display 241. In the following question, out of the given four alternatives, select the one which best expresses the meaning of the given word. Embargo (a) Barrier (b) Prescribe (c) Comfort (d) Justify SSC CGL (Phase-II) 17/02/2018 Ans. (a): Embargo – Synonym - Barrier Meaning of other words– Prescribe – Ordain Comfort – Rest Justify – Confirm 242. Select the most appropriate synonym of the given word. ENDEAVOUR (a) attempt (b) achievement (c) result (d) success SSC CGL (Tier-I) – 13/06/2019 (Shift-I) Ans. (a): Endeavour – Synonym - ‘Attempt’ Meaning of other words– Achievement – Cognition Result – Outcome Success – Pass 243. Select the most appropriate synonym of the given word. EXALT (a) challenge (b) condemn (c) praise (d) extract SSC CGL (Tier-I) – 11/06/2019 (Shift-III) Ans. (c): Exalt - Synonym - ‘Praise’ Meaning of other words– Condemn- Cencure Extract- Quote, Challenge- Defiance 244. Select the synonym of the given word. EXEMPT (a) hinder (b) exclude (c) reduce (d) prevent SSC CGL (Tier-I) – 07/06/2019 (Shift-II) Synonyms

Ans. (b) Exempt - Synonym - 'Exclude' Meaning of other words– Hinder - Stop Reduce - Detract Prevent - Inhibit 245. Select the synonym of the given word. ENTICE (b) entail (a) ensue (c) enrage (d) entrap SSC CGL (Tier-I) – 06/06/2019 (Shift-II) Ans. (d) Entice - Synonym - 'Entrap' Meaning of other words– Ensue - Pursue Enrage - Nettle Entail - Required 246. Select the synonym of the given word. ENGULF (a) encroach (b) enshrine (c) entangle (d) envelop SSC CGL (Tier-I) – 06/06/2019 (Shift-III) Ans. (d) Engulf - Synonym - 'Envelop' Meaning of other words– Encroach - Supersede Enshrine - Cherish Entangle - Confound 247. Select the most appropriate synonym of the given word. ENGULF (a) dry (b) uncover (c) neglect (d) inundate SSC CPO-SI – 09/12/2019 (Shift-II) Ans. (d) Engulf- Synonym - 'Inundate' Meaning of other words– Dry - Arid, Uncover- Uncoil, Neglect- Disregard 248. Select the most appropriate synonyms of the given word. EFFICACIOUS (a) Effective (b) Impotent (c) Voracious (d) Audacious SSC CPO-SI – 13/12/2019 (Shift-I) Ans. (a): Efficacious - Synonym – Effective Meaning of other words– Impotent - Powerless Voracious - Gourmand Audacious - Daring 249. Select the most appropriate synonym of the given word. EXASPERATE (a) Imitate (b) Desperate (c) infuriate (d) Appreciate SSC CPO-SI – 13/12/2019 (Shift-I) Ans. (c): Exasperate - Synonym – Infuriate Meaning of other words– Imitate - Simulate, Desperate - Downcast, Appreciate - Esteem 250. Select the most appropriate synonym of the given word. Enthrall (a) Disgust (b) Free (c) Mesmerize (d) Repel SSC CHSL (Tier-I) –03/07/2019 (Shift-I)

398

YCT

Ans. (c): Enthrall - Synonym- Mesmerize Meaning of other words– Disgust- Hatred Free- Liberated Repel- Setback 251. Select the most appropriate synonym of the given word. ERODE (a) fix (b) disintegrate (c) preserve (d) build SSC CHSL (Tier-I) –03/07/2019 (Shift-II) Ans. (b): Erode – Synonym – Disintegrate Meaning of other words– Fix- Put Preserve- Defend Build- Construct 252. Select the most appropriate synonym of the given word. ENTANGLE (a) exclude (b) clarify (c) implicate (d) release SSC CHSL (Tier-I) –03/07/2019 (Shift-II) Ans. (c): Entangle - Synonym- Implicate Meaning of other words– Exclude- Debar, Clarify- Explain Release- Discharge 253. Select the most appropriate synonym of the given word. EMBED (a) pull (b) bury (c) dig up (d) rise SSC CHSL (Tier-I) –02/07/2019 (Shift-III) Ans. (b) Embed - Synonyms - Bury Meaning of other words– Pull - Draw, Dig up - Grub, Rise - Increase 254. Select the most appropriate synonym of the given word. EXPOSE (a) hide (b) bury (c) reveal (d) protect SSC CHSL (Tier-I) –01/07/2019 (Shift-III) Ans. (c) Expose - Synonyms – Reveal Meaning of other words– Bury- Inter Hide- Conceal Protect- Defend 255. Select the most appropriate synonym of the given word. ENRAGE (a) Infuriate (b) Alleviate (c) Placate (d) Moderate SSC CHSL (Tier-I) 13/10/2020 (Shift-I) Ans. (a): 'Enrage' – Synonym- 'Infuriate' Meaning of other words– Alleviate – Reduce Placate – Console Moderate – Medium 256. Select the most appropriate synonym of the given word. EXPLICIT Synonyms

(a) Lucid (b) Compatible (c) Unintelligible (d) Equivocal SSC CHSL (Tier-I) 18/03/2020 (Shift-III) Ans. (a): ''Explicit''- Synonym - 'Lucid' Meaning of other words– Compatible - Relevant Unintelligible - Abstruse Equivocal - Suspected 257. Select the most appropriate synonym of the given word. Enunciate (a) Confine (b) Create (c) Articulate (d) Engage SSC CHSL (Tier-I) 26/10/2020 (Shift-I) Ans. (c): Enunciate - Synonym- Articulate Meaning of other words– Confine - Limit Create - Fabricate Engage - Infliction 258. Select the most appropriate synonym of the given word. Enormous (a) Puny (b) Tiny (c) Huge (d) Petty SSC MTS – 22/08/2019 (Shift-III) Ans. (c): 'Enormous' Synonym - 'Huge' Meaning of other words– Puny - Tiny Tiny - Lesser Petty - Flippant 259. Select the most appropriate synonym of the given word. Exceptional (a) Extraordinary (b) Usual (c) Mundane (d) Common SSC MTS – 08/08/2019 (Shift-I) Ans. (a): Exceptional- Synonym ‘Extraordinary’ Meaning of other words– Usual - General Mundane - Cosmic Common - Generic 260. Select the most appropriate synonym of the given word : Encumbrance (a) Destroy (b) Despair (c) Obstacle (d) Stimulant SSC MTS – 20/08/2019 (Shift-II) Ans. (c): Encumbrance- synonym - 'Obstacle' Meaning of other words– Destroy - Abrade Despair - Despond Stimulant - Exciting 261. Select the most appropriate synonym of the given word. Engage (a) Dismiss (b) Settle (c) Occupy (d) Release SSC MTS – 14/08/2019 (Shift-I) Ans. (c): 'Engage'- Synonym - 'Occupy' Meaning of other words– Dismiss - Depart Settle - Dwell Release - Discharge

399

YCT

262. Select the synonym of the given word. Enhance (a) Reduce (b) Frighten (c) Shrink (d) Improve SSC MTS – 08/08/2019 (Shift-II) Ans. (d): Enhance - Synonym – Improve Meaning of other words– Reduce - Detract Frighten - Startle Shrink - Hesitation 263. Select the most appropriate synonym of the given word. Eager (a) Satisfied (b) Active (c) Keen (d) Alert SSC MTS – 07/08/2019 (Shift-II) Ans. (c): Eager - Synonym Keen Meaning of other words– Satisfied - Contented Active - Deedful Alert - Warning 264. Select the synonym of the given word. ELAPSED (a) Persuaded (b) Ceased (c) Prolonged (d) Connected SSC GD – 02/03/2019 (Shift-I) Ans. (b): Elapsed - Synonym – Ceased Meaning of other words– Persuaded - Fiducial Prolonged - Lengthy Connected - Cohesive

267. Select the most appropriate synonym of the given word. Feeble (a) Unheedful (b) Weak (c) Strong (d) Baneful SSC CGL (Tier-I) 14/07/2023 (Shift-I) Ans. (b) : Synonym of the 'feeble' is 'weak'. Meaning of other word. Unheedful – Neglected Strong – Powerful Baneful – Calamitous 268. Choose the word that means of same as the given word. Faddish (a) Unpopular (b) Phony (c) Barge (d) Fashionable SSC CHSL (Tier-I) 20/03/2023 (Shift-I) Ans. (d) : Faddish - Synonym - "Fashionable" Phony- Wrong Barge- Hoy Unpopular- Unpleasant 269. Select the most appropriate synonym of the given word. FINAL (a) Allowed (b) Forbidden (c) former (d) Concluding SSC CGL (Tier-I) 18/04/2022 Shift-I Ans. (d): Final (Last)- Synonym- Concluding (End, Finish) Meaning of the other optionsAllowed - Permit Forbidden - Taboo Former - Erstwhile, Prior 265. Choose the word that means the same as the 270. Select the most appropriate synonym of the given word. given word. Fabulously FLUKE (a) Nominally (b) Fantastically (a) Contract (b) Chance (c) Scantily (d) Scarcely (c) Policy (d) Plan SSC MTS 16/05/2023 (Shift-II) SSC CGL (Tier-I) 12/04/2022 Shift-II Ans. (b) : The synonym of the given word- 'Fabulously' Ans. (b): Fluke (Coincidence), Synonym - Chance is 'Fantastically'. (Luck) Meaning of other wordsMeaning of the other optionsNominally- Ostensiblly Contract - Agreement Scantily- Meager, Scant Policy - Scheme Scarcely- Hardly, Barely Plan - Project, Idea Thus, the correct answer is option (b).

F

266. Select the most appropriate word which is nearest in meaning to the underlined word. One should follow the role models in life. (a) Emulate (b) Cherish (c) Admire (d) Reward SSC CGL (Tier-I) 19/07/2023 (Shift-IV) Ans. (a) : The most appropriate nearest meaning for the word 'follow' is 'Emulate'. Other meaning of words are Cherish means - Treasure Admire means - Praise, Applaud Reward means - Bounty, Prize, compensation Synonyms

271. Select the most appropriate synonym of the given word. Facility (a) Happiness (b) Fitting (c) Skill (d) Luck SSC Constable GD-26/11/2021 Shift-I Ans. (c): Facility (Convenience, Aptitude), Synonym Skill (Talent) Meaning of the other optionsHappiness – Pleasure Fitting – Attachment, Connection Luck – Fortune, Fate

400

YCT

272. Select the most appropriate synonym of the given word. FORTHRIGHT (a) secret (b) tricky (c) tactful (d) outspoken SSC Stenographer (Grade C & D) 12.11.2021 Shift-I Ans. (d): Forthright (Candid), Synonym-Outspoken (Explicit) Meaning of the other optionsSecret - Clandestine Tactful - Courteous Tricky - Devious, Crafty 273. Select the most appropriate synonym of the given word. Forbid (a) Close (b) Prohibit (c) Adieu (d) Encourage SSC CHSL (Tier-I) –05/08/2021 (Shift-I) Ans. (b): Forbid - Synonym - Prohibit Encourage - Stimulate Close - Nigh Adieu - Leave 274. Select the most appropriate synonym of the given word. FICTION (a) fantasy (b) fact (c) truth (d) literature SSC CPO-SI – 23/11/2020 (Shift-I) Ans. (a): Fiction - Synonym – Fantasy Fact - Reality, Truth- Sooth, Literature - Letter 275. Select the most appropriate synonym of the given word. FOUNDATION (a) base (b) top (c) structure (d) building SSC CPO-SI – 23/11/2020 (Shift-I) Ans. (a): Foundation - Synonym – Base Meaning of other words– Top - Summit, Structure- Fabrication Building- Construction 276. Select the most appropriate synonym of the given word. FLABBERGASTED (a) astonished (b) motivated (c) disturbed (d) terrified SSC CPO-SI – 25/11/2020 (Shift-II) Ans. (a): Flabbergasted- Synonym – Astonished Meaning of other words– Motivated - Propel Disturbed - Break Terrified - Horrified 278. Select the most appropriate synonym of the given word. FASTEN (a) halt (b) allow (c) affix (d) hurry SSC CPO-SI – 24/11/2020 (Shift-I) Ans. (c): 'Fasten'- Synonym - 'Affix' Meaning of other words– Halt - Stage, Allow - Let, Hurry - Haste Synonyms

279. Select the most appropriate synonym of the given word. FUSE (verb) (a) correct (b) unused (c) break (d) combine SSC CGL (Tier-I) – 13/06/2019 (Shift-III) Ans. (d) Fuse- Synonym - ‘Combine’ Meaning of other words– Correct- Right Unused- Unusable Break- Unroot 280. Select the most appropriate synonym of the given word. FURY (b) anger (a) sorrow (c) cruelty (d) fright SSC CGL (Tier-I) – 07/06/2019 (Shift-III) Ans. (b) Fury - Synonym - 'Anger' Meaning of other words– Sorrow - Pain Cruelty - Atrocity Fright - Fear 281. Select the most appropriate synonym of the given word. FASCINATING (a) appealing (b) boring (c) stupid (d) tiring SSC CPO-SI – 09/12/2019 (Shift-I) Ans. (a): Fascinating - Synonym - 'Appealing' Meaning of other words– Boring - Tedious, Stupid – Silly, Tiring - Exhausting 282. Select the most appropriate synonym of the given word. FLOURISH (a) Thrive (b) Shrivel (c) Wither (d) Languish SSC CPO-SI – 11/12/2019 (Shift-II) Ans. (a): Flourish - Synonym- Thrive Meaning of other words– Shrivel - Dwindle, Wither - Fade, Languish- Sluggard 283. Select the most appropriate synonym of the given word. FATIGUE (a) brightness (b) weariness (c) liveliness (d) freshness SSC CPO-SI – 11/12/2019 (Shift-I) SSC CPO-SI – 12/12/2019 (Shift-I) Ans. (b) Fatigue - Synonym - Weariness Meaning of other words– Brightness - Gleam Liveliness - Vividness Freshness - Recency 284. Select the most appropriate synonym of the given word. FATIGUE (a) energy (b) tiredness (c) lightness (d) warmth SSC CPO-SI – 13/12/2019 (Shift-II) Ans. (b): Fatigue – Synonym – Tiredness Meaning of other words– Energy - Power Lightness - Glitter Warmth - Ardency

401

YCT

285. Select the most appropriate synonym of the given word. FEIGN (a) Foretell (b) Pretend (c) Hinder (d) Instruct SSC CPO-SI – 13/12/2019 (Shift-I) Ans. (b): Feign- Synonym – Pretend Meaning of other words– Foretell - Prophecy Hinder - Stop Instruct - Read 286. Select the most appropriate synonym of the given word. 'FLEETING' (a) lasting (b) ready (c) momentary (d) complete SSC CHSL (Tier-I) –09/07/2019 (Shift-III) Ans. (c) Fleeting- Synonym- 'Momentary' Meaning of other words– Lasting - Stable Ready - Ripe Complete - Finished 287. Select the most appropriate synonym of the given word. Fortify (a) Secure (b) Harm (c) Loosen (d) Neglect SSC CHSL (Tier-I) –03/07/2019 (Shift-I) Ans. (a): Fortify- Synonym- Secure Meaning of other words– Harm- Loss Loosen- Slacken Neglect- Slight 888. Select the most appropriate synonym of the given word: Fraudulent (a) Repellent (b) Deceitful (c) Prevalent (d) Genuine SSC CHSL (Tier-I) 21/10/2020 (Shift-I) Ans. (b): Fraudulent - Synonym – Deceitful Meaning of other words– Repellent – Hateful Prevalent – Current Genuine – Real 289. Select most appropriate synonym of the given word. FURTIVE (a) Positive (b) Secretive (c) Adhesive (d) Abortive SSC CHSL (Tier-I) 12/10/2020 (Shift-II) Ans. (b): Furtive – Synonym - 'Secretive' Meaning of other words– Positive – Cogent Adhesive –Glue Abortive – Useless 290. Select the most appropriate synonym of the given word. FOLIAGE (a) Greenery (b) Firmness (c) Urgency (d) Foolishness SSC CHSL (Tier-I) 13/10/2020 (Shift-II) Synonyms

Ans. (a): Foliage - Synonym – Greenery Meaning of other words– Firmness - Stability Urgency - Longing Foolishness - Stupidity 291. Select the most appropriate synonym of the given word. FESTIVITY (a) Celebration (b) Foundation (c) Calibration (d) Botheration SSC CHSL (Tier-I) 19/03/2020 (Shift-III) Ans. (a): Festivity- Synonym – Celebration Meaning of other words– Foundation – Base, Calibration – Updo, Botheration – Anger 292. In the following question, out of the given four alternatives, select the one which best expresses the meaning of the given word. Fickle. (a) Cognizant (b) Unstable (c) Accidental (d) Steadfast SSC MTS 9-10-2017 (Shift-I) Ans. (b) Fickle - Synonym- Unstable Meaning of other words– Cognizant - Aware, Accidental - Casual, Steadfast Firm 293. Select the most appropriate synonym of the given word. Forgive (a) Forget (b) Accuse (c) Pardon (d) Compel SSC MTS – 07/08/2019 (Shift-I) Ans. (c): Forgive- Synonym - 'Pardon' Meaning of other words– Forget - Delude Accuse - Denounce Compel - Coerce 294. Select the most appropriate synonym of the given word. Former (a) Inferior (b) superior (c) Regular (d) Previous SSC MTS – 05/08/2019 (Shift-I) Ans. (d): 'Former'- Synonym - 'Previous' Meaning of other words– Inferior - Junior Superior - Better Regular - Steady 295. Select the most appropriate synonym of the given word. FEROCIOUS (a) Gentle (b) Domesticated (c) Tame (d) Fierce SSC MTS – 16/08/2019 (Shift-II) Ans. (d): 'Ferocious' - Synonym - 'Fierce' Meaning of other words– Gentle - Noble Domesticated - Retinue Tame - Pet 296. Select the most appropriate synonym of the given word. Fundamental (a) Trivial (b) Extra (c) secondary (d) Basic SSC MTS – 13/08/2019 (Shift-I)

402

YCT

Ans. (d): Fundamental- Synonym – Basic Meaning of other words– Trivial - Niggard Extra - Spare Secondary - Subsidiary 297. Select the most appropriate synonym of the given word. FRUITFUL (a) Useless (b) productive (c) Barren (d) Juicy SSC GD – 11/02/2019 (Shift-III) Ans. (b): Fruitful - Synonym - Productive Meaning of other words– Useless - Idle, Barren - Infertile, Juicy - Succulent 298. Select the most appropriate synonym of the given word. Foyer (a) Kitchen (b) Verandah (c) Bedroom (d) Lobby SSC GD – 11/02/2019 (Shift-III) Ans. (d): Foyer - Synonym – Lobby Meaning of other words– Kitchen - Cook house Verandah - Parch Bedroom - Dormitory

G 299. Select the most appropriate synonym of the given word. Glorify (a) Strengthen (b) Stupefy (c) Simplify (d) Praise SSC CGL (Tier-I) 25/07/2023 (Shift-IV) Ans. (d) : The most appropriate synonym of the given word - 'Glorify' is 'praise.' Meaning of other optionsStrengthen - Sanction, Torrefy, Stupefy - stultify, Astound, Simplify - clarify, illuminate, Thus, the correct answer is option (d). 300. Select the most appropriate synonym of the given word. Gloomy (a) Deperssing (b) Comfortable (c) Bright (d) Intrepid SSC MTS-07/07/2022 Shift-II Ans. (a): Gloomy (Melancholy), Synonym - Depressing (Disappointing) Meaning of the other optionsComfortable – Convenient Bright – Shiny, Glossy Intrepid – Bold, Dauntless 301. Select the most appropriates synonym of the underlined word. Medicines should be ordered by the generic name, not by the proprietary or trade (a) Exclusive (b) General (c) Individual (d) Specific SSC MTS-08/07/2022 Shift-I Synonyms

Ans. (b): Generic (Usual), Synonym - General (Common) Meaning of the other optionsExclusive – Particular Individual – Personal Specific – Special 302. Select the most appropriate synonym of the given word. GRAB (a) lose (b) seize (c) free (d) offer SSC Stenographer (Grade C & D) 15.11.2021 Shift-I Ans. (b): Grab (Capture, Grasp), Synonym- Seize (Hold, Catch) Meaning of the other optionsFree - Release Lose - Miss Offer - Proposal 303. Select the most appropriate synonym of the given word. Generic (a) Definite (b) Specific (c) Precise (d) Universal SSC CGL (Tier-I) –13/08/2021 (Shift-I) Ans. (d): Generic - Synonym - Universal Meaning of other words– Definite - Certain, Specific - Distinct, Precise - Good 304. In the following question, out of the given four alternatives, select the one which best expresses the meaning of the given word. Guzzle (a) Sip (b) Imbibe (c) Starve (d) Release SSC CGL (Phase-II) 17/02/2018 Ans. (b) Guzzle – Synonym - Imbibe Meaning of other words– Sip – Neck Starve – Hungry Release – Acquittal 305. Select the most appropriate synonym of the given word. GRADUAL (a) abrupt (b) uneven (c) continuous (d) hasty SSC CPO-SI – 25/11/2020 (Shift-II) Ans. (c): Gradual - Synonym- Continuous Meaning of other words– Abrupt - Sudden, Uneven - Random, Hasty - Eager 306. Select the most appropriate synonym of the given word. GAUDY (a) proper (b) simple (c) flashy (d) plain SSC CPO-SI – 24/11/2020 (Shift-I) Ans. (c): 'Gaudy'- Synonym - 'Flashy' Meaning of other words– Proper - Advisable Simple - Easy Plain - Decent

403

YCT

307. Select the synonym of the given word. GARRULOUS (a) Talkative (b) Concise (c) Throaty (d) Guttural SSC CGL (Tier-I) – 04/06/2019 (Shift-II) Ans. (a): Garrulous - Synonym - 'Talktive' Meaning of other words– Concise- Brief Throaty- Husky Guttural- Jugular 308. Select the most appropriate synonym of the given word. GLUM (a) Afraid (b) Dismal (c) Bright (d) Nervous SSC CHSL (Tier-I) 19/10/2020 (Shift-III) Ans. (b): Glum – Synonym – Dismal Meaning of other words– Afraid– Fearful, Bright – Radiant, Nervous– Restless 309. Select the most appropriate synonym of the given word. GUILE (a) Intuition (b) Surfeit (c) Deceit (d) Sincerity SSC CHSL (Tier-I) 12/10/2020 (Shift-I) Ans. (c): Guile - Synonym - 'Deceit' Meaning of other words– Intuition - Instinct Surfeit - Excess Sincerity - Reality

H 310. Select the most appropriate synonym of the given word. Humble (a) Bold (b) Assertive (c) Modest (d) Inactive SSC MTS-05/07/2022 Shift-II Ans. (c): Humble (Modest), Synonym – Modest (Submissive) Meaning of the other optionsBold – Courageous Assertive – Confident, Aggressive Inactive – Dormant 311. Select the most appropriate synonym of the given word. Hasty (a) Expeditious (b) Restricted (c) Laidback (d) Locked SSC MTS-05/07/2022 Shift-I Ans. (a): Hasty (Impatient), Synonym - Expeditions (Quickness) Meaning of the other optionsRestricted – Banned Laidback – Relaxed, Calm Locked – Closed, Sealed Synonyms

312. Select the most appropriate synonym of the given word. Haste (a) Hurry (b) Delay (c) Leisure (d) Rest SSC Stenographer (Grade C & D) 15.11.2021 Shift-II Ans. (a): Haste (Hurry, Rush), Synonym - Hurry (Quick) Meaning of the other optionsDelay - Procrastination Leisure - Vacation, Rest Rest - Ease, Comfort 313. Select the most appropriate synonym of the given word. HARMONY (a) unease (b) peace (c) calmness (d) success SSC Stenographer (Grade C & D) 12.11.2021 Shift-II Ans. (b): Harmony (Unison, Adjustment), SynonymPeace (Silence) Meaning of the other optionsUnease - Discomfort Calmness - Serenity, Stillness Success - Victory, Triumph 314. Select the most appropriate synonym of the given word. HOSPITABLE (a) unsocial (b) stingy (c) hostile (d) congenial SSC Stenographer (Grade C & D) 11.11.2021 Shift-II Ans. (d): Hospitable (Welcoming, Cordial), SynonymCongenial (Compatible) Meaning of the other wordsUnsocial - Unfriendly Stingy - Miser Hostile - Enemy, Antagonistic 315. Select the most appropriate synonym of the given word. HUGE (a) big (b) embrace (c) far (d) small SSC Stenographer (Grade C & D) 11.11.2021 Shift-I Ans. (a): Huge (Massive), Synonym- Big (Enormous) Meaning of the other optionsEmbrace - Cuddle, Hug Far - Afar Small - Little 316. Select the most appropriate synonym of the given word. H AMPER (a) restrict (b) absolve (c) adhere (d) punish SSC Stenographer (Grade C & D) 12.11.2021 Shift-I Ans. (a): Hamper (Hurdle), Synonym- Restrict (Inhibit) Meaning of the other wordsAbsolve - Adversary Adhere - Soothsayer Punish - Inflict

404

YCT

317.

In the following question, out of the given four alternatives, select the one which best expresses the meaning of the given word. Hound (a) Critic (b) Migrate (c) Beagle (d) Variable SSC CGL (Phase-II) 17/02/2018 Ans. (c) Hound – Synonym – Beagle Meaning of other words– Critic – Baleful Migrate – Emigrate Variable – Mutable 318. Select the most appropriate synonyms of the given word. HOSTILITY (a) sympathy (b) friendship (c) goodwill (d) enmity SSC CGL (Tier-I) – 13/06/2019 (Shift-II) Ans. (d): Hostility – Synonym - ‘Enmity’ Meaning of other words– Sympathy- Empathy Friendship- Intimacy Good-will- Gracious 319. Select the most appropriate synonym of the given word. HIND (a) near (b) rear (c) first (d) front SSC CHSL (Tier-I) –02/07/2019 (Shift-I) Ans. (b): Hind - Synonym- 'Rear' Meaning of other words– Near- Nigh First- Primarily Front- Afore 320. Select the most appropriate synonym of the given word: HAUGHTY (a) Humble (b) Talkative (c) Arrogant (d) Quiet SSC CHSL (Tier-I) 21/10/2020 (Shift-III) Ans. (c): 'Haughty'- Synonym - 'Arrogant' Meaning of other words– Humble - Meek, Talkative - Chatty, Quiet - Calm 321. Select the most appropriate synonym of the given word. HOLISTIC (a) dissimilar (b) superficial (c) sketchy (d) comprehensive SSC CHSL (Tier-I) 15/10/2020 (Shift-I) Ans. (d): 'Holistic'- Synonym – Comprehensive Meaning of other words– Dissimilar - Different, Superficial - Shallow, Sketchy - Partial 322. Select the most appropriate synonym of the given word. Harass (a) Molest (b) Assist (c) Comfort (d) Scare SSC MTS – 06/08/2019 (Shift-II) Ans. (a): Harass - Synonym – Molest Meaning of other words– Assist- Help, Comfort- Rest, Scare- Frighten Synonyms

323. Select the synonym of the given word. Hate (a) Deny (b) Defer (c) Detest (d) Defile SSC MTS – 08/08/2019 (Shift-II) Ans. (c): 'Hate'- Synonym – Detest Meaning of other words– Deny - Negate Defer - Avoid Defile- Profane 324. Select the most appropriate synonym of the given word. HUMDRUM (a) Synchronic (b) Musical (c) Monotonous (d) Noisy SSC MTS – 20/08/2019 (Shift-I) Ans. (c): Humdrum - Synonym -'Monotonous' Meaning of other words– Synchronic - Simultaneous Noise - Clamor Musical - Interlude 325. Select the synonym of the given word. HANDY (a) Convenient (b) Easy (c) Funny (d) Nice SSC CHSL (Tier-I) –03/07/2019 (Shift-III) SSC GD – 18/02/2019 (Shift-III) Ans. (a): Handy - Synonym – Convenient Meaning of other words– Easy - Simple Nice - Lovely Funny - Queer

I 326. Choose the word that means the same as the given word. Impede (a) Stalwart (b) Stampede (c) Sore (d) Thwart SSC MTS 08/05/2023 (Shift-III) Ans. (d) : The most appropriate Similar meaning for the given word 'Impede' is 'Thwart' (Transversal). Meaning for the other words– Stalwart means – loyal Stampede means – Rush, Haste Sore means – Bitter These, the correct answer is option (d) 327. Select the most appropriate synonym of the given word. Introvert (a) Bold (b) Shy (c) Forward (d) Outgoing SSC GD 13/02/2023 (Shift-I) Ans. (b) : 'Shy' is the most appropriate synonym of the given word– 'Introvert' (turn inside). The meaning of other words is– Bold :- Adventurous, fearless. Forward :- ahead Outgoing :- extrovert.

405

YCT

328. Select the most appropriate synonym of the underlined word. The outcome document of the Rio+20 Conference, the Future We Want, underscores climate change as “an inevitable and urgent global challenge with long-term implications for the sustainable development of all countries”. Through the document, Member States express their concern about the continuous rising of emissions of greenhouse gases and the vulnerability of all countries, particularly developing countries, to the adverse impacts of climate change. (a) escapable (b) immemorable (c) undeniable (d) adventurous SSC CGL (Tier-I) 18/07/2023 (Shift-III) Ans. (c) : The most appropriate synonym of the above underlined word 'inevitable' is 'undeniable'. Meaning of other optionsEscapable - Unnecessary, Immemorable - Unremarkable, Adventurous - Venturesome, 329. Select the most appropriate option that can substitute the underlined segment in the given sentence. We must remember that what we teach our children is what we inculcate in them. (a) endanger (b) import (c) inspire (d) instill SSC CGL (Tier-I) 14/07/2023 (Shift-I) Ans. (d) : In the above sentence most appropriate option of underlined word 'inculcate' is 'instill'. Hence option (d) will be correct answer. 330. Choose the word that means the same as the given word. INSTIGATE (a) Deterrence (b) Settle (c) Raise (d) Complaisance SSC CHSL (Tier-I) 10/03/2023 (Shift-III) Ans. (c) : Instigate - Synonym - Raise Complaisance – Politeness Deterrence – Redressal Settle – Pause/Stop 331. Select the most appropriate synonym of the given word. Intricate (a) Basic (b) Complicated (c) Painted (d) Developed SSC MTS-12/07/2022 Shift-I Ans. (b): Intricate (Complex), Synonym - Complicated (Confusing, Baffling) Meaning of the other optionsBasic – Fundamental Painted – Tinted Developed – Advanced 332. Select the most appropriate synonym of the given word. Imminent (a) Impending (b) Ignorant (c) Distant (d) Unyielding SSC CGL (Tier-I) 11/04/2022 Shift-III Synonyms

Ans. (a): Imminent (Adjacent), Synonym- Impending (Proximate) Meaning of the other optionsIgnorant - Unaware, Unwise Distant - Afar Unyielding - Firm, Obstinate 333. Select the most appropriate synonym of the given word. Inscrutable (a) Inevitable (b) Indelible (c) Inexplicable (d) Incredible SSC CGL (Tier-I) 12/04/2022 Shift-II Ans. (c): Inscrutable (Mysterious), Synonym Inexplicable (Inexpressible) Meaning of the other optionsInevitable - Indispensable Indelible - Ineradicable Incredible - Unbelievable 334. Select the most appropriate synonym of the highlighted word He is inclined to stay with the present organisation. (a) declined (b) motivated (c) struggled (d) mystified SSC CHSL 26.05.2022 Shift-II Ans. (b): Inclined (Sloping, Willing), SynonymMotivated (Inspire) Meaning of the other optionsDeclined – Reject, Refuse Struggled – Clash Mystified – Baffled, perplexed 335. Select the most appropriate synonym of the given word. ISOLATED (a) Open (b) Mingling (c) Secluded (d) Included SSC CHSL (Tier-I) –13/04/2021 (Shift-I) Ans. (c): Isolated - Synonym, Secluded Meaning of other words– Included - Present, Mingling - Conspire, Open - Naked 336. Select the most appropriate synonym of the given word. Irregular (a) Smooth (b) Common (c) Uneven (d) Usual SSC Constable GD-06/12/2021 Shift-III Ans. (c): Irregular (Intermittent, Random), Synonym – Uneven (Dissimilar, Unlike) Meaning of the other optionsSmooth – Sluk Common – General Usual – Normal, Regular 337. Select the most appropriate synonym of the given word. Investigate (a) Supervise (b) Relate (c) Examine (d) Calculate SSC Constable GD-08/12/2021 Shift-I

406

YCT

Ans. (c): Investigate (Interrogate), Synonym - Examine (Inspect) Meaning of the other optionsSupervise - Inspect, Administer Relate - Connect, Associate Calculate - Estimate, Evaluate 338. Select the most appropriate synonym of the given word. Immerse (a) Immediate (b) Submerge (c) Submission (d) Immoral SSC Constable GD-25/11/2021 Shift-III Ans. (b): Immerse (Dip), Synonym – Submerge (Overflow, Flood) Immediate – Instant Submission – Courtesy, Humility Immoral – Unethical 339. Select the most appropriate synonym of the given word. Integrate (a) Assimilate (b) Disburse (c) Distribute (d) Separate SSC CGL (Tier-I) –24/08/2021 (Shift-I) Ans. (a): Integrate - Synonym - Assimilate Meaning of other words– Disburse - Repay, Distribute - Dispense, Separate Apportion 340. Select the most appropriate synonym of the given word. INTRIGUING (a) monotonous (b) investigating (c) simultaneous (d) gripping SSC CPO-SI – 24/11/2020 (Shift-II) Ans. (d): Intriguing - Synonym – Gripping Meaning of other words– Monotonous - Prosaic Investigating - Examine Simultaneous - Synchronic 341. Select the most appropriate synonym of the given word. IMITATION (a) fake (b) genuine (c) real (d) original SSC CPO-SI – 23/11/2020 (Shift-I) Ans. (a): Imitation – Synonym - 'Fake' Meaning of other words– Genuine – Real Real – Actual Original – Pure 342. Select the most appropriate synonym of the given word. INSULT (a) apply (b) offend (c) offer (d) remove SSC CGL (Tier-I) – 13/06/2019 (Shift-II) Ans. (b): Insult - Synonym -‘Offend’ Meaning of other words– Apply- Put Offer- Proposal Remove- Isolation Synonyms

343. Select the most appropriate synonym of the given word. INTRICATE (a) Connected (b) Complete (c) Complex (d) Colorful SSC CGL (Tier-I) – 12/06/2019 (Shift-II) Ans. (c) Intricate – Synonym - ‘Complex’ Meaning of other words– Connected - Fitted, Complete - Finished, Colorful Shiny 344. Select the most appropriate synonym of the given word. INITIATE (a) show (b) start (c) sign (d) slow SSC CGL (Tier-I) – 12/06/2019 (Shift-III) Ans. (b): Initiate - Synonym - ‘Start’ Meaning of other words– Show- Display Sign- Singnal Slow- Gently 345. Select the synonym of the given word. INDELIBLE (a) illegible (b) inerasable (c) ineffective (d) illegal SSC CGL (Tier-I) – 06/06/2019 (Shift-I) Ans. (b): Indelible - Synonym - 'Inerasable' Meaning of other words– Illegible - Vague Ineffective - Sterilized Illegal - Lawless 346. Select the synonym of the given word. INEPT (a) Clumsy (b) fit (c) strong (d) capable SSC CGL (Tier-I) – 04/06/2019 (Shift-III) Ans. (a): Inept - Synonym - 'Clumsy' Meaning of other words– Strong- Firm, Capable- Eligible, Fit- Suitable 347. Select the synonym of the given word. INARTICULATE (a) eloquent (b) incoherent (c) inevitable (d) fluent SSC CGL (Tier-I) – 04/06/2019 (Shift-III) Ans. (b): : Inarticulate - Synonym – Incoherent Meaning of other words– Eloquent- Elocution Inevitable- Mandatory Fluent- Innate 348. Select the most appropriate synonym of the given word. INIMICAL (a) Deliberate (b) Comparable (c) Hostile (d) Influential SSC CPO-SI – 11/12/2019 (Shift-II) Ans. (c) Inimical - Synonym- 'Hostile' Meaning of other words– Deliberate-Intentional, Comparable-Equivalent, Influential- Impressive 349. Select the most appropriate synonym of the given word. INCREDIBLE

407

YCT

(a) unnoticed (b) unwilling (c) disturbing (d) unbelievable SSC CPO-SI – 12/12/2019 (Shift-II) Ans. (d) Incredible - Synonym – Unbelievable Meaning of other words– Unnoticed - Unaware, Unwilling- Unwished, Disturbing- Nervous 350. Select the most appropriate synonym of the given word. INTENSIFY (a) destroy (b) strengthen (c) eliminate (d) terminate SSC CPO-SI – 12/12/2019 (Shift-II) Ans. (b) Intensify - Synonym – Strengthen Meaning of other words– Destroy - Abrade, Eliminate - Delete, Terminate Abolish 351. Select the most appropriate synonym of the given word. IGNOMINY (a) morality (b) virtue (c) disgrace (d) esteem SSC CPO-SI – 12/12/2019 (Shift-I) Ans. (c) Ignominy - Synonym – Disgrace Meaning of other words– Morality - Decorum Virtue - Merits Esteem - Honor 352. Select the most appropriate synonym of the given word. INCESSANT (a) inconstant (b) intermittent (c) persistent (d) transient SSC CPO-SI – 12/12/2019 (Shift-I) Ans. (c) Incessant- Synonym – Persistent Meaning of other words– Inconstant - Shaky Intermittent - Discrete Transient - Momentary 353. Select the most appropriate synonym of the given word. IDENTICAL (a) similar (b) legible (c) detailed (d) perfect SSC CHSL (Tier-I) –10/07/2019 (Shift-I) Ans. (a): Identical- Synonym – Similar Meaning of other words– Legible- Readable Detailed- Extensive Perfect- Good 354. Select the most appropriate synonym of the given word. IDEOLOGY (a) contract (b) belief (c) mark (d) recognition SSC CHSL (Tier-I) –09/07/2019 (Shift-II) Ans. (b) Ideology- Synonym – Belief Meaning of other words– Contract- Agreement Mark- Trail Recognition- Identity Synonyms

355. Select the most appropriate synonym of the given word. INTREPID (a) gallant (b) cowardly (c) insecure (d) invisible SSC CHSL (Tier-I) –08/07/2019 (Shift-I) Ans. (a): Intrepid- Synonym- "Gallant" Meaning of other words– Cowardly- Timid Insecure- Unsafe Invisible- Hidden 356. Select the most appropriate synonym of the given word. IGNOMINIOUS (a) wonderful (b) disgraceful (c) innocuous (d) generous SSC CHSL (Tier-I) –08/07/2019 (Shift-III) Ans. (b): Ignominious- Synonym -'Disgraceful' Meaning of other words– Wonderful- Amazing Innocuous- Unharmed Generous- Liberal 357. Select the most appropriate synonym of the given word. INNUENDO (a) implication (b) evidence (c) verification (d) proof SSC CHSL (Tier-I) –02/07/2019 (Shift-I) Ans. (a): Innuendo - Synonym- Implication Meaning of other words– Evidence- Witness, Verification- Enquire, ProofVindication 358. Select the most appropriate synonym of the given word. Idleness (a) Energy (b) Industry (c) Cheerfulness (d) Lethargy SSC CHSL (Tier-I) 26/10/2020 (Shift-II) Ans. (d): Idleness - Synonym- Lethargy Meaning of other words– Energy - Power Industry - Diligence Cheerfulness - Zeal 359. Select the most appropriate synonym of the given word: Indomitable (a) Accessible (b) Invincible (c) Justifiable (d) Changeable SSC CHSL (Tier-I) 21/10/2020 (Shift-I) Ans. (b): Indomitable - Synonym- Invincible Meaning of other words– Accessible – Obtainable Justifiable – Rational Changeable – Unsettled 360. Select the most appropriate synonym of the given word. INTERSPERSED (a) Scattered (b) Met (c) Collected (d) Wore SSC CHSL (Tier-I) 15/10/2020 (Shift-III) Ans. (a): Interspersed- Synonym - 'Scattered' Meaning of other words– Met - Converge, Collected - Gather, Wore - Put on

408

YCT

361. Select the most appropriate synonym of the given word. INTEGRATE (a) Disappear (b) Unite (c) Increase (d) Divide SSC CHSL (Tier-I) 15/10/2020 (Shift-II) Ans. (b): Integrate - Synonym - 'Unite' Meaning of other words– Disappear – Vanish, Increase - Disappear, Divide Split 362. Select the most appropriate synonym of the given word. Insidious (a) Callous (b) Benevolent (c) Crafty (d) Sincere SSC CHSL (Tier-I) 12/10/2020 (Shift-III) Ans. (c): Insidious - Synonym- Crafty Meaning of other words– Callous - Harsh Benevolent - Generous Sincere - Honest 363. Select the most appropriate synonym of the given word. INSTIGATE (a) Instill (b) Stagnate (c) Kindle (d) Imbibe SSC CHSL (Tier-I) 18/03/2020 (Shift-III) Ans. (c): 'Instigate'- Synonym - 'Kindle' Meaning of other words– Instill - Blob Stagnate - Extravasate Imbibe - Adsorb 364. In the following question, out of given four alternatives, select the one which best expresses the meaning of the given word. Invade (a) Abuse (b) Ravage (c) Trash (d) Shred SSC MTS 10-10-2017 (Shift-I) Ans. (b) Invade- Synonym- Ravage Meaning of other words– Abuse- Rebuke, Trash- Junk, Shred- Crumble 365. Select the most appropriate synonym of the given word. Infuriate (a) Endow (b) Enhance (c) Enrage (d) Engage SSC MTS – 05/08/2019 (Shift-III) Ans. (c): Infuriate- Synonym – Enrage Meaning of other words– Endow - Provide, Enhance - Enlarge, Engage- Inflict 366. Select the most appropriate synonym of the given word. Immense (a) Insignificant (b) Dull (c) Minute (d) Massive SSC MTS – 05/08/2019 (Shift-I) Ans. (d): 'Immense'- Synonym - 'Massive' Meaning of other words– Insignificant - Frivolous, Dull - Tedious, Minute Moment Synonyms

367. Select the most appropriate synonym of the given word. INTRICATE (a) Complicated (b) Simple (c) Plain (d) Straightforward SSC MTS – 16/08/2019 (Shift-II) Ans. (a): Intricate- Synonym – Complicated Meaning of other words– Simple - Easy Plain - Decent Straightforward - Apparent 368. Select the most appropriate synonym of the given word. INTEGRATION (a) Separation (b) Unification (c) Segregation (d) Division SSC MTS – 14/08/2019 (Shift-II) Ans. (b): 'Integration' - Synonym – 'Unification' Meaning of other words– Separation– Segregation, Segregation– Isolation, Division– Partition 369. Select the synonym of the given word. Impolite (a) Intelligent (b) Brave (c) Rude (d) Irrelevant SSC MTS – 09/08/2019 (Shift-II) Ans. (c): Impolite - Synonym – Rude Meaning of other words– Intelligent - Wise Brave - Hero Irrelevant - Futile 370. Select the most appropriate synonym of the given word Inclination (a) Aptitude (b) Indifference (c) Repulsion (d) Aversion SSC MTS – 21/08/2019 (Shift-I) Ans. (a): Inclination - Synonym – Aptitude Meaning of other words– Indifference - Apathy, Repulsion - Dislike, Aversion Malice 371. Select the most appropriate synonym of the given word. Impious (a) Irreligious (b) Devout (c) Skilled (d) Loyal SSC MTS – 21/08/2019 (Shift-II) Ans. (a): Impious - Synonym – Irreligious Meaning of other words– Devout - Godly, Skilled - Adept, Loyal – Faithful 372. Select the synonym of the given word. INDUSTRIOUS (a) Fruitful (b) Practical (c) Prompt (d) Diligent SSC GD – 18/02/2019 (Shift-II) Ans. (d): Industrious - Synonym – Diligent Meaning of other words– Fruitful - Profitable, Prompt - Fast, Practical - Usual 373. Select the synonym of the given word. IMPECCABLE (a) Imperfect (b) Flawless (c) Smooth (d) Clean SSC GD – 14/02/2019 (Shift-I)

409

YCT

Ans. (b): Impeccable - Synonym Meaning of other words– Imperfect - Pending Smooth - Sleek Clean - Neat

– Flawless

J 374. Select the most appropriate synonym of the given word. JUST (a) fair (b) favouring (c) courtly (d) partial SSC CPO-SI – 23/11/2020 (Shift-I) Ans. (a): Just– Synonym - 'fair' Meaning of other words– Favouring – Side Courtly – Decent Partial – Partisan 375. Select the most appropriate synonym of the given word. JEOPARDY (a) prevention (b) protection (c) peril (d) promotion SSC CPO-SI – 12/12/2019 (Shift-I) Ans. (c) Jeopardy - Synonym – Peril Meaning of other words– Prevention - Detention Protection - Security Promotion - Boost 376. Select the most appropriate synonym of the given word. JOVIAL (a) Jeering (b) Joyous (c) Judgmental (d) Jealous SSC CHSL (Tier-I) 18/03/2020 (Shift-I) Ans. (b): 'Jovial'- Synonym - 'Joyous' Meaning of other words– Jeering - Droll Judgmental - Decision Jealous - Hateful 377. Select the most appropriate synonym of the given word. Judicious (a) Prudent (b) Dull (c) Foolish (d) Unfair SSC MTS – 21/08/2019 (Shift-III) Ans. (a): Judicious - Synonym – Prudent Meaning of other words– Dull - Floppy, Foolish - Stupid, Unfair - Undue

K 378. Select the most appropriate synonym of the given word. Kinship (a) Incompatibility (b) Relationship (c) Partnership (d) Viewership SSC Constable GD-26/11/2021 Shift-I Synonyms

Ans. (b): Kinship (Consanguinity), Synonym Relationship (Connection) Meaning of the other optionsIncompatibility – Inconsistency Partnership– Sharing Viewership – Audience 379. Select the most appropriate synonym of the given word. KIT (a) Equipment (b) Information (c) Particle (d) Element SSC CHSL (Tier-I) 18/03/2020 (Shift-II) Ans. (a): 'Kit'- Synonym - 'Equipment' Meaning of other words– Information - Intimation Particle – Atom Element - Part 380. Select the most appropriate synonym of the given word. Knotty (a) Simple (b) Complex (c) Visible (d) Shriek SSC GD – 18/02/2019 (Shift-II) Ans. (b): Knotty - Synonym - Complex Meaning of other words– Simple - Easy, Visible - Clear, Shriek - Scream 381. Select the most appropriate synonym of the given word. Keen (a) Dull (b) Blunt (c) Eager (d) Adept SSC MTS – 05/08/2019 (Shift-III) Ans. (c): Keen - Synonym - Eager Meaning of other words– Dull - Obtuse, Blunt - Morose, Adept - Efficient 382. Select the most appropriate synonym of the given word. Knave (a) Mundane (b) Common (c) Rogue (d) Equivocal SSC CPO-SI (Tier-II) 27/09/2019 (3 pm - 5 pm) Ans. (c): Knave - Synonym - Rogue Meaning of other words– Mundane - Cosmic, Common - General, Equivocal Dubious 383. Select the most appropriate synonym of the given word. Knit (a) Lucid (b) Kit (c) Weave (d) Tremble SSC CHSL (Tier-I) 18/03/2020 (Shift-III) Ans. (c): Knit - Synonym - Weave Meaning of other words– Lucid - Bright, Kit - Fiddle, Tremble - Shiver

410

YCT

L 384. Select the most appropriate synonym of the given word. Leap (a) Run (b) Pile (c) Peek (d) Hop SSC GD 08/02/2023 (Shift-IV) Ans. (d) : 'Hop' is the most appropriate synonym of the given word– 'Leap'. Hop means – to move by jumping on one foot. The meaning of other options is– Run :- compete in a race. Pile :- bundle, pack Peek :- glance, peep. 385. Choose the word that means the same as the given word. Laggard (a) Prompt (b) Hustler (c) Dawdling (d) Expeditious SSC CHSL (Tier-I) 09/02/2023 (Shift-II) Ans. (c) : Laggard Synonym, Dawdling Meaning of other wordsPrompt – Expedite Expeditious – Fast Hustler – Bookmaker, Jamester 386. Select the most appropriate synonym of the given word. Lacuna (a) Apathy (b) Misfortune (c) Languor (d) Hiatus SSC CGL (Tier-I) 13/04/2022 Shift-I Ans. (d): Lacuna (Lack, Blank space), SynonymHiatus (Gap, Break) Meaning of the other optionsApathy – Indifference Misfortune – Adversity Languor – Fstigue 387. Select the most appropriate synonym of the given word. LULL (a) Calm (b) Noise (c) Activity (d) Disturbance SSC CGL (Tier-I) 20/04/2022 Shift-III Ans. (a): Lull (Sooth), Synonym- Calm (Quiet) Meaning of the other optionsNoise - Cacophony Activity - Action, Act Disturbance - Disorder, Obstacle 388. Select the most appropriate synonym of the given word. Loathe (a) Abhor (b) Meddle (c) Abjure (d) Entrench SSC CHSL 25.05.2022 Shift-I Ans. (a): Loathe (Hate), Synonym - Abhor (Detest) Meaning of the other optionsMeddle – Interfere Abjure – Renounce, Abdicate Entrench – Establish, Settle Synonyms

389. Select the most appropriate synonym of the given word. Lethal (a) Lazy (b) Fatal (c) Beneficial (d) Slothful SSC Constable GD-14/12/2021 Shift-II Ans. (b): Lethal (Deadly), Synonym - Fatal (Dangerous, Harmful) Meaning of the other optionsBeneficial - Advantageous, Favorable Slothful - Sluggish Lazy - Idle, Lethargic 390. Select the most appropriate synonym of the given word. LIBERTY (a) captivity (b) bondage (c) freedom (d) slavery SSC Stenographer (Grade C & D) 12.11.2021 Shift-I Ans. (c): Liberty (Freedom), Synonym - Freedom (Independence) Meaning of the other optionsSlavery - Bondage Bondage - Captivity Captivity - Custody 391. Select the most appropriate synonym of the given word. LIVID (a) furious (b) timid (c) mild (d) lively SSC CHSL (Tier-I) –06/08/2021 (Shift-I) Ans. (a): Livid – Synonym – Furious Meaning of other words– Mild - Gentle, Timid - Sneaky, Lively - Fancy 392. Select the most appropriate synonym of the given word. Leverage (a) Suggestion (b) Weakness (c) Conclusion (d) Influence SSC CGL (Tier-I) –24/08/2021 (Shift-I) Ans. (d): Leverage - Synonym - Influence Suggestion - Proposal Weakness - Acratia Conclusion - Apilogue 393. Select the most appropriate synonym of the given word. LOFTY (a) towering (b) humble (c) silent (d) plain SSC CPO-SI – 24/11/2020 (Shift-II) Ans. (a): 'Lofty'- Synonym - 'Towering' Meaning of other words– Humble - Meek Silent - Tacit Plain - Homely 394. Select the most appropriate synonym of the given word. LAMENT (a) afflict (b) mourn (c) torment (d) distress SSC CGL (Tier-I) – 10/06/2019 (Shift-III) Ans. (b): Lament - Synonym - 'Mourn' Meaning of other words– Afflict- Smite Torment- Agony Distressed- Afflicted

411

YCT

401. Select the most appropriate synonym for the highlighted word. His modest nature made all the difference. (a) dry (b) courageous (c) horrific (d) shy SSC CGL (Tier-I) 26/07/2023 (Shift-II) Ans. (d) : The most appropriate synonym for the highlighted word 'Modest' is 'shy'. other words are – Dry means – Tedious Courageous means – Impudent Horrific means –Grueling. 402. Select the word that is similar in meaning (synonym) to the given below 'Malign' (a) Nihilistic (b) Insurgent (c) Baleful (d) Radical SSC CHSL (Tier-I) 15/03/2023 (Shift-III) Ans. (c) : Malign - Synonym - 'Baleful' Meaning of other wordsInsurgent → Rebel Radical → Fiery Nihilistic → Related to drug 403. Select the most appropriate synonym of the given word. Maintain (a) Neglect (b) Ignore (c) Care (d) Release SSC Constable GD-30/11/2021 Shift-III Ans. (c): Maintain (Nurture, Preserve), Synonym - Care (Concern) Meaning of the other optionsNeglect - Disregard Ignore - Avoid Release - Liberation, Discharge 404. Select the most appropriate synonym of the given word. Mandate (a) Question (b) Request (c) Enquire (d) Command SSC MTS-14/07/2022 Shift-III Ans. (d): Mandate (Directive), Synonym – Command (Order) Meaning of the other optionsQuestion – Query Request – Urge Enquire – Investigation 400. Select the most appropriate synonym of the 405. Select the most appropriate synonym of the given word. given word. Manifest Modest (a) Apparent (b) Cryptic (a) Bold (b) Humble (c) Natural (d) Concealed (c) Dismal (d) Grave SSC GD 16/01/2023 (Shift-III) SSC MTS-06/07/2022 Shift-I Ans. (a) : Manifest (Direct, Revealed), SynonymAns. (b): Modest (Gracious), Synonym- Humble Apparent (Clear Evident) (Submissive) Meaning of the other optionsMeaning of the other optionsCryptic Hidden Bold – Adventurous Natural Original, Real Dismal – Depressed Concealed Covered Grave – Serious 395. Select the synonym of the given word. LETHAL (a) harmless (b) healthy (c) fatal (d) strong SSC CGL (Tier-I) – 07/06/2019 (Shift-I) Ans. (c) Lethal- Synonym - 'Fatal' Harmless - Unoffending Healthy - Sane Strong - Sturdy 396. Select the most appropriate synonym of the given word. LOYAL (a) routine (b) devoted (c) varied (d) legal SSC CPO-SI – 12/12/2019 (Shift-II) Ans. (b) Loyal- Synonym – Devoted Routine – Common Varied – Different Legal – Statutory 397. Select the most appropriate SYNONYM of the given word. LURK (a) sing (b) prowl (c) vibrate (d) tremble SSC CHSL (Tier-I) 15/10/2020 (Shift-I) Ans. (b): 'Lurk'- Synonym- 'Prowl Sing - Carol, Vibrate - Oscillate, Tremble- Shiver 398. Select the most appropriate synonym of the given word. Lavish (a) Scarce (b) Bountiful (c) Simple (d) Austere SSC MTS – 05/08/2019 (Shift-II) Ans. (b): Lavish - Synonym – Bountiful Scarce - Scrimp, Simple - Unpretentious, AustereSimple 399. Select the most appropriate synonym of the given word. LUCID (a) Confusing (b) Clear (c) Ambiguous (d) Unclear SSC MTS – 14/08/2019 (Shift-III) Ans. (b): Lucid- Synonym – Clear Confusing - Jitter, Ambiguous-Dubious, UnclearVague

M

Synonyms

412

YCT

406. Select the most appropriate synonym of the given word. Malign (a) Approve (b) Benefit (c) Besmirch (d) Commend SSC MTS-05/07/2022 Shift-III Ans. (c): Malign (Harmful, Evil), Synonym - Besmirch (Sully) Meaning of the other optionsApprove – Sanction, Permit Benefit – Advantage, Profit Commend – Admire 407. Select the most appropriate synonym of the given word. Monotonous (a) Eventful (b) Boring (c) Momentous (d) Compulsory SSC CGL (Tier-I) 19/04/2022 Shift-II Ans. (b): Monotonous (Dull), Synonym- Boring (Humdrum) Meaning of the other optionsEventful - Occurring Momentous - Significant Compulsory - Mandatory 408. Select the most appropriate synonym of the given word. MALIGNANT (a) vicious (b) beneficial (c) loving (d) healthy SSC CHSL (Tier-I) –04/08/2021 (Shift-I) Ans. (a): Malignant - Synonym - Vicious Meaning of other words– Beneficial - Profitable, Loving - Sweet, Healthy Healed 409. Select the most appropriate synonym of the given word. Mandatory (a) Unnecessary (b) Optional (c) Imperative (d) Impervious SSC CHSL (Tier-I) –19/04/2021 (Shift-I) Ans. (c): Mandatory - Synonym - Imperative Meaning of other words– Unnecessary- Useless, Optional - Alternate, Impervious - Potent 410. Select the most appropriate synonym of the given word. Munch (a) Drew (b) Flew (c) Chew (d) Burp SSC CGL (Tier-I) –17/08/2021 (Shift-I) Ans. (c): Munch - Synonym - Chew Meaning of other words– Drew- Exert, Flew- Hover, Burp - Belch 411. Select the most appropriate synonym of the given word. MAGNIFY (a) enlarge (b) withdraw (c) shrink (d) decrease SSC CPO-SI (Tier-II) 27/09/2019 (3 pm - 5 pm) Synonyms

Ans. (a): 'Magnify' - Synonym – Enlarge Meaning of other words– Withdraw - Recant Shrink - Rumple Decrease - Lack 412. Select the most appropriate synonym of the given word. MASSIVE (a) honour (b) majority (c) muscular (d) huge SSC Stenographer Grade C&D –05/02/2019 (3:5pm)

Ans. (d): 'Massive'- Synonym – Huge Meaning of other words– Honour - Reverence Majority - Plurality Muscular - Fleshy 413. Select the most appropriate synonym of the given word. Mandatory (a) optional (b) compulsory (c) secondary (d) free SSC Sel. Post Phase-VII (M.L.) 15.10.2019 (Shift-I)

Ans. (b): Mandatory –Synonym- Compulsory Meaning of other words– Optional - Alternate Secondary - Middle Free - Liberate 414. Select the most appropriate synonym of the given word. MAMMOTH (a) magnificent (b) miniscule (c) perilous (d) gigantic SSC CGL (Tier-I) – 11/06/2019 (Shift-II) Ans. (d): Mammoth– Synonym - ‘Gigantic’ Meaning of other words– Magnificent- Superb, Miniscule- Tiny, Perilousdangerous 415. Select the most appropriate synonym of the given word. MALADY (a) mischief (b) disturbance (c) wickedness (d) illness SSC CPO-SI – 13/12/2019 (Shift-II) Ans. (d): Malady- Synonym – Illness Meaning of other words– Mischief - Harm, Disturbance - Unrest, Wickedness Badness 416. Select the most appropriate synonym of the given word. Miniature (a) Diminutive (b) Mammoth (c) Colossal (d) Gigantic SSC CHSL (Tier-I) –11/07/2019 (Shift-II) Ans. (a): Miniature- Synonym – Diminutive Meaning of other words– Mammoth- Huge, Colossal- Cosmic, Gigantic- Large 417. Select the most appropriate synonym of the given word. MANUAL (a) thesaurus (b) handbook (c) directory (d) dictionary SSC CHSL (Tier-I) –11/07/2019 (Shift-III)

413

YCT

Ans. (b) Manual- Synonym - Handbook Meaning of other words– Thesaurus- Lexicon, Dictionary - Vocabulary, Directory- Catalogue 418. Select the most appropriate synonym of the given word. Meddle (a) manage (b) interfere (c) repair (d) elaborate SSC CHSL (Tier-I) –09/07/2019 (Shift-II) Ans. (b) Meddle- Synonym- Interfere Meaning of other words– Manage- Run Repair- Improve Elaborate- Detailed 419. Select the most appropriate synonym of the given word. MASQUERADED (a) Managed (b) Dropped (c) Acted (d) Disappeared SSC CHSL (Tier-I) 15/10/2020 (Shift-III) Ans. (c): 'Masqueraded'- Synonym - 'Acted' Meaning of other words– Managed - Determinate, Dropped- Erring, Disappeared - Vanished 420. Select the most appropriate synonym of the given word. MOTIF (a) Creation (b) Reason (c) Purpose (d) Design SSC CHSL (Tier-I) 20/10/2020 (Shift-II) Ans. (d): 'Motif'- Synonym - 'Design' Meaning of other words– Creation – Building, Reason – Logic, Purpose – Motive 421. Select the most appropriate synonym of the given word. MOROSE (a) Sullen (b) Cheerful (c) Optimistic (d) Friendly SSC CHSL (Tier-I) 19/10/2020 (Shift-II) Ans. (a): Morose– Synonym – Sullen Meaning of other words– Cheerful– Glad, Optimistic– Sanguine, Friendly– Kind 422. Select the most appropriate synonym of the given word. MANDIBLE (a) Human (b) Praise (c) Glory (d) Jaw SSC CHSL (Tier-I) 14/10/2020 (Shift-I) Ans. (d): 'Mandible'- Synonym – Jaw Meaning of other words– Human - Earthborn, Praise– Admire, Glory- Majesty 423. Select the most appropriate synonym of the given word. MANDATE (a) Official order (b) Last trip (c) Broken promise (d) Man-made SSC CHSL (Tier-I) 14/10/2020 (Shift-III) Synonyms

Ans. (a): Mandate- Synonym - Official-order Meaning of other words– Last trip– Destination Broken promise– Abandon Man-made – Artificial 424. In the following question, out of the given four alternatives, select the one which best expresses the meaning of the given word. Mince (a) Reveal (b) Enlarge (c) Crumble (d) Maximize SSC MTS 11-10-2017 (Shift-I) Ans. (c) Mince- Synonym- Crumble Reveal- Demonstration Enlarge - Enhance Maximize- Utmost 425. Select the most appropriate synonym of the given word. Misfortune (a) Benefit (b) Calamity (c) Support (d) Miracle SSC MTS – 02/08/2019 (Shift-II) Ans. (b): Misfortune - Synonym – Calamity Benefit – Mileage Support – Adherence Miracle - Marvel 426. Select the most appropriate synonym of the given word. Morbid (a) Nasty (b) Prompt (c) Pretty (d) Lively SSC MTS – 20/08/2019 (Shift-III) Ans. (a): Morbid - Synonym - 'Nasty' Meaning of other words– Prompt - Expedite Pretty - Scenic Lively - Fancy 427. Select the synonym of the given word. Massive (a) Huge (b) Weak (c) Tiny (d) Strong SSC MTS – 13/08/2019 (Shift-III) Ans. (a): Massive - Synonym – Huge Meaning of other words– Weak - Feeble Tiny - Puny Strong - Firm 428. Select the most appropriate synonym of the given word. Mock (a) Cheer (b) Taunt (c) Praise (d) Amuse SSC MTS – 22/08/2019 (Shift-I) Ans. (b): 'Mock'- Synonym - 'Taunt' Meaning of other words– Cheer - Glad Praise - Admire Amuse-Entertain 429. Select the synonym of the given word. MITIGATE (a) Excuse (b) Improve (c) Moderate (d) Lessen SSC GD – 18/02/2019 (Shift-II)

414

YCT

Ans. (d): Mitigate - Synonym – Lessen Meaning of other words– Excuse - Motive, Improve - Repair, Moderate Medium 430. Select the synonym of the given word. MELODIOUS (a) Rhythmic (b) Rattling (c) Tuneful (d) Doleful SSC GD – 14/02/2019 (Shift-II) Ans. (c): Melodious - Synonym - Tuneful Meaning of other words– Rhythmic – Magnifier Doleful - Gloomy, Rattling - Stunning

N 431. Select the most appropriate synonym of the underlined word. We had to move slowly through the narrow alley. (a) Imminent (b) Dreary (c) Maximum (d) Cramped SSC GD 16/01/2023 (Shift-III) Ans. (d) : Narrow Synonym- Cramped (Confind Limited) Meaning of the other optionsImminent Adjacent Dreary Dull, Forlorn Maximum Most, Utmost 432. Select the most appropriate synonym of the given word. Noble (a) Normal (b) Ordinary (c) Poor (d) Dignified SSC Constable GD-24/11/2021 Shift-II Ans. (d): Noble (Honorable, Prestigious), Synonym Dignified (Reputed, Distinguished) Meaning of the other optionsNormal - General, Ordinary Ordinary - Mediocre, Common Poor - Indigent, Impoverished 433. Select the most appropriate synonym of the given word. Numerous (a) Numbered (b) Few (c) Totalled (d) Several SSC CGL (Tier-I) –16/08/2021 (Shift-I) Ans. (d): Numerous - Synonym - Several Meaning of other words– Numbered - Enumerated, Few - Some, Totalled - All 434. Select the synonym of the given word. Nebulous (a) Stable (b) Faith (c) Vague (d) Dearly SSC CHSL (Tier-I) 13/10/2020 (Shift-III) Ans. (c) 'Nebulous'- Synonym - 'Vague' Meaning of other words– Stable - Static, Faith - Believe Dearly - Gently Synonyms

435. In the following question out of the given four alternatives, select the one which best expresses the meaning of the given word Nudge (a) Allow (b) Leave alone (c) Poke (d) Withdraw SSC MTS 9-10-2017 (Shift-III) Ans. (c) Nudge- Synonym- Poke Meaning of other words– Allow- Permit Leave alone- Alone Withdraw- Recant 436. In the following question, out of the given four alternatives, select the one which best expresses the meaning of the given word. Notorious (a) Disreputable (b) Ethical (c) Honorable (d) Glorious SSC MTS 10-10-2017 (Shift-II) Ans. (a): Notorious– Synonym- Disreputable Meaning of other words– Ethical – Moral, Honorable – Prestigious, Glorious – August 437. Select the synonym of the given word. NOMADIC (a) Barbaric (b) Brave (c) Wild (d) Roving SSC GD – 18/02/2019 (Shift-III) Ans. (d): Nomadic - Synonym – Roving Meaning of other words– Barbaric - Wild Brave - Daredevil Wild - Wooded 438. Select the most appropriate synonym of the given word. Nexus (a) Aloof (b) apparent (c) Cohesion (d) Adhere SSC CHSL (Tier-I) –10/07/2019 (Shift-I) Ans. (c): Nexus - Synonym - Cohesion Meaning of other words– Apparent - Clear, Aloof - Separate, Adhere - Abide 439. Select the most appropriate synonym of the given word. Nomad (a) Lively (b) Reduce (c) Migrant (d) Weary SSC CHSL (Tier-I) –09/07/2019 (Shift-I) Ans. (c): Nomad - Synonym - Migrant Meaning of other words– Lively - Fervent, Reduce - Restrain, Weary - Dull 440. Select the most appropriate synonym of the given word. Nocturnal (a) Honorable (b) Ethical (c) Nightly (d) Glorious SSC CHSL (Tier-I) 17/03/2020 (Shift-III) Ans. (c): Nocturnal - Synonym - Nightly Meaning of other words– Honorable - Decent, Ethical - Moral, Glorious Renowned

415

YCT

441. Select the most appropriate synonym of the given word. Nonchalant (a) Barbaric (b) Stoic (c) Concerned (d) Distinct SSC MTS – 06/08/2019 (Shift-I) Ans. (b): Nonchalant - Synonym - Stoic Meaning of other words– Barbaric - Wild, Concerned - Related, Distinct Isolated

O 442. Select the most appropriate synonym of the given word. Omnipresent (a) Powerful (b) Unique (c) Contemporary (d) Universal SSC GD 12/01/2023 (Shift-II) Ans. (d) : Omnipresent (Ubiquitous) - Synonym Universal (Global, worldwide) Meaning of the other optionsPowerful Influential, Potent Unique Peerless Contemporary Synchronize Present day 443. Select the most appropriate synonym of the given word. Genial (a) Optimistic (b) Cheerful (c) Despondent (d) Positive SSC CGL (Tier-I) 21/07/2023 (Shift-II) Ans. (b) : The most appropriate synonym of the above given word 'Genial' is 'cheerful.' Meaning of other options Optimistic - Sanguine, Despondent - Hopeless, Positive - useful/definite 444. Choose the word that means the same as the given word. Obligation (a) Imperative (b) Loophole (c) Discharge (d) Quietus SSC CHSL (Tier-I) 17/03/2023 (Shift-IV) Ans. (a) : Obligation - Synonym - imperative Meaning of other wordsDischarge - Release Loophole - Hole Quietus - Death 445. Select the most appropriate synonym of the given word. Obstruct (a) Replace (b) Curb (c) Release (d) Unclog SSC MTS-06/07/2022 Shift-III Ans. (b): Obstruct (Hinder), Synonym - Curb (Restriction, Control) Meaning of the other optionsReplace – Substitute Release – Discharge, Emancipation Unclog – Open Synonyms

446. Select the most appropriate synonym of the given word. Occlude (a) facilitate (b) promote (c) Abstain (d) Obstruct SSC MTS-05/07/2022 Shift-III Ans. (d): Occlude (Deter), Synonym - Obstruct (Hinder) Meaning of the other optionsFacilitate – Case, Simplify Promote – Encourage Abstain – Evade 447. Select the most appropriate synonym of the given word. Obtrusive (a) Pervasive (b) Exclusive (c) Prominent (d) Modest SSC MTS-06/07/2022 Shift-II Ans. (c): Obtrusive (Conspicuous), SynonymProminent (Renowned) Meaning of the other optionsPervasive – Comprehensive Exclusive – Special Modest – Humble 448. Select the most appropriate synonym of the given word. ostentatious (a) showy (b) tasteful (c) sudden (d) quick SSC CGL (Tier-I) 11/04/2022 Shift-I Ans. (a): Ostentatious (Pretentious), Synonym – Showy (Pompous) Meaning of the other optionsTasteful – Delicious Sudden – Unexpected Quick – Fast 449. Select the most appropriate synonym of the given word. Overture (a) Slander (b) Plunk (c) Preamble (d) Conclusion SSC CHSL 30.05.2022 Shift-I Ans. (c): Overture (Proposal), Synonym- Preamble (Prelude, Preface) Meaning of the other optionsSlander - Infamy, Defamation Plunk - Throw, Peal Conclusion - Gist 450. Select the most appropriate synonym of the given word. Obvious (a) Dull (b) Thick (c) Clear (d) Vague SSC Constable GD-08/12/2021 Shift-I Ans. (c): Obvious (Explicit, Evident), Synonym - Clear (Apparent) Meaning of the other optionsDull - Humdrum, Inactive Thick - Plump Vague - Obscure, Ambiguous

416

YCT

451. Select the most appropriate synonym of the given word. OBLIVIOUS (a) ignorant (b) attentive (c) obvious (d) sensitive SSC CPO-SI – 25/11/2020 (Shift-II) Ans. (a): Oblivious - Synonym – Ignorant Meaning of other words– Attentive - Observant, Obvious - Manifest, SensitiveSentient 452. Select the most appropriate synonym of the given word. OBSTACLE (a) benefit. (b) clearance (c) accessory (d) barrier SSC CGL (Tier-I) – 13/06/2019 (Shift-I) Ans. (d) Obstacle– Synonym -‘Barrier’ Meaning of other words– Benefit– Profit Clearance– Regress Accessory– Assistant 453. Select the most appropriate synonym of the given word. OFFEND (a) aid (b) annoy (c) assist (d) assuage SSC CPO-SI – 09/12/2019 (Shift-II) Ans. (b) Offend- Synonym - 'Annoy' Meaning of other words– Aid- Relief, Assist- Help, Assuage- Soothe 454. Select the most appropriate synonym of the given word. OPINION (a) view (b) wisdom (c) chance (d) sense SSC CPO-SI – 12/12/2019 (Shift-II) Ans. (a): Opinion - Synonym – View Meaning of other words– Wisdom - Sagacity, Chance - Opportunity, Sense Spirit 455. Select the most appropriate synonym of the given word. OSTENTATIOUS (a) distinct (b) complete (c) flashy (d) trusted SSC CHSL (Tier-I) –10/07/2019 (Shift-I) Ans. (c) Ostentatious- Synonym - 'Flashy' Meaning of other words– Distinct- Isolated Complete- Total Trusted- Believe 456. Select the most appropriate synonym of the given word. ORTHODOX (a) civilized (b) habitual (c) traditional (d) deliberate SSC CHSL (Tier-I) –09/07/2019 (Shift-I) Ans. (c) Orthodox - Synonym – Traditional Meaning of other words– Civilized - Decent Habitual - Used to Deliberate - Consider Synonyms

457. Select the most appropriate synonym of the given word. OPPORTUNE (a) rarely (b) likely (c) timely (d) barely SSC CHSL (Tier-I) –04/07/2019 (Shift-III) Ans. (c) Opportune- Synonym -'Timely' Meaning of other words– Rarely- Perhaps, Likely- Suitable, Barely- Scarcely 458. Select the most appropriate synonym of the give word. OPULENT (a) Virulent (b) Distant (c) Luxurious (d) Shabby SSC CHSL (Tier-I) 13/10/2020 (Shift-I) Ans. (c): Opulent– Synonym- Luxurious Meaning of other words– Virulent – Spiteful Distant – Far Shabby – Poor 459. Select the most appropriate synonym of the given word. OPULENT (a) Sumptuous (b) Poor (c) Economical (d) Depressed SSC CHSL (Tier-I) 19/10/2020 (Shift-III) Ans. (a): Opulent– Synonym – Sumptuous Meaning of other words– Poor – Penniless, Economical – Frugal, Depressed – Dreary 460. Select the most appropriate synonym of the given word. OCCUR (a) Hinder (b) Hasten (c) Happen (d) Hamper SSC CHSL (Tier-I) 17/03/2020 (Shift-III) Ans. (c): 'Occur'- Synonym - 'Happen' Meaning of other words– Hinder - Stop, Hasten - Hurry, Hamper - Hurdle 461. In the following question, out of the given four alternatives, select the one which best expresses the meaning of the given word. Obligation (a) Denial (b) Refusal (c) Commitment (d) Declination SSC MTS 10-10-2017 (Shift-II) Ans. (c): Obligation – Synonym – Commitment Meaning of other words– Denial – Reject, Refusal – Abnegation, Declination – Tilt 462. Select the most appropriate synonym of the given word. Obvious (a) Vague (b) Various (c) Flexible (d) Evident SSC MTS – 06/08/2019 (Shift-I) Ans. (d): Obvious - Synonym – Evident Meaning of other words– Vague- Obscure, Various- Diverse, Flexible - Elastic

417

YCT

463. Select the most appropriate synonym of the given word. Optimist (a) Critic (b) Distinct (c) Pessimist (d) Idealist SSC MTS – 06/08/2019 (Shift-I) Ans. (d): Optimist - Synonym - Idealist Meaning of other words– Critic - Baleful, Distinct - Specific, Pessimist - Cynic 464. Select the most appropriate synonym of the given word. OBTAIN (a) Achieve (b) Suppress (c) Inspire (d) Lessen SSC MTS – 20/08/2019 (Shift-I) Ans. (a): Obtain- Synonym – Achieve Meaning of other words– Suppress - Stifle Inspire - Induce lessen - Reduce

P 465. Select the appropriate synonym for the underlined word. I didn’t have time to give them proof. (a) snap (b) guess (c) evidence (d) charge SSC MTS 18/05/2023 (Shift-I) Ans. (c) : The appropriate synonym of the underline word ‘proof’ is ‘evidence’. Meaning of other optionsSnap – Incision, Guess – Presumption, Charge – Huff, Imputation, 466. Select the most appropriate option that can substitute the underlined word in the given sentence. She had an ability to persuade others. (a) suppress (b) halt (c) outrage (d) impress SSC CGL (Tier-I) 14/07/2023 (Shift-I) Ans. (d) : In the above sentence 'impress' is most appropriate synonym of underlined word 'persuade'. Hence option (d) will be correct answer. 467. Select the most appropriate synonym of the underlined word. The Sustainable Development Goals are a call for action by all countries—poor, rich and middle-income—to promote prosperity while protecting the planet. (a) conceal (b) growl (c) holistic (d) boost SSC CHSL (Tier-I) 10/08/2023 (Shift-I) Ans. (d) : The synonym of the underlined word promote – (To help something happen or develop) is option (d) Boost – (To increase something in number, value or strength) Meaning contained by other words – Conceal – Hide Growl – Howl Holistic – Integral Synonyms

468. Select the most appropriate synonym of the given word. Precise (a) Ambiguous (b) Accurate (c) General (d) Vague SSC CHSL (Tier-I) 10/08/2023 (Shift-I) Ans. (b) : Among the given options, option (b) Accurate – (Correct and exact and without any mistakes) is the synonym of word 'precise' –( very Careful and accurate, especially about small details) Meaning of the other options are – Ambiguous – Vague General – Usual Vague – Unclear 469. Select the most appropriate synonym of the given word. Prolific (a) Lazy (b) Unsuccessful (c) Productive (d) Unproductive SSC CHSL (Tier-I) 09/08/2023 (Shift-III) Ans. (c) : Option (c) Productive – (Providing a large amount or supply of something) will be the most appropriate synonym of the given word Prolific– (Producing a great number of something). other options have following meaning Lazy – Sluggish Unsuccessful – Jejune unproductive – Barren 470. Select the most appropriate synonym for the underlined word in the following sentence. The politician’s pompous rhetoric during the campaign failed to garner much support from the public. (a) Conciliatory (b) Benign (c) Diplomatic (d) Supercilious SSC CHSL (Tier-I) 02/08/2023 (Shift-I) Ans. (d) : The most appropriate synonym for the given underlined word 'pompous' (claptoap) is 'supercilious'. Meaning of other options – Conciliatory – Placatory, Appeasing Benign – Good, Placable, Diplomatic – Ambassadorial, consular 471. Select the most appropriate synonym of the underlined word. All the employees feel that continuing working there is perilous to their careers. (a) Helpful (b) Suitable (c) Dangerous (d) Convenient SSC CHSL (Tier-I) 08/08/2023 (Shift-II) Ans. (c) : The most appropriate synonym of the above underlined word 'perilous' is 'dangerous'. Meaning of other options – Helpful – Useful, Subsidiary, Suitable – Appropriate, Apt, Convenient – Suitable, Handy, 472. Select the most appropriate synonym for the highlighted word. There were a plethora of options available to the applicant. (a) destructive (b) excess (c) insufficient (d) recess SSC Selection Posts XI-28/06/2023 (Shift-III)

418

YCT

Ans. (b) : In the above sentence, synonym of underlined word "Plethora" is 'excess'. Other options give different meaning. Destructive - Harmful Insufficient - Inadequate Recess - Vacation 473. Select the denotation of the highlighted word. The archer doesn’t get perturbed easily. (a) Agitated (b) Triumphed (c) Animated (d) Initiated SSC CHSL 25.05.2022 Shift-I Ans. (a): Perturbed (Restless, Anxious), Synonym Agitated (Excited) Meaning of the other optionsTriumphed – Succeed, Victory Animated – Live, Stunning Initiated – Begin, Commence 474. Select the most appropriate synonym of the given word. Probe (a) Search (b) Motivate (c) Invite (d) Push SSC MTS-13/07/2022 Shift-I Ans. (a): Probe (Investigation), Synonym - Search (Inspection) Meaning of the other optionsMotivate – Encourage, Inspire Invite – Invitation Push – Force 475. Select the most appropriate synonym of the given word. Paranoid (a) Committed (b) Convinced (c) Trustful (d) Distrustful SSC CGL (Tier-I) 12/04/2022 Shift-I Ans. (d): Paranoid (Suspicious), Synonym - Distrustful (Mistrustful) Meaning of the other optionsCommitted - Restricted Convinced - Persuade Trustful - Faithful 476. Select the most appropriate synonym of the given word. Penniless (a) Broke (b) Reckless (c) Cranky (d) Wealthy SSC MTS-15/07/2022 Shift-I Ans. (a): Penniless (Impoverished), Synonym – Broke (Poor) Meaning of the other optionsReckless – Careless Cranky – Whimsical Wealthy – Rich 477. Select the most appropriate synonym of the given word. Plausible (a) irresistible (b) Irrational (c) Untenable (d) Credible SSC MTS-07/07/2022 Shift-I Synonyms

Ans. (d): Plausible (Believable), Synonym - Credible (Reliable) Meaning of the other optionsIrresistible – Unstable Irrational – Illogical Untenable – Changeable, Indefensible 478. Select the most appropriate synonym of the given word. Perilous (a) Dangerous (b) Impetus (c) Secure (d) Stimulus SSC CHSL 25.05.2022 Shift-I Ans. (a): Perilous (Imperil) Synonym- Dangerous (Hazardous) Meaning of the other optionsImpetus – Encouragement, Inspiration Secure – Safe Stimulus – Provocation, Encouragement 479. Select the most appropriate synonym of the given word. Pace (a) Target (b) Angle (c) Speed (d) Corner SSC CHSL 27.05.2022 Shift-III Ans. (c): Pace (Motion), Synonym - Speed (Momentum, Tempo) Meaning of the other optionsTarget - Aim Angle - Corner, Outlook Corner - Edge 480. Select the most appropriate synonym of the given word. Precarious (a) Stable (b) Strange (c) Insecure (d) Safe SSC Constable GD-24/11/2021 Shift-II Ans. (c): Precarious (Uncertain), Synonym - Insecure (Unsure) Meaning of the other optionsStable - Steady, Constant Strange - Bizarre, Weird Safe - Secure 481. Select the most appropriate synonym of the given word. Promotion (a) Demotion (b) Endearment (c) Elevation (d) Domination SSC Constable GD-06/12/2021 Shift-III Ans. (c): Promotion (Advancement, Rise), Synonym – Elevation (Upgrading) Meaning of the other optionsDemotion – Degradation Endearment – Affection Domination – Supremacy 482. Select the most appropriate synonym of the given word. Paranoid (a) Impeccable (b) Suspicious (c) Insignificant (d) Patronising SSC CHSL (Tier-I) –09/08/2021 (Shift-I)

419

YCT

Ans. (b): Paranoid - Synonym - Suspicious Meaning of other words– Patronising - Protect, Impeccable - Errorless Insignificant - Trivial 483. Select the most appropriate synonym of the given word. PRODIGAL (a) talented (b) modest (c) intelligent (d) wasteful SSC CPO-SI – 25/11/2020 (Shift-I) Ans. (d): Prodigal- Synonym - Wasteful Meaning of other words– Talented - Genius, Modest - Humble, Intelligent - Wise 484. Select the most appropriate synonym of the given word. PERPLEX (a) deceive (b) bewilder (c) complex (d) surprise SSC CGL (Tier-I) – 11/06/2019 (Shift-I) Ans. (b): Perplex - Synonym ‘Bewilder’ Meaning of other words– Deceive-Cheat, Complex- Intricate, Surprise- Wonder 485. Select the most appropriate synonym of the given word. PRECARIOUS (a) dangerous (b) abundant (c) valuable (d) premature SSC CGL (Tier-I) – 10/06/2019 (Shift-I) Ans. (a): "Precarious"- Synonym – Dangerous Meaning of other words– Abundant - Copious Valuable - Precious Premature - Untimely 486. Select the most appropriate synonym of the given word. PENSIVE (a) Spontaneous (b) Spiteful (c) Reflective (d) Tragic SSC CGL (Tier-I) – 10/06/2019 (Shift-II) Ans. (c) Pensive - Synonym - ‘Reflective Meaning of other words– Spontaneous - Instincive, Spiteful - Malicious, Tragic Fierce 487. Select the synonym of the given word. PREVALENT (a) unusual (b) common (c) rare (d) different SSC CGL (Tier-I) – 07/06/2019 (Shift-I) Ans. (b) Prevalent - Synonym- 'Common'Meaning of other words– Unusual - Abnormal Different - Apart Rare - Strange 488. Select the most appropriate synonym of the given word. PARSIMONY (a) readiness (b) bounty (c) generosity (d) frugality SSC CPO-SI – 09/12/2019 (Shift-II) Ans. (d) Parsimony- Synonym - 'Frugality' Meaning of other words– Readiness- Alacrity, Bounty- Leniency, GenerosityInnocuity Synonyms

489. Select the most appropriate synonym of the given word. PERPETUATE (a) Release (b) Prevent (c) Cease (d) Preserve SSC CPO-SI – 11/12/2019 (Shift-II) Ans. (d) Perpetuate- Synonym - 'Preserve' Meaning of other words– Release- Discharge, Prevent- Stop, Cease - Intermit 490. Select the most appropriate synonym of the given word. PITFALL (a) haven (b) retreat (c) hazard (d) refuge SSC CHSL (Tier-I) –08/07/2019 (Shift-II) Ans. (c) Pitfall - Synonym - "Hazard" Meaning of other words– Haven- Shelter Element- Part Refuge- Resort 491. Select the most appropriate synonym of the given word. PATHETIC (a) powerful (b) plentiful (c) purposeful (d) pitiful SSC CHSL (Tier-I) –04/07/2019 (Shift-III) Ans. (d) Pathetic - Synonym - Pitiful Meaning of other words– Powerful- Strong Plentiful- Copious Purposeful- Tenacious 492. Select the most appropriate synonym of the given word. PEDIGREE (a) Mongrel (b) IIIegitimate (c) Lineage (d) Forbidden SSC CHSL (Tier-I) 21/10/2020 (Shift-II) Ans. (c): Pedigree - Synonym – Lineage Meaning of other words– Mongrel – Illegitimate Illegitimate – Lawless Forbidden – Taboo 493. Select the most appropriate synonym of the given word. PENURY (a) Poverty (b) Compatibility (c) Temporality (d) Readability SSC CHSL (Tier-I) 19/03/2020 (Shift-III) Ans. (a): Penury - Synonym – Poverty Meaning of other words– Compatibility- Fitness, Temporality – Secularism Readability – Legibility 494. Select the most appropriate synonym of the given word. Prodigious (a) immense (b) unimportant (c) proud (d) exciting SSC CHSL (Tier-I) 20/10/2020 (Shift-III) Ans. (a): Prodigious- Synonym - 'Immense' Meaning of other words– Unimportant - Small Proud - Haughty Exciting - Stimulant

420

YCT

495. Select the most appropriate synonym of the given word. PERSECUTION (a) Oppression (b) Perfection (c) Stimulation (d) Affliction SSC CHSL (Tier-I) 14/10/2020 (Shift-II) Ans. (a): Persecution– Synonym – Oppression Meaning of other words– Perfection – Fullness, Stimulation– Excitement, Affection– Friendship 496. Select the most appropriate synonym of the given word. PREJUDICE (a) Tolerance (b) Advantage (c) Bias (d) Justice SSC CHSL (Tier-I) 14/10/2020 (Shift-II) Ans. (c): Prejudice– Synonym – Bias Meaning of other words– Tolerance – Patience Advantage– Benefit Justice– Right 497. Select the most appropriate synonym of the given word. PLEASANT (a) Tedious (b) Tiresome (c) Refreshing (d) Exasperating SSC CHSL (Tier-I) 19/03/2020 (Shift-II) Ans. (c): Pleasant– Synonym – Refreshing Meaning of other words– Tedious– Sluggish, Exasperating– Odious, Tiresome– Exhausting 498. In the following question, out of the given four alternatives, select the one which best expresses the meaning of the given word. Proscribe (a) Admit (b) Ban (c) Praise (d) Include SSC MTS 7-10-2017 (Shift-I) Ans. (b): Proscribe– Synonym – Ban Meaning of other words– Praise– Admire, Include- Comprise, Admit- Concede 499. In the following question out of the given four alternatives, select the one which best expresses the meaning of the given word Perish (a) Energize (b) Revive (c) Refresh (d) Decease SSC MTS 9-10-2017 (Shift-III) Ans. (d) Perish - Synonym- 'Decease' Meaning of other words– Energize- Activate Revive- Enchant Refresh- Cheer 500. In the following question out of the given four alternatives, select the one which best expresses the meaning of the given word Panic (a) Calm (b) Contentment (c) Confidence (d) Frenzy SSC MTS 9-10-2017 (Shift-III) Ans. (d) Panic- Synonym- Frenzy Meaning of other words– Calm- Quiet, Contentment- Satisfaction, ConfidenceAdventure Synonyms

501. Select the most appropriate synonym of the given word. POPULAR (a) Particular (b) untidy (c) specific (d) favourite SSC MTS – 19/08/2019 (Shift-III) Ans. (d): 'Popular'- Synonym- Favourite Meaning of other words– Particular - Special Untidy - Dirty Specific - Distinct 502. Select the most appropriate synonym of the given word. Proclaim (a) Oppose (b) Produce (c) Declare (d) Conceal SSC MTS – 07/08/2019 (Shift-III) Ans. (c): Proclaim - Synonym – Declare Meaning of other words– Oppose - Stand Produce - Yield Conceal - Hide 503. Select the most appropriate synonym of the given word Persuade (a) Halt (b) Deplete (c) Coax (d) Impose SSC MTS – 21/08/2019 (Shift-I) Ans. (c) Persuade - Synonym – Coax Meaning of other words– Halt - Stage Deplete - Deplenish Impose - Inflict 504. Select the most appropriate synonym of the given word. Prodigy (a) Anger (b) Tranquil (c) Genius (d) Plight SSC MTS – 21/08/2019 (Shift-II) Ans. (c): Prodigy - Synonym – Genius Meaning of other words– Anger - Rage, Tranquil - Calm, Plight - Hell 505. Select the most appropriate synonym of the given word. Profuse (a) Thrifty (b) Scarce (c) Plenty (d) Sparse SSC MTS – 06/08/2019 (Shift-III) Ans. (c): Profuse - Synonym - 'Plenty' Meaning of other words– Thrifty - Frugal, Scarce - Less, Sparse - Rare 506. Select the synonym of the given word. PRECISE (a) Ambiguous (b) Inaccurate (c) Accurate (d) Erroneous SSC GD – 08/03/2019 (Shift-II) Ans. (c): Precise - Synonym – Accurate Meaning of other words– Ambiguous - Vague, Inaccurate - Wrong, Erroneous - Incorrect

421

YCT

Q

R

507. Select the most appropriate synonym of the given word. Queer (a) Strange (b) Expedite (c) Ordinary (d) Valid SSC GD 01/02/2023 (Shift-II) Ans. (a) : 'Strange' (weird) is the most appropriate synonym of the given word 'Queer'. The meaning of the other options is– Expedite :- hasten, speed up the progress. Ordinary :- common. Valid :- legitimate Hence option (a) is correct answer. 508. Select the most appropriate synonym of the given word. QUEST (a) search (b) path (c) destination (d) route SSC CHSL (Tier-I) –09/07/2019 (Shift-I) Ans. (a): Quest- Synonym – Search Meaning of other words– Destination- Target Route- Path Path- way 509. Select the synonym of the given word. Quarrelsome (a) Pragmatic (b) Querulous (c) Ambiguous (d) Opaque SSC CHSL (Tier-I) 19/03/2020 (Shift-I) Ans. (b): 'Quarrelsome' - Synonym - 'Querulous' Meaning of other words– Pragmatic - Practical, Ambiguous - Vague, Opaque Implicit 510. Select the most appropriate synonym of the given word. Quell (a) Incite (b) Aggravate (c) Soothe (d) Raze SSC CGL (Tier-I) – 07/06/2019 (Shift-III) Ans. (c): Quell - Synonym - Soothe Meaning of other words– Incite - Agitate, Aggravate - Foment, Raze – Tear 511. Select the most appropriate synonym of the given word. Quench (a) Quell (b) Review (c) Persuade (d) Extinguish SSC CHSL (Tier-I) 19/03/2020 (Shift-II) Ans. (d): Quench - Synonym - Extinguish Meaning of other words– Quell - Soothe, Review - Observe, Persuade - Coax 512. Select the most appropriate synonym of the given word. Quotidian (a) Unsure (b) reprimand (c) Ordinary (d) Query SSC MTS – 19/08/2019 (Shift-III) Ans. (c): Quotidian - Synonym - Ordinary Meaning of other words– Unsure - Unstable, Reprimand - Rebuke, Query Question

513. Select the most appropriate synonym to substitute the bracketed word in the given sentence. In the twentieth century, many (radical) intellectuals embraced the mass media. (a) sabbatical (b) egotistical (c) sceptical (d) fanatical SSC MTS 02/05/2023 (Shift-I) Ans. (d) : The most appropriate synonym to substitute the bracketed word 'Radical' is 'Fanatical' which means Dogmatic. Other words are____ Sabbatical means- vacation egotistical means- self centred sceptical means- dubious Thus, the correct answer is option (d). 514. Select the most appropriate synonym of the underlined word. Demand for water is also rapidly rising due to population growth, urbanisation and increasing pressures from the agriculture and energy sector. (a) leisurely (b) hopefully (c) promptly (d) luxuriously SSC CHSL (Tier-I) 10/08/2023 (Shift-I) Ans. (c) : Option (c) Promptly – (Immediately : without delay; Punctually) is synonym of the underlined word 'Rapidly' – (very quickly, at a great rate) given in the sentence. Meaning expressed by other options.– Leisurely – Slowly hopefully – Optimistically Luxuriously – Affluently 515. Select the most appropriate synonym of the given word. Resentment (a) Happiness (b) Calmness (c) Relaxation (d) Anger SSC CHSL (Tier-I) 02/08/2023 (Shift-I) Ans. (d) : The most appropriate synonym of the above given word 'Resentment' is 'Anger'. Meaning of other option – Happiness – Delight, pleasure, Calmness – peace, serenity, Relaxation – Rest, Remission, Hence, the correct option is (d) 516. Choose the word that means the same as the given word. 'Ridicule' (a) Pathology (b) Resolute (c) Derision (d) Derivation SSC CHSL (Tier-I) 09/03/2023 (Shift-IV) Ans. (c) : Ridicule , Synonym , 'Derision' Meaning of other wordPathology – The scientific study of diseases Resolute – Determined Derivation – The origin of something

Synonyms

422

YCT

517. Select the most appropriate synonym of the given word. Retrieve (a) Remove (b) Retain (c) Rescue (d) Recover SSC Constable GD-09/12/2021 Shift-II Ans. (d): Retrieve (Recapture), Synonym – Recover (Convalesce, Improve) Meaning of the other optionsRemove – Eliminate, Abolish Retain – Preserve, Keep Rescue – Save 518. Select the most appropriate synonym of the given word. Recommend (a) Adhere (b) Oppose (c) Approve (d) Reward SSC Constable GD-16/11/2021 Shift-III Ans. (c): Recommend (Suggest, Prescribe), Synonym Approve (Permit) Meaning of the other optionsAdhere - Abide Oppose - Resist Reward - Prize, Regalia 519. Select the most appropriate synonym of the given word. Refuge (a) Entrance (b) Shelter (c) Admit (d) Arrival SSC Constable GD-29/11/2021 Shift-II Ans. (b): Refuge (Asylum), Synonym - Shelter (Sanctuary) Meaning of the other optionsEntrance - Entry Admit - Accept Arrival - Advent 520. Select the most appropriate synonym of the given word. Random (a) Definite (b) Continuous (c) Systematic (d) Chance SSC MTS-06/07/2022 Shift-I Ans. (d): Random (Haphazard), Synonym - Chance (Coincidence) Meaning of the other optionsDefinite – Certain Continuous – Uninterrupted Systematic – Orderly 521. Select the most appropriate synonym of the given word. Retaliate (a) Facilitate (b) react (c) Rotate (d) Clap SSC CGL (Tier-I) 11/04/2022 Shift-I Ans. (b): Retaliate (Respond), Synonym – React (Act, Behave) Meaning of the other optionsFacilitate – Ease Rotate – Revolve Clap – Applaud Synonyms

522. Select the most appropriate synonym of the given word. Redundant (a) Superfluous (b) Ignorant (c) Essential (d) Arrogant SSC CGL (Tier-I) 12/04/2022 Shift-III Ans. (a): Redundant (Noodles), Synonym - Superfluous (Unnecessary) Meaning of the other optionsIgnorant- Unaware, Unwise Essential- Necessary, Compulsory Arrogant - Conceited, Haughty 523. Select the most appropriate synonym of the given word. Retaliate (a) Rotate (b) react (c) Clap (d) Facilitate SSC CGL (Tier-I) 21/04/2022 Shift-III Ans. (b): Retaliate (Respond), Synonym – React (Act, Behave) Meaning of the other optionsRotate - Revolve, Spin Clap - Applaud Facilitate - Ease 524. Select the most appropriate synonym of the given word. Ravenous (a) Satiated (b) Starved (c) Full (d) Content SSC CGL (Tier-I) 20/04/2022 Shift-III Ans. (b): Ravenous (Various), Synonym – Starved (Famished) Meaning of the other optionsSatiated – Satisfy Full – Brimming, Entire Content – Gratified 526. Select the most appropriate synonym of the given word. Resplendent (a) Dazzling (b) Withered (c) Dull (d) Unattractive SSC CHSL (Tier-I) –16/04/2021 (Shift-I) Ans. (a): Resplendent - Synonym - Dazzling Meaning of other words– Withered - Hipped, Dull - Sluggish, Unattractive Wierd 527. Select the most appropriate synonym of the given word. RELEASE (a) Acquit (b) Retain (c) Confine (d) Restrain SSC Stenographer (Grade C & D) 15.11.2021 Shift-I Ans. (a): Release (Free, Emancipation), Synonym– Acquit (Exonerate, Absolve) Meaning of the other optionsRetain - Preserve, Sustain Confine - Bound Restrain - Limit

423

YCT

528. Select the most appropriate synonym of the given word. Regime (a) Space (b) Country (c) Rule (d) Territory SSC CGL (Tier-I) –13/08/2021 (Shift-I) Ans. (c): Regime - Synonym - Rule Meaning of other words– Space - Place, Country - Province, Territory - Area 529. Select the most appropriate synonym of the given word : REVIVE (a) Review (b) Depress (c) Restore (d) Damage SSC CGL (Tier-II) 16/11/2020 (3 pm - 5 pm) Ans. (c): Revive – Synonym – Restore Meaning of other words– Review - Observe Depress - Press Damage - Injury 530. Select the most appropriate synonym of the given word. RAMIFICATION (a) Source (b) Consequence (c) Satisfaction (d) Inception SSC CGL (Tier-II) 16/11/2020 (3 pm - 5 pm) Ans. (b): 'Ramification' Synonym – Consequence Meaning of other words– Source - Origin Satisfaction - Content Inception - Threshold 531. Select the most appropriate synonym of the given word. Robust (a) fragile (b) healthy (c) incapable (d) delicable SSC Sel. Post Phase-VII (M.L.) 15.10.2019 (Shift-I)

Ans. (b): Robust - Synonym- Healthy Meaning of other words– Fragile - Delicate Incapable - Unable Delicable - Pleasant 532. Select the most appropriate synonym of the given word. ROUGH (a) Coarse (b) Soft (c) Foul (d) Feathery SSC Sel. Post Phase-VIII (H.L.) 09.11.2020 (Shift-I)

Ans. (a): Rough - Synonym Coarse Meaning of other words– Soft - Bland Foul - Muddy Feathery - Plumose 533. Select the most appropriate synonym of the given word. RECEIVE (a) Drop (b) Pass (c) Get (d) Lose SSC Sel. Post Phase-VIII (M.L.) 09.11.2020 (Shift-III)

Ans. (c): Receive– Synonym – Get Meaning of other words– Drop - Down Pass - Side Lose - Miss Synonyms

534.

Select the most appropriate synonym of the given word. RENOWN (a) obscurity (b) fame (c) wisdom (d) conceit SSC CGL (Tier-I) – 12/06/2019 (Shift-I) Ans. (b) 'Renown' - Synonym - 'Fame' Meaning of other words– Obscurity - Darkness Wisdom - Intelligence Conceit - Pride 535. Select the most appropriate synonym of the given word. RETICENT (a) silent (b) garrulous (c) extrovert (d) confident SSC CGL (Tier-I) – 07/06/2019 (Shift-III) Ans. (a): Reticent- Synonym -'Silent' Meaning of other words– Garrulous – Talkative Extrovert – Extrorse Confident – Guaranteed 536. Select the most appropriate synonym of the given word. REPUDIATE (a) enforce (b) sanction (c) regret (d) renounce SSC CGL (Tier-I) – 10/06/2019 (Shift-III) Ans. (d): Repudiate - Synonym -'Renounce' Meaning of other words– Enforce- Apply Sanction- Limit Regret- Deplore 537. Select the most appropriate synonym of the given word. REDUNDANT (a) uninterested (b) unsuitable (c) unnecessary (d) unparalleled SSC CPO-SI – 11/12/2019 (Shift-I) Ans. (c) Redundant - Synonym – Unnecessary Meaning of other words– UninterestedTedious, UnsuitableImproper, Unparalleled - Peculiar 538. Select the most appropriate synonym of the given word. REPRIMAND (a) rebuke (b) complete (c) command (d) repeat SSC CPO-SI – 13/12/2019 (Shift-II) Ans. (a): Reprimand- Synonym –Rebuke Meaning of other words– Complete- Finished, Command- Mandate, RepeatRemake 539. Select the most appropriate synonym of the given word. RADIANCE (a) depth (b) dryness (c) sparkle (d) redness SSC CHSL (Tier-I) –10/07/2019 (Shift-II) Ans. (c) Radiance- Synonym - 'Sparkle' Meaning of other wordsDepth- Deepness Dryness- Aridity Redness- Glow

424

YCT

540. Select the most appropriate synonym of the given word. RECAPITULATION (a) summary (b) movement (c) prominence (d) readiness SSC CHSL (Tier-I) –10/07/2019 (Shift-II) Ans. (a): Recapitulation- Synonym - 'Summary' Meaning of other words– Movement- Agitation Prominence- Salience Readiness- Alacrity 541. Select the most appropriate synonym of the given word. RECEDE (a) increase (b) intensify (c) waver (d) wane SSC CHSL (Tier-I) –10/07/2019 (Shift-III) Ans. (d) Recede- Synonym - 'Wane' Meaning of other words– Increase- Rise Intensify- Severity Waver- Uncurl 542. Select the most appropriate synonym of the given word. RESOLUTE (a) unsure (b) determined (c) uncertain (d) hesitant SSC CHSL (Tier-I) –08/07/2019 (Shift-I) Ans. (b) Resolute- Synonym - 'Determined' Meaning of other words– Unsure- Dickey Uncertain- Suspect Hesitant- Indecisive 543. Select the most appropriate synonym of the given word. REPRIMAND (a) eradicate (b) reproach (c) applaud (d) reward SSC CHSL (Tier-I) –05/07/2019 (Shift-III) Ans. (b) Reprimand- Synonym - 'Reproach' Meaning of other words– Eradicate- Destroy Applaud- Appreciate Reward- Prize 544. Select the most appropriate synonym of the given word. Reproach (a) Indulge (b) Berate (c) Supervise (d) Train SSC CHSL (Tier-I) 26/10/2020 (Shift-II) Ans. (b): Reproach - Synonym- Berate Meaning of other words– Indulge - Full, Supervise - Direct, Train – Teach 545. Select the most appropriate synonym of the given word. REMORSE (a) Reproach (b) Affection (c) Regret (d) Indifference SSC CHSL (Tier-I) 19/10/2020 (Shift-I) Ans. (c): 'Remorse'- Synonym - Regret Meaning of other words– Reproach – Disdain Affection – Love Indifference – Apathy Synonyms

546. Select the most appropriate synonym of the given word. RIND (a) Shade (b) Peel (c) Petal (d) Leaf SSC CHSL (Tier-I) 13/10/2020 (Shift-III) Ans. (b) 'Rind'- Synonym – Peel Meaning of other words– Shade – Image Petal – Leaf Leaf – Vane 547. Select the most appropriate synonym of the given word. RESILIENT (a) Baseless (b) Lazy (c) Furious (d) Supple SSC CHSL (Tier-I) 17/03/2020 (Shift-III) Ans. (d): 'Resilient'- Synonym - 'Supple' Meaning of other words– Baseless – Unfounded Lazy – Sluggish Furious - Angry 548. Select the most appropriate synonym of the given word. ROTUND (a) Range (b) Rolled (c) Rocky (d) Round SSC CHSL (Tier-I) 14/10/2020 (Shift-III) Ans. (d): Rotund- Synonym – Round Meaning of other words– Rang– Jiggle, Rocky– Petrous, Rolled - Round 549. Select the most appropriate synonym of the given word. ROWDY (a) Charming (b) Boisterous (c) Calm (d) Amenable SSC CHSL (Tier-I) 16/10/2020 (Shift-II) Ans. (b): 'Rowdy'- Synonym - 'Boisterous' Meaning of other words– Charming - Lovely, Calm – Soothe, Amenable Responsible 550. Select the most appropriate synonym of the given word. Recently (a) Lately (b) Normally (c) Immediately (d) Usually SSC MTS – 20/08/2019 (Shift-II) Ans. (a): Recently- Synonym- 'Lately' Meaning of other words– Normally - Commonly Immediately - Quickly Usually - Ordinary 551. Select the most appropriate synonym of the given word. Revive (a) Repeat (b) Review (c) Ruin (d) Recover SSC MTS – 14/08/2019 (Shift-I) Ans. (d): 'Revive' - Synonym - 'Recover' Meaning of other words– Repeat- Duplicate, Review - Observe, Ruin - Waste

425

YCT

552. Select the most appropriate synonym of the given word. RELEVANT (a) Unconnected (b) Applicable (c) Inapplicable (d) Unrelated SSC MTS – 13/08/2019 (Shift-II) Ans. (b): Relevant- Synonym – Applicable Meaning of other words– Unconnected - Irrelevant Inapplicable - Inappropriate Unrelated - Unconnected 553. Select the synonym of the given word. Remember (a) Recall (b) Retell (c) Review (d) Reign SSC MTS – 09/08/2019 (Shift-II) Ans. (a): Remember - Synonym – Recall Meaning of other words– Retell - Repeat Review - Observe Reign - Rule 554. Select the most appropriate synonym of the given word. Rectify (a) Folly (b) Amend (c) Raid (d) Bend SSC MTS – 06/08/2019 (Shift-III) Ans. (b): Rectify - Synonym – Amend Meaning of other words– Folly - Stupidity, Raid - Assail, Bend - Turn

S 555. Choose the word that means the same as he given word. Soar (a) Sweet (b) Fly (c) Bitter (d) Sour SSC CHSL (Tier-I) 21/03/2023 (Shift-II) Ans. (b) : Soar - Synonym - fly Meaning of other wordsBitter – Spiteful Sour – Acid Sweet – Lovely 556. Choose the word that means the same as the given word. Sacrilege (a) Blasphemy (b) Modification (c) Consecration (d) Reverence SSC CHSL (Tier-I) 09/02/2023 (Shift-II) Ans. (a) : 'Sacrilege' - Synonym, "Blasphemy" Meaning of other wordsConsecration – Purification Modification – Customize Reverence – Venerate, Admire 557. Choose the word that means the same as the given word. Slick (a) Novice (b) Fractions (c) Slippery (d) New SSC CHSL (Tier-I) 09/02/2023 (Shift-II) Synonyms

Ans. (c) : Slick - Synonym - Slippery. Other options have different meaningNew – Fresh Fractions – Post Novice – Beginner 558. Select the appropriate synonym for the underlined word. In the absence of miracles, my frustration spikes. (a) Increases (b) Falls (c) Losses (d) Digs SSC MTS 04/05/2023 (Shift-II) Ans. (a) : The most appropriate synonym for the underlined word 'spikes' is 'Increases'. while other options are incorrect. Falls Plunge Losses Dropping Digs Hollow Thus, the correct answer is option (a). 559. Select the appropriate synonym for the underlined word. The story shows the importance of self-esteem. (a) pride (b) humility (c) disgrace (d) shyness SSC MTS 15/05/2023 (Shift-I) Ans. (a) : The appropriate synonym for the underlined word 'Self-esteem' is 'pride'. Meaning of other options – Humility :- Delicacy, Disgrace :- Offence, Shyness :- Shame, 560. Select the most appropriate synonym of the underlined word in the following sentence. Biosphere reserves are ‘learning places for sustainable development’. (a) maintainable (b) unsuitable (c) temporary (d) brief SSC Selection Posts XI-27/06/2023 (Shift-I) Ans. (a) : The most appropriate meaning of the above underlined word 'sustainable' is 'maintainable'. Other options give different meanings – Unsuitable – Improper, Temporary – Streaky, Brief – Concise 561. Select the most appropriate synonym of the underlined word. The investment scheme had been a scam all along. (a) Plain (b) Successful (c) Failure (d) Swindle SSC CGL (Tier-I) 20/07/2023 (Shift-I) Ans. (d) : The most appropriate Synonym of the underlined word ‘Scam’ is ‘Swindle’. The meaning of the others words is–– Plain means – apparent, evident, Successful means – Accomplished Failure means – Defeat, loss. Thus, the correct answer is option (d).

426

YCT

562. Select the most appropriate synonym of the underlined word. The hunter drew his bow, but only managed to sever the paw of the wolf before the wolf ran off. (a) join (b) ramify (c) bind (d) mix SSC CGL (Tier-I) 20/07/2023 (Shift-I) Ans. (b) : The most appropriate Synonym of the underlined word ‘Sever’ is 'ramify’. The meaning of other words is– Join means – Unite, Combine Bind means – Secure, Attach Mix means – Intermingle. Thus, The correct answer is option (b). 563. Select the most appropriate synonym of the given word. Smooth (a) Flat (b) Rough (c) Uneven (d) Broken SSC CGL (Tier-I) 11/04/2022 Shift-III Ans. (a): Smooth (Flat, Glib), Synonym - Flat (Plan, Level) Meaning of the other optionsRough - Coarse Uneven - Irregular Broken - Split 564. Select the most appropriate synonym of the given word. Succulent (a) Juicy (b) Foul (c) Decent (d) Coarse SSC CGL (Tier-I) 13/04/2022 Shift-II Ans. (a): Succulent (Lush), Synonym - Juicy (Mellow) Meaning of the other optionsFoul - Stench Decent - Civilized Coarse - Rough 565. Select the most appropriate synonym of the given word. Sacred (a) Consecrated (b) Submissive (c) Passionate (d) Adamant SSC MTS-07/07/2022 Shift-I Ans. (a): Sacred (Pious), Synonym - Consecrated (Holy) Meaning of the other optionsSubmissive – Humble Passionate – Enthusiastic, Zealous Adamant – Firm, Resolute 566. Select the most appropriate synonym of the given word. Segregate (a) Submerge (b) Isolate (c) Nullify (d) Integrate SSC MTS-06/07/2022 Shift-II Ans. (b): Segregate (Separate), Synonym - Isolate (Dissociate) Synonyms

Meaning of the other optionsSubmerge – Overflow, Sink Nullify – Invalidate Integrate – Untie, Join 567. Select the most appropriate synonym of the given word. Solemn (a) Dignified (b) Frivolous (c) Excited (d) Trivial SSC CGL (Tier-I) 13/04/2022 Shift-I Ans. (a): Solemn (Serious), Synonym- dignified (Conceited, Serious) Meaning of the other optionsFrivolous –Insignificant Excited – Agitated Trivial – Insignificant 568. Select the most appropriate synonym of the given word. Squawk (a) Suggest (b) Explore (c) Scream (d) Connote SSC CGL (Tier-I) 18/04/2022 Shift-I Ans. (c): The most appropriate synonym of the word ‘Squawk’ (Croak), is 'Scream' (Yell, Shout) Meaning of the other optionsSuggest - Advice Explore - Discover Connote - Declare 569. Select the most appropriate synonym of the word. Sagacious (a) Delicious (b) Precious (c) Judicious (d) Malicious SSC CGL (Tier-I) 21/04/2022 Shift-III Ans. (c): Sagacious (Brilliant), Synonym – Judicious (Sensible, Right) Meaning of the other optionsDelicious – Tasty Precious – Valuable Malicious – Spiteful 570. Select the most appropriate synonym of the given word. Startle (a) Scare (b) Humiliate (c) Shame (d) Surprise SSC CHSL 30.05.2022 Shift-I Ans. (d): Startle (Frighten/Intimidate), SynonymSurprise (Astonish, Amaze) Meaning of the other optionsScare - Terrify Humiliate - Embarrassed Shame - Shyness, Bashful 571. Select the most appropriate synonym of the given word. Sanguinity (a) Despondence (b) Distrust (c) Acrimony (d) Optimism SSC CHSL 26.05.2022 Shift-III

427

YCT

Ans. (d): Sanguinity (Optimism), Synonym- Optimism (Hopeful) Meaning of the other optionsDespondence – Hopeless, Dejected Distrust – Doubt, Suspicion Acrimony – Bitterness 572. Select the most appropriate synonym of the given word. Serendipity (a) Godsend (b) Surreptitious (c) Catastrophe (d) Sibilance SSC CHSL 25.05.2022 Shift-I Ans. (a): Serendipity (Luck), Synonym- Godsend (Blessing) Meaning of the other wordsSurreptitious – Larcenous Catastrophe – Destruction, Calamity Sibilance– Hoot, Catcall 573. Select the most appropriate synonym of the given word. Scared (a) Sacred (b) Afraid (c) Tranquil (d) Solemn SSC CHSL 27.05.2022 Shift-II Ans. (b): Scared (Afraid), Synonym- Afraid (Fearful) Meaning of the other optionsSacred - Pious Tranquil - Quiet Solemn - Serious 574. Select the most appropriate synonym of the given word. Stationary (a) Portable (b) Movable (c) Fixed (d) Paper SSC Stenographer (Grade C & D) 11.11.2021 Shift-II Ans. (c): Stationary (Stable), Synonym-Fixed (Steady) Meaning of the other optionsPortable - Movable, Transferable Movable - Adjustable Paper - Blank Shut 575. Select the most appropriate synonym of the given word. Stun (a) Trick (b) Jest (c) Joke (d) Shock SSC Constable GD-17/11/2021 Shift-II Ans. (d): Stun (Daze, Amaze), Synonym - Shock (Surprise, Jerk) Meaning of the other optionsTrick - Deceive Jest - Joke, Laugh Joke - Mockery, Banter 576. Select the most appropriate synonym of the given word. Substitution (a) Logical (b) Hold (c) Exchange (d) Give SSC Constable GD-16/11/2021 Shift-III Synonyms

Ans. (c): Substitution (Replacement), Synonym Exchange (Swap) Meaning of the other optionsLogical - Rational Hold - Catch Give - Grant, Provide 577. Select the most appropriate synonym of the given word. Sufficient (a) Enough (b) Large (c) Deficient (d) Lacking SSC Constable GD-14/12/2021 Shift-II Ans. (a): Sufficient (Adequate), Synonym - Enough (Ample) Meaning of the other optionsLarge - Big, Huge Deficient - Lacking, Insufficient Lacking - Inadequate 578. Select the most appropriate synonym of the given word. Sanction (a) Degrade (b) Reward (c) Permit (d) Cancel SSC Constable GD-10/12/2021 Shift-III Ans. (c): Sanction (Approval), Synonym - Permit (Allow) Meaning of the other optionsDegrade- Demean Reward- Award, Prize Cancel- Abolish, Finish 579. Select the most appropriate synonym of the given word. SUPPORTING (a) disagreeing (b) applauding (c) arguing (d) agreeing SSC Stenographer (Grade C & D) 11.11.2021 Shift-I Ans. (d): Supporting (Helpful), Synonym - Agreeing (Consent, Accede) Meaning of the other optionsDisagreeing - Dissent, Discard Applauding - Praise, Extol Arguing - Fight, Dispute 580. Select the most appropriate synonym of the given word. SPONTANEOUS (a) impulsive (b) prejudiced (c) planned (d) intended SSC CPO-SI – 25/11/2020 (Shift-I) Ans. (a): Spontaneous - Done naturally without being forced, Synonym - Impulsive Meaning of other words– Prejudiced - Partisan, Intended – Purpose, Planned Organize 581. Select the most appropriate synonym of the given word. SHOVE (a) pull (b) halt (c) shout (d) poke SSC CPO-SI – 24/11/2020 (Shift-II)

428

YCT

Ans. (d): 'Shove'- Synonym – Poke Meaning of other words– Pull - Draw Halt - Stage Shout - Clamor 582. Select the most appropriate synonym of the given word. SUMPTUOUS (a) opulent (b) stingy (c) meager (d) bereft SSC CPO-SI – 24/11/2020 (Shift-I) Ans. (a): 'Sumptuous'- Synonym- 'Opulent' Meaning of other words– Stingy - Frugal, Meager - Scanty, Bereft - Loop 583. Select the most appropriate synonym of the given word. STERILE (b) barren (a) productive (c) sordid (d) pure SSC CGL (Tier-I) – 10/06/2019 (Shift-I) Ans. (b) Sterile - Synonym - 'Barren' Meaning of other words– Productive - Fertile Sordid - Odious Pure - Refined 584. Select the synonyms of the given word. RAMPANT (a) excessive (b) limited (c) rare (d) gentle SSC CGL (Tier-I) – 07/06/2019 (Shift-II) Ans. (a): Rampant - Synonym - 'Excessive' Meaning of other words– Limited - Finited Rare - Scantily Gentle - Noble 585. Select the most appropriate synonym of the given word. SUBSEQUENT (a) consecutive (b) preceding (c) antecedent (d) prior SSC CPO-SI – 09/12/2019 (Shift-I) Ans. (a): Subsequent - Synonym- 'Consecutive' Meaning of other words– Preceding - Precede, Antecedent - Prior, Prior - Former 586. Select the most appropriate synonym of the given word. SCANTY (a) adequate (b) meager (c) plentiful (d) enough SSC CPO-SI – 11/12/2019 (Shift-I) Ans. (b) Scanty - Synonym – Meagre Meaning of other words– Adequate- Sufficient, Plentiful - Abundant, Enough Adequate 587. Select the most appropriate synonym of the given word. SANCTION (a) grace (b) mercy (c) denial (d) approval SSC CPO-SI – 12/12/2019 (Shift-I) Synonyms

Ans. (d) Sanction - Synonym – Approval Meaning of other words– Grace - Favor Mercy - Clemency Denial - Reject 588. Select the most appropriate synonym of the given word. SURREPTITIOUS (a) spacious (b) adventurous (c) secretive (d) aggressive SSC CPO-SI – 13/12/2019 (Shift-II) Ans. (c): Surreptitious - Synonym - 'Secretive' Meaning of other words– Aggressive - Rampant, Spacious - Detailed, Adventurous - Daring 589. Select the most appropriate synonym of the given word. STERN (a) strict (b) mellow (c) proud (d) shrewd SSC CHSL (Tier-I) –05/07/2019 (Shift-I) Ans. (a): 'Stern'- Synonym - "Strict" Meaning of other words– Mellow- Tuneful Proud- Arrogant Shrewd- Canny 590. Select the most appropriate synonym of the given word. SORDID (a) pure (b) solid (c) clean (d) dirty SSC CHSL (Tier-I) –05/07/2019 (Shift-III) Ans. (d) 'Sordid'- Synonym -'Dirty' Meaning of other words– Pure- Refined Solid- Concrete Clean- Neat 591. Select the most appropriate synonym of the given word. Scrap (a) Hoard (b) Save (c) Store (d) Reject SSC CHSL (Tier-I) –04/07/2019 (Shift-I) Ans. (d): Scrap- Synonym - 'Reject' Meaning of other words– Hoard- Gather Save- Protect Store- Stock 592. Select the most appropriate synonym of the given word. SANE (a) sensible (b) crazy (c) sensitive (d) foolish SSC CHSL (Tier-I) –04/07/2019 (Shift-II) Ans. (a): Sane- Synonym - 'Sensible' Meaning of other words– Crazy- Slight Sensitive- Emotional Foolish- Absurd 593. Select the most appropriate of the given word. SUCCINCT (a) pithy (b) terse (c) lengthy (d) curt SSC CHSL (Tier-I) –02/07/2019 (Shift-I)

429

YCT

Ans. (b): Succinct - Synonym- Terse Meaning of other words– Pithy- Pregnant, Curt- Rude, Lengthy - Detailed 594. Select the most appropriate synonym of the given word. SURGE (a) Rush (b) Smooth (c) Hygienic (d) Clean SSC CHSL (Tier-I) 17/03/2020 (Shift-II) Ans. (a): 'Surge' - Synonym- Rush Meaning of other words– Smooth - Sleek Hygienic - Sanative Clean - Neat 595. Select the most appropriate synonym of the given word. SALVAGE (a) Lose (b) Hurt (c) Waste (d) Save SSC CHSL (Tier-I) 20/10/2020 (Shift-I) Ans. (d): 'Salvage'- Synonym - 'Save' Meaning of other words– Lose - Miss, Hurt - Wound, Waste - Spend 596. In the following question, out of the given four alternatives, select the one which best expresses the meaning of the given word. Scion (a) Root (b) Cause (c) Heir (d) Source SSC MTS 7-10-2017 (Shift-I) Ans. (c) Scion – Synonym - Heir Meaning of other words– Root– Foundation, Source– Origin, Cause– Reason 597. In the following question, out of the given four alternatives, select the one which best expresses the meaning of the given word. Shriek (a) Peep (b) Toot (c) Yell (d) Whisper SSC MTS 7-10-2017 (Shift-I) Ans. (c) Shriek – Synonym – Yell Meaning of other words– Peep– Glimpse, Toot– Siren, Whisper– Murmur 598. In the following question, out of the given four alternatives, select the one which best expresses the meaning of the given word. Scream (a) Cry (b) Whisper (c) Hum (d) Murmur SSC MTS 9-10-2017 (Shift-II) Ans. (a): Scream - Synonym- Cry Meaning of other words– Whisper – Mumble, Hum – Murmur, Murmur – Whisper 599. In the following question, out of the given alternatives, select the one which best expresses the meaning of the given word. Smear (a) Discolor (b) Honor (c) Laud (d) Upgrade SSC MTS 9-10-2017 (Shift-I) Synonyms

Ans. (a): Smear - Synonym- Discolor Meaning of other words– Honor- Respect, Upgrade- Progress, Laud– Admire 600. In the following question, out of the given four alternatives select the one which best expresses the meaning of the given word. Smudge (a) Clean (b) Sanitary (c) Pure (d) Stain SSC MTS 10-10-2017 (Shift-III) Ans. (d): 'Smudge'- Synonym - 'Stain' Meaning of other words– Clean - Neat, Sanitary - Medicinal, Pure - Holy 601. In the following question, out of the given four alternatives, select the one which best expresses the meaning of the given word. Savage (a) Civilized (b) Brutal (c) Tame (d) Domesticated SSC MTS 10-10-2017 (Shift-II) Ans. (b): Savage – Synonym- Brutal Meaning of other words– Civilized – Decent, Tame – Pet, Domesticated –Tame 602. In the following question, out of the given four alternatives, select the one which best expresses the meaning of the given word. Subside (a) Descend (b) Enlarge (c) Prolong (d) Ascend SSC MTS 10-10-2017 (Shift-I) Ans. (a): Subside – Synonym – Descend Meaning of other words– Enlarge - Enhance, Prolong- Increase, Ascend- Climb 603. In the following question, out of given four alternatives, select the one which best expresses the meaning of the given word. Steady (a) Vulnerable (b) Flexible (c) Abiding (d) Wobbly SSC MTS 10-10-2017 (Shift-I) Ans. (c) Steady- Synonym- Abiding Meaning of other words– Vulnerable- Penetrable, Flexible- Resilient 604. In the following question, out of the given four alternatives, select the one which best expresses the meaning of the given word. Sneer (a) Mock (b) Compliment (c) Praise (d) Applaud SSC MTS 11-10-2017 (Shift-III) Ans. (a): Sneer- Synonym- Mock Meaning of other words– Compliment – Admiration Praise – Laud Applaud – Appreciate 605. In the following question, out of the given four alternatives, select the one which best expresses the meaning of the given word. Solemn (a) Funny (b) Pensive (c) Laughing (d) Light SSC MTS 11-10-2017 (Shift-III)

430

YCT

Ans. (b): Solemn –Synonym- Pensive Meaning of other words– Funny – Queer Laughing – Jest Light – Illumination 606. In the following question, out of the given four alternatives, select the one which best expresses the meaning of the given word. Scorn (a) Mockery (b) Approval (c) Delight (d) Pleasure SSC MTS 11-10-2017 (Shift-I) Ans. (a): Scorn - Synonym- 'Mockery' Meaning of other words– Approval - Sanction, Delight - Enjoyment, Pleasure 607. In the following question, out of the given four alternatives, select the one which best expresses the meaning of the given word. Sarcastic (a) Kind (b) Nice (c) Approved (d) Arrogant SSC MTS 11-10-2017 (Shift-I) Ans. (d) Sarcastic - Synonym- Arrogant Meaning of other words– Nice - Good, Kind - Species, Approve - Favor 608. Select the synonym of the given word. Swift (a) Slow (b) Harsh (c) Quick (d) Turn SSC MTS – 13/08/2019 (Shift-III) Ans. (c): 'Swift'- Synonym – Quick Meaning of other words– Slow - Sluggish Harsh - Rigid Turn - Bend 609. Select the most appropriate synonym of the given word. Strange (a) Despair (b) Abnormal (c) Conventional (d) Natural SSC MTS – 07/08/2019 (Shift-II) Ans. (b): Strange- Synonym – Abnormal Meaning of other words– Despair - Despond Conventional - Traditional Natural - Original 610. Select the synonym of the given word. SLENDER (a) Narrow (b) Wide (c) Slim (d) Weak SSC MTS – 09/08/2019 (Shift-III) SSC GD – 14/02/2019 (Shift-I) Ans. (c): Slender - Synonym – Slim Meaning of other words– Narrow - Parochial Wide - Broad Weak - Feeble Synonyms

T 611. Select the appropriate synonym for the underlined word. “Little Red Riding Hood” is a fairy tale. (a) story (b) fact (c) tail (d) document SSC MTS 16/05/2023 (Shift-II) Ans. (a) : The appropriate synonym for the underlined word- 'tale' is 'story'. Meaning of other optionsFact- Data, Certainty Tail- Rump, Tang Document- Deed, Script Thus, the correct answer is option (a). 612. Select the most appropriate synonym of the underlined word. It was due to treacherous leaders that he decided to leave the party. (a) Devoted (b) Transient (c) Ravenous (d) Disloyal SSC CGL (Tier-I) 19/07/2023 (Shift-IV) Ans. (d) : The most appropriate synonym of the underlined word 'treacherous' is 'Disloyal'. The meaning of other options is Devoted means - Dedicated Transient Means - Temporary, evanescent, Ravenous means -Hungry, famished. Thus, the correct answer is option (d) 613. Select the most appropriate synonym of the underlined word. The transient rains have caused harm to the crops in the north. (a) Long (b) Perpetual (c) Stable (d) Fleeting SSC CHSL (Tier-I) 09/08/2023 (Shift-III) Ans. (d) : The most appropriate synonym of the underlined word ' Transient – (lasting for a short period of time) will be option (d) 'Fleeting'. fleeting – ( Continuing for a very short time). Meaning of other options are – Perpetual – Continuing forever in the same way Stable – firmly/fixed 614. Read the given sentence carefully and fill in the blank by selecting the correct synonym of the word given in the brackets. The authority refused to pay any heed to the _________ demands made by the workers. (Trivial) (a) conceited (b) insignificant (c) majestic (d) idiotic SSC CGL Mains 26/10/2023 (Shift-I) Ans. (b) : In the above sentence, ''Insignificant' is the synonym of bracteted word 'Trivial', will be used in the blank space. Other options give different meaning. Conceited Arrogant Majestic Impressive Idiotic Silly Correct Sentence The authority refused to pay any heed to the Insignificant demands made by the workers.

431

YCT

615. Select the word that is closest in meaning (SYNONYM) to the word given below. THRIFTY (a) PRUDENT (b) WASTEFUL (c) GENEROUS (d) EXTRAVAGANT SSC CHSL (Tier-I) 20/03/2023 (Shift-I) Ans. (a) : Thrifty - Synonym - Prudent Meaning of other wordsGenerous- Liberal Wasteful- Hurtful Extravagant- Extraordinary 616. Select the most appropriate synonym of the given word. Taciturn (a) Verbose (b) Reserved (c) Lively (d) Noisy SSC CGL (Tier-I) 11/04/2022 Shift-II Ans. (b) : Taciturn (Reticent), Synonym–Reserved (Shy) Meaning of the other optionsVerbose – Talktive Lively– Interesting Noisy – Clamorous 617. Select the most appropriate synonym of the given word. Trendy (a) Foreign (b) Common (c) Popular (d) Familiar SSC CGL (Tier-I) 13/04/2022 Shift-III Ans. (c): Trendy (Stylish), Synonym- Popular (Famous) Meaning of the other optionsForeign - Alien, Exotic Common - General, Usual Familiar - Well-known, Friendly 618. Select the most appropriate synonym of the given word. Tranquil (a) Aware (b) Dazed (c) Alert (d) Calm SSC CGL (Tier-I) 19/04/2022 Shift-III Ans. (d): Tranquil (Calm), Synonym- Calm (Quiet) Meaning of the other optionsAware - Conscious Dazed - Baffled, Confused Alert - Warning, Caution 619. Select the most appropriate synonym of the given word. TRANQUIL (a) sober (b) worried (c) tempered (d) nervous SSC Stenographer (Grade C & D) 11.11.2021 Shift-II Ans. (a): Tranquil (Calm), Synonym- Sober (Quiet, serious) Meaning of the other optionsWorried - Anxious Tempered - Irritate, Angry Nervous - Restless Synonyms

620. Select the most appropriate of the given word. Toil (a) Drive (b) Sweat (c) Rest (d) Climb SSC Constable GD-07/12/2021 Shift-II Ans. (b): Toil (Labor), Synonym - Sweat (Struggle, Diligent) Meaning of the other optionsDrive – Operate, Ride Rest – Comfort, Relief Climb – Ascent 621. Select the most appropriate synonym of the given word. TENSION (a) peace (b) detachment (c) strain (d) comfort SSC Stenographer (Grade C & D) 15.11.2021 Shift-I Ans. (c): Tension (Stress), Synonym- Strain (Exertion) Meaning of the other optionsPeace - Silence, Calm Detachment - Isolation, Segregation Comfort - Rest, Ease 622. Select the most appropriate synonym of the given word. TIMID (a) confident (b) bold (c) courageous (d) humble SSC Stenographer (Grade C & D) 11.11.2021 Shift-II Ans. (d): Timid (Coy, Modest), Synonym-Humble (Meek, Submissive) Confident - Morale, Optimistic Bold - Daring, Brave Courageous - Dauntless, Bold 623. Select the most appropriate synonym of the given word. Transmit (a) Accept (b) Receive (c) Convey (d) Catch SSC CGL (Tier-I) –16/08/2021 (Shift-I) Ans. (c): Transmit - Synonym - Convey Meaning of other words– Accept - Admit, Receive - Get, Catch - Hold 624. Select the most appropriate synonym of the given word. Testify (a) Disprove (b) Affirm (c) Oppose (d) Invalidate SSC CGL (Tier-I) –18/08/2021 (Shift-I) Ans. (b): Testify - Synonym, Affirm Meaning of other words– Disprove - Rebut, Oppose - Stand, Invalidate - Cancel 625. Select the most appropriate synonym of the given word. Tag (n) (a) Syphon (b) Drain (c) Pump (d) Label SSC CGL (Tier-I) –20/08/2021 (Shift-I)

432

YCT

Ans. (d): Tag (n) - Synonym, Label (n) Meaning of other words– Syphon - Adsorb Drain (N) - Gutter Pump (N) - Syringe 626. Select the most appropriate synonym of the given word. Termination (a) Conviction (b) Conclusion (c) Consolation (d) Conduction SSC CGL (Tier-I) –17/08/2021 (Shift-I) Ans. (b): Termination - Synonym - Conclusion Meaning of other words– Conviction - Notion, Consolation - Solace, ConductionConductivity 627. Select the most appropriate synonym of the given word. TUMULT (a) clarity (b) peace (c) uproar (d) surprise SSC CPO-SI – 24/11/2020 (Shift-II) Ans. (c): 'Tumult'- Synonym - 'Uproar' Meaning of other words– Clarity - Lucidity Peace - Serenity Surprise - wonder 628. Select the most appropriate synonym of the given word. TACITURN (a) reticent (b) talkative (c) phlegmatic (d) placid SSC CGL (Tier-I) – 11/06/2019 (Shift-II) Ans. (a): Taciturn - Synonym - ‘Reticent’ Meaning of other words– Talkative- Garrulous, Phlegmatic- Sluggish, PlacidQuiet 629. Select the synonym of the given word. TRIUMPH (a) victory (b) fight (c) attack (d) peace SSC CGL (Tier-I) – 06/06/2019 (Shift-I) Ans. (a): Triumph - Synonym - 'Victory' Meaning of other words– Fight - Battle Attack - Invasion Peace - Calm 630. Select the synonym of the given word. TILT (a) Support (b) Cross (c) Straighten (d) Slant SSC CGL (Tier-I) – 04/06/2019 (Shift-II) Ans. (d) Tilt - Synonym - 'Slant' Meaning of other words– Support- Side Cross- Pass Straighten- Plumb 631. Select the most appropriate synonym of the given word. TRANQUIL (a) nervous (b) agitated (c) calm (d) wild SSC CPO-SI – 09/12/2019 (Shift-I) Synonyms

Ans. (c) Tranquil- Synonym - 'Calm' Meaning of other words– Nervous- Restless, Agitated - Excited, Wild - Wooded 632. Select the most appropriate synonym of the given word. THRIFTY (a) frugal (b) grand (c) rigid (d) discreet SSC CHSL (Tier-I) –05/07/2019 (Shift-I) Ans. (a): Thrifty - Synonym- 'Frugal' Meaning of other words– Grand - Huge Rigid - Stiff Discreet - Humble 633. Select the most appropriate synonym of the given word. TEMPERATE (a) moderate (b) temporary (c) tentative (d) extreme SSC CHSL (Tier-I) –05/07/2019 (Shift-II) Ans. (a): Temperate- Synonym- Moderate Meaning of other words– Temporary- Temporal Tentative- Test Extreme- Ultimate 634. Select the most appropriate synonym of the given word. THWART (a) impede (b) aid (c) support (d) face SSC CHSL (Tier-I) –03/07/2019 (Shift-III) Ans. (a): Thwart- Synonym- "Impede" Meaning of other words– Aid- Help Support- Side Face- Combat 635. Select the most appropriate synonym of the given word. TRUST (n) (a) Safety (b) Ability (c) Belief (d) Welfare SSC CHSL (Tier-I) 17/03/2020 (Shift-I) Ans. (c) 'Trust' - Synonym- Belief Meaning of other words– Safety - Secure Ability - Strength Welfare - Prosperity 636. Select the most appropriate synonym of the given word. TIMID (a) Strong (b) Willful (c) Kind (d) Shy SSC CHSL (Tier-I) 19/10/2020 (Shift-I) Ans. (d): 'Timid'- Synonym - 'Shy' Meaning of other words– Strong – Firm Willful – Deliberately Kind – Gracious 637. Select the most appropriate synonym of the given word. TOLERANCE (a) Aggression (b) Endurance (c) Oppression (d) Impatience SSC CHSL (Tier-I) 15/10/2020 (Shift-II)

433

YCT

Ans. (b): 'Tolerance'- Synonym- Endurance Meaning of other words– Aggression - Invasion, Oppression - Harassment, Impatience - Rashness 638. Select the most appropriate synonym of the given word. TARNISH (a) Heighten (b) Decorae (c) Damage (d) Appease SSC CHSL (Tier-I) 19/03/2020 (Shift-II) Ans. (c): Tarnish– Synonym - Damage Meaning of other words– Heighten– Increase Decorate– Spruce Appease– Coax 639. In the following question, out of the given four alternatives, select the one which best expresses the meaning of the given word. Tackle (a) Avoid (b) Forget (c) Neglect (d) Deal SSC MTS 10-10-2017 (Shift-III) Ans. (d): 'Tackle'– Synonym - 'Deal' Meaning of other words– Avail – Gain Forget – Delude Neglect – Slight 640. Select the most appropriate synonym of the given word. Tangled (a) Orderly (b) Regulated (c) Knotted (d) Organized SSC MTS – 07/08/2019 (Shift-III) Ans. (c): Tangled - Synonym – Knotted Meaning of other words– Orderly - Lackey Regulated - Rank Organized - Unified

U 641. Select the most appropriate synonym for the highlighted word. The government’s utilitarian stance was appreciated by all. (a) fanciful (b) useful (c) thrust (d) distasteful SSC CGL (Tier-I) 18/07/2023 (Shift-III) Ans. (b) : The most appropriate synonym of the above highlited word 'utilitarian' is 'useful'. Meaning of other options Fanciful - Imaginart, Thrust - Push, Propel Distasteful - Unpleasant, 642. Select the most appropriate synonym of the given word. Unbiased (a) Definitive (b) Objective (c) Subjective (d) Organizational SSC CGL (Tier-I) 18/07/2023 (Shift-III) Synonyms

Ans. (b) : The most appropriate synonym of the above word 'Unbiased' (impartial) is 'Objective (fair). Other options given different meaningDefinitive - Certain, Subjective - Personal, Organisational - Combinational 643. Select the most appropriate synonym of the given word. Urge (a) Refuse (b) Protest (c) Appeal (d) Reply SSC CGL (Tier-I) 12/04/2022 Shift-III Ans. (c) : Urge (Request), Synonym-Appeal (Cassation, Plead) Meaning of the other optionsRefuse- Deny Protest- Resist Reply- Answer 644. In the following question, out of the given four alternatives, select the one which best expresses the meaning of the given word. Urge (a) Depress (b) Discourage (c) Divert (d) Impulse SSC MTS 9-10-2017 (Shift-II) Ans. (d): Urge - Synonym – Impulse Meaning of other words– Depress – Deflect, Discourage – Threaten, Divert – Turn 645. In the following question, out of the given four alternatives, select the one which best expresses the meaning of the given word. Unanimous (a) Split (b) Undisputed (c) Cleaved (d) Separate SSC MTS 10-10-2017 (Shift-III) Ans. (b): 'Unanimous'- Synonym - 'Undisputed' Meaning of other words– Split - Divided, Cleaved - Break, Separate - Distinct 646. Select the most appropriate synonym of the given word. Uncanny (a) Strange (b) Unruly (c) Accurate (d) Harmful SSC CGL (Tier-I) – 04/06/2019 (Shift-II) Ans. (a): Uncanny - Synonym - Strange Meaning of other words– Unruly - Rampant, Accurate - Right, Harmful Detrimental 647. Select the most appropriate synonym of the given word. Uproar (a) Vague (b) Veil (c) Timid (d) Clamor SSC CHSL (Tier-I) 17/03/2020 (Shift-I) Ans. (d): Uproar - Synonym - Clamor Meaning of other words– Vague - Obscure, Veil - Cover, Timid - Sneaky

434

YCT

648. Select the most appropriate synonym of the given word. Upbeat (a) Adhere (b) Cheerful (c) Ignore (d) Detach SSC CHSL (Tier-I) –10/07/2019 (Shift-III) Ans. (b): Upbeat - Synonym - Cheerful Meaning of other words– Adhere - Abide, Ignore - Negligence, Detach Segregate

V

SSC Stenographer Grade C&D –05/02/2019 (3:5pm)

649. Select the most appropriate synonym to substitute the bracketed word in the given sentence. Confession had always rested on a clear distinction between and (venial) sins. (a) Intolerable (b) Major (c) Pardonable (d) Genial SSC MTS 18/05/2023 (Shift-I) Ans. (c) : The appropriate synonym of the given bracket word ‘Venial’ is ‘Pardonable’. Meaning of other optionsIntolerable – Unbearable, Major – Main, Staple, Genial – Gregorian, 650. Select the most appropriate synonym of the underlined word. After my first visit to Boston, I spent almost every winter in the North. Once I went on a visit to a New England village with its frozen lakes and vast snow fields. (a) solid (b) minimal (c) extensive (d) shallow SSC CGL (Tier-I) 21/07/2023 (Shift-II) Ans. (c) : The most appropriate synonym of the above underlined word 'vast' is 'extensive'. Meaning of other optionsSolid - Firm, Minimal - Least, Shallow - Undipped, Thus, the correct answer is option (c). 651. Select the word that is closest in meaning (synonym) to the word given below. VAULT (a) Ration (b) Crypt (c) Raisins (d) Monarch SSC CHSL (Tier-I) 15/03/2023 (Shift-I) Ans. (b) : Vault - Synonym - 'Crypt' Meaning of other wordsMonarch → Emperor Ration → Dietary Raisins → Dried fruit 652. Select the most appropriate synonym of the given word. Vertical (a) Upright (b) Parallel (c) Equal (d) Shaky SSC CGL (Tier-I) 11/04/2022 Shift-II Synonyms

Ans. (a): Vertical (Perpendicular), Synonym – Upright (Straight) Meaning of the other optionsParallel – Similar, Alike Equal – Identical Shaky – Unstable 653. Select the most appropriate synonym of the given word : VANQUISH (a) defeat (b) attack (c) victory (d) loss Ans. (a): 'Vanquish' – Synonym - Defeat Meaning of other words– Victory - Profit Attack - Invasion Loss - Harm 654. Select the most appropriate synonym of the given word. VINDICTIVE (a) Watchful (b) Revengeful (c) Forceful (d) Helpful SSC CGL (Tier-I) – 10/06/2019 (Shift-II) Ans. (b) ‘‘Vindictive” –Synonym- ‘Revengeful' Meaning of other words– Watchful – Attentive, Forceful - Potential, Helpful Auxiliary 655. Select the most appropriate synonym of the given word. VENERABLE (a) Valuable (b) Reliable (c) Redeemed (d) Esteemed SSC CPO-SI – 11/12/2019 (Shift-II) Ans. (d) Venerable - Synonym- Esteemed Meaning of other words– Reliable - Credible, Redeemed - Sanctify, Valuable Precious 656. Select the most appropriate synonym of the given word. VITAL (a) needless (b) insignificant (c) trivial (d) essential SSC CHSL (Tier-I) –08/07/2019 (Shift-II) Ans. (d) Vital- Synonym- "Essential" Meaning of other words– Needless - Unnecessary Insignificant - Niggard Trivial - Puny 657. Select the most appropriate synonym of the given word. VIGILANT (a) Negligent (b) Inattentive (c) Watchful (d) Careless SSC MTS – 14/08/2019 (Shift-III) Ans. (c): Vigilant- Synonym – Watchful Meaning of other words– Negligent– Careless Inattentive– Heedless Careless– Reckless

435

YCT

658. Select the synonym of the given word. VAGUE (a) Precise (b) Unclear (c) Incomplete (d) Definite SSC GD – 08/03/2019 (Shift-II) Ans. (b): Vague Synonym – Unclear Meaning of other words– Precise - Exact, Incomplete - Unfinished, Definite Sicker

W 659. Select the most appropriate synonym to substitute the bracketed word in the given sentence. It broke Eleanora's heart to see her daughter become that (wraith) of her former self. (a) Yore (b) Spirit (c) Being (d) Wail SSC MTS 19/05/2023 (Shift-III) Ans. (b) : The most appropriate synonym of the above given word ‘wraith’ is ‘spirit’. Meaning of other options – Yore – Antiquity, bygone, Being – Existence, Entity, Wail – Lament, Bowl, Thus, the correct answer is option (b). 660. Select the most appropriate synonym of the underlined word. I have thus far sketched the events of my life, but I have not shown how much I have depended on books not only for pleasure and for the wisdom they bring to all who read, but also for that knowledge which comes to others through their eyes and their ears. (a) clarity (b) insight (c) folly (d) emotion SSC CGL (Tier-I) 27/07/2023 (Shift-III) Ans. (b) : The most appropriate synonym of the above given underlined word 'wisdom' is 'insight'. Meaning of other optionsClarity - Lucidity, Fally - Stupidity, Emotion - Sensation. Thus, the correct answer is option (b). 661. Select the word that is closest in meaning (synonym) to the word given below. WHIMSICAL (a) WEAK (b) RESPECTFUL (c) HUMILIATED (d) FUNNY SSC CHSL (Tier-I) 10/03/2023 (Shift-III) Ans. (d) : Whimsical - Synonym - 'Funny' Meaning of other wordHumiliated – Disgraced Weak – Feeble Resepectful – Faithful Synonyms

662. Select the most appropriate SYNONYM of the given word. Wily (a) Frank (b) Friendly (c) Crafty (d) Honest SSC Constable GD-01/12/2021 Shift-I Ans. (c): Wily (Cunning), Synonym - Crafty (Clever) Meaning of the other optionsFrank - Candid, Generous Friendly - Amiable, Cordial Honest - Truthful, Genuine 663. Select the most appropriate synonym of the given word. WRY (a) straight (b) frank (c) forthright (d) crooked SSC CHSL (Tier-I) –04/08/2021 (Shift-I) Ans. (d):Wry - Synonym - Crooked Meaning of other words– Straight - Directly, Forthright - Genuine, Frank - Liberal 664. Select the most appropriate synonym of the given word. WRENCH (a) supply (b) wrest (c) reject (d) deter SSC CPO-SI – 24/11/2020 (Shift-I) Ans. (b): 'Wrench' (to pull or twist something), Synonym - 'Wrest' Meaning of other words– Supply - Provision, Reject - Denial, Deter – Delay 665. Select the most appropriate synonym of the given word. WARY (a) rash (b) inattentive (c) lax (d) alert SSC CHSL (Tier-I) –10/07/2019 (Shift-III) Ans. (d) Wary- Synonym - 'Alert' Meaning of other words– Rash- Reckless Inattentive- Careless Lax- Loose 666. Select the most appropriate synonym of the given word. Wizard (a) Vow (b) Magician (c) Wan (d) Vague SSC CPO-SI – 25/11/2020 (Shift-II) Ans. (b): Wizard - Synonym - Magician Meaning of other words– Vow - Oath, Wan - Pallid, Vague - Obscure 667. Select the most appropriate synonym of the given word. Woe (a) Yearn (b) Vie (c) Sorrow (d) Vigorous SSC CHSL (Tier-I) –02/07/2019 (Shift-II) Ans. (c): Woe - Synonym - Sorrow Meaning of other words– Yearn - Sulk, Vie - Emulate, Vigorous - Cogent

436

YCT

668. Select the most appropriate synonym of the given word. Witty 674. Select the most appropriate synonym of the (a) Wan (b) Sagacious given word. (c) Volatile (d) Vicious Zealous SSC CGL (Tier-I) – 12/06/2019 (Shift-II) (a) Enthusiastic (b) Apathetic Ans. (b): Witty - Synonym - Sagacious (c) Indifferent (d) Detached Meaning of other words– SSC CGL (Tier-I) 14/07/2023 (Shift-I) Wan - Pallid, Volatile - Variable, Vicious - Cunning Ans. (a) :Synonym of the word 'zealous' is 'Enthusiastic' meaning of other word Apathetic – Neutral Indifferent – unconcerned 669. Select the most appropriate synonym of the Detached – Disgruntled given word. 675. Select the most appropriate synonym of the Xenial given word. (a) Adept (b) Hospitable ZEST (c) Ridiculous (d) Mend (a) Indifference (b) Detachment SSC CHSL (Tier-I) 21/10/2020 (Shift-III) (c) Unconcern (d) Enthusiasm Ans. (b): Xenial - Synonym - Hospitable SSC MTS – 16/08/2019 (Shift-I) Meaning of other words– Ans. (d): Zest- Synonym - Enthusiasm Adept - Skillful, Ridiculous - Unsightly, Mend - Meaning of other words– Reform Indifference - Apathy 670. Select the most appropriate synonym of the Detachment - Squad Unconcern - Indifference given word. Xanthic 676. Select the most appropriate synonym of the (a) Applause (b) Plentiful given word. (c) Yellowish (d) Generous Zenith SSC MTS – 09/08/2019 (Shift-I) (a) Explicit (b) Impediment (c) Climax (d) Nadir Ans. (c): Xanthic - Synonym - Yellowish – SSC MTS 11-10-2017 (Shift-I) Meaning of other words– Applause - Cheer, Plentiful - Abundant, Generous - Ans. (c): Zenith - Synonym - Climax Meaning of other words– Liberal Explicit - Obvious, Impediment - Hindrance, Nadir Fall 677. Select the most appropriate synonym of the 671. Select the most appropriate synonym of the given word. given word. Zealot Yearn (a) Adverse (b) Pausity (a) Deny (b) Sadden (c) Sectarian (d) Knave (c) Vow (d) Wan SSC CHSL (Tier-I) 13/10/2020 (Shift-II) SSC MTS – 05/08/2019 (Shift-II) Ans. (c): Zealot - Synonym - Sectarian Ans. (b): Yearn - Synonym - Sadden Meaning of other words– Meaning of other words– Adverse - Hastile, Pausity - Lack, Knave – Rogue Deny - Negate, Vow - Oath, Wan - Faded 678. Select the most appropriate synonym of the 672. Select the most appropriate synonym of the given word. given word. Zeal Yawn (a) Apathy (b) Enthusiasm (a) Nap (b) Volatile (c) Impeccable (d) Forego (c) Gape (d) Vicious SSC CGL (Tier-I) – 06/06/2019 (Shift-III) SSC GD – 11/02/2019 (Shift-II) Ans. (b): Zeal - Synonym - Enthusiasm Ans. (c): Yawn - Synonym - Gape Meaning of other words– Meaning of other words– Apathy- Indifference, Impeccable - Perfect, Forego Abandon Nap - Blink, Volatile - Variable, Vicious - Corrupt 673. Select the most appropriate synonym of the 679. Select the most appropriate synonym of the given word. given word. Zigzag Yield (a) Simple (b) Straight (a) Diligent (b) Blame (c) Curve (d) Frank (c) Assume (d) Weary SSC CPO-SI – 11/12/2019 (Shift-I) SSC CPO-SI – 23/11/2020 (Shift-II) Ans. (c): Zigzag - Synonym - Curve Ans. (c): Yield - Synonym - Assume Meaning of other wordsMeaning of other words– Simple - Easy, Frank - Lenient, Straight - Directly Diligent - Industrious, Blame - Flaw, Weary - Dull

Z

X

Y

Synonyms

437

YCT

07. ANTONYMS TCS hewšve& hej DeeOeeefjle (Based On TCS Pattern) Chapterwise

Exam

Question No.

1

CGL (Tier-1) CGL (Tier-2) CHSL (Tier-1) CHSL (Tier-2) Selection Post VII, VIII, XI SSC MTS SSC GD SSC CPO SI CGL (Tier-1) CGL (Tier-2) CHSL (Tier-1) CHSL (Tier-2) Selection Post VII, VIII, XI SSC MTS SSC GD SSC CPO SI CGL (Tier-1) CGL (Tier-2) CHSL (Tier-1) CHSL (Tier-2) Selection Post VII, VIII, XI SSC MTS SSC GD SSC CPO SI CGL (Tier-1) CGL (Tier-2) CHSL (Tier-1) CHSL (Tier-2) Selection Post VII, VIII, XI SSC MTS SSC GD SSC CPO SI CGL (Tier-1) CGL (Tier-2) CHSL (Tier-1) CHSL (Tier-2) Selection Post VII, VIII, XI SSC MTS SSC GD SSC CPO SI

15 5 20 10 9 11 1 2 3 2 1 1 2 5 1 4 10 8 12 6 4 10 2 5 10 3 17 5 5 5 2 5 7 1 10 5 5 9 1 7

CGL (Tier-1) CGL (Tier-2) CHSL (Tier-1) CHSL (Tier-2)

8 2 7 3

A

2

B

3

C

4 D

5

E

6

Antonyms

438

Years

(2017–2023)

(2017–2023)

(2017–2023)

(2017–2023)

(2017–2023)

(2017–2023) YCT

F

7

G

8

H

9

I

10

J

11

K

12 L

13

Antonyms

Selection Post VII, VIII, XI SSC MTS SSC GD SSC CPO SI CGL (Tier-1) CGL (Tier-2) CHSL (Tier-1) CHSL (Tier-2) Selection Post VII, VIII, XI SSC MTS SSC GD SSC CPO SI CGL (Tier-1) CGL (Tier-2) CHSL (Tier-1) CHSL (Tier-2) Selection Post VII, VIII, XI SSC MTS SSC GD SSC CPO SI CGL (Tier-1) CGL (Tier-2) CHSL (Tier-1) CHSL (Tier-2) Selection Post VII, VIII, XI SSC MTS SSC GD SSC CPO SI CGL (Tier-1) CGL (Tier-2) CHSL (Tier-1) CHSL (Tier-2) Selection Post VII, VIII, XI SSC MTS SSC GD SSC CPO SI CGL (Tier-1) CGL (Tier-2) CHSL (Tier-1) CHSL (Tier-2) Selection Post VII, VIII, XI SSC MTS SSC GD SSC CPO SI CGL (Tier-1) CGL (Tier-2) CHSL (Tier-1) CHSL (Tier-2) Selection Post VII, VIII, XI SSC MTS SSC GD SSC CPO SI CGL (Tier-1) CGL (Tier-2) CHSL (Tier-1) 439

4 5 1 5 2 1 3 1 1 1 2 2 1 2 1 1 4 1 4 9 1 10 2 8 5 3 5 1 1 1 1 1 1 1 1 2 1 2 1 1 5 1 3 7 3 5

(2017–2023)

(2017–2023)

(2017–2023)

(2017–2023)

(2017–2023)

(2017–2023)

YCT

M

14

N

15 O

16 P

17 Q

18

R

19

S

Antonyms

CHSL (Tier-2) Selection Post VII, VIII, XI SSC MTS SSC GD SSC CPO SI CGL (Tier-1) CGL (Tier-2) CHSL (Tier-1) CHSL (Tier-2) Selection Post VII, VIII, XI SSC MTS SSC GD SSC CPO SI CGL (Tier-1) CGL (Tier-2) CHSL (Tier-1) CHSL (Tier-2) Selection Post VII, VIII, XI SSC MTS SSC GD SSC CPO SI CGL (Tier-1) CGL (Tier-2) CHSL (Tier-1) CHSL (Tier-2) Selection Post VII, VIII, XI SSC MTS SSC GD SSC CPO SI

2 3 5 1 5 1 1 2 1 2 4 3 2 5 2 5 3 1 2 8 2 12 8 5 10 2 5

CGL (Tier-1) CGL (Tier-2) CHSL (Tier-1) CHSL (Tier-2) Selection Post VII, VIII, XI SSC MTS SSC GD SSC CPO SI CGL (Tier-1) CGL (Tier-2) CHSL (Tier-1) CHSL (Tier-2) Selection Post VII, VIII, XI SSC MTS SSC GD SSC CPO SI CGL (Tier-1) CGL (Tier-2) CHSL (Tier-1) CHSL (Tier-2) Selection Post VII, VIII, XI SSC MTS SSC GD SSC CPO SI

1 1 1 1 1 1 8 2 12 5 10 2 2 5 10 8 22 5 1 4 2 4

440

(2017–2023)

(2017–2023)

(2017–2023)

(2017–2023)

(2017–2023)

(2017–2023)

(2017–2023)

YCT

20

T

21

U

22

V

23

W

24

X

25

Y

26

Z

Antonyms

CGL (Tier-1) CGL (Tier-2) CHSL (Tier-1) CHSL (Tier-2) Selection Post VII, VIII, XI SSC MTS SSC GD SSC CPO SI CGL (Tier-1) CGL (Tier-2) CHSL (Tier-1) CHSL (Tier-2) Selection Post VII, VIII, XI SSC MTS SSC GD SSC CPO SI CGL (Tier-1) CGL (Tier-2) CHSL (Tier-1) CHSL (Tier-2) Selection Post VII, VIII, XI SSC MTS SSC GD SSC CPO SI CGL (Tier-1) CGL (Tier-2) CHSL (Tier-1) CHSL (Tier-2) Selection Post VII, VIII, XI SSC MTS SSC GD SSC CPO SI CGL (Tier-1) CGL (Tier-2) CHSL (Tier-1) CHSL (Tier-2) Selection Post VII, VIII, XI SSC MTS SSC GD SSC CPO SI CGL (Tier-1) CGL (Tier-2) CHSL (Tier-1) CHSL (Tier-2) Selection Post VII, VIII, XI SSC MTS SSC GD SSC CPO SI CGL (Tier-1) CGL (Tier-2) CHSL (Tier-1) CHSL (Tier-2) Selection Post VII, VIII, XI SSC MTS SSC GD SSC CPO SI 441

5 3 6 6 2 4 1 1 1 1 2 1 1 1 1 1 1 1 1 1 1 2 1 1 1 1 1 1 2 1 1 1 1 1 1 1 1 1 1 1 1

(2017–2023)

(2017–2023)

(2017–2023)

(2017–2023)

(2017–2023)

(2017–2023)

(2017–2023)

YCT

Trend Analysis of Questions Topicwise from CGL (Pre & Mains) CHSL (Pre & Mains) Selection Post VII, VIII, XI, SSC MTS, SSC GD & Other Exams (2017-2023)

Antonyms

442

YCT

07. ANTONYMS 5.

A 1.

Select the most appropriate ANTONYM of the given word. Arrogant (a) Humble (b) Haughty (c) Superior (d) Scornful SSC GD 31/01/2023 (Shift-II) Ans. (a) : Arrogant (Conceited, haughty), Antonym Humble (Modest, submissive) Meaning of the other optionsHaughty - Highbrow Superior - Better Scornful - Reproachful, Sardonic 2. Select the most appropriate ANTONYM of the given word. Accuse (a) Incriminate (b) Defend (c) Charge (d) Allege SSC GD 27/01/2023 (Shift-I) Ans. (b) : Accuse (Blame), Antonym- Defend (Protect) Meaning of the other optionsIncriminate Inculpate Charge Cost, allegation Allege Impeach 3. Select the most appropriate ANTONYM of the given word. Alienate (a) Alternate (b) Alter (c) Reconcile (d) Renounce SSC GD 23/01/2023 (Shift-IV) Ans. (c) : Alienate (Separate, Segregate), AntonymReconcile (Combine, integrate) Meaning of the other optionsAlternate Optional Alter Change Renounce Leave/Cessation 4. Select the most appropriate ANTONYM of the given word. Accurate (a) Heist (b) Proficient (c) Unreliable (d) Fiesta SSC GD 16/01/2023 (Shift-III) Ans. (c) : Accurate (correct), Anthonym - Unreliable (Inaccurate Incredible). Meaning of the other optionsHeist Theft Proficient Skilled Fiesta Festival Antonyms

Select the most appropriate ANTONYM of the underlined word. To abate her listlessness, she went for a long walk in the country. (a) subside (b) lessen (c) sustain (d) increase SSC CGL (Tier-I) 19/07/2023 (Shift-IV) Ans. (d) : The most appropriate antonym of the underlined word ' abate' (decrease) is 'Increase.' Meaning of other words areSubside means - diminish Sustain means - endure, maintain Lessen means - Reduce, Thus, the correct answer is option (d). 6. Select the most appropriate ANTONYM of the given word. Absurd (a) Tolerable (b) Adorable (c) Lovable (d) Sensible SSC CGL (Tier-I) 25/07/2023 (Shift-IV) Ans. (d) : The most appropriate antonym for the above given word - 'Absurd' is 'sensible,' Meaning of other optionsAdorable - Placable, Socred, Lovable - Affectionable, Endearment, Thus, the correct answer is option (d). 7. Select the most appropriate ANTONYM of the underlined word in the following sentence. The old port was abandoned a long ago. (a) Quit (b) Kept (c) Shortened (d) Rashed SSC CGL (Tier-I) 20/07/2023 (Shift-I) Ans. (b) : The most appropriate antonym of the underlined word ‘Abandoned’ is ‘kept’. The meaning of the other words is– Quit means – Relinquish, stop Shortened means – Abbreviated. Rashed means – Reckless, hasty. Thus, the correct answer is option (b). 8. Select the most appropriate ANTONYM of the underlined word. My teacher accepted my excuse for being late. (a) Rejected (b) Believed (c) Trusted (d) Agreed SSC CGL (Tier-I) 20/07/2023 (Shift-I) Ans. (a) : The most appropriate antonym of the underlined word ‘accepted’ is ‘Rejected’. Other options are – Believed means – Trusted Trusted means – Reliable Agreed means – Concurred Thus, the correct answer is option (a).

443

YCT

9.

Choose the word that is opposite in meaning to the given word. Analogous (a) Dissimilar (b) Puzzle (c) Straight (d) Matching SSC MTS 08/05/2023 (Shift-III) Ans. (a) : The most appropriate opposite meaning for the given word 'Analogous' (similar) is 'Dissimilar'. other words are– Puzzle means – Riddle Straight means – Square/Directly. Matching means – Collation Thus, the correct answer is option (a) 10. Choose the word that is opposite in meaning to the given word. Abolish (a) Efface (b) Formalize (c) Deface (d) Jettison SSC MTS 04/05/2023 (Shift-II) Ans. (b) : The most appropriate opposite meaning to the given word 'Abolish' (Terminate) to 'Formalize.' (Adorn). Meaning of the other words are___ Efface means- Expunge, erase Deface means- Calumniate, Disgrace Jettison means- Scrap Thus, the correct answer is option (b). 11. Choose the world that is opposite in maning to the given word. "Abort" (a) Commence (b) Collision (c) Commode (d) Command SSC MTS 16/05/2023 (Shift-II) Ans. (a) : The opposite meaning for the given word 'Abort' (stop) is 'Commence'. (start) Meaning of other optionsCallision- Conflict Commode- Bathroom Command- Mandate 12. Select the most appropriate ANTONYM of the underlined word. My niece is an amateur artist. I hope she becomes famous one day. (a) Boring (b) Decent (c) Freelancing (d) Expert SSC CGL (Tier-I) 27/07/2023 (Shift-III) Ans. (d) : The most appropriate antonym of the above given underlined word 'amateur' is 'expert'. Meaning of other optionsBoring - Tedious, Decent - Civilied, Freelancing - Independent. 13. Select the most appropriate ANTONYM of the given word. Affliction (a) Jeopardy (b) Mortgage (c) Predicament (d) Consolation SSC CHSL (Tier-I) 09/08/2023 (Shift-III) Ans. (d) : "Consolation" (Something that given comfort to someone who is sad/ disappointed would be the most apposite antonym of the given word 'Affliction' – (A cause of Persistent pain or distress). Antonyms

Other option are different Jeopardy – danger of loss, harm or failure. Mortgage – A type of loan to buy a flat, land, home etc. Predicament – An unpleasant and difficult situation that is hard to get out of. 14. Select the most appropriate ANTONYM of the given word. Awkward (a) Innocent (b) Meaningful (c) Huge (d) Graceful SSC CHSL (Tier-I) 03/08/2023 (Shift-II) Ans. (d) : The most appropriate antonym of the above given word 'Awkward' is 'Graceful'. Other options give different meanings – Innocent – Candid, Faultless, Meaningful – Significant, Useful Huge – Gaint, Large, 15. Choose the word that is opposite in meaning to the given word. Algid (a) Algae (b) Gelid (c) lgneous (d) Clean SSC CHSL (Tier-I) 21/03/2023 (Shift-II) Ans. (c) : Algid - Antonym - lgneous Meaning of other wordsClean – Neat Algae – Moss Gelid – Wet 16. Choose the word that is opposite in meaning to the given word. Abduct (a) Redeem (b) Impress (c) Sociable (d) Hijack SSC CHSL (Tier-I) 20/03/2023 (Shift-I) Ans. (a) : Abduct - Antonym - Redeem Meaning of other wordsHijack- Dispoil Impress- Affect Sociable- Affable 17. Choose the word that is opposite in meaning to the given word. 'Anomalous' (a) Endemic (b) Outlandish (c) Customary (d) Alien SSC CHSL (Tier-I) 09/03/2023 (Shift-IV) Ans. (c) : 'Anomalous' - Antonym - 'Customary' Meaning of other wordsOutlandish – External Endemic – Local Alien – Foregner 18. Choose the word that is opposite in meaning to the given word. Accessory (a) Substitute (b) Surplus (c) Essential (d) Option SSC CHSL (Tier-I) 09/02/2023 (Shift-II)

444

YCT

Ans. (c) : Accessory, Antonym, Essential Meaning of other wordsSubstitute – Replacement Option – Choice Surplus – Saving 19. Select the most appropriate ANTONYM of the given word. Ally (a) Friend (b) Partner (c) Enemy (d) Associate SSC Constable GD-03/12/2021 Shift-II Ans. (c) : Ally (Friend), Antonym, Enemy (Foe) Meaning of the other OptionsFriend – Companion Partner – Fellow, Associate Associate – Colleague, Accomplice 20. Select the most appropriate ANTONYM of the given word. Assuage (a) Soothe (b) Decrease (c) Relieve (d) Provoke SSC Constable GD-01/12/2021 Shift-I Ans. (d) : Assuage (Soothe) Antonym, Provoke (Aggravate) Meaning of the other OptionsSoothe – Quiet Decrease – Reduce Relieve – Aid, Help 21. Select the most appropriate antonym of the given word. Assert (a) Abandon (b) Request (c) Urge (d) Insist SSC Constable GD-08/12/2021 Shift-I Ans. (a) : Assert (Claim) Antonym - Abandon - (give up, Renounce) Meaning of the other OptionsRequest - Plea, Appeal Urge - Request Insist - Emphasize 22. Select the most appropriate ANTONYM of the given word. Alert (a) Ready (b) Careless (c) Active (d) Aware SSC Constable GD-17/11/2021 Shift-II Ans. (b) : Alert (Careful), Antonym-Careless (Reckless) Meaning of the other OptionsReady - Prepared Active - Energetic/Lively Aware - Conscious 23. Select the most appropriate ANTONYM of the given word. Advance (a) Approach (b) Retreat (c) Assemble (d) Progress SSC Constable GD-14/12/2021 Shift-III Antonyms

Ans. (b) : Advance (Proceed) Antonym – Retreat (Back off) Meaning of the other OptionsApproach – Access Assemble – Amass Progress – Advancement 24.

Select the most appropriate ANTONYM of the given word. Awkward (a) Bulky (b) Uneasy (c) Inelegant (d) Graceful SSC Constable GD-23/11/2021 Shift-III Ans. (d) : Awkward - (Clumsy/ugly), Antonym - Graceful - (Pretty/Beautiful) Meaning of the other OptionsBulky - Heavy Uneasy - Restless Inelegant - Awful

25.

Select the most appropriate ANTONYM of the given word. Aggressive (a) Calm (b) Excited (c) Sharp (d) Simple SSC Constable GD-25/11/2021 Shift-III Ans. (a) : Aggressive (Assailant), Antonym – Calm (Tranquil) Meaning of the other OptionsExcited – Agitated Sharp – Acute/Intense Simple – Easy

26.

Select the most appropriate ANTONYM of the given word. AMPLE (a) Enough (b) Liberal (c) Meagre (d) Sufficient SSC Constable GD-29/11/2021 Shift-II Ans. (c) : Ample (Sufficient), Antonym - Meager (Scant) Meaning of the other OptionsEnough - Sufficient Liberal - Generous Sufficient - Ample 27. Select the most appropriate ANTONYM of the given word. Appal (a) Assure (b) Astound (c) Amaze (d) Alarm SSC CGL (Tier-I) 12/04/2022 Shift-I Ans. (a) : Appal (Horrify), Antonym - Assure (Convince) Meaning of the other OptionsAstound - Amaze/Surprise Amaze - Surprise Alarm - Panic/warning

445

YCT

28.

Select the most appropriate ANTONYM of the given word. Abundant (a) Surplus (b) Scarce (c) Brilliant (d) Rich SSC MTS-15/07/2022 Shift-I Ans. (b) : Abundant (Ample/Plentiful), Antonym – Scarce (Meager/Scanty) Meaning of the other OptionsSurplus – Extra Brilliant – Intelligent Rich – Wealthy

29.

Select the most appropriate ANTONYM of the given word. (a) Present (b) Resent (c) Descent (d) Patent SSC CGL (Tier-I) 13/04/2022 Shift-I Ans. (c) : Ascent (Crescendo), Antonym - descent (Declivity) Meaning of the other OptionsPresent – Current Resent – Rage Patent – Copyright/License

30.

Select the most appropriate ANTONYM of the given word. Astound (a) Clarify (b) Stun (c) Confuse (d) Shock SSC CGL (Tier-I) 21/04/2022 Shift-III Ans. (a) : Astound (amaze/Astonish), Antonym – clarify (Explain) Meaning of the other OptionsStun – Paralyze Confuse – Muddle Shock – Jerk

31.

Select the most appropriate antonym of the given word. Assiduous (a) Anxious (b) Destitute (c) Unique (d) Idle SSC CHSL 27.05.2022 Shift-II Ans. (d) : Assiduous (Hardworking/diligent), Antonym - Idle (Inactive/Dormant) Meaning of the other wordsAnxious - Vexed/worried Destitute - Impoverished/Poor Unique - Special/Exclusive

32.

Select the most appropriate antonym of the given word. AUDACITY (a) rashness (b) bravery (c) boldness (d) timidity SSC Stenographer (Grade C & D) 15.11.2021 Shift-I

Antonyms

Ans. (d) : Audacity (Courage/Impertinent), Antonym timidity (Cowardice/Coy) Meaning of the other wordsbravery - Courageousness boldness - Dauntlessness rashness - Eagerness/Hastiness 33. Select the most appropriate antonym of the given word. ACQUIT (a) replace (b) relieve (c) leave (d) condemn SSC Stenographer (Grade C & D) 15.11.2021 Shift-I Ans. (d) : Acquit (Absolve/Exonerate) Antonym/condemn (Censure/Criticize) Meaning of the other wordsRelieve - Assist/Help Leave - Renounce/Vacation Replace - Displace 34. Select the most appropriate antonym of the given word. APPRISE (a) inform (b) notify (c) enlighten (d) hide SSC Stenographer (Grade C & D) 11.11.2021 Shift-II Ans. (d) : Apprise (Inform/Notify), Antonym- hide (Conceal) Meaning of the other wordsInform - Tell/Report Notify - Apprise/Promulgate Enlighten - Inform 35. Select the most appropriate antonym of the given word. Accustomed (a) Terrific (b) Fabulous (c) Ordinary (d) Unusual SSC CHSL (Tier-I) –05/08/2021 (Shift-I) SSC CGL (Tier-I) –10/06/2019 (Shift-II) Ans. (d): Accustomed - (Habituated), Antonym, Unusual - (Abnormal) Meaning of other words – Terrific - Horrible Fabulous - Tremendous Ordinary - Normal 36. Select the most appropriate antonym of the given word. AFFINITY (a) aversion (b) weakness (c) inclination (d) attraction SSC CHSL (Tier-I) –06/08/2021 (Shift-I) Ans. (a): Affinity - (Congeniality) is Antonym, Aversion - (Hate) Meaning of other words – Attraction - Charm Inclination - Tendency Weakness - Vulnerability 37. Select the most appropriate antonym of the given word. Anxiety (a) Certainty (b) Restlessness (c) Nervousness (d) Panic SSC CGL (Tier-I) –20/08/2021 (Shift-I)

446

YCT

Ans. (a): Anxiety - (Perplexity) is Antonym, Certainty (Assurance/Doubtless) Meaning of other words – Restlessness – Discomfort/Impatience Nervousness - Fidgetiness Panic – Fear/Anxiety 38. Select the most appropriate antonym of the given word. AMBIGUOUS (a) ambitious (b) rare (c) vague (d) clear SSC Stenographer Grade C&D –05/02/2019 (3:5pm)

Ans. (d): 'Ambiguous' – (Vague), Antonym - clear – (Transparent) Meaning of other words – Ambitious – Desirous/Wishful Rare - Scarce Vague - Unclear 39. Select the most appropriate ANTONYM of the given word. ADMONISH (a) warn (b) scold (c) chide (d) applaud SSC CPO-SI – 25/11/2020 (Shift-I) SSC CHSL (Tier-I) –16/10/2012 (Shift-II) Ans. (d): Admonish - (Reprimand/Chide), Antonym Applaud (Praise) Meaning of other words – Warn - Caution Scold - Rebuke Chide - Damn 40. Select the most appropriate ANTONYM of the given word. ALIVE (a) active (b) life (c) dead (d) living SSC CPO-SI – 23/11/2020 (Shift-I) Ans. (c): Alive - (Living), Antonym - Dead (Expired) Meaning of other words – Active - animate Life - Existence Living - Livelihood 41. Select the antonym of the given word. AGONY (a) anxiety (b) distress (c) comfort (d) misery SSC CGL (Tier-I) – 06/06/2019 (Shift-I) Ans. (c): 'Agony' - (Anguish), Antonym - Comfort (Ease) Meaning of other words – Anxiety - Worry Distress - Hazard/Danger Miser - Niggard 42. Select the most appropriate antonym of the given word. ASCENT (a) descent (b) depression (c) decent (d) distant SSC CHSL (Tier-I) – 26/10/2020 (Shift-II) SSC CGL (Tier-I) – 12/06/2019 (Shift-I) Antonyms

Ans. (a): Ascent - (Crescendo/Rise), Antonym Descent- (Declivity) Meaning of other words – Depression - Sadness/Despair Decent - Civilized/Elegant Distant - Far a way 43. Select the most appropriate antonym of the given word. ABSURD (a) Sensitive (b) Sentimental (c) Selfish (d) Sensible SSC CGL (Tier-I) – 12/06/2019 (Shift-II) Ans. (d): Absurd - (Foolish/Stupid), Antonym Sensible - (Conscious/Aware) Meaning of other words – Sensitive – Emotional Sentimental – Soft-hearted Selfish - Egotist 44. Select the most appropriate antonym of the given word. ACQUIT (a) Evict (b) Forgive (c) Convict (d) Clear SSC CGL (Tier-I) – 12/06/2019 (Shift-II) Ans. (c): Acquit- (Exonerate), Antonym - Convict(Denounce/Impute) Meaning of other words – Forgive - Pardon Clear - Apparent Evict – Expel/Banish 45. Select the most appropriate antonym of the given word. ARROGANCE (a) superiority (b) sweetness (c) humility (d) vanity SSC CGL (Tier-I) – 11/06/2019 (Shift-I) Ans. (c): Arrogance - (Conceited/Haughty), Antonym - Humility (Modesty/Courtesy) Meaning of other words – Superiority- Excellence/Supremacy Sweetness- Delicacy/Sugariness Vanity - Pride 46. Select the most appropriate antonym of the given word. ACCEPT (a) Reject (b) Acquire (c) Obtain (d) Release SSC CGL (Tier-I) – 10/06/2019 (Shift-II) Ans. (a): Accept - (Admit), Antonym - Reject (Denial) Meaning of other words – Acquire - Occupy Obtain - Get Release – Discharge/Liberation 47. Select the most appropriate ANTONYM of the given word. APPLAUD (a) denounce (b) magnify (c) commend (d) approve SSC CPO-SI – 09/12/2019 (Shift-I)

447

YCT

Ans. (a): Applaud - (Praise), Antonym - Denounce (impute/Accuse) Meaning of other words – Magnify – Elongate/Increase Commend - Admire Approve - Sanction 48. Select the most appropriate ANTONYM of the given word. AWARE (a) impervious (b) ignorant (c) robust (d) resilient SSC CPO-SI – 12/12/2019 (Shift-II) Ans. (b): Aware - (Conscious/Informed), Antonym Ignorant - (Unknown/Unfamiliar) Meaning of other words – Impervious – Impregnable/Impermeable Robust - Strong Resilient - Flexible 49. Select the most appropriate ANTONYM of the given word. ADEPT (a) proficient (b) deft (c) expert (d) inept SSC CPO-SI – 12/12/2019 (Shift-I) Ans. (d): Adept – (Skillful/Dexterous), Antonym Inept (Unworthy/Unfit) Meaning of other words – Proficient – Experienced/Versed Deft - Efficient Expert – Specialist/Master 50. Select the most appropriate antonym of the given word. ADORN (a) Subdue (b) Disfigure (c) Enrich (d) Grace SSC CPO-SI – 13/12/2019 (Shift-I) Ans. (b): Adorn - (Decorate/Prettify), Antonym Disfigure (Deface/Impair) Meaning of other words – Subdue - Quell Enrich - Aggrandize Grace – Pity/Compassion 51. Select the most appropriate antonym of the given word. Abstract (a) Elusive (b) Concrete (c) Intangible (d) Vague SSC CHSL (Tier-I) –11/07/2019 (Shift-I) Ans. (b): Abstract- (Intangible/Abstract), Antonym Concrete (Ignoble) Meaning of other words – Elusive- Illusory Vague- Ambiguous/Unclear Intangible- Abstract/Immaterial 52. Select the most appropriate antonym of the given word. AMASS (a) collect (b) horde (c) heap (d) distribute SSC CHSL (Tier-I) –10/07/2019 (Shift-III) Antonyms

Ans. (d): Amass- (Collect/Accumulate) AntonymDistribute (Divide) Meaning of other words – Collect- Accumulate/Congregate Horde- Throng/Mob Heap- Pile/Mass 53. Select the most appropriate antonym of the given word. ADVANCE (a) shift (b) move (c) progress (d) retreat SSC CHSL (Tier-I) –09/07/2019 (Shift-I) Ans. (d): Advance- (Progress) Antonym- Retreat (Digress/Recoil) Meaning of other words – Shift- Displace Move- Ploy Progress- Advancement/Rise 54. Select the most appropriate antonym of the given word. ARROGANT (a) humble (b) rough (c) bold (d) uncouth SSC CHSL (Tier-I) –09/07/2019 (Shift-I) Ans. (a): Arrogant- (Conceited/Haughty) Antonymhumble (Modest) Meaning of other words – Uncouth- Clumsy Bold- Dauntless/Courageous Rough- Uncivilized/Rude 55. Select the most appropriate antonym of the given word. ASCENT (a) nascent (b) descent (c) crescent (d) recent SSC CHSL (Tier-I) –08/07/2019 (Shift-I) Ans. (b): Ascent- (Crescendo) Antonym - Descent (Declivity) Meaning of other words – Nascent- Newborn Crescent- Growing Recent- Modern 56. Select the most appropriate Antonym of the given word. Accessible (a) Reachable (b) Stressed (c) Attainable (d) Restricted SSC CHSL (Tier-I) –04/07/2019 (Shift-I) Ans. (d): Accessible- (Handy/Facile), Antonym Restricted (Bounded/Banned) Meaning of other words – Stressed- Emphasize/Pressurize Attainable- Obtainable Reachable- Accessible 57. Select the most appropriate ANTONYM of the given word. ABBREVIATE (a) Expand (b) Dispute (c) Oppose (d) Challenge SSC CHSL (Tier-I) 17/03/2020 (Shift-I)

448

YCT

Ans. (a): Abbreviate– (Brief/Compress), Antonym Expand (Enlarge/Extend) Meaning of other words – Dispute–Controversy/Fight Oppose–Deprecate Challenge–Objection 58. Select the most appropriate ANTONYM of the given word. AGITATE (a) Argue (b) Clam (c) Excite (d) Impress SSC CHSL (Tier-I) 15/10/2020 (Shift-I) Ans. (b): 'Agitate'- (Protest), Antonym- Calm (Quiet) Meaning of other words– 'Argue' – Contend , 'Excite' – Thrill/Stir, 'Impress' - Influence 59. Select the most appropriate ANTONYM of the given word. APEX (a) Asset (b) Base (c) Force (d) Pinnacle SSC CHSL (Tier-I) 20/10/2020 (Shift-II) Ans. (b): 'Apex'- (Acme), Antonym - 'Base' (Foundation)~ Meaning of other words– Asset - Property, Force – Power/Strength, Pinnacle - Summit 60. Select the most appropriate ANTONYM of the given word. ALTRUISTIC (a) Outstanding (b) Selfish (c) Serene (d) Soft-spoken SSC CHSL (Tier-I) 13/10/2020 (Shift-II) Ans. (b): Altruistic– (Unselfish), Antonym - 'Selfish' (Egotist) Meaning of other words – Outstanding –Due/Unpaid Serene - Calm Soft-spoken - Gentle 61. In the following question, out of the given four alternatives, select the one which is opposite in meaning of the given word. Astonish (a) Surprise (b) Shock (c) Startle (d) Calm SSC MTS 9-10-2017 (Shift-II) Ans. (d): Astonish- (Astound), Antonym - Calm (Pacify) Meaning of other words– Surprise - ‘Bewilderment’ Shock or Startle - ‘Flabbergast’~ 62. In the following Question, Out of the given four alternatives, select the one in which is opposite the given word. Auspicious (a) Bright (b) Happy (c) Unlucky (d) Timely SSC MTS 9-10-2017 (Shift-III) Antonyms

Ans. (c): Auspicious- (Propitious), Antonym Unlucky (Unfortunate) Meaning of other words – Bright- Gleaming/Effulgent Happy- Cheerful Timely- Opportune 63. In the following question, out of the given four alternatives, select the one which is opposite in meaning of the given word. Arid (a) Firtlie (b) Desert (c) Thirsty (d) Dusty SSC MTS 11-10-2017 (Shift-II) Ans. (a): Arid- (Barren/Dry), Antonym - Fertile (Fecund) Meaning of other words – Desert – Wilderness/Torrid Thirsty – Parched Dusty – Grubby/Mucky 64. In the following question, out of the given four alternatives, select the one which is opposite in meaning of the given word. Apposite (a) Suitable (b) Inappropriate (c) Timely (d) Relevant SSC MTS 11-10-2017 (Shift-II) Ans. (b): Apposite - (Appropriate), Antonym Inappropriate - (Unsuitable) Meaning of other words– Suitable – Preferable/Ideal Timely – Opportune, Relevant - pertinent 65. Select the most appropriate antonym of the given word. Approve (a) Agree (b) Reject (c) Appeal (d) Confirm SSC MTS – 07/08/2019 (Shift-I) Ans. (b): Approve- (Approbation/Sanction), Antonym - Reject (Deny/Refuse)~ Meaning of other words – Agree - Concur Appeal - Plead Confirm - Vouch 66. Select the most appropriate antonym of the given word. Acute (a) Dark (b) Clever (c) Grave (d) Blunt SSC MTS – 02/08/2019 (Shift-I) Ans. (d): Acute- (Sharp), Antonym - Blunt (Dull/Flat)~ Meaning of other words – Dark - Dim Clever - Adroit Grave – Serious/Vital 67. Select the most appropriate antonym of the word. Attract (a) Charm (b) Repel (c) Praise (d) Lure SSC MTS – 21/08/2019 (Shift-II) SSC MTS – 09/08/2019 (Shift-III)

449

YCT

Ans. (b): Attract (Captivate/Enchant), Antonym Repel (Hate) Meaning of other words– Praise – Admire, Lure - Entice, Charm – Glamour, Attractiveness 68. Select the most appropriate antonym of the word. ABUNDANT (a) Copious (b) Plentiful (c) Scarce (d) Ample SSC MTS – 19/08/2019 (Shift-II) Ans. (c): Abundant- (Bountiful), Antonym - Scarce (Mearge/Scant) Meaning of other words – Copious - Profuse Plentiful - Galore Ample - Plenty 69. Select the most appropriate antonym of the word. APPLAUD (a) Cheer (b) Criticize (c) Praise (d) Clap SSC MTS – 19/08/2019 (Shift-I) Ans. (b): APPLAUD- (Admire), Antonym - Criticize (Denigrate/Lambaste) Meaning of other words – Cheer – Merriment/Joy Praise – Commend Clap - Applause 70. Select the most appropriate antonym of the word. AFFLUENCE (a) Prosperity (b) Luxury (c) Poverty (d) Richness SSC MTS – 16/08/2019 (Shift-III) Ans. (c): Affluence - (Prosperity), Antonym - Poverty (Destitution/Indigence) Meaning of other words– Luxury – Enjoyment, Richness – Wealth/Success 71. Select the most appropriate antonym of the word. AMBIGUOUS (a) Strange (b) Uncertain (c) Clear (d) Indefinite SSC MTS – 14/08/2019 (Shift-II) Ans. (c): 'Ambiguous'- (Unclear/Obscure), Antonym 'Clear' (Comprehensible, Direct) Meaning of other words – Strange-Odd/Peculiar Uncertain-Unsure Indefinite- Unlimited/Infinite 72. Select the antonym of the give word. Adamant (a) Coherent (b) Stubborn (c) Stupid (d) Flexible SSC MTS – 09/08/2019 (Shift-II) Antonyms

Ans. (d): Adamant - (Rigorous), Antonym - Flexible (Pliable/Malleable) Meaning of other words – Coherent – Apparent/Clear Stubborn – Obdurate / Obstinate Stupid – Dumb, Silly 73. Select the antonym of the given word. ASSAULT (a) Defend (b) Offend (c) Strike (d) Agree SSC GD – 02/03/2019 (Shift-I) Ans. (a): Assault - (Attack), Antonym - Defend (Protect/Safeguard) Meaning of other words– Offend- (Dishonor), Strike- Down tool, Agree- Consent 74. Select the antonym of the given word. ABOLISH (a) Build (b) Affirm (c) Cancel (d) Finish SSC GD – 11/02/2019 (Shift-II) Ans. (a): Abolish-(Terminate), Antonym- Build (Construct) Meaning of other words – Affirm – Assert/declare Cancel - Rescind Finish - Conclude

B 75.

Select the most appropriate ANTONYM of the given word. Benign (a) Favorable (b) Wicked (c) Friendly (d) Cheerful SSC GD 12/01/2023 (Shift-II) Ans. (b) : Benign (kind, Gentle), Antonym - Wicked (Vicious) Meaning of the other optionsFriendly - Amiable, cordial Favorable - Compatiable Cheerful - Hilarious 76. Select the most appropriate ANTONYM of the underlined word. As a boy, Charles was a boor who bullied other kids in his neighbourhood. (a) creep (b) scum (c) blighter (d) gentleman SSC GD 01/02/2023 (Shift-II) Ans. (d) : In the given sentence, 'Gentleman' is the most appropriate antonym of the underlined word boor (barbarian). The other options give different meaning– Creep :- crawl. Scum :- worthless people, trash. Blighter :- a persistently annoying person. 77. Select the most appropriate ANTONYM of the capitalised word in the given sentence. Taking a BRISK walk can often induce a feeling of well-being.

450

YCT

(a) Slow (b) Late night (c) Dusky (d) Quick SSC CGL (Tier-I) 19/07/2023 (Shift-IV) Ans. (a) : The most appropriate antonym of the capitalized word 'Brisk' in the given sentence is word 'slow.' The meaning of the option is Late night means - midnight Dusky means - Gloomy, dims Quick means - prompt, brisk Thus, the correct answer is option (a). 78. Select the most appropriate ANTONYM of the underlined word in the following sentence. The doctor assured the patient that the tumour was benign and could be easily removed (a) Sterile (b) Kind (c) Favourable (d) Malignant SSC CHSL (Tier-I) 08/08/2023 (Shift-II) Ans. (d) : The most appropriate antonym of the above underlined word 'benign' (kindly) is 'malignant' (deadly). Meaning of other options – Sterile – Arid, Barren, Kind – Sort, Variant, Favourable –suited, Congruent 79. Select the most appropriate ANTONYM of the underlined word in the following sentence. The fields remained barren despite the efforts of the local administration and agricultural scientists. (a) Damp (b) Fertile (c) Sterile (d) Unforgiving SSC CHSL (Tier-I) 03/08/2023 (Shift-II) Ans. (b) : The most appropriate antonym of the above underlined word 'barren' is 'fertile'. Meaning of other options – Damp – Wet, Moist, Sterile – Neuter, Barren, Unforgiving – Grim, Revengeful, 80. Select the most appropriate antonym of the given word. BABBLE (a) DRIVEL (b) SENSE (c) JARGON (d) GIBBERISH SSC CHSL (Tier-I) 17/03/2023 (Shift-III) Ans. (b) : Babble (Delirium) - Antonym - Sense (Spirit) Meaning of other wordGibberish Splutter Jargon Murmur Drivel Slaver / Haver 81. Select the most appropriate ANTONYM of the given word. Beneficial (a) Fearful (b) Hopeful (c) Harmful (d) Careful SSC Constable GD-30/11/2021 Shift-III Antonyms

Ans. (c) : Beneficial (Advantageous), Antonym→ Harmful (Detrimental) Meaning of the other OptionsFearful - Afraid Hopeful - Optimistic Careful - Wary 82. Select the most appropriate antonym of the given word. BORING (a) enduring (b) exciting (c) supporting (d) preventing SSC Stenographer (Grade C & D) 11.11.2021 Shift-I Ans. (b) : Boring (Dull/Monotonous), Antonymexciting (Thrilling/Interesting) Meaning of the other wordsEnduring - Everlasting/Tolerate Supporting - Help/Assist Preventing - Stop/Avoid 83. Select the most appropriate ANTONYM of the given word. BLUNT (a) Dull (b) Sharp (c) Round (d) Pointless SSC CPO-SI–12/12/2019 (Shift-II) SSC CHSL (Tier-I) –15/04/2021 (Shift-I) Ans. (b): Blunt – Unsharpened/Thick, Antonym, Sharp – Edged/ Pointed Meaning of other words – Dull – Sluggish/Lethargic, Round - Circular, Pointless - Senseless 84. Select the most appropriate antonym of the given word. BRAVERY (a) heroism (b) strength (c) cowardice (d) courage SSC CHSL (Tier-I) –09/08/2021 (Shift-I) Ans. (c): Bravery - (Boldness), Antonym, Cowardice (Timidity) Meaning of other words – Strength - Power, Courage - Velour, Heroism – Valiance 85. Select the most appropriate antonym of the given word. Blemish (a) Perfection (b) Stain (c) Damage (d) Discoloration SSC CGL (Tier-I) –20/08/2021 (Shift-I) Ans. (a): Blemish - (Fault/ disfigurement), Antonym, Perfection - (Excellence) Meaning of other words – Stain - Spot, Damage - Harm, Discoloration – Streak/ Tarnishing 86. Select the most appropriate antonym of the given word. BOLD (a) confident (b) cheeky (c) brave (d) timid SSC CPO-SI (Tier-II) 27/09/2019 (3 pm - 5 pm)

451

YCT

Ans. (d): 'Bold' - Intrepid, Antonym - Timid (Frightened) Meaning of other words – Confident – Optimistic/Poised Cheeky – Pert/Impudent Brave – Courageous 87. Select the most appropriate antonym of the given word. BRUTAL (a) fierce (b) sane (c) savage (d) humane SSC CGL (Tier-I) – 19/06/2019 (Shift-III) Ans. (d): Brutal - (Vicious/Cruel), Antonym Humane (Kind/Considerate) Meaning of other words – Fierce- Aggressive/Dangerous, Savage- Ferocious, Sane- Rational 88. Select the most appropriate antonym of the given word. BESEECH (a) Command (b) Exhort (c) Invoke (d) Implore SSC CPO-SI – 13/12/2019 (Shift-I) Ans. (a): Beseech - (Beg/Implore), Antonym 'Command' (Order) Meaning of other words– Invoke - (Pray), Implore - (Entreat/ Request), Exhort - (Encourage) 89. Select the most appropriate antonym of the given word: BUOYANT (a) Neutral (b) Bouncy (c) Gloomy (d) Cheerful SSC CHSL (Tier-I) 21/10/2020 (Shift-I) Ans. (c): Buoyant- (Blithe/Happy), Antonym Gloomy (Despondent/dejected) Meaning of other words – Neutral – Impartial Bouncy – Springy Cheerful – Joyful 90. Selection the most appropriate ANTONYM of the given word. BENEVOLENT (a) Friendly (b) Generous (c) Liberal (d) Stingy SSC CHSL (Tier-I) 19/10/2020 (Shift-I) Ans. (d): Benevolent - (Gracious), Antonym - Stingy (Parsimonious) Meaning of other words – Friendly– Amiable, Generous– Magnanimous, Liberal– Free/Nonpartisan 91. Select the most appropriate antonym of the word. Better (a) Worse (b) Worst (c) Bad (d) Good SSC MTS – 16/08/2019 (Shift-I) Antonyms

Ans. (a): Better- (Superior), Antonym - Worse (Substandard/Inferior) Meaning of other words – Worst - Lowest Bad - Poor Good - fine 92. Select the most appropriate antonym of the word. BOISTEROUS (a) Stormy (b) Loud (c) Noisy (d) Peaceful SSC MTS – 14/08/2019 (Shift-I) Ans. (d): 'Boisterous'- (Thundery), Antonym 'Peaceful'- (Tranquil) Meaning of other words – Stormy – blustery/Windy Loud – Raucous Noisy - Clamorous 93. Select the antonym of the given word. BLESS (a) Purify (b) Sanctify (c) Wish (d) Curse SSC GD – 14/02/2019 (Shift-II) Ans. (d): Bless - (Benediction), Antonym - Curse (Anathema) Meaning of other words– Purify – Clean/Decontaminate, Sanctify - Consecrate, Wish - Desire

C 94.

Select the appropriate antonym for the underlined word. The arms and legs lay in a careless way. (a) Forgetful (b) Unsafe (c) Neglectful (d) Vigilant SSC MTS 02/05/2023 (Shift-I) Ans. (d) : The most appropriate antonym for the underlined word 'careless' will be 'vigilant' Meaning of the other words areForgetful means- Absent minded Unsafe means- unanimous, perilous Neglectful means- Negligent Thus, the correct answer is option (d) 95. Select the most appropriate ANTONYM of the underlined word. She chided her son for being impatient with the customers. (a) applauded (b) scolded (c) rebuked (d) reproved SSC CGL (Tier-I) 26/07/2023 (Shift-II) Ans. (a) : The most appropriate antonym of the Underlined word 'chided' (Rebuke) is 'applauded' (appreciate)/praise. The meaning of the other words is– Scolded means – Castigated Rebuked means – Reprimand Reproved means – scolded Thus, the correct answer is option (a).

452

YCT

96.

Select the most appropriate ANTONYM of the underlined word in the given sentence. Many confessed after the physical torture by the cops. (a) Revealed (b) Acknowledged (c) Denied (d) Recognised SSC CHSL (Tier-I) 09/08/2023 (Shift-III) Ans. (c) : The most apposite antonym of the underlined word 'confessed' will be {Admission of hauing done something wrong option (c) denied, which means 'refusal to admit or accept something. Meaning of other options are– Revealed – To make a secret known Acknowledged – To admit that something is true or exists Recognized –To perceive to be the same as something previously seen of known 97. Select the most appropriate ANTONYM of the underlined word. Wealthier countries are obligated to fulfil a commitment made in the Paris Agreement to provide $100 billion a year in international climate finance. (a) hale (b) feeble (c) drape (d) breach SSC CHSL (Tier-I) 02/08/2023 (Shift-I) Ans. (d) : The most appropriate antonym of the given underlined word ' commitment' is 'breach' (outrage)' Meaning of other options– Hale – Robust Feeble – Weakly Drape – Hang 98. Identify the most appropriate ANTONYM of the underlined word in the given sentence. Precious stones are expensive because they are not common. (a) Extinct (b) Abundant (c) Natural (d) Rare SSC Selection Posts XI-27/06/2023 (Shift-I) Ans. (d) : The most appropriate antonym of the above underlined word 'Common' is 'Rare'. Meaning of other options – Extinct – Destructed, Abundant –Copious, Natural – Physical, 99. Choose the word that is opposite in meaning to the given word. 'Contempt' (a) Approbation (b) Opprobrium (c) Odium (d) Obloquy SSC CHSL (Tier-I) 15/03/2023 (Shift-III) Ans. (a) : 'Contempt' - Antonym - ''Approbation'' Meaning of other wordOpprobrium → Disdain Obloquy → Insult Odium → Hatred Antonyms

100. Choose the word that is opposite in meaning to the given word. 'Cupidity' (a) Largesse (b) Hatred (c) Greed (d) Avarice SSC CHSL (Tier-I) 15/03/2023 (Shift-III) Ans. (a) : Cupidity - Antonym - 'Largesse' Meaning of other wordAvarice → Solicitation Greed → Seduction Hatred → Disgust 101. Select the most appropriate ANTONYM of the underlined word in the given sentence. The library sessions were conducive for healthy discussions. (a) Invaluable (b) Unfavourable (c) Integrated (d) Exclusive SSC Selection Posts XI-28/06/2023 (Shift-III) Ans. (b) : In the above sentence, Antonym of underlined word 'conducive' (Helpful) is 'Unfavourable' (Repugnant). Other options give different meaning. Invaluable - Priceless Integrated - Unified Exclusive - Unique 102. Select the most appropriate ANTONYM of the underlined word in the given sentence. We had hundreds of courageous scientists. (a) diffident (b) gutsy (c) audacious (d) valiant SSC MTS-06/07/2022 Shift-II Ans. (a) : Courageous (Bold), Antonym - Diffident (Bashful) Meaning of the other OptionsGutsy – Windly Audacious – Daring Valiant – Gallant 103. Select the most appropriate ANTONYM of the given word. Criticise (a) Prevent (b) Forgive (c) Disappoint (d) Praise SSC CGL (Tier-I) 11/04/2022 Shift-III Ans. (d) : Criticize (Condemn), Antonym - Praise (Admire) Meaning of the other OptionsPrevent → Stop/Interrupt Forgive → Pardon/Excuse Disappoint → Sadden 104. Select the most appropriate ANTONYM of the given word. Camaraderie (a) Friendship (b) Companionship (c) Amity (d) Dislike SSC CGL (Tier-I) 11/04/2022 Shift-III Ans. (d) : Camaraderie (Amicability/Friendliness), Antonym - Dislike (Aversion/Enmity) Meaning of the other OptionsFriendship →Cordiality Companionship →Brotherhood Amity → Friendliness

453

YCT

105. Select the most appropriate ANTONYM of the given word. CENTRE (a) Periphery (b) Definite (c) Mainstream (d) Clear SSC CGL (Tier-I) 13/04/2022 Shift-III Ans. (a) : Centre (Hub/Core/midpoint), Antonym Periphery (Circumference) Meaning of the other OptionsDefinite - Certain/Sure Mainstream - Orthodox/Popular Clear - Neat/ Apparent 106. Select the most appropriate ANTONYM of the given word. CLENCH (a) Relax (b) Clasp (c) Tighten (d) Hold SSC CGL (Tier-I) 13/04/2022 Shift-I Ans. (a) : clench (Depress), Antonym - Relax (Calm) Meaning of the other OptionsClasp – Grip Tighten – Stretched Hold – Catch 107. Select the most appropriate ANTONYM of the given word. Constrict (a) Choke (b) Cramp (c) Smite (d) Stretch SSC CGL (Tier-I) 19/04/2022 Shift-II Ans. (d) : Constrict (Narrowness), Antonym - Stretch (Expansion/Elasticity) Meaning of the other wordsChoke – Smother Cramp – Twist/Bend Smite – Kill/beat 108. Select the most appropriate antonym of the given word. Comely (a) Grandiloquent (b) Grotesque (c) Gregarious (d) Gorgeous SSC CHSL 31.05.2022 Shift-III Ans. (b) : Comely - (Delightful/Elegant) Antonym Grotesque - (Bizarre/Clumsy) Meaning of the other wordsGrandiloquent - Bombastic/Pompous Gregarious - Social/Companionable Gorgeous - Splendid/Grandiose 109. Select the most appropriate antonym of the word in bold in the following sentence. Her candid remarks impressed the Board of members. (a) blunt (b) sincere (c) deceitful (d) unconventional SSC CHSL 30.05.2022 Shift-I Antonyms

Ans. (c) : "Candid" (Frank/out spoken) Antonym deceitful (Dishonest/charlatan) Meaning of the other wordsBlunt – Unsharp Sincere – Honest/Genuine Unconventional – Unorthodox 110. Select the most appropriate ANTONYM of the given word. Callous (a) Cold (b) Insensitive (c) Caring (d) Hardened SSC Constable GD-10/12/2021 Shift-III Ans. (c) : Callous (Unkind, ruthless), Antonymcaring- (Loving, Warm-hearted) Meaning of the other OptionsCold- Cool, chill Insensitive- Heartless, Indifferent Hardened- Tough 111. Select the most appropriate ANTONYM of the given word. Certainty (a) Unreasonable (b) Prolong (c) Doubt (d) Divulge SSC Constable GD-01/12/2021 Shift-I Ans. (c) : Certainty (Necessarily), Antonym Doubt (Suspicion) Meaning of the other OptionsUnreasonable – Causeless Prolong – Lengthen Divulge – Reveal, Disclose 112. Select the most appropriate ANTONYM of the given word. Criticise (a) Analyze (b) Oppose (c) Clarify (d) Approve SSC Constable GD-16/11/2021 Shift-III Ans. (d) : Criticize - (Censure/Condemn) Antonym - Approve - (Praise/Confirm) Meaning of the other OptionsAnalyze - Examine/Scrutiny Oppose - Resist Clarify - Elucidate 113. Select the most appropriate ANTONYM of the given word. Crisp (a) Tender (b) Fresh (c) Rotten (d) Hard SSC Constable GD-18/11/2021 Shift-I Ans. (a) : Crisp (Crunchy), Antonym, Tender (Soft) Meaning of the other OptionsFresh – New/Refresh Rotten– Decayed Hard – Difficult/Solid 114. Select the most appropriate antonym of the given word. CONFINE (a) restrain (b) imprison (c) detain (d) liberate SSC Stenographer (Grade C & D) 12.11.2021 Shift-II

454

YCT

Ans. (d) : Confine(Limit/Bound) antonym - liberate (Emancipate/Release) Meaning of the other wordsrestrain–Control/Curb imprison–Jail/Lockup detain–Arrest/Apprehend 115. Select the most appropriate antonym of the given word. CONSERVE (a) maintain (b) support (c) destroy (d) preserve SSC Stenographer (Grade C & D) 12/11/2021 (Shift-I) Ans. (c): Conserve (Protect/Guard), Antonym destroy (Demolish/Damage) Meaning of other words– maintain - Preserve / Sustain support - Help/Aid Preserve - Retain / safeguard 116. Select the most appropriate antonym of the given word. CONGENIAL (a) Merciful (b) Mellow (c) Mild (d) Mean SSC CGL (Tier-II) 08/08/2022 (Shift-II) Ans. (d) : Congenial (Unison/Balancing), Antonym Mean (Insignficant/Frivolous) Meaning of other words– Merciful - Kind/Pitiful Mellow - Melodic/sweet Mild - Gentle/Placid 117. Select the most appropriate antonym of the given word. Clandestine (a) Heavy (b) Honest (c) Dressed (d) Opaque SSC CHSL (Tier-I) –09/08/2021 (Shift-I) Ans. (b): Clandestine - (Surreptitious), Antonym, Honest - (Candid/Upright) Meaning of other words – Heavy - Large, Opaque – Non-Transparent/Blurred, Dressed - Attired 118. Select the most appropriate ANTONYM of the given word. Captivity (a) Prosperity (b) Freedom (c) Tumble (d) Elevate SSC Constable GD-23/11/2021 Shift-III Ans. (b) : Captivity - (Imprisonment), Antonym - freedom (Liberation) Meaning of the other OptionsProsperity - Affluence Tumble - Fall Elevate - Raising 119. Select the most appropriate antonym of the given word. CRUCIAL (a) Essential (b) Trivial (c) Urgent (d) Stable SSC CGL (Tier-II) 16/11/2020 (3 pm - 5 pm) SSC CGL (Tier-I) 11/06/2019 (Shift–II) Antonyms

Ans. (b): 'Crucial' – Pivotal/Important, Antonym Trivial – Insignificant Meaning of other words – Essential - Indispensable Urgent – Emergency/High priority Stable - Permanent 120. Select the most appropriate antonym of the given word. Compassion (a) concern (b) meanness (c) sympathy (d) kindness SSC Sel. Post Phase-VII (M.L.) 15.10.2019 (Shift-I)

Ans. (b): Compassion – (Solicitude/Empathy), Antonym - Meanness - (Enormity) Meaning of other words – Concern - Worry Sympathy- Commiseration, Kindness – Benevolence 121. Select the most appropriate antonym of the given word. CALM (a) Still (b) Excited (c) Restful (d) Cool SSC Sel. Post Phase-VIII (M.L.) 09.11.2020 (Shift-III)

Ans. (b): Calm – (Serene/Quit), Antonym - excited (thrilled/Stir) Meaning of other words – Still - Motionless Restful - Relaxed Cool - Calm 122. Select the most appropriate antonym of the given word. COVETOUS (a) mercenary (b) avaricious (c) acquisitive (d) benevolent SSC CGL (Tier-I) – 11/06/2019 (Shift-II) Ans. (d): Covetous - (Rapacious), Antonym benevolent (Kind) Meaning of other words – Mercenary- Money-Grubbing/Grasping Avaricious- Greedy Acquisitive- Materialistic/Possessive 123. Select the antonym of the given word. COMIC (a) amusing (b) tragic (c) awkward (d) absurd SSC CGL (Tier-I) – 06/06/2019 (Shift-III) Ans. (b): 'Comic' - (Funny, Humorous), AntonymTragic (Disastrous) Meaning of other words – Absurd - Preposterous Amusing - Entertaining/Rollicking Awkward - Clumsy 124. Select the most appropriate ANTONYM of the given word. CONTIGUOUS (a) separated (b) contactual (c) neighbouring (d) adjoining SSC CPO-SI – 09/12/2019 (Shift-II)

455

YCT

Ans. (a): Contiguous- (Adjacent), Antonym - separated (Distant, Segregated )~ Meaning of other words – Contactual- Closest, Abutting, Neighbouring –Nearest, Preserve, Adjoining- Proximate 125. Select the most appropriate antonym of the given word. CONSUME (a) save (b) absorb (c) exhaust (d) devour SSC CPO-SI – 09/12/2019 (Shift-II) Ans. (a): Consume- (Ingest/Dissipate), Antonym- Save (Conserve, Preserve) Meaning of other words – Absorb- Assimilate/Imbibe, Exhaust- Enervate, Fatigue, Devour – Gobble/Munch 126. Select the most appropriate ANTONYM of the given word. CULPABLE (a) offensive (b) innocent (c) reserved (d) disagreeable SSC CPO-SI – 13/12/2019 (Shift-II) Ans. (b): Culpable - (Guilty), Antonym - innocent (Guiltless) Meaning of other words – offensive – hurtful/Derogatory, reserved - Booked, disagreeable - Objectionable 127. Select the most appropriate ANTONYM of the given word. CHIVALROUS (a) annoyed (b) valiant (c) indignant (d) cowardly SSC CPO-SI – 13/12/2019 (Shift-II) Ans. (d): Chivalrous- (Valiant, Bold), Antonym Cowardly (timorous, Pusillanimous) Meaning of other words – Annoyed- (Irritated), (Irked/Piqued) Valiant- Valorous, Indignant(Miffed/Resentful) 128. Select the most appropriate antonym of the given word. COMPLY (a) Defer (b) Adhere (c) Consent (d) Oppose SSC CPO-SI – 13/12/2019 (Shift-I) Ans. (d): Comply- (Obey, Follow), Antonym Oppose (Resist, Dissent, Disobey) Meaning of other words – Defer – (Adjourn, Postpone), Adhere – (Follow, Observe), Consent – (Assent, Accord) 129. Select the most appropriate antonym of the given word. Conclusive (a) Ambiguous (b) Clear (c) Undeniable (d) Decisive SSC CHSL (Tier-I) –11/07/2019 (Shift-III) Ans. (a): Conclusive- (Decisive/Demonstrative), Antonyms - Ambiguous (Equivocal) Meaning of other words – Clear (Lucid, Transparent), decisive (Final, Deciding), Undeniable (Indisputable, Indubitable Antonyms

130. Select the most appropriate antonym of the given word. COMPLACENT (a) humble (b) satisfied (c) confident (d) trusted SSC CHSL (Tier-I) –10/07/2019 (Shift-I) Ans. (a): Complacent- (Content, Smug), Antonyms Humble- (Niggard) Meaning of other words – Satisfied- Contended Confident- Guaranteed Trusted- Credible, Reliable 131. Select the most appropriate antonym of the given word. CHRONIC (a) continuing (b) infrequent (c) routine (d) regular SSC CHSL (Tier-I) –09/07/2019 (Shift-III) Ans. (b): Chronic- (Old) Antonym- Infrequent (rare, Unique) Meaning of other words – Continuing- Ongoing, Relentless Routine- Procedure, Visual, Everyday Regular- Frequent, continual, Perpetual 132. Select the most appropriate antonym of the given word. CENSURE (a) distrust (b) criticism (c) praise (d) shame SSC CHSL (Tier-I) –05/07/2019 (Shift-I) Ans. (c): Censure- (Stricture), Antonym- praise (Admire) Meaning of other words – Distrust- Wariness Criticism- Denunciation Shame- Shyness 133. Select the most appropriate antonym of the given word. CORROBORATE (a) confirm (b) contradict (c) approve (d) verify SSC CHSL (Tier-I) –02/07/2019 (Shift-III) Ans. (b): Corroborate- (Validate,/Confirm), Antonym Contradict (Refute, Rebut) Meaning of other words – Approve (Sanction), Verify (Vindicate, Substantiate), Confirm (Prove, Certify) 134. Select the most appropriate ANTONYM of the given word. CONCEALED (a) Masked (b) Revealed (c) Discussed (d) Hidden SSC CHSL (Tier-I) 16/10/2020 (Shift-II) Ans. (b): 'Concealed' - (Shrouded, Covered), Antonym- 'Revealed' (Exposed, Disclosed) Meaning of other words – Masked - Camouflaged Discussed – Debated, Hidden - Unrevealed

456

YCT

135. Select the most appropriate ANTONYM of the given word. CASUAL (a) Planned (b) Immature (c) Inspirational (d) Informal SSC CHSL (Tier-I) 19/03/2020 (Shift-I) Ans. (a): Casual - (Random, Spontaneous), Antonym - Planned (Orchestrated Devised) Meaning of other words – Immature – Callow, Inexperienced, Inspirational – Motivational, Informal - Causal 136. Select the most appropriate ANTONYM of the given word. CLOUDY (a) Transparent (b) Indefinite (c) Murky (d) Vague SSC CHSL (Tier-I) 18/03/2020 (Shift-III) Ans. (a): Cloudy - (Murky, Dull), Antonym 'transparent' (Clear) Meaning of other words – Indefinite – Infinite, Limitless Murky – Dim, Glormy Vague – Hazy, Indistinct 137. Select the most appropriate ANTONYM of the given word. COMPATRIOT (a) Brethren (b) Countryman (c) Outsider (d) Insider SSC CHSL (Tier-I) 26/10/2020 (Shift-II) Ans. (c): Compatriot – (Countryman), Antonym Outsider (Alien, Stranger) Meaning of other words – Brethren – Sibling Countryman – Fellow, Citizen Insider – Internal 138. Select the most appropriate antonym of the given word: CAUSTIC (a) Sarcastic (b) Kind (c) Scary (d) Bitter SSC CHSL (Tier-I) 21/10/2020 (Shift-I) Ans. (b): CAUSTIC- (Severe/Strict), Antonym - Kind (Gracious) Meaning of other words – Sarcastic – Scathing Scary – Terrifying Bitter – Acrid 139. Select the most appropriate ANTONYM of the given word. CONSPICUOUS (a) Noticeable (b) Distinct (c) Apparent (d) Obscure SSC CHSL (Tier-I) 19/10/2020 (Shift-I) Ans. (d): 'Conspicuous' - (Prominent, Specific), Antonym - Obscure (Unclear, Uncertain) Meaning of other words – Noticeable– Perceptible Distinct– Clear, Marked Apparent– Manifest, Obvious Antonyms

140. Select the most appropriate antonym of the given word: CONTEMPLATE (a) Deify (b) Conceive (c) Ignore (d) Meditate SSC CHSL (Tier-I) 21/10/2020 (Shift-III) Ans. (c): 'contemplate' - (Ponder), Antonym - 'Ignore' (Neglect, Avoid) Meaning of other words– Deify – Venerate, Worship, Conceive - formulate, Pregnant, Meditate - Ruminate 141. Select the most appropriate antonym of the given word: COARSE (a) Common (b) Colourful (c) Refined (d) Vulgar SSC CHSL (Tier-I) 21/10/2020 (Shift-III) Ans. (c): 'Coarse' – (Unrefined), Antonym 'Refined' (Purified) Meaning of other words – Common - General, Colourful - Jazzy, Motley, Vulgar - Indecent 142. Select the most appropriate ANTONYM of the given word. CONVENIENCE (a) Enjoyment (b) Satisfaction (c) Assistance (d) Hindrance SSC CHSL (Tier-I) 20/10/2020 (Shift-I) Ans. (d): 'Convenience'- (Comfort), Antonym 'Hindrance' (Obstruction) Meaning of other words – Enjoyment - Pleasure, Satisfaction - Contentment, Assistance - Aid 143. Select the most appropriate ANTONYM of the given word. CANDID (a) Genuine (b) Frank (c) Truthful (d) Devious SSC CHSL (Tier-I) 20/10/2020 (Shift-I) Ans. (d): Candid - (Frank, Forthright), Antonym Devious (Deceitful) Meaning of other words – Genuine- (Real, Authentic), Frank- (Unequivocal), Truthful - Sincere 144. In the following question, out of the given four alternatives, select the one which is opposite in meaning of the given word. Coarse (a) Cheap (b) Common (c) Foul (d) Gentle SSC MTS 9-10-2017 (Shift-II) Ans. (d): Coarse- (Oafish, Loutish), Antonym Gentle (humble) Meaning of other words – Cheap – Economical, Affordable, Common – General, Foul – Disagreeable, Unscrupulous. 145. In the following question, out of the given four alternatives, select the one which is opposite in meaning of the given word. Cease (a) Pause (b) Desist (c) Terminate (d) Continue SSC MTS 11-10-2017 (Shift-III)

457

YCT

Ans. (d): Cease - (End), Antonym - Continue (Pursue) Meaning of other words – Pause – Halt Desist – Refrain Terminate – Discontinue 146. In the following question, out of the given four alternatives, select the one which is opposite in meaning of the given word. Circuitous (a) Circular (b) Direct (c) Long way (d) Roundabout SSC MTS 11-10-2017 (Shift-II) Ans. (b): Circuitous - (Meandering), Antonym Direct - (Straight) Meaning of other words – Circular – Round, Long way - Distance, Roundabout - Rotary 147. Select the most appropriate antonym of the given word. Crooked (a) Curved (b) Straight (c) Spiral (d) Bent SSC MTS – 08/08/2019 (Shift-I) Ans. (b): Crooked-(Crafty, Dodgy), Antonym Straight (Direct) Meaning of other words – Curved Loop Spiral - Curl, Gyre Bent Twisted 148. Select the most appropriate antonym of the given word. Captivity (a) Liberty (b) Imprisonment (c) Cheer (d) Offense SSC MTS – 07/08/2019 (Shift-I) Ans. (a): 'Captivity'- (Incarceration), Antonym Liberty (Freedom) Meaning of other words – Imprisonment - Confinement Cheer - Encourage Offense - Crime 149. Select the most appropriate antonym of the given word. Cautious (a) Discreet (b) Negligent (c) Prudent (d) Alert SSC MTS – 06/08/2019 (Shift-I) Ans. (b): Cautious - (Heedful, Wary), Antonym Negligent (Careless) Meaning of other words – Discreet – Judicious, Prudent – sagacious, Alert - Careful 150. Select the most appropriate antonym of the given word. Current (a) General (b) Modern (c) Past (d) Present SSC MTS – 05/08/2019 (Shift-III) Ans. (c): Current- (Contemporary), Antonym - past (Previous) Meaning of other words – General - Common, Modern - Recent, Present – Instant, Existing Antonyms

151. Select the most appropriate antonym of the given word. Comic (a) Tickle (b) Happy (c) Tragic (d) Jubilant SSC MTS – 02/08/2019 (Shift-II) Ans. (c): Comic - (Funny), Antonym -Tragic (Disastrous) Meaning of other words – Tickle – Amuse, Excite, Happy - Joyful, Jubilant – Exultant 152. Select the most appropriate antonym of the word. Congested (a) Cleared (b) Crowded (c) Choked (d) Crammed SSC MTS – 19/08/2019 (Shift-I) Ans. (a): 'Congested'- (Crowded), Antonym - cleared (Free, Rid) Meaning of other words – Crowded - Thronged Choked - Block Crammed - Packed 153. Select the most appropriate antonym of the word. CURTAIL (a) Reduce (b) Decrease (c) Enlarge (d) lessen SSC MTS – 14/08/2019 (Shift-II) Ans. (c): 'Curtail'- (Reduce/Cut), Antonym - 'Enlarge' (Expand) Meaning of other words – Reduce– Lower, Shrink, Decrease– Diminish, Lessen– Minimize 154. Select the antonym of the given word. Coarse (a) Ragged (b) Attractive (c) Smooth (d) Shiny SSC MTS – 13/08/2019 (Shift-III) Ans. (c): Coarse- (Rough), Antonym - Smooth (Plain) Meaning of other words – Ragged - Craggy Attractive - Charming Shiny - Bright 155. Select the most appropriate antonym of the word. Confront (a) Avoid (b) Oppose (c) Encounter (d) Meet SSC MTS – 13/08/2019 (Shift-I) Ans. (a): Confront- (Challenge/Face), Antonym Avoid (Shun, Elude, Ignore) Meaning of other words – Oppose - Resist Encounter- Skirmish Meet – Intermingle 156. Select the antonym of the given word. CREATE (a) Develop (b) Deploy (c) Derive (d) Destroy SSC MTS – 08/08/2019 (Shift-II)

458

YCT

Ans. (d): Create- (Generate/ Design), Antonym destroy (Destruct, Demolish) Meaning of other words – Develop - Evolve Deploy - post, Employ, Locate Derive - Obtain, Gain 157. Select the antonym of the given word. CONSENSUS (a) Disagreement (b) Consent (c) Agreement (d) Harmony SSC MTS – 14/08/2019 SSC GD – 08/03/2019 (Shift-III) Ans. (a): 'Consensus'- (Concord), Antonym Disagreement (Dissent, Contention) Meaning of other words – Consent – Assent, Accord Agreement – Concurrence Harmony – Union 158. Select the antonym of the given word. COURTEOUS (a) Impartial (b) Rude (c) Detached (d) Independent SSC GD – 02/03/2019 (Shift-II) Ans. (b): Courteous - (Polite), Antonym - Rude (Insolent) Meaning of other words – Impartial - Neutral, Detached - Disconnected, Independent - Free 159. Select the antonym of the given word CONTRARY (a) Neutral (b) Balanced (c) Agreeable (d) Predetermined SSC GD – 18/02/2019 (Shift-II) Ans. (c): Contrary - (Opposite), Antonym Agreeable (Favorable) Meaning of other words – Neutral - Unbiased, Balanced - Stabilized, Predetermined – Fixed, Prearranged 160. Select the most appropriate antonym of the given word. CONVEX (a) Opaque (b) Dense (c) Concave (d) Transparent SSC GD – 11/02/2019 (Shift-III) Ans. (c): Convex - (Curvilinear, Arched, outcurve), Antonym - Concave (Incurved, Indented) Meaning of other words – Opaque – Filmy, blurred, Dense - Thick, Transparent - Clear

D 161. Select the most appropriate antonym to substitute the (bracketed) word in the given sentence. It would effectively rouse people's ardour to invest, and help to (discourage) the transfer of capital abroad. (a) Intolerable (b) Persuade (c) Industrious (d) Construe SSC MTS 10/05/2023 (Shift-I) Antonyms

Ans. (b) : The most appropriate antonym for the bracketed word 'discourage' is 'persuade' (coax). other words areIntolerable means- unbearable Industrious means- Diligent Construe means- Comment Thus, the correct answer is option (b). 162. Choose the word that is opposite in meaning to the given word. Desert (a) Bereft (b) Forlorn (c) Migrate (d) Populate SSC MTS 19/05/2023 (Shift-III) Ans. (d) : The opposite meaning of the above word – ‘Desert’ is ‘Populate’. Meaning of other optionsBereft – Loopy, Mania, Forlorn – Orphaned, hopeless, Migrate – Transmigrate, Emigrate, Thus, the correct answer is option (d). 163. Select the most appropriate antonym of the given word. Discreet (a) Open (b) Release (c) Lose (d) Zenith SSC MTS 18/05/2023 (Shift-I) Ans. (a) : The most appropriate antonym of the above given word – ‘Discreet’ is ‘open’. Meaning of other optionsRelease – Acquittal, Lose – Mislay, Zenith – Acme, Thus, the correct answer is option (a). 164. Select the appropriate antonym for the underlined word. I stare out into the really amazingly dark night. (a) unusually (b) obviously (c) shockingly (d) astonishingly SSC MTS 15/05/2023 (Shift-I) Ans. (b) : The opposite meaning for the underlined word 'dark' is 'obviously'. Meaning of other options – Unusually :- Remarkably, Shockingly :- disgraceful, Astonishingly :- Amazingly, Thus, the correct answer is options (b). 165. Choose the word that is opposite in meaning to the given word. Doze (a) Wake (b) Cold (c) Snooze (d) Neglect SSC MTS 12/05/2023 (Shift-III) Ans. (a) : The opposite meaning of the above given word 'Doze' is 'Wake.' Doze' means – 'to sleep lightly for a short time meaning of other options Cold – Winter, cool, Snooze – Loaf, Wink, Neglect – Disregard,

459

YCT

166. Select the most appropriate ANTONYM of the given word. Dispel (a) Scatter (b) Remove (c) Disburse (d) Integrate SSC GD 31/01/2023 (Shift-II) Ans. (d) : Dispel (Disperse, Spread), Antonym Integrate (Unite, merge) Meaning of the other optionsScatter - Diffuse, Seperate Remove - Abolish, End Disburse - Repay 167. Select the most appropriate ANTONYM of the given word. Divulgence (a) Repudiation (b) Confirmation (c) Revelation (d) Assertion SSC GD 13/02/2023 (Shift-I) Ans. (a) : 'Repudiation' (renunciation) is the most appropriate antonym of the given word Divulgence (opening). The meaning of other options is– Confirmation :- verification ratification. Revelation :- disclosure. Assertion :- statement. 168. Select the most appropriate ANTONYM of the underlined word. The musician gave a dismal performance of a few old melodies at last night’s concert. (a) exhausting (b) lively (c) melancholy (d) smart SSC CGL (Tier-I) 26/07/2023 (Shift-II) Ans. (b) : The most appropriate antonym of the underlined word ' dismal' is 'lively'. The other words are – Exhausting means – Bothersome melancholy means – Gloom Smart means – Nimble Thus, the correct answer is option (b). 169. Select the most appropriate ANTONYM of the underlined words in the given sentence. Developed countries are at a disadvantage to modern science and technologies. (a) privileged (b) useless (c) powered (d) unhappy SSC CGL (Tier-I) 17/07/2023 (Shift-II) Ans. (a) : The antonym of underlined word 'Disadvantage' is 'privileged' other option give different meaning. useless – vain/Aimless Powered – Enable unhappy – Confortless 170. Select the most appropriate ANTONYM of the given word from the following sentence. Decent My father taught me to be true and ethical in business as dishonesty leads to downfall. Antonyms

(a) ethical (b) downfall (c) true (d) dishonesty SSC CHSL (Tier-I) 09/08/2023 (Shift-III) Ans. (d) : The correct antonym of word 'Decent' Conforming with generally accepted standards of moral Behavior mentioned in the given sentence is option (d) Dishonesty – act without honesty a disposition to lie, cheat or steal. Meanings of other option are– Ethical – Connected with beliefs of what is right or wrong. Downfall – A sudden fall from power True – Being right or Being correct. 171. Select the appropriate ANTONYM of the underlined word to fill in the blank. I wanted to ask if you have an/a _____ bank account. Mine has been dormant for a year. (a) inert (b) side (c) active (d) quick SSC CHSL (Tier-I) 04/08/2023 (Shift-III) Ans. (c) : The most appropriate antonym of the above underlined word 'dorment' (inactive) is 'active'. Meaning of other options– Inert – Inactive, passive, Side – Party, Aspect, Quick – Hasty, At once, 172. Select the most appropriate ANTONYM of the given word. Despicable (a) Rational (b) Deplorable (c) Admirable (d) Offensive SSC CHSL (Tier-I) 04/08/2023 (Shift-III) Ans. (c) : The most appropriate antonym of the above given word 'Despicable' (Sordid) is 'admirable' Meaning of other options – Rational – Logical, Legitimate, Deplorable – Lamentable, Grievous, Offensive – Allack, Derogatory, 173. Select the most appropriate ANTONYM of the given word. Decay (a) Disintegrate (b) Growth (c) Spoilage (d) Breakdown SSC MTS-06/07/2022 Shift-III Ans. (b) : Decay (Destruction), Antonym - Growth (Development) Meaning of the other OptionsDisintegrate – Dispart Spoilage – Damage/Waster Breakdown – Collapse/Failure 174. Select the most appropriate antonym of the given word. Detrimental (a) Baseless (b) Senseless (c) Harmless (d) Meaningless SSC CHSL 24.05.2022 Shift-III

460

YCT

Ans. (c) : Detrimental - (Harmful/Noxious) Antonym Harmless - (Safe/Innocuous) Meaning of the other wordsBaseless - Pointless/Unfounded Senseless - Unconscious/silly Meaningless - Purposeless/Inexpedient 175. Select the most appropriate antonym of the given word. Dwarf (a) Medium (b) Giant (c) Measurable (d) Tiny SSC CHSL 24.05.2022 Shift-III Ans. (b) : Dwarf - (Pygmy), Antonym - 'Giant' (Huge/Enormous) Meaning of the other wordsMedium - Moderate/Intermediate Measurable - Medicare Tiny - Minute/Small 176. Select the most appropriate antonym of the given word. Demolish (a) Destroy (b) Divide (c) Diminish (d) Construct SSC Constable GD-08/12/2021 Shift-I Ans. (d) : Demolish (Destroy), Antonym - construct (Build) Meaning of the other OptionsDestroy - Decimate Divide - Split Diminish - Subtract 177. Select the most appropriate ANTONYM of the given word. Dull (a) Tender (b) Potent (c) Keen (d) Charitable SSC Constable GD-14/12/2021 Shift-II Ans. (c) : Dull - (Sluggish), Antonym- keen (Eager) Meaning of the other OptionsTender - Proposal, Soft Potent - Powerful, Strong Charitable - Liberal, Altruist 178. Select the most appropriate antonym of the given word. Dummy (a) Counterfeit (b) Copy (c) Duplicate (d) Original SSC Constable GD-07/12/2021 Shift-II Ans. (d) : Dummy (Fake) Antonym, original (Real) Meaning of the other OptionsCounterfeit – Forged Copy – Imitate Duplicate – Replica, Copy 179. Select the most appropriate ANTONYM of the given word. Distinctive (a) Similarity (b) Common (c) Original (d) Close SSC Constable GD-26/11/2021 Shift-I Antonyms

Ans. (b) : Distinctive (Special), Antonym– Common (Normal) Meaning of the other OptionsSimilarity – Equality Original – Genuine Close – Near 180. Select the most appropriate ANTONYM of the given word. DISCARD (a) accept (b) ditch (c) scrap (d) eliminate SSC Stenographer (Grade C & D) 12.11.2021 Shift-I Ans. (a) : Discard (Disapprove/Reject). Antonym Accept (Admit/ Concede) Meaning of the other wordsScrap - Trash/ Fragment eliminate - Delete/Remove ditch - Gulf/Entrenchment 181. Select the most appropriate ANTONYM of the given word. Deny (a) Exhaust (b) Agree (c) Deflate (d) Mourn SSC Constable GD-14/12/2021 Shift-III Ans. (b) : Deny (Reject) Antonym–Agree (Consent) Meaning of the other OptionsExhaust– Exit, Tire Deflate –Curtail, minimize Mourn – lament 182. Select the most appropriate antonym of the given word. DEVIOUS (a) Deceitful (b) shrewd (c) straight (d) calculating SSC CHSL (Tier-I) –04/08/2021 (Shift-I) Ans. (c): Devious - (Circuitous, roundabout), Antonym, Straight - (Direct) Meaning of other words – Deceitful – Dishonest, Shrewd - Astute, Calculating – Computing, Enumerating 183. Select the most appropriate antonym of the given word. Disheveled (a) Untidy (b) Cluttered (c) Ordered (d) Unkempt SSC CHSL (Tier-I) –05/08/2021 (Shift-I) Ans. (c): Disheveled - Disordered, Antonym, Ordered (Organized, Arranged) Meaning of other words – Untidy-Messy, Dirty, Unkempt-Scruffy, ClutteredDisarranged 184. Select the most appropriate antonym of the given word. Downcast (a) Distressed (b) Successful (c) Disappointed (d) Cheerful SSC CGL (Tier-I) –18/08/2021 (Shift-I)

461

YCT

Ans. (d): Downcast – Despondent, Antonym, Cheerful – Happy, Jolly Meaning of other words – Distressed - Upset, Successful – Triumphant, Disappointed – Crestfallen 185. Select the most appropriate ANTONYM of the given word. DEPREDATION (a) construction (b) pillage (c) plunder (d) devastation SSC CPO-SI – 24/11/2020 (Shift-I) Ans. (a) : 'Depredation' - Antonym construction (Manufacture, Creation) Meaning of other words – Pillage - Reaving, Plunder - Robbing, devastation - Ruin 186. Select the most appropriate ANTONYM of the given word. DISTRACT (a) beguile (b) divert (c) clarify (d) perplex SSC CPO-SI – 24/11/2020 (Shift-I) Ans. (c): 'Distract' - (Deflect, Divert), Antonym 'Clarify' (Elucidate) Meaning of other words – Beguile – Deceive, divert Deviate, perplex – Baffle 187. Select the most appropriate ANTONYM of the given word. DISHEARTEN (a) humble (b) indispose (c) encourage (d) dampen SSC CPO-SI – 23/11/2020 (Shift-II) Ans. (c): Dishearten- (Discourage), Antonym'Encourage' (Motivate) Meaning of other words Humble - (Modest), Indispose - (Unwell, Sick), Dampen - (Get wet) 188. Select the most appropriate ANTONYM of the given word. DIVERGE (a) stray (b) divide (c) branch (d) collect SSC CPO-SI – 24/11/2020 (Shift-II) Ans. (d): 'Diverge' - (Disunite/Divide), Antonym 'Collect' (Assemble, Unity) Meaning of other words – Stray – Digress wander Divide - Split Branch – Offshoot/Bough 189. Select the most appropriate ANTONYM of the given word. DEFILE (a) degrade (b) debase (c) evade (d) elevate SSC CPO-SI – 24/11/2020 (Shift-II) Ans. (d): 'Defile'- (profane), Antonym - 'Elevate' (Raise, Sanctity) Meaning of other words– Degrade - Demean Debase - Devalue Evade - Elude, Dodge Antonyms

190. Select the most appropriate ANTONYM of the given word. DEFICIENCY (a) excess (b) glitch (c) dearth (d) snag SSC CPO-SI – 24/11/2020 (Shift-II) Ans. (a): 'deficiency'- (Scarcity, Shortage), Antonym 'Excess' (Surplus) Meaning of other words – Glitch – Problem, Issue Dearth – Shortfall, Paucity Snag – Obstacle, Difficulty 191. Select the most appropriate antonym of the given word. DEFICIT (a) adhere (b) credit (c) surplus (d) remove SSC CGL (Tier-I) – 13/06/2019 (Shift-III) Ans. (c): Deficit-(Dearth), Antonym- Surplus (Excess) Meaning of other words – Remove- Eradicate Adhere- Comply Credit- eliminate 192. Select the antonym of the given word. DENSE (a) sparse (b) thick (c) condensed (d) opaque SSC CGL (Tier-I) – 07/06/2019 (Shift-I) SSC CHSL – 03/07/2019 (Shift-II) Ans. (a) 'Dense' - (Thick), Antonym- Sparse (Scarce/meager) Meaning of other words – Opaque - Blurred, filmy Condensed - Compact, Dense Thick - Dense 193. Select the most appropriate antonym of the given word. DEMEAN (a) Debase (b) Admire (c) Belittle (d) Ignore SSC CPO-SI – 11/12/2019 (Shift-II) Ans. (b): Demean - (Humiliate), Antonym - Admire (Applaud) Meaning of other words – Debase - Adulterate, Belittle – Curtail, Ignore – overlook 194. Select the most appropriate antonym of the given word. DUBIOUS (a) Likely (b) Fishy (c) Shady (d) Dusky SSC CPO-SI – 11/12/2019 (Shift-II) Ans. (a): Dubious - (Doubtful), Antonym - 'Likely' (Probable) Meaning of other words – Fishy - Suspicious, Shady – Dabious, Dusky – Murky 195. Select the most appropriate ANTONYM of the given word. DISCARD (a) share (b) distribute (c) adopt (d) waste SSC CPO-SI – 12/12/2019 (Shift-II)

462

YCT

Ans. (c): Discard - (Jettison), Antonym - Adopt (Embrace) Meaning of other words – Share - Part, Distribute - Dispense, Waste – Squander 196. Select the most appropriate antonym of the given word. DESPAIR (a) pride (b) calm (c) hope (d) dearth SSC CHSL (Tier-I) –05/07/2019 (Shift-II) Ans. (c): Despair- (hopelessness), Antonym - Hope (Expectation) Meaning of other words – Pride- Vanity Calm- Quiet Dearth- Paucity 197. Select the most appropriate antonym of the given word. Denounce (a) Criticise (b) Applaud (c) Vilify (d) Blame SSC CHSL (Tier-I) –03/07/2019 (Shift-I) Ans. (b): Denounce- (Impute), Antonym - Applaud(Admire) Meaning of other words– Criticize- Decry/Condemn Vilify- Denigrate/Abuse Blame- Allegation, Accusation 198. Select the most appropriate ANTONYM of the given word. Deport (a) Transport (b) Permit (c) Expel (d) Conduct SSC CHSL (Tier-I) 26/10/2020 (Shift-II) Ans. (b): deport- (Banish), Antonym - permit (Allow) Meaning of other words – Transport – Transit, Carriage, Expel - Evict, Conduct - Behavior 199. Select the most appropriate ANTONYM of the given word. DIVERSE (a) Practical (b) Farcical (c) Identical (d) Parallel SSC CHSL (Tier-I) 13/10/2020 (Shift-I) Ans. (c): Diverse – (Sundry), Antonym - Identical (Similar, Alike) Meaning of other words – Practical – Pragmatic Farcical – Humorous Parallel – Analogous, Similar. 200. Select the most appropriate ANTONYM of the given word. DISABLE (a) Permit (b) Refuse (c) Disapprove (d) Disallow SSC CHSL (Tier-I) 15/10/2020 (Shift-II) Ans. (a): 'Disable'- (Impair, Incapacitate), Antonym 'Permit' (Allow) Meaning of other words – Refuse - Decline, Disapprove - Reproach, Disallow – Reject. Antonyms

201. Select the most appropriate ANTONYM of the given word. DECAY (a) Decline (b) Rotting (c) Corrosion (d) Growth SSC CHSL (Tier-I) 19/10/2020 (Shift-III) Ans. (d): Decay– (Decompose, Deteriorate), Antonym - Growth (Progress) Meaning of other words – Decline – Downfall, Corrosion – Breakdown, Disintegration 202. Select the most appropriate ANTONYM of the given word. DISPUTE (a) Concede (b) Quarrel (c) Strengthen (d) Impute SSC CHSL (Tier-I) 19/03/2020 (Shift-II) Ans. (a): Dispute – (Discord, Controversy), Antonym - concede (Acknowledge, Admit) Meaning of other words – Quarrel – Feud, Fracas, Strengthen– Fortify, Impute– Blame 203. Select the most appropriate ANTONYM of the given word. DISAPPROVE (a) Oppose (b) Commend (c) Refuse (d) Neglect SSC CHSL (Tier-I) 14/10/2020 (Shift-II) Ans. (b): Disapprove– (Reject), Antonym- Commend (Suggest, Endorse) Meaning of other words – Oppose– Oppugn, Refuse– Decline, Neglect– Overlook 204. Select the most appropriate ANTONYM of the given word. Disclose (a) Disguise (b) Complete (c) Disrupt (d) Conceal SSC CHSL (Tier-I) 13/10/2020 (Shift-II) Ans. (d): Disclose– (Divulge), Antonym - Conceal (Hide) Meaning of other words – Disguise – Camouflaged Complete - Total Disrupt - Sabotage 205. Select the most appropriate antonym of the given word. Disillusioned (a) Soured (b) Embittered (c) Disappointed (d) Enthusiastic SSC CHSL (Tier-I) 12/10/2020 (Shift-III) Ans. (d): Disillusioned – (Disenchanted), Antonym Enthusiastic (Avid, Fervent) Meaning of other words – Soured – Tart Embittered – Soured Disappointed – Disheartened

463

YCT

206. Select the most appropriate ANTONYM of the given word. DEMOLISH (a) Abolish (b) Construct (c) Cherish (d) Obstruct SSC CHSL (Tier-I) 12/10/2020 (Shift-I) Ans. (b): Demolish – (Destroy), Antonym - Construct (Build) Meaning of other words – Abolish – Finish, Annul Cherish – Nurse, Adore Obstruct – Block, Clog 207. In the following question, out of the given four alternatives, select the one which is opposite in meaning of the given word. Dilate (a) Stretch (b) Develop (c) Compress (d) Enlarge SSC MTS 7-10-2017 (Shift-I) Ans. (c): Dilate– (Expand, Distend), Antonym Compress (Squeeze, Press Contract) Meaning of other words – Stretch–Lengthen, Widen, Develop – Grow, Enlarge– Lengthen, Elongate 208. In the following question, out of the given four alternatives, select the one which is opposite in meaning of the given word. Defy (a) Brave (b) Yield (c) Defeat (d) Ignore SSC MTS 10-10-2017 (Shift-III) Ans. (b): ‘Defy'- (Flout, Dare), Antonym- 'Yield' Capitulate / Assume Meaning of other words – Brave - Bold, Defeat – Lose, Vanquish, Ignore Overlook 209. In the following question, out of the given four alternatives, select the one which is opposite in the meaning of the given word. Disgrace (a) Honour (b) Spot (c) Blur (d) Stain SSC MTS 11-10-2017 (Shift-I) Ans. (a): Disgrace - (Humiliation Dishonor), Antonym - Honour (Reverence) Meaning of other words – Spot - Mark, Blur - Blot, Stain – Smear 210. Select the most appropriate antonym of the given word. DETER (a) Generalise (b) Dissuade (c) Occupy (d) Incite SSC MTS – 22/08/2019 (Shift-II) Ans. (d): 'Deter'- (Discourage, Daunt), Antonym 'Incite' (Provoke) Meaning of other words – Generalise - Mumble Dissuade - Deter Occupy - Surround Antonyms

211. Select the antonym of the given word. Delicious (a) Insipid (b) Tepid (c) Salty (d) Gross SSC MTS – 09/08/2019 (Shift-I) Ans. (a): Delicious - (Delectable), Antonym - Insipid (Tasteless, Unsavory) Meaning of other words– Tepid - warm Salty - saline Gross - Coarse, Overall 212. Select the most appropriate antonym of the given word. Disperse (a) Gather (b) Neglect (c) Send (d) Hold Ans. (a): Disperse- (Scatter, Diffuse), Antonym Gather (Garner) Meaning of other words – Neglect - overlook Send - Dispatch Hold - catch 213. Select the most appropriate antonym of the given word. DEFICIT (a) Scarcity (b) Manifold (c) Abundance (d) Shortage SSC MTS – 20/08/2019 (Shift-I) Ans. (c): Deficit- (Paucity, Shortage), Antonym Abundance (Plethora) Meaning of other words – Scarcity - Deficiency manifold – Miscellaneous Shortage – Shortfall 214. Select the most appropriate antonym of the word. DISTANT (a) Close (b) Faraway (c) Beyond (d) Remote SSC MTS – 16/08/2019 (Shift-I) Ans. (a): Distant- (Far, Outlying), Antonym - close (Near) Meaning of other words – Faraway - outlying Beyond - outside Remote - distant 215. Select the most appropriate antonym of the word. DIMINISH (a) Increase (b) Decrease (c) Shrink (d) Decline SSC MTS – 13/08/2019 (Shift-II) Ans. (a): Diminish - (Lessen), Antonym - Increase (Grow) Meaning of other words – Decrease - Reduce Shrink - Contract Decline – Fall 216. Select the antonym of the given word. Delicate (a) Flimsy (b) Feeble (c) Fragile (d) Firm SSC MTS – 09/08/2019 (Shift-II)

464

YCT

Ans. (d): Delicate - (Frail), Antonym - Firm (Strong) Meaning of other words – Flimsy - weak Feeble - Powerless Fragile - delicate 217. Select the most appropriate antonym of the given word. Destroy (a) Obliterate (b) Abolish (c) Preserve (d) Shine SSC MTS – 07/08/2019 (Shift-II) Ans. (c): Destroy- (Demolish), Antonym - Preserve (Save) Meaning of other words – obliterate - Eradicate Abolish - Scrap Shine - Glitter

E 218. Select the most appropriate ANTONYM of the word given in the bracket to fill in the blank. My stomach is so ______ (empty) I couldn't possibly eat more. (a) overfed (b) drained (c) flush (d) teeming SSC GD 01/02/2023 (Shift-II) Ans. (a) : In the blank space of the above sentence, 'overfed' would be appropriate which is the antonym of bracketed word 'empty'. The other options give different meaning. Drained :- empty, exhausted. Flush :- flow freely. Teeming :- abundant, aplenty. 219. Select the most appropriate ANTONYM of the given word. Elusive (a) Rigid (b) Explosive (c) Affordable (d) Definite SSC CHSL (Tier-I) 17/08/2023 (Shift-II) Ans. (c) : The antonym of given word Elusive – (not easy to catch , find or remember) is option (c) Affordable – ( believed to be within one's means or reach) Meaning of other options are – Rigid – Not able to be changed or persuaded Explosive – Capable of exploding therefore dangerous Definite – fixed, clearly determined 220. Select the most appropriate ANTONOYM of the underlined word in the given sentence. We were very much enthusiastic about our tour to Agra. (a) anxious (b) obsessed (c) inclined (d) casual SSC CHSL (Tier-I) 17/08/2023 (Shift-II) Ans. (a) : The correct antonym of the underlined word 'Enthusiastic' – (full of excitement and interest in something) in the given sentence is option (a) anxious – (Extreme uneasiness of mind, Brooding fear of contingency). Antonyms

Meaning of other options are – Obsessed – To completely fill your mind so that you cannot think of anything else. Inclined – wanting to behave in a particular way. Casual – without showing great effort or interest. 221. Select the most appropriate ANTONYM of the underlined word in the given sentence. She was in a state of ecstasy when she came to know that her son received a visa on his first attempt. (a) Abash (b) Inspiration (c) Agony (d) Joyfulness SSC CHSL (Tier-I) 04/08/2023 (Shift-III) Ans. (c) : The most approriate antonym of the above underlined word 'ecstasy' (bliss) is 'agony' (pain). Meaning of other options – Abash – Blaush, shame, Inspiration – spur, Stimulus, Joyfulness – Pleasure, Gaiety, Thus, the correct option is (c). 222. Choose the word that is opposite in meaning to the given word. 'Elucidate' (a) Obfuscate (b) Dilate (c) Magnify (d) Outdate SSC CHSL (Tier-I) 15/03/2023 (Shift-I) Ans. (a) : Elucidate - Antonym - 'obfuscate' Meaning of other wordsDilate → Extend Magnify → Amplify Outdate → Outmode 223. Select the most appropriate ANTONYM of the underlined word in the given sentence. He learned to endure hunger and cold. (a) Reconcile (b) Persist (c) Resist (d) Sustain SSC MTS-06/07/2022 Shift-III Ans. (c) : Endure (Suffer/Tolerate), Antonym - Resist (protest) Meaning of the other OptionsReconcile – Adjust/Resolve Persist – Persevere Sustain – Keep/Continue 224. Select the most appropriate ANTONYM of the given word. Evident (a) Obvious (b) Indistinct (c) Stubborn (d) Irritable SSC MTS-12/07/2022 Shift-I Ans. (b) : Evident (Obvious/Candid), Antonym Indistinct (Obscure/Vague) Meaning of the other OptionsObvious – Direct/ Apparent Stubborn – Obstinate Irritable – Fastidious/Touchy

465

YCT

225. Select the most appropriate ANTONYM of the underlined word. The more enlightened employers offer better terms. (a) Civilized (b) Educated (c) Confounded (d) Reasonable SSC MTS-05/07/2022 Shift-I Ans. (c) : Antonym of the underlined ward 'enlightened' (awakened) in the above sentence is 'Confounded' (Puzzled/perplexed). other options have different meaningsCivilized – Decent Educated – Literate Reasonable – Logical 226. Select the most appropriate ANTONYM of the given word. Exonerate (a) Exempt (b) Engage (c) Absolve (d) Acquit SSC CGL (Tier-I) 18/04/2022 Shift-I Ans. (b) : ‘Exonerate’ (Acquit/Absolve), Antonym - Engage (Involve/Capture) Meaning of the other OptionsExempt - free/Immune Absolve - Acquit/Exonerate Acquit - Liberate/Absolve 227. Select the most appropriate antonym of the given word. Ebb (a) Ripe (b) Flow (c) Moisture (d) Stiff SSC CHSL 27.05.2022 Shift-II Ans. (b) : Ebb - (Tide/Declivity), Antonym - flow (Flux) Meaning of the other wordsRipe - Mature/Mellow Moisture - Wetness/humidity Stiff - Hard/Tough 228. Which word in the given sentence is the antonym of exonerate ? It was difficult to convict him of the falsity of his beliefs. (a) beliefs (b) falsity (c) difficult (d) convict SSC CHSL 31.05.2022 Shift-III Ans. (d) : Exonerate - (Absolve), Antonym - Convict (Culprit) Meaning of the other wordsBeliefs - faith/trust Falsity - Myth Difficult - Arduous 229. Select the most appropriate ANTONYM of the given word. Erase (a) Create (b) Delete (c) Abolish (d) Strike SSC Constable GD-10/12/2021 Shift-III Antonyms

Ans. (a) : Erase (Efface), Antonym - Create-(Build, Make) Meaning of the other OptionsDelete- Remove Abolish- Finish, Eradicate Strike- Walkout, Assail 230. Select the most appropriate antonym of the given word. ENDORSE (a) Affirm (b) Renounce (c) Support (d) Advocate SSC CGL (Tier-II) 16/11/2020 (3 pm - 5 pm) Ans. (b): 'Endorse' – (Recommend, Support), Antonym - Renounce (Abdicate) Meaning of other words – Affirm – Assert, Aver Support - Aid Advocate – Supporter, Lawyer 231. Select the most appropriate antonym of the given word. EMACIATED (a) lean (b) fat (c) thin (d) shrunken SSC CPO-SI (Tier-II) 27/09/2019 (3 pm - 5 pm) Ans. (b): 'Emaciated' – Thin, Antonym - fat (Thick) Meaning of other words – Shrunken - Contract Thin – Slender Lean – Slim, willowy 232. Select the most appropriate ANTONYM of the given word. EXONERATE (a) vindicate (b) acquit (c) sentence (d) absolve SSC CPO-SI – 25/11/2020 (Shift-I) Ans. (c): Exonerate - (Absolve, Acquit), Antonym Sentence (Convict, Punish) Meaning of other words – Vindicate - Prove, Acquit - Exculpate, Absolve – Let off 233. Select the most appropriate ANTONYM of the given word. EXASPERATION (a) enjoyment (b) pique (c) resentment (d) wrath SSC CPO-SI – 23/11/2020 (Shift-II) Ans. (a): Exasperation- (Indignation, Fury), Antonym- 'Enjoyment' (Amusement) Meaning of other words – Pique - Irritation Resentment – Bitterness Wrath – Anger 234. Select the most appropriate ANTONYM of the given word. EXPEL (a) remove (b) erupt (c) absorb (d) leave SSC CPO-SI – 24/11/2020 (Shift-II)

466

YCT

Ans. (c): 'expel'- (Ostracize), Antonym - 'Absorb' (Imbibe) Meaning of other words – Remove - Eliminate Erupt - Explode Leave – Relinquish. 235. Select the most appropriate antonym of the given word. EXTRAVAGANT (a) pauper (b) generous (c) thrifty (d) deficit SSC CGL (Tier-I) – 11/06/2019 (Shift-III) Ans. (c): Extravagant- (Profligate), Antonym Thrifty (Economical) Meaning of other words – Pauper(Mendicant, Indigent), Generous(Magnanimous), Deficit - (Dearth) 236. Select the antonym of the given word. EMINENT (a) exalted (b) impressive (c) inconspicuous (d) distinguished SSC CGL (Tier-I) – 06/06/2019 (Shift-I) Ans. (c): 'Eminent' - (Prominent, Famous), AntonymInconspicuous (Unnoticeable Ordinary, Low-Profile) Meaning of other words – Exalted - Advanced, Elated Impressive - Magnificent Distinguish - Peculiar 237. Select the antonym of the given word. EXPANSION (a) extension (b) inflation (c) compression (d) augmentation SSC CGL (Tier-I) – 04/06/2019 (Shift-I) Ans. (c): Expansion- (Augmentation Growth), Antonym- Compression (Constriction, Contraction) Meaning of other words – Extension- Proliferation Inflation- Hike, Rise Augmentation- Multiplication 238. Select the antonym of the given word. ESCALATE (a) Raise (b) Enlarge (c) Reduce (d) heighten SSC CGL (Tier-I) – 04/06/2019 (Shift-II) Ans. (c): Escalate- (Soar, Surge), Antonym- Reduce (Decrease) Meaning of other words – Raise- Mount Enlarge- Grow Heighten- Elevate 239. Select the antonym of the given word. EXCEPTIONAL (a) uncommon (b) Unimaginable (c) unthinkable (d) unremarkable SSC CGL (Tier-I) – 04/06/2019 (Shift-III) Ans. (d): Exceptional- (Outstanding, Extraordinary, Phenomenal), Antonym - Unremarkable (normal, Common, Ordinary) Meaning of other words – Uncommon- Unusual, Unimaginable- inconceivable, Unthinkable- Implausible, Unbelievable Antonyms

240. Select the most appropriate ANTONYM of the given word. ENRAGED (a) pleased (b) livid (c) aggravated (d) infuriated SSC CPO-SI – 09/12/2019 (Shift-I) Ans. (a): Enraged - (Angry), Antonym - pleased (Happy) Meaning of other words – Livid – (Colourless), aggravated – (Provoked), infuriated - Vexed 241. Select the most appropriate ANTONYM of the given word. ERUDITE (a) scholarly (b) high-brow (c) savvy (d) ignorant SSC CPO-SI – 11/12/2019 (Shift-I) Ans. (d): Erudite- (Sapient), Antonym - Ignorant (Uneducated) Meaning of other words – Scholarly- Genius, High brow- Knowledgeable, Savvy - Learned 242. Select the most appropriate ANTONYM of the given word. ENDURANCE (a) tolerance (b) indolence (c) persistence (d) patience SSC CPO-SI – 12/12/2019 (Shift-I) Ans. (b): Endurance - (Forbearance), Antonym Indolence (Lethargy, Laziness) Meaning of other words – Tolerance - bearing / durability Persistence - Perseverance Patience – Fortitude, toleration 243. Select the most appropriate ANTONYM of the given word. EXULTANT (a) wounded (b) delighted (c) energetic (d) depressed SSC CPO-SI – 13/12/2019 (Shift-II) Ans. (d): Exultant - (Gleeful), Antonym - Depressed (Melancholy) Meaning of other words – Delighted - (Glad), Energetic - (Zestful, Zippy), Wounded - (Injured) 244. Select the most appropriate antonym of the given word. EXCEPTIONAL (a) ordinary (b) optional (c) rare (d) unique SSC CHSL (Tier-I) –05/07/2019 (Shift-II) Ans. (a): Exceptional- (Outstanding Extra-Ordinary), Antonym - Ordinary- (Common, Normal) Meaning of other words – Optional- Voluntary, Discretionary Rare- Infrequent Unique- Fearless/Rarebit

467

YCT

245. Select the appropriate antonym of the given word. EXAGGERATE (a) heighten (b) amplify (c) compress (d) overdo SSC CHSL (Tier-I) –03/07/2019 (Shift-II) Ans. (c): Exaggerate- (Hyperbolize, Overemphasize), Antonym - Compress (Brief) Meaning of other words – Heighten- Elevate, Amplify- Magnify OverdoOverstate, Overemphasize 246. Select the most appropriate antonym of the given word. Exotic (a) alien (b) ordinary (c) colourful (d) curious SSC CHSL (Tier-I) –02/07/2019 (Shift-I) Ans. (b): Exotic- (Foreign, Imported), Antonym Ordinary- (Common) Meaning of other words – Alien- (Non-Native, Unknown), Colourful- (Motley, Variegated), Curious- (Eager, Keen, Inquisitive) 247. Select the most appropriate ANTONYM of the given word. EXTENSIVE (a) Intricate (b) Intensive (c) Elaborate (d) Length SSC CHSL (Tier-I) 17/03/2020 (Shift-I) Ans. (b): Extensive – (Broad, Vast), Antonym Intensive (Concentrated, Deep) Meaning of other words – Intricate – (Complex, Entangle) Elaborate – Detailed, Length – Extent 248. Select the most appropriate ANTONYM of the given word. EARTHLY (a) Temporal (b) Celestial (c) Temperamental (d) Peripheral SSC CHSL (Tier-I) 18/03/2020 (Shift-I) Ans. (b): 'Earthly'- (Terrestrial, Temporal, Mundane) Antonym - 'Celestial' (Planetary, Astronomical) Meaning of other words – Temporal – Material, Worldly Temperamental – Mercurial, Volatile Peripheral – Minor, Subsidiary 249. Select the most appropriate ANTONYM of the given word. ENIGMA (a) Make-believe (b) Clarity (c) Mystery (d) Charade SSC CHSL (Tier-I) 12/10/2020 (Shift-II) Ans. (b): 'Enigma' - (Riddle, Puzzle), Antonym Clarity (Lucidity, Precision) Meaning of other words – Make believe – Fantasy, Daydreaming Charade – Sham, Facade Mystery – Secrecy 250. Select the most appropriate ANTONYM of the given word. EXPUNGE (a) Deplete (b) Exhaust (c) Insert (d) Remove SSC CHSL (Tier-I) 20/10/2020 (Shift-II) Antonyms

Ans. (c): Expunge - (Efface, Wipe out), Antonym 'Insert' (Place, Put, Add) Meaning of other words – Deplete – Consume, Expend, Exhaust Exhaust – Fatigue, Weary Remove – Erase 251. Select the most appropriate ANTONYM of the given word. EXHALE (a) Respite (b) Inspire (c) Inhale (d) Expel SSC CHSL (Tier-I) 17/03/2020 (Shift-III) Ans. (c): 'Exhale'- (Suspire), Antonym - 'Inhale' (Respire) Meaning of other words – Respite – Rest, Break, Inspire – (Motivate), Expel Eject 252. In the following question, out of the given four alternatives, select the one which is opposite in the meaning of the given word. Eager (a) Disinterested (b) Desire (c) Restless (d) Zealous SSC MTS 11-10-2017 (Shift-I) Ans. (a): Eager - (Desirous), Antonym - Disinterested (Indifferent) Meaning of other words – Restless – (Restive, Fidgety), Desire - Wish, Zealous – Fervent, Ardent 253. Select the most appropriate antonym of the word : Entrance (a) End (b) Closing (c) Exit (d) Opening SSC MTS – 22/08/2019 (Shift-I) Ans. (c): 'Entrance'- (Entry, Ingress), Antonym ‘Exit’ (Egress, Vent) Meaning of other words – End - finish, close Closing - Concluding Opening - Initial

F 254. Select the most appropriate ANTONYM to substitute the underlined word in the given sentence. This certainly was a feeble attempt to capture the culprit. (a) enviable (b) refraining (c) powerful (d) treble SSC GD 23/01/2023 (Shift-IV) Ans. (c) : The most appropriate antonym to substitute the underlined word feeble (weak) is powerful (strong). Other option give different meanings. Meaning of the other optionsEnviable Desirable Refraining Withhold, Avoid Treble Ternary

468

YCT

255. Select the most appropriate ANTONYM of the given word. Follow (a) Trail (b) Lead (c) Show (d) Heed SSC GD 08/02/2023 (Shift-IV) Ans. (b) : 'Lead' (guide) is the most appropriate antonym of the given word :- 'fellow'. The meaning of other options is– Trail :- track way. Show :- express Heed :- paying attention 256. Select the most appropriate ANTONYM of the underlined word. Her parents forbade her to marry her lover. (a) refused (b) allowed (c) declined (d) prevented SSC CGL (Tier-I) 25/07/2023 (Shift-IV) Ans. (b) : The most appropriate antonym of the underlined word 'forbade' is 'allowed'. Meaning of other optionsRefused - Rejected, Declined - Disown, Deny, Prevented - Fend, Inhibit, Hence, the correct option is (b). 257. Select the most appropriate ANTONYM of the given word. Fundamental (a) Neutral (b) Link (c) Impact (d) Least SSC CHSL (Tier-I) 02/08/2023 (Shift-I) Ans. (d) : The most appropriate antonym of the given word 'Fundamental' (basic) is Least' Meaning of other options – Neutral – Indifferent, Link – clinch, Impact – Effect, 258. Select the word that is opposite in meaning (ANTONYM) to the word given below. 'Fret' (a) Anxious (b) Anguish (c) Relax (d) Pine SSC CHSL (Tier-I) 21/03/2023 (Shift-IV) Ans. (c) : Fret (Peeve) - Antonym - 'Relax' (Rest) Anguism - Distress Pine - Cedar Anxious - Curious 259. Select the word that is opposite in meaning (ANTONYM) to the word given below. Fanatic (a) Militant (b) Moderate (c) Sectarian (d) Zealot SSC CHSL (Tier-I) 21/03/2023 (Shift-II) Ans. (b) : Fanatic - Antonym - Moderate Meaning of the wordsMilitant – Billicose Sectrarian – Communal Zealot – Partisan Antonyms

260. Select the most appropriate ANTONYM of the given word. Frivolous (a) Quiet (b) Petty (c) Wise (d) Idle SSC Constable GD-25/11/2021 Shift-III Ans. (c) : Frivolous (Trivial), Antonym – Wise (Intelligent) Meaning of the other OptionsQuiet – Calm Petty – Meager Idle – Inactive 261. Select the most appropriate ANTONYM of the given word. Flabby (a) Long (b) Plump (c) Firm (d) Stretched SSC MTS-05/07/2022 Shift-III Ans. (c) : Flabby (Soft), Antonym - firm (hard/Resolute) Meaning of the other OptionsLong – Lengthy Plump – Thick Stretched – Straighten 262. Select the most appropriate ANTONYM of the given word. Feeble (a) Fast (b) Wild (c) Strong (d) Fickle SSC CGL (Tier-I) 19/04/2022 Shift-III Ans. (c) : Feeble (Weak), Antonym - Strong (powerful) Meaning of the other wordsFast - Quick/Rapid Wild - Fierce/Feral Fickle - Unsteady/changeable 263. Select the most appropriate ANTONYM of the given word. Famished (a) Volatile (b) Starved (c) Satiated (d) Hesitant SSC CGL (Tier-I) 19/04/2022 Shift-III Ans. (c) : Famished (Starved/Hungry), Antonym Satiated (Satisfied) Meaning of the other OptionsVolatile - Uncertain/unstable Starved - Hungry Hesitant - shy/Unsure 264. Select the most appropriate antonym of the underlined word in the following sentence. The uneducated man was flummoxed by the legal document. (a) happy (b) annoyed (c) comfortable (d) scared SSC CHSL 27.05.2022 Shift-III

469

YCT

Ans. (c) : Flummoxed' (Baffled/at sea), Antonym Comfortable (Comfy/Cozy) Happy - Joyful/Glad/Pleased Annoyed - Angry/Bothered/Irritated Scared - afraid/fearful/Scary 265. Select the most appropriate antonym of the given word. Former (a) Additional (b) Featured (c) Latter (d) External SSC CHSL 26.05.2022 Shift-I Ans. (c) : Former (Priar/First), Antonym - Latter (Hind/Last) Meaning of the other Options Additional – Extra/Moreover Featured – Presented External – Outside/Exterior 266. Select the most appropriate antonym of the given word. Foreign (a) Strange (b) Distant (c) Native (d) Alien SSC Stenographer (Grade C & D) 15.11.2021 Shift-II Ans. (c) : Foreign (Outlandish), Antonym - Native (Aboriginal) Meaning of the other wordsStrange → Bizarre/Weird Distant → Afar/Far Alien → Foreigner 267. Select the most appropriate antonym of the given word. Feeble (a) Weak (b) Strong (c) Frail (d) Soft SSC Stenographer (Grade C & D) 11.11.2021 Shift-II Ans. (b) : Feeble (Weak/Incompetent),AntonymStrong (Powerful/energetic) Meaning of the other wordsWeak - Infirm Frail - Immoral/ lean Soft - Gentle/ Smooth 268. Select the most appropriate ANTONYM of the given word. FRIGID (a) Chilly (b) Warm (c) Frosty (d) Salty SSC CHSL (Tier-I) –13/04/2021 (Shift-I) Ans. (b): Frigid - (Cool, Icy), Antonym, Warm - (Hot) Meaning of other words – Salty - Saline, Chilly – Cool, Frosty – (Cold/Gelid) 269. Select the most appropriate antonym of the given word. Fecund (a) Barren (b) Polite (c) Genial (d) Rude SSC CGL (Tier-I) –23/08/2021 (Shift-I) Antonyms

Ans. (a): Fecund - (Fertile, Prolific), Antonym, Barren - (Sterile, Arid) Meaning of other words – Polite - Modest, Genial – Affable, Cordial, Rude Impolite 270. Select the most appropriate antonym of the given word FORLORN (a) solitary (b) elated (c) desperate (d) bereft SSC Sel. Post Phase-VIII (G.L.) 09.11.2020 (Shift-II)

Ans. (b): Forlorn - (Unhappy, Sad), Antonym - Elated - (Exultant, Exhilarated) Meaning of other words – Bereft – Med/Loopy Desperate – Hopeless Solitary - Lonely 271. Select the most appropriate antonym of the given word. Freed (a) Imprisoned (b) Released (c) Discharged (d) Liberated SSC Sel. Post Phase-VIII (M.L.) 09.11.2020 (Shift-III)

Ans. (a): Freed – Emancipate, Liberate, Antonym Imprisoned (Confined) Meaning of other word – Released – Discharged 272. Select the most appropriate ANTONYM of the given word. FLAW (a) defect (b) weakness (c) perfection (d) feature SSC CPO-SI – 23/11/2020 (Shift-I) Ans. (c): Flaw - (Fault), Antonym - Perfection (Excellence) Meaning of other words – Defect - Imperfection, Weakness - Frailty, Feature Attributes 273. Select the most appropriate ANTONYM of the given word. FEEBLE (a) strong (b) weak (c) foible (d) fault SSC CPO-SI – 23/11/2020 (Shift-I) Ans. (a): Feeble - (Frail), Antonym - Strong (Powerful) Meaning of other words – Weak - Infirm, Foible – Weakness, Fault - Flaw 274. Select the most appropriate ANTONYM of the given word. FLEXIBLE (a) rigid (b) ductile (c) elastic (d) pliable SSC CPO-SI – 25/11/2020 (Shift-II) Ans. (a): Flexible - (Supple), Antonym - Rigid (Stiff, Hard) Meaning of other words – Ductile - Malleable, Elastic – Whippy, Pliable Flexible

470

YCT

275. Select the antonym of the given word. FINITE (b) limited (a) finished (c) bound (d) endless SSC CGL (Tier-I) – 06/06/2019 (Shift-II) Ans. (d): 'Finite' - (Limited), Antonym - Endless (Infinite) Meaning of other words – Finished - Close, Concluded Bound - Restricted Limited - Demarcated/Finite 276. Select the most appropriate antonym of the given word. FOREIGN (a) rustic (b) rural (c) Indian (d) native SSC CGL (Tier-I) – 12/06/2019 (Shift-I) Ans. (d): Foreign - (Unfamiliar, Alien), Antonym "Native - (Domestic) Meaning of other words – Rustic - Pastoral Rural - Agrestic Indian - People of India 277. Select the most appropriate ANTONYM of the given word. FUTILE (a) hollow (b) worthless (c) barren (d) productive SSC CPO-SI – 11/12/2019 (Shift-I) Ans. (d): Futile - (Useless), Antonym - Productive (Fertile) Meaning of other words – Hollow- Void, Gossipy, Worthless- Valueless, Barren – Sterile, Arid 278. Select the most appropriate ANTONYM of the given word. FRAIL (a) fancy (b) elaborate (c) weak (d) robust SSC CPO-SI – 13/12/2019 (Shift-II) Ans. (d): Frail- (Weak, Infirm), Antonym - Robust (Strong) Meaning of other words – Fancy - (Imagination), Elaborate - (complex, Expand), Weak - (Feeble) 279. Select the most appropriate antonym of the given word. FLEXIBLE (a) pliant (b) elastic (c) stiff (d) variable SSC CHSL (Tier-I) –08/07/2019 (Shift-I) Ans. (c): Flexible- (Pliable, Supple), Antonym - Stiff(Rigid, Hard) Meaning of other words – Pliant- Ductile Elastic- Malleable Variable - Alterable 280. Select the most appropriate antonym of the given word. FEROCIOUS Antonyms

(b) cunning (d) gentle SSC CPO–SI–25/11/2020 (Shift-II) SSC CHSL (Tier-I) –05/07/2019 (Shift-III) Ans. (d): Ferocious- (Cruel, fierce), Antonym - Gentle (Modest, Humble) Meaning of other words – Prudent - Wise Cunning – Crafty, Sly Fierce – Violent 281. Select the most appropriate ANTONYM of the given word FRAGILE (a) Strong (b) Fancy (c) Slow (d) Fast SSC CHSL (Tier-I) 15/10/2020 (Shift-III) Ans. (a): Fragile - (Delicate), Antonym - 'Strong' (Robust) Meaning of other words – Fancy – Imagination, Slow – Unhurried, Fast – Quick, Swift 282. Select the correct antonym for the given word. Formidable (a) Appalling (b) Dangerous (c) Trivial (d) Scary SSC CHSL (Tier-I) 20/10/2020 (Shift-III) Ans. (c): 'Formidable'- (Intimidating), Antonym 'Trivial' (Insignificant) Meaning of other words – Appalling – Horrifying, Terrible Dangerous – Perilous Scary – Horrible 283. In the following question, out of the given four alternatives, select the one which is opposite in meaning of the given word. Former (a) Ancient (b) Above (c) Departed (d) After SSC MTS 7-10-2017 (Shift-I) Ans. (d): Former– (Previous), Antonym- After (Latter) Meaning of other words – Ancient – Primitive, Primordial Above – Exceeding Depart – Leave 284. In the following question, out of given four alternatives, select the one which is opposite in meaning of the given word. Forsake (a) Desert (b) Forgo (c) Give up (d) Revert SSC MTS 10-10-2017 (Shift-I) Ans. (d): Forsake – Relinquish, Antonym, Revert / Get back Meaning of other words – Desert – Abandon Forego – Disown, Leave Give up – Renounce 285. Select the antonym of the given word. Fragile (a) Novel (b) Tiny (c) Tough (d) Gross SSC MTS – 08/08/2019 (Shift-III)

471

(a) prudent (c) fierce

YCT

Ans. (c): 'Fragile'- (Feeble, delicate), Antonym 'Tough' (Hard) Meaning of other words – Novel – Fiction / New Tiny – Minuscule Gross – Total 286. Select the most appropriate antonym of the given word. Familiar (a) Usual (b) Common (c) Known (d) Strange SSC MTS – 05/08/2019 (Shift-III) Ans. (d): Familiar - (Intimate), Antonym - strange (Unknown) Meaning of other words – Usual – Normal Common – General Known – Familiar 287. Select the most appropriate antonym of the given word. Friendly (a) Felicity (b) Insanity (c) Hostility (d) Facility SSC MTS – 05/08/2019 (Shift-II) Ans. (c): Friendly - (Affable, Amiable), Antonym Hostility (Animosity) Meaning of other words– Insanity – Lunacy, Madness Facility – Amenity 288. Select the most appropriate antonym of the given word. Fluctuate (a) Stabilize (b) Mobilize (c) Waver (d) Stumble SSC MTS – 02/08/2019 (Shift-III) Ans. (a): Fluctuate- (Waver), Antonym - Stabilize (Steady) Meaning of other words – Mobilize - assemble Waver - Vibrate Stumble - Shuffle 289. Select the most appropriate antonym of the given word. Fertile (a) Barren (b) Civilized (c) Sane (d) Tough SSC CPO SI– 23/11/2020 (Shift-I) SSC MTS – 02/08/2019 (Shift-I) Ans. (a): Fertile - (Fecund), Antonym - Barren (Infertile) Meaning of other words – Civilized – Cultured, Advanced, Sane – Rational, Prudent Tough – Hard 290. Select the most appropriate antonym of the word Fickle (a) Constant (b) Hasty (c) Feeble (d) Mild SSC MTS – 20/08/2019 (Shift-II) Antonyms

Ans. (a): Fickle- (Capricious, Volatile), Antonym Constant (Persistent) Meaning of other words – Hasty – Quick, Hurried Feeble – Weak Mild – Placid / Tender 291. Select the most appropriate antonym of the word. FACILITATE (a) Ease (b) Encourage (c) Aid (d) Hinder SSC MTS – 16/08/2019 (Shift-II) Ans. (d): Facilitate- (Ease, Simplify), Antonym 'Hinder' (Hamper, Impede) Meaning of other words – Ease – Comfort Encourage – Motivate Aid – Help, Assist 292. Select the most appropriate antonym of the word. FEASIBLE (a) Profitable (b) Implausible (c) Attainable (d) Achievable SSC MTS – 14/08/2019 (Shift-I) Ans. (b): 'Feasible'- (Probable, Possible), Antonym 'Implausible' (Improbable, Unlikely) Meaning of other words – Profitable – Beneficial, Attainable – Achievable, Doable Achievable – Accomplishable 293. Select the antonym of the given word. Fatigued (a) Energized (b) Tired (c) Exhausted (d) Deprived SSC GD – 08/03/2019 (Shift-III) Ans. (a): 'Fatigued'- (Weariness, Enervated), Antonym - 'Energized' (Invigorated) Meaning of other words – Tired – Weary Exhausted – Tired/forworn Deprived – Bereaved 294. Select the antonym of the given word. FUTILE (a) Mutual (b) Fruitful (c) Collective (d) Common SSC GD – 02/03/2019 (Shift-II) Ans. (b): Futile - (Vain), Antonym - Fruitful (Useful) Meaning of other words – Mutual – Reciprocal Collective – Combined, Joint Common – General

G 295. Select the most appropriate ANTONYM of the given word. Grumble (a) Navigate (b) Stray (c) Raise (d) Praise SSC GD 27/01/2023 (Shift-I)

472

YCT

Ans. (d) : Grumble (Gabble, Complain) Antonym Praise (Admiration, Compliment) Meaning of the other optionsNavigate Guid Stray Diffuse, Misguided Raise uplift 296. Select the most appropriate ANTONYM of the given word. Gratification (a) Growth (b) Pleasure (c) Dissatisfaction (d) Blessing SSC GD 13/02/2023 (Shift-I) Ans. (c) : 'dissatisfaction' (discontent) is the most appropriate antonym of the given word 'Gratification'. The meaning of other options is– Growth :- developed, increment. Pleasure :- delight, joy. Blessing :- benediction. 297. Select the most appropriate ANTONYM of the given word. Gratitude (a) Appreciation (b) Mercy (c) Thanklessness (d) Care SSC GD 06/02/2023 (Shift-III) Ans. (c) : 'Thanklessness' is the most appropriate antonym of the given word 'Gratitude'. The meaning of other options is– Appreciation :- admiration Mercy :- the feeling that motivates compassion. Care :- guardianship, attention. 298. Select the most appropriate ANTONYM of the given word. Glad (a) Innocent (b) Jive (c) Haunt (d) Sad SSC CHSL (Tier-I) 03/08/2023 (Shift-II) Ans. (d) : The most appropriate antonym of the given word 'Glad' (happy) is 'sad' (gloomy). Meaning of other options – Innocent – Gullible, Guiltless, Jive – Gossip, chat, Haunt – Base, Perch, 299. Select the most appropriate ANTONYM of the given word. Grave (a) Frivolous (b) Serious (c) Native (d) Foreign SSC CHSL (Tier-I) 03/08/2023 (Shift-II) Ans. (a) : The most appropriate antonym of the given word ' Grave' (serious) is 'Frivalous' (Foppish). Other options give different meanings– Serious – Grave, Solemn, Native – maty, Connate, Foreing – Strange, External, 300. Select the most appropriate ANTONYM of the given word. Guilty (a) Ignorant (b) Innocent (c) Sad (d) Clever SSC Constable GD-26/11/2021 Shift-I Antonyms

Ans. (b) : Guilty (Convicted), Antonym, Innocent (Faultless) Meaning of the other OptionsIgnorant – Unaware Sad – Dejected Clever– Cunning 301. Select the most appropriate antonym of the given word. Gradual (a) Abrupt (b) Regular (c) Gentle (d) Slow SSC CGL (Tier-I) –13/08/2021 (Shift-I) Ans. (a): Gradual – (Steady, Successive), Antonym, Abrupt – (Sudden, Instantaneous) Meaning of other words – Regular – Consistent Gentle - Kind Slow – Moderate 302. Select the most appropriate antonym of the given word. GRADUALLY (a) Constantly (b) Abruptly (c) Steadily (d) Readily SSC CGL (Tier-II) 16/11/2020 (3 pm - 5 pm) Ans. (b): 'Gradually' (Progressively), Antonym Abruptly (Immediately, Suddenly) Meaning of other words – Constantly – Consistently Steadily - Uniformly Readily - Quickly 303. Select the antonym of the given word. GATHER (a) display (b) dispute (c) distract (d) disperse SSC CGL (Tier-I) – 06/06/2019 (Shift-III) Ans. (d): Gather - (Collect), Antonym- Disperse (Scatter) Meaning of other words – Dispute - Controversy Distract - Deviate Display - Show 304. Select the most appropriate antonym of the given word. GAUNT (a) Lean (b) Bent (c) Plump (d) Old SSC CHSL (Tier-I) –11/07/2019 (Shift-II) Ans. (c): Gaunt- (Emaciated), Antonym- Plump (Fat, Thick) Meaning of other words – Lean- Slim, Slender Bent- Twisted, Inclination Old- past, former 305. In the following question, out of the given four alternatives, select the one which is opposite in meaning of the given word. Grudge. (a) Spite (b) Averse (c) Dislike (d) Goodwill SSC MTS 9-10-2017 (Shift-I)

473

YCT

Ans. (d): Grudge - (Resentment Pique), Antonym Goodwill / Charity / Pity Meaning of other words– Spite – (Malice / Despise) Dislike – Loathe, Detest, Abhor 306. In the following question, out of the given four alternatives, select the one which is opposite in meaning of the given word. Gather (a) Choose (b) Unite (c) Crowd (d) Divide SSC MTS 10-10-2017 (Shift-II) Ans. (d): Gather – (Amass, Heap), Antonym - Divide (Separate, Split) Meaning of other words – Choose – Pick Unite – Integrate, Combine Crowd – Throng, Mob 307. Select the most appropriate antonym of the given word. Gloomy (a) Cloudy (b) Bright (c) Dull (d) Greedy SSC MTS – 05/08/2019 (Shift-I) Ans. (b): 'Gloomy'- (Dark), Antonym - Bright (Shine) Meaning of other words – Cloudy – Overcast, Murky Dull – Monotonous Greedy – Avaricious 308. Select the most appropriate antonym of the word. Glee (a) Joy (b) Woe (c) Fun (d) Mirth SSC MTS – 21/08/2019 (Shift-II) Ans. (b): Glee - (Delight, Elation), Antonym - woe (Misery, Sorrow) Meaning of other words – Fun – Enjoyment Mirth – Pleasure Joy – Happiness 309. Select the most appropriate antonym of the word. GRADUAL (a) Moderate (b) Rapid (c) Measured (d) Slow SSC MTS – 14/08/2019 (Shift-III) Ans. (b): Gradual - (Successive), Antonym - Rapid (Quick) Meaning of other words – Moderate – Modest Measured –Evaluated Slow – Unhurried Antonyms

H 310. Select the most appropriate ANTONYM of the given word. Hideous (a) Disgusting (b) Shocking (c) Attractive (d) Ghastly SSC MTS-14/07/2022 Shift-III Ans. (c) : Hideous (Abhorrent), Antonym Attractive(Alluring) Meaning of the other OptionsDisquieting – Restless Shocking – Appalling/Terrifying Ghastly – Terrible/Frightful 311. Select the most appropriate ANTONYM of the given word. Hilarious (a) Ridiculous (b) Pretentious (c) Usual (d) Humorless SSC MTS-07/07/2022 Shift-I Ans. (d) : Hilarious (Jubilant), Antonym - Humorless (Sad) Meaning of the other OptionsRidiculous – Humorous Pretentious – Conceited/Impertinent Usual – General 312. Select the most appropriate ANTONYM of the given word. HARMONY (a) Peace (b) Discord (c) Tranquility (d) Good will SSC MTS-06/07/2022 Shift-II Ans. (b) : Harmony (Unison), Antonym - Discord (Strife) Meaning of the other OptionsPeace – Silence Tranquility – Patience/Calmness Good will – Cordiality/Friendliness 313. Select the most appropriate antonym of the given word. Humble (a) Modest (b) Boring (c) Fair (d) Arrogant SSC Constable GD-06/12/2021 Shift-III Ans. (d) : Humble (Modest, Meek), Antonym – Arrogant (conceited, Haughty) Meaning of the other OptionsModest – Meek, Humble Boring – Dull, Monotonous Fair – Impartial, Unbiased 314. Select the most appropriate antonym of the given word. HARMONY (a) creativity (b) discord (c) friendship (d) understanding SSC Stenographer (Grade C & D) 11.11.2021 Shift-II

474

YCT

Ans. (b) : Harmony (unison/Balancing). Antonym Discord (Strife/Dissension) Meaning of the other wordsCreativity - Innovation Friendship - Cordiality/Amity Understanding - Discerning/Comprehension 315. Select the most appropriate antonym of the given word. Hospitable (a) Cordial (b) Gracious (c) Rude (d) Sick SSC Stenographer (Grade C & D) 11.11.2021 Shift-I Ans. (c) : Hospitable (Welcoming/Accommodating) Antonym, Rude (Impolite/uncivilized) Meaning of the other wordsGracious - kind/Submissive Cordial - Amicable/Friendly Sick - ill/Unwell 316. Select the most appropriate antonym of the given word. Harmony (a) Friendship (b) Peace (c) Rapport (d) Hatred SSC CGL (Tier-I) –23/08/2021 (Shift-I) Ans. (d): Harmony - (Unison), Antonym, Hatred (Scorn) Meaning of other words – Friendship – Amity Peace - Calm Rapport - Relation 317. Select the most appropriate ANTONYM of the given word. HUMILITY (a) modesty (b) timidity (c) vanity (d) servility SSC CPO-SI – 25/11/2020 (Shift-II) Ans. (c): Humility - (Modesty), Antonym – Vanity (Conceit) Meaning of other words – Modesty – Humility Timidity – Timorousness Servility – Sycophancy 318. Select the most appropriate antonym of the given word. Hope (a) Daring (b) Courage (c) Despair (d) Confidence SSC CHSL (Tier-I) –11/07/2019 (Shift-I) Ans. (c) : Hope- (Aspiration, Wish), Antonym Despair (Hopelessness) Meaning of other words – Daring – Intrepid Courage- Bravery Confidence- Assuredness 319. Select the most appropriate antonym of the given word. Hasty (a) Sudden (b) Cautious (c) Reckless (d) Urgent SSC CHSL (Tier-I) –04/07/2019 (Shift-I) Antonyms

Ans. (b): Hasty- (Quick, Hurried), Antonym - Cautious (Wary) Meaning of other words – Urgent – Crucial, Vital Sudden – Abrupt Reckless – Careless 320. Select the correct antonym for the given word. Harsh (a) Tough (b) Grave (c) Gentle (d) Bitter SSC CHSL (Tier-I) 26/10/2020 (Shift-I) Ans. (c): Harsh - (Rigid), Antonym - Gentle (Mild) Meaning of other words – Tough - Hard Grave – Serious Bitter- Pungent 321. Select the most appropriate ANTONYM of the given word. HEINOUS (a) Biased (b) Unhealthy (c) Unwilling (d) Agreeable SSC CHSL (Tier-I) 15/10/2020 (Shift-I) Ans. (d): Heinous - (Odious, abominable), Antonym - 'Agreeable' (Pleasant) Meaning of other words – 'Biased' - Prejudiced Unhealthy - Detrimental Unwilling – Unwished 322. Select the most appropriate ANTONYM of the given word. HARSH (a) Energetic (b) Lenient (c) Fragile (d) Robust SSC CHSL (Tier-I) 14/10/2020 (Shift-III) Ans. (b): Harsh– (Cruel), Antonym - Lenient (Liberal) Meaning of other words – Energetic – Lively fragile – Delicate Robust – Strong 323. In the following question, out of the given four alternatives, select the one which is opposite in meaning of the given word. Hail. (a) Rain (b) Storm (c) Greet (d) Dribble SSC MTS 9-10-2017 (Shift-I) Ans. (d): Hail - (Sleet), Antonym -Dribble (Drizzle) Meaning of other words – Rain – Shower Storm– Gale Greet – Welcome 324. Select the most appropriate antonym of the given word. Hidden (a) Concealed (b) Covered (c) Veiled (d) Apparent SSC MTS – 05/08/2019 (Shift-II)

475

YCT

Ans. (d): Hidden - (Secret), Antonym - Apparent (Clear) Meaning of other words – Covered – Masked Veiled – Mantled Concealed – Unrevealed 325. Select the most appropriate antonym of the word. Harsh (a) Honest (b) Dirty (c) Gentle (d) Rough SSC MTS – 20/08/2019 (Shift-III) Ans. (c): Harsh- (Rigid, Brutal), Antonym - Gentle (Lenient) Meaning of other words – Honest - Upright Dirty - Filthy Rough - Coarse 326. Select the most appropriate antonym of the word. HARMONY (a) Strife (b) Accord (c) Consensus (d) Agreement SSC MTS – 16/08/2019 (Shift-II) Ans. (a): 'Harmony'- (Adjustment), Antonym - strife (Discord) Meaning of other words – Accord – Concurrence Consensus – Unanimity, Concord Agreement – Compromise, Contract 327. Select the antonym of the given word. Hopeful (a) Optimistic (b) Pessimistic (c) Cheerful (d) Healthy SSC MTS – 09/08/2019 (Shift-III) Ans. (b): Hopeful - (Optimistic, Sanguine), Antonym - Pessimistic (Despondent) Meaning of other words– Optimistic – Sanguine Cheerful – hilarious, Gleeful Healthy – Fit, well 328. Select the antonym of the given word. HIDEOUS (a) Beautiful (b) Secretive (c) Detestable (d) Ugly SSC GD – 14/02/2019 (Shift-II) Ans. (a): Hideous - (Abhorrent, Abominable), Antonym - Beautiful (Pretty, Lovely) Meaning of other words – Secretive – Demure Detestable – Sordid, Disgusting Ugly – Unattractive, Disfigure 329. Select the antonym of the given word. HORIZONTAL (a) Parallel (b) Financial (c) Official (d) Vertical SSC GD – 11/02/2019 (Shift-II) Ans. (d): Horizontal - (Laterally), Antonym Vertical (Perpendicular) Meaning of other words – Parallel – Similar Financial - Pecuniary Official - Authoritative Antonyms

I 330. Select the appropriate antonym for the underlined word. Amenorrhea can cause a range of issues, including infertility. (a) sterile (b) barrenness (c) stance (d) fecund SSC MTS 08/05/2023 (Shift-III) Ans. (d) : The most appropriate antonym for the underlined word 'infertility' is word 'Fecund' (Fertile) Other words are– Sterile means – Barren Barrenness means – Impoverishment Stance means – attitude. These the correct answer is option (d) 331. Select the most appropriate ANTONYM to substitute the underlined word in the given sentence. The goods being sold are of inferior quality. (a) Superior (b) Anterior (c) Exterior (d) Ulterior SSC GD 12/01/2023 (Shift-II) Ans. (a) : The most appropriate antonym of the underlined word 'inferior' (lousy worse) in the given sentence is 'superior' (better, higher). Meaning of the other optionsAnterior - Anterograde Exterior - Outer Ulterior - Clandestine Correct SentenceThe goods being sold are of superior quality. 332. Select the most appropriate ANTONYM of the given word. Indolent (a) Unskilled (b) Sluggish (c) Lazy (d) Active SSC CGL (Tier-I) 17/07/2023 (Shift-II) Ans. (d) : Indolent (lazy) antonym , Active (Deedful) Meaning of other word – Unskilled – unartful Sluggish – slack Lazy – idle 333. Select the most appropriate ANTONYM to substitute the underlined word in the following sentence. They realised that the ghost was imperturbable. (a) exasperating (b) startling (c) excitable (d) contaminating SSC CHSL (Tier-I) 04/08/2023 (Shift-III) Ans. (c) : The most appropriate antonym of the above given underlined word' Imperturbable' (nonchalant) is 'excilable' (chalant). Meaning of other options– Exasperating – Tedious, Bored, Startling – miraculous, Sereaning Contaminating – Clog, Sully,

476

YCT

334. Choose the word that is opposite in meaning to the given word. Ignominy (a) Esteem (b) Disrepute (c) Stigma (d) Infamy SSC CHSL (Tier-I) 17/03/2023 (Shift-IV) Ans. (a) : Ignominy - Antonym - Esteem Meaning of other wordsDisrepute - Slander Stigma - Blur Infamy - Slander 335. Select the most appropriate antonym of the given word. INDISPENSABLE (a) FUNDAMENTAL (b) CRUCIAL (c) INESSENTIAL (d) BASIC SSC CHSL (Tier-I) 17/03/2023 (Shift-IV) Ans. (c) : Indispensable - Antonym - Inessential Meaning of other wordsBasic - Essential Crucial - Important Fundamental - Basic 336. Choose the word that is opposite in meaning to the given word. Inchoate (a) Artless (b) Refined (c) Articulate (d) Formless SSC CHSL (Tier-I) 17/03/2023 (Shift-III) Ans. (b) : Inchoate (Incomplete) - Antonym - Refined Meaning of other wordFormless - Shapeless Artless - Clumsy Articulate - Expressed, clear 337. Choose the word that is opposite in meaning to the given word. Inept (a) Wanting (b) Competent (c) Barred (d) Callow SSC CHSL (Tier-I) 10/03/2023 (Shift-III) Ans. (b) : Inept - Antonym - Competent Meaning of other wordBarred – Interrupted Callow – Inexperienced Wanting – Needy 338. Select the most appropriate ANTONYM of the bracketed word in the given sentence. It is irrelevance, at least to me, whether the grant was (improvident) or no. (a) tardy (b) thrifty (c) cautious (d) hungry SSC MTS-05/07/2022 Shift-I Ans. (b) : Antonym of the Bracketed word, Improvident (Spendthrift) in the above Sentence will be 'Thrifty' (Economical). Meaning of the other OptionsTardy – Slow/Sluggish Cautions – Wary/Careful Hungry – Famished Antonyms

339. Select the most appropriate ANTONYM of the given word. Impure (a) Contaminated (b) Segregated (c) Sterile (d) Concentrated SSC MTS-13/07/2022 Shift-I Ans. (c) : Impure (Contaminated/Polluted), Antonym Sterile (Germ-free) Contaminated – Muddy Segregated – Separated Concentrated – Thick/Staring 340. Select the most appropriate ANTONYM of the given word. Industrious (a) Persistent (b) Zealous (c) Diligent (d) Indolent SSC CGL (Tier-I) 21/04/2022 Shift-III Ans. (d) : Industrious (Hardworking/Diligent) Antonym-indolent (Inactive/Sluggish) Meaning of the other wordsPersistent – Stubborn/Firm Zealous – Enthusiastic/Ardent Diligent – Assiduous/Sedulous 341. Select the most appropriate antonym of the highlighted word. Hari's intimate nature has earned him many friends. (a) timely (b) unfriendly (c) passionate (d) affectionate SSC CHSL 27.05.2022 Shift-I Ans. (b) : Intimate (Familiar/Acquainted), Antonym 'Unfriendly' (Antisocial/Uncordinal) Meaning of the other wordsTimely - Punctual Passionate - Intense/Pointed Affectionate - Loving/Adoring 342. Select the most appropriate antonym of the highlighted word. It was an impetuous decision to buy a car when he didn't have a job. (a) sensible (b) dense (c) arguable (d) tangible SSC CHSL 26.05.2022 Shift-I Ans. (a) : Impetuous (Imprudent), Antonym - Sensible (practical) Meaning of the other OptionsDense – Thick/Compact Arguable – Disputable/Controversial Tangible – Corporeal/ Real 343. Select the most appropriate antonym of the given word. IMPEDE (a) disrupt (b) block (c) expedite (d) delay SSC Stenographer (Grade C & D) 15.11.2021 Shift-I

477

YCT

Ans. (c) : Impede (Interrupt). opposite/ expedite 348. Select the most appropriate antonym of the given word. (Hasten) Infirm Meaning of the other words(a) Feeble (b) Frail disrupt - Stop/ Obstruct (c) Strong (d) Weak Block - Barrier SSC CGL (Tier-I) –24/08/2021 (Shift-I) Delay - Postpone Ans. (c): Infirm - (Weak, Feeble), Antonym, Strong 344. Select the most appropriate antonym of the (Firm) given word. Meaning of other words – INCREASE Feeble - Infirm Frail - Immoral/ Weak (a) diminish (b) collect Weak - Powerless (c) widen (d) expand SSC Stenographer (Grade C & D) 15.11.2021 Shift-II 349. Select the most appropriate antonym of the given word. Ans. (a) : Increase (raise/Accelerates), Antonym Invade diminish (Curtail/Mitigate) (a) Surrender (b) Seize Meaning of the other words(c) Attack (d) Assault Collect → Gather/Accumulate SSC CGL (Tier-I) –16/08/2021 (Shift-I) Widen → Spread/Expand Ans. (a): Invade - (Infest), Antonym, Surrender (Capitulate) Expand → Enlarge/Extend 345. Select the most appropriate antonym of the Meaning of other words – Seize – Grasp, Snatch give word. Attack - Aggression INCORPORATE Assault - Invasion (a) exclude (b) assimilate 350. Select the most appropriate ANTONYM of the (c) integrate (d) absorb given word. SSC Stenographer (Grade C & D) 15.11.2021 Shift-II IRRELEVANT (a) consequential (b) trivial Ans. (a) : Incorporate (Consolidated) Antonym, (c) pointless (d) immaterial Exclude (Eliminate) SSC CPO-SI – 25/11/2020 (Shift-I) Meaning of the other wordsAns. (a): Irrelevant (Inappropriate ), Antonym – assimilate → Intermix/Imbibe Consequential ( Important ) Integrate → Combine/unite Meaning of other words – absorb → Suck/Soak Trivial – Unimportant 346. Select the most appropriate antonym of the Pointless – Senseless, Useless Immaterial - Insignificant given word. 351. Select the most appropriate ANTONYMN of indolent the given word. (a) energetic (b) listless IGNITE (c) torpid (d) languid (a) light (b) burn SSC CGL (Tier-II) 08.08.2022 Shift-II (c) kindle (d) extinguish Ans. (a) : Indolent (Inactive/Idle). antonym. Energetic SSC CPO-SI – 25/11/2020 (Shift-I) (Active/lively) Ans. (d): Ignite - (Kindle), Antonym - Extinguish Meaning of the other words(Quench) Listless – Indifferent/Disinterested Meaning of other words – Torpid – Inactive/Sluggish Light – Lamp, Illumination Languid – Lethargic/slow Burn – Sear/Singe 347. Select the most appropriate antonym of the Kindle - Inflame / Fire 352. Select the most appropriate ANTONYM of the given word. given word. IMPECCABLE INDUSTRIOUS (a) Blemished (b) Unsullied (a) indolent (b) assiduous (c) Flawless (d) Exquisite (c) diligent (d) productive SSC CGL (Tier-II) 08.08.2022 Shift-II SSC CPO-SI – 25/11/2020 (Shift-II) Ans. (a) : Impeccable (Errorless/Innocent). Antonym. Ans. (a): Industrious - (Hardworking), Antonym Blemished (Tarnish/disfigured) Indolent - (Lazy) Meaning of the other wordsMeaning of other words – Unsullied – Clean/Accurate Assiduous - Plodding Flawless – Faultless/ perfect Diligent - Painstaking Productive - Fruitful Exquisite – Excellent/ superb Antonyms

478

YCT

353. Select the most appropriate ANTONYM of the given word. IGNOBLE (a) abject (b) corrupt (c) craven (d) dignified SSC CPO-SI – 25/11/2020 (Shift-II) Ans. (d): Ignoble - (Despicable), Antonym Dignified (Reputable) Meaning of other words – Abject – Despondent Corrupt – Depraved Craven - Coward 354. Select the most appropriate ANTONYM of the given word. IMPUDENT (a) modest (b) arrant (c) brazen (d) blatant SSC CPO-SI – 23/11/2020 (Shift-II) Ans. (a) : Impudent- (Shameless, Immodest), Antonym - Modest (Meek, Humble) Meaning of other words – Arrant – Notorious Brazen – Shameless, Bold Blatant - Sturdy 355. Select the most appropriate antonym of the given word. ILLUMINATE (a) add (b) erase (c) light (d) darken SSC CGL (Tier-I) – 13/06/2019 (Shift-III) Ans. (d): Illuminate- (Light up), Antonym - Darken (Dim, Blacken) Meaning of other words – Add- Sum Erase- Delete Light- Illumination 356. Select the most appropriate ANTONYM of the given word. INITIATE (a) finish (b) begin (c) trigger (d) commence SSC CPO-SI – 09/12/2019 (Shift-II) Ans. (a): Initiate- (Begin), Antonym- Finish(Complete) Meaning of other words – Begin- Commence Trigger- Activate Commence- Start 357. Select the most appropriate ANTONYM of the given word. INFINITE (a) boundless (b) etemal (c) vast (d) limited SSC CPO-SI – 09/12/2019 (Shift-I) Ans. (d): Infinite - (Endless), Antonym- limited (Bounded, Restricted) Meaning of other words – Boundless – Limitless, Endless Vast – Huge, Extensive Antonyms

358. Select the most appropriate ANTONYM of the given word. INTENTIONAL (a) designed (b) voluntary (c) calculated (d) accidental SSC CPO-SI – 09/12/2019 (Shift-I) Ans. (d): Intentional- (Deliberate, Willful), Antonym accidental-(Casual) Meaning of other words – Designed- Structural Voluntary- Unconstrained Calculated- Premeditated 359. Select the most appropriate antonym of the given word. IMPEDE (a) Thwart (b) Retard (c) Hamper (d) Expedite SSC CPO-SI – 11/12/2019 (Shift-II) Ans. (d): Impede - (Destruct, Hindrance), Antonym Expedite (hasten, Quicken) Meaning of other words – Thwart - Depolarize Retard – Slowdown Hamper - Interrupt 360. Select the most appropriate ANTONYM of the given word. INNATE (a) basic (b) unassuming (c) simple (d) acquired SSC CPO-SI – 12/12/2019 (Shift-II) Ans. (d): Innate - (Inborn), Antonym - Acquired (Earned) Meaning of other words – Basic – Essential, Fundamental Unassuming – Humble Simple – Easy, Straightforward 361. Select the most appropriate ANTONYM of the given word. INSIDIOUS (a) treacherous (b) stealthy (c) sincere (d) astute SSC CPO-SI – 12/12/2019 (Shift-I) Ans. (c): Insidious - (Deceptive), Antonym - Sincere (Honest) Meaning of other words – Treacherous – Disloyal, Faithless Stealthy – Secret, Clandestine Astute – Shrewd, Sharp 362. Select the most appropriate antonym of the given word. INFERIOR (a) ulterior (b) exterior (c) interior (d) superior SSC CHSL (Tier-I) –04/07/2019 (Shift-III) Ans. (d): Inferior- (Lousy, Cheesy), Antonym Superior (better, Higher) Meaning of other words – Ulterior- Ambiguous, Covert Exterior- External Interior- Internal

479

YCT

363. Select the most appropriate antonym of the given word. INSTANT (a) immediate (b) current (c) quick (d) delayed SSC CHSL (Tier-I) –03/07/2019 (Shift-III) Ans. (d): Instant- (Immediate, Instantaneous), Antonym - Delayed (Postponed, suspend) Meaning of other words – Immediate- Quick, Prompt Current- Present, Flow Quick- Rapid 364. Select the most appropriate antonym of the given word. INORDINATE (a) undue (b) extreme (c) exorbitant (d) reasonable SSC CHSL (Tier-I) –02/07/2019 (Shift-II) Ans. (d): Inordinate- (Abnormal, Unusual), Antonym Reasonable (Logical, Rational) Meaning of other words – Undue- Unfair, Inappropriate Exorbitant- Excessive Extreme- Supreme 365. Select the most appropriate ANTONYM of the given word. Invincible (a) Secure (b) Supreme (c) Unstoppable (d) Vulnerable SSC CHSL (Tier-I) 26/10/2020 (Shift-II) Ans. (d): Invincible - (Unbeatable, Unconquerable), Antonym - Vulnerable (Susceptible) Meaning of other words – Secure – Protect, Defend Supreme – Chief, Foremost Unstoppable – Impregnable, Invincible 366. Select the most appropriate ANTONYM of the given word. INCOHERENT (a) Intelligible (b) Puzzling (c) Delightful (d) Inconsistent SSC CHSL (Tier-I) 16/10/2020 (Shift-I) Ans. (a): Incoherent- (Absurd), Antonym 'Intelligible' (Comprehensible) Meaning of other words – Puzzling - Bewilder Delightful – Delicious, Pleasing Inconsistent – Anomalous, Incompatible 367. Select the most appropriate ANTONYM of the given word IMPROMPTU (a) Planned (b) Fast (c) Unplanned (d) Appropriate SSC CHSL (Tier-I) 15/10/2020 (Shift-III) Ans. (a): 'Impromptu' - (Extempore), Antonym 'Planned' (Premeditated) Meaning of other words – Fast- Rapid, Quick Unplanned – Unexpected, Accidental Appropriate – Suitable, Relevant Antonyms

368. Select the most appropriate ANTONYM of the given word. INCITE (a) Instigate (b) Appeal (c) Provoke (d) Restrain SSC CHSL (Tier-I) 15/10/2020 (Shift-II) Ans. (d): 'Incite'- (Provoke, Egg on), Antonym 'Restrain' (Continence) Meaning of other words – Instigate – Stimulate Appeal - Request Provoke - Aggravate 369. Select the most appropriate ANTONYM of the given word. INSTANT (a) Diverse (b) Similar (c) Gradual (d) Prompt SSC CHSL (Tier-I) 13/10/2020 (Shift-III) Ans. (c): 'Instant'- (Immediate), Antonym - 'Gradual' (Slowly) Meaning of other words – Diverse – Various, Miscellaneous Similar – Identical, Alike Prompt – Expedite, Fast 370. Select the correct antonym for the given word. Identical (a) corresponding (b) different (c) similar (d) indistinguishable SSC CHSL (Tier-I) 20/10/2020 (Shift-III) Ans. (b): 'Identical'- (similar/Alike), Antonym 'Different' (Apart) Meaning of other words – Corresponding - Commensurate Similar-Analogous Indistinguishable – Nescient 371. In the following question, out of the given four alternatives, select the one which is opposite in meaning of the given word. Incite. (a) Encourage (b) Arouse (c) Drive (d) Prohibit SSC MTS 9-10-2017 (Shift-I) Ans. (d): Incite - (Agitate), Antonym - Prohibit (Stop/taboo) Meaning of other words – Arouse – Awaken/ Actuate Encourage – Promote Drive – Move/Campaign 372. In the following question, out of the given four alternatives, select the one which is opposite in meaning of the given word. Induce (a) Convince (b) Activate (c) Impel (d) Prevent SSC MTS 10-10-2017 (Shift-II) Ans. (d): Induce – (Motivate/Persuade), Antonym prevent (Stop/Inhibit) Meaning of other words – Convince – Persuade/Assure Activate – Stimulate/Actuate Impel – Propel/Force

480

YCT

373. In the following question, out of given four alternatives, select the one which is opposite in meaning of the given word. Immaculate (a) Clean (b) Bright (c) Pure (d) Foul SSC MTS 10-10-2017 (Shift-I) Ans. (d): Immaculate - (Serene/Clear), Antonym -Foul (Dirty/Muddy) Meaning of other words – Clean – Neat/Spotless Bright – Radiant/Glossy Pure – Pious/Chaste 374. In the following question, out of given four alternatives, select the one which is opposite in meaning of the given word. Impute (a) Defend (b) Accuse (c) Assign (d) Blame SSC MTS 10-10-2017 (Shift-I) Ans. (a): Impute - (Denounce), Antonym -Defend (Protect) Meaning of other words – Accuse – Denounce/Arraign Assign – Deliver/Hand Over Blame – Fault/Flaw 375. Select the most appropriate antonym of the given word. Impartial (a) Biased (b) Judicious (c) Involved (d) Impractical SSC MTS – 05/08/2019 (Shift-I) Ans. (a): 'Impartial'- (Fair/Unbiased), Antonym 'Biased' (Prejudice/partial) Meaning of other words – Judicious – Justifiable/Reasonable Involved – Converge/Conjoin/Interning Impractical – Fictitious/Implausible 376. Select the most appropriate antonym of the given word. Inexpensive (a) Dear (b) Reasonable (c) Popular (d) Liberal SSC MTS – 02/08/2019 (Shift-II) Ans. (a): Inexpensive - (Cheap), Antonym - Dear (Expensive) Meaning of other words – Reasonable – Logical/Justifiable Popular – Famous/Prevalent Liberal – Generous/Gentle 377. Select the most appropriate antonym of the given word. INFALLIBLE (a) Notorious (b) Flawless (c) Imperfect (d) Dependable SSC MTS – 19/08/2019 (Shift-III) Ans. (c): Infallible- (Impeccable/Flawless), Antonym - Imperfect (Deficient/Erroneous) Meaning of other words – Notorious – Infamous/Disreputable Flawless – Faultless/Impeccant Dependable – Reliable/Credible Antonyms

378. Select the most appropriate antonym of the word. ILLICIT (a) Prohibited (b) Lawful (c) Illegal (d) Illegible SSC MTS – 16/08/2019 (Shift-III) Ans. (b): Illicit - (Illegal/Unlawful), antonym - lawful (Legal/Legitimate) Meaning of other words – Prohibited – Taboo/Banned Illegal – Lawless Illegible – Vague/Obscure/Ambiguous 379. Select the most appropriate antonym of the word. ILLUSION (a) Fantasy (b) Hallucination (c) Mirage (d) Reality SSC MTS – 13/08/2019 (Shift-II) Ans. (d): Illusion (Delusion/Fallacy), Antonym Reality (Actuality/Tangibility) Meaning of other words – Fantasy – Imagination/Daydream Hallucination – Illusion/Superstitious Mirage – Delusion/Phantasm 380. Select the antonym of the given word. INDOLENT (a) Lively (b) Sluggish (c) Inert (d) Lifeless SSC GD – 02/03/2019 (Shift-I) Ans. (a): Indolent - (Idle/Lazy), Antonym - Lively (Active/Energetic) Meaning of other words – Sluggish – Lethargic/inactive Inert – Dormant/Indolent/Passive Lifeless – Dead/Inanimate/Deceased

J 381. Select the most appropriate ANTONYM of the underlined word in the given sentence. On the day of independence, a jubilant crowd from all over the country cheered and wept. (a) Triumphant (b) Gloomy (c) Cryptic (d) Elated SSC CHSL (Tier-I) 03/08/2023 (Shift-II) Ans. (b) : The most appropriate antonym of the above given underlined word 'Jubilant' is 'gloomy'. Meaning of other options – Triumphant – winner, Victorious, Gryptic – Hidden, Furtive, Elated – Excited, Agitated, 382. Select the most appropriate antonym of the given word. Jubilant (a) Happy (b) Depressed (c) Delighted (d) Young SSC Stenographer (Grade C & D) 11.11.2021 Shift-II Ans. (b) : Jubilant (Hilarious/overjoyed), Antonym/ Depressed (Gloomy/Sullen) Meaning of the other wordsHappy - Cheerful/Joyful Delighted - Pleased/Glad Young - Juvenile/Adolescent

481

YCT

383. Select the most appropriate antonym of the given word. JEOPARDIZE (a) threaten (b) imperil (c) protect (d) hazard SSC CHSL (Tier-I) –02/07/2019 (Shift-II) Ans. (c): Jeopardize- (Endanger/Risk), Antonym Protect (Defend/secure) Meaning of other words – Threaten- Intimidate/Bluster Imperil- hazard/Endonger Hazard- Danger/Risk 384. Select the most appropriate antonym of the given word. Jovial (a) Hesitant (b) Gloomy (c) Cheerful (d) Tired SSC MTS – 06/08/2019 (Shift-II) Ans. (b): Jovial- (Cheerful/Glad), Antonym - Gloomy (Sorrowful) Meaning of other words – Hesitant- Indecisive/Inconclusive Cheerful- Glad/Jolly Tired- Exhausted/weary 385. Select the most appropriate antonym of the given word. Jest (a) Happy (b) Deride (c) Sadden (d) Joke SSC CHSL (Tier-I) 16/10/2020 (Shift-I) Ans. (c): Jest – (Joke), Antonym, Sadden – (Depress, Melancholy) Meaning of other words – Happy – Pleased/Glad Deride – Taunt/Laugh Joke – Mockery/Humor 386. Select the most appropriate antonym of the given word. Jubilant (a) Loyal (b) Sluggish (c) Disappointed (d) Treason SSC MTS – 05/08/2019 (Shift-II) Ans. (c): Jubilant – (Hilarious), Antonym, Disappointed – (Hopeless/Despondent) Meaning of other words – Loyal – Faithful/True Sluggish – Idle/Dormant/Inactive Treason – Insurgency/Rebellion 387. Select the most appropriate antonym of the given word. Jealous (a) Envious (b) Trusting (c) Lively (d) Inert SSC CPO-SI – 23/11/2020 (Shift-II) Ans. (b): Jealous – (Suspicious) Antonym, Trusting – (Faithful) Meaning of other words – Envious - Spiteful Lively – Animate Inert - Inactive Antonyms

K 388. Choose the word that is opposite in meaning to the given word. 'Knack' (a) Vile (b) Process (c) Lesson (d) Inability SSC CHSL (Tier-I) 09/03/2023 (Shift-IV) Ans. (d) : knack - Antonym - Inability Meaning of other wordVile Despicable Lesson Paragon / Task Process Growth 389. Select the most appropriate antonym of the given word. Knotty (a) Lax (b) Stop (c) Simple (d) Complex SSC CHSL (Tier-I) –04/07/2019 (Shift-II) Ans. (c): Knotty – (Complex), Antonym, Simple – (Easy) Meaning of other words – Lax – Nonchalant Stop - Pause Complex - Intricate 390. Select the most appropriate antonym of the given word. Keen (a) Eager (b) Adept (c) Jovial (d) Dull SSC CPO-SI – 11/12/2019 (Shift-I) Ans. (d): Keen – (Zeal/Eager), Antonym, Dull – (Idle/Inert) Meaning of other words – Eager – Desirous/Avid Adept – Skillful/Dexterous Jovial – Cheerful/Genial 391. Select the most appropriate antonym of the given word. Knave (a) Rogue (b) Cheerful (c) Civilized (d) Blunt SSC MTS – 08/08/2019 (Shift-II) Ans. (c): Knave – (Deceiver/Cheat), Antonym, Civilized – (Decent/Well behaved) Meaning of other words – Rogue – Rascal Cheerful – Gleeful/Glad Blunt – Edgeless/Obtuse 392. Select the most appropriate antonym of the given word. Knack (a) Ability (b) Intimidate (c) Lunacy (d) Inability SSC CHSL (Tier-I) –02/07/2019 (Shift-II) Ans. (d): Knack – (Proficiency/Dexterity), Antonym, Inability – (Incapability/Incompetence) Meaning of other words – Ability – Caliber/Capability Intimidate – Threaten/Browbeat Lunacy – Madness/Insanity

482

YCT

L 393. Select the most appropriate ANTONYM of the given word. Liability (a) Resource (b) Responsibility (c) Asset (d) Strike SSC GD 10/01/2023 (Shift-I) Ans. (c) : Liability (Responsibility, burden), Antonym Asset (Wealth, Money) Meaning of the other optionsResource - Supply, Stockpile Responsibility - Accountability Strike - Punch, hit 394. Select the most appropriate ANTONYM of the underlined word. Lucy presented a lucid account of her achievements before the committee. (a) intelligible (b) transparent (c) ambiguous (d) orderly SSC CGL (Tier-I) 17/07/2023 (Shift-II) Ans. (c) : Antonym of the above underlined word 'Lucid' is 'ambiguous' Meaning of other word– Intelligible – Easy. Transparent – Limpid Orderly – systematic 395. Select the most appropriate antonym of the underlined word in the given sentence. The loyal nature of the princess was one of the reasons that the king trusted her very much. (a) unyielding (b) livid (c) treacherous (d) infamous SSC CHSL (Tier-I) 14/08/2023 (Shift-IV) Ans. (c) : Treacherous – "Guilty of betrayal or involving deception" is the antonym of underlined word loyal – (Faithful in allegiance to someone) given in the sentence. Meaning expressed by other wordsUnyielding –Not gluing way to pursuer, hard or solid. Livid – Extremely angry Infamous – Famous for being bad (Notorious) 396. Select the most appropriate ANTONYM of the given word. Liable (a) Peaceful (b) Invulnerable (c) Tending (d) Responsible SSC CHSL (Tier-I) 04/08/2023 (Shift-III) Ans. (b) : The most appropriate antonym of the above given word 'Liable' (responsible) is 'Invulnerable' (Unhurt). Meaning of other options – Peaceful – Quiet, Steady, Tending – Direction, Keeping, Responsible – Amenable, Liable, Antonyms

397. Choose the word that is opposite in meaning to the given word. Leisure (a) Retire (b) Worship (c) Work (d) Play SSC CHSL (Tier-I) 17/03/2023 (Shift-III) Ans. (c) : Leisure (Leave) - Antonym - Work (Act) Meaning of other wordPlay - Sport Retire - Homage Worship - Refuge 398. Select the most appropriate ANTONYM of the underlined word in the given sentence. I do not feel it is a laudable plan. (a) noticeable (b) precious (c) commendable (d) disrespected SSC MTS-14/07/2022 Shift-III Ans. (d) : Laudable – (Praiseworthy/Admirable), Antonym – Disrespectful (Rude/Uncivilized) Meaning of the other OptionsNoticeable – Noteworthy Precious – Valuable Commendable – Admirable 399. Select the most appropriate ANTONYM of the given word. Linger (a) Annoy (b) Leave (c) Entertain (d) Choose SSC CGL (Tier-I) 12/04/2022 Shift-III Ans. (b) : Linger- (Stay), Antonym- Leave (Departure/Exit) Meaning of the other OptionsAnnoy- Irritate/vex Entertain- Amuse Choose- Prefer/Select 400. Select the most appropriate antonym of the given word. LAVISH (a) excessive (b) luxurious (c) tiny (d) frugal SSC CHSL (Tier-I) –06/08/2021 (Shift-I) Ans. (d): Lavish - (Extravagant/Prodigal), Antonym, Frugal - (Economical/Thrifty) Meaning of other words – Luxurious – Epicure Excessive – Exorbitant Tiny - Minute 401. Select the antonym of the given word. Liberty (a) slavery (b) freedoms (c) autonomy (d) reservation SSC CGL (Tier-I) – 07/06/2019 (Shift-III) Ans. (a): 'Liberty' - (Freedom), Antonym- Slavery (Thrall) Meaning of other words – Autonomy - Self-rule Reservation - Preserve Freedom - Independence

483

YCT

402. Select the most appropriate ANTONYM of the given word. LAMENT (a) bleed (b) bemoan (c) appeal (d) celebrate SSC CPO-SI – 09/12/2019 (Shift-I) Ans. (d): Lament - (Moan/Deplore), Antonym celebrate - (Rejoice) Meaning of other words– Bleed - Lose of blood Bemoan – Lament/weep Appeal – Cassation 403. Select the most appropriate antonym of the given word. LOYALTY (a) faith (b) treason (c) trust (d) fidelity SSC CHSL (Tier-I) –08/07/2019 (Shift-III) Ans. (b): Loyalty- (Devotion/Dedication), AntonymTreason (Insurgency/Rebellion) Meaning of other words – Faith- Belief Trust- Belief/faith Fidelity- Loyalty/Integrity 404. Select the most appropriate antonym of the given word. Loiter (a) Loaf (b) Hasten (c) Dawdle (d) Repel SSC MTS – 07/08/2019 (Shift-III) Ans. (b): Loiter- (Delay), Antonym - Hasten (Hurry) Meaning of other words – Loaf - Bun Dawdle - delay Repel - Setback 405. Select the most appropriate antonym of the given word. Linger (a) Quicken (b) Stop (c) Wait (d) Defer SSC MTS – 06/08/2019 (Shift-II) Ans. (a): Linger- (Stay), Antonym - Quicken (Hasten) Meaning of other words – Stop – Pause Wait – Stay Defer- Avoid 406. Select the antonym of the given word. LATENT (a) Forbidding (b) Hidden (c) Artificial (d) Obvious SSC GD – 18/02/2019 (Shift-III) Ans. (d): Latent - (Secret), Antonym - Obvious (Explicit/direct) Meaning of other words – Forbidding – Undesirable/Deformed Hidden – Invisible Artificial – Synthetical/ Man-made 407. Select the most appropriate antonym of the given word. LENIENT (a) Generous (b) Helpful (c) Strict (d) Defensive SSC GD – 11/02/2019 (Shift-III) Antonyms

Ans. (c): Lenient - (Generous), Antonym - Strict (Harsh/Rigid) Meaning of other words – Generous - Liberal Helpful – Supportive/useful Defensive - Protective

M 408. Choose the word that is opposite in meaning to the given word. Malign (a) Align (b) Cheerful (c) Harangue (d) Praise SSC MTS 10/05/2023 (Shift-I) Ans. (d) : The most appropriate opposite word for 'Malign' is 'praise'. The meaning of the other words isAlign means straighten Cheerful means Glad, happy Harangue means Lecture Thus, the correct answer is option (d). 409. Select the most appropriate ANTONYM of the given word. Meagre (a) Generous (b) Racial (c) Stupid (d) Regional SSC CHSL (Tier-I) 02/08/2023 (Shift-I) Ans. (a) : The most appropriate antonym of the given word 'meagre' (little) is 'Generous' (bountiful). Meaning of other options– Racial – Vernacular, Stupid – Foolish, Regional – Territorial, 410. Select the most appropriate ANTONYM of the underlined word in the given sentence. It is a misconception that the more you learn, the more likely you are to succeed. (a) Fallacy (b) Illusion (c) Certainty (d) Hallucination SSC CHSL (Tier-I) 08/08/2023 (Shift-II) Ans. (c) : The most appropriate antonym of the above underlined word 'misconception' (Illusion) is certainty'. Meaning of other options – Fallacy – Illusion, Phantasm, Illusion –Fallacy, Chimera, Hallucination – Reverie, Delusion, 411. Select the most appropriate ANTONYM of the given word. Moil (a) Cultivate (b) Grow (c) Discourage (d) Work SSC MTS-12/07/2022 Shift-I Ans. (c) : Moil (Toil), Antonym - Discourage (Demoralize/Daunt) Meaning of the other OptionsCultivate – Develop/Evolve Grow – Surmount Work – Duty/Job

484

YCT

412. Select the most appropriate ANTONYM of the given word. Modest (a) Conceited (b) Glum (c) Unhappy (d) Sullen SSC CGL (Tier-I) 11/04/2022 Shift-I Ans. (a) : Modest (Humble), Antonym - conceited (Haughty) Meaning of the other OptionsGlum – Gloomy Unhappy – Sad Sullen – Sorrowful 413. Select the most appropriate antonym of the given word. multiple (a) double (b) single (c) triple (d) many SSC CGL (Tier-I) 20/04/2022 Shift-III Ans. (b) : Multiple (Several/Various), Antonym Single (Solitary) Meaning of the other wordsDouble - Duplex triple - Three fold many - Numerous/Multiple 414. Select the most appropriate antonym of the given word. Mettle (a) Cowardice (b) Caution (c) Stamina (d) Attention SSC Constable GD-02/12/2021 Shift-I Ans. (a) : Mettle (Enthusiasm/Courage), Antonym cowardice (Timidity) Meaning of the other OptionsCaution –Warning/Alert Stamina – Endurance attention – Notice/Concentration 415. Select the most appropriate antonym of the given word. Moderate (a) Easy (b) Extreme (c) Medium (d) Temperate SSC Stenographer (Grade C & D) 11.11.2021 Shift-I Ans. (b) : Moderate (Confined/Medium) antonymextreme (Excessive) Meaning of the other wordsEasy - Simple/Effortless Medium - Middle/Average Temperate - Abstinent/Average 416. Select the most appropriate ANTONYM of the given word. Maintain (a) Preserve (b) Abandon (c) Protect (d) Sustain SSC Constable GD-22/11/2021 Shift-II Antonyms

Ans. (b) : Maintain (Preserve), Antonym, abandon (Renounce) Meaning of the other OptionsPreserve – Protect Protect – Save Sustain – Keep/Continue 417. Select the most appropriate antonym of the given word. MODERN (a) ancient (b) well known (c) creaky (d) popular SSC Stenographer (Grade C & D) 12.11.2021 Shift-II Ans. (a) : 'Modern' (Latest/Recent), antonym - ancient (Primitive/old) Meaning of the other wordsWell known - Familiar Creaky - Crepitate Popular - Famous 418. Select the most appropriate antonym of the given word. Malicious (a) Culpable (b) Biased (c) Perverted (d) Decent SSC CHSL (Tier-I) –19/04/2021 (Shift-I) Ans. (d): Malicious - (Spiteful), Antonym, Decent (Civilized) Meaning of other words – Culpable - Criminal Biased – Prejudiced Perverted – Deterred/Distorted 419. Select the most appropriate ANTONYM of the given word. MENACE (a) balance (b) peril (c) threat (d) comfort SSC CPO-SI – 24/11/2020 (Shift-I) Ans. (d): 'Menace'- (Hazard/Peril), Antonym 'Comfort' (Convenient/Ease) Balance - Equilibrium Peril - Danger Threat - Reprimand 420. Select the most appropriate ANTONYM of the given word. MUSTER (a) convene (b) mobilize (c) disperse (d) assemble SSC CPO-SI – 24/11/2020 (Shift-I) Ans. (c): 'Muster' - (Collection), Antonym 'Disperse' (Spread) Meaning of other words – Convene – Call/Summon Mobilize - Organize Assemble - Amass/Rally 421. Select the most appropriate antonym of the given word. MEAGRE (a) Inadequate (b) plentiful (c) scanty (d) premium SSC CHSL (Tier-I) – 13/10/2022 (Shift-I) SSC CGL (Tier-I) – 11/06/2019 (Shift-I)

485

YCT

Ans. (b): Meager - (Scant/little), Antonym - Plentiful (Abundant/Bountiful) Meaning of other words – Scanty- Merger Premium- Installment Inadequate- Insufficient/Scarce 422. Select the most appropriate ANTONYM of the given word. MORTALITY (a) death (b) birth (c) lethality (d) fatality SSC CPO-SI – 12/12/2019 (Shift-I) Ans. (b): Mortality - (Decease/Death), Antonym - Birth (Genesis/Life) Meaning of other words – Death – Demise/decease Lethality – Deadly/fatal Fatality - Calamity 423. Select the most appropriate antonym of the given word. MENTAL (a) strength (b) physical (c) ability (d) brainy SSC CHSL (Tier-I) –09/07/2019 (Shift-III) Ans. (b): Mental- (psycho/psychic) Antonym- Physical (Material/bodily) Meaning of other words – Ability- Capability/Caliber Strength- Competence/Ability Brainy- Sensible 424. Select the most appropriate antonym of the given word. MISERY (a) bliss (b) poverty (c) censure (d) dearth SSC CHSL (Tier-I) –05/07/2019 (Shift-III) Ans. (a): Misery- (Suffering/distress), Antonym- Bliss (Ecstasy) Meaning of other words – Poverty- Impoverishment/penury Censure- Condemn/Criticism Dearth- Scarcity/Famine 425. Select the most appropriate antonym of the given word. MOROSE (a) sullen (b) cheerful (c) unsatisfied (d) mournful SSC MTS–07/08/2019 (Shift-II) SSC CHSL (Tier-I) –02/07/2019 (Shift-III) Ans. (b): Morose- (Gloomy/Sorrowful), Antonyms Cheerful (Gleeful/Hilarious) Meaning of other words – Mournful – Distressful Unsatisfied – Discontent Sullen – Gloomy/Melancholy 426. Select the most appropriate antonym of the given word. Minuscule (a) dwarf (b) small (c) gigantic (d) meagre SSC CHSL (Tier-I) –01/07/2019 (Shift-III) Antonyms

Ans. (c): Minuscule- (Tiny/Minute), Antonym 'Gigantic' (Huge/Enormous) Meaning of other words – Dwarf- Shorty/Pygmy Small- Miniature/minor Meagre- Scant/Insignificant 427. Select the most appropriate ANTONYM of given word. MAESTRO (a) Teacher (b) Connoisseur (c) Enthusiast (d) Amateur SSC CHSL (Tier-I) 19/03/2020 (Shift-III) Ans. (d): Maestro – (Expert/Artist), Antonym Amateur (Novice/Inexperienced ) Meaning of other words – Connoisseur – Specialist/Expert Enthusiast – Ardent/keen Teacher – Educator/Pedagogue 428. Select the most appropriate antonym of the given word. MEDDLE (a) Fortify (b) Prize (c) Support (d) Ignore SSC CHSL (Tier-I) 17/03/2020 (Shift-II) Ans. (d): Meddle – (Interfere/Intervene), Antonym Ignore (Neglect) Meaning of other words – Fortify – Strengthen Prize – Reward Support –Help/Assist 429. Select the most appropriate ANTONYM of the given word. MALIGNANT (a) Indignant (b) Hesitant (c) Benign (d) Fatal SSC CHSL (Tier-I) 13/10/2020 (Shift-I) Ans. (c): Malignant – (Harmful/Detrimental), Antonym - 'Benign' (Kind/Benevolent) Meaning of other words – Indignant – Angry Hesitant – Indecisive Fatal – Deadly Malignant – Deadly 430. Select the most appropriate ANTONYM of the given word. METROPOLITAN (a) Provincial (b) Federal (c) Insular (d) National SSC CHSL (Tier-I) 13/10/2020 (Shift-III) Ans. (a): Metropolitan- (Municipal/Urban), Antonym Provincial (territorial) Meaning of other words – Federal – Confederate/Combined Insular – Singly National– Countrywide/Governmental 431. Select the antonym of the given word. MODEST (a) Vain (b) Furious (c) Dishonest (d) Simple SSC MTS – 08/08/2019 (Shift-II)

486

YCT

Ans. (a): 'Modest'- (Humble), Antonym - Vain (Egocentric/Conceited) Meaning of other words – Furious - violent/Wrathful Dishonest - Insidious Simple - Easy/General 432. Select the antonym of the given word. MALEVOLENT (a) Kind (b) Positive (c) Cruel (d) Well-mannered SSC GD – 18/02/2019 (Shift-III) Ans. (a): Malevolent - (Evil/Deceitful), Antonym Kind (Generous/Gentle)~ Meaning of other words – Positive - Hopeful Cruel – Heartless/Brutal Well-mannered – Elegant/Courteous

N 433. Select the most appropriate ANTONYM for the underlined word in the following sentence. He neglected his duties and was dismissed from his job. (a) Succeeded (b) Ignored (c) Attended (d) Scorned SSC GD 12/01/2023 (Shift-II) Ans. (c) : The most appropriate antonym for the underlined word 'neglected' (ignored) is 'Attended' pay attention, noticed. Meaning of the other optionsSucceed - Triumph, Accomplish Ignored - Neglected Scorned - Hate Correct SentenceHe attended his duties and was dismissed from his job. 434. Select the most appropriate ANTONYM of the word 'Naive' from teh given sentence. After years of working in politics, she had become cynical and jaded, convinced that all politicians were corrupt and that the system was rigged against the people. (a) jaded (b) Corrupt (c) Convinced (d) Cynical SSC CGL (Tier-I) 27/07/2023 (Shift-III) Ans. (d) : The most appropriate antonym of the above given word 'Naive' is 'Cynical'. Meaning of other optionsJaded - Tired, Sick, Corrupt - Impaired, Depraved, Convinced - Expectant, assured, Thus, the correct answer is option (d). 435. Choose the word that is opposite in meaning to the given word. Nadir (a) Climax (b) Base (c) Bottom (d) Docile SSC CHSL (Tier-I) 09/03/2023 (Shift-IV) Antonyms

Ans. (a) : Nadir - Antonym - 'Climax' Meaning of other wordBottom – Base, Surface Base – Foundation Docile – Courteous 436. Select the most appropriate ANTONYM of the given word. Negligent (a) Careful (b) Indifferent (c) Strange (d) Faithful SSC CGL (Tier-I) 13/04/2022 Shift-II Ans. (a) : Negligent (Careless), Antonym - careful (Wary) Meaning of the other OptionsIndifferent → Apathetic strange → Bizarre Faithful → Reliable 437. Select the most appropriate ANTONYM of the given word. Never (a) Always (b) Rarely (c) Sometimes (d) Seldom SSC Constable GD-09/12/2021 Shift-II Ans. (a) : Never – (Not at all, Nevermore), Antonym, always (Eternally) Meaning of the other OptionsRarely – Infrequently, Seldom Sometimes – Occasionally Seldom – Sometimes 438. Select the most appropriate antonym of the given word. Negligent (a) Careful (b) Calm (c) Careless (d) Composed SSC Stenographer (Grade C & D) 11.11.2021 Shift-I Ans. (a) : Negligent (Reckless/Carefree), Antonym careful (Cautious/alert) Meaning of the other wordsCalm - Quiet/Tranquil Careless - Irresponsible/Neglectful Composed - Calm/Peaceful 439. Select the most appropriate antonym of the given word. NOVEL (a) fresh (b) old (c) original (d) new SSC CHSL (Tier-I) –08/07/2019 (Shift-II) Ans. (b): Novel- (New/Modern), Antonym - Old (Ancient/Aged) Meaning of other words – Fresh- Verdant Original- Genuine/Factual New- Recent/Modern 440. Select the most appropriate antonym of the given word. NEBULOUS (a) Common (b) Clear (c) Mutual (d) Familiar SSC CHSL (Tier-I) 18/03/2020 (Shift-II)

487

YCT

Ans. (b): 'Nebulous'- (Obscure/Vague), Antonym 'Clear' (Apparent/Articulate) Meaning of other words – Common – Usual/General Mutual – Reciprocal/Fraternal Familiar – Well-know/Friendly 441. Select the most appropriate ANTONYM of the given word. NEAT (a) Messy (b) Clear (c) Sharp (d) Strong SSC CHSL (Tier-I) 14/10/2020 (Shift-I) Ans. (a): Neat– (Clean/Fresh), Antonym - 'Messy' (Dirty/Muddy) Meaning of other words – Clear– Apparent/Spotless Sharp– Fast/Intense Strong– Firm/Polent 442. Select the antonym of the given word. Notorious (a) Efficient (b) Popular (c) Optimistic (d) Infamous SSC CGL (Tier-I)– 10/06/2019 (Shift-I) SSC MTS – 13/08/2019 (Shift-III) Ans. (b): Notorious- (Infamous), Antonym - Popular (Famous) Meaning of other words – Efficient - Adept/Skillful Optimistic - Sanguine/Hopeful Infamous - Ignoble

445. Select the most appropriate ANTONYM of the given word. Overrun (a) Surrender (b) Inundate (c) Storm (d) Invade SSC MTS-13/07/2022 Shift-I Ans. (a) : Overrun (Crush), Antonym - Surrender (submit/Capitulate) Meaning of the other OptionsInundate – Flood/Submerge storm – Typhoon/Tornado Invade – Attack/Raid 446. Select the most appropriate ANTONYM of the given word. Obscure (a) Veiled (b) Cloudy (c) Apparent (d) Blurred SSC CGL (Tier-I) 18/04/2022 Shift-I Ans. (c) : Obscure (Vague/Ambiguous), Antonym Apparent (clear/obvious) Meaning of the other OptionsVeiled - Indirect/Latent Cloudy - Hazy/Misty Blurred - Vague/unclear

447. Select the most appropriate ANTONYM of the given word. Obdurate (a) Rigid (b) Amenable (c) Callous (d) Secure SSC CGL (Tier-I) 20/04/2022 Shift-III Ans. (b) : obdurate (Stubborn/Obstinate), Antonym Amenable (Responsible/Docile) 443. Select the most appropriate ANTONYM of the Meaning of the other wordsunderlined word. Her room looks very messy Rigid – Harsh/Tough (a) Organised (b) Chaotic Callous – Unkind/Ruthless (c) Untidy (d) Disordered SSC CGL (Tier-I) 21/07/2023 (Shift-II) Secure – Safe Ans. (a) : The most appropriate antonym of the 448. Select the most appropriate ANTONYM of the underlined word 'messy' is 'organised'. given word. Meaning of other optionsOptional Chaotic - Anarchic, (a) Elective (b) Arbitrary Untidy - Dirty, (c) Compulsory (d) Voluntary Disordered - Unarranged, SSC CGL (Tier-I) 12/04/2022 Shift-II Thus, the correct answer is option (a). Ans. (c) : Optional (Alternative Antonym 444. Select the most appropriate ANTONYM of the Compulsory (Mandatory) given word. Meaning of the other OptionsObedient Elective - Electoral/Qualifying (a) Compliant (b) Submissive Arbitrary - Whimsical/Capricious (c) Cheery (d) Resistant SSC MTS-12/07/2022 Shift-I Voluntary - Wilful/Free of charge Ans. (d) : Obedient (Compliant), Antonym - Resistant 449. Select the most appropriate ANTONYM of the (Obstructive) given word. Meaning of the other OptionsOutstanding Compliant – Dutiful (a) Astounding (b) Ordinary Submissive – Docile/Courteous (c) Remarkable (d) Honest Cheery – Glad SSC MTS-18/07/2022 Shift-I

O

Antonyms

488

YCT

Ans. (b) : Outstanding (Excellent/Impressive), Antonym - Ordinary (Normal/Common) Meaning of the other wordsAstounding – Surprising/Amazing Remarkable – Extraordinary Honest – Frank/Candid 450. Select the most appropriate ANTONYM of the given word. Overlook (a) Neglect (b) Notice (c) ignore (d) Visualize SSC Constable GD-03/12/2021 Shift-II Ans. Ans. (b) : Overlook (Ignore), Antonym– Notice (Attention) Meaning of the other OptionsNeglect – Disregard, Disparage Ignore – Defy, Neglect Visualize – Imagine , Dream 451. Select the most appropriate antonym of the given word. Oppose (a) Assist (b) Quarrel (c) Favour (d) Control SSC Constable GD-06/12/2021 Shift-III Ans. (c) : Oppose (Resist), Antonym, favour (Defend, Bias) Meaning of the other OptionsAssist – Help Quarrel – Fight, Dispute Control – Curb, command 452. Select the most appropriate ANTONYM of the given word. Obscure (a) Doubtful (b) Dim (c) Clear (d) Vague SSC Constable GD-24/11/2021 Shift-II Ans. (c) : Obscure (Vague) Antonym - Clear (Apparent) Meaning of the other OptionsDoubtful - Suspect/Suspicious Dim - Hazy/Misty Vague - Obscure/Unclear

454. Select the most appropriate antonym of the given word Obese (a) fat (b) overweight (c) thin (d) plump SSC Sel. Post Phase-VII (M.L.) 15.10.2019 (Shift-I)

Ans. (c): Obese – Fat, Antonym - Thin - Slender Meaning of other words – Fat - Thick Overweight - Obese Plump - Voluminous 455. Select the most appropriate ANTONYM of the given word. OFFBEAT (a) conventional (b) weird (c) unique (d) freaky SSC CPO-SI – 11/12/2019 (Shift-I) Ans.(a): Offbeat- (Way-out), Antonym - Conventional (Traditional) Meaning of other words – Weird- Peculiar/Bizarre Unique- Exclusive/Matchless Freaky- Strange/Abnormal 456. Select the most appropriate antonym of the given word. OMINOUS (a) foreboding (b) threatening (c) auspicious (d) baleful SSC CHSL (Tier-I) –08/07/2019 (Shift-II) Ans. (c): Ominous - (Jinx/Unblessed), Antonymauspicious (Favorable/Lucky) Meaning of other words – Foreboding- Fore knowledge Threatening- Terrorizing Baleful- Harmful/Detrimental 457. Select the most appropriate antonym of the given word. Opaque (a) Dull (b) Clear (c) Frosty (d) Hazy SSC CHSL (Tier-I) –03/07/2019 (Shift-I) Ans. (b) : Opaque - (Cloudy/Non-Transparent), Antonym - Clear (Apparent/Clean) Meaning of other words – Dull- Inactive Frosty- Cold/Fridge 453. Select the most appropriate antonym of the Hazy- Cloudy/Dim given word. 458. Select the most appropriate ANTONYM of the Obstinate given word. (a) Willful (b) Firm OPTIONAL (c) Flexible (d) Loyal (a) Elective (b) Fanciful SSC Stenographer (Grade C & D) 15.11.2021 Shift-II (c) Original (d) Compulsory SSC CHSL (Tier-I) 16/10/2020 (Shift-I) Ans. (c) : Obstinate (Stubborn), Antonym - Flexible Ans. (d): Optional- (Alternative), Antonym (Resilient/Elastic) Compulsory (Mandatory/Necessary) Meaning of the other wordsMeaning of other words – Willful → Deliberately Elective - Selective Firm → Determined Fanciful – Imaginary/Dreamy Loyal → Faithful Original – Authentic/Genuine Antonyms

489

YCT

459. Select the most appropriate ANTONYM of the given word. OBSCURE (a) Uncertain (b) Ambiguous (c) Vague (d) Clear SSC CHSL (Tier-I) 18/03/2020 (Shift-I) Ans. (d): Obscure- (Vague/Unclear), Antonym Clear (Apparent/Clean) Meaning of other words – Uncertain – Wavering/Doubtful Ambiguous – Vague/Unreadable Vague – Obscure/Indistinct 460. Select the most appropriate ANTONYM of the given word. OUTSPOKEN (a) Ambivalent (b) Candid (c) Equivocal (d) Secretive SSC CHSL (Tier-I) 26/10/2020 (Shift-II) Ans. (d): Outspoken - (Vociferous/Straightforward), Antonym - Secretive (Bashful/shy) Meaning of other words – Ambivalent – Equivocal Candid – Forthright/Frank Equivocal – Ambiguous/Suspicious 461. In the following question, out of the given four alternatives select the one which is opposite in meaning of the given word. Obligate (a) Force (b) Indebt (c) Restrict (d) Let off SSC MTS 10-10-2017 (Shift-III) Ans. (d): 'Obligate'- (Coerce/Constrain), Antonym'Let off' (Pardon/Forgive) Meaning of other words – Force - Emphasis Indebt – Obliging Restrict - Ban 462. Select the most appropriate antonym of the word. Obstinate (a) Stern (b) Stubborn (c) Flexible (d) Adamant SSC MTS – 20/08/2019 (Shift-III) Ans. (c): Obstinate- (Stubborn/Headstrong), Antonym - Flexible (Resilient/Elastic) Meaning of other words – Stern - Harsh/Rigid Stubborn - Importune Adamant - Obdurate 463. Select the antonym of the given word. OPAQUE (a) Turbid (b) Transparent (c) Fogged (d) Murky SSC GD – 14/02/2019 (Shift-I) Ans. (b): Opaque - (Hazy/Obscure), Antonym transparent (Clear/Apparent) Meaning of other words – Turbid – Confused Fogged - Darken Murky - Darksome Antonyms

P 464. Select the appropriate antonym for the underlined word. The peach-tree is most productive when the roots are kept near the surface. (a) potent (b) lush (c) futile (d) fertile SSC MTS 18/05/2023 (Shift-I) Ans. (c) : The appropriate antonym for the underlined word – ‘productive’ is ‘futile’. Meaning of other options – Potent – Blatant, Lush – Succulent, Fertile – Prolific, 465. Select the appropriate antonym for the underlined word. He says we are poor now. (a) baren (b) indigent (c) ruined (d) opulent SSC MTS 16/05/2023 (Shift-II) Ans. (d) : The appropriate antonym for the underlined word 'poor' is 'Opulent'. Meaning of other optionsBarren- Infertile Indigent- Cashless Ruined- Destroyed 466. Select the most appropriate ANTONYM of the given word. Ponderous (a) Obligated (b) Graceful (c) Swollen (d) Adverse SSC GD 23/01/2023 (Shift-IV) Ans. (b) : Ponderous (Dull, weighty) AntonymGraceful (Beautiful, happy) Meaning of the other optionsObligated Constrained Swollen Bloated Adverse Opposite, Contrary 467. Select the most appropriate ANTONYM of the given word. Put forth (a) Stop (b) Against (c) Apply (d) Conceal SSC GD 16/01/2023 (Shift-III) Ans. (d) : Put forth (Apply), Antonym - Conceal (Hide) Meaning of the other optionsStop Pause, End Against Contrary Apply Enforce 468. Select the most appropriate ANTONYM of the given word. Presumale (a) Reasonable (b) Credible (c) Improbable (d) Forgetful SSC GD 06/02/2023 (Shift-III) Ans. (c) : The most appropriate Antonym of the given word 'Presumale' is – improbable. It is clear that 'improbable' is the most suitable antonym of presumale'

490

YCT

The other options give different meaning. Reasonable :- rational, logical. Credible :- believable Forgetful :- unmindful Hence option (c) is correct answer. 469. Select the most appropriate ANTONYM of the given word. Perplex (a) Complex (b) Simplify (c) Comprehensive (d) Classify SSC GD 01/02/2023 (Shift-II) Ans. (b) : 'Simplify' is the most appropriate antonym of the given word 'Perplex' (complicate). The meaning of the other words is– Complex :- difficult, hard. Comprehensive :- including all or everything. (complete) Classify :- arrange or order by classes or categories. 470. Select the most appropriate ANTONYM of the given word Partial (a) Unbiased (b) Unjust (c) Prejudiced (d) Doubtful SSC CGL (Tier-I) 19/07/2023 (Shift-IV) Ans. (a) : The most appropriate 'antonym' of the given word 'partial' (Fragmental) is 'unbiased.' The meaning of other words areUnjust means - unfair Prejudiced means - partisan Doubtfull means - suspect Thus, the correct answer is option (a). 471. Select the most appropriate ANTONYM of the underlined word. He regards himself as a patriod. (a) dutiful (b) traitor (c) nationalist (d) loyal SSC CGL (Tier-I) 17/07/2023 (Shift-II) Ans. (b) : The antonym of the underlined word "patriot" is 'traitor' while the remaining words have different meaning. 472. Select the most appropriate ANTONYM of the underlined word in the given sentence. Everyone feels that he us pompous, but I know the underlying truth that he is a man of great understanding and love. (a) Lofty (b) Haughty (c) Grandiose (d) Modest SSC CHSL (Tier-I) 17/08/2023 (Shift-II) Ans. (d) : Among the given option, option (d) modest – ( Tending not to talk about or make obuious your own abilities and achievements) is the most appropriate antonym of the underlined word, Pompus – ( seeling or showing that you think you are better or more important than other people.) in the sentence. Meaning of other options are– Lofty – Elevated in tharacter an spirit of high moral standerd . Haughty – and looking down on others Acting with superiority Grandiose – Bigger and containing more details than necessary. Antonyms

473. Select the most appropriate ANTONYM of the underlined word in the following sentence. Suraj has always shown a willingness to accept and respect the differences in others' lives with impartiality and avoid prejudice. (a) Avoid (b) Differences (c) Willingness (d) Impartiality SSC Selection Posts XI-28/06/2023 (Shift-III) Ans. (d) : In the above sentence, antonym of the underlined word "Prejudice" (Favoritisim) is "impartiality" (Justice). Meaning of other wordAvoid - Avert Differences - Disparity Willingness - Intentness 474. Select the most appropriate ANTONYM of the given word. Perilous (a) Verbose (b) Unstable (c) cruel (d) Safe SSC MTS-05/07/2022 Shift-I Ans. (d) : Perilous (Hazardous), Antonym - safe (Secure) Meaning of the other OptionsVerbose – Talkative Unstable – Fickle/Changeable Cruel – Brutal 475. Select the most appropriate ANTONYM of the underlined word in the given sentence. The kids have been pestering me to buy them new toys. (a) pacifying (b) perturbing (c) bugging (d) tormenting SSC MTS-05/07/2022 Shift-III Ans. (a) : Pestering Annoy/Bother), Antonym Pacifying (Soothe) Meaning of the other OptionsPerturbing – Puzzle/Harass Bugging – Irritating Tormenting – Afflict/Torture 476. Select the most appropriate ANTONYM of the given word. Prey (a) Loot (b) Sufferer (c) Game (d) Predator SSC CGL (Tier-I) 12/04/2022 Shift-II Ans. (d) : Prey (Hunt), Antonym - Predator - (Vermin) Meaning of the other OptionsLoot - Rob/Plunder Sufferer - Sick person/woeful Game - Sport 477. Select the most appropriate antonym of the highlighted word. Two original manuscripts of this text are still extant. (a) negotiate (b) extinct (c) drool (d) persist SSC CHSL 27.05.2022 Shift-III

491

YCT

Ans. (b) : Extant (Present/In vogue), Antonym - 481. Select the most appropriate antonym of the given word. Extinct (lost/Decedent) Persist Meaning of the other words(a) Discontinue (b) Convince Negotiate – Talk/Settle (c) Enable (d) Incline Drool – Saliva SSC Constable GD-07/12/2021 Shift-II Persist – Persevere Ans. (a) : Persist (Persevere), Antonym – Discontinue 478. Select the most appropriate antonym of the (Interrupt) highlighted word. Meaning of the other OptionsThe boss persists with his loyalists even now. Convince – Persuade Enable – Empower, Permit (a) discontinues (b) continues Incline – Tilt (c) formulates (d) retinues SSC CHSL 27.05.2022 Shift-II 482. Select the most appropriate ANTONYM of the given word. Ans. (a) : persists (perseveres) Antonym - discontinues Possess (Cease/Stop) (a) Release (b) Occupy Meaning of the other words(c) Obtain (d) Maintain Continues - Persevere/persist SSC Constable GD-14/12/2021 Shift-II Formulates - Draw up/map out Ans. (a) : Possess (Own) Antonym, Release - (Liberation, discharge) Retinues - Escort/Entourage Meaning of the other Options479. Select the most appropriate antonym of the Occupy - Divest, Take given word. Obtain - Get, Acquire Pseudo Maintain - Preserve (a) Genuine (b) Artificial 483. Select the most appropriate ANTONYM of the given word. (c) Flawed (d) Corrupt Prudent SSC CHSL 27.05.2022 Shift-II (a) Wasteful (b) Cautious Ans. (a) : Pseudo (Fake/Mock), Antonym - Genuine (c) Clever (d) Shrewd (Real/Actual) SSC Constable GD-16/11/2021 Shift-III Meaning of the other wordsAns. (a) : Prudent - (Discerning) Artificial - Synthetic/False Antonym - Wasteful -(Prodigal) Meaning of the other OptionsFlawed - Faulty/Incorrect Cautious - Alert, Careful Corrupt - Dishonest/Depraved Clever - Cunning, Crafty 480. Select the most appropriate antonym of the Shrewd - Canny highlighted word. 484. Select the most appropriate antonym of the She has a penchant for art and craft. given word. (a) anxiety (b) hatred PREVAIL (c) fear (d) joy (a) prove (b) succeed (c) surrender (d) control SSC CHSL 27.05.2022 Shift-I SSC Stenographer (Grade C & D) 11.11.2021 Shift-II Ans. (b) : Penchant (Passion/fondness/Inclination) Ans. (c) : Prevail (Win/Conquer/popular) Antonym / Antonym - Hatred (Dislike/aversion/Disgust) Surrender (Capitulate/give up) Meaning of the other wordsMeaning of the other wordsAnxiety - Concern/Perplexity Prove - Vindicate/Verify Fear - Phobia/Fright Succeed - Triumph/win Control - Command/Curb Joy - Glad/Glee 481. Select the most appropriate antonym of the Option (c) is correct given word. 485. Select the most appropriate antonym of the given word. Vulgar PRODUCE (verb) (a) Refined (b) Senior (a) harvest (b) yield (c) Ignorant (d) Decreased (c) destroy (d) arrange SSC CHSL 30.05.2022 Shift-I SSC Stenographer (Grade C & D) 11.11.2021 Shift-I Ans. (a) : Vulgar (Indecent/unrefined) Antonym, Ans. (c) : Produce (Make/Build), Antonym destroy 'Refined' (Civilized/cultured) (Decimate/Demolish) Meaning of the other wordsMeaning of the other wordsSenior - Elder/ Superior harvest - Crop Ignorant - Unwise/Unaware yield - Produce arrange - Order Decreased - Reduce/Lessen Antonyms

492

YCT

486. Select the most appropriate ANTONYM of the given word. PUNGENT (a) Sharp (b) Sour (c) Mild (d) Biting SSC CHSL (Tier-I) –16/04/2021 (Shift-I) Ans. (c): Pungent - (Acrimonious/Acute), Antonym, Mild - (Gentle/Soft) Meaning of other words – Sharp - Intense Sour – Tart/Acid Biting - Cutting 487. Select the most appropriate ANTONYM of the given word. PRESTIGE (a) renown (b) disregard (c) dignity (d) repute SSC CPO-SI – 23/11/2020 (Shift-II) Ans. (b): Prestige - (Dignity/Glory), AntonymDisregard (Dishonor) Meaning of other words – Renown - (Glory/Fame), Dignity - (Decoroum/Deference) Repute - (Popularity/Honor) 488. Select the antonym of the given word. PARDON (a) mercy (b) punish (c) grace (d) kindness SSC CGL (Tier-I) – 04/06/2019 (Shift-III) Ans. (b): Pardon- (Excuse), Antonym - Punish(Torment/Chastise) Meaning of other words – Mercy- Pity/Ruth Grace- Elegance/Compassion Kindness- Gentleness/Benevolence 489. Select the most appropriate antonym of the given word. PRUDENT (a) indiscreet (b) judicious (c) tactful (d) practical SSC CGL (Tier-I) – 10/06/2019 (Shift-III) Ans. (a): Prudent - (Discerning/Rational), AntonymsIndiscreet (Impetuous/Indiscriminate) Meaning of other words – Judicious- Sensible/Reasonable Tactful- Courteous/Docile Practical- Experimental/Usual 490. Select the most appropriate antonym of the given word. PUBLIC (a) private (b) ready (c) common (d) restricted SSC CGL (Tier-I) – 13/06/2019 (Shift-II) Ans. (a): Public - (Common/Demos), Antonym Private (Personal/Individual) Meaning of other words – Ready- Prepared Common- Usual Rusticated- Banished Antonyms

491. Select the most appropriate antonym of the given word. PROGRESSIVE (a) moving (b) conservative (c) repeated (d) aristocratic SSC CGL (Tier-I) – 13/06/2019 (Shift-II) Ans. (b): Progressive - (Developing), Antonym Conservative- (Orthodox) Meaning of other words – Moving- Transportable Repeated- Recurrent Aristocratic- Oligarchy 492. Select the most appropriate antonym of the given word. PROLONG (a) prevent (b) allow (c) increase (d) shorten SSC CGL (Tier-I) – 12/06/2019 (Shift-III) Ans. (d): Prolong- (Lengthen/Elongate) Antonym 'Shorten'- (Reduce/Slacken) Meaning of other words – Prevent- Stop Allow- Permit/Assent Increase- Growth/Augment 493. Select the most appropriate antonym of the given word. PERTURB (a) Fluster (b) Soothe (c) Confound (d) Commend SSC CPO-SI – 11/12/2019 (Shift-II) Ans. (b): Perturb - (Discompose/Agitation), Antonym Soothe - (Calm/Quiet) Meaning of other words – Fluster- Quandary/Harass Confound- Perplex Commend – Admire/Applaud 494. Select the most appropriate antonym of the given word. POROUS (a) spongy (b) permeable (c) impermeable (d) absorbent SSC CHSL (Tier-I) –10/07/2019 (Shift-III) Ans. (c): Porous- (Foraminate/Perforated) AntonymImpermeable- (Impenetrable/Unassailable) Meaning of other words – Spongy- Taut Permeable- Penetrable/Accessible Absorbent- Imbibe/Soak 495. Select the most appropriate antonym of the given word. PROCEED (a) ahead (b) recede (c) forward (d) continue SSC CHSL (Tier-I) –09/07/2019 (Shift-II) Ans. (b): Proceed- (Go forward/Go ahead) AntonymRecede- (Retreat/Recoil) Meaning of other words – Ahead- Further Forward- Onward Continue- Sustain

493

YCT

496. Select the most appropriate antonym of the given word. PROSPERITY (a) nobility (b) affinity (c) simplicity (d) adversity SSC CHSL (Tier-I) –05/07/2019 (Shift-I) Ans. (d): Prosperity(Affluence/Flourishing), Antonym- Adversity (Poverty/Lack) Meaning of other words – Nobility- Greatness/Generosity Affinity- Congeniality Simplicity- Naivety 497. Select the most appropriate antonym of the given word. PERPETRATE (a) execute (b) prevent (c) perform (d) commit SSC CHSL (Tier-I) –01/07/2019 (Shift-III) Ans. (b): Perpetrate - (Sin/Crime), Antonym - Prevent (Stop/Clog) Meaning of other words – Execute- Gibbet Perform- Act/Impersonate Commit- Pledge/Promise 498. Select the most appropriate ANTONYM of the given word. PROLIFIC (a) Abundant (b) Unproductive (c) Several (d) Rich SSC CHSL (Tier-I) 21/10/2020 (Shift-II) Ans. (b): Prolific- (Fertile/Productive), Antonym Unproductive (Infertile/Barren) Meaning of other words – Abundant – Bountiful/Ample Several – Various/Diverse Rich – Affluent/Wealthy 499. Select the most appropriate ANTONYM of the given word. PACIFIST (a) Orator (b) Peace-maker (c) Warmonger (d) Nihilist SSC CHSL (Tier-I) 21/10/2020 (Shift-II) Ans. (c): Pacifist- (Peacemaker), Antonym Warmonger (Belligerent) Meaning of other words – Orator – Speaker/Spokes man Peace-maker – Pacificator Nihilist – Emptiness/Vividness 500. Select the most appropriate ANTONYM of the given word. PLAUSIBLE (a) Unlikely (b) Feasible (c) Unmanageable (d) Unlikely SSC CHSL (Tier-I) 19/03/2020 (Shift-I) Ans. (a): Plausible - (Probable/Colorable), Antonym unlikely (Unexpected/Improbable) Meaning of other words – Feasible – Practicable/Doable Unmanageable – Uncontrollable/Unbearable Antonyms

501. Select the most appropriate ANTONYM of the given word. PLACATE (a) Appease (b) Conciliate (c) Propitiate (d) Enrage SSC CHSL (Tier-I) 18/03/2020 (Shift-III) Ans. (d): 'Placate'- (Console/Solace), Antonym Enrage (Infuriate/Anger) Meaning of other words – Appease – Soothe/Calm Conciliate - Compromise Propitiate – satisfy/Pamper 502. Select the most appropriate ANTONYM of the given word. PERISH (a) Vanish (b) Wither (c) Grow (d) Crumble SSC CHSL (Tier-I) 19/10/2020 (Shift-II) Ans. (c): Perish– (Ruin), Antonym - Grow (Surmount/cultivate) Meaning of other words – Vanish– Disappear/Evanish Wither– Fade/Shrivel Crumble– Pulverize/Shred 503. Select the most appropriate ANTONYM of the given word. PROVOKE (a) Stimulate (b) Encourage (c) Appease (d) Beckon SSC CHSL (Tier-I) 14/10/2020 (Shift-II) Ans. (c): Provoke– (Aggravate/Inflame), Antonym Appease– (Quiet/Soothe) Meaning of other words – Stimulate– Excite/Agitate Encourage– Promote/Embolden Beckon– Gesture/Insinuate 504. Select the most appropriate ANTONYM of the given word. PRIM (a) Mature (b) Principle (c) Disheveled (d) Intellectual SSC CHSL (Tier-I) 17/03/2020 (Shift-III) Ans. (c): 'Prim'- (In order), Antonym - Disheveled (Disorderly) Meaning of other words – Mature – Sensible/Practical Principle – Doctrine/Maxim Intellectual – Cognitive/Scholarly 505. In the following question, out of the given four alternatives, select the one which is opposite in meaning of the given word. Persuade (a) Advise (b) Affect (c) Assure (d) Prevent SSC MTS 9-10-2017 (Shift-II) Ans. (d): Persuade- (Expostulate), Antonym Prevent (Stop/Clog) Meaning of other words – Advise – Suggest Affect – Impress/Influence Assure – Ensure/Convince

494

YCT

506. Select the antonym of the given word. Persuade (a) Convince (b) Endorse (c) Dissuade (d) Fume SSC MTS – 08/08/2019 (Shift-III) Ans. (c): Persuade- (Expostulate), Antonym 'Dissuade' (Deter) Meaning of other words – Convince - Persuade Endorse - Support Fume - Smoke 507. Select the most appropriate antonym of the given word. Progress (a) Advancement (b) Growth (c) Betterment (d) Decline SSC MTS – 08/08/2019 (Shift-I) Ans. (d): Progress(Development/Advance), Antonym - Decline (Downfall/Wane) Meaning of other words – Advancement - Enhancement/Rise Growth - Progress/Increase Betterment - Improvement/Reformation 508. Select the most appropriate antonym of the word. Pompous (a) Arrogant (b) Calm (c) Impatient (d) Humble SSC MTS – 21/08/2019 (Shift-III) Ans. (d): Pompous - (Ostentatious), Antonym Humble (Submissive) Meaning of other words – Arrogant – Conceited/Haughty Calm – Quit/Silent Impatient – Restless/Uneasy

511. Select the most appropriate antonym of the given word. Quench (a) Extinguish (b) Ignite (c) Soothe (d) Quiet SSC MTS – 06/08/2019 (Shift-III) Ans. (b): Quench – (Extinguish), Antonym, ignite – (Kindle/Arson) Meaning of other words – Extinguish – Blow out/Quench Soothe – Calm/Quiet Quiet – Tranquil/Silent 512. Select the most appropriate antonym of the given word. Query (a) Nibble (b) Doubt (c) Answer (d) Oust SSC CGL (Tier-I) – 07/06/2019 (Shift-II) Ans. (c): Query – (Question), Antonym–Answer – (Reply) Meaning of other words – Nibble - Suspect/Doubt Doubt – Vacillate Oust – Extrude/Evict 513. Select the most appropriate antonym of the given word. Quotidian (a) Ordinary (b) Reckless (c) Extraordinary (d) Pious SSC CHSL (Tier-I) 19/10/2020 (Shift-III) Ans. (c): Quotidian – (Ordinary/Normal), Antonym, Extraordinary – (Exceptional/Special) Meaning of other words – Ordinary – Normal/Hackneyed Reckless – Careless/Irresponsible 509. Select the most appropriate antonym of the Pious – Holy/Chaste word. Quell (a) Agitate (b) Deny (c) Reduce (d) Entire 514. Select the most appropriate ANTONYM of the SSC MTS – 13/08/2019 (Shift-I) given word. SSC CPO–SI–12/12/2019 (Shift-II) Retain Ans. (a): Quell (Calm/Sooth), Antonym - Agitate (a) Maintain (b) Forget (Excite) (c) Restore (d) Attain Meaning of other words – SSC GD 12/01/2023 (Shift-II) Deny - Refuse/Reject Reduce - Decrease/Slacken Ans. (b) : Retain (Hold, keep), Antonym - Forget Entire - Whole/Total (Bewilder) 510. Select the most appropriate antonym of the Meaning of the other optionsgiven word. Maintain - Preserve, Sustain Quarrel (a) Strife (b) Feud Restore - Renew, riestablish (c) Harmony (d) Bias Attain - Achieve, obtain SSC MTS – 07/08/2019 (Shift-II) 515. Select the most appropriate ANTONYM of the Ans. (c): Quarrel- (Dispute/Fight), Antonym given word. Harmony (Unison/Consistence) Rancid Meaning of other words – (a) Rotten (b) Urgent Strife – Conflict/Discord (c) Fresh (d) Rough Feud – Animosity/Enmity Bias – discrimination/Partiality/Prejudice SSC GD 10/01/2023 (Shift-I)

Q

R

Antonyms

495

YCT

Ans. (c) : Rancid (stale), Antonym - Fresh (New, Original) Meaning of the other optionsRotten - Putrid Urgent - Essential Rough - Rigid, coarse, unrefine 516. Select the most appropriate ANTONYM of the given word. Rustic (a) Depressed (b) Natural (c) Shallow (d) Urbane SSC GD 08/02/2023 (Shift-IV) Ans. (d) : 'Urbane' is the most appropriate antonym of the given word Rustic. The other options give different meaning. Depressed :- gloomy Natural :- Normal Shallow :- lacking physical depth. 517. Select the most appropriate ANTONYM of the given word. Repeal (a) Separation (b) Expense (c) Approval (d) Adversity SSC CHSL (Tier-I) 17/08/2023 (Shift-II) Ans. (c) : "Approval" ( feeling or shoring that you think something is good/Getting agree) will be the antonym of given word Repeal – (To officially make a low no long valid) Other options express following meaning – Separation – The state of things separated. Expense – money that is spent for a particular purpose. Adversity – An unpleasant and difficult situation. 518. Select the most appropriate antonym for 'refuse' in the given sentence. Shama was 'refuse' to visit the accommodation places for the devotees of the shrine. (a) Allowed (b) Shrine (c) Visit (d) Accommodation SSC CHSL (Tier-I) 14/08/2023 (Shift-IV) Ans. (a) : The antonym of word 'refuse' – ( Indicate that one is not willing to do/accept/approve something is option (a) allowed – (To let someone do something or to let something happen or granting Permission.) Meaning possessed by other words Shrine – A place for worship that is holy because of having connection with sacred things or object/person. Accommodation – A place for somebody to live or stay Visit – To go to see a person or place for a period of time. 519. Select the most appropriate antonym of the given word. Raciness (a) Obscenity (b) Lewdness (c) Filth (d) Perfection SSC CHSL (Tier-I) 14/08/2023 (Shift-IV) Antonyms

Ans. (d) : The most appropriate antonym of the given word 'Raciness' – (Behavior or language bordering on indelicacy) is option (d) Perfection (The state of being perfect and without fault). Meaning possessed by other options– obscenity – Sexual words or acts that shock people and cause offence. Lewdness – Behavior that is sexual in offensive way. Filth – unpleasant dirt or sexual words that cause offence. 520. Choose the word that is opposite in meaning to the given word. Recondite (a) Abstruse (b) Phlegm (c) Clear (d) Abstrct SSC CHSL (Tier-I) 21/03/2023 (Shift-II) Ans. (c) : Recondite - Antonym - Clear Meaning of other wordsPhlegm – Mucilage Abstract – Essence Abstruse – Intensive 521. Select the most appropriate ANTONYM of the given word. Retreat (a) Advance (b) Push (c) Promote (d) Strengthen SSC Constable GD-09/12/2021 Shift-II Ans. (a) : Retreat – (Recoil), Antonym – advance – (Progress, Proceed) Meaning of the other OptionsPush – Force Promote – Encourage Strengthen – Invigorate, Fortify 522. Select the most appropriate ANTONYM of the given word. Reign (a) Manage (b) Yield (c) Influence (d) Govern SSC Constable GD-18/11/2021 Shift-I Ans. (b) : Reign (Rule), Antonym - Yield (Agree/Submit) Meaning of the other OptionsManage – Handle/Supervise Influence– Impact/Effect Govern – Reign/Suppress 523. Select the most appropriate ANTONYM of the given word. REMEMBER (a) Learn (b) Forget (c) Memorise (d) Retain SSC Constable GD-22/11/2021 Shift-II Ans. (b) : Remember (Recall), Antonym Forget (Bewilder) Meaning of the other OptionsLearn – Train/Master Memories – Recall Retain – keep/Preserve

496

YCT

524. Select the most appropriate ANTONYM of the given word. Reticent (a) Talkative (b) Hostile (c) Aggressive (d) Controversial SSC Constable GD-29/11/2021 Shift-II Ans. (a) : Reticent (Taciturn), Antonym - Talkative (Garrulous) Meaning of the other OptionsHostile - Rancorous Aggressive - Assailant Controversial - Disputable 525. Select the most appropriate ANTONYM of the given word. Reverence (a) Contempt (b) Respect (c) Awe (d) Admiration SSC Constable GD-24/11/2021 Shift-II Ans. (a) : Reverence (Respect), Antonym - Contempt (Humiliation) Meaning of the other OptionsRespect - Deference Awe - Fear Admiration - Praise 526. Select the most appropriate ANTONYM of the given word. Rely (a) Await (b) Move (c) Distrust (d) Depend SSC CGL (Tier-I) 12/04/2022 Shift-I Ans. (c) : Rely (Believe/Depend), Antonym - Distrust (Doubt) Meaning of the other OptionsAwait - Expect/Anticipate Move - Proceed/Stir Depend - Rely 527. Select the most appropriate ANTONYM of the given word. Raze (a) Comfort (b) Ruin (c) Ease (d) Build SSC CGL (Tier-I) 11/04/2022 Shift-I Ans. (d) : Raze (Destroy), antonym – Build (Creation) Meaning of the other OptionsComfort – Ease Ruin – Destruction Ease – Relief 528. Select the most appropriate ANTONYM of the given word. Refute (a) Endorse (b) Acknowledge (c) Deny (d) Dispute SSC MTS-06/07/2022 Shift-I Ans. (a) : Refute (Contradict), Antonym, Endorse (Support) Meaning of the other OptionsAcknowledge – Admit Deny – Reject Dispute – Controversy Antonyms

529. Select the most appropriate ANTONYM of the given word. ROUGH (a) Smooth (b) Coarse (c) Thick (d) Crude SSC CGL (Tier-I) 11/04/2022 Shift-II Ans. (a) : Rough (Coarse), Antonym – Smooth – (Sleek) Meaning of the other OptionsCoarse – Rugged/ Rough Thick – Plump Crude – Unrefined 530. Select the most appropriate ANTONYM of the given word. Ransack (a) Seize (b) Protect (c) Pilfer (d) Raid SSC CGL (Tier-I) 13/04/2022 Shift-III Ans. (b) : Ransack (Rob/Loot), Antonym - Protect (Secure/Preserve) Meaning of the other OptionsSeize - Confiscate Pilfer - Steal/Embezzle Raid - Assault/charge 531. Select the most appropriate ANTONYM of the given word. Rectify (a) Correct (b) Corrupt (c) Select (d) Assist SSC CGL (Tier-I) 13/04/2022 Shift-II Ans. (b) : Rectify (Amend), Antonym - corrupt (Deprave) Meaning of the other OptionsCorrect - Right Select - Choose Assist - Help 532. Select the most appropriate antonym of the given word. Reinforce (a) Validate (b) Diverge (c) Determine (d) Weaken SSC CHSL 31.05.2022 Shift-III Ans. (d) : Reinforce - (Strengthen/Fortify), Antonym Weaken - (Debilitate) Meaning of the other wordsValidate - Confirm/Ratify Diverge - Deviate/Separate Determine - Decide 533. Select the most appropriate antonym of the given word. REVIVE (a) damage (b) freshen (c) continue (d) restore SSC Stenographer (Grade C & D) 12.11.2021 Shift-I Ans. (a) : Revive (Resurrect/Reanimate). Antonym Damage (Destruction/Demolition) Meaning of the other wordsRestore - Renovate/Repair Freshen - Refreshment/ Recency Continue - Carry on

497

YCT

534. Select the most appropriate ANTONYM of the given word. Inordinate (a) Irrational (b) Disproportionate (c) Reasonable (d) Exorbitant SSC CHSL (Tier-I) –16/04/2021 (Shift-I) Ans. (c): Inordinate - (Unusual/Abnormal), AntonymReasonable - (Rational/Logical) Meaning of other words – Exorbitant-Excessive/Exceeding Irrational- Unreasonable/Illogical Disproportionate – Irrelevant/Inconsequential 535. Select the most appropriate antonym of the given word. Resign (a) Sign (b) Join (c) Accept (d) Reject SSC CHSL (Tier-I) –19/04/2021 (Shift-I) Ans. (b): Resign - (Quit/Abdicate), Antonym- Join (Unite/Add) Meaning of other words – Sign-Hint/Clue Accept-Admit/Confess Reject – Denial/Decline 536. Select the most appropriate antonym of the given word. Revel (a) Mourn (b) Rejoice (c) Enjoy (d) Appreciate SSC CGL (Tier-I) –18/08/2021 (Shift-I) Ans. (a): Revel - (Delight/Enjoy), Antonym, Mourn (Lament/Deplore) Meaning of other words – Rejoice – Pleasure/Joy Enjoy – Relish/Delight Appreciate – Applaud/Admire 537. Select the most appropriate ANTONYM of the given word. REVIVE (a) arouse (b) renew (c) console (d) ruin SSC CPO-SI – 24/11/2020 (Shift-I) Ans. (d): 'Revive'- (Revitalize), Antonym - 'Ruin' (Destroy)~ Meaning of other words – Arouse – Agitate/Wake Renew – Regenerate Console – Solace/placate 538. Select the antonym of the given word. REPRIMAND (a) apprehend (b) reproach (c) compliment (d) blame SSC CGL (Tier-I) – 06/06/2019 (Shift-II) Ans. (c): 'Reprimand' - (Chide/Rebuke), AntonymCompliment (Admiration) Meaning of other words – Reproach - Admonish/Damn Apprehend - Understand/Comprehend Blame - Fault/Flaw Antonyms

539. Select the most appropriate antonym of the given word. REPUGNANT (a) Austere (b) Sombre (c) Pleasant (d) Odious SSC CPO-SI – 11/12/2019 (Shift-II) Ans. (c): Repugnant- (Hateful/Disgusting), Antonym Pleasant (Delightful) Meaning of other words – Austere – Adamant/Straightforward Somber – Desperate/Despondent Odious – Abominable/Disgusting 540. Select the most appropriate ANTONYM of the given word. RECKLESS (a) heedless (b) daring (c) brash (d) cautious SSC CPO-SI – 12/12/2019 (Shift-I) Ans. (d): Reckless - (Careless/Irresponsible), Antonym - Cautious - (Careful/Aware) Meaning of other words – Heedless - Inattentive Daring – Courageous/Adventurous Brash - Fragile/Tender 541. Select the most appropriate antonym of the given word. REPRIMAND (a) Rebuke (b) Reward (c) Reproof (d) Redeem SSC CPO-SI – 13/12/2019 (Shift-I) Ans. (b): Reprimand - (Chide/Rebuke), Antonym Reward (Regalia/Prize) Meaning of other words – Rebuke – Scold/Damn Reproof – Condemn/Censure Redeem – Free/Save 542. Select the most appropriate antonym of the given word. RANDOM (a) formal (b) specific (c) irregular (d) casual SSC CHSL (Tier-I) –10/07/2019 (Shift-II) Ans. (b): Random- (Erratic) Antonym- Specific (Special) Meaning of other words – Formal- Casual Irregular- Desultory Casual- Accidental 543. Select the most appropriate antonym of the given word. RETALIATION (a) reconciliation (b) retribution (c) retrospection (d) regression SSC CHSL (Tier-I) –08/07/2019 (Shift-III) Ans. (a): Retaliation- (Revenge), AntonymReconciliation- (Compromise/Reunion) Meaning of other words – Retribution- Retaliation/Revenge Retrospection- Introspection/Self-reflective Regression- Comeback/Return

498

YCT

544. Select the most appropriate antonym of the given word. RESERVED (a) placid (b) communicative (c) quiet (d) modest SSC CHSL (Tier-I) –03/07/2019 (Shift-III) Ans. (b): Reserved- (Reticent/Taciturn), Antonym Communicative (Sociable/Accommodating) Meaning of other words – Placid- Gentle/Amiable Quiet- Calm/Tranquil Modest- Humble/Demure 545. Select the most appropriate antonym of the given word. RUEFUL (a) Regretful (b) Joyful (c) Spicy (d) Fragrant SSC CHSL (Tier-I) 17/03/2020 (Shift-II) Ans. (b): Rueful – (Gloomy/Sullen), Antonym Joyful (Gleeful/Glad) Meaning of other words – Regretful – Remorseful/Contrite Spicy – Pickled/Marinated Fragrant – Aromatic/Redolent 546. Select the most appropriate ANTONYM of the given word. RESERVED (a) Forgiving (b) Reticent (c) Cowardly (d) Friendly SSC CHSL (Tier-I) 12/10/2020 (Shift-II) Ans. (d): 'Reserved' - (Reticent/Taciturn), Antonym Friendly (Amicable/Coordinal) Meaning of other words – Forgiving – Merciful/Compassionate Reticent – Taciturn Cowardly – Timid/Chiken-hearted 547. Select the most appropriate ANTONYM of the given word. REVERED (a) Cherished (b) Valued (c) Esteemed (d) Despised SSC CHSL (Tier-I) 19/10/2020 (Shift-III) Ans. (d): Revered – (Esteem/Respect), Antonym Despised (Abhor/Dishonor) Meaning of other words – Cherished – Nurture Valued – Precious Esteemed– Respected 548. Select the most appropriate ANTONYM of the given word. RESPONSIBLE (a) irresponsible (b) illresponsible (c) disesponsible (d) unresponsible SSC CHSL (Tier-I) 14/10/2020 (Shift-I) Ans. (a): Responsible(Liable/Accountable), Antonym - 'Irresponsible' (Negligent/Careless) Meaning of other words is not suitableAntonyms

549. In the following question, out of the given four alternatives, select the one in which is opposite the given word. Remarkable (a) Famous (b) Odd (c) Race (d) Normal SSC MTS 9-10-2017 (Shift-III) Ans. (d): Remarkable- (Exceptional/Appreciable), Antonym - Normal (Common/Ordinary) Meaning of other words – Famous- Renowned/Eminent Odd- Unproportional/Peculiar Rare- Scarce/Exceptional 550. In the following question, out of the given four alternatives, select the one which is opposite in meaning of the given word. Repulsive (a) Creepy (b) Hateful (c) Ugly (d) Pleasing SSC MTS 11-10-2017 (Shift-III) Ans. (d): Repulsive- (Obnoxious/Distasteful), Antonym - Pleasing (Charming/Attractive) Meaning of other words – Creepy– Weird/Terrifying Hateful – Despiteful/Disgusting Ugly – Nasty/Misshapen 551. Select the most appropriate antonym of the given word. Relaxed (a) Peaceful (b) Calm (c) Arrogant (d) Tense SSC MTS – 06/08/2019 (Shift-III) Ans. (d): Relaxed - (ÌEase/Slacken), Antonym - Tense - (Stressful/Strained) Meaning of other words – Peaceful – Soothing/Noiseless Calm – Tranquil/Quiet Arrogant – Conceited/Haughty 552. Select the most appropriate antonym of the word : Reluctant (a) Hesitant (b) Uncertain (c) Unwilling (d) Eager SSC MTS – 22/08/2019 (Shift-III) Ans. (d): 'Reluctant'- (Unwilling/Loath), Antonym Eager (Avid/Impatient) Meaning of other words – Hesitant - Pendulous/Indecisive Uncertain - Unsteady/Doubtful Unwilling - Unwanted/Unwished 553. Select the most appropriate antonym of the word. Refined (a) Cruel (b) Crude (c) Fierce (d) Smooth SSC MTS – 21/08/2019 (Shift-III) Ans. (b): Refined - (Pure/Clear), Antonym - Crude (Unrefined/Coarse) Meaning of other words – Cruel – Brutal/Ferocious Fierce – Violent/Ruthless Smooth – Sleek/Glib

499

YCT

554. Select the most appropriate antonym of the given word. REWARD (a) Punish (b) Produce (c) Present (d) Praise SSC MTS – 19/08/2019 (Shift-III) Ans. (a): 'Reward'- (Prize/Regalia), Antonym 'Punish' (Chastise/Torment) Meaning of other words – Produce - Manufacture/Beget Present - Gift/Current Praise - Compliment/Laud 555. Select the antonym of the given word. REVEAL (a) Show (b) Praise (c) Hide (d) Disclose SSC GD – 08/03/2019 (Shift-II) Ans. (c): Reveal - (Disclose/Unfold), Antonym-Hide (Conceal/Secrete) Meaning of other words – Show - Display Praise – Compliment/Loud Disclose – Reveal/Unfold

S 556. Select the appropriate antonym for the underlined word. They must have been more than just suspicious of you. (a) Queer (b) Doubtful (c) Ambiguous (d) Definitive SSC MTS 04/05/2023 (Shift-II) Ans. (d) : The appropriate antonym for the underlined word 'suspicious' will be 'Definitive' (fixed). other words areQueer means- Awkward, clumsy Doubtful means- suspicious Ambiguous means- unclear, obscure. Thus, the correct answer is option (d). 557. Choose the word that is opposite in meaning to the given word. Surrender (a) Waive (b) Stay (c) Withstand (d) Calm SSC MTS 15/05/2023 (Shift-I) Ans. (c) : The opposite meaning for the above given word– 'Surrender' is Withstand'. Meaning of other options – Waive :- Abandon, Expurgate, Stay :- Remain, Calm :- Quiet, Serene, 558. Select the most appropriate ANTONYM for the underlined word in the following sentence. He is a spendthrift and spends money lavishly. (a) Careful (b) Wicked (c) Miser (d) Mean SSC GD 27/01/2023 (Shift-I) Antonyms

Ans. (c) : The most appropriate antonym for the underlined word 'spendthrift' (Prodigal) is 'Miser' (Niggaral). Meaning of the other optionsCareful - Cautious, Wary Wicked - Vicious Mean - Gready, Signify Average 559. Select the most appropriate ANTONYM of the given word. Scanty (a) Meagre (b) Buoyant (c) Abundant (d) Divine SSC GD 06/02/2023 (Shift-III) Ans. (c) : 'Abundant' (plentiful) is the most suitable antonym of 'scanty' (meagre). The meaning of other options is– Meagre :- deficient in amount or quality. Buoyant :- Cheerful Divine :- godly. 560. Select the most appropriate ANTONYM of the underlined word in the given sentence. Henry is so servile that other people take adventage of him. (a) Sheepish (b) Arrogant (c) Cunning (d) Bickering SSC CGL (Tier-I) 14/07/2023 (Shift-I) Ans. (b) : The most appropriate ntonym of above underlined word ‘servile is 'Arrogant'. The Remaining word and their antonym Sheepish – unabashed Cunning – Honest Bickering – peace. 561. Select the most appropriate ANTONYM of the given word. Serendipity (a) Tranquillity (b) Permanency (c) Tangibility (d) Misfortune SSC CHSL (Tier-I) 03/08/2023 (Shift-II) Ans. (d) : The most appropriate antonym of the above given word 'Serendipity' (luck) is 'Misfortune' (adversity). Meaning of other options – Tranguillity – Serenity, Calm, Permanency – Stability, Strength, Tangibility – Reality, Entity, 562. Select the most appropriate antonym of the given word. 'Sorcery' (a) Spell (b) Reality (c) Devilry (d) Dismiss SSC CHSL (Tier-I) 15/03/2023 (Shift-III) Ans. (b) : 'Sorcery' - Antonym - 'Reality' Meaning of other wordDevilry → Malice Spell → Magic Dismiss → Depart

500

YCT

563. Select the most appropriate ANTONYM of the given word. Salient (a) Chief (b) Remarkable (c) Insignificant (d) Eminent SSC MTS-05/07/2022 Shift-II Ans. (c) : Salient (Main), Antonym - Insignificant (Fruitless) Meaning of the other OptionsChief – Main/Major Remarkable – Marvelous/Wonderful Eminent – Famous/Well known 564. Select the most appropriate ANTONYM of the given word. Steady (a) Tabular (b) Shaky (c) Dedicate (d) Constant SSC MTS-07/07/2022 Shift-II Ans. (b) : Steady (Stable), Antonym - Shaky (Unsteady) Meaning of the other OptionsTabular – List/Chart Dedicate – Devotion/Commitment Constant – Stable/Perpetual 565. Select the most appropriate ANTONYM of the given word. Sagacious (a) Dull (b) Profound (c) keen (d) Intelligent SSC MTS-06/07/2022 Shift-II Ans. (a) : Sagacious (Brilliant), Antonym - Dull (Hebetate) Meaning of the other OptionsProfound – Intensive/Deep Keen – eager/Zealous Intelligent – wise/Prudent 566. Select the most appropriate ANTONYM of the underlined word in the following sentence. Eventually the patient succumbed to her illness. (a) surrendered (b) overcame (c) died (d) advanced SSC CHSL 25.05.2022 Shift-I Ans. (b) : Succumbed (Surrender/Pass away) Antonym - Overcame (Conquer/Takeover) Meaning of the other wordsSurrendered – Capitulate/Give up/Submit Died – Pass away/ Expire Advanced – Developed/ Improved 567. Select the most appropriate antonym of the given word. Sceptic (a) Receiver (b) Doubter (c) Disbeliever (d) Believer SSC CHSL 27.05.2022 Shift-III Antonyms

Ans. (d) : Skeptic (Suspicious/Distrust), Antonym 'Believer' (Theist/Deist) Meaning of the other wordsReceiver – Recipient/Beneficiary Doubter – Skeptic/Atheist Disbeliever – Questions/Sceptic 568. Select the most appropriate antonym of the given word. Savage (a) Civilized (b) Adequate (c) Impartial (d) Brutal SSC CHSL 26.05.2022 Shift-I Ans. (a) : Savage (Vandal/Barbarous), Antonym Civilized (Decent/Polite) Meaning of the other OptionsAdequate – Sufficient/Ample Impartial – Fair/ unbiased Brutal – Fierce/Ruthless 569. Select the most appropriate antonym of the highlighted word. They drank some spurious drink and fainted. (a) genuine (b) nefarious (c) curious (d) artificial SSC CHSL 26.05.2022 Shift-I Ans. (a) : 'Spurious' (Countrfeit), Antonym - Genuine (Real/Actual) Meaning of the other OptionsNefarious – Dishonest/Shifty Curious – Eager Artificial – Synthetic 570. Select the most appropriate ANTONYM of the given word. Severe (a) Narrow (b) Mild (c) Short (d) Sharp SSC Constable GD-13/12/2021 Shift-III Ans. (b) : Severe (Serious/Grievous), Antonym 'mild'(Gentle/Placid) Meaning of the other OptionsNarrow - Parochial/Poky Short - Small Sharp - Intense/Fast 571. Select the most appropriate ANTONYM of the given word. Stout (a) Obsess (b) Thin (c) Bulky (d) Heavy SSC Constable GD-30/11/2021 Shift-III Ans. (b) : Stout (fat/Plump), Antonym - Thin (Slender/Lean) Meaning of the other optionsObese - Fat Bulky - Heavy Heavy - Bulky/Cumbersome 572. Select the most appropriate antonym of the given word. SHALLOW (a) superficial (b) hollow (c) spiteful (d) deep SSC Stenographer (Grade C & D) 12.11.2021 Shift-II

501

YCT

Ans. (d) : Shallow(Cursory), antonym- deep (Profound) Meaning of the other wordsSuperficial - Surface/Cursory Hollow - Vacant/Unfilled Spiteful - Malicious/Despiteful 573. Select the most appropriate antonym of the given word. SHRINK (a) expand (b) resume (c) supply (d) apply SSC Stenographer (Grade C & D) 15.11.2021 Shift-II Ans. (a) : Shrink (Wrinkle/Reduce), Antonym - Expand (Enlarge) Meaning of the other wordsResume → Restart Supply → Provide/Contribute Apply → Appeal/Request 574. Select the most appropriate antonym of the given word. SUPERFICIAL (a) praiseworthy (b) genuine (c) trivial (d) shallow SSC Stenographer (Grade C & D) 12.11.2021 Shift-I Ans. (b) : Superficial (Shallow). Antonym Genuine (Real/Actual) Meaning of the other wordstrivial - Insignificant/ Commonplace shallow - perfunctory/Cursory praiseworthy - Commendable/ meritorious 575. Select the most appropriate ANTONYM of the given word. SUSTENANCE (a) refreshment (b) livelihood (c) starvation (d) nourishment SSC CHSL (Tier-I) –04/08/2021 (Shift-I) Ans. (c): Sustenance - (Livelihood/Living), Antonym, Starvation - (Dearth/Famine) Meaning of other words – Nourishment-Nutrition/Foster Livelihood-Living/Subsistence Refreshment-Freshness/Newness 576. Select the most appropriate ANTONYM of the given word. STRETCH (a) Extend (b) Elongate (c) Compress (d) Lengthen SSC CHSL (Tier-I) –13/04/2021 (Shift-I) Ans. (c): Stretch - (Elongate/Lengthen), Antonym Compress - (Brief/Contract) Meaning of other words – Lengthen – Prolong/Increase Extend - Enlarge/Expand Elongate – Amplify/Lengthen 577. Select the most appropriate antonym of the given word Sprightly (a) Lively (b) Lethargic (c) Active (d) Energetic SSC CGL (Tier-I) –24/08/2021 (Shift-I) Antonyms

Ans. (b): Sprightly - (Agile/Merry), Antonym, Lethargic - (Sluggish/Lazy) Meaning of other words – Lively – Energetic/Active Active – Agile/Energetic Energetic – Dynamic/Animated 578. Select the most appropriate antonym of the given word. Slur (a) Insult (b) Compliment (c) Assortment (d) Mixture SSC CGL (Tier-I) –17/08/2021 (Shift-I) Ans. (b): Slur- (Slander/Infamy), AntonymCompliment - (Praise/Laud) Meaning of other words – Insult – Dishonor/Disgrace Assortment – Classification/Taxonomy Mixture – Amalgam/Combination 579. Select the most appropriate antonym of the given word. Stalwart (a) Cowardly (b) Robust (c) Firm (d) Stout SSC CGL (Tier-I) –17/08/2021 (Shift-I) Ans. (a): Stalwart - (Adventurous/Strong), Antonym, Cowardly – (Timid/Fainthearted) Meaning of other words – Robust – Hale/Tangible Firm – Hard/Unyielding/Concrete Stout – Sturdy/Fat 580. Select the most appropriate antonym of the given word. Seize (a) Snatch (b) Catch (c) Grab (d) Loosen SSC CGL (Tier-I) –13/08/2021 (Shift-I) Ans. (d): Seize – (Confiscate/Capture) Antonym, Loosen – (Slacken) Meaning of other words – Snatch – Grab/Grasp Catch – Hold/Grip Grab - Capture 581. Select the most appropriate antonym of the given word. SPARSE (a) thin (b) scant (c) abundant (d) meagre SSC CPO-SI (Tier-II) 27/09/2019 (3 pm - 5 pm) Ans. (c): 'Sparse' – (Scatter/Disperse) Antonym Abundant – (Ample/Bountiful) Meaning of other words – Scant – Meager/Little/Some Thin – Slender/Lean Meager - Scant/Little 582. Select the most appropriate antonym of the given word. SHALLOW (a) dry (b) deep (c) close (d) clean

502

SSC Stenographer Grade C&D –05/02/2019 (3:5pm)

YCT

Ans. (b): 'Shallow' – (Surface) Antonym - Deep – (Profound) Meaning of other words – Dry - Drought Close - Near/Turn off Clean – Pure/Clear 583. Select the most appropriate antonym of the given word : STRANGE (a) durable (b) delicate (c) unknown (d) familiar SSC Stenographer Grade C&D –05/02/2019 (3:5pm)

Ans. (d): 'Strange' – (Bizarre/Abnormal) Antonym Familiar – (Well-known) Meaning of other words – Durable - Lasting Delicate - Fragile Unknown - Unfamiliar 584. In the following question, out of the given four alternatives, select the one which is opposite in meaning of the given word. Swindle (a) Insert (b) Dupe (c) Ample (d) Honesty SSC CGL (Phase-II) 17/02/2018 Ans. (d): Swindle – (Hoax/Betrayal), Antonym Honesty – (Integrity/Veracity) Meaning of other words – Insert – Teem Dupe – Gullible Ample – Sufficient 585. Select the most appropriate ANTONYM of the given word. SPIRITUAL (a) material (b) ethereal (c) sacred (d) divine SSC CPO-SI – 23/11/2020 (Shift-II) Ans. (a): Spiritual- (Metaphysical), AntonymMaterial (Hylic/Physical) Meaning of other words – Ethereal - Celestial/Intangible Sacred – Holy/Chaste Divine – Heavenly/Supernal 586. Select the most appropriate ANTONYM of the given word. SUAVE (a) rude (b) brisk (c) charming (d) polite SSC CPO-SI – 24/11/2020 (Shift-II) Ans. (a): 'Suave'- (Elegant/Urbane), Antonym 'Rude' (Impolite/Boorish) Meaning of other words – Brisk – Agile/Quick Charming – Alluring/Attractive Polite – Humble/Coy 587. Select the antonym of the given word. SWERVE (a) droop (b) lurch (c) deviate (d) straighten SSC CGL (Tier-I) – 07/06/2019 (Shift-II) Antonyms

Ans. (d): 'Swerve' - (Deviate/Yaw), Antonym straighten (Arrange/Align) Meaning of other words – Droop Dwindle/Fade Lurch Stagger/Totter Deviate Straggle/Evade 588. Select the antonym of the given word. SOBER (a) nervous (b) agitated (c) serious (d) calm SSC CGL (Tier-I) – 07/06/2019 (Shift-II) Ans. (b): 'Sober' - (Serious/Solemn), Antonym Agitated (Disturbed/Worried) Meaning of other words – Nervous - Restless/Confuse Serious - Solemn/Sober Calm - Quiet/tranquil 589. Select the most appropriate antonym of the given word. SCANTY (a) profuse (b) small (c) precise (d) concise SSC CGL (Tier-I) – 12/06/2019 (Shift-III) Ans. (a): Scanty- (Meager/Diminutive), Antonym Profuse - (Abundant/Exuberant) Meaning of other words – Small- Miniature/Little Precise- Accurate/Exact Concise- Brief/Abbreviate 590. Select the most appropriate antonym of the given word. STATIONARY (a) tired (b) still (c) shifting (d) motionless SSC CGL (Tier-I) – 19/06/2019 (Shift-III) Ans. (c): Stationary - (Stable/Constant), Antonym shifting, (Transfer/Change) Meaning of other words – Tired Weary/Exhausted Still Yet/Stable Motionless Stationary/Stagnant 591. Select the most appropriate antonym of the given word. SEVERE (a) mediocre (b) meticulous (c) morose (d) mild SSC CGL (Tier-I) – 10/06/2019 (Shift-III) Ans. (d): Severe–(Harsh/Grievous), Antonym- Mild (Gentle/Placid) Meaning of other words – Mediocre- Ordinary/Common Meticulous- Careful Morose- Gloomy/Sullen 592. Select the antonym of the given word. STALE (a) flat (b) fresh (c) dry (d) sour SSC CGL (Tier-I) – 07/06/2019 (Shift-I)

503

YCT

Ans. (b): 'Stale' - (Rancid), Antonym- Fresh (Clean/Neat) Meaning of other words – Flat - Plane/Level Dry - Arid/Drought Sour - Acetic/Tart 593. Select the antonym of the given word. SCARCE (a) seldom (b) scanty (c) few (d) plentiful SSC CGL (Tier-I) – 04/06/2019 (Shift-I) Ans. (d): Scarce- (Meager/Scant), Antonym- Plentiful (Abundant/Bountiful) Meaning of other words – Seldom- Occasionally/Infrequently Scanty- Meager/Little Few- Some/Diminutive 594. Select the most appropriate ANTONYM of the given word. SUBSIDE (a) abate (b) ease (c) diminish (d) enlarge SSC CPO-SI – 09/12/2019 (Shift-II) Ans. (d): Subside- (Lessen/Decrease), AntonymEnlarge- (Enhance/Amplify) Meaning of other words – Abate- Reduce/Decrease Ease- Comfort/Relief Diminish- Slacken/Shorten 595. Select the most appropriate ANTONYM of the given word. STABLE (a) shaky (b) tough (c) reliable (d) even SSC CPO-SI – 09/12/2019 (Shift-II) Ans. (a): Stable- (Stagnant/Permanent), AntonymShaky- (Unstable/Unsteady) Meaning of other words – Tough- Harsh/Rigid Reliable- Credible/Authentic Even- Alike/Equal 596. Select the most appropriate ANTONYM of the given word. SOGGY (a) gloomy (b) unfit (c) dry (d) broken SSC CPO-SI – 13/12/2019 (Shift-II) Ans. (c): Soggy - (Wet/Moist), Antonym - Dry (Arid)~ Meaning of other words – Gloomy – Sullen/Sorrowful Unfit – Incapable/Incompetent Broken – Split/Dissolve 597. Select the most appropriate antonym of the given word. SAGACIOUS (a) Ingenious (b) Prudent (c) Astute (d) Ignorant SSC CPO-SI – 13/12/2019 (Shift-I) Antonyms

Ans. (d): Sagacious - (Brilliant/Intelligent), AntonymIgnorant (Unaware/Unfamiliar) Meaning of other words – Ingenious – Simple/Accurate/Proficient Prudent - Discerning Astute – Nimble/Crafty/Clever 598. Select the most appropriate antonym of the given word. SALIENT (a) negligible (b) prominent (c) striking (d) clear SSC CHSL (Tier-I) –10/07/2019 (Shift-I) Ans. (a): Salient- (Main/Chief), Antonyms Negligible- (Insignificant) Meaning of other words – Prominent- Famous/Renowned Striking- Bizarre/Grotesque Clear- Apparent/Neat/Clean 599. Select the most appropriate antonym of the given word. SHALLOW (a) deep (b) narrow (c) crooked (d) slight SSC CHSL (Tier-I) –10/07/2019 (Shift-II) Ans. (a): Shallow- (Superficial/Cursory) AntonymDeep (Profound/Intensive) Meaning of other words – Narrow- Parochial Crooked- Curved/Devious Slight- Weak/Feeble/Scant 600. Select the most appropriate antonym of the given word. SPARSE (a) dense (b) strong (c) thin (d) weak SSC CHSL (Tier-I) –04/07/2019 (Shift-III) Ans. (a): Sparse- (Dispersed/Diffuse), Antonym Dense- (Compact/Thick) Meaning of other words – Strong- Firm/Powerful Thin- Slender/Lean Weak- Feeble/Powerless 601. Select the most appropriate ANTONYM of the given word. SPIRITUAL (a) Righteous (b) Physical (c) Ghostly (d) Pious SSC CHSL (Tier-I) 16/10/2020 (Shift-III) Ans. (b): 'Spiritual'- (Metaphysical), Antonym Physical (Hylic/Material) Meaning of other words – Righteous – Virtuous/Truthful Ghostly – Phantom/Terrifying Pious – Holy/Chaste 602. Select the most appropriate ANTONYM of the give word. SACRED (a) Pious (b) Profane (c) Solemn (d) Divine SSC CHSL (Tier-I) 19/10/2020 (Shift-II) SSC CGL (Tier-I) – 10/06/2019 (Shift-III)

504

YCT

Ans. (b): Sacred– (Pious/Holy), Antonym - Profane (Unholy/Impious) Meaning of other words – Pious – Pure/Chaste Solemn – Serious/Sedate Divine – Supernal/Providential 603. Select the most appropriate ANTONYM of the given below word SURVEILLANCE (a) Neglect (b) Attack (c) Vigilance (d) Endurance SSC CHSL (Tier-I) 12/10/2020 (Shift-I) Ans. (a): Surveillance – (Observation/Custody), Antonym - Neglect (Disregard/Overlook) Meaning of other words – Attack – Assail/Invasive Vigilance – Watchfulness/Cautious Endurance – Stamina/Fortitude 604. In the following Question, Out of the given four alternatives, select the one in which is opposite the given word. Spare (a) Necessary (b) Free (c) Option (d) Surplus SSC MTS 9-10-2017 (Shift-III) Ans. (a): Spare- (Extra/Excess), Antonym Necessary-(Essential/Compulsory) Meaning of other words – Free- Liberate/Release Option- Choice/Selection Surplus- Additional/Extra 605. In the following question, out of the given four alternatives, select the one which is opposite in meaning of the given word. Stern (a) Bitter (b) Cruel (c) Light (d) Grim SSC MTS 10-10-2017 (Shift-III) Ans. (c): 'Stern'- (Harsh/Firm/Ruthless), Antonym'Light' (Floaty/Simple) Meaning of other words – Bitter – Pungent/Sore Cruel – Ferocious/Brutal Grim – Ruthless/Rigorous 606. In the following question, out of the given four alternatives, select the one which is opposite in meaning of the given word. Solace (a) Pity (b) Disharmony (c) Relief (d) Cheer SSC MTS 10-10-2017 (Shift-II) Ans. (b): Solace– (Consolation/Calmness/Satisfy), Antonym - Disharmony (Discord/Acrimony) Meaning of other words – Pity– Compassion/Ruth Relief– Comfort/Ease Cheer– Joy/Happiness Antonyms

607. Select the most appropriate antonym of the given word. Steep (a) Narrow (b) Rough (c) Shallow (d) Flat SSC MTS – 06/08/2019 (Shift-III) Ans. (d): Steep- (Gradient/Tilt), Antonym - Flat (Plane/Level) Meaning of other words – Narrow – Parochial/Tapered Rough – Harsh/Rigid Shallow – Superficial/Cursory 608. Select the most appropriate antonym of the word. SYMPATHY (a) Tragedy (b) Warmth (c) Cruelty (d) Enthusiasm SSC MTS – 22/08/2019 (Shift-II) Ans. (c): 'Sympathy'- (Compassion/Empathy), Antonym - 'Cruelty' (Barbarity/Ruthlessness) Meaning of other words – Tragedy - Disaster/Calamity Warmth - Friendliness/Hotness Enthusiasm - Zeal/Euphoria 609. Select the most appropriate antonym of the word : Slander (a) Appeal (b) Malign (c) Admire (d) Defame SSC MTS – 22/08/2019 (Shift-I) Ans. (c): 'Slander'- (Tarnish, Calumniate) Antonym Admire (Praise/Extol) Meaning of other words – Appeal – Request/Implore Malign – Evil/Rogue Defame - Denigrate/Calumniate 610. Select the most appropriate antonym of the word. Startle (a) Shock (b) Soothe (c) Amaze (d) Agitate SSC MTS – 21/08/2019 (Shift-I) Ans. (b): Startle - (Flabbergast/Frighten), Antonym Soothe (Calm/Assuage) Meaning of other words – Shock – Astonish/Horrify Amaze – Flabbergast/Daze Agitate – Ignite/Stimulate 611. Select the antonym of the given word. SERENE (a) Superb (b) Splendid (c) Showy (d) Stressed SSC GD – 18/02/2019 (Shift-II) Ans. (d): Serene - (Tranquil/Stable), Antonym Stressed (Stained/Anxious) Meaning of other words – Superb – Stunning/Splendid Splendid – Magnificent/Excellent Showy – Ostentatious/Grandiose 612. Select the antonym of the given word. STINGY (a) Generous (b) Skinny (c) Cheap (d) Mean SSC GD – 14/02/2019 (Shift-I)

505

YCT

Ans. (a): Stingy- (Niggard/Miser), Antonym Generous (Lenient/Liberal) Meaning of other words – Skinny – Thin/Slender Cheap – Inexpensive/Insignificant Mean – Express/Convey

T 613. Select the most appropriate antonym to substitute the bracketed word in the given sentence. She ventured to go back to the tree of the (tryst), the mulberry with the shining white fruit. (a) Separation (b) Rendezvous (c) Engagement (d) Date SSC MTS 19/05/2023 (Shift-III) Ans. (a) : The most appropriate antonym of the above bracketed word- ‘tryst’ is ‘separation’. Meaning of other optionsRendezvous – Assignation Engagement – Betrothal, Date – Period, Visit, Thus, the correct answer is option (a). 614. Select the most appropriate ANTONYM of the given word. Timid (a) Diffident (b) Shy (c) Bold (d) Fearful SSC GD 10/01/2023 (Shift-I) Ans. (c) : Timid (Faithearted, Chicken - hearted), Antonym Bold (Brave) Meaning of the other optionsDifferent - Bashful Shy - Demure, Reticent Fearful - Frightened, Afraid 615. Select the most appropriate ANTONYM of the underlined word. Trivial issues have always interested our boss. (a) Hopeful (b) Fierce (c) Common (d) Significant SSC CGL (Tier-I) 19/07/2023 (Shift-IV) Ans. (d) : The most appropriate antonym of the underlined word 'Trivial' (Negligible) is 'significant.' The other options are Hopeful means - optimistic Fierce means - Furious Common means - Ordinary. Thus, the correct answer is option (d) 616. Select the most appropriate ANTONYM of the given word in the following sentence. Traitor People should vote for honest and loyal leaders not for betrayer and defender. (a) Loyal (b) Honest (c) Betrayer (d) Defender SSC CGL (Tier-I) 17/07/2023 (Shift-II) Antonyms

Ans. (a) : Antonym of 'Traitor' is 'loyal' meaning of other word – Honest – sincere Betrayer – unfaithful Defender – Protector 617. Select the word that is opposite in meaning (antonym) to the word given below 'Timorous' (a) Brazen (b) Scared (c) Gutless (d) Reserved SSC CHSL (Tier-I) 15/03/2023 (Shift-I) Ans. (a) : Timorous - Antonym - 'Brazen' Meaning of other wordReserved → Derm Gutless → Coward Scared → Frightened 618. Select the most appropriate antonym of the given word. 'TIMID' (a) SMELLY (b) COURAGEOUS (c) LUCID (d) CAREFUL SSC CHSL (Tier-I) 15/03/2023 (Shift-I) Ans. (b) : Timid - antonym - 'courageous' Meaning of other wordSmelly → Stinking Lucid → Bright Careful → Wary 619. Select the most appropriate ANTONYM of the underlined word. He carries that sorrow with him now, just under the surface, almost tangible. (a) Detectable (b) Abstract (c) Actual (d) Distinct SSC MTS-06/07/2022 Shift-III Ans. (b) : Tangible (Corporeal), Antonym - Abstract (Intangible) Meaning of the other OptionsDetectable – Noticeable Actual – Real Distinct – Separate 620. Select the most appropriate ANTONYM of the given word. Tenacious (a) Relentless (b) Persistent (c) Yielding (d) Steadfast SSC CGL (Tier-I) 11/04/2022 Shift-II Ans. (c) : Tenacious (Firm/Strict), Antonym – yielding (Surrender/Submit) Meaning of the other OptionsRelentless – Merciless/Ruthless Persistent – Tenacious Steadfast – Resolute 621. Select the most appropriate ANTONYM of the given word. Taut (a) Slack (b) Solid (c) Tight (d) Strong SSC CGL (Tier-I) 19/04/2022 Shift-II

506

YCT

Ans. (a) : Taut (Tight/Stretched), Antonym - Slack (Loose/Lax) Meaning of the other OptionsSolid – Hard/Rigid Tight – Stretched Strong – Powerful/Robust 622. Select the most appropriate antonym of the given word. TUMULT (a) disturbance (b) commotion (c) quarrel (d) calmness SSC Stenographer (Grade C & D) 12.11.2021 Shift-II Ans. (d) : 'Tumult' (Commotion/Uproar), antonym 'calmness' (Peace/Tranquility) Meaning of the other wordscommotion - Disturbance/Fuss quarrel - Fight/Dispute disturbance - Disorder/Confusion 623. Select the most appropriate ANTONYM of the given word. Transparent (a) Opaque (b) Beautiful (c) Clean (d) Shining SSC CHSL (Tier-I) –15/04/2021 (Shift-I) Ans. (a): Transparent - (Clear/Diaphanous), Antonym, Opaque - (Unclear/Cloudy) Meaning of other words – Clean – Clear/Spotless/Neat Shining – Glossy/Bright Beautiful – Lovely/Pretty 624. Select the most appropriate ANTONYM of the given word. Toxic (a) Lethal (b) Venomous (c) Harmful (d) Healthy SSC CGL (Tier-I) –16/08/2021 (Shift-I) Ans. (d): Toxic - (Poisonous/Venomous), Antonym, Healthy- (Well/Fit) Meaning of other words – Lethal – Fatal/Deadly Venomous – Poisonous/Toxicant Harmful – Detrimental/Maleficent 625. Select the most appropriate antonym of the given word. TRANSPARENT (a) thick (b) weary (c) glassy (d) opaque SSC Stenographer Grade C&D –05/02/2019 (3:5pm)

Ans. (d): 'Transparent'– (Clear/Diaphanous),Antonym - Opaque – (Unclear/Cloudy) Meaning of other words – Thick – Plump/Fat Weary – Tired/Exhausted Glassy – Spiritless/Azoic/Lifeless 626. In the following question, out of the given four alternatives, select the one which is opposite in meaning of the given word. Tempestuous (a) Stress (b) Toil (c) Calm (d) Examine SSC CGL (Phase-II) 17/02/2018 Antonyms

Ans. (c): Tempestuous – (Turbulent/Stormy), Antonym - Calm (Tranquil/Quiet) Meaning of other words – Stress – Exertion/Strain/Tension Examine – Scrutiny/Check/Investigate Toil – Hard work/Drudgery 627. Select the most appropriate antonym of the given word. TYRANT (a) rival (b) benefactor (c) patron (d) champion SSC CGL (Tier-I) – 13/06/2019 (Shift-I) Ans. (b): Tyrant- (Dictator), Antonym - Benefactor (Philanthropist) Meaning of other words – Rival- Opponent/Competitor Patron- Guardian/Protector Champion- Paramount/Winner 628. Select the most appropriate antonym of the given word. TRANSIENT (a) stationary (b) temporal (c) celestial (d) permanent SSC CGL (Tier-I) – 07/06/2019 (Shift-III) Ans. (d): 'Transient' - (Momentary/Evanescent), Antonym- Permanent (Eternal/Everlasting) Meaning of other words – Stationary - Stagnant/Unchangeable Temporal - Plebeian/Earthly Celestial - Astronomical/Heavenly 629. Select the antonym of the given word. TENDER (a) Rough (b) Warm (c) Gentle (d) Soft SSC CGL (Tier-I) – 04/06/2019 (Shift-II) Ans. (a): Tender- (Soft/Fragile), AntonymRough- (Unrefined/Harsh) Meaning of other words – Warm- Hot/Tepid Gentle- Mild/Placid Soft- Pulpy/Flexible 630. Select the most appropriate ANTONYM of the given word. TENSE (a) stiff (b) strained (c) close (d) relaxed SSC CPO-SI – 11/12/2019 (Shift-I) Ans. (d): Tense- (Stressful/Anxious), Antonym Relaxed (Ease/Calm) Meaning of other words – Stiff – Hard/Tough Strained – Stretched/Synthetic Close – Near/Shut 631. Select the most appropriate ANTONYM of the given word. TURBID (a) clear (b) confused (c) mixed (d) varied SSC CPO-SI – 12/12/2019 (Shift-II)

507

YCT

Ans. (a): Turbid - (Muddy/Dirty), Antonym - Clear (Clean/Neat) Meaning of other words – Confused – Disturbed/Perplexed Mixed – Complex/Combined/Assorted Varied – Different/Diverse/Various 632. Select the most appropriate antonym of the given word. TACIT (a) inaudible (b) silent (c) mute (d) explicit SSC CHSL (Tier-I) –09/07/2019 (Shift-II) Ans. (d): Tacit- (Still/Silent) Antonym- ‘‘Explicit(Candid/Obvious) Meaning of other words – Inaudible- Unheard/Indistinct Silent- Still/Untold Mute- Dumb/Voiceless 633. Select the most appropriate ANTONYM of the given word. TESTIMONY (a) Approval (b) Difficulty (c) Witness (d) Disproof SSC CHSL (Tier-I) 16/10/2020 (Shift-III) Ans. (d): Testimony- (Evidence/Showing), Antonym 'Disproof' (Rebuttal) Meaning of other words – Approval – Consent/Acceptance Difficulty - Hardship/Trouble Witness – Evidence/Proof 634. Select the most appropriate ANTONYM of the given word. TAME (a) control (b) Upset (c) Restrain (d) Sanctify SSC CHSL (Tier-I) 19/03/2020 (Shift-II) Ans. (b): Tame– (Pet/Calm/Quiet), Antonym - Upset (Trouble/Disturb) Meaning of other words – Control– Operate/Command Restrain– Stop/Inhibit/Prevent Sanctify– Purge/Hallowed 635. Select the most appropriate antonym of the given word. Timid (a) Sincere (b) Bold (c) Punctual (d) Dubious SSC CHSL (Tier-I) 12/10/2020 (Shift-III) Ans. (b): Timid – (Coy/Coward), Antonym - Bold (Dauntless/Daring) Meaning of other words – Sincere – Honest/Genuine Punctual – Scrupulous/Timely Dubious – Doubtful/Suspect 636. In the following question, out of the given four alternatives, select the one which is opposite in meaning of the given word. Taciturn (a) Silent (b) Talkative (c) Reticent (d) Reserved SSC MTS 11-10-2017 (Shift-III) Antonyms

Ans. (b): The most appropriate opposite for 'Taciturn' is 'Talkative' means-'out spoken' while other options are incorrect. Thus, the correct answer is option (b) 637. In the following question, out of the given four alternatives, select the one which is opposite in the meaning of the given word. Thrive (a) Arrive (b) Fail (c) Develop (d) Shine SSC MTS 11-10-2017 (Shift-I) Ans. (b): The given word 'Thrive' is opposite of 'Fail' which means 'Miscarry'. The meanings of other words are Arrive- Come Develop - Cultivate Shine - Glow Thus, the correct answer is option (b) 638. Select the most appropriate antonym of the given word. Tranquil (a) Calm (b) Peaceful (c) Cool (d) Stormy SSC MTS – 06/08/2019 (Shift-I) Ans. (d): The most appropriate Antonym of the word 'Tranquil' is 'Stormy' which means- 'Rampant', while the other option are incorrect. Thus, the correct answer is option (d). 639. Select the most appropriate antonym of the word. TREACHEROUS (a) Deceitful (b) Disloyal (c) Faithful (d) Dangerous SSC MTS – 19/08/2019 (Shift-II) Ans. (c): The most appropriate Antonym of the word 'Treacherous' is 'Faithful'. Treacherous', means'Disloyal', while other options are incorrect. Thus, the correct answer is option (c).

U 640. Select the most appropriate antonym to substitute the bracketed word in the given sentence. They are virtually unimpeachable in the (unbiased), invariably correct reading of a race. (a) prejudiced (b) ceasing (c) captivity (d) stranger SSC MTS 12/05/2023 (Shift-III) Ans. (a) : The opposite meaning of the above word – 'Unbiased' is 'Prejudiced' (partisan). Meaning of other options – Closing – Intermit, captivity – Bondage, Stranger – sturdy, Thence, the correct option is (a) 641. Select the most appropriate ANTONYM of the underlined word. Her dog can climb under the fence. (a) Over (b) Sink (c) Beneath (d) Behind SSC CGL (Tier-I) 14/07/2023 (Shift-I)

508

YCT

Ans. (a) : Antonym of underline word “under” is “over”. The Remaining words and their antonymSink – Rise, Float Beneath – Above, Behind – Ahead of, 642. Select the most appropriate ANTONYM of the given word. Upgrade (a) Advance (b) Lean (c) Demote (d) Elevate SSC MTS-05/07/2022 Shift-III Ans. (c) : Upgrade (progress), Antonym - Demote (degrade/Dismiss) Meaning of the other OptionsAdvance – progress Lean – Slender/weak Elevate – Raise/Upgrade 643. Select the most appropriate ANTONYM of the given word. USURP (a) wrest (b) annex (c) release (d) assume SSC CPO-SI – 25/11/2020 (Shift-I) Ans. (c): The most appropriate Antonym of the word 'Usurp' is 'Release'. The word 'Usurp' means 'Arrogate'. The meanings of other words are Wrest - Inflexion Annex - Supplement Assume - Suppose Thus, the correct answer is option (c) 644. Select the most appropriate antonym of the given word. URBAN (a) rural (b) alien (c) casual (d) modern SSC CHSL (Tier-I) –04/07/2019 (Shift-II) Ans. (a): The most appropriate Antonym of the word 'Urban' is 'Rural'. The meaning of others words are Alien - Foreigner Casual - Accidental Modern - Recent Thus, the correct answer is option (a) 645. Select the most appropriate ANTONYM of the given word. UNWORTHY (a) Trustworthy (b) Helpful (c) Unskilled (d) Deserving SSC CHSL (Tier-I) 19/03/2020 (Shift-III) Ans. (d): The most appropriate Antonym of the word 'Unworthy' is 'Deserving'. Deserving means eligible, while other options are incorrect. Thus, the correct answer is option (d) 646. In the following question, out of the given four alternatives, select the one which is opposite in meaning of the given word. Unitary (a) Whole (b) Incomplete (c) Integral (d) Entire SSC MTS 7-10-2017 (Shift-I) Antonyms

Ans. (b): The most appropriate Antonym of the word 'Unitary' is 'Incomplete'. The meaning of the other words areWhole - Absolute Integral - Undivided Entire - Total Thus, the correct answer is option (b) 647. Select the most appropriate antonym of the given word. Unanimous (a) Identical (b) Vague (c) Terse (d) Individual SSC CGL (Tier-I) – 11/06/2019 (Shift-II) Ans. (d): The most appropriate Antonym of the word 'Unanimous' is 'Individual'. The meanings of other words are Vague - Obscure Terse - Concise Identical - Tantamount Thus, the correct answer is option (d) 648. Select the most appropriate antonym of the given word. Unruly (a) Rampant (b) Rogue (c) Timid (d) Well-behaved SSC MTS 9-10-2017 (Shift-II) Ans. (d): The most appropriate Antonym of the word 'Unruly' is 'Well-behaved' The meanings of other words are Rampant - Aggressive Rogue - Malevolent Timid - Cowardly Thus, the correct answer is option (d) 649. Select the most appropriate antonym of the given word. Uproar (a) Clamor (b) Calm (c) Vague (d) Jovial SSC CHSL (Tier-I) –05/07/2019 (Shift-I) Ans. (b): The most appropriate Antonym of the word. 'Uproar' is 'Calm'. The meanings of other words are Clamour - Cacophony Vague - Ambiguous Jovial - Chromatic Thus, the correct answer is option (b)

V 650. Select the most appropriate antonym of the given word. Viscous (a) Watery (b) Grim (c) Brutal (d) Thick SSC MTS 02/05/2023 (Shift-I) Ans. (a) : The most appropriate antonym of the word 'viscous' (Gummy) is 'watery' (Aquatic). Meaning for the other words Grim means- Awful Brutal means- Ferocious Thick means- Plump Thus, the correct answer is option (a).

509

YCT

651. Select the most appropriate antonym of the underlined word. This novelist is known for his verbosity. (a) Succinctness (b) Candour (c) Vengeance (d) Humility SSC CHSL (Tier-I) 14/08/2023 (Shift-IV) Ans. (a) : Option (a) 'Succinctness' – (The act of expressing something clearly in a few words) is the 'antonym of the underlined word 'verbosity' – (The fact or quality of using more words than needed).] Other words have following meaning Candour – The quality of being open and honest. Vengeance – Punishment inflicted in retaliation for an injury or offence. Humility – The quality of not thinking that you are better than other people. 652. Choose the word that is opposite in meaning to the given word. Vivacity (a) Sap (b) Vim (c) Lush (d) Apathy SSC CHSL (Tier-I) 17/03/2023 (Shift-IV) Ans. (d) : Vivacity - Opposite - Apathy Meaning of other wordsSap - Absorb Lush- Ample Vim - Power 653. Select the most appropriate ANTONYM of the given word. Vanish (a) Disappear (b) Depart (c) conquer (d) Emerge SSC MTS-18/07/2022 Shift-I Ans. (d) : Vanish (Disappear), Antonym – Emerge (Appear) Meaning of the other wordsDisappear – Evanesce Depart – Leave/Go off Conquer – win 654. Select the most appropriate antonym of the given word. Variance (a) Harmony (b) Intellectual (c) Convention (d) Equal SSC CHSL 26.05.2022 Shift-II Ans. (a) : variance (Dissonance), Antonym - Harmony (Balancing) Meaning of the other OptionsIntellectual – Scholarly/Intelligent Convention – Conference/Tradition Equal – Identical/Like/Same 655. Select the most appropriate ANTONYM of the given word. Vulnerable (a) Sensitive (b) Tender (c) Breakable (d) Secure SSC Constable GD-17/11/2021 Shift-II Antonyms

Ans. (d) : Vulnerable - (Unprotected/Unsafe) Antonym - Secure (Safe) Meaning of the other OptionsSensitive - emotional/ Sentimental Tender - Proposal/Soft Breakable - Fragile 656. Select the most appropriate antonym of the given word. Violent (a) Delicious (b) Mighty (c) Urgent (d) Gentle SSC Constable GD-02/12/2021 Shift-I Ans. (d) : Violent (Brutal/Fierce), Antonym - Gentle (Liberal/Decent) Meaning of the other OptionsDelicious - Tasty Mighty - Powerful Urgent - Essential 657. Select the most appropriate antonym of the given word. VIGILANT (a) wary (b) careful (c) inattentive (d) cautious SSC Stenographer (Grade C & D) 15.11.2021 Shift-I Ans. (c) : Vigilant (Watchful/Awake), Antonym inattentive (Neglectful/careless) Meaning of the other wordswary -Alert careful - Aware cautious - Careful 658. Select the most appropriate antonym of the given word. Praise (a) Celebrate (b) Condemn (c) Secure (d) Hail SSC CHSL 24.05.2022 Shift-III Ans. (b) : Praise - (Compliment/Laud), Antonym, Condemn (Censure/Criticize) Meaning of the other wordsCelebrate - Commemorate/Rejoice Secure - Safe/Obtain Hail - welcome/ Hailstorm 659. Select the most appropriate antonym of the given word. VACATE (a) holiday (b) occupy (c) discharge (d) abandon

510

SSC Stenographer Grade C&D –05/02/2019 (3:5pm)

Ans. (b): The most appropriate Antonym of the word 'Vacate' is 'occupy' The meanings of other words are Holiday - Leave Discharge - Release Abandon - Let off Thus, the correct answer is option (b) YCT

660.

In the following question, out of the given four alternatives, select the one which is opposite in meaning of the given word. Vista (a) Confront (b) Contour (c) Blindness (d) Outlook SSC CGL (Phase-II) 17/02/2018 Ans. (c): The most appropriate Antonym of the word 'Vista' is 'Blindness'. Vista means Excursion, while other options are not correct. Thus, the correct answer is option (c) 661. Select the most appropriate antonym of the given word. VIGILANT (a) visible (b) elusive (c) careful (d) careless SSC CGL (Tier-I) – 11/06/2019 (Shift-III) Ans. (d): The most appropriate Antonym of the word 'Vigilant' (sharp) is 'Careless' while other options are incorrect. Elusiuo - Deceptive Thus, the correct answer is option (d) 662. Select the most appropriate antonym of the given word. Virulent (a) Dangerous (b) Amicable (c) Lethal (d) Powerful SSC CHSL (Tier-I) –11/07/2019 (Shift-II) Ans. (b): The most appropriate Antonym of the word. Virulent' (poisonous) is 'Amicable'. The meaning of other words is Dangerous - Hazardous Lethal - Deadly Thus, the correct answer is option (b) 663. Select the most appropriate ANTONYM of the given word. VERSATILE (a) Insincere (b) Truthful (c) Honest (d) Inflexible SSC CHSL (Tier-I) 18/03/2020 (Shift-II) Ans. (d): The most appropriate Antonym of the word 'Versatile' is 'Inflexible'. The other options are incorrect. Thus, the correct answer is option (d) 664. Select the most appropriate antonym of the word Vanity (a) Nobility (b) Honour (c) Conceit (d) Modesty SSC MTS – 21/08/2019 (Shift-I) Ans. (d): The most appropriate Antonym of the word 'Vanity' (arrogant) is 'Modesty' while other options are incorrect. Thus, the correct answer is option (d) 665. Select the most appropriate antonym of the word. Vile (a) Crafty (b) decent (c) Horrid (d) Evil SSC MTS – 20/08/2019 (Shift-II) Ans. (b): The most appropriate Antonym of the word 'Vile' is 'Decent'. The meaning of other words is _____ Crafty - Devious Horrid - Dolorous Evil - Harm. Thus, the correct answer is option (b) Antonyms

W 666. Select the most appropriate ANTONYM of the given word. Wicked (a) Evil (b) Sinful (c) Errant (d) Moral SSC MTS-08/07/2022 Shift-I Ans. (d) : Wicked (Vicious), Antonym, Moral (Ethical) Meaning of the other OptionsEvil – Rogue Sinful – Immoral Errant – Rover 667. Select the most appropriate antonym of the given word. WORSEN (a) Leave (b) Disappear (c) Command (d) Improve SSC Sel. Post Phase-VIII (H.L.) 09.11.2020 (Shift-I)

Ans. (d): The most appropriate Antonym of the word 'Worsen' is 'Improve' while other options are incorrect. Thus, the correct answer is option (d) 668. Select the most appropriate ANTONYM of the given word. WARM (a) hot (b) cool (c) springy (d) spongy SSC CPO-SI – 23/11/2020 (Shift-I) Ans. (b): The most appropriate Antonym of the word 'Warm' is 'Cool'. The meaning of other words is______ Spongy - Porous Springy - Resilient Thus, the correct answer is option (b) 669. Select the most appropriate antonym of the given word. WEAKNESS (a) illness (b) strength (c) disability (d) bravery SSC CGL (Tier-I) – 13/06/2019 (Shift-I) Ans. (b): The most appropriate Antonym of the given word 'Weakness' is 'Strength' while other options are in correct. Thus, the correct answer is option (b) 670. Select the most appropriate ANTONYM of the given word. WANE (a) dwindle (b) abate (c) increase (d) fall SSC CPO-SI – 11/12/2019 (Shift-I) Ans. (c): The most appropriate Antonym of the word 'wane' (reduce) is 'Increase'. The meanings of other words is _____ Dwindle - Dry Abate - Subside Thus, the correct answer is option (c)

511

YCT

X

Z

671. Select the most appropriate antonym of the given word. Xenial (a) Abuse (b) Yearn (c) Inhospitable (d) Vie SSC MTS 11-10-2017 (Shift-II) Ans. (c): The most appropriate Antonym of the word 'Xenial' is 'Inhospitable'. Xenial means 'Hospitable' Thus, the correct answer is option (c) 672. Select the most appropriate antonym of the given word. Xenophile (a) Veteran (b) Xenophobia (c) Commotion (d) Adorn SSC CGL (Tier-I) – 12/06/2019 (Shift-I) Ans. (b): The most appropriate Antonym of the word 'Xenophile' is 'Xenophobia'. Xenophile means- One attract to foreign things. The meaning of other words is ____ Veteran - Experience, Adept Commotion - Excitement Adorn - Decorate. Thus, the correct answer is option (b)

676. Select the most appropriate ANTONYM of the underlined word. Their reverence reaches its zenith here at the birthplace of the country's founder. (a) crest (b) Nadir (c) horizon (d) pinnacle SSC MTS-06/07/2022 Shift-I Ans. (b) : Zenith (Apex), Antonym - Nadir (Bottom) Meaning of the other OptionsCrest – Top/Summit Horizon – Skyline Pinnacle – Vertex/Peak 677. Select the most appropriate antonym of the given word. Zenith (a) Climax (b) Nadir (c) Deny (d) Adorn SSC CHSL (Tier-I) 18/03/2020 (Shift-I) Ans. (b): The most appropriate Antonym of the word 'Zenith' (apex) is 'Nadir' (Fall). The meaning of other words is _____ Climax - Extremity Deny - Negate. Thus, the correct answer is option (b) 678. Select the most appropriate antonym of the given word. Zealot (a) Vicious (b) Nonpartisan (c) Yearn (d) Abuse SSC CPO-SI – 25/11/2020 (Shift-II) Ans. (b): The most appropriate Antonym of the word 'Zealot' is 'Nonpartisan'. Zealot means 'Fanatic'. The meaning of other words is _____ Vicious - Mischievous Yearn - Melancholy. Thus, the correct answer is option (b) 679. Select the most appropriate antonym of the given word. Zeal (a) Forego (b) Paucity (c) Adverse (d) Apathy SSC MTS–14/08/2019 (Shift-III) Ans. (d): The most appropriate Antonym of the word 'Zeal' is 'Apathy'. Zeal means Enthusiasm. The other words are _____ Paucity - Reduction Forego - Abandon Adverse - Hostile Thus, the correct answer is option (d) 680. Select the most appropriate antonym of the given word. Zigzag (a) Curved (b) Volatile (c) Straight (d) Deny SSC CHSL (Tier-I) –04/07/2019 (Shift-I) Ans. (c): The most appropriate Antonym of the word 'Zigzag' is 'Straight'. Zigzag means curved. The meaning of other words is _____ Volatile - Variable, Mutable Deny - Reject Thus, the correct answer is option (c)

Y 673. Select the most appropriate antonym of the word : Yell (a) Shout (b) Whisper (c) Scream (d) Shriek SSC MTS – 22/08/2019 (Shift-III) Ans. (b): The most appropriate Antonym of the word 'Yell' (cry) is. 'Whisper' The meaning of other words is _____ Shout - Yell Scream - Yell Shriek - Yell Thus, the correct answer is option (b) 674. Select the most appropriate antonym of the given word. Yearn (a) Wan (b) Victimize (c) Loathe (d) Veteran SSC CGL (Tier-I) – 06/06/2019 (Shift-I) Ans. (c): The most appropriate Antonym of the word 'Yearn' (wish) is 'Loathe' (dislike). Thus, the correct answer is option (c) 675. Select the most appropriate antonym of the given word. Yield (a) Woes (b) Produce (c) Resist (d) Abuse SSC MTS – 22/08/2019 (Shift-I) Ans. (c): The most appropriate Antonym of the word 'Yield' (agree) is 'Resist' (Protest) while other options are incorrect. Thus, the correct answer is option (c) Antonyms

512

YCT

08. MISPELT WORD TCS hewšve& hej DeeOeeefjle (Based On TCS Pattern) Chapterwise

Exam

Question No.

1

CGL (Tier-1) CGL (Tier-2) CHSL (Tier-1) CHSL (Tier-2) Selection Post VII, VIII, XI SSC MTS SSC GD SSC CPO SI CGL (Tier-1) CGL (Tier-2) CHSL (Tier-1) CHSL (Tier-2) Selection Post VII, VIII, XI SSC MTS SSC GD SSC CPO SI CGL (Tier-1) CGL (Tier-2) CHSL (Tier-1) CHSL (Tier-2) Selection Post VII, VIII, XI SSC MTS SSC GD SSC CPO SI CGL (Tier-1) CGL (Tier-2) CHSL (Tier-1) CHSL (Tier-2) Selection Post VII, VIII, XI SSC MTS SSC GD SSC CPO SI CGL (Tier-1) CGL (Tier-2) CHSL (Tier-1) CHSL (Tier-2) Selection Post VII, VIII, XI SSC MTS SSC GD SSC CPO SI

15 5 17 10 9 11 1 2 3 2 1 1 2 5 1 4 10 8 12 6 4 10 2 5 7 3 17 5 5 5 2 5 7 1 7 5 5 9 1 7

CGL (Tier-1) CGL (Tier-2) CHSL (Tier-1) CHSL (Tier-2)

8 2 7 3

A

2

B

3

C

4 D

5

E

6

Misspelt word

513

Years

(2017–2023)

(2017–2023)

(2017–2023)

(2017–2023)

(2017–2023)

(2017–2023) YCT

F

7

G

8

H

9

I

10

J

11

K

12 L

13

Misspelt word

Selection Post VII, VIII, XI SSC MTS SSC GD SSC CPO SI CGL (Tier-1) CGL (Tier-2) CHSL (Tier-1) CHSL (Tier-2) Selection Post VII, VIII, XI SSC MTS SSC GD SSC CPO SI CGL (Tier-1) CGL (Tier-2) CHSL (Tier-1) CHSL (Tier-2) Selection Post VII, VIII, XI SSC MTS SSC GD SSC CPO SI CGL (Tier-1) CGL (Tier-2) CHSL (Tier-1) CHSL (Tier-2) Selection Post VII, VIII, XI SSC MTS SSC GD SSC CPO SI CGL (Tier-1) CGL (Tier-2) CHSL (Tier-1) CHSL (Tier-2) Selection Post VII, VIII, XI SSC MTS SSC GD SSC CPO SI CGL (Tier-1) CGL (Tier-2) CHSL (Tier-1) CHSL (Tier-2) Selection Post VII, VIII, XI SSC MTS SSC GD SSC CPO SI CGL (Tier-1) CGL (Tier-2) CHSL (Tier-1) CHSL (Tier-2) Selection Post VII, VIII, XI SSC MTS SSC GD SSC CPO SI CGL (Tier-1) CGL (Tier-2) CHSL (Tier-1) 514

4 5 1 2 2 1 3 1 1 1 2 2 1 2 1 1 4 1 4 5 1 7 2 8 5 3 5 1 1 1 1 1 1 1 1 2 1 2 1 1 5 1 3 7 3 5

(2017–2023)

(2017–2023)

(2017–2023)

(2017–2023)

(2017–2023)

(2017–2023)

YCT

M

14

N

15 O

16 P

17 Q

18

R

19

S

Misspelt word

CHSL (Tier-2) Selection Post VII, VIII, XI SSC MTS SSC GD SSC CPO SI CGL (Tier-1) CGL (Tier-2) CHSL (Tier-1) CHSL (Tier-2) Selection Post VII, VIII, XI SSC MTS SSC GD SSC CPO SI CGL (Tier-1) CGL (Tier-2) CHSL (Tier-1) CHSL (Tier-2) Selection Post VII, VIII, XI SSC MTS SSC GD SSC CPO SI CGL (Tier-1) CGL (Tier-2) CHSL (Tier-1) CHSL (Tier-2) Selection Post VII, VIII, XI SSC MTS SSC GD SSC CPO SI

2 3 5 1 5 1 1 2 1 2 4 3 2 3 2 5 3 1 2 4 2 12 8 5 5 2 5

CGL (Tier-1) CGL (Tier-2) CHSL (Tier-1) CHSL (Tier-2) Selection Post VII, VIII, XI SSC MTS SSC GD SSC CPO SI CGL (Tier-1) CGL (Tier-2) CHSL (Tier-1) CHSL (Tier-2) Selection Post VII, VIII, XI SSC MTS SSC GD SSC CPO SI CGL (Tier-1) CGL (Tier-2) CHSL (Tier-1) CHSL (Tier-2) Selection Post VII, VIII, XI SSC MTS SSC GD SSC CPO SI

1 1 1 1 1 1 8 2 6 5 7 2 2 5 6 8 11 5 1 4 2 4

515

(2017–2023)

(2017–2023)

(2017–2023)

(2017–2023)

(2017–2023)

(2017–2023)

(2017–2023)

YCT

20

T

21

U

22

V

23

W

24

X

25

Y

26

Z

Misspelt word

CGL (Tier-1) CGL (Tier-2) CHSL (Tier-1) CHSL (Tier-2) Selection Post VII, VIII, XI SSC MTS SSC GD SSC CPO SI CGL (Tier-1) CGL (Tier-2) CHSL (Tier-1) CHSL (Tier-2) Selection Post VII, VIII, XI SSC MTS SSC GD SSC CPO SI CGL (Tier-1) CGL (Tier-2) CHSL (Tier-1) CHSL (Tier-2) Selection Post VII, VIII, XI SSC MTS SSC GD SSC CPO SI CGL (Tier-1) CGL (Tier-2) CHSL (Tier-1) CHSL (Tier-2) Selection Post VII, VIII, XI SSC MTS SSC GD SSC CPO SI CGL (Tier-1) CGL (Tier-2) CHSL (Tier-1) CHSL (Tier-2) Selection Post VII, VIII, XI SSC MTS SSC GD SSC CPO SI CGL (Tier-1) CGL (Tier-2) CHSL (Tier-1) CHSL (Tier-2) Selection Post VII, VIII, XI SSC MTS SSC GD SSC CPO SI CGL (Tier-1) CGL (Tier-2) CHSL (Tier-1) CHSL (Tier-2) Selection Post VII, VIII, XI SSC MTS SSC GD SSC CPO SI 516

5 3 3 2 2 2 1 1 1 1 2 1 1 1 1 1 1 1 1 1 1 2 1 1 1 1 1 1 2 1 1 1 1 1 1 1 1 1 1 1 1

(2017–2023)

(2017–2023)

(2017–2023)

(2017–2023)

(2017–2023)

(2017–2023)

(2017–2023)

YCT

Trend Analysis of Questions topicwise from CGL (Pre & Mains) CHSL (Pre & Mains) Selection Post VII, VIII, XI, SSC MTS, SSC GD & Other Exams (2017-2023)

Misspelt word

517

YCT

08. MISSPELT WORD 6.

A 1.

Select the correctly spelt word. (a) Aposire (b) Apositte (c) Apposite (d) Aposite SSC MTS 19/05/2023 (Shift-III) Ans. (c) : The above given options are incorrectly spelt except option (c) ‘Apposite’ which means – ‘Suitable and right for the occasion’. Other options are incorrectly spelt. 2. Select the wrongly spelt word. (a) Unconventional (b) Alturistic (c) Ethical (d) Unsavoury SSC MTS 12/05/2023 (Shift-III) Ans. (b) : All the above given options are correct except option (b) 'Alturistic'. The correct word is 'Altruistic; Meaning of other options– Unconventional – stable, Ethical – Moral, Unsavoury – Test less, 3. The sentence given below has one misspell word. Spot the INCORRECTLY spelt word and select its correct spelling. Abreviations are not understood by everyone. (a) Abreviashions (b) Abbriviations (c) Abbreviations (d) Abriviations SSC GD 16/01/2023 (Shift-III) Ans. (c) : The spelling of 'abreviation' in the above sentence is incorrect. Its correct spelling would be 'abbreviation' (summary). Correct SentenceAbbreviation are not understood by everyone. 4. Select the correct spelling of the underlined word in the following sentence. The architectre of the building is worth seeing. (a) Arckitecture (b) Architecture (c) Architecher (d) Architecter SSC GD 08/02/2023 (Shift-IV) Ans. (b) : In the underlined part of the above sentence, 'Architecture' will be correct spelling. Hence, option (b) is correct answer. 5. Select the INCORRECTLY spelt word. (a) Allmost (b) Treble (c) Nozzle (d) Moisture SSC CGL (Tier-I) 21/07/2023 (Shift-II) Ans. (a) : The above given option are correct except option (a) 'Allmost', The correctly spelt word is'Almost' which means 'approximate'. Hence, the correct option is (A). Misspelt word

Select the INCORRECTLY spelt word. (a) Aristocrate (b) Preference (c) Occurrence (d) Advertise SSC CGL (Tier-I) 18/07/2023 (Shift-III) Ans. (a) : The incorrectly spelt word of the above given options is (a) 'Aristocrate'. The correctly spelt is 'Aristorcrat' which means 'a person of high social rank, often with a special title. Other options are correctly spelt. 7. Select the INCORRECTLY splet word. (a) Dearth (b) Corrupt (c) Barely (d) Accepttible SSC CGL (Tier-I) 27/07/2023 (Shift-III) Ans. (d) : All the given options are correct except option (d) 'Acceptible'. The correctly spelt word is 'Acceptable'. Meaning of other optionsDearth - Wane, Corrupt - Impaired, Barely - Hardly. 8. Select the correct spelling of the underline word. They denied having any associasion with the terrorists. (a) association (b) assocciation (c) assosiation (d) asociation SSC CGL (Tier-I) 14/07/2023 (Shift-I) Ans. (a) : The correct spelling of the underlined word is ‘association’. Hence option (a) will be correct answer. 9. Select the INCORRECTLY spelt word. (a) Architecture (b) Ambassador (c) Awful (d) Accomodate SSC CHSL (Tier-I) 09/08/2023 (Shift-III) Ans. (d) : Option (d) 'Accomodate' has been incorrectly spelt. Its correct spelling will be 'Accommodate' which means of have enough space for somebody/ to Provide with something desired/ to make fit adjust. Other options have following meaning– Architecture – The art and technique of designing and building Ambassador – An important person who represents his country in a foreign country Awful – very bad/ Extremely objectionable very great or large. 10. Identify the INCORRECTLY spelt word in the following sentence. Staying healthy and exercising daily enhances an athelet’s ability. (a) athelet’s (b) healthy (c) exercising (d) enhances SSC CHSL (Tier-I) 04/08/2023 (Shift-III)

518

YCT

Ans. (a) : All the above given options are correctly spelt except option (a) 'athelet's'. The correctly spelt word is 'athlete's '. Thus, the correct answer is option (a). 11. Select the wrongly spelt word. (a) Aesthetic (b) Affectation (c) Abase (d) Arei SSC CHSL (Tier-I) 15/03/2023 (Shift-III) Ans. (d) : In the above options, option (d) ''Arei'' is wrongly spelt its correct spelling is ''Airy'' or 'Area'. Other options are correct. 12. Select the wrongly spelt word. (a) Articulate (b) Arduous (c) Arrogate (d) Artefect SSC CHSL (Tier-I) 15/03/2023 (Shift-III) Ans. (d) : In the above options, option (d) ''Artefect'' is wrongly spelt, its correct spelling is ''Artifact'', other options are correct. 13. Choose the incorrectly spelt word. (a) Cramp (b) Damp (c) Agravate (d) Stamp SSC CHSL (Tier-I) 09/03/2023 (Shift-IV) Ans. (c) : For the above options, option (c) 'Agravate' is misspelled its correct spelling is "Aggravate" other options are correctly spelt. 14. Select the INCORRECTLY spelt word. (a) Aggreement (b) Aggression (c) Agitate (d) Aggravate SSC Constable GD-14/12/2021 Shift-II Ans. (a) : The word 'Aggreement' is incorrect because the correct spelling is 'agreement' which means 'contract'. The other words areAggression - Attack Agitate - Incite Aggravate - Inflame 15. Select the INCORRECTLY spelt word. (a) Abbrevations (b) Fluttering (c) Menace (d) Notoriety SSC Constable GD-14/12/2021 Shift-III Ans. (a) : The word 'Abbrevations' is incorrect because the correct spelling is 'Abbreviations' which means 'Summarization'. The other words are Fluttering - Flitter Notoriety - Disrepute / Infamy Thus, the correct answer is options (a). 16. Select the INCORRECTLY spelt word. (a) Arrogent (b) Appeal (c) Immediate (d) Attention SSC Constable GD-13/12/2021 Shift-III Ans. (a) : The word 'Arrogent' is incorrect because the correct spelling is 'Arrogant' which means 'obstinate'/Proud. Thus, the correct answer is options (a). 17. Select the INCORRECTLY spelt word. (a) Accuse (b) Progress (c) Acclame (d) Vicious SSC Constable GD-25/11/2021 Shift-III Misspelt word

Ans. (c) : The word ' Acclame' is incorrect because the correct spelling is 'Acclaim' which means praise'. The other words are Accuse - Denounce Progress - Advance vicious - mischievous Thus, the correct answer is options (c). 18. Select the correctly spelt word. (a) Absolutly (b) Adaptation (c) Abandan (d) Acurate SSC Constable GD-29/11/2021 Shift-II Ans. (b) : Out of the given options only 'Adaptation' is spelt correct. It means 'Adjustment'. While other options are incorrect. Thus, the correct answer is options (b). 19. Select the INCORRECTLY spelt word. (a) IIIusion (b) Brilliant (c) Scrutiny (d) Abandan SSC Constable GD-30/11/2021 Shift-III Ans. (d) : The word 'Abandan' is incorrect because the correct spelling is 'Abandon' which means 'Giveup'. The other words are Illusion - Delusion Brilliant - Bright Scrutiny - Review, Dissection. This, the correct answer is option (d). 20. Select the correctly spelt word. (a) Assembly (b) Asembly (c) Asemmbly (d) Assemmbly SSC MTS-06/07/2022 Shift-III Ans. (a) : Out of the given options, only 'Assembly' is spelt correctly, which means 'congregation'. Thus, the correct answer is options (a). 21. Select the INCORRECTLY spelt word. (a) Aisle (b) Altar (c) Attick (d) Adorn SSC CGL (Tier-I) 13/04/2022 Shift-I Ans. (c) : The word 'Attick' is incorrect because the correct spelling is 'Attic' means a room just below the roof. The other words are Aisle - street Altar - chancel Adorn - decoration Thus, the correct is answer is option (c). 22. Select the INCORRECTLY spelt word. (a) Announcement (b) Anonymous (c) Annivarsary (d) Anguish SSC CGL (Tier-I) 19/04/2022 Shift-III Ans. (c) : The word 'Annivarsary' is incorrect because the correct spelling is 'Anniversary' which means 'commemoration'. The other words are Announcement means 'Declaration'. Anonymous 'Nameless' Anguish 'suffering'. Thus, the correct answer is options (c).

519

YCT

23.

Select the incorrectly spelt word. (a) Extrovert (b) Appology (c) Lugubrious (d) Collaborate SSC CHSL 26.05.2022 Shift-I Ans. (b) : The word 'Appology' is incorrect because the correct spelling is 'Apology' which means 'excuse'. The other words are Extravert - extrorse Lugubrious - mournful Collaborate - Cooperation Thus, the correct answer is options (b). 24. Select the incorrectly spelt word. (a) Secretary (b) Performance (c) Valuable (d) Arguement SSC CHSL 24.05.2022 Shift-III Ans. (d) : The word 'Arguement' is incorrect because the correct spelling is 'Argument' which means 'Debate'. The other words are Performance - Execution/Valuable - psecious / Thus, the correct answer is options (d). 25. Select the INCORRECTLY spelt word. (a) Changeable (b) Acidentaly (c) Tomorrow (d) Possession SSC CHSL (Tier-I) –16/04/2021 (Shift-I) Ans. (b) : The above given option (b) 'Acidentaly' is incorrectly spelt word. The correctly spelt word is 'Accidentally' which means 'without advance planning'. The other correct options arePossession- Rights, Tomorrow - The day after today, Changeable - Mutable Thus, the correct answer is option (b). 26. Select the misspelt word. (a) applause (b) modern (c) arrivel (d) serious SSC CHSL (Tier-I) –06/08/2021 (Shift-I) Ans. (c) : The above given option (c) 'Arrivel' is misspelt word. The correctly spelt word is 'Arrival'. It means - 'access'/Advent. The other given options are correctApplause - Extol, Modern - Recent, Serious - Sedate Thus, the correct answer is option (c). 27. Select the incorrectly spelt word. (a) assimilate (b) readable (c) acquire (d) acknowlege SSC CHSL (Tier-I) –09/08/2021 (Shift-I) Ans. (d) : The incorrectly spelt word is option (d) 'Acknowlege'. The correctly spelt word is 'Acknowledge' which means- 'to accept or admit that something is true. The other correct options areAssimilate - Comprehend, Readable – Legible, Acquire - Obtain. Thus, the correct answer is option (d). 28. Select the incorrectly spelt word. (a) Eligibility (b) Acheivmant (c) Relevant (d) Adamant SSC CHSL (Tier-I) –19/04/2021 (Shift-I) Ans. (b) : The above given option (b) 'Acheivmant' is incorrectly spelt word. The correctly spelt word is 'Achievement' which means- 'Something that you have done successfully through hard work'. Misspelt word

The other correct options areEligibility - Competency, Relevant – Consistent, Adamant - Rigorous hard Thus, the correct answer is option (b). 29. Select the incorrectly spelt word. (a) Bouquet (b) Abridge (c) Pledge (d) Acquarium SSC CGL (Tier-I) –17/08/2021 (Shift-I) Ans. (d) : The above given option (d) 'Acquarium' is incorrectly spelt word. The correctly spelt word is 'Aquarium' which means- 'a glass container filled with water, in which fish and water animals are kept'. The other correct options areBouquet - Nsegay / Pledge - Resolution, Abridge - Expurgation Thus, the correct answer is option (d). 30. Select the wrongly spelt word. (a) pupil (b) capacity (c) teacher (d) abilty SSC CPO-SI – 23/11/2020 (Shift-I) Ans. (d) : The incorrectly spelt word is option (d) 'Ablty'. The correctly spelt word is 'Ability' which means - 'the mental and physical power that makes it possible to do something.' The other correct options arePupil - Student, Capacity - Efficiency, Teacher - Instructor Thus, the correct answer is option (d). 31. Select the misspelt word. (a) adjourn (b) afiliate (c) etiquette (d) inimical SSC CPO-SI – 24/11/2020 (Shift-II) Ans. (b) : The above given option (b) 'Afiliate' is misspelt word. The correctly spelt word is 'Affiliate'. It means - 'modulate'. The other correct options areAdjourn - Postpone / Etiquette - Decorum, Inimical - Antagonistic, Thus, the correct answer is option (b). 32. Select the correctly spelt word. (a) advertizment (b) adversery (c) advantegious (d) adventurous SSC CGL (Tier-I) – 19/06/2019 (Shift-III) Ans. (d) : The correctly spelt word from the given options is (d) 'Adventurous' which means 'Bold/Daring'. The other correct options areAdvertizement - Public announcement, Adversary - Dissident / Advantageous - Remunerative Thus, the correct answer is option (d). 33. Select the correctly spelt word. (a) accomplish (b) hieghten (c) repitition (d) aggrravate SSC CGL (Tier-I) – 12/06/2019 (Shift-III) Ans. (a) : The above given options are incorrectly spelt except option (a) 'Accomplish' which means 'Associative'. The other correct options areHeighten- Amplify Repetition- Revision Aggravate- Provoke Thus, the correct answer is option (a).

520

YCT

34.

Select the correctly spelt word. (a) Arguement (b) Argument (c) Argeument (d) Arguemant SSC CGL (Tier-I) – 11/06/2019 (Shift-III) Ans. (b) : The above given options are incorrectly spelt word except option (b) 'Argument' which means'Contention'. Thus, the correct answer is option (b). 35. Select the correctly spelt word. (a) Adolescence (b) Adolescance (c) Adolescense (d) Adolesense SSC CGL (Tier-I) – 10/06/2019 (Shift-I) Ans. (a): The correctly spelt word is option (a) 'Adolescence' which means - 'Maiden hood'. The other given options are incorrectly speltThus, the correct answer is option (a). 36. Select the correct spelt word. (a) acommodation (b) accomodation (c) accommodation (d) acomodation SSC CGL (Tier-I) – 10/06/2019 (Shift-III) Ans. (c) : The correctly spelt word from the given options is (c) 'Accommodation' which means 'Residence'. The other given options are incorrectly speltThus, the correct answer is option (c). 37. Select the correctly spelt word. (a) attendence (b) abundance (c) arrogence (d) apearance SSC CGL (Tier-I) – 07/06/2019 (Shift-III) Ans. (b): The correctly spelt word from the given options is (b) 'Abundance' which means - 'Sufficiency'. The other correct options areAttendance - Impendence Arrogance - Atrocity Appearance - Coutenance Thus, the correct answer is option (b). 38. Select the wrongly spelt word. (a) Article (b) Precious (c) Audibal (d) Conceal SSC CPO-SI – 11/12/2019 (Shift-II) Ans. (c): The incorrectly spelt word from the given options is (c) 'Audibal'. The correctly spelt word is 'Audible' which means- 'capable of being heard'. The other correct options areArticle - Section, Precious - Valuable, Conceal - Stash. Thus, the correct answer is option (c). 39. Select the wrongly spelt word. (a) Evasion (b) Aviation (c) Omission (d) Avertion SSC CPO-SI – 11/12/2019 (Shift-II) Ans. (d): The above given options are correctly spelt word except option (d) 'Avertion'. The correctly spelt word is 'Aversion' which means Dislike'. The other correct options areEvasion - Pretense Aviation - Aerostation Omission - Concealment Thus, the correct answer is option (d). 40. Select the INCORRECTLY spelt word. (a) Audible (b) Audiance (c) Austere (d) Authority SSC CPO-SI – 11/12/2019 (Shift-I) Misspelt word

Ans. (b): The incorrectly spelt word from the given options is (b) 'Audiance'. The correctly spelt word is 'Audience' which means - 'spectator'. The other correct options areAudible - Capable of being heard, Austere - Unblamable, Authority - Citation Thus, the correct answer is option (b). 41. Select the INCORRECTLY spelt word. (a) amateur (b) audience (c) acheivement (d) audacity SSC CPO-SI – 12/12/2019 (Shift-II) Ans. (c): The above given option (c) 'Acheivement' is incorrectly spelt word. The correctly spelt word is 'Achievement', which means- 'Something that has been accomplished by hard work'. The other correct options areAmateur - Doing something for pleasure Audience - Spectator Audacity - Insolence/Courage Thus, the correct answer is option (c). 42. Select the INCORRECTLY spelt word. (a) argyu (b) tongue (c) vague (d) vogue SSC CPO-SI – 12/12/2019 (Shift-II) Ans. (a): The above given option (a) 'Argyu' is incorrectly spelt word. The correctly spelt word is 'Argue' which means - 'Infer/debate'. The other correct options areVague - Obscure Vogue - Prevalence Tongue - Speech Thus, the correct answer is option (a). 43. Select the INCORRECTLY spelt word. (a) incremental (b) ferocious (c) alignement (d) amenable SSC CPO-SI – 13/12/2019 (Shift-II) Ans. (c) : The above given option (c) 'Alignement' is incorrectly spelt word. The correctly spelt word is 'Alignment' which means - 'Arrangement in a straight line'. The other correct options areIncremental - Gradual Ferocious - Brutal Amenable - Responsible Thus, the correct answer is option (c). 44. Select the correctly spelt word. (a) Acquasition (b) Acquision (c) Acquesition (d) Acquisition SSC CHSL (Tier-I) –11/07/2019 (Shift-III) Ans. (d): The correctly spelt word from the given options is (d) 'Acquisition' which means - 'The act of obtaining or buying something'. The other given options are incorrectly speltThus, the correct answer is option (d). 45. Select the correctly spelt word. (a) achievement (b) acknoledgement (c) accreditted (d) accoustomed SSC CHSL (Tier-I) –09/07/2019 (Shift-I)

521

YCT

Ans. (a): The correctly spelt word from the given options is (a) 'achievement' which means 'Accomplishment'. The other correct options areAcknowledgement - Receipt, Accredited - Authorized, Accustomed - Habituated Thus, the correct answer is option (a). 46. Select the wrongly spelt word. (a) acheive (b) receive (c) believe (d) grieve SSC CHSL (Tier-I) –08/07/2019 (Shift-II) Ans. (a): The above given options are correctly spelt except option (a) 'Achieve'. The correctly spelt word is 'Achieve' which means - 'obtain'. The other correct options areReceive - Enlist Believe - Confide Grieve - Molest Thus, the correct answer is option (a). 47. Select the wrongly spelt word. (a) accumulate (b) accommodate (c) accelarate (d) accomplice SSC CHSL (Tier-I) –05/07/2019 (Shift-III) Ans. (c): The above given options are correctly spelt except option (c) 'Accelarate'. The correct word is 'Accelerate' which means - 'speed up'. The other correct options areAccumulate - Collect Accommodate - Adapt, Accomplice - Associative Thus, the correct answer is option (c). 48. Select the wrongly spelt word. (a) attendance (b) alliance (c) affilate (d) affluence SSC CHSL (Tier-I) –04/07/2019 (Shift-II) Ans. (c): The above given option (c) 'Affilate' is wrongly spelt word. The correctly spelt word is 'Affiliate' which means - connect. The other correct options areAttendance - Impendence, Alliance - Junction Affluence - Prosperity Thus, the correct answer is option (c). 49. Select the wrongly spelt word. (a) aggrassive (b) agriculture (c) agitation (d) agreement SSC CHSL (Tier-I) –03/07/2019 (Shift-II) Ans. (a) : The wrongly spelt word from the given options is (a) 'Aggrassive'. The correctly spelt word is 'Aggressive' which means - 'Rampant'. The other correct options areAgriculture - Husbandry Agitation - Stimulus, Agreement - Annexure Thus, the correct answer is option (a). 50. Select the wrongly spelt word. (a) assurence (b) assurable (c) assure (d) assuredly SSC CHSL (Tier-I) –02/07/2019 (Shift-I) Ans. (a) : The wrongly spelt word from the given options is (a) 'Assurence'. The correctly spelt word is 'Assurance' which means - Insurance/Warranties. Misspelt word

The other correct options areAssurable - Insurable, Assure - Insure, Assuredly - Confidently. Thus, the correct answer is option (a). 51. Select the wrongly spelt word. (a) Advisible (b) Invisible (c) Capable (d) Irritable SSC CHSL (Tier-I) 26/10/2020 (Shift-II) Ans. (a) : The wrongly spelt word from the given options is (a) 'Advisible'. The correctly spelt word is 'Advisable' which means - 'Suitable/Reasonable'. The other correct options areInvisible - Occult Capable - Competent Irritable - Choleric Thus, the correct answer is option (a). 52. Select the incorrectly spelt word. (a) Distance (b) Arguement (c) Commitment (d) Existence SSC CHSL (Tier-I) 16/10/2020 (Shift-II) Ans. (b) : The incorrectly spelt word from the given options is (b) 'Arguement'. The correctly spelt word is 'Argument' which means- 'Debate/Contention'. The other correct options areDistance - Mileage Commitment - Engagement, Existence - Entity Thus, the correct answer is option (b). 53. Select the INCORRECTLY spelt word. (a) Acceptable (b) Beginning (c) Accomodate (d) Quizzes SSC CHSL (Tier-I) 13/10/2020 (Shift-I) Ans. (c) : The above given option (c) 'Accomodate' is incorrectly spelt word. The correctly spelt word is 'Accommodate' which means - 'to have enough space for somebody or something'. The other correct options areAcceptable - Admissible Beginning - Start up, Quizzes - Satirize Thus, the correct answer is option (c). 54. Select the wrongly spelt word. (a) Appreciate (b) Judiciary (c) Virtual (d) Ambasador SSC CHSL (Tier-I) 12/10/2020 (Shift-II) Ans. (d) : The above given option (d) 'Ambasador' is incorrectly spelt word. The correctly spelt word is 'Ambassador' which means - 'Diplomat'. The other correct options areAppreciate - Panegyrize Judiciary - Magisterial Virtual - Genuine Thus, the correct answer is option (d). 55. Select the INCORRECTLY spelt word. (a) Ambiteous (b) Gorgeous (c) Serious (d) Curious SSC CHSL (Tier-I) 20/10/2020 (Shift-I) Ans. (a) : The incorrectly spelt word from the given option is (a) 'Ambiteous'. The correctly spelt word is 'Ambitious' which means 'Desirous'. The other correct options areGorgeous - Splendid Serious - Solemn, Curious - Impatient Thus, the correct answer is option (a).

522

YCT

56.

Select the correctly spelt word. (a) Exquiste (b) Recquirement (c) Acquaintance (d) Squirrell SSC CHSL (Tier-I) 20/10/2020 (Shift-II) Ans. (c) : The above given options are incorrect except option (c) 'Acquaintance' which means - 'A person that you know but who is not a close friend'. The other correct options areExquisite - Best Requirement - Necessity Squirrel - A small animal covered in fur with a long tail. Thus, the correct answer is option (c). 57. Select the wrongly spelt word. (a) Tolerate (b) Temperate (c) Accomodate (d) Promulgate SSC CHSL (Tier-I) 19/03/2020 (Shift-II) Ans. (c) : The incorrectly spelt word from the given options is (c) 'Accomodate'. The correctly spelt word is 'Accommodate' which means- 'to have enough space for somebody or something'. The other correct options areTolerate - Forebear Temperate - Abstemious Promulgate - Announce Thus, the correct answer is option (c). 58. In the following question, a word has been written in four different ways out of which only one is correctly spelt. Select the correctly spelt word. (a) Accedent (b) Akcident (c) Accident (d) Acciant SSC MTS 11-10-2017 (Shift-II) Ans. (c) : All the above given options are incorrect except option (c) 'Accident' which means - on unpleasant event that happens unexpectedly and causes damage, injury or death'. Thus, the correct answer is option (c). 59. In the following question, a word has been written in four different ways out of which only one is correctly spelt. Select the correctly spelt word. (a) Acomplish (b) Accomplish (c) Akcomplish (d) Accomplesh SSC MTS 11-10-2017 (Shift-II) Ans. (b) : All the above given options are incorrect except (b) 'Accomplish' which means - 'a person who helps somebody to do something bad, especially a crime'. Hence, the correct answer is option (b). 60. Select the correctly spelt word. (a) Anniversery (b) Anniversary (c) Anniversiry (d) Aniversary SSC MTS – 09/08/2019 (Shift-I) Ans. (b) : All the above given options are incorrect except option (b) 'Anniversary' which means- 'a date that is remembered or celebrated because a special or notable event occured.' Thus, the correct answer is option (b). 61. Select the correctly spelt word. (a) Adrress (b) Address (c) Addres (d) Adress SSC MTS – 20/08/2019 (Shift-II) Misspelt word

Ans. (b) : All the above given options are incorrect except option (b) 'Address' which means - 'a description of the location of a person or organization'. Thus, the correct answer is option (b). 62. Select the correctly spelt word. (a) Agression (b) Appearence (c) Assasination (d) Assimilate SSC MTS – 16/08/2019 (Shift-II) Ans. (d) : The correctly spelt word from the given options is (d) 'Assimilate' which means- 'adopt/digest'. The other correct options areAggression - Invasion, Appearance - Plausibility Assassination - Murder Thus, the correct answer is option (d). 63. Select the correctly spelt word. (a) Asthetic (b) Already (c) Outragous (d) Orignal SSC MTS – 14/08/2019 (Shift-III) Ans. (b) : The correctly spelt word from the given options is (b) 'Already' which means- 'to emphasis that something happened before something else or earlier than expected' (Previously). The other correct options areAesthetic– Concerned with beauty or art Outrageous– Pejorative Original– Genuine Thus, the correct answer is option (b). 64. Select the correctly spelt word. (a) Administrater (b) Administrator (c) Administratur (d) Administretor SSC MTS – 13/08/2019 (Shift-III) Ans. (b) : All the above given options are incorrect except option (b) 'Administrator' which means- 'a person whose job is to organize or manage a system, a business.' Thus, the correct answer is option (b). 65. Select the wrongly spelt word. (a) Quote (b) Require (c) Queue (d) Accquire SSC MTS – 08/08/2019 (Shift-II) Ans. (d) : All the above given options are correct except option (d) 'Accquire'. The correctly spelt word is 'Acquire' which means- 'to obtain or buy something. The other correct options areQuote - Citation Require - Desire Queue - Line Thus, the correct answer is option (d). 66. Choose the correctly spelt word. (a) Disccusion (b) Suspition (c) Adjourn (d) Procedings SSC MTS – 07/08/2019 (Shift-II) Ans. (c) : The correctly spelt word from the given options is (c) 'Adjourn' which means- 'to put off or suspend until a future time'. The other correct options areDiscussion - Parlance/Debate Suspicion - Doubt Proceedings- Prmopt action Thus, the correct answer is option (c).

523

YCT

67.

Select the correctly spelt word. (a) Acommodate (b) Accmmadate (c) Accomodete (d) Accommodate SSC GD – 18/02/2019 (Shift-III) Ans. (d) : All the above given options are incorrect except option (d) 'Accommodate' which means- 'to have enough space for somebody or something'. Thus, the correct answer is option (d). 68. Select the correctly spelt word. (a) Acheive (b) Achieve (c) Acheeve (d) Achiev SSC GD – 14/02/2019 (Shift-I) Ans. (b) : All the above given options are incorrect except option (b) 'Achieve' which means- 'obtain'. Thus, the correct answer is option (b).

B 69.

Select the INCORRECTLY spelt word. (a) Drifter (b) Haughty (c) Hinterland (d) Bellweather SSC CGL (Tier-I) 26/07/2023 (Shift-II) Ans. (d) : For the above given words, the incorrectly spelt word is 'Bellweather' because the correctly spelt word is 'Bellwether'. Thus, the correct answer is option (a). 70. Identify the sentence with correct spellings. (a) Rahul was ashamed of beehaving so badly. (b) Rahul was ashamed of behaving so badly. (c) Rahul was ashamed of bihaving so badly. (d) Rahul was ashamed of biheving so badly. SSC CGL (Tier-I) 20/07/2023 (Shift-I) Ans. (b) : For the above sentence, the most appropriate option (b) ‘Rahul was ashamed of behaving so badly’. Here the spelling of ‘behaving’ is correct while other options are in correct. Thus, the correct answer is option (b). 71. Choose the incorrectly spelt word. (a) Morning (b) Washing (c) Calling (d) Burnig SSC CHSL (Tier-I) 21/03/2023 (Shift-IV) Ans. (d) : For the above options, option (d) 'burnig' is wrongly spelt its correct spelling is 'burning'. Other options are correct. 72.

Select the INCORRECTLY spelt word. (a) black (b) blade (c) bleame (d) blare SSC Constable GD-10/12/2021 Shift-III Ans. (c) : The word 'Bleame' is incorrect because the correct spelling is 'Blame' which means 'stricture'. Thus, the correct answer is options (c).

73.

Select the correctly spelt word. (a) Bureaucracy (b) Scavange (c) Warrier (d) Inheritence SSC Constable GD-18/11/2021 Shift-I Ans. (a) : Out of the given options, only 'Burecucracy' is spelt correctly.

Misspelt word

The correct words areScavenge - scrub Warrior - Knight Inheritance - patrimony Thus, the correct answer is options (a). 74. Select the sentence in which a word is INCORRECTLY spelt. (a) He hoped that he won the prestigious award. (b) I could never be as talented as him. (c) His breathe smelled of garlic. (d) A woman gasped in horror at the sight of him. SSC MTS-08/07/2022 Shift-I Ans. (c) : A word 'Breathe' is incorrectly spelt because the correct spelling is 'breath' it is used as a noun while 'breathe' used as a verb and after the possessive adjective (His) only 'noun' is used. Thus, the correct sentence is His breath smelled of garlic. 75. Select the INCORRECTLY spelt word. (a) Swimming (b) Borowing (c) Sovereign (d) Preserve SSC MTS-06/07/2022 Shift-II Ans. (b) : The word 'Borowing' is incorrect because the correct spelling is 'Borrowing'. While the remaining options are correct. Thus, the correct answer is option (b) 76. Select the INCORRECTLY spelt word. (a) Mathematician (b) Musician (c) Technician (d) Beautycian SSC CHSL 25.05.2022 Shift-I Ans. (d) : The word 'Beautycian' is incorrect because the correct spelling is 'Beautician' which means makeup artist. The other words are Mathematician - A person skilled in Mathematics Musician - A person who plays a musical instruments Thus, the correct answer is option (d). 77. Select the wrongly spelt word. (a) Geometry (b) Scenery (c) Primary (d) Boundery SSC Sel. Post Phase-VIII (M.L.) 09.11.2020 (Shift-III)

Ans. (d) : The wrongly spelt word from the given options is (d) 'Boundery' and the correctly spelt word is 'boundary' which means - 'Range'. The other options are correctly speltGeometry - The study in mathematics of lines, shapes, curves Scenery - Landscape Primary - Rudimentary Thus, the correct answer is option (d). 78. Select the INCORRECTLY spelt word. (a) Buying (b) Bouncing (c) Boxing (d) Begining SSC Constable GD-09/12/2021 Shift-II Ans. (d) : The word 'Begining' is incorrect because the correct spelling is 'Beginning' which means 'starting'. The other words are Bouncing - Boast Boxing - Pugilism Thus, the correct answer is options (d).

524

YCT

79.

Select the misspelt word. (a) beguile (b) bemoan (c) bereave (d) beseige SSC CPO-SI – 24/11/2020 (Shift-I) Ans. (d) : All the above given options are correct except option (d) 'Beseige'. The correctly spelt word is 'Besiege' which means- 'to surround a place with an army'. The other options are correctly speltBeguile - Charm Bemoan - Weep Bereave - Deprive Thus, the correct answer is option (d). 80. Select the correctly spelt word. (a) bouckey (b) bookay (c) bowquet (d) bouquet SSC CPO-SI – 24/11/2020 (Shift-II) Ans. (d) : All the above given options are incorrect expect option (d) 'Bouquet' which means'a bunch of flowers that is arranged in an attractive way'. Thus, the correct option is (d). 81. Select the wrongly spelt word. (a) bridle (b) brisk (c) bristel (d) brittle SSC CGL (Tier-I) – 07/06/2019 (Shift-II) Ans. (c): The wrongly spelt word from the given options is (c) 'Bristel'. The correctly spelt word is 'Bristle' which means- 'short stiff hair of an animal or plant'. The other given options are correctly speltBridle - Rein, Brisk - Cunning, Brittle - Fragile Thus, the correct answer is option (c). 82. Select the wrongly spelt word. (a) benethe (b) behold (c) bench (d) begin SSC CPO-SI – 09/12/2019 (Shift-II) Ans. (a): The incorrectly spelt word is option (a) 'Benethe'. The correctly spelt word is 'Beneath' which means - 'below or under'. The other options are correctly speltBehold - Peruse, Bench - Law Court, Begin-Initiate Thus, the correct answer is option (a). 83. Select the wrongly spelt word. (a) Burglery (b) Treasury (c) Aviary (d) Nursery SSC CPO-SI – 11/12/2019 (Shift-II) Ans. (a): The wrongly spelt word from the given options is (a) 'Burglery'. The correctly spelt word is 'Burglary' which means- 'The crime of entering a building illegally in order to steal'. The other options are correctly speltTreasury - Coffers Aviary - 'A large cage or area, in which birds are kept' Nursery - Kindergarten/Brooder Thus, the correct answer is option (a). 84. Select the correctly spelt word. (a) Benigne (b) Benine (c) Benign (d) Bennine SSC CPO-SI – 13/12/2019 (Shift-I) Misspelt word

Ans. (c) : The correctly spelt word from the given options is (c) 'Benign' which means- 'kind or gentle' and the other given options are incorrect. Thus, the correct option is (c). 85. Select the wrongly spelt word. (a) fomentation (b) bureaucrecy (c) plateau (d) horizontal SSC CHSL (Tier-I) –10/07/2019 (Shift-II) Ans. (b): All the above given options are correct except option (b) 'Bureaucrecy'. The correctly spelt word is 'Bureaucracy' which means- 'the system of official rules that an organization has for doing something'. The other correct options areFomentation- Passion Plateau- Tableland Horizontal- Vertical Thus, the correct answer is option (b). 86. Select the wrongly spelt word. (a) Bliss (b) Behaive (c) board (d) better SSC CHSL (Tier-I) –03/07/2019 (Shift-I) Ans. (b) The incorrectly spelt word from the given options is (b) 'Behaive'. The correctly spelt word is 'Behave' which means - 'Act in a particular way'. The other options are correctly speltBliss - Pleasance Board- Committee Better- Superior Thus, the correct answer is option (b). 87. Select the wrongly spelt word. (a) berry (b) barren (c) barel (d) barrier SSC CHSL (Tier-I) –02/07/2019 (Shift-I) Ans. (c) : The incorrectly spelt word from the given options is (c) 'Barel'. The correctly spelt word is Barrel' which means- 'a large, round, wooden, plastic or metal container for liquids.' The other options are correctly speltBerry- Jujube Barren- Infertile Barrier- Interrupt Thus, the correct answer is option (c). 88. Select the wrongly spelt word. (a) booty (b) brood (c) blosom (d) boast SSC CHSL (Tier-I) –02/07/2019 (Shift-III) Ans. (c): All the above given options are correct except option (c) 'Blosom'. The correctly spelt word is 'Blossom' which means- 'a flower or bloom, especially of a fruit-bearing plant.' The other options are correctly speltBooty - Plunder, Brood - Progeny, Boast - Ostentation Thus, the correct option is (c). 89. Select the INCORRECTLY spelt word. (a) Balloon (b) Adulteration (c) Irritant (d) Bungalow SSC CHSL (Tier-I) 16/10/2020 (Shift-II) Ans. (d) : The incorrectly spelt word from the give options is (d) 'Bungalow'. The correctly spelt word is 'Bungalow' which means– 'a house that is all on one level, without stairs'.

525

YCT

The other options are correctly speltBalloon - Aircraft, Adulteration - Tincture, Irritant - Thrilling Thus, the correct answer is option (d). 90. Select the correctly spelt word. (a) Bouquet (b) Secquence (c) Acquirum (d) Frequensy SSC CHSL (Tier-I) 15/10/2020 (Shift-I) Ans. (a) : The correctly spelt word from the given options is (a) 'Bouquet' which means- 'Epergne'/Posy. The other correct options are– Sequence - Series Acquarium - Fishbowl, Frequency - Recurrence Thus, the correct answer is option (a). 91. Select the correctly spelt word. (a) Memor (b) Studiuous (c) Restaurent (d) Business SSC CHSL (Tier-I) 13/10/2020 (Shift-III) Ans. (d) The correctly spelt word from the given option is (d) 'Business' which means- 'buying and selling as a way of earning money'. The other correct options are– Memoir – Narration Studious – Educated Restaurant – Canteen Thus, the correct answer is option (d). 92. Select the correctly spelt word. (a) Beautifuli (b) Beautifuly (c) Beautifully (d) Beautefully SSC MTS – 13/08/2019 (Shift-III) Ans. (c) : All the above given options are incorrect except option (c) 'Beautifully' which means– 'in a way that is very attractive.' The other given options are incorrect. Thus, the correct answer is option (c). 93. Select the correctly spelt word. (a) Buraucracy (b) Bureaucracy (c) Bureaucrasy (d) Bureucracy SSC GD – 18/02/2019 (Shift-II) Ans. (b): : The correctly spelt word from the given options is (b) 'Bureaucracy' which means– 'A system for controlling a country or organization that is operted by officials. The other given options are incorrectly spelt. Thus, the correct answer is option (b).

C 94.

Select the correctly spelt word. (a) Concensus (b) Concencus (c) Consencus (d) Consensus SSC MTS 10/05/2023 (Shift-I) Ans. (d) : For the above given options the correctly spelt word is 'consensus' which means Harmony. other options are incorrect. Thus, the correct answer is option (d). 95. Select the correct spelling of the underlined word from the given options to make the sentence meaningfully correct. I am very confidant of my positive result I the examination.

Misspelt word

(b) confidence (d) confiedent SSC GD 13/02/2023 (Shift-I) Ans. (a) : In the given sentence, 'confident' is the correct spelling of the underline word 'confidant. Confident means– reassure, positive. Hence option (a) is correct answer. 96. Identify the correctly spelt sentence. (a) The events arranged or organised in order of time are known as chronoligal order. (b) The events arranged or organised in order of time are known as chronological order. (c) The events arranged or organised in order of time are known as chronologikal order. (d) The events arranged or organised in order of time are known as chronoloigal order. SSC GD 10/01/2023 (Shift-I) Ans. (b) : Among the above options, the spelling of the word 'Chronological' used in the second sentence is correct. Hence the sentence of option (b) is correctly spelt. Correct sentenceThe events arranged or organised in order of time are known as chronological order. 97. Select the INCORRECTLY spelt word. (a) Announce (b) Cotarminous (c) Ambient (d) Felicitate SSC CGL (Tier-I) 25/07/2023 (Shift-IV) Ans. (b) : The above given option are incorrectly spelt except option (b) 'cotarminous.' The correctly spelt word is 'coerminous' which means- 'having the same boundaries or extent in space, time, or meaning. Hence, the correct option is (b) 98. Select the correct spelling from the given options to fill in the blank. Mihika was one of the _______ in the Master Chef competition held in our society last year. (a) contestents (b) contestants (c) kontestants (d) contextants SSC CHSL (Tier-I) 17/08/2023 (Shift-II) Ans. (b) : Option (b) Contestants – ( A person who lakes part in a contest or competition) contains the correct spelling hence, It will be filled in the given void space of the sentence. Correct sentence – mihika was one of the contestants in the Master chef competition held in our society last year. 99. The following sentence has been spilt into four segments. Identify the segment that contains a spelling error. You cannot/ compensate for the/ wrongdoings you have/ comitted in the past. (a) wrongdoings you have (b) comitted in the past. (c) You cannot (d) compensate for the SSC CHSL (Tier-I) 14/08/2023 (Shift-IV)

526

(a) confident (c) confidance

YCT

Ans. (b) : In the option (b) 'comitted in the past' is erroneous and contains spelling error. Its correct spelling would be 'committed' – obligated by a pledge to some course of action. Correct Sentence you con not compensate for the wrongdoings you have committed in the past. 100. Parts of the following sentence have been given as options. Select the option that contains an error. My colleages have lacked the conscience needed to be morally correct on this grave issue. (a) morally (b) conscience (c) grave (d) colleages SSC CHSL (Tier-I) 03/08/2023 (Shift-II) Ans. (d) : In the above sentence, the parts of the given option (d) 'Colleages' is incorrectly spelt. the correctly spelt word is 'Colleagues' which means.– 'one of a group of people who work together.' other options are correctly spelt. 101. Choose the incorrectly spelt word. (a) Clovver (b) Sharpen (c) Sunrise (d) Happen SSC CHSL (Tier-I) 09/02/2023 (Shift-II) Ans. (a) : For the above options, option (a) "Clovver" is misspelled. Its correct spelling is "Clever". Other options are correct. 102. Select the INCORRECTLY spelt word. (a) Absence (b) Conferense (c) Expense (d) Entrance SSC Constable GD-10/12/2021 Shift-III Ans. (b) :The word 'conferense' is incorrect because the correct spelling is 'Conference' which means 'Congress'. The other words areAbsence - Deficiency Expense - Expenditure Entrance - Entry 103. Select the option that corrects the misspelt words in the given sentence. A baloon may liave the earth with a charg. (a) A balloon may leave the earth with a charge. (b) A ballon may leave the earth with a chargge. (c) A balloon may live the earth with a charrge. (d) A ballun may leave the earth with a charge. SSC MTS-12/07/2022 Shift-I Ans. (a) : In the given sentence the correct answer is options (a). The words 'Baloon' 'Liave' and 'charge' are incorrect because the correct spelling are 'Balloon' 'Leave' and 'charge'. Therefore the correct sentence will be– A balloon may leave the earth with a charge. 104. Select the INCORRECTLY spelt word. (a) Innocence (b) Essence (c) Conveyance (d) Conveniance SSC Constable GD-14/12/2021 Shift-II Misspelt word

Ans. (d) : The word 'Conveniance' is incorrect because the correct spelling is 'Convenience' which means facility. The other words are Innocence - Gullibility Essence - Extract Conveyance - Carriage 105. Select the INCORRECTLY spelt word. (a) Cuisine (b) Mundane (c) Beverage (d) Celastiel SSC Constable GD-14/12/2021 Shift-III Ans. (d) : The word 'celastiel' is incorrect because the correct spelling is 'celestial' which means 'Heavenly body'. The other words are Cuisine - Related to food. Mundane - Profane, worldly Beverage - Drinks Thus, the correct answer is options (d). 106. Select the incorrectly spelt word. (a) Vein (b) Quick (c) Certain (d) Contein SSC Constable GD-02/12/2021 Shift-I Ans. (d) : The word 'contein' is incorrect because the correct spelling is 'contain' which means 'imply'. Thus, the correct answer is options (d). 107. Select the INCORRECTLY spelt word. (a) Collection (b) Consolidation (c) Coressponding (d) Complaint SSC Constable GD-17/11/2021 Shift-II Ans. (c) : The word 'coressponding' is incorrect because the correct spelling is 'corresponding', while other options are correct. Thus the correct answer is options (c). 108. Select the correctly spelt word. (a) Imigrant (b) Chauvinist (c) Contemperary (d) Cosmopolition SSC MTS-05/07/2022 Shift-III Ans. (b) : Out of the given options, only 'chauvinist' is spelt correctly. It means 'Jingoism' . The other options are incorrect. Thus, the correct answer is option (b) 109. Select the correctly spelt word. (a) Convect (b) Conect (c) Convert (d) Concent SSC CGL (Tier-I) 18/04/2022 Shift-I Ans. (c) : The correctly spelt word is ‘convert’ means Tranform. The remaining words are incorrect. Thus, the correct answer is (c). 110. Select the sentence which has no spelling error. (a) Neeta represented Global School in the state level debate competition. (b) Gupta empire posed a competetoin to the invaders in ancient India. (c) The cooking compitition was judged by the head chef of Garland Hotels. (d) There is a high market compettion due to the advent of online shopping. SSC CHSL 26.05.2022 Shift-II

527

YCT

Ans. (a) : In the given options, the options (a) is correct answer, because the word 'competition' is correctly spelt in this sentence. While other option are incorrect. 111. Select the incorrectly spelt word. (a) Opportunity (b) Conversation (c) Warrant (d) Collaegue SSC CHSL 26.05.2022 Shift-II Ans. (d) : The word 'Collaegue' is incorrect because the correct spelling is 'colleague' which means comrade. Thus the correct answer is options (d). The remaining words are correct. 112. Select the incorrectly spelt word. (a) Frigidity (b) Corteous (c) Mandatory (d) Amnesia SSC CHSL 27.05.2022 Shift-II Ans. (b) : The word 'Corteous' is in correct because the correct spelling is 'Courteous' which means submissive. The other words are Frigidity - coolness Mandatory - compulsory Amnesia - loss of memory Thus, the comment option is (b). 113. Select the INCORRECTLY spelt word. (a) Elegant (b) Cavalier (c) Charishma (d) Dominate SSC CHSL 26.05.2022 Shift-III Ans. (c) : The word 'Charishma' is the incorrect because the correct spelling is 'Charisma' which means 'talent'. The other words are Elegant - Aesthetic Cavalier - supercilious Dominate - Reign Thus, the correct answer is option (c). 114. Select the incorrectly spelt word. (a) Customary (b) Domicile (c) Tactile (d) Concentratte SSC CHSL 31.05.2022 Shift-III Ans. (d) : The word 'Concentratte' is the incorrect spelt because the correct spelling is 'concentrate' which means 'Heed'. The other options are Customary - commonly practiced Domicile - Habitat Tactile - touchable. Thus, the correct answer is option (d). 115. From among the words given in bold, select the incorrectly spelt word in the following sentence. Small drops trickled down the red and yellow tassels of cannopies and dampened the heads of little boys. (a) trickled (b) tassels (c) cannopies (d) dampened SSC CHSL 24.05.2022 Shift-III Ans. (c) : The word 'cannopies' is incorrect because the correct spelling is 'canopies' which means 'tent', 'sunshade'. Misspelt word

The other words areTrickled - leak Tassels - Cluster Dampened to make something slightly wet. Thus, the correct answer is options (c). 116. Select the incorrectly spelt word. (a) Concensas (b) Articulate (c) Colossal (d) Magnificent SSC CGL (Tier-I) –20/08/2021 (Shift-I) Ans. (a) : The incorrectly spelt word from the given options is (a) 'Concensas'. The correctly spelt word is 'Consensus' which means- 'a general agreement about something'. The other correct options areArticulate - Clear Colossal - Profound Magnificent - Splendid Thus, the correct answer is option (a). 117. Select the incorrectly spelt word. (a) Calender (b) Cadre (c) Commit (d) Carnivorous SSC CGL (Tier-I) –20/08/2021 (Shift-I) Ans. (a) : The incorrectly spelt word from the given options is (a) 'Calender', the correctly spelt word is 'Calendar' which means- 'a list of important events and dates they take place'. The other options are correctly speltCadre - Skeleton Commit - Enunciate Carnivorous - Predatory Thus, the correct answer is option (a). 118. In the following question, four words have been given out of which one word is incorrectly spelt. Select the incorrectly spelt word. (a) Acquaintance (b) Ascendance (c) Claivoyence (d) Countenance SSC CGL (Phase-II) 17/02/2018 Ans. (c): The incorrectly spelt word from the given options is (c) 'Claivoyence'. The correctly spelt word is 'Clairvoyance' which means- 'the supernatural power of seeing objects or actions removed in space or time from natural viewing. The other options are correctly speltAcquaintance – Notion Ascendance – Sovereignty Countenance – Support Thus, the correct answer is option (c). 119. Select the option which is correctly spelt. (a) contageous (b) contegious (c) contagious (d) countagious SSC Sel. Post Phase-VII (M.L.) 15.10.2019 (Shift-I)

Ans. (c) : The correctly spelt word from the given options is (c) 'Contagious' which means- 'a disease that you can get by touching somebody or something. The other given options are incorrect. Thus, the correct answer is option (c). 120. Select the misspelt word (a) Credulous (b) Contagious (c) Consensus (d) Contemptous

528

SSC Sel. Post Phase-VIII (G.L.) 09.11.2020 (Shift-II)

YCT

Ans. (d) : The incorrectly spelt word from the given options is (d) 'Contemptous'. The correctly spelt word is 'Contemptuous' which means- 'feeling or expressing deep hatred or disapproval.' The other correct options areCredulous - Trustful Contagious - Infectious Consensus - Accord Thus, the correct answer is option (d). 121. Select the wrongly spelt word. (a) custody (b) curtesy (c) custom (d) cursory SSC CPO-SI – 25/11/2020 (Shift-I) Ans. (b) : The wrongly spelt word from the given options is (b) 'Curtesy'. The correctly spelt word is 'courtesy' which means- 'polite and pleasant behaviour that shows respect for other people.' The other correct options areCustody - Lockup, Custom - Tradition, Cursory - Shallow Thus, the correct answer is option (b). 122. Select the correctly spelt word. (a) comosion (b) cammosion (c) commotion (d) camotion SSC CPO-SI – 24/11/2020 (Shift-I) Ans. (c) : All the above given options are incorrect except option (c) 'Commotion' which means- 'great noise or excitement'. Thus, the correct answer is option (c). 123. Select the correctly spelt word. (a) cartificate (b) sertificate (c) certificate (d) sartifikate SSC CGL (Tier-I) – 19/06/2019 (Shift-III) Ans. (c) : The correctly spelt word from the given options is (c) 'Certificate' which means- 'an official piece of paper that says that something is true or correct.' The other given options are incorrect. Thus, the correct answer is option (c). 124. Select the correctly spelt word. (a) chooze (b) chooj (c) chuse (d) choose SSC CGL (Tier-I) – 13/06/2019 (Shift-III) Ans. (d): The correctly spelt word from the given options is (d) 'Choose' which means- 'to decide which thing or person you want out of the ones select.' The other given options are incorrectly spelt. Thus, the correct answer is option (d). 125. Select the correctly spelt word. (a) Challeng (b) Challange (c) Chalenge (d) Challenge SSC CGL (Tier-I) – 10/06/2019 (Shift-II) Ans. (d): The correctly spelt word from the given options is (d) 'Challenge' which means- 'something new and different that forces you to make a lot of effort.' The other given options are incorrectly spelt. Thus, the correct answer is option (d). 126. Select the correctly spelt word. (a) Concentious (b) Conscentious (c) Conscientious (d) Consenteous SSC CGL (Tier-I) – 10/06/2019 (Shift-II) Misspelt word

Ans. (c): The correctly spelt word from the given option is (c) 'Conscientious' which means- 'Honest/Sincere'. The other given options are incorrectly spelt. Thus, the correct answer is option (c). 127. Select the correctly spelt word. (a) concious (b) corigible (c) complascent (d) contemptible SSC CGL (Tier-I) – 07/06/2019 (Shift-III) Ans. (d): The correctly spelt word from the given options is (d) 'Contemptible' which means- 'not deserving any respect at all' (Sordid). The other correct options areConscious - Aware Complacent - Cheerful Corrigible - Polite Thus, the correct answer is option (d). 128. Select the wrongly spelt word. (a) controversial (b) conquer (c) contamporary (d) cooperation SSC CGL (Tier-I) – 06/06/2019 (Shift-I) Ans. (c) : The incorrectly spelt word from the given option is (c) 'Contamporary'. The correctly spelt word is 'Contemporary' which means- 'belonging to the same time as somebody or something'. The other given options are correctly spelt. Controversial Disputable Conquer Vanquish Cooperation Collaborate Thus, the correct answer is option (c). 129. Select the wrongly spelt word. (a) callous (b) career (c) calander (d) carriage SSC CGL (Tier-I) – 04/06/2019 (Shift-I) Ans. (c) : The incorrectly spelt word from the given options (c) 'Calander'. The correctly spelt word is 'Calendar' which means- 'Almanac'. The other correct options areCallous - Stiff, Career - Profession, Carriage - Cart Thus, the correct answer is option (c). 130. Select the wrongly spelt word. (a) Comparison (b) Communication (c) Compitition (d) Comparable SSC CGL (Tier-I) – 04/06/2019 (Shift-II) Ans. (c): The incorrectly spelt word from the given option is (c) 'Compitition'. The correctly spelt word is 'Competition' which means- 'Juxtaposition'. The other correct options areComparison- Similitude, Communication- Infusion Comparable- Alike Thus, the correct answer is option (c). 131. Select the wrongly spelt word. (a) comparable (b) committee (c) consceince (d) commission SSC CGL (Tier-I) – 04/06/2019 (Shift-III) Ans. (c) : The incorrectly spelt word from the given option is (c) 'Consceince'. The correctly spelt word is 'Conscience' which means- 'the inner sense of what is right or wrong in one's conduct or motives'. The other given options are correctly spelt–

529

YCT

Comparable - Equivalent, Committee - Council, Commission - Board Thus, the correct answer is option (c). 132. Select the wrongly spelt word. (a) ambitious (b) fictitious (c) capritious (d) nutritious SSC CPO-SI – 09/12/2019 (Shift-II) Ans. (c): The incorrectly spelt word from the given option is (c) 'Capritious'. The correctly spelt word is 'Capricious' which means- 'Changing behaviour suddenly in a way that is difficult to protect'. The other given options are correctly spelt– Ambitious- Wishful Fictitious -Fanciful Nutritious - Alible Thus, the correct answer is option (c). 133. Select the correctly spelt word. (a) Credibel (b) Creddible (c) Credible (d) Credibal SSC CPO-SI – 13/12/2019 (Shift-I) Ans. (c) : The correctly spelt word from the given option is (c) 'Credible' which means- 'that you can believe'. The other given options are incorrectly spelt– Thus, the correct answer is option (c). 134. Select the correctly spelt word. (a) counterfiet (b) countorfelt (c) counterfeit (d) counterfate SSC CHSL (Tier-I) –11/07/2019 (Shift-III) Ans. (c): The correctly spelt word from the given option is (c) 'Counterfeit' which means- 'not genuine, but copied so that it looks like the real thing'. The other given options are incorrectly spelt– Thus, the correct answer is option (c). 135. Select the correctly spelt word. (a) commemorete (b) comemorate (c) commemmorate (d) commemorate SSC CHSL (Tier-I) –10/07/2019 (Shift-II) Ans. (d): The correctly spelt word from the given option is (d) 'Commemorate' which means- 'to make people remember a special event'. (Celebrate) The other given options are incorrectly spelt– Thus, the correct answer is option (d). 136. Select the correctly spelt word. (a) contrapsion (b) cunstruction (c) controversy (d) controdiction SSC CHSL (Tier-I) –10/07/2019 (Shift-III) Ans. (c): All the above given options are incorrect except option (c) 'Controversy' which means'Disagreement about something'. The other correct options areContraption- Appliance Construction- Production Contradiction- Disaccord Thus, the correct answer is option (c). 137. Select the wrongly spelt word. (a) comencement (b) resentment (c) continuation (d) triangulation SSC CHSL (Tier-I) –09/07/2019 (Shift-III) Misspelt word

Ans. (a): The incorrectly spelt word from the given option is (a) 'Comencement'. The correctly spelt word is 'Commencement' which means- 'start'. The other given options are correctly spelt– Resentment- Indignation, Continuation- Seriality, Tringulation- Formation of or division into triangle Thus, the correct answer is option (a). 138. Select the wrongly spelt word. (a) particle (b) consize (c) discern (d) nurture SSC CHSL (Tier-I) –05/07/2019 (Shift-I) Ans. (b): All the given options are correctly spelt except option (b) 'Consize'. The correctly spelt word is 'Concise' which means- 'giving a lot of information in a few words'. The other given options are correctly spelt– Particle- Jot, Discern- Identify, Nurture- Alimentation Thus, the correct answer is option (b). 139. Select the wrongly spelt word. (a) curious (b) curse (c) curry (d) cursore Ans. (d): The incorrectly spelt word from the given option is (d) 'Cursore'. The correctly spelt word is 'Cursor' which means- 'a small sign on a computer screen'. The other given options are correctly spelt– Curious- Eager, Curse- Calumny Curry- Sauce or relish for rice. Thus, the correct answer is option (d). 140. Select the wrongly spelt word. (a) celebrity (b) celebrate (c) celeberation (d) celebrant SSC CHSL (Tier-I) –01/07/2019 (Shift-III) Ans. (c): The incorrectly spelt word from the given option is (c) 'Celeberation'. The correctly spelt word is 'Celebration' which means- 'a joyful occasion for special festivities to mark some happy event'. The other given options are correctly spelt– Celebrity - Magnate, Celebrate - Commemorate, Celebrant - a person who performs a rite Thus, the correct answer is option (c). 141. Select the INCORRECTLY spelt word. (a) Changable (b) Mileage (c) Arrangement (d) Practice SSC CHSL (Tier-I) 16/10/2020 (Shift-III) Ans. (a) : The incorrectly spelt word from the given option is (a) 'Changable'. The correctly spelt word is 'Changeable' which means- 'unsettled'. The other given options are correctly spelt– Mileage - Benefit, Arrangement - System, Practice - Exercise Thus, the correct answer is option (a). 142. Select the INCORRECTLY spelt word. (a) Resultant (b) Constraint (c) Expanse (d) Conscentious SSC CHSL (Tier-I) 21/10/2020 (Shift-II)

530

YCT

Ans. (d) : The incorrectly spelt word from the given option is (d) 'Conscentious'. The correctly spelt word is 'Conscientious' which means- 'Honest'. The other given options are correctly spelt– Resultant – Consequent Constraint – Duress Expanse – Extension Thus, the correct answer is option (d). 143. Select the word with the correct spelling. (a) Inseckts (b) Contemptuous (c) Aggresive (d) Biscits SSC CHSL (Tier-I) 17/03/2020 (Shift-II) Ans. (b) : The correctly spelt word from the given option is (b) 'Contemptuous' which means- 'feeling deep hatred or disapproval'. The other correct options areInsects– Moth, Aggressive–Rampant, Biscuit–Cookie Thus, the correct answer is option (b). 144. Select the wrongly spelt word. (a) Conjugate (b) Corporate (c) Collaborate (d) Comemorate SSC CHSL (Tier-I) 18/03/2020 (Shift-I) Ans. (d) : All the above given options are correctly spelt except option (d) 'Comemorate'. The correctly spelt word is 'Commemorate' which means- 'to make people remember a special events'. The other given options are correctly spelt– Conjugate - Conjoint, Corporate - Collective Collaborate - Cooperate Thus, the correct answer is option (d). 145. Select the wrongly spelt word. (a) Courteous (b) Circueteous (c) Momentous (d) Delicious SSC CHSL (Tier-I) 21/10/2020 (Shift-I) Ans. (b) : The incorrectly spelt word from the given option is (b) 'Circueteous'. The correctly spelt word is 'Circuitous' which means- 'Helicoid'. The other given options are correctly spelt– Courteous – Submissive Momentous – Massive Delicious – Yummy Thus, the correct answer is option (b). 146. Select the wrongly spelt word. (a) Attacked (b) Clearify (c) Provision (d) Promote SSC CHSL (Tier-I) 15/10/2020 (Shift-II) Ans. (b) : The incorrectly spelt word from the given option is (b) 'Clearify'. The correctly spelt word is 'Clarify' which means- 'explain'. The other given options are correctly spelt– Attacked - Strike, Provision - Principle, Promote - Upgrade Thus, the correct answer is option (b). 147. Select the correctly spelt word. (a) Changeable (b) Insigneficant (c) Immpending (d) Unsustanable SSC CHSL (Tier-I) 14/10/2020 (Shift-III) Ans. (a): The correctly spelt word from the given option is (a) 'Changeable' which means- 'Temporary'. The other correct options areInsignificant–Frivolous, Impending – Adjacent, Unsustainable - Temporary Thus, the correct answer is option (a). Misspelt word

148. Select the wrongly spelt word. (a) conveyance (b) conviencing (c) commemorate (d) commensurate SSC CHSL (Tier-I) 20/10/2020 (Shift-III) Ans. (b) : The incorrectly spelt word from the given option is (b) 'Conviencing'. The correctly spelt word is 'Convincing' which means- 'able to make somebody believe something'. The other given options are correctly spelt– Conveyance - Carry, Commemorate - Celebrate, Commensurate - Compatible Thus, the correct answer is option (b). 149. Select the wrongly spelt word. (a) Compromize (b) Sympathize (c) Scrutinize (d) Colonize SSC CHSL (Tier-I) 19/03/2020 (Shift-II) Ans. (a) : The incorrectly spelt word from the given option is (a) 'Compromize'. The correctly spelt word is 'Compromise' which means- 'to adjust or settle by mutual concessions'. The other given options are correctly spelt– Sympathize– Commiserate, Scrutinize– Check, Colonize– Resettle Thus, the correct answer is option (a). 150. Select the word the correct spelling. (a) Capricious (b) Ecstacy (c) Complasent (d) Avalaunche SSC CHSL (Tier-I) 17/03/2020 (Shift-III) Ans. (a) : The correctly spelt word from the given option is (a) 'Capricious' which means- 'Changing behaviour suddenly in a way that is difficult to predict.'. The other correct options areEcstasy - Bliss, Complacent - Cheery, Avalanche - Icefall Thus, the correct answer is option (a). 151. In the following question, a word has been written in four different ways out of which only one is correctly spelt. Select the correctly spelt word. (a) Conservative (b) Conservetive (c) Concervative (d) Conservrative SSC MTS 7-10-2017 (Shift-I) Ans. (a) : The correctly spelt word from the given option, is “Conservative” which means ‘pedantic’. The remaining option are incorrectly spelt. Hence option (a) is correct answer. 152. Select the correctly spelt word. (a) Comittee (b) Commite (c) Committee (d) Commitee SSC MTS – 09/08/2019 (Shift-I) Ans. (c) : The correctly spelt word from the given option, is “Committee” which means “Board/ Commission”. The remaining options are incorrectly spelt. Hence option (c) is correct answer. 153. Choose the correctly spelt word. (a) Convenience (b) Konvenience (c) Convinience (d) Conveniense SSC MTS – 07/08/2019 (Shift-I)

531

YCT

Ans. (a) : The correctly spelt word from the given options, is “Convenience” which means “Facility/Vantage”. The remaining options are incorrectly spelt. Hence option (a) is correct answer. 154. Select the correctly spelt word. (a) Changeble (b) Changable (c) Chageable (d) Changeable SSC GD – 11/02/2019 (Shift-II) Ans. (d) : The correctly spelt word from the given options, is “Changeable” Which means “Unstable”. The remaining options are incorrectly spelt. Hence option (d) is correct answer.

159. Choose the sentence with no spelling error. (a) The joy definitely lies in cooking for others. (b) The joy definitely lyes in cooking for others. (c) The joy defintely lies in cooking for others. (d) The joy definitely lies in cokking for others. SSC CHSL (Tier-I) 10/03/2023 (Shift-III) Ans. (a) : In the above options, option (a) "The joy definitely lies in cooking for others" is correct sentence. Where in other options have spelling mistake. Correct Sentence– The joy definitely lies in cooking for others.

160. Select the INCORRECTLY spelt word. (a) Nascent (b) Malleable (c) Dissiminate (d) Obsequious SSC MTS-07/07/2022 Shift-II 155. Select the INCORRECTLY spelt word. Ans. (c) : Out of the given options incorrect options is (a) Thankful (b) Dutyful 'Dissiminate'. Its correct spelling is 'Disseminate'. (c) Merciful (d) Plentiful SSC GD 23/01/2023 (Shift-IV) Means 'Canvass'. Ans. (b) : Out of the above options, option (b) Dutyful Thus, the correct answer is options (c). is incorrectly spelt. Its correct spelling would be 161. Select the correctly spelt word. 'Dutiful' (Dedicated). (a) Disstract (b) Destarct Meaning of the other options(c) Distract (d) Distarct Thankful Grateful SSC MTS-13/07/2022 Shift-I Merciful Indulgent Ans. (c) :Out of the given (c) 'Distract' is spelt Plentiful Bountiful, Abundant correctly. While the remaining options have incorrect 156. Select the INCORRECTLY spelt word from spelling. the given sentence. Drunkeness, riot and brawling were the order, Thus, the correct answer is option (c)

D

there, every night. (a) Order (b) Drunkeness (c) Brawling (d) Riot SSC CHSL (Tier-I) 08/08/2023 (Shift-II) Ans. (b) : The incorrectly spelt word of the above sentence is option (b) 'Drunkeness.' The correctly spelt word is 'Drunkenness' which means – 'the state of being drunk.' Other options are correctly spelt. 157. Select the wrongly spelt word. (a) Malleable (b) Nascent (c) Obsequious (d) Dissiminate SSC CHSL (Tier-I) 15/03/2023 (Shift-I) Ans. (d) : In the above options, option (d) 'Dissiminate' is spelt wrongly its correct spelling is "Disseminate" (Expand). Other options are correct.

162. Select the INCORRECTLY spelt word. (a) Conscious (b) Opportunity (c) Disturbence (d) Genius SSC CGL (Tier-I) 19/04/2022 Shift-II Ans. (c) : The word 'Disturbence' is incorrect because the correct spelling is 'Disturbance' which means disorder. The other words are Conscious - Aware Opportunity - Occasion Genius - Brillance Thus, the correct answer is option (c).

163. Select the incorrectly spelt word. (a) Neutron (b) Decieve (c) Gothic (d) Frosty SSC CHSL 26.05.2022 Shift-III Ans. (b) : The word 'Decieve' is incorrect because 158. Select the incorrectly spelt word. correct spelling is 'deceive' which means betray/ (a) Deliceous (b) Progress Hoodwink. (c) Present (d) Approve The other words are SSC Constable GD-06/12/2021 Shift-III Neutron - It is a kind of atoms Ans. (a) : The word 'Deliceous' is incorrect because the Gothic - Strange/unusual correct spelling is 'Delicious'. Frosty - very cold The other words areThus, the correct answer is option (b). Progress - advancement 164. Select the incorrectly spelt word. Present - current (a) Discipline (b) Delicious Approve - favor. (c) Desparate (d) Dependence Thus, the correct answer is options (a). SSC CHSL (Tier-I) –15/04/2021 (Shift-I) Misspelt word

532

YCT

Ans. (c) : The incorrectly spelt word is option © “Desparate” and it correct spelling is “Desperate” which means “Hopeless”. The other correct options are– Discipline – Regimentation Delicious – Tasty Dependence – Support 165. Select the option which is correctly spelt. (a) discpline (b) discipiline (c) discipline (d) descripline SSC Sel. Post Phase-VII (M.L.) 15.10.2019 (Shift-I) Ans. (c) : Out of given options, option (c) “discipline’ is spelt correctly it means “Regimentation”. While other options are incorrect. Hence option (c) is the correct answer. 166. Select the wrongly spelt word. (a) decease (b) decibel (c) decieve (d) decency SSC CPO-SI – 25/11/2020 (Shift-I) Ans. (c) : The wrongly spelt word is option (c) “Decieve” and its correct spelling is “Deceive” it means “Hood wink” The remaining option are spelt correctly. Decease – Death Decibel – The unit used to measure the intensity of sound. Decency – Courtesy/Decorum 167. Select the correctly spelt word. (a) desperete (b) disparat (c) desperate (d) desparate SSC CGL (Tier-I) – 11/06/2019 (Shift-II) Ans. (c) : Out of the given options, option (c) “Desperate” is spelt correctly is means “Hopeless” While the remaining options are incorrect. Hence option (c) is the correct answer. 168. Select the wrongly spelt word. (a) definate (b) define (c) deform (d) deflate SSC CGL (Tier-I) – 07/06/2019 (Shift-I) Ans. (a): The wrongly spelt word is “Definate” and the correct spelling is “Definite” it means “Certain/Sure” The remaining option are correct. Define – Declane Deform – Deface Deflate – Negate 169. Select the wrongly spelt word. (a) devine (b) dive (c) deviate (d) divide SSC CPO-SI – 09/12/2019 (Shift-II) Ans. (a): The wrongly spelt word is “Devine” and the correct spelling is “Divine” it means “Heavenly” The remaining options are correct. Dive – Dip Deviate – Straggle Divide – Split 170. Select the wrongly spelt word. (a) describe (b) descern (c) descent (d) desert SSC CPO-SI – 09/12/2019 (Shift-II) Misspelt word

Ans. (b): The wrongly spelt word is “Descern” and the correct spelling “Discern” is means “Identify” The remaining options are correct. Describe – depict Descent – Touch-down /Ancestry Desert – Sands 171. Select the INCORRECTLY spelt word. (a) despise (b) describe (c) desire (d) designe SSC CPO-SI – 11/12/2019 (Shift-I) Ans. (d): The incorrectly spelt word is option (d) “Designe” and its correct spelling is “Design” means “Taxture”. The remaining options are correct. Despise – Hate/Disdain Describe – Depict Desine – Wish 172. Select the INCORRECTLY spelt word. (a) illumination (b) discrimnation (c) hallucination (d) intimation SSC CPO-SI – 12/12/2019 (Shift-II) Ans. (b): The incorrectly spelt word is option (b) “Discrimnation” and its correct spelling is 'Discrimination' means “Partiality”. The remaining options are correct. Illumination - Light Hallucination - Phantasm Intimation - Information 173. Select the INCORRECTLY spelt word. (a) orientation (b) disappointment (c) corroborate (d) disapprooval SSC CPO-SI – 13/12/2019 (Shift-II) Ans. (d) : The incorrectly spelt word is “Disapprooval” and its correct spelling is “Disapproval” means “Rejection”. The remaining options are correct. Orientation – Guideline Disappointment – Despair Corroborate – Stand for 174. Select the INCORRECTLY spelt word. (a) Dimention (b) Jostle (c) Ideology (d) Illiterate SSC CHSL (Tier-I) 16/10/2020 (Shift-I) Ans. (a) : The incorrectly spelt word is “Dimention” and its correct spelling is “Dimension” means “Amplitude”. The remaining options are correct. Jostle – Jerk Ideology – Dogma Illiterate – Uneducated 175. Select the INCORRECTLY spelt word. (a) Fattening (b) Decieve (c) Dessert (d) Business SSC CHSL (Tier-I) 13/10/2020 (Shift-I) Ans. (b) : The incorrectly spelt word is “Decieve” and its correct spelling is “Deceive” means “Hoodwink”. The remaining options are correct. Fattening – Thickness Dessert – Confection Business – Trade/commerce

533

YCT

176. Select the wrongly spelt word. (a) Valediction (b) Selection (c) Conviction (d) Defenition SSC CHSL (Tier-I) 18/03/2020 (Shift-I) Ans. (d) : The wrongly spelt word is “Defenition” and its correct spelling is “Definition” means “Explanation” The remaining options are correct. Valediction- Farewell Selection- Choice/Fleet Conviction- Nation/Belief 177. Select the wrongly spelt word. (a) Transparent (b) Deterent (c) Different (d) Irreverent SSC CHSL (Tier-I) 21/10/2020 (Shift-I) Ans. (b) : The wrongly spelt word is “Deterent” and its correct spelling is “Deterrent” means “Preventative” The remaining options are correct. Transparent – Pellucid Different – Apart/Various Irreverent – Degrading 178. Select the INCORRECTLY spelt word. (a) Harass (b) Heavenly (c) Advisable (d) Disposeable SSC CHSL (Tier-I) 19/10/2020 (Shift-I) Ans. (d) : The incorrectly spelt word is “Disposeable” and its correct spelling is “Disposable” means “Castaway” The remaining options are correct. Harass – Molest/Vex Heavenly – Celestial Advisable – Suitable/Proper 179. Select the INCORRECTLY spelt word. (a) Corruption (b) Discriminition (c) Embodiment (d) Identification SSC CHSL (Tier-I) 15/10/2020 (Shift-I) Ans. (b) : The incorrectly spelt word is “Discriminition” and its correct spelling is “Discrimination” means “Partiality”. The remaining options are correct. Corruption – Malversation Embodiment – Incarnation Identification – Recognition 180. Select the wrongly spelt word. (a) Diferently (b) Accidentally (c) Evidently (d) Prudently SSC CHSL (Tier-I) 21/10/2020 (Shift-III) Ans. (a) : The wrongly spelt word is “Diferently” and its correct spelling is “Differently” means Disparately. The remaining options are correct. Accidentally – Suddenly Evidently – Certainly/Sure Prudently – Carefully 181. Select the INCORRECTLY spelt word. (a) Vicious (b) Obvious (c) Deliceous (d) Courageous SSC CHSL (Tier-I) 19/10/2020 (Shift-III) Ans. (c) : The incorrectly spelt word is “Deliceous” and its correct spelling is “Delicious” means “Tasty”. The remaining options are correct. Obvious – manifest/Undoubted Vicious – Cunning/Corrupt Courageous – Bold/Daring Misspelt word

182. Select the correctly spelt word. (a) Definitly (b) Definitely (c) Definetely (d) Definately SSC MTS – 08/08/2019 (Shift-III) Ans. (b) : Out of the given options, option (b) “Definitely” is spelt correctly. It means “Absolutely” While the remaining options are incorrect. Hence option (b) is the correct answer. 183. Choose the correctly spelt word. (a) Dilema (b) Dilemma (c) Dillema (d) Dielema SSC MTS – 05/08/2019 (Shift-II) Ans. (b) : Out of the given options, option (b) “Dilemma” is spelt correctly. It means “Doubt”. While the remaining options one incorrect. Hence option (b) is the correct answer. 184. Choose the correctly spelt word. (a) Dominent (b) Disiase (c) Deminish (d) Dependent SSC MTS – 02/08/2019 (Shift-III) Ans. (d) : Out of the given options, Option (d) “Dependent” is spelt correctly. It means “Incumbent”. While the remaining options are incorrectly spelt. Dominant – Chief Disease – Illness Diminish – Mitigate Hence option (d) is the correct answer. 185. Select the correctly spelt word. (a) Surrander (b) Priviledge (c) Fasinate (d) Desperate SSC MTS – 16/08/2019 (Shift-III) Ans. (d) : Out of the given options, option (d). 'Desperate' is spelt correctly it means “Downcast”. While the remaining options are incorrectly spelt. Surrender - Capitulation Privilege - Charter Fascinate – Spellbind Hence option (d) is the correct answer. 186. Select the wrongly spelt word. (a) Journey (b) Condemn (c) Anxiety (d) Dimond SSC MTS – 14/08/2019 (Shift-II) Ans. (d) : The wrongly spelt word is “Dimond” and its correct spelling “Diamond” it means “A precious stone”. The remaining options are correct. Journey– Travel Condemn– Objurgating Anxiety– Concern 187. Select the correctly spelt word. (a) Discipline (b) Disatraus (c) Diractory (d) Dormitery SSC MTS – 13/08/2019 (Shift-II) Ans. (a) : Out of the given options, option (a) “Discipline” is spelt correctly. It means “Regimentation”. While the remaining options are incorrectly spelt. Disastrous - Critical Directory - Book of the address and number Dormitory - Hostel

534

YCT

188. Select the correctly spelt word. (a) Devlop (b) Diffrence (c) Discribe (d) Disappear SSC MTS – 13/08/2019 (Shift-II) Ans. (d) : Out of the given options, option (d) “Disappear” is “Vanish” While the remaining options are incorrectly spelt. Develop - Evolve Difference - Distinction Describe - Depict

E 189. Identify the sentence with correct spellings. (a) There was not enough food in the beggar's house. (b) There was not enouf food in the beggar's house. (c) There was not enauf food in the beggar's house. (d) There was not enoufh food in the beggar's house. SSC CGL (Tier-I) 24/07/2023 (Shift-III) Ans. (a) : For the above given sentence, the correct sentence is option (a). 'There was not enough food in the begger's house' because in this sentence no spelling error. Other options have incorrect spelling. 190. Choose the correct spelling of the underlined word in the given sentence. We are going on a hiking expidition. (a) ixpedition (b) expedetion (c) exhipidition (d) expedition SSC Selection Posts XI-27/06/2023 (Shift-I) Ans. (d) : The correct spelling of the above underlined word is option (d) 'expedition' which means 'a long journey for a special purpose; Other given options are not correct. 191. Select the INCORRECTLY spelt word. (a) Erode (b) Eraser (c) Erosion (d) Eratic SSC Constable GD-16/11/2021 Shift-III Ans. (d) : The word 'Eratic' is incorrect because the correct spelling is Erratic which means 'Random'. Thus, the correct answer is options (d). 192. Select the correctly spelt word. (a) Exploitation (b) Conversatian (c) Reveneue (d) Pashion SSC MTS-07/07/2022 Shift-I Ans. (a) : The correct word 'Exploitation' which means 'Absorption'. The other options are incorrect. Thus, the correct answer is option (a) 193. Select the correctly spelt word. (a) Entirely (b) Entierely (c) Entierly (d) Entyrely SSC MTS-15/07/2022 Shift-I Ans. (a) : The correctly spelt word 'Entirely' is correct which means whole/completely. Thus, the correct answer is options (a). Misspelt word

194. Select the correctly spelt word. (a) Unrravel (b) Equilibrium (c) Compeling (d) Esctasy SSC MTS-15/07/2022 Shift-I Ans. (b) : The correct spelt word 'Equilibrium' which means 'Balance'. The other options are incorrect. Thus, the correct answer is options (b). 195. Select the INCORRECTLY spelt word. (a) Miscellaneous (b) Disciplinarian (c) Exclaimation (d) Correspondence SSC CGL (Tier-I) 13/04/2022 Shift-II Ans. (c) : The word 'Exclaimation' is incorrect because the correct spelling is 'Exclamation' it means 'Interjection'. The other words are Miscellaneous - Diverse Disciplinarian - pedant Correspondence - mail /adjustment Thus, the correct answer is options (c). 196. Select the incorrectly spelt word. (a) Environment (b) Encouregement (c) Achievement (d) Advertisement SSC CHSL 26.05.2022 Shift-II Ans. (b) : The word 'Encouregement' is incorrect because the correct spelling is 'Encouragement' which means 'stimulus'. The other words are Environment - atmosphere Achievement - cognition Advertisement - advertising Thus, the correct answer is options (b). 197. Select the incorrectly spelt word. (a) Equine (b) Equilateral (c) Equilibrium (d) Equanimity SSC CHSL 26.05.2022 Shift-II Ans. (c) : The word 'Equilebrium' is incorrect because the correct spelling is 'Equilibrium' which means balance. Thus, the correct answer is options (c). 198. Select the misspelt word. (a) Endevour (b) Acknowledge (c) Individual (d) Auspicious SSC CHSL (Tier-I) –13/04/2021 (Shift-I) Ans. (a) : The misspelt word is “Endevour” and its correct spelling is “Endeavour” it means “Try/Effort” The remaining options are correct. Individual - Personal Acknowledge – Betake/admit Auspicious - Providential 199. Select the incorrectly spelt word. (a) Esscalator (b) Sanctity (c) Automated (d) Refurbish SSC CGL (Tier-I) –23/08/2021 (Shift-I) Ans. (a) : The incorrectly spelt word is “Esscalator” and its correct spelling is “Escalator” it means “moving staircase”. The remaining options are correct. Sanctity - Purity Automated – Working by itself Refurbish – Renovarte

535

YCT

200. Select the option which is correctly spelt. (a) exserssive (b) excesive (c) excessive (d) exsesiv SSC Sel. Post Phase-VII (M.L.) 15.10.2019 (Shift-I)

Ans. (c) : Out of the given options, option (c) “Excessive” spelt correctly it means “Intense” While the remaining options are incorrectly spelt. Hence option (c) is the correct answer. 201. Select the wrongly spelt word (a) Expect (b) Exact (c) Excess (d) Exept SSC Sel. Post Phase-VIII (M.L.) 09.11.2020 (Shift-III)

Ans. (d) : The wrongly spelt word is “Exept” and its correct spelling is “Except” it means “Besides”. The remaining options are correct. Expect – Need/Hope Exact – Appropriate Excess – Extra/More 202. Select the wrongly spelt word. (a) exprimant (b) occurring (c) sediment (d) umbrella SSC CPO-SI – 23/11/2020 (Shift-I) Ans. (a) : The wrongly spelt word is “Exprimant” and its correct spelling is “Experiment” it means 'Praxis/Test'. The remaining options are correct. Occurring – Happening Sediment – Silt/Fag Umbrella – Brolly/Ciborium 203. Select the misspelled word. (a) cautions (b) carriage (c) explotion (d) descent SSC CPO-SI – 25/11/2020 (Shift-II) Ans. (c) : The misspelled word is “Explotion” and its correct spelling is “Explosion” it means “Blast/Eruption” The remaining options are correct. cautions – Warning carriage – Transport/Cart descent – Touch-down/Landing 204. Select the misspelled word. (a) separate (b) existance (c) proceed (d) dependent SSC CPO-SI – 23/11/2020 (Shift-II) Ans. (b) : The misspelled word is “Existance” and its correct spelling is “Existence” it means “Being/Entity The remaining options are correct. separate – Distinct/Aloof proceed – Step forward Dependent – Incumbent 205. Select the correctly spelt word. (b) exhilerate (a) exilarate (c) exilerate (d) exhilarate SSC CGL (Tier-I) – 12/06/2019 (Shift-I) Ans. (d): Out of the given options, option (d) “Exhilarate” is spelt correctly it means “Gladden/Delight” While the remaining options are incorrectly spelt. Hence option (d) is the correct answer. 206. Select the correctly spelt word. (a) afliction (b) tuetion (c) exhibition (d) voilation SSC CGL (Tier-I) – 11/06/2019 (Shift-II) Misspelt word

Ans. (c) : Out of the given options, option (c) “Exhibition” is spelt correctly it means “Exposition/Show” The remaining correctly spelt words– affliction- Torment/Agony Tuition- Personal/Etching violation- Breach/Infraction 207. Select the wrongly spelt word. (a) expire (b) explain (c) expereince (d) except SSC CGL (Tier-I) – 06/06/2019 (Shift-I) Ans. (c) : The wrongly spelt word is “Expereince” and correct spelling is Experience it means “Sense/Intubation. The remaining options are correctly spelt Expire - End/Cease Explain - Decode/Clarify Except - Besides/Barring 208. Select the wrongly spelt word. (a) exploite (b) explode (c) explicit (d) expire SSC CGL (Tier-I) – 04/06/2019 (Shift-I) Ans. (a): The wrongly spelt word is “Exploite” and its correct spelling is “Exploit” it means “Utilize” The remaining options are correctly spelt. explicit - Fulminate explode – Outspoken/Obvious expire – End/ Finish 209. Select the wrongly spelt word. (a) Examplify (b) Example (c) Exhale (d) Exempt SSC CGL (Tier-I) – 04/06/2019 (Shift-II) Ans. (a): The wrongly spelt word is “Examplify” and its correct spelling is “Exemplify” it means Illustrate” The remaining options are correctly spelt. Example- Instance Exhale- Breathe out Exempt- Excul pated 210. Select the wrongly spelt word. (a) exchange (b) exercise (c) exclaim (d) exite SSC CGL (Tier-I) – 04/06/2019 (Shift-III) Ans. (d) : The wrongly spelt word is “Exite” and its correct spelling is “Exit” it means “Egress/Out let” The remaining options are correctly spelt Exchange – Reciprocation Exercise – Practice Exclaim – Cry/yell 211. Select the wrongly spelt word. (a) effigy (b) effort (c) effect (d) effecient SSC CPO-SI – 09/12/2019 (Shift-II) Ans. (d): The wrongly spelt word is “Efficient” and its correct spelling is “Efficient” it means veteran/Versed” The remaining options are correctly spelt. Effigy- Mannequin Effort- Endeavor/Attempt Effect - Impact 212. Select the INCORRECTLY spelt word. (a) economy (b) eclipse (c) echelon (d) ecsentricity SSC CPO-SI – 11/12/2019 (Shift-I)

536

YCT

Ans. (d): The incorrectly spect word is “Ecsentricity” and its correct spelling is “Eccentricity” it means “Mania/Vagary” The remaining options are correctly spelt. economy – Finance/Cheap eclipse – Obscuration echelon – Team/Department 213. Select the INCORRECTLY spelt word. (a) diagnosis (b) accumulate (c) ecstacy (d) athlete SSC CPO-SI – 12/12/2019 (Shift-I) Ans. (c): The incorrectly spelt word is “Ecstacy” and its correct spelling is Ecstasy” it means “Bliss/Jubilance” The remaining options are correctly spelt. Diagnosis – Pathology Accumulate – Store/Collect Athlete – Gymnast 214. Select the INCORRECTLY spelt word. (a) acquaintance (b) acquisition (c) aggregate (d) explaination SSC CPO-SI – 12/12/2019 (Shift-I) Ans. (d): The incorrectly spelt word is “Explaination” and its correct “Explanation” it means Interpretation” The remaining options are correctly spelt. Acquaintance – Familiarization Acquisition – Advantage/Gain Aggregate – Cumulous 215. Select the wrongly spelt word. (a) Embarrass (b) Embelish (c) Embody (d) Embrace SSC CPO-SI – 13/12/2019 (Shift-I) Ans. (b) : The wrongly is “Embelish” and its correct spelling is “Embellish” it means “Adjust”. The remaining options are correct spelt. Embarrass – Embroil/entangle Embody – Incarnate Embrace – Hug/Enclasp 216. Select the correctly spelt word (a) execption (b) ecseption (c) exception (d) ecxeption SSC CHSL (Tier-I) –09/07/2019 (Shift-II) Ans. (c): Out of the given options, option “Except” is spelt correctly it means “Detraction” While the remaining options are incorrect. Hence options (c) is the correct answer. 217. Select the wrongly spelt word. (a) extract (b) eligible (c) extension (d) explaination SSC CHSL (Tier-I) –08/07/2019 (Shift-I) Ans. (d): The wrongly spelt word is “Explaination” and its correct spelling is “Explanation” it means “Interpretation” The remaining options are correctly spelt. Extract- Quotation Eligible- Able/Deserving Extension- Enhancement/Growth 218. Select the wrongly spelt word. (a) missionary (b) jewellery (c) elementery (d) hereditary SSC CHSL (Tier-I) –04/07/2019 (Shift-III) Misspelt word

Ans. (c): The wrongly spelt word is “Elemantery” and its correct spelling is “Elementary” it means “Primary/First”. The remaining options are correct spelt. Missionary- Promulgator Jewellery- Ornament Hereditary- Patrimonial 219. Select the wrongly spelt word. (a) emphasis (b) emolument (c) emporium (d) emmision SSC CHSL (Tier-I) –02/07/2019 (Shift-III) Ans. (d): The wrongly spelt word is “Emmision” and its correct spelling is “Emission” it means “Ejections. The remaining options are correctly spelt. emphasis – Determination/Force emolument – Salary/Wage emporium – Store-house 220. Select the wrongly spelt word. (a) earn (b) eerie (c) early (d) eagel SSC CHSL (Tier-I) –01/07/2019 (Shift-III) Ans. (d): The wrongly spelt word is “Eagel” and its correct spelling is “Eagle” it means “bird” The remaining options are correctly spelt. Earn – Gain Eerie – Terrible Early – Ratheripe 221. Select the wrongly spelt word. (a) Experiment (b) Extinct (c) Extempore (d) Expidition SSC CHSL (Tier-I) 17/03/2020 (Shift-I) Ans. (d): The wrongly spelt word is “Expidition” and its correct spelling is “Expedition” it means “Campaign” The remaining options are correctly spelt. Experiment–Test Extinct–Destructed Extempore – Uncontemplated 222. Select the wrongly spelt word. (a) Envioronment (b) Solvent (c) Residual (d) Pollutant SSC CHSL (Tier-I) 18/03/2020 (Shift-III) Ans. (a) : The wrongly spelt word is “Envioronment” its correct spelling is “Environment” it means “Atmosphere”. The remaining options are correct spelt. Solvent – Merger Residual – Remaining Pollutant – Substance that pollutes 223. Select the wrongly spelt word. (a) Embarassment (b) Announcement (c) Resettlement (d) Undergarment SSC CHSL (Tier-I) 12/10/2020 (Shift-II) Ans. (a) : The wrongly spelt word is “Embarassment” its correct spelling is “Embarrassment” it means “Anxiety/Turmoil” The remaining options are correctly spelt. Announcement – Declaration Resettlement – Rehabilitation Undergarment – Inner clothes

537

YCT

224.

In the following question, a word has been written in four different ways out of which only one is correctly spelt. Select the correctly spelt word. (a) Eighth (b) Eaight (c) Aight (d) Eihght SSC MTS 10-10-2017 (Shift-I) Ans. (a): Out of the given options, option (a) “Eighth” is spelt correctly it means “Eight is a sequence” While the remaining options are incorrect. Hence option (a) is the correct answer. 225. Select the correctly spelt word. (a) Engineer (b) Ingineer (c) Injineer (d) Enginere SSC MTS – 22/08/2019 (Shift-II) Ans. (a) : Out of the given options, option (a) “Engineer” spelt correctly it means “Machinist” While the remaining options are incorrect. Hence options (a) is the correct answer. 226. Choose the correctly spelt word. (a) Axhilarate (b) Exhilarate (c) Axilarate (d) Exilarate SSC MTS – 07/08/2019 (Shift-I) Ans. (b) : Out of the given options, option (b) “Exhilarate” is spelt correctly it means "Delight/cheer" While the remaining options are incorrect. Hence options (a) is the correct answer. 227. Choose the correctly spelt word. (a) Peirce (b) Indevidual (c) Extract (d) Luxeuries SSC MTS – 06/08/2019 (Shift-I) Ans. (c) : Out of the given options, option “Extract” is spelt correctly it means “Quotation”. While the remaining options are incorrect. Pierce –Enter/Perforate Individual – Personal Luxuries – The state of great comfort 228. Choose the correctly spelt word. (a) Extravagant (b) Extrawagant (c) Extravagent (d) Extrevagant SSC MTS – 05/08/2019 (Shift-III) Ans. (a) : Out of the given options, option (a) “Extravagant” is spelt correctly. It means “Superfluous” While the remaining options are incorrect. Hence option (b) is the correct answer. 229. Choose the correctly spelt word. (a) Allert (b) Excead (c) Emphasize (d) Equipped SSC MTS – 05/08/2019 (Shift-II) Ans. (d) : Out of the given options, options “Equipped” is spelt correctly it means “Furnished”. While the remaining options are incorrect. Alert – Warning Exceed – Outreach Emphasize – Intone 230. Choose the correctly spelt word. (a) Bachelar (b) Existence (c) Calender (d) Antedote SSC MTS – 02/08/2019 (Shift-III) Misspelt word

Ans. (b) : Out of the given options, option “Existence” is spelt correctly it means “Entity/Being” While the remaining options are incorrect. Bachelor – Celibate Calendar – Almanack Antidote – Prohibitor 231. Choose the correctly spelt word. (a) Anthusiasm (b) Enthusiesm (c) Enthusiasm (d) Entheusiasm SSC MTS – 02/08/2019 (Shift-II) Ans. (c) : Out of the given options, option “Enthusiasm” is spelt correctly it means “Zeal/Rapture”. While the remaining options are incorrect. Hence option (c) is the correct aswer. 232. Select the correctly spelt word. (a) Ettiquete (b) Etiquete (c) Etiquette (d) Ettiquette SSC MTS – 21/08/2019 (Shift-III) Ans. (c) : Out of the given options, option (c) “Etiquette” is spelt correctly it means “Manners/Decorum”. While the remaining options are incorrect. Hence option (c) is the correct answer. 233. Select the correctly spelt word. (a) Exhoustive (b) Exaustive (c) Exhostive (d) Exhaustive SSC MTS – 20/08/2019 (Shift-I) Ans. (d) : Out of the given options, option (d) “Exhaustive” spelt correctly. It means “Entire/Overall” While the remaining options are incorrect. Hence options (d) is the correct answer. 234. Select the correctly spelt word. (a) Essentail (b) Essencial (c) Essential (d) Essensial SSC MTS – 13/08/2019 (Shift-I) Ans. (c) : Out of the given options, option (c) “Essential” is spelt correctly it means “Necessary”. While the remaining options are incorrect. Hence options (c) is the correct answer. 235. Select the correctly spelt word. (a) Elimantry (b) Elementary (c) Elemantry (d) Elimentary SSC MTS – 09/08/2019 (Shift-III) Ans. (b) : Out of the given options, option (b) “Elementary” is spelt correctly it means “Primary”. While the remaining options are incorrect. Hence options (b) is the correct answer. 236. Choose the correctly spelt word. (a) Explanation (b) Submision (c) Conquor (d) Confidense SSC MTS – 07/08/2019 (Shift-II) Ans. (a) : Out of the given options, option (a) “Explanation” is spelt correctly it means “Interpretation” While the remaining options are incorrect. Submission - Presentment Conquer - Win/Vanquish Confidence - Assuredness 237. Select the correctly spelt word. (a) Energancy (b) Emergency (c) Emeregency (d) Emargency SSC GD – 02/03/2019 (Shift-II)

538

YCT

Ans. (b) : Out of the given options, option (b) “Emergency” is spelt correctly it means “Exigency” While the remaining options are incorrect. Hence option (b) is the correct answer. 238. Select the correctly spelt word. (a) Exaggeration (b) Exageration (c) Exeggeration (d) Exaggaration SSC GD – 18/02/2019 (Shift-III) Ans. (a) : Out of the given options, option (a) “Exaggeration” is spelt correctly it means “Hyperbole”. While the remaining options are incorrect. Hence options (a) is the correct answer. 239. Select the correctly spelt word. (a) Embarrasment (b) Mebassassmment (c) Embarrassment (d) Embarassment SSC GD – 11/02/2019 (Shift-III) Ans. (c) : Out of the given options, option (c) “Embarrassment” is spelt correctly it means “Anxiety/Trepidation”. While the remaining options are incorrect. Hence option (c) is the correct answer.

F 240. Select the INCORRECTLY spelt word. (a) Philosophy (b) Horrendous (c) Passive (d) Farmaceutical SSC GD 06/02/2023 (Shift-III) Ans. (d) : The incorrectly spelt word is – Farmaceutical. The correct word is – 'Pharmaceutical' which means– relating to pharmacy. The meaning of other words is– Philosophy :- Any personal belief about how to live or how to deal with a situation. Horrendous :- awful, dreadful. Passive :- inactive. 241. Select the wrongly spelt word. (a) Invulnerable (b) Intrepid (c) Litigation (d) Fortuous SSC CHSL (Tier-I) 15/03/2023 (Shift-I) Ans. (d) : In the above options, option (d) ''Fortuous'' is wrongly spelt its correct spelling is 'Fortes' (stroby point) other option are correctly spelt. 242. Select the option that corrects the incorrectly spelt words in the given sentence. Fifty people have been taken into polise custoudy. (a) Fivety people have been taken into polise costody. (b) Fiffty people have been taken into polis custoudy. (c) Fifity people have been taken into police custoudy. (d) Fifty people have been taken into police custody. SSC MTS-06/07/2022 Shift-II Misspelt word

Ans. (d) : The correct spelling 'fifty and Police' are used in the sentence (d). While the remaining sentences have incorrect spelling. The correct sentence will beFifty people have been taken into police custody. Thus, the correct answer is option (d). 243. Choose the incorrectly spelt word. (a) Practical (b) Glowing (c) Released (d) Frighter SSC CHSL (Tier-I) 10/03/2023 (Shift-III) Ans. (d) : In the above options, option (d) "Fraighter" spelt incorrectly. Hence its correct spelling is "frighter". Other options are spelt correctly. 244. Select the incorrectly spelt word. (a) Segregation (b) Jingoistic (c) Fecilitation (d) Resplendent SSC CHSL 27.05.2022 Shift-I Ans. (c) : The word 'Fecilitation' is incorrect because the correct spelling is 'Facilitation' which means to make esay. The other words are Segregation - Detachment Jingoistic - patriotism Resplendent - Shiny Thus, the correct answer is options (c). 245. Select the misspelt word : (a) intrigue (b) foreiner (c) summon (d) conjure SSC CGL (Tier-II) 16/11/2020 (3 pm - 5 pm) Ans. (b) : Out of the given options, option (b) “Foreiner” is spelt incorrectly and its correct spelling is “Foreigner” it means “Gringo/Alien” Intrigue – Cabal/ Conspiracy Summon - Convene Conjure – Enchant 246. Select the misspelt word. (a) florish (b) approach (c) partner (d) slander SSC CGL (Tier-II) 16/11/2020 (3 pm - 5 pm) Ans. (a) : The misspelt word is “Florish” and its correct spelling is “Flourish” it means “Effloresce”. The remaining options are correct. Approach – Views/Access Partner – Fellow/Comrade Slander – Infamy 247. Select the word with the correct spelling. (a) favouret (b) favourite (c) favourate (d) favoirite SSC CPO-SI (Tier-II) 27/09/2019 (3 pm - 5 pm) Ans. (b) : Out of the given options, option (b) “Favourite” is spelt correctly it means “Pleasing” While the remaining options are incorrect. Hence option (b) is the correct answer. 248. Select the wrongly spelt word. (a) fascination (b) fashion (c) fascism (d) fastedious SSC CPO-SI – 25/11/2020 (Shift-I)

539

YCT

Ans. (d) : The wrongly spelt word “Fastedious” and its correct spelling is “Fastidious” it means “Touchy” The remaining options are correct. fascism - Nazism fashion – Style/Twig fascination – Spell/Glamor 249. Select the correctly spelt word. (a) fourtyeth (b) fourtieth (c) fortieth (d) fortyth SSC CGL (Tier-I) – 13/06/2019 (Shift-I) Ans. (c): Out of the given options, option (c) “Fortieth” is spelt correctly it means “forty in number” While the remaining options are incorrect Hence option (c) is the correct answer. 250. Select the INCORRECTLY spelt word. (a) generous (b) malicious (c) victorious (d) fallicious SSC CPO-SI – 12/12/2019 (Shift-II) Ans. (d): The incorrectly spelt word is “Fallicious” and its correct spelling is “Fallacious” it means “wrong/Incorrect”. The remaining options are correct. Generous – Liberal Malicious – Malevolent/Spiteful Victorious – Winner 251. Select the wrongly spelt word. (a) Fourtieth (b) Seventeenth (c) Fifteenth (d) Fourteenth SSC CHSL (Tier-I) –11/07/2019 (Shift-I) Ans. (a): The wrongly spelt word is “Fourtieth” and its correct spelling is “Fortieth” it means “forty in number” The remaining options are correct. Hence option (a) is the correct. 252. Select the correctly spelt word. (a) February (b) Janury (c) Octuber (d) Novamber SSC CHSL (Tier-I) –05/07/2019 (Shift-I) Ans. (a): Out of the given options, option (a) “February” is spelt correctly while remaining options are incorrect. Hence option (a) is the correct answer. 253. Select the wrongly spelt word. (a) Flower (b) Flour (c) Flevor (d) Floor SSC CHSL (Tier-I) 26/10/2020 (Shift-I) Ans. (c) : The wrongly spelt word is “Flevor” and its correct spelling is “Flavour” it means “Guatatory”. The remaining options are correct. Flower - Bloom Flour - Farina Floor – Ground part of a building 254. Select the wrongly spelt word. (a) Awfully (b) Unduly (c) Faithfuly (d) Unruly SSC CHSL (Tier-I) 12/10/2020 (Shift-I) Ans. (c) : The wrongly spelt word is “ Fathfuly” and its correct spelling “faithfully” it means “Sincerely” The remaining options are correct Awfully – Fearfully Unduly – Extremely Unruly – Rampant Misspelt word

255. Choose the correctly spelt word. (a) Foreign (b) Eddition (c) Forrward (d) Enfluence SSC MTS – 05/08/2019 (Shift-III) Ans. (a) : Out of the options, option (a) “Foreign” is spelt correctly it means “outside” The remaining options are incorrect. Edition- Accomplishment forward- forth/Onward Influence – Effect/Impact 256. Choose the correctly spelt word. (a) Financialy (b) Financially (c) Finantially (d) Finencially SSC MTS – 02/08/2019 (Shift-II) Ans. (b) : Out of the given options, option (b) “Financially” is spelt correctly it means “Peculiarly” While the remaining options are incorrect. Hence option (b) is the correct answer. 257. Select the correctly spelt word. (a) Feudal (b) Feudel (c) Fuedal (d) Fudal SSC MTS – 20/08/2019 (Shift-I) Ans. (a) : Out of the options, option (a) “Feudal” is spelt correctly it means “Feudatory” While the remaining options are incorrect. Hence options (a) is the correct answer. 258. Select the correctly spelt word. (a) Foriegn (b) Foaurth (c) February (d) Foward SSC MTS – 19/08/2019 (Shift-I) Ans. (c) : Out of the given options, options (c) “February” is spelt correctly. While the remaining options are incorrect. Foreign - Outward Fourth - Four in number Forward - Onward

G 259. Select the INCORRECTLY spelt word in the given sentence. the gathering of gliterring dignitaries, sat in their assigned rows and joked with vegetarian Brahmin apprentices. (a) gliterring (b) assigned (c) apprentices (d) beggars SSC CGL (Tier-I) 14/07/2023 (Shift-I) Ans. (a) : In the above sentence, the incorrectly spelt word is ‘gliterring’, the correct spelling is ‘glittering; Hence option (a) is the correct answer . 260. Select the INCORRECTLY spelt word. (a) Giant (b) Girdle (c) Gingar (d) Giggle SSC CGL (Tier-I) 21/04/2022 Shift-III Ans. (c) : The word 'Gingar' is incorrect because the correct spelling 'Ginger'. The other options are Giant - Gigantic Girdle - waistband Giggle - Grin Thus, the correct answer is option (c).

540

YCT

261. Select the INCORRECTLY spelt word. (a) Guide (b) Guard (c) Geilty (d) Global SSC CHSL 25.05.2022 Shift-I Ans. (c) : The word 'Geilty' is incorrect because the correct spelling is 'Guilty'. Which means vicious. The other words are Guide - pilot Guard - watchman Global - universal Thus, the correct answer is option (c). 262. Select the INCORRECTLY spelt word. (a) Ground (b) Grind (c) Greive (d) Groan SSC CHSL (Tier-I) –13/04/2021 (Shift-I) Ans. (c) : The incorrectly spelt word is “Greive” and its correct spelling “Grieve” it means “Mourn” While the remaining options are correct. Ground – Land Grind – Chafe Groan – Moan 263. Select the wrongly spelt word. (a) Reliant (b) Eliminate (c) Gluton (d) Allergy SSC CPO-SI – 11/12/2019 (Shift-II) Ans. (c): The wrongly spelt word is “Gluton” and its correct spelling is “Glutton” it means “Gourmand”. While the remaining options are correct. Reliant – Trustful Eliminate – Dismiss Allergy – Announce 264. Select the INCORRECTLY spelt word. (a) gadget (b) galaxy (c) gaiety (d) gallary SSC CPO-SI – 11/12/2019 (Shift-I) Ans. (d): The incorrectly spelt word “Gallary” and its correct spelling is Gallery” it means “Corridor” The remaining options are correct. Gadget – Device Galaxy – A system of millions stars Gaiety – Enjoyment 265. Select the INCORRECTLY spelt word. (a) miscellaneous (b) gregerious (c) hilarious (d) multifarious SSC CPO-SI – 12/12/2019 (Shift-II) Ans. (b): The incorrectly spelt word “Gregerious” and its correct spelling is “Gregarious”. The remaining options are correct. hilarious – Jubilant/Jolly multifarious – Diverse miscellaneous – Diverse 266. Select the correctly spelt word. (a) genuine (b) genwin (c) genuin (d) genune SSC CHSL (Tier-I) –08/07/2019 (Shift-III) Ans. (a): Out of the given options, option (a) “Genuine” is spelt correctly it means “Real/Actual” While the remaining option are incorrect. Hence option (a) is the correct answer. Misspelt word

267. Select the correct spelt word. (a) gayety (b) gaiety (c) gaity (d) gaeity SSC CHSL (Tier-I) –05/07/2019 (Shift-III) Ans. (b): Out of the given options option (b) Gaiety” spelt correctly. It means “Enjoyment/Joy” While the remaining options are incorrect. Hence options (b) is the correct answer. 268. Select the correctly spelt word. (a) genuine (b) genaral (c) guillty (d) generus SSC CHSL (Tier-I) –04/07/2019 (Shift-II) Ans. (a): Out of the given options, option (a) “genuine” is spelt correctly. It means “Real/Actual” The remaining options are incorrect. General – Usual Guilty – Vicious Generous – Liberal 269. Select the wrongly spelt word. (a) glory (b) goggles (c) gobbel (d) global SSC CHSL (Tier-I) –03/07/2019 (Shift-II) Ans. (c) : The wrongly spelt word is “Gobbel” and its correct spelling is “Gobble” it means “Gormandize” The remaining options are correct. Glory – Majesty/Prestige Goggles – Spectacles Global – Universal 270. Select the INCORRECTLY spelt word. (a) Gallop (b) Gallary (c) Galley (d) Gallant SSC CHSL (Tier-I) 20/10/2020 (Shift-I) Ans. (b) : The incorrectly spelt word is “Gallary” and correct spelling is “Gallery” it means “Corridor” The remaining options are correct. Gallop- Running/Pace Galley – Long-ship Gallant – Brave/Heroic 271. Select the INCORRECTLY spelt word. (a) Allegation (b) Disapprove (c) Permanent (d) Grammer SSC CHSL (Tier-I) 14/10/2020 (Shift-II) Ans. (d) : The incorrectly spelt word is “Grammer” and its correct spelling is “Grammar” The remaining options are correct. Allegation – Accusation Disapprove – Reject/Deny Permanent – Durable/Lasting 272. In the following question, a word has been written in four different ways out of which only one is correctly spelt. Select the correctly spelt word. (a) Grammatoical (b) Grammatical (c) Grammetical (d) Garammatical SSC MTS 7-10-2017 (Shift-I) Ans. (b) : Out of the given options, option (b) “Grammatical” is spelt correctly it means “Related to grammar” While the remaining options are incorrect. Hence options (b) is the correct answer.

541

YCT

273.

In the following question, a word has been written in four different ways out of which only one is correctly spelt. Select the correctly spelt word. (a) Grammar (b) Grammer (c) Grammured (d) Grammor SSC MTS 10-10-2017 (Shift-I) Ans. (a) : Out of the given options, option (a) “Grammar” is spelt correctly. While the remaining options are incorrect. Hence option (a) is the correct answer. 274. Choose the correctly spelt word. (a) Genuine (b) Geniune (c) Ganuine (d) Jenuine SSC MTS – 05/08/2019 (Shift-I) Ans. (a) : The correctly spelt word from the given options is (a) 'Genuine' which means- 'real; true; The other options are not correctly spelt. Thus, the correct answer is option (a)'. 275. Select the correctly spelt word. (a) Gratefull (b) Greatfull (c) Greatful (d) Grateful SSC GD – 14/02/2019 (Shift-II) Ans. (d) : The correctly spelt word from the given options (d) 'Grateful' which means- 'feeling or showing thanks to somebody.' The other options are not correctly spelt. Thus, the correct answer is option (d).

H 276. Select the wrongly spelt word (a) gauche (b) fallacious (c) erudite (d) harange SSC CHSL (Tier-I) 17/03/2023 (Shift-III) Ans. (d) : For the above options, option (d) 'harange' is wrongly spelt its correct spelling is 'harangue'. Other options are correct. 277. Select the incorrectly spelt word. (a) Horror (b) Hostil (c) Hospital (d) Huddle SSC CHSL 27.05.2022 Shift-II Ans. (b) : The word 'Hostil' is incorrect because the correct spelling is 'Hostile' which means 'Enemy'. The other words are Horror - phobia Hospital - clinic Huddle - mass / Throng Thus, the correct answer is options (b). 278. Select the incorrectly spelt word. (a) Resolute (b) Hautily (c) Discussion (d) Evidence SSC CHSL (Tier-I) –05/08/2021 (Shift-I) Ans. (b) : The incorrectly spelt word is option (b) 'Hautily' and the correctly spelt word is 'Haughtily' which means- 'unfriendly way and seeming to consider yourself better than other people'. (Proudly) The other options are correctly spelt. Resolute – Determined, Discussion – Debate, Evidence – Witness, Thus, the correct answer is option (b). Misspelt word

279. Select the incorrectly spelt word. (a) Seasonal (b) Hygeinic (c) Gymnastic (d) Celestial SSC CGL (Tier-I) –18/08/2021 (Shift-I) Ans. (b) : The incorrectly spelt word from the given option is (b) 'Hygeinic' The correctly spelt word is – 'Hygienic' which means – 'clean' without the bacteria that cause disease.' The other options are correctly spelt– Seasonal – Topical, Gymnastic – Athletics, Celestial – Astronomic, Thus, the correct answer is option (b). 280. Select the wrongly spelt word. (a) hazard (b) hazy (c) hasitate (d) hasty SSC CGL (Tier-I) – 06/06/2019 (Shift-III) Ans. (c) : The Wrongly spelt word from the given options is (c) 'Hasitate' The correctly spelt word is – 'Hesitate' which means – 'not want to do something because you are not sure that it is right or wrong.' The other options are correctly speltHazard - Danger Hazy – Misty Hasty – Restless Thus, the correct answer is option (c). 281. Select the INCORRECTLY spelt word. (a) humour (b) homage (c) hoarse (d) hinderence SSC CPO-SI – 11/12/2019 (Shift-I) Ans. (d): The incorrectly spelt word from the given options is (d) 'Hinderence' The correctly spelt word is 'Hindrance' which means – 'a person or thing that makes it difficult to do something.' (obstacle) The other options are correctly speltHumour – Jocosity Homage – Tribute Hoarse – Gruff Thus, the correct answer is option (d). 282. Select the correctly spelt word. (a) humillated (b) humilated (c) humelated (d) humiliated SSC CPO-SI – 12/12/2019 (Shift-I) Ans. (d): The correctly spelt word from the given options is (d) 'Humiliated' which means- 'to make somebody feel very embarrassed.' The other options are incorrectly spelt. Thus, the correct answer in option (d). 283. Select the wrongly spelt word. (a) Tranquility (b) Conscientious (c) Hinderance (d) Quarrelled SSC CPO-SI – 13/12/2019 (Shift-I) Ans. (c) : The wrongly spelt word from the given optins is (c) 'Hinderance' The correctly spelt word is 'hindrance' which means – 'a person or thing that makes difficult to do something; (obstacle) The other options are correctly spelt – Tranquility- Serenity Conscientious – Honest Quarrelled – Brabble Thus, the correct answer is option (c).

542

YCT

284. Select the wrongly spelt word. (a) Homoepathy (b) Naturopathy (c) Allopathy (d) Telepathy SSC CHSL (Tier-I) 18/03/2020 (Shift-III) Ans. (a) : The wrongly spelt word from the given options is (a) ' Homoepathy' and the correctly spelt word is 'Homeopathy'. The other options are correctly speltNaturopathy – Natural medicine Allopathy – Heteropathy Telepathy – senses communication Thus, the correct answer is option (a). 285. In the following question, a word has been written in four different ways out of which only one is correctly spelt. Select the correctly spelt word. (a) Heinious (b) Heinies (c) Heinous (d) Hienous SSC MTS 9-10-2017 (Shift-I) Ans. (c) The correctly spelt word from the given options is (c) 'Heinous' which means- 'a person or wrongful act, especially a crim. The other options are incorrectly spelt. Thus, the correct answer is option (c). 286. Select the correctly spelt word. (a) Hijean (b) Higiene (c) Hygene (d) Hygiene SSC MTS – 22/08/2019 (Shift-III) Ans. (d) : The correctly spelt word from the given options is (d) 'Hygiene' which means– 'Sanitation' The other options are incorrectly spelt. Thus, the correct answer is option (d). 287. Choose the correctly spelt word. (a) Hotiness (b) Haughtyness (c) Haughtiness (d) Hautiness SSC MTS – 02/08/2019 (Shift-I) Ans. (c) : The correctly spelt word from the given options is (c) 'Haughtiness' which means – 'the appearance of quality of being arrogantly.' (Pride) The other options are not correctly spelt. Thus, the correct answer is option (c). 288. Select the correctly spelt word. (a) Hiearchy (b) Hierarky (c) Hierarchy (d) Hirarchy SSC GD – 11/02/2019 (Shift-III) Ans. (c) : The correctly spelt word from the given options is (c) 'Hierarchy' which means – 'a system or organization that has many levels from the lowest to the highest'. The other options are incorrectly spelt. Thus, the correct answer is option (c).

Meaning of other options– Supply – Provide, Optical – Ocular, Cares – Concern, 290. Parts of the following sentence have been underlined and given as options. Select the option that contains a spelling error. If more is desired than the instantanious impact on the mind, the advertising value of the poster falls sharply, within limits of outdoor publicity, however it is at present unchallenged. (a) unchallenged (b) instantanious (c) advertising (d) desired SSC CGL (Tier-I) 21/07/2023 (Shift-II) Ans. (b) : The underlined parts of the above given sentence 'instantanious' are incorrectly spelt. The correctly spelt word is 'instantaneous' which means'quickly or prompt.' Other options are correctly spelt. 291. Select the correct spelling of the underlined word in the given sentence. Incidently, I happened to bump into my childhood friend at the airport yesterday. (a) Incidentaly (b) Incidentally (c) Incidantly (d) Incidantally SSC CHSL (Tier-I) 10/08/2023 (Shift-I) Ans. (b) : Option (b) Incidentally – (Desire To say more on a topic or transition into a new subject ) is the correct spelling of the underlined word in the given sentence. Have option (b) is found to have contained immaculate spelling. 292. Parts of the following sentence have been given as options. Select the option that contains an error. You need to acknowledge the itinary and accommodation details of your journey as per the given calendar. (a) acknowledge (b) itinary (c) calendar (d) accommodation SSC CHSL (Tier-I) 02/08/2023 (Shift-I) Ans. (b) : All the above given option are correct except option (b) 'itinary'. The correctly spelt word is 'itinerary' which means –'a detailed plan or route of a journey; Other options are correctly spelt. 293. Select the incorrectly spelt word form the given sentence. The city had a hundred thousand inhabitentssome think double as many. (a) Thousand (b) Hundred (c) Double (d) Inhabitents SSC CHSL (Tier-I) 14/08/2023 (Shift-IV) 289. Choose the incorrectly spelt word. Ans. (d) : In the given sentence, Inhabitants has been (a) insigt (b) Supply incorrectly spelt Its correct spelling will be – Inhabitants – A person who occupies a particular place regularly, (c) Optical (d) Cares SSC MTS 18/05/2023 (Shift-I) routinely Ans. (a) : All the above given options are incorrectly Correct sentence spelt except option (a) ‘insigt’. The correctly spelt word The city had a hundred thousand inhabitants some think double as many. is ‘insight’.

I

Misspelt word

543

YCT

294. Select the wrongly spelt word (a) Innocuous (b) Ingenous (c) Inimical (d) Invulnerable SSC CHSL (Tier-I) 17/03/2023 (Shift-III) Ans. (b) : (b) In the above options, option (b) 'Ingenous' is wrongly spelt its correct spelling is 'Ingenious'. Other option are correct. 295. Choose the sentence with no spelling error. (a) I take this issue very serisly (b) I take this issue vary seriusly (c) I take this issue very seriously (d) I take this isue very seriously SSC CHSL (Tier-I) 09/03/2023 (Shift-IV) Ans. (c) : For the above options, option (c) I take this issue very seriously is correct sentence. Whereas in other options have spelling mistakes. 296. Select the INCORRECTLY spelt word. (a) Inaffectual (b) Authentic (c) Authority (d) Illegitimate SSC Selection Posts XI-30/06/2023 (Shift-IV) Ans. (a) : In the above options, option (a) 'Inaffectual' is wrongly spelt. Its correct spelling is 'Inffectual'. Other options are correct. 297. Select the incorrectly spelt word. (a) Imaculate (b) Impediment (c) Immense (d) Immature SSC Constable GD-07/12/2021 Shift-II Ans. (a) : The word 'Imaculate' is incorrect because the correct spelling is 'Immaculate' which means 'pure'. Impediment - hindrance Immense - Gigantic Immature - undeveloped 298. Select the INCORRECTLY spelt word. (a) Accommodate (b) Intregue (c) Unique (d) Quarantine SSC Constable GD-29/11/2021 Shift-II Ans. (b) : The word 'Intregue' is incorrect because the correct spelling is 'Intrigue' it means 'cabal'. The other words are Accommodate - Adapt Unique - peerless Quarantine - Isolation. 299. Select the correctly spelt word. (a) Innability (b) Inabilty (c) Inability (d) Inablity SSC MTS-12/07/2022 Shift-I Ans. (c) : Out of the given options, only 'Inability' is spelt correctly. While other option are incorrect. Thus, the correct answer is options (c).

301. Select the correctly spelt word. (a) Influnce (b) Influence (c) Influance (d) Influennce SSC MTS-06/07/2022 Shift-I Ans. (b) : The correctly spelt word 'Influence' which means 'impact'. The other options are incorrect. Thus, the correct answer is options (b). 302. From among the words given in bold, select the incorrectly spelt word in the following sentence. And more than the muggings, the waiting to be muggged, fearing the dark that transformed shoddy innocous side streets into giant fangs crouching, springing to demolish this one last reminder. (a) shoddy (b) innocous (c) demolish (d) crouching SSC CHSL 27.05.2022 Shift-I Ans. (b) : The word 'Innocous' is incorrect because the correct spelling is 'Innocuous' which means unharmed. The other words are Shoddy - insignificant Demolish - Bring down Crouching - Bend Thus, the correct answer is option (b). 303. In the following question, four words have been given out of which one word is incorrectly spelt. Select the incorrectly spelt word. (a) Irreprassible (b) Irrefutable (c) Irrevocable (d) Irresistible SSC CGL (Phase-II) 17/02/2018 Ans. (a): The incorrectly spelt word is option (a) 'Irreprassible' The correctly spelt word is 'Irrepressible' which means – 'Stable/Stagnant' The other options are correctly speltIrrefutable – Undeniable Irrevocable – Unaltered' Irresistible – Tireless Thus, the correct answer is option (a). 304. Identify the word that is misspelt. (a) Immediate (b) Inosense (c) Special (d) Irrelevant SSC Sel. Post Phase-VIII (H.L.) 09.11.2020 (Shift-I)

Ans. (b) : The incorrectly spelt word from the given options is (b) 'Inosense' The correctly spelt word is – 'Innocence' which means– 'the fact of not being guilty of a crime, (Guiltlessness)'. The other options are correctly spelt – Immediate – Instant Special – Peculiar Irrelevant – absurd Thus, the correct answer is option (b). 300. Select the correctly spelt word. 305. Select the misspelled word. (a) Institute (b) Insitute (a) irritible (b) enmity (c) Instute (d) Insetute (c) gallery (d) earnest SSC MTS-08/07/2022 Shift-I SSC CPO-SI – 25/11/2020 (Shift-II) Ans. (a) : Out of the given options, only 'Institute' is Ans. (a) : The misspelt word is option (a) 'Irritible' spelt correctly. It means 'organization'. The correctly spelt word is ' Irritable' which means'Fastidious'. Thus, the correct answer is options (a). Misspelt word

544

YCT

The other options are correctly speltEnmity – Hostility Gallery – Aisle Earnest – Honest Thus, the correct answer is option (a). 306. Select the misspelled word. (a) immediately (b) virtually (c) infinitly (d) eventually SSC CPO-SI – 23/11/2020 (Shift-II) Ans. (c) : The incorrectly spelt word is option (c) 'Infinitly' The correctly spelt word is 'Infinitely' which means – 'very much'. The other options are correctly spelt – Immediately – Quick Virtually – Realy Eventually –Lastly Thus, the correct answer is option (c). 307. Select the INCORRECTLY spelt word. (a) equipment (b) paraphernalia (c) apparatus (d) impliments SSC CPO-SI – 13/12/2019 (Shift-II) Ans. (d) : The incorrectly spelt word is option (d) 'Impliments'. The correctly spelt word is 'Implements' which means – 'Tools/goods'. The other options are correctly spelt – Equipment – Appliance Paraphernalia – Equipment Apparatus – stuff Thus, the correct answer is option (d). 308. Select the wrongly spelt word. (a) Inefficent (b) Independence (c) Influence (d) Inequality SSC CHSL (Tier-I) –03/07/2019 (Shift-I) Ans. (a): The incorrectly spelt word is option (a) 'Inefficent'. The correcttly spelt word is 'Inefficient' which means'not working or producing results in the best way.' The other options are correctly spelt – Independence – Freedom Influence – Impact Inequality – Inconformity Thus, the correct answer is options (a). 309. Select the word with the correct spelling. (a) Impresionable (b) Propencity (c) Impeccable (d) Suceptible SSC CHSL (Tier-I) 18/03/2020 (Shift-II) Ans. (c) : The correctly spelt word from the given option is (c) 'Impeccable' which means- 'without any mistakes (Errorless).' The other correct options are – Impressionable – affectable, propensity – Tendency Susceptible – Emotional Thus, the correct answer is option (c). 310. Select the INCORRECTLY spelt word. (a) Inadvertent (b) Imigration (c) Unsurpassable (d) Equivalent SSC CHSL (Tier-I) 19/10/2020 (Shift-I) Misspelt word

Ans. (b) : The incorrectly spelt word is option (b) 'Imigration.' the correctly spelt word is 'Immigration' which means – 'the process of coming to live permanently in a country that is not your own.' The other options are correctly spelt – Inadvertent – Heedless Unsurpassable – Unique, Equivalent – Equal / Compeer Thus, the correct answer is option (b). 311. Select the INCORRECTLY spelt word. (a) Appreciation (b) Escalation (c) Irritable (d) Irational SSC CHSL (Tier-I) 15/10/2020 (Shift-III) Ans. (d) : The incorrectly spelt is option (d) 'Irational' the correctly spelt word is 'Irrotional' which means'not based on reason or clear thought.' The correctly spelt options areAppreciation – Gratitude, Escalation – Exacerbation Irritable – Jumpy Thus, the correct answer is option (d). 312. Select the INCORRECTLY spelt word. (a) implicatted (b) pressurised (c) coercion (d) suppression SSC CHSL (Tier-I) 15/10/2020 (Shift-III) Ans. (a) : The incorrectly spelt word is option (a) 'Implicatted' The correctly spelt word is 'Implicated' which means – 'Decoy/Draw in' The other options are correctly spelt – Pressurised – compressed Coercion – Outrage/Rape Suppression – Subjugation Thus, the correct answer is option (a). 313. Select the wrongly spelt word. (a) Interfear (b) Electorate (c) Acrimony (d) Selection SSC CHSL (Tier-I) 15/10/2020 (Shift-II) Ans. (a) : The wrongly spelt word is option (a) 'Interfear; The correctly spelt word is 'Interfere' which means- 'to prevent something from succeeding.' (Intervene) The other options are correctly speltElectorate – Constituency Acrimony – sance Selection – choice Thus, the correct answer is option (a). 314. Select the INCORRECTLY spelt word. (a) Irregular (b) Irradicate (c) Irreverent (d) Irrational SSC CHSL (Tier-I) 19/10/2020 (Shift-III) Ans. (b) : The incrrectly spelt word is option (b) 'Irradicate' The correctly spelt word is 'Eradicate' which means – 'to destroy or get rid of something completely.' The other options are correctly spelt – Irregular – Desultory Irreverent – Degrading Irrational – Inanimate Thus, the correct answer is option (b). 315. Select the INCORRECTLY spelt word. (a) Irritable (b) Irrilegious (c) Irrigate (d) Irrelevant SSC CHSL (Tier-I) 19/10/2020 (Shift-II)

545

YCT

Ans. (b) : The incorrectly spelt word is option (b) 'Irrilegious'. The correctly spelt word is 'Irreligious' which means'indifferent or hostile to religion.' (Atheist) The other options are correctly speltIrritable – jittery Irrigate – Sparge/water Irrelevant – Extraneous Thus, the correct answer is option (b). 316. Select the INCORRECTLY spelt word. (a) Separate (b) Symphony (c) Marriage (d) Intriging SSC CHSL (Tier-I) 14/10/2020 (Shift-II) Ans. (d) : The incorrectly spelt word from the given options is (d) 'Intriging' The correct spelling is 'Intriguing' which means – 'to make some body very interested and wanting to know more.' (Attractive) The other options are correctly speltSeparate – Distinct Symphony – Harmony Marriage – Nuptials Thus, the correct answer is option (d). 317. Select the correctly spelt word. (a) Intention (b) Transishion (c) Fassion (d) Injecshun SSC CHSL (Tier-I) 13/10/2020 (Shift-II) Ans. (a) : The correctly spelt word is option (a) 'Intention' which means – 'what somebody intends or means to do; (motive)'. The other correct options are – Transition – Epidemic, Fashion – Twig, Injection – The act of inject Thus, the correct answer is option (a). 318. Select the correctly spelt word. (a) Independent (b) Independant (c) Indipendent (d) Indepindent SSC MTS – 08/08/2019 (Shift-III) Ans. (a) : The correctly spelt word from the given option is (a) 'Independent' which means – 'free from and not controlled by another person, country etc. The other options are not correctly spelt. Thus, the correct answer is option (a). 319. Choose the correctly spelt word. (a) Exausted (b) Intelligent (c) Equaly (d) Amigrate SSC MTS – 05/08/2019 (Shift-I) Ans. (b) : The correctly spelt word is option (b) 'Intelligent' which means – 'having or showing the ability to understand.' (wise) The other correct options are – Exhausted – tired, Equally – Uniformly, Emigrate – Migrate Thus, the correct answer is option (b). 320. Select the correctly spelt word. (a) Inqusitive (b) Inqustive (c) Inqisitive (d) Inquisitive SSC MTS – 21/08/2019 (Shift-III) Misspelt word

Ans. (d) : The correctly spelt word from the given options is (d) 'Inquisitive' which means – 'too interested in finding out, about what other people are doing' (Curious) The other options are incorrectly spelt. Thus, the correct answer is option (d). 321. Select the correctly spelt word. (a) Individual (b) Indivisual (c) Indevidual (d) Indeviduel SSC MTS – 21/08/2019 (Shift-I) Ans. (a): The correctly spelt word from the given options is (a) 'Individual' which means – 'considered separately rather than as part of a group.' (Personal) The other options are incorrectly spelt. Thus, the correct answer is option (a). 322. Select the correctly spelt word. (a) Infrastracture (b) Infrastructure (c) Imfrastructure (d) Infrastructur SSC GD – 02/03/2019 (Shift-I) Ans. (b) : The correctly spelt word from the given options is (b) 'Infrastructure' which means – 'the basic systems and services that are necessary for a country or an organization.' The other options are incorrectly spelt. Thus, the correct answer is option (b).

J 323. Choose the incorrectly spelt word. (a) Helpful (b) Energetic (c) Juorney (d) Possible SSC MTS 08/05/2023 (Shift-III) Ans. (c) : The most appropriate answer is option (c) because the incorrectly spelt word is 'Journey' The correct spelt word 'Journey' which means 'travel'. 324. Select the INCORRECTLY spelt word. (a) jumble (b) junior (c) journy (d) jovial SSC CPO-SI – 09/12/2019 (Shift-I) Ans. (c): The incorrectly spelt word from the given option is (c) 'Journy' The correctly spelt word is 'Journey' which means – 'Travel'. The other options are correctly speltJumble - Chaos, Junior – Inferiar, Jovial – Glad/Happy Thus, the correct answer is option (c). 325. Select the incorrectly spelt word. (a) Luxury (b) Jugler (c) Boundary (d) Elementary SSC CGL (Phase-II) 17/02/2018 Ans. (b) : The incorrectly spelt word from the given option is (b) 'Jugler' The correctly spelt is 'Juggler' which means– 'Sorcerer / Warlock' The other options are correctly spelt – Luxury – Pleasure, Boundary – Limit/Range, Elementary – Primitive Thus, the correct answer is option (b).

546

YCT

326. Select the incorrectly spelt word. (a) Jelous (b) Labour (c) Leisure (d) Measure SSC CPO-SI – 25/11/2020 (Shift-II) Ans. (a) : The incorrectly spelt word from the given opions is (a) 'Jelous'. The correctly spelt word is 'Jealous' which means- 'Envious, Green-eyed' The other options are correctly speltLabour – Exertion, Leisure – Vacation, Measure – Gauge Thus, the correct answer is option (a). 327. Select the incorrectly spelt word. (a) Endurance (b) Elite (c) Juniar (d) Tolerance SSC CHSL (Tier-I) 13/10/2020 (Shift-II) Ans. (c) : The incorrectly spelt word form the given options is (c) 'juniar'. The correctly spelt word is 'junior' which means – 'having a low or lower position in an organization.' The other options are correctly speltEndurance – Hardiment, Elite – Noble, Tolerance – Patience Thus, the correct answer is option (c).

K 328. Choose the correctly spelt word. (a) knowledge (b) Patner (c) Lisence (d) Neice SSC MTS – 06/08/2019 (Shift-II) Ans. (a) : The correctly spalt word is option (a) 'Knowledge' which means – 'Understanding or skill gained by experience.' The other correct options are – Partner – Fellow, Licence – Rights, Niece – The daughter of your brothre or sister, Thus, the correct answer is option (a). 329. Select the correctly spelt word. (a) Knowlege (b) Knowledge (c) Knoledge (d) Knowdedg SSC GD – 02/03/2019 (Shift-I) Ans. (b) : The correctly spelt word is option (b) 'knowledge' which means – 'Understanding or skill gained by experience.' The other options are incorrectly speltThus, the correct answer is option (b). 330. Select the incorrectly spelt word. (a) Knav (b) Stop (c) Eager (d) Leisure SSC CHSL (Tier-I) 26/10/2020 (Shift-II) Ans. (a) : The incorrectly spelt word from the given options is (a) 'knav'. The correctly spelt word is 'knave' which means – ' a dishonest or shifty man'. The other options are correctly speltStop – Gorge, Eager – Intent, Leisure – Recess Thus, the correct answer is option (a). Misspelt word

331. Select the incorrectly spelt word. (a) Rogue (b) Blunt (c) Knak (d) Dull SSC CPO-SI – 24/11/2020 (Shift-I) Ans. (c) : The incorrectly spelt word from the given options is (c) 'knak'. The correctly spelt word is 'knack' which means – 'skill or ability to do something difficult that you gave naturally or you can learn.' (Habit) The other options are correctly speltRogue – Cunning, Blunt – Obtuse, Dull – Sluggard Thus, the correct answer is option (c).

L 332. Parts of the following sentence have been underlined and given as options. Select the option that contains a spelling error. Most hobbies are to some extent constructive, but that they may be useful is of secondary significance, and that they may be lucreative is a minor consideration. (a) constructive (b) consideration (c) significance (d) lucreative SSC CGL (Tier-I) 19/07/2023 (Shift-IV) Ans. (d) : In the above given underlined words are correct except the word in option (d) 'Lucreative' because the correct spelt is 'Lucrative' which means 'Profitable.' Thus, the correct answer is option (d). 333. Select the option with the correct spelling that can replace the underlined word in the given sentence. Kasturi acts as a liason between patients and staff. (a) liaisan (b) laision (c) liaison (d) liaision SSC CHSL (Tier-I) 02/08/2023 (Shift-I) Ans. (c) : The underlined word 'liaison' is incorrectly spelt. The correctly spelt word is option (c) 'liaison' which means ' a communication between two or more people or groups that together; The other options are incorrectly spelt. 334. Select the incorrectly spelt word. (a) Judicial (b) Zealous (c) laxuryous (d) Innocent SSC CHSL 24.05.2022 Shift-III Ans. (c) : 'Laxuryous' is incorrect because the correct spelling is 'Luxurious' which means 'Epicure'. The other words areJudicial – Juridical, Zealous – enthusiastic, Innocent – gullible, Thus, the correct answer is options (c). 335. Select the incorrectly spelt word. (a) Licenced (b) Aesthetic (c) Haphazard (d) Chandelier SSC CGL (Tier-I) –24/08/2021 (Shift-I) Ans. (a) : The incorrectly spelt word from the given options is (a) 'Licenced'. The correctly spelt word is 'Licensed' which means – 'to give someone official permission to do or have something.'

547

YCT

The other options are correctly speltHaphazard – Disorganized, Aesthetic – Elegant, Chandelier – Pendant Thus, the correct answer is option (a). 336. Select the wrongly spelt word. (a) boundary (b) olfactory (c) elementary (d) laboratary SSC CPO-SI – 25/11/2020 (Shift-I) Ans. (d) : The incorrectly spelt word from the given options is (d) 'Laboratary'. The correctly spelt word is 'Laboratory' which means – 'a room or building that is used for scientific research, testing, experiments etc.' The other options are correctly speltBoundary – Extent, Olfactory – Aromatic, Elementary –seminal Thus, the correct answer is option (d). 337. Select the correctly spelt word. (a) Roberry (b) Leisure (c) Restaurent (d) Meazure SSC CGL (Tier-I) – 12/06/2019 (Shift-II) Ans. (b): The correctly spelt word from the given options is (b) 'Leisure' which means- 'the time when ' you do not have to work; free time.' The other options are incorrectly spelt, the correctly spelt word is – Robbery- Mugging, Restaurant – Mess, Measure – Scale, Thus, the correct answer is option (b). 338. Select the correctly spelt word. (a) luxery (b) luxry (c) lugzury (d) luxury SSC CGL (Tier-I) – 12/06/2019 (Shift-III) Ans. (d) : The correctly spelt word from the given options is (d) 'Luxury' which means 'a very comfortable and enjoyable situation.' The other options are incorrectly spelt. Thus, the correct answer is option (d). 339. Select the INCORRECTLY spelt word. (a) laboratery (b) labour (c) lantern (d) literature SSC CPO-SI – 09/12/2019 (Shift-I) Ans. (a): The incorrectly spelt word from the given options is (a) 'Laboratery'. The correctly spelt word is 'laboratory' which means – 'a room or building that is used for scientific research, testing, experiments etc.' The other options are correctly speltLabour – Effort, Lantern – Taillight, Literature – Letter, Thus, the correct answer is options (a). 340. Select the wrongly spelt word. (a) Palliative (b) Representative (c) Lucretive (d) Productive SSC CHSL (Tier-I) 19/03/2020 (Shift-III) Ans. (c) : The incorrectly spelt word from the given options is (c) 'Lucretive'. The correctly spelt word is 'Lucrative' which means – 'allowing somebody to earn a lot of money.' The other options are correctly speltPalliative – Abator, Representative – Delegate, Productive – Fertile Thus, the correct answer is option (c). Misspelt word

341. Select the INCORRECTLY spelt word. (a) Syllabi (b) Aggravate (c) Zoologist (d) Luxurient SSC CHSL (Tier-I) 26/10/2020 (Shift-II) Ans. (d) : The incorrectly spelt word from the given option is (d) 'Luxurient'. The correctly spelt word is – 'Luxuriant' which means – 'Joyful/Copious'. The other options are correctly spelt– Syllabi – Briefs, Aggravate – Provoke, Zoologist –a person who scientifically studies animals, Thus, the correct answer is option (d), 342. Select the INCORRECTLY spelt word. (a) Liason (b) Magisterial (c) Maintain (d) Jurist SSC CHSL (Tier-I) 26/10/2020 (Shift-II) Ans. (a) : The incorrectly spelt word from the given option is (a) 'Liason'. The correctly spelt word is 'Liaison' which means- ' communication between two or more people or groups that work together'. The other options are correctly speltMagisterial – Judicial, Maintain – Preserve, jurist – a person who is an expert in law, Thus, the correct answer is option (a). 343. In the following question, a word has been written in four different ways out of which only one is correctly spelt. Select the correctly spelt word. (a) Livaeble (b) Lieivable (c) Livable (d) Liveble SSC MTS 9-10-2017 (Shift-II) Ans. (c) : The correctly spelt word from the given optins is (c) 'Livable' wich means – 'Suitable to live in.' The other options are not correctly spelt. hence, the correct answer is option (c). 344. Select the correctly spelt word. (a) Luminescent (b) Luminscent (c) Luminscent (d) Luminascent SSC MTS – 20/08/2019 (Shift-III) Ans. (a) : The correctly spelt word from the given options is (a) 'Luminescent' which means – 'emission of light by certain materials when they are relatively cool. The other options are incorrectly spelt. Thus, the correct answer is option (a). 345. Select the correctly spelt word. (a) Lubrisent (b) Lubricant (c) Bubrecant (d) Lubricent SSC MTS – 09/08/2019 (Shift-II) Ans. (b) : The correctly spelt word from the given options is (b) 'Lubricant' which means – ' a substance (such as grease) capable of reducing friction, heat. and wear.' The other options are incorrectly spelt. Thus, the correct answer is option (b).

M 346. Select the INCORRECTLY spelt word. (a) Spiteful (b) Rotational (c) Magnificient (d) Debatable SSC GD 27/01/2023 (Shift-I)

548

YCT

Ans. (c) : Among the given options, option (c) Magnificient is incorrectly spelt. It's correct spelling is 'Magnificent' (Splendid). Meaning of the other optionsSpiteful Malicious Rotational Circulating Debatable Disputable 347. Select the INCORRECTLY spelt word. (a) Anticipate (b) Calculate (c) Menipulate (d) Instigate SSC GD 06/02/2023 (Shift-III) Ans. (c) : 'Menipulate' is the incorrectly spelt word 'Manipulate' is the correct word which means :influence or control shrewdly or deviously. Hence options (c) is correct answer. 348. Select the INCORRECTLY spelt world. (a) Naturally (b) Laundry (c) Maintenance (d) Jeweller SSC GD 01/02/2023 (Shift-II) Ans. (c) : In the given options, The spelling of 'Maintainace' is incorrect. The correct spelling is 'Maintenance' which means :the act of keeping something in good condition by checking or repairing it regular. The meaning of other options is– Naturally :- normal manner Laundry :- washing, washhouse. Jeweller :- someone who makes jewelry. 349. Select the INCORRECTLY spelt word. (a) Miscelleneous (b) Misalliance (c) Miscreant (d) Misrepresent SSC Constable GD-18/11/2021 Shift-I Ans. (a) : The word 'Miscellenous' is incorrect because the correct spelling is 'Miscellaneous' which means 'various'. Thus, the correct answer is options (a). 350. Select the option that corrects the misspelt words in the given sentence. Surely, he now has earned the repsect 'mastr'. (a) Surely, he now has earned the 'muster'. (b) Surely, he now has earned the 'masterr'. (c) Surely, he now has earned the 'masteer'. (d) Surely, he now has earned the respect 'master'. SSC MTS-05/07/2022 Shift-I Ans. (d) : The correct spelling of 'respect and master' is given in the sentence (d). While the remaining options have incorrect spelling. Hence option (d) is the correct answer. Correct Sentence– Surely, he now has earned the respect 'master'. 351. Select the correctly spelt word. (a) Mechenism (b) Mechanisum (c) Mechanism (d) Machanism SSC Constable GD-23/11/2021 Shift-III Ans. (c) : Out of the given words, only 'Mechanism' is spelt correctly. "While other options are incorrect. Thus, the correct answer is options (c). Misspelt word

352. Select the INCORRECTLY spelt word. (a) Hover (b) Monark (c) Adorn (d) Dreary SSC Constable GD-24/11/2021 Shift-II Ans. (b) : The word 'Monark' is incorrect because the correct spelling is 'Monarch' it means 'king'. The other words are Hover - Haunt Adorn - Decorate Dreary - Deserted Thus, the correct answer is option (b). 353. Select the correctly spelt word. (a) Maintenance (b) Refree (c) Restaraunt (d) Millienum SSC Constable GD-26/11/2021 Shift-I Ans. (a) : Out of the given options only 'Maintenance' which means 'conservation'. The other words are Referee - umpire Restaurant - porterhouse Millennium - one thousand years Thus the correct answer is options (a). 354. Select the incorrectly spelt word. (a) Acquaint (b) Guarantee (c) Monotonous (d) Mamorial SSC CHSL 26.05.2022 Shift-II Ans. (d) : The word 'Mamorial' is incorrect because the correct spelling 'memorial' which means 'Monument'. Which means 'Monument'. The other words are Acquaint - Familiarize Guarantee - Warranty Monotonous - Prosaic Thus, the correct answer is options (d). 355. Select the correctly spelt word. (a) momentery (b) mommentary (c) momentory (d) momentary SSC CPO-SI – 24/11/2020 (Shift-I) Ans. (d) : The correctly spelt word from the given options is (d) 'Momentary' which means – 'lasting for a very short time.' The other options are incorrectly spelt. Hence, the correct answer is option (d). 356. Select the misspelled word. (a) magnificient (b) mediocre (c) millionaire (d) miraculous SSC CPO-SI – 25/11/2020 (Shift-II) Ans. (a) : The incorrectly spelt word from the given options is (a) 'Magnificient.' The correctly spelt word is 'Magnificent' which means – 'extremely impressive and attractive.' The other options are correclty speltMediocre – Normal, Millionaire – a very rich person, Miraculous – Prodigious Thus, the correct answer is option (a). 357. Select the correctly spelt word. (a) mantion (b) mentoin (c) mension (d) mention SSC CGL (Tier-I) – 13/06/2019 (Shift-II)

549

YCT

Ans. (d) : The correctly spelt word from the given options is (d) 'Mention' which means – 'to say or write something about somebody.' The other given options are incorrectly spelt. Thus, the correct answer is option (d). 358. Select the correctly spelt word. (a) manipulate (b) menipulate (c) manepulate (d) manipulete SSC CGL (Tier-I) – 11/06/2019 (Shift-III) Ans. (a) : The correctly spelt word from the given options is (a)' Manipulate' which means – 'to control or influence somebody/something, often in a dishonest way so that they do not realize it .' The other options are incorreclty spelt. Thus, the correct answer is option (a). 359. Select the correctly spelt word. (a) maintainence (b) maintenence (c) maintenance (d) mentainance SSC CGL (Tier-I) – 10/06/2019 (Shift-I) Ans. (c): The correctly spelt word from the given options is (c) 'Maintenance' which means – 'keeping something in good condition.' The other given options are incorrectly spelt. Thus, the correct answer is option (c) 360. Select the wrongly spelt word. (a) megre (b) meadow (c) matter (d) measure SSC CGL (Tier-I) – 06/06/2019 (Shift-II) Ans. (a): The incorrectly spelt word from the given options is (a) 'Megre'. The correctly spelt word is 'Meagre' which means – 'very small or not enough.' The other options are correctly spelt – Meadow – Grassland, Matter – Affair Measure – Scale. Thus, the correct answer is option (a). 361. Select the INCORRECTLY spelt word. (a) ridiculous (b) mischivious (c) humorous (d) mysterious SSC CPO-SI – 09/12/2019 (Shift-I) Ans. (b): The incorrectly spelt word from the given options is (b) 'Mischivious'. The correctly spelt word is 'Mischievous' which means – 'Rogue/Wicked. The other options are correctly spelt Ridiculous- laughable, Humorous – Comic, Mysterious – Occult, Thus, the correct answer is option (b). 362. Select the correctly spelt word. (a) Manegeable (b) Managable (c) Manageble (d) Manageable SSC CHSL (Tier-I) –11/07/2019 (Shift-I) Ans. (d) The correctly spelt word from the given options is (d) 'Manageable' which means –' capable of being managed.' (Administrable) The other options are incorrectly spelt. Thus, the correct answer is option (d). 363. Select the INCORRECTLY spelt word. (a) Immediate (b) Amass (c) Messanger (d) Massage SSC CHSL (Tier-I) 16/10/2020 (Shift-I) Misspelt word

Ans. (c) : The incorrectly spelt word is option (c) 'Messanger' The correctly spelt word is 'Messenger' which means – 'a preson who carries the message.' The other options are correctly speltImmediate – Instant, Amass – collect, Massage – Friction Thus, the correct answer is option (c). 364. Select the wrongly spelt word. (a) Eradication (b) Manipulation (c) Menifestation (d) Regularization SSC CHSL (Tier-I) 19/03/2020 (Shift-III) Ans. (c) : All the above given options are correct except option (c) 'Menifestation. the correctly spelt word is 'Manifestation' which means - 'The public display of emotion or feeling, or something theoretical made real. The other options are correctly spelt. 365. Select the word with the INCORRECT spelling. (a) Medetative (b) Appended (c) Apprehansive (d) Pensive SSC CHSL (Tier-I) 18/03/2020 (Shift-II) Ans. (a) : The incorrectly spelt words from the given option is a 'Medetative'. The correctly spelt word is 'Meditative' which means 'relating to or absorbed in meditation or considered thought'. The other options are correctly speltAppended - Enclosed, Apprehensive - Frightened, Pensive - Joyless, Thus, the correct answer is option (a). 366. In the following question, a word has been written in four different ways out of which only one is correctly spelt. Select the correctly spelt word. (a) Micsellaneous (b) Miscelaneous (c) Miscelanious (d) Miscellaneous SSC MTS 11-10-2017 (Shift-III) Ans. (d) : The correctly spelt word, from the given options is miscellaneous which means "Diverse". The other options are incorrectly spelt. Thus, the correct answer is options (d). 367. In the following question, a word has been written in four different ways out of which only one is correctly spelt. Select the correctly spelt word. (a) Maintane (b) Maintain (c) Muntain (d) Maintein SSC MTS 11-10-2017 (Shift-III) Ans. (b) : The correctly spelt word from the given options is 'Maintain' which means - 'to make something continue at the same level standard.' (Sustain) The other options are incorrectly spelt. Thus, the correct answer is option (b). 368. Choose the correctly spelt word. (a) Jelousy (b) Corupt (c) Intrest (d) Miniature SSC MTS – 06/08/2019 (Shift-I) Ans. (d) : The correctly spelt word form the given options is 'miniature' which means - 'a small copy of something which is much larger'.

550

YCT

The other correct options are Jealousy - Envy, corrupt - vicious Interest - zest, Thus, the correct answer is options (d). 369. Select the correctly spelt word. (a) Maintnance (b) Medival (c) Millenium (d) Moustache SSC MTS – 14/08/2019 (Shift-III) Ans. (d) : The correctly spelt word from the given options is (d) 'Moustache' which means - hair that grows on a man's top lip, between the mouth and nose. The other correct options are Maintenance - conservation, medieval-obsolete. Millennium - a period of one thousand years. Thus, the correct answer is options (d). 370. Select the correctly spelt word. (a) Modern (b) Moddern (c) Modrran (d) Modren SSC MTS – 09/08/2019 (Shift-III) Ans. (a) : The correctly spelt word from the given options is (a) 'modern' which means - 'present or recent time.' The other options are incorrectly spelt. Hence, the correct answer is options (a). 371. Select the correctly spelt word. (a) Mathmetics (b) Mathamatics (c) Mathametics (d) Mathematics SSC GD – 18/02/2019 (Shift-II) Ans. (d) : The correctly spelt word from the given options is (d) 'mathematics' which means 'study of numbers, quantities or shapes. The other options are incorrectly speltThus, the correct answer is options (d). 372. Select the correctly spelt word. (a) Milennium (b) Millenium (c) Millenniumm (d) Millennium SSC GD – 11/02/2019 (Shift-II) Ans. (d) : The correctly spelt word from the given options is (d) 'Millennium' which means - 'a period of one thousand years'. The other options are incorrectly spelt. Thus, the correct answer is options (d).

N 373. Choose the incorrectly spelt word. (a) Neweset (b) Drape (c) Vibe (d) Surround SSC MTS 04/05/2023 (Shift-II) Ans. (a) : For the above given options, option (a) 'Neweset' is in correctly spelt because correctly spelt is 'Newest' which means 'Novel',Recent Thus, the correct answer is option (a). 374. Select the INCORRECTLY spelt word. (a) Niece (b) Father (c) Nefew (d) Uncle SSC CGL (Tier-I) 17/07/2023 (Shift-II) Ans. (c) : In the above options, option (c) 'Nefew' is wrongly spelt, its correct spelling is 'nephew' Misspelt word

Other options are correctly spelt. 375. From among the words given in bold, select the incorrectly spelt word in the following sentence. The Imitations of Horace raise issues of political nautrality, partisanship and moral satire and as such are key texts of the Augustan age. (a) satire (b) partisanship (c) Augustan (d) nautrality SSC CHSL 27.05.2022 Shift-III Ans. (d) : The word 'Nautrality' is given in bold letter, in the sentence is incorrect because the correct spelling is 'Neutrality' which means 'Impartiality'. The other words are satire - Innuendo/Irony Partisanship - Discrimination/Bias Augustan - Spacifie age of english Literature. Thus, the correct answer is options (d). 376. Select the most appropriate spelling that can substitute the underlined word in the given sentence. We love to play with our neighboar's children (a) neighbours (b) naighbour's (c) neighboars (d) neighbour's SSC CHSL 30.05.2022 Shift-I Ans. (d) : 'Neighbours' is incorrect because the correct spelling is 'Neighbours'. The correct sentence will be 'We love to play with our neighboar's children. Thus, the correct answer is options (d). 377. Select the wrongly spelt word. (a) Nervous (b) Never (c) Nature (d) Nuetral SSC Sel. Post Phase-VIII (M.L.) 09.11.2020 (Shift-III)

Ans. (d) : The incorrectly spelt word from the given options is (d) 'Nuetral'. The correctly spelt word is 'Neutral' which means - 'not supporting or belonging to either side in an argument, war, etc. Other options are correctly speltNervous - Jittery, never = Not ever, Nature - Ethos, Thus, the correct answer is options (d). 378. Select the misspelt word. (a) ingenuity (b) committee (c) jeopardy (d) neglegible SSC CPO-SI – 24/11/2020 (Shift-II) Ans. (d) : The misspelt word form the given options is (d) 'Neglegible'. The correctly spelt word is 'negligible' which means - 'very small and therefore not important.' The other options are correctly spelt Ingenuity - simplicity, committee - Board, Jeopardy - Menace, Thus, the correct answer is options (d). 379. Select the wrongly spelt word. (a) notification (b) necessity (c) negociation (d) negligence SSC CGL (Tier-I) – 07/06/2019 (Shift-I) Ans. (c): The wrongly spelt word form the given options is (c) 'Negocitation'. The correctly spelt word is

551

YCT

'Negotiation' which means - 'discussions at which people try to decide or agree something'. The other options are correctly spelt Negligence - Untidiness, Necessity - need, Notification - Reportage, Thus, the correct answer is options (c). 380. Select the wrongly spelt word. (a) negligible (b) nuisance (c) noticable (d) neighbouring SSC CGL (Tier-I) – 07/06/2019 (Shift-II) Ans. (c): The wrongly spelt word form the given options is (c) 'Noticable'. The correctly spelt word is 'Noticeable; which means - 'Noteworthy'. The other options are correctly speltNegligible - Potty, Nuisance - obstacle, Neighbouring - Vicinal Thus, the correct answer is options (c). 381. Select the correctly spelt word. (a) necassery (b) nesessary (c) necessary (d) necessery SSC CHSL (Tier-I) –10/07/2019 (Shift-III) Ans. (c): The correctly spelt word from the given options is (c) 'Necessary' which means - 'that is needed for a purpose of a reason'. The other given options are incorrectly spelt. Hence, the correct answer is options (c). 382. Select the wrongly spelt word. (a) Neither (b) Nibble (c) Nightingale (d) Ninteenth SSC CHSL (Tier-I) –04/07/2019 (Shift-I) Ans. (d) The wronghly spelt word from the given options is (d) 'Ninteenth'. The correctly spelt word is 'Nineteenth' which means- 'Nineteen in number'. The other options are correctly speltNeither - Not either, Nibble - Bite off, Nightingale - a small brown bird, Thus, the correct answer is options (d). 383. Select the wrongly spelt word. (a) nerve (b) neglect (c) negative (d) nerveous SSC CHSL (Tier-I) –03/07/2019 (Shift-III) Ans. (d) : The wrongly spelt word from the given options is (d) 'nerveous'. The correctly spelt word is 'Nervous' which means - 'anxious about something or afraid of something'. The other options are correctly speltNerve - vein, Neglect - disregard, Negative - Privative, Thus, the correct options is (d) 384. Select the wrongly spelt word. (a) Molestation (b) Combination (c) Navigetion (d) Condonation SSC CHSL (Tier-I) 19/03/2020 (Shift-I) Ans. (c) : The wrongly spelt word form the given options is (c) 'Navigetion'. The correctly spelt word is 'Navigation' which means - 'the act of directing a ship, aircraft, etc. The other options are correctly spelt Molestation - Aggravation, Misspelt word

Combination - conjunction, Condonation - Exemption, thus, the correct answer is options (c). 385. Choose the correctly spelt word. (a) Noticable (b) Nauticable (c) Noticeable (d) Noteceable SSC MTS – 06/08/2019 (Shift-III) Ans. (c) : The correctly spelt words form the given options is (c) 'Noticeable' which menas - 'easy to see or notice. The other options are incorrectly spelt. Thus, the correct answer is option (c). 386. Select the correctly spelt word. (a) Newclear (b) Nuclere (c) Neuclear (d) Nuclear SSC MTS – 22/08/2019 (Shift-III) Ans. (d) : The correctly spelt word from the given options is (d) 'Nuclear' which means - 'relating to energy from changing atom's structure, relating to central part of atom. The other options are incorrectly spelt. Thus, the correct answer is option (d). 387. Select the correctly spelt word. (a) Ninetteen (b) Nineteen (c) Ninteen (d) Ninneteen SSC MTS – 21/08/2019 (Shift-II) Ans. (b) : The correctly spelt word from the given options is (b) Nineteen', which means - 'next after the eightieth'. The other options are incorrectly spelt. Thus, the correct answer is options (b). 388. Select the correctly spelt word. (a) Nocturenal (b) Nocturnal (c) Nocternal (d) Nocturnel SSC MTS – 20/08/2019 (Shift-II) Ans. (b) : The correctly spelt word from the given options is (b) 'Nocturnal' which means - 'Nightly'. Thus, the correct option is (b). 389. Select the correctly spelt word. (a) Priorety (b) Jelous (c) Disgise (d) Nourish SSC MTS – 14/08/2019 (Shift-I) Ans. (d) : The correctly spelt word form the given options is (d) 'Nourish' which means - ' to provide people or living things with food in order to make them grow and keep them healthy. The other correct options are Priority - Precursory, Jealous - Envious, Disguise - Mummery Thus, the correct answer is options (d). 390. Choose the correctly spelt word. (a) Peirce (b) Necessary (c) Noticeing (d) Permmit SSC MTS – 07/08/2019 (Shift-III) Ans. (b) : The correctly spelt word is 'Necessary' means 'Essential'. The other correctly options arePurse - Wallet Noticing - Attending Permit - Ordain Thus, the correct answer is option (c).

552

YCT

O 391. Identify the INCORRECTLY spelt word from the given options. (a) Sieve (b) Heave (c) Weave (d) Obssess SSC CGL (Tier-I) 24/07/2023 (Shift-III) Ans. (d) : From the above options, option (d) 'obssess' is wrongly spelt. His correct spelling is 'obsess'. Other options are correctly spelt. 392. Select the INCORRECTLY spelt word. (a) Persistent (b) Repetition (c) Overwelming (d) Audacious SSC CHSL (Tier-I) 04/08/2023 (Shift-III) Ans. (c) : All the above given options are corrctly spelt except option (c) 'Overwelming'. The correctly spelt word is 'Overwhelming.' Hence, the correct option is (c). 393. Select the INCORRECTLY spelt word. (a) Submitted (b) Ommitted (c) Committed (d) Remitted SSC Constable GD-13/12/2021 Shift-III Ans. (b) : The word 'Ommitted' is incorrect because the correct spelling is 'Omitted', while other options are correct. Thus, the correct answer is options (b). 394. Select the incorrectly spelt word. (a) Ascetic (b) Boasting (c) Overide (d) Evidence SSC CGL (Tier-I) –24/08/2021 (Shift-I) Ans. (c) : The incorrectly spelt word form the given options is (c) 'overide'. The correctly spelt word is 'Override' which means - 'to prevail or have dominance over; put own. The other options are correctly spelt. Ascetic- Hermit, Boasting - Splurge, Evidence - 'Testimony' Thus, the correct answer is option (c). 395. Select the wrongly spelt word. (a) difficult (b) supporting (c) optsion (d) easier SSC CPO-SI – 23/11/2020 (Shift-I) Ans. (c) : The wrongly spelt word from the given options is (c) 'optsion'. The correctly spelt word is'Option' which means - 'something that you can choose to do'. The other options are correctly spelt Difficult - Arduous, Supporting - Helpful, Easier - comfortable, Thus, the correct answer is option (c). 396. Select the misspelled word. (a) ommission (b) occurred (c) accommodate (d) oppressed SSC CPO-SI – 25/11/2020 (Shift-II) Ans. (a) : The misspelt word from the given options is (a) 'Ommission'. The correctly spelt word is 'Omission' which means - 'Something that has not been included or has not been done.' Misspelt word

The other options are correctly speltOccurred - happen, accommodate Adapt, Oppressed - Aggrieved, Thus, the correct answer is options (a). 397. Select the correctly spelt word. (a) omelette (b) omelete (c) omelate (d) omlate SSC CPO-SI – 12/12/2019 (Shift-I) Ans. (a) The correctly spelt word from the given options is (a) 'Omelette' which means - a dish made of eggs that have been mixed together very fast and fried.' The other options are incorrectly spelt. Thus, the correct answer is options (a). 398. Select the INCORRECTLY spelt word. (a) violence (b) fierce (c) resemblance (d) occassion SSC CPO-SI – 13/12/2019 (Shift-II) Ans. (d) : The incorrectly spelt word from the given options is (d) 'Occassion'. The correctly spelt word is 'Occasion' which means - 'a Particular time when something happens. The other options are correctly spelt Violence - Disaster, Fierce - Frightful, Resemblance - Congruence, Thus, the correct answer is options (d). 399. Select the wrongly spelt word. (a) Opulence (b) Oppression (c) Oppurtunity (d) Opposition SSC CHSL (Tier-I) –11/07/2019 (Shift-II) Ans. (c) The wrongly spelt word from the given options is (c) 'Oppurtunity'. The correctly spelt word is 'Opportunity', which means- a good chance for advancement or progeness. The other options are incorrectly spelt. Thus, the correct answer is options (b). 400. Select the correctly spelt word. (a) oppertunity (b) opportunity (c) oppurtunity (d) oportunity SSC CHSL (Tier-I) –08/07/2019 (Shift-I) Ans. (b) The correctly spelt word from the given options is (b) 'Opportunity' which means - the possibility of doing something'. The othe options are incorrectly spelt. Thus, the correct answer is option (b). 401. Choose the correctly spelt word. (a) obbstacle (b) Obctacle (c) Obstacal (d) Obstacle SSC MTS – 08/08/2019 (Shift-I) Ans. (d) : The correctly spelt word from the given options is (d) 'Obstacle' which means - 'something that stands in the way of progress or achievement; The other options are incorrectly spelt. Thus, the correct answer is option (d). 402. Select the correctly spelt word. (a) Occassion (b) Ommision (c) Occurrence (d) Occupasion SSC MTS – 19/08/2019 (Shift-II) Ans. (c) : The correctly spelt word from the given options is (c) 'Occurrence' which means - 'something that happens or exists'.

553

YCT

The other correct options are Occasion - chance, Omission - Annihilation, Occupation - Remunerative, Thus, the correct options is (c). 403. Select the correctly spelt word. (a) Godess (b) Villian (c) Obedient (d) Truely SSC MTS – 16/08/2019 (Shift-II) Ans. (c) : The correctly spelt word from the given options is (c) 'Obedient' which means - 'doing what you are told to do? The other correct options are Goddess - Deity, villain - scoundrel, Truly - candidly, Thus, the correct options is (c). 404. Choose the correctly spelt word. (a) Knoledge (b) Jealusy (c) Odour (d) Langauge SSC MTS – 07/08/2019 (Shift-III) Ans. (c) : The correctly spelt word from the given options is (c) 'Odour' which means - 'a smell, often one that is unpleasant'. The other correct options are Knowledge - Intellect, Jealousy - Envy, Language - speech, Hence, the correct answer is options (c). 405. Select the correctly spelt word. (a) Occation (b) Occassion (c) Occasion (d) Ocassion SSC GD – 14/02/2019 (Shift-II) Ans. (c) : All the above given options are incorrect except option (c) 'Occasion' which means - 'a particular time, especially when something happens or has happened'. The other given options are incorrectly spelt. Thus, the correct answer is options (c).

P 406. Select the wrongly spelt word. (a) Activity (b) Protesion (c) Inactivity (d) Unshielded SSC MTS 19/05/2023 (Shift-III) Ans. (b) : The above given options are correctly spelt except option (b) ‘protesion’. Its correct spelling is 'protection'. Meaning of other options – Activity – Stir, Agility Inactivity – Indolence, Sloth Unshielded – Defenseless 407. Choose the INCORRECTLY Spelt word. (a) Falcon (b) Tailor (c) Talon (d) Palour SSC GD 31/01/2023 (Shift-II) Ans. (d) : Among the given options, option (d) 'Palour' is incorrectly spet. Its correct spelling is 'parlor'. Meaning of the other optionsFalcon - Hawk Tailor - Dressmaker, Clothier Talon - Hook Misspelt word

408. Select the INCORRECTLY spelt word. (a) Passage (b) Pancake (c) Pasion (d) Modern SSC GD 12/01/2023 (Shift-II) Ans. (c) : Among the given options, option (c) 'Pasion' is incorrectly spelt. Its correct spelling is 'Passion'. Meaning of the other optionsPassage - Path, Route Pancake - Muffin Modern - Recent, Stylish 409. Select the INCORRECTLY spelt word. (a) Maladjusted (b) Stakeholder (c) Permenent (d) Longitude SSC GD 10/01/2023 (Shift-I) Ans. (c) : Among the above options, option (c) 'permenent' is incorrectly spelt. Its correct spelling is 'Permanent' Meaning of the other optionsMaladjusted - Unbalanced, disturbed Stokeholder - Shareholder Longitude - Vertical segment 410. Identity the correct spelling of the underlined word. It is considered a great phenamina in the field of economics. (a) phynamena (b) fenomena (c) phinomina (d) phenomena SSC CHSL (Tier-I) 03/08/2023 (Shift-II) Ans. (d) : In the above sentence, the underlined word 'phenamina' is incorrectly spelt , the Correctly spelt word is 'phenomena' which means 'a fact or an event in nature, especially one that is not fully understood. Other options are not Correct. 411. Choose the correctly spelt word. (a) TEMPORERY (b) CAMOFLAGUE (c) PSEUDONYM (d) TABBOO SSC CHSL (Tier-I) 20/03/2023 (Shift-I) Ans. (c) : In the above sentence option (c) "Pseudonym" is correctly spelt. Other options are not correct. 412. Parts of the following sentence have been underlined and given as options. Select the option that contains a spelling error. In the unscientific age or community, there are official repositories of wisdom, such as Egyptian preists and Tibetan Lamas. (a) repositories (b) Egyptian (c) preists (d) unscientific SSC Selection Posts XI-27/06/2023 (Shift-I) Ans. (c) : In the above sentence, the underlined word 'preists' is incorrectly spelt. The correctly spelt word is 'priests' which means 'a person who performs religious ceremonies in some religions; Other options are correctly spelt. 413. Choose the incorrectly spelt word. (a) Prepaer (b) Mushy (c) Busiest (d) Matter SSC CHSL (Tier-I) 17/03/2023 (Shift-IV)

554

YCT

Ans. (a) : In the above options, option (a) is incorrect spelling. Its correct spelling is 'Prepare'. Other options are correct. 414. Select the INCORRECTLY spelt word. (a) Palace (b) Peddle (c) Pecket (d) Paint SSC Constable GD-01/12/2021 Shift-I Ans. (c) : The word 'Pecket' is incorrect because the correct spelling is 'packet' which means parcel. Thus, the correct answer is options (c). 415. Select the incorrectly spelt word. (a) Private (b) Priviledge (c) Prevalent (d) Previous SSC Constable GD-06/12/2021 Shift-III Ans. (b) : The word 'Previledge' is incorrect because the correct spelling is 'privilege' means 'special rights'. Thus, the correct answer is option (b). 416. Select the incorrectly spelt word. (a) Procastination (b) Alliteration (c) Superintendent (d) Hallucination SSC Constable GD-08/12/2021 Shift-I Ans. (a) : The word 'Procastination' is in correct because the correct spelling is 'procrastination' which means 'Delay'. The other words are Alliteration - A figure of speech superintendent - president Hallucination - Reverie/Phantosm Thus, the correct answer is options (a). 417. Select the INCORRECTLY spelt word. (a) Commitment (b) Bouquet (c) Proceedure (d) Quarantine SSC Constable GD-16/11/2021 Shift-III Ans. (c) : The word 'Proceedure' is incorrect because the correct spelling is 'procedure' which means .process'. The other words areCommitment - Handover Bouquet - Epergne Quarantine - Isolation Thus, the correct answer is options (c). 418. Select the INCORRECTLY spelt word. (a) Recession (b) Remission (c) Precission (d) Omission SSC Constable GD-17/11/2021 Shift-II Ans. (c) : The word 'Precission' is incorrect because the correct spelling is 'Precision' which means 'chastity'. The other words are Recession - Repayment Remission - Forgiveness Omission - Absence Thus, the correct answer is options (c). 419. Select the INCORRECTLY spelt word. (a) Proactivity (b) Pronounciation (c) Procrastination (d) Prolong SSC Constable GD-25/11/2021 Shift-III Misspelt word

Ans. (b) : The word 'Prononuciation' is incorrect because the correct spelling is 'Pronunciation'. Proactivity - preemptive Procrastination - Dither Prolong - Amplify 420. Select the correctly spelt word. (a) Domicille (b) Penninsula (c) Canibal (d) Pharmacy SSC Constable GD-26/11/2021 Shift-I Ans. (d) : Out of the given options, only 'pharmacy' is seplt correct. While other option are incorrect. Thus, the correct answer is options (d). 421. Select the INCORRECTLY spelt word. (a) Defiance (b) Pretanse (c) Reliance (d) Alliance SSC Constable GD-30/11/2021 Shift-III Ans. (b) : The word 'Pretanse' is incorrect because the correct spelling is 'Pretence' which means 'show/veneer'. The other words are – Defiance - Disobedience Reliance - Dependency Alliance - Treaty Thus, the correct answer is options (b). 422. Select the correctly spelt word. (a) Promenant (b) Prominent (c) Promneant (d) Prominant SSC MTS-05/07/2022 Shift-II Ans. (b) : Out of the given options, option (b) 'prominent' is spelt correctly which means 'famous'. While other options are incorrect. Thus, the correct answer is options (b). 423. Select the correctly spelt word. (a) Envisaige (b) Psychologist (c) Emergense (d) Trancition SSC MTS-13/07/2022 Shift-I Ans. (b) : Out of the given options, only 'Psychologist' is spelt correctly. While the remaining options have incorrect spelling. Thus, the correct answer is options (b). 424. Select the INCORRECTLY spelt word. (a) Perterb (b) Perceive (c) Pigeon (d) Passive SSC MTS-05/07/2022 Shift-I Ans. (a) : The word 'perterb' is incorrect because the correct spelling is 'perturb' which means 'stun'. While the remaining options are correct. Thus, the correct answer is option (a) 425. Select the correctly spelt word. (a) Bezarre (b) Infatuaition (c) Fortiunate (d) Perfection SSC MTS-05/07/2022 Shift-III Ans. (d) : Out of the given options, only 'perfection' is spelt correctly. The other options are incorrect. Thus, the correct answer is option (d)

555

YCT

426. Select the sentence which has no spelling error. (a) Dusty is a prodegious novelist of the present era. (b) His business gives him cash in prodigeous amounts. (c) Beethoven is known to be a prodigious musician of all times. (d) Ashley caught the spectators by his prodigiuous tricks. SSC CHSL 27.05.2022 Shift-III Ans. (c) : There is no spelling error in the option (c). The word 'prodigious' is correct it means 'Bizarre' while the remaining options are incorrect. 427. Select the incorrectly spelt word. (a) Believe (b) Conscious (c) Previlege (d) Discipline SSC CHSL 27.05.2022 Shift-III Ans. (c) : The words 'Previlege' is incorrect because the correct spelling is 'privilege' which means special right. The other words are Believe - Presume Conscious - Aware Discipline - Regimentation Thus, the correct answer is options (c). 428. From among the words given in bold, select the INCORRECTLY spelt word in the following sentence. Almost all the governors of Bengal strongly resented the special privileges enjoyed by English company as it meant a huge loss to the provinciel exchequer. (a) provinciel (b) privileges (c) exchequer (d) resented SSC CHSL 26.05.2022 Shift-III Ans. (a) : The word 'provinciel' is incorrect because correct spelling is 'provincial'. which means regional The other words are Privileges - special right Exchequer - Treasury Resented - Take offense/Rage Thus, the correct answer is options (a). 429. Select the sentence which has no spelling error. (a) The pharmaetuical company develops and market drugs. (b) Many phramaeutical companies supplied vaccines in pandemic. (c) My elder brother works in a pharmaceutical company. (d) Geet runs a pharmasutical chemist shop at New Delhi. SSC CHSL 26.05.2022 Shift-I Ans. (c) : The word 'Pharmaceutical' is the correct spelling which is given in the options (c). Thus, correct sentence will be My elder brother works in a pharmaceutical company. 430. Select the wrongly spelt word. (a) Plantain (b) Platinum (c) Plataue (d) Plaque SSC CGL (Tier-II) 08.08.2022 Shift-II Misspelt word

Ans. (c) : The word ' Plataue' is the incorrect spelt word from the given options. The correct spelt is 'plateau' which means a large high area of flat land. The other options are Plantain - A tropical fruit similar to banana with green skin. Plaque - Bladed. Thus, the correct answer is option (c). 431. Select the incorrectly spelt word. (a) Possesions (b) Irritation (c) Invitation (d) Protection SSC CHSL (Tier-I) –05/08/2021 (Shift-I) Ans. (a) : The incorrectly spelt word from the given options is (a) 'Possesions'. The correctly spelt word is 'Possessions' which means - 'something that is owned or possessed by someone'. The other options are correctly speltIrritation - Exasperation, Invitation - Invite Protection - Tutelage, Thus, the correct options, is (a). 432. Select the incorrectly spelt word. (a) Endurance (b) Tolerance (c) Insistence (d) Parsistance SSC CHSL (Tier-I) –09/08/2021 (Shift-I) Ans. (d) : The incorrectly spelt word from the given options is (d) 'Parsistance'. The correctly spelt word is 'Persistence' which means- 'the quality that allows someone to continue doing something'. The other options are correctly speltEndurance - Patience, Tolerance - Durability, Insistence - Exaction, Thus , the correct answer is options (d). 433. Select the incorrectly spelt word. (a) Concrete (b) Elite (c) Petete (d) Secrete SSC CGL (Tier-I) –23/08/2021 (Shift-I) Ans. (c) : The incorrectly spelt word from the given options is (c) 'Petete'. The correctly spelt word is 'Petite' which means - 'relating to women who is small and thin in an attractive way.' (Beautiful) The other options are correctly spelt Concrete - solid, Elite - Noble, Secrete - Hide, Thus, the correct answer is options (c). 434. Select the incorrectly spelt word. (a) Specified (b) Decrease (c) Creature (d) Pieceful SSC CGL (Tier-I) –13/08/2021 (Shift-I) Ans. (d) : The incorrectly spelt word from the given options is (d) 'Pieceful'. the correctly spelt word is 'Peaceful' which means - 'Tranquil/Calm'. The other options are correctly speltSpecified - Extensive, Decrease - Reduction, Creature - Being, Thus, the correct answer is options (d). 435. Select the word with the correct spelling. (a) permmited (b) plesent (c) pungant (d) punctuation SSC CPO-SI (Tier-II) 27/09/2019 (3 pm - 5 pm)

556

YCT

Ans. (d) : The correctly spelt word from the given options is (d) 'Punctuation' which means - the marks used in writing to separate the sentences and phrases. The other correct options are Permitted - Allow, Pleasant- Charming Pungent - Bitter, Thus, the correct answer is options (d). 436. In the following question, four words have been given out of which one word is incorrectly spelt. Select the incorrectly spelt word. (a) Declension (b) Labyrinth (c) Quintessence (d) Pandemonious SSC CGL (Phase-II) 17/02/2018 Ans. (d): The incorrectly spelt word from the given options is (d) 'Pandemonuous'. The correctly spelt word is 'Pandemonium' which means - 'any place or scene of wild disorder, noise, or confusion'. The other options are correctly spelt Declension - Decadence, Labyrinth - Maza Quintessence - Summary, Thus, the correct answer is options (d). 437. Select the wrongly spelt word. (a) central (b) electric (c) presure (d) irrigation SSC CPO-SI – 23/11/2020 (Shift-I) Ans. (c) : The wrongly spelt word from the given options is (c) 'Presure'. The correctly spelt word is 'Pressure' which means - the action of a force against an opposing force. (Stress) The other options are correctly spelt Central - Pivotal, Electric - Power, Irrigation - Watering, Thus, the correct answer is options (c). 438. Select the correct spelt word. (a) propotion (b) proportion (c) preportion (d) proporsion SSC CGL (Tier-I) – 12/06/2019 (Shift-I) Ans. (b): The correctly spelt word from the options is (b) 'Proportion' which means a part or share of a whole'. The other options are incorrectly spelt. Thus, the correct answer is option (b). 439. Select the correctly spelt word. (a) peivailege (b) previledge (c) previllage (d) privilege SSC CGL (Tier-I) – 11/06/2019 (Shift-I) Ans. (d) : The correctly spelt word from the given options is (d) 'privilege' which means - ' a special right or advantage that only one person or group has'. The other options are incorrectly spelt. Thus, the correct answer is option (d). 440. Select the correct spelt word. (a) perticular (b) particuler (c) particular (d) perticuler SSC CGL (Tier-I) – 10/06/2019 (Shift-III) Ans. (c) : The correctly spelt word from the given options is (c) 'Particular' which means - 'used to emphasize that you are talking about one person, thing time, etc. The other options are incorrectly spelt. Thus, the correct answer is options (c). Misspelt word

441. Select the INCORRECTLY spelt word. (a) puddel (b) pigeon (c) pillion (d) pioneer SSC CPO-SI – 09/12/2019 (Shift-I) Ans. (a): The incorrectly spelt word from the given options is (a) 'Puddel'. The correctly spelt word is 'Puddle' which means - 'a very small pool of usually dirty or muddy water'. The other options are correctly speltPillion - Back seat Pigeon - A bird Pioneer - Precursor, Thus, the correct answer is options (a) 442. Select the wrongly spelt word (a) presidential (b) preferential (c) profeslonal (d) proverbial SSC CHSL (Tier-I) –09/07/2019 (Shift-II) Ans. (c): The wrongly spelt word from the given options is (c) 'Profeslonal'. The correctly spelt word is 'Professional' which means - 'connected with a job that needs a high level of training or education'. The other option are correctly speltThus, the correct answer is options (c). 443. Select the correctly spelt word. (a) persue (b) persoue (c) pursue (d) purseue SSC CHSL (Tier-I) –08/07/2019 (Shift-II) Ans. (c): The correctly spelt word from the given options is (c) 'Pursue' which means - 'to follow somebody in order to catch him'. The other options are incorrectly spelt. Thus, the correct answer is options (c). 444. Select the wrongly spelt word. (a) Migrant (b) Precedent (c) Permanant (d) Adamant SSC CHSL (Tier-I) 26/10/2020 (Shift-II) Ans. (c) : The wrongly spelt word from the given options is (c) 'Permanant'. The correctly spelt word is 'Permanent' which means - 'lasting for a long time or forever, that will not change'. The other options are correctly spelt Migrant - Expatriate, Precedent - Case law, Adamant - corneous, Thus, the correct answer is options (c). 445. Select the INCORRECTLY spelt word. (a) Alligator (b) Privilage (c) Alimony (d) Judicious SSC CHSL (Tier-I) 16/10/2020 (Shift-III) Ans. (b) : The incorrectly spelt word from the given options is (b) 'Privilage'. The correctly spelt word is 'Privilege' which means - 'a special right or advantage that only one person or group has'. The other options are correctly speltAlligator - Crocodile, Alimony - Sustenance, Judicious - Right, Thus, the correct answer is options (b). 446. Select the INCORRECTLY spelt word. (a) Preamble (b) Miscellaneous (c) Deviation (d) Pnemonia SSC CHSL (Tier-I) 21/10/2020 (Shift-II)

557

YCT

Ans. (d) : The incorrectly spelt word from the given option is (d) 'Pnemonia'. The correctly spelt word is 'Pneumonia' which means - 'an infection that affects one or both lungs'. The other options are correctly speltPreamble - Preface, Deviation - Departure, Miscellaneous - diverse, Thus, the correct answer is options (d). 447. Select the word with the correct spelling. (a) Vizibility (b) Certanity (c) Piety (d) Notoreity SSC CHSL (Tier-I) 17/03/2020 (Shift-II) Ans. (c) : The correctly spelt word from the given options is (c) 'piety' which means - a way of behaving that shows a deep respect for God and religion'. The other correct options are Visibility - Pretense, Certainty - Certitude, Notoriety - Infamy, Thus, the correct answer is options (c). 448. Select the wrongly spelt word. (a) Profuse (b) Protrude (c) Profesional (d) Professor SSC CHSL (Tier-I) 26/10/2020 (Shift-I) Ans. (c) : The wrongly spelt word from the given, options is (c) 'Profesional'. The correctly spelt word is 'Professional' which means - 'connected with a job that needs a high level of training or education'. The other options are correctly spelt Profuse - Abundant, Protrude - Expand, Professor - Instructor, Thus, the correct answer is options (c). 449. Select the wrongly spelt word. (a) Privilige (b) Mileage (c) Gait (d) Truism SSC CHSL (Tier-I) 21/10/2020 (Shift-III) Ans. (a) : The wrongly spelt words is options (a) 'Privilige'. The correctly spelt word is 'Privilege' which means - 'a special right or advantage that only one person or group has. The other options are correctly speltMileage - Distance/Advantage, Gait - Demeanor/Speed, Truism - True statement Thus, the correctly answer is options (a). 450. Select the INCORRECTLY spelt word. (a) Boutique (b) Lacquer (c) Placque (d) Rouge SSC CHSL (Tier-I) 13/10/2020 (Shift-II) Ans. (c) : The incorrectly spelt word from the given options is (c) 'Placque'. The correctly spelt word is 'plaque' which means - 'a flat thin piece (as of metal) used for decoration'. The other options are correctly speltBoutique - Retrospect, Lacquer - shellack, Rouge - Chrism, Thus, the correct answer is option (c). 451. Select the wrongly spelt word. (a) Consequence (b) Fixation (c) Musician (d) Providance SSC CHSL (Tier-I) 12/10/2020 (Shift-III) Misspelt word

Ans. (d) : The wrongly spelt word from the given options is (d) 'Providance'. The correctly spelt word is 'Providence' which means - 'divine guidance or care'. The other options are correctly speltConsequence - Result, Fixation - Assessment, Musician - Melodist, Thus, the correct answer is option (d). 452. In the following question, a word has been written in four different ways out of which only one is correctly spelt. Select the correctly spelt word. (a) Procedger (b) Procedure (c) Proceadure (d) Proceger SSC MTS 9-10-2017 (Shift-II) Ans. (b) : The correctly spelt word from the given options is (b) 'Procedure' which means 'the usual or correct way for doing something'. The other given options are incorrectly spelt. Thus, correct answer is option (b). 453. In the following question, a word has been written in four different ways out of which only one is correctly spelt. Select the correctly spelt word. (a) Parllel (b) Paralell (c) Parallel (d) Parralel SSC MTS 11-10-2017 (Shift-I) Ans. (c) The correctly spelt word from the given options is (c) 'Parallel' which means - 'with the same distance between them for all their length'. The other options are incorrectly spelt. Thus, the correct answer is options (c). 454. In the following question, a word has been written in four different ways out of which only one is correctly spelt. Select the correctly spelt word. (a) Proceed (b) Proced (c) Prroceed (d) Procede SSC MTS 11-10-2017 (Shift-I) Ans. (a): The correctly spelt word from the given options is (a) 'Proceed' which means - 'to start doing the next thing after finishing the last one'. The other options are incorrectly spelt. Hence, the correct answer is options (a). 455. Choose the correctly spelt word. (a) Passengger (b) Pasenger (c) Passenger (d) Passinger SSC MTS – 06/08/2019 (Shift-III) Ans. (c) : The correctly spelt word from the given options is (c) 'Passenger' which means - 'a person who is traveling from one place to another in a car, bus, etc'. The other options are incorrectly spelt. Thus, the correct answer is options (c). 456. Choose the correctly spelt word. (a) Philosophy (b) Phelosophy (c) Philosophie (d) Filosophy SSC MTS – 08/08/2019 (Shift-I) Ans. (a) : The correctly spelt word from the given options is (a) 'Philosophy' which means - 'the study of ideas and beliefs about the meaning of life'. The other options are incorrectly spelt. Thus, the correct answer is options (a).

558

YCT

457. Select the wrongly spelt word. (a) Vessel (b) Kettle (c) Brittle (d) Parellel SSC MTS – 19/08/2019 (Shift-III) Ans. (d) : The wrongly spelt word from the given options is (d) 'Parellel'. The correctly spelt word is 'Parallel' which means- 'with the same distance between them for all their length'. The other options are correctly speltVessel - Pot, Kettle - Kettledrum, Brittle - Fragile, Thus, the correct answer is options (d). 458. Select the correctly spelt word. (a) posibilty (b) pompus (c) potantial (d) possession SSC MTS – 19/08/2019 (Shift-II) Ans. (d) : The correctly spelt word from the given options is (d) 'Possession' which means - 'the state of having or owning something'. The other correct options are Possibility - Probability, Pompous - claptoap, Potential - Probable/Capacity Thus, the correct answer is options (d). 459. Select the wrongly spelt word. (a) Perseverence (b) Probable (c) Politician (d) possession SSC GD – 08/03/2019 (Shift-III) Ans. (a) : The wrongly spelt word from the given options is (a) 'Perseverence'. The correctly spelt word is 'Perseverance' which means - 'the quality of continuing to try to achieve a particular aim despite difficulties. The other options are correctly speltProbable - Feasible, Politician - statesman, Possession - Rights 460. Select the correctly spelt word. (a) Permanant (b) Permanent (c) Pirmenent (d) Perminent SSC GD – 14/02/2019 (Shift-I) Ans. (b) : The correctly spelt word from the given options is (b) 'Permanent' which means - 'lasting for a long time or forever, that will not change. The other options are incorrectly spelt. Thus, the correct answer is option (b).

Q 461. The following sentence has been split into four segments. Identify the segment that contains an INCORRECTLY spelt word. Since the health hazards have reduced, / the ministry has relaxed the earlier regulations / and has suggested only a short period / of quarrantine for the affected people. (a) the ministry has relaxed the earlier regulations (b) Since the health hazards have reduced (c) of quarrantine for the affected people (d) and has suggested only a short period SSC Selection Posts XI-30/06/2023 (Shift-IV) Misspelt word

Ans. (c) : In the above sentence 'quarrantine' is wrongly spelt. Its correct spelling is 'quarantine'. Other options have no spelling error. Hence option (c) will be correct. 462. Select the incorrectly spelt word. (a) Rascal (b) Antibodies (c) Brag (d) Quear SSC CHSL 31.05.2022 Shift-III Ans. (d) : The word 'Quear' is the incorrect because the correct spelling is 'Queer' which means 'strange'. The other words are Rascal - scoundrel Antibodies - Antigen Brag - Boast Thus, the correct answer is option (d). 463. Select the correctly spelt word. (a) Qiete (b) Queit (c) Quiet (d) Quiete SSC CGL (Tier-I) – 12/06/2019 (Shift-II) Ans. (c): The correctly spelt word from the given options is (c) 'Quiet' which means - 'the state of being calm and without much noise or activity'. The other options are incorrectly spelt. Thus, the correct answer is options (c). 464. Select the correctly spelt word. (a) queiten (b) questionnaire (c) quallification (d) quantefy SSC CHSL (Tier-I) –10/07/2019 (Shift-I) Ans. (b): The correct spelt word from the given options is (b) 'Questionnaire' which means - 'a list of Questions The other correct options areQuatrain - Tetrapody, Qualification - Ability, Quantify - Admeasure, Thus, the correct answer is options (b). 465. Select the word with the incorrect spelling. (a) Quantity (b) Argument (c) Aisle (d) Quarrell SSC CHSL (Tier-I) 17/03/2020 (Shift-I) Ans. (d): The incorrectly spelt word from the given option is (d) 'Quarrell'. The correctly spelt word is 'Quarrel' which means - argument or disagreement. The other are correctly spelt Quantity - Amount, Argument - Reason, Aisle - Gallery Thus, the correct answer is options (d). 466. Select the incorrectly spelt word. (a) Pious (b) Reckless (c) Quary (d) Nibble SSC CHSL (Tier-I) –11/07/2019 (Shift-II) Ans. (c) : The incorrectly spelt word from the given options is (c), 'Quary'. The correctly spelt word is 'Query' which means - 'a question'. The other options are correctly speltPious - Holy, Reckless - Careless, Nibble - Scrabble. Thus, the correct answer is options (c). 467. Select the incorrectly spelt word. (a) Vague (b) Qoutidian (c) Doubt (d) Identical SSC MTS – 22/08/2019 (Shift-I)

559

YCT

Ans. (b) : The incorrectly spelt word from the given 473. From among the words given in bold, select the INCORRECTLY spelt word in the following options is (b). Qoutidian'. The correctly spelt word is sentence. 'Quotidian' which means - 'Occurring every day or The popular ridikule of Puritanism in daily. burlesque and doggerel is best exemplified in The other options are correctly spelt. Butler’s Hudibras. Vague - obscure, (a) burlesque (b) doggerel Doubt - Suspicion, (c) exemplified (d) ridikule Identical - Similar, SSC CHSL 25.05.2022 Shift-I Thus, the correct answer is options (b). Ans. (d) : The word 'Ridikule' is incorrect because the correct spelling is 'Ridicule' which means 'mock'. The other words areBurlesque - caricature 468. Select the correctly spelt word. Doggerel - comic verse composed in irregular rhythm. (a) resind (b) rescind Exemplified - Illustrate (c) recsind (d) recind SSC MTS 12/05/2023 (Shift-III) Thus, the correct answer is options (d). Ans. (b) : The above given options are incorrect except 474. Select the INCORRECTLY spelt word. (a) Rifel (b) Ride option (b) 'Rescind' which means – 'to officially state (c) Right (d) Ridge that a law, contract, decision, etc. is no longer valid.' SSC CHSL (Tier-I) –16/04/2021 (Shift-I) The other option are incorrectly spelt. Ans. (a) : The incorrectly spelt word from the given 469. Select the correctly spelt word. options is (a) 'Rifel'. The correct spelling is 'Refill' (a) Ricipit (b) Reicipit which means - 'to fill something again. (c) Reciept (d) Receipt The other options are correctly speltSSC GD 23/01/2023 (Shift-IV) Ridge - Braid, Ans. (d) : Out of the above options, option (d) Receipt Right - correct. Ride - Journey, (Quittance, fee) is correctly spelt. Thus, the correct answer is options (a). 470. Select the correctly spelt word. 475. Select the wrongly spelt word. (a) Recemmend (b) Reccommend (a) solution (b) beaker (c) Reccomend (d) Recommend (c) brakes (d) refrence SSC CPO-SI – 23/11/2020 (Shift-I) SSC CHSL (Tier-I) 10/08/2023 (Shift-I) Ans. (d) : Option (d) Recommend – ( To tell somebody Ans. (d) : The wrongly spelt word from the given that something is good or To suggest favorably as options is (d) 'Refrence'. The correctly spelt word is 'Reference' Which means - 'a written or spoken suited for some use) has been correctly spelt. comment that mentions somebody'. Hence, option (d) is correct answer. The other options are correctly spelt471. Select the correctly spelt word. Solution - Settlement, Beaker - Glass, (a) Rudimentory (b) Concious Brakes - slow down, (c) Coalece (d) Reliant SSC MTS-14/07/2022 Shift-III Thus, the correct answer is options (d). 476. Select the misspelled word. Ans. (d) : Out of the given options, only 'Reliant' is (a) pierce (b) riegn spelt correctly and it means trustful'. (c) relieve (d) deceive The other words are SSC CPO-SI – 25/11/2020 (Shift-II) Rudimentary - Basic Ans. (b) : The misspelled word from the given options is (b) 'Riegn'. The correctly spelt and word is 'Reign' Conscious - Aware which means - 'To rule a country. Coalesce - Partake The other options are correctly speltThus, the correct answer is options (d). Pierce - Prick, Relieve - Help, 472. Select the INCORRECTLY spelt word. Deceive - Hoodwink, (a) Retrieve (b) Return Thus, the correct answer is option (b). (c) Retort (d) Revange 477. Select the correctly spelt word. SSC CGL (Tier-I) 11/04/2022 Shift-II (a) richeous (b) riteous Ans. (d) : The word 'Revange' is incorrect because the (c) righteous (d) rightious correct spelling. SSC CPO-SI – 24/11/2020 (Shift-II) 'Revenge' which means 'vendetta'. Ans. (c) : The correctly spelt word from the given The other words are options is (c) which means - 'that you think is morally Retrieve - Rectify good or fair.' Return - Comeback The other options are incorrectly spelt. Retort - Defensive Thus, the correct answer is options (c).

R

Misspelt word

560

YCT

478. Select the correctly spelt word. (a) recommend (b) reccommend (c) recommened (d) recomend SSC CPO-SI – 24/11/2020 (Shift-II) Ans. (a) : The correctly spelt word from the given options is (a) 'Recommend' which means - 'to suggest that someone would be good for a particular job'. The other options are incorrectly spelt. Thus, the correct answer is option (a). 479. Select the wrongly spelt word. (a) rendezvous (b) renassaince (c) remuneration (d) remonstrate SSC CGL (Tier-I) – 06/06/2019 (Shift-II) Ans. (b): The wrongly spelt word from the given options is (b) 'renassaince'. The correctly spelt word is 'Renaissance' which means - 'a revival of classical learning and wisdom'. The other options are correctly spelt. Rendezvous - Appointment, Remuneration - Return/Reward, Remonstrate - Resist, Thus, the correct answer is options (b). 480. Select the wrongly spelt word. (a) remembrance (b) reminascant (c) remnant (d) remittance SSC CGL (Tier-I) – 06/06/2019 (Shift-III) Ans. (b) : The wrongly spelt word from the given options is (b) 'Reminascant'. The correctly spelt word is 'Reminiscent' which means - 'awakening memories of something similar'. Remittance - Dispatch, Remembrance - Souvenir, Remnant - Relics, Thus, the correct answer is options (b). 481. Select the INCORRECTLY spelt word. (a) reliable (b) realm (c) resemble (d) redius SSC CPO-SI – 09/12/2019 (Shift-I) Ans. (d): The incorrectly spelt word from the given options is (d) 'Redius'. The correctly spelt word is 'Radius' which means - 'the distance from the centre of a circle to the outside edge'. The other options are correctly spelt Reliable - Credible, Realm- Region, Resemble - Harmonize, Thus, the correct answer is options (d). 482. Select the wrongly spelt word. (a) Relagate (b) Residue (c) Refurbish (d) Retrospect SSC CPO-SI – 13/12/2019 (Shift-I) Ans. (a) : The wrongly spelt word from the given options is (a) 'Relagate'. The correctly spelt word is 'Relegate' which means - 'to assign to a lower position'. The other options are correctly speltResidue - Remnant, Refurbish - Renovate, Retrospect - Enquiry, Thus, the correct answer is options (a). 483. Select the correctly spelt word. (a) Tantative (b) Substentive (c) Remunerative (d) Speculetive SSC CHSL (Tier-I) –11/07/2019 (Shift-II) Misspelt word

Ans. (c): The correctly spelt is option (c) 'Remunerative' which means - 'Profitable'. The other correct options areTentative - Test, Substantive - Original, speculative - Imaginary, Hence, the correct answer is option (c). 484. Select the correctly spelt word (a) reeciet (b) reciept (c) receipt (d) reiceit SSC CHSL (Tier-I) –09/07/2019 (Shift-I) Ans. (c): The correctly spelt word from the given options is (c) 'Receipt' which means- 'a piece of paper that is given to show that you have paid for something'. The other options are incorrectly spelt. Hence, the correct answer is options (c). 485. Select the correctly spelt word. (a) compitition (b) distruction (c) contruction (d) repetition SSC CHSL (Tier-I) –04/07/2019 (Shift-III) Ans. (d): The correctly spelt word is options (d) 'Repetition' which means - 'doing something again'. The other correct options are Competition - contest, Destruction - Annihilation, Construction - Build Thus, the correct answer is options (d). 486. Select the wrongly spelt word. (a) reproduce (b) reverse (c) require (d) resembel SSC CHSL (Tier-I) –03/07/2019 (Shift-III) Ans. (d) : The wrongly spelt word is options (d) 'Resembel'. The correctly spelt word is 'Resemble which means - ' to be or look like somebody or something else'. The other options are correctly spelt Reproduce - Facsimile, Reverse - Inverse, Require - Need Thus, the correct answer is option (d). 487. Select the INCORRECTLY spelt word. (a) Advantageous (b) Tedious (c) Victorious (d) Rebelleous SSC CHSL (Tier-I) 19/10/2020 (Shift-II) Ans. (d) : The incorrectly spelt word is options (d) 'Rebelleous'. The correctly spelt word is 'Rebellious' which means- 'fighting against a government'. (Mutinous) The other options are correctly speltAdvantageous - profitable, Tedious - Bored. Victorious - Successful, Thus, the correct answer is options (d). 488. Select the wrongly spelt word. (a) Esteem (b) Respectfull (c) Subtle (d) Mimic SSC CHSL (Tier-I) 12/10/2020 (Shift-III) Ans. (b) : The wrongly spelt word is options (b) 'Respectfull'. The correctly spelt word is 'Respectful' which means - 'marked by or showing respect or deference'.

561

YCT

The other options are correctly speltEsteem - Veneration, subtle - meticulous, Mimic - Imitator, Thus, the correct answer is option (b). 489. Select the correctly spelt word. (a) Rupangence (b) Repugnance (c) Repungance (d) Repungence SSC MTS – 22/08/2019 (Shift-I) Ans. (b) : The correctly spelt word from the given options is (b) 'Repugnance' which means - 'Causing a feeling of dislike or disgust'. The other options are incorrectly spelt. Thus, the correct answer is option (b). 490. Select the correctly spelt word. (a) Restaurant (b) Restorant (c) Restaurent (d) Restorent SSC MTS – 22/08/2019 (Shift-I) Ans. (a) : The correctly spelt word fromt he given options is (a) 'Restaurant' which means - 'a place where you can buy and eat a meal'. The other options are incorrectly spelt. Thus, the correct answer is options (a). 491. Select the correctly spelt word. (a) Reverence (b) Reverense (c) Revernce (d) Revernece SSC MTS – 21/08/2019 (Shift-II) Ans. (a): The correctly spelt word from the given options is (a) 'Reverence' which means - a feeling of great respect'. The other options are incorrectly spelt. Thus, the correct answer is options (a). 492. Select the correctly spelt word. (a) Revenue (b) Ravanue (c) Ravenue (d) Reveanu SSC MTS – 19/08/2019 (Shift-III) Ans. (a) : The correctly spelt word from the given options is (a) 'Revenue' which means - 'money regularly receive by a government'. The other options are incorrectly spelt. Thus, the correct answer is options (a). 493. Select the correctly spelt word. (a) Recommend (b) Repitition (c) Referance (d) Reciept SSC MTS – 14/08/2019 (Shift-I) Ans. (a) : The correctly spelt word is options (a) 'Recommend' which means - 'to advice someone to do something'. (Petition) The other correct options are Repetition - Reiterate, Reference - context, Receipt - Quittance, Thus, the correct answer is option (a). 494. Select the wrongly spelt word. (a) Resistence (b) Independent (c) Characteristic (d) Beneficial SSC GD – 08/03/2019 (Shift-III) Ans. (a) : The wrongly spelt word from the given options is (a) 'Resistence' The correctly spelt word is 'Resistance' which means - 'to stop somebody from doing something'. Misspelt word

The other options are correctly speltIndependent - free, characteristic - specialty, Beneficial - profitable, Thus, the correct answer is option (a). 495. Select the wrongly spelt word. (a) Reception (b) Recommend (c) Ridicullous (d) Representative SSC GD – 08/03/2019 (Shift-II) Ans. (c) : The wrongly spelt word from the given options is (c 'Ridicullous'. the correctly spelt word is 'Ridiculous' which means - 'very silly or unreasonable. The other options are correctly speltReception -Greeting, Recommend - Petition, Representative - Delegate, Thus, the correct answer is option (c).

S 496. Choose the incorrectly spelt word. (a) Structurere (b) Havoc (c) Recently (d) Brush SSC MTS 16/05/2023 (Shift-II) Ans. (a) : The above given options are correctly spelt except option (a) 'structrure'. The correctly spelt word is 'structure'. Thus, the correct answer is option (a). 497. Identify the MISSPELT word in the following sentence. In some regions of the world, child poverty remains a serious issue, significently in relation of the geospatial and living standards in which children grow up. (a) Issue (b) Significently (c) Geospatical (d) Regions SSC GD 27/01/2023 (Shift-I) Ans. (b) : In the following sentence the word 'significently' is misspelt. It's correct spelling is 'Significantly'. Meaning of other optionsIssue Matter, Controversy Geospatical Geographically Regions Area, Territory 498. Select the INCORRECTLY spelt word. (a) Nutrient (b) Sitable (c) Seperate (d) Permanent SSC GD 12/01/2023 (Shift-II) Ans. (c) : Among the given options, option (c) seperate is incorrectly spelt. Its correct spelling is 'separate'. Meaning of the other optionsNutrient - Nutritious, alible Suitable - appropriate Permanent - Stable, durable 499. Identify and correct the INCORRECTLY spelt word in the given sentence. Sometimes failure is the steping stone to success. (a) Succes (b) Stepping (c) Failuire (d) Stown SSC GD 08/02/2023 (Shift-IV)

562

YCT

Ans. (b) : 'Steping' is the incorrectly spelt word in the 504. Parts of the following sentence have been given as options. Select the option that contains an given sentence. error. The correct word will be stepping which means :- to The lack of mutual bonding was conspicuous move a short distance. from their shody body language in the The other options– marriage party. Success :- An attainment that is successful. (a) marriage (b) conspicuous Failure :-lack of success. (loser) (c) shody (d) mutual Stown :- stown a scotch form of stolen. SSC CHSL (Tier-I) 09/08/2023 (Shift-III) 500. Select the INCORRECTLY spelt word in the Ans. (c) : Error lies is in part (c) 'shody' of the sentence. It's correct spelling will be- 'Shoddy' which means given sentence. To achieve somthing gigantic, you have to 'showing little respect, thought or core. marriage – Nuptial knot grind for a considerable amount of time. Mutual – Shared by two/involving two people (a) considerable (b) gigantic Conspicuous – Easily seen or noticed– (c) achieve (d) somthing Correct Sentence SSC CGL (Tier-I) 26/07/2023 (Shift-II) The lack of mutual funding was Conspicuous from their Ans. (d) : The correct answer is option (d) 'Somthing' shoddy body language in the marriage party because the correctly spelt word is 'something'. other 505. Select the INCORRECTLY spelt word. (a) Drastic (b) Surrugocy options are correctly spelt. (c) Cryptic (d) Obdurate Thus, the correct answer is option (b). SSC CHSL (Tier-I) 08/08/2023 (Shift-II) 501. Select the INCORRECTLY spelt word. Ans. (b) : The incorrectly spelt word of the above given (a) Sufficiant (b) Syrup options is (b) 'surrugocy.' The correctly spelt word is (c) Superior (d) Shubbery 'surrogacy' which means 'an arrangement by which a SSC CGL (Tier-I) 27/07/2023 (Shift-III) women gives birth to a baby on behalf of some who is Ans. (a) : The incorrectly spelt word of the above given physically unable to have babies themselves.' options is (a) 'Sufficiant'. The correctly spelt word is- Other options are correctly spelt. 'Sufficient'? 506. Select the INCORRECTLY spelt word. (a) Though (b) Psychic Other given options are correctly spelt. (c) Suddeness (d) Truthfully 502. Select the INCORRECTLY spelt word. SSC Selection Posts XI-28/06/2023 (Shift-III) (a) Anxiety (b) Stetionery Ans. (c) : In the above options, option (c) "Suddeness" (c) Preparation (d) Voluntary is wrongly spelt. Its correct spelling is "Suddenness". SSC CGL (Tier-I) 25/07/2023 (Shift-IV) Other options are correctly spelt. Ans. (b) : All the above given option are correctly spelt 507. Choose the incorrectly spelt word. (a) Cream (b) Sleam except option (b) 'Stetionery.' The correct word is (c) Beam (d) Steam 'stationary.' SSC CHSL (Tier-I) 21/03/2023 (Shift-II) Meaning of other options Ans. (b) : (b) In the above options, 'sleam' is incorrectly Anxiety - Impatience, spelt. Its correct spelling is slim. Other options are Preparation - Dispositon, correct. Voluntary - Spontaneous, 508. Choose the sentence with no spelling error. 503. Select the INCORRECTLY spelt word. (a) Souvenir (b) Suseptible (c) Symbolise (d) Sympathetic SSC CHSL (Tier-I) 17/08/2023 (Shift-II) Ans. (b) :Option (b) Suseptible is incorrectly spelt. its correct spelling will be 'susceptible'– (Easily influenced , damaged or affected by something/somebody). Given options contain following meaning – Souvenir – Something kept or serving as a reminder of a place, person or occasion. Symbolise – The use of symbols to represent ideas or qualities Sympathetic – Showing that you understand other peoples feeling , especially their problems. Misspelt word

We can never solvve our problems. We can never solve our problems. We can never solve our problams. We can nver solve our problems. SSC CHSL (Tier-I) 17/03/2023 (Shift-IV) Ans. (b) : In the above options, option (b) is correct sentence. Other options have incorrect word. 509. Select the correctly spelt word. (a) Suddenly (b) Sudenly (c) Sudenlly (d) Suddenlly SSC MTS-07/07/2022 Shift-II Ans. (a) : Out of the given options, only 'suddenly' is spelt correctly. It means 'Abruptly'. Thus, the correct answer is options (a).

563

(a) (b) (c) (d)

YCT

516. Select the option that corrects the misspelt 510. Select the incorrectly spelt word. words in the given sentence. (a) Injury (b) Thrust A number of measres have been sugested by (c) Sheild (d) Attach her. SSC Constable GD-07/12/2021 Shift-II (a) A number of measrues have been sugessted Ans. (c) : The word 'sheild' is incorrect because the by her. correct spelling is 'shield' which means 'armor'. (b) A number of measures have been sujjested by Thus the correct answer is option (c). her. (c) A number of measures have been suggested 511. Select the INCORRECTLY spelt word. by her. (a) Sumpteus (b) Summary (d) A number of mesures have been suggested by (c) Suffering (d) Sundry her. SSC Constable GD-22/11/2021 Shift-II SSC MTS-18/07/2022 Shift-I Ans. (a) : The word 'Sumpteus' is incorrect. Its correct Ans. (c) : There is no error in the option (c) because spelling is 'Sumptuous' which means 'magnificent'. spelling of suggested' and measures are correct while The other options are other options are incorrect. Summary - Essence Thus, the correct answer is options (c). Sundry - various 517. Select the INCORRECTLY spelt word. Thus, the correct answer is options (a). (a) Tongue (b) Heartbeat (c) Secratariat (d) Performance 512. Select the correctly spelt word. SSC CGL (Tier-I) 21/04/2022 Shift-III (a) Spacifying (b) Specifing Ans. (c) : The word 'Secratariat' is incorrect because the (c) Specifying (d) Specefying SSC Constable GD-22/11/2021 Shift-II correct spelling is 'Secretariat' It means 'administrative office'. Ans. (c) : Out of the given options, only 'specifying' is The other words arespelt correctly while other options are incorrect. Thus, the Performance – Execution correct answer is options (c). Heartbeat – Sound of heart Tongue – Accents 513. Select the correctly spelt word. Thus, the correct answer is option (c). (a) Magestic (b) Flexibal 518. Select the INCORRECTLY spelt word. (c) Stringent (d) Faltoring (a) Chauffeur (b) specielist SSC MTS-07/07/2022 Shift-I (c) Anchor (d) Compere Ans. (c) : Out of the given words, only stringent, is SSC MTS-18/07/2022 Shift-I spelt correctly. It means stiff. The other options are Ans. (b) : The word 'Specielist' is incorrect because the incorrect. correct spelling 'specialist' which means expert. Thus, the correct answer is option (c) The other words are 514. Select the INCORRECTLY spelt word. Chauffeur - motor Driver (a) Symtoms (b) Pilgrim Compere - familiarization (c) Progress (d) Mystery 519. Select the incorrectly spelt word. SSC CGL (Tier-I) 12/04/2022 Shift-I (a) Autopsy (b) Anonymous Ans. (a) : The word 'symtoms' is incorrect because the (c) Indefatigable (d) Supercede correct spelling is 'symptoms' which means SSC CHSL 30.05.2022 Shift-I characteristics. Ans. (d) : 'Super cede' is incorrect spelt because the The other words are correct spelling is 'supersede', which means transgress. Pilgrim - Traveler of holy place The other words areProgress - development Autopsy postmortem / anonymous - Nameless. In defatigable Tireless. Mystery - Enigma Thus, the correct answer is options (d). Thus, the correct answer is options (a). 520. Select the incorrectly spelt word. 515. Select the INCORRECTLY spelt word. (a) superior (b) superstition (a) Stationery (b) Stratagy (c) superflous (d) supreme (c) Statutory (d) Stationary SSC CHSL (Tier-I) –04/08/2021 (Shift-I) SSC CGL (Tier-I) 12/04/2022 Shift-III Ans. (c) : The incorrectly spelt word is option (c) Ans. (b) : The word 'stratagy' is incorrect because the 'superflous'. The correctly spelt word is - Superfluous correct spelling is 'Strategy' means politics. means - 'more than is sufficient or required'. The other words are The other options are correctly spelt. Stationery - writing goods Superior - Elevated, Statutory - legal Superstition - Vagary, Supreme - Uppermost, Stationary - stagnant / stable Hence, the correct answer is option (c). Thus, the correct answer is option (b). Misspelt word

564

YCT

521. Select the incorrectly spelt word. (a) Suspicion (b) Saspence (c) Suppress (d) Secrecy SSC CHSL (Tier-I) –19/04/2021 (Shift-I) Ans. (b) : The incorrectly spelt word from the given options is (b) 'saspence'. The correctly spelt word is 'suspense' which means - 'a feeling of worry or easement that you have when you feel that something is going to happen'. The other options are correctly spelt Suspicion - Doubt, Suppress - Stifle, Secrecy - Stealthiness, Thus, the correct answer is option (b). 522. Select the incorrectly spelt word. (a) Chameleon (b) Caterpillar (c) Lizard (d) Scorpeon SSC CGL (Tier-I) –18/08/2021 (Shift-I) Ans. (d) : The incorrectly spelt word is option (d) 'Scorpeon'. The correctly spelt word is - 'Scorpion' which means 'a creature which looks like a large insect and lives in hot countries'. The other options are correctly spelt Chameleon - Chrysotile, Caterpillar - Butterfly. Lizard - Pangolin, Thus, the correct answer is option (d). 523. Select the misspelt word. (a) strech (b) swallow (c) sense (d) survey SSC CGL (Tier-II) 16/11/2020 (3 pm - 5 pm) Ans. (a) : The misspelt word is option (a) 'strech'. The Correctly spelt word is 'stretch' which means 'to pull something so that is becomes longer or wider. The other options are correctly spelt Swallow - Ingest, Sense - Spirit, Survey - Record, Thus, the correct answer is option (a). 524. Select the misspelled word. (a) lustrous (b) succint (c) dilemma (d) enigmatic SSC CPO-SI – 23/11/2020 (Shift-II) Ans. (b) : The misspelt word from the given options is (b) 'succint'. The correctly spelt word is 'Succinct' which means - 'said clearly' in a few words'. The other options are correctly spelt Lustrous - Shiny, Dilemma - Doubt, Enigmatic - Mysterious, Thus, the correct answer is options (b). 525. Select the misspelled word. (a) acquaint (b) spectacles (c) superceed (d) restaurant SSC CPO-SI – 23/11/2020 (Shift-II) Ans. (c) : The incorrectly spelt word from the given options is (c) 'Superceed'. The correct spelling is 'Supercedes' which means - 'to cause to be set aside'. The other options are correctly spelt Acquaint - Deputize, Spectacles - Eyeglasses, Restaurant - Canteen, Thus, the correct answer is option (c). Misspelt word

526. Select the correctly spelt word. (a) taelant (b) cleaver (c) centiment (d) serenity SSC CGL (Tier-I) – 13/06/2019 (Shift-I) Ans. (d): The correctly spelt word is option (d) 'serenity' which means - 'a state of being calm, peaceful a untroubled. The other correct options are Talent - Genius, Clever - Crafty, Sentiment - Emotion, Thus, the correct option is (d) 527. Select the correctly spelt word. (a) scissors (b) sweatter (c) clattering (d) teribble SSC CGL (Tier-I) – 13/06/2019 (Shift-II) Ans. (a) : The correctly spelt word is option (a) 'scissors' which means - a small cutting tool with two sharp blades that are screwed together. The other correct options are Sweater - Pullover, cluttering - Huddle, Terrible - Dire, Thus, the correct answer is option (a). 528. Select the correctly spelt word. (a) jewlry (b) defnition (c) scramble (d) acceptence SSC CGL (Tier-I) – 13/06/2019 (Shift-III) Ans. (c): The correctly spelt word is options (c) 'Scramble' which means - 'to fight or move quickly to get something which a lot of people want'. The other correct options are Jewellery - Ornament, Definition - Term, Acceptance - Assent, Thus, the correct answer is options (c). 529. Select the wrongly spelt word. (a) Sacrelegious (b) Millennium (c) Connoisseur (d) Manoeuvre SSC CPO-SI – 11/12/2019 (Shift-II) Ans. (a): The wrongly spelt word from the given options is (a) 'sacrelegious'. The correctly spelt word is 'Socrilegious' which means - an act of treating a holy thing or place without respect. Millennium - Millenary, Connoisseur - Sampler Manoeuvre - Proficiency, Thus, the correct options is (a). 530. Select the wrongly spelt word. (a) whimsical (b) contiguous (c) spectaculer (d) adjacent SSC CHSL (Tier-I) –10/07/2019 (Shift-I) Ans. (c): The wrongly spelt word is option (c) 'spectaculer' the correct spelling is 'spectacular' which means - 'very impressive to see. The other options are correctly spelt Whimsical - Freak, Contiguous - Embodied, Adjacent - Imminent, Thus, the correct answer is options (c). 531. Select the correctly spelt word. (a) saprate (b) separete (c) saperate (d) separate SSC CHSL (Tier-I) –05/07/2019 (Shift-II)

565

YCT

Ans. (d): The correctly spelt word from the given options is (d) 'Separate' which means - ' to keep people or things apart'. The other options are incorrectly spelt. Thus, the correct answer is options (d). 532. Select the wrongly spelt word. (a) Scenary (b) Scarce (c) Scheme (d) Scanty SSC CHSL (Tier-I) –04/07/2019 (Shift-I) Ans. (a): The wrongly spelt word is options (a) 'Scenary'. The correctly spelt word is 'scenery;' which means 'The natural beauty that you see around you in country'. Scarce - Short, Scheme - Plan, Scanty - Meager, thus, the correct answer is option (a). 533. Select the wrongly spelt word. (a) settle (b) satallite (c) satiate (d) session SSC CHSL (Tier-I) –02/07/2019 (Shift-II) Ans. (b): The wrongly spelt word from the given options is (b) 'satallite'. The correctly spelt word is 'satellite' which means - 'a moon, planet ro machine that orbits a planet or star'. The other options are correctly spelt. Settle - Dwell, Satiate - Satisfy, Session - sitting, Thus, the correct answer is option (b). 534. Select the wrongly spelt word. (a) Simulation (b) Synthetic (c) Simultaneoues (d) Stylistics SSC CHSL (Tier-I) 19/03/2020 (Shift-I) Ans. (c) : The wrongly spelt word is option (c) 'simultaneoues'. The correctly spelt word is 'simultaneous' which means - 'happening or done at exactly the same time as something else. The other options are correctly spelt Simulation - Fake, Synthetic - Artificial, Stylistics - Bombastic, Thus, the correct answer is option (c). 535. Select the INCORRECTLY spelt word. (a) Excess (b) Supportors (c) Inclusion (d) Discriminatory SSC CHSL (Tier-I) 13/10/2020 (Shift-III) Ans. (b): The incorrectly spelt word from the given options is (b) 'Supportors' which means - 'a person who gives supports to someone or something'. Excess - Spare, Discriminatory - Electoral, Inclusion - meeting Thus, the correct answer is option (b). 536. Select the INCORRECTLY spelt word. (a) Succumb (b) Success (c) Surender (d) Succulent SSC CHSL (Tier-I) 14/10/2020 (Shift-III) Ans. (c) : The incorrectly spelt word is option (c) 'Surender'. The correctly spelt wond is 'Surrender', Which means' to stop fighting and admit that you have lost'. Misspelt word

The other options are correctly spelt Succumb - Truckle, Success - Prosperity, Succulent - Lush, Thus, the correct answer is option (c). 537. Select the wrongly spelt word. (a) Popularly (b) Sincearly (c) Beggarly (d) Jocularly SSC CHSL (Tier-I) 12/10/2020 (Shift-I) Ans. (b) : The wrongly spelt word is option (b) 'Sincearly'. the correctly spelt word is 'sincereley' which means 'honestly and without pretending or lying'. The other options are correctly speltPopularly - Famously, Beggarly - Indigent, Jocularly - Humorous, Thus, the correct answer is option is options (b). 538. Select the word with the incorrect spelling (a) Ascertain (b) Denounce (c) Systemetic (d) Combination SSC CHSL (Tier-I) 17/03/2020 (Shift-III) Ans. (c) : The incorrectly spelt word is option (c) 'systemetic'. The correctly spelt word is 'systematic' which means - 'doing using a fixed plan or method. The other options are incorrectly speltAscertain - Ensure, Denounce - Impute, Combination - Annexation, Thus, the correct answer is option (c). 539. In the following question, a word has been written in four different ways out of which only one is correctly spelt. Select the correctly spelt word. (a) Secratary (b) Secretery (c) Secratery (d) Secretary SSC MTS 9-10-2017 (Shift-I) Ans. (d) The correctly spelt word from the given options is (d) 'Secretary' which means - an officer of an organization or society responsible for its records and correspondence. The other options are incorrectly spelt Thus, the correct answer is option (d). 540. In the following Question, a word has been written in four different ways out of which only one is correctly spelled. Select the correct spelled word (a) Sicondary (b) Secondiry (c) Secondary (d) Secondory SSC MTS 9-10-2017 (Shift-III) Ans. (c): The correctly spelt word from the given options is (c) 'secondary' which means - 'less important than something else? The other options are incorrectly speltThus, the correct answer is options (c). 541. In the following Question, a word has been written in four different ways out of which only one is correctly spelled. Select the correct spelled word (a) Strength (b) Strenth (c) Stringth (d) Strentgh SSC MTS 9-10-2017 (Shift-III)

566

YCT

Ans. (a): The correctly spelt word from the given options is (a) 'strength' which means -' the quality or state of being strong'. The other options are incorrectly spelt. Thus, the correct answer is options (a). 542. In the following question, a word has been written in four different ways out of which only one is correctly spelt. Select the correctly spelt word. (a) Sukceed (b) Succed (c) Sucede (d) Succeed SSC MTS 10-10-2017 (Shift-III) Ans. (d) : The correctly spelt word from the given options is (d) 'Succeed' which means 'to manage to achieve what you want to do'. The other option..... 543. In the following question, a word has been written in four different ways out of which only one is correctly spelt. Select the correctly spelt word. (a) Sponsor (b) Esponsor (c) Sponser (d) Sponcer SSC MTS 10-10-2017 (Shift-II) Ans. (a) : The correct spelt word from the given options is (a) 'Sponsor' which means- a person or an organization that helps to pay for a special sports event. The other options are incorrectly spelt. Thus, the correct answer is option (a). 544. In the following question, a word has been written in four different ways out of which only one is correctly spelt. Select the correctly spelt word. (a) Satelite (b) Sattelite (c) Satellite (d) Sattellite SSC MTS 10-10-2017 (Shift-II) Ans. (c) : The correctly spelt word from the given options is (c) 'Satellite' which means - an electronic device that is sent into space and moves around the earth or another planet for a particular purpose'. The other options are incorrectly spelt. Thus, the correct answer is options (c). 545. Select the correctly spelt word. (a) Separate (b) Succesful (c) Seige (d) Superfisial SSC MTS – 16/08/2019 (Shift-I) Ans. (a) : The correctly spelt word is option (a) 'Separate' which means - 'to set or keep apart'. The other correct options areSuccessful - Prosperous, Siege - Blockade, Superficial - Flimsy, Thus, the correct answer is options (a). 546. Select the correctly spelt word. (a) Scienctific (b) scientific (c) Scientefic (d) Sceintific SSC MTS – 13/08/2019 (Shift-I) Ans. (b) : The correctly spelt from the given options is (b) 'scientific' which means - 'relating' to science, or using the organized methods of science'. The other options are incorrectly spelt. Thus, the correct answer is options (b). Misspelt word

547. Select the correctly spelt word. (a) Supplimantry (b) Supplimentry (c) Supplementary (d) Supplemantry SSC MTS – 09/08/2019 (Shift-II) Ans. (c) : The correctly spelt, word from the given options is (c) 'supplementary' which means - 'provided in addition to something else in order to improve or complete it'. The other options are incorrectly spelt. Thus, the correct answer is options (c). 548. Select the wrongly spelt word. (a) Secretary (b) Suspicous (c) Sustain (d) Schedule SSC GD – 08/03/2019 (Shift-II) Ans. (b) : The wrongly spelt word from the given options is (b) 'suspicous', the correctly spelt word is 'suspicious' which means - 'that makes you full that something is wrong, dishonest or illegal. The other options are correctly spelt Secretary - Front Bencher, Sustain - keep Schedule - curriculum, Thus, the correct answer is option (b).

T 549. Choose the incorrectly spelt word. (a) Locker (b) Rift (c) Tihgt (d) Stock SSC MTS 02/05/2023 (Shift-I) Ans. (c) : For the above given options, the incorrectly spelt word is 'Tihgt' because it correct form is 'Tight' which means 'narrow' Thus, the correct answer is option (c). 550. Choose the incorrectly spelt word. (a) Molest (b) Called (c) Reflect (d) Temparory SSC MTS 15/05/2023 (Shift-I) Ans. (d) : The above given options are correctly spelt except options (d) 'Temparory'. The correctly spelt word is 'Temporary.' Thus, the correct answer is option (d). 551. Select the INCORRECTLY spelt word. (a) Sentence (b) Artificial (c) Tendancy (d) Matrimony SSC GD 01/02/2023 (Shift-II) Ans. (c) : In the given options, 'Tendancy' is the incorrectly spelt word. The correct spelling is– Tendency which means– an inclination to do something. The meaning of other options is– Sentence :- pronounce a sentence on (somebody) in a court of law. Artificial :- unread, counterfeit. Matrimony :- spousal relationship. 552. Select the INCORRECTLY spelt word. (a) Thretened (b) Thermometer (c) Thanksgiving (d) Throughout SSC Constable GD-09/12/2021 Shift-II

567

YCT

Ans. (a) : The word 'Thretened' is incorrect because the correct spelling is 'Threatened' which means 'menace/Bluster'. Thus, the correct answer is options (a). 553. Select the INCORRECTLY spelt word. (a) Fatal (b) Taccal (c) Brutal (d) Metal SSC Constable GD-23/11/2021 Shift-III Ans. (b) : The word 'Taccal' is incorrect because the correct spelling is 'Tackle' it means 'deal with'. Other words areFatal - Deadly Brutal - Ferocious Thus, the correct answer is option (b). 554. Select the INCORRECTLY spelt word. (a) Tution (b) Manners (c) Circuit (d) Genuine SSC CGL (Tier-I) 11/04/2022 Shift-I Ans. (a) : The word 'Tution' is incorrect because the correct spelling is 'Tuition' which means teach/preach. The other words are Manners - Decorum Circuit - circle Genuine - original Thus, the correct answer is options (a). 555. Select the INCORRECTLY spelt word. (a) Longitude (b) Manners (c) Contrary (d) Terribal SSC CGL (Tier-I) 13/04/2022 Shift-II Ans. (d) : The word 'Terribal' is incorrect because the correct spelling is 'Terrible' which means horrible'. The other words are Longitude - compass Manners - courtesy / Decorum Contrary - protest Thus the correct answer is option (d). 556. Select the INCORRECTLY spelt work. (a) Temprate (b) Cooperate (c) Syndicate (d) Contemplate SSC CGL (Tier-I) 20/04/2022 Shift-III Ans. (a) : The word 'Temprate' is incorrect because the correct spelling is 'Temperate' mean 'Sober'. The other words are Cooperate - collaborate Syndicate - organization Contemplate - ponder Thus, the correct answer is option (a). 557. Select the incorrectly spelt word. (a) disappear (b) until (c) twelth (d) temperature SSC CHSL (Tier-I) –04/08/2021 (Shift-I) Ans. (c) : The incorrectly spelt word from the given options is (c) 'Twelth'. The correctly spelt word is 'Twelfth'. The other options are correctly speltDisappear - Evanesce, Until - During, Temperature - Climate, Thus, the correct answer is option (c). Misspelt word

558. Select the incorrectly spelt word. (a) aggravate (b) titilate (c) mediate (d) initiate SSC CHSL (Tier-I) –06/08/2021 (Shift-I) Ans. (b) : The incorrectly spelt word is option (b) 'titilate'. The correctly spelt word is 'Titillate' which means 'to excite or interest somebody deliberately, especially in a sexual way'. The other options are correctly speltAggravate - Foment, Thus, the correct answer is options (b). 559. Select the misspelt word. (a) tyrrany (b) vicious (c) resistance (d) pursue SSC CPO-SI – 24/11/2020 (Shift-I) Ans. (a): The misspelt word from the given options is (a) 'Tyrrany'. The correctly spelt word is 'Tyranny' a person or small group to control a country or state. The other options are correctly spelt Vicious - Roguish, Resistance - Opposition, Pursue - Follow, Thus, the correct options is (a). 560. Select the correctly spelt word. (a) tuetion (b) tuttion (c) tuition (d) tution SSC CGL (Tier-I) – 11/06/2019 (Shift-I) Ans. (c) : The correctly spelt word from the given options is (c) 'Tuition' which means - 'teaching' especially to a small group of people. The other options are incorrectly spelt. Thus, the correct answer is options (c). 561. Select the INCORRECTLY spelt word. (a) trenquility (b) schedule (c) acceleration (d) vacuum SSC CPO-SI – 12/12/2019 (Shift-I) Ans. (a): The incorrectly spelt word is option (a) 'Trenquility'. The correctly spelt word is 'Tranquility' which means - 'a feeling of calm'. The other options are correctly speltSchedule - Program, Acceleration - Rapidity, Vacuum - Void, Thus, the correct answer is options (a). 562. Select the correctly spelt word. (a) turbulance (b) turbulense (c) turbulence (d) terbulence SSC CHSL (Tier-I) –09/07/2019 (Shift-III) Ans. (c): The correctly spelt word from the given options is (c) 'Turbulence' which means - 'great commotion or agitation'. The other options are incorrectly spelt. Thus, the correct answer is option (c). 563. Select the wrongly spelt word. (a) trivial (b) tution (c) tyranny (d) typical SSC CHSL (Tier-I) –05/07/2019 (Shift-II) Ans. (b): The wrongly spelt word is option (b) 'Tution'. The correctly spelt word is 'Tuition' which means 'teaching' especially to a small group of people'. The other options are correctly spelt-

568

YCT

Trivial - niggard, Tyranny - Victimization Typical - Tentative, Thus, the correct answer is option (b). 564. Select the INCORRECTLY spelt word. (a) Tumulttous (b) Trousseau (c) Belligerent (d) Pharmaceutical SSC CHSL (Tier-I) 14/10/2020 (Shift-I) Ans. (a) : The incorrectly spelt word is option (a) 'Tumulttous'. The correctly spelt word is 'Tumultuous'. Which means - 'very noisy' because people are excite'. The other options are correctly spelt. Trousseau - attire, Belligerent - Truculent, Pharmaceutical - medicine, Thus, the correct answer is option (a). 565. Select the correctly spelt word. (a) Tution (b) Tuition (c) Tiution (d) Tuetion SSC MTS – 22/08/2019 (Shift-II) Ans. (b) : The correctly spelt word from the given option is (b) 'Tuition' which means- 'the activity of teaching, especially to small groups of students'. Thus, the correct answer is option (b). 566. Select the correctly spelt word. (a) Tendency (b) Tyeranny (c) Twelth (d) Tommorow SSC MTS – 16/08/2019 (Shift-III) Ans. (a) : The correctly spelt word from the given options is (a) 'Tendency' which means - 'a typical or repeated habit, action, or belief'. The other correct options are Tyranny - Enormity, Twelfth - Twelve in number Tomorrow - The next day Thus, the correct answer is options (a). 567. Select the correctly spelt word. (a) Temparary (b) Threshold (c) Temparature (d) Tachnique SSC MTS – 14/08/2019 (Shift-II) Ans. (b) : The correctly spelt word from the given option is (b) 'Threshold' which means - 'the ground at the entrance to a room or building'. The other options are incorrect, the correct spelling is Temporary - Mutable, Temperature - heat, Technique - method, Thus, the correct answer is options (b). 568. Select the wrongly spelt word. (a) Rogue (b) Argue (c) Toungue (d) Vague SSC MTS – 08/08/2019 (Shift-II) Ans. (c) : The wrongly spelt word from the given options is (c) 'Toungue'. The correctly spelt word is 'Tongue' which means - 'the soft, movable part in the mouth that is used for tasting food. The other options are correctly speltRogue - Rascal, Argue - Infer/Debate, Vague - Obscure, Thus, the correct answer is option (c). Misspelt word

569. Choose the correctly spelt word. (a) lgnorence (b) Interupt (c) Thriller (d) Colaborate SSC MTS – 06/08/2019 (Shift-II) Ans. (c) : The correctly spelt word from the given options is (c) 'thriller' which means - 'a type of mystery with a few key differences'. The other correct options areIgnorance - Insensibility, Interrupt - Hindrance, Collaborate - Cooperate, Thus, the correct answer is options (c).

U 570. Select the correct spelling of the underlined word from the options. She is the most underated cricketer in the country. (a) underateed (b) underrated (c) undderated (d) underatted SSC Selection Posts XI-27/06/2023 (Shift-I) Ans. (b) : The correct spelling of the above undrlined word is option (b) 'Underrated' which means 'Undrvalue; Spelling of other options are not correct. 571. Select the incorrectly spelt word. (a) Incriminating (b) Embodiment (c) Unpretensious (d) Cancellation SSC CGL (Tier-I) –17/08/2021 (Shift-I) Ans. (c) : The incorrectly spelt word from the given options is (c) 'unpretensious'. The correctly spelt word is - 'Unpretentious' which means - 'not trying to appear more special or important than you really are'. (simple) The other options are correctly speltIncriminating - Impeach, Embodiment - Incarnation, Canclelation - Rescission, thus, the correct answer is options (c). 572. Select the misspelled word. (a) userp (b) unique (c) impeach (d) implore SSC CPO-SI – 23/11/2020 (Shift-II) Ans. (a) : The misspelt word is options (a) 'Userp'. The correctly spelt word is 'Usurp' which means ' to seize and hold in possession by force or without right'. The other options are correctly spelt Unique - peerless, impeach - charge, Implore - Beseech/Prey, Thus, the correct answer is options (a). 573. Select the INCORRECTLY spelt word. (a) Recurrent (b) Unpretensious (c) Dilemma (d) Excellence SSC CHSL (Tier-I) 20/10/2020 (Shift-II) Ans. (b) : The incorrectly spelt word is option (b) 'Unpretensious'. The correctly spelt word is 'Unpretentious' which means- 'not trying to appear more special or important than you really are'. (Simple)

569

YCT

The other options are correctly spelt. The other options are correctly speltAnxious - Curious, Recurrent - Periodic, Moisture - Humidity, Dilemma - Doubt, Mimic - Imitator, Excellence - Superiority, Thus, the correct answer is option (d). Thus, the correct answer is option (b). 579. Select the correctly spelt word. 574. Select the correctly spelt word. (a) wengence (b) vengance (a) Undoutedly (b) Uniique (c) wengeance (d) vengeance (c) Unforgatable (d) Unfortunately SSC CPO-SI – 24/11/2020 (Shift-I) SSC MTS – 16/08/2019 (Shift-I) Ans. (d) : The correctly spelt word from the given Ans. (d) : The correctly spelt word is opiton (d) options is (d) 'vengeance'; which means - ' a desire for 'Unfortunately' which means - 'with bad luck'. revenge. (Vendetta) The other correct options are The other options are incorrectly spelt. Undoubtedly - certainly, Thus, the correct answer is options (d). Unique - Rare, 580. Select the wrongly spelt word. Unforgettable - Memorable, (a) eager (b) beggar Thus, the correct answer is options (d). (c) Violance (d) value SSC CHSL (Tier-I) –08/07/2019 (Shift-III) Ans. (c): The wrongly spelt word is options (c) 'Violance'. The correctly spelt word is 'Violence' which 575. Select the INCORRECTLY spelt word. means - 'the use of physical force so as to injure, abuse (a) Consonant (b) Similar or destroy'. (c) Treasure (d) Vaccenation SSC GD 13/02/2023 (Shift-I) The other options are correctly speltEager - Avid, Ans. (d) : In the above options, option (d) 'vaccenation' Beggar - mendicant, is used incorrectly spelt word. Value - Cost/Price, The correct word is– vaccinations which means– Thus, the correct answer is options (c). 'inoculation'. 581. Select the wrongly spelt word. The meaning of other options is– (a) video (b) voluntary Consonant :- harmonic (c) voracous (d) valour SSC CHSL (Tier-I) 20/10/2020 (Shift-III) Similar :- alike Treasure :- a collection of precious things. Ans. (c) : The wrongly spelt word is option (c) 576. The given sentence has an error which has been 'Voracous'. The correctly spelt word is 'voracious' underlined. The underlined word is given as which means 'eating or wanting large amounts of food'. The other options are correctly spelt options with some changes. Video - Television, Select the option that rectifies the error. Voluntary - unbidden/Spontaneous, She vaccillates a lot while making any decision. Valour - Bravery, (a) vacillattes (b) vasillates Thus, the correct answer is options (c). (c) vacilates (d) vacillates 582. Select the incorrectly spelt word. SSC CGL Mains 26/10/2023 (Shift-I) (a) Exterminated (b) Voilence (c) Unmongeable (d) wilderness Ans. (d) : In the above sentence 'vacillate' will be used SSC CGL (Tier-I) –13/08/2021 (Shift-I) in place of underlined word vacillates. Other options spelling is wrongly spelt. Hence option (d) will be Ans. (b) : The incorrectly spelt word is options (b) correct. 'Voilence'. The correctly spelt word is 'Violence' which means - 'the use of physical force so as to injure, abuse Correct Sentence or destroy. She vacillates a lot while making any decision. The other options are correctly spelt 577. Select the correctly spelt word. Exterminated - destruct, (a) Metalic (b) Vigorously Unmanageable - Awkward, (c) Crystalisation (d) Carying Wilderness - Forest SSC Constable GD-01/12/2021 Shift-I Thus, the correct answer is options (b). Ans. (b) : Out of the given option only 'vigorously' is spelt correctly while other options are incorrect. Thus, the correct answer is options (b). 583. Select the INCORRECTLY spelt word in the 578. Select the incorrectly spelt word. given sentence. (a) Anxious (b) Moisture It is wierd to overreact on the consequences (c) Mimic (d) Vishious like this, it didn’t even matter at the end of the SSC CGL (Tier-I) –16/08/2021 (Shift-I) day. (a) Matter (b) Wierd Ans. (d) : The incorrectly spelt word is option (d) (c) Consequences (d) Overreact 'Vishious'. The correctly spelt word is 'vicious' which SSC CGL (Tier-I) 18/07/2023 (Shift-III) means - 'Cunning/sly''.

V

W

Misspelt word

570

YCT

Ans. (b) : The incorrectly spelt word of the above given sentence is option (b) 'Wierd'. The correctly spelt word is 'wired'. 584. Select the INCORRECTLY spelt word. (a) Cradle (b) Fruitful (c) Wilfull (d) Unction SSC CGL (Tier-I) 17/07/2023 (Shift-II) Ans. (c) : In the above options, option (c) 'wilfull' is wrongly spelt its correct spelling will be 'willful' . other option is correct. 585. Choose the incorrently spelt word. (a) WASSE (b) FITTER (c) COURIET (d) COURSE SSC CHSL (Tier-I) 20/03/2023 (Shift-I) Ans. (a) : In the above options, option (a) 'wasse' is incorrectly spelt. Its correct spelling is 'wisse'. Other options are not correct. 586. Select the INCORRECTLY spelt word. (a) Develop (b) Occur (c) Regulate (d) Writting SSC Constable GD-03/12/2021 Shift-II Ans. (d) : The word 'writting' is incorrect because the correct spelling 'writing'. Develop - Progress/Evolve Occur - Happen Regulate - Govern Thus, the correct answer is option (d). 587. Select the incorrectly spelt word. (a) Complaint (b) Worried (c) Warior (d) Passenger SSC Constable GD-02/12/2021 Shift-I Ans. (c) : The word 'warior' is incorrect because the correct spelling is 'Warrior' it means 'Fighter'. Thus, the correct answer is option (c) 588. Select the correctly spelt word. (a) Recieive (b) Prepair (c) Cansel (d) Weird SSC CGL (Tier-I) 11/04/2022 Shift-III Ans. (d) : The word 'weird' is correct spelt which means 'peculiar'. The other options are incorrect. Thus, the correct answer is options (d). 589. Select the INCORRECTLY spelt word. (a) Deceive (b) Leisure (c) Wellfare (d) Believe SSC CGL (Tier-I) 13/04/2022 Shift-III Ans. (c) : The word 'Wellfare' is incorrect because the correct spelling is 'welfare' means 'well being'. The other words are Deceive - Betray Leisure - vacation Believe - trust Thus, the most appropriate answer is option (c). 590. Select the INCORRECTLY spelt word. (a) Hasten (b) Whitan (c) Enlighten (d) Brighten SSC CGL (Tier-I) 12/04/2022 Shift-II Misspelt word

Ans. (b) : The word 'Whitan' is incorrect because the correct spelling 'whiten' means to make white. The other words are Hasten - rush / Hurry up Enlighten - Notify / Glitter Brighten - glisten Thus, the correct answer is options (b). 591. Select the INCORRECTLY spelt word. (a) governor (b) conductor (c) inspector (d) waitor SSC CHSL (Tier-I) –15/04/2021 (Shift-I) Ans. (d) : The incorrectly spelt word is option (d) 'waitor'. The correctly spelt word is 'waiter' which means - a man whose job is to serve customers at their tables in a restaurant'. the other options are correctly speltInspector - Investigator, Governor - ruler, Conductor - Check taker, Thus, the correct answer is options (d). 592. Select the incorrectly spelt word. (a) Cerebral (b) Scrutinize (c) Withdrawl (d) Arbitrary SSC CGL (Tier-I) –16/08/2021 (Shift-I) Ans. (c) : The incorrectly spelt word is options (c) 'withdrawl'. The correctly spelt word is 'withdrawal' which means - 'moving or being moved back or away from a place' The other options are correctly speltCerebral - Medullary, Scrutinize - Check, Arbitrary - Licentious, Thus, the correct answer is option (c). 593. Select the correctly spelt word. (a) Witness (b) Wisdum (c) Wimper (d) Welth SSC CHSL (Tier-I) 14/10/2020 (Shift-I) Ans. (a) : The correctly spelt word in option (a) 'witness' which means - 'a person who sees something happen and who can tell other people about it later'. The other correct options are Wisdom - Sagacity, Whimper - crying softly, Wealth - money, Thus, the correct answer is option (a). 594. In the following question, a word has been written in four different ways out of which only one is correctly spelt. Select the correctly spelt word. (a) Woolen (b) Woollen (c) Wollen (d) Wolen SSC MTS 10-10-2017 (Shift-III) Ans. (b) : The correctly spelt word from the given options is (b) 'Woollen' which means - 'Clothes, made of wool'. The other options are incorrectly spelt. Thus, the correct answer is option (b). 595. Select the correctly spelt word. (a) Welfare (b) Wellfare (c) Wellfair (d) Welfair SSC MTS – 21/08/2019 (Shift-I)

571

YCT

Ans. (a): The correctly spelt word from the given options is (a) 'welfare' which means - 'the general health, happiness of a person, (Prosperity)'. The other options are incorrectly spelt. Thus, the correct answer is option (a). 596. Select the correctly spelt word. (a) Wadnesday (b) Worthwile (c) Wayther (d) Weird SSC MTS – 19/08/2019 (Shift-I) Ans. (d) : The correctly spelt word from the given options is (d) 'weird' which means - 'strange and different from anything natural or ordinary'. The other options are incorrect The other correct answer is option (d).

X 597. Select the incorrectly spelt word. (a) Wan (b) Veteran (c) Xanial (d) Vie SSC MTS – 16/08/2019 (Shift-III) Ans. (c) : The incorrectly spelt word is options (c) 'Xanial'. the correctly spelt word is 'Xenial' which means; 'hospitable, especially to visiting strangers or foreigners'. The other options are correctly spelt Veteron - Efficient, Wan - washy, Vie - compete Thus, the correct answer is option (c). 598. Select the incorrectly spelt word. (a) Climax (b) Vicious (c) Xanophobia (d) Adorn SSC CHSL (Tier-I) –04/08/2021 (Shift-I) Ans. (c) : The incorrectly spelt word is options (c) 'Xanophobia'. The correctly spelt word is 'Xenophobia' which means - 'a fear or hatred of foreign people and cultures'. The other options are correctly speltClimax - Zenith, Vicious - Impaired, Adorn - Spruce, Thus, the correct answer is options (c).

The other options are correctly spelt. Ceiling - Roof, Grief - Mourning, Niece - Cousin, Thus, the correct answer is options (c). 601. Select the incorrectly spelt word. (a) Victimize (b) Veteran (c) Commotion (d) Yern SSC CPO-SI – 11/12/2019 (Shift-II) Ans. (d) : The incorrectly spelt word is option (d) 'yern'. The correctly spelt word is 'yearn' which means - 'to want somebody or something very much, especially something that you con not have'. (Sulk) The other options are correctly speltVictimize - Excruciate, Veteran - Adept, Commotion - clutter, Thus, the correct answer is option (d). 602. Select the incorrectly spelt word. (a) Arena (b) Journal (c) Yall (d) Visual SSC GD – 18/02/2019 (Shift-II) Ans. (c) : The incorrectly spelt word is options (c) 'Yall'. The correctly spelt word is 'yell' which means- 'to say something very loudly especially because you are angry, surprised, or trying to get someone's attention. (cry) The other options are correctly spelt Arena - Amphitheater, Journal - Magazine, visual - Scene, Thus, the correct answer is option (c).

Z

603. Select the incorrectly spelt word. (a) Clutch (b) Zelot (c) Precise (d) Cause SSC CGL (Tier-I) – 04/06/2019 (Shift-II) Ans. (b) : The incorrectly spelt word from the given option is (b) 'zelot'. The correctly spelt word is 'zealot' which means - 'one who is passionately devoted to cause'. (Bigot) The other options are correctly speltClutch - Hold, Precise - Appropriate, 599. Select the INCORRECTLY spelt word. Cause - Reason, (a) Batch (b) Catch Hence, the correct answer is option (b). (c) Yatch (d) Snatch 604. Select the incorrectly spelt word. SSC Constable GD-03/12/2021 Shift-II (a) Kudos (b) Zeast Ans. (c) : The word 'yatch' is incorrect because the (c) Acute (d) Rare correct spelling is 'yacht'. SSC MTS – 09/08/2019 (Shift-I) Thus, the correct answer is options (c). Ans. (b) : The incorrectly spelt word is option (b) 600. Select the INCORRECTLY spelt word. 'zeast'. The correctly spelt word is 'zest' which means (a) ceiling (b) grief 'a feeling of enjoyment, excitement and enthusiasm'. (c) yeild (d) niece The other options are correctly speltSSC CPO-SI – 13/12/2019 (Shift-II) Kudos - Glory, Ans. (c) : The incorrectly spelt word is options (c) Acute Intense, 'yeild'. the correctly spelt word is 'yield' which means - Rare - Scantily, 'to produce or provide crops, profits or results'. Hence, the correct answer is option (b).

Y

Misspelt word

572

YCT

09. ONE WORD SUBSSTITUTION TCS hewšve& hej DeeOeeefjle (Based On TCS Pattern) Chapterwise

Exam

Question No.

1

CGL (Tier-1) CGL (Tier-2) CHSL (Tier-1) CHSL (Tier-2) Selection Post VII, VIII, XI SSC MTS SSC GD SSC CPO SI CGL (Tier-1) CGL (Tier-2) CHSL (Tier-1) CHSL (Tier-2) Selection Post VII, VIII, XI SSC MTS SSC GD SSC CPO SI CGL (Tier-1) CGL (Tier-2) CHSL (Tier-1) CHSL (Tier-2) Selection Post VII, VIII, XI SSC MTS SSC GD SSC CPO SI CGL (Tier-1) CGL (Tier-2) CHSL (Tier-1) CHSL (Tier-2) Selection Post VII, VIII, XI SSC MTS SSC GD SSC CPO SI CGL (Tier-1) CGL (Tier-2) CHSL (Tier-1) CHSL (Tier-2) Selection Post VII, VIII, XI SSC MTS SSC GD SSC CPO SI

12 5 17 8 6 11 1 2 3 2 1 1 2 5 1 4 7 5 8 6 4 7 2 5 4 3 8 5 5 5 2 5 7 1 4 5 2 3 1 7

CGL (Tier-1) CGL (Tier-2) CHSL (Tier-1) CHSL (Tier-2)

3 2 2 3

A

2

B

3

C

4 D

5

E

6

One Word Substitution

573

Years

(2017–2023)

(2017–2023)

(2017–2023)

(2017–2023)

(2017–2023)

(2017–2023) YCT

F

7

G

8

H

9

I

10

J

11

K

12 L

13

One Word Substitution

Selection Post VII, VIII, XI SSC MTS SSC GD SSC CPO SI CGL (Tier-1) CGL (Tier-2) CHSL (Tier-1) CHSL (Tier-2) Selection Post VII, VIII, XI SSC MTS SSC GD SSC CPO SI CGL (Tier-1) CGL (Tier-2) CHSL (Tier-1) CHSL (Tier-2) Selection Post VII, VIII, XI SSC MTS SSC GD SSC CPO SI CGL (Tier-1) CGL (Tier-2) CHSL (Tier-1) CHSL (Tier-2) Selection Post VII, VIII, XI SSC MTS SSC GD SSC CPO SI CGL (Tier-1) CGL (Tier-2) CHSL (Tier-1) CHSL (Tier-2) Selection Post VII, VIII, XI SSC MTS SSC GD SSC CPO SI CGL (Tier-1) CGL (Tier-2) CHSL (Tier-1) CHSL (Tier-2) Selection Post VII, VIII, XI SSC MTS SSC GD SSC CPO SI CGL (Tier-1) CGL (Tier-2) CHSL (Tier-1) CHSL (Tier-2) Selection Post VII, VIII, XI SSC MTS SSC GD SSC CPO SI CGL (Tier-1) CGL (Tier-2) CHSL (Tier-1) 574

4 2 1 3 2 1 3 1 1 1 2 2 1 2 1 1 2 1 2 3 1 4 2 4 3 3 3 1 1 1 1 1 1 1 1 2 1 2 1 1 3 1 3 4 3 2

(2017–2023)

(2017–2023)

(2017–2023)

(2017–2023)

(2017–2023)

(2017–2023)

YCT

M

14

N

15 O

16 P

17 Q

18

R

19

S

20 One Word Substitution

CHSL (Tier-2) Selection Post VII, VIII, XI SSC MTS SSC GD SSC CPO SI CGL (Tier-1) CGL (Tier-2) CHSL (Tier-1) CHSL (Tier-2) Selection Post VII, VIII, XI SSC MTS SSC GD SSC CPO SI CGL (Tier-1) CGL (Tier-2) CHSL (Tier-1) CHSL (Tier-2) Selection Post VII, VIII, XI SSC MTS SSC GD SSC CPO SI CGL (Tier-1) CGL (Tier-2) CHSL (Tier-1) CHSL (Tier-2) Selection Post VII, VIII, XI SSC MTS SSC GD SSC CPO SI

2 3 2 1 2 1 1 2 1 2 1 3 2 3 2 2 1 1 2 3 2 4 8 2 3 2 2

CGL (Tier-1) CGL (Tier-2) CHSL (Tier-1) CHSL (Tier-2) Selection Post VII, VIII, XI SSC MTS SSC GD SSC CPO SI CGL (Tier-1) CGL (Tier-2) CHSL (Tier-1) CHSL (Tier-2) Selection Post VII, VIII, XI SSC MTS SSC GD SSC CPO SI CGL (Tier-1) CGL (Tier-2) CHSL (Tier-1) CHSL (Tier-2) Selection Post VII, VIII, XI SSC MTS SSC GD SSC CPO SI CGL (Tier-1)

1 1 1 1 1 1 3 2 3 1 3 2 2 5 3 8 13 5 1 1 2 4 2

575

(2017–2023)

(2017–2023)

(2017–2023)

(2017–2023)

(2017–2023)

(2017–2023)

(2017–2023)

YCT

T

21

U

22

V

23

W

24

X

25

Y

26

Z

One Word Substitution

CGL (Tier-2) CHSL (Tier-1) CHSL (Tier-2) Selection Post VII, VIII, XI SSC MTS SSC GD SSC CPO SI CGL (Tier-1) CGL (Tier-2) CHSL (Tier-1) CHSL (Tier-2) Selection Post VII, VIII, XI SSC MTS SSC GD SSC CPO SI CGL (Tier-1) CGL (Tier-2) CHSL (Tier-1) CHSL (Tier-2) Selection Post VII, VIII, XI SSC MTS SSC GD SSC CPO SI CGL (Tier-1) CGL (Tier-2) CHSL (Tier-1) CHSL (Tier-2) Selection Post VII, VIII, XI SSC MTS SSC GD SSC CPO SI CGL (Tier-1) CGL (Tier-2) CHSL (Tier-1) CHSL (Tier-2) Selection Post VII, VIII, XI SSC MTS SSC GD SSC CPO SI CGL (Tier-1) CGL (Tier-2) CHSL (Tier-1) CHSL (Tier-2) Selection Post VII, VIII, XI SSC MTS SSC GD SSC CPO SI CGL (Tier-1) CGL (Tier-2) CHSL (Tier-1) CHSL (Tier-2) Selection Post VII, VIII, XI SSC MTS SSC GD SSC CPO SI 576

3 2 3 2 1 1 1 1 1 2 1 1 1 1 1 1 1 1 1 1 2 1 1 1 1 1 1 2 1 1 1 1 1 1 1 1 1 1 1 1

(2017–2023)

(2017–2023)

(2017–2023)

(2017–2023)

(2017–2023)

(2017–2023)

(2017–2023)

YCT

Analysis of Questions topicwise from CGL (Pre & Mains) CHSL (Pre & Mains) Selection Post VII, VIII, XI, SSC MTS, SSC GD & Other Exams (2017-2023)

One Word Substitution

577

YCT

09. ONE WORD SUBSTITUTION A 1.

Choose the word that can substitute the given group of word. A person appointed by two parties to solve a problem. (a) Jury (b) Advocate (c) Judge (d) Arbitrator SSC MTS 08/05/2023 (Shift-III) Ans. (d) : For the above given group of words 'A person appointed by two parties to solve a problem' is known as ' Arbitrator'. Meaning of the other words are – Jury – A body of person appointed to hear evidence and give their verdict in trails. Advocate – A lawyer who defends somebody in court of law. Judge - A person who presides over court proceedings, either or as a part of a panel of judges. 2. Select the most appropriate word for the given group of words. Extreme physical or mental sufferings (a) Aviary (b) Autonomy (c) Autocracy (d) Agony SSC MTS 04/05/2023 (Shift-II) Ans. (d) : The most appropriate word for the given group of words 'Extreme physical or Mental sufferings is 'Agony'. Aviary - A place where birds are kept. Autocracy - A country that is ruled by one person who has complete power. Autonomy - The state of being self governing. 3. Choose the word that can substitute the given group of words. A government run by a single person (a) Anocracy (b) Gerontocracy (c) Bureaucracy (d) Autocracy SSC MTS 16/05/2023 (Shift-II) Ans. (d) : The meaning of the above given group of words is 'Autocracy.' Meaning of other optionsGerontocracy- a state or group or group governed by old people. Bureaucracy- a system of government by a large number of officials who are not elected. 4. Choose the word that can substitute the given group of words. A person who does not believe in God (a) Iconoclast (b) Highbrow (c) Atheist (d) Lecher SSC MTS 15/05/2023 (Shift-I) One Word Substitution

Ans. (c) : 'Atheist' is the suitable word for the above given group of words – A person who does not believe in God'. Meaning of other options – Iconoclast - One who breaks the established traditions and image. Highbrow - A person who is only interested in serious art or complicated subject. Lecher - A man who engage in lechery. 5. Select the most appropriate word for the given group of words. Open to more than one interpretation, not having one obvious meaning (a) Altercation (b) Ambiguous (c) Alchemy (d) Amateur SSC MTS 12/05/2023 (Shift-III) Ans. (b) : For the given group of words, the most appropriate option is 'Ambiguous' which means – 'having or expressing more than one possible meaning of interpretations; equivocal. Altercation - A noisy, heated, angry disputed also Alchemy - A form of chemistry in the middle age which involved trying to discover how to change ordinary metals into gold. Amateur - An unprofessional who is unqualified or insufficiently skilled for a task or job. 6. Select the option that can be used as a oneword substitute for the given group of words. Life history written by oneself. (a) Autobiography (b) Autocracy (c) Autography (d) Autophagy SSC CGL (Tier-I) 26/07/2023 (Shift-II) Ans. (a) : The most appropriate one word substitute for the given group of words 'Life history written by one self' is 'autobiography'. Autocracy - A system of government & country in which one person has complete power. Autography - A persons own handwriting Autophagy - A natural process in which body's cell clean ou any danaged or unnecessary components. 7. Select the option that can be used as a oneword substitute for the given group of words. A large cage, building, or enclosure to keep birds (a) Apiary (b) Burrow (c) Aviary (d) Dormitory SSC CGL (Tier-I) 20/07/2023 (Shift-I) Ans. (c) : The most appropriate ‘one-word substitute for the given group of words ‘A large cage, building , or enclosure to keep birds’ is Aviary. The meaning of other words are–

578

YCT

Apiary - A place where bees are kept and managed for the purpose of raising bees. Burrow - Excavation, Hole. Dormitory - Living quarters. Thus, the correct answer is option (c). 8. Choose the word that can substitute the given group of words. A person who given up their beliefs (a) Misanthrope (b) Versatle (c) Apostate (d) Crusade SSC CHSL (Tier-I) 20/03/2023 (Shift-I) Ans. (c) : For the above group of words "A person who has given up their beliefs", option (c) 'Apostate' is appropriate one word. 9. Select the most appropriate 'one word' for the expression given below. Study of human being and their ancestors through time. (a) Anthropology (b) Physiology (c) Biology (d) Ethnology SSC CHSL (Tier-I) 15/03/2023 (Shift-I) Ans. (a) : In the above group of word 'Study of human being and their ancestors through time' is option (a) 'Anthropology' gives an appropriate meaning. Other options give different meaning. Biology → the scientific study of living things Ethnology → the scientific study and comparision of Human races. Physiology → the scientific study of Low living things function. 10.

Select the option that can be used as a oneword substitute for the given group of words. A tank for fish or water plants (a) Aquarium (b) Museum (c) Reservoir (d) Aviary SSC CGL (Tier-I) 13/04/2022 Shift-III Ans. (a) : For the above group of words ‘A tank for fish or water plants’, option (a) Aquarium is appropriate one word.

11.

Select the option that can be used as a oneword substitute for the given group of words. A remedy to counteract the effects of poison (a) Antibody (b) Antidote (c) Antiseptic (d) Antibiotic SSC CHSL 27.05.2022 Shift-II Ans. (b) : For the above group of words ‘A’ remedy to counteract the effects of poison’, option (b) Antidote is appropriate one word. Other options have different meaning. Antibody - A protective protein produced by the immune system in response to the presence of a foreign substance. Antiseptic - A liquid or cream prevents a cat etc from becoming infected. Antibiotic - A medicine which is used for destroying bacteria and caring infections.

One Word Substitution

12.

Select the option that can be used as a oneword substitute for the given group of words. A fictional being from another world (a) Foreigner (b) Alien (c) Stranger (d) Native SSC CGL (Tier-I) 12/04/2022 Shift-II Ans. (b) : For the above group of words, ‘A fictional being form another world’. Option (b) Alien is appropriate one word. Foreigner - A person who belongs to a country that is not your own. Stranger - A person that you do not know Native - A person who was born in a particular place. 13. Select the option that can be used as a oneword substitute for the given group of words. Sharpness and accuracy of judgment (a) Experience (b) Estimation (c) Knowledge (d) Acumen SSC CGL (Tier-I) 13/04/2022 Shift-II Ans. (d) : For the above group of the words ‘ Sharpness and accuracy of Judgment’. Option (d) acumen is appropriate one word. Other options have different meaning. Experience - The knowledge and skill that you have gained through doing something for a period of time. Estimation - Opinion of someone or something. Knowledge - A state of knowing about a particular fact or situation. 14. Select the option that can be used as a oneword substitute for the given group of words. To walk aimlessly (a) Slither (b) Amble (c) Crawl (d) Sprint SSC CGL (Tier-I) 11/04/2022 Shift-I Ans. (b) : For the above group of words, ‘To walk aimlessly’ option (b) Amble is appropriate one word. Other options have different meaning. Slither – To move by sliding from side to side along the ground like a snake Crawl – To move slowly with your body on or close to the ground, or on your hands and knees. 15. Select the option that can be used as a oneword substitute for the given group of words. A glass container to keep fish (a) Atrium (b) Vase (c) Aquarium (d) Beaker SSC CGL (Tier-I) 11/04/2022 Shift-III Ans. (c) : For the above group of words, ‘ A glass container to keep fish’, option (C) Aquarium is appropriate one word. Atrium - A large open central area in public or commercial building. Vase - A container that is used for holding cut flowers. Aquarium - A glass container filled with water in which fish and water animals are kept. 16. Select the option that can be used as a oneword substitute for the given group of words. One who is not sure about God's existence (a) Agnostic (b) Anchor (c) Pantheist (d) Haggard SSC MTS-07/07/2022 Shift-II

579

YCT

Ans. (a) : For the above group of words, one who is not sure about God’s existence, option (a) Agnostic is appropriate one word. Anchor - To fix something firmly so that it cannot move Pantheist - A person who belives in many or all gods. Haggard - Looking very tired because of illness. 17. Select the option that can be used as a oneword substitute for the given group of words. A person who believes in or tries to bring about a state of lawlessness (a) Ephemeral (b) Deluge (c) Gazette (d) Anarchist SSC MTS-12/07/2022 Shift-I Ans. (d): For the above group of words, ‘A person who believes in or tries to bring about a state of lawlessness’, option (d) Anarchist is appropriate one word. Ephemeral - Lasting or used for only a short period of time. Deluge - A sudden very heavy fall of rain. Gazette - A word used in the titles of some newspaper. 18. Select the option that can be used as a oneword substitute for the underlined group of words. Several literary pieces of the Old English period are written by authors whose names are not known. (a) Anonymous (b) Undercover (c) Anomalies (d) Renowned SSC CHSL 27.05.2022 Shift-I Ans. (a) : For the above underlined group of words ‘whose names are not known’, option (a) Anonymous is appropriate one word. Other options have different meaning Undercover - Working done secretly in order to find out information for the police. Anomalies - something that is different from what is normal or usual. Renowned - Famous for something. 19. Select the option that can be used as a oneword substitute for the given group of words. Formally put an end to a system, practice, or institution (a) Kill (b) Destroy (c) Abolish (d) Stop SSC CGL (Tier-I) 12/04/2022 Shift-I Ans. (c) : For the above group of words, option (C) Abolish is appropriate one word. Kill - To make somebody die. Destroy - To damage something so badly that it can no longer be used. Stop - To finish doing something that you were doing. 20. Select the option that can be used as a oneword substitute for the given group of words. Deliberately and maliciously set something (buildings usually) on fire (a) Agrarian (b) Arson (c) Ambush (d) Apex SSC MTS-06/07/2022 Shift-I One Word Substitution

Ans. (b) : For the above group of words, option (b) Arson is appropriate one word. Other options have different meaning. Agrarian – Connected with farming and the use of land for farming. Ambush – A surprise attack from a hidden position. Apex –The top or highest part of something. 21. Select the option that can be used as a oneword substitute for the given group of words. One who is hostile and violent (a) Idle (b) Aggressive (c) Legendary (d) Agreeable SSC MTS-07/07/2022 Shift-I Ans. (b) : For the above group of words, ‘one who is hostile and violent’, option (b) Aggressive is appropriate one word. Idle - Not working or being used Legendary - Very famous and admired or spoken about Agreeable - Able to be accepted by everyone 22. Select the option that can be used as a oneword substitute for the given group of words. A place where government records are kept (a) Cloakroom (b) Cache (c) Hangar (d) Archive SSC MTS-05/07/2022 Shift-III Ans. (d) : For the above group of words, options (d) Archive is appropriate one word. Cloakroom - A room near the entrance to building where you can leave your coat, bags, etc. Cache - A hiding place especially one in the ground, for ammunition, food, treasures. Hangar - A big building where planes are kept. 23. Select the option that can be used as a oneword substitute for the given group of words. Become less intense or widespread (a) Ablaze (b) Abate (c) Abide (d) Abode SSC Constable GD-29/11/2021 Shift-II Ans. (b) : For the above group of words, ‘Become less intense or widespread’ , option (b) Abate is appropriate one word. Ablaze - In the process of burning on fire Abide - Be able to tolerate (someone or something) Abode - The place where you live 24.

Select the option that can be used as a oneword substitute for the given group of words. Person who is not an expert in a field of art or music (a) Amateur (b) Patron (c) Eccentric (d) Pariah SSC Constable GD-01/12/2021 Shift-I Ans. (a) : For the above group of words, ‘ Person who is not an expert in a field of art or music’ option (a) Amateur is appropriate one word. Patron - A person who support with money, gifts, efforts etc. Eccentric - Strange or unusual behavior of somebody. Pariah - A person who is not accepted by a social group.

580

YCT

25.

Select the option that can be used as a oneword substitute for the given group of words. Lack of feeling (a) Empathy (b) Apathy (c) Telepathy (d) Sympathy SSC Constable GD-25/11/2021 Shift-III Ans. (b) : For the above group of words’, ‘Lack of feeling option (b) ‘Apathy’ is appropriate one word. Other options have different meaning. Empathy - The ability to understand and share the feeling of another. Telepathy - The ability to know what is in someone else's mind. Sympathy - An Understanding of other people's feeling especially their problems. 26. Select the option that can be used as a oneword substitute for the given group of words. The branch of physics concerned with the properties of sound (a) Optics (b) Photonics (c) Mechanics (d) Acoustics SSC CHSL (Tier-I) –05/08/2021 (Shift-I) Ans. (d) : Above group of word's The branch of physics of sound means - Acoustics. Hence option (d) 'Acoustics' would be oppropirate other optionsMachanics- A person whose job is repairing machines, especially the engines of vehicles. Photonics - It is the physical science of light waves. Optics - the scientific study of sight and light. 27. Select the option which means the same as the group of words given. Animal that can live both on land and in water (a) Arboreal (b) Amphibian (c) Vertebrate (d) Mammal SSC CHSL (Tier-I) –15/04/2021 (Shift-I) Ans. (b) : Above group of word's Animal that can live both on land and in water' would be ambhibian. The meaning of the other options isVertebrate - (of an animal) having bone. Arboreal - relating to tree; living in trees. Mammal - Any animal of which the female feeds her young on milk from her own body. 28. Select the option that can be used as a oneword substitute for the given group of words. One who disbelieves the existence of God (a) Polytheist (b) Atheist (c) Monotheist (d) Theologist SSC Constable GD-06/12/2021 Shift-III Ans. (b) : For the above group of words, ‘ One who disbelieves the existence of God’ option (b) Atheist is appropriate one word. Other options have different meaning. Polytheist - Someone who believes in many god. Monotheist - Someone who believes that there is only one god. Theologist - Someone who is learned in theology or who speculates about theology. 29. Select the word which means the same as the group of words given. A person who presents and acts as a link in a radio or news programme One Word Substitution

(a) programmer (b) disc jockey (c) anchor (d) actor SSC CHSL (Tier-I) –15/04/2021 (Shift-I) Ans. (c) : The above group of words' A person who presents and acts as a link in a radio or news programme called - Anchor The meaning of the other options is Disc jockey - person who plays popular music Programmer - A person whose job is writing programs for computers. Actor- A person who performs on the stage, on the television or in films movies. 30. Select the option that can be used as a oneword substitute for the given group of words. A person who presents a radio/television programme. (a) Idol (b) Anchor (c) Speaker (d) Star SSC CHSL (Tier-I) –09/08/2021 (Shift-I) Ans. (b) : Above group of words 'A person who presents a radio/television programme would be 'Anchor (displayer)'. The meaning of other options Star- A large ball of burning gas in space that we see as a point of light in the sky at night. Idol- A person or thing that is loved and admired very much. 31. Select the option that can be used as a oneword substitute for the given group of words. One who does not believe in the existence of God. (a) Atheist (b) Cynic (c) Anarchist (d) Egotist SSC CGL (Tier-I) –20/08/2021 (Shift-I) Ans. (a) : Above group of words, 'One who does not believe in the existence of god' - 'Atheist' (Atheist). Other options have different meaningCynic- A person who does not believe that something good will happen. Anarchist - A person who believes that laws and government are not necessary. 32. Select the most appropriate one-word substitution for the given words. A government by the nobles (a) Democracy (b) Oligarchy (c) Aristocracy (d) Monarchy SSC CGL (Tier-II) 16/11/2020 (3 pm - 5 pm) Ans. (c) : The above group of word's A government by nobles' - Aristocracy. Aristocracy would be appropriate one word. Other options have different meaningDemocracy - A system of government in which all the people of a country can vote to elect their representative. Oligarchy - A form of government in which only a small group of people hold all the power. Monarchy - The system of government or rule by a king or queen. 33. Select the most appropriate one-word substitution for the given words. A partner in crime (a) Accomplice (b) Alibi (c) Amateur (d) Agent SSC CGL (Tier-II) 16/11/2020 (3 pm - 5 pm)

581

YCT

Ans. (a) : Above group of words'A partner in crime' - Accomplice. Hence, option (a) 'Accomplice' will be appropriate. Other options have different meaningAmateur - A person who is not skilled. Alibi - An excuse for something that you have done wrong. Agent - A person whose job is to act for, or manage the affairs of other people in business etc. 34. Select the word which means the same as the group of words given. A person who does not believe in God (a) spiritualist (b) atheist (c) hypocrite (d) sceptic SSC CPO-SI (Tier-II) 27/09/2019 (3 pm - 5 pm) Ans. (b) : The above group of words 'A person who does not believe in god - Atheist Hence, option (b) Atheist would be appropriate. Other options have different meaningHypocrite - A person who pretends to have moral standards or opinion that they do not actually have. Sceptic - A person who usually doubts that claims or statements are true, especially those that other people believe in. 35. Select the word which means the same as the group of words given. Branch of physics dealing with the properties of sound (a) mechanics (b) radiation (c) acoustics (d) audition SSC CPO-SI (Tier-II) 27/09/2019 (3 pm - 5 pm) Ans. (c) : The above group of words, branch of physics dealing with the properties of sound' Acoustics. One word 'acoustics' would be appropriate. Other options have different meaning. Mechanics - A person whose job is repairing machines, especially the engines of vehicles. Radiateion - powerful and very dangerous rays that are sent out from radioactive substances. Audition - A short performance given by an actor, a singer etc. 36. In the following question, out of the four alternative, select the alternative which is the best substitute of the phrase. Killing of birds (a) Herbicide (b) Avicide (c) Vulpicide (d) Matricide SSC CGL (Phase-II) 17/02/2018 Ans. (b) The above group of words 'killing of birds means - Avicide. The meaning of the other wordsHerbicide- a chemical that is poisonous to plants, used to kill plants that are growing where they are not wanted. Vulpicide - A person killing a fox by means other than those of hunting with hounds. Matricide - the crime of killing your mother. 37. Select the most appropriate one-word substitution for the given group of words. A person who is trained for travelling into space (a) scientist (b) physicist (c) astronaut (d) pilot SSC CPO-SI – 24/11/2020 (Shift-I) One Word Substitution

Ans. (c) Given group of words, 'A person who is trained for travelling into space' is - Astronaut. The meaning of other optionsScientist - A person who studies one or more of the natural science (for e.g. physics, chemistry) Physicist - A scientist who studies physics. Pilot - A person who operates the controls of an aircraft, especially as a job. 38. Select the most appropriate word which means the same as the group of words given. Extreme mental or physical suffering (a) agony (b) acute (c) rapture (d) ecstasy SSC CPO-SI – 23/11/2020 (Shift-I) Ans. (a) : Given group of words, 'extreme mental or physical suffering, is - Agony. The meaning of the other options is Acute - intelligent and quick to notice and understand things. Rapture - a feeling of extreme pleasure and happiness. (delight) Ecstasy - bliss, a feeling or state of very great happiness. 39. Select the word which means the same as the group of words given. A person appointed by two parties to resolve a dispute (a) broker (b) valuer (c) auditor (d) arbitrator SSC CGL (Tier-I) – 06/06/2019 (Shift-III) Ans. (d) : Given group of words, 'A person appointed by two parties to resolve a dispute is - Arbitrator. The meaning of the other optionsValuer - a person whose job is to estimate how much property, land, etc is worth. Auditor - A person who officially examines the business and financial records of a company. Broker - A person who buys and sells things for other people. 40. Select the word which means the same as the group of words given. A place for storing guns and military equipment (a) apiary (b) arsenal (c) archive (d) aviary SSC CGL (Tier-I) – 11/06/2019 (Shift-II) Ans. (b) : The given group of words, 'a place for storing guns and military equipment is - Arsenal. The meaning of other optionsApiary - A place where bees are kept. Archive - A collection of historical documents or records of a government. aviary - A large building for keeping birds. 41. Select one word substitution for the given words. To give up the throne (a) bequeath (b) usurp (c) consign (d) abdicate SSC CGL (Tier-I) – 07/06/2019 (Shift-III) Ans : (d) The given group of words, 'To give up the throne' is - Abdicate. Meaning of other optionsConsign- to give or send something to somebody.

582

YCT

Usurp - to take somebody's position and or power without having the right to do this. Bequeath - to say in will that you want somebody to have your property, money etc after you die. (leave). 42. Select the word which means the same as the group of words given. An enclosure to keep the birds in (a) stable (b) apiary (c) sanctuary (d) aviary SSC CGL (Tier-I)–06/06/2019 (Shift-I) Ans. (d) : The given group of words, 'An enclosure to keep the birds in' is - Aviary. the meaning of the other optionsStable - a building in which horses are kept. Apiary - a place where bees are kept. Sanctuary - an area where wild birds or animals are protected and encouraged to bread. 43. Select the option which is the most appropriate one word substitution for the given group of words. One who devotes one's life to the welfare of other people. (a) alchemist (b) altruist (c) ambassador (d) martyr SSC CPO-SI – 11/12/2019 (Shift-II) Ans : (b) One word substitution for the given group of words' one who devotes one's life to the welfare of other people is - Altruist. The meaning of the other wordsAlchemist - a person who studied alchemy. Ambassador - an official who lives in a foreign country as the senior representative. 44. Select the option which is the most appropriate one word substitution for the given group of words. One who denies oneself ordinary bodily pleasures. (a) ascetic (b) aesthetic (c) atheist (d) theist SSC CPO-SI – 11/12/2019 (Shift-II) Ans : (a) One word substitution for given group of the words, 'One who denies oneself ordinary bodily pleasure is 'Ascetic'. Other wordsAesthetic- concerned with beauty and art and the understanding of beautiful things. Atheist - A person who believes that god does not exist. Theist - A person who believes in the existence of a god. 45. Select the option which is the most appropriate one word substitution for the given group of words. A shortened form of a word or phrase (a) abbreviation (b) pseudonym (c) precis (d) summary SSC CPO-SI – 11/12/2019 (Shift-II) Ans : (a) The given group of words, 'A shortened form of a word or phrase is - Abbreviation. The meaning of the other words Precis - A short version of a speech or a piece of writing that gives the main points or idea. Summary - A short statement that gives only the main points of something, not the detail. One Word Substitution

46.

Select the most appropriate option that can be used as a one-word substitute for the given group of words. A list of items to be discussed at a formal meeting. (a) topics (b) programme (c) agenda (d) timetable SSC CPO-SI – 12/12/2019 (Shift-II) Ans : (c) One-word substitution for the given group of words. A list of items to be discussed at a formal meeting is - Agenda. The meaning of other words is Topic - a subject that you talk, write or learn about. Programme - a plan of things that will be done. Timetable - A detailed plan showing when events or activities will happen. 47. Select the most appropriate option that can be used as a one-word substitute for the given group of words. A sea or stretch of water having many islands. (a) ocean (b) harbour (c) bay (d) archipelago SSC CPO-SI – 12/12/2019 (Shift-II) Ans : (d) One word substitution for the given group of words, 'A sea or stretch of water having many islands' is - Archipelago. Ocean - The mass of salt water that covers most of the earth's surface. Harbour - A sheltered port where ships can take on or discharge cargo. Bay- a part of the sea or of a large lake, partly surrounded by a wide curve of the land. 48. Select the option which is the most appropriate one word substitution for the given group of words. Statement open to more than one interpretation. (a) ambidextrous (b) ambushed (c) ambiguous (d) ambrosial SSC CPO-SI – 12/12/2019 (Shift-I) Ans : (c) One word substitution of the given group of words, 'statement open to more than one interpretation is - Ambiguous. The meaning of the other words is Ambidextrous - able to use the left hand or the right hand equally well. Ambushed- to make a surprise attack on somebody/ something from a hidden position. Ambrosial - Having a very pleasant taste or smell. 49. Select the option which is the most appropriate one word substitution for the given group of words. A place where weapones and ammunitions are stored (a) Godown (b) Museum (c) Arsenal (d) Hangar SSC CPO-SI – 13/12/2019 (Shift-I) Ans. (c) : One word substitution for the given group of words, 'A place where weapones and ammunitions are stored' is - Arsenal. The meaning of the other words. Godown - a warehouse (building where goods are stored).

583

YCT

Hangar - a large building in which aircraft are kept. Museum - a building in which objects of artistic cultural historical or scientific interest are kept and shown to the public. 50. Select the option which is the most appropriate one word substitution for the given group of words. Giving up one's authority or throne (a) Abdication (b) Abjection (c) Abduction (d) Abjuration SSC CPO-SI – 13/12/2019 (Shift-I) Ans. (a) : The given group of words, 'Giving up one's authority or throne is - Abdication. The meaning of other options is Abjection - a low or downcast state. Abduction - the act of taking somebody away illegally, especially using force. Abjuration - giving up, renunciation. 51. Select the word which means the same as the group of words given. One who can use both hands efficiently. (a) Ambidextrous (b) Skillful (c) Genius (d) Ambivalent SSC CHSL (Tier-I) –11/07/2019 (Shift-III) Ans : (a) The given group of words, 'One who can use both hands efficiently' is - Ambidextrous. the meaning of other options is Skillful - (of a person) good at doing something especially something that a particular ability. Genius - Showing unusually great intelligence. Ambivalent - Having or showing both good and bad felling about somebody something. 52. Select the word which means the same as the group of words given. The original inhabitants of a country (a) slaves (b) immigrants (c) aborigines (d) residents SSC CHSL (Tier-I) –10/07/2019 (Shift-III) Ans : (c) The given group of words, 'the original in habitants of a country is 'Aborigines'. The meaning of the other options isSlaves - a person who is owned by another person and is forced to work for them. Immigrant - a person who has come to live permanently in a country that is not these own. Residents - A person who is staying in a hotel or who has their home there. 53. Select the most appropriate word for the given group of words. A word formed from the initial letters of other words and pronounced as a word (a) ellipsis (b) precise (c) epitome (d) acronym SSC CHSL (Tier-I) –09/07/2019 (Shift-I) Ans : (d) The given words, 'A word formed from the initial letter of others words and pronounced as a word is Acronym. Ellipsis - The act of leaving out a word or words from a sentence deliberately, when the meaning can be understood without them. Epitome - a perfect example of something. Precise - Very careful and accurate. One Word Substitution

54.

Select the most appropriate word for the group of words. An official pardon (a) regicide (b) redemption (c) amnesty (d) amnesia SSC CHSL (Tier-I) –08/07/2019 (Shift-I) Ans : (c) The given group of words, 'An official pardon is - 'Amnesty'. The meaning of other options is Regicide - The crime of killing a king or queen Redemption - the act of saving or state of being saved from the power of evil. Amnesia - a medical condition in which somebody partly or completely loses their memory. 55. Select the most appropriate word for the group of words. Willing to be guided or controlled (a) abominable (b) amenable (c) adamant (d) adorable SSC CHSL (Tier-I) –08/07/2019 (Shift-II) Ans : (b) The given group of words, 'willing to be guided or controlled is - 'Amenable'. The meaning of other options isAbominable - Extremely complaisant and causing disgust. Adamant - determined not to change air mind. Adorable - very attractive and easy to feel love for. 56. Select the word which means the same as the group of words given. Living in air (a) heavenly (b) feathery (c) aerial (d) visual SSC CHSL (Tier-I) –03/07/2019 (Shift-II) Ans. (c) : The given group of words 'Living in air' is Aerial. The meaning of the other options isHeavenly - Connected with the heaven. Feathery - Light and soft. Visual - Connected with seeing or sight. 57. Select the word which means the same as the group of words given. The study of human history and prehistory through the excavation of sites (a) archaeology (b) psephology (c) anthropology (d) geology SSC CHSL (Tier-I) –03/07/2019 (Shift-III) Ans. (a) : The given group of words, the study of human history and prehistory through the excavation of sites' is - archaeology. The meaning of other options is Psephology - The study of how people vote in elections. Anthropology - The study of the human race, especially of its origin, development, custom and beliefs. Geology - The scientific study of the earth, including the origin and history of rocks and soil. 58. Select the word which means the same as the group of words given. A person belonging to a foreign country (a) native (b) resident (c) alien (d) inhabitant SSC CHSL (Tier-I) –01/07/2019 (Shift-III)

584

YCT

Ans : (c) The given group of words, 'A person belonging to foreign country' is Alien. The meaning of other options isResident - A person who lives in a particular place or who has their home there. Native - A person who was born in a particular country or area. Inhabitant - A person or animal that lives in a particular place. 59. Select the word which means the same as the group of words given. A person who is neither well experienced nor professional (a) Veteran (b) Expert (c) Amateur (d) Proficient SSC CHSL (Tier-I) 18/03/2020 (Shift-I) Ans. (c) : The given group of words 'A person who is neither well experienced nor professional is - Amateur. The meaning of other options is Veteran - a person who has a lot of experience in a particular area or activity. Expert - A person with special knowledge, skill or training in something. Proficient - able to do something well because of training and practice. 60. Select the word which means the same as the groups of words given. The life history of a person written by himself. (a) Travelogue (b) Essay (c) Biography (d) Autobiography SSC CHSL (Tier-I) 26/10/2020 (Shift-I) Ans. (d) : The given group of words, 'The life history of a person written by himself, is - Autobiography The meaning of the other options is Travelogue - a film/movie, broadcast or piece of writing about travel. Essay - a short piece of writing on a particular subject, writing in order to be published. Biography - the story of a person's life written by somebody else. 61. Select the word which means the same as the group of words given. One who examines accounts officially (a) Collector (b) Radiator (c) Auditor (d) Aviator SSC CHSL (Tier-I) 19/10/2020 (Shift-I) Ans. (c) : The given group of words, 'One who examines accounts officially - is 'Auditor' The meaning of the other options is Collector - a person who collects things, either as a hobby or as a job. Radiator - a device for cooling the engine of a vehicle or an aircraft. Aviator - a person who flies an aircraft. 62. Select the word which means the same as the group of words given. Repetition of an initial sound in two or more words of a line. (a) Rhyme (b) Assonance (c) Alliteration (d) Rhythm SSC CHSL (Tier-I) 19/10/2020 (Shift-I) One Word Substitution

Ans. (c) : The given group of words, 'Repetition of an initial sound in two or more words of a line' is Alliteration. The meaning of other options is Assonance - it is a figure of speech that is characterized by the use of words having similar vowel sounds consecutively. It can be said to be a variation of alliteration. Rhyme - a words that has the same sand or ends with the same sound as another words. rhythm - a strong regular respected pattern of sounds or movements. 63. Select the most appropriate segment to substitute the underlined segment of the given sentence. In the jungle, I saw a group of lions. (a) a pride (b) a school (c) a herd (d) a flock SSC CHSL (Tier-I) 15/10/2020 (Shift-II) Ans. (a) : The given group of words, 'A group of lions' is 'A pride'. The meaning of other options is Herd - a herd of cows, a herd of great, a herd of deer. School - a school of fish. Flock - a flock of sheep/bird. 64. Select the word which means the same as the group of words given. One who tells future form the stars (a) Miser (b) Personification (c) Astrologer (d) Philanthropist SSC CHSL (Tier-I) 14/10/2020 (Shift-II) Ans. (c) : The given group of words, 'One who tells future form the stars' is - Astrologer' The meaning of other words is Miser - a person who loves money and hates spending it. Personification- the proactive of representing objects qualities etc as human in art and literature; an object quality etc. Philanthropist - a rich person who helps the poor and those in need, especially by giving money. 65. Select the word which means the same as the group of words given. Something that is genuine (a) Authentic (b) Unreal (c) Academic (d) Majestic SSC CHSL (Tier-I) 12/10/2020 (Shift-I) Ans. (a) : the given group of words, something that is genuine' is - Authentic. The meaning of other options is Unreal - not related to reality. Academic - a person who teaches and/or does research at a university or college. Majestic - impressive because of size or beauty. 66. In the following question, out of the four given alternatives, select the alternatives which is the best substitute of the phrase. A diplomatic representative in another country. (a) Spy (b) Spinster (c) Bureaucrat (d) Ambassador SSC MTS 9-10-2017 (Shift-I)

585

YCT

Ans : (d) The given group of words, 'A diplomatic representative in another country' is 'ambassador'. The meaning of other options is Spy - a person who fries to get secret information about another country, organization of person, somebody who is employed by a government or the police. Spinster - a women who is not married bureaucrat - an official working in an organization. Bureaucrat - An administrative official who works for the government. 67. In the following question, out of the four given alternatives, select the alternative which is the best substitute of the phrase. A place where arms are kept. (a) Auditory (b) Armoury (c) Apiary (d) Actuary SSC MTS 10-10-2017 (Shift-I) Ans : (b) The given group of words 'A place where arms are kept' is - Armoury. The meaning of other options is Apiary - A place where bees are kept. Actuary - A person whose job involves calculating insurance risks and payments. Auditory - Connected with hearing. 68. In the following question out of the four given alternatives, select the alternative which is the best substitute of the phrase. Words which are opposite in meaning (a) Synonym (b) Antonym (c) Homonym (d) Pseudonym SSC MTS 11-10-2017 (Shift-II) Ans. (b) : The given group of words, 'words which are opposite in meaning is 'Antonym'. The meaning of other options isSynonym - a word or expression that has the same or nearly the same meaning. Hononym - a word that is spelt like another word but which has a different meaning. Pseudonym - A name used by somebody especially a writer instead of their real name. 69. Select the most appropriate word for the given group of words. The study of stars and planets (a) Archeology (b) Astrology (c) Anatomy (d) Astronomy SSC MTS – 22/08/2019 (Shift-III) Ans. (d) : The given group of words, 'The study of stars and planets, is - 'Astronomy'. The meaning of the other options is Archeology - The study of cultures of the past, and of periods of history by examining the remains of buildings and objects found in the ground. Astrology - The study of the positions of the stars and the movements of the planets. Anatomy - the scientific study of the structure of human or animal bodies. 70. Select the most appropriate word for the given group of words. The lack of government control or rule of law (a) Monarchy (b) Autocracy (c) Anarchy (d) Dictatorship SSC MTS – 22/08/2019 (Shift-II) One Word Substitution

Ans. (c) : The given of words, 'the lack of government control or rule of law' is 'Anarchy'. the meaning of the other options is Monarchy - A system of government by a king or a queen. Autocracy - A system of government of a country in which one person has complete power. Dictatorship - government by dictator. 71. Choose the most appropriate word that can substitute the given group of words. Medicine given to counteract a poison. (a) Antibiotic (b) Antiseptic (c) Antigen (d) Antidote SSC MTS – 08/08/2019 (Shift-I) Ans. (d) : The given group of words, 'Medicine given to counteract a poison is - 'Antidote'. The meaning of the other options Antibiotic - Penicillin, that can destroy or prevent the growth of bacteria and cure infections. Antiseptic - able to prevent infection. Antigen - a substance that enters the body and starts a process that can cause disease. 72. Choose the most appropriate word that can substitute the given group of words. Yearly celebration of a date or an event (a) Gala (b) Anniversary (c) Carnival (d) Bash SSC MTS – 07/08/2019 (Shift-I) Ans. (b) : The given group of words, 'Yearly celebration of a date or an event' is 'anniversary'. The meaning of the other options is Gala - A special public festival usually one that happens at a regular time each year. Carnival - a public festival usually one that happens at a regular time each year. Bash - to hit somebody/something very hard /or/A large party or celebration. 73. Choose the word that can substitute the given sentence. A building where an audience sits (a) Aisle (b) Hanger (c) Gymnasium (d) Auditorium SSC MTS – 02/08/2019 (Shift-II) Ans. (d) : The given group of words, 'A building where an audience sits or A large public building where meetings, concerts are held' is - 'Auditorium'. The meaning of other options is Aisle - a passage between rows of seats in a church, theater train etc. Hangar - a large building in which aircraft are kept. Gymnasium - a room or hall with equipment for doing physical exercise. 74. Select the most appropriate word for the given group of words. A person who rules without consulting the opinion of others. (a) Aristocrat (b) Arbitrator (c) Atheist (d) Autocrat SSC MTS – 21/08/2019 (Shift-III) Ans. (d) : The given group of words, 'A person who rules without consulting the opinion of others is 'Autocrat'.

586

YCT

The meaning of the other options is Aristocrat - People born in the highest social class, who have special titles. Arbitrator - a person who is chosen to settle a disagreement. Atheist - A person who believes that god does not exist. 75. Select the most appropriate word for the given group of words. An institution for the care of people who are mentally ill (a) Aviary (b) Apiary (c) Arsenal (d) Asylum SSC MTS – 20/08/2019 (Shift-III) Ans. (d) : The given group of words, 'An institution for the care of people who are mentally ill' is'Asylum'. The meaning of other options is Aviary - A large cage or building for keeping birds in. Apiary - A place where bees are kept. Arsenal - A building where military weapons and explosives are made or stored. 76. Select the most appropriate word for the given group of words : An account of a person's life written by self (a) Autobiography (b) cartography (c) Biography (d) Calligraphy SSC MTS – 19/08/2019 (Shift-III) Ans. (a) : The given group of words, 'An account of a person's life written by self' is - 'Autobiography'. Meaning of the other options is Cartography - The art of process of drawing or making maps. Biography - The story of a person's life written by somebody. Calligraphy - Beautiful handwriting that you do with a special pen or brush. 77. Select the most appropriate word for the given group of words. Policy of racial discrimination (a) Inequality (b) Segregation (c) Apartheid (d) Supremacy SSC MTS – 16/08/2019 (Shift-III) Ans. (c) : The given group of words, 'Policy of racial discrimination' is - 'Apartheid'. The meaning of the other option is Inequality - the unfair difference between groups of people in society, when some have more wealth, status than others. Segregation - the act or policy of separating people of different races, religions or sexes. Supremacy - a position in which you have more power, authority or states than anyone else. 78. Select the most appropriate word for the given group of words. The stage of growth between boyhood and youth (a) Infancy (b) Childhood (c) Adulthood (d) Adolescence SSC MTS – 14/08/2019 (Shift-II) Ans. (d) : The given group of words, 'The stage of growth between boyhood and youth' is - Adolescence. One Word Substitution

The meaning of other options is Infancy - the time when a child is a body or very youth. Childhood - the period of somebody life when they are child. Adulthood- the state of being an adult. 79. Select the most appropriate word for the given group of words. The branch of science concerned with the bodily structure of humans, animals and other living organisms (a) Anatomy (b) Astronomy (c) Botany (d) Gastronomy SSC MTS – 14/08/2019 (Shift-I) Ans. (a) : The given group of words, 'the branch of science concerned with the bodily structure of humans animals, and other living organisms' is - 'Anatomy'. The meaning of the other option is Astronomy - the scientific study of the sun, moon, stars, planets etc. Botany - the scientific study of plants and their structure. Gastronomy - the art and practice of cooking and eating good food. 80. Select the most appropriate word for the given group of words. One who studies the evolution of mankind (a) Sociologist (b) Archaeologist (c) Dermatologist (d) Anthropologist SSC MTS – 13/08/2019 (Shift-I) Ans. (d) : The given group of words 'One who studies the evolution of mankind' is Anthropologist. The meaning of the other options isSociologist - A person who studies sociology. Archeologist - A person who studies archeology. Dermatologist - A doctor who studies and treats skin diseases. 81. Choose the most appropriate word that can substitute the given group of words. The state of growing to maturity. (a) Immigrant (b) Innocence (c) Adolescence (d) Apartheid SSC MTS – 07/08/2019 (Shift-III) Ans. (c) : The given group of words, 'the state of growing to maturity' is - 'Adolescence'. The meaning of the other options is Immigrant - a person who has come to live permanently in a country that is not their own. Innocence - the lack of knowledge and experience of the world, especially of evil or unpleasant things. Apartheid - the former political system in south Africa in which only white people had full political forced to live away from white people. 82. Select the word which means the same as the group of words given. A person who presents a radio/television programme. (a) Reporter (b) Anchor (c) Alchemist (d) Anarchist SSC GD – 18/02/2019 (Shift-II) Ans. (b) : The given group of words, A person who presents a radio/television programme' would be 'Anchor'.

587

YCT

The meaning of other options is Reporter - a person who collects and reports news for newspapers, radio or television. Alchemist - a person who studies alchemy Anarchist - a person who believes that laws and government are not necessary. 83. Select the word which means the same as the group of words given. A place where fights take place (a) Arsenal (b) Arena (c) Asylum (d) Aquarium SSC GD – 14/02/2019 (Shift-I) Ans. (b) : The given group of words, 'a place where fight take place, would be 'Arena'. The meaning of the other options is Arsenal - a collection of weapons such as guns and explosives. Asylum - protection that a government gives to people who have left their own country, casually because they were in danger for political reasons. Aquarium - a large glass container in which fish and other water creatures and plants are kept.

B 84.

Select the options that can be used as a oneword substitute for the given group of words. A group of guns or missile launchers operated together at one place (a) Psaltery (b) Skittery (c) Cemetery (d) Battery SSC MTS-05/07/2022 Shift-I Ans. (d) : For the above group of words, ‘A group of guns or missile launchers operated together at one place’, option (d) Battery is appropriate one word. Psaltery - An ancient musical instrument like a dulcimer. Skittery - Moving very quickly and lightly or in a way that is unexpected or irregular. Cemetery - A place where dead people or buried. 85. Select the word which means the same as the group of words given. a large single detached house with single or double story (a) Suite (b) Apartment (c) Bungalow (d) Penthouse SSC CGL (Tier-II) 08.08.2022 Shift-II Ans. (c) : For the above group of word, ‘ a large single detached house with single or double story’, option (c) Bungalow is appropriate one word. Suit – A set of clothing worn for a particular activity. Apartment – A set of rooms rented for a holiday. Pent house – An expensive a partment or set of rooms at the top of a hotel or tall building. 86. Select the option that can be used as a oneword substitute for the given group of words. A hole or tunnel dug by a small animal to live in (a) Cave (b) Den (c) Shed (d) Burrow SSC Constable GD-03/12/2021 Shift-II One Word Substitution

Ans. (d) : For the above group of words,’ A hole or tunnel dug by a small animal to live in’, option (d) Burrow is appropriate one word. Cave - A natural void in the ground specially a space large enough for a human to entre. Den - The place where certain wild animal live Shade - An area that is out of direct sunlight and is darken and Cooler than areas in the sun. 87. Select the option that can be used as a oneword substitute for the given group of words. An arrangement of flowers that is usually given as a present. (a) Boutique (b) Bracket (c) Basket (d) Bouquet SSC CGL (Tier-I) –20/08/2021 (Shift-I) Ans. (d) : The given group of words, 'An arrangement of flowers that is usually given as a present would be 'Bouquet'. The meaning of other options is Boutique - (of a business) small products or services of a high quality to a small number of customers. Bracket - either of a pair of marks, (+), placed around extra information in a piece of writing or part of a problem in mathematics. Basket - a container for holding or carrying things. 88. Select the option that can be used as a oneword substitute for the given group of words. Life history of a person written by someone else (a) Autobiography (b) Biography (c) Biodata (d) Resume SSC Constable GD-09/12/2021 Shift-II Ans. (b) : For the above group of words, Life history of a person written by someone else’ , option (b) biography is appropriate one word. Autobiography - The story of persons' life written by that person. Biodata - A collection of basic information about a person and their career. Resume - A short written description of your education, qualifications and previous job. 89. Select the option that can be used as a oneword substitute for the given group of words. The sound made by bees (a) Hiss (b) Coo (c) Buzz (d) Moo SSC Constable GD-24/11/2021 Shift-II Ans. (c) : For the above group of words, The sound made by bees’, option (c) Buzz is appropriate one word. Hiss - to make a sound like a very longs to show that you are angry or do not like something. Coo - To make a soft low sound like a bird. Moo - The sound the a cow makes. 90. Select the word which means the same as the group of words given. An underground hole dug by a small animal as a dwelling (a) drain (b) pit (c) cave (d) burrow SSC CPO-SI (Tier-II) 27/09/2019 (3 pm - 5 pm)

588

YCT

Ans. (d) : The above group of words, 'An underground holl dug by a small animal as a dwelling' would be (burrow). The meaning of other options is Drain - a pipe that carries away dirty water. Pit - A large deep hole in the ground. Cave - a large hole in the side of a hill or under the ground. 91. Select the word means the same as the group of words given. a family of young animals (a) nest (b) clutch (c) brood (d) offspring SSC CGL (Tier-I) – 06/06/2019 (Shift-III) Ans. (c) : The given group of words 'A family of young animates would be 'brood'. The meaning of other options is Nest - a place where insects or other small creatures live and produce their young. clutch - power or control. Offspring - a child of a particular person or couple. 92. Select the word which means the same as the group or words given. An instrument for measuring the atmospheric pressure (a) thermometer (b) altimeter (c) barometer (d) ammeter SSC CGL (Tier-I) – 13/06/2019 (Shift-III) Ans : (c) An instrument for measuring the atmospheric pressure - is 'barometer'. The meaning of other options is Thermometer - an instrument used for measuring the temperature of the air, a person body etc. Altimeter - an instrument for showing height above sea level used especially in an aircraft. Ammeter - an instrument for measuring the strength of an electric current. 93. Select the word which means the same as the group of words given. One who is a great lover of books (a) bibliophile (b) pedophile (c) xenophile (d) hemophile SSC CGL (Tier-I) – 11/06/2019 (Shift-I) Ans. (a) : The given group of words 'One who is a great lover of books' is - bibliophile. The meaning of other options is Pedophile - a person who is sexually attracted to children. Xenophile - One attracted to foreign things (such as style or people) Hemophile - someone who has hemophilia and is subjected to uncontrollable bleeding. 94. Select the word which means the same as the group of words given. An arrangement of flowers that is usually given as a present (a) bouquet (b) wreath (c) garland (d) bundle SSC CGL (Tier-I) – 07/06/2019 (Shift-I) Ans : (a) The given group of words, 'An arrangement of flowers that is usually given as a present' is 'bouquet'. The meaning of other options is One Word Substitution

Wreath - an arrangement of flowers and leaves especially in the shape of a circle. Garland - a circle of flowers and leaves that is worn an the head or around the neck. Bundle - A number of things tied or wrapped together. 95. Select the word which means the same as the group of words given. A cylindrical container bulging out in the middle, traditionally made of wooden staves for keeping oil, beer etc. (a) bushel (b) bale (c) barrel (d) bin SSC CGL (Tier-I) – 04/06/2019 (Shift-III) Ans. (c) : For the above group of words, option (c) barrel is appropriate one word. Bale - a large amount of a light material pressed tightly together and tied up. Bushel - a unit for measuring grain and fruit or a large amount of something. Bin - a container that you waste in. 96. Select the most appropriate word which means the same as the group of words given. A list of the books referred to in a scholarly work (a) bibliography (b) calligraphy (c) holography (d) bibliophile SSC CPO-SI – 11/12/2019 (Shift-I) Ans : (a) The given group of words' A list of the books referred to in a scholarly work' is - bibliography'. Calligraphy - beautiful handwriting that you do with a special pen or brush. Holography - the process or technique of making holograms. Bibliophile - a person who loves or collects book. 97. Select the most appropriate option that can be used as a one-word substitute for the given group of words. A large building or group of buildings used to house soldiers (a) fort (b) dormitory (c) quarters (d) barracks SSC CPO-SI – 12/12/2019 (Shift-II) Ans : (d) The given group of works, 'A large building or group of buildings used to house soldiers is 'barracks'. The meaning of other options is Fort- a building built in order to defend an area against attak. Quarter - one of four equal parts of something. 98. Select the word which means the same as the group of words given. A person who can speak only two languages. (a) Monolingual (b) Multilingual (c) Linguist (d) Bilingual SSC CHSL (Tier-I) –04/07/2019 (Shift-I) Ans : (d) The given group of words, 'A person who can speak only two languages' is - 'Bilingual'. The meaning of other options is linguist - a person who knows several foreign language well. Multilingual - speaking or using several different languages. Monolingual - speaking or using only one language.

589

YCT

99.

Select the word which means the same as the group of words given A rabbit's dwelling. (a) lair (b) burrow (c) sty (d) den SSC CHSL (Tier-I) –03/07/2019 (Shift-I) Ans : (b) The given group of words, 'a rabbit's dwelling' is - 'burrow'. The meaning of other options is Lair - den (a place where a wild animal sleeps or hides) Sty - any filthy place or abode. Den - the hidden home of some types of wild animal. 100. Select the word which means the same as the group of word given. An instrument used for measuring atmospheric pressure (a) thermometer (b) voltmeter (c) altimeter (d) barometer SSC CHSL (Tier-I) –03/07/2019 (Shift-III) Ans. (d) : The given group of words 'An instrument used for measuring atmospheric pressure' is Barometer The meaning of other options is Thermometer - an instrument used for measuring the temperature of the air, a person's body, etc. Voltmeter - an instrument for measuring voltage. (a unit for measuring the farce of an electric current) Altimeter - an instrument for showing height above sea level, used especially in an aircraft. 101. Select the word that means the same as the given group of words. Happening every two years (a) Biannual (b) Centennial (c) Biennial (d) Triennial SSC CHSL (Tier-I) 21/10/2020 (Shift-II) Ans. (c) : The given group of words, 'Happening every two years' is - Biennial. The meaning of other options is Biannual - happening twice a year. Centennial - the 100th anniversary of an event. Triennial - happening every three years. 102. In the following question, out of the given alternatives, the alternative is the best substitute of the phrase. A man who remains unmarried (a) Vidoved (b) Councilor (c) Bachelor (d) Propeller SSC MTS 9-10-2017 (Shift-III) Ans : (c) The given group of words, 'A man who remains unmarried' is - 'Bachelor'. The meaning of other options is Widowed - a women whose husband has died and who gas not married again. Councilor - a member of a council. Propeller - a device that causes a ship or aircraft to move. 103. Select the most appropriate word for the given group of words. One who loves books (a) Orator (b) Altruist (c) Librarian (d) Bibliophile SSC MTS – 14/08/2019 (Shift-III) One Word Substitution

Ans. (d) : The given group of words 'One who loves books' is - 'Bibliophile'. The meaning of other options is Orator - a person who makes formal speeches in public or is good at public speaking. Altruist - a person unselfishly concerned for or devoted to the welfare of others. Librarian - a person who is in charge of or works in library. 104. Select the most appropriate word for the given group of words. A large bundle bound for storage or transport (a) Bunch (b) Bale (c) Bevy (d) Barrel SSC MTS – 13/08/2019 (Shift-I) Ans. (b) : The given group of words, 'A large bundle bound for storage or transport' is - 'Bale'. The meaning of other options is Bunch - a large amount of something; a large number of things or people. Bevy - a large group of people or things of the same kind. Barrel - a large round container, usually made of wood or metal.

C 105. Select the most appropriate word for the given group of words. A local meeting of party members to select candidates, elect convention delegates, etc (a) Claque (b) Caravan (c) Clique (d) Caucus SSC MTS 10/05/2023 (Shift-I) Ans. (d) : For the above given, group of words 'A local meeting of party members to select candidates elect convertion delegates' is known as 'caucus'. other words are____ Claque means- a group of sycophantic followers. Caravan means- A group of people and animals that travel together. Clique means- An exclusive circle of people with a common purpose. 106. Choose the word that can substitute the given group of words. Someone who knows a lot about the arts, food, drink, or some other subject (a) Connoisseur (b) Gourmand (c) Cynic (d) Pacifist SSC MTS 15/05/2023 (Shift-I) Ans. (a) : For the above given group of words, Someone who knows a lot about the arts, food, drink, or some other subject the word is 'Connoisseur' is suitable. Meaning of other options– Gourmand :- A person who enjoy eating and eat large amount of food. Cynic :- A person who believe that people only do things for themselves. Pacifist :- Someone who oppose war or violence. Thus, the correct answer is option (a).

590

YCT

107. Select the option that can be used as a oneword substitute for the given group of words. A person who draws or produces maps (a) Chauffeur (b) Choreographer (c) Calligrapher (d) Cartographer SSC CGL (Tier-I) 18/07/2023 (Shift-III) Ans. (d) : One-word substitute for the above given group of words 'A person who draws or produces maps' is option (d) 'cartographer'. Other options give different meanings - Chauffeur Someone whose job is to drive a car for a rich person. Choreographer - Someone who creates new dances. Calligrapher - The art of writing beautifully with a special pen. 108. Select the option that can be used as a oneword substitute for the given phrase. Gradual recovery of health and strength (a) Potency (b) Rejuvenation (c) Convalescence (d) Benefaction SSC CGL (Tier-I) 20/07/2023 (Shift-I) Ans. (c) : The most appropriate one-word substitute for the given phrase ‘Gradual recovery of health and strength’ is ‘Convalescence’. The meaning of the other words is– Potency – The power that somebody/something has to affect your body or mind. Rejuvenation – The act or process of making somebody look or feel younger. Benefaction – The act of doing good or helping others. 109. Select the option that can be used as a oneword substitute for the given group of words. Collection of written or spoken texts (a) Data bank (b) Dictionary (c) Circa (d) Corpus SSC CHSL (Tier-I) 17/08/2023 (Shift-II) Ans. (d) : For the given group of words, 'collection of written or spoken texts', option (d) corpus, is appropriate one word substitution. other options means the following – circa – (used with dates) about or approximately. Data bank - A repository of information about one or more subjects. Dictionary – A book or digital resource giving information on a particular subject or on a particular class of words. 110. Select the option that can be used as a oneword substitute for the underlined group of words. Kidney beans were boiled quickly when Nisha added a substance that speeds up a chemical reaction without being consumed by the reaction itself. (a) catalyst (b) reactant (c) enzyme (d) solute SSC CHSL (Tier-I) 17/08/2023 (Shift-II) Ans. (a) : One word substitution for the underlined group of words, 'Substance that speeds up a chemical reaction without being consumed by the reaction itself' will be option (a) catalyst. One Word Substitution

other options have following meanings Reactant – A substance that takes part in and undergoes change during a reaction Enzyme – proteins that act as biological catalysts Solute – The minor component in a solution, dissolved in the solvent. 111. Select the option that can be used as a oneword substitute for the given group of words. A person who conforms to accepted behaviour or established practices. (a) Psychologist (b) Rebelist (c) Socialist (d) Conformist SSC CHSL (Tier-I) 03/08/2023 (Shift-II) Ans. (d) : The most appropriate one-word substitute of the given group of words 'A person who conforms to accepted behavior or established practices' is 'Conformist'. Psychologist - A scientist who studies the mind and the way that people behave. Rebelist - A person who refuses to accept the governments power. Socialist - Someone who supports a political or economic philosophy. 112. Choose the word that can substitute the given group of words. A fall of water from a great height (a) River (b) Stream (c) typhoon (d) Cascade SSC CHSL (Tier-I) 21/03/2023 (Shift-II) Ans. (d) : For the above group of words "A fall of water from a great height", option (d) 'Cascade' is appropriate one word. Other options are not correct. River - A large, natural flow of water that goes across land and into the sea. 113. Choose the word that can substitute the given group of words. Compulsory enlistment for military service (a) Militaristic (b) Conscription (c) Armament (d) Abdication SSC CHSL (Tier-I) 09/02/2023 (Shift-II) Ans. (b) : For the above group of words "Compulsory enlistment for military service" option (b) Conscription is an appropriate word. Other options give different meaning. Armament – The process of increasing the amount of weapons an army or a country Abdication – To leave the position of being a king or queen. militaristic – A person skilled in the conduct of war. 114. Select the most appropriate one-word substitution for the given group of words. One who is a center of attraction (a) Exposure (b) Cynosure (c) Commissure (d) Censure SSC CHSL (Tier-I) 17/03/2023 (Shift-III) Ans. (b) : The above group of words, "one who is a centre of attraction" is option (b) "Cynosure" gives appropriate meaning. Other options gives different meaning.

591

YCT

Exposure - The act of making something public Commissure - The joint between two bones Censure - To tell somebody, in a strong and formal way that he/she has done something wrong. 115. Select the option that can be used as a oneword substitute for the given group of words. A style of cooking food, characteristic of a particular country of region (a) Antique (b) Cuisine (c) Menu (d) Creche SSC CGL (Tier-I) 19/04/2022 Shift-II Ans. (b) : For the above group of words, option (b) Cuisine is appropriate one word. Other options have different meaning. Antique – A work of art, piece of furniture made at an earlier period Menu – A list of the food that you can choose at a restaurant Creche - A place where small children are looked after while their parents are working. 116. Select the option that can be used as a oneword substitute for the given group of words. Made in imitation so as to be passed off fraudulently as genuine (a) Counterfeit (b) Staunch (c) Authentic (d) Deliberate SSC MTS-13/07/2022 Shift-I Ans. (a) : For the above group of words, ‘Made in imitation so as to be passed off fraudulently as genuine’, option (a) counterfeit is appropriate one word. Staunch - Very devoted or loyal to a person. Authentic - That you know is real or genuine. Deliberate - To think about or discuss something fully before making a decision. 117. Select the option that can be used as one-word substitute for the given group of words. A Person who makes maps (a) Sinecure (b) Anarchist (c) Cartographer (d) Polyglot SSC MTS-08/07/2022 Shift-I Ans. (c) : For the above group of words, ‘ A person who makes maps,’ option (C) cartographer is appropriate one word. Sinecure - An office or position requiring little or no work Anarchist - A person who rebels against any authority or ruling power. Polyglot - Someone who can speak or use several different languages. 118. Select the option that can be used as a oneword substitute for the given group of words. One who feeds on human flesh (a) Cannibal (b) Hedonist (c) Stoic (d) Samaritan SSC MTS-12/07/2022 Shift-I Ans. (a) : For the above group of words ‘ one who feeds on human flesh’, option (a) cannibal is appropriate one word. Other options have different meaning. One Word Substitution

Hedonist - A person who believes that pleasure is the most important thing in life. Stoic - Suffring pain or difficulty without complaining Samaritan - A charitable or helpful person. 119. Select the word which means the same as the group of words given. a doctor who specializes in heart diseases (a) Urologist (b) Cardiologist (c) Nephrologist (d) Dermatologist SSC CGL (Tier-II) 08.08.2022 Shift-II Ans. (b) : For the above group of words’ a doctor who specializes in heart diseases’, option (b) Cardiologist is appropriate one word. Other options have different meaning. Urologist – Specialist of Urine diseases. Nephrologists – Specialist of kidney diseases. Dermatologist – Specialist of skin diseases. 120. Select the word which means the same as the group of words given. A person who draws or produces maps (a) Chauffer (b) Cartographer (c) Calligrapher (d) Choreographer SSC CHSL (Tier-I) –13/04/2021 (Shift-I) Ans. (b) : The given group of words, 'A person who draws or produces maps' is - 'Cartographer'. The meaning of other options is Chauffeur - a person whose job is to drive a can especially for somebody rich or important. Choreographer - a person whose job involves designing and arranging the steps and movements in dances, especially in ballet. Calligrapher - a person who practices calligraphy the art of writing beautifully with a special pen or bush. 121. Select the option that can be used as a oneword substitute for the given group of words. Relating to the present time (a) Preceding (b) Ancient (c) Corollary (d) Contemporary SSC CHSL (Tier-I) –19/04/2021 (Shift-I) Ans. (d) : The above group of words, 'Relating to the present time' is - 'Contemporary'. The meaning of other options is Preceding - existing or happening before someone or something. Ancient - belonging to a period of history that is thousands of years in the past. Corollary - a situating, an argument or a fact that is the natural and direct result of another one. 122. Select the option that can be used as a oneword substitute for the given group of words. A corporation made up of a number of different companies that operate in diversified fields. (a) Agreement (b) Association (c) Correlation (d) Conglomerate SSC CGL (Tier-I) –24/08/2021 (Shift-I) Ans. (d) : The above group of words, 'A corporation made up of a number of different companies that operate in diversified fields is - 'conglomerate'. The meaning of other options is -

592

YCT

Agreement - an arrangement, a promise or a contract made with somebody. Association - an official group of people who have joined together for a particular purpose. Correlation - a connection between two thinks. 123. Select the most appropriate one-word substitution for the given words. A group of starts that make a pattern (a) Planet (b) Constellation (c) Horizon (d) Comet SSC CGL (Tier-II) 16/11/2020 (3 pm - 5 pm) Ans. (b) : The above group of words, 'A group of starts that make a pattern' is 'constellation'. Planet - a large round object in space that moves around a star and receives light from it. comet - a mass of ice and dust that moves around the sun and dooms like a bright start with a tail. Horizon - the line at the farthest place that you can see, where sky seems to touch the land or see. 124. Select the most appropriate one-word substitution for the given words. A place where gambling games are played (a) Casino (b) Gymnasium (c) Arena (d) Stadium SSC CGL (Tier-II) 16/11/2020 (3 pm - 5 pm) Ans. (a) : The above group of words, 'a place where gambling games are played' is - 'casino'. The meaning of other options is Arena - a place with a flat open area in the middle and seats around it where people can watch and entertainment. Gymnasium - A large room with equipment for exercing. Stadium - a large sports ground surrounded by rows of seats and usually other buildings. 125. Select the word which means the same as the group of words given. Extreme fear of confined places (a) cellophobia (b) claustrophobia (c) chronophobia (d) centrophobia SSC CPO-SI (Tier-II) 27/09/2019 (3 pm - 5 pm) Ans. (b) : The above group of words, 'Extreme fear of confined places' is 'claustrophobia'. The meaning of other option is Chronophobia - fear of time. Centrophobia - a dislike of being in the centre. Cellophobia - To feel stressed and anxious and when your mobile phone runs out of battery. 126. Select the word which means the same as the group of words given. An arrangement of events or dates in the order of their occurrence (a) chronometry (b) charter (c) chronology (d) calendar SSC CPO-SI (Tier-II) 27/09/2019 (3 pm - 5 pm) Ans. (c) : The above group of words, 'An arrangement of events of dates in the order of their occurrence' is 'chronology'. The meaning of other options is Chronometry - The science of accurate time measurement. One Word Substitution

Charter - A written statement describing the rights that a particular group of people. Calendar - a page or series of pages showing the days, weeks and months of a particular year. 127. Select the word which means the same as the group of words given. One who plans the steps and moves in a dance (a) composer (b) choreographer (c) producer (d) director SSC CPO-SI (Tier-II) 27/09/2019 (3 pm - 5 pm) Ans. (b) : The above group of words, 'One who plans the steps and moves in a dance' is - choreographer'. The meaning of other options is Composer - a person who writes music, especially classical music. Producer - a person, a company or a country that grows or makes food, goods or materials. Director - One of a group of senior managers who run a company. 128. In the following question, out of the four alternative, select the alternative which is the best substitute of the phrase. Love for dogs (a) Pedophilia (b) Sinophobe (c) Canophilia (d) Zoophilia SSC CGL (Phase-II) 17/02/2018 Ans. (c) The above group of words, 'Love for dogs' is 'canophilia'. Other options given different meaning. Pedophilia - the condition of being sexually attracted to children; sexual activity with children. Zoophilia - a sexual attraction to animals. 129. In the following question, out of the four alternative, select the alternative which is the best substitute of the phrase. Mutual discourse (a) Soliloquy (b) Obloquy (c) Colloquy (d) Eloquence SSC CGL (Phase-II) 17/02/2018 Ans. (c) The above group of words, 'Mutual discourse' is - 'colloquy'. The meaning of other options is Soliloquy - a speech in a play in which a character, who is alone on the stage, speaks his or her thoughts. Obloquy - strong public criticism Eloquence - the ability to use language and express your opinion well. 130. Select the most appropriate word for the given group of words. A disease which spreads with contact (a) Poisonous (b) Contagious (c) Endemic (d) Contractual

593

SSC Sel. Post Phase-VIII (M.L.) 09.11.2020 (Shift-III)

Ans. (b) : The above group of words, 'A disease which spreads with contact' is - 'contagious'. The meaning of other words is Poisonous - causing death or illness if swallowed or absorbed into the body. Endemic - regularly found in a particular place or among a particular group of people and difficult to get rid of. Contractual - connected with the conditions of a legal written agreement. YCT

131. Select the most appropriate word for the given group of word. A factual written account of important or historical events in the order of their occurrence. (a) chronology (b) journal (c) chronicle (d) register SSC CPO-SI – 25/11/2020 (Shift-I) Ans. (c) : The given group of words ' A factual written account of important or historical events in the order of their occurrence' is - 'chronicle'. The meaning of other options is Chronology - the order in which a series of events happened. Journal - a newspaper or magazine that deals with a particular subject or profession. Register - an official or recorded of names, items etc. 132. Select the most appropriate one-word substitution for the given group of words. Arrangement of events according to the dates of occurrence (a) analogy (b) chronology (c) theology (d) trilogy SSC CPO-SI – 24/11/2020 (Shift-I) Ans. (b) : The given group of words, 'a factual written account or historical events in the order of their occurrence - 'chronology'. The meaning of other options isAnalogy - a comparison of one thing with another thing that has similar features. Theology - the study of religion and beliefs. Trilogy - a group of three books, films/moves etc. 133. Select the most appropriate one-word substitution for the given group of words. Harsh or discordant sound (a) monotony (b) harmony (c) symphony (d) cacophony SSC CPO-SI – 24/11/2020 (Shift-I) Ans. (d) : The given group of words, 'Harsh or discordant sound' is - 'cacophony'. The meaning of other options isMonotony - boring lack of variety. Harmony - a state of peaceful existence and agreement. Symphony - a long complicated piece of music for a large orchestra, in three or four main parts. 134. Select the most appropriate word which means the same as the group of words given. A place where games are played for money (a) casino (b) church (c) university (d) gymnasium SSC CPO-SI – 23/11/2020 (Shift-I) Ans. (a) : The given group of words, 'A place where games are played for money' is - 'casino'. The meaning of other options is Church - a building where Christians go to worship. University - an institution at the highest level of education where you can study for a degree or do research. Gymnasium - A large room with equipment for exercising. One Word Substitution

135. Select the most appropriate one word substitution for the given group of words. A person who believes everything easily. (a) plaussible (b) creditor (c) reliable (d) credulous SSC CPO-SI – 24/11/2020 (Shift-II) Ans. (d) : The given group of words, 'a person who believes everything easily' is - 'credulous'. The meaning of other options is Plausible - reasonable and likely to be true. Creditor - a person, company, etc that somebody owes money to. Reliable - That can be trusted. 136. Select the word which means the same as the group of words given. A person who draws or produces maps (a) calligrapher (b) cartographer (c) lexicographer (d) choreographer SSC CGL (Tier-I) – 04/06/2019 (Shift-III) Ans. (b) : The given group of words, 'a person who draws or produce maps' is - cartographer. The meaning of other options is Lexicographer - a person who writes and edits dictionaries. Calligrapher - a person who practices calligraphy, the art of writing beautifully with a special pen or brush. Choreographer - a person whose job involves designing and arranging the steps and movement in dances. 137. Select the word which means the same as the group of words given. A large, deep, metal pot used for cooking over open fire (a) barrel (b) kettle (c) skillet (d) cauldron SSC CGL (Tier-I) – 07/06/2019 (Shift-I) Ans : (d) The given group of words, ' a large, deep, metal pot used for cooking over open fire, is 'cauldron'. The meaning of other options is Barrel - a large round container, usually made of wood or metal with flat ends and usually curved sides. Kettle - a container with a lid, handle and a spout, used for boiling water. Skillet - frying pan. 138. Select the word which means the same as the group of words given. a large, deep pot used both in the oven and as a serving vessel. (a) sauce-pan (b) casserole (c) skillet (d) cauldron SSC CGL (Tier-I) – 07/06/2019 (Shift-II) Ans : (b) The given group of words, 'A large deep pot used both in the over and as a serving vessel' is 'casserole'. The meaning of the other options is Sauce pan - a deep round metal pot with a lid and one long handle. Caluldron - a large deep pot for boiling liquids or cooking food over a fire. 139. Select the word which means the same as the group of words given. Persons living at the same time

594

YCT

(a) cosmopolitans (b) compatriots (c) colleagues (d) contemporaries SSC CGL (Tier-I) – 06/06/2019 (Shift-II) Ans : (d) The given group of words, 'Persons living at the same time', is - 'contemporaries'. The meaning of other options is Compariot - a person who has experiences of many different parts of the world. Compatriot - countryman. Colleague - a person that you work with, especially in a profession or a business. 140. Select the word which means the same as the group of words given. The sound of a crow (a) cackle (b) creak (c) caw (d) cluck SSC CPO-SI – 13/12/2019 (Shift-II) Ans. (c) : The given group of words, 'the sound of a crow' is - 'caw'. The meaning of other options is Cackle - the loud noise that a chicken makes Creak - a sound, for example that sometimes made by a door when it opens or shut. Cluck - the low, short sounds that a chicken makes. 141. Select the word which means the same as the group of words given. People living at the same time (a) comrades (b) cosmopolitans (c) friends (d) contemporaries SSC CHSL (Tier-I) –11/07/2019 (Shift-III) Ans : (d) The given group of words, 'People living at the same time' is - 'contemporaries'. The meaning of other options is Comrade - a person who is a member of the same communist or socialist political party Cosmopolitans - a person who has experience of many different part of the world. 142. Select the most appropriate word for the given group of words. One who makes and sells sweets and pastries (a) grocer (b) confectioner (c) cobbler (d) barber SSC CHSL (Tier-I) –09/07/2019 (Shift-II) Ans : (b) The given group of words, 'One who makes and sells sweet and pastries' is - 'confectioner'. The meaning of other options is Grocer - a person who owns, manages or works in a shop/store salting food and other things used in the home. Cobbler - a person who repairs shoes. Barber - a person whose job is to cut men's hair and sometimes to shave them. 143. Select the most appropriate word for the given group of words A person who is skilled at writing beautifully (a) calligrapher (b) artist (c) painter (d) cartoonist SSC CHSL (Tier-I) –09/07/2019 (Shift-III) Ans : (a) The given group of words, 'A person who is skilled at writing beautifully' is - 'calligrapher'. The meaning of other options is One Word Substitution

Artist - a person who creates woks of art, especially paintings or drawings. Painter - a person whose job is painting buildings, walls, etc. Cartoonist - A person who draws cartoons 144. Select the most appropriate word for the group of words. The state of remaining unmarried (a) monogamy (b) polygamy (c) feminism (d) celibacy SSC CHSL (Tier-I) –05/07/2019 (Shift-II) Ans : (d) The given group of words, 'the state of remaining unmarried' is 'celibacy'. The meaning of other options Monogamy - the fact or custom of being married to only one person at a particular time. Polygamy - the custom of having more than one wife at the same time. Feminism - the belief and aim that woman should have the dame rights and opportunities as men. 145. Select the most appropriate word for the group of words. One who is in charge of a museum or art gallery (a) monitor (b) curator (c) instructor (d) collector SSC CHSL (Tier-I) –04/07/2019 (Shift-III) Ans : (b) The given group of words, 'One who is in charge of a museum or art gallery' is - 'curator'. The meaning of other options is Monitor - a person whose job is to check that something is done fairly and honestly. Instructor - a person whose job is to teach somebody a practical skill or sport. Collector - a person who collects things, either as a hobby or as a job. 146. Select the word which means the same as the group of words given. A group of singers in a church (a) choir (b) band (c) host (d) troop SSC CHSL (Tier-I) –02/07/2019 (Shift-I) Ans. (a) : The given group of words, 'A group of singers in a church' is - 'choir'. The meaning of other options is band - a small group of musician who play popularly music together. Host - a person who invites guests to a meal, a party etc. Troops - soldiers, especially in large groups. 147. Select the word which means the same as the given group of words. Arrangement of events or dates in the order in which they happen (a) Geology (b) Chronology (c) Archaeology (d) Ecology SSC CHSL (Tier-I) 16/10/2020 (Shift-II) Ans. (b) : The given group of words, ' Arrangement of events or dates in the order in which they happen' is 'Chronology'. Geology - the scientific study of the earth. Archaeology - the study of cultures of the past. Ecology - the relation of plants and living creatures to each other and to their environment.

595

YCT

148. Select the word which means the same as the group of words given. Persons or thing that is the centre of attention (a) Organiser (b) Cynosure (c) Politician (d) Leader SSC CHSL (Tier-I) 18/03/2020 (Shift-II) Ans. (b) : The given group of words, 'Person or thing that is the centre of attention' is - 'cynosure'. The meaning of other options is Organiser - a person who makes the arrangement for something. Politician - a person whose job is concerned with politics. Leader - a person who leads a group of people. 149. Select the word which means the same as the group of words given. Something which is considered to be very important (a) Scanty (b) Cardinal (c) Supplementary (d) Meagre SSC CHSL (Tier-I) 18/03/2020 (Shift-I) Ans. (b) : The given group of words, 'something which is considered to be very important' is - 'Cardinal'. The meaning of other options is Scanty - too little amount for what is needed. Supplementary - Add to something else in order to improve it. Meagre - paltry (small in quantity and poor in quality) 150. Select the word which mean the same as the group of words given: To make perfect or complete (a) Deportment (b) Attainment (c) Impediment (d) Complement SSC CHSL (Tier-I) 21/10/2020 (Shift-I) Ans. (d) : The given group of words, 'to make perfect or complete' is - 'complement'. The meaning of other options is Deportment - the way in which a person stands and moves. Attaniment - success in achieving something. Impediment - obstacle (stop the progress of something). 151. Select the which means the same as the group of words given: Satisfied, with no desire to change or improve (a) Adolescent (b) Complacent (c) Beneficent (d) Innocent SSC CHSL (Tier-I) 21/10/2020 (Shift-I) Ans. (b) : The given group of words, 'satisfied with no desire to change or improve' is - 'complacent'. The meaning of other options is Adolescent - a young person who is developing from a child into an adult. beneficent - generous (giving help). Innocent - not guilty of a crime. 152. Select the word which means the same as the group of words given. A group of people who have come together in a religious building for worship and prayer (a) Throng (b) Crowd (c) Multitude (d) Congregation SSC CHSL (Tier-I) 15/10/2020 (Shift-I) One Word Substitution

Ans. (d) : The given group of words, 'A group of people who have come together in a religious building for worship and prayer' is - 'congregation' The meaning of other options is Throng - a crowd of people. Multitude - an extremely large number of things or people. Crowd - A large number of people in one place. 153. Select the word which means the same as the group of words given. A small compartment in an aircraft for the pilot (a) Canopy (b) Chassis (c) Cockpit (d) Fuselage SSC CHSL (Tier-I) 19/10/2020 (Shift-III) Ans. (c) : The given group of words, 'A small compartment in an aircraft for the pilot' is - 'Cockpit'. The meaning of other options is Canopy - a cover that is fixed or hangs above a bed, seat etc. Chassis - the frame that a vehicle is built an. Fuselage - the main part of a plane 154. Select the word which means the same as the group of words given. A deep fissure in the earth's surface (a) Chasm (b) Earthquake (c) Tremor (d) Valcano SSC CHSL (Tier-I) 20/10/2020 (Shift-II) Ans. (a) : The given group of words, 'A deep fissure in the earth's surface' is - 'chasm'. The meaning of other options is Earthquake - a sudden, violent shaking of the earth's surface. Tremor - a small earthquake in which the ground shakes slightly. (quiver) Volcano - a mountain with a large opening at the top through which gases and lava rare forced out into the air. 155. Select the word which means the same as the group of words given. A person living in the same age with another (a) Confidant (b) Collaborator (c) Partner (d) Contemporary SSC CHSL (Tier-I) 19/03/2020 (Shift-II) Ans. (d) : The given group of words, 'A person living in the same age with another' is - contemporary. The meaning of other options is Confident - Feeling that you are sure about your own abilities. Collaborator - cooperator, get together. partner - better half, mate. 156. Choose the most appropriate word that can substitute the given group of words. The art of creating and arranging dances (a) Geology (b) Choreography (c) Seismology (d) Calligraphy SSC MTS – 05/08/2019 (Shift-III) Ans. (b) : The given group of words, 'The art of creating and arranging dances' is choreography. The meaning of other options is Geology - the origin and history of the rocks and soil of a particular area. Seismology - the scientific study of earthquakes. Calligraphy - beautiful handwriting that you do with a special pen or brush.

596

YCT

157. Choose the most appropriate word that can substitute the given group of words. Group of singers (a) Jury (b) Choir (c) Orchestra (d) Council SSC MTS – 05/08/2019 (Shift-II) Ans. (b) : The given group of words, 'Group of singers' is called - choir. The meaning of other options is Jury - a committee appointed to judge (panel) Orchestra - a large group of people who play various musical instruments tougher. Council - administrative body. 158. Choose the most appropriate word that can substitute the given group of words. A place where nuns live and work (a) Convent (b) Dormitory (c) Hostel (d) Quarter SSC MTS – 02/08/2019 (Shift-I) Ans. (a) The given group of words, 'A place where nuns live and work' is called - 'convent'. The meaning of other options is Dormitory - bedchamber, bedroom, sleeping room. Hostel - inn, lodge, student lodging. Quarter - fourth, one fourth. 159. Select the most appropriate word for the given group of words. A critical judge of any art and craft (a) Connoisseur (b) Curator (c) Crusader (d) Cynic SSC MTS – 21/08/2019 (Shift-I) Ans : (a) The given group of words, 'A critical judge of any art craft' is called 'Connoisseur'. The meaning of the other options is Curator - a person whose job is to be in charge of the objects or works of art in a museum. Crusader - a person who takes parta in a crusade (campaign) Cynic - someone who is critical of the motives of other. (faultfinder) 160. Select the most appropriate word for the given group of words. A person who is recovering after an illness or medical treatment. (a) Athlete (b) Convalescent (c) Senile (d) Altruist SSC MTS – 19/08/2019 (Shift-II) Ans. (b) : The given group of words, 'A person who is recovering after an illness or medical treatment is called - 'convalescent'. The meaning of other options isAthlete - a person trained to compete in sports. Senile - mentally or physically infirm with age. Altruist - philanthropist, donor. 161. Select the most appropriate word for the given group of words. The fear of being enclosed in a small space or room. (a) Hydrophobia (b) Acrophobia (c) Claustrophobia (d) Aerophobia SSC MTS – 19/08/2019 (Shift-II) One Word Substitution

Ans. (c) : The given group of words, ' the fear of being enclosed in a small space or room, is called 'claustrophobia'. The meaning of other options is Hydrophobia - a morbid fear of water. Acrophobia - a morbid fear of great heights. Aerophobia - a morbid fear of air. 162. Select the most appropriate word for the given group of words. A list of names or things in a special order (a) Atlas (b) Catalogue (c) Dictionary (d) Directory SSC MTS – 14/08/2019 (Shift-I) Ans. (b) : The given group of words, 'A list of names or things in special order' is called - 'catalogue'. The meaning of other things is Atlas - a collection of maps in books form (book of maps). Dictionary - a reference book containing an alphabetical list of words with information about them. Directory - an alphabetical list of names and addresses. 163. Select the word which means the same as the group of word given. Persons living at the same time (a) Colleagues (b) Contemporaries (c) Comrades (d) Companions SSC MTS – 08/08/2019 (Shift-II) Ans. (b) : The given group of words, 'persons living at the same time' is called 'contemporaries. The meaning of other options is Colleagues - co- worker, fellow worker. Comrade - companion, fellow Companions - a person or an animal that travel with you or spends a lot of time with you. 164. Select the word which means the same as the group of words given. The dead body of an animal (a) Skull (b) Skeleton (c) Vesper (d) Carcass SSC GD – 08/03/2019 (Shift-III) Ans. (d) : The given group of words, 'The dead body of an animal' is called 'carcass'. The meaning of other animals Skull - the head or the brain. (cranium) Skeleton - frame (a very thin person or animal) Vesper - the service of evening prayer in some Christian churches.

D 165. Select the option that can be used as a oneword substitute for the given group of words. A person who enjoys doing dangerous things, in a way that other people may think is stupid. (a) Cavalier (b) Lunatic (c) Gallant (d) Daredevil SSC CGL (Tier-I) 19/07/2023 (Shift-IV) Ans. (d) : The most appropriate one - word substitute for the given group of words 'A person who enjoys doing dangerous things, in a way that other people may think is stupid' is 'Daredevil'. Other options give different meaning.

597

YCT

Ans. (b) : For the above group of words, ‘The study of different skin diseases’, option (b) Dermatology is appropriate one word. Neurology - The medical field that deals with nervous system disorder. Psychology - The study of mind and behavior Radiology - The branch of medicine that deals with radiant energy 170. Select the option that can be used as a oneword substitute for the given group of words. Something that is strong and lasts a long time without breaking or becoming weaker. (a) Durable (b) Harsh (c) Secure (d) Pliable SSC CGL (Tier-I) 13/04/2022 Shift-II Ans. (a) : For the above group of words, option (a) Durable is appropriate one word. Other options have different meaning. Harsh - Very strict and unkind Secure - Free from worry or doubt Pliable – easy to bend or shape 171. Select the option that can be used as a oneword substitute for the given group of words, Causing a mood of gloom and depression (a) Apprehensive (b) Devastated (c) Dismal (d) Desolate SSC Constable GD-03/12/2021 Shift-II Ans. (c) : For the above group of words, ‘Causing a mood of gloom and depression option (c) 'Dismal' is appropriate word. Other options have different meaning. Apprehensive - feeling worried about something that you are going to do or that is going to happen Devastated - Extremely shocked and upset Desolate - Extremely sad and feeling alone. 172. Select the option that can be used as a oneword substitute for the given group of words. To throw an event into confusion or disorder (a) Detonate (b) Disrupt (c) Explode (d) Erupt SSC CGL (Tier-I) –18/08/2021 (Shift-I) Ans. (b) : The above group of words, 'To throw an event confusion or disorder' is called 'Disrupt'. The meaning of other options is Detonate - To explode or to make a bomb Explode - to brust with a loud noise. Erupt - to start something suddenly. 173. Select the option that can be used as a oneword substitute for the given group of words. A herd or flock of animals being driven in a body (a) Throng (b) Cluster (c) Crowd (d) Drove SSC CGL (Tier-I) –13/08/2021 (Shift-I) Ans. (d) : The above group of words, 'A herd or flock 169. Select the option that can be used as a one- of animal being driven in a body' is called - 'Drove'. The meaning of other options is word substitute for the given group of words. Throng - a large tethering of people (Mob) The study of different skin diseases Cluster - a grouping of a number of similar things. (a) Neurology (b) Dermatology (bunch, climb). (c) Psychology (d) Radiology Crowd - a large number of things or people considered SSC MTS-07/07/2022 Shift-I together. Cavalier - A person trained in horsemanship Lunatic - A person who behaves in stupid way Gallant - A man who flirt with women 166. Select the option that can be used as a oneword substitute for the given group of words. The place that a person treats as his permanent home, or lives in and has a substantial connection with (a) Domicile (b) Embarkment (c) Oasis (d) Resort SSC CGL (Tier-I) 21/07/2023 (Shift-II) Ans. (a) : The correct one-word substitute for the given group of words 'The place that a person treats as his permanent home, or lives in and has a substantial' is 'Domicile'. Other options give different meaning. Meaning of other options Embarkment - an artificial slope mode of stone. Oasis - a fertile or green area in a desert region. Resort - a place were a lot of people go to on holiday. 167. Select the option that can be used as a oneword substitute for the given group of words. Capable of being bent or pulled into different shapes (a) Lissome (b) Lithe (c) Willowy (d) Ductile SSC CGL (Tier-I) 20/07/2023 (Shift-I) Ans. (d) : The most appropriate one-word substitute for the given group of words ‘Capable of being bent or pulled into different shapes’ is ‘Ductile’ Other options are – Lissome– Moving or able to move with grace and ease. Lithe– Moving or bending easily. Willowy - A person who tall and slim. 168. Select the option that can be used as a oneword substitute for the given group of words. The wide variety of shared and different personal and group characteristics among human beings (a) Community (b) Intensity (c) Viscosity (d) Diversity SSC Selection Posts XI-27/06/2023 (Shift-I) Ans. (d) : The most appropriate one-word substitute for the above group of words is option (d) 'Diversity'. Other options give different meaning– Community – A group of people who live in a particular area or place. Internsity – The quality or state of being intense: extreme strength or force. Viscosity – The quality that some liquids have of being thick and sticky.

One Word Substitution

598

YCT

174. Select the most appropriate one-word substitution for the given words. A doctor who treats skin diseases (a) Ophthalmologist (b) Orthodontist (c) Dermatologist (d) Cardiologist SSC CGL (Tier-II) 16/11/2020 (3 pm - 5 pm) Ans. (c) : The above group of words, 'A doctor who treats skin diseases, is called - 'Dermatologist'. The meaning of other option is Ophthalmologist - a medical doctor specializing in the diagnosis an treatment of diseases of the eye (oculist) Orthodontist - a dentist specializing in the prevention or correction of irregularities or the teeth. Cardiologist - heart specialist. 175. Select the most appropriate word for the given group of words. A particular form of a language which is peculiar to a specific region (a) dialect (b) lingo (c) jargon (d) slang SSC CPO-SI – 25/11/2020 (Shift-I) Ans. (a) : The given group of words, 'a particular form of a language which is peculiar to a specific region' is 'dialect'. The meaning of other options is Lingo - a characteristic language of a particular group. Jargon - words or expressions that are ages by a particular profession or group of people. Slang - very informal words and expression that are more common is spoken language. 176. Select the most appropriate word for the given group of words. A young person tending to commit crime, particularly minor crime. (a) crime (b) derelict (c) delinquent (d) convict SSC CPO-SI – 25/11/2020 (Shift-I) Ans. (c) : The given group of words, 'A young person tending to commit crime, particularly minor crime' is delinquent. The meaning of other options is Crime - criminal offense. Derelict - a person without a home, job or property. Convict - a person who has been convicted of a criminal affence. 177. Select the most appropriate one-word substitution for the given group of words. A large bedroom for a number of people in an institution (a) chamber (b) apartment (c) auditorium (d) dormitory SSC CPO-SI – 24/11/2020 (Shift-I) Ans. (d) : The given group of words, 'A large bedroom for a number of people in an institution' is called 'Dormitory'. The meaning of other options isChamber- a natural or artificial enclosed space. Apartment - flat (a set of rooms used for a holiday/vacation) Auditorium - the area of a theater or concert hall where the audience sits. One Word Substitution

178. Select the word which means the same as the group of words given. A person, animal or plant much below the usual height (a) creature (b) witch (c) wizard (d) dwarf SSC CGL (Tier-I) – 04/06/2019 (Shift-I) Ans : (d) The given group of words, 'A person, animal or plant much below the usual height' is - 'dwarf'. The meaning of other option is creature - a human being. witch - a female sorcerer or magician (enchantress). Wizard - a person who skilled in magic 179. Select the word which means the same as the group of words given. That which can be drawn into a thin wire (a) brittle (b) ductile (c) flexible (d) smooth SSC CGL (Tier-I) – 13/06/2019 (Shift-I) Ans : (b) The given group of words, 'That which can be drawn into a thin wire' is called 'ductile'. The meaning of the other options is Brittle - breakable, having little elasticity. Flexible - able to bend easily (yielding) Smooth - having a surface free from roughness. 180. Select the most appropriate word for the given group of words. A game in which no one wins (a) Conquest (b) Draw (c) Triumph (d) Feat SSC CHSL (Tier-I) –11/07/2019 (Shift-II) Ans : (b) The given of words, 'A game in which no one wins' is called - 'draw'. The meaning of other options is Conquest - the act of conquering. (Subjugation) Triumph - a great success or victory Feat - a notable achievement (effort) 181. Select the most appropriate word for the given group of words The statistical study of the population (a) anthropology (b) demography (c) sociology (d) psychology SSC CHSL (Tier-I) –09/07/2019 (Shift-III) Ans : (b) The given group of words, 'the statistical study of the population' is called - 'demography'. The meaning of other options is Anthropology - the study of the human race especially of its origin, development, custom and beliefs. Sociology - the scientific study of the nature and development of society and social behavior. Psychology - the scientific study of the mind and how it influences behavior. 182. Select the word that means the same as the given group of words. Unable to think clearly because of a mental illness (a) Deranged (b) Eccentric (c) Whimsical (d) Balanced SSC CHSL (Tier-I) 26/10/2020 (Shift-II) Ans. (a) : The given group of words, 'unable to think clearly because of a mental illness' is called 'Deranged'.

599

YCT

The meaning of other options is Eccentric - no concentric (not cantered) Whimsical - capricious, impassive. Balanced - being in a state of proper equilibrium. 183. Select the word which means the same as the group of words given. To laugh at something in a cruel way (a) Deprive (b) Decide (c) Defy (d) Deride SSC CHSL (Tier-I) 15/10/2020 (Shift-II) Ans. (d) : The given group of words, 'to laugh at something' is called - 'deride'. The meaning of other options is Deprive - to take away possessions from someone. decide - determine, come to a decision about something. Defy - hold up, resist. 184. Select the word which means the same as the group of words given. A ruler with total power over a country, typically one who has obtained control by force (a) Traitor (b) Dictator (c) Democrat (d) Protector SSC CHSL (Tier-I) 20/10/2020 (Shift-I) Ans. (b) : The given group of words, ' A ruler with total power over a country, typically one who has obtained control by force' is called - Dictator'. The meaning of other options is Traitor - a person who says one thing and does another. (betrayer) Democrat - an advocate of democratic principles. Protector - a person who cares from person or property. (defender). 185. Select the word which means the same as the group of words given. A situation requiring a choice between equally undesirable alternatives or confusions (a) Sophistication (b) Complication (c) Instability (d) Dilemma SSC CHSL (Tier-I) 19/03/2020 (Shift-II) Ans. (d) : The given group of words, 'A situation requiring a choice between equally undesirable alternatives or confusions' is called - 'Dilemma'. The meaning of other options is Sophistication - mundanely (the quality of being sophisticated) Instability - a lack of balance or state of disequilibrium. (imbalance) Complication - the act or process of complicating puzzling complexity. 186. Choose the most appropriate word that can substitute the given group of words. One who can't speak (a) Bevy (b) Quiet (c) Orphan (d) Dumb SSC MTS – 06/08/2019 (Shift-I) Ans. (d) : The given group of words, 'One who can't speak' is called - 'Dumb'. The meaning of other options are Bevy - a flock of birds. Quiet - peaceful. Orphan - a child who has lost both parents. One Word Substitution

187. Choose the word that can substitute the given sentence. A book of names and addresses (a) Diary (b) Directory (c) Dictionary (d) Journal SSC MTS – 02/08/2019 (Shift-II) Ans. (b) : The given group of words, 'A book of names and addresses' is called - 'Directory'. Diary - a daily contrite recorded of experiences and observations. Dictionary - Lexicon Journal - daybook, a news paper or magazine that deals with a particular subject or profession. 188. Select the word which means the same as the group of words given. A contest between two people to settle a point of honour (a) dual (b) duel (c) duo (d) duet SSC MTS – 09/08/2019 (Shift-II) Ans. (b) : The given group of words, 'A contest between two people settle a point of honour' is called 'duel'. The meaning of other options areDual - double, multiple. Due - couple (pair, span) Duet - two performers or singers who performs together.

E 189. Choose the word that can substitute the given group of words. A person who believes in the equality of all people. (a) Philodendron (b) Philologist (c) Philander (d) Egalitarian SSC MTS 12/05/2023 (Shift-III) Ans. (d) : For the above given group of words. A person who believes in the equality of all people, the most appropriate option is (d) Egalitarian ' Meaning of other options – Philodendron – A tropical climbing plant that is evergreen and is often grown indoors. Philologist – The study of language, especially its history and development. Philander – To have casual or illicit sex with a person or many people. 190. Select the option that can be used as a oneword substitute for the given group of words. A thing fit to eat. (a) Digestible (b) Edible (c) Eligible (d) Curable SSC CGL (Tier-I) 14/07/2023 (Shift-I) Ans. (b) : For the above group of word, “A thing fit to eat’ option (b) edible is appropriate word. Other options give different meaning. Digestible – suitable for use a food Eligible – Having the right to do or having something Curable – that can be made better 191. Choose the word that can substitute the given group of words. A speech made without preparation

600

YCT

(a) Parable (b) Eulogy (c) Extempore (d) Chronicle SSC CHSL (Tier-I) 21/03/2023 (Shift-IV) Ans. (c) : For the above group of word, 'A speech made without Preparation' is option (c) 'Extrmpore' gives an appropriate meaning. Other options give different meaning. Eulogy - A speech or piece of writing the praises someone or something very much. Chronicle - An account or record of a series of events. Parable - A short that uses familiar events to illustrate a religious or ethical point. 192. Choose the word that can substitute the given group of words. A lover of oneself, of one's advancement (a) Philistine (b) Mercenary (c) Cynic (d) Egoist SSC CHSL (Tier-I) 10/03/2023 (Shift-III) Ans. (d) : In the above group of words, "A lover of oneself, of one's advancement" is option (d) Egoist gives an appropriate meaning. Other options give different meaning. Mercenary– A person hired to fight for a foreign army etc. Cynic– Someone who believes that people always act selfishly. Philistine– A person who does not like, understand or enjoy the beauty of art, literature, music. 193. Choose the word that can substitute the given group of words. Book giving information about everything (a) Dictionary (b) Encyclopedia (c) Phonebook (d) Thesis SSC CHSL (Tier-I) 10/03/2023 (Shift-III) Ans. (b) : For the above group of word, "Book giving information about everything" option (b) "Encyclopedia" is appropriate word. Other options give different meaning. Dictionary–A similar reference work giving equivalent words in two or more languages. Thesis–A dissertation resulting from original research, esp. When submitted by a candidate for a degree or diploma. 194. Select the most appropriate one word substitution for the given group of words. A man who is womanish in his habits (a) Effeminate (b) Indictment (c) Hooch (d) Infidel SSC CHSL (Tier-I) 10/03/2023 (Shift-III) Ans. (a) : In the above group of words "A man who is womanish in his habits" is option (a) Effeminate is an appropriate word. Other options gove different meaning. Indictment– A written paper that officially accuses somebody of a crime. Infidel– Somebody who reject a theory of doctrine. Hooch– A strong alcoholic drink, especially something that has been made illegally. One Word Substitution

195. Select the options that can be used as a oneword substitute for the given group of words. A book that contains information on various subjects (a) Encyclopaedia (b) Manuscript (c) Thesaurus (d) Dictionary SSC MTS-05/07/2022 Shift-I Ans. (a) : For the above group of word ‘A book that contains information on various subjects’ option(a) Encyclopedia is appropriate one word. Manusript- A copy of a book before it has been printed. Thesaurus- A book that lists words in group that hare similar meaning. Dictionary- A book that explains the words that are used in a particular subject. 196. Select the option that can be used as a oneword substitute for the underlined group of words. Mohan frequently wastes his money on luxurious objects such as cars and digital devices. (a) Capitalist (b) Extravagant (c) Businessman (d) Consumer SSC CHSL 26.05.2022 Shift-III Ans. (b) : For the above underlined group of words, ‘ wastes his money on luxurious objects’ option (b) extravagant is appropriate one word. Other options have different meaning. Capitalist – A person who invests large amounts of money in a business or businesses. Consumer – A person who buys things are uses services. 197. Select the option that can be used as a oneword substitute for the given group of words. Study of insects (a) Entomology (b) Biology (c) Geology (d) Ecology SSC CGL (Tier-I) 13/04/2022 Shift-I Ans. (a) : For the above group of words, ‘Study of insects’ option (a) entomology is appropriate one word. Biology - The scientific study of living things. Geology - The study of rocks and of the way they are formed. Ecology - The study of the relationships between plants, animals, people and their environment. 198. Select the word which means the same as the group of words given. a book or set of books giving information on many subjects or on many aspects of one subject and typically arranged alphabetically. (a) Glossary (b) Dictionary (c) Encyclopedia (d) Thesaurus SSC CGL (Tier-II) 08.08.2022 Shift-II Ans. (c) : For the above group of words, ‘a book or set of books giving information on many subjects or on many aspects of one subject and typically arranged alphabetically’, option (c) Encyclopedia is appropriate one word other options have different meaning.

601

YCT

Glossary– A list of special or unusual word and their meanings, usually at the end of a text or book. Dictionary–A book or electronic resource that gives a list of the words of a language in alphabetical order and explains what they mean or gives a word for them in a foreign language. Thesaurus–A book that contains lists of words and phrases with similar meanings. 199. Select the option that can be used as a oneword substitute for the given group of words. Release someone from a duty or obligation (a) Banish (b) Exonerate (c) Dismiss (d) Expel SSC Constable GD-17/11/2021 Shift-II Ans. (b) : For the above group of words ‘Release someone from a duty or obligation’, option (b) Exonerate is appropriate one word. Banish- To send somebody away (especially out of the country, usually as a punishment. Expel- To force somebody to leave a country, school, club etc. Dismiss- To order an employee to leave his job. 200. Select the option that can be used as a oneword substitute for the given group of words. A change which is the result of an act or some cause (a) Effect (b) Affect (c) Product (d) Addition SSC Constable GD-26/11/2021 Shift-I Ans. (a) : For the above group of word, A change which is the result of an act or some cause, option (a) effect is appropriate one word. Affect- A person or thing, it influences them or causes them to change in some way. Product- A product is something that is produced and sold in large quantities. Addition- A person or thing that is added to something. 201. Select the option that can be used as a oneword substitute for the given group of words. Study of insects (a) Entomology (b) Ornithology (c) Geology (d) Terminology SSC Constable GD-08/12/2021 Shift-I Ans. (a) : For the above group of words, ‘Study of insects’ option (a) Entomology is appropriate one word. Ornithology - The branch of zoology that deals with the study of birds. Geology - The study of rocks and of the way they are formed. Terminology - Sepecial words or expressions used in relation to a particular subject or activity. 202. Select the option that can be used as a oneword substitute for the given group of words. The art of effective speaking (a) Impromptu (b) Allocation (c) Elocution (d) Extempore SSC Constable GD-26/11/2021 Shift-I One Word Substitution

Ans. (c) : For the above group of words,’ The art of effective speaking’, option (c) Elocution is appropriate one word. Impromptu - Without being prepared or organized. Elocution - The ability to speak clearly, correctly and without a strong accent. Extempore - Spoken or written without any prior preparation or thought. 203. Select the option that can be used as a oneword substitute for the given group of words. A person who listens to someone's private conversation without them knowing. (a) Secret agent (b) Spy (c) Eavesdropper (d) Infiltrator SSC CGL (Tier-I) –16/08/2021 (Shift-I) Ans. (c) : The above group of words, 'A person who listens to someone's private conversation without them knowing' is called eavesdropper. The meaning of other options is secret agents - intelligence officer. Spy - undercover agent. Infiltrator - someone who takes a position surreptitiously for the purpose of espionage. 204. Select the most appropriate one-word substitution for the given words. Speech made without preparation (a) Extempore (b) Statement (c) Speech (d) Debate SSC Sel. Post Phase-VIII (H.L.) 09.11.2020 (Shift-I) Ans. (a) : The above group of words, 'speech made without preparation' is called - 'extempore'. The other options give different meaning. Debate - argument, public debate. Statement - the act of affirming or asserting. 205. Select the most appropriate one-word substitution for the given group of words. The dates when days and nights are of equal length (a) equinox (b) solstice (c) stellar (d) eclipse SSC CPO-SI – 24/11/2020 (Shift-I) Ans. (a) : The given group of words, 'The dates when days and night are of equal length' is called - 'Equinox'. The meaning of other options is Solstice - either of the two times of the year when the seen is at its greatest distance from the celestial equator. Stellar - being or relating to stars. (Astral). Eclipse - occultation 206. Select the most appropriate one-word substitution for the given group of words. Study of environment (a) radiology (b) pathology (c) ecology (d) cosmology SSC CPO-SI – 23/11/2020 (Shift-II) Ans. (c) : The given group of words, 'Study of environment' is called - 'ecology'. The meaning of other options is Radiology - The branch of medical science dealing with medical of x-ray or other penetrating radiation. Pathology - the branch of medical science that studies the causes and nature and effects of diseases. Cosmology - the metaphysical study of the origin and nature of the universe.

602

YCT

207. Select the word which means the same as the group of words given. Something which is fit to be eaten. (a) Unpalatable (b) Delicious (c) Edible (d) Tasty SSC CGL (Tier-I) – 10/06/2019 (Shift-II) Ans : (c) The given group of words, 'Something which is fit to be eaten, is called - 'Edible'. The meaning of other options is Unpalatable - not pleasant or acceptable. Delicious - extremely pleasing to the sense of taste. Tasty - palatable, having a strong and pleasant flavor. 208. Select the word which means the same as the group of words given. An inscription on a tombstone written in memory of the deceased (a) slab (b) basilica (c) epitaph (d) pillar SSC CGL (Tier-I) – 06/06/2019 (Shift-I) Ans. (c) : The given group of words, 'An inscription on a tombstone written in memory of the deceased' is called 'Epitaph'. The meaning of other options is Slab - block consisting of a thick piece of something. Basilica - a roman building used for public administration. Pillar - column. 209. Select the option that can be used as a oneword substitute for the given group of words. Words written on a tombstone in the memory of the person who has died (a) prologue (b) epilogue (c) stela (d) epitaph SSC CPO-SI – 09/12/2019 (Shift-I) Ans : (d) The given group of words, 'Words written on a tombstone in the memory of the person who has died' is called 'epitaph'. The meaning of other options is Prologue - an introductory section of a novel or other literary work. Epilogue - a short speech addressed directly to the audience by an actor at the end of a play. 210. Select the most appropriate word which means the same as the group of words given. To prepare written material for publication by correcting or modifying it (a) summarise (b) abridge (c) edit (d) crop SSC CPO-SI – 11/12/2019 (Shift-I) Ans : (c) The given group of words 'To prepare written material for publication by correcting or modifying it' is called 'edit'. Summarise - give a summary of (resume) Abridge - lessen, diminish or curtail. Crop - harvest. 211. Select the most appropriate option that can be used as a one-word substitute for the given group of words. All the people in a country or area who are entitled to vote in an election. (a) electorate (b) inhabitants (c) residents (d) settlers SSC CPO-SI – 12/12/2019 (Shift-II) One Word Substitution

Ans : (a) The given group of words, 'All the people in country or area who are entitled to vote in an election' is called - 'electorate'. The meaning of other options is Inhabitants - dweller, denizen. Resident - occupies, living in a particular place. Settlers - colonist (a person who settle in a new colony or moves into new country) 212. Select the word which means the same as the group of words given. Soon passing out or of a short duration (a) Frostbite (b) Evanescent (c) Extempore (d) Fluke SSC CHSL (Tier-I) 16/10/2020 (Shift-I) Ans. (b) : The given group of words, 'Soon passing out or of a short duration' is called 'Evanescent'. The meaning of other options is Frostbite - cryopathy extempore - impromptu Fluke - good luck 213. Select the word which means the same as the group of words given. Painless death given to patients to relieve suffering. (a) Euphemism (b) Euthanasia (c) Euphoria (d) Euphorbia SSC CHSL (Tier-I) 18/03/2020 (Shift-III) Ans. (b) : The given group of words, 'Painless death given to patients to relieve suffering' is called Euthanasia'. The meaning of other options is Euphemism - the substitution of an agreeable or inoffensive expression. Euphoria - Euphony (a feeling of great elation. Euphorbia - Genus euphorbia. 214. Select the word which means the same as the group of words given. A person who speaks always in praise of himself (a) Conventionalist (b) Evangelist (c) Egotist (d) Nationalist SSC CHSL (Tier-I) 18/03/2020 (Shift-III) Ans. (c) : The given group of words, 'A person who speaks always in praise of himself' is called 'Egotist'. The meaning of other options isConventionalist - following accepted customs and proprietes. Evangelist - a preacher of the christian gospel. Nationalist - patriot. 215. Select the word which means the same as the group of words given. A situation in which many people leave a place at the same time (a) Navigation (b) Immigration (c) Mitigation (d) Exodus SSC CHSL (Tier-I) 15/10/2020 (Shift-II) Ans. (d) : The given group of words, 'A situation in which many people leave a place at the same time' is called 'Exodus'. The meaning of other options is Navigation - the guidance of ships or airplanes from place to place. Immigration - in-migration (a country of which you are not a native in order to settle there).

603

YCT

216. Select the word which means the same as the group of words given. Lasting for a very short time (a) Ephemeral (b) Endless (c) Illusory (d) Infinite SSC CHSL (Tier-I) 13/10/2020 (Shift-III) Ans. (a) The given words, 'Lasting for a very short time' is called 'Ephemeral'. The meaning of other options is Endless - infinite, dateless. illusory - based on or having the nature of an illusion. Infinite - limitless, bound less. 217. Select the word which means the same as the group of words given. Steal or misappropriate money (a) Overcharge (b) Raid (c) Embezzle (d) Manipulate SSC CHSL (Tier-I) 12/10/2020 (Shift-III) Ans. (c) : The given group of words, 'Something which lasts forever' is called 'Embezzle'. The meaning of other options is Overcharge - surcharge (ask an unreasonable price) Raid - a sudden short attack around) Manipulate - control shrewdly or deviously. 218. Choose the most appropriate word that can substitute the given group of words. Something which lasts forever (a) Eternal (b) Temporal (c) Annual (d) Peripheral SSC MTS – 08/08/2019 (Shift-I) Ans. (a) : The given group of words, 'Something which lasts forever' is called Eternal'. The meaning of other options is Temporal - not eternal, temporary. Annual - occuring or payable every year. Peripheral - circumferential. 219. Select the word which means the same as the group of words given. A person who wastes his money on luxury (a) Monarch (b) Expensive (c) Frugal (d) Extravagant SSC MTS – 09/08/2019 (Shift-III) Ans. (d) : The given group of words, 'A person who wastes his money on luxury' is called - 'Extravagant'. The meaning of other options is Monarch - crowned head, sovereign. Expensive - high in price (costly) Frugal - economical, thrifty. 220. Select the word which means the same as the group of words given. A poem of serious reflection, typically a lament for the dead. (a) Hymn (b) Elegy (c) Ballad (d) Sonnet SSC GD – 02/03/2019 (Shift-II) Ans. (b) : The given group of words, 'A poem of serious reflection, typically a lament for the dead' is called 'Elegy'. The meaning of other options is Hymn - a song of praise (to god or to a saint) Ballast - a narrative poem of popular origin (lay) Sonnet - a verse from consisting of 14 lines with a fixed rhyme scheme. One Word Substitution

221. Select the word which means the same as the group of words given. Someone who leaves one's country to settle in another country. (a) Tourist (b) Emigrant (c) Foreigner (d) Explorer SSC GD – 14/02/2019 (Shift-II) Ans. (b) : The given group of words, 'Someone who leaves one's country to settle in another county' is called - 'Emigrant'. The meaning of other options is Foreigner - alien, outlander. Explorer - adventurer. Tourist - someone who travels for pleasure. 222. Select the word which means the same as the group of words given. A poem that expresses lament for the dead (a) Lyric (b) Ode (c) Elegy (d) Sonnet SSC GD – 14/02/2019 (Shift-I) Ans. (c) : The given group of words, 'A poem that expresses lament for the dead' is called 'Elegy'. The meaning of other option is Lyric - of or related to a category of poetry that expresses emotions. Ode - a lyric poem with complex stanza form a poem that speaks to a person or thing. Sonnet - a poem that has 14 lines, each containing 10 syllables, and a fixed pattern of rhyme. 223. Select the word which means the same as the group of words given. A short speech at the end of a play. (a) Eulogy (b) Prologue (c) Epilogue (d) Chorus SSC GD – 11/02/2019 (Shift-III) Ans. (c) : The given group of words, 'A short speech at the end of a play' is called 'Epilogue'. The meaning of other options is Eulogy - a formal expression of praise for someone who has died recently. Prologue - an introductory section of a novel or other literary work. Chorus - a group of people assembled to sing together. 224. Select the word which means the same as the group of words given. A disease that is prevalent in a particular area. (a) Genetic (b) Contagious (c) Endemic (d) Viral SSC GD – 11/02/2019 (Shift-II) Ans. (c) : The given group of words, 'A disease that is prevalent in a particular area' is called 'Endemic'. The meaning of other options is Genetic - relating to the science of genetics. Contagious - contractible, transmissible. Viral - caused by a virus. 225. Select the word which means the same as the group of words given. One who studies and cares about environment (a) Archeologist (b) Ecologist (c) Anthropologist (d) Biologist SSC GD – 11/02/2019 (Shift-II)

604

YCT

Ans. (b) : The given group of words, 'One who studies and cares about environment' is called 'Ecologist'. The meaning of other options is Archeologist - one who studies prehistoric people and their culture. Anthropologist - a person who studies anthropology. Biologist - a scientist who studies living organ. (life scientist).

F 226. Choose the word that can substitute the given group of words. Destruction of a fetus (a) Ellipse (b) Abhor (c) Fratricide (d) Feticide SSC MTS 18/05/2023 (Shift-I) Ans. (d) : The one-word of the given group of words‘Destruction of a fetus’ is ‘Feticide’. Meaning of other optionsEllipse – A regular oval shape. Abhor – To hate something very much. Fratricide – The killing of one's brother. 227. Select the option that can be used as a oneword substitute for the given group of words. Being careful that every detail of something is correct (a) Squeamish (b) Fastidious (c) Impregnable (d) Stubborn SSC CGL (Tier-I) 18/07/2023 (Shift-III) Ans. (b) : The most appropriate one-word substitute of the above group of word 'Being careful that every detail of something is correct' is option (b) 'Fastidious' (irritable). Meaning of other optionsSqueamish- Easily shocked as by the sight of blood. Impreganble - not able to be captured by attack very strong? Stubborn - 'Performed or carried on in an unyieleding, obstinate manner'. 228. Select the option that can be used as a oneword substitute for the given group of words. The constitutional right to vote (a) Voting (b) Buttoning (c) Polling (d) Franchise SSC CHSL (Tier-I) 03/08/2023 (Shift-II) Ans. (d) : A one word substitute for the given group of words 'the constitutional right to vote' is option (d) 'Franchise'. Meaning of other options Voting - To choose somebody for a particular position or prize. Buttoning - To choose somebody for a particular position or prize. Polling - To propel a boat or raft with a pole. 229. Select the option that can be used as a oneword substitute for the underlined part of the given sentence. Dogs are known for possession of the quality of being faithful. (a) fidelity (b) creditability (c) honesty (d) genuineness SSC Selection Posts XI-28/06/2023 (Shift-III) One Word Substitution

Ans. (a) : In the above sentence underlined, group of word the quality of being faithful option (a) "fidelity" gives appropriate meaning. Other options give different meaning. Creditability - Of a quite good standard and deserving praise or approval. Honesty - The quality of being honest. Genuineness - The quality of being honest and sincore. 230. Select the most appropriate one word substitution for the given words. a period of two weeks (a) month (b) century (c) decade (d) fortnight SSC CHSL (Tier-I) –06/08/2021 (Shift-I) Ans. (d) : The above group of words, 'A period of two weeks' is called 'fortnight'. The meaning of other options is Century - a period of 100 years. Decade - a period of 10 years (decennary). Month - one of the twelve divisions of the calendar year. 231. Select the option that can be used as one-word substitute for the given group of words. A person who arranges and sells cut flowers (a) Grocer (b) Flautist (c) Florist (d) Gardener SSC Constable GD-10/12/2021 Shift-III Ans. (c) : For the above group of words, ‘ A person who arranges and sells cut flowers, option (c) florist is appropriate one word. Other options have different meaning. Grocer - A shop that sells food and other things. Flautist - A person who plays a musical instrument. Gardener - A person whose job is to work in a garden. 232. Select the option that can be used as a oneword substitute for the given group of words. Something that is to your advantage but happened by chance (a) Deliberate (b) Fortuitous (c) Occidental (d) Purposeful SSC CGL (Tier-I) –18/08/2021 (Shift-I) Ans. (b) : The above group of words, 'Something that is to your advantage but happened by change' is called 'fortuitous'. The meaning of other options is Deliberate - carefully thought out in advance. Occidental - denoting or characteristic of chantries of Europe and western hemisphere. Purposeful - meaningful (having a meaning). 233. Select the most appropriate word for the given group of words. A group of birds of one kind (a) Crowd (b) Bevy (c) Band (d) Flock

605

SSC Sel. Post Phase-VIII (M.L.) 09.11.2020 (Shift-III)

Ans. (d) : The above group of words, 'A group of birds of one kind' is called 'Flock'. Crowd - gang, posse. Bevy - a flock of birds (especially when gathered close together on the ground) Band - an unofficial association of people or group. YCT

234. Select the word which means the same as the group of words given. easily broken (a) fragile (b) pliable (c) malleable (d) ductile SSC CGL (Tier-I) – 19/06/2019 (Shift-III) Ans. (a) : The given group of words 'easily broken' is called - 'fragile'. The meaning of other options is Pliable - ductile, tensile. Malleable - capable of being shaped or bent. Ductile - pliant, given a form. 235. Select the word which means the same as the group of words given. Someone who believes that people cannot change the way events will happen (a) prophet (b) fatalist (c) fanatic (d) seer SSC CGL (Tier-I) – 06/06/2019 (Shift-II) Ans : (b) The given group of words, 'Someone who believes that people cannot change the way events will happen' is called 'fatalist'. Prophet - someone who speaks by divine inspiration Fanatic - a person who motivated by irrational enthusiasm. Seer - a person with unusual powers of foresight. 236. Select the most appropriate word for the given group of words. A person who sells and arranges cut flowers. (a) Nutritionist (b) Agriculturist (c) Florist (d) Botanist SSC CHSL (Tier-I) –11/07/2019 (Shift-I) Ans : (c) The given group of words, 'A person who sells and arranges cut flowers' is called 'Florist'. The meaning of other options is Nutritionist - a specialist in the study of flood, nourishment and health. (Dietician) Agriculturist - cultivator, grower Botanist - a biologist specializing in the study of plants. 237. Select the most appropriate word for the group of words. One who is difficult to please (a) fastidious (b) fanatic (c) fatalist (d) feminist SSC CHSL (Tier-I) –05/07/2019 (Shift-I) Ans : (a) The given group of words, 'One who is difficult to please' is called - 'fastidious'. The meaning of other options is Fanatic - a person motivated by irrational enthusiasm. (fiend) Fatalist - predestinations. Feminist - a supporter of feminism. 238. Select the word which means the same as the group of words given. A group of ships sailing together (a) Fleet (b) Vessels (c) Navy (d) Cruisers SSC CHSL (Tier-I) 13/10/2020 (Shift-II) Ans. (a) : The given group of words, 'A group of ships sailing together' is called 'Fleet'. One Word Substitution

The meaning of other options is Vessels - an object used as a container (especially for liquids) Navy - naval forces. Cruisers - pleasure boat 239. Choose the most appropriate word that can substitute the given group of words. A group of ships (a) Fleet (b) Suite (c) Army (d) Bevy SSC MTS – 06/08/2019 (Shift-III) Ans. (a) : The given group of words, 'A group of ships' is called 'Fleet'. The meaning of other options is Suite - the group following and attending to some important person (cortege). Army - regular army (a permanent organization of the military, land forces of nations) Bevy - a flock of birds. 240. Select the most appropriate word of the given group of words. An act of murdering one's brother (a) Fratricide (b) Regicide (c) Genocide (d) Patricide SSC MTS – 22/08/2019 (Shift-II) Ans. (a) : The given group of words, 'An act of murdering one's brother' is called 'Fratricide'. The meaning of other options is Regicide - the act of killing a king. Genocide - systematic killing of a crucial (race murder) Patricide - the murder of your father. 241. Select the most appropriate word for the given group of words. Something which leads to death (a) Fatal (b) Trial (c) Penal (d) Vital SSC MTS – 02/08/2019 (Shift-III) Ans. (a) : The given group of words, 'Something which leads to death' is called 'Fatal'. The meaning of other option is Trial - test Penal - Pemishable Vital - life sustaining essential. 242. Select the most appropriate word for the given group of words : A person motivated by irrational enthusiasm (a) Infidel (b) Fanatic (c) Atheist (d) Polytheist SSC MTS – 20/08/2019 (Shift-I) Ans. (b) : The given group of words, 'A person motivated by irrational enthusiasm' is called 'Fanatic'. The meaning of other options. Infidel - a person who does not acknowledge your god. Atheist - someone who denies the existence of god. Polytheist - one who believes in a plurality of gods. 243. Select the word which means the same as the group of words given. A period of two weeks (a) Quarter (b) Decade (c) Fortnight (d) Weekend SSC MTS – 08/08/2019 (Shift-II)

606

YCT

Ans. (c) : The given group of words, 'A period of two weeks' is called 'Fortnight' (two weeks). The meaning of other options is Quarter - One of four equal parts. (Three months) Decade - a period of 10 years Weekend - Saturday and Sunday or slightly longer.

G 244. Select the most appropriate word for the given group of words. Tending to associate with others of one's kind (a) Pungent (b) Venial (c) Gregarious (d) Pedantic SSC MTS 02/05/2023 (Shift-I) Ans. (c) : The most appropriate word for the group of words 'Tending to associate with others of one's kind' is 'Gregarious' other words are ____ pungent- Smelling or testing very strong and sharp. Venial- That may be forgiven, pardonable. Pedantic - Too worried about rules or details. 245. Select the option that can be used as a oneword substitute for the underlined part of the given sentence. There is a place for grains in the backyard of this house. (a) dormitory (b) apiary (c) archive (d) granary SSC CGL Mains 26/10/2023 (Shift-I) Ans. (d) : In the above sentence underlined group of word 'place for grains' is granary. Other options give different meaning. Dormitory – A room for several people to sleep in, especially in school Apiary - A place where bees are kept. Archive - Collection of historical record of a Government. 246. Select the option that can be used as a oneword substitute for the given group of words. The murder of a large number of people from a particular nation or ethnic group. (a) Genocide (b) Honour killing (c) Geronticide (d) Mariticide SSC CGL (Tier-I) 26/07/2023 (Shift-II) Ans. (a) : The most appropriate one word substitute for the given group of words 'The murder of a large number of people from a particular nation or ethnic group ' is ' Genocide'. The meaning of the other words is– Honour Killing means – the Killing of a relative especially a girl. Geronticide means – The killing or euthanasia of the elderly. Mariticide means – The act of Killing one's spouse, especially the murder of a husband by his a life. Thus, the correct answer is option (a). One Word Substitution

247. Select the option that can be used as a oneword substitute for the given group of words. Drawings or writing on a wall in a public place (a) Graffiti (b) Caricature (c) Collage (d) Portrait SSC CGL (Tier-I) 18/04/2022 Shift-I Ans. (a) : For the above group of words, 'Drawings or writing on a wall in a public place' option (a) Graffiti is appropriate one word. Caricature - A representation of a person that is exaggerated for comic effect. Collage - The art of making a picture by sticking pieces of colored paper, cloth or photographs onto a surface. Portrait - A portrait is a painting, drawing or photograph of a particular person. 248. Select the option that can be used as a oneword substitute for the given group of words. Science of heredity (a) Orthopedics (b) Pathology (c) Genealogy (d) Genetics SSC Constable GD-03/12/2021 Shift-II Ans. (d) : For the above group of words, Science of heredity’, option (d) Genetics is appropriate one word. Orthopedics - The treatment or study of bones. Pathology - The scientific study of diseases of the body. Genealogy - A record or account of the ancestry and descent of a person, family or group. 249. Select the word which means the same as the group of words given. A room or building with equipment for doing physical exercise (a) dormitory (b) convent (c) infirmary (d) gymnasium SSC CPO-SI (Tier-II) 27/09/2019 (3 pm - 5 pm) Ans. (d) : The above group of words, 'A room or bulging with equipment for doing physical exercise is called 'gymnasium'. The meaning of other options is Dormitory - residence hall, student residence. Infirmary - a health facility where patients receive treatment (hospital) Convent - a religious residence especially for nun. 250. Select the most appropriate word which means the same as the group of words given. One who is easily deceived (a) delible (b) gullible (c) incorrigible (d) infallible SSC CPO-SI – 09/12/2019 (Shift-II) Ans : (b) The given group of words 'One who is easily deceived' is called 'Gullible'. The meaning of other options is Delible - Able to be deleted. Infalliable - Never making mistakes or being wrong. 251. Select the option which is the most appropriate one word substitution for the given group of words. Animals living in flocks (a) gregarious (b) bohemian (c) social (d) herbivorous SSC CPO-SI – 12/12/2019 (Shift-I)

607

YCT

Ans : (a) 'Animals living in flocks' is called 'Gregarious'. Bohemian - unconventional Herbivorous - feeding only on plants (baccivorous) 252. Select the most appropriate word for the group of words. people who enjoy social company (a) gregarious (b) unanimous (c) gorgeous (d) herbivorous SSC CHSL (Tier-I) –08/07/2019 (Shift-III) Ans : (a) The given group of words 'people who enjoy social company' is called 'gregarious'. The meaning of other options is Unanimous - in complete agreement. Gorgeous - beautiful. Herbivorous - feeding an grasses. 253. Select the most appropriate word for the group of words. One who eats too much (a) overweight (b) glutton (c) corpulent (d) obese SSC CHSL (Tier-I) –08/07/2019 (Shift-III) Ans : (b) The given group of words 'One who eats too much' is called 'Glutton'. Overweight - fleshy, heavy, fat. Corpulent - excessively large obese, weighty) Obese - corpulent, fat. 254. Select the word which means the same as the group of words given. A person who studies the structure of the earth (a) Geographer (b) Gemologist (c) Gynaecologist (d) Geologist SSC CHSL (Tier-I) 19/03/2020 (Shift-III) Ans. (d) : The given group of words 'a Person who studies the Structure of the earth' is called 'Geologist'. The meaning of other options is Geographer - an expert on geographer. Gemologist - One who works in the field of gemology. Gynaecologist - woman's doctor. 255. Select the word which means the same as the group of words given. Tending to associate with others of ones kind (a) Congruous (b) Harmounious (c) Gregarious (d) Genial SSC CHSL (Tier-I) 13/10/2020 (Shift-III) Ans. (c) The given group of words 'Tending to associate with others of ones kind' is called 'Gregarious'. The meaning of other options is Congruous - suitable or appropriate Harmonious - balanced Genial - affable, amiable, cordial 256. Select the word which means the same as the group of words given: A part of the city especially the slum area (a) Ghetto (b) Metropolis (c) Suburb (d) Colony SSC CHSL (Tier-I) 21/10/2020 (Shift-III) Ans. (a) : The given group of words 'A part of the city especially the slum area' is called 'Ghetto'. The meaning of other options is metropolis - Urban city, city. One Word Substitution

Suburb - a residential district located on the act skirt of a city. Colony - a group of organism of the same type living or growing together. 257. Select the most appropriate word for the given group of words. An extremely large mass of ice which moves very slowly, often down a mountain valley (a) Mirage (b) Glacier (c) Island (d) Ocean SSC MTS – 13/08/2019 (Shift-II) Ans. (b) : The given words, 'An extremely large mass of ice which moves very slowly, often down a mountain valley' is called 'Glacier'. The meaning of other options is Mirage - illusion Island - a land mass that is surrounded by water. Ocean - sea.

H 258. Select the option that can be used as a oneword substitute for the given group of words. A place where airplanes are kept for maintenance (a) Scullery (b) Hangar (c) Hanger (d) Aviary SSC CGL (Tier-I) 27/07/2023 (Shift-III) Ans. (b) : The most appropriate one-word substitute for the above given group of words 'A place where airplanes are kept for maintenance' is 'Hangar'. Meaning of other optionScullery- A small room or kitchen in an old house. Hanger- A metal or plastic object with a look that is used for hanging up cloths in a cupboard. Aviary- A large cage in which birds are kept. 259. Select the option that can be used as a oneword substitute for the underlined segment. The company was known for its deserving respect or admiration business practices and honest and fair ethical standards. (a) honourable (b) decent (c) respectable (d) trustworthy SSC CHSL (Tier-I) 04/08/2023 (Shift-III) Ans. (a) : The most appropriate one – word for the underlined segment 'know for its deserving respect or admiration business practices and honest and fair ethical standards' is option (a) 'honourable'. Decent - Socially acceptable or good. Respectable - Having or deserving the respect of other people, estimable, worthy. Trustworthy - Able to be trusted. 260. Choose the word that can substitute the given group of words. Exaggerated claims, not meant to be taken seriously (a) Hyperpyrexia (b) Hyperbole (c) Hypergamy (d) Hyperreal SSC CHSL (Tier-I) 21/03/2023 (Shift-IV)

608

YCT

Ans. (b) : For the above group of word, "Exaggerated claims, not meant, to be taken seriously" is option (b) 'Hyperbole' is given appropriate meaning. Other options give different meaning. Hypergamy – Marrying a spouse of higher caste Hyperpyrexia – High fever Hyperreal – Exaggerating reality 261. Select the most appropriate word for the given group of words. A cage for small animals (a) Cave (b) Hutch (c) Shed (d) Hut SSC MTS-07/07/2022 Shift-II Ans. (b) : For the above group of words ‘A cage for small animals’, option (b) Hutch is appropriate one word. Cave - A hollow in the earth, especially one opening more or less horizontally into a hill, mountain etc. Shed - A small building that is used for keeping things or animal in. Hut - A small building with one room, usually made of wood or metal. 262. Select the option that can be used as a oneword substitute for the given group of words. A person exhibiting excessive worry about one's health (a) Intellectual (b) Hypochondriac (c) Megalomaniac (d) Expert SSC MTS-12/07/2022 Shift-I Ans. (b) : For the above group of words ‘A person exhibiting worry about one’s health’, option (b) Hypochondriac is appropriate one word. Intellectual - Connected with or using a person's ability to think in a logical way and understanding things. Megalomaniac - A person suffering from megalomania. Expert - A person who has a lot of special knowledge or kill. 263. Select the option that can be used as a oneword substitute for the given group of words. The sound of owls (a) Moo (b) Caw (c) Cluck (d) Hoot SSC Constable GD-30/11/2021 Shift-III Ans. (d) : For the above group of words ‘The sound of owls’ option (d) Hoot is appropriate one word. Other options have different meaning. 264. Select the option that can be used as a oneword substitute for the given group of words. A large building with an extensive floor area, typically for housing aircraft (a) Barn (b) Hangar (c) Shed (d) Airport SSC CGL (Tier-I) –16/08/2021 (Shift-I) Ans. (b) : The above group of words, 'A large building with an extensive floor area typically for housing aircraft' is called 'Hangar'. The meaning of other options is Barn - a large farm building for storing grain or keeping animals in. Shed - disgorage, spill. Airport - aerodrome, drome. One Word Substitution

265.

In the following question, out of the four alternative, select the alternative which is the best substitute of the phrase. Preferring or attracted to sunlight (a) Lithophilous (b) Heliophilous (c) Topophilia (d) Homophile SSC CGL (Phase-II) 17/02/2018 Ans. (b) The above group of words, 'Preferring or attracted to sunlight' is called 'Heliophilous'. The meaning of other options is Lithophilous - growing or living in stony places. Topophilia - the lover of or emotional connections with place or physical environment. Hemophile - bleeder (someone who has hemophilia and is subject to uncontrollable bleeding). 266. Select the correct one – word substitution for the given words. Work for which no salary is paid (a) Optional (b) Honorary (c) Mandatory (d) Bonus SSC Sel. Post Phase-VIII (G.L.) 09.11.2020 (Shift-II)

Ans. (b) : The above group of words, 'Work for which no salary is paid' is called 'Honorary'. The other options give different meanings. Optional - not compulsory (elective) Mandatory - compulsory, required. Bonus - incentive. 267. Select the most appropriate word which means the same as the group of words given. One who holds a post without any salary (a) volunteer (b) samaritan (c) honorary (d) highbrow SSC CPO-SI – 09/12/2019 (Shift-II) Ans : (c) The given group of words, 'One who holds a post without any salary' is called 'Honorary'. The meaning of other options is Volunteer - unpaid worker. Samaritan - a person who gives help and sympathy to people who need it. 268. Select the most appropriate word which means the same as the group of words given. To spend winter in a dormant state (a) hoist (b) hibernate (c) convalesce (d) connote SSC CPO-SI – 11/12/2019 (Shift-I) Ans : (b) The given group of words, 'To spend winter in a dormant state' is called - 'hibernate'. The meaning of other options is Convalesce - recover, recuperate. Connote - express or state indirectly. (Imply). 269. Select the word which means the same as the group of words given. A box or cage for rabbits or small animals (a) lair (b) hutch (c) menagerie (d) kennel SSC CPO-SI – 13/12/2019 (Shift-II) Ans. (b) : The given group of words, 'A box or cage for rabbits or small animals' is called 'hutch'. The meaning of other options is Lair - den (The habitation of wild animals) Menagerie - zoo, zoological garden Kennel - dog house.

609

YCT

270. Select the word which means the same as the group of words given. The therapeutic use of sunlight (a) psephology (b) heliotherapy (c) dactylology (d) generalogy SSC CPO-SI – 13/12/2019 (Shift-II) Ans. (b) : The given group of words 'The therapeutic use of sunlight' is called 'heliotherapy'. Psephology - the branch of sociology that studies election trends (as by option polls) Dactylology - the method of using manual sign language, as in communicating with deaf people. Genealogy - the study or investigation of ancestry and family history. 271. Select the most appropriate word for the given group of words Place for collection of dried plant specimens (a) green house (b) nursery (c) warehouse (d) herbarium SSC CHSL (Tier-I) –10/07/2019 (Shift-I) Ans : (d) The given group of words, 'Place for collection of dried plant specimens' is called 'herbarium'. The meaning of other options is Green house - a building with glass walls and roof. (glasshouse) Nursery - a child's room for a baby or greenhouse Warehouse - a storehouse for goods and merchandise. 272. Select the word which means the same as the group of words given. Condition of sleep certain parts of the year (a) Somnambulism (b) Constipation (c) Dehydration (d) Hibernation SSC CHSL (Tier-I) 19/03/2020 (Shift-III) Ans. (d) : The given group of words 'Condition of sleep during certain parts of the year' is called 'Hibernation'. The meaning of other options is Somnambulism - walking by a person who is asleep. (sleepwalking) Constipation - the act of making something futile and useless. Dehydration - the press of extracting moisture. 273. Select the word which means the same as the group of words given. A song or music in praise of God (a) Hymn (b) Epic (c) Elegy (d) Ode SSC CHSL (Tier-I) 19/03/2020 (Shift-I) Ans. (a) : The given group of words, 'A song or music in praise of god' is called 'Hymn'. The meaning of other options is Epic - a long narrative poem telling of a hero's deeds. Elegy - a mournful poem; a lament for the dead. Ode - a lyric poem with comply stanza forms. 274. Select the word which means the same as the group of words given. Things which are of the same kind and of the same dimensions. (a) Homogenous (b) Homophony (c) Homosexual (d) Homonymous SSC CHSL (Tier-I) 19/03/2020 (Shift-I) One Word Substitution

Ans. (a) : The given group of words 'Things which are of the same kind and of the same dimensions.' is called 'Homogenous'. The meaning of other options is Homophony - the same pronunciation for words of different origins. Homosexual - having a sexual attraction of persons of the same sex. (gay woman) Homonymous - having the same designation. 275. Select the word which means the same as the group of words given. Come in the way of (a) Indulge (b) Deprive (c) Hamper (d) Facilitate SSC CHSL (Tier-I) 15/10/2020 (Shift-III) Ans. (c) : The given group of words 'come in the way of' is called 'Hamper'. The meaning of other options is Indulge - enjoy to excess (luxuriate) Deprive - take away (impoverish) Facilitate - make easier (alleviate, ease) 276. Select the word which means the same as the group of words given: Making one feel very sad (a) Heart-to-heart (b) Heart-searching (c) Heart-stopping (d) Heart-wrenching SSC CHSL (Tier-I) 21/10/2020 (Shift-III) Ans. (d) : The given group of words 'Making one feel very sad' is called 'Heart-wrenching'. The other options is not a suitable answer. 277. Select the word which means the same as the group of words given. Something that is extremely unpleasant (a) Heightened (b) Humane (c) Humorous (d) Horrendous SSC CHSL (Tier-I) 20/10/2020 (Shift-III) Ans. (d) : The given group of words 'Something that is extremely unpleasant' is called 'Horrendous'. Heightened - make more extreme. Humane - civilised Humorous - full of or characterized by human (mirthful) 278. In the following question, out of the four given alternatives, select the alternative which is the best substitute of the phrase. A woman who entertains guests. (a) Nurse (b) Waiter (c) Hostess (d) Maid SSC MTS 10-10-2017 (Shift-II) Ans. (c) : The given group of words, 'A woman who entertains guests' is called 'Hostess'. The meaning of other options is Nurse - a person whose job is to take care of sick. Waiter - a person whose job is to serve customers at their table in a restaurant, etc. Maid - a female servant in a house or hotel. 279. In the following question, out of the four given alternatives, select the alternative which is the best substitute of the Phrase. That which eats grass. (a) Carnivore (b) Omnivore (c) Herbivore (d) Photovore SSC MTS 11-10-2017 (Shift-I)

610

YCT

Ans : (c) The given group of words ' That which eats grass.' is called 'Herbivore' The meaning of other options is Carnivore - any animal that eats meat. Omnivore - an animal or a person that eats all types of food. 280. Choose the most appropriate word that can substitute the given group of words. One who pretends to be what he is not (a) Director (b) Hypocrite (c) Creator (d) Acrobat SSC MTS – 02/08/2019 (Shift-I) Ans. (b) : The given Group of words 'One who pretends to be what he is not' is called 'hypocrite'. The meaning of other options is Director - one of a group of senior managers who run a company. Creator - God Acrobat - an entertainer who performs difficult acts such as balancing on high ropes, especially at a circus. 281. Select the most appropriate word for the given group of words. An abnormal fear of water (a) Cacophobia (b) Hydrophobia (c) Algophobia (d) Pyrophobia SSC MTS – 22/08/2019 (Shift-I) Ans. (b) : The given group of words, 'An abnormal fear of water' is called 'Hydrophobia'. The meaning of other options is Cacophobia - an intense fear of ugliness. Algophobia - a morbid fear of pain Pyrophobia - a morbid fear of fire. 282. Select the most appropriate word for the given group of words. Friendly and welcoming to visitors. (a) Hostile (b) Hostel (c) Hospital (d) Hospitable SSC MTS – 20/08/2019 (Shift-II) Ans. (d) : The given group of words, 'Friendly and welcoming to visitors' is called 'Hospitable'. The meaning of other options is Hostile - characterized by enmity or ill will. Hostel - a hotel providing overnight lodging for travelers. (Inn, lodge). Hospital - a health facility where sick or injured people are given medical. 283. Select the word which means the same as the group of words given. One who loses temper very soon (a) Temporal (b) Argumentative (c) Tampered (d) Hot-headed SSC MTS – 13/08/2019 (Shift-III) Ans. (d) : The given group of words, 'One who loses temper very soon, is called - 'hot-heated'. The meaning of other options is Temporal - not eternal (temporary) Argumentative - quarrelsome. Tampered - interfere with. 284. Choose the most appropriate word that can substitute the given group of words. A person pretending to be somebody he isn't. (a) Aristocrat (b) Hypocrite (c) Conservative (d) Bureaucrat SSC MTS – 06/08/2019 (Shift-II) One Word Substitution

Ans. (b) : The given group of words, 'A person pretending to be some body the isn't' is called 'hypocrite'. The meaning of other options is Aristocrat - a member of change (blimpish) Bureaucrat - an official of a bureaucracy 285. Select the word which means the same as the group of words given. A song written in the praise of God (a) Hymn (b) Elegy (c) Ode (d) Epic SSC GD – 11/02/2019 (Shift-III) Ans. (a) : The given group of words, 'A song written in the praise of god' is called 'Hymn' The meaning of the options is elegy - a mournful poem; a lament for the deal. ode - a lyric poem with complex stanza forms. Epic - a long narrative poem telling of a hero's deeds. (heroic poem)

I 286. Select the option that can be used as a oneword substitute for the given group of words. Too strong to be defeated or changed (a) Invincible (b) Sovereign (c) Vigorous (d) Headstrong SSC CGL (Tier-I) 25/07/2023 (Shift-IV) Ans. (a) : The above given group of words 'Too strong to be defeated or changed,' a one word substitute is 'invincible'. Meaning of other options Sovereign - A king, queen or other ruler of a country. Vigorous - Very forceful or energetic. Headstrong - A person who is determined to do things their own way refuses to listen to advice. 287. Select the option that can be used as a oneword substitute for the given group of words. Certain to happen (a) Inevitable (b) Convertible (c) Indispensable (d) Justifiable SSC CGL (Tier-I) 17/07/2023 (Shift-II) Ans. (a) : For the above group of word, "certain to happen' option (a) 'inevitable' gives appropriate meaning other options gives different meaning Convertible – Able to changed into another form Indispensable – very important Justifiable – Reasonable and acceptable 288. Select the option that can be used as a oneword substitute for the given group of words. Done without conscious control; unintentional (a) Invincible (b) Involuntary (c) Invitro (d) Invoke SSC CHSL (Tier-I) 04/08/2023 (Shift-III) Ans. (b) : The most appropriate one–word for the given group of words 'Done without conscious Control; unintentional' is option (b) 'Involuntary'. Invincible - Too strong or power to be defeated. Invitro - Taking place outside a living body, in scientific apparatus. Invoke - To appeal to or cite in support or justification.

611

YCT

289. Choose the word that can substitute the given group of word. One who does not express oneself freely (a) Skeptic (b) Erudite (c) Introvert (d) Pessimist SSC CHSL (Tier-I) 20/03/2023 (Shift-I) Ans. (c) : For the above group of words "One who does not express oneself freely" option (c) "Introvert" is appropriate one word. Skeptic - A person who doubts that something is true, right. Erudite - Showing a lot of knowledge. Pessimist - Thinking that bad things are more likely to happen. 290. Choose the word that can substitute the given group of words. Travelling from place to place (a) Recluse (b) Arcane (c) Reverent (d) Itinerant SSC CHSL (Tier-I) 17/03/2023 (Shift-IV) Ans. (d) : For the above group of words 'Travelling from place to place' option (d) 'Itinerant' is appropriate one word. Recluse - A person who lives alone and deliberately avoids other people. Arcane - Mysterious and know only by a few people. Reverent - Feeling or showing profound respect. 291. Select the option that can be used as a oneword substitute for the underlined group of words in the given sentence. Adversity always presents opportunities for the examination of one's own conscious thoughts and feelings. (a) evaluation (b) cognition (c) judgement (d) introspection SSC Selection Posts XI-27/06/2023 (Shift-I) Ans. (d) : The most appropriate one-word for the above underlined group of words is option (d) 'introspection'. Other options give different meanings– Evaluation – To judge the value or condition of someone or something. Cognition – The use of conscious mental processes. Judgement – The ability to form valuable opinions and make good decisions. 292. Select the option that can be used as a oneword substitute for the given group of words. Someone who attacks cherished ideas or traditional institutions (a) Believer (b) Iconoclast (c) Humanitarian (d) Misanthrope SSC MTS-06/07/2022 Shift-II Ans. (b) : For the above group of words ‘someone who attacks cherished ideas or traditional institutions’, option (b) Iconoclast is appropriate one word. Believer - A person who has religious beliefs. Humanitarian - A person concerned with the welfare of all humankind. Misanthrope - A person who hates or avoids other people. One Word Substitution

293. Select the options that can be used as a oneword substitute for the given group of words. A thing or substance that is liable to catch fire easily (a) Inaudible (b) Inflammable (c) Incombustible (d) Extinguishable SSC MTS-05/07/2022 Shift-I Ans. (b) : For the above group of words, ‘A thing or substance that is liable to catch fire easily’, option (b) inflammable is appropriate one word. Inaudible - Unbale to be heard. Incombustible - Unable to be burn easily. Extinguishable - Capable of being extinguished or killed. 294. Select the option that can be used as a oneword substitute for the given group of words. One who supervises students in an examination hall (a) Inspector (b) Examinee (c) Invigilator (d) Teacher SSC CGL (Tier-I) 11/04/2022 Shift-II Ans. (c) : For the above group of words, option (c) invigilator is appropriate one word. Inspector - Someone who looks into, observes and examines things. Examinee - A person who is examined. Teacher - A person whose job is to teach. 295. Select the option that can be used as a oneword substitute for the given group of words. An image of a God used for worship (a) Temple (b) Model (c) Icon (d) Idol SSC CGL (Tier-I) 13/04/2022 Shift-III Ans. (d) : For the above group of words, ‘An image of a God used for worship’ option (d) Idol is appropriate one word. Temple - A building where people pray to a god or gods. Model - To work or act as a fashion or art model. Icon - A person or thing that is considered to be a symbol of something. 296. Select the option that can be used as a oneword substitute for the given group of words. The examination or observation of one's own mental and emotional processes (a) Inspection (b) Valuation (c) Introspection (d) Assessment SSC CGL (Tier-I) 12/04/2022 Shift-III Ans. (c) : For the above group of words, option (c) introspection is appropriate one word. Other options have different, meaning. Inspection - A careful and critical examination. Valuation – The act of deciding how much money something is worth. Assessment - To judge or decide the amount, value, quality or importance of something.

612

YCT

297. Select the option that can be used as a oneword substitute for the given group of words. A sound that cannot be heard (a) Inedible (b) Audible (c) Indelible (d) Inaudible SSC CGL (Tier-I) 11/04/2022 Shift-III Ans. (d) : For the above group of words, option (d) inaudible is appropriate one word. Other options have different meaning. Inedible - Not suitable to eat. Audible - Able to be heard. Indelible - Impossible to remove or forget. 298. Select the option that can be used as a oneword substitute for the given group of words. Unable to express feeling clearly (a) Inarticulate (b) Eloquent (c) Affluent (d) Verbose SSC CGL (Tier-I) 19/04/2022 Shift-II Ans. (a) : For the above group of words ‘unable to express feeling clearly’, option (a) inarticulate is appropriate one word. Eloquent - Able to use language and express your opinions well. Affluent - Having a lot of money or property. Verbose - Using move words than are necessary. 299. Select the option that can be used as a oneword substitute for the given group of words. Pictures given in a book for the purpose of explaining things (a) Illustration (b) Demonstration (c) Decoration (d) Ornamentation SSC Constable GD-06/12/2021 Shift-III Ans. (a) : For the above group of words, ‘ Picture given in a book for the purpose of explaining things’ option (a) Illustration is appropriate one word. Demonstration - The act of presenting something to sight or view. Decoration - The act of decorating. Ornamentation - The use of objects, designs to decorate something. 300. Select the option that can be used as a oneword substitute for the given group of words. One who supervises in an examination hall (a) Examinee (b) Teacher (c) Invigilator (d) Inspector SSC Constable GD-06/12/2021 Shift-III Ans. (c) : For the above group of words, ‘ one who supervises in an examination hall’ option (c) invigilator is appropriate one word. Examinee - A person taking an examination or test. Teacher - A person whose job is to teach. Inspector - A person who looks into, observes and examines things. 301. Select the option that can be used as a oneword substitute for the given group of words. Not to pay attention to someone or something (a) Argue (b) Ignore (c) Attract (d) Rebel SSC Constable GD-26/11/2021 Shift-I One Word Substitution

Ans. (b) : For the above group of words, Not to pay attention to someone or something’ option (b) Ignore is appropriate one word. Argue - To present reasons for or againts a thing. Attract - To have a romantic interest in somebody. Rebel - To fight against authority, society, a law etc. 302. Select the option that can be used as a oneword substitute for the given group of words. Matter which is against moral values (a) Immortal (b) Mortal (c) Amorous (d) Immoral SSC Constable GD-14/12/2021 Shift-III Ans. (d) : For the above group of words, ‘Matter which is against moral values’, option (d) Immoral is appropriate one word. Immortal - Living or lasting forever. Mortal - Unable to continue living for ever. Amorous - Having strong feelings of love, especially romantic love. 303. Select the word which means the same as the group of words given. That which is incapable of being read or understood (a) Indelible (b) Illegible (c) Illegal (d) Intelligible SSC CHSL (Tier-I) –04/08/2021 (Shift-I) Ans. (b) : The above group of words, 'that which is incapable of being read or understand' is called ' Illegible'. The meaning of other options is Intelligible - capable of being apprehended or understand. Illegal - prohibited by law or by official or accepted rules. (unlawful). 304. Select the option that can be used as a oneword substitute for the given group of words. Certain to happen (a) Inevitable (b) Predicted (c) Necessary (d) Avoidable SSC Constable GD-18/11/2021 Shift-I Ans. (a) : For the above group of words, Certain to happen’ option (a) Inevitable is appropriate one word. Predicted - To state that something is true. Necessary - That is needed for a purpose or a reason. Avoidable - Possible to avoid. 305. Select the option that can be used as a oneword substitute for the given group of words. Having no boundaries or limits (a) Infinite (b) Colossal (c) Enormous (d) Tremendous SSC CGL (Tier-I) –17/08/2021 (Shift-I) Ans. (a) : The above group of words 'Having no bandaries or limits is called 'Infinite'. The meaning of other options is Colossal - big, large, prodigious. Enormous - tremendous (extraordinarily large in size or extent.) Tremendous - Great in amount, size or degree. 306. Select the option that can be used as a oneword substitute for the given group of words. Eskimo’s house that is made of ice blocks (a) Burrow (b) Cave (c) Den (d) Igloo SSC Constable GD-18/11/2021 Shift-I

613

YCT

311. Select the word which means the same as the group of words given. Impossible to satisfy (a) contented (b) satisfiable (c) satisfactory (d) insatiable SSC CGL (Tier-I) – 13/06/2019 (Shift-II) Ans. (d) : The given group of words, 'Insatiable to satisfy' is called 'Insatiable'. The meaning of other options is Contented - satisfied or showing satisfaction with things as they are. (self-satisfied) Satisfiable - capable of being sated (satiate) Satisfactory - giving satisfaction (acceptable). 312. Select the word which means the same as the group of words given. One who does not tire easily (a) indelible (b) indefatigable (c) inflatable (d) indestructible SSC CGL (Tier-I) – 12/06/2019 (Shift-III) Ans. (b) : The given group of words, 'One who does not tire easily' is called - 'Indefatigable'. The meaning of other options is Indelible - not able to be for gotten, removed. Inflatable - designed to be filled with air or gas. (expansive) Indestructible - not easily destroyed (undestroyable) 313. Select the word which means the same as the group of words given. Incapable of being read (a) vague (b) eligible (c) unseen (d) illegible SSC CGL (Tier-I) – 12/06/2019 (Shift-III) Ans. (d) : The given group of words, 'Incapable of being read' is called 'Illegible'. The meaning of other options is Vague - lacking clarity, Indistinct. Eligible - qualified for or allowed or worthy of being chosen. (worthy) Unseen - unobserved. 314. Select the word which means the same as the group of words given. That which cannot be satisfied. (a) incredible (b) insatiable SSC Sel. Post Phase-VIII (H.L.) 09.11.2020 (Shift-I) (c) improbable (d) impossible Ans. (a) : The above group of words, 'a quiet person SSC CGL (Tier-I) – 11/06/2019 (Shift-III) who prefers of spend time alone' is called 'Introveert'. Ans. (b) : The given group of words, 'that which The meaning of other options is cannot be satisfied' is called 'Insatiable'. Extrovert - a person concerned more with practical The meaning of other options is realities than with inner thoughts and feelings. Incredible - extraordinarily good or great used Narcissist - someone who is excessively self centered. especially as intensifiers. (wonderful) 310. Select the most appropriate one-word Impossible - hopeless, unworkable. substitution for the given group of words. Improbable - unbelievable unconvincing. Travelling under a name other than one's own 315. Select the word which means the same as the (a) incredible (b) invisible group of words given. (c) incognito (d) impromptu Something which cannot be understood. SSC CPO-SI – 25/11/2020 (Shift-II) (a) illegible (b) incomprehensible Ans. (c) : The given group of words, 'Travelling under (c) inexplicable (d) infallible a name other than one's own' is called 'Incognito'. SSC CGL (Tier-I) – 10/06/2019 (Shift-II) The meaning of other options is Ans : (b) The given group of words, 'something which Incredible - extraordinarily good great. (marvellous) or cannot be understood' is called 'Incomprehensible'. beyond belief (unbelievable) The meaning of other options is Invisible - Impossible to see. Illegible - unreadable, not legible. Infallible - incapable of failure or error. (fool proof). Impromptu - Without earlier planning or preared. Ans. (d) : For the above group of words, ‘Eskimo’s house that is made of ice blocks’ , option (d) Igloo is appropriate one word. Burrow - A hole or tunnel dug in the ground by a rabbit, fox or other small animal. Cave - A hollow in the earth, especially more or less horizontally into a hill mountain etc. Den - A secret place where wild animals live. 307. Select the option that can be used as a oneword substitute for the given group of words. Something that cannot be avoided (a) Inevitable (b) Impossible (c) Irreparable (d) Intolerable SSC Constable GD-02/12/2021 Shift-I Ans. (a) : For the above group of words ‘Something that cannot be avoided,’ , option (a) Inevitable is appropriate one word. Impossible - Unable to be done, performed, effected etc. Irreparable -Impossible to repair. Intolerable - Unpleasant or difficult to bear or accept. 308. In the following question, out of the four alternative, select the alternative which is the best substitute of the phrase. Lack of civic-mindness or of patriotism (a) Iconomachy (b) Incivism (c) Shag (d) Clergy SSC CGL (Phase-II) 17/02/2018 Ans. (b) The group of words, 'lack of civil mindness or of patriotism' is called 'Incivism'. The meaning of other options is Iconomacy - opposition to the worship of image or Icons. Shag - a matted tangle of hair or fiber Clergy - gn Christianity 309. Select the most appropriate one-word substitution for the given words. A quiet person who prefers to spend time alone (a) Introvert (b) Narcissist (c) Extrovert (d) Braggart

One Word Substitution

614

YCT

316.

Select one word substitution for the given words. A trade that is prohibited by law (a) incredible (b) illusive (c) inapt (d) illicit SSC CGL (Tier-I) – 07/06/2019 (Shift-III) Ans : (d) The given group of words, 'A trade that is prohibited by law' is called 'Illicit'. The meaning of other options is Incredible - Something is very difficult to believe. Illusive - Not real and based on illusion. Inapt - Not suitable for the situation. 317. Select the most appropriate word which means the same as the group of words given. That cannot be approached easily (a) inevitable (b) invincible (c) inaudible (d) inaccessible SSC CPO-SI – 09/12/2019 (Shift-II) Ans : (d) The given group of words, 'that can nto be approached easily' is called 'Inaccessbile'. The meaning of other options is Inevitable - Incapable of being avoided (necessary) Invincible - Incapable of being conquered, overcome or subdued. Inaudible - Unable to be heard. 318. Select the option which is the most appropriate one word substitution for the given group of words. A detailed plan for a journey especially a list of places to visit. (a) inventory (b) directory (c) trajectory (d) itinerary SSC CPO-SI – 11/12/2019 (Shift-II) Ans : (d) 'A detailed plan for a journey especially a list of places to visit' is called 'Itinerary'. The meaning of other options is Inventory - stock-taking, making an itemized list of merchandise or supplies an hand. Trajectory - the path followed by an object moving through space. (flight) Directory - an alphabetical list of names and addresses. 319. Select the option which is the most appropriate one word substitution for the given group of words. A decision which cannot be taken back (a) indelible (b) irrational (c) irrevocable (d) infallible SSC CPO-SI – 12/12/2019 (Shift-I) Ans : (c) The given group of words, 'A decision which cannot be taken back' is called 'irrevocable'. The meaning of other options is Indelible - not able to be forgotten, removed or erased. Irrational - not consistent with or using reason. (Illogical) Infallible - unerring, unfailing. 320. Select the most appropriate word for the group of words. Something that catches fire easily (a) intallible (b) inflammable (c) inflatable (d) incredible SSC CHSL (Tier-I) –05/07/2019 (Shift-I) One Word Substitution

Ans : (b) The given group of words, 'something that catches fire easily' is called - 'inflammable'. The meaning of other options is Inflatable - designed to be filled with air or gas. Incredible - beyond belief or understanding. 321. Select the most appropriate word for the group of words. That which cannot be heard (a) inaudible (b) invisible (c) incorrigible (d) incredible SSC CHSL (Tier-I) –04/07/2019 (Shift-II) Ans : (a) The given group of words, 'That which can not be heard' is called inaudible'. The meaning of other options is Invisible - unseeable hidden. Incorrigible - uncorrectable, unmanageable. 322. Select the word which means the same as the group of words given. One who does not tire easily (a) infallible (b) indefatigable (c) inevitable (d) indelibe SSC CHSL (Tier-I) –02/07/2019 (Shift-I) Ans. (b) : The given group of words, 'One who does not tire easily' is called 'Indefatigable'. The meaning of other options is Indelible - not able to be forgotten, removed. Inevitable - incapable of being avoided or prevented. 323. Select the word which means the same as the group of words given. Impossible or extremely difficult to understand (a) inaudible (b) incomprehensible (c) incompatible (d) inevitable SSC CHSL (Tier-I) –02/07/2019 (Shift-II) Ans : (b) The given group of words, 'Impossible or extremely difficult to understand' is called Incomprehensible'. The meaning of other options is Incompatible - not compatible Inaudible - impossible to near. 324. Select the word which means the same as the group of words given. A type of dome-shaped shelter built from blocks of solid snow (a) chalet (b) wigwam (c) hut (d) igloo SSC CHSL (Tier-I) –02/07/2019 (Shift-II) Ans : (d) The given group of words, 'A type of domeshaped shelter built from blacks of solid snow' is called 'Igloo'. The other options given different meaning. 325. Select the word which means the same as the group of words given. That which cannot be taken by force (a) Impregnable (b) inapprehensible (c) inaccessible (d) immutable SSC CHSL (Tier-I) –02/07/2019 (Shift-III) Ans : (a) The given group of words, 'That which cannot be taken by force' is called 'Impregnable'. The meaning of other options is Inapprehensible - not intelligible (beyond understand) Inaccessible - remote, inaccessible. Immutable - changeless.

615

YCT

326. Select the word which means the same as the group of words given. Something that cannot be avoided (a) indelible (b) infallible (c) inevitable (d) invincible SSC CHSL (Tier-I) –01/07/2019 (Shift-III) Ans : (c) The given group of words, 'something that cannot be avoided is called 'inevitable'. The meaning of other options is Indelible - not able to be forgotten. (ceneradicable) Infallible - incapable of failure or error. Invincible - incapable of being overcome or subdued. 327. Select the word which the same as the group of words given. Covered with a material or substance in order to stop heat, sound etc. From entering or escaping (a) Hydrated (b) Insulated (c) Assimilated (d) Calibrated SSC CHSL (Tier-I) 26/10/2020 (Shift-II) Ans. (b) : The given group of words, 'covered with a material or substance in order to stop heat, sound etc from entering or esc aping, is called 'Insulated'. The meaning of other options is Hydrated - containing combined water (hydrous) Assimilated - absorb, take in Calibrated - marked with or divided into. 328. Select the word which means the same as the group of words given. Something that is expressed in a confusing way (a) Eccentric (b) Inaudible (c) Gullible (d) Incoherent SSC CHSL (Tier-I) 26/10/2020 (Shift-II) Ans. (d) : The given group of words, 'Something that is expressed in a confusing way' is called 'Incoherent'. The meaning of other options is Eccentric - bizarre, freakish. Inaudible - impossible to hear Gullible - easily tricked because of being too trusting (Naive) 329. Select the word which means the same as the group of words given. Incapable of making mistakes (a) Ineligible (b) Negligible (c) Inevitable (d) Infallible SSC CHSL (Tier-I) 16/10/2020 (Shift-III) Ans. (d) : The given group of words 'Incapable of making mistakes' is called 'Infallible'. Ineligible - disqualified. Negligible - paltry, trifling, worthless. Inevitable - necessary (an unavoidable event) 330. Select the word which means the same as the group of words given. One whom it is impossible to defeat (a) Dissatisfied (b) Cowardly (c) Invincible (d) Inimitable SSC CHSL (Tier-I) 16/10/2020 (Shift-I) Ans. (c) : The given group of words, 'One whom it is impossible to defeat' is called - 'Invincible'. The meaning of other options is Dissatisfied - in a state of sulky dissatisfaction. (discontent) Cowardly - lacking courage. Inimitable- matchless, irreproducible. One Word Substitution

331. Select the word that means the same as the given group of words. A person weak and disabled by illness (a) Invalid (b) Petitioner (c) Activist (d) Athlete SSC CHSL (Tier-I) 21/10/2020 (Shift-II) Ans. (a) : The given group of words, 'A person weak and disabled by illness' is called 'Invalid'. The meaning of other options is Petitioner - requester, suppliant. Activist - a reformer who works to achieve social or political change. Athlete - a person trained to compete in sports. 332. Select the word which means the same as the group of words given. Combine to form a whole (a) Scatter (b) Create (c) Integrate (d) Disjoin SSC CHSL (Tier-I) 12/10/2020 (Shift-III) Ans. (c) : The given group of words, 'combine to form a whole' is called 'Integrate'. The meaning of other options is seatter - break up disperse. create - bring into existence (make) Disjoin - become separated 333. Select the most appropriate word of the given group of words. One who does not express himself freely (a) Extrovert (b) Introvert (c) Narcissist (d) Lunatic SSC MTS – 22/08/2019 (Shift-III) Ans. (b) : The given group of words, 'One who does not express himself freely' is called 'Introvert'. Extrovert - being concerned with the social and physical environment. Narcissist - someone who is excessively self centered. Lunatic -madman 334. Select the word which means the same as the group of words given. Resistant to a particular infection (a) Innocent (b) Immune (c) Immortal (d) Innocuous SSC MTS – 08/08/2019 (Shift-III) Ans. (b) : The given group of words, 'Resistant to a particular infection' is called 'Immune'. The meaning of other options is Inocent - free from sin (virtuous) Immortal - deathless, infinite. Innocuous - harmless, innoxicus. 335. Choose the most appropriate word that can substitute the given group of words. One who is unable to read or write. (a) Illiterate (b) Incomprehensible (c) Frugal (d) Illegible SSC MTS – 07/08/2019 (Shift-I) Ans. (a) : The given group of words, 'One who is unable to read or write' is called 'Illiterate'. The meaning of other options is Incomprehensible - difficult to understand. Frugal - economical, thrifty. Illegible - not legible' unreadable.

616

YCT

336. Select the word which means the same as the group of words given. One who works very hard (a) Labourer (b) Industrious (c) Exploited (d) Exhausted SSC MTS – 09/08/2019 (Shift-III) Ans. (b) : The given group of words, 'One who works very hard is called 'Industrious'. The meaning of other options is Labourer - someone who works with their hands. Exploited - developed or used to greatest advantage. Exhausted - depleted of energy, force. 337. Select the word which means the same as the group of words given. A place in a large institution for the care of those who are ill. (a) camp (b) shelter (c) infirmary (d) clinic SSC GD – 02/03/2019 (Shift-I) Ans. (c) : The given group of words, 'A place in large institution for the care of those who was ill' is called 'Infirmary'. The meaning of other options is Camp - cantonment, bivouac. Shelter - temporary housing for homeless or displace persons. Clinic - a medical establishment run by a group of medical specialists.

J 338. Select the option that can be used as a oneword substitute for the underlined segment. The music filled the room with a lot of happiness, expressing joyful melodies that brought smiles to everyone's faces. (a) fear (b) excitement (c) joy (d) surprise SSC CHSL (Tier-I) 03/08/2023 (Shift-II) Ans. (c) : The correct one – word substitute for the above underlined segment 'a lot of happiness' is option (c) 'Joy' Meaning of other options – Fear– 'to be frightened of something of someone unpleasant.' Excitement– The state of being excited. Surprise – An unexpected event. 339. Select the option that can be used as a oneword substitute for the given group of words. Merry, convivial, hearty and good humoured (a) Jowl (b) Jovial (c) Jubilee (d) Joyous SSC CHSL 30.05.2022 Shift-I Ans. (b) : For the above group of words ‘ Merry, convivial, hearty and good humored’ , option (b) Jovial is appropriate one word. Other options have different meaning. Jowl - The lower part of a person's or animals check especially when it is fleshy or dropping. Jubilee - The celebration of the anniversary of a special event. Joyous – A very happy or full of joy. One Word Substitution

340. Select the most appropriate one-word substitution for the given group of words. To cross streets on foot carelessly (a) stroll (b) wander (c) jaywalk (d) catwalk SSC CPO-SI – 25/11/2020 (Shift-II) Ans. (c) : The given group of word; 'To cross streets on foot carelessly' is called 'jaywalk' The meaning of other options is Stroll:- A leisurely walk (usually in some public place) Wander:- Meander, weave, (a short walk in) Catwalk:- The long stage that models walk on during a fashion show. 341. Select the word which means the some as the group of words given. The part of a country's government responsible for its legal system (a) judiciary (b) legislature (c) police (d) executive SSC CGL (Tier-I) – 13/06/2019 (Shift-III) Ans : (a) The given group of words, 'the part of a country's government responsible for its legal system' is called- 'judiciary' The meaning of other options isLegislature:- A group of people who have the power to make and change laws, (Exec dive) Police:- The force of policemen and officers. Executive:- Administrator. 342. Select the word which means the same as the group of words given. The science or philosophy of law (a) justice (b) archive (c) infirmary (d) jurisprudence SSC CPO-SI – 13/12/2019 (Shift-II) Ans. (d) : The given group of words, 'The science or philosophy of law' is called- 'Jurisprudence' The meaning of other options isJustice:- The quality of being fair or reasonable. Archive:- A depository containing historical records and documents. Infirmary:- A health facility where patients receive treatment. 343. Select the most appropriate word for the given group of words. A person who rides in horse races, especially as a profession (a) Jockey (b) Chauffer (c) Champion (d) Cowboy SSC MTS – 16/08/2019 (Shift-III) Ans. (a) : The given group of word, 'a person who rides in bores races. Especially as a profession' is called - 'Jockey' The meaning of other options isChauffer :- A portable stove usually with a great at the bottom. Champion:- Someone who has won the first place in a competition. Cowboy:- Cattleman, cowhand.

617

YCT

Ans. (a) : The above group of words, 'Praise given for achievement' is called- 'kudos' 344. Select the most appropriate one-word The meaning of other options isAcute:- Ending in a sharp point substitution for the given group of words. Anxiety:A vague unpleasant emotion that is Irresistible impulse to steal experienced in anticipation of some misfortune. (a) plunder (b) megalomania (anxiousness) (c) kleptomania (d) sabotage SSC CPO-SI – 25/11/2020 (Shift-II) Ans. (c) : The give group of words, 'Irresistible impulse to steal' is called- kleptomania' 349. Choose the word that can substitute the given The meaning of other options isgroup of words. Plunder:- Steal gods, loot The editor for dictionary Megalomania:- A psychological state characterized by (a) Stenographer (b) Hagiographer delusions of grandeur. (c) Lexicographer (d) Scenographer Sabotage:- Destroy property or hinder normal SSC MTS 08/05/2023 (Shift-III) operations. 345. Select the word which means the same as the Ans. (c) : For the above given group of words 'The editor for a dictionary' is known as 'Lexicographer' group of words given. A natural skill at doing something Meaning of the other words are – (a) Snack (b) Stack Hagiographer means – A writer of the live of the saints. (c) Knack (d) Quack Stenographer means – A professional who transcriber SSC CHSL (Tier-I) 13/10/2020 (Shift-I) the spoken word into coded language. Ans. (c) : The given group of words, 'A natural skill at Scenographer means – the art of perspective doing something' is called- 'knack' representation especially as applied to the design and The meaning of other options ispainting of stage scenery. Snack:- A light in formal meal, 350. Choose the word that can substitute the given Stack:- A pile of something group of words. A unit to measure light Quack:- Unqualified. (a) Watt (b) Lumen 346. Select the most appropriate one word (c) Horse-power (d) Ampere substitution for the given group of words. SSC MTS 18/05/2023 (Shift-I) The sounds of funeral bell is called Ans. (b) : The meaning of the group of words – ‘A unit (a) acute (b) acoustics to measure light’ is ‘Lumen’. (c) knell (d) mechanics SSC CHSL (Tier-I) –01/07/2019 (Shift-III) Meaning of other options – Ans. (c) : The above group of words, 'the sounds of Watt – A unit of electrical power, Ampere – The SI base unit of electrical current, funeral bell is called- 'knell' The meaning of other options isHorse-power – A measurement of the power of an Acute:- Extremely sharp engine. Acoustics:- Relating to music or musical instrument 351. Select the option that can be used as a onewhere sound is not electronic all enhanced. word substitute for the given group of words. Mechanics:- The branch of physics concerned with the One who can read and write motion of bodies in a frame of inference. (a) Illiterate (b) Eligible 347. Select the most appropriate one word (c) Illegible (d) Literate substitution for the given group of words. SSC CGL (Tier-I) 18/04/2022 Shift-I A small open fronted shop where things are Ans. (d) : For the above group of words, ‘one who can soldread and write, option (d) ‘Literate’ is appropriate one (a) Kiosk (b) Precis word. (c) Godown (d) Museum Illiterate - Unable to read or write. SSC GD – 02/03/2019 (Shift-I) Eligible - Fit or proper to be chosen; worthy of choice. Ans. (a) : The above group of words, ' A small open Illegible - Difficult to read. fronted shop where things are sold' is called - 'Kiosk' 352. Select the option that can be used as a oneThe meaning of other options isword substitute for the given group of words. Précis :- Abstract, synopsis. That which is lawful Godown:- A warehouse. (a) Valid (b) Illegal Museum:- A building in which objects of artistic, (c) Proper (d) Legal cultural, historical or scientific interest are Kept. SSC Constable GD-16/11/2021 Shift-III 348. Select the most appropriate one word Ans. (d) : For the above group of words, ‘That which substitution for the given group of words. is lawful’, option (d) legal is appropriate one word. Praise given for achievement Valid - Having some foundation, based on truth. (a) Kudos (b) Acute Illegal - Not allowed by law. (c) Anxiety (d) Rare SSC MTS – 22/08/2019 (Shift-III) Proper - Adapted

K

L

One Word Substitution

618

YCT

353.

In the following question, out of the four alternative, select the alternative which is the best substitute of the phrase. Eating mud (a) Equivorous (b) Limivorous (c) Calcivorous (d) Fructivorous SSC CGL (Phase-II) 17/02/2018 Ans. (b) The above group of words, 'Eating mud' is called- 'limivorous' The meaning of other options isEquivorous:- Feeding or subsisting on horse-flesh. Calciveorous:- Lixing on and eroding lime stone. Fructivorous:- Fruit eating or feeding on fruit. 354. Select the most appropriate word which means the same as the group of words given. Two lengths of rope, bamboo or wood with rungs used for climbing up and down walls, sides of ships etc. (a) bridge (b) steps (c) staircase (d) ladder SSC CPO-SI – 11/12/2019 (Shift-I) Ans : (d) The given group of words, 'two lengths of rope, bamboo or wood with rungs used for climbing up and down walls, sides of ships etc, is called - 'ladder' The meaning of other options:Bridge:- The link between two lenses; rests on the nose Step:- Footstep, pace. Staircase:- A way of access consisting of a set of steps. 355. In the following question, out of the four given alternatives, select the alternative which is the best substitute of the phrase. A Place where clothes are washed and pressed. (a) Foundry (b) Sundry (c) Husbandry (d) Laundry SSC MTS 9-10-2017 (Shift-II) Ans. (d) : The given group of words, 'A place where clothes are washed and pressed' is called- 'laundry' The meaning of other options isFoundry:- Factory where metal castings are produced. Sundry:- miscellaneous, mixed. Husbandry:- agriculture, farming 356. Select the most appropriate word for the given group of words. A gentle song to put a child to sleep. (a) Lullaby (b) Lyre (c) Lyric (d) Laurel SSC MTS – 20/08/2019 (Shift-II) Ans. (a) : The given group of words, 'A gentle song to but a child to sleep' is calledLullaby' The meaning of other options isLyre:- A harp used by ancient Greeks for accompaniment Lyric:- Expressing deep emotion. Laurel:- any of various aromatic tress of the laurel family. 357. Select the word which means the same as the group of words given. One who speaks many languages. (a) Multilingual (b) Monolingual (c) Bilingual (d) Linguist SSC GD – 18/02/2019 (Shift-III) One Word Substitution

Ans. (d) : The given group of words, 'one who speaks many languages is called- 'linguist' The meaning of other options isMonolingual:- Using or knowing only one language. bilingual:- using or knowing two languages.

M 358. Select the option that can be used as a oneword substitute for the given group of words. Any female animal which feeds its young on milk from her own body (a) Vertebrate (b) Fauna (c) Parasite (d) Mammal SSC CGL (Tier-I) 17/07/2023 (Shift-II) Ans. (d) : For the above group of word, 'Any female animal which feeds its young on milk from her own body' option (d) 'Mammal' is appropriate meaning. other options give different meaning. vertebrate – Related to spinal Fauna – Related to Animal for any special region Parasite – Drone 359. Select the option that can be used as a oneword substitute for the given group of words. A book or paper written by hand (a) Edition (b) Periodical (c) Manuscript (d) Paperback SSC MTS-05/07/2022 Shift-III Ans. (c) : For the above group of words, ‘ A book or paper written by hand’, option (C) manuscript is appropriate one word. Edition - A particular version of a book, magazine or news paper. Periodical - A publication that is released at regular intervals, usually weekly or monthly. Paperback - A book with a paper ever, rather than a soft cover. 360. Select the option that can be used as a oneword substitute for the given group of words. One who dies for a noble cause (a) Martyr (b) Heretic (c) Butcher (d) Narcissist SSC MTS-05/07/2022 Shift-III Ans. (a) : For the above group of words, ‘One who dies for a noble cause’, option (a) martyr is appropriate one word. Heretic - A person whose religious beliefs are belived to be wrong or evil. Butcher - A person who sells meat. Narcissist - A person who extremely self centered with an exaggerate sense of self importance. 361. Select the option that can be used as a oneword substitute for the given group of words. One who hates mankind (a) Altruist (b) Compere (c) Malevolent (d) Misanthrope SSC MTS-06/07/2022 Shift-II

619

YCT

Ans. (d) : For the above group of words ‘one who hates mankind’, option (d) misanthrope is appropriate one word. Altruist - A person who is unselfishly convened for the welfare of others. Malevolent - Wishing evil or harm to others. 363. Select the option that can be used as a oneword substitute for the given group of words. Place where objects of importance are exhibited (a) Museum (b) Treasury (c) Fair (d) Exhibition SSC CGL (Tier-I) 20/04/2022 Shift-III Ans. (a) : For the above group of words ‘Place where objects of importance are exhibited’, option (a) museum is appropriate one word. 363. Select the option that can be used as a oneword substitute for the given group of words. Lacking interest or excitement (a) Hostile (b) Mundane (c) Miserly (d) Despot SSC CGL (Tier-I) 20/04/2022 Shift-III Ans. (b) : For the above group of words ‘Lacking interest or excitement’, option (b) mundane is appropriate one word. Other options have different meaning. Hostile – Having very strong feeling against somebody /something. Miserly– A person who loves to have a lot of money but hates to spend it. Despot – A ruler or other person who has a lot of power and who uses it unfairly. 364. Select the word which means the same as the group of words given. person who move from one place to another, with intentions of finding work and settling, permanently or temporarily, at a new location. (a) Nomad (b) Migrant (c) Tramp (d) Vagrant SSC CGL (Tier-II) 08.08.2022 Shift-II Ans. (b) : For the above group of words, option (b) migrant is appropriate one word. Other options have different meaning. Nomad – A member of a people that has no fixed home but wanders from place to place. Tramp – Maverick Note - Nomad, Tramp and Vagrant are synonyms 365. Select the option that can be used as a oneword substitute for the given group of words. A place where antiques are kept (a) Resort (b) Orphanage (c) Museum (d) Hangar SSC Constable GD-01/12/2021 Shift-I Ans. (c) : For the above, group of words, ‘ A place where antiques are kept’ , option (c) Museum is appropriate one word. One Word Substitution

Resort - A place designed to provide recreation, entertainment and accommodation especially to vacationers. orphanage - A home for children whose percents are dead. Hanger - A building where planes are kept. 366. Select the option that can be used as a oneword substitute for the given group of words. Imitate someone's action or words in order to entertain or ridicule (a) Repeat (b) Duplicate (c) Copy (d) Mimic SSC Constable GD-17/11/2021 Shift-II Ans. (d) : For the above group of words, ‘Imitate someone’s action or words in order to entertain or ridicule’ option (d) Mimic is appropriate one word. Repeat - To express or present (oneself) again in the same words, terms or form. Duplicate - To make an exact copy of something. Copy - Something that is made to took exactly live something else. 367. Select the option that can be used as a oneword substitute for the given group of words. Woman who is the head of the family (a) Patriarch (b) Monogamy (c) Polygamy (d) Matriarch SSC Constable GD-02/12/2021 Shift-I Ans. (d) : For the above group of words ‘ Woman who is the head of the family’, option (d) matriarch is appropriate one word. Patriarch - A man who is the head of a family or social group. Monogamy - The fact or custom of being married to only one person at a particular time. Polygamy - The custom of having more than one wife at the same time. 368. Select the word which means the same as the group of words given. Killing of one's mother (a) infanticide (b) patricide (c) regicide (d) matricide SSC CHSL (Tier-I) –06/08/2021 (Shift-I) Ans. (d) : The above group of words, 'killing of one's mother' is called- 'Matricide' The meaning of other options isRegicide:- The act of killing a king Infanticide:- murdering an in font. Patricide:– the murder of your father. 369. Select the option that can be used as a oneword substitute for the given group of words. A place where money is coined (a) Mint (b) Factory (c) Industry (d) Store SSC Constable GD-24/11/2021 Shift-II Ans. (a) : For the above group of words, ‘ A place where money is coined’, option (a) Mint is appropriate one word. Factory - A building or group of buildings where goods are made in large quantities by machine. Industry - A specific branch of economic activity, typically in factories with machinery. Store - A large shop

620

YCT

370. Select the option that can be used as a oneword substitute for the given group of words. Belonging to the middle ages (a) Ancient (b) Primitive (c) Medieval (d) Modern SSC Constable GD-22/11/2021 Shift-II Ans. (c) : For the above group of words, 'Belonging to the middle ages', option (c) medieval is appropriate one word. Ancient - Belonging to a period of history that is thousands of years in the past. Primitive - Connect with a very early stage in the development of humans or animals. Modern - Present or recent times. 371. Select the option that can be used as a oneword substitute for the given group of words. The state of being subject to death (a) Morbidity (b) Immortality (c) Mortality (d) Deadly SSC CGL (Tier-I) –23/08/2021 (Shift-I) Ans. (c) : The above group of words, 'the state of being subject to death' is called- 'Mortality' The meaning of other options isMorbidity:- the quality of being unbeautiful and generally bad for you Immortality:- the quality or state of being immortal. Deadly:- mortal, deathly, fetal. 372. Select the option that can be used as a oneword substitute for the given of words. A musical composition made up of a series of songs or short pieces. (a) Orchestra (b) Lyric (c) Medley (d) Band SSC CGL (Tier-I) –23/08/2021 (Shift-I) Ans. (c) : The above group of words, 'A musical composition made up of a geris of songs or short pieces' is called - 'medley' The meaning of other options isOrchestra:- A musical organization consisting of a group of instrumentalists including string placer. Lyric:- Expressing deep emotions. Band:- an unofficial association of people or group. 373. Select the most appropriate one-word substitution for the given words. A very large impressive residence (a) Igloo (b) Mansion (c) Cabin (d) Cottage SSC CGL (Tier-II) 16/11/2020 (3 pm - 5 pm) Ans. (b) : The above group of words, 'Avery large impressive residence' is called- 'mansion' The meaning of offer options isIgloo:- A small round haves or shelter beret from blocks of hard snow. Cottage:- A small house with a single story. 374. Select the most appropriate one-word substitution for the given words. The state of being married to one person at a time (a) Bigamy (b) Homogamy (c) Polygamy (d) Monogamy SSC CGL (Tier-II) 16/11/2020 (3 pm - 5 pm) One Word Substitution

Ans. (d) : The above group of words, 'The state of being married to one person at a time' is called 'Monogamy.' The meaning of other options isBigamy:- Honing two spouses at the same time. Polygamy:- Having more than one spouse at a time. Homogamy:-Marriage between people from similar sociological or educational backgrounds. 375. Select the word which means the same as the group of words given. Sound of horses (a) grunt (b) screech (c) squeak (d) neigh SSC CPO-SI (Tier-II) 27/09/2019 (3 pm-5 pm) Ans. (d) : The above group of words, 'sound of horses' is called- 'neigh'. The meaning of other options isGrant:- Sound of pigs. Squeak:- Make a shout whirls Cory or noise Screech:- Sound of bats. 376. Select the word which means the same as the group of words given. One who embraces voluntary death for the sake of one’s country (a) martyr (b) diplomat (c) fanatic (d) patriot SSC CGL (Tier-I) – 10/06/2019 (Shift-I) Ans : (a) The given group of words 'One who embraces voluntary death for the sake of one' is called 'martyr.' The meaning of other options isDiplomat:An official engaged in international negotiations. Fanatic:- A person motivated by irrational enthusiasm. Patriot :- One who loves and defends his or her Country. 377. Select the most appropriate word which means the same as the group of words given. A government run by a king or a queen (a) oligarchy (b) aristocracy (c) monarchy (d) plutocracy SSC CPO-SI – 09/12/2019 (Shift-II) Ans : (c) The given group of words, 'A government run by a king or a queen' is called- 'Monarchy'. The meaning of other options isOligarchy:- A political system governed by a few people. Aristocracy:- A privileged class holding hereditary titles. Plutocracy:- A Political system governed by the wealthy people. 378. Select the most appropriate word for the given group of words. a short statement of a general truth or rule of conduct (a) instance (b) illustration (c) maxim (d) parable SSC CHSL (Tier-I) –09/07/2019 (Shift-I) Ans : (c) The given group of words, 'A short statement of general truth or rule of conduct is called 'maxim' The meaning of other options is-

621

YCT

Parable:- A short moral story (often with animal barraters) Illustration:- Artwork that helps make something clear or attractive. Instance:- An item of information that is typical of a closes or group. 379. Select the word which means the same as the group of words given. A large impressive house (a) mansion (b) shack (c) cabin (d) fort SSC CHSL (Tier-I) –02/07/2019 (Shift-III) Ans : (a) The given group of words, ' A lave impressive house' is called- 'mansion' The meaning of other options isShack:- Small crude shelter used as a dwelling. (but) Cabin:- A small house built of wood. Fort:- A fortified defensive structure. 380. Select the word which means the same as the group of words given. Government by a king or a queen (a) Oligarchy (b) Monarchy (c) Plutocracy (d) Aristocray SSC CHSL (Tier-I) 17/03/2020 (Shift-II) Ans. (b) : The given group of words, 'Government by a king or a queen' is called 'Monarchy' The meaning of other options is Oligarchy:- A poetical system governed by a few people Plutocracy:- A political system governed by the wealthy people. Aristocracy:- Nobility. 381. Select the word which means the same as the group of words given. Very careful and precise (a) Tremulous (b) Tumultuous (c) Meticulous (d) Garrulous SSC CHSL (Tier-I) 13/10/2020 (Shift-I) Ans. (c) : The given group of words 'very careful and precise' is called 'Meticulous'. The meaning of other options is Tremulous:- Quivering as from weakness or fear. Tumultuous:- Riotous, troubled, turbulent. Garrulous:- Loquacious, talkative. 382. Select the word which means the same as the group of words given. An unforseen happening which creates some damage (a) Magical (b) Mishap (c) Menace (d) Mystery SSC CHSL (Tier-I) 15/10/2020 (Shift-III) Ans. (b) : The given group of words, 'An unforeseen happening which creates some damage' is called mishap'. The meaning of other options isMagical:- Possessing or using or characteristic of or appropriate to supernatural powers. (magic, wizard) Menace:- Something that is a source of danger. Mastery:- Enigma, secret. One Word Substitution

383. Select the word which means the same as the group of words given. A period of one thousand years (a) Decade (b) Millennium (c) Centenary (d) Century SSC CHSL (Tier-I) 19/10/2020 (Shift-II) Ans. (b) : The given group of words, 'A period of one thousand years' is called- 'Millennium'. The meaning of other options is Decade:- A period of 10 years. Century:- A period of 100 years. Centenary:- The 100th anniversary 384. Select the word which means the same as the group of words given. Movement of people or animals from one area to another (a) Migration (b) Diversification (c) Acclimatisation (d) Population SSC CHSL (Tier-I) 14/10/2020 (Shift-II) Ans. (a) : The given group of words, 'movement of people or animals from once area to another' is called'migration' The meaning of other options isDiversification:- The condition of being varied. Acclimatisation:- Adaptation to a new climate. Population:- A group of organisms of the same species inhabiting a given area. 385. Select the word which means the same as the group of words given. A place where wild animals live (a) Lair (b) Hutch (c) Kennel (d) Cage SSC CHSL (Tier-I) 17/03/2020 (Shift-III) Ans. (a) : The given group of words, 'A place where wild Animal live' is called- 'Lair'. The meaning of other options isHutch:- A cage for small animals, (hovel, hut) Kennel:- A small shelter for a dog to pleep in. Cage:- Something that confine in a cage. 386. In the following question, out of the given alternatives, the alternative is the best substitute of the phrase. A supreme ruler (a) Patriarch (b) Anarchic (c) Monarch (d) Matriarch SSC MTS 9-10-2017 (Shift-III) Ans : (c) The given of words, 'A supreme ruler' is called- 'Monarch' The meaning of other options is Patriarch:- The male head of family or tribe. Anarchic:- Having no government, order or control. Matriarch:- A women who is the head of a family or social group. 387. In the following question, out of the four given alternatives, select the alternative which is the best substitute of the phrase. A reminder of something. (a) Information (b) letter (c) Brochure (d) Memento SSC MTS 10-10-2017 (Shift-II)

622

YCT

Ans. (d) : The given of words, 'A reminder of something ' is called 'Memento'. The meaning of other options isInformation:- Knowledge acquired through study or experience or instruction. letter:- A written message addressed to a person or organization. Brochure:- A small book casually having a paper cover. 388. In the following question, out of the four given alternatives, select the alternatives which is the best substitute of the phrase. Something that serves as a reminder. (a) Moderato (b) Obbligato (c) Memento (d) Crypto SSC MTS 11-10-2017 (Shift-III) Ans. (c) : The given group of words, 'something that hares as a reminder' is called 'memento' The meaning of other options is Moderato:- (used as an instruction) not very quickly or very slowly Obbligato:- A part of the score that must be performed without change or omission. Crypto:- Secret. 389. In the following question, out of the four given alternatives, select the alternative which is the best substitute of the phrase. A building which is constructed to keep alive one's memory. (a) Monument (b) Fort (c) Kingdom (d) Statue SSC MTS 11-10-2017 (Shift-III) Ans. (a) : The given group of words, 'A building which is constructed to keep alive one's memory' is called 'Monument' The meaning of other options isFort:- A fortified defensive structure. Kingdom:- The domain reeled by a king or queen. Statue:- A sculpture representing a human or animal. 390. Choose the word that can substitute the given sentence. An area of grassland where animals graze (a) Forest (b) Park (c) Meadow (d) Garden SSC MTS – 05/08/2019 (Shift-I) Ans. (c) : The given group of words, 'An area of grassland where animals ' is called - 'Meadow' The meaning of other optics isForest:- The trees and other plants in a large densely wooded area. Park:- A large area of land presented in its natural state as public property. Garden:- A plot of ground where plants are cultivated. 391. Select the most appropriate word for the given group of words. A large crowd of people intent on causing trouble or violence (a) Mob (b) Procession (c) Spectators (d) Audience SSC MTS – 19/08/2019 (Shift-I) Ans. (a) : The given group of words. 'A large crowd of people intent on causing trouble or violence' is calledmob. One Word Substitution

The meaning of other options isProcession:- The act of moving forward. (Advance) Spectator:- Someone who looks at something Audience:- A gathering of spectators or listener at performance. 392. Select the word which means the same as the group of words given. A person who loves money and hates spending it (a) Investor (b) Miser (c) Banker (d) Lender SSC MTS – 13/08/2019 (Shift-III) Ans. (b) : The given group of words, ' A person who loves money and hates spending it.' is called 'miser' The meaning of other options isInvestor:- Someone or a person or an organization that invests money in something. Bankers:- A person who owns a bank or has an important job at a bank. Lender:- Someone who lends money or gives credit in business matters.

N 393. Choose the word that can substitute the given group of words. Beginning to exist; recently formed or developed (a) Herald (b) Blasphemy (c) Knave (d) Nascent SSC MTS 04/05/2023 (Shift-II) Ans. (d) : The most appropriate group of words 'Beginning to exist, recently formed or developed' will 'Nascent' other words are ___ Herald- an official announce or messenger. Blasphemy- writting or speaking about God in a way that shows a lack of respect. Knave - a dishonest man 394. Select the most appropriate option that can be used as a one-word substitute for the underlined segment in the following sentence. As I walked through the old town, I couldn’t help but feel a sentimental longing for the simpler times of my childhood. (a) homesickness (b) nostalgia (c) affection (d) inertia SSC CHSL (Tier-I) 17/08/2023 (Shift-II) Ans. (b) : One word substitution for the underlined segment of the sentence 'a sentimental longing' will be option (b) Nostalgia – ( A wistful desire to return in thought or in fact to a former time in one's life.) Meaning of other option are – Homesickness – The feeling of being unhappy because of being away from home for a longer period Affection – A feeling of loving or liking somebody Inertia – An inability to move or change 395. Select the option that can be used as a oneword substitute for the given group of words. One who is new to some trade or profession (a) Novice (b) Martyr (c) Omnipresent (d) Philistine SSC CHSL (Tier-I) 10/08/2023 (Shift-I)

623

YCT

Ans. (a) : "Novice" (a person new to and inexperienced in a job) will be the one word substitution of given group of words 'one who in new to some trade or profession'. Meaning expressed by other words – martyr – a person who sacrifices something of great value and especially life for the sake of principle. Omnipresent – Present everywhere/universal Philistine – A person who refuses to as the beauty or the value of art or culture. 396. Choose the word that can substitute the given group of words. A specialist of kidney (a) Obstetrician (b) Nephrologist (c) Pathologist (d) Primatologist SSC CHSL (Tier-I) 09/02/2023 (Shift-II) Ans. (b) : For the above group of words "A specialist of kidney" option (b) Nephrologist is appropriate word. Other options give different meaning. Obstetrician – A physician specialzes in onbsetetsics Pathologist – An expert in the study of diseases. 397. Select the option that can be used as a oneword substitute for the underlined group of words : Geeta is doubtful about getting hired as she is inexperienced at this job. (a) Ace (b) Expert (c) Novice (d) Professional SSC CHSL 24.05.2022 Shift-III Ans. (c) : For the above underlined group of words’ inexperienced at this job’ option (C) Novice is appropriate one word. Other options have different meaning. Ace - a service that the person playing against you connot lit back. Expert - A person who has a lot of special knowledge. Professional - A person who works in a job that needs a high level of training. 398. Select the option which means the same as the group of words given. So small or unimportant as to be not worth considering (a) Feasible (b) Negligible (c) Invincible (d) Plausible SSC CHSL (Tier-I) –16/04/2021 (Shift-I) Ans. (b) : The above group of words 'so small Unimportant as to be not worth considering' is called 'negligible'. The meaning of other options isFeasible:- Executable, possible Invincible:- Unconquerable, unvanquishable. Plausible:- Apparently reasonable, valid, or truthful. 399. Select the option that can be used as a oneword substitute for the given group of words. One who studies the occult meanings of numbers and their supposed influence on human life (a) Numerologist (b) Astrologer (c) Prophet (d) Astronomer SSC CGL (Tier-I) –17/08/2021 (Shift-I) One Word Substitution

Ans. (a) : The above group of words, 'One who studies the occult meanings of numbers and their supposed in fluency on human life' is called 'numerologist' The meaning of other options isAstrologer:- Someone who predicts the future by the positions of the planets and sun and moon: Prophet:- Someone who is an interpreter of the will of god. Astronomer:- A physicist who studies astronomy. 400. Select the word which means the same as the group of words given. A person having the same name as the other. (a) sibling (b) surname (c) namesake (d) pseudonym SSC Sel. Post Phase-VII (M.L.) 15.10.2019 (Shift-I) Ans. (c) : The above group of words, a person having the same name as the other is called - 'namesake' The meaning of other options is Sibling:- A person's brother or sister. Pseudonym:- A fictitious name used when the person performs a particular social role. Surname:- Family name, last name. 401. Select the most appropriate one-word substitution for the given group of words. Downfall that satisfies natural justice (a) paradox (b) climax (c) nemesis (d) catharsis SSC CPO-SI – 25/11/2020 (Shift-II) Ans. (c) : The given group of words, 'Downfall that satisfies natural justice' is called- nemesis. The meaning of other options isParadox:- A statement that contradicts itself. Climax:- The most exciting or important event or point in time. (orgasm) Catharsis:- Purging the body by the use of a cathartic to stimulate evacuation of the bowels. (Purgation) 402. Select the most appropriate one-word substitution for the given group of words. A person with the same name as another (a) imposter (b) namesake (c) ancestor (d) alias SSC CPO-SI – 23/11/2020 (Shift-II) Ans. (b) : The given group of words, 'a person with the same name as another' is called - 'namesake' The meaning of other options isImposter:- A person who makes deceitful pretenses. (pretender) Ancestor:- A person in your family who lived a long time before you. Alias:- False name use to conceal one's identity. 403. Select the most appropriate one-word substitution for the given group of words. A person of evil reputation. (a) notorious (b) famous (c) icon (d) renowned SSC CPO-SI – 24/11/2020 (Shift-II) Ans. (a) : The given group of words, 'Adperson of evil reputation' is called - 'Notorious' the meaning of other options isFamous:- A person who well known to many people. Icon:- A small picture on a computer screen Renowned:- Famous for something

624

YCT

404. Select the word which means the same as the group of words given. A person, animal or plant belonging originally to a place (a) occupant (b) alien (c) native (d) resident SSC CGL (Tier-I) – 04/06/2019 (Shift-II) Ans : (c) The given group of words, 'A person, animal or plant belonging originally to a place' is called'native' The meaning of other options isOccupant:- A person who lives or works in a particular house room, building, etc. (resident) Alien:- A creature that comes from another planet. Resident:- Occupier (living in a particular place) 405. Select the most appropriate word for the given group of words. A person or thing that has the same name as another (a) pseudonym (b) namesake (c) relative (d) successor SSC CHSL (Tier-I) –10/07/2019 (Shift-II) Ans : (b) The given of words, 'A person or thing that has the same name as another' is called - 'namesake' The meaning of other persons is Pseudonym:- A fictitious name used when the person performs a particular social role. Relative - A member of your family. Successor - Someone that comes after another person 406. Select the word which means the same as the group of words given. Guide the course of a ship especially by using instruments or maps. (a) anchor (b) sail (c) travel (d) navigate SSC CHSL (Tier-I) –04/07/2019 (Shift-I) Ans : (d) The given group of words 'Guide the course of a ship especially by using instruments or maps' is called 'Navigate' The meaning of other options is Sail:- A trip in a boat or ship Travel:- Change location, move, travel. Anchor:- A mechanical device that prevents a vessel from moving. 407. Select the most appropriate word for the given group of words. A sentimental longing or wistful affection for a period in the past (a) Nostalgia (b) Paradise (c) Recollection (d) Harmony SSC MTS – 16/08/2019 (Shift-I) Ans. (a) : The given group of words, 'A sentimental longing or wistful affection for a period in the past' is called- 'Nostalgia' The meaning of other options isParadise:- Any place of complete bliss and delight and peace. Recollection:- Recall, reminiscence harmony:- Agreement of opinions. One Word Substitution

408. Choose the most appropriate word that can substitute the given group of words. Person with no fixed abode (a) Artist (b) Crusader (c) Nomad (d) Tourist SSC MTS – 07/08/2019 (Shift-II) Ans. (c) : The given group of words, 'person with no fixes abode' is called 'Nomad' The meaning of other options is Artist:- A person whose creative work shows sensitivity and imagination Crusader:- A disputant who advocates reform. (reformist) Tourist:- Someone who travels for pleasure.

O 409. Select the word from the options which is similar in meaning to the bracketed part given in the sentence below. I suppose I was looking for (a work that would give me income) and that was going to be an adventure. (a) Calling (b) Activity (c) Occupation (d) Pursuit SSC MTS 10/05/2023 (Shift-I) Ans. (c) : For the above given bracked part of the sentence similar meaning is 'occupation' which means Trade. The meaning of other words isCalling - a strong urge towards a particular way of life or career Activity - A situation in which there is a lot of action pursuit - the action of trying to achieve or get something 410. Choose the word that can substitute the given group of words. A person holding conversational belief especially in matter of religion. (a) Prodent (b) Orthodox (c) Compare (d) Arsonist SSC MTS 19/05/2023 (Shift-III) Ans. (b) : For the above group of words ‘A person holding conversational belief especially in matter of religion’ the word in ‘Orthodox’. Meaning of other optionsProdent – showing good judgment in avoiding risks and uncertainties. Compare – Difference between two or more things. Arsonist – A person who deliberately fire to a building. 411. Select the option that can be used as a oneword substitute for the given group of words. One who knows all (a) Topper (b) Bully (c) Anarchist (d) Omniscient SSC MTS-15/07/2022 Shift-I Ans. (d) : For the above group of words, one who knows all option (d) omniscient is appropriate one word. Topper- Someone or somthing excellent Bully - A person who use his strength power to hurt people who are weaker Anarchist - A person who rebels against any authority

625

YCT

412. Select the option that can be used as a oneword substitute for the underlined group of words. Human beings are usually ones who are naturally able to eat both, plants and flesh. (a) Herbivorous (b) Cannibals (c) Omnivorous (d) Carnivorous SSC CHSL 26.05.2022 Shift-I Ans. (c) : For the above underlined group of words ‘ naturally able to eat both, plants and flesh”, option (C) omnivorous is appropriate one word. Herbivorous - An animal eating plants. Cannibals - An animal that eats the flesh of another animal Carnivorous - An animal that eats meat. 413. Select the option that can be used as a oneword substitute for the given group of words. A person who tends to be hopeful and confident about the future or the success of something. (a) Fanatic (b) Pessimist (c) Optimist (d) Opportunist SSC Constable GD-08/12/2021 Shift-I Ans. (c) : For the above group of words, A person who tends to be hopeful and confident about the future or the success of something.’, option (c) optimist is appropriate one word. Fanatic - A person who is very enthusiastic about something. Pessinist - A person who always thinks that bad things will happen Opportunist - A person who see a chance to gain some advantage from a situation 414. Select the option that can be used as a oneword substitute for the given group of words. One who has unlimited power (a) Omnipresent (b) Omnipotent (c) Omnivore (d) Omniscient SSC Constable GD-22/11/2021 Shift-II Ans. (b) : For the above group of words, ‘one who has unlimited power’, option (b) omnipotent is appropriate one word. Omnipresent - One who present everywhere at the same time. omnivore - An animal that eats both plant and meat Omniscient - one who knowing everything. 415. Select the option that can be used as a oneword substitute for the given group of words. A piece of land where fruit trees are grown (a) Pasture (b) Barnyard (c) Meadow (d) Orchard SSC Constable GD-16/11/2021 Shift-III Ans. (d) : For the above group of words, ‘ A place of land where fruit trees are gown’ , option (d) orchard is appropriate one word. 416. Select the option that can be used as a oneword substitute for the given group of words. That through which light cannot pass (a) Opaque (b) Cloudy (c) Lucid (d) Gloomy SSC Constable GD-22/11/2021 Shift-II One Word Substitution

Ans. (a) : For the above group of words, ‘ That through which light cannot pass’, option (a) opaque is appropriate one word. Cloudy - Full of or overcaste by clouds Lucid - Something that is said or written clear and easy to understand. Gloomy - Dark in way that makes you feel sad. 417. Select the word which means the same as the group of words given. One who has the power to do anything (a) omnipresent (b) multifaceted (c) omnipotent (d) multilingual SSC CHSL (Tier-I) –09/08/2021 (Shift-I) Ans. (c) : The above group of words, 'one who has the power to do anything is called (Omnipotent) The meaning of other options is Omnipresent:- Being present everywhere at once. Multisided:- Having many aspects. Multilingual:- Knowing more than one language 418. Select the most appropriate one-word substitution for the given words. A word or law no longer in use (a) Absolute (b) Obsolete (c) Vintage (d) Venerable SSC CGL (Tier-II) 16/11/2020 (3 pm - 5 pm) Ans. (b) : The above group of words, 'A word or law no longer in use is called 'obsolete' The meaning of other options isAbsolute:- Perfect or complete or pure. Venerable:- Profoundly honored. Vintage:- Out of fashion. 419. In the following question, out of the four alternatives, select the alternative which is the best substitute of the phrase. Obsession for wine (a) Ethnomania (b) Oenomania (c) Egomania (d) Idolomania SSC CGL (Phase-II) 17/02/2018 Ans. (b) The above group of words, obsession for wine, is called 'Oenomania' The meaning of other options isEthromania:- Excessive desire about religion Egomania:- An intense and irresistible love for yourself (yourself) and concern for your own needs. Idol mania:- A devotion to idols. 420. In the following question, out of the four alternative, select the alternative which is the best substitute of the phrase. Study of snakes (a) Philology (b) Phrenology (c) Ophilogy (d) Apology SSC CGL (Phase-II) 17/02/2018 Ans. (c) The above group of words, 'study of snakes' is called 'ophilogy' The meaning of other options isPhilology:- The humanistic study of language. (linguistics) Phrenology:- the study of the shape of the human head, which some people think is a guide to a person's character.

626

YCT

421. Select the most appropriate word which means the same as the group of words given. Be more successful than someone (a) succeed (b) beat (c) win (d) outdo SSC CPO-SI – 23/11/2020 (Shift-I) Ans. (d) : The given group of words, 'be more successful than someone' is called 'outdo' The meaning of other options is Succeed:- Attain success or reach a desired goal. Beat:- Hit repeatedly. Win:- A victory. 422. Select the most appropriate word for the given group of words. Government run by a small group of people (a) aristocracy (b) monarchy (c) plutocracy (d) oligarchy SSC CPO-SI – 25/11/2020 (Shift-I) Ans. (d) : The given group of words 'Government run by a small group of people' is called 'Oligarchy' The meaning of other options isMonarchy:- An autocracy governed by a monarch who usually inherits the authority. Plutocracy:- A political system governed by the wealthy people. Aristocracy:- A privileged class holding hereditary titles. (Nobility) 423. Select the word which means the same as the group of words given. A place where fruit trees are grown (a) garden (b) plantation (c) orchard (d) farm SSC CGL (Tier-I) – 11/06/2019 (Shift-II) Ans. (c) : The given group of words 'a place where free it tree are grown' is called 'orchard' The meaning of other options isPlantation:- an area of land where trees are grown to produce wood Farm:- Cultivate by growing. Garden:- A plot of ground where plants are cultivated. 424. Select the option which is the most appropriate one word substitution for the given group of words. One who holds established opinions (a) missionary (b) orthodox (c) monotheist (d) atheist SSC CPO-SI – 12/12/2019 (Shift-I) Ans : (b) The given group of words, 'One who holds established opinions' is called 'Orthodox'. The meaning of other options is Missionary:- Someone sent on a mission especially a religious or charitable mission to a foreign country. Monotheist:- Believing that there is only one god. Atheist:- Someone who denies the existent of god. 425. Select the option which is the most appropriate one word substitution for the given group of words. That which cannot be seen through (a) transparent (b) opaque (c) impervious (d) translucent SSC CPO-SI – 13/12/2019 (Shift-I) One Word Substitution

Ans. (b) : The given group of words, 'That which cannot be seen through' is called 'opaque' The meaning of other opaque isTransparent:- Transmitting light, able to be seen through with clarity. Impervious:- Not admitting of passage or capable of being affected. Translucent:- Semitransparent. 426. Select the word which means the same as the group of words given. Something like an opening, particularly one in the body such as a nostril (a) obscure (b) oblique (c) orifice (d) obtuse SSC CHSL (Tier-I) 20/10/2020 (Shift-III) Ans. (c) : The given group of words, 'something like an opening particularly one in the body such as a nostril is caused 'orifice' The meaning of other options isObscure:- Not cleanly understood or expensed. Obliges:- Convergent. Obtuse:- Lacking in insight. 427. Select the word which means the same as the group of words given. A garden with fruit trees (a) Vineyard (b) Orchard (c) Grove (d) Nursery SSC CHSL (Tier-I) 19/10/2020 (Shift-III) Ans. (b) : The given group of words, 'a garden with fruit tree' is called 'orchard'. The meaning of other options isVineyard:- A form of grapevines where wine grapes are produced. Grove:- A small growth of trees without underbrush. Nursery:- A chides room for a body. 428. In the following question, out of the four given alternatives, select the alternative which is the best substitute of the phrase. One who see bright side of things. (a) Optimist (b) Superspecialist (c) Conventionalist (d) Educationalist SSC MTS 7-10-2017 (Shift-I) Ans : (a) The given group of words, 'One who see bright side of things' is called 'optimist' The meaning of other options isSuper specialist:- A highly skilled specialist. Conventionalist:- Regularity. Educationalist:- A specialist in the theory of education 429. Select the most appropriate word for the given group of words. Something no longer in use. (a) Obdurate (b) Odious (c) Obsolete (d) Ominous SSC MTS – 02/08/2019 (Shift-III) Ans. (c) : The given group of words, 'something no longer in use' is called 'absolute' the meaning of other options isObdurate:- Stubbornly persistent in wrongdoing. Odious:- Acorn enable, detestable, hateful. Ominous:- its, inauspicious.

627

YCT

430. Choose the most appropriate word that can substitute the given group of words. A polygon with eight sides and eight angles. (a) Triangle (b) Hexagon (c) Octagon (d) Pentagon SSC MTS – 07/08/2019 (Shift-II) Ans. (c) : The given group of words, 'A polygon with eight sides and eight angles' is called 'octagon'. The meaning of other options isTriangle:- A three sided polygon. Hexagon:- A six, sided polygon. Pentagon:- A five, sided polygon. 431. Select the word which means the same as the group of words given. One who is present everywhere (a) Omnipotent (b) Omniscient (c) Omnipresent (d) Omnivorous SSC GD – 14/02/2019 (Shift-II) Ans. (c) : The given group of words, 'one who is present everywhere' is called 'omnipresent'. The meaning of other options isOmnipotent:- Having unlimited power. (almighty) Omnivorous:- Feeding on both plants and animals. Omniscient:- Infinitely wise.

P 432. Choose the word that can substitute the given group of words. A living organism that steals nourishment from other living beings (a) Insect (b) Co - Dependent (c) Vector (d) Parasite SSC MTS 16/05/2023 (Shift-II) Ans. (d) : For the above given group of words, the one word is option (d) 'Parasite'. Meaning of other optionsInsect- A small animal that has six legs. Vector- A measurement or a quantity that has both size and direction. Co-Dependent- A circular relationship in which one person, who in turn needs to be needed. 433. Select the option that can be used as a oneword substitute for the given group of words. A formal agreement between two or more nations or peoples (a) Bond (b) Charter (c) Pact (d) Concord SSC CGL (Tier-I) 24/07/2023 (Shift-III) Ans. (c) : For the above group of word, 'A formal agreement between two or more nations or peoples' option (c) 'pact' is appropriate meaning. Other options give different meaning. Bond - a certificate that you can buy from a government that promises to pay you interest on the money you have given Charter - a written statement of the rights, beliefs and purpose of an organization Concord - the fact of having to have a particular from according to other words in the phrase. One Word Substitution

434. Select the option that can be used as a oneword substitute for the underlined group of words in the following sentence. Denmark has a type of government where power is held by the wealthiest members of society. (a) democracy (b) plutocracy (c) monarchy (d) oligarchy SSC CHSL (Tier-I) 04/08/2023 (Shift-III) Ans. (b) : The most appropriate one - word substitute for the above given underlined group of words 'a type of government ............society' is option (b) 'plutocracy'. Meaning of other options – Democracy – Government by the people. Monarchy –The system of government by a king or queen. Oligarchy – county governed by small groupof people. 435. Select the most appropriate ‘one word’ for the expression given below. A person who can speak multiple languages. (a) Minaret (b) Polyglot (c) Spire (d) Intestate SSC CHSL (Tier-I) 20/03/2023 (Shift-I) Ans. (b) : For the above group of words "A person who can speak multiple languages", option (b) 'Polyglot' is appropriate one word. Minartate - a tall thin tower Spire - A tall pointed tower on the top of a church Intestate - to die without leaving instructions about who should be given your property. 436. Select the option that can be used as a oneword substitute for the given group of words. A remedy for all diseases (a) Medicine (b) Panacea (c) Treatment (d) Diagnosis SSC Constable GD-24/11/2021 Shift-II Ans. (b) : For the above group of words, ‘A remedy for all diseases’ , option (b) Panacea is appropriate one word. Medicine - the science of preventing and treating illness. Treatment - the way that you behave towards somebody or deal with something Diagnosis - The act of saying exactly what illness a person has or what the cause of a problem 437. Select the option that can be used as a oneword substitute for the given group of words. A large number of things placed one on top of another (a) Group (b) Batch (c) Load (d) Pile SSC Constable GD-30/11/2021 Shift-III Ans. (d) : For the above group of words, ‘A large number of things placed one on top of another’, option (d) Pile is appropriate one word. Group - A number of people or thing Batch - A group of similar objects Loa - the quantity of something that can be carried.

628

YCT

438. Select the option that can be used as a oneword substitute for the given group of words. A person who loves his/her country and, if necessary, will fight for it (a) Patriot (b) Patriarch (c) Patron (d) Expatriate SSC Constable GD-29/11/2021 Shift-II Ans. (a) : For the above group of words, option (a) Patriot is appropriate one word. 439. Select the option that can be used as a oneword substitute for the given group of words. The love of man or mankind (a) Philology (b) Philanthropy (c) Philosophy (d) Phylactery SSC CHSL 27.05.2022 Shift-III Ans. (b) : For the above group of words,’ the love of man or mankind’ option (b) philanthropy is appropriate one word. Other options have different meaning. Philology – Study of the history of the language. Philosophy – the study of ideas and beliefs about the meaning of life. Phylactery – a small leather box containing hebrew texts on vellum 440. Select the option that can be used as a oneword substitute for the given group of words. One who collects stamps (a) Numismatist (b) Narcissist (c) Philatelist (d) Collector SSC MTS-06/07/2022 Shift-II Ans. (c) : For the above group of words ‘one who collects stamps’, option (C) philatelist is appropriate one word. Numismatist - A person who collects or studies coins Narcissist - A person who has an excessive interest in or admiration of themselves Collector - A person who collects things as a hobby. 441. Select the option that can be used as a oneword substitute for the given group of words. The state of being full or complete (a) Plebiscite (b) Plunder (c) Pliability (d) Plenitude SSC CHSL 27.05.2022 Shift-II Ans. (d) : For the above group of words, ‘The state of being full or complete, option (d) plenitude is appropriate one word. Other options have different meaning. Plebiscite - a vote by the people of a country or a region on question that is very important Plunder - the action of stealing from people or places Pliability - the quality of being easily bent. 442. Select the options that can be used as a oneword substitute for the given group of words. The science or art of political government (a) Politics (b) Sociology (c) Economics (d) Theology SSC MTS-05/07/2022 Shift-II One Word Substitution

Ans. (a) : For the above group of words, the science or art of political government’, option (a) politics is appropriate one word. Sociology - the study of human societies and social behaviour. Economics - the study of the way money, business Theology - the study of religion 443. Select the option that can be used as a oneword substitute for the given group of words. An imaginary name assumed by an author (a) Nomenclature (b) Almanac (c) Pseudonym (d) Platonic SSC MTS-14/07/2022 Shift-III Ans. (c) : For the above group of words An imaginary name assumed by an author , Option (C) pseudonym is appropriate one word. Nomenclature - a system of naming things Almanac - a book published every year that contains information about the movement of the planets. Platonic - a relationship that is purely spiritual and not physical. 444. Select the option that can be used as a oneword substitute for the underlined group of words. Ravi got the job easily because he speaks many languages. (a) Polynomial (b) Polyglot (c) Polycenric (d) Polysyllabic SSC CHSL 25.05.2022 Shift-I Ans. (b) : For the above group of words, ‘speaks many language’ option (b) 'polyglot' is appropriate one word. Polynomial – an expression which consist of single or multiple terms. Polycentric - Having more than one centre Polysyllabic - A word having more than one syllable. 445. Select the option that can be used as a oneword substitute for the given group of words. Showing great attention to detail and correct behaviour (a) Disciplined (b) Punctilious (c) Punctual (d) Zealous SSC CGL (Tier-I) 12/04/2022 Shift-II Ans. (b) : For the above group of words, option (b) Punctilious is appropriate one word. Disciplined - to train somebody to obey and to behave in controlled way Punctual - doing something or happening at the right time. Zealous - Feeling upset or angry because you thing that somebody you like or love is showing interest in somebody else. 446. Select the option that can be used as a oneword substitute for the given group of words. Eternal; lasting forever or indefinitely (a) Perplex (b) Permissive (c) Peristyle (d) Perpetual SSC CHSL 26.05.2022 Shift-III

629

YCT

Ans. (d) : For the above group of words ‘Eternal lasting forever or indefinitely ; option (d) perpetual is appropriate one word. Perplex – make someone feel completely baffled Permissive – having allowing a lot of freedom Peristyle – a space such as a court that is surrounded by a peristyle. 447. Select the option which means the same as the group of words given. A government runs by the wealthy people (a) oligarchy (b) monarchy (c) plutocracy (d) aristocracy SSC CHSL (Tier-I) –04/08/2021 (Shift-I) Ans. (c) : The above group of words, 'A government runs by the wealthy people' is called 'plutocracy'. The meaning of other options isOligarchy:- A political system governed by a few people. Monarchy:- An autocracy governed by a monarch who usually inherits the authority. Aristocracy:- Nobility. 448. Select the most appropriate one-word substitution for the given words. Safe to drink (a) Feasible (b) Delectable (c) Potable (d) Edible SSC CGL (Tier-II) 16/11/2020 (3 pm - 5 pm) Ans. (c) : The above group of words 'safe to drink' is called 'potable' The meaning of other options isFeasible:- Capable of being done with at hand and circumstances as they are. Delectable:- Extremely pleasing to the sense of taste. Edible:- Eatable. 449. Select the most appropriate one-word substitution for the given words. A group of lions (a) Pride (b) Pack (c) Colony (d) Herd SSC CGL (Tier-II) 16/11/2020 (3 pm - 5 pm) Ans. (a) : The above group of words, 'A group of lions' is called 'pride' The meaning of other options isPack:- Group of dogs. Colony:- Group of ants. Herd:- Group of cattle. 450. Select the most appropriate one-word substitution for the given words : One who does not care for art or literature (a) Philatelist (b) Plagiarist (c) Pessimist (d) Philistine SSC CGL (Tier-II) 16/11/2020 (3 pm - 5 pm) Ans. (d) : The above group of words, 'one who does not care for art or literature' is called 'philistine' The meaning of other options isPessimist:- A person who expects the worst. Philatelist:- Literary pirate. 451. Select the word which means the same as the group of words given. A solution for all difficulties or diseases : (a) medication (b) treatment (c) remedy (d) panacea SSC CPO-SI (Tier-II) 27/09/2019 (3 pm - 5 pm) One Word Substitution

Ans. (d) : The above group of words- 'A solution for all difficulties or diseases is called 'panacea' The meaning of other options isMedication:- Medicine Treatment:- Something that is done to cure an illness or injury. Remedy:- Cure, curative. 452. Select the most appropriate one-word substitution for the given group of words. To go around an area to keep a watch (a) patrol (b) cruise (c) campaign (d) explore SSC CPO-SI – 24/11/2020 (Shift-II) Ans. (a) : The given group of words, ' To go around an area to keep a watch' is called 'patrol' The meaning of other options isCruise:- An ocean trip taken for pleasure. Campaign:- Several related operations aimed at achieving a particular goal. Explore:- Inquire into, search 453. Select the word which means the same as the group of words given. A fictitious name used by an author (a) homonym (b) alibi (c) pesudonym (d) anonymous SSC CGL (Tier-I) – 11/06/2019 (Shift-I) Ans. (c) : The given group of words, 'a fictitious name used by an author' is called 'pseudonym' The meaning of other options is Homonym:- A word that is spelt like another word but which has a different meaning e.q- can:- be able can:put something in a container Alibi:- A defense of some offensive behavior or some failure to keep a promise etc. Anonymous:- Unknown, unnamed. 454. Select the word which means the same as the group of words given. One who is indifferent to art and culture (a) philanderer (b) cynic (c) philistine (d) scientist SSC CGL (Tier-I) – 10/06/2019 (Shift-I) Ans : (c) The given group words, 'one who is indifferent to art and culture' is called- 'philistine' The meaning of other options isPhilanderer:- A man who likes many women and short sexual relationship with them. Cynic:- Faultfinder. Scientist:- A person with advanced knowledge of one or more sciences. 455. Select the word which means the same as the group of words given. Sole right to make and sell some invention (a) copyright (b) inheritance (c) patent (d) heirloom SSC CGL (Tier-I) – 10/06/2019 (Shift-III) Ans. (c) : The given group words, 'sole right to make and sell some invention, is called 'patent' The meaning of other options isInheritance:- Hereditary succession to a title or an office or properly. Copyright:- Right of first publication Heirloom:- Something that has been in a family for generations.

630

YCT

456. Select the option that can be used as a oneword substitute for the given group of words. A person who believes that war and violence are unjustifiable (a) pessimist (b) anarchist (c) optimist (d) pacifist SSC CPO-SI – 09/12/2019 (Shift-I) Ans : (d) The given group of words, 'A person who believes that war and violence are unjustifiable' is called 'pacifist' The meaning of other options isPessimist:- A person who always expects bad things to happen. Anarchist:- A person who believes that laws and government are not necessary. Optimist:- A person who always expects good things to happen. 457. Select the option that can be used as a oneword substitute for the given group of words. A person who held a job or office before the current holder (a) successor (b) ancestor (c) descendant (d) predecessor SSC CPO-SI – 09/12/2019 (Shift-I) Ans : (d) The given group of words, 'A person who hold a job or office before the current holder' is called 'predecessor' The meaning of other options isSuccessor:- A person who follows next in order Ancestor:- A person considered as descended from some ancestor. 458. Select the option which is the most appropriate one word substitution for the given group of words. One who travels on foot (a) propagandist (b) paediatrician (c) pedestrian (d) protagonist SSC CPO-SI – 12/12/2019 (Shift-I) Ans : (c) The given group of words, 'one who travels on foot' is called ' pedestrian' The meaning of other options is Propagandist:- A person who disseminates message calculated to assist some cause. Pediatrician:- A specialist is the care of babies. protagonist:- The principal character in a work of fiction. 459. Select the option which is the most appropriate one word substitution for the given group of words. Careful in performing duties (a) Sincere (b) Punctual (c) Obedient (d) Punctilious SSC CPO-SI – 13/12/2019 (Shift-I) Ans. (d) : The given group of words, 'careful in performing duties' is called 'punctilious' The meaning of other options isSincere:- Honest, genuine Punctual:- Timely, prompt. Obedient:- compliant. One Word Substitution

460. Select the most appropriate word for the group of words. A previous case that might serve as an example or guide in subsequent situations (a) president (b) predecessor (c) precursor (d) precedent SSC CHSL (Tier-I) –05/07/2019 (Shift-III) Ans : (d) The given group of words, 'A previous case that might serve as an example or guide in subsequent situations' is called 'precedent' The meaning of other options isPresident:- The chief executive of a republic. Predecessor:- One who precedes you time. Precursor:- Forerunner. 461. Select the most appropriate word for the group of words. A remedy for all diseases (a) panacea (b) antibiotic (c) antidote (d) patent SSC CHSL (Tier-I) –04/07/2019 (Shift-II) Ans : (a) The given group of words, 'A remedy for all diseases' is called 'panacea' The meaning of other options isAntibiotic:- Containing or using a substance. Antidote:- A substance that controls the effects of a poison or disease. Patent:- A document granting an inventor sole rights to an invention. 462. Select the most appropriate word for the group of words. One whose hobby is stamp collection. (a) phonologist (b) philanthropist (c) philatelist (d) panegyrist SSC CHSL (Tier-I) –04/07/2019 (Shift-III) Ans : (c) The given group of words 'one whose hobby is stamp collection' is called philatelist. The meaning of other options is Philologist:- A specialist in phonology. Philanthropist:- Altruist. Panegyrist:- An orator who delivers eulogies or panegyrics. 463. Select the word which means the same as the group of words given. rapid increase in the number or amount of something (a) proliferation (b) promotion (c) propensity (d) paucity SSC CHSL (Tier-I) –03/07/2019 (Shift-II) Ans. (a) : The given group of word, 'rapid increase in the number or amount of something is called 'proliferation' The meaning of other options is promotion:- The act of raising in rank or position. Propensity:- A natural inclination. (Leaning, Proclivity) Paucity:- Dearth. 464. Select the word which means the same as the given group of words. Someone who freely gives money and help to people who need it (a) Philologist (b) Philanthropist (c) Impressionist (d) Receptionist SSC CHSL (Tier-I) 16/10/2020 (Shift-II)

631

YCT

Ans. (b) : The given group of words, 'someone who freely gives money and help to people who need it' is called 'philanthropist' The meaning of other options is Philologist:- A humanist specializing in classical scholarship. Impressionist:- Relating to or characteristic of impression. Receptionist:- A secretary whose main duty is to answer the telephone 465. Select the word that means the same as the given group of words. A person working on a trial period (a) Probationer (b) Petitioner (c) Practitioner (d) Professional SSC CHSL (Tier-I) 26/10/2020 (Shift-II) Ans. (a) : The given group of words, 'A person working on a trial period' is called 'probationer' The meaning of other options isPetitioners:- Requester, suppliant Practitioner:- Someone who practices a learned profession. professional:- Rerating to a profession. 466. Select the word which means the same as the groups of words given. A remedy for all diseases (a) Tablet (b) Syrup (c) Panacea (d) Nectar SSC CHSL (Tier-I) 26/10/2020 (Shift-I) Ans. (c) : The given group of words, 'A remedy for all diseases' is called 'panacea' The meaning of other is Tablet:- A small hat compressed cake of some substances. Syrup:- A thick sweet sticky liquid. Nectar:- A sweet lingered secretion that is attractive to pollinations. 467. Select the word which means the same as the group of words given. One who studies the elections and voting trends (a) Reviewer (b) Sociologist (c) Psephologist (d) Reseacher SSC CHSL (Tier-I) 14/10/2020 (Shift-III) Ans. (c) : The given group of words 'one who studies the auction and voting trends' is called 'psephologist' The meaning of other options is Review:- Reexamine, retrospect. Sociologist:- A social scientist who studies the institutions and development of human. Researcher:- A scientist who devotes himself. (investigator) 468. Select the word which means the same as the group of words given. A collection of drawings, documents, etc. That represent a person's. especially an artist's work (a) Dossier (b) Profile (c) Portfolio (d) Sketch SSC CHSL (Tier-I) 13/10/2020 (Shift-II) Ans. (c) : The given group of words, 'A collection of drawings, oddments, etc that represent a persons especially an artist's work is called 'portfolio' One Word Substitution

The meaning of other options isDossier:- File Profile:- A description of somebody/something that gives useful in for motion. Sketch:- Study Retort descriptive summary 469. In the following question, out of the four given alternatives, select the alternative which is the best substitute of the phrase. One who loves own country (a) Chariot (b) Adroit (c) Idiot (d) Patriot SSC MTS 7-10-2017 (Shift-I) Ans. (d) : The given group of words, 'one who loves own country' is called 'patriot' The meaning of other options isChariot:- A two wheeled horse drawn battle vehicle Adored:- Quick or skillful or adept in action or thought. Idiot:- A person of subnormal intelligence. 470. In the following question, out of the four given alternatives, select the alternative which is the best substitute of the phrase. A medical examination of a dead body. (a) Premortem (b) Postmortem (c) Prenatal (d) Postnatal SSC MTS 10-10-2017 (Shift-I) Ans : (b) The given group of word 'medical examination of a dead body' is called 'postmortem' The meaning of other options is premortem- Existing or taking place immediately before death. Prenatal:- Occurring or existing before birth. postnatal:- Occurring immediately after birth. 471. Choose the most appropriate word that can substitute the given group of words. Killing of pests (a) Patricide (b) Herbicide (c) Pesticide (d) Suicide SSC MTS – 06/08/2019 (Shift-III) Ans. (c) : The given group of words, 'killing of pests' is called 'pesticide' The meaning of other options isPatricide:- The murder of your father. Herbicide:- A chemical agent that destroys plants or inhibits their growth. Suicide:- The act of killing yourself. 472. Select the most appropriate word for the given group of words. A set of 52 playing cards (a) Bundle (b) Parcel (c) Pack (d) Packet SSC MTS – 22/08/2019 (Shift-I) Ans. (c) : The given group of words, 'A set of 52 playing cards' is called 'pack' The meaning of other options isBundle:- Package, parcel parcel:- A collection of things wrapped or boxed together. Packet:- A small package or bumble. 473. Select the most appropriate word for the given group of words. One who collects stamps (a) Curator (b) Geologist (c) Philatelist (d) Conjurer SSC MTS – 21/08/2019 (Shift-II)

632

YCT

Ans. (c) : The given group of words, 'One who collects stamps' is called 'philatelist' The meaning of other options isCurator:- The custodian of a collection (as a museum or library) Geologist:- A specialist in geology. Conjurer:- Magician. 474. Select the most appropriate word for the given group of words : An introduction to a literary work (a) prologue (b) Blurb (c) Epilogue (d) Sequel SSC MTS – 20/08/2019 (Shift-I) Ans. (a) : The given group of words 'An introduction to a literary work' is called 'prologue' The meaning of other other options is Blurb:- A promotional statement Epilogue:- A short speech (often in yerse) addressed directly to the audience by an actor at the end of a play. Sequel:- Something that follows something else (subsequence) 475. Select the most appropriate word for the given group of words. An animal that lives by killing and eating other animals (a) Amphibian (b) Prey (c) Predator (d) Quadruped SSC MTS – 19/08/2019 (Shift-I) Ans. (c) : The given group of words, 'An animal that lives by killing and eating other animals' is called 'predator' The meaning of other options is Predator:- Any animal that lives by preying on other animals. Amphibian:- A flat bottomed motor vehicle that can travel on land or water. Prey:- Animal hunted or caught for foot. Quadruped:- Having four feet. 476. Select the word which means the same as the group of words given. An introduction to a book usually stating its purpose (a) Bibliography (b) Preface (c) Epilogue (d) Acknowledgment SSC MTS – 09/08/2019 (Shift-II) Ans. (b) : The group of words, 'An introduction to a book usually stating its purpose' is called 'preface' The meaning of other options isBibliography:- A list of the book referred to in a scholarly work. Epilogue:- A short passage added at the end a work. Acknowledgement:- The state of being recognizea. 477. Choose the most appropriate word that can substitute the given group of words. A group of Lions (a) Pack (b) Pride (c) Herd (d) Flock SSC MTS – 06/08/2019 (Shift-II) Ans. (b) : The given group of words, 'A group of lions' is called 'pride' The meaning of other options is Pack:- A convenient package or parcel. Herd:- A group of cattle or sheep. Flock:- A group of birds/ goats. One Word Substitution

478. Select the word which means the same as the group of words given. An animal that lives by killing and eating other animal (a) Predator (b) Amphibian (c) Biped (d) Quadruped SSC GD – 08/03/2019 (Shift-II) Ans. (a) : The given group of words, 'An animal that lives by killing and eating other animal' is called 'predator' The meaning of other options isAmphibian:- An airplane designed to take off and land on water. Biped:- An animal with two feet. Quadruped:- Having four beet. 479. Select the word which means the same as the group of words given. An introduction made by the dramatist at the beginning of the play (a) Dialogue (b) Epilogue (c) Monologue (d) Prologue SSC GD – 02/03/2019 (Shift-II) Ans. (d) : The given group of words 'An introduction made by the dramatist at the beginning of the play' is called 'prologue' The meaning of other options is Dialogue:- A conversation between two persons. Epilogue:- A thorn section added at the end of a literary work. Monologue:- A dramatic speech by a single actor. 480. Select the word which means the same as the group of words given. The method and practice of teaching (a) Methodology (b) Psychology (c) Pedagogy (d) Philosophy SSC GD – 18/02/2019 (Shift-II) Ans. (c) : The given group of words 'The method and practice of teaching' is called 'pedagogy' The meaning of other options is Methodology:- The system of methods followed in a particular discipline. Psychology:- The science of mental life Philosophy:- The rational investigation of outstations about existence and knowledge and ethics.

Q 481. Select the option that can be used as a oneWord substitute for the given group of words. A four- footed animal (a) Reptile (b) Mammal (c) Biped (d) Quadruped SSC CGL (Tier-I) 21/04/2022 Shift-III Ans. (d) : For the above group of words, ‘A four footed animal’, option (d) Quadruped is appropriate one word. Other options have different meaning. Reptile – A group of animals which have scales on their skin and lay eggs. Mammal – An animal that gives birth to live babies, nor eggs, and feeds its young on milk.

633

YCT

482. Select the option that can be used as a oneword substitute for the given group of words. Reject as invalid, especially by legal procedure (a) Reiterate (b) Putrid (c) Quash (d) Putrefy SSC CHSL 30.05.2022 Shift-I Ans. (c) : For the above group of words, ‘ Reject as invalid, especially by legal procedure’, option (c) Quash is appropriate one word. Other options have different meaning. Reiterate - To repeat something that you have already said. Putrid - Very unpleasant. Putrefy – Decays and produces a very unpleasant smell. 483. Select the option that can be used as a oneword substitute for the given group of words. Sound made by ducks (a) Moo (b) Grunt (c) Caw (d) Quack SSC Constable GD-14/12/2021 Shift-II Ans. (d) : For the above group of words, ‘ Sound made by ducks,’ option (d) Quack is appropriate one word. Moo - Sound made by a cow or bull. Grunt - A short, low sound from the throat. Caw - Sound made by crows. 484. Select the most appropriate one word substitution for the given group of words. One who pretends to be a medical practitioner. (a) Culprit (b) Quack (c) Adept (d) Vanity SSC MTS 7-10-2017 (Shift-I) Ans. (b) : The above group of words 'one who pretends to be a medical practitioner' is called 'Quack' The meaning of other options is Culprit:- Someone who perpetrates wrongdoing Adept:- Having or showing knowledge and skill and aptitude. Vanity:- Feeling of excessive pride. 485. Select the most appropriate one word substitution for the given group of words. Divided into four equal or cores pending parts. (a) Tripod (b) Quarter (c) Half (d) Concrete SSC CPO-SI – 24/11/2020 (Shift-II) Ans. (b) : The above group of words 'Divided into four equal or corresponding parts' is called 'quarter' The meaning of other options is Tripod:- A tare legged stand (as for a camera) Half:- One half Concrete:- Tangible, touchable. 486. Select the most appropriate one word substitution for the given group of words. Stop something especially by using force (a) Soothe (b) Quell (c) Extinguish (d) Obtain SSC CGL (Tier-II) 16/11/2020 (3 pm - 5 pm) Ans. (b) : The above group of words 'stop something, especially by using force' is called 'Quell' The meaning of other options is Soothe:- Calm/quiet Extinguish:- Blow out Obtain:- Find, get, incur. One Word Substitution

R 487. Select the option that can be used as a oneword substitute for the given group of words. The murder of a king (a) Regicide (b) Sacrilege (c) Reticule (d) Rebel SSC CHSL (Tier-I) 10/08/2023 (Shift-I) Ans. (a) : One word substitution for the given group of words 'The murder of a king' will be option (a) regicide. Meaning of other options are – Sacrilege – Treating a religious object or place without the respect that it deserves, Reticule – A woman small handbag having a drawstring and decorated with embroidery Rebel – A person who fights against his country's government for the wants of change. 488. Select the most appropriate option that can be used as a one-word substitute for the underlined segment in the following sentence. The king’s coronation ceremony was a grand affair, with all the nobles and dignitaries dressed in their finest insignia of royalty. (a) spectres (b) regals (c) regalia (d) ornaments SSC CHSL (Tier-I) 08/08/2023 (Shift-II) Ans. (c) : The most appropriate one-word substitute for the above underlined segment 'insignia of royalty' is 'regalia' (reward). Meaning of other options – Spectres – Ghost, Demon, Regals – Very impressive, for a king or queen. Ornaments – The act of adorning or being adorned. 489. Select the option that can be used as a one-word substitute for the given group of word. Able to be happy, successful, etc. again after something difficult or bad has happened (a) Resilient (b) Remnant (c) Reluctant (d) Resistant SSC CGL (Tier-I) 19/04/2022 Shift-III Ans. (a) : For the above group of words, option (a) Resilient is appropriate one word. Other options have different meaning Remnant - A small remaining part. Reluctant - Feeling or showing hesitation or unwillingness. Resistant - Not wanting something and trying to prevent something happening. 490. Select the most appropriate one word to substitute the given group of words. Symbols of royalty (a) sceptre (b) relic (c) emblem (d) regalia SSC CGL (Tier-II) 08.08.2022 Shift-II Ans. (d) : For the above group of words ‘symbols of royalty’, option (d) regalia is appropriate one word. Other options have different meaning.

634

YCT

Sceptre – A decorated rod carried by a king or queen on 495. Select the most appropriate one-word substitution for the given group of words. ceremonial occasions as a symbol of their power. A deceased holy person’s remains or Relic – An object, a tradition, a system that has survived belongings kept for reverence from the past. (a) trophy (b) souvenir Emblem – An object or symbol that represents (c) memento (d) relic something. SSC CPO-SI – 24/11/2020 (Shift-II) 491. Select the option that can be used as a one- Ans. (d) : The given group of words 'A deceased holy word substitute for the given group of words. person’s remains or belongings kept for reverence' is One who has fixed opinions and beliefs called 'relic' (a) Broad-minded (b) Flexible The meaning of other options is Trophy:- Prize (c) Rigid (d) Amiable SSC Constable GD-09/12/2021 Shift-II Souvenir:- Something of sentimental value or a Ans. (c) : for the above group of words, ‘ one who has reminder of past events. fixed opinions and beliefs’, option (C) ‘Rigid’ is Memento:- A reminder of past events. appropriate one word. Other options have different 496. Select the word which means the same as the group of words given. meaning. The act of looking back on past time Broad-minded - Willing to accept opinions, beliefs, that (a) retrospection (b) inspection are unusual or different from your own. (c) introspection (d) circumspection Flexible - Able to bend or easily without breaking. SSC CGL (Tier-I) – 11/06/2019 (Shift-III) Amiable - Good-natured and likable. Ans. (a) : The given group of words 'The act of 492. Select the most appropriate one-word looking back on past time' is called 'retrospection' substitution for the given group of words. The meaning of other options is The act of thinking now about something in the Inspection:- A formal or official examination past. Introspection:- Self examination (a) Retrospection (b) Circumspection Circumspection:- Caution. (c) Introspection (d) Inspection 497. Select the most appropriate word which means SSC CHSL (Tier-I) –13/04/2021 (Shift-I) the same as the group of words given. Ans. (a) : The above group of words, 'The act of No revealing one's thoughts or feelings readily thinking now about something in the past' is called (a) forthright (b) evaluative 'retrospection' (c) reticent (d) communicative The meaning of other options is SSC CPO-SI – 11/12/2019 (Shift-I) Introspection:- Self examination. Ans : (c) The given group of words 'No revealing one's Circumspection:- Caution thoughts or feelings readily' is called 'reticent' Inspection:- A formal or optical examination. The meaning of other options is 493. Select the option that can be used as a one- Forthright:- Candid, frank. word substitute for the given group of words. Evaluative:- Critical, appraising Withdraw from a forward position in battle Communicative:- Able or tending to communicate. (a) Retreat (b) Resort 498. Select the word which means the same as the (c) Restore (d) Relocate group of words given. SSC CHSL (Tier-I) –19/04/2021 (Shift-I) A large natural or artificial lake as a source of Ans. (a) : The above group of words, withdraw from a water supply forward position in battle' is called 'retreat' (a) Pool (b) Pond The meaning of other options is (c) Creek (d) Reservoir Resort:- Fall back, recur. SSC CHSL (Tier-I) 17/03/2020 (Shift-I) Restore:- Reestablish, reinstate. Ans. (d) The given group of words 'a large natural or Relocate:- Become establish in a new location. artifical lake used as a source of water supply' is called 494. Select the most appropriate word which means 'reservoir' the same as the group of words given. The meaning of other options is An exact or a very close copy (of something) Pond:- A small lake (a) substitute (b) artificial Geek:- A small river (c) replica (d) forgery Polk:- A small body of standing water (rainwater) or SSC CPO-SI – 23/11/2020 (Shift-I) other livid. Ans. (c) : The given group of words 'An exact or a 499. Select the word which means the same as the very close copy' is called 'replica' group of words given. The meaning of other options is That which is no longer useful Substitute:- Replacement (a) Redundant (b) Abundant Artificial:- Contrived by art rather than nature. (c) Reluctant (d) Dominant Forgery:- Counterfeit. SSC CHSL (Tier-I) 12/10/2020 (Shift-I) One Word Substitution

635

YCT

Ans. (a) : The given group of words 'That which is no longer useful' is called 'redundant' The meaning of other options is Abundant:- Abounding ample Reluctant:- Not eager. Dominant:- Most frequent or common. 500. In the following question, out of the four given alternatives, select the alternatives which is the best substitute of the phrase. Amount paid to a man for his labour. (a) Incentive (b) Remuneration (c) Gratuity (d) Honorarium SSC MTS 9-10-2017 (Shift-I) Ans : (b) The given group of words 'Amount paid to a man for his lab our ' is called 'remuneration' The meaning of other options is Incentive:- A positive motivational influence (motivator) Gratuity:- A positive small amount of money given for services rendered. Honorarium:- A fee paid for a nominally free service. 501. In the following question, out of the four given alternatives, select the alternative which is the best substitute of the Phrase. One who receives something. (a) Gradient (b) Patient (c) Recipient (d) Lenient SSC MTS 11-10-2017 (Shift-I) Ans : (c) The given group of words ' One who receives something' is called 'recipient' The meaning of other options is Gradient:- Slope Patient:- Long suffering Lenient:- Not strict.

S 502. Select the word which means the same as the group of words underlined in the sentence. Only the artists who make sculptures are allowed to take part in exhibitions. (a) Sculptor (b) Painter (c) Potter (d) Artisan SSC CGL (Tier-I) 17/07/2023 (Shift-II) Ans. (a) : For the above group of word, only the artists who make sculptures are allowed to take part in exhibition, option (a) 'sculptor' gives appropriate meaning. Painter - Someone who paints pictures. Potter - A person who makes pots, bowls, plates etc. Artisan - A person skilled in making a product by hand. 503. Select the option that can be used as a oneword substitute for the given group of words. A person whose age is between 70 and 79 years old (a) Aged (b) Septuagenarian (c) Oldie (d) Pensioner SSC CGL (Tier-I) 24/07/2023 (Shift-III) One Word Substitution

Ans. (b) : The most appropriate one word for the given group of words 'A person whose age is between 70 and 79 years old' is 'septuagenarain'. Other options give different meaning. Aged - Geezer/very old Oldie - An old person or thing Pensioner - A person who is receiving a pension. 504. Select the most appropriate one word substitution for the given group of words. The violation or profanation of anything sacred or held sacred (a) Pious (b) Sacramental (c) Holy (d) Sacrilege SSC CHSL (Tier-I) 17/03/2023 (Shift-III) Ans. (d) : In the above group of words, "The violation or Profanation of anything sacred or held Sacred" is option (d) Sacrilege give an appropriate meaning. Other option give different meaning. Pious - Having or showing a deep belief in religion. Sacramental- Relating to an important religious ceremony in the Christian church. Holy- Connected to a god or a religion. 505. Choose the word that can substitute the given group of words. One who helps the needy and the helpless (a) Novice (b) Egotist (c) Samaritan (d) Highbrow SSC CHSL (Tier-I) 15/03/2023 (Shift-III) Ans. (c) : In the above group of words, ''one who helps he needy and the helpless'' is option (c) Samaritan give proper meaning. Other option give different meaning. Egotist → A conceited, boastful person. Highbrow → Highly cultured and sophisticated. Novice → A person who is not experienced in a job or situation. 506. Select the option that can be used as a oneword substitute for the given group of words. The study of earthquakes (a) Seismology (b) Geography (c) Topography (d) Geology SSC CGL (Tier-I) 11/04/2022 Shift-I Ans. (a) : For the above group of words, ‘The study of earth –quakes’, option (a) seismology is appropriate one word. Geography - The scientific study of the earth's surface physical features, divisions, population etc. Topography - The study of the features and forms of land surfaces. Geology - The study of the rocks, and of the way they are formed. 507. Select the option that can be used as a oneword substitute for the given group of words. Indifferent to pleasure and pain (a) Lusty (b) Stoic (c) Cynic (d) Prudent SSC CGL (Tier-I) 12/04/2022 Shift-I Ans. (b) : For the above group of words, ‘ Indifferent to pleasure and pain’, option (b) stoic is appropriate one word. Other options have different meaning.

636

YCT

Lusty - Full of strength and energy. Cynic - A person who has negative opinions about other people. Prudent - Showing good judgment in avoiding risks and uncertainties. 508. Select the most appropriate one word to substitute the given group of words. One who speaks for others (a) spokesperson (b) orator (c) verbose (d) talkative SSC CGL (Tier-II) 08.08.2022 Shift-II Ans. (a) : For the above group of words ‘one who speaks for others, option (a) spokesperson is appropriate other options have different meaning. Orator – A person who is good at making public speeches. Verbose – Containing more words than are needed. Talkative – To talk a lot. 509. Select the most appropriate one word to substitute the given group of words. a case for keeping a sword. (a) sleeve (b) sheath (c) quiver (d) wrapper SSC CGL (Tier-II) 08.08.2022 Shift-II Ans. (b) : For the above group of words, ‘ a case for keeping a sword’, option (b) sheath is appropriate one word. Other options have different meaning. Sleeve – A part of a garment covering an arm. Quiver – To shake slightly. Wrapper – A piece of paper, plastic that is wrapped around something. 510. Select the option that can be used as a oneword substitute for the given group of words. A short rest or sleep taken aften lunch (a) Doze (b) Leisure (c) Slumber (d) Siesta SSC CGL (Tier-I) –24/08/2021 (Shift-I) Ans. (d) : The above group of words, A short rest or sleep taken aften lunch' is called 'siesta' The meaning of other options is Doze:- A light fitful sleep Leisure:- Time available for ease and relaxation. Slumber:- be asleep 511. Select the option that can be used as a oneword substitute for the given group of words. A person who walks in his/her sleep (a) Exorcist (b) Atheist (c) Somnambulist (d) Ventriloquist SSC Constable GD-07/12/2021 Shift-II Ans. (c) : For the group of words ‘A person who walks in his/her sleep ‘ option (c) somnambulist is appropriate one word. Exorcist - A person who makes spirits leave a place by prayers or ceremonies. Atheist - A person who does not believe in the existence of a god. Ventriloquist - Someone who can speak without moving their lips. One Word Substitution

512. Select the most appropriate one-word substitution for the given words. A close-fitting cover for a sword (a) Quiver (b) Shank (c) Sheath (d) Dagger SSC CGL (Tier-II) 16/11/2020 (3 pm - 5 pm) Ans. (c) : The above group of words 'A close-fitting cover for a sword' is called 'sheath' The meaning of other options is Quiver:- Cafe for holding arrows. Shank:- The part of the human leg between the Dagger:- A short knife with a pointed blade used for piercing or stabbing . 513. Select the word which means the same as the group of words given. A large number of fish swimming together (a) herd (b) shoal (c) brood (d) cache SSC CPO-SI (Tier-II) 27/09/2019 (3 pm - 5 pm) Ans. (b) : The above group of words 'A large number of fish swimming together' is called 'shoal' The meaning of other options is Herd:- A group of cattle or sheep brood:- Group of fetching Cache:- A hidden storage space. 514. In the following question, out of the four alternatives, select the alternative which is the best substitute of the phrase. A government by the military class (a) Kratocracy (b) Pantisocray (c) Stratocracy (d) Monocracy SSC CGL (Phase-II) 17/02/2018 Ans. (c) The above group of words, a government by the military class' is called 'stratocracy' The meaning of other options is Pantisocracy:- A form of utopian social organization in which all are equal in social position and responsibility. Kratocracy:- Rule by the strong or those seize power through force. 515. In the following, out of the four alternatives, select the alternative which is the best substitute of the phrase. A person's last performance (a) Prognosis (b) Elegy (c) Memoir (d) Swan song SSC CGL (Phase-II) 17/02/2018 Ans. (d) The above group of words, a person's last performance is called 'swan song' The meaning of other options is Prognoses:- A prediction of the course of a disease (forecast) Elegy:- A mournful poem, a lament for the dead. Memoir:- An account of the author's personal experiences. 516. Select the most appropriate word for the given group of words. A nature reserve for birds of animals (a) retreat (b) asylum (c) oasis (d) sanctuary SSC CPO-SI – 25/11/2020 (Shift-I) Ans. (d) : The given group of words A nature reserve for birds of animals' is called 'sanctuary' The meaning of other options is

637

YCT

Retreat:- Move back Asylum:- A hospital for mentally incompetent or unbalanced people. Oasis:- A shelter serving as a place of safety or sanctuary. 517. Select the word which means the same as the group of words given. One who helps a person in need (a) mercenary (b) veteran (c) samaritan (d) collaborator SSC CGL (Tier-I) – 10/06/2019 (Shift-III) Ans. (c) : The given group of words 'One who helps a person in need' is called 'samaritan' The meaning of other options is Mercenary:- A person hired to fight for another country than their own. Veteran:- A person who has served in the armed forces. (ex-serviceman) Collaborator:- Cooperator, partner. 518. Select the option that can be used as a oneword substitute for the given group of words. A person who can endure pain or hardship without showing his feelings or complaining (a) stoic (b) agnostic (c) ascetic (d) pedant SSC CPO-SI – 09/12/2019 (Shift-I) Ans : (a) The given group of words 'A person who can endure pain or hardship without showing his feelings or complaining' is called stoic. The meaning of other options is Agnostic:- A person who claims that they cannot have true knowledge about the existence of god. Ascetic:- Someone who practice self denial as a spiritual discipline pedant:- A person who pays more attention to formal rules and book learning than they merit. (scholastic) 519. Select the option which is the most appropriate one word substitution for the given group of words. A protected place for birds and animals (a) bunker (b) island (c) sanctuary (d) estuary SSC CPO-SI – 11/12/2019 (Shift-II) Ans : (c) The given group of words 'A protected place for birds and animals' is called 'sanctuary' The meaning of other options isBunker:- A large container for storing fuel. Island:- A land mass that is surrounded by water. Estuary:- The wide part of a river where et nears the sea, fresh ash and salt water. 520. Select the option which is the most appropriate one word substitution for the given group of words. One who is sixty years old (a) Nonagenarian (b) Sexagenarian (c) Octogenarian (d) Septuagenarian SSC CPO-SI – 13/12/2019 (Shift-I) Ans. (b) : The given group of words 'One who is sixty years old' is called 'sexagenarian'. The meaning of other options is Nonagenarian:- (90 - 99 years old) Octogenarian:- (80 - 89 years old) Septuagenarian:- (70 - 79 years old) One Word Substitution

521. Select the most appropriate word for the group of words. An office with high salary but no work (a) honorary (b) sinecure (c) autocracy (d) bureaucracy SSC CHSL (Tier-I) –05/07/2019 (Shift-II) Ans : (b) The given group of words 'An office with high salary but no work' is called 'sinecure'. The meaning of other options is Honorary- Given as an honor without the normal duties Autocracy- A political theory favoring unlimited authority by a single individual Bureaucracy- A system of government in which there are a large number of state officials who are not elected. 522. Select the most appropriate word for the group of words. One who is indifferent to pleasure or pain (a) wise (b) temperate (c) brave (d) stoic SSC CHSL (Tier-I) –05/07/2019 (Shift-III) Ans : (d) the given group of words 'One who is indifferent to pleasure or pain' is called 'stoic' The meaning of other options is Wise- Prudent, omniscient Temperate- Not extreme, moderate, mined. Brave- Courageous 523. Select the word which means the same as the group of words given. Close fitting cover for a sword or knife (a) Shield (b) Guard (c) Screen (d) Sheath SSC CHSL (Tier-I) 17/03/2020 (Shift-I) Ans. (d) The given group of words 'closed fitting cover for a sured or knife' is 'sheath' The meaning of other options isShield–Armor carried on the arm to intercept blows Guard – Watch over or shield from danger or harm, protect. Screen– Partition consisting of a decorative frame or panel that serves to divide a space. 524. Select the word which means the same as the group of words given. A vertical passageway into a mine (a) Lobby (b) Shaft (c) Canal (d) Aisle SSC CHSL (Tier-I) 17/03/2020 (Shift-II) Ans. (b) : The given group of words 'a vertical passageway into a mine' is called 'shaft' The meaning of other options Lobby – Antechamber, anteroom, hall. Canal – Lang and narrow strip of water made for boats or irrigating. Aisle – A passage between rows of seats in a church, theatre, train etc. 525. Select the word which means the same as the group of words given. Small room where dishes are washed. (a) Larder (b) Scullery (c) Pantry (d) Ante-room SSC CHSL (Tier-I) 18/03/2020 (Shift-II)

638

YCT

Ans. (b) : The given group of words 'Small room where dishes are washed' is called 'scullery' The meaning other options isLarder – A small storeroom for storing Pantry – Buttery Ante-room – A large entrance or reception room or area. 526. Select the word which means the same as the group of words given. A person who is blamed for something that someone else has done (a) Competitor (b) Enemy (c) Opponent (d) Scapegoat SSC CHSL (Tier-I) 14/10/2020 (Shift-I) Ans. (d) : The given Group of words 'A person who is blamed for something that someone else has' is calledscapegoat' The meaning options isCompetitor:- Challenger, contender. Enemy:- One who oppose someone. Opponent:- Antagonist, oppose 527. In the following question, out of the four given alternatives, select the alternative which is the best substitute of the phrase. The bony framework of the body. (a) Respiratory system (b) Skeleton (c) Neurons (d) Arteries SSC MTS 10-10-2017 (Shift-III) Ans. (b) : The given group of words 'The bony framework of the body' is called 'skeleton' The meaning of other options is Respiratory system - The system for taking in oxygen and giving off carbon dioxide. Neurons - A all that is specialized to conduct nerve impulses. Arteries - A blood vessel that carries blood from the heart to the body. 528. Select the word which means the same as the group of words given. A speech made to oneself (a) Soliloquy (b) Solitary (c) Eloquent (d) Dialogue SSC MTS – 09/08/2019 (Shift-I) Ans. (a) : The given group of words 'A speech made to oneself' is called soliloquy' The meaning of options isSolitary - Nonsocial Eloquent - Smooth , spoken. Dialogue - A conversation between two persons. 529. Choose the most appropriate word that can substitute the given group of words. Full of criticism and mockery (a) Cliche (b) Praise (c) Satire (d) Eulogy SSC MTS – 05/08/2019 (Shift-II) Ans. (c) : The given Group of words 'Full of criticism and mockery' is called 'satire' The meaning of other options isPraise- An expression of approval and commendation. Eulogy- A formal expression of praise. (paneguyric) One Word Substitution

530. Choose the word that can substitute the given sentence. An internal or external framework of bones (a) Spine (b) Skull (c) Skeleton (d) Skin SSC MTS – 05/08/2019 (Shift-I) Ans. (c) : The given Group of words 'An internal or external framework of bones' is called 'skeleton' The meaning of options isSpine - backbone, spinal Skull - The bony skeleton of the head of vertebrates. Skin - A natural protective body covering and site of the sense of touch 531. Select the most appropriate word for the given group of words. A set of twenty (a) Dozen (b) Score (c) Quarter (d) Decade SSC MTS – 21/08/2019 (Shift-III) Ans. (b) : The given Group of words 'A set of twenty' is called 'score' The meaning of other options isDozen - A set of twelve. Quarter - Each of four equal parts. Decade - A period of ten years. 532. Select the most appropriate word for the given group of words. A large number of fish swimming together (a) Pack (b) Shoal (c) Horde (d) Herd SSC MTS – 21/08/2019 (Shift-II) Ans. (b) : The given group of words 'A large number of fish swimming together' is called 'shoal' The meaning of other options isPack - Bundle, compact Horde -A vast multitude, a nomadic community Herd - A group of cattle or sheep 533. Select the most appropriate word for the given group of words. Government not connected with religious or spiritual matters. (a) Socialist (b) Republic (c) Sovereign (d) Secular SSC MTS – 21/08/2019 (Shift-I) Ans : (d) The given group of words 'Government not connected with religious or spiritual matters' is called 'secular' The meaning of other options isSocialist - Advocating or following the socialist principles. Republic -A form of government whose head of state is not a monarch Sovereign - Crowned head, monarch. 534. Select the word which means the same as the group of words given. A building in which Jewish people worship and study their religion (a) Mosque (b) Temple (c) Church (d) Synagogue SSC GD – 08/03/2019 (Shift-III) Ans. (d) : The given group of words 'A building in which Jewish people worship and study their religion' is called 'synagogue'

639

YCT

The meaning of other options isTemple - Place of worship consisting of an edifice for the worship of a deity. Church- A place for public (especially Christian) worship. Monastery - The residence of a religious community. 535. Select the word which means the same as the group of words given. Not connected with religious or spiritual matters. (a) Secular (b) Veteran (c) Communal (d) Atheist SSC GD – 08/03/2019 (Shift-II) Ans. (a) : The given group of words ' Not connected with religious or spiritual matters' is called 'secular' The meaning of other options is Veteran - A person who has served in the armed forces. Communal - For or by a group rather than individuals Atheist - Someone who denies the existence of god.

T 536. Choose the word that can substitute the given group of words. The young of a frog (a) Tadpole (b) Kitten (c) Antelope (d) Pup SSC MTS 02/05/2023 (Shift-I) Ans. (a) : The most appropriate group of the word for 'The young of a frog' will be 'Tadpole'. other words are___ kitten- The young of a cat. Antelope- An antelope is an animal that resembles a deer, with four legs, hooves and antlers. Pup- The young one of a dog. 537. Select the most appropriate word for the given group of words. Quiet and peaceful. (a) Inundate (b) Incessant (c) Introvert (d) Tranquil SSC MTS 19/05/2023 (Shift-III) Ans. (d) : The most appropriate word for the above given group of word – ‘Quiet and peaceful’. Is ‘Tranquil’. Meaning of other options – Inundate – To cover an area of lord with a large amount of water. Incessant – Never stopping. Introvert – A quiet, shy person who prefers to be alone than with other people. 538. Select the option that can be used as a oneword substitute for the given group of words. An agreement between enemies or opponents to stop fighting, arguing, etc., for a certain period of time (a) Unanimous (b) Graft (c) Truce (d) Surplus SSC MTS-15/07/2022 Shift-I One Word Substitution

Ans. (c) : For the above group of words, option (C) truce is appropriate one word. Unanimous - Agreed to everyone. Graft - A piece cut from a living plant and fixed in a cut made in another plant. Surplus - an extra amount or something. 539. Select the option that can be used as a oneword substitute for the given group of words. Connected with the sense of touch (a) Visual (b) Auditory (c) Tactile (d) Olfactory SSC Constable GD-13/12/2021 Shift-III Ans. (c) : For the above group of words, ‘connected with the sense of touch’, option (c) Tactile is appropriate one word. Visual - Connected with seeing. Auditory - Relating to the sense of hearing. Olfactory - Connected with the sense of smell. 540. Select the option that can be used as a oneword substitute for the given group of words. The sound an elephant makes (a) Bray (b) Grunt (c) Low (d) Trumpet SSC Constable GD-07/12/2021 Shift-II Ans. (d) : For the above group of words, ‘The sound an elephants makes’, option (d) trumpet is appropriate one word. Bray - The loud, harsh cry of a donkey or mule. Grunt - To make a short low sound in the throat. Low - Close to the ground. 541. Select the option that can be used as a oneword substitute for the given group of words. A group of three novels or plays, each complete in itself. (a) Triplet (b) Triumvir (c) Trilogy (d) Trivet SSC CGL (Tier-I) –13/08/2021 (Shift-I) Ans. (c) : The above group of word, A group of three novels or plays, each complete in itself is called 'trilogy' The meaning of other options is Triplet - Triple, triad (a set of three similar things considered as a unit) Triumvir - One of a group of three sharing public administer ration Trivet - A stand, often with short feet used under a hot wish on a table. 542. In the following question, out of the four alternatives, select the alternative which is the best substitute of the phrase. A person who preserves skin of animals (a) Biloquist (b) Philologist (c) Taxidermist (d) Oculist SSC CGL (Phase-II) 17/02/2018 Ans. (c) The above group of words, a person who preserves skin of animals' is called 'taxidermist' The meaning of other options isBiloquist – Person who can speak is two voices Philologist – A humanist specializing in classical scholarship Oculist – Specialist of eyes

640

YCT

543. Select the most appropriate one-word substitution for the given group of words. One who changes sides (a) novice (b) amateur (c) invader (d) turncoat SSC CPO-SI – 23/11/2020 (Shift-II) Ans. (d) : The given group of words 'one who changes sides' is called 'turncoat' The meaning of other options isNovice - Beginner, initiate, tyro Amateur - Someone who pursue a study or sport as a pastime. Invader - Someone who enters by force in order to conquer. 544. Select the word which means the same as the group of words given. Too unimportant to consider (a) dimunitive (b) trivial (c) nothing (d) noticeable SSC CGL (Tier-I) – 13/06/2019 (Shift-I) Ans : (b) The given group of words 'Too important to consider' is called 'trivial' The meaning of other options isDimunitive - Very small Nothing - Nix, zero, nil. Noticeable- Capable of being detected. (perceptible) 545. Select the most appropriate option that can be used as a one-word substitute for the given group of words. A cup or other decorative object awarded as a prize for a victory or success (a) gift (b) crown (c) present (n) (d) trophy SSC CPO-SI – 12/12/2019 (Shift-II) Ans : (d) The given group of words 'A cup or other decorative object awarded as a prize for a victory or success' is called 'trophy' The meaning of other options isGift - The act of giving Crown - A wreath or garland worn on the head to signify victory. Present - Award. 546. Select the most appropriate word for the given group of words A strong and fast moving stream of water (a) rivulet (b) creek (c) torrent (d) trickle SSC CHSL (Tier-I) –11/07/2019 (Shift-II) Ans : (c) The given group of words 'A strong and fast moving stream of water' is called 'torrent' The meaning of other options isRivulet- A small stream. (streamed) Creek- Brook or flow slowly (filter) Trickle- Run or flow slowly (filter) 547. Select the most appropriate word for the given group of words. The art of cleaning and preserving animal skins (a) etymology (b) taxonomy (c) dermatology (d) taxidermy SSC CHSL (Tier-I) –09/07/2019 (Shift-II) One Word Substitution

Ans : (d) The given group of words 'The art of cleaning and preserving animal skins' is called 'texidermy' The meaning of other options isEtymology- A history of a word Taxonomy- A classification of organisms into groups based on similarities of structure or origin. Dermatology- Dermatology the branch of medicine dealing with the skin and its diseases. 548. Select the word which means the same as the group of words given. Relating to the healing of diseases (a) Preventive (b) Immunity (c) Inoculation (d) Therapeutic SSC CHSL (Tier-I) 15/10/2020 (Shift-I) Ans. (d) : The given group of word 'Relating to the healing of diseases' is called 'therapeutic' The meaning of other options isImmunity - Resistance Inoculation- Vaccination Preventive - Tending to prevent or hinder. 549. In the following question, out of the four given alternatives, select the alternative which is the best substitute of the phrase. A portable radio set. (a) transistor (b) transcription (c) Transposition (d) Transaction SSC MTS 10-10-2017 (Shift-III) Ans. (a) : The given group of words 'A portable radio set' is called 'transistor' The meaning of other options isTranscription - Written text. Transposition - Reversal Transaction - Dealings. (action taken by a group of people especially an act of buying or selling) 550. Select the word which means the same as the group of words given. A young child just beginning to walk (a) Toddler (b) Cub (c) Joey (d) Kid SSC MTS – 08/08/2019 (Shift-III) Ans. (a) : The given group of words A young child just beginning to walk' is called 'toddler' The meaning of other options isCub - The young of certain carnivorous mammals such as the bear or wolf or lion Joey - A baby kangaroo or other marsupial. Kid - Young goat (kidskin) 551. Choose the most appropriate word that can substitute the given group of words. A violent windstorm (a) Temperate (b) Drought (c) Draught (d) Tempest SSC MTS – 05/08/2019 (Shift-III) Ans. (d) : The given group of words 'A violent windstorm' is called 'tempest' The meaning of other options isDrought - A shortage of rainfall. Draught - A serving of drink drawn from a keg. Temperate - Moderate (next extreme) 552. Select the most appropriate word for the given group of words. One who abstains from alcoholic drinks

641

YCT

(a) Vegetarian (c) Drunkard

(b) Non-vegetarian Ans. (d) : The given group of words 'Having no moral (d) Teetotaller principles' is called 'unscrupulous' SSC MTS – 16/08/2019 (Shift-I) unaware - Unmindful, oblivious Ans. (d) : The given group of words 'one who abstains Mischievous – Naughty or annoyingly playful. from alcoholic drinks' is called 'teetotaller' Ethical – Conforming to accept standards of social or The meaning of other options isprofessional behavior Vegetarian - Eater of fruits and grains and nuts. Or someone who eats no meat or fish or any animal products. Non-Vegetarian - A person who died includes meat 557. Select the option that can be used as a oneDrunkard - A chronic drink word substitute for the given group of words. One who possesses outstanding technical ability in a particular art or field (a) Virtuoso (b) Amateur 553. Select the word which means the same as the (c) Gourmand (d) Virtuous group of words given. SSC CHSL (Tier-I) 08/08/2023 (Shift-II) An imaginary, perfect state or place Ans. (a) : The most appropriate one-word substitute is (a) Utopia (b) Dystopia 'Virtuoso', for the above group of words – 'One who (c) Nostalgia (d) Arcadia SSC CHSL (Tier-I) –16/04/2021 (Shift-I) possesses outstanding technical ability in a particular art or field.' Ans. (a) : The above group of words 'An imaginary, Meaning of other options – perfect state or place' is called 'utopia' Amateur – 'A person who does something for pleasure, The meaning of other options is Arcadia - A department of state in which central not as a job.' Gourmand – 'Eating good food and lots of it.' Peloponnese. Dystopia - An imaginary place or state in which Virtuous – Behaving in a morally good way.' everything is extremely bad or unpleasant 558. Select the option that can be used as a oneNostalgia - A feeling of sadness mixed with pleasure word substitute for the given group of words. and affection when you think of happy time in the past Person who portrays bad character roles in 554. Select the most appropriate one-word cinema substitution for the given words. (a) Villain (b) Altruist An imaginary ideal society (c) Sadist (d) Warrior (a) Model (b) Flawless SSC Constable GD-29/11/2021 Shift-II (c) Ultimate (d) Utopia Ans. (a) : For the above group of words ‘Person who SSC CGL (Tier-II) 16/11/2020 (3 pm - 5 pm) portrays bad character roles in cinema’, option (a) Ans. (d) : The above group of words 'An imaginary villain is appropriate one word. ideal society' is called 'utopia' Altruist - A person who devoted to the welfare of Model - The act of representing something others. Ultimate - Furthest or highest in degree. Sadist - A person who gets pleasure out of giving pain Flawless - Perfect. to others. 555. Select the word which means the same as the Warrior - A person who fights in a battle. group of words given. An imagined society where everything is 559. Select the option that can be used as a oneword substitute for the given group of words. perfect and everyone is happy In exactly the same words as the original (a) Responsible (b) Hypocrite (a) Verbatim (b) Copy (c) Utopia (d) Evangelist (c) Imitation (d) Duplicate SSC CHSL (Tier-I) 16/10/2020 (Shift-III) SSC CGL (Tier-I) 12/04/2022 Shift-III Ans. (c) : The given group of words 'An imagined Ans. (a) : For the above group of words, ‘In exactly the society where everything is perfect and everyone is same words as the original’ , option (a) vibration is happy' is called 'utopia' appropriate one word. Other options have different The meaning of other options isResponsible - Worthy of or requiring responsibility or meaning. Copy- A thing that is made to be the same as something. trust Imitation- A copy of something real. Hypocrite - Pretender, dissimulator. Duplicate- To make an exact copy of something. Evangelist - A preacher of the Christian gospel. 556. Select the word which means the same as the 560. Select the option that can be used as a onegroup of words given. word substitute for the underlined group of Having no moral principles words. (a) Unaware (b) Mischievous Samantha keeps herself occupied by helping (c) Ethical (d) Unscrupulous other people willingly without expecting any SSC CHSL (Tier-I) 12/10/2020 (Shift-II) payment for her services.

V

U

One Word Substitution

642

YCT

(a) Mercenary (c) Volunteer

(b) Opportunist (d) Iconoclast SSC CHSL 27.05.2022 Shift-II Ans. (c) : For the above group of words, option (c) Volunteer is appropriate one word. Mercenary - A soldier who will fight for any country or group that offers payment. Opportunist - Making use of an opportunity, especially to get an advantage for yourself. Iconoclast - A person who criticizes popular beliefs or established customs and ideas. 561. Select the option that can be used as a oneword substitute for the given group of words. One who gives money or help to another person or cause (a) Agnostic (b) Amateur (c) Benefactor (d) Anarchist SSC CHSL 27.05.2022 Shift-III Ans. (c) : For the above group of words ‘one who gives money or help to another person’. Option (C) Benefactor is appropriate one word. Agnostic - A person who is not sure it God exists or not. Amateur - A person who does something for enjoymeny not money. Anarchist – A person who believes that laws and government not necessary. 562. Select the option that can be used as a oneword substitute for the given group of words. One who has a long experience of any occupation (a) Reverent (b) Practitioner (c) Staunch (d) Veteran SSC MTS-15/07/2022 Shift-I Ans. (d) : For the above group of words, ‘one who has a long experience of any occupation’, option (d) veteran is appropriate one word. Other options have different meaning. Reverent – Showing respect. Practitioner – One who practices a profession. Staunch – Always showing strong support in your opinions and attitude. 563. Select the option that can be used as a oneword substitute for the given group of words. One who is well-versed in the knowledge of plants (a) Gardener (b) Vegetarian (c) Botanist (d) Florist SSC CHSL 26.05.2022 Shift-II Ans. (c) : For the above group of words ‘one who is well versed in the knowledge of plants’, option (c) Botanist is appropriate one word. Gardener - A person who works in a garden. Vegetarian - A person who does not eat meat or fish. Florist - A person who owns or works in a shop that sells flowers and plants. One Word Substitution

564. Select the option that can be used as a one-word substitute for the given group of words. An animal with a spinal cord (a) Vertebrate (b) Invertebrate (c) Mammal (d) Amphibian SSC CGL (Tier-I) 19/04/2022 Shift-III Ans. (a) : For the above group of words, ‘ An animal with a spinal cord, option (a) Vertebrate is appropriate one word. Other options have different meaning. Invertebrate - An animal without a back bone. Mammal - An animal that gives birth to live young and feeds its young on milk. 565. Select the option that can be used as a oneword substitute for the underlined group of words. Nisha participates in almost every activity and in all functions held in the school as she has many skills. (a) Innovative (b) Vulnerable (c) Versatile (d) Fragile SSC CHSL 31.05.2022 Shift-III Ans. (c) : For the above underlined group of words ‘ has many skills’ , option (C) versatile is appropriate one word. Other options have different meaning. Innovative - New and original. Vulnerable - Weak and easy to hurt physically or emotionally. Fragile – Easily damaged or broken. 566. Select the option that can be used as a oneword substitute for the given group of words. A new project or business activity involving some risk. (a) Entrepreneur (b) Venture (n) (c) Experiment (d) Taboo SSC CHSL (Tier-I) –05/08/2021 (Shift-I) Ans. (b) : The above group of words 'A new project or busines activity involving some risk' is called 'venture' The meaning of other options isExperiment - The testing of an idea. Entrepreneur - Someone who organizes a business venture and assumes the risk for it Taboo - An inhibition or ban resulting from social custom or emotional aversion. 567. In the following question, out of the four alternatives, select the alternative which is the best substitute of the phrase. Killer of prophet (a) Mariticide (b) Vaticide (c) Patricide (d) Sororicide SSC CGL (Phase-II) 17/02/2018 Ans. (b) The above phrase 'killer of prophet' is called 'vaticide' The meaning of other options isMatricide - The murder of one's own mother Patricide - The crime of killing your father Sororicide - The killing of one's sister. 568. Select the word which means the same as the group of words given A long sea journey (a) trip (b) voyage (c) excursion (d) expedition

643

SSC Sel. Post Phase-VII (M.L.) 15.10.2019 (Shift-I)

YCT

Ans. (b) : The above group of words, a long sea journey' is called 'voyage' The meaning of other options isTrip - Miss a step and fall or nearly fall a journey to a place and back again. Excursion - A short journey made for pleasure. (tour) Expedition - A journey organized for a particular purpose 569. Select the most appropriate word which means the same as the group of words given. Something done because one wants to, not by force (a) threatened (b) voluntary (c) subjected (d) coered SSC CPO-SI – 23/11/2020 (Shift-I) Ans. (b) : The given group of words 'Something done because one wants to, not by force' is called 'voluntary' The meaning of other options isThreatened - (of flora or fauna) likely in the near future to become endangered. Coered - Force, pressure. Subjected - Defeat people or a country and then control them against their wishes 570. Select the most appropriate one-word substitution for the given group of words. A person who is unduly anxious about his/her health (a) disciplinarian (b) egalitarian (c) valetudinarian (d) antiquarian SSC CPO-SI – 23/11/2020 (Shift-II) Ans. (c) : The given group of words 'a person who is unduly anxious about his/her health' is called 'valetudinarian' The meaning of other options is Disciplinarian - Moralist Egalitarian - Favoring social equality Antiquarian - Relating to antique. (archaist) 571. Select the word which means the same as the group of words given. A person without a settled home or regular work who wanders from place to place and lives by begging. (a) Vagrant (b) Truant (c) Itinerant (d) Migrant SSC CGL (Tier-I) – 04/06/2019 (Shift-II) Ans : (a) The given group of words 'A person without a settled home or regular work who wanders from place to place and lives by begging' is called 'vagrant' The meaning of other options isTruant- One who is absent from school without permission Itinerant- Travelling from place to place to work Migrant- Traveler who moves from one region or cam try to another. 572. Select the most appropriate word for the given group of words. Able to adapt to many different functions or activities (a) expert (b) versatile (c) surrogate (d) deputy SSC CHSL (Tier-I) –10/07/2019 (Shift-II) One Word Substitution

Ans : (b) The given group of words 'Able to adapt to many different functions or activities' is called 'versatile' The meaning of other options isExpert- Pundit (a person with special knowledge) Surrogate- Someone who takes the place of another person. Deputy- A person appointed to represent or act on behalf of others. 573. Select the word which means the same as the group of words given. A plantation of grapes (a) Nursery (b) Orchard (c) Vineyard (d) Grove SSC CHSL (Tier-I) 19/10/2020 (Shift-II) Ans. (c) : The given group of words 'a plantation of grapes' is called 'vineyard' Nursery – A child's room for a baby. (baby's room) Orchard– Garden consisting of a small cultivation wood without undergrowth Grove– A small growth of trees without under bush 574. Select the most appropriate word for the given of words. A person who has long experience in a particular field (a) Veteran (b) Amateur (c) Cosmopolitan (d) Stoic SSC MTS – 14/08/2019 (Shift-III) Ans. (a) : The given Group of words–'A person who has long experience in a particular field' is called 'veteran' The meaning of other options isAmateur– Someone who pursues a study or sport as a pastime Cosmopolitan– Universal, world-wide Stoic– Someone who is seemingly indifferent to emotions. 575. Select the most appropriate word for the given group of words. The decision that is given by the jury or judge at the end of a trial (a) Accusation (b) Punishment (c) Allegation (d) Verdict SSC MTS – 13/08/2019 (Shift-II) Ans. (d) : The given group of words ' The decision that is given by the jury or judge at the end of a trial' is called 'verdict' The meaning of other options isAccusation - An assertion that someone is gaiety of a fault or offence. Punishment -The act of punishing. Allegation - A formal accusation against somebody.

W 576. Select the option that can be used as a oneword substitute for the given phrase. Forsaken or neglected child who has no home and spends most of his/her time on the streets (a) Waif (b) Fugitive (c) Vulnerable (d) Miscreant SSC CGL (Tier-I) 19/07/2023 (Shift-IV)

644

YCT

Ans. (a) : The mast appropriate one word substitute for the given phrase. 'Forsaken or neglected child who has no home and spends most of his/her time on the streets' is 'waif' The meaning of other words areFugitive - A person who has escaped or running Vulnerable - Able to easily heart, influenced or attacked. Miscreant - A person who behaves badly. 577. Select the most appropriate option that can substitute the underlined words in the given sentence. All his bones could be counted because his body was totally shrivelled and wrinkled with age. (a) bony (b) shrunken (c) wizened (d) dried SSC CHSL 25.05.2022 Shift-I Ans. (c) : For the above group of words, ‘shriveled and wrinkled with age’, option (c) wizened is appropriate one word. Bony - Very thin so that the bones can be seen under the skin. Shrunken - Smaller than before and less important. Dried - Free from liquid and especially water. 578. Select the option that can be used as a oneword substitute for the given group of words. To dispute angrily (a) Wrench (b) Wrangle (c) Wrack (d) Wreck SSC CHSL 31.05.2022 Shift-III Ans. (b) : For the above group of words ‘ To dispute angrily’ option (b) wrangle is appropriate one word. Other options have different meaning. Wrench - To pull something strongly and suddenly. Wreck - A vehicle or ship that has been destroyed or badly damaged. 579. Select the most appropriate one-word substitution for the given group of words. To separate husk from the grain (a) whisk (b) willow (c) whip (d) winnow SSC CPO-SI – 25/11/2020 (Shift-II) Ans. (d) : The given group of words 'To separate husk from the grain whisk' is called 'winnow' The meaning of other options isWillow - A tree with long thin branches and long thin leaves, that often grows hear water. Whip - An instrument with a handle and a flexible lash that is used for whipping. 580. Select the word which means the same as the group of words given. Flowers or leaves woven together in a circle for placing on a coffin or a grave (a) garland (b) wreath (c) bunch (d) bouquet SSC CGL (Tier-I) – 07/06/2019 (Shift-II) Ans : (b) The given group of words 'Flowers or leaves woven together in a circle for placing on a coffin or a grave' is called 'wreath' One Word Substitution

The meaning of other options isGarland - Coronal, wreath Bunch - Clump, cluster a grouping of a number of similar thing Bouquet - An arrangement of flowers that is usually given as present. 581. Select the most appropriate one word substitution for the given group of words. A man who has magical powers or skills (a) Amateur (b) Wizard (c) Agent (d) Scientist SSC GD – 14/02/2019 (Shift-I) Ans. (b) : The above group of words 'A man who has magical powers or skills' is called 'wizard' The meaning of other options isAmateur - Nonprofessional Agent - Broker, factor Scientist - A person with advanced knowledge of one or more scions 582. Select the most appropriate one word substitution for the given group of words. Very strange and unusual, unexpected, or not natural (a) weird (b) wizard (c) altruist (d) atheist SSC MTS – 19/08/2019 (Shift-II) Ans. (a) : The above group of words 'Very strange and unusual, unexpected or not natural' is called 'weird' The meaning of other options isWizard - One who practices magic or sorcery Altruist - Philanthropist Atheist - Someone who denies the existence of god. 583. Select the most appropriate one word substitution for the given group of words. Feeling or Showing caution about possible dangers. (a) ambiguous (b) wary (c) genius (d) museum SSC CPO-SI – 12/12/2019 (Shift-II) Ans. (b) : The above group of words 'feeling or showing caution about possible dangers' is called 'wary' The meaning of other options isAmbiguous - Having more than one possible meaning. Genius - Brilliance. Museum - A depository for collecting and displaying objects having scientific or historical

X 584. Select the most appropriate one word substitution for the given group of words. Dislike or prejudice against people from other countries (a) xenophile (b) xenophobia (c) ambivalent (d) skillfull SSC CGL (Tier-I) – 13/06/2019 (Shift-I) Ans. (b) : The above group of words 'Dislike or prejudice against people from other countries' is called 'xenophobia. The meaning of other options isxenophile - One attracted to foreign things. Ambivalent - Uncertain, unsure Skillful - Adept, expert, practiced.

645

YCT

585. Select the most appropriate one word Ans. (c) : The given group of words 'The highest substitution for the given group of words. point' is called 'zenith' of the realtion between a host and guest, Horizon - The line at which the sky and earth appera to friendly meet (a) xenial (b) harbour Paradise - Any place of complete bliss and delight and (c) agenda (d) summary peace (heaven) SSC CHSL (Tier-I) 15/10/2020 (Shift-I) Nadir - An extreme state of adversity Ans. (a) : The above group of words 'of the relation 590. Select the most appropriate one word between a host and guest, friendly' is called 'xenial' substitution for the given group of words. The meaning of other options is Harbour - Seaport, haven Great energy or enthusiasm in pursuit of a Agenda - A list of matters to be taken up cause Summary -A brief statement that presents the main (a) chronology (b) journal points in a concise form. (c) zeal (d) jealous SSC CPO-SI – 23/11/2020 (Shift-I) Ans. (c) : The above group of words 'Great energy or 586. Select the most appropriate one word enthusiasm in pursuit of a cause' is called 'zeal' The meaning of other options issubstitution for the given group of words. chronology - An arrangement of events in time Have an intense feeling for something journal - Diary (a) yearn (b) Proficient (c) adamant (d) adorable jealous - Green eyed, over jealous SSC MTS – 08/08/2019 (Shift-I) 591. Select the most appropriate one word Ans. (a) : The above group of words 'have an intense substitution for the given group of words. feeling for something' is called 'yearn' A person who is fanatical The meaning of other options is(a) zealot (b) director Proficient - Having or showing knowledge and skill and (c) composer (d) charter aptitude. SSC MTS 10-10-2017 (Shift-I) Adamant - Determined not to change your mind Ans. (a) : The above group of words 'A person who is Adorable - Very attractive and easy to feel love for. 587. Select the most appropriate one word fanatical' is called 'zealot' The meaning of other options issubstitution for the given group of words. An amount produced of an agricultural or Director - One of a group of senior managers run a industrial product company. (a) precise (b) yield Composer - A person who writhe music especially (c) native (d) visual classical music SSC CPO-SI (Tier-II) 27/09/2019 (3 pm - 5 pm) Charter - Grant a charter to. Ans. (b) : The above group of words 'An amount 592. Select the most appropriate one word produced of an agricultural or industrial product' is substitution for the given group of words. called 'yield' A line or course having abrupt right and left The meaning of other options isturns Precise - Snarply exact or accurate. (a) reliable (b) zigzag Native - Inborn (c) barrel (d) analogy Visual - Relating to or using sight (optical) SSC CHSL (Tier-I) 26/10/2020 (Shift-I) 588. Select the most appropriate one word substitution for the given group of words. Ans. (b) : The above group of words 'A line or course Shout in a loud, shard way. having abrupt right and left turns' is called 'zigzag' (a) clutch (b) yell The meaning of other options is(c) arena (d) charter Reliable - Worthy of reliance or trust (certain) SSC CHSL (Tier-I) –03/07/2019 (Shift-III) Barrel - A bulging cylindrical shape, hollow with flat Ans. (b) : The above group of words 'shout in a loud, ends. sharp way' is called 'yell' 593. Select the most appropriate one word The meaning of other options issubstitution for the given group of words. Clutch - Take hold of Love for foreign people, manners or cultures Arena - A large structure for open air sport or (a) xenophobia (b) xenophilia entertainment. (c) analogy (d) theology Charter - A contract to hire or lease transportation. SSC CPO-SI – 24/11/2020 (Shift-I) Ans. (b) : The above group of words 'Love for foreign people, manners or cultures' is called 'xenophila' 589. Choose the most appropriate word that can The meaning of other options is substitute the given group of words. Xenophobia - A fear of foreigners or strangers. The highest point Analogy - Drawing a comparison is order to show a (a) Horizon (b) Paradise similarity in some respect. (c) Zenith (d) Nadir SSC MTS – 06/08/2019 (Shift-I) Theology - Divinity

Y

Z

One Word Substitution

646

YCT

10. IDIOM & PHRASE TCS hewšve& hej DeeOeeefjle (Based On TCS Pattern) Chapterwise

Exam

Question No.

1

CGL (Tier-1) CGL (Tier-2) CHSL (Tier-1) CHSL (Tier-2) Selection Post VII, VIII, XI SSC MTS SSC GD SSC CPO SI CGL (Tier-1) CGL (Tier-2) CHSL (Tier-1) CHSL (Tier-2) Selection Post VII, VIII, XI SSC MTS SSC GD SSC CPO SI CGL (Tier-1) CGL (Tier-2) CHSL (Tier-1) CHSL (Tier-2) Selection Post VII, VIII, XI SSC MTS SSC GD SSC CPO SI CGL (Tier-1) CGL (Tier-2) CHSL (Tier-1) CHSL (Tier-2) Selection Post VII, VIII, XI SSC MTS SSC GD SSC CPO SI CGL (Tier-1) CGL (Tier-2) CHSL (Tier-1) CHSL (Tier-2) Selection Post VII, VIII, XI SSC MTS SSC GD SSC CPO SI

12 5 14 8 7 11 1 2 3 2 1 1 2 5 1 4 7 8 6 6 4 5 2 5 5 3 9 5 5 5 2 5 7 1 6 5 5 9 1 7

CGL (Tier-1) CGL (Tier-2) CHSL (Tier-1) CHSL (Tier-2)

4 2 3 3

A

2

B

3

C

4 D

5

E

6

Idoms & Phrases

647

Years

(2017–2023)

(2017–2023)

(2017–2023)

(2017–2023)

(2017–2023)

(2017–2023) YCT

F

7

G

8

H

9

I

10

J

11

K

12 L

13

Idoms & Phrases

Selection Post VII, VIII, XI SSC MTS SSC GD SSC CPO SI CGL (Tier-1) CGL (Tier-2) CHSL (Tier-1) CHSL (Tier-2) Selection Post VII, VIII, XI SSC MTS SSC GD SSC CPO SI CGL (Tier-1) CGL (Tier-2) CHSL (Tier-1) CHSL (Tier-2) Selection Post VII, VIII, XI SSC MTS SSC GD SSC CPO SI CGL (Tier-1) CGL (Tier-2) CHSL (Tier-1) CHSL (Tier-2) Selection Post VII, VIII, XI SSC MTS SSC GD SSC CPO SI CGL (Tier-1) CGL (Tier-2) CHSL (Tier-1) CHSL (Tier-2) Selection Post VII, VIII, XI SSC MTS SSC GD SSC CPO SI CGL (Tier-1) CGL (Tier-2) CHSL (Tier-1) CHSL (Tier-2) Selection Post VII, VIII, XI SSC MTS SSC GD SSC CPO SI CGL (Tier-1) CGL (Tier-2) CHSL (Tier-1) CHSL (Tier-2) Selection Post VII, VIII, XI SSC MTS SSC GD SSC CPO SI CGL (Tier-1) CGL (Tier-2) CHSL (Tier-1) 648

4 5 1 5 2 1 3 1 1 1 2 2 1 2 1 1 4 1 4 5 1 6 2 8 5 3 2 1 1 1 1 1 1 1 1 2 1 2 1 1 5 1 3 3 3 5

(2017–2023)

(2017–2023)

(2017–2023)

(2017–2023)

(2017–2023)

(2017–2023)

YCT

M

14

N

15 O

16 P

17 Q

18

R

19

S

Idoms & Phrases

CHSL (Tier-2) Selection Post VII, VIII, XI SSC MTS SSC GD SSC CPO SI CGL (Tier-1) CGL (Tier-2) CHSL (Tier-1) CHSL (Tier-2) Selection Post VII, VIII, XI SSC MTS SSC GD SSC CPO SI CGL (Tier-1) CGL (Tier-2) CHSL (Tier-1) CHSL (Tier-2) Selection Post VII, VIII, XI SSC MTS SSC GD SSC CPO SI CGL (Tier-1) CGL (Tier-2) CHSL (Tier-1) CHSL (Tier-2) Selection Post VII, VIII, XI SSC MTS SSC GD SSC CPO SI

2 3 5 1 5 1 1 2 1 2 4 3 2 5 2 5 3 1 2 8 2 6 8 5 5 2 5

CGL (Tier-1) CGL (Tier-2) CHSL (Tier-1) CHSL (Tier-2) Selection Post VII, VIII, XI SSC MTS SSC GD SSC CPO SI CGL (Tier-1) CGL (Tier-2) CHSL (Tier-1) CHSL (Tier-2) Selection Post VII, VIII, XI SSC MTS SSC GD SSC CPO SI CGL (Tier-1) CGL (Tier-2) CHSL (Tier-1) CHSL (Tier-2) Selection Post VII, VIII, XI SSC MTS SSC GD SSC CPO SI

1 1 1 1 1 1 4 2 6 5 5 2 2 5 10 8 14 5 1 4 2 4

649

(2017–2023)

(2017–2023)

(2017–2023)

(2017–2023)

(2017–2023)

(2017–2023)

(2017–2023)

YCT

20

T

21

U

22

V

23

W

24

X

25

Y

26

Z

Idoms & Phrases

CGL (Tier-1) CGL (Tier-2) CHSL (Tier-1) CHSL (Tier-2) Selection Post VII, VIII, XI SSC MTS SSC GD SSC CPO SI CGL (Tier-1) CGL (Tier-2) CHSL (Tier-1) CHSL (Tier-2) Selection Post VII, VIII, XI SSC MTS SSC GD SSC CPO SI CGL (Tier-1) CGL (Tier-2) CHSL (Tier-1) CHSL (Tier-2) Selection Post VII, VIII, XI SSC MTS SSC GD SSC CPO SI CGL (Tier-1) CGL (Tier-2) CHSL (Tier-1) CHSL (Tier-2) Selection Post VII, VIII, XI SSC MTS SSC GD SSC CPO SI CGL (Tier-1) CGL (Tier-2) CHSL (Tier-1) CHSL (Tier-2) Selection Post VII, VIII, XI SSC MTS SSC GD SSC CPO SI CGL (Tier-1) CGL (Tier-2) CHSL (Tier-1) CHSL (Tier-2) Selection Post VII, VIII, XI SSC MTS SSC GD SSC CPO SI CGL (Tier-1) CGL (Tier-2) CHSL (Tier-1) CHSL (Tier-2) Selection Post VII, VIII, XI SSC MTS SSC GD SSC CPO SI 650

5 3 2 3 2 4 1 1 1 1 2 1 1 1 1 1 1 1 1 1 1 2 1 1 1 1 1 1 2 1 1 1 1 1 1 1 1 1 1 1 1

(2017–2023)

(2017–2023)

(2017–2023)

(2017–2023)

(2017–2023)

(2017–2023)

(2017–2023)

YCT

Trend Analysis of Questions topicwise from CGL (Pre & Mains) CHSL (Pre & Mains) Selection Post VII, VIII, XI, SSC MTS, SSC GD & Other Exams (2017-2023)

Idoms & Phrases

651

YCT

10. IDIOMS AND PHRASES A 1.

Select the most appropriate meaning of the given idiom. against the clock (a) To take on a task that is a way to big (b) On the point of achievement (c) Rushed and short on time (d) To avoid talking SSC MTS 04/05/2023 (Shift-II) Ans. (c) : The most appropriate meaning of the given idiom 'against the clock' is 'Rushed and short on time.' other options are incorrect. Thus, the correct answer is option (c). 2. In the following question, out of the given four alternatives, select the alternative which best expresses the meaning of the Idiom/Phrase. A blast from the past (a) Something powerfully nostalgic (b) A competition where the outcome is difficult to judge or predict (c) An unresolved argument (d) Punishment for the crime committed in the past SSC MTS 04/05/2023 (Shift-II) Ans. (a) : The most appropriate meaning of the idiom/Phrase 'A blast from the past' is 'something powerfully nostalgic'. while other options are incorrect. Thus, the correct answer is option (a). 3. In the following question, out of the given four alternatives, select the alternative which best expresses the meaning of the Idiom/Phrase. A blind spot (a) A person who is blind (b) A situation where no progress is made (c) A person with no emotional feelings (d) An area in your range of vision that your cannot see properly SSC MTS 02/05/2023 (Shift-I) Ans. (d) : For the above given idiom/phrase 'A blind spot' will be chosen meaning 'An area in your range of vision that your cannot see properly. Thus, the correct answer is option (d). 4. In the following question, out of the given four alternatives, select the alternative which best expresses the meaning of the Idiom/Phrase. At all costs (a) Whatever is needed to achieve something (b) An expensive purchase (c) Not to be trifled with (d) To be master of SSC MTS 18/05/2023 (Shift-I) Idoms & Phrases

Ans. (a) : The best express meaning of the given idiom/phrase ‘At all costs’ is ‘whatever is needed to achieve something’. Other options give different meanings. 5. Select the most appropriate meaning of the given idiom. A night owl (a) Someone who has good eyesight (b) Someone who works very hard (c) Someone who never works (d) A person who prefers to be awake late at night SSC GD 10/01/2023 (Shift-I) Ans. (d) : The most appropriate meaning of the given idiom 'A night owl' is 'A person who prefers to be awake late at night'. Other options are not suitable. 6. Select the most appropriate meaning of the given idiom. At your beck and call (a) To have absolute control over someone (b) To call someone respectfully (c) To have no control over someone (d) To call someone and then deny SSC GD 10/01/2023 (Shift-I) Ans. (a) : The most appropriate meaning of the given idiom 'At your beck and call' is 'To have absolute control our someone.' Other options are not suitable. 7. Select the most appropriate meaning of the given idiom. A perfect storm (a) Appropriate for all occasions (b) The worst possible situation (c) Healing from trauma (d) Pretty good judgment SSC CGL (Tier-I) 21/07/2023 (Shift-II) Ans. (b) : The most appropriate meaning of the above given idiom 'A perfect storm' is 'The worst possible situation.' Other options give different meanings. 8. Identify the option that can be substituted as the correct idiom for the underlined part of the given sentence. My cousin sister Neetu had an aerial view of the trade fair from the top of the giant wheel. (a) A bird inteh gilded cage (b) Bird brain (c) Birds of same feather (d) Bird's eye view SSC CGL (Tier-I) 27/07/2023 (Shift-III)

652

YCT

Ans. (d) : The correct substitute idiom for the underlined part of the given sentence 'aerial view' is option (d) 'Bird's eye view' which means 'an overall or cursory look at something.' Other options give different meanings. Thus, the correct answer is option (d). 9. Select the sentence that uses the given idiom correctly. A dime a dozen (a) Her ideas were worth a dime a dozen and didn't impress the boss. (b) The restaurant had some delicious desserts that were a dime a dozen. (c) The antique store had some rare finds, but they were a dime a dozen. (d) The concert tickets were a dime a dozen and sold out quickly. SSC CGL (Tier-I) 17/07/2023 (Shift-II) Ans. (a) : For the above idiom, 'A dime a dozen' option (a) 'Her ideas were work a dime a dozen and didn't impress the boss' gives appropriate meaning other options give different meaning. 10. Select the most appropriate meaning of the given idiom. a narrow escape (a) Likely to fall in the near futher (b) Do something very risky (c) Just manage to avoid danger or trouble (d) Only just succeed in doing something SSC CHSL (Tier-I) 14/08/2023 (Shift-IV) Ans. (c) : The given idiom 'a narrow escape' means 'just mange to avoid danger or trouble' mentioned in option (c). other options contain different meanings. 11. In the following question, out of the given four alternatives, select the alternative which best expresses the meaning of the Idiom/Phrase. 'As hard as nails' (a) To be stubborn (b) To feel no pain (c) Physically or mentally tough (d) Hard working SSC CHSL (Tier-I) 21/03/2023 (Shift-IV) Ans. (c) : For the above given idiom/phrase "As hard as nails" is option (c) 'Physically or mentally tough' gives an appropriate explanation. Other options give different meaning. 12. Select the most appropriate meaning of the given idiom. A cock and bull story (a) A true story (b) A children’s story (c) A story of animals (d) A foolish and concocted story SSC Selection Posts XI-28/06/2023 (Shift-III) Ans. (d) : For the above Idiom, "A cock and bull story" option (d) 'A foolish and concocted story' gives appropriate meaning. Other options give different meaning. Idoms & Phrases

13.

In the following question, out of the given four alternatives, select the alternative which best expresses the meaning of the idiom/phrase. A bite at the cherry (a) A wrong move (b) An attempt or opportunity to do something (c) to have a leverage over someone (d) A dog bite SSC CHSL (Tier-I) 17/03/2023 (Shift-IV) Ans. (b) : For the above idiom, 'A bite at the cherry' option (b) An attempt or opportunity to do something" gives suitable meaning. Other options are not correct. 14. In the following question;, out of the given four alternatives, select the alternative which best expresses the meaning of the idiom/phrase. Appeal to Caesar (a) An authority given to a foolish person (b) To expect good from a wrong person (c) To live in a false reality (d) Appeal to the highest possible authority SSC CHSL (Tier-I) 17/03/2023 (Shift-III) Ans. (d) : The given idiom, 'Appeal to Caesar' option (d) 'Appeal to the highest possible authority' gives an appropriate explanation. Other options give different meaning. 15. Select the most appropriate meaning of the given idiom. All bark and no bite (a) Someone who sticks to his words (b) Actions are stronger than mere words (c) Not being able to understand anything (d) Full of talk, but low on action SSC CHSL (Tier-I) 17/03/2023 (Shift-III) Ans. (d) : For the above idiom, 'All bark and no bite' is option (d) Full of talk, but low on action gives an appropriate explanation. Other option give different mening. 16. Select the most appropriate meaning of the given idiom. Ace in the hole (a) To create noisy disturbances so that competitors are unable to work. (b) A major advantage that one keeps hidden until an ideal time (c) To attempt to do the impossible (d) To suffer an injury on the lower body SSC CHSL (Tier-I) 10/03/2023 (Shift-III) Ans. (b) : For the above Idiom, "Ace in the hole" option (b) "A major advantage that one keeps hidden until an ideal time" gives an appropriate explanation. Other options give different meaning. 17. Select the most appropriate meaning of the underlined idiom. When those thugs come near you, run at the drop of a hat. (a) Be dressed for a party (b) With all your belongings (c) Go on a shot trip (d) Without delay SSC MTS-08/07/2022 Shift-I

653

YCT

Ans. (d) : For the above underlined idiom' at the drop of a hat', option (d) 'without delay' gives suitable meaning. Other options are not correct. 18. Select the most appropriate meaning of the given idiom. An eye wash (a) In writing (b) A pretence (c) Avoid talking (d) A rare event SSC MTS-06/07/2022 Shift-II Ans. (b) : For the above idiom 'An eye wash', option (b) 'A pretence' gives suitable meaning. Other options are not correct. 19. Select the option that gives the most appropriate meaning of the underlined idiom. You have put me in a tight corner by telling me to keep quiet. (a) An embarrassing situation (b) A secret place (c) A comfortable situation (d) A difficult situation SSC CGL (Tier-I) 13/04/2022 Shift-III Ans. (d) : For the above idiom 'A tight corner', option (d) A difficult situation gives suitable meaning. Other options are not correct. 20. Select the most appropriate meaning of the given idiom. A long shot (a) A perfect aim (b) Little chance of success (c) Very profitable (d) A correct prediction SSC CGL (Tier-I) 12/04/2022 Shift-II Ans. (b) : For the above idiom. A long shot', option (b) little chance of success gives suitable meaning other options are not correct. 21. Select the most appropriate meaning of the given idiom. A hair in the butter (a) A slippery road (b) Easy to handle (c) Sinking in debt (d) A challenging situation SSC CGL (Tier-I) 18/04/2022 Shift-I Ans. (d) : For the above idiom 'A hair in the butter', option (d) 'A challenging situation' gives suitable meaning. Other options are not correct. 22. Select the most appropriate meaning of the given idiom. A snake in the grass (a) An unfortunate accident (b) A hidden enemy (c) A sudden death (d) A good fortune SSC CGL (Tier-I) 19/04/2022 Shift-III Ans. (b) : For the above idiom 'A snake in the grass' option (b) 'A hidden enemy' gives suitable meaning. Other options are not correct. 23. Select the most appropriate meaning of the given idiom. A snake in the grass (a) A deceitful person (b) A waealthy person (c) A timid person (d) A talkative person SSC CHSL 25.05.2022 Shift-I Idoms & Phrases

Ans. (a) : For the above idiom 'A snake in the grass', option (a) A deceitful person gives suitable meaning. Other options are not correct. 24. Select the most appropriate meaning of the given idiom. At one's elbow (a) Next to someone (b) Strong grip (c) Far away (d) Strong bond SSC CHSL 24.05.2022 Shift-III Ans. (a) : For the above idiom 'At one's elbow', option (a) next to someone gives suitable meaning, other options are not correct. 25. Select the most appropriate meaning of the given idiom. All ears (a) trying to listen (b) not able to listen (c) not wanting to listen (d) ready to listen SSC Stenographer (Grade C & D) 15.11.2021 Shift-I Ans. (d) : For the above idiom 'All ears', option (d) ready to listen gives suitable meaning. Other option are not correct. 26. Select the most appropriate meaning of the given idiom. At a snail's pace (a) carelessly (b) peacefully (c) very slowly (d) rapidly SSC Stenographer (Grade C & D) 11.11.2021 Shift-II Ans. (c) : For the above idiom 'At a snail's pace' option (c) very slowly gives suitable meaning. Other options are not correct. 27. Select the most appropriate meaning of the given idiom. A bad egg (a) Someone who regularly makes mistakes (b) Someone who is dishonest and unreliable (c) Someone who doesn't like eggs (d) Someone who likes to break eggs SSC CGL (Tier-I) –20/08/2021 (Shift-I) Ans. (b) : The above idiom, ‘a bad egg’ option (b) 'someone who is dishonest and unreliable' gives an appropriate explanation. Other options have different meaning. 28. Select the most appropriate meaning of the underlined idiom in the given sentence.] Her success as a singer was a nine days' wonder. (a) a proud achievement (b) eternal fame (c) an impossible feat (d) a short-lived sensation SSC CGL (Tier-II) 16/11/2020 (3 pm - 5 pm) Ans. (d) : The above idiom, “a nine days wonder” option(d) a short-lived sensation gives an appropriate explanation. Other options have different meaning. 29. Select the most appropriate meaning of the underlined idiom in the given sentence. She felt like a fish out of water at her new job. (a) happy and free (b) uncomfortable and restless (c) angry and hurt (d) comfortable and relaxed SSC CGL (Tier-II) 16/11/2020 (3 pm - 5 pm)

654

YCT

Ans. (b) : The above idiom, "like fish out of water" option (b) 'uncomfortable and restless' gives an appropriate explanation. Other options have different meaning. 30. Select the most appropriate meaning of the underlined idiom in the given sentence. The apple of discord among the brothers was their father's mansion in the country. (a) reason for quarrel (b) hopeful attention (c) Sincere affection (d) Fruitful discussion SSC CPO-SI 25/11/2020 (Shift II) SSC CPO-SI (Tier-II) 27/09/2019 (3 pm - 5 pm) Ans. (a) : The above idiom, ‘The apple of discord’ option (a) reason for quarrel gives an appropriate explanation. Other options have different meaning. 31. Select the most appropriate meaning of the given idiom. A bird's eye view (a) a cursory look (b) a detailed study (c) a prejudiced view (d) a personal view SSC CPO-SI – 25/11/2020 (Shift-I) Ans. (a) : The above idiom, ‘A bird's eye view’ option (a) 'a cursory look' gives an appropriate explanation. Other options have different meaning. 32. Select the most appropriate meaning of the underlined idiom in the given sentence. Throughout his speech the crowd was all ears. (a) making a lot of noise (b) covering their ears (c) very attentive (d) talking loudly SSC CGL (Tier-I) – 12/06/2019 (Shift-II) Ans : (c) The above idiom, "All ears" option for (c) 'very attentive' gives an appropriate explanation. Other options have different meaning. 33. Select the most appropriate meaning of the underlined idiom in the given sentence. Coming from an affluent family, she found herself a square peg in a round hole when she married a poor farmer and moved to a small village. (a) in a favorable actuation (b) unhappy and regretful (c) a misfit in the environment (d) in a financial crisis SSC CGL (Tier-I) – 10/06/2019 (Shift-II) Ans : (c) The above idiom/Phrase ‘a square peg in a round hole’ option (c) ‘a misfit in the environment’ give an appropriate explanation. Other options have different meaning. 34. Select the most appropriate meaning of the given idiom. At daggers drawn (a) deceiving somebody (b) bitterly hostile (c) friendly with each other (d) without hope SSC CGL (Tier-I) – 04/06/2019 (Shift-I) Ans : (b) The above idiom/phrase 'At daggers drawn' (b) ‘bitterly hostile’ gives an appropriate explanation. Other options have different meaning. Idoms & Phrases

35.

Select the most appropriate meaning of the given idiom. Actions speak louder than words (a) what you do is more important than what you say (b) take up a task that you cannot finish (c) look for solutions in the wrong place (d) do something without planning SSC CGL (Tier-I) – 04/06/2019 (Shift-III) Ans. (a) : The above idiom/phrase ‘Action speak Louder than words’ option (a) what you do is more important than what you 'say' gives an appropriate explanation. Other options have different meaning. 36. Select the most appropriate meaning of the underlined idiom in the given sentence. As the flood waters hit the coastal village, several families had a close shave. (a) a narrow escape (b) to have no livelihood (c) to loss all possessions (d) nothing to eat SSC CPO-SI – 11/12/2019 (Shift-II) Ans : (a) The above idiom/phrase "A close shave" option (a) ‘A narrow escape’ gives an appropriate explanation, other options have different meaning. 37. Select the most appropriate meaning of the underlined idiom in the given sentence. He is completely at sea about where to invest his hard-earned money. (a) confused (b) sure (c) in shock (d) committed SSC CPO-SI – 11/12/2019 (Shift-II) Ans : (a) The above idiom/phrase "At sea" option (a) ‘confused’ gives an appropriate explanation. Other options have different meaning. 38. Select the most appropriate meaning of the underlined idiom in the given sentence. Unless you are above board in your dealings, you will not be able to win the trust of your clients. (a) smart and deceptive (b) bold and reckless (c) honest and frank (d) exceptionally talented SSC CPO-SI – 11/12/2019 (Shift-II) Ans : (c) The above idiom/phrase 'Above board'option (c) ‘Honest and Frank’ gives an appropriate explanation. Other options have different meaning. 39. Select the most appropriate meaning of the given idiom. At the drop of a hat (a) Instantly (b) Usually (c) Frequently (d) Normally SSC MTS-07/08/2019(Shift-III) SSC CPO-SI – 11/12/2019 (Shift-I) Ans : (a) The above idiom/phrase "At the drop of a hat" (a) 'instantly' gives an appropriate meaning. Other option have different meaning. 40. Select the most appropriate meaning of the given idiom. Be an open book (a) one about whom it is easy to know everything (b) one who advises others to be frank and unsecretive

655

YCT

(c) one who is fond of reading even when travelling (d) one who has written an examination without preparation SSC CHSL (Tier-I) –11/07/2019 (Shift-II) Ans : (a) The above idiom/phrase ‘Be an open book’ option (a) one about whom it is easy to know everything gives appropriate meaning. Other options have different meaning. 41. Select the most appropriate meaning of the given idiom at snail's pace (a) do something very carefully (b) be very persistent (c) do something very slowly (d) keep your moves secret SSC CHSL (Tier-I) –10/07/2019 (Shift-II) Ans : (c) The above idiom/phrase "At snail's Pace" option (c) do something very slowly gives an appropriate explanation. Other options have different meaning. 42. Select the most appropriate meaning of the given idiom at arm's length (a) checking the length of fabric (b) as close as one can touch (c) avoiding too much closeness (d) using one's arm to measure length SSC CHSL (Tier-I) –09/07/2019 (Shift-I) Ans : (c) For the above idiom/Phrase 'At arm's length' option (c) ‘avoiding too much closeness’ gives in appropriate meaning. Other options have different meaning. 43. Select the most appropriate meaning of the given idiom acid test (a) a malicious way of taking revenge (b) a test that shows how to harm someone (c) proof that an attack has been made (d) definitive proof of truth or falsehood SSC CHSL (Tier-I) –09/07/2019 (Shift-I) Ans : (d) For the above idiom/phrase “Acid test” option (d) definitive proof of truth or falsehood gives an appropriate meaning. Other options have different meaning. 44. Select the most appropriate meaning of the given idiom. As the crow flies (a) to be uncertain (b) a long winding path (c) the shortest route (d) to be directionless SSC CHSL (Tier-I) –08/07/2019 (Shift-I) Ans : (c) For the above idiom/phrase "As the crow flies" Option (c) 'the shortest route' gives an appropriate explanation. Other options have different meaning. 45. Select the most appropriate meaning of the given idiom. As clear as mud (a) impossible to understand (b) extremely dirty (c) honest and legal (d) completely suitable for someone SSC CHSL (Tier-I) –02/07/2019 (Shift-III) Idoms & Phrases

Ans : (a) For the above idiom/phrase 'As clear as mud' option (a) 'impossible to understand' gives an appropriate explanation. Other options have different meaning. 46. Select the most appropriate meaning of the given idiom. A piece of cake (a) Something amusing (b) Something serious (c) Something difficult (d) Something easy SSC CHSL (Tier-I) 12/10/2020 (Shift-II) Ans. (d) : For the above idiom/phrase "A piece of cake" option (d) 'something easy' gives an appropriate explanation other options have different meaning. 47. Select the most appropriate meaning of the given idiom. Add insult to injury (a) Hurt someone physically (b) Say words that are very offensive (c) Play a prank on someone (d) Make a bad situation even worse. SSCHSL (Shift-I) 06/08/2021 (Shift-I) SSC CHSL (Tier-I) 14/10/2020 (Shift-II) Ans. (d) : For the above idiom/phrase "Add insult to injury" option (d) 'make a bad situation even worse' gives an appropriate meaning. Other options have different meaning. 48. In the following question, out of the given four alternatives, select the alternative which best expresses the meaning of the Idiom/Phrase. A bone of contention. (a) Strong and healthy bones (b) Tough man (c) Cause of dispute (d) Peaceful thing SSC MTS 9-10-2017 (Shift-I) Ans : (c) For the above idiom/phrase "A bone of contention" option (c) 'Cause of dispute' gives an appropriate meaning. Other options have different meaning. 49. In the following question, out of the four given options, select the alternative which means the idiom / phrase. A child's play (a) Very innocent (b) Very annoying (c) Something that is very easy (d) Very irritating SSC MTS 9-10-2017 (Shift-III) Ans : (c) For the above idiom ‘A child's play’ option (c) something that is very easy gives an appropriate explanation. Other options have different meaning. 50. In the following question, out of the four given alternatives, select the alternatives which best express the meaning of the Idiom/Phrase. A burning question (a) A lengthy question (b) An important question (c) A useless question (d) An easy question SSC MTS 10-10-2017 (Shift-II)

656

YCT

Ans. (b) : For the above Idiom/Phrase ‘A burning Question’ means ‘An important question’. Thus, the correct answer is option (b). 51. Select the meaning of the given idiom. Be at your wits' end (a) To be overjoyed (b) Careful thinking (c) Be overwhelmed with problems (d) Do something confidently SSC MTS – 09/08/2019 (Shift-I) Ans. (c) : For the above idiom/phrase "Be at your wits end' option (c) be overwhelmed with problems gives an appropriate explanation. Other options have different meaning. 52. Select the meaning of the given idiom. Apples and oranges. (a) Same things (b) Different things (c) Fresh (d) Nutritious SSC MTS – 08/08/2019 (Shift-III) Ans. (b) : For the above idiom/phrase "Apple and oranges option (b) 'Different things' gives an appropriate explanation. Other options have different meaning. 53. Select the most appropriate meaning of the given idiom. As a matter of fact (a) Actually as it happens (b) To seize a good opportunity (c) To forgive and forget (d) just at the last moment SSC MTS – 06/08/2019 (Shift-I) Ans. (a) : For the above idiom/phrase "As matter of fact "Option (a) 'Actually as it happens' gives an appropriate explanation. Other options have different meaning. 54. Select the most appropriate meaning of the given idiom. Above one's head (a) Intimate and personal (b) Beyond one's understanding (c) To be very tall (d) Overhead expenses SSC MTS – 05/08/2019 (Shift-II) Ans. (b) : For the above idiom/phrase "Above one's head" option (b) 'beyond one's understanding' gives an appropriate explanation. Other options have different meaning. 55. Select the most appropriate meaning of the given idiom. A penny for your thoughts (a) Start something all over again (b) Be glad to see someone leave (c) Way of asking what someone is thinking (d) To make a bad situation worse SSC MTS – 21/08/2019 (Shift-III) Ans. (c) : For the above idiom/phrase "A penny for your thought" option (c) 'way of asking what someone is thinking' gives an appropriate explanation other options have different meaning. 56. Select the most appropriate meaning of the given idiom. A bull in a china shop Idoms & Phrases

An extremely awkward, clumsy person A very disciplined and systematic person A brave person in a group of cowards To be a rude and aggressive person SSC MTS – 20/08/2019 (Shift-I) Ans. (a) : For the above idiom/phrase "A bull in a china shop" option (a) An extremely awkward, clumsy person gives an appropriate explanation. Other options have different meaning. 57. Select the most appropriate meaning of the given idiom. A change of heart (a) To have a heart transplant (b) To fail to fulfill a promise (c) To have a second opinion (d) A change in one's opinion SSC MTS – 19/08/2019 (Shift-II) Ans. (d) : For the above idiom/phrase "A change of heart" option (d) A change in one's opinion gives an appropriate explanation. Other option have different meaning. 58. Select the most appropriate meaning of the given idiom. A needle in a haystack (a) To be of minor importance (b) To be involved in unnecessary activity (c) To create a state of confusion (d) Something that is very difficult to locate SSC MTS – 14/08/2019 (Shift-I) Ans. (d) : For the above idiom/phrase 'A needle in a haystack' option (d) 'something that is very difficult to locate' gives an appropriate explanation. Other options have different meaning. 59. Select the meaning of the given idiom. All and sundry (a) By all means (b) All eager (c) All included (d) All in agreement SSC MTS – 09/08/2019 (Shift-II) Ans. (c) : For the above idiom/phrase "All and sundry" option (c) 'All include' gives an appropriate explanation other options have different meaning. 60. Select the most appropriate meaning of the given idiom. As clear as a bell (a) To have a tinkling sound (b) At a very low price (c) To get fooled (d) Readily understood SSC MTS – 07/08/2019 (Shift-III) Ans. (d) : For the above idiom/phrase 'As clear as a bell' option (d) ‘Readily understood’ gives an appropriate explanation. Other options have different meaning. 61. Select the meaning of the given idiom. A man of spirit is (a) A spiritual person (b) A unique person (c) A very talented man (d) A very courageous man SSC GD – 08/03/2019 (Shift-II) Ans. (d) : For the above idiom/phrase "A man of spirit is" option (d) A very courageous man gives an appropriate explanation. Other options have different meaning.

657

(a) (b) (c) (d)

YCT

Select the option that means the same as the 66. given idiom. A stone's throw away (a) Very far off (b) At a short distance (c) Many miles away (d) At a remote location SSC GD – 18/02/2019 (Shift-III) Ans. (b) : For the above idiom/phrase 'A stone's throw away' option (b) At a short distance gives an appropriate explanation. Other options have different meaning.

62.

B 63.

In the following question, out of the given four alternatives, select the alternative which best expresses the meaning of the Idiom/Phrase. blessing in disguise (a) Hear a good news after a long time (b) A misfortune that unexpectedly turns into good fortune (c) To have a dispute with someone (d) To go from being very poor to being very wealthy SSC MTS 19/05/2023 (Shift-III) Ans. (b) : The appropriate meaning of the above given idiom ‘blessing in disguise’ is ‘A misfortune that unexpectedly turns into good fortune’. Other options give different meanings. 64. Select the most appropriate meaning of the given idiom. Bear a grudge (a) To consider people with kindness and care (b) To feel angry against somebody for a past incident (c) To entertain the guests will utmost happiness (d) To feel satisfied with the achievements SSC GD 23/01/2023 (Shift-IV) Ans. (b) : The most appropriate meaning of the given idiom 'Bear a grudge' is 'To feel angry against somebody for a past incident'. The other options are not suitable. 65. Select the most appropriate meaning of the given idiom. Better the devil you know than the devil you don't. (a) Being cruel is better with known people. (b) It is easier to be in a known bad situation that being in an unkown worse situation. (c) Do not trust unknown ans known devils. (d) Being good is better with evil people. SSC GD 06/02/2023 (Shift-III) Ans. (b) : The most appropriate meaning of the given idiom– 'Better the devil you know than the devil you don't :- is 'It is easier to be in a known bad situation that being in an unknown worse situation', Hence options (b) is correct answer. Idoms & Phrases

Select the sentence that has the most appropriate meaning of the given idiom. Bad blood (a) Sushant always gets in trouble for telling lies but he never learns anything. (b) The gangster had some bitter relations with a local activist so he warned him to stay out of his business. (c) The doctor dropped the bad news to Ravi as he was suffering from a serious disease. (d) She is so scared of cockroaches that she fainted when she saw them crawling near her foot. SSC CGL (Tier-I) 21/07/2023 (Shift-II) Ans. (b) : The most appropriate meaning of the above given idiom 'Bad blood' is option (b) 'The gangster had same bitter relation with a local activist so he warned him to stay out of his business.' Other option give different meanings. 67. Select the most appropriate meaning of the given idiom. It turns out to be a blessing in disguise. (a) When things get better and better when least expecting it (b) Has a very good effect on a situation or a thing (c) When a simple solution solves a problem very well (d) Has a good effect even though at first it seemed it would be bad SSC CGL (Tier-I) 18/07/2023 (Shift-III) Ans. (d) : The most appropriate meaning of the given idiom 'a blessing in disguise is option (d) 'Has a good effect even though at first it seemed it would be bad'. Other options give different meanings. 68. Select the sentence which gives the most appropriate meaning of the given idiom. Bite the bullet (a) The children played in the park on a sunny day. (b) The chef prepared a delicious meal for the guests. (c) Despite the pain, Sarah decided to go ahead with the surgery. (d) Jack and Jill went up the hill to fetch a pail of water. SSC CGL Mains 26/10/2023 (Shift-I) Ans. (c) : For the above idiom, 'Bite the bullet' option (c) despite the pain, sarach decided to go ahead with the surgery gives an appropriate explanation. Other options give different meaning. 69. Select the most appropriate idiom that can substitute the underlined segment in the given sentence. Vinay missed his flight but good luck came to him from bad circumstances as the flight crashed in the sea. (a) It was a tinker's curse (b) It was a blessing and a curse (c) It was counting his blessings (d) It was a blessing in disguise SSC CGL (Tier-I) 24/07/2023 (Shift-III)

658

YCT

Ans. (d) : In the above sentence underlined idiom 'good luck came to him from bad circumstances' option (d) It was a "blessing in disguise" gives appropriate explanation. Other options give different meanings. 70. Select the most appropriate idiom for the following statement. The manager was punished for his negligence in work. (a) Bid defiance (b) Bring to book (c) Burn one’s boats (d) Bolt from the blue SSC CHSL (Tier-I) 17/08/2023 (Shift-II) Ans. (b) : The suitable idiom for the statement 'The manager was punished for his negligence in work would be option (b) bring to book. Idiom, Bring to book means to punish someone and make that person explain their behavior. Idioms given a options have below mentioned meanings – bid defiance – Defy and remain stubborn Burn one's boat – To cut oneself off from all chance of retreat Bolt from the blue – A sudden and unexpected event or piece of news. 71. Select the most appropriate meaning of the given idiom. A piece of information received by word of mouth has no authenticity. (a) in spoken form (b) in written form (c) through rumours (d) through an official order SSC Selection Posts XI-30/06/2023 (Shift-IV) Ans. (a) : For the above underlined Idiom, 'by word of mouth'. Option (a) 'in spoken form' gives appropriate explanation. Other options give different meaning. 72. Select the most appropriate meaning of the given idiom. Bite the bullet (a) To take a risk (b) To endure a painful situation (c) To criticise someone harshly (d) To enjoy something immensely SSC CHSL (Tier-I) 03/08/2023 (Shift-II) Ans. (b) : The most appropriate meaning of the above given idiom 'Bite the bullet' is option (b) 'To endure a painful situation'. Other options give different meaning. 73. Select the most appropriate meaning of the given idiom. He brought his servant to book for breaking the beautiful showpiece. (a) Scolded his servant (b) Relieved his servant (c) Rewarded his servant (d) Sent his servant away SSC Selection Posts XI-27/06/2023 (Shift-I) Ans. (a) : The most appropriate meaning of the above given idiom 'brought his servant to book' is 'Scolded his servant'. Other options give different meanings. Idoms & Phrases

74.

Select the most appropriate meaning of given idiom. Burn your bridges (a) To have a fight (b) To disclose a secret (c) To be puzzle (d) To eliminate the possibility of return or retreat SSC CHSL (Tier-I) 20/03/2023 (Shift-I) Ans. (d) : For the above Idiom, "Burn your bridges", option (d) "To eliminate the possibility to return or retreat". Other options are not correct. 75. Select the most appropriate meaning of the given idiom. 'Blessing is disguise' (a) A good deed rewards as a blessing (b) Bad thing that turns out to be good (c) A good thing introduced into your life as a blessing (d) Something that happens very rarely SSC CHSL (Tier-I) 15/03/2023 (Shift-III) Ans. (b) : For the above idiom, ''Blessing is disguise'' is option (b) ''Bad thing that turns out to be good'' gives an appropriate explanation. Other options give different meaning. 76.

Select the most appropriate meaning of the given idiom. Between Scylla and Charybdis (a) Winning a competition (b) Choice between two pleasant alternatives (c) Choice between two unpleasant alternatives (d) Dividing into equal shares SSC Constable GD-07/12/2021 Shift-II Ans. (c) : For the above idiom 'Between Scylla and charybdis', option (c) 'Choice between two unpleasant alternatives' gives suitable meaning. Other options are not correct.

77.

Select the most appropriate meaning of the given idiom : Blue blood (a) Noble birth (b) Rare sight (c) Bad mood (d) Weak person SSC Constable GD-25/11/2021 Shift-III Ans. (a) : For the above idiom 'Blue blood', option (a 'Noble birth' gives suitable meaning. Other options are not correct.

78.

Select the most appropriate meaning of the given idiom. Big bucks (a) A big loss (b) A big bucket (c) A big function (d) A lot of money SSC Constable GD-18/11/2021 Shift-I Ans. (d) : For the above idiom 'Big bucks', option (d) A lot of money' gives suitable meaning. Other options are not correct.

659

YCT

79.

Select the most appropriate meaning of the given idiom. Back on one's feet (a) To support the losing side (b) Relive previous moments (c) Well or successful again (d) Forced to begin something again SSC MTS-13/07/2022 Shift-I Ans. (c) : For the above idiom' Back on one's feet', option (c) well or successful again' gives suitable meaning other options are not correct.

84.

Select the most appropriate meaning of the given idiom. Below the belt (a) Without any effort (b) Very quickly (c) Cruel and unfair (d) Suspiciously SSC MTS-18/07/2022 Shift-I Ans. (c) : For the above idiom 'Below the belt', option (c) cruel and unfair' gives suitable meaning. Other options are not correct.

83.

88.

Select the most appropriate meaning of the highlighted idiom. The youth involved in the accident escaped by 80. Select the option that gives the most the skin of his teeth. appropriate meaning of the underlined idiom. (a) Always taking calculated decisions I am working on too many projects. I think I have bitten off more than I can chew. (b) Barely managed to escape (a) To take on more than one can handle (c) Deficiency of funds (b) To be happy to multitask (d) Very arrogant (c) To find work dull SSC CHSL 26.05.2022 Shift-II (d) Not able to finish work SSC CGL (Tier-I) 11/04/2022 Shift-III Ans. (b) : For the above idiom 'By the skin of his teeth' option (b) Barely managed to escape gives suitable Ans. (a) : For the above idiom 'Bitten off more then I meaning. Other options are not correct. can chew' option (a) 'to take on more than one can handle' gives suitable meaning. Other options are not 86. Select the most appropriate meaning of the given idiom. correct. Beat around the bush 81. Select the most appropriate meaning of the (a) To search for a lost thing in vain given idiom. (b) To avoid talking about what is important Be hard up (c) To discuss minute details of something (a) Find it very difficult to wake up early (d) To hope for something that is unattainable (b) Have very little money SSC CHSL 27.05.2022 Shift-II (c) Unable to calculate (d) Have difficulty in climbing stairs Ans. (b) : For the above idiom 'Beat around the bush', SSC CGL (Tier-I) 11/04/2022 Shift-I option (b) To avoid talking about what is important' Ans. (b) : For the above idiom 'Be hard up' options (b) gives suitable meaning. Other options are not correct. Have very little money gives suitable meaning, other 87. Select the most appropriate meaning of the options are not correct. given idiom. Break a leg 82. Select the most appropriate meaning of the (a) Work long hours given idiom. (b) Wish someone bad luck Back out (c) Speak directly (a) To throw (b) To withdraw (d) Wish someone good luck (c) To explode (d) To enquire SSC CHSL 31.05.2022 Shift-III SSC CGL (Tier-I) 20/04/2022 Shift-III Ans. (b) : For the above idiom 'Back out', option (b) To Ans. (d) : For the above idiom 'Break a leg', option (d) withdraw gives suitable meaning. Other options are not wish someone good luck' gives suitable meaning. Other correct. options are not correct. Select the most appropriate meaning of the given idiom. Blow your own trumpet (a) Boast about one's own qualities (b) Play on your instrument (c) Keep away the dust from your instruments (d) Practise playing music SSC CGL (Tier-I) 19/04/2022 Shift-III Ans. (a) : For the above idiom 'Blow your own trumpet', option (a) Boast about one's own qualities' gives suitable meaning. Other options are not correct.

Idoms & Phrases

85.

Select the most appropriate meaning of the given idiom. Blow up (a) To stand upright (b) To destroy by an explosion (c) To suffice (d) To live up greatly SSC CHSL 24.05.2022 Shift-III Ans. (b) : for the above idiom 'Blow up', option (b) To destroy by an explosion give suitable meaning. other options are not correct.

660

YCT

89.

Select the most appropriate meaning of the given idiom. Bury the hatchet (a) have enmity (b) make peace (c) start a conflict (d) continue a flight SSC Stenographer (Grade C & D) 15.11.2021 Shift-II Ans. (b) : For the above idiom, Bury the hatchet, option (b) 'make peace' gives suitable meaning. Other options are not correct. 90. Select the most appropriate meaning of the given idiom. bad blood (a) ill feeling (b) poor quality (c) low status (d) hard luck SSC CGL (Tier-II) 08.08.2022 Shift-II Ans. (a) : For the above idiom 'Bad blood', option (a) ill feeling gives suitable meaning. Other options are not correct. 91. Select the most appropriate meaning of the given idiom. bury the hatchet (a) dig a grave (b) hide a treasure (c) forget past quarrels (d) sow the seeds SSC CGL (Tier-II) 08.08.2022 Shift-II Ans. (c) : For the above idiom 'Bury the hatchet' option (c) forget past quarrels gives suitable meaning. Other options are not correct. 92. Select the most appropriate meaning of the given idiom. Bang of the buck (a) A sorrowful heart (b) Dash against something (c) Less value for money (d) More value for money SSC CGL (Tier-I) –13/08/2021 (Shift-I) Ans. (d) : For the above idiom/Phase ‘Bang of the buck’ means ‘more value for money’. Thus, the correct answer is option (d). 93. Select the most appropriate meaning of the underlined idiom in the given sentence. If the audit report shows anomalies, the finance manager will be brought to book. (a) given a promotion (b) held accountable (c) supported fully (d) rewarded suitably SSC CGL (Tier-II) 16/11/2020 (3 pm - 5 pm) Ans. (b) : For the above idiom/phrase "Brought the book" option (b) held accountable gives an appropriate explanation. Other options have different meaning. 94. Select the most appropriate meaning of the underlined idiom in the given sentence. Information technology has developed by leaps and bounds. (a) at a rapid pace (b) in far off places (c) very gradually (d) through unfair means SSC CGL (Tier-II) 16/11/2020 (3 pm - 5 pm) Ans. (a) : For the above idiom/phrase "by leaps and bounds" option (a) ‘at a rapid pace’ gives an appropriate explanation. Other options have different meaning. Idoms & Phrases

95.

Select the most appropriate meaning of the underlined idiom in the given sentence. There was no one in the team who could bell the cat and tell the producer the truth. (a) tame some animals (b) warn the owners (c) do the impossible task (d) ring the bells regularly SSC GD-02/03/2019 (Shift-II) SSC CPO-SI (Tier-II) 27/09/2019 (3 pm - 5 pm) Ans. (c) : For the above idiom/phrase "bell the cat" option (c) ‘do the impossible task’ gives an appropriate explanation. other options have different meaning. 96. Select the most appropriate meaning of the underlined idiom in the given sentence. My blood boils when I hear people abuse children. (a) to be very controlled (b) to be very concerned (c) to be very angry (d) to be very sensitive SSC Sel. Post Phase-VII (M.L.) 15.10.2019 (Shift-I)

Ans. (c) : For the above idiom/phrase "blood boils" option (c) ‘to be very angry’ gives an appropriate explanation. Other options have different meaning. 97. Select the most appropriate meaning of the following idiom. Be in the eye of a storm (a) Be in the middle of a difficult situation (b) Be very worried due to something (c) Be caught in heavy rain (d) Be in conflict with many people SSC Sel. Post Phase-VIII (H.L.) 09.11.2020 (Shift-I)

Ans. (a) : For the above idiom/phrase. "Be in the eye of a storm" option (a) ‘be in middle of a difficult situation’ gives an appropriate explanation. Other options have different meaning. 98. Select the most appropriate meaning of the given idiom. Bear the brunt of (a) bear the maximum fury (b) bear discrimination (c) breed jealousy (d) assume an identity SSC CPO-SI – 24/11/2020 (Shift-II) Ans. (a) : For the above idiom/phrase "Bear the brunt of "Option (a) ‘bear the maximum fury’ gives an appropriate explanation. Other options have different meaning. 99. Select the most appropriate meaning of the underlined idiom in the given sentence. Let us have all the regulations in black and white. (a) painted in colour (b) printed in coloured ink (c) written on the black board (d) in writing SSC CGL (Tier-I) – 13/06/2019 (Shift-II) Ans. (d) : For the above idiom/phrase "in black and white" option (d) 'in writing' gives an appropriate explanation. Other options have different meaning.

661

YCT

100. Select the most appropriate meaning of the underlined idiom in the given sentence. The boss is going to blow his top when he discovers the blatant mistake in the balance sheet. (a) attack fiercely (b) dismiss from job (c) be very embarrassed (d) be very angry SSC CGL (Tier-I) – 11/06/2019 (Shift-II) Ans. (d) : For the above idiom/phrase "blow his top" option(d) ‘to be very angry’ gives an appropriate explanation. Other options have different meaning. 101. Select the most appropriate meaning of the underlined Idiom in the given sentence. The heavy losses in business came like a bolt from the blue. (a) a thunderstorm (b) an unexpected disaster (c) a windfall (d) an ominous warning SSC CHSL –16/04/2021 (Shift-I) SSC CGL–10/07/2019 (Shift-I) SSC CGL (Tier-I) – 07/06/2019 (Shift-III) Ans : (b) For the above idiom/phrase "bolt from the blue" option (b) ‘an unexpected disaster’ gives an appropriate explanation. Other options have different meaning. 102. Select the most appropriate meaning of the given idiom. butterflies in the stomach (a) being excited (b) being nervous (c) being angry (d) being hungry SSC MTS – 14/08/2019 (Shift-I) SSC CGL (Tier-I) – 06/06/2019 (Shift-III) Ans. (b) : For the above idiom/phrase " butterflies in the stomach" option (b) 'being nervous' gives an appropriate explanation. Other options have different meaning. 103. Select the most appropriate meaning of the given idiom. Bed of roses (a) An enjoyable state (b) A difficult situation (c) An unforgettable moment (d) A state of perplexity SSC CPO-SI – 11/12/2019 (Shift-I) Ans : (a) For the above idiom/phrase "Bed of roses" option (a) 'an enjoyable state' gives an appropriate explanation. Other options have different meaning. 104. Select the most appropriate meaning of the given idiom. Be left in the lurch (a) be sent to do work which one doesn't enjoy (b) keep repeating something till it is accepted (c) be tired of doing the same thing over and over again (d) be deserted when one is in trouble SSC CPO-SI – 13/12/2019 (Shift-II) Ans. (d) : For the above idiom/phrase "Be left in the lurch" option (d) deserted when one is in trouble gives an appropriate explanation. Other options have different meaning. Idoms & Phrases

105. Select the most appropriate of the given idiom. To blaze a trail (a) to initiate something new (b) to complete a project (c) to act impulsively (d) to set fire to something SSC CPO-SI – 13/12/2019 (Shift-I) Ans. (a) : For the above idiom/phrase "to blaze a trail" option (a) ‘to initiate something new’ gives an appropriate expatiation. Other options have different meaning. 106. Select the most appropriate meaning of the underlined idiom in the given sentence. Please come to the point, don't beat around the bush. (a) avoid the topic (b) hide behind a bush (c) make excuses (d) accept defeat SSC CHSL (Tier-I) –10/07/2019 (Shift-III) Ans : (a) For the above idiom/phrase "beat around the bush" option (a) ‘avoid the topic’ gives an appropriate explanation. Other options have different meaning. 107. Select the most appropriate meaning of the given idiom. Be an old hat (a) Be superstitious (b) Be outdated (c) Be crazy (d) Be foolish SSC CHSL (Tier-I) –04/07/2019 (Shift-I) Ans : (b) For the above idiom/phrase "be an old hat" option (b) ‘be outdated’ gives an appropriate explanation. Other options have different meaning. 108. Select the most appropriate meaning of the given idiom. To bark up the wrong tree (a) to be wrong about the reason for something (b) to bury money under a tree and forget the place (c) to be mistaken about the name of a tree (d) to eat an unpalateble fruit by mistake SSC CHSL (Tier-I) –04/07/2019 (Shift-III) Ans : (a) For the above idiom/phrase "To bark up the wrong tree" option (a) 'to be wrong about the reason for something' gives an appropriate explanation. Other options have different meaning. 109. Select the most appropriate meaning of the given idiom. bite your tongue (a) talk for a long time (b) be impossible to be understood (c) get bruises in the mouth (d) stop yourself from saying something SSC CHSL (Tier-I) –02/07/2019 (Shift-II) Ans : (d) For the above idiom/phrase "bite your tongue" option (d) ‘stop yourself from saying something’ gives an appropriate explanation. Other options have different meaning. 110. Select the most appropriate meaning of the underlined idiom in the given sentence. A lot of work has gone on behind the scenes for the special event which is being planned next week. (a) To complete the arrangements (b) In the theatre (c) For spreading the news (d) Unknown to everyone SSC CHSL (Tier-I) 17/03/2020 (Shift-I)

662

YCT

Ans. (d) For the above idiom/phrase "behind the scenes" option (d) 'unknown to everyone' gives an appropriate explanation. Other options have different meaning. 111. Select the most appropriate meaning of the given idiom. Bite off more than one can chew (a) To taste something while cooking (b) To eat quickly and run away (c) To take on more than one can deal with (d) To cheat someone and get away SSC CHSL (Tier-I) 13/10/2020 (Shift-I) Ans. (c) : For the above idiom/phrase "Bite off more than one can chew" option (c) ‘to take on more than one can deal with’ gives an appropriate explanation. Other options have different meaning. 112. Select the most appropriate meaning of the underlined idiom in the given sentence. The police was able to find the guilty but the big fish escaped. (a) main leaders (b) large-sized ones (c) tall people (d) plum men SSC CHSL (Tier-I) 18/03/2020 (Shift-II) Ans. (a) : For the above idiom/phrase "The big fish" option (a) ‘main leaders' gives an appropriate explanation. Other options have different meaning. 113. Select the most appropriate meaning of the given idiom. In the blink of an eye (a) Something that is not real (b) Within a very short period of time (c) Within a very long period of time (d) Something that is unavoidable SSC CHSL (Tier-I) 26/10/2020 (Shift-II) Ans. (b) : For the above idiom/phrase "In the blink of an eye" option (b) 'within a very short period of time' gives an appropriate explanation. Other options have different meaning. 114. Select the most appropriate meaning of the given Idiom. Be behind the times (a) Be old fashioned (b) Always dreaming (c) Be very old (d) Not be alert SSC CHSL (Tier-I) 19/10/2020 (Shift-I) Ans. (a) : For the above idiom/phrase "be behind the times" option (a) ‘be old fashioned’ gives an appropriate explanation. Other options have different meaning. 115. Select the most appropriate meaning of the idiom. Back in saddle (a) Learn how to ride (b) Resume duty (c) Teach someone riding (d) Put a new saddle SSC CHSL (Tier-I) 17/03/2020 (Shift-III) Ans. (b) : For the above idiom/phrase "back in saddle" option (b) ‘Resume duty’ gives an appropriate explanation. Other options have different meaning. 116. In the following question, out of the four given options, select the option which means the best of the idiom / phrase. Between the devil and the deep sea Idoms & Phrases

Go fishing and cause trouble Between two great difficulties Speaking sharply To begin working seriously SSC MTS 9-10-2017 (Shift-III) Ans : (b) For the above idiom/phrase "Between the devil and the deep sea" option (b) ‘Between two great difficulties’ gives an appropriate explanation. Other options have different meaning. 117. In the following question, out of the four given alternatives, select the alternatives which best express the meaning of the Idiom/Phrase. By hook or by crook (a) Causing trouble to (b) By all means (c) To flight against (d) To bear patiently SSC MTS 10-10-2017 (Shift-II) Ans. (b) : For the above idiom/phrase "By hook or by crook" option (b) ‘by all means’ gives an appropriate explanation. Other options have different meaning. 118. In the following question, out of the given four alternatives, select the alternative which best expresses the meaning of the Idiom/Phrase. Birds of the same feather (a) People with a generous heart (b) To get into trouble together (c) To be in a totally helpless condition (d) People with similar character SSC MTS – 05/08/2019 (Shift-I) Ans. (d) : For the above idiom/phrase "Birds of the same feather" option (d) ‘people with similar character’ gives an appropriate explanation. Other options have different meaning. 119. Select the most appropriate meaning of the given idiom. Bend over backwards (a) To express sudden shock (b) To exert a lot of effort towards some end (c) To end all activities (d) to confess to a crime SSC MTS – 02/08/2019 (Shift-I) Ans. (b) : For the above/phrase "Bend over backwards" option (b) ‘to exert a lot of efforts towards some end’ gives and appropriate explanation. Other options have different meaning. 120. Select the most appropriate meaning of the given idiom Best thing since sliced bread (a) To take on a task that is way too big (b) To present a counter argument (c) To perform satisfactorily (d) Most useful innovation in a long time SSC MTS – 21/08/2019 (Shift-I) Ans. : (d) For the above idiom/phrase "Best thing since sliced bread" option (d) 'most useful innovation in a long time' gives an appropriate explanation, other options have different meaning. 121. Select the most appropriate meaning of the given idiom. By and large (a) All inclusive (b) By choice (c) In general (d) In short SSC MTS – 16/08/2019 (Shift-III)

663

(a) (b) (c) (d)

YCT

Ans. (c) : For the above idiom/phrase "By and large" option (c) in general gives an appropriate explanation other options have different meaning. 122. Select the most appropriate meaning of the given idiom. Be on cloud nine (a) To be in a state of extreme happiness (b) To be able to climb higher (c) To dream of clouds (d) To think about something unrealistically SSC MTS – 14/08/2019 (Shift-II) Ans. (a) : For the above idiom/Phrase "Be on cloud nine" option (a) To be in a state of extreme happiness gives an appropriate explanation. Other have different meaning. 123. Select the option that means the same as the given idiom. To bury the hatchet (a) To bury old utensils (b) To bury an axe (c) To make someone happy (d) To make peace SSC GD – 02/03/2019 (Shift-I) Ans. (d) : For the above idiom/phrase "To bury the hatchet" option (d) ‘to make peace’ gives an appropriate explanation. Other options have different meaning. 124. Select the most appropriate meaning of the given idiom. A blessing in disguise (a) A hidden misfortune (b) An apparent fortune (c) An apparent blessing (d) A hidden favour SSC GD – 11/02/2019 (Shift-III) Ans. (d) : For the above Idiom/Phase ‘A blessing in disguise’ means ‘A hidden favour’. Thus the correct answer is option (d).

C 125. Select the most appropriate option that can substitute the underlined words in the given sentence. The company is planning to cut back few employees. (a) lay off some (b) reduce some (c) lay down some (d) refuse some SSC CHSL (Tier-I) 14/08/2023 (Shift-IV) Ans. (a) : The underlined words 'cut back few' will be substituted by option (a) lay off some. phrases verb 'Cut something back means to reduce something upto a particular amount . which can be substituted by 'lay off', which means the act of ending a workers job sometimes temporarily, usually because there is not enough work to do. Correct sentence The Company is planning to lay off some employees. 126. Select the most appropriate idiom for the given situation. Aurangzeb, the famous Mughal ruler, found that he had caught someone more powerful than him in the form of the brave Maratha, Shivaji. Idoms & Phrases

(a) Catching a tartar (b) Changing the hands (c) Blazing the trail (d) Nipping in the bud SSC CHSL (Tier-I) 03/08/2023 (Shift-II) Ans. (a) : The most appropriate idiom for the above given situation is option (a ) 'Catching a tartar'. Meaning or other idioms – Changing the hands – To go from one owner to another. Blazing the trail – To be the first one to do something and to show others how to do it. Nipping in the bud – To destroy early. 127. Select the option that is closest in meaning to the underlined idiom/phrase in the given sentence. Please cut down on your expenses now or you will face difficulties in the future. (a) raise (b) recover (c) reduce (d) refuse SSC Constable GD-01/12/2021 Shift-I Ans. (c) : For the above underlined idiom 'cut down on' option (c) 'reduce' gives suitable meaning. Other options are not correct. 128. Select the most appropriate meaning of the given idiom. Crack up (a) Grow old (b) Laugh out loud (c) Find fault (d) Become wise SSC Constable GD-10/12/2021 Shift-III Ans. (b) : For the above idiom 'Crack up', option (b) Laugh out loud' gives suitable meaning other option are not correct. 129. Select the most appropriate meaning of the given idiom. Come to blows (a) Insult someone (b) Return a favour (c) Agree on a plan (d) Get into a fight physically SSC Constable GD-16/11/2021 Shift-III Ans. (d) : For the above idiom 'Come to blows', option (d) Get into a fight physically gives suitable meaning. Other options are not correct. 130. Select the most appropriate meaning of the given idiom. Cool as a cucumber (a) Wise (b) Scared (c) Calm (d) Lazy SSC Constable GD-16/11/2021 Shift-III Ans. (c) : For the above idiom 'Cool as cucumber', option (c) 'Calm' gives suitable meaning. Other options are not correct. 131. Select the most appropriate meaning of the given idiom. Chicken hearted (a) Visibly pleased (b) Slightly risky (c) Readily available (d) Easily scared SSC Constable GD-22/11/2021 Shift-II Ans. (d) : For the above idiom 'Chicken hearted', option (d) 'Easily scared' gives suitable meaning. Other options are not correct.

664

YCT

132. Select the most appropriate meaning of the given idiom. Crocodile tears (a) Hopeless situation (b) Totally different (c) False tears (d) Of less worth SSC MTS-12/07/2022 Shift-I Ans. (c) : For the above idiom "Crocodile tears' option (c) false tears gives suitable meaning. Other options are not correct. 133. Select the most appropriate meaning of the given idiom. Change of heart (a) To narrowly escape disaster (b) An unfair attack (c) Different opinion about someone or something (d) Very similar in appearance SSC MTS-06/07/2022 Shift-II Ans. (c) : For the above idiom 'Change of heart', option (c) 'Different opinion above someone or something' gives suitable meaning. Other options are not correct. 134. Select the most appropriate meaning of the given idiom. Chew the scenery (a) To understate (b) To act overly emotional (c) To beautify (d) To mismanage SSC MTS-05/07/2022 Shift-I Ans. (b) : For the above idiom 'Chew the scenery', option (b) 'To act overly emotional gives suitable meaning other options are not correct. 135. Select the most appropriate meaning of the given idiom. Cake walk (a) Something tasty (b) Something sweet (c) Something easy (d) Something enjoyable SSC CGL (Tier-I) 11/04/2022 Shift-II Ans. (c) : For the above idiom' 'Cake walk', option (c) 'Something easy' gives suitable meaning. Other options are not correct. 136. Select the most appropriate meaning of the given idiom. Cool as a cucumber (a) Calm and composed (b) Irritated and annoyed (c) Happy and excited (d) Nervous and fidgety SSC CGL (Tier-I) 18/04/2022 Shift-I Ans. (a) : For the above idiom 'cool as a cucumber', option (a) calm and composed gives suitable meaning. other options are not correct. 137. Select the most appropriate meaning of the given idiom. cat’s paw (a) A person used as a tool by another (b) To attack someone (c) Move at a fast pace (d) The most important attribute SSC CGL (Tier-I) 21/04/2022 Shift-III Idoms & Phrases

Ans. (a) : For the above idiom 'Cat's paw', option (a) A person used as a tool by another' gives suitable meaning, Other options are not correct. 138. Select the most appropriate meaning of the highlighted idiom. It was a Catch 22 situation for the manager. (a) A positive outcome in a tough situation (b) Hailed as the ultimate champion (c) Worthy successor to the throne (d) A difficult situation in which the solution to a problem is impossible. SSC CHSL 26.05.2022 Shift-I Ans. (d) : For the above idiom 'Catch 22 situation, option (d) A difficult situation in which the solution to a problem is impossible gives suitable meaning. Other options are not correct. 139. Select the most appropriate meaning of the given idiom. Chill out (a) Eat spicy food (b) Feel very cold (c) Relax completely (d) Become stiff SSC Stenographer (Grade C & D) 15.11.2021 Shift-II Ans. (c) : For the idiom 'Chill out', option (c) Relax completely, gives suitable meaning. Other options are not correct. 140. Select the most appropriate meaning of the given idiom. Cross a bridge when you come to it (a) know all the dangers and difficulties likely to be faced and be prepared (b) deal with something when it happens, and not worry about it beforehand (c) use different strategies to cross a bridge safety (d) plan to reach a place and then rest there SSC Stenographer (Grade C & D) 12.11.2021 Shift-II Ans. (b) : For the above idiom 'Cross a bridge when you come to it', option (b) deal with something, gives suitable meaning. Other options are not correct. 141. Select the most appropriate meaning of the given idiom. cut a sorry figure (a) render an apology (b) make a sculpture (c) create a poor impression (d) break a record SSC CGL (Tier-II) 08.08.2022 Shift-II Ans. (c) : For the above idiom 'Cut a sorry figure', option (c) create a poor impression, gives suitable meaning. Other options are not correct. 142. Select the most appropriate meaning of the given idiom. Cook the book. (a) To falsify financial records (b) To be a good writer (c) To write books on cooking (d) To be a good editor SSC CHSL (Tier-I) –09/08/2021 (Shift-I)

665

YCT

Ans. (a) : For the above idiom/phrase "Cook the book option (a) ‘To falsify financial records’ gives an appropriate explanation. Other options have different meaning. 143. Select the most appropriate meaning of the given idiom . To cut a long story short (a) Tell something briefly (b) Like to tell long stories (c) Like to tell short stories (d) Tell something in a roundabout way SSC CGL (Tier-I) –16/08/2021 (Shift-I) Ans. (a) : For the above idiom/phrase "To cut a long story short" option (a) ‘Tell something briefly’ gives an appropriate explanation. Other options have different meaning. 144. Select the most appropriate meaning of the underlined idiom in the given sentence. His comments cast a slur upon the integrity of his manager. (a) praised (b) redeemed (c) damaged (d) improved SSC CGL (Tier-II) 16/11/2020 (3 pm - 5 pm) Ans. (c) : For the above idiom "Cast a slur upon" option (c) ‘damaged’ gives an appropriate explanation other option have different meaning. 145. Select the most appropriate meaning of the underlined idiom in the given sentence. The threat of the pandemic is not just a cry in the wilderness. (a) spreading rumours (b) tears of sorrow (c) an unheeded warning (d) mourning for the loss SSC CGL (Tier-II) 16/11/2020 (3 pm - 5 pm) Ans. (c) : For the above idiom "a cry in the wilderness" option (c) ‘an unheeded warning’ gives an appropriate explanation. Other options have different meaning. 146. Select the most appropriate meaning of the underlined idiom in the given sentence. You must have a concrete project and not build castles in the air if you want to submit an application for a loan. (a) make unfaithful friends (b) talk irresponsibly (c) have unrealistic ideas (d) make unplanned buildings SSC CPO – SI 13/12/2019 (Shift-I) SSC CPO-SI (Tier-II) 27/09/2019 (3 pm - 5 pm) Ans. (c) : For the above idiom "build castle in the air" option (c) ‘have unrealistic ideas’ gives an appropriate explanation. Other options have different meaning. 147. Select the most appropriate meaning of the underlined idiom in the given sentence. In our office it is the Chief Accountant who calls the shots. (a) to be in control (b) one who keeps the secrets (c) to be rational (d) to be aggressive

Ans. (a) : For the above idiom "call the shots" option (a) ‘to be in control’ gives an appropriate explanation. Other options have different meaning. 148. Select the most appropriate meaning of the given idiom. Call on (a) step out (b) telephone (c) announce (d) visit SSC CPO-SI – 23/11/2020 (Shift-I) Ans. (d) : For the above idiom/phrase "call on" option (d) ‘visit’ gives an appropriate explanation. Other option have different meaning. 149. Select the most appropriate meaning of the underlined idiom in the given sentence. The loyal watchman was cut to the quick when he was accused of theft. (a) dismissed (b) imprisoned (c) severely punished (d) hurt intensely SSC CGL (Tier-I) – 10/06/2019 (Shift-I) Ans : (d) For the above idiom/phrase "cut to the quick" option (d) ‘Hurt intensely’ gives an appropriate explanation. Other options have different meaning. 150. Select the most appropriate meaning of the given idiom. Couch potato (a) a person who sleeps all day (b) a person who watches too much television (c) a person who remains relaxed and calm (d) a person who is simple in nature SSC CHSL (Tier-I) – 12/10/2020 (Shift-III) SSC CGL (Tier-I) – 06/06/2019 (Shift-II) Ans : (b) For the above idiom/phrase "Couch potato" option (b) ‘a person who watches too much television’ gives an appropriate explanation. Other options have different meaning. 151. Select the most appropriate meaning of the given idiom. costs an arm and a leg (a) rarely available (b) easy to obtain (c) nothing to lose (d) very expensive SSC CHSL (Tier-I)20/10/2020 SSC CHSL – 16/04/2021 (Shift-I) SSC CGL (Tier-I) – 04/06/2019 (Shift-II) Ans : (d) For the above idiom/phrase "Costs an arm and a leg" option (d) ‘very expensive’ gives an appropriate explanation other options have different meaning. 152. Select the most appropriate meaning of the given idiom. Come rain or shine (a) in bad weather (b) in the monsoon (c) when it does not rain (d) whatever happens SSC CPO-SI – 12/12/2019 (Shift-II) Ans : (d) For the above idiom/phrase "come rain and shine" option (d) ‘whatever happens’ gives an appropriate explanation. Other options have different meaning. 153. Select the most appropriate meaning of the underlined idiom in the sentence. All those who carried the day in reality shows SSC Sel. Post Phase-VII (M.L.) 15.10.2019 (Shift-I) failed to make a mark later.

Idoms & Phrases

666

YCT

(a) (b) (c) (d)

won a victory made a guest appearance participated acted as hosts SSC CPO-SI – 12/12/2019 (Shift-I) Ans : (a) For the above idiom/phrase "Carried the day" option (a) ‘won a victory’ gives an appropriate explanation Other options have different meaning. 154. Select the most appropriate meaning of the given idiom. The crux of the matter (a) The unknown point (b) The lesser issue (c) The critical point (d) The interesting thing SSC CHSL (Tier-I) –03/07/2019 (Shift-I) Ans : (c) For the above idiom/phrase "the crux of the matter" option (c) ‘The critical point’ gives an appropriate explanation. Other options have different meaning. 155. Select the most appropriate meaning of the given idiom. Come hell or high water. (a) Not ready to go on adventures (b) Tired of all kinds of dangers (c) In spite of any obstacles (d) Afraid of the challenges SSC CHSL (Tier-I) 26/10/2020 (Shift-II) Ans. (c) : For the above idiom/phrase "Come hell or high water" option (c) ‘in spite of any obstacles’ gives an appropriate explanation. Other options have different meaning. 156. Select the most appropriate meaning of the given idiom. Call a spade a spade (a) Say things that are untrue and painful (b) Say the truth about something, even if it is not pleasant (c) Plan to attack someone with rude words (d) Lash out unkindly at someone for no reason SSC CHSL (Tier-I) 19/10/2020 (Shift-I) Ans. (b) : For the above idiom/phrase "Call a spade a spade" option (b) ‘say the truth above something, even if it is not pleasant’ gives an appropriate explanation. Other options have different meaning. 157. Select the most appropriate meaning of the given idiom. Cool it (a) To be angry (b) To be nervous (c) To relax (d) To wait SSC CHSL (Tier-I) 19/10/2020 (Shift-III) Ans. (c) : For the above idiom/phrase "Cool it" option (c) ‘to relax’ gives an appropriate explanation. Other options have different meaning. 158. Select the most appropriate meaning of the given idiom. Chill out (a) To calm down (b) To be angry (c) To be nervous (d) To fight SSC CHSL (Tier-I) 19/10/2020 (Shift-II) Ans. (a) : For the above idiom, "Chill out" option (a) ‘To calm down’ gives an appropriate explanation. other options have different meaning. Idoms & Phrases

159. Select the most appropriate meaning of the underlined idiom in the given sentence. To restart his cash-strapped company he decided to apply for a bank loan. (a) expanding (b) impoverished (c) improving (d) expensive SSC CHSL (Tier-I) 14/10/2020 (Shift-I) Ans. (b) : For the above idiom/phrase "Cash strapped" option (b) ‘impoverished’ gives an appropriate explanation. Other options have different meaning. 160. Select the most appropriate meaning of the underlined idiom in the given sentence. When they read about the proposed strike of buses they said, “We’ll cross that bridge when we come to it.” (a) Prevent them (b) Call them for talks (c) Solve the problem (d) Cancel our plans SSC CHSL (Tier-I) 17/03/2020 (Shift-III) Ans. (c) : For the above idiom/phrase "Cross the bridge" option (c) ‘solve the problem’ gives an appropriate explanation. Other options have different meaning. 161. Select the most appropriate meaning of the given idiom. Cut to the chase (a) Talking about the important point and leaving out the detail (b) Sleeping long hours and spoiling the health (c) Cutting long pieces and spoiling the dish cooked (d) Stopping unexpectedly and missing the chase SSC CHSL (Tier-I) 18/03/2020 (Shift-III) Ans. (a) : For the above idiom/phrase "cut to the chase" option (a) ‘talking about the important point and leaving out the detail’ gives an appropriate explanation. Other options have different meaning. 162. Select the most appropriate meaning of the given idiom. Come rain or shine (a) under any circumstances (b) in favorable conditions (c) in difficult situations (d) during bad weather SSC MTS – 22/08/2019 (Shift-II) Ans. (a) : For the above idiom/phrase "Come Rain or shine" option (a) ‘under any circumstances’ gives an appropriate explanation. Other options have different meaning. 163. Select the most appropriate meaning of the given idiom. Cup of tea (a) something that pleases one (b) A feeling of elation (c) A useless search (d) Something refreshing SSC MTS – 08/08/2019 (Shift-I) Ans. (a) : For the above idiom/phrase "cup of tea" option (a) ‘something that pleases one’ gives an appropriate explanation. Other options have different meaning.

667

YCT

164. Select the most appropriate meaning of the given idiom. Cut one short (a) To chop something (b) To interrupt someone (c) To act quickly (d) To be rude SSC MTS – 07/08/2019 (Shift-I) Ans. (b) : For the above idiom/phrase "cut one short" option (b) ‘To interrupt someone’ gives an appropriate explanation. Other options have different meaning. 165. Select the most appropriate meaning of the given idiom. Come to light (a) To be well lit-up (b) To be obscure (c) To be known publicly (d) To pry a secret SSC MTS – 05/08/2019 (Shift-II) Ans. (c) : For the above idiom/phrase "Come to light" option (c) ‘to be known publicly’ gives an appropriate explanation. Other options have different meaning. 166. In the following question, out of the given four alternatives, select the alternative which best expresses the meaning of the Idiom/Phrase. Cat's whiskers (a) A very easy matter (b) To be highly impressive (c) To be very determined (d) Difficult to understand SSC MTS – 05/08/2019 (Shift-I) Ans. (b) : For the above idiom/phrase "Cat's whiskers" option (b) ‘to be highly impressive’ gives an appropriate explanation. Other options have different meaning. 167. Select the most appropriate meaning of the given idiom Cut the mustard (a) To be pessimistic (b) To succeed (c) To take shortcuts (d) To be angry SSC MTS – 20/08/2019 (Shift-III) Ans. (b) : For the above idiom/phrase "Cut the mustard" option (b) ‘to succeed’ gives an appropriate explanation. Other options have different meaning. 168. Select the most appropriate meaning of the given idiom. Child's play (a) Something delightful (b) Something amusing (c) Something easy (d) Something risky SSC MTS – 14/08/2019 (Shift-III) Ans. (c) : For the above idiom/phrase "child's pay" option (c) ‘something easy’ gives an appropriate explanation. Other options have different meaning. 169. Select the most appropriate meaning of the given idiom. Cry over spilt milk (a) cry too much over a petty issue (b) regret about something that happened and cannot be changed (c) unnecessarily worry about the future (d) complain about being forced to drink milk SSC MTS – 13/08/2019 (Shift-I) Ans. (b) : For the above idiom/phrase "cry over spilt milk option (b) ‘regret about something that happened and cannot be changed’ gives an appropriate explanation. Other options have different meaning. Idoms & Phrases

170. Select the meaning of the given idiom. Chip off the old block (a) Resembling parents (b) Having interest in antique furniture (c) Having old fashioned values (d) Looking after parents SSC MTS – 09/08/2019 (Shift-II) Ans. (a) : For the above idiom/phrase "chip off the old block" option (a) ‘Resembling parents’ gives an appropriate explanation. Other options have different meaning. 171. Select the most appropriate meaning of the given idiom. Cutting corners (a) Spending too much money (b) Saving money (c) Making money (d) Lending money SSC GD – 11/02/2019 (Shift-III) Ans. (b) : For the above idiom/phrase "cutting corner" option (b) ‘saving money’ gives an appropriate explanation. Other options have different meaning.

D 172. Select the most appropriate meaning of the given idiom. Devil’s advocate (a) One who is very precise and accurate (b) One who presents very logical arguments (c) One who never argues (d) One who takes an opposing position for the sake of argument SSC CHSL (Tier-I) 08/08/2023 (Shift-II) Ans. (d) : The most appropriate idiom of the above word 'Devil's advocate' is 'one who takes an opposing position for the sake of argument.' other options give different meaning. 173. Select the most appropriate meaning of the given idiom. Drag one's feet (a) Do something deliberately at a slow pace (b) Have a disability in the feet (c) Be hurt badly on the foot (d) Find it difficult to walk fast SSC Constable GD-09/12/2021 Shift-II Ans. (a) : For the above idiom 'Drag one's feet', option (a) Do something deliberately at a slow pace' gives suitable meaning. Other options are not correct. 174. Select the most appropriate meaning of the given idiom. Drop names (a) Delets someone's name from a list (b) Name famous people to impress others (c) Show disrespect to someone's name (d) Assume a false name SSC Constable GD-17/11/2021 Shift-II Ans. (b) : For the above idiom 'Drop names', option (b) 'name famous people to impress others' gives suitable meaning. Other options are not correct.

668

YCT

175. Select the most appropriate meaning of the given idiom. Draw first blood (a) Be the first to gain an advantage or score against an opponent (b) Be the first to donate blood to help a friend (c) Agree to accompany someone for the diagnosis of an illness (d) Take a blood test early in the morning SSC Constable GD-14/12/2021 Shift-II Ans. (a) : For the above idiom 'Draw first blood', option (a) Be the first to gain an advantage or score against an opponent gives suitable meaning. Other options are not correct. 176. Select the most appropriate meaning of the given idiom. Down in the mouth (a) To be angry (b) To be depressed (c) To feel hungry (d) To be happy SSC Constable GD-29/11/2021 Shift-II Ans. (b) : For the above idiom 'Down in the mouth' option (b) 'to be depressed' gives suitable meaning. Other options are not correct. 177. Select the most appropriate meaning of the given idiom. Draw the line (a) To accept someone's idea (b) To give a second chance (c) To move on from a past relationship (d) To set a limit on something SSC MTS-06/07/2022 Shift-I Ans. (d): For the above idiom 'Draw the line', options (d) To set a limit on something gives suitable meaning. Other options are not correct. 178. Select the most appropriate meaning of the given idiom. Dark horse (a) A mean person (b) A slow runner (c) An unexpected winner (d) An honest fellow SSC CGL (Tier-I) 13/04/2022 Shift-I Ans. (c) : For the above idiom 'Dark horse' options (c) an unexpected winner gives suitable meaning. Other options are not correct. 179. Select the most appropriate meaning of the given idiom. Donkey's years (a) Leisure time (b) Bad time (c) Long time (d) Good time SSC CHSL 27.05.2022 Shift-I Ans. (c) : For the above idiom 'Donkey's years', option (c) Long time' gives suitable meaning. Other options are not correct. 180. Select the most appropriate meaning of the given idiom. Dry run Idoms & Phrases

(a) Jogging in a park (b) Running without shoes (c) Rehearsal of an event (d) Running in a desert SSC Stenographer (Grade C & D) 11.11.2021 Shift-II Ans. (c) : For the above idiom 'Dry run', option Rehearsal of an event, gives suitable meaning. Other options are not correct. 181. Select the most appropriate meaning of the given idiom. To drag one's feet (a) to walk unsteadily (b) To walk with a limp (c) To delay taking a decision (d) to pull someone's legs SSC CHSL (Tier-I) –19/04/2021 (Shift-I) Ans. (c) : For the above idiom "To drag one's feet option (c) ‘To delay taking a decision’ gives an appropriate explanation. Other options have different meaning. 182. Select the most appropriate meaning of the given idiom. Draw the line at something (a) Coming to a conclusion (b) Agreeing to an idea (c) Making pencil sketches (d) Accept something up to particular point SSC CGL (Tier-I) –23/08/2021 (Shift-I) Ans. (d) : For the above idiom, "Draw the line at something" option(d) ‘accept something up to particular point’ gives an appropriate explanation. Other options have different meaning. 183. Select the most appropriate meaning of the given idiom. A dry run (a) a poor harvest (b) a rehearsal (c) a slow run (d) a run on dry ground SSC CPO-SI – 23/11/2020 (Shift-I) Ans. (b) : For the above idiom/phrase "A dry run" option (b) ‘a rehearsal’ gives an appropriate explanation. Other options have different meaning. 184. Select the most appropriate meaning of the underlined idiom in the sentence. The aged man preferred to die in harness than lead a rusted life. (a) to keep one's belongings safe (b) to lead a life of comfort (c) to live in perpetual slavery (d) to continue occupation till death SSC CPO-SI – 12/12/2019 (Shift-I) Ans : (d) For the above idiom/phrase "Die in harness option (d) ‘to continue occupation till death’ gives an appropriate explanation. Other options have different meaning. 185. Select the most appropriate meaning of the given idiom. A dark horse (a) a well kept secret (b) an unknown genius (c) a horse of a fine breed (d) an expected outcome SSC CPO-SI – 13/12/2019 (Shift-I)

669

YCT

Ans. (b) : For the above idiom/phrase "A dark horse" option (b) ‘An unknown genius’ gives an appropriate explanation. Other options have different meaning. 186. Select the most appropriate meaning of the given idiom draw a blank (a) be lucky in a game (b) be unable to sketch (c) be happy about something (d) be unsuccessful SSC CHSL (Tier-I) –09/07/2019 (Shift-III) Ans : (d) For the above idiom/phrase "Draw a blank" option (d) ‘be unsuccessful’ gives an appropriate explanation. Other options have different meaning. 187. Select the most appropriate meaning of the given idiom. Dance to someone's tune (a) argue with others on petty matters (b) delay in making a decision (c) do what others want you to do (d) be engaged in an energetic activity SSC CHSL (Tier-I) –01/07/2019 (Shift-III) Ans : (c) For the above idiom/phrase "Dance to someone's tune" option (c) ‘do what others want you to do’ gives an appropriate explanation other options have different meaning. 188. Select the most appropriate meaning of the underlined idiom in the given sentence. We enjoyed watching the film, because it was down-to-earth depiction of modern life. (a) pleasing and entertaining (b) practical and direct (c) fanciful and dreamlike (d) short and meaningful SSC CHSL (Tier-I) 17/03/2020 (Shift-II) Ans. (b) : For the above idiom/phase "down to earth" option (b) ‘practical and direct’ gives an appropriate explanation. Other options have different meaning. 189. In the following question out of the four given alternatives select the alternative which best expresses the meaning of the Idiom/phrase. To doctor the accounts (a) Very costly treatment (b) To manipulate the accounts (c) High class treatment (d) Auditing of accounts SSC MTS 10-10-2017 (Shift-I) Ans : (b) For the above idiom/phrase "doctor the accounts" option (b) ‘to manipulate the accounts’ gives an appropriate explanation. Other options have different meaning.

Ans. (c) : The most appropriate idiom for the above statement 'One who hurries up always gets the best 'is ' early bird catches the worm'. Other options give different meanings. 191. Select the most appropriate meaning of the given idiom. Easier said than done (a) Not as difficult as it appears to be (b) Not as moderate as it appears to be (c) Not as restricted as it appears to be (d) Not as easy as it appears to be SSC Selection Posts XI-27/06/2023 (Shift-I) Ans. (d) : The most appropriate meaning of the above idiom 'Easier said than done' is 'Not as easy as it appears to be; Other options give different meanings. 192. Select the most appropriate meaning of the given idiom. Eagle eye (a) An eye full of curiosity (b) An eye with sharp visual powers (c) An eye full of suspicion (d) An eye with sense of admiration SSC Constable GD-23/11/2021 Shift-III Ans. (b) : For the above idiom 'Eagle eye', option (b) An eye with sharp visual powers, gives suitable meaning. other options are not correct. 193. Select the most appropriate meaning of the given idiom. Egg on (a) continue (b) agree (c) encourage (d) quarrel SSC CPO-SI – 25/11/2020 (Shift-I) Ans. (c) : For the above idiom/phrase "Egg on" option (c) ‘encourage’ gives an appropriate explanation. Options have different meaning. 194. Select the most appropriate meaning of the given idiom. Eat humble pie (a) Make noise while eating (b) Eat something which is not tasty (c) Eat less food (d) Admit that you are wrong SSC GD – 14/02/2019 (Shift-II) SSC CHSL (Tier-I) 15/10/2020 (Shift-III) Ans. (d) : For the above idiom, "Eat humble pie" option (d) 'Admit that you are wrong' gives an appropriate explanation. Other options have different meaning. 195. Select the most appropriate meaning of the given idiom. Every cloud has a silver lining (a) Something irritating is there in any joyous 190. Select the most appropriate idiom for the given situation statement. (b) Something disheartening is there in any One who hurries up always gets the best. happy situation (a) Brevity is the soul of wit. (c) Something interesting is there in any colorful (b) Forewarned is forearmed. situation (c) early bird catches the worm. (d) Something promising is there in any joyous (d) His bark is worse than his bite. situation SSC CHSL (Tier-I) 04/08/2023 (Shift-III) SSC CHSL (Tier-I) 19/03/2020 (Shift-I)

E

Idoms & Phrases

670

YCT

Ans. (d) : For the above idiom/phrase "Every cloud has a silver lining" option (d) ‘something promising is there in any joyous situation’ gives an appropriate explanation. Other options have different meaning. 196. In the following question, out of the given four alternatives, select the alternative which best expresses the meaning of the Idiom/Phrase. To end in smoke. (a) To do one's duty well (b) To not agree to something (c) To be the cause of a quarrel (d) To come to nothing SSC CGL (Tier-I) – 10/06/2019 (Shift-III) SSC MTS – 02/08/2019 (Shift-II) Ans. (d) : For the above idiom/phrase "to end in smoke" option (d) ‘to come to nothing’ gives an appropriate explanation. Other options have different meaning. 197. Select the most appropriate meaning of the given idiom. Eye wash (a) A problem (b) A fact (c) A solution (d) A deception SSC MTS – 06/08/2019 (Shift-II) Ans. (d) : For the above idiom/phrase "Eye wash" option (d) ‘A deception’ gives an appropriate explanation other options have different meaning.

(a) (b) (c) (d)

A person out of accustomed environment To kill somebody To criticize someone A dead person SSC CHSL (Tier-I) 09/03/2023 (Shift-IV) Ans. (a) : For the above Idiom, "A fish out of water" option (a) "A person out of accustomed environment" gives an appropriate explanation. Other options give different meaning. 201. Select the idiom that gives the most appropriate meaning of the underlined phrase in the following sentence. I finally understood how to operate the new smart television. (a) Don’t know (b) Figured out (c) Hit the sack (d) Cast aside SSC Selection Posts XI-30/06/2023 (Shift-IV) Ans. (b) : For the above sentence the underlined word 'finally understood' option (b) 'figure out' is appropriate Idioms/Phrase. Other options give different meaning.

202. Select the most appropriate meaning of the given idiom. Full steam ahead (a) Miss the train before reaching (b) Make a train journey (c) Move onward with determination (d) Move very noisily SSC Constable GD-01/12/2021 Shift-I 198. Select the most appreciate meaning of the idiom in bold in the given sentence. Ans. (c) : For the above idiom 'full steam ahead', option Because the shop is closing down, most of the (c) 'Move onward with determination' gives suitable stock is going for a song. meaning. Other options are not correct. (a) Incredibly expensive 203. Select the most appropriate meaning of the (b) Incredibly inexpensive given idiom. (c) Something that is soothing to your ears Feel at sea (d) Receive punishment (a) To travel always by sea SSC GD 12/01/2023 (Shift-II) (b) To experience sea for the first time Ans. (b) : The most appropriate meaning of the given (c) To feel exhausted idiom 'For a song' means 'Incredibly inexpensive'. (d) To feel lost or confused 199. Selec the most appropriate meaning of the SSC Constable GD-26/11/2021 Shift-I given idiom. Ans. (d) : For the above idiom 'Feel at sea', option (d) Face the music to feel lost or confused' gives suitable meaning. Other (a) Music for peace and health options are not correct. (b) to face the unpleasant consequences of one's 204. Select the most appropriate meaning of the actions underlined idiom in the given sentence. (c) Listening to melodies from music albums He is foaming at the mouth about the (d) Dance according to the tune committee's decision. SSC GD 01/02/2023 (Shift-II) (a) To be enraged and show it Ans. (b) : 'To face the unpleasant consequences of (b) to cause damage to someone one's action' is the most appropriate meaning of the (c) to not recognize something good given idiom 'Face the music'. (d) to be upset about something that happened in The other options give different meaning. the past Hence option (b) is correct answer. SSC MTS-13/07/2022 Shift-I 200. In the following question, out of the given four Ans. (a) : For the above idiom 'foaming at the mouth', alternatives, select the alternative which best option (a) 'To be enraged and show it' gives suitable expresses the meaning of the Idiom/Phrase. meaning. Other options are not correct. fish out of water

F

Idoms & Phrases

671

YCT

205. Select the most appropriate meaning of the given idiom. Foul play (a) Unfair or dishonest behavior (b) A drama which is badly produced (c) Unpleasant weather for playing (d) A bad smelling theatre or playground SSC CGL (Tier-I) 12/04/2022 Shift-I Ans. (a) : For the above idiom 'foul play', option (a) unfair or dishonest behaviors' gives suitable meaning. Other options are not correct. 206. Select the most appropriate meaning of the given idiom : fight shy of (a) to invite (b) to avoid (c) to challenge (d) to perform SSC CGL (Tier-II) 08.08.2022 Shift-II Ans. (b) : For the above idiom 'Fight shy of', option (b) to avoid gives suitable meaning. Other options are not correct. 207. Select the most appropriate meaning of the given idiom. forty winks (a) a worthless object (b) a hot day (c) a short nap (d) a brief statement SSC CGL (Tier-II) 08.08.2022 Shift-II Ans. (c) : For the above idiom 'Forty winks' option (c) 'a short nap' gives suitable meaning. Other option are not correct. 208. Select the most appropriate meaning of the given idiom. Feel at sea (a) Feel angry (b) Feel comfortable while travelling by water (c) Feel seasick (d) Feel lost or confused SSC CGL (Tier-I) –24/08/2021 (Shift-I) Ans. (d) : For the above idiom, "Feel at sea" option(d) ‘feel lost or confused’ gives an appropriate explanation. Other options have different meaning. 209. Select the most appropriate meaning of the given idiom. Full of beans (a) Full of energy (b) A dish made of French beans (c) Full of cowardice (d) A storeroom full of vegetables SSC CGL (Tier-I) –16/08/2021 (Shift-I) Ans. (a) : For the above idiom, Full of beans "option (a) ‘full of energy’ gives an appropriate explanation. Other options have different meaning. 210. Select the most appropriate meaning of the underlined idiom in the given sentence. It was quite a shock for Raju to have to foot the bill for all his friends. (a) prepare everything they wanted (b) kick away the paper bill (c) pay for everything (d) pick up the bill from the floor SSC CGL (Tier-I) – 19/06/2019 (Shift-III) Idoms & Phrases

Ans. (c) : For the above idiom/phrase, "Foot the bill" option (c) ‘pay for everything’ gives an appropriate explanation. Other options have different meaning. 211. Select the most appropriate meaning of the underlined idiom in the given sentence. Many people join politics to feather their own nest. (a) make others' life comfortable (b) promote their own interest (c) utilize black money (d) serve their country SSC CHSL (Tier-I) 04/07/2019 (Shift-I) SSC CGL (Tier-I) – 12/06/2019 (Shift-I) Ans : (b) For the above idiom/phrase "Feather their own nest" option (b) ‘promote their own interest’ gives an appropriate explanation. Other options have different meaning. 212. Select the most appropriate meaning of the underlined idiom in the sentence. My brother has decided to settle in Mumbai for good. (a) forever (b) for new opportunities (c) for a while (d) for better income SSC CPO-SI – 12/12/2019 (Shift-I) Ans : (a) For the above idiom/phrase "For good" option (a) ‘forever’ gives an appropriate explanation. Other options have different meaning. 213. Select the most appropriate meaning of the underlined idiom in the sentence. In spite of the high sounding words, his speech fell flat on the audience. (a) could not be heard clearly (b) appealed to the sentiments of the audience (c) failed to make an impact (d) impressed a lot SSC CPO-SI – 12/12/2019 (Shift-I) Ans : (c) For the above idiom/phrase "Fell flat option (c) ‘failed to make an impact’ gives an appropriate explanation. Other options have different meaning. 214. Select the most appropriate meaning of the given idiom. Four corners of the earth (a) from the very beginning (b) from morning till night (c) from all parts of the world (d) from beginning to end SSC CHSL (Tier-I) –03/07/2019 (Shift-I) Ans : (c) For the above idiom/phrase "Four corners of the earth" option (c) ‘from all parts of the world’ gives an appropriate explanation other options have different meaning. 215. Select the most appropriate meaning of the given idiom. Full of sound and fury (a) Merely loud end angry words but ineffective (b) A fiery and effective speaker (c) Someone who gets angry very quickly (d) An angry and unruly mob SSC CHSL (Tier-I) 21/10/2020 (Shift-II)

672

YCT

Ans. (a) : For the above idiom/phrase "Full of sound and fury" option (a) “merely loud and angry words but ineffective" gives an appropriate explanation. Other options have different meaning. 216. Select the most appropriate meaning of the given idiom. Flying off the handle (a) suddenly becoming enraged (b) day dreaming (c) having ideas (d) flying away being launched from a handle SSC CHSL (Tier-I) 20/10/2020 (Shift-III) Ans. (a) : For the above idiom/phrase "flying of the handle" option (a) ‘suddenly becoming enraged’ gives an appropriate explanation. Other options have different meaning. 217. Select the most appropriate meaning of the given idiom. Falling head over heels (a) Feeling deep sympathy to the needy (b) Finding fault with others (c) Falling deeply in love with someone (d) Fixing models without proper analysis SSC CHSL (Tier-I) 19/03/2020 (Shift-II) Ans. (c) : For the above idiom/phrase "Falling head over heels" option (c) ‘falling deeply in love with someone’ gives an appropriate explanation. Other options have different meaning. 218. Select the most appropriate meaning of the given idiom. Fit as a fiddle (a) Playing a musical instrument (b) Healthy and strong (c) Of the right size (d) Singing happily SSC CHSL (Tier-I) 13/10/2020 (Shift-II) Ans. (b) : For the above idiom/phrase "fit as fiddle" option (b) ‘healthy and strong’ gives an appropriate explanation. Other options have different meaning. 219. Select the most appropriate meaning of the given idiom. To feel at home (a) To feel comfortable (b) To miss one's home (c) To feel out of place (d) To be rarely at home SSC MTS – 05/08/2019 (Shift-III) Ans. (a) : For the above idiom/phrase "To feel at home" option (a) ‘To feel comfortable’ gives an appropriate explanation. Other options have different meaning. 220. Select the most appropriate meaning of the given idiom. Feel a bit under the weather (a) To be unappreciated (b) To feel sick or unhealthy (c) To be happy (d) To be calm and patient SSC MTS – 22/08/2019 (Shift-I) Ans. (b) : For the above idiom/phrase "Feel a bit under the weather" option (b) ‘to fell sick or unhealthy’ gives an appropriate explanation. Other options have different meaning. Idoms & Phrases

221. Select the most appropriate meaning of the given idiom. Forty winks (a) A short nap during the day (b) To blink repeatedly (c) To keep awake (d) A sound sleep at night SSC MTS – 13/08/2019 (Shift-II) Ans. (a) : For the above idiom/phrase "forty winks option (a) ‘A short nap during the day’ gives an appropriate explanation. Other option have different meaning. 222. Select the most appropriate meaning of the given idiom. Far cry from (a) To be very different from (b) To be sad about past things (c) To be away from dear ones (d) To cry over small things SSC MTS – 13/08/2019 (Shift-I) Ans. (a) : For the above idiom/phrase "far cry home" option (a) ‘To be very different from’ gives an appropriate explanation, other options have different meaning. 223. Select the meaning of the given idiom. Fair's fair (a) Fair complexion (b) Just treatment (c) Disappointed (d) White supremacy SSC MTS – 09/08/2019 (Shift-III) Ans. (b) : For the above idiom/phrase "fair's fair" option (b) ‘just treatment’ gives an appropriate explanation. Other options have different meaning.

G 224. Select the most appropriate meaning of the given idiom. Get down to brass tacks (a) Compromise, cooperation between people (b) To become serious about something (c) Get fired from the job (d) To be ready SSC MTS 02/05/2023 (Shift-I) Ans. (b) : The most appropriate meaning of the given idiom get down to brass tacks is 'To become serious about something'. other options are not correct. Thus, the correct answer is option (b). 225. Select the most appropriate meaning of the underlined idiom in the given sentence. It is hard to believe that many smokers did't go cold turkey even after they got lung cancer. (a) Losing sensory inputs (b) Dead (c) Quite addictive or dangerous behavior (d) Withered SSC GD 31/01/2023 (Shift-II) Ans. (c) : The most appropriate meaning of the underlined idiom 'go cold turkey' is 'Quite addictive or dangerous behavior'. Other options are not suitable.

673

YCT

226. Select the most appropriate meaning of the given idiom. Gate-crasher (a) Thick grass (b) A person who enters an event without any invitation (c) A blind person (d) A cobra that lives in gardens SSC GD 23/01/2023 (Shift-IV) Ans. (b) : The most appropriate meaning of the given idiom, 'Gate-crasher' is 'A person who enters an event without any invitation'. The other options are not suitable. 227. Select the most appropriate meaning of the given idiom. Go back to the drawing board (a) Start over (b) Mediate a task (c) Accomplish a task (d) Leave a work unfinished SSC CGL (Tier-I) 19/07/2023 (Shift-IV) Ans. (a) : The most appropriate meaning of the given idiom 'Go back to the drawing board' is 'start over. Other options give different meaning. Thus, the correct answer is option (a). 228. Choose the most appropriate meaning of the underlined phrase. His voice gets on my nerves. (a) Makes me ill (b) Makes me sad (c) Irritates me (d) Pierces my eardrums SSC CGL (Tier-I) 26/07/2023 (Shift-II) Ans. (c) : The most appropriate meaning of the underlined phrase 'gets on my nervier' is ' irritates me' It means 'something is irritating, annoying or causing frustration to the person. Other option give different meaning. 229. Select the most appropriate meaning of the given idiom. Get up on the wrong side of the bed (a) Someone who is having a good day (b) Go to bed or go to sleep (c) Destroy or ruin a plan (d) Someone who is having a horrible day SSC CGL (Tier-I) 27/07/2023 (Shift-III) Ans. (d) : The most appropriate meaning of the above given idiom 'Get up on the wrong side of the bed' is 'Someone who is having a Horrible day.' Other options give different meaning. 230. Select the most appropriate idiom for the underlined segment in the following sentence. Meera is not qualified much but has the art of speaking. (a) Gift of the gab (b) Hope against hope (c) An acid tongue (d) A lick of paint SSC CHSL (Tier-I) 09/08/2023 (Shift-III) Idoms & Phrases

Ans. (a) : Gift of the gab – (Able to Speak easily and confidently and to persuade people.) will be the most appropriate for the underlined segment 'the art of speaking' given in the above mentioned sentence. Meaning of other idioms given as option– Hope against hope – cling to a mere possibility/ Hope with little reason or justification An acid tongue – Bitingly critical or sarcastic A lick of paint – A refurbishment of something's aesthetic appearance. 231. Select the most appropriate meaning of the given idiom. Games is up (a) New scheme has been launched (b) Game is over and winner declared (c) Deception is at an end (d) Plan has been executed SSC Constable GD-09/12/2021 Shift-II Ans. (c) : For the above idiom 'Games is up' option (c) Deception is at an end gives suitable meaning. Other options are not correct. 232. Select the most appropriate meaning of the given idiom. Get out of hand (a) Work hard (b) Slow down (c) Lose control (d) Calm down SSC Constable GD-14/12/2021 Shift-III Ans. (c) : For the above idiom 'Get out of hand', option (c) 'lose control' gives suitable meaning. Other options are not correct. 233. Select the most appropriate meaning of the given idiom. Gift of the gab (a) Ability to draw sketches (b) Ability to speak in a sarcastic tone (c) Ability to speak confidently and clearly (d) Ability to please others SSC Constable GD-25/11/2021 Shift-III Ans. (c) : For the above idiom 'Gift of the Gab', option (c) Ability to speak confidently and clearly' gives suitable meaning. Other option are not correct. 234. Select the most appropriate meaning of the given idiom. Get the axe (a) Relaxed job (b) Lose the job (c) Get the job (d) Burdened job SSC CHSL 30.05.2022 Shift-I Ans. (b) : For the above idiom 'Get the axe' option (b) 'Lose the job' gives suitable meaning. Other options are not correct. 235. Select the most appropriate meaning of the given idiom. Get your act together (a) start taking action slowly (b) organize your work in a better way (c) decide to put on an act of innocence (d) take up a career in acting SSC Stenographer (Grade C & D) 12.11.2021 Shift-I

674

YCT

Ans. (b) : For the above idiom 'Get your act together', option (b) 'organize your work in a better way' is correct explanation. Other options are not correct. 236. Select the most appropriate meaning of the given diom. A good turn (a) A lucky chance (b) A great opportunity (c) A helpful act (d) A pleasant experience SSC CHSL (Tier-I) –15/04/2021 (Shift-I) Ans. (c) : For the above idiom/phrase "A good Turn" option (c) ‘A helpful act’ gives an appropriate explanation. Other options have different meaning. 237. Select the most appropriate meaning of the underlined idiom in the given sentence. My neighbour's son is very mischievous and always getting in everyone's hair. (a) Throwing things at them (b) Pulling their hair (c) Entertaining them (d) Annoying them SSC CPO-SI (Tier-II) 27/09/2019 (3 pm - 5 pm) Ans. (d) : For the above idiom "getting in everyone's hair" option (d) 'Annoying them' gives an appropriate explanation other options have different meaning. 238. Select the most appropriate meaning of the underlined idiom in the given sentence. My aunt has the gift of the gab and can socialize in any group. (a) ability to cook well (b) ability to criticize anyone (c) ability to spend time anywhere (d) ability to speak eloquently SSC CHSL (Tier-I) 05/07/2019 SSC CGL (Tier-I) 24/08/2021 (Shift-I) SSC CPO-SI (Tier-II) 27/09/2019 (3 pm - 5 pm) Ans. (d) : For the above idiom/phrase "gift of the gab" option (d) ‘ability to speak eloquently’ Gives an appropriate explanation. Other options have different meaning. 239. Select the most appropriate meaning of the underlined idiom in the given sentence. I told you not to play the prank but you didn’t listen, now face the music. (a) accept the blame (b) put on earphones (c) listen to the songs (d) sing popular songs SSC CHSL (Tier-I) 05/07/2019 Shift-III SSC CPO-SI (Tier-II) 27/09/2019 (3 pm - 5 pm) Ans. (a) : For the above idiom/phrase "Face the music" option (a) ‘accept the blame’ gives an appropriate explanation. Other options have different meaning. 240. In the following question, out of the given four alternatives, select the alternative which best expresses the meaning of the Idiom/Phrase. Give it a whirl (a) To be engaged in cheating (b) To completely destroy something (c) Attach or criticize someone (d) To try out something SSC CGL (Phase-II) 17/02/2018 Idoms & Phrases

Ans. (d) For the above idiom/phrases ‘Give it a whirl’ ,means ‘to try out something’ thus the correct answer is option (d). 241. Select the most appropriate meaning of the given idiom. Give a hand (a) assist (b) encourage (c) donate an organ (d) clap hands SSC CPO-SI – 23/11/2020 (Shift-I) Ans. (a) : For the above idiom/phrase "Give a hand" option (a) ‘Assist’ gives an appropriate explanation. Other options have different meaning. 242. Select the most appropriate meaning of the given idiom. Grin from ear to ear (a) to make funny faces (b) to have very sharp ears (c) to ridicule someone (d) to smile a lot because of happiness SSC CPO-SI – 23/11/2020 (Shift-II) Ans. (d) : For the above idiom/phrase 'Grin from ear to ear' option (d) ‘to smile a lot because of happiness’ gives an appropriate explanation other options have different meaning. 243. Select the most appropriate meaning of the given idiom. Get your money's worth (a) to get something of poor quality for free (b) to be cheated in a purchase (c) to buy something very expensive (d) to pay and get something of good value SSC CPO-SI – 25/11/2020 (Shift-I) Ans. (d) : For the above idiom/phrase "Get your money's worth" option (d) ‘to pay and get something of good value’ gives an appropriate explanation. Other options have different meaning. 244. Select the most appropriate meaning of the underlined idiom in the given sentence. Mrs. Arora has got the green light from the principal to organics a science conference in the college. (a) Got fired for doing something (b) Got permission to go ahead with something (c) Got the work started (d) Got energy and motivation to do something SSC CPO-SI – 09/12/2019 (Shift-I) Ans : (b) For the above idiom/phrase "got the green light" option (b) ‘got permission to go ahead with something’ gives an appropriate explanation. Other options have different meaning. 245. Select the most appropriate meaning of the underlined idiom in the sentence. 'The situation is getting out of hand' the manager said to us. (a) going to be handled soon (b) not in control anymore (c) can be passed on to someone else (d) not growing fast enough SSC CGL (Tier-I) – 04/06/2019 (Shift-II) SSC CPO-SI – 12/12/2019 (Shift-II)

675

YCT

Ans : (b) For the above idiom/phrase ‘getting out of hand’. option (b) ‘not in control anymore’ gives an appropriate explanation. Other options have different meaning. 246. Select the most appropriate meaning of the given idiom. Go down in flames (a) crash land an Aeroplane (b) fail completely (c) get completely burnt (d) get hit by a burning cracker SSC CPO-SI – 13/12/2019 (Shift-II) Ans. (a) : For the above idiom/phrase "Go down in flames" option (a) ‘crash land an aeroplane’ gives an appropriate explanation. Other options have different meaning. 247. Select the most appropriate meaning of the given idiom. GREEN THUMB (a) to have talent in gardening (b) to have talent in painting (c) to be angry (d) to be envious SSC CHSL (Tier-I) –04/07/2019 (Shift-II) Ans : (a) For the above idiom/phrase "Green thumb" option (a) ‘to have talent in gardening’ gives an appropriate explanation other options have different meaning. 248. Select the most appropriate meaning of the given idiom. To grease the palm (a) to give support (b) to bribe someone (c) to cheat someone (d) to give a massage SSC CHSL (Tier-I) –04/07/2019 (Shift-III) Ans : (b) For the above idiom/phrase "To grease the palm" option (b) ‘to bribe’ someone gives an appropriate explanation. Other options have different meaning. 249. Select the most appropriate meaning of the given idiom. Going over one's head (a) unable to take a decision (b) beyond one's capacity to understand (c) unable to function as one used to (d) something one didn't expect SSC CHSL (Tier-I) –03/07/2019 (Shift-III) Ans. (b) : For the above idiom/phrase "Going over one's head option (b) ‘beyond one's capacity to understand’ gives an appropriate explanation. Other options have different meaning. 250. Select the most appropriate meaning of the given idiom. Get something off your chest (a) Remove something from you as it is troubling you a lot (b) Have difficulty in doing something that is quite easy (c) Express something that has been worrying you, and you want to say (d) Tell people some good news that you know and you want to share SSC CHSL (Tier-I) 16/10/2020 (Shift-III) Idoms & Phrases

Ans. (c) : For the above idiom/phrase "Get something off your chest" option (c) ‘express something that has been worrying you and you want to say’ gives an appropriate explanation. 251. Select the most appropriate meaning of the given idiom. Go off at a tangent (a) go on at great length (b) Change the subject immediately (c) Become boisterous (d) Forget things in between SSC CHSL (Tier-I) 19/10/2020 (Shift-III) Ans. (b) : For the above idiom/phrase "Go off at a tangent" option (b) ‘change the subject immediately’ gives an appropriate explanation. Other options have different meaning. 252. Select the most appropriate meaning of the underlined idiom in the given sentence. It took Rohan a long time to get the hang of the machine in the new factory. (a) Clear and dry (b) Bring someone to run (c) Learn how to use (d) Put up safely SSC CHSL (Tier-I) 13/10/2020 (Shift-II) Ans. (c) : For the above idiom/phrase "Get the hang of" option (c) ‘learn how to use’ gives an appropriate explanation other options have different meaning. 253. In the following question, out of the given four alternatives, select the one which best expresses the meaning of the Idiom/Phrase. To get into hot waters (a) To influence (b) To get into difficulties (c) To became too excited (d) To give a correct decision SSC MTS 11-10-2017 (Shift-I) Ans : (b) For the above idiom/phrase "To get into hot waters" option (b) ‘to get into difficulties’ gives an appropriate explanation. Other options have different meaning. 254. Select the most appropriate meaning of the given idiom. Be glad to see the back of (a) To avoid someone (b) To be happy when a person leaves (c) To be happy to receive guests (d) To support someone SSC MTS – 22/08/2019 (Shift-III) Ans. (b) : For the above idiom/phrase "be glad to see the back of" option (b) ‘to be happy when a person leaves’ gives an appropriate explanation other options have different meaning. 255. Select the most appropriate meaning of the given idiom. Go getter (a) A real achiever (b) One who does the right thing (c) One who runs away (d) A greedy person SSC MTS – 05/08/2019 (Shift-III)

676

YCT

Ans. (a) : For the above idiom/phrase "Go getter" option (a) ‘a real achiever or someone who is energetic and works’ gives an appropriate explanation. Other options have different meaning. 256. Select the most appropriate meaning of the given idiom. Get someone's goat (a) To cheat someone (b) To hurt someone (c) To irritate someone (d) To abuse someone SSC MTS – 21/08/2019 (Shift-II) Ans. (c) : For the above idiom/phrase "Get someone's got" option (c) ‘To irritate someone’ gives an appropriate explanation. Other options have different meaning. 257. Select the most appropriate meaning of the given idiom. Get the ball rolling (a) To spin the ball (b) To cheer someone up (c) To start doing something (d) To have an active lifestyle SSC MTS – 19/08/2019 (Shift-II) Ans. (c): For the above idiom/phrase "Get the ball rolling" option (c) ‘to start doing something’ gives an appropriate explanation. Other options have different meaning. 258. Select the most appropriate meaning of the given idiom. Get a foot in the door (a) To have a chance to do something (b) To block the path (c) To be injured (d) To be lucky SSC MTS – 16/08/2019 (Shift-III) Ans. (a) : For the above idiom/phrase "Get a foot in the door" option (a) ‘to have a chance to do something’ gives an appropriate explanation. Other options have different meaning. 259. Select the most appropriate meaning of the given idiom. Give someone the cold shoulder (a) To display superiority (b) To serve cold food (c) To involve in a verbal dispute (d) To ignore someone SSC CGL (Tier-I) 06/06/2019 (Shift-I) SSC MTS – 14/08/2019 (Shift-II) Ans. (d) : For the above idiom/phrase "Give someone the cold shoulder" option (d) ‘To ignore someone’ gives an appropriate explanation. Other options have different meaning. 260. Select the option that means the same as the given idiom. The grass is greener on the other side (a) Things appear better for others (b) Many things looks like green grass (c) Green grass grows everywhere (d) Only green grass grows well SSC GD – 18/02/2019 (Shift-III) Ans. (a) : For the above idiom/phrase "The grass is greener on the other side option (a) ‘thing appear better for others’ gives an appropriate explanation. Other options have different meaning. Idoms & Phrases

H 261. In the following question, out of the given four alternatives, select the alternative which best expresses the meaning of the Idiom/Phrase. have the heart of stone. (a) To be fearless (b) No being able to make a decision (c) To feel no pity (d) To have a hidden agenda SSC MTS 08/05/2023 (Shift-III) Ans. (c) : For the above given sentence, the best empress meaning of the idiom/ phrase 'have the heart of stone' is ' To feel no pity'. Thus, the correct answer is option (c) 262. In the following question, out of the given four alternatives, select the alternative which best expresses the meaning of the Idiom/Phrase. have a foot in the grave (a) To trip somebody over (b) To take risks (c) To be near death (d) To hire a contract killer SSC MTS 08/05/2023 (Shift-III) Ans. (c) : For the above given sentence, the best empress meaning of the idiom/phrase 'have a foot in the grave' is 'To be near death'. Thus, the correct answer is option (c) 263. Select the most appropriate meaning of the given idiom. Head over heels (a) To be very angry (b) To have immense Patience (c) To be madly in love (d) To have dispute SSC GD 08/02/2023 (Shift-IV) Ans. (c) : 'To be madly in love' is the most appropriate meaning of the given idiom– 'Head over heels'. Head over heels means :- completely in love. Hence option (c) 'To be madly in love' is correct answer. 264. Select the most appropriate meaning of the given idiom. Have your heart in your mouth (a) Feeling extremely nervous (b) Doing immediately (c) Not being of your kind (d) Accusing someone of faults which you have SSC GD 08/02/2023 (Shift-IV) Ans. (a) : 'Feeling extremely nervous' is the most appropriate meaning of the given idiom 'Have your heart in your mouth'. 'Have your heart in your mouth' means :- Anxious and worried. Hence, option (a) is the correct answer.

677

YCT

265. Select the most appropriate meaning of the given idiom. Have a bash (a) to spoil a moment (b) to betray a close friend (c) to make an attempt (d) to put something on hold SSC CHSL (Tier-I) 09/08/2023 (Shift-III) Ans. (c) : The given idiom 'Have a bash' means 'to make an attempt', which has been given in option (c). Have a bash (idiom) is used to say, try to do something you have not done before, despite there being least chance of success. other options do not give appropriate meaning. 266. Select the idiom that gives the most appropriate meaning of the underlined phrase in the following sentence. ‘Honesty is the best policy’ does not remain valid in the present times of corruption and greed. (a) Go through thick and thin (b) Hold good (c) Keep pace with (d) Put up SSC Selection Posts XI-27/06/2023 (Shift-I) Ans. (b) : The most appropriate idiom of the above underlined phrase 'remain valid' is option (b) 'Hold good. Other options give different meanings – Go through thick and thin – In good and bad times, Keep pace with – To move, increase, change, etc. at the same speed as someone/something. Put up – To accept something that is annoying, unpleasant etc. 267. Select the most appropriate meaning of the given idiom. Hang up one's boots (a) Tease an opponent (b) Show disrespect (c) Retire from a sport (d) Challenge someone SSC Constable GD-08/12/2021 Shift-I Ans. (c) : For the above idiom 'Hang up one's boots', option (c) 'Retire from a sport' gives suitable meaning other options are not correct. 268. Select the most appropriate meaning of the given idiom. have a whale of a time (a) Be extremely busy with work (b) A good thing that seems bad at first (c) Have an exceptionally fun or exciting experience (d) Immediate involvement of everyone present SSC MTS-12/07/2022 Shift-I Ans. (c) : For the above idiom 'have a whale of a time' option (c) 'Have an exceptionally fun or exciting experience' gives suitable meaning. Other options are not correct. Idoms & Phrases

269. Select the most appropriate meaning of the given idiom. Hold the key (a) To own a property (b) To have the right of succession (c) To keep a secret (d) To have control of something SSC CGL (Tier-I) 13/04/2022 Shift-II Ans. (d) : For the above idiom' Hold the key', options (d) to have control of something gives suitable meaning. Other options are not correct. 270. Select the most appropriate meaning of the given idiom. Have a change of heart (a) Fall in love with someone (b) Change one's opinion (c) Express happiness about something (d) Undergo a surgery SSC CGL (Tier-I) 20/04/2022 Shift-III Ans. (b) : For the above idiom 'Have a change of heart' option (b) change one's opinion gives suitable meaning. Other options are not correct. 271. Select the most appropriate meaning of the given idiom. Hard and fast (a) Strict (b) Badly affected (c) Quick witted (d) Urgent SSC CHSL 26.05.2022 Shift-I Ans. (a) : For the above idiom 'Hard and fast' option (a) strict gives suitable meaning. Other options are not correct. 272. Select the most appropriate meaning of the given idiom. Hit the nail on the head (a) hit someone hard on the head (b) say something that is exactly right (c) hurt someone with nails (d) say or do something that is painful SSC Stenographer (Grade C & D) 15.11.2021 Shift-I Ans. (b) : For the above idiom 'Hit the nail on the head' option (b) say something that is exactly right gives suitable meaning. Other options are not correct. 273. Select the most appropriate meaning of the given idiom. Hit a dead end (a) reach a situation that leads to nothing further (b) look for a place that is quiet (c) reach a place that is quite dead (d) have a serious problem with someone SSC Stenographer (Grade C & D) 15.11.2021 Shift-I Ans. (a) : For the above idiom 'Hit a head end', option (a) 'reach a situation that leads to nothing further' gives suitable meaning. Other options are not correct. 274. Select the most appropriate meaning of the given idiom. hold your horses (a) not get upset (b) aim high (c) fight trouble (d) slow down SSC CGL (Tier-II) 08.08.2022 Shift-II

678

YCT

Ans. (d) : For the above idiom 'Hold your horses', option (d) 'slow down' gives suitable meaning other options are not correct. 275. Select the most appropriate meaning of the given idiom. Hang up your boots (a) take good care of your shoes (b) keep away your shoes carefully (c) retire from playing a game (d) be tired and go to sleep early SSC Stenographer (Grade C & D) 12.11.2021 Shift-I Ans. (c) : For the above idiom, 'Hang up your boots', option (c) 'retire from playing a game' gives correct explanation. Other options are not correct. 276. Select the most appropriate meaning of the given idiom. To have eggs on one's face (a) To be embarrassed because of one's action (b) To apply egg yolk on one's face for making it look beautiful (c) To throw eggs on a speaker's face (d) To buy eggs and break them SSC CHSL (Tier-I) –05/08/2021 (Shift-I) Ans. (a) : For the above idiom/phrase "To have Eggs on one's face" option (a) ‘to be embarrassed because of one's action’ gives an appropriate explanation other options have different meaning. 277. Select the most appropriate meaning of the given idiom. Herculean task (a) A job that requires a lot of talking and information (b) A job that requires a large amount of physical strength (c) A job that only weak people are assigned (d) A job that makes everyone feel proud SSC CGL (Tier-I) –17/08/2021 (Shift-I) Ans. (b) : For the above idiom/phrase "Herculean task" option (b) ‘A job that requires a large amount of physical strength’ gives an appropriate explanation. Other options have different meaning. 278. Select the most appropriate meaning of the underlined idiom in the given sentence: If we lay our heads together, we will surely find a solution. (a) work in consultation (b) work in isolation (c) take a break (d) rest for a while SSC CGL (Tier-II) 16/11/2020 (3 pm - 5 pm) Ans. (a) : For the above idiom/phrase "Head together" option (a) ‘work in consultation’ gives an appropriate explanation. Other options have different meaning. 279. Select the most appropriate meaning of the underlined idiom in the given sentence. This problem is a hard nut to crack, it will take longer than they imagined. (a) involves breaking nuts (b) is not interesting enough (c) is difficult to solve (d) needs a lot of work SSC CPO-SI (Tier-II) 27/09/2019 (3 pm - 5 pm) Ans. (c) : For the above idiom/phrase "Hard nut to crack" option (c) ‘is difficult to solve’ gives an appropriate explanation. Other options have different meaning. Idoms & Phrases

280. Select the most appropriate meaning of the underlined idiom in the given sentence. Pradeep was so tired that he hit the sack as soon as possible. (a) Left work (b) Went to bed (c) Accepted defeat (d) Kicked the sack SSC CPO-SI (Tier-II) 27/09/2019 (3 pm - 5 pm) Ans. (b) : For the above idiom/phrase "hit the sack" option (b) ‘went to bed’ gives an appropriate explanation. Other options have different meaning. 281. Select the most appropriate meaning of the given idiom. High time (a) past the appropriate time (b) time to celebrate something (c) well in time for something (d) time to fly high SSC CPO-SI – 25/11/2020 (Shift-II) Ans. (a) : For the above idiom/phrase "High time" option (a) ‘Past the appropriate time’ gives an appropriate explanation. Other options have different meaning. 282. Select the most appropriate meaning of the given idiom. Handle with kid gloves (a) to treat someone with extreme care (b) to accomplish something too ambitious (c) to show someone your creativity (d) to remove all hurdles to start something SSC CPO-SI – 09/12/2019 (Shift-II) Ans : (a) For the above idiom/phrase "Handle with kid gloves" option (a) ‘to treat someone with extreme care’ gives an appropriate explanation. Other options have different meaning. 283. Select the most appropriate meaning of the given idiom hand in glove (a) working together in the same office (b) wearing gloves while working (c) in partnership for something dishonest (d) doing work in the garden SSC CHSL (Tier-I) –09/07/2019 (Shift-II) Ans : (c) For the above idiom/phrase "Hand glove" option (c) ‘In partnership for something dishonest’ gives an appropriate explanation. Other options have different meaning. 284. Select the most appropriate meaning of the given idiom. Have one's hands full (a) to be very clumsy (b) to be very busy (c) to be very rich (d) to be very generous SSC CHSL (Tier-I) –05/07/2019 (Shift-III) Ans : (b) For the above idiom/phrase "Have one's hand full 'option (b) ‘to be very busy’ gives an appropriate explanation. Other options have different meaning. 285. Select the most appropriate meaning of the given idiom. HEAD IN THE CLOUDS (a) to be very tall (b) to feel giddy (c) to be very lazy (d) to daydream SSC CHSL (Tier-I) –04/07/2019 (Shift-II)

679

YCT

Ans : (d) For the above idiom/phrase "To have ones head in the clouds" option (d) ‘to daydream’ gives an appropriate explanation. Other options have different meaning. 286. Select the most appropriate meaning of the given idiom. Hanging by a thread (a) in a sorry or humble state (b) be extremely weak (c) unable to act as desired (d) be in a dangerous situation SSC CHSL (Tier-I) –03/07/2019 (Shift-II) Ans. (d) : For the above idiom/phrase "Hanging by a thread" option (d) be in a dangerous situation gives an appropriate explanation. Other options have different meaning. 287. Select the most appropriate meaning of the underlined idiom. Hats off to someone (a) Given hats to (b) Make new hats for (c) Announce a prize for (d) Congratulations to SSC CHSL (Tier-I) 17/03/2020 (Shift-I) Ans. (d) For the above idiom/phrase "Hats off to" option (d) ‘congratulations to' gives an appropriate explanation. Other options have different meaning. 288. Select the most appropriate meaning of the given idiom. Hang up your boots (a) to be rejected (b) To take retirement (c) To dry your shoes (d) To hang upside down SSC MTS – 20/08/2019 (Shift-I) Ans. (b) : For the above idiom/phrase "hang up your boots' option (b) ‘to take retirement’ gives an appropriate explanation. Other options have different meaning. 289. Select the meaning of the given idiom. Hang one's head (a) To be ashamed (b) To be proud (c) To punish (d) To be greedy SSC MTS – 13/08/2019 (Shift-III) Ans. (a) : For the above idiom/phrase "Hang one's head" option (a) ‘to be ashamed’ gives an appropriate explanation. Other options have different meaning.

I 290. Select the most appropriate meaning of the bracketed idiom in the given sentence. That were remarks made (in the heat of the moment). (a) To be dismissed from your job (b) To exert all efforts (c) At time when you cannot think carefully (d) To present a counter argument SSC MTS 19/05/2023 (Shift-III) Ans. (c) : The most appropriate meaning of the bracketed idiom (in the heat of the moment) is ‘At time when you connot think carefully’. Other options give different meanings. Idoms & Phrases

291. Identify the idiom/phrase that can best substitute the underlined segment. I’m going to be in trouble if I don’t submit my homework to the teacher. (a) off base (b) up the creek (c) the lion’s share (d) a devil’s advocate SSC CGL (Tier-I) 19/07/2023 (Shift-IV) Ans. (b) : The most appropriate idiom/phrase for the underlined segment 'in trouble' will be 'up the creek' which means being in a difficult or challenging situation. Thus, the correct answer is option (b). 292. Select the most appropriate option that can substitute the underlined words in the given sentence. Our former minister Mr. Sikhar is in the running from Shrinet Nagar for the post of Member of Legislative Assembly. (a) giving speeches (b) making laws (c) teaching students (d) contesting the elections SSC CHSL (Tier-I) 09/08/2023 (Shift-III) Ans. (d) : The underlined word 'in the running of the given sentence can be substituted by the words mentioned in option (d) contesting the elections. Because 'In the running' is an idiom which means (in contention for victory or position' or contesting for being successful in an election.) 293. Select the most appropriate meaning of given idiom. In Vain (a) to no purpose (b) near to perfection (c) totally dilute (d) remove suddenly SSC CHSL (Tier-I) –05/07/2019 (Shift-II) Ans. (a) : For the above idiom, "In vain" option (a) 'to no purpose' gives an appropriate explanation. Other option have different meaning. 294. Select the most appropriate meaning of the given idiom. In the heat of the moment (a) Saying or doing something without thinking due to anger (b) keep something away from everyone (c) To share information that was previously concealed (d) In a confused or befuddled state of mind SSC MTS-14/07/2022 Shift-III Ans. (a) : For the above idiom 'In the heat of the moment', option (a) 'Saying or doing something without thinking due to anger' gives suitable meaning. Other options are not correct. 295. Select the most appropriate meaning of the given idiom. In the same boat (a) Having fun (b) Sharing a ride (c) In a similar situation (d) To be calm SSC Constable GD-26/11/2021 Shift-I

680

YCT

Ans. (c) : For the above idiom 'In the same boat', option (c) 'In a similar situation;' gives suitable meaning. Other options are not correct. 296. Select the most appropriate meaning of the underlined idiom in the given sentence. I want you to give me this contract in black and white. (a) in writing (b) quickly (c) in the morning (d) at night SSC MTS-07/07/2022 Shift-II Ans. (a) : For the above idiom' In black and white', option (a) 'in writing' gives suitable meaning. Other options are not correct. 297. Select the most appropriate meaning of the given idiom. In a nutshell (a) Extremely pale (b) Very briefly (c) Under protection (d) Without delay SSC MTS-05/07/2022 Shift-III Ans. (b) : For the above idiom 'In a nutshell', option (b) very briefly gives suitable meaning. Other options are not correct. 298. Select the most appropriate meaning of the highlighted idiom. My mother is in raptures over her new stove. (a) completely dissatisfied with (b) extremely delighted (c) extremely angry (d) working vigorously SSC MTS-15/07/2022 Shift-I Ans. (b) : For the above idiom 'in Raptures', option (b) 'extremely delighted' gives suitable meaning. Other options are not correct. 299. Select the most appropriate meaning of the given idiom. In the same breath (a) Try and hold your breath (b) Practice breathing exercise (c) Say two contradictory things at the same time (d) Able to get a foul smell SSC CGL (Tier-I) 11/04/2022 Shift-I Ans. (c) : For the above idiom' In the same breath' option (c) say two contradictory things at the same time' gives suitable meaning. Other options are not correct. 300. Select the most appropriate meaning of the given idiom. In the limelight (a) A well kept secret (b) In a well lit up room (c) The centre of attraction (d) About to be successful SSC CGL (Tier-I) 21/04/2022 Shift-III Ans. (c) : For the above idiom 'In the limelight' option (c) 'the centre of attraction' gives suitable meaning. Other options are not correct. Idoms & Phrases

301. Select the most appropriate meaning of the given idiom. In high spirits (a) mad (b) joyful (c) angry (d) hopeful SSC Stenographer (Grade C & D) 12.11.2021 Shift-I Ans. (b) : For the above idiom 'In high spirits', option (b) 'Joyful' gives correct explanation other options are not correct. 302. Select the most appropriate meaning of the given idiom: In the nick of time (a) The beginning of a month (b) A special occasion (c) At the last possible moment (d) A long delay SSC MTS – 16/08/2019 (Shift-I) SSC Stenographer Grade C&D –05/02/2019 (3:5pm)

Ans. (c) : For the above idiom "In the nick of time" option (c) ‘At the last possible moment’ gives an appropriate explanation. Other options have different meaning. 303. Select the most appropriate meaning of the underlined idiom in the given sentence. We now have an ex-Minister in the running from our constituency for the post of Member of Parliament. (a) jogging everyday (b) giving speeches (c) contesting the seat (d) teaching yoga SSC CGL (Tier-I) – 13/06/2019 (Shift-I) Ans : (c) For the above idiom/phrase 'In the running' option (c) ‘Contesting the seat’ gives an appropriate explanation. Other options have different meaning. 304. Select the most appropriate meaning of the given idiom. in the pink (a) in pink dress (b) in the limelight (c) in a happy mood (d) in good health SSC CGL (Tier-I) – 07/06/2019 (Shift-I) Ans : (d) For the above idiom/phrase "in the pink" option (d) In good health’ gives an appropriate explanation. Other options have different meaning. 305. Select the most appropriate meaning of the underlined idiom in the given sentence. The abrogation of article 370 was achieved in the teeth of tough resistance. (a) in direct opposition to (b) in a short span of time (c) in a state of uncertainty (d) in collaboration with SSC CPO-SI – 11/12/2019 (Shift-II) Ans : (a) For the above idiom/phrase "In the teeth" option (a) ‘in direct opposition to’ gives an appropriate explanation. Other options have different meaning. 306. Select the most appropriate meaning of the given idiom In a nutshell (a) Add nuts into something (b) Put everything in a shell (c) Packed very tightly (d) Express very briefly SSC GD – 10/02/2019 (Shift-II) SSC CHSL (Tier-I) –11/07/2019 (Shift-II)

681

YCT

Ans : (d) For the above idiom/phrase "In a nutshell" 312. Select the most appropriate meaning of the given idiom. option (d) ‘express very briefly’ gives an appropriate Jump on the bandwagon explanation. Other options have different meaning. (a) Do an activity that is fun 307. Select the most appropriate meaning of the (b) Get engaged in an activity that is very given idiom. important In a pickle (c) Do something because circumstances forces (a) in jubilation (b) in deep sorrow you to (c) in trouble (d) in great pain (d) Get involved in an activity because it is likely SSC CHSL (Tier-I) –08/07/2019 (Shift-II) to succeed Ans : (c) For the above idiom/phrase "In a pickle" SSC CHSL (Tier-I) 14/10/2020 (Shift-II) option (c) In trouble’ gives an appropriate explanation. Ans. (d) : For the above idiom/phrase "Jump on the Other options have different meaning. bandwagon" option (d) ‘Get involved in an activity 308. Select the most appropriate meaning of the because it is likely to succeed’ gives an appropriate given idiom. explanation. Other options have different meaning. In a fog 313. Select the meaning of the given idiom. (a) Trapped in foggy weather To have a jaundiced eye (b) Confused and dazed (a) To be very pleased (c) Disturbed and unhappy (b) To have yellowish eyes (d) Sightless and blind (c) To be prejudiced SSC CHSL (Tier-I) 19/03/2020 (Shift-III) (d) To have sore eyes SSC MTS – 08/08/2019 (Shift-II) Ans. (b) : For the above idiom/phrase "In a fog" option (b) ‘confused and dazed’ gives an appropriate Ans. (c) : For the above idiom/phrase "To have a explanation. Other options have different meaning. jaundiced eye" option (c) ‘to be prejudiced’ gives an 309. In the following question, out of the four given appropriate explanation. Other options have different alternatives, select the alternatives which best meaning. express the meaning of the Idiom/Phrase. In deep water (a) In great difficulties 314. In the following question, out of the given four (b) With strength and fury alternatives, select the alternative which best (c) Using unfair means expresses the meaning of the Idiom/Phrase. (d) Struck by thunder Know the ropes SSC MTS 9-10-2017 (Shift-II) (a) To reveal a secret Ans. (a) : For the above idiom/phrase "In deep water" (b) To fight without any reason option (a) 'in great difficulties' gives an appropriate (c) To understand the details explanation. Other options have different meaning. (d) To break the rules SSC MTS 15/05/2023 (Shift-I) Ans. (c) : The meaning of the above given idiom/ 310. Select the most appropriate idiom for the given phrase- 'know the ropes' is 'to understand the details'. situation. 315. In the following question, out of the given four I was very excited about my new assignment. I alternatives, select the alternative which best sent the email without the required attachment. expresses the meaning of the Idiom/phrase. kill What have I done? two birds with one stone. (a) As swift as an eagle (b) Jumped the gun (a) The weak point in a person (c) Tested the waters (d) Shook a leg (b) To accidentally hurt someone's feelings SSC CGL (Tier-I) 20/07/2023 (Shift-I) (c) To achieves two things with a single effort (d) A work which needs tremendous efforts Ans. (b) : The most appropriate idiom for the given SSC MTS 12/05/2023 (Shift-III) situation ‘Jumped the gun’. Which means ‘acting prematurely or taking action before the appropriate Ans. (c) : The most appropriate meaning of the above time. given idiom 'kill two birds with one stone,' is 'To achieves two things with a single effort.' Thus, the correct answer is option (b) 311. Select the most appropriate meaning of the hence, the correct answer is option (c). underlined idiom in the given sentence. 316. Select the most appropriate meaning of the Any jumped the gun and announced their underlined idiom in the following sentence. plans at the dinner table. Keep an eye on the situation around, and take (a) ate quickly (b) started shooting your decision accordingly. (c) acted hastily (d) shouted loudly (a) Ignore SSC CHSL (Tier-I) 15/10/2020 (Shift-I) (b) Observe continually and carefully Ans. (c) : For the above idiom/phrase "Jumped the (c) Cries over spilt milk gun" option (c) ‘acted hastily’ gives an appropriate (d) Be aggressive with red eyes explanation. Other options have different meaning. SSC GD 27/01/2023 (Shift-I)

K

J

Idoms & Phrases

682

YCT

Ans. (b) : The most appropriate meaning of the underlined idiom 'Keep an eye on' is observe continually and carefully. The other options are not suitable. 317. Select the sentence that has the most appropriate meaning of the given idiom. Kill two birds with one stone (a) He ruined his chances of getting into college as he got distracted while studying for two different entrance tests and not perfecting even a single one. (b) Today’s fishing session was a success as both the fishermen were working together. (c) He was so unlucky that he never won a single coin toss in the entire tournament. (d) Maya decided to clean the house but at the end of the day, she realised it was a great workout. SSC Selection Posts XI-28/06/2023 (Shift-III) Ans. (d) : For the above Idiom, "Kill two birds with one stone" option (d) 'Maya decided to clean the house but at the end of the day, she realised it was a great workout'. It gives appropriate meaning. Other options give different meaning. 318. Select the most appropriate meaning of the given idiom. Keep body and soul together (a) Live happily (b) Live as long as possible (c) Be mentally and physically fit (d) Stay alive during hard times SSC MTS-18/07/2022 Shift-I Ans. (d) : For the above idiom 'keep body and soul together' option (d) stay alive during hard times gives suitable meaning. Other options are not correct. 319. Select the most appropriate meaning of the given idiom .......................... knock down. (a) To succeed (b) To accept (c) To demolish (d) To re do SSC CHSL 27.05.2022 Shift-II Ans. (c) : For the above idiom 'knock down', option (c) 'To demolish' gives suitable meaning. Other options are not correct. 320. Select the most appropriate meaning of the given idiom. A kick in the teeth (a) A grave setback (b) Severe toothache (c) Removing tooth cavities (d) Hitting someone hard SSC CGL (Tier-I) –18/08/2021 (Shift-I) Ans. (a) : For the above idiom/phrase "A kick in the teeth option (a) ‘A grave setback’ gives an appropriate explanation. Other options have different meaning. 321. Select the most appropriate meaning of the given idiom. Kith and kin (a) family relations (b) distant enemies (c) neighbours (d) colleagues SSC CPO-SI – 24/11/2020 (Shift-I) Idoms & Phrases

Ans. (a) : For the above idiom/phrase "kith and kin" option (a) ‘family relations’ gives an appropriate explanation. Other options have different meaning. 322. Select the most appropriate meaning of the given idiom. Know like the back of one’s hand (a) to hit someone with the back of one’s hand (b) to be unaware of the situation (c) to copy another’s trade secrets (d) to have detailed knowledge of something SSC CPO-SI – 23/11/2020 (Shift-II) Ans. (d) : For the above idiom/phrase "know like the back of one's hand" option (d) ‘to have detailed knowledge of something or to know something completely’ gives an appropriate explanation. Other options have different meaning. 323. Select the most appropriate meaning of the underlined idiom in the given sentence. Extreme poverty made the poor woman how long she could keep the wolf from the door. (a) live on charity (b) keep wild animals away (c) avoid starvation (d) be safe in her house SSC CGL (Tier-I) – 10/06/2019 (Shift-III) Ans. (c) : For the above idiom/phrase "keep the wolf from the door" option (c) ‘avoid starvation’ gives an appropriate explanation. Other options have different meaning. 324. Select the most appropriate meaning of the underlined idiom in the given sentence. He kicked up a row when he was denied entry to the amusement park. (a) cried with sorrow (b) waited in a queue (c) kicked the guard (d) created a fuss SSC CPO-SI 24/11/2020 (Shift-II) SSC CGL (Tier-I) – 07/06/2019 (Shift-III) Ans : (d) For the above idiom/phrase "kick up a row" option (d) ‘created a fuss’ gives an appropriate explanation. Other options have different meaning. 325. Select the most appropriate meaning of the given idiom. Keep a civil tongue (a) maintain silence when needed (b) Argue with soft words (c) Express dissent mildly (d) Speak with great politeness SSC CHSL (Tier-I) 18/03/2020 (Shift-III) Ans. (d) : For the above idiom/phrase "keep a civil tongue" option (d) ‘speak with great politeness’ gives an appropriate explanation. Other options have different meaning. 326. Select the most appropriate of the given idiom. Keep up appearances (a) To pretend to be happier or richer so as to conceal the real situation (b) To spend a lot of time on appearances so as to lead the fashion world (c) To deliberately appear shabby and not care to impress people (d) To waste a lot of money on mindless shopping and not bother about the consequences SSC CHSL (Tier-I) 26/10/2020 (Shift-II)

683

YCT

Ans. (a) : For the above idiom/phrase “keep up appearance" option (a) ‘to be pretend to be happier or richer so as to conceal the real situation’ gives an appropriate explanation. Other options have different meaning. 327. Select the most appropriate meaning of the given idiom: Keep your shirt on (a) To remain cool (b) To celebrate (c) To keep busy (d) To work hard SSC CHSL (Tier-I) 21/10/2020 (Shift-I) Ans. (a) : For the above idiom/phrase "keep your shirt on" option (a) ‘to remain cool’ gives an appropriate explanation. Other options have different meaning. 328. Select the most appropriate meaning of the underlined idiom in the given sentence. Ashutosh kept a stiff upper lip even when he was reprimanded in the presence of clients. (a) behaved insolently (b) started sniggering (c) kept shouting (d) remained expressionless SSC CHSL (Tier-I) 20/10/2020 (Shift-II) Ans. (d) : For the above idiom/phrase "kept a stiff upper lip" option (d) ‘remained expressionless’ gives an appropriate explanation. Other options have different meaning. 329. Select the most appropriate meaning of the given idiom. Know something inside out (a) To know something from a distance (b) To know something superficially (c) To have only a slight idea about something (d) To know everything about something SSC MTS – 16/08/2019 (Shift-II) Ans. (d) : For the above idiom/phrase "know something inside out" option (d) ‘to know everything about something’ gives an appropriate explanation. Other options have different meaning. 330. Select the option that means the same as the given idiom. To kill two birds with one stone (a) To shoo away birds (b) To shout at two individuals at a time (c) To achieve two aims simultaneously (d) To do many jobs at different places SSC GD – 02/03/2019 (Shift-II) Ans. (c) : For the above idiom/phrase "kill two birds with one stone" option (c) ‘to achieve two aims simultaneously’ gives an appropriate explanation. Other options have different meaning. 331. Select the option that means the same as the given idiom. To keep an eye on (a) To keep a watch (b) To be cautious (c) To follow (d) To stare SSC GD – 11/02/2019 (Shift-II) Ans. (a) : For the above idiom/phrase "keep an eye on" option (a) ‘to keep a watch’ gives an appropriate explanation. Other options have different meaning. Idoms & Phrases

L 332. In the following question, out of the given four alternatives, select the alternative which best expresses the meaning of the Idiom/Phrase. The life and sound of the party (a) Person who brings lots of energy to the party (b) A person who brings lots of gifts (c) The only person who dances in the party (d) Chief guest of the party SSC MTS 10/05/2023 (Shift-I) Ans. (a) : For the above given, idiom 'life and sound' means 'person who brings lots of energy to the party'. Thus, the correct answer is option (a). 333. Select the most appropriate meaning of the given idiom. Light at the end of tunnel (a) Something that signals the end of a difficult situation (b) An opening at the end of a matter (c) New light coming out from darkness (d) Preparing for examinations SSC GD 12/01/2023 (Shift-II) Ans. (a) : The most appropriate meaning of the given idioms, 'Light at the end of tunnel' is something that signals the end of a difficult situation. 334. Select the most appropriate meaning of the given idiom. Lily-livered (a) Naughty (b) Comical (c) Not brave (d) Brave SSC CGL (Tier-I) 14/07/2023 (Shift-I) Ans. (c) : For the above idiom, Lily-livered” option (c) “Not brave gives appropriate explanation. Other options give different meaning. 335. Select the most appropriate meaning of the underlined idiom in the given sentence. I accidentally let the cat out of the bag about your promotion. (a) To state the wrong facts (b) To reveal a secret (c) To say something unrealistic (d) To discourage someone SSC MTS-14/07/2022 Shift-III Ans. (b) : For the above idiom 'let the cat out of the bag' option (b) 'To reveal a secret' gives suitable meaning. Other options are not correct. 336. Select the most appropriate meaning of the underlined idiom in the given sentence. The eldest sister received the lion's share of her father's property. (a) the largest portion (b) the entire amount (c) the smallest part (d) no portion at all SSC MTS-05/07/2022 Shift-II Ans. (a) : For the above idiom 'lion's share', options (a) the largest portion' gives suitable meaning. Other options are not correct.

684

YCT

337. Select the most appropriate meaning of the given idiom. Long in the tooth (a) Too uncomfortable for words (b) Too old to do something (c) Too wise for people (d) Extremely dangerous SSC MTS-07/07/2022 Shift-II Ans. (b) : For the above idiom Long in the tooth option (b) 'Too old do something gives suitable meaning. Other options are not correct. 338. Select the option that gives the most appropriate meaning of the underlined idiom. My friends succeeded because they left no stone unturned in their search for an answer to the problem. (a) depended on many people (b) looked in different places (c) made every possible effort (d) proposed good plans SSC CGL (Tier-I) 13/04/2022 Shift-II Ans. (c) : For the above idiom' Left no stone unturned', option (c) 'made every possible effort' gives suitable meaning. Other options are not correct. 339. Select the most appropriate meaning of the given idiom. Lie low (a) Try not to be noticed (b) Fly a plane at a low altitude (c) Lie down and relax after a tiring day (d) Sit on a low chair SSC CGL (Tier-I) 12/04/2022 Shift-III Ans. (a) : For the above idiom 'Lie low', options (a) 'try not to be notices' gives suitable meaning. Other options are not correct. 340. Select the most appropriate meaning of the given idiom. Let down (a) To fail (b) To care about (c) To nurse (d) To hire SSC CHSL 26.05.2022 Shift-III Ans. (a) : For the above idiom 'Let down' option (a) To fail gives suitable meaning. Other options are not correct. 341. Select the most appropriate meaning of the given idiom. Lose your touch (a) To be passed from one person to another something being very difficult to find (b) Something being very difficult to find (c) Not being as successful as previously (d) Doing someone a favour in hopes that the favour will be returned SSC CHSL 24.05.2022 Shift-III Ans. (c) : For the above idiom 'Lose your touch', option (c) Not being as successful as previously, gives suitable meaning. Other options are not correct. Idoms & Phrases

342. Select the most appropriate meaning of the given idiom. Lion's share (a) Largest share (b) Animal's share (c) Forceful share (d) Limited share SSC CHSL 30.05.2022 Shift-I Ans. (a) : For the above idiom 'Lions share', option (a) 'Largest share' gives suitable meaning Other options are not correct. 343. Select the most appropriate meaning of the given idiom. To look after (a) To anticipate (b) to remove (c) To search for (d) To take care SSC Sel. Post Phase-VIII (M.L.) 09.11.2020 (Shift-III)

Ans. (d) : For the above idiom/phrase "To look after" option (d) ‘to take care’ gives an appropriate explanation. Other options have different meaning. 344. Select the most appropriate meaning of the given idiom. Look out (a) be careful (b) locate (c) find out (d) search SSC CPO-SI – 23/11/2020 (Shift-I) Ans. (a) : For the above idiom/phrase "look out" option (a) 'be careful' gives an appropriate explanation. Other options have different meaning. 345. Select the most appropriate meaning of the given idiom. Little by little (a) Not much (b) a few (c) gradually (d) only once SSC CPO-SI – 23/11/2020 (Shift-I) Ans. (c) : For the above idiom/phrase "Little by little" option (c) ‘gradually’ gives an appropriate explanation. Other options have different meaning. 346. Select the most appropriate meaning of the underlined idiom in the given sentence. It is unfortunate that most people in the country are still living from hand to mouth. (a) saving for the future generations (b) earning wealth by selfish means (c) consuming daily what little is earned (d) doing manual labour SSC CGL (Tier-I) – 10/06/2019 (Shift-I) Ans : (c) For the above idiom/phrase "Living from hand to mouth" option (c) ‘consuming daily what little is earned’ gives an appropriate explanation. Other options have different meaning. 347. Select the most appropriate meaning of the given idiom. Lock, stock and barrel (a) immediately (b) partly (c) rarely (d) completely SSC CGL (Tier-I) – 07/06/2019 (Shift-I) Ans : (d) For the above idiom/phrase "lock, stock and barrel" option (d) ‘completely’ gives an appropriate explanation. Other options have different meaning.

685

YCT

348.

Select the most appropriate meaning of the given idiom. Lend an ear (a) to pay attention to (b) to not make trouble (c) to force someone to listen (d) to not tell someone something SSC CGL (Tier-I) – 07/06/2019 (Shift-II) Ans : (a) For the above idiom/phrase "Lend an ear" option (a) ‘to pay attention to’ gives an appropriate explanation. Other options have different meaning. 349. Select the most appropriate meaning of the given idiom. Lead someone by the nose (a) to do the right thing (b) to force someone to take the blame (c) to dominate someone (d) to quarrel with someone SSC MTS – 07/08/2019 (Shift-II) SSC CGL (Tier-I) – 06/06/2019 (Shift-II) Ans : (c) For the above idiom/phrase "lead someone by the nose" option (c) ‘to dominate someone’ gives an appropriate explanation. Other options have different meaning. 350. Select the most appropriate meaning of the given idiom. Left out in cold (a) to be indecisive (b) to stand out (c) to be criticised (d) to be ignored SSC CPO-SI – 09/12/2019 (Shift-II) Ans : (d) For the above idiom/phrase "left out in cold" option (d) ‘to be ignored’ gives an appropriate explanation. Other options have different meaning. 351. Select the most appropriate meaning of the underlined idiom in the given sentence. Trying to find my lost ring in the college is like looking for a needle in a haystack. (a) Doing something impossible (b) Doing something unimportant (c) Making a big deal of a small thing (d) Doing something secretly SSC CPO-SI – 09/12/2019 (Shift-I) Ans : (a) For the above idiom/phrase "Looking for a needle in a haystack" option (a) ‘doing something impossible’ gives an appropriate explanation. Other options have different meaning. 352. Select the most appropriate meaning of the given idiom left- handed compliment (a) praise that is given directly (b) insulting someone in front of others (c) insulting remark appearing as praise (d) making fun of someone to tease them SSC CHSL (Tier-I) –10/07/2019 (Shift-I) Ans : (c) For the above idiom/phrase 'Left-handed compliment' option (c) ‘Insulting remark appearing as Praise’ gives an appropriate explanation. Other options have different meaning. 353. Select the most appropriate meaning of the given idiom late in the day Idoms & Phrases

too old to work too dark to see anything too delayed to be of any use too ripe to eat SSC CHSL (Tier-I) –09/07/2019 (Shift-III) Ans : (c) For the above idiom/phrase 'Late in the day' option (c) ‘too delayed to be of any use’ gives an appropriate explanation. Other options have different meaning. 354. Select the most appropriate meaning of the given idiom. leave no stone unturned (a) leave the path halfway (b) not make enough efforts (c) turn everything upside down (d) try everything possible SSC CHSL (Tier-I) –01/07/2019 (Shift-III) Ans : (d) For the above idiom/phrase 'leave no stone unturned' option (d) ‘try everything possible’ gives an appropriate explanation. Other options have different meaning. 355. Select the most appropriate meaning of the given idiom. To leave someone in the lurch. (a) To put someone at ease (b) To come to compromise with someone (c) Constant source of annoyance to someone (d) To desert someone in his difficulties SSC CHSL (Tier-I) 16/10/2020 (Shift-II) Ans. (d) : For the above idiom/phrase 'leave someone in the lurch' option (d) ‘to desert someone in his difficulties’ gives an appropriate explanation. Other options have different meaning. 356. Select the most appropriate meaning of the given idiom. Let off the hook (a) Not made to wait for a long time (b) Not allowed to take part in an activity (c) Allowed to escape from blame (d) Made to pay for one's actions SSC CHSL (Tier-I) 16/10/2020 (Shift-I) Ans. (c) : For the above idiom/phrase 'Let off the hook' option (c) ‘allowed to escape from blame’ gives an appropriate explanation. Other options have different meaning. 357. Select the most appropriate meaning of the given idiom. Like talking to a brick wall (a) Fascinated (b) Frightened (c) Curious (d) Inattentive SSC CHSL (Tier-I) 13/10/2020 (Shift-I) Ans. (d) : For the above idiom/phrase 'Like talking to a brick wall' option (d) ‘inattentive’ gives an appropriate explanation. Other options have different meaning. 358. Select the most appropriate meaning of the given idiom. She declared that it was the last straw and fled. (a) the situation had become unbearable (b) it was the last time she would use a straw (c) the situation was improving (d) it was last time she would hold a straw SSC CPO - SI – 13/12/2019 (Shift-II) SSC MTS 20/08/2019 (Shift-III) SSC CHSL (Tier-I) 26/10/2020 (Shift-I)

686

(a) (b) (c) (d)

YCT

Ans. (a) : For the above idiom/phrase 'the last straw' option (a) ‘the situation become unbearable’ gives an appropriate explanation. Other options have different meaning. 359. Select the most appropriate meaning of the given idiom. Look after (a) Take care of (b) Resemble (c) Search for (d) Scold SSC CHSL (Tier-I) 20/10/2020 (Shift-I) Ans. (a) : For the above idiom/phrase 'Look after' option (a) ‘take care of’ gives an appropriate explanation. Other options have different meaning. 360. Select the most appropriate meaning of the given idiom. Look before you leap (a) Prepare yourself before jumping (b) Think carefully of the consequences (c) Pull someone out from a disaster (d) Caution others before taking a step SSC CHSL (Tier-I) 19/10/2020 (Shift-II) Ans. (b) : For the above idiom/phrase 'Look before you leap' option (b) ‘think carefully of the consequences’ gives an appropriate explanation. Other options have different meaning. 361. Select the most appropriate meaning of the given idiom. To lose one's head (a) To be beheaded for disloyalty (b) To lose self control (c) To die valiantly in a war (d) To lose one's position SSC MTS – 19/08/2019 (Shift-III) Ans. (b) : For the above idiom/phrase 'To lose one's head" option (b) ‘to lose self control’ gives an appropriate explanation. Other options have different meaning. 362. Select the most appropriate meaning of the given idiom. Let bygones be bygones (a) Neat and tidy (b) Just at the last moment (c) To remember the past (d) To forgive and forget SSC MTS – 07/08/2019 (Shift-II) Ans. (d) : For the above idiom/phrase "Let bygones be bygones" option (d) ‘to forgive and forget’ gives an appropriate explanation. Other options have different meaning. 363. Select the meaning of the given idiom in the sentence. The detective left no stone unturned to trace the culprit. (a) Took no pains (b) Resorted to unfair practices (c) Used all available means (d) Did irrelevant things SSC GD – 08/03/2019 (Shift-III) Ans. (c) : For the above idiom/phrase "left no stone unturned" option (c) ‘used all available means’ gives an appropriate explanation. Other options have different meaning. Idoms & Phrases

M 364. Select the most appropriate idiom for the underlined words in the given sentence. Playing fantasy games on internet is an easy way to make money quickly but it’s risky. (a) in for a penny in for a pound (b) not fair and square (c) making a quick buck (d) bigger fish to fry SSC CHSL (Tier-I) 10/08/2023 (Shift-I) Ans. (c) : 'An easy way to make money quickly' underlined in the given sentence, can be best described the idiom, 'making a quick buck' mentioned in option (c). Other idioms in the options means the following– In for a penny in for a pound → If one undertakes something, it must be carried through at whatever cost Not fair and square → In an dishonest way Bigger fish to fry → Have other or more important matters to attend to 365. In the following question, out of the given four alternatives, select the alternative whichbest expresses the meaning of the Idiom/Phrase. miss the bus (a) Short lived glory (b) lose an opportunity (c) Escape by a thin margin (d) A problem in a situation SSC CHSL (Tier-I) 21/03/2023 (Shift-II) Ans. (b) : For the above idiom 'To miss the bus', option (b) 'To lose an opportunity' gives suitable meaning. Other options are not correct. 366. Select the option that is closest in meaning to the underlined idiom/phrase in the given sentence. The teacher couldn't make head or tail of the students' answer. (a) appreciate (b) understand (c) overlook (d) recommend SSC Constable GD-13/12/2021 Shift-III Ans. (b) : For the above idiom 'make head or tail of', option (b) 'understand' gives suitable meaning. Other options are not correct. 367. Select the most appropriate meaning of the underlined idiom in the given sentence. When I asked for an expensive dress for my friend’s wedding, my mother reminded me that money does not grow on trees. (a) Money is hard earned and limited (b) Money grows on shrubs and there’s plenty (c) Money is like leaves of a tree and freely available (d) Money is freely available to spend SSC CGL (Tier-I) 12/04/2022 Shift-III Ans. (a) : For the above idiom 'Money does not grow on trees options (a) money is hard earned and limited gives suitable meaning. Other options are not correct.

687

YCT

368. Select the most appropriate meaning of the given idiom. Make a beeline for (a) To stand patiently in a queue (b) To move directly towards something (c) To put bees in a line (d) To try and collect all the free items SSC Stenographer (Grade C & D) 15.11.2021 Shift-I Ans. (b) : For the above idiom 'Make a beeline for', option (b) 'to move directly towards something' gives suitable meaning. Other options are not correct. 369. Select the most appropriate meaning of the given idiom. To make both ends meet (a) To have just enough money to live (b) To try to do two different things at the same time (c) To try do bring two parties together (d) To try to solve a problem between friends SSC Sel post phase – VIII 09/11/2020 (Shift-I) SSC CHSL (Tier-I) –19/04/2021 (Shift-I) Ans. (a) : For the above idiom/phrase "To make both ends meet" option (a) ‘to have just enough money to live’ gives an appropriate explanation. Other options have different meaning. 370. Select the most appropriate meaning of the given idiom : Move heaven and earth (a) Pay for something (b) Make every effort to achieve something (c) Start something new (d) Make difficult choices SSC Stenographer Grade C&D –05/02/2019 (3:5pm)

Ans. (b) : For the above idiom/phrase "Move heaven and earth" option (b) ‘make every effort to achieve something’ gives an appropriate explanation. Other options have different meaning. 371. Select the most appropriate meaning of the given idiom. Make one’s mark (a) leave a clue (b) claim a right (c) sign a deal (d) attain recognition SSC CPO-SI – 24/11/2020 (Shift-I) Ans. (d) : For the above idiom/phrase "Make one's mark" option (d) ‘attain recognition’ gives an appropriate explanation. Other options have different meaning. 372. Select the most appropriate meaning of the given idiom. Make up one's mind (a) to decide (b) to lie (c) to pretend (d) to achieve SSC CPO-SI – 25/11/2020 (Shift-II) Ans. (a) : For the above idiom/phrase "Make up one's mind" option (a) ‘to decide’ gives an appropriate explanation. Other options have different meaning. 373. Select the most appropriate meaning of the underlined idiom in the given sentence. The thieves made off with all the case and jewellery in the house. (a) discovered (b) squandered (c) run away with (d) transferred SSC CPO-SI – 11/12/2019 (Shift-II) Idoms & Phrases

Ans : (c) For the above idiom/phrase "Made off with" option (c) ‘run away with’ gives an appropriate explanation. Other options have different meaning. 374. Select the most appropriate meaning of the given idiom. Make a beeline for (a) Go for freebies (b) Go straight to (c) Stand in a queue (d) Make a line for honey SSC CHSL (Tier-I) 20/10/2020 (Shift-I) SSC CPO-SI – 11/12/2019 (Shift-I) Ans : (b) For the above idiom/phrase "Make a beeline for" option (b) ‘go straight to’ gives an appropriate explanation. Other options have different meaning. 375. Select the most appropriate meaning of the underlined idiom in the sentence. The suspect made a clean breast of it during the enquiry . (a) blamed his companion (b) asked for a fresh shirt (c) confessed his crime (d) took off his shirt SSC CPO-SI – 12/12/2019 (Shift-II) Ans : (c) For the above idiom/phrase "Make a clean breast" option (c) ‘Confess his crime’ gives an appropriate explanation. Other options have different meaning. 376. Select the most appropriate idiom to fill in the blank. The festival expenses are bound to –––– in my pocket. (a) be upfront (b) be out of the blue (c) make a dent (d) give a clean chit SSC CPO-SI – 12/12/2019 (Shift-II) Ans : (c) From these idiom/phrase "Make a dent" is used which means ‘to reduce the amount of something’ it gives an appropriate meaning for the sentence. 377. Select the most appropriate meaning of the given idiom Make no headway (a) check if the head can pass through (b) unable to progress ahead (c) unable to overtake anyone (d) forced to fix a new goal SSC CHSL (Tier-I) –11/07/2019 (Shift-I) Ans : (b) For the above idiom/phrase "Make no head way" option (b) ‘unable to progress ahead’ gives an appropriate explanation. Other options have different meaning. 378. Select the most appropriate meaning of the given idiom Make short work of something (a) Reduce the size (b) Dispose of quickly (c) Edit carefully (d) Shorten some dress SSC CHSL (Tier-I) –11/07/2019 (Shift-I) Ans : (b) For the above idiom/phrase "Make short work of something" option (b) ‘dispose of quickly’ gives an appropriate explanation. Other options have different meaning.

688

YCT

379. Select the most appropriate meaning of the 384. Select the most appropriate meaning of the given idiom. given idiom. Mad as a hatter Not mince (your) words (a) superstitious (b) very upset (a) State your opinion clearly (c) eccentric (d) old fashioned (b) No to have clarity SSC CHSL (Tier-I) –03/07/2019 (Shift-III) (c) Eat a lot clumsily Ans. (c) : For the above idiom/phrase "Mad as a (d) Say words that are confusing hatter" option (c) ‘eccentric’ gives an appropriate SSC CHSL (Tier-I) 15/10/2020 (Shift-II) meaning. Other options have different meaning. 380. Select the most appropriate meaning of the Ans. (a) : For the above idiom/phrase "Not mince given idiom. (your) words" option (a) ‘state your opinion clearly’ Made out of whole cloth gives an appropriate explanation. Other options have (a) Entirely false and fabricated different meaning. (b) Entirely fabricated though believable 385. In the following question, out of the four given (c) Entirely true and convincing alternatives, select the alternatives which best (d) Entirely false though convincing SSC CHSL (Tier-I) 19/03/2020 (Shift-I) express the meaning of the Idiom/Phrase. Ans. (a) : For the above idiom/phrase "Made out of No ifs and buts whole cloth" option (a) ‘Entirely false and fabricated’ (a) Jubilant (b) Low spirited gives an appropriate explanation. Other options have (c) No doubts (d) Screaming different meaning. SSC MTS 9-10-2017 (Shift-II) 381. Select the most appropriate meaning of the Ans. (c) : For the above idiom/phrase "No ifs and given idiom. buts" option (c) ‘No doubts’ gives an appropriate Miss the boat explanation. Other options have different meaning. (a) To miss someone you love (b) To lose something important 386. In the following question, out of the given four (c) To forget something at home alternatives, select the alternative which best (d) To miss an opportunity expresses the meaning of the Idiom/Phrase. SSC GD – 18/02/2019 (Shift III) Nail-biting SSC MTS – 21/08/2019 (Shift-III) (a) Suspenseful (b) Plentiful SSC CHSL 31/05/2022 (Shift-III) (c) Peaceful (d) Joyful Ans. (d) : For the above idiom/phrase "Miss the boat" SSC MTS – 02/08/2019 (Shift-III) option (d) ‘to miss an opportunity’ gives an appropriate explanation. Other options have different Ans. (a) : For the above idiom/phrase "Nail biting:" meaning. option (a) ‘suspenseful’ gives an appropriate explanation. Other options have different meaning. 387. Select the most appropriate meaning of the given idiom. 382. Select the most appropriate meaning of No strings attached following idiom. Nip in the bud (a) Without conditions to restrictions (a) To cut the buds (b) To destroy late (b) Forced to do something (c) Not to destroy (d) To destroy early (c) Involving a choice to be made SSC CGL (Tier-I) 13/04/2022 Shift-III (d) Lacking clarity Ans. (d) : For the above idiom 'Nip in the bud', option SSC MTS – 19/08/2019 (Shift-I) (d) 'to destroy early' gives suitable meaning. Other Ans. (a) : For the above idiom/phrase "No strings options are not correct. attached" option (a) ‘without conditions to restriction’ 383. Select the most appropriate meaning of the gives an appropriate explanation. Other options have given idiom. different meaning. Neither fish nor fowl (a) Something which can neither be purchased 388. Select the most appropriate meaning of the nor be grown underlined idiom in the given sentence. (b) Something which can neither be supplied The idea of making a children's park has been easily nor be made available readily nipped in the bud by the local council. (c) Something which can neither be supplied (a) encouraged strongly easily nor be accepted befittingly (b) included somewhere else (d) Something which can neither be eaten nor be (c) changed completely preserved (d) dropped at an early stage SSC CHSL (Tier-I) 19/03/2020 (Shift-III) SSC CGL (Tier-I) – 13/06/2019 (Shift-I) Ans. (c) : For the above idiom/phrase "Neither fish Ans : (d) For the above idiom/phrase "Nipped in the nor fowl" option (c) ‘something which can neither be categorized easily nor be accepted befittingly’ gives an bud" option (d) ‘dropped at an early stage’ gives an appropriate explanation. Other options have different appropriate explanation. other options have different meaning. meaning.

N

Idoms & Phrases

689

YCT

Ans. (c) : For the above Idiom, "Over and above" option (c) "In addition to" gives an appropriate 389. In the following question, out of the given four explanation. Other options gave different meaning. alternatives, select the alternative which best 394. Select the most appropriate meaning of the expresses the meaning of the Idiom/Phrase. On given idiom. account of Out of the blue (a) To mark one’s territory (b) Because of (a) Completely unexpectedly (c) To make excuses (d) To settle dues (b) Always dressing up in blue clothes SSC MTS 16/05/2023 (Shift-II) (c) Coming out of a blue box Ans. (b) : The meaning of the above given idiom/ phrase- 'On account of' is option (b) 'Because of'. other (d) Caught in a fight options give different meanings. SSC Constable GD-08/12/2021 Shift-I 390. Select the most appropriate meaning of the Ans. (a) : For the above idiom 'Out of the blue', option given idiom. (a) completely unexpectedly gives suitable meaning. On the fence Other options are not correct. (a) Under all conditions

O

(b) Undaunted by anything (c) Halting between two options (d) On very intimate terms SSC GD 13/02/2023 (Shift-I) Ans. (c) : 'Halting between two options' is the most appropriate meaning of the given idiom– 'On the fence'. The other idioms do not give the suitable meaning. Hence option (c) is correct answer. 391. Select the most appropriate meaning of the given idiom. Out of the blue (a) Unconcerned (b) Undoubtedly (c) Unexpectedly (d) Unbelievably SSC CGL (Tier-I) 25/07/2023 (Shift-IV) Ans. (c) : The most appropriate meaning of the above given idiom 'out of the blue' is 'Unexpectedly' Meaning of other optionsUnconcerned - Indifferent, Undoubtedly - Doubles, Unbelievably - Implausibly, Hence, the correct option is (c). 392. Select the most appropriate meaning of the given idiom. On the ball (a) Playing with a ball (b) Exercising using a ball (c) Doing a job in a slow and incorrect manner (d) Doing a job in a quick and competent manner SSC CGL (Tier-I) 20/07/2023 (Shift-I) Ans. (d) : The most appropriate meaning of the given idiom ‘On the ball’ is ‘Doing a Job in a quick and competent manner.’ Other options are incorrect. Thus, the correct answer is option (d). 393. In the following question, out of the given four alternatives, select the alternative which best expresses the meaning of the idiom/phrase. Over and above (a) Above all (b) To sum up the conversation (c) In addition to (d) Highest rank SSC CHSL (Tier-I) 10/03/2023 (Shift-III) Idoms & Phrases

395. Select the most appropriate meaning of the given idiom. Open-ended (a) Ending is twisted (b) Ending is happy (c) Having no planned ending (d) Never ending SSC Constable GD-03/12/2021 Shift-II Ans. (c) : For the above idiom 'Open ended', option (c) 'Having no planned ending' gives suitable meaning. Other option are not correct. 396. Select the most appropriate meaning of the given idiom. Out of question (a) Wishful (b) Doubtful (c) Difficult (d) Impossible SSC Constable GD-13/12/2021 Shift-III Ans. (d) : For the above idiom 'Out of question', option (d) 'impossible' gives suitable meaning. Other options are not correct. 397. Select the most appropriate meaning of the given idiom. On cloud nine (a) very happy (b) very angry (c) very far (d) very worried SSC Constable GD-02/12/2021 Shift-I Ans. (a) : For the above idiom 'On cloud nine', option (a) 'very happy' gives suitable meaning. Other options are not correct. 398. Select the most appropriate meaning of the given idiom. One's cup of tea (a) One's perception about someone (b) One's general principle (c) One's hard-earned money (d) One's chosen or preferred thing SSC MTS-05/07/2022 Shift-I Ans. (d) : For the above idiom 'One's cup of tea', option (d) 'one's chosen or preferred thing' gives suitable meaning other options are not correct.

690

YCT

399. Select the most appropriate meaning of the given idiom. Out for the count (a) Very expensive (b) Prepared for battle (c) In a deep, insensible sleep (d) Ready for scrutiny SSC MTS-06/07/2022 Shift-I Ans. (c) : For the above idiom 'Out for the count', option (c) 'In a deep, insensible sleep' gives suitable meaning. Other options are not correct. 400. Select the most appropriate meaning of the given idiom. On the wane (a) On the rise (b) On the way (c) On the top (d) On the decline SSC CGL (Tier-I) 12/04/2022 Shift-II Ans. (d) : For the above idiom 'On the wane', option (d) 'On the decline' gives suitable meaning. Other options are not correct. 401. Select the most appropriate meaning of the given idiom. On cloud nine (a) Extremely happy (b) Good number (c) Travel by airplane (d) Heavy rain SSC CHSL 31.05.2022 Shift-III Ans. (a) : For the above idiom 'on cloud nine', option (a) Extremely happy gives suitable meaning. Other options are not correct. 402. Select the most appropriate meaning of the given idiom. Once and for all (a) Happily (b) Effectively (c) Out of control (d) Finally SSC Stenographer (Grade C & D) 11.11.2021 Shift-II Ans. (d) : For the above idiom 'Once and for all', option (d) 'finally' gives suitable meaning. Other options are not correct. 403. Select the most appropriate meaning of the given idiom. On tenterhooks (a) Very sad (b) Very tense (c) Very happy (d) Very angry SSC CHSL (Tier-I) –04/08/2021 (Shift-I) Ans. (b) : For the above idiom/phrase "On tenterhooks" option (b) ‘very tense’ gives an appropriate explanation. other options have different meaning. 404. Select the most appropriate meaning of the given idiom. On cloud nine (a) Extremely happy (b) Frequently flying by air (c) Flying with a parachute (d) Extremely sad SSC CGL (Tier-I) –23/08/2021 (Shift-I) Ans. (a) : For the above idiom/phrase "on cloud nine" option (a ‘Extremely happy’ gives an appropriate explanation. Other options have different meaning. Idoms & Phrases

405. Select the most appropriate meaning of the given idiom. Once in a blue moon (a) Always (b) Rarely and infrequently (c) At regular intervals (d) A natural occurrence SSC CHSL 16/10/2020 (Shift-II) SSC Stenographer Grade C&D –05/02/2019 (3:5pm)

Ans. (b) : For the above idiom/phrase "once in a blue moon" option (b) ‘Rarely and infrequently’ gives an appropriate explanation. Other options have different meaning. 406. In the following question, out of the given four alternatives, select the alternative which best expresses the meaning of the Idiom/Phrase. Of the first water (a) Leading a comfortable life. (b) Acting contrary to the one's interests (c) Critically important (d) Of the best quality SSC CGL (Phase-II) 17/02/2018 Ans. (d) For the above idiom/phrase 'of the first water" option (d) ‘of the best quality’ gives an appropriate explanation. Other options have different meaning. 407. Select the most appropriate meaning of the underlined idiom in the given sentence. On the face of it, nothing seems to be fishy about the matter. (a) On deeper scrutiny (b) From facial expressions (c) What appears on the surface (d) In comparison SSC Sel. Post Phase-VIII (G.L.) 09.11.2020 (Shift-II)

Ans. (c) : For the above idiom/phrase "On the face of" option (c) ‘what appears on the surface’ gives an appropriate meaning. Other options have different meaning. 408. Select the most appropriate meaning of the underlined idiom in the given sentence. The man heaved a sigh of relief when he was sure he was out of the woods. (a) discharged from hospital (b) released from prison (c) out of danger (d) out of the forest SSC CGL (Tier-I) – 12/06/2019 (Shift-I) Ans : (c) For the above idiom/phrase 'out of the woods' option (c) ‘out of danger’ gives an appropriate explanation. Other options have different meaning. 409. Select the most appropriate meaning of the given below. Out of bounds (a) Forbidden (b) Within limits (c) Tied up (d) Limitless SSC CHSL (Tier-I) 26/10/2020 (Shift-II) Ans. (a) : For the above idiom/phrase "out of bounds" option (a) 'forbidden' gives an appropriate explanation. Other options have different meaning.

691

YCT

410. Select the most appropriate meaning of the given idiom. Once in a blue moon (a) An incident that is scary (b) Happening only once in a lifetime (c) Happening only rarely (d) An unforgettable experience SSC CHSL (Tier-I) 16/10/2020 (Shift-II) Ans. (c) : For the above idiom/phrase "once in a blue moon" option (c) 'happening only rarely' gives an appropriate explanation. Other options have different meaning. 411. Select the most appropriate meaning of the underlined idiom in the given sentence. If someone is an old hand, then he/she is: (a) experienced (b) weak (c) aged (d) forgetful SSC CHSL (Tier-I) 14/10/2020 (Shift-III) Ans. (a) : For the above idiom/phrase "old hand" option (a) 'experienced' gives an appropriate explanation. Other options have different meaning. 412. In the following question out of the four given alternatives select the alternative which best expresses the meaning of the Idiom/phrase. On the brink of disaster (a) Something still in hand (b) Natural destruction (c) To die (d) Very close to failure SSC MTS 10-10-2017 (Shift-I) Ans : (d) For the above idiom/phrase "On the brink of disaster" option (d) very close to failure gives an appropriate explanation. Other options have different meaning. 413. In the following question, out of the four given alternatives, select the alternative which best expresses the meaning of the Idiom/Phrase. On account of (a) Instead of (b) Despite (c) Because of (d) Rejecting something SSC MTS 11-10-2017 (Shift-III) Ans. (c) : For the above idiom/phrase "on account of" option (c) 'because of' gives an appropriate explanation. Other options have different meaning. 414. In the following question, out of the four given alternatives which best expresses the meaning of the Idiom/Phrase. On the wrong side of sixty (a) To be inexperienced (b) To be older than sixty (c) To be able to convince (d) To be unsuitable SSC MTS 11-10-2017 (Shift-II) Ans. (b) : For the above idiom/phrase "on the wrong side of sixty" option (b) 'to be older than sixty' gives an appropriate explanation. Other options have different meaning. 415. Select the most appropriate meaning of the given idiom. On the tip of my tongue (a) Feeling extremely joyful (b) Readily available in my memory (c) Excessively costly (d) At full speed SSC MTS – 08/08/2019 (Shift-I) Idoms & Phrases

Ans. (b) : For the above idiom "on the tip of my tongue" option (b) Readily available in my memory gives an appropriate explanation. Other options have different meaning. 416. In the following question, out of the given four alternatives, select the alternatives which best expresses the meaning of the Idiom/Phrase. Over my dead body (a) Opposing something fiercely (b) Inheriting of fortune (c) Erecting a grave stone (d) Threatening to kill oneself SSC MTS – 02/08/2019 (Shift-II) Ans. (a) : for the above idiom/phrase "over my dead body" option (a) opposing something fiercely gives an appropriate explanation. Other options have different meaning. 417. Select the most appropriate meaning of the given idiom. Off the cuff (a) To be in a hurry (b) To be lazy (c) Without preparation (d) Without any loss SSC MTS – 21/08/2019 (Shift-I) Ans. : (c) For the above idiom/phrase "of the cuff" option (c) 'without preparation' gives an appropriate explanation. Other options have different meaning. 418. Select the most appropriate meaning of the given idiom. Out of date (a) Being careless (b) Something old fashioned (c) A worthless fellow (d) A date seed SSC MTS – 06/08/2019 (Shift-I) Ans. (b) : For the above idiom/phrase "out of date" option (b) 'something old fashioned' gives an appropriate explanation. Other options have different meaning. 419. Select the most appropriate meaning of the given idiom. Once bitten, twice shy (a) Someone who doesn't learn from his previous experiences (b) An unpleasant experience induces caution (c) Someone who is afraid of taking challenges (d) To keep something precious safely SSC MTS – 22/08/2019 (Shift-III) Ans. (b) : For the above idiom/phrase "once bitten, twice shy" option (b) 'An unpleasant experience induces caution' gives an appropriate explanation. Other options have different meaning.

P 420. Select the most appropriate meaning of the bracketed idiom in the given sentence. Later, Brooks' brother alleged that racism helped put the (pedal to the metal). (a) Very excessive (b) To go full speed, especially hale driving a vehicle (c) Avoid responsibility by giving it to someone else (d) To enjoy yourself by going out SSC MTS 10/05/2023 (Shift-I)

692

YCT

Ans. (b) : The most appropriate meaning of the bracketed idiom 'pedal to the Metal' is 'To go full speed, especially while driving a vehicle'. Thus, the correct answer is option (b). 421. Select the most appropriate meaning of the bracketed idiom in the given sentence. Sure, fixing a flubbed decision isn't always a (piece of cake), but it's rarely impossible. (a) A task that can be accomplished very easily (b) Having no stand, shows favour at one time and unfavour at another (c) A rare event or occurrence (d) The family bond is closer than anything else SSC MTS 12/05/2023 (Shift-III) Ans. (a ) : The most appropriate meaning of the above given idiom – 'piece of cake' is a task that can be accomplished very easily.' Thus, the correct answer is option (a). 422. Select the most appropriate option that can substitute the underlined segment in the given sentence. My mother asked my father to increase her personal use allowance. (a) rank and file (b) pell mell (c) pin money (d) pins and needles SSC CHSL (Tier-I) 17/08/2023 (Shift-II) Ans. (c) : The underlined segment of the sentence 'personal use allowance' can be substituted by 'idiom– 'pin money' given in option (c). Other idioms given as options have the following meaning – Rank and file – The ordinary members/ workers in an organization as apposed to its leaders or managers. Pell mell – In a recklessly hurried manner. Pins and needles – In an agitated state of suspense. 423. Select the most appropriate idiom for the underlined words in the given sentence. The childhood days are peaceful days of our life. (a) hard and fast days (b) halcyon days (c) live wire days (d) all years SSC CHSL (Tier-I) 14/08/2023 (Shift-IV) Ans. (b) : The meaning of underlined words 'peaceful days is being well expressed by idiom 'halcyon days' (A very happy or successful period in the past ) given in option (b). New Sentence The childhood days are halcyon days of our life. 424. Select the most appropriate meaning of the given idiom 'Pedal to the metal' (a) To enjoy yourself by going out. (b) To go full speed, especially while driving a vehicle (c) Avoid responsibility by giving it to someone else (d) Very excessive SSC CHSL (Tier-I) 15/03/2023 (Shift-III) Idoms & Phrases

Ans. (b) : For the above Idiom, 'Pedal to the metal' is option (b) 'To go full speed, especially while driving a vehicle' gives an appropriate explanation. Other options gives different meaning. 425. Select the most appropriate meaning of the given idiom. Play with fire (a) To play a game honestly (b) To do something risky (c) To please someone (d) To waste time on worthless things SSC MTS-06/07/2022 Shift-III Ans. (b) : For the above idiom' Play with fire', option (b) 'To do something risky' gives suitable meaning. Other option are not correct. 426. Select the most appropriate meaning of the given idiom. Play with fire (a) To be courageous (b) To fight an opponent (c) To do something dangerous (d) To spread knowledge SSC CGL (Tier-I) 19/04/2022 Shift-II Ans. (c) : For the above idiom 'Play with fire', (c) to do something dangerous gives suitable meaning. Other options are not correct. 427. Select the most appropriate meaning of the given idiom. Pull yourself together (a) gather people together for support (b) control emotions and behave calmly (c) ensure a large number of people join you (d) decide to fight for a cause alone SSC Stenographer (Grade C & D) 12.11.2021 Shift-II Ans. (b) : For the above idiom 'Pull yourself together' option (b) control emotions and behave calmly gives suitable meaning. Other options are not correct. 428. Select the most appropriate meaning of the given idiom. Pipe dream (a) A dream or idea that is loudly proclaimed (b) A dream or idea that is nearly accomplished (c) A dream or idea that is unlikely to happen (d) A dream or idea that keeps getting repeated SSC CGL (Tier-I) –17/08/2021 (Shift-I) Ans. (c) : For the above idiom/phrase "pipe dream option (c) 'A dream or idea that is unlikely to happen' gives an appropriate explanation. Other options have different meaning. 429. Select the most appropriate meaning of the underlined idiom in the given sentence. The sad part of the agitation is that most of the leaders are only paying lip-service to the issue. (a) saying they agree although they do not support it (b) partially impressing the crowds (c) making long speeches for the cause (d) growing angry and restless with the organisers SSC CPO-SI (Tier-II) 27/09/2019 (3 pm - 5 pm)

693

YCT

Ans. (a) : For the above idiom/phrase "Paying lipservice" option (a) 'saying they agree although they do not support it' gives an appropriate explanation. Other options have different meaning. 430. Select the most appropriate meaning of the given idiom. Pocket an insult (a) bear an insult quietly (b) plan a revenge (c) express true feelings (d) resent an affront SSC CPO-SI – 24/11/2020 (Shift-I) Ans. (a) : For the above idiom/phrase "Pocket an insult" option (a) bear an insult quietly gives an appropriate explanation other options have different meaning. 431. Select the most appropriate meaning of the underlined idiom in the given sentence. Raju was playing with fire when he made speeches against the management. (a) taking a grave risk (b) smoking a cigarette (c) provoking the crowd (d) lighting candles SSC CGL (Tier-I) – 12/06/2019 (Shift-III) Ans. (a) : For the above idiom/phrase "playing with fire" option (a) 'taking a grave risk' gives an appropriate explanation. Other options have different meaning. 432. Select the most appropriate meaning of the underlined idiom in the given sentence. One must learn to prioritize in life. It never pays to put the cart before the horse. (a) postpone till the last moment (b) perform many tasks simultaneously (c) do things spontaneously (d) do last things first SSC CGL (Tier-I) – 11/06/2019 (Shift-III) Ans. (d) : For the above idiom/phrase "put the cart before the horse" option (d) 'do last thing first' gives an appropriate explanation. Other options have different meaning. 433. Select the most appropriate meaning of the given idiom. pull yourself together (a) go to sleep (b) try to understand (c) calm down (d) do a good job SSC CGL (Tier-I) – 06/06/2019 (Shift-I) Ans. (c) : For the above idiom/phrase "pull yourself together" option (c) 'calm down' gives an appropriate explanation. Other options have different meaning. 434. Select the most appropriate meaning of the given idiom. pull someone's leg (a) joke with someone (b) get upset with someone (c) tell someone a secret (d) trust someone SSC CGL (Tier-I) – 04/06/2019 (Shift-III) Ans. (a) : For the above idiom/phrase "Pull someone's leg" option (a) 'joke with someone' gives an appropriate explanation. Other options have different meaning. Idoms & Phrases

435. Select the most appropriate meaning of the given idiom Put your best foot forward (a) take care to dress well (b) walk very cautiously (c) try as hard as one can (d) check every step SSC CHSL (Tier-I) –10/07/2019 (Shift-II) Ans : (c) For the above idiom/phrase "put your best foot forward" option (c) 'try as hard as one can' gives an appropriate explanation other options have different meaning. 436. Select the most appropriate meaning of the underlined idiom in the given sentence. My aunt who orders every one around is a pain in the neck. (a) irritating (b) short tempered (c) curious (d) argumentative SSC CHSL (Tier-I) –10/07/2019 (Shift-III) Ans : (a) For the above idiom/phrase "in the neck" option (a) 'irritating' gives an appropriate explanation. Other options have different meaning. 437. Select the most appropriate meaning of the given idiom. Pass the hat (a) to sell something (b) to collect money (c) to play a game (d) to avoid work SSC CHSL (Tier-I) –08/07/2019 (Shift-I) Ans : (b) For the above idiom/phrase "pass the hat" option (b) 'to collect money' gives an appropriate explanation. Other options have different meaning. 438. Select the most appropriate meaning of the given idiom. Plain as day (a) Very simple and easy to do (b) Visible only in broad daylight (c) Very obvious and easy to understand (d) Very interesting to know SSC CHSL (Tier-I) 16/10/2020 (Shift-III) Ans. (c) : For the above idiom/phrase "Plain as day" option (c) 'very obvious and easy to understand' gives an appropriate explanation. Other options have different meaning. 439. Select the most appropriate meaning of the given idiom. Parthian shot (a) Parting hit (b) Victory (c) Praise (d) Say goodbye SSC CHSL (Tier-I) 16/10/2020 (Shift-I) Ans. (a) : For the above idiom/phrase "Parthian shot" option (a) 'Parting hit' gives an appropriate explanation. Other options have different meaning. 440. Select the most appropriate meaning of the underlined idiom in the given sentence. His guide asked him to pull up his socks if he wanted a good rank. (a) dress more carefully and neatly (b) make a greater effort to improve (c) take care to be more formal (d) give attention to his appearance also SSC CHSL (Tier-I) 17/03/2020 (Shift-II)

694

YCT

Ans. (b) : For the above idiom/phrase "wanted a good rank" option (b) 'make a greater effort to improve' gives an appropriate explanation. Other options have different meaning. 441. Select the most appropriate meaning of the idiom. Pull yourself together (a) Talk about fitness (b) Be friendly with everyone (c) Use a rope ladder (d) Regain composure SSC CHSL (Tier-I) 14/10/2020 (Shift-I) Ans. (d) : For the above idiom/phrase "pull yourself together" option (d) 'regain composure' gives an appropriate explanation. Other options have different meaning. 442. In the following question, out of the four given alternatives, select the alternative which best expresses the meaning of the idiom/Phrase. Pull strings (a) Fixed the issue (b) In a dilemma (c) To use influence (d) Losing hope SSC MTS 7-10-2017 (Shift-I) Ans : (c) For the above idiom/phrase "pull strings" option (c) 'to use influence' gives an appropriate explanation. Other options have different meaning. 443. Select the most appropriate meaning of the given idiom. To pick up the threads (a) To examine as issue seriously (b) Being received very warmly (c) To restart from the previous closing point (d) At a short distance SSC MTS – 06/08/2019 (Shift-III) Ans. (c) : For the above idiom/phrase "To pick up the threads" option (c) 'to restart from the previous closing point' gives an appropriate explanation. Other options have different meaning. 444. Select the most appropriate meaning of the given idiom. Put the cart before the horse (a) To do last things first (b) To lie about a situation (c) To follow the logical order (d) To do impractical things SSC MTS – 16/08/2019 (Shift-I) Ans. (a) : For the above idiom/phrase "put the cart before the horse" option (a) to do last things first gives an appropriate explanation other options have different meaning. 445. Select the most appropriate meaning of the given idiom. To pull someone's legs (a) of a kind and sympathetic nature (b) To use all possible methods (c) To play a joke on someone (d) Not a serious information SSC MTS – 07/08/2019 (Shift-III) Ans. (c) : For the above idiom/phrase "To pull someone's leg "option (c) 'to play a joke on someone' gives an appropriate explanation. Other options have different meaning. Idoms & Phrases

446. Select the most appropriate meaning of the given idiom. Pull up your socks (a) To lose one's temper (b) To put in extra effort (c) To play a joke on someone (d) To suspect some foul play SSC MTS – 06/08/2019 (Shift-II) Ans. (b) : For the above idiom/phrase "pull up your sock's option (b) to put in extra effort gives an appropriate explanation. Other options have different meaning. 447. Select the meaning of the given idiom. To pour oil in troubled water (a) To calm a dispute (b) To create tension (c) To light a fire (d) To raise a controversy SSC MTS – 09/08/2019 (Shift-III) Ans. (a) : For the above idiom/phrase "To pour oil in troubled water" option (a) 'to calm a dispute' gives an appropriate explanation. Other options have different meaning. 448. Select the meaning of the given idiom in the sentence. Harassed by repeated acts of injustice, he decided to put his foot down. (a) Resign (b) Not to yield (c) Withdraw (d) Change his mind SSC GD – 08/03/2019 (Shift-III) Ans. (b) : For the above idiom/phrase "To put his foot down" option (b) 'not to yield' gives an appropriate explanation. Other options have different meaning.

Q 449. Select the most appropriate meaning of given idiom. Queer fish (a) caused fear (b) wasting a vast treasure (c) an activity in motion (d) a person who seems strange or unusual SSC CHSL (Tier-I) –10/07/2019 (Shift-III) Ans. (d) : For the above idiom/phrase 'Queer fish' option (d) 'a person who seems strange or unusual' gives an appropriate explanation. Other options have different meaning. 450. Select the most appropriate meaning of given idiom. A question of time (a) Something that is sure to happen (b) Caused damage (c) Spread rapidly (d) Spread slowly SSC CGL (Tier-I) – 11/06/2019 (Shift-III) Ans. (a) : For the above idiom/phrase "A question of time" option (a) 'something that is sure to happen' gives an appropriate explanation. Other options have different meaning.

695

YCT

R 451. In the following question, out of the given four alternatives, select the alternative which best expresses the meaning of the Idiom/Phrase. Rolling in cash (a) Have a large quantity of money (b) A business magnate (c) To borrow a costly attire (d) Buying things on interest SSC MTS 18/05/2023 (Shift-I) Ans. (a) : The meaning of the above given idiom/phrase- ‘Rolling in cash’ is ‘Have a large quantity of money’ Other options give different meanings. 452. Select the most appropriate meaning of the given idiom. Royal road (a) Achieving something by birth (b) Achieving something by truth (c) Achieving something by false means (d) Achieving something trouble free SSC GD 01/02/2023 (Shift-II) Ans. (d) : 'Achieving something trouble free' is the most appropriate meaning of the given word– 'Royal road'. Hence option (d) is correct answer. 453. Select the most appropriate meaning of the underlined phrase. Though very rich, Alexander always saves for a rainy day. (a) Lives like a miser (b) Spends too much (c) Saves money for later (d) Accumulates less wealth SSC CGL (Tier-I) 26/07/2023 (Shift-II) Ans. (c) : The most appropriate meaning of the underlined Phrase 'saver for a rainy day' is 'saves money for later'. which means set aside money or resources for future unexpected needs of emergencies. Thus, the correct answer is option (c). 454. Select the most appropriate option that can substitute the underlined segment in the given sentence. Disability was viewed as God's retribution for wickedness in the past. (a) payback for doing help (b) punishment for wrongdoing (c) revenge for legal works (d) fine for misbehaviour SSC CGL (Tier-I) 20/07/2023 (Shift-I) Ans. (b) : The most appropriate option for the underlined Segment in the given sentence ‘retribution for wickedness’ is replaced by ‘Punishment for wrong doing’. Thus, the correct answer is option (b). Idoms & Phrases

455. Select the most appropriate meaning of the given idiom. Raining cats and dogs (a) Cat and dog fight (b) Too crowded (c) Raining heavily (d) Raining lightly SSC Constable GD-07/12/2021 Shift-II Ans. (c) : For the above idiom 'Raining cats and dogs', option (c) 'Raining heavily' gives suitable meaning. Other options are not correct. 456. Select the option that gives the most appropriate meaning of the underlined idiom. The Chairman of our company takes care of the rank and file in the company. (a) Ordinary people (b) Documents and files (c) Only the top rank people (d) Officers SSC CGL (Tier-I) 11/04/2022 Shift-II Ans. (a) : For the above idiom 'Rank and file' option (a) ordinary people' gives suitable meaning. Other options are not correct. 457. Select the most appropriate meaning of the given idiom. Red letter day (a) Memorable day (b) Holiday (c) Birthday (d) Rose day SSC CHSL 27.05.2022 Shift-III Ans. (a) : For the above idiom, 'Red letter day', option (a) 'memorable day' gives suitable meaning. Other options are not correct. 458. Select the most appropriate meaning of the given idiom. Read between the lines (a) Be afraid of doing something again (b) Speak the truth even if it is unpleasant (c) Understand something that is not said outright (d) Harm someone who trusts you SSC CHSL 27.05.2022 Shift-III Ans. (c) : For the above idiom' Read between the lines', options (c) understand something that is not said outright' gives suitable meaning. Other options are not correct. 459. Select the most appropriate meaning of the given idiom. Raise an alarm (a) create an alarming situation (b) be punctual and disciplined (c) set the alarm clock to wake one up (d) warn of a dangerous situation SSC Stenographer (Grade C & D) 15.11.2021 Shift-II Ans. (d) : For the above idiom 'Raise an alarm' option (d) warn of a dangerous situation gives suitable meaning. Other options are not correct. 460. Select the most appropriate meaning of the given idiom. Read between the lines (a) understand what is implied but not stated directly (b) read very slowly enjoying each line

696

YCT

(c) read loudly each and every word given in the sentence (d) read carefully each and every word written SSC CHSL (Tier-I) –15/04/2021 (Shift-I) Ans. (a) : For the above idiom/phrase "Read between the line" options (a) 'understand what is implied but not stated directly' gives an appropriate explanation. Other options have different meaning. 461. Select the most appropriate meaning of the given idiom. Raise the bar (a) To raise the price (b) To win a competition (c) To grow taller (d) To set higher goals SSC CGL (Tier-I) –13/08/2021 (Shift-I) Ans. (d) : For the above idiom/phrase "Raise the bar" option (d) 'to set higher goal' gives an appropriate explanation. Other options have different meaning. 462. Select the most appropriate meaning of the underlined idiom in the given sentence. Their relationship has run into rough weather ever since they fell in love with the same girl. (a) experienced difficulties (b) brought cool breeze and rains (c) become stronger and firmer (d) become pleasant and cordial SSC CPO-SI (Tier-II) 27/09/2019 (3 pm - 5 pm) Ans. (a) : For the above underlined idiom "run into rough weather" option (a) 'experienced difficulties' gives an appropriate explanation. Other options have different meaning. 463. In the following question, out of the given four alternatives, select the alternative which best expresses the meaning of the Idiom/Phrase. Reap the whirlwind (a) Providing that other factors or circumstance remain the same (b) Suffer negative consequences as a result of one's actions (c) An indication that something is accepted or regarded favorably (d) Believing that moderation is more satisfying than excess SSC CGL (Phase-II) 17/02/2018 Ans. (b) For the above idiom "Reap the whirlwind option (b) 'suffer negative consequences as a result of one's action' gives an appropriate explanation. Other options have different meaning. 464. Select the most appropriate meaning of the given idiom. Rise to the occasion (a) show you can deal with a situation successfully (b) assess a critical situation carefully (c) challenge an opponent who is strong (d) take a big risk in order to succeed SSC CPO-SI – 24/11/2020 (Shift-I) Ans. (a) : For the above idiom/phrase "Rise to the occasion" option (a) 'show you can deal with situation successfully' gives an appropriate explanation. Other options have different meaning. Idoms & Phrases

465. Select the most appropriate meaning of the given idiom. Run around in circles (a) to work excessively hard by keeping two jobs (b) to remain busy with your work (c) to keep running to set a record (d) to keep doing something without achieving much SSC CPO-SI – 09/12/2019 (Shift-II) Ans : (d) For the above idiom/phrase "Run around in circles" options (d) 'to keep doing something without achieving much' gives an appropriate explanation. Other options have different meaning. 466. Select the most appropriate meaning of the given idiom. To read between the lines (a) to understand the implied meaning (b) to completely misunderstand the meaning (c) to critically analysis a piece of writing (d) to read with great difficulty SSC CPO-SI – 13/12/2019 (Shift-I) Ans. (a) : For the above idiom/phrase "Read between the line" options (a) 'to understand the implied meaning' gives an appropriate explanation. Other options have different meaning. 467. Select the most appropriate meaning of the given idiom. Red letter day (a) a very hot day (b) a frightful day (c) a very special day (d) a very cold day SSC CHSL (Tier-I) –08/07/2019 (Shift-II) Ans : (c) For the above idiom/phrase "Red letter day" option (c) 'a very special day' gives an appropriate explanation. Other options have different meaning. 468. Select the most appropriate meaning of the given idiom. Rule the roost (a) to make all the decisions (b) to be prosperous but generous (c) to frequently pick a quarrel (d) to blindly follow others SSC CHSL (Tier-I) –08/07/2019 (Shift-III) Ans : (a) For the above idiom/phrase "Rule the roost" options (a) 'to make all the decisions' gives an appropriate explanation. Other options have different meaning. 469. Select the most appropriate meaning of the given idiom. Rub the wrong way (a) Rub incorrectly (b) Rub well without oil (c) Annoy someone (d) Please someone SSC CHSL (Tier-I) 15/10/2020 (Shift-II) Ans. (c) : For the above idiom/phrase "Rub the wrong way" option (c) 'Annoy someone' gives an appropriate explanation. Other options have different meaning. 470. Select the most appropriate meaning of the given idiom. Reinventing the wheel (a) Disturbing the system which has already been in use (b) Examining circumstances that have already been understood

697

YCT

(c) Wasting one's time for things that have already been satisfactorily done (d) Analysing various factors of happenings SSC CHSL (Tier-I) 19/03/2020 (Shift-II) Ans. (c) : For the above idiom/phrase "Reinventing the wheel" option (c) 'wasting one's time for thing that have already been satisfactorily done' gives an appropriate explanation. Other options have different meaning. 471. Select the most appropriate meaning of the given idiom. It's raining cats and dogs (a) To rain very heavily (b) To rain continuously (c) To rain mildly (d) To rain at regular intervals SSC MTS – 13/08/2019 (Shift-II) Ans. (a) : For the above idiom/phrase 'raining cats and dogs' option (a) 'To rain very heavily' gives an appropriate explanation. Other options have different meaning. 472. Select the meaning of the given idiom. To run amok (a) To compete in race (b) To participate in a marathon (c) To outrun someone (d) To run about in a frenzy SSC MTS – 08/08/2019 (Shift-II) Ans. (d) : For the above idiom/phrase "To run amok" option (d) 'to run about in a frenzy' gives an appropriate explanation. Other options have different meaning. 473. Select the most appropriate meaning of the given sentence. Sandeep received a new lease of life after moving to Pune. (a) became energetic again (b) started taking healthy food (c) looked for a group of friends (d) started looking for a job SSC CHSL (Tier-I) 14/10/2020 (Shift-III) Ans. (a) : For the above idiom/phrase "received a new lease of life" option (a) 'became energetic again' gives an appropriate explanation. Other options have different meaning.

S 474. Select the most appropriate meaning of the given idiom. A shot in the dark. (a) Shooting in the night (b) Randomly doing something, the outcome of which is unknown (c) A blind man's cane (d) Looking at the stars SSC CHSL (Tier-I) 09/02/2023 (Shift-II) Ans. (b) : For the above Idiom "A shot in the dark" option (b) Randomly doing something the outcome of which is unknown gives an appropriate explanation. Other options give different meaning. Idoms & Phrases

475. In the following question, out of the given four alternatives, select the alternative which best expresses the meaning of the Idiom/Phrase. To scratch one’s head (a) To come up with an idea (b) To think hard in a puzzled way (c) To lose all the wealth (d) To think about good days SSC MTS 15/05/2023 (Shift-I) Ans. (b) : The meaning of the above idiom– 'To scratch one's head' is option (b) 'To think hard in a puzzled way.' Thus, the correct option is (b). 476. Select the most appropriate meaning of the given idiom. Scrape the barrel (a) To be forced to use one's last and weakest resource (b) Trying to find something (c) Using all the ways to achieve the desired result (d) Hiding something SSC CGL (Tier-I) 17/07/2023 (Shift-II) Ans. (a) : For the above idiom "scrape the barrel" option (a) 'To be forced to use one's Last and weakest resource' gives appropriate explanation. other option give different meaning. 477. Select the most appropriate meaning of the given idiom. Spill the beans (a) Open the flow of the stopped water (b) Disclose the secrets accidentally (c) Liberate the controlled emotions (d) Distribute food grains freely SSC CHSL (Tier-I) 08/08/2023 (Shift-II) Ans. (b) : The most appropriate idiom of the above word 'spill the beans' is 'Disclose the secrets accidentally.' Other options give different meanings. 478. Select the most appropriate meaning of the given idiom. Save your breath (a) To make an effeort that will be futile (b) To hold one’s anger (c) To work hard (d) To be resentful SSC CHSL (Tier-I) 20/03/2023 (Shift-I) Ans. (a) : For the above sentence Idiom, "Save your breath", option (a) "To make an effort that will be furtile" gives suitable meaning. Other options are not correct. 479. Select the most appropriate meaning of the given idiom. To show a clean pair of heels (a) To run fast (b) Show clean shoes (c) To clean shoes fast (d) Show clean feet SSC Selection Posts XI-27/06/2023 (Shift-I) Ans. (a) : The most appropriate meaning of the above given idiom is option (a) 'to run fast' Other options are not correct.

698

YCT

480. In the following question, out of the given four alternatives, select the alternative which best expresses the meaning of the Idiom/Phrase. shake in one's shoes (a) To be afraid (b) To leave no remark (c) An unpleasant scenario (d) To fail to understand SSC CHSL (Tier-I) 09/03/2023 (Shift-IV) Ans. (a) : For the above Idiom, "To shake in one's shoes" option (a) "To be afraid" gives an appropriate explanation. Other options give different meaning. 481. Select the most appropriate meaning of the given idiom. Storm in a teacup (a) Making a habit of breaking teacups (b) Thunder and storm during meal time (c) Too little tea poured in a cup (d) Anger or fuss about something trivial SSC Constable GD-06/12/2021 Shift-III Ans. (d) : For the above idiom 'Storm in a teacup', option (d) 'Anger or fuss about something trivial' gives suitable meaning. Other options are not correct. 482. Select the most appropriate meaning of the underlined idiom in the given sentence. My elder brother is so innocent that winning an argument against him is like shooting fish in a barrel. (a) Indulge in an illegal activity (b) Exceptionally easy to do (c) Asking for charity (d) Making someone uneasy SSC MTS-07/07/2022 Shift-I Ans. (b) : For the above idiom, 'shooting fish in a barrel', option (b) 'Exceptionally easy to do gives suitable meaning. Other options are not correct. 483. Select the most appropriate meaning of the given idiom. Stone's throw (a) Hurt slightly (b) Large hurdle (c) Short distance (d) Difficult problem SSC CGL (Tier-I) 13/04/2022 Shift-I Ans. (c) : For the above idiom ;stone's throw', option (c) short distance' gives suitable meaning. Other options are not correct. 484. Select the most appropriate meaning of the given idiom. Safe pair of hands (a) Miss an opportunity (b) Speak rashly without thinking carefully (c) A person who can be trusted to do something efficiently (d) A secret or hidden advantage SSC CHSL 26.05.2022 Shift-III

485. Select the most appropriate meaning of the underlined idiom in the given sentence. Newton's work in the field of physics speaks volumes about his genius. (a) advertises his work (b) claims fame and honour (c) does not gives a detailed report (d) conveys a great deal SSC CHSL (Tier-I) –13/04/2021 (Shift-I) Ans. (d) : For the above idiom 'speaks volumes' option (d) conveys a great deal gives an appropriate explanation. Other options have different meaning. 486. Select the most appropriate meaning of the given idiom. stumbling block (a) An obstacle to progress (b) Removing stones on the way (c) Skipping over a hurdle (d) Putting stones along the way SSC CGL (Tier-I) –18/08/2021 (Shift-I) Ans. (a) : For the above idiom "stumbling block" option (a) 'An obstacle to progress' gives an appropriate explanation. Other options have different meaning. 487. Select the most appropriate meaning of the underlined idiom in the given sentence. The new of a leopard in the city spread like wild fire: (a) caused fear (b) caused damage (c) spread rapidly (d) spread slowly SSC CGL (Tier-II) 16/11/2020 (3 pm - 5 pm) Ans. (c) : For the above idiom, 'spread like wild fire' option (c) 'spread rapidly' gives an appropriate explanation. Other options have different meaning. 488. Select the most appropriate meaning of the given idiom: Smell a rat (a) Have no doubts about something (b) Feel that there is a loss in business (c) Think that there is a dead animal (d) Have a reason to suspect something SSC Stenographer Grade C&D –05/02/2019 (3:5pm)

Ans. (d) : For the above idiom 'smell a rate option (d) 'have a reason to suspect something' gives an appropriate explanation. Other options have different meaning. 489. In the following question, out of the given four alternatives, select the alternative which best expresses the meaning of the Idiom/Phrase. Spin your wheels (a) Set an activity in motion (b) waste your time or efforts (c) A firmly held belief (d) With great affection or enthusiasm SSC CGL (Phase-II) 17/02/2018 Ans. (b) For the above idiom/phrase 'Spin you wheels', option (b) 'waste your time or efforts' gives an appropriate explanation. Other options have different meaning. 490. Select the most appropriate meaning of the given idiom. Ans. (c) : For the above idiom 'Safe pair of hands', Snake in the grass option (c) A person who can be trusted to do something (a) a deceitful person (b) a mean person efficiently gives suitable meaning. Other options are not (c) a slippery person (d) a suspicious person correct. SSC CPO-SI – 25/11/2020 (Shift-I)

Idoms & Phrases

699

YCT

Ans. (a) : For the above idiom/phrase 'snake in the grass' option (a) 'a deceitful person' gives an appropriate explanation. Other options have different meaning. 491. Select the most appropriate meaning of the given idiom. Sitting on a gold mine (a) wasting a vast treasure (b) secretly hunting for something precious (c) to own something potentially valuable (d) not availing of an opportunity SSC CPO-SI – 25/11/2020 (Shift-II) Ans. (c) : For the above idiom/phrase 'Sitting on a gold mine' Option (c) 'to own something potentially valuable' gives an appropriate explanation. Other options have different meaning. 492. Select the most appropriate meaning of the given idiom. Speak one’s mind (a) to put one’s thoughts in writing (b) to speak without thinking (c) to speak in an assembly (d) to voice one’s thoughts plainly SSC CPO-SI – 23/11/2020 (Shift-II) Ans. (d) : For the above idiom/phrase 'Speak one’s mind' option (d) 'to voice one’s thoughts plainly' gives an appropriate explanation. Other options have different meaning. 493. Select the most appropriate meaning of the given idiom. Strike when the iron is hot (a) torture someone with a hot iron (b) wait patiently for the results (c) grab a favourable opportunity promptly (d) sort out differences promptly SSC CPO-SI – 24/11/2020 (Shift-I) Ans. (c) : For the above idiom/phrase "Strike when the iron is hot" option (c) 'grab a favorable opportunity promptly' gives an appropriate explanation. Other options have different meaning. 494. Select the most appropriate meaning of the underlined idiom in the given sentence. "These glasses suit you to a T," said Ria to Vandana. (a) look very good on your face (b) are not suitable for your work (c) make you look weird (d) need to be won with a suit SSC CGL (Tier-I) – 13/06/2019 (Shift-III) Ans : (a) For the above idiom/phrase 'suit you to a T' option (a) 'look very good on your face' gives an appropriate explanation. Other options have different meaning. 495. Select the most appropriate meaning of the underlined idiom in the given sentence. Since he secured the first rank Sudhir has become swollen-headed. (a) famous (b) unwell (c) conceited (d) well-connected SSC CGL (Tier-I) – 12/06/2019 (Shift-II) Idoms & Phrases

Ans : (c) For the above idiom/phrase 'Swollen-headed' option (c) 'conceited' gives an appropriate explanation. Other options have different meaning. 496. Select the most appropriate meaning of the underlined idiom in the given sentence. His success at such a young age speaks volumes for his talent. (a) publically announces (b) boasts a lot (c) gives enough proof (d) showers praise SSC CGL (Tier-I) – 11/06/2019 (Shift-III) Ans. (c) : For the above idiom/phrase 'Speaks Volumes' option (c) 'give enough proof' gives an appropriate explanation. Other options have different meaning. 497. Select the most appropriate meaning of the underlined idiom in the given sentence. When his envious competitor extended a hand of friendship, he smelt a rat. (a) detected something wrong (b) felt very pleased (c) became complaisant (d) behaved arrogantly SSC CGL (Tier-I) – 10/06/2019 (Shift-II) Ans : (a) For the above idiom/phrase 'smelt a rat' option (a) 'detected something wrong' gives an appropriate explanation. Other options have different meaning. 498. Select the most appropriate meaning of the given idiom. To shake off (a) to denounce (b) to get rid of (c) to recollect (d) to pass off SSC CHSL (Tier-I) –03/07/2019 (Shift-II) Ans. (b) : For the above idiom/phrase 'To shake off' option (b) 'to get rid of' gives an appropriate explanation. Other options have different meaning. 499. Select the most appropriate meaning of the given idiom. Spill the beans (a) frighten someone (b) make an excuse (c) reveal a secret (d) create difficulty SSC CHSL (Tier-I) –02/07/2019 (Shift-III) Ans : (c) For the above idiom/phrase 'spill the beans' option (c) 'reveal a secret' gives an appropriate explanation. Other options have different meaning. 500. Select the most appropriate meaning of the given idiom: Sleep on it (a) To make a wrong decision (b) To prefer to sleep than take action (c) To wait before making a decision (d) To make a decision immediately SSC CHSL (Tier-I) 21/10/2020 (Shift-I) Ans. (c) : For the above idiom/phrase 'sleep on it' option (c) 'to wait before making a decision' gives an appropriate explanation other options have different meaning. 501. Select the most appropriate meaning of the given idiom: The straw that broke the camel’s back (a) To show strength and continue without a fuss (b) To be the last in a series of events leading one to feel that it is intolerable

700

YCT

(c) To undertake a difficult journey across a desert (d) To make a wish for an escape from existing conditions SSC CHSL (Tier-I) 21/10/2020 (Shift-III) Ans. (b) : For the above idiom/phrase 'The straw that broke the camel's back' option (b) 'to be the last in a series of event leading one to feel that it is intolerable' gives an appropriate explanation. Other options have different meaning. 502. Select the most appropriate meaning of the given idiom. Stir up a hornet's nest (a) Provoke trouble (b) Lear a revolt (c) Destroy a nest (d) Call for dialogue SSC CHSL (Tier-I) 12/10/2020 (Shift-I) Ans. (a) : For the above idiom/phrase 'stir up a hornet's nest options (a) 'provoke trouble' gives an appropriate explanation. Other options have different meaning. 503. In the following question, out of the four given alternatives, select the alternative which best expresses the meaning of the meaning of the Idiom/Phrase. Snake in the grass (a) Very clear (b) Something mandatory (c) A hidden enemy (d) An exciting moment SSC MTS 10-10-2017 (Shift-III) Ans. (c) : For the above idiom/phrase, 'snake in the grass' option (c) 'A hidden enemy' gives an appropriate explanation. Other options have different meaning. 504. Select the most appropriate meaning of the given idiom. sail in the same boat (a) go for a boat ride (b) be in heavy debt (c) be in the same situation as others (d) pay a very low price for something SSC MTS – 06/08/2019 (Shift-III) Ans. (c) : For the above idiom 'sail in the same boat' options (c) 'be in the same situation as others' gives an appropriate explanation other options have different meaning. 505. Select the most appropriate meaning of the given idiom : Shot in the dark (a) To shoot someone at night (b) To guess something (c) To be unaware (d) To lie about something SSC MTS – 22/08/2019 (Shift-I) Ans. (b) : For the above idiom/phrase , 'shot in the dark" option (b) 'to guess something' gives an appropriate explanation. Other options have different meaning. 506. Select the most appropriate meaning of the given idiom Strike while the iron is hot (a) To wait for someone (b) To act at the right time (c) To do something unimportant (d) To cry all the time SSC MTS – 20/08/2019 (Shift-II) Idoms & Phrases

Ans. (b) : For the above idiom/phrase 'strike while the iron is hot' option (b) 'to act at the right time' gives an appropriate explanation. Other options have different meaning. 507. Select the most appropriate meaning of the given idiom. To spill the beans (a) To give away a secret (b) To waste food items (c) To plan a strategy (d) To drop some food SSC MTS – 19/08/2019 (Shift-III) Ans. (a) : For the above idiom/phrase 'to spill the beans' option (a) 'to give away a secret' gives an appropriate explanation. Other options have different meaning. 508. Select the option that means the same as the given idiom. To set aside (a) to reject an offer (b) to stop temporarily (c) to move sideways (d) to finish a work SSC GD – 18/02/2019 (Shift-II) Ans. (a) : For the above idiom/phrase 'to set aside' option (a) 'to reject an offer' gives an appropriate explanation. Other options have different meaning. 509. Select the option that means the same as the given idiom. To see eye to eye (a) To agree with another (b) To love another (c) To threaten another (d) To flight with another SSC GD – 14/02/2019 (Shift-I) Ans. (a) : For the above idiom/phrase "To see eye to eye" option (a) 'to agree with another' gives an appropriate explanation. Other options have different meaning. 510. Select the most appropriate meaning of the underlined idiom in the given sentence. It is a wonder how some people manage to sit on the fence without committing themselves. (a) to sit lazily and not do anything (b) relax when others are working (c) remain undecided and unsure (d) build fences and walls for sitting on SSC CGL (Tier-I) – 19/06/2019 (Shift-III) Ans. (c) For the above underlined idiom/phrase "to sit on the fence" option (c) 'remain undecided and unsure' gives an appropriate explanation. Other options have different meaning.

T 511. Select the most appropriate meaning of the underlined idiom that can be substitute in the following sentence. My son was disappointed with his failure, but he took heart and tried again. (a) Felt encouraged (b) Loved dearly (c) Was deeply affected (d) Cried miserably SSC GD 27/01/2023 (Shift-I)

701

YCT

Ans. (a) : The most appropriate meaning of the underlined idiom 'took heart' is 'Felt encouraged'. The other options are not suitable. 512. Select the most appropriate meaning of the given idiom. Through thick and thin (a) Hope during trying times (b) Only assist during good times (c) Weak and lonely (d) In good times and in bad times SSC CGL (Tier-I) 18/07/2023 (Shift-III) Ans. (d) : The most appropriate meaning of the above idiom 'Through thick and thin' is 'In good times and in bad times'. Hence option (d) will be correct. Other options give different meanings. 513. Select the most appropriate meaning of the underlined idiom in the given sentence. Pooja tried to explain the problem, but soon she tied herself up in knots. (a) Make no progress in an argument or discussion (b) Become very confused when you are trying to explain something (c) Won’t modify an opinion or agree to even small changes that another person wants (d) Be forced to explain your actions and (probably) punished SSC CGL (Tier-I) 14/07/2023 (Shift-I) Ans. (b) : For the above underlined idiom, 'tied herself up in knots’ option (b) Become very confused when you are trying to explain something' gives appropriate explanation. Other options give different meaning. 514. In the following question, out of he given four alternatives, select the alternative which best expresses the meaning of the Idiom/Phrase. To have a nodding acqaintance (a) To have a great experience (b) To neglect all the parameters (c) Know somebody/something slightly (d) To have nothing in common SSC CHSL (Tier-I) 21/03/2023 (Shift-II) Ans. (c) : For the above idiom, 'To have a nodding acquaintance', option (c) 'know somebody/something slightly' gives suitable meaning. Other options are not correct. 515. In the following question, out of the given four alternatives, select the alternative which best expresses the meaning of the idiom/phrase. To make a mountain of a mole hill (a) To violate the rules (b) To make unnecessary comments (c) To give great importance to trifles (d) To achieve big success SSC CHSL (Tier-I) 17/03/2023 (Shift-IV) Ans. (c) : For the above idiom, "To make a mountain of a mole hill", option (c) "To give great importance to trifles" gives suitable meaning. Other options are not correct. Idoms & Phrases

516. In the following question, out of the given four alternatives, select the alternative which best expresses the meaning of the idiom/phrase. To bridge the gap (a) To reduce differences and reconcile (b) To push people apart (c) To mend a broken bridge (d) To slander someone SSC CHSL (Tier-I) 15/03/2023 (Shift-I) Ans. (a) : For the above idiom, ''To bridge the gap'' option (a) ''To reduce differences and reconcile'' gives an appropriate explanation. Other options give different meaning. 517. In the following question, out of the given four alternatives, select the alternative which best expresses the meaning of the Idiom/Phrase. To burn one's finger (a) To suffer a loss by acting rashly (b) To accidentally touch a hot utensil (c) A question without any agreed answer (d) To have one's finger chopped SSC CHSL (Tier-I) 09/02/2023 (Shift-II) Ans. (a) : For the above Idiom, "To burn one's finger" option (a) "To suffer a loss by acting rashly" gives an appropriate meaning. Other options give different meaning. 518. Identify the idiom/phrase that can best substitute the underlined segment. They discussed everything in the meeting and finalised the next step. (a) through thick and thin (b) the home stretch (c) the whole nine yards (d) the eleventh hour SSC Selection Posts XI-30/06/2023 (Shift-IV) Ans. (c) : For the above sentence, the best substitute idiom of the underlined word 'everything' is option (c) 'the whole nine yards'. Other options give different meaning. 519. Select the most appropriate meaning of the given idiom. To vanish into the air (a) Perpetually postpone (b) Totally dilute (c) Completely disappear (d) Permanently mix SSC CHSL (Tier-I) 18/03/2020 (Shift-I) Ans. (c) : For the above idiom/phrase "To vanish into thin air" option (c) 'completely disappears' gives an appropriate explanation. Other options have different meaning. 520. Select the most appropriate meaning of the given idiom. In a fix (a) In a difficult situation (b) Under repairs (c) In a closed space (d) Under observation SSC Constable GD-03/12/2021 Shift-II

702

YCT

526. Select the most appropriate meaning of the given idiom. To break loose (a) To escape (b) To let go (c) To untie (d) To set free SSC Constable GD-18/11/2021 Shift-I Ans. (a) : For the above idiom 'To break loose', option (a) To escape gives suitable meaning. other options are incorrect. 527. Select the most appropriate meaning of the given idiom. To walk the tight rope (a) Be an expert (b) Be very cautious (c) Be well trained (d) Be very nervous SSC MTS-08/07/2022 Shift-I Ans. (b) : For the above idiom 'To walk the tight rope', option (b) 'be very cautions' gives suitable meaning. Other options are not correct. 528. Select the most appropriate meaning of the given idiom. To get out of hand (a) To get in a trouble (b) To get an award (c) To get angry (d) To get out of control SSC Constable GD-30/11/2021 Shift-III Ans. (d) : For the above idiom 'To get out of hand', option (d) To get out of control' gives suitable meaning. Other options are not correct. 529. Select the most appropriate meaning of the given idiom. Through thick and thin (a) In good and bad times (b) With great pride (c) To work hard to the point of exhaustion (d) In every possible way SSC MTS-05/07/2022 Shift-II Ans. (a) : For the above idiom 'through thick and thin', option (a) In good and bad times' gives suitable meaning. Other options are not correct. 530. Select the most appropriate meaning of thegiven idiom. To lead one by the nose (a) To try again after failing at something (b) To control someone completely (c) To have the freedom to choose (d) To be anxious to start something SSC MTS-06/07/2022 Shift-III Ans. (b) : For the above idiom 'To lead one by the nose', option (b) 'To control someone completely' gives suitable meaning other options are not correct. 531. Select the most appropriate meaning of the given idiom. That ship has sailed (a) To arrive early (b) To reach safely Ans. (c) : For the above idiom, 'To bear fruit', option (c) (c) Too late to do something 'To produce positive results' gives suitable meaning. (d) Too extreme to follow Other option are not correct. SSC MTS-15/07/2022 Shift-I

Ans. (a) : For the above idiom 'In a fix', option (a) 'In a difficult situation' gives suitable meaning. Other options are not correct. 521. Select the most appropriate meaning of the given idiom. To let someone off (a) To punish someone lightly (b) To thank someone (c) To allow someone (d) To scold someone severely SSC Constable GD-06/12/2021 Shift-III Ans. (a) : For the above idiom 'To let someone off', option (a) 'To punish someone lightly' gives suitable meaning. other options are not correct. 522. Select the most appropriate meaning of the given idiom. To get wind of (a) To learn something secret (b) To gain something unfairly (c) To enjoy a nice weather (d) To be clear about something SSC Constable GD-14/12/2021 Shift-III Ans. (a) : For the above idiom 'To get wind of', option (a) 'to learn something secret' gives suitable meaning. Other option are not correct. 523. Select the most appropriate meaning of the given idiom. Take up room (a) Feel choked (b) Use mildly (c) Occupy space (d) Rent a place SSC Constable GD-14/12/2021 Shift-II Ans. (c) : For the above idiom 'Take up room', option (c) 'occupy space' gives suitable meaning. Other options are not correct. 524. Select the most appropriate meaning of the given idiom. To beat around the bush (a) To discuss the main topic (b) To avoid the main issue (c) To avoid noticing someone (d) To explain something in detail SSC Constable GD-17/11/2021 Shift-II Ans. (b) : For the above idiom 'To beat around the bush', option (b) 'To avoid the main issue' gives suitable meaning. Other options are not correct. 525. Select the most appropriate meaning of the given idiom. To bear fruit (a) To accept the truth (b) To be creative (c) To produce positive results (d) To put in hard work SSC Constable GD-23/11/2021 Shift-III

Idoms & Phrases

703

YCT

Ans. (c) : For the above idiom 'That ship has sailed', 537. Select the most appropriate meaning of the given idiom. option (c) too late to do something; gives suitable meaning. Other options are not correct. Tit for tat (a) A wild guess (b) Prepare to leave 532. Select the most appropriate meaning of the (c) Counter attack (d) Go smoothly given idiom. SSC CHSL 25.05.2022 Shift-I to foot the bill (a) To reveal a secret (b) To be friendly Ans. (c) : For the above idiom 'Tit for tat', option (c) (c) To hold grudges (d) To make payment counter attack' gives suitable meaning. Other option are SSC MTS-07/07/2022 Shift-I not correct. Ans. (d) : For the above idiom 'to foot the bill', option 538. Select the most appropriate meaning of the (d) to make payment gives suitable meaning. other given idiom. options are not correct. Take one's hat off to someone 533. Select the most appropriate meaning of the (a) express anger to someone given idiom. (b) show admiration for someone Tricks of the trade (c) display humility to someone (a) A particular profession (d) indicate disapproval of someone (b) A trick to deceive SSC Stenographer (Grade C & D) 15.11.2021 Shift-I (c) Special skills or knowledge Ans. (b) : For the above idiom 'take one's hat off to (d) An easy task someone option (b) show admiration for someone gives SSC MTS-05/07/2022 Shift-III suitable meaning. Other options are not correct. Ans. (c) : For the above idiom' Tricks of the trade', 539. Select the most appropriate meaning of the option (c) 'special skills or knowledge' gives suitable given idiom. meaning. Other options are not correct. In the same boat 534. Select the most appropriate meaning of the (a) getting married given idiom. (b) escaping something together To take after (c) doing something in a team (a) To mock someone (d) sharing the same problem (b) To be similar in appearance SSC Stenographer (Grade C & D) 12.11.2021 Shift-II (c) To chase someone Ans. (d) : For the above idiom 'In the same boat', option (d) To change sides often (d) sharing the same problem' gives suitable meaning. SSC CGL (Tier-I) 12/04/2022 Shift-I Other options are not correct. Ans. (b) : For the above idiom 'To take after', option (b) 540. Select the most appropriate meaning of the to be similar in appearance' gives suitable meaning. given idiom. Other options are not correct. Take a cue from someone 535. Select the option that gives the most (a) take someone's position in the queue appropriate meaning of the underlined idiom. (b) be strongly influenced by someone It is good to take into account all the important (c) wait patiently to listen to someone factors before making a decision. (d) treat someone as very important (a) control (b) leave SSC Stenographer (Grade C & D) 15.11.2021 Shift-II (c) consider (d) select SSC CGL (Tier-I) 11/04/2022 Shift-III Ans. (b) : For the above idiom, 'Take a cue from Ans. (c) : For the above idiom 'take into account', someone option (b) be strongly influenced by someone option (c). 'Consider' gives suitable meaning. Other gives suitable meaning. Other options are not correct. options are not correct. 541. Select the most appropriate meaning of the given idiom. 536. Select the most appropriate meaning of the To make up for given idiom. (a) to praise someone to get something for To hold water oneself (a) To be unsteady and unsure (b) to prepare or arrange something (b) To be sad and depressed (c) to replace something that is lost or damaged (c) To be valid and reasonable (d) to help to make something possible (d) To be large and deep SSC CGL (Tier-I) 19/04/2022 Shift-II SSC Stenographer (Grade C & D) 11.11.2021 Shift-I Ans. (c) : For the above idiom 'To hold water', option Ans. (c) : For the above idiom 'To make up for' option (c) To be valid and reasonable' gives suitable meaning. (c) To replace something that is lost or damaged gives Other options are not correct. suitable meaning. Other options are not correct. Idoms & Phrases

704

YCT

542. Select the most appropriate meaning of the given idiom. Twist someone's arm (a) handle a situation cleverly (b) understand a problematic situation (c) persuade someone to do something (d) hurt someone by mistake SSC Stenographer (Grade C & D) 11.11.2021 Shift-II Ans. (c) : For the above idiom 'twist someone's arm' option (c) persuade somebody to do something gives suitable meaning. Other options give different meaning. 543. Select the most appropriate meaning of the given idiom. To call the shots (a) To take pictures (b) To have control (c) To shoot someone (d) To call someone SSC Stenographer (Grade C & D) 11.11.2021 Shift-II Ans. (b) : For the above idiom 'To call the shots' option (b) To have control gives suitable meaning other options are not correct. 544. Select the most appropriate meaning of the given idiom. To get cold feet (a) To become afraid to do something (b) Not to think clearly (c) To get frost bite in winter (d) To dip one's feet in cold water SSC Stenographer (Grade C & D) 11.11.2021 Shift-I Ans. (a) : For the above idiom 'To get cold feet', option (a) to become afraid to do something gives suitable meaning. Other options are not correct. 545. Select the most appropriate meaning of the given idiom. The apple of someone's eyes (a) the special apple that someone has bought (b) the apple a day that keeps the doctor away (c) the apple that is rotten and irritating one's eye (d) the person that someone loves and is proud of SSC Stenographer (Grade C & D) 11.11.2021 Shift-I Ans. (d) : For the above idiom 'The apple of someone's eyes' option (d) the person that someone loves and is proud of gives suitable meaning. Other options are not correct. 546. Select the most appropriate meaning of the given idiom. To cut corners (a) To reduce corners in a building to save space (b) To save money and economise (c) To cut the corners of a cloth for stitching (d) To remove sharp edges in furniture SSC Stenographer (Grade C & D) 11.11.2021 Shift-I Ans. (b) : For the above idiom 'To cut corners', option (b) 'To save money and economise' gives suitable meaning, other options are not correct. Idoms & Phrases

547. Select the most appropriate meaning of the given idiom. Throw caution to the wind (a) enjoy a good outing (b) not go out when it is windy (c) behave in a reckless manner (d) act in a thoughtful manner SSC Stenographer (Grade C & D) 12.11.2021 Shift-I Ans. (c) : For the above idiom 'Throw caution to the wind option (c) 'behave in a reckless manner' gives correct explanation. Other options are not correct. 548. Select the most appropriate meaning of the given idiom. Thick as thieves (a) Having a close friendship (b) Being very dishonest (c) Never helping anyone (d) Telling lies all the time SSC CHSL (Tier-I) –05/08/2021 (Shift-I) Ans. (a) : For the above idiom, 'Thick as thieves' option (a) 'Having close friendship' gives appropriate explanation. Other options have different meaning. 549. Select the most appropriate meaning of the given idiom. To cut corners (a) cut something easily without spoiling it (b) cut something into pieces (c) be able to do a job effortlessly (d) not do a thing well in order to save money or effort SSC CHSL (Tier-I) –09/08/2021 (Shift-I) Ans. (d) : For the above idiom 'To cut corners' option (d) 'not do a thing well in order to save money or effort' gives an appropriate explanation other options have different meaning. 550. Select the most appropriate meaning of the given idiom. Tide someone over (a) Get a boat ready to cross a river or water body (b) Ask someone for financial assistance (c) Complete a voyage successfully (d) Give temporary help, usually financial SSC CGL (Tier-I) –20/08/2021 (Shift-I) Ans. (d) : For the above idiom/phrase, "Tide someone over' option (d) 'give temporary help, usually financial' gives an appropriate explanation. Other options have different meaning. 551. Select the most appropriate meaning of the given idiom. To turn over a new leaf (a) To change your behaviour in a positive way (b) To become famous (c) To buy new things (d) To understand others' problems

705

SSC Stenographer Grade C&D –05/02/2019 (3:5pm)

Ans. (a) : For the above idiom/phrase "to turn over a new leaf" option (a) 'to change your behaviour in a positive way' gives an appropriate explanation. Other options have different meaning. YCT

552. Select the most appropriate meaning of the given idiom. Through and through (a) partially (b) eventually (c) completely (d) gradually SSC CPO-SI – 25/11/2020 (Shift-I) Ans. (c) : For the above idiom/phrase "Through and through" option (c) 'completely' gives an appropriate explanation. Other options have different meaning. 553. Select the most appropriate meaning of the given idiom. To take stock of (a) to busy stocks (b) to check the accounts (c) to store goods (d) to assess a situation SSC CPO-SI – 25/11/2020 (Shift-II) Ans. (d) : For the above idiom/phrase, 'to take stock of' option (d) 'to assess a situation' gives an appropriate explanation. Other options have different meaning. 554. Select the most appropriate meaning of the given idiom. Through thick and thin (a) going through pain (b) under all circumstances (c) by unfair means (d) under extreme pressure SSC CPO-SI – 24/11/2020 (Shift-II) Ans. (b) : For the above idiom/phrase "through thick and thin" option (b) 'under all circumstances' gives an appropriate explanation other options have different meaning. 555. Select the most appropriate meaning of the given idiom. Tide over (a) to jump over (b) to change sides (c) to deceive (d) to overcome SSC CPO-SI – 24/11/2020 (Shift-II) Ans. (d) : For the above idiom/phrase "Tide over" option (d) 'to overcome' gives an appropriate explanation other options have different meaning. 556. Select the most appropriate meaning of the given idiom. The sword of Damocles (a) a poignant weapon (b) an authentic sword (c) an impending threat (d) a rare artifact SSC CPO-SI – 24/11/2020 (Shift-II) Ans. (c) : For the above idiom/phrase, "The sword of Damocles" option (c) 'an impending threat' gives an appropriate explanation. Other options have different meaning. 557. Select the most appropriate meaning of the underlined idiom in the given sentence. The dog played in the pond to his heart's content. (a) only upto his legs (b) till he was completely sick (c) as much as he wanted to (d) avoiding getting too wet SSC CGL (Tier-I) – 13/06/2019 (Shift-III) Ans : (c) For the above idiom/phrase "to his heart's content" option (c) 'as much as he wanted' gives an appropriate explanation Other options have different meaning. Idoms & Phrases

558. Select the most appropriate meaning of the underlined idiom in the given sentence. The authorities have turned a deaf ear to all our requests. (a) acknowledged (b) accepted (c) presented (d) neglected SSC CGL (Tier-I) – 12/06/2019 (Shift-III) Ans. (d) : For the above idiom/phrase "turned a deaf ear" option (d) 'neglected' gives an appropriate explanation. Other options have different meaning. 559. Select the most appropriate meaning of the given idiom. To play ducks and drakes (a) to use recklessly (b) to change places (c) to act cleverly (d) to be friendly SSC CGL (Tier-I) – 04/06/2019 (Shift-I) Ans : (a) For the above idiom/phrase "To play ducks and drakes" option (a) 'to use recklessly' gives an appropriate explanation. Other options have different meaning. 560. Select the most appropriate meaning of the underlined idiom in the given sentence. In spite of her severe accident she took heart and moved forward. (a) Gathered courage (b) Overcame problems (c) Felt depressed (d) Acted promptly SSC CPO-SI – 09/12/2019 (Shift-I) Ans : (a) For the above idiom/phrase "took heart and moved" option (a) 'gathered courage' gives an appropriate explanation. Other options have different meaning. 561. Select the most appropriate meaning of the given idiom. Till the cows come home (a) For a small part (b) Till the work is finished (c) For a long time (d) In the evening SSC CPO-SI – 11/12/2019 (Shift-I) Ans : (c) For the above idiom/phrase, 'Till the cows come home" option (c) 'for a long time' gives an appropriate explanation. Other options have different meaning. 562. Select the most appropriate meaning of the given idiom. Touch and go (a) Unimportant (b) Unchanging (c) Uncertain (d) Undoubted SSC CPO-SI – 11/12/2019 (Shift-I) Ans : (c) For the above idiom/phrase, "Touch and go" option (c) 'uncertain' gives an appropriate explanation. Other options have different meaning. 563. Select the most appropriate meaning of the given idiom. To be thrown in at the deep end (a) need to do one thing at a time (b) unable to do the given work (c) try to do too many things at the same time (d) learn to do the difficult things too soon SSC CPO-SI – 12/12/2019 (Shift-II)

706

YCT

Ans : (d) For the above idiom/phrase "To be thrown in at the deep end" option (d) 'learn to do the difficult things too soon' give an appropriate explanation. Other options have different meaning. 564. Select the most appropriate meaning of the underlined idiom in the sentence. Simran was under tremendous stress because she had too many irons in the fire. (a) She had too many clothes to iron (b) She was suffering from many ailments (c) She was involved in many activities (d) She had made a lot of risky investments SSC CPO-SI – 12/12/2019 (Shift-I) Ans : (c) For the above idiom/phrase "to many irons in the fire" option (c) 'involved in many activities' gives an appropriate explanation. Other options have different meaning. 565. Select the most appropriate meaning of the underlined idiom in the sentence. His family has supported him through thick and thin. (a) on occasions when he had a lot of money (b) even though he has put on weight (c) when there was a lot of work to be done (d) in times of good and bad fortune SSC CPO-SI – 13/12/2019 (Shift-II) Ans. (d) : For the above idiom/phrase "through thick and thin" option (d) 'in times of good and bad fortune' gives an appropriate explanation. Other options have different meaning. 566. Select the most appropriate meaning of the given idiom. Of the first water (a) of ancient origin (b) of favorable opinion (c) of no significance (d) of the best quality SSC CPO-SI – 13/12/2019 (Shift-I) Ans. (d) : For the above idiom/phrase "of the first water" option (d) 'of the best quality' gives an appropriate explanation. Other options have different meaning. 567. Select the most appropriate meaning of the given idiom. To take the bull by the horns (a) to escape unhurt (b) to act foolishly (c) to enjoy risky sports (d) to face danger boldly SSC CHSL (Tier-I) –05/07/2019 (Shift-I) Ans : (d) For the above idiom/phrase "To take the bull by the horns' option (d) 'to face the danger boldly' gives an appropriate explanation. Other options have different meaning. 568. Select the most appropriate meaning of the given idiom. Turn a deaf ear (a) to make someone deaf (b) to pay attention (c) to pay no heed (d) to listen carefully SSC CHSL (Tier-I) –05/07/2019 (Shift-II) Idoms & Phrases

Ans : (c) For the above idiom/phrase "Turn a deaf ear" option (c) 'to pay no heed' gives an appropriate explanation. Other options have different meaning. 569. Select the most appropriate meaning of the given idiom. Take to task (a) to assign work (b) to give an award (c) to praise (d) to rebuke SSC CHSL (Tier-I) –05/07/2019 (Shift-II) Ans : (d) For the above idiom/phrase "Take to task option (d) 'to rebuke' gives an appropriate explanation. Other options have different meaning. 570. Select the most appropriate meaning of the given idiom. to air dirty linen in public (a) to discuss private affairs in public (b) to hang out clothes in the open (c) to continue to complain (d) to stand up and fight SSC CHSL (Tier-I) –02/07/2019 (Shift-I) Ans. (a) : For the above idiom/phrase "to air dirty linen in public "option (a) 'to discuss private affairs in public gives an appropriate explanation. Other options have different meaning. 571. Select the most appropriate meaning of the given idiom. throw in the towel (a) face the situation (b) think of a solution (c) admit defeat (d) drop something SSC CHSL (Tier-I) –02/07/2019 (Shift-I) Ans. (c) : For the above idiom/phrase "Throw in the towel" option (c) 'admit defeat' gives an appropriate explanation. Other options have different meaning. 572. Select the most appropriate meaning of the given idiom. tit for tat (a) to reward people for the good done (b) to do harm as done to you (c) to make someone angry (d) to take advantage of someone SSC CHSL (Tier-I) –02/07/2019 (Shift-II) Ans : (b) For the above idiom/phrase 'tit for tat' option (b) 'to do harm as done to you' gives an appropriate explanation. Other options have different meaning. 573. Select the most appropriate meaning of the idiom. Touch all bases (a) Imagine you are flying (b) Force someone to do something (c) Include everything (d) Stretch your hands high SSC CHSL (Tier-I) 18/03/2020 (Shift-II) Ans. (c) : For the above idiom/phrase ' touch all bases' option (c) 'Include everything' gives an appropriate explanation. Other options have different meaning. 574. Select the most appropriate meaning of the given idiom. Take one's hat off to someone (a) Show admiration (b) Indicate disapproval (c) Express anger (d) Display humility SSC CHSL (Tier-I) 18/03/2020 (Shift-I)

707

YCT

Ans. (a) : For the above idiom/phrase "Take one's hat off to someone" option (a) 'show admiration' gives an appropriate explanation. Other options have different meaning. 575. Select the most appropriate meaning of the underlined in the given sentence. Akshay is likely to take the heart for having miscalculated the expenses. (a) enjoy warm climate (b) receive criticism (c) feel very hot (d) scold everyone SSC CHSL (Tier-I) 13/10/2020 (Shift-III) Ans. (b) For the above idiom/phrase "Take the heart" option (b) 'receive criticism' gives an appropriate explanation. Other options have different meaning. 576. Select the most appropriate meaning of the given idiom. Take something with a pinch of salt (a) To take long to understand what is being said (b) To believe only part of something (c) To consider everyone a liar (d) To think of oneself as always right SSC CHSL (Tier-I) 21/10/2020 (Shift-II) Ans. (b) : For the above idiom/phrase "Take something with a pinch of salt" option (b) 'to believe only part of something' gives an appropriate explanation. Other options have different meaning. 577. Select the most appropriate meaning of the given idiom. To take one's hat off (a) To fool someone (b) To feel hot (c) To please someone (d) To admire someone SSC MTS – 07/08/2019 (Shift-I) Ans. (d) : For the above idiom/phrase "to take one's hat off" option (d) 'to admire someone' gives an appropriate explanation. Other options have different meaning. 578. Select the most appropriate meaning of the given idiom. The ball is in (one's) court (a) To be responsible for further action (b) To pass the responsibility to another (c) To be fearful of taking any action (d) To risk everything in one venture SSC MTS – 02/08/2019 (Shift-I) Ans. (a) : For the above idiom/phrase "The ball is in (one's) court" option (a) To be responsible for forther action. gives an appropriate explanation. Other have different meaning. 579. Select the most appropriate meaning of the given idiom. Too close to call (a) A very narrow margin (b) Huddled together (c) very close to heart (d) Winning a competition SSC MTS – 21/08/2019 (Shift-II) Ans. (a) : For the above idiom/phrase "To close to call" option (a) 'a very narrow margin' gives an appropriate explanation. Other options have different meaning. Idoms & Phrases

580. Select the most appropriate meaning of the given idiom. Throw in the towel (a) To take a shower (b) To decide firmly (c) To provoke someone (d) To admit defeat or failure SSC MTS – 16/08/2019 (Shift-II) Ans. (d) : For the above idiom/phrase "Throw in the towel" option (d) 'to admit defeat of failure' gives an appropriate explanation. Other options have different meaning. 581. Select the meaning of the given idiom. To be tight-lipped (a) To keep quiet (b) To keep records (c) To keep up the spirit (d) To keep a fast SSC MTS – 13/08/2019 (Shift-III) Ans. (a) : For the above idiom/phrase "To be tightlipped" option (a) 'to keep quiet' gives an appropriate explanation. Other options have different meaning. 582. Select the most appropriate meaning of the given idiom. To take the bull by its horns (a) To deal with a difficult situation directly (b) To accept defeat before a strong adversary (c) To invest money in an inflationary market (d) To go on a hunting expedition SSC MTS – 22/08/2019 (Shift-II) Ans. (a) : For the above idiom/phrase "To take bull by its horns" option (a) 'to deal with a difficult situation directly' gives an appropriate explanation. Other options have different meaning. 583. Select the option that means the same as the given idiom. To turn over a new leaf (a) To be always careful about one's behaviour (b) To change one's behavior for better (c) to make new things happen (d) To turn over the page of a book SSC GD – 02/03/2019 (Shift-I) Ans. (b) : For the above idiom/phrase "To turn over new leaf" option (b) 'to change one's behaviour for better' gives an appropriate explanation. Other options have different meaning.

U 584. Select the most appropriate idiom for the following statement. We should be together to fight any danger. Otherwise, separately, we won’t be able to fight. (a) United we stand, divided we fall. (b) A full purse makes the mouth speak. (c) Nothing ventured, nothing gained. (d) Well begun is half done. SSC CHSL (Tier-I) 10/08/2023 (Shift-I)

708

YCT

Ans. (a) : The appropriate idiom for the statement we should be together to fight any danger. Otherwise separately, we won't be able to fight' will be option (a) United we stand, divided we fall. meaning of other idioms proverbs given in the options are – A full purse makes the mouth speak → wealth can be a source of power and influence and that those who possess it may be more willing to shore their thoughts or ideas with others. Nothing ventured, nothing gained → you cannot expect to achieve anything if you never take any risks. well begun is falf done – Beginning a project/work sell makes it easier to do the rest. 585. Select the most appropriate meaning of the given idiom. Under the weather (a) In good weather (b) Damp and wet (c) Bright and sunny (d) Not feeling well SSC Stenographer (Grade C & D) 11.11.2021 Shift-I Ans. (d) : For the above idiom 'Under the weather', option 'Not feeling well' gives suitable meaning. Other options are not correct. 586. Select the most appropriate meaning of the underlined idiom in the given sentence. His probation was extended because his performance was not up to the mark. (a) more than expected (b) up to the required standard (c) of the desired height (d) lacking in maturity SSC CGL (Tier-II) 16/11/2020 (3 pm - 5 pm) Ans. (b) : For the above idiom/phrase "up to the mark" option (b) 'up to the required standard' gives an appropriate explanation. Other options have different meaning. 587. In the following question, out of the given four alternatives, select the alternative which best expresses the meaning of the Idiom/Phrase. Up against the wall (a) To take care of themselves and their own interests and safety (b) In a disorderly fashion (c) In an inextricable situation (d) Performing well in a difficult or competitive situation SSC CGL (Phase-II) 17/02/2018 Ans. (c) For the above idiom/phrase "up against the wall" option (c) 'in an inextricable situation' gives an appropriate explanation. Other options have different meaning. 588. Select the most appropriate meaning of the underlined idiom in the given sentences. The invigilator did not know that the two boys were exchanging notes under his nose. (a) rolled into small pellets (b) written in small letters (c) wrapped in handkerchief's (d) right in front of him SSC CGL (Tier-I) – 13/06/2019 (Shift-II) Idoms & Phrases

Ans. (d) : For the above idiom/phrase "under his nose" option (d) 'right in front of him' gives an appropriate explanation. Other options have different meaning. 589. Select the most appropriate meaning of the given idiom. Up in arms (a) to be angry (b) to be defensive (c) to be nervous (d) to be rude SSC CPO-SI – 09/12/2019 (Shift-II) Ans : (a) For the above idiom/phrase "Up in arms" option (a) 'to be angry' gives an appropriate explanation. Other options have different meaning. 590. Select the most appropriate meaning of the given idiom. Upset someone's applecart (a) Upsetting someone by dropping the apples (b) Making someone angry by ruining business (c) Doing something that fails someone's plan (d) Spoiling the apples in someone's cart SSC CHSL (Tier-I) 26/10/2020 (Shift-I) Ans. (c) : For the above idiom/phrase "upset someone's applecart" option (c) 'Doing something that fails someone's plane' gives an appropriate explanation. Other options have different meaning. 591. Select the most appropriate meaning of the given idiom: Under the weather (a) To face bad weather conditions (b) To feel wronged by friends (c) Feel strong and energetic (d) To be in low spirits SSC CHSL (Tier-I) 21/10/2020 (Shift-III) Ans. (d) : For the above idiom/phrase "under the weather" options (d) 'to be in low spirits' gives an appropriate explanation. Other options have different meaning. 592. Select the most appropriate meaning of the given idiom. Up in arms (a) Angry about something (b) Throw up arms in joy (c) Given up fighting and surrender (d) Divide into armed group SSC CHSL (Tier-I) 12/10/2020 (Shift-I) Ans. (a) : For the above idiom/phrase "up in arms" option (a) 'Angry about something' gives an appropriate explanation. Other options have different meaning. 593. Select the most appropriate meaning of the given idiom. Under wraps (a) under estimate (b) double layered (c) secretly (d) underhand SSC CHSL (Tier-I) –04/08/2021 (Shift-I) Ans. (c) : For the above idiom/phrase "under wraps" option (c) 'secretly' gives an appropriate explanation. Other options have different meaning.

709

YCT

V 594. Select the most appropriate meaning of the given idiom. Vanish into thin air (a) dilute (b) postpone (c) disappear (d) mix SSC Stenographer (Grade C & D) 12.11.2021 Shift-I Ans. (c) : For the above idiom 'Vanish into thin air, option (c) 'disappear' gives correct explanation. Other options are not correct. 595. Select the most appropriate meaning of the given idiom. Vanish into thin air (a) To completely disappear (b) To remove suddenly (c) To postpone something (d) To begin suddenly SSC Sel. Post Phase-VIII (M.L.) 09.11.2020 (Shift-III)

Ans. (a) : For the above idiom/phrase "To vanish into thin air" Option (a) 'to completely disappear' gives an appropriate explanation. Other options have different meaning. 596. Select the most appropriate meaning of given idiom. Vim and Vigor (a) Near to perfection (b) Energy and enthusiasm (c) Remove suddenly (d) Begin something SSC CPO-SI – 09/12/2019 (Shift-II) Ans. (b) : For the above idiom/phrase "vim and vigor" option (b) 'Energy and enthusiasm' gives an appropriate explanation. Other options have different meaning. 597. Select the most appropriate meaning of given idiom. Vale of tears (a) full of enthusiasm (b) full of troubles (c) wild and wooly (d) walking carefully SSC MTS – 07/08/2019 (Shift-I) Ans. (b) : For the above idiom, "vale the tears" option (b) 'full of trouble' gives an appropriate explanation. Other options have different meaning.

W 598. In the following question, out of the given four alternatives, select the alternative which best expresses the meaning of the Idiom/Phrase. work against the clocks (a) To fill one with terror (b) To work fast (c) To die while still working (d) To move anti-clockwise SSC MTS 16/05/2023 (Shift-II) Ans. (b) : The meaning of the above given idiom/phrase- 'work against the clock' is 'To work fast.' Other options give different meaning. Idoms & Phrases

599. Select the most appropriate idiom for the underlined segment in the given sentence. Without any question, Naresh is my good friend. (a) A good deal of (b) Care a hang (c) Account for (d) Beyond a doubt SSC GD 31/01/2023 (Shift-II) Ans. (d) : The most appropriate idiom of the underlined segment 'Without any question' is Beyond a doubt.' Meaning of the other idiomsA good deal of - a lot of Care a hang - show no cencern or interest Account for - to give a reason or explanation for (something) 600. Select the most appropriate meaning of the given idiom. Will-o-the-wisp (a) A splendid dream (b) An unrealistic or unattainable goal (c) Having a strong will (d) Being greedy for money SSC GD 06/02/2023 (Shift-III) Ans. (b) : The most appropriate meaning of the given idiom 'will-o-the-wisp' means– 'An unrealistic or unattainable goal'. will-o-the-wisp :- something that is impossible to get or achieve. Hence the correct answer is options (b). 601. Choose the most appropriate meaning of the underlined phrase. Many analysts are tipping stable, blue-chip companies to weather the storm. (a) To deal with a simple situation without being harmed (b) To deal with a situation without thinking (c) To deal with a simple situation without harming anyone else (d) To deal with a difficult situation without being harmed SSC CGL (Tier-I) 24/07/2023 (Shift-III) Ans. (d) : For the above underline idiom 'To weather the storm' option (d) 'To deal with a difficult situation without being harmed' gives appropriate explanation. Other options give different meaning. 602. Select the most appropriate idiom for the underlined segment in the following sentence. Brave people never hide at the moment of decision. (a) Turn back the hands of time (b) Beat the clock (c) A hell of time (d) When the crunch comes SSC CHSL (Tier-I) 02/08/2023 (Shift-I) Ans. (d) : The most appropriate idiom of the above underlined segment 'at the moment of decision is option (d) 'when the crunch comes,' which means 'when a situation becomes extremely serious and a decision must be made.' Other options give different meaning.

710

YCT

603. Select the most appropriate idiom to fill in the blank. When I was in Italy for a tour, the tourist guide provided by Azim was ________. Because of him, the whole tour was successful. (a) worth his weight in gold (b) armed to the teeth (c) burning the midnight oil (d) looking for a needle in a haystack SSC CHSL (Tier-I) 04/08/2023 (Shift-III) Ans. (a) : The Most appropriate idion in the above blank space option (a) worth his weight in gold' would be appropriate which means 'to be very useful or helpful.' Meaning of other idioms – Armed to the teath – 'Carrying many weapons.' Burning the midnight oil – 'Read or work late into the night'. Looking for a needle in a haysteck – 'Someone or something. That is very hard to find.' 604. In the following question, out of the given four alternatives, select the alternative which best expresses the meaning of the Idiom/Phrase. The whys and wherefores (a) References (b) Circumstances (c) Requirements (d) Reasons SSC CHSL (Tier-I) 21/03/2023 (Shift-IV) Ans. (d) : For the above idiom/Phrase, 'The whys and wherefores', option (d) ‘Reasons’ gives an appropriate explanation. Other options give different meaning. 605. Select the most appropriate meaning of the given idiom. Wee hours of the day (a) Dusk (b) Midnight (c) Dawn (d) Noon SSC Constable GD-22/11/2021 Shift-II Ans. (c) : For the above idiom 'Wee hours of the day', option (c) Dawn gives suitable meaning. Other options are not correct. 606. Select the most appropriate meaning of the given idiom. Win laurels (a) To gain honour (b) To punish (c) To be decisive (d) To change SSC CHSL 26.05.2022 Shift-II Ans. (a) : For the above idiom 'win laurels', option (a) To gain honour' gives suitable meaning. Other options are not correct. 607. Select the most appropriate meaning of the given idiom. Wipe the floor with someone (a) To defeat someone (b) To cheat someone (c) To criticise someone (d) To help someone SSC CHSL 30.05.2022 Shift-I Ans. (a) : For the above idiom 'Wipe the floor with someone option (a) 'To defeat someone' gives suitable meaning. Other options are not correct. Idoms & Phrases

608. Select the most appropriate meaning of the given idiom. Wash your dirty linen in public (a) Fight with everyone unnecessarily (b) Discuss private matters in front of others (c) Complain about everything (d) Hangclothes outside SSC CHSL (Tier-I) –13/04/2021 (Shift-I) Ans. (b) : For the above idiom, "wash your dirty linen in public" option (b) 'discuss private matters in front of others' gives an appropriate explanation. Other options have different meaning. 609. Select the most appropriate meaning of the underlined idiom in the given sentence. He is very impulsive and does things without rhyme or reason. (a) without proper planning (b) without a reasonable explanation (c) without any resources (d) without anyone's help SSC CHSL (Tier-I) –06/08/2021 (Shift-I) Ans. (b) : For the above idiom, "without rhyme or reason" option (b) 'without a reasonable explanation' give an appropriate explanation other options have different meaning. 610. In the following question, out of the given four alternatives, select the alternative which best expresses the meaning of the Idiom/Phrase. Whoop it up (a) Be continually reminded of an unpleasant topic (b) Enjoying in a noisy way, usually in a group (c) Be totally ignorant or incompetent (d) Used as an exhortation to overcome or be rid of something SSC CGL (Phase-II) 17/02/2018 Ans. (b) For the above idiom, "whoop it up" option (b) 'enjoying in a noisy way, usually in a group' gives an appropriate explanation. Other options have different meaning. 611. In the following question, out of the given four alternatives, select the alternative which best expresses the meaning of the Idiom/Phrase. With a vengeance (a) A complete disaster (b) Having similar views or attitude to something (c) To look out for something without particular attention (d) Used to emphasize the degree to which something occurs SSC CGL (Phase-II) 17/02/2018 Ans. (d) For the above idiom, "with a vengeance' option (d) 'used to emphasize the degree to which something occurs' gives an appropriate explanation. Other options have different meaning. 612. In the following question, out of the given four alternative, select the alternative which best expresses the meaning of the Idiom/Phrase. Weather the storm (a) Be absolutely unwilling to operate (b) Fall over in sudden or dramatic way (c) Survive a period of difficulty (d) Dispute a decision or choice already made SSC CGL (Phase-II) 17/02/2018

711

YCT

Ans. (c) For the above idiom/phrase "weather the storm" option (c) 'survive a period of difficulty' gives an appropriate explanation. Other options have different meaning. 613. In the following question, out of the given four alternatives, select the alternative which best expresses the meaning of the Idiom/Phrase. Wear the green willow (a) To do something for someone as an act of kindness (b) Suffer unrequited love (c) Cause someone to be very frightened (d) Producing a lavish celebrating feast SSC CGL (Phase-II) 17/02/2018 Ans. (b) For the above idiom, "wear the green willow" option (b) 'suffer unrequited love' gives an appropriate explanation. Other options have different meaning. 614. In the following question, out of the given alternatives, select the alternative which best expresses the meaning of the Idiom/Phrase. Wild and woolly (a) Uncouth in appearance or behavior (b) Either too much of something or too little (c) Near to perfection (d) Ignoring all obligations SSC CGL (Phase-II) 17/02/2018 Ans. (a) For the above idiom, "wild and woolly" option (a) 'uncouth in appearance or behavior' gives an appropriate explanation. Other options have different meaning. 615. Select the most appropriate meaning of the given idiom. Walking on thin ice (a) walking carefully (b) performing a ballet (c) slipping on ice (d) doing something risky SSC CPO-SI – 23/11/2020 (Shift-II) Ans. (d) : For the above idiom/phrase "walking on thin ice" option (d) 'doing something risky' gives an appropriate explanation. Other options have different meaning. 616. Select the most appropriate meaning of the underlined idiom the given sentence. A mountaineer has to walk the tight rope as a small slip can prove to be fatal. (a) be an expert (b) be very nervous (c) be very cautious (d) be well trained SSC CGL (Tier-I) – 11/06/2019 (Shift-II) Ans. (c) : For the above idiom/phrase "walk the tight rope "option (c) 'be very cautious' gives an appropriate explanation. Other options have different meaning. 617. Select the most appropriate meaning of the given idiom. Work against the clock (a) work late in the night (b) work against all problems (c) work with concentration (d) work in great hurry SSC CPO-SI – 09/12/2019 (Shift-II) Ans : (d) For the above idiom/phrase "work against the clock" option (d) 'work in great hurry' gives an appropriate explanation. Other options have different meaning. Idoms & Phrases

618. Select the most appropriate meaning of the underlined idiom in the given sentence. Our PM is received with open arms wherever he goes. (a) Harmoniously (b) Promptly (c) Honourably (d) Warmly SSC CPO-SI – 09/12/2019 (Shift-I) Ans : (d) For the above idiom/phrase, "with open arms" option (d) 'warmly' gives an appropriate explanation. Other options have different meaning. 619. In the following question, out of the four given alternatives, select the alternative which best expresses the meaning of the idiom/Phrase. Word of mouth (a) Give money for talking a lot about nothing (b) Through the verbal sharing of information (c) Bring order and discipline in the company (d) To leave someone in the time of difficulty SSC MTS 7-10-2017 (Shift-I) Ans. (b) : For the above idiom/phrase "word of mouth" option (b) 'through the verbal sharing of information' gives an appropriate explanation. Other options have different meaning. 620. In the following question, out of the given four alternatives, select the alternative which best expresses the meaning of the Idiom/Phrase. With open arms (a) Cordially (b) Often (c) Seldom (d) Frequently SSC MTS 9-10-2017 (Shift-I) Ans : (a) For the above idiom/phrase "with open arms" option (a) 'cordially' gives an appropriate explanation. Other options have different meaning. 621. Select the most appropriate meaning of the given idiom. When pigs fly (a) A difficult situation (b) An argument between two people (c) A practical idea (d) A time that will never come SSC MTS – 19/08/2019 (Shift-I) Ans. (d) : For the above idiom/phrase "when pigs fly" option (d) 'A time that will never come' gives an appropriate explanation. Other options have different meaning. 622. Select the most appropriate meaning of the given idiom. Work like a charm (a) To do something faster (b) To delay a task (c) To work very well (d) To do a magic trick SSC MTS – 14/08/2019 (Shift-III) Ans. (c) : For the above idiom "work like a charm" option (c) 'to work very well' gives an appropriate explanation. Other options have different meaning.

712

YCT

623. Select the meaning of the given idiom. A wild-goose chase (a) Useless search (b) Timely action (c) Wise decision (d) Delayed action SSC MTS – 09/08/2019 (Shift-I) Ans. (a) : For the above idiom/phrase "A wild-goose chase" option (a) 'useless search' gives an appropriate explanation. Other options have different meaning.

Ans. (a) : For the above idiom, Yeoman's services option (a) 'excellent work' gives an appropriate explanation. Other options have different meaning. 628. Select the most appropriate meaning of given idiom. Young blood (a) young person (b) inexperienced person (c) very attentive (d) angry and hurt SSC CPO-SI – 25/11/2020 (Shift-II) Ans. (b) : For the above idiom, "Young blood" option (b) 'inexperienced person' gives an appropriate 624. Select the most appropriate meaning of given explanation. Other options have different meaning. idiom. X-factor (a) a special talent or quality 629. Select the most appropriate meaning of the (b) bad quality given idiom. (c) unhappy Zero tolerance (d) without hope (a) To detect someone in the group is betraying SSC CPO-SI – 24/11/2020 (Shift-II) the others Ans. (a) : For the above idiom, "X-factor" option (a) 'a (b) Riding in the front passenger seat of a car special talent or quality' gives an appropriate (c) To escape and not have to pay explanation. Other options have different meaning. (d) A policy of not allowing any violations of a 625. Select the most appropriate meaning of given rule or law idiom. SSC CHSL (Tier-I) 15/03/2023 (Shift-I) X-rated Ans. (d) : For the above idiom, 'Zero tolerance' option (a) angry and hurt (d) ''A Policy of not allowing any violations of a rule or (b) Pornographic or indecent law'' gives an appropriate explanation. Other options (c) eternal fame give different meaning. (d) very attentive SSC CHSL (Tier-I) 19/10/2020 (Shift-I) 630. Select the most appropriate meaning of given idiom. Ans. (b) : For the above idiom, "X-rated" option (b) Zip your lip 'pornographic or indecent' gives an appropriate (a) start talking (b) to stay calm explanation. Other options have different meaning. (c) a tempting offer (d) cursory look SSC CPO-SI – 12/12/2019 (Shift-I) Ans. (b) : For the above idiom, "zip your lip" option (b) 'to stay calm' gives an appropriate explanation. Other 626. Select the most appropriate meaning of the options have different meaning. given idiom. 631. Select the most appropriate meaning of given You snooze, you lose idiom. (a) If you are not alert, you are likely to miss Zonk out (a) to fall asleep (b) angry and hurt opportunities. (c) eternal fame (d) without hope (b) You may lose your money if snoozing is not SSC MTS – 22/08/2019 (Shift-I) stopped Ans. (a) : For the above idiom, "zonk out" option (a) (c) Laziness is not good 'to fall asleep' gives an appropriate explanation. Other (d) If you oversleep, you are likely to lose a job options have different meaning. SSC CGL (Tier-I) 25/07/2023 (Shift-IV) 632. Select the most appropriate meaning of given idiom. Ans. (a) : The most appropriate meaning of the above Zenith of life given idiom 'you snooze, you lose' is option (a) 'if you (a) Important for life are not a lot, you are likely to miss apportunities.' (b) happy and free Other given option are not correct. (c) highest pinnacle of someone's life (d) unhappy and regretful 627. Select the most appropriate meaning of given SSC CGL (Tier-I) – 06/06/2019 (Shift-I) idiom. Ans. (c) : For the above idiom, "zenith of life" option Yeoman's services (c) 'highest pinnacle of someone's life' gives an (a) excellent work (b) happy and free appropriate explanation. Other options have different (c) hopeful attention (d) sincere affection SSC MTS – 21/08/2019 (Shift-II) meaning.

X

Z

Y

Idoms & Phrases

713

YCT

11. READING COMPREHENSION TCS hewšve& hej DeeOeeefjle (Based On TCS Pattern) Chapterwise

Exam

Question No.

Years

1

CGL (Tier-1) CGL (Tier-2) CHSL (Tier-1) CHSL (Tier-2) Selection Post VII, VIII, XI SSC MTS SSC GD SSC CPO SI

48 22 35 15 20 35 25 50

(2017–2023)

Reading comprehension

Sec-III Trend Analysis of Questions topicwise from CGL (Pre & Mains) CHSL (Pre & Mains) Selection Post VII, VIII, XI, SSC MTS, SSC GD & Other Exams (2017-2023)

Reading Comprehension

714

YCT

11. READING COMPREHENSION Comprehension: Read the given passage and answer the questions that follow. “For me, trees have always been the most penetrating preachers. I revere them when they live in tribes and families, in forests and groves. And even more I revere them when they stand alone. They are like lonely persons. Not like hermits who have stolen away out of some weakness, but like great, solitary men, like Beethoven and Nietzsche. In their highest boughs the world rustles, their roots rest in infinity; but they do not lose themselves there, they struggle with all the force of their lives for one thing only: to fulfill themselves according to their own laws, to build up their own form, to represent themselves. Nothing is holier, nothing is more exemplary than a beautiful, strong tree. When a tree is cut down and reveals its naked death-wound to the sun, one can read its whole history in the luminous, inscribed disk of its trunk: in the rings of its years, its scars, all the struggle, all the suffering, all the sickness, all the happiness and prosperity stand truly written, the narrow years and the luxurious years, the attacks withstood, the storms endured. And every young farmboy knows that the hardest and noblest wood has the narrowest rings, that high on the mountains and in continuing danger the most indestructible, the strongest, the ideal trees grow.” 1. Which of the following labels is most appropriate for the author of the given passage? (a) Arrogant theorist (b) Social activist (c) Nostalgic dreamer (d) Environment enthusiast SSC Selection Posts XI-27/06/2023 (Shift-I) Ans. (d) : According to the sense of the above passage, the most appropriate label is option (d) 'Environment enthusiast; Other options give different meanings. 2. Where can one read the complete history of a tree? (a) On its skin when it is counted (b) On its trunk when it is cut down (c) On its branches when it is full-grown (d) On its leaves in sunny days SSC Selection Posts XI-27/06/2023 (Shift-I) Ans. (b) : According to the sense of the above passage, one can read the complete history of a tree. 'on its trunk when it is cut down; Other options give different meanings. 3. What is the tone of the speaker? (a) Sarcastic (b) Vituperative (c) Emotional (d) Apathetic SSC Selection Posts XI-27/06/2023 (Shift-I) Reading Comprehension

Ans. (c) : According to the context of the above passage, the tone of the speaker is 'emotional'. Other options give different meanings. Sarcastic – Sour, Vituperative – Sardonic, Apathetic – Indifferent, 4. Select the most suitable word given in the passage that means ‘very good and perfect’. (a) Luminous (b) Holier (c) Penetrating (d) Exemplary SSC Selection Posts XI-27/06/2023 (Shift-I) Ans. (d) : The most suitable word is option (d) 'Exemplary' which means 'very good and perfect. Other options give different meanings. Luminous – Radiant, Holier – Pure, Penetrating – Acute, 5. Where does the world rustle, according to the given passage? (a) Around the trunk of trees (b) In the deepest roots of trees (c) Over the forest in the sunny days (d) In the highest boughs of trees SSC Selection Posts XI-27/06/2023 (Shift-I) Ans. (d) : According to the sense of the above passage, the world rustle does 'in the highest boughs of trees.' Other options give different meanings. Comprehension: Read the given passage and answer the questions that follow. Satyajit Ray's films are rich in sociological elements which portray themes like faulty social structures, childhood experiences, poverty-stricken India, rural life, complexities in relationships and politics, religion, gender and on. Films like Devi or The Goddess and Mahapurush or The Godman show how religion becomes a breeding ground of superstitions and how the blind faith of people in ‘Godmen’, without analysing the reality, leave people trapped in the end. In Pather Panchali or Song of the Road, Ray shows how Apu and Durga enjoy their childhood despite their poverty. The innocence and mirth of childhood offers a sharp contrast with the disconsolate faces of other adults in the family who are affected by the present harsh situations. A critique of autocracy is found in his film Hirak Rajar Deshe or In the Country of the Diamond King which is an overall political satire. Although Satyajit Ray's films are deeply rooted in Bengali culture, they have a universal significance, and much of it is perhaps because his film making is heavily inspired by neo-realist principles.

715

YCT

6.

According to the passage, how has Mahapurush been translated? (a) Great man (b) The one man (c) Great being (d) Godman SSC Selection Posts XI-28/06/2023 (Shift-III) Ans. (d) : According to the passage, Mahapurush translated as Godman. Other options are not correct. Hence option (d) will be correct. 7. According to the passage, which of the following was a major inspiration for Satyajit Ray? (a) Comics (b) Early Bengali cinema (c) The society (d) Neo realism SSC Selection Posts XI-28/06/2023 (Shift-III) Ans. (d) : According to the passage, A major inspiration for Satyajit Ray was 'Neo realism'. Other options are not correct. Hence option (d) will be correct. 8. Which film by Satyajit Ray deals with the portrayal of childhood? (a) Devi (b) Hirak Rajar Deshe (c) Pather Panchali (d) Mahapurush SSC Selection Posts XI-28/06/2023 (Shift-III) Ans. (c) : According to the passage, 'The Pather Panchali' by Satyajit Ray deals with the portrayal of children. Other options are not correct. Hence option (c) will be correct. 9. Which of the following is NOT a theme of any of Satyajit Ray’s movies, according to the passage? (a) Rural life (b) Complexities in relationships (c) Faulty social structures (d) Motivational life stories SSC Selection Posts XI-28/06/2023 (Shift-III) Ans. (d) : According to the passage, Motivational life stories is not a theme of any of Satyajit Ray's movie. Hence, option (d) will be correct. 10. Which film by Satyajit Ray deals with the theme of autocracy? (a) Devi (b) Mahapurush (c) Hirak Rajar Deshe (d) Pather Panchali SSC Selection Posts XI-28/06/2023 (Shift-III) Ans. (c) : According to the passage, 'Hirak Rajar Deshe' is deals with the theme of autocracy by Satyajit Ray. Other options are not correct. Hence option (c) will be correct. Comprehension: Read the given passage and answer the questions that follow. New York City is on 'high alert' after reporting 100 cases of a rare inflammatory disease apparently tied to COVID-19 in children. The city recently began reporting rising cases of paediatric multi-system inflammatory syndrome, an illness officials say is similar to toxic shock or Kawasaki disease, with symptoms including a fever and abdominal pain and which 'appears to be linked to an immune response to COVID-19', The New York Times reports. Reading Comprehension

City Mayor Bill de Blasio, in a press briefing on Thursday said, "We are now on high alert in addressing the syndrome after the number of confirmed cases of paediatric multi-system inflammatory syndrome in the city has reached 100." "Among those cases, 55 children have tested positive for COVID-19 or the antibodies", de Blasio said, "and one child has died". Two Blasio had reported 82 cases of the syndrome in the city. These developments continue to be 'really troubling' after it was previously thought that "children seemed to have very little effect from this disease," do Blasio said. He urged parents to call their doctor immediately if a child had a persistent fever, a rash abdominal pain, vomiting and especially a combination of these symptoms. 11. When the City Mayor says, "We are now on high alert in addressing the syndrome", means: (a) people are consulting each other in panic (b) the administration understands it's an urgent health issue (c) it's sad for the people who are suffering (d) the city administration is concerned about war SSC Stenographer (Grade C & D) 12.11.2021 Shift-I Ans. (b) : According to the 3rd Paragraph of the sentence, option (b) ‘the administration understands it’s an urgent health issue’ would be the correct answer. 12. It can be said that children are suffering from the disease if they have the following symptoms: 1. Pain in the stomach 2. Fever that does not go 3. Toxic shock 4. Vomiting 5. Pain in the whole body (a) 1, 2 and 4 (b) 2, 3 and 5 (c) 3, 4 and 5 (d) 1, 4 and 5 SSC Stenographer (Grade C & D) 12.11.2021 Shift-I Ans. (a) : According to the passage, it can be said that children are suffering from the disease if they have pain in stomach, fever, and Vomiting. Hence option (a) will be correct. 13. The new disease is said to be a 'multi-system disorder'. This means: (a) it affects many body functions (b) it affects a large number of people (c) there are many cases (d) it requires many tests SSC Stenographer (Grade C & D) 12.11.2021 Shift-I Ans. (a) : According to the passage, ‘multi-system disorder’ means it affect many body function. Hence options (a) would be correct. 14. The word 'paediatric' in the passage refers to: (a) medical care of adults (b) stomach diseases (c) medical care of children (d) research in immunity SSC Stenographer (Grade C & D) 12.11.2021 Shift-I Ans. (c) : According to the passage, ‘Paediatric’ refers to medical care of children. Hence option (c) would be correct.

716

YCT

15.

It was previously thought that 'children seemed to have very little effect from this disease'. This disease refers to: (a) inflammation of organs (b) Covid-19 (c) Kawasaki disease (d) a multi-organ disease SSC Stenographer (Grade C & D) 12.11.2021 Shift-I Ans. (b) : According to the passage, the children seemed to have very little from the disease ‘Covid-19’. Hence option (b) would be correct. Comprehension: Read the given passage and answer the questions that follow. Fine motor skills development allows children to execute vital tasks such as feeding themselves, writing, zipping their garments, and more. Small muscles that govern the hand, fingers, and thumbs are used in fine motor activity. Fine motor abilities assist youngsters in doing crucial tasks such as self-feeding, handling objects, and writing. The capacity to do selfcare and everyday chores utilising fine motor skills promotes the development of a child's self-esteem and confidence. Fine motor skills develop systematically in children. Most new-borns have a reflexive grip at birth and begin to reach for items about three months of age. They will also begin to practise their voluntary grip and twohanded palmar grasps. At 5 months, they should be able to demonstrate a one-handed palmar grip, and at 6 months, they should be able to create a controlled reach. Most new-borns begin to reach and grab for items to put in their mouth between the ages of 6 and 12 months, and they begin to gain the capacity to regulate the release of objects they are gripping. They will also develop a pincer grasp and use it to pick objects up (thumb and one finger).They will also be able to move an object from one hand to the other and drop and pick up their toys. Most youngsters are interested in stacking building blocks between the ages of 12 and 24 months. They will also practise putting rings on a stick, putting pegs on a pegboard, and turning pages a few at a time. They can also begin to draw, paint with their entire arm movement, moving their hands and producing strokes, and feed themselves with little to no help. Toddlers at the age of two can begin to string big beads, turn single pages, cut with scissors, and handle crayons with their thumb and fingers (instead of their fist). They will also be able to use only one hand for most tasks and begin to paint with their wrists, forming dots, lines, and circular strokes. Your youngster will also be able to eat consistently and without help. Most youngsters will be able to cut along the dotted lines on paper by the age of four. They will be able to draw a cross or square shape and write their name and the numbers 1 to 5. They will be able to copy letters and will have mastered their handedness. They will also be able to dress themselves for the first time. 16. Select the most appropriate ANTONYM of the given word. Systematically (a) Evenly (b) Oddly (c) Haphazardly (d) Selectively SSC Selection Posts XI-30/06/2023 (Shift-IV) Reading Comprehension

Ans. (c) : According to the passage, Antonym of given word 'Systematically' (Respectively) is 'Haphazardly' (by chance/casually). Other options give different meaning. Evenly - Equally Oddly - Peculiarly Selectively - Choicely 17. Select the most appropriate title for the passage. (a) Importance of Fine Motor Skills (b) Systematic Development of Children (c) Development of New-borns (d) Development of Fine Motor Skills in Children SSC Selection Posts XI-30/06/2023 (Shift-IV) Ans. (d) : According to the passage, the most appropriate title for the passage is 'Development of fine motor skills in children'. Other options are not correct. Hence, Options (d) will be correct. 18. When do children learn to regulate the release of the objects that they are gripping? (a) 12-24 months (b) 6-12 months (c) 2-4 years (d) 3-6 months SSC Selection Posts XI-30/06/2023 (Shift-IV) Ans. (b) : According to sense of the passage when children in 6-12 months they learn to regulate the release of the objects that they are gripping. Other options are not correct. Hence, option (b) will be correct. 19. When do children get interested in playing with building blocks? (a) 12-24 months (b) 0-6 months (c) 2-4 years (d) 6-12 months SSC Selection Posts XI-30/06/2023 (Shift-IV) Ans. (a) : According to sense of the passage when children 12-24 months they get Interested in playing with building blocks. Other options are not correct. Hence, option (a) will be correct. 20. In light of the above passage, which of the following toys may help in the development of fine motor skills? (a) All of the given options (b) Stacks and building blocks (c) String beads (d) Crayons SSC Selection Posts XI-30/06/2023 (Shift-IV) Ans. (a) : In light of the above passage 'All of the given options (Stacks and building blocks/string beads/Crayons) 'toys may help in the development of fine motor skills.' Hence, option (a) will be correct. Comprehension: Read the given passage and answer the question that follows. Steel is tough, and it can easily be shaped, cut or made into wire. Special alloys of steel can be made by adding small amounts ofother metals such as aluminium, nickel and copper. Alloys give steel unusual hardness, toughness or ability to resist rust. Steel isoften called the backbone of modern industry. Almost everything we use is either made of iron or steel or has been made with toolsand machinery of these metals. Ships, trains, trucks

717

YCT

and autos are made largely of steel. Even the safety pins and the needles youuse are made from steel. Oil wells are drilled with steel machinery. Steel pipelines transport oil. Minerals are mined with steelequipment. Farm machines are mostly steel. Large buildings have steel framework. 21. Why is steel often called the backbone of modern industry? (a) Steel is tough. (b) Steel can be added with any material. (c) It is easy to make steel alloys. (d) Steel is used in almost every tool and machinery. SSC CGL Mains 26/10/2023 (Shift-I) Ans. (d) : According to the passage steel often called the backbone of modern Industry because steel is used in almost every toal and machinery. Hence option (d) will be correct. 22. What is the tone of the passage? (a) Informative (b) Ironical (c) Judgemental (d) Critical SSC CGL Mains 26/10/2023 (Shift-I) Ans. (a) : According to the passage, the tone of the passage is 'Informative' other options are not correct. Hence option (a) will be correct. 23. Select an appropriate title for the passage from the given options. (a) Mineral ores (b) Manufacturing (c) Steel Industry (d) Resources of Steel SSC CGL Mains 26/10/2023 (Shift-I) Ans. (c) : According to the passage, the most appropriate title is 'steel industry'. Hence option (c) will be correct. Comprehension: Read the given passage carefully and answer the question that follows. The discovery of a new bird species has sparked excitement among ornithologists and bird enthusiasts worldwide. The bird, namedthe Sapphire-winged Warbler, discovered in a remote tropical island's rainforests, is a unique bird with vibrant blue feathers and adistinctive melodic song. Birdwatchers and nature enthusiasts have been flocking to the island in hopes of catching a glimpse of thiselusive and captivating creature. The discovery of the Sapphire-winged Warbler highlights the importance of preserving and protectinghabitats. Amidst the excitement surrounding the discovery of the Sapphire-winged Warbler, researchers have embarked on a missionto uncover the bird's migratory patterns. Equipped with satellite tracking devices, they aim to trace the remarkable journey of thisspecies across vast distances. By unravelling its migratory routes and stopover locations, scientists hope to gain insight into thechallenges the Sapphire-winged Warbler faces during its arduous travels. The local community on the tropical island has embracedthe newfound avian resident with great enthusiasm. Efforts are underway to raise awareness about the importance of conservationand to create protected areas that ensure the continued existence of this exquisite species. 24. Identify the structure of the passage. (a) Sequential (b) Cause and effect (c) Classification (d) Descriptive SSC CGL Mains 26/10/2023 (Shift-I) Reading Comprehension

Ans. (d) : According to sense of the passage, the structure of the passage is 'Descriptive' other options give different meaning. Hence option (d) will be correct. 25. Based on the given passage, which of the following inferences can be made? (a) Conservation efforts are solely focused on protecting the Sapphire-winged Warbler. (b) The discovery of the Sapphire-winged Warbler has sparked global interest in birdwatching. (c) The Sapphire-winged Warbler is an aggressive and territorial bird species. (d) The scientific community has lost interest in studying avian species. SSC CGL Mains 26/10/2023 (Shift-I) Ans. (b) : According to the passage, 'The discovery of the sapphire-winged warbler has sparked global interest in bird watching' inference is made. Hence option (b) will be correct. 26. Select the most appropriate title for the given passage. (a) The Curious Behavior of Birds in Remote Islands (b) Conservation Efforts in Tropical Rainforests (c) The Migratory Journey of the Sapphirewinged Warbler (d) The Unique Features of the Sapphire-winged Warbler SSC CGL Mains 26/10/2023 (Shift-I) Ans. (c) : According to sense of the passage, the most appropriate title is 'The migratory journey of the sappire-winged warbler'. Other options are not correct. Hence option (c) will be correct. 27. Which statement best reflects a fact mentioned in the given passage? (a) The Sapphire-winged Warbler is the only bird species on the tropical island. (b) The Sapphire-winged Warbler is the largest bird species in the avian kingdom. (c) Researchers are using satellite tracking devices to study the bird's migratory patterns. (d) Schools on the island organise annual birdwatching competitions. SSC CGL Mains 26/10/2023 (Shift-I) Ans. (c) : According to the passage, statement (c) Researchers are using satellite tracking devices to study the bird's migratory patterns. Other options are not correct. Hence option (c) will be correct. Comprehension: Read the given passage and answer the question that follows. On woody plants known as vines, luscious, smoothskinned berries called grapes grow in dense clusters. In particular, grapes are grown in France, Italy, Spain, Australia, Chile, Romania, Georgia, South Africa and California, all of which have warm summers andmoderate winters. Table grapes are larger and sweeter grapes that are intended for fresh consumption. To create wine, 80% of thegrapes are crushed. The fermentation process turns grapes into wine. To make raisins, grapes must also be dried.

718

YCT

Grapes ripen in the area of California in August. They are cut from the vines and left on trays to dry for two to three weeks. So, to transform them into raisins, they are further cooked and steeped in particular treatments. According to studies, one acre of grapes can make roughly 15,000 glasses of wine on average. Grapes of the Thompson, Flame, Ruby, Perlette and Tokay varieties are popular. 28. Identify the most appropriate ANTONYM of the word ‘stare’ from the passage. (a) Glimpse (b) Excitement (c) Arduousness (d) Vibrancy SSC CGL Mains 26/10/2023 (Shift-I) Ans. (a) : According to the passage, the most appropriate antonym of the word 'stare' is 'glimpse'. Meaning of other options Excitement Stimulation Arduousness Diligence Vibrancy Shaking Thus, the correct answer is option (a). 29. Select the most appropriate title for the passage. (a) Wine making (b) Varieties of grapes (c) Grapes and its uses (d) Grapes and world SSC CGL Mains 26/10/2023 (Shift-I) Ans. (c) : The most appropriate title for the above passage is option (c) 'Grapes and its uses'. Other options are not correct. Thus, the correct answer is option (c). 30. In which region do grapes ripen in August? (a) Chile (b) California (c) France (d) Australia SSC CGL Mains 26/10/2023 (Shift-I) Ans. (b) : According to sense of the above passage, Grapes ripen in the area of California in August. Hence, option (b) will be correct. 31. Based on your reading of the passage, select the most appropriate word which best describes ‘luscious’. (a) Large (b) Juicy (c) Smooth (d) Soft SSC CGL Mains 26/10/2023 (Shift-I) Ans. (b) : According to the passage, the most appropriate meaning of the word 'luscious' is 'Juicy'. Meaning of other options Large Great Smooth Glib Soft Pulpy Thus, the correct answer is option (b). 32. What does the fermentation process do? (a) Turns grapes into raisins (b) Turns grapes into wine (c) Makes grapes larger (d) Crushes the grapes SSC CGL Mains 26/10/2023 (Shift-I) Ans. (b) : According to the sense of the above passage, the fermentation process do turns grapes into wine. Thus, the correct answer is option (b). Reading Comprehension

Comprehension : Read the passage and answer the questions given after it. The savanna landscape is typified by tall grass and short trees. It is rather misleading to call the savanna ‘tropical grassland’, because trees are always present with the luxuriant tall grass. The terms ‘parkland’ or ‘bush-veld’ perhaps describe the landscape better. Trees grow best towards the equatorial humid latitudes or along river banks but decrease in height and density away from the equator. They occur in clumps or as scattered individuals. The trees are deciduous, shedding their leaves in the cool, dry season to prevent excessive loss of water through transpiration, e.g. acacias. Others have broad trunks, with water-storing devices to survive through the prolonged drought such as baobabs and bottle trees. Trees are mostly hard, gnarled and thorny and may exude gum like gum arable. Many trees are umbrella shaped, exposing only a narrow edge to the strong winds. Palms which cannot withstand the drought are confined to the wettest areas or along rivers. Vegetative luxuriance reaches its peak in the rainy season, when trees renew their foliage and flower. In true savanna lands, the grass is tall and coarse, growing 6 to 12 feet high. The elephant grass may attain a height of even 15 feet! The grass tends to grow in compact tufts and has long roots which reach down in search of water. It appears greenish and wellnourished in the rainy season but turns yellow and dies down in the dry season that follows. The grass lies dormant throughout the long, rainless period and springs up again in the next rainy season. In between the tall grass are scattered short trees and low bushes. As the rainfall diminishes towards the deserts the savanna merges into thorny scrub. In Australia, this scrubland is particularly well represented by a number of species: mallee, mulga, spinifex grass and other bushes. The savanna, particularly in Africa, is the home of wild animals. It is known as the ‘big game country’ and thousands of animals are trapped or killed each year by people from all over the world. Some of the animals are tracked down for their skins, horns, tusks, bones or hair, others are captured alive and sent out of Africa as zoo animals, laboratory specimens or pets. There is such a wealth of animal life in Africa that many of the animal films that we see at the cinema are actually taken in the savanna. There are, in fact, two main groups of animals in the savanna, the grass-eating herbivorous animals and the fleshing-eating carnivorous animals. The herbivorous animals are often very alert and move swiftly from place to place insearch of green pastures. They are endowed with great speed to run away from the savage flesh-eaters that are always after them. The leaf and grass-eating animals include the zebra, antelope, giraffe, deer, gazelle, elephant and okapi. Many are well camouflaged species and their presence amongst the tall greenishbrown grass cannot be easily detected. The giraffe with such a long neck can locate its enemies a great distance away, while the elephant is so huge and strong that few animals will venture to come near it. It is well equipped with tusks and trunk for defence. The carnivorous

719

YCT

animals like the lion, tiger, leopard, hyena, panther, jaguar, jackal, lynx and puina have powerful jaws and teeth for attacking other animals. Their natural colorings of light yellowish-brown, often with stripes like the tiger or spots like the leopard, match perfectly with the tawny background of the savanna. They often hide themselves in shady spots up in the branches or amidst the tall bushes, and many wild animals, as well as hunters themselves, are caught unawares in this manner. 33. Why is Savanna is Africa called the "Big Game Country"? (a) There are both carnivores and herbivores. (b) Thousands of animals are hunted here. (c) Animal movies are particularly shot here. (d) It is home to many wild animals. SSC CGL (Tier-II) 08.08.2022 Shift-II Ans. (b) : According to the passage, Savanna in Africa called the ‘Big Game Country’ because thousands of animals are hunted here. Hence option (b) would be correct. 34. The main theme of the passage is : (a) Life in Savnna grassland (b) Grasses of the grassland Savanna (c) Vegetation and animal life in Savanna (d) Wealth of animal life in Africa SSC CGL (Tier-II) 08.08.2022 Shift-II Ans. (c) : According to the passage, the main theme of the passage is vegetation and animal life in Savanna. Hence option (c) would be correct. 35. What kind of a passage is it ? (a) Analytical (b) Informative (c) Narrative (d) Literary SSC CGL (Tier-II) 08.08.2022 Shift-II Ans. (b) : According to the sense, The passage is Informative. Hence option (b) would be correct. 36. Match the words with their meanings. Words - a. luxurian, b. dormant, c. prolonged Meanings - 1. continued, 2. lush, 3. sleeping (a) a-3, b-2, c-1 (b) a-2, b-1, c-3 (c) a-2, b-3, c-1 (d) a-1, b-3, c-2 SSC CGL (Tier-II) 08.08.2022 Shift-II Ans. (c) : Correct Matching for the above question is a – 2, b – 3, c – 1. Hence option (c) will be correct. 37. What helps an elephant fight its enemies ? (a) its legs and ears (b) its tusks and trunk (c) its size and strength (d) is neck and tail SSC CGL (Tier-II) 08.08.2022 Shift-II Ans. (b) : According to the passage Elephants fight its enemies with their tusks and trunk. Hence option (b) would be correct. 38. Which of the following trees has water storing capacity in is broad trunk ? (a) Acacia (b) Baobab (c) Palm (d) Gum arable SSC CGL (Tier-II) 08.08.2022 Shift-II Reading Comprehension

Ans. (b) : According to the passage, ‘Baobab tree’ has water storing capacity in its broad trunk. Hence option (b) would be correct. 39. Which species of vegetation is NOT found in Australian Savanna ? (a) mallee (b) mulga (c) elephant grass (d) spinifex grass SSC CGL (Tier-II) 08.08.2022 Shift-II Ans. (c) : According to the passage, ‘Elephant Grass’ is not found in Australian Savanna. Hence option (c) would be correct 40. Select the most appropriate meaning of the underlined words as it is used in the text. Palms which cannot withsand the drought are confined to the wettest areas or along rivers. (a) Endure (b) Convert (c) Undergo (d) Suffer SSC CGL (Tier-II) 08.08.2022 Shift-II Ans. (a) : Appropriate meaning of the underlined word ‘withstand’ is Endure. Hence option (a) would be correct. Meaning of other worlds– Convert – Modify Undergo – Incur Suffer – Sustain 41. Select the carnivorous animal from the following : (a) zebra (b) lynx (c) okapi (d) gazelle SSC CGL (Tier-II) 08.08.2022 Shift-II Ans. (b) : In the above options, ‘lynx’ is carnivorous animal. Hence option (b) would be correct. 42. What is the main feature of Savanna landscape? (a) tall grass and short trees (b) dense forests with tall trees (c) dry grass and low bushes (d) tall trees and short grass SSC CGL (Tier-II) 08.08.2022 Shift-II Ans. (a) : The main feature of Savanna landscape is tall grass and short trees. Hence option (a) would be correct. Comprehension: Read the given passage and answer the questions that follow. I was on a visit to Calcutta. It was Saturday and I was staying for the weekend at my cousin's. That evening my cousin, who works in a firm, had to fly to Delhi for a meeting. So, his wife drove him to the airport and I accompanied them. On our way back from the airport we stopped by a lake. My cousin's wife said, "Let me show you where we spend some of our evenings." She locked the car and we began to walk towards the lake. There was a small bridge to cross and she led the way. Three young men were coming from the other end of the bridge. As they approached us, one of them said to me, "Can I have a match?" As I put my hand into my pocket for the match box, I heard a cry of alarm. And then I saw what was happening.

720

YCT

One of the other two men had snatched the handbag from my cousin's wife and passed it on the man next to him. He in turn passed it to the third man and all three started running back to where they had come from. My cousin's wife kept shouting and hurling abuses at them but made no attempt to follow them to retrieve her bag. I wanted to chase them but she stopped me saying that such rogues could be armed and dangerous. As it was dark, the three men were soon out of sight. I felt sorry for not being of any help but my cousin's wife assumed me that there was no money or valuables in the bag. She had just kept a couples of apples and a packet of biscuits in the handbag in case I felt hungry. 43. One of the three men asked narrator for a match to: (a) burn the handbag (b) smoke a cigarette (c) stop and divert the narrator's attention (d) pass the bag to his accomplice SSC Stenographer (Grade C & D) 11.11.2021 Shift-II Ans. (c) : According to the passage one of the three men asked narrator for a match to stop and divert the narrator’s attention. Hence option (c) would be correct. 44. After dropping the narrator's cousin, his cousin's wife took him to a lake to: (a) show him the place where she and her husband spent their evenings (b) enjoy the cool breeze in the evening (c) have a picnic by the lakeside (d) have an evening walk on the bridge SSC Stenographer (Grade C & D) 11.11.2021 Shift-II Ans. (a) : According to the passage option (a) would be the correct answer for the above question. 45. Why did the narrator's counsin's wife stop him from chasing the thieves? (a) She wanted to retrieve the bag herself. (b) He couldn't follow them in the dark. (c) She feared they could harm the narrator. (d) There were no valuables in the handbag. SSC Stenographer (Grade C & D) 11.11.2021 Shift-II Ans. (c) : The narrators cousin’s stop him from chasing the thieves because. She feared they could harm the narrator. Hence option (c) is correct. 46. Which of the following statements is not true? (a) There was no real loss in the incident. (b) The handbag contained apples and biscuits. (c) The narrator drove his cousin to the airport. (d) The narrator's cousin lived in Calcutta. SSC Stenographer (Grade C & D) 11.11.2021 Shift-II Ans. (c) : According the passage, option (c) is not correct statement. Other statements are true. 47. How long was the narrator going to stay at his cousin's place? (a) One week (b) Two weeks (c) One day (d) Two days SSC Stenographer (Grade C & D) 11.11.2021 Shift-II

Comprehension: Read the given passage and answer the questions that follow. Difficult times could create feelings of anxiety, insecurity and fear. As we navigate through undesirable situations and uncertainities, we find that negativities surrounding the problems consume our mind. It is in those times that we have to exert ourselves to change our perspective and adopt a more positive and solutiondriven attitude. Wise people have always chosen to leverage adversities as opportunities for growth and progress, rather than succumb to negative thought patterns. Saint Kabir described how different people respond when challenged by problems: "Gold, virtuous people and saints are resilient; you may break them a hundred times and yet they will rejoin. In contrast, negative people and clay pots are fragile; one shattered, they are broken forever." The distinguishing feature of the successful is that they have a mind-set that enables them to remain positive in the face of adversity and even utilise the negative situation for their benefit. The eagle can teach us a few lessons on this. When the storm starts blowing, it does not cower down before it. It flies to a higher point and waits for the wind to blow over. When the storm sets in, the eagle spreads its wings and use the draft of the wind to rise higher and higher in the sky. It does not run away from the storm, rather it uses the storm to its advantage. While the storm rages below, the eagle soars above it. The winning attitude to overcome despair and stress is to realise that adversities have opportunities inherent in them and opportunities do not come without problems. Hence, difficulties and obstacles have a positive side to them as they make us grow from within. A gem gets polished with friction. The finest steel is produced by putting it in fire. Similarly, the more hardships we face, the stronger we become. 48. Which of the following is an ideal way of handling a difficult situation? (a) Moving away from difficulties (b) Making the best use of a situation to your advantage (c) Becoming overcome by stress and negativity (d) Thinking deeply about the impact of adversities SSC Stenographer (Grade C & D) 11.11.2021 Shift-I Ans. (b) : According to the passage, Making the best use of a situation to your advantage’ is an ideal way of handling a difficult situation 49. Saint Kabir believed that saintly people: (a) can be broken easily (b) are not willing to suffer a hundred times (c) can be overcome by fear easily (d) can regain their strength easily SSC Stenographer (Grade C & D) 11.11.2021 Shift-I Ans. (d) : According to the passage, Saint Kabir Ans. (b) : According to the passage, the narrator going believed that saintly people can regain their strength to stay at his cousin’s place for two weeks. Hence easily. Hence option (d) is correct. option (b) is correct.

Reading Comprehension

721

YCT

50.

When there is a storm, an eagle ............. (a) waits for it to end, then starts flying (b) uses it to fly away to safety (c) gets scared and hides to save itself (d) flies high above it, using its force SSC Stenographer (Grade C & D) 11.11.2021 Shift-I Ans. (d) : When there is a storm, an eagle flies high above it, using its force. Hence option (d) is correct. 51. Which of the following is not a message that the passage gives us? (a) Those who are positive are fragile and can be easily shattered by difficulties. (b) We have to change our view point and adopt a more positive and solution-driven attitude towards difficulties. (c) Difficulties enable us to become stronger as we learn to face them. (d) Adversities have opportunities as well as obstacles inherent in them. SSC Stenographer (Grade C & D) 11.11.2021 Shift-I Ans. (a) : According to the passage, option (a) is not a message that the passage gives us. Hence option (a) is correct. 52. As per the passage, when we face difficult times: (a) we look at them as opportunities for improvement (b) our minds get overcome by negative thoughts (c) we start changing our views on life (d) we become passively engaged in overcoming them SSC Stenographer (Grade C & D) 11.11.2021 Shift-I Ans. (b) : As per the passage – When we face difficult times our minds get overcome by negative thoughts. Hence option (b) is correct. Read the given passage and answer the questions that follow. 'What's your name, boy?' said the gentleman in the high chair. Oliver was frightened at the sight of so many gentlemen, which made him tremble and speak in a very low and hesitating voice. 'Listen to me. You know you're an orphan, I suppose? And that you were brought up by the parish, don't you?' 'Yes sir,' replied Oliver, weeping bitterly. 'I hope you say your prayers every night,' said another gentleman in a gruff voice, 'and pray for the people who feed you, and take care of you'. 'Yes, sir', stammered the boy. 'Well! You have come here to be educated, and taught a useful trade,' said the red-faced gentleman in the high chair. Poor Oliver! He little thought, as he lay sleeping, happily unconscious of things around him, that the board had that very day arrived at a decision which would exercise the most material influence over all his future fortunes. Reading Comprehension

But they had. They believed the workhouse was a regular place of public entertainment for the poorer classes; a tavern where there was nothing to pay; a public breakfast, dinner, tea and supper all the year round; a brick and mortar place, where it was all play and no work. 'Oho!' said the board, looking very knowing; 'we are the fellows to set this to rights; we'll stop it all, in no time.' So, they established the rule, that all poor people should be starved by a gradual process in the house. The water-works were asked to lay on an unlimited supply of water; a com-factory asked to supply small quantities of oatmeal; and it was decided to issue three meals of thin gruel a day, with an onion twice a week, and half a roll of bread on Sundays. 53. The changes to be brought about for the inhabitants at the workhouse were that they would be given: 1. thin gruel thrice a day 2. healthy breakfast, lunch and dinner 3. plenty of water to drink 4. free entertainment and fun activities 5. half a roll of bread on Sundays 6. plenty of food from a corn factory (a) a, c and d (b) b, e and f (c) a, c and e (d) c, d and e SSC Stenographer (Grade C & D) 12.11.2021 Shift-II Ans. (c) : According to the passage, The changes to be brought about for the inhabitants at the workhouse were that they would be given thin gruel thrice a day, plenty of water to drink thin gruel thrice a day, plenty of water to drink and half a roll of bread on sundays, which is given in option (c) as it was mentioned in the last stanza of the passage. 54. What was Oliver's situation? (a) He was enjoying a stay at a workhouse (b) He had no family and was living in a home for orphans (c) His education would prepare him for the future (d) He was fortunate to be provided with good meals SSC Stenographer (Grade C & D) 12.11.2021 Shift-II Ans. (b) : According to the 3rd line of the passage, Oliver had no family and was living in a home for orphans. Hence option (b) will be correct. 55. How was Oliver feeling? (a) Unconcerned (b) Happy (c) Frightened (d) Bored SSC Stenographer (Grade C & D) 12.11.2021 Shift-II Ans. (c) : According to the 2nd line of the passage, Oliver was feeling frightened. Hence option (c) will be correct. 56. What was the decision taken by the gentlemen? (a) To slowly starve the boys to death (b) To give the boys a happy home (c) To improve things so the children remained ever grateful (d) To give food, education and care to the boys SSC Stenographer (Grade C & D) 12.11.2021 Shift-II

722

YCT

Ans. (a) : The decision taken by the gentlemen was to slowly starve the boys to death as it was mentioned in the last stanza of the passage. Hence option (a) will be correct. 57. The impression one forms of the gentlemen is that they: (a) followed laid down rules (b) were very thoughtful (c) were kind human beings (d) were cruel and uncaring SSC Stenographer (Grade C & D) 12.11.2021 Shift-II Ans. (d) : According to the passage it could be understood that the gentlemen were cruel and uncaring. Hence option (d) will be correct. Comprehension: Read the given passage carefully and answer the questions that follow. If you pay close attention to soap advertisements, you may hear of its pH value being claimed perfect for human skin. But is there really such a thing? Let's start at the beginning. pH (potential Hydrogen) is defined as the concentration of hydrogen ions in a solution. pH value ranges between 0 and 14.7 is the neutral point, 0 being the most acidic and 14 being the most alkaline. More importantly, your skin isn't exactly pH 5.5; it falls in a range between 4.0 and 7.0, depending on diverse factors like the body part, age, gentics, ethnicity and environment conditions. So, are products formulated at pH 5.5 perfect for skin? The short answer: not really! First, parameters like texture and other ingredients indicate a cleanser's quality, much better than pH alone. Second, though the skin pH rises slightly immediately after cleaning even with plain water, it reverts to its mild acidic pH in an hour. Healthy skin quickly rebalances the 'acid mantle'–a protective layer over the skin – and is unaffected in the long term by the cleanser's pH. Skin modulates pH, making skin products function optimally. So, why market pH 5.5 products as "perfect"? Well, for certain skin types (e.g., oily skin) and certain skin conditions (like acne), an increase in pH can aggravate these skin situations. This might lead to an interpretation that a skin care product needs to be at a 5.5 pH for optimum cleansing. Hence, skincare experts have expressed reservations about pH being the sole criterion of product safety and 'acid mantle' preservation when factor including plain water may contribute to the same. Thus, an ideal product is almost impossible to define. So, look well beyond pH alone. 58. "Is there really such a thing?" By "such a thing" the author is referring to: (a) the perfect pH value for human skin (b) soap advertisements (c) close attention paid to advertisements (d) the perfect soap for all skin types SSC Stenographer (Grade C & D) 15.11.2021 Shift-II Ans. (a) : According to the passage 'the author is referring to the perfect pH value for human skin'. PH is defined as the concentration of hydrogen ions in a solution pH value range between 0 and 14. Reading Comprehension

59.

According to the passage, the pH value of human skin is generally: (a) 5.5 (b) between 0 and 7 (c) between 7 and 14 (d) between 4 and 7 SSC Stenographer (Grade C & D) 15.11.2021 Shift-II Ans. (d) : According to the passage, the pH value of human skin is generally between 4.0 and 7.0. the pH value falls in a range between 4.0 and 7.0, depending on diverse factors like the body part, age, genetics, ethnicity and environment conditions. 60. "Let's start at the beginning." By this, the author means that: (a) we should start discussing soap advertisements (b) we should look at the characteristics of a good soap (c) we should start with the basic facts before going into the issue (d) we should first answer the question "Is there such a thing?" SSC Stenographer (Grade C & D) 15.11.2021 Shift-II Ans. (c) : "Let's start at the beginning." By this, the author means that we should start with the basic fact before going into the issue. 61. According to the passage, the 'acid mantle' refers to: (a) an acidic skin product with the right pH (b) a layer over the skin that is balanced by healthy skin in terms of its pH (c) an acidic cleansing agent with a pH of 5.5 (d) a layer over the skin that keeps the skin pH at 5.5 SSC Stenographer (Grade C & D) 15.11.2021 Shift-II Ans. (b) : According to the passage. Healthy skin quickly rebalances the 'acid mantle-a protective layer over the skin-and is unaffected in the long term by the cleanser's pH. 62. Which of the following is not true according to the passage? (a) There is no perfect pH for the human skin (b) An increase in pH can aggravate certain skin conditions (c) Human skin adjusts the pH so that a cleansing agent functions optimally. (d) The best way to judge the safety of a skin care product is by its pH. SSC Stenographer (Grade C & D) 15.11.2021 Shift-II Ans. (c) : According to the passage. Human skin adjusts the pH so that a cleaning agent functions optimally is not true. Comprehension: Read the given passage carefully and answer the questions that follow. Indian mathematical Nikhil Srivastava, working at the University of California in Berkeley, is among the winners of the Prestigious 2021 Michael and Sheila Held Prize, announced last week by the US National Academy of Science (NAS). Adam W Marcus, EPFL (Swiss Federal Institute of Technology, Lausanne) and Daniel Alan Spielman, Yale University, are the other two winners.

723

YCT

"Marcus, Spielman and Srivastava solved longstanding questions on the Kadison-Singer problem and on Ramanujan graphs, and in the process uncovered a deep new connection between linear algebra, geometry of polynomials, and graph theory that has inspired the next generation of theoretical computer scientists," the NAS said in a statement. An Indian national, Nikhil was born in New Delhi in November 1983 and has attended educational institutions across the world–Syria, UK, Saudi Arabia and US – as his father was an Indian Foreign Services officer, who has served as the Indian ambassador to Uganda and Denmark. At present, Nikhil is an associate professor of mathematics at UC Berkeley. The Michael and Sheila Held Prize is presented annually to honour outstanding, innovative, creative and influential research in the area of combinatorial mathematics. 63. Which of the following is not true of Nikhil Srivastava according to the passage? (a) He is now a faculty at the University of California. (b) He was born in North India in the early 1980s. (c) He has served as the Indian ambassador to Denmark and Uganda. (d) He had his education in many countries around the world. SSC Stenographer (Grade C & D) 15.11.2021 Shift-I Ans. (c) : In the above option, option (c) is not true of Nikhil Srivastava according to the passage. 64. How many mathematicians won the Michael Sheila Prize for 2021? (a) Two (b) Three (c) Four (d) One SSC Stenographer (Grade C & D) 15.11.2021 Shift-I Ans. (b) : According to the passage, Three mathematics won the Michael Sheila Prize for 2021. Hence option (b) would be correct. 65. The Michael and Sheila Held prize is presented: (a) every two years to honor the best researchers in mathematics. (b) every year to honor new thinking and research in a specific area of mathematics. (c) every year to honor those who solve longstanding problems in graph theory and linear algebra (d) twice every year to honor young and influential researchers in mathematics. SSC Stenographer (Grade C & D) 15.11.2021 Shift-I Ans. (b) : According to the passage, The Michel and Sheila held Prize is presented every year to honor new thinking and research in a specific area of mathematics. 66. In what way have the prize winners helped the future theoretical computer scientists? (a) They have done innovative research on Ramanujan graphs and geometry of polynomials. (b) The new connection they have discovered has inspired future theoretical computer scientists. (c) They solved new problems in graph theory used in theoretical computer science. (d) The have found connections between algebra, geometry and computer science. SSC Stenographer (Grade C & D) 15.11.2021 Shift-I Reading Comprehension

Ans. (b) : According to the passage, option (b) will be the correct answer for the above question. 67. What is the connection between the prize winners and the famous Indian mathematical genius Ramanujan? (a) They solved questions on Ramanujan's graphs that had not been solved for a long time. (b) The prize winners were all students of Ramanujan. (c) The prize winners were all admirers of Ramanujan's great contribution. (d) They explained Ramanujan's graphs in solving the Kadison-Singer problem. SSC Stenographer (Grade C & D) 15.11.2021 Shift-I Ans. (a) : According to the passage, Prize winners solved questions on Ramanujan’s graphs that had not been solved for a long time. Comprehension: Read the given passage carefully and answer the questions that follow. When consuming your favourite fruit is forbidden by your doctor or dietician, it is time to find out why rather than trying to have more of it on the sly. Conventional diet advice for weight watchers and diabetics puts mangoes in the red list, and not without reason. Keeping scientific information about mangoes in mind, recommendations about the fruit warrant a relook. This is because of its impact on body weight, blood sugar and health. It's true that ripe mangoes are high in carbohydrates, sugars and have a high glycogenic index. One of the seventh most popular fruits worldwide, it provides roughly 80 kilocalories per 100 g (a medium-sized ripe mango would carry about 130-150 calories), which mainly come from carbohydrates (20 per cent). Due to high sugar content, mangoes are high in calories. But does this mean that diabetics and weight watchers should be banned from having it? The answer is no, if one adjusts for calories, i.e. substitutes it with an equivalent amount of calories and carbohydrates in the meal. Better still, combine it with a low-glycogenic food. Low glycaemic foods include pulses, legumes, low-fat dairy (milk, curd), vegetables, nuts and seeds. So mangoes can make a safe entry into the diet, if one is able to keep the caloric intake and carbohydrates constant, i.e. take the right amounts. With over 2,500 varieties around the world, the popularity of mangoes clearly lies in its aromatic sweetness. This tropical fruit also can boast of a huge array of impressive health benefits. The mango is known to be an excellent source of many vitamins such as vitamin C (ascorbic acid), B vitamins (thiamine, riboflavin and niacin), and Vitamin A (βcarotene). Ripe mangoes generally possess a higher number of phenolics, researchers say. This makes mangoes a good source of antioxidants with anti-cancer and anti-inflammatory properties. With reasonably good fibre content, it provides a good combination of soluble and insoluble fibre. Soluble fibre is good for digestion and its insoluble variant is good for management of blood cholesterol and sugar levels.

724

YCT

But before you start planning a mango party, there is need for caution. Remember that mangoes, being high on sugars, predominantly fructose, should be consumed in appropriate portions. This is true particularly for weight watchers, diabetics and those with high cholesterol levels. 68. The main idea of the passage is that: (a) diabetics should not eat mangoes (b) mangoes should be eaten in appropriate proportion (c) mango is a nutritious fruit (d) mango is one of the most popular fruits SSC Stenographer (Grade C & D) 15.11.2021 Shift-I Ans. (b) : According to the passage, The main idea of the passage is that mangoes should be eaten in appropriate proportion. 69. What advice is given in the passage to the diabetics and weight watchers? (a) Eat mangoes but combine it with a lowglycaemic food. (b) Do not eat mangoes as it is too high in sugar. (c) Eat plenty of mangoes as it has impressive health benefits. (d) Eat mangoes but balance the intake of vitamins SSC Stenographer (Grade C & D) 15.11.2021 Shift-I Ans. (a) : According to the passage, ‘Eat mangoes but combine it with a low glycogenic food’ is the advice to the diabetics and weight watchers. 70. Ripe mangoes have a higher number of phenolics which make them: (a) a high-glycaemic food (b) a good source of antioxidants (c) highly rich in fructose (d) highly nutritious SSC Stenographer (Grade C & D) 15.11.2021 Shift-I Ans. (b) : According to the passage, Ripe mangoes have a higher number of phenolics which make them a good source of antioxidants. Hence option (b) is correct. 71. Mangoes are high in calories. What is its main source of calories? (a) Carbohydrates (b) Fibre (c) Antioxidants (d) Vitamins SSC Stenographer (Grade C & D) 15.11.2021 Shift-I Ans. (a) : According to the passage, mangoes are high in calories its main source of calories is carbohydrates. Hence option (a) is correct. 72. Which of the following nutrients is not present in mangoes? (a) Sugar (b) Vitamins (c) Carbohydrates (d) Fats SSC Stenographer (Grade C & D) 15.11.2021 Shift-I Ans. (d) : According to the passage, fats are not present in mangoes. Hence option (d) would be correct. Comprehension: Read the given passage carefully and answer the questions that follow. Rajasthan's farmers have been banging plates and lighting fires these days. It is their desperate attempt to scare away the swarms of migratory locusts invading their fields and eating away crops. The pest attack has come as a double blow for farmers already reeling under the impact of the Covid lockdown. Reading Comprehension

Millions of locusts, covering 1-4 sq km of the skyline, have reached 10 of the state's 33 districts within three weeks of being first spotted on April 30. Last year, yellow locusts had entered India from Pakistan on the second half of May. The attack had originated from Yemen, coming all the way to West Asia and Pakistan and then India, creating havoc in 17 countries en route. With the harvest nearly over in Rajasthan, the invading population of grasshoppers this May is feeding mainly on sprouting cotton crop sown last month, vegetables and fodder. State officials, though, fear damage to the young millet and moong dal crops due in July and August if the locusts breed. Flying in through the Pakistan border near Jaisalmer, the locust swarms have infiltrated as deep as Kishangarh in Ajmer district. Locusts can cover 200-250 km in a day if they get wind support. Presently, the dust storms sweeping parts of Rajasthan are aiding their flight. Officials said the Union agriculture ministry's Locusts Warning Organization (LWO) is in the process of acquiring 60 vehicles with mounted insecticide sprayers from abroad by July. Ten such vehicles were purchased this January. "Immature pink locusts are very active, which is why they have reached the interiors of Rajasthan (from Pakistan) so quickli," says K.L. Gurjar, deputy director, LWO. Their metabolism slows down during winters, but in summers, they can fly from 4 am to 8 pm, covering long distances and consuming more crops along the way than mature locusts would. The LWO has put 10 insecticides-spraying vehicles on the job in Rajasthan. The Rajasthan government has provided rented tractors, on which insecticide sprayers are mounted, and also the manpower to man them. The Center-state joint operation has so far cleared a few hundred sq km of fields of the locusts. Preliminary estimates put the loss to crops at a few crore rupees. State officials say there is an urgent need to contain the pink locusts before they start breeding in the next few weeks. Indiscriminate aerial spraying of insecticides is illadvised as this can damage crops as well as affect human settlements. One of the strategies being considered now is to use fixedwing aircraft for ultra-low volume spraying of chemicals. The state government may request for such aerial sorties by the air force. Drones have limited capabilities as they can carry only about 20 litres of insecticide at a time, making them effective only over small areas. 73. It can be inferred from the passage that the present swarm of locusts consists of: (a) yellow mature locusts (b) yellow immature locusts (c) pink mature locusts (d) pink immature locusts SSC Stenographer (Grade C & D) 15.11.2021 Shift-I Ans. (d) : According to the passage, the present swarm of locusts consists of pink immature locusts. Hence option (d) would be correct. 74. The main theme of the passage is: (a) breeding of locusts (b) invasion of locusts in Rajasthan (c) measures for dealing with the locusts (d) the damage done by the locusts SSC Stenographer (Grade C & D) 15.11.2021 Shift-I

725

YCT

Ans. (b) : The main theme of the passage is invasion of locusts in Rajasthan. Hence option (b) would be correct. 75. If the locusts breed, which crops will they damage? (a) Vegetables (b) Cotton crop (c) Millet and moong (d) Fodder SSC Stenographer (Grade C & D) 15.11.2021 Shift-I Ans. (c) : According to the passage, if the locusts bread it will damage millet and moong. Hence option (c) is correct. 76. Which of the following measures the government of Rajasthan wants to procure for containing locusts? (a) Drones for spraying small areas with insecticides (b) Insecticide-spraying vehicles (c) Fixed-wing aircraft for spraying of chemicals (d) Rented tractors mounted with insecticide sprayers SSC Stenographer (Grade C & D) 15.11.2021 Shift-I Ans. (c) : Fixed– wing aircraft for spraying of chemicals measures the government of Rajasthan wants to procure for containing locusts. Hence option (c) is correct. 77. After reading this passage it can be said that it is: (a) an epic (b) a government statement (c) a news report (d) a survey analysis SSC Stenographer (Grade C & D) 15.11.2021 Shift-I Ans. (c) : After reading this passage it can be said that it is a news report. Hence option (c) would be correct. Comprehension: Read the given passage and answer the questions that follow. Plato is the earliest important educational thinker, and education is an essential element in ‘The Republic’ (his most important work on philosophy and political theory, written around 360B.C.). In it, he advocates some rather extreme methods: removing children from their mothers' care and raising them as wards of the state, and differentiating children suitable to the various castes, the highest receiving the most education, so that they could act as guardians of the city and care for the less able. He believed that education should be holistic, including facts, skills, physical discipline, music and art. Plato believed that talent and intelligence is not distributed genetically and thus is be found in children born to all classes, although his proposed system of selective public education for an educated minority of the population does not really follow a democratic model. Aristotle considered human nature, habit and reason to be equally important forces to be cultivated in education, the ultimate aim of which should be to produce good and virtuous citizens. He proposed that teachers lead their students systematically, and that repetition be used as a key tool to develop good habits, unlike Socrates' emphasis on questioning his listeners to bring out their own ideas. He emphasized the balancing of the theoretical and practical aspects of subjects taught, among which he explicitly mentions reading, writing, mathematics, music, physical education, literature, history, and a wide range of sciences, as well as play, which he also considered importAntonym Reading Comprehension

78.

Which of these methods is NOT advocated in 'The Republic'? (a) Imparting similar education to all children (b) Bringing up children under state guardianship (c) Keeping children away from mothers (d) Differentiating children based on castes SSC CGL (Tier-II) 16/11/2020 (3 pm - 5 pm) Ans. (a) : According to the passage option (a) "Imparting similar education to all children" is not advocated in ''The Republic''. Hence option (a) is the correct answer. 79. Which of these statements is NOT true? (a) Socrates encouraged the listeners to come up with original ideas (b) Socrates considered music and physical education as important aspects of learning (c) The Republic was written around 360 A.D (d) Plato's methods of education can be called 'extreme'. SSC CGL (Tier-II) 16/11/2020 (3 pm - 5 pm) Ans. (c) : According to the passage "The Republic was written around 360 A.D" is not True. Hence option (c) is the correct answer. 80. Aristotle believed that virtuous citizens could be produced by cultivating : (a) habit and reason (b) theoretical aspects of education (c) art and music (d) mathematics and science SSC CGL (Tier-II) 16/11/2020 (3 pm - 5 pm) Ans. (a) : According to the passage 'habit and reason' could be produced good and virtuous citizen. Hence option (a) will be correct answer. 81. Children who are imparted highest education, would be responsible for : (a) teaching the illiterate (b) developing talent and skills (c) inculcating good habits (d) guarding the city SSC CGL (Tier-II) 16/11/2020 (3 pm - 5 pm) Ans. (d) : According to the passage children who are imparted highest education would be responsible for "They could act as gurdians of the city and care for the less able". Hence option (d) will be correct answer. 82. What tool does Aristotle advocate to teachers to develop good habits in students? (a) Repetition (b) Questioning (c) Reading (d) Writing SSC CGL (Tier-II) 16/11/2020 (3 pm - 5 pm) Ans. (a) : According to the passage "repetition" a key tool to develop good habits in students. Hence option (a) will be correct answer. Comprehension: Read the given passage and answer the questions that follow. Chanhudaro was a tiny settlement (less than 7 hectares) as compared to Mohenjodaro (125 hectares), almost exclusively devoted to craft production, including beadmaking, shell-cutting, metal-working, seal-making and weight-making. The variety of materials used to make

726

YCT

beads is remarkable: stones like carnelian (of a beautiful red colour), jasper, crystal, quartz and steatite; metals like copper, bronze and gold; and shell, faience and terracotta or burnt clay. Some beads were made of two or more stones, cemented together, some of stone with gold caps. The shapes were numerous – disc shaped, cylindrical, spherical, barrel-shaped, segmented. Some were decorated by incising or painting, and some had designs etched onto them. Techniques for making beads differed according to the material. Steatite, a very soft stone, was easily worked. Some beads were moulded out of a paste made with steatite powder. This permitted making a variety of shapes, unlike the geometrical forms made out of harder stones. How the steatite micro bead was made remains a puzzle for archaeologists studying ancient technology. Archaeologists’ experiments have revealed that the red colour of carnelian was obtained by firing the yellowish raw material and beads at various stages of production. Nodules were chipped into rough shapes, and then finely flaked into the final form. Grinding, polishing and finally drilling completed the process. Specialised drills have been found at Chanhudaro, Lothal and more recently at Dholavira. Nageshwar and Balakot, both settlements are near the coast. These were specialised centres for making shell objects –including bangles, ladles and in lay – which were taken to other settlements. Similarly, it is likely that finished products (such as beads) from Chanhudaro and Lothal were taken to the large urban centres such as Mohenjodaro and Harappa. 83. Which is the final process in the production of beads? (a) Polishing (b) Grinding (c) Chipping (d) Drilling SSC CGL (Tier-II) 16/11/2020 (3 pm - 5 pm) Ans. (d) : According to the passage option (d) "Drilling" is the final process of beads. The meaning of other option is different. 84. Which of these stones is very soft and easy to mould? (a) Quartz (b) Steatite (c) Crystal (d) Jasper SSC CGL (Tier-II) 16/11/2020 (3 pm - 5 pm) Ans. (b) : According to the passage option (b) "steatite" a stone is very soft and easy to mould. Other options are not suitable. 85. According to the text, which of these crafts was Not practised in Chanhudaro? (a) Paper making (b) Metal cutting (c) Weight making (d) Seal making SSC CGL (Tier-II) 16/11/2020 (3 pm - 5 pm) Ans. (a) : According to the passage option (a) "Paper making" crafts was not practised in chanhudaro. Other options are not suitable. 86. Shell objects were mostly made at Nageshwar and Balakot because : (a) specialised drills were found here (b) shell were transported here from Lothal (c) shell were readily available in these coastal areas (d) the women loved to wear bangles made of shells SSC CGL (Tier-II) 16/11/2020 (3 pm - 5 pm) Reading Comprehension

Ans. (c) : According to the passage shell objects were mostly made at Nageshwar and Balakot because shells were readily available in these coastal areas. Hence option (c) will be correct answer. 87. Which of these statements is NOT true? (a) Harappa and Mohenjodaro were large urban centres (b) Chanhudaro was exclusively devoted to craft production (c) Beads were made in numerous shapes and sizes (d) Camelian is a beautiful bluish stone SSC CGL (Tier-II) 16/11/2020 (3 pm - 5 pm) Ans. (d) : According to the passage option (d) "Camelian is a beautiful bluish stone" is not true. Hence option (d) is the correct answer. Comprehension: Read the given passage and answer the questions that follow. Her name was Sulekha, but since her childhood everyone had been calling her Bholi, the simpleton. She was the fourth daughter of Ramlal. When she was ten months old, she had fallen off the cot on her head and perhaps it had damaged some part of her brain. That was why she remained a backward child and came to be known as Bholi, the simpleton. At birth, the child was very fair and pretty. But when she was two years old, she had an attack of small-pox. Only the eyes were saved, but the entire body was permanently disfigured by deep black pockmarks. Little Sulekha could not speak till she was five as she was a slow learner, and when at last she learnt to speak, she stammered. The other children often made fun and mimicked her. As a result, she talked very little. Ramlal had seven children — three sons and four daughters, and the youngest of them was Bholi. It was a prosperous farmer’s household and there was plenty to eat and drink. All the children except Bholi were healthy and strong. The sons had been sent to the city to study in schools and later in colleges. Of the daughters, Radha, the eldest, had already been married. The second daughter Mangla’s marriage had also been settled, and when that was done, Ramlal would think of the third, Champa. They were good-looking, healthy girls, and it was not difficult to find bridegrooms for them. But Ramlal was worried about Bholi. She had neither good looks nor intelligence. From her very childhood Bholi was neglected at home. She was seven years old when Mangla was married. The same year a primary school for girls was opened in their village. The Tehsildar sahib came to perform its opening ceremony. He said to Ramlal, “As a revenue official you are the representative of the government in the village and so you must set an example to the villagers. You must send your daughters to school.” That night when Ramlal consulted his wife, she cried, “Are you crazy? If girls go to school, who will marry them?” But Ramlal had not the courage to disobey the Tehsildar. At last his wife said, “I will tell you what to do. Send Bholi to school. As it is, there is little chance of her getting married, with her ugly face and lack of sense. Let the teachers at school worry about her.”

727

YCT

88.

'Backward child' in the passage means : (a) belonging to an underprivileged community (b) physically challenged (c) belonging to a poor family (d) mentally challenged SSC CGL (Tier-II) 16/11/2020 (3 pm - 5 pm) Ans. (d) : According to the passage Backward child means is "mentally challenged" Hence option (d) will be correct answer. 89. Bholi's mother agreed to send her to school because : (a) she wanted to wash her hands off Bholi (b) Bholi was neglected at home (c) she wanted to educate Bholi (d) she cared for Bholi's well-being SSC CGL (Tier-II) 16/11/2020 (3 pm - 5 pm) Ans. (a) : According to the passage Bholi's mother agreed to send her to school because "She wanted to wash her hands off Bholi." Hence option (a) will be correct answer. 90. Why was Ramlal worried about Bholi? (a) It would be difficult to arrange her marriage (b) She was too old to get married (c) It was difficult to comprehend her speech (d) She was not willing to get educated SSC CGL (Tier-II) 16/11/2020 (3 pm - 5 pm) Ans. (a) : According to the passages Ramlal worried about Bholi because "It would be difficult to arrange her marriage". Hence option (a) will be correct answer. 91. Which of these statements is NOT true about Bholi? (a) She was a simpleton (b) She was neglected by her family (c) She was seven when her eldest sister got married (d) She was healthy and strong SSC CGL (Tier-II) 16/11/2020 (3 pm - 5 pm) Ans. (d) : According to the passage "She was healthy and strong" is NOT true about Bholi. Hence option (d) will be correct answer. 92. Sulekha came to be called Bholi because she was: (a) fair and pretty (b) not very intelligent (c) healthy and strong (d) the youngest daughter SSC CGL (Tier-II) 16/11/2020 (3 pm - 5 pm) Ans. (b) : According to the passage Sulekha came to be called Bholi because she was "not very intelligent." Hence option (b) will be correct answer. 93. The word 'disfigured' suggests that Bholi's looks were : (a) preserved (b) improved (c) impaired (d) enhanced SSC CGL (Tier-II) 16/11/2020 (3 pm - 5 pm) Ans. (c) : According to the passage option (c) disfigured - (Deformed) suggest that Bholi were impaired (perverted). Reading Comprehension

94.

At what age did Sulekha damage her brain? (a) Five years (b) Seven years (c) Two years (d) Ten months SSC CGL (Tier-II) 16/11/2020 (3 pm - 5 pm) Ans. (d) : According to the passage "Sulekha damage" her brain in age of Ten month. Hence option (d) will be correct answer. 95. How was Ramlal expected to set an example for the villagers? (a) By sending his daughters to school (b) By marrying off his daughters at an early age (c) By sending his sons to school (d) By treating Bholi as an equal SSC CGL (Tier-II) 16/11/2020 (3 pm - 5 pm) Ans. (a) : According to the passage "By sending his daughters to school" Ramlal expected to set an example for the villagers. Hence option (a) will be correct answer. 96. What did the small pox attack do to Sulekha? (a) It made her dull (b) It made her look ugly (c) It damaged her speech (d) It damaged her eyes SSC CGL (Tier-II) 16/11/2020 (3 pm - 5 pm) Ans. (b) : According to the passage "The small pox attack sulekha made her look ugly." Hence option (b) will be correct answer. 97. Who was invited to inaugurate the girls' school? (a) Village head (b) Revenue official (c) School Headmaster (d) Tehsildar SSC CGL (Tier-II) 16/11/2020 (3 pm - 5 pm) Ans. (d) : According to the passage "Tehsildar" was invited to inavgurate the girl's school. Hence option (d) will be correct answer. Comprehension: Read the given passage and answer the questions that follow. Santiniketan embodies Rabindranath Tagore’s vision of a place of learning that is unfettered by religious and regional barriers. Established in 1863 with the aim of helping education go beyond the confines of the classroom, Santiniketan grew into the Visva Bharati University in 1921, attracting some of the most creative minds in the country. He developed a curriculum that was a unique blend of art, human values and cultural interchange. Even today, in every step, in every brick and in every tree at Santiniketan, one can still feel his presence, his passion, his dedication and his pride in the institution. In 1862, Maharishi Debendranath Tagore, father of Rabindranath, was taking a boat ride through Birbhum, the westernmost corner of Bengal, when he came across a landscape that struck him as the perfect place for meditation. He bought the large tract of land and built a small house and planted some saplings around it. Debendranath Tagore decided to call the place Santiniketan, or the ‘abode of peace’, because of the serenity it brought to his soul. In 1863, he turned it into a spiritual centre where people from all religions, castes and creeds came and participated in meditation.

728

YCT

In the years that followed, Debendranath’s son Rabindranath went on to become one of the most formidable literary forces India has ever produced. He wrote in all literary genres but he was first and foremost a poet. As one of the earliest educators to think in terms of the global village, he envisioned an education that was deeply rooted in one’s immediate surroundings but connected to the cultures of the wider world. Located in the heart of nature, the school aimed to combine education with a sense of obligation towards the larger civic community. Blending the best of western and traditional eastern systems of education, the curriculum revolved organically around nature with classes being held in the open air. Tagore wanted his students to feel free despite being in the formal learning environment of a school, because he himself had dropped out of school when he found himself unable to think and felt claustrophobic within the four walls of a classroom. Nature walks and excursions were a part of the curriculum, special attention was paid to natural phenomena and students were encouraged to follow the life cycles of insects, birds and plants. The rural paradise of Santiniketan, Tagore’s erstwhile home, has become a thriving centre of art, education and internationalism over the years. 98. Which of these statements about Santiniketan is NOT true? (a) Santiniketan is located in the western most part of Bengal (b) Santiniketan grew into Visva Bharati university (c) At Santiniketan, classes were held in the open air (d) Santiniketan was set up by Rabindranath Tagore SSC CGL (Tier-II) 16/11/2020 (3 pm - 5 pm) Ans. (d) : According to the passage option (d) "Santiniketan was set up by Rabindranath Tagore" is NOT ture. Hence option (d) will be correct answer. 99. What did Santiniketan initially serve as? (a) A spiritual centre (b) A summer house (c) A holiday resort (d) A classroom SSC CGL (Tier-II) 16/11/2020 (3 pm - 5 pm) Ans. (a) : According to the passage Santiniketan intially serve as "A spiritual centre". Hence option (a) will be correct answer. 100. What does the name Santiniketan mean? (a) Abode of culture (b) Abode of learning (c) Abode of peace (d) Abode of love SSC CGL (Tier-II) 16/11/2020 (3 pm - 5 pm) Ans. (c) : According to the passage Santiniketan means "Abode of peace" hence option (c) will be correct. 101. The curriculum designed for Santiniketan was a blend of : (a) science and religion (b) human values art and culture (c) westerm education and village practices (d) spiritual and religious exchange SSC CGL (Tier-II) 16/11/2020 (3 pm - 5 pm) Reading Comprehension

Ans. (b) : According to the passage The curriculum designed for Santiniketan was a blend of "human values art and culture". Hence option (b) will be correct answer. 102. The work 'unfettered' in the text suggest : (a) restricted by regional differences (b) bound by religious beliefs (c) free from barriers (d) outside the classroom SSC CGL (Tier-II) 16/11/2020 (3 pm - 5 pm) Ans. (c) : According to the passage the word "unfettered" in the text suggest that 'free from barriers.' Hence option (c) will be correct answer. 103. When did Santiniketan grow into a university? (a) In 1921 (b) In 1862 (c) In 1922 (d) In 1863 SSC CGL (Tier-II) 16/11/2020 (3 pm - 5 pm) Ans. (a) : According to the passage Santiniketan grow into a university in 1921. Hence option (a) will be correct answer. 104. Why did Rabindranath drop out of school? (a) He wanted a formal learning environment (b) He felt stifled within the classroom (c) He found the curriculum too tough (d) He was not interested in studies SSC CGL (Tier-II) 16/11/2020 (3 pm - 5 pm) Ans. (b) : According to the passage Rabindranath drop out of school because he felt stifted within the classroom. Hence option (b) will be correct answer. 105. Rabindranath was a 'formidable' literary force. This implies he was : (a) an avid reader of books (b) an orthodox educationist (c) a wealthy landowner (d) a powerful writer SSC CGL (Tier-II) 16/11/2020 (3 pm - 5 pm) Ans. (d) : According to the passage Rabindranath was a 'formidable' literary force. This implies he was "a powerful writer." Hence option (d) will be correct answer. 106. With what aim was Santiniketan established? (a) To make it the perfect place for meditation (b) To attract the most creative minds (c) To encourage education outside the classroom (d) To nurture plants, birds and insects SSC CGL (Tier-II) 16/11/2020 (3 pm - 5 pm) Ans. (c) : According to the passage the aim was Santiniketan established "To encourage education outside the classroom." Hence option (c) will be correct answer. 107. Tagore's 'erstwhile' home means: (a) rural retreat (b) magnificent house (c) serene abode (d) former home SSC CGL (Tier-II) 16/11/2020 (3 pm - 5 pm) Ans. (d) : According to the passage Tagore's erstwhile home means "former home." Hence option (d) will be correct answer.

729

YCT

Comprehension: Read the passage and answer the questions given below it. The children were to be driven, as a special treat, to the sands at Jagborough. Nicholas was not to be of the party; he was in disgrace. Only that morning he had refused to eat his wholesome bread-and-milk on the seemingly frivolous ground that there was a frog in it. Older and wiser and better people(his aunt), had told him that there could not possibly be a frog in his bread-and-milk and that he was not to talk nonsense; he continued, nevertheless, to talk what seemed complete nonsense, and described with much detail the coloration and markings of the alleged frog. The dramatic part of the incident was that there really was a frog in Nicholas's basin of bread-and-milk; he had put it there himself, so he felt entitled to know something about it. The sin of taking a frog from the garden and putting it into a bowl of wholesome bread-and-milk was enlarged on at great length, but the fact that stood out clearest in the whole affair, as it presented itself to the mind of Nicholas, was that the older, wiser, and better people had been proved to be profoundly in error in matters about which they had expressed the utmost assurance. "You said there couldn't possibly be a frog in my bread-and-milk; there was a frog in my bread and milk," he repeated, with the insistence of a skilled tactician who does not intend to shift from favourable ground. 108. What was the aunt trying to tell Nicholas? (a) That he should play with his cousins (b) That he would be punished if he misbehaved (c) That he should have eaten the bread and milk (d) That he could choose to stay back if he wished SSC CPO-SI (Tier-II) 27/09/2019 (3 pm - 5 pm) Ans. (b) : According to the passage the aunt trying to tell Nicholas "That he would be punished if he misbehaved." Hence option (b) will be correct answer. 109. What excuse did Nicholas give for not eating his bread and milk? (a) there was a frog in it (b) he did not like it (c) there was no sugar in it (d) it was too cold to eat SSC CPO-SI (Tier-II) 27/09/2019 (3 pm - 5 pm) Ans. (a) : According to the passage the excuse given by Nicholas for not eating his bread and milk because "there was a frog in it" Hence option (a) will be correct. 110. What made Nicholas so sure about what he said? (a) He noticed it jumping in (b) He saw someone putting it in (c) His cousins had told him about it (d) He himself had put it in SSC CPO-SI (Tier-II) 27/09/2019 (3 pm - 5 pm) Ans. (d) : According to the passage Nicholas made so sure about what he said that he himself had put it. Hence option (d) will be corrrect answer. Reading Comprehension

111. Why did Nicholas consider his aunt had been proved to be wrong? (a) She was forcing him to eat the bread and milk. (b) She had said there was no frog in the bowl (c) She never listened to him (d) She always scolded him SSC CPO-SI (Tier-II) 27/09/2019 (3 pm - 5 pm) Ans. (b) : Accoding to the passage Nicholas consider his aunt had been proved to be wrong because "She had said there was no frog in the bowl." Hence option (b) will be correct answer. 112. The children were being taken on a special treat to: (a) The circus (b) The zoo (c) A garden (d) The beach SSC CPO-SI (Tier-II) 27/09/2019 (3 pm - 5 pm) Ans. (d) : According to the passage The children were being taken on a special treat to "The beach." Hence option (d) will be correct answer. Comprehension: Read the passage and answer the questions given below it Our civilization is more secure because it is much more widely spread. Most of the previous civilizations came to an end because uncivilized peoples broke in and destroyed them. This was the fate of Babylon, Assyria, India, China, Greece and Rome. Previous civilizations were specialized and limited like an oasis in a surrounding desert of savagery. Eventually, the desert closed in and the oasis was no more. But to-day, it is the oasis which is spreading over the desert. Practically no part of the world is untouched by it. For the first time, the world has now a chance of becoming a single whole, a unity. To-day, the food we eat comes from all over the world. The things in a grocer’s shop are from the ends of the earth. There are oranges from Brazil, grapes from Africa, rice from India, tea from china, sugar from Columbia etc. To-day the world is beginning to look more like one enormous box. Therefore, there is little danger upon our civilization from outside. The danger comes only from within; it is a danger from among us. To-day, the sharing-out of money – the sharing-out of food, clothing, houses and books, is still very unfair. In England alone, in sharing out of the National Income, we find that one half is divided among every sixteen people and the seventeenth person gets the other half. So while some few people live in luxury, many have not even enough to eat and drink and wear. Again, in England today, thousands of people live in dreadful surroundings. There are many families of five or six persons who live in a single room; in this same room they are born and in this same room they die, because, they are too poor to afford another room. Until everyone gets his proper share of necessary and delightful things, our civilization will not be perfect. 113. This’ in the third sentence refers to: (a) Spread of civilizations (b) Destruction of civilizations (c) Increase in security (d) Rise of education SSC CPO-SI (Tier-II) 27/09/2019 (3 pm - 5 pm)

730

YCT

Ans. (b) : According to the passage 'This' in the third sentence refers to "Destruction of civilizations" Hence option (b) will be correct answer. 114. What is one of the greatest disadvantages of the present times? (a) We are at risk of attack from outsiders (b) We have an internal threat from our own kind. (c) We need to bring things from other parts of the world. (d) We cannot grow our own produce SSC CPO-SI (Tier-II) 27/09/2019 (3 pm - 5 pm) Ans. (b) : According to the passage the greatest disadvantages of the present times is "We have an internal threat from our own kind." Hence option (b) will be correct answer. 115. Which important feature of modern life troubles the writer most? (a) There is inequality in the distribution of things. (b) People do not care for their fellow beings. (c) Civilizations have become dependent. (d) Older civilizations do not exist anymore. SSC CPO-SI (Tier-II) 27/09/2019 (3 pm - 5 pm) Ans. (a) : According to the passage the important feature of modern life trouble the writer most "There is inequality in the distribution of things." Hence option (a) will be correct answer. 116. What according to the writer was the desert that surrounded the oasis? (a) The poorer people (b) Some rich countries (c) Civilization (d) Savagery SSC CPO-SI (Tier-II) 27/09/2019 (3 pm - 5 pm) Ans. (d) : According to the writer the desert that surrounding the oasis was "Savagery." Hence option (d) will be correct answer. 117. Which of the following statements is NOT true according to the facts given in the passage. (a) Goods are transported from far- off places. (b) Some ancient civilizations have been destroyed. (c) Most people share their money equally. (d) Some people do not have adequate space to live. SSC CPO-SI (Tier-II) 27/09/2019 (3 pm - 5 pm) Ans. (c) : According to the passage "Most people share their money equally." is NOT true. Hence option (c) will be correct answer. Comprehension: Read the passage and answer the questions given below it No one can look back on his schooldays and say with truth that they were altogether unhappy. I have good memories of St Cyprian's, among a horde of bad ones. Sometimes on summer afternoons there were wonderful expeditions across the Downs to a village called Birling Gap, or to Beachy Head, where one bathed dangerously among the boulders and came home covered with cuts. And there were still more wonderful mid-summer evenings when, as a special treat, we were not driven off Reading Comprehension

to bed as usual but allowed to wander about the grounds in the long twilight, ending up with a plunge into the swimming bathe at about nine o'clock. There was the joy of waking early on summer mornings and getting in an hour's undisturbed reading (Ian Hay, Thackeray, Kipling and H. G. Wells were the favourite authors of my boyhood) in the sunlit, sleeping dormitory. There was also cricket, which I was no good at but with which I conducted a sort of hopeless love affair up to the age of about eighteen. And there was the pleasure of keeping caterpillars — the silky green and purple puss-moth, the ghostly green poplar-hawk, the privet-hawk, large as one's third finger, specimens of which could be illicitly purchased for six pence at a shop in the town — and, when one could escape long enough from the master who was ‘taking the walk’, there was the excitement of dredging the dew-ponds on the Downs for enormous newts with orange-coloured bellies. This business of being out for a walk, coming across something of fascinating interest and then being dragged away from it by a yell from the master, like a dog jerked onwards by the leash, is an important feature of school life, and helps to build up the conviction, so strong in many children, that the things you most want to do are always unattainable. 118. We can infer that the writer was a : (a) disobedient student (b) lonely child (c) mischievous boy (d) studious boy SSC CPO-SI (Tier-II) 27/09/2019 (3 pm - 5 pm) Ans. (c) : According to the passage we can infer that the writer was a "mischievous boy." Hence option (c) will be correct answer. 119. Why does the writer call cricket a hopeless love affair? (a) He was forced to give it up (b) He did not play it at all (c) He played it secretly (d) He tried hard to learn it SSC CPO-SI (Tier-II) 27/09/2019 (3 pm - 5 pm) Ans. (d) : According to the passage the writter call cricket a hopeless love affair because "he tried hard to learn it." Hence option (d) will be correct answer. 120. "Where one bathed dangerously" . Why does the writer call the bathing dangerous? (a) they were punished later (b) there were snakes in the water (c) the boulders gave them bruises (d) the water was too deep SSC CPO-SI (Tier-II) 27/09/2019 (3 pm - 5 pm) Ans. (c) : According to the passage the writter call the bathing dengerous because "the boulders gave them bruise." Hence option (c) will be correct answer. 121. What is the ‘moral’ the boy draws from his childhood experiences? (a) Things you enjoy doing are always unreachable (b) Only do the things your masters tell you to. (c) Do not walk alone in the grounds. (d) Do not collect insects like caterpillars SSC CPO-SI (Tier-II) 27/09/2019 (3 pm - 5 pm)

731

YCT

Ans. (a) : According to the passage the 'moral' is "Things you enjoy doing are always unreachable." Which the boy drows from his children experiences. Hence option (a) will be correct answer. 122. Which of the following did the boys not do on summer mornings? (a) Read in bed (b) Sleep until late (c) play cricket (d) Collect caterpillars SSC CPO-SI (Tier-II) 27/09/2019 (3 pm - 5 pm) Ans. (b) : According to the passage the boys not "Sleep until late" on summer mornings. Hence option (b) will be correct answer. Comprehension: Read the passage and answer the questions given below it. While I waited expectantly, Madame came in and told me that there was no room at all for me in the hotel, not a bed, not a corner. She was extremely friendly and seemed to find a fund of secret amusement in the fact; she looked at me as though expecting me to break into delighted laughter. “To-morrow,” she said, “there may be. I am expecting a young gentleman who is suddenly taken ill to move from number eleven. He is at present at the chemist's perhaps you would care to see the room?” “Not at all,” said I. “Neither shall I wish tomorrow to sleep in the bedroom of an indisposed young gentleman.” “But he will be gone,” cried Madame, opening her blue eyes wide and laughing with that French cordiality so enchanting to English hearing. I was too tired and hungry to feel either appreciative or argumentative. “Perhaps you can recommend me another hotel?” “Impossible!” She shook her head and turned up her eyes, mentally counting over the blue bows painted on the ceiling. “You see, it is the season in Bruges, and people do not care to let their rooms for a very short time” not a glance at my little suit case lying between us, but I looked at it gloomily, and it seemed to dwindle before my desperate gaze become small enough to hold nothing but a collapsible folding tooth-brush. “My large box is at the station,” I said coldly, buttoning my gloves. Madame started. “You have more luggage. Then you intend to make a long stay in Bruges, perhaps?” “At least a fortnight perhaps a month.” I shrugged my shoulders. “One moment,” said Madame. “I shall see what I can do.” She disappeared, I am sure not further than the other side of the door, for she reappeared immediately and told me I might have a room at her private house “just round the corner and kept by an old servant who, although she has a well eye, has been in our family for fifteen years. The porter will take you there, and you can have supper before you go.” I was the only guest in the dining-room. A tired waiter provided me with an omelet and a pot of coffee, then leaned against a sideboard and watched me while I ate, the limp table napkin over his arm seeming to symbolise the very man. “Are you ready, Madame?” asked the waiter. “It is I who carry your luggage.” Reading Comprehension

123. Which sentence tells us that Madame changed her mind suddenly and decided to give accomodation to the writer? (a) I was the only guest in the dining-room. (b) people do not care to let their rooms for a very short time. (c) My large box is at the station (d) she reappeared immediately SSC CPO-SI (Tier-II) 27/09/2019 (3 pm - 5 pm) Ans. (d) : According to the passage "she reappeared immediately." tell us that Madame changed her mind suddenly and decided to give accomodation to the writer. Hence option (d) is the correct answer. 124. How long does the narrator plan to stay in that place? (a) Two days (b) One week (c) Four days (d) A month SSC CPO-SI (Tier-II) 27/09/2019 (3 pm - 5 pm) Ans. (d) : According to the passage the narrator plane to stay in place "A month." Hence option (d) will be correct answer. 125. Who looks after the private house? (a) The waiter (b) The porter (c) Madame (d) An old servant SSC CPO-SI (Tier-II) 27/09/2019 (3 pm - 5 pm) Ans. (d) : According to the passage "An old servant" Looks after the private house. Hence option (d) will be correct answer. 126. Madame says “impossible” when the narrator asks her ______. (a) to recommend another hotel (b) for a porter to carry her bag (c) for space in the hotel from the next day (d) for some supper SSC CPO-SI (Tier-II) 27/09/2019 (3 pm - 5 pm) Ans. (a) : According to the passage Madame says "impossible" when the narrator asks her "to recommend another hotel." Hence option (a) will be correct answer. 127. Madame is probably ______ (a) Nurse at the clinic (b) A saleswoman at the shop (c) Another guest like the narrator (d) A Manager of the hotel SSC CPO-SI (Tier-II) 27/09/2019 (3 pm - 5 pm) Ans. (d) : According to the passage Madame is probable "A manager of the hotel." Hence option (d) will be correct answer. Comprehension: Read the passage and answer the questions given below it. In 18th-century Germany, Baron Munchausen regales his friends, over drinks, with stories of his many adventures. These include his supposed travel to the moon. Long before Neil Armstrong and Edwin Aldrin stepped out of Apollo 11 onto the surface of the moon 50 years ago, on July 20, 1969, the aspiration to travel to this shiny orb in the sky has fired human imagination. A Flight To The Moon by George Fowler, From The Earth To The Moon – and its

732

YCT

sequel, Around The Moon – by Jules Verne, The First Men In The Moon by HG Wells, Prelude To Space by Arthur C Clarke… are only a few of the many fictionalised accounts that have, for centuries, reflected this aspiration. In the 1954 Explorers On The Moon, iconic comic book character, Tintin, reaches the moon. While some writers imagined the moon’s surface to be barren and uninhabited, others have written about it being populated with lunar beings. Even when Apollo 11 mission was being planned and worked on, in 1963, Apollo At Go by Jeff Sutton, presented a realistic fictionalised portrayal of the upcoming landing. The successful landing of Apollo 11 and Neil Armstrong and Buzz Aldrin’s walk on the moon, opened up another exciting possibility – could the common man or nonspace-scientist reach for the moon? Literally? What had hitherto been in the realm of speculation was now a reality and we weren’t satisfied with just second-hand information any more. Between 1969 and 1972, the US sent six successful manned missions to the moon, and with each victory, the lunar destination seemed a little closer within the reach of the common man. 128. When did the US send first successful manned trip to the moon? (a) 1972 (b) 1963 (c) 1969 (d) 1954 SSC CPO-SI (Tier-II) 27/09/2019 (3 pm - 5 pm) Ans. (c) : According to the passage "in 1969" US send first successful manned trip to the moon. Hence option (c) will be correct answer. 129. Who was the first person to speak of travel to the moon? (a) Munchausen (b) Armstrong (c) Sutton (d) Branson SSC CPO-SI (Tier-II) 27/09/2019 (3 pm - 5 pm) Ans. (a) : According to the passage "Munchausen" was the first person to speak to travel to the moon. Hence option (a) will be correct answer. 130. Which of the following statements is not true according to the passage? (a) Man’s walk on the moon inspired scientists to plan more expeditions. (b) All the writers wrote about the moon being inhabited by weird creatures. (c) Someday the common man could make a trip to the moon. (d) Man has always been attracted to the moon. SSC CPO-SI (Tier-II) 27/09/2019 (3 pm - 5 pm) Ans. (b) : According to the passage "All the writers wrote about the moon being inhabited by weird creatures." is NOT true. Hence option (b) will be correct answer. 131. The writer uses the expression ‘literally’ because___ (a) it has been an idiom for desiring something unlikely (b) no one can ever go to the moon (c) it is wrong to build up stories (d) people only dream of doing it SSC CPO-SI (Tier-II) 27/09/2019 (3 pm - 5 pm) Reading Comprehension

Ans. (a) : According to the passage the writer uses the expression 'literally' because "It has been an idiom for desiring something unlikely". Hence option (a) will be correct answer. 132. How many fictional books written about travelling to the moon are referred to here? (a) Seven (b) Six (c) Eight (d) Five SSC CPO-SI (Tier-II) 27/09/2019 (3 pm - 5 pm) Ans. (a) : According to the passage "seven" fictional books written about travelling to the moon are referred to here. Hence option (a) will be correct answer. Comprehension : Read the following passage and answer the questions. Trade fairs are among the most memorable events that take place periodically with the purpose of promoting sales, launching new products, bringing together manufacturers of a particular line of products and educating the public. They are held at all levels – international, national, state and district. The most prominent among them are the India International Trade Fair, World Book Fair, Information Technology fair, Electronic Trade and Technology Fair, Textile Fair, Auto Expo, state level book fairs, district level exhibitions etc. India Trade Promotion Organization (ITPO), which was incorporated in 1992 by the merger of Trade Development Authority (TDA) with the Trade Fair Authority of India (TFAI), has been playing a commendable role in this respect. It can be said without a doubt that sales promotion is the most important purpose of these fairs. Bringing together the largest possible number of manufacturers, suppliers, existing and potential buyers under the same roof helps to promote the products in an effective way. All these people come together on a single platform for a fixed period of time. This offers a unique opportunity to manufacturers and suppliers to display their best products and services and the buyers get a chance to see a wide range of products and services. Conferences, seminars, live product demonstrations and presentations are regular features of these fairs and exhibitions. Besides these, colourful cultural programmes are also important features of such fairs. These fairs give a good opportunity to the artists to showcase their skills and talent at such specially organized programmes. 133. ITPO stands for : (a) Indian Trade and Promotion Order (b) India Trade Promotion Organisation (c) India Trade Promotion Authority (d) India Traders and Products and Organisation

733

SSC Stenographer Grade C&D –05/02/2019 (3:5pm)

Ans. (b) : According to the passage ITPO stands for "India Trade Promotion Organisation." Hence option (b) will be correct answer. YCT

134. What are some of the regular features of such trade fairs? (a) Programmes for launching new products and showcasing skills and talent (b) Conferences, seminars, live product demonstrations and meetings (c) Programmes for launching new products and bringing together manufacturers (d) Conferences, seminars, live product demonstrations and presentations SSC Stenographer Grade C&D –05/02/2019 (3:5pm)

Ans. (d) : According to the passage "Conferences, seminars, live product demonstrations and presentations" are the regular of such trade fairs. Hence option (d) will be correct answer. 135. How is the trade fair an effective way of promoting products? (a) It brings together the maximum number of manufacturers, suppliers and buyers in a single place (b) It brings together the maximum number of buyers to a single place (c) It is conducted by The Trade Fair Authority of India (d) It has seminars, conferences and film shows on the various products SSC Stenographer Grade C&D –05/02/2019 (3:5pm)

Ans. (a) : According to the passage the trade fair an effective way of promoting products is "It brings together the maximum number of manufacturers, suppliers and buyers in a single place." Hence option (a) will be correct answer. 136. Under the same roof means: (a) In an open area (b) In a big building (c) at one place (d) at different places

Here, the purpose is not to offend but to make him understand that there is something funny about his actions or behaviour. Secondly, the cartoonist may target some wrong practices or situations from different walks of life. Here, his aim is to use humour to not only criticize but also correct the wrong practices. In other words, correction through entertainment. The cartoonist can never beat around the bush because he needs to make his point with just a few strokes of his pen. Political cartoons, that is, cartoons making humorous comments on curent political situations and events are a regular feature of both English newspapers and regional language newspapers. They can be found in the editorial pages of a daily newspaper, in news magazines and on political websites. Political cartoons can be very funny, especially if people can understand the message in the cartoon. Their main purpose, though, is not only to amuse him but also make him think about current events and influence his opinion about the events. The best political cartoonist uses humour so skilfully that the reader's own opinions on various political issues are formed even without him even realizing how it happened. 138. What are the aims of the? (a) To criticize people and all their practices (b) To be humorous and offend people. (c) To make positive criticism about people and change wrong practices. (d) To show the people funny things and criticize them. SSC Stenographer Grade C&D –05/02/2019 (3:5pm)

Ans. (c) : According to the passage the aims of the cartoonist are "To make positive criticism about people and change wrong practices." Hence option (c) will be correct answer. 139. What does the passage talk about? (a) Cartoons and their purpose (b) Cartoons in different language papers SSC Stenographer Grade C&D –05/02/2019 (3:5pm) (c) Humorous cartoons Ans. (c) : According to the passage 'Under the same (d) Cartoonists in India roof' is an idiom which means "at one place." Hence SSC Stenographer Grade C&D –05/02/2019 (3:5pm) option (c) will be correct answer. 137. What is the main purpose of organizing trade Ans. (a) : The passage talk about "Cartoons and their purpose". Hence option (a) will be correct answer. fairs? (a) Informing people about the activities of 140. Why does the cartoonist use humour? (a) To influence public opinion traders (b) To help people have ideas about their society (b) Launching international products (c) To make people understand their duties (c) Bringing the manufacturers and suppliers (d) To change the political issues in the country together SSC Stenographer Grade C&D –05/02/2019 (3:5pm) (d) Sales promotion SSC Stenographer Grade C&D –05/02/2019 (3:5pm) Ans. (a) : According to the passage the cartoonist's aim is to use humour to not only criticize but also Ans. (d) : According to the passage the main purpose correct the wrong practice. In other word we can say of organizing trade fair is "sales promiton." Hence the Cartoonist use humour "To influence public option (d) will be correct answer. opinion." Hence option (a) will be correct. Comprehension : 141. Cartoons make the general reader smile Read the following passage and answer the because they make him notice something that questions. is: The general reader enjoys cartoons for two reasons. First, (a) general and unexpected these cartoons make him smile because they draw his (b) curious and unusual attention to something that is unusual and unexpected. (c) beautiful and unusual The cartoonist highlights some aspects of a well-known (d) uncommon and unexpected personality in the field of politics, social work, cinema, sports, business etc. and criticises the person involved. SSC Stenographer Grade C&D –05/02/2019 (3:5pm) Reading Comprehension

734

YCT

(c) He asked them to give the land to all their six sons (d) He asked them to consider him as their sixth son and give him one-sixth of their land.

Ans. (d) : According to the passage Cartoons make the general reader smile because they make him notice something that is "uncommon and unexpected." Hence option (d) will be correct answer. 142. To beat around the bush means: (a) to talk about a lot of unimportant things (b) to give an example (c) to highlight unique features (d) to say what is most important

SSC Stenographer Grade C&D –05/02/2019 (3:5pm)

SSC Stenographer Grade C&D –05/02/2019 (3:5pm)

Ans. (d) : According to the passage Vinoba make an appeal to the landlords "He asked them to consider him as their sixth son and give him one-sixth of their land." Hence option (d) will be correct answer. 145. How can it be said that Vinoba Bhave's movement was successful? (a) By 1969, he had collected 4 million acres for redistribution. (b) By 1951, he had persuaded the wealthy landlords to donate their land (c) By 1969, he had collected 12,000 acres for redistribution (d) In 1951, he had walked 300 miles to attend a meeting.

Ans. (a) : According to the passage "To beat around the bush" is an idiom which means "to talk about a lot of unimportant things." Hence option (a) will be correct answer. Comprehension : Read the following passage and answer the questions. The founder of the Bhoodan Yangna or the Land Gifts Mission was Acharya Vinoba Bhave, a close associate SSC Stenographer Grade C&D –05/02/2019 (3:5pm) and follower of Mahatma Gandhi. This movement, which was one of the greatest land reform movements in Ans. (a) : According to the passage Vinoba Bhave's Independent India, was started in the year 1951 in movement was successful because "by 1969, he had collected 4 million acres for redistribution." Hence Pochampalli, Telangana. In the spring of 1951 there was a meeting of rural option (a) will be correct answer. workers in Hyderabad. Since Vinoba Bhave never used 146. Read the passage and answer the questions by money, he decided to walk to this meeting which was choosing the correct option for each. some 300 miles away from where he lived. On the way, What type of a movement was the Bhoodan in every village through which he passed, he came face Yagna? to face with the misery of the poor, landless farmers. He (a) It was a Spring movement realized that he should leave no stone unturned in his (b) It was a land reforms movement mission to seek justice and land for his poor countrymen. (c) It was a village movement When he reached Hyderabad he went straight to a village (d) It was a land movement and in one of the prayer meetings he appealed to the SSC Stenographer Grade C&D –05/02/2019 (3:5pm) landlords. He said, "If you had five sons and a sixth were born to you, wouldn't you give him a portion of your Ans. (b) : According to the passage the Bhoodan estate? Treat me as your sixth son and give me one-sixth Yagna "was a land reform movement." Hence option of your land for redistribution to the poor." His words (b) will be correct answer. struck a chord among the landlords. 147. To leave no stone unturned means : Land was voluntarily donated and within the two months (a) To find a new path that he spent in Hyderabad, Vinoba received nearly (b) To try and do new things 12,000 acres in trust for the landless. Encouraged by this (c) To walk carefully on a road full of stones success, he travelled across india to convince the wealthy (d) To do everything possible to achieve a good landlords to share a small area of their land with their result poor, landless neighbours. By 1969, the Bhoodan SSC Stenographer Grade C&D –05/02/2019 (3:5pm) movement had collected over 4 million acres of land for Ans. (d) : According to the passage 'To leave no stone redistribution. 143. When and where was the Bhoodan movement unturned' is an idiom which means "To do everything possible to achieve a good result." Hence option (d) started? will be correct answer. (a) 1951, Hyderabad Comprehension : (b) 1951, Pochampalli, Telangana Read the passage carefully and choose the best (c) 1969, Pochampalli, telangana answer to each question out of the four (d) 1969, Hyderabad alternatives. SSC Stenographer Grade C&D –05/02/2019 (3:5pm) Doing an internship at the University of Lille in France, I Ans. (b) : According to the passage Bhoodan movement started' in 1951, Pochampalli, Telangana. almost always found myself stuck whenever I had to speak to non-Indians about India or on anything 'Indian'. Hence option (b) will be correct answer. This was more because of the subtle differences in the 144. How did Vinoba make an appeal to the way the French understood India in comparison to what I landlords? thought was 'Indian'. For instance, when I, or any Indian (a) He asked them to think of the villagers as for that matter, say 'Hindi' is an Indian language, what it their sons and give one-sixth of their land means is that it is one of the languages widely spoken in (b) He insisted that all the land belonged to the India. This need not be similar to the understanding that poor the French would have when they hear of 'Hindi' as an Reading Comprehension

735

YCT

Indian language. Because for them Hindi then becomes the only language spoken in India. This is a natural inference that the French, Germans, Italians and many other European nationals would tend to make, because that is generally how it is in their own respective countries. The risk of such inappropriate generalisations made about 'Indian' is not restricted to language alone but also for India's landscape, cuisine, movies, music, climate, economic development and even political ideologies. The magnitude of diversity of one European country can be easily compared to that of one of the Indian State, isn't it? Can they imagine that India is one country whose diversity can be equated to that of the entire European continent? The onus is upon us to go ahead and clarify the nuances in 'Indianness' while we converse. But why should one do so? How does it even matter to clarify? 148. Why do some French people think that Hindi is the only Indian language? (a) Because that is the way in most European countries (b) That is what is being taught to them (c) They know India is also called as Hindustan so people there must speak only Hindi (d) As most Indians they meet speak Hindi SSC CGL (Phase-II) 17/02/2018 Ans. (a) According to the passage option (a) Because that is the way in most European countries would be correct answer. Other options are not correct. 149. The writer was working at a university in which country? (a) France (b) Germany (c) Italy (d) India SSC CGL (Phase-II) 17/02/2018 Ans. (a) According to the passage the writer was working at a University in ‘France’. Hence option (a) would be correct answer. 150. What wrong with respect to India are the Europeans responsible for? (a) Their hatred towards Indian culture (b) Their complete lack of knowledge regarding India's past (c) That India is economically decades behind the developed world (d) Their inappropriate generalizations SSC CGL (Phase-II) 17/02/2018 Ans. (d) According to the passage the Europeans are responsible for their inappropriate generalization. Hence option (d) would be correct answer. 151. The writer compares diversity of one European country to the diversity of ____________. (a) One major city in India (b) One Indian State (c) The whole continent of Asia (d) The whole world SSC CGL (Phase-II) 17/02/2018 Ans. (b) According to the passage “The writer compares diversity of one European country to the diversity of 'one Indian state.' Other options are not correct. Hence option (b) would be correct answer. Reading Comprehension

152. According to the writer the responsibility of explaining the facts about India to Europeans rests with? (a) Europeans (b) Indians (c) Rest of the world (d) Indian Government SSC CGL (Phase-II) 17/02/2018 Ans. (b) According to the writer the responsibility of explaining the pacts about India to Europeans rests with ‘Indians’. Other options are not correct. Hence option (b) would be correct answer. Comprehension : Read the passage carefully and choose the best answer to each question out of the four alternatives. It was a bittersweet moment for me when I found out that I had been selected for the Sakura Science Exchange Programme, a Robotics and IoT workshop in Japan. A fully-funded opportunity of a lifetime. Fly off to Saitama without a care on the world, and all I had to do was put into practice what I love to do – computer science. The bitter part of the episode – that I would lose two weeks of IB education, an almost literal mountain to cover when I got back – was quickly forgotten when I envisioned myself programming robots in the country that gave us Anime and sushi! It was with the eagerness to have an extended vacation in an un-visited land, and the opportunity to learn more about a subject that I am passionate about, that I headed to the Kempegowda International Airport outside Bengaluru. Little did I know this would be the experience of a lifetime, more for the endearing values of the Japanese culture that made their mark on me than anything else. The first feature of Japanese society that called out to me was the Discipline. Walking into the Narita International Airport, used as I was to the noisy crowds back in India, I quite literally lost my breath to the sight that awaited me. Be it the security check or baggage claim, somehow there was a silence that felt right. Everyone went about their activities without any confusion. And, contrary to the bharatiya custom of lazy pot-bellied officials, every guard and all counter personnel did what they were supposed to do to ensure this flow was maintained. 153. What was it that the writer did not like about his trip to Japan? (a) That the trip was so short (b) That he would be compelled to eat sushi (c) That he would miss a fortnight worth of I B education (d) That Japanese people are too disciplined SSC CGL (Phase-II) 17/02/2018 Ans. (c) According to the passage “The writer did not like about his trip to Japan” that he would miss a fortnight would of IB Education” Hence option (c) would be correct answer. 154. What did the writer notice when he arrived at Narita International Airport? (a) The bags arriving on time in baggage claim (b) The cleanliness (c) That there was no security check (d) The silence SSC CGL (Phase-II) 17/02/2018

736

YCT

Ans. (d) According to the passage “ The writer noticed when he arrived at Narita International Airport “The Silence”. Hence option (d) would be correct. 155. Why was the writer travelling to Japan? (a) On a holiday (b) For two weeks of IB education (c) For employment (d) To attend a robotics workshop SSC CGL (Phase-II) 17/02/2018 Ans. (d) According to the passage “the writer travelling to Japan” To attend a robotics workshope” Hence option (d) would be correct answer. 156. What aspect of Japanese culture left a mark on the writer? (a) Their cuisine (b) Their unity (c) Their health consciousness (d) Their values SSC CGL (Phase-II) 17/02/2018 Ans. (d) According to the passage “ their values of Japanese culture left a mark on the writer. Hence option (d) would be correct. 157. Which country is credited for producing Anime? (a) India (b) Japan (c) America (d) England SSC CGL (Phase-II) 17/02/2018 Ans. (b) According to the passage “ Japan is credited for producing Anime and Sushi. Hence option (b) would be correct. Comprehension : Read the passage carefully and select the best answer to each question out of the given four alternatives. The conclusion of World Trade Organization’s 11th biennial ministerial conference at Buenos Aires was worrisome. From an Indian standpoint, there was no loss as status quo continues in the most important issue: the right to continue the food security programme by using support prices. But the inability of the negotiators to reach even one substantive outcome suggests that WTO’s efficacy is under question. As a 164-country multilateral organisation dedicated to crafting rules of trade through consensus, WTO represents the optimal bet for developing countries such as India. Strengthening WTO is in India’s best interest. Perhaps the biggest threat to WTO’s efficacy today is the attitude of the US. The world’s largest economy appears to have lost faith in the organisation and has begun to undermine one of its most successful segments, the dispute redressal mechanism. This is significant as the US has been directly involved in nearly half of all cases brought to WTO. Separately, large groups of countries decided to pursue negotiations on e-commerce, investment facilitation and removal of trade obstacles for medium and small scale industries. By itself this should not weaken WTO. But it comes at a time when there is growing frustration with gridlock at WTO. Reading Comprehension

India did well to defend its position on its food security programme. The envisaged reform package which will see a greater use of direct cash transfers to beneficiaries will be in sync with what developed countries do. But it’s important for India to enhance its efforts to reinvigorate WTO. In this context, India’s plan to organise a meeting of some countries early next year is a step in the right direction. WTO represents the best available platform to accommodate interests of adiverse set of nations. Therefore, India should be at the forefront of moves to fortify it. 158. Why was the WTO’s 11th binennial ministerial conference worrisome? (a) Denial of states quo. (b) Inability of negotiators to reach to substantive outcome for the problems. (c) Rift in the policies suggested by WTO on IPR. (d) Non-cooperation from Indian government on various matters. SSC CGL (Phase-II) 17/02/2018 Ans. (b) According to the passage ‘the WTO’ 11th biennial ministerial conference Worrisome because “ inability of negotiators to reach to substantive outcome for the problems. Hence option (b) would be correct answer. 159. What is the biggest threat to WTO’s efficacy today? (a) India being not working in its best interest. (b) Lost of faith in WTO by US. (c) Negotiators of WTO are not decision takers. (d) WTO’s lame attitude towards global trade. SSC CGL (Phase-II) 17/02/2018 Ans. (b) According to the passage “ The biggest threat to WTO’s efficacy today is ‘ Last of faith in WTO by us “. Hence option (b) would be correct answer. 160. Which of the following nation is keen to fortify its interest on WTO platform? (a) USA (b) Japan (c) Russia (d) India SSC CGL (Phase-II) 17/02/2018 Ans. (a) According to the sense of the passage “USA” is keen to fortify its interest an WTO Platform. Hence option (a) would be correct answer. 161. Which of the following is the most successful segments of the WTO mentioned in the passage? (a) Dispute redressal mechanism (b) Intellectual Property Rights (c) Reviewer of government’s t rade policies. (d) Agreement on trade in services. SSC CGL (Phase-II) 17/02/2018 Ans. (a) According to the passage “ Dispute redressal mechanim” is the most successful segment of the WTO mentioned in the passage. Hence option (a) would be correct.

737

YCT

162.

According to the passage, which of the following statement is NOT true? (a) 11th WTO conference was held at Nairobi. (b) India’s take on food security programme in the WTO conference was positive. (c) US is termed as the world’s largest economy. (d) US is involved directly in half of the cases brought to WTO SSC CGL (Phase-II) 17/02/2018 Ans. (a) According to the passage, 11th biennial ministerial Conference was held on Buenos Aire Not in Nairobi. Hence option (a) 11th WTO conference was held at Nairobi is Not True statement. Comprehension : Read the passage carefully and select the best answer to each question out of the given four alternatives. One should consciously engage in activities that will nourish your soul. Just as we nourish the body, we need to nurture the soul to connect to the creative power of the universe and to manifest joy in our lives. Often, we forget to address the soul, lost as we are in a jungle of material and sensual pleasures. But the more you embrace what feeds your soul, the happier you become. So if you want to enjoy the abundance of life, engage in what enriches your soul. Nurturing the soul is all about finding calm amidst chaos. There are a number of practices that empower people towards this end including silent contemplation, various forms of meditation, yoga and tai chi However, the rigor and discipline involved in the pursuit of such practices often seems to discourage people. Add to this, the temptations of the material world that leave little time and motivation for anyone to pursue the spiritual path. Poet Walt Whitman declared: “Whatever satisfies the soul is truth”. The good news is that simple, everyday activities can also nutrify the soul — like spending time in the midst of nature, dancing in the rain or just putting thoughts on paper. Do whatever is calming and pleases you. Creative pursuits are particularly appealing as inside each one of us, there is an artist craving for release and awaiting an opportunity for expression. One of the ways to indulge the artist within is to get started with the practice of any one or more of the creative art forms such as music, singing, dancing, acting, drawing, painting, sculpting, poetry, fiction or essay writing. When you engage in such soul nourishing activities, all thought and energy gets focused toward goal accomplishment. At this point, you will find that even unknown forces of the universe are conniving to assist you in your amateurish but sincere attempts. As you progress, you are motivated to do better. You touch and access a faculty, a part of you that you never knew existed. Your inner artist is unleashed, baring the beauty of your soul that has found a fond medium of expression. For instance, a sculptor’s soul is seen in his artwork; a musician’s in his compositions; an actor’s in his acting, a painter’s in his paintings and so on. It is immaterial whether your effort is an immaculate artwork or just a clumsy attempt by a layperson. The idea is to try, be inspired and to create giving free rein to the mind. As Michelangelo remarked: “I saw the angel in the marble and carved until I set him free”! Reading Comprehension

163. According to the passage, what makes us really happy? (a) A soulful music. (b) A soulful sculptures art. (c) Freeing our mind. (d) Embracing what feeds our soul. SSC CGL (Phase-II) 17/02/2018 Ans. (d) According to the Sense of the passage, “Embracing what feeds our soul” makes us really happy. Hence option (d) would be correct. 164. According to the passage, why even our amateurish attempts motivate us? (a) Because even unknown forces of the universe are assisting us in them. (b) Because we never knew that this part ever existed inside us. (c) Because these amateur attempts of ours are insincere ones. (d) Because they still are clumsy and need improvement. SSC CGL (Phase-II) 17/02/2018 Ans. (a) According to the passage, because even unknown forces of the universe are assisting us in them” is correct answer. Hence option (a) would be correct answer. 165. What activities can nutrify soul? (a) Creative activities that needs your involvement. (b) Any activity that doesn't touch the soul. (c) Peaceful and calming activities. (d) Immaculate artwork. SSC CGL (Phase-II) 17/02/2018 Ans. (c) According to the passage “Peaceful and Calming activities can nutrify soul” Hence option (c) would be correct. 166. What can you infer from Michelangelo’s statement - "I saw the angel in the marble and carved until I set him free"? (a) Michelangelo’s soul probably wanted to free the angel and hence his artwork portrayed the same. (b) Michelangelo didn’t like the angel and hence wanted to do away with it (c) Michelangelo wanted his artwork to look as close to real as possible. (d) Michelangelo’s mind felt free after setting the angel free in his artwork. SSC CGL (Phase-II) 17/02/2018 Ans. (a) For the above question “Michelangelo’s soul probably wanted to free angel and hence his artwork Portrayed the same. Hence option (a) would be correct. 167. Why do creative pursuits appeal us? (a) They nutrify our souls. (b) Inside each one of us, is an artist craving to come out. (c) They identify beauty of our body. (d) They are not simple activities. SSC CGL (Phase-II) 17/02/2018

738

YCT

Ans. (b) According to the passage “We do creative pursuits appeal us because inside each one of us, is an artist craving to come out. Hence option (b) would be correct. Comprehension : A passage is given with five questions following it. Read the passage carefully and select the best answer to each question out of the given four alternatives. The saddest part of life lies not in the act of dying, but in failing to truly live while we are alive. Too many of us play small with our lives, never letting the fullness of our humanity see the light of day. I’ve learned that what really counts in life, in the end, is not how many toys we have collected or how much money we’ve accumulated, but how many of our talents we have liberated and used for a purpose that adds value to this world. What truly matters most are the lives we have touched and the legacy that we have left. Tolstoy put it so well when he wrote: “We live for ourselves only when we live for others.” It took me forty years to discover this simple point of wisdom. Forty long years to discover that success cannot really be pursued. Success ensues and flows into your life as the unintended yet inevitable byproduct of a life spent enriching the lives of other people. When you shift your daily focus from a compulsion to survive towards a lifelong commitment to serve, your existence cannot help but explode into success. I still can’t believe that I had to wait until the “half-time” of my life to figure out that true fulfillment as a human being comes not from achieving those grand gestures that put us on the front pages of the newspapers and business magazines, but instead from those basic and incremental acts of decency that each one of us has the privilege to practice each and every day if we simply make the choice to do so. Mother Teresa, a great leader of human hearts if ever there was one, said it best: “There are no great acts, only small acts done with great love.” I learned this the hard way in my life. Until recently, I had been so busy striving, I had missed out on living. I was so busy chasing life’s big pleasures that I had missed out on the little ones, those micro joys that weave themselves in and out of our lives on a daily basis but often go unnoticed. My days were overscheduled, my mind was overworked and my spirit was underfed. 168. According to the passage, what does ''failing to truly live while we are alive means. (a) End up thinking of death all our lives. (b) Never letting the fullness of our humanity see the light of day. (c) Focus on basic and incremental acts of decency. (d) Over scheduling our days and over paying ourselves. SSC CGL (Phase-II) 17/02/2018 Ans. (b) According to the passage ”failing to truly live while we are alive means” Never letting the fullness of our humanity see the light of day” hence option (b) would be correct answer. Reading Comprehension

169. Suggest a suitable title for the passage? (a) True happiness as experienced by Mother Teresa (b) Forty years of discovery Tolstoy (c) Living truly (d) Learning it the hard way SSC CGL (Phase-II) 17/02/2018 Ans. (c) According to the passage “Living truly” is suitable title for the passage. Hence option (c) would be correct answer. 170. According to the passage, what took Tolstoy forty years to discover? (a) Simple point of happiness. (b) That we live for ourselves only when we live for others. (c) That his spirit was undeterred. (d) That he was a great leader of human hearts. SSC CGL (Phase-II) 17/02/2018 Ans. (b) According to the passage option (b) That we live for ourselves only when we live for others is correct answer. Other option give different meaning. 171. What according to the passage is success? (a) Success cannot be pursued. (b) Success is an unintended yet inevitable byproduct of a life spent enriching the lives of others. (c) Success is true fulfillment. (d) Success is incremental act of decency. SSC CGL (Phase-II) 17/02/2018 Ans. (b) According to the passage, “ Success is an Unintended yet inevitable by product of life spent enriching the lives of other. Hence option (b) would be correct answer. 172. A passage is given with five questions following it. Read the passage carefully and select the best answer to each question out of the given four. (a) That there are no great acts, only small acts are done with great love. (b) That she had been so busy striving that she had missed out on living. (c) That her days were over scheduled and her mind was over worked. (d) That she was so busy chasing life’s big pleasures that she had missed out on the little one’s. SSC CGL (Phase-II) 17/02/2018 Ans. (a) According to the passage option (a) That there are no great acts, only small acts are done with great love is correct answer. Other option give different meaning. Comprehension : A passage is given with five questions following it. Read the passage carefully and select the best answer to each question out of the given four alternatives. Teaching about compassion and empathy in schools can help deal with problems of climate change and environmental degradation,” says Barbara Maas, secretary, Standing Committee for Environment and Conservation, International Buddhist Confederation

739

YCT

(IBC). She was in New Delhi to participate in the IBC’s governing council meeting, December 10-11, 2017. “We started an awareness campaign in the year 2005-2006 with H H The Dalai Lama when we learnt that tiger skins were being traded in China and Tibet. At that time, I was not a Buddhist; I wrote to the Dalai Lama asking him to say that ‘this is harmful’ and he wrote back to say, “We will stop this.” He used very strong words during the Kalachakra in 2006, when he said, ‘If he sees people wearing fur and skins, he doesn’t feel like living. ‘This sent huge shock waves in the Himalayan community. Within six months, in Lhasa, people ripped the fur trim of their tubba, the traditional Tibetan dress. The messenger was ideal and the audience was receptive,” says Maas who is a conservationist. She has studied the battered fox’s behavioral ecology in Serengeti, Africa. She heads the endangered species conservation at the Nature and Biodiversity Conservation Union (NABU) International Foundation for Nature, Berlin. “I met Samdhong Rinpoche, The Karmapa, HH the Dalai Lama and Geshe Lhakdor and I thought, if by being a Buddhist, you become like this, I am going for it, “says Maas, who led the IBC initiative for including the Buddhist perspective to the global discourse on climate change by presenting the statement, ‘The Time to Act is Now: a Buddhist Declaration on Climate Change,’ at COP21 in Paris. “It was for the first time in the history of Buddhism that leaders of different sanghas came together to take a stand on anything! The statement lists a couple of important things: the first is that we amass things that we don’t need; there is overpopulation; we need to live with contentment and deal with each other and the environment with love and compassion,” elaborates Maas. She is an ardent advocate of a vegan diet because “consuming meat and milk globally contributes more to climate change than all "transport in the world.” Turning vegetarian or vegan usually requires complete change of perspective before one gives up eating their favorite food. What are the Buddhist ways to bring about this kind of change at the individual level? “To change our behavior, Buddhism is an ideal vehicle; it made me a more contented person,” says Maas, who grew up in Germany, as a sausage chomping, meat-loving individual. She says, “If I can change, so can anybody”. 173. According to the passage, how can studying compassion and empathy in schools help? (a) It can help us understand and connect Buddhism. (b) It can help deal with problems of climate change and environmental degradation. (c) It can change our behaviours and make us more content person. (d) It can help us in turning vegetarian. SSC CGL (Phase-II) 17/02/2018 Ans. (b) According to the passage option (b) It can help deal with problems of climate change and environmental degradation is correct answer. Other option give different meaning. 174. What did HH Dalai Lama said to his followers which came as a blow to them? (a) He said “we need to live with contentment and deal with each other and the environment with love and compassion. Reading Comprehension

(b) He said that if he sees people wearing fur and skins, he doesn’t feel like living. (c) He said Buddhism is an ideal vehicles it makes people more contented. (d) He said “we need to live with contentment and deal with each other an d the environment with love and compassion”. SSC CGL (Phase-II) 17/02/2018 Ans. (b) According to passage, HH Dalai Lama Said to his followers. He said that if he sees people wearing fur and skins, he doesn’t feel like living. Hence option (b) would be correct answer. 175. According to the passage, what do you infer from ''The messenger was ideal and the audience was receptive''? (a) It means that the audience found the messenger attractive and that they wanted to listen to him more and more. (b) It means that audience’s reaction goes hand in hand with the speaker’s effectiveness. (c) It means that HH Dalai Lama was a perfect choice of messenger for the message to be received by the audience. (d) It means that messenger was tested and was working properly. SSC CGL (Phase-II) 17/02/2018 Ans. (c) According to the passage option (c) means that HH Dalai Lama was a perfect choice of messenger for the massage to be received by the audience. Other options give different meaning. 176. Why did Ms. Barbara Maas say “If I can change, so can anybody”? (a) She never wanted to change but she still did, so anyone else can. (b) She was a complete vegan but still turned non vegetarian. (c) She did not believe in Buddhism but the religion attracted her. (d) She grew up eating non vegetarian but turned vegan. SSC CGL (Phase-II) 17/02/2018 Ans. (d) According to the passage, ‘Ms Barbara Maas say “ if I can change, so can anybody because she grow up eating non vegetarian but turned vegan. Hence option (d) would be correct. Comprehension: Read the passage and answer the questions given below it The Manipur government has appointed a nine-year-old girls as the State's Green ambassador, less than a week after a video of her crying for two tress axed for a roadwidening project went viral. An order from H. Balkrisna Singh, Manipur's Director of Information and Public Relations, on August 7 said Valentina Devi of Kakching district had been made the brand ambassador for the Chief Minister's Green Manipur Mission and all official tree-plantation activities taken up in the State. This, the order said, was in recognition of her immense love and affection for trees and for generating mass awareness on conservation of the environment for a "period of one year from the date of issuing the order"

740

YCT

A class V student of Amutombi Divine Life English School in Kakching town, about 45 km southeast of Manipur's capital Imphal, she had planted the saplings of two Gulmohar (royal Poinciana) trees by the side of a river when she was in class I. she had been tending to the saplings, which grew up into healthy trees in about four years. On Saturday, the two trees, along with other vegetation, were axed for a project to widen a road beside the river. Avideographer filmed her crying over the trees. I saw the video of this girl crying like anything, as if someone very near to her had passed away. Her feeling for two felled trees, which we adults would probably not think much about, moved me. I asked Victoria, the district's Superintendent of Police to meet her, console her and give her at least 20 saplings to plant at a suitable place, Manipur's Chief Minister told The Hindu on Thursday. But the felt this was not enough. "Later, I realised there could not be a better ambassdor than her for the Green Manipur Mission we had launched in June. Her passion for trees at such a young age was so infectious that I took a decision to have an order issued and hand it over through government officials to her," he said. The order says Valentina, the daughter of Premkumar singh, will be part of all such advertisements and campaigns that promote an increase in Manipur's green cover. "She will be provided honorarium at government rates, transportation facilities, food and lodging facilities at admissible rates by the sponsoring government department," the order said. 177. Select the option that correctly answer the question. Why did the police officer meet Valentina? (a) to offer her 20 saplings for the trees cut down (b) to make a video on her life (c) to inform her about her becoming the green ambassador (d) to tell her about the importance of the road widening SSC Sel. Post Phase-VII (M.L.) 15.10.2019 (Shift-I)

Ans. (a) : According to the passage, ‘The police officer meet Valentina to offer her so saplings for the trees cut down. Hence option (a) would be correct. 178. Select the option that correctly answers the question. As a Green Ambassador, which of the following will Valentina NOT be entitled to? (a) will get a monthly salary (b) will receive an honorarium (c) will get transport, food and lodging (d) will feature in advertisements SSC Sel. Post Phase-VII (M.L.) 15.10.2019 (Shift-I)

Ans. (a) : According to the passage, Valentina will not be entitled to get a monthly salary. Hence option (a) would be correct. 179. Select the option that correctly answers the question. Which of these would be the primary role of a Green Ambassador? Reading Comprehension

(a) to promote awareness about the conservation of the environment (b) to supervise tree planting (c) to stop trees from being cut down (d) to make films about environment SSC Sel. Post Phase-VII (M.L.) 15.10.2019 (Shift-I)

Ans. (a) : According to the passage, to promote awareness about the conservation of the environment, would be the primary role of a Green Ambassador. Hence option (a) would be correct. 180. Select the option that correctly answers the question. Valentina Devi has been appointed as Manipur's Green Ambassador because (a) She loves trees immensely (b) She planted two gulmohar trees when she was class 2 (c) the trees she planted were cut down (d) a videographer filmed her crying over the trees SSC Sel. Post Phase-VII (M.L.) 15.10.2019 (Shift-I)

Ans. (d) : According to the passage, Valentina Devi has been appointed as Manipur’s Green Ambassador because to make film about environment. Hence option (d) would be correct. Comprehension: Read the passage and answer the questions. The therapeutic value and healing powers of plants were demonstrated to me when I was a boy of about ten. I had developed an acute persistent abdominal pain that did not respond readily to hospital medication. My mother had taken me to the city’s central hospital on several occasions, where different drugs were tried on me. In total desperation, she took me to Egya Mensa, a wellknown herbalist in my home town in the western province of Ghana. This man was no stranger to the medical doctors at the hospital. He had earned the reputation of offering excellent help when they were confronted with difficult cases where Western medicine had failed to cure. After a brief interview, he left us waiting in his consulting room while he went out to the field. He returned with several leaves and the bark of a tree. One of his attendants quickly prepared a decoction and handed me a glass of this preparation. It was extremely bitter but within an hour or so’, I began to feel relieved. The rest of the decoction was put in two large bottles so that I could take doses periodically. Within three days, the frequent abdominal pains stopped and I recall gaining a god appetite. I have appreciated the healing powers of medicinal plants ever since. My experience may sound unusual to those who come from urban areas of the developed world, but for those in less affluent nations such experience are common. Studies by WHO indicate that for 75 to 90 per cent of the rural population of the world, the herbalist is the only person who handles their medical problems. In African culture, traditional medical practitioners are always considered to be influential spiritual leaders as well, using magic and religion along with medicines. Illness is handled with the individual’s hidden spiritual powers and with application of plants containing healing properties.

741

YCT

181. Egya Mensa was reputed to : (a) help where Western medicines failed (b) handle 75 per cent of rural population (c) cure all patients in the central hospital (d) heal people by magic and religion SSC Sel. Post Phase-VIII (G.L.) 09.11.2020 (Shift-II)

Ans. (a) : According to the passage, Egya Mensa was reputed to help where western medicines failed. Other option give different meaning. Hence option (a) would be correct answer. 182. The decoction given to the narrator was: (a) tasty but useless (b) made from roots and herbs (c) bitter but effective (d) made from fresh fruits SSC Sel. Post Phase-VIII (G.L.) 09.11.2020 (Shift-II)

Ans. (c) : According to the passage, The decoction give to the narrator was bitter but effective. Other option give different meaning. Hence option (c) would be correct answer. 183. Who was Egya Mensa? (a) The narrator (b) A spiritual healer (c) A herbal doctor (d) A medical doctor SSC Sel. Post Phase-VIII (G.L.) 09.11.2020 (Shift-II)

Ans. (c) : According to the passage, Egya Mensa was A herbal doctor. Hence option (c) would be correct. 184. The traditional medical practitioners are found: (a) in WHO (b) around the world (c) in African culture (d) in Ghana

The bowls never wanted washing. The boys polished them with their spoons till they shone again; and when they had performed this operation, they would sit staring at the cooper vessel with eager eyes employing themselves, meanwhile, in sucking their fingers. Oliver Twist and his companions suffered the tortures of slow starvation for three months; at last they got so voracious and wild with hunger, that one boy, who was tall for his age, hinted darkly to his companions, that unless he had another basin of gruel, he was afraid he might some night happen to eat the boy who slept next to him. He had a wild, hungry eye; and they implicitly believed. A council was held; lots were cast about who should walk up to the master after supper that evening, and ask for more; and it fell to Oliver Twist. The evening arrived; the boys took their places. The gruel was served. The boys whispered to each other, and winked at Oliver; while his next neighbors nudged him. Oliver rose from the table; and advancing to the master, basin and spoon in hand, said, "Please, sir, I want some more." The master aimed a blow at Oliver's head and shrieked aloud. 186. When Oliver Twist asked for more gruel: (a) The master screamed loudly (b) The other boys Supported him (c) The master threw the gruel at him (d) The other boys laughed at him SSC Sel. Post Phase-VIII (H.L.) 09.11.2020 (Shift-I)

Ans. (a) : According to the passage, Oliver Twist asked for more gruel for the master screamed Loudly. SSC Sel. Post Phase-VIII (G.L.) 09.11.2020 (Shift-II) Hence option (a) would be correct. Ans. (b) : According to the passage, The traditional 187. The boys held a meeting in order to decide: medical practitioners are found around the would. (a) Who would help the master in the hall Hence option (b) would be correct. (b) Who would beat the tall boy for his wildness 185. Which of the following statements is NOT (c) Who would steal the food from the kitchen true? (d) Who would tell the master they wanted more (a) The narrator is sceptic about the healing food powers of plants

SSC Sel. Post Phase-VIII (H.L.) 09.11.2020 (Shift-I) (b) The narrator was asked to take periodic doses Ans. (d) : According to the passage, The boys held a of the decoction meeting in order to decide who would tell the master (c) The narrator complained of frequent they wanted more food. Hence option (d) would be abdominal pains (d) The narrator paid several visits to the city’s correct answer. central hospital 188. The passage throws light on: SSC Sel. Post Phase-VIII (G.L.) 09.11.2020 (Shift-II) (a) How Oliver and other boys were well cared Ans. (a) : According to the passage, The narrator is for in school sceptic about the healing power of plants is Not true (b) Playful and mischievous behavior of the boys statement. Hence option (a) would be correct answer. (c) The starvation faced by boys living in a home Comprehension: for the poor Read the given passage and answer the (d) The kindness of people who looked after the questions that follow. boys The room in which the boys were fed, was a large stone SSC Sel. Post Phase-VIII (H.L.) 09.11.2020 (Shift-I) hall, with a copper vessel at one end, out of which the master ladled the gruel at mealtimes. Of this festive Ans. (c) : According to the passage, The passage composition each boy had one bowl, and no more - throws light on: The starvation faced by boys living in except on occasions of great public rejoicing, when he a home for the poor. Hence option (c) would be correct. had two ounces and a quarter of bread besides.

Reading Comprehension

742

YCT

189. Which of the following is NOT an indication fo the boys being poorly fed? (a) The boys whispered to each other (b) One tall boy said he might eat someone (c) They would sit staring at the copper, with eager eyes (d) The boys starved for three months SSC Sel. Post Phase-VIII (H.L.) 09.11.2020 (Shift-I)

Ans. (a) : According to the passage, The boys whispered to each other is NOT an indication for the boys being poorly fed. Hence option (a) would be correct. 190. The writer says that 'the bowls never wanted washing' because: (a) The boys cleaned them nicely with their hands (b) The boys ate up every little bit in the bowls (c) They were left to be cleaned by the assistants (d) There was a lot of grueld still left in them SSC Sel. Post Phase-VIII (H.L.) 09.11.2020 (Shift-I)

Ans. (b) : According to the passage, The writer says that the bowls never wanted washing because the boys ate up every little bit in the bowls. Hence option (b) would be correct. Comprehension: Read the following passage and answer the questions that follow. During the decade after liberalisation floriculture industries took giant steps in the export arena. This era has seen a dynamic shift from sustenance production to commercial production. The area under floriculture production in India is around 249 thousand hectares with a production of 1659 thousand tonnes loose flowers and 484 thousand tonnes cut flowers. Floriculture is now commercially cultivated in several states with Tamil Nadu (20%), Karnataka (13.5%) West Bengal (12.2%) having gone ahead of other producing states like Madhya Pradesh, Mizoram, Gujarat, Andhra Pradesh, Odisha, Jharkhand, Haryana, Assam and Chhattisgarh. Government of India has identified floriculture as a sunrise industry and accorded it 100% export oriented status. Owing to steady increase in the demand of flowers, floriculture has become one of the important commercial trades in agriculture. Hence commercial floriculture has emerged as hi-tech activity-taking place under controlled climatic conditions inside greenhouse. Floriculture in India, is being viewed as a high growth industry. Commercial floriculture is becoming important from the export angle. The liberalisation of industrial and trade policies paved the way for development of export-oriented production of cut flowers. The new seed policy had already made it feasible to import planting material of international varieties. It has been found that commercial floriculture has higher potential per unit area than most of the field crops and is therefore a lucrative business. Indian floriculture industry has been shifting from traditional flowers to cut flowers for export purposes. The liberalised economy has given an impetus to the Indian entrepreneurs for establishing export oriented floriculture units under controlled climatic conditions. Indian floriculture industry comprises of flowers such as Rose, Tuberose, Glads, Anthurium, Carnations, Marigold etc. Reading Comprehension

India's total export of floriculture was Rs. 571.38 crores/$81.94 million in 2018-19. The major importing countries were United States, Netherlands, United Kingdom, Germany and United Arab Emirates. There are more than 300 export-oriented units in India. More than 50% of the floriculture units are based in Karnataka, Andhra Pradesh and Tamil Nadu. With the technical collaborations from foreign companies, the Indian floriculture industry is poised to increase its share in world trade. 191. The main theme of the passage is: (a) the export of flowers from India (b) commercialisation of floriculture (c) the rise of floriculture industry in India (d) the Government’s policies regarding floriculture SSC CPO-SI – 25/11/2020 (Shift-I) Ans. (c) : According to the passage, The main theme of the passage is, ‘The rise of Floriculture Industry in India. Hence option (c) would be correct. 192. The area under floriculture production in India is about: (a) 571 thousand hectares (b) 1659 thousand hectares (c) 249 thousand hectares (d) 484 thousand hectares SSC CPO-SI – 25/11/2020 (Shift-I) Ans. (c) : According to the passage, The area under floriculture production in India is about 249 thousand hactares. Hence option (c) would be correct. 193. Which of the following statements is NOT true according to the passage? (a) India's total export of floriculture was $571.38 million in 2018-19. (b) More than 50% of the floriculture units are based in Karnataka, Andhra Pradesh and Tamil Nadu. (c) Floriculture has been identified as a sunrise industry by the Government of India. (d) Floriculture has been accorded a 100% export status by the Government. SSC CPO-SI – 25/11/2020 (Shift-I) Ans. (a) : According to the passage, India’s total export of floriculture was $571.38 million in 2018-19 is NOT true statement. Hence option (a) would be correct. 194. What gave a major fillip to the export of Indian floriculture products? (a) The new policy to ease the import of seeds of international varieties (b) The liberalisation of industrial and trade policies by the government (c) The use of technology in cultivating flowers inside greenhouses. (d) The demand from many countries like the US, the UK, and UAE. SSC CPO-SI – 25/11/2020 (Shift-I) Ans. (b) : According to the passage, The liberalization of Industrial and trade policies by the government gave a major fillip to the expert of Indian floriculture products. Hence option (b) would be currect.

743

YCT

195. Which three states are the major cultivators of floriculture products? (a) Tamil Nadu, Assam and Chhattisgarh (b) Tamil Nadu, Karnataka and West Bengal (c) Karnataka, Gujarat and Andhra Pradesh (d) West Bengal, Andhra Pradesh and Odisha SSC CPO-SI – 25/11/2020 (Shift-I) Ans. (b) : According to the passage, Tamil Nadu, Karnataka and West Bengal are three states are the major cultivators of floriculture products. Hence option (b) would be correct answer. Comprehension: Read the passage and answer the questions that follow. Greek religious traditions encompassed a large pantheon of gods, complex mythologies, rituals and cult practices. Greece was a polytheistic society, and looked to its gods and mythology to explain natural mysteries as well as current events. Religious festivals and ceremonies were held throughout the year, and animal sacrifice and votive offerings were popular ways to appease and worship the gods. Religious life, rituals and practices were one of the unifying aspects of Greece across regions and poleis (cities, or city-states, such as Athens and Sparta). Greek gods were immortal beings who possessed humanlike qualities and were represented as completely human in visual art. They were moral and immoral, petty and just, and often vain. The gods were invoked to intervene and assist in all matters large, small, private and public. City-states claimed individual gods and goddesses as their patrons. Temples and sanctuaries to the gods were built in every city. Many cities became cult sites due to their connection with a god or goddess and specific myths. For instance, the city of Delphi was known for its oracle and sanctuary of Apollo, because Apollo was believed to have killed a dragon that inhabited Delphi. The history of the Greek pantheon begins with the primordial deities Gaia (Mother Earth) and Uranus (Father Sky), who were the parents of the first of twelve giants known as Titans. Among these Titans were six males and six females. The males were named Oceanus, Hyperion, Coeus, Crius, Iapetus, and Kronos. The females were named Themis, Mnemosyne, Tethys, Theia, Phoebe, and Rhea. Kronos eventually overthrew Uranus and ruled during a mythological Golden Age. Over time, he and Rhea had twelve children who would become the Olympian gods. However, Kronos heard a prophecy that his son would overthrow him, as he did to Uranus. In an effort to avert fate, he ordered Rhea to allow him to devour each of the children upon their birth. Best known among the pantheon are the twelve Olympian gods and goddesses who resided on Mt. Olympus in northern Greece. Zeus, the youngest son of Rhea and Kronos, was hidden from his father, instead of being swallowed. Once he became a man, he challenged his father’s rule, forcing Kronos to regurgitate the rest of his swallowed children. These children were Zeus’s siblings, and together they overthrew Kronos, making Zeus the father of gods and men. Reading Comprehension

196. Which of the following statements about Greek Gods is NOT true? (a) They were invoked to aid in all matters. (b) They possessed human qualities. (c) Some of them were immoral, petty and vain. (d) Greek Gods were mortals. SSC CPO-SI – 24/11/2020 (Shift-I) Ans. (d) : According to the passage, Greek Gods were mortals is Not True. Other option are true. Hence option (d) would be correct. 197. How did Zeus bring back his siblings? (a) by challenging and slaying Crius (b) by making Kronos vomit the swallowed children (c) by worshipping the sanctuary of Apollo (d) by reviving them with an elixir SSC CPO-SI – 24/11/2020 (Shift-I) Ans. (b) : According to the passage, Zeus bring back his Sibling by making kronos vomit the swallowed Children. Hence option (b) would correct 198. Animal sacrifices were made to Gods to: (a) pacify and worship them (b) understand nature’s mysteries (c) enrage and challenge them (d) know about future events SSC CPO-SI – 24/11/2020 (Shift-I) Ans. (a) : According to the passage, animal sacrifices were to gods to pacify and worship them. Hence option (c) would be correct. 199. The twelve Olympian Gods and Goddesses were the children of: (a) Gaia and Uranus (b) Zeus and Phoebe (c) Kronos and Rhea (d) Hyperion and Theia SSC CPO-SI – 24/11/2020 (Shift-I) Ans. (c) : According to the passage, The twelve Olympian Gods and Goddesses were the Children of kronos and Rhea. Hence option (c) would be correct. 200. Kronos devoured his children at birth because: (a) his son was predicted to overthrow him (b) he was a titan who hated children (c) he did not have faith in his wife (d) it was a cult practice to please the gods SSC CPO-SI – 24/11/2020 (Shift-I) Ans. (a) : According to the passage, kronos devoured his children at birth because his son was predicted to ever throw him. Hence option (a) would be correct. Comprehension: Read the given passage and answer the questions that follow. Many of us have enjoyed visiting a zoo and consider it a day out having fun. Seeing real animals face to face can be thrilling and exciting, but many people wonder whether the animals themselves, while entertaining us, are suffering in the zoos. Many zoos do not have enough money to provide the animals with the basic features of a proper environment. Most animals are put in an artificial environment (e.g., cages). This isolation from their natural habitat can amount to cruelty. In some zoos, we see concrete floors for burrowing animals like rabbits. Cruelty may be shown in ways other than physical abuse or starvation.

744

YCT

Big predators like lions and tigers are created to chase and hunt. By depriving them of these activities, we forget the reason for which they have evolved. These big cats lie in cages in the zoo eating the food given to them and they have nothing else to do. Highly intelligent and curious animals like chimpanzees and polar bears may become frustrated in a zoo; they need a challenging environment. Zoo supporters might argue that zoos provide educational opportunities to the visitors, especially children; if we didn’t have animals in zoos, children wouldn’t see them at all and be ignorant about them. We live at a time when animals in their natural habitat can be seen on TV or computers, even smart phones, while sitting at home. Therefore, the aim should be to conserve animals in the wild, in protected areas called sanctuaries. 201. According to the passage, we visit zoos _____ . (a) to see how animals behave in zoos (b) to see the animals in cages (c) to spend a day out for fun and excitement (d) to watch the frustration of animals is cages SSC CPO-SI – 23/11/2020 (Shift-I) Ans. (c) : According to the passage, we visit Zoos to spend a day out for fear and excitement. Hence option (c) would be correct. 202. According to the passage, animals in zoos suffer cruelty: (a) by being forced to perform for the visitors (b) by being physically tortured or abused (c) by being kept in artificial environments (d) by being starved to death SSC CPO-SI – 23/11/2020 (Shift-I) Ans. (c) : According to the passage, animal in zoos suffer cruelty by being kept in artificial environments. Hence option (c) would be correct. 203. 'Burrowing' animals: (a) are solitary animals (b) dig the earth for food and shelter (c) chase and hunt other animals (d) are intelligent and curious SSC CPO-SI – 23/11/2020 (Shift-I) Ans. (b) : According to the passage, burrowing animals. Are dig the earth for food and shelter. Hence option (b) would be correct answer. 204. Which of the following best states the author’s purpose in the passage? (a) If animals were not kept in zoos, children would not be able to see and know about them at all. (b) Putting endangered species of animals in zoos is the best way of protecting and saving them. (c) One of the main reasons for which big predators have evolved is hunting. (d) We can show our concern for animals better by supporting protected spaces for them in the natural environment. SSC CPO-SI – 23/11/2020 (Shift-I) Ans. (d) : According to the passage, The best states the author’s purpose in the passage, we can show our concern for animals better by supporting protected spaces for them in the natural environment. Hence option (d) would be correct. Reading Comprehension

205. The word 'predators' (in paragraph 3) means: (a) animals which are starving in zoos (b) animals which are killed for food (c) animals which are kept in their natural habitat (d) animals which kill others for food SSC CPO-SI – 23/11/2020 (Shift-I) Ans. (d) : According to the passage, The word ‘predators’ means animal which kill others for food. Hence option (d) would be correct. Comprehension: Read the given passage and answer the questions that follow. Early written symbols were based on pictographs (pictures which resemble what they signify) and ideograms (symbols which represent ideas). Ancient Sumerian, Egyptian, and Chinese civilizations began to adapt such symbols to represent concepts, developing them into logographic writing systems. Pictographs are still in use as the main medium of written communication in some non-literate cultures in Africa, the Americas, and Oceania. Pictographs are often used as simple, pictorial, representational symbols by most contemporary cultures. Pictographs can be considered an art form, or can be considered a written language and are designated as such in Pre-Columbian art, Native American art, Ancient Mesopotamia and Painting in the Americas before Colonization. One example of many is the Rock art of the Chumash people, part of the Native American history of California. An early modern example of the extensive use of pictographs may be seen in the map in the London suburban timetables of the London and North Eastern Railway, 1936-1947, designed by George Dow, in which a variety of pictographs was used to indicate facilities available near each station. Pictographs remain in common use today, serving as pictorial, representational signs, instructions, or statistical diagrams. Because of their graphical nature and fairly realistic style, they are widely used to indicate public toilets, or places such as airports and train stations. Because they are a concise way to communicate a concept to people who speak many different languages, pictograms have also been used extensively at the Olympics since 1964 Summer Olympics, and are redesigned for each set of games. Pictographs can often transcend languages in that they can communicate to speakers of a number of tongues and language families equally effectively, even if the languages and cultures are completely different. This is why road signs and similar pictographic material are often applied as global standards expected to be understood by nearly all. A standard set of pictographs was defined in the international standard ISO 7001: Public Information Symbols. Other common sets of pictographs are the laundry symbols used on clothing tags and the chemical hazard symbols as standardised by the GHS system. Pictograms have been popularised in use on the web and in software, better known as ‘icons’ displayed on a computer screen in order to help users navigate a computer system or mobile device.

745

YCT

206. Pictographs are used in Olympic games because: (a) they are the main medium of written communication (b) they can transcend languages and convey information effectively (c) they can help to navigate computer systems (d) they can be redesigned for each set of games SSC CPO-SI – 25/11/2020 (Shift-II) Ans. (b) : According to the passage, Pictographs are used in Olympic games because the can transcend and convey information effectively hence option (b) would be correct. 207. What is the passage mainly about? (a) The origin of pictographs (b) Limitations of pictographs (c) Use of pictographs in ancient and modern times (d) Pictographs as an art form SSC CPO-SI – 25/11/2020 (Shift-II) Ans. (c) : According to the passage, the passage mainly about use of pictographs in ancient and modern times. Hence option (c) would be correct. 208. Early civilisations used symbols to represent: (a) concepts (b) facilities (c) art (d) culture SSC CPO-SI – 25/11/2020 (Shift-II) Ans. (c) : According to the passage, Early civilization used symbols to represent art. Hence option (c) would be correct. 209. Which of the following statements is NOT true? (a) Rock art of Chumash people is part of Mesopotamian history. (b) Pictographs are widely used to depict road signs. (c) Ideograms are symbols which represent ideas. (d) George Dow used pictographs to represent facilities near each train station. SSC CPO-SI – 25/11/2020 (Shift-II) Ans. (a) : According to the passage, Rock art of Chumash People is part of Mesopotamian history is not true statement. Because they are part of the Native American History of California. Hence option (a) would be correct. 210. Which of the following is an early example of use of pictographs in modern times? (a) Chemical hazard symbols (b) London railway timetable (c) Icons on a computer screen (d) Logographic writing system SSC CPO-SI – 25/11/2020 (Shift-II) Ans. (b) : According to the passage, London railway timetable is an early example of use of pictograph in modern times. Hence option (b) would be correct. Reading Comprehension

Comprehension: Read the given passage and answer the questions that follow. “The two great empires of Lilliput and Blefuscu have been engaged in a most obstinate war for six-and-thirty moons past. It began upon the following occasion. It is allowed on all hands that the primitive way of breaking eggs, before we eat them, was upon the larger end; but his present majesty’s grandfather, while he was a boy, going to eat an egg, and breaking it according to the ancient practice, happened to cut one of his fingers. Whereupon the emperor his father published an edict, commanding all his subjects, upon great penalties, to break the smaller end of their eggs. The people so highly resented this law, that our histories tell us, there have been six rebellions raised on that account; wherein one emperor lost his life, and another his crown. These civil commotions were constantly fomented by the monarchs of Blefuscu; and when they were quelled, the exiles always fled for refuge to that empire. It is computed that eleven thousand persons have at several times suffered death, rather than submit to break their eggs at the smaller end. Many hundred large volumes have been published upon this controversy: but the books of the Big-indians have been long forbidden, and the whole party rendered incapable by law of holding employments. During the course of these troubles, the emperors of Blefusca did frequently expostulate by their ambassadors, accusing us of making a schism in religion, by offending against a fundamental doctrine of our great prophet Lustrog. This, however, is thought to be a mere strain upon the text; for the words are these: ‘that all true believers break their eggs at the convenient end.’ And which is the convenient end, seems, in my humble opinion to be left to every man’s conscience, or at least in the power of the chief magistrate to determine. Low, the Big-indian exiles have found so much credit in the emperor of Blefuscu’s court, and so much private assistance and encouragement from their party here at home, that a bloody war has been carried on between the two empires for six-and-thirty moons, with various success; during which time we have lost forty capital ships, and a much greater number of smaller vessels, together with thirty thousand of our best seamen and soldiers; and the damage received by the enemy is reckoned to be somewhat greater than ours. However, they have now equipped a numerous fleet, and are just preparing to make a descent upon us; and his imperial majesty, placing great confidence in your valour and strength, has commanded me to lay this account of his affairs before you.” Thus spoke the Principal Secretary. I desired the secretary to present my humble duty to the emperor; and to let him know that I thought it would not become me, who was a foreigner, to interfere with parties; but I was ready, with the hazard of my life, to defend his person and state against all invaders. 211. What was the primitive way of eating eggs in Lilliput? (a) Breaking at the larger end (b) Eating it whole (c) Breaking in the middle (d) Breaking at the smaller end SSC CPO-SI – 23/11/2020 (Shift-II)

746

YCT

Ans. (a) : According to the passage, “Breaking at the Larger end” was the primitive way of eating eggs in Lilliput. Hence option (a) would be correct. 212. The empires of Lilliput and Blefuscu are: (a) ruled by one emperor (b) hostile towards each other (c) cordial and friendly neighbours (d) ready to compromise with each other SSC CPO-SI – 23/11/2020 (Shift-II) Ans. (b) : According to the passage, The empires of Lilliput and Blefuscu are hostile towards each other. Hence option (b) would be correct. 213. What was the result of the edict published by the emperor of Lilliput’s great grandfather? (a) People abandoned Lilliput and left for Blefuscu (b) People attacked the emperor and overthrew him (c) People readily agreed to follow his command (d) People rebelled against the new law SSC CPO-SI – 23/11/2020 (Shift-II) Ans. (d) : According to the passage, “People rebelled against the new Law” was the result of the edict published by the emperor of Lilliput’s great grandfather. Hence option (d) would be correct. 214. Which of the following is NOT true? (a) The two empires had been at war for thirtysix moons. (b) The narrator refuses to defend the emperor against the invaders. (c) The doctrine of Prophet Lustrog mentions the 'convenient end' of the eggs. (d) The big-endians found refuge at Blefuscu. SSC CPO-SI – 23/11/2020 (Shift-II) Ans. (b) : According to the passage, The narrator refuse to defend the emperor against the invader is not true. Hence option (b) would be correct. 215. What inference can be drawn from the narrative? (a) Wars may be waged over very trivial issues. (b) On should remain loyal to the monarch. (c) It is important to follow rules. (d) A peaceful solution to the problem is possible. SSC CPO-SI – 23/11/2020 (Shift-II) Ans. (a) : According to the passage, “wars may be waged over trivial issues” can be drawn from the narrative. Hence option (a) would be correct. Comprehension: Read the given passage and answer the questions that follow. Fishing traps, baskets, cradles, bridges, rainproof hats and umbrellas, mats, musical instruments, water pipes— Indians have always used the bamboo in numerous ways. It is used for house construction, fencing and in the making of bullock carts. Low-cost domestic furniture and a vast range of domestic utility items made of bamboo can be easily seen in any of our bazaars. But we do not easily notice the countless little ways this modest material comes to be used by rural people. One can see it Reading Comprehension

being used in the blacksmith’s bellows, or as bamboo pins in carpentry joints or in the fabrication of toys in village markets. But to the British foresters the multidimensional role that “the forest weed” played in the local Indian environment was of no account, as it did not figure in forest revenues. Bamboo also interfered with the growing of teak, an essential part of their colonial forest policy. It was only in the 1920s that the British realised that by mincing bamboo into millimetre shreds, cooking it in chemicals, pulping and flattening it, they could produce sheets of paper. This would bring the British increased forest revenue and ‘development’ (as defined by them) to the so-called backward regions of India. However, they chose to ignore the consequences this activity would have on the health of the forest. So while bamboo was sold at high prices to basket weavers, it was heavily subsidised for the paper industry. Even after Independence, supplying bamboo at extremely low prices to Indian paper mills became a ‘patriotic’ duty of the government, and bamboo supplies were assured for decades at unchanged prices. The disaster that this would cause to the forests, and to the craftsperson, still remained unforeseen. Colonialism affected forests all across India and marginalised their inhabitants and the traditional occupations they practised. As late as the 1970s, the World Bank proposed that 4,600 hectares of natural Sal forest should be replaced by tropical pine to provide pulp for the paper industry. It was only after protests by local environmentalists that the project was stopped. Colonialism was therefore not only about repression, it was also a story of displacement, impoverishment and ecological crisis. 216. Bamboo is referred to as ‘modest material’ because: (a) it can be easily shredded and reduced to a pulp (b) it is inexpensive and not highly acclaimed (c) the British believed it was worthless (d) it is available in abundance in India SSC CPO-SI – 24/11/2020 (Shift-II) Ans. (b) : According to the passage, “Bamboo is referred to as “modest material” because it is in expensive and not highly acclaimed. Hence option (b) would be correct. 217. What does the ‘forest weed’ refer to? (a) sal (b) teak (c) bamboo (d) pine SSC CPO-SI – 24/11/2020 (Shift-II) Ans. (c) : According to the passage, “The forest weed” refer to bamboo. Other options are not true. Hence option (c) would be correct. 218. What was considered to be a patriotic duty of the government after independence? (a) Preventing the displacement of foresters (b) Handling the ecological crisis in India (c) Replacing Sal forests by tropical pines (d) Supplying bamboo at low prices to paper mills SSC CPO-SI – 24/11/2020 (Shift-II)

747

YCT

Ans. (d) : According to the passage, Supplying bamboo at Law prices to paper mills was considered to be a patriotic duty of the government after independence. Hence option (d) would be correct. 219. Which of these is NOT a step in the production of paper? (a) Mincing (b) Pulping (c) Weaving (d) Flattening SSC CPO-SI – 24/11/2020 (Shift-II) Ans. (c) : According to the passage, ‘weaving’ is Not a step in the production of paper. Hence option (c) would be correct. 220. The varied uses of bamboo were disregarded by the British till the 1920s because: (a) they did not use bamboo products at all (b) it did not increase the forest revenue (c) it was used in the backward regions only (d) the bamboo products were cheaply available SSC CPO-SI – 24/11/2020 (Shift-II) Ans. (b) : According to the passage, The varied used of bamboo were disregarded by the British till the 1920 because ‘it did not increase the forest revenue” Hence option (b) would be correct. Comprehension : Read the following passage and answer the questions given after it. India generates at least 25,940 tonnes of plastic waste daily, equivalent to the weight of around 4,300 elephants. Of this, about 60% gets recycled, according to the Union environment ministry. The rest gets dumped in landfills, clogs draings, goes into the ocean as micro-plastics, or is burnt, leading to air pollution. In the absence of a proper waste management system, the plastics that get recycled are often dirty, which makes the re-cycling process water-intensive and expensive. ''It is the process of cleaning the plastics before recycling that makes it resource intensive. A lot of water is required to wash the collected plastics, especially if it is oily or greasy as it has to be cleaned with a solvent,'' said Dr Suneel Pandey, director of environment and waste management, The Energy and Resources Institute (TERI). Experts say proper waste collection and management is at the core of ensuring more plastics get recycled instead of ending up in landfills and oceans. Researchers from various institutes have come up with innovative ways to utilise the plastic waste that cannot be recycled further or are unrecyclable. At IIT Delhi, a group of chemical engineers are working on chemically breaking down plastics to its smaller hydrocarbon molecules and then synthesising diesel out of it. The process uses packaging material, Polyethylene terephthalate (PET) bottles, polystyrene, and multi-layer packaging. The fuel produced can currently be used as a blend in stationary diesel machines like generators and needs further testing and standardisation to be used as commercial diesel in vehicles. A mix of plastic and stone has been used to create a block that can be used in flooring. A CSIR laboratory also used shredded plastic, chemically treated it and mixed with fillers to make tiles. Reading Comprehension

A German chemical producer called BASF is also breaking down waste plastic and using it as raw material for chemicals, instead of using material derived from fossil fuels. One cost-effective solution was developed by Dr Rajagopalan Vasudevan, professor at Thiagarajar College of Engineering in Madurai. The National Highways Authority of India is currently scaling up his technology to use plastic waste with in making roads. He came up with the idea of mixing plastic waste with Bitumen used for constructing roads in 2001. ''That year the Tamil Nadu government had planned to ban plastic and my concern began with the more than 1 lakh people employed by the industry. Since plastic is derived from petroleum just like Bitumen, I thought of using it for road construction. The result, not only plastic waste was getting utilised, the roads were cheaper and steadier.'' he said. The plastic waste does not have to be segregated and even multi-layered plastics can be used in the mix. ''All we need to do is collect the waste, dry it out and use it,'' he said. The construction of every kilometre of road required nine tonnes of Bitumen and one tonne of plastic waste. This means for every kilometre of road, one-tonne Bitumen is saved, which costs about ` 30,000. 221. Which organization makes blocks and tiles from the plastic waste ? (a) CSIR (b) IIT, Delhi (c) BASF (d) TERI SSC CPO-SI – 09/12/2019 (Shift-II) Ans : (a) According to the passage, CSIR makes blocks and tiles from the plastic waste. Hence option (a) would be correct. 222. What has been developed by Dr Rajagopalan Vasudevan, professor at Thiagarajar College of Engineering in Madurai ? (a) method for using plastic to make tiles and blocks (b) method for using plastic for road construction (c) method for making diesel from plastic waste (d) method for extracting raw material for chemicals SSC CPO-SI – 09/12/2019 (Shift-II) Ans : (b) According to the passage, Methods for using plastic for road construction developed by Rajagopalan Vasudevan. Hence option (b) would be correct. 223. Which statement is NOT true according to the passage ? (a) The National Highways Authority of India is planning to use plastic waste in making roads. (b) The fuel produced from plastic waste can be used as a blend in stationary diesel machines like generators. (c) In the construction of one kilometer of road, nine tonnes of bitumen can be saved by using plastic waste. (d) India generates at least 25,940 tonnes of plastic waste daily. SSC CPO-SI – 09/12/2019 (Shift-II) Ans : (c) According to the passage option (C) is not a true statement. Other option are true.

748

YCT

224. The above passage mainly talks about (a) the problems of recycling plastic waste. (b) the need of research to manage plastic waste. (c) the plastic waste generated in India. (d) putting the plastic waste to use by recycling. SSC CPO-SI – 09/12/2019 (Shift-II) Ans : (d) The passage mainly talks about putting the plastic waste to use by recycling. Hence option (d) would be correct. 225. What makes the recycling of plastic resourceintensive ? (a) the breaking down of plastic in molecules (b) the research required for recycling (c) the cleaning of the plastic waste (d) the collection of plastic waste SSC CPO-SI – 09/12/2019 (Shift-II) Ans : (c) According to the passage, the cleaning of the plastic waste makes the recycling of plastic resource – intensive. Hence option (c) would be correct. Comprehension: Read the following passage and answer the questions given after it. The excitement of bird watchers on spotting a particular species for the very first time is unparalleled. With eyes shining and pride bursting, they exclaim, “I had a lifer today”, or “It was a lifer for me”. For the longest time, I couldn’t fathom what all the fuss was about. I mean, at some point in their lives, even a crow or mynah would have been a lifer — seen for the very first time. So, here, I’ve done a bit of jugaad with the term and define it as a bird that you see may be (but not necessarily) for the first time, but which has made a lasting, life-changing impact on you. Then I recalled some of my own “lifers”. Number 1 is the little coppersmith barbet. The first bird I saw through brand new, big and powerful binoculars — and it was solely responsible for my getting interested in birds. The fellow looked like a tubby little clown with hiccups and that just blew me away. I will never forget the first time I saw grey hornbills aeons ago: over sullen grey skies in the Borivali National Park (now called Sanjay Gandhi National Park) — squealing as they flew high up across the sky. They looked as if they had just left Jurassic Park. Or, for that matter, their larger, more glamorous, cousins — Great pied hornbills. Tramping through a streambed in Kalagarh (near Corbett), we suddenly heard this rasping, whooshing, sound. Up there, in the clear blue, were sixseven huge black-and-white birds with colossal yellow beaks flying in tandem across the clear blue sky, their wings making the rasping sound. Say “paradise flycatcher” and a birder’s eyes will begin to glint: “Where? When? Will it be there now?” are questions that will be shot out like machine-gun bullets. The first time I saw a full grown milk-white-and-glossyblack male, with its glamorous 18-inch streamer tail, was at the Sultanpur National Park in Haryana. But I remember better the flycatchers, that made me run around in a tea garden in Palampur, teasingly whistling at me from one end to the other. The nesting pair in Naukuchiatal was more accommodating except that I had to stand kneedeep in the hotel’s garbage dump to get a good view of them flitting to and fro the gully nearby. To compensate, one actually flew nearly down to my feet to snatch up a bluebottle I had missed. Reading Comprehension

Of course, there have been rarities: the highlight of the regular Bharatpur (the Keoladeo National Park) visits was the darshan of VIP Siberian cranes. Then they stopped coming, which was a first indication of their slow extinction — even if it was just “local” to our area. The gloriously uppity Great Indian bustards in the Karera Sanctuary (Madhya Pradesh) were another unforgettable sighting. The sheer disdain with which they flounced away from our howling, jolting jeep and took to their wings was a lesson in being put in your place. Now, not only does the sanctuary not exist anymore but those magnificent muscular birds are crashing to total extinction. 226. What does the word 'lifer' mean for the author? (a) A bird which makes a life-changing impact on you (b) A bird which is becoming extinct (c) A bird that you see for the first time (d) A bird that you love SSC CPO-SI – 09/12/2019 (Shift-I) Ans : (a) In the above passage, ‘lifer’ is a bird which makes a life changing impact on you. Hence option (a) is correct. 227. Which bird is black and white with a huge yellow beak? (a) Great pied hornbill (b) Siberian crane (c) Paradise flycatcher (d) Grey hornbill SSC CPO-SI – 09/12/2019 (Shift-I) Ans : (a) In the above passage, great pied hornbill is black and white with a huge yellow beak. Hence option (a) is correct. 228. Which pair of birds did the author see in Naukuchiatal ? (a) Great Indian bustard (b) Paradise flycatcher (c) Hombill (d) Coppersmith barbet SSC CPO-SI – 09/12/2019 (Shift-I) Ans : (b) According to the passage, The author saw paradise flyeatcher pair of birds in Naukuchiatal. Hence option (b) is correct. 229. According to the passage which sanctuary does NOT exist anymore? (a) Sultanpur National Park, Haryana (b) Sanjay Gandhi National Park (c) Keoladeo National Park, Bharatpur (d) Karera Sanctuary, Madhya Pradesh SSC CPO-SI – 09/12/2019 (Shift-I) Ans : (d) According to the passage, Karera Sanctuary, Madhya Pradesh does not exit anymore. Hence option (d) is correct. 230. Match the words with their meaning. Words Meanings a. fathom 1. team b. aeons 2. understand c. tandem 3. ages (a) a-2, b-1, c-3 (b) a-1, b-3, c-2 (c) a-2, b-3, c-1 (d) a-3, b-2, c-1 SSC CPO-SI – 09/12/2019 (Shift-I)

749

YCT

Ans : (c) Correct matching for the above question is – a-2, b-3, c-1. Hence option (c) would be correct. Comprehension: Read the given passage and answer the questions that follow. The beaver had made a dam about hundred yards long, and it had formed out of the quickly flowing stream, a wide stretch of water about twelve feel deep. The dam was so firm and broad that it was easy for me to walk along it. After a hundred and fifty yards I came to a 'beaver's castle'- a great heap of logs skilfully fitted together, the lower part covered with earth and plants. The upper logs were put loosely together so that the air could pass through to the interior. In the late autumn, before land and water are covered with snow and ice, the beaver has to make its dwelling frostproof. It collects young trees and gnaws the stems in dauble cone until the tree falls. Then the long stems are gnawed into small sections, the thin twigs bitten off and the logs are dragged to the dwelling. There, the timber is piled on the castle, sometimes to a height of ten or twelve feet. Mud is brought up from the bottom of the lake and packed between the long with the animal's clever forepaws. The interior is lined with the finest wood shavings. In this tall shelter, the beaver remains high and dry, and protected from frost throughout the winter. The only access to the dwelling is under the water, and even the forest wolves are not able to pull the firm building apart. The water is the beaver's element. On land it moves slowly and awkwardly. Thus, it is vital for the beaver to have water in which it can swim, and where nature has not provided this condition for it, is creates it with its dam. The beaver's activities can change whole landscapes. With this dam, for instance, on which I was standing, beavers had turned a whole wooded alley into a lake; trees that stood there had been killed by the water and had disappeared. Aquatic game had settled there. Ducks swam past us, and great must have been the number of fish, as swarms of trout had swum past me in the clear stream. Wide stretches of meadow had come into existence on the banks, with flat landing places to which the well-beaten beaver tracks led. Many years of building, gnawing and dragging must have gone into the completion of this immense task, creating a new region for the beaver to live in. 231. Which line in the passage most strongly indicates that weavers simply love being in or near water? (a) The dam was so firm and broad that it was easy to walk along. (b) The water is the beaver's element. (c) The beaver had made a dam out of the quickly flowing stream. (d) The only access to the dwelling is under the water. SSC CPO-SI – 11/12/2019 (Shift-II) Ans : (b) The water is the beaver’s element ‘ strongly indicates that weavers simply love being in or near water. Hence option (b) is correct. Reading Comprehension

232. What evidence in the passage suggests that beavers build strong dwellings? (a) The interior is lined with finest wood shavings. (b) It takes many years of hard work to build the dwelling. (c) Even the forest wolves cannot pull the dwelling apart. (d) The beaver's dwelling is frostproof. SSC CPO-SI – 11/12/2019 (Shift-II) Ans : (c) According to the sense of the passage, option (c) will be the appropriate answer. 233. How does a beaver ensure that its castle is well ventilated? (a) The timber is piled upto ten feet high on the castle. (b) The upper logs are loosely placed. (c) The long are covered with earth and plants. (d) The long stems are gnawed into small sections. SSC CPO-SI – 11/12/2019 (Shift-II) Ans : (b) According to the sense of the passage, option (b) will be the appropriate answer. Other option are not correct. 234. The beavers can change the landscape by: (a) building dams that can convert a wooded alley into a lake. (b) swimming in the water along with ducks and trouts. (c) building castles with great heaps of logs. (d) dragging the logs of wood to the dwelling. SSC CPO-SI – 11/12/2019 (Shift-II) Ans : (a) According to the passage, The beavers can change the landscape by building dams that can convert a wooded alley into a lake. 235. Which of the following statements is NOT true? (a) The beaver can move quickly and easily on land and in water. (b) Wide stretches of meadows may come into existence on the banks of streams. (c) The beaver has to make its dwelling frostproof before it starts snowing. (d) The access to the beaver's dwelling is under the water. SSC CPO-SI – 11/12/2019 (Shift-II) Ans : (a) According to the passage, option (a) is not a true statement. Other option are correct. Comprehension: Read the following passage and answer the questions given after it. Standing in ankle-deep water engulfing his field, Kamal Singh Dhangar takes a wild shot at the likely yield from the soyabean crop that practically lies in ruins behind him. "One bag, two bags with difficulty", notes the 58-year old with a wry smile, minutes after a fresh spell of rains has lashed Guradiya Sirajuddin village in Ashta tehsil of Madhya Pradesh's (MP) Sehore district. While the same three-acre plot had given nearly 16 quintals of soyabean

750

YCT

last year, Dhangar this time isn't sure he can even afford labourers to harvest the negligible produce staring in his face. He draws solace only from his not being alone. Most farmers in Guradiya Sirajuddin as well as neighbouring villages have reported widespread damage to their already matured standing crop. Gulab singh, who owns 10 acress not far from Dhangar's field, is ruing his decision to have taken and additional six acres of land on lease for cultivating soyabean and wheat this year. "I have paid the owner Rs 1.5 lakh. By now, this soyabean crop should have been ready for taking to Ashta mandi (agriculture produce market at the tehsil town, about 10 km away). But my expensive pursuit has proved costly", remarks the 60-year-old, who too, cannot enter his field without wading through water almost touching the knees. Seated at the edge of his flooded field a few hundred meters away, Dev Singh, a sprightly octogenarian, cannot remember the year when the monsoon rains caused such late-stage havoc. "I may have been this big," he says, pointing to a man many decades younger to him. Western MP, in which Sehore falls, has received 1,335.4 mm of average rainfall from 1 June 1 to 25 September 58% more than the region's historical normal of 845.3mm for his period. However, the real story lies in the month-wise figures. In June, the opening month of the southwest monsoon season, the rains were actually 25.6% below the long-period average. In July, they were 31.2% above average. That surplus rose to 63.6% in August, while a whopping 158% more so far till September, confirming Dev Singh's observation. Worse, there's little respite, with more rains predicted over the next few days. The monsoon has taken its toll mainly on soyabean. According to the Union agriculture ministry's data, a total area of 113.449 lakh hectares (lh) has been planted under this leguminous oilseed in the current Kharif season, with the bulk of it accounted for by MP (55.16 lh), Maharashtra (39.595 lh) and Rajasthan (10.608 lh). Within MP, the main soyabean growing districts are Ujjain, Dewas, Indore, Dhar, Ratlam, Mandsaur, Rajgarh, Shajapur, Sehore and Vidisha. The crop in lowlying areas is the one that has been worst affected. "Farmers who had sown early-maturity (80-90 days duration) varieties such as JS 9560 and JS 2034 just after mid-june will take the biggest hit. Their crop would already have matured; the longer it remains in the field, the more the chances of the grain rotting. Also, these farmers will not be able to save this grain for using as seed next year", admits V S Bhatia, director of the Indian Institute of Soyabean Research at Indore. 236. Who among the following took on 6 acres of land for soyabean cultivation in addition to his original 10 acres of land? (a) Gulab singh (b) Vijay Singh (c) Kamal Singh (d) Dev Singh SSC CPO-SI – 11/12/2019 (Shift-I) Ans : (a) Gulab Singh took on 6 acres of land for soya bean cultivation in addition to his original 10 acres of land. Hence option (a) will be correct. Reading Comprehension

237. Match the words with their meaning. Words Meanings a. havoc 1. undertaking b. pursuit 2. submerge c. engulf 3. calamity (a) a-3, b-1, c-2 (b) a-1, b-3, c-2 (c) a-2, b-3, c-1 (d) a-2, b-1, c-3 SSC CPO-SI – 11/12/2019 (Shift-I) Ans : (a) Correct matching for the above question a-3, b-1, c-2. Hence option (a) would be correct. 238. What problem are the farmers of Sehore District in MP facing? (a) They cannot make seeds from their crop. (b) The excessive rain has damaged the soyabean crop. (c) They cannot take their produce to the market. (d) The soyabean yield has diminished. SSC CPO-SI – 11/12/2019 (Shift-I) Ans : (b) The problem of the farmers is ‘The excessive rain has damaged the soyabean crop. Other options are not correct. 239. In Western MP where Sehore is situated what is the normal average rainfall from June to september? (a) 113.4 mm (b) 158 mm (c) 845.3 mm (d) 1,335.4 mm SSC CPO-SI – 11/12/2019 (Shift-I) Ans : (c) The normal average rainfall from June to September is 845.3mm. Hence option (c) would be correct. 240. Which of the following statements is NOT true according to the passage? (a) The three-acre plot of Kamal Singh had given nearly 16 quintals of soyabean in the past year. (b) In July-August the actual rainfall was 25.6% below the long-period average (c) From 1 June to 25 September, 58% more rainfall was recorded than the region's historical normal. (d) By September, the soyabean crop should have been ready for taking to Ashta mandi. SSC CPO-SI – 11/12/2019 (Shift-I) Ans : (b) According to the passage (b) is not a true statement. Other options are correct. Comprehension : Read the given passage and answer the questions that follow : For years, a long-lost chapter of what is said to be the words's first known novel has been quietly tucked away in a chest in a Japanese storeroom, hiding alongside other heirlooms of a Japanese family with feudal lineage. Experts confirmed that the blue-bound text is a chapter of the 11th Century Japanese classic 'Genji Monogatari' or 'The Tale of Genji'. Written by Japanese noblewoman Murasaki Shikibu, the novel depicts the romantic epic of Prince Genji and his love interest Muraski-no-ue. The recently found chapter contains an important part of the novel in which the 18 year old prince meets Murasaki-

751

YCT

no-ue, according to the Japan Times. The original 54 chapter text no longer exists. Transcriptions by Fujiwara no Teika (1162-1241), a Japanese poet, are believed to be the earliest existing manuscripts. In addition to the most recently discovered chapter, four other confirmed chapters of her transcription are registered as national cultural properties. Reizeike Shiguretei Bunko, a cultural preservation foundation, has authenticated the text as a chapter of Teika's trancriptions. In February, 72 year old Motofuyu Okochi found the little book in his family home in Tokyo and reached out to the foundation to have it authenticated. Okochi's lineage stretches back to the feudal era of Japan when, in 1743, the manuscript was passed to the Okochi family from another family, according to Asahi Shimbun. The experts who examined the found chapter were able to confirm that the text is one of Teika's transcriptions in a few ways. Not only did the handwriting match that of the other found chapters, but the cover of the binding matches the blue of the other texts. The small slip of paper that Teika used to identify the front of the book also matches the slips used on the other chapters. 241. Where was the chapter discovered ? (a) In a cultural foundation (b) In a royal palace (c) In a library in Tokyo (d) In a family storeroom SSC CPO-SI – 12/12/2019 (Shift-II) Ans : (d) According to the passage, the chapter discovered in a family storeroom. Hence option (d) is correct. 242. Which of the following was NOT one of the ways for confirming of the authenticity of the chapter ? (a) The handwriting matched that of the other chapters (b) The binding cover was of the same colour (c) The slip used by the manuscript writer was similar to the slips on other chapters (d) The name of the writer was inscribed on it as also on the earlier chapters SSC CPO-SI – 12/12/2019 (Shift-II) Ans : (d) According to the passage, option (d) was not one of the ways for confirming of the authenticity of the chapter. Other are not correct. 243. How many chapters of the novel have been listed as cultural properties ? (a) Five (b) Eighteen (c) Fifty-four (d) Eleven SSC CPO-SI – 12/12/2019 (Shift-II) Ans : (a) Five chapters of the novel have been listed as cultural properties. Hence option (a) would be correct. 244. Which of the following is NOT true according to the passage ? (a) The home where the book was kept safely belonged to a feudal family. (b) The Japanese novel is important because it is believed to be the oldest known novel. Reading Comprehension

(c) The man who found the chapter decided to hand it over to the cultural foundation. (d) The original manuscript written by Murasakino-ue was bound in blue. SSC CPO-SI – 12/12/2019 (Shift-II) Ans : (d) According to the passage, option (d) is not a true statement. Other options are correct. 245. Who was the author of the novel ? (a) Fujiwara no Teika (b) Motofuyu Okochi (c) Murasaki Shikibu (d) Rezeike Shiguretei Bunko SSC CPO-SI – 12/12/2019 (Shift-II) Ans : (c) According to the passage, Murasaki Shikibu was the author of the novel. Hence option (c) is correct. Comprehension : Read the given passage and answer to question that follow : Pollution befouls the air and poisons water. Pollution induces the release of toxicants into the biosphere which makes the air unsuitable for breathing, harms the quality of water and soil, and causes the emission of substances that may cause damage to humans, plants and animals. To cater to the needs of an increasing population, agriculture has been intensified through the use of a wide spectrum of fertilizers and pesticides. Diverse industries have been set up to produce chemicals including those that pose a danger to all life forms. Rapid industrialisation has led to deterioration in the quality of air, Widespread use of coal and fossil fuels in industries and petroleum fuel in motor vehicles has aggravated the air pollution problem. Our atmosphere seems to have become a waste basket into which dust, noxious fumes, toxic gases and other pollutants are callously thrown. The intensity of air pollution in Indian cities is increasing primarily due to our vintage vehicles and their poor performance. Water pollution, too has increased with the growth of our population and also that of our industries. Water pollution has acquired dangerous dimensions ever since sewage and induustrial effluents have started being disposed of into the rivers. Once considered sacred, the rivers are now turning murky and stink. It is sad that almost three-fourths of our fellow citizens have no choice but to drink filthy water. The severely polluted rivers due to mindless dumping of sewage and industrial wastes are a cause for concern not only to us humans but also o myriads of life forms that exist in water. On the French and italian rivieras we can no longer see the sparkling blue waters. The Mediterranean Sea is reported to be turning grey. Rivers and canals pour sewage, detergents and industrial waste into the sea, tankers flush their contents near the river or sea; bottles, rotting garbage and oil slicks are washed into the beaches. The phosphates and nitrates applied to farmlands as inorganic fertilizers, concentrate in lakes and estuaries causing algal blooms due to which wide expanses of water get choked, plants rot, oxygen is used up and fish die.

752

YCT

246. Select the most appropriate title for the passage. (a) We Breathe and Drink Poison (b) The Saga of Sacred Rivers (c) Algal Blooms (d) Impact of Industrialisation SSC CPO-SI – 12/12/2019 (Shift-I) Ans : (a) The most appropriate title for the passage is we breathe and Drink poison. Hence option (a) would be correct. 247. Since when has water pollution acquired dangerous dimensions? (a) Since sacred rivers became murky and lost their sanctily (b) Since the Mediterranean Sea started changing colour (c) Since sewage and industrial effluents started being disposed of in rivers (d) Since oil slicks started being washed on the shores of beaches SSC CPO-SI – 12/12/2019 (Shift-I) Ans : (c) According to the passage, option (c) is correct. Other options are not correct. 248. Which of the following factors is responsible for algal blooms? (a) Using up of oxygen dissolved in the water by algae (b) Farmlands being located near lakes and estuaries (c) Rotting of aquatic plants in lakes and estuaries (d) Concentration of inorganic fertilizers in lakes SSC CPO-SI – 12/12/2019 (Shift-I) Ans : (d) ‘Concentration of inorganic fertilizers in lake’ is responsible for algal blooms. Hence option (d) would be correct. 249. As per the passage, which of these statements is NOT true about air pollution ? (a) Use of coal and fossil fuels aggravates air pollution. (b) Vintage vehicles and their poor performance adds to pollution. (c) Three-fourths of our citizens are breathing poor quality air. (d) Dust and toxic gases released in the air cause air pollution. SSC CPO-SI – 12/12/2019 (Shift-I) Ans : (c) As per the passage, option (c) is not true about air pollution. Other option are correct. 250. Deterioration in the quality of air can mainly be attributed to (a) increase in population (b) rapid industrialisation (c) intensified agricultural practices (d) global climate change SSC CPO-SI – 12/12/2019 (Shift-I) Ans : (b) Deterioration in the quality of air can mainly be attributed to rapid industrialization. Hence option (b) would be correct. Reading Comprehension

Comprehension: Read the passage and answer the questions that follow: To the great delight of children, schools were being hurriedly closed. Children of all ages and sizes were running helter-skelter, screaming joyously, "No school, no school. Tiger, tiger!" They were shouting and laughing and even enjoyed being scared. They seemed to welcome me. I felt like joining them, and bounded away from the restaurant door and trotted along with them, at which they gleefully cried, "The tiger is coming to eat us; let us get back to the school!" I followed them through their school gate white they ran up and shut themselves in the school hall securely. I ascended the steps of the school, saw an open door at the far end of a veranda, and walked in. it happened to be the headmaster's room, I believed, as I noticed a very dignified man jumping on his table and heaving himself up into an attic. I walked in and flung myself on the cool floor, having a partiality for cool stone floor, with my head under the large desk - which gave me the feeling of being back in the Mempi cave … As I drowsed, I was aware of cautious steps and hushed voices all around. I was in no mood to bother about anything. All I wanted was a little moment of sleep; the daylight was dazzling. In half sleep I heard the doors of the room being shut and bolted and locked. I didn't care. I slept. While I slept a great deal of consultation was going on. I learnt about it later through my Master, who was in the crowd - the crowd which had gathered after making sure that I had been properly locked up - and was watching. The headmaster seems to have remarked some days later, "Never dreamt in my wildest mood that I'd have to yield place to a tiger". A wag had retorted, "Might be one way of maintaining better iscipline among the boys." 251. What did a mischievous person say about the headmaster? (a) He should keep the school closed for some time. (b) The school would be better managed by the tiger than by him. (c) He should leave the place and go because of his fear of the tiger. (d) He should allow the tiger to sleep peacefully. SSC CPO-SI – 13/12/2019 (Shift-II) Ans. (b) : A mischievous person said about the headmaster that the school would be better managed by the tiger that by him. Other option are not correct. 252. Where was the tiger just before the ran towards the school? (a) in a restaurant (b) in a cave (c) in the forest (d) At the zoo SSC CPO-SI – 13/12/2019 (Shift-II) Ans. (a) : According to the passage, option (a) will be the appropriate option other are not correct. 253. What was the tiger's reaction to the frenzy around him? (a) He just wanted to sleep. (b) He was afraid of being caught. (c) He looked for someone to attack. (d) He wanted to go back to his cave. SSC CPO-SI – 13/12/2019 (Shift-II)

753

YCT

Ans. (a) : According to the passage, option (a) would be the correct answer. Other options are not correct. 254. Which word describes the tone of the narative when the headmaster's reaction to the tiger's arrival is mentioned? (a) Threatening (b) Solemn (c) Intimate (d) Humorous SSC CPO-SI – 13/12/2019 (Shift-II) Ans. (d) : According to the passage option (d) would be correct. Other options are not correct. 255. What probably caused the greatest excitement among the children? (a) Watching the tiger run after them (b) Seeing a tiger getting into the school (c) Learning that the school would be closed (d) Knowing the tiger was coming to eat them SSC CPO-SI – 13/12/2019 (Shift-II) Ans. (c) : According to the passage, option (c) would be correct. Other options are not correct. Comprehension : Read the given passage and answer the questions that follow. It was the buzz of boardrooms, power lunches and anxious phone calls from the freeway. It was debated by stockbrokers, real estate agents. Hollywood producers and media bigfeet. Mid-level executives who wouldn't leave home without a phone in their pocket - or at their ear - were putting off calls or finding other ways to make them. Sales of cellular phones which had been growing at a sizzling 20% to 70% a year for the past decade were temporarily put on hold. Do cellular phones really cause brain tumours? The safety of the ultimate yuppie accessory was called into question by the news in the U.S. that two prominent executives had been stricken by brain cancer (though the connection to phone use is unclear) and by a well publicised lawsuit in which a Florida man charged that his wife's fatal brain tumour was caused by her cellular phone. It was not the kind of evidence that would be accepted by the new England Journal of Medicine, but it struck a nerve. American viewers tuned in to hear David Reynard, the Florida widower tell the story of his wife's death to Larry King, Bryant Gumbel, Faith Daniels and dozens of radio talk-show hosts. Even wall street took notice, knocking a couple of points off McCaw Cellular, Contel Cellular and Motorola the day after Reynard's appearance on the Larry King live show. The Cellular Telecommunications Industry Association was finally forced to respond, announcing that it would fund new studies and ask the government to review the findings. The phone flap is the latest in a series of scares linking everyday electrical objects (hair dryers, electric razors, electric blankets, home computers) to one dreaded disease or another. Despite the panic, the case against cellular phones is nowhere near as strong as the ones mounted against electric power lines, electric blankets or even hand police radars. Reading Comprehension

256. What was the result of the controversy over cellular phones? (a) The share value of cell phone companies fell in the stock market. (b) David Reynard won the lawsuit against companies. (c) The New England Journal of Medicine accepted that cell phones caused brain tumour. (d) The cellular, phones became more poular. SSC CPO-SI – 13/12/2019 (Shift-I) Ans. (a) : According to the sense of the passage, option (a) would be the correct answer. Other options are not correct. 257. Larry King is probably : (a) a wall street broker (b) an owner of a cellular phone company (c) a famous TV show host (d) a famous criminal lawyer SSC CPO-SI – 13/12/2019 (Shift-I) Ans. (c) : Larry king is probably famous TV show host. Hence option (c) is correct. 258. What was the buzz of boardrooms and the issue for debate among stockbrokers, real estate agents, producers and others? (a) The safety of all electrical gadgets (b) The concern whether cellular phones caused brain tumours (c) Excitement about the popular talk shows on radio (d) Growing sales of cellular phones SSC CPO-SI – 13/12/2019 (Shift-I) Ans. (b) : According to the passage, option (b) would be the correct answer. Other option are not correct. 259. Which of the following statements is Not true? (a) Brain tumour is more probable in cell users than in those living near power lines. (b) David Reynard claimed that his wife's tumour was due to use of cellular phone. (c) The news on adverse effects of cell phones affected the sales. (d) mcCaw, Motorola and Contel are cellphone companies. SSC CPO-SI – 13/12/2019 (Shift-I) Ans. (a) : According to the passage, option (a) is not a true statement. Other option are true. 260. According to the passage which of these is Not linked to any dreaded disease? (a) Electric blanket (b) Electric razor (c) Hair dryer (d) Hair dyes SSC CPO-SI – 13/12/2019 (Shift-I) Ans. (d) : According to the passage, Hair dryer is not linked to any dreaded disease. Hence option (d) would be correct.

754

YCT

12.

CLOZE TEST TCS hewšve& hej DeeOeeefjle (Based On TCS Pattern) Chapterwise

Exam

Question No.

Years

1

CGL (Tier-1) CGL (Tier-2) CHSL (Tier-1) CHSL (Tier-2) Selection Post VII, VIII, XI SSC MTS SSC GD SSC CPO SI

150 105 245 110 10 145 15 165

(2017–2023)

Cloze test

Trend Analysis of Questions topicwise from CGL (Pre & Mains) CHSL (Pre & Mains) Selection Post VII, VIII, XI, SSC MTS, SSC GD & Other Exams (2017-2023)

Cloze Test

755

YCT

12. CLOZE TEST Comprehension : In the following passage some of the words have been deleted. Read the passage carefully and select the correct answeer for the given blank out of the four alternatives. Intriguingly, those who (1) .......... the psychobiotic diet reported they felt less stressed compared (2) ......... those who followed the control diet. There was also a direct correlation between how strictly (3) ............ followed the diet and their perceived stress levels, with those who (4) ........... more psychobiotic foods during the four-week period reporting the (5) ......... reduction in perceived stress levels. 1. Select the most appropriate option to fill in blank number 1. (a) believed (b) casted (c) followed (d) bellowed SSC MTS 10/05/2023 (Shift-I) Ans. (c) : For the above given blank space (1) 'followed' (Pursue) will be used because it gives suitable meaning. Thus, the correct answer is option (c). 2. Select the most appropriate option to fill in blank number 2. (a) of (b) with (c) on (d) by SSC MTS 10/05/2023 (Shift-I) Ans. (b) : According to the passage, the most appropriate option for the blank space (2) preposition 'with' will be used because compare takes preposition 'with'. Thus the correct answer is option (b). 3. Select the most appropriate option to fill in blank number 3. (a) nomads (b) spectators (c) animals (d) participants SSC MTS 10/05/2023 (Shift-I) Ans. (d) : According to the passage, the most appropriate option for the blank space (3) 'participants' will be used because it gives correct meaning of the sentence. Thus, the correct answer is option (d). 4. Select the most appropriate option to fill in blank number -4. (a) staked (b) raked (c) ate (d) eroded SSC MTS 10/05/2023 (Shift-I) Cloze Test

Ans. (c) : According to the sense of above passage, the appropriate option for the blank space 'ate' (V2) will be used because sentense is in past indifinite tense and it gives suitable meaning. Other words arestaked means- spike Raked means- collect Eroded means- Destroy Thus, the correct answer is option (c). 5. Select the most appropriate option to fill in the blank number 5. (a) greater (b) greatest (c) greatly (d) great SSC MTS 10/05/2023 (Shift-I) Ans. (b) : According to the passage, the most appropriate option for the blank space (5) 'greatest' (superlative degree) will be used because when we compare more than two persons or things with one another. It is the highest degree of the quality (superlative degree). Thus, the correct answer is option (b). Comprehension : In the following passage some of the words have been deleted. Read the passage carefully and select the correct answer for the given blank out of the four alternatives. Although the (1) ........ in the Supreme Court arise out of the judgments or orders made by the Subordinate Courts including the High Courts, but of late the Supreme Court has started entertaining (2) ......... in which interest of the public at large is involved and the Court can be moved by any (3) ......... or group of persons either by filing a Write Petition at the Filing Counter of the Court or by (4) .......... a letter to Hon'ble the Chief Justice of India highlighting the question of public importance for (5) ........ this jurisdiction. 6. Select the most appropriate option to fill in blank number 1. (a) arrangement (b) battles (c) punctuality (d) proceedings SSC MTS 08/05/2023 (Shift-III) Ans. (d) : According to the passage, the most appropriate option for the blank space (1) 'proceedings' will be used because it gives correct meaning of the sentence. Thus, the correct answer is option (d).

756

YCT

7.

Select the most appropriate option to fill in blank number 2. (a) jams (b) matters (c) tangents (d) means SSC MTS 08/05/2023 (Shift-III) Ans. (b) : According to the passage, the most appropriate answer for the blank space (2) 'matters' will be used. Meaning of other words are – Jams means – Marmalade. Tangents means – Peripheral Means – denoter Thus, the correct answer is option (b). 8. Select the most appropriate option to fill in blank number 3. (a) individual (b) personnel (c) species (d) fauna SSC MTS 08/05/2023 (Shift-III) Ans. (a) : For the above given blank space (3) 'Individual' will be used because sentence shows the selection of two things 'individual or groups' The correct sentence is – 'Any individual or group of persons' Thus, the correct answer is option (a) 9. Select the most appropriate option to fill in blank number 4. (a) addressing (b) Tagging (c) calculating (d) summoning SSC MTS 08/05/2023 (Shift-III) Ans. (a) : 'According to the passage the most appropriate option for the blank space (4) 'addressing' which means 'negotiating' will be used. Meaning of other words are – Tagging means – Attach Calculating means – shrewd Summoning means – Send for Thus, the correct answer is option (a). 10. Select the most appropriate option to fill in blank number 5. (a) Invoking (b) Joking (c) Halting (d) Rocking SSC MTS 08/05/2023 (Shift-III) Ans. (a) : According to the passage, the most appropriate option for blank space (5) 'Invoking' which means 'Entreat will be used and it gives correct meaning. Thus, the correct answer is option (a) Comprehension: In the following passage, some words have been deleted. Read the passage carefully and select the most appropriate option to fill in each blank. The white tiger is an offspring of Bengal tiger, which has the necessary gene for white colouring. It has beautiful black stripes on (1) _____ white coat. (2)______ species of animal has blue eyes and a pink nose. Cloze Test

A pure white tiger is totally white without any stripes, which is due to the presence of double recessive allele in the genetic code. It happens only in the Bengal tiger subspecies, and only one in 10,000 births can have it naturally. White tigers are rarely seen in the wild, and only twelve of them have been spotted in India (3)______ the last one-hundred years. Tigers have a lifespan of 10-15 years. They are (4) _______ animals, and are fond of hunting at night. Their diet consists of any animals that they can catch, and can eat up to 40 pounds of meat at one time. After such a big meal, they usually remain away for several days from food. They are slow runners, (5) ______ good swimmers. They are poor climbers and cannot climb a tree easily. They live in an area ranging from 10 to 30 square miles depending upon the availability of prey, water and shelter. Habitat loss, poaching and expansion of the human population have together contributed to their becoming endangered. They are widely hunted for their body-parts, which are used in traditional Chinese medicines. 11. Select the most appropriate option to fill in blank number 1. (a) the (b) of (c) an (d) a SSC Selection Posts XI-30/06/2023 (Shift-IV) Ans. (d) : For the above blank space (1) 'a' will be used because it gives appropriate meaning according to sense of the passage. Other options are not correct. 12. Select the most appropriate option to fill in blank number 2. (a) Them (b) Those (c) This (d) These SSC Selection Posts XI-30/06/2023 (Shift-IV) Ans. (c) : For the above blank space (2) 'This' will be used because it gives appropriate meaning. Other options give different meaning. 13. Select the most appropriate option to fill in blank number 3. (a) following (b) behind (c) in (d) next to SSC Selection Posts XI-30/06/2023 (Shift-IV) Ans. (c) : For the above blank space (3) 'in' will be used because it gives appropriate meaning according to sense of the sentence. Other options give different meaning. 14. Select the most appropriate option to fill in blank number 4. (a) solitary (b) unaccompanied (c) separate (d) loner SSC Selection Posts XI-30/06/2023 (Shift-IV)

757

YCT

Ans. (b) : For the above blank space (4) "of" will be Ans. (a) : For the above blank space (4) 'solitary' used because 'of' preposition is used with surface. (alone) will be used because it gives appropriate Other options are not correct. meaning. Other options give different meaning. Note : Surface of something. Unaccompanied - Single/Alone Separate - Distinct 20. Select the most appropriate option for blank number 5. Loner - Bachelor (a) a (b) by 15. Select the most appropriate option to fill in (c) the (d) from blank number 5. SSC Selection Posts XI-28/06/2023 (Shift-III) (a) nor (b) but Ans. (c) : For the above blank space (5) "the" will be (c) either (d) or used because to give definite sense, definite article 'the' SSC Selection Posts XI-30/06/2023 (Shift-IV) is used. Other options not suitable according to sense Ans. (b) : For the above space (2) 'but' will be used of the passasge. because it gives appropriate meaning according to Comprehension: sense of the passage. Other options are not correct. In the following passage, some words have been Comprehension: deleted. Read the passage carefully and In the following passage, some words have been select the most appropriate option to fill in each deleted. Read the passage carefully and select the blank. Power foods are foods that provide (1) most appropriate option to fill in each blank. _______ levels of nutrients like fibre, potassium The earliest kind of print technology was developed and minerals. With people becoming in China. This was a system of hand (1)_____. From increasingly health conscious today, a lot of AD 594 onwards, (2)______ in China were printed by fitness trainers encourage their clients to (3)______ paper, against the inked surface (4)______ include these foods in their daily diet to (2) woodblocks. As both sides of (5)______ thin, porous ________ muscle development. There are sheet could not be printed, the traditional Chinese various ways of incorporating power foods in ‘accordion book’ was folded and stitched at the side. your daily diet. Of course, the key to enjoying 16. Select the most appropriate option for blank power foods is proper preparation of these foods, the (3) _______ of season-fresh foods and number 1. identifying your choice of flavour among power (a) inking (b) reading foods. Some of the recommended power food (c) writing (d) printing combinations are those that are prepared in SSC Selection Posts XI-28/06/2023 (Shift-III) our kitchens on a regular basis. Take for Ans. (d) : For the above blank space (1) "printing" instance, the combination of chickpeas and will be used because in passage we talk about printing onions. This combination is a powerful source technology. Other options give different meaning. of iron which is required by the body to transport oxygen to its various parts. Iron (4) 17. Select the most appropriate option for blank ______ can lead to anaemia, fatigue, brain fog number 2. and tiredness. A study by the Journal of (a) book (b) books Agricultural and Food Chemistry says that (c) page (d) paper sulphur compounds in onion and garlic help in SSC Selection Posts XI-28/06/2023 (Shift-III) the absorption of iron and zinc from chickpeas. Ans. (b) : For the above blank space (2) "books" will The combination is a hit with teenagers who be used because it gives appropriate meaning need to be (5) _______ about getting iron in according to sense of the passage. Other options give their diet. A quick way to prepare this power different meaning. food is to make a chickpea salad with chopped onions, chaat masala and cilantro. 18. Select the most appropriate option for blank number 3. 21. Select the most appropriate option to fill in blank number 1. (a) rubbing (b) writing (a) prosperous (b) rich (c) rub (d) write (c) flourishing (d) thriving SSC Selection Posts XI-28/06/2023 (Shift-III) SSC Selection Posts XI-27/06/2023 (Shift-I) Ans. (a) : For the above blank space (3) "rubbing" Ans. (b) : for the above passage in blank space (1) will be used because 'VI+ing' (Rubbing) is used after 'rich' would be appropriate because it gives the suitable preposition (by). Other options give different meaning. 19. Select the most appropriate option for blank meaning of the sentence. Other options give different meanings. number 4. Prosperous – Felicitous, (a) in (b) of Flourishing – Healthy, (c) by (d) from SSC Selection Posts XI-28/06/2023 (Shift-III) Thriving – Endowed, Cloze Test

758

YCT

22.

Select the most appropriate option to fill in blank number 2. (a) increase (b) magnify (c) intensify (d) spike SSC Selection Posts XI-27/06/2023 (Shift-I) Ans. (a) : For the above passage in blank space (2) 'increase' would be appropriate which means 'to make some thing larger in number or amount.' Other options give different meanings – Magnify – Amplify, Intensify – Enlarge, spike – wedge, 23. Select the most appropriate option to fill in blank number 3. (a) wield (b) handle (c) apply (d) use SSC Selection Posts XI-27/06/2023 (Shift-I) Ans. (d) : for the above passage in blank space (3) 'use' would be appropriate. Other options give different meanings – Wield – Stirred, Handle – Maintain, Apply – Enforce, 24. Select the most appropriate option to fill in blank number 4. (a) drought (b) lack (c) deficiency (d) inadequacy SSC Selection Posts XI-27/06/2023 (Shift-I) Ans. (c) : For the above passage in blank space (4) 'deficiency' would by appropriate, which means 'a falt of weakness in somebody; Other options give different meanings – Drought – Aridity Lack – Scarcity, Inadequacy – Insufficience, 25. Select the most appropriate option to fill in blank number 5. (a) immersed (b) swamped (c) diligent (d) vibrant SSC Selection Posts XI-27/06/2023 (Shift-I) Ans. (c) : For the above passage in blank space (5) 'diligent' would be appropriate. Other options give different meanings – immersed – Engrossed, swamped – Slobed, vibrant – Doddering Comprehension : In the following passage some of the words have been deleted. Read the passage carefully and select the correct answer for the given blank out of the four alternatives. "I am proud to (1) .......... to a religion which has taught the word both tolerance (2) ......... universal acceptance," he declared (3) ......... Cloze Test

September 11, 1893. "We believe not only in universal toleration, (4) ......... we accept all religions as true. I am proud to belong to a nation which has sheltered the persecuted and the (5) ......... of all religious and all nations of the earth." 26. Select the most appropriate option to fill in blank number 1. (a) behest (b) gain (c) belong (d) live SSC MTS 04/05/2023 (Shift-II) Ans. (c) : According to the passage, the appropriate option for the blank space (1) 'belong' will be used because it gives correct meaning of the sentence. Thus, the correct answer is option (c). 27. Select the most appropriate option to fill in blank number 2. (a) also (b) and (c) as well as (d) or SSC MTS 04/05/2023 (Shift-II) Ans. (b) : According to the passage, the most appropriate option for the blank space (2) conjunction 'and' will be used pair cojnuction both ...... and Thus, the correct answer is option (b). 28. Select the most appropriate option to fill in blank number 3. (a) on (b) upon (c) in (d) at SSC MTS 04/05/2023 (Shift-II) Ans. (a) : According to the passage, the most appropriate option for the blank space (3) preposition 'on' will be used because before date preposition. 'on' will be used. Thus, the correct answer is option (a). 29. Select the most appropriate option to fill in the blank number 4. (a) yet (b) though (c) and (d) but SSC MTS 04/05/2023 (Shift-II) Ans. (d) : According to the passage, the most appropriate option for the blank space (4) 'but' because conjunctin 'not only ___ but also' pair will be used. Thus, the correct answer is option (d). 30. Select the most appropriate option to fill in blank number 5. (a) workers (b) refugees (c) peons (d) toilers SSC MTS 04/05/2023 (Shift-II) Ans. (b) : According to passage the most appropriate option for the blank space (5) refugees' (dependent) will be used because it gives correct meaning for the sentence. Thus, the correct answer is option (b).

759

YCT

Comprehension : In the following passage some words have been deleted. Fill in the blanks with the help of the alternatives given. Select the most appropriate option for each number. Salvador Dali (1)................ a prominent Sapnish surrealist painter (2) ............... in Figureres, Catalonia, on 11 May 1904. Dali's father was strict in the eudcation of his children unlike (3) .............. moter. Dali had a brother named Salvador (4) ............. was born nine months before him and died of gastroenteritis. When he was five, Dali was taken to his brother's grave and was told that he was his brother's (5) ................ 31. Select the most appropriate option to fill in the blank No. 1 (a) has (b) is (c) was (d) were SSC MTS 02/05/2023 (Shift-I) Ans. (c) : According to the passage, the most appropriate option for the blank space (1) 'was' will be used because sentence in past tense, and 'salvador Dali' is a singular noun so singular verb of simple past tense is used. Thus, the correct answer is option (c). 32. Select the most appropriate option to fill in the blank No. 2. (a) bought (b) rooted (c) born (d) brought SSC MTS 02/05/2023 (Shift-I) Ans. (c) : According to the passage, the most appropriate option for the blank space (2) 'born' will be used because it gives condusive meaning of the sentence. Thus, the correct answer is option (c). 33. Select the most appropriate option to fill in the blank No. 3. (a) her (b) him (c) their (d) his SSC MTS 02/05/2023 (Shift-I) Ans. (d) : According to the passage, the most appropriate option for the blank space (3) adjectives 'his' will be used because of 'Dali's father' and adjective 'his' will be used before noun (moter). Thus, the correct answer is option (d). 34. Select the most appropriate option to fill in the blank No. 4. (a) who (b) whose (c) that (d) whom SSC MTS 02/05/2023 (Shift-I) Ans. (a) : According to the passage the most appropriate option for the blank space (4) relative pronoun 'who' will be used because it comes for salvador (person). Thus, the correct answer is option (a). Cloze Test

35.

Select the most appropriate option to fill in the blank No. 5 (a) ordination (b) detonation (c) reincarnation (d) stagnation SSC MTS 02/05/2023 (Shift-I) Ans. (c) : According to the passage, the most appropriate option for the blank space (5) 'reincarnation' will be used because it gives appropriate meaning of the sentence. Other words are ____ Ordination means- Sacrament Detonation means- Explosion Stagnation means- Stability Thus, the correct answer is option (c). Comprehension: In the following passage, some words have been deleted. Read the passage carefully and select the most appropriate option to fill ineach blank. The ongoing fight for social justice and equality continues to be a significant global 1. _____________. Movements advocating forracial, gender and LGBTQ+ equality, as well as addressing systemic discrimination and injustice, have 2. _____________ momentumin recent years. People are increasingly recognising the importance of dismantling structures that 3. ________________ inequality and fostering inclusivity in all aspects of society. Calls for justice, representation and equal opportunities have sparked conversationsand initiatives aimed at creating a more equitable world. It is crucial to 4. __________ in meaningful dialogue, challenge biases andactively promote diversity and to build a society where everyone has equal rights and opportunities. 36. Select the most appropriate option to fill in blank no. 1. (a) drift (b) focus (c) divergence (d) dispersion SSC CGL Mains 26/10/2023 (Shift-I) Ans. (b) : For the above blank space (1) 'Focus' will be used because it gives suitable meaning according to the passage. Other options give different meaning. Drift Flow Divergence Deviation, meving Dispersion Spread 37. Select the most appropriate option to fill in blank no. 2. (a) gained (b) provided (c) scattered (d) invoked SSC CGL Mains 26/10/2023 (Shift-I) Ans. (a) : For the above blank space (2) 'gained' will be used because it gives appropriate meaning. Other options give different meaning. Provided Ready/Handy Scattered Expanded Invoked Call

760

YCT

38.

Select the most appropriate option to fill in blank no. 3. (a) refract (b) cease (c) perpetuate (d) terminate SSC CGL Mains 26/10/2023 (Shift-I) Ans. (c) : For the above blank space (3) 'perpetuate' will be used because it gives appropriate meaning according to sense of the passage. Other options give different meaning. Refract Incline Cease Stop Terminate End, Finish 39. Select the most appropriate option to fill in blank no. 4. (a) engage (b) coincide (c) evolve (d) manifest SSC CGL Mains 26/10/2023 (Shift-I) Ans. (a) : For the above blank space (4) engage will be used because it gives appropriate meaning. Other options give different meaning. Coincide Match Evolve Build Manifest Revealed Comprehension: In the following passage some of the words have been deleted. Read the passage carefully and select the correct answer for the given blank out of the four alternatives. You may wonder if anyone would (1) ______ to be diagnosed before they are even aware they have Alzheimer’s (2) ______? A recent survey shows that, indeed, they would. (3) ______ three-quarters of the respondents said they would want (4) ______ if they had Alzheimer’s disease, even if they were not yet experiencing (5) ______. 40. Select the most appropriate option to fill in blank number 1. (a) want (b) surrender (c) got (d) drop SSC MTS 19/05/2023 (Shift-III) Ans. (a) : According to the sense of above passage in blank space (1) ‘want’ would be appropriate. Other options give different meanings – Surrender – Committal, Got – Obtained, Drop – Blob, 41. Select the most appropriate option to fill in blank number 2. (a) disease (b) health (c) contamination (d) bug SSC MTS 19/05/2023 (Shift-III) Cloze Test

Ans. (a) : For the above passage in blank space (2) ‘disease’ (noun) would be appropriate because Disease come here for the ‘Alzheimer’s’. Other options give different meanings – Contamination – Corruption, Bug – Guilt, Fault, 42. Select the most appropriate option to fill in blank number 3. (a) Casually (b) Totally (c) Nearly (d) Firmly SSC MTS 19/05/2023 (Shift-III) Ans. (c) : According to the sense of the above passage in blank space (3) ‘nearly’ would be appropriate. Meaning of other optionsCasually – By chance, Totally – Altogether, Firmly – Sturdily, Thus, the correct option is (c). 43. Select the most appropriate option to fill in blank number 4. (a) known (b) knowing (c) knows (d) to know SSC MTS 19/05/2023 (Shift-III) Ans. (d) : According to the sense of the above passage in blank space (4) ‘to know’ would be appropriate because infinitive (to + V1) is used after the verbI ‘want’. It gives the appropriate meaning of this sentence. Other options are different form of verb. 44. Select the most appropriate option to fill in blank number 5. (a) indexes (b) symptoms (c) evidences (d) marks SSC MTS 19/05/2023 (Shift-III) Ans. (b) : According to the sense of the above passage in blank space (5) ‘symptoms’ would be appropriate, which means – ‘a change in your body that is a sign of illness’. Meaning of other options – Indexes – Catalog, Evidences – Witness, Marks – Numbers, Thus, the correct answer is option (b). Comprehension : In the following passage some words have been deleted. Fill in the blanks with the help of the alternatives given. Select the most appropriate option for each number. By the (1) __________ we reach puberty, the world has reached us and (2) __________ us to a greater extent than we realize. (3) __________ family, friends, school, and society in general (4) __________ by word and example-how to be. But people (5) __________ leaders at that moment when they decide for themselves how to be.

761

YCT

45.

Select the most appropriate option to fill in blank number 1. (a) age (b) time (c) era (d) moment SSC MTS 18/05/2023 (Shift-I) Ans. (b) : According to the sense of the above passage in blank space (1) ‘time’ would be appropriate because ‘By the time’ is prepositional phrase that is followed by a clause, refer to something has already happened at the time that something else happens. 46. Select the most appropriate option to fill in blank number 2. (a) formats (b) contours (c) shapes (d) shaped SSC MTS 18/05/2023 (Shift-I) Ans. (d) : In the above passage, in blank space (2) ‘Shaped’ would be appropriate because the given sentence is in ‘Present perfect tense, therefore, ‘Verb III’ will be used. Other options give different meanings. 47. Select the most appropriate option to fill in blank number 3. (a) Their (b) Our (c) Her (d) There SSC MTS 18/05/2023 (Shift-I) Ans. (b) : According to the sense of the above passage in blank space (3) ‘Our’ would be appropriate because possessive pronoun ‘our’ is used before the noun as a subject. Other options give different meanings. 48. Select the most appropriate option to fill in blank number 4. (a) have tell us (b) has told us (c) have told us (d) have been told us SSC MTS 18/05/2023 (Shift-I) Ans. (c) : For the above sentence in blank space (4) ‘have told us’ would be appropriate because the sentence is present perfect tense and the subject is plural, therefore, the plural verb is used (have told us). Present Perfect tense– Sub. + has/have + VIII + other word. 49.

Select the most appropriate option to fill in blank number 5. (a) begin to become (b) onset to become (c) start to became (d) began to become SSC MTS 18/05/2023 (Shift-I) Ans. (a) : According to the sense of the above passage in blank space (5) ‘begin to become’ is used because the sentence is ‘Simple present’, the VI (begin to become) is used. Other options give different meanings. Thus, the correct option is (a).

Cloze Test

Comprehension: In the following passage some of the words have been deleted. Read the passage carefully and select the correct answer for the given blank our of the four alternatives. After a rejection, we tend to beat ourselves up over the things that might have (1) ______ us to be rejected ______ and might even end up dwelling on these negative (2) ______, a process called rumination. This habit, however, inevitably causes us to (3) ______ worse. "The first thing a lot of people do when they get rejected is to be unkind to (4) ______, and they start coming up with all kind of ideas about what's (5) ______ with them, "Gottlieb notes. 50. Select the most appropriate option to fill in the blank No. 1. (a) let (b) led (c) leading (d) lead SSC MTS 16/05/2023 (Shift-II) Ans. (b) : According to the sence of the above sentence in blank space (1) 'led' would be appropriate because it is the past form of the verb 'lead'. other options give different meanings. 51. Select the most appropriate option to fill in the blank No. 2. (a) emotions (b) endurance (c) stoppages (d) formalities SSC MTS 16/05/2023 (Shift-II) Ans. (a) : According to the sense of the above sentence in blank space (2) 'emotions' (Noun) would be appropriate because it is given the proper meaning of the sentence. Thus, the correct answer is option (a). 52. Select the most appropriate option to fill in the blank No. 3. (a) recognize (b) share (c) swirl (d) feel SSC MTS 16/05/2023 (Shift-II) Ans. (d) : For the above given passage in blank space (3) 'Feel' would be appropriate because according to the sense (to + v1) is used. other options give different meaningRecognize- Distinguish Swirel- Rotate 53. Select the most appropriate option to fill in the blank No. 4. (a) himself (b) themselves (c) itself (d) yourself SSC MTS 16/05/2023 (Shift-II) Ans. (b) : According to the sense of the above passage in blank space (4) 'themselves' would be appropriate because the reflexive pronoun 'them selves' is used as an object of the sentence is the same as the subject. other options give different meanings.

762

YCT

54.

Select the most appropriate option to fill in the blank No. 5. (a) wrong (b) strong (c) syncing (d) taming SSC MTS 16/05/2023 (Shift-II) Ans. (a) : According to the sense of the above passage in blank space (5) 'wrong' would be appropriate. other options give different meanings. Strong- Firm, Energetic Syncing- Coexist, Concur Taming- Docile, Subduing Comprehension: In the following passage some of the words have been deleted. Read the passage carefully and select the correct answer for the given blank out of the four alternatives. With Halloween only (1) ______ days away, I thought this was a perfect time to talk (2) ______ Zombies. For years, zombies have dominated science (3) ______. These creatures, the walking dead, don’t (4) ______ exist, right? Wrong! There are in fact several very real life diseases that could and can make you (5) ______ like a zombie. 55. Select the most appropriate option to fill in blank number 1. (a) a little (b) a few (c) much (d) little SSC MTS 15/05/2023 (Shift-I) Ans. ( b) : According to the sense of the above passage in blank space (1) 'a few' would be appropriate because of is used with a countable noun (days). Note → A little → It is used with a uncountable noun. Thus, the correct answer is option (b). 56. Select the most appropriate option to fill in blank number 2. (a) about (b) for (c) by (d) of SSC MTS 15/05/2023 (Shift-I) Ans. (a) : Blank space (2) of the above passage option 'about' would be appropriate because 'talk about zombies' gave the proper meaning in this sentence. Thus, the correct answer is option (a). 57. Select the most appropriate option to fill in blank number 3. (a) freelance (b) friction (c) fickle (d) fictions SSC MTS 15/05/2023 (Shift-I) Ans. (d) : According to the sense of the above passage in blank space (3) 'fictions' (Novel) would be appropriate because 'Science fictions' is gives proper meaning for the sentence. The other options give different meanings. Cloze Test

58.

Select the most appropriate option to fill in blank number 4. (a) manually (b) suddenly (c) gladly (d) actually SSC MTS 15/05/2023 (Shift-I) Ans. (d) : According to the sense of the above passage in blank space (4) 'actually' (adv.) would be appropriate. Other options give different meanings. 59. Select the most appropriate option to fill in blank number 5. (a) act (b) slake (c) run (d) state SSC MTS 15/05/2023 (Shift-I) Ans. (a) : According to the sense of the above given passage in blank space (5) 'act' (Perform) would be appropriate. Other options give different meanings. Comprehension: In the following passage some of the words have been deleted. Read the passage carefully and select the correct answer for the given blank out of the four alternatives. Network security (1) ______ securing a network and protecting the user’s information about (2) ______ is connected through that network. Over the network (3) ______ steal, the packets of data through sniffing and spoofing (4) ______, man in the middle attack, war driving, etc, and misuse the data for their (5) ______. 60. Select the most appropriate option to fill in blank number 1. (a) declares (b) calls (c) means (d) names SSC MTS 12/05/2023 (Shift-III) Ans. (c) : According to the sense of the above passage in blank space (1) 'means' would be appropriate because 'means' describe Further information about network security. 61. Select the most appropriate option to fill in blank number 2. (a) whose (b) which (c) who (d) that SSC MTS 12/05/2023 (Shift-III) Ans. (c) : In blank space of the above passage (2) 'who' would be appropriate because it is used as a relative pronoun for a person in subjective case. Hence, the correct option is (c) 62. Select the most appropriate option to fill in blank number 3. (a) engineers (b) labourers (c) hackers (d) smugglers SSC MTS 12/05/2023 (Shift-III)

763

YCT

Ans. (c) : According to the sense of the above passage in blank space (3) 'hackers' would be appropriate word. Meaning of other options – laboures – Workers, Smugglers –Thief's, 63. Select the most appropriate option to fill in blank number 4. (a) attacks (b) guards (c) charges (d) raids SSC MTS 12/05/2023 (Shift-III) Ans. (a) : According to the sense of the above passage in blank space (4) 'attacks' would be appropriate word. Meaning of other options– Guards – Pawl, Charges – Expenses, Raids – Invasion 64. Select the most appropriate option to fill in blank number 5. (a) benefits (b) pleasure (c) hindrances (d) banes SSC MTS 12/05/2023 (Shift-III) Ans. (a) : According to the sense of the above passage in blank space (5) 'benefits' would be appropriate because 'Misuse the date for their benefits' give the proper meaning. Meaning of other options– Pleasure – Gaiety, Hindrances – Obstacle, Banes – Rebuke, Comprehensin: In the following passage, some words have even deleted. Read the passage carefully and select the most appropriate option to fill in each blank. The automobile industry is one of the (1) _______ industries at the global level. It plays a crucial role in the development of the global economy because of the high revenues and increased customer demands. The automobile industry helps to (2)_________ economic development of the country; therefore, it is widely recognized as a major economic sector. The automobile industry consists of multiple companies (3)__________ in car manufacturing, as well as the ones involved in marketing and distribution of automobile products, such as cars, buses, vans, trucks, motorcycles, mopeds and (4)_______ bicycles. The global automobile industry incorporates several large car manufacturers' blocs that work (5)______ at the global level. 65. Select the most appropriate option to fill in blank number 1. (a) Jumping (b) Leading (c) Shortening (d) Moving SSC GD 31/01/2023 (Shift-II) Cloze Test

Ans. (b) : In the blank space (1) of the given passage, 'leading' (Prime, main) will be used because according to the sense of the passage it is giving proper meaning. Other options give different meaning. Jumping - Leaping Shortening - Lacking Moving - Dynmic Correct SentenceThe automobile industry is one of the leading industries at the global level. 66. Select the most appropriate option to fill in blank number 2. (a) Release (b) Block (c) Foster (d) Arrest SSC GD 31/01/2023 (Shift-II) Ans. (c) : In the blank space (2) of the given passage, 'Foster' (promote, encourage) will be used because according to the sense of the passage it is giving proper meaning. Other options give different meaning. Release - Discharge Block - Barricade, Interrupt Arrest - Apprehend, Seize Correct SentenceThe automobile industry help to foster economic development of the country. 67. Select the most appropriate option to fill in blank number 3. (a) Unspecialised (b) Specialised (c) Simplified (d) Non-simplified SSC GD 31/01/2023 (Shift-II) Ans. (b) : In the blank space (3) of the given passage 'Specialised' (particularised) will be used because according to the sense of the passage it is giving proper meaning. Other option give different meaning. Unspecialised - Generalised Simplified - Clarified Non-Simplified - Complicated Correct SentenceThe automobile industry consists of multiple companies specialised in car manufacturing, as well as the ones involved in marketing. 68. Select the most appropriate option to fill in blank number 4. (a) fuel-based (b) motorised (c) Non-motorised (d) Non-fuel SSC GD 31/01/2023 (Shift-II) Ans. (b) : In the blank space (4) of the given passage 'motorised' (Self-operating) will be used because according to the sense of the passage it is giving proper meaning. Other options give different meaning. Correct Sentence......distribution of automobile products, such as cars, buses, vans, trucks, motorcycles, mopeds and motorisid bicycles.

764

YCT

69.

Select the most appropriate option to fill in blank number 5. (a) Hurriedly (b) Smoothly (c) Collaboratively (d) Single-handedly SSC GD 31/01/2023 (Shift-II) Ans. (c) : In the blank space (5) the given passage 'Collaboratively' (Cooperatively) will be used because according to the sense of the passage it is giving proper meaning. Other options give different meaning. Hurriedly - Rapidly Smoothly - Easily, effortlessly Single-handedly - Individually, spearately Correct SentenceThe global automobile industry incorporates several large car manufacurer's blocs that work Collaboratively at the global level. Comprehension: In the following passage, some words have been deleted. Read the passage carefully and select the most appropriate option to fill in each blank. Reaching the stall from which the sound was emanating, he saw that it was deserted. It was festooned (1)________ flowers: blue-bells and foxgloves and harebells and daffodils, but also with violets and lilies. The use and function of these flowers is chiefly (2)_______; they bring pleasure; they can be given to a loved one as a token of admiration and affection, and the sound they make is pleasing (3) ________ the ear. Also, they catch the light (4) ________ delightfully. Dustan could not but observe that the colour of sunlight glittering through the purple crystal was inferior in both (5) ________to that of her eyes. 70. Select the most appropriate option to fill in blank number 1. (a) On (b) With (c) by (d) Upon SSC GD 23/01/2023 (Shift-IV) Ans. (b) : In the blank space (1) of the given passage, option (a) 'with' will be used because it is giving proper meaning according to the sense of the passage. Hence option(b) is correct answer. Correct SentenceIt was festooned with flowers. 71. Select the most appropriate option to fill in blank number 2. (a) Admiration and affection (b) Decorative and recreational (c) Glittering (d) Repellent and frivolous SSC GD 23/01/2023 (Shift-IV) Cloze Test

Ans. (b) : In the blank space (2) of the given passage option(b) 'Decorative and recreational' will be used because it is giving proper meaning according to the sense of the passage. Other options give different meanings. Hence option (b) is correct answer. Correct SentenceThe use and function of these flowers is Chiefly decorative and recreational. 72. Select the most appropriate option to fill in blank number 3. (a) upon (b) for (c) to (d) as far as SSC GD 23/01/2023 (Shift-IV) Ans. (c) : In the blank space (3) of the given passage preposition 'to' will be used because the preposition 'to' is used after pleasing. Hence option(c) is correct answer. Correct Sentence..........and the sound they make is pleasing to the ear. 73. Select the most appropriate option to fill in blank number 4. (a) so (b) much (c) such (d) more SSC GD 23/01/2023 (Shift-IV) Ans. (a) : In the blank space (4) of the given passage option (a) 'So' will be used because it is giving proper meaning according to the sense of the passage. Hence option (a) is correct answer. Correct SentenceAlso, they catch the light so delightfully. 74. Select the most appropriate option to fill in blank number 5. (a) Aroma and fragrance (b) Shape and size (c) Texture and touch (d) Hue and shade SSC GD 23/01/2023 (Shift-IV) Ans. (d) : In the blank space (5) of the given passage option (d) 'Hue and shade' will be used because it is giving proper meaning according to the sense of the passage. Hence option (d) is correct answer. Correct Sentence........the purple crystal was inferier in both hue and shade to that of eyes. Comprehension: In the following passage, some words have been deleted. Read the passage carefully and select the most appropriate option to fill in each blank. Maybe you’re bored of bananas, apples and grapes and need a fresh produce pick? A nutrient-rich serving of kiwi fruit may be just what you need. A serving of kiwi fruit (2 kiwis) has twice the vitamin C of an orange, (1)______ potassium as a banana and the fibre of a bowl of whole grain cereal—all for less than 100 calories! The fuzzy fruit is sky-high in both

765

YCT

soluble and insoluble fibre, both of which are essential for promoting heart health, (2) _________ digestion and lowering cholesterol levels—that’s a winning trifecta. Kiwi fruit has also been considered a ‘nutritional all-star,’ as Rutgers University researchers found that kiwi fruit has the best nutrient density of 21 commonly consumed fruits. Along with vitamin C, kiwi fruits are rich in many bioactive compounds that have antioxidant capacity to help protect against free radicals, harmful by-products produced in the body. If you want clean energy, think of kiwi fruit because they’re rich in magnesium, a nutrient essential to (3) ________ food into energy. A kiwi fruit also doubles as a peeper-keeper by supplying your eyes with protective lutein, a carotenoid that’s concentrated in eye tissues and helps protect against harmful free radicals. Kiwi fruit is also (4) ______ with blood pressure-lowering potassium. In fact, a 100gram serving of kiwi fruit—that’s about one large kiwi—provides 15% of the Recommended Daily Allowance (RDA) of potassium. Kiwi fruit has been growing in New Zealand for over 100 years. Once the fruit gained in popularity, other countries started to grow them including Italy, France, Chile, Japan, South Korea and Spain. At first, kiwis were referred to as ‘Yang Tao’ or ‘Chinese Gooseberry,’ but the name was (5) _______ changed to kiwi fruit so that everyone would know where the fruit came from. 75. Select the most appropriate option to fill in blank number 1. (a) lot of (b) as much (c) as much as (d) much of SSC GD 16/01/2023 (Shift-III) Ans. (b) : Option (b) 'as much' will be used in the blank space (1) of the above passage because as much is used to say that an amount is as large as another amount. Correct sentence- A serving of Kiwi fruit has twice the vitamin C of an orange, as much potassium..... 76. Select the most appropriate option to fill in blank number 2. (a) Regulating (b) Repressing (c) Taming (d) Constraining SSC GD 16/01/2023 (Shift-III) Ans. (a) : Option (a) regulating (control) will be used in the blank space (2) of the above passage because it is giving proper meaning according to the sense of the sentence. Other option give different meanings. Repressing Suppressing Taming Subjugating Constraining Compel, Coerce Correct SentenceBoth of which are essential for promoting heart health, regulating digestion........ Cloze Test

77.

Select the most appropriate option to fill in blank number 3. (a) Vary (b) Revise (c) Convert (d) Displace SSC GD 16/01/2023 (Shift-III) Ans. (c) : 'Convert (Change) will be used in the blank space (3) of the passage because it is giving proper meaning according to the sense of the passage. Other options give different meaningsVary Differ Revise Repeat Displace Shift Correct SentenceA nutrient essential to Convert food into energy. 78. Select the most appropriate option to fill in blank number 4. (a) Burdened (b) Condensed (c) Overstuffed (d) Packed SSC GD 16/01/2023 (Shift-III) Ans. (d) : 'Packed (Filled)' will be used in the blank space (4) of the given passage because it is giving proper meaning according to the sense of the passage. Other options give different meanings. Burdened Loaded Condensed Liquidize Overstuffed Overfull Correct SentenceKiwi fruit is also packed with blood pressure lowering potassium. 79. Select the most appropriate option to fill in blank number 5. (a) Anon (b) Presently (c) Ultimately (d) Directly SSC GD 16/01/2023 (Shift-III) Ans. (c) : Ultimately (Finally) will be used in the blank space (5) of the given passage because it is giving proper meaning according to the sense of the passage. Other options give different meanings. Anon - Soon, Early Presently - Now, Soon Directly - Straight Correct Sentencebut the name was ultimately changed...... Comprehension: In the following passage, some words have been deleted. Read the passage carefully and select the most appropriate option to fill in each blank. Notebook software is included (1) ______ the SMART Board interactive education. Professors and students can write on the SMART Board screen suing a digital pen or (3) _____ finger if they have a SMART Board. Notebook files a may be saved in various formats, printed, emailed, or posted, providing students with a total (4) _____ to course materials. Essentially, having a SMART Board available may (5) ______ everything that can be done on a computer.

766

YCT

80.

Select the most appropriate option to fill in blank no. 1. (a) Upon (b) To (c) Along (d) With SSC GD 12/01/2023 (Shift-II) Ans. (d) : In the blank space (1) of the above passage, preposition with will be used because it is giving proper meaning according to the sense of the passage. Correct sentenceNotebook software is included with the SMART board interactive education. 81. Select the most appropriate option to fill in blank no. 2. (a) Total (b) Limit (c) Number (d) Design SSC GD 12/01/2023 (Shift-II) Ans. (c) : In the blank space (2) of the above passage option (c) Number will be used because it is giving proper meaning according to the sense of the passage. Total- Whole, entire Limit - Restriction, Curb Design - Structure, Formation Correct SentenceThis programme includes a number of tools for different aducation. 82. Select the most appropriate option to fill in blank no. 3. (a) Their (b) There (c) Those (d) These SSC GD 12/01/2023 (Shift-II) Ans. (a) : In the blank space (3) of the above passage option (a) Their will be used because it is giving proper meaning according to the sense of the passage. Correct sentence- .........the SMART Board screen using a digital pen or their finger. 83. Select the most appropriate option to fill in blank no. 4. (a) Access (b) Validity (c) Download (d) Exception SSC GD 12/01/2023 (Shift-II) Ans. (a) : In the blank space (4) of the above passage, option (a) 'Access' (approach) will be used because it is giving proper meaning according to the sense of the passage. Validity - Legitinacy, Download - Copy, take Exception - Abnormality, special case Correct sentence...........or posted, providing student with a total access to course materials. 84. Select the most appropriate option to fill in blank no 5. (a) Form (b) Enlighten (c) Create (d) Enhance SSC GD 12/01/2023 (Shift-II) Cloze Test

Ans. (d) : In the blank space (5) of the above passage option (d) 'enhance' will be used because it is giving proper meaning according to the passage. Other option give different meaning. Form- Shap Enlighten - Inform, intimate Create - Build Correct SentenceEssentially, having a SMART Board available may enhance everything that can be done on a computer. Comprehension: In the following passage, some words have been deleted. Read the passage carefully and select the most appropriate option to fill in each blank. The companies in the present era (1) _______ priority, furring recruitment, to the candidates who have Emotional intelligence. El is the ability to understand, analyse and control emotions. Possessing very high El creates a (2) ______ impact on a person's can be nice even in socially awkward situations. As they are very good at controlling emotions, they make others feel (3) ______. It is proven that it will make leaders more effective as they understand the feelings of teammates. It allows one (4) ______ empathy towards others, they can be very good at interpersonal relationship. These people strengthen the relationship (5) ______ the people with their interaction and coordination both in personal and professional life. Companies are well aware of the benefits with the employees who have El. 85. Select the most appropriate option to fill in blank number 1. (a) Has been giving (b) Have been given (c) Are giving (d) Is giving SSC GD 10/01/2023 (Shift-I) Ans. (c) : In the blank space (1) of the above passage, 'are giving' will be used because 'present era' has been used in the given sentence. hence it is clear that the sentence will be in present tense and the subject of the sentence (the companies) is plural. hence the verb will also be used plural. Correct sentenceThe companies in the present era are giving priotity. 86. Select the most appropriate option to fill in blank number 2. (a) Creative (b) Positive (c) Colonial (d) Memorable SSC GD 10/01/2023 (Shift-I) Ans. (b) : In the blank space (2) of the above passage. option (b) 'positive' will be used because it is giving proper meaning according to the sense of the passage. Other options give different meaning.

767

YCT

Creative - Innovative Colonial - Immigrant Memorable - Unforgettable, noteworthy Correct sentencePossessing very high EI creates a positive impact on a person's. 87. Select the most appropriate option to fill in blank number 3. (a) Comfortable (b) Tolerable (c) Favourable (d) Manageable SSC GD 10/01/2023 (Shift-I) Ans. (a) : In the blank space (3) of the above passage, option (a) 'Comfortable' will be used because it is giving proper meaning according to the sense of the passage. Other options give different meaning. Tolerable - Bearable, Favourable - Congenial Manageable - Organizable Correct sentence.....they make others feel comfortable. 88. Select the most appropriate option to fill in blank number 4. (a) Communicate (b) To communicating (c) To be communicated (d) To communicate SSC GD 10/01/2023 (Shift-I) Ans. (d) : 'To communicate' will be used in the blank space (4) of the above passage because the first form of the verb (v1) is used after the infinitive to'. Hence option (d) is correct answer. Correct sentenceIt allows one to communicate empathy towards others. 89. Select the most appropriate option to fill in blank number 5. (a) In reference with (b) Between (c) Along with (d) Among SSC GD 10/01/2023 (Shift-I) Ans. (d) : 'Among' will be used in the blank space (5) of the above passage because among is used for more than two people. Hence option (d) is correct answer. Correct sentenceThese people strengthen the relationship among the people with their interaction. Comprehension: In the following passage, some words have been deleted. Read the passage carefully and select the most appropriate option to fill in each blank. We (1) _____ brought up to fear insects. We regard them as unnecessary creatures that do more harm that good. Man continually (2) ______ was on them, because they contaminate his food, carry diseases, or devour his crops. They sting or bite without provocation; they fly uninvited into our rooms on summer nights or beat against our lighted windows. We live in Cloze Test

dread not only of unpleasant insects like spiders or wasps but of quite (3) _____ ones like moths. Reading about them increases our understanding society does noting to prevent us from being filled with revulsion when we find hordes of them crawling over a carefully prepared picnic lunch. No matter how much we like honey of how much we have read about the uncanny sense of direction which bees possess, we have a horror of being stung. (4) _____ of our fears are fascinating. We enjoy reading about them, especially when we find that, like the praying mantis, they lead perfectly horrible lives. We enjoy staring at them, entranced as they go (5) ______ their business, unaware (we hope) of our presence. 90. Select the most appropriate option to fill in blank 1. (a) Have been (b) Were being (c) Are been (d) Will be being SSC GD 08/02/2023 (Shift-IV) Ans. (a) : According to the sense of the above passage, 'Have been' would be appropriate. The other options do not give the proper sense. Hence option (a) 'Have been' is correct answer. 91. Select the most appropriate option to fill in blank 2. (a) Yields (b) Employs (c) Wages (d) Redresses SSC GD 08/02/2023 (Shift-IV) Ans. (c) : According to the sense of the above passage, 'wages' will be use. Wages means :- earning, payment. Hence options (c) is correct answer. 92. Select the most appropriate option to fill in blank 3. (a) Benignant (b) Harmless (c) Wholesome (d) Nonfatal SSC GD 08/02/2023 (Shift-IV) Ans. (b) : In the blank space of the above passage, 'Harmless' would be appropriate. The other options do not give the suitable meaning. Hence option (b) is correct answer. 93. Select the most appropriate option to fill in blank 4. (a) Few (b) Most (c) Several (d) Some SSC GD 08/02/2023 (Shift-IV) Ans. (b) : In the given passage, 'Most' will be used in the blank space. The meaning of the other options does not give proper meaning. Hence option (b) is correct answer.

768

YCT

94.

Select the most appropriate option to fill in blank 5. (a) Beside (b) near (c) around (d) about SSC GD 08/02/2023 (Shift-IV) Ans. (d) : According to the sense of the above passage, 'About' would be appropriate word for the blank space. The other options give different meaning. Hence option (d) 'about' is correct answer. Comprehension: In the following passage, some words have been deleted. Read the passage carefully and select the most appropriate option to fill in each blank. The oldest hub of carpet weaving in India is Mirzapurin Bhadohi district of Uttar Pradesh. The Gang (1) ____ past this town with its ghats, ancient temples and waterfalls. The genesis of carpet making in Mirzapur can be (2) ____ to the Mughal emperor, Akbar, in the 16th century. A (3) ____ and injured by dacoits on the Grand Trunk Road. The weavers were rescued and given shelter by local villagers. The master weaver of the group decided to settle down here and share his skills with his (5) ____. That is how the renowned carpet industry of Mirzapur took root. 95. Select the most appropriate option to fill in blank 1. (a) sets (b) spreads (c) flows (d) grows SSC GD 01/02/2023 (Shift-II) Ans. (c) : According to the sense of the above passage, 'flows' will be used in the blank space. The other options give different meaning– Sets :- put into a certain place. Spears :- scatter. Grows :- mature, develop. 96. Select the most appropriate option to fill in blank 2. (a) traced (b) found (c) extracted (d) reduced SSC GD 01/02/2023 (Shift-II) Ans. (a) : In the blank space of the above passage, 'traced' (hint) would be appropriate. The meaning of other options is– Found :- set up, establish. Extracted :- draw act, remove. Reduced :- decreased. 97. Select the most appropriate option to fill in blank 3. (a) caravan (b) crowd (c) army (d) band SSC GD 01/02/2023 (Shift-II) Ans. (a) : According to the sense of the above passage, 'caravan' will be used in the blank space. The meaning of other options is– Crowd :- a large number of people considered together. Cloze Test

Army :- A military forces of a country which are trained to fight on land. Band :- a small ground of musician who place popular music together. 98. Select the most appropriate option to fill in blank 4. (a) hindered (b) waylaid (c) encroached (d) lost SSC GD 01/02/2023 (Shift-II) Ans. (b) : In the blank space of the above passage, 'waylaid' would be appropriate word. The meaning of the other options is– Hindered :- obstruct. Encroach :- impinge Lost :- confused, destroyed. 99. Select the most appropriate option to fill in blank 5. (a) recipients (b) beneficiaries (c) sponsors (d) benefactors SSC GD 01/02/2023 (Shift-II) Ans. (d) : According to the sense of the above passage, 'benefactors' will be used in the blank space. The meaning of other options is– Recipient :- a person who receives something. Sponsors :- supporter, patron. Beneficiary :- the recipient of fund or other benefits. Comprehension: In the following passage, some words have been deleted. Read the passage carefully and select the most appropriate option to fill in each blank. When he was in college, a famous poet made a useful distinction for him. He had drunk (1) _______ in the poet's company to be compelled to describe to him a poem he was (2) _______ of. It would be a monologue of sorts, the selfcontemplation of a student on a summer afternoon who is reading Euphuise. The poem itself would be a subtle (3) _______ of euphuisms, translating the heat, the day, the student's concerns, into symmetrical posies; translating even his (4) _______ and boredom with that famously foolish book into a euphuism. The poet nodded his big head in a sympathetic, rhythmic way as this was (5) _______ to him, then told him that there are two kinds of poems. There is the kind you write; there is the kind you talk about in bars. Both kinds have value and both are poems; but it's fatal to confuse them. 100. Select the most appropriate option to fill in blank no. 1. (a) far (b) many (c) since (d) enough SSC CGL (Tier-I) 19/07/2023 (Shift-IV)

769

YCT

Ans. (d) : According to the passage, the most appropriate option for the blank space (1) 'enough' will be used because it gives conducive meaning of the sentence. Other options are incorrect. Thus, the correct answer is option (d). 101. Select the most appropriate option to fill in blank no. 2. (a) straying (b) marking (c) drying (d) thinking SSC CGL (Tier-I) 19/07/2023 (Shift-IV) Ans. (d) : According to the passage, the most appropriate option for the blank space (2) 'thinking' will be used because sentence is in past continuous tense. Thus, the correct answer is option (d). 102. Select the most appropriate option to fill in blank no. 3. (a) races (b) series (c) plains (d) credits SSC CGL (Tier-I) 19/07/2023 (Shift-IV) Ans. (b) : According to the passage, the most appropriate option for the blank space (3) 'series' will be used because it gives conducive meaning of the sentence. Thus, the correct answer is (b). 103. Select the most appropriate option to fill in blank no. 4. (a) contempt (b) stay (c) haven (d) weight age SSC CGL (Tier-I) 19/07/2023 (Shift-IV) Ans. (a) : According to the passage, the most appropriate option for the blank space (4) 'contempt' will be used because it gives conducive meaning of the sentence. Other options not correct. Thus, the correct answer is option (a) 104. Select the most appropriate option to fill in blank no. 5. (a) explained (b) sprang (c) raised (d) theorised SSC CGL (Tier-I) 19/07/2023 (Shift-IV) Ans. (a) : According to the passage, the most appropriate option for the blank space (5) 'explained' will be used because sentence is in passive form. Other options give different meaning. Sprang means - catapulted, bounced Raised means - Elevate Theorised means - Hypothesized Thus, the correct answer is option (a). Comprehension: In the following passage, some words have been deleted. Read the passage carefully and select the most appropriate option to fill in each blank. The basic elements of computers that (1)________ our world are explained in an interactive manner. The book has a Cloze Test

(2)________ approach of colourful and creative picture-based activities. The puzzles, activities and mazes (3)________ the students to develop critical thinking, logical and motor skills. After every lesson, (4)________ exercises are incorporated to (5)________ the conceptual understanding of the students. 105. Select the most appropriate option to fill in blank no. 1. (a) negates (b) empower (c) empowers (d) negate SSC CGL (Tier-I) 21/07/2023 (Shift-II) Ans. (b) : According to the sense of the above given passage in blank space (1) ' empower' (v) is used which means' to give official authority or legal power.' Other options give different meanings. 106. Select the most appropriate option to fill in blank no. 2. (a) complex (b) difficult (c) unique (d) trivial SSC CGL (Tier-I) 21/07/2023 (Shift-II) Ans. (c) : According to the sense of the above passage in blank space (2) 'unique' would be appropriate, which means' existing as the only one.' Other options give different meaning. Meaning of other options Complex - Intricate, Difficult - Stiff, Trivial - Insignificant. 107. Select the most appropriate option to fill in blank no. 3. (a) had helped (b) help (c) helped (d) helps SSC CGL (Tier-I) 21/07/2023 (Shift-II) Ans. (b) : The above passage in blank space (3) 'help' (Vi) would be appropriate because this sentence is in simple present tense and plural verb (help) is used with plural subject (The puzzles etc.). Thus, the correct answer is option (b). 108. Select the most appropriate option to fill in blank no. 4. (a) superficial (b) irrational (c) brainstorming (d) baffling SSC CGL (Tier-I) 21/07/2023 (Shift-II) Ans. (c) : According to the sense of the above given passage in blank space (4) 'brainstorming' (Present participle) would be appropriate. Other options give different meanings. 109. Select the most appropriate option to fill in blank no. 5. (a) evolve (b) evolving (c) evolution (d) evolved SSC CGL (Tier-I) 21/07/2023 (Shift-II)

770

YCT

Ans. (a) : For the above passage in blank space (5) 112. Select the most appropriate option to fill in blank 3. 'evolve' (V1) would be appropriate because infinitive (to (a) personal (b) personnel + V1) is used according to the sense of this sentence. (c) political (d) public Other given options are not correct. SSC CGL (Tier-I) 25/07/2023 (Shift-IV) Comprehension: Ans. (a) : According to the sense of the above given In the following passage, some words have been deleted. Read the passage carefully and passage in blank space (3) 'personal' would be select the most appropriate option to fill in each appropriate which means 'proper.' Other option give different meaning. blank. The massive ransom were attack that has Meaning of other options rippled e-hospital services of AIIMS, Delhi, Personnel - Team, Ponel highlights the (1) _________ vulnerability of Political - Vernacular the country’s healthcare infrastructure, and Public - Municipal possibly other critical IT systems, to 113. Select the most appropriate option to fill in cybercriminals. The premier public healthcare blank 4. institute (2)_________ to around 15 lakh (a) weighty (b) massive outpatients and 80,000 inpatients every (c) bulk (d) grand year. Consequently, SSC CGL (Tier-I) 25/07/2023 (Shift-IV) there are fears that the attackers could sell the now force-encrypted AIIMS databases that Ans. (b) : According to the sense of the above given contain (3) _________ information of atients- passage in blank space (4) 'massive' would be including political leaders, senior appropriate which means 'havy in structure.' Other administrators, and judges, and their healthcare option give different meaning. records on the Dark Web. Of course, what has Meaning of other options happened to AIIMS is hardly a local Weighty - Heavy, Bulky phenomenon. A (4) _________ increase in Bulk - Pile, Heap cyberattacks on healthcare institutes Grand - Magnificant, Super worldwide (5) _________ during the Covid 114. Select the most appropriate option to fill in pandemic. blank 5. The Indian healthcare sector was the second(a) is witnessed (b) has witnessed most targeted globally. It is clear that Indian (c) was witness (d) has been witnessed hospitals are vulnerable to cybersecurity. Government and hospitals need to use best SSC CGL (Tier-I) 25/07/2023 (Shift-IV) practices to ward off future crises. Ans. (d) : In the above given passage in blank space (5) 110. Select the most appropriate option to fill in 'has been witnessed' would be appropriate because the blank 1. sense of this sentence is in passive voice. (Has been + V3). Other option give different meaning. (a) increasing (b) developing Comprehension: (c) strengthening (d) shining In the following passage, some words have been SSC CGL (Tier-I) 25/07/2023 (Shift-IV) deleted. Read the passage carefully and select Ans. (a) : According to sense of the above passage in the most appropriate option to fill in each blank space (1) 'Increasing' would be appropriate. blank. Other options give different meanings. Socrates is flying. No, he is (1) _______ . The wings behind him beat in a calming rhythm (2) 111. Select the most appropriate option to fill in _______ the cool air rushes past. His wings are blank 2. all that matter, (3) _______ at the rushing wind (a) catered (b) catering like the sails of some great sea vessel, the (c) caters (d) has catered feathery appendages all he is and all he will ever want to be. His back muscles flex with the SSC CGL (Tier-I) 25/07/2023 (Shift-IV) (4) _______ that takes him high above the Ans. (c) : According to sense of the above passage in ground. He feels the effort, of course, but blank space (2) 'Caters' (V1) would be appropriate sweeping into the sky does not require much of because in this sentence 'every year shows the given one. The (5) _______ is pleasurable, even sentence is in simple present tense exhilarating. With flight there is freedom beyond description, an ecstasy bordering on Sub. + V1 + s/es + Object. emotional. Hence, the correct option is (c). SubQuestion No : 115-119 Cloze Test

771

YCT

115. Select the most appropriate option to fill in blank no. 1. (a) redundant (b) sluggish (c) pertinent (d) soaring SSC CGL (Tier-I) 18/07/2023 (Shift-III) Ans. (d) : For the above, passage in blank space (1) 'soaring' would be appropriate because this sentence is in present tense and (V1 + ing) 'soaring' is used according to the context of this sentence, which means 'to fly high in the air'. Other options given different meanings. 116. Select the most appropriate option to fill in blank no. 2. (a) unless (b) unless (c) while (d) since SSC CGL (Tier-I) 18/07/2023 (Shift-III) Ans. (c) : For the above passage in blank space (2) 'while' will be used because it is a conjunctive adverb to express two continuous actives that happen simultaneously. Other options given different meanings. 117. Select the most appropriate option to fill in blank no. 3. (a) snapping (b) cleaning (c) losing (d) limiting SSC CGL (Tier-I) 18/07/2023 (Shift-III) Ans. (a) : According to the sense of the above passage in blank space (3) 'snapping' would be appropriate which means. 'to be broken suddenly'. Other options give different meanings. 118. Select the most appropriate option to fill in blank no. 4. (a) litigation (b) affection (c) manipulation (d) effort SSC CGL (Tier-I) 18/07/2023 (Shift-III) Ans. (d) : For the above passage in blank space (4) 'effort' will be used according to the context of this sentence. Other options give different meanings Litigation - case, Affection - Amorousness, Manipulation - Expedient, 119. Select the most appropriate option to fill in blank no. 5. (a) sensation (b) indignation (c) perversion (d) lamination SSC CGL (Tier-I) 18/07/2023 (Shift-III) Ans. (a) : For the above passage in blank space (5) 'sensation' will be used. because it gives appropriate meaning. Other options given different meaning Indignation - Anger, Perversion - Distortion, Lamination - Splitting, Cloze Test

Comprehension: In the following passage, some words have been deleted. Read the passage carefully and select the most appropriate option to fill in each blank. The world is going through a deep recession. At such a time, one thing we need in abundance is jobs for the semi-skilled and unskilled. This is the only way in which equal (1)____ of wealth can take place. The healthcare industry is ideally poised to occupy this position. The IT industry hires people from the upper-middle strata and rich families, usually engineers, (2)____ the health care industry hires nurses, to the tune of eighty per cent of the jobs created, from the lower economic strata. Global health care is a 4.5trillion rupee industry, second only to the agro industry. Even then, health care (3)____ only eight per cent of the world’s population. Policymakers should look at the health care industry as not only an industry which addresses pain but also as one which can (4)____ the economy. The last century was driven by machines that addressed human toil and it is strongly believed that this century will be driven by health care. This, however, will only happen if policy makers make a conscious effort to (5)____ the right policies in place soon. 120. Select the most appropriate option to fill in blank no. 1. (a) spending (b) distribution (c) partition (d) earning SSC CGL (Tier-I) 20/07/2023 (Shift-I) Ans. (b) : According to the passage, the most appropriate option for the blank space (1) ‘distribution’ will be used because ‘wealth’ indicts that the distribution’ is conducive meaning of the sentence. Thus, the correct answer is option (b). 121. Select the most appropriate option to fill in blank no. 2. (a) despite (b) unlike (c) unless (d) whereas SSC CGL (Tier-I) 20/07/2023 (Shift-I) Ans. (d) : According to the passage, the most appropriate option for the blank space (2) ‘Conjunction’ whereas’ will be used because it given reason of the sentence. Thus, the correct answer is option (d). 122. Select the most appropriate option to fill in blank no. 3. (a) calls (b) cures (c) visits (d) reaches SSC CGL (Tier-I) 20/07/2023 (Shift-I)

772

YCT

Ans. (d) : According to the passage, the most appropriate option for the blank (3) ‘reaches’ will be used because word ‘only eight percent’ indicates reaches’ is appropriate words for the sentence Other words are– Calls means – Summons Cures means – Remedies Visits means – Trips. Thus, the correct answer is option (a). 123. Select the most appropriate option to fill in blank no. 4. (a) influence (b) impede (c) ascertain (d) persuade SSC CGL (Tier-I) 20/07/2023 (Shift-I) Ans. (a) : According to the passage, the most appropriate option for the blank space (4) ‘influence’ will be used because It gives correct meaning of the sentence, Thus, the correct answer is option (a) 124. Select the most appropriate option to fill in blank no. 5. (a) desire (b) design (c) cut (d) put SSC CGL (Tier-I) 20/07/2023 (Shift-I) Ans. (d) : According to the passage, the most appropriate option for the blank space (5) ‘put’ will be used because it gives appropriate sense of the sentence. Thus, the correct answer is option (d). Comprehension: In the following passage, some words have been deleted. Read the passage carefully and select the most appropriate option to fill in each blank. Colonialims had a great impact on the lives of aboriginal Australians who wee eventually (1) ______ by the whits with all power and privilege. The colonisers turned their land into rubbish pits and (2) ______ sites for their own betterment. The aborigines were often (3) ______ as sub-humans with low status and dirty habits. The whites not only displaced the tribes off their homeland but also (4) ______ the beauty and balance of the natural world. An increase in deforestation and desttruction of traditional land led (5) ______ like emu, eagle, and kangaroom among many others to dwindle over time. 125. Select the most appropriate option to fill in blank number 1. (a) understood (b) caught (c) raised (d) subjugated SSC CGL (Tier-I) 27/07/2023 (Shift-III) Ans. (d) : According to sense of the above given passage in blank space (1) 'Subjugated' would be appropriate, which means 'the act of bringing someone or something under control or domination through force ir suppression.' Other options give different meanings. Cloze Test

126. Select the most appropriate option to fill in blank number 2. (a) construction (b) dating (c) halting (d) recreation SSC CGL (Tier-I) 27/07/2023 (Shift-III) Ans. (a) : According to the sense of above passage in blank space (2) 'Construction' would be appropriate because in this sentence, it is used with 'sites'. Other options give different meaningsUnderstood - Perceived, Realized, Caught - Captured Snagged, Raised - Eevate, Uplift. 127. Select the most appropriate option to fill in blank number 3. (a) created (b) perceived (c) received (d) led SSC CGL (Tier-I) 27/07/2023 (Shift-III) Ans. (b) : The above given passage in blank space (3) 'Perceived' would be appropriate, which means - 'to notice or realize something'. Other options give different meaningsCreated - Composed, Fabricated Received - Get, Accept, Led - Conduct, Guide, Thus, the correct answer is option (b). 128. Select the most appropriate option to fill in blank number 4. (a) destroyed (b) climbed (c) utilised (d) fed SSC CGL (Tier-I) 27/07/2023 (Shift-III) Ans. (a) : According to the sense of the above sentence in blank space (4) 'Destroyed; would be appropriate which means - 'Ruined or demolished'. Other options give different meaningsClimbed - Ascend, Clamber, Utilised - Emplayed, Adopted, Fed - Sustained, nurtured, Thus, the correct answer is option (a). 129. Select the most appropriate option to fill in blank number 5. (a) pets (b) thugs (c) homies (d) fauna SSC CGL (Tier-I) 27/07/2023 (Shift-III) Ans. (d) : According to the sense of the above given passage in blank space (5) 'fauna' would be appropriate because If refers to all of the animal life within a specified region. Other options give different meanings. Thus, the correct answer is option (d).

773

YCT

Comprehension : Comprehension: In the following passage, some words have In the following passage, some words have been deleted. Read the passage carefully and select the most been deleted. Read the passage carefully and appropriate option to fill in each blank. Ambition is a Select the most appropriate option to fill in vital ingredient for success. Without ambition to push us, each blank. we will not be (1)_____ of great achievements. We all Any (1) –––––– that revolves around a planet need that special something (2)________ ourselves to is a satellite. Mother Earth has a natural give us the willpower to reach a higher goal (3)_____ we satellite in the moon. The earth is circled by a imagine ourselves capable (4)_____. Yet, ambition, lot of man-made or artificial satellites in (5)_____ money and fire, is a good servant, but a bad 1957, the Soviet Sputnik was the master. first to be (2) ––––– into space. There are 130. Select the most appropriate option to fill in currently a number of artificial satellites blank number 1. launched into space by various nations. (a) curable (b) durable These satellites make it possible for us to (c) capable (d) able talk on the phone, watch television, predict the SSC CGL (Tier-I) 17/07/2023 (Shift-II) weather, map the various surfaces of the earth and learn more about the world in Ans. (c) : For the above blank space (1) 'Capable' which we live. Competent will be used because it giver appropriate How does a satellite get started? It is meaning other options give different meaning. typically launched into space by means of a Curable – Provable rocket. After ascending into the thinner air Durable – Lasting layers above the earth, the satellite is able – Eligible released from the rocket. For a successful 131. Select the most appropriate option to fill in satellite launch, a rocket's speed is carefully blank number 2. calculated and controlled. Every object in (a) in (b) with space is pushed toward the earth's centre by (c) within (d) without gravity. The inertia of motion will propel the SSC CGL (Tier-I) 17/07/2023 (Shift-II) object into space if it has its own velocity of Ans. (c) : For the above blank space (2) 'within' will be motion. When the orbital used because it gives appropriate meaning. velocity is exactly right, these two opposing Other options give different meaning. forces are balanced, causing the satellite to (3)________ toward the earth while still 132. Select the most appropriate option to fill in blank number 3. hovering over its surface. A satellite is said to be in geo-stationary orbit when it completes (a) that (b) than one revolution around the earth in a 24-hour (c) then (d) what period; It doesn't change when compared to a SSC CGL (Tier-I) 17/07/2023 (Shift-II) point on the surface of the earth. These orbits Ans. (b) : For the above blank space (3) 'than' will be are typically used by satellites for used because comparative degree is used in the sense of communication and weather. The camera on comparison, other option give different meaning. the weather satellites takes pictures and sends 133. Select the most appropriate option to fill in them back to Earth. These are helping aid in blank number 4. weather forecasting. Radio receivers and (a) off (b) of amplifiers on communication satellites can (4) (c) on (d) about ________ waves back at a frequency that is SSC CGL (Tier-I) 17/07/2023 (Shift-II) different from the one the satellite received. Ans. (b) : For the above blank space (4) 'of' will be used The beaming frequencies of satellites that are because 'of' preposition is used with capable. other 'parked' too close to one another must not options give different meaning. interfere with one another. Television signals 134. Select the most appropriate option to fill in can be transmitted from one location to blank number 5. other using broadcast satellites and (a) such (b) like scientists can (5)________ scientific data (c) as (d) similarly from numerous satellites. SSC CGL (Tier-I) 17/07/2023 (Shift-II) 135. Select the most appropriate option to fill in blank no. 1. Ans. (b) : For the above blank space (5) 'like' will be (a) Piece (b) Object used because it gives appropriate meaning according to (c) Body (d) Being sense of the passage. Other options give different meaning. SSC CGL (Tier-I) 24/07/2023 (Shift-III) Cloze Test

774

YCT

Ans. (b) : For the above blank space (1) 'object' (Thing/Item) will be used because it gives appropriate meaning. Other options give different meanings. 136. Select the most appropriate option to fill in blank no. 2. (a) Tossed (b) Lobbed (c) Launched (d) Floated SSC CGL (Tier-I) 24/07/2023 (Shift-III) Ans. (c) : For the above blank space (2) 'Launched' (Set on) will be used because it gives appropriate meaning. Other options give different meanings. Tossed - Rolled Lobbed - Threw Floated - Sailed 137. Select the most appropriate option to fill in blank no. 3. (a) Stir (b) Migrate (c) Jump (d) Move SSC CGL (Tier-I) 24/07/2023 (Shift-III) Ans. (d) : For the above blank space (3) 'Move' will be used because it gives appropriate meaning according to sense of the passage. Other options give different meanings. 138. Select the most appropriate option to fill in blank no. 4. (a) Send (b) Discharge (c) Deliver (d) Locate SSC CGL (Tier-I) 24/07/2023 (Shift-III) Ans. (a) : For the above blank space (4) 'Send' will be used because it gives appropriate meaning. Other options give different meanings. Discharge - Release Deliver - Forgive/Carry Locate - Detect 139. Select the most appropriate option to fill in blank no. 5. (a) Entry (b) Ingress (c) Access (d) Approach SSC CGL (Tier-I) 24/07/2023 (Shift-III) Ans. (c) : For the above blank space (5) 'Access' will be used because it gives appropriate meaning. Other options give different meanings. Approach - Access/Path Ingress - Enterance Entry - Access Comprehension: In the following passage some words have been deleted. Read the passage carefully and select the most appropriate option to fill in each blank. The entertainment (1)_______ built by Walt Disney is sometimes called the ‘House of Mouse’ in acknowledgment of the leading role played by Disney’s most famous creation, Mickey Cloze Test

Mouse. Mickey’s round mouse ears, either in silhouette or adorning a hat, have become one of the most (2)_______ corporate logos in the world. But if a few business decisions in the 1920s had gone differently, they could have been rabbit ears. In the mid-1920s Disney was a young (3)________ producing a series called the Alice Comedies, short films that combined live-action footage with animated images so that human actors appeared to interact with cartoon characters. But he had grown tired of the series and wanted to move on to making completely animated films. In 1927 he got his wish, (4)_______ a deal with Universal to produce a new series based on the adventures of a character called Oswald the Lucky Rabbit. Both the name and the species of the character had been picked by Universal—'Oswald’ was (5)________ picked out of a hat of names, and the character was made a rabbit because there were already too many cartoon cats on the market. 140. Select the most appropriate option to fill in blank number 1. (a) empire (b) province (c) country (d) power SSC CHSL (Tier-I) 17/08/2023 (Shift-II) Ans. (a) : Option (a) Empire – ( A very large companies or group of companies) will be filled in the blank provided, Because next line of the sentence mentions the name of the entertainment company House of the mouse, built by walt Disney. Hence using 'empire' in the blank is appropriate. meaning of other options are – Province – A principal administrative division of a country Country – An area of land with its own people and government Power – The ability to control people Political control of a country. 141. Select the most appropriate option to fill in blank number 2. (a) visible (b) notifiable (c) recognisable (d) measurable SSC CHSL (Tier-I) 17/08/2023 (Shift-II) Ans. (c) : In the given blank space, option (c) recognizable – (Able to be recognized or identified from precious encounters or knowledge) need to be filled . As in the sentence of the passage, Popular 'logo' of mickey mouse and its well established identification is being discussed. Meaning of other options are – Visible – That can be seen or noticed Modifiable – Required by lane to be reported to official health authorities. Measurable – capable of being assessed / estimated.

775

YCT

142. Select the most appropriate option to fill in blank number 3. (a) artisan (b) animator (c) miniaturist (d) craftsman SSC CHSL (Tier-I) 17/08/2023 (Shift-II) Ans. (b) : Option (b) Animator – ( A person who makes animated / cartoon films) is suitable to be used in the given blank. Because 'Profession or work being practiced by Walt Disney is being discussed in the sentence other options contain following meaning Artisan – A person who makes things skillfully especially with his/ her hands. Miniaturist – a painter/ illuminator of manuscripts. craftsman – A man who practices a trade or handicrafts as a job. 143. Select the most appropriate option to fill in blank number 4. (a) crossing (b) bewitching (c) possessing (d) striking SSC CHSL (Tier-I) 17/08/2023 (Shift-II) Ans. (d) : In the blank space, option (b) striking will be used. because 'Striking a deal' is an idiom which means To successfully achieve a agreement especially at a acceptable price. Following are the meaning of given options – Crossing – A place where roads or railway lines cross bewitching – Attracting somebody very much Possessing – To have or own something. 144. Select the most appropriate option to fill in blank number 5. (a) anatomically (b) outwardly (c) reasonably (d) apparently SSC CHSL (Tier-I) 17/08/2023 (Shift-II) Ans. (d) : Option (d) apparently – ( what people say or to how something appears but perhaps not true). need to be used in the given blank. Meaning of other options are – Anatomically – Related to human body and how its parts are arranged Outwardly – Things seem to be, rather than haw they are inside. reasonably – using good judgment or In a sensible or fair way Comprehension: In the following passage, some words have been deleted. Read the passage carefully and select the most appropriate option to fill in each blank. Once king Krishnadeva Raya (1)______ a parrot of which he was excessively fond. He kept it in a silver cage and fed it fruits and nuts and anything else the bird asked for. The parrot longed for freedom and often asked for it, but the king would always reply, "Ask for Cloze Test

anything else." One day, the parrot said to the king, "Give me freedom and I'II give you three (2)_____ advice that could be of great benefit to you." The king loved the parrot, but he loved money (3)_____. The parrot said, "Never believe everything that is told to you. What you don't know is that I've two priceless gems in my stomach,” said the bird. "Two priceless gems," echoed the king, "Oh, what a fool I was to set you free! I'II regret this for the rest of my life!" I (4)_____ you never to grieve over losses but here you are grieving over losing me," said the parrot, "I advised you never to believe everything you hear, but you immediately believed me when I told you I had two gems in my stomach. (5)____ I have survived if I really had two gems in my stomach?" My third advice is, "Listen, learn to listen with your mind instead of just with your ears.” Saying these words, the parrot flew away. 145. Select the most appropriate option to fill in blank number 1. (a) has (b) will have (c) have (d) had SSC CHSL (Tier-I) 10/08/2023 (Shift-I) Ans. (d) : In the given blank, option (d) had – (possess, own or hold) will be used as had is used in past tense and its provide suitable meaning to the sentence to the passage (king loved the parrot he had). 146. Select the most appropriate option to fill in blank number 2. (a) quantity of (b) digit of (c) pieces of (d) number of SSC CHSL (Tier-I) 10/08/2023 (Shift-I) Ans. (c) : Since advice is uncountable noun and information to be provided by parrot was more than one piece, 'pieces of information' will by appropriate to be used in the provided blank. Note – piece of advice follows 'noun + noun' collocation pattern 147. Select the most appropriate option to fill in blank number 3. (a) more (b) better (c) most (d) extra SSC CHSL (Tier-I) 10/08/2023 (Shift-I) Ans. (a) : option (a) more, which will be used as an adverb in the given blank means 'To a greater degree than usual or than something else'. Therefore ascertaining the degree of love king had for money in comparison with parrot (which obviously was for Money) 'more' will be used. 148. Select the most appropriate option to fill in blank number 4. (a) advise (b) have advised (c) did advised (d) advised SSC CHSL (Tier-I) 10/08/2023 (Shift-I)

776

YCT

Ans. (d) : The given sentence of the passage. display the sense, hence advised (Giving suggestion on an issue) will be used in the provided blank. As the parrot had given three pieces of advice to the king. 149. Select the most appropriate option to fill in blank number 5. (a) Could (b) Might (c) Can (d) May SSC CHSL (Tier-I) 10/08/2023 (Shift-I) Ans. (a) : Option (a) could (To express a possibility in the past) need to be used in the given blank as parrot talks about hypothetical question which comes under type three conditional sentence, the structure of which is– If + Past Perfect, could have + Past participle. Hence using could in the blank is correct . Comprehension: In the following passage, some words have been deleted. Read the passage carefully and select the most appropriate option to fill in each blank. Unemployment arises from a variety of _____(1). One which is always recurring and the effects of which we have had a recent example, is the disorganisation of industry resulting from a long war; this is a serious _____ (2) admitting of no easy_____ (3) at the best of times. Again there is the unemployment which followed a marked diminution in the _____ (4) of any raw product, such as cotton; fewer hands are required in the mills and the factories. We may call this cause of bad harvests. Similar, but more serious is the effect of changes in industry due to the invention of machinery which does more work and require _____ (5) hands. Yet another cause is strike or lockout and this is more to be deplored because such a stoppage is due to a very trivial matter, perhaps the men are working half an hour longer than what their Union permits. 150. Select the most appropriate option to fill in blank number 1. (a) causes (b) types (c) things (d) thoughts SSC CHSL (Tier-I) 02/08/2023 (Shift-I) Ans. (a) : For the above blank space (1) 'causes' will be used because here talking about the reason of unemployment, Other option give different meaning. 151. Select the most appropriate option to fill in blank number 2. (a) nature (b) difference (c) problem (d) discussion SSC CHSL (Tier-I) 02/08/2023 (Shift-I) Ans. (c) : According to the sense of the above passage in blank space (2) 'problem' would be appropriate. Other options give different meanings. Cloze Test

152. Select the most appropriate option to fill in blank number 3. (a) key (b) answer (c) clue (d) solution SSC CHSL (Tier-I) 02/08/2023 (Shift-I) Ans. (d) : For the above blank space (3) 'solution' would be appropriate because according to the passage 'this is a serious problem admitting of no easy solution at the best of times. Other options are not correct. 153. Select the most appropriate option to fill in blank number 4. (a) material (b) quality (c) quantity (d) cloth SSC CHSL (Tier-I) 02/08/2023 (Shift-I) Ans. (c) : According to the sense of the above passage in blank space (4) 'quantity' will be used. other options give different meanings. 154. Select the most appropriate option to fill in blank number 5. (a) a lot of (b) more (c) many (d) fewer SSC CHSL (Tier-I) 02/08/2023 (Shift-I) Ans. (d) : For the above blank space (5) 'fewer' (comparative adjective) will be used because according to the sense of this sentence double comparative is used to show a cause and effect of unemployment. Other options give different meanings. Comprehension : In the following passage, some words have been deleted. Read the passage carefully and select the most appropriate option to fill in each blank. Tea price in the domestic market continue to rule high in the (1) ___ year despite the expectation of a higher production as compared to the previous year. According to a preliminary assessment (2) ____ on the weather (3) ____ in recent months, tea output in the next year may reach to 800 tonnes as (4) ____ 780 tonnes last year. During the past three months, tea prices have shown an (5) ___. Unlike last year, when tea prices were dramatically low, this year, prices seem to have stabilised at rather high levels. 155. Select the most appropriate option to fill in blank number 1. (a) second (b) current (c) first (d) last SSC CHSL (Tier-I) 14/08/2023 (Shift-IV) Ans. (b) : In the provided blank option (b) current – (or the present time; happening now. is apposite to be filled. As the passage discuss the anticipation of higher production of tea in comparison with last year, yet the price of tea has been high this year. mainly passage points out the irony and anomaly of prices of tea being high, be spite hoping for a higher production.

777

YCT

156. Select the most appropriate to fill in blank number 2. (a) based (b) carried (c) shared (d) flown SSC CHSL (Tier-I) 14/08/2023 (Shift-IV) Ans. (a) : option (a) based will be used in the given blank as 'based on' is a phrasal verb which means 'to use an idea a fact or a situation as the point from which something can be developed. hence option (a) is correct to be used in the given blank. 157. Select the most appropriate option to fill in blank number 3. (a) forecast (b) outbreak (c) condition (d) pattern SSC CHSL (Tier-I) 14/08/2023 (Shift-IV) Ans. (a) : According to the structure and sense of the passage, option (a) forecast – (To say what will probably happen in the future. weather forecast comes under 'noun + noun' collocation pattern. Therefore option (a) happens to be correct. 158. Select the most appropriate option to fill in blank number 4. (a) compared (b) about (c) above (d) against SSC CHSL (Tier-I) 14/08/2023 (Shift-IV) Ans. (d) : option (d) Against (Preposition) would be used in the given blank. Because 'as against something' is an idiom which means 'in contrast with something' Ex – Congress got 26 percent of the vote as against 33 percent at the last election. Therefore for comparing tea output in tonner in respective years (last of current year) against will be appropriate to be used. 159. Select the most appropriate to fill in blank number 5. (a) uptrend (b) decline (c) upgrade (d) incline SSC CHSL (Tier-I) 14/08/2023 (Shift-IV) Ans. (a) : In the given blank option (a) uptrend – (An upward tendency , especially a rise in economic value) is to be used as it is describing current year situation, where pieces appears to have touched a higher level on par with prices of last year. hence option (a) happens to be correct. Comprehension: In the following passage, some words have been deleted. Read the passage carefully and select the most appropriate option to fill in each blank. In a busy bus station, there is a little girl sitting all (1)______ on a bench. She stares at the floor as all the busy travellers rush (2)_______ her. Some of the people are in such a big hurry that they do not (3)_______ her. Other people look at her, but continue running to where they need to go. After some time, a man stops next Cloze Test

to the girl. He is very tired after a long day of work, and he just wants to get to his home as quickly as (4)_________ so that he can be with his family. But he cannot just leave this little girl alone. He cannot see her (5)_________ anywhere. The little girl is no older than his own little daughter. 160. Select the most appropriate option to fill in blank number 1. (a) sole (b) alone (c) just (d) unique SSC CHSL (Tier-I) 04/08/2023 (Shift-III) Ans. (b) : According to the sense of the above passage in blank space (1) 'alone' would be appropriate. Other options give different meanings. Thus, the correct option is (b). 161. Select the most appropriate option to fill in blank number 2. (a) above (b) on (c) henceforth (d) around SSC CHSL (Tier-I) 04/08/2023 (Shift-III) Ans. (d) : For the above passage in blank space (2) 'around' would be appropriate because here the sense of this sentence is 'all the busy travellers rush around her.' Other options give different meanings. 162. Select the most appropriate option to fill in blank number 3. (a) inflict (b) notice (c) detect (d) neglect SSC CHSL (Tier-I) 04/08/2023 (Shift-III) Ans. (b) : For the above passage in blank space (3) 'notice' would be appropriate, according to the sense of this sentence. Other options give different meanings. 163. Select the most appropriate option to fill in blank number 4. (a) potential (b) possible (c) believable (d) credible SSC CHSL (Tier-I) 04/08/2023 (Shift-III) Ans. (b) : For the above passage in blank space (4) 'possible' would be appropriate because 'as quickly as possible so' gave the appropriate meaning for this sentence. Other options give different meanings. 164. Select the most appropriate option to fill in blank number 5. (a) self (b) friends (c) relatives (d) parents SSC CHSL (Tier-I) 04/08/2023 (Shift-III) Ans. (d) : For the above passage in blank space (5) 'parents' would be appropriate other options give different meanings. hence, the correct option is (d).

778

YCT

Comprehension: In the following passage some of the words have been deleted. Read the passage carefully and select the correct answer for the given blank out of the four alternatives. According to the Integrated Taxonomic Information System (ITIS) there are more than 60 (1)______ of rat, which means they come in all sizes. They are (2)_____ 5 inches (12 centimeters), or longer, according to Encyclopedia Britannica. The largest species is the Bosavi woolly rat, which was (3)_____ in 2009 in the rainforest of Papua New Guinea (and doesn't have a scientific name yet). It is (4)______ the size of a cat — 32.2 inches (82 cm) from nose to tail and (5)______ around 3.3 lbs. (1.5 kilograms), according to CNN. SubQuestion No : 165-169 165. Select the most appropriate option to fill in blank number 1. (a) entities (b) sections (c) species (d) objects SSC CHSL (Tier-I) 21/03/2023 (Shift-IV) Ans. (c) : For the above blank space (1) 'species' will be used because it gives appropriate meaning. Other options give different meaning. 166. Select the most appropriate option to fill in blank number 2. (a) modestly (b) manually (c) typically (d) surgically SSC CHSL (Tier-I) 21/03/2023 (Shift-IV) Ans. (c) : For the above blank space (2) 'Typically' will be used because it gives appropriate meaning. Other options give different meaning. 167. Select the most appropriate option to fill in blank number 3. (a) discovered (b) invented (c) denuded (d) screened SSC CHSL (Tier-I) 21/03/2023 (Shift-IV) Ans. (a) : For the above blank space (3) 'discovered' will be used because it gives appropriate meaning. Other options give different meaning. 168. Select the most appropriate option to fill in blank number 4. (a) a lot (b) of (c) about (d) some SSC CHSL (Tier-I) 21/03/2023 (Shift-IV) Ans. (c) : For the above blank space (4) 'about' will be used because it gives appropriate meaning. about is used for showing that you are questioning at an amount or number. 169. Select the most appropriate option to fill in blank number 5. (a) evaluates (b) weighs (c) counts (d) measures SSC CHSL (Tier-I) 21/03/2023 (Shift-IV) Cloze Test

Ans. (b) : For the above blank space (5) 'weighs' will be used because it gives appropriate meaning according to sense of the passage. Other options give different meaning. Comprehension: In the following passage some of the words have been deleted. Read the passage carefully and select the correct answer for the given blank out of the four alternatives. Improving your well being is considered to be the fundamental (1) ............. of travelling. Travelling helps to decrease the risks of heart disease and anxiety (2) .............. developing our brain health. There have been studies proving that travel can have a positive impact on our heart (3) ............... The health benefit of travelling is that when you travel to new surroundings you (4) ............. a restart button to your body and mind. This brings in fresh energy when you get back to your normal (5) .............. activities. 170. Select the most appropriate option to fill in blank number 1. (a) rancidity (b) sanctity (c) sanity (d) benefit SSC CHSL (Tier-I) 17/03/2023 (Shift-IV) Ans. (d) : For the above blank space (1), option (d) 'benefit' will be used. Other options are not correct. 171. Select the most appropriate option to fill in blank number 2. (a) as (b) because (c) so (d) while SSC CHSL (Tier-I) 17/03/2023 (Shift-IV) Ans. (d) : For the above blank space (2), option (d) while will be used because While + Ving is used. 172. Select the most appropriate option to fill in blank number 3. (a) health (b) bloom (c) pace (d) being SSC CHSL (Tier-I) 17/03/2023 (Shift-IV) Ans. (a) : For the above blank space (3) 'health' will be used because it gives suitable meaning. 173. Select the most appropriate option to fill in blank number 4. (a) press (b) heap (c) usurp (d) snatch SSC CHSL (Tier-I) 17/03/2023 (Shift-IV) Ans. (a) : For the above blank space (4) 'press' will be used. Other options are not correct. 174. Select the most appropriate option to fill in blank number 5. (a) regular (b) sedentary (c) arbitrate (d) moderate SSC CHSL (Tier-I) 17/03/2023 (Shift-IV) Ans. (a) : For the above blank space (5) 'regular' will be used. Other options are not correct.

779

YCT

Comprehension: In the following passage, some words have been deleted. Fill in the blanks with the help of the alternatives given. Select the most appropriate option for each blank. Data 1. ................ the Pioneer spacecraft of NASA apparently prove the theory that the high surface temperature of Venus is 2. ............... to an atmospheric greenhouse effect caused mainly by a blanket of carbon dioxide. Such a greenhouse 3. ............... is created when energy in the form of sunlight easily passes through a planet's atmosphere, warms its 4. ............... and is converted to heat radiation that is then held in by the atmosphere from top to bottom. Venus has a relatively thin atmosphere like the Earth's, but Venus' atmosphere consists of more than ninety percent carbon dioxide, compared to less than four percent in that of the 5. ................ 175. Select the most appropriate option to fill in the blank no. 1. (a) on (b) across (c) from (d) through SSC CHSL (Tier-I) 17/03/2023 (Shift-III) Ans. (c) : For the above blank space (1) from will be used because it gives suitable meaning. Other options give different meaning. 176. Select the most appropriate option to fill in the blank no. 2. (a) due (b) understood (c) because (d) cause SSC CHSL (Tier-I) 17/03/2023 (Shift-III) Ans. (a) : For the above blank space (2) 'due' will be used because preposition 'to' is used after due. Other options give different meaning. 177. Select the most appropriate option to fill in the blank no. 3. (a) affect (b) displacement (c) contour (d) effect SSC CHSL (Tier-I) 17/03/2023 (Shift-III) Ans. (d) : For the above blank space (3) 'effect' will be used because here need a noun (effect) not a verb (affect). Other options give different meaning. 179. Select the most appropriate option to fill in the blank no. 4. (a) surface (b) walls (c) atmosphere (d) core SSC CHSL (Tier-I) 17/03/2023 (Shift-III) Ans. (a) : For the above blank space (4) 'Surface' will be used because it gives appropriate meaning. Other options give different meaning. 180. Select the most appropriate option to fill in the blank no. 5. (a) Pluto (b) Mars (c) Earth (d) Mercury SSC CHSL (Tier-I) 17/03/2023 (Shift-III) Cloze Test

Ans. (c) : For the above blank space (5) 'Earth' will be used because it gives appropriate meaning according to the passage. Other options give different meaning. Comprehension: In the following passage, some words have been deleted. Fill in the banks with the help of the alternatives given. Select the most appropriate option for each blank. The Games were held every four years from 1924 until 1936, after which they were 1. ............. by World War II. The Olympics 2. .............. in 1948 and was 3. ................. held every four years. Until 1992, the Winter and Summer Olympic Games were held in the same years, but in accordance with a 1986 4. .............. by the International Olympic Committee (IOC) to place the Summer and Winter Games on 5. ................ four-year cycles in alternating even-numbered years, the next Winter Olympics after 1992 was in 1994. 181. Select the most appropriate option to fill in the blank no. 1. (a) complicated (b) interrupted (c) corrupted (d) solicited SSC CHSL (Tier-I) 15/03/2023 (Shift-III) Ans. (b) : For the above blank space (1) 'Interrupted' will be used because it gives appropriate meaning. Other options give different meaning. Complicated- Complex Corrupted- Useless 182. Select the most appropriate option to fill in the blank no. 2. (a) resumed (b) accustomed (c) confirmed (d) transformed SSC CHSL (Tier-I) 15/03/2023 (Shift-III) Ans. (a) : For the above blank space (2) 'resumed' will be used because it gives appropriate meaning. Other option give different meaning. 183. Select the most appropriate option to fill in the blank no. 3. (a) regain (b) bargained (c) again (d) never SSC CHSL (Tier-I) 15/03/2023 (Shift-III) Ans. (c) : For the above blank space (3) 'again' will be used because it gives appropriate meaning. Other options give different meaning. 184. Select the most appropriate option to fill in the blank no. 4. (a) precision (b) incision (c) decision (d) indecision SSC CHSL (Tier-I) 15/03/2023 (Shift-III) Ans. (c) : For the above blank space (4) 'Decision' will be used because it gives appropriate meaning according to sense of the passage. Other options give different meaning.

780

YCT

185. Select the most appropriate option to fill in the blank no. 5. (a) customary (b) matching (c) together (d) separate SSC CHSL (Tier-I) 15/03/2023 (Shift-III) Ans. (d) : For the above blank space (5) 'separate' will be used because it gives appropriate meaning. Other options give different meaning. Comprehension: In the following passage some of the words have been deleted. Read the passage carefully and select the correct answer for the given blank out of the four alternatives. People may seek fashion as part of (1) ......... they are may want to be a fashion lover, which may lead them to spend more time (2) ........ out the latest fashion trends and more money than others on fashion products. We all know the good feeling that goes (3) ........... walking out of a store with a new shade of lipstick or after ordering a killer pair of boots online. They call it retail therapy for a reason and more than one scientific study has ........... (4) and its effectiveness. Shopping takes time and concentration, whether it is being done online (5) ........... in your favourite store. 186. Select the most appropriate option to fill in blank number 1. (a) which (b) whom (c) whose (d) who SSC CHSL (Tier-I) 10/03/2023 (Shift-III) Ans. (d) : For the above blank space (1) 'Who' will be used because Relative Pronoun (Who) is use for living things (People). Other option give different meaning. 187. Select the most appropriate option to fill in blank number 2. (a) freaking (b) seeking (c) doing (d) making SSC CHSL (Tier-I) 10/03/2023 (Shift-III) Ans. (b) : For the above blank space (2) 'Seeking' will be used because it gives appropriate meaning. Other options give different meaning. 188. Select the most appropriate option to fill in blank number 3. (a) off in (b) within (c) up in (d) along with SSC CHSL (Tier-I) 10/03/2023 (Shift-III) Ans. (d) : For the above blank space (3) "along with" will be used because it gives appropriate meaning. Other options give different meaning. 189. Select the most appropriate option to fill in blank number 4. (a) held back (b) summed up (c) backed up (d) sold on SSC CHSL (Tier-I) 10/03/2023 (Shift-III) Cloze Test

Ans. (c) : For the above blank space (4) "Backed up" will be used because it gives appropriate meaning. Other options give different meaning. 190. Select the most appropriate option to fill in blank number 5. (a) if (b) and (c) or (d) so SSC CHSL (Tier-I) 10/03/2023 (Shift-III) Ans. (c) : For the above blank space (5) 'or' will be used because it gives appropriate meaning. Other options give different meaning. Comprehension: In the following passage some of the words have been deleted. Read the passage carefully and select the correct answer for the given blank out of the four alternatives. I have always been at journaling. All my childhood diaries include major gaps: In one (1) ................, I am, 8, visiting my aunt. In the very (2) ..............., I have experienced puberty and have a crush on Mohit, the kid (3) ........... sits behind me in biology. Today, my journal and I are something like estrange acquaintances, who check in every now and then just to keep in (4) .............., without really understanding why. Yep, we say to each other in passing, everything's good. Hope all is (5) ............ with you! 191. Select the most appropriate option to fill in blank number 1. (a) entry (b) section (c) passing (d) terms SSC CHSL (Tier-I) 09/03/2023 (Shift-IV) Ans. (a) : For the above blank space (1) "entry" will be used because it gives appropriate meaning. Other options give different meaning. Section – Part Passing – Death/Path Terms – Precondition 192. Select the most appropriate option to fill in blank number 2. (a) next (b) former (c) earlier (d) over SSC CHSL (Tier-I) 09/03/2023 (Shift-IV) Ans. (a) : For the above blank space (2) 'next' will be used because it gives appropriate meaning. Other options give different meaning. 193. Select the most appropriate option to fill in blank number 3. (a) who (b) whose (c) whom (d) which SSC CHSL (Tier-I) 09/03/2023 (Shift-IV) Ans. (a) : For the above blank space (3) who will be used because Relative Pronoun "who" is used for (Living things) 'The kid'. Other options give different meaning. Whom – Relative pronoun (objective case) which – Relative Pronoun (non - living thins) whose – Possessive adjective.

781

YCT

194. Select the most appropriate option to fill in blank number 4. (a) taste (b) vision (c) glory (d) touch SSC CHSL (Tier-I) 09/03/2023 (Shift-IV) Ans. (d) : For the above blank space (4) "touch" will be used because it gives suitable meaning. Other options give different meaning. Vision – View Taste – Relish Glory – Dignity 195. Select the most appropriate option to fill in blank number 5. (a) well (b) mild (c) virtuous (d) bling SSC CHSL (Tier-I) 09/03/2023 (Shift-IV) Ans. (a) : For the above blank space (5) 'well' will be used because it gives appropriate meaning. Other options give different meaning. mild –Gentle, Soft virtuous – Religious Bling – Flashy Comprehension : In the given passage some words have been deleted. Fill in the blanks with the help of the alternatives given. Select the most appropriate option for each number. Over the past year, Dr Hemmings has been conducting research on the big cat phenomenon and has already 1.________the remains of some wild animals that may have been eaten by 2._______ far larger than any of the country’s known carnivores. The project has 3._________an analysis of twenty skeletal animal remains 4.________ from across Gloucestershire and other nearby counties. The bones were selected because the 5.___________ of their death led people to believe that these animals may have been killed by a big cat. 196. Select the most appropriate option to fill in blank No.1 (a) decided (b) invented (c) settled (d) identified SSC CGL (Tier-II) 08.08.2022 Shift-II Ans. (d) : For the above blank space (1) of the passage, 'identified' will be used, other options are not correct. 197. Select the most appropriate option to fill in blank No. 2 (a) creatures (b) mortals (c) individuals (d) people SSC CGL (Tier-II) 08.08.2022 Shift-II Ans. (a) : For the above blank space (2) of the passage, 'creatures' gives suitable meaning other options are not correct. Cloze Test

198. Select the most appropriate option to fill in blank No. 3 (a) related (b) concerned (c) connected (d) involved SSC CGL (Tier-II) 08.08.2022 Shift-II Ans. (d) : for the above blank space (3) of the passage, involved will be used. Other options are not correct. 199. Select the most appropriate option to fill in blank No. 4 (a) redeemed (b) revoked (c) reclaimed (d) recovered SSC CGL (Tier-II) 08.08.2022 Shift-II Ans. (d) : For the above blank space (4) of the passage, 'recovered' will be used. Other options are not correct. 200. Select the most appropriate option to fill in the blank No. 5 (a) opportunities (b) potential (c) affairs (d) circumstances SSC CGL (Tier-II) 08.08.2022 Shift-II Ans. (d) : For the above blank space (5) of the passage, 'circumstances' will be used. Other options are not correct. Comprehension: In the following passage some words have been deleted. Fill in the blanks with the help of the alternative given. Select the most appropriate option for each blank. The locusts is a insect that belongs to the family of grasshoppers. These insects are essentially harmless (1) ............ they meet certain circumstances under which they (2) .............. more abundant and change their behaviour. India, Pakistan and Iran (3) .......... currently thinking over the locust plague threat (4) ........... over their economies. When these locusts meet (5) ........... environmental conditions, they become gregarious and (6) .............. breeding abundantly. With their population becoming (7) .........., they form swarms and keep moving (8) ........... areas damaging the crops. These swarms attacking (9) .............. and thereby devastating the entire agricultural economy is (10) ............. is commonly referred to as locust plague. 201. Select the most appropriate option for blank no. 1. (a) although (b) when (c) after (d) unless SSC Stenographer (Grade C & D) 15.11.2021 Shift-II Ans. (d) : For the above blank space (1) of the passage, 'unless' will be used. Other options are not correct. 202. Select the most appropriate option for blank no. 2. (a) become (b) have become (c) are becoming (d) became SSC Stenographer (Grade C & D) 15.11.2021 Shift-II

782

YCT

Ans. (a) : for the above blank space (2) of the passage. 'become' will be used because sentence is in present indefinite tense. 203. Select the most appropriate option for blank no. 3. (a) are (b) were (c) was (d) is SSC Stenographer (Grade C & D) 15.11.2021 Shift-II Ans. (a) : For the above blank space (3) of the passage, 'are' will be used because plural verb is used for plural 'noun/pronoun'. 204. Select the most appropriate option for blank no. 4. (a) remaining (b) lingering (c) staying (d) leaving SSC Stenographer (Grade C & D) 15.11.2021 Shift-II Ans. (b) For the above blank space (4) of the passage, 'lingering' will be used. Other options are not correct. 205. Select the most appropriate option for blank no. 5. (a) suiting (b) suitably (c) suitable (d) suited SSC Stenographer (Grade C & D) 15.11.2021 Shift-II Ans. (c) :For the above blank space (5) of the passage, 'suitable' will be used. other options are not correct. 206. Select the most appropriate option for blank no. 6. (a) start (b) create (c) launch (d) set up SSC Stenographer (Grade C & D) 15.11.2021 Shift-II Ans. (a) : For the above blank space (6) of the passage, 'start' will be used. Other options are not correct. 207. Select the most appropriate option for blank no. 7. (a) most dense (b) densest (c) much dense (d) denser SSC Stenographer (Grade C & D) 15.11.2021 Shift-II Ans. (d) : For the above blank space (7), 'denser' will be used. Other options are not correct. 208. Select the most appropriate option for blank no. 8. (a) against (b) about (c) across (d) around SSC Stenographer (Grade C & D) 15.11.2021 Shift-II Ans. (c) : For the above blank space (8), 'across' will be used. Other options ar not correct. 209. Select the most appropriate option for blank no. 9. (a) seeds (b) grains (c) flowers (d) crops SSC Stenographer (Grade C & D) 15.11.2021 Shift-II Ans. (d) : For the above blank space (9), ' crops' , will be used. Other options are not correct. Cloze Test

210. Select the most appropriate option for blank no. 10. (a) whom (b) who (c) whose (d) what SSC Stenographer (Grade C & D) 15.11.2021 Shift-II Ans. (d) : For the above blank space (10), 'what' will be used. Other options are not correct. Comprehension: In the following passage some words have been deleted. Fill in the blanks with the help of the alternatives given. Select the most appropriate option for each blank. Polio was once a disease feared worldwide, striking suddenly and paralysing mainly children (1) ........... life. WHO is a partner in the Global Polio Eradication Initiative, the (2) ........... private-public partnership for health, which has (3) .......... polio by 99%. Polio now survives (4) ......... among the world's three poorest and (5) ............ marganalised communities, where it stalks the most (6) .......... children. These are–Afghanistan, Nigeria and Pakistan. (7) ........... poliovirus transmission is interrupted in these countries, all (8) ............ remain at risk of importation of polio, (9) ......... vulnerable countries with weak public health and immunisation services. WHO's goal is (10) ........... every last child with polio vaccine and ensure a polio-free world for future generations. 211. Select the most appropriate option for blank no. 1. (a) with (b) for (c) about (d) in SSC Stenographer (Grade C & D) 15.11.2021 Shift-I Ans. (b) : For the above blank space (1), ' for will be used. Other options are not correct. 212. Select the most appropriate option for blank no. 2. (a) most large (b) more large (c) larger (d) largest SSC Stenographer (Grade C & D) 15.11.2021 Shift-I Ans. (d) : For the above blank space (2) of the passage, 'largest' will be used because superlative degree is used before article 'the'. 213. Select the most appropriate option for blank no. 3. (a) reducing (b) reduced (c) reduction (d) reduce SSC Stenographer (Grade C & D) 15.11.2021 Shift-I Ans. (b) : For the above blank space (3) of the passage, 'reduced' will be used because the sentence is in Present Perfect tense. 214. Select the most appropriate option for blank no. 4. (a) alone (b) only (c) singly (d) rarely SSC Stenographer (Grade C & D) 15.11.2021 Shift-I

783

YCT

Ans. (b) : For the above blank space (4) of the passage, 'only' will be used. Other options are not correct. 215. Select the most appropriate option for blank no. 5. (a) a lot (b) most (c) more (d) much SSC Stenographer (Grade C & D) 15.11.2021 Shift-I Ans. (b) : For the above blank space (5) of the passage, 'most' will be used. Other options are not correct. 216. Select the most appropriate option for blank no. 6. (a) weak (b) vulnerable (c) pathetic (d) risky SSC Stenographer (Grade C & D) 15.11.2021 Shift-I Ans. (b) : For the above blank space (6) of the passage, 'vulnerable' will be used. Other options are not correct. 217. Select the most appropriate option for blank no. 7. (a) Until (b) While (c) When (d) If SSC Stenographer (Grade C & D) 15.11.2021 Shift-I Ans. (a) : For the above blank space (7), 'until' will be used. Other options are not correct. 218. Select the most appropriate option for blank no. 8. (a) states (b) countries (c) towns (d) villages SSC Stenographer (Grade C & D) 15.11.2021 Shift-I Ans. (b) : For the above blank space (8) 'countries' will be used. Other options are not correct. 219. Select the most appropriate option for blank no. 9. (a) in principle (b) at particular (c) immediately (d) especially SSC Stenographer (Grade C & D) 15.11.2021 Shift-I Ans. (d) : For the above blank space (9) 'especially' will be used. Other options are not correct. 220. Select the most appropriate option for blank no. 10. (a) reached (b) to reach (c) to reaching (d) reach SSC Stenographer (Grade C & D) 15.11.2021 Shift-I Ans. (b) : For the above blank space (10) 'to reach' will be used, because 'to + V1 is used as infinitive. Other options are not correct. Comprehension: In the following passage, some words have been deleted. Read the passage carefully and select the most appropriate option to fill in each blank. Most of the valley had been (1)_________ over the ages by the river waters, (2)_________ rainfall, and fierce winds. All except for one giant monolith, a (3)_________ tower-like mountain of a single rock. It stood at a proud (4)_________ of eight hundred and fifty metres from the valley floor, towering well above the (5)_________ shoulders. Cloze Test

221. Select the most appropriate option to fill in blank number 1. (a) eroded (b) wiped (c) decoded (d) dried SSC CHSL 30.05.2022 Shift-I Ans. (a) : For the above blank space (1), 'eroded' will be used. Other options are not correct. 222. Select the most appropriate option to fill in blank number 2. (a) heavy (b) scanty (c) less (d) empty SSC CHSL 30.05.2022 Shift-I Ans. (a) : For the above blank space, (2), ' heavy' will be used. Other options are not correct. 223. Select the most appropriate option to fill in blank number 3. (a) neat (b) tiny (c) small (d) humongous SSC CHSL 30.05.2022 Shift-I Ans. (d) : For the above blank space, (3) 'humongous' will be used. Other options are not correct. 224. Select the most appropriate option to fill in blank number 4. (a) length (b) height (c) width (d) weight SSC CHSL 30.05.2022 Shift-I Ans. (b) : For the above blank space, (4), 'height' will be used. Other options are not correct. 225. Select the most appropriate option to fill in blank number 5. (a) valley's (b) mountain (c) valley (d) shelf's SSC CHSL 30.05.2022 Shift-I Ans. (a) : For the above blank space (5), 'valley's ' will be used. Other options are not correct. Comprehension: In the following passage, some words have been deleted. Read the passage carefully and select the most appropriate option to fill in each blank. Competition is a necessary prospect for (1)______________ individuals with particular qualities, but only those who can (2)__________ their minds to work, work hard every day, and prove themselves will be able to (3)_____________ this arduous battle. It's a neverending race to the finish line, and only those who put in the effort and dedication will make it to the end. The outcome of any exam does not (4)__________ on the last day, during your paper — your success is ensured with every morning that you put in work. Students must be aware of the tough environment they are entering and prepare (5)______________ in order to improve their chances.

784

YCT

226. Select the most appropriate option to fill in blank number 1. (a) distinguishing (b) distinguish (c) distinguished (d) distinguishes SSC CHSL 24.05.2022 Shift-III Ans. (a) : For the above blank space (1), 'distinguishing' will be used. Other options are not correct. 227. Select the most appropriate option to fill in blank number 2. (a) divert (b) get (c) put (d) made SSC CHSL 24.05.2022 Shift-III Ans. (c) : For the above blank space (2) 'put' will be used. Other options are not correct. 228. Select the most appropriate option to fill in blank number 3. (a) overload (b) overjoyed (c) overpaid (d) overcome SSC CHSL 24.05.2022 Shift-III Ans. (d) : For the above blank space (3) 'overcome' will be used. Other options are not correct. 229. Select the most appropriate option to fill in blank number 4. (a) arrive (b) awake (c) abase (d) arise SSC CHSL 24.05.2022 Shift-III Ans. (a) : For the above blank space (4) 'arrive' will be used. Other optins are not correct. 230. Select the most appropriate option to fill in blank number 5. (a) accordingly (b) exponentially (c) consequently (d) reluctantly SSC CHSL 24.05.2022 Shift-III Ans. (a) : For the above blank space (5) 'accordingly' will be used. Other options are not correct. Comprehension: In the following passage, some words have been deleted. Read the passage carefully and select the most appropriate option to fill in each blank. The Hall of Dharma was in a circular building, built of stone and mortar, with a (1)________ dome. The delicate (2)________of the dome was believed to represent the feminine while the typical temple spire represented the masculine. The hall was also (3)________ All rishis sat as (4)________ without a moderating ‘head’, debating issues openly and without fear, freedom of (5)________at its zenith. 231. Select the most appropriate option to fill in blank number 1. (a) massive (b) passive (c) conclusive (d) intrusive SSC CHSL 31.05.2022 Shift-III Cloze Test

Ans. (a) : For the above blank space (1), 'massive' will be used. Other options are not correct. 232. Select the most appropriate option to fill in blank number 2. (a) elegance (b) vengeance (c) masculinity (d) harshness SSC CHSL 31.05.2022 Shift-III Ans. (a) : For the above blank space (2) 'elegance' will be used. Other options are not correct. 233. Select the most appropriate option to fill in blank number 3. (a) circular (b) vertical (c) peculiar (d) perpendicular SSC CHSL 31.05.2022 Shift-III Ans. (a) : For the above blank space (3), 'circular' will be used. Other options are not correct. 234. Select the most appropriate option to fill in blank number 4. (a) conical (b) crossed (c) equals (d) unequal SSC CHSL 31.05.2022 Shift-III Ans. (c) : For the above blank space (4) 'equals' will be used. Other options are not correct. 235. Select the most appropriate option to fill in blank number 5. (a) expression (b) depression (c) inspiration (d) running SSC CHSL 31.05.2022 Shift-III Ans. (a) : For the above blank space (5) 'expression' will be used. Other options are not correct. Comprehension: In the following passage, some words have been deleted. Read the passage carefully and select the most appropriate option to fill in each blank. Psychologists discovered that there are two routes (1) ___________ achievement: conformity and originality. Conformity means following the crowd (2) __________ conventional paths and (3) ___________ the status quo. Originality is taking the road (4) __________ travelled, championing a set of novel ideas that go against the grain but (5) __________ making things better. 236. Select the most appropriate option to fill in blank number 1. (a) off (b) against (c) on (d) to SSC CHSL 25.05.2022 Shift-I Ans. (d) : For the above blank space (1), 'to' preposition will be used. Other options are not correct. 237. Select the most appropriate option to fill in blank number 2. (a) at (b) down (c) beside (d) since SSC CHSL 25.05.2022 Shift-I

785

YCT

Ans. (b) : For the above blank space, (2) 'down' will be used. Other options are not correct. 238. Select the most appropriate option to fill in blank number 3. (a) serving (b) building (c) supplying (d) maintaining SSC CHSL 25.05.2022 Shift-I Ans. (d) : For the above blank space (3) 'maintaining' will be used. Other options are not correct. 239. Select the most appropriate option to fill in blank number 4. (a) less (b) far (c) quietly (d) frequently SSC CHSL 25.05.2022 Shift-I Ans. (a) : For the above blank space (4) 'lees' will be used. Other options are not correct. 240. Select the most appropriate option to fill in blank number 5. (a) superficially (b) disinterestedly (c) advertently (d) ultimately SSC CHSL 25.05.2022 Shift-I Ans. (d) : For the above blank space, (5) 'ultimately' will be used. Other options are not correct. Comprehension: In the following passage, some words have been deleted. Read the passage carefully and select the most appropriate option to fill in each blank. personification gives (1) ________ qualities to non-living things or ideas. Imagery is a language used by poets, novelists and other writers to create images in the mind of the (2) _______. Rhyme scheme is the (3) ___________ that is created at the end of the lines of poetry. Irony is defined as when (4) _________ meaning is completely (5) __________ from the literal meaning. 241. Select the most appropriate option to fill in blank number 1. (a) human (b) divine (c) supernatural (d) animal SSC CHSL 27.05.2022 Shift-I Ans. (a) : For the above blank space (1), 'human' will be used. Other options are not correct. 242. Select the most appropriate option to fill in blank number 2. (a) humans (b) actors (c) speakers (d) readers SSC CHSL 27.05.2022 Shift-I Ans. (d) : For the above blank space (2) 'readers' will be used. Other options are not correct. 243. Select the most appropriate option to fill in blank number 3. (a) visual (b) word (c) pattern (d) idea SSC CHSL 27.05.2022 Shift-I Cloze Test

Ans. (c) : For the above blank space (3) 'pattern' will be used. Other options are not correct. 244. Select the most appropriate option to fill in blank number 4. (a) superficial (b) unintentional (c) common (d) actual SSC CHSL 27.05.2022 Shift-I Ans. (d) : For the above blank space (4) 'actual' will be used. Other options are not correct. 245. Select the most appropriate option to fill in blank number 5. (a) against (b) different (c) parallel (d) similar SSC CHSL 27.05.2022 Shift-I Ans. (b) : For the above blank space (5) 'different' will be used. Other options are not correct. Comprehension: In the following passage, some words have been deleted. Read the passage carefully and select the most appropriate option to fill in each blank. In April 1945, during the Conference to set up the United Nations (UN) held in San Francisco, representatives of Brazil and China proposed that an international health organization be (1) ______ and a conference to frame its constitution convened. On 15 February 1946, the Economic and Social Council of the UN instructed the Secretary-General to (2)___________ such a conference. A Technical Preparatory Committee met in Paris from 18 March to 5 April 1946 and (3)___________ proposals for the Constitution which were presented to the International Health Conference in New York City (4) _______ 19 June and 22 July 1946. On the basis of these proposals, the Conference drafted and (5)___________ the Constitution of the World Health Organization, signed 22 July 1946 by representatives of 51 Members of the UN and of 10 other nations. 246. Select the most appropriate option to fill blank 1. (a) established (b) establishes (c) establishing (d) establish SSC CHSL 27.05.2022 Shift-III Ans. (a) : For the above blank space (1) 'established' will be used. Other options are not correct. 247. Select the most appropriate option to fill blank 2. (a) disperse (b) convene (c) collect (d) sit SSC CHSL 27.05.2022 Shift-III Ans. (b) : For the above blank space (2) 'convene' will be used. Other options are not correct.

786

YCT

248. Select the most appropriate option to fill blank 3. (a) drew off (b) drew away (c) drew up (d) drew down SSC CHSL 27.05.2022 Shift-III Ans. (c) : For the above blank space (3) 'drew up' will be used. Other options are not correct. 249. Select the most appropriate option to fill blank 4. (a) from (b) while (c) between (d) among SSC CHSL 27.05.2022 Shift-III Ans. (c) : For the above blank space (4) 'between' will be used, because 'between' is used for two people or things. 250. Select the most appropriate option to fill blank 5. (a) adopted (b) adapted (c) revoked (d) maintained SSC CHSL 27.05.2022 Shift-III Ans. (a) : For the above blank space (5) 'adopted' will be used, because it gives suitable meaning for the sentence. Comprehension: In the following passage, some words have been deleted. Fill in the blanks with the help of the options given. Select the most appropriate option for each blank. Tree plantation is the (1)______ of transplanting saplings or seeds to maximise the amount of greenery and focus on promoting (2)______. Planting trees can be a very rewarding and fruitful (3)______! Not only is it (4)______ beneficial for this planet, but it also contributes to helping create an environmentally friendly place to live, as trees help reduce pollutant (5)______. 251. Select the most appropriate option to fill in blank No.1. (a) Aspect (b) Mechanism (c) Process (d) Tool SSC CHSL 26.05.2022 Shift-II Ans. (c) : For the above blank space (1) 'process' will be used. Other options are not correct. 252. Select the most appropriate option to fill in blank No.2. (a) land reservation (b) afforestation (c) deforestation (d) vegetation SSC CHSL 26.05.2022 Shift-II Ans. (b) : For the above blank space (2) 'afforestation' will be used. Other options are not correct. 253. Select the most appropriate option to fill in blank No.3. (a) Order (b) Labour (c) plan (d) Endeavour SSC CHSL 26.05.2022 Shift-II Cloze Test

Ans. (d) : For the above blank space (3) 'Endeavour' will be used. Other options are not correct. 254. Select the most appropriate option to fill in blank No. 4. (a) Physically (b) Socially (c) Economically (d) Ecologically SSC CHSL 26.05.2022 Shift-II Ans. (d) : For the above blank space (4) 'Ecologically' will be used. Other options are not correct. 255. Select the most appropriate option to fill in blank No. 5. (a) calculation (b) method (c) concentration (d) problem SSC CHSL 26.05.2022 Shift-II Ans. (c) : For the above blank space (5) 'concentration' will be used. Other options are not correct. Comprehension: In the following passage, some words have been deleted. Read the passage carefully and select the most appropriate option to fill in each blank. Many forests are cleared to (1) ______ way for palm oil plantations. Palm oil is the most (2) _____ produced vegetable oil and is found in half of all supermarket products. It's cheap, versatile and can be added to (3) _______ food and personal products like lipsticks and shampoo. (4) ________ popularity has (5) ______ people to clear tropical forests to grow more palm trees. 256. Select the most appropriate option to fill in blank number 1. (a) made (b) take (c) make (d) give SSC MTS-18/07/2022 Shift-I Ans. (c) : For the above blank space (1) 'make' will be used. Other options are not correct. 257. Select the most appropriate option to fill in blank number 2. (a) commonly (b) casually (c) lovingly (d) usually SSC MTS-18/07/2022 Shift-I Ans. (a) : For the above blank space (2) 'commonly' will be used. Other options are not correct. 258. Select the most appropriate option to fill in blank number 3. (a) both (b) each (c) one (d) between SSC MTS-18/07/2022 Shift-I Ans. (a) : For the above blank space (3) 'both' will be used. Other options are not correct. 259. Select the most appropriate option to fill in blank number 4. (a) Our (b) Its (c) It's (d) Their SSC MTS-18/07/2022 Shift-I

787

YCT

Ans. (b): For the above blank space (4) 'its' will be Ans. (a) : For the above blank space (5) 'Such' will be used, because possessive adjective 'its' will be used. used. Other options are not correct. 260. Select the most appropriate option to fill in Comprehension: blank number 5. In the following passage, some words have been (a) baffled (b) delighted deleted. Read the passage carefully and select (c) curated (d) prompted the most appropriate option to fill in each SSC MTS-18/07/2022 Shift-I blank. Ans. (d) : For the above blank space (5) 'prompted' will Birbal immediately smiled and went up to be used. Other options are not correct. Akbar. He announced the (1) ____; he said there were twenty-one thousand, five hundred Comprehension : and twenty-three crows (2) ____ the city. When In the following passage, some words have been asked how he (3) ____ the answer, birbal deleted. Road the passage carefully and select replied, “Ask your men to (4) ____ the number the most appropriate option to fill in each blank. of crows. If there are more, then the relatives of A banker taped a picture, drawn by one of his the crows must be visiting them from (5) ____ small children, to his office wall. When he 1. cities. If there are fewer, then the crows from _____ at work the next morning, he found the our city must be visiting their relatives who live 2.______ was covered by a 3.________notice, outside the city.” saying he was in 4. _______ of company policy which required personal items to be put away at 266. Select the most appropriate option to fill in night. 5. ______ a reaction was not just petty, it blank number 1. risked demotivating the banker completely. In (a) question (b) response short, it defied common sense. (c) answer (d) rejoinder 261. Select the most appropriate option to fill in blank SSC CGL (Tier-I) 21/04/2022 Shift-III no. 1. (a)Arrived (c)Reached

(b) Turned (d) Achieved SSC CGL (Tier-I) 19/04/2022 Shift-III Ans. (a) : For the above blank space (1) 'arrived' will be used. Other options are not correct. 262. Select the most appropriate option to fill in blank no. 2. (a) Child (b) Tape (c) Picture (d) Office SSC CGL (Tier-I) 19/04/2022 Shift-III Ans. (c) : For the above blank spce (2) 'Picture' will be used. Other options are not correct. 263. Select the most appropriate option to fill in blank no. 3. (a) Large (b) Considerable (c) Substantial (d) Generous SSC CGL (Tier-I) 19/04/2022 Shift-III Ans. (a) : For the above blank space (3), 'large' will be used. Other options are not correct. 264. Select the most appropriate option to fill in blank no. 4. (a)Violation (b) Damage (c)Interruption (d) Disruption SSC CGL (Tier-I) 19/04/2022 Shift-III Ans. (a) : For the above blank space (4) ' Violation' will be used. Other options are not correct. 265. Select the most appropriate optoin to fill in blank no. 5. (a) Such (b) That (c) This (d) Which SSC CGL (Tier-I) 19/04/2022 Shift-III Cloze Test

Ans. (c) : For the above blank space (1) 'answer' will be used. Other options are not correct. 267. Select the most appropriate option to fill in blank number 2. (a) in (b) on (c) at (d) off SSC CGL (Tier-I) 21/04/2022 Shift-III Ans. (a) : For the above blank space (2) 'in' will be used. Other options are not correct. 268. Select the most appropriate option to fill in blank number 3. (a) Know (b) Knew (c) Knowing (d) Known SSC CGL (Tier-I) 21/04/2022 Shift-III Ans. (b) : For the above blank space (3) 'Knew' will be used. Other options are not correct. 269. Select the most appropriate option to fill in blank number 4. (a) calculate (b) judge (c) count (d) consider SSC CGL (Tier-I) 21/04/2022 Shift-III Ans. (c) : For the above blank space (4) 'count' will be used. Other options are not correct. 270. Select the most appropriate option to fill in blank number 5. (a) nearby (b) close (c) near (d) vicinity SSC CGL (Tier-I) 21/04/2022 Shift-III

788

YCT

Ans. (a) : For the above blank space (5) 'nearby' will be Comprehension: In the following passage, some words have been used. Other options are not correct. deleted. Read the passage carefully and select the Comprehension: most appropriate option to fill in each blank. In the following passage, some words have been The camel and the fox were very good friends deleted. Read the passage carefully and select and very good thieves. One day, the two friends the most appropriate option to fill in each (1) ______ to cross the river so that they could blank. travel to a (2) ______ farm to steal food. The For an average Indian, driving in any of the small fox (3) ______ swim, so the camel said to major Indian cities is equivalent to (1) ____ a his friend, ‘Climb (4) ______ onto my back and daily war. The everyday struggle of (2) ____ I will swim across the (5) ______.’ traffic, pollution and rash drivers is (3) ____ 276. Select the most appropriate option to fill in biggest cause of chronic stress and other (4) blank number 1. _____ problems. On an average a person (a) will decide (b) are decide spends (5) _____ between thirty minutes to two (c) decided (d) deciding hours of their day driving. SSC CGL (Tier-I) 18/04/2022 Shift-I 271. Select the most appropriate option to fill in Ans. (c) : According to the passage, the most blank no. 1. appropriate word for the given blank space (1) is (a) raising (b) making ‘decided’ (V2) because the sense of this sentence is in past indefinite tense. (c) rushing (d) waging SSC CGL (Tier-I) 20/04/2022 Shift-III Thus, the correct answer is option (c). Ans. (d) : For the above blank space (1), 'waging' will 277. Select the most appropriate option to fill in blank number 2. be used. Other options are not correct. (a) nearly (b) nearby 272. Select the most appropriate option to fill in (c) nearest (d) near blank no. 2. SSC CGL (Tier-I) 18/04/2022 Shift-I (a) boarding (b) stopping Ans. (b) : According to the passage, the most (c) lodging (d) dodging appropriate word for the given blank space (2) is SSC CGL (Tier-I) 20/04/2022 Shift-III ‘nearby’. Ans. (d) : For the above blank space (2) 'dodging' will Thus, the correct answer is option (b). be used. Other options are not correct. 278. Select the most appropriate option to fill in blank number 3. 273. Select the most appropriate option to fill in (a) ought to (b) may not blank no. 3. (c) might (d) could not (a) an (b) the SSC CGL (Tier-I) 18/04/2022 Shift-I (c) one (d) a Ans. (d) : According to the passage, the most SSC CGL (Tier-I) 20/04/2022 Shift-III appropriate word for the given blank space (3) is Ans. (b) : For the above blank space (3) 'the' will be ‘could not’ because the model verb 'could' is used to used, because definite article 'the' is used before talk about past ability or capacity. superlative degree. Thus, the correct option is (d). 274. Select the most appropriate option to fill in 279. Select the most appropriate option to fill in blank no. 4. blank number 4. (a) ecological (b) spiritual (a) over (b) above (c) across (d) up (c) psychological (d) societal SSC CGL (Tier-I) 18/04/2022 Shift-I SSC CGL (Tier-I) 20/04/2022 Shift-III Ans. (c) : For the above blank space (4) 'Psychological' Ans. (d) : According to the passage, the most appropriate word for the given blank space (4) is ‘up’. will be used. Other options are not correct. because ‘Climb up’ means ‘to go up something towards 275. Select the most appropriate option to fill in the top.’ blank no. 5. Thus, the correct option is (d). (a) nowhere (b) everywhere 280. Select the most appropriate option to fill in (c) anywhere (d) elsewhere blank number 5. SSC CGL (Tier-I) 20/04/2022 Shift-III (a) ocean (b) river Ans. (c) : For the above blank space (5) 'anywhere' will (c) sea (d) lake be used. Other options are not correct. SSC CGL (Tier-I) 18/04/2022 Shift-I Cloze Test

789

YCT

Ans. (b) : According to the passage, the most suitable word for the given blank space (5) is ‘river’. Other options are different in meaning. Thus, the correct answer is option (b). Comprehension: In the following passage, some words have been deleted. Read the passage carefully and select the most appropriate option of fill in each blank. There is always something so fascinating about trains and journeys. There (1)……….. certain symbols and signages inside and outside of the train. The (2)……….. of most of them might not be known to everyone. (3)……….. you are frequent traveller or not, you may have (4)………… the letter X or a cross mark on the back of the last (5)……….. of the train. 281. Select the most appropriate option to fill in blank number 1. (a) are (b) were (c) will be (d) had been SSC CGL (Tier-I) 13/04/2022 Shift-III Ans. (a) : For the above blank space (1) 'are' will be used. Because sentence will be in present indefinite tense. 282. Select the most appropriate option to fill in blank number 2. (a) meanings (b) senses (c) worth (d) charge SSC CGL (Tier-I) 13/04/2022 Shift-III Ans. (a) :For the above blank space (2)of the passage, 'meanings' will be used. Other options are not correct. 283. Select the most appropriate option to fill in blank number 3. (a) whether (b) Whichever (c) However (d) Whatever SSC CGL (Tier-I) 13/04/2022 Shift-III Ans. (a) : For the above blank space (3) of the passage, 'whether' will be used. Other options are not correct. 284. Select the most appropriate option of fill in bank number 4. (a) recognised (b) known (c) distinguished (d) noticed SSC CGL (Tier-I) 13/04/2022 Shift-III Ans. (d) : For the above blank space (4) of the passage, 'noticed' will be used. Other options are not correct. 285. Select the most appropriate option to fill in blank number 5. (a) booth (b) coach (c) rail (d) bus SSC CGL (Tier-I) 13/04/2022 Shift-III Ans. (b) : For the above blank space (5), 'Coach' will be used. Other options are not correct.

have been (1) ______ on selected roads. The scheme has now been in (2) ______ for a year and has been hailed as a (3) ______ success. Since the new speed limits were (4) ______, the number of accidents in the area have fallen (5) ______. 286. Select the most appropriate option to fill in blank no.1 (a) reduced (b) scaled (c) lessened (d) shortened SSC CGL (Tier-I) 12/04/2022 Shift-III Ans. (a) : For the above blank space (1) of the passage, 'reduced' will be used. Other options are not

correct. 287. Select the most appropriate option to fill in blank no.2. (a) operation (b) activity (c) process (d) agency SSC CGL (Tier-I) 12/04/2022 Shift-III Ans. (a) :For the above blank space (2) of the passage, 'operation' will be used. Other options are not correct. 288. Select the most appropriate option to fill in blank no.3. (a) great (b) more greater (c) greatest (d) greater SSC CGL (Tier-I) 12/04/2022 Shift-III Ans. (a) For the above blank space (3) of the passage, 'great' will be used. Other options are not correct. 289. Select the most appropriate option to fill in blank no.4. (a) dictated (b) enforced (c) exacted (d) compelled SSC CGL (Tier-I) 12/04/2022 Shift-III Ans. (b) :For the above blank space (4) of the passage, 'enforced' will be used. Other options are not correct. 290. Select the most appropriate option to fill in blank no.5. (a) extremely (b) drastically (c) desperately (d) terribly SSC CGL (Tier-I) 12/04/2022 Shift-III Ans. (b) : For the above blank space (5) 'drastically' will be used. Other options are not correct.

Comprehension: In the following passage, some words have been deleted. Read the passage carefully and select the most appropriate option to fill in each blank. The greedy man rushed home to tell his wife and daughter about his wish, all the (1)_____ touching the objects in his path and watching Comprehension: (2)_____convert into gold. Once he got home, In the following passage, some words have been his daughter (3) _______ to greet him. As soon deleted. Read the passage carefully and select the as he bent (4) _____ to scoop her up in his most appropriate option to fill in each blank. arms, she turned into a golden statue. He was (5) ______ and started crying and trying to New cycle routes have been built in and around the bring his daughter back to life. centre of Birmingham and speed limits Cloze Test

790

YCT

291. Select the most appropriate option to fill in blank number 1. (a) interval (b) wayside (c) while (d) bit SSC CGL (Tier-I) 13/04/2022 Shift-II Ans. (c) : For the above blank space (1) 'while' will be used. Other options are not correct. 292. Select the most appropriate option to fill in blank number 2. (a) them (b) their (c) there (d) those SSC CGL (Tier-I) 13/04/2022 Shift-II Ans. (a) For the above blank space (2) 'Them' will be used. Other options are not correct. 293. Select the most appropriate option to fill in blank number 3. (a) rushed (b) has rushed (c) was rush (d) rushing SSC CGL (Tier-I) 13/04/2022 Shift-II Ans. (a) : For the above blank space (3) 'Rushed' will be used. Other options are not correct. 294. Select the most appropriate option to fill in blank number 4. (a) up (b) through (c) down (d) along SSC CGL (Tier-I) 13/04/2022 Shift-II Ans. (c) : For the above blank space (4) 'Down' will be used. Other options are not correct. 295. Select the most appropriate option to fill in blank number 5. (a) demolished (b) devastated (c) destroyed (d) determined SSC CGL (Tier-I) 13/04/2022 Shift-II Ans. (b) : For the above blank space (5) 'Devastated' will be used. Other options are not correct. Comprehension: In the following passage, some words have been deleted. Read the passage carefully and select the most appropriate option to fill in each blank. The rise in the Irrawaddy dolphin (1) ______ in Chilika can be attributed to the eviction of (2) ______ fish enclosures. After thousands of hectares of Chilika lake were made (3) ______ free, Irrawaddy dolphins found unobstructed area for movement. (4) ______, due to the COVID-19 lockdown last year, there were comparatively fewer tourist boats on Chilika lake, which made it (5) ______ for dolphins to move from one part of the lake to another. 296. Select the most appropriate option to fill in blank no.1. (a) natives (b) inhabitants (c) population (d) clan SSC CGL (Tier-I) 11/04/2022 Shift-I Cloze Test

Ans. (c) : For the above blank space (1) 'population' will be used. Other options are not correct. 297. Select the most appropriate option to fill in blank no.2. (a) unwarranted (b) illegitimate (c) unconstitutional (d) illegal SSC CGL (Tier-I) 11/04/2022 Shift-I Ans. (d) : For the above blank space (2) 'illegal' will be used. Other options are not correct. 298. Select the most appropriate option to fill in blank no. 3. (a) confiscation (b) encroachment (c) intervention (d) trespass SSC CGL (Tier-I) 11/04/2022 Shift-I Ans. (b) : For the above blank space (3) 'encroachment' will be used. Other options are not correct. 299. Select the most appropriate option to fill in blank no.4. (a) Nevertheless (b) Moreover (c) However (d) Whereas SSC CGL (Tier-I) 11/04/2022 Shift-I Ans. (b) : For the above blank space (4) 'moreover' will be used. Other options are not correct. 300. Select the most appropriate option to fill in blank no.5 (a) conducive (b) disturbing (c) detrimental (d) hurtful SSC CGL (Tier-I) 11/04/2022 Shift-I Ans. (a) : For the above blank space (5) 'Conducive' will be used. Other options are not correct. Comprehension: In the following passage, some words have been deleted. Read the passage carefully and select the most appropriate option to fill in each blank. How should one read a story to a child? Usually, a personality or character of the story (1) ______ the child’s interest the most. (2) ______, use different voices (3) ______ each character or act out various (4) ______ from the story, to bring life to those characters. (5) ______ out parts from the story will make story sessions fun and memorable for the kids. 301. Select the most appropriate option to fill in blank number 1. (a) catch (b) catches (c) caught (d) has caught SSC CGL (Tier-I) 11/04/2022 Shift-III Ans. (b) : For the above blank space (1) 'Catches' will be used. Other options are not correct. 302. Select the most appropriate option to fill in blank number 2. (a) Either (b) Also (c) So (d) However SSC CGL (Tier-I) 11/04/2022 Shift-III

791

YCT

Ans. (c) : For the above blank space (2) 'so' will be used. Other options are not correct. 303. Select the most appropriate option to fill in blank number 3. (a) with (b) for (c) from (d) by SSC CGL (Tier-I) 11/04/2022 Shift-III Ans. (b) : For the above blank space (3) 'for' will be used. Other prepositions are not correct. 304. Select the most appropriate option to fill in blank number 4. (a) sheets (b) pages (c) parts (d) words SSC CGL (Tier-I) 11/04/2022 Shift-III Ans. (c) : For the above blank space (4) of the pasage, 'parts' will be used. Other options are not correct. 305. Select the most appropriate option to fill in blank number 5. (a) Telling (b) Acting (c) Asking (d) Saying SSC CGL (Tier-I) 11/04/2022 Shift-III Ans. (b) : For the above blank space (5) of the passage, 'Acting' will be used. Other options are not correct. Comprehension: In the following passage, some words have been deleted. Read the passage carefully and select the most appropriate option to fill in each blank. An old man lived in the village. He was one of the (1) ______ unfortunate people in the world. The whole village was tired (2) ______ him; he was always gloomy, he (3) ______ complained and was always in a bad mood. The (4) ______ he lived, the more vile he was becoming and the more (5) ______ were his words. 306. Select the most appropriate option to fill in blank number 1. (a) main (b) utmost (c) most (d) maximum SSC CGL (Tier-I) 12/04/2022 Shift-I Ans. (c) : For the above blank space (1) of the passage. 'most' will be used. Other options are not correct. 307. Select the most appropriate option to fill in blank number 2. (a) at (b) by (c) from (d) of SSC CGL (Tier-I) 12/04/2022 Shift-I Ans. (d) : For the above blank space (2) of the passage, 'of' will be used, because 'of ' preposition is used with tired. 308. Select the most appropriate option to fill in blank number 3. (a) mostly (b) cyclically (c) commonly (d) constantly SSC CGL (Tier-I) 12/04/2022 Shift-I Cloze Test

Ans. (d) : For the above blank space (3)of the passage, 'Constantly' will be used. Other options are not correct. 309. Select the most appropriate option to fill in blank number 4. (a) long (b) lengthy (c) longest (d) longer SSC CGL (Tier-I) 12/04/2022 Shift-I Ans. (d) : For the above blank space (4) of the passage, 'longer' will be used. Other options are not correct. 310. Select the most appropriate option to fill in blank number 5. (a) fatal (b) poisonous (c) mortal (d) toxic SSC CGL (Tier-I) 12/04/2022 Shift-I Ans. (b) : For the above blank space (5) of the passage, 'Poisonous' will be used. Other options are not correct. Comprehension: In the following passage, some words have been deleted. Read the passage carefully and select the most appropriate option to fill in each blank. Anthony Abraham, a 34-year-old crane driver, (1) ______ managed to get out of the way of falling steel. Turning around, he could hardly believe the (2) ______ scene that lay before him. Girders were (3) ______ on top of the crane in a haphazard fashion, like gigantic pick up sticks. All he could see of the machine was its rear wheels and a section of what had been the cab. Abraham (4) ______ to the cab. shouting his friend George Burns' name. By then, three other iron workers had reached the (5) ______ of the carnage. One of them yelled, 'George, can you hear me over the din of the diesel engine, a strained voice responded. 311. Select the most appropriate option to fill in blank number 1. (a) substantially (b) simultaneously (c) barely (d) rarely SSC MTS-15/07/2022 Shift-I Ans. (c) : For the above blank space (1) of the passage, 'barely' will be used. Other options are not correct. 312. Select the most appropriate option to fill in blank number 2. (a) criminal (b) crooked (c) pleasant (d) horrible SSC MTS-15/07/2022 Shift-I Ans. (d) : For the above blank space (2) of the passage, 'horrible' will be used. Other options are not correct. 313. Select the most appropriate option to fill in blank number 3. (a) dived (b) piled (c) duped (d) riled SSC MTS-15/07/2022 Shift-I

792

YCT

Ans. (b) : For the above blank space (3) of the passage, 'Piled' will be used. Other options are not correct. 314. Select the most appropriate option to fill in blank number 4. (a) shoved (b) raced (c) halted (d) assisted SSC MTS-15/07/2022 Shift-I Ans. (b) : For the above blank space (4) of the passage, 'raced' will be used. Other options are not correct. 315. Select the most appropriate option to fill in blank number 5. (a) sign (b) trace (c) scene (d) sight SSC MTS-15/07/2022 Shift-I Ans. (c) : For the above blank space (5) of the passage, 'scene' will be used. Other options are not correct. Comprehension: In the following passage, some words have been deleted. Read the passage carefully and select the most appropriate option to fill in each blank. The Government is taking seriously the (1) ______ of road safety and is committed to reducing the number of people killed in accidents. Forums are being conducted (2) ______ to bring law-makers, law enforcement officers and road users together to discuss ways to (3) ______ the rise in road accidents. Everyone agrees that more needs to be done on road safety in its determination to (4)_____ the rate of road accidents in the country. The proper road culture in children, young adults and the public in general. 316. Select the most appropriate option of fill in blank number 1. (a) release (b) residue (c) issue (d) site SSC MTS-06/07/2022 Shift-I Ans. (c) : For the above blank space (1) of the passage, 'issue' will be used. Other options are not correct. 317. Select the most appropriate option to fill in blank number 2. (a) regularly (b) habitually (c) terminally (d) rarely SSC MTS-06/07/2022 Shift-I Ans. (a) : For the above blank space (2) of the passage, 'regularly' will be used. Other options are not correct. 318. Select the most appropriate option to fill in blank number 3. (a) confess (b) tackle (c) conduct (d) deal SSC MTS-06/07/2022 Shift-I Ans. (b) : For the above blank space (3) of the passage, 'tackle' will be used. Other options are not correct. Cloze Test

319. Select the most appropriate option to fill in blank number 4. (a) curb (b) lure (c) incite (d) promote SSC MTS-06/07/2022 Shift-I Ans. (a) : For the above blank space (4) of the passage, 'curb' will be used. Other options are not correct. 320. Select the most appropriate option to fill in blank number 5. (a) acknowledgements (b) powers (c) efforts (d) struggles SSC MTS-06/07/2022 Shift-I Ans. (c) : For the above blank space (5), 'efforts' will be used. Other options are not correct. Comprehension: In the following passage, some words have been deleted. Read the passage carefully and select the most appropriate option to fill in each blank. Can you (1) ______ a deaf woman writing stories about a war? Well, Laura Redden Searing did this. Moreover, she went to foreign countries to write stories. She was very ill when she was 11 years old. Because of this, she lost her (2) _______. She went to the Missouri Scholl for the Deaf. Laura was 19 years old when she went to work for a newspaper. She wrote about people, places and art. In 1859, most women were just wives and (3) ______. They did not work outside the home; otherwise, their husbands would be angry. Due to this feeling, Laura used (4) ______ name when she wrote her stories. She called herself Howard Glyndon. She thought that unless she used a masculine name, people would be (5) _____ since they not think that women should write for newspapers. 321. Select the most appropriate option to fill in blank number 1. (a) malign (b) create (c) conjure (d) imagine SSC MTS-07/07/2022 Shift-I Ans. (d) : For the above blank space (1), 'imagine' will be used. Other options are not correct. 322. Select the most appropriate option to fill in blank number 2. (a) hearing (b) suffering (c) enduring (d) inspiring SSC MTS-07/07/2022 Shift-I Ans. (a) : For the above blank space (2) 'hearing' will be used. Other options are not correct. 323. Select the most appropriate option to fill in blank number 3. (a) artists (b) writers (c) mothers (d) teachers SSC MTS-07/07/2022 Shift-I

793

YCT

Ans. (c) : For the above blank space (3) 'Mothers' will be used. Other options are not correct. 324. Select the most appropriate option to fill in blank number 4. (a) woman's (b) children's (c) adult's (d) man's SSC MTS-07/07/2022 Shift-I Ans. (d) : For the above blank space (4) 'man's' will be used. Other options are not correct. 325. Select the most appropriate option to fill in blank number 5. (a) infantile (b) versatile (c) hostile (d) infertile SSC MTS-07/07/2022 Shift-I Ans. (c) : For the above blank space (5) 'hostile' will be used. Other options are not correct. Comprehension: In the following passage, some words have been deleted. Read the passage carefully and select the most appropriate option to fill in each blank. A group of researchers has (1) ______ that pleasure and positive states of mind are better for our health. This new intellectual approach to health is not only more powerful, but also has no side effects. Central to this claim are recent findings that even getting an education may add as much as 10 years to your (2) ______. That is why National Geographic featured John de Rosen in its book the Incredible Machine, which discussed old age. De Rosen, an artist, continued to paint until the week he died at age 91. The book notes: "Some scientists believe that retirement to a (3) _______ lifestyle initiates or aggravates medical problems, thus shortening life. According to a study of retired people, adults over 65 can learn a (4) _______skill, like oil painting, as readily as younger students. "So retiring from a job in a sense means retiring from life unless (5) ______ by some other, preferably new activity. 326. Select the most appropriate option to fill in blank number 1. (a) administrated (b) orchestrated (c) defenestrated (d) demonstrated SSC MTS-05/07/2022 Shift-III Ans. (d) : For the above blank space (1) 'demonstrated' will be used. Other options are not correct. 327. Select the most appropriate option to fill in blank number. (a) insurance (b) charisma (c) planet (d) Life SSC MTS-05/07/2022 Shift-III Cloze Test

Ans. (d) : For the above blank space (2) of the passage, 'Life' will be used. Other options are not correct. 328. Select the most appropriate option to fill in blank number 3. (a) complimentary (b) sanitary (c) sedentary (d) segmentary SSC MTS-05/07/2022 Shift-III Ans. (c) : For the above blank space (3), Sedentary will be used. Other options are not correct. 329. Select the most appropriate option to fill in blank number 4. (a) native (b) creative (c) discriminative (d) exaggerative SSC MTS-05/07/2022 Shift-III Ans. (b) : For the above blank space (4) 'creative' will be used. Other options are not correct. 330. Select the most appropriate option to fill in blank number 5. (a) ornamented (b) pigmented (c) supplemented (d) experimented SSC MTS-05/07/2022 Shift-III Ans. (c) : For the above blank space (5) 'supplemented' will be used. Other options are not correct. Comprehension: In the following passage, some words have been deleted. Read the passage carefully and select the most appropriate option to fill in each blank. Mt. Mantap in North Korea has now been reported to have (1) ______ from its original state after an underground nuclear test was carried out under it at the Punggye-ri nuclear test site on 3 September 2017. An international team of scientists (2) ______ a special satellite radar imaging called synthetic aperture radar and seismic recordings to study the effects of the blast which (3) ______ a 5.2 magnitude earthquake. The surface of the mountain was pushed (4) ______ by almost 11 feet and then collapsed leaving the mountain about 20 inches (5)______. 331. Select the most appropriate option to fill in blank number 1. (a) relapsed (b) clasped (c) mapped (d) collapsed SSC MTS-06/07/2022 Shift-II Ans. (d) : For the above blank space (1) 'collapsed' will be used. Other options are not correct. 332. Select the most appropriate option to fill in blank number 2. (a) framed (b) used (c) urged (d) deduced SSC MTS-06/07/2022 Shift-II

794

YCT

Ans. (b) : For the above blank space (2) 'used' will be used. Other options are not correct. 333. Select the most appropriate option to fill in blank number 3. (a) altered (b) neglected (c) caused (d) lost SSC MTS-06/07/2022 Shift-II Ans. (c) : For the above blank space (3) 'caused' will be used. Other options are not correct. 334. Select the most appropriate option to fill in blank number 4. (a) wayward (b) confined (c) aligned (d) outwards SSC MTS-06/07/2022 Shift-II Ans. (d) : For the above blank space (4) 'outwards' will be used. Other options are not correct. 335. Select the most appropriate option to fill in blank number 5. (a) different (b) shorter (c) proper (d) greater SSC MTS-06/07/2022 Shift-II Ans. (b) : For the above blank space (5) 'Shorter' will be used. Other options are not correct. Comprehension: In the following passage, some words have been deleted. Read the passage carefully and select the most appropriate option to fill in each blank. Deforestation in tropical regions can (1) _______ the way water vapour is produced over the canopy, (2) _____ caused reduced rainfall. A 2019 study published in the journal Ecohydrology showed that parts of the Amazon rainforest that (3) ______ converted to agricultural land (4) ______ higher soil and air temperatures, which could (5)______ drought conditions. 336. Select the most appropriate option to fill in blank number 1. (a) hold (b) attack (c) affect (d) effect SSC MTS-05/07/2022 Shift-I Ans. (c) : For the above blank space (1) of the passage, verb form 'affect' will be used. Other options are not correct. 337. Select the most appropriate option to fill in blank number 2. (a) whom (b) which (c) where (d) who SSC MTS-05/07/2022 Shift-I Ans. (b) : For the above blank space (2) of the passage, 'which' will be used, because 'which/that' is used for 'non-living things'. Cloze Test

338. Select the most appropriate option to fill in blank number 3. (a) is (b) were (c) was (d) has SSC MTS-05/07/2022 Shift-I Ans. (b) : For the above blank space (3) 'were' will be used. Because the subject is plural, hence plural helping verb will be used. 339. Select the most appropriate option to fill in blank number 4. (a) had (b) did (c) was (d) has SSC MTS-05/07/2022 Shift-I Ans. (a) : For the above blank space (4) 'had' will be used. Because 'had' is used for possession in past tense. 340. Select the most appropriate option to fill in blank number 5. (a) purify (b) inflame (c) provoke (d) aggravate SSC MTS-05/07/2022 Shift-I Ans. (d) : For the above blank space (5) 'aggravate' will be used. Other options are not correct. Comprehension: In the following passage, some words have been deleted. Read the passage carefully and select the most appropriate option to fill in each blank. The Ministry of Defence has (1) _____ the formal Government Sanction Letter (2) _____ grant of permanent commission (PC) to women officers in the Army. "In anticipation, the Army Permanent Commission Selection Board for affected women officers. The Selection Board will officers exercise their option and complete (5) _____ documentation," the Army said in a statement. 341. Select the most appropriate option to fill in blank number 1. (a) imbibed (b) accused (c) extracted (d) issued SSC MTS-13/07/2022 Shift-I Ans. (d) : For the above blank space (1) 'issued' will be used. Other options are not correct. 342. Select the most appropriate option to fill in blank number 2. (a) since (b) from (c) for (d) to SSC MTS-13/07/2022 Shift-I Ans. (c) : For the above blank space (2) 'for' will be used, because it gives suitable meaning for the sentence. 343. Select the most appropriate option to fill in blank number 3. (a) deposited (b) set (c) located (d) situated SSC MTS-13/07/2022 Shift-I

795

YCT

Ans. (b) : For the above blank space (3) 'set' will be used. Other options are not correct. 344. Select the most appropriate option to fill in fill blank 4. (a) soon (b) late (c) long (d) blind SSC MTS-13/07/2022 Shift-I Ans. (a) : For the above blank space (4) 'soon' will be used. Other options are not correct. 345. Select the most appropriate option to fill in blank number 5. (a) unnecessary (b) optional (c) gratuitous (d) requisite SSC MTS-13/07/2022 Shift-I Ans. (d) : For the above blank space (5) 'requisite' will be used. Other options are not correct. Comprehension: In the following passage, some words have been deleted. Read the passage carefully and select the most appropriate option to fill in each blank. In Delhi, while the bigger schools transport their students in their own or hired bused; others rely ion vans, autos and e-rickshaws where many reckless drivers pack in too many children and drive (1) _____ often causing accidents. Many parents take time off their daily (2) ______ to do school-runs. Some use vehicles while others just walk their children to school. Two months ago, it was reported how in the (3) _____ of road-crossing facilities near schools, a bunch of parents were taking turns to play traffic marshals. Not so long ago, children could just walk or cycle to their schools, private tuition classes or sports practice. In fact, school authorities (4) _____ to admit students living within walk able distances distances so that they wouldn't have to take the school bus or public transport. As Delhi grew, its children (5) ______ not only traffic, broken infrastructure but also violence, air-pollution and overcrowded living conditions. 346. Select the most appropriate option to fill in blank number 1. (a) humbly (b) decisively (c) rashly (d) commonly SSC MTS-08/07/2022 Shift-I Ans. (c): For the above blank space (1) 'rashly' will be used. Other options are not correct. 347. Select the most appropriate option to fill in blank number 2. (a) habit (b) routines (c) problems (d) task SSC MTS-08/07/2022 Shift-I Cloze Test

Ans. (b) : For the above blank space (2) 'routines' will be used. Other options are not correct. 348. Select the most appropriate option to fill in blank number 3. (a) absence (b) non-compliance (c) defiance (d) reference SSC MTS-08/07/2022 Shift-I Ans. (a) : For the above blank space (3) 'absence' will be used. Other options are not correct. 349. Select the most appropriate option to fill in blank number 4. (a) rescued (b) guided (c) added (d) preferred SSC MTS-08/07/2022 Shift-I Ans. (d) : For the above blank space (4) 'Preferred' will be used. Other options are not correct. 350. Select the most appropriate option to fill in blank number 5. (a) faced (b) replied (c) called (d) relied SSC MTS-08/07/2022 Shift-I Ans. (a) : For the above blank space (5) 'faced' will be used. Other options are not correct. Comprehension: In the following passage, some words have been deleted. Read the passage carefully and select the most appropriate option to fill in each blank. Nitrogen is essential to life (1) _______ Earth. It is a component of all proteins, and it can be (2) ______ in all living systems. Nitrogen compounds are (3) _______ in organic materials, foods, fertilisers, explosives and poisons. Nitrogen is crucial to life, (4) ______ in excess it can also be (5) _______ to the environment. 351. Select the most appropriate option to fill in blank number 1. (a) at (b) into (c) on (d) in SSC MTS-12/07/2022 Shift-I Ans. (c) : For the above blank space (1) 'on' will be used, because it gives suitable meaning for the sentence. 352. Select the most appropriate option to fill in blank number 2. (a) founder (b) find (c) founded (d) found SSC MTS-12/07/2022 Shift-I Ans. (d) : For the above blank space (2) 'found' will be used, because 'modal + be + v3' is used. 353. Select the most appropriate option to fill in blank number 3. (a) function (b) instant (c) immediate (d) present SSC MTS-12/07/2022 Shift-I

796

YCT

Ans. (d) : For the above blank space (3) 'Present' will be used. Other options are not correct. 354. Select the most appropriate option to fill in blank number 4. (a) such (b) because (c) but (d) if SSC MTS-12/07/2022 Shift-I Ans. (c) : For the above blank space (4) 'but' will be used, because 'but' conjunction is used for contrast sense. 355. Select the most appropriate option to fill in blank number 5. (a) harmful (b) pliable (c) optimistic (d) rational SSC MTS-12/07/2022 Shift-I Ans. (a) : For the above blank space (5) 'Harmful' will be used. Other options are not correct. Comprehension: In the following passage, some words have been deleted. Read the passage carefully and select the most appropriate option to fill in each blank. Words convey meaning, because they are used in human activities in different forms. Different words may (1) ______ the same thing in different languages and this is what is referred to as 'family resemblance. ' In other words, languages help us (2) _____ reality and then communicate it to others. There are tens and thousands of languages in the world and all of them help us (3) ______ express ourselves, which in turn, given meaning to our life. One's mother tongue is (4) ______ for one's literary growth, but if you want to communicate to a wider audience, then you will (5) ______ to reach out to the target audience in a language they understand. 356. Select the most appropriate option to fill in blank number 1. (a) meaningful (b) meaning (c) mean (d) meant SSC MTS-14/07/2022 Shift-III Ans. (c) : For the above blank space (1) 'mean' will be used, because 'V1' is used after modals (may/might/ could/should etc). 357. Select the most appropriate option to fill in blank number 2. (a) commend (b) depend (c) comprehend (d) amend SSC MTS-14/07/2022 Shift-III Ans. (c) : For the above blank space (2) 'Comprehend' will be used. Other options are not correct. Cloze Test

358. Select the most appropriate option to fill in blank number 3. (a) retrospectively (b) secretly (c) effectively (d) lately SSC MTS-14/07/2022 Shift-III Ans. (c) : For the above blank space (3) 'effectively' will be used. Other options are not correct. 359. Select the most appropriate option to fill in blank number 4. (a) essential (b) unimportant (c) auspicious (d) silent SSC MTS-14/07/2022 Shift-III Ans. (a) : For the above blank space (4) 'essential' will be used. Other options are not correct. 360. Select the most appropriate option to fill in blank number 5. (a) have (b) should (c) has (d) had SSC MTS-14/07/2022 Shift-III Ans. (a) : For the above blank space (5) 'have' will be used. Other options are not correct. Comprehension: In the following passage, some words have been deleted. Read the passage carefully and select the most appropriate option to fill in each blank. Football legend Diego Maradona was buried in a private ceremony after a day of (1) ______ scenes in the Argentine capital Buenos Aires. Only (2) ______ two dozen relatives and close friends (3) ______ the final ceremony on Thursday. But (4) ______ huge crowds turned out to pay (5) ______ respects, with many weeping, blowing kisses and praying as they filed past his coffin. 361. Select the most appropriate option to fill in blank no. 1. (a) expressive (b) impressive (c) emotional (d) exciting SSC Constable GD-30/11/2021 Shift-III Ans. (c) : For the above blank space (1) 'emotional' will be used. Other options are not correct. 362. Select the most appropriate option to fill in blank no. 2. (a) more or less (b) around (c) just (d) roughly SSC Constable GD-30/11/2021 Shift-III Ans. (b) : For the above blank space (2) 'around' will be used. Other options are not correct. 363. Select the most appropriate option to fill in blank no. 3. (a) attend (b) have attended (c) attended (d) attending SSC Constable GD-30/11/2021 Shift-III

797

YCT

Ans. (c) : For the above blank space (3) 'attended' will be used. Other options are not correct. 364. Select the most appropriate option to fill in blank no. 4. (a) earliest (b) early (c) earlier (d) as early SSC Constable GD-30/11/2021 Shift-III Ans. (c) : For the above blank space (4) 'Earlier' will be used. Other options are not correct. 365. Select the most appropriate option to fill in blank no. 5. (a) them (b) his (c) their (d) its SSC Constable GD-30/11/2021 Shift-III Ans. (c) : For the above blank space (5) 'their' will be used. Other options are not correct. Comprehension: In the following passage, some words have been deleted. Read the passage carefully and select the most appropriate option to fill in each blank. (1)______ my thirteenth birthday, my mother made chicken pizza at home. That was the (2)______ pizza I have ever eaten. She had made (3)______ quantity for all my friends and everyone had a plateful. After the party got over, she (4)______ me by gifting a beautiful pullover. The (5)______ visit of my grandmother made my day. 366. Select the most appropriate option to fill in blank number 1. (a) On (b) In (c) From (d) At SSC Constable GD-29/11/2021 Shift-II Ans. (a) : For the above blank space (1) 'On' will be used. Because 'on' preposition is used before my thirteenth birthday. 367. Select the most appropriate option to fill in blank number 2. (a) best (b) better (c) good (d) very better SSC Constable GD-29/11/2021 Shift-II Ans. (a) : For the above blank space (2) 'Best' will be used, because superlative degree is used after. 368. Select the most appropriate option to fill in blank number 3. (a) sufficient (b) insufficient (c) little (d) inadequate SSC Constable GD-29/11/2021 Shift-II Ans. (a) : For the above blank space (3) 'sufficient' will be used. Other options are not correct. 369. Select the most appropriate option to fill in blank number 4. (a) tormented (b) blasted (c) surprised (d) drove SSC Constable GD-29/11/2021 Shift-II Cloze Test

Ans. (c) : For the above blank space (4), 'surprised' will be used. Other options are not correct. 370. Select the most appropriate option to fill in blank number 5. (a) undesirable (b) uneventful (c) unexpected (d) untoward SSC Constable GD-29/11/2021 Shift-II Ans. (c) : For the above blank space (5) 'unexpected' will be used. Other options are not correct. Comprehension: In the following passage, some words have been deleted. Read the passage carefully and select the most appropriate option to fill in each blank. It was a cold, wet January day on (1) ______ Tom went back to school. He carried in his pocket (2) ______ sugar-candies and a doll for little Laura. The (3) ______ was long and boring. Tom wanted to reach as quickly as (4) ______. He couldn’t wait to see Laura with the (5) ______ smile on her face. 371. Select the most appropriate option to fill in blank number 1. (a) which (b) that (c) where (d) when SSC Constable GD-26/11/2021 Shift-I Ans. (a) : For the above blank space (1) 'which' will be used, because pronoun 'which' is used for non living things. 372. Select the most appropriate option to fill in blank number 2. (a) none (b) any (c) some (d) much SSC Constable GD-26/11/2021 Shift-I Ans. (c) : For the above blank space (2) 'some' will be used. Other options are not correct. 373. Select the most appropriate option to fill in blank number 3. (a) adventure (b) visit (c) passage (d) journey SSC Constable GD-26/11/2021 Shift-I Ans. (d) : For the above blank space (3) 'journey' will be used. Other options are not correct. 374. Select the most appropriate option to fill in blank number 4. (a) portable (b) practical (c) probable (d) possible SSC Constable GD-26/11/2021 Shift-I Ans. (d) : For the above blank space (4) 'Possible' will be used. Other options have different meaning. 375. Select the most appropriate option to fill in blank number 5. (a) adorable (b) capable (c) reliable (d) suitable SSC Constable GD-26/11/2021 Shift-I

798

YCT

Ans. (a) : For the above blank space (5) 'adorable' will Elephants regularly swarm to the landfill (1) ______ near a wildlife sanctuary in Sri Lanka. (2) ______ be used. Other options are not correct. In the following passage, some words have been with food scraps, they consume plastic, (3) ______ deleted. Read the passage carefully and select the slowly kills them, officials say. Images (4) ______ the elephants foraging through the waste (5) ______ most appropriate option to fill in each blank. The human skeleton is the (1)______ framework of environmentalists. Plastic from landfill sites is a our body. It gives shape and support to the body. known killer of wild elephants, which are estimated to (2)______, it provides protection, makes blood cells, number about 7,500 in Sri Lanka. stores minerals and regulates hormones. The spinal 381. Select the most appropriate option to fill in blank no. 1. cord is an (3)______ part of our body. The ribcage protects our lungs and heart. To increase the (a) scene (b) scenery (4)______ of our bones, we should include calcium in (c) sight (d) site our diet. All our (5)______ organs are protected by SSC Constable GD-24/11/2021 Shift-II this skeletal system. Ans. (d) : For the above blank space (1) 'site' will be 376. Select the most appropriate option to fill in used. Other options are not correct. blank no. 1. Correct sentence(a) middle (b) internal Elephants regularly swarm to the landfill site near a (c) external (d) superficial SSC Constable GD-25/11/2021 Shift-III wildlife sanctuary. Ans. (c) : For the above blank space (1) 'external' will 382. Select the most appropriate option to fill in blank no. 2. be used. Other options are not correct. (a) Across (b) Next 377. Select the most appropriate option to fill in (c) Along (d) Beside blank no. 2. (a) Beside (b) Besides SSC Constable GD-24/11/2021 Shift-II (c) Unless (d) Although Ans. (c) : For the above blank space (2) ' along' will be SSC Constable GD-25/11/2021 Shift-III used. Other options are not correct. Ans. (b) : For the above blank space (2) 'besides' will 383. Select the most appropriate option to fill in be used, because it gives suitable meaning for sentence. blank no. 3. 378. Select the most appropriate option to fill in (a) what (b) whom blank no. 3. (c) who (d) which (a) Eternal (b) insignificant SSC Constable GD-24/11/2021 Shift-II (c) alert (d) important Ans. (d) : For the above blank space (3) 'which' will be SSC Constable GD-25/11/2021 Shift-III used. Other options are not correct. Ans. (d) : For the above blank space (3) 'important' will be used, because it gives suitable meaning for the Correct Sentence They consume plastic, which slowly kills them. sentence. 379. Select the most appropriate option to fill in 384. Select the most appropriate option to fill in blank no. 4. blank no. 4. (a) ability (b) strength (a) on (b) of (c) value (d) length (c) in (d) for SSC Constable GD-25/11/2021 Shift-III SSC Constable GD-24/11/2021 Shift-II Ans. (b) : For the above blank space (4) 'strength' will Ans. (b) : For the above blank space (4) 'of ' will be be used. Other options are not correct. used. Other options are not correct. 380. Select the most appropriate option to fill in Correct Sentence blank no. 5. Images of the elephants foraging through the waste(a) fundamental (b) secondary 385. Select the most appropriate option to fill in (c) secret (d) vital blank no. 5. SSC Constable GD-25/11/2021 Shift-III (a) have shocked (b) have been shocking Ans. (d) : For the above blank space (5) 'vital' will be (c) has shocked (d) are shocking used. Other options are not correct. SSC Constable GD-24/11/2021 Shift-II Comprehension: Ans. (a) : For the above blank space (5) 'have shocked' In the following passage, some words have been will be used, because 'elephants' is plural, hence have + deleted. Read the passage carefully and select the V will be used. 3 most appropriate option to fill in each blank. Cloze Test

799

YCT

Comprehension: In the following passage, some words have been deleted. Read the passage carefully and select the most appropriate option to fill in each blank. Mahatma Gandhi was born (1)______ 2 October 1869 to Karamchand and Putlibai (2)______ Porbandar. After completing his studies in England, he (3)______ to South Africa. When he suffered (4)______, he started the Civil Disobedience movement (5)______ civil rights of the Indian community. 386. Select the most appropriate option to fill in blank no. 1. (a) from (b) in (c) of (d) on SSC Constable GD-18/11/2021 Shift-I Ans. (d) : For the above blank space (1) 'on' will be used, because preposition 'on' is used before Dates/ Days. 387. Select the most appropriate option to fill in blank no. 2. (a) above (b) at (c) below (d) through SSC Constable GD-18/11/2021 Shift-I Ans. (b) : For the above blank space (2) 'at' will be used, because preposition 'at' is used before small places. 388. Select the most appropriate option to fill in blank no. 3. (a) went (b) going (c) will go (d) gone SSC Constable GD-18/11/2021 Shift-I Ans. (a) : For the above blank space (3) 'went' will be used, because the Passage is in past tense. 389. Select the most appropriate option to fill in blank no. 4. (a) deterioration (b) distinction (c) discrimination (d) delegation SSC Constable GD-18/11/2021 Shift-I Ans. (c) : For the above blank space (4) 'discrimination' will be used. Other options are not correct. 390. Select the most appropriate option to fill in blank no. 5. (a) with (b) for (c) to (d) after SSC Constable GD-18/11/2021 Shift-I Ans. (b) : For the above blank space (5) 'for' will be used, because it gives suitable meaning for the sentence. Comprehension: In the following passage, some words have been deleted. Read the passage carefully and select the most appropriate option to fill in each blank. You should elect me as Class President as I think that everyone’s (1)______ matters. I am a confident speaker and have won prizes Cloze Test

(2)______ public speaking. I will make sure that the school council (3)______ to us. When people come to me with problems, I (4) ______listen carefully. I never judge people who come to me for (5)______. You can count on me for help. 391. Select the most appropriate option to fill in blank no.1. (a) estimate (b) opinion (c) theorem (d) reaction SSC Constable GD-17/11/2021 Shift-II Ans. (b) : For the above blank space (1) of the passage, 'opinion' will be used. Other options are not correct. 392. Select the most appropriate option to fill in blank no.2. (a) on (b) by (c) of (d) for SSC Constable GD-17/11/2021 Shift-II Ans. (d) : For the above blank space (2) of the passage, 'for' will be used. Other options are not correct. 393. Select the most appropriate option to fill in blank no.3. (a) believes (b) hears (c) listens (d) accepts SSC Constable GD-17/11/2021 Shift-II Ans. (c) : For the above blank space (3) 'listens' will be used. Other options are not correct. 394. Select the most appropriate option to fill in blank no.4. (a) always (b) barely (c) seldom (d) never SSC Constable GD-17/11/2021 Shift-II Ans. (a) : For the above blank space (4) 'always' will be used. Other options are not correct. 395. Select the most appropriate option to fill in blank no.5. (a) advice (b) news (c) caution (d) lesson SSC Constable GD-17/11/2021 Shift-II Ans. (a) : For the above blank space (5) 'advice' will be used. Other options are not correct. Comprehension: In the following passage, some words have been deleted. Read the passage carefully and select the most appropriate option to fill in each blank. Riding a bicycle, also called cycling, has a number of health (1) ___. Cycling is an aerobic activity, which means your heart, lungs and blood vessels all get a workout. It also helps (2) ___ stamina and physical strength. Another important advantage is that cycling helps (3) ____ endorphins or the feel-good chemicals in the brain (4) ____ help reduce stress levels. People who (5) ____ cycling as an exercise find that they have more energy for their day-today activities.

800

YCT

396. Select the most appropriate option to fill in blank no. 1. (a) values (b) credits (c) benefits (d) traits SSC Constable GD-13/12/2021 Shift-III Ans. (c) : For the above blank space (1) 'benefits' will be used. Other options are not correct. 397. Select the most appropriate option to fill in blank no. 2. (a) build (b) establish (c) expand (d) grow SSC Constable GD-13/12/2021 Shift-III Ans. (a) : For the above blank space (2) 'build' will be used. Other options are not correct. 398. Select the most appropriate option to fill in blank no. 3. (a) leak (b) realise (c) release (d) clear SSC Constable GD-13/12/2021 Shift-III Ans. (c) : For the above blank space (3) 'release' will be used. Other options are not correct. 399. Select the most appropriate option to fill in blank no. 4. (a) that (b) thus (c) who (d) such SSC Constable GD-13/12/2021 Shift-III Ans. (a) : For the above blank space (4) 'that' will be used. Other options are not correct. 400. Select the most appropriate option to fill in blnak no. 5. (a) take off (b) take up (c) take out (d) take after SSC Constable GD-13/12/2021 Shift-III Ans. (b) : For the above blank space (5) 'take up' will be used. Other options are not correct. Comprehension: In the following passage, some words have been deleted. Read the passage carefully and select the most appropriate option to fill in each blank. A giant plane carrying 43 passengers was (1) ______to crash-land when its landing gear got (2) ______. Only the skill of the pilot was able to (3) ______ a major tragedy. Expert mechanics laboured (4) ______ hours before they could find the (5) ______ of the mechanical failure. 401. Select the most appropriate option to fill in blank no.1. (a) poked (b) pushed (c) forced (d) bumped SSC Constable GD-16/11/2021 Shift-III Ans. (c) : For the above blank space (1) 'forced' will be used, because sentence is in passive voice. Hence 'V3' will be used. Cloze Test

402. Select the most appropriate option to fill in blank no.2. (a) glued (b) fixed (c) caught (d) stuck SSC Constable GD-16/11/2021 Shift-III Ans. (d) : For the above blank space (2) 'stuck' will be used. Other options are not correct. 403. Select the most appropriate option to fill in blank no.3. (a) limit (b) restrict (c) forbid (d) prevent SSC Constable GD-16/11/2021 Shift-III Ans. (d) : For the above blank space (3) 'Prevent' will be used. Other options are not correct. 404. Select the most appropriate option to fill in blank no.4. (a) from (b) over (c) at (d) for SSC Constable GD-16/11/2021 Shift-III Ans. (d) : For the above blank space (4) 'for' will be used. Other options are not correct. 405. Select the most appropriate option to fill in blank no. 5. (a) motive (b) plan (c) agency (d) cause SSC Constable GD-16/11/2021 Shift-III Ans. (d) : For the above blank space (5) 'Cause' will be used. Other options are not correct. Comprehension: In the following passage, some words have been deleted. Read the passage carefully and select the most appropriate option to fill in each blank. Naval architects never claim that a ship is unsinkable, but sinking of the passenger-andcar ferry Estonia in the Baltic surely should never (1)______. It was well designed and carefully (2)______. It carried the proper number of lifeboats. It had been (3)______ inspected the day of its fatal voyage. Yet hours (4)______, the Estonia rolled over and sank in a cold, stormy night. It went down (5)______ quickly that most of those on board, caught in their dark, flooding cabins, had no chance to save themselves. 406. Select the most appropriate option to fill in blank no. 1. (a) has happened (b) happened (c) had happened (d) have happened SSC Constable GD-14/12/2021 Shift-III Ans. (d) : For the above blank space (1) 'have happened' will be used, because it gives suitable meaning for the sentence.

801

YCT

407. Select the most appropriate option to fill in Ans. (d) : For the above blank space (2) 'hope' will be blank no. 2. used. Other options are not correct. (a) maintain (b) maintained 413. Select the most appropriate option to fill in blank No.3 (c) maintaining (d) to be maintained (a) Since (b) On SSC Constable GD-14/12/2021 Shift-III (c) At (d) From Ans. (b) : For the above blank space (2), 'maintained' SSC Constable GD-14/12/2021 Shift-II will be used, because sentence is in passive voice. other Ans. (c) : For the above blank space (3) 'At' will be options are not correct. used. Other options are not correct. 408. Select the most appropriate option to fill in 414. Select the most appropriate option to fill in blank no. 3. blank No.4. (a) thoroughly (b) gradually (a) looking (b) seeing (c) steadily (d) immediately (c) noting (d) watching SSC Constable GD-14/12/2021 Shift-III SSC Constable GD-14/12/2021 Shift-II Ans. (a) : For the above blank space (3) 'thoroughly' Ans. (b) : For the above blank space (4) 'seeing' will will be used. Other options are not correct. be used. Other options are not correct. 409. Select the most appropriate option to fill in 415. Select the most appropriate option to fill in blank No. 5. blank no. 4. (a) take (b) taken (a) earlier (b) before (c) took (d) takes (c) latter (d) later SSC Constable GD-14/12/2021 Shift-II SSC Constable GD-14/12/2021 Shift-III Ans. (c) : For the above blank space (5) 'took' will be Ans. (d) : For the above blank space (4) 'later' will be used. Other options are not correct. used. Other options are not correct. Comprehension: 410. Select the most appropriate option to fill in In the following passage, some words have been blank no. 5. deleted. Read the passage carefully and select (a) such (b) as the most appropriate option to fill in each (c) so (d) very blank. SSC Constable GD-14/12/2021 Shift-III Life is not about making lists and trying to be one step (1) ____ of others. Stop being a Ans. (c) : For the above blank space (5) 'so' will be workaholic. Take a (2) ____. Do whatever used. Other options are not correct. makes you feel happy and (3) ___. Most of all, Comprehension: spend time in the (4) ____ of nature to pick up In the following passage, some words have been some (5) ____ lessons. deleted. Read the passage carefully and select Select the most appropriate option to fill in 416. the most appropriate option to fill in each blank on. 1. blank. (a) forward (b) away His friend did not return even the next day, or (c) ahead (d) front the day after. For several days, the partridge SSC Constable GD-09/12/2021 Shift-II (1)______ worrying, and then gave up (2)______ of his friend's return.(3)______ dusk, Ans. (c) : For the above blank space (1) 'ahead' will be a hare came to the tree and (4)______ an empty used. Other options are not correct. hole at the foot of the tree, (5)______ shelter 417. Select the most appropriate option to fill in inside. Since the partridge had given up hope, blank no. 2. he did not object to the hole being occupied by (a) recess (b) blow the hare. (c) break (d) gap 411. Select the most appropriate option to fill in SSC Constable GD-09/12/2021 Shift-II blank no. 1. (a) did (b) had Ans. (c) : For the above blank space (2) 'break' will be (c) was (d) kept used. Other options are not correct. SSC Constable GD-14/12/2021 Shift-II 418. Select the most appropriate option to fill in Ans. (d) : For the above blank space (1) 'kept' will be blank no. 3. used. Other options are not correct. (a) lenient (b) patient 412. Select the most appropriate option to fill in (c) casual (d) relaxed blank No.2. SSC Constable GD-09/12/2021 Shift-II (a) trial (b) risk Ans. (d) : For the above blank space (3) 'relaxed' will (c) chance (d) hope be used. Other options are not correct. SSC Constable GD-14/12/2021 Shift-II Cloze Test

802

YCT

419. Select the most appropriate option to fill in blank no. 4. (a) group (b) crowd (c) company (d) gathering SSC Constable GD-09/12/2021 Shift-II Ans. (c) : For the above blank space (4) 'company' will be used. Other options are not correct. 420. Select the most appropriate option to fill in blank no. 5. (a) innumerable (b) invaluable (c) inevitable (d) invisible SSC Constable GD-09/12/2021 Shift-II Ans. (b) : For the above blank space (5) 'Invaluable' will be used. Other options are not correct. Comprehension: In the following passage, some words have been deleted. Read the passage carefully and select the most appropriate option to fill in each blank. The power of words (1)______ an enduring effect and can be (2)______ by all men and women. Words are not (3)______ for writers or lovers of literature. Every profession is (4)______ by words. It is impossible to (5)______ oneself without using words. 421. Select the most appropriate option to fill in blank no. 1. (a) are (b) has (c) is (d) have SSC Constable GD-07/12/2021 Shift-II Ans. (b) : For the above blank space (1) 'has' will be used. Other options are not correct. 422. Select the most appropriate option to fill in blank no. 2. (a) pleased (b) impressed (c) looked (d) enjoyed SSC Constable GD-07/12/2021 Shift-II Ans. (d) : For the above blank space (2) 'enjoyed' will be used. Other options are not correct. 423. Select the most appropriate option to fill in blank no. 3. (a) hardly (b) such (c) merely (d) same SSC Constable GD-07/12/2021 Shift-II Ans. (c) : For the above blank space (3) 'merely' will be used. Other options are not correct. 424. Select the most appropriate option to fill in blank no. 4. (a) guided (b) charged (c) managed (d) trained SSC Constable GD-07/12/2021 Shift-II Ans. (a) :For the above blank space (4) 'guided' will be used. Other options are not correct. Cloze Test

425. Select the most appropriate option to fill in blank no. 5. (a) impress (b) depress (c) repress (d) express SSC Constable GD-07/12/2021 Shift-II Ans. (d) : For the above blank space (5) 'express' will be used. Other options are not correct. Comprehension : In the following passage, some words have been deleted. Read the passage carefully and select the most appropriate option to fill in each blank. A biosketch is (1) –––––– brief summary of a person's professional (2) –––––– and recognition. It is meant (3) –––––– highlight the important (4) ––––––– of one's training, experience and (5) –––––– of interest. 426. Select the most appropriate option to fill in the blank number 1. (a) No article required (b) an (c) one (d) a SSC Constable GD-06/12/2021 Shift-III Ans. (d) : For the above blank space (1) article 'a' will be used, because indefinite article 'a' is used before consonant sound. 427. Select the most appropriate option to fill in blank number 2. (a) annoyance (b) establishments (c) accomplishments (d) adventures SSC Constable GD-06/12/2021 Shift-III Ans. (c) : For the above blank space (2) 'accomplishments' will be used. Other options are not correct. 428. Select the most appropriate option to fill in blank number 3. (a) of (b) from (c) to (d) about SSC Constable GD-06/12/2021 Shift-III Ans. (c) : For the above blank space (3) Preposition 'to' will be used. Other options are not correct. 429. Select the most appropriate option to fill in blank number 4. (a) aspects (b) availability (c) austerity (d) assignments SSC Constable GD-06/12/2021 Shift-III Ans. (a) : For the above blank space (4) 'aspects' will be used. Other options are not correct. 430. Select the most appropriate option to fill in blank number 5. (a) areas (b) section (c) space (d) surface SSC Constable GD-06/12/2021 Shift-III Ans. (a) : For the above blank space (5) 'areas' will be used. Other options are not correct.

803

YCT

Comprehension: In the following passage, some words have been deleted. Read the passage carefully and select the most appropriate option to fill in each blank. Joseph was always prepared. His motto was, 'Never throw anything out, you (1) ____know when it might come in (2) _____.' His bedroom was totally (3) _____with flat bicycle tyres, empty boxes, (4) _____basket balls and bent tennis racquets (5) _____leaving any space to get in the door. 431. Select the most appropriate option to fill in blank no. 1. (a) often (b) should (c) would (d) never SSC Constable GD-03/12/2021 Shift-II

Comprehension: In the following passage, some words have been deleted. Read the passage carefully and select the most appropriate option to fill in each blank. Once there were two brothers. One brother was (1)______poor farmer. He worked in the fields, and (2)______the weather was bad, he (3)______no food. But he kind and worked a (4)______The other brother was a rich man (5)______worked on a ship. He was not kind and often cheated people. 436. Select the most appropriate option to fill in blank no. 1. (a) only (b) simply (c) the (d) a SSC Constable GD-01/12/2021 Shift-I Ans. (d) : For the above blank space (1) Article 'a' will be used. Because article 'a' is used for indefinite sense.

Ans. (d) : For the above blank space (1) 'never' will be 437. Select the most appropriate option to fill in blank no. 2. used. Other options are not correct. (a) while (b) when 432. Select the most appropriate option to fill in (c) after (d) what blank no. 2. SSC Constable GD-01/12/2021 Shift-I (a) handy (b) teady Ans. (b) : For the above blank space (2) 'when' will be (c) close (d) nearby used. Other options are not correct. SSC Constable GD-03/12/2021 Shift-II 438. Select the most appropriate option to fill in Ans. (a) : For the above blank space (2) 'handy' will be blank no. 3. used. Because it gives suitable meaning for the (a) has (b) had sentence. (c) may have (d) will have SSC Constable GD-01/12/2021 Shift-I 433. Select the most appropriate option to fill in Ans. (b) : For the above blank space (3) 'had' will be blank no. 3. used. Other options are not correct. (a) cluttered (b) dislocated 439. Select the most appropriate option to fill in (c) confused (d) splashed blank no. 4. SSC Constable GD-03/12/2021 Shift-II (a) much (b) lots Ans. (a) : For the above blank space (3) 'cluttered' will (c) lot (d) hard be used. Because it gives suitable meaning for the SSC Constable GD-01/12/2021 Shift-I sentence. Ans. (c) : For the above blank space (4) 'lot' will be 434. Select the most appropriate option to fill in used. Other options are not correct. blank no. 4. 440. Select the most appropriate option to fill in blank no. 5. (a) deflated (b) wearied (a) who (b) which (c) drained (d) reduced (c) that (d) whom SSC Constable GD-03/12/2021 Shift-II SSC Constable GD-01/12/2021 Shift-I Ans. (a) : For the above blank space (4) 'Deflated' will be Ans. (a) : For the above blank space (5) 'who' will be used. Because it gives suitable meaning for the sentence. used. Other options are not correct. 435. Select the most appropriate option to fill in Comprehension blank no. 5. In the following passage, some words have been deleted. Select the most appropriate option to (a) merely (b) simply fill in the blank. (c) barely (d) slightly Meghalaya a hilly state of India, is located in SSC Constable GD-03/12/2021 Shift-II the northeastern part of the country. It is (1) Ans. (c) : For the above blank space (5) 'Barely' will be ............ by the Indian state of Assam in the used. Other options are not correct. north and northeast and (2) ............ Bangladesh Cloze Test

804

YCT

in the south and southwest. The state capital is (3) .......... hill town of Shillong, located in eastcentral Meghalaya. Meghalaya is an upland area formed by a (4) ........... block of the Deccan plateau, its summits vary in (5) .......... from 4,000 to 6,000 feet (1,220 to 1,830 metres). 441. Select the most appropriate option to fill in the blank 1. (a) bind (b) bonded (c) bounded (d) bounding SSC CHSL (Tier-I) –05/08/2021 (Shift-I) Ans. (c) : For the above blank space (1) 'bounded' will be used. Other options are not correct. 442. Select the most appropriate option to fill in the blank 2. (a) near (b) with (c) beside (d) by SSC CHSL (Tier-I) –05/08/2021 (Shift-I) Ans. (d) : For the above blank space (2) 'By' will be used. Other options are not correct. 443. Select the most appropriate option to fill in the blank 3. (a) a (b) one (c) only (d) the SSC CHSL (Tier-I) –05/08/2021 (Shift-I) Ans. (d) : For the above blank space (3) article 'The' will be used. Other options are not correct. 444. Select the most appropriate option to fill in the blank 4. (a) withdrawn (b) standing (c) removed (d) detached SSC CHSL (Tier-I) –05/08/2021 (Shift-I) Ans. (d) : For the above blank space (4) 'detached' will be used. Other options are not correct. 445. Select the most appropriate option to fill in the blank 5. (a) distance (b) rise (c) elevation (d) increase SSC CHSL (Tier-I) –05/08/2021 (Shift-I) Ans. (c) : For the above blank space (5) 'elevation' will be used. Other options are not correct. Comprehension In the following passage some words have been deleted. Fill in the blanks with the help of the alternatives given. Select the most appropriate option for each blank. Many hundreds of years ago, a Brahmin lived in a town in Northern India. He spent many years (1)____ the ancient books and talking to the learned priests (2) _____ came to visit the city. After much effort, he (3)____ the knowledge of a certain spell. He was (4) ____ proud of this and guarded his secret (5) _____. 446. Select the most appropriate option for blank no. 1. (a) study (b) to study (c) studied (d) studying SSC CHSL (Tier-I) –16/04/2021 (Shift-I) Cloze Test

Ans. (d) : For the above blank space (1) 'studying' will be used. Other options are not correct. 447. Select the most appropriate option for blank no. 2. (a) which (b) who (c) whom (d) that SSC CHSL (Tier-I) –16/04/2021 (Shift-I) Ans. (b) : For the above blank space (2) 'who' will be used. Because 'who' is used in subjective case for living things. 448. Select the most appropriate option for blank no. 3. (a) used (b) found (c) obtained (d) took SSC CHSL (Tier-I) –16/04/2021 (Shift-I) Ans. (c) : For the above blank space (3), ' obtained' will be used. Other options are not correct. 449. Select the most appropriate option for blank no. 4. (a) many (b) very (c) much (d) a lot SSC CHSL (Tier-I) –16/04/2021 (Shift-I) Ans. (b) : For the above blank space (4) 'very' will be used. Other options are not correct. 450. Select the most appropriate option for blank no. 5. (a) gradually (b) deeply (c) excitedly (d) carefully SSC CHSL (Tier-I) –16/04/2021 (Shift-I) Ans. (d) : For the above blank space (5) 'carefully' will be used. Other options are not correct. Comprehension In the following passage, some words have been deleted. Fill in the blanks with the help of the alternatives given. Select the most appropriate option for each blank. Adventure sports play an important role in shaping and developing the personality of (1) ____ individual. They involve great physical and mental (2) ____ along with discipline and decision making. Participation in adventure sports (3) _____ the confidence level, strength, stamina and fitness. Besides, (4) ___ sports also teach one to work in unity and improve social interaction along with group (5) ____ and leadership. 451. Select the most appropriate option for blank No. 1. (a) some (b) an (c) one (d) many SSC CHSL (Tier-I) –15/04/2021 (Shift-I) Ans. (b) : For the above blank space (1) 'an' will be used. Because indefinite article 'an' is used for vowel sound.

805

YCT

452. Select the most appropriate option for blank No. 2. (a) enduring (b) endured (c) endure (d) endurance SSC CHSL (Tier-I) –15/04/2021 (Shift-I) Ans. (d) : For the above blank space (2) 'endurance' will be used. Other options are not correct. 453. Select the most appropriate option for blank No. 3. (a) enhances (b) grows (c) values (d) decreases SSC CHSL (Tier-I) –15/04/2021 (Shift-I) Ans. (a) : For the above blank space (3) 'Enhances' will be used. Other options are not correct. 454. Select the most appropriate option for blank No. 4. (a) those (b) this (c) that (d) these SSC CHSL (Tier-I) –15/04/2021 (Shift-I) Ans. (d) : For the above blank space (4) 'these' will be used. Because it gives suitable meaning for the sentence. 455. Select the most appropriate option for blank No. 5. (a) dynamics (b) games (c) actions (d) feelings SSC CHSL (Tier-I) –15/04/2021 (Shift-I) Ans. (a) : For the above blank space (5) 'dynamics' will be used. Other options are not correct. Comprehension In the following passage some words have been deleted. Fill in the blanks with the help of the alternatives given. Select the most appropriate option for each number. In the 'Kingdom of Fools', both the king and the ministers were complete fools. They didn't want to do things (1) .......... other kingdoms, so they decided to (2) ............ night into day and day into night. They (3) .......... that everyone should be awake at night, (4) ............. their lands and run their (5) .......... only after dark and should go to bed as soon as the sun came up. 456. Select the most appropriate option to fill in the blank no. 1. (a) like (b) than (c) so as (d) same SSC CHSL (Tier-I) –06/08/2021 (Shift-I) Ans. (a) : For the above blank space (1) 'like' will be used. Other options are not correct. 457. Select the most appropriate option to fill in the blank no. 2. (a) adjust (b) develop (c) change (d) agree SSC CHSL (Tier-I) –06/08/2021 (Shift-I) Ans. (c) : For the above blank space (2) 'change' will be used. Other options are not correct. Cloze Test

458. Select the most appropriate option to fill in the blank no. 3. (a) ordered (b) presented (c) ruled (d) believed SSC CHSL (Tier-I) –06/08/2021 (Shift-I) Ans. (a) : For the above blank space (3) 'ordered' will be used. Other options are not correct. 459. Select the most appropriate option to fill in the blank no. 4. (a) till (b) cut (c) dig (d) sow SSC CHSL (Tier-I) –06/08/2021 (Shift-I) Ans. (a) : For the above blank space (4) 'till' will be used. Other options are not correct. 460. Select the most appropriate option to fill in the blank no. 5. (a) employment (b) livelihood (c) career (d) business SSC CHSL (Tier-I) –06/08/2021 (Shift-I) Ans. (d) : For the above blank space (5) 'business' will be used. Other options are not correct. Comprehension In the following passage, some words have been deleted. Select the most appropriate option to fill in each blank. Kanjirapally has the highest number of rainy days in Kerala. It receives the highest (1) .......... of summer rains, winter rains and northeast monsoons in Kerala, (2) .......... it one of the very few (3) ........... in India enjoying equatorial rainforest type climate, with no (4) .......... dry season. The well-distributed rainfall pattern of Kanjirapally is the (5) ........... reason for the phenomenon of high yield of latex from rubber plantations in and around the town. 461. Select the most appropriate option to fill in blank No. 1. (a) amount (b) number (c) expanse (d) extent SSC CHSL (Tier-I) –09/08/2021 (Shift-I) Ans. (a) : For the above blank space (1) 'amount' will be used. Other options are not correct. 462. Select the most appropriate option to fill in blank No. 2. (a) make (b) making (c) made (d) makes SSC CHSL (Tier-I) –09/08/2021 (Shift-I) Ans. (b) : For the above blank space (2) 'making' will be used. Other options are not correct. 463. Select the most appropriate option to fill in blank No. 3. (a) space (b) places (c) spot (d) place SSC CHSL (Tier-I) –09/08/2021 (Shift-I) Ans. (b) : For the above blank space (3) 'places' will be used. Other options are not correct.

806

YCT

464. Select the most appropriate option to fill in blank No. 4. (a) individual (b) divergent (c) diverse (d) distinct SSC CHSL (Tier-I) –09/08/2021 (Shift-I) Ans. (d) : For the above blank space (4) 'distinct' will be used. Other options are not correct. 465. Select the most appropriate option to fill in blank No. 5. (a) insignificant (b) primary (c) minor (d) initial SSC CHSL (Tier-I) –09/08/2021 (Shift-I) Ans. (b) : For the above blank space (5) 'primary' will be used. Other options are not correct. Comprehension In the following passage, some words have been deleted. Select the most appropriate option to fill in each blank. All of us know that the American astronaut Neil Armstrong was the first (1) –––– to set foot on the Moon in July 1969. Human footprints on the lunar (2) ––––––won't disappear for millions of years. That is because there is no rain or wind to (3) ––––– them. The surfae area of the Moon is 25% larger than that of Africa. Astronauts have so far brought back hundreds of kilos of rock and dust from the Moon to (4) – ––––. It takes the Moon 27.3 days to travel around the Earth. As it does so, we see different amounts of its side (5) ––––– by the Sun. That is why it appears to get larger and then smaller. 466. Select the most appropriate option to fill in blank number 1. (a) creature (b) organism (c) object (d) human SSC CHSL (Tier-I) –19/04/2021 (Shift-I) Ans. (d) : For the above blank space (1) 'human' will be used. Other options are not correct. 467. Select the most appropriate option to fill in blank number 2. (a) eclipse (b) rock (c) dust (d) surface SSC CHSL (Tier-I) –19/04/2021 (Shift-I) Ans. (d) : For the above blank space (2) 'surface' will be used. Other options are not correct. 468. Select the most appropriate option to fill in blank number 3. (a) replicate (b) recreate (c) erode (d) restore SSC CHSL (Tier-I) –19/04/2021 (Shift-I) Ans. (c) : For the above blank space (3) 'erode' will be used. Other options are not correct. 469. Select the most appropriate option to fill in blank number 4. (a) show (b) store (c) study (d) sell SSC CHSL (Tier-I) –19/04/2021 (Shift-I) Cloze Test

Ans. (c) : For the above blank space (4) 'study' will be used. Other options are not correct. 470. Select the most appropriate option to fill in blank number 5. (a) heated (b) hidden (c) covered (d) lit SSC CHSL (Tier-I) –19/04/2021 (Shift-I) Ans. (d) : For the above blank space (5) 'lit' will be used. Other options are not correct. Comprehension In the following passage, some words have been deleted. Fill in the blanks with the help of the alternatives given. Select the most appropriate option for each blank. Smoking causes mutation in cells leading to cancer. Quitting smoking can help reverse the (1)______ risk; and it can replenish cells that actually (2)______ those of a person who has never smoked. The (3)______ you smoke the more mutations you have. The (4)______ damage to the cells lining the airway and lungs can (5)______ to cancers in the lungs, oesophagus, larynx and pharynx. 471. Select the most appropriate option to fill in blank No. 1. (a) nominal (b) factual (c) potential (d) impossible SSC CGL (Tier-I) –24/08/2021 (Shift-I) Ans. (c) : For the above blank space (1) 'Potential' will be used. Other options are not correct. 472. Select the most appropriate option to fill in blank No. 2. (a) identify (b) differ (c) disagree (d) resemble SSC CGL (Tier-I) –24/08/2021 (Shift-I) Ans. (d) : For the above blank space (2) 'resemble' will be used. Other options are not correct. 473. Select the most appropriate option to fill in blank No. 3. (a) more (b) most (c) many (d) much SSC CGL (Tier-I) –24/08/2021 (Shift-I) Ans. (a) : For the above blank space (3) 'more' will be used. Because 'the + comparative degree..............The + comparative degree is used. 474. Select the most appropriate option to fill in blank No. 4. (a) broken (b) stable (c) incomplete (d) constant SSC CGL (Tier-I) –24/08/2021 (Shift-I) Ans. (d) : For the above blank space (4) 'Constant' will be used. Other options are not correct. 475. Select the most appropriate option to fill in blank No. 5. (a) direct (b) seize (c) guide (d) lead SSC CGL (Tier-I) –24/08/2021 (Shift-I)

807

YCT

Ans. (d) : For the above blank space (5) 'lead' will be used. Other options are not correct. Comprehension In the following passage, some words have been deleted. Fill in the blanks with the help of the alternatives given. Select the most appropriate option for each blank. Almost everyone in the present world is familiar with the term ‘Global Warming’. It (1)______ to the gradual rise in the overall temperature of the atmosphere (2)______ the planet Earth. It is true that there are (3) _____ activities taking place which have been increasing the temperature (4)______. The first impact of global warming is melting of (5)____ in a rapid way. This is extremely harmful to the earth as well as to its inhabitants. 476. Select the most appropriate option to fill in blank No. 1. (a) connects (b) seems (c) refers (d) means SSC CGL (Tier-I) –23/08/2021 (Shift-I) Ans. (c) : For the above blank space (1) 'refers' will be used. Other options are not correct. 477. Select the most appropriate option to fill in blank No. 2. (a) of (b) in (c) at (d) on SSC CGL (Tier-I) –23/08/2021 (Shift-I) Ans. (a) : For the above blank space (2) 'of ' will be used. Other options are not correct. 478. Select the most appropriate option to fill in blank No. 3. (a) sudden (b) several (c) separate (d) useful SSC CGL (Tier-I) –23/08/2021 (Shift-I) Ans. (b) : For the above blank space (3) 'several' will be used. Other options are not correct. 479. Select the most appropriate option to fill in blank No. 4. (a) purposely (b) properly (c) gradually (d) frequently SSC CGL (Tier-I) –23/08/2021 (Shift-I) Ans. (c) : For the above blank space (4) 'gradually' will be used. Other options are not correct. 480. Select the most appropriate option to fill in blank No. 5. (a) glaciers (b) rivers (c) lakes (d) mountains SSC CGL (Tier-I) –23/08/2021 (Shift-I) Ans. (a) : For the above blank space (5) 'glaciers' will be used. Other options are not correct. Cloze Test

Comprehension In the following passage, some words have been deleted. Fill in the blanks with the help of the alternatives given. Select the most appropriate option for each blank. It is true that the more you study, the more you (1)______ your horizon and hence your point of view undergoes a (2)______. A person with higher education certainly has a better (3)______ and ideas to help in community and societal (4)______. Ideas that are gathered from such people provide a global (5)______ for the nation as a whole. 481. Select the most appropriate option to fill in the blank number 1. (a) expand (b) narrow (c) exchange (d) convert SSC CGL (Tier-I) –16/08/2021 (Shift-I) Ans. (a) : For the above blank space (1) 'expand' will be used. Other options are not correct. 482. Select the most appropriate option to fill in the blank number 2. (a) backwardness (b) succession (c) transformation (d) enlargement SSC CGL (Tier-I) –16/08/2021 (Shift-I) Ans. (c) : For the above blank space (2) 'transformation' will be used. Other options are not correct. 483. Select the most appropriate option to fill in blank number 3. (a) routine (b) outlook (c) luck (d) scope SSC CGL (Tier-I) –16/08/2021 (Shift-I) Ans. (b) : For the above blank space (3) 'outlook' will be used. Other options are not correct. 484. Select the most appropriate option to fill in blank number 4. (a) maturity (b) elaboration (c) development (d) ripening SSC CGL (Tier-I) –16/08/2021 (Shift-I) Ans. (c) : For the above blank space (4) 'development' will be used. Other options are not correct. 485. Select the most appropriate option to fill in blank number 5. (a) prejudice (b) posture (c) perspective (d) proportion SSC CGL (Tier-I) –16/08/2021 (Shift-I) Ans. (c) : For the above blank space (5) 'perspective' will be used. Other options are not correct. Comprehension In the following passage, some words have been deleted. Fill in the blanks with the help of the alternatives given. Select the most appropriate option for each blank. The five senses (1)______ sight, hearing, taste, touch and (2)______. What is sometimes (3)______ to as a ‘sixth sense’ is a power to be aware (4)______ things independently of the five (5)______ senses, a kind of supernatural sense.

808

YCT

486. Select the most appropriate option to fill in blank number 1. (a) are (b) of (c) is (d) in SSC CGL (Tier-I) –20/08/2021 (Shift-I) Ans. (a) : For the above blank space (1) 'are' will be used, because the sentence is in present tense and subject (the five senses) is plural. Hence the plural verb (are) will be used. Other options are not correct. 487. Select the most appropriate option to fill in the blank number 2. (a) happiness (b) smell (c) anger (d) sadness SSC CGL (Tier-I) –20/08/2021 (Shift-I) Ans. (b) : According to the sense of the above passage in blank space (2) 'smell' will be used. Other options are not correct. 488. Select the most appropriate option to fill in the blank number 3. (a) reminded (b) retained (c) referred (d) required SSC CGL (Tier-I) –20/08/2021 (Shift-I) Ans. (c) : For the above blank space (3) 'referred' (V3) will be used. Other options are not correct. 489. Select the most appropriate option to fill in blank number 4. (a) of (b) at (c) on (d) in SSC CGL (Tier-I) –20/08/2021 (Shift-I) Ans. (a) : For the above blank space (4) 'of ' preposition will be used. Becouse 'aware of' something/somebody is used. 490. Select the most appropriate option to fill in blank number 5. (a) geological (b) geographical (c) physical (d) botanical SSC CGL (Tier-I) –20/08/2021 (Shift-I) Ans. (c) : For the above blank space (5) 'physical' will be used. Other options are not correct. Comprehension In the following passage, some words have been deleted. Fill in the blanks with the help of the alternatives given. Select the most appropriate option for each blank. Water, a substance composed of the chemical elements hydrogen and oxygen (1)______ in gaseous, liquid, and solid states. It is (2)______ of the most plentiful and essential (3)______ compounds. A tasteless and odourless liquid at room temperature, it has the important (4)______ to dissolve many other substances. Indeed, the (5)______ of water as a solvent is essential to living organisms. 491. Select the most appropriate option to fill in blank number 1. (a) exists (b) lasts (c) survives (d) endures SSC CGL (Tier-I) –17/08/2021 (Shift-I) Cloze Test

Ans. (a) : For the above blank space (1) 'exists' will be used. Other options are not correct. 492. Select the most appropriate option to fill in blank number 2. (a) unique (b) lone (c) one (d) single SSC CGL (Tier-I) –17/08/2021 (Shift-I) Ans. (c) : For the above blank space (2) 'one' will be used. Other options are not correct. 493. Select the most appropriate option to fill in the blank number 3. (a) by (b) in (c) for (d) of SSC CGL (Tier-I) –17/08/2021 (Shift-I) Ans. (d) : For the above blank space (3) 'of ' will be used. Because of preposition is used for possession. 494. Select the most appropriate option to fill in blank number 4. (a) ability (b) knack (c) proficiency (d) competence SSC CGL (Tier-I) –17/08/2021 (Shift-I) Ans. (a) : For the above blank space (4) 'ability' will be used. Other options are not correct. 495. Select the most appropriate option to fill in blank number 5. (a) velocity (b) validity (c) veracity (d) versatility SSC CGL (Tier-I) –17/08/2021 (Shift-I) Ans. (d) : For the above blank space (5) 'versatility' will be used. Other options are not correct. Comprehension In the following passage, some words have been deleted. Fill in the blanks with the help of the alternatives given. Select the most appropriate option for each blank. What every tourist must take back from Bhutan is pure honey, especially Putka, “Antibiotic” honey produced (1)______ Melipona Bees (stingless bees) a breed found in protected areas (2)______ 700 to 1,500 metres above sea level. Due (3)______ their small size, they can get larger nutrients (4)______ regular honeybees. It has a tangy/sour taste and can (5)______ your sore throat in a matter of minutes. 496. Select the most appropriate option to fill in blank number 1. (a) through (b) with (c) by (d) of SSC CGL (Tier-I) –13/08/2021 (Shift-I) Ans. (c) : For the above blank space (1) 'by' will be used. Because it gives suitable meaning for the sentence. 497. Select the most appropriate option to fill in blank number 2. (a) midst (b) among (c) between (d) after SSC CGL (Tier-I) –13/08/2021 (Shift-I)

809

YCT

Ans. (c) : For the above blank space (2) 'between' will be used. Because preposition 'between' is used for two people. 498. Select the most appropriate option to fill in blank number 3. (a) for (b) to (c) at (d) of SSC CGL (Tier-I) –13/08/2021 (Shift-I) Ans. (b) : For the above blank space (3) 'to' will be used. Other options are not correct. 499. Select the most appropriate to fill in blank number 4. (a) and (b) than (c) then (d) though SSC CGL (Tier-I) –13/08/2021 (Shift-I) Ans. (b) : For the above blank space (4) 'than' will be used. Other options are not correct. 500. Select the most appropriate option to fill in blank number 5. (a) appease (b) calm (c) pacify (d) soothe SSC CGL (Tier-I) –13/08/2021 (Shift-I) Ans. (d) : For the above blank space (5) 'soothe' will be used. Other options are not correct. Comprehension In the following passage some words have been deleted. Fill in the blanks with the help of the alternatives given. Select the most appropriate option for each blank. M. Ananda Kumar, scientist. Nature Conservation Foundation says that elephants are "engineers of the eco-system. He explains : they are seed (1) ____Forests without (2) ____have been observed to not have young (3)_____at all. This is because (4)_____species disperse seeds only through elephants. The animals are phenomenal at (5) ____themselves to new ecological conditions and negotiating a problem. 501. Select the most appropriate option to fill in blank No. (1) (a) eaters (b) makers (c) dispersers (d) collectors SSC CGL (Tier-I) – 13/06/2019 (Shift-I) Ans : (c) For the above blank space (1) 'dispersers' will be used. Other options are not correct. 502. Select the most appropriate option to fill in blank No.(2) (a) seeds (b) elephants (c) engineers (d) conservation SSC CGL (Tier-I) – 13/06/2019 (Shift-I) Ans : (b) For the above blank space (2) 'Elephants' will be used. Other options are not correct. 503. Select the most appropriate option to fill in blank No. (3) (a) saplings (b) seeds (c) species (d) elephants SSC CGL (Tier-I) – 13/06/2019 (Shift-I) Cloze Test

Ans : (a) For the above blank space (3) 'saplings' will be used. Other options are not correct. 504. Select the most appropriate option to fill in blank No. (4) (a) no (b) certain (c) all (d) every SSC CGL (Tier-I) – 13/06/2019 (Shift-I) Ans : (b) For the above blank space (4) 'certain' will be used. Other options are not correct. 505. Select the most appropriate option to fill in blank No. (5) (a) adapting (b) structuring (c) expanding (d) explaining SSC CGL (Tier-I) – 13/06/2019 (Shift-I) Ans : (a) For the above blank space (5) 'adapting' will be used. Other options are not correct. Comprehension In the following passage some words have been deleted. Fill in the blanks will the help of the alternatives given. Select the most appropriate option for each blank. Marine Conservation Society is hosting the Anjarle Turtle festival 2019, aiming to make more people aware about Olive Ridley turtle. The sea (1)__get their names from their olivecoloured (2)__They grow to about two feet in (3)___and reportedly mate at around 1000 kilometer from the Anjarle (4)__These turtles come to the beach to lay (5) __. 506. Select the most appropriate option to fill in blank no. (1) (a) turtle (b) animal (c) fish (d) snakes SSC CGL (Tier-I) – 13/06/2019 (Shift-II) Ans. (a) : For the above blank space (1) 'turtle' will be used. Other options are not correct. 507. Select the most appropriate option to fill in blank No. (2) (a) shells (b) beaks (c) feathers (d) wings SSC CGL (Tier-I) – 13/06/2019 (Shift-II) Ans. (a) : For the above blank space (2) 'shells' will be used. Other options are not correct. 508. Select the most appropriate option to fill in blank no. (3) (a) depth (b) weight (c) growth (d) length SSC CGL (Tier-I) – 13/06/2019 (Shift-II) Ans. (d) : For the above blank space (3) 'length' will be used. Other options are not correct. 509. Select the most appropriate option to fill in blank no. (4) (a) sea (b) sand (c) beach (d) festival SSC CGL (Tier-I) – 13/06/2019 (Shift-II) Ans. (c) : For the above blank space (4) 'beach' will be used. Other options are not correct.

810

YCT

510. Select the most appropriate option to fill in blank no. (5) (a) eggs (b) pebbles (c) young ones (d) shells SSC CGL (Tier-I) – 13/06/2019 (Shift-II) Ans. (a) : For the above blank space (5) 'eggs' will be used. Other options are not correct. Comprehension In the following passage some words have been deleted. Fill in the blanks with the help of the alternatives given. Select the most appropriate option for each blank. Although a wild elephant, Chinna Thambi did not grow up entirely in the wild. For over the last (1)____ he has had easy (2)_____ to food at Thadagam, a village surrounded by the Western Ghats near Coimbatore. Brick (3)_____ tempted him with (4)_____ of water and he loved the palm pith that was used as (5)_____ for baking bricks. 511. Select the most appropriate option to fill in blank No. (1) (a) festival (b) annual (c) decade (d) summer SSC CGL (Tier-I) – 12/06/2019 (Shift-II) Ans : (c) For the above blank space (1) 'decade' will be used. Other options are not correct. Festival - Magnificence Annual - Yearly Summer - Heat Correct sentenceFor over the last decade he has had.... 512. Select the most appropriate option to fill in blank No. (2) (a) entry (b) way (c) approach (d) access SSC CGL (Tier-I) – 12/06/2019 (Shift-II) Ans : (d) For the above blank space (2) 'access' will be used because access to use for deliver eating items. Other options are not correct. Access- the right or opportunity to use or receive something. 513. Select the most appropriate option to fill in blank No. (3) (a) jar (b) tub (c) store (d) kilns SSC CGL (Tier-I) – 12/06/2019 (Shift-II) Ans : (d) For the above blank space (3) 'kilns' will be used because brick is used before it. Other options are not correct. Jar - Pail Tub - Vat, Store - Shop 514. Select the most appropriate option to fill in blank No. (4) (a) load (b) much (c) plenty (d) many SSC CGL (Tier-I) – 12/06/2019 (Shift-II) Cloze Test

Ans : (c) For the above blank space (4) 'plenty' is used, because 'of' preposition is used with plenty. Other options are not correct. Load – Weight Many - Huge Hence, the correct sentence is Brick kilns tumpted him with plenty of water. 515. Select the most appropriate option to fill in blank No. (5) (a) matter (b) logs (c) fuel (d) fodder SSC CGL (Tier-I) – 12/06/2019 (Shift-II) Ans : (c) For the above blank space (2) 'fuel' will be used because in the passage we talk about baking bricks. Fodder - Forage Matter - Stuff Logs- Haft Comprehension In the following passage some words have been deleted. Fill in the blanks with the help of the alternatives given. Select the most approapriate option for each blank. He remembers the first time he saw the elephant. "This was in 2007 at a (1) ........ called Thanni Paarai. Chinna Thambi was (2) ........ water on himself at a waterbody. The (3) ......... was green; not the kind people would drink, "says Abraham. "After a nice shower, the elephant walked (4) ....... to a rock, lifted it, and drank from a (5) ......... under it." 516. Select the most approprite option to fill in blank No. (1) (a) market (b) shop (c) street (d) place SSC CGL (Tier-I) – 12/06/2019 (Shift-III) Ans. (d) : For the above blank space (1) 'place' will be used. Other options are not correct. Correct Sentence.......This was in 2007 at a place called Thanni Paarai..... 517. Select the most appropriate option to fill in blank No. (2) (a) sprinking (b) raining (c) wetting (d) sparkling SSC CGL (Tier-I) – 12/06/2019 (Shift-III) Ans. (a) : For the above blank space (2) 'sprinkling' will be used. Other options are not correct. Correct Sentence....Thambi was sprinkling water on himself..... 518. Select the most appropriate option to fill in blank No. (3) (a) leaf (b) milk (c) flower (d) water SSC CGL (Tier-I) – 12/06/2019 (Shift-III)

811

YCT

Ans. (d) : For the above blank space (3) 'water' will be used, because we talk about water. Other options are not correct. Correct Sentence....The water was green : not the kind.... 519. Select the most appropriate option to fill in blank No. (4) (a) above (b) over (c) near (d) behind SSC CGL (Tier-I) – 12/06/2019 (Shift-III) Ans. (b) : For the above blank space (4) 'over' will be used because 'walked over' is phrasal verb which meaning is something do easily. Correct Sentence....the Elephant walked over to a rock lifted it. 520. Select the most appropriate option to fill in the blank No. (5) (a) spring (b) tap (c) tank (d) pond SSC CGL (Tier-I) – 12/06/2019 (Shift-III) Ans. (a) : For the above blank space (5) 'spring' will be used. Other options are not correct. Correct Sentence.....and drank from a spring under it. Comprehension In the following passage words have been deleted. Fill in the blanks with the help of the alternatives given. Select the most appropriate option for each blank. The Bhopal gas tragedy has been described as the world's (1)––––– industrial disaster. Forty two tonnes of methyl isocyanate (2) –––– from the steel containers (3) –––––– the Union Carbide factory and released a cloud of (4) ––– – gas. It left a legacy of instant and (5) –––– death. 521. Select the most appropriate option to fill in blank No. 1 (a) worse (b) bad (c) best (d) worst SSC CGL (Tier-I) – 11/06/2019 (Shift-III) Ans. (d) : For the above blank space (1) 'worst' will be used according to sense of the santence. Other options are not correct. Correct sentenceThe Bhopal gas tragedy has been described as the world's worst industrial disaster. 522. Select the most appropriate option to fill in blank No. 2 (a) flowed (b) leaked (c) emitted (d) released SSC CGL (Tier-I) – 11/06/2019 (Shift-III) Ans. (b) : For the above blank space (2) 'leaked' will be used because here we talk about methyle iscocyanak gas. Meaning of other option– Flow- Flux Emitted- Exclude Released- Unfettered Cloze Test

523. Select the most appropriate option to fill in blank No. 3 (a) along (b) beside (c) at (d) across SSC CGL (Tier-I) – 11/06/2019 (Shift-III) Ans. (c) : For the above blank space (3) 'at' preposition will be used. Other options are not correct. 524. Select the most appropriate option to fill in blank No. 4 (a) deathly (b) merciless (c) incurable (d) abominable SSC CGL (Tier-I) – 11/06/2019 (Shift-III) Ans. (a) : For the above blank space (4) 'deathly' will be used. Other options are not correct. Meaning of other option– merciless- Heartless incurable- Insoluble Abominable- Abject 525. Select the most appropriate option to fill in blank No. 5 (a) deferred (b) momentary (c) detained (d) averted SSC CGL (Tier-I) – 11/06/2019 (Shift-III) Ans. (a) : For the above blank space (5) 'deferred' will be used. Other options are not correct. Meaning of other options– Momentary- Instant Detain- Stop Avert- Avoid Correct sentenceIt left a legacy of instant and deferred death. Comprehension In the following passage some words have been deleted. Fill in the blanks with the help of the alternatives given. Select the most appropriate option for each blank. Perhaps India should listen to her (1)_____ sages who taught the importance (2)______ balance. This means we should be (3)______ not to take anything too far (4)_____be too radical. The unfortunate results of (5)_______ too far are obvious in the environmental degradation. 526. Select the most appropriate option to fill in blank No. 1 (a) antique (b) initial (c) ancient (d) extinct SSC CGL (Tier-I) – 10/06/2019 (Shift-I) Ans : (c) For the above blank space (1) 'ancient' will be used. Other options are not correct. Correct SentencePerhaps India should listen to her ancient sages...... 527. Select the most appropriate option to fill in blank No. 2 (a) of (b) for (c) in (d) by SSC CGL (Tier-I) – 10/06/2019 (Shift-I)

812

YCT

Ans : (a) For the above blank space (2) 'of' preposition will be used according to the sence of the sentence. Other options are not correct. Correct Sentence......who taught the importance of balance....... 528. Select the most appropriate option to fill in blank No. 3 (a) carefree (b) caretaker (c) careless (d) careful SSC CGL (Tier-I) – 10/06/2019 (Shift-I) Ans : (d) For the above blank space (3) 'careful' will be used. Other options are not correct. Correct sentence.....we should be 'careful' not to take anything....... 529. Select the most appropriate option to fill in blank No. 4 (a) or (b) but (c) so (d) otherwise SSC CGL (Tier-I) – 10/06/2019 (Shift-I) Ans : (a) For the above blank space (4) 'or' will be used according to sense of the sentence. Other options are not correct. Correct sentence.....to take anything too far or be too radical....... 530. Select the most appropriate option to fill in blank No. 5 (a) venturing (b) risking (c) climbing (d) pioneering SSC CGL (Tier-I) – 10/06/2019 (Shift-I) Ans : (a) For the above blank space (5) 'venturing' will be used. Other options are not correct. Correct sentence.....The unfortunate results of venturing too far are..... Comprehension In the following passage some words have been deleted. Fill in the blanks with the help of the alternatives given. Select the most appropriate option for each blank. Portia claims that even if she (1)____ to live as long as Sibylla, she would die as (2)____ as Diana because she can only be claimed (3)____ the manner specified by her father's will. She (4)____ that nobody would be able to crack (5) ____ casket test and so she was bound to remain unmarried. 531. Select the most appropriate option to fill in blank No. 1 (a) has (b) were (c) was (d) is SSC CGL (Tier-I) – 10/06/2019 (Shift-III) Ans. (b) : For the above blank space (1) 'were' will be used because sentence is Hypothetical. Correct sentencePortia claims that even if she were to live as long as sibylla. Cloze Test

532. Select the most appropriate option to fill in blank No. 2 (a) clear (b) real (c) plain (d) pure SSC CGL (Tier-I) – 10/06/2019 (Shift-III) Ans. (d) : For the above blank space (2) 'pure' will be used. Other options are not correct. Correct sentenceShe would die as pure as Diana. 533. Select the most appropriate option to fill in blank No. 3 (a) at (b) in (c) through (d) from SSC CGL (Tier-I) – 10/06/2019 (Shift-III) Ans. (b) : For the above blank space (3) 'in' preposition is used because it gives suitable meaning of the sentence. Correct sentenceShe can only be claimed in the manner specified by her father will. 534. Select the most appropriate option to fill in blank No. 4 (a) will worry (b) has worry (c) worry (d) worries SSC CGL (Tier-I) – 10/06/2019 (Shift-III) Ans. (d) : For the above blank space (4) 'worries' will be used because sentence is in present indefinite tense and (V1 + s/es) is used with singular subject. Correct sentenceShe worries that nobody would be able to..... 535. Select the most appropriate option to fill in blank No. 5 (a) an (b) the (c) one (d) a SSC CGL (Tier-I) – 10/06/2019 (Shift-III) Ans. (b) : For the above blank space (5) 'the' will be used because Casket is a specific noun. Hence before it definite article 'the' is used. Correct sentenceShe worrise that nobody would to able to crack the casket test. Comprehension In the following passage some words have been deleted. Fill in the blanks with the help of the alternatives given. Select the most appropriate option for each blank. The political condition of India in the eighteenth century was one of extreme disunity. (1)____the decline of the Mughal empire, no (2)_____Indian power emerged to take its (3)_____ in strength and prestige and to unite the (4) _____ under a central authority. The Marathas who rose (5)____ the position of preeminence among the new Indian states did not prove capable of fulfilling that task.

813

YCT

536. Select the most appropriate option for blank No. 1 (a) With (b) In (c) When (d) On SSC CGL (Tier-I) – 07/06/2019 (Shift-I) Ans : (a) For the above blank space (1) 'with' preposition will be used because according to the sense of the sentence it gives appropriate meaning. Correct sentenceWith, the decline of the Mugal empire. 537. Select the most appropriate option for blank No. 2 (a) one (b) another (c) other (d) more SSC CGL (Tier-I) – 07/06/2019 (Shift-I) Ans : (c) For the above blank space (2) 'other' will be used. Other options are not correct. Correct sentenceNo other Indian power emerged to take its. 538. Select the most appropriate option for blank No. 3 (a) niche (b) place (c) role (d) seat SSC CGL (Tier-I) – 07/06/2019 (Shift-I) Ans : (b) For the above blank space (3) 'place' will be used because according to the sentence it gives suitable meaning. Correct sentencePlace in strength and prestige and to unite. 539. Select the most appropriate option for blank No. 4 (a) region (b) territory (c) country (d) state SSC CGL (Tier-I) – 07/06/2019 (Shift-I) Ans : (c) For the above blank space (4) 'country' will be used because it gives suitable meaning. Correct sentenceThe country under a central authority. 540. Select the most appropriate option for blank No. 5 (a) to (b) at (c) on (d) over SSC CGL (Tier-I) – 07/06/2019 (Shift-I) Ans : (a) For the above blank space (5) 'to' preposition will be used because it gives appropriate meaning. Correct sentenceThe Marathas who rose to the position of pre-eminence among the new Indian states did not prove capable of fulfilling that task. Comprehension In the following passage some words have been deleted. Fill in the blanks with the help of the alternatives given. Select the most appropriate option for each blank. During the reign of Aurangzeb, the last of the Mughals, there (1) _____revolts against the empire. (2)_____were the revolts of the Cloze Test

marathas, the Sikhs, the Jats and (3)____ others. After the death of Aurangzeb, the Mughal empire began to (4) ____fairly fast. The country soon (5) ____into smaller territories many of which became more or less independent. 541. Select the most appropriate option for blank No. 1 (a) have been (b) was (c) are (d) were SSC CGL (Tier-I) – 07/06/2019 (Shift-II) Ans : (d) For the above blank space (1) 'were' will be used because the subject 'revolts' is plural. Hence plural verb will be used. 542. Select the most appropriate option for blank No. 2 (a) These (b) It (c) That (d) This SSC CGL (Tier-I) – 07/06/2019 (Shift-II) Ans : (a) For the above blank space (2) 'these' will be used because Demonstrative pronoun is used for plural countable noun. Correct sentenceThese were the revolts of the Marathas, the Sikhs, the Jats. 543. Select the most appropriate option for blank No. 3 (a) more (b) many (c) each (d) every SSC CGL (Tier-I) – 07/06/2019 (Shift-II) Ans : (b) For the above blank space (3) 'many' will be used. Other options are not correct. Correct sentenceThese were the revolts of the Marathas, the Sikhs, the Jats and many others. Note- 'Much' is used before the uncountable noun. 544. Select the most appropriate option for blank No. 4 (a) disperse (b) disturb (c) disrupt (d) disintegrate SSC CGL (Tier-I) – 07/06/2019 (Shift-II) Ans : (d) For the above blank space (4) 'Disintegrate' will be used. Other options are not correct. Meaning of other options– Disperse - Spread Disturb - Impede Disrupt - Outrage 545. Select the most appropriate option for blank No. 5 (a) broken in (b) broke off (c) broke out (d) broke up SSC CGL (Tier-I) – 07/06/2019 (Shift-II) Ans : (d) For the above blank space (5) 'broke' up will be used because it gives suitable meaning to the sentence. Broken in – To interrupt Broke off – To separate something Broke out – To scape from a place

814

YCT

Comprehension In the following passage some words have been deleted. Fill in the blanks with the help of the alternatives given. Select the most appropriate option for each blank. There was still a light on in Mo's room. He often stayed (1)_____reading late into the night. Meggie had (2)______ her love of books from him. When she (3) ______ refuge with him from a bad dream, nothing could pull (4)_______ back to sleep better than Mo's calm breathing (5)____ her and the sound of the pages turning. 546. Select the most appropriate option to fill in blank No. 1 (a) up (b) away (c) for (d) out SSC CGL (Tier-I) – 07/06/2019 (Shift-III) Ans : (a) For the above blank space (1) 'up' will be used because stay followed by 'up' preposition. Stay up – Wake up, 547. Select the most appropriate option to fill in blank No. 2 (a) procured (b) inherited (c) extracted (d) gained SSC CGL (Tier-I) – 07/06/2019 (Shift-III) Ans : (b) For the above blank space (2) 'inherited' will be used because it gives appropriate meaning. Other options are not correct. 548. Select the most appropriate option to fill in blank No. 3 (a) takes (b) has taken (c) will take (d) took SSC CGL (Tier-I) – 07/06/2019 (Shift-III) Ans : (d) For the above blank space (3) 'took (V2)' will be used because passage is in past tense. 549. Select the most appropriate option to fill in blank No. 4 (a) her (b) it (c) his (d) them SSC CGL (Tier-I) – 07/06/2019 (Shift-III) Ans : (a) For the above blank space (4) 'her' will be used because the subject of the sentence is 'she' and its objective case 'her'. When she took refuge with him from a bad dream, nothing could pull her back to sleep. 550. Select the most appropriate option to fill in blank No. 5 (a) against (b) around (c) beside (d) behind SSC CGL (Tier-I) – 07/06/2019 (Shift-III) Ans : (c) For the above blank space (5) 'beside' will be used because it gives appropriate meaning. Other options are not correct. Cloze Test

Comprehension In the following passage some words have been deleted. Fill in the blanks with the help of the alternatives given. Select most appropriate option for each blank. In 1914 a war began in Europe which engulfed the entire world. The damage (1)______by this war has no precedent in history. (2)_____ the earlier wars, the civilian populations were (3)______ not involved and the casualties were mostly (4) _______to the warning armies. The war (5)______ began in 1914 was a total war in which all the resources of the warring states were mobilized. 551. Select the most appropriate option for blank No. 1. (b) caused (a) made (c) generated (d) began SSC CGL (Tier-I) – 06/06/2019 (Shift-II) Ans : (b) For the above blank space (1) 'caused' will be used because it gives appropriate meaning. Made- Creation Began- Unleash Generated- Beget 552. Select the most appropriate option for blank No. 2 (a) At (b) Into (c) In (d) On SSC CGL (Tier-I) – 06/06/2019 (Shift-II) Ans : (c) For the above blank space (2) 'In' will be used. Other options are not correct. 553. Select the most appropriate option for blank No. 3 (a) generally (b) broadly (c) chiefly (d) finally SSC CGL (Tier-I) – 06/06/2019 (Shift-II) Ans : (a) For the above blank space (3) 'Generally' will be used. Other options are not correct. Broadly- Richly Chiefly- Specially Finally-Ultimately 554. Select the most appropriate option for blank No. 4 (a) belonged (b) confined (c) reatrained (d) bound SSC CGL (Tier-I) – 06/06/2019 (Shift-II) Ans : (b) For the above blank space (4) 'confined' will be used because it gives appropriate meaning. 555. Select the most appropriate option for blank No. 5 (a) which (b) whom (c) what (d) who SSC CGL (Tier-I) – 06/06/2019 (Shift-II) Ans : (a) For the above blank space (5) 'which' will be used because the use of 'which' is appropriate to denote object and any event.

815

YCT

Comprehension In the following passage some words have been deleted. Fill in the blanks with the help of the alternatives given. Select the most appropriate option for each blank. Canada is three times the size of India (1)____ its population is only about 26 million. The average (2)_____ of population is less than three persons per square kilometer (3)____ Canada. The distribution of population is highly (4)_____. Nearly 80 percent of the people (5)____in a narrow belt less than 300 kilometer wide along the southern border. The rest of the country has a very sparse population due to excessive cold. 556. Select the most appropriate option for blank No. (1) (a) because (b) until (c) so (d) but SSC CGL (Tier-I) – 06/06/2019 (Shift-III) Ans. (d) : For the above blank space (1) 'but' conjunction will be used. Other options are not correct. 557. Select the most appropriate option for blank No. (2) (a) consistency (b) density (c) quantity (d) frequency SSC CGL (Tier-I) – 06/06/2019 (Shift-III) Ans. (b) : For the above blank space (2) 'density' will be used. Other options are not correct. Meaning of other optionsConsistency - Relevancy Quantity - Magnitude Frequency - Recap 558. Select the most appropriate option for blank No. (3) (a) at (b) in (c) on (d) into SSC CGL (Tier-I) – 06/06/2019 (Shift-III) Ans. (b) : For the above blank space (3) 'In' preposition will be used. Other options are not correct. 559. Select the most appropriate option for blank No. (4) (a) uneven (b) differing (c) infrequent (d) rough SSC CGL (Tier-I) – 06/06/2019 (Shift-III) Ans. (a) : For the above blank space (4) 'uneven' will be used. Other options are not correct. Meaning of other optionsRough - Rude Differ - Distinguish Infrequent - Abnormal 560. Select the most appropriate option for blank No. (5) (a) live (b) survive (c) subsist (d) remain SSC CGL (Tier-I) – 06/06/2019 (Shift-III) Cloze Test

Ans. (a) : For the above blank space (5) 'live' will be used. Other options are not correct. Meaning of other optionsSurvive - Outlive Remain - Escape Subsist - Live through Comprehension In the following passage some words have been deleted. Fill in the blanks with the help of he alternatives given. Select the most appropriate option for each blank. An Itallian mayor has been cleaning the streets along with his councilors after their town (1)_____ with no manual workers, it's reported. In fact, (2)_____ was sweeping the piazza in front of the (3) ______ church in preparation for market day, (4)_____the deputy mayor's father and a town councilor armed with a high-pressure hose. The town Zerfallu's last (5) _____retired six months ago and nobody has been hired since then. "We can't do anything - we are blocked by bureaucracy," the mayor says. 561. Select the most appropriate option. (a) leaves (b) was left (c) was leaving (d) has left SSC CGL (Tier-I) – 04/06/2019 (Shift-II) Ans : (b) For the above blank space (1) 'was left' will be used because the sentence is in passive voice of simple past tense. Correct Sentence....their town was left with no manual workers. 562. Select the most appropriate option. (a) they (b) it (c) she (d) he SSC CGL (Tier-I) – 04/06/2019 (Shift-II) Ans : (d) For the above blank space (2) 'he' will be used because the subject of the sentence is 'An Italian Mayor' hence subjective case pronoun 'he' is used. Correct Sentence....In fact he was sweeping the........ 563. Select the most appropriate option. (a) next (b) near (c) neighbour (d) local SSC CGL (Tier-I) – 04/06/2019 (Shift-II) Ans : (d) For the above blank space (3) 'local' will be used. Other options are not correct. Correct Sentence....In front of the local church 564. Select the most appropriate option. (a) as well (b) alongside (c) also (d) along SSC CGL (Tier-I) – 04/06/2019 (Shift-II) Ans : (b) For the above blank space (4) 'alongside' will be used because when two subject work together then alongside is used. Correct Sentence....for market day alongside the deputy Mayor's.....

816

YCT

565. Select the most appropriate option. (a) mayor (b) councillor (c) worker (d) member SSC CGL (Tier-I) – 04/06/2019 (Shift-II) Ans : (c) For the above blank space (5) 'worker' will be used because it gives appropriate meaning. Correct Sentence.....The town zerfalio's last worker retired six..... Comprehension Read the given passage and fill in the blanks with the most appropriate options. The refugees coming by boat to ltaly have already set a record this year. The surge (1)_____ partly from the civil war in Syria, which has forced 3 million people to (2)_____ their homeland. Europe, peaceful, affluent and comparatively close-is a preferred (3) ____. It's where the desperate meets the developed. Often with the aid (4)_____ reckless human traffickers, refugees attempt to cross the waters in rubber dinghies that can (5)_____, plunging the occupants into the sea. 566. Select the most appropriate option to fill in blank No. 1 (a) is resulting (b) has resulted (c) have resulted (d) is resulted SSC CPO-SI – 11/12/2019 (Shift-II) Ans : (b) For the above blank space (1) 'has' resulted will be used is in passive voice (has+V1). 567. Select the most appropriate option to fill in blank No. 2 (a) fleet (b) fled (c) flee (d) fly SSC CPO-SI – 11/12/2019 (Shift-II) Ans : (c) For the above blank space (2) 'flee' will be used because it gives appropriate meaning to the sentence. 568. Select the most appropriate option to fill in blank No. 3 (a) objective (b) destination (c) purpose (d) target SSC CPO-SI – 11/12/2019 (Shift-II) Ans : (b) For the above blank space (3) 'Destination' will be used because it gives appropriate meaning to the sentence. 569. Select the most appropriate option to fill in blank No. 4 (a) by (b) of (c) for (d) on SSC CPO-SI – 11/12/2019 (Shift-II) Ans : (b) For the above blank space (4) 'of' will be used because it gives appropriate meaning. 570. Select the most appropriate option to fill in blank No. 5 (a) inverse (b) convert (c) renege (d) capsize SSC CPO-SI – 11/12/2019 (Shift-II) Cloze Test

Ans : (d) For the above blank space (5) 'capsize' will be used. Other options are not correct. Comprehension Some words have been deleted in the given passage. Fill in the blanks with the given alternatives. Select the most appropriate option for each blank. Polar bears roam the Arctic ice sheets and swim in that region's coastal waters. They are (1)____ strong swimmers, and their large front paws, (2)_____ they use to paddle, are (3)_____ webbed. Some polar bears have been seen (4)____ hundreds of miles from land, though they probably cover most of that (5)_____ by floating on sheets of ice. 571. Select the most appropriate option to fill in blank number (1). (a) very (b) so (c) too (d) much SSC CPO-SI – 13/12/2019 (Shift-II) Ans. (a) : For the above blank space (1) 'very' is used because to specify 'strong' (Adjectiev) 'very' is used as an adverb. 572. Select the most appropriate option to fill in blank number (2). (a) them (b) whom (c) those (d) which SSC CPO-SI – 13/12/2019 (Shift-II) Ans. (d) : For the above blank space (2) 'which' will be used because Relative pronoun 'which' is use for nonliving things (Paws). 573. Select the most appropriate option to fill in blank number (3). (a) faintly (b) slightly (c) dimly (d) nimbly SSC CPO-SI – 13/12/2019 (Shift-II) Ans. (b) : For the above blank space (3) 'slightly' will be used. Other options are not correct. 574. Select the most appropriate option to fill in blank number (4). (a) swimming (b) wandering (c) hunting (d) running SSC CPO-SI – 13/12/2019 (Shift-II) Ans. (a) : For the above blank space (4) 'swimming' will be used according to the sentence. Meaning of other options– Wandering – Errant Hunting - Chevy Running - Effluence 575. Select the most appropriate option to fill in blank number (5). (a) extent (b) extant (c) distance (d) water SSC CPO-SI – 13/12/2019 (Shift-II)

817

YCT

Ans. (c) : For the above blank space (5) 'distance' will be used. It gives appropriate meaning. Other options are not correct. Meaning of other options – Extent-Region Extant- Current Comprehension In the following passage some words have been deleted. Fill in the blanks with the help of the alternatives given. Select the most appropriate option for each blank. If you don't have a lump sum to invest in a fixed deposit, you can (1) ––––– opt for a long term recurring deposit, (2) –––– the minimum amount of monthly investment is (3) –––– low as Rupees 100. At the end of the (4) ––––––– you get back the amount invested (5) –––––––– with the interest earned on it. It is a good way of saving money regularly. 576. Select the most appropriate option to fill in blank No. 1. (a) ensure (b) assured (c) surely (d) sure SSC CPO-SI – 12/12/2019 (Shift-I) Ans : (c) For the above blank space (1) 'surely' will be used because it gives appropriate meaning. Other options are not correct. 577. Select the most appropriate option to fill in blank No. 2. (a) when (b) wherefore (c) where (d) whenever SSC CPO-SI – 12/12/2019 (Shift-I) Ans : (c) For the above blank space (2) 'conjunction' 'where' will be used. Other options are not correct. 578. Select the most appropriate option to fill in blank No. 3. (a) so (b) just (c) as (d) too SSC CPO-SI – 12/12/2019 (Shift-I) Ans : (c) For the above blank space (3) 'as' will be used because as...........as is used as a conjunction in negative and affirmative sentence. 579. Select the most appropriate option to fill in blank. No. 4. (a) interval (b) timing (c) time (d) tenure SSC CPO-SI – 12/12/2019 (Shift-I) Ans : (d) For the above blank space (4) 'tenure' will be used because it gives appropriate meaning. Other options are not correct. 580. Select the most appropriate option to fill in blank No. 5. (a) among (b) as well as (c) along (d) besides SSC CPO-SI – 12/12/2019 (Shift-I) Cloze Test

Ans : (c) For the above blank space (5) 'along' is used because 'along with' gives appropriate meaning to the sentence. Comprehension In the following passage some words have been deleted. Fill in the blanks with the help of the alternatives given. Select the most appropriate option for each blank. Central Park Tower has just become the tallest residential building in the world. The views (1)_____ this building are amazing. The (2)___ of apartments in this skyscraper are (3)______ as the sky. The cheapest apartment (4)______ at $6.7 million, and the most (5)_____ fivebedroom apartment costs $63 million. 581. Select the most appropriate option to fill in blank No. 1. (a) from (b) on (c) up (d) above SSC CPO-SI – 11/12/2019 (Shift-I) Ans : (a) For the above blank space (1) 'from' preposition will be used. Other options give different meaning. 582. Select the most appropriate option to fill in blank No. 2. (a) bills (b) prices (c) values (d) amounts SSC CPO-SI – 11/12/2019 (Shift-I) Ans : (b) For the above blank space (2) 'prices' will be used because it gives appropriate meaning. Other options are not correct. 583. Select the most appropriate option to fill in blank No. 3. (a) as high (b) high (c) highest (d) higher SSC CPO-SI – 11/12/2019 (Shift-I) Ans : (a) For the above blank space (3) 'as high' will be used. Other options are not correct. 584. Select the most appropriate option to fill in blank No. 4. (a) start (b) is starting (c) started (d) starts SSC CPO-SI – 11/12/2019 (Shift-I) Ans : (d) For the above blank space (4) 'starts' will be used because passage is in present indefinite tense and the subject "the cheapest" apartment is singular. Hence singular verb will be used. 585. Select the most appropriate option to fill in blank No. 5. (a) economical (b) extreme (c) expensive (d) excessive SSC CPO-SI – 11/12/2019 (Shift-I) Ans : (c) For the above blank space (5) 'expensive' will be used according to the sense of the sentence. Other options are not correct. Economical - Frugal Extreme - Ultimate Excessive – Exorbitant

818

YCT

Comprehension In the following passage some wods have been deleted. Fill in the blanks with the help of the alternatives given. Select the most appropiate option for each blank. After coffee had been served in different-sized cups and mugs, the chif rose (1) ––––– his place at the head of the table (2) –––– a speech of welcome, (3) ––––– Aziz translated. Scott made a short (4) ––––– which was applauded even before Aziz had been given the (5) –––– to interpet what he had said. 586. Select the most appropriate option to fill in blank No. 1. (a) from (b) on (c) to (d) into SSC CPO-SI – 09/12/2019 (Shift-II) Ans : (a) For the above blank space (1) 'from' will be used because from preposition is used for denote point of place. 587. Select the most appropriate option to fill in blank No. 2. (a) makes (b) made (c) to make (d) make SSC CPO-SI – 09/12/2019 (Shift-II) Ans : (c) For the above blank space (2) 'to make' will be used because (to+v1) infinitive is used as a noun. 588. Select the most appropriate option to fill in blank No. 3. (a) which (b) whom (c) who (d) what SSC CPO-SI – 09/12/2019 (Shift-II) Ans : (a) For the above blank space (3) 'which' will be used as a pronoun. Other options are not correct. Note- ‘Which’ is used to thing, ‘Who’ and 'Whom is' used to person as a pronoun. 589. Select the most appropriate option to fill in blank No. 4. (a) reply (b) rejoin (c) retort (d) respond SSC CPO-SI – 09/12/2019 (Shift-II) Ans : (a) For the above blank space (4) 'reply' will be used because it gives appropriate meaning according to the sentence. 590. Select the most appropriate option to fill in blank No. 5. (a) probability (b) turning (c) chance (d) moment SSC CPO-SI – 09/12/2019 (Shift-II) Ans : (c) For the above blank space (5) 'chance' will be used. Other options are not correct. Cloze Test

Comprehension In the following passage some words have been deleted. Fill in the blanks with the help of the alternatives given. Select the most appropriate option for each blank. If winter seems to be the season for migratory birds to flock to warmer climates, summer time is ideal for wetland birds. (1) ___ fifty kilometers from Warangal in Talangana, Pakhal lake (2) ____ to be playing host to (3) __ wide variety of flora and fauna (4) _____ season. Wetland birds from North and Central India come (5) _____ the south in search of larger water bodies. 591. Select the most appropriate option to fill in blank No. 1 (a) Totally (b) But (c) Nearly (d) Not SSC CHSL (Tier-I) –11/07/2019 (Shift-I) Ans : (c) For the above blank space (1) 'nearly' will be used as an adverb because it gives appropriate meaning. 592. Select the most appropriate option to fill in blank No. 2 (a) will seems (b) seems (c) was seemed (d) seem SSC CHSL (Tier-I) –11/07/2019 (Shift-I) Ans : (b) For the above blank space (2) 'seems' will be used because sentence is in present tense and the subject 'Pakhal Lake' of the sentece is singular hence singular verb will be used. Note- Simple present Tense → Sub. + Verb + s/es + obj. 593. Select the most appropriate option to fill in blank No. 3 (a) one (b) a (c) the (d) an SSC CHSL (Tier-I) –11/07/2019 (Shift-I) Ans : (b) For the above blank space (3) article 'a' will be used because to gives indefinit sense in singular number subject, article 'a' is used. 594. Select the most appropriate option to fill in blank No. 4 (a) their (b) these (c) this (d) that SSC CHSL (Tier-I) –11/07/2019 (Shift-I) Ans : (c) For the above blank space (4) 'this' will be used. Other options are not correct. Note-In this sentence ‘this’ is used as a demonstrative adjective. 595. Select the most appropriate option to fill in blank No. 5 (a) below (b) towards (c) away (d) about SSC CHSL (Tier-I) –11/07/2019 (Shift-I) Ans : (b) For the above blank space (5) 'towards' preposition will be used. Other options are not correct.

819

YCT

Comprehension In the following passage some words have been deleted. Fill in the blanks with the help of the alternatives given. Select the most appropriate option for each blank. Rivers around the world are (1) ____ with dangerous levels of antibiotics, (2) ___ to a recent study. Concentration of antibiotics in some waterways (3) ___ safety levels by 300 times. Researchers looked at several (4) ___ used antibiotics in rivers flowing (5) ___ 72 countries. 596. Select the most appropriate option for blank No. 1. (a) fouled (b) contaminated (c) infected (d) tainted SSC CHSL (Tier-I) –11/07/2019 (Shift-III) Ans : (b) For the above blank space (1) 'contaminated' will be used. Other options are not correct. 597. Select the most appropriate option for blank No. 2. (a) according (b) consistent (c) conforming (d) reported SSC CHSL (Tier-I) –11/07/2019 (Shift-III) Ans : (a) For the above blank space (2) 'according' will be used. Other options are not correct. 598. Select the most appropriate option for blank No. 3. (a) exceeds (b) outpaces (c) outgrows (d) excels SSC CHSL (Tier-I) –11/07/2019 (Shift-III) Ans : (a) For the above blank space (3) 'exceeds' will be used because it gives appropriate meaning. 599. Select the most appropriate option for blank No. 4. (a) badly (b) likely (c) commonly (d) unusually SSC CHSL (Tier-I) –11/07/2019 (Shift-III) Ans : (c) For the above blank space (4) 'commonly' will be used. Other options are not correct. 600. Select the most appropriate option for blank No. 5. (a) through (b) below (c) with (d) above SSC CHSL (Tier-I) –11/07/2019 (Shift-III) Ans : (a) For the above blank space (5) 'through' will be used because it gives appropriate meaning to the sentence. Comprehension In the following passage some words have been deleted. Fill in the blanks with the help of the alternatives given. Select the most appropriate option for each blank. Students of marketing know that the term Unique Selling Proposition, or USP was developed in the 1940s in the United States. USP is (1)_____as the true value (2)_____ a firm. At the core of (3)_____successful business, there is (4)_____ a value proposition, something (5)_____promoters and the managers are proud Cloze Test

601. Select the most appropriate option to fill in blank No. 1 (a) referred (b) repeated (c) regarded (d) reviewed SSC CHSL (Tier-I) –09/07/2019 (Shift-I) Ans : (c) For the above blank space (1) 'regarded' will be used. Other options are not correct. Meaning of other options – Reviewed - Observe Referred – Mentioned Repeated - Encore 602. Select the most appropriate option to fill in blank No. 2 (a) on (b) from (c) by (d) of SSC CHSL (Tier-I) –09/07/2019 (Shift-I) Ans : (d) For the above blank space (2) 'of' is used because value is always followed by 'of' preposition. Other options are not correct. 603. Select the most appropriate option to fill in blank No. 3 (a) large (b) only (c) every (d) sole SSC CHSL (Tier-I) –09/07/2019 (Shift-I) Ans : (c) For the above blank space (3) 'every' will be used because it gives appropriate meaning to the sentence. 604. Select the most appropriate option to fill in blank No. 4 (a) even (b) always (c) ready (d) one SSC CHSL (Tier-I) –09/07/2019 (Shift-I) Ans : (b) For the above blank space (4) 'always' will be used. Other options are not correct. Meaning of other options – Even - Further Ready - Prepared 605. Select the most appropriate option to fill in blank No. 5 (a) there (b) the (c) those (d) this SSC CHSL (Tier-I) –09/07/2019 (Shift-I) Ans : (b) For the above blank space (5) 'the' will be used because when two noun connected with 'and' and verb is plural then article 'the' is used before both nouns. Correct sentence : The promoters and the managers are proud of. Comprehension In the following passage some words have been deleted. Fill in the blanks with the help of the alternatives given. Select the most appropriate option for each blank. Long before the dawn of modern environmental awareness and the stress on recycling, our forefathers developed the practice of eating off banana leaves. It is a timeless practice (1)_____ in every Vedic ritual. The Vedas refer to (2)____ sacred properties of the banana plant, (3)_____ uniquely bears fruit without pollination, the fruits containing no (4)_____. The plant thus (5)_____ fertility and plenty.

820

YCT

606. Select the most appropriate option to fill in blank No. 1. (a) brought (b) read (c) found (d) said SSC CHSL (Tier-I) –09/07/2019 (Shift-II) Ans : (c) For the above blank space (1) 'found' will be used. Other options are not correct. Brought- Fetch Read- Study Said- Aforecited Correct sentence.....It is a timeless practice found in every Vedic ritual. 607. Select the most appropriate option to fill in blank No. 2. (a) an (b) one (c) the (d) a SSC CHSL (Tier-I) –09/07/2019 (Shift-II) Ans : (c) For the above blank space (2) 'the' will be used because "sacred properties" is a definite noun. Hence definite article 'the' will be used. Correct sentenceThe Vedas refer to the sacred properties..... 608. Select the most appropriate option to fill in blank No. 3. (a) which (b) where (c) whom (d) who SSC CHSL (Tier-I) –09/07/2019 (Shift-II) Ans : (a) For the above blank space (3) 'which' will be used because 'which' pronoun is used for non-living things (The Vedas) and 'who' is used for Living things. However 'that' is used both cases. 609. Select the most appropriate option to fill in blank No. 4. (a) branches (b) seeds (c) leaves (d) roots SSC CHSL (Tier-I) –09/07/2019 (Shift-II) Ans : (b) For the above blank space (4) 'seeds' will be used. Other options are not correct. Branches- Knag Leaves- Foliage Roots- Foundation 610. Select the most appropriate option to fill in blank No. 5. (a) refers (b) explains (c) symbolises (d) means SSC CHSL (Tier-I) –09/07/2019 (Shift-II) Ans : (c) For the above blank space (5) 'symbolises' will be used because it gives appropriate meaning. Other options are not correct. Comprehension In the following passage some words have been deleted. Fill in the blank with the help of the alternatives given. Select the most appropriate option for each blank. As the rains receded I got a strange job. Our neighbor, a (1)___ merchant decided to become a politician. But he was (2)____ so he wanted me to (3)___ him everywhere he went and read and write (4)____ him. For my work I was to be paid decent (5)____ Cloze Test

611. Select the most appropriate option to fill in blank number 1 (a) populous (b) prosperous (c) affluent (d) ambitious SSC CHSL (Tier-I) –08/07/2019 (Shift-I) Ans : (b) For the above blank space (1) 'prosperous' will be used. Other options have different meaning. 612. Select the most appropriate option to fill in blank number 2 (a) illogical (b) illegible (c) illegitimate (d) illiterate SSC CHSL (Tier-I) –08/07/2019 (Shift-I) Ans : (d) For the above blank space (2) 'illiterate' will be used. Other options are not correct. Illogical- Irrelevant Illegible- Vague Illegitimate- Clandestine 613. Select the most appropriate option to fill in blank number 3 (a) attend (b) coexist (c) accompany (d) coincide SSC CHSL (Tier-I) –08/07/2019 (Shift-I) Ans : (c) For the above blank space (3) 'accompany' will be used. Other options have different meaning. Attend- Report Coexist- Accounting Coincide- Concur 614. Select the most appropriate option to fill in blank number 4 (a) for (b) from (c) on (d) through SSC CHSL (Tier-I) –08/07/2019 (Shift-I) Ans : (a) For the above blank space (4) 'For' preposition will be used. Other options are not correct. 615. Select the most appropriate option to fill in blank number 5 (a) wages (b) prize (c) reward (d) bonus SSC CHSL (Tier-I) –08/07/2019 (Shift-I) Ans : (a) For the above blank space (5) 'wages' will be used. Other options give different meaning. Meaning of other options – Prize- Guerdon Reward- Bounty Bonus- Regalia Comprehension In the following passage some words have been deleted. Fill in the blanks with the help of the alternatives given. Select the most appropriate option for each blank. Each child deserves love and care. The child who is (1)____ for what he is even if he is slow or (2)____, will grow with confidence in himself. He will (3)____ the ability to make the best use of the (4)____ which come his way and make light of any (5)____ that may pose difficulties in his path.

821

YCT

616. Select the most appropriate option to fill in the blank number 1 (a) appreciated (b) thanked (c) rebuked (d) jeered SSC CHSL (Tier-I) –08/07/2019 (Shift-III) Ans : (a) For the above blank space (1) 'appreciated' will be used. Other options are not correct. Meaning of other options– Thanked- Pay tribute Rebuked- Vilify Jeered- Submissive 617. Select the most appropriate option to fill in the blank number 2 (a) clumsy (b) capable (c) clever (d) courteous SSC CHSL (Tier-I) –08/07/2019 (Shift-III) Ans : (a) For the above blank space (2) 'clumsy' will be used because it gives an appropriate meaning to the sentence. Other options give different meaning. Meaning of other options– Capable- Competent Clever- Devious Courteous- Submissive 618. Select the most appropriate option to fill in the blank number 3 (a) maintain (b) develop (c) conceal (d) consider SSC CHSL (Tier-I) –08/07/2019 (Shift-III) Ans : (b) For the above blank space (3) 'develop' will be used. Other options have different meaning. Meaning of other options – Maintain- Preserve Conceal- Wrap Consider- Ponder 619. Select the most appropriate option to fill in the blank number 4 (a) opportunities (b) dreams (c) favours (d) threats SSC CHSL (Tier-I) –08/07/2019 (Shift-III) Ans : (a) For the above blank space (4) 'opportunities' will be used. Other options have different meaning. Meaning of other options – Dreams- Reverie Favours- Bias Threats- Bluster 620. Select the most appropriate option to fill in the blank number 5 (a) consequences (b) intrusions (c) prohibitions (d) handicaps SSC CHSL (Tier-I) –08/07/2019 (Shift-III) Ans : (d) For the above blank space (5) 'handicaps' will be used because it gives an appropriate meaning to the sentence. Other options have different meaning. Consequences- Outcome Intrusions- Infraction Prohibitions- Taboo Cloze Test

Comprehension In the following passage some words have been deleted. Fill in the blanks with the help of the alternatives given. Select the most appropriate option for each blank. In july 1937, the Japanese used an incident to provoke a war. They (1)_____China and occupied a major portion (2) ____ it. Chinese and Japanese (3)____clashed in northern China throwing the entire country into (4)____Millions of people were shot (5)____ death. 621. Select the most appropriate option to fill in blank number 1 (a) extended (b) included (c) invaded (d) persueded SSC CHSL (Tier-I) –05/07/2019 (Shift-I) Ans : (c) For the above blank space (1) 'invaded' will be used because sentence is in past tense. Other options are not correct. Meaning of other options – Extended- Enhanced Included- Connected Persuaded- Fiducial 622. Select the most appropriate option to fill in blank number 2 (a) for (b) from (c) of (d) by SSC CHSL (Tier-I) –05/07/2019 (Shift-I) Ans : (c) For the above blank space (2) 'of' will be used because 'of' preposition is use with portion and after that noun/pronouns is used. Correct sentence.....A major portion of it..... 623. Select the most appropriate option to fill in blank number 3 (a) mobs (b) troops (c) crowds (d) caravans SSC CHSL (Tier-I) –05/07/2019 (Shift-I) Ans : (b) For the above blank space (3) 'troops' will be used. Other options are not correct. Meaning of other options – Mobs- Horde Crowds- Throng Caravans- Camelcade 624. Select the most appropriate option to fill in blank number 4 (a) tranquility (b) turmoil (c) peace (d) progress SSC CHSL (Tier-I) –05/07/2019 (Shift-I) Ans : (b) For the above blank space (4) 'turmoil' will be used. Other options have different meaning. Meaning of other options – Tranquility- Serenity Peace- Calm Progress - Stride

822

YCT

625. Select the most appropriate option to fill in blank number 5 (a) for (b) on (c) in (d) to SSC CHSL (Tier-I) –05/07/2019 (Shift-I) Ans : (d) For the above blank space (5) 'to' will be used because 'to' preposition is used after 'shot' which means 'fire'. Other options are not correct. Meaning of other option Shoot – from' somethings Shoot- at' somethings Shoot- to' somebody Correct sentence.....millions of people were shot to death. Comprehension In the following passage some words have been deleted. Fill in the blanks with the help of the alternatives given. Select the most appropriate option for each blank. Spain and ltaly have produced the (1)____painters and sculptors because (2)_____ fine arts appeal more to the (3)____an to the intellect. These countries have (4)____ very few men of science in comparison (5)_____ artists and poets. 626. Select the most appropriate option to fill in blank number 1 (a) more greater (b) great (c) greater (d) greatest SSC CHSL (Tier-I) –05/07/2019 (Shift-II) Ans : (d) For the above blank space (1) 'greatest' will be used because superlative degree (greatest) is used as an adjective after article 'the'. Correct sentenceSpain and Italy have produced the greatest painters... 627. Select the most appropriate option to fill in blank number 2 (a) this (b) that (c) what (d) these SSC CHSL (Tier-I) –05/07/2019 (Shift-II) Ans : (d) For the above blank space (2) 'these' will be used according to sense of the sentence. Correct sentence......and sculptors because these fine arts appeal more....... 628. Select the most appropriate option to fill in blank number 3 (a) imagination (b) creation (c) ambition (d) situation SSC CHSL (Tier-I) –05/07/2019 (Shift-II) Ans :(a) For the above blank space (3) 'imagination' will be used. Other options are not correct. Meaning of other options – Creation- Erection Ambition- Vanity Situation- State Cloze Test

629. Select the most appropriate option to fill in blank number 4 (a) produced (b) presented (c) perplexed (d) prepared SSC CHSL (Tier-I) –05/07/2019 (Shift-II) Ans : (a) For the above blank space (4) 'produced' will be used. Other options gives different meaning. Meaning of other options – Presented- Submitted Perplexed- Abashed Prepared- Vigilant 630. Select the most appropriate option to fill in blank number 5 (a) for (b) by (c) to (d) from SSC CHSL (Tier-I) –05/07/2019 (Shift-II) Ans : (c) For the above blank space (5) 'to' will be used because comparision is always followed by to/with proposition. Correct sentence.....men of science in comparison to artists and poets. Comprehension In the following passage some words have been deleted. Fill in the blanks with the help of the alternatives given. Select the most appropriate option for each blank. In one of the most high-profile localities in the capital, hundreds of (1) ___ walk across an unmanned railway track (2) ____ day to get to school. Lakshman (3) ____ his way down the rock steps (4) ____ lead to the railway track behind his settlement. He (5) ____ in the affirmative when asked if the track scares him. "I have fallen down these steps so many times during the monsoon. But these is no other way to walk to school," he says. 631. Select the most appropriate option for blank No. 1 (a) men (b) people (c) women (d) children SSC CHSL (Tier-I) –04/07/2019 (Shift-I) Ans : (d) For the above blank space (1) 'children' will be used. Other options are not correct. Correct sentence..... hundred of children walk across an........ 632. Select the most appropriate option for blank No. 2 (a) some (b) all (c) every (d) whole SSC CHSL (Tier-I) –04/07/2019 (Shift-I) Ans : (c) For the above blank space (2) 'every' will be used in sense of everyday other options give different meaning. Meaning of other options– All day- An entire day Some day- Sometime Whole day- All day Correct sentence....railway track every day to get to school.....

823

YCT

633. Select the most appropriate option for blank No. 3 (a) had made (b) makes (c) was making (d) make SSC CHSL (Tier-I) –04/07/2019 (Shift-I) Ans : (b) For the above blank space (3) 'makes' will be used because singular verb (makes) is used for singular subject (Lakshman). ....Lakshman makes his way down the....... 634. Select the most appropriate option for blank No. 4 (a) whom (b) what (c) who (d) which SSC CHSL (Tier-I) –04/07/2019 (Shift-I) Ans : (d) For the above blank space (4) 'which' will be used because according to sense of the sentence 'which' pronoun gives an appropriate meaning. Meaning of other options – Correct sentence....his way down the rocky steps which lead to the...... 635. Select the most appropriate option for blank No. 5 (a) shakes (b) salutes (c) nods (d) winks SSC CHSL (Tier-I) –04/07/2019 (Shift-I) Ans : (c) For the above blank space (5) 'nods' will be used. Other options are not correct. Shakes- Oscillate Winks- Cilium Salutes- Viva Comprehension In the following passage some words have been deleted. Fill in the blanks with the help of the alternatives given. Select the most appropriate option for each blank. "Tenzing and Hillary have done it! Mount Everest has been conquered!" This news evoked (1)______ reactions. Somehow, there was a sense of disappointment (2)____ the fact that the great mountain had been (3)______. It would have been nice to (4)_____ the Everest as an (5)_____ part of the Earth. 636. Select the most appropriate option to fill in blank number 1 (a) intruding (b) intimate (c) interesting (d) inspired SSC CHSL (Tier-I) –04/07/2019 (Shift-III) Ans : (c) For the above blank space (1) 'interesting' will be used. Other options are not correct. Meaning of other options – Intrude- Encroach Inspired- Embolden 637. Select the most appropriate option to fill in blank number 2 (a) on (b) of (c) at (d) for SSC CHSL (Tier-I) –04/07/2019 (Shift-III) Cloze Test

Ans : (c) For the above blank space (2) 'at' preposition will be used because 'at' preposition is used with disappointment. Other option are not correct. 638. Select the most appropriate option to fill in blank number 3 (a) invaded (b) coallenged (c) occupied (d) conquered SSC CHSL (Tier-I) –04/07/2019 (Shift-III) Ans : (d) For the above blank space (3) 'conquered' will be used. Other option have different meaning. Meaning of other options – Invade- Assail Challenge- Defiance 639. Select the most appropriate option to fill in blank number 4 (a) be leaving (b) left (c) had left (d) have left SSC CHSL (Tier-I) –04/07/2019 (Shift-III) Ans : (d) For the above blank space (4) 'have left' will be used because sentence is in present perfect tense. Correct sentence : It would have been nice to have left the Everest as an unassailable part of Earth. 640. Select the most appropriate option to fill in blank number 5 (a) unavailed (b) unusual (c) unassailable (d) unruly SSC CHSL (Tier-I) –04/07/2019 (Shift-III) Ans : (c) For the above blank space (5) 'unassailable' will be used. Other options give different meaning. Meaning of other options – Unusual- Abnormal Unruly- Rampant Comprehension In the following passage some words have been deleted. Fill in the blanks with the help of the alternatives given. Select the most appropriate option for each blank. People make plastic products by mixing the plastic with additives. The additives (1) –––––– it difficult to recycle the plastic. To recycle plastic things, (2) ––––– grind them up first, but this new (3) ––––– is not easy to use again. A laboratory in California made a new type (4) –––––– plastic to deal with this problem. (5) –– –––– scientists put the new plastic in a highacid solution, the plastic and the additives break apart. People can then easily reuse the plastic. 641. Select the most appropriate option for blank No. 1. (a) make (b) makes (c) is making (d) was making SSC CHSL (Tier-I) –03/07/2019 (Shift-II) Ans. (a) : For the above blank space (1) 'make' will be used because subject is plural and sentence is present tense. Hence verb is used in first form without (s/es). Correct sentence : The additives make it difficult to recycle the plastic.

824

YCT

642. Select the most appropriate option for blank No. 2. (a) persons (b) personnel (c) human beings (d) people SSC CHSL (Tier-I) –03/07/2019 (Shift-II) Ans. (d) : For the above blank space (2) 'people' will be used because people is used as a subject. Correct sentence : To recycle plastic things, people grind them up first. 643. Select the most appropriate option for blank No. 3. (a) component (b) material (c) thing (d) object SSC CHSL (Tier-I) –03/07/2019 (Shift-II) Ans. (b) : For the above blank space (3) 'material' will be used. Other options are not correct. Correct Sentence : But this new material is not easy to use again. 644. Select the most appropriate option for blank No. 4. (a) to (b) with (c) in (d) of SSC CHSL (Tier-I) –03/07/2019 (Shift-II) Ans. (d) : For the above blank space (4) 'of' will be used because 'of' preposition is used with type. Note- Type of something. Correct sentence : A laboratory in California made a new type of plastic to deal with this problem. 645. Select the most appropriate option for blank No. 5. (a) But (b) Until (c) When (d) Then SSC CHSL (Tier-I) –03/07/2019 (Shift-II) Ans. (c) : For the above blank space (5) 'When' will be used. Other options are not correct. Correct sentence : When scientist put the new plastic in a high acid solution. Comprehension In the following passage some words have been deleted. Fill in the blanks with the help of the alternatives given. Select the most appropriate option for each blank. Big Ben is the UK's most iconic symbol. Since 2017, the Elizabeth Tower and the bell inside have been (1) ––––– renovation. The structure is 160 years old and (2) –––– regular maintenance. (3) –––– many years, people had been painting the hands and numbers (4) –––– the clock black, but last Thursday, the workers (5) –––– the clock's original colour, Prussian Blue! 646. Select the most appropriate option for blank No 1. (a) undermining (b) understanding (c) undergoing (d) undertaking SSC CHSL (Tier-I) –02/07/2019 (Shift-I) Cloze Test

Ans. (c) : For the above blank space (1) 'undergoing' will be used. Other options give different meaning. Meaning of other options – Undermining - Reduce Understanding- Perception Undertaking- Initiation Correct Sentence.... the bell inside have been undergoing renovation. 647. Select the most appropriate option for blank No 2. (a) need (b) was needing (c) is needed (d) has needed SSC CHSL (Tier-I) –02/07/2019 (Shift-I) Ans. (d) : For the above blank space (2) 'has needed' will be used because sentence is in present perfect tense. Its rule isSubject + has/have + verbIIIrd form + Object + Other words Correct sentence.... 160 years old and has needed regular maintenance. 648. Select the most appropriate option for blank No 3. (a) About (b) For (c) Before (d) In SSC CHSL (Tier-I) –02/07/2019 (Shift-I) Ans. (b) : For the above blank space (3) 'For' will be used because sentence is in past perfect continuous tense. Its structure isSubject + had been + verb + ing + object + since/for + time Note- 'Since' is used for period of time 'for' is used for point of time. Correct sentenceFor many years people had been painting the hands ..... 649. Select the most appropriate option for blank No 4. (a) to (b) of (c) at (d) for SSC CHSL (Tier-I) –02/07/2019 (Shift-I) Ans. (b) : For the above blank space (4) 'of' will be used because in sense of possessive case 'of' is used. Other options are not correct. Correct Sentence : .... the hands and numbers of the clock black, but .... 650. Select the most appropriate option for blank No 5. (a) imparted (b) invented (c) discovered (d) concealed SSC CHSL (Tier-I) –02/07/2019 (Shift-I) Ans. (c) : For the above blank space (5) 'discovered' will be used because it gives an appropriate meaning to the sentence. Other options are not correct. Meaning of other options – Imparted- Render Invented- Devise Concealed- Cryptic

825

YCT

Comprehension In the following passage some words have been deleted. Fill in the blanks with the help of the alternatives given. Select the most appropriate option for each blank. Trade on the New York Stock Exchange was (1)_____ to a standstill on an autumn day (2)______1994. The culprit for the outage (3)______ a squirrel that had chewed through a power line near the computer center of the (4)____ in Tumbull, Connecticut. The shutdown (5)_____ lasted for half-an-hour, resulted in the NYSE operating at 85 percent capacity. 651. Select the most appropriate option for blank No. 1 (a) worked (b) happened (c) taken (d) brought SSC CHSL (Tier-I) –02/07/2019 (Shift-II) Ans : (d) For the above blank space (1) 'brought' will be used because Passive voice always followed by was/were+v3. Other options are not correct. 652. Select the most appropriate option for blank No. 2 (a) in (b) by (c) at (d) one SSC CHSL (Tier-I) –02/07/2019 (Shift-II) Ans : (a) For the above blank space (2) preposition 'in' will be used because period of time is given. Other option are not true. 653. Select the most appropriate option for blank No. 3 (a) is (b) were (c) was (d) has SSC CHSL (Tier-I) –02/07/2019 (Shift-II) Ans : (c) For the above blank space (3) 'was' will be used because sentence is in past tense and subject is singular. 654. Select the most appropriate option for blank No. 4 (a) town (b) store (c) exchange (d) bank SSC CHSL (Tier-I) –02/07/2019 (Shift-II) Ans : (c) For the above blank space (4) 'exchange' will be used. Other options are not correct. Meaning of other options – Town- Burg Store- Shop Bank- Coast 655. Select the most appropriate option for blank No. 5 (a) which (b) what (c) who (d) whom SSC CHSL (Tier-I) –02/07/2019 (Shift-II) Ans : (a) For the above blank space (5) 'which' will be used because relative pronoun which is used for nonliving things. Correct sentence : The shutdown which lasted for half an hour, resulted in the NYSE operating at 85 percent capacity. Cloze Test

Comprehension In the following passage some words have been deleted. Fill in the blanks with the help of the alternatives given. Select the most appropriate option for each number. The small farming community of Modlareuth, located halfway between Berlin and Munich in rural Germany, is home to around 50 people and has just one pub. And yet it (1)______ tens of thousands of visitors a year. Upon arrival, (2)______ reason for this is clear. Among the 18 houses and farms (3)______ a 100m-long strip of concrete white wall, (4)______ was once part of the former border between East and West Germany meaning that one (5)______. 656. Select the most appropriate option to fill in the blank no. 1. (a) is attracted (b) attract (c) will attract (d) attracts SSC CPO-SI – 25/11/2020 (Shift-I) Ans. (d) : For the above blank space (1) 'attracts' will be used because subject 'it' is singular. Hence singular verb is used. 657. Select the most appropriate option to fill in the blank no. 2. (a) any (b) one (c) a (d) the SSC CPO-SI – 25/11/2020 (Shift-I) Ans. (d) : For the above blank space (2) 'the' will be used because 'reason' is used in definite sense in the sentence. Hence definite article 'the' will be used. 658. Select the most appropriate option to fill in the blank no. 3. (a) are (b) were (c) is (d) was SSC CPO-SI – 25/11/2020 (Shift-I) Ans. (c) : For the above blank space (3) 'is' will be used because when two plural nouns connected with 'and' and article 'the' is used before first noun then it show unit sense.Hence singular verb is used. 659. Select the most appropriate option to fill in the blank no. 4. (a) this (b) whom (c) which (d) who SSC CPO-SI – 25/11/2020 (Shift-I) Ans. (c) : For the above blank space (4) 'which' will be used because Relative pronoun 'which' is used for nonliving things. Other options are not correct. 660. Select the most appropriate option to fill in the blank no. 5. (a) part (b) piece (c) branch (d) component SSC CPO-SI – 25/11/2020 (Shift-I) Ans. (a) : For the above blank space (5) 'part' will be used. Other options are not correct.

826

YCT

Comprehension In the following passage some words have been deleted. Fill in the blanks with the help of the alternatives given. Select the most appropriate option for each number. Malti had to wait long to see her son, Abhik. She had worked hard to (1)______ her husband’s last wish that Abhik should (2)______ his higher education at Cambridge. Abhik (3)______ to be meritorious and hardworking and had secured a seat. Making ends meet was (4)______ and a trip back home in the vacation, (5)____ for the mother and son. 661. Select the most appropriate option to fill in blank no. 1. (a) finish (b) perform (c) fulfil (d) produce SSC CPO-SI – 25/11/2020 (Shift-II) Ans. (c) : For the above blank space (1) 'fulfil' will be used. Other options have different meaning. Meaning of other options – Finish - Fulfill Perform – Execution Produce - Output 662. Select the most appropriate option to fill in blank no. 2. (a) approach (b) pursue (c) delete (d) provide SSC CPO-SI – 25/11/2020 (Shift-II) Ans. (b) : For the above blank space (2) 'pursue' will be used. Other options are not correct. Meaning of other options – Approach - Outlook Delete – Remove Provide - Endow 663. Select the most appropriate option to fill in blank no. 3. (a) prove (b) had proved (c) will prove (d) is proving SSC CPO-SI – 25/11/2020 (Shift-II) Ans. (b) : For the above blank space (3) 'had proved' will be used because sentence is in past perfect tense. 664. Select the most appropriate option to fill in blank no. 4. (a) severe (b) difficult (c) inspiring (d) worthy SSC CPO-SI – 25/11/2020 (Shift-II) Ans. (b) : For the above blank space (4) 'difficult' will be used. Other options are not correct. Meaning of other options – Severe - Dauce Inspiring - Prompt Worthy – Competent 665. Select the most appropriate option to fill in blank no. 5. (a) uncomfortable (b) unbearable (c) unaffordable (d) uncouth SSC CPO-SI – 25/11/2020 (Shift-II) Cloze Test

Ans. (c) : For the above blank space (5) 'unaffordable' will be used. Other options given different meaning. Uncomfortable – Inconvenient Unbearable – Unmanageable Uncouth – Comical Comprehension In the following passage some words have been deleted. Fill in the blanks with the help of the alternatives given. Select the most appropriate option for each number. The Amazon is the world’s largest tropical rainforest. It is (1)______ the size of the continent of Australia and covers (2)______ area of nearly 2.8 million square miles. The Amazon rainforest gets its life from the (3)______ Amazon River which runs through the heart (4)______ the region. The rainforest is simply the drained basin for the river and its (5)______. The vast forest consists of four layers, each featuring its own ecosystems and specially adapted plants and animals. 666. Select the most appropriate option to fill in blank no. 1. (a) roughly (b) specially (c) similarly (d) totally SSC CPO-SI – 23/11/2020 (Shift-II) Ans. (a) : For the above blank space (1) 'roughly' (adverb) will be used. Other options give different meaning. Specially - Mainly Similarly - Alike Totally – Altogether 667. Select the most appropriate option to fill in blank no. 2. (a) some (b) an (c) a (d) one SSC CPO-SI – 23/11/2020 (Shift-II) Ans. (b) : For the above blank space (2) 'an' will be used because indefinte article 'an' is used before which pronounciation is vowel sound. 668. Select the most appropriate option to fill in blank no. 3. (a) major (b) magnetic (c) mejor (d) majestic SSC CPO-SI – 23/11/2020 (Shift-II) Ans. (d) : For the above blank space (3) 'Majestic' will be used because it gives an appropriate meaning to the sentence. 669. Select the most appropriate option to fill in blank no. 4. (a) on (b) in (c) of (d) at SSC CPO-SI – 23/11/2020 (Shift-II) Ans. (c) : For the above blank space (4) 'of' will be used because to show possession preposition 'of' is used.

827

YCT

670. Select the most appropriate option to fill in blank no. 5. (a) tributaries (b) continents (c) tributes (d) rainforests SSC CPO-SI – 23/11/2020 (Shift-II) Ans. (a) : For the above blank space (5) 'tributaries' will be used. Other options have different meaning. Continents – Mainland Tributes – Pewter Rainforests – Tropical woods Comprehension In the following passage some words have been deleted. Fill in the blanks with the help of the alternatives given. Select the most appropriate option for each number. Alfred Hitchcock was a man with vivid imagination, (1)______ skills and a passion for life. With his (2)______ style and God-gifted wit he produced and directed (3)______ of the most thrilling films that had the audience swooning (4)______ fright and falling off their seats with laughter. He was greatly (5)______ by American films and magazines. 671. Select the most appropriate option to fill in blank no. 1. (a) creative (b) clever (c) leading (d) original SSC CPO-SI – 24/11/2020 (Shift-II) Ans. (a) : For the above blank space (1) 'crative' will be used. Other options are not correct. Clever – Crafty Leading – Salient Original – Prime 672. Select the most appropriate option to fill in blank no. 2. (a) separate (b) dull (c) unique (d) ordinary SSC CPO-SI – 24/11/2020 (Shift-II) Ans. (c) : For the above blank space (2) 'unique' will be used according to sense of the sentence. Other option have different meaning. Separate – Distinct Dull - Floppy Ordinary - Mediocre 673. Select the most appropriate option to fill in blank no. 3. (a) any (b) some (c) much (d) more SSC CPO-SI – 24/11/2020 (Shift-II) Ans. (b) : For the above blank space (3) 'some' will be used. Other options are not correct. 674. Select the most appropriate option to fill in blank no. 4. (a) by (b) through (c) of (d) with SSC CPO-SI – 24/11/2020 (Shift-II) Cloze Test

Ans. (d) : For the above blank space (4) 'with' preposition will be used. Note-'with' to explain where things are. Meaning of other options – Through – Process of something Of – Quantify a time or measurement By – Means of something 675. Select the most appropriate option to fill in blank no. 5. (a) influenced (b) determined (c) attached (d) altered SSC CPO-SI – 24/11/2020 (Shift-II) Ans. (a) : For the above blank space (5) 'influenced' will be used. Other options have different meaning. Attached – Engaged Altered – Changed Comprehension In the following passage some words have been deleted. Fill in the blanks with the help of the alternatives given. Select the most appropriate option for each blank. Smart phones have become an essential part of our lives and their (1) _______ usage is now impacting human behaviour. We need to (2) ______ think about digital detox! Yes the concept of disconnecting with (3) ______ virtual world in not just a fad. It's the need of the hour today. The Global Brand Director of the smart phone Vivo in his speech nudged all smart phone users (4) _______ a step backward and think. He said, "He said, "Let's (5) ______ our mobiles and pledge to be closer and nearer to our loved ones." 676. Select the most appropriate option for blank number 1. (a) exceedingly (b) excessive (c) exceeding (d) exceed SSC CHSL (Tier-I) 12/10/2020 (Shift-I) Ans. (b) : For the above blank space (1) 'excessive' will be used. Other options are not correct. 677. Select the most appropriate option for blank number 2. (a) serious (b) seriously (c) series (d) seriousness SSC CHSL (Tier-I) 12/10/2020 (Shift-I) Ans. (b) : For the above blank space (2) 'seriously' will be used because here need an adverb. Other options are different from part of speech. 678. Select the most appropriate option for blank number 3. (a) the (b) any (c) many (d) a SSC CHSL (Tier-I) 12/10/2020 (Shift-I) Ans. (a) : For the above blank space (3) 'the' will be used because 'virtual word' is a definite noun hence definite article 'the' will be used.

828

YCT

679. Select the most appropriate option for blank number 4. (a) takes (b) to take (c) taken (d) to taking SSC CHSL (Tier-I) 12/10/2020 (Shift-I) Ans. (b) : For the above blank space (4) 'to take' will be used because here need a noun form, hence (to+v1) will be used. 680. Select the most appropriate option for blank number 5. (a) switch in (b) switch off (c) switch over (d) switch up SSC CHSL (Tier-I) 12/10/2020 (Shift-I) Ans. (b) : For the above blank space (5) 'witch off' will be used because it gives suitable meaning. Other options are not correct. Correct sentence 'Let's switch off our mobiles' Comprehension In the following passage some words have been deleted. Fill in the blanks with the help of the alternatives given. Select the most appropriate option for each blank. We do not want to condemn the word. It is after all a mighty instrument, it is the means by (1) _______ we tell each other of our emotions the way in which we influence others. Words can do (2) ______ good but also cause terrible injuries. It is true that at first there was the deed, the word came later. It was in some respects a cultural progress (3) _______ the deed became word. But the word was (4)_______ a spell, a magical act, and it (5) _______ much of its power. 681. Select the most appropriate options for blank No. 1. (a) that (b) whose (c) what (d) which SSC CHSL (Tier-I) 12/10/2020 (Shift-III) Ans. (d) : For the above blank space (1) 'which' (Relative pronoun) will be used which is used as a conjunction in the sentence. Other option are not correct. Thus the correct answer is option (d). 682. Select the most appropriate options for blank No. 2. (a) miserable (b) immeasurable (c) numerable (d) countable SSC CHSL (Tier-I) 12/10/2020 (Shift-III) Ans. (b) : For the above blank space (2) 'immeasurable' will be used because here good is acting as an uncountable noun and before it an adjective expressing unaccountability is needed. Other options areCountable - Numerable Numerable - Countable Miserable - Destitute Cloze Test

683. Select the most appropriate options for blank No. 3. (a) such (b) however (c) which (d) when SSC CHSL (Tier-I) 12/10/2020 (Shift-III) Ans. (d) : For the above blank space (3) 'when' (conjunction) will be used because it gives suitable meaning. Other options give diefferet meaning. Thus, the correct answer is option (d) 684. Select the most appropriate options for blank No. 4. (a) quickly (b) lastly (c) originally (d) firstly SSC CHSL (Tier-I) 12/10/2020 (Shift-III) Ans. (c) : For the above blank space (4) 'originally' (Adverb) will be used. Other options give different meaning. Other options areQuickly means Hastily Lastly means Eventually Firstly means At first 685. Select the most appropriate options for blank No. 5. (a) have retained (b) has retained (c) retained (d) will retained SSC CHSL (Tier-I) 12/10/2020 (Shift-III) Ans. (b) : For the above blank space (5) 'has retained' will be used because subject 'it' is singular. Hence singular helping verb 'has' will be used. Comprehension In the following passage some words have been deleted. Fill in the blanks with the help of the alternatives given. Select the most appropriate option for each blank. Hampi is a UNESCO World Heritage Site in India located near Hospet town in the Karnatake state, India. (1) ______ the attractions of Hampi (2) _______ beautiful temples, ruins of palaces (3) ________ of aquatic structures, ancient market streets, royal pavilions, bastions, royal platforms, treasury buildings ..... the list is practically (4) ______. Most of the visitors of Hampi arrive at the well-connected Hospet Junction railway station, (5) _______ about 13km from Hampi. Roads to Hampi are in great shape. Driving all the way to Hampi from the neighbouring cities is not a bad idea either. 686. Select the most appropriate option to fill in blank number 1. (a) Along (b) Among (c) Aside (d) Away SSC CHSL (Tier-I) 13/10/2020 (Shift-II) Ans. (b) : For the above blank space (1) 'among' will be used because plural number noun is used. Thus, the correct answer is option (b)

829

YCT

687. Select the most appropriate option to fill in blank number 2. (a) was (b) is (c) were (d) are SSC CHSL (Tier-I) 13/10/2020 (Shift-II) Ans. (d) : For the above blank space (2) 'are' will be used because sentence is in present tense and the subject 'the attractions of Hampi' of the sentence is in plural. Hence verb also plural. 688. Select the most appropriate option to fill in blank number 3. (a) revenues (b) remains (c) returns (d) refunds SSC CHSL (Tier-I) 13/10/2020 (Shift-II) Ans. (b) : For the above blank space (3) 'remains' will be used because it gives an appropriate meaning. Other options are not correct. Other options areRevenues means Finances Returns - means Back Refunds - means Repay 689. Select the most appropriate option to fill in blank number 4. (a) finite (b) written (c) endless (d) finished SSC CHSL (Tier-I) 13/10/2020 (Shift-II) Ans. (c) : For the above blank space (4) 'endless' will be used. Other options are not correct. Other options areFinite means limited Written means blake and white Finished means Ended 690. Select the most appropriate option to fill in blank number 5. (a) located (b) fitted (c) settled (d) placed SSC CHSL (Tier-I) 13/10/2020 (Shift-II) Ans. (a) : For the above blank space (5) 'located' will be used. Other options are not correct. Other options areFitted means appropriate Settled means stable Placed means space Comprehension In the following passage, some words have been deleted. Fill in the blanks with the help of the alternatives given. Select the most appropriate option for each blank. Roman civilization emerged around the sixth century BC. At the height of (1) ______ power, the Roman Empire ruled over a huge chunk of (2) ______, and all the present-day Mediterranean countries (3) ______ part of ancient Rome. Early Rome was (4) ______ by kings, but later, the people took control over their own city and ruled (5) ______.They introduced a council known as the Senate which ruled over them. From this point, Rome was referred to as the Roman Republic. Cloze Test

691. Select the most appropriate option for blank number 1. (a) their (b) its (c) his (d) one's SSC CHSL (Tier-I) 19/10/2020 (Shift-II) Ans. (b) : For the above blank space (1) 'its' will be used because its use for Roman Civilization. Other options have different meaning. 692. Select the most appropriate option for blank number 2. (a) sea (b) islands (c) country (d) land SSC CHSL (Tier-I) 19/10/2020 (Shift-II) Ans. (d) : For the above blank space (2) 'land' will be used because it gives suitable meaning according to the passage. Other options have different meaning. 693. Select the most appropriate option for blank number 3. (a) have (b) are (c) were (d) had SSC CHSL (Tier-I) 19/10/2020 (Shift-II) Ans. (c) : For the above blank space (3) 'were' will be used because subject (countries) is plural and sentence is in past tense. 694. Select the most appropriate option for blank number 4. (a) controlled (b) regulated (c) governed (d) conducted SSC CHSL (Tier-I) 19/10/2020 (Shift-II) Ans. (c) : For the above blank space (4) 'governed' will be used. Other options have different meaning. Controlled means Disciplined Regulated means To govern Conducted means Control 695. Select the most appropriate option for blank number 5. (a) themselves (b) ourselves (c) himself (d) yourself SSC CHSL (Tier-I) 19/10/2020 (Shift-II) Ans. (a) : For the above blank space (5) 'themselves' will be used 'Reflexive Pronoun' themselves is used for 'the people'. Other options are not correct. Ourselves - we Himself - he Yourself - you Comprehension In the following passage some words have been deleted. Fill in the blanks with the help of the alternatives given. Select the most appropriate option for each blank. The Ken Gharial Sanctuary is a wildlife sanctuary near Panna in Madhya Pradesh. It is part (1) ______ the gharials Conservation association where captive bred gharials (2) ______ released to help regenerate the gharial ecosystem. (3) ______ is located at the Ken River, north of Panna National Park. Here you can also (4) ______ the majestic Raneh Falls on the Ken River. The Sanctuary is (5) ______ for visitors during monsoon season.

830

YCT

696. Select the most appropriate option for blank number 1. (a) with (b) of (c) in (d) at SSC CHSL (Tier-I) 19/10/2020 (Shift-III) Ans. (b) : For the above blank space (1) 'of' preposition will be used because 'part of something' is used. Other options have different meaning. 697. Select the most appropriate option for blank number 2. (a) had (b) are (c) is (d) was SSC CHSL (Tier-I) 19/10/2020 (Shift-III) Ans. (b) : For the above blank space (2) 'are' will be used because its subject (Gharials) is plural and sentence is in present tense. Hence, plural verb will be used. 698. Select the most appropriate option for blank number 3. (a) That (b) It (c) These (d) This SSC CHSL (Tier-I) 19/10/2020 (Shift-III) Ans. (b) : For the above blank space (3) 'It' will be used because it is used for 'The ken Gharial Sanctuary' which is non-living subject and 'it' pronoun is used. 699. Select the most appropriate option for blank number 4. (a) look (b) see (c) climb (d) regard SSC CHSL (Tier-I) 19/10/2020 (Shift-III) Ans. (b) : For the above blank space (4) 'see' will be used. Other options are not suitable for passage. 700. Select the most appropriate option for blank number 5. (a) locked (b) closed (c) finished (d) banned SSC CHSL (Tier-I) 19/10/2020 (Shift-III) Ans. (b) : For the above blank space (5) 'closed' will be used. Other options are not correct. Other options areLocked means Closed Finished means Complete Banned means Restricted Comprehension In the following passage some words have been deleted. Fill in the blanks with the help of the alternatives given. Select the most appropriate option for each blank. The two men bowed to one (1) ______ and parted. Tom found himself in a difficult (2) _______. He was going to (3) _______ three duels in a (4) _______ of three hours. : "Even if I am killed, it will be a (5) _______ death," he thought. 701. Select the most appropriate option to fill in the blank no. 1 (a) another (b) each (c) other (d) self SSC CHSL (Tier-I) 20/10/2020 (Shift-III) Cloze Test

Ans. (a) : For the above blank space (1) 'another' will be used because one-another is used to give suitable meaning of the sentence. 702. Select the most appropriate option to fill in the blank no. 2 (a) station (b) situation (c) relation (d) location SSC CHSL (Tier-I) 20/10/2020 (Shift-III) Ans. (b) : For the above blank space (2) 'situation' will be used. Other options have different meaning. 703. Select the most appropriate option to fill in the blank no. 3 (a) participate (b) think (c) figure (d) fight SSC CHSL (Tier-I) 20/10/2020 (Shift-III) Ans. (d) : For the above blank space (3) 'fight' will be used. Other options are not correct. Other options areParticipate means partake Think means consider Figure means calculation 704. Select the most appropriate option to fill in the blank no. 4 (a) time (b) difference (c) place (d) matter SSC CHSL (Tier-I) 20/10/2020 (Shift-III) Ans. (d) : For the above blank space (4) 'matter' will be used which gives an appriopriate meaning according to sense of the sentence. 705. Select the most appropriate option to fill in the blank no. 5 (a) laborious (b) glorious (c) spurious (d) curious SSC CHSL (Tier-I) 20/10/2020 (Shift-III) Ans. (b) : For the above blank space (5) 'glorious' will be used. Other options have different meaning. Laborious means Studious Spurious means Fictitious Curious means Anxious Comprehension In the following passage some words have been deleted. Fill in the blanks with the help of the alternatives given. Select the most appropriate option for each blank. Jonas Karlsson and Ruth Osborne are two chefs who (1)______ about promoting zero waste in cooking, whether it is (2)______ restaurants or at home. It means using every part of a food (3)______. So for example, you needn’t peel carrots. You can (4)______ scrub off the surface lightly and cook them You can also find ways (5)______ and recycle byproducts like using coffee grounds to make biscuits.

831

YCT

706. Select the most appropriate option for blank No. 1. (a) are passionately (b) was passionate (c) are passionate (d) were passionate SSC CHSL (Tier-I) 21/10/2020 (Shift-III) Ans. (c) : For the above blank space (1) 'are passionate' will be used because when relative pronoun (who, which, that) come in a sentence, then its verb use as per precendent. so 'two chiefs' is plural and verb also use as 'plural'. Other options are not correct. 707. Select the most appropriate option for blank No. 2. (a) upon (b) in (c) into (d) on SSC CHSL (Tier-I) 21/10/2020 (Shift-III) Ans. (b) : For the above blank space (2) 'in' will be used. Which give suitable meaning to the sentence. 708. Select the most appropriate option for blank No. 3. (a) production (b) product (c) productive (d) producer SSC CHSL (Tier-I) 21/10/2020 (Shift-III) Ans. (b) : For the above blank space (3) 'product' will be used because food is used before blank space. Other options have different meaning. Production means Manufacture Productive means Fertile 709. Select the most appropriate option for blank No. 4. (a) just (b) rarely (c) seldom (d) never SSC CHSL (Tier-I) 21/10/2020 (Shift-III) Ans. (a) : For the above blank space (4) 'just' will be used. Other options have different meaning. 710. Select the most appropriate option for blank No. 5. (a) to redo (b) to reuse (c) to redone (d) to reused SSC CHSL (Tier-I) 21/10/2020 (Shift-III) Ans. (b) : For the above blank space (5) 'to reuse' will be used. Other options have different meaning. Note- After Infinitive 'to' verb will be used in basic form. Comprehension In the following passage some words have been deleted. Fill in the blank with the help of the alternatives given. Select the most appropriate option for each blank. A city, predominantly made of red sandstone, Fatehpur Sikri (1)____ founded in the 16th century by the Mughal Emperor, Akbar. It is essentially a fortified city (2)____had been the capital of Akbar (3)_____fifteen years. Now a UNESCO world heritage site (4) _____a famous tourist attraction, it is a fine (5)____of Mughal architecture. Cloze Test

711. Select the most appropriate option to fill in blank number 1. (a) is (b) was (c) had (d) were SSC CHSL (Tier-I) 13/10/2020 (Shift-III) Ans. (b) For the above blank space (1) 'was' will be used because sentence is in past tense and the subject of the sentence is singular. Since sentence is in passive hence "was + v3" will be used. Other options are not correct form of verb. 712. Select the most appropriate option to fill in blank number 2. (a) what (b) who (c) which (d) where SSC CHSL (Tier-I) 13/10/2020 (Shift-III) Ans. (c) For the above blank space (2) 'which' will be used because which is used for non-living thing as a relative pronoun in subjective case. Other options are not correct. 713. Select the most appropriate option to fill in blank number 3. (a) by (b) for (c) from (d) since SSC CHSL (Tier-I) 13/10/2020 (Shift-III) Ans. (b) For the above blank space (3) 'for' will be used. Other options have different meaning. Thus, the correct answer is option (b). 714. Select the most appropriate option to fill in blank number 4. (a) if (b) but (c) nor (d) and SSC CHSL (Tier-I) 13/10/2020 (Shift-III) Ans. (d) For the above blank space (4) 'and' will be used because connected for two clause 'and' conjunction will be used. Other options are not correct. 715. Select the most appropriate option to fill in blank number 5. (a) example (b) infrastructure (c) ruin (d) organization SSC CHSL (Tier-I) 13/10/2020 (Shift-III) Ans. (a) For the above blank space (5) 'example' will be used. Other options have different meaning. Infrastructure means Framework Ruin means Destruction Organization means Institution Comprehension In the following passage some words have been deleted. Fill in the blanks with the help of the alternatives given. Select the most appropriate option for each blank. A recent census of waterbirds conducted as a part of Asian Waterbird Census (1) ––––– an increase in their population in Najafgarh Jheel this year. The census also showed a (2) ––––– in the number of threatened species. Last year, the number of waterbird species were thirtyone which climbed to fifty-four this year, while

832

YCT

the threatened variety (3) –––– from four to six in the same duration. The region also saw a healthy monsoon in 2019, which would have (4) –––––– impacted the quality of habitat, making it (5) –––– to more birds. 716. Select the most appropriate option for blank number 1. (a) reaped (b) played (c) recorded (d) responded SSC CHSL (Tier-I) 15/10/2020 (Shift-III) Ans. (c) : For the above blank space (1) 'recorded' will be used. Other options have different meaning. Thus, the correct answer is option (c). 717. Select the most appropriate option for blank number 2. (a) growth (b) slowdown (c) wealth (d) health SSC CHSL (Tier-I) 15/10/2020 (Shift-III) Ans. (a) : For the above blank space (2) 'growth' will be used. Other options are not correct. Other words areslowdown means Decelecration health means Well being 718. Select the most appropriate option for blank number 3. (a) rose (b) is risen (c) rises (d) had risen SSC CHSL (Tier-I) 15/10/2020 (Shift-III) Ans. (a) : For the above blank space (3) 'Rose' (V2) will be used. Other options have different meaning. Thus, the correct answer is option (b). 719. Select the most appropriate option for blank number 4. (a) decently (b) deftly (c) dutily (d) directly SSC CHSL (Tier-I) 15/10/2020 (Shift-III) Ans. (d) : For the above blank space (4) 'direrctly' will be used because we used 'Adverb Directly' to modify verb. Other options are not correct. Other words areDecently means correctly Deftly means skillfully Dutily means Devoted 720. Select the most appropriate option for blank number 5. (a) unattractive (b) possible (c) attractive (d) impossible SSC CHSL (Tier-I) 15/10/2020 (Shift-III) Ans. (c) : For the above blank space (5) 'attractive' will be used. Other options have different meaning. Other options areunattractive means Uncharming possible means Eventual impossible means Improbable Cloze Test

Comprehension In the following passage some words have been deleted. Fill in the blanks with the help of the alternatives given. Select the most appropriate option for each blank. The choices we make in life determine who we are. One's life is (1) ______ marked by the exemplary choices one has made (2) ______ one's journey. The mega icon Ratan Tata has described his journey (3) ______ being a sevenyear-old boy who played his (4) ______ the freedom struggle to being moved by the plight of a family on a two-wheeler which became the genesis of the world's most (5) ______ car the Tata Nano. 721. Select the most appropriate option for blank number 1. (a) rarely (b) often (c) seldom (d) never SSC CHSL (Tier-I) 12/10/2020 (Shift-II) Ans. (b) : For the above blank space (1) 'often' will be used. Other options are not correct. 722. Select the most appropriate option for blank number 2. (a) upon (b) toward (c) into (d) along SSC CHSL (Tier-I) 12/10/2020 (Shift-II) Ans. (d) : For the above blank space (2) 'along' will be used. Other options are not correct. 723. Select the most appropriate option for blank number 3. (a) from (b) above (c) though (d) for SSC CHSL (Tier-I) 12/10/2020 (Shift-II) Ans. (a) : For the above blank space (3) 'from' (preposition) will be used because it gives suitable meaning according to the sentence. 724. Select the most appropriate option for blank number 4. (a) part with (b) part of (c) part in (d) part from SSC CHSL (Tier-I) 12/10/2020 (Shift-II) Ans. (c) : For the above blank space (4) 'part in' will be used because it is a phrase which meaning "Play part in something". Other options have different meaning. Other words arePart with means To give up control Part of means Element Part from means Abandon 725. Select the most appropriate option for blank number 5. (a) economical (b) economize (c) economist (d) economic SSC CHSL (Tier-I) 12/10/2020 (Shift-II) Ans. (a) : For the above blank space (5) 'economical' will be used because it gives an appropriate meaning according to the sentence. Other options have different meaning.

833

YCT

Comprehension Ans. (d) : For the above blank space (5) 'Dialogues' In the following passage some words have been will be used which means conversation. It gives deleted. Fill in the blanks with the help of the appropriate meaning as per passage. alternatives given. select the most appropriate Other options areoption for each blank. Monologues means A long speech by one person One day Willy stumbled upon a novel in a Prologue means Introduction (1)___bookshop–a Perry Mason book written epilogue means Postscript. by Erle Stanley Gardner. He (2)___the book Thus, the correct answer is option (d). simply because the title of the book cover Comprehension looked comprehension. Although it was an In the following passage some words have been unbridged book willy was able (3)______ it was deleted. Fill in the blanks with the help of the sufficient (4)______ comprehension. This was alternatives given. partly because the novel contained(5) ____ Studies have shown that there is no standard written in simple, conversational language. advice one can (1) ____to parents. Some 726. Select the most appropriate option for blank guidelines however, may (2)_____The first one No.1. is that children are all different and there is no (a) local (b) locals one way (3)____bringing them up. Each child (c) location (d) localize should be treated as (4)____individual The SSC CHSL (Tier-I) 21/10/2020 (Shift-I) second is that heredity is important, both for intelligence and (5)_____. Ans. (a) : For the above blank space (1) 'local' will be used because it gives appropriate meaning. Thus, the 731. Select the most appropriate option for blank No.1. correct answer is option (a). (a) giving (b) give 727. Select the most appropriate option for blank (c) given (d) gave No.2. SSC CHSL (Tier-I) 26/10/2020 (Shift-II) (a) picked on (b) picked at Ans. (b) : For the above blank space (1) 'give' will be (c) picked in (d) picked up SSC CHSL (Tier-I) 21/10/2020 (Shift-I) used because modal verb always take first form of verb can+v1 . Ans. (d) : For the above blank space (2) 'picked up' One can give to parents. will be used which means 'To improve'. Thus, the correct answer is option (b). Other option are732. Select the most appropriate option for blank Picked on means To Criticize No.2. Picked at means To pull or touch something several (a) was provided (b) be provided times. (c) is provide (d) being provided Picked in means To Choose somebody. SSC CHSL (Tier-I) 26/10/2020 (Shift-II) Thus, the correct answer is option (d). 728. Select the most appropriate option for blank Ans. (b) : For the above blank space (2) 'be provided' will be used because the sentence in passive voice. No.3. Modal + be + v3 . (a) attractive (b) attracted Thus, the correct answer is option (b). (c) attract (d) attraction SSC CHSL (Tier-I) 21/10/2020 (Shift-I) 733. Select the most appropriate option for blank No.3. Ans. (a) : For the above blank space (3) 'attractive' will (a) by (b) of be used because it is 'adjective' which qulify book (c) from (d) off cover. SSC CHSL (Tier-I) 26/10/2020 (Shift-II) Thus, the correct answer is option (a). Ans. (b) : For the above blank space (3) preposition 729. Select the most appropriate option for blank 'of' will be used because it gives correct meaning of the No. 4. sentence. (a) reading (b) to reading The correct sentence is(c) to read (d) read SSC CHSL (Tier-I) 21/10/2020 (Shift-I) 'There is no one way of bringing them up.' Ans. (c) : For the above blank space (4) 'To read' will 734. Select the most appropriate option for blank No.4. be used because to+verb1 is used as a 'Noun' which is (a) an (b) many suitable for blank space. (c) a (d) that Thus, the correct answer is option (c). SSC CHSL (Tier-I) 26/10/2020 (Shift-II) 730. Select the most appropriate option for blank Ans. (a) : For the above blank space (4) article 'an' will No. 5. be used because here 'individual' indicates to a vowel (a) monologues (b) prologue sound. (c) epilogue (d) dialogues SSC CHSL (Tier-I) 21/10/2020 (Shift-I) Thus, the correct answer is option (a). Cloze Test

834

YCT

735. Select the most appropriate option for blank No.5. (a) personal (b) personnel (c) personality (d) personalise SSC CHSL (Tier-I) 26/10/2020 (Shift-II) Ans. (c) : For the above blank space (5) 'personality' will be used because the word 'and' indicates that intelligence is a noun then after 'and' word also being noun so 'personality' is used. Thus, the correct answer is option (c). Comprehension In the following passage some words have been deleted. Fill in the blanks with the help of the alternatives given. Select the most appropriate option for each blank. The Great Stupa at Sanchi is one of the oldest stone structures in India. It was originally commissioned (1) ________ Emperor Ashoka in the 3rd centure BCE. Its nucleus (2) _______ a simple hemispherical brick structure built (3) _______ the relics of the Buddha. It was crowned by the chhatri, a parasol-like (4) _______ symbolising high rank, which was (5) ________ to honour and shelter the relics. 736. Select the most appropriate option to fill in blank No. 1. (a) with (b) to (c) by (d) from SSC CHSL (Tier-I) 18/03/2020 (Shift-II) Ans. (c) : For the above blank space (1) preposition 'by' will be used because the sentence is in passive voice. Thus, the correct answer is option (c). 737. Select the most appropriate option to fill in blank No. 2. (a) gave (b) was (c) were (d) kept SSC CHSL (Tier-I) 18/03/2020 (Shift-II) Ans. (b) : For the above blank space (2) 'was' will be used because the whole passage in past tense. Thus, the correct answer is option (b). 738. Select the most appropriate option to fill in blank No. 3. (a) into (b) aside (c) over (d) about SSC CHSL (Tier-I) 18/03/2020 (Shift-II) Ans. (c) : According to the passage the most appropriate option for the blank space (3) 'over' will be used because it gives correct meaning. 739. Select the most appropriate option to fill in blank No. 4. (a) conduit (b) wall (c) structure (d) bridge SSC CHSL (Tier-I) 18/03/2020 (Shift-II) Ans. (c) : According to the passage the most appropriate option for the blank space (4) 'structure' will be used. Cloze Test

Other words areConduit means Tube Wall means Enclosure Bridge means Sea wall 740. Select the most appropriate option to fill in blank No. 5. (a) intended (b) outlined (c) informed (d) predicted SSC CHSL (Tier-I) 18/03/2020 (Shift-II) Ans. (a) : For the above blank space (5) 'intended' will be used because it gives correct meaning. Other words areOutlined means Stimulate Informed means Knowledgeable Predicted means Forecasting Thus, the correct answer is option (a) Comprehension In the following passage some words have been deleted. Fill in the blanks with the help of the alternatives given. Select the most appropriate option for each blank. Everyone wants to be happy. I also want to be happy (1) _________ that's when I am at my best. I know that happiness comes from (2) _______ me, not from outside. The (3) _______ tools I will have to learn to use here will only work if I understand that I'm in (4) _______ of my thoughts, actions and emotions and if I'am willing to do some simple exercises on a (5) _______ basis. 741. Select the most appropriate option for blank No. 1 (a) until (b) nevertheless (c) unless (d) because SSC CHSL (Tier-I) 18/03/2020 (Shift-III) Ans. (d) : For the above blank space (1) 'because' will be used because it shows reason in sentence. Thus, the correct answer is option (d). 742. Select the most appropriate option for blank No. 2. (a) around (b) for (c) within (d) on SSC CHSL (Tier-I) 18/03/2020 (Shift-III) Ans. (c) : For the above blank space (2) 'within' will be used because it gives appropriate meaning. While other option is incorrect. 743. Select the most appropriate option for blank No. 3. (a) affective (b) powerful (c) effortless (d) stable SSC CHSL (Tier-I) 18/03/2020 (Shift-III) Ans. (b) : According to the passage the most appropriate option for the blank space (3) 'powerful' because adjective 'powerful' qualify tools. Thus, the correct answer is option (b).

835

YCT

744. Select the most appropriate option for blank No. 4. (a) custody (b) charge (c) structure (d) frame SSC CHSL (Tier-I) 18/03/2020 (Shift-III) Ans. (b) : For the above blank space (4) 'Charge' will be used because 'in charge of' is phrase which means 'having control of or responsibility for something'. Other words areCustody means Observation Structure means Fabrication Frame means Draft Thus, the correct answer is option (b). 745. Select the most appropriate option for blank No. 5. (a) regular (b) stimulant (c) legal (d) temporary SSC CHSL (Tier-I) 18/03/2020 (Shift-III) Ans. (a) : For the above blank space (5) 'regular' will be used because it gives the correct meaning. Other options areStimulate means encourage Legal means Legitimate Temporary means Changeable Comprehension In the following passage some words have been deleted. Fill in the blanks with the help of the alternatives given. Select the most appropriate option for each blank. Families, particularly those with children and who have the means, are opting to (1) _______ from Delhi driven by the increasing air pollution and (2) _______ impact on health. Delhi's Air Quality Index had (3) ________ a three-year-low this year, post-Diwali, creating a (4) _______ polluted atmosphere which led to (5) _______ problems in many, forcing people to wear masks outdoors and install air purifiers at home. 746. Select the most appropriate option for blank No. 1. (a) relocate (b) reinstate (c) reschedule (d) restart SSC CHSL (Tier-I) 19/03/2020 (Shift-I) Ans. (a) : According to the passage the most appropriate option for the blank space (1) 'Relocate' will be used. Other words areReinstate means Redintegrate Reschedule means Adjourn Restart means Proceed Thus, the correct answer is option (a). 747. Select the most appropriate option for blank No. 2. (a) their (b) its (c) it's (d) their's SSC CHSL (Tier-I) 19/03/2020 (Shift-I) Cloze Test

Ans. (b) : For the above blank space (2) 'its' will be used because 'its' as a passessive adjective. It is qualifying the noun (impost) Thus, the correct answer is option (b). 748. Select the most appropriate option for blank No. 3. (a) hit (b) bagged (c) produced (d) leaped SSC CHSL (Tier-I) 19/03/2020 (Shift-I) Ans. (a) : For the above blank space (3) 'hit' will be used because the verb 'had' represents as past perfect tense. It structure is Sub. + had + v3 + obj + other . So, here 'hit' is used as 'V3' because all form of 'hit' is same. Thus, the correct answer is option (a). 749. Select the most appropriate option for blank No. 4. (a) separate (b) severely (c) conditionally (d) dissatisfied SSC CHSL (Tier-I) 19/03/2020 (Shift-I) Ans. (b) : For the above blank space (4) 'severely' will be used. Which means 'strongly'. Other options areseparate means Detached, Distinct conditionally means Tentatively dissatisfied means Malcontent, disappointed 750. Select the most appropriate option for blank No. 5. (a) abdominal (b) brain (c) renal (d) respiratory SSC CHSL (Tier-I) 19/03/2020 (Shift-I) Ans. (d) : For the above blank space (5) 'respiratory' will be used because it gives appropriate sense of the passage. The correct sentence is'Which led to respiratory' problems. Meaning of other words areAbdominal means Celiac Brain means Intellect Renal means Nephritic Thus, the correct answer is option (d) Comprehension In the following passage some words have been deleted. Fill in the blanks with the help of the alteranatives given. Select the most appropriate option for each blank. The flight was packed. There were (1) _____parents with small children. Specifically. a three-year-old boy (2)_____the row in front of me (3)_____announcing his presence by letting (4)____ear-piercing shrieks every 45 seconds. I settled (5)____hoping the kid would calm down. 751. Select the most appropriate option to fill in blank No.1. (a) any (b) all (c) several (d) most SSC CHSL (Tier-I) 17/03/2020 (Shift-II)

836

YCT

Ans. (c) : According to the passage the most appropriate option for the blank space (1) 'Several' will be used as an adjective. Thus, the correct answer is option (c). 752. Select the most appropriate option to fill in blank No.2. (a) at (b) inside (c) over (d) in SSC CHSL (Tier-I) 17/03/2020 (Shift-II) Ans. (d) : For the above blank space (2) preposition 'in' will be used because it give appropriate meaning of the sentense. The correct sentence isA three year old boy in the row. 753. Select the most appropriate option to fill in blank No.3. (a) were (b) are (c) was (d) had SSC CHSL (Tier-I) 17/03/2020 (Shift-II) Ans. (c) : For the above blank space (3) 'was' will be used because subject of sentence is 'A three year old boy'. The sentence is in the past continuous Tense. Sub. + was/were + v1 + ing + obj. Thus, the correct answer is option (c) 754. Select the most appropriate option to fill in blank No.4. (a) leave (b) across (c) away (d) loose SSC CHSL (Tier-I) 17/03/2020 (Shift-II) Ans. (d) : For the above blank space (4) 'loose' will be used because it gives correct sense as per passage. Thus, the correct answer is option (d) 755. Select the most appropriate option to fill in blank No.5. (a) up (b) on (c) out (d) down SSC CHSL (Tier-I) 17/03/2020 (Shift-II) Ans. (d) : For the above blank space (5) 'down' will be used because 'settle down' is a phrase which means 'Relax'. Thus, the correct answer is option (d). Comprehension In the following passage some words have been deleted. Fill in the blanks with the help of the alternatives given. Select the most appropriate option for each blank. Art is the use of (1)____skill and imagination to create something that's beautiful or emotionally (2)____.This definition needs to be expanded. If you (3)____a little deeper, You'll see that any skill can be considered an art, (4)_____if you exercise a little (5)____and creativity in the process. 756. Select the most appropriate option for blank No.1. (a) creativity (b) destruction (c) intoxication (d) emotion SSC CHSL (Tier-I) 19/03/2020 (Shift-III) Cloze Test

Ans. (a) : According to the passage the most appropriate option for the blank space (1) 'Creativity' will be used because it gives a appropriate sense of the passage. The correct sentence is'Art is the use of 'creativity' skill. 757. Select the most appropriate option for blank No. 2. (a) imaginative (b) tempting (c) creative (d) comforting SSC CHSL (Tier-I) 19/03/2020 (Shift-III) Ans. (d) : For the above blank space (2) 'comforting' will be used because it gives correct meaning of the passage. Meaning of the other options areTempting means Attractive Imaginative means Delusive Creative means Inspired Thus, the correct answer is option (d) 758 . Select the most appropriate option for blank No. 3. (a) dig (b) look (c) play (d) move SSC CHSL (Tier-I) 19/03/2020 (Shift-III) Ans. (a) : For the above blank space (3) 'dig' will be used because it gives correct meaning of the sentence. Thus, the correct answer is option (a) 759. Select the most appropriate option for blank No. 4. (a) commonly (b) at the outset (c) particularly (d) generally SSC CHSL (Tier-I) 19/03/2020 (Shift-III) Ans. (c) : For the above blank space (4) 'particularly' will be used which means 'especially'. Thus, the correct answer is option (b) 760. Select the most appropriate option for blank No. 5. (a) desire (b) dream (c) hope (d) imagination SSC CHSL (Tier-I) 19/03/2020 (Shift-III) Ans. (d) : For the above blank space (5) 'imagination' will be used because when a noun joints 'and', both side are same words. The correct sentence isA little imgination and creativity in the process. Thus, the correct answer is option (d) Comprehension In the following passage some words have been deleted. Fill in the blanks with the help of the alternatives given. Select the most appropriate option for each blank. All her life Ms. Foster had a great fear of missing a train. a place, a boat or (1)____a theatre curtain. In other respects. She was not a (2)____nervous woman. But the mere thought of (3) ____on occasions like these would throw her (4)____such a nervous state that a finy muscle in the corner of her left eye would (5)_____twitch. This would disappear once she had boarded the plane etc.

837

YCT

761. Select the most appropriate option for blank No.1. (a) extremely (b) quite (c) even (d) hardly SSC CHSL (Tier-I) 21/10/2020 (Shift-II) Ans. (c) : In the blank space (1) of the above passage, the option 'even' will be used because it gives appropriate meaning as per passage. Thus, the correct answer is option (c) 762. Select the most appropriate option for blank No.2. (a) usual (b) particular (c) general (d) particularly SSC CHSL (Tier-I) 21/10/2020 (Shift-II) Ans. (b) : In the blank space (2) of the above passage, the option 'particular' will be used because it gives appropriate meaning as per passage. Thus, the correct answer is option (b) 763. Select the most appropriate option for blank No.3. (a) be later (b) be late (c) being late (d) being later SSC CHSL (Tier-I) 21/10/2020 (Shift-II) Ans. (c) : For the above blank space (3) 'being late' will be used because sentence is in Passive form. Thus, the correct answer is option (c) 764. Select the most appropriate option for blank No.4. (a) in (b) into (c) up (d) away SSC CHSL (Tier-I) 21/10/2020 (Shift-II) Ans. (b) : According to the passage the most appropriate option for the blank space (4) 'into' will be used because 'into' is used when we are discussing the destination of the object. Thus, the correct answer is option (b) 765. Select the most appropriate option for blank No.5. (a) begin over (b) begun over (c) begun to (d) begin to SSC CHSL (Tier-I) 21/10/2020 (Shift-II) Ans. (d) : In the blank space (5) of the above passage, the option 'begin to' will be used because the phrase 'begin to' means 'to start'. Thus, the correct answer is option (d). Comprehension In the following passage some words have been deleted. Fill in the blanks with the help of the alternatives given. Select the most appropriate option for each blank. To close the day on a high note, we decided to spend the rest of (1)______ evening at a 100year-old historic watering hole in downtown Juneau. We wanted to (2)______ our bear sightings and how it feels to be in the wild. While raising a toast and reminiscing (3)______ our time in the woods, I felt a deeper Cloze Test

connection with nature. It was a (4)______ experience to walk for miles with no particular reason or intention other than absorbing (5)______ solitude of the jungle, open to the sky. 766. Select the most appropriate option for blank No. 1. (a) their (b) ours (c) our (d) theirs SSC CHSL (Tier-I) 16/10/2020 (Shift-II) Ans. (c) : According to the passage the most appropriate option for the blank space (1) possessive adjective 'our' will be used because subject is 'we'. Thus, the correct answer is option (c) 767. Select the most appropriate option for blank No. 2. (a) celebrated (b) celebration (c) celebrate (d) celebrates SSC CHSL (Tier-I) 16/10/2020 (Shift-II) Ans. (c) : According to the passage the most appropriate option for the blank space (2) 'celebrate' will be used because infinitive 'to' is always take first form of verb (V1). While other options are not correct. Thus, the correct answer is option (c) 768. Select the most appropriate option for blank No. 3. (a) for (b) in (c) with (d) about SSC CHSL (Tier-I) 16/10/2020 (Shift-II) Ans. (d) : According to the passage the most appropriate option for the blank space (3) preposition 'about' will be used. The correct sentence'Reminiscing about our time in the words. Thus, the correct answer is option (d) 769. Select the most appropriate option for blank No. 4. (a) profound (b) sincere (c) comical (d) dismal SSC CHSL (Tier-I) 16/10/2020 (Shift-II) Ans. (a) : According to the passage the most appropriate option for the blank space (4) adjective 'profound' will be used because it is qualified the noun 'experience'. Other options areSincere means Scrupulous Comical means Humorous Dismal means Depressed Thus, the correct answer is option (a) 770. Select the most appropriate option for blank No. 5. (a) some (b) the (c) a (d) much SSC CHSL (Tier-I) 16/10/2020 (Shift-II) Ans. (b) : For the above blank space (5) article 'the' will be used because it comes before noun (solitude). Thus, the correct answer is option (b)

838

YCT

Comprehension In the following passage some words have been deleted. Fill in the blank with the help of the alternatives given. Select the most appropriate option for each blank. One of the most fascinating archaeological sites in Maharashtra, the Ellora Caves, date back (1)____ about 1,500 years ago, and (2)____the epitome of Indian rock-cut architecture. (3)_____34 caves are actually Buddhist, Hindu and Jain religious monuments (4) _____in the rock. They were given the (5)____of World Heritage Site in 1983. Created between the 6th and 10th century, the 12Buddhist, 17 Hindu and 5 jain caves carved in proximity at Ellora are proof of the religious harmony during this period of Indian history. 771. Select the most appropriate option to fill in blank number 1. (a) in (b) to (c) for (d) at SSC CHSL (Tier-I) 17/03/2020 (Shift-I) Ans. (b) According to the passage the most appropriate option for the blank space (1) Preposition 'to' will be used because it gives appropriate meaning of the sentence. Thus, the correct answer is option (b) 772. Select the most appropriate option to fill in blank number 2. (a) are (b) be (c) were (d) is SSC CHSL (Tier-I) 17/03/2020 (Shift-I) Ans. (a) According to the passage the most appropriate option for the blank space (2) Plural Verb 'are' will be used because subject 'The Ellora Caves' also plural. Thus, the correct answer is option (a) 773. Select the most appropriate option to fill in blank number 3. (a) Many (b) Few (c) An (d) The SSC CHSL (Tier-I) 17/03/2020 (Shift-I) Ans. (d) For the above blank space (3) article 'The' will be used because 'The Caves' Indicates The Ellora Caves. Thus, the correct answer is option (d) 774. Select the most appropriate option to fill in blank number 4. (a) created (b) constructed (c) erected (d) carved SSC CHSL (Tier-I) 17/03/2020 (Shift-I) Ans. (d) For the above blank space (4) Verb 'carved' will be used because it gives appropriate meaning of the passage. Thus, the correct answer is option (d) Cloze Test

775. Select the most appropriate option to fill in blank number 5. (a) status (b) symbol (c) function (d) role SSC CHSL (Tier-I) 17/03/2020 (Shift-I) Ans. (a) According to the passage the most appropriate option for the blank space (5) 'Status' will be used. Other option areSymbol means Emblem Function means Ceremony Role means Contribution Thus, the correct answer is option (a) Comprehension In the following passage some words have been deleted. Fill in the blanks with the help of the alternatives given. Select the most appropriate option for each blank. I remember watching a TV series on bringing up children that showed parents being obsessed (1) _______ their children's performance in school exams. The series highlighted how parents often give (2) _______ higher importance to their children's academic performance than their aptitudes. So many parents push and goad their children to study hard and score (3) _______ possible in their exams. They believe that academic (4) _______ guarantees a successful career and they want their children to be the next 'Albert Einstein' not knowing that Einstein (5) _______ was a rebel and didn't follow any of his parents' plans for him. 776. Select the most appropriate option for blank No. 1. (a) of (b) in (c) about (d) to SSC CHSL (Tier-I) 26/10/2020 (Shift-II) Ans. (c) : According to the passage the most appropriate option for the blank space (1) 'about' will be used because it gives appropriate meaning of the sentence. Thus, the correct answer is option (c) 777. Select the most appropriate option for blank No. 2. (a) completely (b) fully (c) significantly (d) largely SSC CHSL (Tier-I) 26/10/2020 (Shift-II) Ans. (c) : According to the passage the most appropriate option for the blank space (2) advarb 'Significantly' will be used because it is qualifying adjective 'higher'. Thus, the correct answer is option (c) 778. Select the most appropriate option for blank No. 3. (a) as higher as (b) as good as (c) as high as (d) as better as SSC CHSL (Tier-I) 26/10/2020 (Shift-II)

839

YCT

Ans. (c) : According to the passage the most appropriate option for the blank space (3) 'as high as' will be used. Thus, the correct answer is option (c) 779. Select the most appropriate option for blank No. 4. (a) direction (b) realization (c) performance (d) completion SSC CHSL (Tier-I) 26/10/2020 (Shift-II) Ans. (c) : According to the passage the most appropriate option for the blank space (4) 'performance' will be used. Other options aredirection means side realization means perception completion means Resignation Thus, the correct answer is option (c) 780. Select the most appropriate option for blank No. 5. (a) himself (b) oneself (c) themselves (d) itself SSC CHSL (Tier-I) 26/10/2020 (Shift-II) Ans. (a) : For the above blank space (5) Reflexive Pronoun 'himself' will be used because it is emphasis the subject 'Einstein'. Thus, the correct answer is option (a) Comprehension In the following passage some words have been deleted. Fill in the blanks with the help of the alternatives given. Select the most appropriate for each blank. Buffer stock refers to a (1) –––– of a commodity that is used to offset price fluctuations (2) –––– unforeseen emergencies. Buffer stock is generally (3) ––––– for essential commodities and necessities like food grains, pulses etc. The Cabinet Committee (4) –––– Economic Affairs (5) –––– the minimum buffer stock on a quarterly basis. 781. Select the most appropriate option for blank No. 1. (a) reservation (b) reserved (c) reserve (d) reserves SSC MTS – 06/08/2019 (Shift-II) Ans. (c) : According to the passage the most appropriate option for the blank space (1) Noun 'Reserve' will be used. Thus, the correct answer is option (c) 782. Select the most appropriate option for blank No. 2. (a) if (b) and (c) also (d) even SSC MTS – 06/08/2019 (Shift-II) Ans. (b) : According to the passage the most appropriate option for the blank space (2) Conjunction 'and' will be used because it is adding two words 'fluctuations' & 'unforeseen'. Thus, the correct answer is option (b) Cloze Test

783. Select the most appropriate option for blank No. 3. (a) maintains (b) maintaining (c) maintained (d) maintain SSC MTS – 06/08/2019 (Shift-II) Ans. (c) : According to the passage the most appropriate option for the blank space (3) past participal 'maintained' will be used. Thus, the correct answer is option (c) 784. Select the most appropriate option for blank No. 4. (a) in (b) for (c) to (d) at SSC MTS – 06/08/2019 (Shift-II) Ans. (b) : According to the passage the most appropriate option for the blank space (4) Preposition 'for' will be used because it gives correct meaning of the sense. Thus, the correct answer is option (b) 785. Select the most appropriate option for blank No. 5. (a) forces (b) guides (c) fixes (d) rules SSC MTS – 06/08/2019 (Shift-II) Ans. (c) : According to the passage the most appropriate option for the blank space (5) Singular Verb 'Fixes' will be used because 'The Cabinet Committee' is a singular (Single) unite. Thus, the correct answer is option (c) Comprehension In the following passage words have been deleted. Fill in the blanks with the help of the alternatives given. Select the most appropriate option for each blank. The Harmandir Sahib, also known as Darbar Sahib is a Gurudwara (1) –––– in the city of Amritsar. It (2) ––––– an eminent pilgrimage site (3) ––––– Sikhism. It is usually (4) –––––– the golden temple in English (5) –––– it is plated with gold. 786. Select the most appropriate option for blank No. 1. (a) locate (b) located (c) locates (d) locating SSC MTS – 07/08/2019 (Shift-II) Ans. (b) : According to the passage the most appropriate option for the blank space (1) 'located' will be used because it gives the appropriate meaning of the sentence. Thus, the correct answer is option (b) 787. Select the most appropriate option for blank No. 2. (a) is (b) was (c) has (d) are SSC MTS – 07/08/2019 (Shift-II)

840

YCT

Ans. (a) : According to the passage the most appropriate option for the blank space (2) Singular Verb 'is' will be used because it is used as a singular verb. Thus, the correct answer is option (a) 788. Select the most appropriate option for blank No. 3. (a) of (b) on (c) by (d) with SSC MTS – 07/08/2019 (Shift-II) Ans. (a) : According to the passage the most appropriate option for the blank space (3) Preposition 'of' will be used because 'site' takes preposition 'of'. Site of somthing Thus, the correct answer is option (a) 789. Select the most appropriate option for blank No. 4. (a) named (b) known (c) regarded (d) called SSC MTS – 07/08/2019 (Shift-II) Ans. (d) : According to the passage the most appropriate option for the blank space (4) verb 'called' will be used because sentence is in passive form. Thus, the correct answer is option (d) 790. Select the most appropriate option for blank No. 5. (a) even (b) also (c) but (d) because SSC MTS – 07/08/2019 (Shift-II) Ans. (d) : According to the passage the most appropriate option for the blank space (5) Conjunction 'because' will be used which shows reason for the sentence. Thus, the correct answer is option (d) Comprehension In the following passage some words have deleted. Fill in the blanks with the help of the alternatives given. Select the most appropriate option for each blank. Plants first appeared on earth 400 million years ago. Insects arrived 100 million years later. Ever (1) –––– a fierce war has raged between the two. At first it (2) –––– not seem likely that the plants would stand (3) ––––– chance in the battle. Plants, unlike insects, cannot move. Plants are (4) –––– outnumbered by insects, an average oak tree will have tens of thousands of insects (5) ––––– on it. 791. Select the most appropriate option to fill in blank No. 1. (a) for (b) on (c) since (d) afterwards SSC MTS – 08/08/2019 (Shift-II) Ans. (c) : According to the passage the most appropriate option for the blank space (1) 'since' will be used because pair words 'ever since' which means continuously since a 'specified time'. Thus, the correct answer is option (c) Cloze Test

792. Select the most appropriate option to fill in blank No. 2. (a) did (b) does (c) do (d) had SSC MTS – 08/08/2019 (Shift-II) Ans. (a) : According to the passage the most appropriate option for the blank space (2) auxiliary verb 'did' will be used because given sentence is in the past tense. Thus, the correct answer is option (a) 793. Select the most appropriate option to fill in blank No. 3. (a) none (b) any (c) some (d) not SSC MTS – 08/08/2019 (Shift-II) Ans. (b) : According to the passage the most appropriate option for the blank space (3) 'any' will be used because 'any' is used in negative sentence. Thus, the correct answer is option (b) 794. Select the most appropriate option to fill in blank No. 4. (a) firmly (b) lightly (c) vastly (d) deeply SSC MTS – 08/08/2019 (Shift-II) Ans. (c) : According to the passage the most appropriate option for the blank space (4) 'vastly' will be used because it gives appropriate meaning. Thus, the correct answer is option (c) 795. Select the most appropriate option to fill in blank No. 5. (a) feeling (b) eating (c) fighting (d) feeding SSC MTS – 08/08/2019 (Shift-II) Ans. (d) : According to the passage the most appropriate option for the blank space (5) 'feeding' will be used because 'feeding' takes preposition 'on'. Note- Feed on something. Thus, the correct answer is option (d) Comprehension In the following passage some words have been deleted. Fill in the blanks with the help of the alternatives given. Select the most appropriate option for each blank. Louis Braille was a French educator and inventor (1) –––– a system of reading and writing for the visually impaired. His system (2) ––––– virtually unchanged to date and is known worldwide simply as 'braille'. Blinded (3) ––––– both eyes as a result of an early childhood (4) ––––– Braille mastered his disability while still a boy. He (5) –––– at school received a scholarship to the Royal Institute for Blind Youth. 796. Select the most appropriate option to fill in blank no. 1. (a) from (b) of (c) by (d) in SSC MTS – 09/08/2019 (Shift-III)

841

YCT

Ans. (b) : According to the passage the most Comprehension In the following passage some words have been appropriate option for the blank space (1) Preposition deleted. Fill in the blanks with the help of the 'of' will be used because it gives appropriate meaning alternatives given. Select the most appropriate of the sentece. option for each blank. The correct sentence isRama was a naughty boy (1) ––––– lived in the 'A French educator and inventor of a system.' village of Tenali. His mother did not (2) ––––– Thus, the correct answer is option (b). what to do with him since he (3) –––– to study 797. Select the most appropriate option to fill in or do any work. One day (4) ––––took him to blank No. 2. see a Guru. She told the (5) –––– man that she (a) leaves (b) rests wanted him to take Rama under his (c) preserves (d) remains guardianship. SSC MTS – 09/08/2019 (Shift-III) 801. Select the most appropriate option to fill in Ans. (d) : According to the passage the most blank No. 1. (a) who (b) which appropriate option for the blank space (2) 'remains' (c) he (d) that will be used because 'His system' is a singular subject. SSC MTS – 13/08/2019 (Shift-I) so verb also be used as singular. Ans. (a) : According to the passage the most Thus, the correct answer is option (d). appropriate option for the blank space (1) Relative 798. Select the most appropriate option to fill in Pronoun 'who' will be used because 'Rama' indicates as blank No. 3. a person. (a) in (b) with Thus, the correct answer is option (a). (c) to (d) on 802. Select the most appropriate option to fill in SSC MTS – 09/08/2019 (Shift-III) blank No. 2. Ans. (a) : According to the passage the most (a) knowing (b) know appropriate option for the blank space (3) 'in' will be (c) knew (d) knows used because 'Blinded' takes preposition in. SSC MTS – 13/08/2019 (Shift-I) Thus, the correct answer is option (a). Ans. (b) : According to the passage the most 799. Select the most appropriate option to fill in appropriate option for the blank space (2) verb 'know' will be used because did not + v1 , here v1 (know) blank No. 4. while other options are not correct. (a) plan (b) accident Thus, the correct answer is option (b). (c) provision (d) occasion SSC MTS – 09/08/2019 (Shift-III) 803. Select the most appropriate option to fill in blank No. 3. Ans. (b) : According to the passage the most (a) wished (b) reminded appropriate option for the blank space (4) 'accident' (c) refused (d) denied will be used gives appropriate meaning of the SSC MTS – 13/08/2019 (Shift-I) sentence. Ans. (c) : According to the passage the most Meaning of the other words areappropriate option for the blank space (3) (VII) Plan means Scheme 'Refused' will be used. Provision means Supply Other option areOccasion means Opportunity wished means Pleasure 800. Select the most appropriate option to fill in remind means Reminisce blank No. 5. denied means refusable (a) excelled (b) talented Thus, the correct answer is option (c). (c) skilled (d) recorded 804. Select the most appropriate option to fill in SSC MTS – 09/08/2019 (Shift-III) blank No. 4. Ans. (a) : According to the passage the most (a) her (b) they appropriate option for the blank space (5) Verb (c) she (d) he 'excelled' will be used. SSC MTS – 13/08/2019 (Shift-I) Meaning of the other words areAns. (c) : According to the passage the most Talented means Brilliant appropriate option for the blank space (4) Pronoun Skilled means Proficient, Adept 'She' will be used because as a subject 'His mother' is Recorded means Documented used before the sentence which is femine gender. Thus, the correct answer is option (a). Thus, the correct answer is option (c). Cloze Test

842

YCT

805. Select the most appropriate option to fill in blank No. 5. (a) lonely (b) lazy (c) learned (d) lively SSC MTS – 13/08/2019 (Shift-I) Ans. (c) : According to the passage the most appropriate option for the blank space (5) 'learned' (participal) will be used. Thus, the correct answer is option (c). Comprehension In the following passage some words have been deleted. Fill in the blanks with the help of the alternatives given. Select the most appropriate option for each blank. Every Monday, on his way back from work, Bepin Choudhury would drop in at Kalicharan's bookshop in New Market to buy books. He (1)_______to buy at least five at a time to last him through the week. He lived alone, was not a good mixer, had few friends, and (2)_______like spending time in (3)______chat. Today, (4)_______Kalicharan's Bepin Babu had the feeling that someone was (5)_______him from close quarters. 806. Select the most appropriate option to fill in blank No.-1. (a) having (b) have (c) had (d) has SSC MTS – 14/08/2019 (Shift-I) Ans. (c) : According to the passage the most appropriate option for the blank space (1) 'had' will be used because it shows compulsion of the past. Hint had to + v1 Thus, the correct answer is option (c) 807. Select the most appropriate option to fill in blank No.2. (a) didn't (b) don't (c) do (d) did SSC MTS – 14/08/2019 (Shift-I) Ans. (a) : According to the passage the most appropriate option for the blank space (2) 'didn't' will be used because subject 'He' is signular form and after the subject 'didn't' words demand in negative sense. Thus, the correct answer is option (a) 808. Select the most appropriate option to fill in blank No. 3. (a) empty (b) idle (c) active (d) free SSC MTS – 14/08/2019 (Shift-I) Ans. (b) : According to the passage the most appropriate option for the blank space (3) 'idle' will be used which means 'inactive' because it gives conducive meaning of the sense. Thus, the correct answer is option (b) 809. Select the most appropriate option to fill in blank No. 4. (a) in (b) at (c) on (d) by SSC MTS – 14/08/2019 (Shift-I) Cloze Test

Ans. (b) : According to the passage the most appropriate option for the blank space (4) Preposition 'at' will be used because it gives conducive meaning. Thus, the correct answer is option (b) 810. Select the most appropriate option to fill in blank No. 5. (a) recognizing (b) chasing (c) looking (d) observing SSC MTS – 14/08/2019 (Shift-I) Ans. (d) : According to the passage the most appropriate option for the blank space (5) 'observing' will be used because it is in past continuous tense (Sub.+was/were+V+ing+Obj.) Other option areRecognize means Distinguish Chase means Pursuit Look means Stare Thus, the correct answer is option (d) Comprehension In the following passage some words have been deleted. Fill in the blanks with the help of the alternatives given. Select the most appropriate option for each blank. Father read out to Swami the news of a young boy (1) ----------- had faced a tiger while returning home (2) ----------------- the jungle path. "What do you (3) --------- to that, Swami?" he asked." I (4) ----------- he must have been a very strong and (5) ----------person, not at all a boy. How could a boy fight a tiger?" said Swami. 811. Select the most appropriate option to fill in blank No. 1 (a) which (b) whom (c) that (d) who SSC MTS – 14/08/2019 (Shift-III) Ans. (d) : According to the passage the most appropriate option for the blank space (1) 'who' would be appropriate because 'who' is a relative pronoun, used to person while 'which' is used to non living things. 812. Select the most appropriate option to fill in blank No. 2 (a) at (b) from (c) for (d) to SSC MTS – 14/08/2019 (Shift-III) Ans. (b) : According to the passage the most appropriate option for the blank space (2) 'from' would be appropriate because 'from' is a preposition, to refer to the place where someone or something starts or originates. While other options give different meaning. 813. Select the most appropriate option to fill in blank No. 3 (a) say (b) talk (c) tell (d) speak SSC MTS – 14/08/2019 (Shift-III)

843

YCT

Ans. (a) : In the above passage in blank space (3) 'Say' would be appropriate, while other options give different meaningstalk - Discourse speak - Tell, Accost 814. Select the most appropriate option to fill in blank No. 4 (a) know (b) regard (c) think (d) suggest SSC MTS – 14/08/2019 (Shift-III) Ans. (c) : According to the sense of the above passage in blank space (4) 'think' would be appropriate, Meaning of other optionsRegard - Deference, Respect Suggest - Allude, Hint 815. Select the most appropriate option to fill in blank No. 5 (a) grown over (b) grown out (c) grown in (d) grown up SSC MTS – 14/08/2019 (Shift-III) Ans. (d) : According to the sense of the above passage in blank space (5) 'grown up' would be appropriate which means- physically and mentally mature. Meaning of other optionsGrown over– greater or bigger Grown out– to allow something to get longer Grown in– advanced in growth Comprehension In the following passage some words have been deleted. Fill in the blanks with the help of the alternatives given. Select the most appropriate option for each blank. Venus is the planet closest to the Earth. It shines very (1) ––––– in the night sky. The surface temperature (2) ––––– Venus is about 400 degrees Celsius. The atmosphere (3) –––– very dense. Its surface cannot be observed directly (4) –––– the planet is permanently covered with a thick (5) –––– of clouds. Venus has active volcanoes and mountains higher than the Everest. 816. Select the most appropriate option to fill in the blank No. 1. (a) loudly (b) heavily (c) brightly (d) cheerfully SSC MTS – 16/08/2019 (Shift-III) Ans. (c) : According to the sense of the above passage in blank space (1) 'brightly' would be appropriate because adverb (brightly) gives the suitable meaning to the verb (Shines). Other options have different meanings. 817. Select the most appropriate option to fill in blank No. 2. (a) around (b) over (c) on (d) above SSC MTS – 16/08/2019 (Shift-III) Cloze Test

Ans. (c) : According to the sense of the above passage in blank space (2) 'on' would be appropriate because the preposition 'on' is refer to a position on a surface. Other options are not suitable. 818. Select the most appropriate option to fill in blank No. 3. (a) was (b) is (c) are (d) were SSC MTS – 16/08/2019 (Shift-III) Ans. (b) : In the above passage in blank space (3) 'is' would be appropriate because the given sentence is 'simple present tense' and subject 'The atmoshphere' is singular, the singular verb is used. Other options are not suitable. 819. Select the most appropriate option to fill in blank No. 4. (a) because (b) so (c) though (d) but SSC MTS – 16/08/2019 (Shift-III) Ans. (a) : According to the sense of the above passage in blank space (4) 'because' would be appropriate, because the subordinating conjunction 'because' is used for explain the reason. Other options give different meanings. 820. Select the most appropriate option to fill in blank No. 5. (a) slab (b) folds (c) layer (d) pads SSC MTS – 16/08/2019 (Shift-III) Ans. (c) : According to the sense of the above passage in blank space (5) 'layer' would be appropriate, because 'a thick layer' give a suitable meaning in this sentence. Meaning of other optionsSlab - Plaque Folds - Bottom, Turn Pads - Seat Comprehension In the following passage some words have been deleted. Fill in the blanks with the help of the alternatives given. Select the most appropriate option for each blank. Throughout the ages, birds have been a source of wonder to all who have (1) –––– their soaring flight or listened to (2) –––– sweet songs. As a group, birds are (3) –––––. They are the only creatures covered (4) ––––– feathers. This evolutionary development (5) –––––– birds from all other animals. 821. Select the most appropriate option to fill in blank No. 1. (a) claimed (b) verified (c) observed (d) supported SSC MTS – 19/08/2019 (Shift-I) Ans. (c) : According to the sense of the above passage in blank space (1) 'observed' would be appropriate, which means- 'notice or perceive something'. Meaning of other optionsClaimed - Demanded Verified - Calibrate, test Supported - Vindication

844

YCT

822. Select the most appropriate option to fill in blank No. 2. (a) them (b) its (c) their (d) they SSC MTS – 19/08/2019 (Shift-I) Ans. (c) : In the above passage in blank space (2) 'their' would be appropriate. The other options are not suitable. Note : In this passage 'They' will be used as pronoun for birds and the possessive adjective, 'their'. will be used, which is qualifying the noun 'sweet song'. 823. Select the most appropriate option to fill in blank No. 3. (a) mundane (b) unique (c) indifferent (d) ordinary SSC MTS – 19/08/2019 (Shift-I) Ans. (b) : According to the sense of the above passage in blank space (3) 'Unique' will be used, which means'Something or someone is unlike anything or anyone else.' Meaning of other optionsMundane - Sublunary Indifferent - Frigid Ordinary - Mediocre 824. Select the most appropriate option to fill in blank No. 4. (a) on (b) at (c) with (d) from SSC MTS – 19/08/2019 (Shift-I) Ans. (c) : According to the sense of the above passage in blank space (4) 'with' will be used because the preposition 'with' is used with 'cover'. Other options are not suitable. 825. Select the most appropriate option to fill in blank No. 5. (a) is separated (b) has been separated (c) separates (d) was separated SSC MTS – 19/08/2019 (Shift-I) Ans. (c) : In the above passage in blank space (5) 'separates' would be appropriate, because the last sentence of this passage in simple present tense therefore, adding s/es at the end of the verb makes it sungular verb. Comprehension In the following passage some words have been deleted. Fill in the blanks with the help of the alternatives given. Select the most appropriate option for each blank. Scarcity is not created by war (1) ––––– is a permanent characteristic of all human (2) –––– and has been faced by the whole human (3) ––– It springs from the fact that the material resources of the world are (4) –––– and that our ability to make use of those resources are even more limited (5) ––––– our ignorance. Cloze Test

826. Select the most appropriate option to fill in blank No. 1. (a) there (b) these (c) those (d) it SSC MTS – 20/08/2019 (Shift-II) Ans. (d) : In the above passage in blank space (1) 'it' would be appropriate because scarcity is a abstract noun, 'it' (pronoun) is used instead of a abstract noun. Other options give different meanings. 827. Select the most appropriate option to fill in blank No. 2. (a) behaviour (b) cities (c) nations (d) societies SSC MTS – 20/08/2019 (Shift-II) Ans. (d) : According to the sense of the above passage in blank space (2) 'societies' would be appropriate, because 'all human societies' give the proper meaning in the sentence. 828. Select the most appropriate option to fill in blank No. 3. (a) space (b) race (c) kingdom (d) beings SSC MTS – 20/08/2019 (Shift-II) Ans. (b) : According to the sense of the above passage in blank space (3) 'race' will be used because 'Human' is a certain race. Meaning of other optionsSpace - Sphere Kingdom - Realm Beings - Creature 829. Select the most appropriate option to fill in blank No. 4. (a) increased (b) limited (c) restricted (d) declined SSC MTS – 20/08/2019 (Shift-II) Ans. (b) : According to the sense of the above passage in blank space (4) 'limited' would be appropriate, because the material resources of the world are limited. Meaning of other optionsIncreased - Growth Restricted - Banned Declined - Reject 830. Select the most appropriate option to fill in blank No. 5. (a) of (b) by (c) for (d) from SSC MTS – 20/08/2019 (Shift-II) Ans. (b) : According to the sense of the above passage in blank space (5) 'by' (preposition) would be appropriate. The other options are not suitable. Comprehension In the following passage some words have been deleted. Fill in the blanks with the help of the alternatives given. Select the most appropriate option for each blank. Arabia is mostly a desert. It is (1) ––––– to walk over it during (2) –––– day. Occasionally, one

845

YCT

may (3) ––––– a fertile patch called an oasis (4) ––––– springs and tall fig and palm trees. The Arabs are nomads (5) ––– live in tents and travel in caravans. 831. Select the most appropriate option to fill in blank No. 1. (a) impossible (b) absurd (c) hopeless (d) useless SSC MTS – 20/08/2019 (Shift-III) Ans. (a) : According to the sense of the above passage in blank space (1) 'Impossible' would be appropriate. The other options are not suitable. 832. Select the most appropriate option to fill in blank No. 2. (a) one (b) the (c) a (d) an SSC MTS – 20/08/2019 (Shift-III) Ans. (b) : In the above passage in blank space (2) 'the' would be appropriate. Rule : In during, admire with parts of day/night is applied, article 'The' is used before them. 833. Select the most appropriate option to fill in the blank no. 3. (a) come across (b) come up (c) come out (d) come away SSC MTS – 20/08/2019 (Shift-III) Ans. (a) : According to the sense of the above passage in blank space (3) 'come across' would be appropriate, which means. 'to meet someone or something by chance'. The other options give different meanings. 834. Select the most appropriate option to fill in blank No. 4. (a) in (b) on (c) for (d) with SSC MTS – 20/08/2019 (Shift-III) Ans. (d) : In the above passage in blank space (4) 'with' would be appropriate, because 'with is a preposition, used in the company or presence of something. 835. Select the most appropriate option to fill in blank No. 5. (a) whom (b) who (c) what (d) which SSC MTS – 20/08/2019 (Shift-III) Ans. (b) : According to the sense of the above passage in blank space (5) 'who' would be appropriate, because 'who' is a relative pronoun, used with human beings. While other options give different meanings. Comprehension In the following passage some words have been deleted. Fill in the blanks with the help of the alternatives given. Select the most appropriate option for each blank. Bacteria are extremely (1) ––––– living things. While we measure our own sizes in inches or centimetres, bacterial size is measured in (2) –– –– one micron is a thousandth of a millimetre, a Cloze Test

pinhead is about a millimetre (3) ––––– Road shaped bacteria are usually from two to four microns (4) –––– while rounded ones are generally one micron in (5) –––– . 836. Select the most appropriate option to fill in blank No. 1. (a) small (b) dirty (c) clean (d) ugly SSC MTS – 21/08/2019 (Shift-II) Ans. (a) : According to the sense of the above passage in blank space (1) 'small' would be appropriate, because Bacteria are a very small living things, therefore 'small' is used with Bacteria. 837. Select the most appropriate option to fill in blank No.2. (a) kilometres (b) feet (c) metres (d) microns SSC MTS – 21/08/2019 (Shift-II) Ans. (d) : According to the sense of the above passage in blank space (2) 'Microns' would be appropriate, because microns is used to measure the subtle length. (1 micron = 10–6 meter) 838. Select the most appropriate option to fill in blank No. 3. (a) Aside (b) Beside (c) Beneath (d) Across SSC MTS – 21/08/2019 (Shift-II) Ans. (d) : In the above passage in blank space (3) 'across' would be appropriate, because the sense of 'one end to the other end' is significant for the length of the bacteria, 'across' (preposition) is used. 839. Select the most appropriate option to fill in blank No. 4. (a) tall (b) short (c) deep (d) long SSC MTS – 21/08/2019 (Shift-II) Ans. (d) : According to the sense of the above passage in blank space (4) 'long' would be appropriate. Note- 'Long' is generally used for horizontal measurement. 840. Select the most appropriate option to fill in blank No. 5. (a) depth (b) diameter (c) weight (d) height SSC MTS – 21/08/2019 (Shift-II) Ans. (b) : In the above passage in blank space (5) 'diameter' will be used, because the length of any rounded object., diameter is used to measure. The meaning of other options is different. Comprehension In the following passage some words have been deleted. Fill in the blanks with the help of the alternatives given. Select the most appropriate option for each blank. A recent study (1) –––– by an independent organisation reported that only 15 percent (2) – –––– college bound women would consider an

846

YCT

(3) –––– girls colleges. It is clear (4) ––––– women's colleges must become co-educational in order (5) –––– survive. 841. Select the most appropriate option to fill in blank No. 1. (a) conducted (b) surveyed (c) interested (d) happened SSC MTS – 21/08/2019 (Shift-III) Ans. (a) : According to the sense of the above passage in blank space (1) 'conducted' would be appropriate. which means 'led by a guide, managed.' Meaning of other optionsSurveyed - Recorded Happened - Become, Occur 842. Select the most appropriate option to fill in blank No. 2. (a) in (b) with (c) of (d) at SSC MTS – 21/08/2019 (Shift-III) Ans. (c) : In the above passage in blank space (2) 'of' will be used. It is a preposition that indicates that relationship between other words. 843. Select the most appropriate option to fill in blank No. 3. (a) every (b) each (c) all (d) any SSC MTS – 21/08/2019 (Shift-III) Ans. (b) : In the above passage in blank space (3) 'each' would be appropriate. Note- 'Each' is used to refer to individual member of a group. ''Every' is used to refer to whole member of a group. 844. Select the most appropriate option to fill in blank No. 4. (a) whether (b) if (c) that (d) this SSC MTS – 21/08/2019 (Shift-III) Ans. (c) : According to the sense of the above passage in blank space (4) 'that' is used because 'that' refer to a purpose. Other options give different meanings. 845. Select the most appropriate option to fill in blank No. 5. (a) in (b) to (c) with (d) of SSC MTS – 21/08/2019 (Shift-III) Ans. (b) : According to the sense of the above passage in blank space (5) 'to' would be appropriate, because 'in order' is used with 'to'. The other options give different meanings. Comprehension In the following passage some words have been deleted. Fill in the blanks with the help of the alternatives given. Select the most appropriate option for each blank. Humanity can be (1) –––– as the quality of being human; the peculiar nature (2) ––––– man, by which he is distinguished (3) –––– Cloze Test

other beings. Being human (4) ––––– not mean that an individual possesses humanity. One of the (5) –––––– outstanding examples of extraordinary humanity in a human being has been portrayed beautifully by Mother Teresa. 846. Select the most appropriate option for blank No. 1. (a) define (b) defining (c) defined (d) defines SSC MTS – 02/08/2019 (Shift-I) Ans. (c) : According to the sense of the above passage in blank space (1) 'defined' (VIII) would be appropriate, because the sentence structer is based on passive voice. Passive Voice ⇒ Can + be + VIII 847. Select the most appropriate option for blank No. 2. (a) of (b) in (c) at (d) to SSC MTS – 02/08/2019 (Shift-I) Ans. (a) : In the above passage in blank space (2) 'of' would be appropriate, because 'of' preposition is used when we want to show that people or things relate with other things or people. The other options give different meanings. 848. Select the most appropriate option for blank No. 3. (a) along (b) for (c) by (d) from SSC MTS – 02/08/2019 (Shift-I) Ans. (d) : In the above passage in blank space (3) 'from' would be appropriate, because the preposition 'from' is used with 'distinguished'. The other options give different meanings. 849. Select the most appropriate option for blank No. 4. (a) had (b) does (c) has (d) did SSC MTS – 02/08/2019 (Shift-I) Ans. (b) : According to the sense of the above passage in blank space (4) 'does' would be appropriate, because the above sentence is in a negative sentence of simple present tense, for the singular subject (being human) is used to singular verb (does). The other options are not suitable. 850. Select the most appropriate option for blank No. 5. (a) more (b) many (c) most (d) much SSC MTS – 02/08/2019 (Shift-I) Ans. (c) : In the above passage in blank space (5) the word 'most' would be appropriate, because 'most' is the superlative form of 'much' and it takes the definite article 'the' and it is used as an adjective, adverb and noun.

847

YCT

Comprehension In the following passage some words have been deleted. Fill in the blanks with the help of the alternatives given. Select the most appropriate option for each blank. Sanitation refers to public health conditions (1) ––––– to clean drinking water and adequate treatment and disposal (2) –––– human waste and sewage. (3) ––––– human contact with faces is part of sanitation. Sanitation system (4) –––– to protect human health by providing a clean environment (5) –––––will stop the transmission of disease. 851. Select the most appropriate option for blank No. 1. (a) relate (b) relation (c) related (d) relates SSC MTS – 02/08/2019 (Shift-II) Ans. (c) : According to the sense of the above passage in blank space (1) 'related' would be appropriate, because the VerbIII (related to clean) is given proper meaning in this sentence. The other options give different meanings. 852. Select the most appropriate option for blank No. 2. (a) of (b) for (c) with (d) In SSC MTS – 02/08/2019 (Shift-II) Ans. (a) : In the above passage in blank space (2) preposition 'of' would be appropriate, because 'disposal' is used with preposition 'of'. The other options are not suitable. 853. Select the most appropriate option for blank No. 3. (a) Preventing (b) Hampering (c) Prohibiting (d) Facilitating SSC MTS – 02/08/2019 (Shift-II) Ans. (a) : According to the sense of the above passage in blank space (3) 'preventing' would be appropriate, which means- 'to stop something from happening or someone from doing something.' While other options give different meanings. 854. Select the most appropriate option for blank No. 4. (a) ignores (b) paves (c) aims (d) directs SSC MTS – 02/08/2019 (Shift-II) Ans. (c) : According to the sense of the above passage in blank space (4) 'aims' would be appropriate, because senitation system aims to protect human health. Meaning of other optionsIgnores - Defy Paves - Pitch 855. Select the most appropriate option for blank No. 5. (a) that (b) who (c) whom (d) these SSC MTS – 02/08/2019 (Shift-II) Cloze Test

Ans. (a) : The use of 'that' in blank space (5) of the above passege would be appropriate, because 'that' is a relative pronoun, used for both person and thing. The other options give different meanings. Comprehension In the following passage some words have been deleted. Fill in the blanks with the help of the alternatives given. Select the most appropriate option for each blank. Deforestation 1. ––––– the permanent destruction of forests in order to 2. ––––– the land available for other uses. It 3. –––––– around the world, though tropical rainforests are particularly 4. –––––. If current deforestation levels 5. ––––, the world's rainforests may completely vanish in as little as 100 years, according to National Geographic. 856. Select the most appropriate option for blank No. 1. (a) have (b) are (c) is (d) has SSC MTS – 02/08/2019 (Shift-III) Ans. (c) : In the above passage in blank space (1) 'is' would be appropriate, because the singular subject (Deforestation) takes the singular verb 'is'. The other options give different meanings. 857. Select the most appropriate option for blank No. 2. (a) makes (b) make (c) made (d) making SSC MTS – 02/08/2019 (Shift-III) Ans. (b) : According to the sense of the above passage in blank space (2) 'make' (V1) would be appropriate, because 'in order to' is a subordinating conjunction. It is used with an infinitive form of a verb to express the purpose of something. 858. Select the most appropriate option for blank No. 3. (a) occurs (b) occurred (c) occurring (d) occur SSC MTS – 02/08/2019 (Shift-III) Ans. (a) : According to the sense of the above passage, the word 'Occurs' will be used in blank space (3), because 'It' is a singular pronoun, the singular verb 'occurs' used with it. The other options are not suitable. 859. Select the most appropriate option for blank No. 4. (a) targeted (b) robbed (c) looted (d) aimed SSC MTS – 02/08/2019 (Shift-III) Ans. (a) : According to the sense of the above passage in blank space (4), 'targeted' would be appropriate which means- 'the act of deciding to attack a particular point, area, or person'. While the other options given different meaningRobbed- Pilfered, Cheated, Looted- Plunder, Ransack, Aimed- Directed, Pointed

848

YCT

860. Select the most appropriate option for blank No. 5. (a) maintain (b) continue (c) follow (d) proceed SSC MTS – 02/08/2019 (Shift-III) Ans. (b) : According to the sense of the above passage in blank space (5) the word 'continue' would be appropriate, while other options meanMaintain- keep up, perpetuate Follow- propel, locomote Proceed- Carryon, Ramble Comprehension In the following passage some words have been deleted. Fill in the blanks with the help of the alternatives given. Select the most appropriate option for each blank. Obesity is a condition where the body stores excess fat leading to a considerable 1. –––– in the body weight. It is increasing 2. –––– a fast pace in our country. It has 3. –––– adverse health effects and can even 4. –––– to premature death. The basic cause of obesity is 5. – ––– imbalance between energy intake and energy expenditure. 861. Select the most appropriate option for blank No. 1. (a) decrease (b) decline (c) raise (d) increase SSC MTS – 05/08/2019 (Shift-II) Ans. (d) : According to the sense of the above passage in blank space (1) 'Increase' would be appropriate, which means growth of something that gets bigger in number or volume. Meaning of other optionsDecrease - Reduction, Scarcity Decline - Downfall, Decadence Raise - Uplift, Distention 862. Select the most appropriate option for blank No. 2. (a) on (b) at (c) by (d) with SSC MTS – 05/08/2019 (Shift-II) Ans. (b) : According to the sense of the above passage in blank space (2) 'At' preposition will be used, because preposition 'at' is used here to denote the situation with increasing. While, the other options are not correct. 863. Select the most appropriate option for blank No. 3. (a) little (b) many (c) few (d) much SSC MTS – 05/08/2019 (Shift-II) Ans. (b) : According to the sense of the above passage in blank space (3) the word 'many' would be appropriate because positive sense is needed here and used with countable sense. While, the other options give different meanings. Cloze Test

864. Select the most appropriate option for blank No. 4. (a) take (b) point (c) lead (d) start SSC MTS – 05/08/2019 (Shift-II) Ans. (c) : According to the sense of the above passage in blank space (4) 'Lead' would be appropriate which means - result in, cause. Leads to - a situation or event, usually on unpleasant one. While, the other options give different meaning. 865. Select the most appropriate option for blank No. 5. (a) no (b) a (c) one (d) the SSC MTS – 05/08/2019 (Shift-II) Ans. (d) : According to the sense of the above passage in blank space (5) 'The' would be appropriate, because definite article 'the' is used to specific or particular noun. While, the other options are not correct. Comprehension In the following passage some words have been deleted. Fill in the blanks with the help of the alternatives given. Select the most appropriate option for each blank. Education makes a person self-reliant. An (1) – ––– person is well equipped to take important decisions (2) –––– life. He is the master of his (3) –––– and fortune. On the other hand an illiterate person (4) ––––– help and guidance at every step. He feels helpless (5) –––– is at the mercy of others. 866. Select the most appropriate option for blank No. 1. (a) illiterate (b) educated (c) wise (d) learned SSC MTS – 05/08/2019 (Shift-III) Ans. (b) : According to the sense of the above passage in blank space (1) 'Educated' would be appropriate. Meaning of other optionsIlliterate - Uneducated Wise - Intelligent, Shrewd Learned - Savant, Scholar 867. Select the most appropriate option for blank No. 2. (a) to (b) in (c) at (d) on SSC MTS – 05/08/2019 (Shift-III) Ans. (b) : In the blank space (2) of the above passage, the word 'in' (preposition) would be appropriate. While, other options give different meaningHence, the correct option is (b). 868. Select the most appropriate option for blank No. 3. (a) fate (b) luck (c) chance (d) lot SSC MTS – 05/08/2019 (Shift-III)

849

YCT

Ans. (a) : According to the sense of the above passage in blank space (3) 'fate' would be appropriate because fortune has been used after fate (fate and fortune). The other options give different meaning. 869. Select the most appropriate option for blank No. 4. (a) has needed (b) is needing (c) need (d) needs SSC MTS – 05/08/2019 (Shift-III) Ans. (d) : According to the sense of the above passage in blank space (4) 'needs' will be used, because its subject (an illiterate person) is singular, so here singular verb (verb1 + s/es) will be used in simple present tense. The other options are not appropriate. 870. Select the most appropriate option for blank No. 5. (a) and (b) unless (c) though (d) but SSC MTS – 05/08/2019 (Shift-III) Ans. (a) : According to the sense of the above passage in blank space (5) 'and' would be appropriate because the use of conjunction 'and' is appropriate to connect two sentences or two word. While, the other options give different meaning. Comprehension In the following passage some words have been deleted. Fill in the blanks with the help of the alternatives given. Select the most appropriate option for each blank. The Narmada River, (1) ––––– called the Rewa was previously known (2) ––––– Nerbudda. It is a river in central India along (3) –––– the Godavari and the Krishna. It is also (4) ––––– as the 'Life Line of Gujarat and Madhya Pradesh' for its huge (5) ––––– to these states. The Narmada rises from Amarkantak Plateau near Anuppur district. 871. Select the most appropriate option for blank No. 1. (a) but (b) or (c) also (d) and SSC MTS – 07/08/2019 (Shift-I) Ans. (c) : According to the sense of the above passage in blank space (1) 'also' would be appropriate because it is an adverb, placed before the main verb or after a helping verb. While, other options give different meanings. 872. Select the most appropriate option for blank No. 2. (a) by (b) as (c) so (d) even SSC MTS – 07/08/2019 (Shift-I) Ans. (b) : According to the context of the above passage in blank space (2) 'as' would be appropriate because known is used with 'as'. While, other options are incorrect. Cloze Test

873. Select the most appropriate option for blank No. 3. (a) to (b) with (c) by (d) beside SSC MTS – 07/08/2019 (Shift-I) Ans. (b) : According to the sense of the above passage in blank space (3) 'with' would be appropriate because along is used with 'with'. It is a compound preposition. While, other options give different meanings. 874. Select the most appropriate option for blank No. 4. (a) told (b) said (c) known (d) spoken SSC MTS – 07/08/2019 (Shift-I) Ans. (c) : According to the sense of the above passage in blank space (4) 'known' will be used because the given sentence is past tense in passive voice. So, the verb (known) is used in simple past in passive voice. While, other options are not correct in this context. 875. Select the most appropriate option for blank No. 5. (a) donation (b) reformation (c) irrigation (d) contribution SSC MTS – 07/08/2019 (Shift-I) Ans. (d) : According to the sense of the above passage in blank space (5) 'contribution' would be appropriate which means- 'the giving or supplying of something'. Meaning of other optionsDonation - Keepsake Reformation - Reclamation Irrigation - Watering Comprehension In the following passage some words have been deleted. Fill in the blanks with the help of the alternatives given. Select the most appropriate option for each blank. The Ashoka pillars are a series of columns (1) – ––––– in northern India by king Ashoka (2) ––– ––– his reign in the 3rd century BCE. (3) ––––– pillars are inscribed with edicts by the Mauryan king. At an average (4) ––––– 50 feet in height and weighing up to 50 tons each, (5) – –––– 19 pillars survive today. 876. Select the most appropriate option for blank No. 1. (a) erected (b) located (c) developed (d) created SSC MTS – 08/08/2019 (Shift-I) Ans. (a) : According to the sense of the above passage in blank space (1) 'erected' would be appropriate which means - 'Established'. Meaning of other optionsLocated - Vested Developed - Evolved Created - Fabricated

850

YCT

877. Select the most appropriate option for blank No. 2. (a) across (b) through (c) along (d) during SSC MTS – 08/08/2019 (Shift-I) Ans. (d) : According to the sense of the above passage in blank space (2) 'during' would be appropriate, because show the 'time duration' preposition 'during' will be used. The other options give the different meaning. 878. Select the most appropriate option for blank No. 3. (a) These (b) That (c) This (d) Those SSC MTS – 08/08/2019 (Shift-I) Ans. (a) : According to the context of the above passage in blank space (3) 'these' will be used, because here demonstrative adjective (these) will be used before the plural noun (pillars). 879. Select the most appropriate option for blank No. 4. (a) by (b) at (c) of (d) on SSC MTS – 08/08/2019 (Shift-I) Ans. (c) : According to the sense of the above passage in blank space (4) 'of' (preposition) would be appropriate because average is used with preposition 'of'. 880. Select the most appropriate option for blank No. 5. (a) only (b) any (c) also (d) even SSC MTS – 08/08/2019 (Shift-I) Ans. (a) : According to the sense of the above passage in blank space (5) 'only' would be appropriate. It is an adverb, used according to sense of sentence. The other options give incorrect meanings. Comprehension In the following passage some words have been deleted. Fill in the blanks with the help of the alternatives given. Select the most appropriate option for each blank. Bengaluru is becoming an exciting destination for higher education for students (1) –––––all over the world. International students (2) –––– close to 20% of the student population (3) –––– the city and this number is (4) ––––– increasing. The city has almost 900 colleges the maximum in any city (5) ––––– the country. 881. Select the most appropriate option to fill in blank No. 1. (a) in (b) for (c) from (d) by SSC MTS – 08/08/2019 (Shift-III) Ans. (c) : According to the sense of the above passage in blank space (1) 'From' would be appropriate, because preposition 'From' shows where something starts, indicates the distance, origin or position. Meaning of other options are not appropriate. Cloze Test

882. Select the most appropriate option to fill in blank No. 2. (a) constitute (b) consist (c) construct (d) collect SSC MTS – 08/08/2019 (Shift-III) Ans. (a) : According to the sense of the above passage in blank space (2) 'Constitute' would be appropriate, which means- 'Something to form a group legally or officially. Meaning of other optionsConsist - Texture, Design Construct - Build, Fabricate Collect - Hoard, Gather 883. Select the most appropriate option to fill in blank No. 3. (a) at (b) in (c) to (d) on SSC MTS – 08/08/2019 (Shift-III) Ans. (b) : According to the sense of the above passage in blank space (3) 'in' would be appropriate, because preposition 'in' is used to a place that is inside. The other options are not correct. 884. Select the most appropriate option to fill in blank No. 4. (a) readily (b) barely (c) rarely (d) steadily SSC MTS – 08/08/2019 (Shift-III) Ans. (d) : According to the sense of the above passage in blank space (4) 'steadily' (adverb) would be appropriate, which means- 'in a gradual, regular way over a period of time'. Meaning of other optionsReadily - Instantly, Apace Barely - Hardly, Scarcely Rarely - Perhaps, Lest 885. Select the most appropriate option to fill in blank No. 5. (a) about (b) along (c) across (d) through SSC MTS – 08/08/2019 (Shift-III) Ans. (c) : According to context of the above passage in blank space (5) 'across' would be appropriate, because it is a preposition, used from one side to the other of something which has sides or limits such as a city. Comprehension In the following passage some words have been deleted. Fill in the blanks with the help of the alternatives given. Select the most appropriate option for each blank. Laura threw herself (1) ––––– the bed and sobbed bitterly. (2) ––––– in her life had she been subjected to (3) ––––– an outrage, never had she been so deeply (4) ––––. She was suspected (5) ––––– stealing.

851

YCT

886. Select the most appropriate option to fill in blank No. 1. (a) to (b) on (c) in (d) at SSC MTS – 22/08/2019 (Shift-II) Ans. (b) : According to the sense of the above passage in blank space (1) preposition 'on' would be appropriate, because it is used 'on' when we refer to a position on a surface, as (on the table, on the bed, on the roof) etc. 887. Select the most appropriate option to fill in blank No. 2. (a) Before (b) Never (c) Always (d) After SSC MTS – 22/08/2019 (Shift-II) Ans. (b) : According to the sense of the above passage in blank space (2) 'Never' would be appropriate, because it is an adverb which means 'not in any circumstances at all'. While, other options give different meanings. 888. Select the most appropriate option to fill in blank No. 3. (a) as (b) like (c) such (d) so SSC MTS – 22/08/2019 (Shift-II) Ans. (c) : In the blank space (3) of the above passage the word 'such' would be appropriate because 'such' is used as a predeterminer. While, other options give different meanings. 889. Select the most appropriate option to fill in blank No. 4. (a) excited (b) engaged (c) insulted (d) happy SSC MTS – 22/08/2019 (Shift-II) Ans. (c) : According to the sense of the above passage in blank space (4) 'insulted' would be appropriate which means- 'offended'. Meaning of other optionsExcited - Agitated, Elated Engaged - Dissipated, Active Happy - Joyous, Lively 890. Select the most appropriate option to fill in blank No. 5. (a) on (b) of (c) in (d) to SSC MTS – 22/08/2019 (Shift-II) Ans. (b) : For the above passage in blank space (5) preposition 'of' would be appropriate because 'of' is used with suspect such as - suspect of doing something. Comprehension In the following passage some words have been deleted. Fill in the blanks with the help of the alternatives given. Select the most appropriate option for each blank. Steamships were first (1) ––––– to the United States in 1807. By the 1830's dozens of steam vessels were in (2) ––––. They offered the traveller (3) –––– transportation in comfortable facilities-a welcome (4) –––– to stagecoach travel, which at best could be described as (5) – –––. Cloze Test

891. Select the most appropriate option to fill in blank No. 1. (a) established (b) built (c) introduced (d) manufactured SSC MTS – 22/08/2019 (Shift-III) Ans. (c) : According to the sense of the above passage in blank space (1) 'Introduced' would be appropriate, while other options give different meanings. 892. Select the most appropriate option to fill in blank No. 2. (a) use (b) stand (c) move (d) transit SSC MTS – 22/08/2019 (Shift-III) Ans. (a) : According to the context of the above passage in blank space (2) 'use' would be appropriate. Meaning of other optionsStand - Pause, Stasis Move - Ruse, Ploy Transit - Change, Variation 893. Select the most appropriate option to fill in blank No. 3. (a) light (b) slow (c) fast (d) leisurely SSC MTS – 22/08/2019 (Shift-III) Ans. (c) : According to the sense of the above passage in blank space (3) the word 'fast' (adjective) would be appropriate. Meaning of other optionsLight - Illumination, Beacon Slow - Gently, Doggo Leisurely - Quietly, Gingerly 894. Select the most appropriate option to fill in blank No. 4. (a) sign (b) signal (c) gesture (d) alternative SSC MTS – 22/08/2019 (Shift-III) Ans. (d) : According to the sense of the above passage in blank space (4) the word 'alternative' will be appropriate answer. Thus, the correct answer is option (d). 895. Select the most appropriate option to fill in blank No. 5. (a) fortunate (b) wretched (c) admirable (d) worthy SSC MTS – 22/08/2019 (Shift-III) Ans. (b) : According to the passage, the most appropriate option in blank space (5) 'wretched' will be used which means 'very unhappy'. Thus, the correct answer is option (b). Comprehension In the following passage some word have been deleted. Fill in the blanks with e help of the alternatives given. Select the most appropriate option for each blank. The Thane Municipal Transport (TMT) has seen a 6% drop (1) ––––– passenger density after the Brihanmumbai Electric Supply and

852

YCT

Transport (BEST) reduced (2) –––– bus fares recently. The drop, though small in enough to set the (3) –––– bells ringing. TMT plies nearly 300 buses and is already facing (4) ––––– in managing regular schedules. The ticket cost cutby BEST (5) –––– TMT routes that overlap that of the BEST. 896. Select the most approrpriate option for blank No. 1. (a) of (b) in (c) on (d) from SSC MTS – 06/08/2019 (Shift-III) Ans. (b) : According to the passage, the most appropriate option for blank space (1) preposition 'in' will be used. The correct sentence isTMT has seen a 6% drop in passenger. Thus, the correct answer is option (b). 897. Select the most appropriate option for blank No. 2. (a) her (b) it's (c) his (d) its SSC MTS – 06/08/2019 (Shift-III) Ans. (d) : According to the passage, the most appropriate option for blank space (2) is 'its' because it is a possessive adjective, used before a noun (bus). While, other options are incorrect. 898. Select the most appropriate option for blank No. 3. (a) clock-tower (b) church (c) temple (d) alarm SSC MTS – 06/08/2019 (Shift-III) Ans. (d) : According to the passage, the most appropriate answer for the blank space (3) is 'alarm'. The correct sentence is'The drop, though small in enough to set the alarm bells ringing. Thus, the correct answer is option (d). 899. Select the most appropriate option for blank No. 4. (a) consequences (b) differences (c) facilities (d) difficulties SSC MTS – 06/08/2019 (Shift-III) Ans. (d) : According to the passage, the most appropriate option for blank space (4) is 'difficulties'. The meaning of other words areConsequences - Results Differences - Diversity Facilities - Accessories Thus, the correct answer is option (d) 900. Select the most appropriate option for blank No. 5. (a) has impacting (b) impact (c) has impacted (d) impacting SSC MTS – 06/08/2019 (Shift-III) Ans. (c) : According to the passage, the most appropriate option for blank space (5) is 'has impacted' because 'has' take the third form of main verb. Thus, the correct answer is option (c). Cloze Test

Comprehension In the following passage some of the words have been left out. Read the passage carefully and select the correct answer for the blank out of the four alternatives. The tremendous ––1–––– in the number of students and of education institutions has given rise to the term 'education explosion'. No doubt, this has resulted in serious problems such as –––2–– of financial resources and infrastructure and dilution of personal attention to the education and characterformation of the students. Also, there is the ––– 3–– side-effect of enormous increase in the number of educated unemployed. However, we cannot –––4––– the advantages of education explosion in India. Mere increase in the percentage of literate people does not indicate a –––5–– change in the educational standards of the people and a real improvement in manpower resources of India. Unemployment problem in India cannot be blamed on the availability of large masses education People in India. 901. The tremendous –––– in the (a) increase (b) decrease (c) neutralize (d) decree SSC MTS 11-10-2017 (Shift-I) Ans : (a) According to the passage, the most appropriate answer is option (a) 'increase' because the sentences is 'The tremendous increase in the…. Decrease - Reduce Neutralize - Unaffected Decree - Order 902. Serious problems such as ––––––––. (a) extravagant (b) inadequacy (c) sufficiency (d) abundant SSC MTS 11-10-2017 (Shift-I) Ans : (b) According to the passage, the most appropriate option for blank space (2) is 'inadequacy' which means 'insufficiency'. The meaning of other words areExtravagant - Squander Sufficiency - Ampleness Abundant - Copious Thus, the correct answer is option (b). 903. Also, there is the –––– side-effect of (a) desirable (b) required (c) unwanted (d) assumable SSC MTS 11-10-2017 (Shift-I) Ans : (c) According to the passage, the most appropriate option for the blank space (3) is 'unwanted' which means 'forbidding'. Thus, the correct answer is option (c).

853

YCT

904. However, we cannot ––––– the advantages (a) overburden (b) overlook (c) overturn (d) overdone SSC MTS 11-10-2017 (Shift-I) Ans : (b) According to the passage, the most appropriate option for the blank space (4) is 'overlook' which means 'disregard'. The meaning of the other words areOverburden - Overwhelm Overturn - Reverse Overdone - Exaggerate Thus, the correct answer is option (b) 905. does not indicate a –––––– change in the educational (a) quantitative (b) situational (c) qualitative (d) seasonal SSC MTS 11-10-2017 (Shift-I) Ans : (a) According to the passage, the most appropriate answer for the blank space (5) is 'Quantitative'. The meaning of the other words areSituational - Circumstantial Qualitative - Approximate Seasonal - Occasional Thus, the correct answer is option (a). Comprehension In the following passage some of the words have been left out. Read the passage carefully and select the correct answer for the given blank out of the four alternatives. Child labor is commonly seen at various places in India. From roadside tea stalls to big and small factories producing ___1__ products like cement, handloom and jute products, carpets, fashionable tawdry armaments and fireworks and many other hazardous jobs. Children are working in ____2___ condition. There have been cases where child labours were severely beaten and offered no medical help in case they complained or try to ____3___. There are practical difficulties in the way of abolishing child labor overnight. Without _____4__ poverty child labor cannot be abolished. There must be a special _____5__ to check the evil. 906. factories producing ________ products (a) waste (b) sundry (c) bring (d) producing SSC MTS 11-10-2017 (Shift-III) Ans. (b) : According to the passage, the most appropriate answer for the blank (1) is 'Sundry' which means 'Various'. The meaning of other words areWaste - Squander Producing - Manufacture Thus, the correct answer is option (a). Cloze Test

907. are working in ________ condition. (a) neat and clean (b) abysmal (c) hopeful (d) good SSC MTS 11-10-2017 (Shift-III) Ans. (b) : According to the passage, the most appropriate option for blank space (2) is the word 'Abysmal' which means 'disappointed'. The correct sentence isAre working in abysmal condition. 908. complain or try to __________. (a) relate (b) negotiate (c) escape (d) innate SSC MTS 11-10-2017 (Shift-III) Ans. (c) : According to the passage, the most appropriate option for the blank space (3) is the word 'escape' which means 'getaway' The meaning of other words areNegotiate - Embargo Innate - Inborn Thus, the correct answer is option (c). 909. Without _____________ poverty child labor (a) initiating (b) decreasing (c) referring (d) eradicating SSC MTS 11-10-2017 (Shift-III) Ans. (d) : According to the passage, the most appropriate word/option for the blank space (4) is 'Eradicating' which means 'Abolish'. Thus, the correct answer is option (d). 910. must be a special ________ to check the evil. (a) war (b) divine (c) force (d) source SSC MTS 11-10-2017 (Shift-III) Ans. (a) : According to the passage, the most appropriate option for the blank space (5) is 'War'. The correct sentence isMust be a special war to check the evil. Comprehension In the following passage Read the passage carefully For centuries of human existence, we have believed or are rather ___1 to believe that humanity has by nature, been divided into____2 or castes and that certain race physical or intellectual endowments in terms of others to superior, The discrimination that follows this belief is called racism or _____3 ince time immemorial. society has divided into different classes and communities. In countries like India, there was a caste system It was believed that God had created people of a particular caste only to do one kind of Job. So the castes were developed with professional lines The people who performed low-key jobs were treated much _____4. This attitude has been so strongly enriched in the Indian psyche that it continues to exist even ____5. to some measure.

854

YCT

911. We have believed or are rather ______ to believe that (a) taught (b) brought (c) thought (d) sort SSC MTS 9-10-2017 (Shift-III) Ans : (a) According to the passage, the most appropriate option for the blank space (1) is 'Taught' while other options areBrought means - past tense of bring Thought means- consideration Sort means- type Thus, the correct answer is option (a). 912. in ______ or castes and that certain races (a) faces (b) races (c) cases (d) braces SSC MTS 9-10-2017 (Shift-III) Ans : (b) According to the passage, the most appropriate option for the blank space (2) is 'Races'. The Correct sentence is'In races or castes and that certain races. 913. This belief is called racism of _____. (a) atheist (b) agnostic (c) apartheid (d) proteid SSC MTS 9-10-2017 (Shift-III) Ans : (c) According to the passage, the most appropriate option for the blank space (3) is 'apartheid' which means discrimination. The meaning of other words areAtheist means- idolater Agnostic means- Secular Thus, the correct answer is option (c). 914. were treated very_____ (a) lovely (b) nicely (c) sincerely (d) badly SSC MTS 9-10-2017 (Shift-III) Ans : (d) According to the passage, the most appropriate option for the blank space (4) is 'badly'. Thus, the correct answer is option (d). 915. it continues to exist even ______, to some measure. (a) tomorrow (b) today (c) present (d) weekday SSC MTS 9-10-2017 (Shift-III) Ans : (b) According to the passage, the most appropriate option for the blank space (5) is 'Today'. The correct sentence isIt continues to exist even today to some measure. Thus, the correct answer is option (b). Comprehension In the following passage some of the words have been left out. Read the passage carefully and select the correct answer for the given blank out of the four alternatives. Co-education helps the boys and girls to.........1 and understand each other well. They become more broad-minded and..........2 towards Cloze Test

the.......3. gender. They interact freely with........4, thereby overcoming hesitation and shyness. Thus, co-education leads to a...........5 and harmonious relationship between boys and girls. 916. The boys and girls to............. (a) intermingle (b) separate (c) divide (d) unmix SSC MTS 7-10-2017 (Shift-I) Ans. (a) : According to the passage, the most appropriate option for the blank space (1) is 'Intermingle' which means 'Harmonize'. Thus, the correct answer is option (a). 917. more broad-minded and........towards. (a) disapproving (b) tolerant (c) biased (d) prejudiced SSC MTS 7-10-2017 (Shift-I) Ans. (b) : According to the passage, the most appropriate option for the blank space (2) is 'Tolerant' which means 'Endure'. The meaning of the other words areDisapproving means- Refusing Biased means- Deflect Prejudiced means- Partisan 918. the..........gender. (a) like (b) akin (c) opposite (d) same SSC MTS 7-10-2017 (Shift-I) Ans : (c) According to the passage, the most appropriate option for the blank space (3) is 'Opposite'. Thus, the correct answer is option (c). 919. freely with...........other, (a) every (b) each (c) some (d) one SSC MTS 7-10-2017 (Shift-I) Ans. (b) : According to the passage, the most appropriate option for the blank space (4) is 'Each' because 'each' is used for two person which is mentioned in the passage as boy and girls. Thus, the correct answer is option (b). 920. co-education leads to a...........and harmonious (a) sick (b) infirm (c) delicate (d) healthy SSC MTS 7-10-2017 (Shift-I) Ans : (d) According to the passage, the most appropriate option for the blank space (5) is 'Healthy'. While other options areSick means- ill Infirm means - Weak Delicate means- Fragile Thus, the sentence isCo-education leads to a healthy and harmonious.

855

YCT

Comprehension In the following some words have been deleted. Fill in the blanks with the help of the alternatives given. Every day for a whole year, all kinds of holy men, hermits, scholars and nobles came and related to the priests their deeds of (1) ______ and the priests in solemn council heard their claims. At last, they decided that the one who seemed to be the greatest lover of (2) _____ was a rich man who had that very year given all his (3) ______ to the poor. So they gave him the plate of gold, but when he took it in his hand, it turned into (4) ______ lead though when he dropped it on the floor, to his (5) _____, it became gold again. 921. Select the most appropriate option that will fill in the blank number 1. (a) charity (b) kindness (c) happiness (d) sympathy SSC GD – 11/02/2019 (Shift-II) Ans. (a) : According to the passage, the most appropriate option for the blank space (1) is 'Charity' which means 'Mercifulness'. The meaning of the other words areKindness means- Gentleness Happiness means- Pleasure Sympathy means- Compassion Thus, the correct answer is option (a). 922. Select the most appropriate option that will fill in the blank number 2. (a) destitute (b) moneyless (c) mankind (d) women SSC GD – 11/02/2019 (Shift-II) Ans. (c) : According to the passage, the most appropriate option for the blank space (2) is 'Mankind'. The meaning of other words areDestitute means- Miserable Moneyless means - Without money Thus, the correct answer is option (c). 923. Select the most appropriate option that will fill in the blank number 3. (a) wealth (b) energy (c) time (d) life SSC GD – 11/02/2019 (Shift-II) Ans. (a) : According to the passage, the most appropriate option for the blank space (3) 'Wealth' which means 'Money'. Thus, the correct answer is option (a). 924. Select the most appropriate option that will fill in the blank number 4. (a) gold (b) worthy (c) worthless (d) valuable SSC GD – 11/02/2019 (Shift-II) Ans. (c) : According to the passage, the most appropriate option that will fill in the blank number (4) 'Worthless' which means 'Useless'. Thus, the correct answer is option (c). Cloze Test

925. Select the most appropriate option that will fill in the blank number 5. (a) confusion (b) happiness (c) admiration (d) amazement SSC GD – 11/02/2019 (Shift-II) Ans. (d) : According to the passage, the most appropriate option for the blank space (5) is 'Amazement' which means 'Surprised'. Thus, the correct answer is option (d). Comprehension In the following passage some words have been deleted. Fill in the blanks with the help of the alternatives given. Without water (1) ____ animal can survive. In desert regions the greatest (2) ______ to life is dying of thirst. But many creatures are able to make use of (3) _____little water that exists in arid areas. One of nature's masterpieces (4) ______ creatures equipped to (5) ______ with desert life is the hardy camel. 926. Select the most appropriate option that will fill in the blank number 1. (a) any (b) some (c) every (d) no SSC GD – 14/02/2019 (Shift-I) Ans. (d) : According to the passage, the most appropriate option for the blank space (1) is 'No'. The sentence isWithout water, no animal can survive. 927. Select the most appropriate option that will fill in the blank number 2. (a) threat (b) warning (c) worry (d) requirement SSC GD – 14/02/2019 (Shift-I) Ans. (a) : According to the passage, the most appropriate option for the blank space (2) is 'Threat' which means 'Menace'. The meaning of the other words areWarning means- Alert Worry means- Disquietness Requirement means - Necessity Thus, the correct answer is option (a). 928. Select the most appropriate option that will fill in the blank number 3. (a) the (b) that (c) very (d) what SSC GD – 14/02/2019 (Shift-I) Ans. (a) : According to the passage, the most appropriate option for the blank space (3) is 'The'. The sentence isThe make use of the little water. 929. Select the most appropriate option that will fill in the blank number 4. (a) besides (b) about (c) among (d) between SSC GD – 14/02/2019 (Shift-I)

856

YCT

Ans. (c) : According to the passage, the most appropriate option for the blank space (4) is 'Among'. Thus, the correct answer is option (c). 930. Select the most appropriate option that will fill in the blank number 5. (a) cope (b) resist (c) adapt (d) live SSC GD – 14/02/2019 (Shift-I) Ans. (a) : According to the passage, the most appropriate option for the blank space (5) is 'Cope' because the complete phrase is 'cope with' which means 'Tolerate'. Thus, the correct answer is option (a). Comprehension The second Anglo-Maratha war had shattered the (1) _____ of the Maratha chiefs, but not their spirit. The loss of their freedom caused them deep ______ (2). They made a last (3) _____ attempt to regain their independence in 1817. The lead in organizing a united front of the Maratha chiefs ______ (4) by the Peshwa who was smarting under the (5) ______ control exercised by the British Resident. 931. Select the most appropriate option that will fill in the blank no 1. (a) patience (b) dignity (c) power (d) time SSC GD – 14/02/2019 (Shift-II) Ans. (c) : According to the passage, the most appropriate option for the blank space (1) is 'Power'. The other options arePatience means - Endurance Dignity means- Gorgeousness Thus, the correct answer is option (c). 932. Select the most appropriate option that will fill in the blank no 2. (a) agony (b) distress (c) fear (d) depression SSC GD – 14/02/2019 (Shift-II) Ans. (a) : According to the passage, the most appropriate option for the blank space (2) is 'Agony' which means 'Anguish'. Thus, the correct answer is option (a). 933. Select the most appropriate option that will fill in the blank no 3. (a) violent (b) massive (c) desperate (d) feeble SSC GD – 14/02/2019 (Shift-II) Ans. (c) : According to the passage, the most appropriate option for the blank space (3) is 'Desperate' which means 'Hopeless'. The meaning of other words areViolent means- Onslaught Massive means- Palatial Feeble means- Feckless Thus, the correct answer is option (c). Cloze Test

934. Select the most appropriate option that will fill in the blank no 4. (a) was taking (b) was taken (c) had taken (d) were taken SSC GD – 14/02/2019 (Shift-II) Ans. (b) : According to the passage, the most appropriate option for the blank space (4) is 'was taken' because sentence is in passive. Thus, the correct answer is option (b). 935. Select the most appropriate option that will fill in the blank no 5. (a) cruel (b) orthodox (c) pleasant (d) rigid SSC GD – 14/02/2019 (Shift-II) Ans. (d) : According to the passage, the most appropriate option for the blank space (5) is 'Rigid'. The meaning of other words areCruel means- Brutal Orthodox means- Fundamentalist Pleasant means- Snug Thus, the correct answer is option (d). Comprehension In the following passage some words have been deleted. Fill in the blanks with the help of the alternatives given. The neglect of one half of mankind costs the world a great deal both, in economic and social (1) ______. A family (2) _____ an educated mother is, in most cases, a poorer family than one in which everyone is educated and can (3) _______ a living. A family with an uneducated mother may (4) ______ in other ways too, because the mother is (5) _____ to help the children in their education. 936. Select the most appropriate option that will fill in the blank number 1. (a) terms (b) aspects (c) areas (d) values SSC GD – 18/02/2019 (Shift-III) Ans. (a) : According to the passage, the most appropriate option for the blank space (1) 'terms' will be used. The meaning of other words areAspects means- Facets Areas means - Surface Values means- Price, Cost Thus, the most appropriate answer is option (a). 937. Select the most appropriate option that will fill in the blank number 2. (a) without (b) of (c) including (d) with SSC GD – 18/02/2019 (Shift-III) Ans. (a) : According to the passage, the most appropriate option for the blank space (2) 'Without' will be used. Thus, the correct answer is option (a).

857

YCT

938. Select the most appropriate option that will fill in the blank number 3. (a) manage (b) earn (c) create (d) plan SSC GD – 18/02/2019 (Shift-III) Ans. (b) : According to the passage, the most appropriate option for the blank space (3) 'earn' will be used. Thus, the correct answer is option (b). 939. Select the most appropriate option that will fill in the blank number 4. (a) suffer (b) destroy (c) fail (d) prosper SSC GD – 18/02/2019 (Shift-III) Ans. (a) : According to the passage, the most appropriate option for the blank space (4) 'Suffer' will be used. The meaning of the other words areDestroy means- Dissipate Prosper means- Thrive Fail means - Failure Thus, the correct answer is option (a). 940. Select the most appropriate option that will fill in the blank number 5. (a) eligible (b) qualified (c) able (d) unable SSC GD – 18/02/2019 (Shift-III) Ans. (d) : According to the passage, the most appropriate option for the blank space (5) 'Unable' will be used. The meaning of the other words areEligible means - Acceptable Qualified means - Deserving Thus, the correct answer is option (d). Comprehension In the following passage some words have been deleted. Fill in the blanks with the help of the alternatives given. Violence is an expression of aggression. There can be various reasons that cause this kind of behavior. Various (1) ______ social situations or circumstances in life affect an individual adversely. The reaction to those situations is (2) ______ those who are short tempered or are unable to cope with the changing environment find it hard to behave (3) ______ this frustration comes out in the form of anger and violent behavior. Though, environmental factors are responsible but only to an (4) ______ Psychologists believe that violent traits are (5) ________ in nature and are likely to surface sooner or later. Cloze Test

941. Select the most appropriate option that will fill in the blank number 1. (a) lucky (b) destined (c) favourable (d) unfavourable SSC GD – 02/03/2019 (Shift-II) Ans. (d) : According to the passage, the most appropriate option for the blank space (1) 'Unfavourable' (Repugnant) will be used. While other words areLucky means- Fortunate Destined means- Fate Favourable means- Congruent 942. Select the most appropriate option that will fill in the blank number 2. (a) pleasant (b) suitable (c) indecisive (d) agreeable SSC GD – 02/03/2019 (Shift-II) Ans. (c) : According to the passage, the most appropriate option for the blank space (2) 'Indecisive' (doubt) will be used. Thus, the correct answer is option (c). 943. Select the most appropriate option that will fill in the blank number 3. (a) eagerly (b) negatively (c) rationally (d) harshly SSC GD – 02/03/2019 (Shift-II) Ans. (d) : According to the passage, the most appropriate option for the blank space (3) 'Harshly' (Strongly) will be used. The meaning of the other words areEagerly means- Dazedly Negatively means- Rejection Rationally means- Contemplatively Thus, the correct answer is option (d). 944. Select the most appropriate option that will fill in the blank number 4. (a) extent (b) range (c) end (d) intent SSC GD – 02/03/2019 (Shift-II) Ans. (a) : According to the passage, the most appropriate option for the blank space (4) 'Extent' will be used. Thus, the correct answer is option (a). 945. Select the most appropriate option that will fill in the blank number 5. (a) extrinsic (b) inherent (c) common (d) general SSC GD – 02/03/2019 (Shift-II) Ans. (b) : According to the passage, the most appropriate option for the blank space (5) 'Inherent' (Implicit) will be used. Thus, the correct answer is option (b).

858

YCT

13. PARAJUMBLES TCS hewšve& hej DeeOeeefjle (Based On TCS Pattern) Chapterwise

Exam

Question No.

Years

1

CGL (Tier-1) CGL (Tier-2) CHSL (Tier-1) CHSL (Tier-2) Selection Post VII, VIII, XI SSC MTS SSC GD SSC CPO SI

75 30 80 15 15 95 8 50

(2017–2023)

Parajumble/P.Q.R.S

Trend Analysis of Questions topicwise from CGL (Pre & Mains) CHSL (Pre & Mains) Selection Post VII, VIII, XI, SSC MTS, SSC GD & Other Exams (2017-2023)

Parajumbles

859

YCT

13. PARAJUMBLES 1.

Sentences of a paragraph are given below in jumbled order. Arrange the sentences in the correct order to form a meaningful and coherent paragraph. A. So the problems of India should not be viewed as the failure of this great nation. B. The uniqueness of this nation lies in its unity in diversity due to which there are problems here and there at times. C. India, a very large country with many religions, many mini-cultures and many languages, has been achieving success in all fields. D. But even small countries with monolithic societies have more problems than this second largest country. (a) CADB (b) CBDA (c) BDAC (d) DBAC SSC CGL (Tier-I) 19/07/2023 (Shift-IV) Ans. (b) : The correct order for jumble sentence is CBDA (b) because it gives appropriate meaning. Thus, the correct answer is (b). 2. Sentences of a paragraph are given below in jumbled order. Arrange the sentences in the correct order to form a meaningful and coherent paragraph. A. The olive tree grows slowly yet lives a long time. B. The normal lifespan of an olive tree is 300-400 years, but olive trees as ancient as 3000 years have been discovered. C. The olive is a well-known Mediterraneannative evergreen tree whose fruit and oil are used in cuisine and cooking. D. As a result, the olive tree is known as the ‘immortal tree’ in mythology and botany. (a) DBCA (b) ABCD (c) CABD (d) CDAB SSC CGL (Tier-I) 19/07/2023 (Shift-IV) Ans. (c) : The most appropriate jumbled order the sentence is option (c) [CABD]. 3. Sentences of a paragraph are given below in jumbled order. Arrange the sentences in the correct order to form a meaningful and coherent paragraph. A. It was the night before the day fixed for his coronation, and the young King was sitting alone in his beautiful chamber. B. And, indeed, it was the hunters who had found him, coming upon him almost by chance as, bare-limbed and pipe in hand, he was following the flock of the poor goat herd who had brought him up, and whose son he had always fancied himself to be. Parajumbles

C. The lad—for he was only a lad, being but sixteen years of age—was not sorry at their departure, and had flung himself back with a deep sigh of relief on the soft cushions of his embroidered couch, lying there, wild-eyed and open-mouthed, like a brown woodland Faun, or some young animal of the forest newly snared by the hunters. D. His courtiers had all taken their leave of him, bowing their heads to the ground, according to the ceremonious usage of the day, and had retired to the Great Hall of the Palace, to receive a few last lessons from the Professor of Etiquette; there being some of them who had still quite natural manners, which in a courtier is, I need hardly say, a very grave offence. (a) CBDA (b) CDBA (c) ADCB (d) ACDB SSC CGL (Tier-I) 21/07/2023 (Shift-II) Ans. (c) : The correct sequence of above jumbled sentences is option (c) [ADCB]. Other options are not correct in jumbled order. 4. Sentences of a paragraph are given below in jumbled order. Arrange the sentences in the correct order to form a meaningful and coherent paragraph. A. In 1944, he was honoured by King George VI with a Knighthood and became Sir Alexander Fleming. B. On receiving this great honour, Fleming said in his usual modest way, “Wherever I go, people thank me for saving their lives. I didn’t do anything; I just found a remedy that was there.” C. Fleming became famous overnight and was regarded as one of the most distinguished scientists of his time. D. The next year, he and his fellow bacteriologists, whose combined efforts had led to the great discovery, were honoured with a joint Nobel Prize. (a) DBCA (b) BCDA (c) ADBC (d) CADB SSC CGL (Tier-I) 21/07/2023 (Shift-II) Ans. (d) : The correct jumbled order of the above given sentences is option (d) [CADB]. 5. Sentences of a paragraph are given below in jumbled order. Arrange the sentences in the correct order to form a meaningful and coherent paragraph. A. This has an impact not just on global poverty, but also on Indians' awareness of the country's poverty for the first time in a decade.

860

YCT

B. And, for the first time in a decade, this update includes India's poverty data. C. From 15 September 2022, a new global poverty line has been established, with the World Bank revising its data in the Poverty and Inequality Platform on this basis. D. According to Sunitha Sinha, Roy and Roy van der Weide of the World Bank's Policy Research Working Paper, "poverty in India had dropped over the recent decade, although not as much as previously thought." (a) CBAD (b) DBAC (c) DBCA (d) CADB SSC CGL (Tier-I) 26/07/2023 (Shift-II) Ans. (a) : The correct sequence of above jumbled sentence is option (a) [CBAD]. because it gives correct meaning. 6. Sentences of a paragraph are given below in jumbled order. Arrange the sentences in the correct order to form a meaningful and coherent paragraph. A. Diwali, referred to as the 'festival of lights', ushers in the winter season with joy, shopping, presents, and a slew of new occasions and celebrations, such as Dussehra, Dhanteras, Govardhan, Bhaidooj, and Narak Chaturdashi. B. In the spiritual world, the event symbolises the triumph of light over darkness’. C. The five-day festival of Diwali is observed during the Hindu month of Kartika as an event. D. In India, Diwali is also known as Deepawali. (a) ACBD (b) BACD (c) CDBA (d) DBCA SSC CGL (Tier-I) 26/07/2023 (Shift-II) Ans. (a) : The correct sequence of above Jumbled sentences is option (a) [ACBD]. 7. Sentences of a paragraph are given below in jumbled order. Arrange the sentences in the correct order to form a meaningful and coherent paragraph. A. According to him, Rupees 75 lakhs has been spent to make the gymnasium fully equipped with state-of-the-art fitness machines. B. The Honorable Commissioner of Police inaugurated the Gymnasium and Swimming pool and appended the management for their efforts in providing such amenities to students. C. A new indoor gymnasium and swimming pool was inaugurated at APJ International School, Goa. D. Mr. Mercia, the Principal of the school familiarised the guests about the cost incurred and the various facilities available. (a) CBDA (b) ADBC (c) BDCA (d) DABC SSC CGL (Tier-I) 26/07/2023 (Shift-II) Parajumbles

Ans. (a) : The correct order for the jumbled sentences is option (a) [CBDA]. Other option are incorrect. 8. Sentences of a paragraph are given below in jumbled order. Arrange the sentences in the correct order to form a meaningful and coherent paragraph. A. Through the development of technologies, telecommunications, transportation, and other areas, globalisation has developed very quickly in the last few decades, resulting in economic, social, political, and cultural integration on a global scale. B. Although technology has had both positive and harmful effects on human life, its negative repercussions must be addressed appropriately. C. Globalisation has made significant positive contributions to economies all across the world. D. For example — Science and technology have made remarkable strides, providing businesses with a fantastic opportunity to expand quickly even outside of national borders. (a) BCDA (b) CDBA (c) ACBD (d) ACDB SSC CGL (Tier-I) 25/07/2023 (Shift-IV) Ans. (d) : The correct jumbled order of the above given sentence is option (d) 'A C D B.' The other options are not correct. 9. Sentences of a paragraph are given below in jumbled order. Arrange the sentences in the correct order to form a meaningful and coherent paragraph. A. Ashwagandha is an adaptogenic herb that has multiple benefits like improving thyroid functioning and reducing blood sugar levels. B. But remember to consult a specialist while deciding the dosage and duration of continued intake. C. It relaxes the nervous system and lowers cortisol (stress hormone) levels in the blood, thus making you feel calm and relaxed overall. D. You can consume Ashwagandha powder or Ashwagandha capsules. (a) ADBC (b) ADCB (c) ACDB (d) DABC SSC CGL (Tier-I) 25/07/2023 (Shift-IV) Ans. (c) : The correct jumbled order of the above given sentences is option (c) 'A C D B.' The other options are not correct. 10. Sentences of a paragraph are given below in jumbled order. Arrange the sentences in the correct order to form a meaningful and coherent paragraph. A. Louis was enthralled by the concept of a raised dots system. B. He made up his mind to use the technique to represent an alphabetic code.

861

YCT

C. Blind persons could read large-letter books 13. that are bulky with the aid of this alphabet code. D. Additionally, he developed the six-dot cell A. technology, which was at his fingertips. (a) DCAB (b) ABCD B. (c) BCDA (d) CDBA SSC CGL (Tier-I) 25/07/2023 (Shift-IV) Ans. (b) : The correct jumbled order of the above given sentences is option (b) 'ABCD'. 11.

Sentences of a paragraph are given below in jumbled order. Arrange the sentences in the correct order to form a meaningful and coherent paragraph. A. But, while the court’s action solved one problem, it created another and because of the closure, many workers lost their jobs. B. For instance, the courts directed industries in residential areas in Delhi to close down or shift out of the city. C. In recent years, while the courts have come out with strong orders on environmental issues, these have sometimes affected people’s livelihoods adversely. D. Several of these industries were polluting the neighbourhood and discharge from these industries were polluting the river Yamuna because they had been set up without following the rules. (a) BDCA (b) CBDA (c) BCAD (d) CDAB SSC CGL (Tier-I) 18/07/2023 (Shift-III) Ans. (b) : The correct jumbled order of the above paragraphs is option (b) [CBDA]. Other options are not correct. 12. Sentences of a paragraph are given below in jumbled order. Arrange the sentences in the correct order to form a meaningful and coherent paragraph. A. Both, Ratan and Moti were idlers and did not work. B. Their wives had introduced order and industry in the house. C. Ratan and Moti’s wives would work hard and would earn the bread for their family. D. This made them lazier. (a) CDAB (b) ABDC (c) ACBD (d) BADC SSC CGL (Tier-I) 20/07/2023 (Shift-I) Ans. (c) : The correct Jumbled order of above sentences is option (c) [ACBD] because it gives correct sense for the arrangement. Parajumbles

Sentences of a paragraph are given below in jumbled order. Arrange the sentences in the correct order to form a meaningful and coherent paragraph. The first is the herbalist, who generally enjoys the prestige and reputation of being the real traditional medical professional. Over the years, I have come to distinguish three types of medical practitioners in African societies and to classify the extent to which each uses medicinal plants. C. Thirdly, the witch doctor, the practitioner who is credited with the ability to intercept the evil deeds of a witch. D. The second group represents the divine healers. (a) BDAC (b) CBAD (c) BADC (d) ACDB SSC CGL (Tier-I) 20/07/2023 (Shift-I) Ans. (c) : The correct Jumbled order of the given sentences is option (c) [BADC]. Other options are not correct. Thus, the correct sentence is option (c). 14. Identify the option that arranges the given parts iteh correct order to form a meaningful and coherent paragraph. (a) In Shakespeare's hands, English drama (b) that first shone fourth in his early history plays (c) Willaim Shakespeare s considered as teh greatest dramatist and poet of English language. (d) achieved a matchless brilliance (a) c, a, d, b (b) a, b, c, d (c) b, c, a, d (d) d, b, a, c SSC CGL (Tier-I) 27/07/2023 (Shift-III) Ans. (a) : The correct meaningful order of the given sentences is option (a) [c, a, d, b]. Other given options are not correct. Thus, the correct answer is option (a). 15. Sentences of a paragraph are given below in jumbled order. Arrange the sentences in the correct order to form a meaningful and coherent paragraph. A. He later tried to franchise his restaurant. B. Colonel Harland Sanders' real-life story of being disappointed numerious times in his life and still making his ambition come true late in life is truly motivating. C. He began selling chicken at teh age of 40, but his dream of opening a restuarant was repeatedly denied owing to conflicts and wars. D. He is a seventh-grade dropout who tried many things in life but found them bitter. (a) BDCA (b) BACD (c) BDAC (d) BCDA SSC CGL (Tier-I) 27/07/2023 (Shift-III) Ans. (a) : The correct jumbled order of the above given sentences is option (a) [BDCA]. Other options are not correct.

862

YCT

16.

Sentences of a paragraph are given below in jumbled order. Arrange the sentences in the correct order to form a meaningful and coherent paragraph. A. My grandmother always went to school with me because the school was attahced to the temple. B. When the both finished, we would be back together. C. While the children sat in rows on either side of the verandah singing the alphabet or prayer in chorus, my grandmother sat inside reading the scriptures. D. The priest taught us the alphabet and the morning prayer. (a) ADBC (b) ABDC (c) ACBD (d) ADCB SSC CGL (Tier-I) 27/07/2023 (Shift-III) Ans. (d) : The correct jumbled order of the above given sentences is option (d) [ADCB]. Other options are not correct. 17. Sentences of a paragraph are given below in jumbled order. Arrange the sentences in the correct order to form a meaningful and coherent paragraph. A. In Bihar and Central India, in particular, every district had smelters that used local deposits of ore to produce iron which was widely used for the manufacture of implements and tools of daily use. B. The smelting was done by men while women worked on the bellows, pumping air that kept the charcoal burning. C. But iron smelting in India was extremely common till the end of the nineteenth century. D. Production of Wootz steel required a highly specialised technique of refining iron. (a) ACDB (b) DCAB (c) BCDA (d) CBDA SSC CGL (Tier-I) 14/07/2023 (Shift-I) Ans. (b) : The correct order of above jumbled sentences is DCAB. 18. Sentences of a paragraph are given below in jumbled order. Arrange the sentences in the correct order to form a meaningful and coherent paragraph. A. He repeated the experiment, increasing the amount until he had weighed up to a thousand pounds. B. During the spring of 1717, the iron foundries in a remote district were often visited by a thin, middle-aged man with a notebook. C. Three cauldrons were next prepared under his directions. D. He would weigh out two pounds of iron, have them heated till they were red-hot and then weigh them again. (a) CABD (b) BDAC (c) ABDC (d) BCAD SSC CGL (Tier-I) 14/07/2023 (Shift-I) Parajumbles

Ans. (b) The correct order of above jumble sentence is BDAC. 19. Sentences of a paragraph are given below in jumbled order. Arrange the sentences in the correct order to form a meaningful and coherent paragraph. A. The top band is made of saffron, which symbolizes power and courage. B. The constituent assembly adopted our national flag, Tiranga, which means tricolor, on 22 july 1947. C. As a symbol of nationalism and freedom, it is fashioned from khadi, which is domestically spun Indian cotton. D. It features three horizontal stripes that are all the same width. (a) BDCA (b) BCDA (c) CBDA (d) CBAD SSC CGL (Tier-I) 14/07/2023 (Shift-I) Ans. (b) : The correct order of above jumbled sentences is BCDA. 20. Sentences of a paragraph are given below in jumbled order. Arrange the sentences in the correct order to form a meaningful and coherent paragraph. A. According to Homer, Achilles was brought up by his mother at Phthia with his inseparable companion Patroclus. B. Achilles was the bravest, handsomest and greatest warrior of the army of Agamemnon in the Trojan War. C. Later non-Homeric tales suggest that Patroclus was Achilles' kinsman or lover. D. Achilles, in Greek mythology, is the son of the mortal Peleus, king of the Myrmidons, and the Nereid Thetis. (a) DABC (b) DBAC (c) CABD (d) ABCD SSC CGL (Tier-I) 17/07/2023 (Shift-II) Ans. (b) : The correct order of above jumbled sentences is DBAC. 21. Sentences of a paragraph are given below in jumbled order. Arrange the sentences in the correct order to form a meaningful and coherent paragraph. A. We recognise some aspects of an SF narrative with the realities of this world and some elements are estranged from the reality so as to take us to an unreal world. B. Therefore, it is often suggested that cognitive estrangement’is a necessary means through which the SF is usually understood. C. Science Fiction is a genre which manifests in itself the use of scientific premises often in a fantastical manner in a narrative. D. It is highly seeped into the reality of this world. (a) CDBA (b) ADBC (c) BCAD (d) DACB SSC CGL (Tier-I) 24/07/2023 (Shift-III)

863

YCT

Ans. (a) : The correct order of above jumbled sentences will be CDBA. 22. Sentences of a paragraph are given below in jumbled order. Arrange the sentences in the correct order to form a meaningful and coherent paragraph. A. The Olympic Games have come to be regarded as the world’s foremost sports competition. B. Olympic Games is an athletic festival that originated in ancient Greece and was revived in the late 19th century. C. Before the 1970s, the Games were officially limited to competitors with amateur status, but in the 1980s many events were opened to professional athletes. D. Currently, the Games are open to all, even the top professional athletes in basketball and football (soccer). (a) BCDA (b) CADB (c) BDCA (d) ABDC SSC CGL (Tier-I) 24/07/2023 (Shift-III) Ans. (a) : The correct sequence of given jumbled sentences is BCDA. 23. Parts of a sentence are given below in jumbled order. Arrange the parts in the correct order to form a meaningful sentence. (O) on a regular basis can improve (P) engaging in physical activity (Q) the risk of chronic diseases, and boost mental health (R) cardiovascular health, reduce (a) OQRP (b) RPOQ (c) PORQ (d) PROQ SSC CHSL (Tier-I) 17/08/2023 (Shift-II) Ans. (c) : The correct sequence of jumbled sentences in a meaningful way is option (c) PORQ. Arranged Sentence Engaging in physical activity on a regular basis can improve cardiovascular health, reduce the risk of chronic diseases, and boost mental health. 24. Parts of a sentence are given below in jumbled order. Select the option that arranges the parts in the correct order to form a meaningful sentence. A) to his neighbors B) first name is Adam C) that no one even D) he is so rude E) knows that his (a) D, A, C, E, B (b) C, E, B, A, D (c) B, D, A, C, E (d) A, C, B, D, E SSC CHSL (Tier-I) 17/08/2023 (Shift-II) Ans. (a) : The correct order of jumbled sentences is option (a) D, A, C, E, B Correct sentence – He is so rude to his neighbors that no one even knows that his first name is Adam. Parajumbles

25.

Parts of a sentence are given below in jumbled order. Arrange the parts in the correct order to form a meaningful sentence. A. and he shone like a beacon B. but also for the whole world C. our country gave birth to a mighty soul D. not only for India (a) ADBC (b) ACDB (c) DBCA (d) CADB SSC CHSL (Tier-I) 10/08/2023 (Shift-I) Ans. (d) : The correct sequence of jumbled sentence is option (d) CADB. Arranged Sentence Our country gave birth to a mighty soul and he shone like a beacon not only for India but also for the whole word. 26. Parts of a sentence are given below in jumbled order. Select the option that arranges the parts in the correct order to form a meaningful sentence. A. By conquest, rather than by economic contract B. Other types of feudalism was the relation C. The principal difference between Indian and D. Between lord and vassal, established in this case (a) B, A, D, C (b) B, D, A, C (c) C, B, D, A (d) C, B, A, D SSC CHSL (Tier-I) 10/08/2023 (Shift-I) Ans. (c) : The jumbled sentences can be arranged in the correct order mentioned in option (c) C, B, D, A. Hence option (c) correct answer. 27. Parts of a sentence are given below in jumbled order. Select the option that arranges the parts in the correct order to form a meaningful sentence. A) said that they are B) hear this news C) very sorry to D) Catherine and David (a) D, A, C, B (b) A, C, D, B (c) C, D, B, A (d) B, D, C, A SSC CHSL (Tier-I) 09/08/2023 (Shift-III) Ans. (a) : The correct order of jumbled sentences after being arranged is option (a) D, A, C, B. Arranged Sentence Catharine and David said that they are very sorry to hear this never. 28. Parts of a sentence are given below in jumbled order, except the first part. Arrange the parts in the correct order to form a meaningful sentence. It is… A. who are continually trying to improve B. not the least talented C. their dialogue skills, D. the most talented people (a) BADC (b) ABDC (c) CDBA (d) DACB SSC CHSL (Tier-I) 09/08/2023 (Shift-III)

864

YCT

Ans. (d) : The Correct order of the given jumbled sentence to form a meaningful sentence will be option (d) DACB. 29. Parts of a sentence are given below in jumbled order, except the first part. Arrange the parts in the correct order to form a meaningful sentence. Such poststructuralist pronouncements… A. not only in the Anglo-American academia B. on the literature of the Third World, C. however, are a common feature of the literary theory practiced D. but also in the Third-World countries (a) BCAD (b) CABD (c) ABCD (d) ADCB SSC CHSL (Tier-I) 02/08/2023 (Shift-I) Ans. (a) : The correct jumbled order of the above given sentence is option (a) [B C A D]. Other given options are not correct. 30. Parts of a sentence are given below in jumbled order. Arrange the parts in the correct order to form a meaningful sentence. A. fell before me B. a carcass of an animal C. when suddenly D. my heart skipped a beat (a) BACD (b) CDBA (c) ABCD (d) DCBA SSC CHSL (Tier-I) 02/08/2023 (Shift-I) Ans. (d) : The correct jumbled order of the above given sentences is option (d) [DCBA]. 31. Parts of a sentence are given below in jumbled order. Select the option that arranges the parts in the correct order to form a meaningful sentence. A. In accordance with hierarchical status B. Of the services rendered C. Rewards are given D. And are not related to the economic value (a) C, B, A, D (b) C, A, D, B (c) A, D, B, C (d) A, B, D, C SSC CHSL (Tier-I) 02/08/2023 (Shift-I) Ans. (b) : The correct jumbled order of the above given sentences is option (b) [C, A, D, B] Other given options are not correct. 32. Sentences of a paragraph are given below in jumbled order. Arrange the sentences in the correct order to form a meaningful and coherent paragraph. A) So then why are not Silicon Valley entrepreneurs clamouring to decode the ancient wisdom? B) The failure rate in finding a new drug is as high as 95%. C) Because these once-thriving cultures are now reduced to an endangered tribe. D) According to industry data, it takes $2.6 billion and, on average, fourteen years to develop a new drug. Parajumbles

(a) ACDB (b) BADC (c) DBAC (d) DCBA SSC CHSL (Tier-I) 02/08/2023 (Shift-I) Ans. (c) : The correct jumbled order of the above sentences is option (c) [DBAC]. Thus, the correct answer is option (c). 33. Parts of a sentence are given below in jumbled order. Arrange the parts in the correct order to form a meaningful sentence. P. the news of her husband's death Q. to break yo her as gently as possible R. great care was taken S. knowing that Mrs. Mallard was affilicted with a heart trouble (a) SPRQ (b) SPQR (c) SRQP (d) SQPR SSC CHSL (Tier-I) 14/08/2023 (Shift-IV) Ans. (c) : The correct order of jumbled sentences after being arranged is option (c) SRQP. 34. Sentences of a paragraph are given below in jumbled order. Select the option that arranges the sentences in the correct logical sequence to form a meaningful and coherent paragraph. A. I once met a gentlemen who told me about a game he plays with his grandchildren. B. He shares a titbit or two and it starts off a conversation. C. He then asks his teenage gradchildren if they know who those people are. D. He picks some names of politicians, rock stars or even criminals from his generation. (a) ACBD (b) BACD (c) ABCD (d) ADCB SSC CHSL (Tier-I) 14/08/2023 (Shift-IV) Ans. (d) : The correct sequence of jumbled sentences is option (d) ADCB. 35. Part of the sentence are given below in jumbled order. Arrange the parts in the correct order to form a meaningful sentence. P. and allows them to come out of their imaginary world Q. which helps in illuminating and enlightening the dark world R. of two blind or visually impaired persons S. eye donation is a very generous act towards society (a) SQRP (b) SRQP (c) SPQR (d) SQPR SSC CHSL (Tier-I) 14/08/2023 (Shift-IV) Ans. (a) : The correct order of jumbled sentences is option (a) SQRP. 36. Parts of a sentence are given below in jumbled order. Select the option that arranges the parts in the correct order to form a meaningful sentence. A. That the only true individual B. Is the holy man C. It has frequently been claimed D. Which the traditional society of India tolerates

865

YCT

(a) D, C, A, B (b) A, B, D, C (c) C, D, A, B (d) C, A, D, B SSC CHSL (Tier-I) 08/08/2023 (Shift-II) Ans. (d) : The correct jumbled order of the above given sentences is option (d) [C, A, D, B] Other options are not correct. 37. Sentences of a paragraph are given below in jumbled order. Select the option that arranges the sentences in the correct order to form a meaningful and coherent paragraph. A. The post-COVID period has caused many asymptomatic diseases. B. Though it cannot be believed for sure; the change in social life may be a major cause. C. These are people who suffered infection once or never at all. D. Some propose that survival post infection has changed the mindset of people. (a) A, C, D, B (b) B, D, C, A (c) B, D, A, C (d) A, C, B, D SSC CHSL (Tier-I) 08/08/2023 (Shift-II) Ans. (a) : The correct jumbled order of the above given sentences is option (a) [A, C, D, B]. Other given options are not correct. 38. Parts of a sentence are given below in jumbled order. Arrange the parts in the correct order to form a meaningful sentence. (P) and the impact of their (Q) the authors’ journey of self-discovery (R) the memoir explores (S) upbringing on their identity (a) SPRQ (b) RQPS (c) QPSR (d) RQSP SSC CHSL (Tier-I) 08/08/2023 (Shift-II) Ans. (b) : The correct jumbled order of the above given sentences is option (b) [RQPS]. Other given options are not correct. 39. Parts of a sentence are given below in jumbled order. Arrange the parts in the correct order to form a meaningful sentence. P. that a glance from the window seemed simply to prove Q. with such dignity of motion R. the great Pullman was whirling onward S. that the plains of Texas were pouring eastward (a) RQPS (b) PRQS (c) RPQS (d) RQSP SSC CHSL (Tier-I) 04/08/2023 (Shift-III) Ans. (a) : The correct jumbled order of the above sentences is option (a) [R Q P S]. Other given options are not correct. 40. Parts of a sentence are given below in jumbled order. Select the option that arranges the parts in the correct order to form a meaningful sentence. A) based on the B) we live in C) nation that D) our nationality is Parajumbles

(a) C, D, A, B (b) D, A, C, B (c) A, B, C, D (d) B, C, A, D SSC CHSL (Tier-I) 04/08/2023 (Shift-III) Ans. (b) : The correct jumbled order of the above given sentences is option (b) [D, A, C, B]. Other given options are not correct. Correct order – 'Our nationality is based on the nation that we live in.' 41. Sentences of a paragraph are given below in jumbled order. Arrange the sentences in the correct order to form a meaningful and coherent paragraph. A. It included a carbon tax on fossil fuels and incentives for businesses and individuals to adopt green practices. B. The new policy aimed to reduce carbon emissions and promote renewable energy sources. C. However, supporters of the policy argued that it was necessary to address the urgent threat of climate change and create a sustainable future for the planet. D. The policy faced opposition from some industries and political groups who argued that it would hurt the economy. (a) DABC (b) BADC (c) DACB (d) BCAD SSC CHSL (Tier-I) 03/08/2023 (Shift-II) Ans. (b) : The correct jumbled order of the above given sentences is option (b) [BADC]. Other options give different meanings. 42. Rearrange the parts of the sentence in correct order. P. bendable electronics and sensing technologies research Q. people using “aerohaptics”, creating feelings of touch with jets of air R. my colleagues and I working in the University of Glasgow’s S. group have now developed a system of holograms of (a) QPSR (b) RPSQ (c) PQRS (d) SRQP SSC CHSL (Tier-I) 21/03/2023 (Shift-IV) Ans. (b) : The appropriate sequence of above Jumblled sentences will be RPSQ. 43. Given below are four jumbled sentences. Pick the option that gives their correct order. P. In july 1977, Marley was found to have a type of malignant melanoma (cancer) under the nail of a toe. Q. He diet on 11 May 1981 at Cedars of Lebanon hospital in Miami (now niversity of Miami Hospital) at the age of 36. R. Despite his illness, he contined torng until his health deteriorated as the cancer had spread throughout his body. S. Marley turned down his doctors' advice to have his toe amputated, citing his religious beliefs.

866

YCT

(a) PSRQ (b) PQSR (c) RSQP (d) PSQR SSC CHSL (Tier-I) 21/03/2023 (Shift-II) Ans. (a) : For the above jumbled sentence, option (a) PSRQ is appropriate sequence. Other options are not correct. 44. Rearrange the parts of the sentence in correct order. Some algal bloms P. beneficial to aquatic life, since these tiny Q. nutrient conditions are optimal, and they can be R. occur naturally when temperature, water, and S. organisms form the base of the food chain (a) RQPS (b) PRSQ (c) QPRS (d) RQSP SSC CHSL (Tier-I) 21/03/2023 (Shift-II) Ans. (a) : For the above jumbled sentences, option (a) RQPS is correct sequence. Other options are not correct. 45. Rearrange the parts of the sentence in correct order. P. undermining the strict application of comparative anthropology Q. in the early 20th century it was almost niversally assumed R. that traditional cultures world be completely assimilated S. into colonial cultures within a few decades, thus (a) PQRS (b) PSQR (c) QPRS (d) QRSP SSC CHSL (Tier-I) 21/03/2023 (Shift-II) Ans. (d) : For the above jumbled sentences, option (d) QRSP is correct sequence. Other options are not correct. 46. Some parts of a few sentences have been jumbled up, and labeled A, B, C and D. Select the option that gives the correct sequence in which these parts can be rearranged to form a meaningful and grammatically correct sentence. My years in my father’s house A. Would have been unbearable had B. The feel of his hand clutching mine, his C. I not had my brother. I never forgot D. Refusal to abandon me. Perhaps he and I would have been close even otherwise. (a) ACBD (b) DBCA (c) CBDA (d) BCDA SSC CHSL (Tier-I) 20/03/2023 (Shift-I) Ans. (a) : For the above sentence Jumbled part, option (a) ACBD is correct order. Other options are not correct. 47. Some parts of one or more sentences have been fumbled up, and labeled A, B, C and D. Select the option that haves the correct sequence in which these parts can be rearranged to form a meaningful and grammatically correct sentence. Their approach sensitized Parajumbles

of it. Similarly, behind them, in the halls, flicked on when they came within ten feet a a switch somewhere and the nursery light lights went on and off as they them behind, with a soft automaticity. (a) DCBA (b) CBAD (c) BADC (d) BCAD SSC CHSL (Tier-I) 20/03/2023 (Shift-I) Ans. (b) : Correct order of above Jumbled parts is option (b) CBAD. Other options are not correct. 48. Rearrange the parts of the sentence in correct order. P. on the US naval base, close to the prison compounds Q. housing the remaining 39 detainees held in the "war on terror" R. inviting tenders for private contractors to run the Migrant Operations Center S. the Immigration and Customs Enforcement (ICE) bureau is (a) RPQS (b) QSRP (c) SPQR (d) SRPQ SSC CHSL (Tier-I) 17/03/2023 (Shift-IV) Ans. (d) : Appropriate order for the above jumbled sentences is option (d) SRPQ. Other options are not correct. 49. Rearrange the parts of the sentence in correct order. It could be P. became a byword for superhuman intellectual ability Q. argued that 1919 was the year when Einstein's name R. making possible the small industry of S. Einstein-themed merchandise that still exists today (a) QPRS (b) PSQR (c) RSPQ (d) PQRS SSC CHSL (Tier-I) 17/03/2023 (Shift-III) Ans. (a) : An Appropriate sequence of above Jumbled Sentences will be QPRS. 50. Given below are four jumbled sentences. Pick the option that gives their correct order. P. The earliest years of a child's life are critical. Q. This is why an establishment has been created for the protection of children in these crucial years, known as ECCE. R. It is during the early years that children develop the cognitive, physical, social and emotional skills that they need to succeed in life. S. These years determine child's survival and thriving in life, and lay the foundations for her/his learning and holistic development. (a) RSPQ (b) PSRQ (c) PSQR (d) QPSR SSC CHSL (Tier-I) 17/03/2023 (Shift-III) Ans. (b) : An appropriate sequence of above Jumbled Sentences will be PSRQ.

867

A. B. C. D.

YCT

51.

Rearrange the parts of the sentence in correct order. The federal government P. currently provides disaster relief for direct impacts Q. of volcanic eruptions and other natural disasters, R. but limited or no assistance for the indirect effects S. experienced from long periods of volcanic unrest (a) QRPS (b) PQRS (c) RQPS (d) SPQR SSC CHSL (Tier-I) 15/03/2023 (Shift-III) Ans. (b) : An appropriate sequence of the above jumbled Sentence will be PQRS. 52. Given below are four jumbled sentences. Pick the option that given their correct order. P. Fasting also leads to a feeling of rejuvenation and extended life expectancy. Q. This might be due to the detoxification effect of fasting. R. A study was performed on earthworms that showed the extension of life thanks to fasting. S. The experiment was performed in the 1930s by isolating one worm an putting it on a cycle of fasting and feeding. The isolated worm outlived the other worms by 19 generations, while still maintaining its freshness and youthful physiological characteristics. (a) PQSR (b) PRSQ (c) PQRS (d) PSRQ SSC CHSL (Tier-I) 15/03/2023 (Shift-III) Ans. (c) : An appropriate sequence of above jumbled Sentence will be PQRS. 53. Given below are four jumbled sentences. Pick the option that gives their correct order. The immediate context of the Dunkirk evacuation was Germany's invasion of the Low Countries and northern France in May 1940. P. The Dutch defenders fell back westward, and by noon on May 12 German tanks were on the outskirts of Rotterdam. Q. Queen Wilhelmina and her government left the country for England on May 13 and the next day the Dutch army surrendered to the Germans. R. On May 10 the German blitzkrieg attack on the Netherlands began with the capture by parachutists of key bridges deep within the country, with the aim of opening the way for mobile ground forces. S. The invasion of Belgium also began on May 10, when German airborne troops landed on the fortress of Eben Emael, immediately opposite Maastricht, and on bridges over the Albert Canal. On May 11 the Belgian front was broken and German tanks ran on westward while Belgian, French and British divisions fell back to a line between Antwerp and Namur. Parajumbles

(a) PSRQ (b) RPQS (c) PRSQ (d) QPRS SSC CHSL (Tier-I) 15/03/2023 (Shift-I) Ans. (b) : An appropriate sequence of Jumble sentence will be RPQS. 54. Rearrange the parts of the sentence in correct order. The team plans to P. flowering the fruit production in apple trees Q. use the data to get a better understanding of R. health-promoting compounds of interest and employ S. biotechnological approaches that could speed up (a) QRSP (b) PQRS (c) RSPQ (d) QSPR SSC CHSL (Tier-I) 15/03/2023 (Shift-I) Ans. (a) : An appropriate sequence of Jumble sentence will be QRSP. 55. Rearrange the parts of the sentence in correct order. P. more stringent mobility control and physical distancing policies Q. East Asian governments (except for Japan) implemented R. as well as more comprehensive testing, tracing S. and isolation policies since the early stages (a) PSRQ (b) QPRS (c) PRSQ (d) QSPR SSC CHSL (Tier-I) 10/03/2023 (Shift-III) Ans. (b) : An appropriate sequence of the above jumbled sentence will be QPRS. 56. Some parts of one or more sentences have been jumbled up, and labelled A, B, C and D. Select the option that gives the correct sequence in which these parts can be rearranged to form a meaningful and grammatically correct sentence. At Paris, just after dark A. of meditation and a meerschaum, in company B. one gusty evening in the autumn C. of 18, I was enjoying the two fold luxury D. with my friend Auguste Dupin, in his little back library (a) CDBA (b) ADBC (c) BCAD (d) DCBA SSC CHSL (Tier-I) 10/03/2023 (Shift-III) Ans. (c) : An appropriate sequence of above Jumbled sentence will be BCAD. 57. Rearrange the parts of the sentence in correct order. P. plastics, and not available in sufficient quantities in India Q. rather one for specific, targeted applications as a majority of R. compostable plastics are more expensive than conventional S. compostable plastic packaging is not a blanket solution, but

868

YCT

(a) SQRP (b) RPQS 61. Select the option that arranges the given sentence in the correct sequence. The first (c) PQRS (d) SRPQ sentence is in the correct position. SSC CHSL (Tier-I) 09/03/2023 (Shift-IV) A. Mrs. Salmon who lived in 15, Northwood Street Ans. (a) : An appropriate sequence of the above had been unable to sleep. jumbled sentence will be SQRP . B. He had a hammer in his hand and had just 58. Rearrange the parts of the sentence in correct come out of her house. order. C. But when she went to the window she saw Adams standing on the steps of Mrs. Parker's P. fish, or other potential food item that happen house. Q. to be floating by with stinging cells called D. She heard a door click shut and thought it was nematocysts her own gate. R. at night and capture tiny organisms like (a) ABCD (b) ADCB zooplankton, small (c) ADBC (d) ACBD S. corals are predators and they extend their SSC Stenographer (Grade C & D) 12.11.2021 Shift-I tentacles Ans. (b) : Appropriate sequence of the above jumbled (a) RSPQ (b) RPQS sentences will be ADCB. (c) PQRS (d) SRPQ SSC CHSL (Tier-I) 09/02/2023 (Shift-II) 62. Given here are four sentences. The first sentence is in correct order, while the rest of Ans. (d) : Appropriate sequence of the above jumbled the sentences are jumbled. Pick the option that sentences will be SRPQ. gives their correct order. 59. Given here are four sentences. The first A. I sat in a police car for the first time in my life. sentence is in correct order, while the rest of B. I reached home and sat on the sofa. the sentence are jumbled. Pick the option that C. I swiped my Metro-card and took the train to gives their correct order. Chambers Street. A. Puller sat back and closed his eyes. D. The inspector dropped me at the Clark Street B. In his mind's eye he took a trip through Fort subway stop. (a) ACBD (b) ADCB Leavenworth, a place he had often been to in (c) ADBC (d) ABCD his career. C. He opened his eyes. SSC Stenographer (Grade C & D) 12.11.2021 Shift-I D. His photographic memory clicking away, Ans. (b) : Appropriate sequence of the above jumbled Puller kept up his mental scroll until he came sentences will be ADCB. to the answer. 63. Given here are four sentences. The first (a) ABDC (b) ACBD sentence is in correct order, while the rest of (c) ADBC (d) ABCD the sentence are jumbled. Pick the option that SSC Stenographer (Grade C & D) 12.11.2021 Shift-I gives their correct order. A. Tilly saw the sea slowly rise, and start to foam Ans. (a) : Appropriate sequence of the above jumbled and bubble. sentences will be ABDC. B. Her Geography teacher had shown her class 60. Select the option that arranges the given the video. sentence in the correct sequence. The first C. She had explained that tsunamis can be caused sentence is in the correct position. by earthquakes, landslides and volcanoes. A. Most large companies these days employ D. She remembered that she had seen this in a trained psychologists. video of a tsunami. B. Later, the psychologists also develop such (a) ABDC (b) ADBC models for lower level positions in the (c) ACBD (d) ABCD company. SSC Stenographer (Grade C & D) 12.11.2021 Shift-I C. To create these competency models, the psychologists ask senior managers to identify Ans. (b) : Appropriate sequence of the above jumbled the capabilities that are typically needed for sentences will be ADBC. such role. 64. Given below are four sentences in jumbled D. Their job is to develop 'competency models' to order. Pick the option that gives their correct aid the companies in identifying, training and order. promoting likely managers who could take on A. An allergy is a reaction of the immune system leadership roles. to a substance which is normally harmless. (a) ABDC (b) ACBD B. These antibodies then attach themselves to (c) ADCB (d) ADBC cells, which contain histamine, which inflames SSC Stenographer (Grade C & D) 12.11.2021 Shift-I the tissues. Ans. (c) : Appropriate sequence of the above jumbled C. One-third of us are affected by an allergy at sentences will be ADCB. some point in our lives. Parajumbles

869

YCT

Given below are four jumbled sentence. Select the option that gives their correct order forming a meaningful and coherent paragraph. A. He had agreed to feed him, pay him twelve rupees a month, and find him somewhere to stay. B. A month's salary was paid in advance. C. Fotik had learned to handle his job quite well in the last five days. D. Upen Babu was a good man which had helped a lot. (a) ADBC (b) CDAB (c) CBDA (d) DACB SSC CGL (Tier-II) 08.08.2022 Shift-II Ans. (b) : Appropriate sequence of the above jumbled sentences will be CDAB. 69. Given below are four sentences in jumbled order. Pick the option that gives their correct order. A. We have flower arrangements to suit everybody at all prices. B. We also make paying easy, and we accept all major credit cards at our shops or you can pay online. C. Whether it is birthdays weddings, or even the sad times like funerals, Fancy Flowers is there to make it easy. D. Established in 2002, Fancy Flowers has branches all over the country. (a) CADB (b) ADBC (c) DBCA (d) DACB SSC CGL (Tier-II) 08.08.2022 Shift-II Ans. (d) : Appropriate sequence of the above jumbled sentences will be DACB. 70. Given below are four jumbled sentences. Out of the given options, pick the one that gives their correct order. A. Susan Reynolds slipped into her car after putting her shopping bags in the trunk. B. She reached her house and carried the shopping bags in. C. It was a thirty-minute ride to her home and the traffic was light. D. She put them down and turned off the alarm. (a) ABDC (b) ACBD (c) ABCD (d) ADCB SSC Stenographer (Grade C & D) 11.11.2021 Shift-II Ans. (b) : Appropriate sequence of the above jumbled sentences will be ACBD. 71. Given here are four sentences. The first sentence is in correct order, while the rest of the sentences are jumbled. Pick the option that gives their correct order. A. At last I reached my uncle's beach house. B. I was determined to enjoy my stay thoroughly. C. But, somehow, I had forgotten to pack any Ans. (d) : Appropriate sequence of the above jumbled sunscreen. sentences will be BDAC. D. So, I had packed everything I might need.

D. The immune system in allergy sufferers makes antibodies against harmless substances because it believes them to be dangerous. (a) DCAB (b) CBDA (c) CADB (d) ACBD SSC CGL (Tier-II) 08.08.2022 Shift-II Ans. (c) : Appropriate sequence of the above jumbled sentences will be CADB. 65. Given below are four jumble sentences. Select the option that give their correct order forming a meaningful and coherent paragraph. A. He went up to the king, bowed and presented a velvet case. B. The king invited the men to make their presentations. C. The court-room was packed with people. D. Krishna Kumar was the first to come forward. (a) CDAB (b) DACB (c) CBDA (d) ADBC SSC CGL (Tier-II) 08.08.2022 Shift-II Ans. (c) : Appropriate sequence of the above jumbled sentences will be CBDA. 66. Given below are four sentences in jumbled order. Pick the option that gives their correct order. A. Another advantage is that we can have an opportunity to learn to speak a new language fluently. B. The main advantage of living somewhere else is that you get the opportunity to experience new cultures. C. Of course, living away from home has its disadvantage too, such as being away from friends and family for long periods of time. D. Should we live in one place for all our life or experience living in different places? (a) DBAC (b) DCAB (c) BCAD (d) BDAC SSC CGL (Tier-II) 08.08.2022 Shift-II Ans. (a) : Appropriate sequence of the above jumbled sentences will be DBAC. 67. Given below are four jumbled sentences. Select the option that gives their correct order forming a meaningful and coherent paragraph. A. In 1943, sudden retrenchment from office due to the war cost him his nine year old job. B. In 1934, Patol Babu gave up his factory job and came to Calcutta with his wife. C. Ever since, Patol Babu had sruggled to make a living. D. The sailing was smooth for some years and Patol Babu was in his boss's good books. (a) ADBC (b) CBDA (c) DACB (d) BDAC SSC CGL (Tier-II) 08.08.2022 Shift-II

Parajumbles

68.

870

YCT

(a) ACDB (b) ABDC (c) ADBC (d) ACBD SSC Stenographer (Grade C & D) 11.11.2021 Shift-II Ans. (b) : Appropriate sequence of the above jumbled sentences will be ABDC. 72. Given here are four sentences. The first sentence is in correct order, while the rest of the sentences are jumbled. Pick the option that gives their correct order. A. The computer company was in a crisis as its sales and profits were falling. B. He immediately set to work chopping jobs and taking other tough decisions. C. Soon things started changingand the company was saved, at least in the short term. D. The board brought in a new CEO with many years of experience who began managing the crisis. (a) ADBC (b) ADCB (c) ABCD (d) ACBD SSC Stenographer (Grade C & D) 11.11.2021 Shift-II Ans. (a) : Appropriate sequence of the above jumbled sentences will be ADBC. 73. Given here are four sentences. The first sentence is in correct order, while the rest of the sentences are jumbled. Pick the option that gives their correct order. A. There are two ways of getting rid of weeds in your garden – the easy way and the not-so-easy way. B. The easy way may be to run a lawn-mower and the lawn looks fine for a while. C. It is time consuming and painful, but the weeds will stay away for a longer time. D. But the not-so-easy way may mean getting down on your knees and pulling out the weeds by the roots. (a) ADBC (b) ACDB (c) ABCD (d) ABDC SSC Stenographer (Grade C & D) 11.11.2021 Shift-II Ans. (d) : Appropriate sequence of the above jumbled sentences will be ABDC. 74. Given here are four sentences. The first sentence is in correct order, while the rest of the sentences are jumbled. Pick the option that gives their correct order. A. We make a lot of choices in our daily life. B. Living on the edge may be exciting but not always the right thing to do. C. These determine either our safety or put us in danger. D. Therefore, following limits is always the safer and better way of life. (a) ACBD (b) ABCD (c) ADBC (d) ABDC SSC Stenographer (Grade C & D) 11.11.2021 Shift-II Ans. (a) : Appropriate sequence of the above jumbled sentences will be ACBD. Parajumbles

75.

Given here are four sentences. The first sentence is in correct order, while the rest of the sentences are jumbled. Pick the option that gives their correct order. A. Raja Ravi Verma was a famous Indian painter. B. He was also known for making affordable prints of his paintings available to the public. C. His works were a fusion of European art and Indian sensibility. D. This enhanced the interest of public in fine arts. (a) ADCB (b) ABCD (c) ADBC (d) ACBD SSC Stenographer (Grade C & D) 11.11.2021 Shift-I Ans. (a) : Appropriate sequence of the above jumbled sentences will be ADCB.

76.

Given here are four sentences. The first sentence is in correct order, while the rest of the sentences are jumbled. Pick the option that gives their correct order. A. The Rajasthani men's style of dressing is very classical. B. How the Pagri is tied varies according to the region the people belong to. C. It is commonly known as 'pagri' or 'safa'. D. The turban forms the most traditional form of their dress. (a) ABCD (b) ACBD (c) ADCB (d) ACDB SSC Stenographer (Grade C & D) 11.11.2021 Shift-I Ans. (c) : Appropriate sequence of the above jumbled sentences will be ADCB.

77.

Given here are four sentences. The first sentence is in correct order, while the rest of the sentences are jumbled. Pick the option that gives their correct order. A. Thick socks and tight shoes are worn in winters. B. They also leave no space to move about in. C. Hence, the skink dries up and tears. D. These prevent the feet from receiving enough air. (a) ABDC (b) ACBD (c) ADBC (d) ACDB SSC Stenographer (Grade C & D) 11.11.2021 Shift-I Ans. (c) : Appropriate sequence of the above jumbled sentences will be ADBC.

78.

871

Select the option that arranges the given sentences in the correct sequence. The first sentence is in the correct position. A. At a school function, after the school prayer, the Principal welcomes the chief guest. B. This is followed by a variety entertainment programme by the school children. C. The function comes to a end with the national anthem. YCT

D. Then, the chief guest gives his speech and distributes prizes to students. E. The assistant principal then proposes the vote of thanks. (a) ABEDC (b) ABDEC (c) AEDCB (d) ACBED SSC Stenographer (Grade C & D) 11.11.2021 Shift-I Ans. (b) : Appropriate sequence of the above jumbled sentences will be ABDEC. 79. Given here are four sentence. The first sentence is in correct order, while the rest of the sentences are jumbled. Pick the option that gives their correct order. A. He dipped his fingers in the milk and thrust them into the fawn's soft wet mouth. B. So, he dipped his fingers again and let the fawn suck them. C. When he withdrew them, the fawn bleated frantically. D. It sucked greedily. (a) ABCD (b) ABDC (c) ADCB (d) ACBD SSC Stenographer (Grade C & D) 11.11.2021 Shift-I Ans. (c) : Appropriate sequence of the above jumbled sentences will be ADCB. 80. Given here are four sentence. The first sentence is in correct order, while the rest of the sentence are jumbled. Pick the option that gives their correct order. A. On the last day of the training, I received an email from my training manager, Jane. B. I wondered if I had done anything wrong. C. She had called me to her office. D. Once I had skipped classes to watch a movie and a football game. (a) ADBC (b) ABCD (c) ACBD (d) ABDC SSC Stenographer (Grade C & D) 12.11.2021 Shift-II Ans. (c) : Appropriate sequence of the above jumbled sentences will be ACBD. 81. Given here are four sentences. The first sentence is in correct order, while the rest of the sentences are jumbled. Pick the option that gives their correct order. A. Following at the man's heels was a big native dog. B. The animal understood and was worried by the great cold and knew that this was no time for travelling. C. Having understood the danger of being out there in the cold better than the man, it started barking. D. It was a wolf dog, gray-coated and not noticeably different from the wild wolf. (a) ACBD (b) ADBC (c) ACDB (d) ABDC SSC Stenographer (Grade C & D) 12.11.2021 Shift-II Ans. (b) : Appropriate sequence of the above jumbled sentences will be ADBC. Parajumbles

82.

Select the option that arranges the given sentences in the correct sequence. The first sentence is in the correct position. A. A traffic policeman usually stands at the junction of roads. B. With these signs, he either stops people and vehicles, or allows them to pass by. C. He wears a white uniform and a pair of white gloves. D. Sometimes he stands on a high platform at the junction or under a flyover. E. In one hand he carries a red or green, lighted rod or a metal sign plate that says 'STOP'. (a) ACBED (b) ADCEB (c) ABCED (d) AECBD SSC Stenographer (Grade C & D) 12.11.2021 Shift-II Ans. (b) : Appropriate sequence of the above jumbled sentences will be ADCEB. 83. Given here are four sentences. The first sentence is in correct order, while the rest of the sentences are jumbled. Pick the option that gives their correct order. A. The Brahman used to teach pupils from time to time. B. This young man knew that the Brahmin was in possession of a certain spell. C. He had one pupil of whom he thought very highly. D. But he didn't know what the spell could do. (a) ADBC (b) ABCD (c) ABDC (d) ACBD SSC Stenographer (Grade C & D) 12.11.2021 Shift-II Ans. (d) : Appropriate sequence of the above jumbled sentences will be ACBD. 84. Select the option that arranges the given sentences in the correct sequence. The first sentence is in the correct position. A. An ancient sage was teaching his disciples the art of archery. B. The first disciple was then asked to describe what he saw. C. He asked his disciples to aim at the eye of the bird. D. He put a wooden bird as the target on the branch of a tree. (a) ADCB (b) ADBC (c) ABCD (d) ACBD SSC Stenographer (Grade C & D) 15.11.2021 Shift-II Ans. (a) : Appropriate sequence of the above jumbled sentences will be ADCB. 85.

872

Given here are four sentences. The first sentence is in correct order, while the rest of the sentences are jumbled. Pick the option that gives their correct order. A. World Environment Day is celebrated on 5 June every year by the UN. B. In 2019, more than 130 countries participated actively and supported it. YCT

C. It is the UN's most impactful campaign for spreading awareness to conserve our environment. D. Over the years this campaign has become the means for public participation. (a) ADCB (b) ADBC (c) ACDB (d) ACBD SSC Stenographer (Grade C & D) 15.11.2021 Shift-I Ans. (c) : Appropriate sequence of the above jumbled sentences will be ACDB. 86. Given here are four sentences. The first sentence is in correct order, while the rest of the sentences are jumbled. Pick the option that gives their correct order. A. Suddenly she discovered, in a black satin box, a superb diamond necklace and her heart throbbed with an immoderate desire. B. Her hands trembled as she took it. C. She fastened it round her throat, outside her high-necked waist, and was lost in ecstasy at her reflection in the mirror. D. Then she asked her friend, hesitating, filled with anxious doubt: "Will you lend me this?" (a) ACBD (b) ABDC (c) ABCD (d) ADBC SSC Stenographer (Grade C & D) 15.11.2021 Shift-I Ans. (c) : Appropriate sequence of the above jumbled sentences will be ABCD. 87. Sentences of a paragraph are given below in jumbled order. Arrange the sentences in the correct order to form a meaningful and coherent paragraph. A. The deeper you immerse it, the greater will be the difference in the water level. B. This difference in the levels in the U-tube is a measure of the pressure applied by your palm. C. If you immerse the funnel into a bucket of water, you will again see the difference in the water level in the U-tube. D. When you gently apply pressure on the rubber film with your palm, you will find that a difference in the water level is created in the manometer. (a) DBCA (b) CBAD (c) BACD (d) ABCD SSC CHSL 30.05.2022 Shift-I Ans. (a) : Appropriate sequence of the above jumbled sentences will be DBCA. 88. Parts of a sentence are given below in jumbled order. Arrange the parts in the correct order to form a meaningful sentence. (A) is credited with saying (B) do not dry your feet (C) If you want to leave your footprint on the sands of time, (D) Dr. APJ Abdul Kalam, former President of India. (a) D, A, C, B (b) D, C, A, B (c) C, A, B, D (d) A, B, C, D SSC CHSL 24.05.2022 Shift-III Parajumbles

Ans. (a) : Appropriate sequence of the above jumbled sentences will be DACB. 89. Sentences of a paragraph are given below in jumbled order. Arrange the sentences in the correct order to form a meaningful and coherent paragraph. A. In 1895, a man named Birsa was seen roaming the forests and villages of Chottanagpur in Jharkhand. B. Soon thousands began following Birsa, believing that he was God and had come to solve all their problems. C. Birsa himself declared that God had appointed him to save his people from trouble, free them from the slavery of ‘dikus’. D. People said he had miraculous powers – he could cure all diseases and multiply grain. (a) CABD (b) ADCB (c) DCBA (d) BADC SSC CHSL 31.05.2022 Shift-III Ans. (b) : Appropriate sequence of the above jumbled sentences will be ADCB. 90. Sentences of a paragraph are given below in jumbled order. Arrange the sentences in the correct order to form a meaningful and coherent paragraph. A. But while it may seem normal to you, your child may be negatively affected by your and your partner's actions. B. There is no telling when a squabble between a couple may aggravate and escalate into something ugly, which when witnessed by a child, might devastate them gravely. C. There's no relationship that does not falter. D. Arguments, disagreements and conflicts are extremely common even in a healthy relationship. (a) BACD (b) CDAB (c) CBAD (d) BCAD SSC CHSL 26.05.2022 Shift-I Ans. (b) : Appropriate sequence of the above jumbled sentences will be CDAB . 91.

873

A.

B. C. D.

Sentences of a paragraph are given below in jumbled order. Arrange the sentences in the correct order to form a meaningful and coherent paragraph. Dumbledore tells McGonagall that someone named Voldemort has killed a Mr. and Mrs. Potter and tried unsuccessfully to kill their baby son, Harry. Dumbledore leaves Harry with an explanatory note in a basket in front of the Dursley home. Mr. Dursley, a well-off Englishman, notices strange happenings on his way to work one day. That night, Albus Dumbledore, the head of a wizardry academy called Hogwarts, meets Professor McGonagall, who also teaches at Hogwarts, and a giant named Hagrid outside the Dursley home. YCT

(a) DCAB (c) CDAB

(b) ADBC C. (d) BCAD SSC CHSL 26.05.2022 Shift-III D. Ans. (c) : Appropriate sequence of the above jumbled sentences will be CDAB .

There are not many leaders in the present world because, unlike ancient times, people are not ready to accept responsibilities. Ancient India produced millions of people with leadership because it is an inner part of Indian culture to create leaders in every sphere of life. Chandragupta Mourya, Gandhi, Buddha, Bose etc. were leaders on this soil (a) ADCB (b) ADBC (c) BCDA (d) BCAD SSC CHSL 27.05.2022 Shift-I Ans. (c) : Appropriate sequence of the above jumbled sentences will be BCDA.

92.

Sentences of a paragraph are given below in jumbled order. Arrange the sentences in the correct order to form a meaningful and coherent paragraph. A. Their brother, John, is more blatantly hostile to Jane, reminding her that she is a poor dependent of his mother. B. Orphaned as an infant, Jane Eyre lives at 95. Identify the option that arranges the given Gateshead with her aunt, Sarah Reed, as the parts in the correct order to form a meaningful novel opens. and coherent paragraph. C. Her female cousins, Georgiana, and Eliza, (A) However, through sheer grit and determination tolerate, but don't love her. (B) Nick was able to get admission to a high school D. Jane is ten years old, an outsider in the Reed (C) Nick Vujicic was prevented from family. (D) attending mainstream middle schools. (a) BDCA (b) DCAB (a) C, D, A, B (b) A, B, C, D (c) BACD (d) DBCA (c) D, C, A, B (d) B, C, A, D SSC CHSL 25.05.2022 Shift-I SSC CHSL 27.05.2022 Shift-III Ans. (a) : Appropriate sequence of the above jumbled Ans. (a) : Appropriate sequence of the above jumbled sentences will be BDCA . sentences will be CDAB.

93.

A. B.

C. D.

Sentence of a paragraph are given below in jumbled order. Arrange the sentences in the correct order to form a meaningful and coherent paragraph. When the fish at last had been safely landed, I asked the brothers if they ate fish. The condition that I put was that they had to help me land another fish for my men. C. On receiving their eager answer that they most certainly did, I told them I would. On receiving their eager answer that they most certainly did, I told them I would give them the fish we had just landed. To this they readily agreed and the wait started again. (a) ACBD (b) BDCA (c) ABCD (d) DCBA SSC CHSL 27.05.2022 Shift-II

Ans. (a) : Appropriate sequence of the above jumbled sentences will be ACBD. 94.

Sentences of a paragraph are given below in jumbled order. Arrange the sentences in the correct order of form a meaningful and coherent paragraph. A. These great leaders never escaped from their responsibilities and challenges because they knew greatness would come through great responsibilities. B. Leadership is one of the crucial elements in the field of management because most of the time success or failure depends on the managerial skills of a leader. Parajumbles

96.

Identify the option that arranges the given parts in the correct order to form a meaningful and coherent paragraph. A) Non-verbal communication is a means B) This is possible by using visual clues C) By which you are able to communicate without speaking D) Such as facial expressions, eye-contact, postures, gestures etc. (a) D, C, A, B (b) A, B, C, D (c) A, C, B, D (d) B, D, C, A SSC CHSL 26.05.2022 Shift-II Ans. (c) : Appropriate sequence of the above jumbled sentences will be ACBD .

97.

Sentences of a paragraph are given below in jumbled order. Arrange the sentences in the correct order to form a meaningful and coherent paragraph. (A) They always shared machinery and goods as needed without a hitch. (B) Then they had the first serious quarrel in 40 years of farming side by side. (C) And unfortunately, the long collaboration fell apart. (D) Two brothers lived on adjoining farms in a small village. (a) DABC (b) BCAD (c) DBCA (d) CADB SSC CGL (Tier-I) 19/04/2022 Shift-III Ans. (a) : Appropriate sequence of the above jumbled sentences will be DABC.

874

YCT

98.

Sentences of a paragraph are given below. While the first and the last sentences are in correct order, the sentences in between are jumbled up. Arrange the sentences in the right order to form a meaningful and coherent paragraph. S1. Ram Mohan was a lover of his country. P. David said that it would be a good plan to have an English school and college. Q. He was talking with David about uplifting the mind of Indians. R. So, he also thought that this subject was very interesting. S. David also had a strong bond with India. S6. Indians and Europeans met next year to put this idea into shape. (a) RSPQ (b) SRPQ (c) RPQS (d) QSRP SSC CGL (Tier-I) 21/04/2022 Shift-III Ans. (d) : Appropriate sequence of the above jumbled sentences will be QSRP.

99.

101. Sentences of a paragraph are given below in jumbled order. Arrange the sentences in the correct order to form a meaningful and coherent paragraph. A. Firstly, it provides air for respiration, serves the sense of smell and conditions the air by filtering, warming and moistening it. B. The nose is the prominent structure between the eyes that serves as the entrance to the respiratory tract. C. For inhalation it has two cavities, separated from one another by a wall of cartilage called the septum. D. Apart from filtering the air, it also cleans itself of foreign debris extracted from inhalations. (a) BADC (b) DBCA (c) BACD (d) ACDB SSC CGL (Tier-I) 18/04/2022 Shift-I Ans. (a) : Appropriate sequence of the above jumbled sentences will be BADC. 102. Sentence of a paragraph are given below in jumbled order. Arrange the sentences in the correct order to form a meaningful and coherent paragraph. A. The interest rates offered depends on the bank, deposit amount and the tenure you choose. B. It not only helps you to save money but also helps you to earn a substantial interest on it. C. One of the best ways to secure your money is by investing in fixed deposits. D. Under the fixed deposit scheme, the depositor deposits the money only once at the time of opening the account. (a) ADCB (b) ACBD (c) CABD (d) CDBA SSC CGL (Tier-I) 12/04/2022 Shift-II Ans. (d) : Appropriate sequence of the above jumbled sentences will be CDBA.

Sentences of a paragraph are given below in jumbled order. Arrange the sentence in the correct order to form a meaningful and coherent paragraph. A. Thus, shared laughter is one of the most effective tools in maintaining healthy relationships. B. Emotional sharing builds strong and lasting bonds. C. It also helps in healing resentments and hurt. D. But sharing laughter brings joy and freshness to relationships. (a) BCAD (b) DCBA (c) BDCA (d) DCAB SSC CGL (Tier-I) 20/04/2022 Shift-III Ans. (c) : Appropriate sequence of the above jumbled sentences will be BDCA. 103. Sentences of a paragraph are given below in jumbled order. Arrange the sentences in the 100. Given below are four jumbled sentences. Select correct order to form a meaningful and the option that gives their correct order. coherent paragraph. A. For example, the sound of footsteps followed by A. They can help you to decide the right the slam of a door are like phrases which products for you. convey some information. B. Advertisements can be extremely useful if B. In films, sound effects play the part of words. they are honest. C. We then know who is performing an action and why, and a clear picture is formed in our C. Teenagers are especially vulnerable to mind's eye. such advertisements. D. Once the sounds are prefaced or followed by a D. However, some advertisements may be dialogue, the sentence is complete. harmful as they try to befool you. (a) DCBA (b) BCDA (a) BCAD (b) BADC (c) BADC (d) CBDA (c) CDAB (d) CADB SSC CGL (Tier-I) 19/04/2022 Shift-II SSC CGL (Tier-I) 11/04/2022 Shift-II Ans. (c) : Appropriate sequence of the above jumbled Ans. (b) : Appropriate sequence of the above jumbled sentences will be BADC. sentences will be BADC. Parajumbles

875

YCT

104. Given below are four jumbled sentences. Select C Another difference is that it is powered by the option that gives their correct order. wind and solar energy A. Finally, it can be termed as the channel for all D A new park called Bharat Darshan Park has been thrown open to the public in New trade, and what is more important, of all ideas. Delhi. B. It also provides framework to all economic (a) BACD (b) ACDB development. (c) DBCA (d) DABC C. It is the road which determines the site of many SSC CGL (Tier-I) 13/04/2022 Shift-I cities and the growth and nourishment of all. D. The road is one of the great fundamental Ans. (d) : Appropriate sequence of the above jumbled institutions of mankind. sentences will be DABC. (a) DCBA (b) BDAC 108. Given below are four jumbled sentences. Select (c) DBAC (d) CABD the option that gives their correct order. SSC CGL (Tier-I) 13/04/2022 Shift-III A. Our amusements have little zest if we engage in them in solitude. Ans. (a) : Appropriate sequence of the above jumbled B. Friendship increases happiness and diminishes sentences will be DCBA. misery. 105. Given below are four sentences which are C. It doubles our joys and divides our grief. jumbled. Pick the option that gives their D. When we do well, it is delightful to have friends correct order. who are pleased with our success. A. Schools are closed for the Christmas and (a) ABDC (b) BCDA winter break at this time of the year. (c) DACB (d) CBAD B. Christmas and New Year are the time of the SSC CGL (Tier-I) 11/04/2022 Shift-III year to celebrate. Ans. (b) : Appropriate sequence of the above jumbled C. All over the city, winter carnivals and sentences will be BCDA. Christmas bazaars lend fun and warmth in the 109. Sentences of a paragraph are given below in cold. jumbled order. Arrange the sentences in the D. For the second time in a row, we are likely to correct order to form a meaningful and see restrained celebrations for fear of coherent paragraph. the pandemic raising its ugly head again. A. However, when areas in Leh began to (a) BACD (b) BCDA experience water shortages, life didn’t (c) BADC (d) ABCD grind to a halt. B. Ladakh is a cold desert with a low average SSC CGL (Tier-I) 12/04/2022 Shift-III annual rainfall. Ans. (a) : Appropriate sequence of the above jumbled C. Thus, glaciers have been the main source sentences will be BACD. of water for the people. 106. Arrange the statements in the correct order to D. This was because Chewang Norphel, a make a meaningful paragraph. retired civil engineer came up with the idea of artificial glaciers. A. But, in developed societies, the childhood (a) BCDA (b) CBDA and adolescence is extended. (c) DABC (d) BCAD B. In rural communities, the customs are SSC CGL (Tier-I) 12/04/2022 Shift-I more uniform. C. This provides more opportunities for Ans. (d) : Appropriate sequence of the above jumbled sentences will be BCAD. education and character development. 110. Given below are four jumbled sentences. Out of D. The practices of child rearing vary from the given options. Pick the one that gives their culture to culture. correct order. (a) BCDA (b) BADC A. He lived in a small house near the main (c) DBAC (d) DABC road of a village SSC CGL (Tier-I) 13/04/2022 Shift-II B. As both, he and his wife, worked hard, they made a small fortune Ans. (c) : Appropriate sequence of the above jumbled C. But one thing caused them great sorrow: sentences will be DBAC. they had no children 107. Given below are four sentence which are D. After he got married, he started a tailor's jumbled. Pick the option that gives their shop correct order. (a) ABDC (b) BDAC (c) ADBC (d) BACD A It is a park quite different from any other SSC CHSL (Tier-I) –04/08/2021 (Shift-I) we have seen. B One difference is that it is made from Ans. (c) : Appropriate sequence of the above jumbled sentences will be ADBC. nearly 250 tons of scrap. Parajumbles

876

YCT

111. Examine the four jumbled sentences. Out of the given options, pick the one that gives their correct order. A. We gently put the fledgling back into the nest. B. One morning, we found one of the four fallen on the ground. C. The robin couple built a nest in our verandah and laid four eggs. D. Once they had hatched, they busily took turns feeding the four. (a) CDBA (b) ACDB (c) BADC (d) DABC SSC CHSL (Tier-I) –04/08/2021 (Shift-I) Ans. (a) : Appropriate sequence of the above jumbled sentences will be CDBA. 112. Sentence of a paragraph are given below in jumbled order. Arrange the sentences in the right order to form a meaningful and coherent paragraph. A. As a result, he had a debt of 500 million US dollars at the time of his death. B. Unfortunately, he began spending lavishly on anything he desired, not what he actually needed. C. Michael Jackson, called the 'King of Pop Music', stayed at the top of his career for many years. D. He was respected for his work culture; he would spend long nights at the studio to fix a note correctly in a song. (a) DBAC (b) CDBA (c) BADC (d) ADCB SSC CHSL (Tier-I) –05/08/2021 (Shift-I) Ans. (b) : Appropriate sequence of the above jumbled sentences will be CDBA. 113. Sentence of a paragraph are given below in jumbled order. Arrange the sentences in the right order to form a meaningful and coherent paragraph. A. Humans work because they have to; they play because they want to. B. The most useful definitions are those that clarify the relationship of sports to play, games and contests. C. "Play", wrote the German theorist Carl Diem, "Is purposeless activity, for its own sake, the opposite of work." D. Sports are part of every culture, past and present, but each culture has its own definition of sports. (a) ABCD (b) DCBA (c) DBCA (d) BADC SSC CHSL (Tier-I) –05/08/2021 (Shift-I) Ans. (c) : Appropriate sequence of the above jumbled sentences will be DBCA. 114. Examine the four jumbled sentences. Out of the given options, pick the one that gives their correct order. A. That abandoned house is the childhood home of the director Achal Misra, who recreates it in this semi-autobiographical film. B. It is spread over three decades. Parajumbles

C. 'Gamak Ghar' is a unique movie telling the story of home. D. These are from 1998 to 2019, wherein the film observes the house age as it is gradually abandoned. (a) ABCD (b) CBDA (c) BACD (d) DBCA SSC CHSL (Tier-I) –16/04/2021 (Shift-I) Ans. (b) : Appropriate sequence of the above jumbled sentences will be CBDA. 115. Given below are four jumbled sentences. Out of the given options, pick the one that gives their correct order. A. To kill a whale, hunters used spears called harpoons. B. Whale hunting is not easy. C. These harpoons had very long and strong lines tied to them. D. It was extremely difficult and risky before modern methods and machines began to be used. (a) BDAC (b) BADC (c) DBAC (d) DCBA SSC CHSL (Tier-I) –16/04/2021 (Shift-I) Ans. (a) : Appropriate sequence of the above jumbled sentences will be BDAC. 116. Given below are four jumbled sentences. Out of the given options, pick the one that gives their correct order. A. Jody reached out one hand and laid it on the soft neck of the fawn. B. He moved forward on all fours until he was close to it. C. He put his arms around its body. D. The touch made him delirious. (a) BDAC (b) BACD (c) ABDC (d) ADBC SSC CHSL (Tier-I) –15/04/2021 (Shift-I) Ans. (d) : Appropriate sequence of the above jumbled sentences will be ADBC. 117. Given below are four jumbled sentences. Out of the given options, select the one that gives their correct order. A. This is because the Dar Assalam mosque in Berlin can welcome only a fraction of Muslim worshippers during Ramzan because of social distancing rules. B. This year, a church in Berlin has opened its doors to let Muslims do Friday prayers while observing social distancing. C. As Muslims around the world prepare to celebrate Id-ul-Fitr on 25th May, the holiday that marks the end of Ramzan, a month of fasting, things are very different. D. The Martha Lutheran church has stepped in to help, saying, "We too have the same concerns. It's beautiful to feel that way about each other." (a) BDCA (b) CBAD (c) CADB (d) BCAD SSC CHSL (Tier-I) –15/04/2021 (Shift-I)

877

YCT

Ans. (b) : Appropriate sequence of the above jumbled sentences will be CBAD. 118. Given below are four sentences in jumbled order. Pick the option that gives their correct order. A. The robber demanded all his valuables or threatened to kill him. B. But he put up a brave front and challenged the robber. C. A merchant was confronted by a highway robber. D. The merchant was scared to death. (a) CADB (b) BCAD (c) ADCB (d) DABC SSC CHSL (Tier-I) –13/04/2021 (Shift-I) Ans. (b) : Appropriate sequence of the above jumbled sentences will be BCAD. 119. Given below are four jumbled sentences. Out of the given options pick the one that gives their correct order. A. As time passed, he grew jealous of his brother's fine appearance and gentle manners and at last he wished to destroy him. B. When Orlando's father, sir Rowland de Boys, died, he left his youngest son to the care of his eldest brother Oliver. C. Oliver however proved an unworthy brother and neglected the boy. D. It was Oliver who persuaded his brother to wrestle with the famous man who had killed so many others in days past: and it was this cruel brother's neglect of him that made Orlando say that he wished to die. (a) CDAB (b) ABCD (c) DCBA (d) BCAD SSC CHSL (Tier-I) –13/04/2021 (Shift-I) Ans. (a) : Appropriate sequence of the above jumbled sentences will be CDAB. 120. Given below are four sentences in jumbled order. Select the option that gives their correct order. A. Then he started on his eighteen kilometer expedition back to his village. B. Mohan first equipped himself with a long, sturdy stick. C. At several points, he lost the road and had to swim his way ahead. D. It was a journey he would never forget where he had to use his stick constantly to locate the road. (a) DCAB (b) ACBC (c) CDBA (d) BADC SSC CHSL (Tier-I) –06/08/2021 (Shift-I) Ans. (d) : Appropriate sequence of the above jumbled sentences will be BADC. 121. Examine the four jumbled sentences. Out of the given option, pick the one that gives their correct order. A. Next, keep tapping the pot on all sides and at the bottom until the plant loosens. B. Finally, slide the plant gently and remove the old potting mix on a newspaper. Parajumbles

C. The first thing to do when repotting is to remove the plant from the current pot. D. Begin by holding the plant gently, and turning the pot sideways. (a) CDAB (b) DBCA (c) BDAC (d) ACDB SSC CHSL (Tier-I) –06/08/2021 (Shift-I) Ans. (a) : Appropriate sequence of the above jumbled sentences will be CDAB. 122. Given below are four jumbled sentences. Out of the given options, select the one that gives their correct order. A. On either side of the trunk there are a few scattered tufts of short gases. B. It is towards one of these fallen trees that I am now looking. C. A very violent cyclonic storm has struck the forest, uprooting a number of trees. D. The trunk of the tree is ripped apart and the branches are lying all around it. (a) ABDC (b) ADBC (c) CBDA (d) CDAB SSC CHSL (Tier-I) –09/08/2021 (Shift-I) Ans. (c) : Appropriate sequence of the above jumbled sentences will be CBDA. 123. Sentence of a paragraph are given below in jumbled order. Arrange the sentence in the right order to form a meaningful and coherent paragraph. A. His father shifted his own ambition to those of his son, providing him with models and support for his first exhibition at the age of 13. B. Pablo Picasso was the son of Jose Ruiz Blasco, a professor of drawing, and Maria Picasso Lopez. C. From that point his ability to experiment with what he learned and to developed new expressive means quickly allowed him to surpass his father's abilities. D. His unusually adeptness for drawing began to manifest itself early, around the age of 10, when he became his father's pupil. (a) BDCA (b) DCAB (c) BADC (d) CDAB SSC CHSL (Tier-I) –09/08/2021 (Shift-I) Ans. (a) : Appropriate sequence of the above jumbled sentences will be BDCA. 124. Sentences of a paragraph are given below in jumbled order. Arrange the sentences in the right order to form a meaningful and coherent paragraph. A. The other aspect touches upon certain issues that have arisen in recent years, such as brain-death, mercy killing and cloning. B. There are two major aspects relating to the medical profession and its practice the world over. C. These issues must be discussed and debated in the public domain in terms of our cultures and value systems.

878

YCT

D. One of these is the enthics of medical practitioners in their day-to-day relationship with their patients and with one another. (a) BDAC (b) DABC (c) CDAB (d) BCAD SSC CHSL (Tier-I) –19/04/2021 (Shift-I) Ans. (a) : Appropriate sequence of the above jumbled sentences will be BDAC. 125. Sentences of a paragraph are given below in jumbled order. Arrange the sentences in the right order to form a meaningful and coherent paragraph. A. But he has reached out to millions of people the world over and helped to improve their lives. B. Born in downtown Los Angeles in 1960, Tony Robins came form very humble beginnings. C. Now he travels by private jet and owns properties in many parts of the world. D. His family had been fed by others when they did not have enough money for food. (a) BDCA (b) CABD (c) DABC (d) ADBC SSC CHSL (Tier-I) –19/04/2021 (Shift-I) Ans. (a) : Appropriate sequence of the above jumbled sentences will be BDCA. 126. Sentences of a paragraph are given below in jumbled order. Arrange the sentences in the right order to form a meaningful and coherent paragraph. A. It is also a rich source of news updates around the world. B. Older people and children remain glued to it. C. It has become a great source of entertainment for all. D. TV has become a powerful medium these days. (a) ABCD (b) DBCA (c) BDCA (d) ACBD SSC CGL (Tier-I) –24/08/2021 (Shift-I) Ans. (b) : Appropriate sequence of the above jumbled sentences will be DBCA. 127. Sentences of a paragraph are given below in jumbled order. Arrange the sentences in the right order to form a meaningful and coherent paragraph. A. “Buy sweet apples, or none at all”, he instructed his servant. B. A rich man sent his servant to an orchard to buy some apples. C. The servant asked, “How am I to know that all of them are sweet if I taste just one?” D. The owner of the orchard said to the servant, “All my apples are sweet. Try one.” (a) BDAC (b) ABCD (c) BADC (d) ACBD SSC CGL (Tier-I) –24/08/2021 (Shift-I) Parajumbles

Ans. (c) : Appropriate sequence of the above jumbled sentences will be BADC. 128. Sentences of a paragraph are given below in jumbled order. Arrange the sentences in the right order to form a meaningful and coherent paragraph. A. Information Communication Technology tools have brought in additional changes along with means of communication. B. These changes have enabled man to communicate faster. C. Communication has not only become faster but easier too. D. The result is a more relaxed life than ever before. (a) DBCA (b) CDBA (c) ABCD (d) ABDC SSC CGL (Tier-I) –23/08/2021 (Shift-I) Ans. (c) : Appropriate sequence of the above jumbled sentences will be ABCD. 129. Sentences of a paragraph are given below in jumbled order. Arrange the sentences in the right order to form a meaningful and coherent paragraph. A. Inspector: “Were you sleeping when the robbers entered the bank?” B. Guard: “The iron grill lock was broken and the bank looked ransacked but the robbers could not take away anything as I had returned quickly.” C. Inspector: “What did you see when you came back?” D. Guard: “No Sir. The bank had closed and I had just gone to relieve myself.” (a) CDAB (b) BADC (c) ADCB (d) ABCD SSC CGL (Tier-I) –23/08/2021 (Shift-I) Ans. (c) : Appropriate sequence of the above jumbled sentences will be ADCB. 130. Sentences of a paragraph are given below in jumbled order. Arrange the sentences in the right order to form a meaningful and coherent paragraph. A. Then I took him by the hand and led him across the street. B. I was going to the market when I saw a blind man trying to cross the street. C. He expressed his gratitude with folded hands. D. I walked across to the blind man. (a) BDAC (b) CBDA (c) DABC (d) ABCD SSC CGL (Tier-I)–16/08/2021 (Shift-I) Ans. (a) : Appropriate sequence of the above jumbled sentences will be BDAC. 131. Sentences of a paragraph are given below in jumbled order. Arrange the sentences in the right order to form a meaningful and coherent paragraph. A. This invention has helped us to heal wounds caused by bacteria. B. Penicillin is one of the most useful drugs man has invented.

879

YCT

C. In the beginning however, very few people knew of this discovery and its use. D. Its use has saved the lives of hundreds of thousands of soldiers. (a) DCBA (b) BADC (c) ABCD (d) CABD SSC CGL (Tier-I) –16/08/2021 (Shift-I) Ans. (b) : Appropriate sequence of the above jumbled sentences will be BADC. 132. Sentences of a paragraph are given below in jumbled order. Arrange the sentences in the right order to form a meaningful and coherent paragraph. A. However, its importance is primarily for its location. B. But unlike other rivers, it is not used for shipping. C. Although not the longest river, the Rio Grande is the most important river in America. D. America has several long river systems. (a) DCAB (b) DCBA (c) ABCD (d) DABC SSC CGL (Tier-I) –18/08/2021 (Shift-I) Ans. (b) : Appropriate sequence of the above jumbled sentences will be DCBA. 133. Sentences of a paragraph are given below in jumbled order. Arrange the sentences in the right order to form a meaningful and coherent paragraph. A. But a dove that happened to be sitting on a neighbouring tree saw the ant's danger and, plucking off a leaf, let it drop into the water before him. B. Just at that time, a fowler was spreading his net and was in the act of ensnaring the dove, when the ant, perceiving his object bit his heel. C. An ant went to a fountain to quench his thirst and, tumbling in, almost drowned. D. The ant mounting upon it, was presently wafted safely ashore. (a) DCAB (b) CADB (c) CBAD (d) ABCD SSC CGL (Tier-I)–18/08/2021 (Shift-I) Ans. (b) : Appropriate sequence of the above jumbled sentences will be CADB. 134. Sentences of a paragraph are given below in jumbled order. Arrange the sentences in the right order to form a meaningful and coherent paragraph. A. According to his hypothesis, now known as the neuron theory, each nerve cell communicates with others through contiguity rather than continuity. B. The watershed of all studies of the nervous system was an observation made in 1889 by Spanish scientist Santiago Ramón y Cajal. C. It has since been proved that Cajal’s theory is not universally true, but his central idea has remained an accurate guiding principle for all further study. Parajumbles

D. That is, communication between adjacent but separate cells must take place across the space and barriers separating them (a) ADCB (b) CDAB (c) BCDA (d) BADC SSC CGL (Tier-I) –20/08/2021 (Shift-I) Ans. (d) : Appropriate sequence of the above jumbled sentences will be BADC. 135. Sentence of a paragraph are given below in jumbled order. Arrange the sentences in the right order to form a meaningful and coherent paragraph. A. Dad: 'Yes, tell me''. B. Laxmi: 'Dad, I need to speak to you'. C. Dad: 'I think you are still to young for it'. D. Laxmi: 'Please buy me a scooter'. (a) BADC (b) BCDA (c) DCBA (d) ABDC SSC CGL (Tier-I) –20/08/2021 (Shift-I) Ans. (a) : Appropriate sequence of the above jumbled sentences will be BADC. 136. Sentences of a paragraph are given below in jumbled order. Arrange the sentences in the right order to form a meaningful and coherent paragraph. A. Most important, however, are its oils, which are extracted, processed, and marketed for culinary, medicinal, and cosmetic uses alike. B. Coconut products are used to make everything from clothing to animal feed to beauty creams. C. The coconut is a very useful plant with a wide range of products being sourced from it. D. Its kernel is harvested for its edible flesh and delicious water, while its husk is used for its strong fibres. (a) ADBC (b) BCAD (c) CBDA (d) CADB SSC CGL (Tier-I) –17/08/2021 (Shift-I) Ans. (c) : Appropriate sequence of the above jumbled sentences will be CBDA. 137. Sentences of a paragraph are given below in jumbled order. Arrange the sentences in the right order to form a meaningful and coherent paragraph. A. But in snapping at the supposed treasure, he dropped the bit he was carrying, and so lost all. B. When he was crossing a river on his way home, he saw his own shadow reflected in the stream below. C. Thinking that it was another dog with another piece of meat, he resolved to make himself master of that also. D. A dog had stolen a piece of meat out of a butcher's shop. (a) DACB (b) BDCA (c) ACDB (d) DBCA SSC CGL (Tier-I) –17/08/2021 (Shift-I)

880

YCT

C. It was beautiful with big and small domes Ans. (d) : Appropriate sequence of the above jumbled sentences will be DBCA. and arches. D. Sudhir and Mahima settled down on the 138. Sentences of a paragraph are given below in sand and each began to make a separate jumbled order. Arrange the sentences in the right order to form a meaningful and coherent castle. paragraph. (a) DACB (b) ABDC A. When we got near, we saw it was the steam (c) CBDA (d) DBCA rising from hot springs. SSC CGL (Tier-II) 16/11/2020 (3 pm - 5 pm) B. We saw in the distance a great column of Ans. (a) : Appropriate sequence of the above jumbled smoke. sentences will be DACB. C. We wondered if it came from a chimney or 142. Given below are four sentences in jumbled a burning house. order. Pick the option that gives their correct D. We thought of taking a bath in the hot order. water. (a) ACDB (b) BCDA A. Some turned their eyes to the TV screen (c) ABCD (d) BCAD waiting for the movie to begin. SSC CGL (Tier-I) –13/08/2021 (Shift-I) B. No sooner had the big jet taken off than the Ans. (b) : Appropriate sequence of the above jumbled passengers began to busy themselves. sentences will be BCDA. C. They knew it would be a long flight to 139. Sentences of a paragraph are given below in Boston. jumbled order. Arrange the sentences in the D. Others took out books and magazines to right order to form a meaningful and coherent read. paragraph. (a) CBDA (b) BCAD A. But the eagle, in wrath, gave the beetle a (c) BDAC (d) CBAD flap of his wing, and straightaway seized SSC CGL (Tier-II) 16/11/2020 (3 pm - 5 pm) upon the hare and devoured him. B. The beetle therefore interceded with the Ans. (b) : Appropriate sequence of the above jumbled eagle, begging of him not to kill the poor sentences will be BCAD. suppliant, and pleaded with him not to kill 143. Given below are four sentences in jumbled so small an animal. order. Pick the option that gives their correct C. When the eagle flew away, the beetle flew order. after him, to learn where his nest was. A. So, those subjected to make-up had to face D. A hare, being pursued by an eagle, took a fiery misery. himself for refuge to the nest of a beetle, B. There were lights at all angles and around whom he begged to save him. half a dozen large mirrors in the room. (a) CBAD (b) DCAB C. They make-up room had the look of a hair(c) ACDB (d) DBAC cutting salon. SSC CGL (Tier-I) –13/08/2021 (Shift-I) D. They were all incandescent lights. Ans. (d) : Appropriate sequence of the above jumbled (a) ADBC (b) DCAB sentences will be DBAC. (c) BADC (d) CBDA 140. Given below are four sentences in jumbled SSC CGL (Tier-II) 16/11/2020 (3 pm - 5 pm) order. Pick the option that gives their correct Ans. (d) : Appropriate sequence of the above jumbled order. sentences will be CBDA. A. People were driving away wild pigs from 144. Given below are four sentences in jumbled the fields by shooting at them. order. Pick the option that gives their correct B. We were passing through the sugarcane order. fields near Mysore. A. But he seems to have become weary when C. We thought that the shooting was over it came to the ears. when a sloth bear cam running towards us. B. The result is that we spend all our hours D. Some got shot and others escaped. hankering after something unattainable, (a) DABC (b) CDAB namely silence. (c) ACDB (d) BADC C. God constructed the human body with a lot SSC CGL (Tier-II) 16/11/2020 (3 pm - 5 pm) of forethought and solicitude. Ans. (d) : Appropriate sequence of the above jumbled D. He left them as the most vulnerable part of sentences will be BADC. a human being. 141. Given below are four sentences in jumbled (a) DABC (b) CADB order. Pick the option that gives their correct (c) BCDA (d) CDBA order. SSC CGL (Tier-II) 16/11/2020 (3 pm - 5 pm) A. Soon, Mahima's castle was ready. B. As for Sudhir's castle, even the walls Ans. (b) : Appropriate sequence of the above jumbled sentences will be CADB. weren't ready yet. Parajumbles

881

YCT

145. Given below are four sentences in jumbled order. Pick the option that gives their correct order. A. "Wow! That's fantastic news," said his family and friends. B. He had gone to Koklata for the inter-state swimming championship. C. Irfan was a swimming champ. D. "He won golds in 100m freestyle, 10 m butterfly and relay," informed his coach. (a) CDAB (b) CABD (c) CBDA (d) CBAD SSC CGL (Tier-II) 16/11/2020 (3 pm - 5 pm) Ans. (c) : Appropriate sequence of the above jumbled sentences will be CBDA. 146. Given below are four sentence in jumbled order. Pick the option that gives their correct order. A. She could not accompany her daughter due to visa restrictions. B. That trip to Florida with her father Yuri launched her on the path to success. C. Maria had not yet celebrated her tenth birthday when she was packed off in a train to the United States. D. But it also required a heart-wrenching two-year separation from her mother Yelena. (a) ABCD (b) CBDA (c) CADB (d) BCAD SSC CGL (Tier-II) 16/11/2020 (3 pm - 5 pm) Ans. (b) : Appropriate sequence of the above jumbled sentences will be CBDA. 147. Given below are four sentences in jumbled order. Pick the option that gives their correct order. A. Although Sonu had many more that he wanted to ask, he realised mummy was tired and became silent. B. She answered a few and then stopped. C. When mummy returned from work, she took Sonu for a walk. D. But she was too tired to answer all his questions. (a) CABD (b) BDAC (c) CDBA (d) DACB SSC CGL (Tier-II) 16/11/2020 (3 pm - 5 pm) Ans. (c) : Appropriate sequence of the above jumbled sentences will be CDBA. 148. Given below are four sentences in jumbled order. Pick the option that gives their correct order. A. Each morning he was driven to Obedience School in a black limousine. B. Each evening he fell asleep in his fur-lined basket in front of the fireplace. C. Each afternoon he was fed two grilled lamb chops for lunch. D. Henry D. Penrose was a dog with a pedigree. (a) DBCA (b) DCAB (c) DACB (d) DABC SSC CGL (Tier-II) 16/11/2020 (3 pm - 5 pm) Parajumbles

Ans. (c) : Appropriate sequence of the above jumbled sentences will be DACB. 149. Given below are four sentences in jumbled order. Pick the option that gives their correct order. A. Soon after they left school, they decided that a small town was no place to make a fortune. B. On reaching Paris, they agreed to separate because they wished to be independent of each other. C. So, they ran away from home and managed to reach France on a cargo ship. D. My father was a year older than his brother Oscar. (a) BDAC (b) DACB (c) BDCA (d) ACBD SSC CGL (Tier-II) 16/11/2020 (3 pm - 5 pm) Ans. (b) : Appropriate sequence of the above jumbled sentences will be DACB. 150. Given below are four sentences in jumbled order. Pick the option that gives their correct order. A. Mahatma Gandhi got a doctor to volunteer his services for six months. B. Health conditions in Champaran were miserable. C. The doctor gave quinine to malaria patients and the ointment with castor oil to those with skin eruptions. D. Only three medicines were available castor oil, quinine and sulphur ointment. (a) CDAB (b) BCDA (c) ACBD (d) BADC SSC CGL (Tier-II) 16/11/2020 (3 pm - 5 pm) Ans. (d) : Appropriate sequence of the above jumbled sentences will be BADC. 151. Given below are four sentences in jumbled order. Pick the option that gives their correct order. A. The other was on the side table near the window. B. When I returned, I was shocked to see one set lying on the floor. C. I was gone for more than an hour. D. I had not completed reading the proofs so I left them on my desk. (a) DBAC (b) CDAB (c) DCBA (d) CBDA SSC CGL (Tier-II) 16/11/2020 (3 pm - 5 pm) Ans. (c) : Appropriate sequence of the above jumbled sentences will be DCBA. 152. Given below are four sentences in jumbled order. Pick the option that gives their correct order. A. She had never seen so large and so golden an apple. B. Hilda was passing through an orchard when an apple fell at her feet. C. So, she wrapped it in her handkerchief and carried it home to store it in her drawer.

882

YCT

D. She held it carefully in her clasped hands and thought it would be a pity to eat it. (a) ABCD (b) BADC (c) DACB (d) BCAD SSC CGL (Tier-II) 16/11/2020 (3 pm - 5 pm) Ans. (b) : Appropriate sequence of the above jumbled sentences will be BADC. 153. Given below are four sentences in jumbled order. Pick the option that gives their correct order. A. Dr Prince became aware of a small figure standing in the aisle beside his seat. B. "Won't your mother be wondering where you are?" he asked her. C. He brought his eyes into focus and saw that it was a girl of seven or eight. D. She was staring at him steadily with large blue eyes. (a) BDAC (b) ABCD (c) CDBA (d) ACDB SSC CGL (Tier-II) 16/11/2020 (3 pm - 5 pm) Ans. (d) : Appropriate sequence of the above jumbled sentences will be ACDB. 154. Given below are four sentences in jumbled order. Pick the option that gives their correct order. A. Someone had been smart enough to remove it before I went on the rampage. B. My hands had been itching to tear down that collage from my bedroom wall. C. But I found the wall bare. D. So, I entered my room in a hurry. (a) DABC (b) ABCD (c) BDCA (d) CBAD SSC CGL (Tier-II) 16/11/2020 (3 pm - 5 pm) Ans. (c) : Appropriate sequence of the above jumbled sentences will be BDCA. 155. Given below are four sentences in jumbled order. Pick the option that gives their correct order. A. But it does not appear to have any effect on the children. B. School teachers try to do their best from the early years by ordering, "Silence!" every few seconds in the class. C. They look sweet and elegant but the moment they open their mouths, they let out a shattering volume of sound. D. Someone noted recently that present day babies are peculiarly loud-throated. (a) DCBA (b) ACBD (c) BACD (d) CABD SSC CGL (Tier-II) 16/11/2020 (3 pm - 5 pm) Ans. (a) : Appropriate sequence of the above jumbled sentences will be DCBA. 156. Given below are four sentences in jumbled order. Pick the option that gives their correct order. A. However, Yusuf comforted himself by imagining the hotel food in store for him the next day. B. First, Yusuf's stomach was occupied by excitement. Parajumbles

C. It was difficult to eat dinner that night. D. Secondly, mother was on one of her health trips and had prepared sprouted beans and youghurt. (a) BADC (b) BCAD (c) ACBD (d) CBDA SSC CGL (Tier-II) 16/11/2020 (3 pm - 5 pm) Ans. (d) : Appropriate sequence of the above jumbled sentences will be CBDA. 157. Given below are four sentences in jumbled order. Pick the option that gives their correct order. A. Therefore, I bought a good house in a respectable neighbourhood. B. This made me eager to welcome the first guest at home and show my hospitality. C. I intended to be a model citizen in the neighbourhood. D. I decided it was time for me to settle down. (a) CBDA (b) DBCA (c) CADB (d) DACB SSC CGL (Tier-II) 16/11/2020 (3 pm - 5 pm) Ans. (d) : Appropriate sequence of the above jumbled sentences will be DACB. 158. Given below are four sentences in jumbled order. Pick the option that gives their correct order. A. Mummy struggled to pull them apart and finally succeeded. B. She tried to get them to make up but they did not listen. C. It was a fierce fight, of course. D. The two children scratched and hit each other. (a) CBDA (b) DBAC (c) CDAB (d) ACBD SSC CGL (Tier-II) 16/11/2020 (3 pm - 5 pm) Ans. (c) : Appropriate sequence of the above jumbled sentences will be CDAB. 159. Given below are four sentences in jumbled order. Pick the option that gives their correct order. A. He had lost his leg in an accident five years back. B. With that accident his dream of becoming the next Carl Lewis had been shattered forever. C. In place of his left leg was a wooden stump. D. Vikas sat up and removed the bed sheet that was covering his leg. (a) ACDB (b) DCAB (c) CDAB (d) BCAD SSC CGL (Tier-II) 16/11/2020 (3 pm - 5 pm) Ans. (b) : Appropriate sequence of the above jumbled sentences will be DCAB. 160. Given below are four jumbled sentences. Pick the option that gives their correct order. A: Suddenly she saw a child begging on the platform. B: She also noticed that there was a book in his bag.

883

YCT

C: Ruhi was getting restless as she still had over two hours to wait before her train came. D: He appeared to be a young boy of about ten years who had a small bag on his shoulder. (a) ABDC (b) CADB (c) DBCA (d) CBAD SSC CPO-SI (Tier-II) 27/09/2019 (3 pm - 5 pm) Ans. (b) : Appropriate sequence of the above jumbled sentences will be CADB. 161. Given below are six sentences 4 of which are jumbled. The first and the last sentence are given. Pick the option that gives the logically correct order of the four sentences. S-1 Once a daughter complained to her father that her life was miserable and that she didn’t know how she was going to tackle it. A: Her father, a chef, took her to the kitchen. B: She was tired of fighting and struggling all the time. C: He filled three pots with water and placed each on a high fire. D: It seemed as if just as one problem was solved, another one followed. S-6 When the three pots began to boil, he placed potatoes in one pot, eggs in the second pot and ground coffee beans in the third pot. (a) BCDA (b) BDAC (c) CBDA (d) ADCB SSC CPO-SI (Tier-II) 27/09/2019 (3 pm - 5 pm) Ans. (b) : Appropriate sequence of the above jumbled sentences will be BDAC. 162. Given below are six sentences 4 of which are jumbled. The first and the last sentence are given. Pick the option that gives the logically correct order of the four sentences. S-1 The salt seller came to understand the trick and decided to teach the donkey a lesson. A: But the dampened cotton became very heavy to carry and the donkey suffered. B: It learnt a lesson. C: Again it played the same trick hoping that the cotton bag would become lighter. D: The next day he loaded a cotton bag on the donkey. S-6 It didn’t play the trick anymore after that day, and the salt seller was happy. (a) CBAD (b) DBCA (c) DCAB (d) ACBD SSC CPO-SI (Tier-II) 27/09/2019 (3 pm - 5 pm) Ans. (c) : Appropriate sequence of the above jumbled sentences will be DCAB. 163. The question below consists a set of labeled sentences. These sentences, when properly sequenced form a coherent paragraph. Select the most logical order of sentences from among the options. Parajumbles

P: We may see alcohol and tobacco advertisements everywhere, on television, in newspaper, on street ads card etc. Q: But we know the truth is alcohol and cigarette are harmful for people's health and sometimes it may bring bad effects to self-impression. R: Alcohol ads usually create several feints to tell people that alcohol is good for people and induce people to drink. S: On the other hand, the malign influence of advertisements shows smoking as something "cool". (a) PRSQ (b) PSQR (c) QRSP (d) RSPQ SSC CGL (Phase-II) 17/02/2018 Ans. (a) Appropriate sequence of the above jumbled sentences will be PRSQ. 164. The question below consists a set of labeled sentences. These sentences, when properly sequenced form a coherent paragraph. Select the most logical order of sentences from among the options. P: If the intention is just to consume whatever comes from the west, then it is harmful. Q: Some of its effects are really helping and positive as it increases initiative and entrepreneurship qualities provided taken in that spirit. R: At the same time state's strategic intervention is essential because more than 40% of people live below poverty line. S: The impact of economic reforms are mixed. (a) SPQR (b) SQPR (c) RQPS (d) QPSR SSC CGL (Phase-II) 17/02/2018 Ans. (b) Appropriate sequence of the above jumbled sentences will be SQPR. 165. The question below consists a set of labeled sentences. These sentences, when properly sequenced form a coherent paragraph. Select the most logical order of sentences from among the options. P: The political awakening cannot be an isolated phenomenon; it requires some changes in social structure so that woman can enjoy as important a place as man occupies. Q: Unfortunately our customs and traditions conspired with her economic dependence to make her unimportant in our society. R: Democracy in India can be a success only when the Indian women are politically awakened. S: Moreover they should be free to express their opinion, to act as they like and to assert themselves in all departments of life. (a) PSQR (b) SQPR (c) PRSQ (d) RPSQ SSC CGL (Phase-II) 17/02/2018

884

YCT

Ans. (d) Appropriate sequence of the above jumbled sentences will be RPSQ. 166. The question below consists a set of labeled sentences. These sentences, when properly sequenced form a coherent paragraph. Select the most logical order of sentences from among the options. P: No partner is expected to air views of a particular group in public. Q: The only requirement is that the coalition partners have to stick to a code of conduct. R: Every coalition party has to own the responsibility for all government’s policies or actions. S: Experience has now shown that a coalition government can run as smoothly as any single party government. (a) QRPS (b) RQPS (c) SQRP (d) PQSR SSC CGL (Phase-II) 17/02/2018 Ans. (c) Appropriate sequence of the above jumbled sentences will be SQRP. 167. The question below consists a set of labeled sentences. These sentences, when properly sequenced form a coherent paragraph. Select the most logical order of sentences from among the options. P: Biographies of great men can also help us in learning good manners. Q: Courtesy and politeness is the key to good manners. R: No doubt these are little words but if they are spoken at the right moment and in a soft an sweet voice, they are bound to work wonders. S: The use of polite words like "Sorry", "Please", "Thank you", "Beg your pardon", "Sir" etc. creates a healthy impact on the minds of others. (a) PRSQ (b) QPSR (c) QPSR (d) SPQR SSC CGL (Phase-II) 17/02/2018 Ans. (c) Appropriate sequence of the above jumbled sentences will be QPSR. 168. The question below consists of a set of labeled sentences. These sentences, when properly sequenced form a coherent paragraph. Select the most logical order of sentences from among the options. P: One has to work hard to establish, develop and lasting bonds with those who matter in one's life. Q: As "Man is not as an island entire of itself", he cannot be happy in isolation. R: Humans live out their life in the company of their family, friends, colleagues, superiors, even total strangers in buses and lifts. S: And, brick by brick, they must build their relationship with each one, to lay a secure foundation for their road to happiness. Parajumbles

(b) RQSP (d) QRSP SSC CGL (Phase-II) 17/02/2018 Ans. (d) Appropriate sequence of the above jumbled sentences will be QRSP. 169. The question below consists of a set of labeled sentences. These sentences, when properly sequenced form a coherent paragraph. Select the most logical order of sentences from among the options. P: Superstitions mean an irrational belief in or notion of the ominous significance of a particular thing, circumstance, or the like. Q: This is a universal phenomenon cutting across caste, communal, and even national boundaries. R: All over the globe, people have superstitions, although the superstitions may vary from country to country or from region to region. S: Nor does education drives away superstitions completely. (a) PQRS (b) SQRP (c) QRPS (d) RPSQ SSC CGL (Phase-II) 17/02/2018 Ans. (a) Appropriate sequence of the above jumbled sentences will be PQRS. 170. The question below consists of a set of labeled sentences. These sentences, when properly sequenced form a coherent paragraph. Select the most logical order of sentences from among the options. P: Superstitons exist throughout the world despite scientific advances. Q: One cannot heed anybody's sneezes if one has to be punctual on duty or 'cannot miss a flight. R: But the spread of science and education among the masses has certainly given rise to a significant number of persons who do not believe in these superstitions. S: The compulsions of modern life also caused some of the superstitions to fade out. (a) RPSQ (b) SQRP (c) SRQP (d) PRSQ SSC CGL (Phase-II) 17/02/2018 Ans. (d) Appropriate sequence of the above jumbled sentences will be PRSQ. 171. The question below consists of a set of labeled sentences. These sentences, when properly sequenced form a coherent paragraph. Select the most logical order of sentences from among the options. P: All these ensure that the world we live in grows a new skin every few years. Q: These influences do, of course, work on humankind but they have the greatest impact on that section of society which is on the threshold of discovery-discovery of the self, of life and of living.

885

(a) SPRQ (c) QSPR

YCT

R: The life and times of two generations cannot be identical or even similar, thanks to modern research, progress in science, faster and easier communications and even distasteful things like inflation and population explosion. S: These changes in due course affect our thinking and influence our attitudes, expectations, behavioral patterns and values. (a) PSQR (b) RPSQ (c) QSPR (d) QPSR SSC CGL (Phase-II) 17/02/2018 Ans. (b) Appropriate sequence of the above jumbled sentences will be RPSQ. 172. The question below consists of a set of labeled sentences. These sentences, when properly sequenced form a coherent paragraph. Select the most logical order of sentences from among the options. P: A certain moral code of conduct is ultimately necessary to ensure that the society does not fall prey to degeneration of values, which would lead to rampant sufferings and ultimately chaos. Q: Society's concerns are our concerns: anything capable of causing a detrimental impact on it the short term or in course of time is ultimately bound to affect us and our children. R: When we talk about social morality of any kind, what comes into play is our ability to feel for the well-being of our society. S: It is a concern to help the society by safeguarding it from unwanted ills and malaise and ensuring its well-being that is at the root of social morality. (a) PSQR (b) RPSQ (c) QPSR (d) PQRS SSC CGL (Phase-II) 17/02/2018 Ans. (b) Appropriate sequence of the above jumbled sentences will be RPSQ. 173. The question below consists of a set of labeled sentences. These sentences, when properly sequenced form a coherent paragraph. Select the most logical order of sentences from among the options. P: Those who say death should be the punishment in some cases, agree that it is to be in special cases alone, the most heinous and gravest of crimes. Q: They feel that it is only fear of server punishment that will deter that will deter the criminal and reduce the occurrence of heinous crimes. R: The advocates and the aboltionists of capital punishment have their own arguments in support of their stand. S: The most prominent argument put forth by the advocates of the death penalty is that of deterrence. (a) RPSQ (b) PQSR (c) SQPR (d) QPSR SSC CGL (Phase-II) 17/02/2018 Parajumbles

Ans. (a) Appropriate sequence of the above jumbled sentences will be RPSQ. 174. The question below consists of a set of labeled sentences. These sentences, when properly sequenced form a coherent paragraph. Select the most logical order of sentences from among the options. P: There was once a time when people looked forward to the lazy evenings. Q: They had a simple choice of programmes on Doordarshan. R: It was entirely up to them-to watch or not to watch the selected presentation. S: Today, the satellite and Cable Television have stormed the media world of information and entertainment. (a) PSQR (b) RQPS (c) PQRS (d) QPRS SSC CGL (Phase-II) 17/02/2018 Ans. (c) Appropriate sequence of the above jumbled sentences will be PQRS. 175. The question below consists of a set of labeled sentences. These sentences, when properly sequenced form a coherent paragraph. Select the most logical order of sentences from among the options. P: He wanted to do something to deliver the humanity from all such misery. He reflected on this problem for long. Q: Siddhartha was greatly touched as he saw an old man, a sick man, and a dead body. R: At last on hearing some words from the mouth of a hermit which encouraged him to renounce the world, he decided to leave the palace and go into the forest for meditation. S: Before going, he had a lasting glance on his beloved wife Yashodhra and son, Rahul who were enjoying a sound sleep at midnight. (a) PRSQ (b) RPQS (c) PQRS (d) QPRS SSC CGL (Phase-II) 17/02/2018 Ans. (d) Appropriate sequence of the above jumbled sentences will be QPRS. 176. The question below consists of a set of labeled sentences. These sentences, when properly sequenced form a coherent paragraph. Select the most logical order of sentences from among the options. P: The main manifestations of violence in recent times have been extremism, terrorism, assassination by bombs and bullets. Q: In the modern world, violence has taken amy forms, each signifying the evil side of man and his design to design to destroy the principle of equity, justice and truth. R: Another related activity is the hijacking of aircrafts, vehicles vehicles ets. S: Statement missing

886

YCT

(a) QPSR (c) PRQS

(b) PSRQ (d) SRPQ SSC CGL (Phase-II) 17/02/2018 Ans. (a) Appropriate sequence of the above jumbled sentences will be QPSR. 177. The question below consists of a set of labeled sentences. These sentences, when properly sequenced form a coherent paragraph. Select the most logical order of sentences from among the options. P: Children with poor phonological skills progress more poorly. Q: The consensus concerns the causal role of phonological skills in young children's reading progress. R: Studies of the factors governing reading development in young children have achieved a remarkable degree of consensus over the past two decades. S: Children who have good phonological skills, or good phonological awareness become good readers and good spellers. (a) QSPR (b) SPQR (c) RQSP (d) PQRS SSC CGL (Phase-II) 17/02/2018 Ans. (c) Appropriate sequence of the above jumbled sentences will be RQSP. 178. The question below consists of a set of labeled sentences. These sentences, when properly sequenced form a coherent paragraph. Select the most logical order of sentences from among the options. P: As a result, they are more able to live in harmony with themselves and with the world that surrounds them. Q: Stabilizing meditation is catalogued as one of the hardest techniques of meditation because followers must keep their minds under total control for long period of time. R: The benefits of this mediation, according to the followers, is that mediators can experience full understanding of their thoughts and ideas. S: This consists of maintaining full attention to their role object focus. (a) QSRP (b) PQRS (c) QPSR (d) PRQS SSC CGL (Phase-II) 17/02/2018 Ans. (a) Appropriate sequence of the above jumbled sentences will be QSRP. 179. The question below consists of a set of labeled sentences. These sentences, when properly sequenced form a coherent paragraph. Select the most logical order of sentences from among the options. P: They feel that during their time, young boys and girls were better behaved, more obedient, and had greater respect for their elders. Parajumbles

Q: Young people on the other hand, feel that they are capable enough to learn on their own rather than lean heavily on the older generation for any guidance. R: The people belonging to the older generation always wonder as to what has gone wrong with the new generation. S: They feel that a lack of respect for the old will ruin and spell disaster for the young. (a) PRSQ (b) SPQR (c) PRQS (d) RPSQ SSC CGL (Phase-II) 17/02/2018 Ans. (d) Appropriate sequence of the above jumbled sentences will be RPSQ. 180. The question below consists of a set of labeled sentences. These sentences, when properly sequenced form a coherent paragraph. Select the most logical order of sentences from among the options. P: About 30 new hydro projects are now under execution with an installed capacity of 5,600 MW. Q: However, with the rapid increase in demand for power, higher priority was given to the pithead super thermal power stations as their gestation period was smaller than that of the hydel schemes. R: Many projects were taken up for execution after independence and at one time (196263), the capacity contribution from hydro schemes was equal to thermal schemes. S: The first hydro generating unit in India was commissioned in Darjeeling (W. Bengal) in 1897. (a) RQPS (b) SRQP (c) QPSR (d) PQRS SSC CGL (Phase-II) 17/02/2018 Ans. (b) Appropriate sequence of the above jumbled sentences will be SRQP. 181. The question below consists of a set of labeled sentences. These sentences, when properly sequenced form a coherent paragraph. Select the most logical order of sentences from among the options. P: Farmers could no longer pay their loans and some banks were closed down. Q: Prices dropped due to increased supply, which was followed by a drought. R: The country was well advanced in technology and farmers increased their production levels. S: The Great Depression in America began with overproduction and low prices in the agricultural sector. (a) RQPS (b) QPSR (c) PSRQ (d) SRQP SSC CGL (Phase-II) 17/02/2018 Ans. (d) Appropriate sequence of the above jumbled sentences will be SRQP.

887

YCT

182. Given below are four sentences in jumbled 186. Given below are four jumbled sentences. Select order. Pick the option that gives their correct the option that gives their correct order. order. A. When you begin to house train your A. She understood my signs, and I could make puppy, follow these steps. her do as I wished. B. Also, always take her outside after meals B. My constant companion was Martha, our or when she wakes from a nap. cook’s daughter. C. Keep the puppy on a regular feeding C. We even helped in feeding the hens which schedule and take away food between crowded around the kitchen steps. meals. D. This pleased me greatly and we spent a lot D. Take your puppy out first thing in the of time together. morning and then once every thirty (a) DCAB (b) ADCB minutes or so. (c) CBDA (d) BADC (a) ADCB (b) ACDB SSC Sel. Post Phase-VIII (G.L.) 09.11.2020 (Shift-II) (c) CDAB (d) DABC SSC CGL (Tier-I) – 19/06/2019 (Shift-III) Ans. (d) : Appropriate sequence of the above jumbled sentences will be BADC. Ans. (a) : Appropriate sequence of the above jumbled 183. Given below are four sentences which are sentences will be ADCB. jumbled. Pick the option that gives the correrct 187. Given below are four jumbled sentences. Select order. the option that gives their correct order. A. Notify your card issuer that your card is A. Its Value is linked to what it is being used missing and that it should be blocked. for. B. Lastly, don't forget to check your account B. Such questions often baffle you. for any unauthorized withdrawals. C. How emotional should one get about C. A lost or stolen debit card can cause a lot of money and its value? anxiety. D. Money on its own has no value. untill you D. So, the first thing you need to do is to have some use for it. contact your bank. (a) DBAC (b) CBDA (a) ADCB (b) ACBD (c) CABD (d) ACDB (c) CABD (d) CDAB SSC CGL (Tier-I) – 13/06/2019 (Shift-I) SSC Sel. Post Phase-VIII (H.L.) 09.11.2020 (Shift-I) Ans : (b) Appropriate sequence of the above jumbled Ans. (a) : Appropriate sequence of the above jumbled sentences will be CBDA. sentences will be ADCB. 188. Given below are four jumbled sentences. Select 184. Given below are jumbled sentences. Select the the option that gives their correct order. option that gives their correct order. A. Only natural dyes are used in Kalamkari A. A man on a bike had to ride on the and it involves several steps. pavement. B. There are two distinctive styles of B. When he tried to plunge onto the road, a Kalamkari in India. car hit him. C. They are the Sri Kalahasti style and the C. As the bus inched through the evening life, Machlipatnam style. the traffic grew. D. Kalamkari is a type of hand-painted or D. There was no space on the road any more. block-printed cotton textile produced in (a) CBAD (b) DBCA the Indian States of Andhra Pradesh and (c) ADBC (d) CDAB Telangana. SSC CGL (Tier-I) – 13/06/2019 (Shift-III) (a) ACBD (b) CBDA Ans : (d) Appropriate sequence of the above jumbled (c) DABC (d) DCBA sentences will be CDAB. SSC CGL (Tier-I) – 13/06/2019 (Shift-I) 185. Given below are four jumbled sentences. Select Ans : (c) Appropriate sequence of the above jumbled the option that gives their correct order. sentences will be DABC. A. The disruptions began to annoy his 189. Given below are four jumbled sentences. Select teachers. the option that given their correct order. B. They were increasingly baffled by his A. He would sit at the edge of her mother's questions. bad an stare into her crib. C. Swami looked forward to disrupting every B. Arvind bumbled down the interminable class. corridor, suddenly reminded of his D. They began to write notes in his handbook daughter in his days after she was born. summoning his parents. C. As a proportion to the fragment of life his (a) CDAB (b) CABD daughter had seen, an hour was a vast (c) BDAC (d) DACB sprawling apace. SSC CGL (Tier-I) – 19/06/2019 (Shift-III) D. Some days he would imagine the world Ans. (b) : Appropriate sequence of the above jumbled through her eyes and he would feel in his sentences will be CABD. heart how long an hour actually was. Parajumbles

888

YCT

(a) BCDA (b) DACB (c) ABCD (d) BADC SSC CGL (Tier-I) – 13/06/2019 (Shift-II) Ans. (d) : Appropriate sequence of the above jumbled sentences will be BADC. 190. Given below are four jumbled sentences. Select the option that gives their correct order. A. It is a popular tourist spot for watching the sunset and sunrise over the ocean. B. Kanyakumari is a coastal town in the State or Tamil Nadu on India's southern tip. C. It is also a noted pilgrimage site, thanks, to its Bagvathi Amman Temple and Our Lady of Ransom Church, a centre of India Catholicism. D. Jutting out into the Laccadive sea, the town was known as Cape Comorin during the British rule. (a) BDAC (b) ACDB (c) BCDA (d) DCAB SSC CGL (Tier-I) – 13/06/2019 (Shift-II) Ans. (a) : Appropriate sequence of the above jumbled sentences will be BDAC. 191. Given below are four jumbled sentences. Select the option that gives their correct order. A. Once the formalities are done, you can start using your account and save time and money. B. Opening a bank account can seem intimidating. C. Getting your account opened is just a matter of providing certain details and funding your account. D. Fortunately, most banks follow a standardized process. (a) BDCA (b) BADC (c) CBDA (d) ADBC SSC CGL (Tier-I) – 13/06/2019 (Shift-III) Ans : (a) Appropriate sequence of the above jumbled sentences will be BDCA. 192. Given below are four jumbled sentences. Select the option that gives their correct order. A. Lyrics help in creating a distinctive narrative, some conventions of which have been carried over from the talkies era. B. Thus, songs have outlived films in people's memories. C. However, songs seem to have acquired a musical grammar of their own, establishing an emotional chord with the listeners. D. In popular Indian cinema, lyrics are to music what the heart is to the body. (a) ABCD (b) DCBA (c) DACB (d) ADBC SSC CGL (Tier-I) – 12/06/2019 (Shift-I) Ans : (c) Appropriate sequence of the above jumbled sentences will be DACB. Parajumbles

193. Given below are four jumbled sentences. Select the option that gives their correct order. A. So, I had thoughts of setting up an Indian restaurant there with my wife's support. B. It took us two months to redesign the place to suit our needs. C. One of the things I really missed when I set up home in Maryland, was a restaurant that served authentic Indian food. D. I decided to pursue this idea seriously and bought an old building in the downtown. (a) BCDA (b) CADB (c) CBAD (d) DACB SSC CGL (Tier-I) – 12/06/2019 (Shift-I) Ans : (b) Appropriate sequence of the above jumbled sentences will be CADB. 194. Given below are four jumbled sentences. Select the option that gives their correct order. A. This network of stories is known in academic circles as 'fiction' or imagined realities'. B. However, an imagined reality is not a lie. C. Over the years, people have woven an incredibly complex network of stories. D. Within this network fiction not only exists but also accumulates immense power. (a) CADB (b) ADBC (c) DBCA (d) DBAC SSC CGL (Tier-I) – 12/06/2019 (Shift-II) Ans : (a) Appropriate sequence of the above jumbled sentences will be CADB. 195. Given below are four jumbled sentences. Select the option that gives their correct order. A. If the mixture becomes too thick add more milk. B. Finally, add sugar and nuts, and your kheer is ready. C. Boil milk in a heavy-bottomed pan and add rice. D. Cook for about twenty minutes stirring every once in a while till the mixture thickens. (a) ADCB (b) CDAB (c) DACB (d) BADC SSC CGL (Tier-I) – 12/06/2019 (Shift-II) Ans : (b) Appropriate sequence of the above jumbled sentences will be CDAB. 196. Given below are four jumbled sentences. Select the option that gives their correct order. A. The market which has remained clogged with vehicles was all clear for pedestrians. B. The road has even been marked with stripes demarcating space for havwkers. C. It is also lined with beautiful potted plants to give it a green look. D. Visitors to the busy Karol Bagh market in Delhi were in for a surprise. (a) DABC (b) ADBC (c) DCBA (d) BDAC SSC CGL (Tier-I) – 12/06/2019 (Shift-III) Ans. (a) : Appropriate sequence of the above jumbled sentences will be DABC.

889

YCT

197. Given below are four jumbled sentences. Select the option that gives their correct order. A. When we can't laugh at the same joke again, why should we cry over the same problems ? B. One day he told them a joke and everyone roared with laughter. C. People came to a wise man to complain about the same problems every time. D. When he repeated the joke twice, nobody laughed anymore. (a) ADBC (b) BACD (c) CBDA (d) DBCA SSC CGL (Tier-I) – 12/06/2019 (Shift-III) Ans. (c) : Appropriate sequence of the above jumbled sentences will be CBDA. 198. Given below are four jumbled sentences. Select the option that gives their correct order. A. But they faced grave danger if they tried to criticize these decisions. B. The nationalists now began to openly criticize the policies of the British. C. The freedom movement changed this situation. D. Under colonial rule, the people had lived in fear of the British government and did not agree with many of the decisions that they took. (a) DACB (b) CBAD (c) DBAC (d) BADC SSC CGL (Tier-I) – 11/06/2019 (Shift-I) Ans. (a) : Appropriate sequence of the above jumbled sentences will be DACB. 199. Given below are four jumbled sentences. Select the option that gives their correct order. A. The brain is active too during sleep, sending messages for the heart to beat regularly. B. However, the body utilizes the sleeping time effectively. C. We spend about one third of our time sleeping. D. It produces energy and releases hormones for repair and growth during the night. (a) ABDC (b) BADC (c) CABD (d) CBDA SSC CGL (Tier-I) – 11/06/2019 (Shift-I) Ans. (d) : Appropriate sequence of the above jumbled sentences will be CBDA. 200. Given below are four jumbled sentences. Select the option that gives their correct order. A. But the rest of the students were rushing past her for a break, like she didn't exist. B. She slowly pushed open the library door and took a seat at one of the tables in the corner. C. And today, Jessica really felt like maybe she didn't exist. D. When the bell rang for lunch, Jessica took her lunch box and started moving towards the library. (a) CADB (b) DBCA (c) BDCA (d) DACB SSC CGL (Tier-I) – 11/06/2019 (Shift-II) Parajumbles

Ans. (d) : Appropriate sequence of the above jumbled sentences will be DACB. 201. Given below are four jumbled sentences. Select the option that gives their correct order. A. Then, they spread as the unmined coal starts burning with oxygen drawn from pores and mine shafts. B. An entire township-Jharia is to be relocated because of the uncontrollable underground fires. C. These fires start mostly from burning trash close to coal pits. D. This is a grave threat as poisonous fumes of carbon monoxide rise up from underground fires. (a) DCAB (b) BCAD (c) BDAC (d) CDAB SSC CGL (Tier-I) – 11/06/2019 (Shift-II) Ans. (b) : Appropriate sequence of the above jumbled sentences will be BCAD. 202. Given below are four jumbled sentences. Select the option that gives their correct order. A. The first of these states that the only way to have a friend is to be one. B. Like all relationships, it is also governed by some well-defined principles. C. Friendship is one of the most coveted relationships on earth. D. Thus, one must be reliable as a friend to expect reliability in others. (a) ACBD (b) BDAC (c) CDBA (d) CBAD SSC CGL (Tier-I) – 11/06/2019 (Shift-III) Ans. (d) : Appropriate sequence of the above jumbled sentences will be CBAD. 203. Given below are four jumbled sentences. Select the option that gives their correct order. A. But that is for an emotional reason. B. Learning our mother tongue is important and everyone should start there. C. This is because proficiency in the language can lead to better opportunities. D. Learning English. However, is important for practical reasons. (a) BADC (b) DBAC (c) DACB (d) BDCA SSC CGL (Tier-I) – 11/06/2019 (Shift-III) Ans. (a) : Appropriate sequence of the above jumbled sentences will be BADC. 204. Given below are four jumbled sentences. Select the option that gives their correct order. A. The students often go about their business, singing along in the corridors. B. But for the students of St. Ambrose High School, it no longer dictates their day. C. A bell can sound gloomy or cheerful depending on when it is rung. D. It has been scrapped in favour of music which now breaks up the timetable of the school. (a) ACBD (b) CBDA (c) BDAC (d) CADB SSC CGL (Tier-I) – 10/06/2019 (Shift-I)

890

YCT

Ans : (b) Appropriate sequence of the above jumbled sentences will be CBDA. 205. Given below are four jumbled sentences. Select the option that gives their correct order. A. For a split second, as the stranger stepped into the light, his furry head was clearly visible. B. At first, all seemed still outside the house, except for the sound of the rain. C. Recognizing him, Joe stood petrified staring at the man wearing a long coat, wet from rain. D. But then, footsteps approached the house and a stranger emerged out of the darkness. (b) CABD (a) DACB (c) BCDA (d) BDAC SSC CGL (Tier-I) – 10/06/2019 (Shift-I) Ans : (d) Appropriate sequence of the above jumbled sentences will be BDAC. 206. Given below are four jumbled sentences. Select the option that gives their correct order. A. Here, the picture perfect French Quarter and heritage laden Tamil streets blend into one another. B. The conflict of coexistence is evident in many metropolitan cities. C. The old and, like two different worlds, surviving side by side yet never crossing that chasm. D. There is a place though- Puducherry, where contrasts are celebrated like nowhere else. (a) BACD (b) BCDA (c) DCBA (d) CDAB SSC CGL (Tier-I) – 10/06/2019 (Shift-II) Ans : (b) Appropriate sequence of the above jumbled sentences will be BCDA. 207. Given below are four jumbled sentences. Select the option that gives their correct order. A. European cloth manufactures therefore had to depend on a plant called wood to make dyes. B. However, cloth dyers preferred indigo which produced a rich blue colour. C. Being a plant of the temperate zones, wood was easily available in Europe. D. Indian indigo was very expensive and only small quantities reached the European markets. (a) CBDA (b) DCAB (c) ACBD (d) DACB SSC CGL (Tier-I) – 10/06/2019 (Shift-II) Ans : (d) Appropriate sequence of the above jumbled sentences will be DACB. 208. Given below are four jumbled sentences. Select the option that gives their correct order. A. Soon, flames ignited the wooden beams of the grand palace built by Xerxes. B. Men and women holding aloft flaming torches, raced up and down the palace terraces. Parajumbles

C. When the fire died out, all that remained of the magnificent palace were the stone columns. D. Looters fought off the heat of the inferno to drag out gold and silver vessels. (a) ADBD (b) CDBA (c) BADC (d) BCAD SSC CGL (Tier-I) – 10/06/2019 (Shift-III) Ans. (c) : Appropriate sequence of the above jumbled sentences will be BADC. 209. Given below are four jumbled sentences. Select the option that gives their correct order. A. It was very unusual as boys were not supposed to be out school at this late hour. B. He moved closer to the boy in anger so that he could recognise the miscreant and punish him. C. He felt angry as teachers ought to be about school rules being broken. D. Mr Oliver, the school teacher saw a lonely boy sitting on a rock, weeping soundlessly. (a) DBAC (b) ABCD (c) ACBD (d) DACB SSC CGL (Tier-I) – 10/06/2019 (Shift-III) Ans. (a) : Appropriate sequence of the above jumbled sentences will be DBAC. 210. Given below are four jumbled sentences. Select the option that gives their correct order. A. The move follows last week's fatal Ethiopian Airlines crash. B. Aircraft manufacturer Boeing has grounded the entire global fleet of its 737 Max aircraft. C. Last October, a plane from the Indonesiabased carrier Lion Air had also crashed under similar circumstances. D. That was the second time in five months a 737 Max has crashed. (a) ABCD (b) BCDA (c) BADC (d) DABC SSC CGL (Tier-I) – 07/06/2019 (Shift-I) Ans : (c) Appropriate sequence of the above jumbled sentences will be BADC. 211. Given below are four jumbled sentences. Select the option that gives their correct order. A. However, new research shows that taking the dog for a walk can have its downsides for seniors. B. Dogs are great companions and provide a healthy excuse to go for a walk and get a bit of exercise. C. It once seemed common sense to believe that having and walking a dog was good for older people. D. A report published in an American medical journal says injuries among seniors related to dog-walking are becoming increasingly prevalent. (a) ABCD (b) CBAD (c) CABD (d) ADCB SSC CGL (Tier-I) – 07/06/2019 (Shift-I)

891

YCT

Ans : (b) Appropriate sequence of the above jumbled sentences will be CBAD. 212. Given below are four jumbled sentences. Select the option that given their correct order. A. Since then Boeing has received over 5,000 orders for the aircraft and has delivered 371 to date B. The Max first started flying commercially in 2017. C. Boeing had huge hopes for its 737 Max aircraft and viewed it as a key part of its future. D. However, the company's market value has plummeted by nearly $26 billion since the crash in Ethiopia. (a) ABCD (b) CBAD (c) DCBA (d) BCDA SSC CGL (Tier-I) – 07/06/2019 (Shift-II) Ans : (b) Appropriate sequence of the above jumbled sentences will be CBAD. 213. Given below are four jumbled sentences. select that gives their correct order. A. It has been a leading producer of both industrial and agricultural goods. B. The United States of America holds a dominant position in the world because of its high economic development. C. Because of its enormous output, the United States has a major share in world trade. D. It is also a leader in the development and application of innovative technology. (a) BCAD (b) ABCD (c) BACD (d) DABC SSC CGL (Tier-I) – 07/06/2019 (Shift-II) Ans : (c) Appropriate sequence of the above jumbled sentences will be BACD. 214. Given below are four jumbled sentences. Select the option that gives their correct order. A. What can we do to avoid anger and provocation? B. Anger and tension are prevalent due to conflicts arising out of these differences. C. The society if full of people who think and differently. D. We need to develop in ourselves the capacity for conversion to turn negative experiences into positive thinking. (a) ACBD (b) CBAD (c) CADB (d) ABDC SSC CGL (Tier-I) – 07/06/2019 (Shift-III) Ans : (b) Appropriate sequence of the above jumbled sentences will be CBAD. 215. Given below are four jumbled sentences. Select the option that gives their correct order. A. Only the fittest creatures can survive while competing for food. B. Why do some species survive and others become extinct? C. His answer was that there is ceaseless struggle for life among all creatures. D. This was the question that Darwin asked himself. Parajumbles

(a) ADCB (b) ABDC (c) BDCA (d) BCDA SSC CGL (Tier-I) – 07/06/2019 (Shift-III) Ans : (c) Appropriate sequence of the above jumbled sentences will be BDCA. 216. Given below are four jumbled sentences. Select the option that gives their correct order. A. Mango, the so-called ''king of fruits'', is something of a national obsession in India. B. There was a bumper crop of mangoes in different states. C. It resulted in prices coming down and sales going up much to the delight of buyers and sellers alike. D. 2017 proved to be a very good year for mango lovers. (a) CADB (b) CDAB (c) ADCB (d) ADBC SSC CGL (Tier-I) – 06/06/2019 (Shift-I) Ans. (d) : Appropriate sequence of the above jumbled sentences will be ADBC. 217. Given below are four jumbled sentences. Select the option that gives their correct order. A. Around 600 million of them live in areas of high to extreme water stress. B. India is suffering from the worst water crisis, with one billion people living in water scarcity. C. This is even more than that of China and US combined. D. The reason is that at 24 per cent, India uses the most groundwater drawn out globally. (a) ACBD (b) ADCB (c) BADC (d) BDAC SSC CGL (Tier-I) – 06/06/2019 (Shift-I) Ans. (c) : Appropriate sequence of the above jumbled sentences will be BADC. 218. Given below are four jumbled sentences. Select the option that gives their correct order. A. Canada accounts for nearly one-third of the world's production of news print. B. Forest products, therefore, have been important export items of Canada for more than a century. C. A large part of Canada is covered with coniferous trees. D. Pulp and paper are the two most important forest products. (a) ABCD (b) DCAB (c) CBDA (d) BACD SSC CGL (Tier-I) – 06/06/2019 (Shift-II) Ans : (c) Appropriate sequence of the above jumbled sentences will be CBDA. 219. Given below are four jumbled sentences. Select the option that gives their correct order. A. In China, thousands of people celebrated it. B. The Lunar New Year of the Pig began on 5th February. C. Others burned the first joss sticks of the year to bring them good fortune. D. Some performers re-enacted historical ceremonies.

892

YCT

(a) ABCD (b) BADC (c) DABC (d) BDCA SSC CGL (Tier-I) – 06/06/2019 (Shift-II) Ans : (b) Appropriate sequence of the above jumbled sentences will be BADC. 220. Given below are four jumbled sentences. Select the option that gives their correct order. A. What was supposed to be a holiday of a lifetime turned into a dangerous experience for 1,300 passengers on a cruise ship off the coast of Norway? B. The Norwegian Government decided to evacuate the passengers with the help of helicopters. C. The ship broke down in rough seas and high-speed winds caused it to rock heavily from side to side. D. From the first 371 evacuated passengers, 17 had to be taken to hospital. (a) ABCD (b) CBDA (c) CABD (d) ACBD SSC CGL (Tier-I) – 06/06/2019 (Shift-III) Ans. (d) : Appropriate sequence of the above jumbled sentences will be ACBD. 221. Given below are four jumbled sentences. Select the option that gives their correct order. A. It is one of the highly developed countries of the world. B. It stretches from the Atlantic Ocean in the east to the Pacific Ocean in the west. C. The USA consists of 50 states including Alaska and Hawai. D. The United States of America (USA) is the 4th largest country in the world. (a) ADCB (b) CBAD (c) CBDA (d) DBCA SSC CGL (Tier-I) – 06/06/2019 (Shift-III) Ans. (d) : Appropriate sequence of the above jumbled sentences will be DBCA. 222. Given below are four jumbled sentences. Select the option that gives their correct order. A. The cafe's owner says he's interested in conservation, and hopes customers will realise the animals are worth saving, even though they often have a bad reputation. B. None of them are venomous, meaning customers can get up close and personal with the reptiles. C. Here you sip your drink in the company of 35 snakes. D. This cafe, which has just opened in Tokyo, is not for the faint-hearted. (a) DCBA (b) ABCD (c) DBCA (d) ABDC SSC CGL (Tier-I) – 04/06/2019 (Shift-I) Ans : (a) Appropriate sequence of the above jumbled sentences will be DCBA. 223. Given below are four jumbled sentences. Select the option that gives their correct order. A. An environmental group performed a necropsy on the animal and found about 40 kilograms of plastic, including grocery bags and rice sacks. Parajumbles

B. A 4.7-metre long whale died on Saturday in Phillipines where it was stranded a day earlier. C. "It's very disgusting and heartbreaking," he said. "We've done necropsies on 61 dolphins and whales in the last 10 years and this is one of the biggest amounts of plastic we've seen." D. "The animal died from starvation and was unable to eat because of the trash filling its stomach," said Darrell Blatchley, Director of D' Bone Collector Museum Inc...... (a) BACD (b) BADC (c) ABCD (d) DABC SSC CGL (Tier-I) – 04/06/2019 (Shift-I) Ans : (b) Appropriate sequence of the above jumbled sentences will be BADC. 224. Given below are four jumbled sentences. Select the option that gives their correct order. A. That sort of pollution, which is also widespread in other south east Asian nations, regularly kills wildlife like whales and turtles that ingest the waste. B. Environmental groups have tagged the Philippines as one of the world's biggest ocean polluters due to its reliance on single-use plastic. C. In Thailand also, a whale died last year after swallowing more than 80 plastic bags. D. In the latest case, a whale with 40 kilos of plastic trash in its stomach died on Saturday in southern Philippines where it was stranded a day earlier. (a) ABCD (b) BADC (c) DABC (d) BCAD SSC CGL (Tier-I) – 04/06/2019 (Shift-II) Ans : (b) Appropriate sequence of the above jumbled sentences will be BADC. 225. Given below are four jumbled sentences. Select the option that gives their correct order. A. The elephant tusks were tracked from the Democratic Republic of Congo for two months. B. Customs officials in Thailand say it's the biggest seizure in the country's history. C. Four tonnes of ivory, with a market value of $6 million - it was an impressive haul. D. Officials say they were being transported to Loas, from where they believed the ivory would be sold to customers across Asia. (a) CBAD (b) ABCD (c) ACDB (d) CABD SSC CGL (Tier-I) – 04/06/2019 (Shift-II) Ans : (a) Appropriate sequence of the above jumbled sentences will be CBAD. 226. Given below are four jumbled sentences. Select the option that gives their correct order. A. And 844 million don't have access to clean water close to home, according to the latest report by Water Aid. B. Around 4 billion people in the world live in physically water-scarce areas.

893

YCT

C. It is because globally we use six times as much water today as we did 100 years ago. D. The world's water crisis is getting worse. (a) BADC (b) BDCA (c) CADB (d) DACB SSC CGL (Tier-I) – 04/06/2019 (Shift-III) Ans. (a) : Appropriate sequence of the above jumbled sentences will be BADC. 227. Given below are four jumbled sentences. Select the option that gives their correct order. A. An estimated 70% of this plastic which enters the sea sinks. B. This is a problem that stretches far beyond India. C. Eight million tonnes of plastic ends up in the world's oceans every year, causing damage to the fragile ecosystem. D. And much of it is not biodegradable. (a) ADBC (b) CADB (c) ABDC (d) CDBA SSC CGL (Tier-I) – 04/06/2019 (Shift-III) Ans. (b) : Appropriate sequence of the above jumbled sentences will be CADB. 228. Given below are four jumbled sentences. Out of the given options pick the one that gives their correct order. A. Using matches however came much later. B. The first great discovery that man probably made was that of fire. C. In olden times fires were made by rubbing two flints against each other till a spark was produced. D. And this spark set fire to dry straw, leaves or wood. (a) CBDA (b) BDAC (c) ADBC (d) BCDA SSC CHSL (Tier-I) –11/07/2019 (Shift-I) Ans : (d) Appropriate sequence of the above jumbled sentences will be BCDA. 229. Given below are four jumbled sentences. Out of the given options pick the one that gives their correct order. A. We had already covered half the distance when we reached a three-road junction. B. I was worried it might pounce on us. C. But thankfully we picked up speed and it got left behind. D. Suddenly, a barking dog began to chase the bike for some distance. (a) ABDC (b) DACB (c) BDAC (d) ADBC SSC CHSL (Tier-I) –11/07/2019 (Shift-I) Ans : (d) Appropriate sequence of the above jumbled sentences will be ADBC. 230. Given below are four jumbled sentences. Out of the given pick the one that gives their correct order. A. Baskets and carts flooded the street turning it into a mobile market. B. Hearing their cries people with bags started coming out from their houses. Parajumbles

C. Suddenly the street came alive. D. From one end to the other cries of the vendors rent the air. (a) CADB (b) ABDC (c) DACB (d) CBAD SSC CHSL (Tier-I) –11/07/2019 (Shift-II) Ans : (a) Appropriate sequence of the above jumbled sentences will be CADB. 231. Given below are four jumbled sentences. Out of the given options pick the one that gives their correct order. A. Today flax is used to make fine linen fabric. B. The Neolithic man first clad himself with animal skins C. He also used flax which had a good fibre for making cloth. D. But in those days cloth made of flax must have been very rough. (a) CADB (b) BDAC (c) DABC (d) BCDA SSC CHSL (Tier-I) –11/07/2019 (Shift-II) Ans : (d) Appropriate sequence of the above jumbled sentences will be BCDA. 232. Given below are four jumbled sentences. Out of the given option pick the one that gives their correct order. A. In fact, there are about 800 million mobile phone users now. B. They have been in use in the country for over two decades. C. Mobile companies are regularly bringing out phones with new features for these users. D. There are billions of mobile phones in India today. (a) DBAC (b) DCAB (c) ABCD (d) ACBD SSC CHSL (Tier-I) –11/07/2019 (Shift-III) Ans : (a) Appropriate sequence of the above jumbled sentences will be DBAC. 233. Given below are four jumbled sentences. Out of the given options pick the one that gives their correct order. A. One population lives in the Bwindi National Park in Uganda. B. Mountain gorillas are some of the most threatened animals on the planet. C. Only two groups of mountain gorillas remain. D. The other group lives in the Virunga Mountains. (a) BDCA (b) BCAD (c) CDBA (d) ABCD SSC CHSL (Tier-I) –11/07/2019 (Shift-III) Ans : (b) Appropriate sequence of the above jumbled sentences will be BCAD.

894

YCT

234. Given below are four jumbled sentences. Out of the given options pick the one that gives their correct order. A. Gone are the days when you stayed with one firm for your entire career and made lifelong friends in the office. B. This phenomenon was noted in a recent research report which found that the median tenure for employees below the age of 30 is just two years. C. The research report stated that young people today are tempted to which jobs quickly. D. As job longevity becomes a phenomenon of the past, there is a change in attitude in the modern generation towards employment. (a) DBCA (b) BADC (c) CDAB (d) DCBA SSC CHSL (Tier-I) –10/07/2019 (Shift-I) Ans : (a) Appropriate sequence of the above jumbled sentences will be DBCA. 235. Given below are four jumbled sentences. Out of the given options pick the one that gives their correct order. A. I should have charged double the amount, but it didn't occur to me then. B. Now, after all the expenses and the gardener's salary, I made a profit of only five hundred rupees. C. The jasmine plants flowered twice and I sold 250 grams of buds just for ten rupees. D. Had I charged at least twenty rupees I would have made a good profit. (a) ADCB (b) BCDA (c) CADB (d) BDAC SSC CHSL (Tier-I) –10/07/2019 (Shift-I) Ans : (c) Appropriate sequence of the above jumbled sentences will be CADB. 236. Given below are four jumbled sentences. Out of the given options pick the one that gives their correct order. A. Today, he is popular for creating scaleddown models of automobiles that are not just showpieces but are working toys. B. "I used to ride it during playtime in the evenings as a five-year old", recalls Arun Kumar. C. As a kid I used to pester my parents for toy cars but my carpenter father often couldn't afford them. D. Then, one day he got an old tricycle from the junkyard, made three wooden wheels and fitted them to the cycle. (a) ACDB (b) CDBA (c) CADB (d) BCDA SSC CHSL (Tier-I) –10/07/2019 (Shift-II) Ans : (b) Appropriate sequence of the above jumbled sentences will be CDBA. Parajumbles

237. Given below are four jumbled sentences. Out of the given options pick the one that gives their correct order. A. As man's intelligence grew so did his power. B. This intelligence made him cleverer and stronger than enormous animals which would otherwise have destroyed him. C. The chief difference between man and the other animals was the intelligence of man. D. Combining intelligence with power, man developed weapons to fight his enemies. (a) DABC (b) CBAD (c) ACDB (d) BCDA SSC CHSL (Tier-I) –10/07/2019 (Shift-II) Ans : (b) Appropriate sequence of the above jumbled sentences will be CBAD. 238. Given below are four jumbled sentences. Out of the given options pick the one that gives their correct order. A. Yes, first get us some fresh juice and some sandwiches. B. May I take your order? C. Not now, maybe a little later. D. Sure. Would you like some coffee too? (a) CDBA (b) ABCD (c) BADC (d) BCDA SSC CHSL (Tier-I) –10/07/2019 (Shift-III) Ans: (c) Appropriate sequence of the above jumbled sentences will be BADC. 239. Given below are four jumbled sentences. Out of the given options pick the one that gives their correct order. A. Then she does some gardening. B. Finally before her mid morning nap, she eats a light breakfast. C. My grandmother is an active eighty year old. D. She begins her morning with meditation. (a) CDBA (b) DCBA (c) CABD (d) CDAB SSC CHSL (Tier-I) –10/07/2019 (Shift-III) Ans : (d) Appropriate sequence of the above jumbled sentences will be CDAB. 240. Given below are four jumbled sentences. Out of the given options pick the one that gives their correct order. A. For them, it's like a magic carpet, their version of a car. B. All kids went to be older, so from an early age they imitate their parents or siblings. C. While cycling they imagine they are doing adult things or being adults. D. Bikes offer them a chance to be independent. (b) DCBA (a) ADCB (c) BDAC (d) BACD SSC CHSL (Tier-I) –09/07/2019 (Shift-I) Ans : (c) Appropriate sequence of the above jumbled sentences will be BDAC.

895

YCT

241. Given below are four jumbled sentences. Out of the given options pick the one that gives their correct order. A. The woman is wearing an evening dress and heavy make up, which is now smeared across her cheeks. B. But, who is she, and how did she get there? C. It's seven in the morning and the Bantrys wake to find the body of a young woman in their library. D. The respectable Bantrys invite Miss Marple to solve the mystery before tongues start to wag. (a) CDBA (b) BACD (c) CABD (d) DCBA SSC CHSL (Tier-I) –09/07/2019 (Shift-I) Ans : (c) Appropriate sequence of the above jumbled sentences will be CABD. 242. Given below are four jumbled sentences. Out of the given options pick the one that gives their correct order. A. His affectionate greeting was surprising. B. Although we were from the same village our relationship was confind to a polite hello whenever our paths crossed. C. As I stepped into the shack, Dorababu beckoned to me, making a little space for me on the bench on which he was sitting. D. "Come here, brother", he greeted me. (a) DBCA (b) CABD (c) BACD (d) CDAB SSC CHSL (Tier-I) –09/07/2019 (Shift-II) Ans : (d) Appropriate sequence of the above jumbled sentences will be CDAB. 243. Given below are four jumbled sentence. Out of the given options pick the one that gives their correct order. A. The first person who thought of trading must have been a clever person. B. He must have found them easy to carry and exchange for goods. C. And gradually paying for things in gold and silver became a regular practice. D. When gold and silver were first found people started using them for trade. (a) DBCA (b) BDCA (c) CDAB (d) DABC SSC CHSL (Tier-I) –09/07/2019 (Shift-II) Ans : (d) Appropriate sequence of the above jumbled sentences will be DABC. 244. Given below are four jumbled sentences. Out of the given options pick the one that gives their correct order. A. Today, man in master of the world and he makes the animals do what the likes. B. Others he eats; and those like the lion and the tiger, he shoots for pleasure. C. Early man, when he first came, must have been surrounded by many huge animals, end he must have lived in fear of them. Parajumbles

D. Some he tames like the horse, the cow, the elephant, the dog, the cat and so many others. (a) CADB (b) ACBD (c) BCDA (d) CBDA SSC CHSL (Tier-I) –09/07/2019 (Shift-III) Ans : (a) Appropriate sequence of the above jumbled sentences will be CADB. 245. Given below are four jumbled sentences. Out of the given options pick the one that gives their correct order. A. But now that his father had asked him to fetch some cheroots, he would have to pass by the high school to reach the shop. B. The teachers and students would all be there, and he was ashamed of being seen outdoors. C. Of late Krishna had confined himself to the four walls of the house. D. Wondering how to get past the School unnoticed, he slowly ventured out. (a) DACB (b) CBDA (c) CABD (d) ACBD SSC CHSL (Tier-I) –09/07/2019 (Shift-III) Ans : (c) Appropriate sequence of the above jumbled sentences will be CABD. 246. Given below are four jumbled sentences. Out of the given options pick the one that gives their correct order. A. But when their short racing careers are over, greyhounds are adopted as household pets. B. Greyhound racing is a popular spectator sport in the United States. C. They worry that the greyhounds would need a large space to run to get sufficient exercise. D. Unfortunately, many people hesitate to adopt a retired racing greyhound as a pet. (a) DACB (b) DBCA (c) BADC (d) BDAC SSC CHSL (Tier-I) –08/07/2019 (Shift-I) Ans : (c) Appropriate sequence of the above jumbled sentences will be BADC. 247. Given below are four jumbled sentences. Out of the given options pick the one that gives their correct order. A. Little did she know that I was out of job once again. B. But all the money soon vanished into the bottomless pit of household needs. C. I returned home with my pockets full of my earnings. D. However, my grandmother was happy that I had become responsible. (a) DBAC (b) CBDA (c) BCDA (d) CDAB SSC CHSL (Tier-I) –08/07/2019 (Shift-I) Ans : (b) Appropriate sequence of the above jumbled sentences will be CBDA.

896

YCT

248. Given below are four jumbled sentences. Out of the given options pick the one that gives their correct order. A. We even fail to recognize that their call of duty keeps them away from their families. B. Yet, we often fail to appreciate these sacrifices that they make. C. They make countless sacrifices for the nation and keep us safe from our enemies. D. The uniformed men and women are our unsung heroes. (b) ABCD (a) DCBA (c) CBAD (d) DABC SSC CHSL (Tier-I) –08/07/2019 (Shift-II) Ans : (a) Appropriate sequence of the above jumbled sentences will be DCBA. 249. Given below are four jumbled sentences. Out of the given options pick the one that gives their correct order. A. As a result, he had gotten into a million activities and hobbies. B. Thus, his parents were now reluctant to spend any more money on his hobbies. C. John hated finishing things almost as much as he loved starting them. D. But he never stuck with any of them for long. (a) ADCB (b) CADB (c) BCAD (d) CDBA SSC CHSL (Tier-I) –08/07/2019 (Shift-II) Ans : (b) Appropriate sequence of the above jumbled sentences will be CADB. 250. Given below are four jumbled sentences. Out of the given options pick the one that gives their correct order. A. Till quite recently the trend was to go to the west that offered advanced medical B. This is because it has on offer the highest level of services, facilities and professional skills. C. But over the last decade the trend has reversed and India is being chosen for medical tourism. D. Today, it is not uncommon to travel to a foreign country for medical treatment. (a) BCDA (b) DACB (c) CBDA (d) ABCD SSC CHSL (Tier-I) –08/07/2019 (Shift-III) Ans: (b) Appropriate sequence of the above jumbled sentences will be DACB. 251. Given below are four jumbled sentences. Out of the given options pick the one that gives their correct order. A. This kindness converted his street into a neighbourhood. B. He and his wife were always ready to lend a helping hand. C. Mohan was an ideal neighbour in every way. D. Whether it was lending tools or watering fields, the couple embodied neighbourly kindness. Parajumbles

(a) DBCA (b) CBDA (c) CDAB (d) BDAC SSC CHSL (Tier-I) –08/07/2019 (Shift-III) Ans : (b) Appropriate sequence of the above jumbled sentences will be CBDA. 252. Given below are four jumbled sentences. Out of the given options pick the one that gives their correct order. A. The fire destroyed many buildings which were rebuilt with red clay tiles. B. In the year 1689, a huge fire engulfed the city. C. Prague in the Czech Republic is known for its striking red roofs. D. Even now, traditional red roofs are included in any rapairs or new construction. (a) CADB (b) DCBA (c) CBAD (d) BADC SSC CHSL (Tier-I) –05/07/2019 (Shift-I) Ans : (c) Appropriate sequence of the above jumbled sentences will be CBAD. 253. Given below are four jumbled sentences. Out of the given options pick the one that gives their correct order. A. This code contains information about its nature. B. Each living organism has a DNA code. C. If this genetic material is altered, it would alter the organism. D. The nature of an individual is determined by the unique genetic material in the code. (a) BCDA (b) BADC (c) DCAB (d) DBCA SSC CHSL (Tier-I) –05/07/2019 (Shift-I) Ans : (b) Appropriate sequence of the above jumbled sentences will be BADC. 254. Given below are four jumbled sentences. Out of the given options pick the one that gives their correct order. A. Yet, whatever was printed was profound and even inspirational. B. In pre-independence India there were two kinds of newspapers. C. The other spoke of the aspirations of the people and pointed out injustice. D. One set was supportive of the colonial power. (a) BACD (b) DCAB (c) ABDC (d) BDCA SSC CHSL (Tier-I) –05/07/2019 (Shift-II) Ans : (d) Appropriate sequence of the above jumbled sentences will be BDCA. 255. Given below are four jumbled sentences. Out of the given options pick the one that gives their correct order. A. Yet, it is India that is the home of the malnourished child. B. Some major ones are lack of food, sanitation and proper healthcare.

897

YCT

C. The reasons for malnutrition among Indian children are many. D. Millions of rupees have been spent on child care in the country. (a) DCAB (b) CBAD (c) ACBD (d) DACB SSC CHSL (Tier-I) –05/07/2019 (Shift-II) Ans : (d) Appropriate sequence of the above jumbled sentences will be DACB. 256. Given below are four jumbled sentences. Out of the given options pick the one that gives their correct order. A. They used thin twigs called chew sticks tooth cleaners. B. The fuzzy end was rubbed against the teeth to keep them clean. C. These sticks were fuzzy at one end. D. How did people clean their teeth before the toothbrush was invented? (a) DACB (b) BCAD (c) ACBD (d) DBCA SSC CHSL (Tier-I) –05/07/2019 (Shift-III) Ans : (a) Appropriate sequence of the above jumbled sentences will be DACB. 257. Given below are four jumbled sentences. Out of the given options pick the one that gives their correct order. A. Her father had been stuck in elevator for six hours. B. Esha had heard from her parents about a blackout caused by a sudden storm. C. She had to grope in the dark and cook by the candlelight. D. Her mother's story had been quite different from his. (a) BDCA (b) ADCB (c) BADC (d) CADB SSC CHSL (Tier-I) –05/07/2019 (Shift-III) Ans : (c) Appropriate sequence of the above jumbled sentences will be BADC. 258. Given below are four jumbled sentences. Out of the given options pick the one that gives their correct order. A. On the morning of April 12 Gopal rose at 5.30 am. B. Then he drove to the Joggers park which was four km away. C. There he met his group of walking enthusiasts. D. He put on his track suit and walking shoes. (a) CBAD (b) ADBC (c) DCAB (d) ACBD SSC CHSL (Tier-I) –04/07/2019 (Shift-I) Ans : (b) Appropriate sequence of the above jumbled sentences will be ADBC. 259. Given below are four jumbled sentences. Out of the given options pick the one that gives their correct order. A. He meets me in the same frank friendly way as always without mentioning it. B. He has owed it to me for twelve months now. Parajumbles

C. I can realise whenever I meet him that he has forgotten that he owes me a dollar. D. My friend Todd owes me a dollar. (a) CBAD (b) CBDA (c) ADBC (d) DBCA SSC CHSL (Tier-I) –04/07/2019 (Shift-I) Ans : (d) Appropriate sequence of the above jumbled sentences will be DBCA. 260. Given below are four jumbled sentences. Out of the given options pick the one that gives their correct order. A. Children are the most adversely affected. B. Their eyesight goes weak from their very childhood. C. Watching television continuously is very harmful for the eyes. D. This debilitating effect on eyes results is severe headaches as well. (a) CABD (b) ACBD (c) ABDC (d) CBDA SSC CHSL (Tier-I) –04/07/2019 (Shift-II) Ans : (a) Appropriate sequence of the above jumbled sentences will be CABD. 261. Given below are four jumbled sentences. Out of the given options pick the one that gives their correct order. A. They have no mind of their own. B. It is misleading to imagine that computers can think like humans. C. However, they make it possible for people to accumulate thoughts. D. They cannot do so. (a) ACBD (b) ABCD (c) BDAC (d) BACD SSC CHSL (Tier-I) –04/07/2019 (Shift-II) Ans : (c) Appropriate sequence of the above jumbled sentences will be BDAC. 262. Given below are four jumbled sentences. Out of the given options pick the one that gives their correct order. A. They had come from different parts of the city. B. Men, tall and skinny, short and fat gathered in Dubai. C. They were all named Mohammadi D. But, all of them had one thing in common. (a) BADC (b) BDAC (c) ADCB (d) ABCD SSC CHSL (Tier-I) –04/07/2019 (Shift-III) Ans : (a) Appropriate sequence of the above jumbled sentences will be BADC. 263. Given below are four jumbled sentences. Out of the given options pick the one that gives their correct order. A. Practicing hard, she hoped to win an Olympic medal some day. B. As she sped down the slope she had a trrible accident. C. But, she didn't know it would be her last run ever.

898

YCT

D. Ann Thompson was going to make her practice ski run before lunch. (a) DCBA (b) BCDA (c) DACB (d) ACDB SSC CHSL (Tier-I) –04/07/2019 (Shift-III) Ans : (c) Appropriate sequence of the above jumbled sentences will be DACB. 264. Given below are four jumbled sentences. Out of the given options pick the one that gives their correct order. A. But after a while they all began to blur in our memories. B. When we arrived in Bankok, we took a tour of the city's famous Buddhist temples. C. However there was one temple, the Temple of Golden Buddha, which left an indelible impression in our hearts and minds. D. We visited numerous temples that day. (a) BACD (b) DCBA (c) ADBC (d) BDAC SSC CHSL (Tier-I) –03/07/2019 (Shift-I) Ans : (d) Appropriate sequence of the above jumbled sentences will be BDAC. 265. Given below are four jumbled sentences. Out of the given option pick the one that gives their correct order. A. There was an old apple tree in the centre of a village. B. The adults also sat in the shade and discussed important things. C. They remembered that they too had played around the tree when they were young. D. Children loved to play around the tree and pick delicious apples. (a) ADBC (b) DCAB (c) CBAD (d) ACBD SSC CHSL (Tier-I) –03/07/2019 (Shift-I) Ans : (a) Appropriate sequence of the above jumbled sentences will be ADBC. 266. Given below are four jumbled sentences. Out of the given options pick the one that gives their correct order. A. But once when Mark disturbed the whole class with his pranks I had to reprimand him. B. He was in third grade when I taught at Saint Mary's School. C. All 34 students were dear to me, but Mark Eklund, was one in a million. D. He had a happy-to-be-alive attitute that made even his occasional mischievousness delightful. (a) CBDA (b) BADC (c) ADBC (d) CADB SSC CHSL (Tier-I) –03/07/2019 (Shift-II) Ans. (a) : Appropriate sequence of the above jumbled sentences will be CBDA. Parajumbles

267. Given below are four jumbled sentences. Out of the given options pick the one that gives their correct order. A. Once Oliver made a big snowman. B. The fat snowman wore an old top hat and a muffler. C. Now, it looked very grand indeed and Oliver's friends loved it. D. It was as big as him but much fatter. (a) ADBC (b) CBAD (c) ACBD (d) DCAB SSC CHSL (Tier-I) –03/07/2019 (Shift-II) Ans. (a) : Appropriate sequence of the above jumbled sentences will be ADBC. 268. Given below are four jumbled sentences. Out of the given options pick the one that gives their correct order. A. The subjects taught in the Gurukul varied from Sanskrit to Mathematics. B. Indian education has its roots in the ancient ages. C. However this system changed when the British came to India. D. In those days the Gurukul system was followed. (a) DCAB (b) BCAD (c) CBAD (d) BDAC SSC CHSL (Tier-I) –03/07/2019 (Shift-III) Ans. (d) Appropriate sequence of the above jumbled sentences will be BDAC. 269. Given below are four jumbled sentences. Out of the given options pick the one that gives their correct order. A. At this speed the rocket soon attained the height of 190 miles above the earth. B. At 9 am the great rocket lifted in the air with a mighty roar. C. It rose smoothly at first, then quickened to a speed of 17,500 miles an hour. D. Gagarin now pulled himself towards the window to look down. (a) ADBC (b) CBAD (c) BCAD (d) BADC SSC CHSL (Tier-I) –03/07/2019 (Shift-III) Ans. (c) Appropriate sequence of the above jumbled sentences will be BCAD. 270. Given below are four jumbled sentences. Out of the given options pick the one that gives their correct order. A. Most of them are in jail because of their circumstances. B. Not all prisoners are hardcore criminals. C. So, they deserve a chance to rehabilitate themselves. D. Therefore, learning a trade would help them reintegrate with society. (a) BDCA (b) ABCD (c) DCBA (d) BACD SSC CHSL (Tier-I) –02/07/2019 (Shift-I) Ans. (d) : Appropriate sequence of the above jumbled sentences will be BACD.

899

YCT

271. Given below are four jumbled sentences. Out of the given options pick the one that gives their correct order. A. He often asked questions which were strange and witty. B. Emperor Akbar was in the habit was in the habit of putting riddles and puzzles to his courtiers. C. Once has asked a strange question that confused everyone. D. It took much wisdom to answer these questions. (a) BADC (b) CABD (c) ABCD (d) BACD SSC CHSL (Tier-I) –02/07/2019 (Shift-I) Ans. (a) : Appropriate sequence of the above jumbled sentences will be BADC. 272. Given below are four jumbled sentences. Out of the given options pick the one that gives their correct order. A. In the opening scene, the young boy Pip meets an escaped convict in a graveyard. B. 'Great Expectations' is a novel by Charles Dickens. C. The novel is set in Kent and London in the early 19th century. D. It depicts the personal growth and development of an orphan named Pip. (a) ABCD (b) DCAB (c) CBAD (d) BCDA SSC CHSL (Tier-I) –02/07/2019 (Shift-II) Ans : (d) Appropriate sequence of the above jumbled sentences will be BCDA. 273. Given blow are four jumbled sentences. Out of the given options pick the one that gives their correct order. A. One fine morning, a hunter was getting ready to go hunting. B. He pulled the blanket over the sleeping child. C. As he did so, he thought the blanket was not warm enough. D. Before departing, he went to see his little baby who was sleeping in a crib. (a) ACBD (b) ADBC (c) DCAB (d) CBAD SSC CHSL (Tier-I)–02/07/2019 (Shift-II) Ans : (b) Appropriate sequence of the above jumbled sentences will be ADBC. 274. Given below are four jumbled sentences. Out of the given options pick the one that gives their correct order. A. Early on Christmas morning Pip returns to the graveyard with the file, a pie and brandy. B. On Christmas Eve, Pip, an orphan who is about seven years old, encounters an escaped convict in the village graveyard. C. The convict scares Pip into stealing food and a file. Parajumbles

D. Pip, who lives with his elder sister and her husband Joe Gargery, a blacksmith, steals a file and some food. (a) DCAB (b) CBAD (c) BDCA (d) BCDA SSC CHSL (Tier-I) –02/07/2019 (Shift-III) Ans : (d) Appropriate sequence of the above jumbled sentences will be BCDA. 275. Given below are four jumbled sentences. Out of the given options pick the one that gives their correct order. A. In a small town in England, there lived a poor little girl. B. She was very excited but her right shoe was worn out. C. So, she went to a cobbler to get it mended. D. Her friend invited her to come to her birthday party. (a) ADBC (b) ACBD (c) DCAB (d) CBAD SSC CHSL (Tier-I) –02/07/2019 (Shift-III) Ans : (a) Appropriate sequence of the above jumbled sentences will be ADBC. 276. Given below are four jumbled sentences. Out of the given options pick the one that gives their correct order. A. Later, they set up their own art school and gallery in the prison. B. The prisoners wanted to display their art work inside the prison. C. They took the help of volunteers from the School of Art. D. Prisoners who were interested in visual arts and painting could study there. (a) BCAD (b) DCAB (c) BDAC (d) CBAD SSC CHSL (Tier-I) –01/07/2019 (Shift-III) Ans : (a) Appropriate sequence of the above jumbled sentences will be BCAD. 277. Given below are four jumbled sentences. Out of the given options pick the one that gives their correct order. A. As he looked at them, their heads began to hang low. B. Akbar glanced at his courtiers in search of an answer. C. Once Akbar asked his courtiers a very strange question. D. The courtiers were dumb-founded by his question. (a) CDBA (b) CADB (c) ACBD (d) DCAB SSC CHSL (Tier-I) –01/07/2019 (Shift-III) Ans : (a) Appropriate sequence of the above jumbled sentences will be CDBA.

900

YCT

278. Given below are four jumbled sentences. Out of the given options select the one that gives their correct order. A. He went into a part of the city I had never seen before and walked around a few alleys. B. At one point I distinctly saw him turn around, as if to make sure no one was following him; then he darted away. C. From my conversation with the cook I had learned that Ram Parshad had started to head out of the house at the same time every evening. D. One day I followed at a distance to know where he went every day. (a) ABCD (b) DBAC (c) CABD (d) CDAB SSC CHSL (Tier-I) 26/10/2020 (Shift-II) Ans. (d) : Appropriate sequence of the above jumbled sentences will be CDAB. 279. Given below are four jumbled sentences. Out of the given options select the one that gives their correct order. A. You can also wear a hat to cover your hair and scalp to protect them from toxic particles that could be harmful. B. Therefore, when you are exposed to bad quality air, try and wear full sleeved clothes to cover as much of your skin as possible. C. All the above precautions could save you from serious skin diseases. D. Air pollution plays a major role in skin and hair related problems for which you need to take precautions. (a) CADB (b) BACD (c) DBAC (d) DCAB SSC CHSL (Tier-I) 26/10/2020 (Shift-II) Ans. (c) : Appropriate sequence of the above jumbled sentences will be DBAC. 280. Given below are four jumbled sentences. Select the option that gives their correct order. A. lbn Al-Haitham wrote a book in seven volumes namely 'Kitab al-Manazir' also known as 'The Book of Optics.' B. However, during the medieval period, the notable work of an Arab writer and scientist, lbn Al-Haitham revolutionised the concept of physics. C. During the ancient period, the development of Physics took place simultaneously with the development of astronomy. D. In this book, lbn Al-Haitham disproved the ancient Greek concept of vision and introduced a new theory. (a) ACBD (b) ADBC (c) CBAD (d) BCDA SSC CHSL (Tier-I) 17/03/2020 (Shift-I) Ans. (d) Appropriate sequence of the above jumbled sentences will be BCDA. Parajumbles

281. Given below are four jumbled sentences, Select the option that gives their correct order. A. "It is mine. I saw it first," claimed one cat. B. Suddenly they spotted a loaf of bready lying beneath a tree. C. Once upon a time, two cats were passing through a street. D. Both pounced upon it and cought the loaf at the same time. (a) DBCA (b) ADBC (c) CBDA (d) BADC SSC CHSL (Tier-I) 17/03/2020 (Shift-I) Ans. (c) Appropriate sequence of the above jumbled sentences will be CBDA. 282. Given below are four jumbled sentences. Out of the given options pick the one that gives their correct order. A. Soon, the idea took its final shape and she started selling barfis under the brand name Harbhajan's and the tagline "You’ll remember your childhood”. B. That's when Raveena suggested that she should start selling her signature 'Besan ki Barfi'. C. Today Harbhajan Kaur is truly living by these words as she now sells tasty barfi and makes good money. D. It all started in 2016, when 94 year old Harbhajan's daughter asked her if she had any regrets in life, to which she expressed that she never earned a penny for herself. (a) DBAC (b) CBAD (c) DCAB (d) BCDA SSC CHSL (Tier-I) 16/10/2020 (Shift-III) Ans. (a) : Appropriate sequence of the above jumbled sentences will be DBAC. 283. Given below are four jumbled sentences. Out of the given options pick the one that gives their correct order. A. Einstein had a deep distaste for enforced learning, and was known to skip many of his college lectures. B. Einstein was also severely critical of the exam-based education system. C. The great physicist Galileo too said, “You cannot teach anybody anything. You can only help them discover it within themselves.” D. He often felt that formal education interfered with his natural desire to learn and explore. (a) DABC (b) ADBC (c) CABD (d) ACDB SSC CHSL (Tier-I) 16/10/2020 (Shift-III) Ans. (b) : Appropriate sequence of the above jumbled sentences will be ADBC.

901

YCT

284. Given below are four jumbled sentences. Out of the given options pick the one that gives their correct order. A. He flew down, searched for her until he found her and then asked her to become his wife. B. A hawk flying about in the sky one day decided that he would like to marry a hen whom he often saw on earth. C. The hawk was delighted and flew away after giving her a ring as an engagement present, and telling her to take good care of it. D. She at once gave her consent on the condition that he would wait until she could grow wings like his, so that she might also fly high. (a) BDCA (b) BCDA (c) BADC (d) ADCB SSC CHSL (Tier-I) 16/10/2020 (Shift-II) Ans. (c) : Appropriate sequence of the above jumbled sentences will be BADC. 285. Given below are four jumbled sentences. Out of the given options pick the one that gives their correct order. A. Some such common events that bring us happiness for a short time include passing an exam, getting a job, going on a vacation or buying a dream house. B. Happiness is more difficult to achieve than peace as we keep striving for it daily. C. Peace, on the other hand, is sustainable for many years, costs less energy, and doesn't depend on external events such as these. D. Events that we struggle for in our daily lives to bring us happiness only come randomly, stay for a short time and subside. (a) BDAC (b) ACDB (c) BCAD (d) BACD SSC CHSL (Tier-I) 16/10/2020 (Shift-II) Ans. (a) : Appropriate sequence of the above jumbled sentences will be BDAC. 286. Given below are four jumbled sentences. Pick the option that gives their correct order. A. Finally, she takes the decision to go searching for livelihood in the nearby town. B. They are: Lakua and his wife, Nathuni. C. The latter, burdened with a paralytic husband and two children. battles to survive. D. The film 'Wayfareres' narrates the journey of a poor couple in Jharkhand. (a)DBAC (b) BACD (c)CDAB (d) DBCA SSC CHSL (Tier-I) 21/10/2020 (Shift-II) Ans. (d) : Appropriate sequence of the above jumbled sentences will be DBCA. Parajumbles

287. Given below are four jumbled sentences. Out of the given options, pick the one that gives their correct order. A. While the most common belief is that these traits remain fixed, research studies show otherwise! B. One such research study suggests that we can change our traits provided we want to change them. C. We all possess certain personality traits that set us apart from the rest. D. These traits, a mix of good and bad, define how we respond to situations and people. (a) ACDB (b) CBDA (c) CDAB (d) BCAD SSC CHSL (Tier-I) 16/10/2020 (Shift-I) Ans. (c) : Appropriate sequence of the above jumbled sentences will be CDAB. 288. Given below are four jumbled sentences. Out of the given options, pick the one that gives their correct order. A. Vijay the bus conductor was always there in one of these trucks. B. He had now quit his old job and joined politics. C. In the week before the elections, trucks bumped up and down the dirty streets of Laxmangarh, full of young men holding microphones and shouting: “Stand up to the rich!” D. That was the thing about Vijay; each time you saw him, he had done better for himself. (b) CBAD (a) CABD (c) ABCD (d) DABC SSC CHSL (Tier-I) 16/10/2020 (Shift-I) Ans. (a) : Appropriate sequence of the above jumbled sentences will be CABD. 289. Given below are four jumbled sentences, Pick the option that gives their correct order. A. The Emperor would view the comic acts from the jharokha, a special window of his palace. B. On the festival of Holi, the Emperor was entertained with mimicry. impersonation and play-acting. C. Bahadur Shah Jafer's life and times are described in a book 'City of my heart.' D. Finally, the most talented of the performers would be generously awarded by the king. (a) CBAD (b) DACB (c) ADBC (d) CDBA SSC CHSL (Tier-I) 21/10/2020 (Shift-II) Ans. (a) : Appropriate sequence of the above jumbled sentences will be CBAD. 290. Given below are four jumbled sentences. Out of the given options select the one that gives their correct order. A. Their classmate, Koushal, hardly studied and spent much of his time wasting on prattles.

902

YCT

B. He made himself busy by collecting old pens and used papers in order to avoid studies. C. Poonam, the class teacher, was very much loved and respected by her students. D. Three of them were very intelligent and spent most of their time studying with Poonam, (a) CDBA (b) CDAB (c) DCAB (d) CADB SSC CHSL (Tier-I) 19/03/2020 (Shift-III) Ans. (b) : Appropriate sequence of the above jumbled sentences will be CDAB. 291. Given below are four jumbled sentences, Out of the given options select the one that gives their correct order. A. The boat floated down the river and reached the other bank. B. Thinking that she was dead, her friends secretly took her and abandoned her in a boat. C. the people on the bank tool her to a hospital and saved her. D. Smelling a flower given by the magician, Shweta fell down unconscious. (a) DBAC (b) DABC (c) ACDB (d) BCAD SSC CHSL (Tier-I) 19/03/2020 (Shift-III) Ans. (a) : Appropriate sequence of the above jumbled sentences will be DBAC. 292. Given below are four jumbled sentences. Select the option that gives their correct order. A. A common sight now is that on the roads of Yangon antiquated cars move alongside shiny new vehicles. B. A newly-built highway connects Mandaly to the city of Rangoon, now called Yangon. C. Similarly, hurriedly constructed glassfronted buildings stand next to old colonial structures. D. Yangon is fairly typical of any developing city in Asia, standing as it does at the crossroads of modernisation. (a) BDAC (b) ADCB (c) CDBA (d) DABC SSC CHSL (Tier-I) 17/03/2020 (Shift-II) Ans. (a) : Appropriate sequence of the above jumbled sentences will be BDAC. 293. Given below are four jumbled sentences. Select the option that gives their correct order. A. From the outer entrance itself he had noticed that there was no one inside. B. It was quite dark by the time Raman turned into the lane and reached the temple. C. On entering the main temple, he prostrated before the goddess, got up and then sat directly in front of the idol. D. However, an oil lamp which was burning before the idol of kaali ma lit up the place and he could go up the steps. Parajumbles

(a) DBCA (b) BADC (c) CABD (d) ABDC SSC CHSL (Tier-I) 17/03/2020 (Shift-II) Ans. (b) : Appropriate sequence of the above jumbled sentences will be BADC. 294. Given below are four jumbled sentences. Select the option that gives their correct order. (A) For example, our project files were kept pending for a few years. (B) However, such hindrances did not stop us from tirelessly working towards the establishment of 450 clinics. (C) We initially constructed over 150 clinics. (D) But very soon many roadblocks were put in our path of expansion. (a) DCAB (b) CDAB (c) CDAB (d) CBDA SSC CHSL (Tier-I) 13/10/2020 (Shift-I) Ans. (c) : Appropriate sequence of the above jumbled sentences will be CDAB. 295. Given below are four jumbled sentences. Select the option that gives their correct order. (A) Some returned in the 80's but they were not allowed to reclaim or buy land. (B) In the 70's many Vietnamese living in Cambodia faced deportation after a change in government. (C) Their only solution was to live in wooden homes floating on the lake. (D) Even today they can be found living on the lake surviving on fishing and tourism. (a) CBDA (b) ABDC (c) BACD (d) DACB SSC CHSL (Tier-I) 13/10/2020 (Shift-I) Ans. (c) : Appropriate sequence of the above jumbled sentences will be BACD. 296. Given below are four jumbled sentences. Out of the given options select the one that given their correct order. A. Navin and Jathin gathered a group of young men and trained them in fighting. B. They wanted to build their own village where people could live without fear. C. They lived in a jungle hideout on the banks of a river in the village. D. Navin and Jatin were brothers and brave fighters. (a) DCAB (b) DCBA (c) DBAC (d) DBCA SSC CHSL (Tier-I) 19/03/2020 (Shift-I) Ans. (a) : Appropriate sequence of the above jumbled sentences will be DCAB. 297. Given below are four jumbled sentences. Out of the given options select the one that gives their correct order. A. Anush was very afraid of stray dogs. B. Anush's mother saw the dog chasing her son. C. One evening, when a street dog barked at him, Anush turned and started running. D. The dog ran after him barking loudly and fiercely.

903

YCT

(a) DCAB (b) ADBC C. "Just take out all the grey strands from my (c) ABCD (d) ACDB beard and make me look very young." SSC CHSL (Tier-I) 19/03/2020 (Shift-I) D. Being the first day, a lot of people came and among them was a man in his early Ans. (d) : Appropriate sequence of the above jumbled fifties with a flowing beard. sentences will be ACDB. (a) DBCA (b) CDAB 298. Given below are four jumbled sentences. Out of (c) ADBC (d) BDAC the given options select the one that gives their SSC CHSL (Tier-I) 18/03/2020 (Shift-II) correct order. A. Though John found none, he was not ready Ans. (c) : Appropriate sequence of the above jumbled sentences will be ADBC. to give in. B. He continued the search till noon. 302. Given below are four jumbled sentences. Out of C. John wanted to have a big well dug. the given options select the one that gives their D. He searched around for workers who correct order. would do the job for him without pay. A. In the evening, Tejaswini would sing songs (a) CADB (b) DCAB praising the Lord. (c) CDAB (d) CDBA B. She would go to the Lord's temple twice a SSC CHSL (Tier-I) 18/03/2020 (Shift-III) day. Ans. (c) : Appropriate sequence of the above jumbled C. Tejaswini was known in the village for her sentences will be CDAB. devotion to the Lord. D. In the morning, she would take with her a 299. Given below are four jumbled sentences. Out of the given options select the one that gives their pot of milk and a bunch of flowers as correct order. offering. A. When Franco was young; he would stay at (a) ACDB (b) DCAB sea all night. (c) CADB (d) CBDA B. Every morning, he would arrive with a big SSC CHSL (Tier-I) 18/03/2020 (Shift-I) catch of fresh fish and sell it in the market. Ans. (d) : Appropriate sequence of the above jumbled C. Franco was a fisherman who lived on the sentences will be CBDA. beach. 303. Given below are four jumbled sentences. Out of D. He had been fishing for as long as he could the given options select the one that gives their remember. correct order. (a) CBDA (b) CDBA A. One day, a strange crow from the west, (c) DCAB (d) CDAB landed on one of the branches. SSC CHSL (Tier-I) 18/03/2020 (Shift-III) B. There was a large mango tree deep inside a Ans. (d) : Appropriate sequence of the above jumbled thick forest. sentences will be CDAB. C. The branches were full of leaves, which 300. Given below are four jumbled sentences. Pick crackled when the storm blew. the option that gives their correct order. D. Its branches spread in all directions, A. Waqar instead gave it to Afridi, who went lobbing a large shadow on the ground. on to score a 37-ball century, the fastest (a) CDBA (b) BCAD ODI century? (c) BDCA (d) ACDB B. That was when his teammate Waqar SSC CHSL (Tier-I) 18/03/2020 (Shift-I) Younis handed him over a bat that Ans. (c) : Appropriate sequence of the above jumbled belonged to Sachin Tendulkar. sentences will be BDCA. C. When Shahid Afridi had flown in to join 304. Given below are four jumbled sentences. Out of the Pakistan team 1996, he did not even the given options pick the one the gives their have a proper bat to play with. correct order. D. Sachin had earlier gifted Waqar his bat to A. He commanded that Birbal should leave take to Sialkot, Pakistan's world famous sports goods manufacturing capital and get his kingdom. a custom-made one replicated there. B. Birbal left, and started working on a farm (a) CBDA (b) DCAB in an unknow village. (c) ACBD (d) BADC C. Once upon a time, Akbar the great, SSC CHSL (Tier-I) 18/03/2020 (Shift-II) became very angry with his minister, Ans. (a) : Appropriate sequence of the above jumbled Birbal. sentences will be CBDA. D. As months passed, Akbar started to miss 301. Given below are four jumbled sentence. Pick his favourite minister. the option that given their correct order. (a) DCBA (b) ABCD A. Mulla Nasruddin opened a hair-cutting (c) CABD (d) DCAB salon and was ready to welcome his SSC CHSL (Tier-I) 26/10/2020 (Shift-I) customers. B. "Mulla", he said sheepishly, "my new wife Ans. (c) : Appropriate sequence of the above jumbled sentences will be CABD. is coming home today." Parajumbles

904

YCT

305. Given below are four jumbled sentences, Out of the given options pick the one that gives their correct order. A. One of the twins was driving better than the other. B. Mahesh kept comparing them every day. C. After two months of driving daily, finally both the twins were at par. D. Mahesh was teaching the twins how to drive. (a) DCBA (b) DABC (c) CDAB (d) ABCD SSC CHSL (Tier-I) 26/10/2020 (Shift-I) Ans. (b) : Appropriate sequence of the above jumbled sentences will be DABC. 306. Given below are four jumbled sentences. Pick the option that gives their correct order. A. He would be accompanied by fifty or sixty torchbearers. B. When dawn broke the Emperor would alight, say his prayers and return by 8 a.m. C. They would all head towards the river Jamuna. D. During the winter months the Badshah Bahadur Shah Jafer would set off on his horse at 3 a.m. (a) DCBA (b) CBAD (c) BACD (d) DACB SSC CHSL (Tier-I) 26/10/2020 (Shift-II) Ans. (d) : Appropriate sequence of the above jumbled sentences will be DACB. 307. Given below are four jumbled sentences. Pick the option that gives their correct order. A. He decides to leave home and sets out to find him with the help of his best friend called Sungreen. B. It narrates the story of a young and sensitive boy in a tough world. C. 'A Boy and Sungreen' is a film by the Chinese filmmaker, Ahn-Ju-Young. D. When he finds that his supposedly dead father is alive he decides to act. (a) DBCA (b) CBDA (c) CABD (d) ABDC SSC CHSL (Tier-I) 26/10/2020 (Shift-II) Ans. (b) : Appropriate sequence of the above jumbled sentences will be CBDA. 308. Given below are four jumbled sentences. Pick the option that gives their correct order. A. It is played by turning around while reciting a rhyme. B. The circling motion is like that of a Sufi dervish. C. Fahim Irshad has made a film called 'Aani Maani'. D. The movie title is based on a game popular among little children in Utter Pradesh. (a) CDAB (b) BDCA (c) CBAD (d) ACDB SSC CHSL (Tier-I) 21/10/2020 (Shift-I) Ans. (a) : Appropriate sequence of the above jumbled sentences will be CDAB. Parajumbles

309. Given below are four jumbled sentences. Pick the option that gives their correct order. A. Since then, Pathak has performed in several plays and films. B. While studying there, he say 'Equus,' a play that changed his life forever. C. Vinay Pathak was pursuing MBA in New York. D. He decided to change tracks and study drama. (a) DCAB (b) BACD (c) CBDA (d) CADB SSC CHSL (Tier-I) 21/10/2020 (Shift-I) Ans. (c) : Appropriate sequence of the above jumbled sentences will be CBDA. 310. Given below are four jumbled sentences. Out of the given options, pick the one that gives their correct order. (A) So, what exactly are we eating? (B) They found that the vegetables contained very high, toxic levels of banned pesticides, and the pesticide levels were more than 1000 times the permissible levels. (C) And is our food giving rise to the many health problems today? (D) A survey conducted by the Food Safety and Standards Authority of India under the Union Agriculture Ministry tested random samples of vegetables in markets all over India. (a) DBAC (b) ACBD (c) DACB (d) ADCB SSC CHSL (Tier-I) 19/10/2020 (Shift-I) Ans. (a) : Appropriate sequence of the above jumbled sentences will be DBAC. 311. Given below are four jumbled sentences. Out of the given options pick the one that gives their correct order. (A) You've probably noticed, with great disappointment, that the battery doesn't seem to last as long as it did when it was brand new. (B) Partial charging is just fine for lithium-ion batteries and can actually have some positive benefits for cell longevity. (C) Smartphone users are forever in search of longer battery life, but unfortunately, battery capacity inevitably declines with age. (D) However, one thing you can do to prolong the life of your battery and handset is partial charging. (a) ACDB (b) CADB (c) CDAB (d) BADC SSC CHSL (Tier-I) 19/10/2020 (Shift-I) Ans. (b) : Appropriate sequence of the above jumbled sentences will be CADB. 312. Given below are four jumbled sentences. Select the option that give their correct order. (A) In order to buy that bicycle, he plans to winsome prize money by teaching an illiterate person how to read and write. (B) He needs a bicycle to win the bicycle race which would help pay off his father's debt.

905

YCT

(C) The film 'Douch' is about a young Iranian village boy. (D) But the only illiterate person is a 90 year old woman who is ill-tempered and doesn't wish to learn anything. (a) CABD (b) DABC (c) ADBC (d) CBAD SSC CHSL (Tier-I) 12/10/2020 (Shift-II) Ans. (d) : Appropriate sequence of the above jumbled sentences will be CBAD. 313. Given below are four jumbled sentences. Select the option that gives their correct order. (A) Behind them come camels carrying drums and small guns. (B) The Emperor is escorted by two decorated elephants who head the procession. (C) At the end come the platoons of soldiers in rows of two. (D) First of all, there is a volley of guns announcing Emperor Bahadur Shah's entry into the mosque. (a) ABDC (b) DBAC (c) BCDA (d) DCBA SSC CHSL (Tier-I) 12/10/2020 (Shift-II) Ans. (b) : Appropriate sequence of the above jumbled sentences will be DBAC. 314. Four jumbled sentences are given. Select the option that gives their correct order. A. Saranda forests, in Singhbum, are a huge catchment for rivers and streams in this state. B. If these forests are opened up for mining purposes the remaining rivers and streams will dry up. C. The state of Jharkand faces water scarcity every summer. D. These forests are India's largest contiguous Sal forests and the current plan says mining should not be allowed in the dense forest as rivers are drying up. (a) CBAD (b) CADB (c) BADC (d) ACDB SSC CHSL (Tier-I) 15/10/2020 (Shift-III) Ans. (b) : Appropriate sequence of the above jumbled sentences will be CADB. 315. Four jumbled sentences are given. Select the option that gives their correct order. A. The minimum temperature in the national capital dropped yesterday as cold north westerly winds continued to blow in from the hills. B. North India has been witnessing a harsh winter this year. C. However, experts said the minimum temperatures would increase over the next few days due to cloud cover. D. A favourable outcome of the wind factor was that there was a slight decrease in air pollution levels due to the wind speeds. (a) CDAB (b) BADC (c) ABDC (d) BCDA SSC CHSL (Tier-I) 15/10/2020 (Shift-III) Parajumbles

Ans. (b) : Appropriate sequence of the above jumbled sentences will be BADC. 316. Given below are four jumbled sentences. Select the option that gives their correct order. A. In India, forests are huge catchment areas for rivers and streams as they help in storing water. B. If these forests continue to be felled indiscriminately, rivers and streams will dry up leading to drought. C. Due to this many states are facing shortage of water in the summer months. D. Unfortunately, these dense forest areas are disappearing. (a) ABCD (b) CADB (c) DCBA (d) ADCB SSC CHSL (Tier-I) 15/10/2020 (Shift-II) Ans. (d) : Appropriate sequence of the above jumbled sentences will be ADCB. 317. Given below are four jumbled sentences. Select the option that gives their correct order. A. However, in today's ever changing world, power looms are replacing the traditional handloom textiles. B. Handloom refers to the use of wooden frames of various kinds by skilled artisans to weave fabric from natural fibres. C. One of the most prominent among Indian handlooms are Banarasi textiles. D. Banarasi textiles are made with gleaming gold threads and weaving techniques as old as time. (a) BDAC (b) DBCA (c) BCDA (d) CDAB SSC CHSL (Tier-I) 15/10/2020 (Shift-II) Ans. (c) : Appropriate sequence of the above jumbled sentences will be BCDA. 318. Given below are four jumbled sentences. Select the option that gives their correct order. A. So, clearly, where you get your news form matters since you can't possibly be expected to verify and authenticate every bit of 'news' you receive. B. Are your really sure those apparently sound bit of medical advice you received as forwards would pass muster with your doctor? C. Yet, we do often forward these in the belief that we are doing a good deed to our friends and relatives. D. Being educated is no guarantee against falling prey to fake news. (a) DBCA (b) CADB (c) ACBD (d) BACD SSC CHSL (Tier-I) 15/10/2020 (Shift-I) Ans. (a) : Appropriate sequence of the above jumbled sentences will be DBCA. 319. Given below are four jumbled sentences. Select the option that gives their correct order. A. While an average person lives up to 72 years of age the life expectance of doctors in only about 59 years the shocking IMA study concluded.

906

YCT

B. The sudden death of Dr. Meena Kumar at an International Conference in London after suffering a cardiac arrest has alarmed the medical community. C. She was a well-known medico recognised for her services as a consultant neurologist with a career of over 25 years. D. Her death brings to the fore the fact that even doctors have risky life styles and a hard life, a study by the Indian Medical Association conducted on doctors, said. (a) DABC (b) CABD (c) ABDC (d) BCDA SSC CHSL (Tier-I) 15/10/2020 (Shift-I) Ans. (d) : Appropriate sequence of the above jumbled sentences will be BCDA. 320. Given below are four jumbled sentences. Select the option that gives their correct order. A. I sow a seed. B. I saw the sapling. C. I saw the sprout. D. I saw the plant. (a) ABCD (b) CABD (c) DBAC (d) ACBD SSC CHSL (Tier-I) 13/10/2020 (Shift-III) Ans. (d) Appropriate sequence of the above jumbled sentences will be ACBD. 321. Given below are four jumbled sentences. Select the option that gives correct order. A. They grew cabbages and carrots in their back yard. B. The family was very poor and was alway hungry because there was a war on. C. They also had a chicken, which provided the family with eggs, but still they ware hungry. D. Their country had been invaded by a foreign army. (a) CADB (b) CDAB (c) CDAB (d) BDAC SSC CHSL (Tier-I) 13/10/2020 (Shift-III) Ans. (d) Appropriate sequence of the above jumbled sentences will be BDAC. 322. Given below are four jumbled sentences. Out of the given options pick the one that gives their correct order. (A) It was known as Lolark Ghat earlier and was renamed in honour of the saint. (B) In 1941, industrialist B.D. Birla undertook the renovation of the ghat. (C) Among the seven ghats that are associated with his life, the Tulsi Ghat is most special. (D) The glorious city of Kashi, famous for its ghats along the Ganges river, is where the 17th century saint-poet, Tulsidas, lived. (a) ABDC (b) CDBA (c) BDCA (d) DCAB SSC CHSL (Tier-I) 14/10/2020 (Shift-III) Ans. (d) : Appropriate sequence of the above jumbled sentences will be DCAB. Parajumbles

323. Given below are four jumbled sentences. Out of the given options pick the one that gives their correct order. (A) One of them said "Let us use our combined knowledge to bring this animal to life." (B) On the way, the three friends passed through a forest. (C) Seeing the bones, the friends thought of testing their knowledge and skills. (D) Under a tree in the forest, they saw the bones of an animal. (a) BCAD (b) ADCB (c) BDCA (d) CABD SSC CHSL (Tier-I) 14/10/2020 (Shift-III) Ans. (c) : Appropriate sequence of the above jumbled sentences will be BDCA. 324. Given below are four jumbled sentences. Pick the option that gives their correct order. A. Soon a special friendship develops between the two girls in war torn Syria. B. While at her grandmother’s place, Suzan meets Rana, a Syrian girl, who lives in a nearby refugee camp. C. ‘Summer with Rana’ is a film by Sanne Vogel, who began her film career as early as 17. D. Her film revolves around Suzan, the young girl who is spending her summer holiday with her grandmother, while her mother has gone to Syria, as a war reporter. (a) ADBC (b) CADB (c) BDAC (d) CDBA SSC CHSL (Tier-I) 21/10/2020 (Shift-III) Ans. (d) : Appropriate sequence of the above jumbled sentences will be CDBA. 325. Given below are four jumbled sentences. Pick the option that gives their correct order. A. At the onset of his journey, his mother guides him how to survive and navigate the world. B. ‘Silo’s Journey’ is a film made by a wildlife film director. C. It follows the journey of a young reindeer’s first year at close range. D. Sharing this journey is his friend-a curious red-haired squirrel. (a) BDCA (b) BCAD (c) CABD (d) DABC SSC CHSL (Tier-I) 21/10/2020 (Shift-III) Ans. (b) : Appropriate sequence of the above jumbled sentences will be BCAD. 326. Given below are four sentences in a jumbled order. Out of the given options pick the one that gives their correct order. A. The police suspected the man to be a thief. B. Then the trial began in the court. C. He was soon arrested by the police. D. The thief was sentenced to two year's imprisonment. (a) BCDA (b) ACBD (c) BACD (d) DCAB SSC CHSL (Tier-I) 20/10/2020 (Shift-III)

907

YCT

Ans. (b) : Appropriate sequence of the above jumbled Ans. (a) : Appropriate sequence of the above jumbled sentences will be ACBD. sentences will be CABD. 327. Given below are four sentences in a jumbled 331. Four jumbled sentences are given. Select the order. Out of the given options pick the one option that gives their correct order. that gives their correct order. (A) The middle sum was the one we usually see A. Add the product to your shopping cart. every day, but the ones on either side were B. Select the brand you like. the Sun's smaller reflections. C. Enter the card details and buy the product (B) It looked like there were three Suns in the online. sky. D. Look for the product you want to purchase. (C) A local meteorological centre explained (a) ABCD (b) DACB that reflection is caused by sunshine high (c) BCDA (d) DBAC in the sky and light passing through snow SSC CHSL (Tier-I) 20/10/2020 (Shift-III) crystals in the air Ans. (d) : Appropriate sequence of the above jumbled (D) A rare celestial phenomenon appeared in sentences will be DBAC. Mongolia. 328. Given below are four jumbled sentences. Out of (a) DBAC (b) ADBC the given options pick the one that gives their (c) DBCA (d) ACBD correct order. SSC CHSL (Tier-I) 19/10/2020 (Shift-III) A. The cable was thick as pack-thread, and Ans. (a) : Appropriate sequence of the above jumbled the bars were the size of knitting needles. B. Then I made fifty hooks by twisting three sentences will be DBAC. bars together at a time. 332. Given below are four jumbled sentences. Select C. I ordered a great quantity of strong cable the option that gives their correct order. and bars of iron. (A) In reality, however, we exist in D. I twisted the cables together and made fifty compartments, separated by walls of caste, strong cords. religion, and geography. (a) CABD (b) CADB (B) But we find that ignorance levels are (c) ABDC (d) ADCB growing as well because the rate at which SSC CHSL (Tier-I) 20/10/2020 (Shift-I) we are accumulating information is not Ans. (b) : Appropriate sequence of the above jumbled commensurate with the rate at which our sentences will be CADB. awareness is growing. 329. Given below are four jumbled sentences. Out of the given options pick the one that gives their (C) Technologically we live in a borderless correct order. world. A. In December 2019, Typhoon Kammuri (D) Another paradox is that we take pride in came to the Philippines. describing ourselves as a knowledge-based B. They closed schools and evacuated hundreds society. of thousands of people from their homes. (a) CBAD (b) ABDC C. Officials closed the international airport in (c) CADB (d) BADC the capital city of Manila for 12 hours. SSC CHSL (Tier-I) 20/10/2020 (Shift-II) D. It flooded streets and damaged many houses and power lines. Ans. (c) : Appropriate sequence of the above jumbled (a) CBAD (b) ADCB sentences will be CADB. (c) ABCD (d) CBDA 333. Given below are four jumbled sentences. Select SSC CHSL (Tier-I) 20/10/2020 (Shift-I) the option that gives their correct order. Ans. (b) : Appropriate sequence of the above jumbled (A) The father then wanted his son to pick him sentences will be ADCB. up at four in the afternoon so that they 330. Four jumbled sentences are given. Select the could go home together. option that gives their correct order. (B) A teenager had just passed his driving test (A) Put equal quantities of water and milk in a and was eager to drive his father's car. sauce pan and keep it on the flame. (C) He asked his son to drop him at a place (B) Now put some ginger, tea powder and sugar as per taste and boil it for a couple of that was some 18 miles from home before minutes. he took the car for servicing. (C) Tea is one of the most common Asian (D) His father agreed. drinks and is very simple to make (a) BDCA (b) CBAD (D) Turn off the flame, strain it into a cup and (c) BCAD (d) ADBC serve it hot. SSC CHSL (Tier-I) 20/10/2020 (Shift-II) (a) CABD (b) BCAD Ans. (a) : Appropriate sequence of the above jumbled (c) ABCD (d) CBAD sentences will be BDCA. SSC CHSL (Tier-I) 19/10/2020 (Shift-III) Parajumbles

908

YCT

334. Given below are four jumbled sentences. Out of the given options pick the one that gives their correct order. (A) People in a small Chinese town were shocked to see a wild elephant wandering in their town. (B) The elephant eventually left the town. (C) Authorities followed the large mammal's every movement. (D) But the locals will probably never forget this special visit. (a) ABDC (b) CABD (c) BDAC (d) ACBD SSC CHSL (Tier-I) 19/10/2020 (Shift-II) Ans. (d) : Appropriate sequence of the above jumbled sentences will be ACBD. 335. Given below are four jumbled sentences. Out of the give options pick the one that gives their correct order. (A) When he returned, there was more than just his dog waiting for him. (B) A koala had joined the dog in the car's more comfortable climate. (C) A winemaker in South Australia left his air-conditioned car open to help keep his dog cool in the heat. (D) Although the human struggled a while to make the koala leave, it did leave his car eventually. (a) CBAD (b) BCDA (c) CABD (d) BACD SSC CHSL (Tier-I) 19/10/2020 (Shift-II) Ans. (c) : Appropriate sequence of the above jumbled sentences will be CABD. 336. Given below are four jumbled sentences. Out of the given options pick the one that gives their correct order. (A) He was on a pilgrimage to Jerusalem. (B) The villagers were kind though doubtful. (C) They offered him some food to eat and a room to stay. (D) One night, he took rest in a village unfamiliar on the way. (a) DCAB (b) CADB (c) ADBC (d) CDBA SSC CHSL (Tier-I) 19/03/2020 (Shift-II) Ans. (c) : Appropriate sequence of the above jumbled sentences will be ADBC. 337. Given below are four jumbled sentences. Out of the given options pick the one that gives their correct order. (A) The driver had a cloudy look and drove off without taking him. (B) He asked a taxi driver to take him to the shopping mall. (C) He told the driver that he could pay the fare by card, not is cash. (D) He arrived on the outskirts of a city. (a) CDBA (b) DBCA (c) ACDB (d) DCAB SSC CHSL (Tier-I) 19/03/2020 (Shift-II) Parajumbles

Ans. (b) : Appropriate sequence of the above jumbled sentences will be DBCA. 338. Given below are four jumbled sentences. Pick the one that gives their correct order. (A) When we wish to achieve something, we cannot expect success in a moment. (B) It is the same with any great achievement. (C) It took several years to build Rome and bring it to the state of glory. (D) Rome, the greatest city of ancient world was not built in a short time. (a) CDAB (b) BACD (c) DCBA (d) ABCD SSC CHSL (Tier-I) 14/10/2020 (Shift-II) Ans. (c) : Appropriate sequence of the above jumbled sentences will be DCBA. 339. Given below are four jumbled sentences. Out of the given options pick the one that gives their correct order. (A) It is also very sad that ill-treatment is meted out to one of your gifts, and everyone indulges in excess!" (B) "I can't take it anymore," he complained. " people abuse me and exploit me, and there is no respect for me-no realisation that I am your prasadam to men! (C) An ancient story tells us that the God of Food came before the Divine Presence, battered, exhausted, and downcast. (D) the Lord smiles and said to him, "Those who abuse you and indulge in excess are destroying themselves." (a) CBAD (b) CABD (c) ACDB (d) BADC SSC CHSL (Tier-I) 14/10/2020 (Shift-II) Ans. (a) : Appropriate sequence of the above jumbled sentences will be CBAD. 340. Given below are four jumbled sentences. Out of the given options pick the one that gives their correct order. (A) Then he would give one leap, two leaps, through the snow, and sit up again on his hind legs and look all around. (B) A grey hare lived in the woods near a village. (C) Then he would prick up the other, jerk his whiskers, snuff, and sit up on his hind legs. (D) During the winter, when night came, he would prick up one ear and listen. (a) BCDA (b) CDBA (c) ABCD (d) BDCA SSC CHSL (Tier-I) 14/10/2020 (Shift-I) Ans. (d) : Appropriate sequence of the above jumbled sentences will be BDCA. 341. Given below are four jumbled sentences. Out of the given options pick the one that gives their correct order. A. According to the EPR, the corporation expects such manufactures to develop a collect back system for plastic waste.

909

YCT

B. The Municipal Corporation has introduced the extended producers responsibility (EPR) for producers and generators of plastic waste. C. The producers are persons engaged in manufacture, or import of carry bags, multi-layered packaging and sheets, and person using them. D. They must formulate a plan to collect reusable waste generated from their products within a prescribed timeframe. (a) ACBD (b) DBCA (c) CADB (d) BCAD SSC CHSL (Tier-I) 14/10/2020 (Shift-I) Ans. (d) : Appropriate sequence of the above jumbled sentences will be BCAD. 342. Given below are four jumbled sentences. Select the option that gives their correct order. A. He put it down to dry and went to count his coins. B. When he came back, it seemed like the teapot had moved. C. Akito walked into an old curiosity shop and bought a teapot and took it home. D. Later that day, he cleaned the teapot till it shone like the sun. (a) CDAB (b) BCDA (c) CABD (d) ABDC SSC CHSL (Tier-I) 13/10/2020 (Shift-II) Ans. (a) : Appropriate sequence of the above jumbled sentences will be CDAB. 343. Given below are four jumbled sentences. Select the option that gives their correct order. A. "Today, respect is shown in different ways", she said, "students have a more questioning bent of mind". B. Vidya Subramaniam occupies a small but privileged place in the dance world that has seen the old guard giving place to the new. C. Back in the day, respect meant not sitting on even the same level as one's gurus but that's no longer true, especially for those who live in Western Milieus. D. For instance, she spoke of how the very idea of 'respect' has changed over time. (a) BCAD (b) BDCA (c) DCAB (d) DBAC SSC CHSL (Tier-I) 13/10/2020 (Shift-II) Ans. (b) : Appropriate sequence of the above jumbled sentences will be BDCA. 344. Given below are four sentences in jumbled order. Out of the given options pick the one that gives their correct order. A. A boy tended his master's dogs near a pond. B. However, he realized that the way he talked to them never made them happy. C. Soon he found the tending and his life there very boring. D. All the could do to kill the boredom was by talking to the dogs. Parajumbles

(a) ABCD (b) ACBD (c) DCAB (d) ADBC SSC CHSL (Tier-I) 12/10/2020 (Shift-III) Ans. (b) : Appropriate sequence of the above jumbled sentences will be ACBD. 345. Given below are four sentences in jumbled order. Out of the given options pick the one that gives their correct order. A. At the metro station he noticed a homeless man. B. He decided to travel to office by the metro train. C. he sent his damaged car to the garage for repairs. D. Johnny had a car accident. (a) DCBA (b) DBAC (c) DACB (d) DBCA SSC CHSL (Tier-I) 12/10/2020 (Shift-III) Ans. (a) : Appropriate sequence of the above jumbled sentences will be DCBA. 346. Given below are four jumbled sentences. Select the option that gives their correct order. A. The most well-known among them is the Purple Frog needing immediate protection. B. The secret Life of Frogs is the first ever film on the Amphibians of India. C. The film has been shot extensively in the Western Ghats. D. It showcases unique species of frogs which are critically endangered. (a) DCBA (b) BCDA (c) BCAD (d) CBDA SSC CHSL (Tier-I) 12/10/2020 (Shift-I) Ans. (b) : Appropriate sequence of the above jumbled sentences will be BCDA. 347. Given below are four jumbled sentences. Select the option that gives their correct order. A. This test is conducted of the fully automated Driving Test designed by Maruti Suzuki B. Each of these specific skills are to be tested and need to be completed successfully in under seven minutes by the candidate. C. It is a fully-automated track having six segments to test specific driving skills. D. The Regional Transport Offices of Delhi have recently introduced the new driving test. (a) DBAC (b) ADBC (c) DACB (d) BADC SSC CHSL (Tier-I) 12/10/2020 (Shift-I) Ans. (c) : Appropriate sequence of the above jumbled sentences will be DACB. 348. Given below are four jumbled sentences. Select the option that gives their correct order. A. Problems in absorption of vitamins is the main cause of vitamin deficiency. B. This is the reason why people with metabolic disorders such as obesity, often suffer from vitamin deficiency.

910

YCT

C. Hence, digestion and the absorption mechanism in the body is very important. D. Vitamin deficiency is not just caused by lack of vitamins in the diet. (a) DABC (b) BCAD (c) CBDA (d) ACBD SSC CHSL (Tier-I) 17/03/2020 (Shift-III) Ans. (a) : Appropriate sequence of the above jumbled sentences will be DABC. 349. Given below are four jumbled sentences. Select the option that gives their correct order. A. Just before the sacrifice, however, he noticed that one of the king’s fingers was missing. B. Once when travelling across a forest, the king was captured by a powerful tribe. C. Since it was not acceptable to sacrifice an incomplete person to God, the king was released. D. The tribal priest decided to sacrifice the king to their God. (a) CDBA (b) ADBC (c) DACB (d) BDAC SSC CHSL (Tier-I) 17/03/2020 (Shift-III) Ans. (d) : Appropriate sequence of the above jumbled sentences will be BDAC. 250. Rearrange the parts of the sentence in correct order : Accept only those economic P : developmental activities that are Q : and economically rewarding R : people friendly, environment friendly (a) QPR (b) RPQ (c) RQP (d) PRQ SSC MTS 7-10-2017 (Shift-I) Ans. (d) : Appropriate sequence of the above jumbled sentences will be PRQ. 351. Rearrange the parts of the sentence in correct order : Pulses are P : vegetarian diet Q : a major source R : of protein in a (a) PQR (b) QRP (c) QPR (d) RQP SSC MTS 7-10-2017 (Shift-I) Ans. (b) : Appropriate sequence of the above jumbled sentences will be QRP. 352. Rearrange the parts of the sentence in correct order. The magnitude of stress P: individual's operational style Q: environmental forces or an R: is determined by the (a) PQR (b) RPQ (c) RQP (d) QPR SSC MTS 9-10-2017 (Shift-II) Ans. (c) : Appropriate sequence of the above jumbled sentences will be RQP. Parajumbles

353. Rearrange the parts of the sentence in correct order. Time management is the P: as to make the most of it Q: technique to use one's R: time efficiently so (a) QRP (b) PQR (c) RPQ (d) QPR SSC MTS 9-10-2017 (Shift-II) Ans. (a) : Appropriate sequence of the above jumbled sentences will be QRP. 354. Rearrange the parts of the sentence in correct order. Friendship P: an asset Q: in life R: is indeed (a) PQR (b) RPQ (c) QPR (d) PRQ SSC MTS 9-10-2017 (Shift-I) Ans : (b) Appropriate sequence of the above jumbled sentences will be RPQ. 355. Rearrange the parts of the sentence in correct order. If you don't want P: freedom either Q: you can't have R: to take responsibility (a) RPQ (b) RQP (c) PRQ (d) PQR SSC MTS 9-10-2017 (Shift-I) Ans : (b) Appropriate sequence of the above jumbled sentences will be RQP. 356. Rearrange the parts of the sentence in correct order. The increase of the P: Shrinking of forest area Q: has led to the R: population of our country (a) QRP (b) PRQ (c) QPR (d) RQP SSC MTS 9-10-2017 (Shift-III) Ans : (d) Appropriate sequence of the above jumbled sentences will be RQP. 357. Rearrange the parts of the sentence in correct order. Deforestation is P: society and environment Q: arising as a major R: when the situation offers (a) QRP (b) PRQ (c) QPR (d) RPQ SSC MTS 10-10-2017 (Shift-III) Ans. (a) : Appropriate sequence of the above jumbled sentences will be QRP. 358. Rearrange the parts of the sentence in correct order. Stress is said to be positive P: an opportunity for Q: one to give something R: when the situation offers

911

YCT

(a) QRP (c) PQR

(b) RPQ (d) RQP SSC MTS 10-10-2017 (Shift-III) Ans. (b) : Appropriate sequence of the above jumbled sentences will be RPQ. 359. Rearrange the parts of the sentence in correct order. A country cannot P: development of its citizens Q: the individual growth and R: grow and develop without (a) PRQ (b) QRP (c) PQR (d) RQP SSC MTS 10-10-2017 (Shift-II) Ans. (d) : Appropriate sequence of the above jumbled sentences will be RQP. 360. Rearrange the parts of the sentence in correct order. Everyone should P: the same opportunities Q: command the same R: respect and enjoy (a) RQP (b) QRP (c) PQR (d) QPR SSC MTS 10-10-2017 (Shift-II) Ans. (b) : Appropriate sequence of the above jumbled sentences will be QRP. 361. Rearrange the parts of the sentence in correct order. If you shun responsibility P: accept slavery in Q: some way or other R: you have to (a) PQR (b) RQP (c) QPR (d) RPQ SSC MTS 10-10-2017 (Shift-I) Ans : (d) Appropriate sequence of the above jumbled sentences will be RPQ. 362. Rearrange the parts of the sentence in correct order. Digital India Programme P: government of India to make this Q: is a big step taken by the R: country a digitally empowered country (a) QPR (b) RPQ (c) PQR (d) RQP SSC MTS 10-10-2017 (Shift-I) Ans : (a) Appropriate sequence of the above jumbled sentences will be QPR. 363. Rearrange the parts of the sentences in correct order. Time management P: one's time efficiently so as Q: to be more productive and organized R: is the ability to utilize. (a) RPQ (b) PQR (c) QPR (d) PRQ SSC MTS 11-10-2017 (Shift-III) Parajumbles

Powered by TCPDF (www.tcpdf.org)

Ans. (a) : Appropriate sequence of the above jumbled sentences will be RPQ. 364. Rearrange the parts of the sentence in correct order. Disaster management is a P: well planned strategy for Q: hazards caused by the disaster R: making efforts to reduce the (a) PQR (b) RQP (c) PRQ (d) QRP SSC MTS 11-10-2017 (Shift-III) Ans. (c) : Appropriate sequence of the above jumbled sentences will be PRQ. 365. Rearrange the parts of the sentence in correct order. Small scale industry P: and marketing of output Q: for procuring raw material R: needs government's support (a) PQR (b) RQP (c) QRP (d) QPR SSC MTS 11-10-2017 (Shift-II) Ans. (b) : Appropriate sequence of the above jumbled sentences will be RQP. 366. Rearrange the parts of the sentence in correct order. Sapna a small P: on her three Q: acres of land R: farmer, grows groundnuts (a) RPQ (b) RQP (c) PQR (d) PRQ SSC MTS 11-10-2017 (Shift-II) Ans. (a) : Appropriate sequence of the above jumbled sentences will be RPQ. 367. Rearrange the parts of the sentence in correct order. Presence of various P: is also causing earth's Q: aerosols in the atmosphere R: surface temperature to increase (a) PQR (b) RPQ (c) QPR (d) RQP SSC MTS 11-10-2017 (Shift-I) Ans : (c) Appropriate sequence of the above jumbled sentences will be QPR. 368. Rearrange the parts of the sentence in correct order. Proper education identifies P: teaches us to live Q: in a more civilized manner R: our career goals and (a) PRQ (b) PQR (c) RPQ (d) RQP SSC MTS 11-10-2017 (Shift-I) Ans : (c) Appropriate sequence of the above jumbled sentences will be RPQ.

912

YCT